NASASpaceFlight.com Forum

General Discussion => New Physics for Space Technology => Topic started by: Chris Bergin on 12/12/2014 11:40 am

Title: EM Drive Developments - related to space flight applications - Thread 2
Post by: Chris Bergin on 12/12/2014 11:40 am
Thread 1:
http://forum.nasaspaceflight.com/index.php?topic=29276.0

Problems with Thread 1 explained (note the things to avoid in the new thread):
http://forum.nasaspaceflight.com/index.php?topic=29276.msg1301657#msg1301657

Thus Thread 2, with more focus on space flight applications to ensure this can have a healthy home here. Remember, this is a space flight site.

Title: Re: EM Drive Developments - related to space flight applications - Thread 2
Post by: RotoSequence on 12/12/2014 05:30 pm
I really do hope they EM drive theorists are onto something; if it works, and depending on how well it works, the space flight applications range from replacing maneuvering thrusters to decreasing the cruising times of missions. If, by some miracle of physics, EM drives work spectacularly well, they could replace chemical rocketry for orbital insertion completely and make space access trivial.

If EM drives offer any degree of thrust, telecommunications and science missions will benefit considerably. If it's possible to supply a lot of thrust, they'll literally change the destiny of humanity. But first, we need to know if they work or not. The theories are certainly intriguing, and there are a fair few anecdotal hints that there is something there, but as the several hundred pages of the previous thread show, figuring out what's going on is a real bear.
Title: Re: EM Drive Developments - related to space flight applications - Thread 2
Post by: RonM on 12/12/2014 06:05 pm
Chris, thanks for cleaning up the old thread. There is a lot of good discussion and information in there.

EM drives would have to push or pull against something. The favorite explanation seems to be the Mach Effect, but is there any experimental evidence that points to the Mach Effect being real? Since the origin of inertia seems to be the key, what about the Higgs field? Local gravitational field?

Although I have my doubts, the potential payoff for spaceflight is so large, this should be looked into, but the experiments need to be rigorously done and repeatable.
Title: Re: EM Drive Developments - related to space flight applications - Thread 2
Post by: MP99 on 12/12/2014 06:06 pm
Thanks, Chris.

I really do want to follow this, on the chance that it bears out. There was some good stuff on that thread, but a low s/n ratio combined with a high post count, at the long-ago point I stopped following.

Hoping this will do better. (Might even suggest you leave the old thread open to allow this one to stay cleaner, but know that wouldn't work!)

Cheers, Martin
Title: Re: EM Drive Developments - related to space flight applications - Thread 2
Post by: Star One on 12/12/2014 06:50 pm
Has there been the least indication yet when we are too get some more results in relation to those who are experimenting in this area?
Title: Re: EM Drive Developments - related to space flight applications - Thread 2
Post by: HMXHMX on 12/13/2014 03:47 am
Has there been the least indication yet when we are too get some more results in relation to those who are experimenting in this area?


This paper, presented at the Joint Propulsion Conference this past summer in Cleveland, should address your question.

Title: Re: EM Drive Developments - related to space flight applications - Thread 2
Post by: cuddihy on 12/13/2014 04:51 am
Wow, great paper, Gary! Thanks for pointing it out. I see SSI's efforts got a mention as well.

Interesting this seems to be picking up steam just as Hollywood put out a Sci-Fi movie that takes Space-time, wormholes, and gravity seriously.
Title: Re: EM Drive Developments - related to space flight applications - Thread 2
Post by: RotoSequence on 12/13/2014 05:43 am
Well, looking at the thermistor data with the PZT stack mach effect thruster, whatever it's doing doesn't look like a transient thermal effect...

Title: Re: EM Drive Developments - related to space flight applications - Thread 2
Post by: SteveKelsey on 12/13/2014 09:01 am
That is a very interesting paper. Very clean results compared to previous presentations and the Hoyl and Narliker approach to theory looks promising. It needs to be emphasised that this is an ME device, not EM, and that the paper reports that the theory is a work in progress.
Title: Re: EM Drive Developments - related to space flight applications - Thread 2
Post by: Notsosureofit on 12/13/2014 01:08 pm
Only had a quick look at the paper.  I would like to know the positioning of the accelerometer.  If this is a displacement or zero set type balance the acceleration during the constant thrust segment should be zero.  Possibly the mounting accounts for this ??

Edit:  OK.  The "accelerometer" is between the PZT.  It is not the system accelerating, just an indication that the transducers are receiving power.
Title: Re: EM Drive Developments - related to space flight applications - Thread 2
Post by: HMXHMX on 12/13/2014 03:50 pm
Well, looking at the thermistor data with the PZT stack mach effect thruster, whatever it's doing doesn't look like a transient thermal effect...



I can attest that it is not thermal.  It works in a vacuum.  It works in a Faraday cage and it works when you reverse the device (the thrust reverses).
Title: Re: EM Drive Developments - related to space flight applications - Thread 2
Post by: HMXHMX on 12/13/2014 03:53 pm
That is a very interesting paper. Very clean results compared to previous presentations and the Hoyl and Narliker approach to theory looks promising. It needs to be emphasised that this is an ME device, not EM, and that the paper reports that the theory is a work in progress.

Quite so.  Perhaps the thread needs to be called "Space Drive" (a la Arthur C. Clarke), or "Propellantless Propulsion."
Title: Re: EM Drive Developments - related to space flight applications - Thread 2
Post by: Rodal on 12/14/2014 06:14 pm
"With more focus on space flight applications " (as per http://forum.nasaspaceflight.com/index.php?topic=36313.msg1301658#msg1301658) the metric chosen by the NASA's "Anomalous" report was the thrust force per power input.

EDIT: Dimensions for Shawyer's and Brady et.al.'s cavities updated as per latest estimates on 05/20/15

Here is a comparison of reported measurements for EM Drives and for the latest report by Fearn, Zachar, Woodward & Wanser.

Notice that the force per power input reported by  Fearn, Zachar, Woodward & Wanser is several orders of magnitude lower than the "EM drives".  Actually it is barely (3.5 times higher) more than the force per power input of a photon rocket:

reported measurement ForcePerPowerInput (milliNewtons/kW)

(* Cannae Superconducting *)             761.9 to 952.4
(* Shawyer Demo *)                               80 to 243
(* Shawyer Experimental *)                   18.82
(* Brady c TE mode *)                             21.31
(* Brady a TM mode*)                               5.396
(* Brady b TM mode*)                               3.000
(*Fearn, Zachar, Woodward & Wanser*) 0.01176




lengths in meter
rfFrequency in 1/second (microwave frequency during test)
power in watts
force in milliNewtons
force per PowerInput in milliNewtons/kW
c= 299705000 m/s (speed of light in air)
c= 299792458 m/s (speed of light in vacuum) (for Cannae Superconducting)
(the difference between c in air compared to c in vacuum is negligible)


Note: SmallDiameter for Shawyer's EM Drives obtained from his reported ShawyerDesignFactor .


Force/PowerInput of a Photon Rocket = 1 / c


(* Cannae Superconducting *)
rfFrequency = 1.047*10^9;
cavityLength = 0.01+0.004+0.006+0.01 = 0.03;
bigDiameter =(22.86-2*(0.00430)) = 0.220;
smallDiameter = bigDiameter-2*0.01=0.200;

power =  10.5
Q = 1.1*(10^7)

measured force = 8 to 10
measured ForcePerPowerInput = 761.9 to 952.4
Force/PowerInput of a Photon Rocket =0.003337
measured ForcePerPowerInput to the one of a photon rocket = 228,400 to 285,500


(* Shawyer Experimental *)
rfFrequency=2.45*10^9;
cavityLength=0.156;(estimated from photographs)
bigDiameter=0.16; (given by Shawyer)
smallDiameter=0.1025; (obtained from the Design Factor, bigDiameter and frequency provided by Shawyer)
Design Factor = 0.497;
power =  850;   
Q = 5900 ;

measured force = 16
measured ForcePerPowerInput = 18.82
Force/PowerInput of a Photon Rocket =0.003337
measured ForcePerPowerInput to the one of a photon rocket =5,640


(* Shawyer Demo *)
rfFrequency=2.45*10^9;
cavityLength=0.345;(estimated from photographs)
bigDiameter=0.28;(given by Shawyer)
smallDiameter=  0.09613; (from the Design Factor, bigDiameter and frequency provided by Shawyer)
Design Factor = 0.844;
power =  421 to 1200;
Q = 45000;

(measured force = 102.30 milliNewtons only reported for  421 watts, 243 milliNewtons/kW )

measured ForcePerPowerInput = 80 to 243
Force/PowerInput of a Photon Rocket =0.003337
measured ForcePerPowerInput to the one of a photon rocket =23,980 to 72,830

All Brady cases have the following dimensions:

cavityLength=0.2286; (provided by Paul March)
bigDiameter=0.2794; (provided by Paul March)
smallDiameter=0.15875; (provided by Paul March)


(* Brady a TM mode*)
rfFrequency=1.9326*10^9;

power =   16.9 
Q = 7320

measured force =  0.0912
measured ForcePerPowerInput = 5.396
Force/PowerInput of a Photon Rocket =0.003337
measured ForcePerPowerInput to the one of a photon rocket =1,617.2


(* Brady b TM mode*)   
rfFrequency=1.9367*10^9;

power = 16.7
Q =  18100

measured force = 0.0501
measured ForcePerPowerInput = 3.000
Force/PowerInput of a Photon Rocket =0.003337
measured ForcePerPowerInput to the one of a photon rocket =899.12


  (* Brady c  TE mode *)
rfFrequency = 1.8804*10^9;

power = 2.6
Q = 22000

measured force = 0.05541
measured ForcePerPowerInput = 21.31
Force/PowerInput of a Photon Rocket =0.003337
measured ForcePerPowerInput to the one of a photon rocket =6,386.7


(* Fearn, Zachar, Woodward & Wanser*)
rfFrequency = 39,300;

power =  170

measured force = 0.002
measured ForcePerPowerInput = 0.01176
Force/PowerInput of a Photon Rocket =0.003337
measured ForcePerPowerInput to the one of a photon rocket = 3.526

 
Title: Re: EM Drive Developments - related to space flight applications - Thread 2
Post by: birchoff on 12/14/2014 07:16 pm
"With more focus on space flight applications " (as per http://forum.nasaspaceflight.com/index.php?topic=36313.msg1301658#msg1301658) the metric chosen by the NASA's "Anomalous" report was the thrust force per power input.


Here is a comparison of reported measurements for EM Drives and for the latest report by Fearn, Zachar, Woodward & Wanser.

Notice that the force per power input reported by  Fearn, Zachar, Woodward & Wanser is several orders of magnitude lower than the "EM drives".  Actually it is barely (3.5 times higher) more than the force per power input of a photon rocket:

reported measurement ForcePerPowerInput (milliNewtons/kW)

(* Cannae Superconducting *)             761.9 to 952.4
(* Shawyer Demo *)                               80 to 243
(* Shawyer Experimental *)                   18.82
(* Brady c TE mode *)                             21.31
(* Brady a TM mode*)                               5.396
(* Brady b TM mode*)                               3.000
(*Fearn, Zachar, Woodward & Wanser*) 0.01176




lengths in meter
rfFrequency in 1/second (microwave frequency during test)
power in watts
force in milliNewtons
force per PowerInput in milliNewtons/kW
c= 299705000 m/s (speed of light in air)
c= 299792458 m/s (speed of light in vacuum) (for Cannae Superconducting)
(the difference between c in air compared to c in vacuum is negligible)


Note: SmallDiameter for Shawyer's EM Drives obtained from his reported ShawyerDesignFactor .


Force/PowerInput of a Photon Rocket = 1 / c


(* Cannae Superconducting *)
rfFrequency = 1.047*10^9;
cavityLength = 0.01+0.004+0.006+0.01 = 0.03;
bigDiameter =(22.86-2*(0.00430)) = 0.220;
smallDiameter = bigDiameter-2*0.01=0.200;

power =  10.5
Q = 1.1*(10^7)

measured force = 8 to 10
measured ForcePerPowerInput = 761.9 to 952.4
Force/PowerInput of a Photon Rocket =0.003336
measured ForcePerPowerInput to the one of a photon rocket = 228,400 to 285,500


(* Shawyer Experimental *)
rfFrequency=2.45*10^9;
cavityLength=0.156;
bigDiameter=0.16;
smallDiameter=0.127546;

power =  850   
Q = 5900

measured force = 16
measured ForcePerPowerInput = 18.82
Force/PowerInput of a Photon Rocket =0.003337
measured ForcePerPowerInput to the one of a photon rocket =5,640


(* Shawyer Demo *)
rfFrequency=2.45*10^9;
cavityLength=0.345;
bigDiameter=0.28;
smallDiameter= 0.128853

power =  421 to 1200
Q = 45000

(measured force = 102.30 milliNewtons only reported for  421 watts, 243 milliNewtons/kW )

measured ForcePerPowerInput = 80 to 243
Force/PowerInput of a Photon Rocket =0.003337
measured ForcePerPowerInput to the one of a photon rocket =23,980 to 72,830

All Brady cases have the following dimensions:

cavityLength=0.332;
bigDiameter=0.397;
smallDiameter=0.244;


(* Brady a TM mode*)
rfFrequency=1.9326*10^9;

power =   16.9 
Q = 7320

measured force =  0.0912
measured ForcePerPowerInput = 5.396
Force/PowerInput of a Photon Rocket =0.003337
measured ForcePerPowerInput to the one of a photon rocket =1,617.2


(* Brady b TM mode*)   
rfFrequency=1.9367*10^9;

power = 16.7
Q =  18100

measured force = 0.0501
measured ForcePerPowerInput = 3.000
Force/PowerInput of a Photon Rocket =0.003337
measured ForcePerPowerInput to the one of a photon rocket =899.12


  (* Brady c  TE mode *)
rfFrequency = 1.8804*10^9;

power = 2.6
Q = 22000

measured force = 0.05541
measured ForcePerPowerInput = 21.31
Force/PowerInput of a Photon Rocket =0.003337
measured ForcePerPowerInput to the one of a photon rocket =6,386.7


(* Fearn, Zachar, Woodward & Wanser*)
rfFrequency = 39,300;

power =  170

measured force = 0.002
measured ForcePerPowerInput = 0.01176
Force/PowerInput of a Photon Rocket =0.003337
measured ForcePerPowerInput to the one of a photon rocket = 3.526

While the force per power input is smaller, at least the ME work seems to be much furhter along. After skimming the newest paper I also noticed that there is an additional paper that was published this year focusing on the experimental results.

http://www.researchgate.net/profile/Heidi_Fearn/publication/264040874_EXP-tests13/links/00b4953c9a2675bcc0000000?ev=pub_ext_doc_dl&origin=publication_list&inViewer=true (http://www.researchgate.net/profile/Heidi_Fearn/publication/264040874_EXP-tests13/links/00b4953c9a2675bcc0000000?ev=pub_ext_doc_dl&origin=publication_list&inViewer=true)

Only half way through reading this paper but it seems like more more effort was given to potential spurious forces, compared to the published information we have available for the EM Drives.
Title: Re: EM Drive Developments - related to space flight applications - Thread 2
Post by: Star One on 12/14/2014 08:51 pm
I'm glad to see the discussion continued in this thread.  I must admit, I felt disappointed at the prospect of losing this resource and the small band of interested theorists and experimentalists exploring what may appear to be the impossible, but which may ultimately result in profound space flight applications.

Might I suggest, that if this forum turns out not to be the right place in the future (e.g., if the thread is removed again), that all those who maintain an interest in the topic to congregate in the http://www.reddit.com/r/emdrive forum.  In fact, I suggest that you bookmark that page now for future reference.

I too do not appreciate personal attacks, which have no place in this kind of discussion, and so I understand the desire to keep the thread clean.  On the other hand, this topic is too important not to keep alive.
Can I just thank you for providing that link as it appears to be a good source of information on this topic.
Title: Re: EM Drive Developments - related to space flight applications - Thread 2
Post by: HMXHMX on 12/15/2014 03:52 am
"With more focus on space flight applications " (as per http://forum.nasaspaceflight.com/index.php?topic=36313.msg1301658#msg1301658) the metric chosen by the NASA's "Anomalous" report was the thrust force per power input.


Here is a comparison of reported measurements for EM Drives and for the latest report by Fearn, Zachar, Woodward & Wanser.

Notice that the force per power input reported by  Fearn, Zachar, Woodward & Wanser is several orders of magnitude lower than the "EM drives".  Actually it is barely (3.5 times higher) more than the force per power input of a photon rocket:

reported measurement ForcePerPowerInput (milliNewtons/kW)

(* Cannae Superconducting *)             761.9 to 952.4
(* Shawyer Demo *)                               80 to 243
(* Shawyer Experimental *)                   18.82
(* Brady c TE mode *)                             21.31
(* Brady a TM mode*)                               5.396
(* Brady b TM mode*)                               3.000
(*Fearn, Zachar, Woodward & Wanser*) 0.01176



...

Quite true.  "And yet, it moves."  :)
Title: Re: EM Drive Developments - related to space flight applications - Thread 2
Post by: Rodal on 12/16/2014 01:59 pm
....
While I was sceptic and still am, I learned a lot of interesting things from the previous thread, and enjoyed it even with its defaults.
It's great to have you posting again here, Frobnicat, as your always thoughtfully scientific and frobnicating posts have been one of the main attractions of this thread.

Welcome back, frobnicat !

I hope that Mulletron & all the others return to post here again as well,  and sooner rather than later !
Title: Re: EM Drive Developments - related to space flight applications - Thread 2
Post by: D_Dom on 12/18/2014 02:35 pm
Excellent stuff indeed, I will be perusing the previous thread in an attempt to better understand the logic. Easy enough for me to ignore the nonsense. Hoping we see more evidence from the many experiments underway to fuel the discussion. Many thanks to all the contributing members, truly a thought provoking thread.
Title: Re: EM Drive Developments - related to space flight applications - Thread 2
Post by: Raj2014 on 12/18/2014 07:46 pm
What is the latest news on the EM Drive? What are they planning to do next? If the EM Drive does work, this will be amazing, space exploration will change.
Title: Re: EM Drive Developments - related to space flight applications - Thread 2
Post by: frobnicat on 12/19/2014 10:43 am
May I suggest that people interested in "Mach effect" (reality of which having obvious consequences on directions advanced spaceflight can take) contribute on a thread with a more appropriate explicit topic (and historical content) : Propellantless Field Propulsion and application. The risk in splitting the somewhat frozen propellantless discussion of late would be to make it even harder to revive some activity, but EM and ME approaches are two very different kind of theories and devices, interwining of both topic discussion in a single thread was one of the reason of the chaos that plagued the 1st EM drive thread.

Anyhow, for those interested in Mach effect, here are two versions of results from attempt of Woodward (2008...) at "proof of effect" exhumed from Propellantless thread:
http://forum.nasaspaceflight.com/index.php?action=dlattach;topic=13020.0;attach=260412
http://forum.nasaspaceflight.com/index.php?action=dlattach;topic=13020.0;attach=260481

Haven't read in detail yet.
I won't crosspost here my posts about that (if I have the courage to address some of it deeper)
http://forum.nasaspaceflight.com/index.php?topic=13020.msg1304798#msg1304798
Title: Re: EM Drive Developments - related to space flight applications - Thread 2
Post by: Rodal on 12/19/2014 01:53 pm
May I suggest that people interested in "Mach effect" (reality of which having obvious consequences on directions advanced spaceflight can take) contribute on a thread with a more appropriate explicit topic (and historical content) : Propellantless Field Propulsion and application. The risk in splitting the somewhat frozen propellantless discussion of late would be to make it even harder to revive some activity, but EM and ME approaches are two very different kind of theories and devices, interwining of both topic discussion in a single thread was one of the reason of the chaos that plagued the 1st EM drive thread.

Anyhow, for those interested in Mach effect, here are two versions of results from attempt of Woodward (2008...) at "proof of effect" exhumed from Propellantless thread:
http://forum.nasaspaceflight.com/index.php?action=dlattach;topic=13020.0;attach=260412
http://forum.nasaspaceflight.com/index.php?action=dlattach;topic=13020.0;attach=260481

Haven't read in detail yet.
I won't crosspost here my posts about that (if I have the courage to address some of it deeper)
http://forum.nasaspaceflight.com/index.php?topic=13020.msg1304798#msg1304798

Good points, frobnicat, besides the PropellantLess thread:

http://forum.nasaspaceflight.com/index.php?topic=13020.0

the Woodward effect fans have had their own threads @ NASASpaceFlight, for example this one, completely dedicated to the Woodward effect:

http://forum.nasaspaceflight.com/index.php?topic=31037.0

The reported "thrust" force/(power input) of the latest Woodward experiments (Fearn, Zachar, Woodward & Wanser) is about 2000 times less than the NASA Brady et.al TE mode and about 20,000 times less than the Shawyer Demo

In other words, it takes (for the latest Woodward experiments) 3 to 4 orders of magnitude greater input power to produce the same level of what is reported as "thrust" force

Title: Re: EM Drive Developments - related to space flight applications - Thread 2
Post by: birchoff on 12/19/2014 11:31 pm
May I suggest that people interested in "Mach effect" (reality of which having obvious consequences on directions advanced spaceflight can take) contribute on a thread with a more appropriate explicit topic (and historical content) : Propellantless Field Propulsion and application. The risk in splitting the somewhat frozen propellantless discussion of late would be to make it even harder to revive some activity, but EM and ME approaches are two very different kind of theories and devices, interwining of both topic discussion in a single thread was one of the reason of the chaos that plagued the 1st EM drive thread.

Anyhow, for those interested in Mach effect, here are two versions of results from attempt of Woodward (2008...) at "proof of effect" exhumed from Propellantless thread:
http://forum.nasaspaceflight.com/index.php?action=dlattach;topic=13020.0;attach=260412
http://forum.nasaspaceflight.com/index.php?action=dlattach;topic=13020.0;attach=260481

Haven't read in detail yet.
I won't crosspost here my posts about that (if I have the courage to address some of it deeper)
http://forum.nasaspaceflight.com/index.php?topic=13020.msg1304798#msg1304798

Good points, frobnicat, besides the PropellantLess thread:

http://forum.nasaspaceflight.com/index.php?topic=13020.0

the Woodward effect fans have had their own threads @ NASASpaceFlight, for example this one, completely dedicated to the Woodward effect:

http://forum.nasaspaceflight.com/index.php?topic=31037.0

The reported "thrust" force/(power input) of the latest Woodward experiments (Fearn, Zachar, Woodward & Wanser) is about 2000 times less than the NASA Brady et.al TE mode and about 20,000 times less than the Shawyer Demo

In other words, it takes (for the latest Woodward experiments) 3 to 4 orders of magnitude greater input power to produce the same level of what is reported as "thrust" force

Why does this matter? I was under the impression that while both lines of inquiry hold incredible promise. They still require more research and testing to get to the point where a wider audience would be willing to accept them.
Title: Re: EM Drive Developments - related to space flight applications - Thread 2
Post by: Rodal on 12/19/2014 11:53 pm

The reported "thrust" force/(power input) of the latest Woodward experiments (Fearn, Zachar, Woodward & Wanser) is about 2000 times less than the NASA Brady et.al TE mode and about 20,000 times less than the Shawyer Demo

In other words, it takes (for the latest Woodward experiments) 3 to 4 orders of magnitude greater input power to produce the same level of what is reported as "thrust" force

Why does this matter? I was under the impression that while both lines of inquiry hold incredible promise. They still require more research and testing to get to the point where a wider audience would be willing to accept them.

As Chris Bergin made clear, the purpose of this forum is to deal with SPACEFLIGHT APPLICATIONS (of EM Drives: those propellantless drives comprised of a microwave cavity as the devices tested by Shawyer in the UK, Prof. Juan Yang in China and Brady, March, White, et.al. at NASA):

Quote from: Chris Bergi
focus on space flight applications to ensure this can have a healthy home here.

It matters because the amount of power required to produce a given level of thrust is of vital importance for spaceflight applications

The fact that  the latest Woodward experiments (Fearn, Zachar, Woodward & Wanser) show that it takes 20,000 times more power to produce a given level of thrust than the power required to produce the same thrust for the EM Drive (Shawyer demo) is extremely important for spaceflight applications.
Title: Re: EM Drive Developments - related to space flight applications - Thread 2
Post by: birchoff on 12/20/2014 01:59 am

The reported "thrust" force/(power input) of the latest Woodward experiments (Fearn, Zachar, Woodward & Wanser) is about 2000 times less than the NASA Brady et.al TE mode and about 20,000 times less than the Shawyer Demo

In other words, it takes (for the latest Woodward experiments) 3 to 4 orders of magnitude greater input power to produce the same level of what is reported as "thrust" force

Why does this matter? I was under the impression that while both lines of inquiry hold incredible promise. They still require more research and testing to get to the point where a wider audience would be willing to accept them.

As Chris Bergin made clear, the purpose of this forum is to deal with SPACEFLIGHT APPLICATIONS (of EM Drives: those propellantless drives comprised of a microwave cavity as the devices tested by Shawyer in the UK, Prof. Juan Yang in China and Brady, March, White, et.al. at NASA):

Quote from: Chris Bergi
focus on space flight applications to ensure this can have a healthy home here.

It matters because the amount of power required to produce a given level of thrust is of vital importance for spaceflight applications

The fact that  the latest Woodward experiments (Fearn, Zachar, Woodward & Wanser) show that it takes 20,000 times more power to produce a given level of thrust than the power required to produce the same thrust for the EM Drive (Shawyer demo) is extremely important for spaceflight applications.

I get that efficiency matters. What ever propulsion device yields the highest thrust for the least amount of input energy is a superior device. That said since you also wrote a critique of the Eagle Works test showing that the results could be thermal effects I would think the Thrust figures of either EM Drives or MET thrusters would be consider as simply a measure of how far both approaches have to go to be useful. It could be that a large part of the EM Drive thrust numbers are thermal and will be reduced when tested in vacuum. It could also be that the MET results are small only because of the materials in use. It could also be that neither of these things will ever work. My point being, I do not see how the thrust figures as they exist today can be used as a useful indicator of which area of research should get preferential treatment. They are simply mile markers that cannot be easily compared between the two types of propulsion ideas.
Title: Re: EM Drive Developments - related to space flight applications - Thread 2
Post by: aceshigh on 12/20/2014 01:08 pm

The reported "thrust" force/(power input) of the latest Woodward experiments (Fearn, Zachar, Woodward & Wanser) is about 2000 times less than the NASA Brady et.al TE mode and about 20,000 times less than the Shawyer Demo

In other words, it takes (for the latest Woodward experiments) 3 to 4 orders of magnitude greater input power to produce the same level of what is reported as "thrust" force

Why does this matter? I was under the impression that while both lines of inquiry hold incredible promise. They still require more research and testing to get to the point where a wider audience would be willing to accept them.

As Chris Bergin made clear, the purpose of this forum is to deal with SPACEFLIGHT APPLICATIONS (of EM Drives: those propellantless drives comprised of a microwave cavity as the devices tested by Shawyer in the UK, Prof. Juan Yang in China and Brady, March, White, et.al. at NASA):

Quote from: Chris Bergi
focus on space flight applications to ensure this can have a healthy home here.

It matters because the amount of power required to produce a given level of thrust is of vital importance for spaceflight applications

The fact that  the latest Woodward experiments (Fearn, Zachar, Woodward & Wanser) show that it takes 20,000 times more power to produce a given level of thrust than the power required to produce the same thrust for the EM Drive (Shawyer demo) is extremely important for spaceflight applications.

question is: are those thrust to power rations result of the physics itself?

if they find a better material than PZT stacks to test Mach Effects, suddenly, the efficiency may be much higher.

to what part the thrust to power ratio of each system is an engineering or a physics problem?
Title: Re: EM Drive Developments - related to space flight applications - Thread 2
Post by: Rodal on 12/20/2014 04:00 pm
As Chris Bergin made clear, the purpose of this forum is to deal with SPACEFLIGHT APPLICATIONS (of EM Drives: those propellantless drives comprised of a microwave cavity as the devices tested by Shawyer in the UK, Prof. Juan Yang in China and Brady, March, White, et.al. at NASA):

Quote from: Chris Bergin
focus on space flight applications to ensure this can have a healthy home here.

Why is this prescription by Chris Bergin being ignored?

Although the latest Woodward experiments (Fearn, Zachar, Woodward & Wanser) show that it takes 20,000 times more power to produce a given level of thrust than the power required to produce the same thrust for the EM Drive (Shawyer demo), and therefore these experiments do not constitute "EM Drive Spaceflight Applications" (as instructed by Chris Bergin) Woodward fans persist on carrying a debate on this thread rather than their own Woodward thread.   

Frankly, I don't understand what is the goal being pursued here by Woodward-effect fans to insist to carry arguments concerning Woodward's theory and experiments in this thread. 

The curtailment of the previous EM Drive thread by this forum's moderator was due to disruptive discussions regarding the validity of the Woodward effect, including a negative review of Woodward's book by a physicist.

As remarked by Frobnicat,

Quote from: Frobnicat
May I suggest that people interested in "Mach effect" (reality of which having obvious consequences on directions advanced spaceflight can take) contribute on a thread with a more appropriate explicit topic (and historical content) :>> << As reminded by managing editor, NSF is about space flight. Given the turn taken by the thread, the admin intervention is quite understandable, and most of the content wasn't thrashed, thanks.  While the "space drives" in general are on topic here if proven, the reality of any effect at all (and with what possible backing theories) is still controversial, and this open question is more a matter of fundamental science and has interest in and by itself beyond space flight. Since discussing those theoretical and experimental aspects are only marginally appropriate on this forum then that could proceed "preemptively" at other more appropriate place. Otherwise some talks will be rather contrived ( how make comment sound "space flight relevant" while it's not directly )

As previously remarked, there is a thread in this forum exclusively dedicated to the Woodward effect:

http://forum.nasaspaceflight.com/index.php?topic=31037.0




If the Woodward fans are looking for subjects to debate in the Woodward thread, here are a few suggestions you can add to such a discussion:


1) Extremely small reported thrust force/power input of the latest Woodward experiments (20.000 times less than Shawyer's demo and only barely more than the thrust of military search light acting as a photon rocket)

2) Recall Woodward's own words on his methodology:

http://forum.nasaspaceflight.com/index.php?action=dlattach;topic=13020.0;attach=260412

Quote from: Woodward
After carrying out a variety of experiments designed to exploit the presumed existence of Mach effects to produce small thrusts that yielded varying results, the decision was taken to carry out an experiment to test for the existence of such effects in the simplest of all possible circumstances. The aim was to settle the question of the existence the effects. Underlying physics suggests that these effects should exist; but in some experiments the effects expected were either not present, or much smaller than predicted. Without experimental evidence that the predicted Mach effects in fact do exist, situations in which they are small or not present may be taken as evidence that Mach effects do not exist.

Due to the extremely small level of thrust/power input (20,000 times less than Shawyer's demo) I am personally not interested in pursuing this Woodward debate, because these experiments show a level of thrust/power input that is so small that it does not constitute "Spaceflight Applications"

If there would be Woodward effect experiments with a similar or higher thrust/power-input as the Shawyer demo, I would be interested in pursuing this.  My approach then would be to model the thermo-mechanical and piezoelectric stresses in the Woodward experiment (for example with finite-element analysis) which -to my knowledge- has not been presented for the Woodward experiments.
Title: Re: EM Drive Developments - related to space flight applications - Thread 2
Post by: Rodal on 12/20/2014 04:13 pm
...That said since you also wrote a critique of the Eagle Works test showing that the results could be thermal effects I would think the Thrust figures of either EM Drives or MET thrusters would be consider as simply a measure of how far both approaches have to go to be useful. It could be that a large part of the EM Drive thrust numbers are thermal and will be reduced when tested in vacuum. ...

The technical paper I wrote concerned a thermo-mechanical effect that can explain the NASA Eagleworks truncated cone experimental results for the TE (transverse electric) mode both quantitatively and the initial impulsive rise time-wise response.  I showed that it is incorrect to posit (as some have done) that a thermal effect cannot produce an impulsive-like rise in the same time-frame as the NASA Eagleworks experiments.

Please also notice that it is incorrect to posit the following (if this was written in reference to my paper)

Quote
It could be that a large part of the EM Drive thrust numbers are thermal and will be reduced when tested in vacuum.

Because the thermo-mechanical effect I addressed in my technical paper does not at all need an atmosphere to take place.  As remarked in my paper, the same results would be expected in a vacuum.




Besides the effect discussed in my technical paper regarding the NASA Eagleworks truncated cone TE experiments, the following paper by Oak Ridge Lab:

http://web.ornl.gov/~webworks/cppr/y2001/pres/111404.pdf

addresses yet another thermal effect, specifically discussing the Woodward/Mahood experiments, which also can take place in a vacuum and does not need an atmosphere to take place.




A thermal effect that can take place in a vacuum that (as far as I know) has not been adequately explored (through numerical analysis) to explain the extremely small thrust/power-input results of the latest Fearn et.al experiments is the type of thermal recoil due to thermal radiation effect that has explained the Pioneer anomaly:

http://journals.aps.org/prl/abstract/10.1103/PhysRevLett.108.241101

Title: Re: EM Drive Developments - related to space flight applications - Thread 2
Post by: birchoff on 12/20/2014 10:09 pm
@Rodal,

Thank you for the clarification. That said I must say I am very cloudy on how anyone can have any interesting "Space Flight Applications" discussions about either of these technologies. Mainly because I am assuming that when someone uses the term "Space Flight Applications" they are talking about how these technologies can be used in different mission profiles. If that is the case then it doesnt matter what the technology is, the only thing that matters is the performance of the technology. Which would lead me to ask why even bother discussing any technology. Why not only have each thread in this forum focusing on a specific performance profile?

I have no objections to moving discussions about Mach Effect propulsion to another thread that is focused on just Mach Effects. The only reason I inquired about your comment regarding thrust comparisons is because given what we know, neither of them are ready for application today or even in the short term.

Anyway, I shall return to my lurker's seat while I await more EmDrive results.
Title: Re: EM Drive Developments - related to space flight applications - Thread 2
Post by: Avron on 12/20/2014 10:21 pm
Some things don't scale - Woodward is one. Another with a much better chance is the Sagnac effect, the only challenge is getting anything useful out of them. At a larger scale it would be awesome if Stirling engines would scale.

EmDrive will stay in the lab
Title: Re: EM Drive Developments - related to space flight applications - Thread 2
Post by: IslandPlaya on 12/20/2014 10:50 pm
Some things don't scale - Woodward is one. Another with a much better chance is the Sagnac effect, the only challenge is getting anything useful out of them. At a larger scale it would be awesome if Stirling engines would scale.

EmDrive will stay in the lab

I will avoid the obvious joke about the fact that your insight doesn't scale...
Woodward effect... And yet it moves! (HMXHMX I think)
Sagnac effect... hmmm
By Stirling engines I presume you mean ASRG. It will never provide multi-KW-MW power. It doesn't scale.
You claim to have knowledge of scaling of systems. It is quite apparent you do not.
Title: Re: EM Drive Developments - related to space flight applications - Thread 2
Post by: Rodal on 12/21/2014 12:13 am
@Birchoff

Thank you for your comments.  If this was my forum, I would also prefer to discuss all these propellant-less drives together, including their R&D and theoretical foundations; I miss Mulletron & others' discussions.  However, this is not our house; we are just guests.  Things got very rowdy, after which  the host closed the door for a few days and warned us that we are welcome here only under these conditions.   Otherwise the host will close the door again.
Title: Re: EM Drive Developments - related to space flight applications - Thread 2
Post by: ThinkerX on 12/21/2014 07:23 am
First, thanks to Chris for opening this new thread.

That said, some definitions might be in order to avoid confusion.

Stirling Engine:  The version I am familiar with is proven old line technology, essentially a temperature difference engine.   I have seen DIY versions on You-Tube and elsewhere, and read of others that can act as low power solar generators, but have no idea how this might pertain to spaceflight.  What is 'ASRG?'

Sagnac effect.  I have not heard of this before. 
Title: Re: EM Drive Developments - related to space flight applications - Thread 2
Post by: IslandPlaya on 12/21/2014 06:15 pm
First, thanks to Chris for opening this new thread.

That said, some definitions might be in order to avoid confusion.

Stirling Engine:  The version I am familiar with is proven old line technology, essentially a temperature difference engine.   I have seen DIY versions on You-Tube and elsewhere, and read of others that can act as low power solar generators, but have no idea how this might pertain to spaceflight.  What is 'ASRG?'

Sagnac effect.  I have not heard of this before.
ASRG is advanced Stirling radioisotope generator. It is purposed to generate more power than the traditional RTG at the cost of having moving parts.
The MSL rover has an RTG capable of 100W. This is the current state-of-the-art wrt RTG's.
Sagnac effect wrt to space flight is bunkum, IMHO...
Title: Re: EM Drive Developments - related to space flight applications - Thread 2
Post by: ThinkerX on 12/21/2014 09:55 pm
Ok, did some googling.

First, to clarify things:  I live in Alaska, right at the edge of the wilderness.  I own property in the wilderness.  Some years ago, I was contemplating putting a cabin on that property - until my back told the rest of me in no uncertain terms that wasn't going to work.  Prior to that, though, I did some research on power generation, which is how I came to be aware of Stirling Engines.  The ones I was looking at were small - about the size of a larger bicycle pump, and not much more complex.  Given a heat source, they could generate electricity, but not much - but then again, they didn't need much of a heat source.  I figured just one wasn't going to do the trick.  Then I thought about it, and realized as small as they were, I could use the same heat source (wood stove) to power maybe six or eight of the things, hence generating six or eight times the power. 

That said, to me ASRG looks both doable and noncontroversial.  I don't grasp how it could be used for spaceflight propulsion, but it should be adequate for a Martian rover, automated station, or even a small manned installation. 

The Sagnac effect also looks like proven technology, useful for determining terrestrial location.  To my mind, its essentially a way of 'tricking light via rotation.'  This gives me half an idea relevant to the EM drive:

Suppose the whole copper frustum was rotating very, very, fast on its axis, AND you had the exterior (?) wrapped in about a quarter mile worth of wire (two wires)?  Seems to me that might be enough to produce an asymmetry in this system - though I have no idea whether that would be a useful asymmetry.
Title: Re: EM Drive Developments - related to space flight applications - Thread 2
Post by: frobnicat on 12/21/2014 11:01 pm
So, curious about this mention of Sagnac effect by Avron and relation to Em-drives, there is a short sentence at end of http://emdrive.com/principle.html :
Quote from: emdrive.com
...the system of EM wave and waveguide can be regarded as an open system, with the EM wave and the waveguide having separate frames of reference.
A similar approach is necessary to explain the principle of the laser gyroscope, where open system attitude information is obtained from an apparently closed system device.

For what it's worth, a comment on Slashdot (http://science.slashdot.org/story/13/02/08/0030240/chinas-radical-new-space-drive) (search Sagnac) about Wired article (http://www.wired.co.uk/news/archive/2013-02/06/emdrive-and-cold-fusion) about 2012 publication of Yang Juan (http://www.emdrive.com/yang-juan-paper-2012.pdf) states :
Quote from: joe_frisch
There is a description of the Sagnac effect on wikipedia, this is the basis of a laser gyroscope. Interestingly Newtonian physics and relativity give the same answer for this. I isn't related to the microwave drive. I think they mention it because laser gyroscopes are conceptually complicated and they hope that the reader won't understand them, and therefor not understand that if anything they are yet more evidence that this trick doesn't work.
Uncorrected for spelling. The comment is about another post musing about the last sentence of emdrive.com principle page about laser gyroscope.

So, is there anything published with some level of detail explaining how a hypothetical "space drive effect", and with "good scaling" at that, could be derived or elucidated from the proven physics of Sagnac effect ?
Title: Re: EM Drive Developments - related to space flight applications - Thread 2
Post by: Econocritic on 12/22/2014 01:15 am
http://www.mehtapress.com/mehtapress/Journals/Journal-of-Space-Exploration/Volume-3-Issue-1/vol_3_issue_1_file_2.pdf

I'm not qualified to judge the space flight applicability of the above article by Dr. Woodward as it pertains to this forum.  A brief history,  theory and recent experiments are detailed. It also contains a critique of quantum vacuum explanations for emdrive propulsion. Just thought I should bring it to this groups attention and I'll return to lurking. I've passively appreciated this discussion for a long time.  Some of the most contentious moments have been the most educational.  I'll miss the perspectives of Ron Stahl and Mullertron assuming they don't return. Thanks to everyone who has contributed. Truly fascinating.
Title: Re: EM Drive Developments - related to space flight applications - Thread 2
Post by: Notsosureofit on 12/22/2014 01:17 am
There is some similarity of the EMDrive to the Sagnac oscillator if the optical fiber has a linear variation in the index of refraction.  The frames of reference are stationary and accelerating rather than constant velocity.  But, I havn't seen any reference or calculation giving rotary forces as yet.  At the moment I'm chasing old photon to graviton papers. 
Title: Re: EM Drive Developments - related to space flight applications - Thread 2
Post by: Notsosureofit on 12/22/2014 01:40 pm
There is some similarity of the EMDrive to the Sagnac oscillator if the optical fiber has a linear variation in the index of refraction.  The frames of reference are stationary and accelerating rather than constant velocity.  But, I havn't seen any reference or calculation giving rotary forces as yet.  At the moment I'm chasing old photon to graviton papers.

The Sagnac effect compares the (rotating) velocity frame of reference in which the opposing photons show the same frequency w/ the (stationary) frame in which the doppler photons are observed.

The Shawyer cavity photons can be compared in the (stationary) dispersion frame w/ the same frequency and in an (accelerated) frame which balances out the dispersion and expresses the doppler shifts.

Title: Re: EM Drive Developments - related to space flight applications - Thread 2
Post by: Rodal on 12/22/2014 04:00 pm
There is some similarity of the EMDrive to the Sagnac oscillator if the optical fiber has a linear variation in the index of refraction.  The frames of reference are stationary and accelerating rather than constant velocity.  But, I havn't seen any reference or calculation giving rotary forces as yet.  At the moment I'm chasing old photon to graviton papers.

The Sagnac effect compares the (rotating) velocity frame of reference in which the opposing photons show the same frequency w/ the (stationary) frame in which the doppler photons are observed.

The Shawyer cavity photons can be compared in the (stationary) dispersion frame w/ the same frequency and in an (accelerated) frame which balances out the dispersion and expresses the doppler shifts.

A classic (1967, Air Force Cambridge Research Lab , Massachusetts) article on the Sagnac effect, by Post:

http://www.orgonelab.org/EtherDrift/Post1967.pdf

Quote
Post admits in his great review article: “The search for a physically meaningful transformation for rotation
is not aided in any way whatever by the principle of general space-time covariance (relativity), nor is it true that the space-time theory of gravitation (general relativity) plays any role in establishing physically correct transformations (relevant to the Sagnac effect).”
(parenthesis and bold added for explanatory purposes)

For a contentious viewpoint, here is a paper claiming interferometric measurements validate the classical approach and invalidate the relativistic approach (W. Engelhardt, Division Head, Wissenschatlicher Angestellter JET, Max-Planck-Institut für Physik)

http://www.kritik-relativitaetstheorie.de/Anhaenge/Wolfgang-Engelhardt-Sagnac.pdf

Quote
In 1925 Michelson and Gale built a huge earth-fixed Sagnac Interferometer in Illinois demonstrating that the light velocity is anisotropic on the rotating earth. For Sagnac this result did not come as a surprise having explained the underlying effect on the basis of the ether theory in 1913. The Special Relativity Theory (SRT), however, had predicted on the basis of the Lorentz Transformation (LT) that the velocity of light is isotropic in all inertial systems

Title: Re: EM Drive Developments - related to space flight applications - Thread 2
Post by: aceshigh on 12/22/2014 04:16 pm
http://www.mehtapress.com/mehtapress/Journals/Journal-of-Space-Exploration/Volume-3-Issue-1/vol_3_issue_1_file_2.pdf

I'm not qualified to judge the space flight applicability of the above article by Dr. Woodward as it pertains to this forum.  A brief history,  theory and recent experiments are detailed. It also contains a critique of quantum vacuum explanations for emdrive propulsion. Just thought I should bring it to this groups attention and I'll return to lurking. I've passively appreciated this discussion for a long time.  Some of the most contentious moments have been the most educational.  I'll miss the perspectives of Ron Stahl and Mullertron assuming they don't return. Thanks to everyone who has contributed. Truly fascinating.

thanks Econocritic, but Rodal just chastised us for posting about Woodward's Mach Effect theory in this thread, at post #32
Quote from: Rodal
the purpose of this forum is to deal with SPACEFLIGHT APPLICATIONS (of EM Drives: those propellantless drives comprised of a microwave cavity as the devices tested by Shawyer in the UK, Prof. Juan Yang in China and Brady, March, White, et.al. at NASA)

...

and therefore these experiments do not constitute "EM Drive Spaceflight Applications" (as instructed by Chris Bergin) Woodward fans persist on carrying a debate on this thread rather than their own Woodward thread.   

Frankly, I don't understand what is the goal being pursued here by Woodward-effect fans to insist to carry arguments concerning Woodward's theory and experiments in this thread. 

He is right about that. This is the EM Drive thread. And my guess is that what he wrote was valid for everyone of us, including new members like you.

So I think you should post your link in another topic, probably the Propellantless Field Propulsion and Applications thread...
http://forum.nasaspaceflight.com/index.php?topic=13020.1830
Title: Re: EM Drive Developments - related to space flight applications - Thread 2
Post by: D_Dom on 12/22/2014 05:30 pm
Woodward discussion thread here;
http://forum.nasaspaceflight.com/index.php?topic=31037.465
Somewhat highjacked but may be returned to the original topic.
Title: Re: EM Drive Developments - related to space flight applications - Thread 2
Post by: Notsosureofit on 12/23/2014 03:03 pm
There is some similarity of the EMDrive to the Sagnac oscillator if the optical fiber has a linear variation in the index of refraction.  The frames of reference are stationary and accelerating rather than constant velocity.  But, I havn't seen any reference or calculation giving rotary forces as yet.  At the moment I'm chasing old photon to graviton papers.

The Sagnac effect compares the (rotating) velocity frame of reference in which the opposing photons show the same frequency w/ the (stationary) frame in which the doppler photons are observed.

The Shawyer cavity photons can be compared in the (stationary) dispersion frame w/ the same frequency and in an (accelerated) frame which balances out the dispersion and expresses the doppler shifts.

A classic (1967, Air Force Cambridge Research Lab , Massachusetts) article on the Sagnac effect, by Post:

http://www.orgonelab.org/EtherDrift/Post1967.pdf

Quote
Post admits in his great review article: “The search for a physically meaningful transformation for rotation
is not aided in any way whatever by the principle of general space-time covariance (relativity), nor is it true that the space-time theory of gravitation (general relativity) plays any role in establishing physically correct transformations (relevant to the Sagnac effect).”
(parenthesis and bold added for explanatory purposes)

For a contentious viewpoint, here is a paper claiming interferometric measurements validate the classical approach and invalidate the relativistic approach (W. Engelhardt, Division Head, Wissenschatlicher Angestellter JET, Max-Planck-Institut für Physik)

http://www.kritik-relativitaetstheorie.de/Anhaenge/Wolfgang-Engelhardt-Sagnac.pdf

Quote
In 1925 Michelson and Gale built a huge earth-fixed Sagnac Interferometer in Illinois demonstrating that the light velocity is anisotropic on the rotating earth. For Sagnac this result did not come as a surprise having explained the underlying effect on the basis of the ether theory in 1913. The Special Relativity Theory (SRT), however, had predicted on the basis of the Lorentz Transformation (LT) that the velocity of light is isotropic in all inertial systems

After reading these papers my conclusions are that you can dismiss the second one as looking to disprove relativity.  The 1967 paper is exhaustive and it shows decisively (I could see no errors in the math) that the closed path of the Sagnac effect is an example of a "closed system" even in the case of a dispersive medium.

Edit:  I suppose I should add that the EMDrive can be treated the same way(s) with the radius and area = to infinity (or the cosmological size ?) but the implications are not immediately obvious.

Edit2: Went back to check a few thoughts.  Nothing was considered that could give any relevant calculation to the EMDrive, unfortunately.  The only dispersion considered here would just cancel out.

Edit3: (the last ? for now)

By way of clarification, I've been following the proposition that the existance of photon dispersion in a (gravitational, for instance) accelerated frame of reference (AFR) can imply the generation of an acceleration in a stationary frame by the presence of a dispersed photon field and it's possible application to the EMDrive.

In practice, the dispersion relation is translated to an AFR in which the wavenumber dispersion disappears and the doppler shifted frequency dispersion can be used to calculate the force exerted per photon.

The required conditions for a cavity resonator would seem, so far, to be the assymetry of the cavity dispersion relation and the existance of higher order modes which exhibit "cutoff frequencies".

The 1967 paper above, and other papers I've looked at so far do not include calculations that meet those criteria.

The conservation of momentum is still to be rationalised.  So far, none of the electro-gravitational papers I've seen have had the interaction arise "naturally" out of General Relativity. (that may be my own predudice, so I'm still looking)

Thanks for your patience.
Title: Re: EM Drive Developments - related to space flight applications - Thread 2
Post by: Rodal on 12/24/2014 07:13 pm
.....

After reading these papers my conclusions are that you can dismiss the second one as looking to disprove relativity.  The 1967 paper is exhaustive and it shows decisively (I could see no errors in the math) that the closed path of the Sagnac effect is an example of a "closed system" even in the case of a dispersive medium.

Edit:  I suppose I should add that the EMDrive can be treated the same way(s) with the radius and area = to infinity (or the cosmological size ?) but the implications are not immediately obvious.

Edit2: Went back to check a few thoughts.  Nothing was considered that could give any relevant calculation to the EMDrive, unfortunately.  The only dispersion considered here would just cancel out.

Edit3: (the last ? for now)

By way of clarification, I've been following the proposition that the existance of photon dispersion in a (gravitational, for instance) accelerated frame of reference (AFR) can imply the generation of an acceleration in a stationary frame by the presence of a dispersed photon field and it's possible application to the EMDrive.

In practice, the dispersion relation is translated to an AFR in which the wavenumber dispersion disappears and the doppler shifted frequency dispersion can be used to calculate the force exerted per photon.

The required conditions for a cavity resonator would seem, so far, to be the assymetry of the cavity dispersion relation and the existance of higher order modes which exhibit "cutoff frequencies".

The 1967 paper above, and other papers I've looked at so far do not include calculations that meet those criteria.

The conservation of momentum is still to be rationalised.  So far, none of the electro-gravitational papers I've seen have had the interaction arise "naturally" out of General Relativity. (that may be my own predudice, so I'm still looking)

Thanks for your patience.
Thanks for the great comment and explanation  !

It is known that an electric charge emits radiation if there is a relative acceleration between the charge and its electric field.  And that emission of radiation produces a force.  Unequal radiation produced in opposite directions should result in a net force in one direction.  (In order to satisfy energy and momentum conservation, the charged particle must experience a recoil at the time of emission. The radiation must exert an additional force on the charged particle: the Abraham–Lorentz force)

This should exist both for a charge accelerated in a free space, as well for a charge supported at rest in an accelerated field, for example, a gravitational field. In both situations, the charges radiate.

But, if this is due to the acceleration of gravity, at first glance it seems like the charge would radiate in a dipolar power pattern that looks like an asymmetric radially-polarized doughnut (a dougnhut with a larger cross-section in one direction and a smaller cross-section in the opposite direction).  A doughnut whose axis is parallel to the direction of the gravity force, hence one should be able to discriminate this by orienting the EM Drive with the central axis of the truncated cone oriented like the force of gravity, and orienting it perpendicular to that direction.  NASA Eagleworks unfortunately did not orient the truncated cone with its longitudinal axis oriented in the vertical direction, parallel to the direction of the acceleration of gravity.  Dr. White said in the NASA Ames 2014 August Conference that he was planning to do this in the next series of trials.  However, Shawyer did orient his EM Drive in the vertical direction and did obtain similar results as in the horizontal direction, which would seem to preclude this explanation?

(http://www.mpl.mpg.de/fileadmin/leuchs/img/4pipac/doughnut.png)
Title: Re: EM Drive Developments - related to space flight applications - Thread 2
Post by: Notsosureofit on 12/25/2014 02:22 am
For any closed cavity the gravitational component is just the "weight" of the RF energy

W=PQahf/c^2

The Shawer effect seems to be something else which requires the dispersion relation to have a specific form.

(just had a minute to quip...more later)

Maxwell's equations are just the tautological relationship between the field and particle representations.

Yes, so much easy info to wait for.  My (Xmas) wish is that they looked for any new frequencies in the spectra. (probably not there but cavity modes are bound states and it would be good to look for transitions anyway)

Correction: W = Nahf/c^2 = (PQ/hf)*(ahf/c^2) = PQa/c^2

(I gotta find a way to get more than a minute free at a time !)
Title: Re: EM Drive Developments - related to space flight applications - Thread 2
Post by: ThinkerX on 12/27/2014 12:56 am
Another blog post from Professor McCulloch.  He links to a paper he just published describing how he believes the EM drive is a manifestation of his theory:

http://physicsfromtheedge.blogspot.com/
Title: Re: EM Drive Developments - related to space flight applications - Thread 2
Post by: Notsosureofit on 12/27/2014 12:26 pm
Another blog post from Professor McCulloch.  He links to a paper he just published describing how he believes the EM drive is a manifestation of his theory:

http://physicsfromtheedge.blogspot.com/

Certainly close enough to be interesting.  I would have liked to see simplified Eq. 6 filled out to include the effect of the cavity modes.
Title: Re: EM Drive Developments - related to space flight applications - Thread 2
Post by: Notsosureofit on 12/27/2014 12:41 pm
Awoke to memories of the '60s, ie. the Sachs-Schwebel version of GR using quaternions having an addl. coupling.  Found at least one ref. this AM.

mendelsachs-fromspecialtogeneralrelativity-macrotoquantumdomains28p-120114095137-phpapp02.pdf

"This expression predicts a coupling of the ‘gravitational field’ (in terms of qk) with the matter field components Tρ to define a gravitational current contribution. The latter is not foreseen in the conventional theories that neglect the gravitational coupling to matter fields."

That expression has a form that is at least "similar" in outline to that from the cavity dispersion relation.  The "gravitational current" might provide the missing momentum.

I need to find a way to get copies of the original papers which were in Il Nuovo Cimento as I recall.

Edit:  Looking for,


    Sachs, M. (1964).Nuovo cimento,31, 98.
    Sachs, M. (1968a).International Journal of Theoretical Physics, Vol. 1, No. 4, p. 387.
    Sachs, M. (1968b).Nuovo cimento,53A, 561.
    Sachs, M. and Schwebel, S. L. (1961).Nuovo cimento, Supplement21, 197.
    Sachs, M. and Schwebel, S. L. (1962).Journal of Mathematics and Physics,3, 843.
    Sachs, M. and Schwebel, S. L. (1963).Nuclear Physics,43, 204.

Also: Schwebel, S. L. INT J THEOR PHYS , vol. 6, no. 1, pp. 61-74, 1972, "Interaction theory of the electromagnetic field"


Title: Re: EM Drive Developments - related to space flight applications - Thread 2
Post by: Star One on 12/27/2014 06:39 pm
If you wanted to build a vehicle using this technology would there be a preferred shape to use?
Title: Re: EM Drive Developments - related to space flight applications - Thread 2
Post by: Rodal on 12/28/2014 12:39 am
Awoke to memories of the '60s, ie. the Sachs-Schwebel version of GR using quaternions having an addl. coupling.  Found at least one ref. this AM.

mendelsachs-fromspecialtogeneralrelativity-macrotoquantumdomains28p-120114095137-phpapp02.pdf

"This expression predicts a coupling of the ‘gravitational field’ (in terms of qk) with the matter field components Tρ to define a gravitational current contribution. The latter is not foreseen in the conventional theories that neglect the gravitational coupling to matter fields."

That expression has a form that is at least "similar" in outline to that from the cavity dispersion relation.  The "gravitational current" might provide the missing momentum.

I need to find a way to get copies of the original papers which were in Il Nuovo Cimento as I recall.

Edit:  Looking for,


    Sachs, M. (1964).Nuovo cimento,31, 98.
    Sachs, M. (1968a).International Journal of Theoretical Physics, Vol. 1, No. 4, p. 387.
    Sachs, M. (1968b).Nuovo cimento,53A, 561.
    Sachs, M. and Schwebel, S. L. (1961).Nuovo cimento, Supplement21, 197.
    Sachs, M. and Schwebel, S. L. (1962).Journal of Mathematics and Physics,3, 843.
    Sachs, M. and Schwebel, S. L. (1963).Nuclear Physics,43, 204.

I found this this paper that Sachs wrote under sponsorship from the Air Force Cambridge Research Lab in Trieste in July 1966 while on leave from B.U., "To be submitted to Nuovo Cimento"

"ON FACTORIZATION OF EINSTEIN'S FORMALISM INTO A PAIR OF QUATERNION FIELD EQUATIONS"

http://streaming.ictp.trieste.it/preprints/P/66/081.pdf

Quote from: M. Sachs
it might be remarked that the quaternion form of the metrical field equations lends itself in a natural way to a unification between the inertial and gravitational manifestations of interacting matter. This is because of the basic expression of the matter fields themselves in terms of the same spinor and quaternion variables.

Mendel Sachs has a website:

http://mendelsachs.com/


In his website he has posted several of his articles.  For example this relatively recent one on the Mach principle and origin of inertia:

http://mendelsachs.com/wp-content/uploads/articles/the-mach-principle.pdf

In that reference, Sachs convincingly argues against the approach to Mach's Principle followed by Woodward (-of course- he does not mention  Woodward), he considers the particle-antiparticle pairs of the quantum vacuum having a most important effect, while the effect of distant stars is negligible:


Quote from: Sachs
I have found in my research program in general relativity, that the primary contribution to the inertial mass of any local elementary matter, such as an ‘electron’, are the nearby particle-antiparticle pairs that constitute what we call the ‘physical vacuum’. [The main developments of this research are demonstrated in my two monographs: General Relativity and Matter, and Quantum Mechanics from General Relativity]. A prediction of this research program is that the main influence of these pairs on the mass of, say, an electron comes from a domain of the ‘physical vacuum’ in its vicinity, whose volume has a radius that is the order of 10^(-15) cm. Of course, the distant stars, billions of light-years away, also contribute to the electron’s mass, though negligibly, just as the Sun’s mass contribution to the weight of a person on Earth is negligible compared with the Earth’s influence on this person’s weight! Nevertheless, it was Mach’s contention that in principle all of the matter of the closed system – the nearby as well as far away constituents – determines the inertial mass of any local matter.
(Bold added for emphasis) ==> this is the anti-thesis of Woodward's approach to Mach's principle!

There are many other interesting references, for example this one on Dirac's Quantum Negative Energy Problem:

http://mendelsachs.com/publication/view/the-quantum-negative-energy-problem-revisited/
Title: Re: EM Drive Developments - related to space flight applications - Thread 2
Post by: Notsosureofit on 12/28/2014 03:33 am
Yes, thanks.  This is great stuff and will require a lot of reading.  (not to mention the brain-busting)
But do look back at that expression in the AM reference, it looks (to me w/o justification) like it could be the "doppler frame" expression.

Edit:  Found the AM paper online, (on the Beardon site, of all places!  I have NO idea what this would have to do w/ free energy.)

http://www.cheniere.org/references/Symmetry_in_Electrodynamics.pdf

 See p.24, the three-current density
Title: Re: EM Drive Developments - related to space flight applications - Thread 2
Post by: Mulletron on 12/28/2014 01:13 pm
So what TRL would you guys say the devices presented in the "Anomalous Thrust Production...." papers are at right now? I'd say probably a TRL 2. Possibly on the way to a TRL 3, hopefully.  :) I base this partly on Dr. White's comments at the 29:55 mark in the NASA Ames Research Director’s Colloquium, August 12, 2014 video posted here recently. He remarks that the Q-thruster concept is potentially higher TRL than the Warp concepts at least.

http://www.nasa.gov/content/technology-readiness-level/
http://en.wikipedia.org/wiki/Technology_readiness_level#NASA_definitions

The reason I'm asking is, what kind of actual real "space flight applications" can we discuss given what we know right now? What can we actually do right now in space with the numbers quoted below?

"With more focus on space flight applications " (as per http://forum.nasaspaceflight.com/index.php?topic=36313.msg1301658#msg1301658) the metric chosen by the NASA's "Anomalous" report was the thrust force per power input.

Here is a comparison of reported measurements for EM Drives.........

reported measurement ForcePerPowerInput (milliNewtons/kW)

(* Brady c TE mode *)                                     21.31
(* Brady a TM mode*)                              5.396
(* Brady b TM mode*)                              3.000


A quick online conversion of the 21 milliNewton figure gives me a 0.0755 ozf figure. That has to be worth something given that solar panels essentially supply electrical power for free (besides building/getting the space vehicle in orbit to begin with). So would this be useful for at least station keeping even now? Or could it power a small probe somewhere interesting, faster than the usual burn and coast method?

According to the oracle: http://en.wikipedia.org/wiki/Hall_effect_thruster
"Devices operating at 1.35 kW produce about 83 mN of thrust" "...hall effect thrusters have input power 1.35–10 kilowatts, exhaust velocity 10–50 kilometers per second, thrust 40–600 millinewtons and efficiency 45–60 percent."

That 21 milliNewton/KW figure is creeping up on what a Hall thruster can do, but with NO propellant.

Title: Re: EM Drive Developments - related to space flight applications - Thread 2
Post by: Rodal on 12/28/2014 02:52 pm
So what TRL would you guys say the devices presented in the "Anomalous Thrust Production...." papers are at right now?....

Hey Mulletron, a very warm WELCOME BACK !
We really missed you !
Glad that you are back
Title: Re: EM Drive Developments - related to space flight applications - Thread 2
Post by: Rodal on 12/28/2014 04:41 pm
Yes, thanks.  This is great stuff and will require a lot of reading.  (not to mention the brain-busting)
But do look back at that expression in the AM reference, it looks (to me w/o justification) like it could be the "doppler frame" expression.

Edit:  Found the AM paper online, (on the Beardon site, of all places!  I have NO idea what this would have to do w/ free energy.)

http://www.cheniere.org/references/Symmetry_in_Electrodynamics.pdf

 See p.24, the three-current density

Very interesting.  Also p. 21:
Quote from: Sachs
The role of the Mach principle is revealed at this stage of the analysis. Since Fρλ depends on the spin curvature tensor Kρλ, which automatically vanishes in a vacuum (i.e. a flat spacetime), the electromagnetic field, and therefore the previously considered electric charge of any quantity of matter in a vacuum must vanish. Thus, not only the inertial mass but also the electric charge of a ‘particle’ of matter does not exist when there is no coupling to other matter. I have generalized this idea in the field theory based on General Relativity, to the case where all previously considered intrinsic properties of discrete matter, in addition to inertial mass and electric charge, vanish identically in a vacuum. This view exorcises all of the remaining features of the discrete, separable ‘elementary particle’ of matter. It is replaced with a view of matter in terms of a closed, continuous field theory, according to the theory of general relativity. I have called this view of matter, whereby all of its previously considered intrinsic properties are explained in terms of coupling within the closed system, ‘the generalized Mach principle’.

Sachs formally answered (in the 1960's !) the criticism that the matter in Quantum Vacuum "does not gravitate": intrinsic properties are explained in terms of coupling within the closed system, they don't really exist (thus they are "virtual") in the vacuum per se, they only become into existence when there is coupling.
Title: Re: EM Drive Developments - related to space flight applications - Thread 2
Post by: birchoff on 12/29/2014 12:24 am
...

Sachs formally answered (in the 1960's !) the criticism that the matter in Quantum Vacuum "does not gravitate": intrinsic properties are explained in terms of coupling within the closed system, they don't really exist (thus they are "virtual") in the vacuum per se, they only become into existence when there is coupling.

Not sure I understand what your saying here. do you mind breaking that down?
Title: Re: EM Drive Developments - related to space flight applications - Thread 2
Post by: Notsosureofit on 12/29/2014 01:14 am
Hmmm, Schwebel had mentioned, at the time, that because of that, there was no need for renormalization, but because it had already been done, no one cared. 
Title: Re: EM Drive Developments - related to space flight applications - Thread 2
Post by: birchoff on 12/29/2014 02:35 am
...
In his website he has posted several of his articles.  For example this relatively recent one on the Mach principle and origin of inertia:

http://mendelsachs.com/wp-content/uploads/articles/the-mach-principle.pdf

In that reference, Sachs convincingly argues against the approach to Mach's Principle followed by Woodward (-of course- he does not mention  Woodward), he considers the particle-antiparticle pairs of the quantum vacuum having a most important effect, while the effect of distant stars is negligible:


Quote from: Sachs
I have found in my research program in general relativity, that the primary contribution to the inertial mass of any local elementary matter, such as an ‘electron’, are the nearby particle-antiparticle pairs that constitute what we call the ‘physical vacuum’. [The main developments of this research are demonstrated in my two monographs: General Relativity and Matter, and Quantum Mechanics from General Relativity]. A prediction of this research program is that the main influence of these pairs on the mass of, say, an electron comes from a domain of the ‘physical vacuum’ in its vicinity, whose volume has a radius that is the order of 10^(-15) cm. Of course, the distant stars, billions of light-years away, also contribute to the electron’s mass, though negligibly, just as the Sun’s mass contribution to the weight of a person on Earth is negligible compared with the Earth’s influence on this person’s weight! Nevertheless, it was Mach’s contention that in principle all of the matter of the closed system – the nearby as well as far away constituents – determines the inertial mass of any local matter.


(Bold added for emphasis) ==> this is the anti-thesis of Woodward's approach to Mach's principle!


So I read the paper expecting the supporting arguments for why Sach's claims that it is the matter closest that has the most important effect. Unfortunately I realized by the end that those arguments actually lie in his books. I read the descriptions of them on amazon and the reviews and I do not believe this is a show stopper for woodward. For the following reasons

* If Sach's is right then Mach's Principle is apart of Einsteins General Relativity theory. Which Woodward is also claiming.

* From my perspective (NOT A PHYSICIST) I get the impression that for the Mach Effects being claimed by woodward it isn't so much that we start with the farthest matter to determine the effects on local matter. Instead it is more important that the inertial mass of any local matter depends on all the matter in the universe.

* In the Quantum Mechanics from General Relativity (http://www.amazon.com/Quantum-Mechanics-General-Relativity-Approximation/dp/9027722471/ref=sr_1_4?s=books&ie=UTF8&qid=1419822072&sr=1-4) book. It looks like from the reviews that Sachs also proves the Feynman & Wheeler Absorber theory (http://en.wikipedia.org/wiki/Wheeler%E2%80%93Feynman_absorber_theory) (which is the "delayed action at a distance" being referenced by the customer review). This is important because it is also being used by Woodward to explain how inertial actions are instantaenous.

Quote from: Customer in review(the longest one)
...

The "delayed action at a distance"of Feynman and Wheeler is restored to currency. The "advanced" solutions take their place beside the "retarded" solutions in a single, complete space-time.

...

From my limited perspective I see two men looking at the same thing from different perspectives. And while I believe this will end up affecting the details. The question really is whose version is a more accurate representation of reality?


P.S. Yes I know this may sound like a ME fan boy defending his sacred cow,but I can guarantee you that I am just as much a fan boy of NASA's EmDrive research and eagerly awaiting more results to digest. That said without a thorough understanding of both Sach's and Woodwards theories the only way I can compare the two is to look at just how similar or different their line of reasoning is. And from where I stand the detail being pointed to seems small enough to not matter. Especially given the extensive experimental work Woodward and Paul March has done. The only questions I have for them in my mind is can they scale it up? What material science issue need to be resolved? And what is their timeline to get to a performance level applicable to micro gravity applications?
Title: Re: EM Drive Developments - related to space flight applications - Thread 2
Post by: birchoff on 12/29/2014 02:46 am
...
Sachs formally answered (in the 1960's !) the criticism that the matter in Quantum Vacuum "does not gravitate": intrinsic properties are explained in terms of coupling within the closed system, they don't really exist (thus they are "virtual") in the vacuum per se, they only become into existence when there is coupling.

so how does one create this coupling?
Title: Re: EM Drive Developments - related to space flight applications - Thread 2
Post by: Notsosureofit on 12/29/2014 01:33 pm
...
Sachs formally answered (in the 1960's !) the criticism that the matter in Quantum Vacuum "does not gravitate": intrinsic properties are explained in terms of coupling within the closed system, they don't really exist (thus they are "virtual") in the vacuum per se, they only become into existence when there is coupling.

so how does one create this coupling?

See:  http://link.springer.com/static-content/lookinside/986/art%253A10.1007%252FBF02755823/000.png
Title: Re: EM Drive Developments - related to space flight applications - Thread 2
Post by: birchoff on 12/29/2014 04:49 pm
...
Sachs formally answered (in the 1960's !) the criticism that the matter in Quantum Vacuum "does not gravitate": intrinsic properties are explained in terms of coupling within the closed system, they don't really exist (thus they are "virtual") in the vacuum per se, they only become into existence when there is coupling.

so how does one create this coupling?

See:  http://link.springer.com/static-content/lookinside/986/art%253A10.1007%252FBF02755823/000.png

Since I am not a physicist, and I didnt sleep at a holiday in express last night. I will not even begin to pretend I understood most of what that reply was attempting to show. All I could get out of it  was Sachs provided a point of clarification on why the complaint of his theory lacking internal consistency was not correct. other than that didnt see an argument for how one could physically force the virtual particles to experience coupling and gain their intrinsic properties. Though I suspect this experiment may light the path towards doing something of the sort

http://www.bibliotecapleyades.net/ciencia/secret_projects/project077.htm (http://www.bibliotecapleyades.net/ciencia/secret_projects/project077.htm)
Title: Re: EM Drive Developments - related to space flight applications - Thread 2
Post by: Rodal on 12/29/2014 11:56 pm
...
Sachs formally answered (in the 1960's !) the criticism that the matter in Quantum Vacuum "does not gravitate": intrinsic properties are explained in terms of coupling within the closed system, they don't really exist (thus they are "virtual") in the vacuum per se, they only become into existence when there is coupling.

so how does one create this coupling?

See:  http://link.springer.com/static-content/lookinside/986/art%253A10.1007%252FBF02755823/000.png

Since I am not a physicist, and I didnt sleep at a holiday in express last night. I will not even begin to pretend I understood most of what that reply was attempting to show. All I could get out of it  was Sachs provided a point of clarification on why the complaint of his theory lacking internal consistency was not correct. other than that didnt see an argument for how one could physically force the virtual particles to experience coupling and gain their intrinsic properties. Though I suspect this experiment may light the path towards doing something of the sort

http://www.bibliotecapleyades.net/ciencia/secret_projects/project077.htm (http://www.bibliotecapleyades.net/ciencia/secret_projects/project077.htm)

Instead of photons colliding with electrons "whizzing down the two-mile-long Stanford accelerator" as in the above paper  (".. The paths of colliding electrons and photons in the experiment ")

it's better to think of low energy two photon physics ( http://en.wikipedia.org/wiki/Two-photon_physics ).  See this article for example on such experiments:

Probing the quantum vacuum with polarized light: a low energy photon-photon collider

http://indico.cern.ch/event/1743/contribution/8/material/slides/0.pdf

Theme
– Vacuum as a “target”: low energy photon-photon collider
• QED interactions
• other interactions?

Aim
– Measure the magnetically induced linear birefringence and linear dichroism (optical rotation) of the Vacuum element (in practice a gas in the zero-pressure limit)
– Possible contributions to macroscopic properties
• photon-photon scattering
• photon splitting
• production of:
– neutral bosons

Main parameters of the apparatus
– magnet
• dipole, 6 T, temp. 4.2 K, 1 m field zone
– cryostat
• rotation frequency ~300 mHz, sliding contacts, warm bore to allow light propagation in the interaction zone
– laser
• 1064 nm, 100 mW, frequency-locked to the F.-P. cavity
– Fabry-Perot optical cavity
• 6.4 m length, finesse ~100000, optical path in the interaction region ~ 60 km
– heterodyne ellipsometer
• ellipticity modulator (SOM) and high extinction (~10-7) crossed polarisers + Quarter Wave Plate (QWP)
• time-modulation of the effect
– detection chain
• photodiode with low-noise amplifier
– DAQ
• Slow: demodulated at low frequency and phase-locked to the magnetic field instantaneous direction
• Fast: high sampling frequency direct acquisition
Title: Re: EM Drive Developments - related to space flight applications - Thread 2
Post by: Stormbringer on 12/30/2014 07:29 am
OT: I found a thing and put it over in the "New Hope For Warp Drive" thread. it does have some interesting details that can be examined in a very loose sort of way. not a technical paper by any means but it does describe the set up and effects. i found it interesting the weight of his test object and magnitude of effect on it.
Title: Re: EM Drive Developments - related to space flight applications - Thread 2
Post by: Star One on 12/30/2014 01:21 pm
Curious that there has been no coverage of this in the mainstream science press for the end of year, not a mention in anyone that I've seen roundups of 2014.

For comparison the detection of primordial gravity waves has been well covered even though this discovery has been heavily disputed since.
Title: Re: EM Drive Developments - related to space flight applications - Thread 2
Post by: Rodal on 12/30/2014 03:21 pm
Curious that there has been no coverage of this in the mainstream science press for the end of year, not a mention in anyone that I've seen roundups of 2014.

For comparison the detection of primordial gravity waves has been well covered even though this discovery has been heavily disputed since.

The reason is well explained by xkcd, see: http://www.explainxkcd.com/wiki/index.php/1404:_Quantum_Vacuum_Virtual_Plasma

(http://www.explainxkcd.com/wiki/images/3/32/quantum_vacuum_virtual_plasma.png)

[Note however that the comic is wildly exaggerated regarding NASA's tests: NASA only "pumped" 2.6 to 17 watts of power into the truncated cone cavity, about 1,000 to 8,000 times less input-power than what the comic shows]

None of these experiments have demonstrated a linear acceleration: all of them have measured rotational accelerations.  None of the EM Drives have been tested in a vacuum.  None of the measured forces are high enough to levitate the drive.

Some of Woodward's experiments have been conducted in a vacuum, however, from a practical-applications viewpoint or proof-of-concept viewpoint, they are even more disappointing: The reported "thrust" force/(power input) of the latest Woodward experiments (Fearn, Zachar, Woodward & Wanser) is about 2000 times less than the NASA Brady et.al TE mode and about 20,000 times less than the Shawyer Demo  In other words, it takes (for the latest Woodward experiments) 3 to 4 orders of magnitude greater input power to produce the same level of what is reported as "thrust" force as for the EM Drives.

(http://angryflower.com/experi.gif)

For comparison, the Wright Brothers experimented with real flying machines, Goddard with real chemical propulsion rockets that although small had enough thrust to lift the rockets, and world-renowned physicists like Freeman Dyson (based on ideas by the world-known mathematician Ulam) successfully conducted explosively loaded experiments to thrust up payloads for Project Orion's explosive-loaded propulsion.

If any of these experimenters ever gets any payload to get thrusted upwards, levitated or at least propelled in outer space, you may then see this covered in the mainstream science press.  Until then, there is really no experimental breakthrough to report in the mainstream science press.  Even NASA's Dr. White titled his experimental paper with the weakly tentative title "Anomalous..." and it was given at an AIAA conference and not published in a peer-reviewed journal.
Title: Re: EM Drive Developments - related to space flight applications - Thread 2
Post by: Notsosureofit on 12/30/2014 04:04 pm
OT: I found a thing and put it over in the "New Hope For Warp Drive" thread. it does have some interesting details that can be examined in a very loose sort of way. not a technical paper by any means but it does describe the set up and effects. i found it interesting the weight of his test object and magnitude of effect on it.

Not much to go by, it seems to be a 2 meter (146 MHz) asymmetric resonant antenna, so it could meet the requirement for a "Sachs-Schwebel" gravitational current generator instead of a warp drive.  It could also be seeing electromagnetic forces between the antenna and the faraday cage walls which are in the near field.  The interferometer tests are interesting, but again, not much data.

There is a paper of sorts: http://swdllc.paresspacewarpresearch.org/PressRelease/Press.htm
Title: Re: EM Drive Developments - related to space flight applications - Thread 2
Post by: bad_astra on 12/30/2014 04:14 pm
Has anyone discussed testing spin using a Woodward effect device, like a pendulum, using a cheap very high altitude balloon of the sort JP Aerospace launch? It's not perfect vacuum, but its cheaper than launching a cubesat. It just seems like navel gazing to me unless the effect does anything practical.
Title: Re: EM Drive Developments - related to space flight applications - Thread 2
Post by: Stormbringer on 12/30/2014 04:27 pm


Not much to go by, it seems to be a 2 meter (146 MHz) asymmetric resonant antenna, so it could meet the requirement for a "Sachs-Schwebel" gravitational current generator instead of a warp drive.  It could also be seeing electromagnetic forces between the antenna and the faraday cage walls which are in the near field.  The interferometer tests are interesting, but again, not much data.

There is a paper of sorts: http://swdllc.paresspacewarpresearch.org/PressRelease/Press.htm
When you say that it means it really was topical for this thread after all.

I am slightly optimistic about it now; but i have seen this guy's web page before. I kind of dismissed it as true fringe, but then I saw the article and the video with video demonstration footage. That moved it up a step or two in my estimation. The guy is not completely nuts but his idea has a giggle factor that has ensured he has been interviewed on tons of fringe kook venues; pod casts and such.
Title: Re: EM Drive Developments - related to space flight applications - Thread 2
Post by: Star One on 12/30/2014 04:37 pm
On another forum they were discussing how such drives could accelerate the end of the universe, I looked at some of the explanations on there as too what was meant but they completely lost me I'm afraid to admit?
Title: Re: EM Drive Developments - related to space flight applications - Thread 2
Post by: Stormbringer on 12/30/2014 04:45 pm
On another forum they were discussing how such drives could accelerate the end of the universe, I looked at some of the explanations on there as too what was meant but they completely lost me I'm afraid to admit?

well...that's a step ahead of what is already a shaky concept or two. but I think anything that is a linear drive like the thing i was just talking about would be an infinitesimal perturbation in the scheme of the whole cosmos. but i have heard arguments about collapsing the vacuum state of the universe. but i think that that was in reference to collider experiments and not warp drives or em drives.
Title: Re: EM Drive Developments - related to space flight applications - Thread 2
Post by: Star One on 12/30/2014 05:00 pm

On another forum they were discussing how such drives could accelerate the end of the universe, I looked at some of the explanations on there as too what was meant but they completely lost me I'm afraid to admit?

well...that's a step ahead of what is already a shaky concept or two. but I think anything that is a linear drive like the thing i was just talking about would be an infinitesimal perturbation in the scheme of the whole cosmos. but i have heard arguments about collapsing the vacuum state of the universe. but i think that that was in reference to collider experiments and not warp drives or em drives.

God help us if the media ever get hold of this, you can imagine the headlines now!
Title: Re: EM Drive Developments - related to space flight applications - Thread 2
Post by: Stormbringer on 12/30/2014 05:33 pm

On another forum they were discussing how such drives could accelerate the end of the universe, I looked at some of the explanations on there as too what was meant but they completely lost me I'm afraid to admit?

well...that's a step ahead of what is already a shaky concept or two. but I think anything that is a linear drive like the thing i was just talking about would be an infinitesimal perturbation in the scheme of the whole cosmos. but i have heard arguments about collapsing the vacuum state of the universe. but i think that that was in reference to collider experiments and not warp drives or em drives.

God help us if the media ever get hold of this, you can imagine the headlines now!
oh they already did. it's just that after the LHC black hole media hype no one paid attention to the "collapse of the vacuum state" scare. i remember seeing articles about it in new scientist if i recall right.
Title: Re: EM Drive Developments - related to space flight applications - Thread 2
Post by: Stormbringer on 12/30/2014 06:15 pm
ok. this is interesting... neutrinos that collide with nuclei at an oblique angle create particles out of the vacuum. evidently said new particles don't annihilate with a virtual twin? is that right?

http://phys.org/news/2014-12-neutrinos-full-on-glancing.html

so here is a real interaction with the vacuum. that was being argued a few pages back WRT Dr White's Theory of how QVPTs work i think.

Title: Re: EM Drive Developments - related to space flight applications - Thread 2
Post by: frobnicat on 12/30/2014 10:35 pm
ok. this is interesting... neutrinos that collide with nuclei at an oblique angle create particles out of the vacuum. evidently said new particles don't annihilate with a virtual twin? is that right?

http://phys.org/news/2014-12-neutrinos-full-on-glancing.html

so here is a real interaction with the vacuum. that was being argued a few pages back WRT Dr White's Theory of how QVPTs work i think.

Ok, let's try this : there is no question that there can be real interaction with the vacuum (vacuum polarization, pair production from strong electric field or gamma photon bouncing on nucleus or photon/photon...) the question is at what energetic cost ? If there is conservation of momentum_energy in those interactions then you are no better of than with the photon rocket (again) or shooting the supposedly available neutrinos backward (almost the same).
Title: Re: EM Drive Developments - related to space flight applications - Thread 2
Post by: wembley on 12/31/2014 08:51 am
So what TRL would you guys say the devices presented in the "Anomalous Thrust Production...." papers are at right now? I'd say probably a TRL 2. Possibly on the way to a TRL 3, hopefully. 

Well, Cannae were talking about flying theirs in a nanosatellite about a year ago, and I suspect that is still ongoing. The Chinese appear to be more cautious but are somewhere around 4+. Shawyer certainly claimed to be at 4 some time ago. And if rumour is to be believed, an aerospace company has pushed Shawyer's work on some from there.
Title: Re: EM Drive Developments - related to space flight applications - Thread 2
Post by: wembley on 12/31/2014 09:30 am
Has there been the least indication yet when we are too get some more results in relation to those who are experimenting in this area?

There is essentially a news blackout on the experimental side from NASA.

The problem is, if this drive really works, the defence implications are huge. Those in charge might feel it would not necessarily be a good idea to go public with it -- especially from the US perspective. Remember that 'Sputnik moment'?
Title: Re: EM Drive Developments - related to space flight applications - Thread 2
Post by: ChrisWilson68 on 12/31/2014 10:27 am
Curious that there has been no coverage of this in the mainstream science press for the end of year, not a mention in anyone that I've seen roundups of 2014.

For comparison the detection of primordial gravity waves has been well covered even though this discovery has been heavily disputed since.

The primordial gravity wave research was peer reviewed and done by scientists who are respected by the rest of the scientific community.  The debate over it was from within the scientific community.  That's why the mainstream media covered it.

The reactionless drive work is entirely outside the scientific mainstream, and every mainstream scientist who has examined it has found it to be without sufficient evidence to claim a discovery.  That's why the mainstream media has not covered it much.
Title: Re: EM Drive Developments - related to space flight applications - Thread 2
Post by: ChrisWilson68 on 12/31/2014 10:30 am
So what TRL would you guys say the devices presented in the "Anomalous Thrust Production...." papers are at right now? I'd say probably a TRL 2. Possibly on the way to a TRL 3, hopefully. 

Well, Cannae were talking about flying theirs in a nanosatellite about a year ago, and I suspect that is still ongoing. The Chinese appear to be more cautious but are somewhere around 4+. Shawyer certainly claimed to be at 4 some time ago. And if rumour is to be believed, an aerospace company has pushed Shawyer's work on some from there.

No, it's TRL-1.  Basic research has not even established sufficient evidence of anything anomalous to convince even a small part of the mainstream physics community.
Title: Re: EM Drive Developments - related to space flight applications - Thread 2
Post by: Stormbringer on 12/31/2014 04:28 pm
ok. this is interesting... neutrinos that collide with nuclei at an oblique angle create particles out of the vacuum. evidently said new particles don't annihilate with a virtual twin? is that right?

http://phys.org/news/2014-12-neutrinos-full-on-glancing.html

so here is a real interaction with the vacuum. that was being argued a few pages back WRT Dr White's Theory of how QVPTs work i think.

Ok, let's try this : there is no question that there can be real interaction with the vacuum (vacuum polarization, pair production from strong electric field or gamma photon bouncing on nucleus or photon/photon...) the question is at what energetic cost ? If there is conservation of momentum_energy in those interactions then you are no better of than with the photon rocket (again) or shooting the supposedly available neutrinos backward (almost the same).
But neutrinos are "free" so there is no real cost or at least not a commensurate cost similar to banging high energy gamma photons together. Further I know no way of increasing the amount ambient neutrinos. Since only an occasional neutrino collides with anything and of those only a few do the glancing blow trick I was not shooting for using them for a drive of any sort. What i was pointing at was the vacuum interaction itself; which you covered nicely in the first line of your reply. :)
Title: Re: EM Drive Developments - related to space flight applications - Thread 2
Post by: savuporo on 12/31/2014 05:15 pm
None of these experiments have demonstrated a linear acceleration: all of them have measured rotational accelerations.  None of the EM Drives have been tested in a vacuum.  None of the measured forces are high enough to levitate the drive.
Forget about levitation. I havent read much or anything about it, but are the forces even in the same ballpark as existing magnetic torquer rods for cubesats ? If yes, in theory this could assist with attitude control in deep space, at least for desaturation.
Title: Re: EM Drive Developments - related to space flight applications - Thread 2
Post by: Star One on 12/31/2014 05:26 pm
Curious that there has been no coverage of this in the mainstream science press for the end of year, not a mention in anyone that I've seen roundups of 2014.

For comparison the detection of primordial gravity waves has been well covered even though this discovery has been heavily disputed since.

The primordial gravity wave research was peer reviewed and done by scientists who are respected by the rest of the scientific community.  The debate over it was from within the scientific community.  That's why the mainstream media covered it.

The reactionless drive work is entirely outside the scientific mainstream, and every mainstream scientist who has examined it has found it to be without sufficient evidence to claim a discovery.  That's why the mainstream media has not covered it much.
I suspect there are people who may disagree with you with that statement, but that's not my debate to have.:)
Title: Re: EM Drive Developments - related to space flight applications - Thread 2
Post by: zen-in on 12/31/2014 11:40 pm
So what TRL would you guys say the devices presented in the "Anomalous Thrust Production...." papers are at right now? I'd say probably a TRL 2. Possibly on the way to a TRL 3, hopefully. 

Well, Cannae were talking about flying theirs in a nanosatellite about a year ago, and I suspect that is still ongoing. The Chinese appear to be more cautious but are somewhere around 4+. Shawyer certainly claimed to be at 4 some time ago. And if rumour is to be believed, an aerospace company has pushed Shawyer's work on some from there.

No, it's TRL-1.  Basic research has not even established sufficient evidence of anything anomalous to convince even a small part of the mainstream physics community.

Writing to his friend, Joseph Priestley, Benjamin Franklin said, “We may perhaps learn to deprive large Masses of their Gravity & give them absolute Levity, for the sake of easy Transport.

Another researcher believes he can build a small craft with much levity.

http://www.omaha.com/news/metro/working-toward-a-warp-drive
-in-his-garage-lab-omahan/article_b6489acf-5622-5419-ac18-0c44474da9c9.html
Title: Re: EM Drive Developments - related to space flight applications - Thread 2
Post by: Mulletron on 01/01/2015 01:12 pm

In his website he has posted several of his articles.  For example this
relatively recent one on the Mach principle and origin of inertia:

http://mendelsachs.com/wp-content/uploads/articles/the-mach-principle.pdf

In that reference, Sachs convincingly argues against the approach to Mach's
Principle followed by Woodward (-of course- he does not mention  Woodward),
he considers the particle-antiparticle pairs of the quantum vacuum having
a most important effect, while the effect of distant stars is
negligible
:


Quote from: Sachs
I have found in my research program in general relativity, that the primary
contribution to the inertial mass of any local elementary matter, such as an
'electron', are the nearby particle-antiparticle pairs that constitute what
we call the 'physical vacuum'. [The main developments of this research are
demonstrated in my two monographs: General Relativity and Matter, and
Quantum Mechanics from General Relativity]. A prediction of this research
program is that the main influence of these pairs on the mass of, say, an
electron comes from a domain of the 'physical vacuum' in its vicinity, whose
volume has a radius that is the order of 10^(-15) cm. Of course, the distant
stars, billions of light-years away, also contribute to the electron's mass,
though negligibly, just as the Sun's mass contribution to the weight of a
person on Earth is negligible compared with the Earth's influence on this
person's weight! Nevertheless, it was Mach's contention that in principle
all of the matter of the closed system - the nearby as well as far away
constituents - determines the inertial mass of any local matter.

(Bold added for emphasis) ==> this is the anti-thesis of Woodward's
approach to Mach's principle!


This is quite an interesting find and it is refreshing to see that it supports our conclusions (not sure who all agreed with me on this) from the previous thread and forward
(http://forum.nasaspaceflight.com/index.php?topic=29276.msg1266237#msg1266237) during the origin of inertia days; that all the competing theories on the origin of inertia are correct, none are exclusively correct; they they all share a component to inertia. The evidence tells me that inertia is the aggregate effect caused by many different types of interactions happening at once. It also falls in line with the Feynman quote, "All mass is interaction." Even inertial mass? Why not I say.

It is incorrect to state (in my humble view at least) that inertia is
Machian (distant matter) or arises only from vacuum interaction or is
intrinsic to matter itself, gravitational, electromagnetic, whatever.

I saw evidence recently that Mach didn't mean JUST the distant stars/matter when he
envisioned inertia......I didn't know this until now! Is this true?!? Did Woodward acknowledge this?
From page 3 here:
http://mendelsachs.com/wp-content/uploads/articles/the-mach-principle.pdf
"In contrast to this, in his Science of Mechanics Mach said that all of the
matter of the universe, not only the distant stars, determines the inertial
mass of any localized matter."

Break:
I've noticed a delve back into theory recently, which I have been avoiding because of the call for a greater focus on spaceflight applications. Of course I support this because I want to keep our moderator happy and this is his home, we're the guests. I don't want our thread (and work) to go poof again either :(

Is theory off the table?

I assure those who are reading, that this theory we're slinging around here occasionally has the potential to eventually support spaceflight applications. But as I eluded to in my first post in this thread, there
simply aren't spaceflight applications for this emerging technology, YET.
The TRL of this is potential technology is at the concept stage. I tried to stretch a potential application out what's known and didn't do too well. There isn't much space flight application to talk about yet; mostly theory. This may be debunked tomorrow. The space flight application of this post is simply, we better figure out this inertia thing if we want to improve upon and build new and exciting means of space flight.

Those of us who continually plug away on this problem are doing it for good reason. It certainly isn't fun at times reading scientific papers for hours until my eyes go cross. My wife certainly isn't happy with the time I devote to this now. So it is a labor of love. I think many of us really care about solving this problem. I can't stand not knowing how something works for one and the potential payoff for others is enormous if it works. So it is very exciting. Even if it is debunked, we still learn something new in the end, which could be rolled into a new impossible thruster. The potential space flight applications of this post and others like it might (humbly think mine won't) apply to real missions at some point. I do feel that these issues concerning the science behind these potential thrusters needs to be discussed and it seems people are reading, as the the view count of the old thread continues to climb.

Break:
I don't like to double post so I'll add one more subject here.
Turning light into matter:
http://phys.org/news/2014-05-scientists-year-quest.html
I remembered this article this morning. I can see a potential space flight application of this in the future. That is if you take the matter that was created and shoot it out the back end, like an ion drive. Instead of carrying around propellant like xenon, you create it on the fly.
Title: Re: EM Drive Developments - related to space flight applications - Thread 2
Post by: Notsosureofit on 01/01/2015 04:18 pm
Trying to put numbers to the Sachs-Schwebel gravitational current, I have not been able to get copies of the Dyatlov (in Russian) papers below but did find a reference to his gravitational polarization density.

This paper is from the old ball-lightning arguments.


http://www.math.nsc.ru/directions/tornado-eng.htm

"The presence of positive and negative gravitational charges gives the vacuum a polarization property in a gravitational field, that, apart from everything else, permits an explanation of the nature of a gravitational current of displacement, arising in gravitational waves.
In his work [11] V. L. Dyatlov gives a high assessment of gravitational polarization of a physical vacuum in the gravitational field of the Earth. By this account, 10^8 kg of negative mass is concentrated onto the upper surface of a cubic volume which is 1 decimeter in size and subjected to a gravitational field, whereas the same amount positive mass is concentrated onto the lower surface of the cubic volume."

[11] Dyatlov V. L.Polarization model of non-homogeneous physical vacuum. Novosibirsk, Publishing of Institute of Mathematics of Siberian branch of Russian Academy of Sciences, 1998, 183 p. (in Russian)

Dyatlov V.L. Linear equations of macroscopic electrogravidynamics.- Moscow, Inst.Teor.Appl.Phys. Acad. Nat.Sci., Preprint No.11, 1995 (in Russian)

Which led to:

http://www.sinor.ru/~che/Vdyatlov1.htm

which references:

52.Terletskiy, Y. P.  Paradoxes of the Theory of Relativity.  Moscow:  Nauka, 1966.  120 pp.

58.Terletskiy, Ya. P., and Yu. P. Rybakov.  Electrodynamics.  Moscow:  Vyssh. shk., 1990.  352 pp.

Which "sounds like" a Russian invention of the Sachs-Schwebel ( quadrigues vs quarternions )

In any event, all I can gather from scanning the equations (so far) is that it looks like they are concerned w/ "gravity currents" as opposed to "gravitational currents".  That makes sense if they are looking at ball-lightning and tornadoes, etc.

I don't know if the polarization density they claim (10^5kg/m^3 @ g) is useful or relevant at this point.




Title: Re: EM Drive Developments - related to space flight applications - Thread 2
Post by: Rodal on 01/02/2015 02:13 pm
...

52.Terletskiy, Y. P.  Paradoxes of the Theory of Relativity.  Moscow:  Nauka, 1966.  120 pp.

58.Terletskiy, Ya. P., and Yu. P. Rybakov.  Electrodynamics.  Moscow:  Vyssh. shk., 1990.  352 pp.

Which "sounds like" a Russian invention of the Sachs-Schwebel ( quadrigues vs quarternions )

...

"The
 Sonobioelectronic and Sonobioelectrogravitic Nature of Life on Earth And Why Pilot-wave Subquantum
Mechanics and Information Is Not Enough" (http://www.researchgate.net/profile/Frank_Maye/publication/242650713_
The_Sonobioelectronic_and_Sonobioelectrogravitic_Nature_of_Life_on_Earth_And_Why_Pilot-wave_Subquantum_Mechanics_and_Information_Is_
Not_Enough/links/00b7d51ce056c06be3000000)
A. R. BORDON
D. H. HASLER
J.A. SANCHEZ
F.M. MAYE
R. B. SOLINGEN

describes the quadrigue (group of four) of particles of Terletskiy as follows (quadrigue, from the Latin: "A Roman racing chariot drawn by four horses abreast"):

Quote
In accepting negative mass, one must therefore accept the quadrigue (group of four) of particles
of Terletskiy (88), which is neatly buttressed by Dirac’s quadrupling of states in his quantum theory. It
is also a remarkable turn of events in the modeling of the physical vacuum as a polarizing medium.
Terletskiy’s quadrigue is actually four particles (or, actually, four electrical charges, masses, magnetic
moments and spin moments), the sum of which equal zero
– from which, interestingly enough, one can
derive any two pairs of particles in which their electrical charges, masses, magnetic and spin moments
equal zero! This allows theorists to build a basic model of cella (or distinguishable compartments) of
the physical vacuum as a whole-cloth neutral polarizing medium, from which then three kinds of
physical vacuums can manifest: (a) an absolute physical vacuum, represented by the quadrigues; (b) a
physical vacuum of matter, which is made up of particles, one of which is an ordinary one; and (c) a
physical vacuum of antimatter, which is made up of particle pairs, one of which is an antiparticle.
What is most remarkable is that the absolute physical vacuum is described quite well by Maxwell and
Heaviside equations, and thus in the absolute physical vacuum, the polarization model breaks down
into two independent models: the Maxwell theory of electromagnetism, and the Heaviside theory of
gravitational spin. But in physical reality, the relationships of electrical and gravitational polarizations
and of magnetic and spin polarization in the absolute- as well as in the physical matter vacuums come
out as quite a unified system of equations that become the basis of a combined electrogravitodynamic
model of matter electrogravitogeneration (via electrical charges and magnetic polarizations) and
continuum dynamics (via masses and moments of the quantity of movement set by way of the
Heaviside equations).

Apparently a different concept than (Hamilton's) quaternion algebra:

i^2 = j^2 = k^2 = ijk = −1

which is non-commutative.

(http://www.euclideanspace.com/maths/algebra/realNormedAlgebra/quaternions/transforms/derivations/vectors/quaternionRotation.png)

One motivation for quaternions is that rotations in 3 dimensions ("3D") are not a 3D vector space (we can't combine rotations in 3D space using vector addition). Representations of 3D rotations using 3 scalar values are non linear, they have singularities and they are difficult to combine. To get around these issues one can embed the representation of 3D rotations into a higher dimensional space, in this case the 4D quaternion space.

http://upload.wikimedia.org/wikipedia/commons/thumb/8/85/Euler2a.gif/170px-Euler2a.gif

http://www.toymaker.info/Games/assets/images/yawpitchroll.jpg
Title: Re: EM Drive Developments - related to space flight applications - Thread 2
Post by: Notsosureofit on 01/02/2015 02:45 pm
http://www.researchgate.net/publication/242544190_Gravity-Like_Fields_and_Space_Propulsion_Concepts

Interesting experimental results.  Found just by accident.

Edit:  They are still at it, so ?

Title: Re: EM Drive Developments - related to space flight applications - Thread 2
Post by: Rodal on 01/02/2015 03:04 pm
http://www.researchgate.net/profile/Arbab_Arbab/publication/
51916988_The_unified_quantum_wave_equation/links/0912f50de92ef30406000000.pdf


Quote
We have investigated in this work the quaternionic momentum eigen value problem in quantum mechanics. We have written the wave function, energy and momentum of the particle as quaternionic quantities. The momentum eigen value equation reveals that the scalar and vector parts of the wavefunction are governed by a new wave equation. This is a new equation that we wish it will describe bosons and fermions. When the interaction of the particle with an electromagnetic field is introduced a spin term appeared in the equation of motion. The scalar equation doesn't change due to the interaction of the electromagnetic field with the particle. However, the vector equations are altered. We generalized the ordinary uncertainty relation to quaternionic one. This generalization provides us with the energy momentum relation of Einstein and the remaining uncertainty relations. Hence, by adopting the quaternionic quantum mechanics, namely, Dirac equation, we arrived at a dissipative or generalized
Klein-Gordon equation with a particle spin. This formalism gives rise to the generation of spin angular momentum of the particle when a photon field is introduced in the equation of motion. Further investigation is going on to explore the physics of these two waves
Title: Re: EM Drive Developments - related to space flight applications - Thread 2
Post by: wembley on 01/02/2015 04:24 pm
So what TRL would you guys say the devices presented in the "Anomalous Thrust Production...." papers are at right now? I'd say probably a TRL 2. Possibly on the way to a TRL 3, hopefully. 

Well, Cannae were talking about flying theirs in a nanosatellite about a year ago, and I suspect that is still ongoing. The Chinese appear to be more cautious but are somewhere around 4+. Shawyer certainly claimed to be at 4 some time ago. And if rumour is to be believed, an aerospace company has pushed Shawyer's work on some from there.

No, it's TRL-1.  Basic research has not even established sufficient evidence of anything anomalous to convince even a small part of the mainstream physics community.

Being believed by the mainstream community has nothing to do with TRL. If it works, it works. And if the Chinese fly their Emdrive (possibly this year?) and get acceleration, I doubt they'll care what anyone else thinks however loudly they say it can't be true.

I suspect these things really will be operational before a lot of the mainstream starts accepting them, the will to disbelieve is that strong.
Title: Re: EM Drive Developments - related to space flight applications - Thread 2
Post by: Notsosureofit on 01/02/2015 04:25 pm
http://www.researchgate.net/profile/Arbab_Arbab/publication/51916988
_The_unified_quantum_wave_equation/links/0912f50de92ef30406000000.pdf


Quote
We have investigated in this work the quaternionic momentum eigen value problem in quantum mechanics. We have written the wave function, energy and momentum of the particle as quaternionic quantities. The momentum eigen value equation reveals that the scalar and vector parts of the wavefunction are governed by a new wave equation. This is a new equation that we wish it will describe bosons and fermions. When the interaction of the particle with an electromagnetic field is introduced a spin term appeared in the equation of motion. The scalar equation doesn't change due to the interaction of the electromagnetic field with the particle. However, the vector equations are altered. We generalized the ordinary uncertainty relation to quaternionic one. This generalization provides us with the energy momentum relation of Einstein and the remaining uncertainty relations. Hence, by adopting the quaternionic quantum mechanics, namely, Dirac equation, we arrived at a dissipative or generalized
Klein-Gordon equation with a particle spin. This formalism gives rise to the generation of spin angular momentum of the particle when a photon field is introduced in the equation of motion. Further investigation is going on to explore the physics of these two waves

Very nice, if I'm reading it right, there may be the implication that the spins of the counter-propagating cavity photons could add to 2 in an interaction.  But, at the moment, I don't see that generating enough force.  (who knows at this point, .... it would need a very high vacuum energy density ?)

Added: (18) to (24) is the argument for translating the cavity dispersion to the "doppler plane" to calculate the accelerated frame of reference. (in a different context)

Insight ??:  It seems here that work must be done to install the cavity photons within the limiting boundary conditions of the cavity and that produces a potential energy component which does not appear in the AFR which eliminates the dispersion and doppler shifts the photons.  Since we are holding the cavity static we do not see the energy expenditure, only the static initial force on the unbalanced cavity.  The work done is a good starting number.  Someone must have done that calculation. (he says w/ a straight face ... been there !)

And that force could drop off dramatically under acceleration w/o knowing the energy. In fact it must as a function of the cavity Q.

But that brings up a further question.  Unless the work done is a scalar function, maintaining the acceleration in an isolated system may introduce that compensating momentum which would render the whole point mute.

So the place to start is probably the radiation pressure integrated over the boundary of the cavity compared to the free photon case. (not quite the Shawyer case)
Title: Re: EM Drive Developments - related to space flight applications - Thread 2
Post by: Star One on 01/02/2015 04:33 pm

So what TRL would you guys say the devices presented in the "Anomalous Thrust Production...." papers are at right now? I'd say probably a TRL 2. Possibly on the way to a TRL 3, hopefully. 

Well, Cannae were talking about flying theirs in a nanosatellite about a year ago, and I suspect that is still ongoing. The Chinese appear to be more cautious but are somewhere around 4+. Shawyer certainly claimed to be at 4 some time ago. And if rumour is to be believed, an aerospace company has pushed Shawyer's work on some from there.

No, it's TRL-1.  Basic research has not even established sufficient evidence of anything anomalous to convince even a small part of the mainstream physics community.

Being believed by the mainstream community has nothing to do with TRL. If it works, it works. And if the Chinese fly their Emdrive (possibly this year?) and get acceleration, I doubt they'll care what anyone else thinks however loudly they say it can't be true.

I suspect these things really will be operational before a lot of the mainstream starts accepting them, the will to disbelieve is that strong.

You could end up getting a situation with people still throwing cold water on this just as someone flies past their window in a flying car. :)
Title: Re: EM Drive Developments - related to space flight applications - Thread 2
Post by: tchernik on 01/03/2015 04:06 am
I agree. Even now, without full assurance of these phenomena's existence (without playing favorites, any single one of them being true would be disruptive), the philosophical rejection it causes seems to be way above the expected from new scientific fields or incipient domains.

Yes, I also think it is very possible engineers could earn the merit of using this first, making the applications come before science at large even accepts its existence.

And this could happen in China before the West. Their own existing results, if confirmed and proven to exist to their own authorities, would be worth of a practical test in the short term.
Title: Re: EM Drive Developments - related to space flight applications - Thread 2
Post by: ThinkerX on 01/03/2015 06:21 am
Ok, paging Doctor Rodal for a translation here (from Not so sure of its' link);

Quote


Source





Article: Physics of Extreme Gravitomagnetic and Gravity-Like Fields for Novel Space Propulsion and Energy Generation 

Jochem Hauser, Walter Dröscher

 
[Hide abstract]
ABSTRACT: In 2006 Tajmar et al. reported on the measurements of extreme gravitomagnetic fields from small Nb rings at cryogenic temperatures that are about 18 orders of magnitude larger than gravitomagnetic fields obtained from GR (general relativity). Cifuolini in 2004 and the NASA-Stanford Gravity Probe-B experiment in 2007 confirmed the Lense-Thirring effect as predicted by GR (gravitomagnetic fields generated by a rotating massive body, i.e. Earth) within some 10%. In 2007 gravitomagnetic fields generated by a rotating cryogenic lead disk were measured by Graham et al. Though these measurements were not conclusive (the accuracy of the laser gyrometer was not sufficient to produce a standard deviation small enough) their experiment seems to have seen the same phenomenon reported earlier by Tajmar et al., termed parity violation. This means that gravitomagnetic fields produced by the cryogenic rotating ring or disk vary substantially and change sign for clockwise and counter-clockwise directions of rotation. The experimental situation therefore occurs to be contradictory. On the one hand GR has been confirmed while at the same time, there seems to be experimental evidence for the existence of extreme gravitomagnetic fields that cannot be generated by the movement of large masses. If these experiments can be confirmed, they give a clear indication for the existence of additional gravitational fields of non-Newtonian nature. As was shown by the GP-B experiment, measuring gravitomagnetic fields from GR poses extreme difficulties. Therefore a novel physical mechanism should exist for the generation of gravity-like fields, which might also provide the key to gravitational engineering similar to electromagnetic technology

Quote
: Emerging Physics for Novel Field Propulsion Science 

Jochem Hauser, Walter Dröscher

 
[Hide abstract]
ABSTRACT: All space vehicles in use today need some kind of fuel for operation. The basic physics underlying this propulsion principle severely limits the specific impulse and/or available thrust. Launch capabilities from the surface of the Earth require huge amounts of fuel. Hence, space flight, as envisaged by von Braun in the early 50s of the last century, will not be possible using this concept. Only if novel physical principles are found can these limits be overcome. Gravitational field propulsion is based on the generation of gravitational (gravity-like) fields by manmade devices. In other words, gravity-like fields should be experimentally controllable. Present physics believes that there are four fundamental interactions: strong (nuclei), weak (radioactive decay), electromagnetism and Newtonian gravitation. As experience has shown for the last six decades, none of these physical interactions is suitable as a basis for novel space propulsion. None of the advanced physical theories like string theory or quantum gravity, go beyond these four known interactions. On the contrary, recent results from causal dynamical triangulation simulations indicate that wormholes in spacetime do not seem to exist, and thus even this type of exotic space travel may well be impossible. Recently, novel physical concepts were published that might lead to advanced space propulsion technology, represented by two additional long range gravitational-like force fields that would be both attractive and repulsive, resulting from interaction of gravity with electromagnetism. A propulsion technology, based on these novel long range fields, would be working without propellant

Title: Re: EM Drive Developments - related to space flight applications - Thread 2
Post by: RotoSequence on 01/03/2015 07:05 am
Ok, paging Doctor Rodal for a translation here (from Not so sure of its' link);

Links to the papers:

Arxiv: Physics of Extreme Gravitomagnetic and Gravity-Like Fields for Novel Space Propulsion and Energy Generation 
 (http://arxiv.org/abs/1104.3247)

PDF link: Emerging Physics for Novel Field Propulsion Science (http://www.hpcc-space.com/publications/documents/AIP2010Hauser.pdf)
Title: Re: EM Drive Developments - related to space flight applications - Thread 2
Post by: Stormbringer on 01/03/2015 08:04 am
if it is the initial martin Tajmar discovery he later recanted due to discovering the flow of his cryogenic coolant was the source of his interesting phenomenon. Unfortunate. However that explanation does not illuminate whether he think there is a nonspurious effect lurking after his error is factored out or not. After all he is definitely not the only researcher who thinks his kind of setup affects gravity. Podkletnov for example. There must be some reason this is so.
Title: Re: EM Drive Developments - related to space flight applications - Thread 2
Post by: Mulletron on 01/03/2015 12:47 pm
http://www.emdrive.com/EmDriveForceMeasurement.pdf

New info:
Some clarification from Shawyer. Helps to clear up some of the arguments we had about how Shawyer uses thrust and reaction, and which way it moves.
Title: Re: EM Drive Developments - related to space flight applications - Thread 2
Post by: Mulletron on 01/03/2015 01:00 pm

Very nice, if I'm reading it right, there may be the implication that the spins of the counter-propagating cavity photons could add to 2 in an interaction.  But, at the moment, I don't see that generating enough force.  (who knows at this point, .... it would need a very high vacuum energy density ?)


I bolded spin 2 above. It immediately caught my eye because gravitons are spin 2 massless particles (if they exist). Don't know what to make of it yet, but it jumped right off of the page when I saw it. Very interesting work you're doing. I really think you were on to something with dispersion too. It makes way more sense to approach radiation pressure differences leading to thrust using your ideas than how Shawyer explains it with his group velocity model.
Title: Re: EM Drive Developments - related to space flight applications - Thread 2
Post by: Stormbringer on 01/03/2015 02:24 pm
http://www.preposterousuniverse.com/blog/2013/10/03/guest-post-lance-dixon-on-calculating-amplitudes/

Quote
Along the way, Zvi, John Joseph and Henrik, thanks to the time-honored method of “just staring at” the loop integrand provided by unitarity, also stumbled on a new property of gauge theory amplitudes, which tightly couples them to gravity. They found that gauge theory amplitudes can be written in such a way that their kinematic part obeys relations that are structurally identical to the Jacobi identities known to fans of Lie algebras. This so-called color-kinematics duality, when achieved, leads to a simple “double copy” prescription for computing amplitudes in suitable theories of gravity: Take the gauge theory amplitude, remove the color factors and square the kinematic numerator factors. Crudely, a graviton looks very much like two gluons laid on top of each other. If you’ve ever looked at the Feynman rules for gravity, you’d be shocked that such a simple prescription could ever work, but it does. Although these relations could in principle have been discovered without unitarity-based methods, the power of the methods to provide very simple expressions, led people to find initial patterns, and then easily test the patterns in many other examples to gain confidence.

These guys won the Sakurai Prize for Physics.
Title: Re: EM Drive Developments - related to space flight applications - Thread 2
Post by: Notsosureofit on 01/03/2015 02:32 pm
All I can say so far is, that I think that if the cavity dispersion relation has an asymmetric variation in at least one spatial dimension, then there exists an accelerated frame of reference such that the dispersion is eliminated [and the cavity acts as an open system ?] (ie. the boundary condition is nullified, the photons act as free photon wavefunctions, the impedance matches that of the vacuum [?]) The Sachs-Schwebel anomalous current "looks like" it might contain this relation.  Conservation of 4-momentum is still open to question.
Title: Re: EM Drive Developments - related to space flight applications - Thread 2
Post by: birchoff on 01/03/2015 04:28 pm
http://www.emdrive.com/EmDriveForceMeasurement.pdf

New info:
Some clarification from Shawyer. Helps to clear up some of the arguments we had about how Shawyer uses thrust and reaction, and which way it moves.

Hold the phone... am I reading this right. You can only ever measure thrust if the cavity is accelerating? So just sitting on a flat floating stationary table top should yield no force, the same with when the cavity is moving at constant velocity. Did I mis interpret that pdf?
Title: Re: EM Drive Developments - related to space flight applications - Thread 2
Post by: Stormbringer on 01/03/2015 04:53 pm
if so that pushes it into Dr Woodward and vis inert or "fictitious" fields such as inertia, and gravity. (I'm currently reading Woodward's book )
Title: Re: EM Drive Developments - related to space flight applications - Thread 2
Post by: Rodal on 01/03/2015 06:46 pm
http://www.emdrive.com/EmDriveForceMeasurement.pdf

New info:
Some clarification from Shawyer. Helps to clear up some of the arguments we had about how Shawyer uses thrust and reaction, and which way it moves.

Hold the phone... am I reading this right. You can only ever measure thrust if the cavity is accelerating? So just sitting on a flat floating stationary table top should yield no force, the same with when the cavity is moving at constant velocity. Did I mis interpret that pdf?
if so that pushes it into Dr Woodward and vis inert or "fictitious" fields such as inertia, and gravity. (I'm currently reading Woodward's book )

Unlike the concepts of displacement, (and its time rates: velocity, acceleration) and strain (which can be mathematically defined in terms of geometry), the concept of "force" is intuitive and has been impossible to mathematically define intrinsically except in terms of other variables. (Although Cauchy and most recently Truesdell made valiant attempts).

Forces can never be measured directly.  Forces are always measured indirectly: for example measuring the resulting acceleration (and calculating the inertial force as F=m*a) or measuring a strain (and calculating an average Force=(cross-sectional Area)*(modulus of Elasticity)*strain ) or measuring a displacement (and calculating Force=(spring stiffness)*displacement).

It is not practical to use a "balance scale" to measure unknown, time-varying-forces

(http://upload.wikimedia.org/wikipedia/commons/thumb/d/d0/Balance_scale_IMGP9755.jpg/220px-Balance_scale_IMGP9755.jpg)

EDIT: Notwithstanding the above (true) statements I find Shawyer's writing to be confusing and easily lead to confusion.  NASA Eagleworks used a torsional pendulum to figure out the force produced by an initially static (truncated cone) EM Drive.   NASA reported to have figured out the EM Drive forces from the EM Drive measured displacement , using a calibration Force=stiffness*displacement.  Unfortunately NASA did not clearly specify whether the displacement (for the truncated cone) was in the direction of the small end or the direction of the large end of the truncated cone.
Title: Re: EM Drive Developments - related to space flight applications - Thread 2
Post by: frobnicat on 01/03/2015 11:14 pm
This "clarification" by Shawyer can hardly be interpreted at all. It states that a reaction force equal and opposite to the thrust is to be expected when not accelerating (reaction from a support) "so is Newtonian" but then plays with orientation conventions and decides that "This situation is unique to a propellantless thruster such as EmDrive and analogies with conventional devices are pointless." Ha !

So we are left wondering what makes "propellantless Newtons" special compared to "usual Newtons". I agree (with Rodal) that, as far as conventional Newtons are concerned, measurement of force needs some kind of displacement, but for constant force such displacement reaches a stable equilibrium (static situation) where R=T and both can then be determined by said dispacement. Shawyer's paper clearly states that this shouldn't be expected from "propellantless Newtons".

Quote from: Shawyer
Clearly, in a static situation, where T and R both exist as forces, they will cancel out. Thus any attempt to measure them by simply placing the thruster vertically on a set of scales will fail. If however the thrust is sufficient such that a=-g,then the thruster could be made to hover above the scales.

So if we have a thruster with a ever so slightly below -g the balance records no change (full weight of thruster) and then an infinitesimally small increase of a above -g makes the thruster hover above the balance that all of a sudden will record a weight of 0. How strange transition ! Moreover the same argument of experiment "at rest" => no measured force is made in horizontal setting (Fig 3).

Quote
It therefore appears that a force measurement can only be made in a dynamic environment, ideally by allowing the thruster to accelerate, ...

But isn't a vertical thruster in a dynamic environment thanks to g ? It's like the  "propellantless Newtons"  can tell the difference between being "restrained" or "unleashed". Not only equivalence principle is broken by such special Newtons, but paradoxes directly relevant to space flight are obvious :
assume such a thruster is mounted at the back of a spacecraft, but between the thruster and this big inertial mass there is a spring (no matter how stiff) and a displacement/stress sensor that tells the Newtons communicated by the thruster to the spacecraft. Take a body small enough (asteroid...) that the thruster is supposed to be able to hover the craft above at constant altitude. In this "static situation" the "propellantless Newtons" can't be measured (by a displacement related to stress of the link between thruster and craft). So I guess a spacecraft expects "conventional Newtons" to hover around: the "propellantless Newtons" can't be communicated from the thruster to useful push on a craft through a conventional link that would allow stress deformation (and therefore conventional measurement)...
Same argument would forbid thrusting at constant speed and constant thrust against a medium (horizontal flight in atmosphere). Worse : if equivalence principle is to hold, can't accelerate a spacecraft in free space as it's the same as hovering at constant altitude in a given gravity field.

Basically, saying a thruster is not stressing a scale's spring is saying that running the thing aboard a ship is useless : the thrust is 0 for all practical purpose. Unless we are speaking of an extending inertia modifying field that goes beyond the inner skin of walls of the thruster and accelerating directly the bulk of the rest of spacecraft (that would then need clarification for the range of such field.)

We could also discuss the orientation conventions : fig1 in the usual sense a reaction pseudo-force like R is opposite to a  (italic for vectors) :  the usual M a = T would go T - M a = 0  and to T + R = 0  (as common sense would tell) that is
R = - Ma and not the opposite.

Now we understand why Shawyer's design of LEO launcher puts pointy end of thruster up, because the thrust is toward the ground (but this is unconventional Newtons...).

This latest paper is moving the goalpost, ideally by allowing the goalpost to accelerate...


Title: Re: EM Drive Developments - related to space flight applications - Thread 2
Post by: Notsosureofit on 01/04/2015 02:08 am
So, before I trundle off to bed...(be'in old an' all)
When we are in the AFR that removes the cavity dispersion at the expense of splitting the frequency into doppler components, and giving us a flat metric tensor, what tensor quantities can we generate which will give zero force in this frame (despite the difference in frequency) and allow us to find the force in the rest frame of the cavity.  Then the question is: can this be done in GR or does it need extensions (such as Sachs-Schwebel, etc)
Title: Re: EM Drive Developments - related to space flight applications - Thread 2
Post by: CW on 01/04/2015 06:51 am
Dear Mr. Shawyer, it is very simple. Put your device PLUS power supply on a balance scale, switch it on. If the scale moves (or most preferably in ~vacuum in LEO, kindly ask your chinese friends to arrange for that - they reported almost a Newton "thrust", didn't they), you become an eternally famous person. If it however does not move, stop deluding yourself and others. Deal?
Title: Re: EM Drive Developments - related to space flight applications - Thread 2
Post by: QuantumG on 01/04/2015 07:02 am
Considering that NASA will fly just about anything of scientific note into space for the cost of the paperwork (and the Ukrainians will fly anything for you for less than $100k) most of these gadgets could be demonstrated in LEO.. if they worked.
Title: Re: EM Drive Developments - related to space flight applications - Thread 2
Post by: Req on 01/04/2015 02:29 pm
ISTM like they may be able to get a clear signal without going all the way into LEO.  Couldn't they send an appropriately sensitive test rig up on a vomit comet or a balloon(then drop it), vacuum chamber included if need be?  1 Newton sounds like a lot with respect to sensitivity/margins/etc in microgravity, so the test rig probably shouldn't need to be terribly complicated or expensive in either case.  At least not when compared to something that's going to LEO and the associated launch/operation costs.
Title: Re: EM Drive Developments - related to space flight applications - Thread 2
Post by: Star One on 01/04/2015 02:40 pm

Considering that NASA will fly just about anything of scientific note into space for the cost of the paperwork (and the Ukrainians will fly anything for you for less than $100k) most of these gadgets could be demonstrated in LEO.. if they worked.

Is this something that could be tested on the outside of ISS?
Title: Re: EM Drive Developments - related to space flight applications - Thread 2
Post by: Rodal on 01/04/2015 03:45 pm
So, before I trundle off to bed...(be'in old an' all)
When we are in the AFR that removes the cavity dispersion at the expense of splitting the frequency into doppler components, and giving us a flat metric tensor, what tensor quantities can we generate which will give zero force in this frame (despite the difference in frequency) and allow us to find the force in the rest frame of the cavity.  Then the question is: can this be done in GR or does it need extensions (such as Sachs-Schwebel, etc)
It would a big surprise if this would be possible within General Relativity: it would certainly go against what Sean M. Carroll and John Baez have stated (particularly concerning frames of reference).  The implication is that extra coupling terms and/or nonlinearities would be needed, such as Sachs-Schwebel.
Title: Re: EM Drive Developments - related to space flight applications - Thread 2
Post by: Mulletron on 01/04/2015 04:59 pm
Things are coming together slowly. I have the connectors and pigtails. Got the N-type female bulkhead threaded solder cup connectors and N-type male to SMA male connectors. Also got the panel type bulkhead connectors which I'm familiar with but don't really like, as a backup. I purchased instead of borrowed and shipping costs more than the actual product. The horn antennas in which to couple the rf across the gap will be homebrew (have to make these myself because standard gain horns are way too expensive; this could make it or break it). Copper sheeting is expensive so I didn't just order a bunch of it yet. Mostly because I don't want to waste material, and honestly I'm being lazy (and am still unsure on wasting expendable cash on a failed experiment), and I spent most of my extra cash on Christmas for the kids. Today I finally got around to planning out the frustum configuration and dims. Attached is a screen shot of the cut sheet utilizing a 2x4' sheet. The full CAD is in the google drive link. I can do a light cone (by the base) and a 45 degree (by the apex) with one sheet and have room for cutouts to make the flat ends. The design intent is to provide a cone in which I can quickly change resonant modes by inserting discs of varying diameter. With this setup I can support 2.4ghz ISM band at the narrow end and support 2 full wavelengths plus more in length. 5ghz is also supported.

Tools used:
https://www.google.com/?gws_rd=ssl#q=right%20circular%20cone%20calc%3A%20find%20A_L&skip=s
http://www.analyzemath.com/Geometry_calculators/surface_volume_frustum.html
http://www.calculatorsoup.com/calculators/geometry-solids/conicalfrustum.php
http://www.cleavebooks.co.uk/scol/calsect.htm
http://en.wikipedia.org/wiki/IEEE_802.11#Channels_and_frequencies
http://www.wavelengthcalculator.com/

CAD drawing with cut sheet:
https://drive.google.com/folderview?id=0B4PCfHCM1KYoTXhSUTd5ZDN2WnM&usp=sharing#list

Copper:
http://basiccopper.com/10-mil-copper-sheet-24-x-4-1.html (not cheap)
http://basiccopper.com/24x416milcos.html (might go with thicker, more expensive 16mil.
 
I think 10 mil is the way to go (I'm going for strength enough to support its own weight, but still be lightweight) but before I drop a hunski on it I'm going to get a sampler http://basiccopper.com/samplerpacks.html. I might go with 16 mil http://basiccopper.com/24x416milcos.html. It depends. Here's why I think 10mil or alternate 16mil is the right choice: http://basiccopper.com/thicknessguide.html


Title: Re: EM Drive Developments - related to space flight applications - Thread 2
Post by: aceshigh on 01/04/2015 05:03 pm
By the way, John Baez (in his blog https://plus.google.com/117663015413546257905/posts/C7vx2G85kr4 , answering correspondence) had some non-flattering things to say about Woodward's approach as well.

which however is not the subject of this thread, as you pointed out at page 2 and 3 of this thread

Quote from: Rodal
As Chris Bergin made clear, the purpose of this forum is to deal with SPACEFLIGHT APPLICATIONS (of EM Drives: those propellantless drives comprised of a microwave cavity as the devices tested by Shawyer in the UK, Prof. Juan Yang in China and Brady, March, White, et.al. at NASA):

Why is this prescription by Chris Bergin being ignored?

Although the latest Woodward experiments (Fearn, Zachar, Woodward & Wanser) show that it takes 20,000 times more power to produce a given level of thrust than the power required to produce the same thrust for the EM Drive (Shawyer demo), and therefore these experiments do not constitute "EM Drive Spaceflight Applications" (as instructed by Chris Bergin) Woodward fans persist on carrying a debate on this thread rather than their own Woodward thread.   

Frankly, I don't understand what is the goal being pursued here by Woodward-effect fans to insist to carry arguments concerning Woodward's theory and experiments in this thread. 

The curtailment of the previous EM Drive thread by this forum's moderator was due to disruptive discussions regarding the validity of the Woodward effect, including a negative review of Woodward's book by a physicist.


seriously Rodal, you chastised us and now posts about Woodward here? Looks like a bait for someone to come defend Woodward theory here...  ::)

if you want to post about Woodward Theory, be it questions, praise, analysis, ARTICLES or CRITICISM, the place is this thread
http://forum.nasaspaceflight.com/index.php?topic=13020.1845

and yes, the article was about EM Drive, but you specifically called out in your comment the criticism by John Baez regarding Woodward Theory, and THAT criticism should be pointed out and discussed in the appropriate thread, as pointed out by yourself in the 2nd and 3rd pages.
Title: Re: EM Drive Developments - related to space flight applications - Thread 2
Post by: zen-in on 01/04/2015 05:06 pm
Good that Shawyer has clarified the force measurement.  Referring to his Fig. 1 from the paper
http://www.emdrive.com/EmDriveForceMeasurement.pdf
and reproduced below, I conclude he is saying the force that is acting on the EM-drive is in the same direction as the vector labeled T.   The only difficulty I have with this is from looking at the videos of the EM=drive.   When it is operating it moves with the small end forward.
https://www.youtube.com/watch?v=57q3_aRiUXs

In the earlier thread I stated that maybe the thrust was due to heat convection from the sloped sides.   This was dismissed because the EM-drive moves in the opposite direction as seen in the video.


Title: Re: EM Drive Developments - related to space flight applications - Thread 2
Post by: Mulletron on 01/04/2015 05:58 pm
I think it is about time for us to get on Eagleworks for an update:

https://www.facebook.com/eagleworksnasa

Title: Re: EM Drive Developments - related to space flight applications - Thread 2
Post by: Stormbringer on 01/04/2015 06:12 pm
@AcesHigh

I think it was because i mentioned Woodward while trying to explain the difference between Vis Vivo forces like electromagnetism, the strong force, weak force and forces that are only apparent when a force or acceleration is applied to a mass such as inertia. such forces are labelled "fictitious" in physics without any negative meaning. everyone realizes there is a such thing as inertia and gravity.

So anyway; i invoked Woodward before Dr Rodal made his reply containing comments on Woodward.
Title: Re: EM Drive Developments - related to space flight applications - Thread 2
Post by: Rodal on 01/04/2015 06:21 pm
Good that Shawyer has clarified the force measurement.  Referring to his Fig. 1 from the paper
http://www.emdrive.com/EmDriveForceMeasurement.pdf
and reproduced below, I conclude he is saying the force that is acting on the EM-drive is in the same direction as the vector labeled T.   The only difficulty I have with this is from looking at the videos of the EM=drive.   When it is operating it moves with the small end forward.
https://www.youtube.com/watch?v=57q3_aRiUXs

In the earlier thread I stated that maybe the thrust was due to heat convection from the sloped sides.   This was dismissed because the EM-drive moves in the opposite direction as seen in the video.

Look at Fig. 2 in http://www.emdrive.com/EmDriveForceMeasurement.pdf:

the acceleration according to Shawyer is directed towards the small end, which is entirely consistent with the movement in the video.

Furthermore, Shawyer himself writes that the experiment in the video with the rotary air-bearing with the SPR Demonstrator calculated the force based on measurement of the acceleration (directed towards the small end):

Quote from: Shawyer
It therefore appears that a force measurement can only be made in a dynamic environment, ideally by allowing the thruster to accelerate, measuring that acceleration, and then calculating the thrust from T =   -  Ma. This is not a very easy method, although the SPR Demonstrator Thruster was successfully tested in this way on a rotary air bearing

As discussed by Frobnicat and me in posts above, the definition of the thrust force by Shawyer ( Shawyer defines the thrust force in the opposite direction as to the movement) is highly unorthodox and Shawyer's force arguments are inconsistent, but anyway, as I discuss above, forces can never be measured directly: what can be measured are displacements, strains or accelerations.  The acceleration as defined by Shawyer in Fig. 2 is entirely consistent with the movement in the video.
Title: Re: EM Drive Developments - related to space flight applications - Thread 2
Post by: Stormbringer on 01/04/2015 07:23 pm
at the risk of raising the ire or EM thruster purists; this is the time to ask:

only apparent in a dynamic environment, only apparent in excelleration and so forth...

and i noticed that thing i posted in the warp thread thrusts in the opposite direction it should by Newtonian physics. The direction of the induced motion is in the same direction the radiated energy travels. (If I understood it right.)

don't these things kind of resemble "vis inert" or "fictitious" forces?

If not; forgive me, I apparently have that book passage on my mind now, perhaps to uncalled for extremes.
Title: Re: EM Drive Developments - related to space flight applications - Thread 2
Post by: IslandPlaya on 01/04/2015 09:19 pm
...and your point is?
Title: Re: EM Drive Developments - related to space flight applications - Thread 2
Post by: Notsosureofit on 01/04/2015 09:24 pm
Had a couple minutes.  Just thought I'd check the consistency of the dispersion idea against the Greg Egan cavity.  No problem.  It's not a general expression, he has the numeric results for a few modes, but that's enough.
Title: Re: EM Drive Developments - related to space flight applications - Thread 2
Post by: Stormbringer on 01/04/2015 10:19 pm
...and your point is?
who me? You trying to get me in trouble? ;)

well... I guess it would mean that the EM bit is somewhat misleading. The real mechanism would be from some ordinarily inert and undetectable source that might be non-local or global in nature. He who shall not be named in this thread would suggest the gravitational influence of mostly distant matter. But that is probably far from the only candidate among so called fictitious fields. The EM details, asymmetric capacitors or wobbly cams or whatever was used to create a dynamic environment would only be acting like the small voltage that biases a transistor or the current that pull the contact arm down in a relay. Once you have an *appropriate* dynamic system  the "fictitious" fields would stop pretending to not exist and make themselves known. It would be the aroused dead energy/vis inert that creates the motive force and not necessarily the EM put through the device.

Title: Re: EM Drive Developments - related to space flight applications - Thread 2
Post by: IslandPlaya on 01/04/2015 10:22 pm
Thank you SB. That makes it all as clear as mud.
Title: Re: EM Drive Developments - related to space flight applications - Thread 2
Post by: Stormbringer on 01/04/2015 10:33 pm
Oh great; then we're both at about the same level of understanding on my ramblings then :)

The references to the effect only being evident when the system was in a dynamic state and others led me to think that that is how inert or dead energies or fictitious energies are described and to speculate that if they sound the same they might be the same. I think it would explain how you can have half a hundred different design approaches that somehow get comparable results. I could go on but I doubt it would be appreciated.
Title: Re: EM Drive Developments - related to space flight applications - Thread 2
Post by: frobnicat on 01/05/2015 01:03 am
ME thrusters are exploring quite a different range of "bulk acceleration" magnitudes (more than 1000s of Gs if I recall, back and forth around the piezos) compared to a freely moving EM drive, horizontally (a few milli Gs ?) or vertically on a scale (1G)... And the "thrusts" (equivalently M times measured a, to deal with recently introduced "special Newtons") should be roughly impacted by a few order of magnitudes less for EM devices than ME devices. That is, if the same "dynamic setting effect" is at play...

Beyond unorthodox pseudo-forces orientation conventions, this "dynamic environment" condition seems quite problematic and ill defined. Ignoring theoretical musings, Shawyer makes 3 phenomenological predictions :
- That a vertical EM thruster on a scale will record no thrust (unless the thruster is efficient enough to fly against 1G)  page 2 "Thus any attempt to measure them by simply placing the thruster vertically on a set of scales will fail"
- That a horizontal EM thruster restrained from accelerating horizontally (through an opposing spring) will record no thrust, page 3 figure 3 "Because the thruster is at rest, no force will be measured on the load cell"
- That a horizontal EM thruster free to move will accelerate horizontally, page 2 figure 2 "In free space, the thruster will simply accelerate at a m/s/s" In this case we are not in free space but on the horizontal axis this is all the same (and claimed results by Shawyer of accelerations on horizontally rotating arms confirm this view from the author)


So if one thing is clear it is that, for him, the definition of "being at rest" is the same whether not moving on a 0G axis (sorry for the ugly wording) as in restrained horizontal situation or not moving in altitude at fixed G pulling, as in vertical situation. Well, maybe that is the common sense most non physicists give to the intuitive concept of "being at rest", but the 2 situations are very different from what equivalence principle would tell : on a horizontal axis a "static object" shows no departure from an inertial rest frame, while on a vertical axis a "static object" (constant altitude) shows an accelerating departure from an inertial rest frame (free falling). Telling the two situations have the same consequence (no differential stressing of springs between device on and off) is either saying that the "effect" does not depend on acceleration relative to inertial rest frames (that is : it is always 0, there is no effect !) or that it breaks equivalence principle.

All right then, equivalence principle is strong but this is not a sacred cow, we can sacrifice it if it's worth. But then there are problems... how can we define "at rest" vs "dynamic environment" in a scientifically sane and quantitatively predictive way as far as measurable quantities are concerned ? Taking the 3 experimental predictions of Shawyer together, certainly not relative to distant matter (à la Machian) since it would depend on local gravity.

Now Shawyer can claim that measuring a 0 net difference on a scale's spring is actually a confirmation of the theory... But if EM drives like to be free, why not just mount the thruster freely in a box, let it accelerate, bump into one end of the box (on a spring) so that the box recoils, and the thruster is free again to accelerate and so on. This cycle would convert "freely accelerating thingy" into net linear momentum, averaged. This box, as seen from outside, could either be itself freely accelerating, or having a net pushing on a spring of a restraining scale as any well behaved thruster is expected to be able to do. Call that a pulsed EM drive.
Title: Re: EM Drive Developments - related to space flight applications - Thread 2
Post by: ChrisWilson68 on 01/05/2015 01:23 am
Looking over the last several posts, I see absolutely nothing related to space flight applications.  All I see is more of exactly what got the previous version of this thread to disappear for a while.

Title: Re: EM Drive Developments - related to space flight applications - Thread 2
Post by: DIYFAN on 01/05/2015 02:09 am
Looking over the last several posts, I see absolutely nothing related to space flight applications.  All I see is more of exactly what got the previous version of this thread to disappear for a while.

I would respectfully disagree.  I find the discussion both highly pertinent to possible space flight applications and quite thought provoking.  We must not lose the forest for the trees, but at the same time, it is difficult to comprehend the forest without the trees.
Title: Re: EM Drive Developments - related to space flight applications - Thread 2
Post by: savuporo on 01/05/2015 02:43 am
None of these experiments have demonstrated a linear acceleration: all of them have measured rotational accelerations.  None of the EM Drives have been tested in a vacuum.  None of the measured forces are high enough to levitate the drive.
Forget about levitation. I havent read much or anything about it, but are the forces even in the same ballpark as existing magnetic torquer rods for cubesats ? If yes, in theory this could assist with attitude control in deep space, at least for desaturation.

Anyone ? Anyone ?

I mean, actual spaceflight application. Desaturation spends fuel. Can we get a propellantless desaturation device, with main attitude control provided by reaction wheels ?

Again, from the claimed experimental setups, are the reported angular momentums even on a scale where they could turn a cubesat - even if it takes a long time to do so ? Its not like torquer rods are fast or anything, but they only work within earths magnetic field.

And if it cannot turn a cubesat, the entire thing is no better than Steorn Orbo, is it ?
Title: Re: EM Drive Developments - related to space flight applications - Thread 2
Post by: Rodal on 01/05/2015 04:05 pm
....

Beyond unorthodox pseudo-forces orientation conventions, this "dynamic environment" condition seems quite problematic and ill defined. Ignoring theoretical musings, Shawyer makes 3 phenomenological predictions :
....
- That a horizontal EM thruster restrained from accelerating horizontally (through an opposing spring) will record no thrust, page 3 figure 3 "Because the thruster is at rest, no force will be measured on the load cell"
....

A spring (with finite stiffness) attached to a wall NEVER prevents an object attached to it from accelerating, thus Shawyer's statement is not well stated or it is incorrect.  Only an infinitely stiff (rigid) spring would prevent an object from accelerating.  Otherwise (for finite spring constant) the system will just obey a solution of the second order differential equation:  m d^2x/dt^2 +c dx/dt +  k x = F. 

If the displacement is a function of time, such that the second order derivative of the displacement with respect to time is not zero, there is an acceleration a = d^2x/dt^2 =( F - c dx/dt -  k x )/m  .   In words: the acceleration equals the applied force minus the viscous force, minus the spring force, all divided by the mass. 

If the displacement is not a function of time, we have simply (the "steady-state" solution) F=k*x (force=springConstant*displacement)

In the NASA Eagleworks tests of the truncated cone:

1) The truncated cone EM thruster was restrained by the torsional spring constant of the torsional pendulum used for measurement.  The torsional spring constant effectively acts like a spring attached to a wall (the difference being that in the NASA Eagleworks test we have a torsional pendulum where the EM Drive performs a rotation instead of a rectilinear motion)

2) The truncated cone EM thruster was initially at rest (prior to power input).

3) A time-varying displacement (effectively due to a rotation around the torsional axis) was recorded and thus a force was calculated by NASA Eagleworks using the known constants of the system.

4) We have verified this: we modeled NASA Eagleworks torsional pendulum, and furthermore analyzed the data using Fourier Transforms to obtain the Power Spectral Density and the Autocorrelation of the response.

Thus:

Shawyer's statement "That a horizontal EM thruster restrained from accelerating horizontally (through an opposing spring) will record no thrust" in http://www.emdrive.com/EmDriveForceMeasurement.pdf is not well stated or it is incorrect, as shown, for example, by the experiments carried out at NASA Eagleworks.

Title: Re: EM Drive Developments - related to space flight applications - Thread 2
Post by: Rodal on 01/05/2015 05:31 pm
None of these experiments have demonstrated a linear acceleration: all of them have measured rotational accelerations.  None of the EM Drives have been tested in a vacuum.  None of the measured forces are high enough to levitate the drive.
Forget about levitation. I havent read much or anything about it, but are the forces even in the same ballpark as existing magnetic torquer rods for cubesats ? If yes, in theory this could assist with attitude control in deep space, at least for desaturation.

Anyone ? Anyone ?

I mean, actual spaceflight application. Desaturation spends fuel. Can we get a propellantless desaturation device, with main attitude control provided by reaction wheels ?

...

As an example you can use for calculations, Shawyer reported (for his "Shawyer Demo") a measured force = 0.1023 Newtons (1/10th of a Newton) for a power input of 421 watts.

NASA Eagleworks (Brady et.al) reported (for the TE mode of the truncated cone) a measured force = 0.00005541 Newtons (55.41 MicroNewtons) for a power input of 2.6 watts.
Title: Re: EM Drive Developments - related to space flight applications - Thread 2
Post by: Notsosureofit on 01/05/2015 06:14 pm
I thought I would make up a summary of the dispersion relation approach, as I keep

doing this in bits and pieces.

1. The initial question was:

  In General Relativity (GR), if an RF cavity subject to an accelerating frame of

reference (AFR) displays asymmetric frequency dispersion, will an asymmetric RF

cavity which exhibits wavelength dispersion generate an accelerating frame of

reference.

2. It turned out that the asymmetric cavity dispersion relation is easily solved

for an AFR in which the wavelength dispersion dissapears, being replaced by a

frequency dispersed Doppler pair.  Energy is conserved.

g = (X[subm,n])^2*(c/4*pi^2)*lambda^2*((1/a^2)-(1/b^2))

where a anb b are the end plate radii and the X are the Bessel function zeros.

X[subm,n] = m-th root of dJ[subn](x)/dx = 0

[1,0]=3.83, [1,1]=1.84, [1,2]=3.05, [2,0]=7.02, [2,1]=5.33, [2,2]=8.54, [3,0]

=10.17, etc.

Lambda < cutoff wavelength.

Lambda is the free space wavelength c/f.

giving thrust per photon:

T = (X[subm,n])^2*(h/4*pi^2)*lambda*((1/a^2)-(1/b^2))


3. The surprise was that the force "on the photons" T, which would be required to

maintain that acceleration closely matched the experimental forces reported for

these cavities. The difference from other calculations is that there is a term

dependent on the particular mode of the cavity, (X[subm,n])^2, not just the area

of the end plates.


T = P*Q*(X[subm,n])^2*(1/c*4*pi^2)*lambda^2*((1/a^2)-(1/b^2))

TM211 T=9.84e-5 vs 9.12e-5 P=16.9 Q=7320
TM211 T=2.39e-4 vs 5.01e-5 P=16.7 Q=18100
TE012 T=1.32e-4 vs 5.54e-5 P=2.6  Q=22000


4. So where do things stand?

If the experimental forces are real and repeatable then since the acceleration of

the "Doppler" frame is considered the "Gravitational Potential" in GR.  The

corresponding term in the rest frame could only be an "Electromagnetic Potential",

calculated from the dispersion relation of the cavity.

That is to say, in GR, there is no valid equation here without introducing a cross

term between gravity and electromagnetism.  So, new physics would be required.

The Sachs-Schwebel "quaternion" formulation of GR has a cross term which "looks

like" it might fit, but it's hard for me to evaluate.  There is no shortage of

other theoretical attempts.


5. Momentum:

Conservation of momentum would have to be satisfied with the cross term.  The

suggestion from the experimental results (assuming they are real), that the

required momentum transfer is the same (or nearly the same) as that available from

the rest frame photons, implies that the cross term contains a field of very high

energy density compared to that of the photons. (to "push off" ?)

   

Title: Re: EM Drive Developments - related to space flight applications - Thread 2
Post by: Rodal on 01/05/2015 06:29 pm
I thought I would make up a summary of the dispersion relation approach, as I keep doing this in bits and pieces.
....

Thanks for the summary, much appreciated.

Quote from: Notsosureofit
The difference from other calculations is that there is a term  dependent on the particular mode of the cavity, (X[subm,n])^2, not just the area of the end plates.

Indeed! It is very impressive that not only your calculations are not that far from actual results but that your theory correctly predicts mode dependence and that the magnitude of the mode dependence corresponds with experimental results: the Transverse Electric TE012 mode produces much more [thrust force/input power] than the Transverse Magnetic TM211 mode , which was confirmed by the NASA Eagleworks experiments!  Neither the Shawyer nor the simplified McCulloch equations show this mode-dependence.
Title: Re: EM Drive Developments - related to space flight applications - Thread 2
Post by: RotoSequence on 01/06/2015 01:03 am
I got some help making those equations look pretty with LaTeX, but I'm not sure it's rendered correctly. Are these accurate?

EDIT: Formulas were rendered inaccurately, and have been unlinked.

Thrust per photon:
http://i.imgur.com/V24BnsY.png
\mathrm T = \left( \mathrm X _{m,n} \right) ^2 \left( \frac{\hslash}{4} \pi ^2 \right) \lambda \left( \frac{1}{a^2} - \frac{1}{b^2} \right)

Force on the photons:
http://i.imgur.com/YvP9Zj9.png
\mathrm{T = P} \mathbb Q \left( \mathrm X_{m,n} \right) ^2  \left( \frac{1}{\mathrm{c}} 4 \pi ^2 \right) \lambda^2  \left( \frac{1}{a^2} - \frac{1}{b^2} \right)

Title: Re: EM Drive Developments - related to space flight applications - Thread 2
Post by: Rodal on 01/06/2015 01:25 am
I got some help making those equations look pretty with LaTeX, but I'm not sure it's rendered correctly. Are these accurate?

Thrust per photon:
(http://i.imgur.com/V24BnsY.png)
\mathrm T = \left( \mathrm X _{m,n} \right) ^2 \left( \frac{\hslash}{4} \pi ^2 \right) \lambda \left( \frac{1}{a^2} - \frac{1}{b^2} \right)

Force on the photons:
(http://i.imgur.com/YvP9Zj9.png)
\mathrm{T = P} \mathbb Q \left( \mathrm X_{m,n} \right) ^2  \left( \frac{1}{\mathrm{c}} 4 \pi ^2 \right) \lambda^2  \left( \frac{1}{a^2} - \frac{1}{b^2} \right)

@RotoSequence: thank you for putting this in Latex.



@Notsosureofit:  did you mean to use the "reduced Planck constant", also called "Dirac constant"  hbar as in the Latex equations above or did you mean to use the Planck constant h as per your post ?

where

 h =  hbar * 2 * Pi

Given the de Broglie wavelength λ of a photon and the speed of light c, the energy E of the photon is
 
E = h c / λ = hbar * 2 * Pi * c / λ

It seems to me that you meant

1) to use h (instead of hbar in the Latex equation)

2) the factors of (Pi^2) in the first Latex equation and 4 Pi^2 in the 2nd Latex equation are in an incorrect position: the factors should be  (h/(4 Pi^2)) in the first equation and  (1/(c 4 Pi^2)) in the second equation

Please check, thanks
Title: Re: EM Drive Developments - related to space flight applications - Thread 2
Post by: Notsosureofit on 01/06/2015 01:35 am
g = (X[subm,n])^2*(c/(4*pi^2))*lambda^2*((1/a^2)-(1/b^2))

T = (X[subm,n])^2*(h/(4*pi^2))*lambda*((1/a^2)-(1/b^2))

T = P*Q*(X[subm,n])^2*(1/(c*4*pi^2))*lambda^2*((1/a^2)-(1/b^2))

I'm not where I can double check, but I usually use h and "My Dear Aunt Sally".

Probably should call the last one NT...
Title: Re: EM Drive Developments - related to space flight applications - Thread 2
Post by: RotoSequence on 01/06/2015 01:53 am
Thrust per photon, with Planck's constant instead of the reduced constant: http://i.imgur.com/Lm9OVYD.png

EDIT: The original image has been unlinked because the formula is rendered inaccurately.
Title: Re: EM Drive Developments - related to space flight applications - Thread 2
Post by: Rodal on 01/06/2015 01:56 am
Thrust per photon, with Planck's constant instead of the reduced constant:

(http://i.imgur.com/Lm9OVYD.png)

I think that the factors of (Pi^2) in the first Latex equation and 4 Pi^2 in the 2nd Latex equation are in an incorrect position (they should be in the denominator instead of the numerator): the factors should be  (h/(4 Pi^2)) in the first equation and  (1/(c 4 Pi^2)) in the second equation.
Title: Re: EM Drive Developments - related to space flight applications - Thread 2
Post by: RotoSequence on 01/06/2015 02:06 am
I think that the factors of (Pi^2) in the first Latex equation and 4 Pi^2 in the 2nd Latex equation are in an incorrect position (they should be in the denominator instead of the numerator): the factors should be  (h/(4 Pi^2)) in the first equation and  (1/(c 4 Pi^2)) in the second equation.

Hmm. Are you sure? Oh well, here they are just the same:

EDIT: Added NotSoSureOfIt's third equation to the list:

(http://i.imgur.com/CigE4HR.png)

(http://i.imgur.com/0AT8Jep.png)

(http://i.imgur.com/DmV4xbE.png)

Old versions:

Changed T to NT http://i.imgur.com/kBUnXiA.png
http://i.imgur.com/NjgzBtl.png
Title: Re: EM Drive Developments - related to space flight applications - Thread 2
Post by: Rodal on 01/06/2015 02:09 am
I think that the factors of (Pi^2) in the first Latex equation and 4 Pi^2 in the 2nd Latex equation are in an incorrect position (they should be in the denominator instead of the numerator): the factors should be  (h/(4 Pi^2)) in the first equation and  (1/(c 4 Pi^2)) in the second equation.

Hmm. Are you sure? Oh well, here they are just the same:

(http://i.imgur.com/0AT8Jep.png)

(http://i.imgur.com/NjgzBtl.png)

Yes, thanks.  That's what I think.  But these are @Notsosureofit equations, hopefully he can double check them and see whether he agrees  :)

PS: I agree with Notsosureofit, the second equation would better read NT, where "N" stands for the thrust of all the photons, instead of the thrust of a single photon "T".
Title: Re: EM Drive Developments - related to space flight applications - Thread 2
Post by: RotoSequence on 01/06/2015 02:20 am
PS: I agree with Notsosureofit, the second equation would better read NT, where "N" stands for the thrust of all the photons, instead of the thrust of a single photon "T".

Edited :)

EDIT: And because my friend who showed me LaTeX insists, c4 is becoming 4c ;)

Also added Notsosureofit's third equation, hopefully without error!

(http://i.imgur.com/CigE4HR.png)
Title: Re: EM Drive Developments - related to space flight applications - Thread 2
Post by: Prober on 01/06/2015 08:25 am
Guys, Guys......I stopped reading some time ago.....now we are into the 2nd thread.

Time to "let it go"........prove it one way or another.   The energy spent back and forth could have been put into a cad file, exported into STEP or IGES format by now....(someone please do it)

Then maybe if time permits i'll print out a test model.

Someone then talk maybe to Nanoracks, and lets get it tested. :)
Title: Re: EM Drive Developments - related to space flight applications - Thread 2
Post by: Notsosureofit on 01/06/2015 01:34 pm
Guys, Guys......I stopped reading some time ago.....now we are into the 2nd thread.

Time to "let it go"........prove it one way or another.   The energy spent back and forth could have been put into a cad file, exported into STEP or IGES format by now....(someone please do it)

Then maybe if time permits i'll print out a test model.

Someone then talk maybe to Nanoracks, and lets get it tested. :)

What are STEP and IGES ?
Title: Re: EM Drive Developments - related to space flight applications - Thread 2
Post by: Rodal on 01/06/2015 02:08 pm
Guys, Guys......I stopped reading some time ago.....now we are into the 2nd thread.

Time to "let it go"........prove it one way or another.   The energy spent back and forth could have been put into a cad file, exported into STEP or IGES format by now....(someone please do it)

Then maybe if time permits i'll print out a test model.

Someone then talk maybe to Nanoracks, and lets get it tested. :)

What are STEP and IGES ?

They are graphics file data formats for supposedly "vendor neutral" purposes for digital exchange of CAD (Computer Aided Design) drawings. (As opposed to, for example vendor-specific AutoCad data files, for which you need vendor-provided software like AutoCad to be able to read them).

Imagine if somebody told airplane or rocket developers [if NASA Eagleworks or Shawyer can be compared [?] to the Wright Brothers or Goddard] :"I stopped reading some time ago, it has now been [months (?) for us] since you guys have been writing about this in this forum.  Time to "let it go": give me detailed engineering-quality drawings showing how to make this so that I can fabricate it to test whether this flying machine or rocket does indeed fly"   :)

For Nanoracks vision, see:  http://nanoracks.com/about-us/our-vision/ or this video:


https://www.youtube.com/watch?v=AdtiVFwlXdw

Title: Re: EM Drive Developments - related to space flight applications - Thread 2
Post by: Notsosureofit on 01/06/2015 02:25 pm
Hmmm...

http://www.nature.com/nphys/journal/vaop/ncurrent/full/nphys3226.html

"A self-accelerating electronic wave packet can acquire a phase akin to the Aharonov–Bohm effect, but in the absence of a magnetic field."
"The vector potential in question is a gauge-dependent quantity, namely a mathematical construct whose form is not uniquely defined."

http://www.nature.com/nphys/journal/vaop/ncurrent/full/nphys3196.html

"The Aharonov–Bohm effect predicts that two parts of the electron wavefunction can accumulate a phase difference even when they are confined to a region in space with zero electromagnetic field. Here we show that engineering the wavefunction of electrons, as accelerating shape-invariant solutions of the potential-free Dirac equation, fundamentally acts as a force and the electrons accumulate an Aharonov–Bohm-type phase—which is equivalent to a change in the proper time and is related to the twin-paradox gedanken experiment. This implies that fundamental relativistic effects such as length contraction and time dilation can be engineered by properly tailoring the initial conditions. As an example, we suggest the possibility of extending the lifetime of decaying particles, such as an unstable hydrogen isotope, or altering other decay processes. We find these shape-preserving Dirac wavefunctions to be part of a family of accelerating quantum particles, which includes massive/massless fermions/bosons of any spin."

See:

http://www.nature.com/nphys/journal/vaop/ncurrent/extref/nphys3196-s1.pdf
Title: Re: EM Drive Developments - related to space flight applications - Thread 2
Post by: Rodal on 01/06/2015 02:52 pm
The main criticism against propellant-less EM Drives has been conservation of momentum.

It is discussed here that conservation of momentum for certain effects (like the Aharonov-Bohm Effect) is neither trivial nor has it been satisfactorily derived

Feynman's conservation of momentum paradox and the Aharonov-Bohm Effect

http://digitalcommons.unl.edu/cgi/viewcontent.cgi?article=1129&context=physicsfacpub

Quote
what is the correct force expression for the force on the solenoid and
second, the assumption that Newton’s third law holds in the sense that the
change of the solenoid’s momentum is compensated by the change of the
electron’s momentum. The discussion of “Feynman’s paradox” shows that
the latter is not always the case. It is possible that a change in field momentum
is an essential part of the Aharonov-Bohm discussion, which is exactly
what Aharonov and Casher claim in 1984 [45]. Many theoretical papers have
discussed this issue [16, 17, 36, 37]. These discussions involve imbalanced
forces, field momentum and relativistic terms, all of which are present in our
above discussion. However, none of the discussions gives an explicit and exact
derivation of the delicate balance of all the momentum terms, but often
resort to a treatment of simplified systems. For example, Aharonov and D.
Rohrlich [16] discuss a flux tube with a radially moving charge, instead of a
charge passing by the flux tube. While the issue of whether the charge distribution
of the solenoid is perturbed has been addressed [17, 36, 46], none of
the discussions mention the relativistic electric field imbalance.
As it is possible to describe a solenoid as a collection of moving charged
particles, the above treatment of the Feynman paradox provides hope to settle
the theoretical discussion on forces. Integration over a solenoidal current
distribution would provide an exact derivation of momentum conservation
for the Aharonov-Bohm case.
Title: Re: EM Drive Developments - related to space flight applications - Thread 2
Post by: savuporo on 01/06/2015 03:18 pm
Guys, Guys......I stopped reading some time ago.....now we are into the 2nd thread.
Not being read or heard. There is absolutely nothing related to spaceflight here.

Again, NASA tested what, a 20 watt setup ? That is within power budget of a 3U cubesat. Flight model ? Demonstrate torque, if not directional thrust ?
Title: Re: EM Drive Developments - related to space flight applications - Thread 2
Post by: bad_astra on 01/06/2015 03:26 pm
Agreed, this thread will end up like the last one unless it stays within the realm of spaceflight. which is odd, because space is a great place to test this.
Title: Re: EM Drive Developments - related to space flight applications - Thread 2
Post by: Star One on 01/06/2015 05:30 pm

Guys, Guys......I stopped reading some time ago.....now we are into the 2nd thread.
Not being read or heard. There is absolutely nothing related to spaceflight here.

Again, NASA tested what, a 20 watt setup ? That is within power budget of a 3U cubesat. Flight model ? Demonstrate torque, if not directional thrust ?

I suggest those of you concerned that NASA is not testing this yet in space make your representations to them.:)
Title: Re: EM Drive Developments - related to space flight applications - Thread 2
Post by: JasonAW3 on 01/06/2015 06:20 pm
"With more focus on space flight applications " (as per http://forum.nasaspaceflight.com/index.php?topic=36313.msg1301658#msg1301658) the metric chosen by the NASA's "Anomalous" report was the thrust force per power input.


Here is a comparison of reported measurements for EM Drives and for the latest report by Fearn, Zachar, Woodward & Wanser.

Notice that the force per power input reported by  Fearn, Zachar, Woodward & Wanser is several orders of magnitude lower than the "EM drives".  Actually it is barely (3.5 times higher) more than the force per power input of a photon rocket:

reported measurement ForcePerPowerInput (milliNewtons/kW)

(* Cannae Superconducting *)             761.9 to 952.4
(* Shawyer Demo *)                               80 to 243
(* Shawyer Experimental *)                   18.82
(* Brady c TE mode *)                             21.31
(* Brady a TM mode*)                               5.396
(* Brady b TM mode*)                               3.000
(*Fearn, Zachar, Woodward & Wanser*) 0.01176




lengths in meter
rfFrequency in 1/second (microwave frequency during test)
power in watts
force in milliNewtons
force per PowerInput in milliNewtons/kW
c= 299705000 m/s (speed of light in air)
c= 299792458 m/s (speed of light in vacuum) (for Cannae Superconducting)
(the difference between c in air compared to c in vacuum is negligible)


Note: SmallDiameter for Shawyer's EM Drives obtained from his reported ShawyerDesignFactor .


Force/PowerInput of a Photon Rocket = 1 / c


(* Cannae Superconducting *)
rfFrequency = 1.047*10^9;
cavityLength = 0.01+0.004+0.006+0.01 = 0.03;
bigDiameter =(22.86-2*(0.00430)) = 0.220;
smallDiameter = bigDiameter-2*0.01=0.200;

power =  10.5
Q = 1.1*(10^7)

measured force = 8 to 10
measured ForcePerPowerInput = 761.9 to 952.4
Force/PowerInput of a Photon Rocket =0.003336
measured ForcePerPowerInput to the one of a photon rocket = 228,400 to 285,500


(* Shawyer Experimental *)
rfFrequency=2.45*10^9;
cavityLength=0.156;
bigDiameter=0.16;
smallDiameter=0.127546;

power =  850   
Q = 5900

measured force = 16
measured ForcePerPowerInput = 18.82
Force/PowerInput of a Photon Rocket =0.003337
measured ForcePerPowerInput to the one of a photon rocket =5,640


(* Shawyer Demo *)
rfFrequency=2.45*10^9;
cavityLength=0.345;
bigDiameter=0.28;
smallDiameter= 0.128853

power =  421 to 1200
Q = 45000

(measured force = 102.30 milliNewtons only reported for  421 watts, 243 milliNewtons/kW )

measured ForcePerPowerInput = 80 to 243
Force/PowerInput of a Photon Rocket =0.003337
measured ForcePerPowerInput to the one of a photon rocket =23,980 to 72,830

All Brady cases have the following dimensions:

cavityLength=0.332;
bigDiameter=0.397;
smallDiameter=0.244;


(* Brady a TM mode*)
rfFrequency=1.9326*10^9;

power =   16.9 
Q = 7320

measured force =  0.0912
measured ForcePerPowerInput = 5.396
Force/PowerInput of a Photon Rocket =0.003337
measured ForcePerPowerInput to the one of a photon rocket =1,617.2


(* Brady b TM mode*)   
rfFrequency=1.9367*10^9;

power = 16.7
Q =  18100

measured force = 0.0501
measured ForcePerPowerInput = 3.000
Force/PowerInput of a Photon Rocket =0.003337
measured ForcePerPowerInput to the one of a photon rocket =899.12


  (* Brady c  TE mode *)
rfFrequency = 1.8804*10^9;

power = 2.6
Q = 22000

measured force = 0.05541
measured ForcePerPowerInput = 21.31
Force/PowerInput of a Photon Rocket =0.003337
measured ForcePerPowerInput to the one of a photon rocket =6,386.7


(* Fearn, Zachar, Woodward & Wanser*)
rfFrequency = 39,300;

power =  170

measured force = 0.002
measured ForcePerPowerInput = 0.01176
Force/PowerInput of a Photon Rocket =0.003337
measured ForcePerPowerInput to the one of a photon rocket = 3.526

Ok, let's break it down a bit simpler.

How many units of energy are being used to produce how many units of thrust?  Newtons as the unit of measure could work.
Title: Re: EM Drive Developments - related to space flight applications - Thread 2
Post by: Mulletron on 01/06/2015 06:47 pm
None of these experiments have demonstrated a linear acceleration: all of them have measured rotational accelerations.  None of the EM Drives have been tested in a vacuum.  None of the measured forces are high enough to levitate the drive.
Forget about levitation. I havent read much or anything about it, but are the forces even in the same ballpark as existing magnetic torquer rods for cubesats ? If yes, in theory this could assist with attitude control in deep space, at least for desaturation.

Anyone ? Anyone ?

I mean, actual spaceflight application. Desaturation spends fuel. Can we get a propellantless desaturation device, with main attitude control provided by reaction wheels ?

Again, from the claimed experimental setups, are the reported angular momentums even on a scale where they could turn a cubesat - even if it takes a long time to do so ? Its not like torquer rods are fast or anything, but they only work within earths magnetic field.

And if it cannot turn a cubesat, the entire thing is no better than Steorn Orbo, is it ?

Trying to elaborate on desaturation which savuporo brought up as a possible application. Here's more info concerning the problem:

http://space.stackexchange.com/questions/323/how-often-must-the-iss-desaturate-its-control-moment-gyros
http://en.wikipedia.org/wiki/Control_moment_gyroscope#International_Space_Station
http://spacestationlive.jsc.nasa.gov/displays/adcoDisplay3.html
http://ntrs.nasa.gov/archive/nasa/casi.ntrs.nasa.gov/20100024204.pdf  slide 21-23 especially.

I remember reading about VASIMR the other day and found that NASA spends $210 million annually in fuel costs just to reboost the ISS. So there's the above potential application, and the cost savings which are potential (if it works) space flight applications.

Guys, Guys......I stopped reading some time ago.....now we are into the 2nd thread.
Not being read or heard. There is absolutely nothing related to spaceflight here.

Again, NASA tested what, a 20 watt setup ? That is within power budget of a 3U cubesat. Flight model ? Demonstrate torque, if not directional thrust ?

I read your post and you are being heard. Please give this unproven technology some time to fail or fly. I don't know what it can do yet. For all we know EMdrives could all be a mistake.
Title: Re: EM Drive Developments - related to space flight applications - Thread 2
Post by: Rodal on 01/06/2015 09:41 pm
....

Ok, let's break it down a bit simpler.

How many units of energy are being used to produce how many units of thrust?  Newtons as the unit of measure could work.

Energy can be expressed in units of Force*displacement (for example, Joule=Newton*meter), and Thrust is a Force of course.  Whether one uses Newtons, KilogramForce, PoundForce, or any other (arbitrary but consistent) unit to measure Force, the unit of Energy/Thrust is the unit of length, because Energy/Thrust = Force*displacement/Force = displacement, and displacement has units of length.

Regarding the experiments, it took a given amount of input power to produce a constant EM Drive force.
For example,Shawyer reported (for his "Shawyer Demo") a measured force = 0.1023 Newtons (1/10th of a Newton) for a power input of 421 watts.  NASA Eagleworks (Brady et.al) reported (for the TE mode of the truncated cone) a measured force = 0.00005541 Newtons (55.41 MicroNewtons) for a power input of 2.6 watts.


Since Energy is the integral of Power with respect to time, the amount of input Energy required to produce the (approximately constant) thrust force, at (effectively) constant power, was (effectively) a linearly increasing function of time

So, it is Power/Thrust -which has units of velocity- (or inversely, Thrust/Power) rather than Energy/Thrust that NASA Eagleworks (Brady, March, White et.al.) used (at its own peril) to scale-up for their proposed EM Drive missions to Mars,  Enceladus, etc.



EDIT: Notice that NASA Eagleworks assumed a 2 MegaWatt Power Input for the crewed Mars and Enceladus missions.  A 2 MegaWatt Power Input seems to require a Nuclear Power Input in the spacecraft.
For the Mars and Enceladus crewed missions they assumed 0.4 N/kW Thrust/PowerInput, which is certainly feasible for a Superconducting EM Drive (the Canae Superconducting experiment reported almost 1 N/kW). Certainly much more conservative than Shawyer's 30000 N/kW assumption used in his projection for a superconducting Shawyer flying car (which assumes a Q of 5 billion). 

(http://4.bp.blogspot.com/-i8ERUjGAbzw/U-F3jAwLrRI/AAAAAAAAxLo/4savWVpmFIM/s1600/nasatestemdrive7.png)
Title: Re: EM Drive Developments - related to space flight applications - Thread 2
Post by: Mulletron on 01/06/2015 10:12 pm
Throwing around ideas/stuff to think about here:
Since the thrust/power ratio of these devices is pretty bad, they are severely limited by the amount of electrical power you can put on orbit.

Here's what I found about the ISS:

" The complete power system, consisting of U.S. and Russian hardware, generates 110 kilowatts (kW) total power, about as much as 55 houses would typically use. Approximately 30 kW are available for research activities."
Source: http://www.nasa.gov/centers/glenn/about/fs06grc.html

Probes:
Cassini launched with 3 RTGs providing ~300W electrical power each. http://spacemath.gsfc.nasa.gov/Modules/8Mod7Prob2.pdf http://saturn.jpl.nasa.gov/spacecraft/safety/solar.pdf
New Horizons launched with a spare Cassini RTG providing ~250W electrical power, 200 at encounter with Pluto. http://en.wikipedia.org/wiki/New_Horizons or 290W/230W at arrival after 9.5 years according to here: http://www.boulder.swri.edu/~tcase/Ottman-Hersman_IECEC_paper.pdf

The newer RTGs, MMRTGs like the on in Curiosity provide ~125W/100W energy after 14 years: http://en.wikipedia.org/wiki/Multi-Mission_Radioisotope_Thermoelectric_Generator
http://en.wikipedia.org/wiki/Curiosity_%28rover%29#Specifications
http://mars.jpl.nasa.gov/files/mep/MMRTG_Jan2008.pdf

So we really don't have very much electrical power to play with in space.

Factoring in efficiencies such as what SLAC (most powerful klystrons I could find, while thinking about manned missions) reports on their Klystrons; http://www.slac.stanford.edu/cgi-wrap/getdoc/slac-pub-10620.pdf the best I see is around 70%.

Another efficient amplifier capable of very high rf power output, a CFA:
http://www.radartutorial.eu/08.transmitters/Crossed-Field%20Amplifier%20%28Amplitron%29.en.html

The future? More power.
Two interesting things I found about high power space flight projects at none other than JSC. What is also interesting is that both of these mention Q-thrusters.

http://ntrs.nasa.gov/archive/nasa/casi.ntrs.nasa.gov/20140004802.pdf
http://ntrs.nasa.gov/archive/nasa/casi.ntrs.nasa.gov/20140004416.pdf
Title: Re: EM Drive Developments - related to space flight applications - Thread 2
Post by: CW on 01/07/2015 09:21 am
Let's better hope that compact (preferably p-B11 direct-conversion) fusion generators will be implemented soon. Lockheed Martin et al. are working on these things and LM have announced a prototype reactor (100MW class) for around 2019/20. 100MW should be enough for a little spaceship. I wonder when new experimental results will be published by NASA.
Title: Re: EM Drive Developments - related to space flight applications - Thread 2
Post by: wembley on 01/07/2015 12:06 pm
Throwing around ideas/stuff to think about here:
Since the thrust/power ratio of these devices is pretty bad, they are severely limited by the amount of electrical power you can put on orbit.

If it works as Shawyer claims, the superconducting version will be producing far more thrust:

http://emdrive.com/faq.html

"The second generation engines will be capable of producing a specific thrust of 30kN/kW. Thus for 1 kilowatt (typical of the power in a microwave oven) a static thrust of 3 tonnes can be obtained, which is enough to support a large car. This is clearly adequate for terrestrial transport applications.
 The static thrust/power ratio is calculated assuming a superconducting EmDrive with a Q of 5 x 109. This Q value is routinely achieved in superconducting cavities."
Title: Re: EM Drive Developments - related to space flight applications - Thread 2
Post by: Rodal on 01/07/2015 12:15 pm
Throwing around ideas/stuff to think about here:
Since the thrust/power ratio of these devices is pretty bad, they are severely limited by the amount of electrical power you can put on orbit.

If it works as Shawyer claims, the superconducting version will be producing far more thrust:

http://emdrive.com/faq.html

"The second generation engines will be capable of producing a specific thrust of 30kN/kW. Thus for 1 kilowatt (typical of the power in a microwave oven) a static thrust of 3 tonnes can be obtained, which is enough to support a large car. This is clearly adequate for terrestrial transport applications.
 The static thrust/power ratio is calculated assuming a superconducting EmDrive with a Q of 5 x 109. This Q value is routinely achieved in superconducting cavities."
(http://4.bp.blogspot.com/-IOhb45CEI9I/URSurOhJNPI/AAAAAAAAh5M/c5oSLgis9qE/s640/emdrivespaceplane3.png)

You meant to write "assuming a superconducting EmDrive with a Q of 5 * 10^9" = 5,000,000,000 (five billion) (rather than Q = 5 x 109 =545).


In his IAC 2014 October 2014 presentation Shawyer (in his slide #3 in http://www.emdrive.com/iac2014presentation.pdf ) instead uses a projected Q =  5*(10^7), which is 100 times less than the value assumed by Shawyer for the above 30kN/kW projection.


Reviewing actual published data, the maximum Q measurement for a superconducting EM Drive I have seen reported is for the Canae superconducting drive experiment which gave a Q = 1.1*(10^7), this is 500 times less than the value assumed by Shawyer for the above 30kN/kW projection.

Also comparing with actual experimental data, Canae's measured ForcePerPowerInput for the superconducting Canae experiment was only 0.7619 to 0.9524 N/kW, which is 31500 times less than the 30kN/kW projection, therefore besides the decrease in thrust by a factor of 500 due to the lower actual Q, there was another factor decreasing thrust (by a factor of 63 times) in the actual experimental reported results (500*63=31500) as compared to Shawyer's optimistic 30kN/kW projection..
Title: Re: EM Drive Developments - related to space flight applications - Thread 2
Post by: RotoSequence on 01/07/2015 12:48 pm
You meant to write "assuming a superconducting EmDrive with a Q of 5 * 10^9" = 5,000,000,000 (five billion) (rather than Q = 5 x 109 =545).

In his IAC 2014 October 2014 presentation Shawyer (in his slide #3 in http://www.emdrive.com/iac2014presentation.pdf ) instead uses a projected Q =  5*(10^7), which is 100 times less than the value assumed by Shawyer for the above 30kN/kW projection.


Reviewing actual published data, the maximum Q actually measured for a superconducting EM Drive I have seen reported is for the Canae superconducting drive experiment which gave a Q = 1.1*(10^7), this is 500 times less than the value assumed by Shawyer for the above 30kN/kW projection.

Also comparing with actual experimental data, Canae's measured ForcePerPowerInput for the superconducting Canae experiment was only 0.7619 to 0.9524 N/kW, which is 31500 times less than the 30kN/kW projection, therefore besides the decrease in thrust by a factor of 500 due to the lower actual Q, there was another factor of decrease in thrust (by a factor of 63 times) in the actual performance of the experiment (500*63=31500).

On the plus side, with the right power source, 0.7619 Newtons from a kilowatt of electricity is still, probably, enough thrust to displace rockets for orbital insertion. It's not competing with airplanes, but it is an exciting number.  :)
Title: Re: EM Drive Developments - related to space flight applications - Thread 2
Post by: Rodal on 01/07/2015 02:56 pm
You meant to write "assuming a superconducting EmDrive with a Q of 5 * 10^9" = 5,000,000,000 (five billion) (rather than Q = 5 x 109 =545).

In his IAC 2014 October 2014 presentation Shawyer (in his slide #3 in http://www.emdrive.com/iac2014presentation.pdf ) instead uses a projected Q =  5*(10^7), which is 100 times less than the value assumed by Shawyer for the above 30kN/kW projection.


Reviewing actual published data, the maximum Q actually measured for a superconducting EM Drive I have seen reported is for the Canae superconducting drive experiment which gave a Q = 1.1*(10^7), this is 500 times less than the value assumed by Shawyer for the above 30kN/kW projection.

Also comparing with actual experimental data, Canae's measured ForcePerPowerInput for the superconducting Canae experiment was only 0.7619 to 0.9524 N/kW, which is 31500 times less than the 30kN/kW projection, therefore besides the decrease in thrust by a factor of 500 due to the lower actual Q, there was another factor of decrease in thrust (by a factor of 63 times) in the actual performance of the experiment (500*63=31500).

On the plus side, with the right power source, 0.7619 Newtons from a kilowatt of electricity is still, probably, enough thrust to displace rockets for orbital insertion. It's not competing with airplanes, but it is an exciting number.  :)

Agreed. 

For the Mars and Titan/Enceladus crewed missions NASA Eagleworks assumed 0.4 N/kW Thrust/PowerInput, which is certainly feasible for a Superconducting EM Drive (the Canae Superconducting experiment reported almost 1 N/kW). Certainly much more conservative than Shawyer's 30000 N/kW assumption used in his projection for a superconducting Shawyer flying car (which assumes a Q of 5 billion).

(NASA Eagleworks also assumed a 2 MegaWatt Power Input for the crewed Mars and Titan/Enceladus missions,  which implies Nuclear Power generation available in the spacecraft.)

(http://i.guim.co.uk/static/w-620/h--/q-95/sys-images/Guardian/Pix/pictures/2012/4/25/1335371334433/Artists-impression-of-pro-009.jpg)

(http://www.esa.int/var/esa/storage/images/esa_multimedia/images/2014/04/inside_enceladus/14362029-1-eng-GB/Inside_Enceladus_node_full_image_2.jpg)

EDIT: and the Chinese (Prof. Juan Yang) have reported (for their ambient-temperature non-superconducting EM Drive) experimental measurements of 0.29 N/kW Thrust/PowerInput, which is not too far off (-28% difference) from the 0.4 N/kW Thrust/PowerInput assumed by NASA Eagleworks for the Mars and Enceladus crewed missions.
Title: Re: EM Drive Developments - related to space flight applications - Thread 2
Post by: Prober on 01/08/2015 04:28 am

Guys, Guys......I stopped reading some time ago.....now we are into the 2nd thread.
Not being read or heard. There is absolutely nothing related to spaceflight here.

Again, NASA tested what, a 20 watt setup ? That is within power budget of a 3U cubesat. Flight model ? Demonstrate torque, if not directional thrust ?

I suggest those of you concerned that NASA is not testing this yet in space make your representations to them.:)

Nah, just point out the opportunities out there.
 :o
Title: Re: EM Drive Developments - related to space flight applications - Thread 2
Post by: savuporo on 01/08/2015 05:02 am
So when in a reasonable setup with a 20 watts of input power, you are hoping to generate zero point zero zero zero zero zero and then some point oh millinewtons of linear thrust, or torque, doesn't really matter ( again, see xkcd #1404 ) .. any mentions of levitating cars and interstellar travel is just eroding the credibility of the whole thing.

Cubesat , even a whopping 3U one is not that difficult to get to orbit, and is demonstrably well within the capabilities of crowdfunding even. Getting a NIAC grant also works, although SBIR might be a stretch, at this point.
Title: Re: EM Drive Developments - related to space flight applications - Thread 2
Post by: Stormbringer on 01/08/2015 05:35 am

The future? More power.
Two interesting things I found about high power space flight projects at none other than JSC. What is also interesting is that both of these mention Q-thrusters.

http://ntrs.nasa.gov/archive/nasa/casi.ntrs.nasa.gov/20140004802.pdf
http://ntrs.nasa.gov/archive/nasa/casi.ntrs.nasa.gov/20140004416.pdf

it may be that i have an outdated reader but those pdfs fail to load for me. the first gets 2/3s of the way loaded according to the progress bar and then hangs. the second gets 1/4 of the way loaded and then hangs.

:(
Title: Re: EM Drive Developments - related to space flight applications - Thread 2
Post by: savuporo on 01/08/2015 05:43 am
it may be that i have an outdated reader but those pdfs fail to load for me. the first gets 2/3s of the way loaded according to the progress bar and then hangs. the second gets 1/4 of the way loaded and then hangs.

:(
NTRS does that a lot with firefox, try chrome or curl/wget
Title: Re: EM Drive Developments - related to space flight applications - Thread 2
Post by: Stormbringer on 01/08/2015 05:44 am
it may be that i have an outdated reader but those pdfs fail to load for me. the first gets 2/3s of the way loaded according to the progress bar and then hangs. the second gets 1/4 of the way loaded and then hangs.

:(
NTRS does that a lot with firefox, try chrome or curl/wget
Thanks. :)
Title: Re: EM Drive Developments - related to space flight applications - Thread 2
Post by: Rodal on 01/09/2015 12:23 am
In this report http://ntrs.nasa.gov/archive/nasa/casi.ntrs.nasa.gov/20110023492.pdf NASA's Dr. White wrote of his continuing "dialogue with the (International Space Station) ISS national labs office for an on orbit DTO (On Orbit Detailed Test Objectives)" of the EM Drive (which he calls "Q-Thruster"):

Quote from: Dr. Harold “Sonny” White, Paul March, Nehemiah Williams, William O’Neill
The near term focus of the laboratory work is focused on gathering performance data to support development of a Q-thruster engineering prototype targeting Reaction Control System (RCS) applications with force range of 0.1-1 N with corresponding input power range of 0.3-3 kW. Up first will be testing of a refurbished test article to duplicate historical performance on the high fidelity torsion pendulum (1-4 mN at 10-40 W). The team is maintaining a dialogue with the ISS national labs office for an on orbit DTO.
How would Q-thrusters revolutionize human exploration of the outer planets? Making minimal extrapolation of performance, assessments show that delivery of a 50 mT payload to Jovian orbit can be accomplished in 35 days with a 2 MW power source [specific force of thruster (N/kW) is based on potential measured thrust performance in lab, propulsion mass (Q-thrusters) would be additional 20 mT (10 kg/kW), and associate power system would be 20 mT (10 kg/kW)]. Q-thruster performance allows the use of nuclear reactor technology that would not require MHD conversion or other more complicated schemes to accomplish single digit specific mass performance usually required for standard electric propulsion systems to the outer solar system. In 70 days, the same system could reach the orbit of Saturn.

(http://www.delhidailynews.com/news_image/1415199916ISS.jpg)
Title: Re: EM Drive Developments - related to space flight applications - Thread 2
Post by: Econocritic on 01/09/2015 03:23 am
In this report http://ntrs.nasa.gov/archive/nasa/casi.ntrs.nasa.gov/20110023492.pdf NASA's Dr. White wrote of his continuing "dialogue with the (International Space Station) ISS national labs office for an on orbit DTO (On Orbit Detailed Test Objectives)" of the EM Drive (which he calls "Q-Thruster"):


The "Q-Thruster" in this case appears to be a Woodward/ME device.   
Title: Re: EM Drive Developments - related to space flight applications - Thread 2
Post by: ThinkerX on 01/09/2015 04:15 am
Quote
The "Q-Thruster" in this case appears to be a Woodward/ME device.   

Yes.  No.  Sort of.

Despite claimed differing operating principles, the 'Q-Thruster' and Woodward/ME device appear to differ in minor detail,  in terms of construction and results.  At least that is my (possibly badly flawed) understanding.

 Heated debates about this in the old thread contributed to its demise. We attempt to keep Woodward/Mach out of this thread, but the equations and thoughts keep heading that way.
Title: Re: EM Drive Developments - related to space flight applications - Thread 2
Post by: Mulletron on 01/09/2015 09:23 am
In this report http://ntrs.nasa.gov/archive/nasa/casi.ntrs.nasa.gov/20110023492.pdf NASA's Dr. White wrote of his continuing "dialogue with the (International Space Station) ISS national labs office for an on orbit DTO (On Orbit Detailed Test Objectives)" of the EM Drive (which he calls "Q-Thruster"):
The "Q-Thruster" in this case appears to be a Woodward/ME device.
Quote
The "Q-Thruster" in this case appears to be a Woodward/ME device.   

Yes.  No.  Sort of.

Despite claimed differing operating principles, the 'Q-Thruster' and Woodward/ME device appear to differ in minor detail,  in terms of construction and results.  At least that is my (possibly badly flawed) understanding.

 Heated debates about this in the old thread contributed to its demise. We attempt to keep Woodward/Mach out of this thread, but the equations and thoughts keep heading that way.

Yeah it's a MLT. You can tell by the pixels caps. Essentially when these devices get through the door over at Eagleworks, it seems the favored theory of operation for all these devices becomes the White model. It is his house after all. And I believe this is for good reason.

Paul March seems to believe these devices are related, yet their theories diverge.

His actual words, "Dr. Woodward maintains that the M-E's mass fluctuations occur in the "squishy" intermolecular chemical bonds of the dielectric and not in the rest mass of the ions in question. Next question is what are these squishy intermolecular chemical bonds made of? They are typically called covalent sharing of molecular electrons and/or an imbalance of ionic electric charges between the charged ions. Ok then what is in between the electrons and ions in these dielectric molecules that is affected by the M-E equation's transient gravity waves, or in other words what do the M-E's pressure transients in the cosmological gravitational field affect in between the molecules that for all practical purposes is a pure vacuum state. A vacuum state filled only with virtual photons of the electric fields and perhaps the virtual e/p pairs of the quantum vacuum. That is why I continue to say that Dr. White in only trying to answer what Woodward's M-E "gravity" pressure waves are effecting at the molecular and subatomic scales. A place that Dr. Woodward refuses to go to this date except perhaps in his musings on the ADM electron structure where the gravitational field is used to counter balance the electrostatic field forces, but once again ignoring the basic question of what either of these fields are composed of. That is supposed to be the realm of quantum gravity, but since no one has come up with an accepted answer for same, Dr. White is free to suggest his own."

The issue is discussed here: http://www.talk-polywell.org/bb/viewtopic.php?t=2949&p=115985 He goes on to mention that Woodward doesn't address some issues relating to ME and how it effects covalent bonds and because of that, Dr. White is free to make his own conclusions to fill in the gaps. Further on in the thread, it becomes clear that the supposed ME/EM dichotomy really boils down to what the origin of inertia is.

Now enter EMdrive developments thread 1 and 2, and a thorough research of the inertia issue using all available evidence, it was concluded by myself and others that these devices do in fact share the same mechanism of operation and that the theories supporting both types (Woodward, White, Shawyer, Fetta) had their own unique issues, but no dichotomy was required.

The same effect was happening with varying success (http://forum.nasaspaceflight.com/index.php?topic=36313.msg1302455#msg1302455) in each of these devices and this success was directly informed by design, further informed by theory held by the inventors. As very recently supported by Dr. Rodal's research, here: http://forum.nasaspaceflight.com/index.php?topic=36313.msg1307817#msg1307817, inertia isn't simply caused by one single thing. Evidence keeps piling up which supports we need to cast off old theories and think differently about things.

All this didn't sit well with some of the Woodward fans, who were very much welcome to participate in the discussion in thread 1. Given the emotional attachment to the theories, which lead to bad behavior by some and then more bad behavior in response from myself included; I support the attempt to keep Woodward/Mach out of this thread. Besides they have their own happy home here on NSF too here: http://forum.nasaspaceflight.com/index.php?topic=13020.1830 and here http://forum.nasaspaceflight.com/index.php?topic=31037.465

Ok, so how does all of this I just banged out relate to space flight applications I ask myself? It is the same as trying to build a better mouse trap. If a lab or inventor is hell bent on building a device that interacts with theoretical X in Y fashion, trying to achieve thrust, to the exclusion of all other ideas, and it continues to not work so well for them...., we're going to go nowhere fast. I think the attachment sums this up nicely. The truth is buried within the noise. Find it? Then we'll have some all electric space flight action.

From what I'm seeing coming out of Eagleworks, they are using sound scientific methods to test/develop these (still not completely vetted) theories and more importantly they are using lessons learned and applying them to the future. For example, in the Eagleworks Newsletter 2013 (https://xa.yimg.com/kq/groups/86787010/513081407/name/Eagleworks+Newsletter+2013.pdf), they concluded the "magnitude of the thrust is dependent on the AC content of the turn-on and turn-off pulse" in the first device and further concluded "...confirms that the RF approach will be a quicker path to long-life flight thrusters" with the Cannae. This shows that an AC signal is important, a key insight.

Next came the Anomalous thrust production...(http://www.libertariannews.org/wp-content/uploads/2014/07/AnomalousThrustProductionFromanRFTestDevice-BradyEtAl.pdf) paper, which continued using the RF approach.

Most importantly what they didn't do was the same thing over and over and over again.
Title: Re: EM Drive Developments - related to space flight applications - Thread 2
Post by: Rodal on 01/09/2015 11:55 am
In this report http://ntrs.nasa.gov/archive/nasa/casi.ntrs.nasa.gov/20110023492.pdf NASA's Dr. White wrote of his continuing "dialogue with the (International Space Station) ISS national labs office for an on orbit DTO (On Orbit Detailed Test Objectives)" of the EM Drive (which he calls "Q-Thruster"):


The "Q-Thruster" in this case appears to be a Woodward/ME device.

Quote
The "Q-Thruster" in this case appears to be a Woodward/ME device.   

Yes.  No.  Sort of.
....

1) The example of "Q-Thruster" pictured in the report is what's called by Woodward followers a "Mach-Lorentz thruster (MLT)".  As such, it is not a Woodward Mach Effect (Piezoelectric) device.  The  Mach-Lorentz thruster (MLT) uses a charging capacitor embedded in a magnetic field created by a magnetic coil. It is claimed that a Lorentz force, cross product between the electric field and the magnetic field, appears and acts upon the ions inside the capacitor dielectric. In such electromagnetic experiments, the power can be applied at frequencies of several megahertz, unlike piezoelectric PZT stack actuators where frequency is presently limited to tens of kilohertz.  This kind of Mach Lorentz thruster (MLT) was nullified by the experiments of Brito, Marini and Galian (who used a classic Cavendish type pendulum where all the power supply was self-contained with the MLT device).  The nullifying experiments of Brito Marini and Galian (http://enu.kz/repository/2009/AIAA-2009-5070.pdf) where repeated and published in a peer-reviewed AIAA journal (Journal of Propulsion and Power) (http://arc.aiaa.org/doi/abs/10.2514/1.46541?journalCode=jpp).  Woodward has instead concentrated on the Mach Effect (Piezoelectric) thruster.

2) To concentrate on spaceflight applications (the subject of this thread) it is of paramount importance the amount of power required to produce a given level of thrust. Notice that the force per power input reported by Woodward's Mach Effect thruster in their latest reported experiments (Fearn, Zachar, Woodward & Wanser) is several orders of magnitude lower than the "EM drives".  Actually it is barely (3.5 times higher) more than the force per power input of a photon rocket (using a military searchlight as a means of propulsion ! ):

reported measurement ForcePerPowerInput (milliNewtons/kW)

MICROWAVE (* Cannae Superconducting *)             761.9 to 952.4
MICROWAVE (* Prof. Juan Yang et.al. China*)            290
MICROWAVE (* Shawyer Demo *)                               80 to 243
MICROWAVE (* Shawyer Experimental *)                   18.82
MICROWAVE (* Brady c TE mode *)                             21.31
MICROWAVE (* Brady a TM mode*)                               5.396
MICROWAVE (* Brady b TM mode*)                               3.000
PIEZOELECTRIC (*Fearn, Zachar, Woodward & Wanser*) 0.01176

3) To understand what is important for spaceflight applications: how small is the force (for a given power input) produced by Woodward's (Fearn, Zachar, Woodward & Wanser) latest Mach Effect experiments: it takes 80000 times greater power to produce the same level of force with the Woodward's (Fearn, Zachar, Woodward & Wanser) device as the superconducting EM Drive.  Also for spaceflight applications comparison, notice that while Dr. White calculated that it would take 2 MW power input (nuclear power generation) for the microwave EM Drive at 0.4 N/kW missions to Mars and Titan/Enceladus, it would take 68 *(10 ^9) watts (68 Gigawatts) for Woodward's (Fearn, Zachar, Woodward & Wanser) device to produce the same thrust, given its tiny 0.00001176 N/kW thrust/PowerInput. (Fearn, Zachar, Woodward & Wanser device gives just 3.5 times the thrust/powerInput of a very inefficient classical-physics means of propellant-less propulsion: using a military searchlight as a photon-rocket ) . 

4) Thus, it is evident, that for spaceflight applications  (the subject of this thread) we should concentrate on the devices that require orders of magnitude (up to  80000 times) less power to produce a given level of thrust: the microwave EM Drives.

5) Dr. White used the term "Q-thruster" in general for all of these devices (including the MLT Thruster, Boeing's Serrano Effect Device and the microwave EM drives).  See the slide by Dr. White enclosed below.  For the above-pointed (and other) reasons, Dr. White at NASA Eagleworks has moved on and is presently concentrating on Microwave-Cavity Drives, as the reported experimental data shows that they are much more promising for spaceflight applications.  This makes sense.

6) The point is that Dr. White wrote that he has a dialogue with the (International Space Station) ISS national labs office for an on orbit DTO (On Orbit Detailed Test Objectives)" of the EM Drive (which he calls "Q-Thruster") and therefore his plan is to eventually test the EM Drive in the ISS: which would obviously constitute a spaceflight demonstration.

Title: Re: EM Drive Developments - related to space flight applications - Thread 2
Post by: tchernik on 01/09/2015 04:03 pm
Great posts by Rodal and Mulletron.

And I concur: the next step is to firmly cement the scientific nature of the mentioned thrusters (them being falsifiable), by providing additional replications and tests. Theories can come later, if empiric evidence is there.

It doesn't matter if H. White's theory is right or wrong, the same as Woodward's (or Shawyer's, which is most likely wrong). What matters is that the phenomena related to those theories would have evidence of being real.

Humanity has used real phenomena nobody can explain for millenia, and this way before even having complete formal descriptions/theories for them. We still do, as we don't know the provenance and nature of inertia and gravity, and they are part of our everyday lives and expected to work for a lot of technology to function as well.
Title: Re: EM Drive Developments - related to space flight applications - Thread 2
Post by: Star One on 01/09/2015 06:56 pm

Great posts by Rodal and Mulletron.

And I concur: the next step is to firmly cement the scientific nature of the mentioned thrusters (them being falsifiable), by providing additional replications and tests. Theories can come later, if empiric evidence is there.

It doesn't matter if H. White's theory is right or wrong, the same as Woodward's (or Shawyer's, which is most likely wrong). What matters is that the phenomena related to those theories would have evidence of being real.

Humanity has used real phenomena nobody can explain for millenia, and this way before even having complete formal descriptions/theories for them. We still do, as we don't know the provenance and nature of inertia and gravity, and they are part of our everyday lives and expected to work for a lot of technology to function as well.

All well & good unless it proves through theory they are doing some fundamental damage. Something raised in fiction by a Star Trek: The Next Generation & the use of warp drive damaging the universe.
Title: Re: EM Drive Developments - related to space flight applications - Thread 2
Post by: tchernik on 01/09/2015 07:24 pm
All well & good unless it proves through theory they are doing some fundamental damage. Something raised in fiction by a Star Trek: The Next Generation & the use of warp drive damaging the universe.

Well, it has happened in the past. For example, that's what happened with those first researching radioactive and nuclear phenomena in general. They first knew they had some new phenomena in their hands and they knew how to produce some of the effects, but they didn't know how bad it was to have it near you. Several people died of radiation poisoning, and some cases even documented by the victim, in a moving display of dedication to science.

These are not precisely universe-destroying dangers per se, but equally life-wrecking stuff.

And we also learned.
Title: Re: EM Drive Developments - related to space flight applications - Thread 2
Post by: Stormbringer on 01/10/2015 12:33 am
 :D There is (however tenuously) a potential real world equivalent. collapse of the vacuum potential of the universe from mucking about with high energy collisions, quantum vacuums and warping space. the odds are about 2/3'rds in an infinity-1 gabbillion chance of it. but it could theoretically happen. :)

EDIT:  But then I think the *vacuum* potential is measured in micro-Hoover (TM) units.
Title: Re: EM Drive Developments - related to space flight applications - Thread 2
Post by: Mulletron on 01/10/2015 08:46 pm
Big difference between figure 5 here: http://ntrs.nasa.gov/archive/nasa/casi.ntrs.nasa.gov/20110023492.pdf (same old Q-thruster paper we've been reading)

and figures 2/3 here: http://www.jhuapl.edu/techdigest/TD/td2804/McNutt.pdf (MPD thrusters/Nuclear power)

The difference between Jupiter transit times is astonishing. Not to mention you don't have to carry aloft ~75 percent of your mass as propellant. Disclaimer though, the empty mass of Jupiter mission spacecraft (fig. 2 McNutt) is ~4081 metric tons (why so massive? anybody know*? This is about half as much as of one of these metal monstrosities http://en.wikipedia.org/wiki/Ticonderoga-class_cruiser), compared to the 90 metric tons (fig. 5 White) (50mT payload,  propulsion mass 20 mT, 2MW power system 20 mT. I'm not sure what the author intended here. I'm assuming a spacecraft with a mass of at least 90 metric tons total though.)

Anybody have any idea how massive this is? http://en.wikipedia.org/wiki/High_beta_fusion_reactor

It would be interesting to put together an imaginary ship using say....a cluster of the Brady c TE mode articles @21.31 milliNewtons/kW + a Lockheed Martin 100MW miracle reactor -reasonable electrical power and RF efficiency losses, to see if anything interesting can be done. Is there even enough information for this to be do-able yet?

Edit: * Found the mass, (McNutt pg. 385) "For the types of systems envisioned in the calculations of Fig. 5, ~1700 t of mass is sufficient to cross from Earth to Neptune in ~2 years (assuming 0.1 kg/kWe and an exhaust speed of 200 km·s−1). The mass of ~420 t is intended to carry food for a crew of six for 5 years, little more shielding than that provided by the structural mass of the ship, and a Boreas-style station that could be used as a science staging base on Triton and left there for any future expeditions."

Seems like the Dr. White example from figure 5 might be a little light.
Title: Re: EM Drive Developments - related to space flight applications - Thread 2
Post by: CW on 01/10/2015 09:04 pm
Big difference between figure 5 here: http://ntrs.nasa.gov/archive/nasa/casi.ntrs.nasa.gov/20110023492.pdf (same old Q-thruster paper we've been reading)

and figures 2/3 here: http://www.jhuapl.edu/techdigest/TD/td2804/McNutt.pdf (MPD thrusters/Nuclear power)

The difference between Jupiter transit times is astonishing. Not to mention you don't have to carry aloft ~75 percent of your mass as propellant. Disclaimer though, the empty mass of Jupiter mission spacecraft (fig. 2 McNutt) is ~4081 metric tons (why so massive? anybody know? This is about half as much as of one of these metal monstrosities http://en.wikipedia.org/wiki/Ticonderoga-class_cruiser), compared to the 90 metric tons (fig. 5 White) (50mT payload,  propulsion mass 20 mT, 2MW power system 20 mT. I'm not sure what the author intended here. I'm assuming a spacecraft with a mass of at least 90 metric tons total though.)

Anybody have any idea how massive this is? http://en.wikipedia.org/wiki/High_beta_fusion_reactor

It would be interesting to put together an imaginary ship using say....a cluster of the Brady c TE mode articles @21.31 milliNewtons/kW + a Lockheed Martin 100MW miracle reactor -reasonable electrical power and RF efficiency losses, to see if anything interesting can be done. Is there even enough information for this to be do-able yet?

As the Lockheed Martin reactor will supposedly fit onto a truck, my guesstimate is 5..10mt with shielding, superconducting magnets and whatnot. If it is a thermal conversion model, then LM will need to use heat exchangers with gas turbines to get the electrical power out (makes it heavier again). My favorite, the direct conversion model with p-B11 fuel, would obviously be more compact. I'm worrying a bit about how to get rid of that immense waste heat in a vacuum, though. The radiators would glow red-hot, I guess. They could also use the photon pressure from the waste heat to accelerate.. .
Title: Re: EM Drive Developments - related to space flight applications - Thread 2
Post by: Mulletron on 01/10/2015 11:48 pm
Big difference between figure 5 here: http://ntrs.nasa.gov/archive/nasa/casi.ntrs.nasa.gov/20110023492.pdf (same old Q-thruster paper we've been reading)

and figures 2/3 here: http://www.jhuapl.edu/techdigest/TD/td2804/McNutt.pdf (MPD thrusters/Nuclear power)

The difference between Jupiter transit times is astonishing. Not to mention you don't have to carry aloft ~75 percent of your mass as propellant. Disclaimer though, the empty mass of Jupiter mission spacecraft (fig. 2 McNutt) is ~4081 metric tons (why so massive? anybody know? This is about half as much as of one of these metal monstrosities http://en.wikipedia.org/wiki/Ticonderoga-class_cruiser), compared to the 90 metric tons (fig. 5 White) (50mT payload,  propulsion mass 20 mT, 2MW power system 20 mT. I'm not sure what the author intended here. I'm assuming a spacecraft with a mass of at least 90 metric tons total though.)

Anybody have any idea how massive this is? http://en.wikipedia.org/wiki/High_beta_fusion_reactor

It would be interesting to put together an imaginary ship using say....a cluster of the Brady c TE mode articles @21.31 milliNewtons/kW + a Lockheed Martin 100MW miracle reactor -reasonable electrical power and RF efficiency losses, to see if anything interesting can be done. Is there even enough information for this to be do-able yet?

As the Lockheed Martin reactor will supposedly fit onto a truck, my guesstimate is 5..10mt with shielding, superconducting magnets and whatnot. If it is a thermal conversion model, then LM will need to use heat exchangers with gas turbines to get the electrical power out (makes it heavier again). My favorite, the direct conversion model with p-B11 fuel, would obviously be more compact. I'm worrying a bit about how to get rid of that immense waste heat in a vacuum, though. The radiators would glow red-hot, I guess. They could also use the photon pressure from the waste heat to accelerate.. .

Lockheed Martin keeps throwing around that it will fit onto a truck. http://www.lockheedmartin.com/us/products/compact-fusion.html As I suspected, that can be a very wide range of weight.

First depending on where you look, an empty 18 wheeler with trailer weights ~15mT.

I did some digging and I found that the max gross weight of every conex box size is ~30mT. http://en.wikipedia.org/wiki/Intermodal_container#Specifications

If I use the federal interstate gross vehicle weight limits, they are ~36mT. Subtract that ~15mT empty truck weight plus another ton to keep things super simple, you get ~20mT.

So to me, "it fits on a truck" sounds like 20-30mT.

If we want to get ridiculous here though, with a permit, using Florida and Texas as examples, you can haul as much as 78mT and 90mT respectively. http://www.fhwa.dot.gov/reports/tswstudy/Vol2-Chapter2.pdf

Now for perspective, the ISS weights in at 924,739 pounds or ~420mT.
http://www.nasa.gov/mission_pages/station/main/onthestation/facts_and_figures.html#.VLG_FnupjSh
http://www.arianespace.com/news-press-release/2014/7-29-2014-VA219-launch-success.asp
Also Skylab, http://en.wikipedia.org/wiki/Skylab ~77mT.

So that kind of tonnage on orbit isn't out of our reach by any means already. Just launched in stages.

Since I know, and I'm sure many of you have seen as well, reality hardly lives up to expectations, so I'm inclined to, for the purpose of this exercise, make the reactor twice as massive and deliver half the power. So double the Dr. White/"it fits on a truck" 20mT figure and factoring in ~50% DEC, http://en.wikipedia.org/wiki/Direct_energy_conversion, 50MW power. A 40mT/50 MWe power system.

@Rodal, care to throw some math at combining this 40mT/50 MWe power system with one of those thrusters you got the calcs on? Honestly the only numbers I trust are the Brady et al ones, not the inventor's numbers. How much RF can you pump into just one of those Shawyer designs before it starts to melt down? >9000 Watts? What about a hundred of them at once? I estimate they weigh ~10lbs each. It seems ~70% RF amp efficiency is a good round number (http://forum.nasaspaceflight.com/index.php?topic=36313.msg1311433#msg1311433).

Edit:
Um I feel like I screwed this up? The final velocity seems kinda um, large. :o I didn't subtract the Sun pulling on it, but I think something else is wrong.

So throwing around numbers using the ISS as an example (just converted it to an interplanetary spaceship) with a mass of 500mT (420mT original + 40mT 50MWe reactor module add on + 20mT propulsion/aux systems add on + 20mT logistics module add on.

Total mass: 500mT
40MWe available to propulsion, 10MWe left in reserve/used/rest lost to heat
Assuming 70% propulsion system efficiency
28MW RF available for drive
Brady c TE mode @21.31 milliNewtons/kW performance propulsion system
Thrust at 28MW RF @0.02131N/kw=596N
Acceleration=0.00119200m/s^2   
Velocity after 1 year=37.59km/s, 135327km/h, 84088.5mph
Displacement 3.96AU
Title: Re: EM Drive Developments - related to space flight applications - Thread 2
Post by: aero on 01/11/2015 01:31 am
Since my some months ago post interchange with Mullerton in which we more or less agreed that I should pursue the possibility of evanescent waves being the source of the EM Drive effect, I have been doing that with some results.

Using FDTD software I found the following salient points. First though, my computer is not powerful enough to perform FDTD calculations in high resolution, so I'm stuck with moderate resolution at best. That rules out resolving the thin copper sheets on the circuit board ends of the cavity. Second, there isn't any data that I can discover describing copper at this frequency and the power level experienced by the cavity ends. Sure, lots of data on copper for shielding, but ...

Consider that operating at Q = 22,000 and drive power of 2.6 watts with the cavity end diameter giving an area of 0.058 m^2 the radiation intensity approaches a kW/cm^2, or almost a MW/m^2. That is a lot of power and it seems no one has researched the behavior of copper under those intense conditions.

With those two points understood, I did calibrate the FDTD model to a photon rocket, giving thrust of very nearly 1/c as it should. This is with the detector plane about two cavity lengths behind the source antenna.

For the reasons given above, I used a perfect metal or ideal conductor as the material for the cavity model. Not surprisingly, when I ran the model totally enclosing the source with perfect metal, there was zero force detected. That result has been calculated analytically and discussed here on NSF.

I went further, considering that the end is bolted onto the cavity, what if it leaks RF? I modelled a narrow slice around the circumference of the cavity cone in the end plate, a variable sized opening but at the smallest resolution my computer will allow, 0.2% of the cavity large end radius.

The simulation detected Force/Power ranging from 2/c to 3/c.

That is two to three times the thrust of an ideal photon rocket. I think the cause of the force is evanescent waves escaping through the very narrow gap in the cavity base.

My data does not answer the question, "What causes the thrust of the EM drive?" It does point in a direction.

I can run more cases with low to moderate resolution and perfect metal, but as I wrote at the top, I need a bigger computer and better knowledge of copper behavior under intense radiation in order to calculate definitive results.

I'm sure someone will ask, so, Yes, I did look at forces generated by similar gaps in the small end of the cavity. Forces exist but only about one forth as great as at the large end. That may be directly due to the gap area, the small end circumference being about half that of the large end, and the dielectric does interfere. I should probably remove the dielectric and run some small end test cases.
Title: Re: EM Drive Developments - related to space flight applications - Thread 2
Post by: Mulletron on 01/11/2015 01:36 am
Welcome back!
Title: Re: EM Drive Developments - related to space flight applications - Thread 2
Post by: aero on 01/11/2015 02:56 am
Welcome back!

Thanks.
Title: Re: EM Drive Developments - related to space flight applications - Thread 2
Post by: ThinkerX on 01/11/2015 06:50 am
Quote
Total mass: 500mT
40MWe available to propulsion, 10MWe left in reserve/used/rest lost to heat
Assuming 70% propulsion system efficiency
28MW RF available for drive
Brady c TE mode @21.31 milliNewtons/kW performance propulsion system
Thrust at 28MW RF @0.02131N/kw=596N
Acceleration=0.00119200m/s^2   
Velocity after 1 year=37.59km/s, 135327km/h, 84088.5mph
Displacement 3.96AU

So...if I am following this correctly, allowing for deceleration, a 3 year trip to Saturn, give or take a month?  Versus a bit over 9 months for the Eagleworks proposal.
Title: Re: EM Drive Developments - related to space flight applications - Thread 2
Post by: ThinkerX on 01/11/2015 06:58 am
Welcome back Aero!

Quote
I went further, considering that the end is bolted onto the cavity, what if it leaks RF? I modelled a narrow slice around the circumference of the cavity cone in the end plate, a variable sized opening but at the smallest resolution my computer will allow, 0.2% of the cavity large end radius.

The simulation detected Force/Power ranging from 2/c to 3/c.

That is two to three times the thrust of an ideal photon rocket. I think the cause of the force is evanescent waves escaping through the very narrow gap in the cavity base

Isn't that about on a par with Woodward's device?  The worst performer of the bunch? 
Title: Re: EM Drive Developments - related to space flight applications - Thread 2
Post by: RotoSequence on 01/11/2015 08:04 am
Welcome back Aero!

Quote
I went further, considering that the end is bolted onto the cavity, what if it leaks RF? I modelled a narrow slice around the circumference of the cavity cone in the end plate, a variable sized opening but at the smallest resolution my computer will allow, 0.2% of the cavity large end radius.

The simulation detected Force/Power ranging from 2/c to 3/c.

That is two to three times the thrust of an ideal photon rocket. I think the cause of the force is evanescent waves escaping through the very narrow gap in the cavity base

Isn't that about on a par with Woodward's device?  The worst performer of the bunch?

I think 2x to 3x photon thrust is pretty much exactly the same number as the Woodward device, actually.
Title: Re: EM Drive Developments - related to space flight applications - Thread 2
Post by: MP99 on 01/11/2015 11:11 am


... the radiation intensity approaches a kW/cm^2, or almost a MW/m^2.

10,000 sq cm in one sq metre, not 1,000.

Cheers, Martin
Title: Re: EM Drive Developments - related to space flight applications - Thread 2
Post by: Mulletron on 01/11/2015 04:38 pm
Quote
Total mass: 500mT
40MWe available to propulsion, 10MWe left in reserve/used/rest lost to heat
Assuming 70% propulsion system efficiency
28MW RF available for drive
Brady c TE mode @21.31 milliNewtons/kW performance propulsion system
Thrust at 28MW RF @0.02131N/kw=596N
Acceleration=0.00119200m/s^2   
Velocity after 1 year=37.59km/s, 135327km/h, 84088.5mph
Displacement 3.96AU

So...if I am following this correctly, allowing for deceleration, a 3 year trip to Saturn, give or take a month?  Versus a bit over 9 months for the Eagleworks proposal.

Not sure because I didn't factor in any orbital mechanics. That was just straight line constant acceleration. I'm surprised it could even move at all within a reasonable amount of time. I'm downloading something called Orbiter right now, slowly to see if I can simulate this kind of stuff. http://orbit.medphys.ucl.ac.uk/zipinstall.html Found a CPA for Saturn/Earth of 8.052au. This was in the past. Next one I don't know. Plugged that into Celestia. See screenshot.
Title: Re: EM Drive Developments - related to space flight applications - Thread 2
Post by: aero on 01/11/2015 05:13 pm


... the radiation intensity approaches a kW/cm^2, or almost a MW/m^2.

10,000 sq cm in one sq metre, not 1,000.

Cheers, Martin

Yes I know. That's why I weasel worded it. If we don't look at the higher power devices from Shawyer or China, just the three Brady devices, the stored power comes out as: 2.12E+006, 9.81E+005, and 5.18E+006 Watts/m^2. All shy of 1 kW/cm^2. On the other hand, other devices operate an 100 times the power level of the Brady device, even 1000 times in one particular case. ( 2.6 watts .cf. 2.5 kW)

My point is that the characteristic behavior of copper in this regime is not easy to find. (Make that impossible to find) It seems that high power radar wave guides should operate in this regime. Does anyone have a Drude  model or test data with which to create one? Or even a known material that in thick sheets, behaves like copper in thin sheets under high power radiation?


Title: Re: EM Drive Developments - related to space flight applications - Thread 2
Post by: aero on 01/11/2015 05:32 pm
Welcome back Aero!

Quote
I went further, considering that the end is bolted onto the cavity, what if it leaks RF? I modelled a narrow slice around the circumference of the cavity cone in the end plate, a variable sized opening but at the smallest resolution my computer will allow, 0.2% of the cavity large end radius.

The simulation detected Force/Power ranging from 2/c to 3/c.

That is two to three times the thrust of an ideal photon rocket. I think the cause of the force is evanescent waves escaping through the very narrow gap in the cavity base

Isn't that about on a par with Woodward's device?  The worst performer of the bunch?

I think 2x to 3x photon thrust is pretty much exactly the same number as the Woodward device, actually.
And I think that producing thrust by making a resonant cavity and putting the end on losely is a lot easier than making a Woodward device. But that is not my point.

My point is that EM drive thrust may be produced by evanescent waves. If only I knew the behavior of copper and had a more powerful computer to run the calculations over thin sheets I could find out for sure. (I think)
Title: Re: EM Drive Developments - related to space flight applications - Thread 2
Post by: Rodal on 01/11/2015 06:26 pm
.....
Total mass: 500mT
40MWe available to propulsion, 10MWe left in reserve/used/rest lost to heat
Assuming 70% propulsion system efficiency
28MW RF available for drive
Brady c TE mode @21.31 milliNewtons/kW performance propulsion system
Thrust at 28MW RF @0.02131N/kw=596N
Acceleration=0.00119200m/s^2   
Velocity after 1 year=37.59km/s, 135327km/h, 84088.5mph
Displacement 3.96AU

Using the following parameters from @Mulletron:

TotalMass                    500000 kgm
Power Input:                 28000000 Watts
ThrustForce/power:      0.00002131 Newton/Watt

we indeed immediately obtain:
ThrustForce:      596.7 Newtons
Acceleration:      0.001193 m/s^2

From, "Human Outer Solar System Exploration via Q-Thruster Technology" by B. Kent Joosten and Harold G. “Sonny” White, http://ntrs.nasa.gov/archive/nasa/casi.ntrs.nasa.gov/20140013174.pdf, their equation (4):

minimum flight time = 2 * Sqrt[(radiusTarget-radiusEarth)/Acceleration]

and the following heliocentric radii:

radiusEarth   1.496E+11 m
radiusMats   2.2794E+11 m
radiusJupiter    7.7833E+11 m
radiusSaturn    1.4246E+12 m

The following flight times for @Mulletron parameters vs the flight times (for the different parameters considered by)  "Human Outer Solar System Exploration via Q-Thruster Technology" by B. Kent Joosten and Harold G. “Sonny” White, http://ntrs.nasa.gov/archive/nasa/casi.ntrs.nasa.gov/20140013174.pdf follow:

From Earth to:        time @Mulletron (days/months)           time Joosten/White (days/months)

Mars                       188/6.3                                                 76/2.5                               
Jupiter                    531/17.7                                             194/6.5
Saturn                    757/25.2                                              277/9.2


The main reason for the ~3x longer flight times for Mulletron is because of the lower thrust/PowerInput considered (Mulletron=0.02131 N/kW vs. White=0.4 N/kW, hence Sqrt[0.4/0.02131]=4 times)

NOTE: The parameters considered by @Mulletron, under all cases considered (including Earth to Saturn in 757 days or 25.2 months) are safely under the "paradox condition" of Joosten/White.  There is no paradox for these Mulletron Missions: the change in kinetic energy is less than the energy consumed

The "paradox condition" of Joosten/White for the parameters considered by @Mulletron occurs at 910 days (30.3 months of continuous acceleration).


The "paradox condition" is defined as the condition for which the change in kinetic energy of the spacecraft exceeds the input energy.

Change of KineticEnergy of Spacecraft >= EnergyInput

Assuming constant thrust force (and hence constant acceleration for a constant spacecraft mass) under constant input power, the energy consumed is simply:

EnergyConsumed=Power*time

And the change in kinetic energy of the spacecraft can be expressed as:

Change of KineticEnergy of Spacecraft = mass*((change in velocity)^2)/2
                                                             = (mass/2)*((ThrustForce/mass)*time)^2)
                                                             = (mass/2)*(((ThrustForce/Power)*Power/mass)*time)^2)
Therefore

ParadoxTime = 2 * mass / (power*((thrustForce/power)^2))

see Appendix A of  "Human Outer Solar System Exploration via Q-Thruster Technology" by B. Kent Joosten and Harold G. “Sonny” White, http://ntrs.nasa.gov/archive/nasa/casi.ntrs.nasa.gov/20140013174.pdf and the post by Frobnicat  http://forum.nasaspaceflight.com/index.php?topic=29276.msg1281724#msg1281724 and by Mulletron http://forum.nasaspaceflight.com/index.php?topic=29276.msg1281736#msg1281736

Also, of course

Quote from: B. Kent Joosten and Harold G. “Sonny” White
should examine specific earth departure and return techniques. Unconstrained Q-Ship spiral trajectories
could expose crewmembers to undesirable levels of radiation exposure due to the Van Allen radiation belts.

(http://www.solpass.org/5s/images/planetss.jpg)
Title: Re: EM Drive Developments - related to space flight applications - Thread 2
Post by: Vultur on 01/11/2015 10:48 pm
What is the evanescent wave effect? Is this a false positive (IE wouldn't work in space)?
Title: Re: EM Drive Developments - related to space flight applications - Thread 2
Post by: Rodal on 01/11/2015 11:32 pm
Since under constant acceleration (where "velocity" stands for "delta velocity"=vfinal-vinitial and "time" stands for "delta time"=tfinal - tinitial)

time=velocity/acceleration

and acceleration=thrustForce/mass

time= velocity*mass/thrustForce

Substituting this into the paradox equation:

time = 2 * mass / (power*((thrustForce/power)^2))

velocity*mass/thrustForce = 2 * mass / (power*((thrustForce/power)^2))

and simplifying:

Paradox velocity = 2*(thrustForce/ power) /(thrustForce/power)^2)
                           = 2/(thrustForce/ power)
                           =2*power/thrustForce

Hence the paradox occurs when the (change in) velocity exceeds 2 times the PowerInput divided by the thrust force

This makes sense of course, since

Quote from: equation 19-1 Rocket propulsion elements 7th edition- Sutton
for any vehicle power P, the thrust that may be provided is:
ThrustForce = 2*Power/velocity



EXAMPLES

For

(Brady TE mode) ThrustForce/power:      0.00002131 Newton/Watt

ParadoxVelocity = 2/0.00002131 m/s = 93853 m/s = 93.85 km/s


For

(Prof. Juan Yang et.al. China)  ThrustForce/power:      0.000290  Newton/Watt

ParadoxVelocity = 2/0.000290 m/s = 6897 m/s = 6.90 km/s

 

For

(Cannae Superconducting)   ThrustForce/power:      0.0009524  Newton/Watt     

ParadoxVelocity = 2/0.0009524 m/s = 2100 m/s = 2.10 km/s     
Title: Re: EM Drive Developments - related to space flight applications - Thread 2
Post by: ThinkerX on 01/11/2015 11:40 pm
Quote
NOTE: The parameters considered by @Mulletron, under all cases considered (including Earth to Saturn in 757 days or 25.2 months) are safely under the "paradox condition" of Joosten/White.  There is no paradox for these Mulletron Missions: the change in kinetic energy is less than the energy consumed

The "paradox condition" of Joosten/White for the parameters considered by @Mulletron occurs at 910 days (30.3 months of continuous acceleration).


The "paradox condition" is defined as the condition for which the change in kinetic energy of the spacecraft exceeds the input energy.

Maybe not the brightest of questions, but could Doctor McCulloch's theory possibly resolve the paradox issue?  Accelerate long enough, and inertial radiation becomes significant?   
Title: Re: EM Drive Developments - related to space flight applications - Thread 2
Post by: aero on 01/11/2015 11:59 pm
What is the evanescent wave effect? Is this a false positive (IE wouldn't work in space)?

Evanescent waves are a solution to Maxwell's equation, commonly named the "near field." So no, it is not a false positive and yes, it should work in space.
Title: Re: EM Drive Developments - related to space flight applications - Thread 2
Post by: Notsosureofit on 01/12/2015 12:19 am
Home w/ a cold so puttering around, looking to add the 3rd mode number into consideration. (cavity length)

Found out that the Wikipedia formulas for the cylindrical cavity resonance frequency is in error.  It has an extra c term.   Edit:  Whoops, nope those are relative numbers.  They are OK.

So redoing everything w/ m,n,p.
Title: Re: EM Drive Developments - related to space flight applications - Thread 2
Post by: Rodal on 01/12/2015 12:25 am
Quote
NOTE: The parameters considered by @Mulletron, under all cases considered (including Earth to Saturn in 757 days or 25.2 months) are safely under the "paradox condition" of Joosten/White.  There is no paradox for these Mulletron Missions: the change in kinetic energy is less than the energy consumed

The "paradox condition" of Joosten/White for the parameters considered by @Mulletron occurs at 910 days (30.3 months of continuous acceleration).


The "paradox condition" is defined as the condition for which the change in kinetic energy of the spacecraft exceeds the input energy.

Maybe not the brightest of questions, but could Doctor McCulloch's theory possibly resolve the paradox issue?  Accelerate long enough, and inertial radiation becomes significant?

I don't think so.  The paradox occurs under conditions of constant acceleration.  Under constant acceleration, the Unruh radiation stays the same in McCulloch's theory.   (Ditto for Hawking radiation under constant acceleration).

The paradox occurs under constant acceleration when the velocity exceeds 2 times the PowerInput divided by the thrust force, since that implies a change in kinetic energy greater than the energy provided.
Title: Re: EM Drive Developments - related to space flight applications - Thread 2
Post by: ThinkerX on 01/12/2015 12:35 am
Quote
I don't think so.  The paradox occurs under conditions of constant acceleration.  Under constant acceleration, the Unruh radiation stays the same in McCulloch's theory.

The paradox occurs when the velocity equals or exceeds 2 times the Power Input divided by the thrust force.

So...How much of the physics textbooks will need rewritten if this device ignores the paradox?  (continues to accelerate with the velocity exceeding twice the power input divided by the thrust force). 

Would that be Free Energy? 
 

Title: Re: EM Drive Developments - related to space flight applications - Thread 2
Post by: Rodal on 01/12/2015 12:38 am
Quote
I don't think so.  The paradox occurs under conditions of constant acceleration.  Under constant acceleration, the Unruh radiation stays the same in McCulloch's theory.

The paradox occurs when the velocity equals or exceeds 2 times the Power Input divided by the thrust force.

So...How much of the physics textbooks will need rewritten if this device ignores the paradox?  (continues to accelerate with the velocity exceeding twice the power input divided by the thrust force). 

Would that be Free Energy?
No, Dr. White does not propose that the EM Drive ignores this energy constraint. 

Effectively, White proposes that the maximum velocity possibly achievable for these EM Drives is 2*PowerInput/ThrustForce.

Quote from: Joosten and White
When this situation occurs, in order to ensure that the input energy is equal to the change in kinetic energy, the thrust to power performance will have to decrease

At constant input power, the thrust, and therefore the acceleration, must decrease with time, to ensure that the spacecraft's (change in) velocity never exceeds 2*Power/Thrust

(This energy constraint is known even from Sutton's textbook on Elements of Rocket Propulsion (equation
19-1 Rocket propulsion elements 7th edition- Sutton)).




The higher the ThrustForce/InputPower, the lower the maximum velocity of an EM Drive:

MAXIMUM VELOCITY OF EM DRIVE = 2 /(thrustForce/PowerInput)

(Brady TE mode) ThrustForce/power:      0.00002131 Newton/Watt

MaximumVelocity = 2/0.00002131 m/s = 93853 m/s = 93.85 km/s


(Prof. Juan Yang et.al. China)  ThrustForce/power:      0.000290  Newton/Watt

MaximumVelocity = 2/0.000290 m/s = 6897 m/s = 6.90 km/s

 

(Cannae Superconducting)   ThrustForce/power:      0.0009524  Newton/Watt     

MaximumVelocity = 2/0.0009524 m/s = 2100 m/s = 2.10 km/s     
Title: Re: EM Drive Developments - related to space flight applications - Thread 2
Post by: Notsosureofit on 01/12/2015 01:10 am
Just a thought, but the higher the Q, the narrower the bandwidth, and the Doppler shift due to actual acceleration (or velocity in the rest frame) would probably limit that amount by exceeding the bandwidth.
Title: Re: EM Drive Developments - related to space flight applications - Thread 2
Post by: Rodal on 01/12/2015 01:38 am
Just a thought, but the higher the Q, the narrower the bandwidth, and the Doppler shift due to actual acceleration (or velocity in the rest frame) would probably limit that amount by exceeding the bandwidth.

To all, let's recall that

ThrustForce/PowerInput ~ Q*otherParameters

in all the formulas (Shawyer's, McCulloch's and NotSoSureOfIt's).  (NotSoSureOfIt's formula also depends explicitly on the microwave's cavity mode of operation).

Therefore the maximum velocity of the EM Drive is inversely proportional to Q:

MAXIMUM VELOCITY OF EM DRIVE = 2 /(thrustForce/PowerInput)

MAXIMUM VELOCITY OF EM DRIVE ~ (2 /(Q*otherParameters))

The higher the Q, the lower the maximum velocity
Title: Re: EM Drive Developments - related to space flight applications - Thread 2
Post by: ThinkerX on 01/12/2015 01:49 am
Quote
At constant input power, the thrust, and therefore the acceleration, must decrease with time, to ensure that the spacecraft's velocity never exceeds 2*Power/Thrust

I was going to start a thread, but there is a photon rocket variant idea I have been batting around for a few months now:

take a long hollow cylinder, closed at one end, open on the other.  Probably several hundred meters long, by three or four meters in diameter.  Running the length of this cylinder, spaced at even intervals are low beams - probably no more than ten or twenty centimeters high.  So the inner edge of the cylinder has...call it a dozen shallow troughs.  At regular intervals - maybe a meter - these beams have specially designed reflective points.  Open end of the cylinder, you have a powerful high frequency laser (or something emitting a focused photon beam) aimed at a 45 degree angle into each trough.  One laser per trough, call it twelve total. 

Now, a laser, like a military searchlight, is also a photon rocket.

Photons, as pointed out in the previous thread are durable little critters, and can bounce around a good 50,000+ times before going wherever it is expired photons go.   And photons can transfer momentum with each bounce.   

So, turn the lasers on.  The initial 'thrust' is backward.   Actually, 'backward and sideways' because of the angle. 

That thrust gets negated at the first bounce point.  Photons hit that (reflective) point, transfer momentum, and head over to the next bounce point, set at a 45 degree angle to the first.

At the second bounce point, the whole thing is moving forward.  Repeat for the length of the cylinder. Because the photons are hitting at an angle, the cylinder might start rotating as well as moving forward, but I don't see that as a major issue.  At the end of the cylinder, the photons hit a shaped surface and bounce back along the tubes center and out into space. 

Did a bit of reading on laser propulsion systems.  A Doctor Bae ran some laboratory tests on this: bouncing laser beams multiplied the 'thrust' by a factor of 3000+ - into EM Drive territory without the physics headache.  He proposed two linked spacecraft, with laser beams between them, something NASA is supposed to be looking into for near earth applications.   My idea is one spacecraft (the cylinder) and a multiplier of about 1500, if the cylinder is long enough. Not sure, but that's should be on a par with the Brady EM drive model.

Alter the angles a bit, test  different lasers/emitters, might get a lot more work out of the photons, increasing thrust further.

Would this violate the paradox?   






Title: Re: EM Drive Developments - related to space flight applications - Thread 2
Post by: Vultur on 01/12/2015 02:18 am
What is the evanescent wave effect? Is this a false positive (IE wouldn't work in space)?

Evanescent waves are a solution to Maxwell's equation, commonly named the "near field." So no, it is not a false positive and yes, it should work in space.

OK, so how does this handle the conservation of momentum issue if it's more efficient than a photon rocket? What's the "reaction"?

At constant input power, the thrust, and therefore the acceleration, must decrease with time, to ensure that the spacecraft's velocity never exceeds 2*Power/Thrust

Velocity measured relative to what?
Title: Re: EM Drive Developments - related to space flight applications - Thread 2
Post by: ChrisWilson68 on 01/12/2015 02:44 am
Quote
At constant input power, the thrust, and therefore the acceleration, must decrease with time, to ensure that the spacecraft's velocity never exceeds 2*Power/Thrust

I was going to start a thread, but there is a photon rocket variant idea I have been batting around for a few months now:

take a long hollow cylinder, closed at one end, open on the other.  Probably several hundred meters long, by three or four meters in diameter.  Running the length of this cylinder, spaced at even intervals are low beams - probably no more than ten or twenty centimeters high.  So the inner edge of the cylinder has...call it a dozen shallow troughs.  At regular intervals - maybe a meter - these beams have specially designed reflective points.  Open end of the cylinder, you have a powerful high frequency laser (or something emitting a focused photon beam) aimed at a 45 degree angle into each trough.  One laser per trough, call it twelve total. 

Now, a laser, like a military searchlight, is also a photon rocket.

Photons, as pointed out in the previous thread are durable little critters, and can bounce around a good 50,000+ times before going wherever it is expired photons go.   And photons can transfer momentum with each bounce.   

So, turn the lasers on.  The initial 'thrust' is backward.   Actually, 'backward and sideways' because of the angle. 

That thrust gets negated at the first bounce point.  Photons hit that (reflective) point, transfer momentum, and head over to the next bounce point, set at a 45 degree angle to the first.

At the second bounce point, the whole thing is moving forward.  Repeat for the length of the cylinder. Because the photons are hitting at an angle, the cylinder might start rotating as well as moving forward, but I don't see that as a major issue.  At the end of the cylinder, the photons hit a shaped surface and bounce back along the tubes center and out into space. 

Did a bit of reading on laser propulsion systems.  A Doctor Bae ran some laboratory tests on this: bouncing laser beams multiplied the 'thrust' by a factor of 3000+ - into EM Drive territory without the physics headache.  He proposed two linked spacecraft, with laser beams between them, something NASA is supposed to be looking into for near earth applications.   My idea is one spacecraft (the cylinder) and a multiplier of about 1500, if the cylinder is long enough. Not sure, but that's should be on a par with the Brady EM drive model.

Alter the angles a bit, test  different lasers/emitters, might get a lot more work out of the photons, increasing thrust further.

Would this violate the paradox?   

You seem to misunderstand the fundamentals of mechanics.

If a photon hits a mirror at a 45 degree angle and reflects off it, the mirror will receive an impulse perpendicular to the plane of the mirror only.  It will not be pushed in the direction of the other component of the photon at all.

It's not just with photons.  If you have a billiard ball and you bounce it off another billiard ball that was stationary so that the original ball end up leaving at a 90 degree angle to its initial direction of travel, the other ball will end up traveling at a 45 degree angle to the path of the original ball.

It's non-intuitive because our intuition is shaped by friction tending to pull things along, but such friction is not a part of purely elastic collisions, and photons bouncing off mirrors are purely elastic.

So, every time your photon bounces off the wall, the momentum it imparts will only be to push outward perpendicular to the axis of the tube.  And it will be cancelled by the next bounce off the opposite wall.

The only effect of the net momentum of the tube is the opposite of whatever momentum the photo has when it finally leaves the tube.  Whatever it does as it bounces around in the tube will have no net effect.
Title: Re: EM Drive Developments - related to space flight applications - Thread 2
Post by: aero on 01/12/2015 05:02 am
What is the evanescent wave effect? Is this a false positive (IE wouldn't work in space)?

Evanescent waves are a solution to Maxwell's equation, commonly named the "near field." So no, it is not a false positive and yes, it should work in space.


OK, so how does this handle the conservation of momentum issue if it's more efficient than a photon rocket? What's the "reaction"?

                                                    ...  snip ...

See this paper - Extraordinary momentum and spin in evanescent waves Middle of page 5.
http://arxiv.org/pdf/1308.0547.pdf (http://arxiv.org/pdf/1308.0547.pdf)
 
Quote
First, note that the evanescent wave (7) possesses longitudinal canonical momentum    pOz∝kz>k,   which exceeds the momentum of a plane wave with the same local intensity.

That's what led me to look in this direction. Evanescent waves exhibit unusual behavior. The answer to your question then is that the photon rocket is not the ultimate in efficiency. Make a photon rocket using evanescent wave photons and the efficiency will be higher.

... Maybe ... that is what we're all here to find out.
Title: Re: EM Drive Developments - related to space flight applications - Thread 2
Post by: Rodal on 01/12/2015 01:11 pm
What is the evanescent wave effect? Is this a false positive (IE wouldn't work in space)?

Evanescent waves are a solution to Maxwell's equation, commonly named the "near field." So no, it is not a false positive and yes, it should work in space.


OK, so how does this handle the conservation of momentum issue if it's more efficient than a photon rocket? What's the "reaction"?

                                                    ...  snip ...

See this paper - Extraordinary momentum and spin in evanescent waves Middle of page 5.
http://arxiv.org/pdf/1308.0547.pdf (http://arxiv.org/pdf/1308.0547.pdf)
 
Quote
First, note that the evanescent wave (7) possesses longitudinal canonical momentum    pOz∝kz>k,   which exceeds the momentum of a plane wave with the same local intensity.

That's what led me to look in this direction. Evanescent waves exhibit unusual behavior. The answer to your question then is that the photon rocket is not the ultimate in efficiency. Make a photon rocket using evanescent wave photons and the efficiency will be higher.

... Maybe ... that is what we're all here to find out.

The linked paper ( http://arxiv.org/pdf/1308.0547.pdf] ) addresses canonical-momentum and angular/spin-momentum of evanescent waves in general.  The studied effect (spin Hall effect of light http://en.wikipedia.org/wiki/Spin_Hall_effect) can be used for example for rotating nanoparticles using an external (to the particles) plasmonic optical-nano-tweezer.   

(http://webwib.com/wp-content/uploads/2014/03/manipulasi-tweezer.jpg)

What remains unclear is how the linked paper -by itself- addresses the issue of conservation of momentum of an EM Drive: I do not recall the linked paper specifically addressing conservation of momentum of an EM Drive cavity where all the microwave photons remain inside the cavity.  If it is claimed that the evanescent waves (resulting from  exponentially-decaying solutions to the electromagnetic field inside cavity) cause the EM Drive (and spacecraft) to accelerate, what balances the change in the EM Drive (and the spacecraft’s) momentum?.

By contrast,

* in Dr. McCulloch's theory momentum of the EM Drive cavity is balanced by emitted Unruh radiation,

* in Dr. White's theory and in other theories investigated by @Mulletron, it is balanced by interaction with the Quantum Vacuum,

* in @NotSoSureOfIt's formulation it may be balanced by terms coupling electromagnetism with gravitational forces.

* people in this forum also investigated (in the earlier thread) momentum balancing of the EM Drive by photon interaction coupling with Dark Matter (specifically: Axions).  [For a far out idea, a "Through the wormhole" episode (minute 11 of the episode, see the video http://www.sciencechannel.com/tv-shows/through-the-wormhole/videos/through-the-wormhole-2-dark-matter/ ) Tim Tate , of UC Irvine, tells us dark matter may be "massless photons spinning in the 4th dimension that appear to us as having mass because they are rapidly spinning in the 4th dimension" see: http://www-conf.slac.stanford.edu/ssi/2011/Tait_072911.pdf]

So the question remains: what balances the momentum change of an EM Drive accelerated by evanescent waves? Are you proposing an interaction with the Quantum Vacuum? or terms coupling electromagnetism with gravitational forces? or something else?
Title: Re: EM Drive Developments - related to space flight applications - Thread 2
Post by: Stormbringer on 01/12/2015 02:17 pm
In relativity; coupling of gravity and electromagnetism is infinitesimal though. That's what got Martin Tajmar's experiment so much attention at first. It suggested gravity-EM coupling billions of times stronger than predicted by RT. and what he got was still extremely tiny too.

Of course later he attributed his results to air currents caused by his cryo-coolant sublimating and retracted his paper.

Parenthetically, Tajmar's general design was also used by Ning Li and Podkletnov but probably differed in the details of cooling and method of obtaining rotation and materials for the disks.

That branch (family?) of gravity-EM drive designs has had bad luck even though there may be something there. Tajmar retracted, Ning Li allegedly absconded with research funds, Podkletnov got the kook treatment and retreated to Russia where he continues to claim results.

EDIT:  in other news a recent experiment designed to test spin gravity failed to detect any difference in gravitation effects on  a particle with a zero spin value and a non zero spin value. fermion and bosons were used...

http://phys.org/news/2014-07-equivalence-principle-effects-spin-gravity-coupling.html
Title: Re: EM Drive Developments - related to space flight applications - Thread 2
Post by: Notsosureofit on 01/12/2015 04:06 pm
In relativity; coupling of gravity and electromagnetism is infinitesimal though. That's what got Martin Tajmar's experiment so much attention at first. It suggested gravity-EM coupling billions of times stronger than predicted by RT. and what he got was still extremely tiny too.

Of course later he attributed his results to air currents caused by his cryo-coolant sublimating and retracted his paper.

Parenthetically, Tajmar's general design was also used by Ning Li and Podkletnov but probably differed in the details of cooling and method of obtaining rotation and materials for the disks.

That branch (family?) of gravity-EM drive designs has had bad luck even though there may be something there. Tajmar retracted, Ning Li allegedly absconded with research funds, Podkletnov got the kook treatment and retreated to Russia where he continues to claim results.

EDIT:  in other news a recent experiment designed to test spin gravity failed to detect any difference in gravitation effects on  a particle with a zero spin value and a non zero spin value. fermion and bosons were used...

http://phys.org/news/2014-07-equivalence-principle-effects-spin-gravity-coupling.html

Classically, you need a gravitational gradient for the interaction.  Maybe they can combine w/
http://phys.org/news/2015-01-gravity-curvature.html#nRlv
Title: Re: EM Drive Developments - related to space flight applications - Thread 2
Post by: aero on 01/12/2015 05:30 pm
So the question remains: what balances the momentum change of an EM Drive accelerated by evanescent waves? Are you proposing an interaction with the Quantum Vacuum? or terms coupling electromagnetism with gravitational forces? or something else?
No - I am proposing that electromagnetic wave momentum depends on group velocity, and that evanescent waves in these circumstances exhibit superluminal group velocity. Nothing is violated - wave momentum reacts against the base plate and energy is inductively drawn from the cavity.

I'll challange you, Dr. Rodel, and Mathematica to demonstrate why/how the energy resolves from the subluminal velocity solution of wave equations. I can help you formulate the problem but we need independent confirmation and you have shown great capability with Mathematica. Do that and you will have solved the EM drive paradox.
Title: Re: EM Drive Developments - related to space flight applications - Thread 2
Post by: as58 on 01/12/2015 06:02 pm
So the question remains: what balances the momentum change of an EM Drive accelerated by evanescent waves? Are you proposing an interaction with the Quantum Vacuum? or terms coupling electromagnetism with gravitational forces? or something else?
No - I am proposing that electromagnetic wave momentum depends on group velocity, and that evanescent waves in these circumstances exhibit superluminal group velocity. Nothing is violated - wave momentum reacts against the base plate and energy is inductively drawn from the cavity.

I'll challange you, Dr. Rodel, and Mathematica to demonstrate why/how the energy resolves from the subluminal velocity solution of wave equations. I can help you formulate the problem but we need independent confirmation and you have shown great capability with Mathematica. Do that and you will have solved the EM drive paradox.

But evanescent waves decay exponentially. If you enclose the whole device in a large (imaginary) box, everything coming out will be just regular plane waves with well-known momentum. So where's the extra momentum coming from?
Title: Re: EM Drive Developments - related to space flight applications - Thread 2
Post by: aero on 01/12/2015 06:56 pm
So the question remains: what balances the momentum change of an EM Drive accelerated by evanescent waves? Are you proposing an interaction with the Quantum Vacuum? or terms coupling electromagnetism with gravitational forces? or something else?
No - I am proposing that electromagnetic wave momentum depends on group velocity, and that evanescent waves in these circumstances exhibit superluminal group velocity. Nothing is violated - wave momentum reacts against the base plate and energy is inductively drawn from the cavity.

I'll challange you, Dr. Rodel, and Mathematica to demonstrate why/how the energy resolves from the subluminal velocity solution of wave equations. I can help you formulate the problem but we need independent confirmation and you have shown great capability with Mathematica. Do that and you will have solved the EM drive paradox.

But evanescent waves decay exponentially. If you enclose the whole device in a large (imaginary) box, everything coming out will be just regular plane waves with well-known momentum. So where's the extra momentum coming from?

I don't think it matters what happens to the photons after they depart the thruster interface. An analogy would be the classic photon rocket, or a chemical rocket engine.

A chemical rocket leaves the launch pad but the momentum of the exhaust gas is immediately diverted. The rocket still works. Same thing for a photon rocket, it doesn't matter if the expelled photon beam strikes a planetary body, a mirror or is adsorbed by a dust cloud, the rocket still works. The same rule should hold for evanescent waves.
Title: Re: EM Drive Developments - related to space flight applications - Thread 2
Post by: wembley on 01/12/2015 07:20 pm

Meanwhile, I gather there are at least two other labs (besides NASA, Yang Juan and Shawyer) working with cavity-type EmDrives. Anyone heard anything? Do we know what Boeing did after testing the one Shawyer sent them? They say they are not working with him, but that doesn't rule out developing his technology.
Title: Re: EM Drive Developments - related to space flight applications - Thread 2
Post by: Rodal on 01/12/2015 07:40 pm

... Anyone heard anything? Do we know what Boeing did after testing the one Shawyer sent them? They say they are not working with him, but that doesn't rule out developing his technology.
David Hambling  http://www.theguardian.com/profile/davidhambling
http://www.wired.co.uk/search/author/David+Hambling

wrote in Aviation Week and Space Technology Nov 5, 2012 (http://aviationweek.com/awin/propellentless-space-propulsion-research-continues):

Quote from: David-Hambling  http://aviationweek.com/awin/propellentless-space-propulsion-research-continues
Such work has evidently ceased. “Phantom Works is not working with Mr. Shawyer,” a Boeing representative says, adding that the company is no longer pursuing this avenue.
Notice that Hambling disclosed not only that “Phantom Works is not working with Mr. Shawyer” but even more final , they added  that "the company (Boeing) is no longer pursuing this avenue."   From what Hambling wrote, it reads to me that Boeing Phantom Works is no longer pursuing this (EM Drive) avenue.  Furthermore, given a previous arrangement between Boeing and Shawyer, it would be highly non-standard for Boeing to continue working on Shawyer's technology without an Intellectual Property arrangement with Shawyer that would allow this.  And, if Boeing would have conducted their own independent microwave cavity EM Drive R&D work prior to the arrangement with Shawyer, it would not make Intellectual Property sense that Boeing would have entered into an arrangement with Shawyer, as large companies usually refuse to discuss inventions with outside inventors because such discussions and arrangements create Intellectual Property issues (Ford Motor Company has an old classic legal case on such IP questions, involving windshield wipers).  Since Boeing is a very large public company with an extensive IP department, this implies that indeed "they are no longer pursuing this (EM Drive) avenue" at all, and if, hypothetically, Boeing were to conduct such development in the future, it would have to be a very different (for Intellectual Property purposes) engineering design than Shawyer's EM Drive.
In other words, Boeing would not be developing "his (Shawyer's) technology" but Boeing's separate, unique, very different (for IP purposes) technology.
Title: Re: EM Drive Developments - related to space flight applications - Thread 2
Post by: Rodal on 01/12/2015 09:01 pm

At constant input power, the thrust, and therefore the acceleration, must decrease with time, to ensure that the spacecraft's velocity never exceeds 2*Power/Thrust

Velocity measured relative to what?
It should read "At constant input power, the thrust, and therefore the acceleration, must decrease with time, to ensure that the spacecraft's (change in) velocity never exceeds 2*Power/Thrust"(I have accordingly added "change in" in the original entry).

It is the change in Kinetic Energy (between the final time under consideration and the initial time) under constant acceleration produced directly by the propulsion engine that matters and therefore it is the change in velocity that matters.  Since only the change in velocity matters, any arbitrary additive constant for defining the initial and final velocities disappears from consideration. Ultimately, the definition of "change in velocity" in this context really follows from the definition of constant acceleration as acceleration=(delta velocity)/(delta time) so that (delta velocity)=acceleration*(delta time). Given acceleration=ThrustForce/Mass, the change in velocity is uniquely defined in terms of: ThrustForce, total Mass of the spacecraft and DeltaTime (regardless of velocity-frame of observance) directly as

(delta velocity)=(ThrustForce/Mass)*(delta time).

Of course, changes in velocity produced by external forces (for example by gravitational sling-shot) are not part of this consideration, only the ThrustForce is under consideration.
Title: Re: EM Drive Developments - related to space flight applications - Thread 2
Post by: frobnicat on 01/12/2015 10:35 pm
Sounds to me the answer to the question by Vultur would then be "relative to initial velocity". Which poses the problem (already mentioned) of what records/holds the initial velocity ?

Say we have 1N/kW, 1kg spacecraft, 4MJ energy onboard to spend at a flow of 1kW (yielding 1N thrust), for a duration of 2000s (we will spend only 2MJ). a=F/m=1/1=1m/s²  Final velocity at 2000s vf=a*2000=2000m/s. DeltaV is 2km/s.
Kinetic energy Ek=.5*1*2000²=2MJ exactly what was spent. Now any more use of the thruster with the remaining 2MJ should yield less than 1N/kW efficiency if we are to be shy of the "energetic paradox".
At this point, first spacecraft meets another spacecraft that was put there (at same velocity) by conventional means. Otherwise, second spacecraft has same design (EM thruster...) and batteries loaded at 2MJ. The two spacecrafts are undistinguishable, same design, same onboard energy of 2MJ, except that one has already used his "EM drive credit" and can't anymore use power at 1N/kW efficiency, while the other is "fresh", and can have 1N/kW for 2000s. There would be a kind of "memory effect" to differentiate those two spacecrafts.

BTW, "added kinetic energy" seems a poor metric, there is always an inertial reference frame in which it is 0 (for instance relative to the mean velocity between Vinitial and Vfinal). To me it is hopeless to save propellantless schemes at thrust/power > 1/c from both energetic "paradoxes" and special rest frame (aether) : if this kind of ratio is possible one of those must be abandoned, either a problem of diminishing returns appears relative to some absolute velocity rest frame (local vector to be determined) or power is pumped from somewhere (with tachyons involved, if relativity is to be respected). Taking together apparent respect of energy conservation and relativity (change in velocity) gives worse paradox than (apparently) breaking energy conservation alone. Equating "change of velocity" with "change of kinetic energy" is ill defined : 2/m*DeltaEk = (Vf-Vr)²-(Vi-Vr)² = Vf²-Vi²+2Vr(Vi-Vf)  with Vr the arbitrary velocity of a change in inertial reference frame, how could deltaEk have any meaning as soon as Vi != Vf ?

If I may quote myself from this post (http://forum.nasaspaceflight.com/index.php?topic=29276.msg1275281#msg1275281), believe this is the proper relativistic derivation (not being challenged so far) :


3/ We are not using a bath or aether, the EMdrive just comes by on its own in nothingness and emits something that is forever lost (can't be closed, closed solutions can't see their centre of energy depart from an inertial trajectory). So if I consider a device in its own rest frame, it is spending a infinitesimal energy E, with this Energy it "creates" and gives velocity to a "thing" of mass m that is leaving and never to come back again. With Gamma the Lorentz factor 1/sqrt(1-v²/c²), v in the rest frame where the device is not (yet) moving.
E = gamma m c²  Energy spent by device to give thing of mass m a velocity of v
p = gamma m v  Momentum of the thing of mass m and velocity leaving the device forever
We agree that conservation of energy and conservation of momentum are the two most fundamental rules to apply. So for the later, the device gets a reaction kick of p momentum, at the price of spent energy E.
Thrust is momentum kicks per second, Power is spent Energy  per second. Hence :
thrust/power = p/E = v/c²

The consequences are
thrust/power < 1/c => v/c² < 1/c => v<c    classical rocket, reaction on usual mass
thrust/power = 1/c => v/c² = 1/c => v=c    photon rocket, reaction on particles of 0 rest mass
thrust/power > 1/c => v/c² > 1/c => v>c    tachyon rocket, reaction on particles of imaginary rest mass


Note that in the above inequalities, the thrust/power is understood in the very particular way (in the case of the classical rocket) that the energy equivalent content of ejected mass is counted in "spent power" (otherwise a classical rocket obviously does much better than 1/c, but it spits quite a lot of valuable energy as mass, never to be recovered...)

Insisting on both relativistic (difference, not absolute velocities) and spent/gained energy balance hypothesis should invariably have tachyons as consequence.
Title: Re: EM Drive Developments - related to space flight applications - Thread 2
Post by: Rodal on 01/12/2015 11:58 pm
1) Ron Maimon answers the question "Is the change in kinetic energy of a particle frame independent?"  with a resounding: NO: "energy isn't conserved the same way in every frame because the energy is not separate from the momentum.  This is clearest in relativity, where the energy is the time-component of the energy-momentum vector. Then if you change frames, what you called energy partly becomes momentum, and it is the conservation law of the total energy-momentum vector which is frame independent."

I would have written this instead as "the total energy-momentum vector is covariant, not invariant".

See:
 http://physics.stackexchange.com/questions/23323/is-the-change-in-kinetic-energy-of-a-particle-frame-independent

and

http://www.quora.com/Does-the-kinetic-energy-of-an-object-depend-on-the-reference-frame-of-the-observer-Why-or-why-not

and

http://physics.stackexchange.com/questions/23734/is-the-energy-conserved-in-a-moving-frame-of-reference

and

http://www.loreto.unican.es/Carpeta2012/TPT%28Tefft%29RelativityGalilean2007.pdf

2) The frame-of-reference kinetic energy issues (for non-relativistic spacecraft speeds) are eliminated by the definition of constant acceleration as acceleration=(delta velocity)/(delta time) so that (delta velocity)=acceleration*(delta time). Since from Newton's 2nd law, acceleration=ThrustForce/Mass, the change in velocity is uniquely defined in terms of: ThrustForce, total Mass of the spacecraft and DeltaTime (regardless of velocity-frame-of-reference) directly as

(delta velocity)=(ThrustForce/Mass)*(delta time).

and the intrinsic definition of the change in KineticEnergy of the spacecraft under constant acceleration:

deltaKineticEnergy = (1/2)*Mass*((delta velocity)^2)
                              = (1/2)*Mass*([(ThrustForce/Mass)*(delta time) ]^2)

where thrustForce, mass and deltaTime have frame-independent unique meanings.  This follows directly from A) the definition of constant acceleration and B) Newton's second law.

EDIT: This relationship for the deltaKineticEnergy follows by arbitrarily defining the end velocity to be equal to the deltaVelocity and arbitrarily defining the initial velocity to be zero:

end velocity = delta velocity
initial velocity = 0

effectively setting the initial kinetic energy KEinitial=0

( which we are free to do since the definition of velocity is arbitrary to within an arbitrary additive constant since the absolute value of velocity is relative)


3) You are correct, for the EM Drive to make any sense as a closed system it implies having memory of its time-history in order to never exceed the critical velocity at which (for constant InputPower throughout the time period (delta time))

(1/2)*Mass*[(ThrustForce/Mass)*(delta time) ]^2 = InputPower*(delta time)


This equation relating thrust force to change in velocity of the spacecraft is consistent with textbooks on Rocket Propulsion, for example: Sutton's textbook on Elements of Rocket Propulsion (equation 19-1 Rocket propulsion elements 7th edition- Sutton)).

Quote from: equation 19-1 Rocket propulsion elements 7th edition- Sutton
for any vehicle power P, the thrust that may be provided is: ThrustForce = 2*Power/velocity

As to how such a memory can arise in a seemingly simple system of microwave photons in a cavity, let's add this to the enigma of conservation of momentum of the EM Drive.   We know of materials that have memory of their time history (of stress, strain, stress rate and strain rate) but they are macroscopic materials much more complex than the apparently much simpler microwave cavity EM Drive.

Maimon's last words maybe particularly pertinent:

Quote from: Ron Maimon
the transformation law shows that if energy is conserved in any one frame for a system which also conserves momentum, then the energy is conserved in another frame. This means that the total energy change is only meaningful for a system where no momentum flows in or out

If the EM drive conserves momentum by interacting with the QuantumVacuum for example, we have an open system where momentum will be flowing in or out of the cavity, and therefore the open system should be taken into account in the conservation of kinetic energy (which we have not done) in order to arrive at a satisfactory answer.
Title: Re: EM Drive Developments - related to space flight applications - Thread 2
Post by: ThinkerX on 01/13/2015 02:01 am
Quote
3) You are correct, for the EM Drive to make any sense it implies having memory of its time-history in order to never exceed the critical velocity at which (for constant InputPower throughout the time period (delta time))

Strangely enough, Doctor McCulloch's latest blog post deals with this issue or its first cousin.
Title: Re: EM Drive Developments - related to space flight applications - Thread 2
Post by: Rodal on 01/14/2015 01:02 am
....
Note that in the above inequalities, the thrust/power is understood in the very particular way (in the case of the classical rocket) that the energy equivalent content of ejected mass is counted in "spent power" (otherwise a classical rocket obviously does much better than 1/c, ...)
....

Just like you take into account the energy of ejected mass from a classical rocket, also for an EM Drive the energy and momentum of the total open system has to be taken into account to reach any conclusions on velocity limitations:

*for Dr.McCulloch's theory, the energy/momentum of the radiated Unruh radiation has to be taken into account

*for theories advocating interaction with the Quantum Vacuum (Dr. White's or the theories considered by @Mulletron) the energy/momentum of the involved Quantum Vacuum has to be taken into account.  An example would be given by Dr.White's analogy of the propeller in a submarine: the energy/momentum of the water propelled by the propeller has to be taken into account.

*ditto for theories advocating coupling between electromagnetism and gravitational forces, or theories advocating coupling with Dark Matter, or any other theories explaining conservation of momentum of the EM Drive: the energy/momentum of the coupled field also needs to be taken into account

Thus my conclusion is that Joosten and White's "paradox" analysis for the EM Drive (Appendix A of  "Human Outer Solar System Exploration via Q-Thruster Technology" by B. Kent Joosten and Harold G. “Sonny” White, http://ntrs.nasa.gov/archive/nasa/casi.ntrs.nasa.gov/20140013174.pdf ) is incorrect/incomplete because they failed to take into account the energy/momentum of the involved Quantum Vacuum (which they advocate as the reason for conservation of momentum yet they fail to take into account when they perform the kinetic energy constraint).

If the EM drive conserves momentum by interacting with the QuantumVacuum, we have an open system where momentum will be flowing in or out of the cavity, and therefore the open system should be taken into account in the conservation of energy in order to arrive at a satisfactory answer concerning any velocity limitation.

(http://history.nasa.gov/SP-4404/p275.jpg)
(http://upload.wikimedia.org/wikipedia/commons/6/66/Cavitating-prop.jpg)
Title: Re: EM Drive Developments - related to space flight applications - Thread 2
Post by: Stormbringer on 01/14/2015 04:55 am
http://www.technologyreview.com/view/426281/giant-casimir-effect-predicted-inside-metamaterials/

http://arxiv.org/abs/1110.1919

With this type of set up could you determine if the casimir effect is a suitable stunt double for exotic energy or mass for purposes of White's warp drive idea?  :-\
Title: Re: EM Drive Developments - related to space flight applications - Thread 2
Post by: Rodal on 01/14/2015 02:23 pm
http://www.technologyreview.com/view/426281/giant-casimir-effect-predicted-inside-metamaterials/

http://arxiv.org/abs/1110.1919

With this type of set up could you determine if the casimir effect is a suitable stunt double for exotic energy or mass for purposes of White's warp drive idea?  :-\

The article was published in 2011.  The authors claim that

Quote from: Tian-Ming Zhao, Rong-Xin Miao
Since the parameters of metamaterials we proposed are quite simple, this experiment would be easily implemented in laboratory

We are now in 2015.  If the experiments to confirm their theoretical expectation are so easy to implement in a laboratory as the authors claimed in their 2011 paper, has anyone heard anything regarding their experimental confirmation or nullification?   :-\
Title: Re: EM Drive Developments - related to space flight applications - Thread 2
Post by: Notsosureofit on 01/14/2015 03:16 pm
http://www.mouser.com/new/teampconnectors/te-connectivity-ariso/?utm_medium=email&utm_source=january2015&utm_campaign=cm-enews-tech2015&utm_content=p2-lm

Just an FYI on contactless connector for garage builds.

Update:
OK! I can get all 3 quantum numbers to agree w/ the Brady results.  The mode numbers they give have to be erroneous as they don't fit w/ the cavity frequency calculations.  That limits the fit.  I'll try to get time to send the whole ball-a-wax by this weekend.
Title: Re: EM Drive Developments - related to space flight applications - Thread 2
Post by: Notsosureofit on 01/17/2015 02:39 pm
This is the latest exposition on the proposition that dispersion caused by an accelerating frame of reference implied an accelerating frame of reference caused by a dispersive cavity resonator. (to 1st order using mass-less, perfectly conducting cavity)

Unlike the previous waveguide attempt, this wasn't done on a series of scribbled post-it notes.

Here I've tried to incorporate all 3 subscripts into the expression and eliminate some of my own confusion about the reported modes. This should allow for some comparison w/ the Dynamical Casimir Effect which also shows Doppler type behavior.

I had a lot of trouble trying to figure out the X's from the "given" modes until I remembered they were in the cavity frequency formulas.  That eliminated most of the possible modes.

Starting with the expressions for the frequency of an RF cavity:

f = (c/(2*Pi))*((X[sub m,n]/R)^2+((p*Pi)/L)^2)^.5

Where I use: R = ((1/2)*(a^2+b^2))^0.5

a and b are the end plate radii and L is the cavity length.

For TM modes, X[sub m,n] = the n-th zero of the m-th Bessel function.
[1,1]=3.83, [0,1]=2.40, [0,2]=5.52 [1,2]=7.02, [2,1]=5.14, [2,2]=8.42, [1,3]=10.17, etc.

and for TE modes, X'[subm,n] = the n-th zero of the derivative of the m-th Bessel function.
[0,1]=3.83, [1,1]=1.84, [2,1]=3.05, [0,2]=7.02, [1,2]=5.33, [1,3]=8.54, [0,3]=10.17, [2,2]=6.71, etc.

So, using these to identify the frequencies, I chose:

Bradya => TM122 or TE022  X[sub m,n] = 7.02 p = 2
Bradyb => TE213           X'[sub m,n] = 3.05 p = 3
Bradyc => TE222           X'[sub m,n] = 6.71 p = 2

Rotate the dispersion relation of the cavity into doppler frame to get the Doppler shifts, that is to say, look at the dispersion curve intersections of constant wave number instead of constant frequency.

df = (1/(2*f))*(c/(2*Pi))^2*(X[sub m,n])^2*((1/a^2)-(1/b^2))

and from there the expression for the acceleration g from:

g = (c^2/L)*(df/f) such that:

g = (c^2/(2*L*f^2))*(c/(2*Pi))^2*(X[sub m,n])^2*((1/a^2)-(1/b^2))

Using the "weight" of the photon in the accelerated frame from:

"W" = (h*f/c^2)*g =>  "W" = T = (h/L)*df

gives thrust per photon:

T = (h/(2*L*f))*(c/(2*pi))^2*(X[sub m,n])^2*((1/a^2)-(1/b^2))

If the number of photons is ((P/f)*Q)/hf then:

NT = P*Q*(1/(2*L*f^3))(c/(2*pi))^2*(X[sub m,n])^2*((1/a^2)-(1/b^2))

Plugging the numbers in we get:

Bradya NT= 1.04e-4 vs 9.12e-5 Pwr= 16.9 Q=7320  TM122 or TE022
Bradyb NT= 4.80e-5 vs 5.01e-5 Pwr= 16.7 Q=18100 TE213
Bradyc NT= 4.78e-5 vs 5.54e-5 Pwr= 2.6  Q=22000 TE222

I kept everything in terms of frequency this time to minimize my own confusion.  Hopefully, I didn't screw it up and can get on to momentum conservation. 

Thanks for your patience.

Title: Re: EM Drive Developments - related to space flight applications - Thread 2
Post by: Mulletron on 01/17/2015 03:06 pm
Keeping my eyes on the conference scene, nothing out of this one: http://www.aiaa-scitech.org/ http://www.aiaa.org/EventsCalendar.aspx?id=79 I scanned the program and didn't spot anything.

I suspect the next two potential ones will be STAIF II http://www.staif2.org/about.html and this one: http://www.aiaa-propulsionenergy.org/default.aspx

Things have been pretty quiet for a few months now. What's going on here? The follow up news after "anomalous thrust..." hasn't been coming. Anyone have any new gouge?

Has there been the least indication yet when we are too get some more results in relation to those who are experimenting in this area?

There is essentially a news blackout on the experimental side from NASA.

The problem is, if this drive really works, the defence implications are huge. Those in charge might feel it would not necessarily be a good idea to go public with it -- especially from the US perspective. Remember that 'Sputnik moment'?

@wembley, do you have any specific info about why you think there is a news blackout?

Also, do you have any further verifiable information from this post: http://forum.nasaspaceflight.com/index.php?topic=29276.msg1265607#msg1265607 stating Shawyer stopped using a dielectric section?
Title: Re: EM Drive Developments - related to space flight applications - Thread 2
Post by: Star One on 01/17/2015 07:16 pm
With my tinfoil hat on;) it's gone into the black and no more will be heard about it. Mind you surely the genie is out of the box now for it as an experiment able to be replicated including by other countries.

With my serious hat on perhaps you should ask on Talk-Polywell as there's probably people on there who might have an answer for you?
Title: Re: EM Drive Developments - related to space flight applications - Thread 2
Post by: RotoSequence on 01/18/2015 10:05 am
Poking around the internet, I ended up at this paper on Arxiv: http://arxiv.org/abs/1411.3987

Quote
Photons that travel in free space slower than the speed of light

That the speed of light in free space is constant is a cornerstone of modern physics. However, light beams have finite transverse size, which leads to a modification of their wavevectors resulting in a change to their phase and group velocities. We study the group velocity of single photons by measuring a change in their arrival time that results from changing the beam's transverse spatial structure. Using time-correlated photon pairs we show a reduction of the group velocity of photons in both a Bessel beam and photons in a focused Gaussian beam. In both cases, the delay is several microns over a propagation distance of the order of 1 m. Our work highlights that, even in free space, the invariance of the speed of light only applies to plane waves. Introducing spatial structure to an optical beam, even for a single photon, reduces the group velocity of the light by a readily measurable amount.

What effect does reduced group velocity in free space have on the momentum of photons?
Title: Re: EM Drive Developments - related to space flight applications - Thread 2
Post by: Mulletron on 01/18/2015 12:17 pm
With my tinfoil hat on;) it's gone into the black and no more will be heard about it. Mind you surely the genie is out of the box now for it as an experiment able to be replicated including by other countries.

With my serious hat on perhaps you should ask on Talk-Polywell as there's probably people on there who might have an answer for you?

Since the genie is out of the bottle there is no way to put it in the black. They already went public with it. But as @Wembley stated, there are huge defense uses for this....why DARPA is involved. The military applications of it will be black, for certain, as they should be.


There is essentially a news blackout on the experimental side from NASA.

The problem is, if this drive really works, the defence implications are huge. Those in charge might feel it would not necessarily be a good idea to go public with it -- especially from the US perspective. Remember that 'Sputnik moment'?

It would be an interesting exercise to see what emdrives could do for say a boomer http://en.wikipedia.org/wiki/Ohio-class_submarine with its S8G reactor onboard http://www.ewp.rpi.edu/hartford/~ernesto/F2010/EP2/Materials4Students/Misiaszek/NuclearMarinePropulsion.pdf http://en.wikipedia.org/wiki/S8G_reactor

So if these emdrives work, they'll likely find their way into applications such as this before they are helping us explore the solar system.

On the flip side, this will flood the arena with dollars and expertise, especially if the Chinese have already one upped us.

Break:
So as far as space flight applications, the other day when, for the purpose of exercise, I converted the ISS to an interplanetary spaceship, would such a thing in the real world make sense? Or is it just plain ridiculous? I was thinking that from an economic standpoint, converting it to that use, and boosting it out of LEO and beyond, would be far cheaper than building a whole new space craft. And since it is modular already, it has to be a pretty straightforward adaptation for an exploration mission.

Found this: http://www.universetoday.com/15561/the-space-station-as-an-interplanetary-transport-vehicle/
Title: Re: EM Drive Developments - related to space flight applications - Thread 2
Post by: Rodal on 01/18/2015 05:03 pm
....
the Brady results.  The mode numbers they give have to be erroneous as they don't fit w/ the cavity frequency calculations.  ...

...
Here I've tried to incorporate all 3 subscripts into the expression and eliminate some of my own confusion about the reported modes. ...I had a lot of trouble trying to figure out the X's from the "given" modes until I remembered they were in the cavity frequency formulas.  That eliminated most of the possible modes...
So, using these to identify the frequencies, I chose:

Bradya => TM122 or TE022  X[sub m,n] = 7.02 p = 2
Bradyb => TE213           X[sub m,n] = 3.05 p = 3
Bradyc => TE222           X[sub m,n] = 6.71 p = 2

...
Bradya NT= 1.04e-4 vs 9.12e-5 Pwr= 16.9 Q=7320  TM122 or TE022
Bradyb NT= 4.80e-5 vs 5.01e-5 Pwr= 16.7 Q=18100 TE213
Bradyc NT= 4.78e-5 vs 5.54e-5 Pwr= 2.6  Q=22000 TE222

...

First of all congratulations on the extremely impressive further development of your theoretical prediction.  Your formulas now are superior to the ones of Shawyer and McCulloch in these important considerations:

1) the only ones that show mode shape dependence
2) closest to NASA's Brady et.al. experimental results.

There is a lot to discuss, but I would like to start with your finding that the Brady mode shape quantum numbers (m, n, p) must be erroneous.

Concerning Brady, White, March, Lawrence and Davies "Anomalous Thrust Production from an RF Test Device Measured on a Low-Thrust Torsion Pendulum", Propulsion and Energy Forum, Cleveland, OH
AIAA 2014-4029, (http://www.libertariannews.org/wp-content/uploads/2014/07/AnomalousThrustProductionFromanRFTestDevice-BradyEtAl.pdf ) I note, that:

A) The NASA authors report that they used the commercial finite element software COMSOL Multiphysics® to calculate the frequencies and mode shapes.   COMSOL Multiphysics® software can indeed calculate the frequencies and it can be used to plot the mode shapes, but COMSOL Multiphysics® does not define the mode shape quantum numbers.   Such mode shape quantum numbers can only be defined by convention.

B) The truncated cone (tapered frustum) is asymptotically closest to a cylindrical waveguide, the two becoming identical at a tapered cone angle of zero.  However we must admit that a truncated cone mode shapes cannot be identical to the mode shapes of a cylindrical cavity.  However, as an engineering approximation, for the tapered cone angle being small it should be acceptable to describe the tapered cone mode shapes with the same terminology as the one used for a cylindrical waveguide.

C) The presence of the dielectric is very important in discussing mode shapes, as the presence of a dielectric necessarily makes the cavity mode shape different from an empty cavity. Hence the description of the cavity mode shapes entails both:

C1) Idealizing the tapered cavity as a cylindrical cavity and
C2) Ignoring the presence of the dielectric: assuming an empty cavity.

D) the standard convention to name electromagnetic modes of a cylindrical cavity in terms of the three quantum numbers m, n and p that I am familiar with is:

The first subscript (m) is the azimuthal mode number: it indicates the number of full-wave patterns around the circumference of the cavity.   It is zero for modes in which there is no variation in the circumferential direction.

The second subscript (n) is the radial mode number: it indicates the number of half-wave patterns across the diameter.  The radial mode number (n) plus one indicates the number of nodes across the diameter (counting as nodes the end nodes).

The third subscript (p) is the longitudinal mode number.  It indicates the number of full half-wave patterns along the longitudinal length of the cavity.  It is zero for modes in which there is no variation in the longitudinal direction.


E) The following file (http://www.engr.sjsu.edu/rkwok/EE172/circular_WG.JPG, created by Dr. Ray Kwok at San Jose State University) shows the contour plots for the first Transverse Electric (TE) and Transverse Magnetic (TM) modes of a cylindrical cavity


F) Comparing the mode shapes determined by @NotSoSureOfIt with the mode shapes reported by NASA's Brady et.al., we show the corresponding mode number contour plot from Dr. Ray Kwok at San Jose State University (where we drop the longitudinal mode number "p" for simplification):


NASA-Test  NASA-mode-reported Kwok#   @NotSoSureOfIt-mode        Kwok#

Brady a       TM21                           7               TM12                                     15
Brady a       TM21                           7               TE02                                      14
Brady b       TM21                           7               TE21                                      3
Brady c        TE01                           5               TE22                                      13

http://www.engr.sjsu.edu/rkwok/EE172/circular_WG.JPG

Title: Re: EM Drive Developments - related to space flight applications - Thread 2
Post by: Notsosureofit on 01/18/2015 06:43 pm
@RODAL

A.  I'm still having conceptual difficulties w/ the mode numbers trying to resolve the waveguide vs cavity versions of this thing.

B. Don't have access to COMSOL anyway

C. Used  square avg to get close to an equivalent vol cylinder

D. Didn't use included dielectric, but assumed it might change the res freq somewhat.  Prob the biggest errors. (enough to change mode numbers?)

E. Sounds like what I wound up using

F. Nice !

G.  Those still might not be the correct modes given that the dielectric is not taken into account along w/ the dimensional approximations.  i didn't try all matches.

@RotoSequence

http://arxiv.org/abs/1411.3987

This is a very interesting paper !
Title: Re: EM Drive Developments - related to space flight applications - Thread 2
Post by: Rodal on 01/18/2015 07:32 pm
Poking around the internet, I ended up at this paper on Arxiv: http://arxiv.org/abs/1411.3987

Quote
Photons that travel in free space slower than the speed of light

That the speed of light in free space is constant is a cornerstone of modern physics. However, light beams have finite transverse size, which leads to a modification of their wavevectors resulting in a change to their phase and group velocities. We study the group velocity of single photons by measuring a change in their arrival time that results from changing the beam's transverse spatial structure. Using time-correlated photon pairs we show a reduction of the group velocity of photons in both a Bessel beam and photons in a focused Gaussian beam. In both cases, the delay is several microns over a propagation distance of the order of 1 m. Our work highlights that, even in free space, the invariance of the speed of light only applies to plane waves. Introducing spatial structure to an optical beam, even for a single photon, reduces the group velocity of the light by a readily measurable amount.

What effect does reduced group velocity in free space have on the momentum of photons?

I agree with @NotSoSureOfIt, very interesting post.  The ending paragraph is also worthy of note:

Quote
The speed of light in free space propagation is a fundamental quantity. It holds a pivotal
role in the foundations of relativity and field theory....
It has previously been experimentally established that single photons travel at the group
velocity. We have now shown that transverse structuring of the photon results in a
decrease in the group velocity along the axis of propagation.
The effect can be derived
from a simple geometric argument, which is also supported by a rigorous calculation of
the harmonic average of the group velocity. Beyond light, the effect observed will have
applications to any wave theory, including sound waves and, potentially, gravitational
waves.
Title: Re: EM Drive Developments - related to space flight applications - Thread 2
Post by: Stormbringer on 01/18/2015 08:57 pm
wait. going from "that" book again: the speed of light is constant in local frames of reference only. If you are way over there and i am way over here we measure light where we happen to be the same (locally invariant.)  But if am over here and i measure the speed of light over there (where you are) i may get a different value for the speed of light (where you are) than you do.

E.G; if you are near the event horizon of a black hole and i am way over here; you still experience light and other things as you would expect but i see light and other things as frozen where you are at.

I have encountered dozens of people who argue for pages and pages that relativistic time dilation causes a stationary observer on earth to experience 1000's of years when a relativistic rocket (at 99% C) travels 4 light years to Alpha Centauri. They get it completely wrong and argue the wrong reference frame for pages as if their lives depended on it. These are people that have at least read books on relativity written at the popular level.

In short -thinking about reference frames can warp your mind and your calculations. I recommend laying down and avoiding them.
Title: Re: EM Drive Developments - related to space flight applications - Thread 2
Post by: ThinkerX on 01/18/2015 10:03 pm
Oh well, time for me to make a spectacle of myself again...

if I am following Notsosureofits and Rodal correctly, then...

1) the experimenters are proceeding from badly flawed assumptions about mode, among other things; and

2) their positive results come about through chance or design accident.

Title: Re: EM Drive Developments - related to space flight applications - Thread 2
Post by: Rodal on 01/18/2015 10:58 pm
Oh well, time for me to make a spectacle of myself again...

if I am following Notsosureofits and Rodal correctly, then...

1) the experimenters are proceeding from badly flawed assumptions about mode, among other things; and

2) their positive results come about through chance or design accident.
The fact that they do not sufficiently explore the complete mode shapes in the cavity does not mean that they proceeded from "badly flawed assumptions".   That is too severe an assessment.

Brady' a and b experiments  (which they labeled with the TM211 mode for reasons that are unclear and they do not explain) had significantly lower thrust/InputPower than Brady's c experiment (which they labeled TE012).  Therefore Brady's c is most important to deal with.  My contention is that Brady c "looks" like a TE01 mode strictly because of the  field in the dielectric.  If you look at the contour plot shown by Brady, the only part of the cavity experiencing significant (yellow and red) field contours is the dielectric.  The rest of the cavity has extremely small (dark blue) levels in comparison. See:

(http://forum.nasaspaceflight.com/index.php?action=dlattach;topic=36313.0;attach=631726;image)

Therefore (just looking at the above picture, based solely on the field within the dielectric and ignoring the field in the rest of the cavity as being negligible) it makes some sense for Brady to label the c mode as TE01.

EDIT: On the other hand, the presence of the dielectric may not be that relevant to what matters: the thrust, because NotSoSureOfIt predicts the reported experimentally measured thrust based on an empty cavity with no dielectric (and hence a very different mode shape: TE22 instead of TE01).

 

What is really most impressive is that NotSoSureOfIt has come up with an analytical solution (which Brady et.al. did not do) that models the experimental results accurately in terms of mode shape quantum numbers, and that NotSoSureOfIt has done this with a minimum of data compared with the data available to Brady et.al.
Title: Re: EM Drive Developments - related to space flight applications - Thread 2
Post by: Notsosureofit on 01/19/2015 12:11 am
For c: TE01 doesn't compute but TE022 is a good fit, maybe better than TE222.  (w/o dielectric that is)

Edit:  Nope, frequency is too high (dielectric might bring it down ?)

For b: TE213 seems the only good fit

For a: Can't tell the difference between TE022 andTM122

Probably as good as this model is gonna get w/o the dielectric (and the COMSOL)
Title: Re: EM Drive Developments - related to space flight applications - Thread 2
Post by: Mulletron on 01/19/2015 03:53 pm
Quick question:
Did someone mention in the past that the equations at the Oracle are no good? Or were they good?
http://en.wikipedia.org/wiki/Microwave_cavity#Cylindrical_cavity

or are these good
Am I better off going with the Kwok slide 13?
http://www.engr.sjsu.edu/rkwok/EE172/Cavity_Resonator.pdf

or this guy's method?
http://www.chrislmueller.com/studies/Jackson8-6.pdf

Playing catch up here. This page is nifty. http://mathworld.wolfram.com/BesselFunctionZeros.html
Title: Re: EM Drive Developments - related to space flight applications - Thread 2
Post by: Rodal on 01/19/2015 05:11 pm
Quick question:
Did someone mention in the past that the equations at the Oracle are no good? Or were they good?
http://en.wikipedia.org/wiki/Microwave_cavity#Cylindrical_cavity

or are these good
Am I better off going with the Kwok slide 13?
http://www.engr.sjsu.edu/rkwok/EE172/Cavity_Resonator.pdf

or this guy's method?
http://www.chrislmueller.com/studies/Jackson8-6.pdf

Playing catch up here. This page is nifty. http://mathworld.wolfram.com/BesselFunctionZeros.html
Quick glance: I took a quick gander at them and they all look identical, insofar as being related to X'm,n and  Xm,n  (the zeros of the derivative of the Bessel function or the zeros of the Bessel function respectively) for TE and TM respectively.  What difference do you see between them?

For example, notice (from http://wwwal.kuicr.kyoto-u.ac.jp/www/accelerator/a4/besselroot.htmlx ) 
that X'1,1 = 1.84118378134065

And therefore in Kwok slide 13 (http://www.engr.sjsu.edu/rkwok/EE172/Cavity_Resonator.pdf ) with radius of the Coke can a = 1.25

kc = X'1,1 / a = 1.8412 / 1.25 = 1.473

f111=(c/(2*Pi))*Sqrt[(kc)^2+(p*Pi/d)^2]

The expressions for ChrisMueller and Wikipedia look identical except for the factor of 2*Pi due to the fact that ChrisMueller calculates the angular frequency (omega) while Wikipedia calculates the frequency (f).   Since omega = 2 *Pi*f, the expressions in Wikipedia and in ChrisMueller are both identical and both are correct. Kwok's expression is missing the factor of the squareRoot of the  relative permeability and relative permittivity of the cavity filling because Kwok explicitly wrote that he is calculating the example for air inside the cavity. (For a vacuum they are exactly unity, for air they are approximately unity).

(http://upload.wikimedia.org/wikipedia/commons/c/c9/AngularFrequency.gif)
Title: Re: EM Drive Developments - related to space flight applications - Thread 2
Post by: Mulletron on 01/19/2015 05:53 pm
I get the air thing which means I can just drop that part. The part that screwed with me was

f = (c/(2*Pi))*((X[sub m,n]/R)^2+((p*Pi)/L)^2)^.5

silly enough the ^.5 part because I'm used to seeing sqrt typed out, but that works too. Thanks for clearing all that up.

This is all looking pretty good, so far as I've gotten. I think it might be time for @Notsosureofit to put some polish on it, add some prose, and make it a packaged deal.

I think it might be time for a spaceflight application minute. Theory is getting pretty heavy once again.
Title: Re: EM Drive Developments - related to space flight applications - Thread 2
Post by: Rodal on 01/19/2015 06:01 pm
...
I think it might be time for a spaceflight application minute...
Good idea, I'm ready :-)
Title: Re: EM Drive Developments - related to space flight applications - Thread 2
Post by: Rodal on 01/19/2015 06:30 pm
...
For TM modes, X[sub m,n] = the n-th zero of the m-th Bessel function.
[1,1]=3.83, [0,1]=2.40, [0,2]=5.52 [1,2]=7.02, [2,1]=5.14, [2,2]=8.42, [1,3]=10.17, etc.

and for TE modes, X[subm,n] = the n-th zero of the derivative of the m-th Bessel function.
[0,1]=3.83, [1,1]=1.84, [2,1]=3.05, [0,2]=7.02, [1,2]=5.33, [1,3]=8.54, [0,3]=10.17, [2,2]=6.71, etc.

...
A minor point.  For further clarity you may want to include a prime symbol (or an apostrophe) on X

"for TE modes, X'[subm,n] = the n-th zero of the derivative of the m-th Bessel function"
instead of
"for TE modes, X[subm,n] = the n-th zero of the derivative of the m-th Bessel function"

to differentiate X' (as used for the TE modes) from X (as used for the TM modes).  :)
Title: Re: EM Drive Developments - related to space flight applications - Thread 2
Post by: Mulletron on 01/19/2015 07:56 pm
...
For TM modes, X[sub m,n] = the n-th zero of the m-th Bessel function.
[1,1]=3.83, [0,1]=2.40, [0,2]=5.52 [1,2]=7.02, [2,1]=5.14, [2,2]=8.42, [1,3]=10.17, etc.

and for TE modes, X[subm,n] = the n-th zero of the derivative of the m-th Bessel function.
[0,1]=3.83, [1,1]=1.84, [2,1]=3.05, [0,2]=7.02, [1,2]=5.33, [1,3]=8.54, [0,3]=10.17, [2,2]=6.71, etc.

...
A minor point.  For further clarity you may want to include a prime symbol (or an apostrophe) on X

"for TE modes, X'[subm,n] = the n-th zero of the derivative of the m-th Bessel function"
instead of
"for TE modes, X[subm,n] = the n-th zero of the derivative of the m-th Bessel function"

to differentiate X' (as used for the TE modes) from X (as used for the TM modes).  :)

Perfect timing, in the RF & Microwave Toolbox app I've been using, I found they messed up exactly what you are saying. Emailed developer. See screenshot.

Still trying to think of another spaceflight application. We need more power Scotty!
(http://www.quickmeme.com/img/e6/e61f7f0012fb504e3292ba7068f661f9e67231b46e415e5b569847b2a8d9bcdf.jpg)
Title: Re: EM Drive Developments - related to space flight applications - Thread 2
Post by: Asteroza on 01/19/2015 10:22 pm
For applications, I would imagine a key figure is practical thrust/weight and thrust/power. It probably isn't enough for a very low orbit PROFAC atmosphere collector, but a high orbit PROFAC might work. Tether/rotovator reboost is another application.
Title: Re: EM Drive Developments - related to space flight applications - Thread 2
Post by: ThinkerX on 01/19/2015 11:55 pm
Space travel applications...

If applied, would 'Notsosureofits's' mode calculations significantly help or hinder the 'Mulletron Mission to Saturn'? 
Title: Re: EM Drive Developments - related to space flight applications - Thread 2
Post by: Notsosureofit on 01/20/2015 12:18 am
Space travel applications...

If applied, would 'Notsosureofit's mode calculations significantly help or hinder the 'Mulletron Mission to Saturn'?

Might help if it could calculate an "optimum" cavity shape.   Such has been done for acoustic cavity refrigerators, etc.  What is the maximum asymmetric dispersion you can get ?  What is the optimum frequency ? (and the highest Q, of course)

Then too, it's not yet apparent why this calculation works at all.

Maybe that's what the Cannae  was about, short cavity w/ high mode numbers ? I could see where they might have been trying to stabilize some mode w/ the fins, sorta like some magnetrons.

Title: Re: EM Drive Developments - related to space flight applications - Thread 2
Post by: Mulletron on 01/20/2015 10:49 am
Space travel applications...

If applied, would 'Notsosureofit's mode calculations significantly help or hinder the 'Mulletron Mission to Saturn'?

Might help if it could calculate an "optimum" cavity shape.   Such has been done for acoustic cavity refrigerators, etc.  What is the maximum asymmetric dispersion you can get ?  What is the optimum frequency ? (and the highest Q, of course)

Then too, it's not yet apparent why this calculation works at all.


I think the other half of the calculation remains to be completed, which is the part concerning how to, and may provide a way to, conserve momentum. The other side of the momentum vector diagram concerns vacuum radiation pressure; well I think it is anyway. At the bottom of page 91 is a nice mashup. We've discussed this in depth in thread one, yet the math remains elusive, but I don't think it is out of our reach.

Google book link to: The Quantum Vacuum: A Scientific and Philosophical Concept, from Electrodynamics to String Theory and the Geometry of the Microscopic World. Luciano Boi
http://bit.ly/1CKmGWX
Title: Re: EM Drive Developments - related to space flight applications - Thread 2
Post by: aero on 01/20/2015 05:27 pm
@RODAL

A.  I'm still having conceptual difficulties w/ the mode numbers trying to resolve the waveguide vs cavity versions of this thing.

B. Don't have access to COMSOL anyway

C. Used  square avg to get close to an equivalent vol cylinder

D. Didn't use included dielectric, but assumed it might change the res freq somewhat.  Prob the biggest errors. (enough to change mode numbers?)

E. Sounds like what I wound up using

F. Nice !

G.  Those still might not be the correct modes given that the dielectric is not taken into account along w/ the dimensional approximations.  i didn't try all matches.

                                                                 ...snip...

@Notsosureofit

I encountered similar problems with the mode numbers. I note here http://mathworld.wolfram.com/BesselFunctionZeros.html (http://mathworld.wolfram.com/BesselFunctionZeros.html) that the first J'0(x) value is given as non-zero in the table but the curve of J0(x) clearly shows a zero for the derivative at x=0. This is explained in the text, but the text does not resolve the question of which value to use for cavity resonance. That of course will change the TE mode numbers.

There is also the remark that I found (somewhere) while reading up on Bessel functions or resonance cavitys. In communicating mode numbers it is important to specify the order of the subscripts. Some people communicate TE0,1 as TE1,0 and so forth.

Regarding your concern W.R.T. frequency changing with dielectric. The dielectric constant for the Brady cavity dielectric is about 1.76, I think. I obtained this value by iteratively running meep for different values of the dielectric constant and using Harminv to calculate the resonant frequency. The cavity resonated at 1880.5 MHz using a dielectric value of 1.76. I called that close enough. Note that is about 50% density of P.E.

I will check using meep again to see how strong an effect the dielectric constant has on the resonance frequency. I have conflicting recall on that subject at the moment.
Title: Re: EM Drive Developments - related to space flight applications - Thread 2
Post by: Rodal on 01/20/2015 07:56 pm
...
For TM modes, X[sub m,n] = the n-th zero of the m-th Bessel function.
[1,1]=3.83, [0,1]=2.40, [0,2]=5.52 [1,2]=7.02, [2,1]=5.14, [2,2]=8.42, [1,3]=10.17, etc.

and for TE modes, X'[subm,n] = the n-th zero of the derivative of the m-th Bessel function.
[0,1]=3.83, [1,1]=1.84, [2,1]=3.05, [0,2]=7.02, [1,2]=5.33, [1,3]=8.54, [0,3]=10.17, [2,2]=6.71, etc.

So, using these to identify the frequencies, I chose:

Bradya => TM122 or TE022  X[sub m,n] = 7.02 p = 2
Bradyb => TE213           X'[sub m,n] = 3.05 p = 3
Bradyc => TE222           X'[sub m,n] = 6.71 p = 2

....

1) I have checked your above-given definitions for Xm,n and  X'm,n vs. the ones obtained from Mathematica and vs. the ones given by University of Kyoto, Japan in http://wwwal.kuicr.kyoto-u.ac.jp/www/accelerator/a4/besselroot.htmlx .  They are all identical, therefore the discrepancies in mode shape definitions that I will discuss below have nothing to do with the definitions of  Xm,n and X'm,n .  You and I are using identical definitions.  (For anybody interested in the reason why Mathematica, the University of Kyoto and others has the J'0(x)@x=0 value as non-zero (as opposed to the old reference: Abramowitz) , see for example Theorem 3.1 in http://www.irjabs.com/files_site/paperlist/r_1259_130901233803.pdf :  the value of the derivative at the origin (x=0) is mathematically undefined as the closed-form solution has zero/zero at the origin.)


2) It appears that the exact geometrical dimensions play a paramount role concerning the actual mode shape for a given frequency, particularly for higher modes, where different mode shapes are more likely to give frequencies close to each other.  This makes sense.

3) Let's define as "Aero geometry" the following definition for the NASA Brady et. al. cavity:

Aero Best estimate as of 11/9/2014    http://forum.nasaspaceflight.com/index.php?topic=29276.msg1285896#msg1285896
   
cavityLength = 0.24173 m
bigDiameter = 0.27246 m
smallDiameter = 0.15875 m

4) Let's define as "Fornaro geometry" the following definition for the NASA Brady et. al. cavity:

Fornaro estimate    http://forum.nasaspaceflight.com/index.php?topic=36313.msg1302455#msg1302455
   
cavityLength = 0.332 m
bigDiameter = 0.397 m
smallDiameter = 0.244 m

5) Let's use the GeometricalMeanDiameter=Sqrt[bigDiameter*smallDiameter] as the equivalent diameter of the equivalent cylindrical cavity

6) Given the experimentally reported frequencies, the geometrical dimensions and the value of speed of light in air, one can invert the frequency equation (see: http://en.wikipedia.org/wiki/Microwave_cavity#Cylindrical_cavity ) to obtain Xm,n and  X'm values as a function of constants and the longitudinal mode shape number "p". Let's define the error difference between these Xm,n and  X'm,n  values and actual Xm,n and  X'm values as:

error= (value of Xm,n or  X'm,n obtained from frequency eqn.)/ (correct value of Xm,n or  X'm,n ) -1

where Xm,n is used for TM modes and X'm,n is used for TE modes.

7) Then I obtain the following mode shapes:


BRADY "A"

Fornaro Geometry

Best result:  TM022  error=+0.956%
2nd best: TM310 error= - 1.206%

Aero Geometry

Best result: TE310  error=+0.255%
2nd best: TM111 or TE011 error= + 4.11%

BRADY "B"

Fornaro Geometry

Best result:  TM310  error= - 0.997%
2nd best: TM022 error= + 1.230%

Aero Geometry

Best result:  TE310  error=+0.468%
2nd best: TM111 or TE011 error= + 4.36%


BRADY "C"

Fornaro Geometry

Best result:  TE412  error= + 1.166%
2nd best: TE313 error= + 1.259%

Aero Geometry

Best result:  TE212  error=+0.859%
2nd best: TE011 or TM111 ;  error= + 0.971%




CONCLUSIONS

1) The mode-shape to frequency relation is very sensitive to the exact geometrical dimensions of the cavity.  The above-given Fornaro and Aero guesses of the dimensions of the Brady et.al cavity give very different mode shapes (all other parameters being the same).

2) Before these calculations the consensus was that Aero's latest estimates of the geometry were superior (for a number of reasons).  These calculations give further confirmation that Brady et.al. actual geometry may be closer to Aero's estimates:

   2a) In all cases examined above, the errors are smaller using the Aero estimate of geometry.

   2b) Aero's estimate of geometry (predicting a smaller Brady et.al. cavity than Fornaro) lead to more stable values  of mode shape with frequency: Aero's estimate gives the same mode shape (TE310 for both Brady "A" and Brady"B", which differ very little in frequency).  Fornaro's geometry gives different mode shapes for Brady "A" and "B".

    2c) Aero's estimate of geometry gives more discrimination between mode shapes for  Brady cases "A" and "B": the mode shape (TE310) with the smallest error has an error off less than 1% while the next closest mode shapes have errors exceeding 4%.  Fornaro's geometry has errors much closer together which do not provide as much power to discriminate between the actual mode shape.

3) It is interesting that I obtained for Brady case "C" (NASA's experiment which gave the largest by far thrust/PowerInput) mode TE01 as the second best mode shape, with an error of less than 1%: just 0.971%.  This coincides with NASA's Brady et.al. mode shape first two quantum numbers (circumferential and radial).  I propose that TE01 may indeed be the actual mode shape for Brady "C" (the difference in errors between TE01 and TE21 is insignificant) because of the effect of the dielectric polymer in the cavity (which we do not take into account).  The dielectric polymer is a circumferential doughnut-shape polymer that must have a TE01 mode shape - obviously (see this picture) 
(http://forum.nasaspaceflight.com/index.php?action=dlattach;topic=36313.0;attach=631726;image).  This dielectric polymer must force the cavity into the TE01 mode shape. As to why I calculate TE011 while NASA Brady et.al give TE012 that also can be readily explained by the polymer dielectric as the dielectric will produce an extra longitudinal full wave in the dielectric polymer, resulting in two full waves in the longitudinal direction of the whole cavity: 1) one full wave within the doughnut-shaped polymer dielectric itself, along the thickness of the doughnut and 2) the other full wave within the longitudinal direction of the rest of the empty cavity that has no dielectric polymer.

4) The Aero geometry estimate of the Brady et.al. cavity (that appear to be the best geometrical estimate) gives TE (transverse electric) mode shapes as the best mode shape estimates for all frequencies tested by NASA Brady et.al.  Actually, the TE01 mode is a close mode shape for all the Brady experiments.  It just happens that the frequency that best excites TE01 happens to be Brady C, that provided the highest thrust/PowerInput.  This is critical, because TE01 is also the mode excited by the dielectric polymer.  This is important also if indeed the thrust force is mainly a result of the internal, centrally located magnetic field (contained within the Transverse Electric (TE) circumferential field) being responsible for the experimental results (as previously argued either as thermal buckling artifact or whether as a result of the central longitudinal magnetic field interacting with the Quantum Vacuum providing quantum vacuum radiation pressure, for example).

5) I have to double-check my Mathematica program for any errors and my above transcription for any errors.  Thank you for your patience  :)
Title: Re: EM Drive Developments - related to space flight applications - Thread 2
Post by: JasonAW3 on 01/20/2015 09:18 pm
Quote
At constant input power, the thrust, and therefore the acceleration, must decrease with time, to ensure that the spacecraft's velocity never exceeds 2*Power/Thrust

I was going to start a thread, but there is a photon rocket variant idea I have been batting around for a few months now:

take a long hollow cylinder, closed at one end, open on the other.  Probably several hundred meters long, by three or four meters in diameter.  Running the length of this cylinder, spaced at even intervals are low beams - probably no more than ten or twenty centimeters high.  So the inner edge of the cylinder has...call it a dozen shallow troughs.  At regular intervals - maybe a meter - these beams have specially designed reflective points.  Open end of the cylinder, you have a powerful high frequency laser (or something emitting a focused photon beam) aimed at a 45 degree angle into each trough.  One laser per trough, call it twelve total. 

Now, a laser, like a military searchlight, is also a photon rocket.

Photons, as pointed out in the previous thread are durable little critters, and can bounce around a good 50,000+ times before going wherever it is expired photons go.   And photons can transfer momentum with each bounce.   

So, turn the lasers on.  The initial 'thrust' is backward.   Actually, 'backward and sideways' because of the angle. 

That thrust gets negated at the first bounce point.  Photons hit that (reflective) point, transfer momentum, and head over to the next bounce point, set at a 45 degree angle to the first.

At the second bounce point, the whole thing is moving forward.  Repeat for the length of the cylinder. Because the photons are hitting at an angle, the cylinder might start rotating as well as moving forward, but I don't see that as a major issue.  At the end of the cylinder, the photons hit a shaped surface and bounce back along the tubes center and out into space. 

Did a bit of reading on laser propulsion systems.  A Doctor Bae ran some laboratory tests on this: bouncing laser beams multiplied the 'thrust' by a factor of 3000+ - into EM Drive territory without the physics headache.  He proposed two linked spacecraft, with laser beams between them, something NASA is supposed to be looking into for near earth applications.   My idea is one spacecraft (the cylinder) and a multiplier of about 1500, if the cylinder is long enough. Not sure, but that's should be on a par with the Brady EM drive model.

Alter the angles a bit, test  different lasers/emitters, might get a lot more work out of the photons, increasing thrust further.

Would this violate the paradox?   

You seem to misunderstand the fundamentals of mechanics.

If a photon hits a mirror at a 45 degree angle and reflects off it, the mirror will receive an impulse perpendicular to the plane of the mirror only.  It will not be pushed in the direction of the other component of the photon at all.

It's not just with photons.  If you have a billiard ball and you bounce it off another billiard ball that was stationary so that the original ball end up leaving at a 90 degree angle to its initial direction of travel, the other ball will end up traveling at a 45 degree angle to the path of the original ball.

It's non-intuitive because our intuition is shaped by friction tending to pull things along, but such friction is not a part of purely elastic collisions, and photons bouncing off mirrors are purely elastic.

So, every time your photon bounces off the wall, the momentum it imparts will only be to push outward perpendicular to the axis of the tube.  And it will be cancelled by the next bounce off the opposite wall.

The only effect of the net momentum of the tube is the opposite of whatever momentum the photo has when it finally leaves the tube.  Whatever it does as it bounces around in the tube will have no net effect.

What about a severe conewith a 5 degree opening? Lasers shot up into th mirrorwould glance off at an approximate 7 degerr angle, still proceeding upwards to the conical apex, where they would then be relected directly outwards.  Obviously the photons would loose energy with each bounceand you'd have to cool the conical mirror, as at best, you MIGHT get a 97% reflectivity from it, but could THAT work as a propulsive system?  (BTW the 7 degree is an approximation of the angle.  Your milage may vary).
Title: Re: EM Drive Developments - related to space flight applications - Thread 2
Post by: Rodal on 01/20/2015 11:19 pm
...
For TM modes, X[sub m,n] = the n-th zero of the m-th Bessel function.
[1,1]=3.83, [0,1]=2.40, [0,2]=5.52 [1,2]=7.02, [2,1]=5.14, [2,2]=8.42, [1,3]=10.17, etc.

and for TE modes, X'[subm,n] = the n-th zero of the derivative of the m-th Bessel function.
[0,1]=3.83, [1,1]=1.84, [2,1]=3.05, [0,2]=7.02, [1,2]=5.33, [1,3]=8.54, [0,3]=10.17, [2,2]=6.71, etc.

So, using these to identify the frequencies, I chose:

Bradya => TM122 or TE022  X[sub m,n] = 7.02 p = 2
Bradyb => TE213           X'[sub m,n] = 3.05 p = 3
Bradyc => TE222           X'[sub m,n] = 6.71 p = 2

....

OK, let's use your terminology to check the labeling of the modes.
The lowest modes that have exactly the same numerical values, according to your definition above are:

For TM modes, X[sub m,n] = the n-th zero of the m-th Bessel function.
[1,1]=3.83 Therefore m=1,n=1 and TM11 corresponds to X1,1=3.83

and for TE modes, X'[subm,n] = the n-th zero of the derivative of the m-th Bessel function.
[0,1]=3.83 Therefore m=0,n=1 and TE01 corresponds to X'0,1=3.83

* Actually, in general X1,n = X'0,n for any n *

*
EDIT:

therefore

frequency of TM1np = frequency of TE0np  for given radius and cavity length, for any value of n and any value of p

*

Therefore the modes that have the same numerical value are TM11 and TE01 (and not  TM12 or TE02).

Conclusion: it appears that the radial quantum number "n" in your mode number is higher than the correct value by one.    (For some reason it also appears that your longitudinal "p" is also inflated by one: it should be TM111 instead of TM122 and it should be TE011 instead of TE022).
Title: Re: EM Drive Developments - related to space flight applications - Thread 2
Post by: Notsosureofit on 01/20/2015 11:45 pm
X[sub 1,a] = X'[sub 0,a]

These are the columns that correspond.

http://mathworld.wolfram.com/BesselFunctionZeros.html

TM111 gives me 1.03 GHz  ditto for TE011

I couldn't find a formula for a tapered cavity.

I'm thinking the volumetric radius might be the best choice as radius

R^2 = (a^2+a*b+b^2)/3

Havn't had time to try recalculations, but if you have Mathematica you can try lots
Title: Re: EM Drive Developments - related to space flight applications - Thread 2
Post by: Rodal on 01/21/2015 12:02 am
...
TM111 gives me 1.03 GHz  ditto for TE011


From http://en.wikipedia.org/wiki/Microwave_cavity#Cylindrical_cavity, the frequency equation gives:

frequencyTM111 =( cAir/(2*Pi))*Sqrt[((X1,1 /(aeroGeometricMeanDiameter/2))^2) +(1*Pi/aeroLength)^2]
                           = 1.8643 GHz instead of 1.03 GHz

 ditto for frequencyTE011

using

m=1

n=1

p=1

X1,1 = 3.83170597020751

cair = 299705000 m/s = (299792458 m/s) / (Sqrt[mur*epsilonr] = c / (Sqrt[mur*epsilonr])

aeroGeometricMeanDiameter = 0.207974 m = Sqrt[ (0.27246 m) * (0.15875 m)]

aeroLength = 0.24173 m
Title: Re: EM Drive Developments - related to space flight applications - Thread 2
Post by: Rodal on 01/21/2015 12:26 am
...
TM111 gives me 1.03 GHz  ditto for TE011


If, were one to use the diameter instead of the radius in the frequency equation one would obain:

frequencyTM111 =( cAir/(2*Pi))*Sqrt[((X1,1 /(aeroGeometricMeanDiameter))^2) +(1*Pi/aeroLength)^2]
                           = 1.07 GHz which is very close to your calculated 1.03 GHz

==> Please consider whether your mode shape calculations may be incorrect perhaps because you inadvertently may have used the diameter instead of the radius in the expression for frequency <===

Remember:


Aero Best estimate as of 11/9/2014    http://forum.nasaspaceflight.com/index.php?topic=29276.msg1285896#msg1285896
   
cavityLength = 0.24173 m
bigDiameter = 0.27246 m
smallDiameter = 0.15875 m
Title: Re: EM Drive Developments - related to space flight applications - Thread 2
Post by: Notsosureofit on 01/21/2015 01:50 am
We are using different radii.  I'm thinking of going to the volumetric, might be closest, still looking for an actual f solution.
Title: Re: EM Drive Developments - related to space flight applications - Thread 2
Post by: Rodal on 01/21/2015 01:59 am
...

I'm thinking the volumetric radius might be the best choice as radius

R^2 = (a^2+a*b+b^2)/3

Havn't had time to try recalculations, but if you have Mathematica you can try lots
Are you thinking this way?

Defining these symbols for the radii of the truncated cone (frustum of a cone):

r=smallRadius=smallDiameter/2
R=bigRadius=bigDiameter/2

The volume of a truncated cone and the volume of a cylinder are:

Volume of the frustum of a cone =Height*Pi*(r^2+r*R+R^2)/3
Volume of a cylinder= Pi*(EquivalentR^2)*Height

Equating these volumes one arrives at an expression for the Equivalent Radius of a cylinder having the same volume as the volume of the frustum of a cone:

           Volume of a cylinder = Volume of the frustum of a cone
Pi*(EquivalentR^2)*Height = Height*Pi*(r^2+r*R+R^2)/3

hence

EquivalentCylindricalRadius = Sqrt[(r^2+r*R+R^2)/3]
                    = Sqrt[(smallDiameter^2+smallDiameter*bigDiameter+bigDiameter^2)/12]

See this example: it makes very little difference what mean to use:

aeroLength=0.24173 meter;aeroBigDiameter=0.27246 meter;aeroSmallDiameter=0.15875 meter;


aeroGeometricMeanDiameter=Sqrt[aeroBigDiameter*aeroSmallDiameter]
                                              = 0.207974 meter

aeroMeanDiameter                =(aeroBigDiameter+aeroSmallDiameter)/2
                                              = 0.215605 meter

aeroVolumetricDiameter=Sqrt[(aeroSmallDiameter^2+aeroSmallDiameter*aeroBigDiameter+aeroBigDiameter^2)/3]
                                              = 0.218089 meter

The differences between these different means (just a few % between them) is less than the level of uncertainty we have for the geometry.  The difference between these means is much less than the difference between Aero and Fornaro's estimates of the geometry.  It would be useful to know what dimensions did you use.
Title: Re: EM Drive Developments - related to space flight applications - Thread 2
Post by: Rodal on 01/21/2015 02:02 am
We are using different radii.  ,,,
I used Aero's and also Fornaro's dimensions.  For example,
Aero Best estimate as of 11/9/2014    http://forum.nasaspaceflight.com/index.php?topic=29276.msg1285896#msg1285896
   
cavityLength = 0.24173 m
bigDiameter = 0.27246 m
smallDiameter = 0.15875 m

What dimensions did you use for the tapered cone (frustum) ?

I don't recall you stating the dimensions in this post:  http://forum.nasaspaceflight.com/index.php?topic=36313.msg1317866#msg1317866 .  Sorry if it is in some of your other posts and I missed it.
Title: Re: EM Drive Developments - related to space flight applications - Thread 2
Post by: ThinkerX on 01/21/2015 02:03 am
Ok...so if I am following the latest exchange correctly, then these devices dimensions MUST be in complete sync with the given frequency in order for them to work at all?  Or do they just function very poorly if the match is not exact?

I also recollect long posts in the other thread about how difficult it was to attain and maintain the correct frequency - something about it shifting?

 

Title: Re: EM Drive Developments - related to space flight applications - Thread 2
Post by: Rodal on 01/21/2015 02:14 am
Ok...so if I am following the latest exchange correctly, then these devices dimensions MUST be in complete sync with the given frequency in order for them to work at all?  Or do they just function very poorly if the match is not exact?

I also recollect long posts in the other thread about how difficult it was to attain and maintain the correct frequency - something about it shifting?

 

These are cavity resonators that use the Quality Factor Q to achieve greater response.  So, definitely you have to be at a peak resonant frequency to achieve highest Q.

But what we have been discussing as of late, besides Q and frequency is the mode shape: the shape of the electric and magnetic fields inside the cavity at a given frequency.  There are an infinite number of possible mode shapes inside a cavity (considering all frequencies).  Even in the MHz and GHz frequency ranges there are several natural frequencies and associated mode shapes to consider.

Take a gander at the attached picture for a few mode shapes

The question is what mode shape may be most effective for thrust generation.  This is the emphasis of our discussion: how to achieve the greatest level of thrust of an EM Drive for a given level of  inputpower.
Title: Re: EM Drive Developments - related to space flight applications - Thread 2
Post by: aero on 01/21/2015 06:11 am
On seeing the cavity geometry revisited, I will just mention that my measurements were of necessity of the outside of the cavity. The inside dimensions would be smaller by the amount of copper thickness. As I recall, we estimated the copper cone to be most likely 1/32" copper and the end copper ground plane on the circuit board to be ~ 2 thousandths of an inch.

I did a Google search for circuit boards, and found that they are rated in ounces of metal per square ? something 1, 2 and 3 oz. plating being common. Translated to thickness it came to 30-40 microns, 65-75 microns and just over 100 microns. I dropped that line of investigation an went with the 2/1000 estimate.

As for the cone thickness of 1/32 inch, I think I gave the pixel count for the 1 1/2 inch square beam end which I believe it puts 1/32 inch on the order of 1 pixel. But my measurements of the outer dimension are also at best, on that order so I don't know what kind of wizardry it would take to pull up a better estimate of the inner dimensions of the cone.

There it is, if anyone wants to take a shot at further correction to the big and small diameters.

 Add: Of course one could just subtract the thickness from the radius. And  I note that the small end diameter is the diameter of the dielectric, so it is already the inside diameter.
Title: Re: EM Drive Developments - related to space flight applications - Thread 2
Post by: wembley on 01/21/2015 10:23 am

There is essentially a news blackout on the experimental side from NASA.

The problem is, if this drive really works, the defence implications are huge. Those in charge might feel it would not necessarily be a good idea to go public with it -- especially from the US perspective. Remember that 'Sputnik moment'?
[/quote]

@wembley, do you have any specific info about why you think there is a news blackout?

Also, do you have any further verifiable information from this post: http://forum.nasaspaceflight.com/index.php?topic=29276.msg1265607#msg1265607 stating Shawyer stopped using a dielectric section?
[/quote]

I been in contact with NASA on this and the lack of direct response has been marked. They will talk about the warp drive and other work, but will not even mention the EmDrive in their replies. It's as though they cannot speak its name...I also have it from other sources that NASA have requested that they do not discuss NASA's work in this area. If you can find any official NASA public comment on anything related to their EmDrive work I will be impressed!  It does look like a blackout to me.

The info about Shawyer not using dielectric section was in an email from the man himself.
Title: Re: EM Drive Developments - related to space flight applications - Thread 2
Post by: wembley on 01/21/2015 10:25 am

I have no way of telling if this would work, and it seems to me that the forces involved are rather high for the device to deal with, but potentially it could produce much more thrust for the same input

 "Here, we review the derivation of the equation of thrust of Shawyer’s thruster, by obtaining a new expression, which includes the indexes of refraction of the two parallel plates in the tapered waveguide. This new expression shows that, by strongly increasing the index of refraction of the plate with the largest area, the value of the thrust can be strongly increased. "

http://www.frandeaquino.org/Shawyer%20Thruster.pdf
Title: Re: EM Drive Developments - related to space flight applications - Thread 2
Post by: Star One on 01/21/2015 11:31 am

There is essentially a news blackout on the experimental side from NASA.

The problem is, if this drive really works, the defence implications are huge. Those in charge might feel it would not necessarily be a good idea to go public with it -- especially from the US perspective. Remember that 'Sputnik moment'?

@wembley, do you have any specific info about why you think there is a news blackout?

Also, do you have any further verifiable information from this post: http://forum.nasaspaceflight.com/index.php?topic=29276.msg1265607#msg1265607 stating Shawyer stopped using a dielectric section?
[/quote]

I been in contact with NASA on this and the lack of direct response has been marked. They will talk about the warp drive and other work, but will not even mention the EmDrive in their replies. It's as though they cannot speak its name...I also have it from other sources that NASA have requested that they do not discuss NASA's work in this area. If you can find any official NASA public comment on anything related to their EmDrive work I will be impressed!  It does look like a blackout to me.

The info about Shawyer not using dielectric section was in an email from the man himself.
[/quote]

Interesting to hear.

What happens if another country wheels a drive out, haven't others been working in this area?
Title: Re: EM Drive Developments - related to space flight applications - Thread 2
Post by: Notsosureofit on 01/21/2015 11:59 am
...
TM111 gives me 1.03 GHz  ditto for TE011


If, were one to use the diameter instead of the radius in the frequency equation one would obain:


You are exactly right, I've been (inadvertantly) calculating w/ DIAMETERS !

I've got to stop using Post-it notes  !
Title: Re: EM Drive Developments - related to space flight applications - Thread 2
Post by: Rodal on 01/21/2015 12:43 pm
...
TM111 gives me 1.03 GHz  ditto for TE011


If, were one to use the diameter instead of the radius in the frequency equation one would obain:


You are exactly right, I've been (inadvertantly) calculating w/ DIAMETERS !

I've got to stop using Post-it notes  !

"The man with insight enough to admit his mistakes comes nearest to perfection."
Johann Wolfgang von Goethe

Keep doing what you have been doing, as it is only thanks to you that we have a thrust equation incorporating the mode shape, and thus thanks to you that we are (only now) able to investigate optimum mode shapes and therefore have greater insight into what is going on !  :)

I plan today to run the calculations again using the volumetric mean instead of the geometric mean (I expect little difference, but just to confirm).  Next I plan to calculate mode shapes for the Shawyer experiments (as nobody has ever discussed the mode shapes involved in Shawyer's experiments).
Title: Re: EM Drive Developments - related to space flight applications - Thread 2
Post by: Notsosureofit on 01/21/2015 03:25 pm
Great !  In my 2 minutes at a time I'm off to figure out why df/c = the momentum, but in the accelerated frame of reference.

Added:  http://scitechdaily.com/way-self-propel-subatomic-particles-without-external-forces/

"We find these shape-preserving Dirac wavefunctions to be part of a family of accelerating quantum particles, which includes massive/massless fermions/bosons of any spin."
Title: Re: EM Drive Developments - related to space flight applications - Thread 2
Post by: Rodal on 01/21/2015 05:54 pm
...
Added:  http://scitechdaily.com/way-self-propel-subatomic-particles-without-external-forces/

Interesting work by MIT PostDoc !

Quote
It turns out that this self-acceleration does not actually violate any physical laws — such as the conservation of momentum — because at the same time the particle is accelerating, it is also spreading out spatially in the opposite direction.

“The electron’s wave packet is not just accelerating, it’s also expanding,” Kaminer says, “so there is some part of it that compensates. It’s referred to as the tail of the wave packet, and it will go backward, so the total momentum will be conserved. There is another part of the wave packet that is paying the price for the main part’s acceleration.”

It turns out, according to further analysis, that this self-acceleration produces effects that are associated with relativity theory: It is a variation on the dilation of time and contraction of space, effects predicted by Albert Einstein to take place when objects move close to the speed of light. An example of this is Einstein’s famous twin paradox, in which a twin who travels at high speed in a rocket ages more slowly than another twin who remains on Earth.
Title: Re: EM Drive Developments - related to space flight applications - Thread 2
Post by: DIYFAN on 01/21/2015 06:13 pm
I been in contact with NASA on this and the lack of direct response has been marked. They will talk about the warp drive and other work, but will not even mention the EmDrive in their replies. It's as though they cannot speak its name...I also have it from other sources that NASA have requested that they do not discuss NASA's work in this area. If you can find any official NASA public comment on anything related to their EmDrive work I will be impressed!  It does look like a blackout to me.

If NASA really decides to go silent on this, that would be a strong indication that the effect is not only real, but imminently useful.  For those in the administration that might feel going silent is the optimal approach, I would suggest to them otherwise.  Attempting to impede and to cover such information in the age of the Internet is futile.  Though it may slow the efforts within the U.S. to reproduce and harness the effect, this is a multi-national effort, and the advancement and evolution of our species will continue in its progression despite such efforts.  To any and all leaders within NASA or other governmental organizations whose first reaction is to suppress, please consider that such an approach is very 20th century.
Title: Re: EM Drive Developments - related to space flight applications - Thread 2
Post by: Rodal on 01/21/2015 06:20 pm
1) I have re-run the calculations using the Volumetric Mean instead of the Geometric Mean. Please compare the results shown below using the Volumetric Mean with the GeometricMean results previously shown in post http://forum.nasaspaceflight.com/index.php?topic=36313.msg1319117#msg1319117 .   



2) We define the Volumetric  Mean as follows:

VolumetricMeanDiameter=Sqrt[(aeroSmallDiameter^2+aeroSmallDiameter*aeroBigDiameter+aeroBigDiameter^2)/3]


DERIVATION OF VOLUMETRIC MEAN

Defining the following symbols for the radii of the truncated cone (frustum of a cone):

r=smallRadius=smallDiameter/2
R=bigRadius=bigDiameter/2

The volume of a truncated cone and the volume of a cylinder are:

Volume of the frustum of a cone =Height*Pi*(r^2+r*R+R^2)/3
Volume of a cylinder= Pi*(EquivalentR^2)*Height

Equating these volumes one arrives at an expression for the Equivalent Radius of a cylinder having the same volume as the volume of the frustum of a cone:

           Volume of a cylinder = Volume of the frustum of a cone
Pi*(EquivalentR^2)*Height = Height*Pi*(r^2+r*R+R^2)/3

hence

VolumetricMeanRadius = EquivalentR
                                    = Sqrt[(r^2+r*R+R^2)/3]
                                    = Sqrt[(smallDiameter^2+smallDiameter*bigDiameter+bigDiameter^2)/12]

or,

VolumetricMeanDiameter=Sqrt[(aeroSmallDiameter^2+aeroSmallDiameter*aeroBigDiameter+aeroBigDiameter^2)/3]



3) Let's define as "Aero geometry" the following definition for the NASA Brady et. al. cavity:

Aero Best estimate as of 11/9/2014    http://forum.nasaspaceflight.com/index.php?topic=29276.msg1285896#msg1285896
   
cavityLength = 0.24173 m
bigDiameter = 0.27246 m
smallDiameter = 0.15875 m

then

aeroGeometricMeanDiameter=Sqrt[aeroBigDiameter*aeroSmallDiameter]
                                              = 0.207974 meter

aeroMeanDiameter                =(aeroBigDiameter+aeroSmallDiameter)/2
                                              = 0.215605 meter

aeroVolumetricMeanDiameter=Sqrt[(aeroSmallDiameter^2+aeroSmallDiameter*aeroBigDiameter+aeroBigDiameter^2)/3]
                                              = 0.218089 meter

4) Let's define as "Fornaro geometry" the following definition for the NASA Brady et. al. cavity:

Fornaro estimate    http://forum.nasaspaceflight.com/index.php?topic=36313.msg1302455#msg1302455
   
cavityLength = 0.332 m
bigDiameter = 0.397 m
smallDiameter = 0.244 m

then

fornaroGeometricMeanDiameter=Sqrt[fornaroBigDiameter*fornaroSmallDiameter]
                                                  = 0.311236 meter

fornaroMeanDiameter                =(fornaroBigDiameter+fornaroSmallDiameter)/2
                                                  = 0.3205 meter

fornaroVolumetricMeanDiameter=Sqrt[(fornaroSmallDiameter^2+fornaroSmallDiameter*fornaroBigDiameter+fornaroBigDiameter^2)/3]
                                                  = 0.323529 meter

5) Given the experimentally reported frequencies, the geometrical dimensions and the value of speed of light in air, one can invert the frequency equation (see: http://en.wikipedia.org/wiki/Microwave_cavity#Cylindrical_cavity ) to obtain Xm,n and  X'm values as a function of constants and the longitudinal mode shape number "p". Let's define the error difference between these Xm,n and  X'm,n  values and actual Xm,n and  X'm values as:

error= (value of Xm,n or  X'm,n obtained from frequency eqn.)/ (correct value of Xm,n or  X'm,n ) -1

where Xm,n is used for TM modes and X'm,n is used for TE modes.



6) Then I obtain the following mode shapes and associated errors:


BRADY "A"

Fornaro Geometry

Best result:  TM311  error= - 0.146%
2nd best:     TM014 error= - 3.00%

Aero Geometry

Best result: TE311 error= - 0.427%
2nd best:    TE212 error= + 3.05%

BRADY "B"

Fornaro Geometry

Best result:  TM311 error= + 0.0780%
2nd best:     TM014 error= -  1.389%

Aero Geometry

Best result: TE311 error= - 0.192%
2nd best:    TE310 error= + 5.35%


BRADY "C"

Fornaro Geometry

Best result:  TM310  error= - 0.0781%
2nd best:     TM012 error= + 1.303%

Aero Geometry

Best result:  TE310 error= + 2.292%
2nd best:     TE311  error= - 3.43%
      TE011 or TM111 error= + 5.88%




CONCLUSIONS

1) At the frequencies tested by Brady et.al. , what mode-shape corresponds to a given frequency is very sensitive to the exact geometrical dimensions of the cavity.   The reason for this is that there are many natural frequencies very close to each other, each of these frequencies having different mode shapes.  Therefore, the above-given Fornaro and Aero guesses of the dimensions of the Brady et.al cavity give very different mode shapes for a given frequency.

2) Using the Volumetric Mean, all cases run by Brady et.al, for the Fornaro dimensions correspond to transverse magnetic (TM) mode shapes, all cases run by Brady et.al, for the Aero dimensions correspond to transverse electric (TE) mode shapes:

Fornaro dimensions ----->  Transverse Magnetic mode shapes : the magnetic field is in the circumferential direction
Aero      dimensions ----->  Transverse Electric mode shapes:  the electric field is in the circumferential direction


EDIT:  Actually, using the Aero dimensions, all cases run by Brady et.al, correspond to transverse electric (TE) mode shapes, using either the Geometric Mean or the Volumetric Mean to estimate the equivalent cylindrical diameter.  I think that the TE mode shapes are the one that should provide thrust  because it is only the TE mode shapes that have the magnetic field directed along the longitudinal direction of the EM Drive.  Physically, an axial magnetic field may result in a measured thrust either 1) as an artifact, because the magnetic field can heat the flat ends of the truncated cone and hence produce thermal buckling or 2) as a real means of propulsion, by the magnetic field coupling with the Quantum Vacuum, for example.


3)  Using the Volumetric Mean, the field mode shapes for the Fornaro dimensions are similar to the field for the Aero dimensions: mode 311 for Brady cases "a" and "b"" and mode 310 for Brady case "c".  The difference between them is that for the Fornaro dimensions the circumferential field is magnetic while for the Aero dimensions the circumferential field is electric.  Furthermore, one can state that the field mode shapes in the circumferential cross section for the Fornaro dimensions are similar to the field for the Aero dimensions for ALL Brady cases: they are all 31. The difference between them is

Brady cases "a" and "b''  ---> one half-wave in the longitudinal direction

Mode 311 ---->  3 full-wave patterns around the circumference
                          1 half-wave pattern across the diameter
                          1 half-wave pattern along the longitudinal length
       ______________________________________________

Brady case "c''  ---> constant in the longitudinal direction

Mode 310 ---->  3 full-wave patterns around the circumference
                          1 half-wave pattern across the diameter
                          constant along the longitudinal length


4) Using the volumetric mean in all cases, the errors are smaller using the Fornaro estimate of geometry, while using the geometric mean in all cases, the errors are smaller using the Aero estimate of geometry.  Conclusion: this may be fortuitous and it may not be something to discriminate between the two estimates of geometry.

5) Using the volumetric mean leads to more stable values of mode shape with variation in frequency than when using the geometric mean: it gives mode shape 31 for all Brady cases.   




6) The most important conclusion: this exercise has made me appreciate the true role and value of the dielectric polymer, and why NASA reported it was so important.  It is evident that at these frequencies there are so many mode shapes bunched next to each other that:

 it would be extremely difficult to predict what mode shape one will get with a given geometry at these frequencies, because small variations in geometry lead to large changes in mode shape.  Certainly it would be impossible to predict the mode shape of an empty cavity with the coarse finite element model used by NASA Brady et.al.
(There are no Bessel functions in a finite element model: the solution is approximated with low power piecewise polynomials in each finite element.  The finite element solution is a Galerkin solution "in an integral sense" and not an exact partial differential solution "point to point through the domain".)

Hence the function of the dielectric polymer is to force the cavity to function into a preferred mode shape.
For NASA Brady et.al. they used a doughnut-shaped dielectric polymer to try to force the cavity to operate in TE01 mode shape (constant electric field in the circumferential direction).  The calculations show that Brady experiment"c" was the closest to be such a mode shape while experiments "a" and "c" were much further apart.  Hence experiment "c" resulted in a much larger thrust per PowerInput with the dielectric polymer having a constant electric field in the circumferential direction and the rest of the cavity having a constant electric field in the circumferential direction with a half wave amplitude in the longitudinal direction, resulting in a mode TE012.  The polymer dielectric will produce an extra longitudinal half wave in the dielectric polymer, resulting in two half waves in the longitudinal direction of the whole cavity: 1) one half wave within the doughnut-shaped polymer dielectric itself, along the thickness of the doughnut and 2) the other half wave within the longitudinal direction of the rest of the empty cavity that has no dielectric polymer.

7) So, an optimal design of an EM Drive would proceed as follows: first decide what mode shape provides the largest thrust per power Input.  Then design a dielectric polymer shape that would best force this mode shape.  Then model the geometry of the cavity such that the rest of the cavity is also in the same cross-sectional mode shape (to amplify the resonance) and accurately model at what frequency this occurs.
Title: Re: EM Drive Developments - related to space flight applications - Thread 2
Post by: tchernik on 01/21/2015 07:00 pm
If NASA really decides to go silent on this, that would be a strong indication that the effect is not only real, but imminently useful.  For those in the administration that might feel going silent is the optimal approach, I would suggest to them otherwise.  Attempting to impede and to cover such information in the age of the Internet is futile.  Though it may slow the efforts within the U.S. to reproduce and harness the effect, this is a multi-national effort, and the advancement and evolution of our species will continue in its progression despite such efforts.  To any and all leaders within NASA or other governmental organizations whose first reaction is to suppress, please consider that such an approach is very 20th century.

Thinking about potential historic parallels, they ought to be seriously considering to avoid another Sputnik moment.

It is possible they would want to use it in military projects in the short term (endless autonomy/re-positioning satellites would revolutionize spy sats and anti-sat warfare).

But there is the Chinese interest in it too. If they verified it works, they could be already working in military applications, and probably on a few civil ones. Unlimited range space probes would be a very good publicity stunt for them, for proving their technical superiority.
Title: Re: EM Drive Developments - related to space flight applications - Thread 2
Post by: Star One on 01/21/2015 07:20 pm
If NASA really decides to go silent on this, that would be a strong indication that the effect is not only real, but imminently useful.  For those in the administration that might feel going silent is the optimal approach, I would suggest to them otherwise.  Attempting to impede and to cover such information in the age of the Internet is futile.  Though it may slow the efforts within the U.S. to reproduce and harness the effect, this is a multi-national effort, and the advancement and evolution of our species will continue in its progression despite such efforts.  To any and all leaders within NASA or other governmental organizations whose first reaction is to suppress, please consider that such an approach is very 20th century.

Thinking about potential historic parallels, they ought to be seriously considering to avoid another Sputnik moment.

It is possible they would want to use it in military projects in the short term (endless autonomy/re-positioning satellites would revolutionize spy sats and anti-sat warfare).

But there is the Chinese interest in it too. If they verified it works, they could be already working in military applications, and probably on a few civil ones. Unlimited range space probes would be a very good publicity stunt for them, for proving their technical superiority.

A case perhaps of shutting the stable door after the horse has bolted.
Title: Re: EM Drive Developments - related to space flight applications - Thread 2
Post by: tchernik on 01/21/2015 07:50 pm
Would you say it's a case of shutting the stable door after the horse has bolted?

It probably is, but I don't think it would be an attempt at censorship by making all information about this a secret. That horse already left the stable, as you say.

In all probability (if they already know this exists and works outside of the public peer review process) they are trying to prevent the spread of sensitive "next step" information, allowing potential enemy powers to quickly develop the same technology, just by reading the same public reports we on the Internet do.

It's not the first time I have read people pleading for keeping the reports about Q-thruster technology a secret, because in case of being true, they would certainly be matters of national security.
Title: Re: EM Drive Developments - related to space flight applications - Thread 2
Post by: Star One on 01/21/2015 08:21 pm
Would you say it's a case of shutting the stable door after the horse has bolted?

It probably is, but I don't think it would be an attempt at censorship by making all information about this a secret. That horse already left the stable, as you say.

In all probability (if they already know this exists and works outside of the public peer review process) they are trying to prevent the spread of sensitive "next step" information, allowing potential enemy powers to quickly develop the same technology, just by reading the same public reports we on the Internet do.

It's not the first time I have read people pleading for keeping the reports about Q-thruster technology a secret, because in case of being true, they would certainly be matters of national security.
So does that mean we shouldn't except to hear anything more from the US on this & if that's the case what does that mean for civil applications?
Title: Re: EM Drive Developments - related to space flight applications - Thread 2
Post by: Mulletron on 01/21/2015 08:22 pm
Great !  In my 2 minutes at a time I'm off to figure out why df/c = the momentum, but in the accelerated frame of reference.

Added:  http://scitechdaily.com/way-self-propel-subatomic-particles-without-external-forces/

"We find these shape-preserving Dirac wavefunctions to be part of a family of accelerating quantum particles, which includes massive/massless fermions/bosons of any spin."

Yeah that article on Physorg http://phys.org/news/2015-01-analysis-self-propel-subatomic-particles.html made me do this (see below), because it is relevant to what we're exploring here and what a surprising result! Also Marin Soljačić is the professor cited in the evanescent wave explorations from thread 1.

This thread is exploding! once again thanks especially to @Notsosureofit and @Rodal. I really think a milestone happened here over the last couple of days. I wish I could bring the bacon like those two have but I'm kinda stuck in a rutt on figuring out this QV deal.

Quote
It turns out that this self-acceleration does not actually violate any physical laws — such as the conservation of momentum — because at the same time the particle is accelerating, it is also spreading out spatially in the opposite direction.

“The electron’s wave packet is not just accelerating, it’s also expanding,” Kaminer says, “so there is some part of it that compensates. It’s referred to as the tail of the wave packet, and it will go backward, so the total momentum will be conserved. There is another part of the wave packet that is paying the price for the main part’s acceleration.”

It turns out, according to further analysis, that this self-acceleration produces effects that are associated with relativity theory: It is a variation on the dilation of time and contraction of space, effects predicted by Albert Einstein to take place when objects move close to the speed of light. An example of this is Einstein’s famous twin paradox, in which a twin who travels at high speed in a rocket ages more slowly than another twin who remains on Earth.

The above quote (it popped out to me too) and what it means, could actually drop the QV model by changing the way we think about how to conserve momentum. It also sounds eerily like dispersion too.

(https://artgalleryfor.us/music/wp-content/uploads/2014/06/throw-papers-meme-150x150.jpg)

Break: I've seen anecdotal evidence that using the math for a resonant cylinder is "close enough" for a conical frustum. I can find references again if you wish where I found this. But I'm not so sure if this is a good approximation. Anyone know anything about cylindrical coordinate systems? I just started trying to figure this out.

zen-in? http://forum.nasaspaceflight.com/index.php?topic=29276.msg1275592#msg1275592
Title: Re: EM Drive Developments - related to space flight applications - Thread 2
Post by: tchernik on 01/21/2015 08:51 pm
So does that mean we shouldn't except to hear anything more from the US on this & if that's the case what does that mean for civil applications?

I don't believe they can prevent any public disclosure of undeniably working prototypes by the Chinese, in a few years (or months). So they probably are just buying some time for having a working version on our side by then.

That or someone on NASA became aware of the potential huge embarrassment this represents, and told all the guys off and ordered them to pretend it never happened.

But as H. White et al seem to be still working at NASA, I somewhat doubt it's a case of disowning a blunder. Or at least, I hope so.
Title: Re: EM Drive Developments - related to space flight applications - Thread 2
Post by: Stormbringer on 01/21/2015 08:55 pm
if you talking about a electron's self energy...

isn't it infinite? So science has sort of looked the other way and declared it a SEP (Hitchhiker's Guide reference meaning someone else's problem) while they got on with the bits of the math that doesn't make them uncomfortable?

http://en.wikipedia.org/wiki/Somebody_Else%27s_Problem

Speaking of physorg... you know there is doozy of an article in there today... :D (the galaxy may be a wormhole)

Title: Re: EM Drive Developments - related to space flight applications - Thread 2
Post by: Mulletron on 01/21/2015 08:59 pm
Quote from: wembley
There is essentially a news blackout on the experimental side from NASA.

The problem is, if this drive really works, the defence implications are huge. Those in charge might feel it would not necessarily be a good idea to go public with it -- especially from the US perspective. Remember that 'Sputnik moment'?
Quote from: Mulletron
@wembley, do you have any specific info about why you think there is a news blackout?

Also, do you have any further verifiable information from this post: http://forum.nasaspaceflight.com/index.php?topic=29276.msg1265607#msg1265607 stating Shawyer stopped using a dielectric section?
Quote from: wembley
I been in contact with NASA on this and the lack of direct response has been marked. They will talk about the warp drive and other work, but will not even mention the EmDrive in their replies. It's as though they cannot speak its name...I also have it from other sources that NASA have requested that they do not discuss NASA's work in this area. If you can find any official NASA public comment on anything related to their EmDrive work I will be impressed!  It does look like a blackout to me.

The info about Shawyer not using dielectric section was in an email from the man himself.

Interesting to hear.

What happens if another country wheels a drive out, haven't others been working in this area?
It means an arms race, with the benefit being new tech making its way into the private sector eventually, as it usually does. I think our academic institutions and Nasa are being careful, either confirming a breakthrough or squashing a blunder.

....fixing all those quotes was a mess, did I screw them up?
Title: Re: EM Drive Developments - related to space flight applications - Thread 2
Post by: Star One on 01/21/2015 09:11 pm
So does that mean we shouldn't except to hear anything more from the US on this & if that's the case what does that mean for civil applications?

I don't believe they can prevent any public disclosure of undeniably working prototypes by the Chinese, in a few years (or months). So they probably are just buying some time for having a working version on our side by then.

That or someone on NASA became aware of the potential huge embarrassment this represents, and told all the guys off and ordered them to pretend it never happened.

But as H. White et al seem to be still working at NASA, I somewhat doubt it's a case of disowning a blunder. Or at least, I hope so.
It could be that the decision, assuming it does work, on what to say and what not to say has gone right the way up the chain of command literally to the very top and in the meantime the blackout is put in place.

This might sound odd but perhaps it's for the best that for now this stays outside of the mainstream of public awareness.
Title: Re: EM Drive Developments - related to space flight applications - Thread 2
Post by: Mulletron on 01/21/2015 09:28 pm
So does that mean we shouldn't except to hear anything more from the US on this & if that's the case what does that mean for civil applications?

I don't believe they can prevent any public disclosure of undeniably working prototypes by the Chinese, in a few years (or months). So they probably are just buying some time for having a working version on our side by then.

That or someone on NASA became aware of the potential huge embarrassment this represents, and told all the guys off and ordered them to pretend it never happened.

But as H. White et al seem to be still working at NASA, I somewhat doubt it's a case of disowning a blunder. Or at least, I hope so.
Yep you nailed it....... This is TOO public so the truth will have to come and come soon. This isn't your typical flash in the pan news story. The potential worth of this kind of discovery is priceless, on the order of Newton and Einstein. Or just a big series of unfortunate mistakes.
Title: Re: EM Drive Developments - related to space flight applications - Thread 2
Post by: Stormbringer on 01/21/2015 09:38 pm
So does that mean we shouldn't except to hear anything more from the US on this & if that's the case what does that mean for civil applications?

I don't believe they can prevent any public disclosure of undeniably working prototypes by the Chinese, in a few years (or months). So they probably are just buying some time for having a working version on our side by then.

That or someone on NASA became aware of the potential huge embarrassment this represents, and told all the guys off and ordered them to pretend it never happened.

But as H. White et al seem to be still working at NASA, I somewhat doubt it's a case of disowning a blunder. Or at least, I hope so.
It could be that the decision, assuming it does work, on what to say and what not to say has gone right the way up the chain of command literally to the very top and in the meantime the blackout is put in place.

This might sound odd but perhaps it's for the best that for now this stays outside of the mainstream of public awareness.
only if this thing can go hyper velocity or relativistic velocity in a flight profile that makes it useful as a weapon of mass destruction. so far the most optimistic projection by Dr White would make it have to fly for over twelve days and then turn around and come back to make it into a WMD. that's not really a realistic flight profile for a sneak attack weapon.
Title: Re: EM Drive Developments - related to space flight applications - Thread 2
Post by: DIYFAN on 01/21/2015 09:38 pm
It could be that the decision, assuming it does work, on what to say and what not to say has gone right the way up the chain of command literally to the very top and in the meantime the blackout is put in place.

This might sound odd but perhaps it's for the best that for now this stays outside of the mainstream of public awareness.

Assuming there really is a blackout from NASA (not convinced of that yet), your scenario seems plausible.  The problem is, opaque governments are becoming a thing of the past.  The Internet, and its communities, are pressuring governments to be more open and transparent.  Secrecy, even in the name of national security, can sometimes backfire, particularly when certain technologies have the capability of lifting people of the world out of poverty.  In other words, withholding technological progress in the civilian space can result in a moral failure.
Title: Re: EM Drive Developments - related to space flight applications - Thread 2
Post by: Asteroza on 01/21/2015 10:23 pm
So does that mean we shouldn't except to hear anything more from the US on this & if that's the case what does that mean for civil applications?

I don't believe they can prevent any public disclosure of undeniably working prototypes by the Chinese, in a few years (or months). So they probably are just buying some time for having a working version on our side by then.

That or someone on NASA became aware of the potential huge embarrassment this represents, and told all the guys off and ordered them to pretend it never happened.

But as H. White et al seem to be still working at NASA, I somewhat doubt it's a case of disowning a blunder. Or at least, I hope so.
It could be that the decision, assuming it does work, on what to say and what not to say has gone right the way up the chain of command literally to the very top and in the meantime the blackout is put in place.

This might sound odd but perhaps it's for the best that for now this stays outside of the mainstream of public awareness.
only if this thing can go hyper velocity or relativistic velocity in a flight profile that makes it useful as a weapon of mass destruction. so far the most optimistic projection by Dr White would make it have to fly for over twelve days and then turn around and come back to make it into a WMD. that's not really a realistic flight profile for a sneak attack weapon.

Well, if it works, then parking a second strike kinetic weapon package at earth-sun L4/L5 is feasible (hell, earth-moon L2 would make the package immune from terrestrial laser attack). This IS the US military we're talking about, who dreamed up Project Horizon to bomb the earth from a moonbase, and that was with a 3 day lag using chemical propulsion. Added fun is using an expendable electrodynamic tether as a power source as you approach earth, since you only need high relative speed at impact (drop into a counterorbit).

Actually, I wonder how applicable that would be, using an ED tether as a power source for orbital capture ops, as a possible alternative to aerocapture...
Title: Re: EM Drive Developments - related to space flight applications - Thread 2
Post by: tchernik on 01/21/2015 11:41 pm

Well, if it works, then parking a second strike kinetic weapon package at earth-sun L4/L5 is feasible (hell, earth-moon L2 would make the package immune from terrestrial laser attack). This IS the US military we're talking about, who dreamed up Project Horizon to bomb the earth from a moonbase, and that was with a 3 day lag using chemical propulsion. Added fun is using an expendable electrodynamic tether as a power source as you approach earth, since you only need high relative speed at impact (drop into a counterorbit).

Actually, I wonder how applicable that would be, using an ED tether as a power source for orbital capture ops, as a possible alternative to aerocapture...

You know, despite the well meaning attempts to place this within the known laws of physics, and telling us this works and respects conservation of momentum (and energy), I'm still not convinced how that could be. Rockets respect conservation of momentum and energy by losing mass (and thus energy) in greater amounts than they gain by expelling that mass. These things don't have any clear way to remember how much energy they have spent, and what relative maximum velocity they can reach in order to not be "overunity". No wonder they face very strong skepticism.

But leaving that part out. Even assuming they are limited in speed, they still seem capable of gathering a fair bit of kinetic energy (tens of kms per second as per a previous post).

Thus the geopolitical impact of this technology being real is evident if you think about it a little.

In the short term, endless autonomy in terms of thrust and re-positioning would completely change the rules of engagement for spy satellites and anti-satellite weapons. You can have limitless observation capabilities and limitless loitering, following, attacking (and dodging) capabilities too. War bots in orbit could finally fulfill Ronald Reagan's dreams for war in space. And not just that.

Even very slight accelerations (as reported by Brady, White et al) with preposterously limited top speeds, they can serve for pushing and placing dormant kinetic bombs far away from Earth, beyond the reach of any feasible enemy's detection or attack; and they would also serve for bringing them back to Earth at their maker's command, for landing at any desired spot, producing nearly any desired amount of damage.

While all of us sit here on Earth, sharing the same biosphere and the same rock under our feet, the development and availability of that weaponry would result in an aggravated and somewhat madder version of MAD. Because even if you could be attacked in secret from any direction, you can very well threaten to respond by attacking all your potential enemies with nuclear or kinetic nastiness, ruining the game and the fun for all your potential attackers.

But if you have parties/adversaries living in space, could you threaten to crack all the planets in the Solar System?, or all the artificial habitats floating in the space between?

Removing the speed limitations (thus conservation of energy) only makes things worse, making it feasible to have interstellar adversaries sending relativistic missiles to you. Those have no way to be stopped and even less, any way to provide adequate or proportionate response (you and all around you sharing the same planet would simply be dead).
Title: Re: EM Drive Developments - related to space flight applications - Thread 2
Post by: DIYFAN on 01/22/2015 03:53 am
Thus the geopolitical impact of this technology being real is evident if you think about it a little.

In the short term, endless autonomy in terms of thrust and re-positioning would completely change the rules of engagement for spy satellites and anti-satellite weapons. You can have limitless observation capabilities and limitless loitering, following, attacking (and dodging) capabilities too. War bots in orbit could finally fulfill Ronald Reagan's dreams for war in space. And not just that.

Even very slight accelerations (as reported by Brady, White et al) with preposterously limited top speeds, they can serve for pushing and placing dormant kinetic bombs far away from Earth, beyond the reach of any feasible enemy's detection or attack; and they would also serve for bringing them back to Earth at their maker's command, for landing at any desired spot, producing nearly any desired amount of damage.

While all of us sit here on Earth, sharing the same biosphere and the same rock under our feet, the development and availability of that weaponry would result in an aggravated and somewhat madder version of MAD. Because even if you could be attacked in secret from any direction, you can very well threaten to respond by attacking all your potential enemies with nuclear or kinetic nastiness, ruining the game and the fun for all your potential attackers.

But if you have parties/adversaries living in space, could you threaten to crack all the planets in the Solar System?, or all the artificial habitats floating in the space between?

Removing the speed limitations (thus conservation of energy) only makes things worse, making it feasible to have interstellar adversaries sending relativistic missiles to you. Those have no way to be stopped and even less, any way to provide adequate or proportionate response (you and all around you sharing the same planet would simply be dead).

I agree that the geopolitical impact of this technology being real is evident.  Consider, however, that the geopolitical impact might be far more positive than supposed.  If asteroids can be mined economically using this technology, can you imagine the impact?  Mining on Earth could therefore be reduced, thereby reducing negative impacts on Earth's ecosystem. 

If sufficient lift can be achieved, freights and shipping become far more economical such that providing goods to the poorest of the world to meet the basic necessities of life all of the sudden becomes quite feasible. 

Wars and poverty often go hand-in-hand.  If you can uplift and help a society become more self-sufficient and civilized, the trajectory toward war diminishes.  Even if this technology (assuming it works) is used for defense or purposes of war, the mutually assured destruction principal would apply equally well here as it would with other dual purpose technologies.   
Title: Re: EM Drive Developments - related to space flight applications - Thread 2
Post by: wembley on 01/22/2015 11:48 am
So does that mean we shouldn't except to hear anything more from the US on this & if that's the case what does that mean for civil applications?

I don't believe they can prevent any public disclosure of undeniably working prototypes by the Chinese, in a few years (or months). So they probably are just buying some time for having a working version on our side by then.



The Chinese will not go public. They have had a fairly tight clampdown for a couple of years now, with the only evidence being odd scientific papers.  They don't want anyone else getting a lead on them either.
Title: Re: EM Drive Developments - related to space flight applications - Thread 2
Post by: Rodal on 01/22/2015 12:30 pm
...Rockets respect conservation of momentum and energy by losing mass (and thus energy) in greater amounts than they gain by expelling that mass. These things don't have any clear way to remember how much energy they have spent, and what relative maximum velocity they can reach in order to not be "overunity"....

Just like one needs to take into account the energy/momentum of ejected mass from a classical rocket (or injected into and ejected from a turbojet), also for an EM Drive the energy and momentum of the total open system has to be taken into account to reach any conclusions on velocity limitations:

*for Dr.McCulloch's EM Drive theory, the energy/momentum of the radiated Unruh radiation has to be taken into account.  According to Dr. McCulloch's theory, there is Unruh radiation being emitted from the EM Drive, that Unruh radiation has energy/momentum, and it is responsible for conservation of momentum. 

*for EM Drive theories advocating interaction with the Quantum Vacuum (Dr. White's or the theories considered by @Mulletron) the energy/momentum of the involved Quantum Vacuum has to be taken into account.  An example would be given by Dr.White's analogy of the propeller in a submarine: the energy/momentum of the water propelled by the propeller has to be taken into account.

*ditto for EM Drive theories advocating coupling between electromagnetism and gravitational forces, or theories advocating coupling with Dark Matter, or any other theories explaining conservation of momentum of the EM Drive: the energy/momentum of the coupled field also needs to be taken into account

Thus my conclusion is that Joosten and White's "paradox" analysis for the EM Drive (Appendix A of  "Human Outer Solar System Exploration via Q-Thruster Technology" by B. Kent Joosten and Harold G. “Sonny” White, http://ntrs.nasa.gov/archive/nasa/casi.ntrs.nasa.gov/20140013174.pdf ) is incorrect/incomplete because they failed to take into account the energy/momentum of the involved Quantum Vacuum (which they advocate as the reason for conservation of momentum yet they fail to take into account when they perform the kinetic energy constraint).  Actually Dr. White in the same article shows how a classical ion rocket suffers the same "paradox" if one fails to take into account the energy/momentum of the ejected propellant.

No, the EM Drives do NOT need to remember how much energy they have spent, and what relative maximum velocity they can reach in order to not be "overunity", that "memory" would be a requirement only if one disregards conservation of momentum, which none of us is prepared to do, because it violates a basic law of physics. 

If the EM drive conserves momentum, for example, by interacting with the QuantumVacuum, we have an open system where momentum will be flowing in or out of the cavity, and therefore the open system should be taken into account in the conservation of energy in order to arrive at a satisfactory answer concerning any velocity limitation.

(http://history.nasa.gov/SP-4404/p275.jpg)
(http://upload.wikimedia.org/wikipedia/commons/6/66/Cavitating-prop.jpg)
Title: Re: EM Drive Developments - related to space flight applications - Thread 2
Post by: Star One on 01/22/2015 01:41 pm
So does that mean we shouldn't except to hear anything more from the US on this & if that's the case what does that mean for civil applications?

I don't believe they can prevent any public disclosure of undeniably working prototypes by the Chinese, in a few years (or months). So they probably are just buying some time for having a working version on our side by then.



The Chinese will not go public. They have had a fairly tight clampdown for a couple of years now, with the only evidence being odd scientific papers.  They don't want anyone else getting a lead on them either.

Sounding like a game of Texas Hold' em between them and the U.S. Wonder if there are any other players of substance in this game?
Title: Re: EM Drive Developments - related to space flight applications - Thread 2
Post by: Cinder on 01/22/2015 04:54 pm
Really hoping that discussing such subjective political prospects and other implications of an as yet very theoretical space propulsion technology, is not going to get this thread locked for good..
Title: Re: EM Drive Developments - related to space flight applications - Thread 2
Post by: Rodal on 01/22/2015 06:22 pm
Really hoping that discussing such subjective political prospects and other implications of an as yet very theoretical space propulsion technology, is not going to get this thread locked for good..
1) As explained here http://forum.nasaspaceflight.com/index.php?topic=29276.msg1301657#msg1301657, the previous long thread (which due to its extreme length would have been eventually locked to start a new thread anyway) was locked because of personal attacks and "stupid" and "pointless" posts "that did not feel like this site's subject matter." Specifically, posts (sometimes involving ad hominem attacks) dealing with Mach-Effect Piezoelectric experiments and theory (which already had dedicated threads to deal specifically with that topic) instead of the thread's subject: (Microwave ) EM Drive developments.

2) Political ramifications of space flight applications (for example NASA's rocket developments, SpaceX developments, or Advanced Concepts like Vasimir development or Fusion Power technology) are routinely discussed in many threads of NASA's SpaceFlight Forum, why shouldn't political ramifications of (Microwave) EM Drive space flight applications be discussed as well? (as long of course as they do not contain ad hominem attacks).

3) This EM Drive technology has now been repeatedly tested (experimentally ) at NASA in the USA, in the UK and in China.  The experimental results of these measurements reportedly produce thrust/PowerInput that are within practical SpaceFlight Applications.  Such spaceflight applications have been discussed in several reports by NASA and in this thread.  So, assuming that the experimental results at NASA, UK and China are not all artifacts, the subject being discussed here is not just a theory (unlike some threads in the "Advanced Concepts" section of the NASA SpaceFlight Forum that discuss topics that are much further away from SpaceFlight applications).
Title: Re: EM Drive Developments - related to space flight applications - Thread 2
Post by: Star One on 01/22/2015 06:51 pm
@Rodal tested in the UK, never knew that?

Who was carrying out those experiments?
Title: Re: EM Drive Developments - related to space flight applications - Thread 2
Post by: Stormbringer on 01/22/2015 07:00 pm
@Rodal tested in the UK, never knew that?

Who was carrying out those experiments?
Isn't Shawyer a UK native?
Title: Re: EM Drive Developments - related to space flight applications - Thread 2
Post by: Rodal on 01/22/2015 07:24 pm
@Rodal tested in the UK, never knew that?

Who was carrying out those experiments?

Satellite Propulsion Research Ltd (SPR Ltd), a UK based company, with partial funding from the UK government for some of their tests.  See http://www.emdrive.com/
Title: Re: EM Drive Developments - related to space flight applications - Thread 2
Post by: Cinder on 01/22/2015 08:05 pm
why shouldn't political ramifications of (Microwave) EM Drive space flight applications be discussed as well?
I don't mind them myself.  But I got the possibly mistaken impression (e.g. people actually asking for this to be locked even after thread v.2) that they put the discussion thread too close to or beyond this website's tolerances.  My bad.

Seeing such a thorough crunching thru the uncertainties of such an advanced and frankly just plain cool potential technology, IMHO is too excellent to lose.  Even if EM Drives all turn out to be nothing, the process of making sense of it as detailed in these two threads is exemplary.
Title: Re: EM Drive Developments - related to space flight applications - Thread 2
Post by: Rodal on 01/22/2015 08:09 pm
why shouldn't political ramifications of (Microwave) EM Drive space flight applications be discussed as well?
I don't mind them myself.  But I got the possibly mistaken impression (e.g. people actually asking for this to be locked even after thread v.2) that they put the discussion thread too close to or beyond this website's tolerances.  My bad.

Seeing such a thorough crunching thru the uncertainties of such an advanced and frankly just plain cool potential technology, IMHO is too excellent to lose.  Even if EM Drives all turn out to be nothing, the process of making sense of it as detailed in these two threads is exemplary.
Happy to hear that you share our excitement of the potential of propellant-less space-propulsion technology  :)
Title: Re: EM Drive Developments - related to space flight applications - Thread 2
Post by: Cinder on 01/22/2015 10:13 pm
The way you guys systematically work every scenario to its end, leaving no stone unturned is really awesome.  Ok, I'll let signal/noise go back to normal now.
Title: Re: EM Drive Developments - related to space flight applications - Thread 2
Post by: aero on 01/23/2015 12:05 am
@Notsosureofit
This is a little information regarding the effect of the dielectric on resonance frequency in a cylindrical cavity.
Using the Wikipedia equations for resonance of a cylindrical cavity, J'1(1) = 1.8412, and L = c/2.45E9/s, with p = 1, I calculated frequency to be 2.45 GHz when R = 0.0377449 m. I think this is the TE1,1,1 mode.

I then modelled this cylindrical cavity in meep, the FDTD software, with a drive frequency of 2.45E9 Hz, electrical. On running the software with the cavity model empty (containing vacuum, no dielectric), Harminv calculated resonance to be 1.85058E+009 GHz. Q-factor was 484476.647518797. I don't know why frequency is not 2.45 GHz but it isn't.

I then inserted a PE dielectric with a constant of 1.76, length of 20% of the cavity length, radius = cavity radius. The dielectric was against one end of the cavity. I ran this in meep and Harminv calculated a resonance frequency of 1.70036E+009 and 3.67196E+009 Hz. Q-factors of 342650.708941864 and 280142.928092836.

Pardon all of the digits. Result is,
Resonate frequency = 1850. MHz, Q = 4.8 E+05 with no dielectric
Resonate frequency = 1700. MHz, Q = 3.4 E+05 with 20 % cavity filled with dielectric, constant 1.76.
And another resonant mode at 3671 MHz, Q = 2.8 E+05.

A corresponding second mode may have existed for the empty cavity. I don't know as I had to extend the frequency band search to find the 1.7 GHz frequency which extended the band upward as well as downward.

In any case, these numbers are one way to look at the effect of the dielectric within the cavity. I did make an effort to change the cavity parameters in order to hold the resonant frequency constant. That's to hard.

I would be curious to know if anyone can explain why the difference between the Wikipedia formula calculation of resonant frequency and the resonant frequency obtained by integrating Maxwell's equations in the time domain then doing Fourier analysis to calculate the resonant frequencies. In a nutshell, meep excites the cavity with Gaussian noise across the search band, waits for the noise to die down, then extracts the 100 strongest frequencies, dropping those for which Q is less than 50. It reports those that remain.
Title: Re: EM Drive Developments - related to space flight applications - Thread 2
Post by: Rodal on 01/23/2015 12:28 pm
....
I would be curious to know if anyone can explain why the difference between the Wikipedia formula calculation of resonant frequency and the resonant frequency obtained by integrating Maxwell's equations in the time domain then doing Fourier analysis to calculate the resonant frequencies. In a nutshell, meep ...
Meep (an acronym for MIT Electromagnetic Equation Propagation) solves the Maxwell partial differential equations with the old Finite Difference numerical technique (developed decades before the Finite Element Method or the Boundary Element Method and other numerical techniques), albeit with an interesting implementation (and it is free, of course).

Before embarking on a solution of a more complicated geometry (truncated cone) and materials (dielectric inside the cavity) it is always advisable to compare your numerical solution for a problem having an exact solution to see whether your finite difference spatial and time discretization have converged, to examine possible ill-conditioning of the matrices and to make sure that there are no human errors of input or bugs or theoretical problems with the software.  Any numerical solution is always inferior to an exact solution, the only point of a numerical solution is to solve problems for which an exact solution is not possible.  Thus you are doing the right thing by testing the solution vs the exact solution for a cylindrical empty (no dielectric) cavity.  Your present numerical results are so far from the exact solution for an empty cylindrical cavity that it is not meaningful yet to discuss your solution for a dielectric included until you can show good convergence between your numerical result towards the exact solution for the empty cylindrical cavity  (of course, besides discretization convergence problems there is always the possibility that you have made a human error somewhere or that the software has a bug or a theoretical problem). 

The starting point of any finite difference solution is the discretization of space and time into a grid. Meep uses the standard Yee grid discretization (see  http://ab-initio.mit.edu/wiki/index.php/Yee_grid ) (http://ab-initio.mit.edu/wiki/images/thumb/6/6f/Yee-cube.png/250px-Yee-cube.png) which staggers the electric and magnetic fields in time and in space, with each field component sampled at different spatial locations, allowing the time and space derivatives to be formulated as center-difference approximations.  Meep further divides the grid into chunks that are joined together into an arbitrary topology via boundary conditions.

To examine the convergence of your solution you should do a convergence study: run different cases with finer spatial grids and smaller time increments (output the frequencies for each spatial and time discretization).  The time discretization is very important as it has been known for a long time that the central finite difference time discretization has a stability problem  (the time increment needs to be small enough for a good result).  This is assuming that one uses the time stepping technique (that I would start with).  For a frequency-domain solver you will have to examine convergence of the frequency-domain solver.  Also, I assume of course that you are using at least double precision. Examining the convergence should give you an idea of how much finer spatial and time discretization you need in order to arrive to results that match the exact solution.  Only once you have been able to accurately match your numerical discretization output with the exact solution for the empty cylindrical cavity you should pursue more complicated geometries containing dielectric materials, problems for which there is no closed-form solution.

Godspeed and carry on  :)


NOTE: The frequency-domain solver assumes a time dependence of e^(−iωt) for all currents and fields, and solves the resulting linear equations for the steady-state response or eigenmodes. Thus if you eventually want to solve problems with nonlinear dielectric materials or active systems, be forewarned that the frequency domain solver is inadequate for nonlinear materials and for active systems in which frequency is not conserved. 

To obtain the frequency-domain response of a cavity with multiple long-lived resonant modes, in the time domain, is very challenging for numerical techniques, like Meep.  These modes require a long simulation to reach steady state, whereas in the frequency domain the resonances correspond to poles that increase the condition number ( http://en.wikipedia.org/wiki/Condition_number ) and hence slow convergence due to ill-conditioning of the matrix.   (R. Barrett, M. Berry, T. Chan, J. Demmel, J. Donato, J. Dongarra, V. Eijkhout, R. Pozo, C. Romine, H. V. der Vorst, Templates for the Solution of Linear Systems: Building Blocks for Iterative Methods, SIAM, Philadelphia, PA, 1994,   http://citeseerx.ist.psu.edu/viewdoc/download?doi=10.1.1.83.867&rep=rep1&type=pdf )
Title: Re: EM Drive Developments - related to space flight applications - Thread 2
Post by: RanulfC on 01/23/2015 02:58 pm
Lightly following this thread but you folks are veering into territory I DO actually understand and I felt the need to correct some misconceptions.

Why is there the "assumption" that no matter how much (or little) an operational EM-Drive would generate it would be of military/geo-political value beyond, say, extending the service life of spy, communications, etc satellites? That it provides for greatly enhanced (or even practical) kinetic bombardment weapons? That it has some sort of huge military purpose that will ignite an arms race?

Really? No. Only if it has enough thrust to power an aircraft with a reasonable (turbine-generator) amount of power and is more efficent AND more powerful than current jet or rocket engines. Otherwise its simply a low-thrust station keeping and manuever system with a long service life :)

A thrust of 0.09lb to 0.9lb per KW as noted in the cited paper isn't that great really. Again the main advantge is you don't have to carry propellant/reaction mass. The main "geopolitical" ramification of the EM-Drive would be that satellite servicing is going to look a lot LESS attractive since with it you'd (supposedly) never have to fill up maneuvering system every again which was the major driver for that concept. :)

Hidden "Rods-From-Gods" in deep space ready to rain down on anyone, anywhere and "undetectable" due to the EM-Drive? Uh, NO just no.

First of all: There is no "stealth" in space. Period.
http://www.projectrho.com/public_html/rocket/spacewardetect.php

Your rod carrier is going to be spotted and tracked. The EM-Drive requires power, which is going to have heat that is going to have to be rejected, which is going to be "visible" to anyone looking in the right direction. The EM-Drive itself (according to one post above) "emits" radiation which can be detected with the right sensor set up. And lastly you CAN see objects in space if you look carefully enough. Even if you used solar panels to provide the power for the EM-Drive they are going to "reflect" some of the energy they recieve AND they are going to be sources of waste heat for the energy they absorb but do not use.

I wonder if anyone has pointed out that IF this "works" the way it would seem to what they've invented is basically the "Space:1889" Ether Propeller :)

Randy
Title: Re: EM Drive Developments - related to space flight applications - Thread 2
Post by: Rodal on 01/23/2015 03:03 pm
http://en.wikipedia.org/wiki/Space:_1889

(http://upload.wikimedia.org/wikipedia/en/thumb/a/af/Space1889rpg.jpg/250px-Space1889rpg.jpg)
Title: Re: EM Drive Developments - related to space flight applications - Thread 2
Post by: JasonAW3 on 01/23/2015 03:24 pm
Lightly following this thread but you folks are veering into territory I DO actually understand and I felt the need to correct some misconceptions.

Why is there the "assumption" that no matter how much (or little) an operational EM-Drive would generate it would be of military/geo-political value beyond, say, extending the service life of spy, communications, etc satellites? That it provides for greatly enhanced (or even practical) kinetic bombardment weapons? That it has some sort of huge military purpose that will ignite an arms race?

Really? No. Only if it has enough thrust to power an aircraft with a reasonable (turbine-generator) amount of power and is more efficent AND more powerful than current jet or rocket engines. Otherwise its simply a low-thrust station keeping and manuever system with a long service life :)

A thrust of 0.09lb to 0.9lb per KW as noted in the cited paper isn't that great really. Again the main advantge is you don't have to carry propellant/reaction mass. The main "geopolitical" ramification of the EM-Drive would be that satellite servicing is going to look a lot LESS attractive since with it you'd (supposedly) never have to fill up maneuvering system every again which was the major driver for that concept. :)

Hidden "Rods-From-Gods" in deep space ready to rain down on anyone, anywhere and "undetectable" due to the EM-Drive? Uh, NO just no.

First of all: There is no "stealth" in space. Period.
http://www.projectrho.com/public_html/rocket/spacewardetect.php

Your rod carrier is going to be spotted and tracked. The EM-Drive requires power, which is going to have heat that is going to have to be rejected, which is going to be "visible" to anyone looking in the right direction. The EM-Drive itself (according to one post above) "emits" radiation which can be detected with the right sensor set up. And lastly you CAN see objects in space if you look carefully enough. Even if you used solar panels to provide the power for the EM-Drive they are going to "reflect" some of the energy they recieve AND they are going to be sources of waste heat for the energy they absorb but do not use.

I wonder if anyone has pointed out that IF this "works" the way it would seem to what they've invented is basically the "Space:1889" Ether Propeller :)

Randy

I don't know Randy.  All the talk about Dark Energy, Dark Matter and Quantum Vacume are starting to sound a lot like the Aether concept!

I guess everything old is new again.
Title: Re: EM Drive Developments - related to space flight applications - Thread 2
Post by: RanulfC on 01/23/2015 03:32 pm
http://en.wikipedia.org/wiki/Space:_1889

(http://upload.wikimedia.org/wikipedia/en/thumb/a/af/Space1889rpg.jpg/250px-Space1889rpg.jpg)

But is it the Edison, Zepplin, or Armstrong type? :)

I don't know Randy.  All the talk about Dark Energy, Dark Matter and Quantum Vacume are starting to sound a lot like the Aether concept!

I guess everything old is new again.

As I recall SEVERAL people working in quantum mechanics have pointed out that its FAR to easy to "slip" into that description when talking to us "laymen," however it has the dangers of being taken TOO far way to fast as is a problem with most analogies :)
It probably does not help though that I've noted many of those same people tend to have "steampunk" leanings :)

Randy
Title: Re: EM Drive Developments - related to space flight applications - Thread 2
Post by: Rodal on 01/23/2015 03:40 pm
But is it the Edison, Zepplin, or Armstrong type? :)
(http://ourworlds.topcities.com/space1889/ships/enterprise-side.gif)
(http://ourworlds.topcities.com/space1889/ships/tempestside.gif)
(http://ourworlds.topcities.com/space1889/ships/york-old.gif)
Title: Re: EM Drive Developments - related to space flight applications - Thread 2
Post by: aero on 01/24/2015 01:10 am
@Rodel
Quote
The first subscript (m) is the azimuthal mode number: it indicates the number of full-wave patterns around the circumference of the waveguide.   It is zero for modes in which there is no variation in the circumferential direction.

The second subscript (n) is the radial mode number: it indicates the number of half-wave patterns across the diameter.  The radial mode number (n) plus one indicates the number of nodes across the diameter (counting as nodes the end nodes).

The third subscript (p) is the longitudinal mode number.  It indicates the number of full-wave patterns along the longitudinal length of the waveguide.  It is zero for modes in which there is no variation in the longitudinal direction.

I did double check everything as you advised and there does not seem to be anything wrong with my meep simulation. Neither could I find any questions related to my problem on the Internet. That leads me to think that my problem is still my understanding of mode shapes and cavity dimensions. I thought I had TE1,1, but from the above, for the mode to be TE 1,1, the cavity radius needs to be 1/4 wavelength and the circumference should support 1 full wave pattern. The wavelength for 2.45 GHz is 0.1223642686 in vacuum. So, for the vacuum filled cavity to resonate at 2.45 GHz in the TE 1,1 mode the radius needs to be 0.0305910671 meters. But simply plugging that radius into the formula calculates a resonance frequency of ~2.92GHz in air. So it seems evident that I am still confused about modes and use of the formula to calculate resonance frequencies. Would you lead me through the example of a resonant cavity dimensions for 2.45 GHz resonance?

I also note that driving the cavity from my previous post at 2.45 GHz, R = 0.0377449,  there is no sign of resonance in the field images. So the cavity does not resonate at 2.45 GHz and therefore my dimensions must be wrong.

Title: Re: EM Drive Developments - related to space flight applications - Thread 2
Post by: Rodal on 01/24/2015 07:57 am
....

I did double check everything as you advised and there does not seem to be anything wrong with my meep simulation.....
Please provide the convergence study you have done, to analyze the convergence rate of your Meep calculation.  I can't help you without seeing, and thus being able to analyze, the convergence study data.
....
So the cavity does not resonate at 2.45 GHz and therefore my dimensions must be wrong.
Incorrect dimensioning is one of several kinds of numerical simulation errors possible.
Title: Re: EM Drive Developments - related to space flight applications - Thread 2
Post by: aero on 01/24/2015 09:03 pm
@Rodel -
I am going to explain my understanding of meep in excruciating detail. If you detect a misunderstanding, PLEASE tell me. First, I am setting my simulation in 2-D, running in 64-bit single precision. To run in double precision would require a re-compilation of meep from source code and I am not prepared to do that.
Here is what I have:
Drive frequency 2.45 E+9 Hz, so wavelength = 0.1223642686 meters.
Geometry actual inside dimensions L= 0.1223642686, Dia = 0.0754898000 meters air filled cylindrical cavity with no dielectric. I am ignoring the difference between speed of light in air and vacuum.
Scale factor, 0.01, but is a parameter to adjust
The above gives geometry simulation dimensions in scaled units = 012.23642686, 007.54898000
There is a boundary layer, PML layer around the inside of the computational lattice.
The PML layer thickness is set to 1.
Through an abundance of caution I have set my computational lattice (cell) to twice the geometry dimensions plus twice the PML layer thickness.
Twice the PML layer is correct because there is a layer within on all 4 sides.
Twice the geometry dimensions separates the geometry by one-half its width/height from the PML layer.
This results in -
Computational cell is 26.8729 x 17.498 x 0 (it is a 2D model)
The geometry model is in the center of the computational cell.
The antenna is located at x = 0. and y offset to the radius of the cylinder, and inward - the length is 0.007 meters, or 0.7 scaled units.
The other significant parameter is Resolution, that is, the density of the pixel grid within each distance unit of the computational cell. If resolution = 1, then there would be about 12 pixels lengthwise and 7.5 pixels crosswise in space within the geometry. That is not actually enough density to resolve the wave pattern across the search bandwidth. The tutorial indicates that meep likes a minimum of 10 pixels per wavelength of the frequency, or the frequency to be detected. It happens that the minimum frequency to be searched for is the drive frequency minus half the search band width. In other words, to detect resonance frequency as low as 1.75 GHz, (wavelength = 0.171309976 meters, 17.13 units). The corresponding highest frequency in the bandwidth is 3.15 GHz. The wavelength at 3.15 GHz ~ .095 meters or 9.5 simulation units. This constrains resolution to be no less than 2. Time descretization is the significant constraint.
Time in meep is normalized to the speed of light, that is, c =1. The meep literature is very confusing on time scaling, but by setting the resolution and simply running the simulation for 1 time unit, meep displays the number of time steps. In this way, I find the the number of time steps = 2 times resolution.

Now, running the simulation, using cylindrical coordinates (effectively a 1D simulation) and setting resolution sequentially to 1, 2, 3, 4 meep gives no results. With resolution = 5, meep does give a result.  I made these low resolution runs for the purpose of this convergence study, in reality, I rarely use resolution less than 40. And from the meep tutorial document, " Note: this error is only the uncertainty in the signal
processing"

Here are my results.
Resolution number of time steps resonant frequency      Q           error
    1               2               none detected
    2               4               none detected
    3               6               none detected
    4               8               none detected
    5              10               1.84921E+009    negative        2 e-4
   10              20               1.85128E+009    negative        2 e-4
   20              40               1.86441E+009    ~ 500           6 e-4
   40              80               1.87262E+009    ~ 1200          3 e-4
   80             160               1.86992E+009    ~ 300          13 e-4
  160             320               1.87042E+009    ~ 80           47 e-4

 The detected frequency bounces around consistently with the error which can be taken as estimating the number of significant digits of the frequency detected.   

The quality is very low. I take that to be a result of the cavity dimensions being incorrect for the resonant frequency detected as they are also incorrect for the drive frequency.

I would seriously like to know how to properly design a resonant cavity for a selected resonant mode. I can tinker with the dimensions in meep and get higher quality factors, but that is not very efficient and forces me to guess the mode by looking at images of the wave pattern.
Title: Re: EM Drive Developments - related to space flight applications - Thread 2
Post by: Rodal on 01/24/2015 09:35 pm
....

Here are my results.
Resolution number of time steps resonant frequency      Q           error
    1               2               none detected
    2               4               none detected
    3               6               none detected
    4               8               none detected
    5              10               1.84921E+009    negative        2 e-4
   10              20               1.85128E+009    negative        2 e-4
   20              40               1.86441E+009    ~ 500           6 e-4
   40              80               1.87262E+009    ~ 1200          3 e-4
   80             160               1.86992E+009    ~ 300          13 e-4
  160             320               1.87042E+009    ~ 80           47 e-4

 The detected frequency bounces around consistently with the error which can be taken as estimating the number of significant digits of the frequency detected.   

The quality is very low. I take that to be a result of the cavity dimensions being incorrect for the resonant frequency detected as they are also incorrect for the drive frequency.

....

Well,  there is a lot of stuff here for me to digest, I will need some time to read your post very carefully, and consider what to do, but my first impression is that there is a convergence problem that is most evident from the Q:

Resolution number of time steps resonant frequency      Q           error
    1               2               none detected
    2               4               none detected
    3               6               none detected
    4               8               none detected
    5              10               1.84921E+009    negative        2 e-4
   10              20               1.85128E+009    negative        2 e-4
   20              40               1.86441E+009    ~ 500           6 e-4
   40              80               1.87262E+009    ~ 1200          3 e-4
   80             160               1.86992E+009    ~ 300          13 e-4
  160             320               1.87042E+009    ~ 80           47 e-4

Notice how the Q is completely wrong ("negative") even for

  10              20               1.85128E+009    negative        2 e-4

then reaches a maximum for:

 40              80               1.87262E+009    ~ 1200          3 e-4

which,  as you wrote, it is still a very low value for Q, and then as you increase the time step discretization, the Q gets worse rather than better:

80             160               1.86992E+009    ~ 300          13 e-4
160           320               1.87042E+009    ~ 80           47 e-4

And notice that the error also increased, as you increased the time step for those two cases.

I don't see how the convergence problem can be due to the geometry.  By "geometry" I mean  the dimensions of the cavity (in meters or whatever consistent unit of length). The geometry stayed constant (I presume) for the different discretizations.  Therefore the wrong geometry can lead to a wrong solution but not to a solution that gets worse (increasing error and decreasing Q) with increased discretization

I'll be back.
Title: Re: EM Drive Developments - related to space flight applications - Thread 2
Post by: Rodal on 01/24/2015 09:55 pm
.. I am setting my simulation in 2-D, running in 64-bit single precision....
Just to be speaking the same language, are you really using 64-bit: Double Precision as defined by the IEEE 754 standard ?

64-bit: Double Precision see http://en.wikipedia.org/wiki/Double-precision_floating-point_format

32-bit Double Precision: Computers with 32-bit storage locations use two memory locations to store a 64-bit double-precision number (a single storage location can hold a single-precision number).

32-bit: Single Precision see http://en.wikipedia.org/wiki/Single-precision_floating-point_format
Title: Re: EM Drive Developments - related to space flight applications - Thread 2
Post by: Rodal on 01/24/2015 11:39 pm
.....
The quality is very low. I take that to be a result of the cavity dimensions being incorrect for the resonant frequency detected as they are also incorrect for the drive frequency.

I would seriously like to know how to properly design a resonant cavity for a selected resonant mode. I can tinker with the dimensions in meep and get higher quality factors, but that is not very efficient and forces me to guess the mode by looking at images of the wave pattern.

What did you use for the bandwidth (df) source around the frequency of interest (Drive frequency 2.45 E+9 Hz)?

Could you try running all these cases again, everything the same as before except with a significantly narrower bandwidth (df) source around the frequency of interest ?  .  Reportedly harminv does a better job the narrower the source is around the frequency of interest .
Title: Re: EM Drive Developments - related to space flight applications - Thread 2
Post by: aero on 01/25/2015 12:57 am
.....
The quality is very low. I take that to be a result of the cavity dimensions being incorrect for the resonant frequency detected as they are also incorrect for the drive frequency.

I would seriously like to know how to properly design a resonant cavity for a selected resonant mode. I can tinker with the dimensions in meep and get higher quality factors, but that is not very efficient and forces me to guess the mode by looking at images of the wave pattern.

What did you use for the bandwidth source around the frequency of interest (Drive frequency 2.45 E+9 Hz)?

Could you try running all these cases again, everything the same as before except with a significantly narrower bandwidth source around the frequency of interest ?  .  Reportedly harminv does a better job the narrower the source is around the frequency of interest .
Ok, I did run it again with bandwidth = 0.2 * Drive frequency, for cases up to resolution of 80, but I didn't get anything. Once I narrow the bandwidth to exclude the resonant frequency at 1.87 GHz, there are no resonances within the bandwidth.
Harminv does work better at identifying the resonant frequency with narrower bandwidth, when the frequency is within the bandwidth. I set the drive frequency to 1.873 GHz, narrowed the bandwidth to 0.07 * frequency and got this:

frequency                             Quality factor               error
1,873,339,229.3075 Hz   18,325,307.0778158    1.673972608680621e-7+0.0i

As you can see the quality factor is much higher and the processing error is much lower. The only problem is that it is not the frequency I had hoped for.

I did some further searching and found two things.
1 - The value of the J'0(1) Bessel function = 1.8411837813 which agrees with the number we have.
2 - Meep doesn't actually excite the cavity with Gaussian noise, rather it uses the derivative of a Gaussian signal, whatever that means.

I really don't think this particular problem is in the meep software. As I wrote before, I searched the discussion list, which goes back at least 8 years, and there is no mention of this particular problem. If it were in meep, some user would have encountered it long ago. A 25% discrepancy is hard to overlook. There is a chance that it is in my general understanding of how to model using meep, but my knowledge of meep is far superior to my knowledge of resonant cavity design so using Occam's razor, it is most likely that my cavity design is the problem.

Dr. Rodal, I really appreciate your efforts on my behalf. I will continue to look into the details of resonant cavity design. Maybe it has something to do with the cavity length. But actually, that doesn't seem very likely at all. What do you know about Gaussian noise derivatives and could that be a simple frequency correction? But, when generated with a continuous wave at 2.45 GHz, the field images don't show any resonance.


Title: Re: EM Drive Developments - related to space flight applications - Thread 2
Post by: Rodal on 01/25/2015 01:13 am
.....
The quality is very low. I take that to be a result of the cavity dimensions being incorrect for the resonant frequency detected as they are also incorrect for the drive frequency.

I would seriously like to know how to properly design a resonant cavity for a selected resonant mode. I can tinker with the dimensions in meep and get higher quality factors, but that is not very efficient and forces me to guess the mode by looking at images of the wave pattern.

What did you use for the bandwidth source around the frequency of interest (Drive frequency 2.45 E+9 Hz)?

Could you try running all these cases again, everything the same as before except with a significantly narrower bandwidth source around the frequency of interest ?  .  Reportedly harminv does a better job the narrower the source is around the frequency of interest .
Ok, I did run it again with bandwidth = 0.2 * Drive frequency, for cases up to resolution of 80, but I didn't get anything. Once I narrow the bandwidth to exclude the resonant frequency at 1.87 GHz, there are no resonances within the bandwidth.
Harminv does work better at identifying the resonant frequency with narrower bandwidth, when the frequency is within the bandwidth. I set the drive frequency to 1.873 GHz, narrowed the bandwidth to 0.07 * frequency and got this:

frequency                             Quality factor               error
1,873,339,229.3075 Hz   18,325,307.0778158    1.673972608680621e-7+0.0i

As you can see the quality factor is much higher and the processing error is much lower. The only problem is that it is not the frequency I had hoped for.

I did some further searching and found two things.
1 - The value of the J'0(1) Bessel function = 1.8411837813 which agrees with the number we have.
2 - Meep doesn't actually excite the cavity with Gaussian noise, rather it uses the derivative of a Gaussian signal, whatever that means.

I really don't think this particular problem is in the meep software. As I wrote before, I searched the discussion list, which goes back at least 8 years, and there is no mention of this particular problem. If it were in meep, some user would have encountered it long ago. A 25% discrepancy is hard to overlook. There is a chance that it is in my general understanding of how to model using meep, but my knowledge of meep is far superior to my knowledge of resonant cavity design so using Occam's razor, it is most likely that my cavity design is the problem.

Dr. Rodal, I really appreciate your efforts on my behalf. I will continue to look into the details of resonant cavity design. Maybe it has something to do with the cavity length. But actually, that doesn't seem very likely at all. What do you know about Gaussian noise derivatives and could that be a simple frequency correction? But, when generated with a continuous wave at 2.45 GHz, the field images don't show any resonance.

Ok, that  (drive frequency to 1.873 GHz, bandwidth to 0.07 * frequency) takes care of the Q and error problems  :

frequency                             Quality factor               error
1,873,339,229.3075 Hz   18,325,307.0778158    1.673972608680621e-7+0.0i

I hope that tomorrow I have some time to go over the equations and the dimensions (which I did not have the time to go over yet) to check if I find any reason why you calculate it should be 2.45 GHz and Meep resonates at1.873 GHz instead.

Title: Re: EM Drive Developments - related to space flight applications - Thread 2
Post by: aero on 01/25/2015 04:07 am
It was partly due to a misunderstand of the formula. I changed modes, to T?-?,?,0 and found that with p=0, while the cylinder length has no effect on frequency from the formula, it has a strong effect in meep. It is understandable that changing length should effect the resonance. In fact the effect of changing length is stronger than the effect of changing radius but that is likely due to having a radius not associated with any given mode. The nearest frequency mode is TM0,1,0
I didn't notice that I was looking at the wrong table of frequencies until after I iterated meep to the 2.45 GHz frequency. I'll look at it again tomorrow, but it's getting tired here now.
Title: Re: EM Drive Developments - related to space flight applications - Thread 2
Post by: Mulletron on 01/25/2015 08:56 am
Went back to investigate what I reported in post #233 http://forum.nasaspaceflight.com/index.php?topic=36313.msg1318683#msg1318683 about the RF and Microwave Toolbox app. I found that the app is reporting the correct solutions for TE and TM. The help file just had a typo. I verified it against the KWOK lectures http://www.engr.sjsu.edu/rkwok/EE172/Cavity_Resonator.pdf slide 16. KWOK and the APP match. So this works as a quick and easy way to find resonant modes! There really is an app for everything.

I remain unconvinced that calculating resonant modes for cylinders is a good approximation for conical frustums though.

Title: Re: EM Drive Developments - related to space flight applications - Thread 2
Post by: Stormbringer on 01/25/2015 09:39 am
people have mentioned Dr Dr. Michael McCulloch a few times. i though i would drop a link to one of his papers. though actual discussion should be taken up elsewhere. it is related to the present topic because he has alternate views of how these EM devices (Shawyer, Cannae, White, the Chinese, etc) work.

http://www.ptep-online.com/index_files/2015/PP-40-15.PDF
Title: Re: EM Drive Developments - related to space flight applications - Thread 2
Post by: Mulletron on 01/25/2015 11:18 am
@Rodel
Quote
The first subscript (m) is the azimuthal mode number: it indicates the number of full-wave patterns around the circumference of the waveguide.   It is zero for modes in which there is no variation in the circumferential direction.

The second subscript (n) is the radial mode number: it indicates the number of half-wave patterns across the diameter.  The radial mode number (n) plus one indicates the number of nodes across the diameter (counting as nodes the end nodes).

The third subscript (p) is the longitudinal mode number.  It indicates the number of full-wave patterns along the longitudinal length of the waveguide.  It is zero for modes in which there is no variation in the longitudinal direction.

I did double check everything as you advised and there does not seem to be anything wrong with my meep simulation. Neither could I find any questions related to my problem on the Internet. That leads me to think that my problem is still my understanding of mode shapes and cavity dimensions. I thought I had TE1,1, but from the above, for the mode to be TE 1,1, the cavity radius needs to be 1/4 wavelength and the circumference should support 1 full wave pattern. The wavelength for 2.45 GHz is 0.1223642686 in vacuum. So, for the vacuum filled cavity to resonate at 2.45 GHz in the TE 1,1 mode the radius needs to be 0.0305910671 meters. But simply plugging that radius into the formula calculates a resonance frequency of ~2.92GHz in air. So it seems evident that I am still confused about modes and use of the formula to calculate resonance frequencies. Would you lead me through the example of a resonant cavity dimensions for 2.45 GHz resonance?

I also note that driving the cavity from my previous post at 2.45 GHz, R = 0.0377449,  there is no sign of resonance in the field images. So the cavity does not resonate at 2.45 GHz and therefore my dimensions must be wrong.

I want to point out a discrepancy I found. Perhaps I'm the discrepancy, because I don't agree with my old post or any of the other sources, which is highly unlikely.

First here's what I have about mode numbering from various sources:

ME from thread 1: T(MorE)mnp. m is the # of 1/2 wavelengths around a half circumference, n is the # of 1/2 wavelengths across a radius, p is the # of 1/2 wavelengths of length of the cavity.

Navy Neets mod 11 (screenshot below): The first subscript indicates the number of full-wave patterns around the circumference of the waveguide. The second subscript indicates the number of half-wave patterns across the diameter.........(p left out).

Oracle: http://en.wikipedia.org/wiki/Transverse_mode  In circular waveguides, circular modes exist and here m is the number of half-wavelengths along a half-circumference and n is the number of half-wavelengths along a radius.......(p left out).

Rodal: The first subscript (m) is the azimuthal mode number: it indicates the number of full-wave patterns around the circumference of the waveguide.   
The second subscript (n) is the radial mode number: it indicates the number of half-wave patterns across the diameter. The third subscript (p) is the longitudinal mode number.  It indicates the number of full-wave patterns along the longitudinal length of the waveguide.

So there is conflicting information. Rodal and the Navy agree, the oracle and me are different. I'll see if I can clear it up.....and find deal here.

Using the coke can example from http://www.engr.sjsu.edu/rkwok/EE172/Cavity_Resonator.pdf slide 17, for a radius of 1.25"(or diameter of 2.5"), depth of 5". This comes out to a TE111 f,res of 3.01ghz, which gives me a wavelength of 3.923". So first, to test the first subscript m, the circumference of a circle with r 1.25" is 7.85". 7.85"inch is 2 wavelengths @3.01ghz.

So it appears that m should be the # of full wavelengths around half a circumference.

or

If you don't do any rounding with the coke can example, @3.01ghz you get 3.923928113636958 inches, multiply that by 2 you get 7.847856227273916 inches, which is just shy of the calculated circumference of 7.85, which technically is not a FULL cycle of 2 wavelengths. Which means this example sits on the edge of TE111 and TE211. Technically that 0 wasn't crossed yet.

So is that the answer? FULL wavelengths must be counted, the rest is dropped? Meaning if you go around 2.6 times for example, you just get an m of 2?

###Resolution:
http://forum.nasaspaceflight.com/index.php?topic=36313.msg1321080#msg1321080
http://forum.nasaspaceflight.com/index.php?topic=36313.msg1321196#msg1321196

This is important because soon I'm going to be cutting copper shapes and making stupid mistakes can be very expensive.

I've found fault with the Navy references before on other things, and we all know that everything on the Oracle needs to be verified, and I'm frequently wrong, but Rodal is usually right. So what's going on there?

As far as n or p go, I'm not even going to look at them until I get some feedback about the m discrepancies. I just want to clear this up. I don't mind getting egg on my face.

Break:
You know, I think this got overlooked: "We performed some very early evaluations without the dielectric resonator (TE012 mode at 2168 MHz, with power levels up to ~30 watts) and measured no significant net thrust."

I got a lot of grief before for my approach to deriving the cavity dimensions (starting with the 6.25 inch small end, using the dimensions of the PE discs from 14 of Brady et al Anomalous thust...., but I think those dimensions, (see screenshot below) are exactly spot on and here's empirical proof. So my calculated cavity length in Autocad after scaling based on 6.25inch small ends size, was 10.88". If you look at the frequency of 2168mhz, you'll find the wavelength is 5.4479". Take two wavelengths of this, you'll arrive at 10.8958, my cavity length was 10.88". Converted to meters, it is:
Dsmall=0.15875m  (0.159m)
Dlarge=0.30098m  (0.3m) amazingly round number
Length=0.27637m (.276m)

###Edit: Added link to resolution with current and correct info.
Title: Re: EM Drive Developments - related to space flight applications - Thread 2
Post by: Stormbringer on 01/25/2015 01:47 pm
The Rational Wiki has an interesting page on these EM drives.

http://rationalwiki.org/wiki/EmDrive

Quote
NASA plans to upgrade their equipment to higher power levels, use vacuum-capable RF amplifiers with power ranges of up to 125 W, and design a new tapered cavity analytically expected to produce thrust in the 0.1 N/kW range. Then, the test article will be shipped to other laboratories for independent verification and continued evaluations of the technology, at Glenn Research Center, the Jet Propulsion Laboratory and the Johns Hopkins University Applied Physics Laboratory.

I know that has been discussed here but thought there might be additional info in it that may have not been noticed before.
Title: Re: EM Drive Developments - related to space flight applications - Thread 2
Post by: Rodal on 01/25/2015 01:55 pm
....
First here's what I have about mode numbering from various sources:

ME from thread 1: T(MorE)mnp. m is the # of 1/2 wavelengths around a half circumference...
Navy Neets mod 11 (screenshot below): The first subscript indicates the number of full-wave patterns around the circumference of the waveguide....

Oracle: http://en.wikipedia.org/wiki/Transverse_mode  In circular waveguides, circular modes exist and here m is the number of half-wavelengths along a half-circumference....

Rodal: The first subscript (m) is the azimuthal mode number: it indicates the number of full-wave patterns around the circumference of the waveguide.   
....
So there is conflicting information. Rodal and the Navy agree, the oracle and me are different....

As far as n or p go, I'm not even going to look at them until I get some feedback about the m discrepancies....
As far as "m" goes, I don't see any discrepancy whatsoever.  They all agree:

Mulletron (from thread 1) "the # of 1/2 wavelengths around a half circumference"
Wikipedia " the number of half-wavelengths along a half-circumference"
US NAVY: the number of full-wave patterns around the full circumference
Rodal: the number of full-wave patterns around the full circumference

" the number of half-wavelengths around a half-circumference" is exactly the same as the number of full-wave patterns around the full circumference of the waveguide because a full wave around the full circumference is exactly one half-wave around the half-circumference, or 1/3 wave around 1/3 the circumference, or 1/nr wave around 1/nr circumference where nr is an arbitrary integer.

And the definition of wavelength is exactly the same length as the definition of "full wave pattern". 
Title: Re: EM Drive Developments - related to space flight applications - Thread 2
Post by: Mulletron on 01/25/2015 02:08 pm
We differ in the full mnp description. Look at n. M is the same between us, but the coke can example has me wondering.
Title: Re: EM Drive Developments - related to space flight applications - Thread 2
Post by: Rodal on 01/25/2015 02:11 pm
We differ in the full mnp description. Look at n. M is the same between us, but the coke can example has me wondering.

Well, I wanted to clear that up, because you had previously written:

Quote
As far as n or p go, I'm not even going to look at them until I get some feedback about the m discrepancies....

and I don't see any "m discrepancies"
Title: Re: EM Drive Developments - related to space flight applications - Thread 2
Post by: Mulletron on 01/25/2015 02:14 pm
I'm mixed up on the m after studying the KWOK example mostly.
Title: Re: EM Drive Developments - related to space flight applications - Thread 2
Post by: Rodal on 01/25/2015 02:15 pm
....

Here are my results.
Resolution number of time steps resonant frequency      Q           error
    1               2               none detected
    2               4               none detected
    3               6               none detected
    4               8               none detected
    5              10               1.84921E+009    negative        2 e-4
   10              20               1.85128E+009    negative        2 e-4
   20              40               1.86441E+009    ~ 500           6 e-4
   40              80               1.87262E+009    ~ 1200          3 e-4
   80             160               1.86992E+009    ~ 300          13 e-4
  160             320               1.87042E+009    ~ 80           47 e-4

 The detected frequency bounces around consistently with the error which can be taken as estimating the number of significant digits of the frequency detected.   

The quality is very low. I take that to be a result of the cavity dimensions being incorrect for the resonant frequency detected as they are also incorrect for the drive frequency.

....

Aero, please tell me again exactly what is the actual  numerical value that you actually inputed into MEEP for the drive frequency for the above calculations.  What is the number that you inputed into MEEP.
Title: Re: EM Drive Developments - related to space flight applications - Thread 2
Post by: Mulletron on 01/25/2015 02:33 pm
So is the m resolved because the KWOK modes were so close but no cigar together? That's my hunch, but I want verify before I let it go. Lesson learned.....don't round off when it comes to mode shapes then?
Title: Re: EM Drive Developments - related to space flight applications - Thread 2
Post by: Rodal on 01/25/2015 02:42 pm
So is the m resolved because the KWOK modes were so close but no cigar together? That's my hunch, but I want verify before I let it go. Lesson learned.....don't round off when it comes to mode shapes then?
I need to take care of $ paying work first   :).  As I get time, my first priority is to deal with aero's problem (it came first  :)  ) and after that I'll take a thorough look at the numbers in KWOK and the "n" and "p" issue you brought up.  Sorry I don't want to give you an answer as an immediate reflex.  I want to give you a thoughtful answer.  I need some time to go carefully over these issues  :). 
Title: Re: EM Drive Developments - related to space flight applications - Thread 2
Post by: Mulletron on 01/25/2015 03:18 pm
FWIW for TE111(mnp) 2.45ghz, cavity dimensions for a cylinder are Length=0.122448m Diameter=0.0827888m in air. Or 4.8208"x 3.2594". Much more precision than necessary or attainable. The exact frequency I could get is 2.450019ghz.

Anyway what you do if you build something like this is build it to the ball park dimensions, then tune it the rest of the way with tuning screws to bring it into resonance and maximize Q. You can't build something easily with those kind of tolerances above.

Edit:
Okay to get a perfect 2.45ghz, TE111 with way too high precision:
L=0.1224489m
D=0.08278945m
Title: Re: EM Drive Developments - related to space flight applications - Thread 2
Post by: aero on 01/25/2015 03:42 pm
....

Here are my results.
Resolution number of time steps resonant frequency      Q           error
    1               2               none detected
    2               4               none detected
    3               6               none detected
    4               8               none detected
    5              10               1.84921E+009    negative        2 e-4
   10              20               1.85128E+009    negative        2 e-4
   20              40               1.86441E+009    ~ 500           6 e-4
   40              80               1.87262E+009    ~ 1200          3 e-4
   80             160               1.86992E+009    ~ 300          13 e-4
  160             320               1.87042E+009    ~ 80           47 e-4

 The detected frequency bounces around consistently with the error which can be taken as estimating the number of significant digits of the frequency detected.   

The quality is very low. I take that to be a result of the cavity dimensions being incorrect for the resonant frequency detected as they are also incorrect for the drive frequency.

....

Aero, please tell me again exactly what is the actual  numerical value that you actually inputed into MEEP for the drive frequency for the above calculations.  What is the number that you inputed into MEEP.
Drive frequency 2.45 E+9 Hz, so wavelength = 0.1223642686 meters.
Geometry actual inside dimensions L= 0.1223642686, Dia = 0.0754898000 meters air filled cylindrical cavity with no dielectric. I am ignoring the difference between speed of light in air and vacuum.

I copied those numbers straight from the control file. This is what I used.
Title: Re: EM Drive Developments - related to space flight applications - Thread 2
Post by: Mulletron on 01/25/2015 03:54 pm

Drive frequency 2.45 E+9 Hz, so wavelength = 0.1223642686 meters.
Geometry actual inside dimensions L= 0.1223642686, Dia = 0.0754898000 meters air filled cylindrical cavity with no dielectric. I am ignoring the difference between speed of light in air and vacuum.

I copied those numbers straight from the control file. This is what I used.

I get TE111, 2.63018ghz from that.
Title: Re: EM Drive Developments - related to space flight applications - Thread 2
Post by: Rodal on 01/25/2015 03:57 pm
....

Here are my results.
Resolution number of time steps resonant frequency      Q           error
    1               2               none detected
    2               4               none detected
    3               6               none detected
    4               8               none detected
    5              10               1.84921E+009    negative        2 e-4
   10              20               1.85128E+009    negative        2 e-4
   20              40               1.86441E+009    ~ 500           6 e-4
   40              80               1.87262E+009    ~ 1200          3 e-4
   80             160               1.86992E+009    ~ 300          13 e-4
  160             320               1.87042E+009    ~ 80           47 e-4

 The detected frequency bounces around consistently with the error which can be taken as estimating the number of significant digits of the frequency detected.   

The quality is very low. I take that to be a result of the cavity dimensions being incorrect for the resonant frequency detected as they are also incorrect for the drive frequency.

....

Aero, please tell me again exactly what is the actual  numerical value that you actually inputed into MEEP for the drive frequency for the above calculations.  What is the number that you inputed into MEEP.
Drive frequency 2.45 E+9 Hz, so wavelength = 0.1223642686 meters.
Geometry actual inside dimensions L= 0.1223642686, Dia = 0.0754898000 meters air filled cylindrical cavity with no dielectric. I am ignoring the difference between speed of light in air and vacuum.

I copied those numbers straight from the control file. This is what I used.

Sorry to be insistent, but just to make clear, could you please confirm that the numerical value that you actually typed as an input for MEEP was 2.45 E+9 ?

I am not asking what the frequency in Hz should be.  I am asking what number you typed as an input to MEEP.
Title: Re: EM Drive Developments - related to space flight applications - Thread 2
Post by: Rodal on 01/25/2015 04:08 pm
....

Here are my results.
Resolution number of time steps resonant frequency      Q           error
    1               2               none detected
    2               4               none detected
    3               6               none detected
    4               8               none detected
    5              10               1.84921E+009    negative        2 e-4
   10              20               1.85128E+009    negative        2 e-4
   20              40               1.86441E+009    ~ 500           6 e-4
   40              80               1.87262E+009    ~ 1200          3 e-4
   80             160               1.86992E+009    ~ 300          13 e-4
  160             320               1.87042E+009    ~ 80           47 e-4

 The detected frequency bounces around consistently with the error which can be taken as estimating the number of significant digits of the frequency detected.   

The quality is very low. I take that to be a result of the cavity dimensions being incorrect for the resonant frequency detected as they are also incorrect for the drive frequency.

....

Aero, please tell me again exactly what is the actual  numerical value that you actually inputed into MEEP for the drive frequency for the above calculations.  What is the number that you inputed into MEEP.
Drive frequency 2.45 E+9 Hz, so wavelength = 0.1223642686 meters.
Geometry actual inside dimensions L= 0.1223642686, Dia = 0.0754898000 meters air filled cylindrical cavity with no dielectric. I am ignoring the difference between speed of light in air and vacuum.

I copied those numbers straight from the control file. This is what I used.

Sorry to be insistent, but just to make clear, could you please confirm that the numerical value that you actually typed as an input for MEEP was 2.45 E+9 ?

I am not asking what the frequency in Hz should be.  I am asking what number you typed as an input to MEEP.

Quote from: http://ab-initio.mit.edu/wiki/index.php/Meep_Introduction
Moreover, since c = 1 in Meep units, a (or a / c) is our unit of time as well. In particular, the frequency f in Meep (corresponding to a time dependence e − i2πft) is always specified in units of c / a

Note: if you use meters as the unit of length in MEEP, then it must follow that the MEEP unit of time is meters and the MEEP unit of frequency is 1/meter.

If you used meters as the unit of length such that your input was

aeroMeepLength = 0.1223642686

aeroMeepDiameter = 0.0754898000

[Showing only 6 signficant digits for calculations from now on, but I am using full Mathematica precision]

aeroMeepRadius = 0.0377449


In meters/second the speeds of light:

cVacuum = c
               = 299792458;


cAir = c / (Sqrt[mur*epsilonr])
       = 299705000

Then your input for frequency to MEEP, in MEEP units, for mode TE111, (m=1,n=1,p=1) should have been:

p = 1;

X'1,1=1.84118378134065;

frequencyTEMeep
             =  (cAir/cVacuum) (1/(2*Pi))*Sqrt[(X'1,1/aeroMeepRadius)^2 +((p*Pi/aeroMeepLength)^2)]
             =  (cAir/cVacuum) (1/(2*Pi))*Sqrt[(1.84118378134065/0.0377449)^2 +((1*Pi/0.122364)^2)]
            =  8.77064

In MEEP units of frequency (1/meter) which is almost 9 orders of magnitude smaller than 2.45 E+9

In other words, as an input to MEEP you must divide the expression for frequency (for example as it appears in Wikipedia) by cVacuum in order to input frequency in MEEP frequency units of 1/length.

Once you have done this, you consequently have to interpret the MEEP output for frequency in MEEP frequency units (which are not Hz, they are 1/length).

For example:  Meep frequency of 8.77064 (1/meter) corresponds to  8.77064*c = 8.77064*cVacuum  = 2.62937 GHz
Title: Re: EM Drive Developments - related to space flight applications - Thread 2
Post by: Rodal on 01/25/2015 05:28 pm
...
Geometry actual inside dimensions L= 0.1223642686, Dia = 0.0754898000 meters air filled cylindrical cavity with no dielectric. I am ignoring the difference between speed of light in air and vacuum.
Scale factor, 0.01, but is a parameter to adjust
The above gives geometry simulation dimensions in scaled units = 012.23642686, 007.54898000
...
I don't follow the need for this "scale factor"  (Scale factor, 0.01) you are using.  It may unnecessarily complicate things -- I would not use it until you have exactly matched the exact solution.

The mention of "scale factor" in MEEP I found was in http://ab-initio.mit.edu/wiki/index.php/Meep_Reference as:

Quote
susceptibility
Parent class for various dispersive susceptibility terms, parameterized by an anisotropic amplitude σ (see Material dispersion in Meep):
sigma [number]
The scale factor σ.

But the exact solution (of an empty cylindrical cavity) we are considering does not consider any anisotropic materials consideration.

You have to be careful as to what MEEP will interpret as inputs ("garbage in" = "garbage out"  :)  ).  If you input to MEEP  L= 0.1223642686 (meters), Dia = 0.0754898000 (meters), then the MEEP frequency should be as per my previous post in 8.77064 1/meters units.  (And you should interpret the output in 1/meters frequency units as well)


If instead you input L = 12.23642686,  D= 7.54898000 you are effectively using centimeters as your input unit, and therefore your MEEP frequency should be input in 1/cm units, giving 0.0877064  1/centimeter ,  but again, what is the need to use a Scale Factor? 
Title: Re: EM Drive Developments - related to space flight applications - Thread 2
Post by: aero on 01/25/2015 06:24 pm
The scale factor, also known as characteristic length - "we pick some characteristic lengthscale in the system, a, and use that as our unit of distance." Or, more detailed, from "Units in Meep" here; http://ab-initio.mit.edu/wiki/index.php/Meep_Introduction (http://ab-initio.mit.edu/wiki/index.php/Meep_Introduction)

Quote
In particular, because Maxwell's equations are scale invariant (multiplying the sizes of everything by 10 just divides the corresponding solution frequencies by 10), it is convenient in electromagnetic problems to choose scale-invariant units (see our online textbook, ch. 2). That means that we pick some characteristic lengthscale in the system, a, and use that as our unit of distance.
Title: Re: EM Drive Developments - related to space flight applications - Thread 2
Post by: Rodal on 01/25/2015 06:30 pm
The scale factor, also known as characteristic length - "we pick some characteristic lengthscale in the system, a, and use that as our unit of distance." Or, more detailed, from "Units in Meep" here; http://ab-initio.mit.edu/wiki/index.php/Meep_Introduction (http://ab-initio.mit.edu/wiki/index.php/Meep_Introduction)

Quote
In particular, because Maxwell's equations are scale invariant (multiplying the sizes of everything by 10 just divides the corresponding solution frequencies by 10), it is convenient in electromagnetic problems to choose scale-invariant units (see our online textbook, ch. 2). That means that we pick some characteristic lengthscale in the system, a, and use that as our unit of distance.

 It unnecessarily complicates things at this point, it presents extra problems of interpretation -- I would not use it until you have exactly matched the exact solution.  If you must, use Scale Factor =1, for the time being: if you input to MEEP  L= 0.1223642686 (meters), Dia = 0.0754898000 (meters), then the MEEP frequency should be as per my previous post in 8.77064 1/meters units.  (And you should interpret the output in 1/meters frequency units as well)


If instead you input L = 12.23642686,  D= 7.54898000 you are effectively using centimeters as your input unit of length, and therefore your MEEP frequency should be input in 1/cm units, giving MEEP Frequency = 0.0877064  1/centimeter, and you would have to multiply the output frequencies by cVacuum = 29979245800 centimeter/second to express the output in Hz.  Unnecessarily messy at this point.
Title: Re: EM Drive Developments - related to space flight applications - Thread 2
Post by: aero on 01/25/2015 07:17 pm
The scale factor, also known as characteristic length - "we pick some characteristic lengthscale in the system, a, and use that as our unit of distance." Or, more detailed, from "Units in Meep" here; http://ab-initio.mit.edu/wiki/index.php/Meep_Introduction (http://ab-initio.mit.edu/wiki/index.php/Meep_Introduction)

Quote
In particular, because Maxwell's equations are scale invariant (multiplying the sizes of everything by 10 just divides the corresponding solution frequencies by 10), it is convenient in electromagnetic problems to choose scale-invariant units (see our online textbook, ch. 2). That means that we pick some characteristic lengthscale in the system, a, and use that as our unit of distance.

 It unnecessarily complicates things at this point, it presents extra problems of interpretation -- I would not use it until you have exactly matched the exact solution.  If you must, use Scale Factor =1, for the time being: if you input to MEEP  L= 0.1223642686 (meters), Dia = 0.0754898000 (meters), then the MEEP frequency should be as per my previous post in 8.77064 1/meters units.  (And you should interpret the output in 1/meters frequency units as well)


If instead you input L = 12.23642686,  D= 7.54898000 you are effectively using centimeters as your input unit of length, and therefore your MEEP frequency should be input in 1/cm units, giving MEEP Frequency = 0.0877064  1/centimeter, and you would have to multiply the output frequencies by cVacuum = 29979245800 centimeter/second to express the output in Hz.  Unnecessarily messy at this point.

It doesn't work that way. I input units in meters, and the scale factor. The input is scaled, then the output that I gave you is "unscaled" to be in SI units. But yes, I can run meep with a scale factor of 1. It gives the same answers but takes more CPU so the runs are longer. Not to bad for this simple 1D calibration problem though.

Here is an example using resolution = 1200, which is quite low resolution.
frequency            quality factor                  error
1.86060E+009   37934.0653626318    7.872026063658947e-6+0.0i
 Had I used this scale factor and geometry to generate 2D images of the developing fields, it would take about 12 hours computer run time, per meep estimate. Generating those images using a scale factor of 0.01 takes about 45 minutes as I recall.

I think the solution is in p, the cavity length. If I understand it at all, the cavity with length less than 1/2 wavelength resonates in the p=0 mode, with cavity length between 1/2 and 3/2 wavelength it can resonate in a p=1 mode, with cavity length between 3/2 and 5/2 wavelength it can resonate in p=2 mode and so forth.

But when I calculate a radius using cavity length = wave length, that radius gives a resonant frequency of about 2.28 GHz. Then I adjust the cavity length to obtain a resonant frequency of 2.45 GHz, the formula (inverted to calculate R) gives a new R. Plugging that new R back into the frequency formula with the adjusted cavity length, the frequency formula gives back the same 2.28 GHz.  I don't understand it yet but I'm working on it.
Title: Re: EM Drive Developments - related to space flight applications - Thread 2
Post by: Rodal on 01/25/2015 07:28 pm
The scale factor, also known as characteristic length - "we pick some characteristic lengthscale in the system, a, and use that as our unit of distance." Or, more detailed, from "Units in Meep" here; http://ab-initio.mit.edu/wiki/index.php/Meep_Introduction (http://ab-initio.mit.edu/wiki/index.php/Meep_Introduction)

Quote
In particular, because Maxwell's equations are scale invariant (multiplying the sizes of everything by 10 just divides the corresponding solution frequencies by 10), it is convenient in electromagnetic problems to choose scale-invariant units (see our online textbook, ch. 2). That means that we pick some characteristic lengthscale in the system, a, and use that as our unit of distance.

 It unnecessarily complicates things at this point, it presents extra problems of interpretation -- I would not use it until you have exactly matched the exact solution.  If you must, use Scale Factor =1, for the time being: if you input to MEEP  L= 0.1223642686 (meters), Dia = 0.0754898000 (meters), then the MEEP frequency should be as per my previous post in 8.77064 1/meters units.  (And you should interpret the output in 1/meters frequency units as well)


If instead you input L = 12.23642686,  D= 7.54898000 you are effectively using centimeters as your input unit of length, and therefore your MEEP frequency should be input in 1/cm units, giving MEEP Frequency = 0.0877064  1/centimeter, and you would have to multiply the output frequencies by cVacuum = 29979245800 centimeter/second to express the output in Hz.  Unnecessarily messy at this point.

It doesn't work that way. I input units in meters, and the scale factor. The input is scaled, then the output that I gave you is "unscaled" to be in SI units. But yes, I can run meep with a scale factor of 1. It gives the same answers but takes more CPU so the runs are longer. Not to bad for this simple 1D calibration problem though.

Here is an example using resolution = 1200, which is quite low resolution.
frequency            quality factor                  error
1.86060E+009   37934.0653626318    7.872026063658947e-6+0.0i
 Had I used this scale factor and geometry to generate 2D images of the developing fields, it would take about 12 hours computer run time, per meep estimate. Generating those images using a scale factor of 0.01 takes about 45 minutes as I recall.

Wait a minute, this is the first time that I see:

frequency            quality factor                  error
1.86060E+009   37934.0653626318    7.872026063658947e-6+0.0i

from you.


do you agree or not, that your input frequency should have been frequencyTEMeep =  8.77064 (1/meters) if your length input is in meters?  ???

and that the problem you had was that you were inputting frequency into Meep in Hertz instead of using consistent MEEP units ?  ???
Title: Re: EM Drive Developments - related to space flight applications - Thread 2
Post by: aero on 01/25/2015 08:24 pm
No it's not, that resolution is likely somewhere between these  lines.

 10              20               1.85128E+009    negative        2 e-4
 20              40               1.86441E+009    ~ 500           6 e-4
 40              80               1.87262E+009    ~ 1200          3 e-4

that I posted 2 pages back.

And no I don't. The meep input frequency must be in the same dimensional units as the geometry, I use SI units.

I have made progress though. Using the frequency formula to adjust cavity length so that the formula gave 2.45GHz, required a cavity length about  0.0965 meters. Iteratively running meep and adjusting the cavity length to force resonance at 2.45 GHZ independently produced a cavity length of 0.0936 meters.

I emphasize that I worked those problems independently of each other so the fact that they are in near agreement is telling. I would like to find a combination length and radius that would give the frequency without the length being so close to 3/4 wavelength. In fact, I would like for the length to be exactly one wavelength and for which I knew the mode. TE 1,1,1 would be good, but TE 1,4,1 might also work. Maybe now that I have found one solution, I can find more solutions.
Title: Re: EM Drive Developments - related to space flight applications - Thread 2
Post by: Rodal on 01/25/2015 08:36 pm
No it's not, that resolution is likely somewhere between these  lines.

 10              20               1.85128E+009    negative        2 e-4
 20              40               1.86441E+009    ~ 500           6 e-4
 40              80               1.87262E+009    ~ 1200          3 e-4

that I posted 2 pages back.

And no I don't. The meep input frequency must be in the same dimensional units as the geometry, I use SI units.

I have made progress though. Using the frequency formula to adjust cavity length so that the formula gave 2.45GHz, required a cavity length about  0.0965 meters. Iteratively running meep and adjusting the cavity length to force resonance at 2.45 GHZ independently produced a cavity length of 0.0936 meters.

I emphasize that I worked those problems independently of each other so the fact that they are in near agreement is telling. I would like to find a combination length and radius that would give the frequency without the length being so close to 3/4 wavelength. In fact, I would like for the length to be exactly one wavelength and for which I knew the mode. TE 1,1,1 would be good, but TE 1,4,1 might also work. Maybe now that I have found one solution, I can find more solutions.

Well, we disagree.  The correct solution for the geometry you input

aeroMeepLength = 0.1223642686 meters
aeroMeepDiameter = 0.0754898000 meters

at TE111 is 2.63 Ghz, I agree with Mulletron. (Mulletron  2.63018 Ghz, Rodal 2.62937 GHz)  Also, you should have quite a lot of frequencies bunched up nearby TE111 at 2.63 Ghz.  It is incorrect to get only one frequency and very far away at 1.87 GHz instead of 2.63 Ghz.

Also, unless you input a finite tan delta (which I did not find in your writing), your Q should be extremely large (for tan delta=0, Q goes to infinity).  The Q's you report are very low.

____

At my next break  :) I will look into the m,n,p issue brought by Mulletron.
Title: Re: EM Drive Developments - related to space flight applications - Thread 2
Post by: aero on 01/25/2015 09:27 pm
My input frequency is converted to meep units in the control file, meep frequency =  0.08172320332354725. That is scaled by the 0.01 factor. But that is not an input. The input is frequency in SI units. The conversion is scale factor/c so I guess the units would be 1/meter internally.

It's difficult for me to come up with 8.77064 1/meters though. That is your 2.63 GHz number and I've not seen it in any of my meep runs that I recall.
Title: Re: EM Drive Developments - related to space flight applications - Thread 2
Post by: Rodal on 01/25/2015 09:36 pm
My input frequency is converted to meep units in the control file, meep frequency =  0.08172320332354725. That is scaled by the 0.01 factor. But that is not an input. The input is frequency in SI units. The conversion is scale factor/c so I guess the units would be 1/meter internally.

It's difficult for me to come up with 8.77064 1/meters though. That is your 2.63 GHz number and I've not seen it in any of my meep runs that I recall.

1) It should be 0.0877064  1/centimeter Meep frequency units instead of 0.0817232.  For 

aeroMeepLength = 0.1223642686 meters
aeroMeepDiameter = 0.0754898000 meters

at TE111, since the frequency is 2.63 Ghz as Mulletron and I get.

2) You should run with your "scale factor" of 1 instead, for as long as that takes, and see what results you get.  If the scale factor of 1 results in a longer run, it means that your scale factor is getting mixed with the finite difference mesh discretization.
Title: Re: EM Drive Developments - related to space flight applications - Thread 2
Post by: Rodal on 01/25/2015 10:03 pm

....

I want to point out a discrepancy I found. Perhaps I'm the discrepancy, because I don't agree with my old post or any of the other sources, which is highly unlikely.

First here's what I have about mode numbering from various sources:

ME from thread 1: T(MorE)mnp. m is the # of 1/2 wavelengths around a half circumference, n is the # of 1/2 wavelengths across a radius, p is the # of 1/2 wavelengths of length of the cavity.

Navy Neets mod 11 (screenshot below): The first subscript indicates the number of full-wave patterns around the circumference of the waveguide. The second subscript indicates the number of half-wave patterns across the diameter.........(p left out).

Oracle: http://en.wikipedia.org/wiki/Transverse_mode  In circular waveguides, circular modes exist and here m is the number of half-wavelengths along a half-circumference and n is the number of half-wavelengths along a radius.......(p left out).

Rodal: The first subscript (m) is the azimuthal mode number: it indicates the number of full-wave patterns around the circumference of the waveguide.   
The second subscript (n) is the radial mode number: it indicates the number of half-wave patterns across the diameter. The third subscript (p) is the longitudinal mode number.  It indicates the number of full-wave patterns along the longitudinal length of the waveguide.

So there is conflicting information. Rodal and the Navy agree, the oracle and me are different. I'll see if I can clear it up.....and find deal here.

Using the coke can example from http://www.engr.sjsu.edu/rkwok/EE172/Cavity_Resonator.pdf slide 17, for a radius of 1.25"(or diameter of 2.5"), depth of 5". This comes out to a TE111 f,res of 3.01ghz, which gives me a wavelength of 3.923". So first, to test the first subscript m, the circumference of a circle with r 1.25" is 7.85". 7.85"inch is 2 wavelengths @3.01ghz.

So it appears that m should be the # of full wavelengths around half a circumference.

or

If you don't do any rounding with the coke can example, @3.01ghz you get 3.923928113636958 inches, multiply that by 2 you get 7.847856227273916 inches, which is just shy of the calculated circumference of 7.85, which technically is not a FULL cycle of 2 wavelengths. Which means this example sits on the edge of TE111 and TE211. Technically that 0 wasn't crossed yet.

So is that the answer? FULL wavelengths must be counted, the rest is dropped? Meaning if you go around 2.6 times for example, you just get an m of 2?


This is important because soon I'm going to be cutting copper shapes and making stupid mistakes can be very expensive.

I've found fault with the Navy references before on other things, and we all know that everything on the Oracle needs to be verified, and I'm frequently wrong, but Rodal is usually right. So what's going on there?

As far as n or p go, I'm not even going to look at them until I get some feedback about the m discrepancies. I just want to clear this up. I don't mind getting egg on my face.

Break:
You know, I think this got overlooked: "We performed some very early evaluations without the dielectric resonator (TE012 mode at 2168 MHz, with power levels up to ~30 watts) and measured no significant net thrust."

I got a lot of grief before for my approach to deriving the cavity dimensions (starting with the 6.25 inch small end, using the dimensions of the PE discs from 14 of Brady et al Anomalous thust...., but I think those dimensions, (see screenshot below) are exactly spot on and here's empirical proof. So my calculated cavity length in Autocad after scaling based on 6.25inch small ends size, was 10.88". If you look at the frequency of 2168mhz, you'll find the wavelength is 5.4479". Take two wavelengths of this, you'll arrive at 10.8958, my cavity length was 10.88". Converted to meters, it is:
Dsmall=0.15875m  (0.159m)
Dlarge=0.30098m  (0.3m) amazingly round number
Length=0.27637m (.276m)

Thank you @Mulletron for looking at this discrepancy, which enables me to correct this mistake:   :)

My description of the quantum mode number "p" for the longitudinal direction of a cylindrical cavity in http://forum.nasaspaceflight.com/index.php?topic=36313.msg1318217#msg1318217 should be corrected from "number of full-wave patterns along the longitudinal length " to "number of half-wave patterns along the longitudinal length "

Quote
The first subscript (m) is the azimuthal mode number: it indicates the number of full-wave patterns around the circumference of the cylindrical cavity.   It is zero for modes in which there is no variation in the circumferential direction.

The second subscript (n) is the radial mode number: it indicates the number of half-wave patterns across the diameter.  The radial mode number (n) plus one indicates the number of nodes across the diameter (counting as nodes the end nodes).

The third subscript (p) is the longitudinal mode number.  It indicates the number of full-wave patterns along the longitudinal length of the waveguide.  It is zero for modes in which there is no variation in the longitudinal direction.


to:

The first subscript (m) is the azimuthal mode number: it indicates the number of full-wave patterns around the circumference of the cavity.   It is zero for modes in which there is no variation in the circumferential direction.

The second subscript (n) is the radial mode number: it indicates the number of half-wave patterns across the diameter.  The radial mode number (n) plus one indicates the number of nodes across the diameter (counting as nodes the end nodes).  The radial mode number (n) minus one indicates the number of middle nodes across the diameter (not counting as nodes the end nodes).  The radial mode number (n) cannot be zero.

The third subscript (p) is the longitudinal mode number.  It indicates the number of half-wave patterns along the longitudinal length of the cavity.  It is zero for modes in which there is no variation in the longitudinal direction.


I double-checked the others (m and n) and I am sure that they are correct. 

The US Navy reference is correct.  GO NAVY !

(http://orangehoodie.com/photo/lifeliberty800.jpg)

Concerning, "m"  "the azimuthal mode number: it indicates the number of full-wave patterns around the circumference of the cavity, all the descriptive references you brought up agree (as we previously discussed in this thread).

Concerning "n" (the radial mode number that "indicates the number of half-wave patterns across the diameter"), the Wikipedia link is incorrect -I didn't have the time to check the history of that Wiki entry to see what is the history of that Wikipedia error. When I have the time I will examine that and I will correct the entry in Wikipedia.


To make the long story short, the quantum mode numbers have to respect the boundary conditions:

1) In the circumferential direction there is really no boundary, the condition is one of periodicity, hence m must be the number of full waves in the circumferential direction.  All references agree

2) In the radial and longitudinal directions, the boundary conditions (electrical conductivity at the surfaces) are at each end of the diametral direction or at each end of the longitudinal direction.  There is no boundary condition at the center of the circular cross-section other than symmetry or antisymmetry (unless it would be a double-concentric cylinder having an inner conductive cylinder in addition to the outer conductive cylindrical surface) hence a half-wave can be supported within the diameter direction and a half-wave can be supported within the longitudinal direction.  Hence the radial and longitudinal directions n and p are the number of half-waves in both directions, just like a rectangular cavity.
Title: Re: EM Drive Developments - related to space flight applications - Thread 2
Post by: Rodal on 01/25/2015 10:10 pm
I have not had a chance to look at your Coke can for the numbers in KWOK yet.
Title: Re: EM Drive Developments - related to space flight applications - Thread 2
Post by: Rodal on 01/25/2015 11:35 pm
I found this very interesting (expired) 1969 patent by Wolf, owned by JOHNSON CONTROLS INTERNATIONAL, that deals with conical cavities !

http://www.google.com/patents/US3425006

It has a lot of interesting practical stuff, for example dealing with spherical ends (like in Greg Egan's solution  http://gregegan.customer.netspace.net.au/SCIENCE/Cavity/Cavity.html  ) instead of flat end plates (NASA Brady et.al., Shawyer(Experimental and Demo) and Prof. Juan Yang in China's EM Drives all have flat ends):

Quote
The naturally conforming reflecting end for a conical Wave-guide is such a sphere, just as the conforming end for a cylindrical wave-guide is a flat plate. It is believed that the reason that the spherical end is superior is that fewer higher order modes need be excited in reflection from such an end than from a fiat end, or that the degree of such excitation is less. It is further believed that higher order modes can be excited in the large end of the resonator which are undetectable through the input and output structures located at the small end, where these modes are below cut-off. They are however, detectable in that such coupling between modes may cause a reduction in the Q factor of the desired mode.

Since the curvature of the end plate cannot be altered as the resonator is tuned, the curvature of this end plate is also a factor which does not scale during the tuning of the cavity. It thus will cause slight perturbation of the modes. It appears to be possible to employ this perturbation to offset the effects of the perturbation due to the gap around the plunger, which also varies during tuning. This generally requires empirical adjustment of the plunger radius of curvature until the Q factor remains good over the tuning range.

As a feature of the third concept, an absorber is provided in the back cavity, behind the plunger or base plate and around the rim of the base plate. Successful use has also been made in experiment of grooves made in the face of the plate and partially filled with absorbing material.

This looks like the earlier Shawyer designs (with the adjustable end):

http://patentimages.storage.googleapis.com/pages/US3425006-0.png




EDIT: it is interesting that the only EM Drive with spherical end sections is the 2014 superconducting design by Shawyer, who after his experience with the more traditional designs has now: 1) replaced the flat ends with spherical ends, 2) discarded the adjustable end, 3) adopted piezoelectric compensation for Doppler shift, 4) uses superconductive inner surfaces to get much higher Q,  5) reportedly (according to @Wembley) discarded the dielectric and 6) it has a length smaller than both the small and big diameters.
Title: Re: EM Drive Developments - related to space flight applications - Thread 2
Post by: ThinkerX on 01/26/2015 12:34 am
I find myself wondering: Shawyer has been at this for a long while, and seems pretty bright.

So...did he discard the dielectric for good, or is he just doing 'prep work' of some sort that doesn't require a dielectric at the moment.

Also, my memory is a bit hazy at the moment, but when you allow for the tortured grammar, it seemed almost like Shawyer was talking about something similar to Doctor McCulloch's theory when it came to operating principle.  (That was more pages back than I care to think about.)
Title: Re: EM Drive Developments - related to space flight applications - Thread 2
Post by: Mulletron on 01/26/2015 10:39 am

Thank you @Mulletron for looking at this discrepancy, which enables me to correct this mistake:   :)

My description of the quantum mode number "p" for the longitudinal direction of a cylindrical cavity in http://forum.nasaspaceflight.com/index.php?topic=36313.msg1318217#msg1318217 should be corrected from "number of full-wave patterns along the longitudinal length " to "number of half-wave patterns along the longitudinal length "

Quote
The first subscript (m) is the azimuthal mode number: it indicates the number of full-wave patterns around the circumference of the cylindrical cavity.   It is zero for modes in which there is no variation in the circumferential direction.

The second subscript (n) is the radial mode number: it indicates the number of half-wave patterns across the diameter.  The radial mode number (n) plus one indicates the number of nodes across the diameter (counting as nodes the end nodes).

The third subscript (p) is the longitudinal mode number.  It indicates the number of full-wave patterns along the longitudinal length of the waveguide.  It is zero for modes in which there is no variation in the longitudinal direction.


to:

The first subscript (m) is the azimuthal mode number: it indicates the number of full-wave patterns around the circumference of the cavity.   It is zero for modes in which there is no variation in the circumferential direction.

The second subscript (n) is the radial mode number: it indicates the number of half-wave patterns across the diameter.  The radial mode number (n) plus one indicates the number of nodes across the diameter (counting as nodes the end nodes).  The radial mode number (n) minus one indicates the number of middle nodes across the diameter (not counting as nodes the end nodes).  The radial mode number (n) cannot be zero.

The third subscript (p) is the longitudinal mode number.  It indicates the number of half-wave patterns along the longitudinal length of the cavity.  It is zero for modes in which there is no variation in the longitudinal direction.


I double-checked the others (m and n) and I am sure that they are correct. 

The US Navy reference is correct.  GO NAVY !

(http://orangehoodie.com/photo/lifeliberty800.jpg)

Concerning, "m"  "the azimuthal mode number: it indicates the number of full-wave patterns around the circumference of the cavity, all the descriptive references you brought up agree (as we previously discussed in this thread).

Concerning "n" (the radial mode number that "indicates the number of half-wave patterns across the diameter"), the Wikipedia link is incorrect -I didn't have the time to check the history of that Wiki entry to see what is the history of that Wikipedia error. When I have the time I will examine that and I will correct the entry in Wikipedia.


To make the long story short, the quantum mode numbers have to respect the boundary conditions:

1) In the circumferential direction there is really no boundary, the condition is one of periodicity, hence m must be the number of full waves in the circumferential direction.  All references agree

2) In the radial and longitudinal directions, the boundary conditions (electrical conductivity at the surfaces) are at each end of the diametral direction or at each end of the longitudinal direction.  There is no boundary condition at the center of the circular cross-section other than symmetry or antisymmetry (unless it would be a double-concentric cylinder having an inner conductive cylinder in addition to the outer conductive cylindrical surface) hence a half-wave can be supported within the diameter direction and a half-wave can be supported within the longitudinal direction.  Hence the radial and longitudinal directions n and p are the number of half-waves in both directions, just like a rectangular cavity.


I figured out the confusion I had over the m subscript after studying the KWOK 3.01ghz coke can, and the 2.45ghz example. The issue is, wavelengths is NOT the same as wave pattern. For both the frequency examples, you get pretty much exactly 2 wavelengths around the circumference, which breaks TE111. So wavelengths is where I got it wrong. My old n subscript was jacked up too. As the Navy example states, and @Rodal did too. It is wave PATTERN. Glad I got that sorted out. So the Navy example is correct, and then add the p subscript from my old post, and it all falls in to place, as Rodal typed it out above the Navy logo, which is perfect.

So around a circumference there is of course no E field boundary condition, so trying to quantify it in wavelengths makes no sense. Pattern variation does. On the other hand, across a diameter or along the length, standard E field boundary conditions apply, so pattern or half wavelengths works equally well. I'm being super cautious before I sink cash on a large copper sheet, so I'm testing everything.
Title: Re: EM Drive Developments - related to space flight applications - Thread 2
Post by: Rodal on 01/26/2015 12:02 pm
..

I figured out the confusion I had over the m subscript after studying the KWOK 3.01ghz coke can, and the 2.45ghz example. The issue is, wavelengths is NOT the same as wave pattern. For both the frequency examples, you get pretty much exactly 2 wavelengths around the circumference, which breaks TE111. So wavelengths is where I got it wrong. My old n subscript was jacked up too. As the Navy example states, and @Rodal did too. It is wave PATTERN. Glad I got that sorted out. So the Navy example is correct, and then add the p subscript from my old post, and it all falls in to place, as Rodal typed it out above the Navy logo, which is perfect.

So around a circumference there is of course no E field boundary condition, so trying to quantify it in wavelengths makes no sense. Pattern variation does. On the other hand, across a diameter or along the length, standard E field boundary conditions apply, so pattern or half wavelengths works equally well. I'm being super cautious before I sink cash on a large copper sheet, so I'm testing everything.

Thanks again @Mulletron for bringing up this issue and the joint effort. This was a great collaboration.   :)

Here are my planned next steps:

1) discuss the issue of modes in a cylindrical cavity (using the Volumetric Mean) in comparison with the actual modes of a tapered cone (frustum).  I would like to post some information on this.  I need time to post it.  Hopefully we can discuss it during the next few days.

2) Calculate the cyindrical cavity mode shapes for the Shawyer Experimental and Demo based on the Volumetric Mean.  Then post the mode shapes for all the cases: NASA Brady and Shawyer, to compare.

3) Calculate the @NotSoSureOfIt expression for NASA Brady and Shawyer Experimental and Demo and compare it with McCulloch's and Shawyer's calculations.

4) I plan to eventually write a program in Mathematica to analyze the tapered cone, but I'm not going to have time for that until weeks from now at the earliest, depending on how much extra time I get.
Title: Re: EM Drive Developments - related to space flight applications - Thread 2
Post by: JasonAW3 on 01/26/2015 02:38 pm
I was wondering if this phenomena might have an effect on this system?

http://gizmodo.com/sub-atomic-particles-could-accelerate-themselves-1681765188 (http://gizmodo.com/sub-atomic-particles-could-accelerate-themselves-1681765188)

Title: Re: EM Drive Developments - related to space flight applications - Thread 2
Post by: Rodal on 01/26/2015 03:45 pm
...
Concerning "n" (the radial mode number that "indicates the number of half-wave patterns across the diameter"), the Wikipedia link is incorrect -I didn't have the time to check the history of that Wiki entry to see what is the history of that Wikipedia error. When I have the time I will examine that and I will correct the entry in Wikipedia.

...
I made the changes in the Wikipedia page  https://en.wikipedia.org/wiki/Transverse_mode#Types_of_modes  , to correct "wavelength" to "wave pattern" and the expressions for m and n for the circular waveguide. I also added the US NAVY reference. Let's see how long these corrections last in the Wikipedia world where anybody gets to be an Oracle :)
Title: Re: EM Drive Developments - related to space flight applications - Thread 2
Post by: aero on 01/26/2015 06:06 pm
I've attached a video that Tom Ligon made for me some time ago. Can anyone tell me what mode this is, in particular, what is "p" . As for the operating frequency - it is consistent within meep and Harminv but these last few pages of discussion have shaken my confidence that it is consistent with the real world for those cavity dimensions.

The cavity is similar to the Brady cavity in perfect metal, but the dielectric constant is to high.

I don't expect to make a new video until I get the frequencies and modes sorted out.

Edit: add cite to the trailer of video.
Title: Re: EM Drive Developments - related to space flight applications - Thread 2
Post by: Rodal on 01/26/2015 06:41 pm
I've attached a video that Tom Ligon made for me some time ago. Can anyone tell me what mode this is, in particular, what is "p" . As for the operating frequency - it is consistent within meep and Harminv but these last few pages of discussion have shaken my confidence that it is consistent with the real world for those cavity dimensions.

The cavity is similar to the Brady cavity in perfect metal, but the dielectric constant is to high.

I don't expect to make a new video until I get the frequencies and modes sorted out.

1) Only the longitudinal-radial (z, r) cross-sectional area (perpendicular to the circumferential direction) is shown (at a fixed azimuthal angle), and it indicates that p=3 since there are three half-wave patterns along the longitudinal axis.

Quote from: Rodal
The third subscript (p) is the longitudinal mode number.  It indicates the number of half-wave patterns along the longitudinal length of the cavity.  It is zero for modes in which there is no variation in the longitudinal direction.

2) The circular cross-section is not shown in the video and therefore there is no way to tell what the circumferential "m" quantum number of the mode shape is.  One needs a circular-cross section contour plot (the cross-sectional area perpendicular to the longitudinal direction) in order to ascertain what m is, and to make sure what is n.    It would be interesting to supply at least 2 cross-sectional circumferential plots: one located at the longitudinal-direction-middle section of the dielectric and the other one in the empty section of the cavity to compare the field in the dielectric and the rest of the cavity.

3) Based on the limited information given by the sole longitudinal-radial (z, r) cross-section shown, it appears that the radial quantum number n =1 since there is one half-wave pattern along the diameter

Quote from: Rodal
The second subscript (n) is the radial mode number: it indicates the number of half-wave patterns across the diameter.  The radial mode number (n) plus one indicates the number of nodes across the diameter (counting as nodes the end nodes).  The radial mode number (n) minus one indicates the number of middle nodes across the diameter (not counting as nodes the end nodes).  The radial mode number (n) cannot be zero.

4) It is interesting that the two interior nodal lines separating the longitudinal half-wave patterns are spherical even though the ends are flat.  It really shows that the cone mode shapes naturally prefer spherical ends, as in Greg Egan's solution, and as discussed in the 1969 Patent by Grant I posted previously.   The flat ends of the EM Drive degenerate the natural mode shapes of the cone cavity.

5) It would be informative to also see plots of the magnetic field
Title: Re: EM Drive Developments - related to space flight applications - Thread 2
Post by: Notsosureofit on 01/26/2015 08:06 pm
I was wondering if this phenomena might have an effect on this system?

http://gizmodo.com/sub-atomic-particles-could-accelerate-themselves-1681765188 (http://gizmodo.com/sub-atomic-particles-could-accelerate-themselves-1681765188)

I've had a chance to go through these papers over the weekend.  I didn't see anything that would rule it out.

The most encouraging aspect is the inclusion of Rindler-like coordinates  ie.  accelerating frame of reference.

The shape-maintaining requirement is met by the physical cavity.

Still a long way from obvious.
Title: Re: EM Drive Developments - related to space flight applications - Thread 2
Post by: Rodal on 01/26/2015 09:45 pm
******EDIT: This study does not take into account the cutoff frequency condition that eliminates several of these mode shapes.  A new study incorporating the cutoff frequency will be posted *****

MODE SHAPE CALCULATION FOR SHAWYER EXPERIMENTAL AND SHAWYER DEMO

1) We define the Volumetric  Mean as follows:

VolumetricMeanDiameter=Sqrt[(SmallDiameter^2+SmallDiameter*BigDiameter+BigDiameter^2)/3]

For a derivation of the Volumetric Mean (equating the volume of an equivalent cylinder to the volume of a truncated cone): See http://forum.nasaspaceflight.com/index.php?topic=36313.msg1319655#msg1319655




2) Let's define as "Shawyer EXPERIMENTAL geometry" the following estimate of the Shawyer Experimental cavity:

shawyerExpLength               = 0.156 meter;
shawyerExpBigDiameter     = 0.16 meter
shawyerExpSmallDiameter = 0.127546 meter;

then

shawyerExpGeometricMeanDiameter =Sqrt[shawyerExpBigDiameter * shawyerExpSmallDiameter]
                                              = 0.142854 meter

shawyerExpMeanDiameter =( shawyerExpBigDiameter + shawyerExpSmallDiameter)/2
                                              = 0.143773 meter

shawyerExpVolumetricDiameter =Sqrt[(shawyerExpSmallDiameter^2+shawyerExpSmallDiameter*shawyerExpBigDiameter+shawyerExpBigDiameter^2)/3]
                                              = 0.144078 meter

Observe that shawyerExpVolumetricDiameter is just 0.21 % greater than shawyerExpMeanDiameter, and 0.86 % greater than shawyerExpGeometricMeanDiameter.


3) Let's define as " Shawyer DEMO geometry" the following definition for the Shawyer Demo cavity:

shawyerDemoLength               = 0.345 meter;
shawyerDemoBigDiameter     = 0.28 meter;
shawyerDemoSmallDiameter = 0.128853 meter;

then

shawyerDemoGeometricMeanDiameter=Sqrt[shawyerDemoBigDiameter*shawyerDemoSmallDiameter]
                                                  = 0.189944 meter

shawyerDemoMeanDiameter=(shawyerDemoBigDiameter+shawyerDemoSmallDiameter)/2
                                                  = 0.204427 meter

shawyerDemoVolumetricDiameter=Sqrt[(shawyerDemoSmallDiameter^2+shawyerDemoSmallDiameter* shawyerDemoBigDiameter+ shawyerDemoBigDiameter^2)/3]
                                                  = 0.209031 meter

Observe that shawyerDemoVolumetricDiameter is 2.25 % greater than shawyerDemoMeanDiameter, and 10.05 % greater than shawyerDemoGeometricMeanDiameter.  The difference between the mean diameter measures is much greater for Shawyer's Demo than for Shawyer's Experimental because Shawyer's Demo has a greater difference between the small and big diameters.


4) The experimentally reported frequency was

rfFrequency = 2.45*10^9 Hz;

for both Shawyer Demo and for Shawyer Experimental.



5) Given the experimentally reported frequency, the geometrical dimensions and the value of speed of light in air, one can invert the frequency equation (see: http://en.wikipedia.org/wiki/Microwave_cavity#Cylindrical_cavity ) to obtain Xm,n and  X'm values as a function of constants and the longitudinal mode shape number "p". Let's define the error difference between these Xm,n and  X'm,n  values and actual Xm,n and  X'm values as:

error= (value of Xm,n or  X'm,n obtained from frequency eqn.)/ (correct value of Xm,n or  X'm,n ) -1

where Xm,n is used for TM modes and X'm,n is used for TE modes.



6) Then I obtain the following mode shapes and associated errors:


Shawyer EXPERIMENTAL

Geometric Mean

Best result:  TE010 or TM110  error= - 4.253%
2nd best:     TM012                   error= - 5.329%

Volumetric Mean

Best result:  TE010 or TM110  error= - 3.433%
2nd best:     TM012                    error= - 4.518%

Mode 010 ---->  constant around the circumference
                          1 half-wave pattern across the diameter
                          constant along the longitudinal length


Shawyer DEMO

Geometric Mean

Best result:  TE313  error= - 1.673%
2nd best:     TM210 error= - 5.015%

Volumetric Mean

Best result: TE411   error= - 0.6458%
2nd best:    TE410   error= + 0.9534%

Mode 411 ---->  4 full-wave patterns around the circumference
                          1 half-wave pattern across the diameter
                          1 half-wave pattern along the longitudinal length
       



CONCLUSIONS

1) At the frequencies tested by Shawyer, what mode-shape corresponds to a given frequency is very sensitive to the exact geometrical dimensions of the cavity.   The reason for this is that there are many natural frequencies very close to each other, each of these frequencies having different mode shapes.  It would be difficult to predict what mode shape one will get with a given geometry at these frequencies, because small variations in geometry lead to large changes in mode shape.   Therefore a dielectric is needed to force a circumferential-cross-section-mode-shape or moving-end to adjust the length of the cavity.  Certainly it would be very difficult to predict the mode shape of an empty cavity with the coarse finite element model used by NASA Brady et.al. or with a Finite Difference code like MEEP (There are no Bessel functions in a finite element or finite difference model: the solution is approximated with low power piecewise polynomials in each finite element.  The finite element solution is a Galerkin solution "in an integral sense" and not an exact partial differential solution "point to point through the domain".)  Finite Difference solutions (like MEEP) have even more discretization and convergence problems than Finite Element models.

2) Both with the Geometric Mean and with the Volumetric Mean, all Shawyer cases: the Shawyer Experimental and the Shawyer Demo experiments, correspond to transverse electric (TE) mode shapes: the electric field is in the circumferential direction and the magnetic field is perpendicular to it. I think that the TE mode shapes are the ones that should provide thrust  because it is only the TE mode shapes that have the magnetic field directed along the longitudinal direction of the EM Drive.  Physically, an axial magnetic field may result in a measured thrust either 1) as an artifact, because the magnetic field can heat the flat ends of the truncated cone by induction heating and hence produce thermal buckling and other heat effects or/and 2) as a real means of propulsion, by the magnetic field coupling with the Quantum Vacuum, among several possible physical mechanisms.

3) Using the volumetric mean leads to lower errors than the geometric mean.   

4) No dielectric was considered, and it is unknown whether Shawyer used a dielectric and if so what material and geometry of dielectric, for these experiments.  The dielectric will produce an extra longitudinal half wave in the dielectric region of the cavity, and it should reduce the number of full-wave patterns around the circumference, probably making the electric field constant in the circumferential direction for a TE mode, essentially forcing TE41 into TE01.  For the DEMO, Shawyer used an adjustable end, that allowed him (unlike NASA, that did not use adjustable ends) to fine-tune the natural frequency of the cavity.
Title: Re: EM Drive Developments - related to space flight applications - Thread 2
Post by: aero on 01/26/2015 11:24 pm
@Rodel
I believe that your answer is that the mode is

TX a,1,3
   where X = (E or M)
   it could be that a = 1, 2, ... or more.
There must be an upper limit on the value of a, but ...

I agree that more video views would be interesting, but as I wrote, I don't plan to make more until I get the frequency question ironed out. I'm still working that problem but have determined that it is either an issue with Harminv or with my use of Harminv. But it's not geometry or frequency input values, rather it is signal amplitude, run time and wait time issues. Most likely all of them. In the video I posted, it is clear that the cavity is resonating strongly at the drive frequency. The drive frequency is only moderately near the Brady experimental drive frequency. I took that to be a result of the relatively higher dielectric constant used, 2.3 compared to the 1.76 value that gives the Brady experimental value of 1.8804 GHz. But now I'm not confident of that assumption either.

I am looking into ParaView, an open source data visualization tool. With it, I might be able to generate some 3D views of interest but movies are to hard.  Making of that movie took 12 hours of CPU, another 8 hours to upload the PNG files to Box, and I don't know how long to download and make the movie, then send it back. To hard just for simple exploration of the data.
Title: Re: EM Drive Developments - related to space flight applications - Thread 2
Post by: Rodal on 01/26/2015 11:34 pm
@Rodel
I believe that your answer is that the mode is

TX a,1,3
   where X = (E or M)
   it could be that a = 1, 2, ... or more.
There must be an upper limit on the value of a, but ...

...

Yes, except:

TX m,1,3

with m=0,1, ...  (the lowest possible m value is 0 meaning constant field in the circumferential direction)

I think that X is most likely to be E (because you have a dielectric in the cavity) and that m is low (because you have a dielectric in the cavity).

So I would bet that it is most likely TE013 (but we would need to see the circular cross-section contour plot to be sure what is "m")

Thank you for the excellent update on your progress  :)

Godspeed  :)


Title: Re: EM Drive Developments - related to space flight applications - Thread 2
Post by: Rodal on 01/26/2015 11:39 pm
.....

I am looking into ParaView, an open source data visualization tool. With it, I might be able to generate some 3D views of interest but movies are to hard.  Making of that movie took 12 hours of CPU, another 8 hours to upload the PNG files to Box, and I don't know how long to download and make the movie, then send it back. To hard just for simple exploration of the data.

Absolutely, we do NOT need a movie, we only need the very last still picture, since we are only interested in the steady-state standing-wave.   

(http://static-p2.photoxpress.com/jpg/00/05/25/40/400_F_5254097_d5Zs7X8V9fTEduGJtCKJk31PZVejXWP0_PXP.jpg)


But it was fun, enjoyable to see the movie of how MEEP proceeds from the transient waves in both directions to finally achieve a standing wave -- very educational (it could be used to teach students how standing waves get formed)  :)
Title: Re: EM Drive Developments - related to space flight applications - Thread 2
Post by: Rodal on 01/27/2015 12:00 am
...'m still working that problem but have determined that it is either an issue with Harminv or with my use of Harminv. But it's not geometry or frequency input values, rather it is signal amplitude, run time and wait time issues. Most likely all of them. In the video I posted, it is clear that the cavity is resonating strongly at the drive frequency. The drive frequency is only moderately near the Brady experimental drive frequency. I took that to be a result of the relatively higher dielectric constant used, 2.3 compared to the 1.76 value that gives the Brady experimental value of 1.8804 GHz. ...

I wonder about the reason why you get much better results for the truncated cone case with the dielectric (compared to experimental frequency) than with the cylindrical cavity with no dielectric (compared to exact solution).  I wonder whether the reason for this is that the matrix that needs to be inverted in MEEP is numerically ill-conditioned (because there are many mode shapes very close together, actually the frequencies for mode shapes TE01p and TM11p are identical ! ) and that placing the dielectric makes the matrix much better numerically conditioned (the dielectric forces the mode into TE and low values for m and n ).  So, for different reasons (numerical conditioning of the matrix to be inverted in the case of MEEP and fine-tuning frequency and mode-shape in the case of the real experiments) the dielectric may be beneficial both for the numerical solution and for the experiment. 

Example of an ill-conditioned matrix:

(http://www.phy.ornl.gov/csep/CSEP/BF/IMG245.GIF)
Title: Re: EM Drive Developments - related to space flight applications - Thread 2
Post by: RotoSequence on 01/27/2015 11:24 am
This discovery for cavity cooling of quantum oscillators looks interesting. Could it be relative to the subject of EM drives?

http://physics.aps.org/articles/v8/8

Quote
Cavity optomechanics studies the interaction between light and mechanical systems, most often mediated by radiation pressure—the force exerted by photons hitting on a mirror. In a typical optomechanics setup, microwave or optical photons travel in a cavity formed by two or more mirrors, one of which is free to move and acts like a mechanical oscillator whose position changes because of radiation pressure and thermal fluctuations. A recent milestone in the field was the cavity cooling of mechanical oscillators to a regime in which they have less than one quantum of motion [1]. This could have important applications, ranging from the observation of quantum behavior in macroscopic systems to the development of ultrasensitive detectors that could reveal the tiny vibrations caused by the still-elusive gravitational waves. However, to-date-demonstrated schemes require that the frequency of the cooling light be lower than the cavity resonant frequency by an amount corresponding to the frequency of the mechanical oscillator. These two frequencies can be measurably different only if the oscillator’s frequency is sufficiently large. This poses an important limitation, as low frequency, heavy oscillators cannot be cooled by this method.

Now a team led by Roman Schnabel at the Albert Einstein Institute in Hannover, Germany has reported an optomechanical cooling scheme that gets rid of this frequency-detuning requirement [2]. The key to their achievement is a novel form of optomechanical coupling. Optomechanics has been mainly investigated using “dispersive coupling,” in which displacements of the oscillating mirror change the resonant frequency of the cavity [3]. Here, the authors rely instead on “dissipative coupling,” in which the mirror oscillations modify the coupling between the cavity and its environment, for instance by modulating the speed at which cavity photons are lost (i.e., the cavity bandwidth).

In the dispersive coupling regime, the most widely used cooling approach is “cavity cooling” [3]. In this method, light cools a mechanical mode via a “parametric” process: quanta of mechanical excitations are up-converted into cavity photons, which are then dissipated through the cavity. The light frequency must be detuned from the cavity resonant frequency: the mechanical oscillator produces lower-frequency (red) and higher-frequency (blue) sidebands, which are shifted from the cavity resonance by multiples of the mechanical frequency ωM. When the driving frequency is tuned to the first red sideband, photons entering the cavity take away phonons with energy ħωM from the mechanical system, cooling the oscillator. In a number of previous studies, this method allowed researchers to bring mechanical oscillators to their quantum ground state (i.e., states in which the average number of mechanical excitations is reduced below unity) [1] and to realize the coherent conversion of photons to mechanical motion and of microwave photons to optical photons [4]. But these realizations required the system to be in the “resolved sideband regime”: for the red sideband to be distinguishable from the cavity frequency, the frequency of the mechanical oscillator (determining the shift of the sidebands) has to be larger than the cavity bandwidth. Ground-state cooling has been achieved for frequencies down to hundreds of megahertz, but reaching lower frequencies (such as the ∼100-kilohertz frequency of the mechanical resonator used by Schnabel’s team) would require cavities with extremely narrow bandwidths, which are currently not available.

However, a recent theory shows that cavity cooling could be possible without the requirement to resolve the mechanical-oscillator sidebands [5] if the optomechanical coupling is dissipative [6]. In dissipatively coupled systems, two types of fluctuating forces act on the mechanical resonator: the noise in the light injected into the cavity and the quantum fluctuation of the cavity field. These two types of noise, which have different spectra, can interfere destructively. With proper parameter choice, such interference can reduce the spectral density of the noise at the frequency (-ωM) of the oscillator, effectively cooling it. This mechanism, which does not require to resolve the sidebands, can work with a low oscillator frequency.
Title: Re: EM Drive Developments - related to space flight applications - Thread 2
Post by: Rodal on 01/27/2015 05:28 pm
...

I have made progress though. Using the frequency formula to adjust cavity length so that the formula gave 2.45GHz, required a cavity length about  0.0965 meters. Iteratively running meep and adjusting the cavity length to force resonance at 2.45 GHZ independently produced a cavity length of 0.0936 meters.

I emphasize that I worked those problems independently of each other so the fact that they are in near agreement is telling. I would like to find a combination length and radius that would give the frequency without the length being so close to 3/4 wavelength. In fact, I would like for the length to be exactly one wavelength and for which I knew the mode. TE 1,1,1 would be good, but TE 1,4,1 might also work. Maybe now that I have found one solution, I can find more solutions.
For:

MeepDiameter = 7.54898 centimeter
MeepLength = 12.2364 centimeter
cAir= (29970500000 centimeter)/second

The exact solution first few mode-shapes and frequencies are:

{{"TE", 1, 1, 0}, 2.32677*10^9},
{{"TE", 1, 1, 1}, 2.62937*10^9},
{{"TM", 0, 1, 0}, 3.03906*10^9},


So that TE111 has a frequency of 2.63 GHz, as @Mulletron and I compute.  Therefore I don't understand why you are inputing an excitation frequency of 2.45 GHz, which does not correspond to any natural frequency, and moreover sits between two natural frequencies: TE110 (2.33 GHz) and TE111 (2.63 GHz).
Your excitation frequency of 2.45 GHz is closer to TE110 than to the mode you said you wanted to excite: TE111.



For:

MeepDiameter = 7.54898 centimeter
MeepLength = 9.65  centimeter
cAir= (29970500000 centimeter)/second

The exact solution first few mode-shapes and frequencies are:

{{"TE", 1, 1, 0}, 2.32677*10^9},
{{"TE", 1, 1, 1}, 2.79737*10^9},
{{"TM", 0, 1, 0}, 3.03906*10^9},

By decreasing the cavity length, you made your excitation frequency of 2.45 GHz further away from the mode you said you wanted to excite (TE111).


For:

MeepDiameter = 7.54898 centimeter
MeepLength = 9.36 centimeter
cAir= (29970500000 centimeter)/second

The exact solution first few mode-shapes and frequencies are:

{{"TE", 1, 1, 0}, 2.32677*10^9},
{{"TE", 1, 1, 1}, 2.82436*10^9},
{{"TM", 0, 1, 0}, 3.03906*10^9},

By decreasing the cavity length even further, you made your excitation frequency of 2.45 GHz even further away from the mode you said you wanted to excite (TE111).



As you decrease the length , the frequency of mode shapes TE110 and TM010 remain exactly the same (2.33 GHz and 3.04 GHz respectively) while the frequency of TE111 increases from 2.63 GHz to 2.82 GHz, so that your excitation frequency of 2.45 GHz remains at the same distance from TE110 but is further and further away from TE111 as you decrease the length.

I'm puzzled as to why you are using an excitation frequency of 2.45 GHz which does not correspond to any natural frequency of the cavity.  If you want to excite TE110 you should use an excitation frequency of  2.32677 GHz (using the speed of light in air, while if you use the speed of light in vacuum it would be 2.32745 GHz).   The natural frequency of mode shape TE110, 2.33 GHz, is independent of the length of the cavity.

If you want to excite mode TE111, which does depend on the cavity length, you should use an excitation frequency significantly higher, as shown above, and as indicated by @Mulletron and me.  Furthermore, if you decrease the cavity length, the TE111 natural frequency increases, so you have to increase the excitation frequency to correspond to it, if you want to excite TE111, otherwise, decreasing the cavity length keeping the excitation frequency constant 2.45GHz should make it more difficult to excite TE111 (rather than easier, which is the opposite of what you wrote you were looking for).
Title: Re: EM Drive Developments - related to space flight applications - Thread 2
Post by: Mulletron on 01/27/2015 06:14 pm
I found this very interesting (expired) 1969 patent by Grant, owned by JOHNSON CONTROLS INTERNATIONAL, that deals with conical cavities !

http://www.google.com/patents/US3425006

It has a lot of interesting practical stuff, for example dealing with spherical ends (like in Greg Egan's solution  http://gregegan.customer.netspace.net.au/SCIENCE/Cavity/Cavity.html  ) instead of flat end plates (NASA Brady et.al., Shawyer(Experimental and Demo) and Prof. Juan Yang in China's EM Drives all have flat ends):


Found some time to read over this patent. It is a gold mine of good info. Great find. A couple takeaways I found is that it confirms that TE modes are highly desirable compared to TM. Also this caught my eye:
Quote
It has been found possible to predict the resonances approximately by defining a phase shift per unit length as 21r/ \g, where Ag is given by the usual formula for circular wave-guides of diameter D, but where D and hence Ag vary along the cone. If this phase shift is integrated from the location of the cut-off diameter to the position of the plunger or movable end wall, resonances will be found when the integral has values of 11 pi."
Looks like math for predicting resonant modes for cones. It looks like some of the text got messed up in the character translation over to Google patents, see the bold part.
Title: Re: EM Drive Developments - related to space flight applications - Thread 2
Post by: Rodal on 01/27/2015 06:23 pm
I found this very interesting (expired) 1969 patent by Grant Wolf, owned by JOHNSON CONTROLS INTERNATIONAL, that deals with conical cavities !

http://www.google.com/patents/US3425006

It has a lot of interesting practical stuff, for example dealing with spherical ends (like in Greg Egan's solution  http://gregegan.customer.netspace.net.au/SCIENCE/Cavity/Cavity.html  ) instead of flat end plates (NASA Brady et.al., Shawyer(Experimental and Demo) and Prof. Juan Yang in China's EM Drives all have flat ends):


Found some time to read over this patent. It is a gold mine of good info. Great find. A couple takeaways I found is that it confirms that TE modes are highly desirable compared to TM. Also this caught my eye:
Quote
It has been found possible to predict the resonances approximately by defining a phase shift per unit length as 21r/ \g, where Ag is given by the usual formula for circular wave-guides of diameter D, but where D and hence Ag vary along the cone. If this phase shift is integrated from the location of the cut-off diameter to the position of the plunger or movable end wall, resonances will be found when the integral has values of 11 pi."
Looks like math for predicting resonant modes for cones. It looks like some of the text got messed up in the character translation over to Google patents, see the bold part.

Glad you agree that this 1969 patent is a gold mine for people interested in EM Drives.

Please see  the attached Adobe Acrobat .pdf file of page 3 of the original patent, top of column 4, for the actual formulas and symbols

The patent states

"n * Pi" instead of "11 * Pi"

"2 * Pi / Lambdag " instead of b]21r/ \g[/b] 

"Lambdag" instead of  "Ag " where Lambdag must mean the waveguide's wavelength
Title: Re: EM Drive Developments - related to space flight applications - Thread 2
Post by: Rodal on 01/27/2015 07:49 pm
....
Found some time to read over this patent. It is a gold mine of good info. Great find. A couple takeaways I found is that it confirms that TE modes are highly desirable compared to TM. Also this caught my eye:
Quote
It has been found possible to predict the resonances approximately by defining a phase shift per unit length as 21r/ \g, where Ag is given by the usual formula for circular wave-guides of diameter D, but where D and hence Ag vary along the cone. If this phase shift is integrated from the location of the cut-off diameter to the position of the plunger or movable end wall, resonances will be found when the integral has values of 11 pi."
Looks like math for predicting resonant modes for cones. It looks like some of the text got messed up in the character translation over to Google patents, see the bold part.

Glad you agree that this 1969 patent is a gold mine for people interested in EM Drives.

Please see  the attached Adobe Acrobat .pdf file of page 3 of the original patent, top of column 4, for the actual formulas and symbols

The patent states

"n * Pi" instead of "11 * Pi"

"2 * Pi / Lambdag " instead of b]21r/ \g[/b] 

"Lambdag" instead of  "Ag "  where Lambdag must mean the waveguide's wavelength

I attach the Adobe Acrobat .pdf file for the whole patent

Publication number   US3425006 A
Publication date   Jan 28, 1969
Filing date   Feb 1, 1967
Priority date   Feb 1, 1967
Inventors           Wolf James M
Original Assignee   Johnson Service Co
Title: Re: EM Drive Developments - related to space flight applications - Thread 2
Post by: aero on 01/27/2015 09:40 pm
Quote
I'm puzzled as to why you are using an excitation frequency of 2.45 GHz which does not correspond to any natural frequency of the cavity.  If you want to excite TE110 you should use an excitation frequency of  2.32677 GHz (using the speed of light in air, while if you use the speed of light in vacuum it would be 2.32745 GHz).   The natural frequency of mode shape TE110, 2.33 GHz, is independent of the length of the cavity.

It's very simple. 2.45 GHz is a given.
Cavity length and radius are the independent variables to be adjusted to establish resonance at 2.45 GHz.
And yes, I'm quite sure I want TE 1,1,1 mode.
Title: Re: EM Drive Developments - related to space flight applications - Thread 2
Post by: Rodal on 01/27/2015 10:35 pm
Quote
I'm puzzled as to why you are using an excitation frequency of 2.45 GHz which does not correspond to any natural frequency of the cavity.  If you want to excite TE110 you should use an excitation frequency of  2.32677 GHz (using the speed of light in air, while if you use the speed of light in vacuum it would be 2.32745 GHz).   The natural frequency of mode shape TE110, 2.33 GHz, is independent of the length of the cavity.

It's very simple. 2.45 GHz is a given.
Cavity length and radius are the independent variables to be adjusted to establish resonance at 2.45 GHz.
And yes, I'm quite sure I want TE 1,1,1 mode.

If you insist in specifying the exciting frequency as 2.45GHz and the diameter of the cylindrical cavity, and having the length as the variable to be adjusted, then you are going the wrong way in reducing the length !

The inverted formula for length in terms of the other variables is simply:

length = pNr/(2 Sqrt[( fr/cMediumNr)^2 - ( xbesselNr/([Pi]*diameterNr))^2])

If you specify:

diameterNr = 7.54898 centimeter
cMediumNr = cAir = (29970500000 centimeter)/second
fr = 2.45*10^9/second

and for TE111 you must have:

xbesselNr = X'11 = 1.84118378134065
pNr = p = 1

Then, this results in a length of

length = pNr/(2 Sqrt[( fr/cMediumNr)^2 - ( xbesselNr/(\[Pi]*diameterNr))^2])
            = 1 /(2 Sqrt[(2.45*10^9/second/ (29970500000 centimeter)/second)^2 - (1.84118378134065/([Pi]* 7.54898 centimeter))^2])
             = 19.531439054546873` centimeters

instead of 12.2364 centimeter.


Conversely, the following inputs

diameter = 7.54898 centimeter
length=19.531439054546873` centimeters
cMedium = cAir = (29970500000 centimeter)/second

give the following lowest modes and frequencies

{{"TE", 1, 1, 0}, 2.32677*10^9},
{{"TE", 1, 1, 1},  2.45*10^9},
{{"TE", 1, 1, 2}, 2.7872*10^9},
{{"TM", 0, 1, 0}, 3.03906*10^9},

which fully verifies that length= 19.531439054546873` centimeters gives a frequency of 2.45 GHz and mode shape TE111, for a diameter = 7.54898 centimeter, in air.



But instead of inputing  the correct length (for TE111) that is length=19.5314 centimeters, first you inputed the incorrect length of 12.2364 centimeter, which was much smaller than the correct length, but then proceeded to make it worse by reducing it even further to  9.36 centimeter which does not make sense.  You should have input length=19.5314 centimeters to get TE111     :)



But in general, it is inadvisable to pose the problem by specifying the frequency, diameter and mode shape to calculate the length, because you are then ill-posing the problem.  If the frequency you specify is low enough, there will be no real length solution.

The lowest frequency you can specify is

fr= (cMediumNr* xbesselNr/([Pi]*diameterNr))

which will result in an infinite length !
Title: Re: EM Drive Developments - related to space flight applications - Thread 2
Post by: aero on 01/27/2015 11:17 pm
Isn't it possible to increase radius while leaving length at or around 12 cm? I would prefer that but can't find a radius that works with 2.45 GHz.
Title: Re: EM Drive Developments - related to space flight applications - Thread 2
Post by: Rodal on 01/27/2015 11:34 pm
Isn't it possible to increase radius while leaving length at or around 12 cm? I would prefer that but can't find a radius that works with 2.45 GHz.

Yes, it's possible. 

If you insist in specifying the exciting frequency as 2.45GHz and the length of the cylindrical cavity, and having the diameter as the variable to be adjusted, then

The inverted formula for diameter in terms of the other variables is, trivially:

diameter= (2   xbesselNr)/([Pi] Sqrt[4 (fr/cMediumNr)^2 - (pNr/lengthNr)^2])

If you specify:

lengthNr = 12 centimeter (Please notice you asked 12 cm instead of the previous 12.2364 cm)
cMediumNr = cAir = (29970500000 centimeter)/second
fr = 2.45*10^9/second

and for TE111 you must have:

xbesselNr = X'11 = 1.84118378134065
pNr = p = 1

Then, this results in a diameter of

length =(2   xbesselNr)/([Pi] Sqrt[4 (fr/cMediumNr)^2 - (pNr/lengthNr)^2])
            = (2    1.84118378134065)/([Pi] Sqrt[4 (2.45*10^9/second/29970500000 centimeter)/second)^2 - (1/(12 centimeter))^2])
             = 8.332965999678832` centimeters

instead of  the diameter=7.54898 centimeter you used.


Conversely, the following inputs

diameter = 8.332965999678832`  centimeter
length=12 centimeters
cMedium = cAir = (29970500000 centimeter)/second

give the following lowest modes and frequencies

{{"TE", 1, 1, 0}, 2.10786*10^9},
 {{"TE", 1, 1, 1}, 2.45*10^9},
{{"TM", 0, 1, 0}, 2.75314*10^9},
{{"TM", 0, 1, 1}, 3.02311*10^9},

which fully verifies that diameter = 8.332965999678832`  centimeter gives a frequency of 2.45 GHz and mode shape TE111, for a length=12 centimeters, in air.

quod erat demonstrandum  :)



But in general, it is inadvisable to pose the problem by specifying the frequency, length and mode shape to calculate the diameter, because you are then ill-posing the problem.  If the frequency you specify is low enough, there will be no real diameter solution.

The lowest frequency (for a specified length) you can specify is

fr= cMediumNr*(pNr/lengthNr) /2

which will result in an infinite diameter!




We have now looked at this three different ways, and all these ways show that you inputed the incorrect parameters to best excite a TE111 natural frequency.
Title: Re: EM Drive Developments - related to space flight applications - Thread 2
Post by: Rodal on 01/28/2015 12:23 am
If you specify:

lengthNr = 12.23642686` centimeter
cMediumNr = cAir = (29970500000 centimeter)/second
fr = 2.45*10^9/second

and for TE111 you must have:

xbesselNr = X'11 = 1.84118378134065
pNr = p = 1

Then, this results in a diameter of  8.277559638179381` centimeter;

Conversely, the following inputs

diameter = 8.277559638179381`  centimeter
length=12.23642686` centimeters
cMedium = cAir = (29970500000 centimeter)/second

give the following lowest modes and frequencies

{{"TE", 1, 1, 0}, 2.12197*10^9},
{{"TE", 1, 1, 1},  2.45*10^9},
{{"TM", 0, 1, 0}, 2.77157*10^9},
{{"TM", 0, 1, 1}, 3.03007*10^9},

quod erat demonstrandum   :)
Title: Re: EM Drive Developments - related to space flight applications - Thread 2
Post by: Mulletron on 01/28/2015 09:46 am
Well according the above posts and others, http://forum.nasaspaceflight.com/index.php?topic=36313.msg1320981#msg1320981 we're all getting pretty adept at using our skills/resources for calculating resonant modes of cylinders, which is a good first step. Well I'm learning new skills as I go along. So I can do cylinders all day, cones...not so much..yet. I certainly didn't come to the table knowing how to calculate resonant modes of anything 5-6 months ago.

I'm trying to figure out how to use the (2 * Pi ) / (Lambda * g) expression from the patent or the Volumetric Mean approach (which is better?) toward calculating resonant modes of conical frustums. I think the holy grail would be a quick and easy correction to convert from cylinder solutions to conical frustums.

So the (2 * Pi ) / (Lambda * g) above, does that mean what when I take 6.28 and divide that by x wavelength, and get a multiple of pi, that diameter or can support a resonant mode?

Trying to figure out how to convert this cylinder to a cone, by keeping the diameter as the small diameter, adjusting the length to arrive at the new large diameter along a 45 or 90 degree cone, and still maintain resonance @ 2.45ghz TE111:
2.45ghz, TE111
L=0.1224489m
D=0.08278945m
Title: Re: EM Drive Developments - related to space flight applications - Thread 2
Post by: Rodal on 01/28/2015 12:06 pm
Well according the above posts and others, http://forum.nasaspaceflight.com/index.php?topic=36313.msg1320981#msg1320981 we're all getting pretty adept at using our skills/resources for calculating resonant modes of cylinders, which is a good first step. Well I'm learning new skills as I go along. So I can do cylinders all day, cones...not so much..yet. I certainly didn't come to the table knowing how to calculate resonant modes of anything 5-6 months ago.

I'm trying to figure out how to use the (2 * Pi ) / (Lambdag ) expression from the patent or the Volumetric Mean approach (which is better?) toward calculating resonant modes of conical frustums. I think the holy grail would be a quick and easy correction to convert from cylinder solutions to conical frustums.

So the (2 * Pi ) / (Lambdag ) above, does that mean what when I take 6.28 and divide that by x wavelength, and get a multiple of pi, that diameter or can support a resonant mode?

Trying to figure out how to convert this cylinder to a cone, by keeping the diameter as the small diameter, adjusting the length to arrive at the new large diameter along a 45 or 90 degree cone, and still maintain resonance @ 2.45ghz TE111:
2.45ghz, TE111
L=0.1224489m
D=0.08278945m

Regarding the Edit of http://forum.nasaspaceflight.com/index.php?topic=36313.msg1320981#msg1320981, glad  :) that we also agree that the diameter should be 8.28 centimeters for that length and TE111 frequency.

As to the comparison between the longitudinal integration (of D and lambdag expressed as a function of the lengthwise coordinate of the cone) as suggested by Wolf's 1969 patent in comparison with the Volumetric Mean approach I have to run the calculations when I get a chance.  But I  first observe that the patent's author (Wolf) admits that the suggested approach is unable to obtain any TM mode that is constant in the length direction: TMmn0 , (notice that he uses indices lmn instead of mnp).  I have some reservations that this integration approach offers any benefit over the Volumetric Mean approach but I will keep an open mind until I get a chance to compare.  [Note: I corrected my nomenclature to lambdag to better show that lambdag means the cylindrical-waveguide's wavelength]

Meanwhile,  I would particularly appreciate @NotSoSureOfIt 's comments regarding Wolf's suggestion on how to calculate the resonances of a truncated cone cavity (lines 60 to 75 of column 3 and lines 1 to 11 of column 4) of the US patent #3,425,006 (which I attach below as an Adobe Acrobat .pdf document).

Publication number   US3425006 A
Publication date   Jan 28, 1969
Filing date   Feb 1, 1967
Priority date   Feb 1, 1967
Inventors           Wolf James M
Original Assignee   Johnson Service Co
Title: Re: EM Drive Developments - related to space flight applications - Thread 2
Post by: Rodal on 01/28/2015 01:46 pm
..

So the (2 * Pi ) / (Lambdag) above, does that mean what when I take 6.28 and divide that by x wavelength, and get a multiple of pi, that diameter or can support a resonant mode?

...

Not precisely.  Instead the patent's author (Wolf) means that resonance occurs when the definite Integral of (2 Pi / lambdag) taken along the lengthwise coordinate of the cone is an integer multiple of Pi (where the cylindrical-waveguide's wavelength lambdag and diameter D are expressed as functions of the lengthwise cone coordinate, under the integral sign).

Since Pi is a constant under the integral, it seems to me that it is simpler to express this as follows: resonance occurs when the definite Integral of (2 / lambdag) taken along the lengthwise coordinate of the cone is an integer multiple of unity: 1, 2, 3, 4, 5, ....(where the cylindrical-waveguide's wavelength lambdag and diameter D are expressed as functions of the lengthwise cone coordinate, under the integral sign).
Title: Re: EM Drive Developments - related to space flight applications - Thread 2
Post by: Rodal on 01/28/2015 02:27 pm
Isn't it possible to increase radius while leaving length at or around 12 cm? I would prefer that but can't find a radius that works...
That's another reason why to compare a numerical solution (by Finite Difference, Finite Element or any other numerical method that relies on a mesh to obtain results) to an exact solution, you should first pick dimensions, discretize the problem to a fine mesh to ensure convergence (this is what takes most of the person's time in a numerical solution) and if necessary, explore different exciting frequencies (keeping dimensions constant, instead of keeping the same excitation frequency and changing dimensions, which means changing the mesh).  For example using the exciting frequency of 2.63GHz for TE111 (for MeepDiameter = 7.54898 centimeter, MeepLength = 12.2364 centimeter) as obtained by @Mulletron and me.

To insist on a fixed excitation frequency for a given mode (like TE111) means that you will have to change dimensions (and hence discretize with a mesh again with the correct dimension that is associated with a natural frequency) if there are no natural frequencies for the frequency you are insisting on, and this only if there is a finite dimension for the frequency you are insisting to get (see previous posts for minimum natural frequency for a given diameter or a given length).
Title: Re: EM Drive Developments - related to space flight applications - Thread 2
Post by: Rodal on 01/28/2015 02:37 pm
...

What did you use for the bandwidth source around the frequency of interest (Drive frequency 2.45 E+9 Hz)?

Could you try running all these cases again, everything the same as before except with a significantly narrower bandwidth source around the frequency of interest ?  .  Reportedly harminv does a better job the narrower the source is around the frequency of interest .
Ok, I did run it again with bandwidth = 0.2 * Drive frequency, for cases up to resolution of 80, but I didn't get anything. Once I narrow the bandwidth to exclude the resonant frequency at 1.87 GHz, there are no resonances within the bandwidth.
Harminv does work better at identifying the resonant frequency with narrower bandwidth, when the frequency is within the bandwidth. I set the drive frequency to 1.873 GHz, narrowed the bandwidth to 0.07 * frequency and got this:

frequency                             Quality factor               error
1,873,339,229.3075 Hz   18,325,307.0778158    1.673972608680621e-7+0.0i

....


A bandwidth of 0.2 * Drive frequency is way too large for the cases you are trying to match.  Look at my message http://forum.nasaspaceflight.com/index.php?topic=36313.msg1322181#msg1322181 for example.

The exact solutions for the lowest modes with those cavity dimensions are:

{{"TE", 1, 1, 0}, 2.12197*10^9},
{{"TE", 1, 1, 1},  2.45*10^9},
{{"TM", 0, 1, 0}, 2.77157*10^9},
{{"TM", 0, 1, 1}, 3.03007*10^9},

Therefore the bandwidth between {{"TE", 1, 1, 1},  2.45*10^9} and {{"TM", 0, 1, 0}, 2.77157*10^9} is

( 2.77157*10^9 - 2.45*10^9) / 2.45*10^9 = 0.13

hence the bandwidth separating those modes is  0.13 * Drive frequency  which is much smaller than the bandwith of 0.2 * Drive frequency you first used (with Drive frequency = 2.45*10^9).

This means that you should first calculate the exact natural frequencies around the frequency you are interested in, and therefore the bandwidth between them.  After that input in Meep an even smaller bandwidth for the excitation frequency (as shown in the above example)  :).  In this case it means that you should input in Meep a bandwidth smaller than 0.13 * Drive frequency

Title: Re: EM Drive Developments - related to space flight applications - Thread 2
Post by: aero on 01/28/2015 02:59 pm
Ok, I'll try that.
I did correct my analytical formula spread sheet while verifying your formula, and now my analytical formula calculations agree with yours in all respects.
Still need agreement from meep/Harminv however.
Title: Re: EM Drive Developments - related to space flight applications - Thread 2
Post by: Notsosureofit on 01/29/2015 12:12 am

Meanwhile,  I would particularly appreciate @NotSoSureOfIt 's comments regarding Wolf's suggestion on how to calculate the resonances of a truncated cone cavity (lines 60 to 75 of column 3 and lines 1 to 11 of column 4) of the US patent #3,425,006 (which I attach below as an Adobe Acrobat .pdf document).

Publication number   US3425006 A
Publication date   Jan 28, 1969
Filing date   Feb 1, 1967
Priority date   Feb 1, 1967
Inventors           Wolf James M
Original Assignee   Johnson Service Co

Mmmm..  That's the argument that I used to come up w/ "volumetric".  Been otherwise occupied, but I'll take a look at it as time permits.

Edit:   Lessee, that's the "guide" wavelength, for equal phase planes.

Basically, you want to solve for the k[sub z] w/ R as a function of z.

like k[sub z]^2 = (omega/c)^2- (X[sub m,n]/R[fn z])^2    or X'

Title: Re: EM Drive Developments - related to space flight applications - Thread 2
Post by: Rodal on 01/29/2015 12:16 pm

Meanwhile,  I would particularly appreciate @NotSoSureOfIt 's comments regarding Wolf's suggestion on how to calculate the resonances of a truncated cone cavity (lines 60 to 75 of column 3 and lines 1 to 11 of column 4) of the US patent #3,425,006 (which I attach below as an Adobe Acrobat .pdf document).

Publication number   US3425006 A
Publication date   Jan 28, 1969
Filing date   Feb 1, 1967
Priority date   Feb 1, 1967
Inventors           Wolf James M
Original Assignee   Johnson Service Co

Mmmm..  That's the argument that I used to come up w/ "volumetric".  Been otherwise occupied, but I'll take a look at it as time permits.

Edit:   Lessee, that's the "guide" wavelength, for equal phase planes.

Basically, you want to solve for the k[sub z] w/ R as a function of z.

like k[sub z]^2 = (omega/c)^2- (X[sub m,n]/R[fn z])^2    or X'
Yes, thank you for looking at it, and formulating the eigenvalue problem :). 

I get an interesting closed-form expression containing an ArcTan term for that integration, so, it looks like it should give different results than the cylindrical cavity formula based on any mean diameter measure like the Mean, Geometric Mean or the Volumetric Mean.  I will double check my closed-form integration and run some numbers when I have time available, to see what difference it makes. 

It is interesting that the patent's author (Wolf) did not actually give the resulting closed-form integral but he just hinted at how to solve the eigenvalue problem for the cone :).
Title: Re: EM Drive Developments - related to space flight applications - Thread 2
Post by: Stormbringer on 01/29/2015 01:03 pm
 ;D

Well inventors often try to deliberately hide stuff in their schematics. Even Leonardo Da Vinchi did (His tank and Orinthopter thing for example) And of course the alchemists  and later chemists did. There is an ancient time honored tradition of trying to screw your would be successors and possible competitors over by booby trapping your paper records.

Title: Re: EM Drive Developments - related to space flight applications - Thread 2
Post by: Rodal on 01/29/2015 01:09 pm
https://en.wikipedia.org/wiki/Ornithopter

(http://media.web.britannica.com/eb-media/40/96240-004-A9CAD60A.jpg)

(https://upload.wikimedia.org/wikipedia/commons/4/41/Skybird.gif)
Title: Re: EM Drive Developments - related to space flight applications - Thread 2
Post by: Rodal on 01/29/2015 06:52 pm
******EDIT: This study does not take into account the cutoff frequency condition that eliminates several of these mode shapes.  A new study incorporating the cutoff frequency will be posted *****


MODE SHAPE STUDY of NASA Brady et.al.'s EXPERIMENTS according to different assumed GEOMETRIES

1) We conduct a thorough study of the mode-shapes of the experiments in NASA Brady et.al.'s " Anomalous Thrust Production ..." report  (http://www.libertariannews.org/wp-content/uploads/2014/07/AnomalousThrustProductionFromanRFTestDevice-BradyEtAl.pdf) , taking into account @Mulletron's assumed geometry as well as the extra experiment conducted at 2168 Mhz.



2) The experimental data can be found in p .18, Table 2. Tapered Cavity Testing :  Summary of Results and in the section on p .18, F.Tapered Cavity RF Evaluation, General Findings and Lessons Learned; of (http://www.libertariannews.org/wp-content/uploads/2014/07/AnomalousThrustProductionFromanRFTestDevice-BradyEtAl.pdf )



3) We define the Volumetric  Mean as follows:

VolumetricMeanDiameter=Sqrt[(SmallDiameter^2+SmallDiameter*BigDiameter+BigDiameter^2)/3]

For a derivation of the Volumetric Mean (equating the volume of an equivalent cylinder to the volume of a truncated cone): See http://forum.nasaspaceflight.com/index.php?topic=36313.msg1319655#msg1319655



4) Let's define as "Mulletron geometry" the following definition for the NASA Brady et. al. cavity:

Mulletron Best estimate as of 11/9/2014   
http://forum.nasaspaceflight.com/index.php?topic=36313.msg1320903#msg1320903
   
cavityLength = 0.27637 m
bigDiameter = 0.30098 m
smallDiameter = 0.15875 m

then

VolumetricMeanDiameter=Sqrt[(SmallDiameter^2+SmallDiameter*BigDiameter+BigDiameter^2)/3]
                                       = 0.2335031034055008` meter



5) Let's define as "Aero geometry" the following definition for the NASA Brady et. al. cavity:

Aero Best estimate as of 11/9/2014    http://forum.nasaspaceflight.com/index.php?topic=29276.msg1285896#msg1285896
   
cavityLength = 0.24173 m
bigDiameter = 0.27246 m
smallDiameter = 0.15875 m

then

VolumetricMeanDiameter=Sqrt[(SmallDiameter^2+SmallDiameter*BigDiameter+BigDiameter^2)/3]
                                       = 0.21808946107809366` meter

6) Let's define as "Fornaro geometry" the following definition for the NASA Brady et. al. cavity:

Fornaro estimate    http://forum.nasaspaceflight.com/index.php?topic=36313.msg1302455#msg1302455
   
cavityLength = 0.332 m
bigDiameter = 0.397 m
smallDiameter = 0.244 m

then

VolumetricMeanDiameter= Sqrt[(SmallDiameter^2+SmallDiameter*BigDiameter+BigDiameter^2)/3]
                                       = 0.32352897860933574` centimeter

7) These are the experimentally reported frequencies and the COMSOL-calculated mode shapes:

frequencyBradyA =  1.9326*10^9 1/second; TM211
frequencyBradyB =  1.9367*10^9 1/second; TM211
frequencyBradyC =  1.8804*10^9 1/second; TE012
frequencyBradyD =  2.168*10^9   1/second; TE012

Notice the contradiction in NASA's report: mode shape TE012 at 1.88GHz, mode shape TM211 at a higher frequency 1.93 GHz and mode shape TE012 again at an even higher frequency 2.17 GHz (? ).  This is impossible.  Our interpretation is that NASA Brady et.al.'s made a typo and they meant to write TM211 (or another mode) for frequencyBradyD =  2.168*10^9 .



8) Given the the geometrical dimensions (using the previously defined Volumetric Mean Diameter), and the value of speed of light in air, we use the frequency equation (see: http://en.wikipedia.org/wiki/Microwave_cavity#Cylindrical_cavity ) to calculate frequencies as a function of the mode shape quantum numbers: circumferential (m), radial (n), and longitudinal (p).   Mode shapes are reported as TXmnp where "X" can stand for E= electric transverse mode or M= magnetic transverse mode.



9) Then I obtain the following mode for the three different assumed geometries:


Mulletron Geometry

{{"TE", 1, 1, 0}, 7.52226*10^8},
{{"TE", 1, 1, 1}, 9.27278*10^8},
{{"TM", 0, 1, 0}, 9.82506*10^8},
{{"TM", 0, 1, 1}, 1.12219*10^9},
{{"TE", 2, 1, 0}, 1.24783*10^9},
{{"TE", 1, 1, 2}, 1.31979*10^9},
{{"TE", 2, 1, 1}, 1.36054*10^9},
{{"TM", 0, 1, 2}, 1.46332*10^9},
{{"TM", 1, 1, 0}, 1.56547*10^9},
{{"TE", 0, 1, 0}, 1.56547*10^9},
{{"TE", 2, 1, 2}, 1.6532*10^9},
{{"TM", 1, 1, 1}, 1.65671*10^9},
{{"TE", 0, 1, 1}, 1.65671*10^9},
{{"TE", 3, 1, 0}, 1.71642*10^9},
{{"TE", 1, 1, 3}, 1.79216*10^9},
{{"TE", 3, 1, 1}, 1.80003*10^9},
{{"TM", 0, 1, 3}, 1.90034*10^9},
{{"TM", 1, 1, 2}, 1.90438*10^9},
{{"TE", 0, 1, 2}, 1.90438*10^9},
{{"TE", 3, 1, 2}, 2.03029*10^9},
{{"TE", 2, 1, 3}, 2.05014*10^9},
{{"TM", 2, 1, 0},2.09819*10^9},
{{"TM", 2, 1, 1}, 2.16712*10^9},
{{"TE", 4, 1, 0}, 2.17252*10^9},

Mode shapes bracketing NASA-reported frequencies:

frequencyBradyA = 1.9326*10^9 1/second;
{{"TE", 0, 1, 2}, 1.9043845840829124`*^9}; {{"TM", 1, 1, 2}, 1.9043845840829124`*^9};
{{"TE", 3, 1, 2}, 2.0302945503765867`*^9};
frequencyBradyB = 1.9367*10^9 1/second;
{{"TE", 0, 1, 2}, 1.9043845840829124`*^9}; {{"TM", 1, 1, 2}, 1.9043845840829124`*^9};
{{"TE", 3, 1, 2}, 2.0302945503765867`*^9};
frequencyBradyC =  1.8804*10^9 1/second;
{{"TE", 3, 1, 1}, 1.8000272999857957`*^9};
{{"TM", 0, 1, 3}, 1.9003447870125527`*^9};
frequencyBradyD =  2.168*10^9 1/second;
{{"TM", 2, 1, 1}, 2.167116717351836`*^9};
{{"TE", 4, 1, 0}, 2.17251756371995`*^9};

Frequencies of NASA-reported mode shapes:

{{"TE", 0, 1, 2}, 1.9043845840829124`*^9}
{{"TM", 2, 1, 1}, 2.167116717351836`*^9}


Aero geometry

{{"TE", 1, 1, 0}, 8.05391*10^8},
{{"TE", 1, 1, 1}, 1.01634*10^9},
{{"TM", 0, 1, 0}, 1.05194*10^9},
{{"TM", 0, 1, 1}, 1.22102*10^9},
{{"TE", 2, 1, 0}, 1.33602*10^9},
{{"TE", 2, 1, 1}, 1.47283*10^9},
{{"TE", 1, 1, 2}, 1.47846*10^9},
{{"TM", 0, 1, 2}, 1.62597*10^9},
{{"TM", 1, 1, 0}, 1.67611*10^9},
{{"TE", 0, 1, 0}, 1.67611*10^9},
{{"TM", 1, 1, 1}, 1.78707*10^9},
{{"TE", 0, 1, 1}, 1.78707*10^9},
{{"TE", 2, 1, 2}, 1.82267*10^9},
{{"TE", 3, 1, 0}, 1.83773*10^9},
{{"TE", 3, 1, 1}, 1.93947*10^9},
{{"TE", 1, 1, 3}, 2.02665*10^9},
{{"TM", 1, 1, 2}, 2.08483*10^9},
{{"TE", 0, 1, 2}, 2.08483*10^9},
{{"TM", 0, 1, 3}, 2.13665*10^9},
{{"TE", 3, 1, 2}, 2.21685*10^9}   

Mode shapes bracketing NASA-reported frequencies:

frequencyBradyA = 1.9326*10^9 1/second;
{{"TE", 3, 1, 0}, 1.8377297671354957`*^9};
{{"TE", 3, 1, 1}, 1.9394709580599272`*^9};
frequencyBradyB = 1.9367*10^9 1/second;
{{"TE", 3, 1, 0}, 1.8377297671354957`*^9};
{{"TE", 3, 1, 1}, 1.9394709580599272`*^9};
frequencyBradyC = 1.8804*10^9 1/second;
{{"TE", 3, 1, 0}, 1.8377297671354957`*^9};
{{"TE", 3, 1, 1}, 1.9394709580599272`*^9};
frequencyBradyD = 2.168*10^9 1/second;
{{"TM", 0, 1, 3}, 2.136646973612425`*^9};
{{"TE", 3, 1, 2}, 2.216853242229605`*^9};

Frequencies of NASA-reported mode shapes:

{{"TE", 0, 1, 2}, 2.0848310776817226`*^9};
{{"TM", 2, 1, 1}, 2.330443792149791`*^9}


Fornaro geometry

{{"TE", 1, 1, 0}, 5.4291*10^8},
{{"TE", 1, 1, 1}, 7.06031*10^8},
{{"TM", 0, 1, 0}, 7.09111*10^8},
{{"TM", 0, 1, 1}, 8.40576*10^8},
{{"TE", 2, 1, 0}, 9.00604*10^8},
{{"TE", 2, 1, 1}, 1.00738*10^9},
{{"TE", 1, 1, 2}, 1.05341*10^9},
{{"TM", 1, 1, 0}, 1.12986*10^9},
{{"TE", 0, 1, 0}, 1.12986*10^9},
{{"TM", 0, 1, 2}, 1.14793*10^9},
{{"TM", 1, 1, 1}, 1.21668*10^9},
{{"TE", 0, 1, 1}, 1.21668*10^9},
{{"TE", 3, 1, 0}, 1.23881*10^9},
{{"TE", 2, 1, 2}, 1.27515*10^9},
{{"TE", 3, 1, 1}, 1.31847*10^9},
{{"TM", 1, 1, 2}, 1.4462*10^9},
{{"TE", 0, 1, 2}, 1.4462*10^9},
{{"TE", 1, 1, 3}, 1.45887*10^9},
{{"TM", 2, 1, 0}, 1.51434*10^9},
{{"TM", 0, 1, 3}, 1.52853*10^9},
{{"TE", 3, 1, 2}, 1.53283*10^9},
{{"TE", 4, 1, 0}, 1.56799*10^9},
{{"TE", 1, 2, 0}, 1.57208*10^9},
{{"TM", 2, 1, 1}, 1.58018*10^9},
{{"TE", 2, 1, 3}, 1.62624*10^9},
{{"TM", 0, 2, 0}, 1.62771*10^9},
{{"TE", 4, 1, 1}, 1.63166*10^9},
{{"TE", 1, 2, 1}, 1.6356*10^9},
{{"TM", 0, 2, 1}, 1.68913*10^9},
{{"TM", 2, 1, 2}, 1.76299*10^9},
{{"TM", 1, 1, 3}, 1.76356*10^9},
{{"TE", 0, 1, 3}, 1.76356*10^9},
{{"TE", 4, 1, 2}, 1.80928*10^9},
{{"TE", 1, 2, 2}, 1.81283*10^9},
{{"TE", 3, 1, 3}, 1.83526*10^9},
{{"TM", 0, 2, 2}, 1.86127*10^9},
{{"TM", 3, 1, 0}, 1.88132*10^9},
{{"TE", 1, 1, 4}, 1.88531*10^9},
{{"TE", 5, 1, 0}, 1.89177*10^9},
{{"TM", 3, 1, 1}, 1.93471*10^9},
{{"TM", 0, 1, 4}, 1.93972*10^9},
{{"TE", 5, 1, 1}, 1.94487*10^9},
{{"TE", 2, 2, 0}, 1.97744*10^9},
{{"TE", 2, 1, 4}, 2.01761*10^9},
{{"TE", 2, 2, 1}, 2.0283*10^9},
{{"TM", 2, 1, 3}, 2.03145*10^9},
{{"TM", 1, 2, 0}, 2.06869*10^9},
{{"TE", 0, 2, 0}, 2.06869*10^9},
{{"TE", 4, 1, 3}, 2.07175*10^9},
{{"TE", 1, 2, 3}, 2.07485*10^9},
{{"TM", 3, 1, 2}, 2.08669*10^9},
{{"TE", 5, 1, 2}, 2.09612*10^9},
{{"TM", 0, 2, 3}, 2.11731*10^9},
{{"TM", 1, 2, 1}, 2.11736*10^9},
{{"TE", 0, 2, 1}, 2.11736*10^9},
{{"TM", 1, 1, 4}, 2.12984*10^9},
{{"TE", 0, 1, 4}, 2.12984*10^9},
{{"TE", 2, 2, 2}, 2.17375*10^9},

Mode shapes bracketing NASA-reported frequencies:

frequencyBradyA = 1.9326*10^9 1/second;
{{"TE", 5, 1, 0}, 1.891774278367002`*^9};
{{"TM", 3, 1, 1}, 1.9347074025015635`*^9};
frequencyBradyB = 1.9367*10^9 1/second;
{{"TM", 3, 1, 1}, 1.9347074025015635`*^9};
{{"TM", 0, 1, 4}, 1.9397152624956703`*^9};
frequencyBradyC = 1.8804*10^9 1/second;
{{"TM", 0, 2, 2}, 1.8612746091297557`*^9};
{{"TM", 3, 1, 0}, 1.8813198077493927`*^9};
frequencyBradyD = 2.168*10^9 1/second;
{{"TE", 0, 1, 4}, 2.129842918761221`*^9};
{{"TE", 2, 2, 2}, 2.173747734247802`*^9};

Frequencies of NASA-reported mode shapes:

{{"TE", 0, 1, 2}, 1.4461980111408324`*^9}
{{"TM", 2, 1, 1}, 1.5801773963619902`*^9}





CONCLUSIONS

1)The "Fornaro Assumed Geometry" gives:

1a) frequencies of NASA-reported mode shapes:

{{"TE", 0, 1, 2}, 1.4461980111408324`*^9}
{{"TM", 2, 1, 1}, 1.5801773963619902`*^9}

that are much lower (25%) than the tested frequencies.  If one takes into account that the NASA's experiments were conducted with a dielectric, whose inclusion will lower the natural frequency, it is clear that NASA's truncated cone cavity must have had dimensions significantly smaller than assumed by Fornaro.

1b)  The mode shapes obtained under the Fornaro geometrical assumptions have the incorrect transverse field:  Transverse Magnetic modes for the experiments that resulted in measured thrust (Brady a, b and c) and Transverse Electric mode for the experiment that resulted in no measured thrust (Brady d).  A correct geometry should result in the opposite: TE modes for Brady a, b and c and TM mode for Brady d. It is clear then that NASA's truncated cone could not have Fornaro's assumed geometry.

1c) The Fornaro geometry has so many natural frequencies in the experimental range, so close to each other, that it would have been extremely difficult to tune the EM Drive to a particular frequency.  What we learn from this is that if one wants to drive EM Drives with these natural frequencies and mode shapes, EM Drives should be small enough (smaller than Fornaro's dimensions) so that natural frequencies are apart enough from each other that the EM Drive can be tuned if one wants to excite some of these frequencies.  To achieve the thrust required for crewed space missions would then require a large number of small EM Drives, rather than a large single EM Drive as pictured in some of Shawyer's concepts or in Aether Drives in steampunk.
One could aim to excite the lowest natural frequency: TE110 with a large EM Drive, which for Fornaro's geometry means 0.54291 GHz (less than 1/3 the frequencies tested at NASA by Brady et.al.), but I don't think that this mode TE110 (see picture below) is likely to produce significant thrust according to any theories that claim coupling with external fields (like the Quantum Vacuum, etc.)



2) The "Mulletron assumed geometry":

2a) achieves an outstanding fit by very closely predicting the frequency of 2.168 GHz for Brady's experiment "d", mentioned in the section on p .18, F.Tapered Cavity RF Evaluation, General Findings and Lessons Learned; of (http://www.libertariannews.org/wp-content/uploads/2014/07/AnomalousThrustProductionFromanRFTestDevice-BradyEtAl.pdf ). This is a very important fit, since it is the only experiment that which we are sure was conducted without a dielectric, and hence it should be best modeled by the exact solution:

frequencyBradyD =  2.168*10^9 1/second;
{{"TM", 2, 1, 1}, 2.167116717351836`*^9};

2b) it predicts that the mode shape should be TM211, which is a Transverse Magnetic mode. Brady et.al reported no thrust measurement at this frequency.  Our explanation is that this may have been due to the fact that Transverse Electric modes are required for thrust measurements (either as an artifact due to heating of the ends by heat induction or as a legitimate propulsion for example by coupling with the Quantum Vacuum).  Hence Transverse Magnetic modes should result in no thrust, which agrees with the experimental results.

Brady et.al. write:

Quote
We performed some very early evaluations without the dielectric resonator (TE012 mode at 2168 MHz, with power levels up to ~30 watts) and measured no significant net thrust.

Brady et.al. report mode shape TE012 for this frequeny. The assumption that the mode shape was TE012 may be an error in the report, because their Table 2. Tapered Cavity Testing: Summary of Results, clearly shows the mode shape TE012 occurring at a lower frequency than TM211, so it doesn't make any sense for Brady et.al. to write that the mode shape TE012 can occur both below and above TM211.  We think that they may have intended to write "TM211 mode at 2168 MHz" which would be in agreement with Mulletron's geometry.

2c) it predicts  mode shape TE012 for the frequencies of NASA Brady et.al.'s experiments a and b:

frequencyBradyA = 1.9326*10^9 1/second;
{{"TE", 0, 1, 2}, 1.9043845840829124`*^9};
frequencyBradyB = 1.9367*10^9 1/second;
{{"TE", 0, 1, 2}, 1.9043845840829124`*^9};

Brady et.al. report mode shape TM211 for these frequencies, based on their numerical calculations using the Finite Element code COMSOL.  However the data on their Figure 16.  "Predicted and Actual S21 plots" shows that their  COMSOL numerical calculations are substantially in error (probably due to an unconverged, insufficiently fine finite-element mesh), particularly at this frequency range in question: 1.9 GHz.   It makes most sense that the actual mode shape was TE012, rather than a magnetic mode (TM211) for the previously discussed reasons.

2d) it has a problem for Brady et.al.'s experiment c, where it predicts mode shape TM013, with mode shape TE311 giving a lower frequency:

frequencyBradyC =  1.8804*10^9 1/second;
{{"TE", 3, 1, 1}, 1.8000272999857957`*^9};
{{"TM", 0, 1, 3}, 1.9003447870125527`*^9};

NASA's Brady et.al. reported mode shape TE012 for this frequency, which makes more sense because this is NASA's experiment that resulted in the highest thrust/PowerInput.  It is possible that the discrepancy is that the exact solution we are using does not include the effect of the dielectric.  Mulletron's geometry predicts a frequency of 1.90 GHz for mode shape TE012 without the dielectric:

{{"TE", 0, 1, 2}, 1.9043845840829124`*^9}

It is very possible that the dielectric lowers the frequency and that the actual mode shape for Brady et.al.'s experiment c at 1.8804 GHz was TE012 (or actually TE013: two half-wave patterns in the empty section of the cavity in the longitudinal direction, like TE012, and another extra half-wave in the dielectric).



3) The "Aero assumed geometry":

3a) The strength of Aero's assumed geometry is that it is the only assumed geometry that very consistently predicts mode shape TE (transverse electric) modes for the Brady et.al. experiments that measured thrust: (experiments a, b and c), while predicting mode shape TM (transverse magnetic) for the Brady experiment that measured no thrust: Brady et.al. experiment d:


Mode shapes bracketing NASA-reported frequencies:

frequencyBradyA = 1.9326*10^9 1/second;
{{"TE", 3, 1, 1}, 1.9394709580599272`*^9};
frequencyBradyB = 1.9367*10^9 1/second;
{{"TE", 3, 1, 1}, 1.9394709580599272`*^9};
frequencyBradyC = 1.8804*10^9 1/second;
{{"TE", 3, 1, 0}, 1.8377297671354957`*^9};
frequencyBradyD = 2.168*10^9 1/second;
{{"TM", 0, 1, 3}, 2.136646973612425`*^9};

3b) The presence of a dielectric inside the cavity may have resulted lowering the frequency predicted for mode shape TE012 from 2.08GHz to 1.88 GHz:

Frequencies of NASA-reported mode shapes:

{{"TE", 0, 1, 2}, 2.0848310776817226`*^9};
{{"TM", 2, 1, 1}, 2.330443792149791`*^9}



4)  It is clear that the actual geometry of NASA's Brady et.al. experiments was smaller than as predicted by Fornaro's assumptions.  Based on the evidence discussed above, the geometry was close to Mulletron's and Aero's assumed geometry, with Mulletron's geometry having an edge for the above-discussed reasons.



Images of modes TE01p,  TM21p, and TE11p  (where p can be any p=0,1,2,3,...):

electric field ________________    solid lines

magnetic field - - - - - - - - - - - - -  dashed lines
Title: Re: EM Drive Developments - related to space flight applications - Thread 2
Post by: aero on 01/30/2015 12:43 am
Very interesting.
Here's another little wrinkle you can play with.
Define x = r/R where
r is the small diameter of the cone,
and R is the big diameter of the cone.
Then let Rcy equal the diameter of the cylindrical cavity. I get
f(x) = (x^2+x+1)
R = sqrt(Rcy^2 /f(x)/3)
Of course the formula can be inverted to solve for Rcy.

For my geometry, (aero geometry) I get x = 0.5826543346
For Mulletron geometry, x = 0.527443684
and for Fornaro geometry, x = 0.6146095718

This is not quite the formula that we need (check it before using), we need something that relates L / Rcy, I think, because I read somewhere that cylindrical cavities won't resonate when L/Rcy < ~ 2.02. It has to do with cut-off frequency as I recall.
Title: Re: EM Drive Developments - related to space flight applications - Thread 2
Post by: Rodal on 01/30/2015 01:22 pm
Isn't it possible to increase radius while leaving length at or around 12 cm? I would prefer that but can't find a radius that works with 2.45 GHz.

Yes, it's possible. 

If you insist in specifying the exciting frequency as 2.45GHz and the length (12 cm) of the cylindrical cavity, and having the diameter as the variable to be adjusted, then

....
             = 8.332965999678832` centimeters

instead of  the diameter=7.54898 centimeter you used.

@Aero, is the reason why you insist in keeping a frequency of 2.45GHz in your modeling because you are looking at making a small EM Drive using a kitchen's microwave's magnetron as the source (which are nominally rated to operate at ~ 2.45 GHz)  ?

(http://jlnlabs.online.fr/plasma/images/2M253A.gif)
Title: Re: EM Drive Developments - related to space flight applications - Thread 2
Post by: aero on 01/30/2015 03:11 pm
Quote
@Aero, is the reason why you insist in keeping a frequency of 2.45GHz in your modeling because you are looking at making a small EM Drive using a kitchen's microwave's magnetron as the source (which are nominally ~ 2.45 GHz)  ?

I certainly won't, but look around. Magnetron technology advances but there are orders of magnitude more magnetrons made for the kitchen microwave than for anything else. That means research and technology advance will and does focus on the 2.45 GHz operating point. Even now there exists all digital 1000W, 2.45 GHz microwave power generators on the market. A little pricy at $39.95 each in lots of 10,000 but price/lot size will come down. The web link, which I don't have available at the moment, indicated that they are pretty slick little devices, digitally tunable, sized for microwave application and long lived.

We are researching EM thruster theory. Its not to early to think about the price point for the manufacture of these devices. Yes, after a few 100 or few 1000 are made and work, then research money will start to flow into the development of the microwave generators at whatever frequency. Until then, as the Chinese experiments illustrate, 2.45 GHz is the most convenient frequency to use for research.
Title: Re: EM Drive Developments - related to space flight applications - Thread 2
Post by: Mulletron on 01/30/2015 06:40 pm
IRT the last couple posts, the unlicensed ISM band is more approachable for enthusiasts. I want folks like us to explore this proposition that we can achieve all electric thrust in the vacuum of space without propellant. I'm awaiting word from our academic institutions and NASA about what is really going on here, but that word may not come. Part of my mission on here, besides hopefully bringing attention to the underlying research which could support this (Tiggelen et al), and trying to figure this out myself with the help of my fellow posters, is also to inspire builders to take up experimentation by providing as much research and backup as possible. At the end, I want this thread (or the derivative of) to mature from "advanced concepts" to proven flight ready in my lifetime. There is NO way humanity is going to be stuck on this planet forever.

I in particular am trying fervently to find a solution for an unloaded conical frustum geometry resonant at 2437mhz at TE111 to start with. 2450mhz is a good start. From there you can perturb the cavity down to 2437mhz at will (mostly trial and error). I am still unsure if angle by the apex is truly important for resonance, but according to Egan, it is: http://www.gregegan.net/SCIENCE/Cavity/Cavity.html But if my hunch about the importance of the QV to spacetime is correct (pretty much has to be), 45 and 90 degrees by the apex is important. Since there appears to not be a ready made solution for this available to me in the world (short of simulating it like buying COMSOL and learning it, or hints from that patent), I(we) have to figure it out, somehow.

So in short. I want to crowdsource this problem. We need help! We all sink or swim together, as they say.
Title: Re: EM Drive Developments - related to space flight applications - Thread 2
Post by: Rodal on 01/30/2015 07:18 pm
....

I in particular am trying fervently to find a solution for an unloaded conical frustum geometry resonant at 2437mhz at TE111 to start with. 2450mhz is a good start. From there you can perturb the cavity down to 2437mhz at will (mostly trial and error)...
I think one has a greater chance of achieving thrust (it that is indeed possible) with mode shape TE01p than with mode TE11p, for any p. The images below illustrate the difference between them. 

Also, NASA reported mode TE012 for their experiment that gave greatest thrust/PowerInput.

More discussion of this here:  http://forum.nasaspaceflight.com/index.php?topic=36313.msg1322952#msg1322952

As to calculations for a cone geometry, there is Greg Egan's http://gregegan.customer.netspace.net.au/SCIENCE/Cavity/Cavity.html .  We will be reporting on a comparison as well.



Images of modes TE01p, and TE11p  (where p can be any p=0,1,2,3,...):

electric field ________________    solid lines

magnetic field - - - - - - - - - - - - -  dashed lines
Title: Re: EM Drive Developments - related to space flight applications - Thread 2
Post by: Rodal on 01/31/2015 03:20 pm
...I am still unsure if angle by the apex is truly important for resonance, but according to Egan, it is: http://www.gregegan.net/SCIENCE/Cavity/Cavity.html But if my hunch about the importance of the QV to spacetime is correct (pretty much has to be), 45 and 90 degrees by the apex is important. ..

@Mulletron, why do you think that the cone angle (thetaw in Egan's nomenclature), of an EM Drive, should (ideally) be 45 degrees ? (I don't recall the reasons(s), please refresh my mind). 

(http://gregegan.customer.netspace.net.au/SCIENCE/Cavity/CavityShape.gif)

In Egan's geometry, the cone angle  (thetaw)  is measured from the longitudinal axis of the cone, "z", therefore this picture should show a negative cone angle  ( - thetaw) on the left, and a positive  ( + thetaw) on the right.



For reference. the tangent of thetaw and the angle thetaw, for the following cases are:


Example (and geometry)                    { Tan[thetaw],thetaw (degrees) }

NASA Brady et.al. (Fornaro)                {0.230422, 12.9757}
NASA Brady et.al. (Aero)                     {0.2352,     13.2354}
NASA Brady et.al. (Mulletron)              {0.257318, 14.4302}

Shawyer Experimental                        {0.104019,   5.93851}
Shawyer Demo                                    {0.219054, 12.3557}
Shawyer Superconducting 2014          {0.7002,     35}

Egan's example                                   {0.36397 ,  20}

We see that the cone angle thetaw for NASA's Brady et.al. truncated cone was about 14 degrees, Shawyer's Experimental  was only 6 degrees, Shawyer's Demo  was 12 degrees, and Egan's only example is 20 degrees (Egan's example has a cone angle much larger than the cone angle of experiments). The "cone angle thetaw" for a cylinder is zero, Shawyer's Experimental was closest to a cylinder, and NASA's Brady et.al. was the experiment with the largest cone angle.

Shawyer's latest (2014) superconducting design (see image, presented at the IAC 2014 conference in Toronto), for which there are no experimental results reported yet, appears to have a significantly larger cone angle than his previous experimental and demo geometries, and significantly larger than NASA's Brady et.al.'s.

EDIT: Shawyer's (2014) superconducting EM Drive design has a cone angle  thetaw of about 35 degrees.

(http://1.bp.blogspot.com/-IUOMkncS1sM/VFLSUw2HeoI/AAAAAAAA3yE/Zl3iaKJyrX0/s1600/emdrivedata2.png)



(*

Fornaro estimate \
http://forum.nasaspaceflight.com/index.php?topic=36313.msg1302455#msg1302455;

fornaroLength=0.332 m
fornaroBigDiameter=0.397 m
fornaroSmallDiameter=0.244 m

*)

(*

Aero Best estimate as of 11/9/2014 \
http://forum.nasaspaceflight.com/index.php?topic=29276.msg1285896#msg1285896 ;

aeroLength=0.24173 m
aeroBigDiameter=0.27246 m
aeroSmallDiameter=0.15875 m

*)

(*
Mulletron Best estimate as of 11/9/2014
http://forum.nasaspaceflight.com/index.php?topic=36313.msg1320903#msg1320903;

mulletronLength=0.27637 m
mulletronBigDiameter=0.30098 m
mulletronSmallDiameter=0.15875 m

*)

(*
"Shawyer EXPERIMENTAL geometry"

shawyerExpLength=0.156 meter;
shawyerExpBigDiameter=0.16 meter
shawyerExpSmallDiameter=0.127546 meter;
*)

(*
" Shawyer DEMO geometry"

shawyerDemoLength=0.345 meter;
shawyerDemoBigDiameter=0.28 meter;
shawyerDemoSmallDiameter=0.128853 meter;

*)
Title: Re: EM Drive Developments - related to space flight applications - Thread 2
Post by: Rodal on 01/31/2015 04:40 pm
Also, the larger the cone angle thetaw, the more important is to have EM Drive spherical ends rather than flat ends.  The curvature of the spherical ends is more pronounced for larger cone angle thetaw.  Shawyer realized this: he has spherical ends in his 2014 superconducting design shown in the above image.
Title: Re: EM Drive Developments - related to space flight applications - Thread 2
Post by: aero on 02/01/2015 06:57 pm
I learned a new trick.  :) Here are some images of the ez field along the x coordinate.

The imaginary and real parts are shown at x=194, because that showed a powerful signal.
The imaginary and real parts are shown at x=216, because that is the big end of the cavity.
The imaginary and real parts are shown at x=39, because that is the just inside the dielectric at the small end.

I'll see if I can get some magnetic field images.
Title: Re: EM Drive Developments - related to space flight applications - Thread 2
Post by: Rodal on 02/01/2015 08:46 pm
I learned a new trick.  :) Here are some images of the ez field along the x coordinate.

The imaginary and real parts are shown at x=194, because that showed a powerful signal.
The imaginary and real parts are shown at x=216, because that is the big end of the cavity.
The imaginary and real parts are shown at x=39, because that is the just inside the dielectric at the small end.

I'll see if I can get some magnetic field images.

The real parts of the electric field shape at the location of highest intensity (x=194) and at the small end dielectric ( x=39) look like mode shape TE12 (see image attached).  Looking forward to seeing the magnetic field, to be sure what mode it is.

electric field ________________    solid lines

magnetic field - - - - - - - - - - - - -  dashed lines
Title: Re: EM Drive Developments - related to space flight applications - Thread 2
Post by: Rodal on 02/01/2015 08:58 pm
I learned a new trick.  :) Here are some images of the ez field along the x coordinate.

The imaginary and real parts are shown at x=194, because that showed a powerful signal.
The imaginary and real parts are shown at x=216, because that is the big end of the cavity.
The imaginary and real parts are shown at x=39, because that is the just inside the dielectric at the small end.

I'll see if I can get some magnetic field images.
Are you running the same dielectric properties, overall geometry and source frequency as in your message http://forum.nasaspaceflight.com/index.php?topic=36313.msg1321460#msg1321460 ?

It the answer is yes, then it looks like the mode shape was TE123 , that is m=1, n=2, p =3. 

Based just on the limited information from the movie's image of the z,r plane I previously thought ( http://forum.nasaspaceflight.com/index.php?action=post;msg=1321476;topic=36313.320 ) the mode shape could be TE013: m=0, n=1, p=3.  Based on the z,r plane previously shown one could not tell what m was.  The field does not look constant in the circumferential direction (which would be needed for m=0).  It seems to have a full wave, which means m=1.  I previously thought that n=1 , but n =2 can look like n=1  on the z,r plane at the right circumferential (azimuthal ) angle. 

To be 100% sure one would need an image of the plane z , r, showing a still picture of the electric field in the longitudinal direction, like you did in the movie (where z is the longitudinal axis of the cone and r is the radial axis) rotated (around the longitudinal axis z) at an angle 90 degrees from the previous circumferential angle.
 
(Or even better, get two new still images showing the electric field in the longitudinal direction:

1) of a plane z , r located at a circumferential angle theta1 and

2) of another plane z, r located at a different circumferential angle theta2

such that theta2 is rotated  (around the longitudinal axis z) by 90 degrees from theta1 so that  theta2=  theta1 + 90 degrees
)


(http://mathworld.wolfram.com/images/eps-gif/CylindricalCoordinates_1001.gif)

(http://www.ibiblio.org/links/devmodules/shared/graphics/cylindrical.jpg)

The magnetic field images will also be helpful to understand what mode shape it is.
Title: Re: EM Drive Developments - related to space flight applications - Thread 2
Post by: aero on 02/01/2015 11:23 pm
I will try to get additional views as time and health permits. My boy brought something home from school and we've all contracted it. Unfortunately for me, I didn't throw it off like the wife and boy did.

This image is in 3D, which means very low resolution, so no its not like the other one. It's the same cavity and same drive frequency though so it should be generally very similar.
Title: Re: EM Drive Developments - related to space flight applications - Thread 2
Post by: Rodal on 02/02/2015 12:39 am
I will try to get additional views as time and health permits. My boy brought something home from school and we've all contracted it. Unfortunately for me, I didn't throw it off like the wife and boy did.

This image is in 3D, which means very low resolution, so no its not like the other one. It's the same cavity and same drive frequency though so it should be generally very similar.
Thanks for updating us on your great progress.  Hope you feel better soon  :)
Title: Re: EM Drive Developments - related to space flight applications - Thread 2
Post by: Rodal on 02/02/2015 02:10 pm
I learned a new trick.  :) Here are some images of the ez field along the x coordinate.

The imaginary and real parts are shown at x=194, because that showed a powerful signal.
The imaginary and real parts are shown at x=216, because that is the big end of the cavity.
The imaginary and real parts are shown at x=39, because that is the just inside the dielectric at the small end.

I'll see if I can get some magnetic field images.


For these truncated cone calculations, you reported (in the movie attachment to http://forum.nasaspaceflight.com/index.php?topic=36313.msg1321460#msg1321460 ) that the value of the relative permittivity (dielectric constant) you used was 2.3.

I did not find the value of relative permeability ( the degree of magnetization of the material ) you used for your truncated cone (NASA Brady et.al.) calculations.

Just to be sure, could you please confirm that you used a value of 1 (one) for the relative permeability in the above calculations ?
Title: Re: EM Drive Developments - related to space flight applications - Thread 2
Post by: aero on 02/02/2015 04:04 pm
I learned a new trick.  :) Here are some images of the ez field along the x coordinate.

The imaginary and real parts are shown at x=194, because that showed a powerful signal.
The imaginary and real parts are shown at x=216, because that is the big end of the cavity.
The imaginary and real parts are shown at x=39, because that is the just inside the dielectric at the small end.

I'll see if I can get some magnetic field images.


For these truncated cone calculations, you reported (in the movie attachment to http://forum.nasaspaceflight.com/index.php?topic=36313.msg1321460#msg1321460 ) that the value of the relative permittivity (dielectric constant) you used was 2.3.

I did not find the value of relative permeability ( the degree of magnetization of the material ) you used for your truncated cone (NASA Brady et.al.) calculations.

Just to be sure, could you please confirm that you used a value of 1 (one) for the relative permeability in the above calculations ?
I used the dielectric constant of 1.76 for the dielectric disk. That number was 2.3 in the movie but I only use 1.76 now that I've decided that 1.76 is the correct value. To investigate resonance of an empty cavity I can replace the dielectric material with "air." The value is 1.76 for the above runs.

In all cases the cone material, which should be copper, is a material defined by meep as a "Perfect Metal."  The content of the cavity is another material defined by meep as "air." The material outside the cavity, the environment, defaults to "vacuum." I do move the antenna location around and often forget to put it back to the most representative location for the run type. I move it because when I run Harminv using Cylindrical coordinates, the antenna must be on the central axis of rotation of the cone. If it is not, then in Cylindrical coordinates, nothing excites the cavity.

You can most likely tell by looking at the images, where the antenna was located for a particular run.
Title: Re: EM Drive Developments - related to space flight applications - Thread 2
Post by: Rodal on 02/02/2015 04:40 pm
I learned a new trick.  :) Here are some images of the ez field along the x coordinate.

The imaginary and real parts are shown at x=194, because that showed a powerful signal.
The imaginary and real parts are shown at x=216, because that is the big end of the cavity.
The imaginary and real parts are shown at x=39, because that is the just inside the dielectric at the small end.

I'll see if I can get some magnetic field images.


For these truncated cone calculations, you reported (in the movie attachment to http://forum.nasaspaceflight.com/index.php?topic=36313.msg1321460#msg1321460 ) that the value of the relative permittivity (dielectric constant) you used was 2.3.

I did not find the value of relative permeability ( the degree of magnetization of the material ) you used for your truncated cone (NASA Brady et.al.) calculations.

Just to be sure, could you please confirm that you used a value of 1 (one) for the relative permeability in the above calculations ?
I used the dielectric constant of 1.76 for the dielectric disk. That number was 2.3 in the movie but I only use 1.76 now that I've decided that 1.76 is the correct value. To investigate resonance of an empty cavity I can replace the dielectric material with "air." The value is 1.76 for the above runs.
..

As I understand, the z,r (axial,radial) plane image in the movie was based on a relative permittivity of 2.3, while the theta, r (circumferential, radial) plane images of your above message are based on the different value of relative permittivity of 1.76.

Therefore it is possible that the mode shape in the z,r (axial,radial) plane image in the movie is different than the mode shape of the theta, r (circumferential, radial) plane images of your above message, the different mode shapes being due to the different values of  relative permittivity that were used in each calculation.
Title: Re: EM Drive Developments - related to space flight applications - Thread 2
Post by: Rodal on 02/02/2015 05:05 pm
I learned a new trick.  :) Here are some images of the ez field along the x coordinate.

The imaginary and real parts are shown at x=194, because that showed a powerful signal.
The imaginary and real parts are shown at x=216, because that is the big end of the cavity.
The imaginary and real parts are shown at x=39, because that is the just inside the dielectric at the small end.

I'll see if I can get some magnetic field images.


For these truncated cone calculations, you reported (in the movie attachment to http://forum.nasaspaceflight.com/index.php?topic=36313.msg1321460#msg1321460 ) that the value of the relative permittivity (dielectric constant) you used was 2.3.

I did not find the value of relative permeability ( the degree of magnetization of the material ) you used for your truncated cone (NASA Brady et.al.) calculations.

Just to be sure, could you please confirm that you used a value of 1 (one) for the relative permeability in the above calculations ?
..." I do move the antenna location around and often forget to put it back to the most representative location for the run type. I move it because when I run Harminv using Cylindrical coordinates, the antenna must be on the central axis of rotation of the cone. If it is not, then in Cylindrical coordinates, nothing excites the cavity.

For the calculations for the movie (with relative permittivity of 2.3) you used an excitation frequency of 1.76365 GHz.

What excitation frequency did you use for the calculations in your above message (with relative permittivity of 1.76) ?
Title: Re: EM Drive Developments - related to space flight applications - Thread 2
Post by: Mulletron on 02/02/2015 07:39 pm
...I am still unsure if angle by the apex is truly important for resonance, but according to Egan, it is: http://www.gregegan.net/SCIENCE/Cavity/Cavity.html But if my hunch about the importance of the QV to spacetime is correct (pretty much has to be), 45 and 90 degrees by the apex is important. ..

@Mulletron, why do you think that the cone angle (thetaw in Egan's nomenclature), of an EM Drive, should (ideally) be 45 degrees ? (I don't recall the reasons(s), please refresh my mind). 


Keeping first principles in mind concerning the QV model for how these thrusters may work. If they do interact with the QV somehow, their design should obviously be complimentary to the geometry of spacetime (there are crazy folks out there http://www.onlyspacetime.com/, I'm one of them, who believe spacetime emerges from the quantum world.) I wanted to test whether opening angle had any significance. When faced with choosing an opening angle amongst seemingly arbitrary angles I've found amongst Shawyer's prototypes, I wondered if a light cone opening angle of 45 degrees (measured from the longitudinal axis for a total of 90 as measured from outside) would be of any benefit or not. http://en.wikipedia.org/wiki/Minkowski_diagram After seeing that a light cone setup would be a ginormous cone after factoring in the necessary conical sections for front and end walls, I further halved it to 22.5 degrees (total of 45). That's why I have 2 drawn up in CAD. So it really came down to a question of what angle to pick out of so many choices. Those two I want to try.

Trying to reconcile the above ideas with simultaneously hunting a viable rf solution is proving daunting. Mostly due to the lack of resources. I may not be afforded the option to choose an opening angle and stay on freq and within reasonable size limits. As I'm researching this it is becoming clear that opening angle will be dominated by chosen frequency and practical considerations.

After following up on Aero's post: http://forum.nasaspaceflight.com/index.php?topic=29276.msg1274400#msg1274400 about this: http://www.emdrive.com/NWPU2010translation.pdf trying to see if I can find a good solution for calculating exact solutions for conical frustums, I learned that no such method exists to find closed form solutions to that problem. The Egan method is similar to what we need but it doesn't address the problem. None of what we're dealing here has spherical end caps. And honestly, the Egan way is way too high speed for me.
Quote
"Currently have two ways to find the electromagnetic field of the rectangular and circular waveguides, the eigen-value equation which is an analytical method and numerical solution, when finding solution for the resonator, Maxwell equation in is need to be created in a spherical coordinate system, because the complexity of the spherical coordinate fielder equation, has not found anyone using eigen-value method to calculated the distribution of the resonant field. Only find in Paper [4] using asymptotic method for conical waveguide. That method assume a equivalent radius ae, believes field of wavefront sphere of cone waveguide Eo,EФ,Ho,HФ can use its wavefront position radius ae equivalent circular waveguide field Er,EФ,Hr,HФ, this method of finding the field distribution within the conical resonator can be used as reference, but the accuracy reduced as the cone half opening angle increases. Using finite element to numerically simulate the Maxwell electromagnetic equation for the idealised conical resonator, the distribution of electromagnetic can be obtained directly, this method is not limited by the cavity structure and microwave mode."

Quote
By keep the diameter of the Small End constant, increase the large end of the cavity, in order to have the same resonant frequency, cavity height must be reduced, quality factor also reduce.

They're basically saying: (1) That I'm hosed trying to calculate such things. Simulating the conical frustum using FEM software is the way to go. Which I simply don't have access to. (2) Also they're saying that as the opening angle opens up, approximating the resonant modes becomes more and more difficult. (3) You have to shorten the cavity height as opening angle increases to maintain resonance at desired frequency, but it lowers Q. So I should probably (for now) re-think using such wide opening angles.

So I'm switching gears a bit using what I've learned from the above reporting.
(1) Keeping Cannae in mind, who says we need a cone anyway? We've discussed the commonality between Shawyer and Cannae in thread 1. http://forum.nasaspaceflight.com/index.php?topic=29276.msg1298712#msg1298712 So I'm thinking it would be smart to use what we've learned about cylinders and try a cylinder experiment.
(2) Instead of trying to optimize right out of the gate by throwing around light cones. It would be smarter to use the dims we already have for Shawyer experimental and demo, which you provided on the previous page.
(3) If I ever get this build going, I'm going to have to easter egg it anyway using a sig-gen and a power meter to find the resonant frequency (treat it like a filter, tune it until I get get an output from sample port), so I don't need to have exact calculations. I just need to be close enough to be within tunable limits.
Title: Re: EM Drive Developments - related to space flight applications - Thread 2
Post by: Mulletron on 02/02/2015 08:02 pm

(*
"Shawyer EXPERIMENTAL geometry"

shawyerExpLength=0.156 meter;
shawyerExpBigDiameter=0.16 meter
shawyerExpSmallDiameter=0.127546 meter;
*)

(*
" Shawyer DEMO geometry"

shawyerDemoLength=0.345 meter;
shawyerDemoBigDiameter=0.28 meter;
shawyerDemoSmallDiameter=0.128853 meter;

*)

These were both 2450mhz experiments. Where did the small diameters and lengths come from? I see the Large diameters here: http://www.emdrive.com/yang-juan-paper-2012.pdf.
Title: Re: EM Drive Developments - related to space flight applications - Thread 2
Post by: Rodal on 02/02/2015 08:06 pm

(*
"Shawyer EXPERIMENTAL geometry"

shawyerExpLength=0.156 meter;
shawyerExpBigDiameter=0.16 meter
shawyerExpSmallDiameter=0.127546 meter;
*)

(*
" Shawyer DEMO geometry"

shawyerDemoLength=0.345 meter;
shawyerDemoBigDiameter=0.28 meter;
shawyerDemoSmallDiameter=0.128853 meter;

*)

These were both 2450mhz experiments. Where did the small diameters and lengths come from? I see the Large diameters here: http://www.emdrive.com/yang-juan-paper-2012.pdf.

If I my memory is correct, we worked this out in Thread 1, the major contribution by far being from aero (aero deserves all the praise, if there are any mistakes in the above figures, they are mine).  I recall feeling very confident about the rationale that aero used to estimate these numbers, which I remember had a very solid foundation. 
Title: Re: EM Drive Developments - related to space flight applications - Thread 2
Post by: Mulletron on 02/02/2015 08:13 pm
Well if they are correct, they are a ready made solution for ISM band experimentation.

Break:
So I swore off the theory bug a few weeks ago but I've got a bug I need swatting concerning the behavior of confined photons vs the usual free range variety. Here's a quote and a video, both of which have bells ringing and light bulbs floating over my head.
Quoting John A. Macken, from http://www.onlyspacetime.com/HiggsBoson
Quote
Suppose that there was a box with hypothetical 100% reflecting internal walls. It would be possible to trap some light energy in such a box. A freely propagating photon is a massless particle, but what about a “confined photon” trapped in the box. That photon is forced to have the box’s specific frame of reference. A calculation at the end of chapter 1 shows that the photon pressure exerted on the walls of the box is uniform if the box is not accelerating, but the pressure becomes unequal if the box is accelerated. This difference in pressure results in a net force which resists acceleration. This is the inertia of the confined photon energy and it exactly equals the inertia of an equal amount of energy in the form of matter particles. This is not a coincidence.
That scenario sure sounds familiar......like within magic resonant cavity thrusters. I read that a few days ago and really didn't believe any of it, but I filed it away for later.

Now I'm going to re arrange a portion of the above quote and put out a RFC on it:
Quote
.......the photon pressure exerted on the walls of the box is uniform if the box is not accelerating, but the pressure becomes unequal if the box is accelerated. This difference in pressure results in a net force which resists acceleration.

Let's assume that if the author were correct, is it logical to say the flipside is also correct?

.......if the photon pressure exerted on the walls of the box is non uniform if the box is not accelerating accelerates, but the pressure becomes unequal if when the box is accelerated. This difference in pressure results in a net force which resists enables acceleration.

Perhaps it is better to find his calculation at the end of chapter 1 and rearrange it and see if it still works.

And this video @ 3:15
https://www.youtube.com/watch?v=nYDokJ2A_vU

I think I can safely believe Fermilab. Exchange photons are confined between two particles, so indeed they are also confined photons.

I never really approached photons from this perspective. I always assumed and was told they were always massless. What's the deal with confined photons? Confined photons seem to behave like massive particles.

Massive particles flowing into cavity, bouncing around a few thousand times, and then flowing out of a resonant cavity (by being absorbed) surely could make it move while conserving momentum. I picture a thrust nozzle when I think of it like this. Mass comes in, bounces around a bunch, finally comes out....and you have thrust.

Thanks for your patience. Would love some comments on this.

Edit:
Aha! http://www.livescience.com/45287-how-to-trap-light.html
Quote
A photon trapped in such a cavity behaves as if it had mass; in other words, the cavity creates a "trapping potential," keeping the photons from escaping.
http://www.desy.de/user/projects/Physics/Relativity/SR/light_mass.html
Quote
However, if light is trapped in a box with perfect mirrors so the photons are continually reflected back and forth in both directions symmetrically in the box, then the total momentum is zero in the box's frame of reference but the energy is not.  Therefore the light adds a small contribution to the mass of the box.
But then what? Can one really say with confidence that there exists a condition of "mass flow?"
Title: Re: EM Drive Developments - related to space flight applications - Thread 2
Post by: wembley on 02/03/2015 01:29 pm

A potentially interesting sidelight:

http://okomov.livejournal.com/577.html

The physics is descrived in vvery different terms ("Leonov’s superunification theory"?? "Antigravity"???) this has similar performance to what Shawyer predicts for a superconducting EmDrive thruster, i.e. 500 to 700 kg for 1kW power input.

Moreover, as soon as it starts to accelerate,  the thrust ceases, hence the pulsed operation in the video, which is what Shawyer claims for a high Q EmDrive thruster without Doppler compensation.

What does the team think...?
Title: Re: EM Drive Developments - related to space flight applications - Thread 2
Post by: Rodal on 02/03/2015 10:01 pm
....
Keeping first principles in mind concerning the QV model for how these thrusters may work. If they do interact with the QV somehow, their design should obviously be complimentary to the geometry of spacetime (there are crazy folks out there http://www.onlyspacetime.com/, I'm one of them, who believe spacetime emerges from the quantum world.) I wanted to test whether opening angle had any significance. When faced with choosing an opening angle amongst seemingly arbitrary angles I've found amongst Shawyer's prototypes, I wondered if a light cone opening angle of 45 degrees (measured from the longitudinal axis for a total of 90 as measured from outside) would be of any benefit or not. http://en.wikipedia.org/wiki/Minkowski_diagram After seeing that a light cone setup would be a ginormous cone after factoring in the necessary conical sections for front and end walls, I further halved it to 22.5 degrees (total of 45). That's why I have 2 drawn up in CAD. So it really came down to a question of what angle to pick out of so many choices. Those two I want to try.

Trying to reconcile the above ideas with simultaneously hunting a viable rf solution is proving daunting. Mostly due to the lack of resources. I may not be afforded the option to choose an opening angle and stay on freq and within reasonable size limits. As I'm researching this it is becoming clear that opening angle will be dominated by chosen frequency and practical considerations.

After following up on Aero's post: http://forum.nasaspaceflight.com/index.php?topic=29276.msg1274400#msg1274400 about this: http://www.emdrive.com/NWPU2010translation.pdf trying to see if I can find a good solution for calculating exact solutions for conical frustums, I learned that no such method exists to find closed form solutions to that problem. The Egan method is similar to what we need but it doesn't address the problem. None of what we're dealing here has spherical end caps. And honestly, the Egan way is way too high speed for me.
Quote
"Currently have two ways to find the electromagnetic field of the rectangular and circular waveguides, the eigen-value equation which is an analytical method and numerical solution, when finding solution for the resonator, Maxwell equation in is need to be created in a spherical coordinate system, because the complexity of the spherical coordinate fielder equation, has not found anyone using eigen-value method to calculated the distribution of the resonant field. Only find in Paper [4] using asymptotic method for conical waveguide. That method assume a equivalent radius ae, believes field of wavefront sphere of cone waveguide Eo,EФ,Ho,HФ can use its wavefront position radius ae equivalent circular waveguide field Er,EФ,Hr,HФ, this method of finding the field distribution within the conical resonator can be used as reference, but the accuracy reduced as the cone half opening angle increases. Using finite element to numerically simulate the Maxwell electromagnetic equation for the idealised conical resonator, the distribution of electromagnetic can be obtained directly, this method is not limited by the cavity structure and microwave mode."

Quote
By keep the diameter of the Small End constant, increase the large end of the cavity, in order to have the same resonant frequency, cavity height must be reduced, quality factor also reduce.

They're basically saying: (1) That I'm hosed trying to calculate such things. Simulating the conical frustum using FEM software is the way to go. Which I simply don't have access to. (2) Also they're saying that as the opening angle opens up, approximating the resonant modes becomes more and more difficult. (3) You have to shorten the cavity height as opening angle increases to maintain resonance at desired frequency, but it lowers Q. So I should probably (for now) re-think using such wide opening angles.

So I'm switching gears a bit using what I've learned from the above reporting.
(1) Keeping Cannae in mind, who says we need a cone anyway? We've discussed the commonality between Shawyer and Cannae in thread 1. http://forum.nasaspaceflight.com/index.php?topic=29276.msg1298712#msg1298712 So I'm thinking it would be smart to use what we've learned about cylinders and try a cylinder experiment.
(2) Instead of trying to optimize right out of the gate by throwing around light cones. It would be smarter to use the dims we already have for Shawyer experimental and demo, which you provided on the previous page.
(3) If I ever get this build going, I'm going to have to easter egg it anyway using a sig-gen and a power meter to find the resonant frequency (treat it like a filter, tune it until I get get an output from sample port), so I don't need to have exact calculations. I just need to be close enough to be within tunable limits.

1) Concerning a solution for the truncated cone EM Drive (the geometry used by Shawyer, NASA and Juan Yang in China), I can report the following progress.  I have derived an exact, closed-form solution for the integral of the longitudinal wavenumber "kz" (as proposed by @NotSoSureOfIt and hinted in the 1969 patent of Wolf) for a truncated cone or a cone. The solution contains square root terms and ArcTan terms.  There is no question that when an exact solution is available it is always superior to any numerical method like Finite Element, Finite Difference, Boundary Element, etc. Even when exact solutions are not available, it is a standard methodology in any engineering department to start the design process with exact solutions to simplified geometries because of the considerable amount of time that it takes to generate a converged solution with Finite Element, Finite Difference, etc. Also because one can gain a much better understanding of the problem with a closed-form solution (to quickly understand the influence of parameters).

2) It is not possible to solve the eigenvalue problem for the truncated cone cavity, or for a cone cavity, directly in terms of standard functions of the frequency (or the mode shape) because the longitudinal wavenumber kz expression for the truncated cone or a cone cannot be inverted (since the frequency is nonlinearly embedded in a number of square root and ArcTan terms).

3) The eigenvalue problem for the truncated cone can be solved by obtaining a numerical solution (finding roots of the nonlinear equation that arises from equating the longitudinal wavenumber "kz" to the mode shape quantum number "p" = 0,1,2,3,4...).

4) I will post the solution and numerical results in more detail, but meanwhile here is the main result:

the exact solution for the truncated cone gives results that are less than 1% different (for typical geometries with a cone angle equal or less than 20 degrees from the longitudinal axis -comprising the NASA Brady et.al. , China (Juan Yang) and the UK (Shawyer) experiments) from the exact solution of a cylinder, if the cylinder diameter is expressed as the GeometricMean of the big and small diameters of the truncated cone.  I have explored several mean measures, for example:

H, the Harmonic Mean ( https://en.wikipedia.org/wiki/Harmonic_mean ) ,
G, the Geometric Mean ( https://en.wikipedia.org/wiki/Geometric_mean ),
L, the Logarithmic Mean ( https://en.wikipedia.org/wiki/Logarithmic_mean ),
A, the Arithmetic Mean ( https://en.wikipedia.org/wiki/Arithmetic_mean ),
V, the Volumetric Mean (http://forum.nasaspaceflight.com/index.php?topic=36313.msg1319655#msg1319655 ),
R, the Root Mean Square ( https://en.wikipedia.org/wiki/Root_mean_square ),
C, the Contraharmonic mean ( https://en.wikipedia.org/wiki/Contraharmonic_mean ),

where:

SmallDiameter < H < G < L < A < V < R < C < BigDiameter

The closest results to the exact solution for the truncated cone are obtained using the GeometricMean of its big and small diameters, as the equivalent diameter of the cylinder cavity equation.  I also found the value of the exponent of the Stolarsky Mean ( https://en.wikipedia.org/wiki/Stolarsky_mean )  that minimizes the error even further than the Geometric Mean, but the improvement is not dramatic and the Stolarsky exponent depends on the cone geometry, hence, for simplicity, one might as well use the GeometricMean=Sqrt[BigDiameter*SmallDiameter] in the cylinder equation to model the frequency and mode shapes of the truncated cone.

(http://www.gogeometry.com/geometry/geometric_mean_arithmetic.gif)
(http://forum.nasaspaceflight.com/index.php?action=dlattach;topic=29276.0;attach=620202;image)
Title: Re: EM Drive Developments - related to space flight applications - Thread 2
Post by: francesco nicoli on 02/04/2015 03:01 pm
As for the previous one, the level and quality of these comments is so high that the thread has become unreadable for non-physicists.
Could you eventually, for the sake of not losing the majority of us too much behind, make a quick non-technical summary of what are you discussing? is there any progress or barely nothing?

Thanks! :)
Title: Re: EM Drive Developments - related to space flight applications - Thread 2
Post by: Rodal on 02/04/2015 03:17 pm
As for the previous one, the level and quality of these comments is so high that the thread has become unreadable for non-physicists.
Could you eventually, for the sake of not losing the majority of us too much behind, make a quick non-technical summary of what are you discussing? is there any progress or barely nothing?

Thanks! :)

1) We are exploring the validity of the claims of the different experimenters (NASA in the USA, Shawyer in the UK and Juan Yang in China) by systematically analyzing their experiments.

2) Some of the people in this forum are also designing and working to make their own prototypes and experiments.

3) One of the people in this forum (@NotSoSureOfIt), has made an outstanding contribution by independently deriving an equation that is not far from the claimed experimental results.

4) There are a number of possible physical reasons for the experimental results to be valid for space propulsion as well as for the results to be an experimental artifact that may not produce any propulsion in space.

5) There have been no further reports from NASA on the experiments that were supposed to be replicated at their other centers (JPL and Glenn) or at John Hopkins University.  Actually @wembley, who is an aerospace technology reporter, reports that it is his opinion that NASA has a "news blackout" on this matter, and China's Juan Yang is not saying much either.  The Chinese seem to be much further along than NASA, as they reported much greater thrust and they have conducted more thorough experiments (including being the first to numerically report the effects of temperature and temperature gradients with thermocouples embedded in the metal).  Shawyer in the UK seems to be much further ahead than NASA as he claims he is exploring a superconducting design.  Shawyer (UK) made a presentation in Canada late last year where he showed his latest design (using superconductivity) which he claims will result in much greater thrust/PowerInput (see image below).
Title: Re: EM Drive Developments - related to space flight applications - Thread 2
Post by: tchernik on 02/04/2015 04:02 pm

A potentially interesting sidelight:

http://okomov.livejournal.com/577.html

The physics is descrived in vvery different terms ("Leonov’s superunification theory"?? "Antigravity"???) this has similar performance to what Shawyer predicts for a superconducting EmDrive thruster, i.e. 500 to 700 kg for 1kW power input.

Moreover, as soon as it starts to accelerate,  the thrust ceases, hence the pulsed operation in the video, which is what Shawyer claims for a high Q EmDrive thruster without Doppler compensation.

What does the team think...?

I'm a newbie and not an expert, but I can give my opinion nonetheless: another proposal that needs strong evidence to give it any credence.

The material is somewhat scarce on technical details about the posited thruster, except the description of the author's theory.

It's not clear for me if this is another version of the Emdrive or something very different.

The videos of the moving chariot with inert wheels are interesting, but they can be done with many other things here on Earth, by using friction.
Title: Re: EM Drive Developments - related to space flight applications - Thread 2
Post by: saucyjack on 02/04/2015 05:46 pm
Long-time lurker here; I have to thank you all for your tireless contributions with very little data to go on.  While it's too early to tell if anything will come of this, the potential payoff is earth-changing. 

It does appear to me that @wembley is correct and there is indeed a de facto, if not formal, news blackout from NASA.  Given the enormous level of interest in this over the last few months, isn't it odd that we haven't seen so much as a terse announcement to the effect of "We decided not to test this further at JPL after all, due to the fact we think it's nonsense, we have budget constraints, etc.".  But as far as we know, they have said nothing at all since the initial announcement.

For people more familiar with NASA, is that common?  Does NASA as a civilian agency have a history (or policy?) of suppressing experimental results that, like this, may have military applications?   In other words, can the lack of any news at all one way or the other, be taken as a sign that there might be something to this?  Or is that just conspiratorial nonsense?
Title: Re: EM Drive Developments - related to space flight applications - Thread 2
Post by: DIYFAN on 02/04/2015 06:11 pm
For people more familiar with NASA, is that common?  Does NASA as a civilian agency have a history (or policy?) of suppressing experimental results that, like this, may have military applications?   In other words, can the lack of any news at all one way or the other, be taken as a sign that there might be something to this?  Or is that just conspiratorial nonsense?

Rockets have military applications, but NASA does not suppress experimental results of rockets.  To the contrary, they partner with companies such as SpaceX and share information for the advancement of space exploration.  I'm not convinced there is a black out for EM drive technology.  There appears to be a delay, which could have many causes.  If there are people in the administration advocating for a black out, I think it is a mistake, and probably too late given the amount of information already in the public domain.
Title: Re: EM Drive Developments - related to space flight applications - Thread 2
Post by: matthewpapa on 02/04/2015 06:20 pm
This is the latest official NASA media we have I think

Dr. Harold "Sonny" White - Eagleworks Laboratories: Advanced Propulsion
https://www.youtube.com/watch?v=Wokn7crjBbA

NASA Ames Research Director’s Colloquium, August 12, 2014

It is basically a rehash of the statements released in July, but gives some spaceflight context. In this presentation Dr White is very matter of fact about this concept. He seems to leave little doubt on how he feels about the effect's existence.

I hope he is right. He talks about some of the mission profiles.

But this year we have not heard anything since. It has been nearly 6 months, surely they know yay or nay by now. As some readers have pointed out, perhaps it has gone underground? Or maybe the results are so incredible that no one wants to be the first to report it for fear of ruining their career if they are wrong. If thats the case we just need someone to break the dam.
Title: Re: EM Drive Developments - related to space flight applications - Thread 2
Post by: Mulletron on 02/04/2015 06:27 pm
Interesting account relating to the history of Nasa and Darpa involvement with developing/testing advanced propulsion concepts.
http://www.bbc.com/future/story/20120321-searching-for-a-starship

There is a lot of intriguing reading if you research the history and associations of the people named in the article.
Title: Re: EM Drive Developments - related to space flight applications - Thread 2
Post by: Star One on 02/04/2015 06:38 pm
For people more familiar with NASA, is that common?  Does NASA as a civilian agency have a history (or policy?) of suppressing experimental results that, like this, may have military applications?   In other words, can the lack of any news at all one way or the other, be taken as a sign that there might be something to this?  Or is that just conspiratorial nonsense?

Rockets have military applications, but NASA does not suppress experimental results of rockets.  To the contrary, they partner with companies such as SpaceX and share information for the advancement of space exploration.  I'm not convinced there is a black out for EM drive technology.  There appears to be a delay, which could have many causes.  If there are people in the administration advocating for a black out, I think it is a mistake, and probably too late given the amount of information already in the public domain.

To be fair to NASA if this does work it's such a revolutionary technology that the reporting of it is not something to be rushed into. No doubt every result is having to be checked, checked again & then checked again etc etc which no doubt slows down the release of information. Especially if on top of that there are national security implications that have to be considered as well.
Title: Re: EM Drive Developments - related to space flight applications - Thread 2
Post by: Mulletron on 02/05/2015 05:26 pm
Anyone have the pdf?
http://arc.aiaa.org/doi/abs/10.2514/6.2014-3853

Also not exactly mainstream news but interesting nonetheless.
http://m.disclose.tv/news/NASA_Impossible_Quantum_Space_Engine_Actually_Works/113761#DTV
Title: Re: EM Drive Developments - related to space flight applications - Thread 2
Post by: OSE on 02/05/2015 05:39 pm
Anyone have the pdf?
http://arc.aiaa.org/doi/abs/10.2514/6.2014-3853

Also not exactly mainstream news but interesting nonetheless.
http://m.disclose.tv/news/NASA_Impossible_Quantum_Space_Engine_Actually_Works/113761#DTV

I have access to that conference paper. I won't give it out but if you have any specific questions about the contents I can try to answer them. (This topic is outside my area of expertise)

I only very briefly skimmed over the paper but it has a very amateur feel to it. For example the author claims ``numerical method analysis'' was used. Towards the end of the paper they clarify that they used the software packages HFSS and Analyst under an evaluation license. This does not give me high hopes that they were used correctly.
Title: Re: EM Drive Developments - related to space flight applications - Thread 2
Post by: wembley on 02/06/2015 07:32 am
Anyone have the pdf?
http://arc.aiaa.org/doi/abs/10.2514/6.2014-3853


Yes, anything in particular you were after?

You may also want to look at his patents, eg from 2012

https://www.google.com/patents/US20140013724?dq=cannae+fetta&hl=en&sa=X&ei=4HvUVKaFIauV7AaCqYHABA&ved=0CCQQ6AEwAQ

and the earlier 2007 version which references Shawyer
https://www.google.com/patents/WO2007089284A2?cl=en&dq=fetta+thruster&hl=en&sa=X&ei=QHzUVJjuAsWP7AbMy4GYBQ&ved=0CEIQ6AEwBQ


Title: Re: EM Drive Developments - related to space flight applications - Thread 2
Post by: Star-Drive on 02/06/2015 12:58 pm
For people more familiar with NASA, is that common?  Does NASA as a civilian agency have a history (or policy?) of suppressing experimental results that, like this, may have military applications?   In other words, can the lack of any news at all one way or the other, be taken as a sign that there might be something to this?  Or is that just conspiratorial nonsense?

Rockets have military applications, but NASA does not suppress experimental results of rockets.  To the contrary, they partner with companies such as SpaceX and share information for the advancement of space exploration.  I'm not convinced there is a black out for EM drive technology.  There appears to be a delay, which could have many causes.  If there are people in the administration advocating for a black out, I think it is a mistake, and probably too late given the amount of information already in the public domain.

To be fair to NASA if this does work it's such a revolutionary technology that the reporting of it is not something to be rushed into. No doubt every result is having to be checked, checked again & then checked again etc etc which no doubt slows down the release of information. Especially if on top of that there are national security implications that have to be considered as well.

Folks:

The Eagleworks Lab is still working on the copper frustum thruster that was reported on last summer at the AIAA/JPC.  We have now confirmed that there is a thrust signature in a hard vacuum (~5.0x10^-6 Torr) in both the forward direction, (approx. +50 micro-Newton (uN) with 50W at 1,937.115 MHz), and the reversed direction, (up to -16uN with a failing RF amp), when the thruster is rotated 180 degrees on the torque pendulum.  However we continue to fight through RF amplifier failures brought on by having to operate them in a hard vacuum with few $$$ resources to fix them when they break, so the desired data is coming along very slowly.  We are still working on obtaining enough data though that will allow us to go to Glenn Research Center (GRC) for a replication effort in the next few months.  However that will only happen if we can make the thrust signature large enough since the GRC thrust stand can only measure down to ~50uN, so we have to get the thrust signature up to at least 100uN before we can go to GRC. 

As to the theoretical side of Q-Thrusters, Dr. White has just developed the first cut at a quantum vacuum (QV) based plasma code written in C+ under Windows/Unix and VMD visualization software that utilizes the COMSOL E&M derived field data for a given thruster geometry that allows one to track the movement and velocity of a subset of the QV's electron/positron neutral plasma pairs in the thruster over time as they respond to the applied time varying RF E&M fields in the copper frustum resonant cavity and to each other.  This package also allows one to calculate the expected thrust for a given input power and quality factor of the frustum resonant cavity based of standard plasma rocket physics.  So far the estimated thrust verses experimental observations are within 2% for the first experimental data run I compared it to, but we still have a long, long road ahead of us of experimental validation before we have any real confidence in this very new Q-Thruster design tool.

Best, Paul March   
Title: Re: EM Drive Developments - related to space flight applications - Thread 2
Post by: Rodal on 02/06/2015 02:44 pm
Wanted to examine the inventor's theory of operation. We spent a lot of time discussing Shawyer's theory, but Fetta didn't get much discussion. Probably because his paper is paywalled. Also @Wembley, thanks for the patents. There's plenty of free info there.

Break:
Neat I can hit zero every half wavelength with any frequency like this. (top)
Or if I mirror the diagonal of a cylinder, creating a cone I get pretty close sometimes after adjusted down 1 degree. (bottom)

http://goo.gl/jF8ZJB (shortened link to ebay)
Found these puppies. Emailed the seller to see what the dimensions are and to see if they are the manufacturer and whether they do custom sizes. Might be cheaper and less headache than buying a copper sheet.

Something very nice about the exact solution I obtained for the  frequencies and mode shapes of a truncated cone, or for a cone, is that it implicitly, automatically, incorporates the condition of cutoff frequencies.  The exact solution for a cylinder in Wikipedia (http://en.wikipedia.org/wiki/Microwave_cavity#Cylindrical_cavity) does not explicitly refer to the need for the condition of cutoff frequencies, and therefore the cutoff frequency condition  (http://en.wikipedia.org/wiki/Cutoff_frequency ) has to be included in (http://en.wikipedia.org/wiki/Microwave_cavity#Cylindrical_cavity) as an additional condition (which we did not do in previous discussions of mode shapes: some of the mode shapes we discussed for certain geometries do not have Q resonance because they are cutoff ! ).  Not all values of the longitudinal mode shape quantum number "p"=0,1,2,3,... are allowed.  Low values of "p" (starting with p=0) are disallowed by the cutoff condition for small diameters.

The cutoff frequency condition (http://en.wikipedia.org/wiki/Cutoff_frequency )  is that the frequency of the cavity has to be greater than the speed of light in the medium times the appropriate zero Bessel function , divided by Pi*Diameter:

fmnp > cmedium Xfunction / (Diameter * Pi)

where

cmedium = c / Sqrt[epsilonr * mur] where epsilonr is the relative electric permittivity (dielectric constant), and mur is the relative magnetic permeability, and c  is the speed of light in vacuum

and where

Xfunction=Xmn for TM modes and Xfunction = X'mn for TE modes

Now, for a cylinder it is clear what "diameter" means, since the diameter is constant along the length.  However, what is the cutoff condition for a truncated cone?  This is included in the exact solution I obtained for a truncated  cone and it is simply dictated by the small diameter of the truncated cone (which one would expect based on physical reasons):

fmnp > cmedium Xfunction / (Dsmall Pi)

where Xfunction=Xmn for TM modes and Xfunction = X'mn for TE modes

This has very important consequences for a truncated cone, because as the small Diameter approaches zero, the cutoff frequency approaches infinity.  This means that for a pointy cone, there can be no resonant Q solution possible.  A pointy cone can only have evanescent waves (since only real solutions to the eigenvalue equation are standing-wave resonant Q solutions, and the complex value solutions to the eigenvalue equation are evanescent waves).


EDIT:
This can also be stated as a cutoff wavelength, where the wavelength lambdamnp is defined as the ratio of the speed of light in the medium, to the frequency fmnp:

 lambdamnp = cmedium / fmnp

substituting this, we obtain the following condition

 lambdamnp <  (Dsmall * Pi) / Xfunction 

the wavelength for a given mode mnp needs to be smaller than the smallDiameter*Pi /Xfunction

longer wavelengths get cutoff



Another very important consequence is that the cutoff frequency:

fmnp > cmedium Xfunction / (Dsmall Pi)

implies a minimum diameter for the truncated cone to support a given mode shape at a given frequency:

Dsmall > cmedium Xfunction / ( fmnp Pi)


where Xfunction=Xmn for TM modes and Xfunction = X'mn for TE modes

For example, if one wants resonance at fmnp = 2.45 GHz, as @aero was proposing, then it immediately follows that the minimum diameter has to be

Dsmall > cmedium Xfunction / ( 2.45 GHz)

Since the Xfunction increases with m and n (except for one particular X' value for the TE mode with m=1:  X'1n or TE1n) , small diameters cut off frequencies such that the mn modes can only occur at higher p values, that's why @aero found that the longitudinal mode shape quantum number p=3 for the NASA truncated cone with the dielectric, instead of p=0 or p=1 or p=2. 



Dsmall > cmedium Xfunction / ( fmnp Pi)

or, since cmedium = c / Sqrt[epsilonr * mur]

Dsmall > c * Xfunction / (Sqrt[epsilonr * mur] * fmnp * Pi)

The higher the value of the dielectric constant epsilonr (everything else held constant), the lower the Diameter can be for a given frequency

The higher the value of the dielectric constant epsilonr (everything else held constant), the lower the frequency will be for a given mode shape mnp.


This clearly shows the function of the dielectric in the EM Drive: the dielectric allows mode shapes to occur at frequencies that would otherwise be cutoff for a given geometry.


The dielectric allows mode shapes to occur for smaller values of the small diameter. 

Also, the cutoff frequency condition clearly shows why NASA  and Shawyer located the dielectric at the small end of the cavity, since the cutoff frequency is a function of the the dielectric constant, such that lower dielectric constant allows for lower cutoff frequencies or equivalently smaller diameters. The best  place to put the dielectric is at the small diameter in order to allow mode shapes to occur that would otherwise be precluded because of the cutoff frequency condition.


It is very interesting that there is a "sweet spot" of geometries for these EM Drives: too large a diameter leads to too many mode shapes very close to each other (and hence very difficult to tune and keep the EM Drive at a given resonant frequency) while too small diameters lead to cutoff of mode shapes.  A pointy cone cannot have Q resonant modes, a pointy cone can only have evanescent waves.  The closest one can come up to a pointy cone Q resonance would be to calculate and design a geometry with a dielectric located at the small end and a small but finite Dsmall diameter.  Dsmall must be greater than zero, and for practical solutions this implies a minimum diameter.

(http://i.ebayimg.com/00/s/ODQ5WDg2Ng==/$(KGrHqZHJCkE63YKjOZMBO65s07Fn!~~60_35.JPG)

(http://forum.nasaspaceflight.com/index.php?action=dlattach;topic=29276.0;attach=620202;image)

I will write more on this and give numerical examples and mode shapes for the NASA and the Shawyer experiments.  It is all falling into place very nicely now  :)
Title: Re: EM Drive Developments - related to space flight applications - Thread 2
Post by: Mulletron on 02/06/2015 03:11 pm
Here's what I know about cutoff freqs of circular waveguide at least:
"The cutoff wavelength of a circular guide is 1.71 times the diameter of the
waveguide. Since the "a" dimension of a rectangular waveguide is approximately one half-wavelength at
the cutoff frequency, the diameter of an equivalent circular waveguide must be 2 ÷ 1.71, or approximately
1.17 times the "a" dimension of a rectangular waveguide."

Page 1-26
http://www.navymars.org/national/training/nmo_courses/nmo1/module11/14183_ch1.pdf

I've set the small diameter as 1 wavelength for the top cone. The bottom was set from calculating cylinders, so the guide wavelength in diameter is different than multiples of .5. See previous post.

The whole point of this exercise is to calculate the diameter of the "blanks" to insert into a cone of X angle and be resonant at the desired frequency and mode. I'm trying to figure this out by holding the small diameter as a constant.

Break:
@ Paul March, thank you for chiming in. All of us are chomping at the bit for ANY news whatsoever. Maybe Eagleworks could leverage its social media presence (https://www.facebook.com/eagleworksnasa) to keep the public engaged. As far as the lack of $$$ and resources go, I understand that Eagleworks works on a shoestring budget. Wish we could help. It probably isn't acceptable for us to try and crowdfund a government agency. Hopefully you achieve results that will turn heads. Godspeed.

Title: Re: EM Drive Developments - related to space flight applications - Thread 2
Post by: RotoSequence on 02/06/2015 03:12 pm
It is all falling into place very nicely now  :)

https://www.youtube.com/watch?v=EPX5IBbfmEQ  ;)

In theory, could a dialectric be used to suppress resonance modes that might otherwise be present in an over-sized cavity?
Title: Re: EM Drive Developments - related to space flight applications - Thread 2
Post by: Mulletron on 02/06/2015 03:58 pm

http://goo.gl/jF8ZJB (shortened link to ebay)
Found these puppies. Emailed the seller to see what the dimensions are and to see if they are the manufacturer and whether they do custom sizes. Might be cheaper and less headache than buying a copper sheet.

Got a supplier! They will do custom sizes and have their own shop. What they make is far superior than what I could slap together.

Quote
"apx. dimensions small= 6 5/8" x 3 3/8" medium= 8" x 5" large=11" x 7 3/4" all are 7 3/4" tall . yes we do make them @ our sheet metal shop & sure we can make custom sizes. Thanks Rick"
Title: Re: EM Drive Developments - related to space flight applications - Thread 2
Post by: Rodal on 02/06/2015 04:01 pm
..

Folks:

The Eagleworks Lab is still working on the copper frustum thruster that was reported on last summer at the AIAA/JPC.  We have now confirmed that there is a thrust signature in a hard vacuum (~5.0x10^-6 Torr) in both the forward direction, (approx. +50 micro-Newton (uN) with 50W at 1,937.115 MHz), and the reversed direction, (up to -16uN with a failing RF amp), when the thruster is rotated 180 degrees on the torque pendulum.  However we continue to fight through RF amplifier failures brought on by having to operate them in a hard vacuum with few $$$ resources to fix them when they break, so the desired data is coming along very slowly.  We are still working on obtaining enough data though that will allow us to go to Glenn Research Center (GRC) for a replication effort in the next few months.  However that will only happen if we can make the thrust signature large enough since the GRC thrust stand can only measure down to ~50uN, so we have to get the thrust signature up to at least 100uN before we can go to GRC. 

As to the theoretical side of Q-Thrusters, Dr. White has just developed the first cut at a quantum vacuum (QV) based plasma code written in C+ under Windows/Unix and VMD visualization software that utilizes the COMSOL E&M derived field data for a given thruster geometry that allows one to track the movement and velocity of a subset of the QV's electron/positron neutral plasma pairs in the thruster over time as they respond to the applied time varying RF E&M fields in the copper frustum resonant cavity and to each other.  This package also allows one to calculate the expected thrust for a given input power and quality factor of the frustum resonant cavity based of standard plasma rocket physics.  So far the estimated thrust verses experimental observations are within 2% for the first experimental data run I compared it to, but we still have a long, long road ahead of us of experimental validation before we have any real confidence in this very new Q-Thruster design tool.

Best, Paul March
Thanks Paul for an excellent update, much appreciated.

Congratulations to the Eagleworks team for obtaining experimental force measurements in the torsional pendulum in a hard vacuum (~5.0x10^-6 Torr) !

(http://i262.photobucket.com/albums/ii105/scoozna/misc/applause.gif)

It would be most helpful to the scientific/technical community if you could provide the dimensions of the frustum of a cone (truncated cone) used in the Brady et.al. "Anomalous ..." report.

Of several estimates of the geometry, we have established that this is the best estimate so far:

Aero Best estimate as of 11/9/2014    http://forum.nasaspaceflight.com/index.php?topic=29276.msg1285896#msg1285896
   
cavityLength = 0.24173 m
bigDiameter = 0.27246 m
smallDiameter = 0.15875 m

where "cavity length" is the axial length of the frustum measured along the longitudinal axis of the cone, perpendicular to both the small and big diameters.  In other words, the dimension labeled "h" in the following image: (http://offshoremechanics.asmedigitalcollection.asme.org/data/Journals/JMOEEX/28400/031603_1_1.jpeg)

It would be most helpful if you could provide the actual (internal) dimensions of the frustum (or at least if you could provide its approximate dimensions).

Thanks  :)
Title: Re: EM Drive Developments - related to space flight applications - Thread 2
Post by: Rodal on 02/06/2015 04:12 pm
It is all falling into place very nicely now  :)

...

In theory, could a dialectric be used to suppress resonance modes that might otherwise be present in an over-sized cavity?

The dielectric reduces the frequencies at which resonance occurs, so it helps to avoid cutoff of mode shapes, and therefore there are more mode shapes with the dielectric (looking at all frequencies) if the relative permittivity > 1. Looking at all frequencies, if a given geometry has mode shapes cut off without the dielectric, the inclusion of a dielectric can bring some of the mode shapes back.  This is only possible within the available material properties, for example for polymers, it reduces the frequencies (compared to the ones in air) by a factor of Sqrt[2.3] = 1.52.  From the list below, Neoprene Rubber seems to have the highest dielectric constant of the solid materials in the list, it appears that Neoprene Rubber may reduce frequencies by a factor of Sqrt[6]=2.45. The liquids seem to have much higher dielectric constants.

The Wikipedia list ( See http://en.wikipedia.org/wiki/Relative_permittivity for a list of relative permittivites.) has listed "Calcium copper titanate" as having a dielectric constant of > 250,000 ! which would imply a factor of Sqrt[250000]=500 times !.  It gives this link ( http://oatao.univ-toulouse.fr/698/1/boulos_698.pdf ) that describes this multi-phase ceramic material.

Unfortunately, relative permittivity strongly decreases with increasing frequency.  Inspection of the graphs in ( http://oatao.univ-toulouse.fr/698/1/boulos_698.pdf ) shows that the value of relative permittivity of this ceramic at frequencies > 10^5 Hz is less than 10,000  (Sqrt[10,000]=100).  Unfortunately, it does not give values in the range of interest for EM Drives (10^9 Hz.)

To supress resonant modes (as compared to air or vacuum) one would need a dielectric with a relative permittivity less than one (resulting in an equivalent speed of light in the medium greater than the speed of light in vacuum "c" ).

(http://www.thelearningpit.com/hj/plcs_files/plcs-72.gif)
Title: Re: EM Drive Developments - related to space flight applications - Thread 2
Post by: Rodal on 02/06/2015 05:40 pm
Here's what I know about cutoff freqs of circular waveguide at least:
"The cutoff wavelength of a circular guide is 1.71 times the diameter of the
waveguide. ...
The US Navy reference should be corrected to read instead:

"The cutoff wavelength for the dominant mode TE11 of a circular guide is 1.71 times the diameter of the waveguide."

The dominant mode of a circular waveguide or cavity is the mode with the lowest value of Xfunction.  This corresponds to X'11 which has a value of X'11  = 1.84118 (see:  http://wwwal.kuicr.kyoto-u.ac.jp/www/accelerator/a4/besselroot.htmlx ) . Therefore the dominant mode is TE11.

Using the equation in my post  (  http://forum.nasaspaceflight.com/index.php?topic=36313.msg1326637#msg1326637 )

Dsmall > (cmedium * Xfunction) / (fmnp * Pi)

and since the wavelength is

lambda = cmedium /  fmnp

where cmedium is the speed of light in the medium and fmnp is the frequency

this is equivalent to

Dsmall > (lambda* Xfunction) /  Pi

For the dominant mode TE11, Xfunction =  X'11  = 1.84118 hence

Dsmall > (lambda*  1.84118) /  Pi = lambda / 1.7063

or inverting:

lambda < 1.71 * Dsmall

"The cutoff wavelength for the dominant mode  TE11 of a circular guide is 1.71 times the diameter of the waveguide."

quod erat demonstrandum




For a truncated cone, I would state:

"The cutoff wavelength for the dominant mode TE11 of a truncated cone cavity is 1.71 times the small diameter of the truncated cone."




Contour Plot of mode shape TE11:

Electrical field ____________(solid lines)

Magnetic field - - - - - - - - - - - (dashed lines)
Title: Re: EM Drive Developments - related to space flight applications - Thread 2
Post by: Mulletron on 02/06/2015 06:40 pm
Got the sample pack in the mail today. Will report on performance of 10 or 16 mil soon, specifically the ability to hold shape under its own weight while being as light as possible. Now, since I found the supplier, who uses heavier 22 Mil/16 Ounce Copper (see above), these will probably end up as end caps. I don't know what I'm going to do yet, buy or roll'n'solder myself. There's tradeoffs to consider. Like time and weight. So shiny  :)

Edit:
It just occurred to me that I should have enough material in hand right now to make something happen if I can find a solution to a cone based off this cylinder: http://forum.nasaspaceflight.com/index.php?topic=36313.msg1320981#msg1320981

What do you think @Rodal? Do we have enough to confidently calculate a frustum of a cone that works and will fit within a 12"x12" sheet? The cylinder fits at least. I'm quite happy staying with 2.45ghz, TE111 for now. I don't care about finding the best mode for max thrust, just any thrust.
Title: Re: EM Drive Developments - related to space flight applications - Thread 2
Post by: Rodal on 02/06/2015 07:42 pm
Got the sample pack in the mail today. Will report on performance of 10 or 16 mil soon, specifically the ability to hold shape under its own weight while being as light as possible. Now, since I found the supplier, who uses heavier 22 Mil/16 Ounce Copper (see above), these will probably end up as end caps. I don't know what I'm going to do yet, buy or roll'n'solder myself. There's tradeoffs to consider. Like time and weight. So shiny  :)

Edit:
It just occurred to me that I should have enough material in hand right now to make something happen if I can find a solution to a cone based off this cylinder: http://forum.nasaspaceflight.com/index.php?topic=36313.msg1320981#msg1320981

What do you think @Rodal? Do we have enough to confidently calculate a frustum of a cone that works and will fit within a 12"x12" sheet? The cylinder fits at least. I'm quite happy staying with 2.45ghz, TE111 for now. I don't care about finding the best mode for max thrust, just any thrust.

First congratulations !

My main concern is being able to tune the EM Drive cavity to achieve Q resonance in the experiment. 
Based on NASA's results I think that only the TE modes should provide a force measurement.  I don't think that the TM modes will provide a force measurement.
As you know, the magnetron is not going to exactly deliver the nominal  frequency of 2.45Ghz, and geometry imperfections are going to affect the natural frequency so that it will be different than the natural frequency of a perfect geometrical cone.   Shawyer for the Demo had and adjustable end at the small end of the truncated cone that allowed him to mechanically tune the resonant, natural frequency of the drive (to match his nominal 2.45GHz excitation frequency). 

(http://www.wired.com/images_blogs/dangerroom/images/2008/09/24/emdrive_2.jpg)

NASA had the ability to drive the cavity at different frequencies in order to find the natural frequency resonace "sweet spot"...
(http://d34s6f1vnlnqxq.cloudfront.net/11015598/e9f77817-25f6-444b-bc2c-7415f8edd698/norm/172484402_8.png)


 It would be ideal to have both of those abilities as well  :)
Title: Re: EM Drive Developments - related to space flight applications - Thread 2
Post by: Mulletron on 02/06/2015 07:58 pm
Not using a magnetron. I have test equipment. I can find the resonant frequency if we can calculate geometry close enough. The question is, in your opinion, is the math there to make an exact calculation yet?
Title: Re: EM Drive Developments - related to space flight applications - Thread 2
Post by: Rodal on 02/06/2015 08:08 pm
Not using a magnetron. I have test equipment. I can find the resonant frequency if we can calculate geometry close enough. The question is, in your opinion, is the math there to make an exact calculation yet?
Definitely yes.   :)

Please wait another week for me to post the numerical results for NASA and Shawyer vs equations.
Title: Re: EM Drive Developments - related to space flight applications - Thread 2
Post by: Mulletron on 02/06/2015 09:21 pm
Brainstorming some ideas here. So as far as an at home Cavendish, I want something that has the capability of spinning for days/weeks on wireless power (I understand the wireless power may affect the experiment so I'll have to control for it.) without getting wound up. I was thinking spherical magnets at the very top of the fishing line. Any thoughts? Or something better? The reason for this is. I'm using ultra low power, like 100mw low. The Rf source is going to have to ride along. Available electrical power will limit available rf power. Indeed if there is any thrust, it will be much smaller than anyone ever measured before. If it even works at all, it will probably take a long time to become evident on the balance. I want to give the device time to gain speed and keep going before I stop it and flip the "thruster" over. If it gets wound up, it'll just oscillate.

https://www.fourmilab.ch/gravitation/foobar/
http://www.intalek.com/Index/Projects/Research/CavendishExperiment.htm
http://www.ebay.com/bhp/sphere-magnets
http://www.amazon.com/Beautyforall-Wireless-Supply-Module-Charger/dp/B00L19HE5I/ref=sr_1_37?ie=UTF8&qid=1423260545&sr=8-37&keywords=wireless+power
Title: Re: EM Drive Developments - related to space flight applications - Thread 2
Post by: matthewpapa on 02/06/2015 10:13 pm
Break:
@ Paul March, thank you for chiming in. All of us are chomping at the bit for ANY news whatsoever. Maybe Eagleworks could leverage its social media presence (https://www.facebook.com/eagleworksnasa) to keep the public engaged. As far as the lack of $$$ and resources go, I understand that Eagleworks works on a shoestring budget. Wish we could help. It probably isn't acceptable for us to try and crowdfund a government agency. Hopefully you achieve results that will turn heads. Godspeed.

I would like to second this sentiment. Updates would be great!
Paul, If you feel crowdfunding would be worth it I am sure you could get lots of donors (including me). But like Mullerton said, may not be possible due to red tape.
And if not that we can call our congressmen and ask them to get this funded (for what its worth)
Title: Re: EM Drive Developments - related to space flight applications - Thread 2
Post by: Mulletron on 02/07/2015 04:22 am
Well here's an open one...... http://scharstein.eng.ua.edu/electromagnetic.pdf

Title: Re: EM Drive Developments - related to space flight applications - Thread 2
Post by: supersubie on 02/07/2015 10:22 am
Found a nice little update article on nextbigfuture about the EM drive work. Dunno if you guys have seen this already but it might be of interest to a few of you following along intermittently! Enjoy.

http://nextbigfuture.com/2015/02/update-on-emdrive-work-at-nasa.html#more
Title: Re: EM Drive Developments - related to space flight applications - Thread 2
Post by: Jilpeter on 02/07/2015 10:59 am
For people more familiar with NASA, is that common?  Does NASA as a civilian agency have a history (or policy?) of suppressing experimental results that, like this, may have military applications?   In other words, can the lack of any news at all one way or the other, be taken as a sign that there might be something to this?  Or is that just conspiratorial nonsense?

Rockets have military applications, but NASA does not suppress experimental results of rockets.  To the contrary, they partner with companies such as SpaceX and share information for the advancement of space exploration.  I'm not convinced there is a black out for EM drive technology.  There appears to be a delay, which could have many causes.  If there are people in the administration advocating for a black out, I think it is a mistake, and probably too late given the amount of information already in the public domain.

To be fair to NASA if this does work it's such a revolutionary technology that the reporting of it is not something to be rushed into. No doubt every result is having to be checked, checked again & then checked again etc etc which no doubt slows down the release of information. Especially if on top of that there are national security implications that have to be considered as well.

Folks:

The Eagleworks Lab is still working on the copper frustum thruster that was reported on last summer at the AIAA/JPC.  We have now confirmed that there is a thrust signature in a hard vacuum (~5.0x10^-6 Torr) in both the forward direction, (approx. +50 micro-Newton (uN) with 50W at 1,937.115 MHz), and the reversed direction, (up to -16uN with a failing RF amp), when the thruster is rotated 180 degrees on the torque pendulum.  However we continue to fight through RF amplifier failures brought on by having to operate them in a hard vacuum with few $$$ resources to fix them when they break, so the desired data is coming along very slowly.  We are still working on obtaining enough data though that will allow us to go to Glenn Research Center (GRC) for a replication effort in the next few months.  However that will only happen if we can make the thrust signature large enough since the GRC thrust stand can only measure down to ~50uN, so we have to get the thrust signature up to at least 100uN before we can go to GRC. 

As to the theoretical side of Q-Thrusters, Dr. White has just developed the first cut at a quantum vacuum (QV) based plasma code written in C+ under Windows/Unix and VMD visualization software that utilizes the COMSOL E&M derived field data for a given thruster geometry that allows one to track the movement and velocity of a subset of the QV's electron/positron neutral plasma pairs in the thruster over time as they respond to the applied time varying RF E&M fields in the copper frustum resonant cavity and to each other.  This package also allows one to calculate the expected thrust for a given input power and quality factor of the frustum resonant cavity based of standard plasma rocket physics.  So far the estimated thrust verses experimental observations are within 2% for the first experimental data run I compared it to, but we still have a long, long road ahead of us of experimental validation before we have any real confidence in this very new Q-Thruster design tool.

Best, Paul March


Have you considered crowdfunding? I would absolutely donate to this testing endeavor.

How much money do you need to raise?
Title: Re: EM Drive Developments - related to space flight applications - Thread 2
Post by: Star One on 02/07/2015 12:09 pm
It's criminal in my view that this research isn't being better funded especially now that results are coming through.
Title: Re: EM Drive Developments - related to space flight applications - Thread 2
Post by: RotoSequence on 02/07/2015 12:11 pm
I don't think funding discussions necessarily belong in this thread. Theory, potential application, and research results are one thing - figuring out how to pay for research on something that is not yet proven to create thrust (via the process of peer review and outside verification) is drifting dangerously close to territory that will earn a surefire thread closure.
Title: Re: EM Drive Developments - related to space flight applications - Thread 2
Post by: Star One on 02/07/2015 01:20 pm

I don't think funding discussions necessarily belong in this thread. Theory, potential application, and research results are one thing - figuring out how to pay for research on something that is not yet proven to create thrust (via the process of peer review and outside verification) is drifting dangerously close to territory that will earn a surefire thread closure.

But can you disentangle one from the other as easily as you suggest. I would say not.
Title: Re: EM Drive Developments - related to space flight applications - Thread 2
Post by: Jilpeter on 02/07/2015 01:36 pm
We now have the shwayer, yang juan, fetta, nasa and vacuum test. This many tests almost are intriguing enough to warrant funding and lend credibility to the claims.

The only thing that remains is to build more prototypes and create a theory of operation that can be peer reviewed.

If the project lacks funding and it takes another 10 years to build the next prototypes we will have seriously hampered out development as a civilization.
Title: Re: EM Drive Developments - related to space flight applications - Thread 2
Post by: Star-Drive on 02/07/2015 02:08 pm
..

Folks:

The Eagleworks Lab is still working on the copper frustum thruster that was reported on last summer at the AIAA/JPC.  We have now confirmed that there is a thrust signature in a hard vacuum (~5.0x10^-6 Torr) in both the forward direction, (approx. +50 micro-Newton (uN) with 50W at 1,937.115 MHz), and the reversed direction, (up to -16uN with a failing RF amp), when the thruster is rotated 180 degrees on the torque pendulum.  However we continue to fight through RF amplifier failures brought on by having to operate them in a hard vacuum with few $$$ resources to fix them when they break, so the desired data is coming along very slowly.  We are still working on obtaining enough data though that will allow us to go to Glenn Research Center (GRC) for a replication effort in the next few months.  However that will only happen if we can make the thrust signature large enough since the GRC thrust stand can only measure down to ~50uN, so we have to get the thrust signature up to at least 100uN before we can go to GRC. 

As to the theoretical side of Q-Thrusters, Dr. White has just developed the first cut at a quantum vacuum (QV) based plasma code written in C+ under Windows/Unix and VMD visualization software that utilizes the COMSOL E&M derived field data for a given thruster geometry that allows one to track the movement and velocity of a subset of the QV's electron/positron neutral plasma pairs in the thruster over time as they respond to the applied time varying RF E&M fields in the copper frustum resonant cavity and to each other.  This package also allows one to calculate the expected thrust for a given input power and quality factor of the frustum resonant cavity based of standard plasma rocket physics.  So far the estimated thrust verses experimental observations are within 2% for the first experimental data run I compared it to, but we still have a long, long road ahead of us of experimental validation before we have any real confidence in this very new Q-Thruster design tool.

Best, Paul March
Thanks Paul for an excellent update, much appreciated.

Congratulations to the Eagleworks team for obtaining experimental force measurements in the torsional pendulum in a hard vacuum (~5.0x10^-6 Torr) !

(http://i262.photobucket.com/albums/ii105/scoozna/misc/applause.gif)

It would be most helpful to the scientific/technical community if you could provide the dimensions of the frustum of a cone (truncated cone) used in the Brady et.al. "Anomalous ..." report.

Of several estimates of the geometry, we have established that this is the best estimate so far:

Aero Best estimate as of 11/9/2014    http://forum.nasaspaceflight.com/index.php?topic=29276.msg1285896#msg1285896
   
cavityLength = 0.24173 m
bigDiameter = 0.27246 m
smallDiameter = 0.15875 m

where "cavity length" is the axial length of the frustum measured along the longitudinal axis of the cone, perpendicular to both the small and big diameters.  In other words, the dimension labeled "h" in the following image: (http://offshoremechanics.asmedigitalcollection.asme.org/data/Journals/JMOEEX/28400/031603_1_1.jpeg)

It would be most helpful if you could provide the actual (internal) dimensions of the frustum (or at least if you could provide its approximate dimensions).

Thanks  :)

The copper frustum we built and now are using has the following internal copper surface dimensions.
Large OD: 11.00" (0.2794m), Small OD: 6.25" (0.1588m) & Length: 9.00" (0.2286m)  also see the attached slide with notes on the copper frustum's internal construction.  I've also attached a slide with the TM212 E&M resonant mode we are currently exploring in this copper frustum cavity with a screen shot of the end on VMD display that shows the 100,000th simulation increment for this resonant mode.  Each step in this plasma code is 1/72 of a full RF cycle, i.e., every 5.0 degrees of phase shift so each time step at 1,937.115 MHz is ~7.12 pico-seconds (10^-12s).  Next is a picture of the forward thrust signature from this copper frustum taken in a ~5.0x10^-6 Torr vacuum.  However we are currently trying to investigate the thermal response of the is copper frustum on the baseline of the torque pendulum after learning that I made a poor choice in how I built and mounted the copper frustum since it makes these thermal effects more pronounced in these thrust plots than they had to be.  I'm appending a partial COMSOL thermal analysis of the copper frustum and I would like to get someone better versed in the art of thermodynamics than I to see if they calculate the expected thermal expansion of the copper frustum AND the polyethylene discs over a 60 second data run with ~50W of 1,937.115 MHz RF applied inside the cavity via a 14mm OD magnetic loop antenna made from 20 gauge magnet wire.   

BTW, we have found that both the TE and TM E&M modes of this copper frustum can produce a thrust signature, but so far the TM modes appear to be the better performer, at least for the few modes we have been able to study to date.  (Shawyer and the Chinese used the magnetron excited TE012 mode in their frustum cavities without dielectrics being present.)

Lastly, like any busy lab, Eagleworks could always use extra funding to deal with its daily heart burns and required salaries to keep it going.  However we are currently a NASA sponsored facility, which sadly precludes being able to accept crowd sourcing or any other outside source of funding, unless it's through a commercial NASA Space Act Agreement that has to be approved up through NASA headquarters in Washington DC.  In the meantime we limp along with the meager funding we are allotted until we either run out of time, or we finally prove our QVF/MHD conjecture is close enough to the reality so that we can start building Q-Thrusters with large enough thrusts, (tens to thousands of Newton), to be used on manned spaceflight missions.
Title: Re: EM Drive Developments - related to space flight applications - Thread 2
Post by: Rodal on 02/07/2015 03:28 pm
...
The copper frustum we built and now are using has the following internal copper surface dimensions.
Large OD: 11.00" (0.2794m), Small OD: 6.25" (0.1588m) & Length: 9.00" (0.2286m)  also see the attached slide with notes on the copper frustum's internal construction.  I've also attached a slide with the TM212 E&M resonant mode we are currently exploring in this copper frustum cavity with a screen shot of the end on VMD display that shows the 100,000th simulation increment for this resonant mode.  Each step in this plasma code is 1/72 of a full RF cycle, i.e., every 5.0 degrees of phase shift so each time step at 1,937.115 MHz is ~7.12 pico-seconds (10^-12s).  Next is a picture of the forward thrust signature from this copper frustum taken in a ~5.0x10^-6 Torr vacuum.  However we are currently trying to investigate the thermal response of the is copper frustum on the baseline of the torque pendulum after learning that I made a poor choice in how I built and mounted the copper frustum since it makes these thermal effects more pronounced in these thrust plots than they had to be.  I'm appending a partial COMSOL thermal analysis of the copper frustum and I would like to get someone better versed in the art of thermodynamics than I to see if they calculate the expected thermal expansion of the copper frustum AND the polyethylene discs over a 60 second data run with ~50W of 1,937.115 MHz RF applied inside the cavity via a 14mm OD magnetic loop antenna made from 20 gauge magnet wire.   

BTW, we have found that both the TE and TM E&M modes of this copper frustum can produce a thrust signature, but so far the TM modes appear to be the better performer, at least for the few modes we have been able to study to date.  (Shawyer and the Chinese used the magnetron excited TE012 mode in their frustum cavities without dielectrics being present.)

Lastly, like any busy lab, Eagleworks could always use extra funding to deal with its daily heart burns and required salaries to keep it going.  However we are currently a NASA sponsored facility, which sadly precludes being able to accept crowd sourcing or any other outside source of funding, unless it's through a commercial NASA Space Act Agreement that has to be approved up through NASA headquarters in Washington DC.  In the meantime we limp along with the meager funding we are allotted until we either run out of time, or we finally prove our QVF/MHD conjecture is close enough to the reality so that we can start building Q-Thrusters with large enough thrusts, (tens to thousands of Newton), to be used on manned spaceflight missions.

Thanks so much, Paul for this excellent information.

(http://s3.amazonaws.com/rem-production/attachments/9800/full/9781618101204.jpg?1370989043)


I hope that all postings in this #2 thread will continue to abide by the guidelines here: http://forum.nasaspaceflight.com/index.php?topic=29276.msg1301657#msg1301657.  The previous thread was locked because of "personal attacks" and "stupid" and "pointless" posts "that did not feel like this site's subject matter."

I hope that we can continue this thread in a professional manner concentrating on technical matters regarding EM Drive Developments related to space flight applications so that we maintain a healthy and open channel of communication with you (Paul March at NASA).  Also with all others interested in the science and technology of  EM Drive Developments related to space flight applications.
Title: Re: EM Drive Developments - related to space flight applications - Thread 2
Post by: Star-Drive on 02/07/2015 04:01 pm
Rodal:

As a follow up to my previous post and in the spirit of open disclosure, I'm including our last null-thrust test that ran the RF amp at 10.0Adc while its RF power was being dissipated in a 100W, 50 ohm  dummy load positioned in place of the test article on the torque pendulum (TP), a picture of the new heat shields for our torque pendulum's upper and lower torsion springs, (more belts and suspenders to mitigate thermal drifts in the TP baseline), the reversed test setup drawing and the best reversed thrust plot obtained just before or during when our second and last 120W max RF amplifier was dying from internal corona discharges around its RF output circulator.  Apparently the RF amp's internal gas pressure had gone down from 1 Bar to an estimated 10 Torr or less after a few days leaking air in a hard vacuum.  And 0.1-to-10.0 Torr is where glow discharges are the easiest to ignite with RF signals.  So much for EMPower's "hermetic" sealed RF amplifiers...

Best, Paul March
Title: Re: EM Drive Developments - related to space flight applications - Thread 2
Post by: birchoff on 02/07/2015 04:50 pm
Rodal:

As a follow up to my previous post and in the spirit of open disclosure, I'm including our last null-thrust test that ran the RF amp at 10.0Adc while its RF power was being dissipated in a 100W, 50 ohm  dummy load positioned in place of the test article on the torque pendulum (TP), a picture of the new heat shields for our torque pendulum's upper and lower torsion springs, (more belts and suspenders to mitigate thermal drifts in the TP baseline), the reversed test setup drawing and the best reversed thrust plot obtained just before or during when our second and last 120W max RF amplifier was dying from internal corona discharges around its RF output circulator.  Apparently the RF amp's internal gas pressure had gone down from 1 Bar to an estimated 10 Torr or less after a few days leaking air in a hard vacuum.  And 0.1-to-10.0 Torr is where glow discharges are the easiest to ignite with RF signals.  So much for EMPower's "hermetic" sealed RF amplifiers...

Best, Paul March

First off congratulations, and thank you very much for the information.

One question though. The use of a dummy load to the best of my understanding provides evidence to support that the thrust measurement device is not generating false positive data. Is it possible to run the Frustum in a null configuration? If so, is that in the plans before the next report is published?
Title: Re: EM Drive Developments - related to space flight applications - Thread 2
Post by: Star One on 02/07/2015 05:09 pm
Please be aware it wasn't me intention to take this thread off topic so apologies for that.
Title: Re: EM Drive Developments - related to space flight applications - Thread 2
Post by: aero on 02/07/2015 05:46 pm
Has anyone notified Dr. M re. precise cavity dimensions and latest forces from Paul? I would but don't have his link handy.
Title: Re: EM Drive Developments - related to space flight applications - Thread 2
Post by: Rodal on 02/07/2015 05:47 pm
Has anyone notified Dr. M re. precise cavity dimensions and latest forces from Paul? I would but don't have his link handy.
I sent Dr. McCulloch a message as soon as I saw the dimensions.  Thanks
Title: Re: EM Drive Developments - related to space flight applications - Thread 2
Post by: Rodal on 02/07/2015 06:11 pm
Well here's an open one...... http://scharstein.eng.ua.edu/electromagnetic.pdf

Thank you.   That paper deals with the surface current induced by an incident plane wave on a truncated cone.


For electromagnetic transmission properties of spherical transverse electric(TE) and transverse-magnetic (TM) eigenmodes within a truncated cone, see the enclosed paper.

Quote from:  Xiahui Zeng and Dianyuan Fan
(1) We have developed an exact analytical approach for the description of the electromagnetic
fields inside a hollow metallic waveguide with a taper. Analytical expressions for the spatital
distributions of electromagnetic field components, attenuation constant, phase constant and
wave impedance are derived.

(2)According to our theory the modes configurations inside a tapered hollow metallic
waveguide are similar to those in a cylindrical hollow metallic waveguide, but the
transmission characteristics and engergy densities distributions along propagating direction
have a different behavior. It is shown that all modes run continuously from a propagating
through a transition to an evanescent region and the value of the attenuation increases as the
distance from the cone vertex and the cone angle desrease. A strict distinction between pure
propagating and pure evanescent modes can not be achieved. There is no well-defined cutoff
wavelength but rather a cutoff radius. It is interesting to note that the magnitude of the cutoff
radius is related to the wavelength and the cone half-angle. The values of attenuation and
phase constants for the spherical TE and TM modes inside the tapered hollow metallic
waveguide depend on the cone half-angle very seriously. As the cone half-angle decreases, the
value of the attenuation increases. The smaller the cone half-angle is, the faster the modes
attenuate. This can explain why large taper angle may improve the light throughout in
aperture probe which finds an important application in scanning near-field optical microscopy

(3) As follows from our calculations, we find that in the propagating region the attentuation of
some modes decays faster than those of others, and one mode after the other reaches cutoff in
the tapered hollow metallic waveguide as the distance from the cone vertex decreases.

4) In the tapered hollow metallic waveguide, light is well confined in the hollow core (air
region) because it is reflected back to the core by a metal wall.
Title: Re: EM Drive Developments - related to space flight applications - Thread 2
Post by: Star-Drive on 02/07/2015 06:17 pm
Rodal:

As a follow up to my previous post and in the spirit of open disclosure, I'm including our last null-thrust test that ran the RF amp at 10.0Adc while its RF power was being dissipated in a 100W, 50 ohm  dummy load positioned in place of the test article on the torque pendulum (TP), a picture of the new heat shields for our torque pendulum's upper and lower torsion springs, (more belts and suspenders to mitigate thermal drifts in the TP baseline), the reversed test setup drawing and the best reversed thrust plot obtained just before or during when our second and last 120W max RF amplifier was dying from internal corona discharges around its RF output circulator.  Apparently the RF amp's internal gas pressure had gone down from 1 Bar to an estimated 10 Torr or less after a few days leaking air in a hard vacuum.  And 0.1-to-10.0 Torr is where glow discharges are the easiest to ignite with RF signals.  So much for EMPower's "hermetic" sealed RF amplifiers...

Best, Paul March

First off congratulations, and thank you very much for the information.

One question though. The use of a dummy load to the best of my understanding provides evidence to support that the thrust measurement device is not generating false positive data. Is it possible to run the Frustum in a null configuration? If so, is that in the plans before the next report is published?

Birchoff:

"Is it possible to run the Frustum in a null configuration? If so, is that in the plans before the next report is published?"

Yes and yes.  In fact it was one of the requests made by the blue ribbon panel of PhDs that NASA/EP hired to review the Eagleworks Lab's theoretical and experimental work last summer.  Even if will take a new mounting arrangement to get it accomplished. 

Overall though the blue ribbon panel's experimentalists appeared to be pleased with our previous and upcoming lab work.  However they ripped into Sonny's QVF/MHD conjecture because it relies on the quantum vacuum being mutable and engineer-able whereas the current physics mainstream thinks that the quantum vacuum is an immutable ground energy state of the universe that can-NOT be used to convey energy or momentum as proposed by Dr. White.   However they brushed aside Sonny's QVF based derivation of the Bohr hydrogen atom electron radius as a "mathematical coincidence" and didn't have a word to say what the Casimir effect and other quantum vacuum phenomenon were caused by, that can only occur only if the QV is mutable and can convey energy and momentum.   So Sonny and Jerry Vera took it upon themselves last fall to increase this mathematical coincidence from one to more than 47 times as they explored the QV created atomic electron shell radii for atoms up to atomic number 7 all based on the QV being the root cause for all of it including the origins of the electron and all other subatomic particles.   

BTW, IMO Jim Woodward's Mach-Effect (M-E) conjecture that is based primarily on SRT and GRT, is still in the running for a way to explain his and our test results to date.  However the M-E also has its detractors since it requires that instantaneous Wheeler/Feynman radiation reaction forces being required between a local time varying mass and all the other mass/energy in the casually connected universe, since this mechanism is used to balance the M-E's energy & momentum conservation books.  In the end analysis though I think that the ME will rest on the quantum nature of space-time, since in Woodward's eyes the gravitational field IS space-time, and in our eyes GRT's space-time is in reality the quantum vacuum that probably has at least 4 spatial dimensions and one time dimension! 

Best, Paul March
Title: Re: EM Drive Developments - related to space flight applications - Thread 2
Post by: Rodal on 02/07/2015 06:44 pm
Thank you for the very interesting information about the "blue ribbon panel of PhDs that NASA/EP hired to review the Eagleworks Lab's theoretical and experimental work last summer."   This is very relevant information to EM Drive Developments.



Paul, in your absence, the prior thread on EM Drives was derailed by polemical discussion of Woodward's Mach-Effect conjecture (more info here: http://forum.nasaspaceflight.com/index.php?topic=29276.msg1301657#msg1301657  ). 

To avoid such issues, it may be preferable to continue discussion of that conjecture (Woodward's Mach-Effect) at this thread, dedicated exclusively to Woodward's Mach-Effect:  http://forum.nasaspaceflight.com/index.php?topic=31037.460 )
Title: Re: EM Drive Developments - related to space flight applications - Thread 2
Post by: birchoff on 02/07/2015 07:25 pm
Rodal:

As a follow up to my previous post and in the spirit of open disclosure, I'm including our last null-thrust test that ran the RF amp at 10.0Adc while its RF power was being dissipated in a 100W, 50 ohm  dummy load positioned in place of the test article on the torque pendulum (TP), a picture of the new heat shields for our torque pendulum's upper and lower torsion springs, (more belts and suspenders to mitigate thermal drifts in the TP baseline), the reversed test setup drawing and the best reversed thrust plot obtained just before or during when our second and last 120W max RF amplifier was dying from internal corona discharges around its RF output circulator.  Apparently the RF amp's internal gas pressure had gone down from 1 Bar to an estimated 10 Torr or less after a few days leaking air in a hard vacuum.  And 0.1-to-10.0 Torr is where glow discharges are the easiest to ignite with RF signals.  So much for EMPower's "hermetic" sealed RF amplifiers...

Best, Paul March

First off congratulations, and thank you very much for the information.

One question though. The use of a dummy load to the best of my understanding provides evidence to support that the thrust measurement device is not generating false positive data. Is it possible to run the Frustum in a null configuration? If so, is that in the plans before the next report is published?

Birchoff:

"Is it possible to run the Frustum in a null configuration? If so, is that in the plans before the next report is published?"

Yes and yes.  In fact it was one of the requests made by the blue ribbon panel of PhDs that NASA/EP hired to review the Eagleworks Lab's theoretical and experimental work last summer.  Even if will take a new mounting arrangement to get it accomplished. 

Overall though the blue ribbon panel's experimentalists appeared to be pleased with our previous and upcoming lab work.  However they ripped into Sonny's QVF/MHD conjecture because it relies on the quantum vacuum being mutable and engineer-able whereas the current physics mainstream thinks that the quantum vacuum is an immutable ground energy state of the universe that can-NOT be used to convey energy or momentum as proposed by Dr. White.   However they brushed aside Sonny's QVF based derivation of the Bohr hydrogen atom electron radius as a "mathematical coincidence" and didn't have a word to say what the Casimir effect and other quantum vacuum phenomenon were caused by, that can only occur only if the QV is mutable and can convey energy and momentum.   So Sonny and Jerry Vera took it upon themselves last fall to increase this mathematical coincidence from one to more than 47 times as they explored the QV created atomic electron shell radii for atoms up to atomic number 7 all based on the QV being the root cause for all of it including the origins of the electron and all other subatomic particles.   

BTW, IMO Jim Woodward's Mach-Effect (M-E) conjecture that is based primarily on SRT and GRT, is still in the running for a way to explain his and our test results to date.  However the M-E also has its detractors since it requires that instantaneous Wheeler/Feynman radiation reaction forces being required between a local time varying mass and all the other mass/energy in the casually connected universe, since this mechanism is used to balance the M-E's energy & momentum conservation books.  In the end analysis though I think that the ME will rest on the quantum nature of space-time, since in Woodward's eyes the gravitational field IS space-time, and in our eyes GRT's space-time is in reality the quantum vacuum that probably has at least 4 spatial dimensions and one time dimension! 

Best, Paul March

Thanks for the insight. what does Dr. White believe he has to do in order to prove or disprove that his conjecture explains the behavior observed when the truncated frustum is energized appropriately? Assuming all the testing currently being done is successful and you get a successful set of replications from other labs. The only thing we would be able to conclusively claim is that the device, built as describes, provides thrust. How does Dr. White plan to show that the device behaves as described by his theory? Also, if Dr. White cannot prove that his theory completely explains the observations, what would be the next steps to find an explanation for the observation? And would we need to have such a theory before we begin using this thing in well defined use cases like ISS or satelite station keeping?
Title: Re: EM Drive Developments - related to space flight applications - Thread 2
Post by: Star-Drive on 02/07/2015 07:28 pm
Thank you for the very interesting information about the "blue ribbon panel of PhDs that NASA/EP hired to review the Eagleworks Lab's theoretical and experimental work last summer."   This is very relevant information to EM Drive Developments.



Paul, in your absence, the prior thread on EM Drives was derailed by polemical discussion of Woodward's Mach-Effect (M-E) conjecture (more info here: http://forum.nasaspaceflight.com/index.php?topic=29276.msg1301657#msg1301657  ). 

To avoid such issues, it may be preferable to continue discussion of that conjecture (Woodward's Mach-Effect) can be pursued at this thread, dedicated exclusively to Woodward's Mach-Effect:  http://forum.nasaspaceflight.com/index.php?topic=31037.460 )

Sorry, I didn't know that Jim Woodward's work had become a hot potato, so I'll go to the M-E thread if the M-E topic comes up again.

Best, PM
Title: Re: EM Drive Developments - related to space flight applications - Thread 2
Post by: Rodal on 02/07/2015 07:59 pm
Thank you for the very interesting information about the "blue ribbon panel of PhDs that NASA/EP hired to review the Eagleworks Lab's theoretical and experimental work last summer."   This is very relevant information to EM Drive Developments.



Paul, in your absence, the prior thread on EM Drives was derailed by polemical discussion of Woodward's Mach-Effect (M-E) conjecture (more info here: http://forum.nasaspaceflight.com/index.php?topic=29276.msg1301657#msg1301657  ). 

To avoid such issues, it may be preferable to continue discussion of that conjecture (Woodward's Mach-Effect) can be pursued at this thread, dedicated exclusively to Woodward's Mach-Effect:  http://forum.nasaspaceflight.com/index.php?topic=31037.460 )

Sorry, I didn't know that Jim Woodward's work had become a hot potato, so I'll go to the M-E thread if the M-E topic comes up again.

Best, PM

I was trying to convey how pleased to hear back from you and how excited I was about the news you conveyed and how I wanted you to continue posting.  It was my poor attempt at preventing this communication from being shut down again  :)
Title: Re: EM Drive Developments - related to space flight applications - Thread 2
Post by: Mulletron on 02/07/2015 08:19 pm
Indeed we are all very delighted to hear from you Paul. This thread, and thread 1 has been a fury of activity trying to explore this proposition that we can possibly achieve all electric thrust in the vacuum of space without carrying propellant, and what it all means for science and humanity's future.

Given the reported results in vacuum, this is progress.

Patiently looking forward to reading "Anomalous Thrust Production from an RF Test Device Measured on a Low-Thrust Torsion Pendulum"
Title: Re: EM Drive Developments - related to space flight applications - Thread 2
Post by: birchoff on 02/07/2015 08:25 pm
Indeed we are all very delighted to hear from you Paul. This thread, and thread 1 has been a fury of activity trying to explore this proposition that we can possibly achieve all electric thrust in the vacuum of space without carrying propellant, and what it all means for science and humanity's future.

Given the reported results in vacuum, this is progress.

Patiently looking forward to reading "Anomalous Thrust Production from an RF Test Device Measured on a Low-Thrust Torsion Pendulum"

seconded
Title: Re: EM Drive Developments - related to space flight applications - Thread 2
Post by: Star-Drive on 02/07/2015 10:21 pm
Rodal:

As a follow up to my previous post and in the spirit of open disclosure, I'm including our last null-thrust test that ran the RF amp at 10.0Adc while its RF power was being dissipated in a 100W, 50 ohm  dummy load positioned in place of the test article on the torque pendulum (TP), a picture of the new heat shields for our torque pendulum's upper and lower torsion springs, (more belts and suspenders to mitigate thermal drifts in the TP baseline), the reversed test setup drawing and the best reversed thrust plot obtained just before or during when our second and last 120W max RF amplifier was dying from internal corona discharges around its RF output circulator.  Apparently the RF amp's internal gas pressure had gone down from 1 Bar to an estimated 10 Torr or less after a few days leaking air in a hard vacuum.  And 0.1-to-10.0 Torr is where glow discharges are the easiest to ignite with RF signals.  So much for EMPower's "hermetic" sealed RF amplifiers...

Best, Paul March

First off congratulations, and thank you very much for the information.

One question though. The use of a dummy load to the best of my understanding provides evidence to support that the thrust measurement device is not generating false positive data. Is it possible to run the Frustum in a null configuration? If so, is that in the plans before the next report is published?

Birchoff:

"Is it possible to run the Frustum in a null configuration? If so, is that in the plans before the next report is published?"

Yes and yes.  In fact it was one of the requests made by the blue ribbon panel of PhDs that NASA/EP hired to review the Eagleworks Lab's theoretical and experimental work last summer.  Even if will take a new mounting arrangement to get it accomplished. 

Overall though the blue ribbon panel's experimentalists appeared to be pleased with our previous and upcoming lab work.  However they ripped into Sonny's QVF/MHD conjecture because it relies on the quantum vacuum being mutable and engineer-able whereas the current physics mainstream thinks that the quantum vacuum is an immutable ground energy state of the universe that can-NOT be used to convey energy or momentum as proposed by Dr. White.   However they brushed aside Sonny's QVF based derivation of the Bohr hydrogen atom electron radius as a "mathematical coincidence" and didn't have a word to say what the Casimir effect and other quantum vacuum phenomenon were caused by, that can only occur only if the QV is mutable and can convey energy and momentum.   So Sonny and Jerry Vera took it upon themselves last fall to increase this mathematical coincidence from one to more than 47 times as they explored the QV created atomic electron shell radii for atoms up to atomic number 7 all based on the QV being the root cause for all of it including the origins of the electron and all other subatomic particles.   

BTW, IMO Jim Woodward's Mach-Effect (M-E) conjecture that is based primarily on SRT and GRT, is still in the running for a way to explain his and our test results to date.  However the M-E also has its detractors since it requires that instantaneous Wheeler/Feynman radiation reaction forces being required between a local time varying mass and all the other mass/energy in the casually connected universe, since this mechanism is used to balance the M-E's energy & momentum conservation books.  In the end analysis though I think that the ME will rest on the quantum nature of space-time, since in Woodward's eyes the gravitational field IS space-time, and in our eyes GRT's space-time is in reality the quantum vacuum that probably has at least 4 spatial dimensions and one time dimension! 

Best, Paul March

Thanks for the insight. what does Dr. White believe he has to do in order to prove or disprove that his conjecture explains the behavior observed when the truncated frustum is energized appropriately? Assuming all the testing currently being done is successful and you get a successful set of replications from other labs. The only thing we would be able to conclusively claim is that the device, built as describes, provides thrust. How does Dr. White plan to show that the device behaves as described by his theory? Also, if Dr. White cannot prove that his theory completely explains the observations, what would be the next steps to find an explanation for the observation? And would we need to have such a theory before we begin using this thing in well defined use cases like ISS or satelite station keeping?

Birchoff:

"What does Dr. White believe he has to do in order to prove or disprove that his conjecture explains the behavior observed when the truncated frustum is energized appropriately?"

Exactly what he and I have been doing.  Fleshing out his QVF conjecture in papers for the appropriate peer reviewed journals. He is also continuing the generation of the COMSOL E&M and QVF based C++ plasma code that will allow us to compare the resonant cavity lab results with the QVF based force predictions using the volume integral of the ejected semi-virtual e/p pairs for the resonant cavity geometry in question. 

We have already performed the first step along this path with the preliminary results I provided in an earlier post today.  In that slide which is based on the copper frustum cavity running in its TM212 mode with 50W of 1,937.188 MHz RF power applied, we showed that the predicted thrust that took over 18 hours to run the 150k time samples on an i5 PC, was 54uN and the average for five real data runs at 50W was 55uN.  Is that another mathematical coincidence?  I don't think so, but we won't know for sure until I have time to compare the rest of the 30W, 40W, and 60W averages on the attached slide with the same computer code that will take 17 hours to run on my lab PC for each additional example.  We will also be looking at modeling and comparing the results of the Cannae test articles we tested in 2013 & 2014, along with the Shawyer/Chinese EM-Drive results with and without dielectrics in the resonant cavities.  If our plasma code predictions nail all those tests to say within +/-10% of the experimental results then we can start using it to optimize the thrust output of these QVF/MHD based thrusters.   

BTW, it appears that the dielectric discs may act as QV e/p pair reflectors that aid in the conical frustum shape's force symmetry breaking and force rectification process.  Left to its own devices, the QV e/p pair spray generated by the applied RF energy tends to want to go in all directions instead of the desired tightly collimated unidirectional propellant beam that goes in one direction while the thruster back-reacts in the opposite direction according to Newton's third law. 

Best, Paul March
Title: Re: EM Drive Developments - related to space flight applications - Thread 2
Post by: birchoff on 02/07/2015 10:54 pm
Rodal:

As a follow up to my previous post and in the spirit of open disclosure, I'm including our last null-thrust test that ran the RF amp at 10.0Adc while its RF power was being dissipated in a 100W, 50 ohm  dummy load positioned in place of the test article on the torque pendulum (TP), a picture of the new heat shields for our torque pendulum's upper and lower torsion springs, (more belts and suspenders to mitigate thermal drifts in the TP baseline), the reversed test setup drawing and the best reversed thrust plot obtained just before or during when our second and last 120W max RF amplifier was dying from internal corona discharges around its RF output circulator.  Apparently the RF amp's internal gas pressure had gone down from 1 Bar to an estimated 10 Torr or less after a few days leaking air in a hard vacuum.  And 0.1-to-10.0 Torr is where glow discharges are the easiest to ignite with RF signals.  So much for EMPower's "hermetic" sealed RF amplifiers...

Best, Paul March

First off congratulations, and thank you very much for the information.

One question though. The use of a dummy load to the best of my understanding provides evidence to support that the thrust measurement device is not generating false positive data. Is it possible to run the Frustum in a null configuration? If so, is that in the plans before the next report is published?

Birchoff:

"Is it possible to run the Frustum in a null configuration? If so, is that in the plans before the next report is published?"

Yes and yes.  In fact it was one of the requests made by the blue ribbon panel of PhDs that NASA/EP hired to review the Eagleworks Lab's theoretical and experimental work last summer.  Even if will take a new mounting arrangement to get it accomplished. 

Overall though the blue ribbon panel's experimentalists appeared to be pleased with our previous and upcoming lab work.  However they ripped into Sonny's QVF/MHD conjecture because it relies on the quantum vacuum being mutable and engineer-able whereas the current physics mainstream thinks that the quantum vacuum is an immutable ground energy state of the universe that can-NOT be used to convey energy or momentum as proposed by Dr. White.   However they brushed aside Sonny's QVF based derivation of the Bohr hydrogen atom electron radius as a "mathematical coincidence" and didn't have a word to say what the Casimir effect and other quantum vacuum phenomenon were caused by, that can only occur only if the QV is mutable and can convey energy and momentum.   So Sonny and Jerry Vera took it upon themselves last fall to increase this mathematical coincidence from one to more than 47 times as they explored the QV created atomic electron shell radii for atoms up to atomic number 7 all based on the QV being the root cause for all of it including the origins of the electron and all other subatomic particles.   

BTW, IMO Jim Woodward's Mach-Effect (M-E) conjecture that is based primarily on SRT and GRT, is still in the running for a way to explain his and our test results to date.  However the M-E also has its detractors since it requires that instantaneous Wheeler/Feynman radiation reaction forces being required between a local time varying mass and all the other mass/energy in the casually connected universe, since this mechanism is used to balance the M-E's energy & momentum conservation books.  In the end analysis though I think that the ME will rest on the quantum nature of space-time, since in Woodward's eyes the gravitational field IS space-time, and in our eyes GRT's space-time is in reality the quantum vacuum that probably has at least 4 spatial dimensions and one time dimension! 

Best, Paul March

Thanks for the insight. what does Dr. White believe he has to do in order to prove or disprove that his conjecture explains the behavior observed when the truncated frustum is energized appropriately? Assuming all the testing currently being done is successful and you get a successful set of replications from other labs. The only thing we would be able to conclusively claim is that the device, built as describes, provides thrust. How does Dr. White plan to show that the device behaves as described by his theory? Also, if Dr. White cannot prove that his theory completely explains the observations, what would be the next steps to find an explanation for the observation? And would we need to have such a theory before we begin using this thing in well defined use cases like ISS or satelite station keeping?

Birchoff:

"What does Dr. White believe he has to do in order to prove or disprove that his conjecture explains the behavior observed when the truncated frustum is energized appropriately?"

Exactly what he and I have been doing.  Fleshing out his QVF conjecture in papers for the appropriate peer reviewed journals. He is also continuing the generation of the COMSOL E&M and QVF based C++ plasma code that will allow us to compare the resonant cavity lab results with the QVF based force predictions using the volume integral of the ejected semi-virtual e/p pairs for the resonant cavity geometry in question. 

We have already performed the first step along this path with the preliminary results I provided in an earlier post today.  In that slide which is based on the copper frustum cavity running in its TM212 mode with 50W of 1,937.188 MHz RF power applied, we showed that the predicted thrust that took over 18 hours to run the 150k time samples on an i5 PC, was 54uN and the average for five real data runs at 50W was 55uN.  Is that another mathematical coincidence?  I don't think so, but we won't know for sure until I have time to compare the rest of the 30W, 40W, and 60W averages on the attached slide with the same computer code that will take 17 hours to run on my lab PC for each additional example.  We will also be looking at modeling and comparing the results of the Cannae test articles we tested in 2013 & 2014, along with the Shawyer/Chinese EM-Drive results with and without dielectrics in the resonant cavities.  If our plasma code predictions nail all those tests to say within +/-10% of the experimental results then we can start using it to optimize the thrust output of these QVF/MHD based thrusters.   

BTW, it appears that the dielectric discs may act as QV e/p pair reflectors that aid in the conical frustum shape's force symmetry breaking and force rectification process.  Left to its own devices, the QV e/p pair spray generated by the applied RF energy tends to want to go in all directions instead of the desired tightly collimated unidirectional propellant beam that goes in one direction while the thruster back-reacts in the opposite direction according to Newton's third law. 

Best, Paul March

Fantastic...

One last question if you dont mind indulging me in a bit of educated speculation. Can the copper frustum be made smaller while maintaining the same thrust performance? I would expect that the smaller frustum would lead to different parameters. But what I am trying to gauge is if there is some lower limit on the size of the frustum.
Title: Re: EM Drive Developments - related to space flight applications - Thread 2
Post by: Mulletron on 02/07/2015 11:59 pm

BTW, it appears that the dielectric discs may act as QV e/p pair reflectors that aid in the conical frustum shape's force symmetry breaking and force rectification process.  Left to its own devices, the QV e/p pair spray generated by the applied RF energy tends to want to go in all directions instead of the desired tightly collimated unidirectional propellant beam that goes in one direction while the thruster back-reacts in the opposite direction according to Newton's third law. 

Best, Paul March

Kinda sounds like sail, like this:
http://forum.nasaspaceflight.com/index.php?topic=29276.msg1299424#msg1299424

There is a LOT of research in thread 1 that we dug up concerning transferring momentum from the QV to dielectrics and how the symmetry breaking works to enable momentum transfer under EM fields. I can dig it all up again for an executive summary.
Title: Re: EM Drive Developments - related to space flight applications - Thread 2
Post by: frobnicat on 02/08/2015 12:49 am

Birchoff:

"What does Dr. White believe he has to do in order to prove or disprove that his conjecture explains the behavior observed when the truncated frustum is energized appropriately?"

Exactly what he and I have been doing.  Fleshing out his QVF conjecture in papers for the appropriate peer reviewed journals. He is also continuing the generation of the COMSOL E&M and QVF based C++ plasma code that will allow us to compare the resonant cavity lab results with the QVF based force predictions using the volume integral of the ejected semi-virtual e/p pairs for the resonant cavity geometry in question. 
...

Thank you for taking time giving some valuable info on ongoing work to followers of this line of research. We all have certainly a ton of questions, hope you can shed some light on the frame of reference aspects (and energetic consequences) : in the present apparently successful simulation model (quantitatively predictive), in what frame of reference the e/p pairs are "found" before they are asymmetrically accelerated and ejected by the drive's power ? Are they always harvested "at rest" relative to the frustum  ?

This would allow for more kinetic energy at the end of a mission than was injected into the drive (at the cost of a disturbed vacuum in the wake...). Sorry if it may sound controversial but I dare say we see two possible outcomes : tapping energy from the vacuum, or strange memory effect of "initial velocity" (for preventing such apparent overunit yield). Sounds like the working model validates the former and not the later ?

Third possibility : a preferred rest frame for the vacuum, since the specific speed of the present experimental thrust is 1000km/s we should expect some thrust/power variations relative to sidereal time (a few tens of % if this "aether" is cosmological, a few % if it is bound to galaxy rotation...) Any other alternative discussed ?

I guess those energetic aspects are asked again and again by sceptics, the "propeller" analogy needs clarification as for the implied instant velocity of the medium pushed against. I am sceptic but without an agenda. Just wonder where it leads.
Title: Re: EM Drive Developments - related to space flight applications - Thread 2
Post by: birchoff on 02/08/2015 01:01 am
Rodal:

As a follow up to my previous post and in the spirit of open disclosure, I'm including our last null-thrust test that ran the RF amp at 10.0Adc while its RF power was being dissipated in a 100W, 50 ohm  dummy load positioned in place of the test article on the torque pendulum (TP), a picture of the new heat shields for our torque pendulum's upper and lower torsion springs, (more belts and suspenders to mitigate thermal drifts in the TP baseline), the reversed test setup drawing and the best reversed thrust plot obtained just before or during when our second and last 120W max RF amplifier was dying from internal corona discharges around its RF output circulator.  Apparently the RF amp's internal gas pressure had gone down from 1 Bar to an estimated 10 Torr or less after a few days leaking air in a hard vacuum.  And 0.1-to-10.0 Torr is where glow discharges are the easiest to ignite with RF signals.  So much for EMPower's "hermetic" sealed RF amplifiers...

Best, Paul March

First off congratulations, and thank you very much for the information.

One question though. The use of a dummy load to the best of my understanding provides evidence to support that the thrust measurement device is not generating false positive data. Is it possible to run the Frustum in a null configuration? If so, is that in the plans before the next report is published?

Birchoff:

"Is it possible to run the Frustum in a null configuration? If so, is that in the plans before the next report is published?"

Yes and yes.  In fact it was one of the requests made by the blue ribbon panel of PhDs that NASA/EP hired to review the Eagleworks Lab's theoretical and experimental work last summer.  Even if will take a new mounting arrangement to get it accomplished. 

Overall though the blue ribbon panel's experimentalists appeared to be pleased with our previous and upcoming lab work.  However they ripped into Sonny's QVF/MHD conjecture because it relies on the quantum vacuum being mutable and engineer-able whereas the current physics mainstream thinks that the quantum vacuum is an immutable ground energy state of the universe that can-NOT be used to convey energy or momentum as proposed by Dr. White.   However they brushed aside Sonny's QVF based derivation of the Bohr hydrogen atom electron radius as a "mathematical coincidence" and didn't have a word to say what the Casimir effect and other quantum vacuum phenomenon were caused by, that can only occur only if the QV is mutable and can convey energy and momentum.   So Sonny and Jerry Vera took it upon themselves last fall to increase this mathematical coincidence from one to more than 47 times as they explored the QV created atomic electron shell radii for atoms up to atomic number 7 all based on the QV being the root cause for all of it including the origins of the electron and all other subatomic particles.   

BTW, IMO Jim Woodward's Mach-Effect (M-E) conjecture that is based primarily on SRT and GRT, is still in the running for a way to explain his and our test results to date.  However the M-E also has its detractors since it requires that instantaneous Wheeler/Feynman radiation reaction forces being required between a local time varying mass and all the other mass/energy in the casually connected universe, since this mechanism is used to balance the M-E's energy & momentum conservation books.  In the end analysis though I think that the ME will rest on the quantum nature of space-time, since in Woodward's eyes the gravitational field IS space-time, and in our eyes GRT's space-time is in reality the quantum vacuum that probably has at least 4 spatial dimensions and one time dimension! 

Best, Paul March

Thanks for the insight. what does Dr. White believe he has to do in order to prove or disprove that his conjecture explains the behavior observed when the truncated frustum is energized appropriately? Assuming all the testing currently being done is successful and you get a successful set of replications from other labs. The only thing we would be able to conclusively claim is that the device, built as describes, provides thrust. How does Dr. White plan to show that the device behaves as described by his theory? Also, if Dr. White cannot prove that his theory completely explains the observations, what would be the next steps to find an explanation for the observation? And would we need to have such a theory before we begin using this thing in well defined use cases like ISS or satelite station keeping?

Birchoff:

"What does Dr. White believe he has to do in order to prove or disprove that his conjecture explains the behavior observed when the truncated frustum is energized appropriately?"

Exactly what he and I have been doing.  Fleshing out his QVF conjecture in papers for the appropriate peer reviewed journals. He is also continuing the generation of the COMSOL E&M and QVF based C++ plasma code that will allow us to compare the resonant cavity lab results with the QVF based force predictions using the volume integral of the ejected semi-virtual e/p pairs for the resonant cavity geometry in question. 

We have already performed the first step along this path with the preliminary results I provided in an earlier post today.  In that slide which is based on the copper frustum cavity running in its TM212 mode with 50W of 1,937.188 MHz RF power applied, we showed that the predicted thrust that took over 18 hours to run the 150k time samples on an i5 PC, was 54uN and the average for five real data runs at 50W was 55uN.  Is that another mathematical coincidence?  I don't think so, but we won't know for sure until I have time to compare the rest of the 30W, 40W, and 60W averages on the attached slide with the same computer code that will take 17 hours to run on my lab PC for each additional example.  We will also be looking at modeling and comparing the results of the Cannae test articles we tested in 2013 & 2014, along with the Shawyer/Chinese EM-Drive results with and without dielectrics in the resonant cavities.  If our plasma code predictions nail all those tests to say within +/-10% of the experimental results then we can start using it to optimize the thrust output of these QVF/MHD based thrusters.   

BTW, it appears that the dielectric discs may act as QV e/p pair reflectors that aid in the conical frustum shape's force symmetry breaking and force rectification process.  Left to its own devices, the QV e/p pair spray generated by the applied RF energy tends to want to go in all directions instead of the desired tightly collimated unidirectional propellant beam that goes in one direction while the thruster back-reacts in the opposite direction according to Newton's third law. 

Best, Paul March

What about testing for a "wake"? I vaguely remember watching a presentation Dr. White gave where he said that one way of verifying conservation is being maintained would be to demonstrate the frustum creating some sort of "wake" or its equivalent. Will that also be tested before the next report?
Title: Re: EM Drive Developments - related to space flight applications - Thread 2
Post by: Notsosureofit on 02/08/2015 02:02 am
Just checked in for a bit.  WOW!  Things have been busy!

Quickly plugged in those measurements.  Used TM212, geometric mean.  Needed effective n=1.38 to get the freq w/ dielectric.  Q is down to 1000 for 50mmN at 50W.

Been chasing the self-acceleration papers, which looks good so far.

Hope to get some free time, too much nano going on.
Title: Re: EM Drive Developments - related to space flight applications - Thread 2
Post by: Mulletron on 02/08/2015 02:03 am
Those are all good observations. Just as the speed of light is Lorentz invariant so too should be the vacuum. If symmetries were broken, there should be particles or phonons emerge that are detectable. Noether's theorem comes into play here and if the Emdrive does in fact work, we need to re-examine what we know about symmetries and conservation laws, because in the end, experiment always wins. Aside from being a neat thruster, it could be an instrument which enables a better understanding of the QV.

http://www.ovaltech.ca/pdfss/The_Challenge_to_Create_the_Space_Drive.pdf

Quote
One approach to conserve momentum is to consider space
itself as the reaction mass. This approach evokes the old idea
of an "ether." To be strictly consistent with empirical
evidence, such as the Michelson-Morely experiment, any
further research to revisit the idea of an ether would have to
impose the condition that an ether is electromagnetically
Lorentz invariant- Note that this condition is a characteristic
of the ZPF [7].
So that approach is out because it is in opposition to Michelson-Morely.

Quote
An alternative to considering space as the reaction mass is
to further develop Mach's Principle. Mach's Principle asserts
that surrounding matter gives rise to inertial frames, and that
the inertial frames are somehow connected to the surrounding
matter [9]. Mach wrote that although he felt a connection to
the surrounding matter was required for the property of inertia
to be detectable, he also admitted that such a treatment was
not necessary to satisfactorily describe the laws of motion
[20]. Specifically, to be useful for propulsion physics, a
formalism of Mach's Principle is required that provides a
means to wansmit reaction forces to surrounding matter. This
implies developing a quantitative description for how the
surrounding matter creates an inertial frame, and how pushing
against that flame with a space drive is actually pushing
against the distant surrounding matter.
That sounds pretty good, but it leaves out two very important "sources" of inertia, which to Mach, were not directly observable to him. The quantum world, and the bound energy within the nucleus of atoms, and of course the QV. The philosophy of Mach valued what was directly observable. In modern times, materialism and physicalism are relics. So this approach is incomplete, thus a no go.

Quote
It is also possible to consider the very structure of
spacetime itself as a candidate for propulsive interactions. If it
were possible, for example, to create asymmetries in the very
properties of spacetime which give rise to inertial frames, it
may be possible to create net inertial forces. This is similar
to the"warp drive" suggested by Alcubierre [4]
Much better....

Uneven radiation pressure (from the QV and the RF simultaneously) across the cavity and the dielectric seems much more plausible than trying to accelerate virtual particle pairs in the style of MHD. Like Chuck Norris, you don't mess with virtual particles, virtual particles mess with you. :)
Title: Re: EM Drive Developments - related to space flight applications - Thread 2
Post by: Notsosureofit on 02/08/2015 02:31 am
I still favor direct application of the Equivalence Principle ...night all
Title: Re: EM Drive Developments - related to space flight applications - Thread 2
Post by: Star-Drive on 02/08/2015 03:12 am
Folks:

If the quantum vacuum is degradable and malleable as we think it should be, then to conserve momentum a QV wake has to be generated in the QV media as a Q-Thruster goes by just like a ship's propeller leaves a disturbance in the water as it goes by.  We think that the density of the QV is normally around its cosmological average of 9.1x10^-27 kg/m^3, but its density can be greatly increased by the presence of E&M fields and especially very strong and fast time-varying E&M fields that occur is microwave resonant cavities with large Quality Factors greater than say 1,000, or around elementary charged particles like electrons or protons where the QV density goes up to nuclear mass density as you approach the surface of the particle.  Suggest anything?  However in the paper we are now trying to get published with no takers so far, we find that the QV density should drop off very rapidly from a high density volume like a proton and in fact it follows the same drop off in density with distance as the Casimir effect does, i.e., 1 / r^4 where r = the distance from the resonant cavity boundary.  With that being the case it would be near impossible to detect the QV wake behind a Q-Thruster only generating milliNewtons or Newtons or even in tens of Newtons. 

So what's to do?  To detect a QV wake from a Q-thruster at even short distances from the source we think we will have to use another RF excited resonant cavity in a form of QV parametric amplification that is designed to produce a high density QV state just like in a Q-Thruster, but not to produce thrust.  Instead it will be optimized to monitor its time varying QV density as various very weak QV wake fields come in, are amplified and detected, then pass out of it again to go back to the low density QV state once again.  This has some interesting implications especially when you finish reading the attached paper from a PhD from Rice University here in Houston.

Last topic for the night for me.  Someone on this list asked if one could extract energy from the QV.  If the QV is GRT space-time, and space-time is the cosmological gravitational field that is created by all the causally connected mass/energy in our section of the universe, then we live in a high pressure sea of gravitational energy.  Now if the QV energy state is degradable and locally changeable, then one can posit the possibility of a thermodynamic energy conversion cycle that can extract energy from a pressure difference created in this QV media relative to the QV background average pressure, with a net decrease in this universal gravitational pressure or temperature reflective of the amount of energy so extracted.  And try to remember that gravitational energy is negative energy.  I'll leave the rest to you folks to draw your own conclusions from what this might mean...

Best, Paul March
Title: Re: EM Drive Developments - related to space flight applications - Thread 2
Post by: Mulletron on 02/08/2015 03:24 am
This news article got a lot of attention here:
http://www.sciencedaily.com/releases/2014/07/140722091425.htm

A couple of us posted that it feels very relevant to the dynamics inside of a resonant cavity. What is interesting is the end result of confining vacuum fluctuations:
Quote
In that case, the fluctuation-mediated attraction between the atoms becomes orders of magnitude stronger than in free space. Usually, the force decreases rapidly with increasing distance between the atoms. Due to the transmission line, it falls off with one over the distance cubed, instead of one over the seventh power of the distance, as in the usual case.

As far as the difference in vacuum energy goes, we've discussed the possibility that there exists a "more negative" energy condition at the small end of the cavity WRT the large end. Less modes fit small end vs large end. No calculations were made.

http://forum.nasaspaceflight.com/index.php?topic=29276.msg1298712#msg1298712
Quote
Either way, I know there is vacuum energy difference in potential from the top to the bottom of the cavity and those relative differences are all that matter. From there, after mathematical conversion to momentum, it doesn't take a mathematician to know that the competing vacuum and RF momentum contributions to the dielectric aren't exactly equal.
Anyway it is just words without math to back it up.
Title: Re: EM Drive Developments - related to space flight applications - Thread 2
Post by: sghill on 02/08/2015 03:45 am
Folks:

If the quantum vacuum is degradable and malleable as we think it should be, then to conserve momentum a QV wake has to be generated in the QV media as a Q-Thruster goes by just like a ship's propeller leaves a disturbance in the water as it goes by.  We think that the density of the QV is normally around its cosmological average of 9.1x10^-27 kg/m^3, but its density can be greatly increased by the presence of E&M fields and especially very strong and fast time-varying E&M fields that occur is microwave resonant cavities with large Quality Factors greater than say 1,000, or around elementary charged particles like electrons or protons where the QV density goes up to nuclear mass density as you approach the surface of the particle.  Suggest anything?  However in the paper we are now trying to get published with no takers so far, we find that the QV density should drop off very rapidly from a high density volume like a proton and in fact it follows the same drop off in density with distance as the Casimir effect does, i.e., 1 / r^4 where r = the distance from the resonant cavity boundary.  With that being the case it would be near impossible to detect the QV wake behind a Q-Thruster only generating milliNewtons or Newtons or even in tens of Newtons. 

So what's to do?  To detect a QV wake from a Q-thruster at even short distances from the source we think we will have to use another RF excited resonant cavity in a form of QV parametric amplification that is designed to produce a high density QV state just like in a Q-Thruster, but not to produce thrust.  Instead it will be optimized to monitor its time varying QV density as various very weak QV wake fields come in, are amplified and detected, then pass out of it again to go back to the low density QV state once again.  This has some interesting implications especially when you finish reading the attached paper from a PhD from Rice University here in Houston.

Last topic for the night for me.  Someone on this list asked if one could extract energy from the QV.  If the QV is GRT space-time, and space-time is the cosmological gravitational field that is created by all the causally connected mass/energy in our section of the universe, then we live in a high pressure sea of gravitational energy.  Now if the QV energy state is degradable and locally changeable, then one can posit the possibility of a thermodynamic energy conversion cycle that can extract energy from a pressure difference created in this QV media relative to the QV background average pressure, with a net decrease in this universal gravitational pressure or temperature reflective of the amount of energy so extracted.  And try to remember that gravitational energy is negative energy.  I'll leave the rest to you folks to draw your own conclusions from what this might mean...

Best, Paul March

Thank you for participating in the  forum Paul. As far as the paper goes, why not publish publicly and let your peers see it and validate it without the "Star Chamber" reviewers?

Regarding the QV wake, does measuring it really matter in terms of validity if tens of Newtons of thrust (or more) are predictably being measured?
 [Serious question]

As to your last few sentences. Woah!!!.......
Title: Re: EM Drive Developments - related to space flight applications - Thread 2
Post by: birchoff on 02/08/2015 04:35 am
Folks:

If the quantum vacuum is degradable and malleable as we think it should be, then to conserve momentum a QV wake has to be generated in the QV media as a Q-Thruster goes by just like a ship's propeller leaves a disturbance in the water as it goes by.  We think that the density of the QV is normally around its cosmological average of 9.1x10^-27 kg/m^3, but its density can be greatly increased by the presence of E&M fields and especially very strong and fast time-varying E&M fields that occur is microwave resonant cavities with large Quality Factors greater than say 1,000, or around elementary charged particles like electrons or protons where the QV density goes up to nuclear mass density as you approach the surface of the particle.  Suggest anything?  However in the paper we are now trying to get published with no takers so far, we find that the QV density should drop off very rapidly from a high density volume like a proton and in fact it follows the same drop off in density with distance as the Casimir effect does, i.e., 1 / r^4 where r = the distance from the resonant cavity boundary.  With that being the case it would be near impossible to detect the QV wake behind a Q-Thruster only generating milliNewtons or Newtons or even in tens of Newtons. 

So what's to do?  To detect a QV wake from a Q-thruster at even short distances from the source we think we will have to use another RF excited resonant cavity in a form of QV parametric amplification that is designed to produce a high density QV state just like in a Q-Thruster, but not to produce thrust.  Instead it will be optimized to monitor its time varying QV density as various very weak QV wake fields come in, are amplified and detected, then pass out of it again to go back to the low density QV state once again.  This has some interesting implications especially when you finish reading the attached paper from a PhD from Rice University here in Houston.

Last topic for the night for me.  Someone on this list asked if one could extract energy from the QV.  If the QV is GRT space-time, and space-time is the cosmological gravitational field that is created by all the causally connected mass/energy in our section of the universe, then we live in a high pressure sea of gravitational energy.  Now if the QV energy state is degradable and locally changeable, then one can posit the possibility of a thermodynamic energy conversion cycle that can extract energy from a pressure difference created in this QV media relative to the QV background average pressure, with a net decrease in this universal gravitational pressure or temperature reflective of the amount of energy so extracted.  And try to remember that gravitational energy is negative energy.  I'll leave the rest to you folks to draw your own conclusions from what this might mean...

Best, Paul March

Thank you for participating in the  forum Paul. As far as the paper goes, why not publish publicly and let your peers see it and validate it without the "Star Chamber" reviewers?

Regarding the QV wake, does measuring it really matter in terms of validity if tens of Newtons of thrust (or more) are predictably being measured?
 [Serious question]

As to your third to last sentence. Woah!!!.......

Well you have two issues at play here. First off is whether or not the device produces thrust. The second one is whether or not Dr. White's theory is the correct explanation for why the device produces thrust. being able to measure tens of newtons of thrust would simply get you easily verifiable proof that the device produces thrust. On the other hand it will not get your proof that Dr. White's proposed theory is the correct explanation. That said, showing a strong correlation between the observed scaling with predicted scaling and using the proposed theory to predict the null configuration of the device (verified by observation). All would strengthen the case for why the proposed theory that explains the operation of the device is correct. Assuming Dr. White had all those things, being able to observe some sort of "Wake" would be the icing on the cake, especially if the device used to detect the wake is predicted by the theory.

Title: Re: EM Drive Developments - related to space flight applications - Thread 2
Post by: flux_capacitor on 02/08/2015 01:51 pm
Hi Paul,

Glad to see some improvement on experiments!

In January 2015, Roger Shawyer put a document online entitled A Note on the Principles of EmDrive force measurement (http://www.emdrive.com/EmDriveForceMeasurement.pdf).
Please leave out his theoretical explanation about group velocities for now and let's focus on his experimental claims about measurements of reaction forces. He basically says:

1. The EmDrive creates two opposite forces:
- a thrust T
- a reaction force R = Ma
where M = mass of the thruster, and a = acceleration of the thruster.

2. Not net force can be measured if the cavity is absolutely static, because in this case T and R cancel out.

3. The cavity needs to move (to accelerate) even a bit for the forces to appear, even if the acceleration is tiny and due for example to a thermal expansion of the cavity walls.

4. According to the experimental setup, either T or R can be measured, hence a difference in the direction of the force (toward the smaller or bigger plate of the cavity).

What do you think about those claims? Especially the one about the reaction force R.
Title: Re: EM Drive Developments - related to space flight applications - Thread 2
Post by: Notsosureofit on 02/08/2015 02:06 pm
@ Mulletron

["As far as the difference in vacuum energy goes, we've discussed the possibility that there exists a "more negative" energy condition at the small end of the cavity WRT the large end. Less modes fit small end vs large end. No calculations were made."]

This is the dispersion relation calculation.  The evaluation is made as the difference from one end of the cavity to the other.  A "boost" is made to an accelerated frame of reference which eliminates that difference, ie. "v is everywhere close to c"  This is just the "trivial" approximation as in:

* Hydrodynamics of the Vacuum_0409292v2.pdf

" However, the vacuum is a Lorentz invariant medium; it has no rest frame. The appropriate frame for the NFA is determined solely by the initial conditions. If in some frame the NFA conditions are satisfied at t = 0 then they will remain satisfied at all later times. One may trivially take a NFA solution and boost it by a large Lorentz boost to obtain an approximate solution to the original relativistic equations in which v is everywhere close to 1. Only when the range of v values is a significant fraction of unity is it necessary to abandon the NFA and return to the relativistic equations, (4.26, 4.27)."

The (static) force then appears as the equivalent "weight" of the photons in the AFR.

Edit:  To get to the QM version you need to evaluate a total wavefunction and show that it satisfies (for example) the self-accelerating condition, or the Sachs-Schwebel current, or some equivalent particle-pair generation criteria.  The self-acceleration looks good so far, and conserves momentum.
Title: Re: EM Drive Developments - related to space flight applications - Thread 2
Post by: Rodal on 02/08/2015 02:23 pm
@ Mulletron

["As far as the difference in vacuum energy goes, we've discussed the possibility that there exists a "more negative" energy condition at the small end of the cavity WRT the large end. Less modes fit small end vs large end. No calculations were made."]

This is the dispersion relation calculation.  The evaluation is made as the difference from one end of the cavity to the other.  A "boost" is made to an accelerated frame of reference which eliminates that difference, ie. "v is everywhere close to c"  This is just the "trivial" approximation as in:

* Hydrodynamics of the Vacuum_0409292v2.pdf

" However, the vacuum is a Lorentz invariant medium; it has no rest frame. The appropriate frame for the NFA is determined solely by the initial conditions. If in some frame the NFA conditions are satisfied at t = 0 then they will remain satisfied at all later times. One may trivially take a NFA solution and boost it by a large Lorentz boost to obtain an approximate solution to the original relativistic equations in which v is everywhere close to 1. Only when the range of v values is a significant fraction of unity is it necessary to abandon the NFA and return to the relativistic equations, (4.26, 4.27)."

The (static) force then appears as the equivalent "weight" of the photons in the AFR.

Good point, that's in page 8 of Hydrodynamics of the Vacuum_0409292v2.pdf

Also in page 9:

Quote from: page 9 of Hydrodynamics of the Vacuum_0409292v2.pdf
Although the flow velocity is nonrelativistic (v ≪ 1), disturbances tend to “propagate” superluminally, at 1/v. Hence, the NFA here is not a normal nonrelativistic reduction. The resulting equations are “anti-Galilean” invariant...This is certainly strange, and takes some getting used to, but one should simply view it
as an approximation to the full Lorentz transformations, valid in the stated context. One
is used to dealing with small objects that move slowly, so that their density distributions
vary rapidly in space, but slowly in time. In the present case one is dealing with large
objects, slowly varying in space, but relatively rapidly varying in time. This is related to
the fact that the Higgs vacuum, as a spontaneous Bose-Einstein condensate, has almost
all its particles in the same quantum state. Small disturbances of this state involve vast
numbers of particles, spread over long distances, all moving nearly in lockstep, so that
the disturbance varies only slowly with position while the whole collective has the same,
relatively rapid time dependence.
(Bold added for emphasis)
Title: Re: EM Drive Developments - related to space flight applications - Thread 2
Post by: Notsosureofit on 02/08/2015 02:42 pm
Yes, I have to look at that in the self-acceleration.  Seems I thought they were using instantaneous, but I'll have to check.  It's pretty heavy going for an old guy.
Title: Re: EM Drive Developments - related to space flight applications - Thread 2
Post by: Mulletron on 02/08/2015 03:17 pm
Paul, so last summer it was reported that TE012 was a top performer yet difficult to work with:
Quote
The measured power applied to the test article was measured to be 2.6 watts, and the (net) measured thrust was 55.4 micronewtons. With an input power of 2.6 watts, correcting for the quality factor, the predicted thrust is 50 micronewtons. However, since the TE012 mode had numerous other RF modes in very close proximity, it was impractical to repeatedly operate the system in this mode, so the decision was made to evaluate the TM211 modes instead.

The landscape appears to have changed somewhat:
Quote
approx. +50 micro-Newton (uN) with 50W at 1,937.115 MHz
Quote
BTW, we have found that both the TE and TM E&M modes of this copper frustum can produce a thrust signature, but so far the TM modes appear to be the better performer, at least for the few modes we have been able to study to date. 

So a couple questions from this. It appears that TM modes are the top dogs now and at the same time performance has gone down significantly since vacuum testing began. See table below for what I mean. TM212 reported now vs TM211 reported then? Do you have any insight about this? Did the vacuum serve to eliminate artifact thrust signals significantly?

Also is TE012 still a good q-thruster performer, but it is just a dog to work with? It showed promising results with only 2.6 watts input producing 55.4uN of thrust (only 1 run though). Is TE012 not so good after all? If so, what made the difference?

Finally, quoting from last summer's paper:
Quote
The tapered thruster has a mechanical design such that it will be able to hold pressure at 14.7 pounds per square inch (psi) inside of the thruster body while the thruster is tested at vacuum to preclude glow discharge within the thruster body while it is being operated at high power.

(http://forum.nasaspaceflight.com/index.php?action=dlattach;topic=36313.0;attach=634622;image)

So it appears the cavity contains atmospheric pressure air inside. Given the thin walls of the cavity and the end sections, did you experience any issues with the resonant cavity expanding under suction of vacuum? And did it affect your testing significantly?

Thanks in advance. There's likely to be a lot of questions coming. We really appreciate all that you have done so far.

On a separate note @Notsosureofit made a calculation based off the dispersion relation inside the cavity that yielded pretty tight results starting here: http://forum.nasaspaceflight.com/index.php?topic=36313.msg1317866#msg1317866


Title: Re: EM Drive Developments - related to space flight applications - Thread 2
Post by: Rodal on 02/08/2015 03:48 pm
Paul, so last summer it was reported that TE012 was a top performer yet difficult to work with:
...
The landscape appears to have changed somewhat:
Quote
approx. +50 micro-Newton (uN) with 50W at 1,937.115 MHz
Quote
BTW, we have found that both the TE and TM E&M modes of this copper frustum can produce a thrust signature, but so far the TM modes appear to be the better performer, at least for the few modes we have been able to study to date. 

So a couple questions from this. It appears that TM modes are the top dogs now and at the same time performance has gone down significantly since vacuum testing began. See table below for what I mean. TM212 reported now vs TM211 reported then? Do you have any insight about this? Did the vacuum serve to eliminate artifact thrust signals significantly?

Some caveats here:

1) When discussing mode shapes in experiments, the first question is whether the mode shape has been actually verified with experimental measurements.  My reading of the information is that the answer is NO.   There has been no experimental verification of what the actual mode shape was in the NASA experiments (either in the original "Anomalous ..." Brady et.al. report or in the new experiments).  Is my reading correct? Has anyone actually measured the electric and magnetic fields inside the EM Drive during the experiments to experimentally verify what the actual mode shape was?  Are there experimental measurements showing the orientation, gradient, and magnitude of the electrical and magnetic fields inside the EM Drive that can enable NASA to plot contour plots of the mode shapes to asses what particular mode shapes have taken place?

2) The mode shapes discussed in the original "Anomalous ..." Brady et.al. report or in the new experiments are based on COMSOL Finite Element modeling of the experiments.  Having written Finite Element computer programs, having been involved in their theoretical formulation for very nonlinear problems at MIT and elsewhere, as well as their numerical implementation, and having used (and written constitutive equation subroutines for) commercial codes like ADINA, ANSYS, and others, I'm quite aware of issues dealing with a) theoretical formulation, b) numerical implementation and c) convergence of the Finite Element mesh.  By no means one can accept a Finite Element solution as a correct modeling of reality.  As a minimum one needs to a) show convergence of the finite element solution and preferably b) comparison of the Finite Element solver to exact solutions.  In this case, there is a readily available solution for cylindrical cavities.  I would like to see a comparison of COMSOL Finite Element (using a similar space and time domain discretization) to the exact solution. 

3) For the problem at hand (truncated cone cavity) we know that exact solutions show that the truncated cone has mode shapes that are similar to those in a cylindrical cavity, but that the mode shapes in a truncated cone  have a transition to an evanescent region. A strict distinction between pure propagating and pure evanescent modes in a truncated cone can not be achieved.  Hence from a rigorous point of view, it is not correct to use the same terminology for the modes in a truncated cone than as used for the modes in a cylindrical cavity (TM212 or TE012, for example).  If we are going to use the same terminology, we need to better define our convention: is this terminology used in the sense that only real solutions to the eigenvalue problem in a truncated cone are taken into account?

Therefore, there are very important issues discussing what the actual mode shape in these experiments are.

I suggest to proceed as follows.  Let's first start with addressing whether the actual mode shapes were measured in the experiments and if so how they were measured. 

If the answer is no, that they were not measured, then let's then proceed to validate a numerical solution.  Can NASA show a comparison of a COMSOL Finite Element solution for a cylindrical cavity (with similar discretization as used for the truncated cone) vs. the exact solution?

Once that has been done, we should proceed to the next question: is the COMSOL Finite Element solution solving the eigenvalue problem for the truncated cone taking into account evanescent modes, that is, is it considering complex value solutions to the eigenvalue problem? or is it only considering real solutions to the eigenvalue problem?
Title: Re: EM Drive Developments - related to space flight applications - Thread 2
Post by: Star-Drive on 02/08/2015 04:25 pm
Rodal:

Look at the copper frustum part-2 COMSOL & IR thermal study that I submitted to this group yesterday for an answer to your "Have you experimentally verified that we are using the TM212 mode as predicted by our COMSOL simulations?  The answer BTW is yes for the TM212 mode, but no for the TE012 mode, but since COMSOL predicted the right PE loaded resonant frequency for the TE212 mode as verified by my IR camera studies 1 & 2 of the copper frustum, I would assume that it got it right for the TE012 mode as well.  In fact I should have provided you my IR study-1 first, so find it attached.

Best, Paul M. 
Title: Re: EM Drive Developments - related to space flight applications - Thread 2
Post by: Star One on 02/08/2015 04:33 pm

Folks:

If the quantum vacuum is degradable and malleable as we think it should be, then to conserve momentum a QV wake has to be generated in the QV media as a Q-Thruster goes by just like a ship's propeller leaves a disturbance in the water as it goes by.  We think that the density of the QV is normally around its cosmological average of 9.1x10^-27 kg/m^3, but its density can be greatly increased by the presence of E&M fields and especially very strong and fast time-varying E&M fields that occur is microwave resonant cavities with large Quality Factors greater than say 1,000, or around elementary charged particles like electrons or protons where the QV density goes up to nuclear mass density as you approach the surface of the particle.  Suggest anything?  However in the paper we are now trying to get published with no takers so far, we find that the QV density should drop off very rapidly from a high density volume like a proton and in fact it follows the same drop off in density with distance as the Casimir effect does, i.e., 1 / r^4 where r = the distance from the resonant cavity boundary.  With that being the case it would be near impossible to detect the QV wake behind a Q-Thruster only generating milliNewtons or Newtons or even in tens of Newtons. 

So what's to do?  To detect a QV wake from a Q-thruster at even short distances from the source we think we will have to use another RF excited resonant cavity in a form of QV parametric amplification that is designed to produce a high density QV state just like in a Q-Thruster, but not to produce thrust.  Instead it will be optimized to monitor its time varying QV density as various very weak QV wake fields come in, are amplified and detected, then pass out of it again to go back to the low density QV state once again.  This has some interesting implications especially when you finish reading the attached paper from a PhD from Rice University here in Houston.

Last topic for the night for me.  Someone on this list asked if one could extract energy from the QV.  If the QV is GRT space-time, and space-time is the cosmological gravitational field that is created by all the causally connected mass/energy in our section of the universe, then we live in a high pressure sea of gravitational energy.  Now if the QV energy state is degradable and locally changeable, then one can posit the possibility of a thermodynamic energy conversion cycle that can extract energy from a pressure difference created in this QV media relative to the QV background average pressure, with a net decrease in this universal gravitational pressure or temperature reflective of the amount of energy so extracted.  And try to remember that gravitational energy is negative energy.  I'll leave the rest to you folks to draw your own conclusions from what this might mean...

Best, Paul March

Thank you for participating in the  forum Paul. As far as the paper goes, why not publish publicly and let your peers see it and validate it without the "Star Chamber" reviewers?

Regarding the QV wake, does measuring it really matter in terms of validity if tens of Newtons of thrust (or more) are predictably being measured?
 [Serious question]

As to your last few sentences. Woah!!!.......

As too publishing publicly, where publicly would be the best place to publish to get the most eyes on it from the type of people you want to see it?
Title: Re: EM Drive Developments - related to space flight applications - Thread 2
Post by: Rodal on 02/08/2015 04:48 pm
Rodal:

Look at the copper frustum part-2 COMSOL & IR thermal study that I submitted to this group yesterday for an answer to your "Have you experimentally verified that we are using the TM212 mode as predicted by our COMSOL simulations?  The answer BTW is yes for the TM212 mode, but no for the TE012 mode, but since COMSOL predicted the right PE loaded resonant frequency for the TE212 mode as verified by my IR camera studies 1 & 2 of the copper frustum, I would assume that it got it right for the TE012 mode as well.  In fact I should have provided you my IR study-1 first, so find it attached.

Best, Paul M.

Paul, thank you.  This is the first time I see the attached IR study  ("Comparison of COMSOL Predictions of Copper Frustrum Heat Dissipation with Dec 30 IR Data") .  It was an excellent idea for NASA to conduct this  experimental study to verify the TM212 mode.  I congratulate you for that because it is of the utmost importance to understand the actual mode shapes being excited.

I attach the TM21 mode for a cylindrical cavity for comparison with TM21 in the truncated cone

Magnetic field: - - - - - - dashed lines
Electric field:   _______solid  lines

PS: Concerning whether COMSOL's discretization predicting TE012 was correct, my attitude (based on conducting experiments and numerical analysis) is always I'm from Missouri "show me"  :), so I would rather also have experimental verification for that experiment as well.  But considering your tight budget constraints, you deserve congratulations for what has been done.
Title: Re: EM Drive Developments - related to space flight applications - Thread 2
Post by: Rodal on 02/08/2015 05:33 pm
Rodal:

Look at the copper frustum part-2 COMSOL & IR thermal study that I submitted to this group yesterday for an answer to your "Have you experimentally verified that we are using the TM212 mode as predicted by our COMSOL simulations?  The answer BTW is yes for the TM212 mode, but no for the TE012 mode, but since COMSOL predicted the right PE loaded resonant frequency for the TE212 mode as verified by my IR camera studies 1 & 2 of the copper frustum, I would assume that it got it right for the TE012 mode as well.  In fact I should have provided you my IR study-1 first, so find it attached.

Best, Paul M.

Paul, thank you.  This is the first time I see the attached IR study  ("Comparison of COMSOL Predictions of Copper Frustrum Heat Dissipation with Dec 30 IR Data") .  It was an excellent idea for NASA to conduct this  experimental study to verify the TM212 mode.  I congratulate you for that because it is of the utmost importance to understand the actual mode shapes being excited.

I attach the TM21 mode for a cylindrical cavity for comparison with TM21 in the truncated cone

Magnetic field: - - - - - - dashed lines
Electric field:   _______solid  lines

PS: Concerning whether COMSOL's discretization predicting TE012 was correct, my attitude (based on conducting experiments and numerical analysis) is always I'm from Missouri "show me"  :), so I would rather also have experimental verification for that experiment as well.  But considering your tight budget constraints, you deserve congratulations for what has been done.

Notice from:

"Anomalous Thrust Production from an RF Test Device Measured on a Low-Thrust Torsion Pendulum" ( http://www.libertariannews.org/wp-content/uploads/2014/07/AnomalousThrustProductionFromanRFTestDevice-BradyEtAl.pdf )



TE012  mode shape at 1.8804 GHz Table 2. Tapered Cavity Testing Summary (with the dielectric)
TM211 mode shape at 1.9326 GHz Table 2. Tapered Cavity Testing Summary (with the dielectric)
TM211 mode shape at 1.9367 GHz Table 2. Tapered Cavity Testing Summary (with the dielectric)
TM212 mode shape at 1.937188 GHz (Jan 2015 data (with the dielectric))


(TM212 mode shape at 1.937188 GHz from here http://forum.nasaspaceflight.com/index.php?topic=36313.msg1327177#msg1327177 )

(or an inconsequential difference 0.5%: TM212 mode shape at 1.946647 GHz http://forum.nasaspaceflight.com/index.php?action=dlattach;topic=36313.0;attach=634723 )

TM211 and TM212 have the same number of wave patterns in the circumferential and radial directions, but

TM211 has one half-wave pattern in the longitudinal direction
TM212 has two half-wave patterns in the longitudinal direction  (twice as many as TM211)

yet they are reported to take place at practically the same frequency ?

Now, mode shapes also depend on:

a)geometry (I understand that the same tapered cone was used for all the above experiments, so that was not a variable)
b) relative electric permittivity and relative magnetic permeability (I understand that the same dielectric  was used for all the above experiments, so that was not a variable)

So, if the geometry and the dielectric were the same in the above experiments:

QUESTION: 1.937188 GHz is only 0.025% different from  1.9367 GHZ. 
It is not possible to have TM211 at 1.9367 GHz and TM212 at just 0.025% higher frequency, because TM211 has 1 half-wave in the longitudinal direction, while TM212 has 2 half-waves in the longitudinal direction. Twice the number of half-waves in the longitudinal direction imply a significantly higher frequency keeping the number of wave-patterns constant in the circumferential and radial directions (the same m=2 and n=1). Therefore doubling the number of half-waves in the longitudinal direction with just a  0.025%  increase in frequency cannot be justified.

Therefore it appears that the labeling of mode TM211 in the Brady et.al. NASA report was an error, and it really should have been TM212 based on the Jan 2015 IR data
If so, was there was an error in the COMSOL analysis for TE012 as well as for TM211  in the report since they may have used the same COMSOL analysis and mesh?

(http://www.optique-ingenieur.org/en/courses/OPI_ang_M01_C03/res/fig02_1.jpg)



Also, in "Anomalous Thrust Production from an RF Test Device Measured on a Low-Thrust Torsion Pendulum" ( http://www.libertariannews.org/wp-content/uploads/2014/07/AnomalousThrustProductionFromanRFTestDevice-BradyEtAl.pdf )

page 18 (bottom) states (without the dielectric)

TE012 mode shape at  2.168 GHz . 

So that the same mode shape (TE012) is stated to take place with the dielectric at 1.8804 GHz and without the dielectric at (15% higher frequency) 2.168 GHz, but there was no measurable response without the dielectric.

However,

Quote from: page 18 of Brady et.al."Anomalous Thrust Production from an RF Test Device Measured on a Low-Thrust Torsion Pendulum"

Numerous COMSOL® analysis runs also indicated a strong dependency between thrust magnitude and antenna type, location, orientation, and number of antenna feeds. Slight changes in antenna design and number of feeds changed the COMSOL® thrust prediction by a factor of three which forced our team to implement tighter configuration control protocols during testing to ensure close representation of as built hardware to the analyzed configuration.

Finally, our experience with the TE012 mode indicated that it is important to design the RF prototype such that any target mode of operation is as isolated as possible in the frequency domain to help ensure that the system can be effectively tuned manually. This also protects for the ability to implement and use a phase lock loop (PLL) automated frequency control circuit. Due to the slow process commensurate with manual tuning, our future test articles will make use of a PLL whenever practical in order to increase the amount of data that can be collected for a given test article configuration and operating condition during a given amount of test time
(Bold added for emphasis)

So it is not clear whether the "no appreciable response without the dielectric at 2.168 GHz" may have been due to "slight changes in antenna location and orientation" in the  2.168 GHz experiment without the dielectric compared to the experiment at 1.8804 GHz with the dielectric.

Also the above-mentioned report states:

Quote from: page 17 of Brady et.al."Anomalous Thrust Production from an RF Test Device Measured on a Low-Thrust Torsion Pendulum"
the TE012 mode had numerous other RF modes in very close proximity, it was impractical to repeatedly operate the system in this mode, so the decision was made to evaluate the TM211 modes instead.
(Bold added for emphasis)

perhaps throwing further uncertainty as to what resonance took place at  1.8804 GHz
Title: Re: EM Drive Developments - related to space flight applications - Thread 2
Post by: Mulletron on 02/08/2015 06:19 pm
Quote
TM212 mode shape at 1.937188 GHz (Jan 2015 data (with the dielectric))

That comsol plot above says 1946.647. Where's the disconnect between numbers reported I wonder? Also unless comsol is taking into account all the little intricacies like heat expansion, bowing and buckling, simulation won't yield an exact result in reality. The thermal image pretty much nails whether or not the excited mode is actually being excited though.
Title: Re: EM Drive Developments - related to space flight applications - Thread 2
Post by: Rodal on 02/08/2015 06:29 pm
Quote
TM212 mode shape at 1.937188 GHz (Jan 2015 data (with the dielectric))

That comsol plot above says 1946.647. Where's the disconnect between numbers reported I wonder? Also unless comsol is taking into account all the little intricacies like heat expansion, bowing and buckling, simulation won't yield an exact result in reality.

1) I took the 1.937188 GHz figure from here:  http://forum.nasaspaceflight.com/index.php?topic=36313.msg1327177#msg1327177

Quote from: Star-Drive http://forum.nasaspaceflight.com/index.php?topic=36313.msg1327177#msg1327177
In that slide which is based on the copper frustum cavity running in its TM212 mode with 50W of 1,937.188 MHz RF power applied

But 1946.647 makes practically NO difference.  One cannot have TM212 at only 0.5% higher frequency than TM211

2) My reading is that the COMSOL Multiphysics simulation did not take into account buckling.  Buckling is an instability that requires a nonlinear stability analysis.

3) "simulation won't yield an exact result in reality".  No way this can justify  TM212 at only 0.5% higher frequency than TM211 for the same known truncated cone geometry and identical dielectric material .  A Finite Element calculation converges from below, it may produce too low a frequency for a given mode shape if the finite element mesh is too crude, but it is not going to give TM212 at only 0.5% higher frequency than TM211 for a cavity with the known dimensions of the NASA truncated cone, since TM212 has twice the number of longitudinal half-wave patterns as TM211
Title: Re: EM Drive Developments - related to space flight applications - Thread 2
Post by: Mulletron on 02/08/2015 06:33 pm
I get that, but with a cavity loaded with a dielectric, you can't add up wavelengths trying to satisfy E field requirements like we do with empty waveguides. With the dielectric inserted, there is a dielectric resonator inside the cavity resonator which means the solution is more complicated to figure out by hand.
Title: Re: EM Drive Developments - related to space flight applications - Thread 2
Post by: Rodal on 02/08/2015 06:36 pm
I get that, but with a cavity loaded with a dielectric, you can't add up wavelengths trying to satisfy E field requirements like we do with empty waveguides. With the dielectric inserted, there is a dielectric resonator inside the cavity resonator which means the solution is more complicated to figure out by hand.
I am not figuring out the solution by hand.  I am using Mathematica.  Also, I have based my above statements on my extensive experience developing Finite Element formulations for nonlinear analysis,  writing code for Finite Element programs and using commercial Finite Element codes to solve practical problems. 

Concerning the dielectric, COMSOL uses the dielectric constant: the electric permittivity.  My understanding is that the same dielectric was used for both the experiments for which COMSOL is stated as giving TM211 and TM212

If NASA used substantially-different truncated cone geometry or  substantially-different dielectric geometry or materials for the experiments reportedly giving TM211 and TM212, it is useful to find out.

If I have made an error somewhere, I would appreciate it being pointed out. 

If there is an error in the NASA report, it is useful to find out. It is not uncommon at all for articles to have errata, and in my experience, authors (certainly in the R&D field), are always appreciative when such errors are pointed out.
Title: Re: EM Drive Developments - related to space flight applications - Thread 2
Post by: Star-Drive on 02/08/2015 07:00 pm
All:

Please note that our first COMSOL analyst who was volunteering his time for this activity while holding down his NASA day job, transcribed the dimensions for the copper frustum PE discs incorrectly when he did the analysis for the TE012 mode.  He used 6.0" OD by 1.0" thick whereas the actual dimensions for the PE discs was 6.13" OD by 1.062" thick.  The extra volume in the two PE discs lowered the actual observed resonant frequencies for all the resonant modes in the cavity down by about 8-to-10 MHz from COMSOL calculated.   When you have to beg for help, one can't be too critical of the results.  As to the TM212 mode analysis it was performed by another volunteer, so again I'm not going to complain that he didn't get these calculated frequencies spot on to what was measured with our Agilent Field-Fox Vector Network Analyzer (VNA) measured.

Rodal:

The next time we look at the TE012 mode I will perform the same IR camera survey I did for the TM212 mode.  The reason I didn't do the IR camera survey of the TE012 mode the first time around was that we didn't have that capability during March of 2014 when we ran that test series, since the IR camera didn't come along until the summer of 2014.

Best, Paul M.
Title: Re: EM Drive Developments - related to space flight applications - Thread 2
Post by: Mulletron on 02/08/2015 07:08 pm
OMG that is the best thing ever!!!! :) :D  Thanks!

(http://forum.nasaspaceflight.com/index.php?action=dlattach;topic=36313.0;attach=634742;image)


(http://forum.nasaspaceflight.com/index.php?action=dlattach;topic=36313.0;attach=634740;image)
Wow, what a day. Thanks Paul March. Seriously thanks.
And there ya go, ISM band dimensions. Any competent tinkerer should be able to try it for themselves if they are so inclined. Be safe.

Let's not forget that if these Emdrives do work (looking pretty likely), Shawyer had the vision to make them happen. The folks at NASA had the courage to take a look.
Title: Re: EM Drive Developments - related to space flight applications - Thread 2
Post by: Rodal on 02/08/2015 07:14 pm
All:

Please note that our first COMSOL analyst who was volunteering his time for this activity while holding down his NASA day job, transcribed the dimensions for the copper frustum PE discs incorrectly when he did the analysis for the TE012 mode.  He used 6.0" OD by 1.0" thick whereas the actual dimensions for the PE discs was 6.13" OD by 1.062" thick.  The extra volume in the two PE discs lowered the actual observed resonant frequencies for all the resonant modes in the cavity down by about 8-to-10 MHz from COMSOL calculated.   When you have to beg for help, one can't be too critical of the results.  As to the TM212 mode analysis it was performed by another volunteer, so again I'm not going to complain that he didn't get these calculated frequencies spot on to what was measured with our Agilent Field-Fox Vector Network Analyzer (VNA) measured.

Rodal:

The next time we look at the TE012 mode I will perform the same IR camera survey I did for the TM212 mode.  The reason I didn't do the IR camera survey of the TE012 mode the first time around was that we didn't have that capability during March of 2014 when we ran that test series, since the IR camera didn't come along until the summer of 2014.

Best, Paul M.

Paul,

Thanks for the response.  It is a pleasure communicating with you. 

It is therefore my understanding (based on the IR data) that the NASA report should have read TM212 instead of TM211

"Anomalous Thrust Production from an RF Test Device Measured on a Low-Thrust Torsion Pendulum" ( http://www.libertariannews.org/wp-content/uploads/2014/07/AnomalousThrustProductionFromanRFTestDevice-BradyEtAl.pdf )


TM211 mode shape at 1.9326 GHz Table 2. Tapered Cavity Testing Summary (with the dielectric)

should read:

TM212 mode shape at 1.9326 GHz Table 2. Tapered Cavity Testing Summary (with the dielectric)
Title: Re: EM Drive Developments - related to space flight applications - Thread 2
Post by: ThinkerX on 02/08/2015 08:21 pm
If I am following the last couple pages correctly at all, it seems that the cavity dimensions relative to the frequency is of supreme importance to making this device work. 

I am reminded of the precision machining required to properly re-bore cylinders in car engines: get that wrong, even by a very tiny fraction, and the engine won't run properly.  I've known a number of people who didn't get that right.  It seems the same degree of precision is required here. 
Title: Re: EM Drive Developments - related to space flight applications - Thread 2
Post by: Star One on 02/08/2015 08:25 pm
If I am following the last couple pages correctly at all, it seems that the cavity dimensions relative to the frequency is of supreme importance to making this device work. 

I am reminded of the precision machining required to properly re-bore cylinders in car engines: get that wrong, even by a very tiny fraction, and the engine won't run properly.  I've known a number of people who didn't get that right.  It seems the same degree of precision is required here.
Sounds like it may make them tricky to reproduce at least initially.
Title: Re: EM Drive Developments - related to space flight applications - Thread 2
Post by: Mulletron on 02/08/2015 08:38 pm
If I am following the last couple pages correctly at all, it seems that the cavity dimensions relative to the frequency is of supreme importance to making this device work. 

I am reminded of the precision machining required to properly re-bore cylinders in car engines: get that wrong, even by a very tiny fraction, and the engine won't run properly.  I've known a number of people who didn't get that right.  It seems the same degree of precision is required here.
Sounds like it may make them tricky to reproduce at least initially.

Yes and here is how you tune a resonant cavity:
http://www.navymars.org/national/training/nmo_courses/nmo1/module11/14183_ch1.pdf

(http://www.wired.com/images_blogs/dangerroom/images/2008/09/24/emdrive_2.jpg)
The cylinder part of the thruster above is for volume tuning.
If your frequency source is fixed, you have to tune your cavity. If you can change your frequency source, the cavity can remain fixed.
Title: Re: EM Drive Developments - related to space flight applications - Thread 2
Post by: Rodal on 02/08/2015 09:20 pm
If I am following the last couple pages correctly at all, it seems that the cavity dimensions relative to the frequency is of supreme importance to making this device work. 

I am reminded of the precision machining required to properly re-bore cylinders in car engines: get that wrong, even by a very tiny fraction, and the engine won't run properly.  I've known a number of people who didn't get that right.  It seems the same degree of precision is required here.

Sorry, but in order to have clarity, I am obliged to point out that I find it difficult to believe that a marginal change like this one:

" 6.0" OD by 1.0" thick whereas the actual dimensions for the PE discs was 6.13" OD by 1.062" thick"

can produce a change from TM211 into TM212 mode.

I think that what happened is that in a truncated cone the (electric and magnetic field) strengths attenuate   (the fields become weaker) considerably as one gets closer to the apex of the cone (going from the Big Diameter towards the Small Diameter).  This is shown in the literature of exact solutions for the truncated cone.  It is a feature of the truncated cone, and I think it maybe a reason why a truncated cone performs better as an EMDrive because this field attenuation towards the apex plays an important role.

Therefore, due to this field attenuation, it becomes difficult to distinguish TM211 from TM212 (and even from TM213) for a truncated cone.



I have to think hard, taking a look at the pictures below, whether the mode is TM211 or whether it is TM212.

This is very different from a cylindrical cavity, because in a cylindrical cavity there is no such attenuation of field strength toward a conical apex.

Therefore the error is a human error of distinguishing what happens near the small diameter (near the apex of the cone), and the error from the contour plot imaging (enough contours have to be available to tell the small gradient near the apex of the cone).

What really matters in a truncated cone, then is A) whether the mode is transverse electric (TE) or transverse magnetic (TM) annd B) what are the number of wave patterns in the circumferential and radial directions.

I would advise to look at the contour plots on the Big Diameter, to establish whether the mode is TM or TE, and what is the number of wave patterns in the circumferential and radial directions at the Big Diameter location, and not to put too much emphasis on the number of wavepatterns in the longitudinal direction for a truncated cone.

Please take a gander at the field strengths towards the small diameter: do you see the very small magnitude gradient of the half-wave mode (deep blue compared to lighter blue) in the longitudinal direction near the small diameter?

Compare that to the much stronger gradient near the Big Diameter (from red to blue).
Title: Re: EM Drive Developments - related to space flight applications - Thread 2
Post by: Notsosureofit on 02/08/2015 09:40 pm
@ RODAL

Did you say you had an exact analytic solution for the truncated cavity ? 

["This is shown in the literature of exact solutions for the truncated cone. "]
Title: Re: EM Drive Developments - related to space flight applications - Thread 2
Post by: Rodal on 02/08/2015 09:45 pm
@ RODAL

Did you say you had an exact analytic solution for the truncated cavity ?
Yes, and I am working on an exact solution including the dielectric as well.

I obtained a closed-form solution for the integral of the wavenumber kz.  The expression contains square root terms and ArcTan terms.  Because of this one cannot invert the expression for the eigenvalue problem directly in terms of frequency and one has to solve the eigenvalue problem using a root-finding equation like Newton-Raphson.  (Similarly to what one has to do to solve eigenvalue problems in vibrations of beams and shells).

The inclusion of the dielectric is easy because the dielectric properties are constant.

Cutoff frequency conditions do not need to be added.  The cutoff is evident because real value solutions are only possible for high enough number of "p" (the longitudinal direction quantum number).

The solution produces evanescent modes (for complex values of the solution of the eigenvalue problem) and conventional resonant modes (TE or TM) for real solutions of the eigenvalue problem.

I have yet to compare my solution with the ones obtained by the Chinese (see: http://forum.nasaspaceflight.com/index.php?action=dlattach;topic=36313.0;attach=634645  ) but I have compared it to the cylindrical cavity and have established that the GeometricMean of the diameters plugged into the cylndrical cavity gives frequencies that are less than 1.5% from the truncated cone, for cone half-angle less than 20 degrees. 

Title: Re: EM Drive Developments - related to space flight applications - Thread 2
Post by: Notsosureofit on 02/08/2015 09:53 pm
@ RODAL

Can you put it in terms of a dispersion relation that can be evaluated at the ends ?
Title: Re: EM Drive Developments - related to space flight applications - Thread 2
Post by: Rodal on 02/08/2015 10:02 pm
@ RODAL

Can you put it in terms of a dispersion relation that can be evaluated at the ends ?

Concerning your question, the non-dimensional parameters in the solution are:


1) fmnp * Length / cmedium    (where cmedium is the speed of light in the medium, therefore this non-dimensional parameter is the ratio of the length to the wavelength)

2) either Xmn/Pi  or X' mn/Pi (for TM and TE modes respectively)

3) (Dbig - Dsmall)/Length (the tangent of the half angle of the cone)

4) Dsmall/Length (the ratio of the small diameter to the length)



My plan:

1) work out the expression including the dielectric
2) compute and compare to NASA experiment, post here the results and the expression
3) compare to Egan and to the Chinese

 
Title: Re: EM Drive Developments - related to space flight applications - Thread 2
Post by: Notsosureofit on 02/08/2015 10:06 pm
@  RODAL

That should keep you busy !  (but worth it)
Title: Re: EM Drive Developments - related to space flight applications - Thread 2
Post by: Rodal on 02/08/2015 10:11 pm
@  RODAL

That should keep you busy !  (but worth it)

Notice that my solution is in terms of Xmn and X'mn.  Rigorously speaking the solution should be in terms of spherical Bessel functions of the third kind.  It should not make much difference for truncated cones that are close enough to a cylinder, such that the ends are practically flat.  Basically it comes down again as to whether the truncated cone has flat ends or spherical ends.


NASA and Shawyer (initially) used flat ends instead of spherical ends.  Shawyer is using spherical ends for his superconducting version
Title: Re: EM Drive Developments - related to space flight applications - Thread 2
Post by: Notsosureofit on 02/08/2015 10:21 pm
@ RODAL

Do you have the dielectric constants (@ freq) and the dimensions ?

Title: Re: EM Drive Developments - related to space flight applications - Thread 2
Post by: Rodal on 02/08/2015 10:30 pm
@ RODAL

Do you have the dielectric constants (@ freq) and the dimensions ?

Here are the dimensions supplied by Paul March http://forum.nasaspaceflight.com/index.php?action=dlattach;topic=36313.0;attach=634621

Concerning the dielectric constant, thanks for reminding me to ask Paul, see my message below
Title: Re: EM Drive Developments - related to space flight applications - Thread 2
Post by: Rodal on 02/08/2015 10:35 pm
...
Best, Paul M.

Paul : did NASA (or an outside lab) measure with a dielectrometer the dielectric constant and the tan delta of the High Density PE you used for the dielectric  ?

Otherwise, do you have the relative electrical permittivity and the tan delta of the HD PE at a frequency of ~ 2 GHz at the temperatures experienced during the experiment, from some other source ?

If not, do you have the molecular weight of the HD PE used for the dielectric or some form of further characterization from the dielectric supplier ?

Thanks
Title: Re: EM Drive Developments - related to space flight applications - Thread 2
Post by: Mulletron on 02/08/2015 11:37 pm
Still going to have to test it on a ballistic pendulum. http://er.jsc.nasa.gov/seh/Sgoddard.htm
(http://www.enasco.com/prod/images/products/CA/AC046461.jpg)
Title: Re: EM Drive Developments - related to space flight applications - Thread 2
Post by: Mulletron on 02/09/2015 01:24 am
Just want to put this out there in case it helps. During all the research of literature concerning QV momentum, these two materials came up again and again. Cr2O3 and FeGaO3. Or if that is too high speed, PTFE or PVDF. To get over the 100uN hump, it might be time to dump the PE and try something else.

Title: Re: EM Drive Developments - related to space flight applications - Thread 2
Post by: Rodal on 02/09/2015 01:49 am
Just want to put this out there in case it helps. During all the research of literature concerning QV momentum, these two materials came up again and again. Cr2O3 and FeGaO3. Or if that is too high speed, PTFE or PVDF. To get over the 100uN hump, it might be time to dump the PE and try something else.
Or if that doesn't work...
Why should there be a target to achieve a given thrust with a single EMDrive?
Why not just achieve the required thrust at Glenn by using n drives to add up to the required thrust?
For 100uN one needs two EM Drives each providing 50uN, or three EM Drives each providing 33uN and so on
Title: Re: EM Drive Developments - related to space flight applications - Thread 2
Post by: Star-Drive on 02/09/2015 03:22 am
...
Best, Paul M.

Paul : did NASA (or an outside lab) measure with a dielectrometer the dielectric constant and the tan delta of the High Density PE you used for the dielectric  ?

Otherwise, do you have the relative electrical permittivity and the tan delta of the HD PE at a frequency of ~ 2 GHz at the temperatures experienced during the experiment, from some other source ?

If not, do you have the molecular weight of the HD PE used for the dielectric or some form of further characterization from the dielectric supplier ?

Thanks

What I used for the HDPE permittivity and loss tangent data was hand book based since we don't have the capability of accurately measuring these parameters in our lab.  So find attached the source for the HDPE we used and the dielectric constants used in our analysis.

BTW, the mass of the HDPE discs was around 486 grams each, which yields a density of 0.947 grams/cc.
Title: Re: EM Drive Developments - related to space flight applications - Thread 2
Post by: Star-Drive on 02/09/2015 03:43 am
Just want to put this out there in case it helps. During all the research of literature concerning QV momentum, these two materials came up again and again. Cr2O3 and FeGaO3. Or if that is too high speed, PTFE or PVDF. To get over the 100uN hump, it might be time to dump the PE and try something else.
Or if that doesn't work...
Why should there be a target to achieve a given thrust with a single EMDrive?
Why not just achieve the required thrust at Glenn by using n drives to add up to the required thrust?
For 100uN one needs two EM Drives each providing 50uN, or three EM Drives each providing 33uN and so on


"Why should there be a target to achieve a given thrust with a single EM-Drive? Why not just achieve the required thrust at Glenn by using n drives to add up to the required thrust?"

The month plus of hand labor total provided by three different home workshops required to make each one of these sheet metal based copper frustums.  And we don't have the funding to contract out the duplication of more copper test articles until we prove to our management that this approach is viable.

BTW, the copper frustum is vented, so its internal pressure matches the chamber pressure after a short time period at vacuum.

Best,  Paul M.
Title: Re: EM Drive Developments - related to space flight applications - Thread 2
Post by: NovaSilisko on 02/09/2015 03:47 am
So... I haven't really done a good long read of this thread in a huge amount of time, since before thread 1 reached a few pages long, even. Would it be possible for someone to produce a 1-2 paragraph summary of what's been going on here over the past few months? Are we... building our own EmDrive, now?
Title: Re: EM Drive Developments - related to space flight applications - Thread 2
Post by: Mulletron on 02/09/2015 04:18 am
Dang! Shawyer didn't provide you a sample!? +10 more respect points for that build. Fantastic job.
Title: Re: EM Drive Developments - related to space flight applications - Thread 2
Post by: sanman on 02/09/2015 05:27 am
Is copper the ideal material for the frustum? Or are there even better materials, theoretically?
Title: Re: EM Drive Developments - related to space flight applications - Thread 2
Post by: ThinkerX on 02/09/2015 05:58 am
Quote
So... I haven't really done a good long read of this thread in a huge amount of time, since before thread 1 reached a few pages long, even. Would it be possible for someone to produce a 1-2 paragraph summary of what's been going on here over the past few months? Are we... building our own EmDrive, now?

This is Doctor Rodal's summary as of page 20 or so of this thread.


Quote
1) We are exploring the validity of the claims of the different experimenters (NASA in the USA, Shawyer in the UK and Juan Yang in China) by systematically analyzing their experiments.

2) Some of the people in this forum are also designing and working to make their own prototypes and experiments.

3) One of the people in this forum (@NotSoSureOfIt), has made an outstanding contribution by independently deriving an equation that is not far from the claimed experimental results.

4) There are a number of possible physical reasons for the experimental results to be valid for space propulsion as well as for the results to be an experimental artifact that may not produce any propulsion in space.

5) There have been no further reports from NASA on the experiments that were supposed to be replicated at their other centers (JPL and Glenn) or at John Hopkins University.  Actually @wembley, who is an aerospace technology reporter, reports that it is his opinion that NASA has a "news blackout" on this matter, and China's Juan Yang is not saying much either.  The Chinese seem to be much further along than NASA, as they reported much greater thrust and they have conducted more thorough experiments (including being the first to numerically report the effects of temperature and temperature gradients with thermocouples embedded in the metal).  Shawyer in the UK seems to be much further ahead than NASA as he claims he is exploring a superconducting design.  Shawyer (UK) made a presentation in Canada late last year where he showed his latest design (using superconductivity) which he claims will result in much greater thrust/PowerInput (see image below).

The principle development since then is the return of 'Star Drive' who actually works with the Eaglework's team and was able to confirm this device does produce thrust in a hard vacuum.  (albeit at a reduced level).  Trying to figure out why this is so is more or less the main topic of discussion at the moment.
Title: Re: EM Drive Developments - related to space flight applications - Thread 2
Post by: francesco nicoli on 02/09/2015 01:56 pm
I forgot to thanks Rodal and the others for having taken the time to explain it to profanes like myself.

So:

Thanks! :)
Title: Re: EM Drive Developments - related to space flight applications - Thread 2
Post by: DIYFAN on 02/10/2015 03:11 am
Is copper the ideal material for the frustum? Or are there even better materials, theoretically?

I think maybe it was Mulletron that previously pointed out this paper:

https://hal.archives-ouvertes.fr/hal-01074608/document (https://hal.archives-ouvertes.fr/hal-01074608/document)

The conjecture in that paper is that building the larger diameter plate (R1) out of metglas 2714A, a significant amplification of the effect would result. Perhaps there is a courageous person or group with some funds to put this to the test.  It would be interesting to see an experiment with the partial metglas construction to rule the theory in or out.

Given that metglas 2714A is a room-temperature material, it would be considerably easier to achieve an amplification that way than lining the interior of the test article with superconducting film and cooling to liquid nitrogen temperatures. 

Title: Re: EM Drive Developments - related to space flight applications - Thread 2
Post by: Mulletron on 02/10/2015 03:53 am
Is copper the ideal material for the frustum? Or are there even better materials, theoretically?

I think maybe it was Mulletron that previously pointed out this paper:

https://hal.archives-ouvertes.fr/hal-01074608/document (https://hal.archives-ouvertes.fr/hal-01074608/document)

The conjecture in that paper is that building the larger diameter plate (R1) out of metglas 2714A, a significant amplification of the effect would result. Perhaps there is a courageous person or group with some funds to put this to the test.  It would be interesting to see an experiment with the partial metglas construction to rule the theory in or out.

Given that metglas 2714A is a room-temperature material, it would be considerably easier to achieve an amplification that way than lining the interior of the test article with superconducting film and cooling to liquid nitrogen temperatures.

As far as building the cavity itself, copper is good enough. It is easy to get and work with and has great electrical properties. Anything better would be precious metals which isn't worth the money. And superconductors aren't exactly "home workshop" compatible for most.

As far as something in place of the polyethylene dielectric, I'd suggest something that exhibits a higher relative permittivity or even a material that exhibits magnetochiral dichroism. http://www.academia.edu/1084905/Probing_magnetochirality
Title: Re: EM Drive Developments - related to space flight applications - Thread 2
Post by: DIYFAN on 02/10/2015 04:09 am
Is copper the ideal material for the frustum? Or are there even better materials, theoretically?

I think maybe it was Mulletron that previously pointed out this paper:

https://hal.archives-ouvertes.fr/hal-01074608/document (https://hal.archives-ouvertes.fr/hal-01074608/document)

The conjecture in that paper is that building the larger diameter plate (R1) out of metglas 2714A, a significant amplification of the effect would result. Perhaps there is a courageous person or group with some funds to put this to the test.  It would be interesting to see an experiment with the partial metglas construction to rule the theory in or out.

Given that metglas 2714A is a room-temperature material, it would be considerably easier to achieve an amplification that way than lining the interior of the test article with superconducting film and cooling to liquid nitrogen temperatures.

As far as building the cavity itself, copper is good enough. It is easy to get and work with and has great electrical properties. Anything better would be precious metals which isn't worth the money. And superconductors aren't exactly "home workshop" compatible for most.

As far as something in place of the polyethylene dielectric, I'd suggest something that exhibits a higher relative permittivity or even a material that exhibits magnetochiral dichroism. http://www.academia.edu/1084905/Probing_magnetochirality

Copper may very well be good enough for the cavity itself.  Or maybe not.  There is at least a theory that metglass would be better if used for one of the frustrum's plates (i.e., the larger plate).  Do you see a reason why someone shouldn't make an experiment along these lines?
Title: Re: EM Drive Developments - related to space flight applications - Thread 2
Post by: Mulletron on 02/10/2015 06:57 am

Copper may very well be good enough for the cavity itself.  Or maybe not.  There is at least a theory that metglass would be better if used for one of the frustrum's plates (i.e., the larger plate).  Do you see a reason why someone shouldn't make an experiment along these lines?

There is no reason not to try Metglas 2714A, especially since it is supported by a paper written by a Professor Emeritus of Physics who makes sweeping claims like this:
Quote
If the microwave source radiates pulses with 10 megawatts power then the thrust can
reach up to 100kN.
.
He could be right for all we know.

Given that NO theory of operation has been proven correct, any reasonable suggestion is equally plausible. Experimentation might nail down the reason behind all this "anomalous thrust." Thrust which is way in excess of a photon rocket. That should be sending alarm bells throughout the scientific community. Instead the attitude we get from the likes of Sean Carroll and John Baez is dismissal and snarky one liners.* Obviously they have all the science all figured out already. Apparently there is no need to challenge our assumptions these days. The hardest inertia to overcome is our own. Scientists should be stumbling over each other trying to figure this Emdrive out because after all the replications, it seems likely there could be a Nobel awaiting someone if there is any new physics involved or at least a better understanding of current physics. At the very least, they could empirically put the smack down on some pseudoscience.

Quoting @GoatGuy over at NBF:
Quote
Any time the K/Ko exceeds 1.0 … is the point where almost all physicists will pause, turn their heads and say, what? Yep. The K/Ko in the lowest wattage, highest Q experiment was not just a little bit greater than 1.0, but was an almost implausible 6,388× larger.

If this were SETI, we'd call that the “Wow! signal”

* http://en.m.wikipedia.org/wiki/Quantum_vacuum_plasma_thruster
https://plus.google.com/app/basic/stream/z13gzfm4xt2tuxehl04chvywcofjhhzhwbk
http://blogs.discovermagazine.com/outthere/2014/08/06/nasa-validate-imposible-space-drive-word/#.VNoM1MO3M0N
Title: Re: EM Drive Developments - related to space flight applications - Thread 2
Post by: Rodal on 02/10/2015 02:13 pm
Is copper the ideal material for the frustum? Or are there even better materials, theoretically?

I think maybe it was Mulletron that previously pointed out this paper:

https://hal.archives-ouvertes.fr/hal-01074608/document (https://hal.archives-ouvertes.fr/hal-01074608/document)

The conjecture in that paper is that building the larger diameter plate (R1) out of metglas 2714A, a significant amplification of the effect would result. Perhaps there is a courageous person or group with some funds to put this to the test.  It would be interesting to see an experiment with the partial metglas construction to rule the theory in or out.

Given that metglas 2714A is a room-temperature material, it would be considerably easier to achieve an amplification that way than lining the interior of the test article with superconducting film and cooling to liquid nitrogen temperatures.
Interesting conjecture.  Did the author of the paper advocating the use of Metglas 2714A take into account that its ultra-high magnetic permeability which he quotes in the paper (apparently for his calculations) as μ=1,000,000 occurs at very low frequencies and that its magnetic permeability decreases significantly at higher frequencies ? 

See the attached plot, unfortunately the available data ( http://www.metglas.com/assets/pdf/2714a.pdf ) goes up to only 10^5 Hz while the EM Drive operates at 20000 times higher frequency: 2*10^9 Hz.  At the much lower frequency of 10^5 Hz the magnetic permeability has decreased by a factor of 50 from the maximum DC permeability of μ =1,000,000 to μ =20,000. If it decreases by another factor of 50 (it will probably decrease more due to the steepening shape of the curve ?) the magnetic permeability at 2*10^9 Hz may be μ ~ 400 instead of 1,000,000.

(Optimistically ?) assuming that  μ ~ 400 for Metglas 2714A at 2*10^9 Hz, then the author's prediction of 1,000 times higher "thrust" (?) than Shawyer’s thruster would be reduced by Sqrt[(10^6)/400]=50 times, so that the increase in thrust would be 20 times over Shawyer's experiments (assuming that the author's conjecture and theory are correct, and assuming that the EM Drive can be (practically) tuned to stay in Q resonance at a suitable mode-shape with such a material).

Still, a factor of 20 times higher force is nothing to ignore, if the author's theory is correct, and if the magnetic permeability is at least μ ~ 400 for Metglas 2714A at 2*10^9 Hz, this material may enable to exceed the minimum requirements for testing the Eagleworks frustum at NASA's Glenn.
Title: Re: EM Drive Developments - related to space flight applications - Thread 2
Post by: Rodal on 02/10/2015 03:55 pm
Is copper the ideal material for the frustum? Or are there even better materials, theoretically?

I think maybe it was Mulletron that previously pointed out this paper:

https://hal.archives-ouvertes.fr/hal-01074608/document (https://hal.archives-ouvertes.fr/hal-01074608/document)

The conjecture in that paper is that building the larger diameter plate (R1) out of metglas 2714A, a significant amplification of the effect would result. Perhaps there is a courageous person or group with some funds to put this to the test.  It would be interesting to see an experiment with the partial metglas construction to rule the theory in or out.

Given that metglas 2714A is a room-temperature material, it would be considerably easier to achieve an amplification that way than lining the interior of the test article with superconducting film and cooling to liquid nitrogen temperatures.
Interesting conjecture.  Did the author of the paper advocating the use of Metglas 2714A take into account that its ultra-high magnetic permeability which he quotes in the paper (apparently for his calculations) as μ=1,000,000 occurs at very low frequencies and that its magnetic permeability decreases significantly at higher frequencies ? 

See the attached plot, unfortunately the available data ( http://www.metglas.com/assets/pdf/2714a.pdf ) goes up to only 10^5 Hz while the EM Drive operates at 20000 times higher frequency: 2*10^9 Hz.  At the much lower frequency of 10^5 Hz the magnetic permeability has decreased by a factor of 50 from the maximum DC permeability of μ =1,000,000 to μ =20,000. If it decreases by another factor of 50 (it will probably decrease more due to the steepening shape of the curve ?) the magnetic permeability at 2*10^9 Hz may be μ ~ 400 instead of 1,000,000.

(Optimistically ?) assuming that  μ ~ 400 for Metglas 2714A at 2*10^9 Hz, then the author's prediction of 1,000 times higher "thrust" (?) than Shawyer’s thruster would be reduced by Sqrt[(10^6)/400]=50 times, so that the increase in thrust would be 20 times over Shawyer's experiments (assuming that the author's conjecture and theory are correct, and assuming that the EM Drive can be (practically) tuned to stay in Q resonance at a suitable mode-shape with such a material).

Still, a factor of 20 times higher force is nothing to ignore, if the author's theory is correct, and if the magnetic permeability is at least μ ~ 400 for Metglas 2714A at 2*10^9 Hz, this material may enable to exceed the minimum requirements for testing the Eagleworks frustum at NASA's Glenn.

This paper (http://infoscience.epfl.ch/record/55950/files/76.pdf ) has data up to 30 MHz (that is 67 times lower than the frequency of operation of the EM Drive, hence much closer to the EM Drive's frequency than the data from the manufacturer at 100KHz, considering that the drop of magnetic permeability depends on the Log of frequency, the EM Drive Log frequency is just 1.8 times higher) showing a magnetic relative permeability of  μ =6000 at 30 MHz, indicating about  μ =2400 at 2*10^9 Hz

If the author's conjecture and theory are correct, this extrapolated value ( μ =2400 at 2*10^9 Hz) would result in an improvement of 1000/Sqrt[(10^6)/2400] = 1000/20.4 = 49 times greater thrust force than Shawyer's experiments.

This information, coupled with the one in the previous post, indicates  that that (if Aquino's theory is correct) using amorphous metal (http://en.wikipedia.org/wiki/Amorphous_metal) Metglas 2714A sheet ( http://en.wikipedia.org/wiki/Metglas ) may result in 20 to 50 times greater thrust force than using copper sheet.

(http://www.metglas.com/assets/images/slider4.jpg)
Title: Re: EM Drive Developments - related to space flight applications - Thread 2
Post by: Rodal on 02/10/2015 05:27 pm
I still favor direct application of the Equivalence Principle ...night all
@Notsosureofit

Concerning <<I still favor direct application of the Equivalence Principle >> I would appreciate your comments/observations concerning Univ. of Massachusetts, John F. Donoghue, Barry R. Holstein, R. W. Robinett's 1985 (awarded ?) paper

"The principle of equivalence at finite temperature"   (http://www.gravityresearchfoundation.org/pdf/awarded/1984/donoghue_holstein_robinett.pdf)

citation history here:
http://inspirehep.net/record/14777/citations

where they claim to demonstrate that the equivalence principle is violated by radiative corrections to the gravitational and inertial masses at finite temperature. (How about Unruh radiation as per Dr. McCulloch's theory ?).

They argue that this result can be attributed to the Lorentz noninvariance of the finite temperature vacuum.

Max Jammer refers to Donoghue's paper in an authoritative manner with very interesting discussion in pages 138 to 140 of Jammer's classic book (in the Princeton University 2000 edition, not the Dover paperback reprint of the 1961 edition  !!) "Concepts of Mass in Contemporary Physics and Philosophy" (see: this link  http://bit.ly/1y8vaCO )

Title: Re: EM Drive Developments - related to space flight applications - Thread 2
Post by: DIYFAN on 02/10/2015 05:28 pm
(Optimistically ?) assuming that  μ ~ 400 for Metglas 2714A at 2*10^9 Hz, then the author's prediction of 1,000 times higher "thrust" (?) than Shawyer’s thruster would be reduced by Sqrt[(10^6)/400]=50 times, so that the increase in thrust would be 20 times over Shawyer's experiments (assuming that the author's conjecture and theory are correct, and assuming that the EM Drive can be (practically) tuned to stay in Q resonance at a suitable mode-shape with such a material).

Still, a factor of 20 times higher force is nothing to ignore, if the author's theory is correct, and if the magnetic permeability is at least μ ~ 400 for Metglas 2714A at 2*10^9 Hz, this material may enable to exceed the minimum requirements for testing the Eagleworks frustum at NASA's Glenn.

This paper (http://infoscience.epfl.ch/record/55950/files/76.pdf ) has data up to 30 MHz (that is 67 times lower than the frequency of operation of the EM Drive, hence much closer to the EM Drive's frequency than the data from the manufacturer at 100KHz, considering that the drop of magnetic permeability depends on the Log of frequency, the EM Drive Log frequency is just 1.8 times higher) showing a magnetic relative permeability of  μ =6000 at 30 MHz, indicating about  μ =2400 at 2*10^9 Hz

If the author's conjecture and theory are correct, this extrapolated value ( μ =2400 at 2*10^9 Hz) would result in an improvement of 1000/Sqrt[(10^6)/2400] = 1000/20.4 = 49 times greater thrust force than Shawyer's experiments.

This information, coupled with the one in the previous post, indicates  that that (if Aquino's theory is correct) using amorphous metal (http://en.wikipedia.org/wiki/Amorphous_metal) Metglas 2714A sheet ( http://en.wikipedia.org/wiki/Metglas ) may result in 20 to 50 times greater thrust force than using copper sheet.

(http://www.metglas.com/assets/images/slider4.jpg)

Thank you for distilling this information down and also for noticing and highlighting the frequency-dependent drop in magnetic permeability.  This aspect must have been overlooked by Aquino.  I agree that despite the precipitous drop, it may still very well be interesting and possibly superior to copper.  Since Dr. Aquino's suggestion is to use the Metglas 2714A sheet with only the larger diameter plate (i.e., the "R1" plate), it seems that those who have already constructed a copper cavity could simply cut a Metglas 2714A sheet to fit the inner dimensions of the R1 plate, and then form-fit it to the interior of the already constructed cavity.  This wouldn't require a significant rebuild of the apparatus.     
Title: Re: EM Drive Developments - related to space flight applications - Thread 2
Post by: Rodal on 02/10/2015 06:01 pm
I still favor direct application of the Equivalence Principle ...night all
@Notsosureofit

Concerning <<I still favor direct application of the Equivalence Principle >> I would appreciate your comments/observations concerning Univ. of Massachusetts, John F. Donoghue, Barry R. Holstein, R. W. Robinett's 1985 (awarded ?) paper

"The principle of equivalence at finite temperature"   (http://www.gravityresearchfoundation.org/pdf/awarded/1984/donoghue_holstein_robinett.pdf)

citation history here:
http://inspirehep.net/record/14777/citations

where they claim to demonstrate that the equivalence principle is violated by radiative corrections to the gravitational and inertial masses at finite temperature. (How about Unruh radiation as per Dr. McCulloch's theory ?).

They argue that this result can be attributed to the Lorentz noninvariance of the finite temperature vacuum.

Max Jammer refers to Donoghue's paper in an authoritative manner with very interesting discussion in pages 138 to 140 of Jammer's classic book "Concepts of Mass in Contemporary Physics and Philosophy" (see: this link  http://bit.ly/1y8vaCO )

Quote from:  John F. Donoghue, Barry R. Holstein, R. W. Robinett
The fundamental ideas which led to the equivalence principle include the impossibility of defining absolute motion through the vacuum and the indistinguishability of acceleration and gravitational force. However, one can measure absolute velocity and acceleration relative to the heat bath (as has been done for the velocity of the Earth in the 3°K photon distribution left over from the early universe). Thus the conditions under which the equivalence principle was formulated are not met at finite temperature. The fact that we do live in a universe at a nonzero ternperature could in principle have led to unexpected results in the Eotvos experiments if it were not for the fact that the correction is too small to be detected at present temperatures.

Title: Re: EM Drive Developments - related to space flight applications - Thread 2
Post by: Notsosureofit on 02/10/2015 09:00 pm
@ RODAL

My understanding was that the Doppler shifted radiation of the cosmic background would constitute an increasing drag (and thus a limit) on the accelerated system, (as Unruh) not that it invalidated the equivalence principle.  But, I'll look and see what they say.
Title: Re: EM Drive Developments - related to space flight applications - Thread 2
Post by: Rodal on 02/10/2015 09:08 pm
@ RODAL

My understanding was that the Doppler shifted radiation of the cosmic background would constitute an increasing drag (and thus a limit) on the accelerated system, (as Unruh) not that it invalidated the equivalence principle.  But, I'll look and see what they say.
Yes, you are entirely correct.  Sorry if my poor wording conveyed that impression.   I brought up Donoghue's paper because it may give yet another possibility: escaping the equivalence principle  (but the effect appears to be very small )
Title: Re: EM Drive Developments - related to space flight applications - Thread 2
Post by: Notsosureofit on 02/10/2015 09:29 pm
@ RODAL

My understanding was that the Doppler shifted radiation of the cosmic background would constitute an increasing drag (and thus a limit) on the accelerated system, (as Unruh) not that it invalidated the equivalence principle.  But, I'll look and see what they say.
Yes, you are entirely correct.  Sorry if my poor wording conveyed that impression.   I brought up Donoghoue's paper because it may give yet another possibility: escaping the equivalence principle  (but the effect appears to be very small )

A quick read but you can probably get their results by using the photon bath as a viscous fluid exchanging energy at a given temperature.   Nice quantum argument.  In any event, the cosmic background does not constitute a temperature "bath" under velocity (or acceleration) because of the vector form (directionality) of the velocity.  Have to look at the citations..
Title: Re: EM Drive Developments - related to space flight applications - Thread 2
Post by: Rodal on 02/10/2015 10:18 pm
@ RODAL

My understanding was that the Doppler shifted radiation of the cosmic background would constitute an increasing drag (and thus a limit) on the accelerated system, (as Unruh) not that it invalidated the equivalence principle.  But, I'll look and see what they say.
Yes, you are entirely correct.  Sorry if my poor wording conveyed that impression.   I brought up Donoghoue's paper because it may give yet another possibility: escaping the equivalence principle  (but the effect appears to be very small )

A quick read but you can probably get their results by using the photon bath as a viscous fluid exchanging energy at a given temperature.   Nice quantum argument.  In any event, the cosmic background does not constitute a temperature "bath" under velocity (or acceleration) because of the vector form (directionality) of the velocity.  Have to look at the citations..

" the cosmic background does not constitute a temperature "bath" under velocity (or acceleration) because of the vector form (directionality) of the velocity"  Good point  :)
Title: Re: EM Drive Developments - related to space flight applications - Thread 2
Post by: Mulletron on 02/10/2015 11:06 pm
While trying to figure out the answer to my question here: http://forum.nasaspaceflight.com/index.php?topic=36313.msg1327384#msg1327384
Quote
It appears that TM modes are the top dogs now and at the same time performance has gone down significantly since vacuum testing began.
The answer to the bold part in particular. I don't think the answer is fully due to the elimination of air currents, as the thrust signature rise and fall times were instantaneous during the non vacuum tests reported last summer.

Also armed with the knowledge from @Star-Drive that the the copper frustum is vented.
Quote
BTW, the copper frustum is vented, so its internal pressure matches the chamber pressure after a short time period at vacuum.

I accidentally stumbled upon this report:
https://hal.archives-ouvertes.fr/hal-00551421v1/document
Quote
We report the first observation of the anisotropy of the velocity of light, induced in a gas by electric
and magnetic fields.....Using a high finesse ring cavity, we have
measured the magneto-electric directional anisotropy of molecular Nitrogen at ambient temperature
and atmospheric pressure
This alone is an amazing result. Note that it is an observation.
They predict the following:
Quote
Moreover, the same effect is expected in quantum vacuum......Our long term goal is to search for
the magneto-electric directional anisotropy of quantum.
The QV prediction would be huge if confirmed.

I'm gathering facts still and I welcome your comments, but does this shed light on why the thrust signature is different between air filled and vacuum filled resonant cavities? This difference in measured thrust and the above experimental observations, may shed light on why Shawyer's devices appear to work without dielectric and also may lend support for the transfer of momentum from the QV approach which we discussed at length.

I'm still trying to find which reference it was where any of this implied non-reciprocity.
Title: Re: EM Drive Developments - related to space flight applications - Thread 2
Post by: Rodal on 02/10/2015 11:42 pm
..I don't think the answer is fully due to the elimination of air currents, as the thrust signature rise and fall times were instantaneous during the non vacuum tests reported last summer. ...
That "instantaneous" length of time was long enough, that it could be fully explained as a thermal instability, as I showed in my report.  Thermal instability that does not need thermal convection as a transport mechanism.  The IR experimental measurements were done without air inside or outside the EM Drive, and the COMSOL analysis showing heating of the big diameter surface was modeled without air inside or outside the EM Drive, is that right?

(http://forum.nasaspaceflight.com/index.php?action=dlattach;topic=36313.0;attach=635194;image)
Title: Re: EM Drive Developments - related to space flight applications - Thread 2
Post by: Rodal on 02/10/2015 11:49 pm
...

I accidentally stumbled upon this report:
https://hal.archives-ouvertes.fr/hal-00551421v1/document
Quote
We report the first observation of the anisotropy of the velocity of light, induced in a gas by electric
and magnetic fields.....Using a high finesse ring cavity, we have
measured the magneto-electric directional anisotropy of molecular Nitrogen at ambient temperature
and atmospheric pressure
This alone is an amazing result. Note that it is an observation.
...

Very interesting.  What determines the anisotropy directions?  If the gas molecules are constantly bouncing around, I would have expected that their random motion would result in isotropy of the continuum of the gas molecules.  Are these experiments done at such a low temperature that the gas molecules are practically "frozen" ? Apparently not, since the paper states T = 300 K ?

Is it that a single Nitrogen gas molecule has no time to move during the experiment? and that the anisotropy being measured is the anisotropy of a single gas molecule (not the anisotropy of a large collection of gas molecules randomly oriented from each other)
Title: Re: EM Drive Developments - related to space flight applications - Thread 2
Post by: Mulletron on 02/10/2015 11:58 pm
As far as heat goes, Eagleworks repeatedly reported that heat was controlled for and wasn't an issue. So I'm forced to trust the experiment was legit. Vacuum eliminates hot air currents but not radiation or expansion.

A simple falsification of all this transferring momentum from the qv business is simply to evacuate all the air from the resonant cavity and remove the dielectric and see if the thrust goes away.
Title: Re: EM Drive Developments - related to space flight applications - Thread 2
Post by: Rodal on 02/11/2015 12:02 am
As far as heat goes, Eagleworks repeatedly reported that heat was controlled for and wasn't an issue. So I'm forced to trust the experiment was legit. Vacuum eliminates hot air currents but not radiation.


Yes, "Vacuum eliminates hot air currents but not radiation," for example, a magnetic field can heat copper by induction heating. Thermal instability is still a possibility, is it not?

A simple falsification of all this transferring momentum from the qv business is simply to evacuate all the air from the resonant cavity and remove the dielectric and see if the thrust goes away.

As I showed in my paper on Thermal Instability, the dielectric HD PE is acting as an effective shield, that shields the small diameter surface from being heated by the electromagnetic field.

If you remove the  dielectric HD PE , the small diameter surface is also going to get heated, instead of just the big diameter.  Also you would be removing the thermal expansion effect (the thermal expansion of HD PE is much higher than the one of the metal).  Hence if the thrust force is decreased  or eliminated it may still be due to thermal effects.

An analysis of these effects is necessary...
Title: Re: EM Drive Developments - related to space flight applications - Thread 2
Post by: Mulletron on 02/11/2015 12:09 am
Is 50 watts worth of ir radiation enough to mimic the thrust signal? You established that the measured thrust is in excess of a photon rocket. So all that is left is the thermal instability you mentioned. Hope Paul comments on that.
Title: Re: EM Drive Developments - related to space flight applications - Thread 2
Post by: Rodal on 02/11/2015 12:17 am
Is 50 watts worth of ir radiation enough to mimic the thrust signal? You established that the measured thrust is in excess of a photon rocket. So all that is left is the thermal instability you mentioned. Hope Paul comments on that.

I wish that NASA Glenn would pay, now, to have several EM Drives tested in their unit, to be able (in principle) to generate enough thrust force to get a reading at Glenn with all the drives together being pointed in the same direction.  My reading of what Paul March wrote is that this would mean at least 3 EM Drives.  With three (or more) EM Drives one could also conduct several interesting tests with orientation of each drive: all of them oriented in parallel, oriented in series (is there any measurable wake ?), oriented opposite to each other, etc.

Edison conducted >10,000 experiments for the incandescent light bulb, thousands of tests for the alkaline battery, etc.
Title: Re: EM Drive Developments - related to space flight applications - Thread 2
Post by: RotoSequence on 02/11/2015 02:28 am
Could the thermal thrust theory be tested by insulating the device with some sort of thermal blanket, such as Mylar?
Title: Re: EM Drive Developments - related to space flight applications - Thread 2
Post by: zen-in on 02/11/2015 03:56 am
Is 50 watts worth of ir radiation enough to mimic the thrust signal? You established that the measured thrust is in excess of a photon rocket. So all that is left is the thermal instability you mentioned. Hope Paul comments on that.

I wish that NASA Glenn would pay, now, to have several EM Drives tested in their unit, to be able (in principle) to generate enough thrust force to get a reading at Glenn with all the drives together being pointed in the same direction.  My reading of what Paul March wrote is that this would mean at least 3 EM Drives.  With three (or more) EM Drives one could also conduct several interesting tests with orientation of each drive: all of them oriented in parallel, oriented in series (is there any measurable wake ?), oriented opposite to each other, etc.

Edison conducted >10,000 experiments for the incandescent light bulb, thousands of tests for the alkaline battery, etc.

From a space flight application perspective what is the TRL of the EM Drive?

Here is the list:

TRL 1.    Basic principles observed and reported
TRL 2.    Technology concept and/or application formulated
TRL 3.    Analytical & experimental critical function and/or characteristic proof-of-concept
TRL 4.    Component and/or breadboard validation in laboratory environment
TRL 5.    Component and/or breadboard validation in relevant environment
TRL 6.    System/subsystem model or prototype demonstration in a relevant environment (ground or space)
TRL 7.    System prototype demonstration in a space environment
TRL 8.    Actual system completed and "Flight qualified" through test and demonstration (ground or space)
TRL 9.    Actual system "Flight proven" through successful mission operations

I haven't seen enough experimental data to be able to say what TRL level the EM-Drive would have.   The EM tether has been deployed in orbit so therefore it must have a TRL level of at least 7. 

Edison started with a theory of how an electric light could be built that many thought was impossible.   It was thought to be impossible not because the physics was believed to be wrong but because so many others had tried and failed.
Title: Re: EM Drive Developments - related to space flight applications - Thread 2
Post by: Rodal on 02/11/2015 11:44 am
Could the thermal thrust theory be tested by insulating the device with some sort of thermal blanket, such as Mylar?
Thermal instability results from the electromagnetic fields heating the interior surfaces of the (big diameter) copper surface.  Since the small diameter surface is shielded by the HD PE dielectric polymer, what gets heated is the big diameter interior surface (and, by thermal conduction, the exterior surface as well, as shown in the IR measurement, attached below).  I understand that the IR measurement was done from the outside, with the IR camera looking at the composite polymer surface of the circuit board surface they had on the exterior of the big diameter flat end.  Since this composite polymer has much lower thermal conductivity and much lower thermal diffusivity than copper, please take into account that these IR measurements represent a temperature and temperature gradients significantly lower than those present on the inner (copper) surface of the big diameter flat end.  In other words, the composite polymer circuit board surface being measured with the IR camera acts like an insulating surface concealing the higher temperature of the inner copper surface.


In my report I proposed that one way to eliminate thermal instabiltity of thin copper sheets is to use copper thick enough to eliminate a thermal instability. 

Quoting my report:

Quote from: Dr. J. Rodal
I have shown that a thermo-mechanical effect (thermal buckling of the base of the truncated cone) can account for some of the "anomalous" results reported by NASA's Brady et.al.  I have shown that the buckling time is under 1 second for copper thicknesses under 0.84 mm (33 thousands of an inch) and just 2.6 watt power input.  I have shown that the buckling temperature increase required is of the order of 1 deg C or less.  I have shown that thermal buckling can produce a sudden output response.

I have shown that the calculated buckling forces agree with the measured force (55.4 microNewtons).  The buckling force is a very strong function of plate thickness (to the fourth power), to prevent thermal buckling from occurring it suffices to have a thicker copper sheet (1/8 inch or thicker would completely prevent this thermal buckling under these input powers).

This thermal buckling effect does not depend at all on air as a conducting medium; it will take place in a complete vacuum as well
(Bold added for emphasis)

Now that NASA is using a higher input power (50 watts) than in the Brady et.al. report, it appears that using a 1/4  inch thick (0.25 inches) copper plate for flat ends would prevent this thermal instability, and hence eliminate this artifact from consideration.

(http://forum.nasaspaceflight.com/index.php?action=dlattach;topic=36313.0;attach=635194;image)
Title: Re: EM Drive Developments - related to space flight applications - Thread 2
Post by: Star-Drive on 02/11/2015 12:44 pm
Could the thermal thrust theory be tested by insulating the device with some sort of thermal blanket, such as Mylar?
Thermal instability results from the electromagnetic fields heating the interior surfaces of the (big diameter) copper surface.  Since the small diameter surface is shielded by the HD PE dielectric polymer, what gets heated is the big diameter interior surface (and, by thermal conduction, the exterior surface as well, as shown in the IR measurement, attached below).  I understand that the IR measurement was done from the outside, with the IR camera looking at the composite polymer surface of the circuit board surface they had on the exterior of the big diameter surface.  Since this composite polymer has much lower thermal conductivity and much lower thermal diffusivity than copper, please take into account that these IR measurements represent a temperature and temperature gradients significantly lower than those present on the inner (copper) surface of the big diameter flat end.


In my report I proposed that one way to eliminate thermal instabiltity of thin copper sheets is to use copper thick enough to eliminate a thermal instability. 

Quoting my report:

Quote from: Dr. J. Rodal
I have shown that a thermo-mechanical effect (thermal buckling of the base of the truncated cone) can account for some of the "anomalous" results reported by NASA's Brady et.al.  I have shown that the buckling time is under 1 second for copper thicknesses under 0.84 mm (33 thousands of an inch) and just 2.6 watt power input.  I have shown that the buckling temperature increase required is of the order of 1 deg C or less.  I have shown that thermal buckling can produce a sudden output response.

I have shown that the calculated buckling forces agree with the measured force (55.4 microNewtons).  The buckling force is a very strong function of plate thickness (to the fourth power), to prevent thermal buckling from occurring it suffices to have a thicker copper sheet (1/8 inch or thicker would completely prevent this thermal buckling under these input powers).

This thermal buckling effect does not depend at all on air as a conducting medium; it will take place in a complete vacuum as well
(Bold added for emphasis)

Now that NASA is using a higher input power (50 watts) than in the Brady et.al. report, it appears that using a 1/4  inch thick (0.25 inches) copper plate for flat ends would prevent this thermal instability, and hence eliminate this artifact from consideration.

(http://forum.nasaspaceflight.com/index.php?action=dlattach;topic=36313.0;attach=635194;image)

Please note that what was used for the both the large and small endplates in the Eagleworks copper frustum was 0.063 inch thick FR4 printed circuit board with 1.0 oz copper, (~35 microns thick of Cu epoxied to the FR4 fiberglass), facing the interior of the cavity.

Question: How long does this copper buckling effect last for a 50W run before the frustum test article relaxes back to the torque pendulum's at rest position?  Just as a note, we've already tried re-enforcing the frustum endplates with angle aluminum mounted on their outside surfaces and we didn't notice any marked change in its thrust response.

Next item is back to thermal expansion issues.  Granted that the PE has ~10X the thermal expansion per degree C than copper does, but the PE only seeing ~0.1C rise in part of its volume during a 50W run, whereas the copper frustum endures up to a ~15C rise over its 9.0 inch length, so the copper expansion still dominates.

Next, measureable thrust was not observed when the PE or Teflon discs were removed from the copper frustum while in air with up to 30W of RF using our Mini-Circuit RF amp.  As to why the vacuum test were observing less thrust than in air tests. please note the difference in the RF amps there were driving each test series.  The 30W Mini-Circuit Class-A RF amp was used for the in-air series reported in the 2014 JPC paper, whereas a 100W EMPower Class-A/B RF amplifier was used in the vacuum tests to date.   So how could a less powerful RF amp produce more thrust than a more powerful one?  Now think about what the driven magnitude of the time rate of change of the energy in the vacuum state might bring to this question...

Got to run...

Best, Paul M.
Title: Re: EM Drive Developments - related to space flight applications - Thread 2
Post by: Rodal on 02/11/2015 12:53 pm
...  Just as a note, we've already tried re-enforcing the frustum endplates with angle aluminum mounted on their outside surfaces and we didn't notice any marked change in its thrust response.
...
This was expected not to make any significant difference.

As I wrote in my report:

Quote from: Dr. J. Rodal
Cotterell and Parkes (based on Cotterell's Ph.D. thesis at the University of Cambridge) correctly point out that the distribution of the heat flux "is not significant in the problem" of thermal buckling of a circular plate, whether the heating takes place uniformly over the whole circular plate or is concentrated in a central region.  Cotterell chose a distribution with a heatedDiameterRatio =1/0.3=3.333 instead of the heatedDiameterRatio=1 analyzed by Noda et.al.  The fact that the exact distribution is not significant for the deltaT that will produce buckling or for the buckling displacement follows from equilibrium: the membrane stress (=E*alpha*deltaT) force resultant (the integral of the membrane stress through the thickness) is reacted at the simply supported edges (that constrain the in-plane displacement).  The membrane force resultant is uniform and it is equal in the polar radial and angular (azimuthal) directions.  If only a central area is heated, the membrane stress is still equilibrated throughout.  If the plate has uniform thickness and isotropic material properties, the strain in the non heated area prior to buckling is the same as in the heated area.

See:

Cotterell, B., and Parkes, E. W.,  Thermal Buckling of Circular Plates, (United Kingdom's) Aeronautical Research Council, Ministry Of Aviation, Reports and Memoranda No. 3245, September, 1960



QUESTION: Why did you use a glass-fiber-reinforced polymer printed circuit board as the end plate ? << 0.063 inch thick FR4 printed circuit board with 1.0 oz copper, (~35 microns thick of Cu epoxied to the FR4 fiberglass)>>

1) The IR measurement was done from the outside, with the IR camera looking at the composite polymer surface of the circuit board surface they had on the exterior of the big diameter flat end.  Since this composite polymer has much lower thermal conductivity and much lower thermal diffusivity than copper, please take into account that these IR measurements represent a temperature and temperature gradients significantly lower than those present on the inner (copper) surface of the big diameter flat end.  In other words, the composite polymer circuit board surface being measured with the IR camera acts like an insulating surface concealing the higher temperature of the inner copper surface. Moreover, due to very low thermal diffusivity of the  glass-fiber-reinforced polymer printed circuit board, measurement of its exterior surface presents a considerable time delay of the interior temperature vs. time profile (as it takes time for the heat to conduct through the thickness of the very low diffusivity  of the  glass-fiber-reinforced polymer printed circuit board).


2) The modulus of elasticity of the glass-fiber-reinforced polymer printed circuit board is much lower than the modulus of elasticity of the copper.  The glass-fiber-reinforced polymer printed circuit board has orders of magnitude lower thermal conductivity and thermal diffusivity than the copper. (Comparison noted below).

3) Why not get rid of the fiber-reinforced-polymer printed circuit board and just simply use a 1/4  inch thick (0.25 inches) copper plate for flat ends to prevent this thermal instability, and hence eliminate this artifact from consideration ?

As to your questions, I would need some time to give them the analytical consideration they deserve and to calculate, rather than give you an impulsive, reflexive, answer that may be incorrect.



NOTE:  FR-4 is a composite material made with woven fiberglass cloth embedded in an epoxy resin (polymer) matrix.  The in-plane Young's modulus of FR4 is 3.0×10^6 psi , about six times smaller than Copper's Young modulus of 17.0×10^6 psi.  The modulus of elasticity in the thickness direction is much lower, practically as low as the modulus of elasticity of epoxy.    FR4's coefficient of thermal expansion - x-axis   1.4×10^(−5) 1/K, Coefficient of thermal expansion - z-axis   7.0×10^(−5) 1/K

The thermal conductivity is a tiny  0.29 W/m·K in the thickness direction, due to the low thermal conductivity of the epoxy resin.  Copper has a thermal conductivity of 401  W/m·K, that is 1400 times higher than the thermal conductivity of FR4
Title: Re: EM Drive Developments - related to space flight applications - Thread 2
Post by: Rodal on 02/11/2015 02:32 pm
...measureable thrust was not observed when the PE or Teflon discs were removed from the copper frustum while in air with up to 30W of RF using our Mini-Circuit RF amp. ...

1) This was done at a significantly higher frequency than the tests that showed a force measurement (done at 1880.4 MHz to 1936.7 MHz).  Therefore it is not a rigorously valid comparison, particularly since the test done at the frequency of 1936.7 showed a significantly lower performance (Force/PowerInput) than the test done at 1880.4 MHz , and we have no IR imaging of the ends for these tests.

Frequency( MHz)       Thrust (uN)   Power (W)   Thrust/PowerInput
1880.4                     55.4                 2.6         21
1936.7                     50.1               16.7           3
2168                         0 (?)              30              0

1a) It should also have been done at the same frequency.

1b) IR camera temperature imaging of the flat end should have been done for the above cases to ascertain exactly what mode shape took place in each case, in order to better understand the very significant decrease in Thrust/PowerInput from 21 @ 1880.4 MHz , to 3 @1936.7 MHz, to 0 @2168 MHz , and the reported difficulties in reproducing the test at 1880.4 MHz.


Quote from: page 18 of http://www.libertariannews.org/wp-content/uploads/2014/07/AnomalousThrustProductionFromanRFTestDevice-BradyEtAl.pdf
We performed some very early evaluations without the dielectric resonator (TE012 mode at 2168 MHz, with power levels up to ~30 watts) and measured no significant net thrust.

2) Even if done at the same frequency, removing the dielectric polymer and observing much less or no force thrust is not a conclusive way to eliminate thermal instability because removing the dielectric polymer removes its effective shielding of the small diameter flat end of the frustum, which will therefore get heated up and also be subjected to thermal instability, producing a buckling force in the opposite direction than the buckling force from the instability on the big diameter end .

3) The effective way to remove thermal instability as an artifact is to get rid of the fiber-reinforced-epoxy boards at the flat ends and instead use a 1/4  inch thick (0.25 inches) copper plate for flat ends to prevent this thermal instability, and hence eliminate this artifact from consideration
Title: Re: EM Drive Developments - related to space flight applications - Thread 2
Post by: aero on 02/11/2015 04:12 pm
Quote
3) The effective way to remove thermal instability as an artifact is to get rid of the fiber-reinforced-epoxy boards at the flat ends and instead use a 1/4  inch thick (0.25 inches) copper plate for flat ends to prevent this thermal instability, and hence eliminate this artifact from consideration

No - Don't do that. Step back and look at the issue. We have a situation where we are reasonably certain (assuming a real effect) that something is either:

1 - Tunnelling through the copper end plates or,
2 - Going around the copper end plates,  (via the QV).

In the first case, a quarter inch thick end plate would eliminate tunnelling and eliminate the thrust.
In the second case, who knows, except that logically a thick end plate would shield the thrust effect.

Of course the thick end plate would eliminate thermal effects but it would be a situation of "Throwing out the baby with the bath water."

I personally hold to the "Tunnelling through" concept via evanescent waves, to which point I intend to start posting information next.

Title: Re: EM Drive Developments - related to space flight applications - Thread 2
Post by: Rodal on 02/11/2015 04:27 pm
Quote
3) The effective way to remove thermal instability as an artifact is to get rid of the fiber-reinforced-epoxy boards at the flat ends and instead use a 1/4  inch thick (0.25 inches) copper plate for flat ends to prevent this thermal instability, and hence eliminate this artifact from consideration

No - Don't do that. Step back and look at the issue. We have a situation where we are reasonably certain (assuming a real effect) that something is either:

1 - Tunnelling through the copper end plates or,
2 - Going around the copper end plates,  (via the QV).

In the first case, a quarter inch thick end plate would eliminate tunnelling and eliminate the thrust.
In the second case, who knows, except that logically a thick end plate would shield the thrust effect.

Of course the thick end plate would eliminate thermal effects but it would be a situation of "Throwing out the baby with the bath water."

I personally hold to the "Tunnelling through" concept via evanescent waves, to which point I intend to start posting information next.
Not doing a test that would eliminate thermal instability as a variable because of...assuming that a "Tunnelling through" conjecture may also be eliminated?

The R&D approach is to fearlessly perform many tests to eliminate possible artifacts and alternative explanations and to confirm and reproduce valid tests, not to conduct only a few tests that agree with an assumed theory.
Title: Re: EM Drive Developments - related to space flight applications - Thread 2
Post by: Mulletron on 02/11/2015 05:09 pm
Quoting @Star-Drive
Quote
As to why the vacuum test were observing less thrust than in air tests. please note the difference in the RF amps there were driving each test series.  The 30W Mini-Circuit Class-A RF amp was used for the in-air series reported in the 2014 JPC paper, whereas a 100W EMPower Class-A/B RF amplifier was used in the vacuum tests to date. So how could a less powerful RF amp produce more thrust than a more powerful one?

This tells me that the presence of an air/PE interface is an important feature and could be exploited to great effect ala Casimir-Lifshitz repulsion attraction. The different permittivity of these two is what's important. I mean, you get thrust with the PE dielectric inserted, you get more with air and PE inserted and none with just air, though I bet you'd get thrust under high power with just air like Shawyer does. The presence of air is clearly doing something good.

If these Emdrives are using the QV to achieve propulsion, we need to use what Casimir, Polder and Lifshitz have taught us and think like them. We can't think like Goddard on this one as we're not shooting anything out the back end. I know this sounds far out but the literature supporting all this is deep and wide and supported by direct observation. Much of this was covered in thread 1. A wide sample of this research is below, basically to show this research is real and to back up my claims. The key thing to really figure out is how these tiny forces are amplified and rectified within a resonant cavity, and only hints (http://www.tuwien.ac.at/en/news/news_detail/article/8908/) of this kind of research have been found so far. It all seems to boil down to confinement. None of the above would make an Emdrive want to thrust and move through space, the final bit is what we uncovered in thread 1 concerning casimir forces with different geometries. We uncovered the casimir force is positive and repulsive inside spheres, corners and cones, unlike parallel plates where it is negative and attractive.

http://www.pnas.org/content/111/29/10485.full possible lead to how the cavity boosts casimir forces
http://en.wikipedia.org/wiki/Dielectric_gas stuff better than air
http://arxiv.org/abs/1004.5243 interesting stuff about PTFE repulsion
http://cnls.lanl.gov/casimir/PresentationsSF/Repulsive_force09.pdf
http://arxiv.org/ftp/quant-ph/papers/0109/0109134.pdf
http://www.nature.com/news/2009/090107/full/news.2009.4.html
http://blogs.discovermagazine.com/80beats/2009/01/08/to-levitate-nano-objects-researchers-exploit-a-force-of-quantum-repulsion/#.VNt5NCzzlah
http://adsabs.harvard.edu/abs/2009Natur.457..170M
http://arxiv.org/abs/1202.6415v2
http://www.newscientist.com/article/dn16374-repulsive-quantum-effect-finally-measured.html#.VNt5Gyzzlai
http://www.nature.com/nature/journal/v457/n7226/abs/nature07610.html
http://journals.aps.org/prb/abstract/10.1103/PhysRevB.89.045430

Casimir geometry stuff:
http://merlin.fic.uni.lodz.pl/concepts/2005_3_4/2005_3_4_137.pdf
http://arxiv.org/pdf/1306.4370v1.pdf
http://scholar.lib.vt.edu/theses/available/etd-04272004-200428/unrestricted/casimir-effect.pdf
https://aphyr.com/media/pwl-2014-casimir.pdf
https://aphyr.com/media/comps.pdf


Edit: Figured it out, it's attractive between the PE and copper. Really stupid mistake.
Title: Re: EM Drive Developments - related to space flight applications - Thread 2
Post by: Rodal on 02/11/2015 05:40 pm
... None of the above would make an Emdrive want to thrust and move through space, the final bit is what we uncovered in thread 1 concerning casimir forces with different geometries. We uncovered the casimir force is positive and repulsive inside spheres, corners and cones, unlike parallel plates where it is negative and attractive.
....

Substitute below copper 1, air 3, PE 2.

Thanks for supplying this picture analogy as it greatly helps to understand what is being proposed.

Please help me understand the proposed analogy: 

EM Drive copper                              => substitute for egg-shaped object 1 in picture below,
Dielectric HD PE inside the EM Drive => substitute for object 2 in picture below,
Air inside the EM Drive                     => substitute for fluid 3 in picture below,

(http://forum.nasaspaceflight.com/index.php?action=dlattach;topic=36313.0;attach=702377;image)

the egg-shaped object 1 is free to move like an object in a rigid body motion as it is being repelled by object 2.  However, in the EM Drive, the big diameter flat end (object "1") is rigidly attached (by the curved walls of the truncated cone) to the dielectric HD PE (object "2")

(http://forum.nasaspaceflight.com/index.php?action=dlattach;topic=36313.0;attach=635478;image)

 so that while in the picture object 1 can be repelled and move away with a rigid body motion, in the EM Drive the big diameter plate is rigidly attached to the rest of the EM Drive, which is rigidly attached to the dielectric (object "2").  Moreover, the dielectric is inside the EM Drive. So it appears that, even if there would be a repulsive force between the dielectric and the big diameter end, this could not result in a rigid body motion of the EM Drive anymore than a spacecraft cannot move because an astronaut pushes inside it against one of its walls, or because an astronaut inside the spacecraft throws balls at one of its walls or because an astronaut uses a magnet inside the spacecraft to repel another magnet (with the same magnetic pole) that is attached to an internal wall.  They (the spacecraft, the astronaut and the balls and the magnets) are all part of a closed system.  The dielectric HD PE is inside of the EM Drive, hence it is in a closed system.  While on the picture, the egg-shaped object 1 is free to move with rigid body motion as it is being repelled by object 2.
Title: Re: EM Drive Developments - related to space flight applications - Thread 2
Post by: JPLeRouzic on 02/11/2015 06:11 pm
...
I accidentally stumbled upon this report:
https://hal.archives-ouvertes.fr/hal-00551421v1/document
Like they say on amazon, if you have been interested in this article, you might want to look at this one:
http://arxiv.org/abs/1411.2107

Title: Re: EM Drive Developments - related to space flight applications - Thread 2
Post by: aero on 02/11/2015 06:16 pm
Quote
3) The effective way to remove thermal instability as an artifact is to get rid of the fiber-reinforced-epoxy boards at the flat ends and instead use a 1/4  inch thick (0.25 inches) copper plate for flat ends to prevent this thermal instability, and hence eliminate this artifact from consideration

No - Don't do that. Step back and look at the issue. We have a situation where we are reasonably certain (assuming a real effect) that something is either:

1 - Tunnelling through the copper end plates or,
2 - Going around the copper end plates,  (via the QV).

In the first case, a quarter inch thick end plate would eliminate tunnelling and eliminate the thrust.
In the second case, who knows, except that logically a thick end plate would shield the thrust effect.

Of course the thick end plate would eliminate thermal effects but it would be a situation of "Throwing out the baby with the bath water."

I personally hold to the "Tunnelling through" concept via evanescent waves, to which point I intend to start posting information next.
Not doing a test that would eliminate thermal instability as a variable because of...assuming that a "Tunnelling through" conjecture may also be eliminated?

The R&D approach is to fearlessly perform many tests to eliminate possible artifacts and alternative explanations and to confirm and reproduce valid tests, not to conduct only a few tests that agree with an assumed theory.

I agree in principle, but in the real world, such tests are designed very carefully to avoid adversely effecting the device under test. Once we develop sufficient knowledge of the cause of the thrust effect that we are certain that the thin end plates are not a key component causing or contributing to the thrust effect, then will be the time to eliminate possible thermal effects using heavy metal as you propose. Until that time, an equally effective test to eliminate thermal effects is to simply turn off the RF power. That's silly, but until we know that thin end plates are not essential, major modification of the cavity entails a great risk of generating false data and is not logical. 
Title: Re: EM Drive Developments - related to space flight applications - Thread 2
Post by: aero on 02/11/2015 06:23 pm
First an obligatory cite.
Data provided in this and any following posts has been generated by Meep : A flexible free-software package for electromagnetic simulations, developed at MIT under  Grants from the National Science Foundation, contracts from the Army Research Office, and DARPA monitored contracts from the Office of Naval Research. For detailed description of Meep, see -
http://web.ics.purdue.edu/~pbermel/pdf/Farjadpour09.pdf

Now that that is handled, I have modeled the "Copper Kettle" EM cavity using dimensions provided by Paul March, "Star Drive," on February 7, 2015 here on NSF. Using Meep's built-in function, calculate force/flux external to the cavity excited by a magnetic source at the inside face of the dielectric. I obtain a force of ~ 0.25 mu-N/Watt. This compares to about 1 mu-N/Watt measured experimentally.

You may ask, "Why the big discrepancy?"  While Meep is sufficiently robust to deal with the EM thruster cavity, it was not designed to operate efficiently at those frequency/geometry conditions. More significantly, Meep was designed to measure relative differences in effects rather than absolute values but most importantly, no one knows exactly what it is that should be measured so in seeking an effect it was necessary to make approximations as to how the effect could be generated.

I believe that the EM thruster force results from evanescent waves tunneling through the thin copper end plates or minute gaps in the cavity joins of the end plates thereby exchanging momentum with the cavity and the decelerating photons of the evanescent waves. (to be explained.) Because I do not have access to computational resources required to model thin (35 micron) end surfaces, nor do I have a Meep compatible model valid for copper at 2 GHz, I used thicker "Perfect Metal" to model the cavity geometry. But, because perfect metal is impervious to radiation at all frequencies, I added small gaps at various places around the cavity, mainly in places where gaps could be expected, at the joins between the conic section and the end plates. The detected force changes and is strongly dependent on the placement of the gap.

I have attached a generated drawing of the cavity model used, showing placement of the gaps. This is the placement for which the 0.25 mu-N/Watt force/power was detected, though the gap, at 1.4 mm, is 10 times larger than actually used. That's to make it visible.

Edited to reformat.
Title: Re: EM Drive Developments - related to space flight applications - Thread 2
Post by: Mulletron on 02/11/2015 06:38 pm
@Rodal, the point is to illustrate a positive repulsive attractive force wrt the rest of the vacuum (the universe) in which the Emdrive resides. So if you have a positive vacuum pressure in proximity to a negative vacuum pressure, aligned fore to aft, what do you suppose would happen next? I'd bet something is compelled to move, especially when more energy is pumped into the system as RF. The point is not whether or not the copper/PE can move because they are rigidly attached. Get it? Don't forget about the QV, in which the Emdrive is supposedly getting momentum from, eg an open system.

In the ball example, the ball levitates in order to conserve momentum, which is brought on by symmetries in the laws of physics, after the ball hovers everything is balanced. In the emdrive case there is an asymmetry which enables continuous momentum transfer to the dielectric and the air but the amounts are not equal, and on the front side of the dielectric against the copper is the area of repulsion attraction. It can't move because the PE disc is attached, which doesn't seem to matter to me really. Just the positive region seems important. I barely get it too. It is hard to put all these concepts into perspective.

About half the time when I think this through, the darn thing thrusts backwards. What I need to figure out is how to describe two different unbalances in radiation pressure across the cavity at the same time.
1. Unbalanced Casimir pressure 2. The Shawyer unbalanced radiation pressure.

The best thing they could do is get Dr. Peter Milonni on the horn and ask him for help.

Edit: Figured it out, it's attractive. Hence why I couldn't figure out which way the thrust should go. Stupid mistake.
Title: Re: EM Drive Developments - related to space flight applications - Thread 2
Post by: Rodal on 02/11/2015 06:46 pm
...

I have attached a generated drawing of the cavity model used, showing placement of the
gaps. This is the placement for which the 0.25 mu-N/Watt force/power was detected,
though the gap, at 1.4 mm, is 10 times larger than actually used. That's to make it visible.

MEEP is a finite-difference solver that divides space and time into a regular grid to solve the time evolution of Maxwell’s equations.  A solution of Maxwell's equations in a closed-system (the EM Drive) cannot produce a force that will make it accelerate as a rigid body through space.  Otherwise that would imply that solutions of Maxwell's equations in closed-systems allow breaking conservation of momentum.  Conservation of momentum is one of the main principles in Physics, not violated both in Quantum Mechanics and in General Relativity. 

MEEP calculates the optical force using Maxwell's stress tensor.  The optical force should be reacted within the closed-system of the EM Drive so that the center of mass of the EM Drive should have a zero net force.

Besides Maxwell's equations, and the EM Drive and its internals (the dielectric inside it, etc.), what else have you included in your model and how was it included?
Title: Re: EM Drive Developments - related to space flight applications - Thread 2
Post by: aero on 02/11/2015 07:13 pm
I'm not sure that I understand what you are asking for, but you may be interested in looking at the fields generated, both internal and external to the cavity. The first drawing shows a fully enclosed cavity, the second shows a cavity with gaps of 140 microns placed as illustrated in the previous drawing. Both cavities are driven by a magnetic source at the inside face of the dielectric disk, at 1.937115E+009 GHz. Also, both images are at the completion of the 32-nd half period of the drive frequency.

Look closely at the second image. Note the standing waves on both end plates and the RF energy looping from one end to the other, outside the cavity. And also, please read this paper, in particular page 15-16 but the complete paper is pertinent. http://wwwsis.lnf.infn.it/pub/INFN-FM-00-04.pdf (http://wwwsis.lnf.infn.it/pub/INFN-FM-00-04.pdf)
Title: Re: EM Drive Developments - related to space flight applications - Thread 2
Post by: Rodal on 02/11/2015 07:24 pm
I'm not sure that I understand what you are asking for, but you may be interested in looking at the fields generated, both internal and external to the cavity. The first drawing shows a fully enclosed cavity, the second shows a cavity with gaps of 140 microns placed as illustrated in the previous drawing. Both cavities are driven by a magnetic source at the inside face of the dielectric disk, at 1.937115E+009 GHz. Also, both images are at the completion of the 32-nd half period of the drive frequency.

Look closely at the second image. Note the standing waves on both end plates and the RF energy looping from one end to the other, outside the cavity. And also, please read this paper, in particular page 15-16 but the complete paper is pertinent. http://wwwsis.lnf.infn.it/pub/INFN-FM-00-04.pdf (http://wwwsis.lnf.infn.it/pub/INFN-FM-00-04.pdf)

It looks to me that you have performed a similar analysis as Prof. Juan Yang in China and Fetta in the US, who solved Maxwell's equations in an EM Drive and also obtained resulting forces, because they did not model the EM Drive as a deformable body.  They just solved Maxwell's equations and obtained a force from Maxwell's Stress Tensor.

It seems to me that you just used MEEP to solve Maxwell's equations.  MEEP considers the EM Drive as a rigid body (you did not input the modulus of elasticity or thermal expansion coefficient or the electro- and magnetorestrictive material constants into the computer code). In the real world the EM Drive deforms due to the electromagnetic field.

This deformation of the EM Drive is quite real, it is because of this deformation that NASA Eagleworks (and others) have trouble keeping the EM Drive in resonance.

As a real body deforms when it is subject to an electromagnetic field you have to include electro- and magnetorestrictive forces in your analysis to model the real-world, in order to satisfy conservation of momentum.  When one does that, the mechanical force on the center of mass of the EM Drive will turn out to be exactly zero, to satisfy conservation of momentum.

To properly do this, you would need a multi-physics computer code (to include the deformation of the EM Drive) like ANSYS, or COMSOL.  (By the way my understanding of the COMSOL analysis done for NASA is that they included the COMSOL programs for Maxwell's equation and heat conduction, but that they did not (?) calculate the coupled deformation of the EM Drive either).

(http://www.comsol.com/model/image/366/big.png)

For the closed EM Drive to move through space you would need to, for example, to radiate Unruh radiation (as in Dr. McCulloch's theory), or couple to a frame-dependent (?) Quantum Vacuum (as per Dr. White's theory, etc.), or to have electromagnetic coupling with gravity, or some other form of coupling to an external field, or a Woodward Mach-Effect.  Just solving Maxwell's equations in a closed system won't do it.
Title: Re: EM Drive Developments - related to space flight applications - Thread 2
Post by: SleeperService on 02/11/2015 07:50 pm
Maybe of interest?

New method to generate high-power microwaves...
http://phys.org/news/2015-02-high-power-microwave.html

Some interesting tidbits how they did the modelling in the abstract...
http://link.springer.com/article/10.1140%2Fepjd%2Fe2014-50798-5

Full paper is paywalled however  :(
Title: Re: EM Drive Developments - related to space flight applications - Thread 2
Post by: aero on 02/11/2015 07:52 pm
I'm not sure that I understand what you are asking for, but you may be interested in looking at the fields generated, both internal and external to the cavity. The first drawing shows a fully enclosed cavity, the second shows a cavity with gaps of 140 microns placed as illustrated in the previous drawing. Both cavities are driven by a magnetic source at the inside face of the dielectric disk, at 1.937115E+009 GHz. Also, both images are at the completion of the 32-nd half period of the drive frequency.

Look closely at the second image. Note the standing waves on both end plates and the RF energy looping from one end to the other, outside the cavity. And also, please read this paper, in particular page 15-16 but the complete paper is pertinent. http://wwwsis.lnf.infn.it/pub/INFN-FM-00-04.pdf (http://wwwsis.lnf.infn.it/pub/INFN-FM-00-04.pdf)

It looks to me that you have performed a similar analysis as Prof. Juan Yang in China and Fetta in the US, who solved Maxwell's equations in an EM Drive and also obtained resulting forces, because they did not model the EM Drive as a deformable body.  They just solved Maxwell's equations and obtained a force from Maxwell's Stress Tensor.

It seems to me that you just used MEEP to solve Maxwell's equations.  MEEP considers the EM Drive as a rigid body (you did not input the modulus of elasticity or thermal expansion coefficient or the electro- and magnetorestrictive material constants into the computer code). In the real world the EM Drive deforms due to the electromagnetic field.

This deformation of the EM Drive is quite real, it is because of this deformation that NASA Eagleworks (and others) have trouble keeping the EM Drive in resonance.

As a real body deforms when it is subject to an electromagnetic field you have to include electro- and magnetorestrictive forces in your analysis to model the real-world, in order to satisfy conservation of momentum.  When one does that, the mechanical force on the center of mass of the EM Drive will turn out to be exactly zero, to satisfy conservation of momentum.

To properly do this, you would need a multi-physics computer code (to include the deformation of the EM Drive) like ANSYS, or COMSOL.  (By the way my understanding of the COMSOL analysis done for NASA is that they included the COMSOL programs for Maxwell's equation and heat conduction, but that they did not (?) calculate the coupled deformation of the EM Drive either).

(http://www.comsol.com/model/image/366/big.png)


While you are correct in your assumption that Meep treats the cavity as a rigid body, it is incorrect to assume that this data is similar to Prof. Yang's data. Professor Yang calculated forces on the interior of the cavity. Meep has calculated forces in the axial coordinate direction exterior to the cavity. As you must know, evanescent waves are a solution to Maxwell's equations and the forces generated and obtained by integration of the Poynting vector are also represented by solutions to Maxwell's equations. You may wish to take the time to study the paper which I referenced. Perhaps you can tell me where the error in that paper is.

Edit add: The paper develops a mathematical formula giving the superluminal velocity of evanescent waves in a waveguide at low GHz frequencies. It further formulated the problem to calculate that velocity and gives very summary results of a Mathematica solution to that problem.

And I too, doubt the validity of Prof. Yang's and Shawyer's theories.

Oh, and just for clarity, I have measured the forces outside the fully enclosed cavity shown first above. The forces are identically zero for the fully enclosed cavity as has been proven mathematically time and again.
Title: Re: EM Drive Developments - related to space flight applications - Thread 2
Post by: Mulletron on 02/11/2015 07:57 pm
If I could find direct evidence for a giant casimir effect in a resonant cavity, we'd be all set. I suspect it exists but I gotta find proof. Here's the full paper (at the bottom) about confining vacuum fluctuations to transmission lines that was in the news. http://www.tuwien.ac.at/en/news/news_detail/article/8908/

http://www.pnas.org/content/111/29/10485.full
Title: Re: EM Drive Developments - related to space flight applications - Thread 2
Post by: Mulletron on 02/11/2015 08:11 pm

From a space flight application perspective what is the TRL of the EM Drive?

Here is the list:

TRL 1.    Basic principles observed and reported
TRL 2.    Technology concept and/or application formulated
TRL 3.    Analytical & experimental critical function and/or characteristic proof-of-concept
TRL 4.    Component and/or breadboard validation in laboratory environment
TRL 5.    Component and/or breadboard validation in relevant environment
TRL 6.    System/subsystem model or prototype demonstration in a relevant environment (ground or space)
TRL 7.    System prototype demonstration in a space environment
TRL 8.    Actual system completed and "Flight qualified" through test and demonstration (ground or space)
TRL 9.    Actual system "Flight proven" through successful mission operations

I haven't seen enough experimental data to be able to say what TRL level the EM-Drive would have.   The EM tether has been deployed in orbit so therefore it must have a TRL level of at least 7. 

Edison started with a theory of how an electric light could be built that many thought was impossible.   It was thought to be impossible not because the physics was believed to be wrong but because so many others had tried and failed.

I'd say barely a 2.
Title: Re: EM Drive Developments - related to space flight applications - Thread 2
Post by: Notsosureofit on 02/11/2015 08:14 pm
Maybe of interest?

Some interesting tidbits how they did the modelling in the abstract...
http://link.springer.com/article/10.1140%2Fepjd%2Fe2014-50798-5

Full paper is paywalled however  :(

This looks worth pursuing. (at least from the dispersion viewpoint)
Title: Re: EM Drive Developments - related to space flight applications - Thread 2
Post by: Rodal on 02/11/2015 09:32 pm
.....

Oh, and just for clarity, I have measured the forces outside the fully enclosed cavity shown first above. The forces are identically zero for the fully enclosed cavity as has been proven mathematically time and again.

<< I have measured the forces outside the fully enclosed cavity shown first above. The forces are identically zero for the fully enclosed cavity>>  Does this mean that you agree that the force on the EM Drive's center of mass is zero? if you agree that it is zero, end of story.

If instead you meant that the forces due to the internal electromagnetic resonant (standing wave) fields are zero, but you still maintain that there is a net force on the EM Drive's center of mas due to evanescent waves on the exterior of the EM Drive, then one has to question

A) the Boundary Conditions you have on the exterior rectangular box surrounding the EM Drive and
B) the dimensional extent of this rectangular box. 

If the EM Drive is in outer space (ignoring a frame dependent (?) Quantum Vacuum or any other frame dependent field that you did not model, did you (?)) there is no such finite-extent box surrounding the EM Drive is there?  So one would have to model a medium that extends to infinity for all practical purposes rather than a box surrounding the EM Drive which has dimensions comparable to the EM Drive.

What happens to the force on the EM Drive's center of mass if you make that box 10, or 100 times larger?

Do the evanescent waves decay exponentially outside the EM Drive (as any evanescent wave should) ?

(http://forum.nasaspaceflight.com/index.php?action=dlattach;topic=36313.0;attach=705863;image)



PS: I see a smiling face on that picture, with two eyes at the upper corners of the big diameter flat end of the EM Drive  and a mouth at the bottom of the EM Drive :)
Title: Re: EM Drive Developments - related to space flight applications - Thread 2
Post by: aero on 02/11/2015 10:03 pm
Quote
PS: I see a smiling face on that picture, with two eyes at the upper corners of the big diameter flat end of the EM Drive  and a mouth at the bottom of the EM Drive :)
I have looked through all of the 34 images generated, one each half period. There are several that stimulate the imagination.

Quote
Do the evanescent waves decay exponentially outside the EM Drive (as any evanescent wave should) ?

Yes, they do, or at least the forces do decay. Whether exponentially or by some other rule, they do decay.


I am suggesting that the EM thruster generates thrust force as follows:

1 - Electromagnetic energy departs the cavity via superluminal evanescent waves. See the paper that I quoted previously for justification of superluminal evanescent waves.
2 - The evanescent waves remain attached to the cavity (they do not propagate) and reach a maximum distance at which point they collapse and the energy returns to the cavity, either at the point of exit or some other attachment point. I think this is reasonably well accepted by theory.
3 - The evanescent waves, departing the cavity carried superluminal momentum which was reacted against the cavity. However, once the waves reach maximum distance and return to the cavity, there is no driving energy to cause them to return superluminally. Once the wave has fully collapsed, it has returned momentum p = m c, to the cavity, acting on the cavity.

It is the difference between p = m vsl reacting on the cavity, and p = m c acting on the cavity that results in a thrust force from the EM thruster.

This force is easy to calculate, it is p = m * ( vsl - c). What is harder to calculate is the value of vsl but the referenced paper gives formulas to calculate that value based on engineering parameters.
Title: Re: EM Drive Developments - related to space flight applications - Thread 2
Post by: Rodal on 02/12/2015 11:36 am
Quote
PS: I see a smiling face on that picture, with two eyes at the upper corners of the big diameter flat end of the EM Drive  and a mouth at the bottom of the EM Drive :)
I have looked through all of the 34 images generated, one each half period. There are several that stimulate the imagination.

Quote
Do the evanescent waves decay exponentially outside the EM Drive (as any evanescent wave should) ?

Yes, they do, or at least the forces do decay. Whether exponentially or by some other rule, they do decay.


I am suggesting that the EM thruster generates thrust force as follows:

1 - Electromagnetic energy departs the cavity via superluminal evanescent waves. See the paper that I quoted previously for justification of superluminal evanescent waves.
2 - The evanescent waves remain attached to the cavity (they do not propagate) and reach a maximum distance at which point they collapse and the energy returns to the cavity, either at the point of exit or some other attachment point. I think this is reasonably well accepted by theory.
3 - The evanescent waves, departing the cavity carried superluminal momentum which was reacted against the cavity. However, once the waves reach maximum distance and return to the cavity, there is no driving energy to cause them to return superluminally. Once the wave has fully collapsed, it has returned momentum p = m c, to the cavity, acting on the cavity.

It is the difference between p = m vsl reacting on the cavity, and p = m c acting on the cavity that results in a thrust force from the EM thruster.

This force is easy to calculate, it is p = m * ( vsl - c). What is harder to calculate is the value of vsl but the referenced paper gives formulas to calculate that value based on engineering parameters.

Can you please show results for force on the center of mass of the EM Drive and evanescent-wave images (showing the electromagnetic fields inside and outside the EM Drive) for the same case without the dielectric (for an empty cavity) ?
Title: Re: EM Drive Developments - related to space flight applications - Thread 2
Post by: Rodal on 02/12/2015 01:00 pm
....

This force is easy to calculate, it is p = m * ( vsl - c). What is harder to calculate is the value of vsl but the referenced paper gives formulas to calculate that value based on engineering parameters.
Well (skipping the controversial subject of a superluminal speed  vsl and the discussion in the referenced paper involving tachyons), just addressing the force, as you know p is the momentum, while the force is the rate at which momentum changes with respect to time (F = dp/dt) : therefore the average force would be given by that momentum change divided by the time interval in which that momentum change takes place.
Title: Re: EM Drive Developments - related to space flight applications - Thread 2
Post by: aero on 02/12/2015 02:50 pm
....

This force is easy to calculate, it is p = m * ( vsl - c). What is harder to calculate is the value of vsl but the referenced paper gives formulas to calculate that value based on engineering parameters.
Well (skipping the controversial subject of a superluminal speed  vsl and the discussion in the referenced paper involving tachyons), just addressing the force, as you know p is the momentum, while the force is the rate at which momentum changes with respect to time (F = dp/dt) : therefore the average force would be given by that momentum change divided by the time interval in which that momentum change takes place.

Yes - my bad.

We also know from our previous work that F is proportional to Q and I suspect that Q in free space for the evanescent waves could be considered to equal 1, although Q is probably not defined in that case. I'm not sure where to go with that at the moment.
Title: Re: EM Drive Developments - related to space flight applications - Thread 2
Post by: aero on 02/12/2015 03:10 pm
Quote
Can you please show results for force on the center of mass of the EM Drive and evanescent-wave images (showing the electromagnetic fields inside and outside the EM Drive) for the same case without the dielectric (for an empty cavity) ?

I can try. The run will take several hours and may not give useful images.

As you know from our discussions of a few days (weeks) ago, Harminv does not calculate resonant frequency very accurately. (I did get independent confirmation on that point.) So my best guess will be to use the same drive frequency as before?? That is what I used for yesterday's image but that data was from Paul March and seems to be pretty solid. In that case, Harminv calculated Q = O(1000) , much lower that Paul's data, but by looking at the images each half- period, the cavity looked to be in resonance. I don't know what I'll see without the dielectric and I am assuming you'd like to see results for a vacuum filled cavity? As opposed to air filled?

I'm pursuing a work-around to determine resonance without using Harminv, but I'm not there yet.

It will be a little while before I set-up and start the run so if you have further guidance, it might be helpful.

Edit: Just a further note, the difference between air filled and vacuum filled cavity is not really detectable from looking at the field patterns.
Title: Re: EM Drive Developments - related to space flight applications - Thread 2
Post by: Rodal on 02/12/2015 03:38 pm
Quote
Can you please show results for force on the center of mass of the EM Drive and evanescent-wave images (showing the electromagnetic fields inside and outside the EM Drive) for the same case without the dielectric (for an empty cavity) ?

I can try. The run will take several hours and may not give useful images.

As you know from our discussions of a few days (weeks) ago, Harminv does not calculate resonant frequency very accurately. (I did get independent confirmation on that point.) So my best guess will be to use the same drive frequency as before?? That is what I used for yesterday's image but that data was from Paul March and seems to be pretty solid. In that case, Harminv calculated Q = O(1000) , much lower that Paul's data, but by looking at the images each half- period, the cavity looked to be in resonance. I don't know what I'll see without the dielectric and I am assuming you'd like to see results for a vacuum filled cavity? As opposed to air filled?

I'm pursuing a work-around to determine resonance without using Harminv, but I'm not there yet.

It will be a little while before I set-up and start the run so if you have further guidance, it might be helpful.

Edit: Just a further note, the difference between air filled and vacuum filled cavity is not really detectable from looking at the field patterns.

Concerning

"the difference between air filled and vacuum filled cavity is not really detectable from looking at the field patterns",

this is to be expected since all that MEEP knows about the difference between air and vacuum are the values of  electric permittivity and  magnetic permeability, which are very close for air and vauum.

Therefore it does not really make much difference whether you would like to run it with air or vacuum.  To make your job simpler for comparison, I would say run it with the same medium (air or vacuum) you had in the previous comparison you showed for the evanescent waves.

Concerning excitation frequency, we are interested in two frequencies:

A) a run with an excitation frequency of 2168 MHz to compare with NASA's experiment at that frequency without the dielectric. 

B) We are also interested in a run with the same excitation frequency you already used for previous comparison you showed for the evanescent waves.

Since B) may be easier, more expedient, for you to run first (all you have to do is to set the dielectric property: electric permittivity equal to the one you used for the empty section of the cavity) you might as well run case "B" first and run "A" after that.

(http://forum.nasaspaceflight.com/index.php?action=dlattach;topic=36313.0;attach=705863;image)
Title: Re: EM Drive Developments - related to space flight applications - Thread 2
Post by: Notsosureofit on 02/12/2015 04:38 pm

Quickly plugged in those measurements.  Used TM212, geometric mean.  Needed effective n=1.38 to get the freq w/ dielectric.  Q is down to 1000 for 50mmN at 50W.



@ RODAL

Is that Q estimate from the last run (vacuum) relevant ?
Title: Re: EM Drive Developments - related to space flight applications - Thread 2
Post by: Rodal on 02/12/2015 05:32 pm

Quickly plugged in those measurements.  Used TM212, geometric mean.  Needed effective n=1.38 to get the freq w/ dielectric.  Q is down to 1000 for 50mmN at 50W.



@ RODAL

Is that Q estimate from the last run (vacuum) relevant ?

@Notsosureofit

Sorry, I don't understand the question.

Are you referring to the Q = 6726  referred to in this image ?

When asking whether it is relevant, do you mean whether it is a reasonably valid Q assuming resonance at that frequency of 1.937115 GHz for mode TM212?

Given that NASA Eagleworks has reported:

Mode   Frequency (MHz)  Quality Factor, Q   Input Power (W)  Mean Thrust (μN)   Medium
TE012     1880.4               22000                         2.6                55.4                   Air
TM2112  1932.6                 7320                       16.9                91.2                   Air
TM2112  1936.7               18100                       16.7                50.1                   Air
TM212    1937.115             6726                       50                   66                      Vacuum

(http://forum.nasaspaceflight.com/index.php?action=dlattach;topic=36313.0;attach=636345;image)



Or is your question in reference to @aero's statement ( http://forum.nasaspaceflight.com/index.php?topic=36313.msg1330521#msg1330521 )

Quote from: aero
That is what I used for yesterday's image but that data was from Paul March and seems to be pretty solid. In that case, Harminv calculated Q = O(1000) , much lower that Paul's data, but by looking at the images each half- period, the cavity looked to be in resonance.
?

And your question is what is the relevance of this Harminv calculated Q = O(1000)  ?
Title: Re: EM Drive Developments - related to space flight applications - Thread 2
Post by: Rodal on 02/12/2015 06:07 pm

Quickly plugged in those measurements.  Used TM212, geometric mean.  Needed effective n=1.38 to get the freq w/ dielectric.  Q is down to 1000 for 50mmN at 50W.



@ RODAL

Is that Q estimate from the last run (vacuum) relevant ?

If your question is in reference to @aero's statement ( http://forum.nasaspaceflight.com/index.php?topic=36313.msg1330521#msg1330521 )

Quote from: aero
That is what I used for yesterday's image but that data was from Paul March and seems to be pretty solid. In that case, Harminv calculated Q = O(1000) , much lower that Paul's data, but by looking at the images each half- period, the cavity looked to be in resonance.
?

Here is an attempt to answer:

Aero may be expressing concern as to whether his MEEP modeling is showing the same resonant frequency and mode shapes modeled by COMSOL at NASA Eagleworks.

I don't recall seeing an identical replication by aero with MEEP of the mode shape and frequency of resonance modeled by COMSOL of the tests at NASA Eagleworks. 

The comparison I recall for the NASA tests done in air showed aero's MEEP modeling (the posted movie) to be at a significantly lower frequency and what is even more important a very different TE mode shape, having 3 half-waves in the longitudinal direction of the cone.  This is different from the latest COMSOL modeling showing (barely) 2 half-waves in the longitudinal direction for TM212, and different from the prior COMSOL modeling showing 2 half-waves in the longitudinal direction for TE012  . 

The big difference is that COMSOL shows attenuation in the longitudinal direction , with a low intensity field in the dielectric for a TM212 mode, while aero's MEEP run did not show such attenuation for a TE mode.  And the mode shape shown by MEEP was not the TE012 mode shape modeled by COMSOL (in the Brady report) either, because the MEEP analysis showed half-waves in the circumferential direction, while TE012 should have a constant electric field in the circumferential direction.  Given these large differences between aero's MEEP and COMSOL's resonance ( differences in mode shape and frequency) it is probably unwarranted for me to say anything about the Q shown by aeros' present MEEP analysis. 

My recollection is that aero in the past showed some runs with extremely high values of Q at resonance.  So if aero got a Q=1000 it may mean that this latest run is signficantly away from the resonant peak (everything else being the same).
Title: Re: EM Drive Developments - related to space flight applications - Thread 2
Post by: aero on 02/12/2015 07:03 pm
Actually, my concern is that the resonance frequency calculated by Harminv is as much as 3% off the peak. I don't know how that is related to the Q value it calculates but I worry that Harminv is sampling amplitude at that frequency to compute Q, and for a very sharp resonance peak, 3% off the peak would be way down on the shoulder. So the Q value might be meaningless.

As I wrote above, I am pursuing a work-around but I can't do meep runs and also load new software at the same time, let alone test the results against meep. i'm currently limping along with suspect software. Fortunately, Harminv is an independent module so it is not a part of the meep runs showing the field images. It does not run unless invoked by the control file.

Yes, I have shown some very high Q values as might be expected from a lossless (perfect metal) cavity. That was before I suspected a problem. Now the question arises, "Was that Q value actually for the resonance frequency calculated or was it for an adjacent resonance frequency which failed the minimum Q test, (Q > 50). Or even one that didn't fail, but was just not very interesting relatively?

The empty cavity is running now, hopefully we'll have some field images soon.
Title: Re: EM Drive Developments - related to space flight applications - Thread 2
Post by: Rodal on 02/12/2015 07:18 pm
Actually, my concern is that the resonance frequency calculated by Harminv is as much as 3% off the peak.
I'm not clear about the meaning of this.  Please clarify:

How do you know that the resonance frequency calculated by Harminv is as much as 3% off the peak?
What peak are you referring to?
Are you calculating a peak by other means than Harminv?
Are you referring to the frequency reported by NASA?

Title: Re: EM Drive Developments - related to space flight applications - Thread 2
Post by: SleeperService on 02/12/2015 07:30 pm
Guys.
I'm reluctant to say this, but maybe some of the previous posts would be better of as PM's to each other?
I follow this thread avidly and I don't want to see it shut down again.
Maybe have a think?
Title: Re: EM Drive Developments - related to space flight applications - Thread 2
Post by: JasonAW3 on 02/12/2015 07:38 pm
Guys.
I'm reluctant to say this, but maybe some of the previous posts would be better of as PM's to each other?
I follow this thread avidly and I don't want to see it shut down again.
Maybe have a think?

I'm not so sure that we shouldn't just let them go with this.  The math has gone way beyond me now, but it's fascinating watching science evolve right in front of us.

When you combine what's being learned here with Dark Matter and Dark energy theory, it begins to look like there may actually be something to the old Ether Theory.

At least that's what it's starting to sound like.
Title: Re: EM Drive Developments - related to space flight applications - Thread 2
Post by: aero on 02/12/2015 07:38 pm
I mentioned earlier that I had obtained independent confirmation of the Harminv discrepancy. Dr. Filip Dominic made some runs sampling the cylindrical resonator fields from meep, then analysed them using software independent of the meep package. He worked on it for about a week and determined that the Harminv values were off. They come closer as resolution is increased but my computer won't run this model at the high resolution needed to get closer.

Dr. Dominic sent me the software that he developed to do the analysis and that is part of my work around so I hope to have confidence in the resonance frequency. The only real problem I have is that his software is in Python, a language that I will need to practice with before I can have any certainty.
Title: Re: EM Drive Developments - related to space flight applications - Thread 2
Post by: Rodal on 02/12/2015 07:43 pm
I mentioned earlier that I had obtained independent confirmation of the Harminv discrepancy. Dr. Filip Dominic made some runs sampling the cylindrical resonator fields from meep, then analysed them using software independent of the meep package. He worked on it for about a week and determined that the Harminv values were off. They come closer as resolution is increased but my computer won't run this model at the high resolution needed to get closer.

Dr. Dominic sent me the software that he developed to do the analysis and that is part of my work around so I hope to have confidence in the resonance frequency. The only real problem I have is that his software is in Python, a language that I will need to practice with before I can have any certainty.
For a perfectly cylindrical resonator we have a simple exact solution (see:  http://en.wikipedia.org/wiki/Microwave_cavity#Cylindrical_cavity ) all that one needs to do is to compare the frequency calculated by MEEP for a cylindrical cavity with the exact solution.
Title: Re: EM Drive Developments - related to space flight applications - Thread 2
Post by: Rodal on 02/12/2015 07:53 pm
Guys.
I'm reluctant to say this, but maybe some of the previous posts would be better of as PM's to each other?
I follow this thread avidly and I don't want to see it shut down again.
Maybe have a think?

1) As explained here http://forum.nasaspaceflight.com/index.php?topic=29276.msg1301657#msg1301657, the previous long thread (which due to its extreme length would have been eventually locked to start a new thread anyway) was locked because of personal attacks and "stupid" and "pointless" posts "that did not feel like this site's subject matter." Specifically, posts (sometimes involving ad hominem attacks) not dealing  with this thread's subject: (Microwave ) EM Drive developments.

2) We are pursuing an examination of the EM Drive related to space flight applications, specifically an examination of the latest test results by NASA Eagleworks.  There are no personal attacks involved or stupid posts, hence I fail to see on what grounds you are concerned about present shutdown of the thread.

3) This EM Drive technology has now been repeatedly tested (experimentally ) at NASA in the USA, in the UK and in China.  The latest results announced by Paul March just a few days ago show that NASA has validated the EM Drive in a vacuum, which eliminates the explanation (rampant in the media) that the EM Drive results were due to thermal convection currents in the air. 
The experimental results of these measurements reportedly produce thrust/PowerInput that are within practical SpaceFlight Applications.  Such spaceflight applications have been discussed in several reports by NASA and in this thread.  So, assuming that the experimental results at NASA, UK and China are not all artifacts, the subject being discussed here is not just a theory (unlike some threads in the "Advanced Concepts" section of the NASA SpaceFlight Forum that discuss topics that are much further away from SpaceFlight applications).
Title: Re: EM Drive Developments - related to space flight applications - Thread 2
Post by: SleeperService on 02/12/2015 07:55 pm
Guys.
I'm reluctant to say this, but maybe some of the previous posts would be better of as PM's to each other?
I follow this thread avidly and I don't want to see it shut down again.
Maybe have a think?

I'm not so sure that we shouldn't just let them go with this.  The math has gone way beyond me now, but it's fascinating watching science evolve right in front of us.

When you combine what's being learned here with Dark Matter and Dark energy theory, it begins to look like there may actually be something to the old Ether Theory.

At least that's what it's starting to sound like.
I agree! But am worried about the thread. I'm sure Chris B will see it like yourself and I do...
Exciting times!

@Rodal
In awe about the stuff discussed so far. I retract... It is good to hear you peeps discussing and arguing in the open.
Title: Re: EM Drive Developments - related to space flight applications - Thread 2
Post by: Rodal on 02/12/2015 07:58 pm
...

@Rodal
In awe about the stuff discussed so far. I retract... It is good to hear you peeps discussing and arguing in the open.
Great  :)
Title: Re: EM Drive Developments - related to space flight applications - Thread 2
Post by: sghill on 02/12/2015 08:04 pm
Guys.
I'm reluctant to say this, but maybe some of the previous posts would be better of as PM's to each other?
I follow this thread avidly and I don't want to see it shut down again.
Maybe have a think?

I'm not so sure that we shouldn't just let them go with this.  The math has gone way beyond me now, but it's fascinating watching science evolve right in front of us.


I could not agree more.  I'm enjoying the conversation immensely, and as it's on topic, I hope the mods are enjoying it too!

I am also extremely interested hearing an in-depth discussion about in aero's statement in bold here:

3 - The evanescent waves, departing the cavity carried superluminal momentum which was reacted against the cavity. However, once the waves reach maximum distance and return to the cavity, there is no driving energy to cause them to return superluminally. Once the wave has fully collapsed, it has returned momentum p = m c, to the cavity, acting on the cavity.

It is the difference between p = m vsl reacting on the cavity, and p = m c acting on the cavity that results in a thrust force from the EM thruster.

If aero is correct, is this thing also a tachyon generator?!?
Title: Re: EM Drive Developments - related to space flight applications - Thread 2
Post by: Mulletron on 02/12/2015 08:10 pm
This old dog has to be dealt with. There isn't much left besides this:

Thermal instability technical paper from Rodal:
http://forum.nasaspaceflight.com/index.php?topic=29276.msg1293349#msg1293349

Title: Re: EM Drive Developments - related to space flight applications - Thread 2
Post by: Rodal on 02/12/2015 08:13 pm
This old dog has to be dealt with. There isn't much left besides this:

Thermal instability technical paper from Rodal:
http://forum.nasaspaceflight.com/index.php?topic=29276.msg1293349#msg1293349
Agreed, all that NASA Eagleworks has to do to eliminate this artifact is:

...get rid of the fiber-reinforced-polymer printed circuit board and just simply use a 1/4  inch thick (0.25 inches) copper plate for flat ends to prevent this thermal instability, and hence eliminate this artifact from consideration

http://forum.nasaspaceflight.com/index.php?topic=36313.msg1329239#msg1329239

http://forum.nasaspaceflight.com/index.php?topic=36313.msg1329424#msg1329424
Title: Re: EM Drive Developments - related to space flight applications - Thread 2
Post by: Giovanni DS on 02/12/2015 08:18 pm
I have a question about how the thrust is usually measured. Is it a pendulum? if so, could a physical cavity deformation simply move the barycenter? that could be mistaken as thrust.

Far from being an expert, I am still enjoying the thread :)
Title: Re: EM Drive Developments - related to space flight applications - Thread 2
Post by: Mulletron on 02/12/2015 08:25 pm
What I don't understand still is how that thermal buckling could appear as a sustained false thrust signature over 30-45 second runs, instead of just a quick impulse. Is it because the CG moved or something?
Title: Re: EM Drive Developments - related to space flight applications - Thread 2
Post by: Rodal on 02/12/2015 08:26 pm
I have a question about how the thrust is usually measured. Is it a pendulum? if so, could a physical cavity deformation simply move the barycenter? that could be mistaken as thrust.

Far from being an expert, I am still enjoying the thread :)

It is a horizontal torsional pendulum, with the torsional axis of rotation in the vertical direction  (call it "z"), paralel to the direction of the force of gravity and hence perpendicular to the floor.  The EM Drive's force produces a rotation around that vertical axis "z", such that the motion of the EM Drive occurs mainly in the x-y plane parallel to the floor, perpendicular to "z".   There is a small amount of coupling between the rotations around z, rotation around x and rotation around y due to the moments of inertia.  I wrote a program in Mathematica taking into account the nonlinear coupled equations of motion, thinking that the forces could be an artifact due to nonlinear coupling of the equations of motion but I after comparing with the data I rejected that possibility.
Title: Re: EM Drive Developments - related to space flight applications - Thread 2
Post by: Rodal on 02/12/2015 08:34 pm
What I don't understand still is how that thermal buckling could appear as a sustained false thrust signature over 30-45 second runs, instead of just a quick impulse. Is it because the CG moved or something?
Yes, good question.  For a perfectly elastic pendulum it should restore itself, as you say.  For it to persist for 45 seconds, one needs to posit some nonlinear non-conservative mechanism, perhaps in the nonlinear coupled equations of motion or perhaps also involving parasitic electromagnetic effects.

EDIT: There is another explanation that was offered by Oak Ridge involving thermal expansion, shifting the center of mass, we should discuss that sometime... (Thermal expansion is slower, but it could be that the answer is due to 1) thermal buckling (fast) + 2) thermal expansion (slower))
Title: Re: EM Drive Developments - related to space flight applications - Thread 2
Post by: SleeperService on 02/12/2015 08:43 pm
I have a question about how the thrust is usually measured. Is it a pendulum? if so, could a physical cavity deformation simply move the barycenter? that could be mistaken as thrust.

Far from being an expert, I am still enjoying the thread :)

It is a horizontal torsional pendulum, with the torsional axis of rotation in the vertical direction  (call it "z"), paralel to the direction of the force of gravity and hence perpendicular to the floor.  The EM Drive's force produces a rotation around that vertical axis "z", such that the motion of the EM Drive occurs mainly in the x-y plane parallel to the floor, perpendicular to "z".   There is a small amount of coupling between the rotations around z, rotation around x and rotation around y due to the moments of inertia.  I wrote a program in Mathematica taking into account the nonlinear coupled equations of motion, thinking that the forces could be an artifact due to nonlinear coupling of the equations of motion but I after comparing with the data I rejected that possibility.
Could you share your Mathmatica code nevertheless?
Title: Re: EM Drive Developments - related to space flight applications - Thread 2
Post by: Star One on 02/12/2015 08:46 pm

This old dog has to be dealt with. There isn't much left besides this:

Thermal instability technical paper from Rodal:
http://forum.nasaspaceflight.com/index.php?topic=29276.msg1293349#msg1293349
Agreed, all that NASA Eagleworks has to do to eliminate this artifact is:

...get rid of the fiber-reinforced-polymer printed circuit board and just simply use a 1/4  inch thick (0.25 inches) copper plate for flat ends to prevent this thermal instability, and hence eliminate this artifact from consideration

http://forum.nasaspaceflight.com/index.php?topic=36313.msg1329239#msg1329239

http://forum.nasaspaceflight.com/index.php?topic=36313.msg1329424#msg1329424

Be interested to hear Star Drive's response to that particular enquiry.
Title: Re: EM Drive Developments - related to space flight applications - Thread 2
Post by: SleeperService on 02/12/2015 08:50 pm
I think we are getting close to a truth here...
Title: Re: EM Drive Developments - related to space flight applications - Thread 2
Post by: aero on 02/12/2015 09:01 pm
Attached find the fields from the Copper Kettle at the 32-nd half-period of the drive frequency, 1.937115E+009 Hz. This would compare directly to the situation presented yesterday, except the dielectric has been removed.

I note that by watching the development of the fields from the first half-period, the cavity resonance does not seem to be as strong. That is, the fields drift with time. The strong blue point moves.
Title: Re: EM Drive Developments - related to space flight applications - Thread 2
Post by: aero on 02/12/2015 09:08 pm
Something I should mention. In watching the development of the evanescent waves and the propagating waves within the cavity, I can see no evidence that one moves any faster than the other.

I think what that means is that as soon as the evanescent wave exits the gap in the end plates, its velocity reverts to the classic solution. The equations needed to determine this are in the paper I referenced yesterday,
http://wwwsis.lnf.infn.it/pub/INFN-FM-00-04.pdf (http://wwwsis.lnf.infn.it/pub/INFN-FM-00-04.pdf) though it would take some tinkering to formulate them to correctly make this determination.

Anyone want to tackle that problem?
Title: Re: EM Drive Developments - related to space flight applications - Thread 2
Post by: aero on 02/12/2015 09:36 pm
@ Rodal
Quote
For a perfectly cylindrical resonator we have a simple exact solution (see:  http://en.wikipedia.org/wiki/Microwave_cavity#Cylindrical_cavity ) all that one needs to do is to compare the frequency calculated by MEEP for a cylindrical cavity with the exact solution.

Yes, that fact was used in Dr. Dominic's analysis. However, he also wanted to verify that the meep fields were as they were supposed to be and not the cause of the discrepancy. They are OK and peaks match the cylindrical cavity resonance frequencies nicely.
Title: Re: EM Drive Developments - related to space flight applications - Thread 2
Post by: Rodal on 02/12/2015 09:36 pm
Attached find the fields from the Copper Kettle at the 32-nd half-period of the drive frequency, 1.937115E+009 Hz. This would compare directly to the situation presented yesterday, except the dielectric has been removed.

I note that by watching the development of the fields from the first half-period, the cavity resonance does not seem to be as strong. That is, the fields drift with time. The strong blue point moves.

You calculated a force with the dielectric in the cavity,
(http://forum.nasaspaceflight.com/index.php?action=dlattach;topic=36313.0;attach=705863;image)

what force do you calculate without the dielectric?
(http://forum.nasaspaceflight.com/index.php?action=dlattach;topic=36313.0;attach=746935;image)

How does the force compare?
Title: Re: EM Drive Developments - related to space flight applications - Thread 2
Post by: aero on 02/12/2015 09:41 pm
Attached find the fields from the Copper Kettle at the 32-nd half-period of the drive frequency, 1.937115E+009 Hz. This would compare directly to the situation presented yesterday, except the dielectric has been removed.

I note that by watching the development of the fields from the first half-period, the cavity resonance does not seem to be as strong. That is, the fields drift with time. The strong blue point moves.

You calculated a force with the dielectric in the cavity, what force do you calculate without the dielectric?

How does the force compare?

Interesting question.

That is another time consuming computer job. I might have answers for you tonight, if not, tomorrow.
Title: Re: EM Drive Developments - related to space flight applications - Thread 2
Post by: Mulletron on 02/12/2015 10:14 pm

I accidentally stumbled upon this report:
https://hal.archives-ouvertes.fr/hal-00551421v1/document

I'm still trying to find which reference it was where any of this implied non-reciprocity.

It does imply non-reciprocity:

http://arxiv.org/pdf/1101.1174v1.pdf
Section 2.1.
Quote
Magneto-electric directional anisotropy can be induced in all media, including centrosymmetric
ones. It was first predicted by G. E. Stedman and coworkers [11] and observed for the first time in
a crystal by G. L. J. A. Rikken and coworkers [12]. This non-reciprocal effect is supposed to be independent on light polarization.

I'm sure any optics expert would tell me, well duh!
Title: Re: EM Drive Developments - related to space flight applications - Thread 2
Post by: Rodal on 02/12/2015 10:42 pm

I accidentally stumbled upon this report:
https://hal.archives-ouvertes.fr/hal-00551421v1/document

I'm still trying to find which reference it was where any of this implied non-reciprocity.

It does imply non-reciprocity:

http://arxiv.org/pdf/1101.1174v1.pdf
Section 2.1.
Quote
Magneto-electric directional anisotropy can be induced in all media, including centrosymmetric
ones. It was first predicted by G. E. Stedman and coworkers [11] and observed for the first time in
a crystal by G. L. J. A. Rikken and coworkers [12]. This non-reciprocal effect is supposed to be independent on light polarization.

I'm sure any optics expert would tell me, well duh!

Yes. 

I also read the (C. Robilliard and G. Bailly) paper (Table 1) as stating that the calculated magneto-electric linear birefringence MELB = nB − nE for the Quantum Vacuum is  3*10^(-10) of the value for Nitrogen: ten orders of magnitude smaller for the Quantum Vacuum than for Air.
Title: Re: EM Drive Developments - related to space flight applications - Thread 2
Post by: aero on 02/12/2015 11:38 pm
Attached find the fields from the Copper Kettle at the 32-nd half-period of the drive frequency, 2.168E+009Hz. In this case, dielectric has been removed and the drive frequency has been increased. Increased drive frequency means that the actual simulated time is less than in the previous cases because the period is shorter.
Title: Re: EM Drive Developments - related to space flight applications - Thread 2
Post by: Rodal on 02/12/2015 11:53 pm
Attached find the fields from the Copper Kettle at the 32-nd half-period of the drive frequency, 2.168E+009Hz. In this case, dielectric has been removed and the drive frequency has been increased. Increased drive frequency means that the actual simulated time is less than in the previous cases because the period is shorter.
Thanks. It appears then that the evanescent fields are strong and unsymmetric (fore and aft of the EM Drive) with a resonant cavity with the dielectric at 1.9 GHz

(http://forum.nasaspaceflight.com/index.php?action=dlattach;topic=36313.0;attach=705863;image)



The evanescent fields are weaker and their intensity much more symmetric (fore and aft of the EM Drive) with the dielectric removed at the same frequency 1.9 GHz.  There is no strong resonance now because with the dielectric removed the natural frequencies for the same mode shapes occur at higher frequencies.

(http://forum.nasaspaceflight.com/index.php?action=dlattach;topic=36313.0;attach=746935;image)


With the dielectric removed, at the higher frequency of 2.1 GHz, the evanescent field intensities are unsymmetric again (fore and aft) like with the dielectric at 1.9 GHz but the fields don't seem to be as strong

(http://forum.nasaspaceflight.com/index.php?action=dlattach;topic=36313.0;attach=750447;image)

1) You need to give us a contour field mapping rule: what do the colors mean in numerical terms, to further understand the contourplot. For example: what is the intensity of the white areas? of the red areas? of the orange areas? of the blue areas?

2) It would be great to get the forces on the center of mass of the EM Drive you compute for the three different cases
Title: Re: EM Drive Developments - related to space flight applications - Thread 2
Post by: Mulletron on 02/12/2015 11:56 pm
I think we have shown that both of these conditions have been met within EMdrive, which means that the research of Tiggelen, Rikken, Donaire and Shen directly apply to this.

http://arxiv-web3.library.cornell.edu/find/quant-ph/1/au:+Tiggelen_B/0/1/0/all/0/1
http://arxiv-web3.library.cornell.edu/find/quant-ph/1/au:+Rikken_G/0/1/0/all/0/1
http://arxiv-web3.library.cornell.edu/find/quant-ph/1/au:+Donaire_M/0/1/0/all/0/1
http://ptp.oxfordjournals.org/content/119/3/351.full.pdf

1) magneto-electric medium in crossed external fields
https://hal.archives-ouvertes.fr/hal-00551421v1/document (nitrogen)
http://arxiv.org/pdf/1101.1174v1.pdf (nitrogen, dielectrics)

2) chiral object in magnetic field
http://www.esrf.eu/UsersAndScience/Publications/Highlights/2011/scm/scm4
http://www.nature.com/srep/2013/130313/srep01444/full/srep01444.html
http://content.elsevierjournals.intuitiv.net/content/files/s003238610400713x-19125334.pdf
http://goo.gl/JegLXS
http://chemed.chem.purdue.edu/genchem/topicreview/bp/1polymer/terms.html

I am not implying this answers all the questions but I do think this is a significant lead in the right direction.
Source of slide http://qvg2013.sciencesconf.org/conference/qvg2013/program/Donaire_qvg2013.pdf

This also points the way toward ways of improving test results if this research does apply. It appears that only a weak coupling of QV momentum is present within the test article att.
1)Replace the pseudo-chiral PE with a truly chiral enantiopure material, preferably one exhibiting "strong magnetochiral dichroism"
2)Place the material in an area of highest magnetic field intensity.
3)Repressurize the cavity if possible.
4)Go back to TE012 mode.



Edit: added enantiopure and #4.
Title: Re: EM Drive Developments - related to space flight applications - Thread 2
Post by: Mulletron on 02/13/2015 12:12 am
Is there anything useful right now, today..... that can be done with an always on 50uN of thrust in space?
Title: Re: EM Drive Developments - related to space flight applications - Thread 2
Post by: Rodal on 02/13/2015 12:15 am
Is there anything useful right now, today..... that can be done with an always on 50uN of thrust in space?
Why limit oneself to just one EM Drive engine in space?.  How about posing the question as:

Is there anything useful right now, today..... that can be done with an always on N*50uN of thrust in space where N is an integer multiple (1,2,3,4,...) of 50 watts per engine.

Thrust: N*50 uN
Power: N*50 Watts

Thust/Power= 1 uN per Watt

For example, 4 engines: total thrust 200uN at total power 200 Watts, 10 engines 500 uN at 500 Watts total.
Title: Re: EM Drive Developments - related to space flight applications - Thread 2
Post by: Mulletron on 02/13/2015 12:20 am
Is there anything useful right now, today..... that can be done with an always on 50uN of thrust in space?
Why limit oneself to just one EM Drive engine in space?.  How about posing the question as:

Is there anything useful right now, today..... that can be done with an always on N*50uN of thrust in space where N is an integer multiple (1,2,3,4,...) of 50 watts per engine.

For example, 4 engines: total thrust 100uN at total power 200 Watts, 10 engines 500 uN at 500 Watts total.
IIRC, the ISS has 30KWe extra to play with.
Title: Re: EM Drive Developments - related to space flight applications - Thread 2
Post by: Rodal on 02/13/2015 12:24 am
Is there anything useful right now, today..... that can be done with an always on 50uN of thrust in space?
Why limit oneself to just one EM Drive engine in space?.  How about posing the question as:

Is there anything useful right now, today..... that can be done with an always on N*50uN of thrust in space where N is an integer multiple (1,2,3,4,...) of 50 watts per engine.

For example, 4 engines: total thrust 200uN at total power 200 Watts, 10 engines 500 uN at 500 Watts total.
IIRC, the ISS has 30KWe extra to play with.

That's 600 EM Drives giving 0.03 Newtons total thrust :)

They need to increase the Thrust/Power to decrease the number of EM Drives necessary.
Title: Re: EM Drive Developments - related to space flight applications - Thread 2
Post by: aero on 02/13/2015 12:49 am
Quote
2) It would be great to get the forces on the center of mass of the EM Drive you compute for the three different cases

I'm working on it. Resonance, Forces and Field patterns are separate runs, so it takes time. The empty cavity force run only has 10249.3s to go so it will be done tonight.
Title: Re: EM Drive Developments - related to space flight applications - Thread 2
Post by: RonM on 02/13/2015 12:59 am
Is there anything useful right now, today..... that can be done with an always on 50uN of thrust in space?
Why limit oneself to just one EM Drive engine in space?.  How about posing the question as:

Is there anything useful right now, today..... that can be done with an always on N*50uN of thrust in space where N is an integer multiple (1,2,3,4,...) of 50 watts per engine.

For example, 4 engines: total thrust 200uN at total power 200 Watts, 10 engines 500 uN at 500 Watts total.
IIRC, the ISS has 30KWe extra to play with.

That's 600 EM Drives giving 0.03 Newtons total thrust :)

They need to increase the Thrust/Power to decrease the number of EM Drives necessary.

If the experimental results hold and this thrust is real, it is very low. Before anyone can increase the thrust/power, someone will have to develop a theory that actually explains what is going on. Only then can the process be improved.

It is similar to work in high temperature superconductors. For years chemists have developed new high temperature superconductors with various properties pretty much by trial and error. They also need a working theory.

Once a functional theory is confirmed, one can predict the properties via math and have better results.

We are on the edge of a golden age of spaceflight with EM drives and superconducting capacitors. Or maybe not, but at least it is fun to discuss it.  :)

Title: Re: EM Drive Developments - related to space flight applications - Thread 2
Post by: Rodal on 02/13/2015 01:20 am
Quote
1) You need to give us a contour field mapping rule: what do the colors mean in numerical terms, to further understand the contourplot. For example: what is the intensity of the white areas? of the red areas? of the orange areas? of the blue areas?

Attached is the best I can do on short notice. I've looked at the available color palates in HDFView and they are weak, to say the least. Or maybe I just don't know how to scale them. I would like to know paraview better because I think it could display the data and in the format you wanted. Lines of constant value would be good. iso - whatevers.
So blue is the most negative and red is the most positive in that scale. 

What is white ? What is black?

White may be the most positive (off-scale) and black the most negative (off-scale), but I would like you to confirm...

(http://forum.nasaspaceflight.com/index.php?action=dlattach;topic=36313.0;attach=751862;image)
Title: Re: EM Drive Developments - related to space flight applications - Thread 2
Post by: aero on 02/13/2015 02:26 am
Quote
1) You need to give us a contour field mapping rule: what do the colors mean in numerical terms, to further understand the contourplot. For example: what is the intensity of the white areas? of the red areas? of the orange areas? of the blue areas?

Attached is the best I can do on short notice. I've looked at the available color palates in HDFView and they are weak, to say the least. Or maybe I just don't know how to scale them. I would like to know paraview better because I think it could display the data and in the format you wanted. Lines of constant value would be good. iso - whatevers.

Hopefully this will be more usable

Quote
What is white ? What is black?

White may be the most positive (off-scale) and black the most negative (off-scale), but I would like you to confirm...

I don't know the answer to your question but I think that your guess is right. I know that the black boarders of the cavity are perfect metal which is confusing as it is not field strength.  But the colors are not single valued. The palates seem to be designed for "Pretty" and not for information. There are two single valued palates in hdfview, both gray and both wash out the differences in field strength. The h5topng color palates also wash out the color, and don't even show the evanescent waves. So I'm not using that program. Again, I need to explore paraview's capabilities.

If it were easy, everyone would do it, no?
Title: Re: EM Drive Developments - related to space flight applications - Thread 2
Post by: Star-Drive on 02/13/2015 04:43 am
...  Just as a note, we've already tried re-enforcing the frustum endplates with angle aluminum mounted on their outside surfaces and we didn't notice any marked change in its thrust response.
...
This was expected not to make any significant difference.

As I wrote in my report:

Quote from: Dr. J. Rodal
Cotterell and Parkes (based on Cotterell's Ph.D. thesis at the University of Cambridge) correctly point out that the distribution of the heat flux "is not significant in the problem" of thermal buckling of a circular plate, whether the heating takes place uniformly over the whole circular plate or is concentrated in a central region.  Cotterell chose a distribution with a heatedDiameterRatio =1/0.3=3.333 instead of the heatedDiameterRatio=1 analyzed by Noda et.al.  The fact that the exact distribution is not significant for the deltaT that will produce buckling or for the buckling displacement follows from equilibrium: the membrane stress (=E*alpha*deltaT) force resultant (the integral of the membrane stress through the thickness) is reacted at the simply supported edges (that constrain the in-plane displacement).  The membrane force resultant is uniform and it is equal in the polar radial and angular (azimuthal) directions.  If only a central area is heated, the membrane stress is still equilibrated throughout.  If the plate has uniform thickness and isotropic material properties, the strain in the non heated area prior to buckling is the same as in the heated area.

See:

Cotterell, B., and Parkes, E. W.,  Thermal Buckling of Circular Plates, (United Kingdom's) Aeronautical Research Council, Ministry Of Aviation, Reports and Memoranda No. 3245, September, 1960



QUESTION: Why did you use a glass-fiber-reinforced polymer printed circuit board as the end plate ? << 0.063 inch thick FR4 printed circuit board with 1.0 oz copper, (~35 microns thick of Cu epoxied to the FR4 fiberglass)>>

1) The IR measurement was done from the outside, with the IR camera looking at the composite polymer surface of the circuit board surface they had on the exterior of the big diameter flat end.  Since this composite polymer has much lower thermal conductivity and much lower thermal diffusivity than copper, please take into account that these IR measurements represent a temperature and temperature gradients significantly lower than those present on the inner (copper) surface of the big diameter flat end.  In other words, the composite polymer circuit board surface being measured with the IR camera acts like an insulating surface concealing the higher temperature of the inner copper surface. Moreover, due to very low thermal diffusivity of the  glass-fiber-reinforced polymer printed circuit board, measurement of its exterior surface presents a considerable time delay of the interior temperature vs. time profile (as it takes time for the heat to conduct through the thickness of the very low diffusivity  of the  glass-fiber-reinforced polymer printed circuit board).


2) The modulus of elasticity of the glass-fiber-reinforced polymer printed circuit board is much lower than the modulus of elasticity of the copper.  The glass-fiber-reinforced polymer printed circuit board has orders of magnitude lower thermal conductivity and thermal diffusivity than the copper. (Comparison noted below).

3) Why not get rid of the fiber-reinforced-polymer printed circuit board and just simply use a 1/4  inch thick (0.25 inches) copper plate for flat ends to prevent this thermal instability, and hence eliminate this artifact from consideration ?

As to your questions, I would need some time to give them the analytical consideration they deserve and to calculate, rather than give you an impulsive, reflexive, answer that may be incorrect.



NOTE:  FR-4 is a composite material made with woven fiberglass cloth embedded in an epoxy resin (polymer) matrix.  The in-plane Young's modulus of FR4 is 3.0×10^6 psi , about six times smaller than Copper's Young modulus of 17.0×10^6 psi.  The modulus of elasticity in the thickness direction is much lower, practically as low as the modulus of elasticity of epoxy.    FR4's coefficient of thermal expansion - x-axis   1.4×10^(−5) 1/K, Coefficient of thermal expansion - z-axis   7.0×10^(−5) 1/K

The thermal conductivity is a tiny  0.29 W/m·K in the thickness direction, due to the low thermal conductivity of the epoxy resin.  Copper has a thermal conductivity of 401  W/m·K, that is 1400 times higher than the thermal conductivity of FR4

Rodal:

QUESTION: Why did you use a glass-fiber-reinforced polymer printed circuit board as the end plate ? << 0.063 inch thick FR4 printed circuit board with 1.0 oz copper, (~35 microns thick of Cu epoxied to the FR4 fiberglass)>>

3) Why not get rid of the fiber-reinforced-polymer printed circuit board and just simply use a 1/4  inch thick (0.25 inches) copper plate for flat ends to prevent this thermal instability, and hence eliminate this artifact from consideration ?

Answer: I used the FR4 PCB with 1.0oz copper end-plates to minimize payload mass while maximizing the thrust to weight ratio of the copper frustum assembly AND the signal to noise ratio of the torque pendulum system.  I used the 1.0oz copper thickness because the ac skin depth of RF at 1.5 GHz is about 2.0 microns, so 5X that depth or 10 microns of copper should contain 99% of the ac currents at this frequency.  And I still had an additional 25 microns of copper thickness as non-current carrying thermal mass to stabilize its performance. 

Now if you care to look at my pictures of the large OD end of the Eagleworks copper frustum, the mass of the entire frustum assembly without the PE discs is listed as 1.606 kg.  Your 0.25" thick solid copper end plates would add over 3.0 kg of dead mass to this figure just for the small and large OD ends plates and cost us $333.50 for a 12"x 24"x0.25" copper plate stock from McMaster-Carr needed to make them.  As I've said before we have tried aluminum angles and even 0.090" thick AL plates across the existing PCB end caps and noticed no change in its performance except for the increase in seismic noise pickup that the extra mass such payload-mass increasing modifications always bring to the table.

Best, Paul M.
 
Title: Re: EM Drive Developments - related to space flight applications - Thread 2
Post by: Rodal on 02/13/2015 12:18 pm
...the large OD end of the Eagleworks copper frustum, the mass of the entire frustum assembly without the PE discs is listed as 1.606 kg.  Your 0.25" thick solid copper end plates would add over 3.0 kg of dead mass to this figure just for the small and large OD ends plates and cost us $333.50 for a 12"x 24"x0.25" copper plate stock from McMaster-Carr needed to make them.

You don't need to replace both the small and big diameter flat plates.  You only need to change the big diameter flat end plate to test whether thermal instability is an artifact in the NASA experiments.  The dielectric HD PE shields the small diameter end plate from heating.  I understood from prior communication with you that you could have ~ 20 pounds = 9 kg on the platform. If a big diameter 1/4 inch thick copper  end plate adds more mass  than what your stand can support, you may still be able to test a 1/8 inch thick copper end plate (just for the Big Diameter end plate) as I proposed in my study based on the previous smaller power input. The mass of a 1/8 thick copper end for the big diameter would be 1.74 kg, hence the added mass will be 1 kg (calculating that the mass of the discarded FR4 with copper is ~0.73 kg). As I show in my report, it will significantly increase the time at which any thermal instability can take place, and this would be very noticeably in the experiments.  Another benefit of using a 1/8 inch thick copper end plate is that copper has several orders of magnitude greater thermal diffusivity than the FR4 epoxy composite you are using.  The FR4 is masking the true temperature profile (vs. time) of the end plate.  Eliminating the FR4 would give you a much more realistic temperature and temperature gradient profile vs time.

(The IR measurement was done from the outside, with the IR camera looking at the composite polymer surface of the circuit board surface they had on the exterior of the big diameter flat end.  Since this composite polymer has much lower thermal conductivity and much lower thermal diffusivity than copper, please take into account that these IR measurements represent a temperature and temperature gradients significantly lower than those present on the inner (copper) surface of the big diameter flat end.  In other words, the composite polymer circuit board surface being measured with the IR camera acts like an insulating surface concealing the higher temperature of the inner copper surface. Moreover, due to very low thermal diffusivity of the  glass-fiber-reinforced polymer printed circuit board, measurement of its exterior surface presents a considerable time delay of the interior temperature vs. time profile (as it takes time for the heat to conduct through the thickness of the very low diffusivity  of the  glass-fiber-reinforced polymer printed circuit board).

FR4 thermal conductivity is a tiny  0.29 W/m·K in the thickness direction, due to the low thermal conductivity of the epoxy resin.  Copper has a thermal conductivity of 401  W/m·K, that is 1400 times higher than the thermal conductivity of FR4. )


we have tried aluminum angles and even 0.090" thick AL plates across the existing PCB end caps and noticed no change in its performance except for the increase in seismic noise pickup that the extra mass such payload-mass increasing modifications always bring to the table.

If thermal buckling occurs, it will bend the end plate towards the interior of the EM Drive, the center of the end plate will move towards the inside (not the outside) of the EM Drive (reference: my report).  It moves towards the inside because the inside surface of the microwave EM Drive is hotter than the outside surface, hence there is a thermal gradient through the thickness which will bend the surface towards the inside such that the hotter (inner) surface expands more than the cooler (outside) surface. As such, installing aluminum angles or stiffeners on the outside of the big diameter end plate is not going to help, because the thermal buckling deflection will be away from it, not towards it.  Even if you would try to install stiffeners on the inside copper surface of the big diameter end plate (which may interfere with the EM Drive electromagnetic field) it would not be a good solution because the buckling deflections are very small (hence the stiffeners would need to be welded or rigidly adhered to the surface of the end plate all across, to make a difference).


Best regards and Godspeed

Title: Re: EM Drive Developments - related to space flight applications - Thread 2
Post by: sghill on 02/13/2015 01:24 pm
We are on the edge of a golden age of spaceflight with EM drives and superconducting capacitors. Or maybe not, but at least it is fun to discuss it.  :)

Today's XKCD beat you to the punch. :)

http://www.xkcd.com/1486/

Title: Re: EM Drive Developments - related to space flight applications - Thread 2
Post by: Rodal on 02/13/2015 01:58 pm
Quote
1) You need to give us a contour field mapping rule: what do the colors mean in numerical terms, to further understand the contourplot. For example: what is the intensity of the white areas? of the red areas? of the orange areas? of the blue areas?

Attached is the best I can do on short notice. I've looked at the available color palates in HDFView and they are weak, to say the least. Or maybe I just don't know how to scale them. I would like to know paraview better because I think it could display the data and in the format you wanted. Lines of constant value would be good. iso - whatevers.

Hopefully this will be more usable

Quote
What is white ? What is black?

White may be the most positive (off-scale) and black the most negative (off-scale), but I would like you to confirm...

I don't know the answer to your question but I think that your guess is right. I know that the black boarders of the cavity are perfect metal which is confusing as it is not field strength.  But the colors are not single valued. The palates seem to be designed for "Pretty" and not for information. There are two single valued palates in hdfview, both gray and both wash out the differences in field strength. The h5topng color palates also wash out the color, and don't even show the evanescent waves. So I'm not using that program. Again, I need to explore paraview's capabilities.

If it were easy, everyone would do it, no?

An alternative suggestion is that you could output (from MEEP) numerical values of the evanescent field at chosen locations (for example where you see maxima and minima from the graph, or where you see off-limit values like in the white zones, or at interesting locations, like for example near the corners of the EM Drive) to ascertain what are the contour magnitudes.

(http://forum.nasaspaceflight.com/index.php?action=dlattach;topic=36313.0;attach=705863;image)
Title: Re: EM Drive Developments - related to space flight applications - Thread 2
Post by: aero on 02/13/2015 04:30 pm
What I need is more hands and more skills, more skills for sure.

I tried to attach the digital output source of the final fields for the copper kettle as designed. It is only 125.4 MB, I would like to share the much more interesting file showing the evolution of the fields from start-up, but it is 4.4 GB so it might not be something Chris would want me to do. Actually, Chris doesn't want people uploading .h5 files at all. I discovered after waiting through the complete upload that .h5 is not an allowed format.

Title: Re: EM Drive Developments - related to space flight applications - Thread 2
Post by: Rodal on 02/13/2015 05:24 pm
What I need is more hands and more skills, more skills for sure.

I tried to attach the digital output source of the final fields for the copper kettle as designed. It is only 125.4 MB, I would like to share the much more interesting file showing the evolution of the fields from start-up, but it is 4.4 GB so it might not be something Chris would want me to do. Actually, Chris doesn't want people uploading .h5 files at all. I discovered after waiting through the complete upload that .h5 is not an allowed format.
I was suggesting you to look at a few values, if you had the time and if you were interested on what those contours mean.  Otherwise I understand  :)
No way I suggested to attach GB-long files. 

We look forward to seeing the forces for the different cases,

Best regards,
Title: Re: EM Drive Developments - related to space flight applications - Thread 2
Post by: aero on 02/13/2015 06:40 pm
@Rodal -
You have asked to see forces for empty cavities at two frequencies. I am working on it but I need to caution you to not take the results to seriously. If my proposition is right then there are several factors that effect the force values. Remember, I have toyed with forces and the gap locations for some time. I have selected locations that generate the largest forces within the bounds of resolution and meep digitization. I don't know of any reason to think that the experimental thrusters would have accidental gaps allowing RF power to escape the cavity at those exact locations.

I can imagine RF power leaking around the gasket where the end is joined to the cone, and because this leakage is accidental and unmonitored, I seriously doubt that it would be the same two times in a row when the end plate is removed and replaced. It would have been removed and replaced in order to insert the dielectric after measuring no thrust without said dielectric. Well, maybe the gap sizes were different which dramatically alters the force. The smaller the gap, the larger the force detected.

While I was toying with forces in isolation, it was OK for me to put the gaps where I wanted, but now that we are more seriously exploring the concept of evanescent waves related to force, I should step back and approach the model more realistically. By that I mean that I should close the gaps in the base plate and model the gasket between the end plates and the cone body using the data that I now have from Paul March. All I need in order to do that is the dielectric constant of the gasket material, and I expect that is readily available. I already have the gasket modelled as an air or vacuum gap but have not run cases using both gasket gaps simultaneously. I think I should do this so that any answer obtained can be related more directly to the experimental data that we have. I know from previous calculations that the force on the CG of the thruster is very much smaller when simulating gaps on only one end of the thruster ~ O(2-3)/c, that is, only slightly more than a classic photon rocket. I have ran that case for a single gasket gap. Similar results obtain for single end plate gaps though slightly larger.

The other thing that would be most useful would be a verification of the superluminal velocity proposition of evanescent waves. If that proposition could be mathematically confirmed then it might be possible to formulate a closed form (or nearly so) solution to the force generation of EM thrusters. Meep can not do that.

As an aside, another thing I know from prior runs is that the gap close to the corners of the EM thruster causes much greater force to be detected than gaps away from the corners. By corners, I mean the join between the cone body and the base plate. I think that is related to the thought that evanescent waves naturally obtain in the inside corners of the cavity but not in the center of the base plate.

I am proposing then, that I drop this "side" investigation of gaps in the base plate and return to a higher fidelity model of the EM thruster cavity as we now know it.

I would hope that you and Mathematica would pursue the calculation of velocity of the evanescent waves through small gaps. I expect that if you do so, you will obtain numbers that are unbelievably large, O (10^15) more or less. Also, depending on your assumptions, I expect that you will determine that the evanescent wave velocity drops to c or less immediately upon exiting the gap. It would not do for me to run this calculation because this whole idea needs independent verification, but you and Mathematica are totally independent of me and meep, so that should help forward our progress.

If you really need for me to pursue the forces on the empty cavity caused by the base plate gaps, I will do so but recognize in light of the above, that such results will not be meaningfully related to the EM thruster experimental results that we have available.

Title: Re: EM Drive Developments - related to space flight applications - Thread 2
Post by: Notsosureofit on 02/13/2015 07:19 pm
FYI

I have a copy of F. de Fornel, Evanescent Waves From Newtonian Optics to Atomic Optics. (a review copy from 2001)

More for instrumentation than anything else, but it's here.
Title: Re: EM Drive Developments - related to space flight applications - Thread 2
Post by: JasonAW3 on 02/13/2015 07:41 pm
Aero,

     From what I am reading you keep mentioning "superluminal velocity proposition of evanescent waves".  Would I be correct in assuming that you are talking about the apparent velocity of teh wavform themselves and not the matter or energy that the waveforms themselves are composed of?

     Otherwise, this put's a whole new slant on this EM drive debate.
Title: Re: EM Drive Developments - related to space flight applications - Thread 2
Post by: aero on 02/13/2015 07:53 pm
Aero,

     From what I am reading you keep mentioning "superluminal velocity proposition of evanescent waves".  Would I be correct in assuming that you are talking about the apparent velocity of teh wavform themselves and not the matter or energy that the waveforms themselves are composed of?

     Otherwise, this put's a whole new slant on this EM drive debate.

I'm referring to superluminal velocity as presented here http://wwwsis.lnf.infn.it/pub/INFN-FM-00-04.pdf (http://wwwsis.lnf.infn.it/pub/INFN-FM-00-04.pdf)
I need an interpretation from someone more knowledgeable than I, in order to know exactly what it is that I'm talking about. Perhaps you can tell me?
Title: Re: EM Drive Developments - related to space flight applications - Thread 2
Post by: JasonAW3 on 02/13/2015 08:21 pm
Aero,

     From what I am reading you keep mentioning "superluminal velocity proposition of evanescent waves".  Would I be correct in assuming that you are talking about the apparent velocity of teh wavform themselves and not the matter or energy that the waveforms themselves are composed of?

     Otherwise, this put's a whole new slant on this EM drive debate.

I'm referring to superluminal velocity as presented here http://wwwsis.lnf.infn.it/pub/INFN-FM-00-04.pdf (http://wwwsis.lnf.infn.it/pub/INFN-FM-00-04.pdf)
I need an interpretation from someone more knowledgeable than I, in order to know exactly what it is that I'm talking about. Perhaps you can tell me?

In this case I wouldn't profess any sort of knowledge.  I't's only fairly recently that I've come to accept that Space can expand faster than the velocity of light.  Although in retrospect, considering the age of the universe and it's apparent size, the concept is pretty much self evident.

I would speculate that it may be possible that what is being done is inducing a Space/Time wave form that could be transferring momentum to the drive itself by causing a high frequency oscillation of compression and expansion of Space itself.  It may be possible that compression and expansion of Space could be induced via electromagnetic frequencies rather than oscillated Strange Matter.  In other words, the drive is essentially being dragged along by the expansion of compressed space.  Catching the wave as it were.

Mind you, as far as I can tell, this would not violate any currently accepted laws of physics, and Einstein himself even noted that there seems to be a link between gravity and electromagnetism.  It's possible that electromagnetism may be able to stretch and compress space in a similar fashion to gravity.  The only reason that this hasn't been found out is nobody ever did an experiment quite like this one before.

Title: Re: EM Drive Developments - related to space flight applications - Thread 2
Post by: Mulletron on 02/13/2015 08:24 pm
This is an op ed.
Putting this all in perspective, the successful measurement of a thrust signature in hard vacuum helped me gain real confidence that the Emdrive and Cannae* are in fact producing a real thrust signature which begs explanation. For now it appears to work, but barely. Not enough to make people take notice, even though it works >6000x better than a photon rocket. This has potential to be HUGE. It would be irresponsible to not take this seriously now. Yes this is high risk, but it is also very high reward. I've been reading about crowdsourcing science lately after hearing a piece about it on the SGU podcast. Wouldn't it be nice if we could crowdsource research into the Emdrive.

I'm thinking crowdsourcing because there is a taboo associated with this subject. Because it is assumed to go against established scientific concepts. Academic institutions and professionals would no doubt be hesitant to publicly acknowledge involvement in such research, without sufficient evidence this is real. Put another way, they won't touch it until someone before them assumes the risk first. BZ to Eagleworks for having the courage to at least take a look. This is the public attitude, but if you really examine the literature, Emdrive can be explained via established principles and only serves as experimental evidence supporting the quantum foundations of reality.

As I've said, this isn't just some neat thruster, if it works it is also an instrument which could give immense insight into the nature of space and time itself. We really need contributions from experts in optics, materials science and QFT, with open minds. If we go the crowdsource route, we need a platform and we need leadership. Just like the hyperloop. This problem is to be figured out or put to rest. I have confidence that Eagleworks can eventually figure this out, but at the same time, we need to provide forceful backup**. The last thing we need is for this potential world changing technology to fall prey to an unworkable theory which leads to no results and time running out***. I know this sounds harsh, and I mean no disrespect. Do we want to be famous or correct? Both have to be true, to be true. In this writer's opinion, this is exactly what happened to ME. The inability or unwillingness to adapt one's theory in the face of new information and scrutiny. Can you imagine the space flight applications that could come from this potential technology, if it is in fact a reality and we can figure it out? Can you imagine what could have happened if we had not let it slip through the cracks for several more years? We (humanity) need to slough off our scientific hubris. We don't know everything yet; we only think we do. Quoting the controversial Rupert Sheldrake, "The science delusion is the belief that science already understands the nature of reality in principle, leaving only the details to be filled in. This is a very widespread belief in our society........." He is pointing out our hubris, how we think we know it all already, even as we are such a young, immature species. We don't know jack. Our collective ego surpasses our wisdom. The insight to be gained from our universe is as infinite as the universe itself, our comprehension is unfortunately finite.

http://www.nbcnews.com/tech/innovation/hyperloop-may-become-crowdsourced-reality-within-decade-n272116
http://www.usatoday.com/story/news/nation/2014/07/25/ozy-crowdsourcing-science/13143465/
http://www.nextscientist.com/3-examples-crowdsourcing-science/
http://www.abc.net.au/news/2014-09-10/crowd-sourced-research-more-trouble-than-it-is-worth-study/5734444
*We need to analyze Emdrive and Cannae equally; eg does thermal instability hold for both?
**Naval leadership concept
*** http://arxiv.org/pdf/1411.5359v1.pdf (the QVPT model has officially "got served")
Title: Re: EM Drive Developments - related to space flight applications - Thread 2
Post by: Rodal on 02/13/2015 09:09 pm
.....
*** http://arxiv.org/pdf/1411.5359v1.pdf (the QVPT model has officially "got served")
Interesting find.  The paper concludes with this advice:

Quote from: T. Lafleur
Although the extraction of a net momentum has been postulated in inhomogeneous vacuums [35] (due to a different mechanism than that discussed in this paper), the effect was found to immeasurably small, and it remains unclear whether this small non-zero value is an artefact of the field regularization techniques used.Regardless though, if a continuous net force is indeed being produced in the experiments in Refs. [38–41], one might expect that this should also imply an anomalously high power loss from the electric circuits of the thruster device. This power loss would be in addition to any standard power losses associated with such things as: ohmic heating, eddy current losses, dielectric and ferrite heating, radiation losses, etc. Consequently, a dedicated effort to isolate known power losses from the total input power of the device should identify any additional anomalous losses.

[35] B.A. van Tiggelen, G.L.J.A. Rikken, and V. Krsti´c, Phys. Rev. Lett. 96, 130402 (2006).

[38] H. White, P. March, N. Willians, and W. ONeil, Eagleworks laboratories: advanced propulsion physics research, JANNAF Joint Propulsion Meeting, 5-9 December 2011, Huntsville, Alabama.

[39] D.A. Brady, H.G. White, P. March, J.T. Lawrence, and F.J. Davies, Anomalous thrust production from an RF test device measured on a low-thrust torsion pendulum, 50th AIAA/ASME/SAE/ASEE Joint Propulsion Conference & Exhibit, 28-30 July 2014, Cleveland, Ohio.

[40] P. March and A. Palfreyman, AIP Conf. Proc. 813, 1321 (2006).

[41] Y. Juan, W. Yu-Quan, M. Yan-Jie, L. Peng-Fei, Y. Le, W. Yang, and H. Guo-Qiang, Chin. Phys. B 22, 050301 (2013).



Note: curiously the paper places the above-mentioned references 38 through 41 all under the same bracket (literally) but they are different models, some of them not even referring to the Quantum Vacuum (supposedly the subject of the paper: "Can the quantum vacuum be used as a reaction medium to generate thrust"):

[38-39] refer to White's Quantum Vacuum model, while

[40] refers to Mach-Lorentz Effect, and

[41] (Prof. Juan Yang in China) does not advocate the Quantum Vacuum model but instead numerically solves Maxwell's equations in the cavity (not clear whether Yang advocates Shawyer's proposal or the Minkowsky formulation of the energy-momentum tensor).
Title: Re: EM Drive Developments - related to space flight applications - Thread 2
Post by: Mulletron on 02/13/2015 09:27 pm
Yeah we've talked about the Lafleur rebuttal before*, but nothing has changed. Everybody knows that you can't accelerate something that 1) isn't material 2) has no rest frame to begin with. The QV isn't hydrazine or other rocket propellant, this is the very nature of space and time itself; the PT part of CPT symmetry! It is the ground state of all known fields. It is pure field theory. This is a rocket scientist trying to accelerate mass out the back end trying to naively use virtual particles as the RM; hence why we need more eyes on this.

* The one that refutes Dr. White (bottom of post): http://forum.nasaspaceflight.com/index.php?topic=29276.msg1300330#msg1300330
Title: Re: EM Drive Developments - related to space flight applications - Thread 2
Post by: aero on 02/13/2015 11:23 pm
Folks, please look at this model and tell me if your read of Paul March's description of the cavity joins are as shown, or is there a cone flange pinched under the Teflon gasket which is held in place by a thicker copper ring.

Thanks.
Title: Re: EM Drive Developments - related to space flight applications - Thread 2
Post by: Rodal on 02/14/2015 01:42 am
Dr. McCulloch has derived a new expression taking into account the three-dimensional nature of the truncated cone EM Drive and constructed a new comparison table. 

P=Power Input
Q=Quality Factor of Resonance
c=speed of light
wb=Big Diameter
wc=Small Diameter
L=Length

The McCulloch thrust (F), was (2-D expression)

F = PQL/c * (1/wb - 1/ws)                                   (1)

and it is now (3-D expression)

F = 6PQL/c * ( 1/(L+4wb) - 1/(L+4ws) )             (2)




See

http://physicsfromtheedge.blogspot.com/2015/02/mihsc-vs-emdrive-data-3d.html

(http://2.bp.blogspot.com/-Dnqt3RKcybk/VN4zCFC2JqI/AAAAAAAAAEs/kW12ut-78cc/s1600/DatavsMiHsC3.jpg)

Title: Re: EM Drive Developments - related to space flight applications - Thread 2
Post by: Notsosureofit on 02/14/2015 02:36 am
Bumbling off to bed, more convinced that the self-acceleration paper (Ido Kaminer, et al) has something going for it.

Finally getting the feel of how it conserves momentum.  With, luck I'll be snowed in w/ work caught up temporarily this weekend.
Title: Re: EM Drive Developments - related to space flight applications - Thread 2
Post by: zen-in on 02/14/2015 04:37 am


Rodal:

QUESTION: Why did you use a glass-fiber-reinforced polymer printed circuit board as the end plate ? << 0.063 inch thick FR4 printed circuit board with 1.0 oz copper, (~35 microns thick of Cu epoxied to the FR4 fiberglass)>>

3) Why not get rid of the fiber-reinforced-polymer printed circuit board and just simply use a 1/4  inch thick (0.25 inches) copper plate for flat ends to prevent this thermal instability, and hence eliminate this artifact from consideration ?

Answer: I used the FR4 PCB with 1.0oz copper end-plates to minimize payload mass while maximizing the thrust to weight ratio of the copper frustum assembly AND the signal to noise ratio of the torque pendulum system.  I used the 1.0oz copper thickness because the ac skin depth of RF at 1.5 GHz is about 2.0 microns, so 5X that depth or 10 microns of copper should contain 99% of the ac currents at this frequency.  And I still had an additional 25 microns of copper thickness as non-current carrying thermal mass to stabilize its performance. 

Now if you care to look at my pictures of the large OD end of the Eagleworks copper frustum, the mass of the entire frustum assembly without the PE discs is listed as 1.606 kg.  Your 0.25" thick solid copper end plates would add over 3.0 kg of dead mass to this figure just for the small and large OD ends plates and cost us $333.50 for a 12"x 24"x0.25" copper plate stock from McMaster-Carr needed to make them.  As I've said before we have tried aluminum angles and even 0.090" thick AL plates across the existing PCB end caps and noticed no change in its performance except for the increase in seismic noise pickup that the extra mass such payload-mass increasing modifications always bring to the table.

Best, Paul M.

A lurker here trying to keep an open mind.   I agree that replacing the single-sided FR4 endplate with a .25" or .125" thick plate of copper is not practical because it would shift the CM too much and may require a complete rebuild of the apparatus.   It is a good idea to perform experiments that prove or disprove alternative explanations for the observed effect.   Playing devil's advocate is a good thing for science.   I can think of a couple of ways to do this without upsetting the balance of this test setup very much. 

1) Instead of applying RF power to the cavity use thin film heating elements to apply an equivalent amount of thermal energy to the inside of the cavity.   There are several ways this can be done.  To address Dr. Rodal's thermal mechanical movement hypothesis you could apply 10 - 20 Watts of heat to the center of the endplate, by gluing thin film heating elements to the copper.    The kapton ones are very lightweight.   In the Aug. paper a baseline test was described where the RF power was terminated in a dummy load.   However the dummy load was not placed inside the cavity, if I remember correctly.   Maybe you have since done this test.

2) Thermally isolate the inside copper of the large end plate by mounting a similar dimensioned piece of copper foil (.010" or less).  The copper foil could be held in place with double sticky tape and small pieces of FR4 (pcb material).    The copper foil would be heated by the RF instead of the thin copper on the inside of the endplate and a small gap between the endplate and the copper foil will insulate the thin copper of the endplate.
Title: Re: EM Drive Developments - related to space flight applications - Thread 2
Post by: RotoSequence on 02/14/2015 04:47 am
A copper plate is unwieldy, heavy, and expensive, but what about an aluminum plate? Its bulk electrical characteristics shouldn't be too dissimilar from copper's in this application, right?
Title: Re: EM Drive Developments - related to space flight applications - Thread 2
Post by: MathieuA on 02/14/2015 08:31 am
I've been reading this thread with great interest, and thank you all for all the information you brought. (i've learnt a bunch of things and been happy to read back my physics courses again)

BUT: I don't understand why the simplest explanation here:

http://forum.nasaspaceflight.com/index.php?topic=29276.msg1293349#msg1293349
NASA'S MICROWAVE PROPELLANT-LESS THRUSTER ANOMALOUS RESULTS:
CONSIDERATION OF A THERMO-MECHANICAL EFFECT

wouldn't explain all the results obtained so far (in ambiant air pressure and in a lab's vacuum)

Is there a need to find other complicated explanations before this one is proven wrong ?

Title: Re: EM Drive Developments - related to space flight applications - Thread 2
Post by: Mulletron on 02/14/2015 10:29 am
Well if I were an enterprising entrepreneur (which I'm not), I would definitely use the insight brought forth from the discussion and investigation in this thread and apply it toward exploring the possibility of building (additive or subtractive manufacturing) a device consisting of thousands of YBCO resonant cavities in a cluster and pumped with microwave and infrared radiation. Especially since it has been shown recently that YBCO will superconduct at room temperature when exposed to pulsed IR.

http://www.mpg.de/8785897/superconductivity-room-temperature
http://www.nature.com/nature/journal/v516/n7529/full/nature13875.html

Now go build me a hovercar!
Title: Re: EM Drive Developments - related to space flight applications - Thread 2
Post by: mclumber1 on 02/14/2015 12:11 pm
This is an op ed.
Putting this all in perspective, the successful measurement of a thrust signature in hard vacuum helped me gain real confidence that the Emdrive and Cannae* are in fact producing a real thrust signature which begs explanation. For now it appears to work, but barely. Not enough to make people take notice, even though it works >6000x better than a photon rocket. This has potential to be HUGE. It would be irresponsible to not take this seriously now. Yes this is high risk, but it is also very high reward. I've been reading about crowdsourcing science lately after hearing a piece about it on the SGU podcast. Wouldn't it be nice if we could crowdsource research into the Emdrive.

I'm thinking crowdsourcing because there is a taboo associated with this subject. Because it is assumed to go against established scientific concepts. Academic institutions and professionals would no doubt be hesitant to publicly acknowledge involvement in such research, without sufficient evidence this is real. Put another way, they won't touch it until someone before them assumes the risk first. BZ to Eagleworks for having the courage to at least take a look. This is the public attitude, but if you really examine the literature, Emdrive can be explained via established principles and only serves as experimental evidence supporting the quantum foundations of reality.

As I've said, this isn't just some neat thruster, if it works it is also an instrument which could give immense insight into the nature of space and time itself. We really need contributions from experts in optics, materials science and QFT, with open minds. If we go the crowdsource route, we need a platform and we need leadership. Just like the hyperloop. This problem is to be figured out or put to rest. I have confidence that Eagleworks can eventually figure this out, but at the same time, we need to provide forceful backup**. The last thing we need is for this potential world changing technology to fall prey to an unworkable theory which leads to no results and time running out***. I know this sounds harsh, and I mean no disrespect. Do we want to be famous or correct? Both have to be true, to be true. In this writer's opinion, this is exactly what happened to ME. The inability or unwillingness to adapt one's theory in the face of new information and scrutiny. Can you imagine the space flight applications that could come from this potential technology, if it is in fact a reality and we can figure it out? Can you imagine what could have happened if we had not let it slip through the cracks for several more years? We (humanity) need to slough off our scientific hubris. We don't know everything yet; we only think we do. Quoting the controversial Rupert Sheldrake, "The science delusion is the belief that science already understands the nature of reality in principle, leaving only the details to be filled in. This is a very widespread belief in our society........." He is pointing out our hubris, how we think we know it all already, even as we are such a young, immature species. We don't know jack. Our collective ego surpasses our wisdom. The insight to be gained from our universe is as infinite as the universe itself, our comprehension is unfortunately finite.

http://www.nbcnews.com/tech/innovation/hyperloop-may-become-crowdsourced-reality-within-decade-n272116
http://www.usatoday.com/story/news/nation/2014/07/25/ozy-crowdsourcing-science/13143465/
http://www.nextscientist.com/3-examples-crowdsourcing-science/
http://www.abc.net.au/news/2014-09-10/crowd-sourced-research-more-trouble-than-it-is-worth-study/5734444
*We need to analyze Emdrive and Cannae equally; eg does thermal instability hold for both?
**Naval leadership concept
*** http://arxiv.org/pdf/1411.5359v1.pdf (the QVPT model has officially "got served")

Has anyone here considered a kickstarter campaign?
Title: Re: EM Drive Developments - related to space flight applications - Thread 2
Post by: Notsosureofit on 02/14/2015 12:37 pm
Dr. McCulloch has derived a new expression taking into account the three-dimensional nature of the truncated cone EM Drive and constructed a new comparison table. 

P=Power Input
Q=Quality Factor of Resonance
c=speed of light
wb=Big Diameter
wc=Small Diameter
L=Length

The McCulloch thrust (F), was (2-D expression)

F = PQL/c * (1/wb - 1/ws)                                   (1)

and it is now (3-D expression)

F = 6PQL/c * ( 1/(L+4wb) - 1/(L+4ws) )             (2)




See

http://physicsfromtheedge.blogspot.com/2015/02/mihsc-vs-emdrive-data-3d.html

(http://2.bp.blogspot.com/-Dnqt3RKcybk/VN4zCFC2JqI/AAAAAAAAAEs/kW12ut-78cc/s1600/DatavsMiHsC3.jpg)

Do we have a complete set of data like Mike's table, including mode numbers ?
Title: Re: EM Drive Developments - related to space flight applications - Thread 2
Post by: Mulletron on 02/14/2015 01:11 pm

Has anyone here considered a kickstarter campaign?

Kinda need expertise instead of money. A workable theory to help improve the performance is what is needed so as to be useful in space flight applications and other uses.

Shawyer and Fetta own the invention.
Title: Re: EM Drive Developments - related to space flight applications - Thread 2
Post by: Rodal on 02/14/2015 01:43 pm
...

Do we have a complete set of data like Mike's table, including mode numbers ?

Hi @Notsosureofit,

Since the mode shapes are very important for your formula, and since we only have experimental confirmation for one experimental test for mode shape TM21p (the hard vacuum test presented by Paul March, where the thermal signature was measured with an IR camera showing TM21p for the big diameter cross-section -p may be 2 according to COMSOL, but there is no measurement of p), I thought it crucial to independently compute the mode shapes.  There is also a disagreement between the different COMSOL calculations run for NASA whether the mode shape  is TM212 or TM211

Also, if this EM Drive works due to some physical principle, it seems to me that the mode shapes may be very important concerning the amount of thrust for a given power input.  Hence it is very important to understand what mode shapes are involved.

Based on the hint in the 1969 patent by Grant, I integrated the differential wavenumber of the cross-section of a circular wavequide to obtain a closed-form solution for a truncated cone (which has a diameter variation along the longitudinal axis), including the dielectric.  However, what I found was, that this expression:

1) Gives mode shapes and frequencies that are less than 1.5% different from the cylindrical cavity using the Geometric Mean of the big and small diameters

2) There is a substantial cut-off of mode shapes in the truncated cone.  The solution shows that the TM01p mode shapes displayed by Greg Egan would be cut-off, yet Egan displays them as not being cut-off.

3) The frequencies for TE012 and TM212 are different than the NASA COMSOL calculations.

4) The solution does not display attenuation of the field strength from the big diameter to the small diameter end (instead modes are just cut-off completely).

Therefore I decided to now compute the exact solution of the truncated cone using spherical wave transmission, as done by Greg Egan and others, to assess what is really going on in the truncated cone.  Please note that the MEEP movie shown by @aero shows spherical wave transmission through the truncated cone.  The exact solution using spherical wave transmission involves Legendre functions  and spherical Bessel functions, and it involves the solution of two eigenvalue problems (one eigenvalue problem in terms of Legendre functions of the cosine of the cone half-angle "theta" and another eigenvalue problem in terms of spherical Bessel functions of the radial spherical coordinate "r" along which the curved surface of the cone is defined). 

Rather than give partial results and exhaust everybody's patience, I thought it best to take another week to establish really what is going on.

The benefit of all this would be that the exact solution can be computed in a second while the COMSOL and MEEP solution take hours, and that the closed-form solution is exact while the COMSOL and MEEP solutions are discrete models that can only be approximations, as they always involve convergence issues.  For example, COMSOL's Finite Element solution is based on a Galerkin approximation rather than an exact solution point-to-point. (Yet, it maybe close enough for practical purposes, we will see...)

Once I have performed calculations for the mode shapes at given frequencies using the exact solution, I will calculate a table for the different expressions.

(http://gregegan.customer.netspace.net.au/SCIENCE/Cavity/CavityShape.gif)
Title: Re: EM Drive Developments - related to space flight applications - Thread 2
Post by: frobnicat on 02/14/2015 01:54 pm
I've been reading this thread with great interest, and thank you all for all the information you brought. (i've learnt a bunch of things and been happy to read back my physics courses again)

BUT: I don't understand why the simplest explanation here:

http://forum.nasaspaceflight.com/index.php?topic=29276.msg1293349#msg1293349
NASA'S MICROWAVE PROPELLANT-LESS THRUSTER ANOMALOUS RESULTS:
CONSIDERATION OF A THERMO-MECHANICAL EFFECT

wouldn't explain all the results obtained so far (in ambiant air pressure and in a lab's vacuum)

Is there a need to find other complicated explanations before this one is proven wrong ?

From what I understand (Rodal please correct me if wrong as I may have skimmed that too fast) the thermal buckling can very well explain the initial steep step thrust response to power-on. The steepness of the initial measured thrust on power-on were used as an argument by Eagleworks to rule out any thermal effect (as any thermal effect was supposed too slow to explain the sharp rise). Rodal's work shows that a thermal effect is not ruled out as far as initial steep rise is concerned : thermal buckling can account for the magnitude and rate of the initial rise (same ballpark).

The question remaining is the sustained level of apparent thrust after the rise, sustained for 35 or 40 s at approx. constant magnitude. I'm not sure this was tackled and ruled out as possibly being produced by pure thermal buckling/dilatation explanation but my feeling (shared ?) is that it is hardly possible. A constant thrust of 50µN for 40s means a mass of 1kg (out of my hat, ballpark of heated mass) accelerating at constant a=F/m=50e-6 m/s² during the thrust plateau of 40s, that is an integrated displacement of (some mass of 1kg relative to rest of the frustum + balance arm) x=0.5 a t² = 4 cm (more than an inch).

Also to have a (more or less) nice fall to flat baseline at power-off (as observed) would mean this moving mass would stop accelerating but continue at its gained velocity (relative to rest of frustum + balance arm) and ideally stop very gently, at opposite acceleration with magnitude much lower than the previous one. This continuation of movement would add even more to the total displacement, I'd say a few times 4cm (like 10 or 20 cm...). Otherwise if the moving mass where to stop quickly at power-off, we would observe a sharp fall of the thrust signature much below the initial 0 baseline. Generally you could tell such "thrust from displacement of one part of the drive relative to another part of drive (1)" by seeing a 0 net thrust when integrating during the power-on and well after the power-off : you would see a part above the baseline (power-on period) and a part below (after power-off) and the later would cancel the former.


             
Either this : sharp stop of displacement at power-off, sharp thrust opposite to initial direction.
           -----
          |     |
...-------      |      --------...
                |   ---
                | --
                 -

Or this : gentle stop of displacement at power-off, harder to see but implies higher total integrated displacement (many centimetres)
           -----
          |     |
...-------      |          ----...
                 ----------   

While what is observed looks like a net imbalance (more above the baseline than below)
           -----
          |     |
...-------       --------------...



It would certainly be possible to hide a moving mass inside the frustum, with a system to make it move the right displacement (many cm) to mimic a thrust signature with apparent net imbalance as seen from the outside (for not too long after power-off and with a "drifting baseline" to blur the opposite thrust period), but it's hard to see how such a huge displacement could happen by accident or stay unnoticed by experimenters.

I would say that a thermo-mechanical effect involving a simple linear displacement of some driver's part mass is rather well ruled-out as an explanation for the sustained part (40 s) and early after decay of the signal. Maybe some more complex classical explanation might still apply.

(1) by "drive" I consider the whole system mounted on the balance's arm and the arm itself
Title: Re: EM Drive Developments - related to space flight applications - Thread 2
Post by: Rodal on 02/14/2015 02:11 pm
I've been reading this thread with great interest, and thank you all for all the information you brought. (i've learnt a bunch of things and been happy to read back my physics courses again)

BUT: I don't understand why the simplest explanation here:

http://forum.nasaspaceflight.com/index.php?topic=29276.msg1293349#msg1293349
NASA'S MICROWAVE PROPELLANT-LESS THRUSTER ANOMALOUS RESULTS:
CONSIDERATION OF A THERMO-MECHANICAL EFFECT

wouldn't explain all the results obtained so far (in ambiant air pressure and in a lab's vacuum)

Is there a need to find other complicated explanations before this one is proven wrong ?

From what I understand (Rodal please correct me if wrong as I may have skimmed that too fast) the thermal buckling can very well explain the initial steep step thrust response to power-on. The steepness of the initial measured thrust on power-on were used as an argument by Eagleworks to rule out any thermal effect (as any thermal effect was supposed too slow to explain the sharp rise). Rodal's work shows that a thermal effect is not ruled out as far as initial steep rise is concerned : thermal buckling can account for the magnitude and rate of the initial rise (same ballpark).

The question remaining is the sustained level of apparent thrust after the rise, sustained for 35 or 40 s at approx. constant magnitude. I'm not sure this was tackled and ruled out as possibly being produced by pure thermal buckling/dilatation explanation but my feeling (shared ?) is that it is hardly possible. A constant thrust of 50µN for 40s means a mass of 1kg (out of my hat, ballpark of heated mass) accelerating at constant a=F/m=50e-6 m/s² during the thrust plateau of 40s, that is an integrated displacement of (some mass of 1kg relative to rest of the frustum + balance arm) x=0.5 a t² = 4 cm (more than an inch).

Also to have a (more or less) nice fall to flat baseline at power-off (as observed) would mean this moving mass would stop accelerating but continue at its gained velocity (relative to rest of frustum + balance arm) and ideally stop very gently, at opposite acceleration with magnitude much lower than the previous one. This continuation of movement would add even more to the total displacement, I'd say a few times 4cm (like 10 or 20 cm...). Otherwise if the moving mass where to stop quickly at power-off, we would observe a sharp fall of the thrust signature much below the initial 0 baseline. Generally you could tell such "thrust from displacement of one part of the drive relative to another part of drive (1)" by seeing a 0 net thrust when integrating during the power-on and well after the power-off : you would see a part above the baseline (power-on period) and a part below (after power-off) and the later would cancel the former.


             
Either this : sharp stop of displacement at power-off, sharp thrust opposite to initial direction.
           -----
          |     |
...-------      |      --------...
                |   ---
                | --
                 -

Or this : gentle stop of displacement at power-off, harder to see but implies higher total integrated displacement (many centimetres)
           -----
          |     |
...-------      |          ----...
                 ----------   

While what is observed looks like a net imbalance (more above the baseline than below)
           -----
          |     |
...-------       --------------...



It would certainly be possible to hide a moving mass inside the frustum, with a system to make it move the right displacement (many cm) to mimic a thrust signature with apparent net imbalance as seen from the outside (for not too long after power-off and with a "drifting baseline" to blur the opposite thrust period), but it's hard to see how such a huge displacement could happen by accident or stay unnoticed by experimenters.

I would say that a thermo-mechanical effect involving a simple linear displacement of some driver's part mass is rather well ruled-out as an explanation for the sustained part (40 s) and early after decay of the signal. Maybe some more complex classical explanation might still apply.

(1) by "drive" I consider the whole system mounted on the balance's arm and the arm itself

Yes, as discussed previously, the classical thermo-mechanical explanation would need to comprise:

1) Thermal instability (buckling) to explain the initial fast rise

plus

2) Thermal expansion (dielectric HD PE thermal expansion playing a crucial role), as proposed by Oak Ridge National Labs, to explain the later, slow rise, more steady force

Therefore, we need to closely examine the Oak Ridge National Lab thermal expansion explanation as to its validity. I understood Paul March as asking for this thermal expansion calculation to be made and critically examined it to rule it out.

See pages 6 through 11 of http://web.ornl.gov/~webworks/cppr/y2001/pres/111404.pdf

and pages 5 through 7 of http://www.otherhand.org/wp-content/uploads/2012/04/McKeever-theory-critique.pdf

J. H. Whealton, J. W. McKeever, M. A. Akerman, J. B. Andriulli*
National Transportation Research Center
2360 Cherahala Boulevard
Oak Ridge National Laboratory, Knoxville, TN 37932-6472
and
D. B. Hamilton, EE-32
Forrestal Building, MS 6A-116
U.S. Department of Energy, Washington, D.C. 20585-0121

Note:these papers also contain a rebuttal of the Mach Effect theory, which does not pertain to this thread.  Please let's stay on point here: let's discuss only the Oak Ridge thermal expansion explanation for the EM Drive (as relating to EM Drive Developments - related to space flight applications).  Any discussions of the Mach Effect should be conducted on its own thread (here:  http://forum.nasaspaceflight.com/index.php?topic=31037.460  )
Title: Re: EM Drive Developments - related to space flight applications - Thread 2
Post by: Star-Drive on 02/14/2015 03:21 pm
Rodal & Crew.

There is a major problem with your explanations of the copper frustum's "thrust" traces being strictly due to the frustum's large OD end-cap's immediate inward oil-canning to the left, see attached thermal expansion diagram, which is then followed by the copper frustum's longer-term thermal expansion to the left of its 0.024" thick copper cone material.  And this problem is that these thermal effect explanations for the generated torque pendulum (T.P.) movements are in the wrong direction to account for the observed thrust traces.  I.e., they are fighting the observed thrust traces, not adding to or creating them.

Now Newton's third law still states that for every action there is an equal and opposite reaction.  So when the copper frustum's large OD end-cap's prompt and inward oil canning action, followed by the slower frustum cone thermal expansions, they both push the copper frustum's Center of Mass (CoM) to the left as viewed from the front of the Eagleworks' vacuum chamber looking back at the test article and torque pendulum, while noting how the copper frustum is bolted on to the T.P..  These thermally induced actions to the left requires the torque pendulum's arm to move to the right to maintain the balance of the torque pendulum's arm in the lab's 1.0 gee gravity field, since we also use the Earth's g-field to help null the pendulum's movements. 

However the observed up-going in the Y-axis thrust traces, see attached 50W slide, require the T.P. to move to the left, whereas the thermal expansion induced left-going CoM actions require the T.P. to move to the right.  These thermal expansion actions should REDUCE and modify over time the magnitude of the observed thrust signal coming from the copper frustum's accelerations of the QV plasma to the right and out of the frustum.  And you can see this negative longer term frustum thermal drift by noting the downward going baseline slope of the thrust trace even after the RF power is removed from the copper frustum.

Best, Paul M.
Title: Re: EM Drive Developments - related to space flight applications - Thread 2
Post by: Rodal on 02/14/2015 03:29 pm
Rodal & Crew.

There is a major problem with your explanations of the copper frustum's "thrust" traces being strictly due to the frustum's large OD end-cap's immediate inward oil-canning to the left, see attached thermal expansion diagram, which is then followed by the copper frustum's longer-term thermal expansion to the left of its 0.024" thick copper cone material.  And this problem is that these thermal effect explanations for the generated torque pendulum (T.P.) movements are in the wrong direction to account for the observed thrust traces.  I.e., they are fighting the observed thrust traces, not adding to or creating them.

Now Newton's third law still states that for every action there is an equal and opposite reaction.  So when the copper frustum's large OD end-cap's prompt and inward oil canning action, followed by the slower frustum cone thermal expansions, they both push the copper frustum's Center of Mass (CoM) to the left as viewed from the front of the Eagleworks' vacuum chamber looking back at the test article and torque pendulum, while noting how the copper frustum is bolted on to the T.P..  These thermally induced actions to the left requires the torque pendulum's arm to move to the right to maintain the balance of the torque pendulum's arm in the lab's 1.0 gee gravity field, since we also use the Earth's g-field to help null the pendulum's movements. 

However the observed up-going in the Y-axis thrust traces, see attached 50W slide, require the T.P. to move to the left, whereas the thermal expansion induced left-going CoM actions require the T.P. to move to the right, which should REDUCE and modify over time the magnitude of the observed thrust signal coming from the copper frustum actions of accelerating QV plasma to the right and out of the frustum.  And you can see this negative longer term frustum thermal drift by noting the downward going baseline slope of the thrust trace even after the RF power is removed from the copper frustum.

Best, Paul M.

Paul March,

An active subject of discussion (all the way back to thread one) was to know which way the truncated cone actually moved in the NASA Eagleworks experiment.

I don't recall seeing anything in the Brady report concerning in which direction it moved: did it move towards the big diameter end ? or did it move towards the small diameter end?

To add to the confusion as to which way the truncated cone moves, there is the difficult-to-follow reports from Shawyer as to which way his truncated cone moved in different experiments (actually Shaywer's definition of thrust force seems to be in the opposite direction as NASA's definition of thrust force). (See :  http://www.emdrive.com/EmDriveForceMeasurement.pdf  ). 

So, could you please let us know which way did the truncated cone move in the NASA experiments? did it move toward the big diameter end  or towards the small diameter end? Do you define your force in the same direction as the movement ? (or do you define it opposite to the movement?)
.


If you have attempted to answer this in your previous message, please forgive my insistence, to answer it again as simply as follows:

1) In the NASA experiments the truncated cone's center of  mass moved towards the [  ? ] diameter end  (where ? stands for big or small)

2) In the NASA experiments, we at NASA Eagleworks define the thrust force direction to be in the  [? ] direction as the movement of the truncated cone's center of  mass  (where ? stands for same or opposite)
Title: Re: EM Drive Developments - related to space flight applications - Thread 2
Post by: Mulletron on 02/14/2015 03:59 pm
What about Cannae? It has completely different construction. Thrust was measured from it too. How does buckling explain Cannae?

The Cannae "null" also worked which put the inventor's theory of operation to bed. Most importantly, the null gave support for the importance of the dielectric and thus a clue to the true mechanism for operation.

When the dummy load (which efficiency converts all the rf to heat) was put in, the strong thrust signature disappeared. That also rules out spurious electromagnetic coupling by cables...etc.

The fact that thrust vanished without the dielectric under low power testing by Nasa combined with knowing that Shawyer's testing with high power and no dielectric shows that the relative permittivity of the material in the cavity is important. Air is dielectric too.

In my view, the thrust signal appears to be genuine and needs to be acknowledged. The results have held up under intense scrutiny. They have been replicated by others. After months of studying the test results and finding published scientific literature from experts supporting the notion that momentum can be gained from the QV.....This is real.
Title: Re: EM Drive Developments - related to space flight applications - Thread 2
Post by: aero on 02/14/2015 04:02 pm
@Rodal -
Its easy to overlook items when there is a lot of new data to evaluate, but Thrust is to the left. See attached, top center of the image, second line.
Title: Re: EM Drive Developments - related to space flight applications - Thread 2
Post by: Rodal on 02/14/2015 04:04 pm
@Rodal -
Its easy to overlook items when there is a lot of new data to evaluate, but Thrust is to the left. See attached.
I would appreciate getting an answer directly from Paul March to both of the following questions, (even if you think that it has already been answered with a diagram) as follows:

If you have attempted to answer this in your previous message, please forgive my insistence, to answer it again as simply as follows:

1) In the NASA experiments the truncated cone's center of  mass moved towards the [  ? ] diameter end  (where ? stands for big or small)

2) In the NASA experiments, we at NASA Eagleworks define the thrust force direction to be in the  [? ] direction as the movement of the truncated cone's center of  mass  (where ? stands for same or opposite)




@aero: "Thrust is to the left" does not answer this question at all, since the definition of thrust force is opposite for Shaywer (for example) than from standard sources, and I have no idea what convention March is using

Shawyer shows "thrust to the right" (towards the Big Diameter) for the NASA experiments while March shows "thrust to the left"  (towards the Small Diameter)

I think that the laws of physics should be the same for both Shawer's and NASA's experiments, hence it is important to answer the above questions (the way I carefully stated them).
Title: Re: EM Drive Developments - related to space flight applications - Thread 2
Post by: Rodal on 02/14/2015 04:16 pm
What about Cannae? It has completely different construction. Thrust was measured from it too. How does buckling explain Cannae? ...
Where are the dimensions, and materials, of the Cannae device tested at NASA Eagleworks so that this question can be answered numerically ?
Title: Re: EM Drive Developments - related to space flight applications - Thread 2
Post by: Mulletron on 02/14/2015 04:31 pm
We have Cannae dimensions. Page 6 of the anomalous thrust paper has some.
Title: Re: EM Drive Developments - related to space flight applications - Thread 2
Post by: Rodal on 02/14/2015 05:33 pm
We have Cannae dimensions. Page 6 of the anomalous thrust paper has some.
How can one can conduct a thermal instability calculation with this information ?:

Quote from: page 6 of Brady et.al.
Each Cannae test article is approximately 11 inches in diameter and 4-5 inches between the ends of the beam pipes, not counting beam pipe extensions or antenna mounts.

It does not even include the most important buckling parameter: the thickness of the test article.
Title: Re: EM Drive Developments - related to space flight applications - Thread 2
Post by: Flyby on 02/14/2015 05:42 pm
https://www.youtube.com/watch?v=57q3_aRiUXs

From the video that Shawyer made of his test configuration  , many years ago, it is clear that the cone is moving towards the small diameter.

Not sure to why he wanted to represent his findings in a drawing that resembles a rocket engine (action- reaction) where there clearly is no exhaust force.. it kinda puzzles me...

Let's hope Paul March gives a conclusive answer to that so we can leave that confusion behind us.

Btw, ever since the EMdrive got worldwide publicity, a few months ago, I've been trying to follow the different discussions pro and contra between "believers" and "non-believers". I find the discussions here, on the NASA forum, by far the most constructive discussion.
Having only a basic scientific background, it is indeed sometimes tough to follow the discussions between professionals when they venture into the higher levels of mathematics, radiation physics and quantum mechanics, but even with only a 50% understanding it is great to see things unfold.

Strangely enough it gives me the same degree of excitement, then when I watched the live feed on the discovery confirmation of the Higgs-boson. Maybe, just maybe, we are indeed on the edge of a new chapter in space exploration...

Fantastic what you people established here. Really!
It is inspiring to see the collaboration from different fields in action, almost in real time...

Not sure, but if I may do a prudent suggestion, but in the discussion about magnetic permeability (some pages ago), wouldn't it make more economical sense to use iron instead of metglas. I believe it would be far easier to shape a slab of iron then finding a plate metglas 2714a. On condition iron scales linear compared to metglas, an iron backplate would still perform roughy 10 times better then copper.
Enough to validate all discussions till now for a reasonable price... ?

Geert
Title: Re: EM Drive Developments - related to space flight applications - Thread 2
Post by: Rodal on 02/14/2015 05:49 pm
https://www.youtube.com/watch?v=57q3_aRiUXs

From the video that Shawyer made of his test configuration  , many years ago, it is clear that the cone is moving towards the small diameter.

Not sure to why he wanted to represent his findings in a drawing that resembles a rocket engine (action- reaction) where there clearly is no exhaust force.. it kinda puzzles me...

Let's hope Paul March gives a conclusive answer to that so we can leave that confusion behind us.

Btw, ever since the EMdrive got worldwide publicity, a few months ago, I've been trying to follow the different discussions pro and contra between "believers" and "non-believers". I find the discussions here, on the NASA forum, by far the most constructive discussion.
Having only a basic scientific background, it is indeed sometimes tough to follow the discussions between professionals when they venture into the higher levels of mathematics, radiation physics and quantum mechanics, but even with only a 50% understanding it is great to see things unfold.

Strangely enough it gives me the same degree of excitement, then when I watched the live feed on the discovery confirmation of the Higgs-boson. Maybe, just maybe, we are indeed on the edge of a new chapter in space exploration...

Fantastic what you people established here. Really!
It is inspiring to see the collaboration from different fields in action, almost in real time...

Not sure, but if I may do a prudent suggestion, but in the discussion about magnetic permeability (some pages ago), wouldn't it make more economical sense to use iron instead of metglas. I believe it would be far easier to shape a slab of iron then finding a plate metglas 2714a. On condition iron scales linear compared to metglas, an iron backplate would still perform roughy 10 times better then copper.
Enough to validate all discussions till now for a reasonable price... ?

Geert

Welcome to the forum and thanks for the great post  :)

You are correct that we can ascertain a direction of movement from the above Shawyer video.

Unfortunately (see image below) Shawyer's presentation muddles the picture because, that is the "SPR Demonstrator Engine" which Shawyer says had forces (and hence movement ?) in both directions towards the small diameter and also towards the big diameter  :(

Also according to Shawyer, the Chinese EM Drive and the Shawyer "Flight Thruster" move in opposite direction to all the following EM Drives: the NASA EM Drive, to the Cannae (both room temperature and superconducting) and to the SPR tapered cavity with dielectric.

I look forward to Paul March's answer to the two questions I posed above.
Title: Re: EM Drive Developments - related to space flight applications - Thread 2
Post by: aero on 02/14/2015 05:55 pm
Do we know what the Eagleworks vacuum chamber is made of? I think I remember it being a Stainless Steel chamber but I can't find confirmation of that at the moment. It is kind of important to me because the evanescent fields from my simulation runs overlap the edges of the vacuum chamber. That means to me, that I need to add the vacuum chamber to my meep model. And expect different answers as a result.
Title: Re: EM Drive Developments - related to space flight applications - Thread 2
Post by: Rodal on 02/14/2015 05:57 pm
Do we know what the Eagleworks vacuum chamber is made of? I think I remember it being a Stainless Steel chamber but I can't find confirmation of that at the moment. It is kind of important to me because the evanescent fields from my simulation runs overlap the edges of the vacuum chamber. That means to me, that I need to add the vacuum chamber to my meep model. And expect different answers as a result.
That's an excellent question, @aero, thanks for bringing it up.  I agree with your reason for asking it.

It is likely to be either Stainless Steel or Aluminum

 Look forward to March's answer.
Title: Re: EM Drive Developments - related to space flight applications - Thread 2
Post by: TMEubanks on 02/14/2015 07:22 pm
Hello;

I have been following this for a while, but this is my first post. (I have corresponded with José Rodal about this in the past.) My congratulations on a spirited and informative discussion of some thorny issues.

It is apparent that the E&M waves on the outside of the drive extend far enough to interact with the walls of the vacuum chamber (not to mention the support, pendulum and other mechanisms inside the chamber). Thus, to verify (or rule out) the reality of the measured thrust, it will be crucial to calculate the force caused by these interactions. As I was reading through the forum, this point impressed itself upon me, and I was going to ask if it has been done but, of course, if we don't know the material of the chamber, it hasn't.

MEEP should be adequate to do this, as this is entirely a conventional physics effect, and I would encourage someone to take this on.
Title: Re: EM Drive Developments - related to space flight applications - Thread 2
Post by: Rodal on 02/14/2015 07:46 pm
Hello;

I have been following this for a while, but this is my first post. (I have corresponded with José Rodal about this in the past.) My congratulations on a spirited and informative discussion of some thorny issues.

It is apparent that the E&M waves on the outside of the drive extend far enough to interact with the walls of the vacuum chamber (not to mention the support, pendulum and other mechanisms inside the chamber). Thus, to verify (or rule out) the reality of the measured thrust, it will be crucial to calculate the force caused by these interactions. As I was reading through the forum, this point impressed itself upon me, and I was going to ask if it has been done but, of course, if we don't know the material of the chamber, it hasn't.

MEEP should be adequate to do this, as this is entirely a conventional physics effect, and I would encourage someone to take this on.

Welcome to the forum, Marshall.  It is great to have an MIT Astrophysicist in this thread who has a main-belt asteroid named after him.  I hope you stay here for a while and continue giving us your guidance, suggestions and comments :)
Title: Re: EM Drive Developments - related to space flight applications - Thread 2
Post by: Star-Drive on 02/14/2015 09:06 pm
All:

Sorry I didn't make the time to participate in this ME-Drive forum for the last 6-to-8 months up, but I will try to catch up with everyone else in due course.  That said lets try to answer the questions that popped up since my morning post.

1.  I was not the lead author for the Eagleworks' 2014 AIAA/JPC paper and in fact I only supplied pictures and data for same during that period because Dr. White thought that my time was best spent in the lab gathering data instead of report writing.  Thus some of the details that Dr. Rodal is looking for may have been lost or garbled in the report writing by the others on the author list.

2.0  The thrust vector for the four resonant modes examined in detail, (the cavity's fundamental TM010, TE012, TM211 & TM212 for our copper frustum is normally in the frustum's large OD to small OD direction for most, but not all the E&M resonant modes checked.  However, one can also reverse this thrust vector for this copper frustum by just changing which excited resonant mode is used and/or mounting the dielectric discs at the large OD end of the cavity instead of the small OD end, see attached resonant mode map.  Sorry, but a one size fits all solution to this EM-Drive thrust direction is not available in this venue because of the importance of the ExB phase relationship of the expressed Lorentz forces between the excited E&M fields and the possible dielectric and QV plasma flow phenomenon that may be at work in each resonant mode expressed.  That is why this type of E&M thruster is so hard to get a handle on, for there are far too many degrees of freedom in the system to track let alone directly control.

3. The Eagleworks vacuum chamber's main body is made from 304L stainless steel while its swing out door is made from aluminum.  Most of the nuts and bolts in the vacuum chamber are also made from 18-8, 304 or 316 stainless steel alloys. 

Now to try to answer Dr. Rodal's specific questions:

"1) In the NASA experiments the truncated cone's center of  mass moved towards the [  ? ] diameter end  (where ? stands for big or small)"

For the TE012 and TM212 excited resonant modes, our copper frustum's center of mass moved toward the small OD end of the frustum when RF power was applied to the copper frustum.

"2) In the NASA experiments, we at NASA Eagleworks define the thrust force direction to be in the  [? ] direction as the movement of the truncated cone's center of  mass  (where ? stands for same or opposite)"

For just the TE012 & TM212 excited resonant modes, the thrust force direction AKA thrust vector was observed to be in the same direction as the movement of the frustum's center of mass when RF power was applied to the frustum's magnetic loop antenna.

If I missed a question along the way keep asking, but I'll be in and out of the house for the rest of the day, so I may not get to answer them until late this evening or tomorrow afternoon USA based CST.

Best, Paul M.
Title: Re: EM Drive Developments - related to space flight applications - Thread 2
Post by: Rodal on 02/14/2015 09:10 pm
All:

Sorry I didn't make the time to participate in this ME-Drive forum for the last 6-to-8 months up, but I will try to catch up with everyone else in due course.  That said lets try to answer the questions that popped up since my morning post.

1.  I was not the lead author for the Eagleworks' 2014 AIAA/JPC paper and in fact I only supplied pictures and data for same during that period because Dr. White thought that my time was best spent in the lab gathering data instead of report writing.  Thus some of the details that Dr. Rodal is looking for may have been lost or garbled in the report writing by the others on the author list.

2.0  The thrust vector for the four resonant modes examined in detail, (the cavity's fundamental TM010, TE012, TM211 & TM212 for our copper frustum is normally in the frustum's large OD to small OD direction for most, but not all the E&M resonant modes checked.  However, one can also reverse this thrust vector for this copper frustum by just changing which excited resonant mode is used and/or mounting the dielectric discs at the large OD end of the cavity instead of the small OD end, see attached resonant mode map.  Sorry, but a one size fits all solution to this EM-Drive thrust direction is not available in this venue because of the importance of the ExB phase relationship of the expressed Lorentz forces between the excited E&M fields and the possible dielectric and QV plasma flow phenomenon that may be at work in each resonant mode expressed.  That is why this type of E&M thruster is so hard to get a handle on, for there are far too many degrees of freedom in the system to track let alone directly control.

3. The Eagleworks vacuum chamber's main body is made from 304L stainless steel while its swing out door is made from aluminum.  Most of the nuts and bolts in the vacuum chamber are also made from 18-8, 304 or 316 stainless steel alloys. 

Now to try to answer Dr. Rodal's specific questions:

"1) In the NASA experiments the truncated cone's center of  mass moved towards the [  ? ] diameter end  (where ? stands for big or small)"

For the TE012 and TM212 excited resonant modes, our copper frustum's center of mass moved toward the small OD end of the frustum when RF power was applied to the copper frustum.

"2) In the NASA experiments, we at NASA Eagleworks define the thrust force direction to be in the  [? ] direction as the movement of the truncated cone's center of  mass  (where ? stands for same or opposite)"

For just the TE012 & TM212 excited resonant modes, the thrust force direction AKA thrust vector was observed to be in the same direction as the movement of the frustum's center of mass when RF power was applied to the frustum's magnetic loop antenna.

If I missed a question along the way keep asking, but I'll be in and out of the house for the rest of the day, so I may not get to answer them until late this evening or tomorrow afternoon USA based CST.

Best, Paul M.

Thank you for the very clear answers !

and thanks again for two outstanding contributions just on the last few months:

1) The first time (to my knowledge) that an EM Drive is tested under hard vacuum, with experimentally confirmed  thrust (at a thrust/PowerInput level amply exceeding the one for a photon rocket).

2) The first time (to my knowledge) that the actual mode shape (TM21 at the big diameter) of an EM Drive is experimentally confirmed.
Title: Re: EM Drive Developments - related to space flight applications - Thread 2
Post by: Star-Drive on 02/14/2015 09:25 pm
PS:

The Cannae pill box cavity with asymmetric RF feeds was made from 2.0mm thick spun copper and 1.0 inch ID copper plumbing pipe and fittings. 

Correction: The thrust vector for this Cannae test article for its ~928MHz, TM010 resonant mode was always along its centerline toward its long 1.0" ID  diameter tube end where the coaxial E-field probe RF power input antenna with a Teflon dielectric cylinder used for 50 ohm impedance matching was mounted.  The opposite end of this Cannae pill box resonant cavity had a shorter 1.0" ID copper tube that housed the RF sense antenna.   

And this thruster was mounted almost on top of the torque pendulum arm's centerline, so most of its left/right thermal expansions should have canceled out.

Best, Paul M.
Title: Re: EM Drive Developments - related to space flight applications - Thread 2
Post by: Rodal on 02/14/2015 09:58 pm
....
If I missed a question along the way keep asking, but I'll be in and out of the house for the rest of the day, so I may not get to answer them until late this evening or tomorrow afternoon USA based CST.

...
@aero had this question,

Quote from: @aero
please look at this model and tell me if your read of Paul March's description of the cavity joins are as shown, or is there a cone flange pinched under the Teflon gasket which is held in place by a thicker copper ring.

answering it would help better model evanescent waves with MEEP.  (Please click on the original picture inserted in his message:  http://forum.nasaspaceflight.com/index.php?topic=36313.msg1331345#msg1331345, to better appreciate the geometry he is interested in)

(http://forum.nasaspaceflight.com/index.php?action=dlattach;topic=36313.0;attach=768546;image)
Title: Re: EM Drive Developments - related to space flight applications - Thread 2
Post by: Mulletron on 02/14/2015 10:10 pm
The fact that it has reverse too pretty much clinches it. That speaks volumes.

(https://encrypted-tbn0.gstatic.com/images?q=tbn:ANd9GcQaH7F0WjOYKzk6tzWMguQUCEVZ8XBeBNg0F_kpNmYfvMmQIbZluwUzt4k)
(https://encrypted-tbn3.gstatic.com/images?q=tbn:ANd9GcSqY-ef16FoCA8if0gj8HzYI0wwOQ4NTT5R2Be-0gxw6QlhuM03EEKil7A_PA)
Title: Re: EM Drive Developments - related to space flight applications - Thread 2
Post by: Rodal on 02/14/2015 10:59 pm
The fact that it has reverse too pretty much clinches it. That speaks volumes.

(https://encrypted-tbn0.gstatic.com/images?q=tbn:ANd9GcQaH7F0WjOYKzk6tzWMguQUCEVZ8XBeBNg0F_kpNmYfvMmQIbZluwUzt4k)
(https://encrypted-tbn3.gstatic.com/images?q=tbn:ANd9GcSqY-ef16FoCA8if0gj8HzYI0wwOQ4NTT5R2Be-0gxw6QlhuM03EEKil7A_PA)
You are right that "it speaks volumes", I agree, the good news about the EM Drive just keeps on coming from Eagleworks. but before "clinching it" we still have to address the electromagnetic field interaction, e.g. evanescent wave interaction with the vacuum chamber and other fixed components, as the ExB field phase relationship changes, don't we ?
Title: Re: EM Drive Developments - related to space flight applications - Thread 2
Post by: SleeperService on 02/14/2015 11:29 pm
Watching closely..
I think a zero-gee experiment is what we need.
Mr Musk would be up for this sort of thing as a payload for one of his re-used rockets. For free.
Can we come up with a self-contained payload to test this?
If not, why not?
Title: Re: EM Drive Developments - related to space flight applications - Thread 2
Post by: Mulletron on 02/14/2015 11:51 pm
I can think of 3 ways to test the evanescent wave theory. 1) Is the measured thrust the same with the chamber door open and closed? 2) Is the thrust still there when the test article is rolled out of the chamber. Not sure if 2 is possible.....3) Change the conditions near the resonant cavity; like wrap the thing in thick foam and then wrap all that with foil, see what the thrust does.

I know that evanescent modes won't couple with something that is not capable of accepting the resonant mode; they are by definition, non propagating, so that likely rules out the support structures. Aluminum beams aren't resonant structures or transmission lines. This is how wireless phone charging systems are safe to be around. They only can couple with the receiving antenna, not other stuff or people. Also the effect is evident in those videos I posted way back about quantum tunneling between prisms pushed together.
 
At the same time, it probably does not rule out the chamber. From what I've read, evanescent modes are strongest within 1/3 of a wavelength and falls off exponentially with the distance, so how far away is the chamber from any point of the test article? I dropped the evanescent wave thing long ago because it is just too far fetched IMHO and proving it or disproving it didn't seem possible at the time. What really ruled it out for me was that only the H (magnetic) component of ExH can get through the cavity walls, which means it could never propagate. Aero seems close to an answer, which is nice.

http://milesmathis.com/evane.pdf
http://en.wikipedia.org/wiki/Resonant_inductive_coupling
http://forum.nasaspaceflight.com/index.php?topic=29276.msg1284471#msg1284471 way back machine
Title: Re: EM Drive Developments - related to space flight applications - Thread 2
Post by: SleeperService on 02/14/2015 11:51 pm
I'm thinking an xU cubesat with a big battery. Spin stabilized even at 50 micro newtons we would see positive results within an hour, tracked from the ground.
Title: Re: EM Drive Developments - related to space flight applications - Thread 2
Post by: Mulletron on 02/15/2015 12:19 am
http://en.m.wikipedia.org/wiki/Marin_Solja%C4%8Di%C4%87

Ask him. He's an evanescent wave expert at MIT.

Below are the approximate resonant modes of the vacuum chamber. Not taking into account perturbation. Doesn't look like the vacuum chamber can accept the modes within the test article.
Title: Re: EM Drive Developments - related to space flight applications - Thread 2
Post by: SleeperService on 02/15/2015 12:36 am
I'm thinking an xU cubesat with a big battery. Spin stabilized even at 50 micro newtons we would see positive results within an hour, tracked from the ground.
I hoped to be experimental about this.
Experiment trumps theory every time.
With what we have, lets have an experiment in zero-g
Title: Re: EM Drive Developments - related to space flight applications - Thread 2
Post by: RotoSequence on 02/15/2015 12:47 am
I'm thinking an xU cubesat with a big battery. Spin stabilized even at 50 micro newtons we would see positive results within an hour, tracked from the ground.
I hoped to be experimental about this.
Experiment trumps theory every time.
With what we have, lets have an experiment in zero-g

I'd rather wait to see a confidence building ground experiment before committing to something of that scale. It shouldn't be too hard (or too expensive) to hit the target thrust levels for third party university verification, and it would almost certainly be much less expensive.
Title: Re: EM Drive Developments - related to space flight applications - Thread 2
Post by: Mulletron on 02/15/2015 12:47 am
I'm thinking an xU cubesat with a big battery. Spin stabilized even at 50 micro newtons we would see positive results within an hour, tracked from the ground.
I hoped to be experimental about this.
Experiment trumps theory every time.
With what we have, lets have an experiment in zero-g
http://cannae.com/2-uncategorised/48-cubesat
Title: Re: EM Drive Developments - related to space flight applications - Thread 2
Post by: SleeperService on 02/15/2015 12:52 am
Someone have a word with Elon.... He must know a thing or two... The Cannae stuff seems like a scam...
Title: Re: EM Drive Developments - related to space flight applications - Thread 2
Post by: Mulletron on 02/15/2015 01:05 am
Someone have a word with Elon.... He must know a thing or two... The Cannae stuff seems like a scam...

Why?
Title: Re: EM Drive Developments - related to space flight applications - Thread 2
Post by: SleeperService on 02/15/2015 01:07 am
Put it this way.
We are trying to determine whether or not we can get thrust out of microwave energy.
Endless discussions...
NASA Eagleworks.
I agree!! But we need an experiment to prove these things.
In Zero-g we trust!

Title: Re: EM Drive Developments - related to space flight applications - Thread 2
Post by: Mulletron on 02/15/2015 01:09 am
Given that thrust follows the dielectric and can be controlled at will; it is clear that this is no artifact.
Title: Re: EM Drive Developments - related to space flight applications - Thread 2
Post by: SleeperService on 02/15/2015 01:11 am
Cool. Elon should be all over this then...
Title: Re: EM Drive Developments - related to space flight applications - Thread 2
Post by: ThinkerX on 02/15/2015 01:15 am
Ok...we are under direct mandate here from Chris to keep this thread geared towards 'space based applications.'

With that in mind perhaps its time to revamp the 'Mulletron Mission to Saturn' and combine that with the vacuum test results? 

Yes, the vacuum tests are not as great as hoped for, but they are a place to start from, and multiple improvements / ways of boosting this devices efficiency have been suggested.   So maybe a 'Mulletron Mission' using the unmodified vacuum test info, and a second based on...'reasonable best guess' improvements?

Also, with all the images on the web, there must be one out there that captures the essence of the Mulletron craft.
Title: Re: EM Drive Developments - related to space flight applications - Thread 2
Post by: Cinder on 02/15/2015 01:54 am
Cool. Elon should be all over this then...

Probably not while it's still this early in TRL.  There's not much to lose in waiting for what seem to be imminent new empirical data.
Title: Re: EM Drive Developments - related to space flight applications - Thread 2
Post by: Notsosureofit on 02/15/2015 01:56 am
Given that thrust follows the dielectric and can be controlled at will; it is clear that this is no artifact.

Being that I just probably wasted the entire evening trying to find a way to incorporate the dielectric into the tapered cavity calculation on scribble sheets, you just stimulated the thought that one only needs a cylindrical cavity plus dielectric to get an asymmetric dispersion relation which should be relatively easy to calculate.  (he says while nodding off)
Title: Re: EM Drive Developments - related to space flight applications - Thread 2
Post by: SleeperService on 02/15/2015 02:03 am
Elon should employ Rodal, Mullertron, NotSureOfIt et al to examine anomalous thrust.
To not do so would be bad business.
Title: Re: EM Drive Developments - related to space flight applications - Thread 2
Post by: RotoSequence on 02/15/2015 02:15 am
Elon should employ Rodal, Mullertron, NotSureOfIt et al to examine anomalous thrust.
To not do so would be bad business.

Elon's already weighed in on the subject (https://twitter.com/elonmusk/status/559554023544483841):

Quote from: Elon Musk
‏@elonmusk
If u saw @TheSimpsons and wonder why @SpaceX doesn't use an electric rocket to reach orbit, it is cuz that is impossible

Title: Re: EM Drive Developments - related to space flight applications - Thread 2
Post by: SleeperService on 02/15/2015 02:21 am
And whatever you may call it.. The jury is out on if it is impossible...
Title: Re: EM Drive Developments - related to space flight applications - Thread 2
Post by: M_Puckett on 02/15/2015 02:59 am
The fact that it has reverse too pretty much clinches it. That speaks volumes.

(https://encrypted-tbn0.gstatic.com/images?q=tbn:ANd9GcQaH7F0WjOYKzk6tzWMguQUCEVZ8XBeBNg0F_kpNmYfvMmQIbZluwUzt4k)
(https://encrypted-tbn3.gstatic.com/images?q=tbn:ANd9GcSqY-ef16FoCA8if0gj8HzYI0wwOQ4NTT5R2Be-0gxw6QlhuM03EEKil7A_PA)

So you really can put the silly thing into reverse?  Don't tell Dodgers.
Title: Re: EM Drive Developments - related to space flight applications - Thread 2
Post by: Star-Drive on 02/15/2015 04:17 am
Folks:

When we can replicate the Chinese 1.0 Newton/kWe demonstrated performance levels that they reported in their 2012 and 2013 tests reports, see attached example from same, perhaps that will be the time to go to the power brokers to see what deal can be had in developing this technology into what it could become. 

In the meantime, lets ask why 60 watts of relatively harmonic free sine-wave RF power at the 1,937.118 MHz AKA the TM212 resonant frequency in this copper frustum cavity, can only generate a paltry ~60uN, whereas the Chinese claimed to have produce 160,000uN using just ~150 watts of 2,450 MHz RF signals from a magnetron?  The magnetron RF signal source that is anything but a pure sine-wave generator, that instead has a modulated FM bandwidth of at least +/-30 MHz that is also concurrently amplitude modulated (AM) with thermal electron noise.  IMO maximizing the time rate of change of the E&M energy spectra working in the resonant cavity, along with the optimization of the TBD process that converts this time varying energy density into a unidirectional force pointed in just one controllable direction will hand us the keys to the solar system and beyond. 

BTW, I have no doubt now that this quantum vacuum derived propulsion system will be able to meet and ultimately surpass my conjectured WarpStar-I concept vehicle performance that I wrote about in my STAIF-2007 paper based on Woodward's Mach Lorentz Thrusters (MLT) of the day.  A vehicle that could go from the surface of the Earth to the surface of the Moon with a crew of two and six passengers with luggage in under four hours and then return to the surface of the Earth in another 4 hours with the same payload using just one load of H2/O2 fuel cell derived electrical power assuming 500-to-1,000 N/kWe efficiency MLTs or Q-Thrusters.  And yes, I know that's a mighty big leap from the 1.0uN/Watt we currently have demonstrated at the Eagleworks Lab, but if Dr. White's QVF/MHD conjecture is anywhere close to reality, it will be doable, at least in the long term.

Best, Paul M.

 
Title: Re: EM Drive Developments - related to space flight applications - Thread 2
Post by: aero on 02/15/2015 05:08 am
@Paul March,
It is kind of important to my results if you could confirm that the Teflon Rubber gaskets are installed as illustrated in the attached model. Click on the image, it will expand so you can see detail - but of course it is mostly black so use the sliders to move around to find a corner. :)

Right now, I am using a 12.5 mm coaxial dipole antenna at the inner face of the dielectric disk. I know you used a loop of some sort. How much do you think this difference matters considering that I am running a digital model?

It is also important that I correctly model the width of the Teflon Rubber gasket filled gap. You wrote that the gasket was .064." Was that after installed, or did you compress it when you tightened down the retaining ring. If so, what would you estimate the actual distance is, between the copper cone and copper base plate, as installed? I know that sounds like a nonsense question, but my simulation shows thrust force is dramatically sensitive to just a small changes in the gap width. I'd like for my model to be as close as is possible to your Copper Kettle thruster.

My final question (I hope) re. the gasket is, "Do you know what the dielectric constant is for the actual Teflon Rubber that you used?" (Did your supplier document it, perhaps.) I find values ranging from 2.1 to 2.5 and while force is not very sensitive to this value, it does have an effect.

And while I'm at it, I read that the vacuum chamber is 30 inches by 36 inches, diameter by length. Is that inside or outside dimensions?
Title: Re: EM Drive Developments - related to space flight applications - Thread 2
Post by: Mulletron on 02/15/2015 07:48 am
Folks:

In the meantime, lets ask why 60 watts of relatively harmonic free sine-wave RF power at the 1,937.118 MHz AKA the TM212 resonant frequency in this copper frustum cavity, can only generate a paltry ~60uN, whereas the Chinese claimed to have produce 160,000uN using just ~150 watts of 2,450 MHz RF signals from a magnetron?  The magnetron RF signal source that is anything but a pure sine-wave generator, that instead has a modulated FM bandwidth of at least +/-30 MHz that is also concurrently amplitude modulated (AM) with thermal electron noise. 


Taking a critical look at this question, and knowing that the spectral shape of a magnetron looks like (see below) compared to a CW spike. It seems evident that a CW spike isn't the best waveform to use if you want to maximize thrust. Dollars to donuts says the Chinese are making full use of the available bandwidth of their resonant cavity by using that noisy magnetron. Magnetrons have lots of phase noise too. You can't easily use them on phased array radars because of that for example.

Now to put this idea to test, Q: What is the bandwidth of the resonant cavity and what is the 90 percent power bandwidth of the signal you are driving it with? What kind of sig gen are you using? Can it do FM? Can you do any advanced waveforms like a PSK waveform? Do you have a way to produce wideband noise or a spread spectrum carrier for your testing? Can you do any waveforms like at the bottom?

Also during researching other possible theories which could explain Emdrive we found ample literature stating that molecules acquire a kinetic momentum during the switching of the magnetic field as a result of its interaction with the vacuum field. If correct, that may well be a very significant lead. So that raises the question, how does one increase the switching rate? What about phase shifting? http://en.wikipedia.org/wiki/Phase-shift_keying

Phase shifting seems important.
https://www.viasat.com/files/assets/web/datasheets/EBEM_MD-1366_043_web.pdf
One of these driving your amp would be helpful. They go up to 2ghz.

(http://www.zdnet.com/i/story/60/03/000578/noise-from-microwave.png)
(https://encrypted-tbn2.gstatic.com/images?q=tbn:ANd9GcTVRN3U7Ai7Zbu56xVIVRQhS_nMGHQFi5EjSx1BFaxm8GZOlY9Q)
(http://www.propagation.gatech.edu/ECE6390/project/Fall2008/DipoleDetectors/Di_Pole%20Detectors/images/DataLink_Spectrum.JPG)
(http://www.cse.iitk.ac.in/users/dheeraj/cs425/fig.lec01/bpsk.gif)
Title: Re: EM Drive Developments - related to space flight applications - Thread 2
Post by: Star One on 02/15/2015 09:51 am
When the time comes do you think there will be any difficulty in getting papers on this published?
Title: Re: EM Drive Developments - related to space flight applications - Thread 2
Post by: Rodal on 02/15/2015 01:43 pm
Folks:

In the meantime, lets ask why 60 watts of relatively harmonic free sine-wave RF power at the 1,937.118 MHz AKA the TM212 resonant frequency in this copper frustum cavity, can only generate a paltry ~60uN, whereas the Chinese claimed to have produce 160,000uN using just ~150 watts of 2,450 MHz RF signals from a magnetron?  The magnetron RF signal source that is anything but a pure sine-wave generator, that instead has a modulated FM bandwidth of at least +/-30 MHz that is also concurrently amplitude modulated (AM) with thermal electron noise. 


Taking a critical look at this question, and knowing that the spectral shape of a magnetron looks like (see below) compared to a CW spike. It seems evident that a CW spike isn't the best waveform to use if you want to maximize thrust. Dollars to donuts says the Chinese are making full use of the available bandwidth of their resonant cavity by using that noisy magnetron. Magnetrons have lots of phase noise too. You can't easily use them on phased array radars because of that for example.

...
I agree with Mulletron that the answer to Paul March's question is that it is much more effective to have a distributed power spectral density than the power concentrated at a single frequency spike.  When the natural frequency changes in an unpredictable manner, it is much more effective to have a distributed power spectral density of excitation (it is the power spectral density ( http://en.wikipedia.org/wiki/Spectral_density#Power_spectral_density ) over the spectrum of changing natural frequencies that matters).

The reason for this is that (as has been verified by Prof. Juan Yang in China by inserting thermocouples at different places in the EM Drive) the EM Drive is subjected to a very non-uniform temperature distribution, with the temperature increasing with time, that results in significant non-uniform thermal expansion of the EM Drive, and therefore the natural frequencies must shift with temperature (and therefore shift with time as the temperature changes with time) as the EM Drive expands non-uniformly with time.  Therefore, having the power concentrated at a single frequency spike (NASA) is bound to be non-efficient as the resonant frequency changes with time, the EM Drive is going to move out of resonance even if one happens to excite it at the correct frequency to start with.  The COMSOL calculations do not provide the natural frequency to enough precision within the extremely narrow bandwidth of a high Q resonance (the higher the Q, the narrower the resonant bandwidth) for NASA to know exactly the natural frequency for a given mode shape.  More importantly, the COMSOL calculations do not provide the information needed for NASA to know how to shift the frequency with time, as the EM Drive thermally expands non-uniformly to stay at peak resonance.

This is evident from the very low Q's reported by NASA (7K to 22K) compared with the Chinese, who report a Q=117K

Quote from: Juan Yang
the resonant frequency and quality factor of the independent microwave resonator system are 2.44895 GHz and 117495.08 respectively

Compare this with NASA's reported Q:

Mode   Frequency (MHz)  Quality Factor, Q   Input Power (W)  Mean Thrust (μN)   Medium      Efficiency(uN/W)
TE012     1880.4               22000                         2.6                55.4                   Air           21
TM2112  1932.6                 7320                       16.9                91.2                   Air             5
TM2112  1936.7               18100                       16.7                50.1                   Air             3
TM212    1937.115             6726                       50                   66                      Vacuum      1

NASA's reported Q for the vacuum experiment is a meager Q = 6726, which is 17 times smaller than the Chinese reported Q = 117495.

Also note that the most efficient mode reported by NASA Eagleworks is the Transverse Electric mode which gave a Mean Thrust of 55 uN with only 2.6 Watts.



The Chinese also report that they used the Transverse Electric mode



Instead, NASA Eagleworks has been running most of the experiments in the Brady report in the Transverse Magnetic mode, and the vacuum experiment also in the Transverse Magnetic mode, which NASA's own data (see above) shows to be the most inefficient mode.

Why is NASA running the vacuum experiment in the most inefficient mode (Transverse Magnetic) rather than the most efficient mode (Transverse Electric) ?  Because they report difficulties in tuning the EM Drive under the Transverse Electric mode.

Quote from: Brady et.al page 17
Prior to the TM211 evaluations, COMSOL® analysis indicated that the TE012 was an effective thrust generation mode for the tapered cavity thruster being evaluated, so this mode was explored early in the evaluation process. Figure 22 shows a test run at the TE012 mode with an operating frequency of 1880.4 MHz. The measured quality factor was ~22,000, with a COMSOL prediction of 21,817. The measured power applied to the test article was measured to be 2.6 watts, and the (net) measured thrust was 55.4 micronewtons. With an input power of 2.6 watts, correcting for the quality factor, the predicted thrust is 50 micronewtons. However, since the TE012 mode had numerous other RF modes in very close proximity, it was impractical to repeatedly operate the system in this mode, so the decision was made to evaluate the TM211 modes instead.

Why does NASA have difficulties running the EM Drive in the more efficient mode (the Transverse Electric mode) ? Because the most efficient mode results in greater shifting of its natural frequency with time.  Hence I agree with Mulletron that instead of having the power concentrated at a frequency, for a problem where we know that the natural frequency of the EM Drive changes with time in a difficult to calculate and predict (with enough precision) manner, the best solution is to have the power distributed over a wider spectrum of frequencies, as done by Prof. Juan Yang in China.

(http://forum.nasaspaceflight.com/index.php?action=dlattach;topic=36313.0;attach=778913;image)
Title: Re: EM Drive Developments - related to space flight applications - Thread 2
Post by: Rodal on 02/15/2015 02:14 pm
As discussed in these previous posts:

http://forum.nasaspaceflight.com/index.php?topic=29276.msg1293800#msg1293800


http://forum.nasaspaceflight.com/index.php?topic=29276.msg1293809#msg1293809

Resonance experiment on a microwave resonator system
Shi Feng Yang Juan Tang Ming-Jie Luo Li-Tao Wang Yu-Quan
(College of Astronautics, Northwestern Polytechnic University, Xi’an 710072, China)
Acta Phys. Sinica Vol. 63, No. 15 (2014)
(Attached below as the last attachment)


Quote
Abstract
A microwave resonator system is made, which has a tapered resonant cavity, a microwave source, and a transmission device. Because of the electromagnetic pressure gradient on the tapered resonant cavity, a net electromagnetic force along the axis of the cavity may be observed, which is needed to verify experimentally the use of the independent microwave resonator system. It is also needed to keep the independent microwave resonator system in resonating state, which is the important procedure to demonstrate the possibility of net electromagnetic force. Thus, a low-signal resonating experiment on the tapered resonant cavity combined with resonating parts is completed to accurately find out the resonant frequency of 2.45 GHz and to analyze the influence of temperature on the resonant state. Experimental result shows that the resonant frequency and quality factor of the independent microwave resonator system are 2.44895 GHz and 117495.08 respectively. When the temperature of the tapered resonant cavity wall rises, the resonant frequency will be decreased and the quality factor changed separately.

Notice that China's Prof. Juan Yang shows the force "F" directed towards the small base of the truncated cone. The same end (the small end) that gets heated up the most and gets heated much faster in the Chinese experiment: the small end..   

Notice that the increase of temperature vs. time (31 deg C = 56 deg F increase in 40 seconds) of this truncated cone is much higher than the increase in temperature reported by NASA (this is expected because the Chinese run their experiment with higher input power than NASA), because they run their experiment in Transverse Electric rather than Transverse Magnetic mode,  and because the Chinese measured the temperature at the correct location: on the inside of the EM Drive with thermocouples, while NASA reports the temperature measured by an IR camera on the outside surface of FR4 fiber-reinforced epoxy, which has very low thermal diffusivity and hence masks the true inner temperature.

The Chinese report that they run their experiment in Transverse Electric (TE) modes.

The Chinese paper also repeatedly states that

Quote
"the electromagnetic field intensity calculated in the vicinity of the axis is larger, therefore the center of the small end has a higher thermal energy, thus heating quickly,"

....

"we measured the cavity surface surface temperature at different locations as shown in Figure 11 with a thermocouple.   [Fig. 11, which I attach below shows the positions of the thermocouples noted as "1" to "6"] Temperature measurements at different points vs.  time, are shown in FIG 12 [Fig 12 which I attach below shows the temperature vs time at the 6 different thermocouple locations]. The temperature at the center of small end [thermocouple #1] first began rising rapidly."

Notice that the temperature of the small end (thermocouple #1) rises much more than the temperature of the big end (thermocouple #6).

Notice that the rise of the temperature vs time at the center of the small end is nonlinear (the upper temperature curve) while the temperature measured at the other locations show fairly linear behavior.


Clearly, there is a large temperature difference between the small end (thermocouple #1) and the large end (thermocouple #6)

Maximum temperature is reached at the center of the small end.

The heating profile is completely different from the one shown by Egan. 
Title: Re: EM Drive Developments - related to space flight applications - Thread 2
Post by: Notsosureofit on 02/15/2015 02:38 pm
FYI for no good reason, here what I get w/o dielectric

Mode   Frequency (MHz)  Quality Factor, Q   Input Power (W)  Mean Thrust (μN)   Calc
TE012  1880.4                  22000                  2.6                       55.4                        16.9
TM212  1932.6                   7320                  16.9                      91.2                        60.5
TM212  1936.7                 18100                  16.7                      50.1                       146.9
TM212  1937.115               6726                  50                         66                          163.3

How much of the table do we have for the Chinese ?
Title: Re: EM Drive Developments - related to space flight applications - Thread 2
Post by: Star-Drive on 02/15/2015 02:54 pm
@Paul March,
It is kind of important to my results if you could confirm that the Teflon Rubber gaskets are installed as illustrated in the attached model. Click on the image, it will expand so you can see detail - but of course it is mostly black so use the sliders to move around to find a corner. :)

Right now, I am using a 12.5 mm coaxial dipole antenna at the inner face of the dielectric disk. I know you used a loop of some sort. How much do you think this difference matters considering that I am running a digital model?

It is also important that I correctly model the width of the Teflon Rubber gasket filled gap. You wrote that the gasket was .064." Was that after installed, or did you compress it when you tightened down the retaining ring. If so, what would you estimate the actual distance is, between the copper cone and copper base plate, as installed? I know that sounds like a nonsense question, but my simulation shows thrust force is dramatically sensitive to just a small changes in the gap width. I'd like for my model to be as close as is possible to your Copper Kettle thruster.

My final question (I hope) re. the gasket is, "Do you know what the dielectric constant is for the actual Teflon Rubber that you used?" (Did your supplier document it, perhaps.) I find values ranging from 2.1 to 2.5 and while force is not very sensitive to this value, it does have an effect.

And while I'm at it, I read that the vacuum chamber is 30 inches by 36 inches, diameter by length. Is that inside or outside dimensions?

Aero:

"It is kind of important to my results if you could confirm that the Teflon Rubber gaskets are installed as illustrated in the attached model."

I think what you are talking about is the initial pressure seal design for our aluminum frustum cavity that later went to a silicone O-ring and metal to metal compression shorting pad just inside the O-ring for both the large and small OD ends of the frustum. 

The Eagleworks copper frustum is not a gas sealed unit, so all it has for its large and small OD end-cap interfaces are copper metal to copper metal interface with #6-32 brass cap-screws, nuts and bronze internal star lock washers spaced an average of 1.0" apart on the frustum's 0.50" wide copper flanges.  As to the average air gap between these copper flanges due to their out of plane irregularities, (These copper flanges are only 0.040" thick.), my guess is that it can be no larger than 0.002" midway between the cap-screws.     

"Right now, I am using a 12.5 mm coaxial dipole antenna at the inner face of the dielectric disk. I know you used a loop of some sort. How much do you think this difference matters considering that I am running a digital model?"

I've used various OD magnetic loops made from #20 AWG copper magnet wire soldered to SMA bulkhead connector that is mounted on the copper frustum's conical side wall, 15% of the of frustum Z-axis height from the large OD end of the frustum cavity, see attached picture.  Currently we are using a 14.0mm OD loop antenna for our TM212 work at 1,937.118 MHz work.

"And while I'm at it, I read that the vacuum chamber is 30 inches by 36 inches, diameter by length. Is that inside or outside dimensions?"

The Eagleworks vacuum chamber interior dimensions are as noted except the distance front aluminum door to the rear domed portion of the 304L stainless steel spun end cap is ~38.0", see attached Kurt J. Lesker drawing.  However, Our vacuum door is hinged on the right side of the chamber as viewed from the door end.
Title: Re: EM Drive Developments - related to space flight applications - Thread 2
Post by: Star-Drive on 02/15/2015 03:36 pm
Folks:

In the meantime, lets ask why 60 watts of relatively harmonic free sine-wave RF power at the 1,937.118 MHz AKA the TM212 resonant frequency in this copper frustum cavity, can only generate a paltry ~60uN, whereas the Chinese claimed to have produce 160,000uN using just ~150 watts of 2,450 MHz RF signals from a magnetron?  The magnetron RF signal source that is anything but a pure sine-wave generator, that instead has a modulated FM bandwidth of at least +/-30 MHz that is also concurrently amplitude modulated (AM) with thermal electron noise. 


Taking a critical look at this question, and knowing that the spectral shape of a magnetron looks like (see below) compared to a CW spike. It seems evident that a CW spike isn't the best waveform to use if you want to maximize thrust. Dollars to donuts says the Chinese are making full use of the available bandwidth of their resonant cavity by using that noisy magnetron. Magnetrons have lots of phase noise too. You can't easily use them on phased array radars because of that for example.

...
I agree with Mulletron that the answer to Paul March's question is that it is much more effective to have a distributed power spectral density than the power concentrated at a single frequency spike.  When the natural frequency changes in an unpredictable manner, it is much more effective to have a distributed power spectral density of excitation (it is the power spectral density ( http://en.wikipedia.org/wiki/Spectral_density#Power_spectral_density ) over the spectrum of changing natural frequencies that matters).

The reason for this is that (as has been verified by Prof. Juan Yang in China by inserting thermocouples at different places in the EM Drive) the EM Drive is subjected to a very non-uniform temperature distribution, with the temperature increasing with time, that results in significant non-uniform thermal expansion of the EM Drive, and therefore the natural frequencies must shift with temperature (and therefore shift with time as the temperature changes with time) as the EM Drive expands non-uniformly with time.  Therefore, having the power concentrated at a single frequency spike (NASA) is bound to be non-efficient as the resonant frequency changes with time, the EM Drive is going to move out of resonance even if one happens to excite it at the correct frequency to start with.  The COMSOL calculations do not provide the natural frequency to enough precision within the extremely narrow bandwidth of a high Q resonance (the higher the Q, the narrower the resonant bandwidth) for NASA to know exactly the natural frequency for a given mode shape.  More importantly, the COMSOL calculations do not provide the information needed for NASA to know how to shift the frequency with time, as the EM Drive thermally expands non-uniformly to stay at peak resonance.

This is evident from the very low Q's reported by NASA (7K to 22K) compared with the Chinese, who report a Q=117K

Quote from: Juan Yang
the resonant frequency and quality factor of the independent microwave resonator system are 2.44895 GHz and 117495.08 respectively

Compare this with NASA's reported Q:

Mode   Frequency (MHz)  Quality Factor, Q   Input Power (W)  Mean Thrust (μN)   Medium      Efficiency(uN/W)
TE012     1880.4               22000                         2.6                55.4                   Air           21
TM2112  1932.6                 7320                       16.9                91.2                   Air             5
TM2112  1936.7               18100                       16.7                50.1                   Air             3
TM212    1937.115             6726                       50                   66                      Vacuum      1

NASA's reported Q for the vacuum experiment is a meager Q = 6726, which is 17 times smaller than the Chinese reported Q = 117495.

Also note that the most efficient mode reported by NASA Eagleworks is the Transverse Electric mode which gave a Mean Thrust of 55 uN with only 2.6 Watts.



The Chinese also report that they used the Transverse Electric mode



Instead, NASA Eagleworks has been running most of the experiments in the Brady report in the Transverse Magnetic mode, and the vacuum experiment also in the Transverse Magnetic mode, which NASA's own data (see above) shows to be the most inefficient mode.

Why is NASA running the vacuum experiment in the most inefficient mode (Transverse Magnetic) rather than the most efficient mode (Transverse Electric) ?  Because they report difficulties in tuning the EM Drive under the Transverse Electric mode.

Quote from: Brady et.al page 17
Prior to the TM211 evaluations, COMSOL® analysis indicated that the TE012 was an effective thrust generation mode for the tapered cavity thruster being evaluated, so this mode was explored early in the evaluation process. Figure 22 shows a test run at the TE012 mode with an operating frequency of 1880.4 MHz. The measured quality factor was ~22,000, with a COMSOL prediction of 21,817. The measured power applied to the test article was measured to be 2.6 watts, and the (net) measured thrust was 55.4 micronewtons. With an input power of 2.6 watts, correcting for the quality factor, the predicted thrust is 50 micronewtons. However, since the TE012 mode had numerous other RF modes in very close proximity, it was impractical to repeatedly operate the system in this mode, so the decision was made to evaluate the TM211 modes instead.

Why does NASA have difficulties running the EM Drive in the more efficient mode (the Transverse Electric mode) ? Because the most efficient mode results in greater shifting of its natural frequency with time.  Hence I agree with Mulletron that instead of having the power concentrated at a frequency, for a problem where we know that the natural frequency of the EM Drive changes with time in a difficult to calculate and predict (with enough precision) manner, the best solution is to have the power distributed over a wider spectrum of frequencies, as done by Prof. Juan Yang in China.

(http://forum.nasaspaceflight.com/index.php?action=dlattach;topic=36313.0;attach=778913;image)

Dr. Rodal:

You seem to ask a lot of "why" questions that could be better answered by getting yourself in the lab and finding out the sought after answers for yourself.  Be that as it may, the main reason that we went with the lower-Q TM modes was because they consistently produced higher thrust levels for a given input power than the TE modes.  I will grant you though that getting the most thrust out of a particular resonant mode depended very painfully on the size, placement and rotational orientation of the loop antenna in the frustum cavity.  And it may just have been that I didn't know how best to optimize their operations at the time, since this has been a rather large learning experience for me over these last three years in the Eagleworks Lab.

BTW, thanks much for the pointer to the 2014 Chinese report.  Is there an English translation of same out in public yet?  Also, in the 2013 Chinese report that had been translated into English, see attached, you will find that their large hundreds of milli-Newton thrust results were obtained with a loaded quality factor of just ~1,530 at 2.45 GHz, see figure 13 in their 2013 report.  We think that is because that like any ac electric induction motor, this device has to load down its input energy/power source as it is generating thrusting work.  Which brings up another point.  That being all the calculated Q-Factors given in the Chinese papers, unless otherwise stated, is the very idealized unloaded Q-factors that implies that no energy is being extracted from the resonant cavity.   We must keep that fact in mind as well...

Best, Paul M.
Title: Re: EM Drive Developments - related to space flight applications - Thread 2
Post by: Star-Drive on 02/15/2015 03:58 pm
Folks:

In the meantime, lets ask why 60 watts of relatively harmonic free sine-wave RF power at the 1,937.118 MHz AKA the TM212 resonant frequency in this copper frustum cavity, can only generate a paltry ~60uN, whereas the Chinese claimed to have produce 160,000uN using just ~150 watts of 2,450 MHz RF signals from a magnetron?  The magnetron RF signal source that is anything but a pure sine-wave generator, that instead has a modulated FM bandwidth of at least +/-30 MHz that is also concurrently amplitude modulated (AM) with thermal electron noise. 


Taking a critical look at this question, and knowing that the spectral shape of a magnetron looks like (see below) compared to a CW spike. It seems evident that a CW spike isn't the best waveform to use if you want to maximize thrust. Dollars to donuts says the Chinese are making full use of the available bandwidth of their resonant cavity by using that noisy magnetron. Magnetrons have lots of phase noise too. You can't easily use them on phased array radars because of that for example.

Now to put this idea to test, Q: What is the bandwidth of the resonant cavity and what is the 90 percent power bandwidth of the signal you are driving it with? What kind of sig gen are you using? Can it do FM? Can you do any advanced waveforms like a PSK waveform? Do you have a way to produce wideband noise or a spread spectrum carrier for your testing? Can you do any waveforms like at the bottom?

Also during researching other possible theories which could explain Emdrive we found ample literature stating that molecules acquire a kinetic momentum during the switching of the magnetic field as a result of its interaction with the vacuum field. If correct, that may well be a very significant lead. So that raises the question, how does one increase the switching rate? What about phase shifting? http://en.wikipedia.org/wiki/Phase-shift_keying

Phase shifting seems important.
https://www.viasat.com/files/assets/web/datasheets/EBEM_MD-1366_043_web.pdf
One of these driving your amp would be helpful. They go up to 2ghz.

(http://www.zdnet.com/i/story/60/03/000578/noise-from-microwave.png)
(https://encrypted-tbn2.gstatic.com/images?q=tbn:ANd9GcTVRN3U7Ai7Zbu56xVIVRQhS_nMGHQFi5EjSx1BFaxm8GZOlY9Q)
(http://www.propagation.gatech.edu/ECE6390/project/Fall2008/DipoleDetectors/Di_Pole%20Detectors/images/DataLink_Spectrum.JPG)
(http://www.cse.iitk.ac.in/users/dheeraj/cs425/fig.lec01/bpsk.gif)


Mulletron:

When using our current 14mm loop antenna optimized for the TM212 resonance at 1,937.118 MHz in our copper frustum, there were four RF resonances spaced +/- 30 MHz around the 2.45 GHz center frequency.  And I assume that would also be the case using a higher power slot antenna in a similar location as the Chinese and Shawyer have done with their frustum resonant cavities.  So yes, a wide bandwidth RF source seems to be called for and one that can be both AM and FM modulated at the same time.  From my readings to date, that appears to be a hard nut to crack for solid state RF amplifiers at the desired kW power levels due to their limited RF power bandwidth capabilities, so we may be forced into using magnetrons and just learn how best to feed their 4-to-20 kV high voltage anode requirements while working in a hard vacuum.  However the more difficult problems are finding ways of reducing their mass and size so we can "fly" them on our torque pendulum.  Cooling the magnetrons in a hard vacuum is also another problem we need to deal with since air cooling is out of the question and liquid cooling is a giant pain to deal with as well.  About the only other way to cool these beasts in a hard vacuum is to use phase change material like paraffin wax that could give us several minutes of run times before we had to let the accumulated heat in the paraffin radiate to the vacuum chamber walls.

BTW, the paraffin wax phase change cooling was used to good advantage on the lunar moon buggy used by NASA astronauts during their lunar explorations in the late 1960s and early 1970s during the USA Apollo Moon program.

Best, Paul M.
Title: Re: EM Drive Developments - related to space flight applications - Thread 2
Post by: Rodal on 02/15/2015 04:08 pm
...
Dr. Rodal:

You seem to ask a lot of "why" questions that could be better answered by getting yourself in the lab ...
When I was in charge of R&D laboratories for private companies I always welcomed such questions, particularly questions from outsiders  that perhaps had not been asked by internal staff, because the purpose of R&D is always to find the truth as fast as possible, hence I saw it as a great benefit to our R&D efforts to get such questions.   That was also the attitude of the Professors at MIT, I was lucky to work with.
Title: Re: EM Drive Developments - related to space flight applications - Thread 2
Post by: Rodal on 02/15/2015 04:09 pm
... the main reason that we went with the lower-Q TM modes was because they consistently produced higher thrust levels for a given input power than the TE modes.  ...
The data reported by NASA Eagleworks contradicts that TM modes produce higher thrust levels than TE modes for a given input power:

Mode   Frequency (MHz)  Quality Factor, Q   Input Power (W)  Mean Thrust (μN)   Medium      Efficiency(uN/W)
TE012     1880.4               22000                         2.6                55.4                   Air           21
TM2112  1932.6                 7320                       16.9                91.2                   Air             5
TM2112  1936.7               18100                       16.7                50.1                   Air             3
TM212    1937.115             6726                       50                   66                      Vacuum      1

Apparently, the emphasis should be on the use of the word "consistently" either as that

a) the reported Brady et.al. data for TE012 was inconsistent, in which case the Mean Thrust was not 55.4 uN as reported by Brady et.al. but apparently there is other unreported data that NASA has, giving significantly lower values of the thrust (and accordingly the reported Mean by Brady et.al. was not the true Mean of the TE012 experiments with the dielectric at NASA)

or

b) my interpretation of the Brady et.al.'s statement that NASA Eagleworks had trouble staying tuned at the natural frequency near  1880.4  MHz

Quote from: Brady et.al page 17
Prior to the TM211 evaluations, COMSOL® analysis indicated that the TE012 was an effective thrust generation mode for the tapered cavity thruster being evaluated, so this mode was explored early in the evaluation process. Figure 22 shows a test run at the TE012 mode with an operating frequency of 1880.4 MHz. The measured quality factor was ~22,000, with a COMSOL prediction of 21,817. The measured power applied to the test article was measured to be 2.6 watts, and the (net) measured thrust was 55.4 micronewtons. With an input power of 2.6 watts, correcting for the quality factor, the predicted thrust is 50 micronewtons. However, since the TE012 mode had numerous other RF modes in very close proximity, it was impractical to repeatedly operate the system in this mode, so the decision was made to evaluate the TM211 modes instead.
Title: Re: EM Drive Developments - related to space flight applications - Thread 2
Post by: zen-in on 02/15/2015 04:11 pm

BTW, thanks much for the pointer to the 2014 Chinese report.  Is there an English translation of same out in public yet?  Also, in the 2013 Chinese report that had been translated into English, see attached, you will find that their large hundreds of milli-Newton thrust results were obtained with a loaded quality factor of just ~1,530 at 2.45 GHz, see figure 13 in their 2013 report.  We think that is because that like any ac electric induction motor, this device has to load down its input energy/power source as it is generating thrusting work.  Which brings up another point.  That being all the calculated Q-Factors given in the Chinese papers, unless otherwise stated, is the very idealized unloaded Q-factors that implies that no energy is being extracted from the resonant cavity.   We must keep that fact in mind as well...

Best, Paul M.

There may be a more conventional reason for the Q to decrease.  Excess RF power may just be getting dissipated inside the cavity as heat.   If the Chinese test device was in a static test fixture, like a torsion balance, and there was no movement in the direction of the measured force, then no work was done. 
Title: Re: EM Drive Developments - related to space flight applications - Thread 2
Post by: Rodal on 02/15/2015 04:14 pm
BTW, thanks much for the pointer to the 2014 Chinese report.  Is there an English translation of same out in public yet?
I have not seen an English translation of this paper posted in the Internet:

Resonance experiment on a microwave resonator system
Shi Feng Yang Juan Tang Ming-Jie Luo Li-Tao Wang Yu-Quan
(College of Astronautics, Northwestern Polytechnic University, Xi’an 710072, China)
Acta Phys. Sinica Vol. 63, No. 15 (2014)

I have only seen the Chinese original that I attached.  As to translations, my experience is that it is always best to seek a meaningful translation from people skilled in the art: in this case from people conversant in the language of the paper who are also engineers/scientists in the same field discussed by the paper.
Title: Re: EM Drive Developments - related to space flight applications - Thread 2
Post by: aero on 02/15/2015 04:22 pm
@Paul March,
It is kind of important to my results if you could confirm that
             ... snip ...
Paul - Thank you for this very helpful information

Quote
Aero:

"It is kind of important to my results if you could confirm that the Teflon Rubber gaskets are installed as illustrated in the attached model."

I think what you are talking about is the initial pressure seal design for our aluminum frustum cavity that later went to a silicone O-ring and metal to metal compression shorting pad just inside the O-ring for both the large and small OD ends of the frustum. 

The Eagleworks copper frustum is not a gas sealed unit, so all it has for its large and small OD end-cap interfaces are copper metal to copper metal interface with #6-32 brass cap-screws, nuts and bronze internal star lock washers spaced an average of 1.0" apart on the frustum's 0.50" wide copper flanges.  As to the average air gap between these copper flanges due to their out of plane irregularities, (These copper flanges are only 0.040" thick.), my guess is that it can be no larger than 0.002" midway between the cap-screws.     

Good good, very good. I have ran many cases, attached find a curve showing the EM Thruster force sensitivity to gap size. Ignore the units - this curve is only representative of force .vs. gap size for what I would consider to be a bare thruster in isolation, perhaps in orbit for example. But do note that the smallest gap corresponds to 0.011 inches, significantly larger than 0.002 inches. This was a 2D meep simulation, limited by the computer power of my home desktop machine.

Quote
"Right now, I am using a 12.5 mm coaxial dipole antenna at the inner face of the dielectric disk. I know you used a loop of some sort. How much do you think this difference matters considering that I am running a digital model?"

I've used various OD magnetic loops made from #20 AWG copper magnet wire soldered to SMA bulkhead connector that is mounted on the copper frustum's conical side wall, 15% of the of frustum Z-axis height from the large OD end of the frustum cavity, see attached picture.  Currently we are using a 14.0mm OD loop antenna for our TM212 work at 1,937.118 MHz work.

Thank you - I'll look again at my antenna placement. Unfortunately I don't know of a way to simulate the loop antenna in 2D if it is not coplanar.
 
Quote

"And while I'm at it, I read that the vacuum chamber is 30 inches by 36 inches, diameter by length. Is that inside or outside dimensions?"

The Eagleworks vacuum chamber interior dimensions are as noted except the distance front aluminum door to the rear domed portion of the 304L stainless steel spun end cap is ~38.0", see attached Kurt J. Lesker drawing.  However, Our vacuum door is hinged on the right side of the chamber as viewed from the door end.

 Hmm. So it is even larger than I am simulating. That will add CPU cycles to the simulation runs. But not so much because the model is already to large to run in 3D on my machine. I've looked at renting a compute engine from Google to overcome this problem but ...
Title: Re: EM Drive Developments - related to space flight applications - Thread 2
Post by: Rodal on 02/15/2015 04:56 pm
FYI for no good reason, here what I get w/o dielectric

Mode   Frequency (MHz)  Quality Factor, Q   Input Power (W)  Mean Thrust (μN)   Calc
TE012  1880.4                  22000                  2.6                       55.4                        16.9
TM212  1932.6                   7320                  16.9                      91.2                        60.5
TM212  1936.7                 18100                  16.7                      50.1                       146.9
TM212  1937.115               6726                  50                         66                          163.3

How much of the table do we have for the Chinese ?
The dimensions of the EM Drive are needed to estimate thrust in your formula as well as Shawyer's and McCulloch's formula.   I could not find all the dimensions needed of the Chinese EM Drive tested by Prof. Juan Yang in any of their Chinese papers.  My recollection is that @aero also tried to look for those dimensions in the Yang papers translated to English, and was unable to find the dimensions either.

We (colleagues in Thread 1 and 2 of EM Drive Developments at NASA Spaceflight.com) never worked out an estimate for the dimensions of the Chinese EM Drive as it was done for the estimates of the NASA Eagleworks truncated cone (an estimate by @aero and @Muletron that turned out to be very close to the actual dimensions recently reported by Paul March) or for Shawyer's Experimental and Demonstration truncated cones.

Dr. McCulloch has used the same dimensions for the EM Drive tested by Prof. Juan Yang as the dimensions of the Shawyer Demonstration EM Drive (see  http://physicsfromtheedge.blogspot.com/2015/02/mihsc-vs-emdrive-data-3d.html) to estimate thrust of the Chinese EM Drive, and gets a reasonable comparison.

If we assume that the drawing from the Chinese paper is to scale, that gives us the ratios of the dimensions, and as a consequence we only need to estimate one of the dimensions (for example one of the diameters). It looks from this drawing (see below) that the axial length of the Juan Yang truncated cone is significantly shorter than the small diameter, and therefore the Juan Yang truncated cone is significantly different from the one tested at NASA Eagleworks and Shawyer's Demo and Experimental truncated cones.

I estimate from this drawing:

Big Diameter     = D1
Small Diameter = D2
Axial Length      = H


Ratio of Big Diameter to Small Diameter = 1.61
Ratio of Big Diameter to axial Length =  2.38




Hence
Ratio of Small Diameter to axial Length =  1.48
(Big Diameter - Small Diameter)/axialLength = 0.91
Tangent of Cone's half angle = 0.4538
Cone's half-angle = 24 degrees



For reference. the tangent of the cone's half angle thetaw and the cone's half angle thetaw, for the following cases are:


Example (and geometry)                    { Tan[thetaw],thetaw (degrees) }

Shawyer Experimental                        {0.104019,   5.93851}
Shawyer Demo                                   {0.219054, 12.3557}
NASA Eagleworks frustum                   {0.263889, 14.7827}
Egan's example                                  {0.36397 ,  20}
Prof. Juan Yang  (2014)                      {0.4538,     24.4 }
Shawyer Superconducting 2014          {0.7002,     35}


(http://gregegan.customer.netspace.net.au/SCIENCE/Cavity/CavityShape.gif)

Shawyer's latest (2014) superconducting design (see image, presented at the IAC 2014 conference in Toronto), for which there are no experimental results reported yet, appears to have a significantly larger cone angle than his previous experimental and demo geometries, and significantly larger than NASA's Brady et.al.'s.

Shawyer's (2014) superconducting EM Drive design has a cone angle  thetaw of about 35 degrees.  It is the only EM Drive designed with spherical ends, which make much more sense -from an electromagnetic wave propagation viewpoint, particularly for large cone angle geometries- than the flat ends used by NASA Eagleworks, and the other EM Drives

(http://1.bp.blogspot.com/-IUOMkncS1sM/VFLSUw2HeoI/AAAAAAAA3yE/Zl3iaKJyrX0/s1600/emdrivedata2.png)





(*
"Shawyer EXPERIMENTAL geometry"

shawyerExpLength=0.156 meter;
shawyerExpBigDiameter=0.16 meter
shawyerExpSmallDiameter=0.127546 meter;
*)

(*
" Shawyer DEMO geometry"

shawyerDemoLength=0.345 meter;
shawyerDemoBigDiameter=0.28 meter;
shawyerDemoSmallDiameter=0.128853 meter;

*)
Title: Re: EM Drive Developments - related to space flight applications - Thread 2
Post by: aero on 02/15/2015 05:18 pm
Quote
It looks from this drawing (see below) that the axial length of the Juan Yang truncated cone is significantly shorter than the small diameter, and therefore the Juan Yang truncated cone is significantly different from the one tested at NASA Eagleworks and Shawyer's Demo and Experimental truncated cones.

Which means to me that the mode is probably like TX y,z,1. That is, the cavity is only one-half wavelength in height.
Title: Re: EM Drive Developments - related to space flight applications - Thread 2
Post by: RotoSequence on 02/15/2015 05:36 pm
I wonder what kind of cash it costs to get some quality time with an additive/subtractive hybrid manufacturing 3D Printer/5 axis mills like the Lasertech series by DMG MORI, seen here fabricating a turbine housing:

https://www.youtube.com/watch?v=s9IdZ2pI5dA

These kinds of printers are capable of working with a variety of materials, including aluminum, steel alloys, inconel, titanium, etc. I suspect hybrid manufacturing of an Inconel cavity or other alloy could be the easiest, least expensive way of acquiring high precision resonance cavities with relatively consistent operational characteristics, but I have no idea how much it costs to work with these sorts of machines.
Title: Re: EM Drive Developments - related to space flight applications - Thread 2
Post by: aero on 02/15/2015 06:00 pm
I wonder what kind of cash it costs to get some quality time with an additive/subtractive hybrid manufacturing 3D Printer/5 axis mills like the Lasertech series by DMG MORI, seen here fabricating a turbine housing:

https://www.youtube.com/watch?v=s9IdZ2pI5dA

These kinds of printers are capable of working with a variety of materials, including aluminum, steel alloys, inconel, titanium, etc. I suspect hybrid manufacturing of an Inconel cavity or other alloy could be the easiest, least expensive way of acquiring high precision resonance cavities with relatively consistent operational characteristics, but I have no idea how much it costs to work with these sorts of machines.


As I have remarked before, to make high quality precision cavities, all you need is a spinner machine from the 20th century. Just spin the copper sheet metal over a steel mandrel, then punch-press the end caps, or use the same circuit boards that Eagleworks uses. Problem with the circuit board is that the gaps are in the horizontal direction which is less efficient at producing thrust than gaps in the vertical direction. At least that is true if meep and the differential Maxwell equations are correctly characterizing the forces.

Making cavities in this way should cost less than $5 each once the machinist has been paid for turning out the mandrel and the punch tools. Of course we could ask Mulletron for an estimate of the cost of copper sheet in bulk, it might be more.
Title: Re: EM Drive Developments - related to space flight applications - Thread 2
Post by: Rodal on 02/15/2015 06:08 pm
I wonder what kind of cash it costs to get some quality time with an additive/subtractive hybrid manufacturing 3D Printer/5 axis mills like the Lasertech series by DMG MORI, seen here fabricating a turbine housing:

https://www.youtube.com/watch?v=s9IdZ2pI5dA

These kinds of printers are capable of working with a variety of materials, including aluminum, steel alloys, inconel, titanium, etc. I suspect hybrid manufacturing of an Inconel cavity or other alloy could be the easiest, least expensive way of acquiring high precision resonance cavities with relatively consistent operational characteristics, but I have no idea how much it costs to work with these sorts of machines.

A 3D printer that would enable fabrication of an EM Drive with spherical ends and a cone half angle~35 degrees or higher would be most advantageous (Invar, as you suggest because it would greatly reduce thermal expansion, and hence allow easier tuning to a stable natural frequency).  Also one that would allow fabrication of a superconducting cavity (Invar+YBCO).
Title: Re: EM Drive Developments - related to space flight applications - Thread 2
Post by: TMEubanks on 02/15/2015 06:30 pm
Thoughts on Axions. (I apologize if this has been already covered.)

The question as to whether the EM Drive could be coupling to the Axion background came up on a different forum. I am dubious, and thought it would be useful to post why.

A recent review of axions as CDM: http://www.pnas.org/.../2015/01/07/1308788112.full.pdf

Current constraints on the axion mass constrain it to be in the range ~ 1 - 1000 micro-eV, and a 2 GHz axion would correspond to 8.2 micro-eV, so that's OK, maybe the EM drive couples to the axion mass. But, check out the Axion Dark-Matter Experiment in the PNAS article. That is much much more sensitive than the EM Drive - they are looking for yoctowatts (10^-24) of RF power in the 2 - 20 micro-eV range, precisely the range of the EM Drive, by tuning the cavity's resonant frequency to the axion mass. There is simply no way that the Drive is coupling to the axion background - the ADMX would see a whopping signal. Now, maybe the ADMX is producing lots of axions, at a low enough velocity to evade the photon rocket limits. That would mean that the EM Drive can convert watts of RF -> axions, while the ADMX is NOT converting 10^-24 watts of dark matter axions -> RF power. While I guess it is possible, I just don't buy it. Nobody is accidentally lucky by 20+ orders of magnitude. (Note also that the theory is pretty clear here - if axions have these kinds of 1 - 10 micro-eV masses, they will supply most or all of the DM, so there will be a background.)

However, there is an easy test - just take an EM drive to the University of Washington, and see if you get a reaction from the ADMX. That should done at some point if the drive continues to pass experimental muster.
Title: Re: EM Drive Developments - related to space flight applications - Thread 2
Post by: Rodal on 02/15/2015 06:45 pm
...
The question as to whether the EM Drive could be coupling to the Axion background came up on a different forum. I am dubious, and thought it would be useful to post why.

A recent review of axions as CDM: http://www.pnas.org/.../2015/01/07/1308788112.full.pdf

...

Link broken ?

I get the message "Proceedings of the National Academy of Sciences of the United States of America
Not Found The page you were trying to reach could not be found." using either Google Chrome or Mozilla Firefox
Title: Re: EM Drive Developments - related to space flight applications - Thread 2
Post by: SleeperService on 02/15/2015 06:54 pm
Maybe the error message should have said:

Not found: The axions you are looking for do not exist.

 ;)
Title: Re: EM Drive Developments - related to space flight applications - Thread 2
Post by: Star-Drive on 02/15/2015 06:58 pm
... the main reason that we went with the lower-Q TM modes was because they consistently produced higher thrust levels for a given input power than the TE modes.  ...
The data reported by NASA Eagleworks contradicts that TM modes produce higher thrust levels than TE modes for a given input power:

Mode   Frequency (MHz)  Quality Factor, Q   Input Power (W)  Mean Thrust (μN)   Medium      Efficiency(uN/W)
TE012     1880.4               22000                         2.6                55.4                   Air           21
TM2112  1932.6                 7320                       16.9                91.2                   Air             5
TM2112  1936.7               18100                       16.7                50.1                   Air             3
TM212    1937.115             6726                       50                   66                      Vacuum      1

Apparently, the emphasis should be on the use of the word "consistently" either as that

a) the reported Brady et.al. data for TE012 was inconsistent, in which case the Mean Thrust was not 55.4 uN as reported by Brady et.al. but apparently there is other unreported data that NASA has, giving significantly lower values of the thrust (and accordingly the reported Mean by Brady et.al. was not the true Mean of the TE012 experiments with the dielectric at NASA)

or

b) my interpretation of the Brady et.al.'s statement that NASA Eagleworks had trouble staying tuned at the natural frequency near  1880.4  MHz

Quote from: Brady et.al page 17
Prior to the TM211 evaluations, COMSOL® analysis indicated that the TE012 was an effective thrust generation mode for the tapered cavity thruster being evaluated, so this mode was explored early in the evaluation process. Figure 22 shows a test run at the TE012 mode with an operating frequency of 1880.4 MHz. The measured quality factor was ~22,000, with a COMSOL prediction of 21,817. The measured power applied to the test article was measured to be 2.6 watts, and the (net) measured thrust was 55.4 micronewtons. With an input power of 2.6 watts, correcting for the quality factor, the predicted thrust is 50 micronewtons. However, since the TE012 mode had numerous other RF modes in very close proximity, it was impractical to repeatedly operate the system in this mode, so the decision was made to evaluate the TM211 modes instead.


Dr. Rodal:

Yes, consistency of results has been a large problem for us.  And its cause is centered around consistently frequency locking the voltage controlled oscillator (VCO) frequency to the always drifting high-Q resonant frequency of the resonant cavity as it heats up while under power.  We tried building a mixer based phased lock loop (PLL) circuit and box that will work in the vacuum chamber, but found that for proper and consistent frequency lock-on, it was very sensitive to the amplitude of the sense antenna signal coming back from the resonant cavity.  Running out of budget & time, we had to give up on it for the moment when it became apparent that we also had to design, build and integrate an automatic gain control circuit for the PLL that could fit into the already physically constrained PLL box.  Thus, right now we are still using the "Paul Locked Loop" approach with me acting as the frequency control feedback loop for each test.  We've talked to our RF group here at JSC about better solutions, and they suggested using a S11 return loss based sensing circuit, which is much less sensitive to input amplitude variations, but again we don't have the budget to have one designed & built for our vacuum based application.  In other words you do what you can do and then move on...

Best, Paul M.
Title: Re: EM Drive Developments - related to space flight applications - Thread 2
Post by: Notsosureofit on 02/15/2015 06:59 pm
Just direct search

1308788112.full.pdf
Title: Re: EM Drive Developments - related to space flight applications - Thread 2
Post by: Notsosureofit on 02/15/2015 07:02 pm
@ Paul

Back in the 60's we just used a cavity based oscillator  (that one was reentrant)

Edit:  50's,  memory must be slippin
Title: Re: EM Drive Developments - related to space flight applications - Thread 2
Post by: Rodal on 02/15/2015 07:05 pm
...
The question as to whether the EM Drive could be coupling to the Axion background came up on a different forum. I am dubious, and thought it would be useful to post why.

A recent review of axions as CDM: http://www.pnas.org/.../2015/01/07/1308788112.full.pdf

...

[/quote]
Just direct search

1308788112.full.pdf

Thanks  :)

This is the link to the paper on Dark Matter search posted by Mashall:

http://www.pnas.org/content/early/2015/01/07/1308788112.full.pdf

Dark-matter QCD-axion searches
Leslie J Rosenberg

Abstract
In the late 20th century, cosmology became a precision science. Now, at the beginning of the next century, the parameters describing how our universe evolved from the Big Bang are generally known to a few percent. One key parameter is the total mass density of the universe. Normal matter constitutes only a small fraction of the total mass density. Observations suggest this additional mass, the dark matter, is cold (that is, moving nonrelativistically in the early universe) and interacts feebly if at all with normal matter and radiation. There’s no known such elementary particle, so the strong presumption is the dark matter consists of particle relics of a new kind left over from the Big Bang. One of the most important questions in science is the nature of this dark matter. One attractive particle dark-matter candidate is the axion. The axion is a hypothetical elementary particle arising in a simple and elegant extension to the standard model of particle physics that nulls otherwise observable CP-violating effects (where CP is the product of charge reversal C and parity inversion P) in quantum chromo dynamics (QCD). A light axion of mass 10−(6–3) eV (the invisible axion) would couple extraordinarily weakly to normal matter and radiation and would therefore be extremely difficult to detect in the laboratory. However, such an axion is a compelling dark-matter candidate and is therefore a target of a number of searches. Compared with other particle dark-matter candidates, the plausible range of axion dark-matter couplings and masses is narrowly constrained. This focused search range allows for definitive searches, where a nonobservation would seriously impugn the dark-matter QCD-axion hypothesis. Axion searches use a wide range of technologies, and the experiment sensitivities are now reaching likely dark-matter axion couplings and masses. This article is a selective overview of the current generation of sensitive axion searches. Not all techniques and experiments are discussed, but I hope to give a sense of the current experimental landscape of the search for dark-matter axions.
Title: Re: EM Drive Developments - related to space flight applications - Thread 2
Post by: SleeperService on 02/15/2015 07:24 pm
Sorry if I confused you.
Are we to have axions or MiHsC?
Both are speculative but only one or none can be true, I think.
What do you chaps think?
Title: Re: EM Drive Developments - related to space flight applications - Thread 2
Post by: Rodal on 02/15/2015 07:34 pm
Sorry if I confused you.
Are we to have axions or MiHsC?
Both are speculative but only one or none can be true, I think.
What do you chaps think?

1) Eubanks posted a paper about Dark Matter, stating <<While I guess it is possible, I just don't buy it>>.

2) We have been posting the predictive formulas of Dr. McCulloch, Shawyer and @NotSoSureOfIt not as partisans for one or another, but in order to find out the truth, whatever the truth ends up to be.

3) This technology is such a breakthrough that it holds my interest if it would be possible to use it for spaceflight.  Count me as an initial skeptic that carefully would like to rule out all classical physics explanations first, one by one.  If the explanation requires exotic physics, this will naturally manifest on its own, just like it happened with Quantum Mechanics at the early 20th century. There is no need for a scientist to be partisan to any given theory.  Experiments will tell the truth.

I think that there is a truth out there in the Universe independent of mankind. Scientists are like explorers trying to find it.  People with passionate beliefs are more unlikely to find it than those with an open mind.
Title: Re: EM Drive Developments - related to space flight applications - Thread 2
Post by: TMEubanks on 02/15/2015 07:47 pm
More thoughts on Dark Matter & the EM Drive (thanks for fixing the dead link!)

Suppose DM is some particle (axions or whatever) with a mass of 2 GHz = 8 micro-eV = 1.48 x 10^-41 kg. The Halo dark matter flux is ~ 2 x 10^-17 kg / (m^2 s sr), so that's 1.4 x 10^24 particles / (m^2 s sr) each (on average) at 300 km/sec. So, the thrust flux from these, if you absorbed them perfectly, would be M * v * that number flux ~ 6 x 10^-12 Newtons / (m^2 sr)*. Given the images of the drive, I can't see its cross section being > 1 m^2 (and the Halo flux is by no means focused), and this is thus ~6 orders of magnitude too small (and, note, also much less than a solar sail).

So, I don't see how a coupling to DM microparticles can explain the drive thrust. 

* Note that that is actually independent of the mass of the DM particles, but I like to show how to work through things. Also note that if you _reflected_ the DM particles, that estimate would increase by a factor of 2, which is not nearly enough to change things. 
Title: Re: EM Drive Developments - related to space flight applications - Thread 2
Post by: aero on 02/15/2015 09:04 pm
After making several runs in an effort to assure myself that the meep model was correct with lattice size and resolution set properly, I have a result of thrust force of the EM thruster inside the stainless steel vacuum chamber, in vacuum. This is a preliminary number but interesting enough that I will share it.

Force = 0.33845 muN/Watt. That is about 1/3 of the experimental value obtained.

The fields within the vacuum chamber are pictured, attached. Again, fields are shown at the end of the 32nd half period of the magnetic source drive frequency, 1.937115E+009 Hz. The meep model for this data still used the Teflon rubber seal, though compressed by 20%.

My next task will be to remove the Teflon rubber gasket and model metal to metal contact with a small gap, and make runs as close to 0.002 inches gap size as my computer will allow. It is my understanding from Paul March's recent post that the EM thruster cavity actually tested was constructed this way. I will then remove the vacuum chamber from the model and make runs simulating the EM thruster in free space. I will be interested to see that result.

While this single run detected only 1/3 of the experimental measured value, I will brazenly write that I think we can forget about axions and dark matter. We can forget about exotic theory and consider that the thrust force likely results from a little understood characteristic of evanescent waves and the forces they generate. Please pardon me if that steps on anyone's toes, but unless there is a serious problem with meep or my model of the thruster and vacuum chamber, well, these forces obtained from integrating the differential Maxwell equations and the MST are to large to overlook.

I don't mean to say that we do not need to know what frequencies and mode shapes to drive the cavity with, we certainly do; meep can only evaluate a model. The drive frequency and cavity needs to be designed before it can be modelled in meep.
Title: Re: EM Drive Developments - related to space flight applications - Thread 2
Post by: Mulletron on 02/15/2015 09:17 pm
Folks:

In the meantime, lets ask why 60 watts of relatively harmonic free sine-wave RF power at the 1,937.118 MHz AKA the TM212 resonant frequency in this copper frustum cavity, can only generate a paltry ~60uN, whereas the Chinese claimed to have produce 160,000uN using just ~150 watts of 2,450 MHz RF signals from a magnetron?  The magnetron RF signal source that is anything but a pure sine-wave generator, that instead has a modulated FM bandwidth of at least +/-30 MHz that is also concurrently amplitude modulated (AM) with thermal electron noise. 


Taking a critical look at this question, and knowing that the spectral shape of a magnetron looks like (see below) compared to a CW spike. It seems evident that a CW spike isn't the best waveform to use if you want to maximize thrust. Dollars to donuts says the Chinese are making full use of the available bandwidth of their resonant cavity by using that noisy magnetron. Magnetrons have lots of phase noise too. You can't easily use them on phased array radars because of that for example.

Now to put this idea to test, Q: What is the bandwidth of the resonant cavity and what is the 90 percent power bandwidth of the signal you are driving it with? What kind of sig gen are you using? Can it do FM? Can you do any advanced waveforms like a PSK waveform? Do you have a way to produce wideband noise or a spread spectrum carrier for your testing? Can you do any waveforms like at the bottom?

Also during researching other possible theories which could explain Emdrive we found ample literature stating that molecules acquire a kinetic momentum during the switching of the magnetic field as a result of its interaction with the vacuum field. If correct, that may well be a very significant lead. So that raises the question, how does one increase the switching rate? What about phase shifting? http://en.wikipedia.org/wiki/Phase-shift_keying

Phase shifting seems important.
https://www.viasat.com/files/assets/web/datasheets/EBEM_MD-1366_043_web.pdf
One of these driving your amp would be helpful. They go up to 2ghz.


Mulletron:

When using our current 14mm loop antenna optimized for the TM212 resonance at 1,937.118 MHz in our copper frustum, there were four RF resonances spaced +/- 30 MHz around the 2.45 GHz center frequency.  And I assume that would also be the case using a higher power slot antenna in a similar location as the Chinese and Shawyer have done with their frustum resonant cavities.  So yes, a wide bandwidth RF source seems to be called for and one that can be both AM and FM modulated at the same time.  From my readings to date, that appears to be a hard nut to crack for solid state RF amplifiers at the desired kW power levels due to their limited RF power bandwidth capabilities, so we may be forced into using magnetrons and just learn how best to feed their 4-to-20 kV high voltage anode requirements while working in a hard vacuum.  However the more difficult problems are finding ways of reducing their mass and size so we can "fly" them on our torque pendulum.  Cooling the magnetrons in a hard vacuum is also another problem we need to deal with since air cooling is out of the question and liquid cooling is a giant pain to deal with as well.  About the only other way to cool these beasts in a hard vacuum is to use phase change material like paraffin wax that could give us several minutes of run times before we had to let the accumulated heat in the paraffin radiate to the vacuum chamber walls.

BTW, the paraffin wax phase change cooling was used to good advantage on the lunar moon buggy used by NASA astronauts during their lunar explorations in the late 1960s and early 1970s during the USA Apollo Moon program.

Best, Paul M.

The genius of your setup is that it's using low power to suss out a very subtle effect. One of the reasons I dismiss the Chinese results is because pumping the resonant chamber with hundreds of watts creates a cone shaped space heater, which doesn't help suss out a subtle effect from amongst the thermal effects.

There are numerous ways you can introduce a wideband rf signal into your test article without resorting to big blubbering magnetrons. I suggested the modem for example, because I know that Nasa owns them (saw the Nasa FBO request to buy them online) and the rest of the govt owns hundreds of them; their modulator will create precise wideband waveforms at will, which takes care of the wideband problem and the phase shifting idea. Any L band modulator can do this. Think of them as dual use for this application. Even an expensive signal generator can't do that.

There are also commercially available RF noise sources out there. http://www.pasternack.com/high-output-noise-generators-1-2-ghz-pe8505-p.aspx albeit hopefully cheaper than this one I hope. The wideband waveform is to ensure the bandwidth of the cavity is being fully exploited, even under changes due to heat expansion. Call me crazy, but here at home, I'm just going to use the wifi on an amp from the remote camera used to view the experiment. Cheap and effective? Which reminds me, I need to spend more time doing stuff rather than reading and typing stuff if I'm ever going to get anything done.

So here's what you mention was used to test Cannae: http://194.75.38.69/pdfs/ZX95-1015+.pdf
What are you using for the tapered frustum?
Title: Re: EM Drive Developments - related to space flight applications - Thread 2
Post by: TMEubanks on 02/15/2015 10:41 pm
Dear aero;

Superb! My congratulations. Clearly, until proven otherwise, we now have to regard the anomalous results as a function of E&M interactions with the walls of the vacuum chamber.
Title: Re: EM Drive Developments - related to space flight applications - Thread 2
Post by: aero on 02/15/2015 11:13 pm
Dear aero;

Superb! My congratulations. Clearly, until proven otherwise, we now have to regard the anomalous results as a function of E&M interactions with the walls of the vacuum chamber.

Well, partly perhaps, but lets wait until I run the same model without the vacuum chamber. I have ran enough cases already that I feel confident that there will be forces in that case to. My question is, how much force is due only to the vacuum chamber. It takes 6 or more hours to get one number, so first I make some runs to verify the model, (I've changed the cone/base attachment, removing the Teflon gasket.) Then I'll re-run the chamber model, then I'll remove the chamber by setting its dielectric constant to 1, and run the case with it removed. It will be tomorrow at the earliest before I have that key number.

Then we should have an idea.

But thank you Sir.
Title: Re: EM Drive Developments - related to space flight applications - Thread 2
Post by: Mulletron on 02/15/2015 11:26 pm
As I was recently reminded, 1) Shawyer didn't use a chamber 2) Neither did the Chinese. Regardless of how I feel about their experimental methods or motivations, they didn't use a chamber. Also evanescent modes will not couple with a structure that can't resonate at the same mode. http://forum.nasaspaceflight.com/index.php?topic=36313.msg1331719#msg1331719 Also the test article walls are way thicker than 5x skin depth thickness which rules this out. We have covered the skin depth of copper at the operating frequencies in depth in thread 1.

http://www.emdrive.com/yang-juan-paper-2012.pdf pg 4.

Quote
We can forget about exotic theory and consider that the thrust force likely results from a little understood characteristic of evanescent waves and the forces they generate. Please pardon me if that steps on anyone's toes
This is hardly a workable theory of operation. There is nothing to back it up. Just conjecture. Not even one citation. Did anyone digest the reported facts that the device's thrust can be reversed simply by changing the resonant mode and/or the placement of the dielectric puck?

@Paul March....Which modes were reverse?

Title: Re: EM Drive Developments - related to space flight applications - Thread 2
Post by: Notsosureofit on 02/15/2015 11:51 pm


 Did anyone digest the reported facts that the device's thrust can be reversed simply by changing the resonant mode and/or the placement of the dielectric puck?

@Paul March....Which modes were reverse?

One would presume it was only w/ reverse placement of the dielectric puck?
Title: Re: EM Drive Developments - related to space flight applications - Thread 2
Post by: Mulletron on 02/16/2015 12:27 am

However, one can also reverse this thrust vector for this copper frustum by just changing which excited resonant mode is used and/or mounting the dielectric discs at the large OD end of the cavity instead of the small OD end, see attached resonant mode map. 

This is non trivial.
Title: Re: EM Drive Developments - related to space flight applications - Thread 2
Post by: Notsosureofit on 02/16/2015 12:54 am

However, one can also reverse this thrust vector for this copper frustum by just changing which excited resonant mode is used and/or mounting the dielectric discs at the large OD end of the cavity instead of the small OD end, see attached resonant mode map. 

This is non trivial.

Yes, I can see the EXB bit, but what did they actually observe ?  Did they only calculate a reversal?  The description and the chart seem to be at odds.  (I don't get a reversal w/o moving the dielectric a lot!)

Title: Re: EM Drive Developments - related to space flight applications - Thread 2
Post by: aero on 02/16/2015 12:56 am
Quote
This is hardly a workable theory of operation. There is nothing to back it up. Just conjecture. Not even one citation. Did anyone digest the reported facts that the device's thrust can be reversed simply by changing the resonant mode and/or the placement of the dielectric puck?

I did so provide a citation - to meep. That citation tells us what some of the software's capabilities are.
I have also provided null results from running the model completely enclosed with no leaks. The lack of forces in that case shows it is not something within the cavity internally causing the thrust.
I have also cited this paper,  http://wwwsis.lnf.infn.it/pub/INFN-FM-00-04.pdf (http://wwwsis.lnf.infn.it/pub/INFN-FM-00-04.pdf) several times. No feedback what-so-ever.
Quote
However, one can also reverse this thrust vector for this copper frustum by just changing which excited resonant mode is used and/or mounting the dielectric discs at the large OD end of the cavity instead of the small OD end, see attached resonant mode map. 

Perhaps I failed to mention the fact that meep solutions exhibit this characteristic. Probably because I didn't want to muddy the water toward my objective of demonstrating force, thinking perhaps it was a resolution issue.

In any case, I did run a lot of cases with various cylindrical cavities looking for a particular resonance mode. Not infrequently, I found the force reversed in sign. Now that we are confident that force reversal is a real phenomena, I'll report it when it occurs again. 

And by the way, isn't a changed resonant frequency or mode the natural result from moving the dielectric from one end to the other in the tapered cavity?
Title: Re: EM Drive Developments - related to space flight applications - Thread 2
Post by: Notsosureofit on 02/16/2015 01:20 am


And by the way, isn't a changed resonant frequency or mode the natural result from moving the dielectric from one end to the other in the tapered cavity?

Yes, that's when I see a reversal if the dispersion reverses.   But not w/ EXB.

(I should say I'm looking at this as a GR result.  The mechanism could easily be evanescent, or other.  The boost transformation self-accelerating wavefunctions, for instance, exhibit exponential decay)
Title: Re: EM Drive Developments - related to space flight applications - Thread 2
Post by: TMEubanks on 02/16/2015 01:40 am
aero - I heartily encourage you to keep at it.
Title: Re: EM Drive Developments - related to space flight applications - Thread 2
Post by: aero on 02/16/2015 02:07 am
aero - I heartily encourage you to keep at it.

Thank You. I've been working on this learning meep since October. It's not likely that I will stop now.

Developing a consistent theory? That's a different question as I am not a theorist.
Title: Re: EM Drive Developments - related to space flight applications - Thread 2
Post by: Rodal on 02/16/2015 02:33 am
...
I have also cited this paper,  http://wwwsis.lnf.infn.it/pub/INFN-FM-00-04.pdf (http://wwwsis.lnf.infn.it/pub/INFN-FM-00-04.pdf) several times. No feedback what-so-ever.
....
(Bold added for emphasis)

aero - I heartily encourage you to keep at it.

I fully agree with heartily encouraging the pursuit of MEEP (or other) calculations to examine whether the EM Drive measurements are the result of evanescent waves interacting with the vacuum chamber or other components (how about the Neodymium (NdFeB Grade N42) block magnets used to dampen the oscillations of NASA Eagleworks torque pendulum, for example ?).

Concerning asking for feedback on   http://wwwsis.lnf.infn.it/pub/INFN-FM-00-04.pdf ,

The paper attempts to address Tolman's paradox (Tachyonic antitelephone http://en.wikipedia.org/wiki/Tachyonic_antitelephone  <<According to the current understanding of physics, no such faster-than-light transfer of information is actually possible. For instance, the hypothetical tachyon particles which give the device its name do not exist even theoretically in the standard model of particle physics, due to tachyon condensation, and there is no experimental evidence that suggests that they might exist. The problem of detecting tachyons via causal contradictions was treated scientifically>>)

 I can't see how the author's attempt to address Tolman's causal paradox (involving superluminal tachyons) can be used to support the EM Drive to work in outer space without any objects or extraneous-to-the-EMDrive fields around it to interact with the evanescent waves.  As far superlumnal tachyons, my understanding is that they are  hypothetical particles that always moves faster than light,  and no compelling evidence for their existence has been found.

The only way that Tolman's antitelephone could exist, I think, is by branching into another timeline in the context of the many-worlds interpretation of reality, and even if one were to accept that, I don't see the connection with the EM Drive.

If Marshall or others think that Recami's discussion of Tolmans's causal paradox with superluminal tachyons ( http://wwwsis.lnf.infn.it/pub/INFN-FM-00-04.pdf ) can be used to justify EM Drive motion in outer space  (without any surrounding objects, or extraneous-to-the-EMDrive electromagnetic fields, to interact with the evanescent waves),  I would be very interested in learning about it.
Title: Re: EM Drive Developments - related to space flight applications - Thread 2
Post by: ThinkerX on 02/16/2015 02:47 am
Ok...time for me to embarrass myself here yet again.

Assuming for the moment, this device works not just in a vacuum chamber, but in space...

1) The frequency necessary to generate thrust varies with the internal (?) temperature.

2) The longer this device runs, the hotter it gets - 'Star Drive' reports major heating issues with low power test setups running for a few minutes (?) at a time.  We are talking about a space version running nonstop for months or even years.  To me, that sounds like a pretty extreme temperature increase - hundreds (?) thousands (?) of degrees, maybe. 

So...would continual high temperatures damage the frustum over time?  Or other parts of the device?

And might not the high temperatures...hmmm...act somehow to 'put the brakes' on the continual acceleration - increase to the point where the required frequency for thrust becomes unattainable, or at least reduced?

Also...heat is energy, and this device, run for long periods in space, looks to produce a lot of heat.  Could a portion of that heat be tapped and converted into electrical power, thus reducing the over all power requirements?  Talking supplement here, not perpetual motion.

Ok, time to quit while I'm behind.
Title: Re: EM Drive Developments - related to space flight applications - Thread 2
Post by: TMEubanks on 02/16/2015 03:17 am
Well, since you asked, I didn't like that paper much. I have a feeling it is either wrong, or one of those papers that uses its own form of concepts so that it appears to say one thing while really saying another, but in either case it hasn't appealed to me enough to figure out exactly where it is wrong/misleading.

There are of course a number of cases where phase velocities are > c, and yet no information can be transmitted > c because the group velocity isn't. There are also cases where weird things can happen because the speed of light in a medium is < c, and energy can be transmitted faster than the speed of light _in the medium_.  I feel quite certain, however, that group velocities > c mean (at least informational) time travel. I don't see anyway around that.

So, specifically

I can't see how the author's attempt to address Tolman's causal paradox (involving superluminal tachyons) can be used to support the EM Drive to work in outer space without any objects or extraneous-to-the-EMDrive fields around it to interact with the evanescent waves. 

Fully agree.

As far superlumnal tachyons, my understanding is that they are  hypothetical particles that always moves faster than light,  and no compelling evidence for their existence has been found.

Fully agree.

Now, in this case, the entire chamber is in the near field of these 15 cm waves. Near fields can be pretty non-intuitive (for example, not weakening as 1/R^2). I would be especially wary of what's going on in the reactive near field (within 2 cm or so of the drive), which is likely to be non-intuitive even for near-fields.

Do we know if MEEP adequately models the reactive near field? I don't see signs of it in the plots, but I do not trust my intituition, so things may be fine, but I thought I would ask.

Also, please don't forget that static fields can cause forces. "It would be nice" to see a breakdown of force by cause here.
Title: Re: EM Drive Developments - related to space flight applications - Thread 2
Post by: aero on 02/16/2015 03:34 am
Quote
Do we know if MEEP adequately models the reactive near field? I don't see signs of it in the plots, but I do not trust my intituition, so things may be fine, but I thought I would ask.

To my knowledge, meep integrates the differential Maxwell equations so that anything addressed by the Maxwell equations is "solved" by numerical integration. This includes evanescent waves which are a solution. It integrates the Maxwell Stress Tensor similarly, so I am laboring under the impression that yes, meep does treat the reactive near fields. Some of the earlier plots I posted from a few days ago show the signs of what I think is the near field. You might look at this image, for example:
 http://forum.nasaspaceflight.com/index.php?topic=36313.msg1329561#msg1329561 (http://forum.nasaspaceflight.com/index.php?topic=36313.msg1329561#msg1329561)
Title: Re: EM Drive Developments - related to space flight applications - Thread 2
Post by: frobnicat on 02/16/2015 09:47 am
Ok...time for me to embarrass myself here yet again.

Assuming for the moment, this device works not just in a vacuum chamber, but in space...

1) The frequency necessary to generate thrust varies with the internal (?) temperature.


From what I understand, the resonant frequency shifts with geometry (dimensions of cavity) which shifts with temperature because of thermal dilatation. Such thermal dilatation can be hard to model because of non uniform heating (depending on standing waves mode) and non linear thermo-mechanical effects (buckling...). At high Q the resonant frequencies have very narrow bands, the slightest thermal shift in geometry of cavity will throw it off a constant frequency narrow band excitation. If it is necessary to excite at resonant frequency to generate thrust then the exciting frequency has to follow temperature (indirectly, because of geometry changing). Also the electric characteristic of dielectric (if any) would change a little (a lot ?) with temperature change and modify the target frequency.

Quote
2) The longer this device runs, the hotter it gets - 'Star Drive' reports major heating issues with low power test setups running for a few minutes (?) at a time.  We are talking about a space version running nonstop for months or even years.  To me, that sounds like a pretty extreme temperature increase - hundreds (?) thousands (?) of degrees, maybe. 

So...would continual high temperatures damage the frustum over time?  Or other parts of the device?

And might not the high temperatures...hmmm...act somehow to 'put the brakes' on the continual acceleration - increase to the point where the required frequency for thrust becomes unattainable, or at least reduced?

This is not an indefinite thermal accumulator, the higher the temperature the higher heat flow goes from the system to the cold surrounding at Tc, as for any energy dissipating system it will reach stationary equilibrium at some temperature Te.


Te                     -------------------
                  -----
               ---
             --
            -
Tc ---------

Temperature rising is a transient.  Passive or active radiators efficient in vacuum are heavy and bulky but not impossible : with enough "heat flow per Kelvin" a low enough equilibrium temperature is reached (for frustum and for microwave generator) and would allow the system to be operated indefinitely at constant power (and probably constant thrust). Unfortunately most (maybe not some of Shawyer's?) emdrive experiments so far have been too short in duration to show this plateau of thermal equilibrium, but this is "just" a matter of cost and weight of radiators with good dissipation.

Quote
Also...heat is energy, and this device, run for long periods in space, looks to produce a lot of heat.  Could a portion of that heat be tapped and converted into electrical power, thus reducing the over all power requirements?  Talking supplement here, not perpetual motion.

Ok, time to quit while I'm behind.

Thermocouples could recover of fraction of dissipated power... not sure if it would help or impede thermal management overall.
Title: Re: EM Drive Developments - related to space flight applications - Thread 2
Post by: Rodal on 02/16/2015 01:13 pm
...
Now, in this case, the entire chamber is in the near field of these 15 cm waves. Near fields can be pretty non-intuitive (for example, not weakening as 1/R^2). I would be especially wary of what's going on in the reactive near field (within 2 cm or so of the drive), which is likely to be non-intuitive even for near-fields.

Do we know if MEEP adequately models the reactive near field? I don't see signs of it in the plots, but I do not trust my intituition, so things may be fine, but I thought I would ask.
Good observation.  I do not see any evidence of the reactive near field at the boundaries  of the upper picture (attached below) either.
Perhaps it is due to the low fidelity of the color contour plots that only display color intensity and do not display contour lines, it would be better to also plot contour lines as contour lines would better show the reactive near field than just color intensity.

Perhaps it is due to how the boundary conditions were set up. I do not know how the boundary conditions for the Finite Difference solution were set up. 

I would be interested in both the near field on the EM Drive as well as the field lines next to the vacuum chamber.  Again, it may be due to the lack of contour lines, but what I see are waves going through the vacuum chamber as if there would be no boundaries in the upper picture below.  I agree with @aero that the bottom picture below shows near fields (particularly near the vacuum chamber corners) that make some sense to me.

To be rigorous, MEEP does not integrate Maxwell's equations, MEEP uses a finite difference solver to solve Maxwell's equations by finite differences both in space and time.

The answer to <<Do we know if MEEP adequately models the reactive near field>> is that since MEEP is a Finite Difference solver, its adequateness is completely dependent on the mesh discretization.  Just because a MEEP 3-D solution takes 20 hours for example, it does not mean that is near convergence to the correct solution.

(http://forum.nasaspaceflight.com/index.php?action=dlattach;topic=36313.0;attach=788387;image)

(http://forum.nasaspaceflight.com/index.php?action=dlattach;topic=36313.0;attach=705863;image)



If this is a 3-D solution, it would help to also see the circular cross-section of the vacuum chamber, perpendicular to the longitudinal axis of the vacuum chamber, to also better understand what is going on.  It is very difficult to tell what is going on in a 3-D problem when one only can see a rectangular cross-section (is it the rectangular cross-section that crosses the center of the vacuum chamber ?)

...
Also, please don't forget that static fields can cause forces. "It would be nice" to see a breakdown of force by cause here.

Another excellent point.  That would also help to better understand what is going on.

Good effort, please carry on  It is laudable to have someone at this forum dedicate so much time and effort towards a classical physics explanation of the EM Drive.
Title: Re: EM Drive Developments - related to space flight applications - Thread 2
Post by: aero on 02/16/2015 02:00 pm
Including the vacuum chamber requires a lattice size of 26x31 x no-size, so all runs will be 2D with the vacuum chamber included. I tried to run a 3D but ran out of memory before resolution got as large as 10. That is not enough resolution to detect small features of the thruster cavity model and so is not useful for our purposes.

After I finish my current set of runs I can remove the vacuum chamber from the model and shrink the lattice to perhaps get some useful 3D data of just the thruster cavity. Even then in 3D the resolution will be low.

As for whether or not meep integrates the equations, I think it is a matter of terminology. When I went to school we called what meep does "numerical integration." If you want to change my terminology and re-educate me so be it, but a finite difference solver is new terminology to me, and is very similar if not exactly what we called numerical integration in the early days of digital computers.
Title: Re: EM Drive Developments - related to space flight applications - Thread 2
Post by: Reactionless on 02/16/2015 02:01 pm
While this single run detected only 1/3 of the experimental measured value, I will brazenly write that I think we can forget about axions and dark matter. We can forget about exotic theory and consider that the thrust force likely results from a little understood characteristic of evanescent waves and the forces they generate.
Well, partly perhaps, but lets wait until I run the same model without the vacuum chamber. I have ran enough cases already that I feel confident that there will be forces in that case to.

Pardon the ignorance of this non-scientist, but with these results are you suggesting that the drive's thrust is only an interaction with the testing apparatus? In the second quote you say that you are confident that forces will be generated without a vacuum chamber. Does this mean that the drive would produce thrust in free space?
Title: Re: EM Drive Developments - related to space flight applications - Thread 2
Post by: TMEubanks on 02/16/2015 02:19 pm
In these particular runs (which I believe is just the Drive and the chamber) the reactive near field is probably irrelevant, but for modeling the Drive + pendulum suspension it won't be.

Does MEEP support variable mesh sizes? It may be necessary to decrease the mesh size near structures.

And, for that matter, what is the mesh size in these plots? I think the rule of thumb is < lambda / 40, which is a little less than 4 mm.

(Note, by the way, that evanescent, or exponentially decaying, waves are just one part of near field behavior and not all near field waves have to rapidly decay, particularly if you have something like a waveguide set up.)
Title: Re: EM Drive Developments - related to space flight applications - Thread 2
Post by: aero on 02/16/2015 02:23 pm
While this single run detected only 1/3 of the experimental measured value, I will brazenly write that I think we can forget about axions and dark matter. We can forget about exotic theory and consider that the thrust force likely results from a little understood characteristic of evanescent waves and the forces they generate.
Well, partly perhaps, but lets wait until I run the same model without the vacuum chamber. I have ran enough cases already that I feel confident that there will be forces in that case to.

Pardon the ignorance of this non-scientist, but with these results are you suggesting that the drive's thrust is only an interaction with the testing apparatus? In the second quote you say that you are confident that forces will be generated without a vacuum chamber. Does this mean that the drive would produce thrust in free space?

Lets wait a few days. Then I should be able to answer that with data.
Title: Re: EM Drive Developments - related to space flight applications - Thread 2
Post by: aero on 02/16/2015 02:31 pm
In these particular runs (which I believe is just the Drive and the chamber) the reactive near field is probably irrelevant, but for modeling the Drive + pendulum suspension it won't be.

Does MEEP support variable mesh sizes? It may be necessary to decrease the mesh size near structures.

And, for that matter, what is the mesh size in these plots? I think the rule of thumb is < lambda / 40, which is a little less than 4 mm.

(Note, by the way, that evanescent, or exponentially decaying, waves are just one part of near field behavior and not all near field waves have to rapidly decay, particularly if you have something like a waveguide set up.)

No - meep does not support variable mesh size. I suggested that it be added and was pointed to the source code and invited to program it in, or alternatively to commercial software that does support it.

Needless to say, I'm running meep with a uniform mesh. In this vacuum chamber/EM thruster model that characteristic can be viewed as a limitation because most of the lattice volume is empty so would need only a relatively few points to fully characterize the fields. But meep is free ...
Title: Re: EM Drive Developments - related to space flight applications - Thread 2
Post by: Rodal on 02/16/2015 02:34 pm
...

As for whether or not meep integrates the equations, I think it is a matter of terminology...
Numerical Integration involves approximating definite integrals by  summing discretized areas.  This is not what the Finite Difference method does. 

Sorry about being perhaps overly rigorous in the use of the word "integrate". I point this (finite? pun-intended  :) ) difference because it is important to understand the convergence problems with Finite Difference methods (as opposed to integration methods like the Boundary Element method, for example, or methods based on variational principles like the Finite Element Method).
What the Finite Difference method does is instead to approximate solutions to differential equations using finite differences to approximate derivatives. 
The idea of a finite difference method is the transformation of a continuity domain to a discrete set of points, the mesh. In every grid point the given differential operator is approximated by a difference-operator.

The issue is that numerically, numerical differentiation is always a much trickier problem than numerical integration (from a convergence viewpoint).

The Finite Difference method is a very old method (references going back to the 19th century) but great progress was made using it during and after World War II, due to the development of the digital computer, due to Von Neuman and Friederichs, mainly due to the Manhattan Project.

At MIT's ASRL very complex Finite Difference codes were developed, for example the PETROS code:
http://bit.ly/1AJ5Vgt in addition to Finite Element and other types of numerical analyses.
Title: Re: EM Drive Developments - related to space flight applications - Thread 2
Post by: TMEubanks on 02/16/2015 02:35 pm
Dear reactionless

Quote
Pardon the ignorance of this non-scientist, but with these results are you suggesting that the drive's thrust is only an interaction with the testing apparatus? In the second quote you say that you are confident that forces will be generated without a vacuum chamber. Does this mean that the drive would produce thrust in free space?

1.) "are you suggesting that the drive's thrust is only an interaction with the testing apparatus?" I personally feel that that is the way to bet. In any case, such interactions must be exhaustively ruled out before we can consider this to be a "real" (i.e., new physics) effect. This is a small effect (10 micronewtons is the force a 1 mm chunk of ice exerts sitting on your hand), and this will not be trivial.

2.) "In the second quote you say that you are confident that forces will be generated without a vacuum chamber. Does this mean that the drive would produce thrust in free space?" Not necessarily (that is, after all, what we are trying to find out). I did not say this, but I believe that previous MEEP runs did estimate a force for a Drive only run (which was  an artifact, not a real free space force), and that was what was being referred to.

This is not trivial, and fully solving it may be beyond the resources that the amateurs on this forum can reasonably be expected to bring to bear, but it doesn't mean we can't have some fun, and maybe bring some useful things to light, in the mean time. 
Title: Re: EM Drive Developments - related to space flight applications - Thread 2
Post by: Notsosureofit on 02/16/2015 02:53 pm
...

As for whether or not meep integrates the equations, I think it is a matter of terminology...
Numerical Integration involves approximating definite integrals by  summing discretized areas.  This is not what the Finite Difference method does. 
I point the difference because it is important to understand the convergence problems with Finite Difference methods (as opposed to integration methods like the Boundary Element method, for example, or methods based on variational principles like the Finite Element Method).
What the Finite Difference method does is instead to approximate solutions to differential equations using finite differences to approximate derivatives. 
The idea of a finite difference method is the transformation of a continuity domain to a discrete set of points, the mesh. In every grid point the given differential operator is approximated by a difference-operator.
The Finite Difference method is a very old method (references going back to the 19th century) but great progress was made using it during and after World War II, due to the development of the digital computer, due to Von Neuman and Friederichs.

At MIT's ASRL very complex Finite Difference codes were developed, for example the PETROS code:
http://bit.ly/1AJ5Vgt in addition to Finite Element and other types of numerical analyses.

Yes, I remember having a Digital Differential Analyzer (w/ a magnetic drum memory) but replaced it w/ a PDP-4 (Fortran 4K memory) before getting it running.
Title: Re: EM Drive Developments - related to space flight applications - Thread 2
Post by: Reactionless on 02/16/2015 02:59 pm
1.) "are you suggesting that the drive's thrust is only an interaction with the testing apparatus?" I personally feel that that is the way to bet. In any case, such interactions must be exhaustively ruled out before we can consider this to be a "real" (i.e., new physics) effect. This is a small effect (10 micronewtons is the force a 1 mm chunk of ice exerts sitting on your hand), and this will not be trivial.

Is there any way for this to be controlled for at this point in the experimentation, or does it require a larger or differently configured vacuum chamber (does the GRC have one that would control for this)?
Title: Re: EM Drive Developments - related to space flight applications - Thread 2
Post by: Rodal on 02/16/2015 03:15 pm
Including the vacuum chamber requires a lattice size of 26x31 x no-size, so all runs will be 2D with the vacuum chamber included. I tried to run a 3D but ran out of memory before resolution got as large as 10. That is not enough resolution to detect small features of the thruster cavity model and so is not useful for our purposes.

....
Does 2-D here mean a

1) a flat plane geometry (as in a Euclidean x,y coordinate plane), or are you solving

2) the  problem in the circumferential direction by assuming axisymmetry, using Fourier analysis in the circumferential space coordinate?

==> My guess is that the answer is 1) (a flat plane) and that's why it is not possible to produce circular cross-sections of the results

Does 3-D mean

3)a solution in a Euclidean "brick" x , y , z ? or does it mean

4) a solution assuming axysymmetry?

Thanks
Title: Re: EM Drive Developments - related to space flight applications - Thread 2
Post by: WBY1984 on 02/16/2015 03:18 pm
I've been following this thread as a lurker for a little while, but my knowledge is serverely lacking and have a lot of difficulty understanding what is being said. Is it the consensus at this point that it isn't a real effect, but merely experimental artefact?
Title: Re: EM Drive Developments - related to space flight applications - Thread 2
Post by: TMEubanks on 02/16/2015 03:20 pm
Well, you can calculate it, or you can try and devise better experiments (e.g., the same test in a fiberglass chamber). In the end, I suspect it will take a test in space to really be sure, but that's expensive, so it is entirely proper to make it jump through all kinds of hoops here on the ground first. (And, note, it is quite possible that it will either be rejected or just fade away in the process.)
Title: Re: EM Drive Developments - related to space flight applications - Thread 2
Post by: Rodal on 02/16/2015 03:22 pm
...
No - meep does not support variable mesh size. ..
That's useful information, as a uniform Finite Difference mesh size implies extremely long computer running times to get convergence in regions of steep gradient of the electromagnetic field.  Otherwise steep gradient regions will not be well modeled.
Title: Re: EM Drive Developments - related to space flight applications - Thread 2
Post by: TMEubanks on 02/16/2015 03:27 pm
Quote
That's useful information, as a uniform Finite Difference mesh size implies extremely long computer running times to get convergence in regions of steep gradient of the electromagnetic field.  Otherwise steep gradient regions will not be well modeled.

This raises the question - do we know if NASA is doing this kind of modeling? Maybe we could get computational resources from NASA.
Title: Re: EM Drive Developments - related to space flight applications - Thread 2
Post by: RotoSequence on 02/16/2015 03:29 pm
I've been following this thread as a lurker for a little while, but my knowledge is serverely lacking and have a lot of difficulty understanding what is being said. Is it the consensus at this point that it isn't a real effect, but merely experimental artefact?

It's generally safe to assume that anomalous outcomes are the product of an artifact in the experiment, rather than hitherto undiscovered or unrecognized physical effect(s). Right now, the hope is to produce conclusive proof that the positive thrust reports are either the product of interactions with the testing apparatus, or are the real product of new physics. We don't have a clear answer yet, but we're making headway.  :)
Title: Re: EM Drive Developments - related to space flight applications - Thread 2
Post by: Notsosureofit on 02/16/2015 03:37 pm
... or a new product of old physics
Title: Re: EM Drive Developments - related to space flight applications - Thread 2
Post by: Rodal on 02/16/2015 03:40 pm
Well, you can calculate it, or you can try and devise better experiments (e.g., the same test in a fiberglass chamber). In the end, I suspect it will take a test in space to really be sure, but that's expensive, so it is entirely proper to make it jump through all kinds of hoops here on the ground first. (And, note, it is quite possible that it will either be rejected or just fade away in the process.)

Marshall, how about these methods proposed by Mulletron to test whether the EM Drive thrust is due to evanescent wave interaction:

I can think of 3 ways to test the evanescent wave theory.

1) Is the measured thrust the same with the chamber door open and closed?

2) Is the thrust still there when the test article is rolled out of the chamber. Not sure if 2 is possible.....

3) Change the conditions near the resonant cavity; like wrap the thing in thick foam and then wrap all that with foil, see what the thrust does.

...
Title: Re: EM Drive Developments - related to space flight applications - Thread 2
Post by: TMEubanks on 02/16/2015 04:06 pm
Quote
Marshall, how about these methods proposed by Mulletron to test whether the EM Drive thrust is due to evanescent wave interaction:

Jose, these are all sensible, but it is not clear if they will be conclusive. E.g., if you open the door you get air currents, and thus a different anomalous thrust.

I worry about the mount and pendulum. Can those be insulated? Made of plastic?

Have any tests been done reversing the drive?
Title: Re: EM Drive Developments - related to space flight applications - Thread 2
Post by: TMEubanks on 02/16/2015 04:09 pm
Quote
Change the conditions near the resonant cavity; like wrap the thing in thick foam and then wrap all that with foil, see what the thrust does.

I would definitely recommend something like this be done.
Title: Re: EM Drive Developments - related to space flight applications - Thread 2
Post by: aero on 02/16/2015 04:11 pm
@TMEubanks
Regarding your question:
Quote
what is the mesh size in these plots? I think the rule of thumb is < lambda / 40, which is a little less than 4 mm.

We can get a handle on that by looking at the image properties, knowing that meep computes a data point for each pixel of the image, and that the general dimensions of the vacuum chamber are 30" x 36". The image pixel dimensions are 3612 x 4354. That converts to about 0.23 mm/pixel which is spacial resolution. (The boundry layer adds some overhead.) It is a rough order of magnitude higher resolution than required by the fields, but considering that the 0.002" gaps at the attachment points of the base plates and cone body is only 0.0508 mm I need to run at very high resolution or those gaps won't be resolved. Even so, I am now using 5.5 times the design gap size in order that it will be resolved and allow RF energy to escape via the gap in the model. I'm only slightly concerned about this because the forces detected by meep get larger as the gap size gets smaller. If it were the other way around then it would be a bigger concern.

Of course in the real world, RF energy can escape via gaps at much smaller sizes or so I think.

@Rodal
 Meep can compute in one, two or three dimensions, x - x,y - x,y,z Cartesian and cylindrical.

Meep advertises a built-in method to utilize symmetry of the model but the vacuum chamber/EM thruster model is not symmetric. And in any case there seems to be a problem in reconstructing the output files correctly for the magnetic fields when mirror symmetry is utilized. Dr. Dominic, whom I might call my meep mentor, avoids using symmetry in his models, hence so do I.
Title: Re: EM Drive Developments - related to space flight applications - Thread 2
Post by: Reactionless on 02/16/2015 04:12 pm
Have any tests been done reversing the drive?

http://nextbigfuture.com/2015/02/update-on-emdrive-work-at-nasa.html

Quote
They have now confirmed that there is a thrust signature in a hard vacuum (~5.0x10^-6 Torr) in both the forward direction, (approx. +50 micro-Newton (uN) with 50W at 1,937.115 MHz), and the reversed direction, (up to -16uN with a failing RF amp), when the thruster is rotated 180 degrees on the torque pendulum.
Title: Re: EM Drive Developments - related to space flight applications - Thread 2
Post by: Rodal on 02/16/2015 04:13 pm
.....

Have any tests been done reversing the drive?
Yes, and they have done it (reversing the drive) under hard vacuum.

(http://forum.nasaspaceflight.com/index.php?action=dlattach;topic=36313.0;attach=635481;image)

(http://forum.nasaspaceflight.com/index.php?action=dlattach;topic=36313.0;attach=635478;image)
Title: Re: EM Drive Developments - related to space flight applications - Thread 2
Post by: Star One on 02/16/2015 04:15 pm

.....

Have any tests been done reversing the drive?
Yes, and they have done it (reversing the drive) under hard vacuum.

Which seems to be a rather significant point.
Title: Re: EM Drive Developments - related to space flight applications - Thread 2
Post by: Reactionless on 02/16/2015 04:19 pm
Which seems to be a rather significant point.

If it was only an interaction with the vacuum chamber or pendulum, would the thrust direction being reversed on reversing the drive be something that was expected as well, or is this a strike against the chamber/pendulum interaction explanation?

They also observed a reversal of thrust under different modes, even when they kept the orientation the same.
Title: Re: EM Drive Developments - related to space flight applications - Thread 2
Post by: TMEubanks on 02/16/2015 04:20 pm
Quote
They have now confirmed that there is a thrust signature in a hard vacuum (~5.0x10^-6 Torr) in both the forward direction, (approx. +50 micro-Newton (uN) with 50W at 1,937.115 MHz), and the reversed direction, (up to -16uN with a failing RF amp), when the thruster is rotated 180 degrees on the torque pendulum.

This needs to be redone.

+50 and -16 is evidence of systematic errors. It is simply not good enough to say (or imply), well, if we could redo it properly, it would be +- 50. That needs to be demonstrated (or not).
Title: Re: EM Drive Developments - related to space flight applications - Thread 2
Post by: Star One on 02/16/2015 04:23 pm

Quote
They have now confirmed that there is a thrust signature in a hard vacuum (~5.0x10^-6 Torr) in both the forward direction, (approx. +50 micro-Newton (uN) with 50W at 1,937.115 MHz), and the reversed direction, (up to -16uN with a failing RF amp), when the thruster is rotated 180 degrees on the torque pendulum.

This needs to be redone.

+50 and -16 is evidence of systematic errors. It is simply not good enough to say (or imply), well, if we could redo it properly, it would be +- 50. That needs to be demonstrated (or not).

Give them a chance. Your enthusiasm is good but time is the best response here. Anyway Star Drive may if he is able to answer more specifically.:)
Title: Re: EM Drive Developments - related to space flight applications - Thread 2
Post by: Star-Drive on 02/16/2015 05:32 pm

Quote
They have now confirmed that there is a thrust signature in a hard vacuum (~5.0x10^-6 Torr) in both the forward direction, (approx. +50 micro-Newton (uN) with 50W at 1,937.115 MHz), and the reversed direction, (up to -16uN with a failing RF amp), when the thruster is rotated 180 degrees on the torque pendulum.

This needs to be redone.

+50 and -16 is evidence of systematic errors. It is simply not good enough to say (or imply), well, if we could redo it properly, it would be +- 50. That needs to be demonstrated (or not).

Give them a chance. Your enthusiasm is good but time is the best response here. Anyway Star Drive may if he is able to answer more specifically.:)

Folks:

The copper frustum reversed thrust data was started just before our second EMPower Model-1165 RF amp started  dying due to internal corona discharges around its RF output line created when its internal pressure had leaked down from 760 Torr on the Friday afternoon just before the MLK Holiday here in the USA.  This data shot was obtained about an hour after the vacuum pumps were turned on Friday evening.  When I cam back on the following Tuesday with the vacuum pumps going all weekend, I tried the same test, but the amp would just turn on and drawn the right dc current, but would only deliver about 25 milli-watts of RF no matter what the input level was.  Why? Because the RF was just creating a nice blue glow in the amplifier...

We should have the repaired RF amps back in the lab by Wednesday morning and the reverse thrust testing is where I'll be picking up again at that time.

Best, Paul M.
Title: Re: EM Drive Developments - related to space flight applications - Thread 2
Post by: TMEubanks on 02/16/2015 07:23 pm
Excellent news! Good luck.
Title: Re: EM Drive Developments - related to space flight applications - Thread 2
Post by: Mulletron on 02/16/2015 07:53 pm
How hard would it be to put the 1165 inside a more vacuum resilient container?

http://www.empowerrf.com/pdfs/1165.pdf

Notice the paper says max altitude 50K feet!

It is operating beyond specs in that vacuum.....why they keep dying!

We gotta figure this out, pronto!
Title: Re: EM Drive Developments - related to space flight applications - Thread 2
Post by: frobnicat on 02/16/2015 08:41 pm
Well, you can calculate it, or you can try and devise better experiments (e.g., the same test in a fiberglass chamber). In the end, I suspect it will take a test in space to really be sure, but that's expensive, so it is entirely proper to make it jump through all kinds of hoops here on the ground first. (And, note, it is quite possible that it will either be rejected or just fade away in the process.)

Marshall, how about these methods proposed by Mulletron to test whether the EM Drive thrust is due to evanescent wave interaction:

I can think of 3 ways to test the evanescent wave theory.

1) Is the measured thrust the same with the chamber door open and closed?

2) Is the thrust still there when the test article is rolled out of the chamber. Not sure if 2 is possible.....

3) Change the conditions near the resonant cavity; like wrap the thing in thick foam and then wrap all that with foil, see what the thrust does.

...

As for 3) : how about putting aluminium plates fixed to the chamber (not on the balance arm) very close to the frustum at its rest position, an inch or so from the small and/or big end. If the thrust is due to coupling of leaked radiation between moving frustum and nearby fixed parts (chamber rest frame) this could show a strong correlation. This should be relatively easy to do and wouldn't modify the weight on balance or other dynamic parameters of experiment.

Maybe Aero you could tell from simulation what kind of influence on apparent thrust would be expected from putting such shield very close to frustum ?



Title: Re: EM Drive Developments - related to space flight applications - Thread 2
Post by: birchoff on 02/16/2015 08:43 pm
...
Have any tests been done reversing the drive?

Both the test run that was reported on in Brady et al and the test run that Star-Drive talked about have included running the drive in a reverse orientation.
Title: Re: EM Drive Developments - related to space flight applications - Thread 2
Post by: Mulletron on 02/16/2015 08:56 pm
Plugged in some numbers to come up with the near field reactive and near field radiating using 2ghz and largest dimension of frustum 280mm.

Near field reactive: .024m
Near field radiating: .261m

But there is a problem with this near field scenario. This is for antennas. The frustum isn't an antenna. The outer surface is also grounded through the coax cable at a minimum. Thus this type of evanescent wave interaction is not an option.
References:
https://www.ic.gc.ca/eic/site/smt-gst.nsf/eng/sf10112.html
http://en.wikipedia.org/wiki/Near_and_far_field#Near-field_characteristics



Another possible method of evanescent wave production by TIRF is an option however, which is an optical phenomenon associated with boundary conditions of two different refractive indexes. I remain unconvinced that this applies to Emdrive solely because the air/copper/air boundary does not fit this scenario. Copper is a conductor, thus any evanescent E field component will completely vanish. Here's info from an expert over at Polywell:
http://www.talk-polywell.org/bb/viewtopic.php?f=10&t=2949&start=270#p118973

Quote
Yes, evanescent waves are propagating waves. They propagate in the direction of the interface, so their momentum is parallel to the boundary. It is just their amplitude that decays exponentially away from the interface layer.
So in this context, evanescent waves "hug" the boundary. The propagate parallel to the boundary. They do not radiate away from the boundary. Their amplitude falls exponentially with distance, unlike radiating waves which fall off with inverse square. The above quote from Polywell matches any check of the literature online.

I am willing to play devil's advocate for a second and accept that evanescent waves do appear on the outer surface of the conical frustum.
Quoting the oracle, http://en.wikipedia.org/wiki/Evanescent_wave
Quote
They are formed at the boundary between two media with different wave motion properties, and are most intense within one third of a wavelength from the surface of formation.
The 1/3 wavelength figure also appears in other references. Evanescent fields would be most intense within .05m or ~2 inches from the conical frustum. Any structure within that range is subject to scrutiny.

We know, as was recent reported that this kind of evanescent wave can impart momentum and spin on particles in the following manner.

Quote
Momentum and spin represent fundamental dynamic properties of quantum particles and fields. In particular, propagating optical waves (photons) carry momentum and longitudinal spin determined by the wave vector and circular polarization, respectively. Here we show that exactly the opposite can be the case for evanescent optical waves. A single evanescent wave possesses a spin component, which is independent of the polarization and is orthogonal to the wave vector. Furthermore, such a wave carries a momentum component, which is determined by the circular polarization and is also orthogonal to the wave vector.

Thus for this type of evanescent wave to be applicable to Emdrive, there would have to be circularly polarized wave in use. The Emdrive is excited by linearly polarized plane waves. If it were excited by circularly polarized radiation, the momentum vector imparted on any particle nearby would be orthogonal to the direction of measured thrust. Under vacuum testing, these particles were removed. Thus this evanescent wave hypothesis does not hold.

http://www.nature.com/ncomms/2014/140306/ncomms4300/full/ncomms4300.html
http://arxiv.org/ftp/arxiv/papers/1308/1308.0547.pdf

(http://cdn.phys.org/newman/gfx/news/2014/extraordinar.jpg)

Now that I'm done playing devil's advocate, these are the only ways I can find in the literature to "tunnel" radiation out of a waveguide. You have to run it at cutoff and use metamaterials. None of these conditions fit Emdrive.
http://personal.us.es/marques/2005-PhysRevB_72_075116.pdf
http://people.ee.duke.edu/~cummer/reprints/074_Liu08_PRL_EMTunnelingMeasurements.pdf

A final method of producing evanescent waves is to operate the waveguide at or below cutoff. Given the .159 meter diameter of the small end, cutoff is 1886.79mhz. Any frequency lower than this would go evanescent inside the cavity. The lowest frequency in which a certain mode can propagate is the cutoff frequency of that mode. Evanescent modes are modes below the cutoff frequency. They cannot propagate down the waveguide for any distance, dying away exponentially. The only test that got close to cutoff was the TE012 test at 1880.4 from Brady et al, but this does not count as the frustum was loaded with PE, which displaced the E and H fields, changing the resonant frequency. So this does not count as running the device below cutoff, as it wouldn't resonate anyway if it were in cutoff.

Quote
Shawyer's statement, "The small end diameters are set just above the cut-off diameter corresponding to the mode and frequency of the design."
http://forum.nasaspaceflight.com/index.php?topic=29276.msg1275094#msg1275094

Lastly it is well known from studying evanescent wave coupling and resonant inductive coupling that evanescent modes will NOT couple to another medium unless the other medium is resonant at that exact mode. So this mode of interaction is not a possibility.
http://en.wikipedia.org/wiki/Evanescent_wave#Evanescent-wave_coupling
http://en.wikipedia.org/wiki/Resonant_inductive_coupling
http://en.wikipedia.org/wiki/Wireless_power

I think this definitively puts evanescent wave theories to bed.



Title: Re: EM Drive Developments - related to space flight applications - Thread 2
Post by: Rodal on 02/16/2015 08:57 pm
...
Have any tests been done reversing the drive?

Both the test run that was reported on in Brady et al and the test run that Star-Drive talked about have included running the drive in a reverse orientation.
To be thorough, if "the drive" means the pillbox-shaped cavity that's correct, if the "the drive" means the truncated cone (frustum), no:

Only the Cannae pillbox cavity was reported to run in a reversed configuration in the Brady et.al. report ( http://www.libertariannews.org/wp-content/uploads/2014/07/AnomalousThrustProductionFromanRFTestDevice-BradyEtAl.pdf ), on test runs 1B and 2B, as reported in the text, and summarized in Table 1. Cannae Testing: Summary of Results and Conclusions and illustrated on Figure 11. Cannae Test Article on Torsion Pendulum (thrust to the right, a.k.a. reverse orientation).

The Brady et.al. paper does not report a single test with the truncated cone (frustum) run in the reverse configuration.

The first instance in which the truncated cone was reported to have been run in a reverse configuration is the recent disclosure by Paul March, under hard vacuum.

(http://forum.nasaspaceflight.com/index.php?action=dlattach;topic=36313.0;attach=635481;image)
Title: Re: EM Drive Developments - related to space flight applications - Thread 2
Post by: birchoff on 02/16/2015 09:33 pm
...
Have any tests been done reversing the drive?

Both the test run that was reported on in Brady et al and the test run that Star-Drive talked about have included running the drive in a reverse orientation.
To be thorough, if "the drive" means the pillbox-shaped cavity that's correct, if the "the drive" means the truncated cone (frustum), no:

Only the Cannae pillbox cavity was reported to run in a reversed configuration in the Brady et.al. report ( http://www.libertariannews.org/wp-content/uploads/2014/07/AnomalousThrustProductionFromanRFTestDevice-BradyEtAl.pdf ), on test runs 1B and 2B, as reported in the text, and summarized in Table 1. Cannae Testing: Summary of Results and Conclusions and illustrated on Figure 11. Cannae Test Article on Torsion Pendulum (thrust to the right, a.k.a. reverse orientation).

The Brady et.al. paper does not report a single test with the truncated cone (frustum) run in the reverse configuration.

The first instance in which the truncated cone was reported to have been run in a reverse configuration is the recent disclosure by Paul March, under hard vacuum.

(http://forum.nasaspaceflight.com/index.php?action=dlattach;topic=36313.0;attach=635481;image)

Thanks for setting me straight on that.  I believe I did get the pill box and frustum results mixed up in my memory.
Title: Re: EM Drive Developments - related to space flight applications - Thread 2
Post by: birchoff on 02/16/2015 09:38 pm
Aside from re running the reverse orientation tests. Are there any other tests that Eagleworks needs to run. I am aware that they need to get to a certain performance level before then can hand off for replication attempt. But for the life of me the only other test case I can think of wanting results for is

* Re Run frustum reverse orientation in Hard vacuum
* Run forward and reverse orientation of frustum in a null configuration
Title: Re: EM Drive Developments - related to space flight applications - Thread 2
Post by: Star One on 02/16/2015 09:44 pm

Aside from re running the reverse orientation tests. Are there any other tests that Eagleworks needs to run. I am aware that they need to get to a certain performance level before then can hand off for replication attempt. But for the life of me the only other test case I can think of wanting results for is

* Re Run frustum reverse orientation in Hard vacuum
* Run forward and reverse orientation of frustum in a null configuration

If it is handed off for a replication attempt, is this to be done in more than one other location, in other words are multiple teams to attempt this or just one?
Title: Re: EM Drive Developments - related to space flight applications - Thread 2
Post by: birchoff on 02/16/2015 09:55 pm

Aside from re running the reverse orientation tests. Are there any other tests that Eagleworks needs to run. I am aware that they need to get to a certain performance level before then can hand off for replication attempt. But for the life of me the only other test case I can think of wanting results for is

* Re Run frustum reverse orientation in Hard vacuum
* Run forward and reverse orientation of frustum in a null configuration

If it is handed off for a replication attempt, is this to be done in more than one other location, in other words are multiple teams to attempt this or just one?

My understanding is given what was said in the conclusion of the Brady et al paper is that they want to create a testable unit to be used by Glen Research Center and JPL and Johns Hopkins. However, I believe of the two NASA centers only Glen has signed on. No clue on whether or not Johns Hopkins has also signed on. Which is the reason they need to get the thrust levels up because the Balance at Glen has a much higher floor of detectable thrust.
Title: Re: EM Drive Developments - related to space flight applications - Thread 2
Post by: Rodal on 02/16/2015 10:34 pm
How hard would it be to put the 1165 inside a more vacuum resilient container?

http://www.empowerrf.com/pdfs/1165.pdf

Notice the paper says max altitude 50K feet!

It is operating beyond specs in that vacuum.....why they keep dying!

We gotta figure this out, pronto!
This appears to be a great finding, Mulletron, thanks for pointing it out.  If the Amp that Eagleworks has been using is not rated for the hard vacuum (5*10^(-4) Torr) in their tests, do you have a best suggestion on how they should proceed?
Title: Re: EM Drive Developments - related to space flight applications - Thread 2
Post by: Mulletron on 02/16/2015 10:38 pm
How hard would it be to put the 1165 inside a more vacuum resilient container?

http://www.empowerrf.com/pdfs/1165.pdf

Notice the paper says max altitude 50K feet!

It is operating beyond specs in that vacuum.....why they keep dying!

We gotta figure this out, pronto!
This appears to be a great finding, Mulletron, thanks for pointing it out.  If the Amp that Eagleworks has been using is not rated for the hard vacuum (5*10^(-4) Torr) in their tests, do you have a best suggestion on how they should proceed?
Nope, sealing things up like that is beyond my expertise. No clue how to help. Anyone?

So why does it have to be inside the chamber again? Can't rf be piped in somehow?
Title: Re: EM Drive Developments - related to space flight applications - Thread 2
Post by: D_Dom on 02/16/2015 11:23 pm
Piping RF into the chamber is interesting, here is a link with various ports as an example.
http://www.designright.com/vacuum1.html

edited with a better source
Title: Re: EM Drive Developments - related to space flight applications - Thread 2
Post by: aero on 02/16/2015 11:51 pm
@Mulletron
Quote
I think this definitively puts evanescent wave theories to bed.

It does if we credit you with knowing everything that is to be known about evanescent waves.

I recognize that you have researched the literature on this topic extensively and while I am willing to credit you with knowing everything that is known about evanescent waves, I'm not prepared to take that last step and credit that that is everything that is to be known.

We know that evanescent waves couple quite strongly with identically generated waves out of phase, if the sources are near enough to each other. There could be and probably is this type of coupling between the two ends of the cavity. That coupling should only stress the cavity material though and I don't know how it would produce thrust.

Dr. Rodel seems to have shot down my thought of evanescent wave photons escaping the cavity superluminally even though there have been several papers published claiming that evanescent waves, under the right conditions, do carry superluminal momentum. (Google it.)

So we're coming around to the thought that there may be a whole bunch of electromagnetic energy in the vacuum chamber in the form of waves, some of them of the correct wavelength and phase to couple with the evanescent waves escaping from the thruster cavity. If such waves are transient, then could the ... and so forth. Isn't this Dr. White's theory?
Title: Re: EM Drive Developments - related to space flight applications - Thread 2
Post by: Star-Drive on 02/16/2015 11:57 pm
How hard would it be to put the 1165 inside a more vacuum resilient container?

http://www.empowerrf.com/pdfs/1165.pdf

Notice the paper says max altitude 50K feet!

It is operating beyond specs in that vacuum.....why they keep dying!

We gotta figure this out, pronto!
This appears to be a great finding, Mulletron, thanks for pointing it out.  If the Amp that Eagleworks has been using is not rated for the hard vacuum (5*10^(-4) Torr) in their tests, do you have a best suggestion on how they should proceed?
Nope, sealing things up like that is beyond my expertise. No clue how to help. Anyone?

So why does it have to be inside the chamber again? Can't rf be piped in somehow?

Folks:

The reason I thought that the EMPower unit could take vacuum is that the first page of the data sheet indicated that they are "hermetically" sealed.  At NASA hermetically sealed always means vacuum rated.  The Air Force and her contractors obviously have another definition of that word, but my bad for assuming it meant the NASA definition.  In any regards and as I stated earlier, EMPower has given us permission to just drill a hole into the top plate of their amp's chassis so it can vent to hard vacuum conditions when operating in same, since there are no components in their unit that would degrade over time in a hard vacuum such as electrolytic caps.

Best, Paul M.
Title: Re: EM Drive Developments - related to space flight applications - Thread 2
Post by: Notsosureofit on 02/17/2015 12:01 am
@ RODAL

Arrgh, Mondays !

Looked over my bleary weekend, noticed I was using diameters AGAIN !

Mode   Frequency (MHz)  Quality Factor, Q   Input Power (W)  Mean Thrust (μN)   Calculated w/o
                                                                                                                          dielectric
TE012     1880.4               22000                         2.6                55.4                   10.8
TM212   1932.6                 7320                        16.9                91.2                   38.5
TM212   1936.7               18100                        16.7                50.1                   93.5
TM212    1937.115             6726                       50                   66                    104.0

Anyway, shows it pays to rewrite everything in the same place !

@ aero

In the equivalence calc, the lower Doppler freq component, as seen in the accelerated frame, is evanescent. (that is its less than the cutoff in the rest frame)
Title: Re: EM Drive Developments - related to space flight applications - Thread 2
Post by: Star-Drive on 02/17/2015 12:05 am
@Mulletron
Quote
I think this definitively puts evanescent wave theories to bed.

It does if we credit you with knowing everything that is to be known about evanescent waves.

I recognize that you have researched the literature on this topic extensively and while I am willing to credit you with knowing everything that is known about evanescent waves, I'm not prepared to take that last step and credit that that is everything that is to be known.

We know that evanescent waves couple quite strongly with identically generated waves out of phase, if the sources are near enough to each other. There could be and probably is this type of coupling between the two ends of the cavity. That coupling should only stress the cavity material though and I don't know how it would produce thrust.

Dr. Rodel seems to have shot down my thought of evanescent wave photons escaping the cavity superluminally even though there have been several papers published claiming that evanescent waves, under the right conditions, do carry superluminal momentum. (Google it.)

So we're coming around to the thought that there may be a whole bunch of electromagnetic energy in the vacuum chamber in the form of waves, some of them of the correct wavelength and phase to couple with the evanescent waves escaping from the thruster cavity. If such waves are transient, then could the ... and so forth. Isn't this Dr. White's theory?

Aero:

What field strength are you estimating to be in between the frustum and the interior of the vacuum chamber in watts per square meter to get the forces you originally calculated for your evanescent wave solution when assuming the that there were Teflon seals in between the two end caps and the copper frustum body?  We've already measured that value at several locations in the chamber...

Best,  Paul M.
Title: Re: EM Drive Developments - related to space flight applications - Thread 2
Post by: Mulletron on 02/17/2015 12:08 am
@Mulletron
Quote
I think this definitively puts evanescent wave theories to bed.

It does if we credit you with knowing everything that is to be known about evanescent waves.

I recognize that you have researched the literature on this topic extensively and while I am willing to credit you with knowing everything that is known about evanescent waves, I'm not prepared to take that last step and credit that that is everything that is to be known.

We know that evanescent waves couple quite strongly with identically generated waves out of phase, if the sources are near enough to each other. There could be and probably is this type of coupling between the two ends of the cavity. That coupling should only stress the cavity material though and I don't know how it would produce thrust.

Dr. Rodel seems to have shot down my thought of evanescent wave photons escaping the cavity superluminally even though there have been several papers published claiming that evanescent waves, under the right conditions, do carry superluminal momentum. (Google it.)

So we're coming around to the thought that there may be a whole bunch of electromagnetic energy in the vacuum chamber in the form of waves, some of them of the correct wavelength and phase to couple with the evanescent waves escaping from the thruster cavity. If such waves are transient, then could the ... and so forth. Isn't this Dr. White's theory?

I'm not hostile to evanescent waves. I've spent months reading about them. I have lots to learn. The only analogue to have evanescent waves production like this is using light and thin metal films. Thin metals every time I look. For example, copper actually does have a refractive index if it is really thin. Thin films of metal behave like dielectrics. This would satisfy the surface plasmon polaritron idea. But the thickness of the frustum is many many times thicker than skin depth at 2ghz of 1.45um to 1.67um depending on what resource I use.
http://en.wikipedia.org/wiki/Surface_plasmon
http://en.wikipedia.org/wiki/Surface_plasmon_polariton
http://en.wikipedia.org/wiki/Surface_plasmon_resonance
http://refractiveindex.info/?shelf=main&book=Cu&page=Rakic
http://www.microwaves101.com/encyclopedias/skin-depth-calculator

I'm very sure it isn't evanescent waves. However that doesn't stop anybody from proving me wrong.
Title: Re: EM Drive Developments - related to space flight applications - Thread 2
Post by: aero on 02/17/2015 12:18 am
@Mulletron
Quote
I think this definitively puts evanescent wave theories to bed.

It does if we credit you with knowing everything that is to be known about evanescent waves.

I recognize that you have researched the literature on this topic extensively and while I am willing to credit you with knowing everything that is known about evanescent waves, I'm not prepared to take that last step and credit that that is everything that is to be known.

We know that evanescent waves couple quite strongly with identically generated waves out of phase, if the sources are near enough to each other. There could be and probably is this type of coupling between the two ends of the cavity. That coupling should only stress the cavity material though and I don't know how it would produce thrust.

Dr. Rodel seems to have shot down my thought of evanescent wave photons escaping the cavity superluminally even though there have been several papers published claiming that evanescent waves, under the right conditions, do carry superluminal momentum. (Google it.)

So we're coming around to the thought that there may be a whole bunch of electromagnetic energy in the vacuum chamber in the form of waves, some of them of the correct wavelength and phase to couple with the evanescent waves escaping from the thruster cavity. If such waves are transient, then could the ... and so forth. Isn't this Dr. White's theory?

I'm not hostile to evanescent waves. I've spent months reading about them. I have lots to learn. The only analogue to have evanescent waves production like this is using light and thin metal films. Thin metals every time I look. For example, copper actually does have a refractive index if it is really thin. Thin films of metal behave like dielectrics. This would satisfy the surface plasmon polaritron idea. But the thickness of the frustum is many many times thicker than skin depth at 2ghz of 1.45um to 1.67um depending on what resource I use.
http://en.wikipedia.org/wiki/Surface_plasmon
http://en.wikipedia.org/wiki/Surface_plasmon_polariton
http://en.wikipedia.org/wiki/Surface_plasmon_resonance
http://refractiveindex.info/?shelf=main&book=Cu&page=Rakic
http://www.microwaves101.com/encyclopedias/skin-depth-calculator

I'm very sure it isn't evanescent waves. However that doesn't stop anybody from proving me wrong.

And the problem with applying the thought of evanescent wave coupling to the quantum vacuum is that meep detects a force, but my meep simulation does not include anything to represent the quantum vacuum so that thought is ruled out. Resorting to the quantum vacuum to explain the force is not necessary.

I've brought this up before regarding the skin depth of copper. It seems to me that that data point (skin depth) is taken under rather benign conditions. The EM fields within the frustum are anything but benign, what with 50 Watts at a Q factor of over 6000. That is like 300 kW of radiant power bouncing around in the cavity. Perhaps under these conditions the electrons within the copper are excited at depth not normally measured by the techniques used measure skin depth. That is, maybe the skin depth is much larger at this high power?
Title: Re: EM Drive Developments - related to space flight applications - Thread 2
Post by: Mulletron on 02/17/2015 12:21 am
Evanescent waves and near field effects are virtual.
http://en.wikipedia.org/wiki/Near_and_far_field#Quantum_field_theory_view
http://en.wikipedia.org/wiki/Virtual_particle#Manifestations

Sorry to be pushing the oracle on you so much but I'm moving fast on my feet. There are other sources.

http://iopscience.iop.org/0295-5075/76/2/189;jsessionid=74CB8C68DB78D80A8FF4200696294DAC.c1

And you were probably right about evanescent waves being superluminal. There is ample evidence that virtual photons can be superluminal. I'm agnostic.
http://math.ucr.edu/home/baez/physics/Quantum/virtual_particles.html

We just met in the vacuum.

It is time to believe in the reality that this Q-thruster is as real as the QV itself. There is ample theory to support it dating back decades.

(https://encrypted-tbn2.gstatic.com/images?q=tbn:ANd9GcQAmSBIz4Jxu0wx3vB40vNHedzVmj3sabyt8VrAUE3YgGd7yXfY)
Title: Re: EM Drive Developments - related to space flight applications - Thread 2
Post by: Rodal on 02/17/2015 01:40 am
@ RODAL

Arrgh, Mondays !

Looked over my bleary weekend, noticed I was using diameters AGAIN !

Mode   Frequency (MHz)  Quality Factor, Q   Input Power (W)  Mean Thrust (μN)   Calculated w/o
                                                                                                                          dielectric
TE012     1880.4               22000                         2.6                55.4                   10.8
TM212   1932.6                 7320                        16.9                91.2                   38.5
TM212   1936.7               18100                        16.7                50.1                   93.5
TM212    1937.115             6726                       50                   66                    104.0

Anyway, shows it pays to rewrite everything in the same place !

....

Great !

In order to understand the above, (please correct me if I am wrong), you used in your formula the actual frequency and mode shapes that took place in the EM Drive experiment with the dielectric so in that sense you did calculate with the dielectric in a very restricted sense.

PS: Very unusual Monday in the Triangle (Raleigh/Durham/Chapel Hill) it is snowing over here  :)
Title: Re: EM Drive Developments - related to space flight applications - Thread 2
Post by: Notsosureofit on 02/17/2015 01:41 am
@ RODAL

Correct !  Hmm, note the inverse rel between the 2nd and 3rd.  Typo ? (or just loading)

Edit:  sums are close 262.7 vs 246.8    ~6%

PS:  about 7 ft here so far (acc to the news)

Title: Re: EM Drive Developments - related to space flight applications - Thread 2
Post by: Mulletron on 02/17/2015 04:12 am
http://forum.nasaspaceflight.com/index.php?topic=36313.msg1330846#msg1330846
To drive the point home, here are some observations reported of PT symmetry breaking featuring non-reciprocity of light. Keep in mind that light also carries momentum.

http://www.nature.com/nphys/journal/v11/n1/abs/nphys3152.html
http://arxiv.org/abs/1308.4564
http://adsabs.harvard.edu/abs/2014NatPh..10..394P
http://www.nature.com/nphys/journal/v6/n3/abs/nphys1515.html
http://goo.gl/gYGJ40

Quote
One of seven types of hypothetical space drives suggested by Marc Millis of the Breakthrough Propulsion Physics Program at NASA's Glenn Research Center (see Millis drives).
The radiation pressure on one side of the induction sail would be increased by some yet undiscovered means, and the pressure on the other side lowered. The spacecraft would move toward the low-pressure region.
http://www.daviddarling.info/encyclopedia/I/induction_sail.html
http://goo.gl/Sb2rRg (Popsci source of image)
http://www.nasa.gov/centers/glenn/technology/warp/ideachev.html

Eagleworks has an induction sail sitting in that vacuum chamber.

Hopefully it will scale up and be useful in some space flight applications someday.
Title: Re: EM Drive Developments - related to space flight applications - Thread 2
Post by: zen-in on 02/17/2015 04:18 am
How hard would it be to put the 1165 inside a more vacuum resilient container?

http://www.empowerrf.com/pdfs/1165.pdf

Notice the paper says max altitude 50K feet!

It is operating beyond specs in that vacuum.....why they keep dying!

We gotta figure this out, pronto!
This appears to be a great finding, Mulletron, thanks for pointing it out.  If the Amp that Eagleworks has been using is not rated for the hard vacuum (5*10^(-4) Torr) in their tests, do you have a best suggestion on how they should proceed?
Nope, sealing things up like that is beyond my expertise. No clue how to help. Anyone?

So why does it have to be inside the chamber again? Can't rf be piped in somehow?

Folks:

The reason I thought that the EMPower unit could take vacuum is that the first page of the data sheet indicated that they are "hermetically" sealed.  At NASA hermetically sealed always means vacuum rated.  The Air Force and her contractors obviously have another definition of that word, but my bad for assuming it meant the NASA definition.  In any regards and as I stated earlier, EMPower has given us permission to just drill a hole into the top plate of their amp's chassis so it can vent to hard vacuum conditions when operating in same, since there are no components in their unit that would degrade over time in a hard vacuum such as electrolytic caps.

Best, Paul M.

If the 1165 amplifier is "NASA" hermetically sealed the reason for a 50,000 ft ceiling may be to limit distortion of the internal compartments of the unit, resulting in out of spec performance.   Drilling one hole may not do it because there is no guarantee every part of the amplifier will vent.   

Heat dissipation is a lot more difficult in a vacuum since just about all the heat has to escape by radiation.   Maybe pre-cooling the copper cavity will help.    A class C amplifier would be more efficient and would work just as well if a CW output was used.   But class C amplifiers are not linear amps.   The output goes from a low level to the maximum design power level with hardly any change in input level.

Getting back to the 1165 amp:   It is class AB so has an efficiency < 75% provided the load is 50 Ohms resistive.   It also can't handle an SWR > 3:1.    One way to protect the amplifier is to put a circulator between it and the em-drive.   The reflected wave from the em-drive gets dissipated as heat in the circulator instead of the amplifier.   Or worse the resulting high RF voltages at the output of the amplifier cause arcing.   

Since the em-drive has such a high Q it's next to impossible to drive it with the right frequency.   The frequency will always be off so the complex impedance at the input of the em-drive will almost never be 50 Ohms resistive.   This results in most of the power being reflected back to the amplifier, damaging it.    , Instead of using a signal generator, if  the cavity was the frequency determining element then locking to the desired frequency might be easier.   Cavity oscillators have been around for a long time.   High power cavity oscillators use tubes.  (Eimac) 
Title: Re: EM Drive Developments - related to space flight applications - Thread 2
Post by: Star One on 02/17/2015 08:33 am


Aside from re running the reverse orientation tests. Are there any other tests that Eagleworks needs to run. I am aware that they need to get to a certain performance level before then can hand off for replication attempt. But for the life of me the only other test case I can think of wanting results for is

* Re Run frustum reverse orientation in Hard vacuum
* Run forward and reverse orientation of frustum in a null configuration

If it is handed off for a replication attempt, is this to be done in more than one other location, in other words are multiple teams to attempt this or just one?

My understanding is given what was said in the conclusion of the Brady et al paper is that they want to create a testable unit to be used by Glen Research Center and JPL and Johns Hopkins. However, I believe of the two NASA centers only Glen has signed on. No clue on whether or not Johns Hopkins has also signed on. Which is the reason they need to get the thrust levels up because the Balance at Glen has a much higher floor of detectable thrust.

Thanks. Hopefully they all sign on for the best verification.
Title: Re: EM Drive Developments - related to space flight applications - Thread 2
Post by: Notsosureofit on 02/17/2015 11:36 am
@ RODAL

Arrgh, Mondays !

Looked over my bleary weekend, noticed I was using diameters AGAIN !

Mode   Frequency (MHz)  Quality Factor, Q   Input Power (W)  Mean Thrust (μN)   Calculated w/o
                                                                                                                          dielectric
TE012     1880.4               22000                         2.6                55.4                   10.8
TM212   1932.6                 7320                        16.9                91.2                   38.5
TM212   1936.7               18100                        16.7                50.1                   93.5
TM212    1937.115             6726                       50                   66                    104.0

Anyway, shows it pays to rewrite everything in the same place !

....

Great !

In order to understand the above, (please correct me if I am wrong), you used in your formula the actual frequency and mode shapes that took place in the EM Drive experiment with the dielectric so in that sense you did calculate with the dielectric in a very restricted sense.


FYI

Cleanup and de-typo of the take on applying the Equivalence Principle.


The proposition that dispersion caused by an accelerating frame of reference implied an accelerating frame of reference caused by a dispersive cavity resonator. (to 1st order using massless, perfectly conducting cavity, no dielectric)


Starting with the expressions for the frequency of a cylindrical RF cavity:

f = (c/(2*Pi))*((X/R)^2+((p*Pi)/L)^2)^.5

For TM modes, X = X[sub m,n] = the n-th zero of the m-th Bessel function.
[1,1]=3.83, [0,1]=2.40, [0,2]=5.52 [1,2]=7.02, [2,1]=5.14, [2,2]=8.42, [1,3]=10.17, etc.

and for TE modes, X = X'[sub m,n] = the n-th zero of the derivative of the m-th Bessel function.
[0,1]=3.83, [1,1]=1.84, [2,1]=3.05, [0,2]=7.02, [1,2]=5.33, [1,3]=8.54, [0,3]=10.17, [2,2]=6.71, etc.

Rotate the dispersion relation of the cavity into Doppler frame to get the Doppler shifts, that is to say, look at the dispersion curve intersections of constant wave number instead of constant frequency.

df = (1/(2*f))*(c/(2*Pi))^2*X^2*((1/Rs^2)-(1/Rb^2))

and from there the expression for the acceleration g from:

g = (c^2/L)*(df/f) such that:

g = (c^2/(2*L*f^2))*(c/(2*Pi))^2*X^2*((1/Rs^2)-(1/Rb^2))

Using the "weight" of the photon in the accelerated frame from:

"W" = (h*f/c^2)*g =>  "W" = T = (h/L)*df

gives thrust per photon:

T = (h/(2*L*f))*(c/(2*pi))^2*X^2*((1/Rs^2)-(1/Rb^2))

If the number of photons is (P/hf)*(Q/2*pi*f) then:

NT = P*Q*(1/(4*pi*L*f^3))*(c/(2*pi))^2*X^2*((1/Rs^2)-(1/Rb^2))


This does fit (as far as I've gotten) the concept of a self-accelerating Dirac wavepacket (which does conserve momentum).

Slow goin', thanks for your patience.



Title: Re: EM Drive Developments - related to space flight applications - Thread 2
Post by: Rodal on 02/17/2015 11:40 am
How hard would it be to put the 1165 inside a more vacuum resilient container?

http://www.empowerrf.com/pdfs/1165.pdf

Notice the paper says max altitude 50K feet!

It is operating beyond specs in that vacuum.....why they keep dying!

We gotta figure this out, pronto!
This appears to be a great finding, Mulletron, thanks for pointing it out.  If the Amp that Eagleworks has been using is not rated for the hard vacuum (5*10^(-4) Torr) in their tests, do you have a best suggestion on how they should proceed?
Nope, sealing things up like that is beyond my expertise. No clue how to help. Anyone?

So why does it have to be inside the chamber again? Can't rf be piped in somehow?

Folks:

The reason I thought that the EMPower unit could take vacuum is that the first page of the data sheet indicated that they are "hermetically" sealed.  At NASA hermetically sealed always means vacuum rated.  The Air Force and her contractors obviously have another definition of that word, but my bad for assuming it meant the NASA definition.  In any regards and as I stated earlier, EMPower has given us permission to just drill a hole into the top plate of their amp's chassis so it can vent to hard vacuum conditions when operating in same, since there are no components in their unit that would degrade over time in a hard vacuum such as electrolytic caps.

Best, Paul M.

If the 1165 amplifier is "NASA" hermetically sealed the reason for a 50,000 ft ceiling may be to limit distortion of the internal compartments of the unit, resulting in out of spec performance.   Drilling one hole may not do it because there is no guarantee every part of the amplifier will vent.   

Heat dissipation is a lot more difficult in a vacuum since just about all the heat has to escape by radiation.   Maybe pre-cooling the copper cavity will help.    A class C amplifier would be more efficient and would work just as well if a CW output was used.   But class C amplifiers are not linear amps.   The output goes from a low level to the maximum design power level with hardly any change in input level.

Getting back to the 1165 amp:   It is class AB so has an efficiency < 75% provided the load is 50 Ohms resistive.   It also can't handle an SWR > 3:1.    One way to protect the amplifier is to put a circulator between it and the em-drive.   The reflected wave from the em-drive gets dissipated as heat in the circulator instead of the amplifier.   Or worse the resulting high RF voltages at the output of the amplifier cause arcing.   

Since the em-drive has such a high Q it's next to impossible to drive it with the right frequency.   The frequency will always be off so the complex impedance at the input of the em-drive will almost never be 50 Ohms resistive.   This results in most of the power being reflected back to the amplifier, damaging it.    , Instead of using a signal generator, if  the cavity was the frequency determining element then locking to the desired frequency might be easier.   Cavity oscillators have been around for a long time.   High power cavity oscillators use tubes.  (Eimac)
It will help to improve radiation of heat by adding low emissivity heat sinks to the locations that get most heated.   Lowering the emissivity of what is usually shiny aluminum,  will improve radiation and decrease the temperature. A thin coat of non reflective, flat paint on the inside and outside surfaces, and a thin coat on the hot parts will allow up to six times the amount of power to radiate for the same component temperature rise.  Anodizing works almost as well, provided the anodized surface is about .001” thick.

Material                      Emissivity (the higher the emissivity the better radiation of heat)
Silver, polished               0.01
Gold                               0.02
Aluminum, brush         0.15
Black Lacquer .24mil       0.48
Polished Steel                  0.53
Anodized Aluminum           0.81
White Flat Paint 1mil    0.94

Warning: emissivity tables have conflicting numbers.  A very extensive one is here:  http://www.engineeringtoolbox.com/emissivity-coefficients-d_447.html  while the one in the Emissivity article in Wikipedia is very restricted.  I would advise to consult several published values and place particular emphasis on published emissivity values in articles for heat radiation in electronics : experimental values used for heat radiation in electronics should rule !  Thickness (of the paint coat or thickness of anodizing layer) is critical for effective heat removal (you don't want to have a thick layer of coating which would trap the heat).
Title: Re: EM Drive Developments - related to space flight applications - Thread 2
Post by: Rodal on 02/17/2015 11:58 am
@ RODAL

Arrgh, Mondays !

Looked over my bleary weekend, noticed I was using diameters AGAIN !

Mode   Frequency (MHz)  Quality Factor, Q   Input Power (W)  Mean Thrust (μN)   Calculated w/o
                                                                                                                          dielectric
TE012     1880.4               22000                         2.6                55.4                   10.8
TM212   1932.6                 7320                        16.9                91.2                   38.5
TM212   1936.7               18100                        16.7                50.1                   93.5
TM212    1937.115             6726                       50                   66                    104.0

Anyway, shows it pays to rewrite everything in the same place !

....

Great !

In order to understand the above, (please correct me if I am wrong), you used in your formula the actual frequency and mode shapes that took place in the EM Drive experiment with the dielectric so in that sense you did calculate with the dielectric in a very restricted sense.


FYI

Cleanup and de-typo of the take on applying the Equivalence Principle.


The proposition that dispersion caused by an accelerating frame of reference implied an accelerating frame of reference caused by a dispersive cavity resonator. (to 1st order using massless, perfectly conducting cavity, no dielectric)


Starting with the expressions for the frequency of a cylindrical RF cavity:

f = (c/(2*Pi))*((X/R)^2+((p*Pi)/L)^2)^.5

For TM modes, X = X[sub m,n] = the n-th zero of the m-th Bessel function.
[1,1]=3.83, [0,1]=2.40, [0,2]=5.52 [1,2]=7.02, [2,1]=5.14, [2,2]=8.42, [1,3]=10.17, etc.

and for TE modes, X = X'[sub m,n] = the n-th zero of the derivative of the m-th Bessel function.
[0,1]=3.83, [1,1]=1.84, [2,1]=3.05, [0,2]=7.02, [1,2]=5.33, [1,3]=8.54, [0,3]=10.17, [2,2]=6.71, etc.

Rotate the dispersion relation of the cavity into Doppler frame to get the Doppler shifts, that is to say, look at the dispersion curve intersections of constant wave number instead of constant frequency.

df = (1/(2*f))*(c/(2*Pi))^2*X^2*((1/Rs^2)-(1/Rb^2))

and from there the expression for the acceleration g from:

g = (c^2/L)*(df/f) such that:

g = (c^2/(2*L*f^2))*(c/(2*Pi))^2*X^2*((1/Rs^2)-(1/Rb^2))

Using the "weight" of the photon in the accelerated frame from:

"W" = (h*f/c^2)*g =>  "W" = T = (h/L)*df

gives thrust per photon:

T = (h/(2*L*f))*(c/(2*pi))^2*X^2*((1/Rs^2)-(1/Rb^2))

If the number of photons is (P/hf)*(Q/2*pi) then:

NT = P*Q*(1/(4*pi*L*f^3))*(c/(2*pi))^2*X^2*((1/Rs^2)-(1/Rb^2))


This does fit (as far as I've gotten) the concept of a self-accelerating Dirac wavepacket (which does conserve momentum).

Slow goin', thanks for your patience.

Excellent!  Thank you for posting the complete equations.

One suggestion:  In the expression NT = P*Q*(1/(4*pi*L*f^3))*(c/(2*pi))^2*X^2*((1/Rs^2)-(1/Rb^2))

the speed of light in vacuum "c" appears in the numerator without being divided by the SquareRoot of the relative electric permittivity and relative magnetic permeability.

Since the relative electric permittivity of the dielectric is 2.3, this would decrease the values in the table by a factor of Sqrt[2.3]=1.52 if the whole cavity would be occupied by the dielectric.  Granted that only a portion of the truncated cone contains the dielectric, which will decrease the dividing factor, but any amount will reduce the effective value of c in the medium, giving lower thrust and hence values closer to the experimental measurements. 

For example, very roughly, assuming that 1/3 of the longitudinal length is occupied by the dielectric, and using the average as a medium with those average properties, Sqrt[(2.3*1/3)+1*(2/3)]=1.20, the thrust values would be reduced by a factor of 1.20, so for the most important test (the one in recently performed in vacuum, -the other experimental values may have been affected by thermal convection effects in the air and are therefore less reliable-), instead of 104 μN you would get 87 μN, which better compares with the experimental value of 66 μN.  
Title: Re: EM Drive Developments - related to space flight applications - Thread 2
Post by: Notsosureofit on 02/17/2015 12:11 pm
Correct !  I just don't want to get into trying to get the exact dispersion relation.  Got enough on the plate looking for a mechanism.
Title: Re: EM Drive Developments - related to space flight applications - Thread 2
Post by: Rodal on 02/17/2015 12:29 pm
Mulletron was kind enough to send me personally this very interesting post that I would like to share:

http://physics.stackexchange.com/questions/130662/emdrive-cavity-modes

Quickly, so as not to abuse your attention:

1)The author of the Physics Stack Exchange post finds out that it is difficult to satisfy the Boundary Conditions and reaches the (false) conclusion:

Quote
When I apply boundary conditions, Er must be zero at the side walls so P0n(cos(θw)) = 0 can only happen for specific angles. If the wall is not at the right angle, Er must be zero every where. Same is true for Eϕ, but that has zeros for P1n(cos(θw)) so it would be a different mode.
...
In other words, if the EmDrive guys don't build the cavity to specific angles, it will simply reflect all power and won't have any RF in it at all!

This conclusion is incorrect.   Greg Egan found a clever way to satisfy all the Boundary Conditions for any arbitrary angle, by adding a parameter, and solving two eigenvalue problems (one eigenvalue problem for the Legendre Functions and another eigenvalue problem for the spherical Bessel functions and an arbitrary parameter).  Greg Egan's conclusion is correct and there are other papers in the literature as well.  Actually the basis of the exact solution goes back to the great US engineer Schelnukoff in 1938 .   Unfortunately nobody at http://physics.stackexchange.com/ set the author of the post straight regarding the Boundary Condition problem.

2) The fact that the truncated cone does resonate is also confirmed by the COMSOL Finite Element studies performed for NASA.

(http://forum.nasaspaceflight.com/index.php?action=dlattach;topic=36313.0;attach=635195;image)

3) I'm making good progress with the exact solution.

4) One benefit of the exact solution is that I am able to plot the modes as a contour line to show the wave and the number of zeros, to ascertain the exact meaning of the mode shape quantum numbers m, n and p.  I have found out that while m and p can be characterized as we previously did, the meaning of n depends on whether the mode is TM or TE and whether n is even or odd.  No wonder then that Mulletron and NotSoSureOfIt found confusion in the literature concerning the meaning of the mode shape numbers.  I may have to correct the article on Wikipedia once again.
Title: Re: EM Drive Developments - related to space flight applications - Thread 2
Post by: Rodal on 02/17/2015 01:17 pm
....
This does fit (as far as I've gotten) the concept of a self-accelerating Dirac wavepacket (which does conserve momentum).
...
Please tell us more when you have a chance about the <<self-accelerating Dirac wavepacket (which does conserve momentum)>> as conservation of momentum has been the biggest problem of the scientific media (Prof. Baez and Sean Carroll for example) and with the serious science-fiction media (Greg Egan) with the EMDrive.  How does momentum get conserved in the EMDrive when there is nothing coming out of the EM Drive?

http://physics.stackexchange.com/questions/160702/self-accelerating-wavepackets-what-are-they-and-can-they-impulse-a-spaceship

http://www.nature.com/nphys/journal/vaop/ncurrent/fig_tab/nphys3196_F1.html

http://newsoffice.mit.edu/2015/self-accelerating-particles-0120

Or are you considering that evanescent waves coming out of the EM Drive in outer space (with no fields or matter nearby to interact with) are responsible for conservation of momentum with an effectiveness much greater than a photon rocket?
Title: Re: EM Drive Developments - related to space flight applications - Thread 2
Post by: Notsosureofit on 02/17/2015 01:51 pm
@ RODAL

Thanks for the refs.  I was not aware of self-acceleration when we started.  (but I spent a year as a NASA Fellow w/ S. Schwebel in the 60's on lift in a grav. grad., later did thesis on acoustic cavity resonator solutions for AF model atmosphere)
Title: Re: EM Drive Developments - related to space flight applications - Thread 2
Post by: Rodal on 02/17/2015 01:59 pm
@ RODAL

Arrgh, Mondays !

Looked over my bleary weekend, noticed I was using diameters AGAIN !

Mode   Frequency (MHz)  Quality Factor, Q   Input Power (W)  Mean Thrust (μN)   Calculated w/o
                                                                                                                          dielectric
TE012     1880.4               22000                         2.6                55.4                   10.8
TM212   1932.6                 7320                        16.9                91.2                   38.5
TM212   1936.7               18100                        16.7                50.1                   93.5
TM212    1937.115             6726                       50                   66                    104.0

Anyway, shows it pays to rewrite everything in the same place !

....

Great !

In order to understand the above, (please correct me if I am wrong), you used in your formula the actual frequency and mode shapes that took place in the EM Drive experiment with the dielectric so in that sense you did calculate with the dielectric in a very restricted sense.


FYI

Cleanup and de-typo of the take on applying the Equivalence Principle.


The proposition that dispersion caused by an accelerating frame of reference implied an accelerating frame of reference caused by a dispersive cavity resonator. (to 1st order using massless, perfectly conducting cavity, no dielectric)


Starting with the expressions for the frequency of a cylindrical RF cavity:

f = (c/(2*Pi))*((X/R)^2+((p*Pi)/L)^2)^.5

For TM modes, X = X[sub m,n] = the n-th zero of the m-th Bessel function.
[1,1]=3.83, [0,1]=2.40, [0,2]=5.52 [1,2]=7.02, [2,1]=5.14, [2,2]=8.42, [1,3]=10.17, etc.

and for TE modes, X = X'[sub m,n] = the n-th zero of the derivative of the m-th Bessel function.
[0,1]=3.83, [1,1]=1.84, [2,1]=3.05, [0,2]=7.02, [1,2]=5.33, [1,3]=8.54, [0,3]=10.17, [2,2]=6.71, etc.

Rotate the dispersion relation of the cavity into Doppler frame to get the Doppler shifts, that is to say, look at the dispersion curve intersections of constant wave number instead of constant frequency.

df = (1/(2*f))*(c/(2*Pi))^2*X^2*((1/Rs^2)-(1/Rb^2))

and from there the expression for the acceleration g from:

g = (c^2/L)*(df/f) such that:

g = (c^2/(2*L*f^2))*(c/(2*Pi))^2*X^2*((1/Rs^2)-(1/Rb^2))

Using the "weight" of the photon in the accelerated frame from:

"W" = (h*f/c^2)*g =>  "W" = T = (h/L)*df

gives thrust per photon:

T = (h/(2*L*f))*(c/(2*pi))^2*X^2*((1/Rs^2)-(1/Rb^2))

If the number of photons is (P/hf)*(Q/2*pi) then:

NT = P*Q*(1/(4*pi*L*f^3))*(c/(2*pi))^2*X^2*((1/Rs^2)-(1/Rb^2))


This does fit (as far as I've gotten) the concept of a self-accelerating Dirac wavepacket (which does conserve momentum).

Slow goin', thanks for your patience.

Excellent!  Thank you for posting the complete equations.

One suggestion:  In the expression NT = P*Q*(1/(4*pi*L*f^3))*(c/(2*pi))^2*X^2*((1/Rs^2)-(1/Rb^2))

the speed of light in vacuum "c" appears in the numerator without being divided by the SquareRoot of the relative electric permittivity and relative magnetic permeability.

Since the relative electric permittivity of the dielectric is 2.3, this would decrease the values in the table by a factor of Sqrt[2.3]=1.52 if the whole cavity would be occupied by the dielectric.  Granted that only a portion of the truncated cone contains the dielectric, which will decrease the dividing factor, but any amount will reduce the effective value of c in the medium, giving lower thrust and hence values closer to the experimental measurements. 

For example, very roughly, assuming that 1/3 of the longitudinal length is occupied by the dielectric, and using the average as a medium with those average properties, Sqrt[(2.3*1/3)+1*(2/3)]=1.20, the thrust values would be reduced by a factor of 1.20, so for the most important test (the one in recently performed in vacuum, -the other experimental values may have been affected by thermal convection effects in the air and are therefore less reliable-), instead of 104 μN you would get 87 μN, which better compares with the experimental value of 66 μN.  
Or, taking the highest thrust value (rather than the truncated upper mean of the experimental trace), (disregarding turn on transients) which is 78 μN (see trace below) one would get an excellent comparison with NotSoSureOfIt's dielectric-weighted prediction in vacuum of 87 μN:  barely 11% difference !

(http://forum.nasaspaceflight.com/index.php?action=dlattach;topic=36313.0;attach=636345;image)
Title: Re: EM Drive Developments - related to space flight applications - Thread 2
Post by: Notsosureofit on 02/17/2015 02:06 pm
@ RODAL

PS:  Got plenty of big vacuum chambers here, (couple 6' x 8' cylinders, etc.) no torsion balances left though (not put together anyway .. might have parts) and a big lack of time !
Title: Re: EM Drive Developments - related to space flight applications - Thread 2
Post by: Rodal on 02/17/2015 02:22 pm
@ RODAL

PS:  Got plenty of big vacuum chambers here, (couple 6' x 8' cylinders, etc.) no torsion balances left though (not put together anyway .. might have parts) and a big lack of time !
That makes for such an attractive setup, that anyone would gladly trudge through 7 ft of snow !  :)
Title: Re: EM Drive Developments - related to space flight applications - Thread 2
Post by: Notsosureofit on 02/17/2015 02:25 pm
@ RODAL

PS:  Got plenty of big vacuum chambers here, (couple 6' x 8' cylinders, etc.) no torsion balances left though (not put together anyway .. might have parts) and a big lack of time !
That makes for such an attractive setup, that anyone would gladly trudge through 7 ft of snow !  :)

Time and trudging are an absolute necessity for keeping the doors open on a small (15.5k sq ft, 4 people) R&D "skunkworks" like this.  Been running at a loss lately.

But, back to emdrive spaceflight ...
Title: Re: EM Drive Developments - related to space flight applications - Thread 2
Post by: SWGlassPit on 02/17/2015 02:40 pm
...

As for whether or not meep integrates the equations, I think it is a matter of terminology...
Numerical Integration involves approximating definite integrals by  summing discretized areas.  This is not what the Finite Difference method does. 

Sorry about being perhaps overly rigorous in the use of the word "integrate". I point this (finite? pun-intended  :) ) difference because it is important to understand the convergence problems with Finite Difference methods (as opposed to integration methods like the Boundary Element method, for example, or methods based on variational principles like the Finite Element Method).
What the Finite Difference method does is instead to approximate solutions to differential equations using finite differences to approximate derivatives. 
The idea of a finite difference method is the transformation of a continuity domain to a discrete set of points, the mesh. In every grid point the given differential operator is approximated by a difference-operator.

The issue is that numerically, numerical differentiation is always a much trickier problem than numerical integration (from a convergence viewpoint).

The Finite Difference method is a very old method (references going back to the 19th century) but great progress was made using it during and after World War II, due to the development of the digital computer, due to Von Neuman and Friederichs, mainly due to the Manhattan Project.

At MIT's ASRL very complex Finite Difference codes were developed, for example the PETROS code:
http://bit.ly/1AJ5Vgt (http://bit.ly/1AJ5Vgt) in addition to Finite Element and other types of numerical analyses.

To expand on this, Finite Difference schemes suffer from a deep, inherent weakness that is not true of Finite Element methods: the solution spaces are vastly different.

In Finite Difference schemes, the solution is defined in terms of the function value (i.e., a plain old number) at a finite number of discrete points -- there is no mathematical basis to define the correct interpolation between points.  In Finite Element methods, the solution space is a collection of functions, defined across the entire domain.  Aside from providing a mathematical basis for interpolation, having a solution space as a collection of functions allows you to estimate the approximation error in the solution (in fact, and this is outside the scope of this thread, provided it is properly applied, Finite Element method has the "best approximation" property, which means that the method will give you the least possible error for the discretization you supply).  This is most easily done by taking the L2-norm (in no way related to NSF.com's L2  ;) ) of the residual that results from plugging the solution back into the original differential equation.  To be specific, (using LaTeX notation), if

A \phi - f = 0,

where A is the differential operator, \phi is the (infinite-dimensional) solution vector, and f is the forcing function (or load vector), then the Finite Element method gives a finite-dimensional approximation \phi_h, which gives the equation:

A \phi_h - f = e,

where e is the residual, a (generally non-zero) function that represents the error that results from substituting the approximation into the differential equation.  By squaring this value and integrating it across the problem domain (which gives the square of the L2-norm), we have a reliable measure of how good our answer is -- it is strictly non-negative, a lower number represents a better approximation, and it is only zero if our answer is exact.

To-wit: Finite Element methods give you both an approximation to the exact solution and a measure of how good that approximation is, even if the exact solution is not easily obtained.  Finite Difference schemes provide no such information regarding the correctness of the results they provide.
Title: Re: EM Drive Developments - related to space flight applications - Thread 2
Post by: Rodal on 02/17/2015 03:00 pm
....
To-wit: Finite Element methods give you both an approximation to the exact solution and a measure of how good that approximation is, even if the exact solution is not easily obtained.  Finite Difference schemes provide no such information regarding the correctness of the results they provide.
That's why it is critical, to assess results of a numerical solution, to compare the results of a numerical solution (for example Finite Difference method) to an exact solution.  In this case, an exact solution to a cylindrical cavity exists  (http://en.wikipedia.org/wiki/Microwave_cavity#Cylindrical_cavity), and it would be worthwhile to compare how far is the MEEP solution for a resonant cylindrical cavity, say of a diameter=Sqrt[BigDiameterOfTruncatedCone * SmallDiameterOfTruncatedCone] and same axial length, with the same material inputs and mesh as used for the Finite Difference solution of the Truncated Cone.

An exact solution to a truncated cone also exists, but it is much more complicated to solve (as it involves the solution of two eigenvalue problems) than the cylindrical cavity.

If the MEEP Finite Difference solution (with the same mesh, inputs and dimensions, as discussed above) cannot match the exact solution of a cylindrical cavity, how can one be confident of the solution for a more complicated problem for which there is no exact solution to compare with?

Another issue is using a flat two-dimensional simulation rather than a three-dimensional simulation, because of the huge run-times involved in a 3-D simulation.  This involves a very severe assumption that a 2-D model is sufficient, and comparison with the exact solution to a cylindrical cavity (http://en.wikipedia.org/wiki/Microwave_cavity#Cylindrical_cavity) is therefore of paramount importance.

This is only meant as a constructive suggestion. 

The very time-consuming and dedicated work of @aero deserves very strong praise, as having shown that evanescent waves is a possible explanation for the EM Drive results.  We sincerely hope that he continues with it.
Title: Re: EM Drive Developments - related to space flight applications - Thread 2
Post by: birchoff on 02/17/2015 03:50 pm
Has anyone looked at rangling some cloud vm time to run these processes.

You can find the amazon compute vm prices here
http://aws.amazon.com/ec2/pricing/

and the azure compute vm prices here
http://azure.microsoft.com/en-us/pricing/details/virtual-machines/

not sure what the runtime looks like but I would hazard a guess that you could figure out a cheap enough solution that allows you to get results in the least amount of time.


Title: Re: EM Drive Developments - related to space flight applications - Thread 2
Post by: aero on 02/17/2015 05:31 pm
Has anyone looked at rangling some cloud vm time to run these processes.

You can find the amazon compute vm prices here
http://aws.amazon.com/ec2/pricing/

and the azure compute vm prices here
http://azure.microsoft.com/en-us/pricing/details/virtual-machines/

not sure what the runtime looks like but I would hazard a guess that you could figure out a cheap enough solution that allows you to get results in the least amount of time.

I've looked at it but I'm not going to take the responsibility of paying for and trying to figure out how to use their systems, and to install and use meep on those systems.

I do have an opinion. The most understandable documentation of the available capabilities is from Google.
https://cloud.google.com/compute/pricing (https://cloud.google.com/compute/pricing)

But at only 100 GB memory for a high memory compute configuration, I'd be concerned about size of the model. For a 3D model, Meep memory requirements go up by a factor of 8 for each doubling of the resolution and compute requirements by a factor of 16 for the same doubling.

If someone wanted to do this, it would be necessary to establish the model at low resolution on a convenient machine, then calculate the resources needed by the problem running at the resolution required for viable results.

Meep was designed to run massive problems at high resolution on supercomputers. Sixteen processors each with 6.5 GB ram is not really a very impressive supercomputer. And I wonder, can these cloud based compute engines guarantee model execution synchronization for the duration of a run that may consume hours of CPU? If it can't be synchronized then AIUI all the CPU's wait for the slowest partition to keep up. That could get costly in a hurry.

I have designed and priced a custom computer that could provide a very good basis for estimating the resources needed to run high fidelity problems. It was priced at $2038.96 (USD), a firm quote, tax included. It is about 1/3 the machine referred to above. (Six cores with 32 GB DDR4 memory) I'm not going to take the responsibility for paying for that machine, either, though I would love to have it.
Title: Re: EM Drive Developments - related to space flight applications - Thread 2
Post by: birchoff on 02/17/2015 06:58 pm
Has anyone looked at rangling some cloud vm time to run these processes.

You can find the amazon compute vm prices here
http://aws.amazon.com/ec2/pricing/

and the azure compute vm prices here
http://azure.microsoft.com/en-us/pricing/details/virtual-machines/

not sure what the runtime looks like but I would hazard a guess that you could figure out a cheap enough solution that allows you to get results in the least amount of time.

I've looked at it but I'm not going to take the responsibility of paying for and trying to figure out how to use their systems, and to install and use meep on those systems.

I do have an opinion. The most understandable documentation of the available capabilities is from Google.
https://cloud.google.com/compute/pricing (https://cloud.google.com/compute/pricing)

But at only 100 GB memory for a high memory compute configuration, I'd be concerned about size of the model. For a 3D model, Meep memory requirements go up by a factor of 8 for each doubling of the resolution and compute requirements by a factor of 16 for the same doubling.

If someone wanted to do this, it would be necessary to establish the model at low resolution on a convenient machine, then calculate the resources needed by the problem running at the resolution required for viable results.

Meep was designed to run massive problems at high resolution on supercomputers. Sixteen processors each with 6.5 GB ram is not really a very impressive supercomputer. And I wonder, can these cloud based compute engines guarantee model execution synchronization for the duration of a run that may consume hours of CPU? If it can't be synchronized then AIUI all the CPU's wait for the slowest partition to keep up. That could get costly in a hurry.

I have designed and priced a custom computer that could provide a very good basis for estimating the resources needed to run high fidelity problems. It was priced at $2038.96 (USD), a firm quote, tax included. It is about 1/3 the machine referred to above. (Six cores with 32 GB DDR4 memory) I'm not going to take the responsibility for paying for that machine, either, though I would love to have it.

Nice to know someone has looked at it. Was just wondering if these computation limitations could be simply solved by the application of a little sprinkle of the cloud. As for paying for access to the resources, I guess the question is how badly do we want accurate results.
Title: Re: EM Drive Developments - related to space flight applications - Thread 2
Post by: Mulletron on 02/17/2015 07:32 pm
https://iafastro.directory/iac/archive/browse/IAC-13/C4/P/16863/

Quote
The EmDrive's resonant cavity has the characteristics as of cutoff  waveguide. By reference to the phenomena of electromagnetic wave anomalous propagation  in the cutoff waveguide, the fact that the  electromagnetic wave can be reflected without metal  surface in the cutoff waveguide is presented in the paper.At the same time, another fact that the electromagnetic wave distribution in the EmDrive's resonant cavity showing a characteristic of evanescent wave  is presented also. It is  a kind of electromagnetic wave anomalous propagation. This anomalous propagation can be described by the photon tunneling effect, consistent with quantum field theory.At last,the opinion that EmDrive revealing some properties of background vacuum is put forward in the paper,and the introduction of the virtual photon process may be a new method to analyze the momentum conservation of EmDrive.

He's right you know, you really don't even need the small diameter end plate. As the tapered frustum diameter reduces to cutoff, it imposes a natural boundary beyond which modes that can't resonate will go evanescent. Standing waves will still appear within the cavity. Just like how reflected power appears in a cut waveguide. Might even be useful feature. Food for thought at least. I only have the abstract.

So this approach described above certainly would apply to a waveguide excited by a wideband RF signal, such as one would get from a magnetron. For a narrowband CW excitation like at Eagleworks, there wouldn't be generation of evanescent modes in the manner described by the Chinese.

The fact that Eagleworks is reporting that in TE modes in particular, there are numerous resonant modes in very close proximity to each other should be taken to heart (which makes sense given the continuously changing E field boundary condition), as this may very well be a useful feature for Q-thrusters.

I think from the research presented in this forum, we've managed to suss out a testable framework of how best to implement Q thrusters driven by RF.
-TE modes are important. In particular because that places the magnetic field longitudinally. Is TE012 the best? The Chinese, Shawyer, and a hint from Eagleworks suggests yes it is. (Technical problems aside like maintaining resonance, ways to overcome have been suggested.)
-I personally think that exciting as many TE modes as possible simultaneously will yield best results.
-Driving the cavity with a wideband signal is important.
-The above serves to maximize field localization and intensity within the resonant cavity.*
-The final is choosing materials which are strongly magnetoelectric and/or magnetochiral.**
* http://adsabs.harvard.edu/abs/2014NatPh..10..394P 
** https://vtechworks.lib.vt.edu/bitstream/handle/10919/25258/1.2337996.pdf?sequence=1


There are still a lot of questions that need answering as to why the Chinese are reporting much better performance from using magnetrons, and this kind of thinking my provide an answer, at least in part.

There's lot of stuff out there about evanescent tunneling in undersized waveguides. I have no idea what this has to do with thrust, might lead to another unknown interaction or a new way of describing the already familiar.
http://www.popularscience.co.uk/features/feat11.htm
http://www.hindawi.com/journals/ijo/2013/947068/
http://www.ifac.cnr.it/toq-www/guide1-eng.htm
http://goo.gl/qlYrjn

I think @Aero will be pleased.  ;)
Title: Re: EM Drive Developments - related to space flight applications - Thread 2
Post by: Rodal on 02/17/2015 07:58 pm
https://iafastro.directory/iac/archive/browse/IAC-13/C4/P/16863/

...
Great article, Mulletron, thank you for bringing it to our attention.

I only found the abstract (sorry if I missed where the actual paper is linked). Is there a link to the actual paper ?  (If the actual paper is in Chinese (language) that's fine as well)

PS: making great progress on the exact solution for the truncated cone.

EDIT: for the NASA cavity dimension some modes are purely resonant, some modes are purely evanescent and some modes go from resonant to evanescent.
Title: Re: EM Drive Developments - related to space flight applications - Thread 2
Post by: Mulletron on 02/17/2015 08:00 pm
It says you have to email him and ask for it.
Title: Re: EM Drive Developments - related to space flight applications - Thread 2
Post by: tchernik on 02/17/2015 08:00 pm

Nice to know someone has looked at it. Was just wondering if these computation limitations could be simply solved by the application of a little sprinkle of the cloud. As for paying for access to the resources, I guess the question is how badly do we want accurate results.

If the cost of the simulation approaches that of an actual experiment, you'd better off doing the experiment!

Anyone has any idea of how much would it cost to make a decent DIY replication?
Title: Re: EM Drive Developments - related to space flight applications - Thread 2
Post by: aero on 02/17/2015 08:01 pm
Has anyone looked at rangling some cloud vm time to run these processes.

You can find the amazon compute vm prices here
http://aws.amazon.com/ec2/pricing/

and the azure compute vm prices here
http://azure.microsoft.com/en-us/pricing/details/virtual-machines/

not sure what the runtime looks like but I would hazard a guess that you could figure out a cheap enough solution that allows you to get results in the least amount of time.

I've looked at it but I'm not going to take the responsibility of paying for and trying to figure out how to use their systems, and to install and use meep on those systems.

I do have an opinion. The most understandable documentation of the available capabilities is from Google.
https://cloud.google.com/compute/pricing (https://cloud.google.com/compute/pricing)

But at only 100 GB memory for a high memory compute configuration, I'd be concerned about size of the model. For a 3D model, Meep memory requirements go up by a factor of 8 for each doubling of the resolution and compute requirements by a factor of 16 for the same doubling.

If someone wanted to do this, it would be necessary to establish the model at low resolution on a convenient machine, then calculate the resources needed by the problem running at the resolution required for viable results.

Meep was designed to run massive problems at high resolution on supercomputers. Sixteen processors each with 6.5 GB ram is not really a very impressive supercomputer. And I wonder, can these cloud based compute engines guarantee model execution synchronization for the duration of a run that may consume hours of CPU? If it can't be synchronized then AIUI all the CPU's wait for the slowest partition to keep up. That could get costly in a hurry.

I have designed and priced a custom computer that could provide a very good basis for estimating the resources needed to run high fidelity problems. It was priced at $2038.96 (USD), a firm quote, tax included. It is about 1/3 the machine referred to above. (Six cores with 32 GB DDR4 memory) I'm not going to take the responsibility for paying for that machine, either, though I would love to have it.

Nice to know someone has looked at it. Was just wondering if these computation limitations could be simply solved by the application of a little sprinkle of the cloud. As for paying for access to the resources, I guess the question is how badly do we want accurate results.

Not so much accurate results. The results I have presented are accurate to second order, for the problem evaluated. More like more representative problems, higher fidelity models (3D, and resolving smaller gaps, for example), that could be achieved with more computing power.

I have calculated error bounds, the magnitude of step size squared for my current run, which I hope to post in about 6 hours.

delta s ~= 0.023 mm = 0.000023 meters, (delta s)^2 =5.29E-010
delta t ~= 0.0001071422 sec, (delta t)^2 = 1.1479455161248E-008 s^2

Those error bounds seem acceptable to me for the problems I am evaluating. The errors inherent in the problem formulated to represent characteristics of the actual EM thurster/vacuum chamber are far, far larger than the computational errors of the simulation.

I kind of wish people would stop posting their concern about numerical computational deficiencies here. If they have a valid concern then they really should take it up with MIT, where the codes were written, or perhaps ONR, DARPA who spent the money to pay for the development. Maybe ONR/DARPA should ask for their money back?

Or PM me, I can point them to several online sites giving results that scrutinise meep accuracy.



Title: Re: EM Drive Developments - related to space flight applications - Thread 2
Post by: birchoff on 02/17/2015 08:20 pm
Has anyone looked at rangling some cloud vm time to run these processes.

You can find the amazon compute vm prices here
http://aws.amazon.com/ec2/pricing/

and the azure compute vm prices here
http://azure.microsoft.com/en-us/pricing/details/virtual-machines/

not sure what the runtime looks like but I would hazard a guess that you could figure out a cheap enough solution that allows you to get results in the least amount of time.

I've looked at it but I'm not going to take the responsibility of paying for and trying to figure out how to use their systems, and to install and use meep on those systems.

I do have an opinion. The most understandable documentation of the available capabilities is from Google.
https://cloud.google.com/compute/pricing (https://cloud.google.com/compute/pricing)

But at only 100 GB memory for a high memory compute configuration, I'd be concerned about size of the model. For a 3D model, Meep memory requirements go up by a factor of 8 for each doubling of the resolution and compute requirements by a factor of 16 for the same doubling.

If someone wanted to do this, it would be necessary to establish the model at low resolution on a convenient machine, then calculate the resources needed by the problem running at the resolution required for viable results.

Meep was designed to run massive problems at high resolution on supercomputers. Sixteen processors each with 6.5 GB ram is not really a very impressive supercomputer. And I wonder, can these cloud based compute engines guarantee model execution synchronization for the duration of a run that may consume hours of CPU? If it can't be synchronized then AIUI all the CPU's wait for the slowest partition to keep up. That could get costly in a hurry.

I have designed and priced a custom computer that could provide a very good basis for estimating the resources needed to run high fidelity problems. It was priced at $2038.96 (USD), a firm quote, tax included. It is about 1/3 the machine referred to above. (Six cores with 32 GB DDR4 memory) I'm not going to take the responsibility for paying for that machine, either, though I would love to have it.

Nice to know someone has looked at it. Was just wondering if these computation limitations could be simply solved by the application of a little sprinkle of the cloud. As for paying for access to the resources, I guess the question is how badly do we want accurate results.

Not so much accurate results. The results I have presented are accurate to second order, for the problem evaluated. More like more representative problems, higher fidelity models (3D, and resolving smaller gaps, for example), that could be achieved with more computing power.

I have calculated error bounds, the magnitude of step size squared for my current run, which I hope to post in about 6 hours.

delta s ~= 0.023 mm = 0.000023 meters, (delta s)^2 =5.29E-010
delta t ~= 0.0001071422 sec, (delta t)^2 = 1.1479455161248E-008 s^2

Those error bounds seem acceptable to me for the problems I am evaluating. The errors inherent in the problem formulated to represent characteristics of the actual EM thurster/vacuum chamber are far, far larger than the computational errors of the simulation.

I kind of wish people would stop posting their concern about numerical computational deficiencies here. If they have a valid concern then they really should take it up with MIT, where the codes were written, or perhaps ONR, DARPA who spent the money to pay for the development. Maybe ONR/DARPA should ask for their money back?

Or PM me, I can point them to several online sites giving results that scrutinise meep accuracy.

I wasn't criticizing numerical accuracy of the work your doing. you are correct to my usage of the term accurate. I just noticed the remark on lack of computational resources for certain levels of resolution and thought I would make a suggestion.
Title: Re: EM Drive Developments - related to space flight applications - Thread 2
Post by: Rodal on 02/17/2015 08:29 pm
...Actually the basis of the exact solution (for the truncated cone microwave cavity) goes back to the great US engineer Schelnukoff in 1938 .   ...

More interesting information about the great Russian/American scientist/engineer Schelkunoff, who made it possible to obtain an exact solution for the  truncated cone (frustum) microwave cavity (the geometry of the EM Drive that is being tested and proposed by NASA, Shawyer in the UK and Juan Yang in China, for space flight applications):

http://en.wikipedia.org/wiki/Sergei_Alexander_Schelkunoff

His book about Electromagnetic Waves published in 1943 in the middle of WWII has higher quality information than many contemporary books.

Quote
He crossed Siberia into Manchuria and then Japan before settling into Seattle in 1921. There he received bachelor's and master's degrees in mathematics from the State College of Washington, now the University of Washington, and in 1928 received his Ph.D. from Columbia University for his dissertation On Certain Properties of the Metrical and Generalized Metrical Groups in Linear Spaces of n Dimension.
....

, Schelkunoff joined Western Electric's research wing, which became Bell Laboratories. In 1933 he and Sally P. Mead began analysis of waveguide propagation discovered analytically by their colleague George C. Southworth. Their analysis uncovered the transverse modes. Schelkunoff appears to have been the first to notice the important practical consequences of the fact that attenuation in the TE01 mode decays inversely with the 3/2 power of the frequency. In 1935 he and his colleagues reported that coaxial cable, then new, could transmit television pictures or up to 200 telephone conversations.

(http://images.ookaboo.com/photo/s/Schelkunoff_s.jpg)

Schelkunoff is officially listed as being a member of Bell Labs Mathematical Center from 1929 to 1963:
http://cm.bell-labs.com/cm/ms/center/frmdir.html
Title: Re: EM Drive Developments - related to space flight applications - Thread 2
Post by: Rodal on 02/17/2015 08:45 pm
...

I have calculated error bounds, the magnitude of step size squared for my current run, which I hope to post in about 6 hours.

delta s ~= 0.023 mm = 0.000023 meters, (delta s)^2 =5.29E-010
delta t ~= 0.0001071422 sec, (delta t)^2 = 1.1479455161248E-008 s^2

Those error bounds seem acceptable to me for the problems I am evaluating. The errors inherent in the problem formulated to represent characteristics of the actual EM thurster/vacuum chamber are far, far larger than the computational errors of the simulation.

I kind of wish people would stop posting their concern about numerical computational deficiencies here. If they have a valid concern then they really should take it up with MIT, where the codes were written, or perhaps ONR, DARPA who spent the money to pay for the development. Maybe ONR/DARPA should ask for their money back?

...
As discussed here: http://forum.nasaspaceflight.com/index.php?topic=36313.msg1332799#msg1332799, those error bounds do not tell you about what can be the difference between a Finite Difference solution and an exact solution to the problem.  Magnitude of step size is instead related, for example, to the stability problems of the Finite Difference operator, as it was shown by Friedrichs and Lax.  For example, the central-difference operator has stability problems that mandate the time step to be smaller than a certain bound, because it is an explicit (as opposed to implicit) finite difference operator.  However, having a step small enough to avoid instability of the finite difference operator does not tell you about how far can the finite difference solution be from an exact solution.

The accuracy issues we are discussing are not related to any bugs or issues concerning the people at MIT that wrote the program, they are issues inherent to the Finite Difference method.  All numerical methods have numerical issues of different kinds. 

The purpose to discuss these issues here in an open forum is to examine the numerical solutions concerning EM Drive for space flight applications, just like we examine the experiments and the theoretical explanations.   :)

We are discussing the NASA experiments, and the proposed theoretical explanations, asking and examining all kinds of questions.  Numerical solutions (related to EM Drive Developments - space flight applications) deserve equal examination, not less, than the examination of experiments and the examination of theoretical explanations.

I fully understand that such examination can get frustrating. We are all interested in finding how the EM Drive generates thrust in experiments, and our only goal is to enable space flight applications as soon as possible.

Title: Re: EM Drive Developments - related to space flight applications - Thread 2
Post by: aero on 02/17/2015 09:51 pm
Quote
We are discussing the NASA experiments, and the proposed theoretical explanations, asking and examining all kinds of questions.  Numerical solutions (related to EM Drive Developments - space flight applications) deserve equal examination, not less, than the examination of experiments and the examination of theoretical explanations.

So what do you expect me to do about it? My 4 poor little processors have been sprinting at 99.7 to 100% capacity since I awoke this morning and started this current run. They won't finish for another 2-3 hours and are very tired. I can not increase the resolution for this run to study convergence. I could cut the resolution but I doubt that would calculate an answer.

What I plan to do is reduce the lattice size after this run (which is checking the forces without the vacuum chamber - lattice size, resolution and all else remaining the same). Once I reduce the lattice size I should be able to run higher resolution to check convergence. Maybe even generate a 3D image of the fields but that is problematic for any gap sizes remotely representative of the Eagleworks test article.

So what do you propose as a resolution to your critique?
Title: Re: EM Drive Developments - related to space flight applications - Thread 2
Post by: Rodal on 02/17/2015 10:04 pm
...
So what do you propose as a resolution to your critique?
Same thing I proposed here:

http://forum.nasaspaceflight.com/index.php?topic=36313.msg1332813#msg1332813

.. it is critical, to assess results of a numerical solution, to compare the results of a numerical solution (for example Finite Difference method) to an exact solution.  In this case, an exact solution to a cylindrical cavity exists  (http://en.wikipedia.org/wiki/Microwave_cavity#Cylindrical_cavity), and it would be worthwhile to compare how far is the MEEP solution for a resonant cylindrical cavity, say of a diameter=Sqrt[BigDiameterOfTruncatedCone * SmallDiameterOfTruncatedCone]=0.21060 m and same axial length, with the same material inputs and mesh as used for the Finite Difference solution of the Truncated Cone.
....

I propose a MEEP analysis for a resonant cylindrical cavity (no dielectric), with  diameter=Sqrt[BigDiameterOfTruncatedCone * SmallDiameterOfTruncatedCone] and same axial length=0.2286 m as the NASA cavity, with the same material inputs and mesh as you used for the Finite Difference solution of the Truncated Cone. 


Actual geometry
Large OD : 11.00 " (0.2794m),
Small OD: 6.25" (0.1588 m)
Length : 9.00 " (0.2286m)
Geometric Mean Diameter: 0.2106056741875679 m


If the MEEP mesh for the truncated cone cannot be used to obtain a  MEEP solution close to the exact solution for a cylindrical cavity of similar dimensions (an easier problem to solve than the truncated cone), then that mesh and solution (2D?) cannot get a reliable solution for the EM Drive truncated cone, concerning EM Drive for space flight applications. (The cylindrical cavity is an easier problem because the mode shapes are either purely resonating (real solutions) or evanescent (imaginary solutions) while for a truncated cone there are modes that go from resonating to evanescent, and because the truncated cone displays interesting attenuation and focusing properties).
Title: Re: EM Drive Developments - related to space flight applications - Thread 2
Post by: SWGlassPit on 02/17/2015 10:24 pm

The accuracy issues we are discussing are not related to any bugs or issues concerning the people at MIT that wrote the program, they are issues inherent to the Finite Difference method.  All numerical methods have numerical issues of different kinds. 

The purpose to discuss these issues here in an open forum is to examine the numerical solutions concerning EM Drive for space flight applications, just like we examine the experiments and the theoretical explanations.   :)

We are discussing the NASA experiments, and the proposed theoretical explanations, asking and examining all kinds of questions.  Numerical solutions (related to EM Drive Developments - space flight applications) deserve equal examination, not less, than the examination of experiments and the examination of theoretical explanations.


I wanted to highlight and expand on this --

Finite Difference schemes have the weakness that I described previously -- so why do people use them at all?  They are cheap and easy to implement.  Finite Element methods, for all their glory, are quite expensive in comparison, especially when the differential operator is non-self-adjoint or nonlinear (for things like fluid mechanics, they can be hideously expensive). 

The point of this discussion is not to say that you are wrong to use a particular method.  The point is that all numerical methods have flaws and drawbacks.  Intelligent use of them requires knowledge and understanding of these problems to develop strategies to mitigate them.
Title: Re: EM Drive Developments - related to space flight applications - Thread 2
Post by: Mulletron on 02/17/2015 10:54 pm
....
This does fit (as far as I've gotten) the concept of a self-accelerating Dirac wavepacket (which does conserve momentum).
...
Please tell us more when you have a chance about the <<self-accelerating Dirac wavepacket (which does conserve momentum)>> as conservation of momentum has been the biggest problem of the scientific media (Prof. Baez and Sean Carroll for example) and with the serious science-fiction media (Greg Egan) with the EMDrive.  How does momentum get conserved in the EMDrive when there is nothing coming out of the EM Drive?

http://physics.stackexchange.com/questions/160702/self-accelerating-wavepackets-what-are-they-and-can-they-impulse-a-spaceship

http://www.nature.com/nphys/journal/vaop/ncurrent/fig_tab/nphys3196_F1.html

http://newsoffice.mit.edu/2015/self-accelerating-particles-0120

Or are you considering that evanescent waves coming out of the EM Drive in outer space (with no fields or matter nearby to interact with) are responsible for conservation of momentum with an effectiveness much greater than a photon rocket?

This is certainly interesting but there are no "specially engineered phase masks" in Emdrive.

Same people: http://arxiv.org/pdf/1205.2112.pdf
Title: Re: EM Drive Developments - related to space flight applications - Thread 2
Post by: aero on 02/17/2015 10:58 pm
...
So what do you propose as a resolution to your critique?
Same thing I proposed here:

http://forum.nasaspaceflight.com/index.php?topic=36313.msg1332813#msg1332813

.. it is critical, to assess results of a numerical solution, to compare the results of a numerical solution (for example Finite Difference method) to an exact solution.  In this case, an exact solution to a cylindrical cavity exists  (http://en.wikipedia.org/wiki/Microwave_cavity#Cylindrical_cavity), and it would be worthwhile to compare how far is the MEEP solution for a resonant cylindrical cavity, say of a diameter=Sqrt[BigDiameterOfTruncatedCone * SmallDiameterOfTruncatedCone]=0.21060 m and same axial length, with the same material inputs and mesh as used for the Finite Difference solution of the Truncated Cone.
....

I propose a MEEP analysis for a resonant cylindrical cavity (no dielectric), with  diameter=Sqrt[BigDiameterOfTruncatedCone * SmallDiameterOfTruncatedCone] and same axial length=0.2286 m as the NASA cavity, with the same material inputs and mesh as you used for the Finite Difference solution of the Truncated Cone. 


Actual geometry
Large OD : 11.00 " (0.2794m),
Small OD: 6.25" (0.1588 m)
Length : 9.00 " (0.2286m)
Geometric Mean Diameter: 0.2106056741875679 m


If the MEEP mesh for the truncated cone cannot be used to obtain a  MEEP solution close to the exact solution for a cylindrical cavity of similar dimensions (an easier problem to solve than the truncated cone), then that mesh and solution (2D?) cannot get a reliable solution for the EM Drive truncated cone, concerning EM Drive for space flight applications. (The cylindrical cavity is an easier problem because the mode shapes are either purely resonating (real solutions) or evanescent (imaginary solutions) while for a truncated cone there are modes that go from resonating to evanescent, and because the truncated cone displays interesting attenuation and focusing properties).

As you must know, there are several more-or-less independent modules in meep. The core FDTD algorithm that we should be concerned about is not the same as the Harminv algorithm which determines resonance frequencies and as you know, is suspect, at least by me and Dr. Dominic and I think, yourself. Proving again that the Harminv module is suspect will not show anything one way or the other about the core FDTD algorithm or the Flux and Forces modules. In other words, You already know the answer, so what is your point? 

In reading this I detect a hint of frustration, but I am.
Title: Re: EM Drive Developments - related to space flight applications - Thread 2
Post by: Mulletron on 02/17/2015 11:10 pm
This is a goldmine of information concerning non-reciprocal materials!

http://arxiv.org/abs/1211.0530

The visual is to help communicate the concept.
Title: Re: EM Drive Developments - related to space flight applications - Thread 2
Post by: Rodal on 02/17/2015 11:12 pm
....
As you must know, there are several more-or-less independent modules in meep. The core FDTD algorithm that we should be concerned about is not the same as the Harminv algorithm which determines resonance frequencies and as you know, is suspect, at least by me and Dr. Dominic and I think, yourself. Proving again that the Harminv module is suspect will not show anything one way or the other about the core FDTD algorithm or the Flux and Forces modules. ....

If the MEEP mesh discretization (distance between nodes and time increment between steps, and type of finite difference operator) used for the truncated cone cannot also be used to obtain a MEEP solution close to the exact solution for a cylindrical cavity of similar dimensions (an easier problem to solve than the truncated cone), then that mesh and solution (2D?) cannot get a reliable solution for the EM Drive truncated cone, concerning EM Drive for space flight applications. 

We shouldn't accept that the reason is due to anything wrong with the Harminv module, the reason may be inherent with the finite difference discretization (and 2D?) model being used I only recently realized that you are using a 2-D model instead of 3-D model. If your cavity analysis was also 2-D I would focus more on the fact that the 2-D model is modeling the cavity as a flat rectangle instead of a 3D cylinder. It would be incorrect to assume that Maxwell's equations in 2D would give similar results as Maxwell's equations in 3D

If the  cylinder was modeled as 3-D and results were obtained far away from the exact solution for a cylindrical cavity and you suspect Harminv, then what should be done is to conduct a time-stepping solution, going through the initial transient state until a steady state solution is obtained.  The frequency and mode shapes can then be obtained from inspecting the data output from MEEP, instead of relying on Harminv.  Actually if the problem would be nonlinear, for example, you would not be allowed to use Harminv anyway and you would have to resort precisely to this type of analysis: examining the response until a steady state is obtained to then ascertain the frequency and mode shape.

Once a good comparison with MEEP has been obtained for the easier problem of the cylindrical cavity which has a simple exact solution, one can then proceed to the more difficult problem of the truncated cone, to have confidence in the solution as applicable to EM Drive for space flight applications. 
Title: Re: EM Drive Developments - related to space flight applications - Thread 2
Post by: Notsosureofit on 02/17/2015 11:17 pm
....
This does fit (as far as I've gotten) the concept of a self-accelerating Dirac wavepacket (which does conserve momentum).
...
Please tell us more when you have a chance about the <<self-accelerating Dirac wavepacket (which does conserve momentum)>> as conservation of momentum has been the biggest problem of the scientific media (Prof. Baez and Sean Carroll for example) and with the serious science-fiction media (Greg Egan) with the EMDrive.  How does momentum get conserved in the EMDrive when there is nothing coming out of the EM Drive?

http://physics.stackexchange.com/questions/160702/self-accelerating-wavepackets-what-are-they-and-can-they-impulse-a-spaceship

http://www.nature.com/nphys/journal/vaop/ncurrent/fig_tab/nphys3196_F1.html

http://newsoffice.mit.edu/2015/self-accelerating-particles-0120

Or are you considering that evanescent waves coming out of the EM Drive in outer space (with no fields or matter nearby to interact with) are responsible for conservation of momentum with an effectiveness much greater than a photon rocket?

This is certainly interesting but there are no "specially engineered phase masks" in Emdrive.

Same people: http://arxiv.org/pdf/1205.2112.pdf

Ahhh, but there are, they are the boundary conditions set by the cavity walls. 
Title: Re: EM Drive Developments - related to space flight applications - Thread 2
Post by: Mulletron on 02/17/2015 11:18 pm
Quote
It has been recently argued that the
quantum vacuum can possess momentum
http://arxiv-web3.library.cornell.edu/abs/0908.4390

As far as the theoretical side of things, as the Emdrive isn't ejecting anything and appears to violate conservation of momentum, we need to critically examine the above quote.

Experimental replications of Emdrive seem to suggest it is gaining momentum via unconventional means; like the quantum vacuum or somewhere else.

If the vacuum can be shown to possess momentum, as the authors claim, we are in business.
Title: Re: EM Drive Developments - related to space flight applications - Thread 2
Post by: Notsosureofit on 02/17/2015 11:20 pm
This is a goldmine of information concerning non-reciprocal materials!

http://arxiv.org/abs/1211.0530

The visual is to help communicate the concept.

Yes, this is a goldmine to be explored !!!
Title: Re: EM Drive Developments - related to space flight applications - Thread 2
Post by: Mulletron on 02/18/2015 12:53 am
Quote
It has been recently argued that the
quantum vacuum can possess momentum
From the most basic concepts, I argue that yes the quantum vacuum can possess momentum.

(http://upload.wikimedia.org/math/b/b/b/bbb1b74b511f868d0882a668de23c111.png)

Can one gain momentum from the quantum vacuum? NO. Due to the universal symmetry of the quantum vacuum...........Can anyone else venture to guess which two combined symmetries you have to break in order to gain momentum from the QV?

Out of the 3 to choose from, Charge, Parity, and Time.....which two have multiple researchers saying you can gain momentum from the QV by breaking?


Edit: It's PT. I have been saying for months now that you can't gain momentum from the QV under normal symmetry conditions. And that research from multiple parties suggests that if you simultaneously break PT symmetry of the vacuum, you can gain momentum from the QV. The multiple replications of Emdrive are experimental evidence which support their claims that it is in fact possible to gain momentum from the QV, and that arguments claiming Emdrive violates conservation of momentum are false. The conditions inside Emdrive fit the theory, and theory and experiment reinforce each other.
Title: Re: EM Drive Developments - related to space flight applications - Thread 2
Post by: mvpel on 02/18/2015 02:15 am
You should reach out to the Center for High-Throughput Computing at the University of Wisconsin. They've got a statewide federation of computing resources that's been used, for example, to find new radio pulsars in Arecibo data.
http://einstein.phys.uwm.edu/forum_thread.php?id=10397
Title: Re: EM Drive Developments - related to space flight applications - Thread 2
Post by: Notsosureofit on 02/18/2015 03:09 am
FYI before bed:

Noticed that the ratio of work done (from start to static peak) versus energy (per photon on the 50W, Q=6726) was ~7x10-5 ??

Pretty small % or did I miscalculate ?

Sunrise: Probably should be half that. (integrating against linear spring)

The curve "looks like" a sudden rise followed by a (thermal?) drift through the resonance.

The power output in that initial rise looks (very roughly) like 3x10-7 W.

Back when I was driving cavities (and Hector was a pup) I used a double balanced modulator to lock the half power points of the resonance using the "bow tie" display.  No "rotation", meant the center freq was correct and a minimum of the "crossing point" meant the bandwidth was correct.  This gave "precision" measurements of the resonances.  The freq standard then was the LORAN station on Cape Cod !  (this was for transport property measurements in gases)  The odd thing that sticks in my mind was that von Hipple at MIT wanted to apply the techniques to "an idea" he had for RF cavities, but he never got funding.  (and he never said what the idea was)

Just musings over coffee.............

Title: Re: EM Drive Developments - related to space flight applications - Thread 2
Post by: Mulletron on 02/18/2015 07:37 am
I've tried to understand the totality of the emdrive mystery in a lot of different ways ranging from it being pure bs to some quirk with inertia. The only way it makes sense is to apply that "momentum from the qv" theory I've been going on about. It is the only one left that still passes the smell test and doesn't try and overturn established science. It seems like a perfect fit for the conditions and two unconnected teams of researchers make the same claims.

Most importantly, it makes falsifiable predictions.
Title: Re: EM Drive Developments - related to space flight applications - Thread 2
Post by: ChrisWilson68 on 02/18/2015 08:28 am
I've tried to understand the totality of the emdrive mystery in a lot of different ways ranging from it being pure bs to some quirk with inertia. The only way it makes sense is to apply that "momentum from the qv" theory I've been going on about. It is the only one left that still passes the smell test and doesn't try and overturn established science. It seems like a perfect fit for the conditions and two unconnected teams of researchers make the same claims.

Most importantly, it makes falsifiable predictions.

Except that it *does* "try and overturn established science".  Momentum from the quantum vacuum is not compatible with generally accepted physics.

Unconnected teams can easily be mislead by similar mistakes.  Just look at how many independent observers claimed to see canals on the surface of Mars when they were on the edge of the signal/noise boundary of optical telescopes.
Title: Re: EM Drive Developments - related to space flight applications - Thread 2
Post by: Star One on 02/18/2015 11:51 am

I've tried to understand the totality of the emdrive mystery in a lot of different ways ranging from it being pure bs to some quirk with inertia. The only way it makes sense is to apply that "momentum from the qv" theory I've been going on about. It is the only one left that still passes the smell test and doesn't try and overturn established science. It seems like a perfect fit for the conditions and two unconnected teams of researchers make the same claims.

Most importantly, it makes falsifiable predictions.

Except that it *does* "try and overturn established science".  Momentum from the quantum vacuum is not compatible with generally accepted physics.

Unconnected teams can easily be mislead by similar mistakes.  Just look at how many independent observers claimed to see canals on the surface of Mars when they were on the edge of the signal/noise boundary of optical telescopes.

No it doesn't necessarily try and overturn establish science. Your example of canals on Mars is not really relevant to the experimental work seen on this thread.
Title: Re: EM Drive Developments - related to space flight applications - Thread 2
Post by: Mulletron on 02/18/2015 12:22 pm
Well it has been accepted for publication at least.
http://iopscience.iop.org/0953-8984/page/Forthcoming%20articles#Special_isCasimir_fo

And the other citations are already published.
http://lpm2c.grenoble.cnrs.fr/spip.php?page=publications&id_auteur=18&clepubli=van%20Tiggelen&lang=fr

So we're not dealing with a crank here.
Title: Re: EM Drive Developments - related to space flight applications - Thread 2
Post by: Star One on 02/18/2015 12:23 pm
Well it has been accepted for publication at least.
http://iopscience.iop.org/0953-8984/page/Forthcoming%20articles#Special_isCasimir_fo

And the other citations are already published.
http://lpm2c.grenoble.cnrs.fr/spip.php?page=publications&id_auteur=18&clepubli=van%20Tiggelen&lang=fr

So we're not dealing with a crank here.

Well unfortunately anything that seems a radical step forward in an area can attract proclamations of crank science even when not warranted.

That first link keeps coming up invalid parameters?
Title: Re: EM Drive Developments - related to space flight applications - Thread 2
Post by: Star-Drive on 02/18/2015 12:32 pm
Folks:

While you all talk about various ways to accomplish the E&M simulations of these frustum cavities, I thought you might like to take a look at the COMSOL derived resonances of the Eagleworks Lab's copper frustum resonant cavity driven with a ~16mm OD loop antenna located 15% up the side wall of the frustum from the large OD end of the cavity. 

BTW, the EMPower amplifiers were delivered to the Lab yesterday and I'll be calibrating the power metering for one that was installed yesterday on the torque pendulum.

Best, Paul M,
Title: Re: EM Drive Developments - related to space flight applications - Thread 2
Post by: Rodal on 02/18/2015 12:51 pm
Folks:

While you all talk about various ways to accomplish the E&M simulations of these frustum cavities, I thought you might like to take a look at the COMSOL derived resonances of the Eagleworks Lab's copper frustum resonant cavity driven with a ~16mm OD loop antenna located 15% up the side wall of the frustum from the large OD end of the cavity. 

BTW, the EMPower amplifiers were delivered to the Lab yesterday and I'll be calibrating the power metering for one that was installed yesterday on the torque pendulum.

Best, Paul M,

Thank you, Paul, for posting this, the attached pdf with COMSOL plots (which I first missed, my bad, thanks to Mulletron for pointing it out to me  :) ) is very useful for all of us (those like me working on the exact solution of the frustum, and those working with numerical methods like MEEP, COMSOL, ANSYS-multiphysics, etc.).

I could not find in the pdf attached to your post, or in your post. language to indicate whether the COMSOL calculations in the attachment (http://forum.nasaspaceflight.com/index.php?action=dlattach;topic=36313.0;attach=796287) include or do not include a dielectric inside the truncated cone (frustum). My impression is that these COMSOL calculations do not include the dielectric inside the truncated cone (this impression is based on the frequency calculated for TE012).

Still a clarification for some readers like me, may be useful (to those to whom what I write below is obvious, please forgive me for taking your time):




1) Clarification: the jpg image you posted are not "COMSOL derived resonances of the Eagleworks Lab's copper frustum resonant cavity", the COMSOL plots are in the pdf attached to your post (http://forum.nasaspaceflight.com/index.php?action=dlattach;topic=36313.0;attach=796287)

2) The mode shapes in the slide are for a different geometry: a circular waveguide (instead of a truncated cone/frustum like NASA's EM Drive) , that had been plotted in published journals since 1936, without using a digital computer. The actual reference's date is 1985 (instead of 1966): http://ieeexplore.ieee.org/xpl/articleDetails.jsp?arnumber=1132998.  Since COMSOL was started in 1986, Lee, Lee and Chuang did not use COMSOL: they had no need to use a finite element program, because an exact solution exists for what they plotted: the mode shapes of a circular waveguide.

The circular waveguide exact solution is present in the 1943 textbook by Schelkunoff, and in papers he published in the 1930's.  The mode shapes look identical to the plots that Schelkunoff published in the 1930's without the benefit of a digital computer: http://forum.nasaspaceflight.com/index.php?topic=36313.msg1332981#msg1332981.  The article by Lee, Lee and Chuang states: "The earliest plots of modal field distribution in rectangular/circular waveguides were given by Southworth (1936), Barrow (1936), Schelkunoff (1937), and Chu and Barrow (1937)"

3) The solution for a truncated cone (the geometry of the NASA frustum, and the geometry analyzed by COMSOL in the attached pdf) is different from the solution for a circular waveguide in several respects: the natural frequencies are quite different, and while in a circular waveguide different mode shapes are either resonant or cutoff, in a truncated cone some mode shapes are resonant, some are evanescent and some have a transition from resonant to evanescent (a characteristic not present in circular waveguides).  Also, the attenuation and focusing aspects of the truncated cone (frustum) are not present in the circular waveguide.

Title: Re: EM Drive Developments - related to space flight applications - Thread 2
Post by: JasonAW3 on 02/18/2015 01:19 pm
Ok, maybe I'm a bit slower than I ought to be lately, but are you guys telling us that they are, essentially, developing a propellentless thrust system based on a RADAR SYSTEM?
Title: Re: EM Drive Developments - related to space flight applications - Thread 2
Post by: Mulletron on 02/18/2015 01:25 pm
Ok, maybe I'm a bit slower than I ought to be lately, but are you guys telling us that they are, essentially, developing a propellentless thrust system based on a RADAR SYSTEM?

You never noticed that the ship rocks harder when the radars are turnin' and burnin'? Shoot, they should just sector SPY back aft and cut the engines. :-)

http://en.wikipedia.org/wiki/Arleigh_Burke-class_destroyer

Based on electromagnetic waves. Waveguide are found in numerous applications besides radars. Sorry back on topic...
Title: Re: EM Drive Developments - related to space flight applications - Thread 2
Post by: Notsosureofit on 02/18/2015 02:18 pm
Ok, maybe I'm a bit slower than I ought to be lately, but are you guys telling us that they are, essentially, developing a propellentless thrust system based on a RADAR SYSTEM?

You never noticed that the ship rocks harder when the radars are turnin' and burnin'? Shoot, they should just sector SPY back aft and cut the engines. :-)

http://en.wikipedia.org/wiki/Arleigh_Burke-class_destroyer

Based on electromagnetic waves. Waveguide are found in numerous applications besides radars. Sorry back on topic...

Wasn't that how Shawyer got the idea, only from a satellite not a ship?

Title: Re: EM Drive Developments - related to space flight applications - Thread 2
Post by: SleeperService on 02/18/2015 02:25 pm
Cross-posted from an EP thread..
http://pepl.engin.umich.edu/pdf/2014_Florenz_Thesis.pdf
Wouldn't it be cool if Mr. March and others could have just half the toys/funding to play with...
Title: Re: EM Drive Developments - related to space flight applications - Thread 2
Post by: Notsosureofit on 02/18/2015 02:26 pm
@ Star-Drive

Trying to put together a reference table.

Did they calculate Q values for all of the COMSOL modes shown ?

As RODAL asked, I'm assuming these are w/o dielectric ?

Also, can they calculate a dispersion relation for the cavity including the dielectric ?

Thanks
Title: Re: EM Drive Developments - related to space flight applications - Thread 2
Post by: lele on 02/18/2015 02:32 pm
If I understood correctly some things in this thread, EM radiations heat the frustum, which changes its shape, which changes the resonant frequency of the cavity, so it's difficult to keep the EM source tuned.

Would cooling (with flowing water for example) the copper plates 1) reduce the variation of the resonant frequency, by reducing the variation of temperature, thus reducing the geometrical changes of the frustum 2) not interfere with the thrust produced ?
Title: Re: EM Drive Developments - related to space flight applications - Thread 2
Post by: Mulletron on 02/18/2015 02:38 pm

Wasn't that how Shawyer got the idea, only from a satellite not a ship?

Background of emdrive.
http://emdrive.com/background.html

I was joking about the ship rocking more.
Title: Re: EM Drive Developments - related to space flight applications - Thread 2
Post by: Rodal on 02/18/2015 02:39 pm
If I understood correctly some things in this thread, EM radiations heat the frustum, which changes its shape, which changes the resonant frequency of the cavity, so it's difficult to keep the EM source tuned.

Would cooling (with flowing water for example) the copper plates 1) reduce the variation of the resonant frequency, by reducing the variation of temperature, thus reducing the geometrical changes of the frustum 2) not interfere with the thrust produced ?
Cooling from the outside may actually be counterproductive, as it may increase the thermal gradient through the thickness (it would be induction-heated from the inside and cooled from the outside hence a large thermal gradient) and deform the very thin NASA EM Drive shell due to the non-uniform aspect of the induction heating due to the non-uniform EM field.

One solution (already offered by Shawyer) is to use Invar, an old alloy that has very low thermal expansion, another solution is to increase the thickness of the EM Drive.  NASA was constrained to use very thin copper for the EM Drive because they wanted to minimize the mass on their torque pendulum.
Title: Re: EM Drive Developments - related to space flight applications - Thread 2
Post by: SleeperService on 02/18/2015 02:45 pm
If I understood correctly some things in this thread, EM radiations heat the frustum, which changes its shape, which changes the resonant frequency of the cavity, so it's difficult to keep the EM source tuned.

Would cooling (with flowing water for example) the copper plates 1) reduce the variation of the resonant frequency, by reducing the variation of temperature, thus reducing the geometrical changes of the frustum 2) not interfere with the thrust produced ?
Water cooling would either have to be internal to the thrust measuring device or external.
If external, then you have the problem of the coupling of the pipework and the externally powered thrust of the coolant.
If internal then you get the added mass of coolant, cooling jackets, pumps and radiators. And you have an pump running in close proximity to the cavity.
Title: Re: EM Drive Developments - related to space flight applications - Thread 2
Post by: Mulletron on 02/18/2015 02:45 pm
Or a phase locked loop circuit.
http://en.m.wikipedia.org/wiki/Phase-locked_loop#Frequency_synthesis
Title: Re: EM Drive Developments - related to space flight applications - Thread 2
Post by: Notsosureofit on 02/18/2015 02:50 pm
Or a phase locked loop circuit.
http://en.m.wikipedia.org/wiki/Phase-locked_loop#Frequency_synthesis

Yep, very easy these days.  In fact if they have VCO already, they must have it on the to-do list.

We lock to a satellite signal here w/ a tiny card.
Title: Re: EM Drive Developments - related to space flight applications - Thread 2
Post by: SleeperService on 02/18/2015 02:53 pm
Or a phase locked loop circuit.
http://en.m.wikipedia.org/wiki/Phase-locked_loop#Frequency_synthesis
How would apply this to tracking the resonant freq of the cavity to the input freq of the loop antenna?
Title: Re: EM Drive Developments - related to space flight applications - Thread 2
Post by: SleeperService on 02/18/2015 03:00 pm
"The function of the PLL is to compare the distributed clock to the incoming reference clock, and vary the phase and frequency of its output until the reference and feedback clocks are phase and frequency matched."
At which spatial point shall we measure the freq. to compare with the incoming freq.?
Please forgive me if I am wrong and/or naive about this...
Title: Re: EM Drive Developments - related to space flight applications - Thread 2
Post by: Rodal on 02/18/2015 03:00 pm
Or a phase locked loop circuit.
http://en.m.wikipedia.org/wiki/Phase-locked_loop#Frequency_synthesis
How would apply this to tracking the resonant freq of the cavity to the input freq of the loop antenna?
I also like Mulletron's original idea of distributing the spectral power density of the input into a spectral band instead of a spike at a single frequency.  As Mulletron pointed out, magnetrons used by Juan Yang in China and Shawyer in the UK deserve attention.
Title: Re: EM Drive Developments - related to space flight applications - Thread 2
Post by: Giovanni DS on 02/18/2015 03:06 pm
A feedback circuit would be required in order to keep the VCO on the resonant frequency. If such a feedback is possible then a simple MCU could be programmed to do that using an ADC as output, it would not be a PLL anyway but more similar to an FLL.
Title: Re: EM Drive Developments - related to space flight applications - Thread 2
Post by: SleeperService on 02/18/2015 03:07 pm
Maybe that was behind Cannae's technique of putting notches in his cavity. It produces more thrust with a broad, 'dirty' spectrum from a magnetron than a 'pure' sinusoidal source..
Title: Re: EM Drive Developments - related to space flight applications - Thread 2
Post by: SleeperService on 02/18/2015 03:09 pm
A feedback circuit would be required in order to keep the VCO on the resonant frequency. If such a feedback is possible then a simple MCU could be programmed to do that using an ADC as output, it would not be a PLL anyway but more similar to an FLL.
Where does the feedback signal come from?
Title: Re: EM Drive Developments - related to space flight applications - Thread 2
Post by: matthewpapa on 02/18/2015 03:18 pm
I have 2 high performance boxes at my house you may want to consider using for simulation purposes
2p Sandy Bridge (16 cores, 32 threads total @2.8 GHz) about 50 GB of DDR3 memory
2p Haswell (20 cores, 40 threads total @3.1 GHz) and 32 GB of DDR4 memory

PM me if that can help at some point
Title: Re: EM Drive Developments - related to space flight applications - Thread 2
Post by: Giovanni DS on 02/18/2015 03:20 pm
A feedback circuit would be required in order to keep the VCO on the resonant frequency. If such a feedback is possible then a simple MCU could be programmed to do that using an ADC as output, it would not be a PLL anyway but more similar to an FLL.
Where does the feedback signal come from?

This is the problem :) I do understand controls and realtime systems but microwaves and resonant cavities are not my field. How do you know it is resonating?

In theory the thrust could be your feedback, you tune the VCO in order to maximize that.
Title: Re: EM Drive Developments - related to space flight applications - Thread 2
Post by: SleeperService on 02/18/2015 03:34 pm
A feedback circuit would be required in order to keep the VCO on the resonant frequency. If such a feedback is possible then a simple MCU could be programmed to do that using an ADC as output, it would not be a PLL anyway but more similar to an FLL.
Where does the feedback signal come from?

This is the problem :) I do understand controls and realtime systems but microwaves and resonant cavities are not my field. How do you know it is resonating?

In theory the thrust could be your feedback, you tune the VCO in order to maximize that.
I agree, the only way you could do it is to lock the thrust to the freq. How you would do that though to avoid false maxima is beyond me.
Title: Re: EM Drive Developments - related to space flight applications - Thread 2
Post by: Mulletron on 02/18/2015 03:40 pm
Resonant cavities are also filters. You monitor a sample port to ensure the cavity is resonant at the desired frequency.
(https://encrypted-tbn1.gstatic.com/images?q=tbn:ANd9GcT1Qw2QkTBGA5bNweG7oyxFemEjFEJmgG9E3SPGH3i3UP6AXP8W)
Another control loop compensating for doppler drift is needed. Shawyer mentions this type of control loop over at emdrive.com.
Title: Re: EM Drive Developments - related to space flight applications - Thread 2
Post by: SleeperService on 02/18/2015 03:44 pm
Resonant cavities are also filters. You monitor a sample port to ensure the cavity is resonant at the desired frequency.

Another control loop compensating for doppler drift is needed. Shawyer mentions this type of control loop over at emdrive.com.
It seems to me that it matters where you put the sample port with a truncated frustum.
Am I wrong? Would a sample port to feedback the PLF work in any location?
I'll leave Doppler drift for another time... :)
Title: Re: EM Drive Developments - related to space flight applications - Thread 2
Post by: Mulletron on 02/18/2015 03:54 pm
I think we're getting on a tangent. But here's your answer.

http://www.radartutorial.eu/03.linetheory/tl11.en.html
Title: Re: EM Drive Developments - related to space flight applications - Thread 2
Post by: SleeperService on 02/18/2015 04:03 pm
I think we're getting on a tangent. But here's your answer.

http://www.radartutorial.eu/03.linetheory/tl11.en.html
Not really, with respect.
You can't have a small probe like you link and expect it to optimise on a freq. sweep for resonance. The resonance shapes are much too complex (Like Star Drive presented.) Do you see what I mean, or am I being dumb?
Title: Re: EM Drive Developments - related to space flight applications - Thread 2
Post by: Rodal on 02/18/2015 04:04 pm
A feedback circuit would be required in order to keep the VCO on the resonant frequency. If such a feedback is possible then a simple MCU could be programmed to do that using an ADC as output, it would not be a PLL anyway but more similar to an FLL.
Where does the feedback signal come from?

This is the problem :) I do understand controls and realtime systems but microwaves and resonant cavities are not my field. How do you know it is resonating?

In theory the thrust could be your feedback, you tune the VCO in order to maximize that.

This is a really a good point for a number of reasons.  Not least of which is the fact that NASA reports that thrust is not proportional to resonance Q.  Some of the experiments show greater thrust at a lower Q. If these experimental results are statistically significant(?) then using thrust as feedback, rather than insisting on Q resonance, is the way to go:

Mode   Frequency (MHz)  Quality Factor, Q   Input Power (W)  Mean Thrust (μN)   Pressure
TE012     1880.4               22000                         2.6                55.4                 Ambient
TM212    1932.6                 7320                        16.9                91.2                 Ambient
TM212    1936.7               18100                        16.7                50.1                 Ambient
TM212    1937.115             6726                        50                   66                    Vacuum

notice that

TM212    1932.6                 7320                        16.9                91.2 

produces practically twice the thrust of

TM212    1936.7               18100                        16.7                50.1       

which has more than twice the Q at practically the same Input Power and frequency
Title: Re: EM Drive Developments - related to space flight applications - Thread 2
Post by: SleeperService on 02/18/2015 04:11 pm
Higher thrust at lower Q seems to hint at heating/other reasons unfortunately...
Title: Re: EM Drive Developments - related to space flight applications - Thread 2
Post by: Rodal on 02/18/2015 04:13 pm
Higher thrust at lower Q seems to hint at heating/other reasons unfortunately...
I agree, but still, using thrust as feedback is superior to relying on resonance at this stage of knowledge. 

If thrust is proportional to Q, as expected by a number of theoretical formulas (Shawyer, McCulloch and NotSoSureOfIt) it does not make a difference whether to use thrust or resonance as feedback, while if thrust is not proportional to Q, then feedback based on thrust is superior, because of course, what one is after is space propulsion by the EM Drive and not just an EM Drive resonating in space.
Title: Re: EM Drive Developments - related to space flight applications - Thread 2
Post by: SleeperService on 02/18/2015 04:20 pm
Dr. Rodal,
What do you think about the false maxima that we could get in the thrust of the EM Drive?
Title: Re: EM Drive Developments - related to space flight applications - Thread 2
Post by: Rodal on 02/18/2015 04:26 pm
Dr. Rodal,
What do you think about the false maxima that we could get in the thrust of the EM Drive?
Need you to elaborate further: false because...it is a local maximum and not the true maxima...false because of error...false because of lag...
Title: Re: EM Drive Developments - related to space flight applications - Thread 2
Post by: SleeperService on 02/18/2015 04:29 pm
Dr. Rodal,
What do you think about the false maxima that we could get in the thrust of the EM Drive?
Need you to elaborate further: false because...it is a local maximum and not the true maxima...false because of error...false because of lag...
Indeed! We need to get the computing resources together to help us...
Title: Re: EM Drive Developments - related to space flight applications - Thread 2
Post by: Mulletron on 02/18/2015 04:30 pm
Thrust is not only proportionate to Q. Evidence suggests that thrust depends on the complex interplay of Q, mode shape and field density, and dielectric location wrt mode topology.

For unloaded magnetron fed cavities, thrust depends on Q. This is because it is resonant at many modes at once.

If Eagleworks wants to improve the performance of their emdrive, get more VCXOs and run them into a combiner, then to the amp.

Or brute force it with a magnetron or noise source.
Title: Re: EM Drive Developments - related to space flight applications - Thread 2
Post by: SleeperService on 02/18/2015 04:38 pm
What hope do we have then?!?!
Title: Re: EM Drive Developments - related to space flight applications - Thread 2
Post by: SleeperService on 02/18/2015 04:43 pm
Another question which I had...
I can see into my microwave thru a mesh that is < 1/2 the uWave frq.
Why are we considering RF energy leaking out with tiny gaps?
Does this happen?
Title: Re: EM Drive Developments - related to space flight applications - Thread 2
Post by: Giovanni DS on 02/18/2015 04:46 pm
About false maxima, it could work like a spectrum analyzer: perform a wide frequency sweep on an initial window, find the maximum (more than one peak could be present) and restrict the window around that, then repeat. When the signal is diminishing then the window could be enlarged and a new sweep performed.

I am sure a good algorithm could be found for this problem.

Thrush measurement problem is that the loop would be probably slow, the sweeps could take time.
Title: Re: EM Drive Developments - related to space flight applications - Thread 2
Post by: Flyby on 02/18/2015 05:23 pm
Not really my expertise field, but can somebody tell me if the volumetric resonance patterns are static or dynamic?
iow, does the combination of internal frustrum shape and wavelength(s) cause the resonance pattern to shift left or right (Is that the "reverse" ppl were talking about?), or does it stay immobile (static) once the interference patterns are established ??

I understand that the shape and aspect of the resonance pattern is very very sensitive and can easily change aspect (and direction?) caused by the slightest change (either wavelength or geometry of the frustrum).

As some of you talked about dielectric materials possibly improving performance, I suppose a force could be generated from the high intensity microwave resonance spots colliding with the frustrum? Am I correct on this assumption?

If that is the case, and looking at it from a distance, shouldn't we speak of a "magnetic pulse engine" as the high-intesity magnetic fields slam into frustrum wall, pushing it forward ?

or.. am i talking completely BS here? :)
Just trying to understand while feeling very much out of comfort zone...
Title: Re: EM Drive Developments - related to space flight applications - Thread 2
Post by: SleeperService on 02/18/2015 05:35 pm
Not really my expertise field, but can somebody tell me if the volumetric resonance patterns are static or dynamic?
iow, does the combination of internal frustrum shape and wavelength(s) cause the resonance pattern to shift left or right (Is that the "reverse" ppl were talking about?), or does it stay immobile (static) once the interference patterns are established ??

I understand that the shape and aspect of the resonance pattern is very very sensitive and can easily change aspect (and direction?) caused by the slightest change (either wavelength or geometry of the frustrum).

As some of you talked about dielectric materials possibly improving performance, I suppose a force could be generated from the high intensity microwave resonance spots colliding with the frustrum? Am I correct on this assumption?

If that is the case, and looking at it from a distance, shouldn't we speak of a "magnetic pulse engine" as the high-intesity magnetic fields slam into frustrum wall, pushing it forward ?

or.. am i talking completely BS here? :)
Just trying to understand while feeling very much out of comfort zone...
As far as I know the resonant freq. is very sensitive to the cavity properties.
Thermal buckling would change this a lot...
Basically you are correct...
Title: Re: EM Drive Developments - related to space flight applications - Thread 2
Post by: Mulletron on 02/18/2015 05:57 pm
Strong H field at dielectric disc. Just like theory suggests.

Quote
we have found that the chiral molecule acquires a kinetic
momentum during the switching of the external magnetic field as a result of its
interaction with the vacuum field. On the other hand we have shown that, as a
result of the conservation of the total momentum K, there exists a transfer of linear
momentum from the vacuum field to the molecule.
http://arxiv.org/pdf/1404.5990v1.pdf
http://arxiv.org/abs/1304.6767
http://ptp.oxfordjournals.org/content/119/3/351.full.pdf

But it isn't JUST chiral molecules, as I've shown experimental evidence of the observation of magneto-electric non-reciprocity in molecular nitrogen gas.
https://hal.archives-ouvertes.fr/hal-00551421v1/document
http://arxiv.org/abs/1101.0712

Which means that air confined to a resonant cavity falls into their magneto-electric metric too. Any condition which creates an anisotropic electromagnetic vacuum environment works.

Which explains why unloaded cavities tested by the Chinese and Shawyer work too.
This implies that a magneto-electric directionally anisotropic electromagnetic environment also absolutely implies an anisotropic vacuum environment, as the authors suggest is true and aim to measure.
Quote
Moreover, the same effect is expected in quantum vacuum.

Only I found evidence last night that the vacuum anisotropy experiment has already been done in the 80s.
http://heart-c704.uibk.ac.at/LV/Quantenoptik/Kapitel_8/Jhe_etal_PRL_58_666_%281987%29.pdf

This Emdrive operates exactly as I have suggested and existing theory supports. I've beat this dead horse enough.

(http://media.giphy.com/media/IOCXHPvn3WErm/giphy.gif)
Edit: Put Shen in there.
Title: Re: EM Drive Developments - related to space flight applications - Thread 2
Post by: tchernik on 02/18/2015 06:59 pm
Strong H field at dielectric disc. Just like theory suggests.

Quote
we have found that the chiral molecule acquires a kinetic
momentum during the switching of the external magnetic field as a result of its
interaction with the vacuum field. On the other hand we have shown that, as a
result of the conservation of the total momentum K, there exists a transfer of linear
momentum from the vacuum field to the molecule.
http://arxiv.org/pdf/1404.5990v1.pdf
http://arxiv.org/abs/1304.6767

But it isn't JUST chiral molecules, as I've shown experimental evidence of the observation of magneto-electric non-reciprocity in molecular nitrogen gas.
https://hal.archives-ouvertes.fr/hal-00551421v1/document
http://arxiv.org/abs/1101.0712

Which means that air confined to a resonant cavity falls into their magneto-electric metric too. Any condition which creates a chiral electromagnetic vacuum environment works.

Which explains why unloaded cavities tested by the Chinese and Shawyer work too.
This implies that a magneto-electric directionally anisotropic electromagnetic environment also absolutely implies an anisotropic vacuum environment, as the authors suggest is true and aim to measure.
Quote
Moreover, the same effect is expected in quantum vacuum.

Only I found evidence last night that the vacuum anisotropy experiment has already been done in the 80s.
http://heart-c704.uibk.ac.at/LV/Quantenoptik/Kapitel_8/Jhe_etal_PRL_58_666_%281987%29.pdf

This Emdrive operates exactly as I have suggested and existing theory supports. I've beat this dead horse enough.

Wow. Those are quite remarkable papers. It seems like a very likely cause for the thrust, with previous published results availing it.

I wonder why the research wasn't pursued to its logical conclusion (a thruster). Maybe the researchers didn't believe they could get usable forces?

Also, does this mean the Emdrive actually works better with air inside the cavity?  it may be so, just not for the reason everyone is afraid of. The QV may actually impinge momentum onto the air molecules contained by the thruster.

Edit:
I just noticed the research is fairly new (except the link from the 80s). So that could explain why the experiments haven't resulted in setups with forces perceptible in the macroscopic scale.
Title: Re: EM Drive Developments - related to space flight applications - Thread 2
Post by: Flyby on 02/18/2015 07:09 pm
huh... you really don't make it easy for me , don't you?  :)

I either have to wrestle through an english translation (bad bad idea) or i need to look up and understand scientific explanation of the english scientific concepts/words you guys are using...
As my last science courses date of 30 years ago...It's not that obvious to brush up my - old and outdated - knowledge of science in a foreign language, but hey, never stepped aside for a bit of a challenge....

Thanks to this very topic, I'm offering myself a crash course in electromagnetic (micro)waves, simply by trying to look up the words and terminology and attempting to understand what you guys are talking about...

glad there is Wikipedia to assist me in these dark moments when I stare at these topics like a rabbit in a lightbox... blink.. blink.. fascinating... blink ..blink...
Title: Re: EM Drive Developments - related to space flight applications - Thread 2
Post by: Rodal on 02/18/2015 07:35 pm
Folks:

While you all talk about various ways to accomplish the E&M simulations of these frustum cavities, I thought you might like to take a look at the COMSOL derived resonances of the Eagleworks Lab's copper frustum resonant cavity driven with a ~16mm OD loop antenna located 15% up the side wall of the frustum from the large OD end of the cavity. 

BTW, the EMPower amplifiers were delivered to the Lab yesterday and I'll be calibrating the power metering for one that was installed yesterday on the torque pendulum.

Best, Paul M,

Paul,  I just calculated the natural frequency for mode TE012 without the dielectric, from the exact solution to the truncated cone, based on the geometry you provided us recently:

Large OD : 11.00 " (0.2794m)
Small OD: 6.25" (0.1588 m)
Length : 9.00 " (0.2286m)

and I obtained:

EXACT SOLUTION TE012:             2.20244 GHz
F.Davies/NASA/COMSOL TE012:   2.1794 GHz

Difference from COMSOL F.E.A.: (2.1794/2.20244 - 1)*100 = -1.05%

I am extremely impressed with the accuracy obtained by Frank Davies NASA/JSC/EP5 using COMSOL (http://forum.nasaspaceflight.com/index.php?action=dlattach;topic=36313.0;attach=796287 ): only 1% difference from the exact solution for this mode TE012.   :)

The fact that the COMSOL solution gives a little lower frequency makes perfect sense because the Finite Element solution converges from below (a finer mesh would result in a higher frequency, to reach the exact solution takes in the limit an infinite number of finite elements).
(http://forum.nasaspaceflight.com/index.php?action=dlattach;topic=36313.0;attach=796989;image)



For TE011:

EXACT SOLUTION TE011:             1.78972 GHz
F.Davies/NASA/COMSOL TE011:   1.77048 GHz

Difference from COMSOL F.E.A.: (1.77048/1.78972 - 1)*100 = -1.08%

_____

Note:

1) found some errors on Greg Egan's formulas affecting higher modes but his posted example results are OK, and his methodology is impecable
2) I am working on an exact solution for the truncated cone with the dielectric
Title: Re: EM Drive Developments - related to space flight applications - Thread 2
Post by: Notsosureofit on 02/18/2015 07:50 pm
@ RODAL

Can you generate a dispersion relation from the exact solution ?
Title: Re: EM Drive Developments - related to space flight applications - Thread 2
Post by: Rodal on 02/18/2015 07:53 pm
@ RODAL

Can you generate a dispersion relation from the exact solution ?
(Almost) anything is possible if I get the time  :)

This work done whenever I take a break from $$$ paying work  :)

It continues to snow over here  ;)
Title: Re: EM Drive Developments - related to space flight applications - Thread 2
Post by: Rodal on 02/18/2015 08:02 pm

This clapping dedicated to Paul March, Dr. White and the NASA Eagleworks team for being the first EM Drive International team to realize that it was important to calculate the mode shapes of the EM Drive, and to  Frank Davies NASA/JSC/EP5 for his excellent Finite Element Analysis using COMSOL, only 1% away from the exact solution for modes TE011 and TE012:

(http://i262.photobucket.com/albums/ii105/scoozna/misc/applause.gif)
Title: Re: EM Drive Developments - related to space flight applications - Thread 2
Post by: Notsosureofit on 02/18/2015 08:03 pm
@ RODAL

Can you generate a dispersion relation from the exact solution ?
(Almost) anything is possible if I get the time  :)

This work done whenever I take a break from $$$ paying work  :)

It continues to snow over here  ;)

Same here, and snowing again. 
Title: Re: EM Drive Developments - related to space flight applications - Thread 2
Post by: Mulletron on 02/18/2015 08:09 pm
At 16:30 the dispersion relation is focused on building on the last several minutes.

https://m.youtube.com/watch?v=Fmwu6AKPpBI

It should be able to be adapted to a cone using the same methodology.

Probably need to break out a straight edge and graph paper.
Title: Re: EM Drive Developments - related to space flight applications - Thread 2
Post by: Rodal on 02/18/2015 09:13 pm
@ RODAL

Can you generate a dispersion relation from the exact solution ?
@NotSoSureOfIt

I think that the relation between the wavenumber k and the angular frequency omega is perfectly linear for a truncated cone homogeneously filled with a medium having constant, isotropic, electric permittivity and constant magnetic permeability.  It follows from the homogeneous electromagnetic wave equation, which for the truncated cone is solved in spherical coordinates via spherical waves (this involves an assumption of spherical ends, instead of flat ends).

Is your question what is the effective dispersion relation for a truncated cavity containing a dielectric (with constant properties) filling only a portion of the truncated cone?

Or is your question what is the dispersion relation for a truncated cavity containing a dielectric with nonlinear properties? Or anisotropic properties?

Or am I wrong, or missing something, ...

Title: Re: EM Drive Developments - related to space flight applications - Thread 2
Post by: Notsosureofit on 02/18/2015 11:11 pm
@ RODAL

Can you generate a dispersion relation from the exact solution ?
@NotSoSureOfIt

I think that the relation between the wavenumber k and the angular frequency omega is perfectly linear for a truncated cone homogeneously filled with a medium having constant, isotropic, electric permittivity and constant magnetic permeability.  It follows from the homogeneous electromagnetic wave equation, which for the truncated cone is solved in spherical coordinates via spherical waves (this involves an assumption of spherical ends, instead of flat ends).

Is your question what is the effective dispersion relation for a truncated cavity containing a dielectric (with constant properties) filling only a portion of the truncated cone?

Or is your question what is the dispersion relation for a truncated cavity containing a dielectric with nonlinear properties? Or anisotropic properties?

Or am I wrong, or missing something, ...

I was just thinking of the simplest case but based on your exact solution, rather than using that of the cylindrical cavity evaluated at both ends, which is what I have been using.  The radii of both ends would presumably be already present in the single (presumably quadratic ?) expression.
Title: Re: EM Drive Developments - related to space flight applications - Thread 2
Post by: Rodal on 02/19/2015 12:05 am
@ RODAL

Can you generate a dispersion relation from the exact solution ?
@NotSoSureOfIt

I think that the relation between the wavenumber k and the angular frequency omega is perfectly linear for a truncated cone homogeneously filled with a medium having constant, isotropic, electric permittivity and constant magnetic permeability.  It follows from the homogeneous electromagnetic wave equation, which for the truncated cone is solved in spherical coordinates via spherical waves (this involves an assumption of spherical ends, instead of flat ends).

Is your question what is the effective dispersion relation for a truncated cavity containing a dielectric (with constant properties) filling only a portion of the truncated cone?

Or is your question what is the dispersion relation for a truncated cavity containing a dielectric with nonlinear properties? Or anisotropic properties?

Or am I wrong, or missing something, ...

I was just thinking of the simplest case but based on your exact solution, rather than using that of the cylindrical cavity evaluated at both ends, which is what I have been using.  The radii of both ends would presumably be already present in the single (presumably quadratic ?) expression.

1) The resonant cylindrical cavity is solved in terms of its natural coordinates: a cylindrical coordinate system rho(radius), phi(angle) and z (longitudinal coordinate).
(http://upload.wikimedia.org/wikipedia/commons/thumb/0/0e/Coord_system_CY_1.svg/240px-Coord_system_CY_1.svg.png)

1a) The resonant cylindrical cavity (with a single medium inside it) has a linear dispersion relationship between frequency and wavenumber

2a) the eigenvalue problem has a closed form solution (http://en.wikipedia.org/wiki/Microwave_cavity#Cylindrical_cavity) expressing the frequency in terms of the SquareRoot of the sum of the square of the zero Bessel function m.n (Xm,n and X'm,n are the eigenvalues ) divided by the Radius of the cylinder plus the Square of the longitudinal quantum number "p" divided by the cylinder's Length

3a) the only issue with the closed form solution (http://en.wikipedia.org/wiki/Microwave_cavity#Cylindrical_cavity) is a side equation necessary to satisfy cut-off of mode shapes that cannot take place in the cylindrical cavity ( http://en.wikipedia.org/wiki/Cutoff_frequency#Waveguides )






2)  The resonant truncated cone cavity is solved in terms of its natural coordinates: a spherical coordinate system r(radius),  theta (polar angle), and phi(azimuthal angle).  Please note that the radial coordinate "r" is entirely different in the spherical coordinate system than the radius "rho" in the cylindrical system. In the spherical system the radius "r" defines the length of the walls of the truncated cone.
(http://upload.wikimedia.org/wikipedia/commons/thumb/4/4f/3D_Spherical.svg/240px-3D_Spherical.svg.png)

2a) The resonant truncated cone cavity  (with a single medium inside it) has a linear dispersion relationship between frequency and wavenumber

2b) There are two eigenvalue problems to solve in the truncated cone:  one eigenvalue problem deals with the  spherical angle theta (polar angle) and another eigenvalue problem deals with the spherical radius "r".  Neither of these two eigenvalue problems has a closed-form solution.  Hence the frequency for the truncated cone cannot be expressed as a closed-form solution (using classical functions).  The two eigenvalue problems need to be solved numerically.  .  There is an exact solution, but it is not closed-form.

3c) while in the cylindrical cavity, the radius (or the diameter) of the cylindrical cavity appears, in the truncated cone this is not the case.  In the truncated cone the

cylindrical coordinate radius "rho1" of the small base of the cone = r1*Sin[thetaw]
and the
cylindrical coordinate radius "rho2" of the big base of the cone = r2*Sin[thetaw],

where thetaw is the cone-half-angle and r1 is the radial distance from the origin of the spherical coordinate system to the small base and r2 is  the radial distance from the origin of the spherical coordinate system to the big base.  The cone-half-angle appears in the eigenvalue problem for the polar angle theta and the radial distances r1 and r2 appear in the eigenvalue problem for the spherical radius r.  Therefore the big and small diameter do not appear explicitly in the truncated cone solution.  Instead the truncated cone solution parameters are the cone-half-angle thetaw and the spherical radii r1 and r2. And there is no closed-form solution: the frequency is the result of the numerical solution of two eigenvalue problems (which can be solved with numerical root finding procedures).

3d) The first eigenvalue problem contains Legendre functions (http://en.wikipedia.org/wiki/Legendre_function) of the Cosine of the cone-half angle.

3e) The second eigenvalue problem contains Spherical Bessel functions (http://mathworld.wolfram.com/SphericalBesselFunctionoftheFirstKind.html) in terms of the wavenumber K and the spherical radius.

3f) No cut-off auxiliary equation is necessary for the solution of the truncated cone problem.  Purely evanescent modes get cut-off automatically when solving the eigenvalue problem if one is seeking real solutions.  There are also interesting degenerate modes in the truncated cone which are resonant within a portion of the cavity and evanescent on another portion of the cavity.

(http://gregegan.customer.netspace.net.au/SCIENCE/Cavity/CavityShape.gif)
Title: Re: EM Drive Developments - related to space flight applications - Thread 2
Post by: Notsosureofit on 02/19/2015 12:32 am
Mmmm...I see the problem.   
Title: Re: EM Drive Developments - related to space flight applications - Thread 2
Post by: Rodal on 02/19/2015 12:42 am
Mmmm...I see the problem.

One could construct a table of eigenvalues (as done for the cylindrical cavity for Xm,n and X'm,n) in terms of discrete values of the spherical polar angle theta at the walls of the cone (Boundary Conditions thetaw the half-angle of the cone), and some papers for conical waveguides (as in radar) have done this.  This is of limited usefulness because it is only useful if one's truncated cone half-angle is in the table (otherwise one has to interpolate), and that solves only the first eigenvalue problem, but for a resonant cavity you still need to solve the second eigenvalue problem for spherical Bessel functions to get the frequency.  It would be unwieldy to build a table of eigenvalues for the second eigenvalue problem of the truncated cone in terms of the wavenumber k and the spherical radius r with boundary conditions at r1 and r2.
And notice that the table of eigenvalues would not be in terms of diameters or radii of the bases of the truncated cone.
Title: Re: EM Drive Developments - related to space flight applications - Thread 2
Post by: Notsosureofit on 02/19/2015 02:15 am
Might need to rethink this approach if the goal is to find the optimum thrust cavity configuration rather than an analytic solution for this particular cavity.
Title: Re: EM Drive Developments - related to space flight applications - Thread 2
Post by: Star-Drive on 02/19/2015 03:39 am
Folks:

While you all talk about various ways to accomplish the E&M simulations of these frustum cavities, I thought you might like to take a look at the COMSOL derived resonances of the Eagleworks Lab's copper frustum resonant cavity driven with a ~16mm OD loop antenna located 15% up the side wall of the frustum from the large OD end of the cavity. 

BTW, the EMPower amplifiers were delivered to the Lab yesterday and I'll be calibrating the power metering for one that was installed yesterday on the torque pendulum.

Best, Paul M,

Thank you, Paul, for posting this, the attached pdf with COMSOL plots (which I first missed, my bad, thanks to Mulletron for pointing it out to me  :) ) is very useful for all of us (those like me working on the exact solution of the frustum, and those working with numerical methods like MEEP, COMSOL, ANSYS-multiphysics, etc.).

I could not find in the pdf attached to your post, or in your post. language to indicate whether the COMSOL calculations in the attachment (http://forum.nasaspaceflight.com/index.php?action=dlattach;topic=36313.0;attach=796287) include or do not include a dielectric inside the truncated cone (frustum). My impression is that these COMSOL calculations do not include the dielectric inside the truncated cone (this impression is based on the frequency calculated for TE012).

Still a clarification for some readers like me, may be useful (to those to whom what I write below is obvious, please forgive me for taking your time):




1) Clarification: the jpg image you posted are not "COMSOL derived resonances of the Eagleworks Lab's copper frustum resonant cavity", the COMSOL plots are in the pdf attached to your post (http://forum.nasaspaceflight.com/index.php?action=dlattach;topic=36313.0;attach=796287)

2) The mode shapes in the slide are for a different geometry: a circular waveguide (instead of a truncated cone/frustum like NASA's EM Drive) , that had been plotted in published journals since 1936, without using a digital computer. The actual reference's date is 1985 (instead of 1966): http://ieeexplore.ieee.org/xpl/articleDetails.jsp?arnumber=1132998.  Since COMSOL was started in 1986, Lee, Lee and Chuang did not use COMSOL: they had no need to use a finite element program, because an exact solution exists for what they plotted: the mode shapes of a circular waveguide.

The circular waveguide exact solution is present in the 1943 textbook by Schelkunoff, and in papers he published in the 1930's.  The mode shapes look identical to the plots that Schelkunoff published in the 1930's without the benefit of a digital computer: http://forum.nasaspaceflight.com/index.php?topic=36313.msg1332981#msg1332981.  The article by Lee, Lee and Chuang states: "The earliest plots of modal field distribution in rectangular/circular waveguides were given by Southworth (1936), Barrow (1936), Schelkunoff (1937), and Chu and Barrow (1937)"

3) The solution for a truncated cone (the geometry of the NASA frustum, and the geometry analyzed by COMSOL in the attached pdf) is different from the solution for a circular waveguide in several respects: the natural frequencies are quite different, and while in a circular waveguide different mode shapes are either resonant or cutoff, in a truncated cone some mode shapes are resonant, some are evanescent and some have a transition from resonant to evanescent (a characteristic not present in circular waveguides).  Also, the attenuation and focusing aspects of the truncated cone (frustum) are not present in the circular waveguide.


Dr. Rodal:

Sorry I wasn't more explicit with my commentary this morning, but I plead not enough coffee!  :)  But to answer your first and second questions, the circular waveguide picture was supplied just as a marker from where we started from, that led us to the conclusion that we would have to model the E&M resonances in the copper frustum in the best manner possible before going forward with our testing.  I was NOT trying to imply that COMSOL was used to generate this circular waveguide modes table and the source given was where I found it. 

That said the NASA/JSC/EP group where we work had one COMSOL license seat that had to be shared amongst all the engineers who had a need for its analytical capabilities.  We were also lucky enough to have one EP5 electrical engineer, Frank Davies, from Rice University who had taken an interest into what Sonny and I were doing in the Eagleworks Lab because his office mate also had a deep interest in the topic.  And as it turned out Frank also knew how to work COMSOL for his battery research work that he does for his day job at NASA/JSC.  Where this interest led is demonstrated in Frank's COMSOL summary report I posted this morning.   And yes this COMSOL analysis was for the copper frustum without any dielectric discs in in it.

For question 3, I agree with you on all counts, but you have to remember that we were a bunch of electrical and aerospace engineers that only vaguely remembered their E&M from 20 to 40 years ago so we had to learn or relearn a lot of E&M basics before getting around to nit picking the details you are worried about.

Best,  Paul
Title: Re: EM Drive Developments - related to space flight applications - Thread 2
Post by: Mulletron on 02/19/2015 06:13 am
Well this is a gut check moment and there is no doubt this post will be controversial and might get me in trouble for being off topic and for the subject matter. I am being serious here. I'm no dummy. Either we didn't figure out jack or we actually did. It seems very likely that we did. If we did, we weren't the first.

We all know how the world works. If a bunch of guys on an internet forum can figure out the basic principles of "q-thrusters".... someone else has already perfected them and kept them black; rightfully so. The cat is now out of the bag gentlemen. We want to explore space. I want to see human space missions to the planets. We all do. Please give what you have to the world.

This isn't tin foil hat wearing nonsense. NOT UFO conspiracy theory bs. It is no different than this:
http://www.aviationweek.com/technology/skunk-works-reveals-compact-fusion-reactor-details
http://en.wikipedia.org/wiki/Skunk_Works

#giveusthegoods

Sorry moderators. This won't happen again. I don't want the thread to devolve into this kind of discussion. I am pointing out the obvious. I'll delete the post if asked to do so.
Title: Re: EM Drive Developments - related to space flight applications - Thread 2
Post by: JPLeRouzic on 02/19/2015 07:35 am
Strong H field at dielectric disc. Just like theory suggests.

Wow. Those are quite remarkable papers. It seems like a very likely cause for the thrust, with previous published results availing it.

I wonder why the research wasn't pursued to its logical conclusion (a thruster). Maybe the researchers didn't believe they could get usable forces?
Symetry violations are hot topics in physic since decades.
For example for someone who don't attempt to detect electrons, Wu's experiment may looks like some kind of propellantless drive, but actually it isn't: http://en.wikipedia.org/wiki/Wu_experiment

Maybe EMDrive is some weird version of Wu's device, but as I wrote nearly one year ago, there are many possible explanations and one man life is not enough to try all of them. It's much more efficient to carefully select and focus on problems that would help us create new knowledge and to build controlled experimentations based on what we already know. That the way science is done since centuries. One step at a time.


Title: Re: EM Drive Developments - related to space flight applications - Thread 2
Post by: Star One on 02/19/2015 11:39 am
Well this is a gut check moment and there is no doubt this post will be controversial and might get me in trouble for being off topic and for the subject matter. I am being serious here. I'm no dummy. Either we didn't figure out jack or we actually did. It seems very likely that we did. If we did, we weren't the first.

We all know how the world works. If a bunch of guys on an internet forum can figure out the basic principles of "q-thrusters".... someone else has already perfected them and kept them black; rightfully so. The cat is now out of the bag gentlemen. We want to explore space. I want to see human space missions to the planets. We all do. Please give what you have to the world.

This isn't tin foil hat wearing nonsense. NOT UFO conspiracy theory bs. It is no different than this:
http://www.aviationweek.com/technology/skunk-works-reveals-compact-fusion-reactor-details
http://en.wikipedia.org/wiki/Skunk_Works

#giveusthegoods

Sorry moderators. This won't happen again. I don't want the thread to devolve into this kind of discussion. I am pointing out the obvious. I'll delete the post if asked to do so.

All I'll say is if it wasn't meant to be discussed publicly it wouldn't be, the fact that is, is your answer and that's the end of that OT post by me. Sorry for going off track.
Title: Re: EM Drive Developments - related to space flight applications - Thread 2
Post by: Notsosureofit on 02/19/2015 11:59 am
FYI

https://www.dropbox.com/s/xiw9mq7dawhp00c/other1.jpg?dl=0


What I would really like to do is stack these 2 puppies on the left into a single vertical chamber and try the Cavandish type experiment under battery power w/ a solid-state oscillator.

OK, couldn't get the pic to post but the link works ... so that would be my retirement plan (fat chance)
Title: Re: EM Drive Developments - related to space flight applications - Thread 2
Post by: Rodal on 02/19/2015 12:16 pm
FYI

https://www.dropbox.com/s/xiw9mq7dawhp00c/other1.jpg?dl=0


What I would really like to do is stack these 2 puppies on the left into a single vertical chamber and try the Cavandish type experiment under battery power w/ a solid-state oscillator.

OK, couldn't get the pic to post but the link works ... so that would be my retirement plan (fat chance)
(http://cosmic-fitness.com/wp-content/uploads/2013/09/go-for-it.jpg)
Title: Re: EM Drive Developments - related to space flight applications - Thread 2
Post by: Notsosureofit on 02/19/2015 01:25 pm
(http://www.dropbox.com/s/cxfcrf822n0dpsa/IMAG0359.jpg)

?? can't get pics to post directly .. must be the Alzhiemers !

(Chris Edit: Hmmm, needs to be attached from your hard drive, as I tried correcting the link, but it doesn't like showing embedded via dropbox it seems).
Title: Re: EM Drive Developments - related to space flight applications - Thread 2
Post by: Notsosureofit on 02/19/2015 01:31 pm
A little less fantasy;

https://www.dropbox.com/s/cxfcrf822n0dpsa/IMAG0359.jpg?dl=0

I'm (slowly) collecting parts for an X-band miniature version in the cans shown in the middle of the picture.

Higher frequency and lower power. (we have a lot of experience w/ battery power in vacuum and/or plasma)

Anybody have a short taper section from X to something smaller ?
Title: Re: EM Drive Developments - related to space flight applications - Thread 2
Post by: Rodal on 02/19/2015 01:40 pm
A little less fantasy;

https://www.dropbox.com/s/cxfcrf822n0dpsa/IMAG0359.jpg?dl=0

I'm (slowly) collecting parts for an X-band miniature version in the cans shown in the middle of the picture.

Higher frequency and lower power. (we have a lot of experience w/ battery power in vacuum and/or plasma)

Anybody have a short taper section from X to something smaller ?

<<the frequency range of X band is rather indefinitely set at approximately 7.0 to 11.2 gigahertz (GHz). In radar engineering, the frequency range is specified by the IEEE at 8.0 to 12.0 GHz >>

For similar dimensions as present, that's going to result in high natural frequency mode shapes with lots of different mode shapes bunched up next to each other at nearby frequencies...

It will indeed have to be a smaller diameters, shorter section...

If you have a truncated cone geometry in mind, I can compute the frequencies and mode shapes from the exact solution to the truncated cone (I need: axial length, Big Diameter and Small Diameter)
Title: Re: EM Drive Developments - related to space flight applications - Thread 2
Post by: Notsosureofit on 02/19/2015 01:58 pm
Yes, oscillator called for, mode = unknown, F down by 1000, but more sensitive ??

The power heads are on X-band rectangular waveguide fittings, so a rectangular taper is what I was thinking as these are out there (EBay ?)
Title: Re: EM Drive Developments - related to space flight applications - Thread 2
Post by: Rodal on 02/19/2015 02:01 pm
(https://www.dropbox.com/s/cxfcrf822n0dpsa/IMAG0359.jpg?dl=0)

?? can't get pics to post directly .. must be the Alzhiemers !

Only .jpg, .png, .gif etc. file formats can show.  For example .bmp file extension does not post.

You have to insert the following: [img] before the link (no spacing allowed)

and you have to insert:   [/img] at the end of the link (no spacing allowed)

For example:

[img]http://ep.yimg.com/ca/I/yhst-37994427369291_2241_23717082[/img]

will show this image

(http://ep.yimg.com/ca/I/yhst-37994427369291_2241_23717082)

This can also be done by clicking on the InsertImage button right underneath the B (bold) button, if you prefer

Try it again like this, it's fun and informative  :)


Or you can attach the image where it says "Attach" with a button saying "Choose File"


EDIT: Apparently the problem is with the way that Dropbox posts the image in a webpage: the dropbox link does not show like other links
Title: Re: EM Drive Developments - related to space flight applications - Thread 2
Post by: Chris Bergin on 02/19/2015 02:21 pm
Firstly, I want to thank people on here for really focusing this thread, especially after the first thread had some problems. I can tell you it has the attention of some big hitters at NASA - who I've invited to post in this thread (at least one is already, but I'm not allowed to name him).

It has also been suggested we look at summarizing the progress made in this thread into a news article. Obviously, this is out of my comfort zone of things like current launch vehicles, but I'm going to sit down this weekend and read every single post in this thread. ;D

Regardless, as I've always said, there's so many members here who would be able to write better articles than I, because they are well versed in the specific subject - which is half the battle. That is even more the case with this thread.

So, per the suggestion, I'm looking to crowd source members posting in this thread to look at being part of creating an article to cover this subject. I already have one offer to be part of that team and if you're interested in helping out, then please PM me and we'll set up a joint PM between all interested people as we go.....because probably setting up a thread (private or otherwise) to build it.

This is a 81,000 viewed thread, but that's over time and the news site article is where you can get that in a day. The benefit to all is it would get it into a much wider audience and likely bring in a lot more folk who may be able to help with the thread, by nature of the news site being interlinked with the forum.
Title: Re: EM Drive Developments - related to space flight applications - Thread 2
Post by: Cinder on 02/19/2015 02:49 pm
EDIT: Apparently the problem is with the way that Dropbox posts the image in a webpage: the dropbox link does not show like other links
It is probably simplest to do as elsewhere on the forum: attach/upload the pictures directly.  Especially now that the effort has probably snowballed for good.
Title: Re: EM Drive Developments - related to space flight applications - Thread 2
Post by: Star One on 02/19/2015 03:59 pm
Can I just add to this thread that though my understanding barely scratches the surface of this topic that I consider it a great privilege to be able to sit in on the discussion of far more learned minds than mine.:)
Title: Re: EM Drive Developments - related to space flight applications - Thread 2
Post by: FuturePass on 02/19/2015 05:56 pm
Isn't frustum a misnomer since frustum refers to a cone, not a four-sided structure...?

Is the frustum-under-test (fut) just a truncated pyramid structure?

Scaling of the frustron could be done
by dimension (monolith) or by array of smaller units.
The following is a raw propulsion calculation only
and does not address or include power, weight, heat transfer
and other system engineering issues.

Imagine a frustum approximately the size of the Giza pyramid with dimensions is

(b x h) 146 m x 230 m  (450 ft x 750 ft).

If the current frustum is approximately 0.146 m x 0.230 m ,
then scaling factor for the frustum volume to the full pyramid
can be grossly calculated by approximately 146 / 0.146 or about 1000 per dimension.


For 100 mN thrust, the total scale factor is

(l w h) == 1000 x 1000 x 1000 = 1 B scaling factor.

If the a frustum produces 100 mN thrust,
a Giza size structure  might produce thrust of 100 million Newtons.

In comparison, a Saturn V First stage
produces approximately 34 kN (7,648,000 lbf)

If we scale up by a only a factor of 10 or 100,
we have respectively thrust maximums of 100 N or 100 kN.


YMMV
Title: Re: EM Drive Developments - related to space flight applications - Thread 2
Post by: Rodal on 02/19/2015 06:05 pm
Isn't frustum a misnomer since frustum refers to a cone, not a four-sided structure...?

Is the frustum-under-test (fut) just a truncated pyramid structure?

...
Welcome to this thread.

It is not a misnomer, what is called the "frustum" by NASA's Brady et.al. is a frustum of a cone, or a truncated cone.  It is not a truncated pyramid, as shown by the images below for NASA's truncated cone (ditto for Shawyer's EM Drive in the UK and Prof. Juan Yang's in China)

Frustum:  "the portion of a cone or pyramid that remains after its upper part has been cut off by a plane parallel to its base, or that is intercepted between two such planes"
Title: Re: EM Drive Developments - related to space flight applications - Thread 2
Post by: Rodal on 02/19/2015 06:19 pm
...

If the a frustum produces 100 mN thrust,
a Giza size structure  might produce thrust of 100 million Newtons.

In comparison, a Saturn V First stage
produces approximately 34 kN (7,648,000 lbf)

If we scale up by a only a factor of 10 or 100,
we have respectively thrust maximums of 100 N or 100 kN.
...
The thrust of the EM Drive (if it is not an artifact, and if indeed it can be used for space propulsion) it is not expected to scale geometrically.  On the contrary, (as discussed previously in this thread), the larger the diameter, the lower the natural frequencies of the first few modes.  A significantly larger EM Drive truncated cone would result in either very low natural frequencies for the first few modes, or if one would attempt to excite the EM Drive in the GHz range, these natural frequencies would correspond to very high mode shapes that would be bunched up together in the frequency spectrum and it would be even much more difficult to keep the EM Drive in resonance with a particular mode shape.  Also, very high natural frequency mode shapes are extremely difficult to calculate accurately (even with the exact solution, and almost hopeless with the Finite Element method because of difficult issues associated with ill-conditioning in inverting the matrix for very large eigenvalues).

What is envisioned (if it is not an artifact, and if indeed it can be used for space propulsion) , rather than a very large EM Drive, is to maximize the thrust of the EM Drive (perhaps using superconductivity, and/or high magnetic permeability materials for the big flat end, as well as much better, anisotropic, dielectric) and to use a number of EM Drives to achieve a large overall thrust.

Unlike liquid propellant rocket engines, there are no issues of fuels and oxidizers hydraulic lines,  pumps, injectors, combustion instability, ignition, hydraulic pressure, etc., associated with using a large number of EM Drives, of course, particularly if ambient temperature EM Drives are used (and superconducting low temperature solutions can be avoided).
Title: Re: EM Drive Developments - related to space flight applications - Thread 2
Post by: Mulletron on 02/19/2015 06:21 pm
Not really my expertise field.............
Welcome to the group. This is very much a multidisciplinary effort. What are you good at?
Title: Re: EM Drive Developments - related to space flight applications - Thread 2
Post by: Notsosureofit on 02/19/2015 06:47 pm
FYI

http://www.intelligent-aerospace.com/articles/2015/02/boeing-to-build-all-electric-propulsion-satellite-for-ses.html
Title: Re: EM Drive Developments - related to space flight applications - Thread 2
Post by: Rodal on 02/19/2015 06:49 pm
FYI

http://www.intelligent-aerospace.com/articles/2015/02/boeing-to-build-all-electric-propulsion-satellite-for-ses.html
Does it work by work by electrically expelling propellant (reaction mass) at high speed?

http://spacenews.com/35894electric-propulsion-satellites-are-all-the-rage/
Title: Re: EM Drive Developments - related to space flight applications - Thread 2
Post by: TescoBag on 02/19/2015 06:50 pm
...

If the a frustum produces 100 mN thrust,
a Giza size structure  might produce thrust of 100 million Newtons.

In comparison, a Saturn V First stage
produces approximately 34 kN (7,648,000 lbf)

If we scale up by a only a factor of 10 or 100,
we have respectively thrust maximums of 100 N or 100 kN.
...
The thrust of the EM Drive (if it is not an artifact, and if indeed it can be used for space propulsion) it is not expected to scale geometrically.  On the contrary, (as discussed previously in this thread), the larger the diameter, the lower the natural frequencies of the first few modes.  A significantly larger EM Drive truncated cone would result in either very low natural frequencies for the first few modes, or if one would attempt to excite the EM Drive in the GHz range, these natural frequencies would correspond to very high mode shapes that would be bunched up together in the frequency spectrum and it would be even much more difficult to keep the EM Drive in resonance with a particular mode shape.  Also, very high natural frequency mode shapes are extremely difficult to calculate accurately (even with the exact solution, and almost hopeless with the Finite Element method because of difficult issues associated with ill-conditioning in inverting the matrix for very large eigenvalues).

What is envisioned (if it is not an artifact, and if indeed it can be used for space propulsion) , rather than a very large EM Drive, is to maximize the thrust of the EM Drive (perhaps using superconductivity, and/or high magnetic permeability materials for the big flat end, as well as much better, anisotropic, dielectric) and to use a number of EM Drives to achieve a large overall thrust.

Unlike liquid propellant rocket engines, there are no issues of fuels and oxidizers hydraulic lines,  pumps, injectors, combustion instability, ignition, hydraulic pressure, etc., associated with using a large number of EM Drives, of course, particularly if ambient temperature EM Drives are used (and superconducting low temperature solutions are avoided).

Hi,

Just want to say that I am enjoying this thread immensely even though a lot of the science is a bit above my pay grade!

If the thrust is not an artifact of the experiment, in theory what level of thrust are we expecting in the best circumstances?
Title: Re: EM Drive Developments - related to space flight applications - Thread 2
Post by: aceshigh on 02/19/2015 06:52 pm
A question... is the Q-Thruster that was being worked by EagleWorks Lab similar to the EM Drive they tested? Are them completely different things with different principles? I have the vague notion of seeing a photo of it long ago (while the main subject of the article was in fact Dr White's Warp Drive experiments) and I don´t remember it having a cone similar to Shawyer's EM Drive, so I guess they are totally different principles? I wonder if the former is still being pursued? If it is "considered" an EM Drive? Maybe Paul March can clear this up (if noone else knows the answer)?
Title: Re: EM Drive Developments - related to space flight applications - Thread 2
Post by: Star One on 02/19/2015 06:52 pm
Would one practical use, if they do work, be a satellite with clusters of EM drives on different axis used for station keeping?

Of how much practical use would they be in reality for interplanetary probes.
Title: Re: EM Drive Developments - related to space flight applications - Thread 2
Post by: Mulletron on 02/19/2015 06:56 pm
FYI

http://www.intelligent-aerospace.com/articles/2015/02/boeing-to-build-all-electric-propulsion-satellite-for-ses.html
Does it work by work by electrically expelling propellant (reaction mass) at high speed?

http://spacenews.com/35894electric-propulsion-satellites-are-all-the-rage/

I'm familiar with the Boeing 702, they use Xenon Ion Propulsion System (XIPS). The only kind of satellite I know, no wait, also the old DSCS III ones too.
http://en.wikipedia.org/wiki/Wideband_Global_SATCOM#Segments
http://en.wikipedia.org/wiki/Boeing_702#Customers
Title: Re: EM Drive Developments - related to space flight applications - Thread 2
Post by: Rodal on 02/19/2015 06:58 pm
.
The thrust of the EM Drive (if it is not an artifact, and if indeed it can be used for space propulsion) it is not expected to scale geometrically.  On the contrary, (as discussed previously in this thread), the larger the diameter, the lower the natural frequencies of the first few modes.  A significantly larger EM Drive truncated cone would result in either very low natural frequencies for the first few modes, or if one would attempt to excite the EM Drive in the GHz range, these natural frequencies would correspond to very high mode shapes that would be bunched up together in the frequency spectrum and it would be even much more difficult to keep the EM Drive in resonance with a particular mode shape.  Also, very high natural frequency mode shapes are extremely difficult to calculate accurately (even with the exact solution, and almost hopeless with the Finite Element method because of difficult issues associated with ill-conditioning in inverting the matrix for very large eigenvalues).

What is envisioned (if it is not an artifact, and if indeed it can be used for space propulsion) , rather than a very large EM Drive, is to maximize the thrust of the EM Drive (perhaps using superconductivity, and/or high magnetic permeability materials for the big flat end, as well as much better, anisotropic, dielectric) and to use a number of EM Drives to achieve a large overall thrust.

Unlike liquid propellant rocket engines, there are no issues of fuels and oxidizers hydraulic lines,  pumps, injectors, combustion instability, ignition, hydraulic pressure, etc., associated with using a large number of EM Drives, of course, particularly if ambient temperature EM Drives are used (and superconducting low temperature solutions are avoided).

Hi,

Just want to say that I am enjoying this thread immensely even though a lot of the science is a bit above my pay grade!

If the thrust is not an artifact of the experiment, in theory what level of thrust are we expecting in the best circumstances?

Welcome to the forum.

There is no unifying theory yet.  Some expect it to be an experimental artifact.  Some propose ambitious crewed missions to the outer planets. Dr. White at NASA proposed the following mission parameters as feasible

2MW NEP (90t spacecraft) Crewed Titan/Enceladus Mission with 0.4N/kWe thrust to power

9 month trip each way

P= 2000 kW

(http://1.bp.blogspot.com/-qpqga7NnEow/U-F3iMmhokI/AAAAAAAAxLg/aEdtFbkcqF4/s1600/nasatestemdrive6.png)
Title: Re: EM Drive Developments - related to space flight applications - Thread 2
Post by: aceshigh on 02/19/2015 07:07 pm
those mission parameters presupposed electrical power from solar panels? I guess that some good fission or fusion reactors (whenever fusion is available) would allow the addition of even more EM Drives and cut that mission time several times.
Title: Re: EM Drive Developments - related to space flight applications - Thread 2
Post by: Star One on 02/19/2015 07:08 pm
This is what confusing about this as some proponents of the technology talk of grand projects others speak of far more modest expectations and to the layperson that's rather confusing.
Title: Re: EM Drive Developments - related to space flight applications - Thread 2
Post by: TescoBag on 02/19/2015 07:12 pm
This is what confusing about this as some proponents of the technology talk of grand projects others speak of far more modest expectations and to the layperson that's rather confusing.

I would think that the larger, more optimistic predictions are relying on technology that hasn't been developed yet but is in theory possible.
Title: Re: EM Drive Developments - related to space flight applications - Thread 2
Post by: RotoSequence on 02/19/2015 07:17 pm
This is what confusing about this as some proponents of the technology talk of grand projects others speak of far more modest expectations and to the layperson that's rather confusing.

There's a lot of uncertainty about how scalable these drives will be (assuming they work). It hasn't proven easy to create a consistent dataset with existing test articles and equipment, and there's little knowledge about how much potential EM drives have.

One order of magnitude improvement will enable an interesting subset of station keeping thrusters and low-acceleration drives. Two orders of magnitude yields what we see in the above the Saturnian mission profile. Three or more orders of magnitude would make rockets obsolete.
Title: Re: EM Drive Developments - related to space flight applications - Thread 2
Post by: Star One on 02/19/2015 07:19 pm
This is what confusing about this as some proponents of the technology talk of grand projects others speak of far more modest expectations and to the layperson that's rather confusing.

I would think that the larger, more optimistic predictions are relying on technology that hasn't been developed yet but is in theory possible.

Good point. The other point of confusion is this purely a technology if it works that is only useful outside the atmosphere because of the conditions of its operation. I've read it couldn't actual get anything on its own off the ground, so it's of no use for a making a launch vehicle. We aren't going to get George Jetson getting in his flying car and nipping off to the moon.
Title: Re: EM Drive Developments - related to space flight applications - Thread 2
Post by: TescoBag on 02/19/2015 07:29 pm
This is what confusing about this as some proponents of the technology talk of grand projects others speak of far more modest expectations and to the layperson that's rather confusing.

I would think that the larger, more optimistic predictions are relying on technology that hasn't been developed yet but is in theory possible.

Good point. The other point of confusion is this purely a technology if it works that is only useful outside the atmosphere because of the conditions of its operation. I've read it couldn't actual get anything on its own off the ground, so it's of no use for a making a launch vehicle. We aren't going to get George Jetson getting in his flying car and nipping off to the moon.

Shawyer has seemed convinced that this drive is able to replace commercial airliner engines etc. Obviously as he is the main proponent of the drive so this might be taken with a pinch of salt or two but if the drive is proven to be real he definitely seems to be ahead of the curve with the technology.
Title: Re: EM Drive Developments - related to space flight applications - Thread 2
Post by: Mulletron on 02/19/2015 07:34 pm
A question... is the Q-Thruster that was being worked by EagleWorks Lab similar to the EM Drive they tested? Are them completely different things with different principles? I have the vague notion of seeing a photo of it long ago (while the main subject of the article was in fact Dr White's Warp Drive experiments) and I don´t remember it having a cone similar to Shawyer's EM Drive, so I guess they are totally different principles? I wonder if the former is still being pursued? If it is "considered" an EM Drive? Maybe Paul March can clear this up (if noone else knows the answer)?

It appears that when METs, MLT, SFEs, Cannaes and Emdrives go through the doors of Eagleworks, they get rolled into the QVPT conjecture. In reality, they come from different sources, different inventors with different theories of operation. Dr. White, IMHO is right to try and unify these different types of "thrusters" under the same paradigm. I think they are all unified too under the same interaction, maybe not QVPT per se, but they share the QV as a common means of interaction.
https://www.linkedin.com/pub/hector-serrano/29/69b/9a5
https://xa.yimg.com/kq/groups/86787010/513081407/name/Eagleworks+Newsletter+2013.pdf
http://ntrs.nasa.gov/archive/nasa/casi.ntrs.nasa.gov/20110023492.pdf
http://ntrs.nasa.gov/archive/nasa/casi.ntrs.nasa.gov/20140009930.pdf
Title: Re: EM Drive Developments - related to space flight applications - Thread 2
Post by: matthewpapa on 02/19/2015 07:37 pm
Would one practical use, if they do work, be a satellite with clusters of EM drives on different axis used for station keeping?

Of how much practical use would they be in reality for interplanetary probes.

That, or have it use a gyroscope.
Since no propellant is emitted, this should be feasible I think. Could be really cool!

Could be great for interplanetary probes since they can just go from destination to destination. Worst case it has to come back to earth every once in a while for new fuel rods, but you essentially have unlimited delta-v. As long as the force of your EM drive output force is greater than any nearby gravity well you arent orbiting
Title: Re: EM Drive Developments - related to space flight applications - Thread 2
Post by: Rodal on 02/19/2015 07:51 pm
A question... is the Q-Thruster that was being worked by EagleWorks Lab similar to the EM Drive they tested? Are them completely different things with different principles? I have the vague notion of seeing a photo of it long ago (while the main subject of the article was in fact Dr White's Warp Drive experiments) and I don´t remember it having a cone similar to Shawyer's EM Drive, so I guess they are totally different principles? I wonder if the former is still being pursued? If it is "considered" an EM Drive? Maybe Paul March can clear this up (if noone else knows the answer)?

It appears that when METs, MLT, SFEs, Cannaes and Emdrives go through the doors of Eagleworks, they get rolled into the QVPT conjecture. In reality, they come from different sources, different inventors with different theories of operation. Dr. White, IMHO is right to try and unify these different types of "thrusters" under the same paradigm. I think they are all unified too under the same interaction, maybe not QVPT per se, but they share the QV as a common means of interaction.
https://www.linkedin.com/pub/hector-serrano/29/69b/9a5
https://xa.yimg.com/kq/groups/86787010/513081407/name/Eagleworks+Newsletter+2013.pdf
http://ntrs.nasa.gov/archive/nasa/casi.ntrs.nasa.gov/20110023492.pdf
http://ntrs.nasa.gov/archive/nasa/casi.ntrs.nasa.gov/20140009930.pdf

OK with Eagleworks doing that, but for this thread let's continue to keep the focus on EM Drive Developments - related to space flight applications, as per SpaceFLight Forum Admin., with EM Drive defined as microwave cavity resonators , of converging focusing type, as the Cannae pillbox and the truncated cones with flat ends at NASA, Shawyer (UK) and Juan Yang (China) or Shawyer's superconducting EM Drive with spherical ends.

Discussions of piezoelectric MET can be conducted in this thread:  http://forum.nasaspaceflight.com/index.php?topic=31037.460

so as to avoid getting into the problems suffered in Thread 1.
Title: Re: EM Drive Developments - related to space flight applications - Thread 2
Post by: tchernik on 02/19/2015 08:07 pm
Shawyer has seemed convinced that this drive is able to replace commercial airliner engines etc. Obviously as he is the main proponent of the drive so this might be taken with a pinch of salt or two but if the drive is proven to be real he definitely seems to be ahead of the curve with the technology.

And if we believe the Chinese results, the tech is squarely at the "useful for probes and satellite station-keeping/ re-positoning" phase now.

The current validation work at NASA, besides the potential synergies with other similar efforts, and even the talks of smart people from Internet forums (not me, I'm just happy to be here), should allow interested and currently acting people to perceive potential vectors in the space of improvement, resulting in increases of  the much sought Newtons per Watt ratio.

That's of the utmost importance, because the higher such ratio, the easier to show this phenomenon's existence without the shadow of a doubt.
Title: Re: EM Drive Developments - related to space flight applications - Thread 2
Post by: Mulletron on 02/19/2015 08:09 pm
I guess not everyone realizes that a resonant cavity can be represented as an LC circuit. They're all the same thing.

(https://encrypted-tbn1.gstatic.com/images?q=tbn:ANd9GcSeF0z098aKsq-NFkxYS7IypVicisG8IM3AiidTeKKRbh8RFIT1B1y5Ju3J)
Title: Re: EM Drive Developments - related to space flight applications - Thread 2
Post by: Rodal on 02/19/2015 08:25 pm
I guess not everyone realizes that a resonant cavity can be represented as an LC circuit. They're all the same thing.

(https://encrypted-tbn1.gstatic.com/images?q=tbn:ANd9GcSeF0z098aKsq-NFkxYS7IypVicisG8IM3AiidTeKKRbh8RFIT1B1y5Ju3J)
It can be represented by a simple LC circuit only for simple uniform cavities, with uniform cross-sections, as for example the rectangular cross section cavity or the cylindrical cavity. 

The truncated cone (frustum) shape used by NASA, Shawyer in the UK and Prof. Juan Yang in China displays degenerate modes that go from resonant to evanescent, and it displays modes that do not conform to the same TEmnp or TMmnp designation as in cylindrical cavities.  Actually in reviewing the mode shapes assigned in the COMSOL study for NASA I am now reviewing some interesting cases (the frequencies and images computed by COMSOL are excellent, but the designation of some of the modes is not straightforward, as the NASA engineer realized when designating some of the modes as "X").

The truncated cone and the Cannae pillbox effect also display very interesting attenuation and focusing effects not present in the uniform-cross section cavities.  These focusing effects may be very important for the EM Drive.
Title: Re: EM Drive Developments - related to space flight applications - Thread 2
Post by: Far Reach on 02/19/2015 08:34 pm
FYI

http://www.intelligent-aerospace.com/articles/2015/02/boeing-to-build-all-electric-propulsion-satellite-for-ses.html

Lots of SEP here on NSF>

http://www.nasaspaceflight.com/?s=%28SEP%29

Including Aerojet on its potential
http://www.nasaspaceflight.com/2012/01/aerojet-solar-electric-propulsion-enabler-exploration-gateway/
Title: Re: EM Drive Developments - related to space flight applications - Thread 2
Post by: Notsosureofit on 02/19/2015 08:37 pm
I guess not everyone realizes that a resonant cavity can be represented as an LC circuit. They're all the same thing.

(https://encrypted-tbn1.gstatic.com/images?q=tbn:ANd9GcSeF0z098aKsq-NFkxYS7IypVicisG8IM3AiidTeKKRbh8RFIT1B1y5Ju3J)
It can be represented by a simple LC circuit only for simple uniform cavities, with uniform cross-sections, as for example the rectangular cross section cavity or the cylindrical cavity. 

The truncated cone (frustum) shape used by NASA, Shawyer in the UK and Prof. Juan Yang in China displays degenerate modes that go from resonant to evanescent, and it displays modes that do not conform to the same TEmnp or TMmnp designation as in cylindrical cavities.  Actually in reviewing the mode shapes assigned in the COMSOL study for NASA I am now reviewing some interesting cases (the frequencies and images computed by COMSOL are excellent, but the designation of some of the modes is not straightforward, as the NASA engineer realized when designating some of the modes as "X").

There is some flexibility.  I remember working on delay lines w/ parameters varying w/ length.

That was odd stuff, I wonder if it can make a resonant circuit that way?
Title: Re: EM Drive Developments - related to space flight applications - Thread 2
Post by: Rodal on 02/19/2015 08:45 pm
I guess not everyone realizes that a resonant cavity can be represented as an LC circuit. They're all the same thing.

(https://encrypted-tbn1.gstatic.com/images?q=tbn:ANd9GcSeF0z098aKsq-NFkxYS7IypVicisG8IM3AiidTeKKRbh8RFIT1B1y5Ju3J)
It can be represented by a simple LC circuit only for simple uniform cavities, with uniform cross-sections, as for example the rectangular cross section cavity or the cylindrical cavity. 

The truncated cone (frustum) shape used by NASA, Shawyer in the UK and Prof. Juan Yang in China displays degenerate modes that go from resonant to evanescent, and it displays modes that do not conform to the same TEmnp or TMmnp designation as in cylindrical cavities.  Actually in reviewing the mode shapes assigned in the COMSOL study for NASA I am now reviewing some interesting cases (the frequencies and images computed by COMSOL are excellent, but the designation of some of the modes is not straightforward, as the NASA engineer realized when designating some of the modes as "X").

There is some flexibility.  I remember working on delay lines w/ parameters varying w/ length.

That was odd stuff, I wonder if it can make a resonant circuit that way?

I agree, that's why I wrote "It can be represented by a simple LC circuit only".  Yes, with a circuit complicated enough we could probably simulate most electromagnetic wave phenomena, just like the few analog computers that still were being used at MIT Labs in the early 1970's to solve differential equations.  I remember those  :).   Reconfiguring the analog computer to solve a different equation required actual handwork unlike just writing software for digital computers :-)

(http://www.research.rutgers.edu/~imacs/index_files/anolog50S.jpg)
Title: Re: EM Drive Developments - related to space flight applications - Thread 2
Post by: Notsosureofit on 02/19/2015 08:52 pm
I guess not everyone realizes that a resonant cavity can be represented as an LC circuit. They're all the same thing.

(https://encrypted-tbn1.gstatic.com/images?q=tbn:ANd9GcSeF0z098aKsq-NFkxYS7IypVicisG8IM3AiidTeKKRbh8RFIT1B1y5Ju3J)
It can be represented by a simple LC circuit only for simple uniform cavities, with uniform cross-sections, as for example the rectangular cross section cavity or the cylindrical cavity. 

The truncated cone (frustum) shape used by NASA, Shawyer in the UK and Prof. Juan Yang in China displays degenerate modes that go from resonant to evanescent, and it displays modes that do not conform to the same TEmnp or TMmnp designation as in cylindrical cavities.  Actually in reviewing the mode shapes assigned in the COMSOL study for NASA I am now reviewing some interesting cases (the frequencies and images computed by COMSOL are excellent, but the designation of some of the modes is not straightforward, as the NASA engineer realized when designating some of the modes as "X").

There is some flexibility.  I remember working on delay lines w/ parameters varying w/ length.

That was odd stuff, I wonder if it can make a resonant circuit that way?

I agree, that's why I wrote "It can be represented by a simple LC circuit only".  Yes, with a circuit complicated enough we could probably simulate most electromagnetic wave phenomena, just like the few analog computers that still were being used at MIT Draper Labs in the early 1970's to solve differential equations.  I remember those  :).   Reconfiguring the analog computer to solve a different equation required actual handwork unlike just writing software for digital computers :-)

I guess I can't stop thinking about the 1/ f^3 in the thrust equation.

Dual 12at7's I think....
Title: Re: EM Drive Developments - related to space flight applications - Thread 2
Post by: Rodal on 02/19/2015 08:57 pm

It can be represented by a simple LC circuit only for simple uniform cavities, with uniform cross-sections, as for example the rectangular cross section cavity or the cylindrical cavity. 

The truncated cone (frustum) shape used by NASA, Shawyer in the UK and Prof. Juan Yang in China displays degenerate modes that go from resonant to evanescent, and it displays modes that do not conform to the same TEmnp or TMmnp designation as in cylindrical cavities.  Actually in reviewing the mode shapes assigned in the COMSOL study for NASA I am now reviewing some interesting cases (the frequencies and images computed by COMSOL are excellent, but the designation of some of the modes is not straightforward, as the NASA engineer realized when designating some of the modes as "X").

There is some flexibility.  I remember working on delay lines w/ parameters varying w/ length.

That was odd stuff, I wonder if it can make a resonant circuit that way?

I agree, that's why I wrote "It can be represented by a simple LC circuit only".  Yes, with a circuit complicated enough we could probably simulate most electromagnetic wave phenomena, just like the few analog computers that still were being used at MIT Draper Labs in the early 1970's to solve differential equations.  I remember those  :).   Reconfiguring the analog computer to solve a different equation required actual handwork unlike just writing software for digital computers :-)

I guess I can't stop thinking about the 1/ f^3 in the thrust equation.

Dual 12at7's I think....


gives thrust per photon:

T = (h/(2*L*f))*(c/(2*pi))^2*X^2*((1/Rs^2)-(1/Rb^2))

If the number of photons is (P/hf)*(Q/2*pi) then:

NT = P*Q*(1/(4*pi*L*f^3))*(c/(2*pi))^2*X^2*((1/Rs^2)-(1/Rb^2))
Title: Re: EM Drive Developments - related to space flight applications - Thread 2
Post by: Rodal on 02/19/2015 09:00 pm
...

I guess I can't stop thinking about the 1/ f^3 in the thrust equation.

Dual 12at7's I think....


gives thrust per photon:

T = (h/(2*L*f))*(c/(2*pi))^2*X^2*((1/Rs^2)-(1/Rb^2))

If the number of photons is (P/hf)*(Q/2*pi) then:

NT = P*Q*(1/(4*pi*L*f^3))*(c/(2*pi))^2*X^2*((1/Rs^2)-(1/Rb^2))

If the thrust really decreases as the cube of the frequency...wouldn't you get much less thrust operating your experiment in the X band? ( 8.0 to 12.0 GHz  instead of 2 GHz implies 64 to 216 times less thrust, of course that's with everything else being the same which is not going to be...)
Title: Re: EM Drive Developments - related to space flight applications - Thread 2
Post by: Notsosureofit on 02/19/2015 09:09 pm
...

I guess I can't stop thinking about the 1/ f^3 in the thrust equation.

Dual 12at7's I think....


gives thrust per photon:

T = (h/(2*L*f))*(c/(2*pi))^2*X^2*((1/Rs^2)-(1/Rb^2))

If the number of photons is (P/hf)*(Q/2*pi) then:

NT = P*Q*(1/(4*pi*L*f^3))*(c/(2*pi))^2*X^2*((1/Rs^2)-(1/Rb^2))

If the thrust really decreases as the cube of the frequency...wouldn't you get much less thrust operating your experiment in the X band? ( 8.0 to 12.0 GHz  instead of 2 GHz implies 64 to 216 times less thrust, of course that's with everything else being the same which is not going to be...)

L goes down as well, but then the x-band sources are usually not as efficient either. How much more sensitive is the Cavendish setup?
Title: Re: EM Drive Developments - related to space flight applications - Thread 2
Post by: Rodal on 02/19/2015 09:14 pm
...

I guess I can't stop thinking about the 1/ f^3 in the thrust equation.

Dual 12at7's I think....


gives thrust per photon:

T = (h/(2*L*f))*(c/(2*pi))^2*X^2*((1/Rs^2)-(1/Rb^2))

If the number of photons is (P/hf)*(Q/2*pi) then:

NT = P*Q*(1/(4*pi*L*f^3))*(c/(2*pi))^2*X^2*((1/Rs^2)-(1/Rb^2))

If the thrust really decreases as the cube of the frequency...wouldn't you get much less thrust operating your experiment in the X band? ( 8.0 to 12.0 GHz  instead of 2 GHz implies 64 to 216 times less thrust, of course that's with everything else being the same which is not going to be...)

L goes down as well, but then the x-band sources are usually not as efficient either. How much more sensitive is the Cavendish setup?

0.174 uN according to Wikipedia:

<<Cavendish's equipment was remarkably sensitive for its time.[9] The force involved in twisting the torsion balance was very small, 1.74 x 10–7 N,[11] about 1/50,000,000 of the weight of the small balls[12] or roughly the weight of a large grain of sand.[13] To prevent air currents and temperature changes from interfering with the measurements, Cavendish placed the entire apparatus in a wooden box about 2 feet (0.61 m) thick, 10 feet (3.0 m) tall, and 10 feet (3.0 m) wide, all in a closed shed on his estate. Through two holes in the walls of the shed, Cavendish used telescopes to observe the movement of the torsion balance's horizontal rod. The motion of the rod was only about 0.16 inches (4.1 mm).[14] Cavendish was able to measure this small deflection to an accuracy of better than one hundredth of an inch using vernier scales on the ends of the rod>>
Title: Re: EM Drive Developments - related to space flight applications - Thread 2
Post by: Flyby on 02/19/2015 10:30 pm
Not really my expertise field.............
Welcome to the group. This is very much a multidisciplinary effort. What are you good at?
Well, mmm...  of formation, I'm an architect, but always had a strong interest in what happening on the nuclear science field and spaceflight development. But the last 25 years I've specialized in 3D visualizations, 3Dmodeling and 3Dprinting (8 years).
I fear that on the real hard science level, i might have little to nothing to add to the high standards here, but if there is anything I can do as far as visual communication goes, I might be of help or assistance, if needed...
I also have considerable experience modeling for and working with a powder 3Dprinter, but I suppose NASA has much more advanced systems to their disposal.
And  ofc, If any of the DIY builders here would require assistance in building 3Dmodels for 3dprinting i'll gladly assist.
Title: Re: EM Drive Developments - related to space flight applications - Thread 2
Post by: Rodal on 02/19/2015 10:46 pm
Not really my expertise field.............
Welcome to the group. This is very much a multidisciplinary effort. What are you good at?
Well, mmm...  of formation, I'm an architect, but always had a strong interest in what happening on the nuclear science field and spaceflight development. But the last 25 years I've specialized in 3D visualizations, 3Dmodeling and 3Dprinting (8 years).
I fear that on the real hard science level, i might have little to nothing to add to the high standards here, but if there is anything I can do as far as visual communication goes, I might be of help or assistance, if needed...
I also have considerable experience modeling for and working with a powder 3Dprinter, but I suppose NASA has much more advanced systems to their disposal.
And  ofc, If any of the DIY builders here would require assistance in building 3Dmodels for 3dprinting i'll gladly assist.
Great  :), we needed (and we still need) help with estimating dimensions of test articles from photographs that often have parallax problems and undimensioned drawings (technical authors often fail to detail the dimensions in their technical papers).   We are very fortunate that Paul March was kind enough to supply the dimensions of the NASA EM Drive truncated cone, but we will need help to estimate other devices from the UK, China and the USA.  Still @aero and @Mulletron did a fantastic job in estimating the NASA EM Drive dimensions just from the photographs.

Perhaps when we are ready to calculate expressions again for other devices, you can help us (at that time in the future).   :)

Thanks
Title: Re: EM Drive Developments - related to space flight applications - Thread 2
Post by: Flyby on 02/19/2015 10:48 pm

Shawyer has seemed convinced that this drive is able to replace commercial airliner engines etc. Obviously as he is the main proponent of the drive so this might be taken with a pinch of salt or two but if the drive is proven to be real he definitely seems to be ahead of the curve with the technology.

Shawyer has yet to prove that the performance of his "engine" scales linear with a dramatically increased Q.

Personally, I have my doubts his supercooled 2nd generation engine will scale to such a point that it creates 1ton of thrust from 1kW, as he claims...'sounds too good to be true...
But.. only time will tell, i suppose...


(http://emdrive.com/images/hybrid1.jpg)
Title: Re: EM Drive Developments - related to space flight applications - Thread 2
Post by: Rodal on 02/19/2015 11:14 pm
...
Shawyer has yet to prove that the performance of his "engine" scales linear with a dramatically increased Q.

..

Good point, on the other hand all theoretical formulas so far scale linearly with Q:

@Notsosureoit:  http://forum.nasaspaceflight.com/index.php?topic=36313.msg1332746#msg1332746

McCulloch MiHsC :  http://physicsfromtheedge.blogspot.com/2015/02/mihsc-vs-emdrive-data-3d.html

Shawyer:   http://www.emdrive.com/IAC-08-C4-4-7.pdf

Notice particularly the table in  McCulloch MiHsC :  http://physicsfromtheedge.blogspot.com/2015/02/mihsc-vs-emdrive-data-3d.html

There is a huge range of Q from Q= 5900 all the way to Cannae's Superconducting which has Q=11000000 
that is a range differing by a factor of almost 2000 from Min to Max


The big problem is delivering a high enough thrust/InputPower, for the reasons we have been discussing (starting with whether the results are an artifact, to thermal distortion, difficulty with keeping the EM Drive at the Q peak, etc etc).  We will not know until experiments demonstrate it.
Title: Re: EM Drive Developments - related to space flight applications - Thread 2
Post by: Notsosureofit on 02/19/2015 11:47 pm
...
Shawyer has yet to prove that the performance of his "engine" scales linear with a dramatically increased Q.

..

Good point, on the other hand all theoretical formulas so far scale linearly with Q:

@Notsosureoit:  http://forum.nasaspaceflight.com/index.php?topic=36313.msg1332746#msg1332746

....

I keep checking back and ..... there's another bloody typo !     N=P*Q/(2*pi*h*f^2)         [ f^2 NOT f ]

If I could type I'd be dangerous !

Going back and scaling term by term it looks like (at constant power) only Q and X, everything else is  a wash between distances and freq.
Title: Re: EM Drive Developments - related to space flight applications - Thread 2
Post by: Rodal on 02/20/2015 12:48 am
...
Shawyer has yet to prove that the performance of his "engine" scales linear with a dramatically increased Q.

..

Good point, on the other hand all theoretical formulas so far scale linearly with Q:

@Notsosureoit:  http://forum.nasaspaceflight.com/index.php?topic=36313.msg1332746#msg1332746

....

I keep checking back and ..... there's another bloody typo !     N=P*Q/(2*pi*h*f^2)         [ f^2 NOT f ]

If I could type I'd be dangerous !

Going back and scaling term by term it looks like (at constant power) only Q and X, everything else is  a wash between distances and freq.

That makes perfect sense because both Q and X are the only non-dimensional parameters in your equation.

Actually, now that you mention it we should non-dimensionalize all the equations (yours, Shawyer's and McCulloch's) in order to express them in term of non-dimensional parameters (Buckingham Pi theorem:   if there is a physically meaningful equation involving a certain number, n, of physical variables, and k is the rank of the dimensional matrix, then the original expression is equivalent to an equation involving a set of p = n − k  dimensionless parameters)
Title: Re: EM Drive Developments - related to space flight applications - Thread 2
Post by: Star-Drive on 02/20/2015 03:24 am
those mission parameters presupposed electrical power from solar panels? I guess that some good fission or fusion reactors (whenever fusion is available) would allow the addition of even more EM Drives and cut that mission time several times.

Aceshigh:

When it comes to manned solar electric propulsion missions, the largest to date proposed photovoltaic arrays for a human crewed missions are in the 300 kilowatt electric (kW-e) power range.  However most of the Q-Thruster solar system based mission analysis we did last year assumed using a U235 fission based reactor with a closed-cycle brayton or rankine thermal to electrical power conversion cycle system in the 1.0-to-2.0 Megawatt electric (MW-e) class output with at least a 10 year lifetime.  Now it's true that such a space based nuclear power reactor system has never been fielded, but that was due to a lack of a funded mission for same like sending humans to Mars, but more importantly, the lack of the political will to do so.  However building such a space-based closed cycle electrical power reactor is technologically feasible and has been since the 1980s. 

Now like all naval  propulsion system, once you've established the viability of a propulsion technique like the EM-Drive, and have an urgent need to do so, what comes next is developing the long lead items needed to power it.  And all these electric space propulsion concepts for deep-space human missions, be they conventional ion, Hall or VASMIR type plasma rocket thrusters, or Q-thruster like space drives, go begging for an already developed nuclear power plant that is sized from 1.0 MWe up to 100 MWe output dependent on the mission scenario.  I know that this sounds like a lot of power to the space community, but when it's compared to what the US Navy already builds for its nuclear submarine fleet like its "Boomer' Ohio class ICBM vehicles, which sport the 220 MW-thermal (MW-t) S8G reactor that fits in a 42 foot diameter by 55 feet long container and lasts for up to 30 years between refueling, its not a lot to ask technically if we just had the real need to do so.  And that will always be a political and business decision, not a technical one.

Best, Paul M.
Title: Re: EM Drive Developments - related to space flight applications - Thread 2
Post by: MichaelBlackbourn on 02/20/2015 03:58 am
This thread is awesome. I'm about to glue a bunch of pennies to a funnel, throw it in my microwave and grab a seat on top. High for two minutes for a quick trip to ISS?

Seriously though. I've been following this stuff for a while (and other related tech, MET, Polywell) and really would love to see a EMdrive ship powered by something like a pb11 polywell.

Will be following closely.
Title: Re: EM Drive Developments - related to space flight applications - Thread 2
Post by: Mulletron on 02/20/2015 04:04 am
We briefly discussed the potential for q-thrusters onboard subs. With that kind of power plant available, if q-thrusters can deliver, that could be huge. That should give the NRL pause. Would give a whole new meaning to silent service.
You see I'm trying to whet people's appetites to take a look at q-thrusters.

We've uncovered a plausible theory of operation of these things. Experimental results from multiple teams show performance way in excess of a photon rocket. Every research lab in the country should be taking notice. Never hurts to look.

It's boom or bust time for Emdrive.
Title: Re: EM Drive Developments - related to space flight applications - Thread 2
Post by: Hauerg on 02/20/2015 04:06 am
those mission parameters presupposed electrical power from solar panels? I guess that some good fission or fusion reactors (whenever fusion is available) would allow the addition of even more EM Drives and cut that mission time several times.

Aceshigh:

When it comes to manned solar electric propulsion missions, the largest to date proposed photovoltaic arrays for a human crewed missions are in the 300 kilowatt electric (kW-e) power range.  However most of the Q-Thruster solar system based mission analysis we did last year assumed using a U235 fission based reactor with a closed-cycle brayton or rankine thermal to electrical power conversion cycle system in the 1.0-to-2.0 Megawatt electric (MW-e) class output with at least a 10 year lifetime.  Now it's true that such a space based nuclear power reactor system has never been fielded, but that was due to a lack of a funded mission for same like sending humans to Mars, but more importantly, the lack of the political will to do so.  However building such a space-based closed cycle electrical power reactor is technologically feasible and has been since the 1980s. 

Now like all naval  propulsion system, once you've established the viability of a propulsion technique like the EM-Drive, and have an urgent need to do so, what comes next is developing the long lead items needed to power it.  And all these electric space propulsion concepts for deep-space human missions, be they conventional ion, Hall or VASMIR type plasma rocket thrusters, or Q-thruster like space drives, go begging for an already developed nuclear power plant that is sized from 1.0 MWe up to 100 MWe output dependent on the mission scenario.  I know that this sounds like a lot of power to the space community, but when it's compared to what the US Navy already builds for its nuclear submarine fleet like its "Boomer' Ohio class ICBM vehicles, which sport the 220 MW-thermal (MW-t) S8G reactor that fits in a 42 foot diameter by 55 feet long container and lasts for up to 30 years between refueling, its not a lot to ask technically if we just had the real need to do so.  And that will always be a political and business decision, not a technical one.

Best, Paul M.
S8G weight 2750 tons?
BiiiiiiiiiiiigFR needed.
Title: Re: EM Drive Developments - related to space flight applications - Thread 2
Post by: Mulletron on 02/20/2015 05:49 am
I guess not everyone realizes that a resonant cavity can be represented as an LC circuit. They're all the same thing.

(https://encrypted-tbn1.gstatic.com/images?q=tbn:ANd9GcSeF0z098aKsq-NFkxYS7IypVicisG8IM3AiidTeKKRbh8RFIT1B1y5Ju3J)
It can be represented by a simple LC circuit only for simple uniform cavities, with uniform cross-sections, as for example the rectangular cross section cavity or the cylindrical cavity. 

The truncated cone (frustum) shape used by NASA, Shawyer in the UK and Prof. Juan Yang in China displays degenerate modes that go from resonant to evanescent, and it displays modes that do not conform to the same TEmnp or TMmnp designation as in cylindrical cavities.  Actually in reviewing the mode shapes assigned in the COMSOL study for NASA I am now reviewing some interesting cases (the frequencies and images computed by COMSOL are excellent, but the designation of some of the modes is not straightforward, as the NASA engineer realized when designating some of the modes as "X").

There is some flexibility.  I remember working on delay lines w/ parameters varying w/ length.

That was odd stuff, I wonder if it can make a resonant circuit that way?

I agree, that's why I wrote "It can be represented by a simple LC circuit only".  Yes, with a circuit complicated enough we could probably simulate most electromagnetic wave phenomena, just like the few analog computers that still were being used at MIT Labs in the early 1970's to solve differential equations.  I remember those  :).   Reconfiguring the analog computer to solve a different equation required actual handwork unlike just writing software for digital computers :-)


The LC circuit approximation of a resonant cavity is to show the similar underlying mode of operation of all the previous iterations of "q-thrusters." Where Emdrive was more successful than the previous generations is the (unintentional) leveraging of cavity QED. The Emdrive utilizes the principle of confinement to great effect. This isn't to point out any particular failure. These are all valuable lessons learned. Evidence keeps piling up that quantum fluctuations under confinement are amplified if they are resonant or spatially confined. And at the same time the cavity serves to maximize the magnetic field localization and intensity, especially TE012.

http://goo.gl/lpI10F
http://goo.gl/gL4WJY  see 1.1.2
http://www.lancaster.ac.uk/users/esqn/windsor10/lectures/Girvin2.pdf
https://www.physik.hu-berlin.de/nano/lehre/copy_of_quantenoptik09/Chapter12 @605
http://www.pnas.org/content/111/29/10485.full
Title: Re: EM Drive Developments - related to space flight applications - Thread 2
Post by: Giovanni DS on 02/20/2015 08:25 am
I wonder if the cavity itself could be used as part of the oscillator using a Gunn diode: http://en.wikipedia.org/wiki/Gunn_diode

The resulting device would be incredibly compact and self contained, tuning would be inherently a non-issue.
Title: Re: EM Drive Developments - related to space flight applications - Thread 2
Post by: frobnicat on 02/20/2015 09:48 am
those mission parameters presupposed electrical power from solar panels? I guess that some good fission or fusion reactors (whenever fusion is available) would allow the addition of even more EM Drives and cut that mission time several times.

Aceshigh:

When it comes to manned solar electric propulsion missions, the largest to date proposed photovoltaic arrays for a human crewed missions are in the 300 kilowatt electric (kW-e) power range.  However most of the Q-Thruster solar system based mission analysis we did last year assumed using a U235 fission based reactor with a closed-cycle brayton or rankine thermal to electrical power conversion cycle system in the 1.0-to-2.0 Megawatt electric (MW-e) class output with at least a 10 year lifetime.  Now it's true that such a space based nuclear power reactor system has never been fielded, but that was due to a lack of a funded mission for same like sending humans to Mars, but more importantly, the lack of the political will to do so.  However building such a space-based closed cycle electrical power reactor is technologically feasible and has been since the 1980s. 

Now like all naval  propulsion system, once you've established the viability of a propulsion technique like the EM-Drive, and have an urgent need to do so, what comes next is developing the long lead items needed to power it.  And all these electric space propulsion concepts for deep-space human missions, be they conventional ion, Hall or VASMIR type plasma rocket thrusters, or Q-thruster like space drives, go begging for an already developed nuclear power plant that is sized from 1.0 MWe up to 100 MWe output dependent on the mission scenario.  I know that this sounds like a lot of power to the space community, but when it's compared to what the US Navy already builds for its nuclear submarine fleet like its "Boomer' Ohio class ICBM vehicles, which sport the 220 MW-thermal (MW-t) S8G reactor that fits in a 42 foot diameter by 55 feet long container and lasts for up to 30 years between refueling, its not a lot to ask technically if we just had the real need to do so.  And that will always be a political and business decision, not a technical one.

Best, Paul M.

But unlike all the conventional electric space propulsion concepts(1) propellantless schemes have a potential for practical unlimited power generation as soon as around tpr = 1N/kWe is proven, by mounting driver(s) on a rotor linked to a generator of efficiency n and having a tangential speed above 1/(n * tpr). See attached picture.

The only caveat I see would be if wakes of successive passages at same place would interfere and lower the effective thrust power ratio below over-unit cycle. Even in this case I don't see how an over-unit power generating system would still be impossible : use a larger rotor (tethered... many km apart if needed) to give more time for hypothetical wakes to dissipate, use a linear scheme...

So, assuming better than 1N/kW can be reached, why bother with conventional energy generators ? Because we would want to convince ourselves that this is not breaking energy conservation and avoid the "free energy" tag ? But it is not "worse" to posit apparent cheap energy than to posit apparent cheap momentum, and from an engineering point of view it makes no sense to use the later and refusing to see the possibility to use the former.

Sorry, this is a repost, for details : http://forum.nasaspaceflight.com/index.php?topic=29276.msg1284676#msg1284676 (http://forum.nasaspaceflight.com/index.php?topic=29276.msg1284676#msg1284676)

(1) except electrodynamic tether used as generator, but at the cost of absorbed deltaV : that is a 0 sum game...
Title: Re: EM Drive Developments - related to space flight applications - Thread 2
Post by: Mulletron on 02/20/2015 11:09 am
I think this explains how Emdrive can never go over unity.

http://usersguidetotheuniverse.com/?p=2865
Title: Re: EM Drive Developments - related to space flight applications - Thread 2
Post by: Notsosureofit on 02/20/2015 11:19 am
I wonder if the cavity itself could be used as part of the oscillator using a Gunn diode: http://en.wikipedia.org/wiki/Gunn_diode

The resulting device would be incredibly compact and self contained, tuning would be inherently a non-issue.

That's exactly what I have in mind.
Title: Re: EM Drive Developments - related to space flight applications - Thread 2
Post by: Chrochne on 02/20/2015 11:26 am
Hello everyone,

First of all. I am just one of many enthusiasts of EmDrive (from central Europe). I think I read all I can about it and follow it for quite a while.

I am wondering about few things. Mr. Paul March, if you can answer, that would be great as well. I see that you said you lack the funds (resources) to continue important tests on the EmDrive. This is definitely a project that cought attention of many in the world (and deserves much more media attention as it can be really revolutionary). Many projects that need to be financed heavily go for support at Kickstarter and many other similar websites. Perhaps it can help you? Also it can bring more media attention. I know that in the past NASA did few of those steps that help them to bring attention to them one way or the other to succeed.

There is also another thing I am wondering about and I would be glad if you guys can help me answer it.
NASA do not mention too much Mr. Roger Shawyer. Is it perhaps to try to stay as much independent as possible in their research? I hope that some time in the future all the scientits and engineers that work on this kind of technology (from Mr. Shawyer, Mr. Fetta, NASA team and chinese team as well) will make some nice conference on this. It can bring you a lot of public attention (and not enough space for proper reasearch I guess :) ).

Have a nice weekend,

With kind regards,

Standa V.
Title: Re: EM Drive Developments - related to space flight applications - Thread 2
Post by: Rodal on 02/20/2015 12:04 pm

...

But unlike all the conventional electric space propulsion concepts(1) propellantless schemes have a potential for practical unlimited power generation as soon as around tpr = 1N/kWe is proven, by mounting driver(s) on a rotor linked to a generator of efficiency n and having a tangential speed above 1/(n * tpr). See attached picture.

The only caveat I see would be if wakes of successive passages at same place would interfere and lower the effective thrust power ratio below over-unit cycle. Even in this case I don't see how an over-unit power generating system would still be impossible : use a larger rotor (tethered... many km apart if needed) to give more time for hypothetical wakes to dissipate, use a linear scheme...

So, assuming better than 1N/kW can be reached, why bother with conventional energy generators ? Because we would want to convince ourselves that this is not breaking energy conservation and avoid the "free energy" tag ? But it is not "worse" to posit apparent cheap energy than to posit apparent cheap momentum, and from an engineering point of view it makes no sense to use the later and refusing to see the possibility to use the former.

Sorry, this is a repost, for details : http://forum.nasaspaceflight.com/index.php?topic=29276.msg1284676#msg1284676 (http://forum.nasaspaceflight.com/index.php?topic=29276.msg1284676#msg1284676)

(1) except electrodynamic tether used as generator, but at the cost of absorbed deltaV : that is a 0 sum game...

Similarly, Maxwell himself conceived a demon (as a thought experiment) to demonstrate how to violate the Second Law of Thermodynamics: a container of gas molecules at equilibrium is divided into two parts by an insulated wall, with a door that can be opened and closed by what came to be known as "Maxwell's demon". The demon opens the door to allow only the faster than average molecules to flow through to a favored side of the chamber, and only the slower than average molecules to the other side, causing the favored side to gradually heat up while the other side cools down, thus decreasing entropy.

One of the famous responses was by Leon Brillouin that shows that the second law of thermodynamics will not actually be violated, if a more complete analysis is made of the whole system including the demon. Any demon must "generate" more entropy segregating the molecules than it could ever eliminate by the method described by Maxwell.

In the case of the EM Drive, to answer the "overunity" question one has to perform a complete analysis of the whole system.   This cannot be done in isolation, without a theory of operation for the EM Drive.  It would be like arguing that a windmill or a sailing boat cannot move (in an analysis ignoring the wind) because it would be a source of practical unlimited power.  Same thing with extracting energy from the ocean.  In the case of the EM Drive, due consideration of the total system is required: whether coupling with the Quantum Vacuum, or coupling with any other external field. 

At this early stage, no theory proposed has quantitatively and scientifically answered how momentum is conserved, or has properly addressed the entire system to answer whether the 2nd law of thermodynamics is respected.

(http://image.wikifoundry.com/image/1/bla-PnfxsfKa5fKdSCAePA74003/GW250H275)
(http://www.pynchon.pomona.edu/entropy/demon.gif)
(http://krisdedecker.typepad.com/.a/6a00e0099229e888330120a5c4802d970b-320wi)
Title: Re: EM Drive Developments - related to space flight applications - Thread 2
Post by: frobnicat on 02/20/2015 12:46 pm
I think this explains how Emdrive can never go over unity.

http://usersguidetotheuniverse.com/?p=2865

So energy in a box (whatever forms) has inertial and gravitational mass, all right. But I fail to see how it explains how em drive cant go over unity. I don't see why it shouldn't go apparently energy over unity in the first place, it is not a more inconvenient problem than it goes apparently non 0 net momentum.

If you insist on this apparent net energy generation being impossible, then please explain how a emdrive used as a linear thruster fed by conventional energy source can impart a (hypothetically) velocity invariant fixed thrust/power ratio to a spaceship, and the same thruster mounted tangentially on a flywheel wouldn't show the same thrust/power ratio ? Where and why exactly the depicted "unlimited energy generator" would fail ?

If the energy generation scheme don't work then reciprocally it would imply that there is some dependence of the thrust/power ratio on some preferred rest frame and/or total integrated "use" of the effect, and that would seriously impede mission profiles (of emdrive fed by conventional mean) : all published mission profiles so far have worked under the assumption of constant thrust/power ratio even beyond the point of acquiring more kinetic energy than spent electric power. And that is precisely what makes them much better than conventional high Isp action/reaction, like Vasimr, most on overunity energy gain, not a lot on saving propellant mass. In the very same papers, the attempts at considering limitations to prevent such apparent over-unity were not applied in the mission profiles chapters.

So my point is, this should be consistent :
- Either thrust/power is invariant, the mission profiles are OK and apparent overunity generator is also possible to power them up.
- Thrust/power is not invariant, apparent overunity impossibility of energy generation might be enforced, but the mission profiles are to be seriously shaved down.

In any case, a system wide analysis would show both momentum and energy conservation (taking into account QV state before and after operation, for instance), the net momentum and energy gain being only apparent. We shouldn't shy away from apparent energy overunity gain, this is not more "free energy" than it is "free momentum".
Title: Re: EM Drive Developments - related to space flight applications - Thread 2
Post by: frobnicat on 02/20/2015 12:57 pm
In the case of the EM Drive, to answer the "overunity" question one has to perform a complete analysis of the whole system.   This cannot be done in isolation, without a theory of operation for the EM Drive.  It would be like arguing that a windmill or a sailing boat cannot move (in an analysis ignoring the wind) because it would be a source of practical unlimited power.  Same thing with extracting energy from the ocean.  In the case of the EM Drive, due consideration of the total system is required: whether coupling with the Quantum Vacuum, or coupling with any other external field. 


Exactly, so we can't rule out the possibility of apparent energy overunity before (the reality of the effect is proven beyond any doubt and) there is a working theory than can encompass the "total system". But the point is, even before we have such working theory that would show how both energy and momentum conservation are guaranteed, we have an experimental/phenomenological prediction of a given thrust/power in a (Lorentz invariant) vacuum, if this is to hold for propulsion (and giving a ship more kinetic energy than spent electric power) then this is to hold for energy generation, regardless of the deeper theories.

I don't mean to sound insistent, but I feel a lot of resistance about that important empirical fact, more for sociological reasons than real scientific logic.

Title: Re: EM Drive Developments - related to space flight applications - Thread 2
Post by: Rodal on 02/20/2015 12:59 pm
In the case of the EM Drive, to answer the "overunity" question one has to perform a complete analysis of the whole system.   This cannot be done in isolation, without a theory of operation for the EM Drive.  It would be like arguing that a windmill or a sailing boat cannot move (in an analysis ignoring the wind) because it would be a source of practical unlimited power.  Same thing with extracting energy from the ocean.  In the case of the EM Drive, due consideration of the total system is required: whether coupling with the Quantum Vacuum, or coupling with any other external field. 


Exactly, so we can't rule out the possibility of apparent energy overunity before (the reality of the effect is proven beyond any doubt and) there is a working theory than can encompass the "total system". But the point is, even before we have such working theory that would show how both energy and momentum conservation are guaranteed, we have an experimental/phenomenological prediction of a given thrust/power in a (Lorentz invariant) vacuum, if this is to hold for propulsion (and giving a ship more kinetic energy than spent electric power) then this is to hold for energy generation, regardless of the deeper theories.

I don't mean to sound insistent, but I feel a lot of resistance about that important empirical fact, more for psychological reasons than real scientific logic.

At this early stage, no published theory has precisely, mathematically modeled how momentum is conserved, or has mathematically modeled the entire system to answer how the 2nd law of thermodynamics is respected in the EM Drive.

Physics (since Newton) is an applied mathematical as well as an experimental science.  Until these questions have been answered mathematically, solely and precisely , they are still a subject of conjecture and hypothesis.


Quote from: Von Neumann
The sciences do not try to explain, they hardly even try to interpret, they mainly make models. By a model is meant a mathematical construct which, with the addition of certain verbal interpretations, describes observed phenomena. The justification of such a mathematical construct is solely and precisely that it is expected to work.
"Method in the Physical Sciences", in The Unity of Knowledge (1955), ed. L. G. Leary (Doubleday & Co., New York), p. 157

No mathematical model has been published yet that "solely and precisely" mathematically models the observed phenomena of the EM Drive.  To make an imprecise analogy, before Heissenberg, Schrodinger and Von Neumann published mathematical solutions, and the theoretical framework of Quantum Mechanics, big questions about the then mysterious Quantum Mechanics could not be answered cathegorically either.

(http://upload.wikimedia.org/wikipedia/commons/d/d6/JohnvonNeumann-LosAlamos.jpg)
Title: Re: EM Drive Developments - related to space flight applications - Thread 2
Post by: Star-Drive on 02/20/2015 01:39 pm
those mission parameters presupposed electrical power from solar panels? I guess that some good fission or fusion reactors (whenever fusion is available) would allow the addition of even more EM Drives and cut that mission time several times.

Aceshigh:

When it comes to manned solar electric propulsion missions, the largest to date proposed photovoltaic arrays for a human crewed missions are in the 300 kilowatt electric (kW-e) power range.  However most of the Q-Thruster solar system based mission analysis we did last year assumed using a U235 fission based reactor with a closed-cycle brayton or rankine thermal to electrical power conversion cycle system in the 1.0-to-2.0 Megawatt electric (MW-e) class output with at least a 10 year lifetime.  Now it's true that such a space based nuclear power reactor system has never been fielded, but that was due to a lack of a funded mission for same like sending humans to Mars, but more importantly, the lack of the political will to do so.  However building such a space-based closed cycle electrical power reactor is technologically feasible and has been since the 1980s. 

Now like all naval  propulsion system, once you've established the viability of a propulsion technique like the EM-Drive, and have an urgent need to do so, what comes next is developing the long lead items needed to power it.  And all these electric space propulsion concepts for deep-space human missions, be they conventional ion, Hall or VASMIR type plasma rocket thrusters, or Q-thruster like space drives, go begging for an already developed nuclear power plant that is sized from 1.0 MWe up to 100 MWe output dependent on the mission scenario.  I know that this sounds like a lot of power to the space community, but when it's compared to what the US Navy already builds for its nuclear submarine fleet like its "Boomer' Ohio class ICBM vehicles, which sport the 220 MW-thermal (MW-t) S8G reactor that fits in a 42 foot diameter by 55 feet long container and lasts for up to 30 years between refueling, its not a lot to ask technically if we just had the real need to do so.  And that will always be a political and business decision, not a technical one.

Best, Paul M.
S8G weight 2750 tons?
BiiiiiiiiiiiigFR needed.

We used 10.0 kg/kW-e in our Q-Thruster mission analysis for our nuclear power plant specific mass, see chart.  This figure could be reduced further to around 5.0 kg/kW-e or even lower with sufficient R&D funding, but still based on a fission based uranium/nitride (UN) fuel and a CO2 supercritical power conversion cycle.

Best, Paul M.
Title: Re: EM Drive Developments - related to space flight applications - Thread 2
Post by: JasonAW3 on 02/20/2015 01:54 pm
Ok,

     For those of us who have become completely lost in the math, can you, in layman's terms, explain what everyone seems to SUSPECT is going on when it comes to converting electrical current into motive force?

     I've tried to follow the math, but was lost a while back, and the best I can understand is that somehow it involves Vaccume Energy and possibley Dark Matter
Title: Re: EM Drive Developments - related to space flight applications - Thread 2
Post by: lele on 02/20/2015 02:34 pm
Assuming that the EM Drive effectively work*, it means you are somehow "stealing" energy from somewhere, and that you can create something which looks like perpetual motion. Let's say that the "somewhere" is the quantum void (QV). Since it can't have an infinite energy, it must mean that you can somehow deplete QV energy.

I'm not a specialist, but I think the question "What is the value of the vacuum energy of free space?" becomes relevant here. Additionally it raises disturbing (for me at least) questions, like "What happens when QV energy is depleted?".

Edit: *what I mean here by "work" is "is a system which, given electrical power, produce a force, regardless of its medium, non-relativistic speed, orientation etc..."
Title: Re: EM Drive Developments - related to space flight applications - Thread 2
Post by: RotoSequence on 02/20/2015 03:10 pm
Ok,

     For those of us who have become completely lost in the math, can you, in layman's terms, explain what everyone seems to SUSPECT is going on when it comes to converting electrical current into motive force?

     I've tried to follow the math, but was lost a while back, and the best I can understand is that somehow it involves Vaccume Energy and possibley Dark Matter

Much of the work in this thread has not been focused on theories of operation. Most of the recent math has been focused on attempts to amplify the observed effect, for the sake of demonstrating that it is a real effect, rather than experimental artifact.

There is not a consensus on the cause of the apparent thrust, be it real or imaginary. Suggested causes of real thrust include interactions with dark matter, harnessing linear momentum from the quantum vacuum, evanescent waves creating inertial moments, tapping into photons being the source of momentum in the universe in general... there's a lot of ideas out there. What all of these theories are short on is experimental data. Because there is no money (Eagleworks is not allowed to accept crowd funding), right now is the time to think.  :P
Title: Re: EM Drive Developments - related to space flight applications - Thread 2
Post by: Rodal on 02/20/2015 03:40 pm
Ok,

     For those of us who have become completely lost in the math, can you, in layman's terms, explain what everyone seems to SUSPECT is going on when it comes to converting electrical current into motive force?

     I've tried to follow the math, but was lost a while back, and the best I can understand is that somehow it involves Vaccume Energy and possibley Dark Matter

Much of the work in this thread has not been focused on theories of operation. Most of the recent math has been focused on attempts to amplify the observed effect, for the sake of demonstrating that it is a real effect, rather than experimental artifact.

There is not a consensus on the cause of the apparent thrust, be it real or imaginary. Suggestions for causes of real thrust include interactions with dark matter, harnessing linear momentum from the quantum vacuum, evanescent waves creating inertial moments, tapping into photons being the source of momentum in the universe in general... there's a lot of ideas out there. What all of these theories are short on is experimental data. Because there is no money (Eagleworks is not allowed to accept crowd funding), right now is the time to think.  :P

As an analogy, what does "everyone seems to suspect" is Dark Matter?  what does "everyone seems to suspect" is Dark Energy?  what does "everyone seems to suspect" is the Unified Theory that could unify Quantum Mechanics and Relativity? what does everyone think is Time? what does everyone think is the ultimate nature of the Universe: continuous or discrete ? and so on...

There is no consensus in the scientific community at large either on these questions, and many more, to which much, much more money and brains have been dedicated to.  The answers to the above questions, from respected scientists, cover a large number of hypotheses (some of the theories on "Dark Matter" are that it is not really matter, in our spatial universe).  Similarly here in this thread no consensus can be expected.  What can be expected is to eliminate theoretical explanations, and to examine theory, numerical models and experiments in detail, with scientific thoroughness.  Some possibilities for how an EM Drive could work for space propulsion have been strongly shot down in this thread.  One of the possibilities that has been studied in some detail, mostly in Thread 1 and some in thread 2 is that the possibility that Dark Matter is responsible for EM Drive thrust measurements is very, very remote, see for example the posts by Astrophysicist Marshall Eubanks:

http://forum.nasaspaceflight.com/index.php?topic=36313.msg1331938#msg1331938

http://forum.nasaspaceflight.com/index.php?topic=36313.msg1331973#msg1331973

(https://str.llnl.gov/str/Sep07/gifs/GAPS_fig1.jpg)
Title: Re: EM Drive Developments - related to space flight applications - Thread 2
Post by: Mulletron on 02/20/2015 04:13 pm
The key take away from the light in a box page was the red shift/blue shift. The same thing is presented by Shawyer. Also the theory paper presented (see Colbert) explains precisely how momentum is conserved. There is no free/cheap momentum.

Asserting there are conservation issues /while at the same time not understanding the interaction doesn't make sense. That's like passing legislation governing the use of time travel and warp drive before they become real. Just speculation.

You just have to read the paper.
Title: Re: EM Drive Developments - related to space flight applications - Thread 2
Post by: tchernik on 02/20/2015 04:18 pm
Assuming that the EM Drive effectively work, it means you are somehow "stealing" energy from somewhere, and that you can create something which looks like perpetual motion. Let's say that the "somewhere" is the quantum void (QV). Since it can't have an infinite energy, it must mean that you can somehow deplete QV energy.

I'm not a specialist, but I think the question "What is the value of the vacuum energy of free space?" becomes relevant here. Additionally it raises disturbing (for me at least) questions, like "What happens when QV energy is depleted?".

I don't think you are looking at it correctly.  Are you stealing from the earth's gravity when you do a push-up?  Of course not.

Gaining potential energy by acting against gravity is well known to respect conservation of energy.

The Emdrive, if it works as a legit interaction with the QV as H. White and some posters here believe, would act more like a windmill or a sail ship. You could potentially be taking energy from the quantum vacuum, but for you it would indeed be "free energy". In the same sense as a windmill provides you with "free energy" from an external source (the wind).

With the difference the external source for the Emdrive would be the QV, which is everywhere.

I have to note that besides the actual existence of this phenomenon, it is not clear either if the momentum gain is limited or not or if it is restricted with respect some background field/reference frame. I'm assuming it's not, that is, the amount of acceleration is the same regardless of the inertial frame of reference.
Title: Re: EM Drive Developments - related to space flight applications - Thread 2
Post by: Flyby on 02/20/2015 04:56 pm

Quote from: Von Neumann
The sciences do not try to explain, they hardly even try to interpret, they mainly make models. By a model is meant a mathematical construct which, with the addition of certain verbal interpretations, describes observed phenomena. The justification of such a mathematical construct is solely and precisely that it is expected to work.
"Method in the Physical Sciences", in The Unity of Knowledge (1955), ed. L. G. Leary (Doubleday & Co., New York), p. 157

Funny... that's exactly the main idea I "learned" from reading Stephin Hawking's "Grand design" : the fact that we as humans are trying to come up with ever new models or concepts of how the world supposedly functions or looks like.  I'd pull it even larger and include religious concepts together with science models as being examples of our irresistible urge to explain things with models. I'd call them "mental constructs" and are inventions of our imagination.
 
Some those models correlated better with our observations then others, but who am I to say what's better then others? Today's concept(s) might be outdated or corrected in a few years...
Look at the concept of the "aether". When I was getting my science classes, the old greek concept by Aristotle was met with laughter... now 3 to 4 decades later, it kinda resurfaces as being a very valid concept to explain dark matter...go figure... :)

I realize that ancient philosophy has little to offer to the current science development, but science most definitely should have more impact on contemporary philosophy, on what impact current developments in science can have on our view of the "world" (or universe, in this case)

about the "free energy concept" : it is not because you, as 21th century human, do not see where the energy originates from that it should be "energy from nothing".
Transport yourself 100 years back and then explain to any people  in the street how you can gain 17kiloton TNT equivalent energy out of 1kg of uranium...where does that energy comes from? today we know, but back then we didn't...
All we know today is that after careful calculations scientists discovered that only 4.9% of the universe is matter like we know it...
Good question then is... what with all the rest? what's that? can or do we interact with it? does it hold energy?  Lots of questions needing lots of answers...
Title: Re: EM Drive Developments - related to space flight applications - Thread 2
Post by: frobnicat on 02/20/2015 09:13 pm
The key take away from the light in a box page was the red shift/blue shift. The same thing is presented by Shawyer. Also the theory paper presented (see Colbert) explains precisely how momentum is conserved. There is no free/cheap momentum.

Asserting there are conservation issues /while at the same time not understanding the interaction doesn't make sense. That's like passing legislation governing the use of time travel and warp drive before they become real. Just speculation.

You just have to read the paper.

What paper ? Colbert ? Where ?
Not trying to be obtuse here but you'll have to connect the dots for me (and for some other readers I guess)... still not understanding how this is supposed to make an energy generating scheme to fail (under the assumption that an emdrive could give 1N of thrust when fed 1kW microwave power for instance).

Some doppler shift is supposed to modifie the thrust/power ratio when emdrive is accelerated ? Does it pertain to accelerations vectors orthogonal to the axis of the frustum (as would be the case for a frustum mounted on a flywheel) ? Does it pertain to arbitrarily low accelerations (as the proposed device would work at, say, fixed tangential velocity V=2 km/s, acc=V²/R so by extending the radius, radial acceleration can be made arbitrarily low) ? Does it pertain to an emdrive that is not accelerating, that is on straight trajectory, at constant velocity and thrusting at constant force that is pulling a tether that is rotating an electric generator ?

I'm not saying there is conservation issue, I'm concerned about consistency of a phenomenological model that's implied when mission profiles are proposed, this model is very simple : at a given microwave power there is a given thrust. Thrust=f(Power). Not Thrust=f(Power, Velocity wrt?, Acceleration, ...?) Granted, this model is speculative, but within this speculation it is not a speculation to state that the same effect used to accelerate a spacecraft can be used in a system that generates energy. It is not logical to operate under this assumption to devise mission profiles and not to devise unlimited energy generators.

From your previous remark :
I think this explains how Emdrive can never go over unity.

http://usersguidetotheuniverse.com/?p=2865

So you seem to think that "Emdrive can never go over unity", isn't it a bit speculative at this stage ? Regardless of why, that would imply that you consider that the Emdrive can't be used to accelerate a spacecraft at constant thrust/power ratio. What would be the proposed Thrust=f(Power, Velocity wrt?, Acceleration, ...?)

To be clear, I'm not trying to make a case that emdrive makes apparent energy conservation breaking and that therefore it must be bogus. If the effect is bogus or not is decided by experience, not by armchair physicists. But armchair physicist is legitimate to foretell that if effect is not bogus for space flight application (thrusting for deltaV) then it is not bogus for energy generation : the two situations can be made undistinguishable from the point of view of the device. This is a package. Take both or leave both.  Otherwise makes the claims appear not serious. Just saying.

For the later (energy generation) limitation on material makes the idea practical around 1N/kW above 1km/s speed (wrt local energy harvesting frame). So maybe the effect proves possible but not at such levels and that "prevents" the practical energy generation potential of the package. But even 50µN/50W is already enough to apparently break energy conservation, if not practically, at least in some physically possible setup.
Title: Re: EM Drive Developments - related to space flight applications - Thread 2
Post by: birchoff on 02/20/2015 09:59 pm
The key take away from the light in a box page was the red shift/blue shift. The same thing is presented by Shawyer. Also the theory paper presented (see Colbert) explains precisely how momentum is conserved. There is no free/cheap momentum.

Asserting there are conservation issues /while at the same time not understanding the interaction doesn't make sense. That's like passing legislation governing the use of time travel and warp drive before they become real. Just speculation.

You just have to read the paper.

What paper ? Colbert ? Where ?
Not trying to be obtuse here but you'll have to connect the dots for me (and for some other readers I guess)... still not understanding how this is supposed to make an energy generating scheme to fail (under the assumption that an emdrive could give 1N of thrust when fed 1kW microwave power for instance).

Some doppler shift is supposed to modifie the thrust/power ratio when emdrive is accelerated ? Does it pertain to accelerations vectors orthogonal to the axis of the frustum (as would be the case for a frustum mounted on a flywheel) ? Does it pertain to arbitrarily low accelerations (as the proposed device would work at, say, fixed tangential velocity V=2 km/s, acc=V²/R so by extending the radius, radial acceleration can be made arbitrarily low) ? Does it pertain to an emdrive that is not accelerating, that is on straight trajectory, at constant velocity and thrusting at constant force that is pulling a tether that is rotating an electric generator ?

I'm not saying there is conservation issue, I'm concerned about consistency of a phenomenological model that's implied when mission profiles are proposed, this model is very simple : at a given microwave power there is a given thrust. Thrust=f(Power). Not Thrust=f(Power, Velocity wrt?, Acceleration, ...?) Granted, this model is speculative, but within this speculation it is not a speculation to state that the same effect used to accelerate a spacecraft can be used in a system that generates energy. It is not logical to operate under this assumption to devise mission profiles and not to devise unlimited energy generators.

From your previous remark :
I think this explains how Emdrive can never go over unity.

http://usersguidetotheuniverse.com/?p=2865

So you seem to think that "Emdrive can never go over unity", isn't it a bit speculative at this stage ? Regardless of why, that would imply that you consider that the Emdrive can't be used to accelerate a spacecraft at constant thrust/power ratio. What would be the proposed Thrust=f(Power, Velocity wrt?, Acceleration, ...?)

To be clear, I'm not trying to make a case that emdrive makes apparent energy conservation breaking and that therefore it must be bogus. If the effect is bogus or not is decided by experience, not by armchair physicists. But armchair physicist is legitimate to foretell that if effect is not bogus for space flight application (thrusting for deltaV) then it is not bogus for energy generation : the two situations can be made undistinguishable from the point of view of the device. This is a package. Take both or leave both.  Otherwise makes the claims appear not serious. Just saying.

For the later (energy generation) limitation on material makes the idea practical around 1N/kW above 1km/s speed (wrt local energy harvesting frame). So maybe the effect proves possible but not at such levels and that "prevents" the practical energy generation potential of the package. But even 50µN/50W is already enough to apparently break energy conservation, if not practically, at least in some physically possible setup.

What is the end goal of this line of reasoning?

Does the model presented with the mission profile, potentially lead to a situation where the EmDrive could be used as an over Unity power generator? Maybe. But lets for arguments sake agree that it does. What use is this information? I ask this question because if anything is clear from the discussions in this thread and the reported results to date. We only have a few rough guestimates at what is going on within the EmDrive. To prove or disprove the over unity power generation assertion you would need to completely characterize everything that is going on in the system. Right now the most we know is that when we pump EM into the cavity we get some thrust. We don't know beyond a reasonable doubt where the thrust is actually coming from. We don't know what we are actually interacting with. Now there are probably a whole lot of potential consequences of this research if it holds up to experimental scrutiny. But right now, the one thing I could probably guarantee, is that there will be a subset of those potential consequences that will not be possible once we have a better understanding of what is ACTUALLY happening.

On the flip side, should the researchers be allowed to propose mission profiles that could potentially not pan out due to a fundamental lack of understanding of what is actually going on? Maybe. Though from my perspective I think I am willing to give the primary investigators some leeway in painting a picture of what could be possible. As long as they assume the responsibility of dealing with the fall out if their vision is never realized.
Title: Re: EM Drive Developments - related to space flight applications - Thread 2
Post by: lele on 02/20/2015 11:27 pm
I think that frobnicat is simply saying that
IF there is a system (any system) that generate a given thrust when it's fed a given electrical power* (with no other parameters affecting the thrust) THEN it can be used to generate power, at least theoretically.

This implication doesn't necessitate knowledge of quantum physics, it's simply basic physics.

The interesting question is then "Is EMdrive a system that generate a given thrust when it's fed a given electrical power?"
If it is, we can generate power with EMdrive. It is an useful information: it means that, since free energy doesn't exist, we take this energy from somewhere.
If it isn't, knowing which parameters affect the thrust will probably give us some insight into how EMdrive generate thrust.

(as usual, if we assume that the thrust is not an artefact)

Edit: precision, see frobnicat's post below
* and the thrust/power ratio is better than that of a phton rocket (3.33 µN/kW)
Title: Re: EM Drive Developments - related to space flight applications - Thread 2
Post by: Flyby on 02/20/2015 11:35 pm
To get away from this overunity discussion , which is - in the current situation - a mere distraction with an apparently high polarized discussion risk... :)
 
the following idea popped up in my mind: what if instead of a truncated cone, other shapes are used?

What would be the implications of fe, using a paraboloid (or a hyperboloid) ?
http://mathworld.wolfram.com/Paraboloid.html
A paraboloid has a natural focal point, which probably would create an ever higher H-field intensity, as far as the resonance patterns go?
(http://www.pbs.org/wgbh/nova/teachers/activities/images/3406_solar_focal1.gif)

I also would like to understand to why the curved bottoms of Shawyer's new test rig "makes more sense" , from microwave resonance point of view?


As for the suggestion to analyze the available pictures of the shawyer and chinese EMdrives...
challenge accepted, i'll have a closer look at them. ? 8)

Finding pictures of shawyer's test device(s) is not that hard, but can any1 actually confirm that this is the Chinese truncated cone? or is it completely unrelated?

(http://tommytoy.typepad.com/.a/6a0133f3a4072c970b01a511f54ee6970c-800wi)
Title: Re: EM Drive Developments - related to space flight applications - Thread 2
Post by: Mulletron on 02/20/2015 11:37 pm
Last post converted to PM.
Here's all I have to say about this.

Quote
“The law that entropy always increases, holds, I think, the supreme position among the laws of Nature. If someone points out to you that your pet theory of the universe is in disagreement with Maxwell's equations—then so much the worse for Maxwell's equations. If it is found to be contradicted by observation—well, these experimentalists do bungle things sometimes. But if your theory is found to be against the second law of thermodynamics, I can give you no hope; there is nothing for it but to collapse in deepest humiliation.” Sir Arthur Eddington (The Nature of the Physical World, 1915)
http://web.mit.edu/keenansymposium/overview/background/
Title: Re: EM Drive Developments - related to space flight applications - Thread 2
Post by: frobnicat on 02/20/2015 11:41 pm
What is the end goal of this line of reasoning?

This stems from the (maybe false) impression that enthusiasts consider that cheap momentum (thrusting for deltaV) is good while cheap energy is wrong (or that matter should be kept silent or mysterious). Such bias would introduces inconsistencies in the way to think about both theoretical aspects and engineering (space flight) prospects, and also degrade the perception from the general scientific public. If the impression of bias is false, and/or my ramblings aren't helping to reduce inconsistencies of thought or improving the sceptical public perception, then I apologize.

Quote
Does the model presented with the mission profile, potentially lead to a situation where the EmDrive could be used as an over Unity power generator? Maybe.

Certainly. How could it be otherwise ? Why let a doubt fly over that immediate consequence of the model ? The attempts to hide or confuse that fact in the published papers can only lower the academic credits of this line of research.

Quote
But lets for arguments sake agree that it does. What use is this information?

Consistency : momentum and energy conservation are to be taken together to build well behaved theories of operation that would help devise clear experiments for refutation/confirmation of said theories. So far, the few papers dealing with the "energy generation" addressed this aspect as a problem to avoid, not as a feature to incorporate. This yields inconsistent ideas and confusing writings, like thrust depending on velocity (with no clear way to tell relative to what), or like telling that classical action/reaction propulsion suffers "the same problem". The only clean simple consistent model is constant thrust at given power, regardless of velocity/trajectory/history of the device, and that simple predictive model predicts possibility of energy generation, but this is seemingly embarrassing people. Why ?

Quote
I ask this question because if anything is clear from the discussions in this thread and the reported results to date. We only have a few rough guestimates at what is going on within the EmDrive. To prove or disprove the over unity power generation assertion you would need to completely characterize everything that is going on in the system. Right now the most we know is that when we pump EM into the cavity we get some thrust. We don't know beyond a reasonable doubt where the thrust is actually coming from. We don't know what we are actually interacting with. Now there are probably a whole lot of potential consequences of this research if it holds up to experimental scrutiny. But right now, the one thing I could probably guarantee, is that there will be a subset of those potential consequences that will not be possible once we have a better understanding of what is ACTUALLY happening.

Well, there are a lot of unknowns. It is quite possible that space flight application is part of the "subset of those potential consequences that will not be possible".

Quote
On the flip side, should the researchers be allowed to propose mission profiles that could potentially not pan out due to a fundamental lack of understanding of what is actually going on? Maybe. Though from my perspective I think I am willing to give the primary investigators some leeway in painting a picture of what could be possible. As long as they assume the responsibility of dealing with the fall out if their vision is never realized.

At the moment the painting is framed in such a way as not to show the whole picture...
One of the clearer answer I got so far (from those questions (http://forum.nasaspaceflight.com/index.php?topic=36313.msg1327228#msg1327228)) :

Last topic for the night for me.  Someone on this list asked if one could extract energy from the QV.  If the QV is GRT space-time, and space-time is the cosmological gravitational field that is created by all the causally connected mass/energy in our section of the universe, then we live in a high pressure sea of gravitational energy.  Now if the QV energy state is degradable and locally changeable, then one can posit the possibility of a thermodynamic energy conversion cycle that can extract energy from a pressure difference created in this QV media relative to the QV background average pressure, with a net decrease in this universal gravitational pressure or temperature reflective of the amount of energy so extracted.  And try to remember that gravitational energy is negative energy.  I'll leave the rest to you folks to draw your own conclusions from what this might mean...

Best, Paul March

As I understand this line of reasoning is high-risk/low-payoff I will stop there.
Title: Re: EM Drive Developments - related to space flight applications - Thread 2
Post by: Notsosureofit on 02/20/2015 11:42 pm
Entropically speaking, are there any other shapes for which we can write an analytic dispersion relation ?

And where did I leave my Gunn diode oscillator ?

Edit: and so far the work done/energy input is ~10-7, a long way from Newtonian unity.
Title: Re: EM Drive Developments - related to space flight applications - Thread 2
Post by: Rodal on 02/20/2015 11:49 pm
Entropically speaking, are there any other shapes for which we can write an analytic dispersion relation ?

And where did I leave my Gunn diode oscillator ?

I understand you mean a closed form expression in terms of readily available computable functions...

cylindrical

parallelopipedal

and...


 the spherical cavity

EDIT: and the paraboloid cavity (the solution involves Kummer’s confluent hypergeometric function, so not a simple function...   http://arxiv.org/pdf/physics/0101011.pdf  )
Title: Re: EM Drive Developments - related to space flight applications - Thread 2
Post by: frobnicat on 02/20/2015 11:51 pm
I think that frobnicat is simply saying that
IF there is a system (any system) that generate a given thrust when it's fed a given electrical power (with no other parameters affecting the thrust) THEN it can be used to generate power, at least theoretically.

Basically, yes.
To be precise, any system that is not expelling mass, that is thrusting at better that tpr = 1/c = 3.33 µN/kW (photon rocket) can be used to generate power if it can be "hooked" to a generator at a relative speed above 1/tpr. For 50µN/50W = 1000 µN/kW that speed is 1000 km/s (hard to hook to anything). For a photon rocket 1/tpr=c : nothing can go > c so the energy generating scheme is intrinsically impossible for a photon rocket. An Emdrive with 1N/kW starts to be overunity above 1km/s. Spinning it at 2km/s tangential speed (engineering feat but not impossible) would leave ample margin for inefficiencies in the closed cycle and still have a small net margin>0.

Quote
This implication doesn't necessitate knowledge of quantum physics, it's simply basic physics.

Yes, and anybody with a background in mechanics will see that.

Quote
The interesting question is then "Is EMdrive a system that generate a given thrust when it's fed a given electrical power?"
If it is, we can generate power with EMdrive. It is an useful information: it means that, since free energy doesn't exist, we take this energy from somewhere.
If it isn't, knowing which parameters affect the thrust will probably give us some insight into how EMdrive generate thrust.

(as usual, if we assume that the thrust is not an artefact)

Right. Thank you !
Title: Re: EM Drive Developments - related to space flight applications - Thread 2
Post by: flux_capacitor on 02/20/2015 11:52 pm
Finding pictures of shawyer's test device(s) is not that hard, but can any1 actually confirm that this is the Chinese truncated cone? or is it completely unrelated?

(http://tommytoy.typepad.com/.a/6a0133f3a4072c970b01a511f54ee6970c-800wi)

I think it's from Shawyer and improperly attributed to Juan Yang on some other web sites: http://emdrive.com/flightprogramme.html (http://emdrive.com/flightprogramme.html)

Someone should ask Roger Shawyer directly about the Chinese version (especially the dimensions of the cavity). He knows a lot about it, since he went there to speak with Juan Yang and give her some advice.
Title: Re: EM Drive Developments - related to space flight applications - Thread 2
Post by: Rodal on 02/20/2015 11:59 pm
....
I also would like to understand to why the curved bottoms of Shawyer's new test rig "makes more sense" , from microwave resonance point of view?

....
Because the type of electromagnetic waves that satisfy the boundary conditions at the lateral curved surfaces of a cone are spherical waves.  The great Russian/American scientist/engineer Schelkunoff showed this in the 1930's (http://forum.nasaspaceflight.com/index.php?topic=36313.msg1332981#msg1332981).  The natural Boundary Conditions for the bases of the cone are thus spherical surfaces.

The Finite Element and the Finite Difference calculations of NASA's EM Drive confirm that the electromagnetic fields are spherical standing waves away from the flat ends.

For a given spherical radius, the bigger the cone's angle, the more different is the spherical surface from a flat end, and the more important is to have spherical ends.  Kudos to Shawyer for being the first EM Drive researcher to realize this.

(http://gregegan.customer.netspace.net.au/SCIENCE/Cavity/TM3.gif)

(http://gregegan.customer.netspace.net.au/SCIENCE/Cavity/TE1.gif)
Title: Re: EM Drive Developments - related to space flight applications - Thread 2
Post by: Rodal on 02/21/2015 12:21 am
..

What would be the implications of fe, using a paraboloid (or a hyperboloid) ?
http://mathworld.wolfram.com/Paraboloid.html
A paraboloid has a natural focal point, which probably would create an ever higher H-field intensity, as far as the resonance patterns go?
(http://www.pbs.org/wgbh/nova/teachers/activities/images/3406_solar_focal1.gif)

...
Because of Maxwell's equations, and the paraboloid boundary conditions, the paraboloid cavity actually has the central axis as a forbidden zone for electromagnetic waves,  see these mode shapes:

authors: Jens U. Nockel, Isabelle Robert, Jean-Marie Moison, and Izo Abram
Title: Re: EM Drive Developments - related to space flight applications - Thread 2
Post by: Rodal on 02/21/2015 12:56 am
Finding pictures of shawyer's test device(s) is not that hard, but can any1 actually confirm that this is the Chinese truncated cone? or is it completely unrelated?

...


I think it's from Shawyer and improperly attributed to Juan Yang on some other web sites: http://emdrive.com/flightprogramme.html (http://emdrive.com/flightprogramme.html)

Someone should ask Roger Shawyer directly about the Chinese version (especially the dimensions of the cavity). He knows a lot about it, since he went there to speak with Juan Yang and give her some advice.


Yes, that Shawyer's Flight Thruster development programme. A 3.85GHz thruster weighing 2.92 Kg,.

(http://emdrive.com/images/thruster1.jpg)

http://emdrive.com/flightprogramme.html

I don't recall estimated dimensions for it. If anybody estimated the dimensions, @aero is the most likely one to have done it.
Title: Re: EM Drive Developments - related to space flight applications - Thread 2
Post by: aero on 02/21/2015 01:28 am
Finding pictures of shawyer's test device(s) is not that hard, but can any1 actually confirm that this is the Chinese truncated cone? or is it completely unrelated?

...


I think it's from Shawyer and improperly attributed to Juan Yang on some other web sites: http://emdrive.com/flightprogramme.html (http://emdrive.com/flightprogramme.html)

Someone should ask Roger Shawyer directly about the Chinese version (especially the dimensions of the cavity). He knows a lot about it, since he went there to speak with Juan Yang and give her some advice.


Yes, that Shawyer's Flight Thruster development programme. A 3.85GHz thruster weighing 2.92 Kg,.

(http://emdrive.com/images/thruster1.jpg)

http://emdrive.com/flightprogramme.html

I don't recall estimated dimensions for it. If anybody estimated the dimensions, @aero is the most likely one to have done it.

I did not. I see nothing to use as a reference. Perhaps someone could estimate ratios. Big/small, big/height or whatever.
Title: Re: EM Drive Developments - related to space flight applications - Thread 2
Post by: lasoi on 02/21/2015 02:15 am
The photo is a bit blurry and that makes estimating a bit challenging, and there are lens distortions to the photo, but nothing too major.  If the concrete block happened to be the standard width of 440 mm, cited by wikipedia, then the dimensions would be roughly as estimated in the chart.

I'm an artist, not a physicist.  If these dimensions seem wrong and you have a different guess for the width of the concrete block, let me know and I'll recalculate based on your width standard.
Title: Re: EM Drive Developments - related to space flight applications - Thread 2
Post by: Rodal on 02/21/2015 02:20 am
The photo is a bit blurry and that makes estimating a bit challenging, and there are lens distortions to the photo, but nothing too major.  If the concrete block happened to be a common standard size, which of course is no given, then the dimensions would roughly be:

Excellent !  Thank you lasoi !!!

(http://nplhinc.org/images/bravo.jpg)
Title: Re: EM Drive Developments - related to space flight applications - Thread 2
Post by: lasoi on 02/21/2015 02:22 am
Happy to help.  I love reading your posts.  Keep up the great work!
Title: Re: EM Drive Developments - related to space flight applications - Thread 2
Post by: ThinkerX on 02/21/2015 03:57 am
Oh well, time for me to demonstrate my ignorance again, this time with a suggestion:

All through these calculations, the thrust is given as 'thrust per photon.' 

Photons seem to be durable little critters, capable of a lot of bounces (50,000+?) before becoming absorbed. 

Bouncing photons are a feature of the laser driven drives proposed by a Doctor David Bae (?) have to look this up, who reported a 3000 fold increase in thrust in a laboratory setting from this mechanism.

Unlike the EM drive, his work is based on known technology and principles.

Given this, might not a 'bouncing photon scale' be of some use in measuring the performance of the EM drive?

Run the thrust per photon calculation, assuming each 'bounce' adds a (decreasing) unit of thrust.  The upper theoretical limit should be well above what Bae reported. 

If the EM Drive stays within this scale in a vacuum, then maybe photon bouncing has something to do with its operation.

Ok, I better call it quits while I'm behind. 
Title: Re: EM Drive Developments - related to space flight applications - Thread 2
Post by: Mulletron on 02/21/2015 07:57 am
http://eec.wustl.edu/aboutthedepartment/Pages/news-story.aspx?news=7577&source=admin

http://adsabs.harvard.edu/abs/2014NatPh..10..394P

http://ieeexplore.ieee.org/iel7/4563994/4814557/06690216.pdf

Man we need way stronger magnetic fields and better materials in that thing. The more I keep reading about this subject of PT symmetry breaking the clearer it becomes that this is routine.
Title: Re: EM Drive Developments - related to space flight applications - Thread 2
Post by: Flyby on 02/21/2015 09:19 am
huh... :o was going to start on it today after getting the feedback, but it seems Lasoi has beaten me to it while i was sleeping...
but I'll have a look at it myself anyway...

Thanks for all those replying that it was NOT the Chinese test engine. It kinda confirmed my suspicions that something wasn't right...

The problem with most of the drawings i see about the resonance cavities is that they are most likely drawings of concepts, they come in all shapes and angles but none give me the impression to be actual engineering based drawings.
I do not think it is wise attempting to extract any realworld info from conceptual drawings.

fe, There is a clear visual difference between the superconductivity cavity drawing from Shawyer (see higher up) and fe, the Chinese drawings. Shawyer's drawing is clearly based on an engineering drawing, where the Chinese drawing is way more schematically build. I'd be much more confident in basing assumptions on Shawyer's drawing then on the Chinese one...
My gut feeling (i know, hardly a scientific approach) says that the chinese drawing has little to no correlation to the real testunit they've build.
Title: Re: EM Drive Developments - related to space flight applications - Thread 2
Post by: Flyby on 02/21/2015 09:30 am

The Finite Element and the Finite Difference calculations of NASA's EM Drive confirm that the electromagnetic fields are spherical standing waves away from the flat ends.

For a given spherical radius, the bigger the cone's angle, the more different is the spherical surface from a flat end, and the more important is to have spherical ends.  Kudos to Shawyer for being the first EM Drive researcher to realize this.


Got it! It is all about having equal distances for the bouncing waves between the 2 top/bottom surfaces. Basically, it is a truncated spherical cone...
http://mathworld.wolfram.com/SphericalCone.html

thnx for the clarification...
Title: Re: EM Drive Developments - related to space flight applications - Thread 2
Post by: Rodal on 02/21/2015 12:31 pm
....
My gut feeling (i know, hardly a scientific approach) says that the chinese drawing has little to no correlation to the real testunit they've build.
What Chinese drawing are you referring to? Could you please give a link or a reference to it?  Thanks
Title: Re: EM Drive Developments - related to space flight applications - Thread 2
Post by: Rodal on 02/21/2015 12:40 pm

The Finite Element and the Finite Difference calculations of NASA's EM Drive confirm that the electromagnetic fields are spherical standing waves away from the flat ends.

For a given spherical radius, the bigger the cone's angle, the more different is the spherical surface from a flat end, and the more important is to have spherical ends.  Kudos to Shawyer for being the first EM Drive researcher to realize this.


Got it! It is all about having equal distances for the bouncing waves between the 2 top/bottom surfaces. Basically, it is a truncated spherical cone...
http://mathworld.wolfram.com/SphericalCone.html

thnx for the clarification...

The correct shape to have perfect propagation of electromagnetic waves (respecting all Boundary Conditions) is a spherical cone truncated at its narrow end by another section of a sphere. 

<< It is all about having equal spherical radial distances for the standing spherical waves between the 2 top/bottom surfaces>>

Yes, that should be called a "truncated spherical cone".  Thanks for the  http://mathworld.wolfram.com/SphericalCone.html  reference.

The truncated cone with flat ends previously used by Shawyer (before its present superconducting design) and by NASA Eagleworks and Juan Yang in China do not satisfy the ideal shape for the truncated ends which should be spherical. 

For a given spherical radius, the larger the cone angle, the more important is this difference.
Title: Re: EM Drive Developments - related to space flight applications - Thread 2
Post by: Notsosureofit on 02/21/2015 12:40 pm
http://eec.wustl.edu/aboutthedepartment/Pages/news-story.aspx?news=7577&source=admin

http://adsabs.harvard.edu/abs/2014NatPh..10..394P

http://ieeexplore.ieee.org/iel7/4563994/4814557/06690216.pdf

Man we need way stronger magnetic fields and better materials in that thing. The more I keep reading about this subject of PT symmetry breaking the clearer it becomes that this is routine.

To my mind, the most interesting aspect of these papers is the incorporation of non-linear materials generating additional frequency components.
Title: Re: EM Drive Developments - related to space flight applications - Thread 2
Post by: Rodal on 02/21/2015 12:43 pm
http://eec.wustl.edu/aboutthedepartment/Pages/news-story.aspx?news=7577&source=admin

http://adsabs.harvard.edu/abs/2014NatPh..10..394P

http://ieeexplore.ieee.org/iel7/4563994/4814557/06690216.pdf

Man we need way stronger magnetic fields and better materials in that thing. The more I keep reading about this subject of PT symmetry breaking the clearer it becomes that this is routine.

To my mind, the most interesting aspect of these papers is the incorporation of non-linear materials generating additional frequency components.

Agreed.  From the point of view of solving the dispersion problem, the nonlinearity of the materials involved is what makes a difference, at least at first sight ?.
Title: Re: EM Drive Developments - related to space flight applications - Thread 2
Post by: flux_capacitor on 02/21/2015 01:03 pm
....
My gut feeling (i know, hardly a scientific approach) says that the chinese drawing has little to no correlation to the real testunit they've build.
What Chinese drawing are you referring to? Could you please give a link or a reference to it?  Thanks

I think Flyby refers to the fact that the various EmDrive drawings from the Chinese do not look alike each others. See for example the compilation attached. The two last drawings, largely different, even come from the same 2015 paper. All we can say looking at those pictures is the length of their cavity is perhaps shorter than Shawyer's or Eagleworks designs. But the angles and proportions are all different.
Title: Re: EM Drive Developments - related to space flight applications - Thread 2
Post by: Notsosureofit on 02/21/2015 01:09 pm
http://eec.wustl.edu/aboutthedepartment/Pages/news-story.aspx?news=7577&source=admin

http://adsabs.harvard.edu/abs/2014NatPh..10..394P

http://ieeexplore.ieee.org/iel7/4563994/4814557/06690216.pdf

Man we need way stronger magnetic fields and better materials in that thing. The more I keep reading about this subject of PT symmetry breaking the clearer it becomes that this is routine.


To my mind, the most interesting aspect of these papers is the incorporation of non-linear materials generating additional frequency components.

Agreed.  From the point of view of solving the dispersion problem, the nonlinearity of the materials involved is what makes a difference, at least at first sight ?.

As a starting ferinstance, how much native copper oxide is present on the inner surfaces of the cavity ?  Enough to convert 10-7 ?
Title: Re: EM Drive Developments - related to space flight applications - Thread 2
Post by: Rodal on 02/21/2015 01:10 pm
....
My gut feeling (i know, hardly a scientific approach) says that the chinese drawing has little to no correlation to the real testunit they've build.
What Chinese drawing are you referring to? Could you please give a link or a reference to it?  Thanks

"the Chinese (drawings) do not look alike each others"
Yes, because drawings that do not look like each other in that compilation refer to different EM Drive designs, as explained in the Chinese text of the different published articles by Juan Yang's team.  The fact that Juan Yang has improved on her design and has tested different EM Drive designs does not necessarily mean that there is "little to no correlation to the real test unit they've build".  She documents a large number of tests (much more numerous than the tests conducted at NASA Eagleworks, for example).

Shawyer has also conducted experimental tests with different EM Drive geometries: the "Demo", the "Flight Thruster", the "Experimental", etc., and now the "Superconducting 2014 design".

Similarly, NASA Eagleworks has tested recently the Cannae pillbox design and the truncated cone frustum design and reports experiments for both (drastically different geometries) in the same report "Anomalous..." by Brady et.al.. 

It does not necessarily follow from that that there is "little to no correlation to the real test unit they've build" all it may mean is that they built and tested different geometries.  Unless somebody has some reported information that I have missed, in which case I'll be happy to be better informed.
Title: Re: EM Drive Developments - related to space flight applications - Thread 2
Post by: Rodal on 02/21/2015 01:21 pm
http://eec.wustl.edu/aboutthedepartment/Pages/news-story.aspx?news=7577&source=admin

http://adsabs.harvard.edu/abs/2014NatPh..10..394P

http://ieeexplore.ieee.org/iel7/4563994/4814557/06690216.pdf

Man we need way stronger magnetic fields and better materials in that thing. The more I keep reading about this subject of PT symmetry breaking the clearer it becomes that this is routine.


To my mind, the most interesting aspect of these papers is the incorporation of non-linear materials generating additional frequency components.

Agreed.  From the point of view of solving the dispersion problem, the nonlinearity of the materials involved is what makes a difference, at least at first sight ?.

As a starting ferinstance, how much native copper oxide is present on the inner surfaces of the cavity ?  Enough to convert 10-7 ?

Excellent question. 
Title: Re: EM Drive Developments - related to space flight applications - Thread 2
Post by: Notsosureofit on 02/21/2015 01:34 pm
http://eec.wustl.edu/aboutthedepartment/Pages/news-story.aspx?news=7577&source=admin

http://adsabs.harvard.edu/abs/2014NatPh..10..394P

http://ieeexplore.ieee.org/iel7/4563994/4814557/06690216.pdf

Man we need way stronger magnetic fields and better materials in that thing. The more I keep reading about this subject of PT symmetry breaking the clearer it becomes that this is routine.


To my mind, the most interesting aspect of these papers is the incorporation of non-linear materials generating additional frequency components.

Agreed.  From the point of view of solving the dispersion problem, the nonlinearity of the materials involved is what makes a difference, at least at first sight ?.

As a starting ferinstance, how much native copper oxide is present on the inner surfaces of the cavity ?  Enough to convert 10-7 ?

Excellent question.

While the formula I've been using is based on satisfying General Relativity, it does not tell us anything about the mechanism of momentum conservation.. PT asymmetry, as Mulletron mentions, is a viable candidate, and nonlinear frequency effects could (in theory) satisfy the requirement.


Title: Re: EM Drive Developments - related to space flight applications - Thread 2
Post by: Rodal on 02/21/2015 01:46 pm
....
While the formula I've been using is based on satisfying General Relativity, it does not tell us anything about the mechanism of momentum conservation.. PT asymmetry, as Mulletron mentions, is a viable candidate, and nonlinear frequency effects could (in theory) satisfy the requirement.
Can anybody present quantitative experimentally-measured data showing significant PT asymmetry or nonlinear frequency effects for a bulk High Density Polyethylene (purchased commercially from McMaster Carr, if my memory serves me correctly ?) used as the dielectric by NASA Eagleworks in their tests ?

That could be a Rosetta Stone...
Title: Re: EM Drive Developments - related to space flight applications - Thread 2
Post by: Notsosureofit on 02/21/2015 01:56 pm
A good question might be how "pristine" the inner surface of a Shawyer cavity might be compared to a NASA one.  NASA is quite used to super-clean surfaces....( their chambers compared to mine ferinstance.....Gulp !!!)
Title: Re: EM Drive Developments - related to space flight applications - Thread 2
Post by: Rodal on 02/21/2015 02:16 pm
The photo is a bit blurry and that makes estimating a bit challenging, and there are lens distortions to the photo, but nothing too major.  If the concrete block happened to be the standard width of 440 mm, cited by wikipedia, then the dimensions would be roughly as estimated in the chart.

I'm an artist, not a physicist.  If these dimensions seem wrong and you have a different guess for the width of the concrete block, let me know and I'll recalculate based on your width standard.
Thanks to @lasoi, from these dimensions for Shawyer's Flight Thruster (a 3.85GHz thruster weighing 2.92 Kg), we can calculate the most important parameter: the cone's half angle thetaw (as per image below), which is, trivially, a function of the flat base diameters and the axial length:

thetaw=ArcTan[(BigDiameter-SmallDiameter)/(2*AxialLength)],

and add it to my table below (note the cone's half-angle, as a dimensionless quantity, is robust insensitive to the length magnitude of the cinder blocks in the wall on the background of the picture):

For reference. the tangent of the cone's half angle thetaw and the cone's half angle thetaw, in ascending order, for the following cases are:

(Notice how Shawyer progressively increased the cone's half-angle, with time, in his experimental designs, by a factor of 7 in the tangent of the half-angle)

Example (and geometry)                    { Tan[thetaw],thetaw (degrees) }

Shawyer Experimental                        {0.104019,   5.93851}
Shawyer Fligth Thruster                      {0.19086,   10.8055}
Shawyer Demo                                   {0.219054, 12.3557}
NASA Eagleworks frustum                   {0.263889, 14.7827}
Egan's example                                  {0.36397 ,  20}
Prof. Juan Yang  (2014)                      {0.4538,     24.4 }
Shawyer Superconducting 2014          {0.7002,     35}


(http://gregegan.customer.netspace.net.au/SCIENCE/Cavity/CavityShape.gif)

(http://forum.nasaspaceflight.com/index.php?action=dlattach;topic=36313.0;attach=802951;image)
(http://forum.nasaspaceflight.com/index.php?action=dlattach;topic=36313.0;attach=802794;image)
Title: Re: EM Drive Developments - related to space flight applications - Thread 2
Post by: Notsosureofit on 02/21/2015 03:17 pm
There are a lot of references to nonlinear effects in polyethylene, but most seem to be due to impurities of one sort or another.
Title: Re: EM Drive Developments - related to space flight applications - Thread 2
Post by: Rodal on 02/21/2015 03:44 pm
There are a lot of references to nonlinear effects in polyethylene, but most seem to be due to impurities of one sort or another.
If so, nonlinearity of the HD PE used for the dielectric in the NASA Eagleworks tests, rather than occurring by intelligent design, by intentional doping, would be by accidental impurity, due to lax quality control in the manufacture of the industrially supplied bulk HD PE used by NASA Eagleworks.

EDIT: Still, what types and how much impurity would be necessary to take place in order to have significant nonlinear effects?
Title: Re: EM Drive Developments - related to space flight applications - Thread 2
Post by: Notsosureofit on 02/21/2015 03:47 pm
There are a lot of references to nonlinear effects in polyethylene, but most seem to be due to impurities of one sort or another.
So nonlinearity of the HD PE, rather than being by intelligent design, would be by accidental impurity, due to lax quality control in the manufacture of the industrially supplied bulk HD PE used by NASA Eagleworks.

That's a possibility.  Could also explain the difference between some chambers w/ and w/o dielectric ???
Title: Re: EM Drive Developments - related to space flight applications - Thread 2
Post by: Rodal on 02/21/2015 03:48 pm
There are a lot of references to nonlinear effects in polyethylene, but most seem to be due to impurities of one sort or another.
So nonlinearity of the HD PE, rather than being by intelligent design, would be by accidental impurity, due to lax quality control in the manufacture of the industrially supplied bulk HD PE used by NASA Eagleworks.

That's a possibility.  Could also explain the difference between some chambers w/ and w/o dielectric ???

Still, what types and how much impurity would be necessary to take place in order to have significant nonlinear effects?
Title: Re: EM Drive Developments - related to space flight applications - Thread 2
Post by: Notsosureofit on 02/21/2015 03:59 pm
There are a lot of references to nonlinear effects in polyethylene, but most seem to be due to impurities of one sort or another.
So nonlinearity of the HD PE, rather than being by intelligent design, would be by accidental impurity, due to lax quality control in the manufacture of the industrially supplied bulk HD PE used by NASA Eagleworks.

That's a possibility.  Could also explain the difference between some chambers w/ and w/o dielectric ???

Still, what types and how much impurity would be necessary to take place in order to have significant nonlinear effects?

Good question.  One of the papers Mulletron brought up:

http://ieeexplore.ieee.org/stamp/stamp.jsp?arnumber=6690216

has interesting curves showing the frequency shifts.   Maybe Mulletron has a PT input on this point ?

" More interestingly, at the PT phase transition point, also called spontaneous PT phase breaking point, a unidirectional invisible medium was realized"
Title: Re: EM Drive Developments - related to space flight applications - Thread 2
Post by: Star-Drive on 02/21/2015 05:08 pm
There are a lot of references to nonlinear effects in polyethylene, but most seem to be due to impurities of one sort or another.
So nonlinearity of the HD PE, rather than being by intelligent design, would be by accidental impurity, due to lax quality control in the manufacture of the industrially supplied bulk HD PE used by NASA Eagleworks.

That's a possibility.  Could also explain the difference between some chambers w/ and w/o dielectric ???

Still, what types and how much impurity would be necessary to take place in order to have significant nonlinear effects?

Good question.  One of the papers Mulletron brought up:

http://ieeexplore.ieee.org/stamp/stamp.jsp?arnumber=6690216

has interesting curves showing the frequency shifts.   Maybe Mulletron has a PT input on this point ?

Dr. Rodal & Notsosureofit:

We had an interesting failure in the Eagleworks lab yesterday.  That being I was getting ready to pull a vacuum on our copper frustum mounted in its "reverse" or to the right thrust vector position and ran a preliminary data un to see if it was performing in air as it had two weeks ago just before our last RF amplifier died.  Sadly it wasn't for it was producing less than half of what it did before and in the wrong direction!   

I had Dr. White come in and take a look over my latest test article installation last night and he found that the center 1/4"-20 nylon PE disc mounting bolt that holds the second PE disc to the small OD frustum's PCB endplate was no-longer tensioned as it had been before.  In fact it had partially melted at the interface between the two PE discs thus relieving the strain induced by its bolts threads and nut.  (There are three ~1.00" 1/4-20 nylon bolts mounted on a ~2.00" radius spaced every 120 degrees that hold the first PE disc to the PCB end cap.   There is then a layer of 3/4" wide office scotch tape at the interface between the first and second PE discs and the center 1/4"-20 nylon bolt that hold second PE disc to the first PE disc.) 

Apparently not having the PE discs firmly mounted to the frustum's small OD end cap hindered the thrust producing mechanism that conveys the generated forces in the PE to the copper frustum.  And/or the melted nylon was hogging all the RF energy in the PE discs due to its higher dissipation factor in its semiliquid state.  Either way it looks like there is a high E-field volume where this center nylon bolt hangs out while running in the TM212 resonant mode.  Too bad Teflon bolts are so weak even in comparison to the nylon, for its dissipation factor is at least two orders of magnitude lower than the nylon's.

Best, Paul M. 
Title: Re: EM Drive Developments - related to space flight applications - Thread 2
Post by: Notsosureofit on 02/21/2015 05:21 pm
That would certainly throw all kinds of motion induced errors into the mix.  Best to carefully rebuild the original configuration.


Anyway, there is an interesting, if probably coincidence, between the reported results in the above paper using a 19 period optical cavity.  When you compare their (PT transition) frequency shift to a 2 period cavity you get 2.1 GHz which is of the order of magnitude of the shift (~ 1.7 GHz) the calculation for the Brady cavity.

Probably means nothing....but curious.

Title: Re: EM Drive Developments - related to space flight applications - Thread 2
Post by: Rodal on 02/21/2015 06:05 pm
That would certainly throw all kinds of motion induced errors into the mix.  Best to carefully rebuild the original configuration.


Anyway, there is an interesting, if probably coincidence, between the reported results in the above paper using a 19 period optical cavity.  When you compare their (PT transition) frequency shift to a 2 period cavity you get 2.1 GHz which is of the order of magnitude of the shift (~ 1.7 GHz) the calculation for the Brady cavity.

Probably means nothing....but curious.

Using a unidirectional reflectionless PT grating with a nonlinear silicon distributed Bragg reflector (DBR) cavity consisting of: 20 modulation periods of 2 Pi /q = 0.27 micro meters



EDIT;

<<the emergence of the Optical Rogue Waves (ORWs) ( http://en.wikipedia.org/wiki/Optical_rogue_waves  ) has been explored based on the system parameters>>

http://arxiv.org/ftp/arxiv/papers/1312/1312.3400.pdf



(http://upload.wikimedia.org/wikipedia/commons/thumb/f/f2/Arrayed-Waveguide-Grating.svg/680px-Arrayed-Waveguide-Grating.svg.png)
Title: Re: EM Drive Developments - related to space flight applications - Thread 2
Post by: Rodal on 02/21/2015 07:08 pm
There are a lot of references to nonlinear effects in polyethylene, but most seem to be due to impurities of one sort or another.
So nonlinearity of the HD PE, rather than being by intelligent design, would be by accidental impurity, due to lax quality control in the manufacture of the industrially supplied bulk HD PE used by NASA Eagleworks.

That's a possibility.  Could also explain the difference between some chambers w/ and w/o dielectric ???

Still, what types and how much impurity would be necessary to take place in order to have significant nonlinear effects?

Good question.  One of the papers Mulletron brought up:

http://ieeexplore.ieee.org/stamp/stamp.jsp?arnumber=6690216

has interesting curves showing the frequency shifts.   Maybe Mulletron has a PT input on this point ?

Dr. Rodal & Notsosureofit:

We had an interesting failure in the Eagleworks lab yesterday.  That being I was getting ready to pull a vacuum on our copper frustum mounted in its "reverse" or to the right thrust vector position and ran a preliminary data un to see if it was performing in air as it had two weeks ago just before our last RF amplifier died.  Sadly it wasn't for it was producing less than half of what it did before and in the wrong direction!   

I had Dr. White come in and take a look over my latest test article installation last night and he found that the center 1/4"-20 nylon PE disc mounting bolt that holds the second PE disc to the small OD frustum's PCB endplate was no-longer tensioned as it had been before.  In fact it had partially melted at the interface between the two PE discs thus relieving the strain induced by its bolts threads and nut.  (There are three ~1.00" 1/4-20 nylon bolts mounted on a ~2.00" radius spaced every 120 degrees that hold the first PE disc to the PCB end cap.   There is then a layer of 3/4" wide office scotch tape at the interface between the first and second PE discs and the center 1/4"-20 nylon bolt that hold second PE disc to the first PE disc.) 

Apparently not having the PE discs firmly mounted to the frustum's small OD end cap hindered the thrust producing mechanism that conveys the generated forces in the PE to the copper frustum.  And/or the melted nylon was hogging all the RF energy in the PE discs due to its higher dissipation factor in its semiliquid state.  Either way it looks like there is a high E-field volume where this center nylon bolt hangs out while running in the TM212 resonant mode.  Too bad Teflon bolts are so weak even in comparison to the nylon, for its dissipation factor is at least two orders of magnitude lower than the nylon's.

Best, Paul M.

Great information, thank you so much for sharing it.   :)

Great discoveries have been made in the past as a result of "failures"

The information that:

" not having the PE discs firmly mounted to the frustum's small OD end cap hindered the thrust producing mechanism that conveys the generated forces in the PE to the copper frustum."

is very valuable and telling of the importance, not only of the dielectric, but of the dielectric being connected to the small flat end.  One idea that had been floated in this thread was to do a test without the small flat end to see what happens.  This "failure" indicates that removal of the copper (on the inside) small flat end may result in significant loss of thrust force.
Title: Re: EM Drive Developments - related to space flight applications - Thread 2
Post by: Mulletron on 02/21/2015 07:21 pm
....
While the formula I've been using is based on satisfying General Relativity, it does not tell us anything about the mechanism of momentum conservation.. PT asymmetry, as Mulletron mentions, is a viable candidate, and nonlinear frequency effects could (in theory) satisfy the requirement.
Can anybody present quantitative experimentally-measured data showing significant PT asymmetry or nonlinear frequency effects for a bulk High Density Polyethylene (purchased commercially from McMaster Carr, if my memory serves me correctly ?) used as the dielectric by NASA Eagleworks in their tests ?

That could be a Rosetta Stone...
http://www.mcmaster.com/#standard-plastic-rods/=w0bzy0
Website won't let me direct link to it. Can't find that 6.25" dimension. See Rigid HDPE Polyethylene.

Quote
Can anybody present quantitative experimentally-measured data showing significant PT asymmetry or nonlinear frequency effects....
I'd suggest changing significant to any. I doubt this will come because PT symmetry breaking doesn't come out of the box, it is a spontaneous symmetry break brought on by the specific conditions the material is subject to. I honestly think an expert is going to have to examine the spontaneous PT symmetry breaking in Emdrive in order to conclusively rule it in or out. I think the best way to figure it out is to substitute the PE with a perfectly ordered chiral material and see where the thrust goes. As we've discussed, PT PE and PTFE have an achiral backbone, yet gain chirality through twisting around the C=C bonds. So there is much room for improvement.
http://www.esrf.eu/UsersAndScience/Publications/Highlights/2011/scm/scm4
http://goo.gl/p7VVKJ
http://goo.gl/aO9E2Z pg94

I'm digging around in PT symmetry more to see if there any analogues to Emdrive to be found, like asymmetric waveguide or high filed strengths, which might hint that this is or is not in fact what is happening. Got hints to those ones in bold from the optical literature posted. It is clear from researching the issue that the phenomenon is well understood.

Given the eerie performance of TE012 combined with the high magnetic field strength at the small end, and the theory making use of the magnetic field to "transfer momentum" it seems likely but not proven this is really happening.

Title: Re: EM Drive Developments - related to space flight applications - Thread 2
Post by: Mulletron on 02/21/2015 07:28 pm
Apparently not having the PE discs firmly mounted to the frustum's small OD end cap hindered the thrust producing mechanism that conveys the generated forces in the PE to the copper frustum. ...........

Best, Paul M.
That dielectric is definitely doing something important. Would be interesting what would happen if a piece of gold leaf was sandwiched behind the PE, just to see what happens to the thrust. This is a hint to an earlier post I made about the perceived importance of the air/dielectric/copper interface, which might me true after all, if there is a giant Casimir force inside the cavity.
http://forum.nasaspaceflight.com/index.php?topic=36313.msg1329454#msg1329454


(http://www.minternetmarketing.com/Portals/55042/images/spock_fascinating-resized-600.jpg)
Title: Re: EM Drive Developments - related to space flight applications - Thread 2
Post by: Rodal on 02/21/2015 07:40 pm
....
While the formula I've been using is based on satisfying General Relativity, it does not tell us anything about the mechanism of momentum conservation.. PT asymmetry, as Mulletron mentions, is a viable candidate, and nonlinear frequency effects could (in theory) satisfy the requirement.
Can anybody present quantitative experimentally-measured data showing significant PT asymmetry or nonlinear frequency effects for a bulk High Density Polyethylene (purchased commercially from McMaster Carr, if my memory serves me correctly ?) used as the dielectric by NASA Eagleworks in their tests ?

That could be a Rosetta Stone...
http://www.mcmaster.com/#standard-plastic-rods/=w0bzy0
Website won't let me direct link to it. Can't find that 6.25" dimension. See Rigid HDPE Polyethylene.

...

Under Performance of Plastics it directs you to a .pdf  that lists

450-1,800  Dielectric Strength, volts/0.001inch

that is 17.72 million V/m (on the low end) that's a little lower than the lowest value listed here:  http://hypertextbook.com/facts/2009/CherryXu.shtml but the same minimum listed here: http://www.rfcafe.com/references/electrical/dielectric-constants-strengths.htm


************

Biggest diameter they list is 6.00 inches:

Rigid HDPE Polyethylene Rod, 6" Diameter

Length, ft.
1
2
3
4
5
6
7
8
Title: Re: EM Drive Developments - related to space flight applications - Thread 2
Post by: Flyby on 02/21/2015 07:57 pm
....
My gut feeling (i know, hardly a scientific approach) says that the chinese drawing has little to no correlation to the real testunit they've build.
What Chinese drawing are you referring to? Could you please give a link or a reference to it?  Thanks

I think Flyby refers to the fact that the various EmDrive drawings from the Chinese do not look alike each others. See for example the compilation attached. The two last drawings, largely different, even come from the same 2015 paper. All we can say looking at those pictures is the length of their cavity is perhaps shorter than Shawyer's or Eagleworks designs. But the angles and proportions are all different.

Good to see all Chinese drawings at once, I was not familiar with all of them... thnx for that, flux_

The only one that strikes me to be a real world technical drawing would be the bottom right one.
All the rest will most likely be pure schematic/concept drawings, intended rather to explain something and not intended to represent actual build systems.
I'm not doubting they made several designs and tests, but I seriously doubt that every drawing made relates back to a real test or model. Most of these drawings simply do not hold enough technical info to be credible "as build" plans. You can not use those drawings and proportions to base a real build model on them.
Technical drawings do follow a certain code and stick to general conventions.
The majority of those drawings do not follow those... Hence why I say that they "do not correlate" with any real made object. They're not construction drawings, but function as communication drawings, explaining the workings, concepts and layouts. You should not base dimensional extrapolations for a real world object on them.

But hey, I'm merely sharing 30 years of experience handling technical drawings... if you feel I'm wrong on that assumption... I can live with that...the earth will continue spinning.. ;)
Title: Re: EM Drive Developments - related to space flight applications - Thread 2
Post by: Rodal on 02/21/2015 08:44 pm
....
My gut feeling (i know, hardly a scientific approach) says that the chinese drawing has little to no correlation to the real testunit they've build.
What Chinese drawing are you referring to? Could you please give a link or a reference to it?  Thanks

I think Flyby refers to the fact that the various EmDrive drawings from the Chinese do not look alike each others. See for example the compilation attached. The two last drawings, largely different, even come from the same 2015 paper. All we can say looking at those pictures is the length of their cavity is perhaps shorter than Shawyer's or Eagleworks designs. But the angles and proportions are all different.

Good to see all Chinese drawings at once, I was not familiar with all of them... thnx for that, flux_

The only one that strikes me to be a real world technical drawing would be the bottom right one.
All the rest will most likely be pure schematic/concept drawings, intended rather to explain something and not intended to represent actual build systems.
I'm not doubting they made several designs and tests, but I seriously doubt that every drawing made relates back to a real test or model. Most of these drawings simply do not hold enough technical info to be credible "as build" plans. You can not use those drawings and proportions to base a real build model on them.
Technical drawings do follow a certain code and stick to general conventions.
The majority of those drawings do not follow those... Hence why I say that they "do not correlate" with any real made object. They're not construction drawings, but function as communication drawings, explaining the workings, concepts and layouts. You should not base dimensional extrapolations for a real world object on them.

But hey, I'm merely sharing 30 years of experience handling technical drawings... if you feel I'm wrong on that assumption... I can live with that...the earth will continue spinning.. ;)
1) I was just asking for a link to understand what you were referring to. 

2) I was not intending to use any of those drawings to build anything but instead to conduct calculations that take a fraction of a second to calculate.  Calculating the frequencies and mode shapes from an estimated geometry and comparing it with the published results quickly tells me how representative was the drawing of the actual build system.  That's how for example, we quickly eliminated the @Fornaro estimate of geometry and zeroed in on the @aero and @Mulletron estimates, for example.

Thanks
Title: Re: EM Drive Developments - related to space flight applications - Thread 2
Post by: Notsosureofit on 02/21/2015 08:49 pm
@ Star-Drive

We've battled the Nylon vs Teflon fasteners for years in our plasma chambers.  These days we replace the Nylon ones every run.  Teflon holds up very well, just won't take much mechanical load.
Title: Re: EM Drive Developments - related to space flight applications - Thread 2
Post by: Notsosureofit on 02/21/2015 09:31 pm
That would certainly throw all kinds of motion induced errors into the mix.  Best to carefully rebuild the original configuration.


Anyway, there is an interesting, if probably coincidence, between the reported results in the above paper using a 19 period optical cavity.  When you compare their (PT transition) frequency shift to a 2 period cavity you get 2.1 GHz which is of the order of magnitude of the shift (~ 1.7 GHz) the calculation for the Brady cavity.

Probably means nothing....but curious.

Using a unidirectional reflectionless PT grating with a nonlinear silicon distributed Bragg reflector (DBR) cavity consisting of: 20 modulation periods of 2 Pi /q = 0.27 micro meters



EDIT;

<<the emergence of the Optical Rogue Waves (ORWs) ( http://en.wikipedia.org/wiki/Optical_rogue_waves  ) has been explored based on the system parameters>>

http://arxiv.org/ftp/arxiv/papers/1312/1312.3400.pdf



(http://upload.wikimedia.org/wikipedia/commons/thumb/f/f2/Arrayed-Waveguide-Grating.svg/680px-Arrayed-Waveguide-Grating.svg.png)


Interesting!

Quick read makes me think we would be below the PT transition.  At some power level the divergence would increase dramatically and so would the thrust.  But this is very speculative!!
Title: Re: EM Drive Developments - related to space flight applications - Thread 2
Post by: Flyby on 02/21/2015 10:13 pm

1) I was just asking for a link to understand what you were referring to. 

2) I was not intending to use any of those drawings to build anything but instead to conduct calculations that take a fraction of a second to calculate.  Calculating the frequencies and mode shapes from an estimated geometry and comparing it with the published results quickly tells me how representative was the drawing of the actual build system.  That's how for example, we quickly eliminated the @Fornaro estimate of geometry and zeroed in on the @aero and @Mulletron estimates, for example.

Thanks
Sigh...rereading my post, my answer does sound unnecessarily aggressive.... My apologies for that.
Lame excuse, but it's getting late here...I'd better go get some good night of sleep... :-X
Title: Re: EM Drive Developments - related to space flight applications - Thread 2
Post by: Mulletron on 02/22/2015 12:52 am
I think the following concerns by @seanmcarroll have been addressed in this thread:

Quote
There is no such thing as a ‘quantum vacuum virtual plasma,’ so that should be a tip-off right there. There is a quantum vacuum, but it is nothing like a plasma. In particular, it does not have a rest frame, so there is nothing to push against, so you can’t use it for propulsion. The whole thing is just nonsense. They claim to measure an incredibly tiny effect that could very easily be just noise.” There is no theory to support the result, and there is no verified result to begin with.
source: http://blogs.discovermagazine.com/outthere/2014/08/06/nasa-validate-imposible-space-drive-word/#.VOk1XC6pjSg

We know it isn't a plasma. We know it doesn't have a rest frame, and accept you can't push against it, but you likely can use it for propulsion under the circumstances we've explored here. The device has also been successfully tested in vacuum.

Title: Re: EM Drive Developments - related to space flight applications - Thread 2
Post by: Rodal on 02/22/2015 01:42 am
Well Sean Carroll, himself wrote, when discussing the Quantum Vacuum stability:

Quote from: Sean Carroll
observers shouldn’t take too seriously the grandiose claims of theorists about what is and is not possible; they should do their experiments and see what the data imply. It would be a shame to miss out on a fantastic discovery because you believed some theorist who told you it couldn’t possibly be there.

http://www.preposterousuniverse.com/blog/2004/09/14/vacuum-stability/
Title: Re: EM Drive Developments - related to space flight applications - Thread 2
Post by: Rodal on 02/22/2015 02:03 am
I think the following concerns by @seanmcarroll have been addressed in this thread:

Quote
...and there is no verified result to begin with.
source: http://blogs.discovermagazine.com/outthere/2014/08/06/nasa-validate-imposible-space-drive-word/#.VOk1XC6pjSg
...The device has also been successfully tested in vacuum.

Probably Sean Carroll still will maintain that "there is no verified result to begin with" because the test under hard vacuum has not been verified at independent institutions, particularly in academia.  That's why I always thought that replicating the tests #1) at John Hopkins (if given a choice between Glenn, JPL and John Hopkins) and #2) at JPL, and #3) NASA Glenn, would have provided the most credibility regarding the subject of independent verification.

He will probably remind readers of the Cold Fusion claims (and similar examples) where Martin Fleischmann (then one of the world's leading electrochemists) reported "anomalous heat" (interesting: same word used in Brady's report title !), then many scientists tried to replicate their experiment but hopes fell with the large number of negative replications, and the withdrawal of many positive replications.
Title: Re: EM Drive Developments - related to space flight applications - Thread 2
Post by: Rodal on 02/22/2015 02:09 am
Oh, actually, there is a much better #1 in my book, than those three places: to independently verify the NASA Eagleworks test at Notsosureofit's lab  !

If Notsosureofit verifies it, then it is a slam dunk, a guaranteed sure thing, and we are on our way !
Title: Re: EM Drive Developments - related to space flight applications - Thread 2
Post by: Star-Drive on 02/22/2015 05:17 am
@ Star-Drive

We've battled the Nylon vs Teflon fasteners for years in our plasma chambers.  These days we replace the Nylon ones every run.  Teflon holds up very well, just won't take much mechanical load.

Notsosureofit:

We've fried a number of nylon bolts and have found that the best way to keep them from getting cooked is to keep them out of the high E-field regions in the cavity.  For Instance we tested the copper frustum in its TM010 mode and mounted a 5.0 inch OD by 1.0" thick PTFE disk at the center of the large OD end cap of the copper frustum with one 1/4-20 nylon bolt.  We got some large thrust signatures in that configuration, see attached slide, but the dam nylon bolt kept melting and dropping the PTFE discs into the main body of the cavity.  Brother did that look like a magnitude 9 earthquake on our uN resolution force measurement system! 

That said, I'm wondering if the nylon bolts themselves could be contributing to the measured force we are seeing?  It has a much smaller volume than the PE and PTFE discs, but they have a much higher dissipation factor than PE or PTFE that could translate into more work done converting E&M momentum into mechanical forces.

Best, Paul M.
Title: Re: EM Drive Developments - related to space flight applications - Thread 2
Post by: DIYFAN on 02/22/2015 05:21 am
... then many scientists tried to replicate their experiment but hopes fell with the large number of negative replications, and the withdrawal of many positive replications.

Sad but true.  There is, however, somewhat of a renaissance occurring right now with LENR+ (commercially viable LENR).  The current most promising method uses a mixture of nickel powder, iron powder, and LiAlH4 heated to 1100 C under pressure.  Multiple universities in the U.S. have recently opened programs with full funding to investigate LENR+.  Don't count this phenomena out entirely.  Coupled with an EM Drive, the space flight applications become very interesting.  Admittedly, controversy abounds, and it is still early to say what the probability of success and implications might be, but nonetheless, it doesn't hurt to keep a finger on the pulse on the most current efforts underway. 
Title: Re: EM Drive Developments - related to space flight applications - Thread 2
Post by: RotoSequence on 02/22/2015 05:35 am
We've fried a number of nylon bolts and have found that the best way to keep them from getting cooked is to keep them out of the high E-field regions in the cavity.  For Instance we tested the copper frustum in its TM010 mode and mounted a 5.0 inch OD by 1.0" thick PTFE disk at the center of the large OD end cap of the copper frustum with one 1/4-20 nylon bolt.  We got some large thrust signatures in that configuration, see attached slide, but the dam nylon bolt kept melting and dropping the PTFE discs into the main body of the cavity.

Are there low E-field regions across multiple resonance modes that can be used as mounting points to secure the PTFE discs?
Title: Re: EM Drive Developments - related to space flight applications - Thread 2
Post by: Rodal on 02/22/2015 12:02 pm
We've fried a number of nylon bolts and have found that the best way to keep them from getting cooked is to keep them out of the high E-field regions in the cavity.  For Instance we tested the copper frustum in its TM010 mode and mounted a 5.0 inch OD by 1.0" thick PTFE disk at the center of the large OD end cap of the copper frustum with one 1/4-20 nylon bolt.  We got some large thrust signatures in that configuration, see attached slide, but the dam nylon bolt kept melting and dropping the PTFE discs into the main body of the cavity.

Are there low E-field regions across multiple resonance modes that can be used as mounting points to secure the PTFE discs?

@Rotosequence and @Star-Drive

Excellent question by @Rotosequence.  Yes, for a given mode-shape there are E-field regions with very low E-Field that could be used as mounting points to secure the PTFE discs.  For example, for TM212, here they are (low E field is represented by the dark blue color):

(http://forum.nasaspaceflight.com/index.php?action=dlattach;topic=36313.0;attach=635196;image)

To answer whether they exist "across multiple resonance modes" , depends on the natural frequency being excited and the modes being excited.  If NASA could consistently excite a TE (transverse electric) mode instead of a TM (transverse magnetic) mode, there would be no electric field whatsoever on the entire surface of the flat ends.  So that's another advantage for exciting mode TE012 (if possible to be done consistently), which is the mode in the Brady et.al. report reported to have, by far, the largest thrust per power input.  Also, the experiments conducted by Prof. Juan Yang in China, that reached the highest thrust forces (by far) reported for an EM drive, were reported to be TE modes.

If NASA for the time being (due to the problems of consistently being able to excite a given mode) cannot consistently excite TE012 or other TE modes, and needs to continue exciting TM modes like TM212, then one possible solution would be to keep the antenna in exactly the same place for every test and place the bolts in the locations of minimum E-field.   Because the azimuthal (circumferential) location of the low electric field regions rotate in the azimuthal direction (with the center of the flat circular end as the axis of rotation) with changes in the azimuthal location of the antenna. With the antenna in a fixed location, and for a fixed frequency and mode shape, one could then mount the bolts in the locations of minimum E-field.
Title: Re: EM Drive Developments - related to space flight applications - Thread 2
Post by: aero on 02/22/2015 02:27 pm
Perhaps a more general solution to the problem of melting nylon bolts would be to search diligently for a material that does not melt.

While perhaps more difficult to obtain (read that as "custom manufacture") bolts can be made from almost any solid. I can just imagine Paul with his pen-knife carving a bolt from a wooden dowel rod. Or baking a ceramic bolt in this wife's kitchen oven.

Seriously though, the bolts problem is a materials issue. What are the necessary characteristics of the bolts, then what are the various materials that satisfy those specifications?

Now that we know the specifications, who wants to volunteer to make 4 bolts?

Edit Add: Or maybe there exists an epoxy (glue) that would serve to attach the dielectric disks?
Title: Re: EM Drive Developments - related to space flight applications - Thread 2
Post by: AnalogMan on 02/22/2015 03:02 pm
Perhaps a more general solution to the problem of melting nylon bolts would be to search diligently for a material that does not melt.

While perhaps more difficult to obtain (read that as "custom manufacture") bolts can be made from almost any solid. I can just imagine Paul with his pen-knife carving a bolt from a wooden dowel rod. Or baking a ceramic bolt in this wife's kitchen oven.

Seriously though, the bolts problem is a materials issue. What are the necessary characteristics of the bolts, then what are the various materials that satisfy those specifications?

Now that we know the specifications, who wants to volunteer to make 4 bolts?

Edit Add: Or maybe there exists an epoxy (glue) that would serve to attach the dielectric disks?

You can buy ceramic machine screws/bolts/nuts (if you can afford them!)

(for example: http://www.amazon.com/Ceramic-Machine-Finish-External-Threads/dp/B00DD44VPA (http://www.amazon.com/Ceramic-Machine-Finish-External-Threads/dp/B00DD44VPA))
Title: Re: EM Drive Developments - related to space flight applications - Thread 2
Post by: Rodal on 02/22/2015 03:22 pm
Please take into account, that as Paul March (@Star-Drive) stated:

the nylon bolts themselves could be contributing to the measured force we are seeing?  It has a much smaller volume than the PE and PTFE discs, but they have a much higher dissipation factor than PE or PTFE that could translate into more work done converting E&M momentum into mechanical forces.

Hence using ceramic bolts (if their cost can be afforded) could provide a straightforward way to test this possibility.
Title: Re: EM Drive Developments - related to space flight applications - Thread 2
Post by: Star-Drive on 02/22/2015 03:26 pm
All:

I tried epoxy and superglue bonding the PE and PTFE discs to the frustum end-caps, but these two plastics just happen to be the slickest and hardest plastics to bond to anything else.  Drat!  And yes I tried to find aluminum oxide or other low-loss dielectric ceramics bolts & nuts in long enough lengths (~2.50") to work in my application with no luck, but I'll admit I didn't look very hard at the time for I had many other things to do.

Aero: I'm not going to start carving out bolts from wood dowels! :)  But that does make me wonder if any wood would have a low enough dissipation factor at 2.0 GHz to be useable...  So many things to think about... 

Best,  Paul M.
Title: Re: EM Drive Developments - related to space flight applications - Thread 2
Post by: Rodal on 02/22/2015 03:30 pm
All:

I tried epoxy and superglue bonding the PE and PTFE discs to the frustum end-caps, but these two plastics just happen to be the slickest and hardest plastics to bond to anything else.  Drat!  And yes I tried to find aluminum oxide or other low-loss dielectric ceramics bolts & nuts in long enough lengths (~2.50") to work in my application with no luck, but I'll admit I didn't look very hard at the time for I had many other things to do.

Aero: I'm not going to start carving out bolts from wood dowels! :)  But that does make me wonder if any wood would have a low enough dissipation factor at 2.0 GHz to be useable...  So many things to think about... 

Best,  Paul M.

Concerning using an adhesive to adhere the HD PE to the copper,yes,  Polyolefin polymers in general are the most common type of hard to bond plastics because of their low surface energy.  HD PE is a Polyolefin polymer and hence hard to bond to due to its low surface energy.  Adhesion would involve surface preparation techniques as well.  Adhesion of copper to HD PE would involve: temperature, humidity, fixture time, chemicals (sometimes carcinogenic ones, particularly for the solvent primer), they can be messy. 

Most importantly, the adhesion would be permanent and removing the dielectric from the copper base plate would involve damage.

Surface roughening of the copper and of the HD PE results in mechanical interlocking sites and causes bond
strength to increase dramatically. A surface roughness of approximately 60-125 microinches is often used as a guideline for assemblies that are to be bonded with adhesives.  Shotblasting of the copper surface is usually needed.  This is an insurmountable problem for your extremely thin copper surface: there is practically no thickness of copper on the fiber-reinforced flat end to be able to roughen the copper.

Primers are solvent-based systems in which a reactive species is dissolved. Applied to a surface using a brush or spray, the primer’s solvent evaporates, leaving behind the reactive species on the substrate. The reactive species acts as a linking pin or bridge between an adhesive and the substrate Polyolefin primers are frequently used on hard-to-bond substrates joined with traditional and/or light curing cyanoacrylates.

Paul, I have experience with bonding hard to bond polymers, it can be done successfully, but it is not trivial, as you have also experienced.  This combined with the permanent nature of an adhesive, would also lead me to seek bolt alternatives, particularly for an R&D effort were you are going to have to dismantle, disassemble  and reconfigure your setup from time to time.
Title: Re: EM Drive Developments - related to space flight applications - Thread 2
Post by: Dagger on 02/22/2015 03:38 pm
What about Vespel Bolts? Temperature resistant up to 288°C (350°F) continuous and 480°C (900°F) intermittent.

These guys at Extreme Bolt & Fastener can custom make it:
http://www.extreme-bolt.com/Products-PI-Bolts.html

They can also custom make ceramic bolts.
Title: Re: EM Drive Developments - related to space flight applications - Thread 2
Post by: flux_capacitor on 02/22/2015 03:43 pm
Surface roughening of the copper and of the HD PE results in mechanical interlocking sites and causes bond
strength to increase dramatically. A surface roughness of approximately 60-125 microinches is often used as a guideline for assemblies that are to be bonded with adhesives.  Shotblasting of the copper surface is usually needed.  This is an insurmountable problem for your extremely thin copper surface: there is practically no thickness of copper on the fiber-reinforced flat end to be able to roughen the copper.

I see another problem apart from the wall being too thin: wouldn't roughening the internal face of the small copper plate (in order for the glue to stick better) dramatically decrease its reflecting capacity, hence the Q factor of the cavity?

Paul, I have experience with bonding hard to bond polymers, it can be done successfully, but it is not trivial, as you have also experienced.  This combined with the permanent nature of an adhesive, would also lead me to seek bolt alternatives, particularly for an R&D effort were you are going to have to dismantle, disassemble  and reconfigure your setup from time to time.

The beauty of this copper cavity lays in its extreme versatility. It's not a superconducting cavity machined from one piece of expensive material. The small end plate is just that: a thin copper plate, with a diameter of a few inches, attached to the end of the body with bolts at rim. Although I think like you that bolts would be better than glue to attach the internal dielectric, I also think even Eagleworks could afford several end plates, and test them for comparison: one small plate without any dielectric at all, one with a dielectric bonded with some glue as you suggested, another one fixed with bolts of any type, etc.
Title: Re: EM Drive Developments - related to space flight applications - Thread 2
Post by: Notsosureofit on 02/22/2015 03:44 pm
If you would like I can sputter coat the mating surface to the end plate w/ copper (or something that would make it easier to adhesive bond).  Only cost you the postage.....
Title: Re: EM Drive Developments - related to space flight applications - Thread 2
Post by: frobnicat on 02/22/2015 04:54 pm
Do the screws to fix the dielectric inside the frustum go through the end PCB plate ?

In this case, even if the hole in the copper is much smaller than the wavelength (in nylon ?), wouldn't that allow a significant amount of em energy to escape and bounce around between the frustum and vacuum chamber walls ? Aero's simulation with Meep seemed to say that evanescent waves could funnel through much thinner cracks, if not to let escape real travelling photons at least for near field interactions with things outside the frustum...
Title: Re: EM Drive Developments - related to space flight applications - Thread 2
Post by: zen-in on 02/22/2015 04:59 pm
All:

I tried epoxy and superglue bonding the PE and PTFE discs to the frustum end-caps, but these two plastics just happen to be the slickest and hardest plastics to bond to anything else.  Drat!  And yes I tried to find aluminum oxide or other low-loss dielectric ceramics bolts & nuts in long enough lengths (~2.50") to work in my application with no luck, but I'll admit I didn't look very hard at the time for I had many other things to do.

Aero: I'm not going to start carving out bolts from wood dowels! :)  But that does make me wonder if any wood would have a low enough dissipation factor at 2.0 GHz to be useable...  So many things to think about... 

Best,  Paul M.

The marine adhesive made by 3M  (5200 or 5220 fast cure) will stick to anything; even under water.   I have used it on HDP.  Most adhesives will just drop off once they have cured.   One trick I learned for gauging the dissipation factor of a plastic is to put it in a microwave oven at high power for 10 minutes; or less.  But I think the problem you are seeing is not from the dissipation factor of the plastic because your power level is relatively low.  Try putting one of the nylon bolts in a microwave and see if it melts.   What may be happening is the thin Copper coating is getting very hot and melting the nylon bolt.   The radiation pattern from the loop antenna inside the cavity directs most of the rf power to the large  end, irrespective of the mode.
Title: Re: EM Drive Developments - related to space flight applications - Thread 2
Post by: RotoSequence on 02/22/2015 05:08 pm
Do the screws to fix the dielectric inside the frustum go through the end PCB plate ?

In this case, even if the hole in the copper is much smaller than the wavelength (in nylon ?), wouldn't that allow a significant amount of em energy to escape and bounce around between the frustum and vacuum chamber walls ? Aero's simulation with Meep seemed to say that evanescent waves could funnel through much thinner cracks, if not to let escape real travelling photons at least for near field interactions with things outside the frustum...

Assuming something similar hasn't been done already, it should be possible to test for photon emissions by shortening the nylon bolts and securing the outside ends with cap nuts.

Title: Re: EM Drive Developments - related to space flight applications - Thread 2
Post by: wembley on 02/22/2015 06:53 pm
... then many scientists tried to replicate their experiment but hopes fell with the large number of negative replications, and the withdrawal of many positive replications.

Sad but true.  There is, however, somewhat of a renaissance occurring right now with LENR+ (commercially viable LENR).  The current most promising method uses a mixture of nickel powder, iron powder, and LiAlH4 heated to 1100 C under pressure.  Multiple universities in the U.S. have recently opened programs with full funding to investigate LENR+.  Don't count this phenomena out entirely.  Coupled with an EM Drive, the space flight applications become very interesting.  Admittedly, controversy abounds, and it is still early to say what the probability of success and implications might be, but nonetheless, it doesn't hurt to keep a finger on the pulse on the most current efforts underway.

Amusingly enough, some of the LENR work is taking place at NASA -- in a very low-level, part-time-project fashion.

The comparisons between developments in the two are quite striking -- mainstream skepticism, lack of a solid theoretical base, amazing claims by one-man-show inventors, and steady drip-feed of 'anomalous' results. Which of course, doesn't mean that either is real.
Title: Re: EM Drive Developments - related to space flight applications - Thread 2
Post by: Star-Drive on 02/22/2015 07:20 pm
If you would like I can sputter coat the mating surface to the end plate w/ copper (or something that would make it easier to adhesive bond).  Only cost you the postage.....

Notsosureofit:

Send me a private note with your name and address and I'll gladly send you a PE and PTFE disc to have them sputter copper coated on one flat side only in your lab.  When returned we can then silver epoxy or even Pb/Sn solder the discs to a new set of 0.090" thick small OD copper end-caps we have sitting in the lab.  And thanks much for the offer!

Best, Paul M.
Title: Re: EM Drive Developments - related to space flight applications - Thread 2
Post by: Star-Drive on 02/22/2015 07:25 pm
All:

I tried epoxy and superglue bonding the PE and PTFE discs to the frustum end-caps, but these two plastics just happen to be the slickest and hardest plastics to bond to anything else.  Drat!  And yes I tried to find aluminum oxide or other low-loss dielectric ceramics bolts & nuts in long enough lengths (~2.50") to work in my application with no luck, but I'll admit I didn't look very hard at the time for I had many other things to do.

Aero: I'm not going to start carving out bolts from wood dowels! :)  But that does make me wonder if any wood would have a low enough dissipation factor at 2.0 GHz to be useable...  So many things to think about... 

Best,  Paul M.

The marine adhesive made by 3M  (5200 or 5220 fast cure) will stick to anything; even under water.   I have used it on HDP.  Most adhesives will just drop off once they have cured.   One trick I learned for gauging the dissipation factor of a plastic is to put it in a microwave oven at high power for 10 minutes; or less.  But I think the problem you are seeing is not from the dissipation factor of the plastic because your power level is relatively low.  Try putting one of the nylon bolts in a microwave and see if it melts.   What may be happening is the thin Copper coating is getting very hot and melting the nylon bolt.   The radiation pattern from the loop antenna inside the cavity directs most of the rf power to the large  end, irrespective of the mode.

Zen-In:

Thanks for the recommendation and I'll give this 3M marine adhesive a try to see how it might work with difficult to bond plastics like PE and PTFE, hopefully without ALL the preparation outlined by Rodal.

Best, Paul, M.
Title: Re: EM Drive Developments - related to space flight applications - Thread 2
Post by: Mulletron on 02/22/2015 07:47 pm
http://www.tara.tcd.ie/handle/2262/38886
http://www.ncbi.nlm.nih.gov/pubmed/16711970
http://journals.aps.org/prl/abstract/10.1103/PhysRevLett.96.130402

Quote
A recent publication [Phys. Rev. Lett. 92, 020404 (2004)PRLTAO0031-900710. 1103/PhysRevLett.92.020404] raises the possibility of momentum transfer from zero-point quantum fluctuations to matter, controlled by applied electric and magnetic fields. We present a Lorentz-invariant description using field-theoretical regularization techniques. We find no momentum transfer for homogeneous media, but predict a very small transfer for a Casimir-type geometry.

An oldie but goodie. These are the older papers. Note the bold part.

I bet that having those PE discs smashed up against each other and the copper is important! From what Paul reported, it is. Here's why I think so:
http://forum.nasaspaceflight.com/index.php?topic=36313.msg1329454#msg1329454
http://forum.nasaspaceflight.com/index.php?topic=36313.msg1329531#msg1329531
Quoting me.
Quote
About half the time when I think this through, the darn thing thrusts backwards.

Quoting @Paul March http://forum.nasaspaceflight.com/index.php?topic=36313.msg1335190#msg1335190
Quote
.....it was producing less than half of what it did before and in the wrong direction!......Apparently not having the PE discs firmly mounted to the frustum's small OD end cap hindered the thrust producing mechanism that conveys the generated forces in the PE to the copper frustum.

Paul, would you be willing to introduce a small gap between the copper and PE as a test? Like tighten down the Nylon bolts and wedge a wooden shim in there?
Also, I know this is kinda high speed, but do you have any gold leaf? To wedge between the PE and Copper? http://en.wikipedia.org/wiki/Gold_leaf#Culinary_uses
The reason I'm saying all this is, if we're gonna call these things Quantum Vacuum thrusters, we need to treat it like a Casimir experiment, instead of a plasma thruster.

Break:
IRT the new stuff I've been posting about PT symmetry, this is a nice plain english rundown of how it works from different people, saying the same thing as the other researchers.
http://www.lap.physik.uni-erlangen.de/lap/?page=research_krstic_chiral&language=en (Also see those references at the bottom, this is very exciting stuff. I think I may actually be right on this one ;) )

And onto the subject of the Nylon bolts:
So there's a bunch of different types of nylon. Anyone know what kind of nylon those bolts might be made of? Wouldn't it be something if nylon was doing some of the thrusting the whole time.
Is it these? http://www.mcmaster.com/#nylon-6/6-screws/=w0xvlx

I feel really goofy right now holding a mirror up to the computer screen.
Title: Re: EM Drive Developments - related to space flight applications - Thread 2
Post by: Mulletron on 02/22/2015 08:29 pm
@ Star-Drive

We've battled the Nylon vs Teflon fasteners for years in our plasma chambers.  These days we replace the Nylon ones every run.  Teflon holds up very well, just won't take much mechanical load.

Notsosureofit:

We've fried a number of nylon bolts and have found that the best way to keep them from getting cooked is to keep them out of the high E-field regions in the cavity.  For Instance we tested the copper frustum in its TM010 mode and mounted a 5.0 inch OD by 1.0" thick PTFE disk at the center of the large OD end cap of the copper frustum with one 1/4-20 nylon bolt.  We got some large thrust signatures in that configuration, see attached slide, but the dam nylon bolt kept melting and dropping the PTFE discs into the main body of the cavity.  Brother did that look like a magnitude 9 earthquake on our uN resolution force measurement system! 

That said, I'm wondering if the nylon bolts themselves could be contributing to the measured force we are seeing?  It has a much smaller volume than the PE and PTFE discs, but they have a much higher dissipation factor than PE or PTFE that could translate into more work done converting E&M momentum into mechanical forces.

Best, Paul M.

Look at the Comsol plot of TM010 with PTFE at large end showing large thrust. Note the strong magnetic field in that region. Do I need to bring Colbert back again?  ;)
(http://forum.nasaspaceflight.com/index.php?action=dlattach;topic=36313.0;attach=804661;image)
Title: Re: EM Drive Developments - related to space flight applications - Thread 2
Post by: Star-Drive on 02/22/2015 11:17 pm
http://www.tara.tcd.ie/handle/2262/38886
http://www.ncbi.nlm.nih.gov/pubmed/16711970
http://journals.aps.org/prl/abstract/10.1103/PhysRevLett.96.130402

Quote
A recent publication [Phys. Rev. Lett. 92, 020404 (2004)PRLTAO0031-900710. 1103/PhysRevLett.92.020404] raises the possibility of momentum transfer from zero-point quantum fluctuations to matter, controlled by applied electric and magnetic fields. We present a Lorentz-invariant description using field-theoretical regularization techniques. We find no momentum transfer for homogeneous media, but predict a very small transfer for a Casimir-type geometry.

An oldie but goodie. These are the older papers. Note the bold part.

I bet that having those PE discs smashed up against each other and the copper is important! From what Paul reported, it is. Here's why I think so:
http://forum.nasaspaceflight.com/index.php?topic=36313.msg1329454#msg1329454
http://forum.nasaspaceflight.com/index.php?topic=36313.msg1329531#msg1329531
Quoting me.
Quote
About half the time when I think this through, the darn thing thrusts backwards.

Quoting @Paul March http://forum.nasaspaceflight.com/index.php?topic=36313.msg1335190#msg1335190
Quote
.....it was producing less than half of what it did before and in the wrong direction!......Apparently not having the PE discs firmly mounted to the frustum's small OD end cap hindered the thrust producing mechanism that conveys the generated forces in the PE to the copper frustum.

Paul, would you be willing to introduce a small gap between the copper and PE as a test? Like tighten down the Nylon bolts and wedge a wooden shim in there?
Also, I know this is kinda high speed, but do you have any gold leaf? To wedge between the PE and Copper? http://en.wikipedia.org/wiki/Gold_leaf#Culinary_uses
The reason I'm saying all this is, if we're gonna call these things Quantum Vacuum thrusters, we need to treat it like a Casimir experiment, instead of a plasma thruster.

Break:
IRT the new stuff I've been posting about PT symmetry, this is a nice plain english rundown of how it works from different people, saying the same thing as the other researchers.
http://www.lap.physik.uni-erlangen.de/lap/?page=research_krstic_chiral&language=en (Also see those references at the bottom, this is very exciting stuff. I think I may actually be right on this one ;) )

And onto the subject of the Nylon bolts:
So there's a bunch of different types of nylon. Anyone know what kind of nylon those bolts might be made of? Wouldn't it be something if nylon was doing some of the thrusting the whole time.

I feel really goofy right now holding a mirror up to the computer screen.

Mulletron:

Treat this thing as a large collection of Casimir Cavities, might work if I can convince Dr. White to do so, and only after we both read through your referenced papers on the topic.  And oh yes, get to Glenn Research Center for a successful replication of what we've seen to date before the end of March, or I may find myself in retirement before I was ready...

As to the source of my nylon cap screws it is http://www.mcmaster.com/#nylon-cap-screws/=w0xvi2.  As to what plastics vendor they use for their nylon supplier and what type of nylon is used in their cap-screws and bolts is TBD.

PS: How big a gap are you thinking about between the PE and copper plate?  Microns or mm...??

Best,  Paul M.     
Title: Re: EM Drive Developments - related to space flight applications - Thread 2
Post by: Mulletron on 02/22/2015 11:26 pm
Just a few mm on one side, like what you get from a spudger tip. To be clear, the gap is intended to break the thrust. From what you're saying, it would break the thrust. It serves to provide a clear answer whether Casimir forces play any part in the thrust or not, or if the PT momentum transfer/anisotropic vacuum idea is enough.

What I picture going on is that the Casimir forces at the PE/air/copper interface isn't the sole enabler of thrust, it is a major amplifying feature of the thrust.

Let's say that there is a few mm gap between the PE and the copper all around. If that breaks the thrust, then it is clear Casimir forces are important.
(http://upload.wikimedia.org/wikipedia/commons/thumb/8/87/Spudgers.jpg/220px-Spudgers.jpg)

Well anyway, this is neat to watch. https://nanohub.org/resources/21370/watch?resid=21371
Title: Re: EM Drive Developments - related to space flight applications - Thread 2
Post by: Rodal on 02/22/2015 11:49 pm
....

Treat this thing as a large collection of Casimir Cavities, might work if I can convince Dr. White to do so, and only after we both read through your referenced papers on the topic.  And oh yes, get to Glenn Research Center for a successful replication of what we've seen to date before the end of March, or I may find myself in retirement before I was ready...

..

I put the emphasis on:

 get to Glenn Research Center for a successful replication of what we've seen to date before the end of March, or I may find myself in retirement

That would be a horrible waste of human talent and experience, somebody extremely difficult to replace, particularly after demonstrating that the EM Drive works in a hard vacuum, and given the better funded efforts in China, that the US should match or exceed.

Those who think that the EM Drive technology is being pursued "under wraps" in the US appear to  be misinformed and disconnected from Aerospace Companies Mgt and federally-funded R&D.

There is a shortfall of funds at NASA, and federally-funded R&D in general under present budget conditions.

There is barely more than one month left until the end of March.

People interested in the EM Drive: this shortfall of funds is real, and EM Drive R&D in the US is in clear and present danger.

(http://images.bwbx.io/cms/2014-11-26/pol_researchchart49_315.jpg)

(https://leadingspace.files.wordpress.com/2010/10/budget3.png)
Title: Re: EM Drive Developments - related to space flight applications - Thread 2
Post by: DIYFAN on 02/23/2015 02:51 am
....

Treat this thing as a large collection of Casimir Cavities, might work if I can convince Dr. White to do so, and only after we both read through your referenced papers on the topic.  And oh yes, get to Glenn Research Center for a successful replication of what we've seen to date before the end of March, or I may find myself in retirement before I was ready...

..

I put the emphasis on:

 get to Glenn Research Center for a successful replication of what we've seen to date before the end of March, or I may find myself in retirement

That would be a horrible waste of human talent and experience, somebody extremely difficult to replace, particularly after demonstrating that the EM Drive works in a hard vacuum, and given the better funded efforts in China, that the US should match or exceed.

Those who think that the EM Drive technology is being pursued "under wraps" in the US appear to  be misinformed and disconnected from Aerospace Companies Mgt and federally-funded R&D.

There is a shortfall of funds at NASA, and federally-funded R&D in general under present budget conditions.

There is barely more than one month left until the end of March.

People interested in the EM Drive: this shortfall of funds is real, and EM Drive R&D in the US is in clear and present danger.

(http://images.bwbx.io/cms/2014-11-26/pol_researchchart49_315.jpg)

(https://leadingspace.files.wordpress.com/2010/10/budget3.png)

By the looks of things, there will be another "sputnik" moment--but this time it will be the Chinese not the Russians, and the EM Drive not the satellite.  As much as I'd like to see the US stay out ahead on this one, if the new sputnik-EM Drive-China moment needs to happen to revive the support of the US government and spur a corresponding rise in NASA's budget, then let's hope it happens, and soon.
Title: Re: EM Drive Developments - related to space flight applications - Thread 2
Post by: Mulletron on 02/23/2015 03:06 am

By the looks of things, there will be another "sputnik" moment--but this time it will be the Chinese not the Russians, and the EM Drive not the satellite.  As much as I'd like to see the US stay out ahead on this one, if the new sputnik-EM Drive-China moment needs to happen to revive the support of the US government and spur a corresponding rise in NASA's budget, then let's hope it happens, and soon.

Well if the reports from China are true, if we don't roll out something impressive soon, they already beat us to the punch.  :(


(https://encrypted-tbn0.gstatic.com/images?q=tbn:ANd9GcSy7QjdoZl-0e9HJLqKx4pkexxWDGg_7Km6Qr1eZ_kR_p28tiY0RCN0Fcrcfg)
http://www.economist.com/node/11791539
Title: Re: EM Drive Developments - related to space flight applications - Thread 2
Post by: aceshigh on 02/23/2015 03:50 am
What are the reports coming from China? I guess I missed the links
Title: Re: EM Drive Developments - related to space flight applications - Thread 2
Post by: Star-Drive on 02/23/2015 04:04 am
Dr. Rodal:

Thanks for the concern over my employment, but everyone is replaceable to one degree or another.  And there are at least two federal civil servants involved with the Eagleworks Lab that will carry on, albeit a bit more slowly, if I have to find other activities to fill my golden years.  Activities like finishing my home lab that has been on hold since going back to work at the NASA/JSC/Eagleworks Lab for the last four years helping mature Dr. White's QVF vision.  What will be, will be.  Now back to trying to understand how these EM-Drives might actually work...

Best, Paul M.
Title: Re: EM Drive Developments - related to space flight applications - Thread 2
Post by: Rodal on 02/23/2015 02:58 pm
Dr. Rodal:

Thanks for the concern over my employment, but everyone is replaceable to one degree or another.  And there are at least two federal civil servants involved with the Eagleworks Lab that will carry on, albeit a bit more slowly, if I have to find other activities to fill my golden years.  Activities like finishing my home lab that has been on hold since going back to work at the NASA/JSC/Eagleworks Lab for the last four years helping mature Dr. White's QVF vision.  What will be, will be.  Now back to trying to understand how these EM-Drives might actually work...

Best, Paul M.

"Experience without theory is blind, but theory without experience is mere intellectual play."

Immanuel Kant
Title: Re: EM Drive Developments - related to space flight applications - Thread 2
Post by: lasoi on 02/23/2015 03:12 pm

By the looks of things, there will be another "sputnik" moment--but this time it will be the Chinese not the Russians, and the EM Drive not the satellite.  As much as I'd like to see the US stay out ahead on this one, if the new sputnik-EM Drive-China moment needs to happen to revive the support of the US government and spur a corresponding rise in NASA's budget, then let's hope it happens, and soon.

Well if the reports from China are true, if we don't roll out something impressive soon, they already beat us to the punch.  :(

(https://encrypted-tbn0.gstatic.com/images?q=tbn:ANd9GcSy7QjdoZl-0e9HJLqKx4pkexxWDGg_7Km6Qr1eZ_kR_p28tiY0RCN0Fcrcfg)
http://www.economist.com/node/11791539
It's almost as though the NASA establishment is reluctant to put too many resources into it for fear of looking bad if it doesn't pan out.  You hope that isn't the case.  If it doesn't pan out it doesn't pan out.  But you don't have to be a rocket scientist to see the potential if it does happen to work.  I've read that NASA doesn't or can't accept crowdfunding, which is a shame if true- and a bit odd since its entire budget is basically crowd funding through tax revenues.
Title: Re: EM Drive Developments - related to space flight applications - Thread 2
Post by: MikeMcCulloch on 02/23/2015 03:21 pm
Dear excellent NSF forum. This is just to say (for those who don't know) that I've suggested a specific new model for inertia that predicts galaxy rotation without dark matter (it is called MiHsC) and I recently compared its predictions with the 9 EmDrive results with 'some' (not perfect) success. You can see the results by looking at the Table here:

http://physicsfromtheedge.blogspot.co.uk/2015/02/mihsc-vs-emdrive-data-3d.html

I've also published a paper summarising this comparison (slightly out of date now) here

http://www.ptep-online.com/index_files/2015/PP-40-15.PDF

I'd encourage those with other interesting explanations of the EmDrive to make a similar comparison between predictions and data, so we can compare using the facts. If you have any more data points to add, or if you disagree with the numbers in my Table, please let me know.
Title: Re: EM Drive Developments - related to space flight applications - Thread 2
Post by: Rodal on 02/23/2015 03:47 pm
Dear excellent NSF forum. This is just to say (for those who don't know) that I've suggested a specific new model for inertia that predicts galaxy rotation without dark matter (it is called MiHsC) and I recently compared its predictions with the 9 EmDrive results with 'some' (not perfect) success. You can see the results by looking at the Table here:

http://physicsfromtheedge.blogspot.co.uk/2015/02/mihsc-vs-emdrive-data-3d.html

I've also published a paper summarising this comparison (slightly out of date now) here

http://www.ptep-online.com/index_files/2015/PP-40-15.PDF

I'd encourage those with other interesting explanations of the EmDrive to make a similar comparison between predictions and data, so we can compare using the facts. If you have any more data points to add, or if you disagree with the numbers in my Table, please let me know.
Yes, as (arguably) the greatest Mathematician/Physicist in recent times (John Von Neumann) wrote:

Quote from: John Von Neumann
The sciences do not try to explain, they hardly even try to interpret, they mainly make models. By a model is meant a mathematical construct which, with the addition of certain verbal interpretations, describes observed phenomena. The justification of such a mathematical construct is solely and precisely that it is expected to work.

Papers with theories that can't predict anything, according to Von Neumann, are not science, because science is all about mathematical models that precisely describe the observed phenomena.

However people may find rigorous fault with any of them, presently there are only three publicly disclosed mathematical formulations that predict a given magnitude of thrust for the EM Drive: Shawyer's, McCulloch's and Dr. Notsosureofit's.

The formulation by Dr. Notsosureofit is the only one known to incorporate the electromagnetic mode shapes of operation.

Here is the formulation of Dr. "Notsosureofit" and a table of results.

@ RODAL

Arrgh, Mondays !

Looked over my bleary weekend, noticed I was using diameters AGAIN !

Mode   Frequency (MHz)  Quality Factor, Q   Input Power (W)  Mean Thrust (μN)   Calculated w/o
                                                                                                                          dielectric
TE012     1880.4               22000                         2.6                55.4                   10.8
TM212   1932.6                 7320                        16.9                91.2                   38.5
TM212   1936.7               18100                        16.7                50.1                   93.5
TM212    1937.115             6726                       50                   66                    104.0

Anyway, shows it pays to rewrite everything in the same place !

....

Great !

In order to understand the above, (please correct me if I am wrong), you used in your formula the actual frequency and mode shapes that took place in the EM Drive experiment with the dielectric so in that sense you did calculate with the dielectric in a very restricted sense.


FYI

Cleanup and de-typo of the take on applying the Equivalence Principle.


The proposition that dispersion caused by an accelerating frame of reference implied an accelerating frame of reference caused by a dispersive cavity resonator. (to 1st order using massless, perfectly conducting cavity, no dielectric)


Starting with the expressions for the frequency of a cylindrical RF cavity:

f = (c/(2*Pi))*((X/R)^2+((p*Pi)/L)^2)^.5

For TM modes, X = X[sub m,n] = the n-th zero of the m-th Bessel function.
[1,1]=3.83, [0,1]=2.40, [0,2]=5.52 [1,2]=7.02, [2,1]=5.14, [2,2]=8.42, [1,3]=10.17, etc.

and for TE modes, X = X'[sub m,n] = the n-th zero of the derivative of the m-th Bessel function.
[0,1]=3.83, [1,1]=1.84, [2,1]=3.05, [0,2]=7.02, [1,2]=5.33, [1,3]=8.54, [0,3]=10.17, [2,2]=6.71, etc.

Rotate the dispersion relation of the cavity into Doppler frame to get the Doppler shifts, that is to say, look at the dispersion curve intersections of constant wave number instead of constant frequency.

df = (1/(2*f))*(c/(2*Pi))^2*X^2*((1/Rs^2)-(1/Rb^2))

and from there the expression for the acceleration g from:

g = (c^2/L)*(df/f) such that:

g = (c^2/(2*L*f^2))*(c/(2*Pi))^2*X^2*((1/Rs^2)-(1/Rb^2))

Using the "weight" of the photon in the accelerated frame from:

"W" = (h*f/c^2)*g =>  "W" = T = (h/L)*df

gives thrust per photon:

T = (h/(2*L*f))*(c/(2*pi))^2*X^2*((1/Rs^2)-(1/Rb^2))

If the number of photons is (P/hf)*(Q/2*pi) then:

NT = P*Q*(1/(4*pi*L*f^3))*(c/(2*pi))^2*X^2*((1/Rs^2)-(1/Rb^2))


This does fit (as far as I've gotten) the concept of a self-accelerating Dirac wavepacket (which does conserve momentum).

Slow goin', thanks for your patience.

Excellent!  Thank you for posting the complete equations.

One suggestion:  In the expression NT = P*Q*(1/(4*pi*L*f^3))*(c/(2*pi))^2*X^2*((1/Rs^2)-(1/Rb^2))

the speed of light in vacuum "c" appears in the numerator without being divided by the SquareRoot of the relative electric permittivity and relative magnetic permeability.

Since the relative electric permittivity of the dielectric is 2.3, this would decrease the values in the table by a factor of Sqrt[2.3]=1.52 if the whole cavity would be occupied by the dielectric.  Granted that only a portion of the truncated cone contains the dielectric, which will decrease the dividing factor, but any amount will reduce the effective value of c in the medium, giving lower thrust and hence values closer to the experimental measurements. 

For example, very roughly, assuming that 1/3 of the longitudinal length is occupied by the dielectric, and using the average as a medium with those average properties, Sqrt[(2.3*1/3)+1*(2/3)]=1.20, the thrust values would be reduced by a factor of 1.20, so for the most important test (the one in recently performed in vacuum, -the other experimental values may have been affected by thermal convection effects in the air and are therefore less reliable-), instead of 104 μN you would get 87 μN, which better compares with the experimental value of 66 μN.  
Title: Re: EM Drive Developments - related to space flight applications - Thread 2
Post by: Rodal on 02/23/2015 08:53 pm
This thread has resulted in great synergy between NSF contributors and Paul March at NASA, and I wanted to take this opportunity to name a partial list of accomplishments and thank everybody that has contributed -including a few humble persons who privately asked me not to list their numerous valuable contributions -(please forgive me if I am missing important contributions, tell me what I missed, or if you disagree, what I should correct):

Consideration was made of whether the experimental measurements of thrust force were the result of an artifact.  Dr. Rodal, one of the contributors, solved the nonlinear, fully coupled system of differential equations (including magnetic damping) of an inverted torque pendulum with Mathematica  to examine whether parasitic modes or nonlinear dynamics could be involved.  Chaotic motion and strange attractors were also examined.  Comparison with the experimental results showed that none of these nonlinear dynamics effects were involved in the experimental measurements at NASA Eagleworks, and therefore a nonlinear dynamics cause was eliminated.  Dr. Rodal (in the US) also conducted Power Spectral Density and Autocorrelation analysis of NASA's experimental data and worked with another contributor, @frobnicat, (in France) to examine the dynamics of the experimental response.  They concluded that indeed NASA's experimental measurements exhibited the expected response of NASA's torque pendulum as excited by an initial thrust force impulse followed by a thrust force response during the 30 to 40 sec length of the experiments.   Dr. Rodal  analyzed possible thermal instability (thermal buckling of the flat ends) as a cause for the measured thrust and reported this at NSF and at ResearchGate (https://www.researchgate.net/publication/268804028_NASA%27S_MICROWAVE_PROPELLANT-LESS_THRUSTER_ANOMALOUS_RESULTS_CONSIDERATION_OF_A_THERMO-MECHANICAL_EFFECT).   A thermo-mechanical effect (thermal buckling) is shown that occurs in less than 1 second (for the copper thickness employed for the microwave cavity), with a temperature increase of a degree C or less and that results in forces of the same magnitude as reportedly measured by NASA.  Moreover, this thermal instability produces forces in the same direction as measured, and it will occur in a vacuum (since the heating can be due either to induction heating from the axial magnetic field in a TE mode or resistive heating due to the axial electric field in a TM mode).  However, this effect can only explain the initial impulsive force and cannot explain the longer 30 to 40 second measured force.  Thus the thrust force measured for up to 40 second is not nullified by this explanation either.

Thermal expansion effect as posited by a team from Oak Ridge National Labs for another propellant-less set of experiments was also eliminated as a possible source by the NSF contributors because it would result in forces in the complete opposite direction as the forces measured by NASA.

One of the participants in the NSF forum is Dr. McCulloch (an academic from the UK), who independently developed a tentative theoretical explanation for the EM Drive: assuming that photons have inertial mass, which is caused by Unruh radiation, whose wavelengths must fit inside the EM Drive cone, more Unruh waves fit in at the wide end of the EM Drive, so photons traveling along the axis would always gain mass going towards the wide end and lose it going the other way.  This is equivalent to expelling mass towards the wide end, so the EM Drive must move towards its narrow end to conserve momentum.  This agrees with the (forward) direction of movement of the EM Drive in reported NASA experiments.  Dr. McCulloch derived a simple formula to predict the thrust force and published his theory in the journal Progress in Physics (http://www.ptep-online.com/index_files/2015/PP-40-15.PDF).  In his blog and published paper he acknowledges the help from NSF participants (@aero, and @Fornaro) in estimating the geometrical dimensions of the EM Drives tested in the US, UK, and China.

Another participant in the NSF forum, @Notsosureofit (Ph.D. Physics) developed and posted an analysis of the EM Drive thrust considering an accelerating frame of reference caused by a dispersive cavity resonator and obtained a formula to predict the thrust of the EM Drive that takes into account the electromagnetic mode shape of the EM Drive (unlike the formulas of McCulloch and Shawyer that do not explicitly include mode shape information).

@frobnicat and Dr. Rodal conducted statistical analyses of the experimental data.  @frobnicat wrote a computer program that included hundreds of possible combinations of the experimental parameters (such as power input, frequency, Q (quality factor of resonance), geometrical dimensions, etc.) to the first few powers.  Interestingly the best fitting formulas were similar to the theoretically derived formula by Notsosureofit and also McCulloch's formula.

Astrophysicist TMEubanks examined whether the EM Drive could be coupling to the (Dark Matter) Axion background.  He concluded that this is very unlikely (by up to 20 orders of magnitude) due to the findings of the Axion Dark-Matter experiment, looking for yoctowatts (10^-24) of RF power in the 2 - 20 micro-eV range, precisely the range of the EM Drive, by tuning the cavity's resonant frequency to the axion mass. There is simply no way that the Drive is coupling to the axion background - the ADMX would see a whopping signal.

Dr. Rodal obtained an exact solution for the electromagnetic modes in a cavity with similar geometry as the NASA's EM Drive using Mathematica and the theory of spherical waves developed by the Russian/American scientist Schelnukoff.  The resulting equations are very similar to the ones posted by Greg Egan (http://gregegan.customer.netspace.net.au/SCIENCE/Cavity/Cavity.html) .  Dr. Rodal calculated the natural frequency for electromagnetic modes tested in the EM Drive, from this exact solution, to compare with the NASA predictions by Frank Davies NASA/JSC/EP5 using Finite Element Analysis with the computer code COMSOL.  The exact solution results calculated by Dr. Rodal at the NSF forum are only 1% different from the NASA calculations using COMSOL.  This confirms the validity of NASA's COMSOL analysis, and that the finite element mesh used was fine enough to result in predicted frequencies that are less than 1% from the exact solution, hence confidence can be had on those calculations.  The validity of NASA's COMSOL calculations has been simultaneously confirmed by experimental comparison with the IR thermal camera image produced for mode shape TM212.

NSF member @aero is using MIT's Finite Difference computer code MEEP to calculate the force produced by evanescent waves escaping from the EM Drive and interacting with the stainless steel vacuum chamber.  This work is in progress.

Despite considerable effort at NSF to dismiss the reported thrust as an artifact the EM Drive results have yet to be falsified.  After consistent reports of thrust measurements from EM Drive experiments in the US, UK, and China, at thrust levels several thousand times in excess of a photon rocket, and now under hard vacuum conditions, the question of where the thrust is coming from deserves serious inquiry.
Title: Re: EM Drive Developments - related to space flight applications - Thread 2
Post by: aero on 02/23/2015 10:52 pm
@Rodal - You get 3 "atta boys" for that post.  ;D
Title: Re: EM Drive Developments - related to space flight applications - Thread 2
Post by: Flyby on 02/23/2015 11:02 pm
Nice to have such an overview.
It helps to understand the sometimes overwhelming flood of information and the often complex and difficult to understand discussion points.
A sincere thanks for that, Dr. Rodal...(and all other contributors ofc)
Title: Re: EM Drive Developments - related to space flight applications - Thread 2
Post by: aero on 02/23/2015 11:04 pm
Since Dr. March told us about the broken bolt holding the dielectric disks together, meep has been concerned with how such breaks might effect the forces detected in the simulation. The attached curves show the detected forces with broken dielectric compared to a nominal case.

All three graphs include the nominal case (labeled "Rim gap") where the dielectric is intact but the gap in metal/metal contact between the cone body and the copper end plate varies. Due to computer issues, this gap is much larger than any gap that occurs in the real hardware so the Force/Power shown are only relative to each other, not close in magnitude to the real device.

In all cases the gap varies over a fraction of the thruster height, from 0.003 to 0.01 times height. In the runs made with gaps in the dielectric, "Rim gap" was set equal to 0.004, hence if computer resolution would allow the dielectric gap to go to zero, one would expect the curve to reach the value of approximately -0.0752378657. As is evident, the curves are far from converging to this value.
 

The first case, the nylon bolt is loose so the gap is between the two dielectric disks. This curve is labeled "Broken Bolt."

The second case, the dielectric is intact but has came loose from the copper end plate. This curve is labeled "Gap at CU face."

The third case is not very meaningful but interesting none-the-less. This curve is labeled "Break=rim gap" and was made in error, with the two dielectric disks separated by an amount equal to the gap in the metal to metal contact of the cone body and the end plate. Both gaps varied from 0.003 to 0.01 times the thruster height. I include it here with the thought that sometimes accidents tell us more than successful plans, though I don't know what this accident is saying.

Title: Re: EM Drive Developments - related to space flight applications - Thread 2
Post by: sanman on 02/23/2015 11:09 pm
This stuff is all an interesting thought exercise, but there's no way it's a Sputnik moment when bulk of the physics community are scoffing at it. Just as with the Hafnium controversy (http://en.wikipedia.org/wiki/Hafnium_controversy), it would be nice to see the mainstream physics community roped in to at least do an authoritative disproof on it, rather than leaving things to linger on under imagination and speculation.

Title: Re: EM Drive Developments - related to space flight applications - Thread 2
Post by: Notsosureofit on 02/23/2015 11:45 pm
The hafnium controversy never had any replication .....  somewhat different puzzle from either "cold fusion" or EMDrive.
Title: Re: EM Drive Developments - related to space flight applications - Thread 2
Post by: Mulletron on 02/24/2015 12:00 am
This stuff is all an interesting thought exercise, but there's no way it's a Sputnik moment when bulk of the physics community are scoffing at it. Just as with the Hafnium controversy (http://en.wikipedia.org/wiki/Hafnium_controversy), it would be nice to see the mainstream physics community roped in to at least do an authoritative disproof on it, rather than leaving things to linger on under imagination and speculation.

Indeed, the goal of many of us on here was/is to try and explain away the reports of thrust from Emdrive, and if we could not, find a more plausible explanation for the thrust, and try to convince others to at least attempt a replication.

To date, Emdrive has yet to be falsified, a few plausible models have been suggested.....the final goal is yet to be accomplished.
Title: Re: EM Drive Developments - related to space flight applications - Thread 2
Post by: Stormbringer on 02/24/2015 12:43 am
I thought somewhere Dr March had said that GRS's balance sensitivity required an improvement of the thrust signal by 100 percent in order to get successful replication. Did I get that right? I cannot find it now and I have been looking back through the thread for 20 minutes or more. -Anyway; with a deadline of EOM, march; does that mean there is an improved test article about to or in the process of undergoing additional testing at Eagleworks? and can someone link to where that fact (GRS sensitivity limits incapable of detecting current thrust signal level) was posted for reference?
Title: Re: EM Drive Developments - related to space flight applications - Thread 2
Post by: Mulletron on 02/24/2015 12:45 am
http://forum.nasaspaceflight.com/index.php?topic=36313.msg1326608#msg1326608
(http://forum.nasaspaceflight.com/index.php?action=dlattach;topic=36313.0;attach=804661;image)
Looks like they're almost there.
Title: Re: EM Drive Developments - related to space flight applications - Thread 2
Post by: Stormbringer on 02/24/2015 12:46 am
http://forum.nasaspaceflight.com/index.php?topic=36313.msg1326608#msg1326608
thank you :)
Title: Re: EM Drive Developments - related to space flight applications - Thread 2
Post by: Notsosureofit on 02/24/2015 01:22 am
Anyway, still thinking about a possible experiment in vacuum using:

https://www.dropbox.com/s/cmxyj7re8rrb6dh/IMAG0361.jpg?dl=0

to make a stack chamber like:

https://www.dropbox.com/s/wf8wv226h8138n1/IMAG0360.jpg?dl=0

but on an isolation stand.

The idea would be to then balance a tapered oscillator (Gunn Diode ?) cavity and a battery pack w/ a remote photo-switch on a beam held by a suspension fiber and monitor the rotation.  I'm thinking one could get some sensitivity multiplication by switching the cavity on and off in time w/ the oscillation frequency of the assembly (easy to do w/ a digital camera) and see if the oscillation amplitude increases w/ time.

Anyone want to try a sensitivity calculation ? Comments ?

 
Title: Re: EM Drive Developments - related to space flight applications - Thread 2
Post by: Rodal on 02/24/2015 01:48 am
Anyway, still thinking about a possible experiment in vacuum using:

https://www.dropbox.com/s/cmxyj7re8rrb6dh/IMAG0361.jpg?dl=0

to make a stack chamber like:

https://www.dropbox.com/s/wf8wv226h8138n1/IMAG0360.jpg?dl=0

but on an isolation stand.

The idea would be to then balance a tapered oscillator (Gunn Diode ?) cavity and a battery pack w/ a remote photo-switch on a beam held by a suspension fiber and monitor the rotation.  I'm thinking one could get some sensitivity multiplication by switching the cavity on and off in time w/ the oscillation frequency of the assembly (easy to do w/ a digital camera) and see if the oscillation amplitude increases w/ time.

Anyone want to try a sensitivity calculation ? Comments ?
Do you have this report by Brito, Marini and Gallian, using a Cavendish balance, batteries for power input, oil damping, counterweight and laser measurements ?  http://enu.kz/repository/2009/AIAA-2009-5070.pdf
Title: Re: EM Drive Developments - related to space flight applications - Thread 2
Post by: Lee Jay on 02/24/2015 01:51 am
I've tried to do searches of multiple types, and have read a few dozen posts, but I still can't answer what seems to be a very simple question, so I'm going to ask.

Has anyone verified that this isn't turning itself into some type of amplified photonic thruster?
Title: Re: EM Drive Developments - related to space flight applications - Thread 2
Post by: Rodal on 02/24/2015 01:56 am
.....

The idea would be to then balance a tapered oscillator (Gunn Diode ?) cavity


Disassembled radar speed gun. The grey assembly attached to the end of the copper-colored horn antenna is the Gunn diode oscillator which generates the microwaves.

From Wikipedia http://en.wikipedia.org/wiki/Gunn_diode
Title: Re: EM Drive Developments - related to space flight applications - Thread 2
Post by: Notsosureofit on 02/24/2015 01:59 am
Anyway, still thinking about a possible experiment in vacuum using:

https://www.dropbox.com/s/cmxyj7re8rrb6dh/IMAG0361.jpg?dl=0

to make a stack chamber like:

https://www.dropbox.com/s/wf8wv226h8138n1/IMAG0360.jpg?dl=0

but on an isolation stand.

The idea would be to then balance a tapered oscillator (Gunn Diode ?) cavity and a battery pack w/ a remote photo-switch on a beam held by a suspension fiber and monitor the rotation.  I'm thinking one could get some sensitivity multiplication by switching the cavity on and off in time w/ the oscillation frequency of the assembly (easy to do w/ a digital camera) and see if the oscillation amplitude increases w/ time.

Anyone want to try a sensitivity calculation ? Comments ?
Do you have this report by Brito, Marini and Gallian, using a Cavendish balance, batteries for power input, oil damping, counterweight and laser measurements ?  http://enu.kz/repository/2009/AIAA-2009-5070.pdf

I do now!  Thanks!
Title: Re: EM Drive Developments - related to space flight applications - Thread 2
Post by: Rodal on 02/24/2015 02:04 am
I've tried to do searches of multiple types, and have read a few dozen posts, but I still can't answer what seems to be a very simple question, so I'm going to ask.

Has anyone verified that this isn't turning itself into some type of amplified photonic thruster?
The amplification is the Q resonance.  The problem is that there are supposed to be no photons whatsoever coming out of the thruster if the EM drive is completely enclosed as a resonant cavity.

If evanescent waves escape from the EM drive through small gaps, the problem is that there can be no amplification of these waves since evanescent waves are exponentially decaying (not resonant) by definition (they are the imaginary solution of the eigenvalue problem, while the resonant modes are the real solutions of the eigenvalue problem of the wave equation). 

And even if photons were to escape, how could the performance be better than the one of a photon rocket?

For more on this, please discuss with our resident Photon Rocket expert: @frobnicat
Title: Re: EM Drive Developments - related to space flight applications - Thread 2
Post by: aero on 02/24/2015 02:05 am
I've tried to do searches of multiple types, and have read a few dozen posts, but I still can't answer what seems to be a very simple question, so I'm going to ask.

Has anyone verified that this isn't turning itself into some type of amplified photonic thruster?

I have suggested something like that. Photons with superluminal momentum or superluminal velocity (a different reference) escaping the cavity via evanescent waves. The references haven't gotten much traction and I've busy with meep so haven't delved into the math. http://wwwsis.lnf.infn.it/pub/INFN-FM-00-04.pdf (http://wwwsis.lnf.infn.it/pub/INFN-FM-00-04.pdf) (See Appendix B, page 15.) People who've looked at the reference get hung up on the causality paradox and can't seem to get past that to look at the math.

My opinion on that is that if the math gives good estimates of the thruster performance, then maybe the paper's author has a valid argument about the causality paradox. Or maybe delve into the tachyon math to see if the paradox really exists in this case.

@Rodal -
 
Quote
there can be no amplification of these waves since evanescent waves are exponentially decaying (not resonant) by definition

But I think that is the part of the point where the math of superluminal evanescent waves diverge from the classic math. The reference claims to show that the velocity of the wave is a function of the cavity diameter and the gap diameter. If that function of velocity is anything like diameter divided by gap size then the momentum of a single photon could be amplified enormously. Huge momentum from a small number of photons and small number of photons from a small power consumed.

I guess I'll have to set meep aside and delve into MAXIMA just to satisfy myself.
Title: Re: EM Drive Developments - related to space flight applications - Thread 2
Post by: Notsosureofit on 02/24/2015 02:23 am
.....

The idea would be to then balance a tapered oscillator (Gunn Diode ?) cavity


Disassembled radar speed gun. The grey assembly attached to the end of the copper-colored horn antenna is the Gunn diode oscillator which generates the microwaves.

From Wikipedia http://en.wikipedia.org/wiki/Gunn_diode

Yup!  That should do it.  Haven't found where I put that Gunn oscillator yet (just had it a couple months ago)

If anyone has one of those speed guns in the drawer I won't ask where you got it .......
Title: Re: EM Drive Developments - related to space flight applications - Thread 2
Post by: ThinkerX on 02/24/2015 03:09 am
Quote
Do you have this report by Brito, Marini and Gallian, using a Cavendish balance, batteries for power input, oil damping, counterweight and laser measurements ?  http://enu.kz/repository/2009/AIAA-2009-5070.pdf

Possibly a stupid question, but was this team using the most productive 'mode/frequency' for the size of their frustum? Having the right mode seems to be crucial to the correct functioning of this device. 

Title: Re: EM Drive Developments - related to space flight applications - Thread 2
Post by: zen-in on 02/24/2015 03:23 am
.....

The idea would be to then balance a tapered oscillator (Gunn Diode ?) cavity


Disassembled radar speed gun. The grey assembly attached to the end of the copper-colored horn antenna is the Gunn diode oscillator which generates the microwaves.

From Wikipedia http://en.wikipedia.org/wiki/Gunn_diode

Yup!  That should do it.  Haven't found where I put that Gunn oscillator yet (just had it a couple months ago)

If anyone has one of those speed guns in the drawer I won't ask where you got it .......

Gunn cavities used in speed sensors have a varactor diode and a detector diode inside the cavity; along with the Gunn diode.   They don't put out much power - only about 10 - 100 mW; since all the power comes from a small diode and there is no power amp.    24 GHz Gunn horns are also available.   One of the interesting effects that low power 10 GHz radiation has is local heating of the skin.   If you point a small ( <10 mW ) Gunn horn at the back of your hand you will feel a slight burning sensation; as if the business end of a soldering iron touched your skin.   

High power Gunn diodes are available so it is possible to swap out the diode for a high power one.    Here are some pictures of a 10 GHz Gunn horn with 3 diodes mounted in it.   The label labeled tune connects to the varactor diode and input to the Gunn diode.   The detector diode is on the opposite side.

Good luck to the the Eagleworks team.   I hope you are able to continue your investigations.
Title: Re: EM Drive Developments - related to space flight applications - Thread 2
Post by: Star-Drive on 02/24/2015 04:09 am
I thought somewhere Dr March had said that GRS's balance sensitivity required an improvement of the thrust signal by 100 percent in order to get successful replication. Did I get that right? I cannot find it now and I have been looking back through the thread for 20 minutes or more. -Anyway; with a deadline of EOM, march; does that mean there is an improved test article about to or in the process of undergoing additional testing at Eagleworks? and can someone link to where that fact (GRS sensitivity limits incapable of detecting current thrust signal level) was posted for reference?

Stormbringer:

The Glenn Research Center (GRC) torque pendulum test rig has a reported sensitivity of ~50 micro-Newton (uN), so we will need a consistent copper frustum thrust performance of at least 100uN to be assured of a successful replication effort at the GRC facility.  And no we don't have another test article that performs better than the copper frustum...

BTW, thank you for the PhD title compliment, but I've never earned the title of PhD at any university or other place of higher learning.  I can claim a BS in Electrical Engineering from UT at Arlington (1972), and 43 years working in the US aerospace industry in a number of engineering & R&D fields including the last 17 years chasing the experimental verifications of the Mach Effect and now these quantum based thrusters both in my home lab, and at the Eagleworks Lab that I helped create and work in over the last 3.8 years.

Best,  Paul March
Title: Re: EM Drive Developments - related to space flight applications - Thread 2
Post by: ThinkerX on 02/24/2015 04:29 am
Quote
The Glenn Research Center (GRC) torque pendulum test rig has a reported sensitivity of ~50 micro-Newton (uN), so we will need a consistent copper frustum thrust performance of at least 100uN to be assured of a successful replication effort at the GRC facility.  And no we don't have another test article that performs better than the copper frustum

You may have to resort to the Chinese method, and latch in the most powerful magnetron you can lay hands on.
Title: Re: EM Drive Developments - related to space flight applications - Thread 2
Post by: Stormbringer on 02/24/2015 04:55 am
I've tried to do searches of multiple types, and have read a few dozen posts, but I still can't answer what seems to be a very simple question, so I'm going to ask.

Has anyone verified that this isn't turning itself into some type of amplified photonic thruster?

I have suggested something like that. Photons with superluminal momentum or superluminal velocity (a different reference) escaping the cavity via evanescent waves. The references haven't gotten much traction and I've busy with meep so haven't delved into the math. http://wwwsis.lnf.infn.it/pub/INFN-FM-00-04.pdf (http://wwwsis.lnf.infn.it/pub/INFN-FM-00-04.pdf) (See Appendix B, page 15.) People who've looked at the reference get hung up on the causality paradox and can't seem to get past that to look at the math.

My opinion on that is that if the math gives good estimates of the thruster performance, then maybe the paper's author has a valid argument about the causality paradox. Or maybe delve into the tachyon math to see if the paradox really exists in this case.

@Rodal -
 
Quote
there can be no amplification of these waves since evanescent waves are exponentially decaying (not resonant) by definition

But I think that is the part of the point where the math of superluminal evanescent waves diverge from the classic math. The reference claims to show that the velocity of the wave is a function of the cavity diameter and the gap diameter. If that function of velocity is anything like diameter divided by gap size then the momentum of a single photon could be amplified enormously. Huge momentum from a small number of photons and small number of photons from a small power consumed.

I guess I'll have to set meep aside and delve into MAXIMA just to satisfy myself.
would deacclerating superluminal  waves release Cherenkov radiation? If so; where is the blue glowy stuff? (i like blue glowy stuff. ) Wouldn't a light intensifying camera detect Cherenkov radiation even if there is not enough to be visible to the naked eye?

i know if you remove the IR filter from a regular electronic camera you can turn a normal camera (on a cell phone for example) into a night vision camera. Perhaps there is a way to hack a camera to do the same thing with UV light.
Title: Re: EM Drive Developments - related to space flight applications - Thread 2
Post by: Paul451 on 02/24/2015 10:25 am
where is the blue glowy stuff? (i like blue glowy stuff. )

A question I ask myself every day.

i know if you remove the IR filter from a regular electronic camera you can turn a normal camera (on a cell phone for example) into a night vision camera. Perhaps there is a way to hack a camera to do the same thing with UV light.

No hack required. Most DSLRs should be able to accept a UV filter. Experiment with the settings against a known UV source and you're golden. (Or purplish-blue.)

[Correction, a digital UV filter is a combination of a UV-tuned optical-filter ("black" filter) and an IR blocker. Quirk of digital cameras, apparently. The red channel is also sensitive to UV and the blue channel is also sensitive to IR. D'oh. Nonetheless, no actual camera hacking is required, just suitable filters and a making sure you sample the raw colour channels separately.]
Title: Re: EM Drive Developments - related to space flight applications - Thread 2
Post by: Star-Drive on 02/24/2015 12:22 pm
Quote
The Glenn Research Center (GRC) torque pendulum test rig has a reported sensitivity of ~50 micro-Newton (uN), so we will need a consistent copper frustum thrust performance of at least 100uN to be assured of a successful replication effort at the GRC facility.  And no we don't have another test article that performs better than the copper frustum

You may have to resort to the Chinese method, and latch in the most powerful magnetron you can lay hands on.

Thinker-X:

That possibility has been discussed, but mounting a ~1.0kWe magnetron with a non-fan cooling subsystem and 4.0kVdc power supply on a torque pendulum is not doable at the moment primarily due to time constraints, and yes, the lack of a material budget as well.  However this Gunn diode approach being discussed in this forum is a possibility and one that might be doable if we could generate several hundreds watts of wideband RF centered at say 2.45 GHz with just a 28Vdc input.

Hmmm, I read a book by Paul A. LaViolette a while back entitled "Secrets of Antigravity Propulsion" where the author explored in his Figure 8.4 the use of a Gunn diode being amplified by a parametric amplifier that was then fed into a phase conjugate microwave resonator integrated into a propulsion waveguide system developed for a 1970s US black R&D propulsion project.   Perhaps it's time to dust off my copy of that book again and take a look-see of how one might at least create the Gunn diode / parametric amplifier portion of this system and then integrate it into our copper frustum.

Best, Paul M.
Title: Re: EM Drive Developments - related to space flight applications - Thread 2
Post by: JasonAW3 on 02/24/2015 02:16 pm
Quote
The Glenn Research Center (GRC) torque pendulum test rig has a reported sensitivity of ~50 micro-Newton (uN), so we will need a consistent copper frustum thrust performance of at least 100uN to be assured of a successful replication effort at the GRC facility.  And no we don't have another test article that performs better than the copper frustum

You may have to resort to the Chinese method, and latch in the most powerful magnetron you can lay hands on.

I take it that this is a hanging Pendulum?  If these are metal pendulums, is it possible that the magnetic field produced could be tweaking the piviot of the pendulum?  probably a dumb question, but I haven't seen any mention of eliminating transient magnetic field generated that could cause such a movement.

Not trying to be obtuse, but sometimesminor things get overlooked.  Like a loose cable at CERN.
Title: Re: EM Drive Developments - related to space flight applications - Thread 2
Post by: Chrochne on 02/24/2015 02:46 pm
Secrets of Antigravity Propulsion Figure 8.4 - copy past this into the Google search. Click the first link it opens up the chapter and the figure 8.4 (interesting picture) Mr. Paul March spoke about. Very interesting reading Mr. Paul, thank you.
Title: Re: EM Drive Developments - related to space flight applications - Thread 2
Post by: Rodal on 02/24/2015 02:54 pm
...
I take it that this is a hanging Pendulum?  ...
NASA Eagleworks has not used a hanging pendulum for their measurements.
They have used a low thrust torsion pendulum.  See the report for further details:   http://www.libertariannews.org/wp-content/uploads/2014/07/AnomalousThrustProductionFromanRFTestDevice-BradyEtAl.pdf
Title: Re: EM Drive Developments - related to space flight applications - Thread 2
Post by: JasonAW3 on 02/24/2015 03:46 pm
...
I take it that this is a hanging Pendulum?  ...
NASA Eagleworks has not used a hanging pendulum for their measurements.
They have used a low thrust torsion pendulum.  See the report for further details:   http://www.libertariannews.org/wp-content/uploads/2014/07/AnomalousThrustProductionFromanRFTestDevice-BradyEtAl.pdf

Could a magnetic field cause issus with this sort of pendulum?  Especially some form of rotating field?
Title: Re: EM Drive Developments - related to space flight applications - Thread 2
Post by: Rodal on 02/24/2015 03:53 pm
...
I take it that this is a hanging Pendulum?  ...
NASA Eagleworks has not used a hanging pendulum for their measurements.
They have used a low thrust torsion pendulum.  See the report for further details:   http://www.libertariannews.org/wp-content/uploads/2014/07/AnomalousThrustProductionFromanRFTestDevice-BradyEtAl.pdf

Could a magnetic field cause issus with this sort of pendulum?  Especially some form of rotating field?

For details on the NASA measurements take a look at the link attached above that contains the NASA report on the EM Drive.

It is not the sort of pendulum that matters but the materials used: what is their magnetic permeability.

Aluminum is a non-ferromagnetic material. 

Aluminum is "paramagnetic." Laymen consider Aluminum to be "non-magnetic." Aluminum doesn't stick to magnets.

Aluminum has a relative permeability of mur=1.000022

Copper is "diamagnetic", it has a relative permeability of mur=0.999994

This is very, very close to the relative permeability of free space--> mur = 1.

For comparison, wood has a relative permeability of mur=1.00000043



(Almost pure) Iron has a relative permeability mur=5000

Carbon Steel has a relative permeability mur=100

Metglas has a relative permeability=1000000 (at low frequencies)
Title: Re: EM Drive Developments - related to space flight applications - Thread 2
Post by: Rodal on 02/24/2015 04:32 pm
The Eagleworks vacuum chamber's main body is made from 304L stainless steel while its swing out door is made from aluminum

According to

http://www.aksteel.com/pdf/markets_products/stainless/austenitic/304_304l_data_sheet.pdf,
and also according to:
http://www.northamericanstainless.com/wp-content/uploads/2010/10/Grade-304-304L.pdf

the relative magnetic permeability of 304L

Magnetic Permeability (@H = 200 Oersteds) Annealed, is a maximum of  1.02

(304 St.St.  permeability increases with cold working, but I imagine that the vacuum steel chamber is annealed StSt)

Title: Re: EM Drive Developments - related to space flight applications - Thread 2
Post by: Rodal on 02/24/2015 04:54 pm
@aero is using MIT's Finite Difference computer code MEEP to calculate the force produced by evanescent waves escaping from the EM Drive and interacting with the stainless steel vacuum chamber.  This work is in progress.
Last I heard. my understanding is that it was a 2-dimensional model (because it is a Finite Difference model in space and time, hence takes very long computer running times). The 2-D model simulates the EM Drive as a flat surface with a trapezium for boundaries, and the cylindrical vacuum chamber is being modeled as a flat surface with a rectangle for boundaries.

I don't recall whether the relative permeability used in that analysis for the 304L StSt vacuum chamber material was given. 

The experiments performed in China and the UK were not performed inside a vacuum chamber.
Title: Re: EM Drive Developments - related to space flight applications - Thread 2
Post by: wembley on 02/24/2015 06:08 pm
Actually the Hafnium one was replicated and DARPA concluded it was real. This was not widely publicised and there was political opposition to it. However, for a variety of interesting reasons, it never went forward (main issue being expense plus lack,of a chain reaction). Work continues though.
Title: Re: EM Drive Developments - related to space flight applications - Thread 2
Post by: flux_capacitor on 02/24/2015 08:22 pm
Quote
Do you have this report by Brito, Marini and Gallian, using a Cavendish balance, batteries for power input, oil damping, counterweight and laser measurements ?  http://enu.kz/repository/2009/AIAA-2009-5070.pdf

Possibly a stupid question, but was this team using the most productive 'mode/frequency' for the size of their frustum? Having the right mode seems to be crucial to the correct functioning of this device.

Brito et al. did not test a microwave resonator frustum aka EmDrive at that time, their experiment was about testing some particular design of Woodward's MLT (Mach-Lorentz Thruster) involving magnetic coils and capacitors, a former design Pr. Woodward later abandoned in favor of a full solid state design aka MET (Mach-Effect Thruster). Hence no waveguide modes.
Title: Re: EM Drive Developments - related to space flight applications - Thread 2
Post by: flux_capacitor on 02/24/2015 09:35 pm
Finding pictures of shawyer's test device(s) is not that hard, but can any1 actually confirm that this is the Chinese truncated cone? or is it completely unrelated?

...


I think it's from Shawyer and improperly attributed to Juan Yang on some other web sites: http://emdrive.com/flightprogramme.html (http://emdrive.com/flightprogramme.html)

Someone should ask Roger Shawyer directly about the Chinese version (especially the dimensions of the cavity). He knows a lot about it, since he went there to speak with Juan Yang and give her some advice.


Yes, that Shawyer's Flight Thruster development programme. A 3.85GHz thruster weighing 2.92 Kg,.

(http://emdrive.com/images/thruster1.jpg)

http://emdrive.com/flightprogramme.html

I don't recall estimated dimensions for it. If anybody estimated the dimensions, @aero is the most likely one to have done it.

I did not. I see nothing to use as a reference. Perhaps someone could estimate ratios. Big/small, big/height or whatever.
The photo is a bit blurry and that makes estimating a bit challenging, and there are lens distortions to the photo, but nothing too major.  If the concrete block happened to be the standard width of 440 mm, cited by wikipedia, then the dimensions would be roughly as estimated in the chart.

I'm an artist, not a physicist.  If these dimensions seem wrong and you have a different guess for the width of the concrete block, let me know and I'll recalculate based on your width standard.

I finally found the real dimensions of this thruster test article in two reference papers from Shawyer on emdrive.com:
- Shawyer, Roger (27 October 2009). "The Emdrive Programme – Implications for the Future of the Aerospace Industry" (http://www.emdrive.com/CEAS2009paper.doc) (Word document). CEAS 2009 European Air and Space Conference. Manchester, U.K.: Royal Aeronautical Society.
- Shawyer, Roger (10 June 2010). "The EmDrive - A New Satellite Propulsion Technology" (http://www.emdrive.com/Toulouse2010paper01.doc) (Word document). Toulouse Space Show'10, 2nd Conference on Disruptive Technology in Space Activities (TECHNO DIS 2010). Toulouse, France: CNES.

So here are the numbers to know:

Overall diameter265 mm
Overall height164 mm
Weight2.92 kg
Operating frequency3.85 GHz
Unloaded Q60,000
Input power150 to 450 W
Max. measured thrust174×10−3 N
Average specific thrust326×10−3 N/kW

So the max. diameter is larger (+18 mm) and the height is smaller (-22 mm) than lasoi's results based on the information in the picture. Hence the ratio of the frustum in the picture is wrong, probably due to the lens of the camera that was used to take it. I think lasoi could get the right dimension of the smaller end diameter, tweaking the image so it could fit those new numbers.
Title: Re: EM Drive Developments - related to space flight applications - Thread 2
Post by: flux_capacitor on 02/24/2015 10:03 pm
Secrets of Antigravity Propulsion Figure 8.4 - copy past this into the Google search. Click the first link it opens up the chapter and the figure 8.4 (interesting picture) Mr. Paul March spoke about. Very interesting reading Mr. Paul, thank you.

Here is the picture attached.

What about other sources of microwaves easily available besides the Gunn diode (http://en.wikipedia.org/wiki/Gunn_diode) and the magnetron (http://en.wikipedia.org/wiki/Cavity_magnetron)? What do you think of:

- Klystron (http://en.wikipedia.org/wiki/Klystron). Bigger but with also very high power. Klystrons offer "cleaner" and narrower frequencies than magnetrons. This feature may be actually a tradeoff since Mulletron (http://forum.nasaspaceflight.com/index.php?topic=36313.msg1331792#msg1331792) and Dr. Rodal (http://forum.nasaspaceflight.com/index.php?topic=36313.msg1331838#msg1331838) showed the wideband of the noisy magnetron in Chinese experiments may compensate the detuning of the cavity due to its deformation while being heated.

- TWTA (http://en.wikipedia.org/wiki/Traveling-wave_tube) (traveling-wave tube amplifiers). Quite compact and powerful. Shawyer used two TWTAs to feed the test article I talked about in my previous post. Quoting his 2010 conference paper:
Quote
A major part of the work is in the development of the frequency tracking algorithm. This is needed to ensure the input frequency matches the resonant frequency of the high Q (60,000) cavity, over the full input power range and the qualification temperature specification.

The thruster is designed to be powered from existing flight qualified TWTAs, which are driven from a dual redundant frequency generator unit (FGU) The FGU  includes a frequency control loop using feedback signals from the thruster.
Title: Re: EM Drive Developments - related to space flight applications - Thread 2
Post by: lasoi on 02/24/2015 10:38 pm
According to those figures the frustum height is only 62% of the base diameter.  But the height of the frustum in the photo is 75% of its base diameter, and that's excluding the nuts atop the small endplate.  If we included those then the percentage would be even greater, closer to 80% of the base diameter.  Are we sure those are the correct measures for the frustum in the photo?  The distortion in the photo, which is primarily around the periphery, isn't nearly enough to account for that much discrepancy.  I don't mind being wrong but I don't see how those numbers jive with the photo.

Edit 1: I see what he's done.  He has excluded the height of the base of the frustum and the height of the top of the frustum from the frustum's overall height.  If we exclude these in the photo, then the frustum height is almost exactly 62% of the base diameter.  So it works out perfectly.  (Sorry for the misunderstanding.  I'm an artist, not a scientist, lol.)

And the diameter of the top cap is 189mm, give or take a mm or two.

Edit 2: If by "overall diameter", Shawyer meant the diameter of the widest part of the frustum excluding the base, then we again have a discrepancy between his given numbers and the photo.  But I'm assuming he includes the diameter of the base in "overall", since the photo and his given numbers fit perfectly that way.
Title: Re: EM Drive Developments - related to space flight applications - Thread 2
Post by: Rodal on 02/24/2015 11:39 pm
According to those figures the frustum height is only 62% of the base diameter.  But the height of the frustum in the photo is 75% of its base diameter, and that's excluding the nuts atop the small endplate.  If we included those then the percentage would be even greater, closer to 80% of the base diameter.  Are we sure those are the correct measures for the frustum in the photo?  The distortion in the photo, which is primarily around the periphery, isn't nearly enough to account for that much discrepancy.  I don't mind being wrong but I don't see how those numbers jive with the photo.

Edit, I see what he's done.  He has excluded the height of the base of the frustum and the height of the top of the frustum from the frustum's overall height.  If we exclude these in the photo, then the frustum height is almost exactly 62% of the base diameter.  So it works out perfectly.  (Sorry for the misunderstanding.  I'm an artist, not a scientist, lol.)

And the diameter of the top cap is 189mm, give or take a mm or two.
For clarity, could you please let us know what you think are the best estimates?

Yes, what matters are the interior dimensions, so the thickness of both end plates should be subtracted from the axial length (height), and the diameters should be measured at the intersections of the cone with the interior surfaces of the flat plates.

Big Diameter = 265 mm ?
Small Diameter =  ?
Height =  164 mm ?

Thanks ! :)
Title: Re: EM Drive Developments - related to space flight applications - Thread 2
Post by: lasoi on 02/24/2015 11:41 pm
Big diameter is 265 mm
Small diameter is 189 mm
Height is 164 mm.

If the diameter of the base and top are measured from where the frustum meets the base and top caps, then Shawyer's numbers (265mm base and 164mm height) don't fit the photo.  But if the diameter of the base and top are measured from the widest parts of the end caps, then they fit perfectly with the photo.

If you like I'll make another chart to show you what I mean but it'd take twenty minutes or so.
Title: Re: EM Drive Developments - related to space flight applications - Thread 2
Post by: Rodal on 02/24/2015 11:45 pm
Big diameter is 265 mm
Small diameter is 189 mm
Height is 164 mm.

So, for Shawyer's Flight Thruster,

Cone Half-Angle:

ArcTan[(BigDiameter - SmallDiameter)/(2*Length)] = ArcTan[0.2317] = 13.05 degrees




Thanks to @flux_capacitor and to @lasoi, we can update the table below

For reference. the tangent of the cone's half angle thetaw and the cone's half angle thetaw, in ascending order, for the following cases are:

(Notice how Shawyer progressively increased the cone's half-angle, with time, in his experimental designs, by a factor of 7 in the tangent of the half-angle)

Example (and geometry)                    { Tan[thetaw],thetaw (degrees) }

Shawyer Experimental                        {0.104019,   5.93851}
Shawyer Demo                                   {0.219054, 12.3557}
Shawyer Fligth Thruster                      {0.231707, 13.04564}
NASA Eagleworks frustum                   {0.263889, 14.7827}
Egan's example                                  {0.36397 ,  20}
Prof. Juan Yang  (2014)                      {0.4538,     24.4 }
Shawyer Superconducting 2014          {0.7002,     35}


(http://gregegan.customer.netspace.net.au/SCIENCE/Cavity/CavityShape.gif)

(http://forum.nasaspaceflight.com/index.php?action=dlattach;topic=36313.0;attach=807019;image)
(http://forum.nasaspaceflight.com/index.php?action=dlattach;topic=36313.0;attach=802794;image)
Title: Re: EM Drive Developments - related to space flight applications - Thread 2
Post by: Notsosureofit on 02/25/2015 12:31 am
Big diameter is 265 mm
Small diameter is 189 mm
Height is 164 mm.

If the diameter of the base and top are measured from where the frustum meets the base and top caps, then Shawyer's numbers (265mm base and 164mm height) don't fit the photo.  But if the diameter of the base and top are measured from the widest parts of the end caps, then they fit perfectly with the photo.

If you like I'll make another chart to show you what I mean but it'd take twenty minutes or so.

Hmm, just playing "guess the mode"...the results look reasonably close, but for Micro not Milli Newtons ?????

Are those internal numbers ?
 
Title: Re: EM Drive Developments - related to space flight applications - Thread 2
Post by: lasoi on 02/25/2015 12:39 am
Sorry to keep harping on this, I'll soon be done and out of the way.  But I just wanna make sure I get this right.  Can someone answer the question on the chart?
Title: Re: EM Drive Developments - related to space flight applications - Thread 2
Post by: frobnicat on 02/25/2015 12:49 am
...
I take it that this is a hanging Pendulum?  ...
NASA Eagleworks has not used a hanging pendulum for their measurements.
They have used a low thrust torsion pendulum.  See the report for further details:   http://www.libertariannews.org/wp-content/uploads/2014/07/AnomalousThrustProductionFromanRFTestDevice-BradyEtAl.pdf

Makes me musing around pendulum considerations again...

Both the pivotal movement (around vertical axis) and the active position restoring torque (around the vertical axis of rotation of the arm) is given by flexural pivots of this kind :
(http://www.pages.drexel.edu/~garfinkm/PivotWebFig/FP.1.gif)

There are two of them mounted (on the same vertical axis obviously) for added rigidity along the two others (horizontal) axis. The role of this tandem flexural pivot is, well, pivotal in the experiment and deserve careful consideration. While those pivots can exert a torque on the arm (restoring : 0 at equilibrium), they are by definition close to the axis, and it would be hard for them to impose a fast action to the rotation of arm (high "apparent inertia" from their place). It's difficult to see a scheme that would make them the source of a "step" response in position (fast rise and fast fall of displacement of arm at on and off power as observed)

It was explained by Paul March (http://forum.nasaspaceflight.com/index.php?topic=29276.msg1254143#msg1254143) in thread one that the IR heating of those bearings by the microwave amplifier (just behind them) would slowly (due to thermal inertia) alter the temperature of their spring and therefore their rest (equilibrium) position and was the reason of the systematic "drift" (slowly moving baseline) seen on the records. Only partially successful attempts at mitigating the problem was reported in this post.

Later on thread 2 (related to spaceflight) while explaining why the buckling analysed by Rodal would 1) not explain a sustained force for ~40s and 2) would make a (short) thrust in opposite direction to the one observed by the pendulum, March added 3) that the "baseline slope" (observed systematic drift) was a consequence of the continued thermal expansion of the frustum (displacement of centre of mass) :

...
And you can see this negative longer term frustum thermal drift by noting the downward going baseline slope of the thrust trace even after the RF power is removed from the copper frustum.
...
(http://forum.nasaspaceflight.com/index.php?action=dlattach;topic=36313.0;attach=776497;image)

I vaguely remember it was made mention between the two posts of a better management of the IR induced pivots heating drift but can't find that (is it my imagination ?)
In this post I find arguments for points 1) and 2) convincing but 3) counter intuitive : would the frustum continue to increase at accelerated rate in bulk temperature after power off for more than 40s to explain the observed overall regular slope as a result of displacement of centre of mass ? Anyone with an intuition on the thermal dynamics of the frustum in vacuum to rule that out, or should we try a quantitative assessment ? This drifting baseline is annoying, maybe not important because slow, but still better to have a better understanding of the whole system. It looks like it is still due to IR heating of flexure pivots.

Argument :

Apart from constrained dynamics (buckling), displacement of centre of mass y(t) is roughly proportional to temperature T(t). Apparent thrust F(t) due to such a displacement is proportional to second derivative of y(t). To get an apparent thrust with a constant down rate would need a displacement to the left (temperature) rising in t^3. Can't see that happening long after power-off. The other way around, any "reasonable" bulk temperature curve would show at least an inflexion point between power on and power off (or soon after power off), that would imply an added apparent thrust first to the right then to the left. Any bound part that accelerates (reverse) has to stop accelerating (forward) and has to regain it's original place in the long run (forward then reverse). Where forward is the thrust toward the small end.

Sorry incomplete and confuse thoughts, to be continued...
Title: Re: EM Drive Developments - related to space flight applications - Thread 2
Post by: Notsosureofit on 02/25/2015 12:58 am
Sorry to keep harping on this, I'll soon be done and out of the way.  But I just wanna make sure I get this right.  Can someone answer the question on the chart?

At least for the internal height, the junction between the plates is the place to measure from.
The internal diameters need to know the wall thickness of the cone etc.

That whole thing looks like he used (standard ?) heavy duty vacuum/pressure components.  They look to be wire seal flanges, but need to know the manufacturer to find the exact sizes.  Could be CF also, a bit more standard.  They make size transition sections which could well be the cone.

20 bolts implies 202.4mm OD 148.2mm ID
24 bolts 253.2mm OD 198.9mm ID
32 implies 304.0 OD 247.6mm ID


Title: Re: EM Drive Developments - related to space flight applications - Thread 2
Post by: Rodal on 02/25/2015 01:00 am
....
Later on thread 2 (related to spaceflight) while explaining why the buckling analysed by Rodal ... would make a ... thrust in opposite direction to the one observed by the pendulum, ...
That's incorrect.

The buckling analysis (thermal instability) gives a force precisely in the same direction as measured.

It was the thermal expansion explanation (advanced by Oak Ridge Labs in another context) that gives a force in the completely opposite direction.

Please refer back to the original image to see that the thermal expansion force (of the dielectric, which is what thermally expands the most, as per Oak Ridge) is the one in the wrong direction.

The buckling on the end plate is in the opposite direction to the thermal expansion of the dielectric.

Buckling movement of the large diameter end is towards the left, and the buckling force is towards the left. (Paul indicates it as "oil canning" of the flat plate).

Thermal expansion of the HD PE dielectric is towards the right.

Movement of the EM Drive is towards the left, exactly the same direction as the buckling movement, and the same as the buckling force.

The buckling force is analogous to somebody pushing (actually bending) the center of the end plate towards the left, which will deflect the plate inwards.  If the structure (the EM Drive) is free to move, it will move to the left, as a consequence of the plate being pushed inwards towards the left.



The buckling force perfectly explains the initial impulse magnitude, time duration of the impulse, and direction of travel.  The buckling force cannot explain the sustained 40 sec force, hence the buckling force explanation is rejected on the grounds that it cannot explain the 40 sec duration of the force.



The thermal expansion explanation is rejected, upon inspection, on several grounds that the thermal expansion movement (of the HD PE) is in the opposite direction as the movement of the EM Drive, rejected on the basis that the HD PE has a free surface, hence free to expand, and therefore there should be no force arising from an unrestrained isothermal  homogeneous thermal expansion.  Thermal stresses arise in restrained materials or those under a temperature gradient or those with anisotropic coefficients of thermal expansion.  This follows from the equations of thermoelasticity.  The equations presented in the Oak Ridge report do not abide by the equations of thermoelasticity (Boley and Wiener).



(http://forum.nasaspaceflight.com/index.php?action=dlattach;topic=36313.0;attach=776495;image)
Title: Re: EM Drive Developments - related to space flight applications - Thread 2
Post by: lasoi on 02/25/2015 01:07 am
Sorry to keep harping on this, I'll soon be done and out of the way.  But I just wanna make sure I get this right.  Can someone answer the question on the chart?

At least for the internal height, the junction between the plates is the place to measure from.
The internal diameters need to know the wall thickness of the cone etc.

That whole thing looks like he used (standard ?) heavy duty vacuum/pressure components.
Thanks.  I'll assume the interior walls are the same thickness as the frustum walls where they flare out and attach to the plate.  We can see their thickness there.
Title: Re: EM Drive Developments - related to space flight applications - Thread 2
Post by: aero on 02/25/2015 01:31 am
An interesting bit of information over on TP, here:
http://www.talk-polywell.org/bb/viewtopic.php?f=10&t=5830 (http://www.talk-polywell.org/bb/viewtopic.php?f=10&t=5830)
I would post to the link but for some reason my computer is forbidden to access the arxiv web site. Anyone know why that might be?

Anyway, seems that in looking at the Juno fly-by anomaly they have came up with a "new" force relating angular and linear momentum. Seemingly would also work for electrical phenomenon and it might even be of the right order of magnitude to be of interest to us here.
Title: Re: EM Drive Developments - related to space flight applications - Thread 2
Post by: Notsosureofit on 02/25/2015 01:39 am
Check the Edits above for some "standard" dimensions

That whole thing looks like he used (standard ?) heavy duty vacuum/pressure components.  They look to be wire seal flanges, but need to know the manufacturer to find the exact sizes.  Could be CF also, a bit more standard.  They make size transition sections which could well be the cone.

20 bolts implies 202.4mm OD 148.2mm ID
24 bolts 253.2mm OD 198.9mm ID
32 implies 304.0 OD 247.6mm ID
Title: Re: EM Drive Developments - related to space flight applications - Thread 2
Post by: aero on 02/25/2015 01:46 am
In the past we have been interested in the dielectric constant of the disks in the thruster. I made some meep runs showing force over a range of gap sizes with different values of the dielectric constant. Gaps are at the metal/metal contact between the end copper sheet and the cone body as a fraction of thruster height.

The value, 1.76, is the one I've used since I first looked for resonance at the frequency of 1880.4 MHz. It is clear that the dielectric constant has a strong effect of the meep force/power, but I speculate that it may be simply moving the resonance frequency so that the cavity is out of resonance for the other values of dielectric constant. Anyway, the data is attached.
Title: Re: EM Drive Developments - related to space flight applications - Thread 2
Post by: lasoi on 02/25/2015 01:57 am
I've probably got the thickness of the frustum walls wrong but it's difficult to gauge because where the frustum walls flare out at the bottom and attach to the base cap they appear to be thinner than where they flare out at the top and attach to the top cap.  I wish the photo was from a lower point of view so we could see beneath the top rim of the thing.

At any rate I can't get the dimensions of the inner cavity in the photo to jive with Shawyer's numbers.  So I'm at a loss.  I will say that it's a pretty remarkable coincidence that his numbers work precisely with the measurements in the photo if we use the diameters of the base and top caps, rather than the estimated diameters of the inner cavity, whatever they may be.

I'll post the chart just for the sake of having it visible.  Like I said, the two diameter scales must be incorrect.  My best guess for "overall" diameter of the top, would be 189mm.
Title: Re: EM Drive Developments - related to space flight applications - Thread 2
Post by: Rodal on 02/25/2015 02:20 am
An interesting bit of information over on TP, here:
http://www.talk-polywell.org/bb/viewtopic.php?f=10&t=5830 (http://www.talk-polywell.org/bb/viewtopic.php?f=10&t=5830)
I would post to the link but for some reason my computer is forbidden to access the arxiv web site. Anyone know why that might be?

Anyway, seems that in looking at the Juno fly-by anomaly they have came up with a "new" force relating angular and linear momentum. Seemingly would also work for electrical phenomenon and it might even be of the right order of magnitude to be of interest to us here.

http://www.sciencedirect.com/science/article/pii/S0273117714004153

http://phys.org/news/2014-09-anomaly-satellite-flybys-confounds-scientists.html

http://en.wikipedia.org/wiki/Flyby_anomaly
Title: Re: EM Drive Developments - related to space flight applications - Thread 2
Post by: frobnicat on 02/25/2015 10:15 am
....
Later on thread 2 (related to spaceflight) while explaining why the buckling analysed by Rodal ... would make a ... thrust in opposite direction to the one observed by the pendulum, ...
That's incorrect.

Maybe, but it appears Paul March in this post (http://forum.nasaspaceflight.com/index.php?topic=36313.msg1331580#msg1331580) says that it is opposite, and I don't see that this disagreement was acknowledged or settled. I understand his argument, not yours (on this particular point).

Quote from: Rodal
The buckling analysis (thermal instability) gives a force precisely in the same direction as measured.

It was the thermal expansion explanation (advanced by Oak Ridge Labs in another context) that gives a force in the completely opposite direction.

Please refer back to the original image to see that the thermal expansion force (of the dielectric, which is what thermally expands the most, as per Oak Ridge) is the one in the wrong direction.

The buckling on the end plate is in the opposite direction to the thermal expansion of the dielectric.

Buckling movement of the large diameter end is towards the left, and the buckling force is towards the left. (Paul indicates it as "oil canning" of the flat plate).

If we leave aside thermal expansion of the dielectric at small end (admittedly I forgot to take that one into account in my post...) the point made by March is that a movement to the left will first appear as a thrust to the right (action/reaction).

Now Newton's third law still states that for every action there is an equal and opposite reaction.  So when the copper frustum's large OD end-cap's prompt and inward oil canning action, followed by the slower frustum cone thermal expansions, they both push the copper frustum's Center of Mass (CoM) to the left as viewed from the front of the Eagleworks' vacuum chamber looking back at the test article and torque pendulum, while noting how the copper frustum is bolted on to the T.P..  These thermally induced actions to the left requires the torque pendulum's arm to move to the right to maintain the balance of the torque pendulum's arm in the lab's 1.0 gee gravity field, since we also use the Earth's g-field to help null the pendulum's movements. 

The last part " in the lab's 1.0 gee gravity field, since we also use the Earth's g-field to help null the pendulum's movements." remains obscure, are we to understand that the system is not adjusted to keep the balance arm's rotation axis as vertical as possible and that a small bias is introduced so that the heavier end of arm rests at a lower position than the lighter one (rotation not on horizontal plane but slightly inclined one) ?

Anyhow, if a centre of mass displacement induced thrust signature is to be understood as an action/reaction effect, the thrust would indeed be to the right, at least initially during a phase of acceleration of CoM to the left.  Or are you considering that the buckling induces another apparent thrust than an action/reaction effect ?

Quote from: Rodal
Thermal expansion of the HD PE dielectric is towards the right.

Movement of the EM Drive is towards the left, exactly the same direction as the buckling movement, and the same as the buckling force.

That's paradoxical, why a displacement to the right of CoM of dielectric (relative to frustum cone) would induce a thrust signature in the same direction as a displacement to the left of CoM of end plate (relative to frustum cone) ?

Quote from: Rodal
The buckling force is analogous to somebody pushing (actually bending) the center of the end plate towards the left, which will deflect the plate inwards.  If the structure (the EM Drive) is free to move, it will move to the left, as a consequence of the plate being pushed inwards towards the left.

That's I don't get. The forces that induce the inward deflection can't reciprocally push on anything fixed (vacuum chamber rest frame). The reciprocal push (first radial but then with an axial component as soon as buckling occurs) is on the rim of the cone, in the moving frame of pendulum's arm.

If someone stands on a bank and pushes to the left a heavy chain that hangs from the mast of a boat, the chain will be deflected to the left and the boat will move to the left. Yes. But if someone stands on the deck of the boat, and pushes the same chain to the left, the chain is deflected to the left and the boat will recoil to the right. And buckling a large "oil canning" plate would show same effects as a freely hanging chain : depending on whether one is pushing from the bank or from the boat.

Quote from: Rodal


The buckling force perfectly explains the initial impulse magnitude, time duration of the impulse, and direction of travel.  The buckling force cannot explain the sustained 40 sec force, hence the buckling force explanation is rejected on the grounds that it cannot explain the 40 sec duration of the force.



The thermal expansion explanation is rejected, upon inspection, on several grounds that the thermal expansion movement (of the HD PE) is in the opposite direction as the movement of the EM Drive, rejected on the basis that the HD PE has a free surface, hence free to expand, and therefore there should be no force arising from an unrestrained isothermal  homogeneous thermal expansion.  Thermal stresses arise in restrained materials or those under a temperature gradient or those with anisotropic coefficients of thermal expansion.  This follows from the equations of thermoelasticity.  The equations presented in the Oak Ridge report do not abide by the equations of thermoelasticity (Boley and Wiener).

(http://forum.nasaspaceflight.com/index.php?action=dlattach;topic=36313.0;attach=776495;image)

I have to think again about all that. You seem to make a difference whether thermal expansion is "restrained" or not on how to account for forces as seen from the outside, while it seems to me that, when all stresses and buckling have been taken into account within the system to predict a given displacement (of one part relative to another), then "recoil force" as seen from the outside depends only on those displacements of CoM (no memory if such displacement where due to constrained dynamics or not, the constraints forces "stay" inside the system). I understand March's statements as having the same difficulty to understand your take on that.
Title: Re: EM Drive Developments - related to space flight applications - Thread 2
Post by: Notsosureofit on 02/25/2015 11:43 am
An interesting bit of information over on TP, here:
http://www.talk-polywell.org/bb/viewtopic.php?f=10&t=5830 (http://www.talk-polywell.org/bb/viewtopic.php?f=10&t=5830)
I would post to the link but for some reason my computer is forbidden to access the arxiv web site. Anyone know why that might be?

Anyway, seems that in looking at the Juno fly-by anomaly they have came up with a "new" force relating angular and linear momentum. Seemingly would also work for electrical phenomenon and it might even be of the right order of magnitude to be of interest to us here.

Don't really see a "new" force there  (not up on flybys) but FYI, there is an "old force" interaction between angular momentum and a gravitational gradient that is "always" forgotten ......

This is the "Geezer" in me talking (Ya young pups) feel free to ignore ... did some of those calculations in the '60s

In those days it was thought to be able to help lift from the earths surface,...needed a source of coherent angular momentum (he he, like Hafnium which wasn't considered as I recall)
Title: Re: EM Drive Developments - related to space flight applications - Thread 2
Post by: Rodal on 02/25/2015 11:50 am
....
Later on thread 2 (related to spaceflight) while explaining why the buckling analysed by Rodal ... would make a ... thrust in opposite direction to the one observed by the pendulum, ...
That's incorrect.

Maybe, but it appears Paul March in this post (http://forum.nasaspaceflight.com/index.php?topic=36313.msg1331580#msg1331580) says that it is opposite, and I don't see that this disagreement was acknowledged ...
(Bold added for emphasis)

The fact that the thermal instability (buckling force) cannot be used to nullify the EM Drive thrust, was explicitly acknowledged by me and explained here http://forum.nasaspaceflight.com/index.php?topic=36313.msg1336196#msg1336196:

Quote
Dr. Rodal  analyzed possible thermal instability (thermal buckling of the flat ends) as a cause for the measured thrust and reported this at NSF and at ResearchGate (https://www.researchgate.net/publication/268804028_NASA%27S_MICROWAVE_PROPELLANT-LESS_THRUSTER_ANOMALOUS_RESULTS_CONSIDERATION_OF_A_THERMO-MECHANICAL_EFFECT).   A thermo-mechanical effect (thermal buckling) is shown that occurs in less than 1 second (for the copper thickness employed for the microwave cavity), with a temperature increase of a degree C or less and that results in forces of the same magnitude as reportedly measured by NASA.  Moreover, this thermal instability produces forces in the same direction as measured, and it will occur in a vacuum (since the heating can be due either to induction heating from the axial magnetic field in a TE mode or resistive heating due to the axial electric field in a TM mode).  However, this effect can only explain the initial impulsive force and cannot explain the longer 30 to 40 second measured force.  Thus the thrust force measured for up to 40 second is not nullified by this explanation either.

Thermal expansion effect as posited by a team from Oak Ridge National Labs for another propellant-less set of experiments was also eliminated as a possible source by the NSF contributors because it would result in forces in the complete opposite direction as the forces measured by NASA.
(Bold and color added for emphasis)
Title: Re: EM Drive Developments - related to space flight applications - Thread 2
Post by: Rodal on 02/25/2015 12:32 pm
.....
I have to think again about all that. ...
The mathematical equations I used are in the aforementioned paper, if you object to any of the equations and assumptions we would have a  better basis for discussion.

To state that the buckling forces produces a force (towards the right in the picture), in the opposite direction to the measured movement and force of the EM Drive (towards the left in the picture) is, I think, untenable.

Here are two tenable positions that can be taken:

1) The buckling force cannot produce any motion of the center of mass of an unconstrained EM Drive free-floating in space. If one assumes simply supported boundary conditions at the edges of the big diameter plate, just before instability takes place,  the end plate is flat and hence the membrane forces are balanced by the rigid circular rim.  These membrane forces are directed radially from the rim towards the center, they are self-balanced.  There are no forces perpendicular to the plate.  If one assumes that the buckling instability takes place instantaneously , superluminally, in the buckled state, there are no membrane forces if the plate is infinitesimally thin, such that it will take whatever bent position is necessary to accommodate the required membrane thermal expansion.  Since there are no membrane forces in the final buckled state, there are no forces at the simply supported edges at all in the buckled state.  Hence there are no forces to move the EM Drive before and after the buckled state, and that's all, because the buckling motion is assumed to take place instantaneously (hence there are no forces to be considered to take place in between the initial flat state and the final buckled state).

2) If one assumes that buckling cannot take place instantaneously, but that its maximum speed has to be limited by the velocity of stress waves in the material, that is, by the speed of sound in the material, then the minimum time interval for buckling to take place is governed by the speed of sound.  In the paper I derived a speed and acceleration for the buckling motion under some assumptions that are explicitly stated (see equations).  Those assumptions can indeed be criticized.  The equations can also be criticized, and you are invited to do so.  The derived speed is below the speed of sound in the material, so it satisfies this physical condition (that the buckling speed should be lower than the speed of sound in the material).  After the onset of buckling, and during this (short time) buckling motion, the membrane force decreases, from a maximum at the onset of buckling to a minimum at the final buckling state.  During this (short time) the plate is in a bent condition, with increased bending shape with time.  At the edges of the plate, the bending of the plate produces a slope at the simply supported edges.  The membrane force at the edges can be decomposed into two directions: A) a component that is in the original flat direction.  This component is self-balanced.  B) The other force vector component is directed in the same direction (towards the left) as the motion of the EM Drive.   This is a very short-time impulsive force, calculated to be of the same magnitude as the initial transient rise force in the experiments.
Title: Re: EM Drive Developments - related to space flight applications - Thread 2
Post by: Mulletron on 02/25/2015 12:36 pm
We're being talked about over there.

http://forums.sufficientvelocity.com/threads/em-drive-system-further-confirmed.14456/page-9

The thread starts with some mathematical analysis of the Dr. White conjecture which is pretty good. At page 9 they tear into me. They think the magnetic field is literally external to the copper frustum. Later they rip into MiHsC. Glad to see people are fans.
Title: Re: EM Drive Developments - related to space flight applications - Thread 2
Post by: MazonDel on 02/25/2015 02:18 pm
Greetings all, new to the forums.

I am a Robotics Engineer and so not nearly as well versed on physics as you all and I was kind of hoping for a clarification on the current discussion about the thermal buckling and such.

My current understanding of the how the experiment is measured (based on the image that Rodal has posted regarding the Areas of Thermal Expansion) is that any thrust that is generated will cause the torque pendulum to twist, and this is what is being measured (via the laser distance interferometer). I note that the center of mass of the frustum is centered over the toque pendulum.

From what I have read it seems as though the buckling of the frustum wall can be a source of some initial force, but cannot account for the constant thrust detected over the course of the experiment. What I have been wondering is if this buckling could cause enough of a shift of the COM towards the left to throw off the balance? As I have seen some discussions about the COM after the posts that made me think of this, I am trying to confirm that this is at some level what is being discussed.

Thanks! Keep up the good work!
Title: Re: EM Drive Developments - related to space flight applications - Thread 2
Post by: Flyby on 02/25/2015 02:56 pm
I've probably got the thickness of the frustum walls wrong but it's difficult to gauge because where the frustum walls flare out at the bottom and attach to the base cap they appear to be thinner than where they flare out at the top and attach to the top cap.  I wish the photo was from a lower point of view so we could see beneath the top rim of the thing.

At any rate I can't get the dimensions of the inner cavity in the photo to jive with Shawyer's numbers.  So I'm at a loss.  I will say that it's a pretty remarkable coincidence that his numbers work precisely with the measurements in the photo if we use the diameters of the base and top caps, rather than the estimated diameters of the inner cavity, whatever they may be.

I'll post the chart just for the sake of having it visible.  Like I said, the two diameter scales must be incorrect.  My best guess for "overall" diameter of the top, would be 189mm.

As you already started on this before i could actually find time, i did no longer bother to continue on this, but applying perspective rules on a picture needs some corrections first...

The image suffers from 3 type of distortions :

-there is a distinct "barrel" distortion, very noticeable on the back-wall horizontal join, which shows as a curve instead of a straight line. the further away from the center of the image, the more severe the distortion. There exists software to correct this with 100% accuracy (PTLens), but the problem is that you need to know the camera type and lens type...

-There is a tilt-down distortion (up-down), causing all verticals to skew together. The further down of the horizon line, the bigger the distortion. Again, you can correct this with software, but due to the yawn (left/right movement) of the camera the vertical lines cant be corrected symmetrically. But I think i could come up with an approximation to solve that problem

-Then there is a yawn-distortion, this causes the horizontal axis to shift. Moving the camera target to the right (out of the center of the image) cause the horizontal axis to shift left-down and right-up. So your  axis of the ellipses (circle in perspective) should not be horizontal, but slightly angled , due to the camera target not being in the dead center.

If you're interested, I'll finish up a corrected image this evening which should be a "close enough" approximation. Too bad we do not have any information about the camera and the used lens...
Title: Re: EM Drive Developments - related to space flight applications - Thread 2
Post by: frobnicat on 02/25/2015 03:15 pm
....
Later on thread 2 (related to spaceflight) while explaining why the buckling analysed by Rodal ... would make a ... thrust in opposite direction to the one observed by the pendulum, ...
That's incorrect.

Maybe, but it appears Paul March in this post (http://forum.nasaspaceflight.com/index.php?topic=36313.msg1331580#msg1331580) says that it is opposite, and I don't see that this disagreement was acknowledged ...
(Bold added for emphasis)

The fact that the thermal instability (buckling force) cannot be used to nullify the EM Drive thrust, was explicitly acknowledged by me and explained here http://forum.nasaspaceflight.com/index.php?topic=36313.msg1336196#msg1336196:

Quote
Dr. Rodal  analyzed possible thermal instability (thermal buckling of the flat ends) as a cause for the measured thrust and reported this at NSF and at ResearchGate (https://www.researchgate.net/publication/268804028_NASA%27S_MICROWAVE_PROPELLANT-LESS_THRUSTER_ANOMALOUS_RESULTS_CONSIDERATION_OF_A_THERMO-MECHANICAL_EFFECT).   A thermo-mechanical effect (thermal buckling) is shown that occurs in less than 1 second (for the copper thickness employed for the microwave cavity), with a temperature increase of a degree C or less and that results in forces of the same magnitude as reportedly measured by NASA.  Moreover, this thermal instability produces forces in the same direction as measured, and it will occur in a vacuum (since the heating can be due either to induction heating from the axial magnetic field in a TE mode or resistive heating due to the axial electric field in a TM mode).  However, this effect can only explain the initial impulsive force and cannot explain the longer 30 to 40 second measured force.  Thus the thrust force measured for up to 40 second is not nullified by this explanation either.

Thermal expansion effect as posited by a team from Oak Ridge National Labs for another propellant-less set of experiments was also eliminated as a possible source by the NSF contributors because it would result in forces in the complete opposite direction as the forces measured by NASA.
(Bold and color added for emphasis)

That "the thermal instability (buckling force) cannot be used to nullify the EM Drive thrust" was acknowledged by everyone, I agree, but on the ground of the duration of the observed thrust alone. Disagreement (respectful) is not on that. Maybe it would be best if Paul March himself confirmed or restated, or some other native Newtonian and English reader could confirm the same reading ( I may have misinterpreted ) but appears to me (and makes sense) that the actual disagreement is explicit enough :
...
Now Newton's third law still states that for every action there is an equal and opposite reaction.  So when the copper frustum's large OD end-cap's prompt and inward oil canning action, followed by the slower frustum cone thermal expansions, they both push the copper frustum's Center of Mass (CoM) to the left as viewed from the front of the Eagleworks' vacuum chamber looking back at the test article and torque pendulum, while noting how the copper frustum is bolted on to the T.P..  These thermally induced actions to the left requires the torque pendulum's arm to move to the right to maintain the balance of the torque pendulum's arm in the lab's 1.0 gee gravity field, since we also use the Earth's g-field to help null the pendulum's movements. 
...

Don't you read like me that Paul March says that the effect to be expected (by a short duration buckling) by a displacement of the centre of mass of the end-cap to the left is a displacement of the arm to the right, at least when things start to move ? Doesn't it make sense from an action/reaction momentum conservation standpoint ? So what I say is that there is disagreement not on the fact that buckling can't account for duration of observed pendulum's displacement (induced by forces or apparent transient forces when the frustum centre of mass moves relative to fixation point on pendulum's arm) but on the fact that the very direction of such effect would be expected to push the pendulum's arm to the right, at least initially : that would mean a dip (step down) on the chart, not a rise as observed  (step up, increasing distance, arm moving to left).

So to make my position explicit in regard to observables : I predict that the consequence of an "inward oil canning" is a sharp plunge on the record charts (or to add a sharp plunge term to other possible effects that move the arm). And I also think Paul March sees that the same way. Will take time to further reply of your next post, unless other contributors feel it clutters the main thread and we could discuss that by PM.
Title: Re: EM Drive Developments - related to space flight applications - Thread 2
Post by: lasoi on 02/25/2015 03:24 pm
@Flyby

The second type distortion you refer to is a perspective distortion.  All of that is true and in addition the camera wasn't held on a flat level when the photo was taken.  I've rotated it a few degrees clockwise to compensate for this problem but I might still be off of the horizontal by a fraction of a degree.  At any rate my experience is that these distortions are so small that they don't significantly affect the accuracy of the estimate, which is a just that, a rough estimate give or take a couple mms.

If you have the software and are able to correct for some of the distortion, I'll recalculate.  I'm not sure that it matters much but I'd be happy to do so.
Title: Re: EM Drive Developments - related to space flight applications - Thread 2
Post by: Rodal on 02/25/2015 03:29 pm
....
So to make my position explicit in regard to observables : I predict that the consequence of an "inward oil canning" is a sharp plunge on the record charts (or to add a sharp plunge term to other possible effects that move the arm). And I also think Paul March sees that the same way. Will take time to further reply of your next post, unless other contributors feel it clutters the main thread and we could discuss that by PM.

I explcitly gave you analytical arguments that stating that the buckling force would be to the right, as you maintain, is untenable, in this post:  http://forum.nasaspaceflight.com/index.php?topic=36313.msg1337106#msg1337106

Tenable positions are that for an EM Drive unrestrained in space either 1) there is no buckling force on the center of mass or 2) there is a very short-time minute buckling force to the left, in the same direction as measured.

You are not addressing my arguments in this post http://forum.nasaspaceflight.com/index.php?topic=36313.msg1337106#msg1337106 but instead re-stating your reading of Paul March (he can argue this for himself, if he wants) and re-stating your belief.  It would be more constructive to analytically address the arguments that I posted here http://forum.nasaspaceflight.com/index.php?topic=36313.msg1337106#msg1337106 and/or the equations and assumptions in my paper, rather than repeatedly stating your prediction
Title: Re: EM Drive Developments - related to space flight applications - Thread 2
Post by: frobnicat on 02/25/2015 03:34 pm
.....
I have to think again about all that. ...
The mathematical equations I used are in the aforementioned paper, if you object to any of the equations and assumptions we would have a  better basis for discussion.

To state that the buckling forces produces a force (towards the right in the picture), in the opposite direction to the measured movement and force of the EM Drive (towards the left in the picture) is, I think, untenable.
...
Quickly, before I address the rest of your post in more depth... I wouldn't insist on that point if I didn't feel that the opposite is untenable  :)

I state that the buckling forces impart a (transient) momentum transfer from the frustum to balance's arm and that this transient momentum transfer is to the right, as a result the balance arm would like to recoil (move) to the right. If this buckling was the only action taking place, the observed step would be a fall, not a rise. Since what is observed is a rise, there must be other action taking place and imparting more momentum to the left than the buckling imparts to the right. Force is only "transferred momentum flux" but maybe my point is more clear if you see it as momentum transfer rather than force.

Sorry, partial answers encourages posting past each other, I will address you points as analytically as possible...
Title: Re: EM Drive Developments - related to space flight applications - Thread 2
Post by: Notsosureofit on 02/25/2015 04:20 pm
FYI

Anytime I hear the term "Two-Photon Loss"...........

http://arxiv.org/pdf/1412.4633v1.pdf

"In conclusion, we have realized a non-linearly driven-dissipative oscillator which spontaneously evolves towards the quantum manifold spanned by two coherent states."

And

"This was achieved by attaining the regime in which the photon pair exchange rate is of the same order as the single photon decay rate. The ratio between these two rates can be further improved within the present technology by using a higher Q oscillator and increasing its on-linear coupling to the bath."

For whatever it's worth.  (requires non-linear elements.  surface ? also ignored some terms ?)
Title: Re: EM Drive Developments - related to space flight applications - Thread 2
Post by: Rodal on 02/25/2015 05:11 pm
FYI

Anytime I hear the term "Two-Photon Loss"...........

http://arxiv.org/pdf/1412.4633v1.pdf

"In conclusion, we have realized a non-linearly driven-dissipative oscillator which spontaneously evolves towards the quantum manifold spanned by two coherent states."

And

"This was achieved by attaining the regime in which the photon pair exchange rate is of the same order as the single photon decay rate. The ratio between these two rates can be further improved within the present technology by using a higher Q oscillator and increasing its on-linear coupling to the bath."

For whatever it's worth.  (requires non-linear elements.  surface ? also ignored some terms ?)

Excellent point.  I completely agree.

Concerning the dielectric: perhaps it is not just a question of the well knonw dielectric constant: relative permittivity.  Perhaps there are nonlinear terms involving material constants characterizing the nonlinear material behavior of the dielectric that are not usually measured, (because such nonlinear terms are usually neglected) and hence we have not taken into account.  Helical, chiral anisotropy of the dielectric, as mentioned by Mulletron, is always an issue, as the analysis has so far been isotropic.  Anisotropic analysis would involve many more material constants.
Title: Re: EM Drive Developments - related to space flight applications - Thread 2
Post by: Flyby on 02/25/2015 05:31 pm
@Flyby

The second type distortion you refer to is a perspective distortion.  All of that is true and in addition the camera wasn't held on a flat level when the photo was taken.  I've rotated it a few degrees clockwise to compensate for this problem but I might still be off of the horizontal by a fraction of a degree.  At any rate my experience is that these distortions are so small that they don't significantly affect the accuracy of the estimate, which is a just that, a rough estimate give or take a couple mms.

If you have the software and are able to correct for some of the distortion, I'll recalculate.  I'm not sure that it matters much but I'd be happy to do so.
Well, it might be interesting to pretend we do not know the real dimensions and see if such a photo-touch up can be used to make more accurate measures.. or... that it proves to be totally useless...

I do feel a bit uncomfortable about the tilt-correction, but ignoring it would mean you have to resort to a 3 point perspective, which is highly uncomfortable (horizon points far far away) considering you have a near 1-point perspective...

but... let's give it a spin...if it hits a wall, then we'll know... :)
Title: Re: EM Drive Developments - related to space flight applications - Thread 2
Post by: RanulfC on 02/25/2015 05:51 pm
FYI

Anytime I hear the term "Two-Photon Loss"...........

Strangely enough every time I hear/see that my mind switches it to:
"Two Photon's enter... One Photon leaves" and Tina Turner singing in the background...

Weird :)

Randy
Title: Re: EM Drive Developments - related to space flight applications - Thread 2
Post by: Rodal on 02/25/2015 06:40 pm
@Flyby

The second type distortion you refer to is a perspective distortion.  All of that is true and in addition the camera wasn't held on a flat level when the photo was taken.  I've rotated it a few degrees clockwise to compensate for this problem but I might still be off of the horizontal by a fraction of a degree.  At any rate my experience is that these distortions are so small that they don't significantly affect the accuracy of the estimate, which is a just that, a rough estimate give or take a couple mms.

If you have the software and are able to correct for some of the distortion, I'll recalculate.  I'm not sure that it matters much but I'd be happy to do so.
Well, it might be interesting to pretend we do not know the real dimensions and see if such a photo-touch up can be used to make more accurate measures.. or... that it proves to be totally useless...

I do feel a bit uncomfortable about the tilt-correction, but ignoring it would mean you have to resort to a 3 point perspective, which is highly uncomfortable (horizon points far far away) considering you have a near 1-point perspective...

but... let's give it a spin...if it hits a wall, then we'll know... :)

What software did you use to do that?

I have in the past used Mathematica to transform images like that but one has to write Mathematica code to get it just right like you did.
Title: Re: EM Drive Developments - related to space flight applications - Thread 2
Post by: frobnicat on 02/25/2015 06:51 pm
.....
I have to think again about all that. ...
The mathematical equations I used are in the aforementioned paper, if you object to any of the equations and assumptions we would have a  better basis for discussion.

Well first, thanks for taking time trying to answer my concerns, and it's only natural you link back to your previous work on the topic for me to work my argument. But frankly I would have a hard time following all this work (thermal buckling of end caps) on details. I'll try an illustrated qualitative attempt at explaining my view and we will see if it's worth going on to equations.

Quote from: Rodal
To state that the buckling forces produces a force (towards the right in the picture), in the opposite direction to the measured movement and force of the EM Drive (towards the left in the picture) is, I think, untenable.

Here are two tenable positions that can be taken:

1) The buckling force cannot produce any motion of the center of mass of an unconstrained EM Drive free-floating in space. If one assumes simply supported boundary conditions at the edges of the big diameter plate, just before instability takes place,  the end plate is flat and hence the membrane forces are balanced by the rigid circular rim.  These membrane forces are directed radially from the rim towards the center, they are self-balanced.  There are no forces perpendicular to the plate.  If one assumes that the buckling instability takes place instantaneously , superluminally, in the buckled state, there are no membrane forces if the plate is infinitesimally thin, such that it will take whatever bent position is necessary to accommodate the required membrane thermal expansion.  Since there are no membrane forces in the final buckled state, there are no forces at the simply supported edges at all in the buckled state.  Hence there are no forces to move the EM Drive before and after the buckled state, and that's all, because the buckling motion is assumed to take place instantaneously (hence there are no forces to be considered to take place in between the initial flat state and the final buckled state).

This would be a model very far from reality. In all cause, if a part mass is sent to movement (relative to a free floating assembly) instantaneously rather than progressively, as a limit case, then one would observe the same total recoil for the rest of assembly (at equal end relative speed) only that the force would be concentrated in a Dirac delta of same integrated momentum value. Not understanding where you are heading with that one. We are (well, I am) concerned by the first initial answer of the pendulum from the initial part of buckling (when there is a growing acceleration of mass of plate relative to frustum). The limit case of instantaneous move would make hard to see that we have first a recoil (opposite direction from that the part is leaving) then a recoil in the same direction as the part's relative speed when it breaks (if it is not forever leaving the assembly, at some point it will have to break). The integrated sum of the observed force (of the moving object on rest of assembly) for a bound object between an initial rest position and a final rest position would be 0, is it what you are saying in green ?

Quote
2) If one assumes that buckling cannot take place instantaneously, but that its maximum speed has to be limited by the velocity of stress waves in the material, that is, by the speed of sound in the material, then the minimum time interval for buckling to take place is governed by the speed of sound.  In the paper I derived a speed and acceleration for the buckling motion under some assumptions that are explicitly stated (see equations).  Those assumptions can indeed be criticized.  The equations can also be criticized, and you are invited to do so.  The derived speed is below the speed of sound in the material, so it satisfies this physical condition (that the buckling speed should be lower than the speed of sound in the material).  After the onset of buckling, and during this (short time) buckling motion, the membrane force decreases, from a maximum at the onset of buckling to a minimum at the final buckling state.  During this (short time) the plate is in a bent condition, with increased bending shape with time.  At the edges of the plate, the bending of the plate produces a slope at the simply supported edges.  The membrane force at the edges can be decomposed into two directions: A) a component that is in the original flat direction.  This component is self-balanced.  B) The other force vector component is directed in the same direction (towards the left) as the motion of the EM Drive.   This is a very short-time impulsive force, calculated to be of the same magnitude as the initial transient rise force in the experiments.

If necessary I will try to dive in you equations but please try to understand what I'm saying, the specifics don't make difference for the argument.
On attached pictures I will use a simplified model of buckling end Plate as a pure mass P connected to massless springs S all around. Those springs hook on the Rim R (lighter blue) that is part of a solid bloc comprised of the frustum + arm (where I assume all in blue has no deformation). This could be freely floating in deep space but I figure the connection to Chamber C through a spring B (Balance's flexure pivots that tend to keep the system at a given rest position figured by 0). This model's springs S are pure tension/compression with no flexure stiffness, this is obviously oversimplified for the final rest position but right now I'm not interested in the final rest position (when buckling is ended). This system does exhibit buckling.
That's what I have in mind when I say that inward buckling to the left will appear as a kick to the right for the system :

1. System at rest before power on, Springs at their rest length, no forces (gravity neglected).

2. Heating of springs increase their rest length, they are now too short, they work in compression, but the symmetry makes opposite forces equilibrate, with no horizontal components. Note the forces of mass P on rim R are equal but opposite to the forces of rim R on mass P (point of application shifted for better clarity). No movement of arm yet, d2=d1.

3. The symmetry breaks leftward, the buckling starts. Mass P move to the left, in order for this move to occur, at least initially (regardless of quantitative equations or details of model) the mass P has to accelerate to the left (red arrow to the left with high magnitude). This is consistent with the now inclined springs still working in compression : the sum of the Frp forces (rim on plate CoM P) cancel radially but are indeed axially oriented to the left. Conversely the sum of the Fpr forces (plate's CoM P on rim) cancel radially but are axially oriented to the right. Action/reaction, a mass starts to leave leftward because the system push it leftward, the leaving mass "pushes back" on the system rightward. The whole balance system sees a force exerted on it that is oriented to the right. The balance departs from it's initial rest position to the right. The restoring torque of the balance flexure pivots will start to oppose this movement ( Fcr chamber on arm ) but significant displacement of balance has to occur first, Fcr < Frp in magnitude (Fcr is proportional to the deviation of balance relative to rest position).
d3<d2

4. As the buckling continues at its finite speed (not only because of inertia of mass P but also because the heating, ie increase of springs' rest length, is progressive and accompanies the movement) the acceleration of P to the left will slow down as the effective length of springs start to match their (ever increasing) rest length. Horizontal sum of Fpr that pushes to the right decreases, and the pendulum's arm travelled enough to the right that the restoring force Fcr now can be higher magnitude than Fpr, the arm is still travelling to the right but is now slowing down (acceleration to the left, as per the vector sum Fpr+Fcr)

5. ... I'm exhausted, to be continued (if worth)

From 3 to 4 : the consequence of an "inward oil canning" alone is a kick to the right initially. Do I have to write down equations ? For me this is more a matter of proper forces orientation conventions than anything else. I hope this is clear enough as to why I said that buckling would kick opposite to the observed thrust. So, what's wrong with this way of seeing orientations ?
Title: Re: EM Drive Developments - related to space flight applications - Thread 2
Post by: lasoi on 02/25/2015 07:02 pm
In straightening up the vertical lines, the program created more distortion than it corrected.  The bottom left side of the engine is pulled down to the left, noticeably.

I would do more calculations but unfortunately my day job is preventing me at the moment.  I'd be interested to see any estimates you might get from the altered photo however.
Title: Re: EM Drive Developments - related to space flight applications - Thread 2
Post by: Rodal on 02/25/2015 07:19 pm
...
That's what I have in mind when I say that inward buckling to the left will appear as a kick to the right for the system :

....
From 3 to 4 : the consequence of an "inward oil canning" alone is a kick to the right initially. Do I have to write down equations ? For me this is more a matter of proper forces orientation conventions than anything else. I hope this is clear enough as to why I said that buckling would kick opposite to the observed thrust. So, what's wrong with this way of seeing orientations ?
Get rid of the pendulum and everything else for the time being, as it further muddles the picture. 
Just consider a free-body diagram for the EM Drive. http://en.wikipedia.org/wiki/Free_body_diagram
Just consider the flat plate and a rigid ring around it and ask yourself in which direction you have to push the plate's center for the plate's center to move towards the left.  Of course that one has to have a force moving to the left.

All those words and images above are associated with a discussion of a reaction from the pendulum
You could use all those words to also talk about a reaction from the pendulum concerning EM Drive's thrust, and get into a similar mambo jambo as Shawyer's discussion of thrust in EM Drives.

First you have to address in which direction is the movement of the buckled shape (it is towards the left), and when it moves towards the left , in which direction it is pulling the supports (towards the left).  The buckled material cannot move towards the left without pulling on the supports towards the left: it is all connected.

What you are discussing instead is what is the effect of the buckling force pulling the supports to the left, on the pendulum.  That is an entirely different question.  You can ask yourself exactly the same question regarding what is the effect of the EM Drive moving towards the left, with a force pulling towards the left, on the pendulum.

After you have accepted that fact, you can talk about the reaction, which involves elastic deformation, and it is also present for the EM Drive thrust.

And, it is unclear why you think that this is so relevant, because the flat plate can theoretically buckle towards the left or towards the right (if there would be no plastic insulation on the outside and if the copper would be very thin) depending on initial imperfections.  An even if you think that when the buckled plate moves towards the left it gives a force towards the right (which is a misscommunication based on the fact that you are focusing on the pendulum's reaction rather than the buckling force itself, but for argument sake's) then even per your admission then if the plate buckles towards the right according to you would produce a force towards the left.

It seems to me that if you wanted to argue about this for intellectual reasons, a much safer ground would  be to say that the buckling force is self-balanced and that it produces no forces on the center of mass, either to the left or to the right (for the aforementioned reasons in my  above post).

EDIT: And having said that, the thermal expansion explanation by Oak Ridge does not make any sense in this case for the HD PE  (to think that an unrestrained, homogeneous, isotropic, free to expand material, will produce a force when expanding.  Thermal expansion changes the VOLUME of a material.  The mass stays the same.  If free to expand, then the density of the HD PE will change (larger volume, same mass = lower density).  Thermal expansion produces forces only when there is a thermal gradient through the material or the material is anisotropic, or the most general case: when the material is constrained so that it cannot expand !!!!! )

Title: Re: EM Drive Developments - related to space flight applications - Thread 2
Post by: frobnicat on 02/25/2015 11:37 pm
...
That's what I have in mind when I say that inward buckling to the left will appear as a kick to the right for the system :

....
From 3 to 4 : the consequence of an "inward oil canning" alone is a kick to the right initially. Do I have to write down equations ? For me this is more a matter of proper forces orientation conventions than anything else. I hope this is clear enough as to why I said that buckling would kick opposite to the observed thrust. So, what's wrong with this way of seeing orientations ?
Get rid of the pendulum and everything else for the time being, as it further muddles the picture.  Just consider the flat plate and a rigid ring around it and ask yourself in which direction you have to push the plate's center for the plate's center to move towards the left.  Of course that one has to have a force moving to the left.

To make the plate start to move to the left, I have to push the plate to the left. Certainly.
Now, on what else am I pushing when I push on the plate ? Either there is nothing else, in this case I am only "pushing on my inertia" i.e. recoil to the right. Or I am also pushing with my feet on a wall of the spacecraft and in this case me+spacecraft recoils to the right (a bit less, but still).

Quote
All those words and images above are associated with a discussion of a reaction from the pendulum
You could use all those words to also talk about a reaction from the pendulum concerning EM Drive's thrust, and get into a similar mambo jambo as Shawyer's discussion of thrust in EM Drives.

No, I am very clear and very standard as for the conventions of usual solid mechanics. When an object A and an object B exchange a force, the force A on B is noted Fab. The observable acceleration of object B (assuming no other forces) will be in the same direction as Fab, this leaves no mambo jambo to speak of. Is it a test ?

Now Newton's third law still states that for every action there is an equal and opposite reaction.

We have a clear unambiguous way to tell the orientations conventions from mambo jambo (observable acceleration) and we have Fab = -Fba (vectors).

If the pendulums restoring spring creates confusion, the very same arguments as for the direction of the start of the apparent displacement of rim of a free floating frustum are valid, see attached picture with pendulum's spring removed.

Note how a displacement of green mass (representing the plate) to the left implies at least initially an acceleration to the left. Being only linked to the rim, this acceleration of the plate must be due to a force F_rim_on_plate oriented to the left, and by Newton's third law there is a force F_plate_on_rim that is oriented to the right, and by definition (of the above mentioned usual convention) the rim and rest of frustum (assumed solid) accelerates to the right. Hope some passer-bys will enjoy.

Quote
First you have to address in which direction is the movement of the buckled shape (it is towards the left), and when it moves towards the left , in which direction it is pulling the supports (towards the left).  The buckled material cannot move towards the left without pulling on the supports towards the left: it is all connected.

Yes the movement is toward the left, and when it move toward the left it is not pulling the support, because if it would be pulling the support when departing (to the left) of its initial rest position then it would not be departing from its rest position in the first place ! That would be the case if the springs where in tension, and a coplanar plate were a stable situation (like for a drum). Only because the rise in temperature increases the rest length of the springs the springs will be compressed, and the situation becomes unstable as the spring want to push.

Now all right, this is only a toy model, the real situation is much more complex because plate's weight is distributed on the whole surface, because plate is very stiff in compression and more supple in flexure so that for instance when a static equilibrium is reached (when there is no more buckling movement) there are still radial forces (but no longer axial ones, that we agree) while in my toy model the complete absence of flexure stiffness leaves no radial forces when the static equilibrium is reached. + copper/epoxy left/right asymmetry of the sheet... damping...

But even if considering a quasi-static evolution in the real situation, where the rise of temperature is slow enough for the plate to always be near static equilibrium, near equilibrium is not exactly equilibrium. That's the whole point : however small is the deviation from perfect static equilibrium, this is this deviation that makes inertial things move in the first place. And whenever a part of mass m of a free floating system moves to the left a distance x, the rest of the system of mass M recoils to the right a distance X=x*m/M (where x and X are measured relative to initial inertial rest frame of the free floating system before the part started to move).

Quote
What you are discussing instead is what is the effect of the buckling force pulling the supports to the left, on the pendulum.  That is an entirely different question.  You can ask yourself exactly the same question regarding what is the effect of the EM Drive moving towards the left, with a force pulling towards the left, on the pendulum.

? What I was discussing is the effect of the buckling recoil (momentum...) pushing the supports to the right, transiently. Main point. Then the plate would slow down, giving an equal amount of momentum in the opposite direction (to the left) and neutralise the speed acquired by the initial kick. That's for a free floating device.

Quote
After you have accepted that fact, you can talk about the reaction, which involves elastic deformation, and it is also present for the EM Drive thrust.

And, it is unclear why you think that this is so relevant, because the flat plate can theoretically buckle towards the left or towards the right (if there would be no plastic insulation on the outside and if the copper would be very thin) depending on initial imperfections.  An even if you think that when the buckled plate moves towards the left it gives a force towards the right (which is a misscommunication based on the fact that you are focusing on the pendulum's reaction rather than the buckling force itself, but for argument sake's) then even per your admission then if the plate buckles towards the right according to you would produce a force towards the left.

I do think that for the buckled plate to move (have a speed) toward the left it has to accelerate to the left, and that when the buckled plate accelerates towards the left it gives to the frustum (and attached mass) an apparent force towards the right (the frustum accelerates to the right, regardless of later dynamics due to it being attached to position restoring torque, ie a pendulum, so please forget pendulum if you want, my argument still holds).

This is so relevant at least because we are discussing here a potentially groundbreaking device that has an enigmatic relation to momentum conservation : the least we can do is to put such mundane engineering matter straight, on a forum read by people who breath (conventional) action/reaction... and contributions are useful if they stand correct (or can be read correctly if it is case of miscommunication).
If the buckling effect adds an initial downward step to the other effects, this is good to know, no ? An other interesting effect would look differently (ie. more square) when this negative term would be subtracted...

Quote
It seems to me that if you wanted to argue about this for intellectual reasons, a much safer ground would  be to say that the buckling force is self-balanced and that it produces no forces on the center of mass, either to the left or to the right (for the aforementioned reasons in my  above post).

By self-balanced you mean quasi-static, always near equilibrium ?
Anyway, if you prefer (and yes, this is probably more elegant) :
Self repeat : whenever a part of mass m of a free floating system moves to the left a distance x, the rest of the system of mass M recoils to the right a distance X=x*m/M (where x and X are measured relative to initial inertial rest frame of the free floating system before the part started to move).
Because : the centre of mass of the whole system m+M can't depart from its initial inertial trajectory.

Quote
EDIT: And having said that, the thermal expansion explanation by Oak Ridge does not make any sense in this case for the HD PE  (to think that an unrestrained, homogeneous, isotropic, free to expand material, will produce a force when expanding.  Thermal expansion changes the VOLUME of a material.  The mass stays the same.  If free to expand, then the density of the HD PE will change (larger volume, same mass = lower density).  Thermal expansion produces forces only when there is a thermal gradient through the material or the material is anisotropic, or the most general case: when the material is constrained so that it cannot expand !!!!! )

??? When thermal expansion displaces a part of a system relative to the rest of a system, the rest of the system will recoil. Whatever displaces a part of a system relative to the rest of the system, this displacement implies a force F_system_part. The rest of the system will recoil. Recoil is the acceleration due to the opposite force (F_part_system). An unrestrained, homogeneous, isotropic, free to expand but in one direction material, will produce a force when expanding against the wall against which it rests.

Why is it so hard to reach consensus now ?
Title: Re: EM Drive Developments - related to space flight applications - Thread 2
Post by: Rodal on 02/26/2015 01:14 am
...
I like your drawings with the buckled condition idealized as a beam with three hinges: one hinge at the center and a hinge at each end. :)

Thank you very much for taking the time to make those drawings.   :)

I should have looked at that initially. I agree with your drawing.  To the extent that my prior wording disagreed with your drawing, my prior words were incorrect, when and if they referred to the thermal force Fpr.

Let's then address what happens in the opposite case that the plate instead buckles to the right:

The flat plate can theoretically buckle towards the left or towards the right (if the copper is thin enough) depending on initial imperfectionsIf when the buckled plate moves towards the left it gives a force towards the right then  if the plate buckles towards the right it would produce a force towards the left, in the same direction as the EM Drive's motion, do you agree?.

Then, if this buckling analysis is correct, it gives transient force that is towards the left when the plate buckles towards the right and it gives a transient force to the right when the plate buckles towards the left.

Now I have to give further thought to which direction the plate buckles when heated.  The real plate has a neutral surface that is not at the middle of the cross-section.  It is really a bi-material thermostat: with the epoxy expanding much more than the very thin copper coating.   If this unsymmetric laminate would be exposed to a uniform temperature it would expand towards the outside, producing a force towards the left, like the EM Drive force.  The question is what happens under the superposed thermal gradient through the thickness. The IR Camera shows very pronounced heating on the outside surface....
Title: Re: EM Drive Developments - related to space flight applications - Thread 2
Post by: Rodal on 02/26/2015 01:33 am
...
??? When thermal expansion displaces a part of a system relative to the rest of a system, the rest of the system will recoil. Whatever displaces a part of a system relative to the rest of the system, this displacement implies a force F_system_part. The rest of the system will recoil. Recoil is the acceleration due to the opposite force (F_part_system). An unrestrained, homogeneous, isotropic, free to expand but in one direction material, will produce a force when expanding against the wall against which it rests.

Why is it so hard to reach consensus now ?
With respect to the HD PE dielectric, now I see your thinking.  I think you think of it as a dynamic problem, for example, if instead of the HD PE we would have an explosive inside the cavity glued to the left wall, the effect of the explosive would be to force the wall towards the left.

However, thermal expansion does not work like that.  There are no dynamic forces caused by a material experiencing thermal expansion such that it is free to expand.  Thermal expansion is a very slow process that does not involve second order derivatives with respect to time.  If a uniaxial isotropic material is glued to a wall, it will just expand without producing any force whatsoever on the wall it is glued to. 

If free, it will just expand, strain = alpha *deltaT.

No stress if it is free to expand.

Can't use thermal expansion as a form of propellant-less propulsion.
Title: Re: EM Drive Developments - related to space flight applications - Thread 2
Post by: ThinkerX on 02/26/2015 03:41 am
Going from the last eight or ten posts, this device really needs to be tested in a vacuum with a different measuring device, just to put paid to these 'tilt' / thermal issues.
Title: Re: EM Drive Developments - related to space flight applications - Thread 2
Post by: zen-in on 02/26/2015 05:04 am
I have some new observations and theory.   I lifted the first picture from the Aug. '14 paper, which shows the linear displacement sensor.   This device tracks the position of a reflected laser dot on a CMOS image sensor.  This is usually done by calculating the center of luminance of the laser dot; a measurement that has an accuracy of a small fraction of the width of a pixel.   The second picture describes a typical LDS that has sub-micron accuracy.

The only control or "NULL" experiment described in the Aug. 14 paper related to the Eagleworks device was when the dummy load was used instead of sending RF into the cavity.   This of course shields the RF very well.   The dummy load is 50 Ohms and so the SWR is 1:1.   However when the cavity is loaded and the dielectric material is inside the cavity it's possible the SWR is much higher.    This would result in RF being reflected back to the amplifier and being radiated from the shield of the RF cable.   This is what happens when the SWR is not 1:1.   It's possible this RF noise is interfering with the LDS.  When the dielectric is not inside the cavity the SWR is lower so no interference takes place.    This theory agrees with the results of the cannae test as well.

The last picture, also from the Aug. '14 paper shows a negative slope on the baseline position ( no thrust) after each RF pulse.   The first one appears to level off just before the final RF pulse.   After that pulse it heads down again.   I believe the thermally induced change in the CoM of the emdrive causes the balance arm to rotate.   This very slight rotation reduces the reflection distance for the laser beam.
Title: Re: EM Drive Developments - related to space flight applications - Thread 2
Post by: RotoSequence on 02/26/2015 07:35 am
I have some new observations and theory.   I lifted the first picture from the Aug. '14 paper, which shows the linear displacement sensor.   This device tracks the position of a reflected laser dot on a CMOS image sensor.  This is usually done by calculating the center of luminance of the laser dot; a measurement that has an accuracy of a small fraction of the width of a pixel.   The second picture describes a typical LDS that has sub-micron accuracy.

The only control or "NULL" experiment described in the Aug. 14 paper related to the Eagleworks device was when the dummy load was used instead of sending RF into the cavity.   This of course shields the RF very well.   The dummy load is 50 Ohms and so the SWR is 1:1.   However when the cavity is loaded and the dielectric material is inside the cavity it's possible the SWR is much higher.    This would result in RF being reflected back to the amplifier and being radiated from the shield of the RF cable.   This is what happens when the SWR is not 1:1.   It's possible this RF noise is interfering with the LDS.  When the dielectric is not inside the cavity the SWR is lower so no interference takes place.    This theory agrees with the results of the cannae test as well.

The last picture, also from the Aug. '14 paper shows a negative slope on the baseline position ( no thrust) after each RF pulse.   The first one appears to level off just before the final RF pulse.   After that pulse it heads down again.   I believe the thermally induced change in the CoM of the emdrive causes the balance arm to rotate.   This very slight rotation reduces the reflection distance for the laser beam.

Glad to see people still attempting to come up with null-thrust ideas on here.  ;) How does this one account for the measured loss of thrust when the PTFE disk in the resonator cavity came loose after the nylon support bolt melted?
Title: Re: EM Drive Developments - related to space flight applications - Thread 2
Post by: MikeAtkinson on 02/26/2015 08:17 am
frobnicat I like your diagrams, very clear.

Note that as the buckling completes the end plate will be at rest with the frustum. This means that there will be an acceleration of the end plate in the opposite direction (at some time after your diagram 4).

After the buckling completes (and all accelerations are zero) the centre of mass will not have changed. However the position of the centre of mass relative to the connection point to the torsion balance may have changed.

From this I think the torsion balance should see a force in one direction, then some time later a force in the other direction. These forces integrated should be zero.

After the buckling has completed the torsion balance should be showing no force, however as the shape of the frustum + end plate has changed the position of the external surfaces has also changed. If the force is being measured as a change in the position of the frustum then a small force would erroneously be shown (I don't think this is the case, but something to check in the experimental set-ups).
Title: Re: EM Drive Developments - related to space flight applications - Thread 2
Post by: Flyby on 02/26/2015 10:29 am

What software did you use to do that?

I have in the past used Mathematica to transform images like that but one has to write Mathematica code to get it just right like you did.
While formulating the steps i've used to get to that result, I've concluded I made an error in the last correction step. I took the wrong side for the horizontal (camera yawn distortion) perspective correction.
I'll correct it this evening, local time.. too much to do atm...
Title: Re: EM Drive Developments - related to space flight applications - Thread 2
Post by: Mulletron on 02/26/2015 08:44 pm

Dr. Rodal & Notsosureofit:

We had an interesting failure in the Eagleworks lab yesterday.  That being I was getting ready to pull a vacuum on our copper frustum mounted in its "reverse" or to the right thrust vector position and ran a preliminary data un to see if it was performing in air as it had two weeks ago just before our last RF amplifier died.  Sadly it wasn't for it was producing less than half of what it did before and in the wrong direction!   

I had Dr. White come in and take a look over my latest test article installation last night and he found that the center 1/4"-20 nylon PE disc mounting bolt that holds the second PE disc to the small OD frustum's PCB endplate was no-longer tensioned as it had been before.  In fact it had partially melted at the interface between the two PE discs thus relieving the strain induced by its bolts threads and nut.  (There are three ~1.00" 1/4-20 nylon bolts mounted on a ~2.00" radius spaced every 120 degrees that hold the first PE disc to the PCB end cap.   There is then a layer of 3/4" wide office scotch tape at the interface between the first and second PE discs and the center 1/4"-20 nylon bolt that hold second PE disc to the first PE disc.) 

Apparently not having the PE discs firmly mounted to the frustum's small OD end cap hindered the thrust producing mechanism that conveys the generated forces in the PE to the copper frustum.  And/or the melted nylon was hogging all the RF energy in the PE discs due to its higher dissipation factor in its semiliquid state.  Either way it looks like there is a high E-field volume where this center nylon bolt hangs out while running in the TM212 resonant mode.  Too bad Teflon bolts are so weak even in comparison to the nylon, for its dissipation factor is at least two orders of magnitude lower than the nylon's.

Best, Paul M.

This was probably one of the most important revelations to come out of Eagleworks yet, other than it works in vacuum. For those who are trying to figure out how it works, wow. It has really gotten my gears turning over the last few days. I hope Paul comes back with more about this, like what happens if a tiny gap is intentionally placed between the PE and copper. The PE and copper should be Casimir attractive while in close contact, but it looks like it became repulsive? somehow upon introduction of the tiny gap introduced by the bolt not holding. This is a significant discovery if it can be repeated, and even better......explained. It has really had me scratching. I don't get the reverse part. What does this mean!??

This link got me chasing down leads. I wish MIT would have provide a reference to what they're talking about, top image: http://www.mit.edu/~kardar/research/seminars/Casimir2010/talks/MITlunch/OffEquil.html

There's a lot of similar stuff out there about this kind of "induced repulsion" like this:
http://arxiv.org/find/quant-ph/1/au:+Chen_F/0/1/0/all/0/1 and this
http://arxiv.org/pdf/1201.5585.pdf refs 27,28

But so far nothing that seems to fit the conditions described within emdrive.
Title: Re: EM Drive Developments - related to space flight applications - Thread 2
Post by: Mulletron on 02/27/2015 01:29 am
It was never explicitly stated why Shawyer got rid of the dielectric section. Thinking about it tells me it was probably due to heat. And the thruster still works without it.
I've been reading up more on Cr2O3 lately due to its magnetoelectric properties and mention throughout the literature. I discovered that it can withstand really high temperatures. I wonder if this material would help improve the thrust.
http://en.wikipedia.org/wiki/Chromium(III)_oxide
Title: Re: EM Drive Developments - related to space flight applications - Thread 2
Post by: zen-in on 02/27/2015 02:55 am
I have some new observations and theory.   I lifted the first picture from the Aug. '14 paper, which shows the linear displacement sensor.   This device tracks the position of a reflected laser dot on a CMOS image sensor.  This is usually done by calculating the center of luminance of the laser dot; a measurement that has an accuracy of a small fraction of the width of a pixel.   The second picture describes a typical LDS that has sub-micron accuracy.

The only control or "NULL" experiment described in the Aug. 14 paper related to the Eagleworks device was when the dummy load was used instead of sending RF into the cavity.   This of course shields the RF very well.   The dummy load is 50 Ohms and so the SWR is 1:1.   However when the cavity is loaded and the dielectric material is inside the cavity it's possible the SWR is much higher.    This would result in RF being reflected back to the amplifier and being radiated from the shield of the RF cable.   This is what happens when the SWR is not 1:1.   It's possible this RF noise is interfering with the LDS.  When the dielectric is not inside the cavity the SWR is lower so no interference takes place.    This theory agrees with the results of the cannae test as well.

The last picture, also from the Aug. '14 paper shows a negative slope on the baseline position ( no thrust) after each RF pulse.   The first one appears to level off just before the final RF pulse.   After that pulse it heads down again.   I believe the thermally induced change in the CoM of the emdrive causes the balance arm to rotate.   This very slight rotation reduces the reflection distance for the laser beam.

Glad to see people still attempting to come up with null-thrust ideas on here.  ;) How does this one account for the measured loss of thrust when the PTFE disk in the resonator cavity came loose after the nylon support bolt melted?

I haven't seen the data from that experiment so am not able to comment on it.   The laser distance sensor has a very high resolution and an event like that would be difficult to interpret if a plot was disclosed.

I think the LDS is probably an interferometer as diagrammed below.    For red light (650 nM) it would take a movement of just 325 nM for each peak of the fringe.  For a slowly moving TP the interferometer's output sine wave would be in the audio region.  However a simple interferometer is not able to detect what direction the mirror is moving.   Another LDS uses different waveforms and a lot of post processing to get direction of movement as well as relative distance.   Both use transimpedance amplifiers that are sensitive to interference.   If RF interference is the explanation for this anomalous force one would expect the force indication to be in both directions.   However there may be some aspect of the experimental setup or the post processing inside the LDS that precludes this.    Since there are so few experimental results disclosed (waveforms), it's difficult to know what is happening.
Title: Re: EM Drive Developments - related to space flight applications - Thread 2
Post by: aero on 02/27/2015 04:37 am
It's to bad that we can't find a way that one of the little known or unknown solutions to Maxwell's equations can cause  a momentum. Meep indicates that it does, and if we had a mechanism then it would be straight forward to design more effective thrusters using meep. For example, I modified the Copper Kettle only a little and meep got a 100 times increase in the force over the design tested. The cavity drawing is attached as is the force/power curve for gaps ranging from 1% down to 0.2% of the cavity height, not counting the added height from the pipes.

Note that the "Rim gap" line on the force/power curve is the same one I showed before for the Copper Kettle as Paul presented it. The "pipe" force is about 100 times larger, so the rim gap force plots as a straight line at zero by comparison. Oh, and I didn't just accidentally arrive at the pipe design, I have seen this and similar results for some time. It's now just a good time to share.

And yes, the force indicated by meep is 70 muN/Watt. That is about 21,000 times 1/c, a photon rocket. The cavity was simulated in a space environment, nothing near to interact with except vacuum.

I know people don't like the idea of evanescent waves causing the thruster force, and by looking at the field patterns it seems that the fields exterior to the cavity extend well beyond 1/3 wavelength. Maybe it is something else but still a solution to Maxwell's equations. I can't guess what that might be.

Unless of course it is surface electrons excited by the high power resonant RF, tunnelling through the 35 micron copper ends.
Title: Re: EM Drive Developments - related to space flight applications - Thread 2
Post by: JPLeRouzic on 02/27/2015 09:48 am
It's to bad that we can't find a way that one of the little known or unknown solutions to Maxwell's equations can cause  a momentum.
There is obviously momentum in electromagnetic waves. There are also some little known electromagnetic effects that create torque.

Quote
Unless of course it is surface electrons excited by the high power resonant RF, tunnelling through the 35 micron copper ends.
Yes, and many other things may happen were not tested nor even proposed. I wonder how people can know that a lot of energy is pumped in this device and imagine nothing will get out. At the very least thermal effects should happen. Testing it in (near) vacuum doesn't eliminate the thermal hypothesis. Even Pioneer's acceleration that was due to thermal effects after all: http://spectrum.ieee.org/aerospace/astrophysics/finding-the-source-of-the-pioneer-anomaly
Another thing that strikes me is that people search for a unique cause explaining everything, which is a bit unlikely.

One last thought: If a simulator shows results, build this device and publish results in a mainstream conference. Interesting things may happen ;-)
Title: Re: EM Drive Developments - related to space flight applications - Thread 2
Post by: frobnicat on 02/27/2015 09:51 am
I have some new observations and theory.   I lifted the first picture from the Aug. '14 paper, which shows the linear displacement sensor.   This device tracks the position of a reflected laser dot on a CMOS image sensor.  This is usually done by calculating the center of luminance of the laser dot; a measurement that has an accuracy of a small fraction of the width of a pixel.   The second picture describes a typical LDS that has sub-micron accuracy.

I thought the linear displacement sensor was based on photons time of flight of reflected light rather than parallax of diffused light dot position :
Quote from: anomalous thrust page 3...
Displacement of the pendulum arm is measured via a Linear Displacement Sensor (LDS). The primary LDS components consist of a combined laser and optical sensor on the fixed structure and a mirror on the pendulum arm. The LDS laser emits a beam which is reflected by the mirror and subsequently detected by the optical sensor. The LDS software calculates the displacement (down to the sub-micrometer level) based upon the beam reflection time. Prior to a test run data take, the LDS is positioned to a known displacement datum (usually 500 micrometers) via mechanical adjustments to its mounting platform. Gross adjustments are performed via set screws. Fine adjustments are performed using manually - operated calibrated screw mechanisms and a remotely controlled motorized mechanism that can be operated with the chamber door closed and the chamber at vacuum. The remote adjustment capability is necessary since the LDS datum will change whenever a change to the test facility environment affects the roll - out table or the chamber – e.g., whenever the chamber door is closed or latched and whenever the chamber is evacuated. Once the LDS displacement is adjusted in the final test environment, further adjustment between test run data takes is usually not required.

Do we have other precisions concerning the type of LDS used ? If is ToF (time of flight) reaching sub-micrometer would mean interferometry of some sort ? I see zen-in you have same conclusion in latest post.

Quote
The only control or "NULL" experiment described in the Aug. 14 paper related to the Eagleworks device was when the dummy load was used instead of sending RF into the cavity.   This of course shields the RF very well.   The dummy load is 50 Ohms and so the SWR is 1:1.   However when the cavity is loaded and the dielectric material is inside the cavity it's possible the SWR is much higher.    This would result in RF being reflected back to the amplifier and being radiated from the shield of the RF cable.   This is what happens when the SWR is not 1:1.   It's possible this RF noise is interfering with the LDS.  When the dielectric is not inside the cavity the SWR is lower so no interference takes place.    This theory agrees with the results of the cannae test as well.

If condition for the hypothesis is : amount of RF power radiated from the feed cable interferes with LDS values, then we need to know some precise details on LDS and how such interference could take place.

Also, do we know how the RF amplifier deals with varying degrees of back reflected power. Could it be conceivable that this back reflected power generates or modifies some current DC component somewhere and that this DC components push to a varying degree on damper's magnetic field ?

We know of at least one significant DC current component interaction with the damper's magnetic field : around 10µN upward (increasing distance) due to 5.6 A DC current in RF amplifier's power cable. Could this drawn DC current be changed (in significant ratio) due to reflected RF power levels ?

Another thing, those reflected power levels of varying degrees could change the heating rates of RF amplifier radiators, that are known to IR heat the flexure bearing and change the rest equilibrium point of balance. Albeit slowly.

Quote
The last picture, also from the Aug. '14 paper shows a negative slope on the baseline position ( no thrust) after each RF pulse.   The first one appears to level off just before the final RF pulse.   After that pulse it heads down again.   I believe the thermally induced change in the CoM of the emdrive causes the balance arm to rotate.   This very slight rotation reduces the reflection distance for the laser beam.

When integrating the momentum needed to keep the balance away from it's rest equilibrium point you see that relative movement a part's CoM relative to fixation point would have to be huge to account for such long term drift against a fixed rest equilibrium point. The most obvious explanation for those drifts is a change of rest equilibrium point of the balance (heating of flexure bearing blocks).

If we have  very roughly an apparent force drift of 30µN during 30s, that's 1µN/s, then F(t) = - 1e-6*t
If such F(t) is to be explained by a recoil of a moving part, such moving part of mass m would have to be accelerated (in the opposite direction, to the left) at  a(t) = -F(t) / m   that's v(t)=5e-7/m t²  and  x(t)= 1.7e-7/m t^3
The moving part's CoM would have to change position as the cube of time to explain such drifting force (against a stable equilibrium rest position of balance) as a recoil effect.
x(t)*m = 1.7e-7 t^3 = 5.6e-3 kg m at t=30s,  that is 1kg moving 5.6mm  or  5.6kg moving 1mm, at least (the drifting baseline shows now sign of reversing...) if rest equilibrium position is supposed stable.

So the most obvious explanation so far (as first given by Paul March) is that this drifting component of measured distance  is not due to a varying force but due to a varying equilibrium rest position (like tuning or detuning a scale when nobody is on it moves the readings).

Edits in blue, for clarification.
Title: Re: EM Drive Developments - related to space flight applications - Thread 2
Post by: Notsosureofit on 02/27/2015 01:16 pm
Weekend coming:

Where do we stand on the frequency, dimensions, power, mode and Q for the Shawyer cavity discussed above ?
Title: Re: EM Drive Developments - related to space flight applications - Thread 2
Post by: frobnicat on 02/27/2015 01:56 pm
frobnicat I like your diagrams, very clear.

Note that as the buckling completes the end plate will be at rest with the frustum. This means that there will be an acceleration of the end plate in the opposite direction (at some time after your diagram 4).

Yes, for a buckling of the right plate to the right (outward) there would be initially an impulse to the left (as seen from the rest of the frustum) then an impulse of equal but opposite momentum to the right when the buckling slows and stops. See attached charts.

Quote
After the buckling completes (and all accelerations are zero) the centre of mass will not have changed. However the position of the centre of mass relative to the connection point to the torsion balance may have changed.

The CoM of the whole system will not have changed for a free floating system, yes. Maybe it's better then to see the other way around since the whole system's CoM is immobile it can play a role of reference : the position of the connection point to the torsion balance relative to the centre of mass may have changed.
But the system's CoM is inertial (not changing) only if the system is isolated (free floating)

Quote
From this I think the torsion balance should see a force in one direction, then some time later a force in the other direction. These forces integrated should be zero.

Yes the force (kg m/s²) when integrated is the momentum (kg m/s) and there can be no net momentum difference in total between initial and final immobile positions of the frustum. Again, this is only valid for free floating system.

The balance arm experiences a recoil force, but we don't see a force, only a deviation in position caused by force. I'm not sure the deviation has to integrate to 0 (so we have to remember that deviation != force).

Quote
After the buckling has completed the torsion balance should be showing no force, however as the shape of the frustum + end plate has changed the position of the external surfaces has also changed. If the force is being measured as a change in the position of the frustum then a small force would erroneously be shown (I don't think this is the case, but something to check in the experimental set-ups).

Given the rest position restoring spring of the balance, a moving part can only impart a transient signal, when the moving part stops moving (relative to rest of system), the balance will restore to it's rest state equilibrium, ie showing a 0 force reading. There is no way for an ended (non longer evolving) deformation in the system on top of the balance arm to make a long lasting deviation from rest equilibrium.

It can be modelled as a part of mass m (green disc) being driven at varying distance L(t) relative to rest of the system of mass M (blue square), rest of the system being linked by a spring and damper to inertial rest frame of infinite mass (earth).

In the charts,
horizontal axis : time
vertical axis magenta : force of mass m on mass M
vertical axis black : distance between mass m and mass M,  L(t) that drives the excitation
vertical axis blue : acceleration of mass M
vertical axis green : velocity of mass M
vertical axis red : position of mass M, what would be measured as a result

Deviations upward correspond to movements to the left. All scales arbitrary at the moment (for illustration purpose only). At same scale, with m<<M the black step on L(t) would appear much greater than the resulting steplike response of position of arm (rpos, in red). For instance it would take a step of 10cm to the left of a mass m=100g to make a mass M=1kg to step to the right by 1cm.

cap1 : stiffness and damping to 0 (free floating)
cap2 : some stiffness (position restoring spring) but no damping
cap3 : slightly underdamped system
cap4 : slightly overdamped system

Note in the last case how it's not obvious that the integrated surface between red curve (positions readings) and the 0 axis (not shown, sorry) is indeed 0. I'll need to check. But if one is to interpret naively the red curve (position of balance arm) directly as force it would be tempting to say there is a net leftward average force. I will try to calibrate my model on the real values of the experiment.
Title: Re: EM Drive Developments - related to space flight applications - Thread 2
Post by: Rodal on 02/27/2015 02:08 pm
...
I like your drawings with the buckled condition idealized as a beam with three hinges: one hinge at the center and a hinge at each end. :)

Thank you very much for taking the time to make those drawings.   :)

I should have looked at that initially. I agree with your drawing.  To the extent that my prior wording disagreed with your drawing, my prior words were incorrect, when and if they referred to the thermal force Fpr.

Let's then address what happens in the opposite case that the plate instead buckles to the right:

The flat plate can theoretically buckle towards the left or towards the right (if the copper is thin enough) depending on initial imperfectionsIf when the buckled plate moves towards the left it gives a force towards the right then  if the plate buckles towards the right it would produce a force towards the left, in the same direction as the EM Drive's motion, do you agree?.

Then, if this buckling analysis is correct, it gives transient force that is towards the left when the plate buckles towards the right and it gives a transient force to the right when the plate buckles towards the left.

Now I have to give further thought to which direction the plate buckles when heated.  The real plate has a neutral surface that is not at the middle of the cross-section.  It is really a bi-material thermostat: with the epoxy expanding much more than the very thin copper coating.   If this unsymmetric laminate would be exposed to a uniform temperature it would expand towards the outside, producing a force towards the left, like the EM Drive force.  The question is what happens under the superposed thermal gradient through the thickness. The IR Camera shows very pronounced heating on the outside surface....
I have calculated the variables that govern which way the circular plate will buckle.

Using S. Timoshenko's classic solution to the bimaterial thermal expansion problem (Journal of the Optical Society of America, JOSA, Vol. 11, Issue 3, pp. 233-255 (1925) ) the radius of curvature can be expressed as:

radiusOfCurvature = (t1+t2)*(3(1+m)+(1+m*n)(m^2 +1/(m*n)))/(6*deltaAlpha*deltaT*((1+m)^2))

where I have expressed the following variables as non-dimensional ratios (allowing use of any consistent system of units, and revealing the important parameters, instead of the expression in Wikipedia that is awkwardly expressed in dimensional units, and expressed for the curvature instead of its reciprocal):

m=t1/t2  Thickness ratio
n=E1(1-nu1^2)/(E2(1-nu2^2))  Plate stiffness-ratio

and I have used the (plates and shells) stifness E1(1-nu1^2) taking into account the Poisson's ratio, since we are dealing with a plate instead of an uniaxial beam (this makes practically no difference for this case due to the very small values of the Poisson's ratio involved for FR4 and the very thin layer of copper).

and the differences in thermal expansion and temperature:

deltaAlpha=alpha2 - alpha1 
deltaT=T - To

t1 and t2 are the thicknesses of the layers
E1 and E2 are the Young's modulii
nu1 and nu2 are the Poisson's ratios

From Paul March we know:

t1=0.00138 inches
t2=0.063 inches

the material "1" is copper

Wikipedia and Engineering Toolbox give E1=17*10^6 psi

Wikipedia, NBS (Hidnert &Krider's classic article) and Engineering Toolbox give alpha1= 17*10^(-6) 1/degK 

and Wikipedia gives nu1=0.33

and that the material "2" is FR4

The following references give:

Wikipedia
ECW = 3*10^6 psi   alphaCW= 14*10^(-6) 1/degK   nuLW=0.136
ELW = 3.5*10^6 psi alphaLW= 12*10^(-6) 1/degK  nuCW=0.118

P-M Services (UK)
                               alphaCW= 15*10^(-6) 1/degK
                               alphaLW= 11*10^(-6) 1/degK

Leiton (Germany)
                               alphaCW= 17*10^(-6) 1/degK
                               alphaLW= 12*10^(-6) 1/degK

It is trivial to show that the sign of the curvature (which way the plate is going to buckle) is governed only by the difference in thermal expansion coefficients between the two layers (and of course whether deltaT is positive or negative).

Bimaterial thermal bending will take place along the anisotropic in-plane direction with the highest thermal expansion and lowest modulus.  It is obvious that this is the CW direction.

It is immediately obvious, that using Leiton's (Germany) coefficient of thermal expansions, the copper/FR4 laminate circular plate will not experience any bimaterial bending whatsoever because according to Leighton alphaCW of FR4 is exactly the same as the universally accepted alpha1 of copper.

The thermal IR camera shows temperature readings ranging from below 79 deg F to a concentrated maximum at certain small spots of 94.3 deg F.

Using Wikipedia's properties, the plate will experience an extremely small amount of bimaterial bending towards the inside, for a temperature increase from 68 deg F to 79 deg F, with a huge radius of curvature exceding 7000 inches (practically flat, in relation to the thickness of only 0.064 inches).

Also, using Wikipedia's properties, the plate will experience an extremely small amount of bimaterial bending towards the inside, for a temperature increase from 68 deg F to 94.3 deg F, with a huge radius of curvature exceeding 3000 inches (practically flat, in relation to the thickness of only 0.064 inches).

CONCLUSION: due to the fact that FR4 has a coefficient of thermal expansion very similar to the one of copper, and that the thickness of the copper is extremely small compared to the thickness of FR4, the circular plate will experience either no bimaterial thermal bending whatsoever, or it will be extremely small (will stay practically flat) under the measured changes in temperature.  Therefore, bi-material bending due to a change in temperature is irrelevant to the buckling problem.  Buckling is instead governed by the plate's initial imperfect flatness

Title: Re: EM Drive Developments - related to space flight applications - Thread 2
Post by: Rodal on 02/27/2015 02:13 pm
...
I like your drawings with the buckled condition idealized as a beam with three hinges: one hinge at the center and a hinge at each end. :)

Thank you very much for taking the time to make those drawings.   :)

I should have looked at that initially. I agree with your drawing.  To the extent that my prior wording disagreed with your drawing, my prior words were incorrect, when and if they referred to the thermal force Fpr.

Let's then address what happens in the opposite case that the plate instead buckles to the right:

The flat plate can theoretically buckle towards the left or towards the right (if the copper is thin enough) depending on initial imperfectionsIf when the buckled plate moves towards the left it gives a force towards the right then  if the plate buckles towards the right it would produce a force towards the left, in the same direction as the EM Drive's motion, do you agree?.

Then, if this buckling analysis is correct, it gives transient force that is towards the left when the plate buckles towards the right and it gives a transient force to the right when the plate buckles towards the left.

Now I have to give further thought to which direction the plate buckles when heated.  The real plate has a neutral surface that is not at the middle of the cross-section.  It is really a bi-material thermostat: with the epoxy expanding much more than the very thin copper coating.   If this unsymmetric laminate would be exposed to a uniform temperature it would expand towards the outside, producing a force towards the left, like the EM Drive force.  The question is what happens under the superposed thermal gradient through the thickness. The IR Camera shows very pronounced heating on the outside surface....
I have calculated the variables that govern which way the circular plate will buckle.

Using S. Timoshenko's classic solution to the bimaterial thermal expansion problem (Journal of the Optical Society of America, JOSA, Vol. 11, Issue 3, pp. 233-255 (1925) ) the radius of curvature can be expressed as:

radiusOfCurvature = (t1+t2)*(3(1+m)+(1+m*n)(m^2 +1/(m*n)))/(6*deltaAlpha*deltaT*((1+m)^2))

where I have expressed the following variables as non-dimensional ratios (allowing use of any consistent system of units, and revealing the important parameters, instead of the expression in Wikipedia that is awkwardly expressed in dimensional units, and expressed for the curvature instead of its reciprocal):

m=t1/t2  Thickness ratio
n=E1(1-nu1^2)/(E2(1-nu2^2))  Plate stiffness-ratio

and the differences in thermal expansion and temperature:

deltaAlpha=alpha2 - alpha1 
deltaT=T - To

t1 and t2 are the thicknesses of the layers
E1 and E2 are the Young's modulii
nu1 and nu2 are the Poisson's ratios

From Paul March we know:

t1=0.00138 inches
t2=0.063 inches

the material "1" is copper

Wikipedia and Engineering Toolbox give E1=17*10^6 psi

Wikipedia, NBS (Hidnert &Krider's classic article) and Engineering Toolbox give alpha1= 17*10^(-6) 1/degK 

and Wikipedia gives nu1=0.33

and that the material "2" is FR4

The following references give:

Wikipedia
ECW = 3*10^6 psi   alphaCW= 14*10^(-6) 1/degK   nuLW=0.136
ELW = 3.5*10^6 psi alphaLW= 12*10^(-6) 1/degK  nuCW=0.118

P-M Services (UK)
                               alphaCW= 15*10^(-6) 1/degK
                               alphaLW= 11*10^(-6) 1/degK

Leiton (Germany)
                               alphaCW= 17*10^(-6) 1/degK
                               alphaLW= 12*10^(-6) 1/degK

It is trivial to show that the sign of the curvature (which way the plate is going to buckle) is governed only by the difference in thermal expansion coefficients between the two layers.

Bimaterial thermal bending will take place along the anisotropic in-plane direction with the highest thermal expansion and lowest modulus.  It is obvious that this is the CW direction.

It is immediately obvious, that using Leiton's (Germany) coefficient of thermal expansions, the copper/FR4 laminate circular plate will not experience any bimaterial bending whatsoever because according to Leighton alphaCW of FR4 is exactly the same as the universally accepted alpha1 of copper.

The thermal IR camera shows temperature readings ranging from below 79 deg F to a concentrated maximum at certain small spots of 94.3 deg F.

Using Wikipedia's properties, the plate will experience an extremely small amount of bimaterial bending towards the inside, for a temperature increase from 68 deg F to 79 deg F, with a huge radius of curvature exceding 7000 inches (practically flat, in relation to the thickness of only 0.064 inches).

Also, using Wikipedia's properties, the plate will experience an extremely small amount of bimaterial bending towards the inside, for a temperature increase from 68 deg F to 94.3 deg F, with a huge radius of curvature exceeding 3000 inches (practically flat, in relation to the thickness of only 0.064 inches).

CONCLUSION: due to the fact that FR4 has a coefficient of thermal expansion very similar to the one of copper, and that the thickness of the copper is extremely small compared to the thickness of FR4, the circular plate will experience either no bimaterial thermal bending whatsoever, or it will be extremely small (will stay practically flat) under the measured changes in temperature.  Therefore, bi-material bending due to a change in temperature is irrelevant to the buckling problem.  Buckling is instead governed by the plate's initial imperfect flatness
Therefore, since the bimaterial thermal bending is either non-existent (according to Leiton-Germany properties) or completely negigible (according to Wikipedia thermal expansion properties for FR4), it is obvious that buckling of the circular plate will be governed by initial imperfections.

In this case, initial imperfections are most likely the result of manufacturing (due to small unsymmetric differences in the FR4 laminate cross-section fiberglass reinforcement, or to the residual stresses from deposition of the copper).

Therefore, the circular plate will buckle either:

A) inwards in which case the force will be to the right
or
b) outwards, in which case the force will be to the left, in the same direction as the EM Drive measured force

There is no basis at this point to decide whether the force will be towards the left (same as the measurement) or towards the right (opposite to the measurement).  It is entirely dependent on the initial non-flatness of the plate..

Statements that the buckling force will be in the same or opposite direction as the measured EM Drive's force are dependent on the unjustified assumption that the buckling will be in a given direction (outwards or inwards, respectively).  That is unsupported by the facts.  The plate can buckle outwards or inwards, depending on the initial imperfect flatness.  Therefore the force can be towards the left (same direction as the EM Drive's measured force) or in the opposite direction.

ANOTHER ARTIFACT BITES THE DUST, and another one, and another one   :)

Therefore the buckling force explanation for the measured EM Drive's thrust is nullified on two accounts:

A) It could only explain the magnitude of the initial transient force but not the longer duration (40 sec) force

B) The direction of the buckling  force can be either in the same direction as the measured EM Drive's force or in the opposite direction, dependent on the initial imperfect flatness of the circular plate.  Since this imperfect flatness should be random for different plates used in the US, UK and China, one must conclude that it would be an unlikely coincidence that all measurements were conducted with plates that were originally imperfect in such a way as to produce buckling forces in the same direction as measured.



Furthermore, to avoid confusion, it should be pointed out again that the buckling force effect cannot be used for propellant-less propulsion. It was submitted only as an explanation for an experimental artifact. The center of mass of an EM Drive free in space should not move during or after buckling.  The center of mass of an EM Drive free in space should be completely unaffected by any buckling .
Title: Re: EM Drive Developments - related to space flight applications - Thread 2
Post by: Notsosureofit on 02/27/2015 02:27 pm
FYI

Found my Gunn Oscillator !!
Title: Re: EM Drive Developments - related to space flight applications - Thread 2
Post by: Rodal on 02/27/2015 02:29 pm
FYI

Found my Gunn Oscillator !!
Where was it hidden? Inquiring minds would like to know... :)
Title: Re: EM Drive Developments - related to space flight applications - Thread 2
Post by: Notsosureofit on 02/27/2015 02:53 pm
FYI

Found my Gunn Oscillator !!
Where was it hidden? Inquiring minds would like to know... :)

On the back of a variable attenuator. ( ~50mW)
https://www.dropbox.com/s/vdueo9mi6l3s4kb/2015-02-27%2011.18.24.jpg?dl=0

"CAVITY" is on the way !
http://www.ebay.com/itm/201065780928?_trksid=p2059210.m2749.l2649&ssPageName=STRK%3AMEBIDX%3AIT

Looks like short WR90 to WR75 that can be machined. (Any short WR51 to WR90 would also work if anyone has one around)
Title: Re: EM Drive Developments - related to space flight applications - Thread 2
Post by: aero on 02/27/2015 04:26 pm
Here is another bit of data from meep that I found interesting. This was from very early on so the gap used (2% base radius) was larger than ideal. The curve shows the Force/Power generated with a gap in the base plate, as the gap moves from the center of the base to the edge of the base. As you can see, whatever it is that causes the thrust effect as detected by meep is concentrated at the edge, in the corner of the cavity between the base and the cone body.

Speaking to the magnitude of the force on this graph. At the time I took this data I was using gaps only in the large end of the cavity. Later data shows that gaps in both ends of the cavity multiplies the detected force by a large factor.

As for what is causing the force? Don't know but this drawing indicates that the copper only needs to be thin or non-existent at the edge of the base. Elsewhere it can be thick with little effect.

Oh - and I should mention that on my cavity with the pipes extending axially from the ends, the drawing scale shows those pipes to be 1 inch long from the interior of the cavity. What I should mention is that length doesn't matter. Changing the length of the pipe (1" to 1/4") only changes the Force/Power calculated by meep in the third or fourth decimal place. A simple gap in the thin 35 micron copper works almost as well.

The force calculated is more strongly dependent on the direction of the pipes, however. From looking at the data I have posted you can see that the force is reduced from 70 to 0.08 as the angle changes from axial to outward horizontal. I did calculate one intermediate angle and found that the force seems to reduce as the cosine of the angle off vertical in the outward direction. I have yet to angle the pipes inward so no data for that instance.
Title: Re: EM Drive Developments - related to space flight applications - Thread 2
Post by: aero on 02/27/2015 06:44 pm
While looking into the tunnelling of electrons through a barrier, I observe that the mathematics for tunnelling of electrons is very similar to the mathematics presented for propagation  evanescent waves through a restriction in an RF wave-guide. See page 15, here http://wwwsis.lnf.infn.it/pub/INFN-FM-00-04.pdf (http://wwwsis.lnf.infn.it/pub/INFN-FM-00-04.pdf) for evanescent wave propagation and sheet 8, here http://tuttle.merc.iastate.edu/ee439/topics/tunneling.pdf (http://tuttle.merc.iastate.edu/ee439/topics/tunneling.pdf) for tunnelling of electrons through a barrier. (I would paste the math here, but both papers are PDF's, so I can't.)

I guess I shouldn't be surprised that the math is similar, if not the same, as tunnelling electrons are evanescent matter waves so the math should be similar. The big difference in the two math formulations is that the author of the first paper, evanescent wave propagating through a restriction in the wave guide, relies on extended special relativity to keep the exp (+term) in the wave equation while the author of the electron tunnelling paper relies on a physical observation to retain that term.

This brings up a question. "Since we are concerned with the mass of the thruster, why isn't it made from a light metal, aluminium for example?" The work function of aluminium is very close to that of copper, is all metals. Granted that aluminium oxidized when exposed to air so that its work function more than doubles after exposure, but a very thin layer of gold electro deposited on both sides would eliminate that problem with the result of a much less massive thruster.

Can anyone suggest a reason that an aluminium cavity would not work?
Title: Re: EM Drive Developments - related to space flight applications - Thread 2
Post by: Rodal on 02/27/2015 06:53 pm
While looking into the tunnelling of electrons through a barrier, I observe that the mathematics for tunnelling of electrons is very similar to the mathematics presented for propagation  evanescent waves through a restriction in an RF wave-guide. See page 15, here http://wwwsis.lnf.infn.it/pub/INFN-FM-00-04.pdf (http://wwwsis.lnf.infn.it/pub/INFN-FM-00-04.pdf) for evanescent wave propagation and sheet 8, here http://tuttle.merc.iastate.edu/ee439/topics/tunneling.pdf (http://tuttle.merc.iastate.edu/ee439/topics/tunneling.pdf) for tunnelling of electrons through a barrier. (I would paste the math here, but both papers are PDF's, so I can't.)

I guess I shouldn't be surprised that the math is similar, if not the same, as tunnelling electrons are evanescent matter waves so the math should be similar. The big difference in the two math formulations is that the author of the first paper, evanescent wave propagating through a restriction in the wave guide, relies on extended special relativity to keep the exp (+term) in the wave equation while the author of the electron tunnelling paper relies on a physical observation to retain that term.

This brings up a question. "Since we are concerned with the mass of the thruster, why isn't it made from a light metal, aluminium for example?" The work function of aluminium is very close to that of copper, is all metals. Granted that aluminium oxidized when exposed to air so that its work function more than doubles after exposure, but a very thin layer of gold electro deposited on both sides would eliminate that problem with the result of a much less massive thruster.

Can anyone suggest a reason that an aluminium cavity would not work?
Their magnetic properties, for example, are opposite: Copper is (weakly) diamagnetic while Aluminum is (weakly) paramagnetic.  

What relative magnetic permeability did you input into MEEP for copper? Did you input a value less than one?  Did you report to us that value?  I don't recall.

Stainless Steel 304L (the material of the vacuum chamber) is weakly paramagnetic (the opposite of copper). What relative magnetic permeability did you input into MEEP for the StSt 304L for the chamber? Did you input a value greater than one?  Did you report to us that value?  I don't recall.

These properties (copper diamagnetic, StSt paramagnetic are relevant to the force due to evanescent ineraction problem you are solving with MEEP)

Diamagnetic materials (like copper) create an induced magnetic field in a direction opposite to an externally applied magnetic field, and they are repelled by the applied magnetic field. In contrast, the opposite behavior is exhibited by paramagnetic materials (like aluminum): they are attracted to it.

On the other hand, it has been proposed that the best material (just) for the flat big end is a strongly ferromagnetic material: preferably Metglas, or otherwise iron.  This would result in a diamagnetic repellent copper material for the small flat end and the curved conical surfaces and a strongly attractive ferromagnetic material (Metglas thin film coated or just iron) for the big flat end. To my knowledge, this has not been tested.

Since there is really no proven theory of how would an EM Drive generate thrust in space, thereby apparently violating the law of conservation of momentum, and we are still debating whether the experimental measurements are an artifact, the best way to cut to the chase is to test an EM Drive made of Aluminum, and to also test an iron (or a material coated with an interior thin film of Metglas) for the big flat end.

(http://upload.wikimedia.org/wikipedia/en/6/65/Diamagnetic_levitation_diagram.gif)
Title: Re: EM Drive Developments - related to space flight applications - Thread 2
Post by: aero on 02/27/2015 08:12 pm
Quote
What relative magnetic permeability did you input into MEEP for copper? Did you input a value less than one?

Unfortunately I don't have a Drude model for copper at 2 GHz. We are cautioned in the meep literature that meep becomes unstable for magnetic permeability less than, or very much less than 1. I'm still using perfect metal and thick sheets at that. It's the computer resolution issue still, not enough memory and not enough CPU.

Thickness of 0.002 * height, or 0.002 * 9 inches = 0.018 inches is the best I can do in under a day of CPU time.

And FYI, I went ahead and calculated the probability of an electron tunnelling through the thin copper (I used Maxima) and found the probability to be 0.0. I guess no one is surprised by that result. That result could be off by a few orders of magnitude, but then what is 0E10? :)
Title: Re: EM Drive Developments - related to space flight applications - Thread 2
Post by: Rodal on 02/27/2015 08:23 pm
Quote
What relative magnetic permeability did you input into MEEP for copper? Did you input a value less than one?

Unfortunately I don't have a Drude model for copper at 2 GHz. We are cautioned in the meep literature that meep becomes unstable for magnetic permeability less than, or very much less than 1. I'm still using perfect metal and thick sheets at that. It's the computer resolution issue still, not enough memory and not enough CPU.

Thickness of 0.002 * height, or 0.002 * 9 inches = 0.018 inches is the best I can do in under a day of CPU time.

And FYI, I went ahead and calculated the probability of an electron tunnelling through the thin copper (I used Maxima) and found the probability to be 0.0. I guess no one is surprised by that result. That result could be off by a few orders of magnitude, but then what is 0E10? :)
Is Maxima a descendant of Macsyma?  Are they competing with Mathematica and Maple ?
(I remember using Macsyma at MIT in the late 1970's  :)  )
Title: Re: EM Drive Developments - related to space flight applications - Thread 2
Post by: Mulletron on 02/27/2015 08:24 pm
It's to bad that we can't find a way that one of the little known or unknown solutions to Maxwell's equations can cause  a momentum.
There is obviously momentum in electromagnetic waves. There are also some little known electromagnetic effects that create torque.

http://arxiv.org/abs/0807.1310

Quoting @Aero
Quote
Unless of course it is surface electrons excited by the high power resonant RF, tunnelling through the 35 micron copper ends.
Quoting @JPLeRouzic
Quote
Yes, and many other things may happen were not tested nor even proposed. I wonder how people can know that a lot of energy is pumped in this device and imagine nothing will get out. At the very least thermal effects should happen. Testing it in (near) vacuum doesn't eliminate the thermal hypothesis. Even Pioneer's acceleration that was due to thermal effects after all: http://spectrum.ieee.org/aerospace/astrophysics/finding-the-source-of-the-pioneer-anomaly
Another thing that strikes me is that people search for a unique cause explaining everything, which is a bit unlikely.

One last thought: If a simulator shows results, build this device and publish results in a mainstream conference. Interesting things may happen ;-)

There are absolutely thermal effects. For every watt pumped in, you get that much heat out as IR. An RF dummy load is an efficient converter of RF to heat. The systemic effects slide with the dummy load attached is the control for heat. Thermal artifacts were a major area of exploration in both threads. Paul March provided lots of data to show that thermal effects were extensively studied over at Eagleworks. There's a thermal analysis on this thread, one of many indications that heat was controlled for: http://forum.nasaspaceflight.com/index.php?topic=36313.msg1326997#msg1326997 Plus the math that has been done numerous times for a photon rocket doesn't add up. So the vacuum did eliminate the thermal hypothesis.
Title: Re: EM Drive Developments - related to space flight applications - Thread 2
Post by: Mulletron on 02/27/2015 09:00 pm
http://www.ebay.com/itm/AERCOM-Microwave-RF-Isolator-Circulator-2-4GHz-20dB-isolation-Low-I-L-TESTED-/281549538390?ssPageName=ADME:L:OU:US:1120
Picked up one of these puppies on Ebay to protect my amp. Another example of broken time reversal symmetry in action.

Got about an oz of very expensive liquid metal from here:
http://www.amazon.com/Gallium-Indium-Eutectic-GaInSn-68-5%25/dp/B00KN92MWW/ref=sr_1_3?ie=UTF8&qid=1425074693&sr=8-3&keywords=galinstan

So back to the copper from way back: http://forum.nasaspaceflight.com/index.php?topic=36313.msg1326742#msg1326742

I've determined that both the 10 and 16 mil are too thin to serve as frustum walls, thus they will become end caps.
https://docs.google.com/file/d/0B4PCfHCM1KYoZUs5dklaRFdoM3M/edit?pli=1

Been working with the supplier with a machine shop I posted about way back:
http://forum.nasaspaceflight.com/index.php?topic=36313.msg1326669#msg1326669
I'm going that route. The quote I got is: price: $120.00 layout + $51.63 for part + freight. So I have to pay the layout, then anyone else who wants one of these:
https://docs.google.com/file/d/0B4PCfHCM1KYoN2VURmltbVlfa3c/edit?pli=1
but built in 16oz copper, with a smooth butt seam inside, and 1/4" flange around edges, can get one for about 50 bucks plus shipping. If all this works out, it'll fulfill my goal of making a replication by DIYers easier. For me, paying the layout plus price about breaks even with buying the sheet myself and fumblefuddeling around trying to solder up a cone at home. So I'm happy. I'll get back with more later, when the items are at home.

Again thanks to @Paul March for providing the dimensions for his DUT over at Eagleworks:
http://forum.nasaspaceflight.com/index.php?topic=36313.msg1327467#msg1327467

Edit:
Links to pics on google drive keep disappearing, hopefully fixed it.
Title: Re: EM Drive Developments - related to space flight applications - Thread 2
Post by: aero on 02/27/2015 09:03 pm
Quote
Is Maxima a descendant of Macsyma?  Are they competing with Mathematica and Maple ?
(I remember using Macsyma at MIT in the late 1970's  :)  )

Yes, Maxima is the direct descendent of Macsyma. No, I wouldn't say "competing" since Maxima is free, hence limited expert support. But yes, Maxima runs Mathematica code. I used Maxima to calculate the masses of the elementary particles using the Heim theory formulas and a .mac file that had been written for Mathematica. Google it. Overcoming the support question was easy once I found a quite good tutorial document for it, here.
http://math.stanford.edu/~paquin/MaximaBook.pdf (http://math.stanford.edu/~paquin/MaximaBook.pdf)
Lengthy at 245 pages, but seemingly comprehensive. Actually, I'm using wxmaxima, maxima with a windows like GUI. I found it pretty easy to use once I got past the "shift enter" used to give output; xmaxima installs simultaneously with wxmaxima. Further, it is free and easily installs on either Ubuntu or Windows. I did both just to see which was easier to use. They are the same.

Quote
Stainless Steel 304L (the material of the vacuum chamber) is weakly paramagnetic (the opposite of copper). What relative magnetic permeability did you input into MEEP for the StSt 304L for the chamber? Did you input a value greater than one? Did you report to us that value?  I don't recall.

Same answer as for copper. And I've given up on trying to evaluate evanescent wave coupling to the vacuum chamber. The results I got indicated that, "Yes, there is some coupling, but it is small." Greater than the calculated forces for the as tested "Copper Kettle" but orders of magnitude below the forces calculated for the simulated "pipe" model that I posted yesterday. Therefore, since we can't have one without the other, cavity/cavity coupling via evanescent waves may happen but not significantly.

 The pipe model was simulated in a space environment, vacuum with nothing near. I even moved the PML boundary layer away from the cavity. Bigger lattice, longer runs, but this way I'm confident that the PML is not absorbing or numerically reflecting the RF energy to significantly affect the calculations.

Edit Add: And note that the "pipes" are not really pipes, rather the cone body has been extended on both ends with a cylindrical section and the shaped base plates pressed into the cylindrical section. This is like a cork in a bottle, or more like the old fashion coffee can or peanut can lids that pressed into the end of the can while keeping the length of the resonant cavity as designed.
Title: Re: EM Drive Developments - related to space flight applications - Thread 2
Post by: aero on 02/27/2015 09:30 pm
This is what I have for copper:

(define myCu (make dielectric (epsilon 1)
(polarizations
 (make polarizability
(omega 1e-20) (gamma 0.024197) (sigma 4.3873e+41))
(make polarizability
(omega 0.23471) (gamma 0.30488) (sigma 84.489))
(make polarizability
(omega 2.385) (gamma 0.85172) (sigma 1.395))
(make polarizability
(omega 4.2747) (gamma 2.5915) (sigma 3.0189))
(make polarizability
(omega 9.0173) (gamma 3.4722) (sigma 0.59868))
)))
;Additional Information
;Normalization length=1e-06 in meter
;Material_used_is_Cufrom Rakic et al.,Applied Optics (1998)
;Plasma Angular Frequency (and plasma wave vector,kp) in normalized units=6.6236


But this is for much higher frequency, THz and above, not even close to copper behavior at 2 GHz. And it is a Drude-Lorenz model. I don't think I need the extra terms, just the first two lines of data, but normalized at 3E-2, instead of 1E-6. I can normalize the data if I had Gamma and sigma for frequency = 2 GHz. And the first line of data, values for DC, shouldn't change, it's the second line...
Title: Re: EM Drive Developments - related to space flight applications - Thread 2
Post by: Rodal on 02/27/2015 09:41 pm
...
I've determined that both the 10 and 16 mil are too thin to serve as frustum walls, thus they will become end caps.
(https://lh3.googleusercontent.com/FHha_JWui320K0ZM7Zt_GA8MSaKKbn6LFjhuCU7Ul6QHwbdFXRgp60qOJ_SVm026udzIJw=w1566-h632)

Been working with the supplier with a machine shop I posted about way back:
http://forum.nasaspaceflight.com/index.php?topic=36313.msg1326669#msg1326669
...
Are you planning to conduct an experiment with a dielectric inside a cylindrical cavity?

Somebody should do so!

Such an experiment would disprove all the theories with formulas advanced so far.  All the formulas (Shawyer's, McCulloch and Notsosureofit...) give zero thrust if both diameters are the same.
Title: Re: EM Drive Developments - related to space flight applications - Thread 2
Post by: Mulletron on 02/27/2015 10:01 pm
...
I've determined that both the 10 and 16 mil are too thin to serve as frustum walls, thus they will become end caps.

Been working with the supplier with a machine shop I posted about way back:
http://forum.nasaspaceflight.com/index.php?topic=36313.msg1326669#msg1326669
...
Are you planning to conduct an experiment with a dielectric inside a cylindrical cavity?

Somebody should do so!

Such an experiment would disprove all the theories with formulas advanced so far.  All the formulas (Shawyer's, McCulloch and Notsosureofit...) give zero thrust if both diameters are the same.

Yes, a cylinder experiment absolutely has to be done.
Title: Re: EM Drive Developments - related to space flight applications - Thread 2
Post by: Rodal on 02/27/2015 10:11 pm
Notsosureofit:

Your equation gives zero thrust for a cylinder (constant diameter along its length). Thus, according to your formula a cylinder will give no thrust, only a geometrical object with decreasing diameter will (ditto for Shawyer's and McCulloch's)

But your formula does not explicitly include a dielectric.

QUESTION: would your line of thinking also give zero thrust for a cylinder with an inserted dielectric at one end of the cavity with constant, homogeneous, isotropic dielectric properties ?  (With no nonlinearities)
Title: Re: EM Drive Developments - related to space flight applications - Thread 2
Post by: Notsosureofit on 02/27/2015 10:18 pm
Notsosureofit:

Your equation gives zero thrust for a cylinder (constant diameter along its length). Thus, according to your formula a cylinder will give no thrust, only a geometrical object with decreasing diameter will (ditto for Shawyer's and McCulloch's)

But your formula does not explicitly include a dielectric.

QUESTION: would your line of thinking also give zero thrust for a cylinder with an inserted dielectric at one end of the cavity with constant, homogeneous, isotropic dielectric properties ?  (With no nonlinearities)

Presumably no, as long as you have frequency dispersion along the axis you should predict a force.

Edit: But it adds complication to the formula in that the second term doesn't cancel out.
Title: Re: EM Drive Developments - related to space flight applications - Thread 2
Post by: aero on 02/27/2015 10:43 pm
...
I've determined that both the 10 and 16 mil are too thin to serve as frustum walls, thus they will become end caps.

Been working with the supplier with a machine shop I posted about way back:
http://forum.nasaspaceflight.com/index.php?topic=36313.msg1326669#msg1326669
...
Are you planning to conduct an experiment with a dielectric inside a cylindrical cavity?

Somebody should do so!

Such an experiment would disprove all the theories with formulas advanced so far.  All the formulas (Shawyer's, McCulloch and Notsosureofit...) give zero thrust if both diameters are the same.

Yes, a cylinder experiment absolutely has to be done.

Hey - Good! I've ran some meep numbers with a cylindrical cavity with good results. Let me run some fresh numbers and I'll post what I find. If you have parameters in mind let me know, then I can run your numbers.
Title: Re: EM Drive Developments - related to space flight applications - Thread 2
Post by: Rodal on 02/27/2015 11:15 pm
Notsosureofit:

Your equation gives zero thrust for a cylinder (constant diameter along its length). Thus, according to your formula a cylinder will give no thrust, only a geometrical object with decreasing diameter will (ditto for Shawyer's and McCulloch's)

But your formula does not explicitly include a dielectric.

QUESTION: would your line of thinking also give zero thrust for a cylinder with an inserted dielectric at one end of the cavity with constant, homogeneous, isotropic dielectric properties ?  (With no nonlinearities)

Presumably no, as long as you have frequency dispersion along the axis you should predict a force.

Edit: But it adds complication to the formula in that the second term doesn't cancel out.

Since your existing formula:

NT = P*Q*(1/(4*pi*L*f^3))*(c/(2*pi))^2*X^2*((1/Rs^2)-(1/Rb^2))

gives NT=0 (zero thrust force) for Rs=Rb (a cylinder), just like Shawyer's and McCulloch's also go to zero for that case, then my understanding from your reply is that this formula is a simplification and that there are terms you neglected that do not go to zero for a cylinder.

Do you know the 2nd order term that will still be finite for Rs=Rb (a cylinder) ?   

Or is the issue deriving an equation for the dispersion including the dielectric?

Title: Re: EM Drive Developments - related to space flight applications - Thread 2
Post by: flux_capacitor on 02/28/2015 12:27 am
Are you planning to conduct an experiment with a dielectric inside a cylindrical cavity?

Somebody should do so!

Such an experiment would disprove all the theories with formulas advanced so far.  All the formulas (Shawyer's, McCulloch and Notsosureofit...) give zero thrust if both diameters are the same.

Before removing the internal dielectric, Shawyer considered a cylindrical EmDrive, in which a conical dielectric would be mandatory for thrust. See his 1990 UK Patent Application GB2229865 "Electrical Propulsion Unit for Spacecraft", attached below.
Title: Re: EM Drive Developments - related to space flight applications - Thread 2
Post by: Notsosureofit on 02/28/2015 12:39 am
Notsosureofit:

Your equation gives zero thrust for a cylinder (constant diameter along its length). Thus, according to your formula a cylinder will give no thrust, only a geometrical object with decreasing diameter will (ditto for Shawyer's and McCulloch's)

But your formula does not explicitly include a dielectric.

QUESTION: would your line of thinking also give zero thrust for a cylinder with an inserted dielectric at one end of the cavity with constant, homogeneous, isotropic dielectric properties ?  (With no nonlinearities)

Presumably no, as long as you have frequency dispersion along the axis you should predict a force.

Edit: But it adds complication to the formula in that the second term doesn't cancel out.

Since your existing formula:

NT = P*Q*(1/(4*pi*L*f^3))*(c/(2*pi))^2*X^2*((1/Rs^2)-(1/Rb^2))

gives NT=0 (zero thrust force) for Rs=Rb (a cylinder), just like Shawyer's and McCulloch's also go to zero for that case, then my understanding from your reply is that this formula is a simplification and that there are terms you neglected that do not go to zero for a cylinder.

Do you know the 2nd order term that will still be finite for Rs=Rb (a cylinder) ?   

Or is the issue deriving an equation for the dispersion including the dielectric?

You need to start back at the dispersion formula and follow the same steps. (sorry, I just got back)
Title: Re: EM Drive Developments - related to space flight applications - Thread 2
Post by: DIYFAN on 02/28/2015 01:02 am
Since there is really no proven theory of how would an EM Drive generate thrust in space, thereby apparently violating the law of conservation of momentum, and we are still debating whether the experimental measurements are an artifact, the best way to cut to the chase is to test an EM Drive made of Aluminum, and to also test an iron (or a material coated with an interior thin film of Metglas) for the big flat end.

As much as I applaud any and all experimentalists who are dedicating time, often with little compensation, part of the problem with the Eagleworks experimental results as they compare to those reported elsewhere (Shawyer and the Chinese) is the relatively small response signal.  Not only does this situation threaten the future of the experiment in the U.S. as other labs might refuse to attempt replications, this situation also leads to in-depth discussions about thermal effects, evanescent waves, and other unlikely explanations for the effect.  If the signal response can be amplified and made easily repeatable, more focus could then be placed on improving the effect, and eventually incorporating it into real world applications.  I fully agree, therefore, with Dr. Rodal that an experiment that tests an iron (or a material coated with an interior thin film of Metglas) for the big flat end should be made of highest priority.  Dr. Aquino proposed a theory of why this would amplify the effect.  It ought to be tested as soon as possible, and preferably in the U.S.
Title: Re: EM Drive Developments - related to space flight applications - Thread 2
Post by: ThinkerX on 02/28/2015 01:05 am
Seems like wherever we go with this project, Shawyer has already been there - and like as not, built a test model.   Given that, is there any way to bring his reasoning/math in line with conventional physics, or do they remain fundamentally irreconcilable?  I keep getting the impression he's on to something major, but consistently fails to communicate that 'something' in a coherent manner. 
Title: Re: EM Drive Developments - related to space flight applications - Thread 2
Post by: Rodal on 02/28/2015 01:08 am
Are you planning to conduct an experiment with a dielectric inside a cylindrical cavity?

Somebody should do so!

Such an experiment would disprove all the theories with formulas advanced so far.  All the formulas (Shawyer's, McCulloch and Notsosureofit...) give zero thrust if both diameters are the same.

Before removing the internal dielectric, Shawyer considered a cylindrical EmDrive, in which a conical dielectric would be mandatory for thrust. See his 1990 UK Patent Application GB2229865 "Electrical Propulsion Unit for Spacecraft", attached below.

Great find and thanks  :)  for bringing it to our attention, this is the first time I see this patent.

(http://forum.nasaspaceflight.com/index.php?action=dlattach;topic=36313.0;attach=808990;image)

Very thought provoking, and interesting in its own right.

I note that Shawyer's patent was filed in 1988 !, that's 27 years ago !

This man has been working at this for a long, long time



Now, on a separate matter:

I observe that although the outside metal cavity is indeed cylindrical, it is really not a cylindrical cavity for resonance purposes because the dielectric inside it is conical in shape, so it is a case again of a cavity with non-uniform inner dimensions along its axial length, for resonance purposes.  Shawyer and McCulloch would still calculate "thrust" for this type of cavity.

What I had in mind for people to test is whether

a cylindrical cavity

                               A) empty
                               or
                               B) with a cylindrical dielectric inside at one end (the dielectric OD having the same dimensions as                                     the cavity ID)

will exhibit thrust.

My understanding from Paul March's experiments is that A) will not produce any thrust (as NASA did not get thrust with the empty cavity).

My (rough) reading of the papers brought up by @Mulletron would say B) may produce thrust.  My reading of Shawyer's experience is that B) will not produce significant thrust (because Shawyer has gone through the trouble of using conical dielectrics inside cylindrical cavities, and tapered cones with progressively larger cone angles).   The above patent reinforces this point of view: Shawyer must have surely tested cylindrical cavities with cylindrical dielectrics fitting inside it.  Shawyer must be pursuing the conical shape (with increasing cone angles) for a good reason.
Title: Re: EM Drive Developments - related to space flight applications - Thread 2
Post by: Notsosureofit on 02/28/2015 01:37 am
Just catching up.  Yes that dispersion should give thrust (Shawyer !)  I get a formula (scribbled at the moment) that depends on the difference in index of the tapered dielectric (and the equivalent length which doesn't look like it will cancel things out, so far)

Probably can be simplified a bit but not tonight........(mode dependence is still in there)
Title: Re: EM Drive Developments - related to space flight applications - Thread 2
Post by: Rodal on 02/28/2015 01:53 am
Seems like wherever we go with this project, Shawyer has already been there - and like as not, built a test model.   Given that, is there any way to bring his reasoning/math in line with conventional physics, or do they remain fundamentally irreconcilable?  I keep getting the impression he's on to something major, but consistently fails to communicate that 'something' in a coherent manner.
Well, Edison was one of the greatest inventors.  He was completely self-taught (with visits to the Cooper Union). Edison was born only 16 years after Maxwell.  Edison was responsible for some of the greatest electromagnetic inventions ever, yet he was not a theoretical physicist like Maxwell or Hertz. Who was the inventor? Edison or Maxwell?.    Why should Shawyer  be the one expected to explain the physics of why it works (if it does work)?
Title: Re: EM Drive Developments - related to space flight applications - Thread 2
Post by: zen-in on 02/28/2015 02:11 am
http://www.ebay.com/itm/AERCOM-Microwave-RF-Isolator-Circulator-2-4GHz-20dB-isolation-Low-I-L-TESTED-/281549538390?ssPageName=ADME:L:OU:US:1120
Picked up one of these puppies on Ebay to protect my amp. Another example of broken time reversal symmetry in action.

Got about an oz of very expensive liquid metal from here:
http://www.amazon.com/Gallium-Indium-Eutectic-GaInSn-68-5%25/dp/B00KN92MWW/ref=sr_1_3?ie=UTF8&qid=1425074693&sr=8-3&keywords=galinstan

So back to the copper from way back: http://forum.nasaspaceflight.com/index.php?topic=36313.msg1326742#msg1326742

I've determined that both the 10 and 16 mil are too thin to serve as frustum walls, thus they will become end caps.
(https://lh3.googleusercontent.com/FHha_JWui320K0ZM7Zt_GA8MSaKKbn6LFjhuCU7Ul6QHwbdFXRgp60qOJ_SVm026udzIJw=w1566-h632)

Been working with the supplier with a machine shop I posted about way back:
http://forum.nasaspaceflight.com/index.php?topic=36313.msg1326669#msg1326669
I'm going that route. The quote I got is: price: $120.00 layout + $51.63 for part + freight. So I have to pay the layout, then anyone else who wants one of these:
(https://lh6.googleusercontent.com/X8z7ACoAfs4KhtbcY2o7xCdzx0sTbIevAqhrVKmsst-dPVhd_MauiiDh6V5ZrQkd9bIkhw=w1549-h632)
but built in 16oz copper, with a smooth butt seam inside, and 1/4" flange around edges, can get one for about 50 bucks plus shipping. If all this works out, it'll fulfill my goal of making a replication by DIYers easier. For me, paying the layout plus price about breaks even with buying the sheet myself and fumblefuddeling around trying to solder up a cone at home. So I'm happy. I'll get back with more later, when the items are at home.

Again thanks to @Paul March for providing the dimensions for his DUT over at Eagleworks:
http://forum.nasaspaceflight.com/index.php?topic=36313.msg1327467#msg1327467

You should see if you can find a 2 meter reject cavity.   The can is 3 feet high by over a foot dia and made from 16 or 18 gauge Copper.   There is a seam but you would want to cut an angle on each side of it so you can form it into a cone.   I think the Eagleworks people used a form for bending their cavity; or they had a sheet metal shop do the work.     Getting a sheet of Copper big enough to make the cone part from one piece can be expensive.
Title: Re: EM Drive Developments - related to space flight applications - Thread 2
Post by: ThinkerX on 02/28/2015 02:11 am
Quote
Well, Edison was one of the greatest inventors.  He was completely self-taught (with visits to the Cooper Union). Edison was born only 16 years after Maxwell.  Edison was responsible for some of the greatest electromagnetic inventions ever, yet he was not a theoretical physicist like Maxwell or Hertz. Who was the inventor? Edison or Maxwell?.    Why should Shawyer  be the one expected to explain the physics of why it works (if it does work)?

People here and elsewhere say a coherent theory is needed in order to advance with this project.

Seems to me, what Shawyer is doing is amassing a pile of data from differing versions of the EM Drive.  If those results could somehow be reconciled with mainstream physics, even if it means tweaking Relativity or some such, then a workable theory is that much closer.  Or to phrase it a bit differently, Shawyer probably has the data, but the data needs interpretation by somebody with a different skill set.   He definitely seems to be ahead of everybody else in EM drive development. 
Title: Re: EM Drive Developments - related to space flight applications - Thread 2
Post by: Rodal on 02/28/2015 11:58 am
Quote
Well, Edison was one of the greatest inventors.  He was completely self-taught (with visits to the Cooper Union). Edison was born only 16 years after Maxwell.  Edison was responsible for some of the greatest electromagnetic inventions ever, yet he was not a theoretical physicist like Maxwell or Hertz. Who was the inventor? Edison or Maxwell?.    Why should Shawyer  be the one expected to explain the physics of why it works (if it does work)?

People here and elsewhere say a coherent theory is needed in order to advance with this project.

...
The majority of contributors to this thread say that what's needed are much more experiments: mainly to confirm that the thrust is real or to nullify it, by verifying the experiments at another NASA center (Glenn or JPL) and a univeristy (John Hopkins, for example) and put this EM Drive phenomena to bed, or continue its engineering progress.  Actually a number of contributors are planning their own experiments and have shown photographs of their progress.  The number of experiments conducted for the EM Drive and the R&D budget are infinitesimal compared with the number of experiments regularly conducted in R&D projects both commercial, academic or government agency and in comparison with normal R&D budgets.  Not a single experiment has been conducted at a US or European University.  Here are a number of experiments that could be conducted:  cylindrical cavity without dielectric, cylindrical cavity with cylindrical dielectric, cylindrical cavity with conical dielectric, truncated cone with one end made of cast iron, truncated cone made with one end internally coated with thin film of Metglas, truncated cone made with Aluminum instead of copper, comparison of excitation produced by Gunn diode, magnetron, etc.; examination of thrust force vs. cone angle of truncated cone, examination of thrust force in a cone (untruncated) cavity. Should I continue?
Title: Re: EM Drive Developments - related to space flight applications - Thread 2
Post by: JPLeRouzic on 02/28/2015 12:51 pm
It's to bad that we can't find a way that one of the little known or unknown solutions to Maxwell's equations can cause  a momentum.
There is obviously momentum in electromagnetic waves. There are also some little known electromagnetic effects that create torque.

http://arxiv.org/abs/0807.1310

What do you mean? It's known since 19th century there is momentum in electromagnetic waves. It's not something new.

Quote
Quoting @Aero
Quote
Unless of course it is surface electrons excited by the high power resonant RF, tunnelling through the 35 micron copper ends.
Quoting @JPLeRouzic
Quote
Yes, and many other things may happen were not tested nor even proposed. I wonder how people can know that a lot of energy is pumped in this device and imagine nothing will get out. At the very least thermal effects should happen. Testing it in (near) vacuum doesn't eliminate the thermal hypothesis. Even Pioneer's acceleration that was due to thermal effects after all: http://spectrum.ieee.org/aerospace/astrophysics/finding-the-source-of-the-pioneer-anomaly
Another thing that strikes me is that people search for a unique cause explaining everything, which is a bit unlikely.

One last thought: If a simulator shows results, build this device and publish results in a mainstream conference. Interesting things may happen ;-)

There are absolutely thermal effects. For every watt pumped in, you get that much heat out as IR. An RF dummy load is an efficient converter of RF to heat. The systemic effects slide with the dummy load attached is the control for heat. Thermal artifacts were a major area of exploration in both threads. Paul March provided lots of data to show that thermal effects were extensively studied over at Eagleworks. There's a thermal analysis on this thread, one of many indications that heat was controlled for: http://forum.nasaspaceflight.com/index.php?topic=36313.msg1326997#msg1326997 Plus the math that has been done numerous times for a photon rocket doesn't add up. So the vacuum did eliminate the thermal hypothesis.
5*10-6 Torr is hardly a vacuum good enough to eliminate the thermal hypothesis. That's what you encounter at 1000 km high or so.
Pioneer lives in a vacuum *one million* times better that 5*10-6 Torr and there are still thermal effects.

Title: Re: EM Drive Developments - related to space flight applications - Thread 2
Post by: flux_capacitor on 02/28/2015 02:00 pm
Great find and thanks  :)  for bringing it to our attention, this is the first time I see this patent.

Very thought provoking, and interesting in its own right.

I note that Shawyer's patent was file in 1988 !, that's 27 years ago !

This man has been working at this for a long, long time

You're welcome. To my knowledge Roger Shawyer filed 4 patent applications related to the EmDrive, here they are attached at the bottom of this message. I think everyone who'd want to replicate this work should read them first.

- GB application 2229865, Shawyer, Roger John, "Electrical propulsion unit for spacecraft", filed 1st November 1988, published 3 October 1990
- GB application 2334761, Shawyer, Roger John, "Microwave thruster for spacecraft", filed 29 April 1988, published 1st September 1999.
- GB application 2399601, Shawyer, Roger John, "Thrust producing device using microwaves", file 13 March 2003, published 22 September 2004
- GB application 2493361, Shawyer, Roger John, "A high Q microwave radiation thruster", filed 1st August 2011, published 6 February 2013

You'll notice the 1999 patent (the second one, with a truncated cone cavity and an internal truncated cone dielectric) was actually the first filed, more than ten years before publication. He was followed by the filling of the patent with the cylindrical cavity only six months later, but nine years separated the two publications.

IMHO the problem with Shawyer and the scientific community which has not even given the EmDrive a try in the past 25 years (!) is not with the EmDrive itself as an experiment, but with Shawyer's attempts to explain the anomalous thrust (i.e. propellantless thrust and the conservation issues it implies) in terms of a radiation pressure imbalance, where a relativistic effect would be caused on end plates by the action of group velocity in different frames of reference (see his theory paper (http://www.emdrive.com/theorypaper9-4.pdf)). Some scientists strongly disagree with that explanation. But this does not imply the EmDrive does not work and could be explained otherwise. Your comment about Edison illustrates the situation well.

We can't judge Shawyer too badly for his theoretical paper, as he is an engineer, not a physicist. Most importantly, the contract he signed with UK government Department of Trade and Industry in July 2001, to develop the EmDrive, specifically asked him to write such a document for the preliminary analysis phase.

We are lucky Shawyer even published some data as patents and conference papers. Well, obviously he needed some money to start all this and he had to publicize his work at that time. He regularly presented progress in experiments at various international conferences, in Brighton in 2005 [1] (http://www.emdrive.com/Brighton2005paper.doc); at the 59th International Astronautical Congress (IAC) in Glasgow in 2008 [2] (http://www.emdrive.com/IAC-08-C4-4-7.pdf); the CEAS 2009 European Air and Space Conference held in Manchester [3] (http://www.emdrive.com/CEAS2009paper.doc); the 2nd Conference on Disruptive Technology in Space Activities (TECHNO DIS) at CNES' Toulouse Space Show, France in 2010 [4] (http://www.emdrive.com/Toulouse2010paper01.doc); IAC 2013 in Beijing, China [5] (http://www.emdrive.com/IAC13paper17254.v2.pdf)[6] (http://www.emdrive.com/IAC13poster17254.pdf); and IAC 2014, Toronto, Canada [7] (http://www.emdrive.com/iac2014presentation.pdf).

I wonder how the tests of superconducting cavities with very high Q are going on now. I emailed Shawyer to ask some information about Chinese cavity dimensions and other news, but he closed the tap and politely declined to comment. After all, he runs a private company (SPR Ltd) and wants to make money of this, so I think he won't disclose sensitive information any further. We'll see.

Guido P. Fetta also silently runs a private company (Cannae LLC) and has an approach different than a tapered frustum and dielectrics.

The Chinese team publishes information in peer-reviewed academic journals, but we are not protected against any disinformation or cover-up by the military if they make a real breakthrough. Pr. Juan Yang also declines to comment any other information than what she already published.

Eagleworks publish their results as regular reports and conference papers, and it's a very good thing. Moreover Paul March kindly shares additional information and wise thinking here with us. Too bad this research is not better funded by NASA. Having such a close deadline for results (end of March 2015 if I remember) and being slowed down by dying power amplifiers, not having the funds to just repair or change them, is really a shame for Science and maybe also for the future of mankind.

So it is a very good thing now some people express willpower to conduct independent verification and validation of such a potential disruptive technology in their own lab.
Title: Re: EM Drive Developments - related to space flight applications - Thread 2
Post by: Left Field on 02/28/2015 02:06 pm
Hi. Newcomer here but I have been following since thread 1. Hats off to all of you and your dedication to finding answers - it is massively entertaining.

I just saw a new post by dustinthewind in Advanced Concepts and thought it may be related:

http://forum.nasaspaceflight.com/index.php?topic=36911.0

It contains links to a patent with the following abstract:

Quote
An electronic propulsion engine that creates a propulsive force or thrust using forces or electrostatic electromagnetic forces, with an effect that is similar to the thrust of a jet or rocket engine. Forces are generated using electromagnets or capacitor plates that are separated by dielectric spacer cores and are operated with two modulated currents. The two modulated currents are synchronized, but with a relative phase such that the forces on the two magnets or capacitor plates are not balanced. Included are techniques to reduce circuit impedance and control field dispersion, such as tuned LCR circuits, dielectric core materials between the magnets or capacitor plates, and RF superconductors result in high propulsion efficiencies. The system operates at RF frequencies and can also be used as a communication device.
Title: Re: EM Drive Developments - related to space flight applications - Thread 2
Post by: Rodal on 02/28/2015 02:14 pm
Hi. Newcomer here but I have been following since thread 1....
Welcome to this thread  :)
Title: Re: EM Drive Developments - related to space flight applications - Thread 2
Post by: Rodal on 02/28/2015 02:18 pm
...To my knowledge Roger Shawyer filed 4 patent applications related to the EmDrive, here they are attached at the bottom of this message....

What an excellent post, full of information, with attached documents and very well-written  :)  Thanks !
Title: Re: EM Drive Developments - related to space flight applications - Thread 2
Post by: Star-Drive on 02/28/2015 03:16 pm
Dr. Rodal & Crew:

The Eagleworks team has already build a 6061 aluminum frustum cavity with 1/4" thick walls and O-ring end caps meant to hold a 1 Bar pressure differential with internal nickel/copper/silver/gold plating system on all interior surfaces with plating thickness of 10-to-15 microns for the first three layers and 0.5 microns for exposed to the RF gold layer.  Sadly the gold layer was just as thick as the rest of the plated layers and textured as well, so as far as the applied ~2.0 GHz RF was concerned it was only interacting with the rough gold layer.  This had the effect of cutting the resonant Q-factor for this aluminum frustum by almost a factor of three over our copper frustum for the resonances of interest. 

At the same time we also tried using a smaller volume, higher-K (e-r=~40) ceramic dielectric resonator discs in the Al cavity mounted at its small OD end, while driving it at its TE011 mode if memory serves.  Bottom line was that this configuration was a total bust in regards to thrust production in our torque pendulum system running at this resonant mode.  This aluminum frustum design also turned out to be ~4X times the mass of the thin walled copper cavity even while using lower density aluminum for its construction.  This exercise was a tribute to the fact that one should never ASSUME that you know what you are doing until proven otherwise!  And oh yes, and only try one variation in the design at a time or one will get lost, fast!

Best, Paul M.
Title: Re: EM Drive Developments - related to space flight applications - Thread 2
Post by: Rodal on 02/28/2015 03:21 pm
So the vacuum did eliminate the thermal hypothesis.
...
5*10-6 Torr is hardly a vacuum good enough to eliminate the thermal hypothesis. That's what you encounter at 1000 km high or so.
Pioneer lives in a vacuum *one million* times better that 5*10-6 Torr and there are still thermal effects.


@Mulletron must have meant "So the vacuum did eliminate the [natural convection] thermal hypothesis" .  He wrote "thermal hypothesis" without the words "natural convection" because he thought that it was obviously understood he was referring to convection in the context he was discussing:  what is the difference between ambient pressure and  5*10-6 Torr.  Obviously this pressure makes a huge difference for natural convection currents, as a trivial calculation can show that 5*10-6 Torr vacuum does eliminate the natural convection hypothesis that was advanced in most media (including Prof. Baez's blog) as the most likely explanation for the measurements.  @Frobnicat, was proposing the thermal convection hypothesis as a possible artifact, and he worked some equations in Thread 1.  I do not recall @Frobnicat objecting to a 5*10-6 Torr vacuum nullifying the hypothesis of natural thermal convection from the EM Drive being able to produce the measured forces. 


Concerning <<Pioneer lives in a vacuum *one million* times better that 5*10-6 Torr and there are still thermal effects>>, yes the Pioneer anomaly was successfully explained by a team of JPL engineers/scientists on the basis of thermal radiation effects (not natural convection thermal effects). 

Thermal radiation, an entirely different (from convection) means of thermal transport, will take place even in a perfect vacuum, as thermal radiation is electromagnetic radiation and therefore does not need a gas or a fluid as a transport medium.  On the other hand natural convection is completely dependent on fluid or gas medium to carry it, and without a fluid or a gas, there cannot be any thermal convection.  Thermal convection involves fluid advection.

Radiative heat transfer is the only form of heat transfer that can take place in a vacuum.  Radiative heat transfer depends on the difference between the fourth power of the temperatures between the radiating surface and the irradiated surface, the surface areas and the emissivities of the radiating and irradiated surfaces and the Stefan–Boltzmann constant .  It may be important for the Pioneer anomaly: an extremely small effect comprising an acceration towards the Sun, of only 8.74×10^(−10) m/s^2, due to asymmetrical radiation of heat from the Radioisotope thermoelectric generators or from the spacecraft electronics reflecting from the back of the spacecraft's dish-like main antenna, causing a recoil like sunlight striking a solar sail.

But in the case of the EM Drive, how much does the temperature difference between radiated and irradiated surfaces have to be to to justify the measured thrust forces ?
Title: Re: EM Drive Developments - related to space flight applications - Thread 2
Post by: MathieuA on 02/28/2015 06:30 pm
http://www.ebay.com/itm/AERCOM-Microwave-RF-Isolator-Circulator-2-4GHz-20dB-isolation-Low-I-L-TESTED-/281549538390?ssPageName=ADME:L:OU:US:1120
Picked up one of these puppies on Ebay to protect my amp. Another example of broken time reversal symmetry in action.

Got about an oz of very expensive liquid metal from here:
http://www.amazon.com/Gallium-Indium-Eutectic-GaInSn-68-5%25/dp/B00KN92MWW/ref=sr_1_3?ie=UTF8&qid=1425074693&sr=8-3&keywords=galinstan

So back to the copper from way back: http://forum.nasaspaceflight.com/index.php?topic=36313.msg1326742#msg1326742
...
Been working with the supplier with a machine shop I posted about way back:
http://forum.nasaspaceflight.com/index.php?topic=36313.msg1326669#msg1326669
I'm going that route. The quote I got is: price: $120.00 layout + $51.63 for part + freight. So I have to pay the layout, then anyone else who wants one of these:
(https://lh4.googleusercontent.com/xpJIaQNfcmecDoHrIDnoYqVs1ccOghkxSpkgXhJ8MrnoyaK61MQ9rSfiBnuIhyayCIIEaQ=w1549-h745)
but built in 16oz copper, with a smooth butt seam inside, and 1/4" flange around edges, can get one for about 50 bucks plus shipping. If all this works out, it'll fulfill my goal of making a replication by DIYers easier. For me, paying the layout plus price about breaks even with buying the sheet myself and fumblefuddeling around trying to solder up a cone at home. So I'm happy. I'll get back with more later, when the items are at home.

Hello Mulletron,

Hope you'll be able to conduct another test, and I guess there are many other people like me who encourage you to continue.
How do you intend to measure the (very tiny) thrust ? Reading back NASA's paper, it seems they had to run the experiment in a lab with complex (and expensive) tools to remove all parasite effects that would interfere with the thrust from the apparatus. Maybe you have access to such equipment ?

--
Mathieu
Title: Re: EM Drive Developments - related to space flight applications - Thread 2
Post by: flux_capacitor on 02/28/2015 07:30 pm
How do you intend to measure the (very tiny) thrust ? Reading back NASA's paper, it seems they had to run the experiment in a lab with complex (and expensive) tools to remove all parasite effects that would interfere with the thrust from the apparatus. Maybe you have access to such equipment ?

- Roger Shawyer achieved a thrust of 174 mN @ 450 W of input power in 2010 (he then operated its engine up to 600 W but thrust levels are not known fort that power).
- Chinese NWPU achieved 720 mN @ 2500 W in 2010.
- Eagleworks achieved 116 µN @ 17 W in 2014.

So if you're gonna try an EmDrive at home, you'd better feed it with at least severals hundred of watts of electric power to be able to measure tens to hundreds of millinewtons. Staying below 100 W seems to produce thrusts in the micronewton range, way too low and something only a dedicated lab like Eagleworks can detect with expensive apparatus. At that level you detect any low-frequency vibrations like small seismic events. Indeed you noticed in the paper that Eagleworks used an isolated test rig from the Apollo program era with an extremely sensitive torsion pendulum, and on windy days they were able to detect the waves from the Gulf of Mexico, about 25 miles southeast of JSC… This low-power experiment was intended for a specific purpose: test the EmDrive in a hard vacuum. Only a small device with a compact and light onboard power amplifier could be tested, hence the very low power used compared to previous experiments by Shawyer and the Chinese.

The Chinese pursue a different path: they try to brutally produce the maximum thrust available with cheap but high power magnetrons. This way they cannot eliminate convection current or test their drive in a hard vacuum, but they can show the thrust achieved is too high to be accounted only for trivial reasons. Besides they showed the engine could compete with modern ion thrusters and perhaps one day overtake them.

You can also let the test article rotate. But it's not easy. Shawyer conducted such dynamic tests on a rotating rig. The whole device weighted 100 kg, comprising the thruster and a cooling system mounted on a beam, supported on a low-friction air bearing. The device reportedly consumed 300 W of power and produced a force of 96.1 mN, a maximum speed of 2 cm/s over 185 cm during testing in October 2006. See this 2007 article by Eureka magazine (http://www.eurekamagazine.co.uk/design-engineering-features/technology/no-propellant-drive-prepares-for-space-and-beyond/9657/) who covered the story, and this article from Wired (http://www.wired.com/2008/10/video-impossibl/) the year after, which displays a video where you can see the mammoth.

Interesting thing to note: Shawyer claims the engine starts to accelerate only when the magnetron frequency locks to the resonant frequency of the thruster, following an initial warm up period where it does not move; thus according to him eliminating possible spurious forces.

We'd need some data of tests made with high-Q superconducting cavities. But only Cannae LLC briefly talked about such test results before shutting down their web site. In January 2011, they sent 10.5 watt power pulses of 1047.335 MHz RF power into a resonant superconducting cavity suspended in a liquid helium-filled dewar, detecting a reduction in compressive force on the load cells consistent with a thrust of 8-10 mN. Did someone read their paper published at the 50th AIAA/ASME/SAE/ASEE Joint Propulsion Conference in 2014?

Fetta, Guido P. (30 August 2014). Numerical and Experimental Results for a Novel Propulsion Technology Requiring no On-Board Propellant. 50th AIAA/ASME/SAE/ASEE Joint Propulsion Conference. American Institute of Aeronautics and Astronautics. doi:doi: 10.2514/6.2014-3853 (http://arc.aiaa.org/doi/abs/10.2514/6.2014-3853)
Title: Re: EM Drive Developments - related to space flight applications - Thread 2
Post by: Rodal on 02/28/2015 07:45 pm
How do you intend to measure the (very tiny) thrust ? Reading back NASA's paper, it seems they had to run the experiment in a lab with complex (and expensive) tools to remove all parasite effects that would interfere with the thrust from the apparatus. Maybe you have access to such equipment ?

- Roger Shawyer achieved 174 mN of thrust at an input power of 450 W in 2010 (he then operated its engine up to 600 W but thrust levels are not known fort that power).
- Chinese NWPU achieved 720 mN @ 2500 W in 2010.
- Eagleworks achieved 116 µN @ 17 W in 2014.

So if you're gonna try an EmDrive at home, you'd better feed it with at least severals hundred of watts of electric power to be able to measure tens to hundreds of millinewtons. Staying below 100 W seems to produce thrusts in the micronewton range, way too low and something only a dedicated lab like Eagleworks can detect with expensive apparatus. At that level you detect any low-frequency vibrations like small seismic events. Indeed you noticed in the paper that Eagleworks used an isolated test rig from the Apollo program era with an extremely sensitive torsion pendulum, and on windy days they were able to detect the waves from the Gulf of Mexico, about 25 miles southeast of JSC… This low-power experiment was intended for a specific purpose: test the EmDrive in a hard vacuum. Only a small device with a compact and light onboard power amplifier could be tested, hence the very low power used compared to previous experiments by Shawyer and the Chinese.

The Chinese pursue a different path: they try to brutally produce the maximum thrust available with cheap but high power magnetrons. This way they cannot eliminate convection current or test their drive in a hard vacuum, but they can show the thrust achieved is too high to be accounted only for trivial reasons. Besides they showed the engine could compete with modern ion thrusters and perhaps one day overtake them.

You can also let the test article rotate. But it's not easy. Shawyer conducted such dynamic tests on a rotating rig. The whole device weighted 100 kg, comprising the thruster and a cooling system mounted on a beam, supported on a low-friction air bearing. The device reportedly consumed 300 W of power and produced a force of 96.1 mN, a maximum speed of 2 cm/s over 185 cm during testing in October 2006. See this 2007 article by Eureka magazine (http://www.eurekamagazine.co.uk/design-engineering-features/technology/no-propellant-drive-prepares-for-space-and-beyond/9657/) who covered the story, and this article from Wired (http://www.wired.com/2008/10/video-impossibl/) the year after, which displays a video where you can see the mammoth.

Interesting thing to note: Shawyer claims the engine starts to accelerate only when the magnetron frequency locks to the resonant frequency of the thruster, following an initial warm up period where it does not move; thus according to him eliminating possible spurious forces.

We'd need some data of tests made with high-Q superconducting cavities. But only Cannae LCC briefly talked about such test results before shutting down their web site.
The experimenters may contemplate using the simple Cavendish balance as done by Cavendish in his experiment performed more than 200 years ago (in 1797–98) http://en.wikipedia.org/wiki/Cavendish_experiment.

John Hopkins Applied Physics Laboratory was mentioned in Brady's "Anomalous ..." report as the only University of the three sites being contemplated to replicate the NASA Eagleworks experiment, in the Cavendish apparatus at John Hopkins.

Brady,  March, White, et.al. wrote:  <<The Johns Hopkins University Applied Physics Laboratory has also expressed an interest in performing a Cavendish Balance style test with the IV&V shipset>>

The force involved in twisting the torsion balance by Cavendish was very small:  0.174 microNewtons (certainly small enough to detect the small forces produced by the low power experiments at NASA Eagleworks).

(http://upload.wikimedia.org/wikipedia/commons/thumb/d/dd/Cavendish_Experiment.png/250px-Cavendish_Experiment.png)

Vibrations in the environment are an issue.  The vibrations produced by nature were also in the environment during Cavendish's time but nowadays we also have vibrations due to heavy trucks on nearby roads, etc.

Another approach is to use the simple hanging pendulum with oil bath damping used by Brito, Marini and Galian (see attached image) in their battery powered MLT-thruster
Title: Re: EM Drive Developments - related to space flight applications - Thread 2
Post by: Rodal on 02/28/2015 07:57 pm
....

Interesting thing to note: Shawyer claims the engine starts to accelerate only when the magnetron frequency locks to the resonant frequency of the thruster, following an initial warm up period where it does not move; thus according to him eliminating possible spurious forces.

....

That's very interesting stuff.  Thank you for bringing it up.

Do you have a reference as to where Shawyer  made that interesting claim?
Title: Re: EM Drive Developments - related to space flight applications - Thread 2
Post by: Notsosureofit on 02/28/2015 09:12 pm
@ RODAL

Still have question about the Shawyer "Demo" cavity w/ 174mN.  What are the current estimates of the cone dimensions, frequency (3.85GHz?), and Q (6000 est?).  When I put in TM02 and 450W, I get 174.8microN, rather than the 174milliN reported.  I would like to recheck those numbers.

Even w/ Q=45000, I need to get X up around (65 Very high mode) to get those numbers.  Is that possible w/ 3.85GHz ??

Thanks
Title: Re: EM Drive Developments - related to space flight applications - Thread 2
Post by: JPLeRouzic on 02/28/2015 09:22 pm
So the vacuum did eliminate the thermal hypothesis.
...
5*10-6 Torr is hardly a vacuum good enough to eliminate the thermal hypothesis. That's what you encounter at 1000 km high or so.
Pioneer lives in a vacuum *one million* times better that 5*10-6 Torr and there are still thermal effects.


@Mulletron must have meant "So the vacuum did eliminate the [natural convection] thermal hypothesis" .  He wrote "thermal hypothesis" without the words "natural convection" because he thought that it was obviously understood he was referring to convection in the context he was discussing:  what is the difference between ambient pressure and  5*10-6 Torr.  Obviously this pressure makes a huge difference for natural convection currents, as a trivial calculation can show that 5*10-6 Torr vacuum does eliminate the natural convection hypothesis that was advanced in most media (including Prof. Baez's blog) as the most likely explanation for the measurements.  @Frobnicat, was proposing the thermal convection hypothesis as a possible artifact, and he worked some equations in Thread 1.  I do not recall @Frobnicat objecting to a 5*10-6 Torr vacuum nullifying the hypothesis of natural thermal convection from the EM Drive being able to produce the measured forces. 


Concerning <<Pioneer lives in a vacuum *one million* times better that 5*10-6 Torr and there are still thermal effects>>, yes the Pioneer anomaly was successfully explained by a team of JPL engineers/scientists on the basis of thermal radiation effects (not natural convection thermal effects). 

Thermal radiation, an entirely different (from convection) means of thermal transport, will take place even in a perfect vacuum, as thermal radiation is electromagnetic radiation and therefore does not need a gas or a fluid as a transport medium.  On the other hand natural convection is completely dependent on fluid or gas medium to carry it, and without a fluid or a gas, there cannot be any thermal convection.  Thermal convection involves fluid advection.

Radiative heat transfer is the only form of heat transfer that can take place in a vacuum.  Radiative heat transfer depends on the difference between the fourth power of the temperatures between the radiating surface and the irradiated surface, the surface areas and the emissivities of the radiating and irradiated surfaces and the Stefan–Boltzmann constant .  It may be important for the Pioneer anomaly: an extremely small effect comprising an acceration towards the Sun, of only 8.74×10^(−10) m/s^2, due to asymmetrical radiation of heat from the Radioisotope thermoelectric generators or from the spacecraft electronics reflecting from the back of the spacecraft's dish-like main antenna, causing a recoil like sunlight striking a solar sail.

But in the case of the EM Drive, how much does the temperature difference between radiated and irradiated surfaces have to be to to justify the measured thrust forces ?
You are indeed right about the two thermal effects you mention, and I don't want to be pushed into territories that I don't master and that were not the subject of my initial post.
However I believe (and in writing this, I do something stupid  :)   ) that thermal convection means implicitly that the mean molecular free path is negligible so thermal convection anyway can't exist at all at this pressure.

However that doesn't means we should stop here. At 5*10-6 Torr there are still 5*10^11 molecules per cm3 and mean free path is in the 5m range. I did rule of thumb calcs (that indeed I will never publish 8)), and the thrust is still in the 5-10 µNewton range. My calcs are obviously wrong but indicate that one cannot conclude easily there is absolutely no thermal effect possible at 5*10-6 Torr.   
Title: Re: EM Drive Developments - related to space flight applications - Thread 2
Post by: Rodal on 02/28/2015 09:33 pm
So the vacuum did eliminate the thermal hypothesis.
...
5*10-6 Torr is hardly a vacuum good enough to eliminate the thermal hypothesis. That's what you encounter at 1000 km high or so.
Pioneer lives in a vacuum *one million* times better that 5*10-6 Torr and there are still thermal effects.


@Mulletron must have meant "So the vacuum did eliminate the [natural convection] thermal hypothesis" .  He wrote "thermal hypothesis" without the words "natural convection" because he thought that it was obviously understood he was referring to convection in the context he was discussing:  what is the difference between ambient pressure and  5*10-6 Torr.  Obviously this pressure makes a huge difference for natural convection currents, as a trivial calculation can show that 5*10-6 Torr vacuum does eliminate the natural convection hypothesis that was advanced in most media (including Prof. Baez's blog) as the most likely explanation for the measurements.  @Frobnicat, was proposing the thermal convection hypothesis as a possible artifact, and he worked some equations in Thread 1.  I do not recall @Frobnicat objecting to a 5*10-6 Torr vacuum nullifying the hypothesis of natural thermal convection from the EM Drive being able to produce the measured forces. 


Concerning <<Pioneer lives in a vacuum *one million* times better that 5*10-6 Torr and there are still thermal effects>>, yes the Pioneer anomaly was successfully explained by a team of JPL engineers/scientists on the basis of thermal radiation effects (not natural convection thermal effects). 

Thermal radiation, an entirely different (from convection) means of thermal transport, will take place even in a perfect vacuum, as thermal radiation is electromagnetic radiation and therefore does not need a gas or a fluid as a transport medium.  On the other hand natural convection is completely dependent on fluid or gas medium to carry it, and without a fluid or a gas, there cannot be any thermal convection.  Thermal convection involves fluid advection.

Radiative heat transfer is the only form of heat transfer that can take place in a vacuum.  Radiative heat transfer depends on the difference between the fourth power of the temperatures between the radiating surface and the irradiated surface, the surface areas and the emissivities of the radiating and irradiated surfaces and the Stefan–Boltzmann constant .  It may be important for the Pioneer anomaly: an extremely small effect comprising an acceration towards the Sun, of only 8.74×10^(−10) m/s^2, due to asymmetrical radiation of heat from the Radioisotope thermoelectric generators or from the spacecraft electronics reflecting from the back of the spacecraft's dish-like main antenna, causing a recoil like sunlight striking a solar sail.

But in the case of the EM Drive, how much does the temperature difference between radiated and irradiated surfaces have to be to to justify the measured thrust forces ?
You are indeed right about the two thermal effects you mention, and I don't want to be pushed into territories that I don't master and that were not the subject of my initial post.
However I believe (and in writing this, I do something stupid  :)   ) that thermal convection means implicitly that the mean molecular free path is negligible so thermal convection anyway can't exist at all at this pressure.

However that doesn't means we should stop here. At 5*10-6 Torr there are still 5*10^11 molecules per cm3 and mean free path is in the 5m range. I did rule of thumb calcs (that indeed I will never publish 8)), and the thrust is still in the 5-10 µNewton range. My calcs are obviously wrong but indicate that one cannot conclude easily there is absolutely no thermal effect possible at 5*10-6 Torr.   
For a third opinion on this matter, it will be interesting to hear comments from @frobnicat (who is also from France  :)  ?), using Frobnicat's thermal convection calculations (discussed in Thread 1) using an atmosphere at  5*10-6 Torr .

If we convert the Torr units used by Paul March to "standard atmosphere" units (we live under 1 atmospheric pressure), the vacuum condition tested by Paul March was:

6.6*10^(-9) atmosphere = 0.0000000066 atmosphere
Title: Re: EM Drive Developments - related to space flight applications - Thread 2
Post by: frobnicat on 02/28/2015 09:34 pm
So the vacuum did eliminate the thermal hypothesis.
...
5*10-6 Torr is hardly a vacuum good enough to eliminate the thermal hypothesis. That's what you encounter at 1000 km high or so.
Pioneer lives in a vacuum *one million* times better that 5*10-6 Torr and there are still thermal effects.


@Mulletron must have meant "So the vacuum did eliminate the [natural convection] thermal hypothesis" .  He wrote "thermal hypothesis" without the words "natural convection" because he thought that it was obviously understood he was referring to convection in the context he was discussing:  what is the difference between ambient pressure and  5*10-6 Torr.  Obviously this pressure makes a huge difference for natural convection currents, as a trivial calculation can show that 5*10-6 Torr vacuum does eliminate the natural convection hypothesis that was advanced in most media (including Prof. Baez's blog) as the most likely explanation for the measurements.  @Frobnicat, was proposing the thermal convection hypothesis as a possible artifact, and he worked some equations in Thread 1.  I do not recall @Frobnicat objecting to a 5*10-6 Torr vacuum nullifying the hypothesis of natural thermal convection from the EM Drive being able to produce the measured forces. 
...

My specific proposition at the time was a pressure difference build-up between inside cavity and outside, enough to "jet" supposedly warmed air through small apertures of the cavity. I recall having said about 5 month ago I would give a definite answer within 2 month (so much for my credibility with deadlines). This specific "warm jet" hypothesis would have been easily nullified even by a very more modest rough vacuum level of a few Torr. So this specific hypothesis is now ruled out, and I'm glad the experimental progress at Eaglworks outpaced my episodic armchair scientist investment on that matter  :D

For anything pressure related (even complex anisotropic molecular flow like in a Crookes radiometer effect) a characteristic value would be an extreme case of a completely asymmetric pressure difference around 5e-6 Torr = 6.65e-4 Pa on a surface of 11'' diameter = 6.13e-2 m² (roughly, cross section of the frustum) that yields about 41µN. Unfortunately still in the ballpark of the signal... so maybe hardcore sceptics could still require 2 orders of magnitude better vacuum, just to be sure... (edit : I see JPLeRouzic reach similar conclusion)

Anyway, it's clear that it becomes difficult to find remaining thermo-aerodynamic effects that would need better than 5e-6 Torr to be ruled out : this is good enough to put possible asymmetric gas flow effects much below the observed signal.

Quote

Concerning <<Pioneer lives in a vacuum *one million* times better that 5*10-6 Torr and there are still thermal effects>>, yes the Pioneer anomaly was successfully explained by a team of JPL engineers/scientists on the basis of thermal radiation effects (not natural convection thermal effects). 

Thermal radiation, an entirely different (from convection) means of thermal transport, will take place even in a perfect vacuum, as thermal radiation is electromagnetic radiation and therefore does not need a gas or a fluid as a transport medium.  On the other hand natural convection is completely dependent on fluid or gas medium to carry it, and without a fluid or a gas, there cannot be any thermal convection.  Thermal convection involves fluid advection.

Radiative heat transfer is the only form of heat transfer that can take place in a vacuum.  Radiative heat transfer depends on the difference between the fourth power of the temperatures between the radiating surface and the irradiated surface, the surface areas and the emissivities of the radiating and irradiated surfaces and the Stefan–Boltzmann constant .  It may be important for the Pioneer anomaly: an extremely small effect comprising an acceration towards the Sun, of only 8.74×10^(−10) m/s^2, due to asymmetrical radiation of heat from the Radioisotope thermoelectric generators or from the spacecraft electronics reflecting from the back of the spacecraft's dish-like main antenna, causing a recoil like sunlight striking a solar sail.

But in the case of the EM Drive, how much does the temperature difference between radiated and irradiated surfaces have to be to to justify the measured thrust forces ?

Very clear.
For the last question, I would be tempted to say : temperatures needing 300 times the actual operating power of the system, since the force/power ratios are ~300 times better than a photon rocket.

Unless the space between the frustum and the vacuum chamber's walls could amplify the effect by bouncing hotter IR photons around a few times and make high temperature, high radiation pressure "traps". IR photons are emitted around all the time, but at thermal equilibrium those radiative pressures have equal contribution on all side of an object in the bath. Anyone knows the typical reflectivity of more or less polished metals in thermal IR ? That would put an upper bound on the "boost" factor compared to the photon rocket equivalent thrust, likely much below 300.
Title: Re: EM Drive Developments - related to space flight applications - Thread 2
Post by: Rodal on 02/28/2015 09:40 pm
So the vacuum did eliminate the thermal hypothesis.
...
5*10-6 Torr is hardly a vacuum good enough to eliminate the thermal hypothesis. That's what you encounter at 1000 km high or so.
Pioneer lives in a vacuum *one million* times better that 5*10-6 Torr and there are still thermal effects.


@Mulletron must have meant "So the vacuum did eliminate the [natural convection] thermal hypothesis" .  He wrote "thermal hypothesis" without the words "natural convection" because he thought that it was obviously understood he was referring to convection in the context he was discussing:  what is the difference between ambient pressure and  5*10-6 Torr.  Obviously this pressure makes a huge difference for natural convection currents, as a trivial calculation can show that 5*10-6 Torr vacuum does eliminate the natural convection hypothesis that was advanced in most media (including Prof. Baez's blog) as the most likely explanation for the measurements.  @Frobnicat, was proposing the thermal convection hypothesis as a possible artifact, and he worked some equations in Thread 1.  I do not recall @Frobnicat objecting to a 5*10-6 Torr vacuum nullifying the hypothesis of natural thermal convection from the EM Drive being able to produce the measured forces. 
...

My specific proposition at the time was a pressure difference build-up between inside cavity and outside, enough to "jet" supposedly warmed air through small apertures of the cavity. I recall having said about 5 month ago I would give a definite answer within 2 month (so much for my credibility with deadlines). This specific "warm jet" hypothesis would have been easily nullified even by a very more modest rough vacuum level of a few Torr. So this specific hypothesis is now ruled out, and I'm glad the experimental progress at Eaglworks outpaced my episodic armchair scientist investment on that matter  :D

For anything pressure related (even complex anisotropic molecular flow like in a Crookes radiometer effect) a characteristic value would be an extreme case of a completely asymmetric pressure difference around 5e-6 Torr = 6.65e-4 Pa on a surface of 11'' diameter = 6.13e-2 m² (roughly, cross section of the frustum) that yields about 41µN. Unfortunately still in the ballpark of the signal... so maybe hardcore sceptics could still require 2 orders of magnitude better vacuum, just to be sure... (edit : I see JPLeRouzic reach similar conclusion)

Anyway, it's clear that it becomes difficult to find remaining thermo-aerodynamic effects that would need better than 5e-6 Torr to be ruled out : this is good enough to put possible asymmetric gas flow effects much below the observed signal.

Quote

Concerning <<Pioneer lives in a vacuum *one million* times better that 5*10-6 Torr and there are still thermal effects>>, yes the Pioneer anomaly was successfully explained by a team of JPL engineers/scientists on the basis of thermal radiation effects (not natural convection thermal effects). 

Thermal radiation, an entirely different (from convection) means of thermal transport, will take place even in a perfect vacuum, as thermal radiation is electromagnetic radiation and therefore does not need a gas or a fluid as a transport medium.  On the other hand natural convection is completely dependent on fluid or gas medium to carry it, and without a fluid or a gas, there cannot be any thermal convection.  Thermal convection involves fluid advection.

Radiative heat transfer is the only form of heat transfer that can take place in a vacuum.  Radiative heat transfer depends on the difference between the fourth power of the temperatures between the radiating surface and the irradiated surface, the surface areas and the emissivities of the radiating and irradiated surfaces and the Stefan–Boltzmann constant .  It may be important for the Pioneer anomaly: an extremely small effect comprising an acceration towards the Sun, of only 8.74×10^(−10) m/s^2, due to asymmetrical radiation of heat from the Radioisotope thermoelectric generators or from the spacecraft electronics reflecting from the back of the spacecraft's dish-like main antenna, causing a recoil like sunlight striking a solar sail.

But in the case of the EM Drive, how much does the temperature difference between radiated and irradiated surfaces have to be to to justify the measured thrust forces ?

Very clear.
For the last question, I would be tempted to say : temperatures needing 300 times the actual operating power of the system, since the force/power ratios are ~300 times better than a photon rocket.

Unless the space between the frustum and the vacuum chamber's walls could amplify the effect by bouncing hotter IR photons around a few times and make high temperature, high radiation pressure "traps". IR photons are emitted around all the time, but at thermal equilibrium those radiative pressures have equal contribution on all side of an object in the bath. Anyone knows the typical reflectivity of more or less polished metals in thermal IR ? That would put an upper bound on the "boost" factor compared to the photon rocket equivalent thrust, likely much below 300.
I agree.

The comments from JPLeRouzic  are well taken. @frobnicat's comments above are well stated.

And this is predicated on the interior of the cavity being at the same pressure as the exterior, as previously discussed with and by Paul March.
Title: Re: EM Drive Developments - related to space flight applications - Thread 2
Post by: Rodal on 02/28/2015 10:16 pm
@ RODAL

Still have question about the Shawyer "Demo" cavity w/ 174mN.  What are the current estimates of the cone dimensions, frequency (3.85GHz?), and Q (6000 est?).  When I put in TM02 and 450W, I get 174.8microN, rather than the 174milliN reported.  I would like to recheck those numbers.

Even w/ Q=45000, I need to get X up around (65 Very high mode) to get those numbers.  Is that possible w/ 3.85GHz ??

Thanks

Concerning dimensions for Shawyer's flight thruster, these are the latest estimates I recall:

Big diameter is 265 mm
Small diameter is 189 mm
Height is 164 mm.

From:  http://forum.nasaspaceflight.com/index.php?topic=36313.msg1336809#msg1336809 and http://forum.nasaspaceflight.com/index.php?topic=36313.msg1336754#msg1336754

Shawyer's Flight Thruster development programme. A 3.85GHz thruster weighing 2.92 Kg,

Mean specific thrust = 326mN/kW  (Tests were carried out over an input power range of 150W to 450W.)

At 450W, this thrust/power mean gives a thrust of 146.7 mN  instead of the 174milliN you report for the "Demo".  However, the plot on  http://emdrive.com/flightprogramme.html gives about 170 milliNewton, as you report for the "Demo". I do not see any Q reported on these tests.

This is the only reference I have for data for the Flight Thruster: http://emdrive.com/flightprogramme.html



Concerning Shawyer's Experimental

bigDiameter = 16  centimeter;
smallDiameter = 12.7546  centimeter;
length = 15.6  centimeter;



Concerning Shawyer's Demo

cavityLength = 0.345;
bigDiameter = 0.28;
smallDiameter = 0.128853 cm;



Concerning the data for the Demo and for the Experimental tests of Shawyer, they are in this report: http://www.emdrive.com/IAC-08-C4-4-7.pdf

For the Demo Shawyer states  (notice a number of important differences with the numbers you quoted, including frequency.  Please consider whether you are referring to another test by Shawyer, perhaps to the flight thruster, instead of the Demo):

<<The engine was built to operate at 2.45 GHz, with a design factor of 0.844 and has a measured Q of 45,000 for an overall diameter of 280 mm. The microwave source is a water cooled magnetron with a variable output power up to a maximum of 1.2 kW>>

<<Fig 7 gives test results for 3 Vertical Thrust test runs under the same input and tuner conditions but for thrust vectors in the Up, Down and Horizontal directions. This clearly illustrates the loss of measured weight for the Up vector, the increase in measured weight for the Down vector, and a mean weight change close to zero, for the horizontal vector. These early comparative tests yielded specific thrusts around 80mN/kW.>>

<<Fig 8 shows the results for a test run with the engine on the balance and then with it suspended above the balance. This illustrates the thrust measurements were not subject to EMC effects. Specific thrust for this test was 214mN/kW.>>

<<Fig 10 gives the result of a typical test run, where the Demonstrator Engine produced a thrust of 10.4 gm against a calibrated friction torque of 7.1 gm. Input power was 421W, giving a specific thrust of 243 mN/kW.
The frequency offset curve shows that initial magnetron thermal drift ends with frequency lock. At this point, 130 secs into the test run, the velocity data shows the start of acceleration under power. The prior thermal drift period, with no acceleration, shows that the thrust is not a result of spurious thermal effects. When the power is turned off, at 210 secs, there is a coast period as the slosh effects of 5kg of coolant maintain a reduced acceleration. This is followed by the deceleration due to the friction torque. A maximum velocity of 2cm/s was achieved and a total distance of 185cm was “flown”. The direction of acceleration was opposite to the direction of thrust, thus conclusively proving that the engine obeys Newton’s laws, and that although no reaction mass is ejected, the engine is not a reactionless machine. An  electrical reaction occurs between the EM wave and the reflector surfaces of the resonator, resulting in an input impedance change with acceleration. This is seen in the
power curve in fig 10.>>
Title: Re: EM Drive Developments - related to space flight applications - Thread 2
Post by: flux_capacitor on 02/28/2015 10:34 pm
....

Interesting thing to note: Shawyer claims the engine starts to accelerate only when the magnetron frequency locks to the resonant frequency of the thruster, following an initial warm up period where it does not move; thus according to him eliminating possible spurious forces.

....

That's very interesting stuff.  Thank you for bringing it up.

Do you have a reference as to where Shawyer  made that interesting claim?

Yes, it is in the same 2008 paper you have just cited in the message above this one, entitled "MICROWAVE PROPULSION – PROGRESS IN THE EMDRIVE PROGRAMME":
http://www.emdrive.com/IAC-08-C4-4-7.pdf
Where Shawyer noted:
Quote
The frequency offset curve shows that initial magnetron thermal drift ends with frequency lock. At this point, 130 secs into the test run, the velocity data shows the start of acceleration under power. The prior thermal drift period, with no acceleration, shows that the thrust is not a result of spurious thermal effects. When the power is turned off, at 210 secs, there is a coast period as the slosh effects of 5kg of coolant maintain a reduced acceleration. This is followed by the deceleration due to the friction torque. A maximum velocity of 2cm/s was achieved and a total distance of 185cm was "flown".
Title: Re: EM Drive Developments - related to space flight applications - Thread 2
Post by: Rodal on 02/28/2015 10:40 pm
....

I think that @Notsosureofit's numbers are for the Flight Thruster (instead of the Demo he references to).  I base that on the frequency (3.85GHz) quoted by @Notsosureofit.

Do you have a reference giving frequency, Q, power and thrust measurements for the Flight Thruster?

This is the only reference I have for data for the Flight Thruster: http://emdrive.com/flightprogramme.html,  I can't find a Q reported for these Flight programme tests.

(http://emdrive.com/images/thruster1.jpg)
Title: Re: EM Drive Developments - related to space flight applications - Thread 2
Post by: flux_capacitor on 02/28/2015 11:07 pm
Do you have a reference giving frequency, Q, power and thrust measurements for the Flight Thruster?

This is the only reference I have for data for the Flight Thruster: http://emdrive.com/flightprogramme.html,  I can't find a Q reported.

Several of them for the "Flight Thruster Programme":

Shawyer's CEAS 2009 paper (http://www.emdrive.com/CEAS2009paper.doc) stated, page 9:
Quote
The Flight thruster programme covers the design and development of a 300 Watt C Band flight thruster. This has a specified thrust of 85 mN, and a mass of 2.92Kg. Overall dimensions are 265mm diameter at the baseplate and a height of 164mm.

Then in the 2010 Toulouse TECHNO DIS paper (http://www.emdrive.com/Toulouse2010paper01.doc), page 8:
Quote
Development testing of the unit, up to a power of 600 W, is under way, and to date, has given a mean specific thrust of 330 mN/kW.
[…]
This is needed to ensure the input frequency matches the resonant frequency of the high Q (60,000) cavity, over the full input power range and the qualification temperature specification.

And in the IAC 2013 paper (http://www.emdrive.com/IAC13paper17254.v2.pdf), page 4:
Quote
The Dynamic performance of the non superconducting Flight Test model, manufactured and tested by SPR Ltd, and described in REF 3 [N.B.: 2010 Toulouse TECHNO DIS paper] was modeled with a cavity Qu = 50,000 and Fres=3.85 GHz.

Finally the mean specific thrust of 326mN/kW over 19 test runs of up to 90 secs duration from 150 W to 450 W was found on the web page http://emdrive.com/flightprogramme.html
As well as the diagram which shows the maximum thrust achieved @ 450 W:
(http://emdrive.com/images/results2.jpg)
Title: Re: EM Drive Developments - related to space flight applications - Thread 2
Post by: Rodal on 02/28/2015 11:10 pm
....
Excellent.  You are our resident expert on Shawyer's tests.  We are lucky to have you on the thread  :)

Title: Re: EM Drive Developments - related to space flight applications - Thread 2
Post by: Rodal on 02/28/2015 11:14 pm
@ RODAL

Still have question about the Shawyer "Demo" cavity w/ 174mN.  What are the current estimates of the cone dimensions, frequency (3.85GHz?), and Q (6000 est?).  When I put in TM02 and 450W, I get 174.8microN, rather than the 174milliN reported.  I would like to recheck those numbers.

Even w/ Q=45000, I need to get X up around (65 Very high mode) to get those numbers.  Is that possible w/ 3.85GHz ??

Thanks

 Can't calculate mode shape today.  Maybe tomorrow.  More likely next week
Title: Re: EM Drive Developments - related to space flight applications - Thread 2
Post by: frobnicat on 02/28/2015 11:20 pm
...
??? When thermal expansion displaces a part of a system relative to the rest of a system, the rest of the system will recoil. Whatever displaces a part of a system relative to the rest of the system, this displacement implies a force F_system_part. The rest of the system will recoil. Recoil is the acceleration due to the opposite force (F_part_system). An unrestrained, homogeneous, isotropic, free to expand but in one direction material, will produce a force when expanding against the wall against which it rests.

Why is it so hard to reach consensus now ?
With respect to the HD PE dielectric, now I see your thinking.  I think you think of it as a dynamic problem, for example, if instead of the HD PE we would have an explosive inside the cavity glued to the left wall, the effect of the explosive would be to force the wall towards the left.

However, thermal expansion does not work like that.  There are no dynamic forces caused by a material experiencing thermal expansion such that it is free to expand.  Thermal expansion is a very slow process that does not involve second order derivatives with respect to time.  If a uniaxial isotropic material is glued to a wall, it will just expand without producing any force whatsoever on the wall it is glued to. 

If free, it will just expand, strain = alpha *deltaT.

No stress if it is free to expand.

Can't use thermal expansion as a form of propellant-less propulsion.

Sorry I'm lagging in response, read your posts carefully though, and agree on :

If buckles to the left -> initial impulse to the right (contrary to observed movement)
If buckles to the right -> initial impulse to the left (helping observed movement initially, but can't explain the sustained displacement against the balance rest equilibrium)
In free space the CoM of the whole system don't move, but I would add : an outside hull's position can still be displaced if some part is moved inside. No net momentum gained at the end (no speed gained overall).

For the part in blue : I don't see the qualitative difference you make between an explosive detonating on a left wall and a plastic block glued to a left wall and thermally expanding, in both cases it is thermal expansion. A material that expands is never completely free to expands, it must at least overcome its own inertia, even if slow quasi-static expansion occurs... Take for instance a slice of the material at the right of the slab, when the whole slab is heating, its distance from the left wall increase (meaning non 0 velocity), it has a mass, it was initially at velocity 0, so going from velocity 0 to some non null velocity implies an acceleration at some time, and this slice has a mass, so the rest of the slab had to push it, hence stress. The slower the process the weaker the forces, but weak is not 0. So maybe there is no leftward force of the plastic block expanding rightward during the phase of heating at constant rate if that means constant velocity of block's CoM, but at some time between start of heating and constant rate there has been a leftward force (on the wall). Likewise at some time between this phase of constant rate heating (constant expansion velocity) and stabilisation at constant position (end of movement to the right, 0 velocity again) there must be a deceleration, that is a leftward acceleration that corresponds to the block pulling the wall toward the right : if it wasn't glued it couldn't decelerate and would leave the left wall.

The integrated forces between initial static position (at 0 relative velocity) and final static position (at 0 relative velocity) will be 0, since by definition the force (kg m/s²) when integrated is the exchanged momentum (kg m/s). So yes "Can't use thermal expansion as a form of propellant-less propulsion" in the sense that it can't give long lasting momentum (deltaV) to a spacecraft. But it can lend small temporary momentum to be paid back a little later, since it can give momentary speed, it can give long lasting displacement : by integrating the momentary "bump" in momentum(t) the craft of mass M will have shifted its position by X while the moving part of mass m has moved x, with MX=-mx  (in kg m units). And from this point of view, this displacement is the same whether the mass m moved distance x fast or ultra-slowly. Equalling force terms to 0 because movement is slow would fail to predict that : weak forces for long time do the same "displacement job" as strong forces for short time.

So a spacecraft A that is already at exact same velocity than spacecraft B but a few inches apart could dock to it just by shifting an inner part's mass. Obviously the mass shift can't be recycled to "inchworm" spacecraft's position again and again : to put the part at it's original position means losing the whole system's gained displacement. But the point is : that don't depend on the specifics of what makes the part move.

So what's the point ? Regardless of what exactly would make a part move relative to another in the frustum, we know that 50µN for 45s, interpreted as a slowly occurring recoil effect (ever-accelerating part), amounts for an integrated "mass displacement" of .5*50e-6*45²=0.05 kg m   that is either 50 grams moving 1m, or 1kg moving 5cm, or 10kg moving 5mm. Sorry, this is a repeat.

But I have 2 new ideas why a much lower displacement could explain an apparently long lasting force reading on the balance, not as a recoil effect, but as a side effect of the displacement of CoM relative to fixation point when taking into account gravity and balance system as a whole... unsure if it holds water or not, will make some drawings  :)
Title: Re: EM Drive Developments - related to space flight applications - Thread 2
Post by: lasoi on 02/28/2015 11:32 pm
For what it's worth, I don't think the given numbers (265mm base diameter and 164mm axial height) can possibly be right for the inner cavity of the thruster in the photo- not unless whatever plates that are inserted within the ends are several centimeters thick.  The only way that the height is only .6188 or 62% of the base diameter is if the 265mm base diameter is measured from the widest part of the exterior of the base plate, and the height is measured from where the slope of the exterior walls flattens out on top and bottom as roughly indicated by the yellow arrows in the attached photo.  If that happened to be the case, then 189mm would be the exterior diameter of the top plate.

But there's almost no way, in my humble opinion, that 265mm, 164mm and 189mm are correct for the inner cavity.  Unless, again, the plates (diaelectric?) inserted inside the end plates in the inner cavity are several centimeters thick. I'm pretty confident in saying that.  The distortion in the photo isn't nearly enough to account the discrepancy.

I only point this out to maybe prevent some confusion, and now retreat to my recliner and leave the real work to minds more learned than mine.
Title: Re: EM Drive Developments - related to space flight applications - Thread 2
Post by: Rodal on 02/28/2015 11:46 pm
For what it's worth, I don't think the given numbers (265mm base diameter and 164mm axial height) can possibly be right for the inner cavity of the thruster in the photo- not unless whatever plates that are inserted within the ends are several centimeters thick.  The only way that the height is only .6188 or 62% of the base diameter is if the 265mm base diameter is measured from the widest part of the exterior of the base plate, and the height is measured from where the slope of the exterior walls flattens out on top and bottom as roughly indicated by the yellow arrows in the attached photo.  If that happened to be the case, then 189mm would be the exterior diameter of the top plate.

But there's almost no way, in my humble opinion, that 265mm, 164mm and 189mm are correct for the inner cavity.  Unless, again, the plates (diaelectric?) inserted inside the end plates in the inner cavity are several centimeters thick. I'm pretty confident in saying that.  The distortion in the photo isn't nearly enough to account the discrepancy.

I only point this out to maybe prevent some confusion, and now retreat to my recliner and leave the real work to minds more learned than mine.
In summary, what do you think are the most likely inner dimensions?
Title: Re: EM Drive Developments - related to space flight applications - Thread 2
Post by: Notsosureofit on 02/28/2015 11:49 pm
Thanks guys !

So I have data for everything except the mode(s), and temp size for the flight cone.

I need to use X numbers of 21, 26, and 77, respectively, for the proto, demo and flight cones to get those numbers. (w/o dielectric that is)

What that means is TBD of course.  There may be another variable involved, in or out of favor of a real effect.
Title: Re: EM Drive Developments - related to space flight applications - Thread 2
Post by: lasoi on 02/28/2015 11:58 pm
In summary, what do you think are the most likely inner dimensions?
To be honest I'm a little baffled that the given numbers (265mm base and 164mm height) are so irreconcilable with the measured dimensions of the thruster in the photo.  I'm pretty confident with my pixel scales.  Is it possible that the dielectric plates you guys keep referring to, located within the end plates, are that thick?

Is it possible that Shawyer would post the exterior diameter for the base instead of the inner diameter of the resonance cavity?

Also, from reading Mullerton's posts, it seems that the frustum walls are maybe only a few mm's thick.  Is this correct?
Title: Re: EM Drive Developments - related to space flight applications - Thread 2
Post by: Rodal on 03/01/2015 12:00 am
For the part in blue : I don't see the qualitative difference you make between an explosive detonating on a left wall and a plastic block glued to a left wall and thermally expanding, in both cases it is thermal expansion. A material that expands is never completely free to expands, it must at least overcome its own inertia, even if slow quasi-static expansion occurs... Take for instance a slice of the material at the right of the slab, when the whole slab is heating, its distance from the left wall increase (meaning non 0 velocity), it has a mass, it was initially at velocity 0, so going from velocity 0 to some non null velocity implies an acceleration at some time, and this slice has a mass, so the rest of the slab had to push it, hence stress. The slower the process the weaker the forces, but weak is not 0. So maybe there is no leftward force of the plastic block expanding rightward during the phase of heating at constant rate if that means constant velocity of block's CoM, but at some time between start of heating and constant rate there has been a leftward force (on the wall). Likewise at some time between this phase of constant rate heating (constant expansion velocity) and stabilisation at constant position (end of movement to the right, 0 velocity again) there must be a deceleration, that is a leftward acceleration that corresponds to the block pulling the wall toward the right : if it wasn't glued it couldn't decelerate and would leave the left wall.

The integrated forces between initial static position (at 0 relative velocity) and final static position (at 0 relative velocity) will be 0, since by definition the force (kg m/s²) when integrated is the exchanged momentum (kg m/s). So yes "Can't use thermal expansion as a form of propellant-less propulsion" in the sense that it can't give long lasting momentum (deltaV) to a spacecraft. But it can lend small temporary momentum to be paid back a little later, since it can give momentary speed, it can give long lasting displacement : by integrating the momentary "bump" in momentum(t) the craft of mass M will have shifted its position by X while the moving part of mass m has moved x, with MX=-mx  (in kg m units). And from this point of view, this displacement is the same whether the mass m moved distance x fast or ultra-slowly. Equalling force terms to 0 because movement is slow would fail to predict that : weak forces for long time do the same "displacement job" as strong forces for short time.

So a spacecraft A that is already at exact same velocity than spacecraft B but a few inches apart could dock to it just by shifting an inner part's mass. Obviously the mass shift can't be recycled to "inchworm" spacecraft's position again and again : to put the part at it's original position means losing the whole system's gained displacement. But the point is : that don't depend on the specifics of what makes the part move.

So what's the point ? Regardless of what exactly would make a part move relative to another in the frustum, we know that 50µN for 45s, interpreted as a slowly occurring recoil effect (ever-accelerating part), amounts for an integrated "mass displacement" of .5*50e-6*45²=0.05 kg m   that is either 50 grams moving 1m, or 1kg moving 5cm, or 10kg moving 5mm. Sorry, this is a repeat.


For a given temperature change, the "speed of thermal expansion" is constant (it is proportional to the speed of sound in the material), the derivative of a constant is zero.

There is no first order derivative with respect to time for the speed of sound in a material: it is zero.  The speed of sound is a constant.

For a given temperature change,, there is no second order derivative with respect to time for thermal expansion: it is zero, because it proceeds at constant speed.

There is no "acceleration of sound" in a material, not in High Density Polyethylene.

For a given temperature change, there is no "thermal expansion acceleration".

An explosive blast has acceleration.  For a given temperature change, thermal expansion does not.

Not everything has acceleration.  Some phenomena proceed at constant speed.  Sound has a constant speed in a material.  Light also has a constant speed.  For a given temperature change, thermal expansion proceeds at a constant speed. 

Title: Re: EM Drive Developments - related to space flight applications - Thread 2
Post by: lasoi on 03/01/2015 12:58 am
If the 265mm base diameter that Shawyer gave does in fact refer the the exterior diameter of the base plate and if the height is measured from the yellow arrows indicated in the aforementioned photo, which is the only way I can reconcile his figures with the photo, then my best guess for the inner resonance cavity is:

224mm base diameter
145mm top diameter
164mm height (given by Shawyer).

Here's the chart if anyone is interested, cluttered though it has admittedly become.
Title: Re: EM Drive Developments - related to space flight applications - Thread 2
Post by: Notsosureofit on 03/01/2015 01:20 am
If the 265mm base diameter that Shawyer gave does in fact refer the the exterior diameter of the base plate and if the height is measured from the yellow arrows indicated in the aforementioned photo, which is the only way I can reconcile his figures with the photo, then my best guess for the inner resonance cavity is:

224mm base diameter
145mm top diameter
164mm height (given by Shawyer).

Here's the chart if anyone is interested, cluttered though it has admittedly become.

Thanks!  That brings that X number down to ~55.  Still a very high mode, but 3.85GHz !
Title: Re: EM Drive Developments - related to space flight applications - Thread 2
Post by: Rodal on 03/01/2015 01:25 am
If the 265mm base diameter that Shawyer gave does in fact refer the the exterior diameter of the base plate and if the height is measured from the yellow arrows indicated in the aforementioned photo, which is the only way I can reconcile his figures with the photo, then my best guess for the inner resonance cavity is:

224mm base diameter
145mm top diameter
164mm height (given by Shawyer).

Here's the chart if anyone is interested, cluttered though it has admittedly become.

Thanks!  That brings that X number down to ~55.  Still a very high mode, but 3.85GHz !

McCulloch's equation gives 148 milliNewtons for Shawyer's demo, comparing pretty well with Shawyer's reported measurements of 80 to 214 millinNewtons, see:

http://physicsfromtheedge.blogspot.com/2015/02/mihsc-vs-emdrive-data-3d.html

I understand that you need to use X=26 in your equation for Shawyer's Demo, also a high mode.

On the other's hand McCulloch's equation gives thrust forces greater by a factor of 2 to 20 times for NASA Eagleworks test results, and your equation may compare better than McCulloch's for NASA's tests.
Title: Re: EM Drive Developments - related to space flight applications - Thread 2
Post by: Notsosureofit on 03/01/2015 01:36 am
If the 265mm base diameter that Shawyer gave does in fact refer the the exterior diameter of the base plate and if the height is measured from the yellow arrows indicated in the aforementioned photo, which is the only way I can reconcile his figures with the photo, then my best guess for the inner resonance cavity is:

224mm base diameter
145mm top diameter
164mm height (given by Shawyer).

Here's the chart if anyone is interested, cluttered though it has admittedly become.

Thanks!  That brings that X number down to ~55.  Still a very high mode, but 3.85GHz !

McCulloch's equation gives 148 milliNewtons for Shawyer's demo, comparing pretty well with Shawyer's reported measurements of 80 to 214 millinNewtons, see:

http://physicsfromtheedge.blogspot.com/2015/02/mihsc-vs-emdrive-data-3d.html

I understand that you need to use X=26 in your equation for Shawyer's Demo, also a high mode.

On the other's hand McCulloch's equation is off by a factor >10 for NASA Eagleworks test results.

That is an interesting fit.

Of course I havn't seen Mike's derivation in 3D.  I only follow the Equivalence argument the best I can w/o fudge factors and see what comes out.  At the moment I think these are maximums if you have all the parameters and I like the NASA results because they seem to have eliminated more sources of error.  Still, it may all be fiction which is what we want to find out.
Title: Re: EM Drive Developments - related to space flight applications - Thread 2
Post by: frobnicat on 03/01/2015 01:37 am


Please, I'm not accustomed to such tools as those used for thermoacoustics. There is not enough of a life to learn all... Just applying more rudimentary reasoning : my slab of PTFE, volumetrically heated, free floating alone, would expand as much leftward than rightward. If glued to a left wall inside an heavy hull, it will expand more rightward than leftward (in the same inertial reference frame) :  necessarily it means that at some time there have been a force of the wall on the slab, otherwise a solid object as a wall wouldn't have been needed to prevent the slab to expand leftward. If such wall exerted no force at all, the slab would have expanded through the wall as if it didn't existed. Ahem.

So if by "speed of thermal expansion is constant" you mean a phase when in all place there is a constant rate dT/dt, and therefore all positions are also changing at constant rate (dx/dt constant), well yes, in such phase there is no acceleration and no force. But what is going on between dT/dt = 0 and dT/dt =cst, that is between an instant where dx/dt=0 and an instant where dx/dt=cst ? Surely at some time there is a force involved. Now, maybe it's usually neglected because it is negligible for all practical purpose, but if we are to integrate µN ...

So if there is no "thermal expansion acceleration" how do you go from a state of no thermal expansion to a state of ongoing thermal expansion ? Step like ? But this step would still represent some momentum impulse...
Please define the "speed of thermal expansion" that is supposed to be constant, I don't get it.

For instance if one is heating volumetrically a mass of material at linearly growing power at constant rate P(t)=cst t, then we have a heat energy Q that goes as t², and so does temperature T, and if we assume a slow (quasi-static) rate, the material will be at length  l(t)=cst + cst*t²  hence ldot=velocity=cst*t hence acceleration=cst hence constant inertial force to overcome...
Title: Re: EM Drive Developments - related to space flight applications - Thread 2
Post by: Rodal on 03/01/2015 02:01 am


Please, I'm not accustomed to such tools as those used for thermoacoustics. There is not enough of a life to learn all... Just applying more rudimentary reasoning : my slab of PTFE, volumetrically heated, free floating alone, would expand as much leftward than rightward. If glued to a left wall inside an heavy hull, it will expand more rightward than leftward (in the same inertial reference frame) :  necessarily it means that at some time there have been a force of the wall on the slab, otherwise a solid object as a wall wouldn't have been needed to prevent the slab to expand leftward. If such wall exerted no force at all, the slab would have expanded through the wall as if it didn't existed. Ahem.

So if by "speed of thermal expansion is constant" you mean a phase when in all place there is a constant rate dT/dt, and therefore all positions are also changing at constant rate (dx/dt constant), well yes, in such phase there is no acceleration and no force. But what is going on between dT/dt = 0 and dT/dt =cst, that is between an instant where dx/dt=0 and an instant where dx/dt=cst ? Surely at some time there is a force involved. Now, maybe it's usually neglected because it is negligible for all practical purpose, but if we are to integrate µN ...

So if there is no "thermal expansion acceleration" how do you go from a state of no thermal expansion to a state of ongoing thermal expansion ? Step like ? But this step would still represent some momentum impulse...
Please define the "speed of thermal expansion" that is supposed to be constant, I don't get it.

For instance if one is heating volumetrically a mass of material at linearly growing power at constant rate P(t)=cst t, then we have a heat energy Q that goes as t², and so does temperature T, and if we assume a slow (quasi-static) rate, the material will be at length  l(t)=cst + cst*t²  hence ldot=velocity=cst*t hence acceleration=cst hence constant inertial force to overcome...
What changes with time (very slowly, due to the low thermal diffusivity of HD PE) is the temperature.

So, one has to solve Fourier's equation (temperature vs time). And the thermal expansion and thermal stresses follow from that.  There is no second order derivative with respect to time in Fourier's equation.  There is only a first order derivative with respect to time, and other terms containing a second order derivative with respect to space.

There is no "acceleration of temperature term in Fourier's equation".  Hence no intrinsic "thermal wave Fourier equation". No "inertia of temperature" term and no "inertia of thermal expansion" term.

You may compute a thermal strain changing with time due to the temperature changing with time: T(t) hence epsilon(t)=alpha*T(t), and a strain rate also changing with time (due to the change with time of the temperature).
The coefficient of thermal expansion is very small, by several orders of magnitude. There is nothing there to produce a large acceleration of thermal deformation term.  The "acceleration of thermal deformation" in HD PE should be negligible, and when you multiply it by the mass it should give a negligible force. Such dynamic terms are neglected in thermal problems.  Even in the Pioneer anomaly problem, involving extremely small accelerations, what is discussed is the thermal radiation producing recoil forces, but nobody has brought up "thermal expansion forces due to the thermal expansion changing with time", one reason for this being that such thermal expansion cannot accelerate the center of mass for an object free in space, of course.  But no issues of "acceleration of thermal expansion" bringing dynamic forces on the antenna either.
Title: Re: EM Drive Developments - related to space flight applications - Thread 2
Post by: zen-in on 03/01/2015 04:02 am
....

Interesting thing to note: Shawyer claims the engine starts to accelerate only when the magnetron frequency locks to the resonant frequency of the thruster, following an initial warm up period where it does not move; thus according to him eliminating possible spurious forces.

....

That's very interesting stuff.  Thank you for bringing it up.

Do you have a reference as to where Shawyer  made that interesting claim?

Yes, it is in the same 2008 paper you have just cited in the message above this one, entitled "MICROWAVE PROPULSION – PROGRESS IN THE EMDRIVE PROGRAMME":
http://www.emdrive.com/IAC-08-C4-4-7.pdf
Where Shawyer noted:
Quote
The frequency offset curve shows that initial magnetron thermal drift ends with frequency lock. At this point, 130 secs into the test run, the velocity data shows the start of acceleration under power. The prior thermal drift period, with no acceleration, shows that the thrust is not a result of spurious thermal effects. When the power is turned off, at 210 secs, there is a coast period as the slosh effects of 5kg of coolant maintain a reduced acceleration. This is followed by the deceleration due to the friction torque. A maximum velocity of 2cm/s was achieved and a total distance of 185cm was "flown".


I don't understand this "slosh effects of 5kg of coolant maintain a reduced acceleration".   For any acceleration to occur there has to be a force acting on the device.   The coolant is part of the device; it is balanced on an air bearing like everything else.   When does the coolant pump switch on?   If the coolant pump is controlled by a temperature sensor maybe it gets switched on at 25 Sec. when the magnetron gets to a setpoint temperature.   That could produce a torque since a mass is being accelerated.   The dynamic test results, fig. 10 of the IAC '08 paper referenced in the paragraph above this quote does show continuing acceleration after the RF power was switched off.    Pumping coolant out of a reservoir and through the magnetron's cooling channels could be the reason for the change in velocity.

A quick search finds examples of injection locking of magnetrons driving a cavity.  This is done by feeding a low level signal at the desired frequency into the magnetron cavity.   Locking a magnetron to the resonant frequency of an external cavity is something else.    If Shawyer's device is locking the magnetron to the desired frequency it shouldn't take 25 Sec.    Maybe someone can correct me here.
Title: Re: EM Drive Developments - related to space flight applications - Thread 2
Post by: Chrochne on 03/01/2015 10:24 am
EmDrive is already discussed as well on the LinkedIn. In the group called "Advanced propulsion" There are 1600 + members in this group. There is already one recent article on the EmDrive and it brings the attention of people from science and public community.

I recently wrote very enthusiastic article about the EmDrive there. https://www.linkedin.com/groups/EmDrive-Confirmation-It-works-in-104738.S.5977756203273441280?trk=groups_most_recent-0-b-ttl&goback=.gde_104738_member_5952824904813465605.gmp_104738.gmr_104738

I am doing this mostly to try to bring more attention of the media and public to this technology and perhaps more funds for this project at NASA.

Also please - If you can like the article  :P . I know it is way too enthusiatic, but that is the way to bring the attention of public. More likes bring the article to wider public.

With kind regards,

Standa V.
Title: Re: EM Drive Developments - related to space flight applications - Thread 2
Post by: frobnicat on 03/01/2015 10:33 am


Please, I'm not accustomed to such tools as those used for thermoacoustics. There is not enough of a life to learn all... Just applying more rudimentary reasoning : my slab of PTFE, volumetrically heated, free floating alone, would expand as much leftward than rightward. If glued to a left wall inside an heavy hull, it will expand more rightward than leftward (in the same inertial reference frame) :  necessarily it means that at some time there have been a force of the wall on the slab, otherwise a solid object as a wall wouldn't have been needed to prevent the slab to expand leftward. If such wall exerted no force at all, the slab would have expanded through the wall as if it didn't existed. Ahem.

So if by "speed of thermal expansion is constant" you mean a phase when in all place there is a constant rate dT/dt, and therefore all positions are also changing at constant rate (dx/dt constant), well yes, in such phase there is no acceleration and no force. But what is going on between dT/dt = 0 and dT/dt =cst, that is between an instant where dx/dt=0 and an instant where dx/dt=cst ? Surely at some time there is a force involved. Now, maybe it's usually neglected because it is negligible for all practical purpose, but if we are to integrate µN ...

So if there is no "thermal expansion acceleration" how do you go from a state of no thermal expansion to a state of ongoing thermal expansion ? Step like ? But this step would still represent some momentum impulse...
Please define the "speed of thermal expansion" that is supposed to be constant, I don't get it.

For instance if one is heating volumetrically a mass of material at linearly growing power at constant rate P(t)=cst t, then we have a heat energy Q that goes as t², and so does temperature T, and if we assume a slow (quasi-static) rate, the material will be at length  l(t)=cst + cst*t²  hence ldot=velocity=cst*t hence acceleration=cst hence constant inertial force to overcome...
What changes with time (very slowly, due to the low thermal diffusivity of HD PE) is the temperature.

So, one has to solve Fourier's equation (temperature vs time). And the thermal expansion and thermal stresses follow from that.  There is no second order derivative with respect to time in Fourier's equation.  There is only a first order derivative with respect to time, and other terms containing a second order derivative with respect to space.

There is no "acceleration of temperature term in Fourier's equation".  Hence no intrinsic "thermal wave Fourier equation". No "inertia of temperature" term and no "inertia of thermal expansion" term.

You may compute a thermal strain changing with time due to the temperature changing with time: T(t) hence epsilon(t)=alpha*T(t), and a strain rate also changing with time (due to the change with time of the temperature).
The coefficient of thermal expansion is very small, by several orders of magnitude. There is nothing there to produce a large acceleration of thermal deformation term.  The "acceleration of thermal deformation" in HD PE should be negligible, and when you multiply it by the mass it should give a negligible force. Such dynamic terms are neglected in thermal problems.  Even in the Pioneer anomaly problem, involving extremely small accelerations, what is discussed is the thermal radiation producing recoil forces, but nobody has brought up "thermal expansion forces due to the thermal expansion changing with time", one reason for this being that such thermal expansion cannot accelerate the center of mass for an object free in space, of course.  But no issues of "acceleration of thermal expansion" bringing dynamic forces on the antenna either.

I begin to see more clearly why we are not really talking of the same thing, I never said there is a large acceleration of thermal deformation term, I said however small it is nonetheless !=0 and accompanying reaction force!=0. This is usually negligible and generally neglected (assumed to be exactly 0) because for most situations it makes no significant difference, and indeed even for very small acceleration effects like for the Pioneer anomaly this would be irrelevant, because in this case what is to be explained is a very small but long lasting acceleration leading to accumulated deltaV. Pioneer telemetry can track extremely accurate velocity and velocity changes but wouldn't mind a one shot position shift of a mm or so (that occurred only once, at warm-up).

But please consider the following situation, a 11kg free floating spacecraft A with its hull 500µm to the left from hull of spacecraft B. Both velocities exactly the same (500µm stable distance, neglecting gravitational or other coupling). On a left inner wall of A is glued a 10cm sided cube of hdpe, about 1kg (give or take). Now this block is heated so that its temperatures goes from 20°C to 30°C. Let's say thermal expansion coefficient (http://www.engineeringtoolbox.com/pipes-temperature-expansion-coefficients-d_48.html) of hdpe is 120e-6 (m/m)/°C  (this is for piping, maybe a little different for a cube) the horizontal size of the cube is now 120e-6*.1m*10°C = 120 µm longer. The CoM of the cube will be 60µm to the right of initial position, as a result of conservation of system wide Com, the rest of the spacecraft A of mass 11-1=10kg will have seen a shift in position to the left of 1kg/10kg * 60µm = 6µm. Hull of A is now 506 µm from hull of B. This is one shot displacement, if the cube is to cool down to 20°C and contract to initial position, the initial hull to hull distance of 500µm is restored.

There is no way that the distance between A and B will go from 500µm to 506µm if the inertial forces associated to thermal expansion are said to be 0 exactly. Now I'm not saying that such forces should be taken into account to calculate the thermal dynamics. The deformation of the cube under the mechanical stress of overcoming its own inertia when going from static to expanding are negligible indeed. I'm saying that, however slow and whatever is the exact time evolution of F_cube_on_hull(t)  the first integral between initial static at 20°C and final static at 30°C will be 0, but the second integral will be 60e-6 kg m. A force F(t) always at 0 would tell no such conclusion. It doesn't depend on the specifics, whether such part's mass CoM shift is driven by a coil, by a servomotor, by a shape memory alloy spring, by a piston, by a thermal expansion...

In this very specific case, which is arguably quantitatively relevant for the experiment at hand, what is usually neglected can't be neglected. F(t) != 0. But I would agree that it makes no difference in the way to proceed with calculations : when assessing the dynamics of heat conduction and expansion, first proceed as usual (negligible inertia relative to expansion accelerations). Then from those position(t) curves, the tiny inertial reaction forces can be derived, if needed.

Edit: to be perfectly accurate, since the hull of mass M is recoiling leftward X absolute while the block of mass m is shifting rightward x absolute, if we know there will be a relative displacement rd=x+X, mx=MX gives m(rd-X)=MX  =>  (M+m)X=m rd  =>  X=m/(M+m) rd    so with rd=60µm  X=5.45µm : the distance between A and B will go from 500µm to 505.45µm.
Title: Re: EM Drive Developments - related to space flight applications - Thread 2
Post by: frobnicat on 03/01/2015 11:19 am
I wonder if the following effect has been quantitatively assessed :
if a thermal displacement (again) of a part of significant mass relative to the fixation point can occur, this will change the torque seen by the balance arm around the x axis. (See attached picture for naming conventions). While the flexure bearings make for a very stiff reaction around x and y axis (compared to the natural rotation around z) no stiffness is infinite. Likewise, the Faztek beam could twist a little around the x axis. Since the distance reported in the charts is measured (along the y axis) from a plate below the beam, any increased rotation clockwise would increase this distance.

Contrary to dynamical recoil effects that needs constant acceleration to mimic sustained force, it suffices the deformation is kept at constant position (relative to fixation point) to yield an apparent long lasting force reading. As for the orientation, in this hypothesis one would have to explain a displacement of some mass to the right (relative to fixation point) to get an upward reading (corresponding to a leftward force interpretation). Expansion of frustum's copper walls alone would shift some mass to the left relative to fixation point, so would run contrary to the observed directions. Expansion of a left wall (small end) glued dielectric would go in the right direction, but Paul March reported of a try (http://forum.nasaspaceflight.com/index.php?topic=36313.msg1335583#msg1335583) with the dielectric on the right wall (big end) and the apparent thrust haven't inverted or even reduced...

So, this is on shaky ground on orientations considerations alone, but I'd be curious to know the amount of rotation stiffness around the x axis, maybe that was discussed already ?

Side note : I'm trying to know the exact model of flexure bearings used. From this post (http://forum.nasaspaceflight.com/index.php?topic=29276.msg1253522#msg1253522) we know that <<The Riverhawk C-flex torsion bearing's spring constant is a nominal 0.007 in-Lb/deg.>> but that doesn't add up with the vertical readings in the charts (from calibration pulses, between 0.033µm and 0.1µm per µN on vertical scale, with still unexplained disparity from chart to chart).
Title: Re: EM Drive Developments - related to space flight applications - Thread 2
Post by: Notsosureofit on 03/01/2015 01:27 pm
Thanks guys !

So I have data for everything except the mode(s), and temp size for the flight cone.

I need to use X numbers of 21, 26, and 77, respectively, for the proto, demo and flight cones to get those numbers. (w/o dielectric that is)

What that means is TBD of course.  There may be another variable involved, in or out of favor of a real effect.

Realized this morning I'd been missing something after rereading Shawer's flight paper and the Magnetron "lock" time.

Kicking myself for not recognizing a "Q multiplier circuit" (I've used enough of them)  The long lock time (time constant) is the tip-off if this is what he's doing and the results are real.  The trade off in that case is spectral purity vs the frequency stability of the oscillator since you track the cavity.

Needs only a modest *10 for the proto and demo cases.  The flight system needs *100 which is not out of the question at all.

Again, this is pure speculation on my part w/o confirmation from Shawyer, but it puts the mode back in a reasonable range.

Title: Re: EM Drive Developments - related to space flight applications - Thread 2
Post by: zen-in on 03/01/2015 02:52 pm
I wonder if the following effect has been quantitatively assessed :
if a thermal displacement (again) of a part of significant mass relative to the fixation point can occur, this will change the torque seen by the balance arm around the x axis. (See attached picture for naming conventions). While the flexure bearings make for a very stiff reaction around x and y axis (compared to the natural rotation around z) no stiffness is infinite. Likewise, the Faztek beam could twist a little around the x axis. Since the distance reported in the charts is measured (along the y axis) from a plate below the beam, any increased rotation clockwise would increase this distance.

Contrary to dynamical recoil effects that needs constant acceleration to mimic sustained force, it suffices the deformation is kept at constant position (relative to fixation point) to yield an apparent long lasting force reading. As for the orientation, in this hypothesis one would have to explain a displacement of some mass to the right (relative to fixation point) to get an upward reading (corresponding to a leftward force interpretation). Expansion of frustum's copper walls alone would shift some mass to the left relative to fixation point, so would run contrary to the observed directions. Expansion of a left wall (small end) glued dielectric would go in the right direction, but Paul March reported of a try (http://forum.nasaspaceflight.com/index.php?topic=36313.msg1335583#msg1335583) with the dielectric on the right wall (big end) and the apparent thrust haven't inverted or even reduced...

So, this is on shaky ground on orientations considerations alone, but I'd be curious to know the amount of rotation stiffness around the x axis, maybe that was discussed already ?

Side note : I'm trying to know the exact model of flexure bearings used. From this post (http://forum.nasaspaceflight.com/index.php?topic=29276.msg1253522#msg1253522) we know that <<The Riverhawk C-flex torsion bearing's spring constant is a nominal 0.007 in-Lb/deg.>> but that doesn't add up with the vertical readings in the charts (from calibration pulses, between 0.033µm and 0.1µm per µN on vertical scale, with still unexplained disparity from chart to chart).

Nice illustrations and yes I think this effect could be happening.   Earlier I proposed this same effect to explain the negative slope in the baseline of the thrust waveform.   It appears to have a long time constant so may be from a thermally induced change in CoM.
Title: Re: EM Drive Developments - related to space flight applications - Thread 2
Post by: Rodal on 03/01/2015 03:10 pm
I wonder if the following effect has been quantitatively assessed :
if a thermal displacement (again) of a part of significant mass relative to the fixation point can occur, this will change the torque seen by the balance arm around the x axis. (See attached picture for naming conventions). While the flexure bearings make for a very stiff reaction around x and y axis (compared to the natural rotation around z) no stiffness is infinite. Likewise, the Faztek beam could twist a little around the x axis. Since the distance reported in the charts is measured (along the y axis) from a plate below the beam, any increased rotation clockwise would increase this distance.

Contrary to dynamical recoil effects that needs constant acceleration to mimic sustained force, it suffices the deformation is kept at constant position (relative to fixation point) to yield an apparent long lasting force reading. As for the orientation, in this hypothesis one would have to explain a displacement of some mass to the right (relative to fixation point) to get an upward reading (corresponding to a leftward force interpretation). Expansion of frustum's copper walls alone would shift some mass to the left relative to fixation point, so would run contrary to the observed directions. Expansion of a left wall (small end) glued dielectric would go in the right direction, but Paul March reported of a try (http://forum.nasaspaceflight.com/index.php?topic=36313.msg1335583#msg1335583) with the dielectric on the right wall (big end) and the apparent thrust haven't inverted or even reduced...

So, this is on shaky ground on orientations considerations alone, but I'd be curious to know the amount of rotation stiffness around the x axis, maybe that was discussed already ?

Side note : I'm trying to know the exact model of flexure bearings used. From this post (http://forum.nasaspaceflight.com/index.php?topic=29276.msg1253522#msg1253522) we know that <<The Riverhawk C-flex torsion bearing's spring constant is a nominal 0.007 in-Lb/deg.>> but that doesn't add up with the vertical readings in the charts (from calibration pulses, between 0.033µm and 0.1µm per µN on vertical scale, with still unexplained disparity from chart to chart).

My very first post on this forum was motivated by the rotation produced by the change in location of the center of mass:  http://forum.nasaspaceflight.com/index.php?topic=29276.msg1252386#msg1252386

This rotation results in nonlinear coupling of degrees of freedom.  I solved the nonlinear coupled differential equations using Mathematica.  The coupling is very small because the rotational stiffness for the motion you are considering is much stiffer than the torsional stiffness of Eagleworks torsional pendulum around the vertical axis perpendicular to the ground. (Great drawings  :), I wish I would have had those drawings to explain what I was discussing when I was posting about this).

Work out the equations, work out the numbers and see what numbers you arrive at.
Title: Re: EM Drive Developments - related to space flight applications - Thread 2
Post by: Rodal on 03/01/2015 03:11 pm
....   Earlier I proposed this same effect to explain the negative slope in the baseline of the thrust waveform.   It appears to have a long time constant so may be from a thermally induced change in CoM.

My very first post on this forum was motivated by the rotation produced by the change in location of the center of mass:  http://forum.nasaspaceflight.com/index.php?topic=29276.msg1252386#msg1252386

This rotation results in nonlinear coupling of degrees of freedom.  I solved the nonlinear coupled differential equations using Mathematica.  The coupling is very small because the rotational stiffness for the motion you are considering is much stiffer than the torsional stiffness of Eagleworks torsional pendulum around the vertical axis perpendicular to the ground.
Title: Re: EM Drive Developments - related to space flight applications - Thread 2
Post by: Star-Drive on 03/01/2015 03:57 pm
Thanks guys !

So I have data for everything except the mode(s), and temp size for the flight cone.

I need to use X numbers of 21, 26, and 77, respectively, for the proto, demo and flight cones to get those numbers. (w/o dielectric that is)

What that means is TBD of course.  There may be another variable involved, in or out of favor of a real effect.

Realized this morning I'd been missing something after rereading Shawer's flight paper and the Magnetron "lock" time.

Kicking myself for not recognizing a "Q multiplier circuit" (I've used enough of them)  The long lock time (time constant) is the tip-off if this is what he's doing and the results are real.  The trade off in that case is spectral purity vs the frequency stability of the oscillator since you track the cavity.

Needs only a modest *10 for the proto and demo cases.  The flight system needs *100 which is not out of the question at all.

Again, this is pure speculation on my part w/o confirmation from Shawyer, but it puts the mode back in a reasonable range.



Notsosureofit:

Could you describe to me how one goes about building a "Q-multiplier Circuit" for a 1.90 GHz RF amplifier circuit?  I've used the old Heathkit QF-1 Q-Multiplier for my old shortwave radio receiver back in high school, see: http://tubularelectronics.com/Heath_Manual_Collection/Heath_Manuals_O-RX/QF-1/QF-1.pdf ,  but I've never thought to use one to enhance the Q-Factor of a microwave frustum cavity before...

Best, Paul M.
Title: Re: EM Drive Developments - related to space flight applications - Thread 2
Post by: frobnicat on 03/01/2015 04:03 pm
I wonder if the following effect has been quantitatively assessed :
if a thermal displacement (again) of a part of significant mass relative to the fixation point can occur, this will change the torque seen by the balance arm around the x axis. (See attached picture for naming conventions). While the flexure bearings make for a very stiff reaction around x and y axis (compared to the natural rotation around z) no stiffness is infinite. Likewise, the Faztek beam could twist a little around the x axis. Since the distance reported in the charts is measured (along the y axis) from a plate below the beam, any increased rotation clockwise would increase this distance.

Contrary to dynamical recoil effects that needs constant acceleration to mimic sustained force, it suffices the deformation is kept at constant position (relative to fixation point) to yield an apparent long lasting force reading. As for the orientation, in this hypothesis one would have to explain a displacement of some mass to the right (relative to fixation point) to get an upward reading (corresponding to a leftward force interpretation). Expansion of frustum's copper walls alone would shift some mass to the left relative to fixation point, so would run contrary to the observed directions. Expansion of a left wall (small end) glued dielectric would go in the right direction, but Paul March reported of a try (http://forum.nasaspaceflight.com/index.php?topic=36313.msg1335583#msg1335583) with the dielectric on the right wall (big end) and the apparent thrust haven't inverted or even reduced...

So, this is on shaky ground on orientations considerations alone, but I'd be curious to know the amount of rotation stiffness around the x axis, maybe that was discussed already ?

Side note : I'm trying to know the exact model of flexure bearings used. From this post (http://forum.nasaspaceflight.com/index.php?topic=29276.msg1253522#msg1253522) we know that <<The Riverhawk C-flex torsion bearing's spring constant is a nominal 0.007 in-Lb/deg.>> but that doesn't add up with the vertical readings in the charts (from calibration pulses, between 0.033µm and 0.1µm per µN on vertical scale, with still unexplained disparity from chart to chart).

My very first post on this forum was motivated by the rotation produced by the change in location of the center of mass:  http://forum.nasaspaceflight.com/index.php?topic=29276.msg1252386#msg1252386

This rotation results in nonlinear coupling of degrees of freedom.  I solved the nonlinear coupled differential equations using Mathematica.  The coupling is very small because the rotational stiffness for the motion you are considering is much stiffer than the torsional stiffness of Eagleworks torsional pendulum around the vertical axis perpendicular to the ground.

Yes I recall that, but can't find the values you used, don't remember if you published or just PM to someone asking. At the moment my concern is not that much on coupling, but on absolute stiffness around the x axis (as seen from torque around x applied 10'' from the z axis). Do you know or have derived the exact flexure bearing model ? Is it a tandem of 2 C-Flex E-10 or B-20 at .0037 Lb-in/degree each as found there http://www.c-flex.com/companyproducts.pdf or a tandem of 2 RiveHawk like 5005-600 (.0035) or 5006-660 (.0037) or 5010-800 (.0036).


From this post :
 (http://forum.nasaspaceflight.com/index.php?topic=29276.msg1253522#msg1253522)
Quote from: Star-Drive
The Riverhawk C-flex torsion bearing's spring constant is a nominal 0.007 in-Lb/deg., but that varies with the mass load mounted on the torque pendulum arm and selected balance point of the test article mass and its counterbalance mass on the other end of the pendulum arm relative to the torque pendulum’s center of rotation.  Each bearing block is rated for ~25.0 Lb of vertical mass load, so we nominally restrict ourselves to a 25 pound total load limit on the torque pendulum arm to give ourselves a 100% support mass margin.

Have we anything more specific ?
For vertical mass load (ie. axial load) E-10 is rated 36.48 Lb, B-20 is 19.6 Lb (page 11 C-Flew spec sheet). From axial load indication could be A-30 or C-20, A-30 is too stiff...
Nearest fit (if 0.007 in-Lb/deg is for each) would then be C-20 : 0.0060 in-Lb/deg and 27.90 Lb axial

For RiverHawk I don't find axial load ratings, neither at here (http://flexpivots.com/wp-content/uploads/2014/07/Riverhawk-FlexuralPivotCatalog.pdf) nor there (http://flexpivots.com/wp-content/uploads/2014/07/Riverhawk-FlexuralPivotEngineeringData.pdf).

Anyhow, even if we have 2 times .007 in-Lb/deg. that's 9.06e-2 Nm/rad (please check as I'm not used to imperial units) and a 29.1µN (calibration pulse) at 10'' from z axis is 7.39e-6 Nm, so we should have 8.16e-5 rad, that is 20.7 µm deviation (at 10'' from z axis, LDS is a bit further so it should be even a bit more).

Yet we see on the charts that the calibration pulses give between 1 to 2.5 µm deviation on the vertical scale. We don't know why this should be varying from chart to chart, and there is one order of magnitude difference with the given stiffness. So maybe the indicated vertical scale is irrelevant... but then it becomes difficult to model the system. Looks like the flexure bearing are stiffer than 0.007 in-Lb/deg. (around z).
Title: Re: EM Drive Developments - related to space flight applications - Thread 2
Post by: Rodal on 03/01/2015 04:23 pm
I wonder if the following effect has been quantitatively assessed :
if a thermal displacement (again) of a part of significant mass relative to the fixation point can occur, this will change the torque seen by the balance arm around the x axis. (See attached picture for naming conventions). While the flexure bearings make for a very stiff reaction around x and y axis (compared to the natural rotation around z) no stiffness is infinite. Likewise, the Faztek beam could twist a little around the x axis. Since the distance reported in the charts is measured (along the y axis) from a plate below the beam, any increased rotation clockwise would increase this distance.

Contrary to dynamical recoil effects that needs constant acceleration to mimic sustained force, it suffices the deformation is kept at constant position (relative to fixation point) to yield an apparent long lasting force reading. As for the orientation, in this hypothesis one would have to explain a displacement of some mass to the right (relative to fixation point) to get an upward reading (corresponding to a leftward force interpretation). Expansion of frustum's copper walls alone would shift some mass to the left relative to fixation point, so would run contrary to the observed directions. Expansion of a left wall (small end) glued dielectric would go in the right direction, but Paul March reported of a try (http://forum.nasaspaceflight.com/index.php?topic=36313.msg1335583#msg1335583) with the dielectric on the right wall (big end) and the apparent thrust haven't inverted or even reduced...

So, this is on shaky ground on orientations considerations alone, but I'd be curious to know the amount of rotation stiffness around the x axis, maybe that was discussed already ?

Side note : I'm trying to know the exact model of flexure bearings used. From this post (http://forum.nasaspaceflight.com/index.php?topic=29276.msg1253522#msg1253522) we know that <<The Riverhawk C-flex torsion bearing's spring constant is a nominal 0.007 in-Lb/deg.>> but that doesn't add up with the vertical readings in the charts (from calibration pulses, between 0.033µm and 0.1µm per µN on vertical scale, with still unexplained disparity from chart to chart).

My very first post on this forum was motivated by the rotation produced by the change in location of the center of mass:  http://forum.nasaspaceflight.com/index.php?topic=29276.msg1252386#msg1252386

This rotation results in nonlinear coupling of degrees of freedom.  I solved the nonlinear coupled differential equations using Mathematica.  The coupling is very small because the rotational stiffness for the motion you are considering is much stiffer than the torsional stiffness of Eagleworks torsional pendulum around the vertical axis perpendicular to the ground.

Yes I recall that, but can't find the values you used, don't remember if you published or just PM to someone asking. At the moment my concern is not that much on coupling, but on absolute stiffness around the x axis (as seen from torque around x applied 10'' from the z axis). Do you know or have derived the exact flexure bearing model ? Is it a tandem of 2 C-Flex E-10 or B-20 at .0037 Lb-in/degree each as found there http://www.c-flex.com/companyproducts.pdf or a tandem of 2 RiveHawk like 5005-600 (.0035) or 5006-660 (.0037) or 5010-800 (.0036).


From this post :
 (http://forum.nasaspaceflight.com/index.php?topic=29276.msg1253522#msg1253522)
Quote from: Star-Drive
The Riverhawk C-flex torsion bearing's spring constant is a nominal 0.007 in-Lb/deg., but that varies with the mass load mounted on the torque pendulum arm and selected balance point of the test article mass and its counterbalance mass on the other end of the pendulum arm relative to the torque pendulum’s center of rotation.  Each bearing block is rated for ~25.0 Lb of vertical mass load, so we nominally restrict ourselves to a 25 pound total load limit on the torque pendulum arm to give ourselves a 100% support mass margin.

Have we anything more specific ?
For vertical mass load (ie. axial load) E-10 is rated 36.48 Lb, B-20 is 19.6 Lb (page 11 C-Flew spec sheet). From axial load indication could be A-30 or C-20, A-30 is too stiff...
Nearest fit (if 0.007 in-Lb/deg is for each) would then be C-20 : 0.0060 in-Lb/deg and 27.90 Lb axial

For RiverHawk I don't find axial load ratings, neither at here (http://flexpivots.com/wp-content/uploads/2014/07/Riverhawk-FlexuralPivotCatalog.pdf) nor there (http://flexpivots.com/wp-content/uploads/2014/07/Riverhawk-FlexuralPivotEngineeringData.pdf).

Anyhow, even if we have 2 times .007 in-Lb/deg. that's 9.06e-2 Nm/rad (please check as I'm not used to imperial units) and a 29.1µN (calibration pulse) at 10'' from z axis is 7.39e-6 Nm, so we should have 8.16e-5 rad, that is 20.7 µm deviation (at 10'' from z axis, LDS is a bit further so it should be even a bit more).

Yet we see on the charts that the calibration pulses give between 1 to 2.5 µm deviation on the vertical scale. We don't know why this should be varying from chart to chart, and there is one order of magnitude difference with the given stiffness. So maybe the indicated vertical scale is irrelevant... but then it becomes difficult to model the system. Looks like the flexure bearing are stiffer than 0.007 in-Lb/deg. (around z).
Based on my interpretation of early discussions with Paul March in Thread 1, I assumed that the compliance for rotation around your x axis was due to the aluminum beams and that the RiverHawk bearings provided an effective clamp condition for rotation around the x axis. 

My recollection (I have not gone back and checked this) is that March said that there were two RiverHawk bearings providing a clamped condition (for rotations around your x axis and for rotations around your y axis).

(Any fault with this assumption rests squarely on my own shoulders as the initial exchanges with Paul March were very unfortunately brought to an abrupt end due to disruptive posters in Thread 1 who wanted to argue with Paul March about General Relativity theory, Mach Effects, Quantum Vacuum, and what Chris described as "stupid posts" instead of obtaining engineering data and instead of performing any calculations.  Paul March withdrew shortly thereafter for a long period of time, so I relied on my own interpretations of the initial few posts by March and on my own assumptions).

(http://forum.nasaspaceflight.com/index.php?action=dlattach;topic=36313.0;attach=809641;image)
Title: Re: EM Drive Developments - related to space flight applications - Thread 2
Post by: Notsosureofit on 03/01/2015 05:08 pm
Thanks guys !

So I have data for everything except the mode(s), and temp size for the flight cone.

I need to use X numbers of 21, 26, and 77, respectively, for the proto, demo and flight cones to get those numbers. (w/o dielectric that is)

What that means is TBD of course.  There may be another variable involved, in or out of favor of a real effect.

Realized this morning I'd been missing something after rereading Shawer's flight paper and the Magnetron "lock" time.

Kicking myself for not recognizing a "Q multiplier circuit" (I've used enough of them)  The long lock time (time constant) is the tip-off if this is what he's doing and the results are real.  The trade off in that case is spectral purity vs the frequency stability of the oscillator since you track the cavity.

Needs only a modest *10 for the proto and demo cases.  The flight system needs *100 which is not out of the question at all.

Again, this is pure speculation on my part w/o confirmation from Shawyer, but it puts the mode back in a reasonable range.



Notsosureofit:

Could you describe to me how one goes about building a "Q-multiplier Circuit" for a 1.90 GHz RF amplifier circuit?  I've used the old Heathkit QF-1 Q-Multiplier for my old shortwave radio receiver back in high school, see: http://tubularelectronics.com/Heath_Manual_Collection/Heath_Manuals_O-RX/QF-1/QF-1.pdf ,  but I've never thought to use one to enhance the Q-Factor of a microwave frustum cavity before...

Best, Paul M.

"Q-multiplier" is the Heath term for adding feedback, short of oscillation, to an IF amplifier to narrow the bandwidth
and enhance its "Q" as a filter.  (had one on an HRO receiver in the 50's.  HRO long gone but the Q-Multiplier is still in the pile somewhere)

In the case of greater feed back you get an oscillator. (had a 220MHz re-entrant cavity oscillator at that time)

The right feedback loop will improve the phase coherence of the oscillation and the "circuit Q".  You pay for this with an increased time constant and are limited by the (thermal ?) drift rate of the cavity.  The "flight" cavity might be heavily built for that reason ?

Hopefully there is a radar guy (like Shawyer) on tap that could give a better explanation. (I'm pulling this out of memories of my misspent youth...)

I was trying to remember something about radar systems (Russian ?) that had a dielectric resonator suspended in a microwave cavity.............

Title: Re: EM Drive Developments - related to space flight applications - Thread 2
Post by: DIYFAN on 03/01/2015 06:26 pm
Dr. Rodal & Crew:

The Eagleworks team has already build a 6061 aluminum frustum cavity with 1/4" thick walls and O-ring end caps meant to hold a 1 Bar pressure differential with internal nickel/copper/silver/gold plating system on all interior surfaces with plating thickness of 10-to-15 microns for the first three layers and 0.5 microns for exposed to the RF gold layer.  Sadly the gold layer was just as thick as the rest of the plated layers and textured as well, so as far as the applied ~2.0 GHz RF was concerned it was only interacting with the rough gold layer.  This had the effect of cutting the resonant Q-factor for this aluminum frustum by almost a factor of three over our copper frustum for the resonances of interest. 

At the same time we also tried using a smaller volume, higher-K (e-r=~40) ceramic dielectric resonator discs in the Al cavity mounted at its small OD end, while driving it at its TE011 mode if memory serves.  Bottom line was that this configuration was a total bust in regards to thrust production in our torque pendulum system running at this resonant mode.  This aluminum frustum design also turned out to be ~4X times the mass of the thin walled copper cavity even while using lower density aluminum for its construction.  This exercise was a tribute to the fact that one should never ASSUME that you know what you are doing until proven otherwise!  And oh yes, and only try one variation in the design at a time or one will get lost, fast!

Best, Paul M.

Dr. March,

I'm a big fan of what you and Eagleworks are doing, particularly in the face of tremendous skepticism.  It takes some truly courageous scientists to do what you are doing.  We all hope for an eventual proof or nullification of this effect.

There has been some discussion here about Dr. Aquino's theory of how to amplify the effect (simply search this thread for "aquino").  I hope you don't mind me asking if you or others at Eagleworks have tested this theory.  Specifically, have you tried using a metglas coating or sheet for the big flat end of the frustum?  Alternatively, have you tried using an iron coating or sheet for the big flat end of the frustum?  (The iron suggestion was proposed by a forum member as an alternative to metglas, albeit an inferior alternative.)  Are either of these feasible to attempt in your experimental setup?  Do you think it is worth the time to test Dr. Aquino's theory or do you have other higher priority tasks at hand?  Dr. Rodal used Dr. Aquino's conjecture to predict a 20-50 times amplification of the effect based on an extrapolation of the permeability characteristics of metglas, specifically metglas 2714A.  Metglas 2714A might be hard to procure, but iron certainly would not be.  It might also be quite easy to procure a metglas brazing foil, which could be fitted to the interior of the large flat plate.
Title: Re: EM Drive Developments - related to space flight applications - Thread 2
Post by: Rodal on 03/01/2015 07:07 pm
See this post:  http://forum.nasaspaceflight.com/index.php?topic=36313.msg1328595#msg1328595
regarding Aquino's conjecture, links to his paper and to published material constant values for Metglas.

Aquino, when advocating the use of Metglas 2714A failed to take into account that its ultra-high magnetic permeability which he quotes in the paper (apparently for his calculations) as μ=1,000,000 occurs at very low frequencies and that its magnetic permeability decreases significantly at higher frequencies.   I extrapolated published data to come up with a more reasonable estimate (if Aquino's conjecture would be correct).
Title: Re: EM Drive Developments - related to space flight applications - Thread 2
Post by: Star-Drive on 03/01/2015 08:35 pm
DIYFAN:

Once the test series we are working on is finished, I will suggest to Dr. White that we try the use of the more readily available NiFeCo mu-metal from McMaster-Carr (See: http://www.mcmaster.com/#mu-metal-foil/=w4hfa3 ) for such a test.  However I think we will have to copper plate the side of the mu-metal facing the interior of the cavity with about 10 microns of copper or silver to keep this large OD end-cap from greatly reducing the Q-Factor of the copper frustum.  Mu-metal resistivity is much higher than copper...

Best,  Paul M.
Title: Re: EM Drive Developments - related to space flight applications - Thread 2
Post by: frobnicat on 03/01/2015 09:12 pm
Dr. March,

I'm trying to model various aspects of the whole system to put upper bounds on thermal effects, and possibly also reconstruct the thrust(t) original signal from the distance(t) given in the charts. It would be a nice boost to this (amateur level) effort if you could confirm either :
- That the flexure bearings have a stiffness of 0.007 in-Lb/deg ? Each ? Both together ? Do you know the exact model reference ?
- That the vertical scale in the charts (indicated in µm, around 500) are relevant or not relevant.

I ask this question because I find a contradiction between the stiffness around the vertical axis and the recorded deviation from the 30µN calibration pulses (at .007 in-Lb/deg the deviation of the linear displacement sensor would be above 40µm, at .014 in-Lb/deg still above 20µm). The readings amount for between 1 to 2.5 µm for the 30µN calibration pulses. So I'm stuck.

While I'm at it : is the plane in which the arm rotates kept as horizontal as possible (ie the axis of rotation as vertical as possible) or is there a small slope voluntarily introduced leading to some pendulum effect against g (for stabilisation or tuning purpose) ? That could explain the varying deviation (in µm) for the same calibration pulses thrusts. Also wondered if this is what was implied in this post (http://forum.nasaspaceflight.com/index.php?topic=36313.msg1331580#msg1331580) :
Quote from: Star-Drive
...
These thermally induced actions to the left requires the torque pendulum's arm to move to the right to maintain the balance of the torque pendulum's arm in the lab's 1.0 gee gravity field, since we also use the Earth's g-field to help null the pendulum's movements.
...

Thanks
Title: Re: EM Drive Developments - related to space flight applications - Thread 2
Post by: Rodal on 03/01/2015 09:44 pm
DIYFAN:

Once the test series we are working on is finished, I will suggest to Dr. White that we try the use of the more readily available NiFeCo mu-metal from McMaster-Carr (See: http://www.mcmaster.com/#mu-metal-foil/=w4hfa3 ) for such a test.  However I think we will have to copper plate the side of the mu-metal facing the interior of the cavity with about 10 microns of copper or silver to keep this large OD end-cap from greatly reducing the Q-Factor of the copper frustum.  Mu-metal resistivity is much higher than copper...

Best,  Paul M.
I couldn't find magnetic permeability values for mu metal close to the GHz range, except the frequent warning (also included in the Wikipedia article) <<The high permeability makes mu-metal useful for shielding against static or low-frequency magnetic fields>> (bold added for emphasis).

I wonder what is the relative magnetic permeability of mu metal in the GHz range, and how effective it will be for the purposes described by Aquino.
Title: Re: EM Drive Developments - related to space flight applications - Thread 2
Post by: Flyby on 03/01/2015 10:31 pm
Small update on the image correcting..

I decided to take a different approach as I clearly need to have a more methodical approach to be convincing. :-\
Tbh, as mentioned, something about the tilt correction didn't match up, so I took a different angle for handeling the problem...

What remains are the camera distortions:  the barrel distortion and the tilt-distortion.

BARREL DISTORTION
The barrel distortion is camera lens dependent , and as we do not know the type of camera and lens used, a general correction on the barrel distortion has been performed, using pincushion lens distortion filter for photoshop.
The pincushion distortion is the opposite of the barrel distortion and is able to cancel out the original distortion. Notice on how the mortar line on the back wall is curved on the original picture and straightened on the corrected one...
A zoom lens will tend to cause pincushion distortion, a wide angle lens tends to cause barrel distortion...

The effect is that the height of the corrected cone will be a tiny bit less then on the non corrected one

TILT DISTORTION
To understand - and illustrate - what's happening with the tilt distortion, I've recreated an approximation of the scene in my 3Dsoftware.
I exaggerated the effect a bit to make it clearly visible what's happening :
Due to the tilting of the camera, the normally vertical lines no longer converge at an infinite distance (= apparent perpendicular), but at a point much closer to the horizon.
What makes it very interesting is that the vertical (red) lines on the back wall and the vertical (yellow) lines on the cylinder converge to the same point.
Also interesting to know is that a wide angle lens (fe 28mm) causes more distortion then a normal lens (50mm)

Similar to the test file, I've used the vertical brick lines of the wall to find the converging point in the photograph...



When you apply that knowledge to the barrel corrected image and you imagine the cone to be a cylinder, you can find all measures in a correct relation with each other...

Sadly, it's getting very, very late again... so I didn't have time yet to put it all in proportional numbers yet.
More tomorrow...hopefully...

But I think it already demonstrates that you should not underestimate/neglect the amount of distortion caused by the tilting of the camera...




BTW.. curiosity killed the cat...An interview with Paul March...
http://beforeitsnews.com/science-and-technology/2014/10/paul-march-reactionless-drive-interview-2726212.html

 
Title: Re: EM Drive Developments - related to space flight applications - Thread 2
Post by: Rodal on 03/01/2015 11:22 pm
Here are my calculations, from the exact solution, for the mode shape presently being tested by NASA Eagleworks, and for which they obtained thermal IR camera imaging.  I show the resultant of both components of the magnetic field, in the polar (theta) (perpendicular to the cone walls) and azimuthal (phi) (circumferential)  directions of the truncated frustum. The resultant is the square root of the sum of the squares of each component.  These results are for a truncated cone without dielectric.  The frequency for the mode TM221 mode shape (without the dielectric) is 2.00709 GHz.

I show density plots (also called intensity plots) with two different color schemes (plum colors gradient and a blue-white-yellow-orange-red gradient) for comparison. The relative scale numbers associated with the image are correct but their magnitude is not (I have to multiply the numbers by a scale factor based on integrating the electromagnetic field to equal the applied power).  I also show the NASA calculations done with Finite Element (COMSOL) analysis and the thermal IR camera measurements for comparison.

On this cross-section, TM221, TM222 and TM223, etc, look practically identical, except for different intensity magnitude for the higher frequency modes with TM22p where p>1.

(http://forum.nasaspaceflight.com/index.php?action=dlattach;topic=36313.0;attach=635194;image)

(http://forum.nasaspaceflight.com/index.php?action=dlattach;topic=36313.0;attach=635195;image)
Title: Re: EM Drive Developments - related to space flight applications - Thread 2
Post by: Rodal on 03/01/2015 11:41 pm
You may notice that I labeled the mode TM22 instead of TM21.  This is intentional.  When using a numerical method like the Finite Element Method (used by NASA Eagleworks for labeling the mode as TM21), the analyst is not given the mode shape number by the (COMSOL) solver.  The analyst has to resort to ascertaining what mode shape it is by inspecting the figure (which is not trivial to do).  In this case, this is the first TM2n appearing, and therefore it is understandable that the analyst may interpret "n" as n=1.  However, one of the advantages of an exact solution (which is what I am using) is that the exact values of m, n, p are given by the solution.  One does not have to resort to visual inspection to ascertain them.  In this case, n=2 (instead of n=1) because the solution for the eigenvalue problem for the Legendre function shows that there is a first root (corresponding to n=1) occurring at a value of the integer 1 (exactly) in the abscissa (images shown below for first root at the abscissa value of 1, and below it is the image for the next few roots, occurring at irrational numbers).  This eigenvalue at n=1 corresponds to a value of zero of the Legendre function, therefore the TM21 mode is cut-off.  That's why TM21 does not appear and that's why TM22 is the mode with the lowest n value for TM2n.
Title: Re: EM Drive Developments - related to space flight applications - Thread 2
Post by: Rodal on 03/01/2015 11:55 pm
The density (or intensity) plot shown above for mode TM221 is, as explained above, the magnetic field resultant of the polar and azimuthal compoments magnetic field components.

Here I show the polar (theta) (perpendicular direction to the cone walls) and the azimuthal (phi) (circumferential) components of the magnetic field at the Big Base, separately, as contour plots.

You may notice that n=2 based on the fact that the polar (theta) component has two half-waves patterns fitting in the diameter.  Equivalently, there are 2 full-wave patterns of the azimuthal component fitting in the diameter.  These are alternative ways to determine the proper value of n.

But after looking at a lot of results from the exact solution for different modes, there is no doubt that the best way is to ascertain the proper values of the quantum numbers m, n and p from the eigenvalues, since there is no human judgement involved in doing so.
Title: Re: EM Drive Developments - related to space flight applications - Thread 2
Post by: Rodal on 03/02/2015 12:11 am
This is the Electric Field in the radial direction (plane at azimuthal angle phi=0).  We see that the electric field in the radial direction is concentrated at the Big End.  Notice that the ends are spherical (instead of flat) because the exact solution uses spherical Bessel functions and assumes wave propagation occurs as spherical waves .

No wonder that the screws are being damaged by the electric field !
Title: Re: EM Drive Developments - related to space flight applications - Thread 2
Post by: Star-Drive on 03/02/2015 12:21 am
Dr. March,

I'm trying to model various aspects of the whole system to put upper bounds on thermal effects, and possibly also reconstruct the thrust(t) original signal from the distance(t) given in the charts. It would be a nice boost to this (amateur level) effort if you could confirm either :
- That the flexure bearings have a stiffness of 0.007 in-Lb/deg ? Each ? Both together ? Do you know the exact model reference ?
- That the vertical scale in the charts (indicated in µm, around 500) are relevant or not relevant.

I ask this question because I find a contradiction between the stiffness around the vertical axis and the recorded deviation from the 30µN calibration pulses (at .007 in-Lb/deg the deviation of the linear displacement sensor would be above 40µm, at .014 in-Lb/deg still above 20µm). The readings amount for between 1 to 2.5 µm for the 30µN calibration pulses. So I'm stuck.

While I'm at it : is the plane in which the arm rotates kept as horizontal as possible (ie the axis of rotation as vertical as possible) or is there a small slope voluntarily introduced leading to some pendulum effect against g (for stabilisation or tuning purpose) ? That could explain the varying deviation (in µm) for the same calibration pulses thrusts. Also wondered if this is what was implied in this post (http://forum.nasaspaceflight.com/index.php?topic=36313.msg1331580#msg1331580) :
Quote from: Star-Drive
...
These thermally induced actions to the left requires the torque pendulum's arm to move to the right to maintain the balance of the torque pendulum's arm in the lab's 1.0 gee gravity field, since we also use the Earth's g-field to help null the pendulum's movements.
...

Thanks


Frobnicat:

To answer your question:

" - That the flexure bearings have a stiffness of 0.007 in-Lb/deg ? Each ? Both together ? Do you know the exact model reference?"

The two torsion bearings used in or torque pendulum are supposed to have a stiffness of 0.007 in-Lb/deg, +/-10% and is made by the Riverhawk Co. in New York USA.  As to their model number find the data sheet for same attached. 

"- That the vertical scale in the charts (indicated in µm, around 500) are relevant or not relevant."

The Philtec D63 fiber-optic displacement sensor measures distance from its target mirror in microns, so the numbers on the left hand side of the force plots measure the distance from the end of the fiber-optic laser head to its mirror target mounted on the torque pendulum arm.  The data sheet for same is attached.

"While I'm at it : is the plane in which the arm rotates kept as horizontal as possible (ie the axis of rotation as vertical as possible) or is there a small slope voluntarily introduced leading to some pendulum effect against g (for stabilization or tuning purpose)?"

The design of our Torque pendulum follows what JPL and Busek Co did at their respective facility, see attached report from Busek.  We found that if we tried to keep the arm completely horizontal though that the pendulum's neutral point would wonder erratically and make alignments near impossible.  So yes I balance the pendulum arm so there is always a slight tilt in it, however this tilt angle magnitude is not controlled as well as it probably should.

Best, Paul M.
Title: Re: EM Drive Developments - related to space flight applications - Thread 2
Post by: aero on 03/02/2015 12:40 am
@Mulletron

I made some cylindrical cavity runs. Meep is still saying what it has been saying to me for the past month. Attached find a graphic of the cavity. The dimensions are given on the F/P charts. Maximum thrust/power, in meep micro-Newtons/Watt are:

Magnetic source - 4,841
Electric source  - 11,952

Note that I used a large gap of 0.008 times height in order to keep the run times down. ( About half-hour per data point as is.) Anyway, I didn't want the numbers to be excessively astounding as they might be at reasonable small gap sizes.
Title: Re: EM Drive Developments - related to space flight applications - Thread 2
Post by: Rodal on 03/02/2015 01:00 am
@Mulletron

I made some cylindrical cavity runs.....
Are you modeling the geometrical shape now with a 3-D (three-dimensional) MEEP mesh or is your present MEEP model a flat plane with a rectangular boundary (2-D two-dimensional model) ?

If it is a 3-D (three-dimensional) mesh, can you also show the solution plots for the circular cross-section of the cylindrical cavity ?
Title: Re: EM Drive Developments - related to space flight applications - Thread 2
Post by: aero on 03/02/2015 01:14 am
No, its still a 2D model. The wife hasn't bought me a new computer yet. This model is smaller than the Copper Kettle thruster so I could likely make one good 3D run in a day's time. Like to know what you'd like to see, re-drive frequency, dielectric constant and so forth, before I do that. And what Mulletron has to say.

Still having trouble calculating the resonant frequency, too. I've picked the frequency off of the F/P curves and run the field patterns from that. It looks to be close to resonance but the frequencies I used for the field patterns are clearly not in resonance. I save a frame every 1/6 th of the period of the drive frequency (Using vacuum light speed for the Period calculation) so I can tell almost when a cycle completes. But the pattern is never quite the same six frames later.

Oh, I used my old stand-by dielectric constant of 1.76 in these runs, for what its worth.
Title: Re: EM Drive Developments - related to space flight applications - Thread 2
Post by: Notsosureofit on 03/02/2015 02:15 am
Thanks guys !

So I have data for everything except the mode(s), and temp size for the flight cone.

I need to use X numbers of 21, 26, and 77, respectively, for the proto, demo and flight cones to get those numbers. (w/o dielectric that is)

What that means is TBD of course.  There may be another variable involved, in or out of favor of a real effect.

Realized this morning I'd been missing something after rereading Shawer's flight paper and the Magnetron "lock" time.

Kicking myself for not recognizing a "Q multiplier circuit" (I've used enough of them)  The long lock time (time constant) is the tip-off if this is what he's doing and the results are real.  The trade off in that case is spectral purity vs the frequency stability of the oscillator since you track the cavity.

Needs only a modest *10 for the proto and demo cases.  The flight system needs *100 which is not out of the question at all.

Again, this is pure speculation on my part w/o confirmation from Shawyer, but it puts the mode back in a reasonable range.



Notsosureofit:

Could you describe to me how one goes about building a "Q-multiplier Circuit" for a 1.90 GHz RF amplifier circuit?  I've used the old Heathkit QF-1 Q-Multiplier for my old shortwave radio receiver back in high school, see: http://tubularelectronics.com/Heath_Manual_Collection/Heath_Manuals_O-RX/QF-1/QF-1.pdf ,  but I've never thought to use one to enhance the Q-Factor of a microwave frustum cavity before...

Best, Paul M.

"Q-multiplier" is the Heath term for adding feedback, short of oscillation, to an IF amplifier to narrow the bandwidth
and enhance its "Q" as a filter.  (had one on an HRO receiver in the 50's.  HRO long gone but the Q-Multiplier is still in the pile somewhere)

In the case of greater feed back you get an oscillator. (had a 220MHz re-entrant cavity oscillator at that time)

The right feedback loop will improve the phase coherence of the oscillation and the "circuit Q".  You pay for this with an increased time constant and are limited by the (thermal ?) drift rate of the cavity.  The "flight" cavity might be heavily built for that reason ?

Hopefully there is a radar guy (like Shawyer) on tap that could give a better explanation. (I'm pulling this out of memories of my misspent youth...)

I was trying to remember something about radar systems (Russian ?) that had a dielectric resonator suspended in a microwave cavity.............

The "Q multiplier" effect comes from the calculation of the loaded resonator Q, as f/2 times the loop gain phase slope.

"Loaded-Q represents the width of the resonance curve, or phase slope, including the
effects of external components. In this case the Q is determined mostly by the external
components."

In typical references that I found, ie.low noise frequency sources, the loaded Q is between 20% to 50% of the unloaded Q.

So for the "Shawyer" type application, to get a maximized loaded Q in a cavity oscillator, one should maximize the loop gain phase slope.  (use the highest gain-bandwidth amp you can find ?)

One question that comes to mind at the moment is;  What is the loaded Q of the present (non oscillator) experiments since that is presumably the relevant parameter.
Title: Re: EM Drive Developments - related to space flight applications - Thread 2
Post by: Rodal on 03/02/2015 02:49 am
DIYFAN:

Once the test series we are working on is finished, I will suggest to Dr. White that we try the use of the more readily available NiFeCo mu-metal from McMaster-Carr (See: http://www.mcmaster.com/#mu-metal-foil/=w4hfa3 ) for such a test.  However I think we will have to copper plate the side of the mu-metal facing the interior of the cavity with about 10 microns of copper or silver to keep this large OD end-cap from greatly reducing the Q-Factor of the copper frustum.  Mu-metal resistivity is much higher than copper...

Best,  Paul M.
I couldn't find magnetic permeability values for mu metal close to the GHz range, except the frequent warning (also included in the Wikipedia article) <<The high permeability makes mu-metal useful for shielding against static or low-frequency magnetic fields>> (bold added for emphasis).

I wonder what is the relative magnetic permeability of mu metal in the GHz range, and how effective it will be for the purposes described by Aquino.

Here is an example of a Korean high permeability material (not mu metal) with an initial relative permeability of 15000.  The report shows that the relative permeability drops precipitously at frequencies over 100 KHz and goes to zero at frequencies above 1 MHz


http://www.samwha.co.kr/electronics/product/material/Material%20characteristics_SM-150.pdf
Title: Re: EM Drive Developments - related to space flight applications - Thread 2
Post by: Star-Drive on 03/02/2015 03:46 am
Dr. Rodal:

Duly noted on mu-metal's drastic falloff in permeability with frequency especially above 1.0 MHz.  Looks Like I will have to continue the search if I perform this Aquino experiment...

Best, Paul M.
Title: Re: EM Drive Developments - related to space flight applications - Thread 2
Post by: aero on 03/02/2015 06:03 am
@Rodal, @notsosureofit, and everyone else.
I've been wondering how to treat the dielectric to compute the resonant frequency of the cylindrical cavity containing a dielectric. After some Google time I find that the velocity factor, VF, of a material is given by the inverse square root of relative permittivity. That's the dielectric constant! And I found that VF for a solid polyethylene core transmission cable is 0.66. That's right for the dielectric constant = 2.3.

So with this, the velocity of the RF waves in the dielectric is V = c *VF, or V = 0.66 c (c - speed of light). Isn't that the same as saying the equivalent vacuum distance within the dielectric, D = 1.515 d, where d is the geometry of the dielectric disk? So a cylinder of 1 unit high and 1 unit diameter containing a dielectric disk of 1/2 unit high and 1 unit diameter would respond to RF as though it were (1/2 +1.515/2) units high and 1.515 units diameter at the end containing the dielectric and 1 unit in diameter at the empty end. So we now have a frustum 1.515 x 1 units diameters x 1.251 high. Is that right? If so, we can use Dr. Rodal's formula to calculate the resonant frequency.

Maybe Dr. Rodal will tell me what the resonant frequency should be for the cylindrical cavity with dielectric that I posted data for earlier today?
Title: Re: EM Drive Developments - related to space flight applications - Thread 2
Post by: Rodal on 03/02/2015 12:17 pm


Maybe Dr. Rodal will tell me what the resonant frequency should be for the cylindrical cavity with dielectric that I posted data for earlier today?
Please fill-in the following data (question marks ? below) for the cylindrical cavity with a dielectric section having the same diameter as the cavity's ID and located at one end of the cavity:

GEOMETRY

Inner Diameter of cylindrical cavity =   ? meters (cavity has a constant, same diameter throughout)
Total Inner Length of cavity            =   ? meters (Length including dielectric length)
Length of dielectric section              =   ? meters

CONSTITUTIVE PROPERTIES

Relative electric permittivity of dielectric =      ?     (dimensionless)  (for HD PE it is reported as 2.3)
Relative magnetic permeability of dielectric =      ?     (dimensionless)  (1 ?)
Relative electric permittivity of empty section =   ?   (dimensionless)  (air or vacuum ?)
Relative magnetic permeability of empty section =   ?   (dimensionless)  (air or vacuum ?)

FREQUENCY of interest

There are an infinite number of resonant frequencies for a resonant cylindrical cavity. 
What resonant frequency are you referring to?     ?  (The lowest natural frequency? )
Title: Re: EM Drive Developments - related to space flight applications - Thread 2
Post by: Notsosureofit on 03/02/2015 02:35 pm
@ Star-Drive


Realized this morning I'd been missing something after rereading Shawer's flight paper and the Magnetron "lock" time.

Kicking myself for not recognizing a "Q multiplier circuit" (I've used enough of them)  The long lock time (time constant) is the tip-off if this is what he's doing and the results are real.  The trade off in that case is spectral purity vs the frequency stability of the oscillator since you track the cavity.

Needs only a modest *10 for the proto and demo cases.  The flight system needs *100 which is not out of the question at all.

Again, this is pure speculation on my part w/o confirmation from Shawyer, but it puts the mode back in a reasonable range.


Notsosureofit:

Could you describe to me how one goes about building a "Q-multiplier Circuit" for a 1.90 GHz RF amplifier circuit?  I've used the old Heathkit QF-1 Q-Multiplier for my old shortwave radio receiver back in high school, see: http://tubularelectronics.com/Heath_Manual_Collection/Heath_Manuals_O-RX/QF-1/QF-1.pdf ,  but I've never thought to use one to enhance the Q-Factor of a microwave frustum cavity before...

Best, Paul M.

"Q-multiplier" is the Heath term for adding feedback, short of oscillation, to an IF amplifier to narrow the bandwidth
and enhance its "Q" as a filter. In the case of greater feed back you get an oscillator. The right feedback loop will improve the phase coherence of the oscillation and the "circuit Q".  You pay for this with an increased time constant and are limited by the (thermal ?) drift rate of the cavity.  The "flight" cavity might be heavily built for that reason ?

The "Q multiplier" effect comes from the calculation of the loaded resonator Q, as f/2 times the loop gain phase slope.

"Loaded-Q represents the width of the resonance curve, or phase slope, including the effects of external components. In this case the Q is determined mostly by the external
components."

In typical references that I found, ie. low noise frequency sources, the loaded Q is between 20% to 50% of the unloaded Q.

So for the "Shawyer" type application, to get a maximized loaded Q in a cavity oscillator, one should maximize the loop gain phase slope.  (use the highest gain-bandwidth amp you can find ?)


The oscillator model is the same as the tuned amp model w/ the cavity used as a filter in the feedback loop.  In a quick search, I was only able to find one worked out example of using feedback to increase the loaded Q of an oscillator over that of the unloaded Q.

  www.zen22142.zen.co.uk/Design/vcqmswo.pdf
Title: Re: EM Drive Developments - related to space flight applications - Thread 2
Post by: Rodal on 03/02/2015 02:52 pm
....
The oscillator model is the same as the tuned amp model w/ the cavity used as a filter in the feedback loop.  In a quick search, I was only able to find one worked out example of using feedback to increase the loaded Q of an oscillator over that of the unloaded Q.

  www.zen22142.zen.co.uk/Design/vcqmswo.pdf
Very interesting that this is the only example readily available in a quick search.  The example comes from Lima, Peru.  One cannot help but wonder whether the worked out analysis in the example is flawless and whether it can work in practice to increase the loaded Q over that of the unloaded Q of the EM Drive.

Have the loaded Q's always been lower than the unloaded Q's in all the EM Drive experiments performed in the UK, US and China?  There seems to be a paucity of data on this, is my memory is correct...
Title: Re: EM Drive Developments - related to space flight applications - Thread 2
Post by: aero on 03/02/2015 03:09 pm


Maybe Dr. Rodal will tell me what the resonant frequency should be for the cylindrical cavity with dielectric that I posted data for earlier today?
Please fill-in the following data (question marks ? below) for the cylindrical cavity with a dielectric section having the same diameter as the cavity's ID and located at one end of the cavity:

GEOMETRY

Inner Diameter of cylindrical cavity =   ? meters (cavity has a constant, same diameter throughout)
Total Inner Length of cavity            =   ? meters (Length including dielectric length)
Length of dielectric section              =   ? meters

CONSTITUTIVE PROPERTIES

Relative electric permittivity of dielectric =      ?     (dimensionless)  (for HD PE it is reported as 2.3)
Relative magnetic permeability of dielectric =      ?     (dimensionless)  (1 ?)
Relative electric permittivity of empty section =   ?   (dimensionless)  (air or vacuum ?)
Relative magnetic permeability of empty section =   ?   (dimensionless)  (air or vacuum ?)

FREQUENCY of interest

There are an infinite number of resonant frequencies for a resonant cylindrical cavity. 
What resonant frequency are you referring to?     ?  (The lowest natural frequency? )

I just printed these numbers out from my program so this is what I used when generating the posted data.
Inner diameter of cylindrical cavity, 0.08278945,m
 total inner length of cavity, 0.1224489,m
 Length of dielectric section 0.027282494103102, m

 Relative electric permittivity of dielectric =1.76


Relative magnetic permeability of dielectric =      1     
Relative electric permittivity of empty section =   vacuum   (a meep program option)
Relative magnetic permeability of empty section = vacuum

I know that the dielectric constant of hdpe is 2.3. I will make some runs later using that value but for now I am using 1.76.

The resonances I'm looking for are those at the peaks of the force/power curves above, near 1.8 Ghz, 1.9 GHz and 2.4 GHz using an electric source, and about 2.3 GHz with the magnetic source.

It could be that there are no resonances in those frequency ranges but everything we know right now says that there will be.

And thanks.
Title: Re: EM Drive Developments - related to space flight applications - Thread 2
Post by: Rodal on 03/02/2015 05:03 pm


Maybe Dr. Rodal will tell me what the resonant frequency should be for the cylindrical cavity with dielectric that I posted data for earlier today?
Please fill-in the following data (question marks ? below) for the cylindrical cavity with a dielectric section having the same diameter as the cavity's ID and located at one end of the cavity:

GEOMETRY

Inner Diameter of cylindrical cavity =   ? meters (cavity has a constant, same diameter throughout)
Total Inner Length of cavity            =   ? meters (Length including dielectric length)
Length of dielectric section              =   ? meters

CONSTITUTIVE PROPERTIES

Relative electric permittivity of dielectric =      ?     (dimensionless)  (for HD PE it is reported as 2.3)
Relative magnetic permeability of dielectric =      ?     (dimensionless)  (1 ?)
Relative electric permittivity of empty section =   ?   (dimensionless)  (air or vacuum ?)
Relative magnetic permeability of empty section =   ?   (dimensionless)  (air or vacuum ?)

FREQUENCY of interest

There are an infinite number of resonant frequencies for a resonant cylindrical cavity. 
What resonant frequency are you referring to?     ?  (The lowest natural frequency? )

I just printed these numbers out from my program so this is what I used when generating the posted data.
Inner diameter of cylindrical cavity, 0.08278945,m
 total inner length of cavity, 0.1224489,m
 Length of dielectric section 0.027282494103102, m

 Relative electric permittivity of dielectric =1.76


Relative magnetic permeability of dielectric =      1     
Relative electric permittivity of empty section =   vacuum   (a meep program option)
Relative magnetic permeability of empty section = vacuum

I know that the dielectric constant of hdpe is 2.3. I will make some runs later using that value but for now I am using 1.76.

The resonances I'm looking for are those at the peaks of the force/power curves above, near 1.8 Ghz, 1.9 GHz and 2.4 GHz using an electric source, and about 2.3 GHz with the magnetic source.

It could be that there are no resonances in those frequency ranges but everything we know right now says that there will be.

And thanks.

GEOMETRICAL INPUT for cylindrical cavity:

bigDiameter = 8.278945 centimeter;
smallDiameter = bigDiameter;
length = 12.24489 centimeter;
dielectricThickness = 2.7282494103102 centimeter;



You can verify this case directly from the equation in Wikipedia http://en.wikipedia.org/wiki/Microwave_cavity#Cylindrical_cavity

First four mode shapes and frequencies for Relative electric permittivity of dielectric =1 (Same thing as no dielectric)

{{"TE", 1, 1, 0}, 2.12223*10^9},
{{"TE", 1, 1, 1}, 2.44998*10^9}, {{"TM", 0, 1, 0}, 2.77191*10^9}, {{"TM", 0, 1, 1}, 3.03019*10^9}



First three mode shapes and frequencies for Relative electric permittivity of dielectric =1.76; dielectricThickness = 2.7282494103102 centimeter

{{"TE", 1, 1, 1}, 2.31958*10^9}, {{"TE", 1, 1, 2}, 3.03479*10^9}, {{"TM", 0, 1, 2}, 3.47425*10^9}



First three mode shapes and frequencies for Relative electric permittivity of dielectric =2.3; dielectricThickness = 2.7282494103102 centimeter

{{"TE", 1, 1, 1}, 2.26774*10^9}, {{"TE", 1, 1, 2}, 2.93557*10^9}, {{"TM", 0, 1, 2}, 3.37114*10^9}


Title: Re: EM Drive Developments - related to space flight applications - Thread 2
Post by: Rodal on 03/02/2015 05:33 pm
 Perhaps you are getting lower frequencies because you are using Maxwell's equations in 2-D ?, and you are modeling the cavity as a flat plane bounded by a rectangle instead of 3-D cylindrical cavity under 3-D Maxwell's equations

Intuition would say that 2-D would lead to higher frequencies (the opposite of what you get), because there is less possible motion (everything is restricted to the plane).  If natural frequencies are higher in 3-D than 2_D that means that there are more constraints in Maxwell's eqns. in 3-D than in 2-D.  ....Mmmmm....I would need to know more about the 2-Dimensional model, and how it attempts to replicate a 3-Dimensional reality.

This brings back memories from a thought-provoking book, (but it didn't deal with Maxwell's equations in 2-D and 3-D, the people in Flatland are indeed more constrained and everything is stiffer there   :)  ) :

http://www.amazon.com/Flatland-Romance-Dimensions-Thrift-Editions/dp/048627263X/ref=sr_1_1?ie=UTF8&qid=1425320304&sr=8-1&keywords=flatland+abbott
Title: Re: EM Drive Developments - related to space flight applications - Thread 2
Post by: aero on 03/02/2015 05:46 pm


Maybe Dr. Rodal will tell me what the resonant frequency should be for the cylindrical cavity with dielectric that I posted data for earlier today?
                                  ... snip ...

                       ... snip ...

GEOMETRICAL INPUT for cylindrical cavity:

bigDiameter = 8.278945 centimeter;
smallDiameter = bigDiameter;
length = 12.24489 centimeter;
dielectricThickness = 2.7282494103102 centimeter;



You can verify this case directly from equation in Wikipedia http://en.wikipedia.org/wiki/Microwave_cavity#Cylindrical_cavity

First four mode shapes and frequencies for Relative electric permittivity of dielectric =1 (No dielectric)

{{"TE", 1, 1, 0}, 2.12223*10^9},
{{"TE", 1, 1, 1}, 2.44998*10^9}, {{"TM", 0, 1, 0}, 2.77191*10^9}, {{"TM", 0, 1, 1}, 3.03019*10^9}



First three mode shapes and frequencies for Relative electric permittivity of dielectric =1.76; dielectricThickness = 2.7282494103102 centimeter

{{"TE", 1, 1, 1}, 2.31958*10^9}, {{"TE", 1, 1, 2}, 3.03479*10^9}, {{"TM", 0, 1, 2}, 3.47425*10^9}



First three mode shapes and frequencies for Relative electric permittivity of dielectric =2.3; dielectricThickness = 2.7282494103102 centimeter

{{"TE", 1, 1, 1}, 2.26774*10^9}, {{"TE", 1, 1, 2}, 2.93557*10^9}, {{"TM", 0, 1, 2}, 3.37114*10^9}


Perhaps you are getting much lower frequencies because you are using Maxwell's equations in 2-D ?, and you are modeling the cavity as a flat plane bounded by a rectangle instead of 3-D cylindrical cavity under 3-D Maxwell's equations

Thank you for running those numbers. You are likely correct in that 2D and 3D do seem to give different answers, although I find the 2D results indicative of what the 3D will be. I will need to run a few 3D samples to see what happens in detail though.
Title: Re: EM Drive Developments - related to space flight applications - Thread 2
Post by: frobnicat on 03/03/2015 12:27 am
Dr. March,

I'm trying to model various aspects of the whole system to put upper bounds on thermal effects, and possibly also reconstruct the thrust(t) original signal from the distance(t) given in the charts. It would be a nice boost to this (amateur level) effort if you could confirm either :
- That the flexure bearings have a stiffness of 0.007 in-Lb/deg ? Each ? Both together ? Do you know the exact model reference ?
- That the vertical scale in the charts (indicated in µm, around 500) are relevant or not relevant.

I ask this question because I find a contradiction between the stiffness around the vertical axis and the recorded deviation from the 30µN calibration pulses (at .007 in-Lb/deg the deviation of the linear displacement sensor would be above 40µm, at .014 in-Lb/deg still above 20µm). The readings amount for between 1 to 2.5 µm for the 30µN calibration pulses. So I'm stuck.

While I'm at it : is the plane in which the arm rotates kept as horizontal as possible (ie the axis of rotation as vertical as possible) or is there a small slope voluntarily introduced leading to some pendulum effect against g (for stabilisation or tuning purpose) ? That could explain the varying deviation (in µm) for the same calibration pulses thrusts. Also wondered if this is what was implied in this post (http://forum.nasaspaceflight.com/index.php?topic=36313.msg1331580#msg1331580) :
Quote from: Star-Drive
...
These thermally induced actions to the left requires the torque pendulum's arm to move to the right to maintain the balance of the torque pendulum's arm in the lab's 1.0 gee gravity field, since we also use the Earth's g-field to help null the pendulum's movements.
...

Thanks


Frobnicat:

To answer your question:

" - That the flexure bearings have a stiffness of 0.007 in-Lb/deg ? Each ? Both together ? Do you know the exact model reference?"

The two torsion bearings used in or torque pendulum are supposed to have a stiffness of 0.007 in-Lb/deg, +/-10% and is made by the Riverhawk Co. in New York USA.  As to their model number find the data sheet for same attached. 

"- That the vertical scale in the charts (indicated in µm, around 500) are relevant or not relevant."

The Philtec D63 fiber-optic displacement sensor measures distance from its target mirror in microns, so the numbers on the left hand side of the force plots measure the distance from the end of the fiber-optic laser head to its mirror target mounted on the torque pendulum arm.  The data sheet for same is attached.

"While I'm at it : is the plane in which the arm rotates kept as horizontal as possible (ie the axis of rotation as vertical as possible) or is there a small slope voluntarily introduced leading to some pendulum effect against g (for stabilization or tuning purpose)?"

The design of our Torque pendulum follows what JPL and Busek Co did at their respective facility, see attached report from Busek.  We found that if we tried to keep the arm completely horizontal though that the pendulum's neutral point would wonder erratically and make alignments near impossible.  So yes I balance the pendulum arm so there is always a slight tilt in it, however this tilt angle magnitude is not controlled as well as it probably should.

Best, Paul M.

Thank you very much for those precious informations. The tilt angle magnitude can probably be inferred from the deviation against the calibration pulses, if we can model the gravitational pendulum component on top of the flexure bearing restoring torque component.

For that we need to know :

Mass of :
  frustum, without dielectric : 1.606 kg
  microwave power amplifier : below 8kg ? 
  faztek horizontal beam : 2.18 Lb (from 1.09Lb/Ft) ?
  Ideally, Total mass with a rough estimate of position of each part...
 
Distances along the arm from vertical axis of rotation to the centre of :
  Long end of arm (frustum side) : 15.5''
  Short end of arm (amplifier side) : 8.5''
  Frustum : 15.5 - 4 = 11.5'' ?
  Electrostatic Fins Calibration System : 15.5-4 = 11.5'' ?
  Linear Displacement Sensor : 15.5-1 = 14.5'' ?
  microwave power amplifier : between 4.25'' and 8.5'' ? 

Stiffness of flexure bearings : .014 in-Lb/deg total  (2 times .007 each)

In short : what is the total mass of the whole rotating assembly, where is the centre of mass of the whole rotating assembly relative to axis of rotation, and what is the moment of inertia around the (almost) vertical axis of rotation (for the later, to assess the dynamics and not just the equilibrium).

green : explicitly provided value
orange : inferred from pictures or derived by me from faztek sellers, to be confirmed
red : not found, do we have better than bounds for those ?


Title: Re: EM Drive Developments - related to space flight applications - Thread 2
Post by: zen-in on 03/03/2015 03:30 am



"- That the vertical scale in the charts (indicated in µm, around 500) are relevant or not relevant."

The Philtec D63 fiber-optic displacement sensor measures distance from its target mirror in microns, so the numbers on the left hand side of the force plots measure the distance from the end of the fiber-optic laser head to its mirror target mounted on the torque pendulum arm.  The data sheet for same is attached.

"While I'm at it : is the plane in which the arm rotates kept as horizontal as possible (ie the axis of rotation as vertical as possible) or is there a small slope voluntarily introduced leading to some pendulum effect against g (for stabilization or tuning purpose)?"

The design of our Torque pendulum follows what JPL and Busek Co did at their respective facility, see attached report from Busek.  We found that if we tried to keep the arm completely horizontal though that the pendulum's neutral point would wonder erratically and make alignments near impossible.  So yes I balance the pendulum arm so there is always a slight tilt in it, however this tilt angle magnitude is not controlled as well as it probably should.

Best, Paul M.

The torque pendulum arm had a slight tilt so that alignment was easier.  The Philtec D63 fiber-optic displacement sensor measures distance from its target mirror by measuring the intensity of the reflected light.   If the change in the center of mass reduces the pendulum arm tilt, the light intensity may increase.  This would explain the negative slope of the baseline waveform (apparent movement closer) after the RF is switched off.   It would be interesting to see what the thrust waveform looks like when the cavity is turned around.
Title: Re: EM Drive Developments - related to space flight applications - Thread 2
Post by: frobnicat on 03/03/2015 08:35 am

"- That the vertical scale in the charts (indicated in µm, around 500) are relevant or not relevant."

The Philtec D63 fiber-optic displacement sensor measures distance from its target mirror in microns, so the numbers on the left hand side of the force plots measure the distance from the end of the fiber-optic laser head to its mirror target mounted on the torque pendulum arm.  The data sheet for same is attached.

"While I'm at it : is the plane in which the arm rotates kept as horizontal as possible (ie the axis of rotation as vertical as possible) or is there a small slope voluntarily introduced leading to some pendulum effect against g (for stabilization or tuning purpose)?"

The design of our Torque pendulum follows what JPL and Busek Co did at their respective facility, see attached report from Busek.  We found that if we tried to keep the arm completely horizontal though that the pendulum's neutral point would wonder erratically and make alignments near impossible.  So yes I balance the pendulum arm so there is always a slight tilt in it, however this tilt angle magnitude is not controlled as well as it probably should.

Best, Paul M.

The torque pendulum arm had a slight tilt so that alignment was easier.  The Philtec D63 fiber-optic displacement sensor measures distance from its target mirror by measuring the intensity of the reflected light.   If the change in the center of mass reduces the pendulum arm tilt, the light intensity may increase.  This would explain the negative slope of the baseline waveform (apparent movement closer) after the RF is switched off.   It would be interesting to see what the thrust waveform looks like when the cavity is turned around.

What I understand is that the main vertical axis of rotation Z of balance arm is tilted so that the pendulum gains in stability : if the CoM of the rotating assembly is in front of the axis (frustum side, X+) then the tilt would be to the front, the arm rotates in a plane XY with a slight downward slope toward the frustum (X+). If the CoM of the rotating assembly is back of the axis (amplifier side, X-) then the tilt would be to the back, the arm rotates in a plane XY with a slight downward slope toward the back (X-).

(http://forum.nasaspaceflight.com/index.php?action=dlattach;topic=36313.0;attach=809641;image)

I wouldn't say that a shift in centre of mass would change (significantly) the tilt (ie XY plane's slope) as the flexure bearing have high stiffness around Y and X, but my concern specifically is that a shift in centre of mass (tangential relative to arm rotation, along Y) would change significantly the equilibrium point within the sloped XY plane. It's well known that for movements constrained in a plane with slope Alpha relative to horizontal (ie air cushion inclined table) there is an acceleration equivalent to a reduced gravity of magnitude  g*sin(Alpha).

We know from this post (http://forum.nasaspaceflight.com/index.php?topic=36313.msg1327012#msg1327012) of one chart with the frustum mounted in reverse (small end toward the right, Y+) that after power-on (short and of low magnitude in this case) following drift is upward:
(http://forum.nasaspaceflight.com/index.php?action=dlattach;topic=36313.0;attach=635481;image)

While in the "forward" configuration ( small end of frustum toward the left, Y-) the baseline drift after power-on is generally downward. See for instance from this post (http://forum.nasaspaceflight.com/index.php?topic=36313.msg1326997#msg1326997)
(http://forum.nasaspaceflight.com/index.php?action=dlattach;topic=36313.0;attach=636345;image)

This would imply that this likely thermal drift may not or no longer entirely be explained by a heating of flexure bearings by IR of amplifier : wouldn't direction of such effect don't depend on orientation of frustum ?
Or maybe not, if mounting frustum in reverse alters torque around X... needs coupling analysis, the flexure bearings are not spiral springs...

Anyhow, considerable effort has gone into isolating the flexure bearings from amplifier IR heating, as seen in picture attached with this post (http://forum.nasaspaceflight.com/index.php?topic=36313.msg1327012#msg1327012) :
(http://forum.nasaspaceflight.com/index.php?action=dlattach;topic=36313.0;attach=635476;image)

Such effort were already relatively successful before such thermal shrinkwrapping if we are to judge from the dummy load test  taken before from same post (http://forum.nasaspaceflight.com/index.php?topic=36313.msg1327012#msg1327012)
(http://forum.nasaspaceflight.com/index.php?action=dlattach;topic=36313.0;attach=635474;image)

BTW in this last dummy load test, we don't see the stray 9.6µN : "average null force is 9.6 micronewtons due to 5.6A DC current in power cable" figure 20 page 16 in Brady et al report (anomalous... (http://www.libertariannews.org/wp-content/uploads/2014/07/AnomalousThrustProductionFromanRFTestDevice-BradyEtAl.pdf))
Was that cancelled somehow ? Are we supposed non longer to subtract 9.6µN to equilibrium thrusts ?
Title: Re: EM Drive Developments - related to space flight applications - Thread 2
Post by: lele on 03/03/2015 11:03 am
What do you think will be needed to prove there is (or not) a real effect, or at least to get enough scientists interested so that a proof or a debunk of the effect is done relatively quickly?
Can something simpler than an on-orbit demonstration be enough?

It's a quite general question which can be asked for any scientific controversy, but since the existence of an EM Drive would be at odd with what 99% of scientists hold to be true, it seems to me they don't want to be caugh anywhere near what seems to be a crackpot theory. Even if building and testing an EM Drive seems relatively cheap and fast*, especially when compared to various fusion initiatives for example.
Actually the fact that it's cheap may not help: it looks like a relatively mundane way to reach fundamentals things of the universe, maybe it would be better accepted if it was a billion-dollars initiative...

I feel a little silly posting that because I'm not adding any concrete value to the thread  :) In any case, I want to thank the people here for working quite hard on this problem and telling us what they're finding, since it's the best way to reach a definitive answer.


*But still an important effort for one person
Title: Re: EM Drive Developments - related to space flight applications - Thread 2
Post by: Stormbringer on 03/03/2015 11:48 am
well; one of the slides that Mr March provided showed 98 micronewtons. He has also said that due to sensitivity thresholds of the balance equipment at the Glenn Research Center facilities the Eagle Works team need consistent thrust signals of 100 micronewtons before they can send it out GRC for independant replication. So while on the face of it they should be pretty close if the 98 micronewton signal from that slide is consistently obtained; I don't think they have consistent output at that level though.  So they are doing everything they can to improve thier device but there is a deadline of EOM march. Not much time.

In my opinion such a low tempo low priority side project that could probably be fully funded from the couch change from break room lounge chairs should not be micromanaged like it's a multi-billion dollar resource intensive space probe project and put under deadline pressure like that.


Title: Re: EM Drive Developments - related to space flight applications - Thread 2
Post by: Rodal on 03/03/2015 01:55 pm
Having successfully compared our exact solution with both the COMSOL Finite Element solution and the experimental thermal IR camera results presented by Paul March ( http://forum.nasaspaceflight.com/index.php?topic=36313.msg1339803#msg1339803), we now compare our exact solution with Greg Egan's exact solution results ( http://gregegan.customer.netspace.net.au/SCIENCE/Cavity/Cavity.html ), as follows:


Numerical data for example TM modes, r1=2.5 cm, r2=10 cm, θw = 20°

GREG EGAN            RODAL

TM011
n = 6.38323            6.38323         
k = 0.861947 cm–1         0.861947 cm–1
freq = 4.12 GHz         4.11265 GHz

TM012
n = 6.38323            6.38323
k = 1.2847 cm–1         1.28475 cm–1
freq = 6.13 GHz         6.12998 GHz

TM013
n = 6.38323            6.38323
k = 1.6409 cm–1         1.64095 cm–1
freq = 7.83 GHz         7.82954 GHz


Numerical data for example TE modes, r1=2.5 cm, r2=10 cm, θw = 20°

GREG EGAN            RODAL

TE011
n = 10.4885            10.4885
k = 1.55771 cm–1         1.55771 cm–1   
freq = 7.438 GHz         7.43236 GHz   

TE012
n = 10.4885            10.4885
k = 1.96024 cm–1         1.96024 cm–1
freq = 9.359 GHz         9.35299 GHz

TE013
n = 10.4885            10.4885
k = 2.326 cm–1         2.32619 cm–1
freq = 11.10 GHz         11.099 GHz


The comparison is excellent.  Below is the comparison of the contour plots for the TM (transverse magnetic) modes:

(http://gregegan.customer.netspace.net.au/SCIENCE/Cavity/TM1.gif)
(http://gregegan.customer.netspace.net.au/SCIENCE/Cavity/TM2.gif)
(http://gregegan.customer.netspace.net.au/SCIENCE/Cavity/TM3.gif)
Title: Re: EM Drive Developments - related to space flight applications - Thread 2
Post by: Rodal on 03/03/2015 01:59 pm
Here is the comparison of the contour plots for the TE (transverse electric) modes

(http://gregegan.customer.netspace.net.au/SCIENCE/Cavity/TE1.gif)
(http://gregegan.customer.netspace.net.au/SCIENCE/Cavity/TE2.gif)
(http://gregegan.customer.netspace.net.au/SCIENCE/Cavity/TE3.gif)
Title: Re: EM Drive Developments - related to space flight applications - Thread 2
Post by: Rodal on 03/03/2015 02:32 pm
....

In my opinion such a low tempo low priority side project that could probably be fully funded from the couch change from break room lounge chairs should not be micromanaged like it's a multi-billion dollar resource intensive space probe project and put under deadline pressure like that.

Probably the "micromanaging deadline" you are referring to is due to internal pressure at NASA because there are other NASA centers that were specifically chartered with "Advanced Space Propulsion" R&D: NASA Johnson was not chartered by management to conduct such "Advanced Space Propulsion" R&D, but instead it was chartered with human spaceflight training, research, and flight control.  Due to the demise of the Space Shuttle operations, there has been a curtailing of Johnson's funding.  By contrast, for example,  the George C. Marshall Space Flight Center (MSFC) was chartered as the U.S. government's civilian rocketry and spacecraft propulsion research center.

It is probably a response to criticism that NASA has too many centers spread through the country, created during the Apollo era when the budget for NASA was much larger (as a proportion of the US budget) and the ensuing in-fighting for where (at what NASA centers) should  "Advanced Space Propulsion" be conducted in an era of falling budgets.
Title: Re: EM Drive Developments - related to space flight applications - Thread 2
Post by: Mulletron on 03/03/2015 03:17 pm
http://www.ebay.com/itm/AERCOM-Microwave-RF-Isolator-Circulator-2-4GHz-20dB-isolation-Low-I-L-TESTED-/281549538390?ssPageName=ADME:L:OU:US:1120
Picked up one of these puppies on Ebay to protect my amp. Another example of broken time reversal symmetry in action.

Got about an oz of very expensive liquid metal from here:
http://www.amazon.com/Gallium-Indium-Eutectic-GaInSn-68-5%25/dp/B00KN92MWW/ref=sr_1_3?ie=UTF8&qid=1425074693&sr=8-3&keywords=galinstan

So back to the copper from way back: http://forum.nasaspaceflight.com/index.php?topic=36313.msg1326742#msg1326742
...
Been working with the supplier with a machine shop I posted about way back:
http://forum.nasaspaceflight.com/index.php?topic=36313.msg1326669#msg1326669
I'm going that route. The quote I got is: price: $120.00 layout + $51.63 for part + freight. So I have to pay the layout, then anyone else who wants one of these:
(https://lh4.googleusercontent.com/xpJIaQNfcmecDoHrIDnoYqVs1ccOghkxSpkgXhJ8MrnoyaK61MQ9rSfiBnuIhyayCIIEaQ=w1549-h745)
but built in 16oz copper, with a smooth butt seam inside, and 1/4" flange around edges, can get one for about 50 bucks plus shipping. If all this works out, it'll fulfill my goal of making a replication by DIYers easier. For me, paying the layout plus price about breaks even with buying the sheet myself and fumblefuddeling around trying to solder up a cone at home. So I'm happy. I'll get back with more later, when the items are at home.

Hello Mulletron,

Hope you'll be able to conduct another test, and I guess there are many other people like me who encourage you to continue.
How do you intend to measure the (very tiny) thrust ? Reading back NASA's paper, it seems they had to run the experiment in a lab with complex (and expensive) tools to remove all parasite effects that would interfere with the thrust from the apparatus. Maybe you have access to such equipment ?

--
Mathieu

Going to do a Cavendish experiment:
http://www.intalek.com/Index/Projects/Research/CavendishExperiment.htm
https://www.fourmilab.ch/gravitation/foobar/ (link is down right now, hope it comes back)
https://www.youtube.com/watch?v=dyLYbvZIYoU

Don't need expensive gear. You can do amazing things with some fishing line and ingenuity.
Title: Re: EM Drive Developments - related to space flight applications - Thread 2
Post by: Notsosureofit on 03/03/2015 03:50 pm
http://www.ebay.com/itm/AERCOM-Microwave-RF-Isolator-Circulator-2-4GHz-20dB-isolation-Low-I-L-TESTED-/281549538390?ssPageName=ADME:L:OU:US:1120
Picked up one of these puppies on Ebay to protect my amp. Another example of broken time reversal symmetry in action.

Got about an oz of very expensive liquid metal from here:
http://www.amazon.com/Gallium-Indium-Eutectic-GaInSn-68-5%25/dp/B00KN92MWW/ref=sr_1_3?ie=UTF8&qid=1425074693&sr=8-3&keywords=galinstan

So back to the copper from way back: http://forum.nasaspaceflight.com/index.php?topic=36313.msg1326742#msg1326742
...
Been working with the supplier with a machine shop I posted about way back:
http://forum.nasaspaceflight.com/index.php?topic=36313.msg1326669#msg1326669
I'm going that route. The quote I got is: price: $120.00 layout + $51.63 for part + freight. So I have to pay the layout, then anyone else who wants one of these:
(https://lh4.googleusercontent.com/xpJIaQNfcmecDoHrIDnoYqVs1ccOghkxSpkgXhJ8MrnoyaK61MQ9rSfiBnuIhyayCIIEaQ=w1549-h745)
but built in 16oz copper, with a smooth butt seam inside, and 1/4" flange around edges, can get one for about 50 bucks plus shipping. If all this works out, it'll fulfill my goal of making a replication by DIYers easier. For me, paying the layout plus price about breaks even with buying the sheet myself and fumblefuddeling around trying to solder up a cone at home. So I'm happy. I'll get back with more later, when the items are at home.

Hello Mulletron,

Hope you'll be able to conduct another test, and I guess there are many other people like me who encourage you to continue.
How do you intend to measure the (very tiny) thrust ? Reading back NASA's paper, it seems they had to run the experiment in a lab with complex (and expensive) tools to remove all parasite effects that would interfere with the thrust from the apparatus. Maybe you have access to such equipment ?

--
Mathieu

Going to do a Cavendish experiment:
http://www.intalek.com/Index/Projects/Research/CavendishExperiment.htm
https://www.fourmilab.ch/gravitation/foobar/ (link is down right now, hope it comes back)
https://www.youtube.com/watch?v=dyLYbvZIYoU

Don't need expensive gear. You can do amazing things with some fishing line and ingenuity.

Same here.  The EBay units I've got are #201065780928 and #131442703325 so far in case anyone want to try the same system.
Title: Re: EM Drive Developments - related to space flight applications - Thread 2
Post by: Rodal on 03/03/2015 04:58 pm
If one looks closely at the contour plots of the TM modes in http://forum.nasaspaceflight.com/index.php?topic=36313.msg1340906#msg1340906, one will notice that Egan's contour plots have the maximum of the magnetic field occurring at the cone surface while my contour plots have the maximum of the magnetic field occurring at a distance from the cone's surface.  I have examined the source of this discrepancy.  My conclusion is that Egan's plots are incorrect regarding this feature, and they are inconsistent with the equations on Egan's post.  If anybody is interested to know why, I can post the mathematical proof and discussion for this conclusion.
Title: Re: EM Drive Developments - related to space flight applications - Thread 2
Post by: flux_capacitor on 03/03/2015 08:41 pm
Does anyone clearly understand the reaction force R vs opposite thrust T in Shawyer's newest document "A Note on the Principles of EmDrive force measurement" (http://www.emdrive.com/EmDriveForceMeasurement.pdf) that I attach to this message? Especially, the problem of the "restrained thruster" of fig. 3 where no movement could be detected at all?

Shawyer seems to imply that the EmDrive needs to experience an acceleration (even the slightest one) to create a force that would me measurable. A fully restrained cavity would not move at all. It reminds me of the the Mach effect thruster, where according to Woodward the material needs to undergo a proper acceleration while being energized, otherwise the transient mass fluctuation does not occur.

Evidently, Eagleworks' RF resonant cavity test article is fixed to the rest of the apparatus, but Shawyer explains any small thermal effect makes the walls move hence the center of gravity of the cavity also move a bit, and this would be enough for any small force to appear.
Title: Re: EM Drive Developments - related to space flight applications - Thread 2
Post by: frobnicat on 03/03/2015 09:25 pm
Does anyone clearly understand the reaction force R vs opposite thrust T in Shawyer's newest document "A Note on the Principles of EmDrive force measurement" (http://www.emdrive.com/EmDriveForceMeasurement.pdf) that I attach to this message? Especially, the problem of the "restrained thruster" of fig. 3 where no movement could be detected at all?

Shawyer seems to imply that the EmDrive needs to experience an acceleration (even the slightest one) to create a force that would me measurable. A fully restrained cavity would not move at all. It reminds me of the the Mach effect thruster, where according to Woodward the material needs to undergo a proper acceleration while being energized, otherwise the transient mass fluctuation does not occur.

Evidently, Eagleworks' RF resonant cavity test article is fixed to the rest of the apparatus, but Shawyer explains any small thermal effect makes the walls move hence the center of gravity of the cavity also move a bit, and this would be enough for any small force to appear.

From memory, nobody clearly understands what Shawyer is saying about forces and accelerations in this paper. I recall at least two comments on the apparent contradictions if you haven't seen them, by me and Rodal, here (http://forum.nasaspaceflight.com/index.php?topic=36313.msg1310015#msg1310015) and there (http://forum.nasaspaceflight.com/index.php?topic=36313.msg1310242#msg1310242).
Stormbringer also makes an attempt here (http://forum.nasaspaceflight.com/index.php?topic=36313.msg1309951#msg1309951). Maybe better to unwind from the start (http://forum.nasaspaceflight.com/index.php?topic=36313.msg1309452#msg1309452) with Mulletron bringing up this article.
Sorry if I forgot other contributors take on that.

To make my current line of inquiry explicit : if small thermal effect makes the walls move hence the center of gravity of the cavity also move a bit, then this might be enough for sustained small deviations of the linear displacement readings to appear, because the measures on Eagleworks balance have a gravitational pendulum component. Such sustained deviation would not need sustained net thrust.
Title: Re: EM Drive Developments - related to space flight applications - Thread 2
Post by: Rodal on 03/03/2015 09:28 pm
Does anyone clearly understand the reaction force R vs opposite thrust T in Shawyer's newest document "A Note on the Principles of EmDrive force measurement" (http://www.emdrive.com/EmDriveForceMeasurement.pdf) that I attach to this message? Especially, the problem of the "restrained thruster" of fig. 3 where no movement could be detected at all?

Shawyer seems to imply that the EmDrive needs to experience an acceleration (even the slightest one) to create a force that would me measurable. A fully restrained cavity would not move at all. It reminds me of the the Mach effect thruster, where according to Woodward the material needs to undergo a proper acceleration while being energized, otherwise the transient mass fluctuation does not occur.

Evidently, Eagleworks' RF resonant cavity test article is fixed to the rest of the apparatus, but Shawyer explains any small thermal effect makes the walls move hence the center of gravity of the cavity also move a bit, and this would be enough for any small force to appear.

No, it is indecipherable to me.  No wonder people asked him about this: other people have difficulty understanding Shawyer regarding these forces.

And what do Shawyer and W..... (if  W..... was also involved in this) mean by "fully restrained" ???????????

There is no infinitely rigid restraint in nature.  In NASA's Eagleworks tests the EM Drive is restrained by the torsional stiffness of the torsional pendulum's couple of RiverHawks.     Fully-restrained is mambo-jambo wording to me.  It would be better to use equations. A minimum acceleration is needed? What is the magnitude of that minimum acceleration and what governs its magnitude?  Need a stiff restraint to have no force? How stiff?  Is there a number? What governs the number?

But it may be my fault because I would much prefer an analytical paper where equations and mathematical symbols would greatly outweigh the number of words.   

Title: Re: EM Drive Developments - related to space flight applications - Thread 2
Post by: flux_capacitor on 03/03/2015 09:35 pm
Thanks you two. I was under the bad impression I was silly for not understanding Shawyer's explanations.
Title: Re: EM Drive Developments - related to space flight applications - Thread 2
Post by: Rodal on 03/04/2015 12:01 am
Thanks you two. I was under the bad impression I was silly for not understanding Shawyer's explanations.
There is an easy way to test this.
Paul March said that they need to maximize the thrust for the experiment to be verified at NASA Glenn.
Paul March measures the displacement vs. time with an optical method as previously discussed in detail by zen-in  "Philtec D63 fiber-optic displacement sensor measures distance from its target mirror by measuring the intensity of the reflected light.".

If Shawyer is correct that the less restraint the better, then Paul March could lessen the current restraint in his experiments by replacing the RiverHawk bearing with another one (if available) having less stiffness.  If Shawyer is correct, the decrease in restraint should produce an increase in the measured response.
Title: Re: EM Drive Developments - related to space flight applications - Thread 2
Post by: Rodal on 03/04/2015 12:55 am
If one looks closely at the contour plots of the TM modes in http://forum.nasaspaceflight.com/index.php?topic=36313.msg1340906#msg1340906, one will notice that Egan's contour plots have the maximum of the magnetic field occurring at the cone surface while my contour plots have the maximum of the magnetic field occurring at a distance from the cone's surface.  I have examined the source of this discrepancy.  My conclusion is that Egan's plots are incorrect regarding this feature, and they are inconsistent with the equations on Egan's post.  If anybody is interested to know why, I can post the mathematical proof and discussion for this conclusion.
OK.  I will interpret the "likes" I got for the previous post, as meaning that they would like me to explain why I have arrived at the conclusion that Egan's contour plots that show the maximum of the magnetic field occurring at the cone surface are incorrect, and why my contour plots have the maximum of the magnetic field occurring at a distance from the cone's surface.

The solution to the resonant truncated cone 3-D cavity can be most conveniently expressed in spherical coordinates because the electromagnetic waves propagate as spherical waves inside it.  Spherical coordinates having three components: a spherical radius r (the radius of the sphere around the origin and two angles: a polar angle θ and an azimuthal angle φ.  The azimuthal angle φ describes a rotation around the longitudinal axis of the cone.  If you look at a cross-section (at a given value of the azimuthal angle φ, for example at φ=0) of the truncated cone that looks like a trapezium, the polar angle θ is perpendicular to the longitudinal axis.  We take θ =0 at the longitudinal axis, such that θ = - θw when touching the cone wall at the left and θ= + θw when touching the cone wall at the right.

(http://gregegan.customer.netspace.net.au/SCIENCE/Cavity/CavityShape.gif)

The solution for the electromagnetic fields can be expressed in terms of three separate multiplicative functions: one function in terms of the azimuthal angle φ, another function in terms of the spherical radius r and a function in terms of the polar angle θ.

Clearly the problem we are facing (with respect to the location of the maximum of the magnetic field in the θ direction) has to do with the function that depends on θ.  This function is an associated Legendre function.  Greg Egan names this function Q(θ), see http://gregegan.customer.netspace.net.au/SCIENCE/Cavity/Cavity.html.

The eigenvalue problem is finding a value "n" such that a function that Egan names Q1, where Q1=n(n+1)Q(θ), is zero at the edges (at the walls).   

Below I show a plot of Q1=n(n+1)Q(θ) as a function of θ, showing that indeed Q1=0 at the walls: at θ= - 20 degrees and at θ=20 degrees.

The function that determines the variation of the transverse magnetic field with θ, is the derivative of Q(θ) with respect to θ . Egan names this derivative Q'(θ).

I show below a plot of this function, Q'(θ), calculated by Mathematica analytically as the derivative of Q(θ) with respect to θ.  One can observe that this function (that governs the variation of the magnetic field in the polar angle direction θ) reaches an extremum before the ends.

I also show below a plot of the analytical expression that Greg Egan used to calculate this derivative:

Q'(θ) = n [cos θ Pn(cos θ) – Pn–1(cos θ)] / sin θ

Observe that this plot is exactly the same as the previous plot.  Greg Egan's analytic expression for the derivative Q'(θ) is correct: it agrees with Mathematica's calculation (one can also show this equality analytically with Mathematica by using == which gives "True" as a result).

So, why is Greg Egan showing the magnetic field reaching a maximum at the walls?

It occurred to me that perhaps this was due to Egan not including the sin  θ term (perhaps because sin θ goes to zero for θ going to zero, and therefore the expression for  Q'(θ) becomes 0/0 at θ=0, which needs special handling in a numerical code.  Analytically, however,  Limit  Q'(θ)  for θ -->0 is Q' -->0 )

Therefore, I show a plot of the numerator of Q'(θ) :
 
 Q'(θ) = n [cos θ Pn(cos θ) – Pn–1(cos θ)] / sin θ

Numerator Q'(θ) = n [cos θ Pn(cos θ) – Pn–1(cos θ)]

CONCLUSION: the plot of the numerator of Q'(θ)  (which therefore doesn't include sin θ occurring in the denominator) reaches an extemum at the walls, just like Greg Egan's plots.  Greg Egan's expression for Q'(θ) is correct: it should not give an extremum at the wall.  What are incorrect are the plots for the transverse magnetic fields for the TM modes.

(http://gregegan.customer.netspace.net.au/SCIENCE/Cavity/TM1.gif)

Greg Egan's plots for the transverse modes may be based on using the numerator of Q'(θ) and neglecting sin θ in the denominator.  There is one issue remaining: the plot of the numerator is symmetric around the center, if Greg Egan's plots are due to only using the numerator, he would have had to further write the code such that for negative values of the polar angle ( θ < 0 ) the negative of the numerator is used instead, in order to result in the asymmetric magnetic field (the magnetic field has different signs for negative θ than for positive θ).
Title: Re: EM Drive Developments - related to space flight applications - Thread 2
Post by: frobnicat on 03/04/2015 01:49 am
Thanks you two. I was under the bad impression I was silly for not understanding Shawyer's explanations.
There is an easy way to test this.
Paul March said that they need to maximize the thrust for the experiment to be verified at NASA Glenn.
Paul March measures the displacement vs. time with an optical method as previously discussed in detail by zen-in  "Philtec D63 fiber-optic displacement sensor measures distance from its target mirror by measuring the intensity of the reflected light.".

If Shawyer is correct that the less restraint the better, then Paul March could lessen the current restraint in his experiments by replacing the RiverHawk bearing with another one (if available) having less stiffness.  If Shawyer is correct, the decrease in restraint should produce an increase in the measured response.

The "restraint", understood as the rest equilibrium restoring torque, is more a matter of inclination of the plane XY of rotation of the arm of balance in gravity than from the flexure bearings stiffness :

The application point of the calibration pulses is 11.5''=.292m from the axis, and the linear displacement measured at 14.5''=.368m from the axis (inferred from the nice lateral view in this post (http://forum.nasaspaceflight.com/index.php?topic=36313.msg1335190#msg1335190)) and knowing the arm's end is 15.5'' from axis and faztek beam is 1.5'' sided.

Calibration pulse of 29.1µN at .2921m gives a torque of 8.50e-6 Nm
The 2 flex bearings are given for .007 in-Lb/deg each (+-10%), that is 9.06e-2 Nm/rad (total)
Under a torque of 8.50e-6 the angle of deflection would be 8.50e-6/9.064e-2=9.38e-5 rad
That would make a linear deviation of 34.5 µm

In all charts, the deviation associated with calibration pulse of 29.1µN is roughly between 1 and 2.5 µm, one order of magnitude less.

The other way around :
For charts with 2.5µm deviation on cal. pulse, deflection angle=6.79e-6 rad, that would make the flexure bearings restoring torque at 6.16e-7 Nm (given the stiffness). Compared to 8.50e-6 Nm of the torque induced by the cal. pulse that represents only 7.25%


µm dev / cal. pulse       ratio of restoring torque due to stiffness of flexure bearings
2.5                                7.25 %
2                                   5.80 %
1                                   2.90 %


The only thing that I see that can account for the apparent lacking torque is the equilibrating torque induced by the inclination of the plane of rotation of the arm toward the CoM of the rotating assembly, that is equivalent to a hanging pendulum. The way it is used, the balance is more than 90% a hanging pendulum and less than 10% a torsion pendulum driven by flexure stiffness.

If this analysis holds, small changes in stiffness of flexure bearings would make for a minor impact on results. Changes in inclination would be the major way to tune the (linearised hanging pendulum equivalent) stiffness.
Title: Re: EM Drive Developments - related to space flight applications - Thread 2
Post by: Rodal on 03/04/2015 01:56 am

....
The only thing that I see that can account for the apparent lacking torque is the equilibrating torque induced by the inclination of the plane of rotation of the arm toward the CoM of the rotating assembly, that is equivalent to a hanging pendulum. The way it is used, the balance is more than 90% a hanging pendulum and less than 10% a torsion pendulum driven by flexure stiffness.

If this analysis holds, small changes in stiffness of flexure bearings would make for a minor impact on results. Changes in inclination would be the major way to tune the (linearised hanging pendulum equivalent) stiffness.

A hanging pendulum hangs from a rigid support located above the weight.  Its period depends only on the length of the pendulum's arm (and g , the acceleration of gravity, which is practically constant on Earth).  The flexural stiffness of the pendulum's arm is negligible.

But here nothing is hanging from the stainless steel chamber "rigid ceiling" supported by arms with negligible flexural stiffness.

What I see is the EM Drive weight supported by a frame of Faztek aluminum beams, Faztek beams that are supported from below, not from the stainless steel chamber ceiling.



What rigid support (located above the EM Drive) is the EM Drive hanging from ?  (Where is the "hanging pendulum" rigid support located ?)

What constitutes the arm of the "hanging pendulum"?  Why does it have negligible flexural stiffness? (The flexural stiffness of the aluminum Faztek beams is far from being negligible)

Do you really mean a hanging pendulum (whose period depends only on the length of the arm)?
Or do you mean a flexural pendulum (whose period depends on the stiffness of the arm)?

And if you agree that the flexural stiffness of the arms are not negligible, why take into account only the portion above the weight?  What about the flexural stiffness below the weight? 
Aren't the Faztek beams supported from below?

What I see is your "z" axis going up to a Faztek frame or "bridge" and the "bridge" being supported by two vertical Faztek beams, and those vertical Faztek beams are supported from below, not from above.

Is that then really an inverted flexural/torsional pendulum since it is made by Faztek aluminum beams supported from below?

(http://forum.nasaspaceflight.com/index.php?action=dlattach;topic=36313.0;attach=809641;image)
(http://forum.nasaspaceflight.com/index.php?action=dlattach;topic=36313.0;attach=803832;image)
Title: Re: EM Drive Developments - related to space flight applications - Thread 2
Post by: aero on 03/04/2015 02:04 am
Do you mean that this might actually be right? I've ignored the fields that look like this because I thought they were supposed to be symmetric.
Title: Re: EM Drive Developments - related to space flight applications - Thread 2
Post by: Rodal on 03/04/2015 02:24 am
Do you mean that this might actually be right? I've ignored the fields that look like this because I thought they were supposed to be symmetric.

electric fields are asymmetric like this about the longitudinal axis like this (electric azimuthal field TE013):

it has to be asymmetric because the electric field in the circumferential (azimuthal) direction is constant for TE013, hence when looking at the cross-section, the field vector (at a given distance away from the bases) is coming out of the paper at the right and it is going into the paper at the left, and viceversa for the other (total of 3) different fields along the axis)

(http://forum.nasaspaceflight.com/index.php?action=dlattach;topic=36313.0;attach=811074;image)
Title: Re: EM Drive Developments - related to space flight applications - Thread 2
Post by: Star-Drive on 03/04/2015 02:42 am
Dr. March,

I'm trying to model various aspects of the whole system to put upper bounds on thermal effects, and possibly also reconstruct the thrust(t) original signal from the distance(t) given in the charts. It would be a nice boost to this (amateur level) effort if you could confirm either :
- That the flexure bearings have a stiffness of 0.007 in-Lb/deg ? Each ? Both together ? Do you know the exact model reference ?
- That the vertical scale in the charts (indicated in µm, around 500) are relevant or not relevant.

I ask this question because I find a contradiction between the stiffness around the vertical axis and the recorded deviation from the 30µN calibration pulses (at .007 in-Lb/deg the deviation of the linear displacement sensor would be above 40µm, at .014 in-Lb/deg still above 20µm). The readings amount for between 1 to 2.5 µm for the 30µN calibration pulses. So I'm stuck.

While I'm at it : is the plane in which the arm rotates kept as horizontal as possible (ie the axis of rotation as vertical as possible) or is there a small slope voluntarily introduced leading to some pendulum effect against g (for stabilisation or tuning purpose) ? That could explain the varying deviation (in µm) for the same calibration pulses thrusts. Also wondered if this is what was implied in this post (http://forum.nasaspaceflight.com/index.php?topic=36313.msg1331580#msg1331580) :
Quote from: Star-Drive
...
These thermally induced actions to the left requires the torque pendulum's arm to move to the right to maintain the balance of the torque pendulum's arm in the lab's 1.0 gee gravity field, since we also use the Earth's g-field to help null the pendulum's movements.
...

Thanks


Frobnicat:

To answer your question:

" - That the flexure bearings have a stiffness of 0.007 in-Lb/deg ? Each ? Both together ? Do you know the exact model reference?"

The two torsion bearings used in or torque pendulum are supposed to have a stiffness of 0.007 in-Lb/deg, +/-10% and is made by the Riverhawk Co. in New York USA.  As to their model number find the data sheet for same attached. 

"- That the vertical scale in the charts (indicated in µm, around 500) are relevant or not relevant."

The Philtec D63 fiber-optic displacement sensor measures distance from its target mirror in microns, so the numbers on the left hand side of the force plots measure the distance from the end of the fiber-optic laser head to its mirror target mounted on the torque pendulum arm.  The data sheet for same is attached.

"While I'm at it : is the plane in which the arm rotates kept as horizontal as possible (ie the axis of rotation as vertical as possible) or is there a small slope voluntarily introduced leading to some pendulum effect against g (for stabilization or tuning purpose)?"

The design of our Torque pendulum follows what JPL and Busek Co did at their respective facility, see attached report from Busek.  We found that if we tried to keep the arm completely horizontal though that the pendulum's neutral point would wonder erratically and make alignments near impossible.  So yes I balance the pendulum arm so there is always a slight tilt in it, however this tilt angle magnitude is not controlled as well as it probably should.

Best, Paul M.

Thank you very much for those precious informations. The tilt angle magnitude can probably be inferred from the deviation against the calibration pulses, if we can model the gravitational pendulum component on top of the flexure bearing restoring torque component.

For that we need to know :

Mass of :
  frustum, without dielectric : 1.606 kg
  microwave power amplifier : below 8kg ? 
  faztek horizontal beam : 2.18 Lb (from 1.09Lb/Ft) ?
  Ideally, Total mass with a rough estimate of position of each part...
 
Distances along the arm from vertical axis of rotation to the centre of :
  Long end of arm (frustum side) : 15.5''
  Short end of arm (amplifier side) : 8.5''
  Frustum : 15.5 - 4 = 11.5'' ?
  Electrostatic Fins Calibration System : 15.5-4 = 11.5'' ?
  Linear Displacement Sensor : 15.5-1 = 14.5'' ?
  microwave power amplifier : between 4.25'' and 8.5'' ? 

Stiffness of flexure bearings : .014 in-Lb/deg total  (2 times .007 each)

In short : what is the total mass of the whole rotating assembly, where is the centre of mass of the whole rotating assembly relative to axis of rotation, and what is the moment of inertia around the (almost) vertical axis of rotation (for the later, to assess the dynamics and not just the equilibrium).

green : explicitly provided value
orange : inferred from pictures or derived by me from faztek sellers, to be confirmed
red : not found, do we have better than bounds for those ?




Some of your questions are answered below in ( ).

Mass of:
frustum, without dielectric : 1.606 kg
microwave power amplifier : below 8kg ? (7.64118 kg + 0.200 kg mass)
faztek horizontal beam : 2.18 Lb (from 1.09Lb/Ft) ?  (0.92338 kg)
Ideally, Total mass with a rough estimate of position of each part...

Distances along the arm from vertical axis of rotation to the centre of :
Long end of arm (frustum side) : 15.5''    (14.75”)
Short end of arm (amplifier side) : 8.5''    (9.25”)
Frustum : 15.5 - 4 = 11.5'' ?                   (11.25”)
Electrostatic Fins Calibration System : 15.5-4 = 11.5'' ?  (11.25”)
Linear Displacement Sensor : 15.5-1 = 14.5'' ?  (13.88”)
Center of microwave power amplifier assembly: between 4.25'' and 8.5'' ? (~7.5”)

Best, Paul M.
Title: Re: EM Drive Developments - related to space flight applications - Thread 2
Post by: aero on 03/04/2015 03:19 am
Thank you Dr. Rodal - So here is the complete set for some magnetic source run. I didn't record any details except I can see  that the antenna in in the location of the magnetic antenna I use.  Is it possible that all of the images are correct? If so that would increase my confidence in the meep output.

Title: Re: EM Drive Developments - related to space flight applications - Thread 2
Post by: Flyby on 03/04/2015 04:49 am
I'm currently trying to finish up the cross section drawing on the flight demonstrator, but i kinda fail to understand how it fits together....

Looking closely (top down wise) at the lower rim it seems to me there is :

-small shiny rim (could be the edge of the alu cone?)
-brownish plate (copper plate?)
-small shiny rim
-thick plate (most probably holding the screw thread)

I'm puzzled about the second small, shiny rim...
Why would you need an additional small slab of alu under the (supposedly) copper plating?
Any one has an idea?
Title: Re: EM Drive Developments - related to space flight applications - Thread 2
Post by: Notsosureofit on 03/04/2015 11:24 am
The "Copper" is just the reflection of the floor from the side of the flange.  The 2 bright lines are reflections from the edge bevels.  If they used the Copper CF (or wire) seals they are located inside of the bolt line.
Title: Re: EM Drive Developments - related to space flight applications - Thread 2
Post by: Rodal on 03/04/2015 11:45 am
Thank you Dr. Rodal - So here is the complete set for some magnetic source run. I didn't record any details except I can see  that the antenna in in the location of the magnetic antenna I use.  Is it possible that all of the images are correct? If so that would increase my confidence in the meep output.
Yes, they can all be correct.  But we need your help in identifying the images you posted.  The electric and magnetic fields are vectors in 3-D space, with orthogonal base vector components.


For example, what does this represent? Is this a contour plot of the electric field component oriented along the axial direction  (the vector component oriented along the "y" axis)? The reason why I think it is the axial component is because the axial component should be symmetric about the "y" axis (which it is)

Notice how although you impose flat faces, the electromagnetic field inside the cavity wants to be spherical (thus the 2 curved boundaries between the 3 contour regions).  Left to its own, Nature will do what it wants to do: to propagate as spherical waves.  The radii of the 2 curved boundaries look correct.  The radii of curvature seem to have the same center as the focal point of  intersection of the sides of the truncated cone (the vertex or apex of the cone).

(http://forum.nasaspaceflight.com/index.php?action=dlattach;topic=36313.0;attach=811566;image)
Title: Re: EM Drive Developments - related to space flight applications - Thread 2
Post by: Flyby on 03/04/2015 12:12 pm
nah, it is not to be dismissed as reflection of the surroundings or floor. Reflections do no behave like that on a cylindrical object...
Believe me, you'd need a very odd shaped floor to achieve such type of reflections on a cylindrical object..

I added 2 renders that use raytracing, and from different height to catch a glimps of the bright surroundings and the wood table. No way you're getting that parallel bright edge as seen on the original picture.

there is only 1 way to explain it : it is a different, brownish material that is sandwiched between high reflective surfaces...
maybe, but highly improbable, it is a brown-ish tape at the edge...

But it is NOT a reflection.
The similarity of colour between the strip and the wood table is merely coincidence..
Title: Re: EM Drive Developments - related to space flight applications - Thread 2
Post by: Stormbringer on 03/04/2015 12:58 pm
Grenade!

http://phys.org/news/2015-03-theorist-gravitational-casimir.html

Title: Re: EM Drive Developments - related to space flight applications - Thread 2
Post by: Rodal on 03/04/2015 01:14 pm
Grenade!

http://phys.org/news/2015-03-theorist-gravitational-casimir.html
I attach the actual paper (available at arXiv) by James Quach on the Gravitational Casimir effect for those interested to read it for personal research purposes.  All others should access it through the American Physical Society at http://journals.aps.org/prl/abstract/10.1103/PhysRevLett.114.081104

@NOTSOSUREOFIT: Notice that the paper begins with a Maxwell-equation-like formulation of the linearized Einstein field equations known as gravitoelectromagnetism

Quote from: James Quach
if experiments show the Casimir pressure to be an order of magnitude larger than that predicted from the photonic contribution alone, this would be the first experimental evidence for the validity of the H-C theory and the existence of gravitons. This would open a new field in the way of graviton detection.
Title: Re: EM Drive Developments - related to space flight applications - Thread 2
Post by: Notsosureofit on 03/04/2015 01:34 pm
Grenade!

http://phys.org/news/2015-03-theorist-gravitational-casimir.html

Hmm, one might suppose that if the graviton interaction is enhanced by the superconducting currents, then perhaps the graviton interaction is also enhanced by the existence of RF induced currents ??
Title: Re: EM Drive Developments - related to space flight applications - Thread 2
Post by: aero on 03/04/2015 02:32 pm
Thank you Dr. Rodal - So here is the complete set for some magnetic source run. I didn't record any details except I can see  that the antenna in in the location of the magnetic antenna I use.  Is it possible that all of the images are correct? If so that would increase my confidence in the meep output.
Yes, they can all be correct.  But we need your help in identifying the images you posted.  The electric and magnetic fields are vectors in 3-D space, with orthogonal base vector components.


For example, what does this represent? Is this a contour plot of the electric field component oriented along the axial direction  (the vector component oriented along the "y" axis)? The reason why I think it is the axial component is because the axial component should be symmetric about the "y" axis (which it is)

Notice how although you impose flat faces, the electromagnetic field inside the cavity wants to be spherical (thus the 2 curved boundaries between the 3 contour regions).  Left to its own, Nature will do what it wants to do: to propagate as spherical waves.  The radii of the 2 curved boundaries look correct.  The radii of curvature seem to have the same center as the focal point of  intersection of the sides of the truncated cone (the vertex or apex of the cone).

(http://forum.nasaspaceflight.com/index.php?action=dlattach;topic=36313.0;attach=811566;image)

Sorry - I was tired when I posted that.

The file names give the information I have, unless the meep documentation gives more. Meep'c coordinate system is a little confusing. For 2D it is (x, y, no-size). When I place objects in the model, x is the axial direction and y is the in-plane perpendicular. There is no z in the geometry. However, the field you identified is named ez.i.jpeg which means that it is the imaginary component of the electric field in the z, or out of plane direction. Then the fields are identified from the file name as "e" for electric and "h" for magnetic and then "i" for imaginary and "r" for real components of the fields.
As the run is in 2D the other 3 fields can't be generated. That is, no ex, ey or hz.
Title: Re: EM Drive Developments - related to space flight applications - Thread 2
Post by: Rodal on 03/04/2015 02:33 pm
I'm currently trying to finish up the cross section drawing on the flight demonstrator, but i kinda fail to understand how it fits together....

Looking closely (top down wise) at the lower rim it seems to me there is :

-small shiny rim (could be the edge of the alu cone?)
-brownish plate (copper plate?)
-small shiny rim
-thick plate (most probably holding the screw thread)

I'm puzzled about the second small, shiny rim...
Why would you need an additional small slab of alu under the (supposedly) copper plating?
Any one has an idea?

Perhaps it is a compliant (less stiff than the joined flange and the thick flat plate) material to prevent leakage by providing a more uniform stress distribution in the circumferential direction (the screws otherwise produce stress concentrations).  If a gasket, the compliant material would have to satisfy the required electromagnetic properties between the joined components.

But, if the brown material serves as a gasket (whether a soft metal or another material), the flange would have to be unusually thin (which is unlikely in my opinion as it would be difficult to keep the flange flat if it would be that thin).

Perhaps the brown material is later inserted into a thin, shallow, groove in the previously joined and bolted metal flange and plate, that is only then filled with a brown (initially a viscous liquid) adhesive to prevent leakage (and metal conductivity was ensured by the fact that there is metal behind the shallow, thin, brown adhesive).

Whatever it is, it seems to be associated with preventing leaking, as there is a huge number of bolts holding the flange to the flat plate, as if somebody was concerned with leaking and wanted to ensure a uniform fit without leaking.

(http://forum.nasaspaceflight.com/index.php?action=dlattach;topic=36313.0;attach=811634;image)
Title: Re: EM Drive Developments - related to space flight applications - Thread 2
Post by: Notsosureofit on 03/04/2015 03:07 pm
FYI

https://www.dropbox.com/s/rtp9gx844yxu6ef/IMAG0372.jpg?dl=0

(http://www.koreavac.com/new/kor/images/product/comp_flange_CF_dimen.gif)

(http://www.koreavac.com/new/kor/images/product/compo_flange_CF_3.gif)

That one is a 13.25".

See also:

(http://www.idealvac.com/files/images/CF_ConicalReducerDrawing572012_1.jpg)

And wire seal dimensions:

http://www.n-c.com/resources/spec/FlangesAndFittings_WireSealFlanges.pdf

The top flange of the cone seems to fit the dimension ratio of a 12.375" (standard) wire seal flange.  The plate on the top end could be just a plate drilled to fit.

Unfortunately, if they used "standard" flanges the cone dimensions are quite a bit larger than what would seem reasonable from the image calculations and Shawyer's numbers.  The wire seal flanges from different manufacturers can vary an enormous amount.  This could easily be someone's "aerospace" version or completely nonstandard to Shawer's specs.

Title: Re: EM Drive Developments - related to space flight applications - Thread 2
Post by: Rodal on 03/04/2015 03:09 pm
FYI

https://www.dropbox.com/s/rtp9gx844yxu6ef/IMAG0372.jpg?dl=0
I see the brown reflection of the wood-like table on the left side of the plate, and the shiny rim.

Can't tell whether that is real solid wood, a thin veneer, or Formica laminate countertop   :)
Title: Re: EM Drive Developments - related to space flight applications - Thread 2
Post by: Notsosureofit on 03/04/2015 03:17 pm
Doesn't mean that it isn't a copper cone that has been plated on the outside for anti corrosion.  Often this is a Ni coating.
Title: Re: EM Drive Developments - related to space flight applications - Thread 2
Post by: frobnicat on 03/04/2015 04:45 pm

....
The only thing that I see that can account for the apparent lacking torque is the equilibrating torque induced by the inclination of the plane of rotation of the arm toward the CoM of the rotating assembly, that is equivalent to a hanging pendulum. The way it is used, the balance is more than 90% a hanging pendulum and less than 10% a torsion pendulum driven by flexure stiffness.

If this analysis holds, small changes in stiffness of flexure bearings would make for a minor impact on results. Changes in inclination would be the major way to tune the (linearised hanging pendulum equivalent) stiffness.

A hanging pendulum hangs from a rigid support located above the weight.  Its period depends only on the length of the pendulum's arm (and g , the acceleration of gravity, which is practically constant on Earth).  The flexural stiffness of the pendulum's arm is negligible.

But here nothing is hanging from the stainless steel chamber "rigid ceiling" supported by arms with negligible flexural stiffness.

What I see is the EM Drive weight supported by a frame of Faztek aluminum beams, Faztek beams that are supported from below, not from the stainless steel chamber ceiling.



What rigid support (located above the EM Drive) is the EM Drive hanging from ?  (Where is the "hanging pendulum" rigid support located ?)

What constitutes the arm of the "hanging pendulum"?  Why does it have negligible flexural stiffness? (The flexural stiffness of the aluminum Faztek beams is far from being negligible)

Do you really mean a hanging pendulum (whose period depends only on the length of the arm)?
Or do you mean a flexural pendulum (whose period depends on the stiffness of the arm)?

And if you agree that the flexural stiffness of the arms are not negligible, why take into account only the portion above the weight?  What about the flexural stiffness below the weight? 
Aren't the Faztek beams supported from below?

What I see is your "z" axis going up to a Faztek frame or "bridge" and the "bridge" being supported by two vertical Faztek beams, and those vertical Faztek beams are supported from below, not from above.

Is that then really an inverted flexural/torsional pendulum since it is made by Faztek aluminum beams supported from below?
... pics ...

It's not about faztek beams compliance. It's about introducing a tilt in the Z axis of rotation (the tilt is around the Y axis) so that the CoM Centre of Mass of the whole rotating assembly, which is not exactly on the Z axis (from the values given by Paul March) but behind the axis (X-) will be lowest when at rest equilibrium position and will have to climb the gravitational potential (ie work against) if it is to deviate from this position (by rotating around the Z axis).

This could be analysed in terms of potential energy, but my Newtonian bias is toward forces and accelerations, see attached picture : movements in a sloped plane XY with theta the angle of slope (0 for horizontal, pi/2 for vertical) and assuming the forces in Z all cancel (since trajectories don't depart from XY) and don't couple to forces on X or Y (no friction) are equivalent to movements in XY plane that would be vertical in a reduced gravity of g sin(theta). This equivalence is used for instance with inclined air cushioned tables frictionless dynamics.

So in the end we have a position restoring torque that is dcom mcom g sin(theta) sin(alpha). For small deviations alpha (and indeed we know they are small), this can be linearised
torque/alpha = dcom mcom g sin(theta)
This behaves like an added spring constant of dcom mcom g sin(theta) [Nm/rad] to the stiffness of the flexure bearings 9.06e-2 [Nm/rad]

But, there is a very strange thing happening with this line of reasoning, when I do quantitative comparison, if flexure bearings really are .007Lb-in/degree each = total 9.06e-2 Nm/rad then it is so low under the required stiffness (in some charts, only 1µm measured linear displacement for 29.1µN cal. pulses) that it would require an added stiffness equivalent (from this pseudo-hanging pendulum effect) that amounts to an inclination angle theta up to 25° (degrees). Wow ! This would certainly not have remained unnoticed in the pictures ! I'm puzzled.

@ Star-Drive : you say you haven't really controlled the amount of tilt used to "stabilize" the pendulum, can you confirm that it is indeed tilted with the frustum (X+) getting higher and the electronic stack (X-) going lower, that is when looking in front of the vacuum chamber the downward slope would be from the front to the back ? Do you have an upper bound for the slope introduced ? Do you see why there would be such a huge disparity between the low stiffness of flexure bearings given at .007 In-Lb/degree each, that would allow for a reading of .353*(29.1e-6* 0.286)/9.06e-2 = 32.4µm in response to 29.1µN cal. pulses and the apparent higher stiffness of between 1µm to 2.5µm response to cal. pulses (depending on the charts) ?

Thank you for the latest complement of informations. For confirmation : 7.64118 kg + 0.200 kg mass is the mass of the complete electronic stack on the back of pendulum's arm ? Mass of small diameter dielectric would be around 1kg ? Total mass of rotating assembly, with dielectric, would be just a bit above 25Lb ?
Title: Re: EM Drive Developments - related to space flight applications - Thread 2
Post by: sghill on 03/04/2015 04:47 pm
That thick bottom-most plate looks like its a painted rubber or silicon footplate to me.  The cracks in the paint are a dead giveaway that the thick bottom footer is some sort of flexible material that's been painted and then cracked after it flexed. 

It makes sense to have a flexible footer to avoid any sort of damage when handling the thing from time to time.

Here's an example:
Title: Re: EM Drive Developments - related to space flight applications - Thread 2
Post by: Rodal on 03/04/2015 05:08 pm
....
It's not about faztek beams compliance. It's about introducing a tilt in the Z axis of rotation (the tilt is around the Y axis) so that the CoM Centre of Mass of the whole rotating assembly, which is not exactly on the Z axis (from the values given by Paul March) but behind the axis (X-) will be lowest when at rest equilibrium position and will have to climb the gravitational potential (ie work against) if it is to deviate from this position (by rotating around the Z axis). ....
Thank you for taking the time to draw these excellent pictures.  A picture is worth 1000 words.

What compliance is responsible for the tilt in the Z axis of rotation around the Y axis) ?  You state that the tilt is not due to the Faztek beam compliance (although if the Faztek beam would be compliant enough it certainly would tilt).

If it is not due to the Faztek beam compliance, then my understanding is that you are saying that it is due to the compliance of the Riverhawk bearing.  I had interpreted what was written about the Faztek bearings as providing a clamp condition (no tilt). 

Do you have quantitative information from the Riverhawk bearing manufacturer as to what is the magnitude of the torsional stiffness for a rotation around the Y axis  provided by two Riverhawks that would allow such a tilt around the Y axis?

NOTE: if such a tilt occurs, due to compliance of the Riverhawk bearings around the Y axis, it would be analogous to a flexural pendulum, with flexural stiffness given by the magnitude of the Riverhawk torsional stiffness for a rotation around the Y axis.   
Title: Re: EM Drive Developments - related to space flight applications - Thread 2
Post by: Flyby on 03/04/2015 05:44 pm
The "Copper" is just the reflection of the floor from the side of the flange.  The 2 bright lines are reflections from the edge bevels.  If they used the Copper CF (or wire) seals they are located inside of the bolt line.

Assuming they are indeed the beveled edges, like you say, the second rim should then be the top bevel of the screw thread holding plate. That could indeed make sense then...

But as you got all the dimensions figured out already, I'll rest my case and switch back to observer modus... :-X
Title: Re: EM Drive Developments - related to space flight applications - Thread 2
Post by: Rodal on 03/04/2015 05:57 pm
Do you have a reference giving frequency, Q, power and thrust measurements for the Flight Thruster?

This is the only reference I have for data for the Flight Thruster: http://emdrive.com/flightprogramme.html,  I can't find a Q reported.

Several of them for the "Flight Thruster Programme":

Shawyer's CEAS 2009 paper (http://www.emdrive.com/CEAS2009paper.doc) stated, page 9:
Quote
The Flight thruster programme covers the design and development of a 300 Watt C Band flight thruster. This has a specified thrust of 85 mN, and a mass of 2.92Kg. Overall dimensions are 265mm diameter at the baseplate and a height of 164mm.

Then in the 2010 Toulouse TECHNO DIS paper (http://www.emdrive.com/Toulouse2010paper01.doc), page 8:
Quote
Development testing of the unit, up to a power of 600 W, is under way, and to date, has given a mean specific thrust of 330 mN/kW.
[…]
This is needed to ensure the input frequency matches the resonant frequency of the high Q (60,000) cavity, over the full input power range and the qualification temperature specification.

And in the IAC 2013 paper (http://www.emdrive.com/IAC13paper17254.v2.pdf), page 4:
Quote
The Dynamic performance of the non superconducting Flight Test model, manufactured and tested by SPR Ltd, and described in REF 3 [N.B.: 2010 Toulouse TECHNO DIS paper] was modeled with a cavity Qu = 50,000 and Fres=3.85 GHz.

Finally the mean specific thrust of 326mN/kW over 19 test runs of up to 90 secs duration from 150 W to 450 W was found on the web page http://emdrive.com/flightprogramme.html
As well as the diagram which shows the maximum thrust achieved @ 450 W:
(http://emdrive.com/images/results2.jpg)

If the 265mm base diameter that Shawyer gave does in fact refer the the exterior diameter of the base plate and if the height is measured from the yellow arrows indicated in the aforementioned photo, which is the only way I can reconcile his figures with the photo, then my best guess for the inner resonance cavity is:

224mm base diameter
145mm top diameter
164mm height (given by Shawyer).

Here's the chart if anyone is interested, cluttered though it has admittedly become.

Thanks!  That brings that X number down to ~55.  Still a very high mode, but 3.85GHz !


McCulloch's equation gives 148 milliNewtons for Shawyer's demo, comparing pretty well with Shawyer's reported measurements of 80 to 214 millinNewtons, see:

http://physicsfromtheedge.blogspot.com/2015/02/mihsc-vs-emdrive-data-3d.html

I understand that you need to use X=26 in your equation for Shawyer's Demo, also a high mode.

On the other's hand McCulloch's equation is off by a factor >10 for NASA Eagleworks test results.

That is an interesting fit.

Of course I havn't seen Mike's derivation in 3D.  I only follow the Equivalence argument the best I can w/o fudge factors and see what comes out.  At the moment I think these are maximums if you have all the parameters and I like the NASA results because they seem to have eliminated more sources of error.  Still, it may all be fiction which is what we want to find out.

@ RODAL

Still have question about the Shawyer "Demo" cavity w/ 174mN.  What are the current estimates of the cone dimensions, frequency (3.85GHz?), and Q (6000 est?).  When I put in TM02 and 450W, I get 174.8microN, rather than the 174milliN reported.  I would like to recheck those numbers.

Even w/ Q=45000, I need to get X up around (65 Very high mode) to get those numbers.  Is that possible w/ 3.85GHz ??

Thanks

Using the geometrical dimensions for Shawyer's flight thruster:

Big Diameter = 224 mm
Small Diameter = 145 mm
Axial Length = 164 mm height (given by Shawyer)

Having successfully compared my exact solution results with the COMSOL's Finite Element Analysis by NASA and thermal IR camera imaging results  and to the exact solution results of Greg Egan, I now calculate some Mode Shapes and associated frequencies (in vacuum with NO dielectric) for Shawyer's flight thruster, for comparison with @Notsosureofit's formula

TM551 4.10811 GHz
TM552 4.76536 GHz
TM553 5.25893 GHz

TM441 3.57892 GHz
TM442 4.20942 GHz
TM443 4.67270 GHz

TM331 3.03740 GHz
TM332 3.63559 GHz
TM333 4.05446 GHz

TM011 1.23005 GHz
TM012 1.63637 GHz
TM013 2.24872 GHz

And for @Notsosureofit's comparison, here are the first m to 10, n to 5 Bessel zeros and Bessel derivative zeros up to 15 digits:  http://wwwal.kuicr.kyoto-u.ac.jp/www/accelerator/a4/besselroot.htmlx, therefore we get:

TM55 --> X55=22.2177998965612
TM44 --> X44=17.6159660498048
TM33 --> X33=13.0152007216984
Title: Re: EM Drive Developments - related to space flight applications - Thread 2
Post by: frobnicat on 03/04/2015 06:36 pm
....
It's not about faztek beams compliance. It's about introducing a tilt in the Z axis of rotation (the tilt is around the Y axis) so that the CoM Centre of Mass of the whole rotating assembly, which is not exactly on the Z axis (from the values given by Paul March) but behind the axis (X-) will be lowest when at rest equilibrium position and will have to climb the gravitational potential (ie work against) if it is to deviate from this position (by rotating around the Z axis). ....
Thank you for taking the time to draw these excellent pictures.  A picture is worth 1000 words.

What compliance is responsible for the tilt in the Z axis of rotation around the Y axis) ?  You state that the tilt is not due to the Faztek beam compliance (although if the Faztek beam would be compliant enough it certainly would tilt).

If it is not due to the Faztek beam compliance, then my understanding is that you are saying that it is due to the compliance of the Riverhawk bearing.  I had interpreted what was written about the Faztek bearings as providing a clamp condition (no tilt). 

Do you have quantitative information from the Riverhawk bearing manufacturer as to what is the magnitude of the torsional stiffness for a rotation around the Y axis  provided by two Riverhawks that would allow such a tilt around the Y axis?

NOTE: if such a tilt occurs, due to compliance of the Riverhawk bearings around the Y axis, it would be analogous to a flexural pendulum, with flexural stiffness given by the magnitude of the Riverhawk torsional stiffness for a rotation around the Y axis.

And a thousand English words is a lot of sweat for me  :)
To answer your questions : while I consider looking for compliance aspects, both for faztek structural elements and for the bearings, there is no compliance implied by my latest post, the tilt would be voluntarily introduced for the whole experiment platform, including the bearing supporting fixed parts (the supporting axis is tilted, not just the arm or arm's rotation axis). That's how I read the answer of Star-Drive :

...
While I'm at it : is the plane in which the arm rotates kept as horizontal as possible (ie the axis of rotation as vertical as possible) or is there a small slope voluntarily introduced leading to some pendulum effect against g (for stabilisation or tuning purpose) ? That could explain the varying deviation (in µm) for the same calibration pulses thrusts. Also wondered if this is what was implied in this post (http://forum.nasaspaceflight.com/index.php?topic=36313.msg1331580#msg1331580) :
Quote from: Star-Drive
...
These thermally induced actions to the left requires the torque pendulum's arm to move to the right to maintain the balance of the torque pendulum's arm in the lab's 1.0 gee gravity field, since we also use the Earth's g-field to help null the pendulum's movements.
...

...
The design of our Torque pendulum follows what JPL and Busek Co did at their respective facility, see attached report from Busek.  We found that if we tried to keep the arm completely horizontal though that the pendulum's neutral point would wonder erratically and make alignments near impossible.  So yes I balance the pendulum arm so there is always a slight tilt in it, however this tilt angle magnitude is not controlled as well as it probably should.

Best, Paul M.

Just tilting the arm relative to a still vertical axis Z would introduce no stabilization, for stabilization the axis of rotation itself has to be tilted (to some degree) at the level of the fixed support. I suppose this is done by raising or lowering the 3 (?) base damped platforms against which the whole experiment weight rests. Hope I'm not misinterpreting...
Title: Re: EM Drive Developments - related to space flight applications - Thread 2
Post by: Rodal on 03/04/2015 06:46 pm
....
It's not about faztek beams compliance. It's about introducing a tilt in the Z axis of rotation (the tilt is around the Y axis) so that the CoM Centre of Mass of the whole rotating assembly, which is not exactly on the Z axis (from the values given by Paul March) but behind the axis (X-) will be lowest when at rest equilibrium position and will have to climb the gravitational potential (ie work against) if it is to deviate from this position (by rotating around the Z axis). ....
Thank you for taking the time to draw these excellent pictures.  A picture is worth 1000 words.

What compliance is responsible for the tilt in the Z axis of rotation around the Y axis) ?  You state that the tilt is not due to the Faztek beam compliance (although if the Faztek beam would be compliant enough it certainly would tilt).

If it is not due to the Faztek beam compliance, then my understanding is that you are saying that it is due to the compliance of the Riverhawk bearing.  I had interpreted what was written about the Faztek bearings as providing a clamp condition (no tilt). 

Do you have quantitative information from the Riverhawk bearing manufacturer as to what is the magnitude of the torsional stiffness for a rotation around the Y axis  provided by two Riverhawks that would allow such a tilt around the Y axis?

NOTE: if such a tilt occurs, due to compliance of the Riverhawk bearings around the Y axis, it would be analogous to a flexural pendulum, with flexural stiffness given by the magnitude of the Riverhawk torsional stiffness for a rotation around the Y axis.

And a thousand English words is a lot of sweat for me  :)
To answer your questions : while I consider looking for compliance aspects, both for faztek structural elements and for the bearings, there is no compliance implied by my latest post, the tilt would be voluntarily introduced for the whole experiment platform, including the bearing supporting fixed parts (the supporting axis is tilted, not just the arm or arm's rotation axis). That's how I read the answer of Star-Drive :....


OK, let's say that you tilt the axis Z axis by rotating it around the Y axis, and that you meet no resistance torque whatsoever in doing so.... Mmmmm... hard to believe particularly when we are discussing microNewton forces (less than the weight of a grain of sand).... hard to believe that there is no torque resistance

But for discussion's sake, if there is no resistance met in rotating the Z axis around the Y axis as you posit, what prevents the axis from continuously rotating to an arbitrary angle (if there is no resistance) ?

If it is posited that it is "balanced" ....Mmmmm.... it maybe balanced statically, but is it stable ?
is the position neutrally stable?
and what about dynamics?  wouldn't you get dynamic oscillations? maybe that's what you meant by hanging pendulum?
and how do we know whether it is dynamically stable ?  (it maybe statically neutrally stable but dynamically unstable)

If this is the reason for the baseline's slope, why does it slowly creep ? what governs the speed of movement of the slope? is there damping of this rotation? what is the source of this rotation's damping?

Is it being proposed that pendulum oscillations for Z axis rotation around the Y axis that are only governed by the length of the arm ?

Would like to have more input from Paul March about this....QUESTION TO PAUL: what keeps the Z axis stable at a given angle?  Is it just weight distribution? Or is there a stiffness associated with it?. Is there damping associated with this rotation? if so, what is the source of the damping?   Why is it that the Riverhawk bearings don't prevent this rotation? Can one adjust the Riverhawk bearing settings to decrease or eliminate this movement?  Can one increase the stiffness of the Riverhawk bearings to accomplish this?
Title: Re: EM Drive Developments - related to space flight applications - Thread 2
Post by: frobnicat on 03/04/2015 08:46 pm
@ Rodal :
See attached picture to share my mental image. Tilt over-exaggerated for illustration.

Grey : solid rotating assembly (no deformation implied)
Orange/brown : fixed assembly (no deformation implied)
Blue : the ground slab of the vacuum chamber (no deformation implied)

For now, assume a perfect axis of rotation around Z : only one degree of freedom of Grey relative to Orange, the "official" rotation around Z, no compliance implied, Grey kept in the XY plane, plane has same tilt as Orange (XY parallel to Orange platform).

@Star-Drive
Can you confirm this is a correct way to understand that there is a tilt in the axis of rotation ?
Title: Re: EM Drive Developments - related to space flight applications - Thread 2
Post by: frobnicat on 03/04/2015 10:53 pm
Follow-up on the apparent higher stiffness of balance (displacement against a given force) than due to the tandem of .007 Lb-in/degree flexure bearings. One set of robust measures are the mass distributions (as per latest Star-Drive precisions (http://forum.nasaspaceflight.com/index.php?topic=36313.msg1341256#msg1341256)) and the oscillation pseudo-period of around 4.5s clearly visible in Fig. 19 page 20 of Brady et al anomalous... (http://www.libertariannews.org/wp-content/uploads/2014/07/AnomalousThrustProductionFromanRFTestDevice-BradyEtAl.pdf)

All those figures show a linear displacement reading of approximately 1µm for the calibration pulses at 29.1µN, some other charts show between 1µm to 2.5µm for the same calibration force, the charts like those of fig. 19 are the "stiffer" to explain, but the following argument does not depend on this apparent sensitivity (ie would be the same even if calibrations pulses and reading magnitudes where completely off scale)

If we assume the pseudo-period of 4.5s as the result of a underdamped rotating harmonic motion (around Z), the same period (well, almost the same) could be observed from an undamped rotating harmonic motion :
 I * alpha_dot_dot= -k * alpha  where the right term is a linear restoring torque (proportional to angular deviation alpha) and I is the moment of inertia around Z. Pulsation omega of that harmonic oscillator is such that  omega = sqrt(k/I) = 2pi/T   or T (period) = 2pi sqrt(I/k)   but since I'm interested in the value of k (stiffness around Z)   k = 4 pi² I / T²

Rough estimation of I around Z axis (assuming frustum and electronic stack as punctual mass and faztek arm as a thin element) :
Frustum with dielectric : 2.60*0.286² =   8.18e-2
Front Faztek arm part : 0.567*0.375²/3 = 2.66e-2
Back Electronic stack : 7.84*0.190² =   3.61e-2
Back Faztek arm part : 0.356*0.235²/3 =0.65e-2
Total I=0.151 kg m²

with T=4.5s  =>  k = 0.294  Nm/rad

Independently : for a deflection of 1µm @ .353m (LDS distance from Z) that is 2.83e-6 rad,  as a result of 29.1µN @ .286m (cal. force distance from Z) that is 8.32e-6 Nm, needs a restoring torque with a stiffness of 8.32e-6/2.83e-6 = 2.94 Nm/rad

Surprisingly 10 times more than the previous. Have I skipped a power of 10 somewhere ??

And two .007 Lb-in/degree flexure bearings make for only 0.0906 Nm/rad

I'm lost in conjectures...
Title: Re: EM Drive Developments - related to space flight applications - Thread 2
Post by: Notsosureofit on 03/04/2015 11:50 pm
 Trying to post emdrive5.xls
Title: Re: EM Drive Developments - related to space flight applications - Thread 2
Post by: Rodal on 03/05/2015 12:13 am
Trying to post emdrive5.xls

I yellowed the frequencies closest to operating frequency (reported as 3.85GHz ) showing @Notsosureofit's formular results from 19 to 52 mN/kW for Shawyer's Flight Thruster

Shawyer reported 326mN/kW.
Title: Re: EM Drive Developments - related to space flight applications - Thread 2
Post by: Notsosureofit on 03/05/2015 12:35 am
The flight thruster is shown as what appears to be an oscillator configuration, so it is still possible that the loaded Q is higher than the unloaded Q.  But that is speculation.  I do worry about his concept of "no static thrust" as well.

(and yes the simple dispersion relation drops dependence on "p".  Maybe we can get that in there w/ your exact solution ??)
Title: Re: EM Drive Developments - related to space flight applications - Thread 2
Post by: Rodal on 03/05/2015 12:39 am
The flight thruster is shown as what appears to be an oscillator configuration, so it is still possible that the loaded Q is higher than the unloaded Q.  But that is speculation.  I do worry about his concept of "no static thrust" as well.

(and yes the simple dispersion relation drops dependence on "p".  Maybe we can get that in there w/ your exact solution ??)
Shawyer's EM Drive was never tested in a vacuum as far as I know.  One wonders whether thermal effects (convection current generation) are very significant for Shawyer's and the Chinese tests due to the very large power input, in other words: is a big portion of Shawyer's (and the Chinese) thrust measurement due to thermal effects ?

Question: your expression has mode-dependence (unlike Shawyer's and McCulloch's expressions which have no mode dependence).  Why is there a mode dependence, and why does it depend only on the circular cross-section quantum numbers "m" and "n" but it does not depend on the axial quantum number "p"?
Title: Re: EM Drive Developments - related to space flight applications - Thread 2
Post by: Rodal on 03/05/2015 12:49 am
...

(and yes the simple dispersion relation drops dependence on "p".  Maybe we can get that in there w/ your exact solution ??)
I understand that your expression is based on the exact solution for a cylindrical cavity.  Is there a reason why a cylindrical cavity thrust should depend on the cross-sectional quantum numbers "m" and "n" but not depend on the axial quantum number "p"?
The exact solution for the cylindrical cavity has a frequency dependent on "p".  Why isn't thrust dependent on "p" ?
Title: Re: EM Drive Developments - related to space flight applications - Thread 2
Post by: Notsosureofit on 03/05/2015 12:51 am
The mode dependence is already there in the waveguide dispersion relation.  "p" drops out when you evaluate it at the k values of each end.  That's the simple case.  Higher terms in a more exact solution (or a dielectric solution) need not do that.
Title: Re: EM Drive Developments - related to space flight applications - Thread 2
Post by: Notsosureofit on 03/05/2015 12:54 am
...

(and yes the simple dispersion relation drops dependence on "p".  Maybe we can get that in there w/ your exact solution ??)
I understand that your expression is based on the exact solution for a cylindrical cavity.  Is there a reason why a cylindrical cavity thrust should depend on the cross-sectional quantum numbers "m" and "n" but not depend on the axial quantum number "p"?
The exact solution for the cylindrical cavity has a frequency dependent on "p".  Why isn't thrust dependent on "p" ?

Thrust for a completely cylindrical cavity is zero, all the terms cancel out.  "p" cancels out in the simple tapered cavity, but that is not an exact solution.

[now if you were to calculate the thrust of a cylindrical cavity in an accelerated frame of reference you would have an exact solution but the "thrust" would oppose the acceleration, ie. Nhfg/c^2]
Title: Re: EM Drive Developments - related to space flight applications - Thread 2
Post by: Rodal on 03/05/2015 12:59 am
...

(and yes the simple dispersion relation drops dependence on "p".  Maybe we can get that in there w/ your exact solution ??)
The problem is that the exact solution is not closed-form: it requires the solution of two eigenvalue problems which have no closed-form solution.

Without a closed-form solution for the frequency and mode shape for a truncated cone there is no hope of a closed-form solution for the thrust.

Maybe when I have the time I can come up with a closed-from piecewise approximation (comprising several simple piecewise functions in subregions that can be spliced together) that can closely approximate the exact solution. 
Title: Re: EM Drive Developments - related to space flight applications - Thread 2
Post by: ThinkerX on 03/05/2015 01:07 am
So, then in order to produce 'thrust,' this device MUST be a truncated cone - preferably with a rounded base?

Also a wild and stupid thought about Sawyer's commentary:  maybe he envisions this device as some sort of 'turbo-charger?'   That is something that adds to an already existing velocity, but won't function well, if at all when 'at rest.'  Say you have a spacecraft in motion at speed X.  Flip this drive on, and the speed becomes X+1. 
Title: Re: EM Drive Developments - related to space flight applications - Thread 2
Post by: Notsosureofit on 03/05/2015 01:10 am
So, then in order to produce 'thrust,' this device MUST be a truncated cone - preferably with a rounded base?

Also a wild and stupid thought about Sawyer's commentary:  maybe he envisions this device as some sort of 'turbo-charger?'   That is something that adds to an already existing velocity, but won't function well, if at all when 'at rest.'  Say you have a spacecraft in motion at speed X.  Flip this drive on, and the speed becomes X+1.

The device must have an asymmetric dispersion relation in some axis.  A truncated cone is one example.

I havn't been able to get my head around the velocity based explanations.
Title: Re: EM Drive Developments - related to space flight applications - Thread 2
Post by: Rodal on 03/05/2015 01:23 am
...maybe he envisions this device as some sort of 'turbo-charger?'   That is something that adds to an already existing velocity, but won't function well, if at all when 'at rest.'  ...
A turbocharger, is a device that increases an engine's efficiency and power by forcing extra air into the combustion chamber. 

What is the "engine" that the EM Drive is turbocharging?

The bizarre nature of something needing to be free to accelerate for it to produce a force doesn't apply to the turbocharger or to the engine:  the engine that is being turbocharged does not need to be accelerating or even be in rigid body motion. Its center of mass can be completely stationary, and the turbocharged engine can then be used for electric power generation, for example, instead of for transporting people as in an automobile.
Title: Re: EM Drive Developments - related to space flight applications - Thread 2
Post by: Rodal on 03/05/2015 01:43 am
...
(and yes the simple dispersion relation drops dependence on "p".  Maybe we can get that in there w/ your exact solution ??)

...
The device must have an asymmetric dispersion relation in some axis.

.....
I think I might be able to obtain a closed-form expression for the frequency of a cylindrical cavity with a dielectric at one end. (Or really a cylindrical cavity having two dielectrics in contact with each other: one dielectric at one end and another dielectric with different permitivity and permeability at the other end).

Would that be of any use to you to improve your equation?
Title: Re: EM Drive Developments - related to space flight applications - Thread 2
Post by: zen-in on 03/05/2015 03:21 am
@ Rodal :
See attached picture to share my mental image. Tilt over-exaggerated for illustration.

Grey : solid rotating assembly (no deformation implied)
Orange/brown : fixed assembly (no deformation implied)
Blue : the ground slab of the vacuum chamber (no deformation implied)

For now, assume a perfect axis of rotation around Z : only one degree of freedom of Grey relative to Orange, the "official" rotation around Z, no compliance implied, Grey kept in the XY plane, plane has same tilt as Orange (XY parallel to Orange platform).

@Star-Drive
Can you confirm this is a correct way to understand that there is a tilt in the axis of rotation ?

One aspect of this tilt in the axis of rotation that hasn't been discussed is the requirement for imbalance.   The difference in moments on either side of the beam only has to be very slight for it to always come to rest at the same location +/- a micron or two.

A small CW tilt along the X axis of the balance arm would explain the apparent drift in the baseline seen in some of the thrust waveforms.   When the cavity has the orientation shown below its CM shifts to the left.  This would reduce the tilt, resulting in an increase in brightness of the reflected light the LDS measures; due to the mirror position being closer to an optimal perpendicular position wrt the light beam.   The increase in brightness corresponds to a decrease in distance; hence the negative slope.   With the device mounted the other way the shift in CM increases the tilt.  This reduces the reflected light and is registered as an increase in distance.   No actual motion of the beam occurs.  This apparent motion is an optical artifact.   This assumes the Philtec distance sensor is used on the far side.   If it is used on the near side a small CCW tilt along the X axis of the beam would produce the same effect; except requiring much less rotation from the change in CM.
Title: Re: EM Drive Developments - related to space flight applications - Thread 2
Post by: Star-Drive on 03/05/2015 03:56 am
@ Rodal :
See attached picture to share my mental image. Tilt over-exaggerated for illustration.

Grey : solid rotating assembly (no deformation implied)
Orange/brown : fixed assembly (no deformation implied)
Blue : the ground slab of the vacuum chamber (no deformation implied)

For now, assume a perfect axis of rotation around Z : only one degree of freedom of Grey relative to Orange, the "official" rotation around Z, no compliance implied, Grey kept in the XY plane, plane has same tilt as Orange (XY parallel to Orange platform).

@Star-Drive
Can you confirm this is a correct way to understand that there is a tilt in the axis of rotation ?


Yes there is a tilt in the torque pendulum's (TP) axis of rotation with the RF power supply lower than the test article as you have it shown in your schematic diagram.  Using my 24" long level I use about a 1/4 bubble of tilt.  From that point on its a matter of fine tuning the TP response by adding or removing small weights around the test article and/or adjusting the tilt angle with a micrometer adjusting the length of the TP support under the test article.

Best,  Paul M. 
Title: Re: EM Drive Developments - related to space flight applications - Thread 2
Post by: ThinkerX on 03/05/2015 06:37 am
Ok, its late...but...
Quote
What is the "engine" that the EM Drive is turbocharging?

The bizarre nature of something needing to be free to accelerate for it to produce a force doesn't apply to the turbocharger or to the engine:  the engine that is being turbocharged does not need to be accelerating or even be in rigid body motion. Its center of mass can be completely stationary, and the turbocharged engine can then be used for electric power generation, for example, instead of for transporting people as in an automobile.

Maybe what initially sets the spacecraft in motion doesn't matter.  What matters is that the craft is in motion (inertial radiation/force as per Doctor McCulloch's theory, maybe).  Maybe this device acts as a sort of amplifier/concentrator for that force when in operation.  But if inertial force/radiation is absent, the device either doesn't work, or works poorly because there is nothing to amplify/concentrate.

Like I said, a wild and stupid idea. 
Title: Re: EM Drive Developments - related to space flight applications - Thread 2
Post by: Notsosureofit on 03/05/2015 11:13 am
...
(and yes the simple dispersion relation drops dependence on "p".  Maybe we can get that in there w/ your exact solution ??)

...
The device must have an asymmetric dispersion relation in some axis.

.....
I think I might be able to obtain a closed-form expression for the frequency of a cylindrical cavity with a dielectric at one end. (Or really a cylindrical cavity having two dielectrics in contact with each other: one dielectric at one end and another dielectric with different permitivity and permeability at the other end).

Would that be of any use to you to improve your equation?

Absolutely !  That should be a very interesting case as it is 2 coupled cavities.  Most of these kind of problems use iterative solutions but I think that case may have some unique analytic solutions !
 
Title: Re: EM Drive Developments - related to space flight applications - Thread 2
Post by: Rodal on 03/05/2015 01:35 pm
...
(and yes the simple dispersion relation drops dependence on "p".  Maybe we can get that in there w/ your exact solution ??)

...
The device must have an asymmetric dispersion relation in some axis.

.....
I think I might be able to obtain a closed-form expression for the frequency of a cylindrical cavity with a dielectric at one end. (Or really a cylindrical cavity having two dielectrics in contact with each other: one dielectric at one end and another dielectric with different permitivity and permeability at the other end).

Would that be of any use to you to improve your equation?

Absolutely !  That should be a very interesting case as it is 2 coupled cavities.  Most of these kind of problems use iterative solutions but I think that case may have some unique analytic solutions !
Yes, it is amenable to a closed-form solution since it involves a quartic: 4th order polynomial.  OK, exact, closed-form solution for a cylindrical cavity containing two coupled dielectrics coming your way soon, courtesy of Rodal-exact-solutions.  I have the solution now for the 4 roots in general.  Just have to figure out which root is the real root for which cases.

Solution will contain SquareRoots and our friends Xmn and X'mn the Bessel and Bessel Prime zeros. :)
Title: Re: EM Drive Developments - related to space flight applications - Thread 2
Post by: frobnicat on 03/05/2015 05:54 pm
@ Rodal :
See attached picture to share my mental image. Tilt over-exaggerated for illustration.

Grey : solid rotating assembly (no deformation implied)
Orange/brown : fixed assembly (no deformation implied)
Blue : the ground slab of the vacuum chamber (no deformation implied)

For now, assume a perfect axis of rotation around Z : only one degree of freedom of Grey relative to Orange, the "official" rotation around Z, no compliance implied, Grey kept in the XY plane, plane has same tilt as Orange (XY parallel to Orange platform).

@Star-Drive
Can you confirm this is a correct way to understand that there is a tilt in the axis of rotation ?

(http://forum.nasaspaceflight.com/index.php?action=dlattach;topic=36313.0;attach=811757;image)

One aspect of this tilt in the axis of rotation that hasn't been discussed is the requirement for imbalance.   The difference in moments on either side of the beam only has to be very slight for it to always come to rest at the same location +/- a micron or two.

Yes, a difference in centre of mass can make long lasting difference in position. The following statement tries to summarize rigorously :
Statement A : Any shift in centre of mass of a part (relative to fixation to the balance arm) along a direction orthogonal to a compliant (not infinite stiffness) axis that is not strictly vertical can induce a change in angular rest position around said compliant axis.

This can be quite important as a (thermal) longitudinal (Y+ or Y-) shift in CoM of the frustum assembly would be orthogonal to axis Z, but so long as this most compliant by far axis Z (the natural axis of the pendulum) was believed to be strictly vertical there would be no change in angular rest position of the arm (around Z). Only transient angular positions shifts could be induced by such thermal CoM shifts, the arm position (around Z) would be quickly enough restored to unmodified rest position (return to baseline) by the spring restoring torque of flexure bearings, and then time of ~45s during which a sustained displacement where recorded could be used as an effective argument that this couldn't be due to thermal expansions alone.

Note that now that the Z axis is no longer vertical the argument is no longer valid.

Quote
A small CW tilt along the X axis of the balance arm would explain the apparent drift in the baseline seen in some of the thrust waveforms.   When the cavity has the orientation shown below its CM shifts to the left.  This would reduce the tilt, resulting in an increase in brightness of the reflected light the LDS measures; due to the mirror position being closer to an optimal perpendicular position wrt the light beam.   The increase in brightness corresponds to a decrease in distance; hence the negative slope.   With the device mounted the other way the shift in CM increases the tilt.  This reduces the reflected light and is registered as an increase in distance.   

Is this what you are saying ?
(http://forum.nasaspaceflight.com/index.php?action=dlattach;topic=36313.0;attach=809643;image)

Indeed the X axis is not vertical (it is horizontal, or almost), so the above statement A does apply if we consider rotations around X are not against infinite stiffness. But preliminary rough estimations gave me relative displacements of rest optical length of at least one order of magnitude below signal. Even if not infinitely stiff, the added torsion compliance of faztek beam around X and compliance of tandem flexure bearings around X is not enough. For reasonable thermal CoM's shifts, this is stiff enough , and the optical lever is small enough (optical length d measured at small distance below X) that it would make very little contribution to the signal.

Quote
No actual motion of the beam occurs.  This apparent motion is an optical artifact.   This assumes the Philtec distance sensor is used on the far side.   If it is used on the near side a small CCW tilt along the X axis of the beam would produce the same effect; except requiring much less rotation from the change in CM.

For this twist around X effect, no actual rotation of the beam around Z would occur, but the beam is deformed. I wouldn't qualify that as an "optical artifact". Sorry I'm becoming quite finicky on wordings lately  :)  That would be a real mechanical motion, only not a motion around the "official" Z axis.

We have all reasons to believe the Philtec linear sensor is used in the far range, see attached chart's horizontal units. The initial report (anomalous...) clearly states that it is used around 500µm, the vertical readings of the charts are consistent, and Paul March confirms :
...
- That the vertical scale in the charts (indicated in µm, around 500) are relevant or not relevant.
...
...
The Philtec D63 fiber-optic displacement sensor measures distance from its target mirror in microns, so the numbers on the left hand side of the force plots measure the distance from the end of the fiber-optic laser head to its mirror target mounted on the torque pendulum arm.  The data sheet for same is attached.
...

Still, there is a factor 10 disparity between the apparent stiffness readings (LDS readings against calibrations pulses alone) and the stiffness needed to explain the 4.5s pseudo-period of oscillations, so this can put a doubt on the readings of vertical scale, a factor 10 here could explain a lot of things.

Regardless of this nagging problem of late, now that we know that Z deviates from strict verticality by a quarter bubble, from statement A above there is now a real possibility that a thermal CoM shift along Y changes angular rest position around the official Z, of relative magnitude compatible with signals, as rotation around Z has a very low stiffness (the restoring torque of flexure bearing around their natural axis of rotation). And such change in angular rest position around Z would record as false "sustained thrusts" in the charts.

Statement B : The validity of real sustained thrust signals now rests on the amount of deviation of Z from vertical, resolution of the contradictions between apparent stiffness around Z, and careful assessment of thermal expansions in vacuum. The sustained duration alone no longer suffice.
Title: Re: EM Drive Developments - related to space flight applications - Thread 2
Post by: Rodal on 03/05/2015 06:52 pm
EXACT, CLOSED-FORM SOLUTION FOR A CYLINDRICAL CAVITY ELECTROMAGNETIC RESONATOR CONTAINING TWO COUPLED ADJOINING DIELECTRICS FILLING THE CAVITY  by Jose' J. Rodal, Ph.D.

There are two positive roots for the frequency fm,n,p to consider, if root1 is real take frequency = root1,  otherwise take root2 (if it is real).  If both roots are imaginary, there is no resonance.  Remember to use the cutt-off frequency rule to cut-off frequencies whose wavelength do not fit.

fm,n,p  = root1 if root1 and root2 are both real numbers,
fm,n,p  = root2 if root1 is not a real number, and root 2 is a real number
fm,n,p  = 0 if neither root1 and root2 are real numbers



root1= Sqrt[((-4*b^2*(dDielectric1 - dDielectric2)*(dDielectric1 + dDielectric2)*(dielectricLength1/cMedium1 - dielectricLength2/cMedium2)*(dielectricLength1/cMedium1 + dielectricLength2/cMedium2))/
     (dDielectric1^2*dDielectric2^2) + (dielectricLength1^2/cMedium1^2 + dielectricLength2^2/cMedium2^2)*p^2 -
    2*Sqrt[(-4*b^2*(dielectricLength1/cMedium1 - dielectricLength2/cMedium2)*(dielectricLength1/cMedium1 + dielectricLength2/cMedium2)*((dDielectric1*dielectricLength1)/cMedium1 - (dDielectric2*dielectricLength2)/cMedium2)*
         ((dDielectric1*dielectricLength1)/cMedium1 + (dDielectric2*dielectricLength2)/cMedium2)*p^2)/(dDielectric1^2*dDielectric2^2) + (dielectricLength1^2*dielectricLength2^2*p^4)/(cMedium1^2*cMedium2^2)])/
   (dielectricLength1^2/cMedium1^2 - dielectricLength2^2/cMedium2^2)^2]/2


root2= Sqrt[((-4*b^2*(dDielectric1 - dDielectric2)*(dDielectric1 + dDielectric2)*(dielectricLength1/cMedium1 - dielectricLength2/cMedium2)*(dielectricLength1/cMedium1 + dielectricLength2/cMedium2))/
     (dDielectric1^2*dDielectric2^2) + (dielectricLength1^2/cMedium1^2 + dielectricLength2^2/cMedium2^2)*p^2 +
    2*Sqrt[(-4*b^2*(dielectricLength1/cMedium1 - dielectricLength2/cMedium2)*(dielectricLength1/cMedium1 + dielectricLength2/cMedium2)*((dDielectric1*dielectricLength1)/cMedium1 - (dDielectric2*dielectricLength2)/cMedium2)*
         ((dDielectric1*dielectricLength1)/cMedium1 + (dDielectric2*dielectricLength2)/cMedium2)*p^2)/(dDielectric1^2*dDielectric2^2) + (dielectricLength1^2*dielectricLength2^2*p^4)/(cMedium1^2*cMedium2^2)])/
   (dielectricLength1^2/cMedium1^2 - dielectricLength2^2/cMedium2^2)^2]/2

where we define the following dimensionless ratios as follows:

dDielectric1 = diameter/dielectricLength1;
dDielectric2 = diameter/dielectricLength2;

and where

dielectricLength1 = length - dielectricLength2;

where length is the total internal length of the cylindrical cavity.

The quantity "b" is defined as follows:

b := If[modetype == "TM", xbesselzeros[[m + 1, n]]/Pi,  If[modetype == "TE", xprimebesselzeros[[m + 1, n]]/Pi]]

Where  Xmn=xbesselzeros[[m+1,n]] (the zeros of the Bessel function)

xbesselzeros = {{2.40483, 5.52008, 8.65373, 11.7915, 14.9309}, {3.83171, 7.01559,
  10.1735, 13.3237, 16.4706}, {5.13562, 8.41724, 11.6198, 14.796,
  17.9598}, {6.38016, 9.76102, 13.0152, 16.2235, 19.4094}, {7.58834,
  11.0647, 14.3725, 17.616, 20.8269}, {8.77148, 12.3386, 15.7002,
  18.9801, 22.2178}, {9.93611, 13.5893, 17.0038, 20.3208,
  23.5861}, {11.0864, 14.8213, 18.2876, 21.6415, 24.9349}, {12.2251,
  16.0378, 19.5545, 22.9452, 26.2668}, {13.3543, 17.2412, 20.807,
  24.2339, 27.5837}, {14.4755, 18.4335, 22.047, 25.5095, 28.8874}}

and where  X'mn=xprimebesselzeros [[m+1,n]] (the zeros of the derivative of the Bessel function)

xprimebesselzeros = {{3.83171, 7.01559, 10.1735, 13.3237, 16.4706}, {1.84118, 5.33144,
  8.53632, 11.706, 14.8636}, {3.05424, 6.70613, 9.96947, 13.1704,
  16.3475}, {4.20119, 8.01524, 11.3459, 14.5858, 17.7887}, {5.31755,
  9.2824, 12.6819, 15.9641, 19.196}, {6.41562, 10.5199, 13.9872,
  17.3128, 20.5755}, {7.50127, 11.7349, 15.2682, 18.6374,
  21.9317}, {8.57784, 12.9324, 16.5294, 19.9419, 23.2681}, {9.64742,
  14.1155, 17.774, 21.2291, 24.5872}, {10.7114, 15.2867, 19.0046,
  22.5014, 25.8913}, {11.7709, 16.4479, 20.223, 23.7607, 27.182}}

For example, for mode TE01, b= xprimebesselzeros[[1,1]]/Pi= 3.83171 / Pi =  3.83171 / 3.14159




NUMERICAL EXAMPLE: we take the case used by @aero, containing a dielectric2 of HD PE material with dielectric constant (relative electric permitivity) = 2.3, and a dielectric1 being the empty portion of the cavity, under vacuum:

diameter = 0.08278945 meter;
length =  0.1224489 meter;
dielectricLength2 = 0.027282494103102 meter;
cMedium1 = cVacuum; cVacuum = 299792458 meter/second;
cMedium2 = cVacuum/Sqrt[2.3] ;  (relative electric permittivity=2.3;relative magnetic permeability=1)



Results:

modetype = "TE"; m = 1; n = 1; p = 1; root1 = 2.26774 GHz
modetype = "TE"; m = 1; n = 1; p = 2; root1 = 2.93557 GHz
modetype = "TM"; m = 0; n = 1; p = 2; root1= 3.37114 GHz

For this case, root1 is real, there is no need to consider root2. A number of modes are cut-off, for example modes TE011 and TM011 are cut-off
Title: Re: EM Drive Developments - related to space flight applications - Thread 2
Post by: Notsosureofit on 03/05/2015 08:52 pm
@ RODAL

Can you check this ?  (working it down)

root1= Sqrt[(

       (-4*b^2*(dD1^2-dD2^2)*((L1/c1)^2-(L2/c2)^2))
       / (dD1^2*dD2^2)
 
       + ((L1/c1)^2+(L2/c2)^2)*p^2

       - 2*Sqrt[

       (-4*b^2*((L1/c1)^2-(L2/c2)^2)*((dD1*L1/c1)^2-(dD2*L2/c2)^2)*p^2)
       / (dD1^2*dD2^2)

       + (L1^2*L2^2*p^4)
       / (c1^2*c2^2)])  >>>>>>>>>>>>>>>>>>>>>>>>>>>>> )] ??

       / (L1^2/c1^2-L2^2/c2^2)^2]/2


root2= Sqrt[(

       (-4*b^2*(dD1^2-dD2^2)*((L1/c1)^2-(L2/c2)^2))
       / (dD1^2*dD2^2)

       + ((L1/c1)^2+(L2/c2)^2)*p^2

       + 2*Sqrt[

       (-4*b^2*((L1/c1)^2 - (L2/c2)^2)*((dD1*L1)/c1)^2-(dD2*L2/c2)^2)*p^2)
       / (dD1^2*dD2^2)

       + (L1^2*L2^2*p^4)
       / (c1^2*c2^2)])  >>>>>>>>>>>>>>>>>>>>>>>>>>>>> )] ??

       / (L1^2/c1^2-L2^2/c2^2)^2]/2

where we define the following dimensionless ratios as follows:

dD1 = diameter/L1;
dD2 = diameter/L2;

and where

L1 = length - L2;

where length is the total internal length of the cylindrical cavity.

The quantity "b" is defined as follows:

b := If[modetype == "TM", xbesselzeros[[i + 1, j]]/Pi,  If[modetype == "TE", xprimebesselzeros[[i + 1, j]]/Pi]]

Thanks
Title: Re: EM Drive Developments - related to space flight applications - Thread 2
Post by: Rodal on 03/05/2015 08:58 pm
@ RODAL

Can you check this ?  (working it down)

root1= Sqrt[(

       (-4*b^2*(dD1^2-dD2^2)*((L1/c1)^2-(L2/c2)^2))
       / (dD1^2*dD2^2)
 
       + ((L1/c1)^2+(L2/c2)^2)*p^2

       - 2*Sqrt[

       (-4*b^2*((L1/c1)^2-(L2/c2)^2)*((dD1*L1/c1)^2-(dD2*L2/c2)^2)*p^2)
       / (dD1^2*dD2^2)

       + (L1^2*L2^2*p^4)
       / (c1^2*c2^2)])  >>>>>>>>>>>>>>>>>>>>>>>>>>>>> )] ??

       / (L1^2/c1^2-L2^2/c2^2)^2]/2


root2= Sqrt[(

       (-4*b^2*(dD1^2-dD2^2)*((L1/c1)^2-(L2/c2)^2))
       / (dD1^2*dD2^2)

       + ((L1/c1)^2+(L2/c2)^2)*p^2

       + 2*Sqrt[

       (-4*b^2*((L1/c1)^2 - (L2/c2)^2)*((dD1*L1)/c1)^2-(dD2*L2/c2)^2)*p^2)
       / (dD1^2*dD2^2)

       + (L1^2*L2^2*p^4)
       / (c1^2*c2^2)])  >>>>>>>>>>>>>>>>>>>>>>>>>>>>> )] ??

       / (L1^2/c1^2-L2^2/c2^2)^2]/2

where we define the following dimensionless ratios as follows:

dD1 = diameter/L1;
dD2 = diameter/L2;

and where

L1 = length - L2;

where length is the total internal length of the cylindrical cavity.

The quantity "b" is defined as follows:

b := If[modetype == "TM", xbesselzeros[[i + 1, j]]/Pi,  If[modetype == "TE", xprimebesselzeros[[i + 1, j]]/Pi]]

Thanks

root1 = Sqrt[((-4*b^2*(dD1 - dD2)*(dD1 + dD2)*(L1/c1 - L2/c2)*(L1/c1 + L2/c2))/(dD1^2*dD2^2) + (L1^2/c1^2 + L2^2/c2^2)*p^2 -
    2*Sqrt[(-4*b^2*(L1/c1 - L2/c2)*(L1/c1 + L2/c2)*((dD1*L1)/c1 - (dD2*L2)/c2)*((dD1*L1)/c1 + (dD2*L2)/c2)*p^2)/(dD1^2*dD2^2) + (L1^2*L2^2*p^4)/(c1^2*c2^2)])/(L1^2/c1^2 - L2^2/c2^2)^2]/2


root2 = Sqrt[((-4*b^2*(dD1 - dD2)*(dD1 + dD2)*(L1/c1 - L2/c2)*(L1/c1 + L2/c2))/(dD1^2*dD2^2) + (L1^2/c1^2 + L2^2/c2^2)*p^2 +
    2*Sqrt[(-4*b^2*(L1/c1 - L2/c2)*(L1/c1 + L2/c2)*((dD1*L1)/c1 - (dD2*L2)/c2)*((dD1*L1)/c1 + (dD2*L2)/c2)*p^2)/(dD1^2*dD2^2) + (L1^2*L2^2*p^4)/(c1^2*c2^2)])/(L1^2/c1^2 - L2^2/c2^2)^2]/2


and yes, you can always substitute, if you prefer

(A-B)(A+B)=A^2 - B^2
Title: Re: EM Drive Developments - related to space flight applications - Thread 2
Post by: Rodal on 03/05/2015 09:13 pm
Since the denominator is the SquareRoot of a square, you can take the denominator out of the Square Root


Take a gander (http://ih2.redbubble.net/image.9999479.1628/flat,550x550,075,f.jpg)
at the image below:

(it reads ROOT2 and ROOT4 because there are 4 roots and root1 above is the 2nd root of the 4 roots).  I only showed two of the roots, since the other two roots give negative frequencies.
Title: Re: EM Drive Developments - related to space flight applications - Thread 2
Post by: Notsosureofit on 03/05/2015 11:01 pm
Great !

Next need to solve for the expression (f0^2 - f2^2) where (f0 => L1+L2=0 and f2 =>L1+L2=L ??)

Hmmm, let me think about that for a second.  What we want to do is isolate the (p*pi/L)^2 term so it will cancel out leaving the Doppler shifts in the accelerated frame. ie. (f0^2 - f2^2) where f0^2 is evaluated at one end and f2^2 at the other.  In short, we want to find the frame in which the dispersion from end to end disappears.  The complication is the terms from the reflections at the c1,c2 interface.

Title: Re: EM Drive Developments - related to space flight applications - Thread 2
Post by: Rodal on 03/05/2015 11:16 pm
Great !

Next need to solve for the expression (f0^2 - f2^2) where (f0 => L1+L2=0 and f2 =>L1+L2=L ??)

Hmmm, let me think about that for a second.  What we want to do is isolate the (p*pi/L)^2 term so it will cancel out leaving the Doppler shifts in the accelerated frame. ie. (f0^2 - f2^2)

There are 3 terms involving p, two of them are proportional to p^2 and the third one is proportional to p^4

Two of the terms multiplying p only involve L1/c1 and L2/c2, another term (the negative second p^2 term, inside the second SquareRoot) involves a lot of stuff..


Sorry, I don't understand what you wrote...  What is what f0 and f2 ?

If you would like me to calculate a Limit of an expression, as a variable goes to a certain value, I can do that very precisely.  I can also do a perturbation analysis if you want to pursue a series expansion as a parameter goes to zero...
Title: Re: EM Drive Developments - related to space flight applications - Thread 2
Post by: Rodal on 03/06/2015 12:03 am
I don't understand how to go from here for the cylindrical RF cavity with only one medium:

f = (c/(2*Pi))*((X/R)^2+((p*Pi)/L)^2)^.5

to here:

df = (1/(2*f))*(c/(2*Pi))^2*X^2*((1/Rs^2)-(1/Rb^2))

What is df ?

How do you rotate the dispersion relation of the cavity into a Doppler frame to get the Doppler shifts ?, looking at the dispersion curve intersections of constant wave number instead of constant frequency.



Where do the terms Rs^2 and Rb^2 come from?
Title: Re: EM Drive Developments - related to space flight applications - Thread 2
Post by: Notsosureofit on 03/06/2015 12:03 am
@ RODAL

I'm struggling w/ the best way to explain it.  We want to get the expression (f0^2 - f2^2) such that the frequencies

f0 and f2 are evaluated at the two ends of the cavity at the same wavenumber.  They are the frequencies (energies) that would appear at the ends of the cavity if it were accelerated to the point that the dispersion between the ends were to disappear.

Yes, dispersion curve intersections at constant k.

No by df I just mean f1-f2 ie del f

From f^2 evaluations at the ends  (ie (f1^2-f2^2) = del f * 2f0  etc.

[Excuse the quick..I'm being otherwise distracted at the moment]
Title: Re: EM Drive Developments - related to space flight applications - Thread 2
Post by: Rodal on 03/06/2015 12:18 am
@ RODAL

I'm struggling w/ the best way to explain it.  We want to get the expression (f0^2 - f2^2) such that the frequencies

f0 and f2 are evaluated at the two ends of the cavity at the same wavenumber.  They are the frequencies (energies) that would appear at the ends of the cavity if it were accelerated to the point that the dispersion between the ends were to disappear.

Yes, dispersion curve intersections at constant k.

No by df I just mean f1-f2 ie del f

From f^2 evaluations at the ends  (ie (f1^2-f2^2) = del f * 2f0  etc.

[Excuse the quick..I'm being otherwise distracted at the moment]

This time, instead of purely geometrical terms for the truncated cone with only one medium like

(b^2)*((1/Rs)^2 - (1/Rb)^2)

one gets terms due to the different speed of light in mediums 1 and 2 for example

(b^2)*((L1/c1)^2 -(L2/c2)^2)


=> this is quite pedagogical to understand your point, and Mulletron's point that a cylindrical cavity with two different dielectric mediums maybe as or more effective than a truncated cone with only one medium
Title: Re: EM Drive Developments - related to space flight applications - Thread 2
Post by: frobnicat on 03/06/2015 12:27 am
Follow up on the Z tilted component of the pendulum as used at Eagleworks for the EM drive reported experiments.

For recall we have the following situation :
(http://forum.nasaspaceflight.com/index.php?action=dlattach;topic=36313.0;attach=811757;image)

The tilt in this illustration is obviously grossly exaggerated. The only quantitative estimation I have from Star-Drive (http://forum.nasaspaceflight.com/index.php?topic=36313.msg1341835#msg1341835) is that the platform deviates from horizontal, downward toward X- (as illustrated), for ~ .25 a bubble of a 24'' level. It depends on the level, I have a 60 cm level and .25 bubble corresponds to 12mm raise of one end, that is 2% slope or about 1.15°. Anyway, it can't be 10° and it can't be .1° so I will assume 1° as an order of magnitude first estimate of this angle I call theta.

Assuming Z is a perfect axis without stiffness (we know this is not true, there is a restoring torque of 2*0.007 Lb-in/degree in the flexure bearings) a tilted pendulum like that with a CoM (centre of mass) for the rotating assembly that is lowered by the tilt (that is the case, the CoM is toward X-) is stable and is strictly equivalent to a vertical hanging pendulum with a reduced gravity of g sin(theta). The problem can therefore be studied in the plane XY as if there was a gravity toward X-. It will also be equivalent when I will add the restoring torque component of the flexure bearings but at the moment I leave that aside and will treat it as a separate angular spring constant on top of a perfect tilted Z axis pendulum. The following drawing from this post (http://forum.nasaspaceflight.com/index.php?topic=36313.msg1341514#msg1341514) illustrates the principle.

Too big picture, still struggling to insert a new attached picture inside the body... please follow this link (http://forum.nasaspaceflight.com/index.php?action=dlattach;topic=36313.0;attach=811712;image) if needed.


Why is it so important ? Any shift along Y (tangential) of the CoM of the test article relative to its point of attach on the arm will change the rest equilibrium position. This change is "amplified" by the proximity of the global CoM (whole rotating assembly) relative to axis Z : the angle of deviation of rest angular position is actually (a bit) more than the angle of the test article's CoM shift as seen from the axis. A thermal shift of 5µm would record more than 5µm on the LDS. And given the orientations, a shift of CoM toward Y-, for instance an inward buckling of the big end cap, but also the global expansion along the length of the frustum, has an effect in reverse of what everyone thought : it will actually raise the LDS readings.

In the attached schematics below, approximately to scale (apart from deviations greatly exaggerated), green dot represents the CoM A of electronic stack and whole Faztek beam, yellow dot represents the CoM B of test article, black cross represents the CoM AB of the whole rotating assembly, it is the barycentre of the weighted green dot and yellow dot positions.

1. Initial stable equilibrium rest position, note that we have a hanging pendulum in a reduced gravity of g sin(theta). From latest weight and positions values (http://forum.nasaspaceflight.com/index.php?topic=36313.msg1341256#msg1341256) where I lack only the mass of dielectric (small end, should have asked, I assumed 1kg, anyone ?) the CoM AB is 6cm below the axis.

2. A thermal expansion of test article would shift CoM B to the right (exaggerated here). As a result the global CoM AB will be shifted to the right, the torque will be clockwise.

3. The rest equilibrium position needs CoM AB to be at the vertical ( as defined by vector g projection on XY, that is the vector a of magnitude g sin(theta) ) below the axis, so that the Force vector F = M g sin(theta) don't have torque relative to the rotation, M being the total mass of assembly. Assuming theta=1° as a first guess, g sin(theta) is worth 0.17 m/s² (0.017 g) and F = 1.95 N.

Obviously (but I prefer to state it explicitly) this is not 2. instant final shift  then 3. new rest equilibrium. The thermal shift would occur gradually and the equilibrium position would follow gradually. Dynamic aspects should be studied too, but 1. and 3. are two valid static situations (1. before heating, 3. either just after heating stops or at new thermal equilibrium under power load and temperature has reached a plateau).

Now if we compare the equivalent stiffness ( restoring torque per angular deviation ratio in Nm/rad ) of this "tilted hanging pendulum" component with the stiffness of the flexure bearings :

(F application point 6cm from axis, linearised .06m/rad * 1.95N = 0.117 Nm/rad)
Tilted hanging pendulum : 0.117 Nm/rad (from a guesstimate of tilt 1°)
Two flexure bearings 0.007 Lb-in/degree each : 0.0906 Nm/rad

The tilted hanging pendulum component is in the same ballpark as the flexure bearings. The flexure bearings can "dilute" the impact of a test article's CoM thermal shift on the sustained displacement of rest angular equilibrium but only to a limited ratio. Even if µm displacements of LDS readings relative to µm shifts of tests article CoM is not 1 to 1, it may very well be  1 to 2 or 1 to 3. And those effects are direct, it's not like the sustained deviations in readings are to be accounted by a constant second derivative of CoM's position as would be the case with pure recoil effects. No longer. Now, every single sustained 1µm deviation of readings might be explained by a sustained expansion (ie a body at or near new thermal equilibrium) displacement of a few µm at most.

Now, if only we could explain the discrepancies of the vertical axis readings of between 1µm to 2.5µm relative to the calibration pulses of 29.1µN... Could anyone do detailed check and see a blunder or a flaw ? (http://forum.nasaspaceflight.com/index.php?topic=36313.msg1341723#msg1341723)
Title: Re: EM Drive Developments - related to space flight applications - Thread 2
Post by: Notsosureofit on 03/06/2015 12:32 am
@ RODAL

I'm struggling w/ the best way to explain it.  We want to get the expression (f0^2 - f2^2) such that the frequencies

f0 and f2 are evaluated at the two ends of the cavity at the same wavenumber.  They are the frequencies (energies) that would appear at the ends of the cavity if it were accelerated to the point that the dispersion between the ends were to disappear.

Yes, dispersion curve intersections at constant k.

No by df I just mean f1-f2 ie del f

From f^2 evaluations at the ends  (ie (f1^2-f2^2) = del f * 2f0  etc.

[Excuse the quick..I'm being otherwise distracted at the moment]

This time, instead of purely geometrical terms for the truncated cone with only one medium like

(b^2)*((1/Rs)^2 - (1/Rb)^2)

one gets terms due to the different speed of light in mediums 1 and 2 for example

(b^2)*((L1/c1)^2 -(L2/c2)^2)


=> this is quite pedagogical to understand your point, and Mulletron's point that a cylindrical cavity with two different dielectric mediums maybe as or more effective than a truncated cone with only one medium

Yes, and the accelerated frame of reference is just an easy way to think about the thrust/photon; T = h*[del f]/L   It cancels out of the derivation.
Title: Re: EM Drive Developments - related to space flight applications - Thread 2
Post by: aero on 03/06/2015 12:55 am
Quote
this is quite pedagogical to understand your point, and Mulletron's point that a cylindrical cavity with two different dielectric mediums maybe as or more effective than a truncated cone with only one medium

You can add meep to that camp. Or at least for all indications you can but I should make some controlled runs to confirm it. If I ran a truncated cone and a cylinder, what frequency and cylindrical dimensions would I use in order that it be a fair comparison to the truncated cone? Dielectric constant? Or, if you prefer, what cylinder and cone dimensions are consistent? Same resonant frequency and mode or same "size" "volume" but since we're addressing a space thruster, maybe integrated structural mass? Cylinders tend to be smaller and simpler.
Title: Re: EM Drive Developments - related to space flight applications - Thread 2
Post by: Rodal on 03/06/2015 01:03 am
Quote
this is quite pedagogical to understand your point, and Mulletron's point that a cylindrical cavity with two different dielectric mediums maybe as or more effective than a truncated cone with only one medium

You can add meep to that camp. Or at least for all indications you can but I should make some controlled runs to confirm it. If I ran a truncated cone and a cylinder, what frequency and cylindrical dimensions would I use in order that it be a fair comparison to the truncated cone? Dielectric constant? Or, if you prefer, what cylinder and cone dimensions are consistent? Same resonant frequency and mode or same "size" "volume" but since we're addressing a space thruster, maybe integrated structural mass? Cylinders tend to be smaller and simpler.

The best thing (if you have not already done so) is to run exactly the same cylinder geometry with at least two runs: one run with a single medium and another run with two mediums inside it.

What force do you get when you run the cylinder with only one medium (for example just vacuum) in MEEP  as compared to the same cylinder diameter and length with two mediums inside it (vacuum at one end and a dielectric at the other end)?
Title: Re: EM Drive Developments - related to space flight applications - Thread 2
Post by: Notsosureofit on 03/06/2015 02:15 am
@ RODAL

Heading off, but I thought I should clarify that the f vs k curve I've been using has the third axis (into the paper) where k is an "instantaneous" function of x from 0 to L, evaluated at 0 and L  for f1, f2, which are (hopefully) the extremes.

Title: Re: EM Drive Developments - related to space flight applications - Thread 2
Post by: Star-Drive on 03/06/2015 03:24 am
@ Rodal :
See attached picture to share my mental image. Tilt over-exaggerated for illustration.

Grey : solid rotating assembly (no deformation implied)
Orange/brown : fixed assembly (no deformation implied)
Blue : the ground slab of the vacuum chamber (no deformation implied)

For now, assume a perfect axis of rotation around Z : only one degree of freedom of Grey relative to Orange, the "official" rotation around Z, no compliance implied, Grey kept in the XY plane, plane has same tilt as Orange (XY parallel to Orange platform).

@Star-Drive
Can you confirm this is a correct way to understand that there is a tilt in the axis of rotation ?

(http://forum.nasaspaceflight.com/index.php?action=dlattach;topic=36313.0;attach=811757;image)

One aspect of this tilt in the axis of rotation that hasn't been discussed is the requirement for imbalance.   The difference in moments on either side of the beam only has to be very slight for it to always come to rest at the same location +/- a micron or two.

Yes, a difference in centre of mass can make long lasting difference in position. The following statement tries to summarize rigorously :
Statement A : Any shift in centre of mass of a part (relative to fixation to the balance arm) along a direction orthogonal to a compliant (not infinite stiffness) axis that is not strictly vertical can induce a change in angular rest position around said compliant axis.

This can be quite important as a (thermal) longitudinal (Y+ or Y-) shift in CoM of the frustum assembly would be orthogonal to axis Z, but so long as this most compliant by far axis Z (the natural axis of the pendulum) was believed to be strictly vertical there would be no change in angular rest position of the arm (around Z). Only transient angular positions shifts could be induced by such thermal CoM shifts, the arm position (around Z) would be quickly enough restored to unmodified rest position (return to baseline) by the spring restoring torque of flexure bearings, and then time of ~45s during which a sustained displacement where recorded could be used as an effective argument that this couldn't be due to thermal expansions alone.

Note that now that the Z axis is no longer vertical the argument is no longer valid.

Quote
A small CW tilt along the X axis of the balance arm would explain the apparent drift in the baseline seen in some of the thrust waveforms.   When the cavity has the orientation shown below its CM shifts to the left.  This would reduce the tilt, resulting in an increase in brightness of the reflected light the LDS measures; due to the mirror position being closer to an optimal perpendicular position wrt the light beam.   The increase in brightness corresponds to a decrease in distance; hence the negative slope.   With the device mounted the other way the shift in CM increases the tilt.  This reduces the reflected light and is registered as an increase in distance.   

Is this what you are saying ?
(http://forum.nasaspaceflight.com/index.php?action=dlattach;topic=36313.0;attach=809643;image)

Indeed the X axis is not vertical (it is horizontal, or almost), so the above statement A does apply if we consider rotations around X are not against infinite stiffness. But preliminary rough estimations gave me relative displacements of rest optical length of at least one order of magnitude below signal. Even if not infinitely stiff, the added torsion compliance of faztek beam around X and compliance of tandem flexure bearings around X is not enough. For reasonable thermal CoM's shifts, this is stiff enough , and the optical lever is small enough (optical length d measured at small distance below X) that it would make very little contribution to the signal.

Quote
No actual motion of the beam occurs.  This apparent motion is an optical artifact.   This assumes the Philtec distance sensor is used on the far side.   If it is used on the near side a small CCW tilt along the X axis of the beam would produce the same effect; except requiring much less rotation from the change in CM.

For this twist around X effect, no actual rotation of the beam around Z would occur, but the beam is deformed. I wouldn't qualify that as an "optical artifact". Sorry I'm becoming quite finicky on wordings lately  :)  That would be a real mechanical motion, only not a motion around the "official" Z axis.

We have all reasons to believe the Philtec linear sensor is used in the far range, see attached chart's horizontal units. The initial report (anomalous...) clearly states that it is used around 500µm, the vertical readings of the charts are consistent, and Paul March confirms :
...
- That the vertical scale in the charts (indicated in µm, around 500) are relevant or not relevant.
...
...
The Philtec D63 fiber-optic displacement sensor measures distance from its target mirror in microns, so the numbers on the left hand side of the force plots measure the distance from the end of the fiber-optic laser head to its mirror target mounted on the torque pendulum arm.  The data sheet for same is attached.
...

Still, there is a factor 10 disparity between the apparent stiffness readings (LDS readings against calibrations pulses alone) and the stiffness needed to explain the 4.5s pseudo-period of oscillations, so this can put a doubt on the readings of vertical scale, a factor 10 here could explain a lot of things.

Regardless of this nagging problem of late, now that we know that Z deviates from strict verticality by a quarter bubble, from statement A above there is now a real possibility that a thermal CoM shift along Y changes angular rest position around the official Z, of relative magnitude compatible with signals, as rotation around Z has a very low stiffness (the restoring torque of flexure bearing around their natural axis of rotation). And such change in angular rest position around Z would record as false "sustained thrusts" in the charts.

Statement B : The validity of real sustained thrust signals now rests on the amount of deviation of Z from vertical, resolution of the contradictions between apparent stiffness around Z, and careful assessment of thermal expansions in vacuum. The sustained duration alone no longer suffice.


Frobnicat:

Your analysis and comments made me go back and look at the current alignment of the Eagleworks torque pendulum and the attached picture indicates that my recollection of the leveling of the bottom beam of the TP being a quarter bubble high at the vacuum chamber door end was a bit exaggerated due to parallax error and a just plain bad memory.  It looks now like less than tenth of bubble low at the door end of the vacuum chamber, but part of that apparent tilt may in reality be due to actual bending of the 1.50 inch square aluminum beam.

Best, Paul M.
Title: Re: EM Drive Developments - related to space flight applications - Thread 2
Post by: zen-in on 03/06/2015 04:22 am


For this twist around X effect, no actual rotation of the beam around Z would occur, but the beam is deformed. I wouldn't qualify that as an "optical artifact". Sorry I'm becoming quite finicky on wordings lately  :)  That would be a real mechanical motion, only not a motion around the "official" Z axis.

We have all reasons to believe the Philtec linear sensor is used in the far range, see attached chart's horizontal units. The initial report (anomalous...) clearly states that it is used around 500µm, the vertical readings of the charts are consistent, and Paul March confirms :
...
- That the vertical scale in the charts (indicated in µm, around 500) are relevant or not relevant.
...
...
The Philtec D63 fiber-optic displacement sensor measures distance from its target mirror in microns, so the numbers on the left hand side of the force plots measure the distance from the end of the fiber-optic laser head to its mirror target mounted on the torque pendulum arm.  The data sheet for same is attached.
...

Still, there is a factor 10 disparity between the apparent stiffness readings (LDS readings against calibrations pulses alone) and the stiffness needed to explain the 4.5s pseudo-period of oscillations, so this can put a doubt on the readings of vertical scale, a factor 10 here could explain a lot of things.

Regardless of this nagging problem of late, now that we know that Z deviates from strict verticality by a quarter bubble, from statement A above there is now a real possibility that a thermal CoM shift along Y changes angular rest position around the official Z, of relative magnitude compatible with signals, as rotation around Z has a very low stiffness (the restoring torque of flexure bearing around their natural axis of rotation). And such change in angular rest position around Z would record as false "sustained thrusts" in the charts.

Statement B : The validity of real sustained thrust signals now rests on the amount of deviation of Z from vertical, resolution of the contradictions between apparent stiffness around Z, and careful assessment of thermal expansions in vacuum. The sustained duration alone no longer suffice.

I don't think the tilt of the balance beam about the X axis would have to be very much for the resulting change in received light amplitude to register as a shift in position.   The LDS has fractional micron resolution.    Assuming the light pattern from the fiber optic cable has a circular gaussian distribution, for any given distance the maximum light level hitting the detector is when the plane of the mirror is perpendicular to the central axis of the light beam.   Any small deviation ( arc-second) will reduce the light amplitude; which registers as a displacement.     It is impossible to align the LDS perfectly so the angle between the mirror and the light beam is never exactly 90 degrees in X and Y.    The expansion of the cavity due to heating has a very long time constant.   The slow drift in the position between RF pulses looks like a thermal response.
Title: Re: EM Drive Developments - related to space flight applications - Thread 2
Post by: DIYFAN on 03/06/2015 04:32 am
Same here.  The EBay units I've got are #201065780928 and #131442703325 so far in case anyone want to try the same system.

I've decided to try and live up to my screen name and throw my lot in with the replicators.  I have a few questions before I kick off my effort:

1) Why did you decide to go with #201065780928 rather than a frustrum of a cone?  The #201065780928 part looks more like a rectangular slit shape rather than a cone shape.
2) How do you plan on hooking the MA86751B X band oscillator up to the waveguide assembly?
3) It looks like the MA86751B X band oscillator is tunable from 9.9 GHz to 10.6 GHz with power output levels from 10 mW to 100 mW powered by 9-10 Volts DC.  Did you pick this particular oscillator for a reason?
4) Do you expect there to be a resonant frequency within the 9.9 GHz to 10.6 GHz frequency band?
5) Do you plan on putting a dielectric toward one end of the waveguide assembly?
6) With what material do you plan on capping each end of the waveguide assembly?

Although I direct this friendly set of questions to Notsosureofit, Mulletron, and others who are attempting replications, I welcome any forum members to chime in with recommendations or insights.  Can you imagine what would happen if the effect can be shown on such a small scale?  It could cause some ripples and raise a quite a few eyebrows around the world.
Title: Re: EM Drive Developments - related to space flight applications - Thread 2
Post by: zen-in on 03/06/2015 05:01 am
Same here.  The EBay units I've got are #201065780928 and #131442703325 so far in case anyone want to try the same system.

I've decided to try and live up to my screen name and throw my lot in with the replicators.  I have a few questions before I kick off my effort:

1) Why did you decide to go with #201065780928 rather than a frustrum of a cone?  The #201065780928 part looks more like a rectangular slit shape rather than a cone shape.
2) How do you plan on hooking the MA86751B X band oscillator up to the waveguide assembly?
3) It looks like the MA86751B X band oscillator is tunable from 9.9 GHz to 10.6 GHz with power output levels from 10 mW to 100 mW powered by 9-10 Volts DC.  Did you pick this particular oscillator for a reason?
4) Do you expect there to be a resonant frequency within the 9.9 GHz to 10.6 GHz frequency band?
5) Do you plan on putting a dielectric toward one end of the waveguide assembly?
6) With what material do you plan on capping each end of the waveguide assembly?

Although I direct this friendly set of questions to Notsosureofit, Mulletron, and others who are attempting replications, I welcome any forum members to chime in with recommendations or insights.  Can you imagine what would happen if the effect can be shown on such a small scale?  It could cause some ripples and raise a quite a few eyebrows around the world.

It doesn't look like that Gunn oscillator (the MA86751B X band oscillator) has a varactor diode.   That would make it difficult to tune.
Title: Re: EM Drive Developments - related to space flight applications - Thread 2
Post by: Notsosureofit on 03/06/2015 11:33 am
Same here.  The EBay units I've got are #201065780928 and #131442703325 so far in case anyone want to try the same system.

I've decided to try and live up to my screen name and throw my lot in with the replicators.  I have a few questions before I kick off my effort:

1) Why did you decide to go with #201065780928 rather than a frustrum of a cone?  The #201065780928 part looks more like a rectangular slit shape rather than a cone shape.
2) How do you plan on hooking the MA86751B X band oscillator up to the waveguide assembly?
3) It looks like the MA86751B X band oscillator is tunable from 9.9 GHz to 10.6 GHz with power output levels from 10 mW to 100 mW powered by 9-10 Volts DC.  Did you pick this particular oscillator for a reason?
4) Do you expect there to be a resonant frequency within the 9.9 GHz to 10.6 GHz frequency band?
5) Do you plan on putting a dielectric toward one end of the waveguide assembly?
6) With what material do you plan on capping each end of the waveguide assembly?

Although I direct this friendly set of questions to Notsosureofit, Mulletron, and others who are attempting replications, I welcome any forum members to chime in with recommendations or insights.  Can you imagine what would happen if the effect can be shown on such a small scale?  It could cause some ripples and raise a quite a few eyebrows around the world.

1.  it matches the osc. waveguide and 1st cheap one on EBay.
2.  just bolt them together
3.  I had this one already
4.  probably,  have to calculate when I get the waveguide
5. that's an option
6. TBD  flat pate, detector fitting etc etc

I'll just try a long ( 20 ft ?) pendulum First and see what happens.   Vacuum later.
Title: Re: EM Drive Developments - related to space flight applications - Thread 2
Post by: Rodal on 03/06/2015 11:40 am
....

I'll just try a long ( 20 ft ?) pendulum First and see what happens.   Vacuum later.
That would also be my first choice: a long pendulum with damping provided by an oil bath. The experiment used by Brito, Marini and Galian (they had battery power, self-contained) to nullify a Mach Lorentz Thruster:
Title: Re: EM Drive Developments - related to space flight applications - Thread 2
Post by: DIYFAN on 03/06/2015 11:41 am
http://www.ebay.com/itm/AERCOM-Microwave-RF-Isolator-Circulator-2-4GHz-20dB-isolation-Low-I-L-TESTED-/281549538390?ssPageName=ADME:L:OU:US:1120
Picked up one of these puppies on Ebay to protect my amp. Another example of broken time reversal symmetry in action.

Got about an oz of very expensive liquid metal from here:
http://www.amazon.com/Gallium-Indium-Eutectic-GaInSn-68-5%25/dp/B00KN92MWW/ref=sr_1_3?ie=UTF8&qid=1425074693&sr=8-3&keywords=galinstan

So back to the copper from way back: http://forum.nasaspaceflight.com/index.php?topic=36313.msg1326742#msg1326742
...
Been working with the supplier with a machine shop I posted about way back:
http://forum.nasaspaceflight.com/index.php?topic=36313.msg1326669#msg1326669
I'm going that route. The quote I got is: price: $120.00 layout + $51.63 for part + freight. So I have to pay the layout, then anyone else who wants one of these:
(https://lh4.googleusercontent.com/xpJIaQNfcmecDoHrIDnoYqVs1ccOghkxSpkgXhJ8MrnoyaK61MQ9rSfiBnuIhyayCIIEaQ=w1549-h745)
but built in 16oz copper, with a smooth butt seam inside, and 1/4" flange around edges, can get one for about 50 bucks plus shipping. If all this works out, it'll fulfill my goal of making a replication by DIYers easier. For me, paying the layout plus price about breaks even with buying the sheet myself and fumblefuddeling around trying to solder up a cone at home. So I'm happy. I'll get back with more later, when the items are at home.

Mulletron,

Can you provide me with the contact information (email) of the maker of the custom cone?  What dimensions did you settle on?  I appreciate that you are footing the bill for the layout, thereby lowering the cost for the rest of us.  Kudos to you for that.
Title: Re: EM Drive Developments - related to space flight applications - Thread 2
Post by: Rodal on 03/06/2015 12:19 pm
Same here.  The EBay units I've got are #201065780928 and #131442703325 so far in case anyone want to try the same system.

I've decided to try and live up to my screen name and throw my lot in with the replicators.  I have a few questions before I kick off my effort:

....
3) It looks like the MA86751B X band oscillator is tunable from 9.9 GHz to 10.6 GHz with power output levels from 10 mW to 100 mW powered by 9-10 Volts DC.  Did you pick this particular oscillator for a reason?
4) Do you expect there to be a resonant frequency within the 9.9 GHz to 10.6 GHz frequency band?
.....
Unless your cavity has an unusually small diameter, the problem with 9.9 GHz to 10.6 GHz will be a luxury of resonances rather than a scarcity of natural frequencies:  there may be too many resonant frequencies very close to each other at such a high frequency and therefore difficult to keep the cavity resonating in a pure mode.

No experimenter in the US, UK or China has operated an EM Drive at such a high frequency, and therefore one would be covering new ground.

@Notsosureofit's formula is proportional to the inverse of the frequency to the third power: this would mean (10/2)^3=125 times less thrust.  That's compensated by the square of the Bessel zero function which also appears in the expression, so, depending on the mode shape excited you may end up with higher thrust  (if the excited mode has many wave patterns through the circumference), similar thrust,  or lower thrust (if the excited mode has few wave patterns in the circumferential direction and many wave patterns in the longitudinal direction)  as present experiments...
Title: Re: EM Drive Developments - related to space flight applications - Thread 2
Post by: Mulletron on 03/06/2015 01:01 pm
http://www.ebay.com/itm/AERCOM-Microwave-RF-Isolator-Circulator-2-4GHz-20dB-isolation-Low-I-L-TESTED-/281549538390?ssPageName=ADME:L:OU:US:1120
Picked up one of these puppies on Ebay to protect my amp. Another example of broken time reversal symmetry in action.

Got about an oz of very expensive liquid metal from here:
http://www.amazon.com/Gallium-Indium-Eutectic-GaInSn-68-5%25/dp/B00KN92MWW/ref=sr_1_3?ie=UTF8&qid=1425074693&sr=8-3&keywords=galinstan

So back to the copper from way back: http://forum.nasaspaceflight.com/index.php?topic=36313.msg1326742#msg1326742
...
Been working with the supplier with a machine shop I posted about way back:
http://forum.nasaspaceflight.com/index.php?topic=36313.msg1326669#msg1326669
I'm going that route. The quote I got is: price: $120.00 layout + $51.63 for part + freight. So I have to pay the layout, then anyone else who wants one of these:
(https://lh4.googleusercontent.com/xpJIaQNfcmecDoHrIDnoYqVs1ccOghkxSpkgXhJ8MrnoyaK61MQ9rSfiBnuIhyayCIIEaQ=w1549-h745)
but built in 16oz copper, with a smooth butt seam inside, and 1/4" flange around edges, can get one for about 50 bucks plus shipping. If all this works out, it'll fulfill my goal of making a replication by DIYers easier. For me, paying the layout plus price about breaks even with buying the sheet myself and fumblefuddeling around trying to solder up a cone at home. So I'm happy. I'll get back with more later, when the items are at home.

Mulletron,

Can you provide me with the contact information (email) of the maker of the custom cone?  What dimensions did you settle on?  I appreciate that you are footing the bill for the layout, thereby lowering the cost for the rest of us.  Kudos to you for that.

The cone is the same dims as the DUT at Eagleworks. What was built was based off of this: https://docs.google.com/file/d/0B4PCfHCM1KYoN2VURmltbVlfa3c/edit?pli=1

It is in shipping to me att. I intend to make sure it is good to go, then if no changes are required, the guy who made it will list it on Ebay for around 50 bucks or so. Please give it a few days to arrive. I want to make sure there aren't any problems before it gets put up for sale. That way I accept the risk first. Here's what it looks like.
Title: Re: EM Drive Developments - related to space flight applications - Thread 2
Post by: Rodal on 03/06/2015 01:05 pm
http://www.ebay.com/itm/AERCOM-Microwave-RF-Isolator-Circulator-2-4GHz-20dB-isolation-Low-I-L-TESTED-/281549538390?ssPageName=ADME:L:OU:US:1120
Picked up one of these puppies on Ebay to protect my amp. Another example of broken time reversal symmetry in action.

Got about an oz of very expensive liquid metal from here:
http://www.amazon.com/Gallium-Indium-Eutectic-GaInSn-68-5%25/dp/B00KN92MWW/ref=sr_1_3?ie=UTF8&qid=1425074693&sr=8-3&keywords=galinstan

So back to the copper from way back: http://forum.nasaspaceflight.com/index.php?topic=36313.msg1326742#msg1326742
...
Been working with the supplier with a machine shop I posted about way back:
http://forum.nasaspaceflight.com/index.php?topic=36313.msg1326669#msg1326669
I'm going that route. The quote I got is: price: $120.00 layout + $51.63 for part + freight. So I have to pay the layout, then anyone else who wants one of these:
(https://lh4.googleusercontent.com/xpJIaQNfcmecDoHrIDnoYqVs1ccOghkxSpkgXhJ8MrnoyaK61MQ9rSfiBnuIhyayCIIEaQ=w1549-h745)
but built in 16oz copper, with a smooth butt seam inside, and 1/4" flange around edges, can get one for about 50 bucks plus shipping. If all this works out, it'll fulfill my goal of making a replication by DIYers easier. For me, paying the layout plus price about breaks even with buying the sheet myself and fumblefuddeling around trying to solder up a cone at home. So I'm happy. I'll get back with more later, when the items are at home.

Mulletron,

Can you provide me with the contact information (email) of the maker of the custom cone?  What dimensions did you settle on?  I appreciate that you are footing the bill for the layout, thereby lowering the cost for the rest of us.  Kudos to you for that.

The cone is the same dims as the DUT at Eagleworks. What was built was based off of this: https://docs.google.com/file/d/0B4PCfHCM1KYoN2VURmltbVlfa3c/edit?pli=1

It is in shipping to me att. I intend to make sure it is good to go, then if no changes are required, the guy who made it will list it on Ebay for around 50 bucks or so. Please give it a few days to arrive. I want to make sure there aren't any problems before it gets put up for sale. That way I accept the risk first. Here's what it looks like.

If one were to drive the above geometrical dimensions (same dims as the DUT at Eagleworks)

(http://forum.nasaspaceflight.com/index.php?action=dlattach;topic=36313.0;attach=812043;image)

 at the 9.9 GHz to 10.6 GHz previously discussed:

....
3) It looks like the MA86751B X band oscillator is tunable from 9.9 GHz to 10.6 GHz with power output levels from 10 mW to 100 mW powered by 9-10 Volts DC.  Did you pick this particular oscillator for a reason?
4) Do you expect there to be a resonant frequency within the 9.9 GHz to 10.6 GHz frequency band?
.....

instead of the ~1.9 GHz frequency tested at NASA Eagleworks,  one would be exciting very high natural frequencies, with lots of wave-patterns through the circumference and/or through the longitudinal axis: there would be a lot of very different mode shapes bunched up next to each other and it would be extremely difficult to keep it at a pure resonant mode.  A cone with these dimensions would be very difficult to calculate, very difficult to analyze and very difficult to experiment with at 10 GHz. 

Better to excite this cone in the ~2 GHz range.
Title: Re: EM Drive Developments - related to space flight applications - Thread 2
Post by: Mulletron on 03/06/2015 01:09 pm
I want to test my own shapes, and cylinders too but I feel it is most important to get to the Eagleworks/Shawyer one first. I want to make it appealing for others to attempt a replication, so I think this is a good way to start.
Title: Re: EM Drive Developments - related to space flight applications - Thread 2
Post by: frobnicat on 03/06/2015 02:05 pm
...

I don't think the tilt of the balance beam about the X axis would have to be very much for the resulting change in received light amplitude to register as a shift in position.   The LDS has fractional micron resolution.    Assuming the light pattern from the fiber optic cable has a circular gaussian distribution, for any given distance the maximum light level hitting the detector is when the plane of the mirror is perpendicular to the central axis of the light beam.   Any small deviation ( arc-second) will reduce the light amplitude; which registers as a displacement.     It is impossible to align the LDS perfectly so the angle between the mirror and the light beam is never exactly 90 degrees in X and Y.    The expansion of the cavity due to heating has a very long time constant.   The slow drift in the position between RF pulses looks like a thermal response.

Interesting hypothesis. You say that in this case the LDS readings would be more sensitive to a change in angle of the reflective mirror than a change in distance. The LDS is a small distance below the X axis, so a small angular deviation around X would amount to tiny displacement (d' - d) but the small angular deviation would itself tilt the mirror and that could change the LDS readings that is roughly proportional to the reflected light.

But then this coupling between angular effect and displacement would also be present for the main movements around Z. LDS is at 35cm from Z axis, I would say it is one order of magnitude more than the distance of mirror below X axis, the lever effect of displacement/angle ratio would be lower (less angle for same displacement) but still if angle deviation is to have a significant effect in case around X it should also have a significant effect around Z.

Unfortunately the documentation for the Philtec D63 displacement sensor (http://forum.nasaspaceflight.com/index.php?action=dlattach;topic=36313.0;attach=810350) seems to give no information on reflective surface angular deviation impact on measurements. It indicates Ø 1.6 mm Target Spot Size. This is 3 times more than nominal operating distance of 500µm (0.5mm) so your drawing is a bit misleading with proportions. In reality the reflected dot is big relative to distance from reflector, would an angular deviation still impact the amount of light swallowed back ?

Is it you drawing or you found it elsewhere ? I ask the question because you mention the reflecting surface as being mirror. It does appear as a mirror in the pictures of pendulum at Eagleworks and this is stated explicitly in Brady's report page 3 (together with that LDS is photon time of flight, which is not, but let's proceed). But I don't find in the D63 documentation that a mirror should or could be used. It does say that These specifications represent best case performance where: the target is flat, smooth and highly reflective
This implies that the target may not be highly reflective (at the cost of decreased performance) but does highly reflective imply that a mirror is the best case ? My reading (but I'm not native English reader) is that reflective can mean a matte white material, that is a surface that does bounce near 100% of incoming photons, but in a more or less scattering manner, not necessarily as per perfect reflection. See attached picture : top the situation at hand at Eagleworks, bottom the default situation for which the LDS might be initially calibrated.

This could explain a lot about the disparity between vertical readings in the charts and stiffness parameters : when charts indicate ~1µm LDS deviation for 29.1µnN calibration pulses for instance, all could be much better explained and consistent if it meant 10µm instead.

To me there is no doubt that the LDS is operating in the far range : the cal. pulses are attractive by nature, and correspond to a displacement to the right (Y+) and that lowers the measured distance, as in the charts. Also operating in the near range would not only reverse this consistent orientation but would also increase the sensitivity : that would go against what is to be explained (apparent higher µm/µN stiffness than compatible with other known parameters, for 2 separate predictions from the dynamic periods of oscillations and the known stiffness of flexure bearings)

So, what anyone wanting to understand clearly the charts need is a way to explain a reduced sensitivity of the LDS, that would show 1µm displacement when in fact it is 10µm. Following on the D63 doc, there is a calibration procedure :
The effect of changing target reflectance is to shift the voltage output higher or lower. Factory calibrations have
the Peak Voltage set to 5.000 volts. A gain control is provided for calibration of the sensor output to various target surfaces. In-situ calibration is performed simply, by adjusting the sensor’s tip-to-target gap until the peak output voltage is attained, and then by using the gain control to set the peak voltage to full scale (5.000 volts). After setting the peak to 5 volts, the factory gap calibration chart applies for the target being measured. This procedure allows the sensor to be used to perform precision linear motion measurements on most materials.


I read "most materials : including smooth but scattering surfaces", not only mirrors. Actually, mirrors may be a very special case. My question is now if the gain control of the system is consistent with the use of a mirror reflecting target. As seen qualitatively from attached figure : a perfect mirror would bounce back in the signal detector a lot more photons than a matte white. If the gain is calibrated against matte material (say, as hypothesis, default factory settings) the baseline distance from the mirror to have a tension that shows as 500µm would be much greater than 500µm.

The protocol of tuning for nominal 500µm distance between optical fibre head and mirror is on page 3 of Brady et al report (anomalous...) :
Quote from: Brady et al. Anomalous thrust...
Prior to a test run data take, the LDS is positioned to a known displacement datum (usually 500 micrometers) via mechanical adjustments to its mounting platform. Gross adjustments are performed via set screws. Fine adjustments are performed using manually - operated calibrated screw mechanisms and a remotely controlled motorized mechanism that can be operated with the chamber door closed and the chamber at vacuum. The remote adjustment capability is necessary since the LDS datum will change whenever a change to the test facility environment affects the roll - out table or the chamber – e.g., whenever the chamber door is closed or latched and whenever the chamber is evacuated. Once the LDS displacement is adjusted in the final test environment, further adjustment between test run data takes is usually not required.

How this displacement of 500µm is known ? Has the procedure followed the step (as per D63 documentation) of homing to the distance corresponding to peak signal (whatever it's absolute level), calibrating the gain so that at this peak the signal is now 5V, and then stepping back until meeting ~4.3 V corresponding to 500µm nominal distance ? If this was not followed, or if the D63 is not mean to be operated with perfect mirror target (doubtful it wouldn't be mentioned in documentation, but who knows ?), it is possible the LDS is operating in a far range that has a much lower sensitivity (and linearity) than the nominal -2.7 mv/µm.

Edit : BTW, how comes that at some very near distance there start to be less photons bouncing back in the signal detector ? Thank zen-in for bringing up interesting questions about LDS...
Title: Re: EM Drive Developments - related to space flight applications - Thread 2
Post by: DIYFAN on 03/06/2015 03:21 pm
Same here.  The EBay units I've got are #201065780928 and #131442703325 so far in case anyone want to try the same system.

I've decided to try and live up to my screen name and throw my lot in with the replicators.  I have a few questions before I kick off my effort:

1) Why did you decide to go with #201065780928 rather than a frustrum of a cone?  The #201065780928 part looks more like a rectangular slit shape rather than a cone shape.
2) How do you plan on hooking the MA86751B X band oscillator up to the waveguide assembly?
3) It looks like the MA86751B X band oscillator is tunable from 9.9 GHz to 10.6 GHz with power output levels from 10 mW to 100 mW powered by 9-10 Volts DC.  Did you pick this particular oscillator for a reason?
4) Do you expect there to be a resonant frequency within the 9.9 GHz to 10.6 GHz frequency band?
5) Do you plan on putting a dielectric toward one end of the waveguide assembly?
6) With what material do you plan on capping each end of the waveguide assembly?

Although I direct this friendly set of questions to Notsosureofit, Mulletron, and others who are attempting replications, I welcome any forum members to chime in with recommendations or insights.  Can you imagine what would happen if the effect can be shown on such a small scale?  It could cause some ripples and raise a quite a few eyebrows around the world.

1.  it matches the osc. waveguide and 1st cheap one on EBay.
2.  just bolt them together
3.  I had this one already
4.  probably,  have to calculate when I get the waveguide
5. that's an option
6. TBD  flat pate, detector fitting etc etc

I'll just try a long ( 20 ft ?) pendulum First and see what happens.   Vacuum later.

Okay, thanks for those answers/thoughts.  Following up on 2 and 6, given that they just bolt together, doesn't that mean that the oscillator occupies one of the ends of the cavity?  It may sound like an obvious question, but I wonder how this would impact the potential effect when other models and equations seem to rely on there being two flat plates on each end of the cavity to reflect the waves.  I'm writing this without the luxury of having the MA86751B X band oscillator in hand, so I don't know the shape or material of the underside (which is not shown in the picture posted on ebay and I can't seem to find other pictures of it online).  Maybe it is just time to start ordering parts and see what it all looks like.   :D

Edit: what do you think of something like this, which would allow a flat plate to be placed on either end?

http://www.ebay.com/itm/WAVEGUIDE-WR-90-T-/131444026986?pt=LH_DefaultDomain_0&hash=item1e9aacae6a
Title: Re: EM Drive Developments - related to space flight applications - Thread 2
Post by: Notsosureofit on 03/06/2015 03:48 pm
Hopefully the whole thing acts as a cavity oscillator.  The intrusion acts as the "antennas". (my old cavity oscillator had a "lighthouse" tube stuck into it.  Wish I could find that thing)

Don't know what to do w/ a "Tee" ?

While the 1/f^3 factor is in there, the geometry of the smaller cavity at higher frequency should help.

The power is only ~50mW though.

Title: Re: EM Drive Developments - related to space flight applications - Thread 2
Post by: frobnicat on 03/06/2015 04:00 pm
http://www.philtec.com/downloadssupport/documentlibrary/documents/philtecmanuals/newanalogD.pdf
This answers some of the questions I just asked about LDS, reflectivity, calibration...  :D

Yes the D63 can measure displacements from matte surfaces (even black rubber !), it can also work with metal mirror polished surfaces. The later (mirror vs matte) having the higher reflectance, that is the higher signals, to be compensated by proper gain adjustment. The interpretation charts µm in function of analog volts output depends on the gain adjustment 5V at peak reflexion (between "near side" and "far side").

If expecting to see ~4.3V corresponding to 500µV with a correct gain (black curve and magenta tangent) then a bad gain much too high would get us more to the right on the red curve (believing we are measuring 500µm when actually it is more than 1mm) but this wouldn't give a lower sensitivity (slope actually higher). To get a slope lower, a gain smaller than the ideal one (result in green) would bring the functioning point (believed to be 500µm) closer to the peak. That would lower sensitivity and could explain a huge discrepancy between vertical readings and known actual stiffness bounds.

This could be easily checked by looking for the actual raw peak analog tension output, when slowly closing the gap the max should be 5V. And when inputing ~4.3V to the display it should show ~500.000 on vertical scale. If both hold, then I'm in mental confusion again.
Title: Re: EM Drive Developments - related to space flight applications - Thread 2
Post by: aero on 03/06/2015 05:28 pm
I just finished running a meep case this morning which gave results that I don't understand at all. The detected forces were so large as to indicate an error in the model set-up. Probably a resolution issue again with the detectors within the same cells as the structure. But I can handle that.

What was weird were the field patterns. The cylindrical cavity appeared to be in perfect resonance with the energy range of the complex fields being about 3 times higher than the energy of the real fields. But the complex fields lagged the real fields by at least a sixth of a cycle. That is about 60 degrees.

Any guesses as to what that might mean and could it relate to the larger forces detected?
Title: Re: EM Drive Developments - related to space flight applications - Thread 2
Post by: frobnicat on 03/06/2015 05:55 pm
...

Frobnicat:

Your analysis and comments made me go back and look at the current alignment of the Eagleworks torque pendulum and the attached picture indicates that my recollection of the leveling of the bottom beam of the TP being a quarter bubble high at the vacuum chamber door end was a bit exaggerated due to parallax error and a just plain bad memory.  It looks now like less than tenth of bubble low at the door end of the vacuum chamber, but part of that apparent tilt may in reality be due to actual bending of the 1.50 inch square aluminum beam.

Best, Paul M.

The X+ (test article) beam end appears lower at Height2= 3.97'' than toward X- (under electronic stack) at Height1= 4.00, as if the assembly is tilted downward toward test article. This appears in contradiction to the previous hypothesis that the electronic stack was lower, and the bubble up orientation to the right if the top image is oriented with the test article (X+) on the right, ie has same orientation as bottom pictures (hard to tell). Also a tilt with downward slope toward test article would lower the stability of the system as the global centre of mass of rotating assembly is toward the backward electronic stack relative to axis (~6cm behind using your latest data on positions and weights). I will try to incorporate those new data points in the model.

Your answers to questions about factual aspects of the ongoing experiments are greatly appreciated here on this thread. I understand you wouldn't want or have time to respond to all interrogations about possible models, but to proceed with my attempts at quantifying thermal expansions impact on sustained displacement signals, I really need your point of view on the immediately appearing contradictions of stiffness around Z :

A calibration pulse of 29.1µN / 4.45 = 6.54µLbf applied at 11.25'' gives a torque of 6.54*11.25 = 73.6 µLbf-In.
Two 0.007 Lb-In/° flexure bearing in tandem give 0.014 Lb-In/°. At rest equilibrium the angular displacement of the arm to equilibrate the cal. pulse torque should be  73.6 * 10^-6 / 0.014 = 5.26 * 10^-3 ° = 9.17 * 10^-5 rad. This would correspond to a displacement at the LDS of 9.17*10^-5*0.352 = 32.3 µm (LDS distance from Z is 13.88''=.352m)

In most charts, the vertical scale reads between 1µm to 2.5µm deviation in response to 29.1µN pulses. In one chart we even have 5µm. See attached pictures. Why is there an apparent disparity from chart to chart ? Why do we have an order of magnitude less actual stiffness (from the flexure bearings, but that would agree also with oscillation periods) than apparent stiffness (apparent LDS readings relative to µN cal. pulses). ? Could it be that the vertical scale numbers are not correct and should read ~10µm when saying 1µm ?

My previous post (http://forum.nasaspaceflight.com/index.php?topic=36313.msg1342556#msg1342556) makes an attempt at a possible explanation (output gain of LDS...) for a 10 times factor on the vertical scale.

The disparity from chart to chart could then be explained by a varying Z tilt angle theta and/or CoM's of rotating assembly position D behind Z axis (by adding or removing small weights). Gravitational effects around a Z tilted pendulum would be equivalent to an horizontal pendulum with added stiffness k' = D M g sin(theta). This component could change enough (with differing test articles...) to explain the disparity between charts. But if the vertical scale remain as is, then it would imply variations of tilts of more than 10° !

Many thanks.
Title: Re: EM Drive Developments - related to space flight applications - Thread 2
Post by: Notsosureofit on 03/06/2015 09:01 pm
@ RODAL

Still struggling as to how to get the exact equation into the form;

f^2 = k^2 + [asymmetric terms(x)] such that, (f(0)^2 - f(L)^2) = [asymmetric terms(0)] - [asymmetric terms(L)]

That is, how to evaluate at constant k.   Any ideas ?
Title: Re: EM Drive Developments - related to space flight applications - Thread 2
Post by: Rodal on 03/06/2015 10:00 pm
@ RODAL

Still struggling as to how to get the exact equation into the form;

f^2 = k^2 + [asymmetric terms(x)] such that, (f(0)^2 - f(L)^2) = [asymmetric terms(0)] - [asymmetric terms(L)]

That is, how to evaluate at constant k.   Any ideas ?
The force derivation for a truncated cone seems to have been of a physical, intuitive character (if not so, please correct me).  From a physical standpoint, one can observe the following:

1) My understanding of your prior derivation is that it modeled the truncated cone as being traveled by plane waves (*), just like a plane wave travels a cylindrical cavity from one end to the other.  The difference is that in the truncated cone, the plane wave continuously, gradually experiences a lower natural frequency as it travels from the small diameter (cross-section with the highest natural frequency) end to the big diameter end (cross-section with the lowest natural frequency).  It should also continuously, gradually experience attenuation (loss in intensity) as it travels from the small end to the big end.'

___________
(*) Note: truncated cones have spherical waves traveling inside them, rather than plane waves.  So the model assumes a very large radius of curvature, such that the spherical wave is approximately flat.

CONICAL PIPES (like the truncated cone: continuous gradual change in diameter)
(http://i00.i.aliimg.com/wsphoto/v0/1546678994/carbon-conical-welded-steel-tubes.jpg)


2) The behavior in a cylindrical pipe containing two dielectric mediums is fundamentally different: instead of the continuous, gradual change in natural frequency of the truncated cone, the  cylindrical pipe containing two dielectric mediums experiences a discontinuous, step-change, a brutal change, going from medium 1 to dielectric medium 2.  This would be more like a pipe experiencing a discontinuous step-change in diameter all of a sudden, causing a sudden change in natural frequency and a sudden change in attenuation.


STEP-PIPE (the geometrical analog of the discontinuous step-change produced by a pipe having two dielectric mediums)

(http://www.feked.com/images/detailed/28/reducer_2.jpg)
Title: Re: EM Drive Developments - related to space flight applications - Thread 2
Post by: Stormbringer on 03/06/2015 10:07 pm
I know its a stretch thinking this fits in here but you guys have to see this:  http://www.sciencedaily.com/releases/2015/03/150306091617.htm

because QM and GRT reconciled? and gravitons. mustn't forget gravitons.
Title: Re: EM Drive Developments - related to space flight applications - Thread 2
Post by: Rodal on 03/06/2015 10:25 pm
@ RODAL

Still struggling as to how to get the exact equation into the form;

f^2 = k^2 + [asymmetric terms(x)] such that, (f(0)^2 - f(L)^2) = [asymmetric terms(0)] - [asymmetric terms(L)]

That is, how to evaluate at constant k.   Any ideas ?
The force derivation for a truncated cone seems to have been of a physical, intuitive character (if not so, please correct me).  From a physical standpoint, one can observe the following:

1) My understanding of your prior derivation is that it modeled the truncated cone as being traveled by plane waves (*), just like a plane wave travels a cylindrical cavity from one end to the other.  The difference is that in the truncated cone, the plane wave continuously, gradually experiences a lower natural frequency as it travels from the small diameter (cross-section with the highest natural frequency) end to the big diameter end (cross-section with the lowest natural frequency).  It should also continuously, gradually experience attenuation (loss in intensity) as it travels from the small end to the big end.'

___________
(*) Note: truncated cones have spherical waves traveling inside them, rather than plane waves.  So the model assumes a very large radius of curvature, such that the spherical wave is approximately flat.

CONICAL PIPES (like the truncated cone: continuous gradual change in diameter)
(http://i00.i.aliimg.com/wsphoto/v0/1546678994/carbon-conical-welded-steel-tubes.jpg)


2) The behavior in a cylindrical pipe containing two dielectric mediums is fundamentally different: instead of the continuous, gradual change in natural frequency of the truncated cone, the  cylindrical pipe containing two dielectric mediums experiences a discontinuous, step-change, a brutal change, going from medium 1 to dielectric medium 2.  This would be more like a pipe experiencing a discontinuous step-change in diameter all of a sudden, causing a sudden change in natural frequency and a sudden change in attenuation.


STEP-PIPE (the geometrical analog of the discontinuous step-change produced by a pipe having two dielectric mediums)

(http://www.feked.com/images/detailed/28/reducer_2.jpg)
@Mulletron and @Star-Drive advocate dielectrics inside these microwave cavity resonators.

So far these dielectrics (I assume for expediency) have had uniform dielectric properties.

Thinking along the lines of the discussion above, one could think of whether a functionally-graded dielectric, with a continuous change in dielectric properties along the longitudinal axis, may provide superior functionality than a dielectric with uniform properties.

Although a functionally-graded dielectric will be more expensive to procure, this could be tested by approximation, by using several coupled dielectric material segments with increasing dielectric constants, their total (additive) length equaling the length of the single dielectric used so far with uniform properties.
Title: Re: EM Drive Developments - related to space flight applications - Thread 2
Post by: RotoSequence on 03/06/2015 10:38 pm
I know its a stretch thinking this fits in here but you guys have to see this:  http://www.sciencedaily.com/releases/2015/03/150306091617.htm

because QM and GRT reconciled? and gravitons. mustn't forget gravitons.

I found the full paper online, if anyone wants to take a look: https://www.academia.edu/8604226/The_Schwarzschild_Solution_to_the_Nexus_Graviton_Field
Title: Re: EM Drive Developments - related to space flight applications - Thread 2
Post by: Notsosureofit on 03/06/2015 11:26 pm

2) The behavior in a cylindrical pipe containing two dielectric mediums is fundamentally different: instead of the continuous, gradual change in natural frequency of the truncated cone, the  cylindrical pipe containing two dielectric mediums experiences a discontinuous, step-change, a brutal change, going from medium 1 to dielectric medium 2.  This would be more like a pipe experiencing a discontinuous step-change in diameter all of a sudden, causing a sudden change in natural frequency and a sudden change in attenuation.



Yes, the case of a gradually changing dielectric constant is very similar to that of the tapered cavity.

The step-change involves a much more complicated situation w/ the necessity of including the reflected waves from the discontinuity.  But, it may well offer enhanced performance. (?)  These types of calculations have been done w/ iterative matrix math.  I'm going to try some ideas w/ programs I have to see if I can get a better feel for this case.  (1D calculations at least)

Added: 1D modes suggest that we only have to consider terms which contain b  (ie. X or X')  The others are plain waves and cancel.

 
Title: Re: EM Drive Developments - related to space flight applications - Thread 2
Post by: zen-in on 03/07/2015 04:06 am




Interesting hypothesis. You say that in this case the LDS readings would be more sensitive to a change in angle of the reflective mirror than a change in distance. The LDS is a small distance below the X axis, so a small angular deviation around X would amount to tiny displacement (d' - d) but the small angular deviation would itself tilt the mirror and that could change the LDS readings that is roughly proportional to the reflected light.

But then this coupling between angular effect and displacement would also be present for the main movements around Z. LDS is at 35cm from Z axis, I would say it is one order of magnitude more than the distance of mirror below X axis, the lever effect of displacement/angle ratio would be lower (less angle for same displacement) but still if angle deviation is to have a significant effect in case around X it should also have a significant effect around Z.

I read "most materials : including smooth but scattering surfaces", not only mirrors. Actually, mirrors may be a very special case. My question is now if the gain control of the system is consistent with the use of a mirror reflecting target. As seen qualitatively from attached figure : a perfect mirror would bounce back in the signal detector a lot more photons than a matte white. If the gain is calibrated against matte material (say, as hypothesis, default factory settings) the baseline distance from the mirror to have a tension that shows as 500µm would be much greater than 500µm.


Edit : BTW, how comes that at some very near distance there start to be less photons bouncing back in the signal detector ? Thank zen-in for bringing up interesting questions about LDS...

Here is a revised drawing that better illustrates how I think the LDS works.   The distance from the end of the LDS to the mirror would actually be much less but the proportion of the beam width at the mirror to distance is similar.  The light from the optical fiber, (green) has a higher dispersion near the target than a laser beam would.   The reflected pattern is equally dispersive, which has the effect of magnifying the reflected dot.  So in effect the receiver is sampling just a small area of the dot.   Small angular changes will produce a variation in amplitude of the received light if the dot pattern is gaussian because of this magnifying effect.   So just a few arc-seconds of rotation will register as a displacement even though the actual distance has not changed.

I think the reason why the LDS output has an opposite slope at close distances (likely < 10 µm) is because the two surfaces start acting like a waveguide when they are in close proximity.   Beamsplitters are made using this principle.
Title: Re: EM Drive Developments - related to space flight applications - Thread 2
Post by: Rodal on 03/07/2015 12:41 pm

2) The behavior in a cylindrical pipe containing two dielectric mediums is fundamentally different: instead of the continuous, gradual change in natural frequency of the truncated cone, the  cylindrical pipe containing two dielectric mediums experiences a discontinuous, step-change, a brutal change, going from medium 1 to dielectric medium 2.  This would be more like a pipe experiencing a discontinuous step-change in diameter all of a sudden, causing a sudden change in natural frequency and a sudden change in attenuation.



Yes, the case of a gradually changing dielectric constant is very similar to that of the tapered cavity.

The step-change involves a much more complicated situation w/ the necessity of including the reflected waves from the discontinuity.  But, it may well offer enhanced performance. (?) ....

If one argues that the truncated cone has enhanced performance over a cylindrical cavity containing two adjoining coupled dielectrics, then it can be argued that the dielectrics should be functionally graded such that the dielectric constant is a continuous function of the axial length, and the dielectric constant changes gradually in that direction. Or, as an approximation, to have short segments of dielectric materials with gradually increasing dielectric constant in the axial direction.

(http://peswiki.com/images/0/0b/Electromagnetic_drive_400.jpg)



Conversely, if the discontinuous jump in dielectric constant encountered in a cavity  containing two adjoining coupled dielectrics offers enhanced performance over the gradual geometrical change of a truncated cone, then it can be argued that the optimum geometrical shape would be a step-change in geometry, going from a small diameter to suddenly, discontinuously, to a larger diameter, for the same reason that it is argued that the discontinuous change in dielectric constant offers enhanced performance.

(http://www.feked.com/images/detailed/28/reducer_2.jpg)




Something like this is accomplished in the Cannae cavity design (practically a sudden change in geometry, going from a small diameter to a large diameter).  However, the dielectric in the Cannae cavity was located at the wrong location: with the dielectric, for example located at the right end of the cylindrical section:

CANNAE:

Tested design: natural frequency reaches a maximum between the ends:

Large Diameter (lower natural frequency) --> Small Diameter (higher natural frequency) --> Small Diameter & Dielectric (lower natural frequency)



It would have made more sense to have the dielectric at the large diameter end, in order to have this:

Natural frequency increases from left end to right end:

Large Diameter & Dielectric (lowest natural frequency)-->Large Diameter (intermediate natural frequency) --> Small Diameter (highest natural frequency) 


Title: Re: EM Drive Developments - related to space flight applications - Thread 2
Post by: Rodal on 03/07/2015 01:09 pm
...maybe he envisions this device as some sort of 'turbo-charger?'   That is something that adds to an already existing velocity, but won't function well, if at all when 'at rest.'  ...
A turbocharger, is a device that increases an engine's efficiency and power by forcing extra air into the combustion chamber. 

What is the "engine" that the EM Drive is turbocharging?

The bizarre nature of something needing to be free to accelerate for it to produce a force doesn't apply to the turbocharger or to the engine:  the engine that is being turbocharged does not need to be accelerating or even be in rigid body motion. Its center of mass can be completely stationary, and the turbocharged engine can then be used for electric power generation, for example, instead of for transporting people as in an automobile.

Of course, there are many kinds of EM-Drives:

here is an older type of EM-Drive    :)     :

Notice that "The strings of the guitar need to be moving for the EM Drive to produce a sound"   :)

(http://files.effectsdatabase.com/gear/pics/tmg_emdrive_001.jpg)

https://www.youtube.com/watch?v=JvDlp46rQ30
Title: Re: EM Drive Developments - related to space flight applications - Thread 2
Post by: Star-Drive on 03/07/2015 01:41 pm
...

I don't think the tilt of the balance beam about the X axis would have to be very much for the resulting change in received light amplitude to register as a shift in position.   The LDS has fractional micron resolution.    Assuming the light pattern from the fiber optic cable has a circular gaussian distribution, for any given distance the maximum light level hitting the detector is when the plane of the mirror is perpendicular to the central axis of the light beam.   Any small deviation ( arc-second) will reduce the light amplitude; which registers as a displacement.     It is impossible to align the LDS perfectly so the angle between the mirror and the light beam is never exactly 90 degrees in X and Y.    The expansion of the cavity due to heating has a very long time constant.   The slow drift in the position between RF pulses looks like a thermal response.

Interesting hypothesis. You say that in this case the LDS readings would be more sensitive to a change in angle of the reflective mirror than a change in distance. The LDS is a small distance below the X axis, so a small angular deviation around X would amount to tiny displacement (d' - d) but the small angular deviation would itself tilt the mirror and that could change the LDS readings that is roughly proportional to the reflected light.

But then this coupling between angular effect and displacement would also be present for the main movements around Z. LDS is at 35cm from Z axis, I would say it is one order of magnitude more than the distance of mirror below X axis, the lever effect of displacement/angle ratio would be lower (less angle for same displacement) but still if angle deviation is to have a significant effect in case around X it should also have a significant effect around Z.

Unfortunately the documentation for the Philtec D63 displacement sensor (http://forum.nasaspaceflight.com/index.php?action=dlattach;topic=36313.0;attach=810350) seems to give no information on reflective surface angular deviation impact on measurements. It indicates Ø 1.6 mm Target Spot Size. This is 3 times more than nominal operating distance of 500µm (0.5mm) so your drawing is a bit misleading with proportions. In reality the reflected dot is big relative to distance from reflector, would an angular deviation still impact the amount of light swallowed back ?

Is it you drawing or you found it elsewhere ? I ask the question because you mention the reflecting surface as being mirror. It does appear as a mirror in the pictures of pendulum at Eagleworks and this is stated explicitly in Brady's report page 3 (together with that LDS is photon time of flight, which is not, but let's proceed). But I don't find in the D63 documentation that a mirror should or could be used. It does say that These specifications represent best case performance where: the target is flat, smooth and highly reflective
This implies that the target may not be highly reflective (at the cost of decreased performance) but does highly reflective imply that a mirror is the best case ? My reading (but I'm not native English reader) is that reflective can mean a matte white material, that is a surface that does bounce near 100% of incoming photons, but in a more or less scattering manner, not necessarily as per perfect reflection. See attached picture : top the situation at hand at Eagleworks, bottom the default situation for which the LDS might be initially calibrated.

This could explain a lot about the disparity between vertical readings in the charts and stiffness parameters : when charts indicate ~1µm LDS deviation for 29.1µnN calibration pulses for instance, all could be much better explained and consistent if it meant 10µm instead.

To me there is no doubt that the LDS is operating in the far range : the cal. pulses are attractive by nature, and correspond to a displacement to the right (Y+) and that lowers the measured distance, as in the charts. Also operating in the near range would not only reverse this consistent orientation but would also increase the sensitivity : that would go against what is to be explained (apparent higher µm/µN stiffness than compatible with other known parameters, for 2 separate predictions from the dynamic periods of oscillations and the known stiffness of flexure bearings)

So, what anyone wanting to understand clearly the charts need is a way to explain a reduced sensitivity of the LDS, that would show 1µm displacement when in fact it is 10µm. Following on the D63 doc, there is a calibration procedure :
The effect of changing target reflectance is to shift the voltage output higher or lower. Factory calibrations have
the Peak Voltage set to 5.000 volts. A gain control is provided for calibration of the sensor output to various target surfaces. In-situ calibration is performed simply, by adjusting the sensor’s tip-to-target gap until the peak output voltage is attained, and then by using the gain control to set the peak voltage to full scale (5.000 volts). After setting the peak to 5 volts, the factory gap calibration chart applies for the target being measured. This procedure allows the sensor to be used to perform precision linear motion measurements on most materials.


I read "most materials : including smooth but scattering surfaces", not only mirrors. Actually, mirrors may be a very special case. My question is now if the gain control of the system is consistent with the use of a mirror reflecting target. As seen qualitatively from attached figure : a perfect mirror would bounce back in the signal detector a lot more photons than a matte white. If the gain is calibrated against matte material (say, as hypothesis, default factory settings) the baseline distance from the mirror to have a tension that shows as 500µm would be much greater than 500µm.

The protocol of tuning for nominal 500µm distance between optical fibre head and mirror is on page 3 of Brady et al report (anomalous...) :
Quote from: Brady et al. Anomalous thrust...
Prior to a test run data take, the LDS is positioned to a known displacement datum (usually 500 micrometers) via mechanical adjustments to its mounting platform. Gross adjustments are performed via set screws. Fine adjustments are performed using manually - operated calibrated screw mechanisms and a remotely controlled motorized mechanism that can be operated with the chamber door closed and the chamber at vacuum. The remote adjustment capability is necessary since the LDS datum will change whenever a change to the test facility environment affects the roll - out table or the chamber – e.g., whenever the chamber door is closed or latched and whenever the chamber is evacuated. Once the LDS displacement is adjusted in the final test environment, further adjustment between test run data takes is usually not required.

How this displacement of 500µm is known ? Has the procedure followed the step (as per D63 documentation) of homing to the distance corresponding to peak signal (whatever it's absolute level), calibrating the gain so that at this peak the signal is now 5V, and then stepping back until meeting ~4.3 V corresponding to 500µm nominal distance ? If this was not followed, or if the D63 is not mean to be operated with perfect mirror target (doubtful it wouldn't be mentioned in documentation, but who knows ?), it is possible the LDS is operating in a far range that has a much lower sensitivity (and linearity) than the nominal -2.7 mv/µm.

Edit : BTW, how comes that at some very near distance there start to be less photons bouncing back in the signal detector ? Thank zen-in for bringing up interesting questions about LDS...

Folks:

The Philtec fiber-optic cable used in their D63 reflective displacement sensor uses TWO (2) fiber optic bundles in the armored cable with one bundle being used as the transmit and the other bundle the receiver.  A description of how Philtec uses these two fiber-optic bundles to measure a distance with sub-micron resolutions is provided at their website: 

http://www.philtec.com/downloadssupport/documentlibrary/documents/applicationnotes/AboutTheSensors.pdf

Hope that helps

Best, Paul M.
Title: Re: EM Drive Developments - related to space flight applications - Thread 2
Post by: zen-in on 03/07/2015 03:39 pm


Folks:

The Philtec fiber-optic cable used in their D63 reflective displacement sensor uses TWO (2) fiber optic bundles in the armored cable with one bundle being used as the transmit and the other bundle the receiver.  A description of how Philtec uses these two fiber-optic bundles to measure a distance with sub-micron resolutions is provided at their website: 

http://www.philtec.com/downloadssupport/documentlibrary/documents/applicationnotes/AboutTheSensors.pdf

Hope that helps

Best, Paul M.

That would explain the near side of the response curve better than my theory.   The response to small angular shifts of the mirror would be similar.   If the change in CM from heating caused the angle of the mirror to shift closer to a perpendicular wrt to the LDS, it would register as a decreasing displacement.   This is an attempt to explain the small negative deviations in position as shown in the curve below.
Title: Re: EM Drive Developments - related to space flight applications - Thread 2
Post by: Rodal on 03/07/2015 07:43 pm

2) The behavior in a cylindrical pipe containing two dielectric mediums is fundamentally different: instead of the continuous, gradual change in natural frequency of the truncated cone, the  cylindrical pipe containing two dielectric mediums experiences a discontinuous, step-change, a brutal change, going from medium 1 to dielectric medium 2.  This would be more like a pipe experiencing a discontinuous step-change in diameter all of a sudden, causing a sudden change in natural frequency and a sudden change in attenuation.



Yes, the case of a gradually changing dielectric constant is very similar to that of the tapered cavity.

The step-change involves a much more complicated situation w/ the necessity of including the reflected waves from the discontinuity.  But, it may well offer enhanced performance. (?)  These types of calculations have been done w/ iterative matrix math.  I'm going to try some ideas w/ programs I have to see if I can get a better feel for this case.  (1D calculations at least)

Added: 1D modes suggest that we only have to consider terms which contain b  (ie. X or X')  The others are plain waves and cancel.

For the geometry and dielectric considered by @aero:

diameter = 8.278945 centimeter;
length = 12.24489 centimeter;
dielectricLength2 = 2.7282494103102 centimeter;
cVacuum = 299792458 meter/second;
cMedium1 = cVacuum*100 centimeter/meter;
cMedium2 = cVacuum*100/Sqrt[2.3] centimeter/meter;

The first two terms inside the first square root are:

(+(3.96427*10^(-19))* b^2)/(6.67825*10^(-39))

and

(+(1.19817*10^(-19)* p^2)/(6.67825*10^(-39))

The two terms inside the second square root (which is the third term inside the first square root) are

+4.65027*10^(-39)* (b^2)* (p^2) /((6.67825*10^(-39))^2)

and

+1.91947*10^(-39)*(p^4) /((6.67825*10^(-39))^2)

So, for this geometry and dielectric, all these terms are positive.



Where

b := If[modetype == "TM", xbesselzeros[[m + 1, n]]/Pi,  If[modetype == "TE", xprimebesselzeros[[m + 1, n]]/Pi]]

Where  Xmn=xbesselzeros[[m+1,n]] (the zeros of the Bessel function)

xbesselzeros = {{2.40483, 5.52008, 8.65373, 11.7915, 14.9309}, {3.83171, 7.01559,
  10.1735, 13.3237, 16.4706}, {5.13562, 8.41724, 11.6198, 14.796,
  17.9598}, {6.38016, 9.76102, 13.0152, 16.2235, 19.4094}, {7.58834,
  11.0647, 14.3725, 17.616, 20.8269}, {8.77148, 12.3386, 15.7002,
  18.9801, 22.2178}, {9.93611, 13.5893, 17.0038, 20.3208,
  23.5861}, {11.0864, 14.8213, 18.2876, 21.6415, 24.9349}, {12.2251,
  16.0378, 19.5545, 22.9452, 26.2668}, {13.3543, 17.2412, 20.807,
  24.2339, 27.5837}, {14.4755, 18.4335, 22.047, 25.5095, 28.8874}}

and where  X'mn=xprimebesselzeros [[m+1,n]] (the zeros of the derivative of the Bessel function)

xprimebesselzeros = {{3.83171, 7.01559, 10.1735, 13.3237, 16.4706}, {1.84118, 5.33144,
  8.53632, 11.706, 14.8636}, {3.05424, 6.70613, 9.96947, 13.1704,
  16.3475}, {4.20119, 8.01524, 11.3459, 14.5858, 17.7887}, {5.31755,
  9.2824, 12.6819, 15.9641, 19.196}, {6.41562, 10.5199, 13.9872,
  17.3128, 20.5755}, {7.50127, 11.7349, 15.2682, 18.6374,
  21.9317}, {8.57784, 12.9324, 16.5294, 19.9419, 23.2681}, {9.64742,
  14.1155, 17.774, 21.2291, 24.5872}, {10.7114, 15.2867, 19.0046,
  22.5014, 25.8913}, {11.7709, 16.4479, 20.223, 23.7607, 27.182}}

For example, for mode TE01, b= xprimebesselzeros[[1,1]]/Pi= 3.83171 / Pi =  3.83171 / 3.14159
Title: Re: EM Drive Developments - related to space flight applications - Thread 2
Post by: Notsosureofit on 03/07/2015 08:59 pm
@ RODAL

Miss. rambling looking for terms that might drop out:

f^2 = (1/4)*((-4*b^2*(dD1^2-dD2^2)*((L1/c1)^2-(L2/c2)^2))/(dD1^2*dD2^2)

      +(L1^2/c1^2 + L2^2/c2^2)*p^2

      (-/+)2*Sqrt[(-4*b^2*((L1/c1)^2-(L2/c2)^2)*((dD1*L1/c1)^2-(dD2*L2/c2)^2)*p^2)

      /(dD1^2*dD2^2)

      +(L1^2*L2^2*p^4)/(c1^2*c2^2))]

      /((L1/c1)^2-(L2/c2)^2)^2

If b = 0 then;

f^2 = [((L1/c1)^2+(L2/c2)^2)*p^2 (-/+) 2*(L1/c1)*(L2/c2)*p^2)] /(4*(L1^2/c1^2-L2^2/c2^2)^2)

If c1 = c2 = 1 then;

f^2 = [(L1^2+L2^2)*p^2 (-/+) 2*L1*L2*p^2] /(4*(L1^2-L2^2)^2)

    = [p^2/4] * [L1^2 (-/+) 2*L1*L2 + L2^2] / [(L1^2-L2^2)*(L1^2-L2^2)]

    = [p^2/4] * [(L1 (-/+) L2)^2] / [(L1-L2)^2*(L1+L2)^2]

    = [p^2/4] / [(L1 (-/+) L2)^2]

implies this p ==> p*2*pi,  that is to say my "f's" are angular frequencies and probably should be "w's".

EDIT:  Nope, just me forgetting I made c = 1.   Must be gettin' (really) old !
Title: Re: EM Drive Developments - related to space flight applications - Thread 2
Post by: frobnicat on 03/07/2015 10:10 pm


Folks:

The Philtec fiber-optic cable used in their D63 reflective displacement sensor uses TWO (2) fiber optic bundles in the armored cable with one bundle being used as the transmit and the other bundle the receiver.  A description of how Philtec uses these two fiber-optic bundles to measure a distance with sub-micron resolutions is provided at their website: 

http://www.philtec.com/downloadssupport/documentlibrary/documents/applicationnotes/AboutTheSensors.pdf

Hope that helps

Best, Paul M.

That would explain the near side of the response curve better than my theory.   The response to small angular shifts of the mirror would be similar.   If the change in CM from heating caused the angle of the mirror to shift closer to a perpendicular wrt to the LDS, it would register as a decreasing displacement.   This is an attempt to explain the small negative deviations in position as shown in the curve below.

Yes that makes perfect sense as for the change of slope when approaching nearer than 175µm absolute distance from target (not close enough for wavelength effects of such magnitude). But if the hypothesis is that a change of CoM can make a tilt around an axis (my bet would be around Z much greater than X) then the question to answer quantitatively is the ratio of measurement change due to angle and change due to linear displacement. Since we are talking about tilts around an axis, the two amounts of variations are mechanically linked, we can't have an angle variation of delta_alpha radians without having a linear displacement of delta_d = D delta_alpha, where D is the distance from the axis. In other words there can't be a significant delta_alpha without a significant delta_d for either rotation around X (D=a bit above 1'' ?) or Z (D=13.88''). With such constraint, what would impact more the measurement, the delta_d or the delta_alpha, or both about same order of magnitude ? I would tend to say that the delta_d influence is linear delta_measure = cst1 * delta_d, while the delta_alpha would be quadratic (cosine like) delta_measure = - cst2 * delta_alpha² since it is symmetric around a maximum. For small deltas that would strongly favor delta_d influence, but that don't preclude -cst2 * delta_alpha² being the leader depending on constant factors cst1 and cst2, and this is only valid for a delta_alpha starting from a perfect perpendicularity...

Quote from: zen-in
If the change in CM from heating caused the angle of the mirror to shift closer to a perpendicular wrt to the LDS, it would register as a decreasing displacement.

Interestingly, depending on the starting angle Alpha (0 for perpendicular), a strong delta_alpha dependent variation could be either in a direction or in the opposite direction : for cosine like effect measure1=cst3 -cst2*Alpha², measure2=cst3-cst2*(Alpha+delta_alpha)²  =>  delta_measure=-cst2*(delta_alpha+2*Alpha)*delta_alpha. If starting angle Alpha is not strictly 0 and delta_alpha is kept small relative to this rest default bias then delta_measure=-2*cst2*Alpha*delta_alpha. The sense of variation of delta_measure relative to delta_alpha would depend on the direction in which the initial non perpendicularity is biased. So no argumentation about orientations could be settled without knowing the direction of the angle bias relative to perfect perpendicularity.

But it is still unsure if the magnitude of the delta_angle effect is enough to be significant at all relative to the delta_d effect, need to be quantified, from the documentation. Unfortunately the doc kindly pointed to by Paul March (http://www.philtec.com/downloadssupport/documentlibrary/documents/applicationnotes/AboutTheSensors.pdf) give quantitative perpendicularity dependence information only for models D20 and D170, we are interested in an intermediate D63. Bottom of post I attached screen grab of the relevant charts. Is 63 to be interpolated between 20 and 170 ? From clearer D170 charts, a quadratic regression (http://www.had2know.com/academics/quadratic-regression-calculator.html) from the values of the peaks depending on angle  (0°->5, 1°->4.82, 2°->4.57, 3°->3.88, 4°->3.15, 5°->2.17) gives peak_tension_in_volts = 4.9559 - 0.1128*alpha² with alpha in degrees. I overlay the outputs of this quadratic model as red horizontal lines above the chart, the fit is consistent with a cosine effect, seems a good model as a first approximation. D20 is murkier... and how would we be supposed to interpolate the quadratic parameters of D63 between the quadratic parameters of D20 and D170 ? Or anyone with happy google karma to find those curves for D63 ?

Anyhow, if the LDS was calibrated perpendicularly and later used at another angle that could explain the apparent lack of punch of the vertical scale : indicating delta 1µm when really it should rather indicate more like delta 10µm. I will insist on that aspect until it is clarified, as it is of paramount importance to validate any sound mechanical model to rule out or not thermal displacements as sources of sustained displacements interpreted as sustained thrusts. The yellow curve at 2° on the perpendicularity dependence chart (for D170, bottom) looks a lot like the situation I drawn in green for a weak gain :
(http://forum.nasaspaceflight.com/index.php?action=dlattach;topic=36313.0;attach=812059;image)

A gain smaller than the ideal one (result in green) would bring the functioning point (believed to be 500µm) closer to the peak. That would lower sensitivity and could explain a huge discrepancy between vertical readings and known actual stiffness bounds.

This could be easily checked by looking for the actual raw peak analog tension output, when slowly closing the gap the max should be 5V. And when inputing ~4.3V to the display it should show ~500.000 on vertical scale.





Title: Re: EM Drive Developments - related to space flight applications - Thread 2
Post by: frobnicat on 03/07/2015 11:20 pm
Follow-up on the respective contribution to the signal of changes delta_alpha vs delta_d knowing that at a distance D from a pivot delta_d = D delta_alpha. Taking the model D170 (much more sensitive to angle) with a huge bias angle of starting Alpha=5° as a worst case, from my previous post delta_measure=-2*cst2*Alpha*delta_alpha and cst2 = .1128 is in good agreement with documentation. Provided that the calibration to the gain was done according to the procedure described in Philtec docs (check that the peak is 5V) at this bias angle of 5°, the relative lower reflectance due to angle would be compensated by the gain. At 5° (for D170) this is a relative gain of 5/2.17=2.30. Not clear if all is scaled 2.30 times by this compensating calibration including the cst2 factor... have to think about it. Let's say it is, cst2 is now .1128*2.30=.26.

We now have an effect for angle delta_measure(volts)=-2*cst2*Alpha*delta_alpha=+or-2.6*delta_alpha (in degrees, +or- depending on Alpha being -5° or +5°)
For same delta_alpha in degrees, at distance D from pivot the displacement is delta_d = D delta_alpha*pi/180. The sensitivity of D63 at nominal distance  is -2.7 mV/µm=2700V/m : delta_measure(volts)=2700*delta_d=47 D delta_alpha. At D=13.88'' (~.35m) around Z that gives delta_measure(volts)=16.6 delta_alpha. At D=1'' around X (?) delta_measure(volts)=1.2 delta_alpha.

Ahem, relatively inconclusive, I expected a clear bound of an order of magnitude or more, but the blue values of the effect of linear displacement alone are not that higher than the red value of the effect of rotation. Note however that would be with a highly angle sensitive D170 angle dependence values, inclined at a worst bias angle of 5°.

Unless anyone has courage to check those laborious calculations and find a blunder, I would consider the case settled : A reasonably well perpendicular (within a few °) D63 that measures tangential movements of a beam at 1'' or more from a pivot will be mostly sensitive to the linear displacement and not to the angular deviation linked to such linear displacement.

This doesn't preclude a few ° angle deviation from perpendicularity between calibration phase and usage to make the D63 work at a less than optimal absolute distance to target, nearer to the peak, at decreased linear sensitivity : we should really concentrate on the vertical scale immediate contradictions (in µm/µN) with calibration pulses of 29.1µN giving readings between 1µm to 5µm displacements instead of the expected 32.3µm from the flexure bearings alone (http://forum.nasaspaceflight.com/index.php?topic=36313.msg1342605#msg1342605). I'm waiting for Star-Drive comment on that.
Title: Re: EM Drive Developments - related to space flight applications - Thread 2
Post by: zen-in on 03/08/2015 04:09 am
The Philtec datasheets describe the RC type distance sensors (RC90, etc) as having 2 adjacent bundles of fibers.  The distance sensor used by Eagle Labs is a D63.  For a Philtec sensor with 2 bundles of fibers the change in output voltage due to an angular change is dependent on the orientation of the sensor. 

Whether the small negative slope in the thrust waveform after RF power is applied is from a change in CM causing a tilt in the mirror or is actually a displacement is impossible to determine emperically.   However it is there - along with the anomalous thrust signature, the magnetic interaction at 5.6 A., and the calibration waveforms from the capacitor.  Maybe what looks like a thermally induced drift is just random movement.

In the Aug. AIAA paper one of the thrust waveforms (shown below) has an unusual shape.    It is bracketed by 300 V. calibration pulses on the capacitor.  The calibration pulses have some overshoot/undershoot and ringing that are the typical response of an underdamped system to step responses.   This response is determined by the time constant and damping of the apparatus and should not change whatever the driving function is.   For example if a fly bumped into the TP and imparted the same momentum as the capacitor with its 300V did, the response should be virtually identical.    So why is the response from the RF being switched on and then off (anomalous thrust) so different?
Title: Re: EM Drive Developments - related to space flight applications - Thread 2
Post by: frobnicat on 03/08/2015 10:00 am
The Philtec datasheets describe the RC type distance sensors (RC90, etc) as having 2 adjacent bundles of fibers.  The distance sensor used by Eagle Labs is a D63.  For a Philtec sensor with 2 bundles of fibers the change in output voltage due to an angular change is dependent on the orientation of the sensor. 

So ? Are you suggesting the D type should be replaced by a RC type ? The D type that is used now at Eagleworks shows no such dependence on angular change relative to some preferred axis since (from this documentation (http://www.philtec.com/downloadssupport/documentlibrary/documents/applicationnotes/AboutTheSensors.pdf)) ...D model probes with one round fiberoptic bundle... We don't know the details but "round" implies the bundle is rotationally symmetric around its axis.

Quote
Whether the small negative slope in the thrust waveform after RF power is applied is from a change in CM causing a tilt in the mirror or is actually a displacement is impossible to determine emperically.

You don't acknowledge the fact that at distance D from a pivot there is a mechanical link between tangential displacement delta_d and "tilt" delta_alpha as per delta_d=D*delta_alpha ? To explain a tilt you would have to explain a tangential displacement as well, unless D=0 or very small (less than 1''). Do you see a reason why, due to a CoM position change, the mirror would tilt around a pivot less that 1'' from its centre ? See attached drawing. Given the mechanical relation between delta_alpha and delta_d, my 2 previous posts show that for D>1'' and a D63 at less than 5° the delta_d aspect is very likely dominant over the delta_alpha impact on measures.

Quote
   However it is there - along with the anomalous thrust signature, the magnetic interaction at 5.6 A., and the calibration waveforms from the capacitor.  Maybe what looks like a thermally induced drift is just random movement.

It was acknowledged as thermally induced drift by Paul March, one time (in thread 1) (http://forum.nasaspaceflight.com/index.php?topic=29276.msg1254143#msg1254143) as a change in equilibrium rest position due to heating of flexure bearings by IR radiations, another one (in thread 2) (http://forum.nasaspaceflight.com/index.php?topic=36313.msg1331580#msg1331580) as a change of CoM linked to a continued expansion of test article even after power was stopped (when commenting about the orientations of consequences of buckling). Anyway, it appears too systematic to be just "random movement".

Quote
In the Aug. AIAA paper one of the thrust waveforms (shown below) has an unusual shape.    It is bracketed by 300 V. calibration pulses on the capacitor.  The calibration pulses have some overshoot/undershoot and ringing that are the typical response of an underdamped system to step responses.   This response is determined by the time constant and damping of the apparatus and should not change whatever the driving function is.   For example if a fly bumped into the TP and imparted the same momentum as the capacitor with its 300V did, the response should be virtually identical.    So why is the response from the RF being switched on and then off (anomalous thrust) so different?

Yes this fig. 22 chart (TE012, 2.6W, 55.4µN reported) is unusual, not time stamped, no vertical scale indications, cal. pulses at 300V 60.1µN instead of the usual 200V 29.1µN (note the non relative proportionality), and very smooth rises and falls for thrust pulses compared to cal. pulses. The overshoot and undershoot would be the same for responses to step rises and falls. The most likely explanation for the lack of ringing for the thrust pulse compared to cal. pulses is that the thrust case the driving function would not be steplike. That particular chart was discussed in thread 1 : http://forum.nasaspaceflight.com/index.php?topic=29276.msg1280094#msg1280094. And even for more "ringing" thrusts responses (more consistent with cal. pulses, like in fig.19) there is still not enough ringing to be explained by a pure step thrust. See bottom pic. in this link (http://forum.nasaspaceflight.com/index.php?topic=29276.msg1280037#msg1280037) to have an idea of what might be the shape of the driving function to explain that. My recollection is that all this semi-quantitative reasoning on relative ringing overshoots hasn't reached general consensus.

Title: Re: EM Drive Developments - related to space flight applications - Thread 2
Post by: zen-in on 03/08/2015 05:02 pm
Yes this fig. 22 chart (TE012, 2.6W, 55.4µN reported) is unusual, not time stamped, no vertical scale indications, cal. pulses at 300V 60.1µN instead of the usual 200V 29.1µN (note the non relative proportionality), and very smooth rises and falls for thrust pulses compared to cal. pulses. The overshoot and undershoot would be the same for responses to step rises and falls. The most likely explanation for the lack of ringing for the thrust pulse compared to cal. pulses is that the thrust case the driving function would not be steplike. That particular chart was discussed in thread 1 : http://forum.nasaspaceflight.com/index.php?topic=29276.msg1280094#msg1280094. And even for more "ringing" thrusts responses (more consistent with cal. pulses, like in fig.19) there is still not enough ringing to be explained by a pure step thrust. See bottom pic. in this link (http://forum.nasaspaceflight.com/index.php?topic=29276.msg1280037#msg1280037) to have an idea of what might be the shape of the driving function to explain that. My recollection is that all this semi-quantitative reasoning on relative ringing overshoots hasn't reached general consensus.

The driving function consists of step functions.   The RF is switched on with a predetermined power level and after a few seconds is switched off.     For the emdrive to exhibit a different response to the RF step function compared to the capacitive calibration step function there would have to be some energy storage/release mechanism beyond just having a high Q.   And for different RF frequencies this appears to change because the response waveforms are different.    The first waveform does show a step-like response initially, but without as much ringing.   Then there is a ramp function.  It looks like there there are a couple of things happening there.    Other thrust waveforms have more similarity to the shape of the step response from the capacitor at different voltages, but are less damped.
Title: Re: EM Drive Developments - related to space flight applications - Thread 2
Post by: Notsosureofit on 03/08/2015 08:55 pm
FYI

Just to get a feel for dielectrics I tried the straight cylinder with a uniform dielectric variation from end to end.

Using the same simpleminded approximation as the tapered cylinder gives:

del f = ( f/(2*c^2)) * (c1^2-c2^2)

T =  (h*f/(2*L*c^2)) * (c1^2-c2^2)

NT = (P*Q/(4*pi*L*f*c^2)) * (c1^2-c2^2)

Which is a lot simpler than I might have expected.  Perhaps the approximation is too simple or it represents a special case with " uniform dielectric variation from end to end "  rather than a straight conical insert.

In any case, it seems to be telling me to look for the proper integral form to be doing these.


Title: Re: EM Drive Developments - related to space flight applications - Thread 2
Post by: Rodal on 03/08/2015 10:08 pm
FYI

Just to get a feel for dielectrics I tried the straight cylinder with a uniform dielectric variation from end to end.

Using the same simpleminded approximation as the tapered cylinder gives:

del f = ( f/(2*c^2)) * (c1^2-c2^2)

T =  (h*f/(2*L*c^2)) * (c1^2-c2^2)

NT = (P*Q/(4*pi*L*f*c^2)) * (c1^2-c2^2)

Which is a lot simpler than I might have expected.  Perhaps the approximation is too simple or it represents a special case with " uniform dielectric variation from end to end "  rather than a straight conical insert.

In any case, it seems to be telling me to look for the proper integral form to be doing these.

where, presumably,

c1 = c/Sqrt[relativeElectricPermittivity1*relativeMagneticPermeability1]

and

c2 = c/Sqrt[relativeElectricPermittivity2*relativeMagneticPermeability2]

therefore:

NT = (P*Q/(4*pi*L*f)) * ((1/(relativeElectricPermittivity1*relativeMagneticPermeability1))-(1/(relativeElectricPermittivity2*relativeMagneticPermeability2)))

for dielectrics such that

relativeMagneticPermeability1 = relativeMagneticPermeability2 =1

this simplifies further to:

NT = (P*Q/(4*pi*L*f)) * ((1/(relativeElectricPermittivity1))-(1/(relativeElectricPermittivity2)))

For example, for a cavity containing two coupled sections, one empty with vacuum and the other one with HD PE with relativeElectricPermittivity2 = 2.3

one gets

NT = (P*Q/(4*pi*L*f)) *(1 - (1/2.3))
     = (P*Q/(L*f)) *0.04498

which goes to infinity for L->0....

Mmmm,  how could that be ?

What happened to the expression (L1/c1 -L2/c2) appearing in the solution of the cylindrical cavity with coupled dielectrics?  How did L1 and L2 disapear ?
Title: Re: EM Drive Developments - related to space flight applications - Thread 2
Post by: Notsosureofit on 03/08/2015 11:12 pm
Well, this isn't going to work for a step function the way it is.  We need to find the integral representation along the axis to be able to do that.  This is only a close (?) approx for the uniform variation.

L may go down but f goes up.......

c is the only thing that varies here (no step)

Might try a fourier expansion next.  Have to think about that.....

ADDED: Can you generate f^4 for the step function?  Hard to do by hand on Post-it Notes !
Title: Re: EM Drive Developments - related to space flight applications - Thread 2
Post by: Rodal on 03/09/2015 12:21 am
Well, this isn't going to work for a step function the way it is.  We need to find the integral representation along the axis to be able to do that.  This is only a close (?) approx for the uniform variation.

L may go down but f goes up.......

c is the only thing that varies here (no step)

Might try a fourier expansion next.  Have to think about that.....

ADDED: Can you generate f^4 for the step function?  Hard to do by hand on Post-it Notes !

Not sure what you mean.

A fourier expansion of some function in terms of frequency as a parameter?  What function of frequency?

Or, expand (the expression for the frequency of a cylindrical cavity containing two dielectrics) in terms of a Fourier expansion up to the fourth term.  A Fourier expansion of the frequency in terms of what parameter?

Or, would you like me to express "p" as a function of frequency ? (this can be done: it is a closed-form expression)
Title: Re: EM Drive Developments - related to space flight applications - Thread 2
Post by: Mulletron on 03/09/2015 08:44 am
http://ind-tech.com/wp-content/uploads/2014/11/Investment-Memorandum-final.pdf

Guess this didn't work out. It is about 10 years old. There is useful information to be gained from the names and claims made in this document.


On a separate note, in my continuing effort to find pre-existing science to explain the "anomalous thrust" I am surprised that I never uttered the words "cavity optomechanics" until today. This is a much narrower focus than "cavity QED"...which is the framework I am trying to place the "QV momentum transfer" mechanism into. Lots of interesting reading in that very narrow field of study.

I'll post some work related to "cavity optomechanics" in further posts.

I'm not on here to write my own theory of how these thrusters might work; just dig up existing theory. I believe there is plenty of existing science around to do that. I just don't see the point of trying to write new equations which try to fit the data (bouncing around well understood photons is old news * and alone doesn't make thrusters move), when what is really called for is an existing mechanism of action that makes sense. Kinda like this guy said: http://forums.sufficientvelocity.com/posts/2867270/

That is a good thread BTW. Those guys are harsh and I like it. I value reading outside opinions because I don't want to get caught up in my own little bubble and lose insight, like I did for a large part of thread 1.

I believe that a theory explaining Emdrive doesn't require any new science; just application of existing science. When I place myself in the shoes of some physicist out there, and try to view things from that perspective, the last thing I would want to read is some "theory in a bottle" that fully explains Emdrive. I'd rather see established concepts put forth by many experts in their own fields, that can be applied toward an experiment.

I fully believe that marrying cavity optomechanics, with the vacuum momentum mechanisms dug up, and the insight gained from the performance metrics of different mode shapes (experiment matched model and model agreed to calcs done here), and the experimental work at Eagleworks (who is an unconnected 3rd party to the inventors) is enough to warrant further explorations of Emdrive.

To sum all that up, we don't need to invent a new wheel here. There's plenty of wheel parts laying around to build a nice shiny new one that does new things.


It cost me a few hundred bucks to gather the materials to put together a simple test, that is nothing for a lab with an actual science budget. If I'm willing to risk some cash (less than the cost of a new smart phone) trying to answer a question, what is holding up the pros out there?

What is it going to take to get a school or a lab to make or break this silly copper kettle thruster already?

I'm not willing to pass up an opportunity to test something which could go on to be the key to unlocking Humanity's Manifest Destiny; however unlikely. Neither should anyone else with the resources to do the same.

At least Boeing had a look and turned him down, so kudos for that. I haven't seen a clear reason why (they probably already had one,  ;) ). The reason is probably buried by an NDA. (If anyone has insight into the Boeing part of this, please provide.)
BZ to Eagleworks for looking, where work is ongoing.

I still can't believe how Shawyer had to go to China to further his efforts.

Okay so for those naysayers out there who think Emdrive is total baloney and there is no science to explain it and that it must violate conservation of momentum, therefore the experiments are all flawed........those arguments are now invalid until proven otherwise.

*Disclaimer: The work being done here with calculating mode shapes, simulation and application thereof is new and very valuable.

Edit:
In contrast to my old somewhat naive interpretation of the physics behind Emdrive (mostly in thread 1 playing around with inertia), I see no evidence which suggests any modification of old physics is required, nor is any new physics.



Title: Re: EM Drive Developments - related to space flight applications - Thread 2
Post by: Notsosureofit on 03/09/2015 03:39 pm
COFFEE BREAK:

Let me back up a bit.

My initial question was:

In General Relativity (GR), if an RF cavity subject to an accelerating frame of reference (AFR) displays asymmetric frequency dispersion, will an asymmetric RF cavity which exhibits wavelength dispersion generate an accelerating frame of reference ?

The two "simple" approximations, "tapered cylindrical cavity" and "cylindrical cavity w/ uniformly varying dielectric" can be evaluated at the end plates because the difference can be viewed as the result of an integration along the asymmetric axis.

When the cavity is much more complicated, as in the L1,c1 plus L2,c2 case, for which we have an exact frequency solution thanks to a super effort by RODAL, an evaluation just at the end plates is no longer valid. The internal details must now be included in the integrated function.

How to generate the required function along the asymmetric axis is my current concern.  A guess might be something like dF(r,theta,z)/dz, which might be represented by a Fourier series expansion in z ?? 
Title: Re: EM Drive Developments - related to space flight applications - Thread 2
Post by: Rodal on 03/09/2015 04:07 pm
EXACT, CLOSED-FORM SOLUTION FOR A CYLINDRICAL CAVITY ELECTROMAGNETIC RESONATOR CONTAINING TWO COUPLED ADJOINING DIELECTRICS FILLING THE CAVITY

Continuing from http://forum.nasaspaceflight.com/index.php?topic=36313.msg1342159#msg1342159

Here I post both

a) the expression for “p” (mode shape quantum number for longitudinal direction) in terms of a given frequency fmnp, where m, n, p are quantum mode shape numbers such that: "m" is the number corresponding to the circumferential-polar-direction, "n" is the number corresponding to the  radial--polar-direction and "p" is the number corresponding to the axial-polar-direction.

and

b) the expression for frequency fmnp in terms of a given “p”

where symbols are defined as follows:

L1 = dielectricLength1
L2 = dielectricLength2

and where  L1 + L2 = length where "length" is the total internal length of the cylindrical cavity, such that:

L1 = length - L2;      (dielectricLength1 = length - dielectricLength2;)
 
and where we define the following dimensionless ratios:

dD1 = diameter/L1;  (dDielectric1 = diameter/dielectricLength1;)
dD2 = diameter/L2;  (dDielectric2 = diameter/dielectricLength2;)

The dimensionless quantity "b" is defined as follows:

b := If[modetype == "TM", xbesselzeros[[m + 1, n]]/Pi,  If[modetype == "TE", xprimebesselzeros[[m + 1, n]]/Pi]]

where Pi=3.14159265359...

and where  Xmn=xbesselzeros[[m+1,n]] (the zeros of the Bessel function)

xbesselzeros = {{2.40483, 5.52008, 8.65373, 11.7915, 14.9309}, {3.83171, 7.01559,
  10.1735, 13.3237, 16.4706}, {5.13562, 8.41724, 11.6198, 14.796,
  17.9598}, {6.38016, 9.76102, 13.0152, 16.2235, 19.4094}, {7.58834,
  11.0647, 14.3725, 17.616, 20.8269}, {8.77148, 12.3386, 15.7002,
  18.9801, 22.2178}, {9.93611, 13.5893, 17.0038, 20.3208,
  23.5861}, {11.0864, 14.8213, 18.2876, 21.6415, 24.9349}, {12.2251,
  16.0378, 19.5545, 22.9452, 26.2668}, {13.3543, 17.2412, 20.807,
  24.2339, 27.5837}, {14.4755, 18.4335, 22.047, 25.5095, 28.8874}}

and where  X'mn=xprimebesselzeros [[m+1,n]] (the zeros of the derivative of the Bessel function)

xprimebesselzeros = {{3.83171, 7.01559, 10.1735, 13.3237, 16.4706}, {1.84118, 5.33144,
  8.53632, 11.706, 14.8636}, {3.05424, 6.70613, 9.96947, 13.1704,
  16.3475}, {4.20119, 8.01524, 11.3459, 14.5858, 17.7887}, {5.31755,
  9.2824, 12.6819, 15.9641, 19.196}, {6.41562, 10.5199, 13.9872,
  17.3128, 20.5755}, {7.50127, 11.7349, 15.2682, 18.6374,
  21.9317}, {8.57784, 12.9324, 16.5294, 19.9419, 23.2681}, {9.64742,
  14.1155, 17.774, 21.2291, 24.5872}, {10.7114, 15.2867, 19.0046,
  22.5014, 25.8913}, {11.7709, 16.4479, 20.223, 23.7607, 27.182}}

For example, for mode TE01, b= xprimebesselzeros[[1,1]]/Pi= 3.83171 / Pi =  3.83171 / 3.14159




NUMERICAL EXAMPLE: we take the case used by @aero, containing a dielectric2 of HD PE material with dielectric constant (relative electric permitivity) = 2.3, and a dielectric1 being the empty portion of the cavity, under vacuum:

diameter = 0.08278945 meter;
length =  0.1224489 meter;
dielectricLength2 = 0.027282494103102 meter;
cMedium1 = cVacuum; cVacuum = 299792458 meter/second;
cMedium2 = cVacuum/Sqrt[2.3] ;  (relative electric permittivity=2.3;relative magnetic permeability=1)



Results:

modetype = "TE"; m = 1; n = 1; p = 1; root1 = 2.26774 GHz
modetype = "TE"; m = 1; n = 1; p = 2; root1 = 2.93557 GHz
modetype = "TM"; m = 0; n = 1; p = 2; root1= 3.37114 GHz

For this case, root1 is real, there is no need to consider root2. A number of modes are cut-off, for example modes TE011 and TM011 are cut-off

Title: Re: EM Drive Developments - related to space flight applications - Thread 2
Post by: Mulletron on 03/09/2015 05:06 pm
Well I called in my old buddies from school to help out. I hope they answer the call. Those men are bonafide geniuses.
Title: Re: EM Drive Developments - related to space flight applications - Thread 2
Post by: aero on 03/09/2015 05:20 pm
I came across an interesting bit of information while doing background reading on evanescent wave forces.

Quote
An intriguing possibility for further increasing the
magnitude of the force is slow-light enhancement,
since the force increases as 1/vg for fixed input power.20
We note that the electrostatic force due to
trapped or induced charges in Si waveguides is esti-
mated to be at least an order of magnitude smaller
than the optically induced force. The Casimir–
Lifshitz force is even smaller. Optical evanescent ...

Quoting from this document:
http://math.mit.edu/~stevenj/papers/PovinelliLo05.pdf (http://math.mit.edu/~stevenj/papers/PovinelliLo05.pdf)
Title: Re: EM Drive Developments - related to space flight applications - Thread 2
Post by: Cinder on 03/09/2015 09:34 pm
That is a good thread BTW. Those guys are harsh and I like it. I value reading outside opinions because I don't want to get caught up in my own little bubble and lose insight, like I did for a large part of thread 1.
Not up to NSF standards, so hopefully if some of them decide to contribute they won't get this thread locked like thread #1.
Title: Re: EM Drive Developments - related to space flight applications - Thread 2
Post by: Rodal on 03/09/2015 10:37 pm
I came across an interesting bit of information while doing background reading on evanescent wave forces.

Quote
An intriguing possibility for further increasing the
magnitude of the force is slow-light enhancement,
since the force increases as 1/vg for fixed input power.20
We note that the electrostatic force due to
trapped or induced charges in Si waveguides is esti-
mated to be at least an order of magnitude smaller
than the optically induced force. The Casimir–
Lifshitz force is even smaller. Optical evanescent ...

Quoting from this document:
http://math.mit.edu/~stevenj/papers/PovinelliLo05.pdf (http://math.mit.edu/~stevenj/papers/PovinelliLo05.pdf)

Highly conjugated polymers with giant electronic orbitals make these materials electroactive yet very stable. The paper below shows a mode of electronic polarizability a giant nomadic polarization, making these organic polymers among the most polarizable materials ever encountered. They claim that one can tailor giant orbital polymers to produce pure and highly stable super-dielectrics with high dielectric constants, from 6 to 100,000.

One more thing for Paul March and others to try...


http://link.springer.com/article/10.1007%2FBF02659632

This is a PhD Thesis (2011) from the University of California on the engineering design of conjugated polymers for organic electronics

http://escholarship.org/uc/item/58c3184d#page-3

This is an article on Super Dielectrics made from inexpensive materials, unfortunately their super-dielectric properties occur at very low frequencies (<10^(−2) Hz))


http://arxiv.org/ftp/arxiv/papers/1403/1403.6862.pdf

http://www.mdpi.com/1996-1944/7/12/8197/pdf
Title: Re: EM Drive Developments - related to space flight applications - Thread 2
Post by: RotoSequence on 03/09/2015 11:07 pm
We've been laser-focused on materials with greater dielectric constants, but what about materials with similar dielectric constants, like fused quartz? Will the EM drives behave differently?
Title: Re: EM Drive Developments - related to space flight applications - Thread 2
Post by: Rodal on 03/09/2015 11:32 pm
We've been laser-focused on materials with greater dielectric constants, but what about materials with similar dielectric constants, like fused quartz? Will the EM drives behave differently?

If Paul March discussed testing with dielectric materials other than Teflon and HD PE, I don't recall.  It would be interesting if Paul could comment (or if Paul already discussed this, if somebody could bring the experimental results to our attention).

I understand that Roger Shawyer tested non-polymer materials as dielectrics, but the specific results and the dimensions and material properties of the dielectrics tested were not disclosed (again, if anyone has more specific details, please bring them to our attention).
Title: Re: EM Drive Developments - related to space flight applications - Thread 2
Post by: Notsosureofit on 03/10/2015 01:29 am
@ RODAL

Just got a minute but from your p expression;

If L1/c1 = L2/c2

del f = (1/2*f)*((c1*c2)/(L1*L2))*b^2*((1/dD1^2)-(1/dD2^2))

might be a solution ??

Got to check the thinking later.

Night !
Title: Re: EM Drive Developments - related to space flight applications - Thread 2
Post by: Rodal on 03/10/2015 01:42 am
@ RODAL

Just got a minute but from your p expression;

If L1/c1 = L2/c2

del f = (1/2*f)*((c1*c2)/(L1*L2))*b^2*((1/dD1^2)-(1/dD2^2))

might be a solution ??

Got to check the thinking later.

Night !

I find your previous expression

del f = ( f/(2*c^2)) * (c1^2-c2^2)

more physically appealing, since it goes to zero for equal dielectric constants, regardless or their dielectric length,

while on the other hand

del f = (1/2*f)*((c1*c2)/(L1*L2))*b^2*((1/dD1^2)-(1/dD2^2))

goes to zero for equal dielectric lengths, regardless of their dielectric constants.
Title: Re: EM Drive Developments - related to space flight applications - Thread 2
Post by: Star-Drive on 03/10/2015 03:18 am
We've been laser-focused on materials with greater dielectric constants, but what about materials with similar dielectric constants, like fused quartz? Will the EM drives behave differently?

If Paul March discussed testing with dielectric materials other than Teflon and HD PE, I don't recall.  It would be interesting if Paul could comment (or if Paul already discussed this, if somebody could bring the experimental results to our attention).

I understand that Roger Shawyer tested non-polymer materials as dielectrics, but the specific results and the dimensions and material properties of the dielectrics tested were not disclosed (again, if anyone has more specific details, please bring them to our attention).

Dr. Rodal:

We've only tried polyethylene, Teflon, neoprene rubber and aluminum oxide discs so far, with PE and PTFE being the most productive.  However what dielectric if any will prove to be optimal in generating the most thrust in these EM-Drive like thrusters is really dependent on what physics is really driving their operation.  So far the dielectrics with the largest electrostrictive coefficient combined with a largest Q-factor appear to be the winners.  This implies to me that fused quartz may be a good candidate, since it has a large Q-factor with moderate electrostrictive coefficient and piezoelectric responses. 

BTW, these dielectrics may prove to be the E&M/gravity field to mechanical converters needed to generate thrust.  On the other hand if Shawyer and the Chinese are right in their statements that they used no dielectrics in their tens to hundreds of milli-Newton thrusters, then these dielectrics may just be means of amplifying the underlying effects that are generated just by the action of the E&M fields on the copper or silver atoms in the walls of the frustum resonant cavity.  Remember that though copper and silver only have a real permittivity of 1.0, in temporal space they have a complex permittivity of greater than 100.  and this is the parameter that drives E to B-field phase shifting over very small distances at microwave frequencies, (~2 microns deep at 2.0 GHz).  I.e. these metallic atoms can undergo very large cyclic accelerations around their crystal lattice positions as the E&M wave fronts are dissipated in them.

Title: Re: EM Drive Developments - related to space flight applications - Thread 2
Post by: RotoSequence on 03/10/2015 05:03 am
Is it a good idea to suggest theories of operation at the current level of understanding of EM drive's operational parameters? I suppose a relatively straightforward experiment for testing electrogravitation would be to place the EM drive test article near a gravimeter (or other physically separated accelerometer), switch it on, and see if/how the gravimeter's readings change.
Title: Re: EM Drive Developments - related to space flight applications - Thread 2
Post by: Mulletron on 03/10/2015 07:32 am
I haven't pointed out the reason why I'm digging into cavity QED and cavity optomechanics. What interests me is the well known strong interaction between light and matter.... the strong coupling observed within cavities is what I'm interested in.

Which also includes vacuum fluctuations and Casimir-Polder forces as indicated by other work I dug up earlier in the thread.

That momentum transfer from the qv work that was dug up can't stand on its own outside of this framework.

I really wish I could get somebody else on board with this idea because I absolutely will fail on my own.
Title: Re: EM Drive Developments - related to space flight applications - Thread 2
Post by: frobnicat on 03/10/2015 09:50 am
@Star-Drive :

I'm still asking about the huge discrepancy (http://forum.nasaspaceflight.com/index.php?topic=36313.msg1342605#msg1342605) between the expected linear displacement of 32.3µm (assuming only flexure bearings restoring torque) in response to a calibration pulse of 29.1µm and the actual readings on chart vertical scale of a linear displacement between 1µm to 2.5µm for the calibration pulses.

There is an inconsistency of an order of magnitude between values as expected from flexure stiffness and as recorded on the display. Is it acknowledged or investigated by the team at Eagleworks ?

I'm not objecting that the thrusts measurements would not be proportional (ie for twice the deviation of cal. pulse => twice 29.1µN) but since the fixed ratio of µm displacement per µN of thrust is at the heart of the experiment, such discrepancy can only weaken the case for the charts published so far. This needs to be clarified anyhow. It could be a problem of calibration of the Philtec D63 gain, or a biased scaling factor between the analog outputs of the D63 and the final rendering of vertical scale on display...
Title: Re: EM Drive Developments - related to space flight applications - Thread 2
Post by: RotoSequence on 03/10/2015 10:04 am
I really wish I could get somebody else on board with this idea because I absolutely will fail on my own.

I'm sorry that I can't be of much help in this area, but for what it's worth, I think your theories are worth exploring by those who have the knowledge, background, and ability to do so.
Title: Re: EM Drive Developments - related to space flight applications - Thread 2
Post by: Notsosureofit on 03/10/2015 11:20 am
@ RODAL

Just got a minute but from your p expression;

If L1/c1 = L2/c2

del f = (1/2*f)*((c1*c2)/(L1*L2))*b^2*((1/dD1^2)-(1/dD2^2))

might be a solution ??

Got to check the thinking later.

Night !

I find your previous expression

del f = ( f/(2*c^2)) * (c1^2-c2^2)

more physically appealing, since it goes to zero for equal dielectric constants, regardless or their dielectric length,

while on the other hand

del f = (1/2*f)*((c1*c2)/(L1*L2))*b^2*((1/dD1^2)-(1/dD2^2))

goes to zero for equal dielectric lengths, regardless of their dielectric constants.

The previous expression is only valid approximation for a "uniformly varying dielectric".  There is no L1 and L2 in that case.

What do you think might maximize the second expression ? (valid only for L1/c1 = L2/c2 )
Title: Re: EM Drive Developments - related to space flight applications - Thread 2
Post by: Mulletron on 03/10/2015 11:47 am
This is a link to another type of resonant cavity microwave thruster that DOES use propellant. I'm sharing it because the paper parallels what is being discussed in this thread to a great degree, such as calculating resonant modes, COMSOL analysis...etc. There is a bunch of useful stuff in there.

https://etda.libraries.psu.edu/paper/12292/7584

Edit: Unshortened the link. Google had a mile long search string to it. Some people are having ssl problems with the link, so it is attached. I'd rather save NSF the bandwidth by not downloading the large PDF from here if possible.

DESIGN AND DEVELOPMENT OF A 30-GHz MICROWAVE ELECTROTHERMAL THRUSTER
A Thesis in Aerospace Engineering
by
Erica E. Capalungan
Title: Re: EM Drive Developments - related to space flight applications - Thread 2
Post by: Mulletron on 03/10/2015 12:06 pm
http://forum.nasaspaceflight.com/index.php?topic=29276.msg1274400#msg1274400

Quoting from the Chinese paper linked to in the above post from @Aero
Quote
With the Small End of the cavity unchanged, the quality factor and thrust decrease with the increase in the Large End.

Ok, so with that in mind, why is Shawyer progressively opening up the angle of this designs? What gives?

http://forum.nasaspaceflight.com/index.php?topic=36313.msg1336811#msg1336811
Title: Re: EM Drive Developments - related to space flight applications - Thread 2
Post by: Star-Drive on 03/10/2015 12:19 pm
@Star-Drive :

I'm still asking about the huge discrepancy (http://forum.nasaspaceflight.com/index.php?topic=36313.msg1342605#msg1342605) between the expected linear displacement of 32.3µm (assuming only flexure bearings restoring torque) in response to a calibration pulse of 29.1µm and the actual readings on chart vertical scale of a linear displacement between 1µm to 2.5µm for the calibration pulses.

There is an inconsistency of an order of magnitude between values as expected from flexure stiffness and as recorded on the display. Is it acknowledged or investigated by the team at Eagleworks ?

I'm not objecting that the thrusts measurements would not be proportional (ie for twice the deviation of cal. pulse => twice 29.1µN) but since the fixed ratio of µm displacement per µN of thrust is at the heart of the experiment, such discrepancy can only weaken the case for the charts published so far. This needs to be clarified anyhow. It could be a problem of calibration of the Philtec D63 gain, or a biased scaling factor between the analog outputs of the D63 and the final rendering of vertical scale on display...


Frobnicat & Crew:

Dr. White and his NASA interns are the folks who performed the original force calibration work on this torque pendulum, so your questions might be bettered answered by him.  However, it's my understanding that the torque pendulum's actual micron displacement observed for each test run is dependent on its specific total active mass load, balance weights and all their locations on the torque pendulum arm for the test run in question.  So as long as we reference the near constant calibration force from our electrostatic fin calibration system before and after each test run, and then use that specific displacement yardstick of the moment as the true measure of the test article's generated forces, it doesn't matter what the actual micron displacement turns out to be for each data run.  And that has been what we've used to date report our generated forces.  If there is a major problem with that approach please let us know. 

Best, Paul M.
Title: Re: EM Drive Developments - related to space flight applications - Thread 2
Post by: Rodal on 03/10/2015 12:23 pm
@ RODAL

Just got a minute but from your p expression;

If L1/c1 = L2/c2

del f = (1/2*f)*((c1*c2)/(L1*L2))*b^2*((1/dD1^2)-(1/dD2^2))

might be a solution ??

Got to check the thinking later.

Night !

I find your previous expression

del f = ( f/(2*c^2)) * (c1^2-c2^2)

more physically appealing, since it goes to zero for equal dielectric constants, regardless or their dielectric length,

while on the other hand

del f = (1/2*f)*((c1*c2)/(L1*L2))*b^2*((1/dD1^2)-(1/dD2^2))

goes to zero for equal dielectric lengths, regardless of their dielectric constants.

The previous expression is only valid approximation for a "uniformly varying dielectric".  There is no L1 and L2 in that case.

What do you think might maximize the second expression ? (valid only for L1/c1 = L2/c2 )
I see the wisdom in your hint that we should approach this starting from an earlier point, with the resonating field as a function of the longitudinal polar coordinate variable.  In the case of only one dielectric the field is just a harmonic function of the longitudinal variable, hence your idea to perhaps do a Fourier series expansion.

I have to take care of some $$$ paying work first.  I'll eventually be back at this in a more comprehensive way.
Title: Re: EM Drive Developments - related to space flight applications - Thread 2
Post by: Mulletron on 03/10/2015 12:37 pm
http://www.emdrive.com/NWPU2010translation.pdf

In the conclusion area, last sentence in 3rd paragraph, next to last page.
Quote
With the Small End of the cavity unchanged, the quality factor and thrust decrease with the increase in the Large End.

Seems to be a summary of this on page 8:
Quote
By keep the diameter of the Small End constant, increase the large end of the cavity, in order to have the same resonant frequency, cavity height must be reduced, quality factor also reduce.
Title: Re: EM Drive Developments - related to space flight applications - Thread 2
Post by: Rodal on 03/10/2015 01:13 pm
http://www.emdrive.com/NWPU2010translation.pdf

In the conclusion area, last sentence in 3rd paragraph, next to last page.
Quote
With the Small End of the cavity unchanged, the quality factor and thrust decrease with the increase in the Large End.

Seems to be a summary of this on page 8:
Quote
By keep the diameter of the Small End constant, increase the large end of the cavity, in order to have the same resonant frequency, cavity height must be reduced, quality factor also reduce.
OK, I took a look at the above quotations.

THESE QUOTED STATEMENTS ARE NOT BASED ON ANY EXPERIMENTS

It is clear from the text that these statements are only based on their Finite Element analysis numerical solution of the eigenvalue problem to solve Maxwell's equations, from which the author's derive a thrust force.  The authors then change the dimensions in their finite element model, which gives different values of the numerically calculated thrust force.

It is trivial to show that a solution that is based only on solving the eigenvalue problem of Maxwell's equations in a truncated cone cavity cannot result in a thrust force on the center of mass.  Greg Egan shows this with closed-form exact equations ( http://gregegan.customer.netspace.net.au/SCIENCE/Cavity/Cavity.html ).  The thrust force should be zero.  This is the reason why prominent physicists and academia are so skeptical of the EM Drive, as a thrust force from such a propellant-less cavity would violate conservation momentum (unless there is coupling with an external field -external fields which the authors did not take into account in their numerical analysis-)

The fact that the authors obtain a thrust force from the solution of the eigenvalue problem for Maxwell's equations (without any consideration of external coupling fields) can be due to ill-conditioning of the eigenvalue matrix (inverted in the FEM solution) and/or problems with improper setting of boundary conditions (as their FEM analysis cannot deal with rigid body modes and they have to be suppressed to get a solution), or it could be due for example to reduced integration of the finite element model (for example spurious deformation modes like the hourglass mode of finite elements that finite element experts are familiar with, but that users of finite element packages may be unaware of). 

In a few words:

*the statements are not based on experiments

*the statements are based on a numerical solution that  should have given zero thrust. Since the numerical solution gave a finite thrust (with extremely small magnitude ! ), the conclusion that increasing the cone angle decreases the thrust force has to be treated as an unsupported outcome of their incorrect numerical solution.


NOTE: The quotation is still very interesting as the flawed numerical analysis by Juan Yang et.al. comes to a completely different conclusion than R. Shawyer's equation (as well as the equations of McCulloch and Notsosureofit).  The fact that R. Shawyer prominently uses Juan Yang's reference in his website to validate the EM Drive, while Juan Yang's numerical results contradict Shawyer's (regarding the angle of the truncated cone) is noteworthy.
Also, if Juan Yang believes her numerical results, why didn't she decrease the angle of the truncated cone (or downright use a cylindrical cavity instead of a truncated cone in her later experiments?). If anything the angle of the truncated cone used by Juan Yang seems to have increased rather than decreased in her more recent experiments.

Title: Re: EM Drive Developments - related to space flight applications - Thread 2
Post by: Notsosureofit on 03/10/2015 03:21 pm
FYI   Cavity gravimetry  ??

http://scitation.aip.org/content/aip/magazine/physicstoday/news/10.1063/PT.5.7149?utm_source=Physics+Today&utm_medium=email&utm_campaign=5417485_Physics+Today%3a+The+week+in+Physics+2-6+March&dm_i=1Y69,3845P,E1MTSN,BJSKS,1

Title: Re: EM Drive Developments - related to space flight applications - Thread 2
Post by: frobnicat on 03/10/2015 05:03 pm
@Star-Drive :

I'm still asking about the huge discrepancy (http://forum.nasaspaceflight.com/index.php?topic=36313.msg1342605#msg1342605) between the expected linear displacement of 32.3µm (assuming only flexure bearings restoring torque) in response to a calibration pulse of 29.1µm and the actual readings on chart vertical scale of a linear displacement between 1µm to 2.5µm for the calibration pulses.

There is an inconsistency of an order of magnitude between values as expected from flexure stiffness and as recorded on the display. Is it acknowledged or investigated by the team at Eagleworks ?

I'm not objecting that the thrusts measurements would not be proportional (ie for twice the deviation of cal. pulse => twice 29.1µN) but since the fixed ratio of µm displacement per µN of thrust is at the heart of the experiment, such discrepancy can only weaken the case for the charts published so far. This needs to be clarified anyhow. It could be a problem of calibration of the Philtec D63 gain, or a biased scaling factor between the analog outputs of the D63 and the final rendering of vertical scale on display...


Frobnicat & Crew:

Dr. White and his NASA interns are the folks who performed the original force calibration work on this torque pendulum, so your questions might be bettered answered by him.  However, it's my understanding that the torque pendulum's actual micron displacement observed for each test run is dependent on its specific total active mass load, balance weights and all their locations on the torque pendulum arm for the test run in question.  So as long as we reference the near constant calibration force from our electrostatic fin calibration system before and after each test run, and then use that specific displacement yardstick of the moment as the true measure of the test article's generated forces, it doesn't matter what the actual micron displacement turns out to be for each data run.  And that has been what we've used to date report our generated forces.  If there is a major problem with that approach please let us know. 

Best, Paul M.

Well, the pendulum is basically a device that, ideally, converts force into displacement. What is measured is displacement  D=Cst*F. The electrostatic calibration system looks like a robust way to achieve a stable reference force Fcal of 29.1µN. So there is no question that  Dthrust=Cst*Fthrust together with Dcal=Cst*Fcal yields Fthrust=Fcal*Dthrust/Dcal where Cst cancels. That's a good point for the design, there is no need to know Cst precisely. But not needing to know precisely is one thing, having an order of magnitude discrepancy in the absolute value of Cst is another. Cst is at the core of the conversion of µN (what we want to know) into µm (what we measure) : at the very least there is a central aspect of the balance that seems very poorly characterised.

Add to that the fact that the whole apparatus is slightly tilted. With the actual vertical scale charts readings (that are in contradiction with both the known flexure bearing stiffness and the natural oscillation period of ~4.5s which is clearly visible on some charts) there is no way to reconstruct this tilt from the data. You seem to imply this tilt is quite low, nevertheless it does play a significant role in the rest equilibrium point of the balance since a perfectly horizontal setting wouldn't allow to have a stable rest position (which makes perfect sense since there is no angular tuning at the axis and flexure bearings rest will drift thermally and with loads). So gravity plays a role. So thermal displacements in centre of mass of a part relative to fixation point can have a significant impact on the rest position, recording as sustained displacements at the LDS. Frankly, from the shapes of the responses to the tests I don't really believe that thermal displacements alone can explain the whole response, but others may be more finicky about that, and to clearly assess those aspects needs a correct characterization of the Cst between µN thrusts and µm readings. For instance if vertical scale is showing 1µm when really it should show 10µm then one would have to explain 300g moving only 0.1mm instead of 300g moving 1mm. The correction (if it is indeed needed) would actually make it harder to explain effects from thermal displacements.

A change of stiffness at the level of flexure bearings due to axial load conditions can't explain the 4.5s period in the charts. I can't find Riverhawks charts indicating how stiffness would change with axial load, but explaining the 1µm reading for 29.1µN cal. as due to an added stiffness (30 times more) depending on axial load would be in direct contradiction with the dynamics of the chart and using moment of inertia from mass distribution (even if it's uncertain up to 50%).

This very apparent discrepancy could be due to a simple scaling factor in display, it could also indicate a problem in the operating conditions of the LDS, with an LDS working not around 500µm but nearer the peak, at reduced sensitivity and reduced linearity.

So while I wouldn't qualify the problem as major as far as Fthrust=Fcal*Dthrust/Dcal is concerned, it would still appear as a manifest and serious consistency problem for anyone looking in some depth at the data and requiring a proper characterization of the system. It could be also indicative that LDS is operating at less than ideal conditions and that alone would make it worth further inquiry.

Side note : I'm only an anonymous contributor trying to understand what is going on, I do have a background in mechanical engineering, including metrology, but can't claim this is my professional activity (neither in aerospace). I do believe my arguments are correct and worth of consideration, but this does not preclude a blunder somewhere. I understand with limited time and resources the team at Eagleworks has to weigh the priorities. Anyhow, thank you for taking time reading and answering my concerns.
Title: Re: EM Drive Developments - related to space flight applications - Thread 2
Post by: Rodal on 03/10/2015 05:21 pm
We've been laser-focused on materials with greater dielectric constants, but what about materials with similar dielectric constants, like fused quartz? Will the EM drives behave differently?

If Paul March discussed testing with dielectric materials other than Teflon and HD PE, I don't recall.  It would be interesting if Paul could comment (or if Paul already discussed this, if somebody could bring the experimental results to our attention).

I understand that Roger Shawyer tested non-polymer materials as dielectrics, but the specific results and the dimensions and material properties of the dielectrics tested were not disclosed (again, if anyone has more specific details, please bring them to our attention).

Dr. Rodal:

We've only tried polyethylene, Teflon, neoprene rubber and aluminum oxide discs so far, with PE and PTFE being the most productive.  However what dielectric if any will prove to be optimal in generating the most thrust in these EM-Drive like thrusters is really dependent on what physics is really driving their operation.  So far the dielectrics with the largest electrostrictive coefficient combined with a largest Q-factor appear to be the winners.  This implies to me that fused quartz may be a good candidate, since it has a large Q-factor with moderate electrostrictive coefficient and piezoelectric responses. 

BTW, these dielectrics may prove to be the E&M/gravity field to mechanical converters needed to generate thrust.  On the other hand if Shawyer and the Chinese are right in their statements that they used no dielectrics in their tens to hundreds of milli-Newton thrusters, then these dielectrics may just be means of amplifying the underlying effects that are generated just by the action of the E&M fields on the copper or silver atoms in the walls of the frustum resonant cavity.  Remember that though copper and silver only have a real permittivity of 1.0, in temporal space they have a complex permittivity of greater than 100.  and this is the parameter that drives E to B-field phase shifting over very small distances at microwave frequencies, (~2 microns deep at 2.0 GHz).  I.e. these metallic atoms can undergo very large cyclic accelerations around their crystal lattice positions as the E&M wave fronts are dissipated in them.

Thank you, Paul.  Great, very valuable information.

Having personally worked on R&D compounding of rubber and thus being very much aware of the great variety of possible properties for neoprene rubber depending on compounding, I would not draw too much general conclusions based on results for particular Neoprene rubber samples (without knowing the specific compound tested).

The information on (ceramic) aluminum oxide discs being inferior (regarding the measured thrust force) to polymers like PTFE and HD PE is very valuable, particularly in light of reports like the following that have comprehensively characterized the dielectric properties of Aluminum Oxide over a very broad frequency range utilizing multiple analytical testing techniques:

Broadband Dielectric Characterization of Aluminum Oxide (Al2O3)
Khalid Z. Rajab, Mira Naftaly, Edmund H. Linfield, Juan C. Nino, Daniel Arenas, David Tanner, Raj Mittra, and Michael Lanagan

J. Micro. and Elect. Pack. 5, 101–106 (2008)

http://www.phys.ufl.edu/~tanner/PDFS/Rajab08jmep-Al2O3.pdf


From the few reports where I saw the dielectric materials tested by R. Shawyer, what I recall is Shawyer using inorganic dielectric materials.  This is important to understand the reports that Shawyer has abandoned the use of dielectric materials.  Without knowing specifically what dielectric materials did Shawyer abandon, the fact that he abandoned them is not that useful.
Title: Re: EM Drive Developments - related to space flight applications - Thread 2
Post by: Notsosureofit on 03/10/2015 05:39 pm
Good 10GHz data !
Title: Re: EM Drive Developments - related to space flight applications - Thread 2
Post by: Mulletron on 03/10/2015 08:24 pm
I really wish I could get somebody else on board with this idea because I absolutely will fail on my own.

I'm sorry that I can't be of much help in this area, but for what it's worth, I think your theories are worth exploring by those who have the knowledge, background, and ability to do so.

I just need to more rigorously show applicability of these concepts presented in the literature I've been posting. I'm getting into the realm where I can't find any further precedent in which I can deep dive into because the applicable work simply hasn't been written yet (or I can't find it). I'm not interested in finding anything other than established theory so this might be the wall. I'm really searching for that clincher that will be undeniable.
Title: Re: EM Drive Developments - related to space flight applications - Thread 2
Post by: aero on 03/10/2015 10:06 pm
@Dr. Rodel - Did you use your magic formulas to calculate the resonance frequencies of the newest Eagleworks cavity from Paul? Including dielectric effects? If so, what were the numbers?
Title: Re: EM Drive Developments - related to space flight applications - Thread 2
Post by: Rodal on 03/10/2015 10:21 pm
@Dr. Rodel - Did you use your magic formulas to calculate the resonance frequencies of the newest Eagleworks cavity from Paul? Including dielectric effects? If so, what were the numbers?
I obtained exact solutions so far for:

1) Truncated cone with only one dielectric medium in the cavity
2) Cylindrical cavity with two dielectric coupled mediums

I haven't had the time to derive the formula for the Truncated Cone with two dielectric mediums to consider the case with vacuum and HD-PE. 

I did calculate the Eagleworks cavity without the HD-PE dielectric section.
Title: Re: EM Drive Developments - related to space flight applications - Thread 2
Post by: aero on 03/10/2015 10:26 pm
@Dr. Rodel - Did you use your magic formulas to calculate the resonance frequencies of the newest Eagleworks cavity from Paul? Including dielectric effects? If so, what were the numbers?
I obtained exact solutions so far for:

1) Truncated cone with only one dielectric medium in the cavity
2) Cylindrical cavity with two dielectric coupled mediums

I haven't had the time to derive the formula for the Truncated Cone with two dielectric mediums to consider the case with vacuum and HD-PE. 

I did calculate the Eagleworks cavity without the HD-PE dielectric section.

Quote
I did calculate the Eagleworks cavity without the HD-PE dielectric section.

And what number did you get?
Title: Re: EM Drive Developments - related to space flight applications - Thread 2
Post by: Rodal on 03/10/2015 10:39 pm
....
Quote
I did calculate the Eagleworks cavity without the HD-PE dielectric section.

And what number did you get?
See http://forum.nasaspaceflight.com/index.php?topic=36313.msg1339803#msg1339803

The frequency for the TM221 mode shape (without the dielectric) is 2.00709 GHz.

Is there another particular mode you are interested to know the frequency for?
Title: Re: EM Drive Developments - related to space flight applications - Thread 2
Post by: aero on 03/10/2015 11:17 pm
....
Quote
I did calculate the Eagleworks cavity without the HD-PE dielectric section.

And what number did you get?
See http://forum.nasaspaceflight.com/index.php?topic=36313.msg1339803#msg1339803

The frequency for the TM221 mode shape (without the dielectric) is 2.00709 GHz.

Is there another particular mode you are interested to know the frequency for?

I'll need to think on that. It seems, from the images you posted, that the mode is the same for you and Eagleworks. So what (if anything) is close to freq  = 1.937115E+009 Hz? And was the Eagleworks photos taken with that drive frequency?
Title: Re: EM Drive Developments - related to space flight applications - Thread 2
Post by: Rodal on 03/10/2015 11:52 pm
....
Quote
I did calculate the Eagleworks cavity without the HD-PE dielectric section.

And what number did you get?
See http://forum.nasaspaceflight.com/index.php?topic=36313.msg1339803#msg1339803

The frequency for the TM221 mode shape (without the dielectric) is 2.00709 GHz.

Is there another particular mode you are interested to know the frequency for?

I'll need to think on that. It seems, from the images you posted, that the mode is the same for you and Eagleworks. So what (if anything) is close to freq  = 1.937115E+009 Hz? And was the Eagleworks photos taken with that drive frequency?

Take a look at the enclosed report from Paul March for a mode which NASA's COMSOL analyst identified as TM112 "like TM110 at top, and TM111 at bottom" with a frequency of 1.9355 GHz.

I only calculated a few modes.

Regarding your question "And was the Eagleworks photos taken with that drive frequency?" I presume that the photo was taken for the EM Drive with a dielectric (which would give a slightly lower frequency) while my computation was without the dielectric.
Title: Re: EM Drive Developments - related to space flight applications - Thread 2
Post by: Notsosureofit on 03/11/2015 12:18 am
@ RODAL

Just got a minute but from your p expression;

If L1/c1 = L2/c2

del f = (1/2*f)*((c1*c2)/(L1*L2))*b^2*((1/dD1^2)-(1/dD2^2))

might be a solution ??

Got to check the thinking later.

Night !

I find your previous expression

del f = ( f/(2*c^2)) * (c1^2-c2^2)

more physically appealing, since it goes to zero for equal dielectric constants, regardless or their dielectric length,

while on the other hand

del f = (1/2*f)*((c1*c2)/(L1*L2))*b^2*((1/dD1^2)-(1/dD2^2))

goes to zero for equal dielectric lengths, regardless of their dielectric constants.

The previous expression is only valid approximation for a "uniformly varying dielectric".  There is no L1 and L2 in that case.

What do you think might maximize the second expression ? (valid only for L1/c1 = L2/c2 )

Mmmm, so for L1/c1 = L2/c2 we can rewrite as:

del f = (b^2/(2*f*D^2))*(c1^2-c2^2)   ==> 0 as c1 => c2   so max c1 vs c2 ??

But we still need a general solution instead of a case by case by inspection.

Still, they both vary as (c1^2-c2^2) and are almost opposite cases.

Title: Re: EM Drive Developments - related to space flight applications - Thread 2
Post by: Rodal on 03/11/2015 12:32 am
....
Quote
I did calculate the Eagleworks cavity without the HD-PE dielectric section.

And what number did you get?
See http://forum.nasaspaceflight.com/index.php?topic=36313.msg1339803#msg1339803

The frequency for the TM221 mode shape (without the dielectric) is 2.00709 GHz.

Is there another particular mode you are interested to know the frequency for?

I'll need to think on that. It seems, from the images you posted, that the mode is the same for you and Eagleworks. So what (if anything) is close to freq  = 1.937115E+009 Hz? And was the Eagleworks photos taken with that drive frequency?

Take a look at the enclosed report from Paul March for a mode which NASA's COMSOL analyst identified as TM112 "like TM110 at top, and TM111 at bottom" with a frequency of 1.9355 GHz.

I only calculated a few modes.

Regarding your question "And was the Eagleworks photos taken with that drive frequency?" I presume that the photo was taken for the EM Drive with a dielectric (which would give a slightly lower frequency) while my computation was without the dielectric.

I compute a frequency of 1.97396 GHz for mode TM112 without the dielectric.
Title: Re: EM Drive Developments - related to space flight applications - Thread 2
Post by: Star-Drive on 03/11/2015 02:46 am
Dr. Rodal:

"From the few reports where I saw the dielectric materials tested by R. Shawyer, what I recall is Shawyer using inorganic dielectric materials.  This is important to understand the reports that Shawyer has abandoned the use of dielectric materials.  Without knowing specifically what dielectric materials did Shawyer abandon, the fact that he abandoned them is not that useful."

Shawyer used a commercial ceramic dielectric resonator with an e-r = ~38 and a Q of greater than 10,000.  We tried a similar commercial grade ceramic dielectric resonator material like the ones from Temex-ceramics and generated zip with same.  Hard ceramics with low electrostrictive coefficients and no piezoelectric response apparently are dead-ends for this EM-Drive application.

http://www.temex-ceramics.com/site/en/dielectric-resonators-cermatmenu-28.html

http://www.temex-ceramics.com/site/fichiers/dielectric.pdf

Acknowledge that "neoprene" rubber covers a multitude of formulations, but it turns out that most if not all of them have low Q-factors at microwave frequencies that disqualify them from this application even if they have a large electrostrictive coefficient.  What counts in this low power EM-drive dielectric "amplifier" appears to be a dielectric with the largest electrostrictive/piezoelectric coefficient combined with a high Q-factor at microwave frequencies.

Best, Paul M.
Title: Re: EM Drive Developments - related to space flight applications - Thread 2
Post by: aero on 03/11/2015 04:43 am
@Rodal

Can there be a resonance in the empty cavity at the low frequency of 1.6859 GHz?
Title: Re: EM Drive Developments - related to space flight applications - Thread 2
Post by: Rodal on 03/11/2015 10:50 am
@Rodal

Can there be a resonance in the empty cavity at the low frequency of 1.6859 GHz?

Take a gander at page 3 of the attachment (from a previous post by Paul March) to http://forum.nasaspaceflight.com/index.php?topic=36313.msg1344168#msg1344168
Title: Re: EM Drive Developments - related to space flight applications - Thread 2
Post by: Rodal on 03/11/2015 11:31 pm
Continuing from message, http://forum.nasaspaceflight.com/index.php?topic=36313.msg1339803#msg1339803, using the exact solution information, I plotted the information as a Vector Density Plot to better understand the magnetic field in modes TM22p, where p can be 1, 2, 3 etc.  On this cross-section, TM221, TM222 and TM223, etc, look practically identical, except for different intensity magnitude for the higher frequency modes with TM22p where p>1.

The vector plot shows the direction and magnitude of the magnetic vector field  on the base for the mode presently being tested by NASA Eagleworks.  It helps understand the reason for the hot and cold intensities.

Below I show:

1) The NASA Finite Element prediction and NASA's thermal IR imaging measurement

2) Vector Intensity Plot of mode TM222 using the exact solution (click the image to magnify it)

3) An Intensity Plot of mode TM221 using the exact solution (with another color scheme)

4) Another Intensity Plot of mode TM221 using the exact solution (with another color scheme)

(http://forum.nasaspaceflight.com/index.php?action=dlattach;topic=36313.0;attach=635194;image)
Title: Re: EM Drive Developments - related to space flight applications - Thread 2
Post by: Rodal on 03/12/2015 01:30 pm
....

Dr. Rodal:

....

Acknowledge that "neoprene" rubber covers a multitude of formulations, but it turns out that most if not all of them have low Q-factors at microwave frequencies that disqualify them from this application even if they have a large electrostrictive coefficient.  What counts in this low power EM-drive dielectric "amplifier" appears to be a dielectric with the largest electrostrictive/piezoelectric coefficient combined with a high Q-factor at microwave frequencies.

Best, Paul M.

Paul,

(*) Neoprene® is Dupont's trade name  for Chloroprene rubber (CR), so in the rest of this post I will refer to it by its technical name Chloroprene rubber (CR). 

The fact that NASA Eagleworks tested a dielectric made of Chloroprene rubber (CR) and found out that this rubber material is "disqualified for this (EM Drive) application" because the measured thrust using this rubber material as a dielectric is insignificant compared to using PTFE or HD-PE has important consequences that should be pointed out.

Since Chloroprene rubber (CR) has a coefficient of thermal expansion up to 158% greater than the coefficient of thermal expansion of HD PE:



THERMAL EXPANSION COEFFICIENT

Chloroprene rubber (CR)  125*10^(-6) to 190*10^(-6) 1/(deg K)  http://techcenter.lanxess.com/docs/pdft/e5-14.pdf
High Density Polyethylene (HD PE)  120*10^(-6) 1/(deg K)
 http://www.engineeringtoolbox.com/pipes-temperature-expansion-coefficients-d_48.html


This experimental finding further nullifies the  conjecture that the experimental measurements are an artifact due to movement of the center-of-mass by thermal expansion of the dielectric.

(This conjecture has been recently explored by @frobnicat.  According to @frobnicat's conjecture, based on thermal expansion, under no condition should the thrust force measurement have been smaller for Chloroprene rubber (CR)  than for HD PE)




(*)

LIST of MANUFACTURER TRADENAMES for Chloroprene rubber (CR)

Lanxess (Bayer AG)      BAYPRENE
Denka Kagaku Kogyo    CHLOROPRENE
Showa Denko               SHOPRENE
TOSOH Corporation      SKYPRENE
DuPont                        NEOPRENE

http://en.wikipedia.org/wiki/Chloroprene
Title: Re: EM Drive Developments - related to space flight applications - Thread 2
Post by: Rodal on 03/12/2015 02:59 pm
....

Dr. Rodal:

....

Acknowledge that "neoprene" rubber covers a multitude of formulations, but it turns out that most if not all of them have low Q-factors at microwave frequencies that disqualify them from this application even if they have a large electrostrictive coefficient.  What counts in this low power EM-drive dielectric "amplifier" appears to be a dielectric with the largest electrostrictive/piezoelectric coefficient combined with a high Q-factor at microwave frequencies.

Best, Paul M.

Paul,

(*) Neoprene® is Dupont's trade name  for Chloroprene rubber (CR), so in the rest of this post I will refer to it by its technical name Chloroprene rubber (CR). 

The fact that NASA Eagleworks tested a dielectric made of Chloroprene rubber (CR) and found out that this rubber material is "disqualified for this (EM Drive) application" because the measured thrust using this rubber material as a dielectric is insignificant compared to using PTFE or HD-PE has important consequences that should be pointed out.

Since Chloroprene rubber (CR) has a coefficient of thermal expansion up to 158% greater than the coefficient of thermal expansion of HD PE:



THERMAL EXPANSION

Chloroprene rubber (CR)  125*10^(-6) to 190*10^(-6) 1/(deg K)  http://techcenter.lanxess.com/docs/pdft/e5-14.pdf
High Density Polyethylene (HD PE)  120*10^(-6) 1/(deg K)
 http://www.engineeringtoolbox.com/pipes-temperature-expansion-coefficients-d_48.html


This experimental finding further nullifies @frobnicat's conjecture that the experimental measurements are an artifact due to thermal expansion of the dielectric.

(According to @frobnicat's conjecture, based on thermal expansion, under no condition should the thrust force measurement have been smaller for Chloroprene rubber (CR)  than for HD PE)




(*)

LIST of MANUFACTURER TRADENAMES for Chloroprene rubber (CR)

Lanxess (Bayer AG)      BAYPRENE
Denka Kagaku Kogyo    CHLOROPRENE
Showa Denko               SHOPRENE
TOSOH Corporation      SKYPRENE
DuPont                        NEOPRENE

http://en.wikipedia.org/wiki/Chloroprene

@Frobnicat's conjecture is based on the shifting of the center of mass due to thermal expansion.

Not only Chloroprene-Rubber (CR) has a higher coefficient of thermal expansion than High-Density-Polyethylene-HDPE, but Chloroprene-Rubber (CR) also has a higher density than High-Density-Polyethylene-HDPE.

So this further nullifies the center-of-mass movement by thermal expansion conjecture.

According to @frobnicat's conjecture, the denser, higher-thermal-expansion material ("Neoprene®") should have produced a higher measured "thrust", but NASA Eagleworks results show the opposite: the denser, higher-thermal expansion material ("Neoprene®") resulted in insignificant thrust.



DENSITY (g/cm^3)   


Chloroprene-Rubber-CR ("Neoprene®")     
                                                       1.25 to 1.37 http://www.westernrubber.com/wp-content/uploads/OVERSEAS-COMPOUNDS-Publication-558-Rev-A.pdf
                                                       1.4 to 1.6 http://www.makeitfrom.com/compare/Chloroprene-Rubber-CR-Neoprene/High-Density-Polyethylene-HDPE/




High-Density-Polyethylene-HDPE       

                                                     0.93 to 0.97   http://en.wikipedia.org/wiki/High-density_polyethylene
                                                     0.947 to 0.965 http://www.dow.com/polyethylene/na/en/prod/hdpe.htm
                                                     0.95 to 1.27   http://www.makeitfrom.com/compare/Chloroprene-Rubber-CR-Neoprene/High-Density-Polyethylene-HDPE/
Title: Re: EM Drive Developments - related to space flight applications - Thread 2
Post by: Polonius on 03/12/2015 06:38 pm
Hey fellas, long time lurker, first time poster here.

I'm not even going to pretend to understand half the maths that have been tossed around in this thread. I'm in complete awe of the work you guys have been doing to solve this mystery. I do have a question though, and forgive me for my ignorance.

Does anyone know if this effect scales with the size of the frustum? Could the frequency of the microwaves be adjusted to allow the same level of force in a microscopic frustum that is shown Nasa's macroscopic frustum? If this is the case, would a sequence of many millions of tiny frustums not provide a great deal more force than one large frustum?

Title: Re: EM Drive Developments - related to space flight applications - Thread 2
Post by: Notsosureofit on 03/12/2015 06:43 pm
Hey fellas, long time lurker, first time poster here.

I'm not even going to pretend to understand half the maths that have been tossed around in this thread. I'm in complete awe of the work you guys have been doing to solve this mystery. I do have a question though, and forgive me for my ignorance.

Does anyone know if this effect scales with the size of the frustum? Could the frequency of the microwaves be adjusted to allow the same level of force in a microscopic frustum that is shown Nasa's macroscopic frustum? If this is the case, would a sequence of many millions of tiny frustums not provide a great deal more force than one large frustum?

Hopefully, that's one of the things we want to try and find out.
Title: Re: EM Drive Developments - related to space flight applications - Thread 2
Post by: Rodal on 03/12/2015 07:05 pm
Hey fellas, long time lurker, first time poster here.

I'm not even going to pretend to understand half the maths that have been tossed around in this thread. I'm in complete awe of the work you guys have been doing to solve this mystery. I do have a question though, and forgive me for my ignorance.

Does anyone know if this effect scales with the size of the frustum? Could the frequency of the microwaves be adjusted to allow the same level of force in a microscopic frustum that is shown Nasa's macroscopic frustum? If this is the case, would a sequence of many millions of tiny frustums not provide a great deal more force than one large frustum?
One thing one knows for sure is that the smaller the frustum, the higher the natural frequency, so in order to excite a given mode shape, one would have to use higher excitation frequencies for smaller frustrums.

@Notsosureofit has disclosed in this forum's thread his plans to test a smaller cavity with a higher excitation frequency, and his plan to use a Gunn diode (http://en.wikipedia.org/wiki/Gunn_diode) to generate the microwaves inside the cavity.

Concerning the use of literally "millions" of these devices (as in an Integrated Circuit), this could no longer involve microwaves (if possible at all) but it would mean much higher frequencies, because the wavelengths get cut-off due to the size of the cavity hence a much smaller cavity implies that the lowest natural frequency must be much higher.

Title: Re: EM Drive Developments - related to space flight applications - Thread 2
Post by: frobnicat on 03/12/2015 07:08 pm
....
....
...

@Frobnicat's conjecture is based on the shifting of the center of mass due to thermal expansion.

Yep.
here (http://forum.nasaspaceflight.com/index.php?topic=36313.msg1341514#msg1341514), there (http://forum.nasaspaceflight.com/index.php?topic=36313.msg1342118#msg1342118) in answer to zen-in's concerns around X axis that it would be actually worse around Z, and there (http://forum.nasaspaceflight.com/index.php?topic=36313.msg1342331#msg1342331)

This last one being important because it shows 3 things :
- The rest angular deviation of the arm from its initial position because of CoM's shifts in test article is worse than the angle of said CoM's shift as seen from the axis Z. 1µm deviation of CoM will cause more than one µm LDS deviation.
- Such LDS deviation will be interpreted as a sustained force when there is just a mass standing at some new place.
- The LDS deviation is counter-intuitive for orientation. A mass travelling to the right (like an expanding dielectric disc on the small end) will not increase the LDS distance but will decrease it. So the idea that the dielectric expansion could drive the rise actually observed is very well nullified already. It is further nullified by the weak relative power dissipated at the small end, and by the high thermal resistivity of the dielectric that makes only an extremely thin layer to heat in contact to copper, and from some initial heat conduction simulation of mine gets initially less than .01µm per second of CoM shift (for the whole dielectric block) that is clearly too weak to get the LDS readings anywhere.

Quote

Not only Chloroprene-Rubber (CR) has a higher coefficient of thermal expansion than High-Density-Polyethylene-HDPE, but Chloroprene-Rubber (CR) also has a higher density than High-Density-Polyethylene-HDPE.

So this further nullifies @Frobnicat's conjecture.

According to @frobnicat's conjecture, the denser, higher-thermal-expansion material ("Neoprene®") should have produced a higher measured "thrust", but NASA Eagleworks results show the opposite: the denser, higher-thermal expansion material ("Neoprene®") resulted in insignificant thrust.

...


Nope. (and I dare say, we are yet to see the charts recording those "insignificant thrusts" with or without such or such dielectric thereof, but this is an aside)

My conjecture is not about dielectric disc's CoM shift playing an important role in the thrust (did I say that ? Where ?) My conjecture is about some test article part's CoM shifting to the left (toward the small end) relative to fixation on the arm. Now what is susceptible to move to the left ? You know that better than anyone, you "invented" the inward buckling of the big end cap. The part that is the most heavily heated (granted this is not by a blowtorch !) and that has a boundary constraint such that, in first approximation, there is a square root between the delta expansion in plane and the resulting displacement perpendicular to plane : buckling is a very efficient amplifier. Under such buckling or near buckling conditions, the mass*displacement of the big end cap would play the major part of test article CoM's shift. Quantitative estimates ongoing...

The problem with thermal explanations is that, in particular in vacuum, given the low temperature deltas (a few °C) the evacuated heat rates are quite low relative to the received powers. The time constants to thermal equilibrium appear way beyond the 45s of a whole run. Therefore the fact that on some "thrusts" rises we see what looks like a thermal first order constant rate heat charge against a proportional loss don't hold water. At 45s the various parts are still swallowing heat at constant rate and evacuating near to none, we would have a near linear rise in temperature wrt time all way through. So if LDS delta is proportional to Com shift (as per the tilted pendulum component), Com shift proportional to expansion, expansion proportional to temperature, and temperature proportional to time, we should see a linear rise, and not a "step". Yes but the buckling could make Com shift proportional to square root of expansion. Now look at the chart below and see the step not as a cst-cst*exp(-cst*t) as per a naive thermal explanation but as a cst*sqrt(cst*t).

So the "attack" and the "sustain" can both be very well explained by progressive thermal expansion near buckling conditions and by a slightly tilted Z axis. Now for the fall (decay) : for those still believing that thermal explanations are irrelevant, how is it possible that the decay is lingering at high LDS values for so long after power off ? But, with so low thermal radiation for cooling, there is no reason (from my conjectures so far) that there would be any significant decay at all : from my hypothesis the signal should stay constantly high at power off, only starting falling at a very small rate (much smaller that the rise rate).

This is why I said in previous post to Star-Drive that I don't believe in thermal effect as being the only cause of observed signal, from the shapes. Not because of rise and sustain (square root buckling amplification + tilted Z allowing for sustained "thrusts" by sustained relative Com's displacements) but because of decay. I do have an idea to explain that : Rodal have you considered that the supporting copper ring around the FR4 big end cap would also expand thermally ? What would happen if there was a (thermal conduction driven) temperature "delay" between the cap and the ring so that when the power stops the difference between cap temperature and ring temperature falls fast enough to be compatible with the time constant of the observed decay ? This is my leading conjecture. I now do believe again in the possibility of a purely thermal explanation wholly consistent with both magnitude and shape of signal.

(http://forum.nasaspaceflight.com/index.php?action=dlattach;topic=36313.0;attach=636345;image)

Edit : also not clear how the drifting baseline would fit in this framework of moving CoMs, would need a test article CoM rightward overall before power on and well after power off, looks like a contradiction of some of the hypothesis above...
Title: Re: EM Drive Developments - related to space flight applications - Thread 2
Post by: aero on 03/12/2015 07:49 pm
Hey fellas, long time lurker, first time poster here.

I'm not even going to pretend to understand half the maths that have been tossed around in this thread. I'm in complete awe of the work you guys have been doing to solve this mystery. I do have a question though, and forgive me for my ignorance.

Does anyone know if this effect scales with the size of the frustum? Could the frequency of the microwaves be adjusted to allow the same level of force in a microscopic frustum that is shown Nasa's macroscopic frustum? If this is the case, would a sequence of many millions of tiny frustums not provide a great deal more force than one large frustum?
One thing one knows for sure is that the smaller the frustum, the higher the natural frequency, so in order to excite a given mode shape, one would have to use higher excitation frequencies for smaller frustrums.

@Notsosureofit has disclosed in this forum's thread his plans to test a smaller cavity with a higher excitation frequency, and his plan to use a Gunn diode (http://en.wikipedia.org/wiki/Gunn_diode) to generate the microwaves inside the cavity.

Concerning the use of literally "millions" of these devices (as in an Integrated Circuit), this could no longer involve microwaves (if possible at all) but it would mean much higher frequencies, because the wavelengths get cut-off due to the size of the cavity hence a much smaller cavity implies that the lowest natural frequency must be much higher.

There is also the materials issue. The behavior of copper, for example, and the dielectric is much changed at much higher frequencies. By much, I mean "a lot" not "a little bit." By the time you got 4000 of them in the space of the current frustum, none of your materials would behave as they do in the current frustum.
Title: Re: EM Drive Developments - related to space flight applications - Thread 2
Post by: Rodal on 03/12/2015 09:16 pm
....
My conjecture is not about dielectric disc's CoM shift playing an important role in the thrust (did I say that ? Where ?) ...
We had a discussion with reference to a drawing presented by Paul March showing thermal expansion of the dielectric.

(http://forum.nasaspaceflight.com/index.php?action=dlattach;topic=36313.0;attach=776495;image)

 I recall you arguing that thermal expansion of the dielectric would produce a force in the same direction as the measured force.  I argued against it, on the basis that the dielectric has a free surface and it is free to expand unrestrained.  A homogenous isotropic material free to expand uniaxially produces no thermal stresses. Only restrained materials produce thermal stresses.  Furthermore I argued that the maximum speed of thermal expansion is governed by the speed of sound and that there is no second order derivative with respect to time due to thermal expansion.  There is no "acceleration of thermal expansion".  And that Fourier's equation contains only a first order derivative with respect to time, no second order derivative.
 
What's important is that the experiments by Paul March showing insignificant thrust force measurements when using the Chloroprene-Rubber-CR ("Neoprene®")  as compared to when using High Density Polyethylene, invalidate the conjecture of the measurements being an artifact due to thermal expansion (as for example advanced by the Oak Ridge group  (section 7 of http://web.ornl.gov/~webworks/cppr/y2001/pres/111404.pdf)).

http://forum.nasaspaceflight.com/index.php?topic=36313.msg1344997#msg1344997

http://forum.nasaspaceflight.com/index.php?topic=36313.msg1344942#msg1344942
Title: Re: EM Drive Developments - related to space flight applications - Thread 2
Post by: Notsosureofit on 03/12/2015 09:52 pm
http://web.ornl.gov/~webworks/cppr/y2001/pres/111404.pdf)).


The subject of thermal "pumping" is one that will have to be carefully considered in any Cavandish experiment.  What procedures are best for it's elimination ?  Making sure the CG of the heated object doesn't move ?

Title: Re: EM Drive Developments - related to space flight applications - Thread 2
Post by: frobnicat on 03/12/2015 10:32 pm
@Rodal,

Yes, that was about recoil forces, my arguments are still valid, a block of material starting to expand thermally will push a wall against which it is resting, whether the expanding material is the hot gas of an explosion or a block of PTFE heating from the left against copper at 0.1°C/s, it's only a (huge) difference of magnitude, not of nature (as far as recoil is concerned). Done some number, such recoil is not significant given inertia of pendulum. Like 0.1µN on the 1st second of heating. Kind of, order of magnitude. Please call it 0 if you feel so inclined. But I will continue to stand on my position on italic statement (separate post later to avoid mixing).

About dielectric role : Paul March is not really showing insignificant thrust force measurements, he is stating that there is insignificant thrust, we are yet to see the corresponding charts of those situations of insignificant thrust. If it's a flat line as flat as for the 50Ohm null test, very well, that will put all speculation about thermal expansions at rest (provided the exciting frequencies are adapted to the absence of dielectric to get similar modes and therefore similar heating patterns). We really lack a catalog of hundred of "events".

Now if you can leave recoil effects aside for a moment (I know we had quite a discussion on that), isn't it speaking to you that there is now a situation where LDS_reading(t)=cst1*Thrust(t)+cst2*CoMPosition(t) where cst1 and cst2 are in the same ballpark ? That we are (well I am) no longer speaking of Force(t)=M*d²CoMPosition(t)/dt² (recoil) where CoMs displacement can't sustain a force for a significant amount of time but we are now in "direct drive" from displacement of CoM to LDS reading ?

The Oak ridge group analysis is irrelevant (recoil effects, not considering a tilted pendulum), the dielectric material density, expansion coefficient, thermal conductivity are probably not relevant directly as this is on the cold end. They are relevant for microwave modes and heating patterns.

Now, please please please, comment on that : LDS_reading(t)=cst1*Thrust(t)+cst2*CoMPosition(t) where cst1 and cst2 are in the same ballpark
Do you agree or not ? If not why. If yes, do you think it is important or not, if not why. And I'm speaking of the hot end. We have a really rich thermal phenomenology here. Please try to understand what I'm saying, if you are still interested in possible conventional explanations and not just to quickly dismiss such possibilities. Such possibility would require careful understanding of the pendulum system and test article, not confusion of CoMPosition(t) with d²CoMPosition(t)/dt².

Title: Re: EM Drive Developments - related to space flight applications - Thread 2
Post by: Rodal on 03/12/2015 11:05 pm
http://web.ornl.gov/~webworks/cppr/y2001/pres/111404.pdf)).


The subject of thermal "pumping" is one that will have to be carefully considered in any Cavandish experiment.  What procedures are best for it's elimination ?  Making sure the CG of the heated object doesn't move ?

Concerning movement of the Center of Mass of the object by thermal expansion, such movement should very slow unless you use very thin shells (for which, the issue of thermal buckling should be considered).  Unless you use very thin shells and shapes that can change shape suddenly due to thermal instability, such movement should be negligible when considering the response of a few seconds. By all means avoid thin flat plates that are constrained at the edges. Curved thin shells are much better in this respect than thin flat plates.



Equations 14, 15 and 16 of the above Oak Ridge reference are incorrect.  Such equations cannot be found in any classic work on thermoelasticity (for example on Boley and Wiener's monograph), and for good reason.

The authors begin by converting the expression for the coefficient of thermal expansion:

LengthFinal - LengthInitial= alpha*(Tfinal - Tinitial)

to a differential form

dL /L= alpha *dT 

This is equivalent to stating that the differential of the logarithmic strain equals alpha*dT

Then, the authors use Newton's second law:

F = m * a = m * d^2 x / dt^2  to obtain a force for the unrestrained thermal expansion using the acceleration, as follows

dL/dt = L * alpha * dT/dt

d^2L/dt^2 = L * alpha * d^2T/dt^2

This is wrong, because thermal expansion proceeds at a constant speed: the speed of sound in the material.
The derivative of a constant is zero. Hence its derivative is zero, and not  L * alpha * d^2T/dt^2.  The equations that follow from the authors with terms involving  mass* alpha * d^2T/dt^2 are therefore non-existent.

That's why such "thermal expansion recoil force" equations do not appear in any classic reference on thermoelasticity.
Title: Re: EM Drive Developments - related to space flight applications - Thread 2
Post by: Rodal on 03/12/2015 11:11 pm
@Rodal,

Yes, that was about recoil forces, my arguments are still valid, a block of material starting to expand thermally will push a wall against which it is resting, whether the expanding material is the hot gas of an explosion or a block of PTFE heating from the left against copper at 0.1°C/s, it's only a (huge) difference of magnitude, not of nature (as far as recoil is concerned). Done some number, such recoil is not significant given inertia of pendulum. Like 0.1µN on the 1st second of heating. Kind of, order of magnitude. Please call it 0 if you feel so inclined. But I will continue to stand on my position on italic statement (separate post later to avoid mixing).

....
:)
Title: Re: EM Drive Developments - related to space flight applications - Thread 2
Post by: zen-in on 03/13/2015 05:39 am

Nope. (and I dare say, we are yet to see the charts recording those "insignificant thrusts" with or without such or such dielectric thereof, but this is an aside)

My conjecture is not about dielectric disc's CoM shift playing an important role in the thrust (did I say that ? Where ?) My conjecture is about some test article part's CoM shifting to the left (toward the small end) relative to fixation on the arm. Now what is susceptible to move to the left ? You know that better than anyone, you "invented" the inward buckling of the big end cap. The part that is the most heavily heated (granted this is not by a blowtorch !) and that has a boundary constraint such that, in first approximation, there is a square root between the delta expansion in plane and the resulting displacement perpendicular to plane : buckling is a very efficient amplifier. Under such buckling or near buckling conditions, the mass*displacement of the big end cap would play the major part of test article CoM's shift. Quantitative estimates ongoing...

The problem with thermal explanations is that, in particular in vacuum, given the low temperature deltas (a few °C) the evacuated heat rates are quite low relative to the received powers. The time constants to thermal equilibrium appear way beyond the 45s of a whole run. Therefore the fact that on some "thrusts" rises we see what looks like a thermal first order constant rate heat charge against a proportional loss don't hold water. At 45s the various parts are still swallowing heat at constant rate and evacuating near to none, we would have a near linear rise in temperature wrt time all way through. So if LDS delta is proportional to Com shift (as per the tilted pendulum component), Com shift proportional to expansion, expansion proportional to temperature, and temperature proportional to time, we should see a linear rise, and not a "step". Yes but the buckling could make Com shift proportional to square root of expansion. Now look at the chart below and see the step not as a cst-cst*exp(-cst*t) as per a naive thermal explanation but as a cst*sqrt(cst*t).

So the "attack" and the "sustain" can both be very well explained by progressive thermal expansion near buckling conditions and by a slightly tilted Z axis. Now for the fall (decay) : for those still believing that thermal explanations are irrelevant, how is it possible that the decay is lingering at high LDS values for so long after power off ? But, with so low thermal radiation for cooling, there is no reason (from my conjectures so far) that there would be any significant decay at all : from my hypothesis the signal should stay constantly high at power off, only starting falling at a very small rate (much smaller that the rise rate).

This is why I said in previous post to Star-Drive that I don't believe in thermal effect as being the only cause of observed signal, from the shapes. Not because of rise and sustain (square root buckling amplification + tilted Z allowing for sustained "thrusts" by sustained relative Com's displacements) but because of decay. I do have an idea to explain that : Rodal have you considered that the supporting copper ring around the FR4 big end cap would also expand thermally ? What would happen if there was a (thermal conduction driven) temperature "delay" between the cap and the ring so that when the power stops the difference between cap temperature and ring temperature falls fast enough to be compatible with the time constant of the observed decay ? This is my leading conjecture. I now do believe again in the possibility of a purely thermal explanation wholly consistent with both magnitude and shape of signal.


It's amazing how long this discussion has gone on.   All the brilliant individuals who have contributed complex physical and mathematical methods of analyzing this device; yet there is still not an accepted explanation for the anomalous thrust.   

There are several observations that can't be disputed:

1)  The thrust signature for both the Cannae device and the Eagleworks cone shaped cavity are similar to the thrust signature of the capacitor calibrator.   All are underdamped and have nearly the same natural frequency.    This is to be expected because all are step responses.   However this is only true for thrust waveforms from the AIAA Aug 2014 paper.

2)  The correction for error thrust due to PA current interacting with the damping magnets is not done for the vacuum tests.   It appears this error source was eliminated.

3)  The thrust signature for the vacuum tests are very different.   There is no overshoot or ringing, the rise time is slow, and a residual thrust remains after the RF is turned off.    This test, if it was to confirm earlier results, would have to have the same thrust signature (overdamped).   Instead it looks like a thermal effect.   The displacement is very small (from 1 - 4 micrometers).

One possible explanation is that the PA current went up when the dielectric material was put in the cavity, due to a higher SWR.   If the PA current was above the level that was used to measure the correction factor then the anomalous thrust seen was just more magnetic torque.   Since this correction was not used with the vacuum tests it can be assumed the problem was mitigated - maybe by coaxializing the power leads.   Frobnicat has proposed an explanation for the residual force seen in the vacuum measurements.   With no air surrounding the cavity it takes much longer for the heat to escape.   Some kind of long duration thermal flexing is causing a change in the LDS reading.   It may be because the alignment of the beam changes ever so slightly when the CM changes.   

The indisputable fact that the two sets of experiments show a thrust waveform with very different shape even though the TP has not been changed invalidates the claims.
Title: Re: EM Drive Developments - related to space flight applications - Thread 2
Post by: Rodal on 03/13/2015 12:29 pm

....


It's amazing how long this discussion has gone on.  ....yet there is still not an accepted explanation for the anomalous thrust...   There are several observations that can't be disputed:

....
Given 1) the extremely small funding (by any objective assessment) of the NASA Eagleworks project,  2) the small magnitude of the measurements involved, 3) the outstanding claims (including issues of conservation of momentum), and 4) the barely 6 months since the release of the "Anomalous..." report and the discussion in this thread, I don't find it at all "amazing" that there is no uniformly accepted mechanism for the measurements.  Actually I don't understand what "acceptance" could possibly mean in this context (an anonymous Internet forum thread).

So, an objective definition of "acceptance" must mean scientific community acceptance.  As far as acceptance in the scientific community at large, such "acceptance" could only be expected after attempts have been made to reproduce the results at academic institutions, and for that to happen (given the small level of funding for this project) it would take much more than 6 months (certainly a Ph.D. thesis at a major university would take years).  We are not even at the point where there is certainty that NASA Glenn will try to replicate the tests (and much less at the point of replicating the tests at leading universities)

A fair assessment acknowledges that progress has been made in eliminating proposed explanations for the thrust measurement being an experimental artifact.  For example:

Observation #1) The explanation (e.g. the jet model from a poster in this thread, and alternative air current conjectures in several blogs in the Internet) that the thrust measurements were due to thermal convection in the air has been nullified by NASA Eagleworks tests in a vacuum at  6.6*10^(-9) standard atmosphere = 0.0000000066 standard atmosphere

...


Karl Popper: The Logic of Scientific Discovery (1934).

There is always some uncertainty associated with scientific conclusions; physical sciences never absolutely prove anything (unlike Mathematics).  What physical sciences do is nullify false explanations.



John von Neumann

Quote
The sciences do not try to explain, they hardly even try to interpret, they mainly make models. By a model is meant a mathematical construct which, with the addition of certain verbal interpretations, describes observed phenomena.
"Method in the Physical Sciences", in The Unity of Knowledge (1955), ed. L. G. Leary (Doubleday & Co., New York), p. 157
Title: Re: EM Drive Developments - related to space flight applications - Thread 2
Post by: zen-in on 03/13/2015 01:00 pm

....


It's amazing how long this discussion has gone on.  ....
There are several observations that can't be disputed:


There are several observations that can't be disputed:

1)  The thrust signature for both the Cannae device and the Eagleworks cone shaped cavity are similar to the thrust signature of the capacitor calibrator.   All are underdamped and have nearly the same natural frequency.    This is to be expected because all are step responses.   However this is only true for thrust waveforms from the AIAA Aug 2014 paper.

2)  The correction for error thrust due to PA current interacting with the damping magnets is not done for the vacuum tests.   It appears this error source was eliminated.

3)  The thrust signature for the vacuum tests are very different.   There is no overshoot or ringing, the rise time is slow, and a residual thrust remains after the RF is turned off.    This test, if it was to confirm earlier results, would have to have the same thrust signature (overdamped).   Instead it looks like a thermal effect.   The displacement is very small (from 1 - 4 micrometers).

One possible explanation is that the PA current went up when the dielectric material was put in the cavity, due to a higher SWR.   If the PA current was above the level that was used to measure the correction factor then the anomalous thrust seen was just more magnetic torque.   Since this correction was not used with the vacuum tests it can be assumed the problem was mitigated - maybe by coaxializing the power leads.   Frobnicat has proposed an explanation for the residual force seen in the vacuum measurements.   With no air surrounding the cavity it takes much longer for the heat to escape.   Some kind of long duration thermal flexing is causing a change in the LDS reading.   It may be because the alignment of the beam changes ever so slightly when the CM changes.   

The indisputable fact that the two sets of experiments show a thrust waveform with very different shape even though the TP has not been changed invalidates the claims.
Actually, an objective, fair assessment starts by acknowledging that progress has been made in eliminating proposed explanations for the thrust measurement being an experimental artifact.  For example:

Observation #1) The explanation (e.g. @frobnicat's jet model, and by several blogs in the Internet) that the thrust measurements were due to thermal convection in the air has been nullified by NASA Eagleworks tests in a hard vacuum at  6.6*10^(-9) standard atmosphere = 0.0000000066 standard atmosphere


While the vacuum test did invalidate the theory of anomalous thrust being the result of air currents it also invalidated the anomalous thrust.   The earlier STP tests and the vacuum test were done with the same hardware and had the same driving function.    The system response should be similar but they aren't.  There are two different causes for the indication of an anomalous thrust.   Therefore there is no thrust.
Title: Re: EM Drive Developments - related to space flight applications - Thread 2
Post by: RotoSequence on 03/13/2015 02:43 pm
While the vacuum test did invalidate the theory of anomalous thrust being the result of air currents it also invalidated the anomalous thrust.   The earlier STP tests and the vacuum test were done with the same hardware and had the same driving function.    The system response should be similar but they aren't.  There are two different causes for the indication of an anomalous thrust.   Therefore there is no anomalous thrust.

The behavior of the hardware is inconsistent in different atmospheric conditions, but there is a consistent, small measurement that shows up after the power is turned off, therefore it must be thermal, ergo there is nothing happening?  :o

That sounds like a mighty big leap, to me.
Title: Re: EM Drive Developments - related to space flight applications - Thread 2
Post by: Superfastjellyfish on 03/13/2015 04:22 pm

....


It's amazing how long this discussion has gone on.  ....
There are several observations that can't be disputed:

....
Actually, an objective, fair assessment starts by acknowledging that progress has been made in eliminating proposed explanations for the thrust measurement being an experimental artifact.  For example:

Observation #1) The explanation (e.g. @frobnicat's jet model, and by several blogs in the Internet) that the thrust measurements were due to thermal convection in the air has been nullified by NASA Eagleworks tests in a hard vacuum at  6.6*10^(-9) standard atmosphere = 0.0000000066 standard atmosphere

...


Karl Popper: The Logic of Scientific Discovery (1934).

There is always some uncertainty associated with scientific conclusions; physical sciences never absolutely prove anything (unlike Mathematics).  What physical sciences do is nullify false explanations.



John von Neumann

Quote
The sciences do not try to explain, they hardly even try to interpret, they mainly make models. By a model is meant a mathematical construct which, with the addition of certain verbal interpretations, describes observed phenomena.
"Method in the Physical Sciences", in The Unity of Knowledge (1955), ed. L. G. Leary (Doubleday & Co., New York), p. 157

While the vacuum test did invalidate the theory of anomalous thrust being the result of air currents it also invalidated the anomalous thrust.   The earlier STP tests and the vacuum test were done with the same hardware and had the same driving function.    The system response should be similar but they aren't.  There are two different causes for the indication of an anomalous thrust.   Therefore there is no anomalous thrust.

I'm probably misunderstanding you, but in the following post, Paul March says:

http://forum.nasaspaceflight.com/index.php?topic=36313.msg1329225#msg1329225
"As to why the vacuum test were observing less thrust than in air tests. please note the difference in the RF amps there were driving each test series.  The 30W Mini-Circuit Class-A RF amp was used for the in-air series reported in the 2014 JPC paper, whereas a 100W EMPower Class-A/B RF amplifier was used in the vacuum tests to date. "
Title: Re: EM Drive Developments - related to space flight applications - Thread 2
Post by: JPLeRouzic on 03/13/2015 04:56 pm
Observation #1) The explanation (e.g. the jet model from a poster in this thread, and alternative air current conjectures in several blogs in the Internet) that the thrust measurements were due to thermal convection in the air has been nullified by NASA Eagleworks tests in a vacuum at  6.6*10^(-9) standard atmosphere = 0.0000000066 standard atmosphere
If some kind of perfect Ion engine expels a gas with a density as thin as the one in the NASA Eagleworks vacuum chamber, through a one cm3 nozzle and at a relativistic speed (let say 200,000 km/sec) the thrust due to this very thin gas is in the order of several Newtons.

A far more imperfect Ion engine can certainly provide 100,000 less thrust by accident at this level of pseudo-vacuum.
Title: Re: EM Drive Developments - related to space flight applications - Thread 2
Post by: Notsosureofit on 03/13/2015 05:31 pm
FYI

The 10GHz chamber just arrived ...

Not exactly the profile I would have hoped for but .... as long as I don't get my bells and hammers mixed up ...
Title: Re: EM Drive Developments - related to space flight applications - Thread 2
Post by: zen-in on 03/13/2015 06:30 pm

I'm probably misunderstanding you, but in the following post, Paul March says:

http://forum.nasaspaceflight.com/index.php?topic=36313.msg1329225#msg1329225
"As to why the vacuum test were observing less thrust than in air tests. please note the difference in the RF amps there were driving each test series.  The 30W Mini-Circuit Class-A RF amp was used for the in-air series reported in the 2014 JPC paper, whereas a 100W EMPower Class-A/B RF amplifier was used in the vacuum tests to date. "
Welcome to the discussion
Both amplifiers supply a CW RF pulse to the cavity.  Both drive the cavity with the same kind of drive.   A close analogy would be comparing the sound of a bronze bell when it's hit with a lead hammer vs a hardwood hammer.   The sound is the same.   The step response of a system - be it a circuit, a mechanical system, or almost anything else- is determined by the nature of the system.   I don't believe it is correct to say that subtle differences in the RF drive has resulted in very different responses.    That requires more new science.   

Occam's razor states that among competing hypotheses that predict equally well, the one with the fewest assumptions should be selected.    The vacuum tests do not show the underdamped response that was seen earlier.   Something was done to mitigate the thrust error caused by the amplifier current generating a torque against the damping magnets.   The picture below shows a large attenuator on what looks like the output of the amp.   This was used possibly to minimize the affects of high SWR.   A high SWR results in the amplifier drawing excessive current.   Earlier posts describe this problem and that the amp had to be returned for repairs.    Something was done to mitigate the thrust error from amplifier current in the vacuum tests.   This attenuator may have been installed for that reason.   With 2 different step responses we are left with the choice of:
1)  Elaborating on the theory of this device further to explain why the system response changes.
2)  Accepting that 2 different experiments have produced results that contradict each other.

Of these two choices Occam requires we choose the one with the fewest new assumptions, which is #2.
Title: Re: EM Drive Developments - related to space flight applications - Thread 2
Post by: Mulletron on 03/13/2015 06:45 pm
Ok, I got the frustum in the mail today. This is my impartial review. No post purchase rationalization here.

The item was well protected in a sturdy double box with brown paper as padding. I don't expect one receiving a dented frustum if further ones are packaged the same way. Attached are some pics showing zoomed in shots on a 1/16" steel ruler showing tolerances. The machine shop promised me +-1/8" tolerances, he surpassed beat those tolerances. I'm not too concerned about tolerances anyway as I am going to brute force this with a wideband RF source. This cone is unique, and so will be the next one that is built. This is a strength for replication purposes.

I had to push and pull in order to return the frustum to true roundness. It was about 1/8-3/16" out from true round what I got it. I was able to get it round with minor effort. This could be avoided with round rings around the frustum in at least two places, soldered in place, like the one at Eagleworks, with added cost of course. It holds itself true after adjustment. The frustum is nearly as stiff as an aluminum bucket as is. (Think of those ones at Joe's Crab Shack) It takes substantial force to correct it.

The frustum is pictured on my glass top kitchen table, which is extremely flat and true. the frustum sits flush (no rocking, no gaps) against the table on the small and large ends. I flattened out small aberrations in the trueness of the flange (which might cause gaps with the end plate) by drawing the flat edge of a very large/heavy cold chisel handle (in one of the pics) across the copper flange. Copper is very ductile, so minor adjustments are pretty easy.

The butt seam is composed of a 1/2" strip of copper soldered to the outside of the frustum, holding the walls together, allowing a smooth interface inside.

My biggest area of concern is at the ends of the seams, on the inside, there was a very slight bowing in/bending out of the copper. I was able to correct this with some slight pressure from an impromptu tool (a 9" socket extension) which allowed me to flatten it out. The smooth chromed tool didn't make too many tool marks so I'm happy.

The final area of concern is a small air gap (equal to the thickness of the metal) which will form at the seam when the end places plates are clamped in place. A small hole isn't a make or break, but it will bug me (Rf doesn't leak out of holes like liquid does), so I intend to correct for this by using a small 1/4" wide piece of conductive copper tape around/over the gap. I might just drop a bead of solder in there too. Doesn't matter really.

The 16oz copper used in the construction does not deform under its own weight. The inside is smooth and shiny, which is where it counts. The tolerances are pretty tight. I'm happy with it. I don't think I could have done a better job within the first 5 tries at really nuking trying to make one on my own.

In all, I'd give this frustum a 7/10. It is in my opinion capable of being a decent resonant cavity. What would make it perfect is actual heavy gauge flanges and both ends and reinforcement rings, both of which are nice but not necessary. I'm going to be busy over the next several days, installing rf connectors, building a balance and doing a cursory sweep of this with a signal generator at work, looking for its own unique unloaded resonant frequency. In shipping there are sheets of UHMW PE, HD PE, PVDF, PTFE, PP and other stuff to play with. Then I will do loaded characterizations of the frustum.

When all the materials arrive, I am going to hop on here so we can design a test protocol, starting with a Cavendish type gravity experiment in order to characterize the balance. Then the Eagleworks device (as close as we can get). I'm trying to do this all on 2.4ghz ISM band if possible (maybe not the best mode shapes and not optimized, but it is a start). If there is success, then maybe I'll in vest in another RF source.

I am very very anxious that I will be able to find a 2450mhz +-50 mhz resonance with an Eagleworksish loaded frustum, with 2 HD PE 1" discs installed. It is a leap of faith as of now. I am banking on the data Eagleworks gave me and knowing that higher order resonances are there and the number is infinite. I might fall squarely on my face. I need exact solutions!

There is a lot of work to do and challenges to overcome. I want this experiment to be a NSF driven effort. That means I need your expertise. After we get past the "does it move or not" phase, we'll have to do force calculations using what we learned from the balance. I plan to do the gravity experiment first, then replicate the Eagleworks device, the rest is devoted to finding what works and what doesn't (where all the other polymers and Chromium Oxide come in). I will take clear pictures of the products, like the Trilene 4 and 6lb test monofilament line because we need data on these. Going to do 4lb first and 6lb if 4lb fails.

There will be an Internet camera monitoring the action at (url is secret until later....com) Rodal knows. Due to limited bandwidth, the username/password will be shared amongst the top 10 here on the forum. I simply don't have high speed internet where I live overseas. If there is a way to do something like Ustream with my Foscam camera, please let me know so I can make it fully public. This is the best option. It is a technical challenge. Can I push just one stream to a provider and cheaply have them host the live video?? I only have 512k to reliably play with.

Now about the vacuum chamber. I'm not selling a car in order to get a vacuum chamber and I want the cavity chamber walls out of the picture anyway. There will be a dedicated sealed off room for the tests. I am an American living overseas in Italy. Italian houses are rock solid and have no ventilation so the place is stable and there will be no air currents. The test device will be in a box in a box, probably in a box. I have to control for convection in a cheaper/smarter way. Simple as that. If Eagleworks already did vacuum tests, I don't see why it absolutely has to be done every single time. It should work nicely. I'm going to seal the room with zipper doors too.

Lastly, this effort is to crowdsource the answer to a question. It doesn't qualify as an academic institution or a national lab performing research. I have questions. I work with you guys, who also have questions. We each have our strengths and weaknesses, and together I think we are the total package. I'm trying to get others to replicate the Shawyer/Eagleworks/Chinese tests, so I am trying to lead by example. I think this is the goal of many of us on here, to get others to take a look....so it is time to take charge and move on this.

I'm going to messenger the machine shop here in a bit and have him list the thing on Ebay for other replicators who want a way in to testing something with the characteristics of the Shawyer/Eagleworks device. I'll post further details later on. Oh and I have no business affiliation with this guy other than he took my money, and he gave me a copper kettle.





Title: Re: EM Drive Developments - related to space flight applications - Thread 2
Post by: frobnicat on 03/13/2015 07:08 pm
Follow up on thermal effects around the big diameter PCB cap :

I ran simulations on a small simplified model : a 28cm round PCB plate alone in vacuum starting at thermal equilibrium in a 20° surrounding, 35µm copper on top of 1.6mm FR4 support, with copper side uniformly heated by 30W. The model uses 2 discrete layers for copper, and 20 layers for the FR4 support, time step integration of 1µs, running on personal code, first order integrator for heat conduction through plane and radiation losses (as per Stefan-Boltzmann law in T^4).

First run, two first attached pictures : 1600 s of constant power.
We see that on power on it takes many minutes to reach a new thermal equilibrium plateau where the radiative losses equal the constant 30W feed. When this stationary situation is reached, the temperature gradient is constant through the thickness of the PCB as each layer gets as much power from its left as it loses on its right. Comment : this is why it is not possible to interpret the smooth steps in some charts as a reach to some stationary thermal equilibrium, the thermal time constants are above one order of magnitude above the recorded dynamics.

Second run, two last attached pictures : 5s without power, 40 s with power 30W, then off again.
Here on temperatures vs time we will see only the initial situation of previous chart, where the temperatures are not high enough for radiative losses to be significant. In this regimen the various layers just integrate the constant rate of heat, all temperatures are rising near linearly with time, but the outside temperature of the PCB lags after the temperature of copper. Side note, we see on the profile that in this situation far from stationary equilibrium the temperature profile is not a constant gradient through the depth. If we look the difference between temperature copper side and the outer side of the PCB, roughly the gradient, this gradient does exhibit a rise (at power on) and a fall (at power off) with time constants and shapes in the ball park of the frustum's LDS response recorded in vacuum.

This is the green curve, bottom attached image. There is a fast enough but not instantaneous rise to near constant plateau above 1°C difference, and a near symmetric decay. So what ? Well, a gradient of 1°C side from side on 1.6mm thick plate will induce a stress in flexion. If the plate was a thin stripe and not rigidly fixed at the rims it would bend into an arc shape, with a deflection at the belly of L²/(8*d) * ExpanCoef*(T2-T1) where T2 is hot side, T1 cold side, L the rest length (28cm), d the depth (1.6mm) and ExpanCoef is about 1.3e-5m/m/K (FR4 inplane, neglecting the copper for now). That is around 80µm for 1°C difference.

Obviously for a round plate and not a thin stripe, the stiffness in tangential deformations will lower that quite a bit, and the plate is rigidly linked at its rim, not only in position but also in flexion. So there's still some work to know the exact magnitude and dynamic by integrating those constraints (and the rim will heat differentially too).

Title: Re: EM Drive Developments - related to space flight applications - Thread 2
Post by: Rodal on 03/13/2015 07:38 pm
....
2)  Accepting that 2 different experiments have produced results that contradict each other.
....
Occam's razor states that among competing hypotheses that predict equally well, the one with the fewest assumptions should be selected. 

1) As per above, the experiments are not exactly the same: besides testing in a relative vacuum (6.6*10^(-9) standard atmosphere) vs. one standard atmosphere, there are a number of components that have changed (for example, besides " The 30W Mini-Circuit Class-A RF amp was used for the in-air series reported in the 2014 JPC paper, whereas a 100W EMPower Class-A/B RF amplifier was used in the vacuum tests to date" you yourself point out the following change: " Something was done to mitigate the thrust error caused by the amplifier current generating a torque against the damping magnets.   The picture below shows a large attenuator on what looks like the output of the amp.  ").  Thus, care should be taken to address all the changes when applying classical logic to examine the results.   It is not a question of A) the experiment are exactly the same (this is definitely not the case) or B) the experiments are completely different (this is not the case either), but a case of C) the experiments have many aspects of commonality and a few different aspects (that need to be scientifically addressed).

2) As used in classical logic: "a contradiction consists of a logical incompatibility between two or more propositions. It occurs when the propositions, taken together, yield two conclusions which form the logical, usually opposite inversions of each other"   Thus contradiction usually implies an inversion of results.  There is no inversion of results here.  Both generate a thrust force, in the same direction under the same geometry and materials.  There is a difference in magnitude between the experiments that needs to be explained. There is a wide range of explanations for these results that do not imply a contradiction.  Non-contradictory explanations comprise the full range of possibilities: from the experiments in atmospheric conditions and the ones in a relative vacuum being both experimental artifacts to both being valid demonstrations of a space thruster.  For example, a non-contradictory explanation is that thermal convection effects were responsible for some fraction of the total measured response in the US, (and a greater fraction in the UK and China due to their use of much higher power), and therefore the magnitude of the measured response in a relative vacuum is smaller.

3) When applying Occam's razor, the focus should be on simplicity overall, as per the original statement of Occam.  Thus, it is not just the number of assumptions but most importantly the simplicity of the assumptions that matter, and of course the overall consistency of the arguments. 



Albert Einstein's famously stated:

Everything should be made as simple as possible, but not simpler.




Title: Re: EM Drive Developments - related to space flight applications - Thread 2
Post by: flux_capacitor on 03/13/2015 07:44 pm
Does anyone know if this effect scales with the size of the frustum? Could the frequency of the microwaves be adjusted to allow the same level of force in a microscopic frustum that is shown Nasa's macroscopic frustum? If this is the case, would a sequence of many millions of tiny frustums not provide a great deal more force than one large frustum?

Submillimeter-wide multiple terahertz radiation cavities covering the entire structure of the ship, actually being the ship, without any thruster visible. What an interesting and provoking thought!
Title: Re: EM Drive Developments - related to space flight applications - Thread 2
Post by: Rodal on 03/14/2015 12:53 am
Does anyone know if this effect scales with the size of the frustum? Could the frequency of the microwaves be adjusted to allow the same level of force in a microscopic frustum that is shown Nasa's macroscopic frustum? If this is the case, would a sequence of many millions of tiny frustums not provide a great deal more force than one large frustum?

Submillimeter-wide multiple terahertz radiation cavities covering the entire structure of the ship, actually being the ship, without any thruster visible. What an interesting and provoking thought!


Quote from: MIT News
What these satellites lack is a viable propulsion system, says MIT aeronautics and astronautics alumna Natalya Brikner PhD ’15, co-founder and CEO of Accion Systems. “You can make a satellite the size of a softball with a surprising amount of capabilities, but it can’t maneuver properly and falls from orbit quickly,” she says. “People are waiting for a solution.”
Now Accion has developed a commercial electrospray propulsion system — their first is about the size of a pack of gum — made of tiny chips that provide thrust for small satellites. Among other advantages, Accion’s module can be manufactured for significantly less than today’s alternatives.
This technology could enable low-cost satellites, such as those known as “CubeSats,” to become more viable for various commercial and research applications, including advanced imaging and communications, where numerous satellites could provide global coverage. “That requires propulsion, but something so small that it won’t interfere with the small volume and resources a small satellite already has,” says Accion technical advisor Paulo Lozano, an associate professor of aeronautics and astronautics who invented the underlying technology.

http://newsoffice.mit.edu/2015/accion-systems-thruster-for-small-satellites-0311#.VQGgo9sCC8A.linkedin

http://www.leonarddavid.com/tiny-thruster-offers-big-promise/

http://newsoffice.mit.edu/2012/microthrusters-could-propel-small-satellites-0817

https://www.youtube.com/watch?v=BHVkc2JwAuI

(http://www.nanowerk.com/news2/space/id39382.jpg)

(http://www.leonarddavid.com/wp-content/uploads/2014/09/MICROTHRUSTER-300x190.jpg)

(http://www.leonarddavid.com/wp-content/uploads/2014/09/MICROTHRUSTER-2-300x168.png)

(http://www.leonarddavid.com/wp-content/uploads/2014/09/microthruster-3-300x200.jpg)

(http://static1.squarespace.com/static/5446faa2e4b025843cfc6731/t/549f0c4fe4b0d2e1e20f5fcf/1419709519668/)
Title: Re: EM Drive Developments - related to space flight applications - Thread 2
Post by: zen-in on 03/14/2015 01:24 am
....
Occam's razor states that among competing hypotheses that predict equally well, the one with the fewest assumptions should be selected.    The vacuum tests do not show the underdamped response that was seen earlier.   Something was done to mitigate the thrust error caused by the amplifier current generating a torque against the damping magnets.   The picture below shows a large attenuator on what looks like the output of the amp.   This was used possibly to minimize the affects of high SWR.   A high SWR results in the amplifier drawing excessive current.   Earlier posts describe this problem and that the amp had to be returned for repairs.    Something was done to mitigate the thrust error from amplifier current in the vacuum tests.   This attenuator may have been installed for that reason.   With 2 different step responses we are left with the choice of:
1)  Elaborating on the theory of this device further to explain why the system response changes.
2)  Accepting that 2 different experiments have produced results that contradict each other.

Of these two choices Occam requires we choose the one with the fewest new assumptions, which is #2.....
Occam's razor states that among competing hypotheses that predict equally well, the one with the fewest assumptions should be selected. 

1) As per above, the experiments are not exactly the same: besides testing in a relative vacuum (6.6*10^(-9) standard atmosphere) vs. one standard atmosphere, there are a number of components that have changed (for example, besides " The 30W Mini-Circuit Class-A RF amp was used for the in-air series reported in the 2014 JPC paper, whereas a 100W EMPower Class-A/B RF amplifier was used in the vacuum tests to date" you yourself point out the following change: " Something was done to mitigate the thrust error caused by the amplifier current generating a torque against the damping magnets. 

2) As used in classical logic: "a contradiction consists of a logical incompatibility between two or more propositions. It occurs when the propositions, taken together, yield two conclusions which form the logical,

3) When applying Occam's razor, the focus should be on simplicity overall,



If the Eagleworks device was actually producing a thrust the STP and vacuum test would have been very similar.   The reason for doing a vacuum test was to show the device worked the same in a vacuum and that the results were not because of air currents, etc.   This concept of achieving consistent results is common to many activities.   If for example you had two FFT programs and they gave you different results for the same input where would you be?   So this is an important concept for theoreticians.    People who get their hands dirty doing experiments, writing software, or designing hardware have the same goal of achieving consistent results.   Without it you have no benchmark to measure success.

Using a different RF amp and adding an attenuator would not change the basic nature of the test.   They are applying an RF pulse of predetermined frequency and power level to the cavity.    What determines the step response of the system is the TP, what it is loaded with, and what kind of damping it has.   

The Aug. 2014 AIAA paper presented several thrust waveforms as evidence of an anomalous thrust.    Taking the Eagleworks team at their word we have examined these thrust waveforms and noted they have an overshoot, ringing, and an undershoot.   This is entirely a function of the TP, its damping, etc.   The same response is seen with the capacitor calibrator and a similar response is seen with the Cannae drive.   Earlier an error thrust waveform was also shown.   This error waveform was acquired when the dummy load was used.   Further tests subtracted this error waveform.   It also had the same overshoot, ringing, and undershoot (underdamped) as all the other waveforms.

The thrust waveforms from the vacuum test are not underdamped like the earlier thrust waveforms.   However the calibration waveform in the same picture does have an underdamped response.   It appears to me that by mitigating the error thrust in some way the underdamped characteristics have been removed from the thrust waveform.   From this I conclude the earlier thrust waveforms were actually error waveforms that were not corrected for because the amplifier current was higher during the test runs than it was when the correction factor was acquired (the dummy load test).    This is a logical inference which I understand some people may not want to accept.   But the fact remains the two set of experiments have produced inconsistent results.

The vacuum tests resulted in very different waveforms.   Thrust continues the after RF is off.   This is very unusual.  How is momentum stored in the cavity?   The earlier tests didn't show this.

A problem with this experiment is the extremely small displacements that indicate a thrust.   A displacement of 4 micrometers has the TP beam move through just 1.7 arcSec. of rotation.   If a laser beam was reflected off the LDS moving mirror and someone was 1 km away they would see the reflected dot move just a few mm.   The LDS is just as sensitive to angular changes of the mirror.   An experiment of this type requires repeatable, consistent results with a signal level far above what is currently seen to provide proof of this proposed theory of its operation. 
Title: Re: EM Drive Developments - related to space flight applications - Thread 2
Post by: DIYFAN on 03/14/2015 05:24 am
An experiment of this type requires repeatable, consistent results with a signal level far above what is currently seen to provide proof of this proposed theory of its operation.

While I generally agree with you, I've been around long enough to know that unless you decide to try something for yourself, it is difficult to gain a true understanding.  Perceiving what is real in a world awash with conjecture, theories, political agendas, financial agendas, greed, and outright lies, is quite a challenge (not accusing anyone on this forum of such things, just commenting in a generalized manner). 

So how does one gain a sure knowledge?  Attempt a replication.  Measure it for yourself.  Convince yourself that it works or doesn't work.  The materials are not cheap, but they are also not super expensive, and are very attainable.  Don't have all of the expertise needed?  Find a friend or co-worker who might.  Don't have the time?  Cut down on some other optional activities.  Don't have the motivation?  Can't really help there, but there are some great self help books out there. 

If we can get say 10 people attempting replications of the general concept and selflessly willing to share their results, imagine how much data can be aggregated and shared among the group.  Imagine where we could take this if the effect is real.  And if the effect cannot be shown and cannot be reproduced after diligent efforts, then that is also a good contribution to the store of human knowledge as well.
Title: Re: EM Drive Developments - related to space flight applications - Thread 2
Post by: frobnicat on 03/14/2015 09:48 am
While replication attempts by DIYers might add very valuable data and insight into the measured effects, there is also a lot that has been made and still can be made by community around models and simulations, thanks to the data published from Eagleworks and some answers later shared by Paul March.

It didn't took building and launching a specifically instrumented clone of pioneer probe to resolve (classically) the pioneer anomaly, it took "only" a reasonably accurate model from known parameters and integrating some relevant classical phenomenons in a simulation to get a convincing qualitative and quantitative explanation of the anomaly. As always, a number of people will find those results not convincing enough and will cling to exotic extensions of physics indefinitely... a classical explanation is less fun obviously, but reality isn't necessarily fun or exciting.

For recall (http://forum.nasaspaceflight.com/index.php?topic=36313.msg1345829#msg1345829) we have a thermal gradient across the thickness of PCB big end cap that has a dynamic that smoothly follows microwave power pumped into the system, see attached picture. The situation is close enough to that of a bimetallic strip, also here it is the difference in temperature across the thickness rather than the difference of thermal expansion that would drive thermal constraints. Also this is not a strip but a round membrane. Anyway, thermal gradient implies stress implies deformations implies more or less proportional CoMPosition(t) shifts. Exact shape and magnitude of which remains to be assessed from thermo-mechanical model of frustum, including PCB cap and supporting rim and copper cone, taking into account gradients across thickness.

Since the apparatus is tilted enough that the tilt plays a role in the stability of pendulum's arm at rest equilibrium, we also have LDS_reading(t)=cst1*Thrust(t)+cst2*CoMPosition(t) where cst1 and cst2 are in the same ballpark. Meaning that a CoMPosition(t) signal alone can explain a step for LDS_reading(t) even while Thrust(t) is 0 and remains 0.

Am I seriously saying those two ideas taken together can account for the major part of what is interpreted as thrust pulses ? Yes, this is seriously close enough to a viable explanation that it deserve further inquiry. But I'm reaching my limits in terms of simulation tools...

Title: Re: EM Drive Developments - related to space flight applications - Thread 2
Post by: Star-Drive on 03/14/2015 02:23 pm
....
Occam's razor states that among competing hypotheses that predict equally well, the one with the fewest assumptions should be selected.    The vacuum tests do not show the underdamped response that was seen earlier.   Something was done to mitigate the thrust error caused by the amplifier current generating a torque against the damping magnets.   The picture below shows a large attenuator on what looks like the output of the amp.   This was used possibly to minimize the affects of high SWR.   A high SWR results in the amplifier drawing excessive current.   Earlier posts describe this problem and that the amp had to be returned for repairs.    Something was done to mitigate the thrust error from amplifier current in the vacuum tests.   This attenuator may have been installed for that reason.   With 2 different step responses we are left with the choice of:
1)  Elaborating on the theory of this device further to explain why the system response changes.
2)  Accepting that 2 different experiments have produced results that contradict each other.

Of these two choices Occam requires we choose the one with the fewest new assumptions, which is #2.....
Occam's razor states that among competing hypotheses that predict equally well, the one with the fewest assumptions should be selected. 

1) As per above, the experiments are not exactly the same: besides testing in a relative vacuum (6.6*10^(-9) standard atmosphere) vs. one standard atmosphere, there are a number of components that have changed (for example, besides " The 30W Mini-Circuit Class-A RF amp was used for the in-air series reported in the 2014 JPC paper, whereas a 100W EMPower Class-A/B RF amplifier was used in the vacuum tests to date" you yourself point out the following change: " Something was done to mitigate the thrust error caused by the amplifier current generating a torque against the damping magnets. 

2) As used in classical logic: "a contradiction consists of a logical incompatibility between two or more propositions. It occurs when the propositions, taken together, yield two conclusions which form the logical,

3) When applying Occam's razor, the focus should be on simplicity overall,



If the Eagleworks device was actually producing a thrust the STP and vacuum test would have been very similar.   The reason for doing a vacuum test was to show the device worked the same in a vacuum and that the results were not because of air currents, etc.   This concept of achieving consistent results is common to many activities.   If for example you had two FFT programs and they gave you different results for the same input where would you be?   So this is an important concept for theoreticians.    People who get their hands dirty doing experiments, writing software, or designing hardware have the same goal of achieving consistent results.   Without it you have no benchmark to measure success.

Using a different RF amp and adding an attenuator would not change the basic nature of the test.   They are applying an RF pulse of predetermined frequency and power level to the cavity.    What determines the step response of the system is the TP, what it is loaded with, and what kind of damping it has.   

The Aug. 2014 AIAA paper presented several thrust waveforms as evidence of an anomalous thrust.    Taking the Eagleworks team at their word we have examined these thrust waveforms and noted they have an overshoot, ringing, and an undershoot.   This is entirely a function of the TP, its damping, etc.   The same response is seen with the capacitor calibrator and a similar response is seen with the Cannae drive.   Earlier an error thrust waveform was also shown.   This error waveform was acquired when the dummy load was used.   Further tests subtracted this error waveform.   It also had the same overshoot, ringing, and undershoot (underdamped) as all the other waveforms.

The thrust waveforms from the vacuum test are not underdamped like the earlier thrust waveforms.   However the calibration waveform in the same picture does have an underdamped response.   It appears to me that by mitigating the error thrust in some way the underdamped characteristics have been removed from the thrust waveform.   From this I conclude the earlier thrust waveforms were actually error waveforms that were not corrected for because the amplifier current was higher during the test runs than it was when the correction factor was acquired (the dummy load test).    This is a logical inference which I understand some people may not want to accept.   But the fact remains the two set of experiments have produced inconsistent results.

The vacuum tests resulted in very different waveforms.   Thrust continues the after RF is off.   This is very unusual.  How is momentum stored in the cavity?   The earlier tests didn't show this.

A problem with this experiment is the extremely small displacements that indicate a thrust.   A displacement of 4 micrometers has the TP beam move through just 1.7 arcSec. of rotation.   If a laser beam was reflected off the LDS moving mirror and someone was 1 km away they would see the reflected dot move just a few mm.   The LDS is just as sensitive to angular changes of the mirror.   An experiment of this type requires repeatable, consistent results with a signal level far above what is currently seen to provide proof of this proposed theory of its operation. 

Zen-in:

"The thrust waveforms from the vacuum test are not underdamped like the earlier thrust waveforms.   However the calibration waveform in the same picture does have an underdamped response.   It appears to me that by mitigating the error thrust in some way the underdamped characteristics have been removed from the thrust waveform."

The various dummy load tests have never been performed in a vacuum, only in-air, so the dynamic response of the torque pendulum (TP) for these dummy load tests will always look like the in-air TP examples.  I suppose we could run the dummy load test in vacuum as well just for consistency, but there are so many other things to do in this business, that it didn't seem a wise use of time and resources.

BTW, our COMSOL thermal analysis of the copper frustum operating in-air indicated that there was a thermally induced air circulation primarily generated by the large-OD face of the frustum due to its much higher change in temp than the small OD end-cap.  This air circulation over the vertical wall of the large OD of the frustum creates a low pressure zone over the large OD end-cap that pulls the frustum towards the large-OD side of the frustum which subtracts from the frustum's normal thrust generation force vector.  We think that this is why for a given input power and the TM212 resonant mode that the in-air thrust measurements are ~2.5X smaller than the in-vacuum cases.   

Lastly find attach a slide with the results of this week's test that demonstrate that the copper frustum still generates a thrust signature when it is not in the stainless steel vacuum chamber walls.  And as you will note the forward thrust signature is similar in magnitude for the same 50W case in-air in the vacuum chamber, so I think we can start to put to bed the idea that standard E&M evanescent waves interactions with the vacuum chamber walls are the cause of these thrust signatures.

Best, Paul M.
Title: Re: EM Drive Developments - related to space flight applications - Thread 2
Post by: Star-Drive on 03/14/2015 02:39 pm
Dr. Rodal:

Looking over some of our test results from last spring and summer just now, I realized that I had forgotten to provide this forum a few test runs at some of the other resonant modes we've looked at that used different dielectric discs and locations including polycarbonate which I hadn't mentioned before.  These examples are attached and you will note that these other copper frustum resonant modes (TM010 & TE012) have very different thermal torque pendulum responses than the TM212 (your TM221 I think) cases we've discussed of late.  That fact might be of importance when discussing whether these thrust signals are real or just artifacts...

PS: The magnitude of the torque pendulum's overshoot is directly related to what version of the magnetic damper that was used for the particular test in question and how thick the copper damper blade was at the time.  At the moment we are using an arrangement that is slightly over-damped in an attempt to smooth out some of the pesky low frequency seismic noise in the system.

DPS:  The magnitude of the RF amp's dc current induced thrust signal offset is directly related to how much leakage current is going through the torque pendulum's steel torsion bearings.  As I became aware of this problem over the last year, I've found various ground wiring tricks to mitigate this dc current induced negative going offset, but I've never found a way to get rid of all of it.

Best, Paul M   
Title: Re: EM Drive Developments - related to space flight applications - Thread 2
Post by: Rodal on 03/14/2015 03:40 pm
... have very different thermal torque pendulum responses than the TM212 (your TM221 I think) cases we've discussed of late.  That fact might be of importance when discussing whether these thrust signals are real or just artifacts......
Thanks Paul.  Great reply with quantitative, factual, objective, helpful information.   :)


Clarification: concerning your designated TM212 mode which I interpret as TM222 we completely agree on what the mode looks like (this is the most important thing, besides any convention for designation), and we agree that it is transverse magnetic, and we agree that the azimuthal mode number should be designated m=2 and we agree that the longitudinal mode number should be designated p=2.  We only differ on how "n" (on TMm n p) should be designated: your designation is n=1 while my designation is n=2.
Title: Re: EM Drive Developments - related to space flight applications - Thread 2
Post by: Mulletron on 03/14/2015 05:36 pm
While researching, I found @Rotosequence found this clue first:

http://forum.nasaspaceflight.com/index.php?topic=36313.msg1321774#msg1321774

Gotta give credit where credit is due.
Title: Re: EM Drive Developments - related to space flight applications - Thread 2
Post by: Notsosureofit on 03/14/2015 09:53 pm
FYI

Still bumbling along.  This is a shot at an integral form for a generalized cylindrical cavity where c and R are functions of the long axis.  Once you have del f, the thrust/photon and thrust are straightforward.

For:

A = (p/2*L)^2 and B = (X/2*pi)^2 where X = X[sub m,n](ie. TE) or X'[sub m,n](ie. TM)

and c and R are functions of z such that c = c(z) and R = R(z) then

(del f)*f[sub m,n,p]) = B*Int(0,L)[(c/R^2)*(dc/dz)-(c^2/R^3)*(dR/dz)]dz
                      + A*Int(0,L)[c*(dc/dz)]dz

Anyway, give it a look.  I'll be trying cases as time permits.

Thanks  (and watch for typos)

Title: Re: EM Drive Developments - related to space flight applications - Thread 2
Post by: frobnicat on 03/15/2015 01:00 am
Rodal I need your expertise ;D

On the magnetic field maps given by Eagleworks simulations and from your exact solutions, is it correct to interpret that the heating power per unit area of frustum inner wall will depend locally as the square of the magnetic field magnitude ? Is the scalar intensity value map below enough to get the heating rate at each point or is the vector map required ? Will we have a smooth transition (in heating rate per unit area) between the PCB plate and cone in spite of the angle they make at the rim ?

(http://forum.nasaspaceflight.com/index.php?action=dlattach;topic=36313.0;attach=635195;image)

Trying to get a more accurate view on displacements : the PCB plate can buckle by thermal expansion against a rigid rim, but the rim is itself heating and expanding, will try an axisymmetric simulation taking both into account + thermal gradient (across thickness) induced "warp".
Title: Re: EM Drive Developments - related to space flight applications - Thread 2
Post by: Mulletron on 03/15/2015 12:31 pm
Okay I'm going to use E field drive and sense probes (instead of loops) within the cavity placed at 1.36" from the large plate.

It seems to me that the connector should be at the small end. I'm kind of torn on this. It probably doesn't matter. Shawyer/Eagleworks have the connectors at the large end, so I will too.

There is a nifty calculator here to make it easy:
http://www.turnpoint.net/wireless/cantennahowto.html
http://www.lincomatic.com/wireless/homebrewant.html (more info)

Also some backgrounders for those who are interested:
http://www.radartutorial.eu/03.linetheory/tl11.en.html
http://www.maritime.org/doc/neets/mod11.pdf

This is also in agreement with the 15% of 9" cavity height from the large end which came out to 1.35", brought up here: http://forum.nasaspaceflight.com/index.php?topic=36313.msg1331854#msg1331854

The length of the probe will be 1.21" or 31mm, based on a 1/4 wavelength of the desired frequency 2450mhz.

The type of connector will be an N-type female in the top picture, as opposed the one in the bottom picture. That way I only have to drill one hole instead of five per connector. The size of the center hole required for both is within 1/16" so I'm not going to bother with the 4 hole bulkhead connectors I got. I wanted to see them both before I decided on which one to use.

The connectors were purchased from here, I guess they sold out because they now say unavailable:
http://www.amazon.com/gp/product/B00OOF54QW/ref=oh_aui_detailpage_o00_s00?ie=UTF8&psc=1
http://www.amazon.com/gp/product/B006Z95L8Q/ref=oh_aui_detailpage_o02_s00?ie=UTF8&psc=1

I'm sure there are more on Amazon, but if not, I've purchased from these guys before and had a good experience:
http://www.fab-corp.com/home.php?cat=274



Title: Re: EM Drive Developments - related to space flight applications - Thread 2
Post by: Star-Drive on 03/15/2015 01:56 pm
Okay I'm going to use E field drive and sense probes (instead of loops) within the cavity placed at 1.36" from the large plate.

It seems to me that the connector should be at the small end. I'm kind of torn on this. It probably doesn't matter. Shawyer/Eagleworks have the connectors at the large end, so I will too.

There is a nifty calculator here to make it easy:
http://www.turnpoint.net/wireless/cantennahowto.html
http://www.lincomatic.com/wireless/homebrewant.html (more info)

Also some backgrounders for those who are interested:
http://www.radartutorial.eu/03.linetheory/tl11.en.html
http://www.maritime.org/doc/neets/mod11.pdf

This is also in agreement with the 15% of 9" cavity height from the large end which came out to 1.35", brought up here: http://forum.nasaspaceflight.com/index.php?topic=36313.msg1331854#msg1331854

The length of the probe will be 1.21" or 31mm, based on a 1/4 wavelength of the desired frequency 2450mhz.

The type of connector will be an N-type female in the top picture, as opposed the one in the bottom picture. That way I only have to drill one hole instead of five per connector. The size of the center hole required for both is within 1/16" so I'm not going to bother with the 4 hole bulkhead connectors I got. I wanted to see them both before I decided on which one to use.

The connectors were purchased from here, I guess they sold out because they now say unavailable:
http://www.amazon.com/gp/product/B00OOF54QW/ref=oh_aui_detailpage_o00_s00?ie=UTF8&psc=1
http://www.amazon.com/gp/product/B006Z95L8Q/ref=oh_aui_detailpage_o02_s00?ie=UTF8&psc=1

I'm sure there are more on Amazon, but if not, I've purchased from these guys before and had a good experience:
http://www.fab-corp.com/home.php?cat=274


Mulletron:

Dependent on the output power of your RF system, you might also consider using TNC connectors for your frustum RF power injection port.  SMA connectors are good to about 100W, TNC are good to 400W and Type-N connectors can go up to 1,000W dependent on its operating frequency.  As to their preferred location, that all depends on what frustum resonant mode you are interested in exciting and whether you want to drive it via an E-field whip or magnetic loop.  If you look at the Frank Davies frustum mode presentation already on this forum, you will observe that each resonant mode and frequency has strong E-field and B-fields locations and weaker locations and you want to drive it at one of stronger locations.

BTW, I've tried E-field whip antennas as the main RF input and I found that due to the high quality factors in these resonant system, you can literally burn off the tip of the whip antenna from the high E-fields created as the attached picture of the Cannae E-field injection antenna will attest when I drove it with ~80W at 932 MHz.  That is why I now use magnetic loops or side wall slots being driven by a waveguide feed.  I've also gone to B-field loops because I can rotate them and thus directly control the coupling by rotating the loop through 180 degrees. 

With an E-field probe you have to start long and trim the whip length to get a minimum VSWR.  And if you try a different lower frequency, you then have to go with a new longer whip antenna and start trimming again.  Loops can be just as fussy in their required OD, but you only end up with five or six loops of various diameters that can be used for most of the frustum resonances of interest. 

Best, Paul M.
Title: Re: EM Drive Developments - related to space flight applications - Thread 2
Post by: Rodal on 03/15/2015 03:13 pm
Rodal I need your expertise ;D

On the magnetic field maps given by Eagleworks simulations and from your exact solutions, is it correct to interpret that the heating power per unit area of frustum inner wall will depend locally as the square of the magnetic field magnitude ? Is the scalar intensity value map below enough to get the heating rate at each point or is the vector map required ? Will we have a smooth transition (in heating rate per unit area) between the PCB plate and cone in spite of the angle they make at the rim ?

(http://forum.nasaspaceflight.com/index.php?action=dlattach;topic=36313.0;attach=635195;image)

Trying to get a more accurate view on displacements : the PCB plate can buckle by thermal expansion against a rigid rim, but the rim is itself heating and expanding, will try an axisymmetric simulation taking both into account + thermal gradient (across thickness) induced "warp".

1) Paul March kindly supplied both the COMSOL Finite Element dissipated heat (W/m^2) per unit area (the finite element model takes into account geometrical changes like the "angle" you are concerned with) as well as the NASA Eagleworks thermal IR camera measurements:

(http://forum.nasaspaceflight.com/index.php?action=dlattach;topic=36313.0;attach=635194;image)

so you don't need to rely solely on the magnetic field

2) Assuming an adiabatic process, the volumetric power dissipation Pdissipated in the material due to the applied magnetic field should be:

 Pdissipated = Pi*f*Chi"*(B^2)/(muo*(mur^2)) =  Pi*f*Chi"*(H^2)*muo

f=frequency (Hz)
Chi"= out-of-phase susceptibility (complex component of the susceptibility) of the material
mu = magnetic permeability = muo*mur
mur = relative permeability of the material
B = magnitude of induction = mu* H
H = magnitude of magnetic field strength

So yes, assuming an adiabatic process, the volumetric power dissipation goes like the square of the magnitude of the magnetic field.

3) The magnitude of the total vector resultant matters (one needs to take into account the resultant of the vector components, which has been done in the plot supplied by Paul March).  The direction doesn't matter.

4) Questions like "Will we have a smooth transition (in heating rate per unit area) between the PCB plate and cone in spite of the angle they make at the rim ?" and other statements in this and the other posts I can't follow without seeing the unstated equations, assumptions and values, you use for your models.  For example, I don't understand why you are concerned by "a smooth transition" due to the geometrical angle when we have discontinuous material properties between the epoxy and the copper) :)
Title: Re: EM Drive Developments - related to space flight applications - Thread 2
Post by: Mulletron on 03/15/2015 03:19 pm
I gotta say it is an honor receiving Q-thruster advice from the Paul March. I plan using no more than 2 watts. I like N-type connectors though, because they're good for low and high power. So if I ever get a hankering to try high power later (like tapping off the waveguide of a microwave like this guy http://gbppr.dyndns.org/mil/emp3/index.html), the holes for the larger connector will already be there. I'd prefer to keep it safe and just use low power.

The middle pic in this post was extremely helpful and thanks!
http://forum.nasaspaceflight.com/index.php?topic=36313.msg1327467#msg1327467

A tip I wanted to make sure I pass along for any further DIYers out there. Thin copper and drill bits don't mix. A drill bit will ruin your day if you don't step it up slowly. I used the reamers in the pics below for most of the work.

I'm still waiting on word back from the guy on Ebay to list his "microwave resonant cavity" for research. He makes these things usually, but I was lucky enough to get him to build a custom job and he did a good job:
http://www.ebay.com/sch/1952rickey/m.html?item=121586024858&rt=nc&_trksid=p2047675.l2562

Title: Re: EM Drive Developments - related to space flight applications - Thread 2
Post by: frobnicat on 03/15/2015 05:42 pm
Rodal I need your expertise ;D

...

1) Paul March kindly supplied both the COMSOL Finite Element dissipated heat (W/m^2) per unit area (the finite element model takes into account geometrical changes like the "angle" you are concerned with) as well as the NASA Eagleworks thermal IR camera measurements:
...
so you don't need to rely solely on the magnetic field

Yes, this is a very informative instantaneous map of temperature, but I wanted to reconstruct the dynamic (transients) and also have the gradients through thickness. Preliminary simulations I am doing show the gradients through thickness are negligible for the cone part since copper is such good thermal conductor but through the PCB plate at any time there easily can be more than 1°C difference between the copper side and the other side. This will induce stress in flexion (known as "bow" or "warp" in PCB literature) and tend to make the PCB flex toward the copper (inward). Magnitude of that effect relative to buckling (in plane PCB expansion relative to rim support) I'm still struggling to assert, especially if the PCB as even a very modest inward warp to start with (which is a concern for single sided boards) like a few 10s of microns at belly.
Why I wanted to start again from the primary heat flux dissipated within the skin depth copper side, even if it is with far lower numerical tools than provided by Comsol results. I wish Eagleworks had added a deformation study from the very accurate thermal maps.

Quote
2) Assuming an adiabatic process, the volumetric power dissipation Pdissipated in the material due to the applied magnetic field should be:

 Pdissipated = Pi*f*Chi"*(B^2)/(mu*mur) =  Pi*f*Chi"*(H^2)*muo

f=frequency (Hz)
Chi"= out-of-phase susceptibility (complex component of the susceptibility) of the material
mur = relative permeability of the material
B = magnitude of induction = permeability * H
H = magnitude of magnetic field strength

So yes, assuming an adiabatic process, the volumetric power dissipation goes like the square of the magnitude of the magnetic field.

3) The magnitude of the total vector resultant matters (one needs to take into account the resultant of the vector components, which has been done in the plot supplied by Paul March).  The direction doesn't matter.

Nice, so we do have access to the primary heat flux at each point. Thanks a lot.

Quote
4) Questions like "Will we have a smooth transition (in heating rate per unit area) between the PCB plate and cone in spite of the angle they make at the rim ?" and other statements in this and the other posts I can't follow without seeing the unstated equations, assumptions and values, you use for your models.  For example, I don't understand why you are concerned by "a smooth transition" due to the geometrical angle when we have discontinuous material properties between the epoxy and the copper) :)

Yes there is certainly a huge discontinuity in thermal characteristics of the walls here, I just wanted to know the discontinuity in terms of heat flux due to vector orientation relative to normal of surface elements. Since the maps provided by Paul March take the vector resultant into account the answer to 3) makes 4) not relevant. For my model I wasn't specifically concerned by the transition in power flux being smooth or not, just trying to interpret correctly the plots. At the moment I'm running an axisymmetric model for thermal conduction (in materials) and thermal radiation (in vacuum) comprising the big end PCB plate + portion of cone, but have not yet tools for deformations induced by the thermal and coefficient of expansion gradients.

For the rest I don't see how I could make the statements more clear (more synthetic yes probably), I posted a number of equations showing how a Z tilted pendulum would quantitatively react to a displacement of centre of mass, with shifts toward small end of frustum giving higher LDS readings and not the opposite. The tilt is not known precisely, but it could be reconstructed, if it wasn't for the apparent stiffness/vertical scale issue. I've shown with simple enough equations that there is a very poorly characterised core parameter in the system, by more than an order of magnitude. With significant thermally induced centre of mass displacements (with time constants compatible with those of the rise and fall of the signal) this poorly explained core parameter is not just a serious concern but becomes a major issue. That I explain that well enough or not to be convincing don't change the fact. I know this latest statement is not convincing or "helpful" by itself, just a feeling. But the equations are here. There is just one lacking (the exact ratio LDS_shift/CoMposition_shift) that needs the issue of vertical scale to be resolved. That there is an issue with the apparent stiffness/vertical scale is for me beyond doubt.
Title: Re: EM Drive Developments - related to space flight applications - Thread 2
Post by: Rodal on 03/15/2015 06:37 pm
....

For the rest I don't see how I could make the statements more clear (more synthetic yes probably)...
Speaking personally, it would be clearer if you would state the equations that you are using for your posted calculations, defining the variables in the equations, and the values of the parameters you used to arrive at a solution. (e.g. what were the material property inputs you used and the equations you used for the personal code thermal calculations?)
Title: Re: EM Drive Developments - related to space flight applications - Thread 2
Post by: Mulletron on 03/15/2015 06:53 pm
Ok so the connectors are cut and soldered in. I'm going to leave it this way unless an issue comes up that prompts change. I'm not thrilled about having that nut inside the frustum. I saw pics from Eagleworks where they have the nut inside their frustum here:
http://forum.nasaspaceflight.com/index.php?topic=36313.msg1331854#msg1331854

So maybe it is fine. I like to nuke things.

If it becomes a problem, I'll solder in place the outside nut (which I'm using as a spacer to compensate for the thin frustum) in place and use it to hold the connector in place, while at the same time ditching the inside nut, allowing a smoother presentation on the inside of the cavity.

The whip antenna was salvaged from the 12ga ground wire from a 4 wire Romex cable I had laying around from an old project. The 12ga wire is a perfect fit in the solder cup. Like a glove. I straightened out the wire in a vise, which had a groove in one side of the clamps. A couple rotations and the wires were straight.

I don't want to upload hundreds of pics so most of the build log will be here:
https://drive.google.com/folderview?id=0B4PCfHCM1KYoTXhSUTd5ZDN2WnM&usp=sharing

I'm taking lots of pics to serve as pointers for other DIYers to learn from my mistakes and successes.
Title: Re: EM Drive Developments - related to space flight applications - Thread 2
Post by: Rodal on 03/15/2015 07:01 pm
Yes, this is a very informative instantaneous map of temperature, but I wanted to reconstruct the dynamic (transients) and also have the gradients through thickness.....
Paul March gave us several plots, including:

1) Calculated dissipated power W/m^2 vs. location
2) Measured temperature vs location
3) Calculated Magnetic field vs location

The fact is that Paul March gave us the calculated dissipated power W/m^2 vs. location.  In your prior post you referred to Paul's data for the calculated magnetic field, and in this post you acknowledge the measured temperature.

....
Why I wanted to start again from the primary heat flux dissipated within the skin depth copper side, even if it is with far lower numerical tools than provided by Comsol results. I wish Eagleworks had added a deformation study from the very accurate thermal maps.
EDIT (hat tip to @frobnicat): To be precise, Eagleworks did not provide COMSOL calculations (to my knowledge) of a thermal map (temperature vs. location).  What Eagleworks provided are (COMSOL Finite Element) calculations for the dissipated power per unit area (in units of power per surface area: Watt/m^2) surface losses throughout the whole 3-Dimensional surface (both the large and small diameter ends as well as the lateral round conical surface).  And it is the volumetric power dissipation that goes like the square of the magnitude of the magnetic field, which was your question.  So, Paul March had already given you the elements to answer your question: both the magnetic field and the power dissipation density.

The thermal map is a result of IR thermal measurements.

(http://forum.nasaspaceflight.com/index.php?action=dlattach;topic=36313.0;attach=635194;image)
Title: Re: EM Drive Developments - related to space flight applications - Thread 2
Post by: Rodal on 03/15/2015 07:24 pm
... I'm still struggling to assert, especially if the PCB as even a very modest inward warp to start with (which is a concern for single sided boards) like a few 10s of microns at belly.
...
Since the bending deformation of the end plate (due to a thermal gradient through its thickness, referred to as "oil canning" in the picture below) is inwards, and you had arrived at the conclusion that such an inward deformation produces a force that is in the opposite direction to the measured force , why are you so interested in performing this calculation?

If your purpose is to show that the actual EM Drive force (due to the Quantum Vacuum or whatever else may make it work in space?) is larger than the measured force, then I  lost track of the discussion somewhere because I recall you stating that you wanted to show the opposite: that the measured force was an artifact.

?

(http://forum.nasaspaceflight.com/index.php?action=dlattach;topic=36313.0;attach=776495;image)
Title: Re: EM Drive Developments - related to space flight applications - Thread 2
Post by: frobnicat on 03/15/2015 08:36 pm
....

For the rest I don't see how I could make the statements more clear (more synthetic yes probably)...
Speaking personally, it would be clearer if you would state the equations that you are using for your posted calculations, defining the variables in the equations, and the values of the parameters you used to arrive at a solution. (e.g. what were the material property inputs you used and the equations you used for the personal code thermal calculations?)

By "the rest" I was speaking of the previous post where I defined axis conventions, tilt, displacements... dispersed in a few post, so a synthesis might be needed, agree on that.

For the later results (thermal conduction), I'm not expecting anyone give too much credit in the accuracy of a personal simulation code, I did check for stability and consistency at equilibrium (it's ok) but this was just a basis of discussion, not presented as definitive proof. I don't have know how using more serious simulation frameworks.  The equations are those of thermal transfers proportional to temperature differences and conductions, and Stefan-Boltzmann for radiation (assuming a bath at 20°). For information those are the values for the materials (Tp = Through plane, Ip = In plane) :

COPPER
density          8960 kg/m^3
specHeat          385 J/kg/K
conductionTp      385 W/m/K
conductionIp      385 W/m/K
expansionTp   16.6e-6 m/m/K
expansionIp   16.6e-6 m/m/K
emissivity       0.05 coef<1

FR4
density          1850 kg/m^3
specHeat          600 J/kg/K
conductionTp      0.3 W/m/K
conductionIp     0.85 W/m/K
expansionTp   70.0e-6 m/m/K
expansionIp   13.0e-6 m/m/K
emissivity        0.9 coef<1

Title: Re: EM Drive Developments - related to space flight applications - Thread 2
Post by: frobnicat on 03/15/2015 08:53 pm
The fact is that Paul March gave us the calculated dissipated power W/m^2 vs. location.  In your prior post you referred to Paul's data for the calculated magnetic field, and in this post you acknowledge the measured temperature.

....
Why I wanted to start again from the primary heat flux dissipated within the skin depth copper side, even if it is with far lower numerical tools than provided by Comsol results. I wish Eagleworks had added a deformation study from the very accurate thermal maps.
To be precise, Eagleworks did not provide COMSOL calculations (to my knowledge) of a thermal map (temperature vs. location).  What Eagleworks provided are (COMSOL Finite Element) calculations for the dissipated power per unit area (in units of power per surface area: Watt/m^2) surface losses throughout the whole 3-Dimensional surface (both the large and small diameter ends as well as the lateral round conical surface).  And it is the volumetric power dissipation that goes like the square of the magnitude of the magnetic field, which was your question.  So, Paul March had already given you the elements to answer your question: both the magnetic field and the power dissipation density.

The thermal map is a result of IR thermal measurements.


Yes, this is exactly what I was looking for (dissipated power by unit area at each location). The document Thermal Analysis 1.pdf (http://forum.nasaspaceflight.com/index.php?action=dlattach;topic=36313.0;attach=634627) (from this post of Star-Drive (http://forum.nasaspaceflight.com/index.php?topic=36313.msg1326997#msg1326997)) do provide COMSOL calculations of a thermal map (temperature vs. location) and I was referring to those about "the very accurate thermal maps".

I share your substantiated opinion that Eagleworks team should definitely substitute the PCB end caps by a plain copper plate, at least for a test or two and see how the results change or not (all other conditions being equal as most as possible).
Title: Re: EM Drive Developments - related to space flight applications - Thread 2
Post by: ThinkerX on 03/15/2015 09:33 pm
Ok, despite my best efforts (which, admittedly are rather feeble), I can no longer tell what 'frobnicat' is attempting to accomplish.  Is he arguing that all of the 'thrust' produced by this device is some sort of thermal artefact?   From what I can tell, the goalposts seem to have moved at least once.

Executive summary time, please.
Title: Re: EM Drive Developments - related to space flight applications - Thread 2
Post by: frobnicat on 03/15/2015 09:35 pm
... I'm still struggling to assert, especially if the PCB as even a very modest inward warp to start with (which is a concern for single sided boards) like a few 10s of microns at belly.
...
Since the bending deformation of the end plate (due to a thermal gradient through its thickness, referred to as "oil canning" in the picture below) is inwards, and you had arrived at the conclusion that such an inward deformation produces a force that is in the opposite direction to the measured force , why are you so interested in performing this calculation?

You complain that my hypothesis and variables and equations are not clearly put, but do you read me seriously ? A movement to the left of a part of mass M will make a force to the right as recoil : F(t)=-M d²CoMPosition(t)/dt². Measuring F and ComPosition on same oriented Y axis. Shift to the left (Y-) Force to the right (Y+). Is it clear enough ? Such F(t) is small, very small, even by µN standards. It depends on the second derivative of the shift (wrt time).

Now, if we have the right to go past those negligible recoil effects :
A shift in CoMPosition to the left (Y-) will change the equilibrium rest position in such a way that the LDS distance will increase (Y-). A slow inward deformation don't produces a force  (ah ah, it does, but is too small to be significant). I'm no longer talking of force. I'm talking change of rest equilibrium position (by rest equilibrium I mean : the stable position when there is no force). This effect on LDS reading is not dependent on second nor first derivative of the shift (wrt time).

This was explained here (http://forum.nasaspaceflight.com/index.php?topic=36313.msg1342331#msg1342331), what is not clear ? Now you can object that in this model I ignored the restoring torque of the flexure bearings. This stiffness will lower the relative magnitude of the effect but not 0 it. I'm waiting for a proper characterization of the pendulum before I can give such magnitude, so far, at 1µm LDS reading per 29.1µN cal. pulse + 4.5s oscillation in charts (when underdamped) the pendulum is not properly characterised, there is a contradiction in the LDS readings data in µm and other known parameters.

(http://forum.nasaspaceflight.com/index.php?action=dlattach;topic=36313.0;attach=811757;image)
(http://forum.nasaspaceflight.com/index.php?action=dlattach;topic=36313.0;attach=811960;image)
Edit : the three bottom charts are roughly to scale concerning the placement of CoMs along the arm (X axis) from data provided by Paul March. The deformations and movements are obviously greatly exaggerated for illustration purpose. The charts seems to imply that the test article CoM shift would be due to frustum copper cone part expansion, but any other significant thermal displacement would play an equivalent role (actually the lengthening of cone may play a minor contribution relative to lighter but "amplified" buckling/warping effect at the big end PCB plate).

Quote
If your purpose is to show that the actual EM Drive force (due to the Quantum Vacuum or whatever else may make it work in space?) is larger than the measured force, then I  lost track of the discussion somewhere because I recall you stating that you wanted to show the opposite: that the measured force was an artifact.

?

I wanted to show that the measured "force" (LDS reading really) might not be a force at all, but a change in equilibrium rest position, as per the diagram showing how the equilibrium rest position is different depending on test article CoM position is here or there along Y axis. No derivative or second derivative implied. CoM sits here, LDS shows a reading, CoM sits there, LDS shows another reading. CoM shifts, LDS follows. Same direction, CoM shifts to left, LDS reading rises.
Title: Re: EM Drive Developments - related to space flight applications - Thread 2
Post by: Rodal on 03/15/2015 09:36 pm
....

For the later results (thermal conduction), I'm not expecting anyone give too much credit in the accuracy of a personal simulation code,....
FR4
density          1850 kg/m^3
specHeat          600 J/kg/K
...

It is not a question of giving personal credit, but my understanding of your posting such calculations is to check whether the experimental results are an artifact or whether they are due to a real thrust that can work for outer space propulsion.  If you don't post the equations and the material properties you use, then how can the reader ascertain how to evaluate what you post?

To give one example, we see now that you used a value of 600 J/kg/K for the specific heat of FR-4 (no reference as to where this came from).  However, Rebecka Domeij B¨ackryd at LINK¨OPING University (Sweden) used a value twice as high: 1200 J/kg/K (see page 14 of http://www.diva-portal.org/smash/get/diva2:18631/FULLTEXT01.pdf )
Title: Re: EM Drive Developments - related to space flight applications - Thread 2
Post by: frobnicat on 03/15/2015 09:44 pm
....

For the later results (thermal conduction), I'm not expecting anyone give too much credit in the accuracy of a personal simulation code,....
FR4
density          1850 kg/m^3
specHeat          600 J/kg/K
...

It is not a question of giving personal credit, but my understanding of your posting such calculations is to check whether the experimental results are an artifact or whether they are due to a real thrust that can work for outer space propulsion.  If you don't post the equations and the material properties you use, then how can the reader ascertain how to evaluate what you post?

To give one example, we see now that you used a value of 600 J/kg/K for the specific heat of FR-4 (no reference as to where this came from).  However, Rebecka Domeij B¨ackryd at LINK¨OPING University (Sweden) used a value twice as high: 1200 J/kg/K (see page 14 of http://www.diva-portal.org/smash/get/diva2:18631/FULLTEXT01.pdf )

sigh
note taken
Title: Re: EM Drive Developments - related to space flight applications - Thread 2
Post by: Rodal on 03/15/2015 09:45 pm
....
You complain that my hypothesis and variables and equations are not clearly put, but do you read me seriously ?...
I just stated that I personally have a hard time following your train of thought, and that I personally would prefer to see more equations.  You could justifiably answer that it is my fault because of my lack of reading comprehension.  I don't recall me stating that it was your fault, I actually recognized that it is probably my fault because in physics and technical science I can much better understand equations than words, particularly about things like forces (I surely don't understand Roger Shawyer's discussion of forces in the EM Drive either).



Example, of things I have a problem understanding  (my fault, probably the majority of other readers can understand you perfectly well):

Quote
A movement to the left of a part of mass M will make a force to the right as recoil : F(t)=-M d²CoMPosition(t)/dt². "  OK, so the force is to the right and it is opposite to the measured force


..."Such F(t) is small, very small, even by µN standards. It depends on the second derivative of the shift (wrt time).

===> So now I understand from this last sentence that you think that the "recoil" force was actually negligible (close to zero).  I had the (incorrect) understanding that the reason why you were discussing recoil was because you thought it was IMPORTANT. (I try to only argue about things that I think are important to me and I incorrectly projected that :)  ).

Quote
Now, if we have the right to go past those negligible recoil effects :
... A slow inward deformation don't produces a force  (ah ah, it does, but is too small to be significant). I'm no longer talking of force. I'm talking change of rest equilibrium position (by rest equilibrium I mean : the stable position when there is no force)

===> So you are not talking of forces.  Sorry, I didn't understand that prior to your latest posting. :)
Title: Re: EM Drive Developments - related to space flight applications - Thread 2
Post by: frobnicat on 03/15/2015 09:51 pm
....
You complain that my hypothesis and variables and equations are not clearly put, but do you read me seriously ?...
I just stated that I personally have a hard time following your train of thought, and that I personally would prefer to see more equations.  You could justifiably answer that it is my fault because of my lack of reading comprehension.  I don't recall me stating that it was your fault, I actually recognized that it is probably my fault because in physics and technical science I can much better understand equations than words.

Sorry mister Rodal, my english is chaotic somehow and I really wish we had some telepathy of concepts at hand. Best.
Title: Re: EM Drive Developments - related to space flight applications - Thread 2
Post by: Rodal on 03/15/2015 10:28 pm
....

I wanted to show that the measured "force" (LDS reading really) might not be a force at all, but a change in equilibrium rest position, as per the diagram showing how the equilibrium rest position is different depending on test article CoM position...
Sorry again for not recalling your prior posts and needing a brief summary.

QUESTION1: have you calculated the change in Center Of Mass position for these different cases?  (yes or no ?)

 if your answer to QUESTION1 is no, stop reading.

 if your answer to QUESTION1 is yes, then,

QUESTION2: do your calculations for the change in center of mass position give a displacement that is close to the displacement vs. time measurements at NASA Eagleworks?

Title: Re: EM Drive Developments - related to space flight applications - Thread 2
Post by: frobnicat on 03/15/2015 10:31 pm
Ok, despite my best efforts (which, admittedly are rather feeble), I can no longer tell what 'frobnicat' is attempting to accomplish.  Is he arguing that all of the 'thrust' produced by this device is some sort of thermal artefact?   From what I can tell, the goalposts seem to have moved at least once.

Executive summary time, please.

First we were on recoil effects. Recoil effects depend on a mass accelerating and keeping a constant acceleration if they are to mimic thrust effects. There is not enough thermal expansion room (by far) to account for such constant acceleration that could explain a sustained "thrust" for 40s or so.

The disparity of values in the vertical scale (µm readings) from chart to chart, for identical 29.1µN calibration pulses, have made me to wonder if the whole balance was tilted, and that the variation of tilt and/or offbalance could account for that disparity. I asked to Star-Drive if the balance was tilted or not and he replied that it was, that it was necessary because a perfectly horizontal arm (perfectly vertical axis) wouldn't settle to stable rest position (before runs). Whether this tilt/offbalance does indeed explain (in whole or in part) the vertical variations (µm readings) from chart to chart is still unconfirmed.

What we know for sure is that there is a tilt and an offbalance, enough to play a role in the rest equilibrium of the arm. That means that gravity plays a role in the angular rest position and that the angular rest position (the position when there is no force and no thrust imposed to the arm) will react to changes in position of masses of the test article relative to fixation point on the arm. The orientation of this change is counter-intuitive : when looking the apparatus in front ( frustum's small end to the left, big end to the right) a shift of mass to the left will shift the rest equilibrium to a new position with the front arm leftward, thus increasing the measured distance (a rise in the charts). This is because the global centre of mass of the whole rotating assembly is behind the axis of rotation and the tilt is such that this backward part of the arm is below the front part. If all is inverted (meaning putting more weight in front, and lowering front relative to back) then the orientation would be also inverted.

Anyway, there is here a mechanism by which a slow displacement of mass from point A to point B (typically 1kg by a few µm, or 100g by a few 10s of µm) will change the readings from a distance to another distance. What would appear as a sustained "thrust" would in fact be a mass just sitting at another place when heated relative to when not heated. Is it certain that such mechanism could account for whole the signal ? No, this is not certain for me, at this point. It would take an accurate simulation of thermal deformations in function of time and a proper characterisation of the pendulum dynamic (tilt included) together to reconstruct the signal that "change of rest position effect" would give. I would say that it is at least likely messing with the thrust signal (if there is one).

To add confusion even more, we already know of another "change of rest position effect" that is not linked to thermal change of mass position (also this is not completely sure it isn't) but rather to heating of the flexure bearings either by IR at the back of electronic amplifiers (thread 1, mitigated since) and/or by return ground currents. This is the cause of the infamous "drifting baseline", effect that easily reaches in magnitude the magnitude of the signal (also with a different shape).

So yes the goalposts moved a little bit. And my leading hypothesis of thread 1 (warm air jet) and a number of other hypothesis are now nullified by Eagleworks working in vacuum. Great. This is advancing.
Title: Re: EM Drive Developments - related to space flight applications - Thread 2
Post by: ThinkerX on 03/15/2015 11:02 pm
Just to keep things straight, a list of experimental artefacts that have been fully or partially accounted for might be in order.  Otherwise, I see a fair possibility of 're-inventing the wheel' so to speak.
Title: Re: EM Drive Developments - related to space flight applications - Thread 2
Post by: frobnicat on 03/15/2015 11:48 pm
....

I wanted to show that the measured "force" (LDS reading really) might not be a force at all, but a change in equilibrium rest position, as per the diagram showing how the equilibrium rest position is different depending on test article CoM position...
Sorry again for not recalling your prior posts and needing a brief summary.

QUESTION1: have you calculated the change in Center Of Mass position for these different cases?  (yes or no ?)

 if your answer to QUESTION1 is no, stop reading.
Er, yes, no, this is in progress but would need a simulation with professional level tools I haven't knowledge of to be assessed with any certainty. Needs both buckling and thickness gradient flexing deformations, with asymmetrical thin plate modelling (single sided PCB). Amongst others, Code Aster is a free simulation framework of more than 1000000 lines code that could do the job but require quite a training to be used correctly. Right now I'm trying to do order of magnitude estimates with small personal code ~500 lines of code. Is it in the ballpark or not ? Preliminary results say maybe.

I haven't "published" yet because, you know, people want to know the details ( rightly so  :) ) and it takes time. The problem with this line of reasoning is not the magnitude but rather the shapes : with low thermal evacuation (in vacuum) the temperatures tend to rise linearly for the whole run of 40s power-on, and then tend to stay at their new acquired temperature at power-off rather than falling back to equilibrium in a symmetrical manner. It gives linear ramp up followed by slowly decay plateau rather than (more or less) quick rise to plateau followed by (more or less) quick decay back to baseline, as observed. That's for the absolute temperatures. But some temperature differences (gradients) show a temporal shape more like the expected shape. See for instance this very rough and simplified model (FR4 heat capacity at 600, will try the value of 1200 you found later, my value came from another source that looked serious, don't have the link at hand).

(http://forum.nasaspaceflight.com/index.php?action=dlattach;topic=36313.0;attach=814602;image)

A gradient across the thickness will make the PCB plate warp. How much ? I don't know : the PCB plate is attached on a rim... if it was a thin strip just floating in vacuum with a cylindrical deformation (rather than a stiffer plate with a spherical deformation) deflection at the belly = L²/(8*d) * ExpanCoef*(T2-T1) where T2 is hot side, T1 cold side, L the rest length (28cm), d the depth (1.6mm) and ExpanCoef is about 1.3e-5m/m/K (FR4 inplane, neglecting the copper for now). That is around 80µm for 1°C difference across the thickness. How much less for a real plate fixed to rim ? All I can say at this point is, we are not talking below µm thermal effects, thermal effects with the appropriate dynamic shape.

The "active part" of PCB (that can warp) of 28cm diameter should weigh around 200g. Very very roughly (order of magnitude) rest of test article is in the 2kg, 10 times more. So the displacement of CoM of test article is roughly a tenth of the displacement of CoM of PCB due to buckling/warping. To "explain" a 1µm deviation in rest position at the LDS could possibly need a bit less than 1µm change of CoM position of the test article. This last claim is unsubstantiated by equations so far, it is just an observation of the 3 bottom diagram above (http://forum.nasaspaceflight.com/index.php?topic=36313.msg1346487#msg1346487) (to scale), and will depend on the relative stiffness of flexure bearings (why I insist so much on a clarification on this point). Let's say this is in the same ball park. Conclusion (fragile) : sustained 10µm displacement of mean PCB membrane position relative to rim could account for 1µm sustained plateau above baseline on the charts.

Quote
if your answer to QUESTION1 is yes, then,

QUESTION2: do your calculations for the change in center of mass position give a displacement that is close to the displacement vs. time measurements at NASA Eagleworks?

It could.
At least it needs to be properly assessed before it can be ruled out. Also it is important to understand that the magnitude of the effect don't depend linearly on the tilt, if it wasn't for the flexure bearings restoring torque, the same deltaLDS/deltaCoM would be observed whether the tilt is 0.1° or 1° or 10°. Since we have the flexure bearings restoring torques, the magnitude will depend on tilt and rotating assembly's CoM distance from Z and bearing's stiffness. And doing the calculations one will invariably fall on a visible inconsistency of the data : the apparent stiffness in µm/µN (whatever part is due to bearings vs g tilted pendulum position restoring torque) from the cal pulses is in contradiction with the reasonably well known moment of inertia of the rotating assembly + clearly visible ~4.5s pseudoperiod on underdamped charts. (+ in contradiction with the known .007lb-in/° stiffness of bearings + in contradiction with tilt needed to reach such equivalent stiffness that would be above 10°)
 
Title: Re: EM Drive Developments - related to space flight applications - Thread 2
Post by: Giovanni DS on 03/16/2015 08:42 am
Would be possible to mount the thruster on the pendulum in a way that its main axis is aligned with force of gravity? this would put to rest this thermal expansion hypothesis or confirm it.

It could be mounted like on scales, with a passive mass on the other side.
Title: Re: EM Drive Developments - related to space flight applications - Thread 2
Post by: Rodal on 03/17/2015 01:16 pm
Observation #1) The explanation (e.g. the jet model from a poster in this thread, and alternative air current conjectures in several blogs in the Internet) that the thrust measurements were due to thermal convection in the air has been nullified by NASA Eagleworks tests in a vacuum at  6.6*10^(-9) standard atmosphere = 0.0000000066 standard atmosphere
If some kind of perfect Ion engine expels a gas with a density as thin as the one in the NASA Eagleworks vacuum chamber, through a one cm3 nozzle and at a relativistic speed (let say 200,000 km/sec) the thrust due to this very thin gas is in the order of several Newtons.

A far more imperfect Ion engine can certainly provide 100,000 less thrust by accident at this level of pseudo-vacuum.
Did you calculate the thrust that your "thought-experiment ion-engine" would produce with "air propellant expelled through a 1 cm^3 nozzle" stored at 1 standard atmosphere and heated in the microwave cavity?  (the same "imperfect Ion engine" that would produce the measured thrust force of 50 microNewtons thrust with "air propellant expelled through a 1 cm^3 nozzle" stored at 0.0000000066 standard atmosphere "by accident"?)

Is the thrust produced by your "thought-experiment ion-engine" consistent with both the NASA Eagleworks results at 1 standard atmosphere and at 0.0000000066 standard atmosphere?



In your "thought-experiment ion-engine", if the directional thrust force is due to an accidental  1 cm^3 opening acting as a nozzle  at one end of the truncated cone , how is it possible for the experimenters to reverse the direction of the thrust force at will?

(http://www.autoguide.com/blog/wp-content/uploads//2012/06/week_in_reverse3.jpeg)
Title: Re: EM Drive Developments - related to space flight applications - Thread 2
Post by: Rodal on 03/17/2015 02:55 pm
...
Er, yes, no, this is in progress but would need a simulation with professional level tools I haven't knowledge of to be assessed with any certainty...It could.
At least it needs to be properly assessed before it can be ruled out. ...
Suppose (for argument's sake) that your conjecture for shifting center of mass due to thermal expansion could explain the measured response in one direction, which will call the "forward" direction.

(http://forum.nasaspaceflight.com/index.php?action=dlattach;topic=36313.0;attach=811960;image)

Then, how could it possibly explain the measured response in the opposite direction, with the experimenters able to put the EM Drive into "reverse" at will?

Wouldn't that necessitate a cooling effect or a thermal contraction in order for your conjecture to still hold when the experimenters put the EM Drive in reverse?

Due to the laws of thermodynamics, there can only be power dissipation which leads to heat generation, which leads to temperature increase , the microwave electromagnetic field cannot lead to a cooling effect, and hence not to a temperature decrease when put in reverse.

Also the temperature increase leads to a thermal expansion in these materials because they have a positive coefficient of thermal expansion, therefore it is not possible for the electromagnetic field to lead to a thermal contraction.

Hence it appears that your conjecture has already been nullified by NASA Eagleworks reverse experiment, insofar as explaining the total measurement as an artifact: it may only serve to explain a % portion of the measurement as a biasing artifact::

MeasurementFORWARD= EMDriveFORWARDThust + @frobnicatMechanism
MeasurementREVERSE= EMDriveREVERSEThust + @frobnicatMechanism

Assume that the absolute value of the EMDrive thrust is really the same in the forward and reverse directions such that:

+ EMDriveFORWARDThust = - EMDriveREVERSEThust , then:

EMDriveThrust = (MeasurementFORWARD - MeasurementREVERSE) / 2

frobnicatMechanism =  (MeasurementFORWARD + MeasurementREVERSE) / 2



Here is a numerical example, to quantify the above statements:

Consider the Reverse Experiment in relative vacuum where

MeasurementREVERSE = -9.9 microNewtons at 30 W input Power

Consider the Forward Experiment in relative vacuum, taking the initial onset force (see data below) such that:

MeasurementFORWARD = + 30 microNewtons at 30 W input Power

Then

EMDriveThrust = (MeasurementFORWARD - MeasurementREVERSE) / 2 = (+30 - (- 9.9))/2 = 19.95 microNewtons at 30 Watt = 19.95/30 = 67 % of the MeasurementFORWARD

@frobnicatMechanism  = (MeasurementFORWARD + MeasurementREVERSE) / 2 = (+30 + (- 9.9))/2 = 10.05 microNewtons at 30 Watt = 10.05/30 = 33 % of the MeasurementFORWARD

Therefore, @frobnicat's mechanism could be responsible for 1/3 of the forward measurement.

2/3 of the forward measurement could be due to a mechanism of another nature (a mechanism capable of reversing direction).


(http://forum.nasaspaceflight.com/index.php?action=dlattach;topic=36313.0;attach=635481;image)

(http://forum.nasaspaceflight.com/index.php?action=dlattach;topic=36313.0;attach=635484;image)


(http://www.autoguide.com/blog/wp-content/uploads//2012/06/week_in_reverse3.jpeg)
Title: Re: EM Drive Developments - related to space flight applications - Thread 2
Post by: Notsosureofit on 03/17/2015 06:45 pm
FYI

http://scitechdaily.com/researchers-hope-to-provide-new-insights-into-quantum-gravity/

But, it's a spherical cavity.......
Title: Re: EM Drive Developments - related to space flight applications - Thread 2
Post by: frobnicat on 03/17/2015 11:09 pm
...
Suppose (for argument's sake) that your conjecture for shifting center of mass due to thermal expansion could explain the measured response in one direction, which will call the "forward" direction.

(http://forum.nasaspaceflight.com/index.php?action=dlattach;topic=36313.0;attach=811960;image)

Then, how could it possibly explain the measured response in the opposite direction, with the experimenters able to put the EM Drive into "reverse" at will?

Please define "at will". By the turn of a knob ? I vaguely recall claim that for some modes the thrust is not toward the small end but toward the big end, with Star-Drive concluding something in the line of "there is not one simple explanation fit all". I don't find back in what post there was such statement, maybe you have better luck at it. I certainly don't recall having seen a chart showing such microwave driven mode reversal, where the thrust is in the opposite direction, toward the big end. ( I collect piously all charts published ) and have no recall either of any explicit detailed statement specifying precise data for such a microwave driven mode reversal where the thrust is toward the big end (power, frequency, mode, thrust, duration, dielectric, pressure, conditions ?). If you are aware of such data, please link to, since it could be certainly a decisive information.

The chart you just posted is not a microwave mode driven reversal, it is not a reversal of thrust "at will" by the turn of a knob. It is simply that the test article is turned 180°. Naturally any local effect (barren interactions with nearby vacuum chamber wall) would also reverse, whether real thrust or thermal expansions effects. That the chart you are posting (Chasing Thrust Reversal...) corresponds to a situation where the test article is turned 180°, with the big end to the left and small end to the right, is my understanding of the post by Paul March that disclosed the chart (http://forum.nasaspaceflight.com/index.php?topic=36313.msg1327012#msg1327012) :

Quote from: Star-Drive
...  a picture of the new heat shields for our torque pendulum's upper and lower torsion springs, (more belts and suspenders to mitigate thermal drifts in the TP baseline), the reversed test setup drawing and the best reversed thrust plot obtained just before or during when our second and last 120W max RF amplifier was dying from internal corona discharges around its RF output circulator.

I added bold for the same use of qualifier reversed concerning the second and third image here :

bigger (http://forum.nasaspaceflight.com/index.php?action=dlattach;topic=36313.0;attach=634630;image)
(http://forum.nasaspaceflight.com/index.php?action=dlattach;topic=36313.0;attach=635476;image)
Here we see that the 01/16/2015 between 16:15 and 16:45 the test article is mounted with the small end toward the right.

bigger (http://forum.nasaspaceflight.com/index.php?action=dlattach;topic=36313.0;attach=634632;image)
(http://forum.nasaspaceflight.com/index.php?action=dlattach;topic=36313.0;attach=635478;image)
This is qualified as "reversed", small end toward the right. Thrust toward the right. : "Thrust is Right or Down in Plots". Reverse don't qualifies a device that thrusts toward big end. The thrust is always toward the small end. Don't indulge ourselves in absurd controversies on things thrusting in a way, but accelerating the other since reversed and so on. Let's define the thrust by the direction relative to small end. The orientation to the left or to the right of said small end should then be irrelevant.

BTW : we see from comments on top right that Paul March is still (or was at the time of this comment) analysing the impact of thermal expansions on pendulum responses as recoils. Such recoils, while not strictly 0, will have a negligible transient contribution relative to the Z tilted pendulum CoMShift component, this latest greater and sustained effect being in the opposite direction of what is given by the (correct) comments about recoil.

bigger (http://forum.nasaspaceflight.com/index.php?action=dlattach;topic=36313.0;attach=634634;image)
(http://forum.nasaspaceflight.com/index.php?action=dlattach;topic=36313.0;attach=635481;image)
This is qualified as "reversed", is there any possible doubt that this corresponds to the small end to the right again ? Date/time is 01/16/2015 17:53. About an hour or so after the previous pictures were taken, where the small end was to the right. And the door wide open...
So, the dip in the plot is a thrust toward the small end (again). My thermal effect will (or will not) show an effect toward the small end, whether the small end is left or right is irrelevant (should be irrelevant).


I would agree with the 98.7% of the rest of your comments about frobnicat's conjecture being 2/3 nullified if it wasn't for the "reversal of thrust" being a 180° turn of test article.

Obviously, and I hope you will make me the courtesy of not engaging me on lengthy quibble on the fact that a 180° turn of test article, that is defining Forward as "small end to the left" and Reverse as "small end to the right" will make not only
+ EMDriveFORWARDThrust = - EMDriveREVERSEThrust
but also
+ @frobnicatFORWARDMechanism = - @frobnicatREVERSEMechanism

So such 180° "reversal" will not discriminate between EMDriveThrust and frobnicatMechanism

But it could discriminate between an effect purely local to test article (be it EMDriveThrust or frobnicatMechanism) where we would expect the same magnitude, only in reverse, and a vicinity effect with the vacuum chamber walls, where we could expect vicinityFORWARD != - vicinityREVERSE (the position of test article is not symmetric in chamber in both situations). So if we want to take the single test "Chasing Thrust Reversal" chart seriously, it nullifies EMDriveThrust as much as frobnicatMechanism...

SO :

1) Do you agree that the only serious data we have about "reverse" is in fact just a 180° turn of test article ?
2) That for a 180° turn of the test article, it will expand in the opposite direction (for a same heating)
3) That a 180° turn of the test article will therefore reverse a thermal effect as much as it reverses a thrust ?
4) That we don't have precise data concerning a situation where some sustained level of apparent thrust would be toward the big end of test article ? I do recall some mention by Paul March to such thrust toward big end was made, would appreciate if you or anyone find that.

I do agree that precise and consistent data on point 4 would indeed nullify a lot of my conjectures.
Title: Re: EM Drive Developments - related to space flight applications - Thread 2
Post by: Rodal on 03/17/2015 11:34 pm

1) Do you agree that the only serious data we have about "reverse" is in fact just a 180° turn of test article ?
2) That a 180° turn of the test article will reverse a thermal effect as much as it reverses a thrust ?


DEFINTION OF THE WORD REVERSE:

re·verse
rəˈvərs/
verb
1.
move backward.
"the truck reversed into the back of a bus"
synonyms:   back, back up, drive back/backward, move back/backward
"the car reversed into a lamppost"


1) Please refer to the image I provided: of a car's shift mechanism, showing a REVERSE shift. I certainly do NOT consider turning the EM Drive around by 180 degrees to equate "running in reverse", any more than turning a car engaged in "D"  (or in 1st shift) around by 180 degrees would make it run in reverse: the car would still run FORWARD (but in the opposite direction).  Reverse, as indicated by the graph I provided refers to going backwards: a car's reverse !  That's why I attached the huge image showing a car's reverse shift (the significance of the "reverse" R cannot be misconstrued )

(http://www.autoguide.com/blog/wp-content/uploads//2012/06/week_in_reverse3.jpeg)

2) Of course, "That a 180° turn of the test article will reverse turn a thermal effect as much as it reverses turns a thrust ?"
So what ?

That has nothing to do with running in reverse (from the definition of the word reverse), in so much as than turning a car around by 180 degrees still runs forward (in the opposite direction) when engaged in "D" or in "1st shift"



Your post above questions whether the EM Drive can be run in reverse.  You state:

Quote from: @frobnicat
..we don't have precise data concerning a situation where some sustained level of apparent thrust would be toward the big end of test article ? I do recall some mention by Paul March to such thrust toward big end was made, would appreciate if you or anyone find that.

@Mulletron [hat tip for being the first one in the thread to use the car's reverse shift image]  has a number of posts also interpreting that the EM Drive had been run in reverse, for example certain modes run the EM Drive forward and certain modes run the EM Drive in reverse.  One can, in principle, then run the EM Drive in reverse, at will, by "turning the (virtual) knob" of the exciting frequency such as to excite a natural frequency that has a mode shape such as producing an EM Drive's thrust in reverse

I will let Paul March:

1) describe how else the EM Drive can be run in reverse at will [using the literal definition of "reverse"].

2) determine whether my interpretation of the "reverse" measurement of -9.9 microNewtons is indeed a reverse thrust measurement (my interpretation was based on a literal interpretation of the word "reverse") or whether it is a measurement with the EM Drive turned around by 180 degrees (your interpretation).  The image for "reverse" shows the same frequency as for the "forward" images, so if the frequency is correct it means that it was run in the same mode shape both forward and backwards.   Either A) you are correct that the image is really not an EM Drive in reverse (but instead it is the EM Drive turned around and still running forward), or B) if it is indeed running in reverse it must have been accomplished by some other means, other than by changing frequency.

3) if my interpretation of the image showing  "reverse" measurement of -9.9 microNewtons was NOT a reverse thrust measurement, but instead was forward thrust with the EM Drive turned around 180 degrees, then could Paul March please point out to where is the data for the EM Drive run in Reverse. 




PS: No need to warn about "absurd controversies" or about "indulging ourselves."  Such warning (and your discussion about turning the EM Drive around by 180 degrees) was unnecessary if you take into account 1) the definition of the word reverse, and 2) the image I provided of the car's shift mechanism showing a Reverse shift.  The meaning of "R" reverse in a car cannot be misconstrued.

No need for us to get upset about this. It is important to settle this issue, as you recognize:

Quote from: @frobnicat
I do agree that precise and consistent data on point 4 [Reverse tests data] would indeed nullify a lot of my conjectures.
Title: Re: EM Drive Developments - related to space flight applications - Thread 2
Post by: Stormbringer on 03/18/2015 12:31 am
FYI

http://scitechdaily.com/researchers-hope-to-provide-new-insights-into-quantum-gravity/

But, it's a spherical cavity.......

same trick. different pony:  http://phys.org/news/2015-03-nanospheres-cooled-explore-limits-quantum.html
Title: Re: EM Drive Developments - related to space flight applications - Thread 2
Post by: aero on 03/18/2015 12:52 am
Regarding running these thrusters in reverse, I for one am not the least surprised by that result. See the attached graph of meep generated data showing force/power over a range of gap widths around the perimeter on the ends of the frustum. I think it does similar things for constant gap with frequency/mode changes but I don't have the data handy and probably didn't keep it. If someone cares to provide a set of frequencies I'd run them just to document what the curve looks like. Meep gives the the following frequencies as flux generators.

freq. Detected
for epsilon = 1, in Hz
1.5486E+009
1.7422E+009
1.9358E+009
2.1294E+009
2.3229E+009
That is, for the Copper Kettle thruster model with no dielectric.

I do have a question though. Meep measures both flux and force. Each carries it's own sign and is always measured in the positive coordinate direction. In calculating  the direction of the force should I use the signed values, or should I take the sign of the force and just the magnitude of the flux?
Title: Re: EM Drive Developments - related to space flight applications - Thread 2
Post by: zen-in on 03/18/2015 02:40 am
The thrust waveforms have changed significantly in the past few months.  The first picture below is from April 2014.   19.9 Watts of RF power resulted in  a peak thrust of 105 microNewtons.   There is no mention what the error thrust due to magnetic interaction is however.   The waveform has a fast rise time and has an overshoot and other characteristics of an underdamped response.   It is very similar to the calibration drive and the error thrust.

The next thrust waveform, in a vacuum, is very different.   I think it is the result of  a long duration thermal effect and doesn't indicate any thrust at all.    With 50 Watts and a vacuum surrounding the cavity a thermal response would be expected.

The reverse test, also with 50 Watts, brings us back to the underdamped response, but at a very low level.

With no consistency in any of these data the only thing anyone knows for sure is:
1)  Heat is being dissipated in the cavity.
2)  DC current near a magnet generates torque.
Title: Re: EM Drive Developments - related to space flight applications - Thread 2
Post by: Left Field on 03/18/2015 06:40 am
I disagree that "reverse" only means to move backwards, so may I suggest that a different description such as "flipped", "turned around" or "rotated" be used to better describe the experimental setup, especially given the potential for there to be a way to change the directionality electrically (and if that is true then perhaps we could follow the circuit symbol for a cell to help us communicate such directionality i.e. thrust towards the big end is positive; small end is negative)

http://www.thefreedictionary.com/reverse (http://www.thefreedictionary.com/reverse)

adj.
1. Turned backward in position, direction, or order: the reverse side of the poster.
2. Moving, acting, or organized in a manner contrary to the usual: in reverse order.
3. Causing backward movement: a reverse gear.
4. Printing Printed in such a way that the normally colored part appears white against a colored or black background.
n.
1. The opposite or contrary: All along we thought Sue was older than Bill, but just the reverse was true.
2.
a. The back or rear part: the reverse of the flyer.
b. The side of a coin or medal that does not carry the principal design; the verso.
3. A change to an opposite position, condition, or direction.
4. A change in fortune from better to worse; a setback: suffered financial reverses.
5.
a. A mechanism, such as a gear in a motor vehicle, that is used to reverse movement.
b. The position or operating condition of such a mechanism.
c. Movement in an opposite direction.
Title: Re: EM Drive Developments - related to space flight applications - Thread 2
Post by: frobnicat on 03/18/2015 09:02 am
...
1) Please refer to the image I provided: of a car's shift mechanism, showing a REVERSE shift. I certainly do NOT consider turning the EM Drive around by 180 degrees to equate "running in reverse", any more than turning a car engaged in "D"  (or in 1st shift) around by 180 degrees would make it run in reverse: the car would still run FORWARD (but in the opposite direction). 

Very well put, this would also be my definition of reverse, but this is not how the qualifier "reversed" was used by Paul March on the chart "Chasing Thrust Reversal". Paul March called reversed a result that was just a 180° turn. Do you agree or not on that this is the (unusual) specific use of of the qualifier reverse by Paul March in regard to the chart you used as a proof of (true usual) reversed mode ? That therefore the chart "Chasing Thrust Reversal" is not reversed in the usual sense and brings next to nothing to the discussion (about thrust vs thermal expansions)

Quote
Reverse, as indicated by the graph I provided refers to going backwards: a car's reverse !  That's why I attached the huge image showing a car's reverse shift (the significance of the "reverse" R cannot be misconstrued )

No, the graph your provided (if we are speaking the one titled "chasing thrust reversal") does not use the qualifier reversal in this sense, it only means turned 180°, small end toward the right. The car's reverse shift analogy is irrelevant as far as this chart and accompanying data is concerned. The significance of reverse R on a car's reverse shift cannot be misconstrued. The significance of "reverse" as used (in an unusual way) on the chart "chasing thrust reversal" was misleading. Insofar, all we have is a car that was turned 180°.

Quote
2) Of course, "That a 180° turn of the test article will reverse turn a thermal effect as much as it reverses turns a thrust ?"
So what ?

Frankly, sometimes things that seem so obvious by one get misconstrued by another. I just wanted to make sure that we agree on that. Now we just have to agree (or disagree) on the fact that on published charts so far "reverse" was not reverse, it was just 180° turn, that we don't have data about "true" reverse modes.

Quote
...
Your post above questions whether the EM Drive can be run in reverse.  You state:

Quote from: @frobnicat
..we don't have precise data concerning a situation where some sustained level of apparent thrust would be toward the big end of test article ? I do recall some mention by Paul March to such thrust toward big end was made, would appreciate if you or anyone find that.

@Mulletron [hat tip for being the first one in the thread to use the car's reverse shift image]  has a number of posts also interpreting that the EM Drive had been run in reverse, for example certain modes run the EM Drive forward and certain modes run the EM Drive in reverse.  One can, in principle, then run the EM Drive in reverse, at will, by "turning the (virtual) knob" of the exciting frequency such as to excite a natural frequency that has a mode shape such as producing an EM Drive's thrust in reverse

Very well, I'm tired of reading the archives again and again. Now you want to make a point about the test article being driven in true reverse having such or such consequence (that I would agree), but you post a chart that only shows data that is not true reverse but only 180° turn, that is therefore irrelevant for your argument.

To make your point stand, please find the most accurate data about a true reverse situation,  certain modes run the EM Drive forward and certain modes run the EM Drive in reverse : what modes ? what frequencies ? what powers ? what magnitude ? what durations ? what shapes ?

Quote
I will let Paul March:

1) describe how else the EM Drive can be run in reverse at will [using the literal definition of "reverse"].

Actual charts and very clear statements about direction of thrust relative to geometry of frustum would be greatly appreciated. Clear as with "thrust toward the big end".

Quote
2) determine whether my interpretation of the "reverse" measurement of -9.9 microNewtons is indeed a reverse thrust measurement (my interpretation was based on a literal interpretation of the word "reverse") or whether it is a measurement with the EM Drive turned around by 180 degrees (your interpretation).  The image for "reverse" shows the same frequency as for the "forward" images, so if the frequency is correct it means that it was run in the same mode shape both forward and backwards.   Either A) you are correct that the image is really not an EM Drive in reverse (but instead it is the EM Drive turned around and still running forward), or B) if it is indeed running in reverse it must have been accomplished by some other means, other than by changing frequency.

I'm also waiting for Paul March confirming either A or B, but frankly do you seriously believe, with the elements I gave in my last post, that B stands correct ? If it takes hours and hours of discussion to settle such simple matter...

Quote
3) if my interpretation of the image showing  "reverse" measurement of -9.9 microNewtons was NOT a reverse thrust measurement, but instead was forward thrust with the EM Drive turned around 180 degrees, then could Paul March please point out to where is the data for the EM Drive run in Reverse. 

It is my understanding no such data (complete, detailed) were disclosed so far.

Quote


PS: No need to warn about "absurd controversies" or about "indulging ourselves."  Such warning (and your discussion about turning the EM Drive around by 180 degrees) was unnecessary if you take into account 1) the definition of the word reverse, and 2) the image I provided of the car's shift mechanism showing a Reverse shift.  The meaning of "R" reverse in a car cannot be misconstrued.

I'm warning that a leading author driving the discussion has a use of the word Reverse that is misleading. The meaning of "R" reverse on Eagleworks plots not only can but has been misconstrued.

Quote
No need for us to get upset about this. It is important to settle this issue, as you recognize:

Quote from: @frobnicat
I do agree that precise and consistent data on point 4 [Reverse tests data]
 would indeed nullify a lot of my conjectures.

I'm actually (a bit) upset indeed that we are not really requiring of the leading authors half of the consistency in wording that seems to be required from me. That ambiguities, when they occur, seem to be hastily interpreted in favour of EMdriveThrust hypothesis and in disfavour of competing schemes, instead of being recognized as ambiguities that need proper and neutral investigation.

Edit : My claims, while needing further justifications and clarifications, and maybe experimentally wrong in the end, are within proven frameworks of physics. The claims of real EMDrive thrust are the extraordinary claims, needing extraordinary clarity in result reports and consistency of data. For recall we have a 30 time factor unexplained at the core of the process of measuring pendulum displacement vs calibrations pulse, we have varying (by 1 to 5) vertical scale readings from chart to chart, we have a signal that should be symmetric when turning the test article 180° and that is not symmetric, we have a Z tilted axis (of weakly characterised tilt angle) that transforms stationary shifts in mass parts of test article into sustained displacement readings, we have undisclosed charts for the "0 thrusts" when operating without dielectric disc (or with neoprene), we have not seen the 20 charts of the 5 test at each power 30W 40W 50W 60W of the "forward thrust campaign" that would allow to average curves and make some statistics instead of arguing on noise plagued single cases, we have a disparity of thrust in vacuum relative to air... All those aspects must be investigated, not just the easy ones. I say that as I'm sure it must be itching to answer the easy ones and forget the hard ones or sweep them under the carpet (http://forum.nasaspaceflight.com/index.php?topic=36313.msg1343900#msg1343900).


To clarify things : I am not thinking that Paul March used the word "reverse" in a unusual way voluntarily to mislead the reader, I think it was very clear to him that he was speaking about a 180° turn, and that it would be equally clear to reader. Reading his post (http://forum.nasaspaceflight.com/index.php?topic=36313.msg1327012#msg1327012) and attached picture it was indeed relatively clear and straightforward to me that, in this chart, "reverse" was just a 180° turn, even if a bit awkward formulation.
Title: Re: EM Drive Developments - related to space flight applications - Thread 2
Post by: frobnicat on 03/18/2015 09:17 am
The thrust waveforms have changed significantly in the past few months.  The first picture below is from April 2014.   19.9 Watts of RF power resulted in  a peak thrust of 105 microNewtons.   There is no mention what the error thrust due to magnetic interaction is however.   The waveform has a fast rise time and has an overshoot and other characteristics of an underdamped response.   It is very similar to the calibration drive and the error thrust.

The next thrust waveform, in a vacuum, is very different.   I think it is the result of  a long duration thermal effect and doesn't indicate any thrust at all.    With 50 Watts and a vacuum surrounding the cavity a thermal response would be expected.

The reverse test, also with 50 Watts, brings us back to the underdamped response, but at a very low level.

With no consistency in any of these data the only thing anyone knows for sure is:
1)  Heat is being dissipated in the cavity.
2)  DC current near a magnet generates torque.

This is what we have (same color, same topic) :

Dr. Rodal:

Looking over some of our test results from last spring and summer just now, I realized that I had forgotten to provide this forum a few test runs at some of the other resonant modes we've looked at that used different dielectric discs and locations including polycarbonate which I hadn't mentioned before.  These examples are attached and you will note that these other copper frustum resonant modes (TM010 & TE012) have very different thermal torque pendulum responses than the TM212 (your TM221 I think) cases we've discussed of late.  That fact might be of importance when discussing whether these thrust signals are real or just artifacts...

PS: The magnitude of the torque pendulum's overshoot is directly related to what version of the magnetic damper that was used for the particular test in question and how thick the copper damper blade was at the time.  At the moment we are using an arrangement that is slightly over-damped in an attempt to smooth out some of the pesky low frequency seismic noise in the system.

DPS:  The magnitude of the RF amp's dc current induced thrust signal offset is directly related to how much leakage current is going through the torque pendulum's steel torsion bearings.  As I became aware of this problem over the last year, I've found various ground wiring tricks to mitigate this dc current induced negative going offset, but I've never found a way to get rid of all of it.

Best, Paul M
Title: Re: EM Drive Developments - related to space flight applications - Thread 2
Post by: frobnicat on 03/18/2015 09:28 am
I disagree that "reverse" only means to move backwards, so may I suggest that a different description such as "flipped", "turned around" or "rotated" be used to better describe the experimental setup, especially given the potential for there to be a way to change the directionality electrically (and if that is true then perhaps we could follow the circuit symbol for a cell to help us communicate such directionality i.e. thrust towards the big end is positive; small end is negative)


Yes, reverse is polysemous. I agree for a clear convention. Using negative to mean "toward the small end" would have the advantage of being compatible with my (relatively arbitrary) direct OXYZ conventions on my drawings, with thrust measured relative to  axis Y, when the small end is to the left (as seen from front of experiment) which was the case for all the tests qualified as "forward" (most of the tests). It has the disadvantage of most of the time talking of negative thrusts, since all the published data so far was toward the small end. The only way to make it crystal clear is to state explicitly "thrust toward the small end" (all published data so far) or "thrust toward the big end" (data yet to be seen), and saying if small-end is mounted toward the left (most of the experiments) or to the right (the so called "reversed" experiment).
Title: Re: EM Drive Developments - related to space flight applications - Thread 2
Post by: Rodal on 03/18/2015 11:16 am
...

Quote from:  Star-Drive
one can also reverse this thrust vector for this copper frustum by just changing which excited resonant mode is used

(http://forum.nasaspaceflight.com/index.php?action=dlattach;topic=36313.0;attach=778913;image)

(http://www.autoguide.com/blog/wp-content/uploads//2012/06/week_in_reverse3.jpeg)

I will calmly wait for the person that conducted the experiments (Paul March "Star-Drive") to confirm whether the EM Drive can indeed be run at will (by changing the exciting frequency) with the thrust force directed towards the big base (instead of towards the small base) of the truncated cone, and if so, to point out the relevant data demonstrating that fact.

Mr. Frobnicat, with all due respect, since you did not perform these experiments, you cannot write with pontificating authority to resolve this matter.

If Paul March answers that the EM Drive cannot be run with the thrust force directed towards the big base (instead of towards the small base) of the truncated cone, I will be very pleased to have learned this fact and to have corrected my misunderstanding.

On the other hand if Paul March answers that the EM Drive can indeed be run with the thrust force directed towards the big base (instead of towards the small base) of the truncated cone, by changing the exciting frequency, your conjecture (if it pretends to explain the total measurement) will be shown to have been already nullified by NASA Eagleworks.  In that case, your conjecture might, at best, serve to explain a fraction of the measurement as a biasing artifact. 

Title: Re: EM Drive Developments - related to space flight applications - Thread 2
Post by: aceshigh on 03/18/2015 04:12 pm
regarding the earlier quote by Paul March saying he could retire (or be retired?) from NASA by the end of March...

what are the conditions for that happening? Not getting positive results for EM Drive?

is the whole Eagleworks Lab being shut down? Which would also mean the end of the Warp Drive and QM Vacuum Thruster Experiments and also the EM Cannae Drive experiments?



Title: Re: EM Drive Developments - related to space flight applications - Thread 2
Post by: Mulletron on 03/18/2015 07:12 pm
Before today, I've never used my set of tin snips for anything more than hacking difficult cables in two. I gotta say, there is definitely a skill set required on how to use tin snips properly if you want to make precision cuts.

There is a lot of bad information out there on the internet, like the red ones only cut left, or green ones only cut right. The real info is that red ones push waste material to the left, and green ones push waste material to the right. They can cut either way. Make sure the waste side is away from your work. That is all that matters.

So I watched a couple youtube videos and practiced on some extra sheet metal I had laying around and got good enough to cut out some circles from expensive copper sheet. It is pretty easy. I made some perfect circles no sweat. Lucky for me, I like to hoard tools, so those tin snips I got 2-3 years ago finally got used for something useful. So you potential replicators out there, make sure you're good with tin snips before you cut, it ain't as easy as scissors. Just widdle down so there is no more than 1/8" of excess around your work then, go in for detail work.

I had trouble seeing my score marks I made with the sheet metal dividers, even with the room well lit. I discovered that an LED head lamp set to red makes the score marks POP, so I could easily see them and follow the cut. White light just blinded me. I didn't want to use a sharpie. I wanted sub millimeter accuracy.

I opted against using the left over sheets from the sample pack I got to be used as end plates. Instead I got 2 12"x12" sheets of 16oz copper on Amazon to make the end plates. The whole copper kettle is 16oz copper.

For now (and possibly forever) the end plates are clamped on with numerous stainless steel paper clamps in. I know I'll probably get some crap over using the clamps, but I have to tell you that these things are crazy strong. I can't pull them apart with my finger tips. This isn't a flight ready thruster, it is a rapid prototype for research. All I care about is keeping the ends firmly against the frustum and that the entire assembly is electrically short and mechanically true. Drilling >9000 bolt holes doesn't seem appealing to me right now; especially since the clamps hold over a wide surface area and do a good job.

After putting the whole thing together, it is extremely sturdy. Thanks to 1952rickey on Ebay for building me the frustum. He did a really good job. He still hasn't answered me back about listing the microwave resonant cavity for research, yet. Feel free to send him a message if you want one too. I want to be clear, he is making resonant cavities, not thrusters or Emdrives.

Ok, so I was very careful to never touch the inside of the cavity without gloved hands. The outside has some fingerprints. Is there anything I could use to wipe down the inside and outside to stave off corrosion?.......while not messing up the RF characteristics? Copper loves to turn green. I need the Q to be as high as possible.

To do, I have all the dielectric sheets in except the UHMW PE. My to do list for tomorrow is to jig saw out 2, 6.25" HD PE circles and drill/mount them. I think this 1" thick sheet of plastic is going to do in my saw. This sheet is very dense and heavy. Wish I had a band saw.

I got a length of very lightweight extruded rectangular aluminum. It is 150cm long. I'm going to cut it down to 1 meter. This will serve as the platform of the balance. This thing is extremely light and doesn't bend.

I have a scale on the way which can/will accurately weigh all this stuff down the sub gram accuracy. It is important to know how much all this stuff weighs as accurately as possible.

Pics are here:
https://drive.google.com/folderview?id=0B4PCfHCM1KYoTXhSUTd5ZDN2WnM&usp=sharing



Title: Re: EM Drive Developments - related to space flight applications - Thread 2
Post by: timbatman1960 on 03/18/2015 09:05 pm
All:

Sorry I didn't make the time to participate in this ME-Drive forum for the last 6-to-8 months up, but I will try to catch up with everyone else in due course.  That said lets try to answer the questions that popped up since my morning post.

1.  I was not the lead author for the Eagleworks' 2014 AIAA/JPC paper and in fact I only supplied pictures and data for same during that period because Dr. White thought that my time was best spent in the lab gathering data instead of report writing.  Thus some of the details that Dr. Rodal is looking for may have been lost or garbled in the report writing by the others on the author list.

2.0  The thrust vector for the four resonant modes examined in detail, (the cavity's fundamental TM010, TE012, TM211 & TM212 for our copper frustum is normally in the frustum's large OD to small OD direction for most, but not all the E&M resonant modes checked.  However, one can also reverse this thrust vector for this copper frustum by just changing which excited resonant mode is used and/or mounting the dielectric discs at the large OD end of the cavity instead of the small OD end, see attached resonant mode map.  Sorry, but a one size fits all solution to this EM-Drive thrust direction is not available in this venue because of the importance of the ExB phase relationship of the expressed Lorentz forces between the excited E&M fields and the possible dielectric and QV plasma flow phenomenon that may be at work in each resonant mode expressed.  That is why this type of E&M thruster is so hard to get a handle on, for there are far too many degrees of freedom in the system to track let alone directly control.

3. The Eagleworks vacuum chamber's main body is made from 304L stainless steel while its swing out door is made from aluminum.  Most of the nuts and bolts in the vacuum chamber are also made from 18-8, 304 or 316 stainless steel alloys. 

Now to try to answer Dr. Rodal's specific questions:

"1) In the NASA experiments the truncated cone's center of  mass moved towards the [  ? ] diameter end  (where ? stands for big or small)"

For the TE012 and TM212 excited resonant modes, our copper frustum's center of mass moved toward the small OD end of the frustum when RF power was applied to the copper frustum.

"2) In the NASA experiments, we at NASA Eagleworks define the thrust force direction to be in the  [? ] direction as the movement of the truncated cone's center of  mass  (where ? stands for same or opposite)"

For just the TE012 & TM212 excited resonant modes, the thrust force direction AKA thrust vector was observed to be in the same direction as the movement of the frustum's center of mass when RF power was applied to the frustum's magnetic loop antenna.

If I missed a question along the way keep asking, but I'll be in and out of the house for the rest of the day, so I may not get to answer them until late this evening or tomorrow afternoon USA based CST.

Best, Paul M.

I understand that we need to wait for Paul March to explicitly "re-verify" the manner in which thrust reversal was achieved, but it does appear to me that he has already  addressed this in the above quote from Feb 14.  Quoting Mr. March:  "one can also reverse this thrust vector for this copper frustum by just changing which excited resonant mode is used and/or mounting the dielectric discs at the large OD end of the cavity instead of the small OD end".   To my admittedly untrained eye,  this appears to be in agreement with Dr. Rodal's earlier statements, and in conflict with Mr. Frobnicat's contention that reversal was achieved by simply rotating the entire mechanism 180 degrees.
Title: Re: EM Drive Developments - related to space flight applications - Thread 2
Post by: frobnicat on 03/18/2015 10:22 pm

Quote from:  Star-Drive
one can also reverse this thrust vector for this copper frustum by just changing which excited resonant mode is used

Thank you for bringing up this citation that allows to find the original mention of "true reverse" by mode switching.

...
2.0  The thrust vector for the four resonant modes examined in detail, (the cavity's fundamental TM010, TE012, TM211 & TM212 for our copper frustum is normally in the frustum's large OD to small OD direction for most, but not all the E&M resonant modes checked.  However, one can also reverse this thrust vector for this copper frustum by just changing which excited resonant mode is used and/or mounting the dielectric discs at the large OD end of the cavity instead of the small OD end, see attached resonant mode map.  Sorry, but a one size fits all solution to this EM-Drive thrust direction is not available in this venue because of the importance of the ExB phase relationship of the expressed Lorentz forces between the excited E&M fields and the possible dielectric and QV plasma flow phenomenon that may be at work in each resonant mode expressed.  That is why this type of E&M thruster is so hard to get a handle on, for there are far too many degrees of freedom in the system to track let alone directly control.
...

"1) In the NASA experiments the truncated cone's center of  mass moved towards the [  ? ] diameter end  (where ? stands for big or small)"

For the TE012 and TM212 excited resonant modes, our copper frustum's center of mass moved toward the small OD end of the frustum when RF power was applied to the copper frustum.
...

That is not really stating explicitly that any experiment showed such thrust toward the big end. Attached "resonant mode map" is not an experimental one. It just shows the theoretical result of a calculation. If we are to interpret the blue plot on top as thrusting toward small end when above 0 and thrusting toward the big end when below 0, then this is contradicting the theoretical formula that the TM modes results published so far all thrust toward small end as well as the TE modes.

If a clear signal toward big end was actually recorded, it seems rather strange that such experimental plot would not have been disclosed as it may confirm the theoretical blue plot, and even if wasn't confirming this formula that would still be a strong case against a lot of classical explanations for the signal.

In the above quoted post by Star-Drive it is a matter of interpretation to understand that it is explicitly stated that there was actual toward big end thrusts or not explicitly stated. My reading is that it is not explicitly stated, but the wording is ambiguous enough that the reverse reading could be argued. I hope Paul March can settle this matter in the clearest manner : yes or no was there at some point an experimentally recorded thrust toward the big end ?

In any case, the following plot is not experimental proof of anything.

bigger (http://forum.nasaspaceflight.com/index.php?action=dlattach;topic=36313.0;attach=778912;image)
(http://forum.nasaspaceflight.com/index.php?action=dlattach;topic=36313.0;attach=778913;image)

Quote from: Rodal
I will calmly wait for the person that conducted the experiments (Paul March "Star-Drive") to confirm whether the EM Drive can indeed be run at will (by changing the exciting frequency) with the thrust force directed towards the big base (instead of towards the small base) of the truncated cone, and if so, to point out the relevant data demonstrating that fact.

Mr. Frobnicat, with all due respect, since you did not perform these experiments, you cannot write with pontificating authority to resolve this matter.

I did brought to the subject a fresh and rational third party look, as you did, as many contributors here did, and I'm not the first of the thread to give the impression of thinking having decisive arguments. My style is what it is, but I'm making perfectly sensible, substantiated, articulated arguments from the available data and what Star-Drive had the courtesy to share with us. Any sceptical person with mechanical engineering background caring to dig the disclosed informations would arrive at similar questions and doubts. So while I said I a lot of time lately (regrettably, would rather hear we), this is not a matter of me.

Quote
If Paul March answers that the EM Drive cannot be run with the thrust force directed towards the big base (instead of towards the small base) of the truncated cone, I will be very pleased to have learned this fact and to have corrected my misunderstanding.

On the other hand if Paul March answers that the EM Drive can indeed be run with the thrust force directed towards the big base (instead of towards the small base) of the truncated cone, by changing the exciting frequency, your conjecture (if it pretends to explain the total measurement) will be shown to have been already nullified by NASA Eagleworks.  In that case, your conjecture might, at best, serve to explain a fraction of the measurement as a biasing artifact.

Basically I agree.

But can is a can of worms. Star-Drive already stated that "However, one can also reverse this thrust vector for this copper frustum by just changing which excited resonant mode is used and/or mounting the dielectric discs at the large OD end of the cavity instead of the small OD end, see attached resonant mode map." which is a theoretical model map... The end of the sentence immediately transforms can into could in my brain. Am I badly wired ? A clear answer would be that the EM Drive has indeed been experimentally recorded running with the thrust force directed towards the big base (with such or such conditions).

I'm not splitting hairs for pleasure. If I learned one thing in those discussions is the degree of misunderstanding that can (does) creep into the arguments and projected motivations. As for the 0 force of "unconstrained thermal expansion" discussions we had. It seems you still think that I was arguing that for vacuous intellectual righteousness reasons, that in fact I believed, like you, that the subject wasn't actually important in the context of EMdrive discussions. Actually I believed that the subject was important in the context of EMdrive. Then (later) I made quantitative assessment of the unrestrained thermal expansions recoil momentums, and am now convinced that it is not, quantitatively, significant. Paul March is apparently still struggling to explain some of the thermal aspects of signal by recoil effects. So I know I'm not the only one to have taken the matter of unrestrained thermal expansions recoil momentums as, a priori, important. Oh, and while I'm at it, the Oakridge paper explaining some of the results of ME drives on pendulum by classical thermal expansion recoil effects (synchronised on many oscillating periods: not applicable for Eagleworks results) makes perfect sense to me, their equations are impeccable  :)
Title: Re: EM Drive Developments - related to space flight applications - Thread 2
Post by: frobnicat on 03/18/2015 10:37 pm

I understand that we need to wait for Paul March to explicitly "re-verify" the manner in which thrust reversal was achieved, but it does appear to me that he has already  addressed this in the above quote from Feb 14.  Quoting Mr. March:  "one can also reverse this thrust vector for this copper frustum by just changing which excited resonant mode is used and/or mounting the dielectric discs at the large OD end of the cavity instead of the small OD end".   To my admittedly untrained eye,  this appears to be in agreement with Dr. Rodal's earlier statements, and in conflict with Mr. Frobnicat's contention that reversal was achieved by simply rotating the entire mechanism 180 degrees.

So maybe I'm badly wired. But please take care to read carefully the later post by Paul March (http://forum.nasaspaceflight.com/index.php?topic=36313.msg1327012#msg1327012) where the only experimental data about "reversal" is disclosed. See the orientations of the frustum on the second and third attached pictures ? The plot with the reversed signal just follows, it was taken within an hour of the second picture (time stamps). Same qualifier "reversed" is used. Also in the same phrase : "...the reversed test setup drawing and the best reversed thrust plot obtained...". The drawing of setup shows small end toward right, this is a 180° turn from the other experiments, the "reversed" thrust plot is therefore quite clearly stated as being done with a setup with small end to the right, reverse as in "turned 180°", my understanding is that the plot is not about a thrust toward big end. Would seem rather contrived or carelessly misleading otherwise.

The previous comment you quote is more ambiguous. But I hope you understand the reasons of my understanding (even if not sharing it) as far as the only experimental "reversed" result published so far is concerned.
Title: Re: EM Drive Developments - related to space flight applications - Thread 2
Post by: Notsosureofit on 03/18/2015 10:47 pm
Paul's statement that "For the TE012 and TM212 excited resonant modes, our copper frustum's center of mass moved toward the small OD end of the frustum when RF power was applied to the copper frustum."  Would seem to explicitly contradict the calculated chart as to the direction for those modes.
Title: Re: EM Drive Developments - related to space flight applications - Thread 2
Post by: Rodal on 03/18/2015 11:25 pm
....
Welcome to NASA SpaceFlight EM Drive Developments' thread  :)
Title: Re: EM Drive Developments - related to space flight applications - Thread 2
Post by: frobnicat on 03/18/2015 11:26 pm
Paul's statement that "For the TE012 and TM212 excited resonant modes, our copper frustum's center of mass moved toward the small OD end of the frustum when RF power was applied to the copper frustum."  Would seem to explicitly contradict the calculated chart as to the direction for those modes.

Actually the calculated chart does predict thrust toward small end for TE012 (blue plot up) but the TM212 is nowhere to be seen (is it ?). The original article (anomalous thrust...) shows only thrusts toward small end, including TM211, that do appear on the calculated chart as thrusting toward big end (blue plot down) : so yes some experiments seem to invalidate the used formula.
Title: Re: EM Drive Developments - related to space flight applications - Thread 2
Post by: Notsosureofit on 03/19/2015 12:18 am
Paul's statement that "For the TE012 and TM212 excited resonant modes, our copper frustum's center of mass moved toward the small OD end of the frustum when RF power was applied to the copper frustum."  Would seem to explicitly contradict the calculated chart as to the direction for those modes.

Ahhh, I see, I've been reading the (small print) TM211A as TM212A,... my mistake.  I might add that I never get any forces out of ExB calculations, that's why I like plain GR Equivilance, but only time (and more experiments) will tell.
Title: Re: EM Drive Developments - related to space flight applications - Thread 2
Post by: ThinkerX on 03/19/2015 12:26 am
To me, it seems like 'frobnicat' came very close to 'reinventing the wheel' with these recent posts.

We really do need a glossary of the known points about this device.

   
Title: Re: EM Drive Developments - related to space flight applications - Thread 2
Post by: Rodal on 03/19/2015 12:29 am
To me, it seems like 'frobnicat' came very close to 'reinventing the wheel' with these recent posts.

We really do need a glossary of the known points about this device.

 

We should resist a rush to judgement (this goes for Mr. Fronicat as well), until Paul March clarifies his experimental program.

Were experiments conducted that showed a force towards the small base of the truncated cone for a given exciting frequency and a force in the opposite direction, towards the big base of the truncated cone at a different exciting frequency ?

Are there "force vs. time" (really "position vs. time") data plots for the EM Drive showing a force towards the big end? If yes, which plots correspond to the experiments where the force was directed towards the big base of the truncated cone?
Title: Re: EM Drive Developments - related to space flight applications - Thread 2
Post by: aero on 03/19/2015 12:41 am
To me, it seems like 'frobnicat' came very close to 'reinventing the wheel' with these recent posts.

We really do need a glossary of the known points about this device.

 
I started to try to do this the other day. We need everyone to pitch in and add what they can, and we really need a wiki format to update.

A look at the aspects of the EM thruster hypotheses discussed.

On the 250 odd pages of this and the previous thread, we have discussed many hypotheses related to the EM thrusters. This is my poor attempt to outline those topics as related to the detected thrust. First, these topics fall into two broad categories, real and artifact. Others may have a more logical summary categorization.

My categories are:

I. Artefact -
    A. Thermal - I will not attempt to break down this. Many considered thermal artifacts.
    B. Device - Includes analysis of Eagleworks torsion pendulum/chamber.
        1.
        2. Attraction of cavity to chamber - Nullified by Eagleworks open air experiment.
II. Real -
    A. Thrust magnitude -N/Watt- factors
        1. Resonance modes
        2.
        3.
    B. Momentum exchange mechanism
        1. Classic known physics
            i. Shawyer theory
            ii. Chinese theory
            iii. Evanescent Wave theory
            iv. Dynamic Casimer theory
            v. Others I don't recall at the moment.
        2. Classic unknown physics
            i. Casimer related
            ii. Evanescent wave related
            iii. Mach effect (GR + gravinertial Wheeler-Feynman field, i.e. inertia)y
        3. New Physics
            i. Quantum Vacuum -Quantum plasma theory
            ii. MiHsC theory
            iii. Dark Matter - Nullified based on insufficient density of DM.
            iv. Scalar–tensor theory (GR + scalar field)

edit Add: GR, DM,

 
Title: Re: EM Drive Developments - related to space flight applications - Thread 2
Post by: Notsosureofit on 03/19/2015 12:51 am
Well, under classic unknown, you can put General Relativity..
Title: Re: EM Drive Developments - related to space flight applications - Thread 2
Post by: Rodal on 03/19/2015 01:03 am
But it is interesting that Roger Shawyer reports forces in different directions (sometimes towards the small end and sometimes towards the big end) for different experimental conditions.

Are these differences really due to the Poynting vector ExB being different for different mode shapes?

(Shawyer -unlike NASA Eagleworks- did not report mode shapes computational studies)

Shawyer reports forces in opposite directions (towards the big end and towards the small end of practically the same magnitude: 214 mN/kW and 243 mN/kW) for his Demonstrator engine.  This seems to give a death knell to mechanistic conjectures like Mr. Frobnicat's mechanistic conjecture.


(http://forum.nasaspaceflight.com/index.php?action=dlattach;topic=36313.0;attach=776944;image)
Title: Re: EM Drive Developments - related to space flight applications - Thread 2
Post by: aero on 03/19/2015 01:49 am
Re. Hypotheses looked at - where does our lengthy discussions of dark matter fit?

http://forum.nasaspaceflight.com/index.php?topic=36313.msg1347946#msg1347946 (http://forum.nasaspaceflight.com/index.php?topic=36313.msg1347946#msg1347946)
Title: Re: EM Drive Developments - related to space flight applications - Thread 2
Post by: Stormbringer on 03/19/2015 01:53 am
Re. Hypotheses looked at - where does our lengthy discussions of dark matter fit?

http://forum.nasaspaceflight.com/index.php?topic=36313.msg1347946#msg1347946 (http://forum.nasaspaceflight.com/index.php?topic=36313.msg1347946#msg1347946)
we had a lengthy discussion of dark matter and i didn't know about it?!

http://www.sciencedaily.com/releases/2015/03/150306091617.htm

http://phys.org/news/2015-03-mini-black-holes-lhc-parallel.html
Title: Re: EM Drive Developments - related to space flight applications - Thread 2
Post by: zen-in on 03/19/2015 02:26 am

Quote from:  Star-Drive
one can also reverse this thrust vector for this copper frustum by just changing which excited resonant mode is used

Thank you for bringing up this citation that allows to find the original mention of "true reverse" by mode switching.

...
2.0  The thrust vector for the four resonant modes examined in detail, (the cavity's fundamental TM010, TE012, TM211 & TM212 for our copper frustum is normally in the frustum's large OD to small OD direction for most, but not all the E&M resonant modes checked.  However, one can also reverse this thrust vector for this copper frustum by just changing which excited resonant mode is used and/or mounting the dielectric discs at the large OD end of the cavity instead of the small OD end, see attached resonant mode map.  Sorry, but a one size fits all solution to this EM-Drive thrust direction is not available in this venue because of the importance of the ExB phase relationship of the expressed Lorentz forces between the excited E&M fields and the possible dielectric and QV plasma flow phenomenon that may be at work in each resonant mode expressed.  That is why this type of E&M thruster is so hard to get a handle on, for there are far too many degrees of freedom in the system to track let alone directly control.
...
...

If a clear signal toward big end was actually recorded, it seems rather strange that such experimental plot would not have been disclosed as it may confirm the theoretical blue plot, and even if wasn't confirming this formula that would still be a strong case against a lot of classical explanations for the signal.

In the above quoted post by Star-Drive it is a matter of interpretation to understand that it is explicitly stated that there was actual toward big end thrusts or not explicitly stated. My reading is that it is not explicitly stated, but the wording is ambiguous enough that the reverse reading could be argued. I hope Paul March can settle this matter in the clearest manner : yes or no was there at some point an experimentally recorded thrust toward the big end ?

Quote from: Rodal
I will calmly wait for the person that conducted the experiments (Paul March "Star-Drive") to confirm whether the EM Drive can indeed be run at will (by changing the exciting frequency) with the thrust force directed towards the big base (instead of towards the small base) of the truncated cone, and if so, to point out the relevant data demonstrating that fact.

Mr. Frobnicat, with all due respect, since you did not perform these experiments, you cannot write with pontificating authority to resolve this matter.

I did brought to the subject a fresh and rational third party look, as you did, as many contributors here did, and I'm not the first of the thread to give the impression of thinking having decisive arguments. My style is what it is, but I'm making perfectly sensible, substantiated, articulated arguments from the available data and what Star-Drive had the courtesy to share with us. Any sceptical person with mechanical engineering background caring to dig the disclosed informations would arrive at similar questions and doubts. So while I said I a lot of time lately (regrettably, would rather hear we), this is not a matter of me.

Quote
If Paul March answers that the EM Drive cannot be run with the thrust force directed towards the big base (instead of towards the small base) of the truncated cone, I will be very pleased to have learned this fact and to have corrected my misunderstanding.

On the other hand if Paul March answers that the EM Drive can indeed be run with the thrust force directed towards the big base (instead of towards the small base) of the truncated cone, by changing the exciting frequency, your conjecture (if it pretends to explain the total measurement) will be shown to have been already nullified by NASA Eagleworks.  In that case, your conjecture might, at best, serve to explain a fraction of the measurement as a biasing artifact.


What I would like to see are thrust waveforms with the cavity position turned 180° around.   This is what I thought was meant by reverse thrust waveforms.    If the thrust waveforms with the cavity position turned 180° around are in the opposite direction and have the same absolute magnitude as the others, given the same RF power and dielectric insert, then it would go a long way towards proving this device actually does produce thrust.   Sorry for being such a doubter but that just comes with old age.

I don't believe further claims of reverse thrust when different modes are used invalidates any alternative theory of the anomalous thrust because all the waveforms seen represent very small TP rotations.   The only way alternative theories can be invalidated is by performing counter experiments and baseline tests such as:

1) Measure the error thrust by using a dummy load under the same conditions as a just completed thrust test.
2) Performing controlled tests with the cavity reversed and forward
3) Apply heat to the inside of the cavity and record any change in the LDS reading.
4) Determine the sensitivity of the LDS to changes in CM.  What affect do small ( ~arcSec ) rotations of the TP beam, with the TP position fixed, have on the LDS reading?
Title: Re: EM Drive Developments - related to space flight applications - Thread 2
Post by: aero on 03/19/2015 04:16 am
Re. Hypotheses looked at - where does our lengthy discussions of dark matter fit?

http://forum.nasaspaceflight.com/index.php?topic=36313.msg1347946#msg1347946 (http://forum.nasaspaceflight.com/index.php?topic=36313.msg1347946#msg1347946)
we had a lengthy discussion of dark matter and i didn't know about it?!

http://www.sciencedaily.com/releases/2015/03/150306091617.htm

http://phys.org/news/2015-03-mini-black-holes-lhc-parallel.html

Well, I just searched the complete advanced concepts forum and found no mention of dark matter on any EM drive developments thread, so I guess either my memory is faulty or there has been some heavy clean-up of what many consider to be "Uggy-Bogy" science. That means we won't be including it in "hypotheses discussed."
Title: Re: EM Drive Developments - related to space flight applications - Thread 2
Post by: ThinkerX on 03/19/2015 04:39 am
Quote
Well, I just searched the complete advanced concepts forum and found no mention of dark matter on any EM drive developments thread, so I guess either my memory is faulty or there has been some heavy clean-up of what many consider to be "Uggy-Bogy" science. That means we won't be including it in "hypotheses discussed."

That is strange.  I remember bringing it up in the first thread.  We had to max out the amount of 'local' dark matter to get even close to the results reported by this device.  Somebody - I think it was you, Aero - proposed a sort of 'dark matter ramjet' in connection with all this, but ultimately we went in other directions.  More recently, Doctor Rodal's colleague Marshal pretty much ruled out a Dark Matter Axiom solution for the EM Drive.    Maybe the thread was edited?  In any event, Dark Matter looks to be something we pretty much rejected months ago.

 
Title: Re: EM Drive Developments - related to space flight applications - Thread 2
Post by: Stormbringer on 03/19/2015 04:47 am
Axions are not the only game in town. What about nexus gravitons? That looks like it could produce a type or ersatz thrust but it's beyond me how the RF thing would coax a high energy nexus graviton into shedding lower energy ones. If it did a thrust like thing might happen due to the space/time expansion property postulated in that article. I mean what do the things (should they even exist) interact with that an engineer can get their hands on with essentially little better than garden shed workshop level gear? Does the theory allow for EM interaction? thermal? Some sort of emergent entity like a phonon, etc?
Title: Re: EM Drive Developments - related to space flight applications - Thread 2
Post by: RotoSequence on 03/19/2015 06:26 am
Quote
Well, I just searched the complete advanced concepts forum and found no mention of dark matter on any EM drive developments thread, so I guess either my memory is faulty or there has been some heavy clean-up of what many consider to be "Uggy-Bogy" science. That means we won't be including it in "hypotheses discussed."

That is strange.  I remember bringing it up in the first thread.  We had to max out the amount of 'local' dark matter to get even close to the results reported by this device.  Somebody - I think it was you, Aero - proposed a sort of 'dark matter ramjet' in connection with all this, but ultimately we went in other directions.  More recently, Doctor Rodal's colleague Marshal pretty much ruled out a Dark Matter Axiom solution for the EM Drive.    Maybe the thread was edited?  In any event, Dark Matter looks to be something we pretty much rejected months ago.

If I recall correctly, the Dark Matter explanation was quickly ruled out in this thread, because the operating frequencies of EM drives are some of the most extensively studied portions of the electromagnetic spectrum, and, IIRC, one of the first places people looked for signs of Dark Matter's existence. If it was responsible for the functionality of EM drives at the frequencies tested, shouldn't we have seen dark matter before people started looking at EM drive?
Title: Re: EM Drive Developments - related to space flight applications - Thread 2
Post by: Mulletron on 03/19/2015 08:30 am
I promised to post some material about cavity optomechanics here:
http://forum.nasaspaceflight.com/index.php?topic=36313.msg1343469#msg1343469

This subject covers a wide swath of applications from suppressing Brownian Motion of atoms to detecting gravitational waves. I'm not concerned with most of those applications besides learning the basic concepts and applying them in the right direction.

It doesn't take too much of a stretch of the imagination to see where Emdrives could fit in with this when you consider that matter is known to couple with electromagnetic radiation on both the real and vacuum side. The transfer of momentum from the quantum vacuum interaction (which conserves momentum) has been elucidated, what is left is to find proof that this, combined with resonant cavity physics, can make something move.....a thruster.

Admittedly, it hasn't been written in the literature yet that one can build a device which moves uni-directionally using these concepts. This part of what I've been saying hasn't been written as far as I can tell, hence the wall I've hit looking for further pre-existing science. This is the holy grail that I'm searching for.

Get ready for information overload, I'm not posting everything I have, just the primers:

http://arxiv.org/abs/1303.0733 (a very broad basic overview of the field)
http://aspelmeyer.quantum.at/docs/82/downloads/revmodphys.pdf (published version of above)
http://www.cqom-itn.net/wp-content/uploads/2013/02/Diavolezza2013-Nunnenkamp.pdf (good overview)
http://authors.library.caltech.edu/32352/1/PTO000029.pdf (easy read)
http://www.umich.edu/~mctp/SciPrgPgs/events/2010/MQSS10/Talks/Stamper-Kurn_Optomechanics.pdf
http://ultracold.physics.berkeley.edu/pubs/ThierryThesis.pdf (very exhaustive thesis)
http://dash.harvard.edu/bitstream/handle/1/11158247/Woolf_gsas.harvard_0084L_10812.pdf?sequence=1 (another very exhaustive thesis)
http://www.academie-sciences.fr/activite/cr/CRPhys_thema_micro.pdf (foreword is very helpful)
http://www.cqom-itn.net/wp-content/uploads/2013/02/Diavolezza-2013-Kippenberg.pdf

Little more focused:
http://aip2014.org.au/cms/uploads/presentation/ray%20simmonds.pdf (close but no cigar)
http://phys.scichina.com:8083/sciGe/fileup/PDF/14yg2276.pdf (getting warmer)
http://arxiv.org/abs/1308.4503 (trapping dielectric spheres)

During all this research I've found that Caltech is heavily involved in cavity optomechanics, and the LIGO experiment. I suspect, that if the Emdrive effect is in fact real, it will be discovered independently by researchers like this by accident.

https://www.vahala.caltech.edu/Research/Nonlinear
https://www.vahala.caltech.edu/Research/Optomechanics
http://www.ligo.caltech.edu/



Title: Re: EM Drive Developments - related to space flight applications - Thread 2
Post by: flux_capacitor on 03/19/2015 10:10 am
        2. Classic unknown physics
            i. Casimer related
            ii. Evanescent wave related
            iii. General Relativity
        3. New Physics
            i. Quantum Vacuum -Quantum plasma theory
            ii. MiHsC theory
            iii. Dark Matter - Nullified based on insufficient density of DM.

May you modify/add:

2. Classic unknown physics
   iii. Mach effect (GR + gravinertial Wheeler-Feynman field, i.e. inertia)
and
3. New Physics
   iv. Scalar–tensor theory (GR + scalar field)

"Classic unknown physics" and "New Physics" should be under the same bullet point IMO.

As for Mach effects (M-E): Ron Stahl (https://www.linkedin.com/pulse/mach-effect-physics-conservation-concerns-3-important-ron-stahl?trk=prof-post), who is willing to replicate Woodward's experiments, stated about the EmDrive:
Quote
I would note that in order to generate thrust according to M-E theory, the dielectric really needs to act as a quarter wave mechanical/acoustic resonator. It's hard to imagine this happening by accident, but it could so act even if very imperfectly. Paul [March] is aware of all this, and has had a hand in designing the experiment, so there is a strong possibility what they're seeing is M-E thrust

As for the scalar-tensor theory of gravity, predictions (falsifiability) for the EmDrive have been calculated by Pr. Fernando Minotti in his peer-reviewed paper Scalar-tensor theories and asymmetric resonant cavities (http://arxiv.org/pdf/1302.5690v3.pdf).
Title: Re: EM Drive Developments - related to space flight applications - Thread 2
Post by: Rodal on 03/19/2015 10:44 am
......

As for the scalar-tensor theory of gravity, predictions (falsifiability) for the EmDrive have been calculated by Pr. Fernando Minotti in his peer-reviewed paper Scalar-tensor theories and asymmetric resonant cavities (http://arxiv.org/pdf/1302.5690v3.pdf).

This is the citation for the peer-reviewed version of the article:

Scalar-tensor theories and asymmetric resonant cavities
F. O. Minotti
Gravitation and Cosmology
July 2013, Volume 19, Issue 3, pp 201-208
Date: 29 Aug 2013

The author recognizes that his theory is "highly speculative" (his own words) and that "The weakest part of the theory seems to be that there is no clear way of preventing large gravitational effects due to the magnetic field of the Earth" (large gravitational effects due to the Earth's magnetic field which are contrary to present evidence)



Here is a description of this journal, from its publisher (Springer):

"Gravitation and Cosmology is a peer-reviewed periodical, dealing with the full range of topics of gravitational physics and relativistic cosmology and published under the auspices of the Russian Gravitation Society and Peoples Friendship University of Russia. The journal publishes research papers, review articles and brief communications on the following fields: theoretical (classical and quantum) gravitation; relativistic astrophysics and cosmology, exact solutions and modern mathematical methods in gravitation and cosmology, including Lie groups, geometry and topology; unification theories including gravitation; fundamental physical constants and their possible variations; fundamental gravity experiments on Earth and in space; related topics. It also publishes selected old papers which have not lost their topicality but were previously published only in Russian and were not available to the worldwide research community."

EDITOR-IN-CHIEF:
Vitaly N. Melnikov
Dr. Sci., Prof., Head of the Center for Gravitation and Fundamental Metrology, VNIIMS; and Deputy Director of the Institute of Gravitation and Cosmology of Peoples’ Friendship University of Russia (PFUR), Moscow, Russia
Title: Re: EM Drive Developments - related to space flight applications - Thread 2
Post by: Rodal on 03/19/2015 11:15 am
Re. Hypotheses looked at - where does our lengthy discussions of dark matter fit?

http://forum.nasaspaceflight.com/index.php?topic=36313.msg1347946#msg1347946 (http://forum.nasaspaceflight.com/index.php?topic=36313.msg1347946#msg1347946)
we had a lengthy discussion of dark matter and i didn't know about it?!

http://www.sciencedaily.com/releases/2015/03/150306091617.htm

http://phys.org/news/2015-03-mini-black-holes-lhc-parallel.html

Well, I just searched the complete advanced concepts forum and found no mention of dark matter on any EM drive developments thread, so I guess either my memory is faulty or there has been some heavy clean-up of what many consider to be "Uggy-Bogy" science. That means we won't be including it in "hypotheses discussed."
NASA's spaceflight internal "search" capabilities for information posted in these threads is, unfortunately, practically worthless.  It is not uncommon to search for terms and get no answers.

It is actually better to use Google to search NASA's Spaceflight forum.  For example, to search for what you are looking for, if you enter the following into Google:

EM Drive NASA Spaceflight dark matter

you will get a number of hits:



EM Drive Developments - related to space flight applications ...
forum.nasaspaceflight.com/index.php?topic...660
NASASpaceFlight.com
Feb 15, 2015 - 20 posts - ‎7 authors
But, check out the Axion Dark-Matter Experiment in the PNAS article. That is much much more sensitive than the EM Drive - they are looking for ...
160   20 posts   Nov 13, 2014
76   20 posts   Oct 5, 2014
56   20 posts   Sep 29, 2014
More results from forum.nasaspaceflight.com
Title: Re: EM Drive Developments - related to space flight applications - Thread 2
Post by: Mulletron on 03/19/2015 11:30 am
All:

Sorry I didn't make the time to participate in this ME-Drive forum for the last 6-to-8 months up, but I will try to catch up with everyone else in due course.  That said lets try to answer the questions that popped up since my morning post.

1.  I was not the lead author for the Eagleworks' 2014 AIAA/JPC paper and in fact I only supplied pictures and data for same during that period because Dr. White thought that my time was best spent in the lab gathering data instead of report writing.  Thus some of the details that Dr. Rodal is looking for may have been lost or garbled in the report writing by the others on the author list.

2.0  The thrust vector for the four resonant modes examined in detail, (the cavity's fundamental TM010, TE012, TM211 & TM212 for our copper frustum is normally in the frustum's large OD to small OD direction for most, but not all the E&M resonant modes checked.  However, one can also reverse this thrust vector for this copper frustum by just changing which excited resonant mode is used and/or mounting the dielectric discs at the large OD end of the cavity instead of the small OD end, see attached resonant mode map.  Sorry, but a one size fits all solution to this EM-Drive thrust direction is not available in this venue because of the importance of the ExB phase relationship of the expressed Lorentz forces between the excited E&M fields and the possible dielectric and QV plasma flow phenomenon that may be at work in each resonant mode expressed.  That is why this type of E&M thruster is so hard to get a handle on, for there are far too many degrees of freedom in the system to track let alone directly control.

3. The Eagleworks vacuum chamber's main body is made from 304L stainless steel while its swing out door is made from aluminum.  Most of the nuts and bolts in the vacuum chamber are also made from 18-8, 304 or 316 stainless steel alloys. 

Now to try to answer Dr. Rodal's specific questions:

"1) In the NASA experiments the truncated cone's center of  mass moved towards the [  ? ] diameter end  (where ? stands for big or small)"

For the TE012 and TM212 excited resonant modes, our copper frustum's center of mass moved toward the small OD end of the frustum when RF power was applied to the copper frustum.

"2) In the NASA experiments, we at NASA Eagleworks define the thrust force direction to be in the  [? ] direction as the movement of the truncated cone's center of  mass  (where ? stands for same or opposite)"

For just the TE012 & TM212 excited resonant modes, the thrust force direction AKA thrust vector was observed to be in the same direction as the movement of the frustum's center of mass when RF power was applied to the frustum's magnetic loop antenna.

If I missed a question along the way keep asking, but I'll be in and out of the house for the rest of the day, so I may not get to answer them until late this evening or tomorrow afternoon USA based CST.

Best, Paul M.

I understand that we need to wait for Paul March to explicitly "re-verify" the manner in which thrust reversal was achieved, but it does appear to me that he has already  addressed this in the above quote from Feb 14.  Quoting Mr. March:  "one can also reverse this thrust vector for this copper frustum by just changing which excited resonant mode is used and/or mounting the dielectric discs at the large OD end of the cavity instead of the small OD end".   To my admittedly untrained eye,  this appears to be in agreement with Dr. Rodal's earlier statements, and in conflict with Mr. Frobnicat's contention that reversal was achieved by simply rotating the entire mechanism 180 degrees.

And let's not forget about the "interesting failure" which produced thrust in the "wrong direction."
http://forum.nasaspaceflight.com/index.php?topic=36313.msg1335190#msg1335190

It would be nice to have the actual data concerning the thrust reversals without flipping the device. For me, word from the experimenter is good enough at least for now. Let's just hope this thing continues to be tested after the end of March.

I think that (until proven otherwise) the broken bolt also broke the Casimir-type geometry mentioned here:
http://journals.aps.org/prl/abstract/10.1103/PhysRevLett.96.130402

One of the things I intend to test if I ever get this copper kettle I have to move in the first place.
Title: Re: EM Drive Developments - related to space flight applications - Thread 2
Post by: Mulletron on 03/19/2015 02:46 pm
These 1" thick by 6.25" (actually 6 5/16" because I left them a little wide for milling) wide HDPE circles need a ride in a lathe. Trouble is, I don't have a lathe.

Cutting these with a jigsaw was not fun. Because the HDPE was so thick, the waste material would just bunch up around the cut in long ribbons and it was very tough to see the cut. Normally such thick material would be cut at low speed, but running the saw at full speed helped to clear out the pesky waste material so I could see better.

Rest of pics:
https://drive.google.com/folderview?id=0B4PCfHCM1KYoTXhSUTd5ZDN2WnM&usp=sharing
Title: Re: EM Drive Developments - related to space flight applications - Thread 2
Post by: Rodal on 03/19/2015 05:06 pm
These 1" thick by 6.25" (actually 6 5/16" because I left them a little wide for milling) wide HDPE circles need a ride in a lathe. Trouble is, I don't have a lathe.

Cutting these with a jigsaw was not fun. Because the HDPE was so thick, the waste material would just bunch up around the cut in long ribbons and it was very tough to see the cut. Normally such thick material would be cut at low speed, but running the saw at full speed helped to clear out the pesky waste material so I could see better.

Rest of pics:
https://drive.google.com/folderview?id=0B4PCfHCM1KYoTXhSUTd5ZDN2WnM&usp=sharing
Great idea to put the pictures in the google drive to be able to share them !

Thanks and Godspeed  :)
Title: Re: EM Drive Developments - related to space flight applications - Thread 2
Post by: Rodal on 03/19/2015 07:14 pm
Paul's statement that "For the TE012 and TM212 excited resonant modes, our copper frustum's center of mass moved toward the small OD end of the frustum when RF power was applied to the copper frustum."  Would seem to explicitly contradict the calculated chart as to the direction for those modes.

Please notice

1) The chart shows TM211 instead of TM212

2) The direction of the highest intensity Poynting vector fields is opposite in modes TM211 and TM212 (which I think should be labeled TM221 and TM222)

(http://www.blazelabs.com/pics/empropagation2.gif)

(http://wwwhome.cs.utwente.nl/~ptdeboer/ham/tn/tn07fig3.png)

https://www.youtube.com/watch?v=Hd29jEaGERk

TM211 (without a dielectric) has a clear Poynting vector field (at 45, 135, 225 and 315 degrees in the azimuthal direction) pointing towards the small base of the truncated cone. It looks like TM211 would work even without a dielectric section.

TM212 (without a dielectric) has the maximum intensity Poynting vector field group (at 45, 135, 225 and 315 degrees in the azimuthal direction) pointing towards the big base of the truncated cone.  The Poynting vector groups for TM212 seem to cancel out (by eyesight, I would have to integrate to be sure) or be slightly towards the small base.  I would not be surprised if TM212 needs a dielectric to enhance the Poynting vector (will need to confirm this conjecture by computation).


The direction of the electromagnetic vectors should be irrelevant to @Frobnicat's conjecture (*) but it is of critical important to the ExB electromagnetic Poynting vector.

I am attaching my calculations for these two modes showing that the Poynting vector fields are in opposite directions for these two modes.

EDIT: corrected some minor issues with the vector field plots on 03/23/2015

Therefore the importance of these electromagnetic modes: if the EM Drive experiences forces in opposite directions for these two modes, this imparts a death knell to any mechanistic conjecture to explain the anomalous force  (including the mechanistic conjectures of Mr. Frobnicat), as they would show that the force is electromagnetic and not mechanical in nature.



(*) @Frobnicat asked whether the direction of the electromagnetic vector should matter for heat production due to electromagnetic power dissipation.  The answer is NO !, because of the 2nd Law of Thermodynamics: a reversal of the direction of the electromagnetic field cannot possibly lead to a cooling or a thermal contraction. 
Title: Re: EM Drive Developments - related to space flight applications - Thread 2
Post by: frobnicat on 03/20/2015 01:10 am
...
(*) @Frobnicat asked whether the direction of the electromagnetic vector should matter for heat production due to electromagnetic power dissipation.  The answer is NO !, because of the 2nd Law of Thermodynamics: a reversal of the direction of the electromagnetic field cannot possibly lead to a cooling or a thermal contraction.

Never thought about cooling for even one second. That's not about direction (toward or away, this is AC anyway so symmetrically going back and forth), I was caring about the vector field going back and forth tangent to surface or normal to surface, as far as heating magnitude is concerned. And your reply was very clear and helpful, thank you.

What do you think of the formula used in the plot titled "S21 and surface integral of cavity with 1W input power" (http://forum.nasaspaceflight.com/index.php?action=dlattach;topic=36313.0;attach=778912;image) ? There appear to be no cross-product but a sum of e and b(h actually) fields' energy densities taken separately. Is it equivalent ? It looks to me like a piston model with electromagnetic energy considered pushing like pressure against displacement along the axis of the frustum (Z in the expression). There is electromagnetic pressure of course, but isn't it supposed to integrate to 0, at least in known frameworks ? Do you see what motivates this particular formula ?

Anyway, factually, using Eagleworks mode nomenclature for convenience, the reported experiments of "anomalous thrust..." show both TE012 and TM211 modes in the same direction (the small end). And the blue plot says it should really reverse between TE012 and TM211. I won't go into battle as saying that it definitely - how you say that ? ah yes - imparts a death knell to this blue plot, but those initial results do weaken the theoretical statement, to put that in diplomatic terms.
Title: Re: EM Drive Developments - related to space flight applications - Thread 2
Post by: Rodal on 03/20/2015 03:09 pm
...
What do you think of the formula used in the plot titled "S21 and surface integral of cavity with 1W input power" (http://forum.nasaspaceflight.com/index.php?action=dlattach;topic=36313.0;attach=778912;image) ?
....
Do you see what motivates this particular formula ?
...
Concerning the equation used in http://forum.nasaspaceflight.com/index.php?action=dlattach;topic=36313.0;attach=778912;image, the discussion should preferably be carried on the following basis:

1) Preferably the author of the equation should explain what is the background, context and significance of the equation and the assumptions under which it holds. 

2) The peers should review the equation (and the assumptions under which it holds).  Review means "the equation appears in the following references" or "the equation cannot be found in the existing literature".  Review means "the equation conflicts with experimental knowledge and/or accepted scientific principles and/or scientific logic etc."

While waiting for Paul March (@Star-Drive) to explain the background, context and significance of the equation, the following is apparent:

The terms in the  http://forum.nasaspaceflight.com/index.php?action=dlattach;topic=36313.0;attach=778912;image equation coincide with the Kronecker's delta (the unit dyadic) negative terms of Maxwell's stress tensor ("times the normal component in the z direction").  The remaining dyadic terms of Maxwell's stress tensor component are missing from this expression. 

See:

 Minkowski, H. (1908). "Die Grundgleichungen für die elektromagnetischen Vorgänge in bewegten Körpern". Nachrichten von der Gesellschaft der Wissenschaften zu Göttingen, Mathematisch-Physikalische Klasse: 53–111.

http://en.wikisource.org/wiki/Translation:The_Fundamental_Equations_for_Electromagnetic_Processes_in_Moving_Bodies

 Abraham, M. (1909). "Zur Elektrodynamik bewegter Körper". Rendiconti del Circolo Matematico di Palermo 28: 1–28. doi:10.1007/bf03018208.

http://en.wikisource.org/wiki/Translation:On_the_Electrodynamics_of_Moving_Bodies_%28Abraham%29

There is electromagnetic pressure of course, but isn't it supposed to integrate to 0, at least in known frameworks ?

What should integrate to zero (for no external forces, assuming a closed system of conservative forces) is the divergence of Maxwell's stress tensor minus the time derivative of Poynting's vector.  The equation in question does not include all the terms in Maxwell's stress tensor (it lacks the dyadic product of E (or D) with E( or D) and the dyadic product of B (or H) with B (or H)), these missing terms have a different sign than the included terms, so no, there is no reason why the posted equation should integrate to zero (and what should integrate to zero anyway is the divergence of Maxwell's stress tensor minus the time derivative of Poynting's vector).

There appear to be no cross-product but a sum of e and b(h actually) fields' energy densities taken separately. Is it equivalent ?

You maybe referring to equivalence of scalar components, for example, but there is no such apparent equivalence here either (for the equation in question and the scalar terms of Poynting's vector).

---

Poynting's vector is the cross-product of E (or D) and B (or H): it is indeed a vector (a first order tensor).  Maxwell's stress tensor is a 2nd order tensor involving dyadic product of base vectors. 
A vector has a scalar component (its magnitude) and a direction (given by the base vectors).
A second order tensor has scalar terms, vector terms (perpendicular to the faces of the unit dyadic), and a dyadic basis (the dyadic product of the base vectors).

Strictly speaking, a vector cannot be equivalent to a  2nd order tensor anymore than a scalar can be equivalent to a vector.  The divergence of Maxwell's stress tensor is a vector that is not equivalent to Poynting's vector.  Actually, the divergence of Maxwell's stress tensor minus the time derivative of Poynting's vector gives the electromagnetic force.  Strictly speaking, none of the above are equivalent (different quantities: different scalars, different vectors and a second order tensor: Maxwell's stress tensor). 

Title: Re: EM Drive Developments - related to space flight applications - Thread 2
Post by: Rodal on 03/20/2015 04:14 pm
....
Anyway, factually, using Eagleworks mode nomenclature for convenience, the reported experiments of "anomalous thrust..." show both TE012 and TM211 modes in the same direction (the small end). And the blue plot says it should really reverse between TE012 and TM211. I won't go into battle as saying that it definitely - how you say that ? ah yes - imparts a death knell to this blue plot, but those initial results do weaken the theoretical statement, to put that in diplomatic terms.
But this thread should really be focused on:

EM Drive Developments - related to space flight applications

A discussion about EM Drive experiments to try to ascertain their validity and significance for space flight applications.

1) Experiments (like reversing at will the direction of the thrust from the small base to the big base directions) that would nullify yet another mechanism (like the one you proposed) pretending to show the EM Drive experiment as an artifact, are very important in that they advance our state of knowledge about "EM Drive Developments - related to space flight applications."

On the other hand, discrepancies in sign between a blue plot and other plots, do not rise to the same level of significance.

One explanation, for example, could be simply that one (or both) of the plots have the wrong sign. Such errors in presentation do not rise to the same level of significance.

Do you know how many plots and equations contain errata in Feynman's original QED and path integral publications? 

Ultimately what matters is whether the EM Drive experiments show phenomena that can be used for spaceflight applications, and ultimately what will matter least is whether plots were mislabeled.

When finding a contradiction between plots, one should not rush to judgement to toll the bell that the experiment is an artifact: it may simply be an innocent error in labeling a plot.



2) That there may be labeling questions is shown by:

2a) What NASA Eagleworks labels as mode TM212 I have shown should be labeled TM222

2b) The mode labeled as TM211 in the "Anomalous ..." report was reported to occur at 1.9326 GHz and 1.9367
 GHz, with COMSOL FEA frequency calculated at 1.947 GHz.  Yet now they report mode labeled TM212 to be tested and to occur at about the same frequency range.
I calculate that the mode labeled "TM212" (which I think should be labeled TM222) -which does not appear in http://forum.nasaspaceflight.com/index.php?action=dlattach;topic=36313.0;attach=778912;image should occur at a significantly higher frequency than TM211, never at the same frequency ( "TM212" at 2.49 GHz without dielectric and "TM211" at 2.01 GHz without dielectric).  These modes were analyzed and labeled (I understand) by different engineers. It is possible that they may be one and the same mode that has been mislabeled.




3)All the emphasis you place on the signs on the blue NASA Eagleworks plot and the other plots (which could be just an error in labeling the plots), yet you ignore the fact that Roger Shawyer published experimental information showing his EM Drive Demonstrator Engine showed practically the same magnitude thrust was experimentally observed to be reversed: Shawyer reports forces in opposite directions (towards the big end and towards the small end of practically the same magnitude: 214 mN/kW and 243 mN/kW) for his Demonstrator engine.

The data being ignored (showing  forces in opposite directions, towards the big end and towards the small end of practically the same magnitude), what should I say  :) ,  oh yes,  defenestrates, mechanical explanations for the EM Drive measurements being a mechanical artifact.

(http://forum.nasaspaceflight.com/index.php?action=dlattach;topic=36313.0;attach=776944;image)
Title: Re: EM Drive Developments - related to space flight applications - Thread 2
Post by: frobnicat on 03/20/2015 11:14 pm
Thank for later part of previous answer. It does answer clearly to some aspects of my innocent questions, for the other aspects I'll have to learn German. Thank for editing the beginning. Unfortunately the tone is becoming a bit harsh so it becomes difficult to talk casually about just, you know, "what you think of ... patati patata", without resorting to lawyers. So I will stop asking casual questions.

...

1) Experiments (like reversing at will the direction of the thrust from the small base to the big base directions) that would nullify yet another mechanism (like the one you proposed) pretending to show the EM Drive experiment as an artifact, are very important in that they advance our state of knowledge about "EM Drive Developments - related to space flight applications."

Yes. The problem is that we have no experimental data (from Eagleworks) concerning such EM mode induced reversal.

Quote
On the other hand, discrepancies in sign between a blue plot and other plots, do not rise to the same level of significance.

If the discrepancies prevent to ascertain or leave ambiguity about the the experimental existence of reversal, then it becomes significant.

Quote
One explanation, for example, could be simply that one (or both) of the plots have the wrong sign. Such errors in presentation do not rise anywhere to the same level of significance.

The same blue plot is up for TE modes and down for TM modes. The sign changes in function of mode in the theoretical prediction (never mind up and down, a change of orientation is what is expected) and don't change in function of mode in the experiments.

Absence of proof is not proof of absence : that's why I say it is not a death blow to the theoretical formula used to get this plot. The magnitude also, of 10µN/W up or 20µN/W down is much higher than those recorded. That is also not by itself a nullification of the formula. It could be that the theory behind the formula is ok but that the exciting frequency is not right on peak, or too spread, or not enough spread. Very well. But excuse me, at some point, one is to say the case for the formula is weakened. Not more, not less. If saying that, after spending quite a lot of time of reading and checking the publicly available data, is immaterial, then all what we say on this forum is immaterial, and we should then all shut up and wait the next peer reviewed publication of those research.

I'm a very reasonable person. I do approach the thrust anomaly with a sceptical eye, but I wouldn't be here in the first place if I hadn't a little excitation at the prospect that it might be worth a discovery (of new physics or new possibility inside known physics). Most sceptical people hearing about the anomaly will dismiss the case on short notice, and having forged an opinion that it must be an artefact just plain don't care.

Sceptics that do care like me don't even require the EM thrust to exhibit reverse modes. It could be (in the hypothesis that there is indeed a thrust) that the still unknown principle of operation involved prevents any possibility of thrust toward the big end of the asymmetrical cavity (when it is a frustum). If the effect is checked on other scales, at other labs, is phenomenologically consistent and reproducible, all people that do care (sceptics and enthusiasts along) will be very pleased, even if there is no reverse mode. If it does show mode reversal, all the better, mechanistic artefact hypothesis (as unique source of signal) defenestrate right now. Other artefacts might hold. If it don't reverse, well then it don't reverse, and then neither EM thrust nor purely mechanistic explanations are ruled out, yet. Isn't it reasonable ?

Quote
Do you know how many plots and equations contain errata in Feynman's original QED and path integral publications? 

No I don't. But I'm sure that it attracted a lot more competent and sceptical peers and third parties to get involved than the experiments at Eagleworks so far.

Quote
Ultimately what matters is whether the EM Drive experiments show phenomena that can be used for spaceflight applications, and ultimately what will matter least is whether plots were mislabeled.

Showing a phenomena requires a minimum of care in the communication of the results. I find the communication from Eagleworks is reasonably clear and consistent most of the time, but unfortunately a bit scarce on some data, we would really like to see the "negligible thrusts" charts of tests without dielectric, as this could by itself be enough to "slaughter" the mechanistic thermal hypothesis. Also it would have been nice that the report gave the "detail" that the horizontal pendulum wasn't quite horizontal after all and that the tilt in axis played a role in stabilizing the equilibrium rest position. I doubt that without my attempts at clarifying the mechanical aspects of the experiment and my attention to "insignificant" problems of labels and scale we would have this little piece of extra info, with due thanks to Paul March for answering with clarity to my request on that (and taking time to check and take pictures, unfortunately hardly workable due to parallax). Before savagely assassinating the competing artefactual hypothesis shouldn't we be eager to learn all the gory details below the signal ?

Ultimately, if 10 years from now, the EM thrust is still in a state of indetermination, with people believing in real EM thrust in spite of inconsistencies of EM thrust signals in one camp, and people believing in artefactual explanations in spite of inconsistencies wrt some experimental results in the other camp, science (and spaceflight) is not the winner. The EM drive effect could float a long time in such a state of indetermination before it floats in space.

Quote
When finding a contradiction between plots, one should not rush to judgement to toll the bell that the experiment is an artifact: it may simply be an innocent error in labeling a plot.

So when finding a contradiction between a WIP classical explanation and some interpretation of ambiguous statements (or the reverse) about the results, one should jump to the throat of the classical explanation...

If I really wanted to pinpoint and quibble miserably on insignificant label contradiction, long time I would have raised the fact that the file name (in the picture, the Browse box of the display) in one of the charts (http://forum.nasaspaceflight.com/index.php?action=dlattach;topic=36313.0;attach=634625;image) states 3 10^-3 Torr when the picture is commented 5 10^-6 Torr, and that in another one (hot topic recently) (http://forum.nasaspaceflight.com/index.php?action=dlattach;topic=36313.0;attach=634634;image) we have "In-Air" vs 5 10^-4 Torr. As anyone who saw that apparent disparity, I just classified that in the "innocent error", probably that was the previous file recorded (before the new conditions). That is insignificant and mean. Can do that. Significance of other discrepancies I raised and put emphasis on, you are not the only judge.

Quote


2) That there may be labeling questions is shown by:

2a) What NASA Eagleworks labels as mode TM212 I have shown should be labeled TM222

2b) The mode labeled as TM211 in the "Anomalous ..." report was reported to occur at 1.9326 GHz and 1.9367
 GHz, with COMSOL FEA frequency calculated at 1.947 GHz.  Yet now they report mode labeled TM212 to be tested and to occur at about the same frequency range.
I calculate that the mode labeled "TM212" (which I think should be labeled TM222) -which does not appear in http://forum.nasaspaceflight.com/index.php?action=dlattach;topic=36313.0;attach=778912;image should occur at a significantly higher frequency than TM211, never at the same frequency ( "TM212" at 2.49 GHz without dielectric and "TM211" at 2.01 GHz without dielectric).  These modes were analyzed and labeled (I understand) by different engineers. It is possible that they may be one and the same mode that has been mislabeled.

Yes, all right, sounds possible. So ? There is no clear experimental data showing thrust toward big end, thermal is not dead, EM thrust is not dead, blue plot is not dead but weakened (not all mode tested but no reversal so far TE vs TM as seen in the plot)

Quote

3)All the emphasis you place on the signs on the blue NASA Eagleworks plot and the other plots (which could be just an error in labeling the plots), yet you ignore the fact that Roger Shawyer published experimental information showing his EM Drive Demonstrator Engine showed practically the same magnitude thrust was experimentally observed to be reversed: Shawyer reports forces in opposite directions (towards the big end and towards the small end of practically the same magnitude: 214 mN/kW and 243 mN/kW) for his Demonstrator engine.

The data being ignored (showing  forces in opposite directions, towards the big end and towards the small end of practically the same magnitude), what should I say  :) ,  oh yes,  defenestrates, mechanical explanations for the EM Drive measurements being a mechanical artifact.


Excuse me but when I see how Shawyer treats the sign conventions with his "special Newtons" that do accelerate (but the other way) and don't know how to push on a spring, I wouldn't care defenestrating any attempt at explaining that classically. Why do we spend so much time here on Eagleworks results that are 2 orders of magnitude below the µN/W yields of Shawyer ? That was your own interrogation (long ago, thread 1). Maybe because the work at Eagleworks is much more transparent, is in vacuum, is rather clearly communicated, gives a better impression of reliability overall ?

I won't trash a mechanical explanation specific to Eagleworks' balance in vacuum just to be consistent with Shawyer's partially known apparatus and inconsistent sign conventions in results reporting. Hell, at 200µN/W how Shawyer hasn't succeeded in convincing the 3 or 4 hard headed sceptics needed to get a chain reaction of replications all over the world ? Wake up, it's reality we are talking about. Those things have been around for years and years, and always failed to get ... momentum.

Also if I can't put emphasis on what IMO I find weak in the publicly available data, then end of discussion. I don't deter you from putting emphasis on whatever you see fit. If some of my emphasis points seem insignificant to you, don't answer them.

Best
Title: Re: EM Drive Developments - related to space flight applications - Thread 2
Post by: Rodal on 03/20/2015 11:52 pm

Quote


2) That there may be labeling questions is shown by:

2a) What NASA Eagleworks labels as mode TM212 I have shown should be labeled TM222

2b) The mode labeled as TM211 in the "Anomalous ..." report was reported to occur at 1.9326 GHz and 1.9367
 GHz, with COMSOL FEA frequency calculated at 1.947 GHz.  Yet now they report mode labeled TM212 to be tested and to occur at about the same frequency range.
I calculate that the mode labeled "TM212" (which I think should be labeled TM222) -which does not appear in http://forum.nasaspaceflight.com/index.php?action=dlattach;topic=36313.0;attach=778912;image should occur at a significantly higher frequency than TM211, never at the same frequency ( "TM212" at 2.49 GHz without dielectric and "TM211" at 2.01 GHz without dielectric).  These modes were analyzed and labeled (I understand) by different engineers. It is possible that they may be one and the same mode that has been mislabeled.

Yes, all right, sounds possible. So ?...
So we have to wait for either Paul March or another report from NASA Eagleworks to clear up this issue of where is the experimental data (position vs. time) for the truncated cone exhibiting a force (and acceleration) directed towards the big end.  Are some of the plots in this thread experiments  exhibiting a force (and acceleration) directed towards the big end ? At what frequencies and mode shapes ?  Which plots (if any)  exhibited a force (and acceleration) directed towards the big end due to the placement of the dielectric in the opposite end?

If none of the plots examined in this thread exhibited a force (and acceleration) directed towards the big end...do such plots exist? Can they make them available?

Or perhaps we just have to wait for @Mulletron and @Notsosureofit's experiments to clear up this matter  :)

Title: Re: EM Drive Developments - related to space flight applications - Thread 2
Post by: Rodal on 03/21/2015 12:03 am
Thank for later part of previous answer. It does answer clearly to some aspects of my innocent questions, for the other aspects I'll have to learn German....
No need to learn German.  I included the English translation links  (by the way, these are excellent translations), which I copy again here:

See:

 Abraham, M. (1909).

http://en.wikisource.org/wiki/Translation:On_the_Electrodynamics_of_Moving_Bodies_%28Abraham%29

 Minkowski, H. (1908).

http://en.wikisource.org/wiki/Translation:The_Fundamental_Equations_for_Electromagnetic_Processes_in_Moving_Bodies

These are excellent papers.  The paper by Abraham is very comprehensive (it includes a review of efforts by Minkowski, Lorentz, Einstein, Hertz, etc.). 

Title: Re: EM Drive Developments - related to space flight applications - Thread 2
Post by: Rodal on 03/21/2015 12:53 am
Thank for later part of previous answer. It does answer clearly to some aspects of my innocent questions, for the other aspects I'll have to learn German. Thank for editing the beginning. Unfortunately the tone is becoming a bit harsh so it becomes difficult to talk casually about just, you know, "what you think of ... patati patata", without resorting to lawyers....
Thanks for the (yes, strong, but very) illuminating exchange.  I learned many aspects of the Eagleworks experiments that I had not realized before you brought them up to our attention, and I better understand your viewpoint.   Thanks for your patience and for taking your time to explain this :)
Title: Re: EM Drive Developments - related to space flight applications - Thread 2
Post by: Flyby on 03/21/2015 11:29 am
Such a pity there is - currently- no way to dramatically increase the power input on the eagleworks setup.
It would be so much easier to have clear significant output results, be them positive or negative.

The lack of clear results makes us spend tremendous time on debating and arguing about measurement procedures, measurement errors and anomalies, etc.
Simple because it is so hard to separate the current results from potential background noise and secondary effects...

If only some one, besides the Chinese and Shawyer would have the ability to either prove or disprove the observed thrust effect, by simply putting 2-3kW instead of 50-100w.
It would at least generate enough interest AND funds to continue fundamental research.
once you develop 720mN of thrust all discussions about thermal or other secondary effects fall away, that much is clear to me.

What I fear is that due to lacking a clear result, research on this will get marginalized and slide into the "yet another crackpot theory". Not that I particularly biased for this to be successful, but I do find previous results intriguing enough to see it continued.

Judging on what Shawyer is currently trying to develop, i think that is exactly what he's trying to do : to produce such results that it can no longer be dismissed as being an error in measurement, or a secondary effect. But we need it to be repeatable...

And even if it fails to produce anything substantial,  then we'll all be able to put it finally aside as a wrongly tracked scientific experiment. No real harm done there, except from a few bruised ego's...
Title: Re: EM Drive Developments - related to space flight applications - Thread 2
Post by: Rodal on 03/21/2015 12:07 pm
...
What do you think of the formula used in the plot titled "S21 and surface integral of cavity with 1W input power" (http://forum.nasaspaceflight.com/index.php?action=dlattach;topic=36313.0;attach=778912;image) ?
....
Do you see what motivates this particular formula ?
...
....
While waiting for Paul March (@Star-Drive) to explain the background, context and significance of the equation, the following is apparent:

The terms in the  http://forum.nasaspaceflight.com/index.php?action=dlattach;topic=36313.0;attach=778912;image equation coincide with the Kronecker's delta (the unit dyadic) negative terms of Maxwell's stress tensor ("times the normal component in the z direction").  The remaining dyadic terms of Maxwell's stress tensor component are missing from this expression. 

....
Wikipedia has a pretty good article on Maxwell's stress tensor, see:  http://en.wikipedia.org/wiki/Maxwell_stress_tensor

See the expression for the electromagnetic force per unit volume f at the very end of point 4 of the Motivation section http://en.wikipedia.org/wiki/Maxwell_stress_tensor#Motivation, just before the start of point 5 (which I reproduce with symbols as an attachment below -too bad that NASA Spaceflight does not allow one to use mathematical symbols in these posts  :(  -):

\mathbf{f} = \epsilon_0\left[  (\boldsymbol{\nabla}\cdot \mathbf{E} )\mathbf{E} + (\mathbf{E}\cdot\boldsymbol{\nabla}) \mathbf{E} \right] + \frac{1}{\mu_0} \left[(\boldsymbol{\nabla}\cdot \mathbf{B} )\mathbf{B} + (\mathbf{B}\cdot\boldsymbol{\nabla}) \mathbf{B} \right] - \frac{1}{2} \boldsymbol{\nabla}\left(\epsilon_0 E^2 + \frac{1}{\mu_0} B^2 \right)
- \epsilon_0\frac{\partial}{\partial t}\left( \mathbf{E}\times \mathbf{B}\right).

Observe that:

1) All the terms containing the divergence (nabla) of vector field B vanish (because there are no magnetic monopoles in Maxwell's equations).

2) All the terms containing the divergence (nabla) of vector field E vanish (assuming that there are no point charges inside the microwave cavity).

3) The term epsilono*d(ExB)dt is the time derivative of Poynting's vector.  Since Poynting's vector time dependence is (Cos[omega*t])^2, the time dependence of its time derivative is proportional to Sin[omega*t], whose average over a complete cycle is zero, so this term (the time derivative of Poynting's vector) also vanishes over a complete cycle.

So, all one is left for the electromagnetic force vector per unit volume f in the microwave cavity is the term:

f = - (1/2) Nabla (epsilono E^2 + (1/muo) B^2 )

So, this term is the negative divergence of the expression in the http://forum.nasaspaceflight.com/index.php?action=dlattach;topic=36313.0;attach=778912;image plot of NASA Eagleworks:


(epsilono E^2 + (1/muo) B^2 )

(where the scalar expression E^2 =( realPart[Ex] )^2 + (imaginaryPart[Ex]) ^2 +  realPart[Ey] )^2 + (imaginaryPart[Ey]) ^2 +  realPart[Ex] )^2 + (imaginaryPart[Ez]) ^2 ) is the AbsoluteValue of the complex expression for E; and ditto for the scalar expression B)

(http://forum.nasaspaceflight.com/index.php?action=dlattach;topic=36313.0;attach=778913;image)

Since, due to conservation of momentum, the electromagnetic force in the microwave cavity should be zero, then


f = 0 =  - (1/2) Nabla (epsilono E^2 + (1/muo) B^2 )

which means that the divergence ("Nabla") of the expression in NASA Eagleworks http://forum.nasaspaceflight.com/index.php?action=dlattach;topic=36313.0;attach=778912;image  plot should be zero.

This still allows  the expression in NASA Eagleworks http://forum.nasaspaceflight.com/index.php?action=dlattach;topic=36313.0;attach=778912;image  itself to be non-zero, as long as the sum of its spatial derivatives (due to the divergence operator) are zero.
Title: Re: EM Drive Developments - related to space flight applications - Thread 2
Post by: Rodal on 03/21/2015 12:49 pm
For yet another related interpretation, see the article "Electromagnetic stress–energy tensor" http://en.wikipedia.org/wiki/Electromagnetic_stress%E2%80%93energy_tensor.

"In relativistic physics, the electromagnetic stress–energy tensor is the contribution to the stress–energy tensor due to the electromagnetic field."

The expression in NASA Eagleworks http://forum.nasaspaceflight.com/index.php?action=dlattach;topic=36313.0;attach=778912;image  is (the energy density): the first diagonal entry T11, purely due to the time coordinate , of the 3+1 spacetime  relativistic-physics electromagnetic stress–energy tensor  (in free space and flat space–time), (the other entries comprise Maxwell's stress tensor sigma in the lower sub-quadrant (these Maxwell's stress tensor sigma components are only related to the space coordinates)  and Poynting's vector S divided by the speed of light c, as the remaining off-diagonal entries due to time and space coupling).


relativistic-time component ==>  energy density in NASA Eagleworks http://forum.nasaspaceflight.com/index.php?action=dlattach;topic=36313.0;attach=778912;image 

relativistic-space components==> Maxwell's stress tensor sigma

coupling of relativistic-space with relativistic-time components ==> Poynting's vector components S/c

(http://upload.wikimedia.org/math/9/0/6/9067342a3c3e13deacfc7cded6b5da36.png)

(http://forum.nasaspaceflight.com/index.php?action=dlattach;topic=36313.0;attach=778913;image)

Still, the conservation laws, comprise the divergence of this relativistic electromagnetic stress–energy tensor, see

http://en.wikipedia.org/wiki/Electromagnetic_stress%E2%80%93energy_tensor#Conservation_laws

As to @frobnicat's question regarding "motivation" what I see here is either


1) A highly conceptualized, MagnetoHydroDynamics model of the Quantum Vacuum interaction for flat-space time as per Harold White's model (which he considers defensible as an engineering computational tool only for modeling purposes).

2) Allowing a non-flat spacetime, another motivation maybe a coupling of electromagnetism to gravity, see for example:

http://physics.stackexchange.com/questions/55660/einstein-field-equations-and-electromagnetic-stress-energy-tensor

and

http://en.wikipedia.org/wiki/Einstein_field_equations#Einstein.E2.80.93Maxwell_equations
Title: Re: EM Drive Developments - related to space flight applications - Thread 2
Post by: Rodal on 03/21/2015 02:58 pm
Still, another thing for Paul March/NASA Eagleworks to explain is, in the expression in NASA Eagleworks http://forum.nasaspaceflight.com/index.php?action=dlattach;topic=36313.0;attach=778912;image, which also  is (the energy density): the first diagonal entry T11 in

(http://upload.wikimedia.org/math/9/0/6/9067342a3c3e13deacfc7cded6b5da36.png)


and taking the square of the AbsoluteValue E^2 and B^2 are necessarily positive scalar expressions:

epsilono* E^2+(1/muo)*B^2 >= 0

where does the change in sign come from for different modes?

Did they calculate the square of the Absolute Value for each electromagnetic field component:

(RealPart[Ex])^2+(ImaginaryPart[Ex])^2 (which is necessarily always positive)

or did they calculate instead

RealPart[Ex^2]+ImaginaryPart[Ex^2] (which can be negative)

and if so, why did they calculate RealPart[(Ex]^2]+ImaginaryPart[(Ex]^2]  instead of the AbsoluteValue?



This is an issue of the order in which mathematical expressions should be interpreted:

I don't know what code did they use for the expression  http://forum.nasaspaceflight.com/index.php?action=dlattach;topic=36313.0;attach=778912;image,

but is there a code where the expression can be interpreted as

RealPart[Ex^2]+ImaginaryPart[Ex^2] +.... (which can be negative)

instead of

(RealPart[Ex])^2+(ImaginaryPart[Ex])^2 +... (which is necessarily always positive) ?
Title: Re: EM Drive Developments - related to space flight applications - Thread 2
Post by: Notsosureofit on 03/21/2015 04:05 pm
The +/- would seem to come from the multiplication of the surface energy by the z component of the (surface) normal vector.  Why ?  Beyond me ?
Title: Re: EM Drive Developments - related to space flight applications - Thread 2
Post by: Rodal on 03/21/2015 05:31 pm
The image  http://forum.nasaspaceflight.com/index.php?action=dlattach;topic=36313.0;attach=778912;image is dated 05/23/2014 but the image does not appear in the July 28-30, 2014 (not enough time to go to press), "Anomalous Thrust Production from an RF Test Device Measured on a Low-Thrust Torsion Pendulum" paper.


The text of the image reads:  "Surface integral (numeric) of this expression:.....This is the integral of the sum of the magnetic and electric field energies multiplied by the z component of the normal vector at that point"

The surface integral? the integral over what surface?  The total surface of the truncated cone?

What normal vector at what point?  The vector normal to the  truncated cone surface? No, because the geometry is the same for different modes, so that cannot change for different modes.

The normal vector of the Electric field for a TM mode? the normal vector of the Magnetic field for a TE mode? Or vice-versa ?  Why ?  Over what surface(s) and why?

What is the meaning of this integral anyway?  It is not a force.

EDIT: As @frobnicat correctly points out, this surface integration has the units of a force, but what I mean by "it is not a force" is that the electromagnetic force should be the divergence of the expression used by NASA Eagleworks in this image, and they did not take the divergence, which at first glance does not appear correct: since if one does not take the divergence of the expression, one is not satisfying conservation of energy/momentum

The electromagnetic force is related to the divergence of these expressions.  The divergence operation is not taken...

For that component, T11, the divergence derivative is with respect to the relativistic time coordinate, (technically speaking, it is the spacetime contravariant rate of the energy density) so the electromagnetic force for that component is related to the derivative with respect to time of the energy density expression.  But no time derivative is shown to us.  But if the electromagnetic components are a harmonic function of time, over a complete cycle, that time derivative would give zero, anyway.




Title: Re: EM Drive Developments - related to space flight applications - Thread 2
Post by: Notsosureofit on 03/21/2015 05:45 pm
Sounds to me kind of like:

 "Surface integral (numeric) of this expression:.....This is the integral of the sum of the magnetic and electric field surface energy densities multiplied by the z component of the vector normal to the  truncated cone surface at that point"

  The vector normal to the  truncated cone surface does not change for different modes.

That's what it sounds like anyway.................
Title: Re: EM Drive Developments - related to space flight applications - Thread 2
Post by: Rodal on 03/21/2015 05:52 pm
Sounds to me kind of like:

 "Surface integral (numeric) of this expression:.....This is the integral of the sum of the magnetic and electric field surface energy densities multiplied by the z component of the vector normal to the  truncated cone surface at that point"

  The vector normal to the  truncated cone surface does not change for different modes.

That's what it sounds like anyway.................

So what is missing here to get a force is coupling: for example coupling to the Quantum Vacuum in a MagnetoHydroDynamics model, as per H. White.  Coupling to an external field (and non-zero differentiation with respect to time of the energy density shown) are necessary to get a force. 

Well,  one probable answer will be: it is all in the dielectric PE for it to produce such force ;

non-harmonic change of the electromagnetic density with respect to time ? (so that the time derivative does not have a period over which it averages zero value?)

Will they claim a nonlinear non-harmonic non-periodic response in the dielectric coupling with the Quantum Vacuum such that the time derivative of the energy density does not have a zero time average?

The image below (described this time as a volumetric force integral)  looks nonlinear or non-harmonic non-periodic:

(http://forum.nasaspaceflight.com/index.php?action=dlattach;topic=36313.0;attach=636339;image)
Title: Re: EM Drive Developments - related to space flight applications - Thread 2
Post by: frobnicat on 03/21/2015 10:55 pm
Sounds to me kind of like:

 "Surface integral (numeric) of this expression:.....This is the integral of the sum of the magnetic and electric field surface energy densities multiplied by the z component of the vector normal to the  truncated cone surface at that point"

  The vector normal to the  truncated cone surface does not change for different modes.

That's what it sounds like anyway.................

If "sum of the magnetic and electric field surface energy densities" (a scalar) is replaced by "pressure" (as scalar pressure in ideal gas at thermal equilibrium) then the formula would read as the surface integral of the pressure multiplied by the z component of the vector normal to the  truncated cone surface at that point. Pressure is N/m² scalar, integrated over a surface yields N. The normal_z factor (dimensionless) leaves only the z component of the force at each point.  Over a patch of surface the integral result would be the z component of the force exerted by such gas on that patch. For the inner surface of a closed cavity filled by a gas at thermal equilibrium and at rest (and without pressure gradient due to gravity or acceleration), that should integrate to 0.

If this analogy holds, the units of "the sum of the magnetic and electric field surface energy densities" should be like a pressure, or equivalently energy volumetric density, in N/m² or equivalently J/m^3 (passing by Nm/m^3 and Nm is J).

Maybe this is more clear to understand "volumetric energy densities (in J/m^3) taken at the surface" rather than "surface energy densities" as the later seems to imply surface densities (in J/m²).

1) Integrating J/m^3 over a surface gives N (m²*J/m^3 -> J/m -> Nm/m -> N)
2) Integrating J/m^2 over a surface gives J, equivalent to Nm, not to N

Only 1) makes the formula dimensionally consistent (to give N in the end)

My musings, can't go much further, why I asked dr Rodal his views on the matter. Latest posts are quite readable even when lacking any decent basis with electromagnetic stress-energy tensors.
Title: Re: EM Drive Developments - related to space flight applications - Thread 2
Post by: ThinkerX on 03/21/2015 11:11 pm
Doctor McCulloch put forth another blog post on the EM Drive, using a 'one wave' version of his theory.  By and large, his predictions with this new model seem much closer to the mark than before.  There are a few tests he put in red because of uncertainty over the geometry.

http://physicsfromtheedge.blogspot.com/

At times, it seems almost like this device operates in the murky area between gravity and electromagnetic radiation.
Title: Re: EM Drive Developments - related to space flight applications - Thread 2
Post by: Rodal on 03/21/2015 11:31 pm
Sounds to me kind of like:

 "Surface integral (numeric) of this expression:.....This is the integral of the sum of the magnetic and electric field surface energy densities multiplied by the z component of the vector normal to the  truncated cone surface at that point"

  The vector normal to the  truncated cone surface does not change for different modes.

That's what it sounds like anyway.................

If "sum of the magnetic and electric field surface energy densities" (a scalar) is replaced by "pressure" (as scalar pressure in ideal gas at thermal equilibrium) then the formula would read as the surface integral of the pressure multiplied by the z component of the vector normal to the  truncated cone surface at that point. Pressure is N/m² scalar, integrated over a surface yields N. The normal_z factor (dimensionless) leaves only the z component of the force at each point.  Over a patch of surface the integral result would be the z component of the force exerted by such gas on that patch. For the inner surface of a closed cavity filled by a gas at thermal equilibrium and at rest (and without pressure gradient due to gravity or acceleration), that should integrate to 0.

If this analogy holds, the units of "the sum of the magnetic and electric field surface energy densities" should be like a pressure, or equivalently energy volumetric density, in N/m² or equivalently J/m^3 (passing by Nm/m^3 and Nm is J).

Maybe this is more clear to understand "volumetric energy densities (in J/m^3) taken at the surface" rather than "surface energy densities" as the later seems to imply surface densities (in J/m²).

1) Integrating J/m^3 over a surface gives N (m²*J/m^3 -> J/m -> Nm/m -> N)
2) Integrating J/m^2 over a surface gives J, equivalent to Nm, not to N

Only 1) makes the formula dimensionally consistent (to give N in the end)

My musings, can't go much further, why I asked dr Rodal his views on the matter. Latest posts are quite readable even when lacking any decent basis with electromagnetic stress-energy tensors.
I agree, you are correct that it has the units of a force.
But the electromagnetic force should be the divergence of the expression used by NASA Eagleworks in that image (including other terms that may be zero because div B = 0 and div E = 0 and the time rate of Poyinting's vector is zero for harmonic electromagnetic standing waves), and they did not take the divergence, which at first glance does not appear correct: they would not be calculating the correct force: since if one does not take the divergence of the expression, one is not satisfying conservation of energy/momentum


http://en.wikipedia.org/wiki/Electromagnetic_stress%E2%80%93energy_tensor#Conservation_laws



Unless they assume some non-conventional time-dependence for the electromagnetic waves such that the time derivative equals the original function.  Essentially that would imply a time-dependence going like e^t (the derivative with respect to time of e^t equals e^t) instead of harmonic dependence e^(i*omega*t).

e^t dependence would not be a standing harmonic wave, but it would be a disturbance growing exponentially with time !

or for time dependence of e^(-a*t) it would be exponentially time-decaying, and in that case if they don't take the divergence they would be off by sign and by the factor a.

These alternatives don't appear to make sense...
Title: Re: EM Drive Developments - related to space flight applications - Thread 2
Post by: Stormbringer on 03/22/2015 12:22 am
Doctor McCulloch put forth another blog post on the EM Drive, using a 'one wave' version of his theory.  By and large, his predictions with this new model seem much closer to the mark than before.  There are a few tests he put in red because of uncertainty over the geometry.

http://physicsfromtheedge.blogspot.com/

At times, it seems almost like this device operates in the murky area between gravity and electromagnetic radiation.

The GRT coupling between EM and Gravity is miniscule; something like billionths of a trilionth or some ridiculously small amount like that. (source: popular articles on Martin Tajmar's retracted gravity effects paper.)

Are these present effects in that slender range or are they outliers to what GRT allows? If it is it's not necessarily invalidating because GRT does not describe dark sector interactions for example. But it's worth exploring how they vary from predictions.
Title: Re: EM Drive Developments - related to space flight applications - Thread 2
Post by: RotoSequence on 03/22/2015 01:10 am
Doctor McCulloch put forth another blog post on the EM Drive, using a 'one wave' version of his theory.  By and large, his predictions with this new model seem much closer to the mark than before.  There are a few tests he put in red because of uncertainty over the geometry.

http://physicsfromtheedge.blogspot.com/

At times, it seems almost like this device operates in the murky area between gravity and electromagnetic radiation.

The GRT coupling between EM and Gravity is miniscule; something like billionths of a trilionth or some ridiculously small amount like that. (source: popular articles on Martin Tajmar's retracted gravity effects paper.)

Are these present effects in that slender range or are they outliers to what GRT allows? If it is it's not necessarily invalidating because GRT does not describe dark sector interactions for example. But it's worth exploring how they vary from predictions.

Is it too much to hope for a strong divergence from predictions?  ;D

http://home.web.cern.ch/about/updates/2015/03/lhcbs-new-analysis-confirms-old-puzzle
Title: Re: EM Drive Developments - related to space flight applications - Thread 2
Post by: aero on 03/22/2015 02:42 am
Quote
Unless they assume some non-conventional time-dependence for the electromagnetic waves such that the time derivative equals the original function.  Essentially that would imply a time-dependence going like e^t (the derivative with respect to time of e^t equals e^t) instead of harmonic dependence e^(i*omega*t).

e^t dependence would not be a standing harmonic wave, but it would be a disturbance growing exponentially with time !

Isn't that exactly the assumption for superluminal evanescent waves and for matter waves tunnelling through a barrier? Yes, I think it is.
Title: Re: EM Drive Developments - related to space flight applications - Thread 2
Post by: Stormbringer on 03/22/2015 03:10 am




Is it too much to hope for a strong divergence from predictions?  ;D

http://home.web.cern.ch/about/updates/2015/03/lhcbs-new-analysis-confirms-old-puzzle

No. No.... It's not. I certainly hope something turns our understanding of gravity on it's head. That's why I'm always spamming links to any article that seems to point in that direction; like the nexus graviton thing.  :)
Title: Re: EM Drive Developments - related to space flight applications - Thread 2
Post by: Star-Drive on 03/22/2015 03:38 am
Folks:

While you think about possible explanations for the gathering body of data that surrounds the EM-Drive, I've been given permission by Dr. John Brandenburg to post his latest conjecture on how he thinks the EM-Drive AKA Q-Thruster can generate a thrust signal that does not violate the conservations laws.  It's not too far off what Dr. White is proposing, but it's just different enough to be of interest especially to the GRT crowd.

Best, Paul M.
Title: Re: EM Drive Developments - related to space flight applications - Thread 2
Post by: Stormbringer on 03/22/2015 05:45 am
Folks:

While you think about possible explanations for the gathering body of data that surrounds the EM-Drive, I've been given permission by Dr. John Brandenburg to post his latest conjecture on how he thinks the EM-Drive AKA Q-Thruster can generate a thrust signal that does not violate the conservations laws.  It's not too far off what Dr. White is proposing, but it's just different enough to be of interest especially to the GRT crowd.

Best, Paul M.

Fascinating. This looks like it could actually tie into the article I posted earlier.

EDIT:  This one right here:

http://www.sciencedaily.com/releases/2015/03/150306091617.htm

Especially:

Quote
Moreover the Nexus graviton can also be considered as a globule of vacuum energy which can merge and de-merge with others in a process that resembles cytokineses in cell biology.

The Nexus graviton is Dark Matter and constitutes space-time. The emission of a graviton of least energy by a high energy graviton results in the expansion of the high energy graviton as it assumes a lower energy state. This process manifests as Dark Energy and takes place throughout space-time as the theory explains.

EDIT 2:  Random thoughts on the GEMS pdf: what would be the effects on the gravitational curves if these things were stacked in a linear fashion with the same orientation? Or placed like end to like end? Or nested coaxially? If the GEMS theory can make predictions for this it could be tested. Not to mention it might be additive or even multiplicative of the "thrust" signal. (A gravity effect would not really be thrust would it? I would describe it as analogous to thrust.)

EDIT 3: Even if the thrust is not a result of some GEMS thing what's to stop the team from stacking these things in an effort to break the GRC sensitivity threshold?

EDIT 4:  @Star-Drive: up-thread the participants were trying to get your attention on the reverse thrust condition.

EDIT 5:  If the effect is gravity related would the gravity gradient extend out of the frustrum? Could it be measured somewhat distant from the physical device? Change orientation to up and down and measure for weight differences of independent objects in proximity to the device?

EDIT 6:  Well if it is a QVPT then this  http://www.sciencedaily.com/releases/2014/07/140722091425.htm may be helpful.

because this sure sounds a lot like the frustrum: 
Quote
Ephraim Shahmoon, Gershon Kurizki (Weizmann Institute of Science) and Igor Mazets calculated what happens to vacuum forces between atoms when they are placed in the vicinity of an electrical transmission line such as a coaxial cable or a coplanar waveguide (a device used in cavity quantum electrodynamics experiments as an open transmission line), cooled to very low temperatures. "In that case, the fluctuations are effectively confined to one dimension," says Igor Mazets. The virtual particles will be forced to go into the direction of the other atom.

In that case, the fluctuation-mediated attraction between the atoms becomes orders of magnitude stronger than in free space. Usually, the force decreases rapidly with increasing distance between the atoms. Due to the transmission line, it falls off with one over the distance cubed, instead of one over the seventh power of the distance, as in the usual case.

Title: Re: EM Drive Developments - related to space flight applications - Thread 2
Post by: Mulletron on 03/22/2015 11:57 am
Attached are VSWR plots for the unloaded frustum configuration currently in my possession between 2400-2500mhz. The cursors are at the MIN VSWR. The peaks of each plot is a VSWR of 99.

Fully calibrated test equipment used for test is as follows:
HP Agilent 8757E SNA
HP 83752B Sweep generator @+10dbm
HP 85027C Directional Bridge
Gigatronics 8542C power meter w/80301A sensor

50 ohm termination
50 ohm short (for cal)
2 high quality 6' test cables 1.2db loss @2450mhz. (open and shorted during cal)

The rest of the data is here:
(where I marked max VSWR and frequency ranges where VSWR is less than 1.5)
https://drive.google.com/folderview?id=0B4PCfHCM1KYoNG5DdWdZb3BLazg&amp;usp=sharing&tid=0B4PCfHCM1KYoTXhSUTd5ZDN2WnM

700-2700mhz VSWR plot is here:
(disclaimer, my E field probe is not optimized for all these frequencies, still useful to see what other frequencies couple to the cavity with low VSWR using my 31mm probe/cavity combination)
https://drive.google.com/folderview?id=0B4PCfHCM1KYoX0RKcU5BRnphYm8&amp;usp=sharing&tid=0B4PCfHCM1KYoTXhSUTd5ZDN2WnM

Why you want to properly terminate the sample port on your frustum:
https://drive.google.com/folderview?id=0B4PCfHCM1KYoWXFTeFhEbXVsUG8&amp;usp=sharing&tid=0B4PCfHCM1KYoTXhSUTd5ZDN2WnM

Tonight, I'll post an amplitude response test, and numerous spectrum analyzer screenshots with more data.

Later, after I get the dielectric discs properly milled down to precisely 6.25", I'll do the same tests for a loaded frustum. If I can still resonate at my desired frequency range in that configuration, then it will be time to balance and fly some copper kettles  :)

Edit: Added another test cable. The test used 2 cables.


Title: Re: EM Drive Developments - related to space flight applications - Thread 2
Post by: Notsosureofit on 03/22/2015 12:17 pm
@ Star-Drive

"EDIT 4:  @Star-Drive: up-thread the participants were trying to get your attention on the reverse thrust condition."

-from Stormbringer

Thanks
Title: Re: EM Drive Developments - related to space flight applications - Thread 2
Post by: Rodal on 03/22/2015 01:25 pm
Attached are VSWR plots for the unloaded frustum configuration currently in my possession between 2400-2500mhz. The cursors are at the MIN VSWR. The peaks of each plot is a VSWR of 99.

...
Would like to perform some calculations of Poynting vector, etc.

What are the geometrical dimensions of your truncated cone?

Internal Diameter of Big Base = ?
Internal Diameter of Small Base = ?
Internal Axial Length (perpendicular to the bases) = ?
Title: Re: EM Drive Developments - related to space flight applications - Thread 2
Post by: Mulletron on 03/22/2015 01:27 pm
@Paul March, what do you recommend doing with the sample port on the frustum during actual force measurements?

It seems that leaving a 50ohm termination on there during the actual force testing will just ensure energy will get coupled back out of the cavity and into the load, which is what I don't want to happen.

Should I leave it terminated with a termination with a very low rating? Or should I remove the probe inside? Or should I just remove the connector and tape over it with copper tape? Connect it to some test equipment? Something else?

What do you do?

Thanks for all that you are doing.
Title: Re: EM Drive Developments - related to space flight applications - Thread 2
Post by: Mulletron on 03/22/2015 01:37 pm
Attached are VSWR plots for the unloaded frustum configuration currently in my possession between 2400-2500mhz. The cursors are at the MIN VSWR. The peaks of each plot is a VSWR of 99.

...
Would like to perform some calculations of Poynting vector, etc.

What are the geometrical dimensions of your truncated cone?

Internal Diameter of Big Base = ?
Internal Diameter of Small Base = ?
Internal Axial Length (perpendicular to the bases) = ?

I posted pics of the dims earlier, it was built based off of this which came from Eagleworks:
https://drive.google.com/folderview?id=0B4PCfHCM1KYoTXhSUTd5ZDN2WnM&amp;usp=sharing
(see frustum.jpg)

Measurements by me are here:
http://forum.nasaspaceflight.com/index.php?topic=36313.msg1345818#msg1345818
https://drive.google.com/folderview?id=0B4PCfHCM1KYoTl90eDBuMklOeTg&amp;usp=sharing&tid=0B4PCfHCM1KYoTXhSUTd5ZDN2WnM

Internal Diameter of Big Base = 11"
Internal Diameter of Small Base = 6.25"
Internal Axial Length (perpendicular to the bases) = 9"

Title: Re: EM Drive Developments - related to space flight applications - Thread 2
Post by: Notsosureofit on 03/22/2015 01:37 pm
@Paul March, what do you recommend doing with the sample port on the frustum during actual force measurements?

It seems that leaving a 50ohm termination on there during the actual force testing will just ensure energy will get coupled back out of the cavity and into the load, which is what I don't want to happen.

Should I leave it terminated with a termination with a very low rating? Or should I remove the probe inside? Or should I just remove the connector and tape over it with copper tape? Connect it to some test equipment? Something else?

What do you do?


Thanks for all that you are doing.

Not NASA here, of course, but I would just leave the connector "open", ie. high impedance.  (what impedance are you looking into as a monitor ?)

My personal favorite would be to use it as feedback in an oscillator configuration.  (I'm "biased" )
Title: Re: EM Drive Developments - related to space flight applications - Thread 2
Post by: Mulletron on 03/22/2015 01:55 pm
@Paul March, what do you recommend doing with the sample port on the frustum during actual force measurements?

It seems that leaving a 50ohm termination on there during the actual force testing will just ensure energy will get coupled back out of the cavity and into the load, which is what I don't want to happen.

Should I leave it terminated with a termination with a very low rating? Or should I remove the probe inside? Or should I just remove the connector and tape over it with copper tape? Connect it to some test equipment? Something else?

What do you do?


Thanks for all that you are doing.

Not NASA here, of course, but I would just leave the connector "open", ie. high impedance.  (what impedance are you looking into as a monitor ?)

My personal favorite would be to use it as feedback in an oscillator configuration.  (I'm "biased" )

Everything I am working is 50ohm, which has to be matched *. With the sample port in place (with a properly configured probe capable of providing a good sample) that is open (not terminated) or shorted you get this on the input for VSWR.
https://drive.google.com/folderview?id=0B4PCfHCM1KYoWXFTeFhEbXVsUG8&amp;usp=sharing&tid=0B4PCfHCM1KYoTXhSUTd5ZDN2WnM
(see not terminated.jpg.... If I direct link to a pic in my Google Drive, Google pulls the plug eventually due to quota or something.)

This is bad.

I suspect the right thing to do is to put a 1/4 watt terminator on there, that is if the frustum isn't properly coupled to a piece of test equipment like a spectrum analyzer being protected by an attenuator.

*I don't intend on using a monitor or a feedback circuit. If for example the cavity falls off resonance due to heat, the wideband signal it is getting fed with will be enough to compensate.

The signal looks like this:
(http://www.ece.vt.edu/swe/mcms/2410ism_52_freq.jpg)
Title: Re: EM Drive Developments - related to space flight applications - Thread 2
Post by: Notsosureofit on 03/22/2015 02:06 pm
Can you use your analyser in a reflectometer configuration ?   Or is that what you are already doing ?
Title: Re: EM Drive Developments - related to space flight applications - Thread 2
Post by: Rodal on 03/22/2015 02:10 pm
Folks:

While you think about possible explanations for the gathering body of data that surrounds the EM-Drive, I've been given permission by Dr. John Brandenburg to post his latest conjecture on how he thinks the EM-Drive AKA Q-Thruster can generate a thrust signal that does not violate the conservations laws.  It's not too far off what Dr. White is proposing, but it's just different enough to be of interest especially to the GRT crowd.

Best, Paul M.

The author has an experimental box: "the Morningstar Energy Box" (which he also refers to in the above cited publication) which he claims (in a 2014 publication http://www.mehtapress.com/mehtapress/Journals/Journal-of-Space-Exploration/index.html see below) produced a steady state 14 pound reduction out of 190 pounds or 7.3% and a transient loss of 12% of the total weight.  They claim to have observed "nonlinear energy Box phenomenon similar to Russian claims"

http://www.mehtapress.com/mehtapress/Journals/Journal-of-Space-Exploration/Volume-3-Issue-1/vol_3_issue_1_file_1.pdf


Russian "box"

The "Magnetic Energy Converter" designed by V. V. Roshchin and S. M. Godin

http://www.google.com/patents/US6822361?dq=6822361

(http://pesn.com/2011/03/04/9501779_Morningstar_Energy_Box_Announcement--Russian_MEC_Partially_Replicated/Russian_Magnetic_Energy_Converter_300.jpg)
Title: Re: EM Drive Developments - related to space flight applications - Thread 2
Post by: Mulletron on 03/22/2015 02:12 pm
Can you use your analyser in a reflectometer configuration ?

Um, no I don't think it can do that. I have an Agilent E4443A, and also an 8562A and 8562EC to use. I can't take test equipment home from work anyway. So all I can do is test at work to get data, then use the data at home.
I've had to work all weekend, so that's why I'm in test mode instead of building things and trying to find a lathe to use..... ;)
Title: Re: EM Drive Developments - related to space flight applications - Thread 2
Post by: Notsosureofit on 03/22/2015 02:40 pm
Can you use your analyser in a reflectometer configuration ?

Um, no I don't think it can do that. I have an Agilent E4443A, and also an 8562A and 8562EC to use. I can't take test equipment home from work anyway. So all I can do is test at work to get data, then use the data at home.
I've had to work all weekend, so that's why I'm in test mode instead of building things and trying to find a lathe to use..... ;)

Hmmm, if you have to use a 50ohm load on the port then I think the cavity will just be acting as a matching network for the load.  Can you sketch up a layout of your connections.  (I had to go reflectometer for a customer a while back and just made up a rig to adapt my analyser...  so maybe ?)

Added:  Just thinking, I may still have it lying around, I'll look tomorrow !  (50ohm if I remember !)

It's just a 3-port bridge using Mini-Circuits  (have to check the freq range)

 
Title: Re: EM Drive Developments - related to space flight applications - Thread 2
Post by: Star-Drive on 03/22/2015 02:49 pm
Mulletron:

In regards to your sense antenna port on the copper frustum, when not being used for taking VNA S21 frequency sweep plots I either leave our sense antenna as an open circuit, AKA in a high-Z state, or use it as a feedback control signal for our phase locked loop (PLL) circuit.  If the latter, I still have to make the E-field sense antenna in the frustum very small, (like just the solder cup on an SMA bulkhead connector), so as to not overload the PLL's mixer input.   Even with this small of an antenna, I still have to add at least 40 dB of 2W each 5-to-10 dB attenuator pads to the sense antenna's output to keep the PLL's mixer circuit from overloading.  And the higher the Q-factor of the mode under study, the more sense antenna padding isolation that is required.

Best,  Paul M.
Title: Re: EM Drive Developments - related to space flight applications - Thread 2
Post by: Star-Drive on 03/22/2015 03:49 pm
@ Star-Drive

"EDIT 4:  @Star-Drive: up-thread the participants were trying to get your attention on the reverse thrust condition."

-from Stormbringer

Thanks


Notsosureofit:

The reverse thrust signals for the 1937.15MHz thrust signals are not the negative going mirror images of the forward or positive going thrust signatures as one would have liked.  I'm still trying to understand why this is so, but I think it means that the force measurement system is not symmetric in its response to left and right going force inputs at the low thrust magnitudes we are currently having to put up with, since I've already shown that the frustum produces similar thrust values both inside and outside the stainless steel vacuum chamber.  I've appended two slides to this post that show the forward and reverse signals in-air on one slide at 50W and then just the reverse thrust signal in-vacuum at 35W.  I've also appended the frustum out of the vacuum chamber thrust run, noisy as it is for your inspection. 

Bottom line to all this is we need more thrust to work with and from our current COMSOL/QV-Plasma Code runs now for the TM010 mode at 942 MHz, its becomes very apparent we really need to increase our power levels into the 1.0-to-10kW-rf range or even higher.  And yes they may validate the magnitude of thrust data that Shawyer's and Chinese's reported even without dielectrics in the cavity.  Of course one data point is only suggestive, but it sure points us to where we have to go to truly validate these conjectures.  However that course of action requires resources that are at present not available to the Eagleworks lab... 

BTW, these COMSOL/QV-Plasma Code thrust predictions may be reporting up to an order of magnitude low from what the real test article will generate at the noted power levels as shown in the last appended slide.

Best, Paul M.
Title: Re: EM Drive Developments - related to space flight applications - Thread 2
Post by: Rodal on 03/22/2015 03:58 pm
@ Star-Drive

"EDIT 4:  @Star-Drive: up-thread the participants were trying to get your attention on the reverse thrust condition."

-from Stormbringer

Thanks


Notsosureofit:

The reverse thrust signals for the 1937.15MHz thrust signals are not the negative going mirror images of the forward or positive going thrust signatures as one would have liked.  I'm still trying to understand why this is so, but I think it means that the force measurement system is not symmetric in its response to left and right going force inputs at the low thrust magnitudes we are currently having to put up with, since I've already shown that the frustum produces similar thrust values both inside and outside the stainless steel vacuum chamber.  I've appended two slides to this post that show the forward and reverse signals in-air on one slide at 50W and then just the reverse thrust signal in-vacuum at 35W.  I've also appended the frustum out of the vacuum chamber thrust run, noisy as it is for your inspection. 

Bottom line to all this is we need more thrust to work with and from our current COMSOL/QV-Plasma Code runs now for the TM010 mode at 942 MHz, its becomes very apparent we really need to increase our power levels into the 1.0-to-10kW-rf range or even higher.  And yes they may validate the magnitude of thrust data that Shawyer's and Chinese's reported even without dielectrics in the cavity.  Of course one data point is only suggestive, but it sure points us to where we have to go to truly validate these conjectures.  However that course of action requires resources that are at present not available to the Eagleworks lab... 

BTW, these COMSOL/QV-Plasma Code thrust predictions may be reporting up to an order of magnitude low from what the real test article will generate at the noted power levels as shown in the last appended slide.

Best, Paul M.

What does "reversed" mean in the above discussion and plots?  does it mean:

1) Turning around by 180 degrees the EM Drive, such that the thrust force and thrust motion still points towards the small base of the truncated cone?

(just like turning around by 180 degrees a car driving in "D" will result in it moving forward in the opposite direction)

(http://upload.wikimedia.org/wikipedia/commons/thumb/8/80/Bootleg-turn.svg/200px-Bootleg-turn.svg.png)

or

2) The "reversed" thrust force and thrust motion points towards the big base of the truncated cone. 

(just like putting a car in reverse "R" will make the car move backward instead of forward)

Where one defines 

forward thrust = thrust force and thrust motion pointing towards the small base of the truncated cone

reverse thrust = thrust force and thrust motion pointing towards the big base of the truncated cone

(http://www.autoguide.com/blog/wp-content/uploads//2012/06/week_in_reverse3.jpeg)
Title: Re: EM Drive Developments - related to space flight applications - Thread 2
Post by: Star One on 03/22/2015 04:17 pm
If I'm understanding this right is there a kind of catch 22 at work here. To validate if this drive works or not you need a greater power input, but to obtain access to a greater power input it has to be proved that it works first?
Title: Re: EM Drive Developments - related to space flight applications - Thread 2
Post by: Notsosureofit on 03/22/2015 04:26 pm
@ Star-Drive

Have there been any experimental runs that show force towards the big end ??
Title: Re: EM Drive Developments - related to space flight applications - Thread 2
Post by: Star-Drive on 03/22/2015 05:11 pm
@ Star-Drive

Have there been any experimental runs that show force towards the big end ??


All:

1. For my post of earlier today reversed thrust means physically reversing the frustum test article 180 degrees from its previous "forward" position.

2.  I've also observed several test setups that generated reversed thrust signatures without physically turning the thruster around 180 degrees in its mount.  One of these instances was with the current copper frustum and the others were with my two Mach Lorentz Thrusters (MLT) as reported in my STAIF-2006 paper.  Thrust reversal was obtained in those test setups by reversing the E-field & B-field drive polarities via relay control.

Best, Paul M.
Title: Re: EM Drive Developments - related to space flight applications - Thread 2
Post by: Rodal on 03/22/2015 05:33 pm
@ Star-Drive

Have there been any experimental runs that show force towards the big end ??


All:

1. For my post of earlier today reversed thrust means physically reversing the frustum test article 180 degrees from its previous "forward" position.

2.  I've also observed several test setups that generated reversed thrust signatures without physically turning the thruster around 180 degrees in its mount.  One of these instances was with the current copper frustum and the others were with my two Mach Lorentz Thrusters (MLT) as reported in my STAIF-2006 paper.  Thrust reversal was obtained in those test setups by reversing the E-field & B-field drive polarities via relay control.

Best, Paul M.

The bell just tolled: my understanding of what "reverse" meant in your plots was thereby defenestrated corrected  :)

Is there a position vs. time plot of the "One of these instances ... with the current copper frustum" with the force  directed towards the big end?  If so, can you make that plot available?

What was the frequency, mode shape, thrust force, Q, input power and testing environment corresponding to the case exhibiting a thrust force pointed towards the big base?

It is very important to report this case because it defenestrates nullifies mechanistic theories proposed to explain the NASA Eagleworks measurements as an experimental mechanical artifact having no application whatsoever for spaceflight applications.
Title: Re: EM Drive Developments - related to space flight applications - Thread 2
Post by: Mulletron on 03/22/2015 06:15 pm
Mulletron:

In regards to your sense antenna port on the copper frustum, when not being used for taking VNA S21 frequency sweep plots I either leave our sense antenna as an open circuit, AKA in a high-Z state, or use it as a feedback control signal for our phase locked loop (PLL) circuit.  If the latter, I still have to make the E-field sense antenna in the frustum very small, (like just the solder cup on an SMA bulkhead connector), so as to not overload the PLL's mixer input.   Even with this small of an antenna, I still have to add at least 40 dB of 2W each 5-to-10 dB attenuator pads to the sense antenna's output to keep the PLL's mixer circuit from overloading.  And the higher the Q-factor of the mode under study, the more sense antenna padding isolation that is required.

Best,  Paul M.

That's good advice about making the sense antenna small. What I have right now, the sense antenna is as good at coupling energy out of the cavity as the feed antenna is at getting energy into the cavity. They are both the same and interchangeable.

During my testing I noted that the terminated sense antenna port played a huge role on how much power was reflected back into the directional bridge connected to the frustum input port.

Based on what I learned today from testing and from you, the sense antenna port should be impedance matched to whatever is driving the frustum and what is attached to the sense port, but it need not be (and shouldn't be) an efficient coupler of energy out of the cavity for the sake of the cavity storing as much energy as possible and for the sake of the test equipment attached. This lesson learned applies to during force testing with higher power levels.

During low power VSWR testing and other tests, a matched input/sense port is ideal for an accurate test. In the next post, I'll show how I can efficiently couple energy into and out of the cavity (from sweep gen to power meter/spectrum analyzer) at certain frequencies and only lose a couple db of power in the round trip. Exact figures will be in that post.

Based on what you're saying, I'm going to make another (hobbled) sense antenna, for now it will be just a solder cup. Then I will repeat the same VSWR test to see how things change.

I will also do a VSWR test with NO sense antenna, the hole will be covered in conductive copper tape on both sides similar to what's in this shot here:
http://forum.nasaspaceflight.com/index.php?action=dlattach;topic=36313.0;attach=785112;image
http://forum.nasaspaceflight.com/index.php?topic=36313.msg1331854#msg1331854

I won't be working with much power so I have to be certain I don't waste even a milliwatt of power, so that's why I'm digging into the sense antenna and also making sure that I get the very lowest VSWR as possible on the input.



Title: Re: EM Drive Developments - related to space flight applications - Thread 2
Post by: Chrochne on 03/22/2015 07:05 pm
Quoted:
Bottom line to all this is we need more thrust to work with and from our current COMSOL/QV-Plasma Code runs now for the TM010 mode at 942 MHz, its becomes very apparent we really need to increase our power levels into the 1.0-to-10kW-rf range or even higher.  And yes they may validate the magnitude of thrust data that Shawyer's and Chinese's reported even without dielectrics in the cavity.  Of course one data point is only suggestive, but it sure points us to where we have to go to truly validate these conjectures.  However that course of action requires resources that are at present not available to the Eagleworks lab... 
(By Star-Drive)

I noticed that Mr. Roger Shawyer suggested something similar, when it comes to high power tests of his second generation engine.
http://emdrive.com/secondgenengines.html . Is there something similar in the USA as European Tesla accelerator? Just a thought.

Additionaly, I am starting to see some similarities between the NASA, Chinese and Mr. Shawyer. All are suggesting a high power tests might really be needed. Question now is how to do that. I pressume it can be really expensive. But NASA might have better chance to reach European Tesla accelerator than Mr. Shawyer do. It would be nice to see some sort of cooperation between above suggested parties on this.

Anyway, thank you all for continuing dabate. For me personaly it gives hope and great insight into the keen minds of the people. Thank you.
 

Title: Re: EM Drive Developments - related to space flight applications - Thread 2
Post by: Mulletron on 03/22/2015 07:08 pm
So here is the amplitude response test which measures gain/loss over a frequency range. This test is centered on 2450mhz with a span of 125mhz. This is an automated test and uses the following:

*HP Agilent 8757E SNA with (1) *11664A detector and (1) *85025A detector.
*HP 11667A power divider
*HP 83752B Sweep generator @0dbm
*Gigatronics 8542C power meter w/*80301A sensor
Laptop running test software controlling sweep generator and SNA
*6db and *3db attenuators to protect gear, 6db on input cable, 3db on detector (accounted for in cal loop)
(1) 6' high quality test cable
*=Calibrated by a lab

All gains and losses not from the DUT are accounted for in the cal loop.

There was a calibrated 0dbm of power input to the frustum as measured by a power meter. The output of the frustum is measured by a detector on the sample port of the frustum. The sweep is obtained by comparing a reference detector attached to the signal generator output at power divider port, with the detector attached to the frustum output.

Before the test is run, a cal loop is run, where the DUT is replaced by a N-type female bullet adaptor. So the input cable is directly connected to the output detector by an rf bullet adaptor.

After the cal run, the bullet is replaced by the DUT.

This gives a nice overview of what is happening in the frustum, but doesn't give much detail.

A post will follow with data from a spectrum analyzer showing the same amplitude response with more detail. This is essentially the same test but done manually without an SNA.
Title: Re: EM Drive Developments - related to space flight applications - Thread 2
Post by: Star-Drive on 03/22/2015 07:16 pm
@ Star-Drive

Have there been any experimental runs that show force towards the big end ??


All:

1. For my post of earlier today reversed thrust means physically reversing the frustum test article 180 degrees from its previous "forward" position.

2.  I've also observed several test setups that generated reversed thrust signatures without physically turning the thruster around 180 degrees in its mount.  One of these instances was with the current copper frustum and the others were with my two Mach Lorentz Thrusters (MLT) as reported in my STAIF-2006 paper.  Thrust reversal was obtained in those test setups by reversing the E-field & B-field drive polarities via relay control.

Best, Paul M.

The bell just tolled: my understanding of what "reverse" meant in your plots was thereby defenestrated corrected  :)

Is there a position vs. time plot of the "One of these instances ... with the current copper frustum" with the force  directed towards the big end?  If so, can you make that plot available?

What was the frequency, mode shape, thrust force, Q, input power and testing environment corresponding to the case exhibiting a thrust force pointed towards the big base?

It is very important to report this case because it defenestrates nullifies mechanistic theories proposed to explain the NASA Eagleworks measurements as an experimental mechanical artifact having no application whatsoever for spaceflight applications.

Dr. Rodal:

The copper frustum thrust reversal due to only its dielectric placement came when I was experimenting with the TM010 mode, when I cut back the HDPE disc count to one, while the copper frustum was mounted in its little OD pointed to the right or forward position.  I then removed the HDPE disc and replaced it with one 5.13" OD by 1.06" thick PTFE discs that was mounted at the large OD end of the copper frustum with the frustum still mounted in the same direction as the first TM010 test, i.e., with its small OD end pointed toward the right in the vacuum chamber.  The attached slides summarizes these two test series at 954MHz (1 HDPE Disc) and 901 MHz (1 Teflon Discs at Large OD end).

Best,
Title: Re: EM Drive Developments - related to space flight applications - Thread 2
Post by: Mulletron on 03/22/2015 08:22 pm
And here's a sample of the spectrum analyzer test. All this test involves is injecting a calibrated 0dbm input signal to the frustum, then seeing what comes out the other side on the spectrum analyzer.

*HP 83752B Sweep generator @0dbm
*Agilent E4443A spectrum analyzer
*Gigatronics 8542C power meter w/*80301A sensor
(2) 6' high quality test cables
*=Calibrated by a lab

Combine this with the VSWR data and the behavior of the unloaded frustum between 2400-2500mhz becomes apparent. The points where the VWSR is really poor is evident in the amplitude response, seen in the spectrum analyzer shots, so these two tests validate each other.

Rest is here:
https://drive.google.com/folderview?id=0B4PCfHCM1KYoUEx5dzlVTG81a2c&usp=sharing&tid=0B4PCfHCM1KYoTXhSUTd5ZDN2WnM

Other frequencies here:
(disclaimer, my E field probe is not optimized for all these frequencies, still useful to see what other frequencies couple to the cavity with low VSWR using my 31mm probe/cavity combination)
The best peaks of other frequencies are labeled similar to the bottom screen shot.
https://drive.google.com/folderview?id=0B4PCfHCM1KYoVkRzUGNuMVBLbVk&usp=sharing&tid=0B4PCfHCM1KYoTXhSUTd5ZDN2WnM

Title: Re: EM Drive Developments - related to space flight applications - Thread 2
Post by: frobnicat on 03/22/2015 09:29 pm
Thank you Paul March for clarifying the situation concerning "true" reversal and sharing precious experimental data points.

Before everyone agrees about what is to be explained in those results, maybe you could further confirm the proper interpretation of orientations (for which clear communication and universal reading is surprisingly difficult) :

Dr. Rodal:

The copper frustum thrust reversal due to only its dielectric placement came when I was experimenting with the TM010 mode, when I cut back the HDPE disc count to one, while the copper frustum was mounted in its little OD pointed to the right or forward position. 

Usually in your reports the "forward" direction implies the small OD toward the left, (as seen when standing in front of vacuum chamber). The 2 frustum setup attached pictures show the small OD toward the left. Should we understand here that the small OD end is actually to the left ? Pictures are self consistent and consistent with the usual use of forward in previous reports. Saying "little OD pointed to the right" seems contradicting. Is it correct to understand that the small OD end is on the left (as seen on pictures), and that the first frustum thrust picture shows a thrust toward the big OD end (red arrow) ? There is in this experiment 1 PE disc, is it at the small OD end or at the big OD end ?
bigger (http://forum.nasaspaceflight.com/index.php?action=dlattach;topic=36313.0;attach=817226;image)
(http://forum.nasaspaceflight.com/index.php?action=dlattach;topic=36313.0;attach=817227;image)

Quote
I then removed the HDPE disc ...

That was at small OD end ? Big OD end ?

Quote
... and replaced it with one 5.13" OD by 1.06" thick PTFE discs that was mounted at the large OD end of the copper frustum with the frustum still mounted in the same direction as the first TM010 test, i.e., with its small OD end pointed toward the right in the vacuum chamber.
 

Again saying "small OD end pointed toward the right" seems to contradict the picture (while all the rest is consistent).
bigger (http://forum.nasaspaceflight.com/index.php?action=dlattach;topic=36313.0;attach=817230;image)
(http://forum.nasaspaceflight.com/index.php?action=dlattach;topic=36313.0;attach=817231;image)

Quote
The attached slides summarizes these two test series at 954MHz (1 HDPE Disc at which OD end ?) and 901 MHz (1 Teflon Discs at Large OD end).

Sorry for this "nitpicking", this is to avoid long arguments on interpretation, and discuss conjectures on common ground.

Best
Title: Re: EM Drive Developments - related to space flight applications - Thread 2
Post by: Star-Drive on 03/23/2015 02:13 am
My bad, I meant to the left....

(That's what I get when I'm in a hurry and let my dyslexia get out.)

Best, Paul M
Title: Re: EM Drive Developments - related to space flight applications - Thread 2
Post by: Stormbringer on 03/23/2015 05:10 am
Someone else is making an EM drive:  http://www.scientificamerican.com/article/worlds-first-flying-saucer/

  It's probably the april first addition.  ;) and it works like one of those 400 dollar ionic wind Dyson fans.

It's not the april first addition but its from july 2008.
Title: Re: EM Drive Developments - related to space flight applications - Thread 2
Post by: Damon Hill on 03/23/2015 05:59 am
http://www.dtic.mil/dtic/tr/fulltext/u2/a564120.pdf?

"Demonstration of a wingless electromagnetic air vehicle"

Pretty much proof-of-concept, goes into lots more detail than I have endurance to read.
Title: Re: EM Drive Developments - related to space flight applications - Thread 2
Post by: Star-Drive on 03/23/2015 11:05 am
http://www.dtic.mil/dtic/tr/fulltext/u2/a564120.pdf?

"Demonstration of a wingless electromagnetic air vehicle"

Pretty much proof-of-concept, goes into lots more detail than I have endurance to read.

Damon:

Good grief man, this is a great find!!  And an R&D activity I didn't even know was going on.  Now to figure out what is conventional plasma physics and what might be extended EM-Drive physics hiding in this University of Florida paper.

Thanks again,

Best, Paul M.   
Title: Re: EM Drive Developments - related to space flight applications - Thread 2
Post by: Rodal on 03/23/2015 11:19 am

This technology (http://www.dtic.mil/dtic/tr/fulltext/u2/a564120.pdf ) uses classical physics for propulsion: electrodes that cover its surface to ionize the surrounding air into plasma.   

It cannot work in outer space because there is no surrounding air to ionize and use for lifting purposes.

There is no connection to the profound issues (conservation of momentum) and the proposed esoteric explanations (Quantum Vacuum, Mach Effect, coupling of electromagnetics with gravity, Unruh radiation, etc.) of the EM Drive, because the EM Drive does not carry on-board any propellant and it is supposed to not rely on surrounding air for propulsion.

MIT's Aero & Astro Department has micro thrusters  (made of tiny chips) technology using electrospray propulsion to use in outer space, but it is also based on classical physics:

http://newsoffice.mit.edu/2015/accion-systems-thruster-for-small-satellites-0311#.VQGgo9sCC8A.linkedin

http://www.leonarddavid.com/tiny-thruster-offers-big-promise/

http://newsoffice.mit.edu/2012/microthrusters-could-propel-small-satellites-0817

https://www.youtube.com/watch?v=BHVkc2JwAuI

(http://www.nanowerk.com/news2/space/id39382.jpg)

(http://www.leonarddavid.com/wp-content/uploads/2014/09/MICROTHRUSTER-300x190.jpg)

(http://www.leonarddavid.com/wp-content/uploads/2014/09/MICROTHRUSTER-2-300x168.png)

(http://www.leonarddavid.com/wp-content/uploads/2014/09/microthruster-3-300x200.jpg)

(http://static1.squarespace.com/static/5446faa2e4b025843cfc6731/t/549f0c4fe4b0d2e1e20f5fcf/1419709519668/)
Title: Re: EM Drive Developments - related to space flight applications - Thread 2
Post by: flux_capacitor on 03/23/2015 11:32 am
The Wingless Electromagnetic Air Vehicles (http://www.dtic.mil/dtic/tr/fulltext/u2/a564120.pdf) (WEAV) is a recent concept of electrohydrodynamic (EHD) flying disc, derived from plasma actuators. It uses the electrophoretic coulomb and polarization forces to ionize air and accelerate the resulting plasma, literally sucking it radially from top of the device, where it creates a lower pressure area, hence a lift.

You can also produce similar bur greater suction using magnetohydrodynamics (MHD) with Lorentz forces F = q (E + v × B). Well, it would cost more than EHD because you need to create a strong magnetic field in the multi-tesla range; and the electrothermal instability becomes really tricky in magnetized non-thermal plasmas. But such MHD forces can accelerate a plasma to hypersonic speeds. I know that quite well because I used to work with a plasma physicist, expert in this field since 1975. Attached, a paper summarizing that research (scroll to section VII. Induction MHD aerodynes, pp. 12-15):

J.P. Petit; J. Geffray, F. David (19-22 October 2009). "MHD Hypersonic Flow Control for Aerospace Applications", AIAA-2009-7348, in 16th AIAA/DLR/DGLR International Space Planes and Hypersonic Systems and Technologies Conference (HyTASP), Bremen, Germany.

Whatever, I second Dr Rodal: despite using advanced concepts, with only electricity and no fuel on board, and having an unusual odd shape, those conceptual flying saucers are not propellantless thrusters, since they have to accelerate ambient air for propulsion. This is leading-edge, but still classical physics.

[EDIT]: A 20ft diameter MHD disc would be quite heavy, requiring about 100 MWe of (preferably fusion) electric power on board in order to overcome its weight. May I suggest to add one… hum no, three EmDrives below the hull (at 120° from each other) in order to lighten a bit that flying mammoth? ;)
Title: Re: EM Drive Developments - related to space flight applications - Thread 2
Post by: Rodal on 03/23/2015 12:10 pm
Quote
This finding indicates that the photons all moved at the same speed, even though different photons had different energies. This is one of the best measurements ever of the independence of the speed of light from the energy of the light particles.
Beyond confirming the general theory of relativity, the observation rules out one of the interesting ideas concerning the unification of general relativity and quantum theory. While these two theories are the pillars of physics today, they are still inconsistent, and there is an intrinsic contradiction between the two that is partially based on Heisenberg's uncertainty principle that is at the heart of quantum theory.
One of the attempts to reconcile the two theories is the idea of "space-time foam." According to this concept, on a microscopic scale space is not continuous, and instead it has a foam-like structure. The size of these foam elements is so tiny that it is difficult to imagine and is at present impossible to measure directly. However light particles that are traveling within this foam will be affected by the foamy structure, and this will cause them to propagate at slightly different speeds depending on their energy.
Yet this experiment shows otherwise. The fact that all the photons with different energies arrived with no time delay relative to each other indicates that such a foamy structure, if it exists at all, has a much smaller size than previously expected.



http://phys.org/news/2015-03-einstein-scientists-spacetime-foam.html?utm_source=nwletter&utm_medium=email&utm_content=ctgr-item&utm_campaign=daily-nwletter


These conclusions involve dispersion (variation of speed with frequency) for those wavelengths comparable to the characteristic length of the assumed foam structure of space-time.
Title: Re: EM Drive Developments - related to space flight applications - Thread 2
Post by: tchernik on 03/23/2015 03:49 pm
Whatever, I second Dr Rodal: despite using advanced concepts, with only electricity and no fuel on board, and having an unusual odd shape, those conceptual flying saucers are not propellantless thrusters, since they have to accelerate ambient air for propulsion. This is leading-edge, but still classical physics.

Yep. And I think a great deal of effort in this thread and the previous defunct one, has been going precisely into determining if this was (or not) yet another instance of the Biefeld-Brown effect (ionic wind) or due to spurious thermal effects/convection, easily mistaken with thrust at such low acceleration regime.

That last point is still unclear, if I'm reading the discussion correctly, but some bits of evidence seem to hint it is not another instance of those known-to-be-fake "reactionless" thrusters from the past.
Title: Re: EM Drive Developments - related to space flight applications - Thread 2
Post by: Rodal on 03/23/2015 04:54 pm
....
The copper frustum thrust reversal due to only its dielectric placement came when I was experimenting with the TM010 mode,...
Paul,

Thanks so much for the very clear and complete response.

Concerning the designation of mode TM010 for this mode shape, please allow me to make this note, not because of nitpicking but perhaps to come to a common understanding (where for example I could be shown to be wrong, in which case I would appreciate learning about something I may have missed or misinterpreted).

I think this mode has been innocently mislabeled, in the efforts by the COMSOL FEA analyst to designate modes on the basis of vector field plots (for FEA analysis the mode shape numbers have to be designated by the analyst while in an exact solution the mode shape numbers come automatically from the solution: there is no room for interpretation).  I think that this mode shape correct designation is TM011 instead of TM010 because:

1) There can be no TMmn0 modes for a truncated cone.  TMmn0 modes need to have a constant electromagnetic field in the longitudinal direction of the cavity.  This is possible for a cylindrical cavity (containing no other dielectrics inside besides the cavity medium) because it has constant geometrical and material properties in the longitudinal direction.  But a conical cavity has variable cross-section in the longitudinal direction, therefore the  TMnn0 mode is not possible.  The first possible mode (if it is not cut-off) is TMmn1.  The exact solution for the truncated cone shows this.  See for example:  http://gregegan.customer.netspace.net.au/SCIENCE/Cavity/Cavity.html   :The quantum mode number "p" for a truncated cone is related to k.   k cannot be zero for a truncated cone, since k = ω/c for a truncated cone , therefore k = 0 implies zero frequency for a truncated cone. 

2) As the attached plot shows, COMSOL's FEA plot shows an electromagnetic field that is not constant in the longitudinal direction, therefore this is not TM010, it looks instead as TM011 upon closer inspection.

(Therefore the analyst's designation for TM011 should be changed to TM012 and so forth for TM01p modes)

Title: Re: EM Drive Developments - related to space flight applications - Thread 2
Post by: Mulletron on 03/23/2015 04:58 pm
Hey @Rodal, would you mind telling us more about what sources and methods you are using or have developed that allowed you to generate these original and colorful simulations in these posts:

http://forum.nasaspaceflight.com/index.php?topic=36313.msg1348295#msg1348295
http://forum.nasaspaceflight.com/index.php?topic=36313.msg1340906#msg1340906
http://forum.nasaspaceflight.com/index.php?topic=36313.msg1340909#msg1340909
http://forum.nasaspaceflight.com/index.php?topic=36313.msg1341244#msg1341244
http://forum.nasaspaceflight.com/index.php?topic=36313.msg1344664#msg1344664

It is quite evident that over the course of this lengthy discussion, you have developed tools and methods which can be helpful to many others. Can you give us a summary of what you can do now?
Title: Re: EM Drive Developments - related to space flight applications - Thread 2
Post by: Rodal on 03/23/2015 05:08 pm
Hey @Rodal, would you mind telling us more about what sources and methods you are using or have developed that allowed you to generate these original and colorful simulations in these posts:

http://forum.nasaspaceflight.com/index.php?topic=36313.msg1348295#msg1348295
http://forum.nasaspaceflight.com/index.php?topic=36313.msg1340906#msg1340906
http://forum.nasaspaceflight.com/index.php?topic=36313.msg1340909#msg1340909
http://forum.nasaspaceflight.com/index.php?topic=36313.msg1341244#msg1341244
http://forum.nasaspaceflight.com/index.php?topic=36313.msg1344664#msg1344664

It is quite evident that over the course of this lengthy discussion, you have developed tools and methods which can be helpful to many others. Can you give us a summary of what you can do now?
Yes, it would be a good idea to summarize them (when I have a chance to do it  :)  )

My analysis is all based on exact solutions based on classical Electromagnetism (*), which I have discussed earlier in the thread. 

I think that @Notsosureofit may have followed some of my earlier discussions.

I wrote Mathematica code to numerically solve the eigenvalue problems in the exact solutions and to plot the solutions.

It is a work in progress: I started with contour plots for fields and later on I wrote the code for the vector plots.

Since the natural coordinates of the exact solution are in spherical coordinates attached to the cone, to plot these fields in Mathematica one has to transform the coordinates and the vectors to a Cartesian field x, y, z.  This transformation actually took more time to get just right (because of nonlinearities involving SquareRoots and ArcTan functions that have multiple values) than the time it took to get the exact solutions.

In the course of one of our latest post discussions with @Frobnicat I started by showing the vector components of E and B to make a point about Poynting's vector (no pun intended) and I told myself: hey why don't I just calculate and plot Poynting's vector ? (NASA Eagleworks did not show us COMSOL plots of Poynting's vector: don't know whether COMSOL can do this easily ...)

My next step was going to be to calculate more exact solutions (including the dielectric) but I am spending more time on these plots.  For example, depending on the discussion it may be interesting to plot Maxwell stress tensors components and/or the components of the 3+1 spacetime relativistic energy-stress tensor ...

And I'm immediately interested in examining Poynting's vector direction for a number of modes in NASA Eagleworks truncated cone to see whether there is any mode clearly pointing in the direction of the big base.  So far all the modes I have examined have a Poyinting's vector very clearly directed towards the small base and other ones have a Poynting's vector that may be zero or very small in the opposite direction.

Which means that I should also write Mathematica code to integrate Poynting's vector for the cases in which it is not visually clear which way the overall integral over the volume is pointing.

______________
(*) As discussed previously, for example the electromagnetic force depends on the derivative of Poynting's vector with respect to time and on the divergence of Maxwell's stress tensor.  Poynting's vector goes like (Cos[ωt])^2 so its derivative with respect to time (2 Cos[ωt]*Sin[ωt] = Sin[2ωt]) should average zero over each half cycle Pi/ω = 1/(2 f).  The divergence of Maxwell's stress tensor should be zero if there are no electromagnetic sources inside the cavity.  (Notice that Brandenburg assumes sources inside the cavity to arrive at his explanation).
Title: Re: EM Drive Developments - related to space flight applications - Thread 2
Post by: Flyby on 03/23/2015 05:11 pm
If I'm understanding this right is there a kind of catch 22 at work here. To validate if this drive works or not you need a greater power input, but to obtain access to a greater power input it has to be proved that it works first?

I fear it is worse then that, because when you resort to more power input the thermal effects on the frustrum will also increase, with a dramatic negative effect on the Q.
And according to Shawyer, there is a real relation between a high Q and the force the "effect" produces.

That is exactly why Shawyer's latest experiment is aiming for a liquid nitrogen cooled device.
Needless to say it increases the cost price considerably.... ::)
The practical challenge to keep such a thruster operational is how to keep the Q  (and the frequency) constant, regardless of the temperature increase...

As Dr.Rodal already suggested, Shawyer has a considerably lead in experimental results, considering he already opted for a more efficient spherical truncated cone and supercooled surfaces. The home build devices we've seen so far is where Shawyer was in 2003....
Luckily, we can learn from his try and error experiments to have a fast forward in concepts...

I think the cheapest alternative would be a medium powered device that has water cooling. I was thinking along a thermal past embedded tubular copper pipe around the cone, using water cooling systems used for PC overclocking. those are easy and readily available online..
Sadly, I do not have the means to start such an undertaking, neither technically, nor financially..

As said before,  way too much energy is spend trying to separate and validate an actual thrust signal from potential background noise.... A brute force approach is needed just to identify whether or not it is working.

If, just like the Chinese, some one succeeds in yanking 720mN out of it (72g force) then all the discussions about the validity of measurements are over and you'll get enough momentum for a world wide focus on the theoretical aspect if this mysterious force...

Title: Re: EM Drive Developments - related to space flight applications - Thread 2
Post by: JasonAW3 on 03/23/2015 05:28 pm
Ok, here's a crazy idea;

     Is it not possible that what is being seen with the EM drive is not so much actual thrust, although the effect detects as such, but possibly an alteration of space itself?  Perhaps compactifycation on an almost infitismal scale?

     From what I've been able to gather, it would detect as actual thrust due to movement of the test article, due to the tiny amount of actual distortion, it wouldn't even be visible optically, unless using an extremely sensitive spectrometer for light frequency shifting in the area of the device.  I realize that the power calculated to create such a distortion are several orders of magnitude greater than what is being used here, but Einstien suggested that Gravity should be able to be manipulated in a fashion similar to electromagnetism.

     Is it possible that someone has stumbled onto just such a method USING electromagnetism?  Or, it could be some form of enhancing of the Nuclear Strong or Nuclear Weak forces.  However, I suspect that manipulation of a local EM field of sufficent local intesity, could alter space within the same volume, assuming the concept of Inflation is correct.
Title: Re: EM Drive Developments - related to space flight applications - Thread 2
Post by: tchernik on 03/23/2015 05:44 pm
Ok, here's a crazy idea;

     Is it not possible that what is being seen with the EM drive is not so much actual thrust, although the effect detects as such, but possibly an alteration of space itself?  Perhaps compactifycation on an almost infitismal scale?

If that's the case, then it could be a good idea to stick fully enclosed accelerometers close to the Emdrive, at several distances, for finding out if there is any residual effect in its vicinity...

Has anyone done such a thing?
Title: Re: EM Drive Developments - related to space flight applications - Thread 2
Post by: Stormbringer on 03/23/2015 06:14 pm
Ok, here's a crazy idea;

     Is it not possible that what is being seen with the EM drive is not so much actual thrust, although the effect detects as such, but possibly an alteration of space itself?  Perhaps compactifycation on an almost infitismal scale?

If that's the case, then it could be a good idea to stick fully enclosed accelerometers close to the Emdrive, at several distances, for finding out if there is any residual effect in its vicinity...

Has anyone done such a thing?

I think it might be worth doing but i see a few problems.

The current investigation at the device itself is attempting to detect micro-newton scale thrust signals. gravity effects fall of drastically at distance. so presumably any gravitic/spacetime distortion effect at distance would be all that much smaller and therefore that much harder to detect.

Dr White's other experiment is based on just such an effect and it has not risen according to publicly disclosed reports comfortably above sigma and that laser interferometry is extremely sensitive.

The current activity is under deadline pressure. I think they have until the end of the month to get a 100 micro-newton signal to justify independent replication efforts. This makes changing the protocol drastically perhaps a destructive course of action. Something does need to be done but probably on stuff like the power input or waveforms or the device itself rather than the detection protocol. Unless it can be done without delaying the work or invalidating the data.
Title: Re: EM Drive Developments - related to space flight applications - Thread 2
Post by: Rodal on 03/23/2015 06:22 pm
Hey @Rodal, would you mind telling us more about what sources and methods you are using or have developed that allowed you to generate these original and colorful simulations in these posts:

http://forum.nasaspaceflight.com/index.php?topic=36313.msg1348295#msg1348295
http://forum.nasaspaceflight.com/index.php?topic=36313.msg1340906#msg1340906
http://forum.nasaspaceflight.com/index.php?topic=36313.msg1340909#msg1340909
http://forum.nasaspaceflight.com/index.php?topic=36313.msg1341244#msg1341244
http://forum.nasaspaceflight.com/index.php?topic=36313.msg1344664#msg1344664

It is quite evident that over the course of this lengthy discussion, you have developed tools and methods which can be helpful to many others. Can you give us a summary of what you can do now?
Yes, it would be a good idea to summarize them (when I have a chance to do it  :)  )

My analysis is all based on exact solutions based on classical Electromagnetism (*), which I have discussed earlier in the thread. 

I think that @Notsosureofit may have followed some of my earlier discussions.

I wrote Mathematica code to numerically solve the eigenvalue problems in the exact solutions and to plot the solutions.

It is a work in progress: I started with contour plots for fields and later on I wrote the code for the vector plots.

Since the natural coordinates of the exact solution are in spherical coordinates attached to the cone, to plot these fields in Mathematica one has to transform the coordinates and the vectors to a Cartesian field x, y, z.  This transformation actually took more time to get just right (because of nonlinearities involving SquareRoots and ArcTan functions that have multiple values) than the time it took to get the exact solutions.

In the course of one of our latest post discussions with @Frobnicat I started by showing the vector components of E and B to make a point about Poynting's vector (no pun intended) and I told myself: hey why don't I just calculate and plot Poynting's vector ? (NASA Eagleworks did not show us COMSOL plots of Poynting's vector: don't know whether COMSOL can do this easily ...)

My next step was going to be to calculate more exact solutions (including the dielectric) but I am spending more time on these plots.  For example, depending on the discussion it may be interesting to plot Maxwell stress tensors components and/or the components of the 3+1 spacetime relativistic energy-stress tensor ...

And I'm immediately interested in examining Poynting's vector direction for a number of modes in NASA Eagleworks truncated cone to see whether there is any mode clearly pointing in the direction of the big base.  So far all the modes I have examined have a Poyinting's vector very clearly directed towards the small base and other ones have a Poynting's vector that may be zero or very small in the opposite direction.

Which means that I should also write Mathematica code to integrate Poynting's vector for the cases in which it is not visually clear which way the overall integral over the volume is pointing.

______________
(*) As discussed previously, for example the electromagnetic force depends on the derivative of Poynting's vector with respect to time and on the divergence of Maxwell's stress tensor.  Poynting's vector goes like (Cos[omega*t])^2 so its derivative with respect to time (2 Cos[omega*t]*Sin[omega*t]) should average zero over each half 2Pi/omega cycle.  The divergence of Maxwell's stress tensor should be zero if there are no electromagnetic sources inside the cavity.  (Notice that Brandenburg assumes sources inside the cavity to arrive at his explanation).
I just made some minor improvements to the he Poynting vector plotting code and thereby updated the images shown on http://forum.nasaspaceflight.com/index.php?topic=36313.msg1348295#msg1348295
Title: Re: EM Drive Developments - related to space flight applications - Thread 2
Post by: JasonAW3 on 03/23/2015 06:39 pm
Ok, here's a crazy idea;

     Is it not possible that what is being seen with the EM drive is not so much actual thrust, although the effect detects as such, but possibly an alteration of space itself?  Perhaps compactifycation on an almost infitismal scale?

If that's the case, then it could be a good idea to stick fully enclosed accelerometers close to the Emdrive, at several distances, for finding out if there is any residual effect in its vicinity...

Has anyone done such a thing?

I think it might be worth doing but i see a few problems.

The current investigation at the device itself is attempting to detect micro-newton scale thrust signals. gravity effects fall of drastically at distance. so presumably any gravitic/spacetime distortion effect at distance would be all that much smaller and therefore that much harder to detect.

Dr White's other experiment is based on just such an effect and it has not risen according to publicly disclosed reports comfortably above sigma and that laser interferometry is extremely sensitive.

The current activity is under deadline pressure. I think they have until the end of the month to get a 100 micro-newton signal to justify independent replication efforts. This makes changing the protocol drastically perhaps a destructive course of action. Something does need to be done but probably on stuff like the power input or waveforms or the device itself rather than the detection protocol. Unless it can be done without delaying the work or invalidating the data.

However, if the effect is not gravitic in nature but simply a distortion of space, gravity itself may not be involved but the electromagnetic forces themselves may be causing a localized distortion of space, either compression in front or expansion in back.  again, this assumes the theory of Spacial Inflation is accurate.
Title: Re: EM Drive Developments - related to space flight applications - Thread 2
Post by: Stormbringer on 03/23/2015 06:41 pm
Ok, here's a crazy idea;

     Is it not possible that what is being seen with the EM drive is not so much actual thrust, although the effect detects as such, but possibly an alteration of space itself?  Perhaps compactifycation on an almost infitismal scale?

     From what I've been able to gather, it would detect as actual thrust due to movement of the test article, due to the tiny amount of actual distortion, it wouldn't even be visible optically, unless using an extremely sensitive spectrometer for light frequency shifting in the area of the device.  I realize that the power calculated to create such a distortion are several orders of magnitude greater than what is being used here, but Einstien suggested that Gravity should be able to be manipulated in a fashion similar to electromagnetism.

     Is it possible that someone has stumbled onto just such a method USING electromagnetism?  Or, it could be some form of enhancing of the Nuclear Strong or Nuclear Weak forces.  However, I suspect that manipulation of a local EM field of sufficent local intesity, could alter space within the same volume, assuming the concept of Inflation is correct.

Gads. Given the thread history I hesitate to even go here. If we are talking about  strong interactions and some sort of gravity connection there is something I always wanted to try -but it is straight out of Ko0k central territory. Fringe lore says that bismuth has an unusual nucleonic structure that slightly exposes normally inaccessible strong force flux lines to potential manipulation and that this would have a gravity or warping effect under the assertion that the strong force and gravity are related. If I had the money to start a kook garden shed laboratory I would buy some bismuth and try zapping it in creative ways to see if there was anything to it.  I might try to melt it and aline the structure with magnets and electricity to get the nuclei aligned with each other to perhaps amplify it's kook-alleged  properties.

At any rate if a bit of bismuth found it's way into some of the frustrum replication efforts i certainly would not mind. If nothing interesting happens no one need ever know such a thing was seriously tried. No reputations or credentials need be risked.
Title: Re: EM Drive Developments - related to space flight applications - Thread 2
Post by: Mulletron on 03/23/2015 06:44 pm
If I'm understanding this right is there a kind of catch 22 at work here. To validate if this drive works or not you need a greater power input, but to obtain access to a greater power input it has to be proved that it works first?
I fear it is worse then that, because when you resort to more power input the thermal effects on the frustrum will also increase, with a dramatic negative effect on the Q.

Yes I believe there is enormous value in doing low power experiments.
http://forum.nasaspaceflight.com/index.php?topic=36313.msg1331993#msg1331993

As power is increased, so too are thermal effects. I think the disconnect here with the LDS at Nasa is that the vacuum chamber and LDS setup is a "complex system" with many (hidden) variables (convection, unbalanced arm, thermal expansion....). Many of these variables have been sussed out in this thread and the previous one.

I firmly believe that low power is the way to go for now, in order to tease out a subtle effect * (which has the potential to be studied and optimized if conclusively verified) which, under high power experiments will get overwhelmed by heat.

I've taken my own advice, and taken a gamble on using low power and using what I've learned from Cavendish (http://en.wikipedia.org/wiki/Cavendish_experiment), hopefully I can build a sensitive (and simple enough) balance. Honestly for the sake of not screwing this up (for others), I prefer to keep hubris at bay and admit to what I know and what I don't know. So yes, apart from the RF stuff I'm doing (which is my day job) the balance and force measurements is uncharted territory for me.

As they say, "No man is an island" ** and I want this to be a NSF effort (and I want others to build too dangit!) because until these copper kettles go bang or bust, there is a huge QUESTION which must be answered.

Gentlemen, we will get our answer to these questions one of these days.


*5th force? or unification of EM and Gravitation? http://en.wikipedia.org/wiki/Fundamental_interaction
I'm a dreamer, I know......
**What I would give to have selfless professionals (who have open minds) donate some time and effort to this problem. Seriously.
Title: Re: EM Drive Developments - related to space flight applications - Thread 2
Post by: Stormbringer on 03/23/2015 06:49 pm


However, if the effect is not gravitic in nature but simply a distortion of space, gravity itself may not be involved but the electromagnetic forces themselves may be causing a localized distortion of space, either compression in front or expansion in back.  again, this assumes the theory of Spacial Inflation is accurate.

I am no expert but I think there is no way that any distortion of space time is anything but gravitic due to the EEP. The equivalence principle states that accelerations are indistinguishable from accelerations due to gravity. Einstein says gravity is distorted space/time. Anything that distorts space is a gravity drive. That's my understanding of it.
Title: Re: EM Drive Developments - related to space flight applications - Thread 2
Post by: ThinkerX on 03/23/2015 11:04 pm
For my part, the thought that keeps crossing what's left of my mind is this device may be employing some sort of 'bouncing photon effect' - but that leads directly to a head-on collision with conservation of energy, which means that can't be the case, and yet the presence of photon's, bouncing or otherwise inside the frustum is one of the few things we do know for certain about this device.   I dunno...is there some sort of exotic loophole in the laws of thermodynamics this device could be taking advantage of? 
Title: Re: EM Drive Developments - related to space flight applications - Thread 2
Post by: dustinthewind on 03/24/2015 09:19 am

I've taken my own advice, and taken a gamble on using low power and using what I've learned from Cavendish (http://en.wikipedia.org/wiki/Cavendish_experiment), hopefully I can build a sensitive (and simple enough) balance.


I was thinking earlier if you all might have designed a flexible beam and have a mirror mounted on the engine.  You could then use interferometry to get very accurate changes in position when the beam flexes if there is a force acting on it.  There might need to be a lot of dampening to reduce noise from vibrations though. 


Another thought hit me regarding why this drive would make a force.  It strikes me as similar to the idea in the thread "E/M propellant-less propulsion using delayed information/dielectrics (patent)" in that there may be some time delay of information going on inside the cavity plates.  It appears there are circulating currents in the device due the changing magnetic fields.  However, that information doesn't travel instantaneously.  Maybe what could be happening is that the circulating currents in the bottom plate observe the circulating current in the top plate as circulating in the same direction and so they are attracted to the top plate.  However, the top plate may observe the bottom plate having current circulating in the opposite direction and so it is repelled from the bottom plate.  The result is a unidirectional force.  In other words the currents in the top and bottom plates are about 90 degrees out of phase? 

I don't know if the delay in propulsion fits with that but maybe it takes a bit for the cavity to reach its final resonant state and then a while for the radiation to die away inside. 

If not the top plate interacting with the bottom plate then maybe the top plate interacting some distance down the side walls such that the magnetic fields are 90 degrees out of phase in time. 
Title: Re: EM Drive Developments - related to space flight applications - Thread 2
Post by: Left Field on 03/24/2015 02:02 pm

I've taken my own advice, and taken a gamble on using low power and using what I've learned from Cavendish (http://en.wikipedia.org/wiki/Cavendish_experiment), hopefully I can build a sensitive (and simple enough) balance.


I was thinking earlier if you all might have designed a flexible beam and have a mirror mounted on the engine.  You could then use interferometry to get very accurate changes in position when the beam flexes if there is a force acting on it.  There might need to be a lot of dampening to reduce noise from vibrations though. 


Another thought hit me regarding why this drive would make a force.  It strikes me as similar to the idea in the thread "E/M propellant-less propulsion using delayed information/dielectrics (patent)" in that there may be some time delay of information going on inside the cavity plates.  It appears there are circulating currents in the device due the changing magnetic fields.  However, that information doesn't travel instantaneously.  Maybe what could be happening is that the circulating currents in the bottom plate observe the circulating current in the top plate as circulating in the same direction and so they are attracted to the top plate.  However, the top plate may observe the bottom plate having current circulating in the opposite direction and so it is repelled from the bottom plate.  The result is a unidirectional force.  In other words the currents in the top and bottom plates are about 90 degrees out of phase? 

I don't know if the delay in propulsion fits with that but maybe it takes a bit for the cavity to reach its final resonant state and then a while for the radiation to die away inside. 

If not the top plate interacting with the bottom plate then maybe the top plate interacting some distance down the side walls such that the magnetic fields are 90 degrees out of phase in time.

Hi dustinthewind. Just so you know, I did link to your thread from here on the 28th Feb as I thought it could be relevant or spark an idea.

@aero

Is there any way the community here can help by donating some computer time to run different simulation scenarios? Can they be packaged up so that an inexperienced user can just run them and then email/post the results?
Title: Re: EM Drive Developments - related to space flight applications - Thread 2
Post by: JasonAW3 on 03/24/2015 02:50 pm
And here's a sample of the spectrum analyzer test. All this test involves is injecting a calibrated 0dbm input signal to the frustum, then seeing what comes out the other side on the spectrum analyzer.

*HP 83752B Sweep generator @0dbm
*Agilent E4443A spectrum analyzer
*Gigatronics 8542C power meter w/*80301A sensor
(2) 6' high quality test cables
*=Calibrated by a lab

Combine this with the VSWR data and the behavior of the unloaded frustum between 2400-2500mhz becomes apparent. The points where the VWSR is really poor is evident in the amplitude response, seen in the spectrum analyzer shots, so these two tests validate each other.

Rest is here:
https://drive.google.com/folderview?id=0B4PCfHCM1KYoUEx5dzlVTG81a2c&usp=sharing&tid=0B4PCfHCM1KYoTXhSUTd5ZDN2WnM

Other frequencies here:
(disclaimer, my E field probe is not optimized for all these frequencies, still useful to see what other frequencies couple to the cavity with low VSWR using my 31mm probe/cavity combination)
The best peaks of other frequencies are labeled similar to the bottom screen shot.
https://drive.google.com/folderview?id=0B4PCfHCM1KYoVkRzUGNuMVBLbVk&usp=sharing&tid=0B4PCfHCM1KYoTXhSUTd5ZDN2WnM

Has anyone considered an optical spectrum analysis?  Just a point of curiosity.  While there will be a predicitbale amount of infrared that will show up during testing, I am curious as to weather or noth the amount on either end of the device is either higher or lower than would be predicted.  If expansion or compression of Space is involved, then there should be a difference in the predicted IR, or infact ANY optical wavelengths, output on one or possibly both ends of the device.

(Added: "or infact ANY optical wavelengths," on 4/24/2015)
Title: Re: EM Drive Developments - related to space flight applications - Thread 2
Post by: aero on 03/24/2015 03:06 pm
Quote
@aero

Is there any way the community here can help by donating some computer time to run different simulation scenarios?

I don't know. The physical dimensions of the thruster models range over several orders of magnitude from the smallest dimension of interest to the overall size of the cavity. That makes it almost a supercomputer problem. I could do better work with a faster computer but that computer runs $2500 USD and I'm sure there are a lot of members here who would like to have a new computer. And that faster computer would only allow me to set-up problems at low resolution which should be verified on a supercomputer.
Quote

Can they be packaged up so that an inexperienced user can just run them and then email/post the results?

If you look here, http://ab-initio.mit.edu/wiki/index.php/Meep_Tutorial (http://ab-initio.mit.edu/wiki/index.php/Meep_Tutorial), you will see that the Meep tutorial strategy is to pose and explain example programs. That makes it pretty easy to copy/paste the examples and run them. The trick is understanding what you have when you are finished and the online support for questions about the Meep results is not very helpful.

Yes, problems could be packaged up and ran elsewhere. Figuring out what the answers mean, if anything, is the trick.
Title: Re: EM Drive Developments - related to space flight applications - Thread 2
Post by: Rodal on 03/24/2015 03:58 pm
....
 Maybe what could be happening is that the circulating currents in the bottom plate observe the circulating current in the top plate as circulating in the same direction and so they are attracted to the top plate.  However, the top plate may observe the bottom plate having current circulating in the opposite direction and so it is repelled from the bottom plate.  The result is a unidirectional force.  In other words the currents in the top and bottom plates are about 90 degrees out of phase? 

I don't know if the delay in propulsion fits with that but maybe it takes a bit for the cavity to reach its final resonant state and then a while for the radiation to die away inside. 

If not the top plate interacting with the bottom plate then maybe the top plate interacting some distance down the side walls such that the magnetic fields are 90 degrees out of phase in time.
We can readily calculate the electromagnetic fields (and associated electromagnetic quantities) for the EM Drive.
It takes less than 3 seconds in my PC.
I attach plots for the mode currently tested by NASA Eagleworks in a partial vacuum, showing the magnetic field at the Big and Small bases and the electric field through the truncated cone.

I don't understand the (analogy ?) in the following statement:

Quote
the circulating currents in the bottom plate observe the circulating current in the top plate as circulating in the same direction and so they are attracted to the top plate.  However, the top plate may observe the bottom plate having current circulating in the opposite direction and so it is repelled from the bottom plate.  The result is a unidirectional force. 

How are the circulating currents in the bottom plate "observing" the circulating currents in the top plate?
The magnetic fields are out of phase (see the plots below).  Why would the bottom circulating current "observe" the top one as circulating in the same direction: "in phase"?


And most important, how does this get around the conservation of momentum problem (if one considers the EM Drive as a closed system with no internal sources) ?

Assuming no internal magnetic sources (no magnetic monopoles) and no internal electric sources, the divergence of Maxwell's stress tensor is zero (due to the lack of internal sources inside the cavity).
The derivative with respect to time of Poynting's vector is zero (if the electromagnetic fields are a harmonic function of time). These conditions lead to no electromagnetic force on the center of mass, even when considering a fully general-relativistic formulation of the principle of conservation of energy-momentum (http://en.wikipedia.org/wiki/Electromagnetic_stress%E2%80%93energy_tensor#Conservation_laws ), because the divergence (in 3D+1 spacetime) of the stress–energy tensor is zero under those previously-stated conditions.

In order to have a force and acceleration of the EM Drive one needs electromagnetic sources inside the EM Drive cavity (as assumed for example in Brandenburg's equations) and/or the electromagnetic fields to be a nonlinear non-harmonic function of time.
Title: Re: EM Drive Developments - related to space flight applications - Thread 2
Post by: Mulletron on 03/24/2015 04:06 pm
Struck out on finding a lathe to use. My belt sander did a fantastic job too. So now I have 2, 1" thick x 6.25" wide HDPE discs. Now I have to figure out how to mount them....

Where there's a will, there's a way.



Title: Re: EM Drive Developments - related to space flight applications - Thread 2
Post by: Rodal on 03/24/2015 06:38 pm
Struck out on finding a lathe to use. My belt sander did a fantastic job too. So now I have 2, 1" thick x 6.25" wide HDPE discs. Now I have to figure out how to mount them....

Where there's a will, there's a way.

QUESTION 1: Before mounting the discs, could you please run a couple of tests without the HDPE discs?

If, so I can send you a post showing the mode shapes at the two frequencies near 2.45 GHz and why .

There is a mode shape at 2.49 GHz (or at 2.46 GHz according to NASA's COMSOL calculations) that should produce no force without the dielectric disc (because the Poynting vector practically cancels out)

There is a mode shape at 2.46 GHz (or at 2.41 GHz according to NASA's COMSOL calculations) that may produce an electromagnetic force without the dielectric disc (because the Poynting vector does not cancel out)

QUESTION 2: Do you have access to a thermal IR camera (to identify the mode shapes, etc.)  ?
Title: Re: EM Drive Developments - related to space flight applications - Thread 2
Post by: jmossman on 03/24/2015 06:53 pm
Struck out on finding a lathe to use. My belt sander did a fantastic job too. So now I have 2, 1" thick x 6.25" wide HDPE discs. Now I have to figure out how to mount them....

Where there's a will, there's a way.

Congratulations on your build progress!

As for the HDPE mounting solution...

If the initial intention is to replicate the Eagleworks' configuration, I'd propose mimicking the Eagleworks' mounting technique.  I've seen at least two different bolt hole patterns in posted images, and some of the verbiage suggesting that different bolt hole patterns are best for different excited modes.  (as an aside, since the bolt holes pierce both the dielectric and the copper frustum's plate, I've wondered what the RF field looks like outside of the cavity near those holes; I don't recall seeing any bolt holes in the previously posted/discussed meep sims)

http://forum.nasaspaceflight.com/index.php?action=dlattach;topic=36313.0;attach=803831 (http://forum.nasaspaceflight.com/index.php?action=dlattach;topic=36313.0;attach=803831)
Quote
(There are three ~1.00" 1/4-20 nylon bolts mounted on a ~2.00" radius spaced every 120 degrees that hold the first PE disc to the PCB end cap.   There is then a layer of 3/4" wide office scotch tape at the interface between the first and second PE discs and the center 1/4"-20 nylon bolt that hold second PE disc to the first PE disc.)
Quote
it looks like there is a high E-field volume where this center nylon bolt hangs out while running in the TM212 resonant mode.

(Image 1 below)

http://forum.nasaspaceflight.com/index.php?topic=36313.msg1335785#msg1335785 (http://forum.nasaspaceflight.com/index.php?topic=36313.msg1335785#msg1335785)
Quote
we tested the copper frustum in its TM010 mode and mounted a 5.0 inch OD by 1.0" thick PTFE disk at the center of the large OD end cap of the copper frustum with one 1/4-20 nylon bolt.


(Image 2 below)




[Mod Edit:  Images were stretching page, moved to attachments.  Please attach rather than embed in the future.]
Title: Re: EM Drive Developments - related to space flight applications - Thread 2
Post by: Rodal on 03/24/2015 07:02 pm
Struck out on finding a lathe to use. My belt sander did a fantastic job too. So now I have 2, 1" thick x 6.25" wide HDPE discs. Now I have to figure out how to mount them....

Where there's a will, there's a way.

Congratulations on your build progress!

As for the HDPE mounting solution...

If the initial intention is to replicate the Eagleworks' configuration, I'd propose mimicking the Eagleworks' mounting technique.  I've seen at least two different bolt hole patterns in posted images, and some of the verbiage suggesting that different bolt hole patterns are best for different excited modes. ...

@Mulletron:

Please clarify whether your intention is indeed to identically "replicate Eagleworks" tests (as per the post above).  In other words: are you going to be testing at exactly the same frequencies (under the same configuration) as NASA Eagleworks?

My understanding was that you were not going to (initially at least) be testing at the same frequencies as NASA Eagleworks but instead you were going to be testing near 2.45 GHz.

If you are going to be testing near ~2.45 GHz (instead of ~1.9 Ghz) then the mode shapes are going to be different, and therefore one should not use the same bolt mounting configuration as Eagleworks.

 I can send you images of the mode shapes at nearby frequencies, if you confirm at what frequency you are going to test.


Title: Re: EM Drive Developments - related to space flight applications - Thread 2
Post by: Mulletron on 03/24/2015 07:08 pm
Struck out on finding a lathe to use. My belt sander did a fantastic job too. So now I have 2, 1" thick x 6.25" wide HDPE discs. Now I have to figure out how to mount them....

Where there's a will, there's a way.

QUESTION 1: Before mounting the discs, could you please run a couple of tests without the HDPE discs?

If, so I can send you a post showing the mode shapes at the two frequencies near 2.45 GHz and why .

There is a mode shape at 2.49 GHz (or at 2.46 GHz according to NASA's COMSOL calculations) that should produce no force without the dielectric disc (because the Poynting vector practically cancels out)

There is a mode shape at 2.46 GHz (or at 2.41 GHz according to NASA's COMSOL calculations) that may produce an electromagnetic force without the dielectric disc (because the Poynting vector does not cancel out)

QUESTION 2: Do you have access to a thermal IR camera (to identify the mode shapes, etc.)  ?

I'm going to try and run it with and without the HDPE discs in there. I can zero in on modes using the test equipment at work....but there's no way to verify really.

At home, there's no way of zeroing in on and exciting particular modes during the force testing. I'm just brute forcing this with wideband RF. I'm only interested in frequencies between 2400-2500mhz. I'm going for the same effect as a magnetron, but with much less power. So clearly defined mode shapes are out for now.

If a force is evident while the frustum is on the balance, I'll invest in one of those USB programmable signal generators. But for now, I'll just be happy if it even moves. I'm using the crawl, walk, run approach to things.

I don't have an IR camera btw.
Title: Re: EM Drive Developments - related to space flight applications - Thread 2
Post by: Mulletron on 03/24/2015 07:17 pm
Struck out on finding a lathe to use. My belt sander did a fantastic job too. So now I have 2, 1" thick x 6.25" wide HDPE discs. Now I have to figure out how to mount them....

Where there's a will, there's a way.

Congratulations on your build progress!

As for the HDPE mounting solution...

If the initial intention is to replicate the Eagleworks' configuration, I'd propose mimicking the Eagleworks' mounting technique.  I've seen at least two different bolt hole patterns in posted images, and some of the verbiage suggesting that different bolt hole patterns are best for different excited modes.  (as an aside, since the bolt holes pierce both the dielectric and the copper frustum's plate, I've wondered what the RF field looks like outside of the cavity near those holes; I don't recall seeing any bolt holes in the previously posted/discussed meep sims)


My leading intention is to use the 3 Nylon bolt approach like Paul March, but it really pains me to drill holes into perfectly good dielectric resonators. All the Nylon bolts I have at the moment aren't long enough, and if the holes are too big, RF can escape.

I'm looking for a better way if there is one.
Title: Re: EM Drive Developments - related to space flight applications - Thread 2
Post by: Rodal on 03/24/2015 07:22 pm
Struck out on finding a lathe to use. My belt sander did a fantastic job too. So now I have 2, 1" thick x 6.25" wide HDPE discs. Now I have to figure out how to mount them....

Where there's a will, there's a way.

Congratulations on your build progress!

As for the HDPE mounting solution...

If the initial intention is to replicate the Eagleworks' configuration, I'd propose mimicking the Eagleworks' mounting technique.  I've seen at least two different bolt hole patterns in posted images, and some of the verbiage suggesting that different bolt hole patterns are best for different excited modes.  (as an aside, since the bolt holes pierce both the dielectric and the copper frustum's plate, I've wondered what the RF field looks like outside of the cavity near those holes; I don't recall seeing any bolt holes in the previously posted/discussed meep sims)


My leading intention is to use the 3 Nylon bolt approach like Paul March, but it really pains me to drill holes into perfectly good dielectric resonators. All the Nylon bolts I have at the moment aren't long enough, and if the holes are too big, RF can escape.

I'm looking for a better way if there is one.

Why don't you try this:

A single bolt through the center (which for sure is a node ).  See whether that's enough.

If it is not enough:

 Adhesion could be permanent and removing the dielectric from the copper base plate may involve damage.

Primers are solvent-based systems in which a reactive species is dissolved. Applied to a surface using a brush or spray, the primer’s solvent evaporates, leaving behind the reactive species on the substrate. The reactive species acts as a linking pin or bridge between an adhesive and the substrate Polyolefin primers are frequently used on hard-to-bond substrates joined with traditional and/or light curing cyanoacrylates ("superGlue").

Look at page 51 of this report from Loctite: http://www.henkelna.com/us/content_data/237471_LT2197_Plastic_Guide_v6_LR7911911.pdf

which are specifically recommended for polyethylene (a very hard to bond polymer)

(Ethyl Cyanoacrylate) Loctite 401 Prism from Amazon:

http://www.amazon.com/Loctite-Instant-Adhesive-Prism-Bottle/dp/B006GOKRSY/ref=pd_sim_sbs_indust_1?ie=UTF8&refRID=14RMFHYMSB2780RR33CY

Primer for Loctite 401 Prism from Amazon:

http://www.amazon.com/Loctite-LOC-18396-770-Cyanoacrylate-Adhesive/dp/B001OBQ8VO/ref=sr_1_fkmr0_1?s=hi&ie=UTF8&qid=1427237585&sr=1-1-fkmr0&keywords=Loctite%C2%AE+770%E2%84%A2+Prism%C2%AE+Primer
Title: Re: EM Drive Developments - related to space flight applications - Thread 2
Post by: Notsosureofit on 03/24/2015 07:34 pm
??  Replaceable end plates ?  One w/ and one w/o glued disks ?

Check w/ http://www.masterbond.com/?utm_source=jan14&utm_medium=email&utm_content=signature&utm_campaign=cpf
Title: Re: EM Drive Developments - related to space flight applications - Thread 2
Post by: Mulletron on 03/24/2015 07:39 pm
Something cool I just found and have been fiddling around with:

http://www.i-logic.com/conecalc.htm (http://www.i-logic.com/conecalc.htm) (online version)
http://www.i-logic.com/conecalc/conecalc.htm (http://www.i-logic.com/conecalc/conecalc.htm) (desktop download)

Plugged in the inside dimensions of the Eagleworks/Shawyer replication frustum, see screenshot. It generated a cut sheet. I added in some dimensions and a .25" offset in red, but not dimensioned. The sheet is 24"x48". The green rectangle is the sheet rotated for more efficient use of the copper.

http://basiccopper.com/22mil16oz24x4.html (http://basiccopper.com/22mil16oz24x4.html)

Files are here: (click this line) (https://drive.google.com/folderview?id=0B4PCfHCM1KYoflVVNmtieXdBY0NfM245bFpVWFFJal9ZcFR5R1VsZ1hXdnJnWEd2OUx5YVk&usp=sharing&tid=0B4PCfHCM1KYoTXhSUTd5ZDN2WnM)




[Mod Edit:  Link was stretching page, changed to hyperlink]
Title: Re: EM Drive Developments - related to space flight applications - Thread 2
Post by: flux_capacitor on 03/24/2015 07:54 pm
If your copper in the endplate is thick enough, surface roughening of the copper and of the HD PE results in mechanical interlocking sites and causes bond strength to increase dramatically.

I asked this before but my question wasn't answered (perhaps it is a stupid question):
Wouldn't roughening the copper internal surface of the end plates (to better bond the glue) dramatically reduce the surface reflectivity, hence the Q factor of the cavity?
Title: Re: EM Drive Developments - related to space flight applications - Thread 2
Post by: jmossman on 03/24/2015 08:02 pm
My leading intention is to use the 3 Nylon bolt approach like Paul March, but it really pains me to drill holes into perfectly good dielectric resonators. All the Nylon bolts I have at the moment aren't long enough, and if the holes are too big, RF can escape.

I'm looking for a better way if there is one.

http://forum.nasaspaceflight.com/index.php?topic=36313.msg1335766#msg1335766
Quote
Quote
tried epoxy and superglue bonding the PE and PTFE discs to the frustum end-caps, but these two plastics just happen to be the slickest and hardest plastics to bond to anything else.  Drat!
The marine adhesive made by 3M  (5200 or 5220 fast cure) will stick to anything

I too suspect 5200/5220 will work, but would eliminate any option of removing the HDPE discs without causing some damage.   High-temperature automotive RTV also comes to mind.  How transparent 5200/5220 or RTV are at these RF frequencies, I do not know.
Title: Re: EM Drive Developments - related to space flight applications - Thread 2
Post by: dustinthewind on 03/24/2015 08:11 pm

I don't understand the (analogy ?) in the following statement:

Quote
the circulating currents in the bottom plate observe the circulating current in the top plate as circulating in the same direction and so they are attracted to the top plate.  However, the top plate may observe the bottom plate having current circulating in the opposite direction and so it is repelled from the bottom plate.  The result is a unidirectional force. 

How are the circulating currents in the bottom plate "observing" the circulating currents in the top plate?
The magnetic electromagnetic fields are out of phase (see the plots below).  Why would the bottom circulating current "observe" the top one as circulating in the same direction: "in phase"?

(Dustinthewind - [bold with my name is me] in other words information only travels at the speed of light or , dx=c*dt.  The key to the image attached is:
Reference Numeral - Diagram illustrations as shown in FIGS. 1-2:
1 - Electromagnetic force
2 - Current in upper wire
3 - Current in middle wire
4 - Current in lower wire
5 - Apparent current of the upper wire
6 - Apparent current of the middle wire
7 - Apparent current of the lower wire
8 - Frame one of six
9 - Frame two of six
10 - frame three of six
11 - frame four of six
12 - frame five of six
13 - frame six of six

The image only considers parallel wires but the idea could be extended to make the wires equivalent to the circulating currents in the top and bottom plates or maybe the top plate and some distance down the sidewalls.

P.S. the diagram considers three wires but the simplest to consider is only two wires.  Ignore one color if you like as they are color coded depending on the wire and limit to two wires.  In the three wire diagram the top and bottom wires are out of phase 180 degrees so there is no unidirectional force between them.  The wires next to each other are only 90 degrees out of phase.  In time what happens is the bottom wire observes the current above it as moving in the same direction = attraction.  The top wire observes the current below it as moving in the opposite direction = repulsion
)


And most important, how does this get around the conservation of momentum problem (if one considers the EM Drive as a closed system with no internal sources) ?

(Dustinthewind - here is a quote from the other thread, "A scientific paper that clarifies how newtons 3rd law does not apply to the time delay of information and how it can be used for electromagnetic propulsion.  It provides a mathematical background for the time delayed magnetic fields but first illustrating how the static equations miss the effect.  https://scholar.google.com/scholar?cluster=7136673109349846373&hl=en&as_sdt=0,48" it is titled, "Newton's Third Law in the Framework of Special Relativity" ) - Dustinthewind - Please disregard this reference as I was in error in assuming this paper was dealing with two current loops changing in time 90 degrees out of phase.

Assuming no internal magnetic sources (no magnetic monopoles) and no internal electric sources, the divergence of Maxwell's stress tensor is zero (due to the lack of internal sources inside the cavity).
The derivative with respect to time of Poynting's vector is zero (if the electromagnetic fields are a harmonic function of time). These conditions lead to no electromagnetic force on the center of mass, even when considering a fully general-relativistic formulation of the principle of conservation of energy-momentum (http://en.wikipedia.org/wiki/Electromagnetic_stress%E2%80%93energy_tensor#Conservation_laws ), because the divergence (in 3D+1 spacetime) of the stress–energy tensor is zero under those previously-stated conditions.

In order to have a force and acceleration of the EM Drive one needs electromagnetic sources inside the EM Drive cavity (as assumed for example in Brandenburg's equations) and/or the electromagnetic fields to be a nonlinear non-harmonic function of time.

(Dustinthewind - It sounds like you have already considered the time dependent interactions from the sound of it so maybe I'm wrong in speculating on the propulsive effect.)
Title: Re: EM Drive Developments - related to space flight applications - Thread 2
Post by: Rodal on 03/24/2015 09:04 pm
If your copper in the endplate is thick enough, surface roughening of the copper and of the HD PE results in mechanical interlocking sites and causes bond strength to increase dramatically.

I asked this before but my question wasn't answered (perhaps it is a stupid question):
Wouldn't roughening the copper internal surface of the end plates (to better bond the glue) dramatically reduce the surface reflectivity, hence the Q factor of the cavity?

Good point, I presume that surface roughness features with height comparable to the penetration length would have a significant effect on the power loss ratio, so I deleted the mention of roughening to improve adhesion.
The effect of roughening on polyethylene does not seem to have universal acceptance: Loctite (see report below) states that they did not find a statistical difference when bonding LDPE (they don't say anything about roughening HD PE)

That leaves solvent-based primers as a good option to improve adhesion.

Look at page 51 of this report from Henkel North America (Loctite): http://www.henkelna.com/us/content_data/237471_LT2197_Plastic_Guide_v6_LR7911911.pdf

which are specifically recommended for polyethylene (a very hard to bond polymer)

(Ethyl Cyanoacrylate) Loctite 401 Prism from Amazon:

http://www.amazon.com/Loctite-Instant-Adhesive-Prism-Bottle/dp/B006GOKRSY/ref=pd_sim_sbs_indust_1?ie=UTF8&refRID=14RMFHYMSB2780RR33CY

Primer for Loctite 401 Prism from Amazon:

http://www.amazon.com/Loctite-LOC-18396-770-Cyanoacrylate-Adhesive/dp/B001OBQ8VO/ref=sr_1_fkmr0_1?s=hi&ie=UTF8&qid=1427237585&sr=1-1-fkmr0&keywords=Loctite%C2%AE+770%E2%84%A2+Prism%C2%AE+Primer

THIS GLUE IS STRONG AND FAST ACTING - MAKE SURE TO WEAR GLOVES AND PROTECT YOURSELF
Title: Re: EM Drive Developments - related to space flight applications - Thread 2
Post by: Rodal on 03/25/2015 12:42 am
If your copper in the endplate is thick enough, surface roughening of the copper and of the HD PE results in mechanical interlocking sites and causes bond strength to increase dramatically.

I asked this before but my question wasn't answered (perhaps it is a stupid question):
Wouldn't roughening the copper internal surface of the end plates (to better bond the glue) dramatically reduce the surface reflectivity, hence the Q factor of the cavity?

Good point, I presume that surface roughness features with height comparable to the penetration length would have a significant effect on the power loss ratio, so I deleted the mention of roughening to improve adhesion.
The effect of roughening on polyethylene does not seem to have universal acceptance: Loctite (see report below) states that they did not find a statistical difference when bonding LDPE (they don't say anything about roughening HD PE)

That leaves solvent-based primers as a good option to improve adhesion.

Look at page 51 of this report from Henkel North America (Loctite): http://www.henkelna.com/us/content_data/237471_LT2197_Plastic_Guide_v6_LR7911911.pdf

which are specifically recommended for polyethylene (a very hard to bond polymer)

(Ethyl Cyanoacrylate) Loctite 401 Prism from Amazon:

http://www.amazon.com/Loctite-Instant-Adhesive-Prism-Bottle/dp/B006GOKRSY/ref=pd_sim_sbs_indust_1?ie=UTF8&refRID=14RMFHYMSB2780RR33CY

Primer for Loctite 401 Prism from Amazon:

http://www.amazon.com/Loctite-LOC-18396-770-Cyanoacrylate-Adhesive/dp/B001OBQ8VO/ref=sr_1_fkmr0_1?s=hi&ie=UTF8&qid=1427237585&sr=1-1-fkmr0&keywords=Loctite%C2%AE+770%E2%84%A2+Prism%C2%AE+Primer

THIS GLUE IS STRONG AND FAST ACTING - MAKE SURE TO WEAR GLOVES AND PROTECT YOURSELF

Notice that http://www.henkelna.com/us/content_data/237471_LT2197_Plastic_Guide_v6_LR7911911.pdf
shows on page 51 that adhesion to HDPE for Loctite 401 Prism alone gives 50 psi adhesive shear strength, while Loctite 401 Prism   with Loctite® 7701™ Prism® Primer (the medical version of 770 primer) gives 2000 psi adhesive shear strength (40 times greater adhesive shear strength when using the primer).

This adhesive + primer combination should amply exceed the adhesive shear strength to HDPE (and to PTFE "Teflon") of polyurethane-based marine adhesives according to published data.
Title: Re: EM Drive Developments - related to space flight applications - Thread 2
Post by: Rodal on 03/25/2015 11:01 am
https://scholar.google.com/scholar?cluster=7136673109349846373&hl=en&as_sdt=0,48....
....

@frobnicat: please notice this paper, published in The European Physical Journal Plus
November 2014, 129:240; which was brought up by @dustinthewind:

https://scholar.google.com/scholar?cluster=7136673109349846373&hl=en&as_sdt=0,48

that states:

Quote from: Miron Tuval and Asher Yahalom Newton’s Third Law in the Framework of Special
Relativity
Most locomotive systems of today are based on open systems. A rocket sheds exhaust gas to propel itself, a speeding bullet generates recoil. A car pushes the road with the same force that is used to accelerate it, the same is true regarding the interaction of a plane with air and of a ship with water. However, the above relativistic considerations suggest’s a new type of motor which is not based on a open system but rather on a closed one
....
As a final remark we will address the problem of achieving constant force which may be of interest for locomotive applications. A constant force may be achieved by having a direct current in one loop and a current of uniform second derivative on the other. For the choice of values given in table 2 we obtained FT z ∼= 2.74 Newton.
Obviously the switching time may represent some difficulty which one may overcome with advanced enough switching technology perhaps using low resistivity superconducting materials. Another possibility for constructing a relativistic motor is using numerous modular solid-state devices each with fast switching and small current such that an appreciable amount of cumulative forcing will result.

The European Physical Journal arose in 1998 as a merger and continuation of the very prestigious journals Zeitschrift für Physik, Journal de Physique,  Il Nuovo Cimento, and other journals.

The authors are associated with the renowned Isaac Newton Institute for Mathematical Sciences, an international research institute for mathematics and theoretical physics adjoining the Cambridge University Centre for Mathematical Sciences. In 1993 the British mathematician Andrew Wiles announced at the Institute his proof of Fermat's last theorem. Its director as of May 2012 was Cambridge University Professor John Toland (who is famous for formally proving in 1978, Stokes' conjecture on the existence of gravity waves of maximum height on deep water, a previously open problem in mathematical hydrodynamics which dated back to the 19th century).

QUESTION: Can you identify fast switching (for tauc=10 nano seconds -> 2.7 Newtons; hence for tauc=3 microseconds -> 30 microNewtons, but for 100 Amps ?) and/or non-harmonic time-response of the electromagnetic fields in these EM Drive experiments?

(If there would be nonlinear, non-harmonic time response of the electromagnetic fields, then the derivative with respect to time of the Poynting vector would not be null, which indeed would result in a net longitudinal force, all that would be needed is special relativity, without the need of any esoteric physics: no need for Quantum Vacuum esoterica, no need for the "Mach Effect", no need for coupling of electromagnetism with gravity, no need for Unruh radiation, no need for superluminal tachyons, no need for extra dimensions of space (Kaluza-Klein or multidimensional branes), no need to appeal to Dark Energy, etc.)
Title: Re: EM Drive Developments - related to space flight applications - Thread 2
Post by: Notsosureofit on 03/25/2015 01:00 pm
FYI

http://www.technologyreview.com/view/536091/spacecraft-traveling-close-to-light-speed-should-be-visible-with-current-technology-say/?utm_source=feedburner&utm_medium=email&utm_campaign=Feed%3A+arxivblog%2FGmoU+%28The+Physics+arXiv+Blog%29
Title: Re: EM Drive Developments - related to space flight applications - Thread 2
Post by: bad_astra on 03/25/2015 01:52 pm
I wonder if the harmonics of the microwave pulses from an EMDRIVE would sound like a WOW signal
Title: Re: EM Drive Developments - related to space flight applications - Thread 2
Post by: Rodal on 03/25/2015 05:10 pm
....
The copper frustum thrust reversal due to only its dielectric placement came when I was experimenting with the TM010 mode,...
... I think that this mode shape correct designation is TM011 instead of TM010 because:

1) There can be no TMmn0 modes for a truncated cone.  TMmn0 modes need to have a constant electromagnetic field in the longitudinal direction of the cavity.  This is possible for a cylindrical cavity (containing no other dielectrics inside besides the cavity medium) because it has constant geometrical and material properties in the longitudinal direction.  But a conical cavity has variable cross-section in the longitudinal direction, therefore the  TMnn0 mode is not possible.  The first possible mode (if it is not cut-off) is TMmn1.  The exact solution for the truncated cone shows this.  See for example:  http://gregegan.customer.netspace.net.au/SCIENCE/Cavity/Cavity.html   :The quantum mode number "p" for a truncated cone is related to k.   k cannot be zero for a truncated cone, since k = ω/c for a truncated cone , therefore k = 0 implies zero frequency for a truncated cone. 

2) As the attached plot shows, COMSOL's FEA plot shows an electromagnetic field that is not constant in the longitudinal direction, therefore this is not TM010, it looks instead as TM011 upon closer inspection.

(Therefore the analyst's designation for TM011 should be changed to TM012 and so forth for TM01p modes)
I have been examining higher modes for the truncated cone cavity and mode shapes for truncated cones with larger cone angle and for smaller base diameters at the same cone angle.

For these cases (mode shapes with higher "m" azimuthal quantum number, and those with smaller base diameters compared to the big base diameter) the difference between the truncated mode shapes and the cylindrical mode shape becomes more significant.

Ultimately this was unavoidable, because the only quantum number that the truncated cone and the cylindrical cavity have in common is "m" the azimuthal quantum mode number.  The variation along the circumference is described by a harmonic function in both the truncated cone and the cylindrical cavity.

On the other hand, the variation along "n" is described by Associated Legendre P functions in terms of the cone's angle for the truncated cone while it is described in terms of zero Bessel functions of the radial polar coordinate for the cylindrical cavity.  Different functions in terms of different variables.

And the variation along "p" is described by Spherical Bessel functions in terms of the spherical radial coordinate for the truncated cone, while it is described in terms of harmonic functions of the longitudinal polar coordinate for the cylindrical cavity.  Different functions in terms of different variables.

I have not found in the literature a commonly accepted nomenclature to designate mode shapes for the truncated cone.

Furthermore, for this thread's audience, the cylindrical mode shapes are something that the audience can more immediately relate to, since the cylindrical mode shape nomenclature is found in the literature, and @Notsosureofit's formula is based on an analogy to the cylindrical cavity mode shapes (using the Bessel zero functions to characterize the mode shapes).

Therefore, to better communicate these mode shapes, and to avoid confusion I have decided to identify the truncated mode shapes in future communications as follows:

1. Always specifying the frequency at which they take place.
2. Whenever possible to provide plots to illustrate the actual mode shape.
3. Whenever possible to provide the designation for the closest cylindrical mode shape.  I will identify these as "Cyl. TMmnp"

Of course, for high mode numbers and/or large cone angles we will find mode shapes that cannot be described in terms of a cylindrical cavity analogy.  In those cases those modes are best described by their frequency and a plot showing the actual mode shape.




Therefore, I will describe what NASA Eagleworks describes as mode TM212 as "Cyl. TM212" from now on.

There is still a discrepancy regarding what NASA Eagleworks describes as mode TM010, which I think should be described "Cyl. TM011" because this mode shape for the truncated cone is not constant along the longitudinal axis of the truncated cone (as NASA Eagleworks's own plot shows).


Title: Re: EM Drive Developments - related to space flight applications - Thread 2
Post by: flux_capacitor on 03/25/2015 07:47 pm
@frobnicat: please notice this paper, published in The European Physical Journal Plus
November 2014, 129:240; which was brought up by @dustinthewind:

https://scholar.google.com/scholar?cluster=7136673109349846373&hl=en&as_sdt=0,48

that states:

Quote from: Miron Tuval and Asher Yahalom Newton’s Third Law in the Framework of Special
Relativity
Most locomotive systems of today are based on open systems. A rocket sheds exhaust gas to propel itself, a speeding bullet generates recoil. A car pushes the road with the same force that is used to accelerate it, the same is true regarding the interaction of a plane with air and of a ship with water. However, the above relativistic considerations suggest’s a new type of motor which is not based on a open system but rather on a closed one
....
As a final remark we will address the problem of achieving constant force which may be of interest for locomotive applications. A constant force may be achieved by having a direct current in one loop and a current of uniform second derivative on the other. For the choice of values given in table 2 we obtained FT z ∼= 2.74 Newton.
Obviously the switching time may represent some difficulty which one may overcome with advanced enough switching technology perhaps using low resistivity superconducting materials. Another possibility for constructing a relativistic motor is using numerous modular solid-state devices each with fast switching and small current such that an appreciable amount of cumulative forcing will result.

The European Physical Journal arose in 1998 as a merger and continuation of the very prestigious journals Zeitschrift für Physik, Journal de Physique,  Il Nuovo Cimento, and other journals.

The authors are associated with the renowned Isaac Newton Institute for Mathematical Sciences, an international research institute for mathematics and theoretical physics adjoining the Cambridge University Centre for Mathematical Sciences. In 1993 the British mathematician Andrew Wiles announced at the Institute his proof of Fermat's last theorem. Its director as of May 2012 was Cambridge University Professor John Toland (who is famous for formally proving in 1978, Stokes' conjecture on the existence of gravity waves of maximum height on deep water, a previously open problem in mathematical hydrodynamics which dated back to the 19th century).

Very interesting paper! Especially considering what Roger Shawyer says about conservation of momentum in "open systems"… To summarize his point of view, standard Newtonian mechanics and thus the law of conservation of momentum indicate that, no matter what shape the cavity is, the forces exerted upon it from within must balance to zero. Shawyer claims this statement ignores special relativity, in which separate frames of reference have to be applied when velocities approach the speed of light. He declares that in the EmDrive, the system of electromagnetic waves and the waveguide can be regarded as an open system, both having separate frames of reference. He also says this effect is similar to the principle of the laser gyroscope, which is also an apparently closed system device, but where the beams act as if having an external frame of reference (which they have, since the speed of light is constant).

However, Shawyer was severely criticized by the scientific community for this "not even wrong" idea. And frankly, I wonder if Tuval & Yahalom's initial assumption regarding the "transmission of any information limited by the speed of light" is really correct, since it is established the Abraham–Lorentz force and inertial reaction forces are instantaneous. Another counterexample involving instantaneousness is quantum entanglement.

[EDIT]I didn't saw at first that Roger Shawyer's idea is opposite (the "open system") than the "closed system" treatment in special relativity by Miron Tuval & Asher Yahalom. It's weird because both seem to claim the same idea: decoupling the EM effects between two interacting electric circuits.
Title: Re: EM Drive Developments - related to space flight applications - Thread 2
Post by: Notsosureofit on 03/25/2015 08:00 pm
The idea is correct, often used to demonstrate the twin paradox in General Relativity not Special Relativity.  (My pet peeve)  Special is only for Inertial frames.  Forces and acceleration require General Relativity, so they wind up using the limit of an approximation instead.

Edit: I have to read this paper in more detail, but first glance, it looks like you might get an impulse during this rather select interval but I don't (yet) see any consideration of the recovery of initial conditions w/o reversing it.
Title: Re: EM Drive Developments - related to space flight applications - Thread 2
Post by: dustinthewind on 03/25/2015 09:07 pm
I believe this article has more to do with two currents out of phase by 90 degrees than the first article I posted.  I'll be sure to review it again to make sure.  http://arxiv.org/pdf/1502.06288v1.pdf .
Title: Re: EM Drive Developments - related to space flight applications - Thread 2
Post by: frobnicat on 03/25/2015 11:11 pm
The idea is correct, often used to demonstrate the twin paradox in General Relativity not Special Relativity.  (My pet peeve)  Special is only for Inertial frames.  Forces and acceleration require General Relativity, so they wind up using the limit of an approximation instead.

Edit: I have to read this paper in more detail, but first glance, it looks like you might get an impulse during this rather select interval but I don't (yet) see any consideration of the recovery of initial conditions w/o reversing it.

I second you on the (also at first glance) apparent problem of having any kind of stationary net average force in a direction with this approach. From the article (at least what is on arxiv (http://arxiv.org/pdf/1302.2537v1.pdf)) : the meat is equation (66)  F=cst*I1bar*I2bar/tau² (number of turns assumed constant).

I don't know the academic history and prestige of the journal in which this is published, but unless the published version is significantly edited, this is a case of notation abuse and almost deceit in conclusion drawing from correct equations, (I assume they are correct, would took me some weeks to understand and check in details). The bar notation usually denotes a time averaged value of a time varying stationary value, be it periodic, or quasiperiodic, or random, but it must be that  ( integral(0 to T) V(t)dt )/T converges to some value when T->+infinity for Vbar to have a meaning. Unless there are other usages I'm unaware of ? Please educate me if so.

Anyway (bottom page 12) : A constant force may be achieved by having a direct current in one loop I1(t) =I1bar and a current of uniform second derivative on the other I2(t)=1/2 I2bar*t²/tau²
Green sensible use of bar notation
Red deceiving use of bar notation : I2(t) defined as having a uniform second derivative (wrt time) can't be stationary. I2bar is ill defined. For instance if a given I2(t) is defined by I2bar=1 and tau=1, the exact same I2(t) could also be defined by by I2bar=100 and tau=10. There are 2 "free parameters" introduced when in fact saying "uniform second derivative" requires only one : I2(t) = 1/2 CurAcc*t²  where CurAcc (Current Acceleration) is the only needed parameter and has units of A/s². This might be ugly, maybe a "representative" current value in A (I2bar) + "representative" time value in s (tau) is more "expressive" and elegant. As absolute values, it's not clear what they are representative of though, since 1A 1s is equivalent to 100A 10s

But abuse of notation and introducing more parameters than necessary is not the problem by itself, and the equation (66) is still as valid written with I2bar/tau² term  as it would be with CurAcc term. Equations all right. But then we forget that I2bar notation was just a commodity, that there is no real stationary I2(t) behind that, and that the force given by the expression can only be a transient, since 1/2 CurAcc*t² will soon exceed any reasonable current carrying capacity (superconducting or not). And assuming a clever possibility, like cutting the current abruptly (in the loop) to put it back to 0 periodically, is doomed to fail as much as any attempt to transform repetitive mechanical effects inside a capsule into net acceleration (Dean drive...). What was gained during an acceleration phase will be lost by a deceleration phase, if the driving parameter (Intensity, Position...) is to be made periodic.

This derives clearly from eq. (51) (from which eq. (66) itself derives) :
F(t) proportional to I1(t)*I2dotdot(t) - I1dotdot(t)*I2(t)
Is it conceivable to have 2 stationary I1(t) and I2(t) (in the sense defined above that I1bar and I2bar have a real meaning) that yield Fbar different from 0 ? I don't see a clever elegant mathematical proof to that but I'm rather sure it is impossible. If someone disagree please state a clear explicit counter example I1(t) I2(t) (for any t) with Fbar different from 0 and I1 and I2 stationary, that is ( integral(0 to T) I(t)dt )/T converges to some value (0 or whatever) when T->+infinity.

The example I1 constant and I2 "uniformly accelerated" given by the paper don't meet the requirement : I2 not stationary. This is a one shot effect. It's not really different from shooting a bullet at constant m/s² in a mass driver fixed to the hull of a spacecraft. As long as the bullet is accelerating in the mass driver, the mass driver gives a constant "thrust" to the spacecraft. But then what ? Either the bullet is free to leave the spacecraft : conventional reaction propulsion, spacecraft loses propellent mass. Or else it is "recycled", which implies some deceleration of it to 0 (wrt spacecraft), which will cancel exactly the gained momentum, be it hard or soft is irrelevant (damages or sparks aside).

BTW we also understand that playing with "delayed" forces in relativity it's not surprising to get non null instant forces on a system. If "a system" is an ensemble of elements separated by some distance, then there is not really such thing as an instant force on the system as a whole since there is no such thing as an instant for the system. SR is enough to show that the notion of intrinsic instant is ill defined wrt the system's parts : "instant sum of local forces seen by each part" depend on arbitrary inertial rest frame.

What is going on with hard science those days ? Do brilliant people make a living of writing correct complicated equations to hint at spectacular but delusional conclusions from them ?
Title: Re: EM Drive Developments - related to space flight applications - Thread 2
Post by: RonM on 03/26/2015 12:11 am
FYI

http://www.technologyreview.com/view/536091/spacecraft-traveling-close-to-light-speed-should-be-visible-with-current-technology-say/?utm_source=feedburner&utm_medium=email&utm_campaign=Feed%3A+arxivblog%2FGmoU+%28The+Physics+arXiv+Blog%29

Quote
The movement of a relativistic spacecraft will have another effect. It should scatter the cosmic microwave background in a way that produces a unique signature. “As a baryonic spacecraft travels at relativistic speeds it will interact with the CMB through scattering to cause a frequency shift that could be detectable on Earth with current technology,” say Yurtsever and Wilkinson.

Please excuse the somewhat off topic question, but would this also apply to the interaction of an Alcubierre drive or similar manipulation of space-time with the CMB?
Title: Re: EM Drive Developments - related to space flight applications - Thread 2
Post by: Rodal on 03/26/2015 12:27 am
The idea is correct, often used to demonstrate the twin paradox in General Relativity not Special Relativity.  (My pet peeve)  Special is only for Inertial frames.  Forces and acceleration require General Relativity, so they wind up using the limit of an approximation instead.

Edit: I have to read this paper in more detail, but first glance, it looks like you might get an impulse during this rather select interval but I don't (yet) see any consideration of the recovery of initial conditions w/o reversing it.

I second you on the (also at first glance) apparent problem of having any kind of stationary net average force in a direction with this approach. From the article (at least what is on arxiv (http://arxiv.org/pdf/1302.2537v1.pdf)) : the meat is equation (66)  F=cst*I1bar*I2bar/tau² (number of turns assumed constant).

I don't know the academic history and prestige of the journal in which this is published, but unless the published version is significantly edited, this is a case of notation abuse and almost deceit in conclusion drawing from correct equations, (I assume they are correct, would took me some weeks to understand and check in details). The bar notation usually denotes a time averaged value of a time varying stationary value, be it periodic, or quasiperiodic, or random, but it must be that  ( integral(0 to T) V(t)dt )/T converges to some value when T->+infinity for Vbar to have a meaning. Unless there are other usages I'm unaware of ? Please educate me if so.

Anyway (bottom page 12) : A constant force may be achieved by having a direct current in one loop I1(t) =I1bar and a current of uniform second derivative on the other I2(t)=1/2 I2bar*t²/tau²
Green sensible use of bar notation
Red deceiving use of bar notation : I2(t) defined as having a uniform second derivative (wrt time) can't be stationary. I2bar is ill defined. For instance if a given I2(t) is defined by I2bar=1 and tau=1, the exact same I2(t) could also be defined by by I2bar=100 and tau=10. There are 2 "free parameters" introduced when in fact saying "uniform second derivative" requires only one : I2(t) = 1/2 CurAcc*t²  where CurAcc (Current Acceleration) is the only needed parameter and has units of A/s². This might be ugly, maybe a "representative" current value in A (I2bar) + "representative" time value in s (tau) is more "expressive" and elegant. As absolute values, it's not clear what they are representative of though, since 1A 1s is equivalent to 100A 10s

But abuse of notation and introducing more parameters than necessary is not the problem by itself, and the equation (66) is still as valid written with I2bar/tau² term  as it would be with CurAcc term. Equations all right. But then we forget that I2bar notation was just a commodity, that there is no real stationary I2(t) behind that, and that the force given by the expression can only be a transient, since 1/2 CurAcc*t² will soon exceed any reasonable current carrying capacity (superconducting or not). And assuming a clever possibility, like cutting the current abruptly (in the loop) to put it back to 0 periodically, is doomed to fail as much as any attempt to transform repetitive mechanical effects inside a capsule into net acceleration (Dean drive...). What was gained during an acceleration phase will be lost by a deceleration phase, if the driving parameter (Intensity, Position...) is to be made periodic.

This derives clearly from eq. (51) (from which eq. (66) itself derives) :
F(t) proportional to I1(t)*I2dotdot(t) - I1dotdot(t)*I2(t)
Is it conceivable to have 2 stationary I1(t) and I2(t) (in the sense defined above that I1bar and I2bar have a real meaning) that yield Fbar different from 0 ? I don't see a clever elegant mathematical proof to that but I'm rather sure it is impossible. If someone disagree please state a clear explicit counter example I1(t) I2(t) (for any t) with Fbar different from 0 and I1 and I2 stationary, that is ( integral(0 to T) I(t)dt )/T converges to some value (0 or whatever) when T->+infinity.

The example I1 constant and I2 "uniformly accelerated" given by the paper don't meet the requirement : I2 not stationary. This is a one shot effect. It's not really different from shooting a bullet at constant m/s² in a mass driver fixed to the hull of a spacecraft. As long as the bullet is accelerating in the mass driver, the mass driver gives a constant "thrust" to the spacecraft. But then what ? Either the bullet is free to leave the spacecraft : conventional reaction propulsion, spacecraft loses propellent mass. Or else it is "recycled", which implies some deceleration of it to 0 (wrt spacecraft), which will cancel exactly the gained momentum, be it hard or soft is irrelevant (damages or sparks aside).

BTW we also understand that playing with "delayed" forces in relativity it's not surprising to get non null instant forces on a system. If "a system" is an ensemble of elements separated by some distance, then there is not really such thing as an instant force on the system as a whole since there is no such thing as an instant for the system. SR is enough to show that the notion of intrinsic instant is ill defined wrt the system's parts : "instant sum of local forces seen by each part" depend on arbitrary inertial rest frame.

What is going on with hard science those days ? Do brilliant people make a living of writing correct complicated equations to hint at spectacular but delusional conclusions from them ?
After your demolishing review, I decided to look into more detail into the Jounal, and I noticed the "Plus" at the end (instead of A or B).

The following may answer your question: (from http://www.springer.com/physics/applied+%26+technical+physics/journal/13360?detailsPage=societies)
concerning <<What is going on with hard science those days ? Do brilliant people make a living of writing correct complicated equations to hint at spectacular but delusional conclusions from them ?>>

Quote
What can we look forward to this year in regards to your journal’s content or development?

EPJ Plus is a newly born journal of the EPJ series, a continuation of Il Nuovo Cimento B formely published by the Italian Physical Society. It has been launched in 2011 as an electronic-only journal, with a new interdisciplinary approach in terms of topics and a wider portfolio of possible article formats. EPJ Plus is also meant as a "cascade" journal for the other EPJs. In the last two years, the number of articles published in EPJ Plus has significantly grown and its impact factor expectations are definitely promising. According to the recent editorial policy of the journal, invited contents, grouped in topical "Focus Points", will be boosted and contributions from new fields, such as accelerator physics or physics applied to cultural heritage or to energy, will be strongly encouraged.

The authors have a patent application in the US:

https://www.google.com/patents/US20140152227?dq=US20140152227&hl=en&sa=X&ei=D2ITVcTUJ8uigwSb34PoAQ&ved=0CB8Q6AEwAA
Title: Re: EM Drive Developments - related to space flight applications - Thread 2
Post by: dustinthewind on 03/26/2015 12:43 am
(Dustinthewind - here is a quote from the other thread, "A scientific paper that clarifies how newtons 3rd law does not apply to the time delay of information and how it can be used for electromagnetic propulsion.  It provides a mathematical background for the time delayed magnetic fields but first illustrating how the static equations miss the effect.  https://scholar.google.com/scholar?cluster=7136673109349846373&hl=en&as_sdt=0,48" it is titled, "Newton's Third Law in the Framework of Special Relativity" )

I'll have to apologize if I wasted anyone's time.  I realized the paper is not what I thought it was about after frobnicat's review.  I expected two current loops changing in time out of phase but per the review I went back and sure enough it is about a current loop constantly on while one changes.   :-[ 
Title: Re: EM Drive Developments - related to space flight applications - Thread 2
Post by: Rodal on 03/26/2015 01:36 am
(Dustinthewind - here is a quote from the other thread, "A scientific paper that clarifies how newtons 3rd law does not apply to the time delay of information and how it can be used for electromagnetic propulsion.  It provides a mathematical background for the time delayed magnetic fields but first illustrating how the static equations miss the effect.  https://scholar.google.com/scholar?cluster=7136673109349846373&hl=en&as_sdt=0,48" it is titled, "Newton's Third Law in the Framework of Special Relativity" )

I'll have to apologize if I wasted anyone's time.  I realized the paper is not what I thought it was about after frobnicat's review.  I expected two current loops changing in time out of phase but per the review I went back and sure enough it is about a current loop constantly on while one changes.   :-[

No need to apologize  :)

Welcome to our forum.

We have discussed more "out there" papers than this one (which was peer-reviewed at a well-known European parent journal and whose authors claimed association with a renowned European Mathematical Institute).  A number of papers, theories and experiments that have been explored in this thread have appeared in less renowned publications and/or have not been peer-reviewed  :)
Title: Re: EM Drive Developments - related to space flight applications - Thread 2
Post by: ThinkerX on 03/26/2015 01:57 am
Quote
Very interesting paper! Especially considering what Roger Shawyer says about conservation of momentum in "open systems"… To summarize his point of view, standard Newtonian mechanics and thus the law of conservation of momentum indicate that, no matter what shape the cavity is, the forces exerted upon it from within must balance to zero. Shawyer claims this statement ignores special relativity, in which separate frames of reference have to be applied when velocities approach the speed of light. He declares that in the EmDrive, the system of electromagnetic waves and the waveguide can be regarded as an open system, both having separate frames of reference. He also says this effect is similar to the principle of the laser gyroscope, which is also an apparently closed system device, but where the beams act as if having an external frame of reference (which they have, since the speed of light is constant).

However, Shawyer was severely criticized by the scientific community for this "not even wrong" idea. And frankly, I wonder if Tuval & Yahalom's initial assumption regarding the "transmission of any information limited by the speed of light" is really correct, since it is established the Abraham–Lorentz force and inertial reaction forces are instantaneous. Another counterexample involving instantaneousness is quantum entanglement.

[EDIT]I didn't saw at first that Roger Shawyer's idea is opposite (the "open system") than the "closed system" treatment in special relativity by Miron Tuval & Asher Yahalom. It's weird because both seem to claim the same idea: decoupling the EM effects between two interacting electric circuits.

Ok, my interest here is in what Shawyer has to say.  It seems fairly apparent he is ahead of the pack when it comes to building/testing these devices - each time we considered a different approach, Shawyer has been there and built a relevant prototype, like as not.  Given this, is there any way to salvage his operational theory here that doesn't involve direct violation of the laws of physics?  Is there some legitimate way his explanation could be shoehorned into the gaps between GR and SR and QM?  Keeping in mind he is an experimenter first and theorist second. 
Title: Re: EM Drive Developments - related to space flight applications - Thread 2
Post by: zen-in on 03/26/2015 02:28 am

The authors have a patent application in the US:

https://www.google.com/patents/US20140152227?dq=US20140152227&hl=en&sa=X&ei=D2ITVcTUJ8uigwSb34PoAQ&ved=0CB8Q6AEwAA

What they have is some kind of motor or electromagnetic actuator.   In the description section the theory of operation is described:

"The scope of this invention rests on the well known phenomenology of electromagnetic asymmetric systems that do not abide by Newton's Third Law. It should be noted that the law of momentum conservation is maintained during operation of the system of the present invention, as the momentum gained by the material system is counter-balanced by an opposite momentum gained by the electromagnetic field. This provides that the total momentum of the entire physical system, which is composed of both matter and field, is conserved."

While it is true that momentum can be transferred via a magnetic field, the momentum is transferred to whatever physical entity maintains said magnetic field.   When two magnets repel or attract each other the magnets move to positions that minimize the potential energy of the magnetic fields.   
Title: Re: EM Drive Developments - related to space flight applications - Thread 2
Post by: Stormbringer on 03/26/2015 02:42 am
FYI

http://www.technologyreview.com/view/536091/spacecraft-traveling-close-to-light-speed-should-be-visible-with-current-technology-say/?utm_source=feedburner&utm_medium=email&utm_campaign=Feed%3A+arxivblog%2FGmoU+%28The+Physics+arXiv+Blog%29

Quote
The movement of a relativistic spacecraft will have another effect. It should scatter the cosmic microwave background in a way that produces a unique signature. “As a baryonic spacecraft travels at relativistic speeds it will interact with the CMB through scattering to cause a frequency shift that could be detectable on Earth with current technology,” say Yurtsever and Wilkinson.

Please excuse the somewhat off topic question, but would this also apply to the interaction of an Alcubierre drive or similar manipulation of space-time with the CMB?
i think it would be somewhat similar. i always envisioned it as producing a burst of gamma rays and x rays at the start of a FTL movement and a burst when it comes out of FTL mode. but now you have to add a moving and rising emission spectra as it gets to its top speed prior to warp. and a reverse of this on the other end as it may deaccelerate at the destination.
Title: Re: EM Drive Developments - related to space flight applications - Thread 2
Post by: Star-Drive on 03/26/2015 02:49 am
....
The copper frustum thrust reversal due to only its dielectric placement came when I was experimenting with the TM010 mode,...
... I think that this mode shape correct designation is TM011 instead of TM010 because:

1) There can be no TMmn0 modes for a truncated cone.  TMmn0 modes need to have a constant electromagnetic field in the longitudinal direction of the cavity.  This is possible for a cylindrical cavity (containing no other dielectrics inside besides the cavity medium) because it has constant geometrical and material properties in the longitudinal direction.  But a conical cavity has variable cross-section in the longitudinal direction, therefore the  TMnn0 mode is not possible.  The first possible mode (if it is not cut-off) is TMmn1.  The exact solution for the truncated cone shows this.  See for example:  http://gregegan.customer.netspace.net.au/SCIENCE/Cavity/Cavity.html   :The quantum mode number "p" for a truncated cone is related to k.   k cannot be zero for a truncated cone, since k = ω/c for a truncated cone , therefore k = 0 implies zero frequency for a truncated cone. 

2) As the attached plot shows, COMSOL's FEA plot shows an electromagnetic field that is not constant in the longitudinal direction, therefore this is not TM010, it looks instead as TM011 upon closer inspection.

(Therefore the analyst's designation for TM011 should be changed to TM012 and so forth for TM01p modes)
I have been examining higher modes for the truncated cone cavity and mode shapes for truncated cones with larger cone angle and for smaller base diameters at the same cone angle.

For these cases (mode shapes with higher "m" azimuthal quantum number, and those with smaller base diameters compared to the big base diameter) the difference between the truncated mode shapes and the cylindrical mode shape becomes more significant.

Ultimately this was unavoidable, because the only quantum number that the truncated cone and the cylindrical cavity have in common is "m" the azimuthal quantum mode number.  The variation along the circumference is described by a harmonic function in both the truncated cone and the cylindrical cavity.

On the other hand, the variation along "n" is described by Associated Legendre P functions in terms of the cone's angle for the truncated cone while it is described in terms of zero Bessel functions of the radial polar coordinate for the cylindrical cavity.  Different functions in terms of different variables.

And the variation along "p" is described by Spherical Bessel functions in terms of the spherical radial coordinate for the truncated cone, while it is described in terms of harmonic functions of the longitudinal polar coordinate for the cylindrical cavity.  Different functions in terms of different variables.

I have not found in the literature a commonly accepted nomenclature to designate mode shapes for the truncated cone.

Furthermore, for this thread's audience, the cylindrical mode shapes are something that the audience can more immediately relate to, since the cylindrical mode shape nomenclature is found in the literature, and @Notsosureofit's formula is based on an analogy to the cylindrical cavity mode shapes (using the Bessel zero functions to characterize the mode shapes).

Therefore, to better communicate these mode shapes, and to avoid confusion I have decided to identify the truncated mode shapes in future communications as follows:

1. Always specifying the frequency at which they take place.
2. Whenever possible to provide plots to illustrate the actual mode shape.
3. Whenever possible to provide the designation for the closest cylindrical mode shape.  I will identify these as "Cyl. TMmnp"

Of course, for high mode numbers and/or large cone angles we will find mode shapes that cannot be described in terms of a cylindrical cavity analogy.  In those cases those modes are best described by their frequency and a plot showing the actual mode shape.




Therefore, I will describe what NASA Eagleworks describes as mode TM212 as "Cyl. TM212" from now on.

There is still a discrepancy regarding what NASA Eagleworks describes as mode TM010, which I think should be described "Cyl. TM011" because this mode shape for the truncated cone is not constant along the longitudinal axis of the truncated cone (as NASA Eagleworks's own plot shows).


Dr. Rodal:

I asked our first COMSOL analyst his opinion on this resonant mode naming issue and attached is Frank's comments on same.

Best, Paul M.
Title: Re: EM Drive Developments - related to space flight applications - Thread 2
Post by: Stormbringer on 03/26/2015 03:04 am
Mr March; I assume you are far far far more familiar with Dr Woodward's nailing down spurious signals than I but I just got through that portion of his book. He basically nuked every potential source of spurious signal in his apparatus and because he is using a nearly identical set up until you get to the frustrum itself his analysis is germain to this project. He appears to have authoritatively disproved every thing we have considered here WRT spurious signals; leaving the problem of what the thrust signal really results from.

That does not give us the answer to what is going on but it does tell us what it is not.

 I guess in replication everyone has to redo that work but I cannot help but feel that it is sort of a waste of time to do that all over again. likewise; since I am pretty sure Dr White and yourself know all about Dr Woodward's spurious signal source crushing that the Eagleworks team has also done the same.

I would therefore assume you guys are very confident you have something real and that it is not explained by mundane errors.
Title: Re: EM Drive Developments - related to space flight applications - Thread 2
Post by: Star-Drive on 03/26/2015 11:44 am
Mr March; I assume you are far far far more familiar with Dr Woodward's nailing down spurious signals than I but I just got through that portion of his book. He basically nuked every potential source of spurious signal in his apparatus and because he is using a nearly identical set up until you get to the frustrum itself his analysis is germain to this project. He appears to have authoritatively disproved every thing we have considered here WRT spurious signals; leaving the problem of what the thrust signal really results from.

That does not give us the answer to what is going on but it does tell us what it is not.

 I guess in replication everyone has to redo that work but I cannot help but feel that it is sort of a waste of time to do that all over again. likewise; since I am pretty sure Dr White and yourself know all about Dr Woodward's spurious signal source crushing that the Eagleworks team has also done the same.

I would therefore assume you guys are very confident you have something real and that it is not explained by mundane errors.

Stormbringer:

"Mr. March; I assume you are far far far more familiar with Dr Woodward's nailing down spurious signals than I but I just got through that portion of his book. He basically nuked every potential source of spurious signal in his apparatus and because he is using a nearly identical set up until you get to the frustrum itself his analysis is germain to this project."

As you probably know Dr. White and I would not be in this business if it were not for Dr. James F. Woodward's valiant efforts in first bringing to light in 1988 his Mach-Effect conjecture, gladly sharing his vision and knowledge with others like us, and then Jim's tireless efforts to experimentally prove his M-E conjecture while suffering through his stage-4 lung and other cancer treatments for the last nine years plus.  And you are also correct in saying that Dr. Woodward has addressed and/or corrected every known error source in his toque pendulum setup over the last 15 years.  In other words IMO Jim has a near bullet proof experimental test setup and Sonny and I have tried to use as much of Dr. Woodward's experimental insights on our test setup in the Eagleworks Lab's torque pendulum as was applicable.   

"Likewise; since I am pretty sure Dr White and yourself know all about Dr. Woodward's spurious signal source crushing that the Eagleworks team has also done the same."

Jim Woodward and I have been talking through front and back channels since March of 1998 and continue to do so to this day as needed.  In fact my wife and I  stayed with Jim and his lovely wife Carole for a few days in his CO retreat during our summer break last August to be with good friends and to discuss his ongoing work and tell Jim about our project at Eagleworks Lab.  And we've tried to incorporate as much of this corporate knowledge that Dr. Woodward has developed over the years that my poor brain can assimilate, for Dr. Woodward is one brilliant experimenter to which I can only aspire to emulate.  However Dr. White and Dr. Woodward had a professional falling out back in 2007 when Sonny decided that the Mach-Effect was not the whole story and that the quantum mechanical aspects of this business had to be addressed as well, where Dr. Woodward was content to rely on just Special & General Relativity Theory and Wheeler/Feynman radiation reaction forces for his explanation for the observed data.

"I would therefore assume you guys are very confident you have something real and that it is not explained by mundane errors."
   
As we say in the lab, WE think there IS a there, there and yes, we feel quite strongly about that point and continue to build a case for the reality of the effects we've been able to bring to light and to find ways to scale them up in magnitude.  Of course whether the effects we are examining are just SRT/GRT based, or whether we have to also evoke known and obscure quantum effects and perhaps even new physics to explain these observed phenomenon is currently what's in play.  But in my opinion we will end up coming to terms with what the thing we call gravity really is by the time this project is finally put into the history books.

Best, Paul M.

Title: Re: EM Drive Developments - related to space flight applications - Thread 2
Post by: dustinthewind on 03/26/2015 06:41 pm
I still think we can violate newtons 3rd law in a way but in another way it is not violated because the propulsion device projects radiation out one end.  This looks like radiation propulsion but by sticking a dielectric between the two current loops we can change the speed of light making the two current loops closer or lowering the frequency needed while also getting near field effects?  What this does for the radiation projected I'm not exactly sure but I would assume it should intensify.  This is assuming none of the current loops have constant current but are both changing in time and out of phase pi/2 (see figure EM Propulsion 2.png). 


I guess the idea was if there was something similar going on inside the radiation cavity though I can't quite say that there is.  There is also the issue of the idea that radiation projected is conserving momentum but this is inside a cavity.  (see figure EM Propulsion 3.png)

Edit: sorry, changed pi/4 to pi/2
Title: Re: EM Drive Developments - related to space flight applications - Thread 2
Post by: Rodal on 03/26/2015 07:09 pm
...

Dr. Rodal:

I asked our first COMSOL analyst his opinion on this resonant mode naming issue and attached is Frank's comments on same.

Best, Paul M.
What a fantastic response !
Paul, please thank Frank for taking the time to write a comprehensive answer.

I think we have arrived to a full understanding and agreement.

I attach 3 plots below:

1) "First Mode TM011a (Eagleworks TM010 at 0.9598 GHz) Small Base at r=0.3008, Big Base at r=0.5294"

2) "Second Mode TM011b (Eagleworks TM011 at 1.3435 GHz) Small Base at r=0.3008, Big Base at r=0.5294"

3) "Third Mode TM012 (Eagleworks TM012 at 1.7160 GHz) Small Base at r=0.3008, Big Base at r=0.5294"

which show the magnetic field strength B at the conical wall (which takes place only in the azimuthal direction, since these are transverse magnetic modes) as a function of the spherical coordinate in the r direction defining the truncated cone (see image for the significance of "r"):

(http://gregegan.customer.netspace.net.au/SCIENCE/Cavity/CavityShape.gif)

The last plot, for "Third Mode TM012 (Eagleworks TM012 at 1.7160 GHz)" shows that the mode shape for this mode is a full-wave pattern: or two half-wave patterns, therefore this mode should indeed be designated as TM012 in an analogy to a cylindrical cavity (because the last quantum number p=2 indicates the number of half-wave patterns in the longitudinal direction).  Thus, there is no doubt about this mode and we fully agree on this one.

The first two plots show that the first two modes both exhibit one half-wave pattern, therefore from that point of view they are both TM011 modes (because the last quantum number p=1 indicates the number of half-wave patterns in the longitudinal direction).  Since they are both TM011 modes, I will designate them TM011a and TM011b.

When the analyst is tasked with identifying the mode shapes that are closest to a cylindrical cavity I can fully see why Frank would identify TM011b as TM011, because TM011b exhibits a change of sign of the magnetic field going from the small base to the big base.  On the other hand, TM011a, although it has a changing magnetic field strength in the longitudinal direction (changing as a half-wave pattern), it does not change sign going from the small base to the big base.

What happens is very neat:

a) strictly speaking, the transverse magnetic field in a conical cavity cannot be constant, hence there cannot be a TM010 mode in a cone.  However, for cones that have a cone angle sufficiently small (that is: truncated cones that are close enough to a cylindrical cavity) there are two TM011 modes, the lowest TM011 mode is closest to a constant field, closest to TM010, and the higher TM011 mode is closest to a true TM011 mode.  Exactly the same designation chosen by Frank: TM010 for TM011a and TM011 for TM011b.

b) For a cone angle approaching zero degrees (a cone approaching a cylindrical cavity) the magnetic field becomes constant in the longitudinal direction, and TM011a becomes TM010.  The same designation chosen by Frank.

c) For higher cone angles (for example for the example considered by Greg Egan http://gregegan.customer.netspace.net.au/SCIENCE/Cavity/Cavity.html) with a cone angle >= 20 degrees, the lower TM011 mode ("TM011a") disappears completely, as it is cut-off.
Title: Re: EM Drive Developments - related to space flight applications - Thread 2
Post by: Rodal on 03/26/2015 07:34 pm
....
....

What happens is very neat:

a) strictly speaking, the transverse magnetic field in a conical cavity cannot be constant, hence there cannot be a TM010 mode in a cone.  However, for cones that have a cone angle sufficiently small (that is: truncated cones that are close enough to a cylindrical cavity) there are two TM011 modes, the lowest TM011 mode is closest to a constant field, closest to TM010, and the higher TM011 mode is closest to a true TM011 mode.  Exactly the same designation chosen by Frank: TM010 for TM011a and TM011 for TM011b.

b) For a cone angle approaching zero degrees (a cone approaching a cylindrical cavity) the magnetic field becomes constant in the longitudinal direction, and TM011a becomes TM010.  The same designation chosen by Frank.

c) For higher cone angles (for example for the example considered by Greg Egan http://gregegan.customer.netspace.net.au/SCIENCE/Cavity/Cavity.html) and certainly for a cone angle > 30 degrees, the lower TM011 mode ("TM011a") disappears completely, as it is cut-off.


@Notsosureofit

The above discussion and plots are relevant to the extent that one can pursue an approximate formula for the truncated cone using a cylindrical cavity as a close analogue, as in your approximate formula.  It helps understand and quantify the approximation involved.

You can see from the plots that even for the NASA Eagleworks truncated cone differences appear for the lowest TM mode:  it is no longer constant in the longitudinal direction.  This lowest TM011a mode that looks like TM010 completely disappears for a cone angle >=20 degrees.

Although the magnetic field variation in the longitudinal direction for a cylindrical cavity goes like Cos[p Pi z/L] (where "p" is the quantum mode shape number in the longitudinal direction and z is the longitudinal coordinate and L the axial length of the cylindrical cavity), the longitudinal variation in the truncated cone with the NASA Eagleworks dimensions is a distorted wave-pattern (described by spherical Bessel functions and its derivative): it does no longer posses the symmetry of the Cosine function: mode shapes have a stronger field at one base than at the other base.
Title: Re: EM Drive Developments - related to space flight applications - Thread 2
Post by: Notsosureofit on 03/26/2015 07:55 pm
@ RODAL

Thinking about that right now.....
Title: Re: EM Drive Developments - related to space flight applications - Thread 2
Post by: tchernik on 03/26/2015 08:23 pm
Good to see this discussion is still moving into interesting directions.

As an interested but ignorant person in the Internet, what I take from the last posts, is that the effect (if it indeed exists) could be strongly affected by the geometry of the truncated cone cavity (e.g. the slope angle), which strongly impacts the resonance modes and frequencies producing them.

Does that  mean that not any cone shape would do? and that for every shape, there is a probably small set of frequencies for which the right resonating modes would arise?

It would be interesting to know which geometries and frequencies produce the best force results... something maybe R. Shawyer has a lot more experience in, given he has been tinkering with this for years and maybe decades.

That characterization is potentially very interesting for anyone else trying replications, because without taking that into account and assuming the phenomenon does exist, some may see something, others may see nothing at all, or just something below the force measurement noise.
Title: Re: EM Drive Developments - related to space flight applications - Thread 2
Post by: Notsosureofit on 03/26/2015 08:31 pm
It's the unending search for the perfect shape giving maximum thrust !
Title: Re: EM Drive Developments - related to space flight applications - Thread 2
Post by: Rodal on 03/26/2015 09:48 pm
I still think we can violate newtons 3rd law in a way but in another way it is not violated because the propulsion device projects radiation out one end.  This looks like radiation propulsion but by sticking a dielectric between the two current loops we can change the speed of light making the two current loops closer or lowering the frequency needed while also getting near field effects?  What this does for the radiation projected I'm not exactly sure but I would assume it should intensify.  This is assuming none of the current loops have constant current but are both changing in time and out of phase pi/2 (see figure EM Propulsion 2.png). 


I guess the idea was if there was something similar going on inside the radiation cavity though I can't quite say that there is.  There is also the issue of the idea that radiation projected is conserving momentum but this is inside a cavity.  (see figure EM Propulsion 3.png)

Edit: sorry, changed pi/4 to pi/2
There are many things to discuss here.

Of course  the electric field and magnetic fields are two interrelated aspects of a single object: the electromagnetic field tensor in 3D+1=4 spacetime. 

But if your loops and images are to be interpreted literally only in terms of electric current loops, please notice that your image then could be interpreted as saying that only TE (transverse electric) modes would produce the effect you are seeking, because only TE modes have the electric field in the azimuthal (circumferential) direction.  But notice that NASA Eagleworks is currently successfully testing in a vacuum TM modes (and actually I understand they have a preference for TM modes), for which the magnetic field B has a component only in the azimuthal (circumferential) direction while the electric field E has components perpendicular to the magnetic B field.  The electric field has zero component in the circumferential direction for TM modes.
Title: Re: EM Drive Developments - related to space flight applications - Thread 2
Post by: Rodal on 03/27/2015 12:43 am
Struck out on finding a lathe to use. My belt sander did a fantastic job too. So now I have 2, 1" thick x 6.25" wide HDPE discs. Now I have to figure out how to mount them....

Where there's a will, there's a way.

QUESTION 1: Before mounting the discs, could you please run a couple of tests without the HDPE discs?

If, so I can send you a post showing the mode shapes at the two frequencies near 2.45 GHz and why .

There is a mode shape at 2.49 GHz (or at 2.46 GHz according to NASA's COMSOL calculations) that should produce no force without the dielectric disc (because the Poynting vector practically cancels out)

There is a mode shape at 2.46 GHz (or at 2.41 GHz according to NASA's COMSOL calculations) that may produce an electromagnetic force without the dielectric disc (because the Poynting vector does not cancel out)

...

So here is the data:

All cases: empty cavity, NO polymer dielectric inside

Mode        Frequency (GHz) [Exact sltn.]     Frequency (GHz) [COMSOL FEA]       Poynting Vector
Cyl. TM311    2.45835                                      2.4068                                                 Towards Small Base
Cyl. TM212    2.49342                                      2.4575                                                 ~ 0

Notice:

1)  The Poynting vector  (http://en.wikipedia.org/wiki/Poynting_vector) (ExB) is a quadratic function of the harmonic function Cos[ ω t], such that the variation with time goes like (Cos[ ω t])^2 so that it never changes sign and therefore does not change orientation with time, even for this AC field.

(http://www.blazelabs.com/pics/empropagation2.gif)

(http://wwwhome.cs.utwente.nl/~ptdeboer/ham/tn/tn07fig3.png)

2) The Poynting  vector field changes magnitude around the circumference (the "azimuthal direction" which is perpendicular, "out of the plane of the truncated cone section").  At the azimuthal angle φ=0 (and multiples of it), the Poynting vector component perpendicular to the cone's curved walls is more than 3 times larger than the Poyting vector component in the longitudinal ("radial") direction of the cone.  Since the  Poynting vector component perpendicular to the cone's curved walls is self-cancelling, all that is left is the radial component (which is also shown).

(http://gregegan.customer.netspace.net.au/SCIENCE/Cavity/CavityShape.gif)

3) At azimuthal angle φ=45 degrees (and multiples) the Poynting vector component perpendicular to the cone's curved walls is zero,  The only Poynting vector component at those angles is directed in the radial direction.

4) The finite element analysis (FEA) solution converges "from below".  As the finite element mesh is made finer (more finite elements are used), the finite element solution becomes less stiff and it approaches the exact solution.  Hence the frequency calculated by FEA is always lower than the exact solution frequency for a finite mesh.

5) The exact solution assumes that the small and big bases are spherical sections (which is the ideal shape because the electromagnetic waves are spherical inside a cone).   The actual geometry used by NASA Eagleworks and modeled by COMSOL FEA is flat surfaces both for the small and big bases of the truncated cone.

6) The exact solution takes less than 3 seconds to calculate in a PC, much less time than what a Finite Element Solution takes. 
Title: Re: EM Drive Developments - related to space flight applications - Thread 2
Post by: dustinthewind on 03/27/2015 02:04 am
I still think we can violate newtons 3rd law in a way but in another way it is not violated because the propulsion device projects radiation out one end.  This looks like radiation propulsion but by sticking a dielectric between the two current loops we can change the speed of light making the two current loops closer or lowering the frequency needed while also getting near field effects?  What this does for the radiation projected I'm not exactly sure but I would assume it should intensify.  This is assuming none of the current loops have constant current but are both changing in time and out of phase pi/2 (see figure EM Propulsion 2.png). 


I guess the idea was if there was something similar going on inside the radiation cavity though I can't quite say that there is.  There is also the issue of the idea that radiation projected is conserving momentum but this is inside a cavity.  (see figure EM Propulsion 3.png)

Edit: sorry, changed pi/4 to pi/2
There are many things to discuss here.

Of course  the electric field and magnetic fields are two interrelated aspects of a single object: the electromagnetic field tensor in 3D+1=4 spacetime. 

But if your loops and images are to be interpreted literally only in terms of electric current loops, please notice that your image then could be interpreted as saying that only TE (transverse electric) modes would produce the effect you are seeking, because only TE modes have the electric field in the azimuthal (circumferential) direction.  But notice that NASA Eagleworks is currently successfully testing in a vacuum TM modes (and actually I understand they have a preference for TM modes), for which the magnetic field B has a component only in the azimuthal (circumferential) direction while the electric field E has components perpendicular to the magnetic B field.  The electric field has zero component in the circumferential direction for TM modes.

Oh wow!  I just realized what your saying and after looking at the website, "http://gregegan.customer.netspace.net.au/SCIENCE/Cavity/Cavity.html" TM modes I suspect might also provide propulsion.  If it does exist I suspect it's a relativistic effect.  I didn't want to bring it up as I wanted to first confirm the TM effect would work and hence my quote from my other thread, "If this effect exists (TE) I have another concept that could possibly provide even more propulsion than this but again I am eager to see if even this effect (TM) can be shown to exist. " At the time I didn't understand TE and TM. 

I don't want to just jump up and say that is what this is but it almost looks like the other method I was thinking about.  I am still uncertain of the current phase evolution in the image in time if my assumption on the current in the image is correct.  I do suspect it could provide propulsion in a similar way to TE.  Attached is an image of the gregegan website above and I marked up one of his images (all due respect) to show the current directions which are hopefully accurate.  The currents towards/away from each other are what I suspect could push each other when out of sync by 90 degrees but I don't think it is magnetic for my own reasons.  I'm not certain the effect exists though I suspect it should. 

One thing that bothers me is at the moment I don't think I could explain by the phase concept how the one on the far left could provide propulsion if it does.  ... unless its like a rail gun but I thought they had recoil... unless maybe the phase delay violates the action reaction?  mmm, I don't even know for sure if there is a phase delay is my biggest problem.  Maybe there is a phase delay assuming things are alternating between capacitance then inductance and the radiation has to be injected from a certain direction. 
Title: Re: EM Drive Developments - related to space flight applications - Thread 2
Post by: Stormbringer on 03/27/2015 04:53 am
Random silly question:  If Eagleworks can build a torque balance with sufficient sensitivity to measure the current level of thrust above sigma why can't GRC? It seems to me that GRC would have every motive to do so since it would be of more use than the one they have now. They have one now. Therefore they need one. If they need one it stands to reason they would want the most sensitive one they could manage.
Title: Re: EM Drive Developments - related to space flight applications - Thread 2
Post by: Mulletron on 03/27/2015 08:58 am
Ok, here's my plan for the way forward:

I have all the data I need now in order to justify moving forward with the unloaded cavity test runs. To accomplish this: I must build the balance and build the Galinstan slip ring for powering the gear riding the balance.

I intend to use the leftover material from the sheet of 1" thick HDPE to construct the slip ring. I will provide photos and drawings of what I have in mind at a later time. For now the concept is still in my head. My 3 day weekend will be devoted to building the slip ring. I have 30grams of Galinstan to play with.

So this weekend.
-Build the slip ring
-Set up the balance
-Integrate everything together.

The slip ring is essentially as follows:
1) 3" diameter HDPE disc with a center hole to hold a small puddle of Ganinstan. Around this hole is a narrow ring, also to hold Galinstan. On the bottom of the HDPE disc, power wires will be run through a groove I mill, and will protrude through drilled/sealed holes into the Galinstan. Around all this a wider channel which will be filled with oil to act as the damper.

2) 3" diameter HDPE disc attached to bottom of the balance at COM/CORotation, with 2 polished and chromed electrodes to pick up power from center hole and ring in other HDPE disc. Power wires will exit from the back of the HDPE disc, through a small hole in the underside of the balance to the topside of the balance.

Next I will fabricate what goes into the oil filled channel of the damper below. I haven't decided what exactly to do with this yet. I want to build a damper system that is adjustable. So that idea is still not settled.

If I can pull this off, the only resistance to rotation on the balance will come from the viscosity of the Galinstan and from the oil damper. If contact is made between the rotating power wires and the HDPE, this will be mitigated by using chromed electrical pickups and the low friction HDPE. Hopefully the Galinstan doesn't corrode the electrical contacts.

Now for the way forward with future loaded cavity testing. I have decided to create a mounting solution for the small end which will support rapidly bolting and unbolting dielectric discs to a heavier sheet of copper, without causing damage. This will use the 3 bolt approach which I learned from Paul March but will be modified in the following way.

On the way is a thicker sheet of 0.043" copper which will be cut to a 6.75" diameter disc. 3 holes will be drilled at 120 degrees, equally spaced. The difference is that, over these holes, I will permanently solder 3, steel nuts and washers to the outside face. Then any dielectric disc will be bolted through from the inside into these nuts. The mounting bolts will be countersunk into the dielectric, in order to provide a smooth interface. The ends of the nuts (on the outside) will be trimmed and covered with conducting copper tape.

Thicker sheet:
http://www.amazon.com/gp/product/B000H9N44I/ref=oh_aui_detailpage_o01_s00?ie=UTF8&psc=1

Bolts: (note the countersinking bolt)
http://www.amazon.com/gp/product/B00NQR0MJI/ref=oh_aui_detailpage_o00_s00?ie=UTF8&psc=1

Challenge to overcome: I am missing one of two lead bricks which were supposed to be control for testing the balance. One arrived in the mail, the other?

Time to build.



Title: Re: EM Drive Developments - related to space flight applications - Thread 2
Post by: SpoCk0nd0pe on 03/27/2015 01:11 pm
Is there any news about the independent verification and validation? It's very hard to find anything reliable on the web :(
Title: Re: EM Drive Developments - related to space flight applications - Thread 2
Post by: Donosauro on 03/27/2015 01:54 pm
Hopefully the Galinstan doesn't corrode the electrical contacts.

"http://www5vip.inl.gov/technicalpublications/documents/3314568.pdf" says Galinstan pits copper. The Wikipedia page on Galinstan says it corrodes aluminum.

"http://www.rgmd.com/msds/msds.pdf" says it weakens aluminum, and lists a number of other metals that it tends to alloy with, including gold and silver.
Title: Re: EM Drive Developments - related to space flight applications - Thread 2
Post by: Rodal on 03/27/2015 02:28 pm
Is there any news about the independent verification and validation? It's very hard to find anything reliable on the web :(
If you are referring to the paragraph:

Quote from: Brady et.al. Anomalous Thrust Production from an RF Test Device Measured on a Low-Thrust Torsion Pendulum
The current plan is to support an IV&V test campaign at the Glenn Research Center (GRC) using their low thrust torsion pendulum followed by a repeat campaign at the Jet Propulsion Laboratory (JPL) using their low thrust torsion pendulum. The Johns Hopkins University Applied Physics Laboratory has also expressed an interest in performing a Cavendish Balance style test with the IV&V shipset.

I get the impression (from discussion in this thread with Star-Drive and others) that there are NO present plans at the Jet Propulsion Laboratory (JPL) and/or the Johns Hopkins University Applied Physics Laboratory of doing any verification of the NASA Eagleworks tests.

Only Glenn Research Center (GRC) is being considered if (and only if) NASA Eagleworks can give them a drive capable of producing more than 100 microNewtons force in a consistent manner to enable measurements at Glenn Research Center (GRC).  It appears that Glenn Research Center (GRC) does not have the budget and/or interest in modifying their testing device to verify NASA Eagleworks tests at forces lower than 100 microNewtons.

To follow the scientific method, experimental and theoretical results must be replicated by several others in the scientific community for the experimental and theoretical results to be accepted.   Adequate funding and interest are necessary for those experimental replications to take place.

Even if the EM Drive is verified, to enable the crewed space missions to the outer planets proposed by NASA Eagleworks, a political consensus will be necessary to enable the necessary nuclear power for the EM Drive, to make this vision attainable.
Title: Re: EM Drive Developments - related to space flight applications - Thread 2
Post by: Mulletron on 03/27/2015 08:03 pm
Hopefully the Galinstan doesn't corrode the electrical contacts.

"http://www5vip.inl.gov/technicalpublications/documents/3314568.pdf" says Galinstan pits copper. The Wikipedia page on Galinstan says it corrodes aluminum.

"http://www.rgmd.com/msds/msds.pdf" says it weakens aluminum, and lists a number of other metals that it tends to alloy with, including gold and silver.

Hey good looking out and thanks for the tips!

Progress for today:
https://drive.google.com/folderview?id=0B4PCfHCM1KYoTl90eDBuMklOeTg&usp=sharing&tid=0B4PCfHCM1KYoTXhSUTd5ZDN2WnM

Title: Re: EM Drive Developments - related to space flight applications - Thread 2
Post by: zen-in on 03/27/2015 11:27 pm
On the way is a thicker sheet of 0.043" copper which will be cut to a 6.75" diameter disc. 3 holes will be drilled at 120 degrees, equally spaced. The difference is that, over these holes, I will permanently solder 3, steel nuts and washers to the outside face. Then any dielectric disc will be bolted through from the inside into these nuts. The mounting bolts will be countersunk into the dielectric, in order to provide a smooth interface. The ends of the nuts (on the outside) will be trimmed and covered with conducting copper tape.

Time to build.

You are motivated!   Instead of steel nuts why not use bronze or brass ones?   Bronze hardware is a bit pricey and you would have to find a chandlery (maybe an online store).   It is a lot easier to solder (lead-tin assumed) to bronze or brass,  Steel requires several applications of acid flux to get it fully tinned.   It likely won't make any difference to the internal wave pattern if the nuts are on the outside.   Non-metallic bolts would be needed inside though.   You don't want a couple of resonant antennas.   Have fun!   I own a lathe and mill that I use for my own mad scientist projects.   I would offer you some help but I seem to recall you saying you were in Germany.    Here's a picture of the lathe after I moved it into my garage.  A South Bend 10L made in 1941 and honorably discharged from the Navy.  I have a piece of bronze in the milling adapter.    The mini-mill was a freeby.  I had to order some missing pieces and made the table.
Title: Re: EM Drive Developments - related to space flight applications - Thread 2
Post by: Flyby on 03/28/2015 03:25 pm
Just an attempt to see it from another angle or perspective...

So far, the focus of the discussion has been concentrated on the small and big plate of the frustum for receiving the presumed forces generated by the electromagnetic fields, but what is that wasn't the case?

What if the internally generated magnetic field forces interact with the sidewalls instead of the front/back ends?
Because of the angled sides, the internal forces on the sides would be diverted towards the front plate.

Compare it to squeezing  a soap cone: if a circular force is applied from the outside it will move toward the large plate, however if a force is applied from the inside, it will move towards the smaller plate....(just like Shawyer's rotating test rig)
It would also possibly explain to why there is no force detected in a cylindrical cavity.

I have not yet seen any reasoning (maybe i missed it) to why we're all assuming that forces are generated on the front/back end plates - as currently been discussed - and not on the sides walls?
Title: Re: EM Drive Developments - related to space flight applications - Thread 2
Post by: Rodal on 03/28/2015 04:55 pm
Just an attempt to see it from another angle or perspective...

So far, the focus of the discussion has been concentrated on the small and big plate of the frustum for receiving the presumed forces generated by the electromagnetic fields, but what is that wasn't the case?

What if the internally generated magnetic field forces interact with the sidewalls instead of the front/back ends?
Because of the angled sides, the internal forces on the sides would be diverted towards the front plate.

Compare it to squeezing  a soap cone: if a circular force is applied from the outside it will move toward the large plate, however if a force is applied from the inside, it will move towards the smaller plate....(just like Shawyer's rotating test rig)
It would also possibly explain to why there is no force detected in a cylindrical cavity.

I have not yet seen any reasoning (maybe i missed it) to why we're all assuming that forces are generated on the front/back end plates - as currently been discussed - and not on the sides walls?

Thanks for bringing this up as it allows to clarify a few matters.

In my analysis: see for example http://forum.nasaspaceflight.com/index.php?topic=36313.msg1351261#msg1351261 there are no assumptions whatsoever regarding

<<assuming that forces are generated on the front/back end plates - as currently been discussed - and not on the sides walls>>.

As long as they invoke  Poynting's vector  (http://en.wikipedia.org/wiki/Poynting_vector), there is no such assumption (that forces are generated at the base plates) made at NASA Eagleworks either (either by Dr. White or by Paul March).

As the images in  http://forum.nasaspaceflight.com/index.php?topic=36313.msg1351261#msg1351261
show, the (non self-cancelling component of the) Poynting vector is radially oriented towards the small base of the truncated cone.

Since the components of the electric field E parallel to a copper surface (either the wall or the bases) must be zero at the surface, the Poynting vector component perpendicular  to a copper surface (either the wall or the bases) must be zero at the copper surface  (either the wall or the bases) .

Let me repeat that: the Poynting vector component perpendicular to the small and the big bases of the truncated cone must be zero at those surfaces (must be zero at the small base and must be zero at the big base).

For a Transverse Magnetic (TM) mode the Poynting vector parallel to the surface doesn't have to be zero.  Actually, as the images show, in some cases the maximum Poynting vector occurs at the wall for a TM mode, and for a TM mode the Poynting vector at the wall must be parallel to the wall.


For example:

(http://forum.nasaspaceflight.com/index.php?action=dlattach;topic=36313.0;attach=818359;image)


On the other hand, for Transverse Electric (TE) modes both components of the Poynting vector (parallel to the wall and perpendicular to the wall) must be zero at the copper surfaces (either the wall or the bases) .  For TE modes the Poynting vector is zero at all copper surfaces: zero at the walls and zero at both of the truncated cone bases.

For example, see the Poynting vector component in the radial direction for Transverse Electric mode TE012 for NASA Eagleworks cavity (without a dielectric).  Notice that Poynting's vector is zero at all the surfaces: at walls and at both bases.
Title: Re: EM Drive Developments - related to space flight applications - Thread 2
Post by: Rodal on 03/28/2015 11:44 pm
Notice that on page 18 of NASA's report "Anomalous Thrust Production from an RF Test Device Measured on a Low-Thrust Torsion Pendulum" (http://www.libertariannews.org/wp-content/uploads/2014/07/AnomalousThrustProductionFromanRFTestDevice-BradyEtAl.pdf ) the authors write:


Quote from: David A. Brady, Harold G. White, Paul March, James T. Lawrence, and Frank J. Davies
There appears to be a clear dependency between thrust magnitude and the presence of some sort of dielectric RF
resonator in the thrust chamber. The geometry, location, and material properties of this resonator must be evaluated using numerous COMSOL® iterations to arrive at a viable thruster solution. We performed some very early evaluations without the dielectric resonator (TE012 mode at 2168 MHz, with power levels up to ~30 watts) and measured no significant net thrust.

The readers should not jump to the conclusion that this single experimental evidence means that the EM Drive must work only with a dielectric polymer (like HDPE or PTFE used by NASA).

Examination of the Poynting vector radial component shows that for this particular mode (TE012) without a dielectric, the Poynting's vector is self-cancelling and hence it is not a surprise that NASA measured no thrust force for this TE012 mode without a dielectric, since according to NASA Eagleworks' own theory (relying on Poynting's vector as per Dr. White's papers) there should not be a thrust force without a dielectric for mode TE012 because Poynting's vector self-cancels for this mode.

(Notice that there are two columns of cells, each column having 4 cells in the North-South vertical direction of the truncated cone section.  The columns have left-right symmetry around the central axis, so we only need to refer to one of the columns.

Looking at one column of cells, the upper two cells have Poynting vectors of similar magnitude pointing in different directions:

Uppermost cell: Poynting vector pointing towards small base. Medium Magnitude.
Second cell from the top: Poynting vector pointing towards big base.  Medium Magnitude.

Ditto for the bottom two cells in the same column:

Third cell from the top: Poynting vector pointing towards small base.  High Magnitude.
Fourth cell from the top: Poynting vector pointing towards big base.  High Magnitude.

)

However, for other modes (TM311 for example), Poynting's vector is not self-cancelling, but it is pointed towards the small base.  This justifies the fact that Shawyer communicates that he is presently not using a dielectric, since a dielectric does not appear necessary for certain modes.



@frobnicat: does this experiment (NASA reporting no experimental thrust for TE012 without a dielectric, but reporting thrust force with a dielectric) nullify mechanistic theories such as the one you recently proposed? (or did I miss something that salvages your mechanistic conjecture for this case?)
Of course, electromagnetic artifact explanations that rely on Poynting's vector are still viable, if somebody can come up with such an artifact explanation.


Title: Re: EM Drive Developments - related to space flight applications - Thread 2
Post by: frobnicat on 03/29/2015 10:13 am

...

@frobnicat: does this experiment (NASA reporting no experimental thrust for TE012 without a dielectric, but reporting thrust force with a dielectric) nullify mechanistic theories such as the one you recently proposed? (or did I miss something that salvages your mechanistic conjecture for this case?)
Of course, electromagnetic artifact explanations that rely on Poynting's vector are still viable, if somebody can come up with such an artifact explanation.


Job taking a revenge, don't have much time to explore the consequences of surprising latest precisions by Paul March. For a purely thermal mechanistic theory explaining in part or in whole the signal there is now need to integrate a few facts and data points reported, while some of them could be fragile or fluke as there is not much samples for those cases.

From the point of view of purely thermal mechanistic, on a (even very slightly) tilted pendulum, what is to be explained is not "thrust" but "displacement of LDS reading", remembering that they are not equivalent and that sustained recorded displacements would be induced by sustained shift in position of some part wrt fixation to pendulum arm. That is we would have displacement (vertical axis on charts) proportional to some part(s) position(s), not to first derivative of said position(s) since there is no viscous friction, nor second derivative of said position(s) since the magnitude of recoil effects are too weak and too short lived to play significant role in sustained readings.

In experimental plots, vertical scale varies as a factor from 1µm to 5µm for same cal pulse of 29.1µN. The cal pulse wrt to the scale (apparent total stiffness) is an indirect indicator for the constant that links CoM shift to LDS displacement readings since it gives tilted pendulum component relative to flexure bearings component (components of rest equilibrium restoring torque).  Expect that the more the LDS displaces in µm for the same 29.1µN (real force), the less the tilted pendulum component, ie. the less the system reacts to CoM's shift.

Few surprising facts and data points for this set of hypothesis :

- That was known from beginning (and always was a difficulty for purely thermal mechanistic) : for some modes there is "no significant net thrust" without dielectric while same mode with dielectric exhibits thrust. This is known for TE012. The absence of thrust without dielectric was for "some very early evaluations", the experimental plot is not published. A TE012 mode with dielectric and thrust is reported in Brady's report. The experimental plot in question, fig. 22 p.18 is unusual in a few respects : the calibrations pulses are 300V and valued at equivalent 60.1µN (instead of 200V 29.1µN for all other published experiments). The vertical scale in µm is absent. The reported "thrust"/power ratio is quite high (compared to other experiments). The rise and fall have a fast component and a slow component, the slow component is very slow (time constant clearly > 30s) and of huge magnitude wrt fast one. Fast component is not fast enough to "ring the bell" the same way the cal. pulses do. All this would hint at "mainly thermal". But then we would expect some comparable thing without dielectric (unless with/without dielectric would alter the mode enough so that heating are quite different). We still haven't seen an experimental plot (and accompanying data) of "no significant net thrust" without dielectric.. Also : apart from TE012 do we know another mode explicitly reported as having no thrust without dielectric and thrust with ? BTW what would be the heating profile for TE012, with and without dielectric ?

- The "turned 180°" experiment, with the small end toward right, should report the same thrust profile but in the opposite. It isn't. This would hint at interaction with vacuum chamber walls (the only apparent introduced asymmetry while turning test article 180°), that would nullify both thermal mechanistic and EMdrive effects. But the cardboard box experiment tend to show otherwise : metallic walls in the vicinity apparently don't play an important role. There is apparent contradiction. Also the "180°" turn experiment was reported as having a dysfunctional RF amplifier (from recollection, anyone can confirm ?) why it is much shorter in duration than usual (?). Would it be possible that this plot is a fluke and not representative ? The idea that pendulum arm is not responsive in the same manner when driven from left to right than the other way around seems quite unlikely : that would mean that the (mechanical) system is behaving in a very far from linear fashion (for instance with solid friction). The cal. pulses are here to show that it is not the case, added on top of a drifting baseline, the same dip is shown whether starting a place or another : quite linear apparently. Another asymmetry introduced by the 180° turn is the position of nylon bolts relative to vertical : for a TM212 (cyl.) the 3 120° spaced bolts that hold dielectric don't bath at same level of microwave heating (because TM212 is not a 120° symmetric mode around axis). If the 180° turn wasn't around Z but around X (as it appear to be from the pictures), a more heated bolt that was above is now below (or the opposite, have to check). Anyway, that could make a difference. And if the "180° turn" plot is to be taken as reliable, this shows that it would make quite a difference. That would hint at a central role of nylon bolts, that are hot enough to melt sometimes, and therefore can be quite often travelling through their glass transition temp. (much lower), with strong nonlinear (wrt Temperature) evolution of Young modulus.

- Last but not least, the "real reversal" by putting dielectric at one end or the other. Note on the plot given with a thrust toward big end (the only plot showing that) that the displacements show a very unusual step-down on top of the dip of "reversed thrust".

(http://forum.nasaspaceflight.com/index.php?action=dlattach;topic=36313.0;attach=817227;image)

The position of LDS reading seems "permanently" changed by a "thrust" pulse. This hints at a remanence. Magnetic ? Maybe. From purely thermal mechanistic hypothesis this looks like permanent plastic deformation or hysteresis remanence. The only other place I see indication for a thrust "in the wrong direction" is in this post (http://this post) where there is question of partial melting of nylon bolts... Again, if some experiment go up to melting, then quite a lot could actually be operating around glass transition and some of them near melting. From this site (http://www.ptonline.com/columns/the-effects-of-temperature) : we see here that between around 50°C and 100°C the drop in rigidity is huge, this is much lower than the actual melting (220°C). Glass transition is reversible (I think) but may show hysteresis (no ?). How would a nylon bolt under stress (ie. tensioned) behave in length when cycling around the glass transition, would it loosen the fixed dielectric then hold it tight again (against springy slightly warped end PCB plate, we are talking µm...) ? 

(http://d2n4wb9orp1vta.cloudfront.net/resources/images/cdn/cms/0811ptMaterials1a.jpg)

Don't throw thermal mechanistic through the window. Experimental data can put it to the ground, but through the stairway, one downstep at a time. This post will inevitably raise more questions and objections, this is just ongoing speculations, I won't have time to really support all that in the coming days.
Title: Re: EM Drive Developments - related to space flight applications - Thread 2
Post by: Notsosureofit on 03/29/2015 01:28 pm
FYI

http://www.ebay.com/itm/261252706326?_trksid=p2059210.m2749.l2649&ssPageName=STRK%3AMEBIDX%3AIT

Adding this to the pile as my analyzer only goes to 1GHz.  Thinking of macining off one end for testing dielectrics.
Title: Re: EM Drive Developments - related to space flight applications - Thread 2
Post by: Rodal on 03/29/2015 01:53 pm
....
The position of LDS reading seems "permanently" changed by a "thrust" pulse. This hints at a remanence. Magnetic ? Maybe. From purely thermal mechanistic hypothesis this looks like permanent plastic deformation or hysteresis remanence. The only other place I see indication for a thrust "in the wrong direction" is in this post (http://this post) where there is question of partial melting of nylon bolts... Again, if some experiment go up to melting, then quite a lot could actually be operating around glass transition and some of them near melting. From this site (http://www.ptonline.com/columns/the-effects-of-temperature) : we see here that between around 50°C and 100°C the drop in rigidity is huge, this is much lower than the actual melting (220°C). Glass transition is reversible (I think) but may show hysteresis (no ?). How would a nylon bolt under stress (ie. tensioned) behave in length when cycling around the glass transition, would it loosen the fixed dielectric then hold it tight again (against springy slightly warped end PCB plate, we are talking µm...) ? 

(http://d2n4wb9orp1vta.cloudfront.net/resources/images/cdn/cms/0811ptMaterials1a.jpg)

Don't throw thermal mechanistic through the window. Experimental data can put it to the ground, but through the stairway, one downstep at a time. This post will inevitably raise more questions and objections, this is just ongoing speculations, I won't have time to really support all that in the coming days.



So what we have, are two viscoelastic polymers HDPE and Nylon 6.  Both of them suffer from a viscoelastic relaxation at operating temperatures.  You have an excellent point that this calls into question the effectiveness of the Nylon 6 bolts to establish contact between the HDPE and the copper surface, as the amount of compressive stress applied by the Nylon 6 bolts decreases with temperature.

Since the E' modulus of HDPE also decreases with temperature the amount of compressive stress necessary to establish contact decreases with temperature.  As the temperature increases the HDPE dielectric becomes more compliant and hence less stress is necessary to establish contact. But since the E' reduction due to glass transition of the Nylon 6 bolts may be greater, it looks like separation could occur at some higher temperature.  This would be a function of the amount of pre-stress applied on the bolts as well as the temperature experienced during operation.

It would have been better to use a bolt material that does not experience a glass transition temperature in the operating temperature range.  The choice of a polymer bolt that has greater amount of amorphous % than the dielectric is questionable.  Also, the location chosen for the the outer Nylon 6 bolts is non-optimal.  The only optimally located Nylon 6 bolt is the one located at the center.


I mentioned using the following adhesive as an alternative to bolts:

Look at page 51 of this report from Henkel North America (Loctite): http://www.henkelna.com/us/content_data/237471_LT2197_Plastic_Guide_v6_LR7911911.pdf

which are specifically recommended for polyethylene (a very hard to bond polymer)

(Ethyl Cyanoacrylate) Loctite 401 Prism from Amazon:

http://www.amazon.com/Loctite-Instant-Adhesive-Prism-Bottle/dp/B006GOKRSY/ref=pd_sim_sbs_indust_1?ie=UTF8&refRID=14RMFHYMSB2780RR33CY

Use of this primer, Primer for Loctite 401 Prism from Amazon, increases the adhesive shear strength by a factor of 40 times, from 50 psi to 2000 psi  !!!!!:

http://www.amazon.com/Loctite-LOC-18396-770-Cyanoacrylate-Adhesive/dp/B001OBQ8VO/ref=sr_1_fkmr0_1?s=hi&ie=UTF8&qid=1427237585&sr=1-1-fkmr0&keywords=Loctite%C2%AE+770%E2%84%A2+Prism%C2%AE+Primer



The percentage of crystallinity of a polymer depends on many factors including chemical structure, interaction between polymer chains and processing conditions.  This Wikipedia page http://en.wikipedia.org/wiki/Crystallization_of_polymers#Degree_of_crystallinity gives the amounts for some materials of interest:

Nylon 6 (shown in the example above quoted by @frobnicat, which is relevant to the nylon bolts) 35 to 45%
HDPE (used as a dielectric in published NASA Eagleworks tests for the truncated cone) 70 to 80 %
PTFE (used as a dielectric in a few NASA Eagleworks tests) 60 to 80%


Since  only the amorphous part contributes to the glass transition, the glass transition (Tg) for highly crystalline polymers like High Density Polyethylene (HDPE) is weaker than for more amorphous polymers like Nylon 6.

For the dependence of E' (or G') vs. temperature, obtained from DTMA tests, see:

This 1995 article shows a gradual decay from 0 deg C to 120 deg C of E'  for extruded HDPE samples : http://www.unt.edu/LAPOM/publications/pdf%20articles/Boiko95PE.pdf






A minor point: strictly speaking, the ordinate in the example shown above for Nylon 6 and Polycarbonate should be labeled E' (the in--phase viscoelastic "storage modulus" component) , or, if they made the necessary adjustment, E* (the "dynamic modulus") instead of "Elastic Modulus", since the material is viscoelastic



Since these are thermoplastic polymers (as opposed to thermoset polymes like epoxy for example) the amorphous transition is indeed reversible.  There should be some amount of hysteresis due to viscoelasticity, which would be due to the out-of-phase component E" (or due to tan delta  = E"/E' depending on how you express it).

The viscoelastic hysteresis looks like an ellipse (instead of the piecewise hysteresis of elastic-plastic materials).  The area of the ellipse is related to the viscoelastically dissipated power in the polymer.

What kind of hysteresis do you have in mind ?

1) Due to the extremely high frequency (GHz) of the time-dependence of the applied electromagnetic field responsible for heating?

or

2) due to the extremely slow (in comparison) cycling of heating and cooling due to every experiment?

I imagine that your point refers to #2 (the slow cycling of heating and cooling due to every experiment).



I do not recall whether NASA Eagleworks checked the torque on the bolts, with a torque wrench, after every experiment to compare that measurement with the initial torque on the Nylon 6 bolts?

EDIT: If the torque wrench would show that the torque on the Nylon 6 bolts is lower after the test than prior to the test this decrease would be due to viscoelastic deformation of the Nylon 6 bolts due to the higher temperatures experienced during the test.
 
Title: Re: EM Drive Developments - related to space flight applications - Thread 2
Post by: Damon Hill on 03/29/2015 03:33 pm
http://www.dtic.mil/dtic/tr/fulltext/u2/a564120.pdf?

"Demonstration of a wingless electromagnetic air vehicle"

Pretty much proof-of-concept, goes into lots more detail than I have endurance to read.

Damon:

Good grief man, this is a great find!!  And an R&D activity I didn't even know was going on.  Now to figure out what is conventional plasma physics and what might be extended EM-Drive physics hiding in this University of Florida paper.

Thanks again,

Best, Paul M.   

As has been pointed out, this isn't a massless drive, of course.  Now that I'm thinking about it, this may simply be a basis for a useful type of thruster of some sort, but it also could be a caveat about unintended side effects of a strong RF field over a conductive surface that generates some tiny amount of thrust in an atmosphere.  On the opposite side of the surface, even.  That might lead to measurement errors.

In and of itself, the propulsion principle apparently demonstrated might lead somewhere, or not.  I should live so long to see it levitate something of serious mass, or an orbiting RF resonant cavity raise its orbit.  Maybe tack against the solar wind?

Mainly I was hoping this paper might contain some useful clues to someone.  Propulsion without moving parts or mass of any sort is slightly spooky to me.  For that matter, so is a Peltier cell, and I own one of those.

--Damon
Title: Re: EM Drive Developments - related to space flight applications - Thread 2
Post by: frobnicat on 03/29/2015 04:20 pm
...

...

What kind of hysteresis do you have in mind ? Due to the extremely high frequency (GHz) of the time-dependence of the applied electromagnetic field responsible for heating? Or due to the extremely slow (in comparison) cycling of heating and cooling due to every experiment?

Slow cycling of heating and cooling. Also note that the nylon bolts, at equivalent local EM excitation conditions (smoothed as "DC" heat flow with much higher time constant than ns) seem to swallow much more microwave than copper/hdpe/ptfe. Typical overall temperatures raises (from initial ambient) actually measured (from the outside) are an order of magnitude below the temperature of nylon fusing, yet it has been shown experimentally (http://forum.nasaspaceflight.com/index.php?topic=36313.msg1335190#msg1335190) that some part of nylon bolts at the small end was heated enough to melt, in spite of the AC EM fields amplitude being low in the small end. Could it be due to the water content of nylon ?

(http://www.toray.jp/plastics/en/amilan/technical/images/ami_tec_050.gif)

(http://www.toray.jp/plastics/en/amilan/technical/images/ami_tec_051.gif)

This later is for "nylon 6 pellets" exposed to air, from this page (http://www.toray.jp/plastics/en/amilan/technical/tec_007.html), don't know exactly the characteristic size of pellets for injection-molding, same order of magnitude as the bolts ?
 
What would be the effect of vacuum, compared to in air at usual lab ambient humidity ? Vacuum conditions would likely degas the water content of nylon, depending on the time during which nylon bolts are in vacuum they would  be dryer than when in air. Do the water absorbed in nylon couple to microwave heating as bulk water does ?

Questions here : is it because of water content that nylon seems to heat a lot in the frustum (compared to other parts) ? And if yes, what would be the difference in microwave heating when in vacuum due to water content degasing, depending on vacuum conditions duration ? That could explain the difference in magnitude of the effects in vacuum relative to in air (not jumping to conclusions here, this would just be a fact compatible with conjecture). Also, what would be the complete stress response material history when mounting and tensioning the bolts in air with some water content, evacuating (drying the bolts), heating above glass transition, cooling below glass transition, cycling around glass transition a few times.

What is interesting from a thermal mechanical viewpoint with the nylon bolts is twofold :
- the temperature would be an order of magnitude higher the other temperature raises, that also means that the apparent time constant are much lower, in the seconds rather than 10s of seconds (how long would it take a nylon bolt at 100°C to cool below 50°C at power off ?)
- the nonlinearity of glass transition makes for a nice plateau : this plateau would translate in the plots (while just considering temperature one sees ramps more than plateaus)

I was sure that your knowledge of polymers would prove useful to polish this line of reasoning  :)
Unfortunately, as I said, paid job leaves me little spare time those days... bookmarking your precisions for later use.
Title: Re: EM Drive Developments - related to space flight applications - Thread 2
Post by: Rodal on 03/29/2015 04:50 pm
....

Few surprising facts and data points for this set of hypothesis :

- That was known from beginning (and always was a difficulty for purely thermal mechanistic) : for some modes there is "no significant net thrust" without dielectric while same mode with dielectric exhibits thrust. This is known for TE012. The absence of thrust without dielectric was for "some very early evaluations", the experimental plot is not published. A TE012 mode with dielectric and thrust is reported in Brady's report.....

What is new is the reporting of Poynting vector plots, to show the Poynting vector field for different modes and their significance.

http://forum.nasaspaceflight.com/index.php?topic=36313.msg1352243#msg1352243

http://forum.nasaspaceflight.com/index.php?topic=36313.msg1352368#msg1352368

Although Dr. White published several papers predicting that the force on the EM Drive would depend on the Poynting vector, for example,

Quote from: p.10 of Brady  et.al.'s  "Anomalous Thrust Production from an RF Test Device Measured on a Low-Thrust Torsion Pendulum"
Consideration of the dynamic fields in the ¼ wave resonance tube shows that there is always a net Poynting vector meaning that the RF launcher tube assembly with dielectric cylinder common to both the slotted and smooth test articles is potentially a Q-thruster where the pillbox is simply a matching network.

I have not seen any published calculations by Dr. White and his group of the Poynting vector field for the EM Drive. There are no Poynting vector plots shown in the paper "Anomalous Thrust Production from an RF Test Device Measured on a Low-Thrust Torsion Pendulum" by Brady et.al. and there have been no Poynting vector plots shown by Paul March in our threads either.

The Poynting vector (its divergence and its vorticity) plays a prominent role in Dr. John Brandenburg's paper (see attachment to http://forum.nasaspaceflight.com/index.php?topic=36313.msg1349142#msg1349142 ) but again, there is no computation and plots of the Poynting vector field for the truncated cone for different electromagnetic modes.

Shawyer has not published any Poynting vector calculations either. 

Prof. Juan Yang and her colleagues mention the Poynting vector in their calculations, but I have not seen any plots of the Poynting vector field reported in Prof. Juan Yang's papers either.

Greg Egan did not publish any plots of the Poynting vector field either, in his article:  http://gregegan.customer.netspace.net.au/SCIENCE/Cavity/Cavity.html

Consideration of the boundary conditions for the Poynting vector seem to also have not been mentioned in the published literature of the EM Drive.

I have not seen the following previously mentioned in any EM Drive report:

Quote
Since the components of the electric field E parallel to a copper surface (either the wall or the bases) must be zero at the surface, the Poynting vector component perpendicular  to a copper surface (either the wall or the bases) must be zero at the copper surface  (either the wall or the bases) .

Let me repeat that: the Poynting vector component perpendicular to the small and the big bases of the truncated cone must be zero at those surfaces (must be zero at the small base and must be zero at the big base).

For a Transverse Magnetic (TM) mode the Poynting vector parallel to the surface doesn't have to be zero.  Actually, as the images show, in some cases the maximum Poynting vector occurs at the wall for a TM mode, and for a TM mode the Poynting vector at the wall must be parallel to the wall.

On the other hand, for Transverse Electric (TE) modes both components of the Poynting vector (parallel to the wall and perpendicular to the wall) must be zero at the copper surfaces (either the wall or the bases) .  For TE modes the Poynting vector is zero at all copper surfaces: zero at the walls and zero at both of the truncated cone bases.


The following points do not appear to have been previously made either:

Quote from: Rodal
Examination of the Poynting vector radial component shows that for this particular mode (TE012) without a dielectric, the Poynting's vector is self-cancelling and hence it is not a surprise that NASA measured no thrust force for this TE012 mode without a dielectric, since according to NASA Eagleworks' own theory (relying on Poynting's vector as per Dr. White's papers) there should not be a thrust force without a dielectric for mode TE012 because Poynting's vector self-cancels for this mode.

....
However, for other modes (TM311 for example), Poynting's vector is not self-cancelling, but it is pointed towards the small base.  This justifies the fact that Shawyer communicates that he is presently not using a dielectric, since a dielectric does not appear necessary for certain modes.

Actually, Prof. Yang writes the complete opposite conclusion (http://www.emdrive.com/NWPU2010translation.pdf ):

Quote from: Yang Juan,Yang Le,Zhu Yu,Ma Nan
Using finite element numerical method to numerical analyse the classical Maxwell equation of electric field of the idealised conical resonator, to obtain the model and practical of the distribution of the electric field of the cavity under 1000W. By analyse the properties under different modes and the different properties. Calculation show that under the four modes, TE011, TE012, TE111 and TM011, the quality factor of TE012 is highest and with highest thrust, followed by TE011. With the Small End of the cavity unchanged, the quality factor and thrust decrease with the increase in the Large End

Prof. Juan Yang writes that her Finite Element calculations show mode TE012 as having the highest thrust (without dielectric).  My exact solution calculations show that the Poynting vector fields are self cancelling for TE012 without dielectric.

Furthermore NASA Eagleworks experiments confirm the self-cancellation of the Poynting vector field for mode TE012: NASA, using the truncated cone without dielectric, and excited at the TE012 frequency, "measured NO significant net thrust": the complete opposite of Prof. Juan Yang's conclusion, but in full accordance with my calculations of the Poynting vector distribution for mode TE012.
Title: Re: EM Drive Developments - related to space flight applications - Thread 2
Post by: Mulletron on 03/29/2015 08:12 pm
Galinstan slipring prototype constructed:

I milled out the sheet of 1" thick HDPE mostly by hand. I had the aid of a Dewalt hand drill and a Dremel type of tool.

30 grams Galinstan was exactly enough. The center Galinstan channel is 8mm deep and 16mm wide. The center channel is shaped inside like an inverted cone. This will aid in maintaining center.

The ring Galinstan channel is 8mm deep/6mm wide which is plenty and gives me room to adjust so the 3mm wide contacts don't touch anything but the liquid metal. Attached are photos showing open circuit, short circuit.

The Galinstan channel side walls and bottom are smooth. The oil damper channel side walls are smooth, but the bottom was left lumpy for now. I might just keep it that way. I'm torn on whether the effort to smooth out the bottom might take away the useful turbulence I could get from that lumpy bottom of the channel.

I'm keeping the Galinstan in the slipring from now on to make sure it will remain stable. Tolerances between the penetrating wires and the holes through the HDPE are extremely tight so I expect no leaks just on account of that. I took the extra measure of sealing the bottom and top of the holes with hot glue, then sandwiched a like bottom plate over the wires, further sealed by hot glue.

The wires are flush with the bottom of the Galinstan channel. Approximately 4mm of HDPE was milled out around the tops of the wire holes to allow liquid metal to make maximum contact with the copper wires.

Also attached are pics of balance overview and zoomed in shot. In the zoomed in shot, those wires will be cut to length and terminated with chromed contacts.

The damper fins/blades haven't been built yet. Should be pretty easy to pull off. The oil damper channel is 20mm wide with 10mm useful depth.

Next up is to hang it, energize it and see what systemic effects this approach causes. I can do AC or DC with this. If it works out, then great. If it don't work out (like it becomes a motor), I'll fly a battery.

Starting to feel like I'm finally making some progress on this.

Rest here:
https://drive.google.com/folderview?id=0B4PCfHCM1KYoTl90eDBuMklOeTg&usp=sharing&tid=0B4PCfHCM1KYoTXhSUTd5ZDN2WnM
Title: Re: EM Drive Developments - related to space flight applications - Thread 2
Post by: MazonDel on 03/30/2015 02:39 pm
Mulletron: I'm not too aware of the electrical properties of Galinstan and I might just be misinterpreting your wording, but if the bottom of the channel that the Galinstan is contained in is rough enough to have 'significant' height variance couldn't this cause possible issues for the cleanliness of your signal through it? A bit like using a potentiometer as a wire as the changing depth of the Galinstan would thus result in changing resistance over its length. So as your contact in the Galinstan moved across the channel, the changing resistance would add noise to it. I don't know how rough the surface is or if the noise in question is outside the realm of your tolerances, but I figured I should mention it.

Edit: It also occurs to me, that if you do want that roughness on the bottom as you mentioned, you should be able to use some filters on the rig to clean up the noise if you need to.
Title: Re: EM Drive Developments - related to space flight applications - Thread 2
Post by: Mulletron on 03/30/2015 07:06 pm
Mulletron: I'm not too aware of the electrical properties of Galinstan and I might just be misinterpreting your wording, but if the bottom of the channel that the Galinstan is contained in is rough enough to have 'significant' height variance couldn't this cause possible issues for the cleanliness of your signal through it? A bit like using a potentiometer as a wire as the changing depth of the Galinstan would thus result in changing resistance over its length. So as your contact in the Galinstan moved across the channel, the changing resistance would add noise to it. I don't know how rough the surface is or if the noise in question is outside the realm of your tolerances, but I figured I should mention it.

Edit: It also occurs to me, that if you do want that roughness on the bottom as you mentioned, you should be able to use some filters on the rig to clean up the noise if you need to.

Hi and welcome. The Galinstan channels are smooth and consistent with respect to depth and width. This was by design.


The Galinstan channel side walls and bottom are smooth.

Galinstan is recommended as a safer alternative to mercury in applications such as this due to its liquid metal behavior at room temperature and excellent electrical properties. I tested the resistance of Galinstan with my meter and found it performs as well as a wire. See pics above.

Electrical conductivity: 3.46x106 S/m (from the Oracle) http://en.wikipedia.org/wiki/Galinstan#cite_note-scitoys-3

Quote
A bit like using a potentiometer as a wire as the changing depth of the Galinstan would thus result in changing resistance over its length. So as your contact in the Galinstan moved across the channel, the changing resistance would add noise to it.

As far as the potentiometer reference. You can use a VERY long length of wire to make a pot, as even copper wire or Galinstan is considered a conductor, still has resistance. The resistance measured between 1mm or 1cm of wire or Galinstan is negligible. You can't even see it on the meter. Normally the resistive element for a potentiometer is graphite, or some other resistor. You can use wire, but you needs boat loads of wire in a coil.

I'm using this setup to couple power across a moving gap, like what you would find in a radar for power/status in the antenna. For the purposes intended, signal noise makes no difference to me. The power contacts which I have pictured are rated for 240VAC 15amps, and I am able to completely able to submerge the contacts in the liquid and not make contact with the bottom or the edges.

I only need 6VDC, 2 amps.

If I wanted to, I could literally run a space heater using this.

I'm prepared/equipped for both AC and DC. I have a 12VDC 15A power supply I use for my LIPO charger. I'm going to try AC first. I'm comfortable with (but still respect) high voltage.

I tested this with a meter and found no issues. I'll let you know if I encounter problems, the reason why, and if there is a fix/work around.

I'm more worried about spilling the liquid metal and/or bumping it and creating a short circuit. So I will develop protocols to ensure I don't have any accidents as this stuff is $90 for 30grams.

For others out there who don't want to use Galinstan, yet want to attempt a replication, there are numerous sliprings on Ebay which use carbon brushes, but as this balance is designed to measure very small forces, I opted to use Galinstan. When I get around to the Cavendish experiment, I'll know how sensitive the balance actually will be.

Title: Re: EM Drive Developments - related to space flight applications - Thread 2
Post by: MazonDel on 03/30/2015 07:23 pm
Thanks! Consider my concern withdrawn. I wasn't quite certain exactly to what degree small effects might factor into the design. At some point I'd love to end up doing some experimentation on this all myself, but I am currently involved in several other projects so mostly I'm just a lurker. Keep up the good work!
Title: Re: EM Drive Developments - related to space flight applications - Thread 2
Post by: Mulletron on 03/30/2015 07:47 pm
....

Few surprising facts and data points for this set of hypothesis :

- That was known from beginning (and always was a difficulty for purely thermal mechanistic) : for some modes there is "no significant net thrust" without dielectric while same mode with dielectric exhibits thrust. This is known for TE012. The absence of thrust without dielectric was for "some very early evaluations", the experimental plot is not published. A TE012 mode with dielectric and thrust is reported in Brady's report.....

What is new is the reporting of Poynting vector plots, to show the Poynting vector field for different modes and their significance.

http://forum.nasaspaceflight.com/index.php?topic=36313.msg1352243#msg1352243

http://forum.nasaspaceflight.com/index.php?topic=36313.msg1352368#msg1352368

Although Dr. White published several papers predicting that the force on the EM Drive would depend on the Poynting vector, for example,

Quote from: p.10 of Brady  et.al.'s  "Anomalous Thrust Production from an RF Test Device Measured on a Low-Thrust Torsion Pendulum"
Consideration of the dynamic fields in the ¼ wave resonance tube shows that there is always a net Poynting vector meaning that the RF launcher tube assembly with dielectric cylinder common to both the slotted and smooth test articles is potentially a Q-thruster where the pillbox is simply a matching network.

I have not seen any published calculations by Dr. White and his group of the Poynting vector field for the EM Drive. There are no Poynting vector plots shown in the paper "Anomalous Thrust Production from an RF Test Device Measured on a Low-Thrust Torsion Pendulum" by Brady et.al. and there have been no Poynting vector plots shown by Paul March in our threads either.

The Poynting vector (its divergence and its vorticity) plays a prominent role in Dr. John Brandenburg's paper (see attachment to http://forum.nasaspaceflight.com/index.php?topic=36313.msg1349142#msg1349142 ) but again, there is no computation and plots of the Poynting vector field for the truncated cone for different electromagnetic modes.

Shawyer has not published any Poynting vector calculations either. 

Prof. Juan Yang and her colleagues mention the Poynting vector in their calculations, but I have not seen any plots of the Poynting vector field reported in Prof. Juan Yang's papers either.

Greg Egan did not publish any plots of the Poynting vector field either, in his article:  http://gregegan.customer.netspace.net.au/SCIENCE/Cavity/Cavity.html

Consideration of the boundary conditions for the Poynting vector seem to also have not been mentioned in the published literature of the EM Drive.

I have not seen the following previously mentioned in any EM Drive report:

Quote
Since the components of the electric field E parallel to a copper surface (either the wall or the bases) must be zero at the surface, the Poynting vector component perpendicular  to a copper surface (either the wall or the bases) must be zero at the copper surface  (either the wall or the bases) .

Let me repeat that: the Poynting vector component perpendicular to the small and the big bases of the truncated cone must be zero at those surfaces (must be zero at the small base and must be zero at the big base).

For a Transverse Magnetic (TM) mode the Poynting vector parallel to the surface doesn't have to be zero.  Actually, as the images show, in some cases the maximum Poynting vector occurs at the wall for a TM mode, and for a TM mode the Poynting vector at the wall must be parallel to the wall.

On the other hand, for Transverse Electric (TE) modes both components of the Poynting vector (parallel to the wall and perpendicular to the wall) must be zero at the copper surfaces (either the wall or the bases) .  For TE modes the Poynting vector is zero at all copper surfaces: zero at the walls and zero at both of the truncated cone bases.


The following points do not appear to have been previously made either:

Quote from: Rodal
Examination of the Poynting vector radial component shows that for this particular mode (TE012) without a dielectric, the Poynting's vector is self-cancelling and hence it is not a surprise that NASA measured no thrust force for this TE012 mode without a dielectric, since according to NASA Eagleworks' own theory (relying on Poynting's vector as per Dr. White's papers) there should not be a thrust force without a dielectric for mode TE012 because Poynting's vector self-cancels for this mode.

....
However, for other modes (TM311 for example), Poynting's vector is not self-cancelling, but it is pointed towards the small base.  This justifies the fact that Shawyer communicates that he is presently not using a dielectric, since a dielectric does not appear necessary for certain modes.

Actually, Prof. Yang writes the complete opposite conclusion (http://www.emdrive.com/NWPU2010translation.pdf ):

Quote from: Yang Juan,Yang Le,Zhu Yu,Ma Nan
Using finite element numerical method to numerical analyse the classical Maxwell equation of electric field of the idealised conical resonator, to obtain the model and practical of the distribution of the electric field of the cavity under 1000W. By analyse the properties under different modes and the different properties. Calculation show that under the four modes, TE011, TE012, TE111 and TM011, the quality factor of TE012 is highest and with highest thrust, followed by TE011. With the Small End of the cavity unchanged, the quality factor and thrust decrease with the increase in the Large End

Prof. Juan Yang writes that her Finite Element calculations show mode TE012 as having the highest thrust (without dielectric).  My exact solution calculations show that the Poynting vector fields are self cancelling for TE012 without dielectric.

Furthermore NASA Eagleworks experiments confirm the self-cancellation of the Poynting vector field for mode TE012: NASA, using the truncated cone without dielectric, and excited at the TE012 frequency, "measured NO significant net thrust": the complete opposite of Prof. Juan Yang's conclusion, but in full accordance with my calculations of the Poynting vector distribution for mode TE012.

These seem important and I don't want these posts to slip from discussion.
I've only had time to peruse the above post and these (see below) and the one I'm quoting, but it seems like you're saying is that we should discard the Poynting vector? If so, in your opinion, what is a better way?
http://forum.nasaspaceflight.com/index.php?topic=36313.msg1352243#msg1352243
http://forum.nasaspaceflight.com/index.php?topic=36313.msg1352368#msg1352368

In your opinion, do you see any correlation between magnetic field strength at the large/small end as seen in the Comsol plots and the measured thrust levels?

A separate question, do you think ponderomotive forces are important here?

Title: Re: EM Drive Developments - related to space flight applications - Thread 2
Post by: Rodal on 03/30/2015 07:57 pm
....BTW what would be the heating profile for TE012, with and without dielectric ?.....

THERMAL PROFILES FOR DIFFERENT ELECTROMAGNETIC MODE SHAPES FOR A TRUNCATED CONE EM-DRIVE

Please find attached below the square of the norm of the magnetic field at the big base of the truncated cone without a polymer dielectric section for the following electromagnetic mode shapes:

Cyl. TE012: this is the mode shape we were discussing.  This is the  mode shape discussed by NASA Eagleworks in the "Anomalous..." report that was tested with and without a polymer dielectric section.  When tested without a polymer dielectric section they measured no thrust force.

Cyl. TM212: the only mode for which the temperature profile was reported by Paul March, as measured by an external thermal infrared camera (with a polymer dielectric section inside the cavity).   This is also a mode shape that takes place, without a dielectric near the frequency that @Mulletron is planning to test.

Cyl. TM311: This is also a mode shape that takes place, without a dielectric near the frequency that @Mulletron is planning to test.

The temperature distribution through the circular cross-section should match the (square of the norm of the) magnetic field distribution because the surface gets heated by the magnetic field induction heating producing eddy currents in the copper.

Frequency for cases: empty cavity, NO polymer dielectric inside

Mode        Frequency (GHz) [Exact sltn.]     Frequency (GHz) [COMSOL FEA]       Poynting Vector

Cyl. TE012     2.20244                                      2.1794                                                0
Cyl. TM311    2.45835                                      2.4068                                                Longitudinal
Cyl. TM212    2.49342                                      2.4575                                                 ~ 0

The frequency obtained by the Finite Element method ranges from 1% (for TE012) to 2% (for TM311) less than the exact solution.  The Finite Element solution is more stiff (the FEA converges from below).  Higher modes (like TM311) require a finer mesh to obtain the same level of accuracy as lower modes.


NOTE 1: The experimental results obtained by Paul March at NASA Eagleworks (see attachment to http://forum.nasaspaceflight.com/index.php?topic=36313.msg1327406#msg1327406) with a thermal infrared camera pointed at the big base of the truncated cone for mode TM212, fully confirm that heating of the big base of the truncated cone is due to the magnetic field heating the big base by induction heating: producing eddy currents on the base.  See these images


(http://forum.nasaspaceflight.com/index.php?action=dlattach;topic=36313.0;attach=635194;image)

(http://forum.nasaspaceflight.com/index.php?action=dlattach;topic=36313.0;attach=635195;image)



Note 2:  I have not yet had the time to write the computer program to calculate the exact solution for electromagnetic modes in the truncated cavity with a polymer dielectric insert, but based on the fact that both the reported thermal IR camera measurements by Paul March and the COMSOL Finite Element calculations by NASA were for TM212 with a polymer dielectric insert in the EM Drive cavity, and that both the measured temperature distribution and the Finite Element calculated distribution are in perfect agreement with my exact solution calculations for the EM Drive cavity without the polymer dielectric insert, it looks like the heat distribution for mode shape TE012 should be the same with and without the polymer dielectric section.
This conclusion is also supported by my exact solutions for the cylindrical cavity with and without the dielectric.

This conclusion is also supported on theoretical grounds, since the polymer dielectric insert is homogenous and isotropic HDPE (or at least transversely-isotropic), and hence it should not affect the mode shape circular cross-section distribution.

Title: Re: EM Drive Developments - related to space flight applications - Thread 2
Post by: Rodal on 03/30/2015 08:07 pm
... it seems like you're saying is that we should discard the Poynting vector? ...

I have not see any other calculations and plots of the Poynting vector for the EM Drive.

I showed results that for mode shape TM311, with the cavity without the polymer dielectric section, the Poynting vector points clearly towards the small base half of the time and towards the big base the other half of the time.  This mode has not been reported as having been experimentally tested yet.

I showed results that for mode shape TM212, the mode shape presently being tested by NASA Eagleworks in a partial vacuum, with the cavity without the polymer dielectric section, the Poynting vector practically self-cancels.  I predict that NASA Eagleworks would measure practically no thrust with the polymer dielectric removed.  NASA Eagleworks reported that when the Nylon 6 bolts melted, they lost thrust force.  This appears to be consistent with these results.

I showed the fact that boundary conditions for the transverse electric (TE) electromagnetic mode shapes all Poynting vector components vanish at the walls of the EM Drive and the fact that for mode shape TE012 without the dielectric the Poynting vector is self-cancelling.  This is consistent with NASA Eagleworks results and supports NASA Eagleworks preference for the transverse magnetic (TM) modes over the transverse electric (TE) modes.

When I have the time I also would like to calculate and show the distribution of the divergence of Poynting's vector as well as the vorticity of Poynting's vector for the truncated cone's EM Drive.
Title: Re: EM Drive Developments - related to space flight applications - Thread 2
Post by: Rodal on 03/31/2015 01:35 am
.....
In your opinion, do you see any correlation between magnetic field strength at the large/small end as seen in the Comsol plots and the measured thrust levels?
.....
One should not seek a correlation between the magnetic field vector by itself and the thrust force, even in MagnetoHydroDynamics, or for the EM Drive, because the magnetic field vector oscillates like a harmonic function, hence it spends as much time going in the negative direction as the amount of time that it spends going in the positive direction.  Ditto for the electric field vector by itself.

Instead, one should consider the quadratic functions that appear in the 3+1 spacetime form of the (contravariant) energy-stress tensor Tμν , which contains the Poynting vector (S) and Maxwell's stress tensor (σ) .   Quadratic functions such as E^2 and B^2 that vary as (Cos[ωt])^2 or (Sin[ωt])^2 have a positive time average.  However the quadratic function (Cos[ωt])(Sin[ωt]) is still self-cancelling over a period since it spends as much time with negative magnitude as with positive magnitude.  Poynting's vector varies as (Cos[ωt])(Sin[ωt]) = (Sin[2ωt])/2 and therefore oscillates at twice the frequency of the electromagnetic fields and it self-cancels over a period of oscillation

Both the energy and momentum conservation laws can be simply expressed in relativity in terms of the divergence (in 3+1 spacetime) of the energy-stress tensor  Tμν

(http://upload.wikimedia.org/math/9/0/6/9067342a3c3e13deacfc7cded6b5da36.png)

See:  http://en.wikipedia.org/wiki/Electromagnetic_stress%E2%80%93energy_tensor#Conservation_laws

The  energy-stress tensor  Tμν is also neat because it directly shows how energy and momentum are inexorably tied together in relativity: lack of conservation of momentum is tantamount to a lack of conservation of energy, as @frobnicat and others have repeatedly pointed out.
Title: Re: EM Drive Developments - related to space flight applications - Thread 2
Post by: Rodal on 03/31/2015 02:04 am
....

A separate question, do you think ponderomotive forces are important here?
I have not contemplated ponderomotive forces yet.
Title: Re: EM Drive Developments - related to space flight applications - Thread 2
Post by: ThinkerX on 03/31/2015 02:09 am
Doctor Rodal -

Could the discrepancy between your calculations and the Chinese results be due to a typo or translation error?  Given that both happen, and keeping in mind the now resolved dispute between your numbers and those of Eagleworks.
Title: Re: EM Drive Developments - related to space flight applications - Thread 2
Post by: Rodal on 03/31/2015 02:19 am
Doctor Rodal -

Could the discrepancy between your calculations and the Chinese results be due to a typo or translation error?  Given that both happen, and keeping in mind the now resolved dispute between your numbers and those of Eagleworks.
I don't think that this particular, specific pointed discrepancy (that mode TE012 without a dielectric resulted in no significant force in the NASA experiment and that it also has a self-cancelling Poynting vector field according to my calculations, while Prof. Juan Yang wrote in her 2010 paper that TE012 was the mode shape giving the highest thrust force without a dielectric) is due to a translation error.  I also doubt that it is a typo because that statement is made in the body of the article as well as in the conclusions and it can also be ascertained from the plots shown in her 2010 paper.

Title: Re: EM Drive Developments - related to space flight applications - Thread 2
Post by: ThinkerX on 03/31/2015 02:25 am
Quote
I don't think that the specific  pointed discrepancy (that mode TE012 without a dielectric resulted in no significant force in the NASA experiment and that it also has a self-cancelling Poynting vector field, while Prof. Juan Yang wrote in her 2010 paper that TE012 was the mode shape giving the highest thrust force) is due to a translation error.  I also doubt that it is a typo because that statement is made in the body of the article as well as in the conclusions and it can also be ascertained from the plots shown in the paper.

Wild speculation, then:

The Chinese, if I recollect correctly, are pumping a lot more energy into their device than Eagleworks.

I have seen repeated mention here before this results in a lot more heat, and the frequency changes with heat.  So maybe this morphs TE012 into something else?

 
Title: Re: EM Drive Developments - related to space flight applications - Thread 2
Post by: Rodal on 03/31/2015 02:29 am
Quote
I don't think that the specific  pointed discrepancy (that mode TE012 without a dielectric resulted in no significant force in the NASA experiment and that it also has a self-cancelling Poynting vector field, while Prof. Juan Yang wrote in her 2010 paper that TE012 was the mode shape giving the highest thrust force) is due to a translation error.  I also doubt that it is a typo because that statement is made in the body of the article as well as in the conclusions and it can also be ascertained from the plots shown in the paper.

Wild speculation, then:

The Chinese, if I recollect correctly, are pumping a lot more energy into their device than Eagleworks.

I have seen repeated mention here before this results in a lot more heat, and the frequency changes with heat.  So maybe this morphs TE012 into something else?

That's the first thing that jumps to mind, if one has to speculate.  Also she uses different equations to calculate the force.  Her equations also lead to another contrarian statement she makes in her 2010 paper (and in the conclusions of that paper) that keeping the small base at constant diameter and keeping the same axial length, increasing the diameter of the big base decreases the thrust.  (I wonder whether she still believes those calculations as her later experiments involve truncated cones with increased -rather than decreased-  cone angle of the EM Drive).
 
Title: Re: EM Drive Developments - related to space flight applications - Thread 2
Post by: Rodal on 03/31/2015 02:52 am
Quote
I don't think that the specific  pointed discrepancy (that mode TE012 without a dielectric resulted in no significant force in the NASA experiment and that it also has a self-cancelling Poynting vector field, while Prof. Juan Yang wrote in her 2010 paper that TE012 was the mode shape giving the highest thrust force) is due to a translation error.  I also doubt that it is a typo because that statement is made in the body of the article as well as in the conclusions and it can also be ascertained from the plots shown in the paper.

Wild speculation, then:

The Chinese, if I recollect correctly, are pumping a lot more energy into their device than Eagleworks.

I have seen repeated mention here before this results in a lot more heat, and the frequency changes with heat.  So maybe this morphs TE012 into something else?

That's the first thing that jumps to mind, if one has to speculate.  Also she uses different equations to calculate the force.  Her equations also lead to another contrarian statement she makes in her 2010 paper (and in the conclusions of that paper) that keeping the small base at constant diameter and keeping the same axial length, increasing the diameter of the big base decreases the thrust.  (I wonder whether she still believes those calculations as her later experiments involve truncated cones with increased -rather than decreased-  cone angle of the EM Drive).

But there are alternate explanations as well.  For example, many of her statements in her 2010 paper are based on her calculations, and not based on experiments.  It could be that her calculations (in that specific paper) that TE012 should give the highest thrust are incorrect.  It could also be that her calculation that she was testing TE012 mode was incorrect: there are no IR thermal images of her EM Drive to verify what mode shape she actually tested, the geometrical dimensions are not given (hence not possible to double check independently) and there is just not enough experimental detail given (unlike the NASA Eagleworks tests were a lot of information was given) to speculate and much less to know for certain.
Title: Re: EM Drive Developments - related to space flight applications - Thread 2
Post by: Mulletron on 03/31/2015 03:48 am
Following up on this post:
http://forum.nasaspaceflight.com/index.php?topic=36313.msg1349268#msg1349268

....and the quote below.


Based on what you're saying, I'm going to make another (hobbled) sense antenna, for now it will be just a solder cup. Then I will repeat the same VSWR test to see how things change.


Here's before and after shots showing the difference between matching drive and sense antennas compared with just a solder cup as the sense antenna.

I was able to confirm that when using just a solder cup as the sense antenna, it makes no difference in VSWR whether or not the sense antenna is terminated, left open, or shorted.

Differences include finding less resonant frequencies (because the sense antenna is also a resonant structure), and resonant frequencies change.

I noted that the output as measured at the sense antenna was attenuated by 32db compared to the input, which is good. That means more energy is staying in the cavity, instead of being coupled out.

So essentially, when drive and sense antennas are the same, the cavity is a really efficient cavity filter. Data from the previous run showed performance as good <2db loss from input to output. Since both antennas are the same, this ensures that energy is coupled into and out of the cavity with ease......not what we want here.

I also sacrificed a connector by nibbling down the solder cup 1mm at a time to see if the VSWR got better or worse. That testing showed no improvement whatsoever, actually it made things worse.

A bare solder cup is optimal.

So my most likely candidate for unloaded testing is 2413.5mhz. I have no clue what mode is being excited here.

http://www.telecomhall.com/what-is-vswr.aspx
Table at bottom of this link converts to reflected power.

Rest here:
https://drive.google.com/folderview?id=0B4PCfHCM1KYoaHhFNGprNlEzcm8&amp;usp=sharing&tid=0B4PCfHCM1KYoTXhSUTd5ZDN2WnM

Spectrum analyzer shots of this round of testing will be posted in there later.


Title: Re: EM Drive Developments - related to space flight applications - Thread 2
Post by: dustinthewind on 03/31/2015 08:00 am
I'm not sure MJ Pinheiro does consider this project related to the concept I was talking about.  Here is a quote from, "The Physical Basis of Electromagnetic Propulsion", http://arxiv.org/abs/1502.06288, "However, DEF don't work on the basis of EM radiation, like is supposed to propel the NASA and Chinese projects. IT is supposed to work on the near- fields propelling effect;".  Abbreviations are, " deferential electromagnetic force (DEF)".  If there is DEF going on maybe it would answer the question how propulsion could be happening inside the cavity without the emission of radiation.  I'm trying to find a link that indicates an overall poynting vector imbalance possibly connecting the two ideas but so far I'm having trouble putting my finger on it.  There are some other related papers I was looking over such as "Dimensional Analysis of Thrusting by Electromagnetic Inertia, http://arxiv.org/abs/1502.01917".  It looks like I should read up on this also, "http://ntrs.nasa.gov/search.jsp?R=20110023492"
Title: Re: EM Drive Developments - related to space flight applications - Thread 2
Post by: Rodal on 03/31/2015 02:00 pm
....

So my most likely candidate for unloaded testing is 2413.5mhz. I have no clue what mode is being excited here.

....

Yes you have been given a strong clue in this thread  :) ( http://forum.nasaspaceflight.com/index.php?topic=36313.msg1352878#msg1352878 ) as to what mode is being excited here:

Mode        Frequency (GHz) [Exact sltn.]     Frequency (GHz) [COMSOL FEA]       

Cyl. TE012     2.20244                                      2.1794                                               
Cyl.TM311    2.45835                                      2.4068                                                
Cyl. TM212     2.49342                                      2.4575                                               


Clearly this frequency ( 2.4135 GHz) falls right in the range of the calculated frequency for mode Cyl. TM311, if the dimensions of your truncated cone are within 1% of the assumed dimensions (Big Diameter=11 inches, Small Diameter=6.25 inches and Axial Length=9 inches).

(http://forum.nasaspaceflight.com/index.php?action=dlattach;topic=36313.0;attach=820280;image)

It is the best mode to excite (in that frequency range) with a cavity lacking a polymer dielectric, because this mode shape (TM311) has a clear Poynting vector.  The other modes have zero Poynting vectors in the longitudinal direction of the truncated cone.

As to why the measured frequency (if correct) is closer to the Finite Element Analysis (FEA) solution, here are possible reasons:

1) Internal dimensions of truncated cone may be 1% larger than the assumed internal dimensions in the exact solution analysis (Big Diameter=11 inches, Small Diameter=6.25 inches and Axial Length=9 inches).

2) Flatness of the big base and the small base.  The exact solution assumes spherical section surfaces for the bases while the Finite Element solution assumes them to be perfectly flat. This can be shown to make a small difference, and its sign (increasing or decreasing the frequency) depends on the electromagnetic mode shape.

3) Internal Damping: damping decreases the frequency of the damped solution as compared to the undamped solution.  The exact solution assumes infinite Q (undamped conditions). 

I attach again the predicted heating profile (at the big base) for this mode (TM311) from the exact solution
Title: Re: EM Drive Developments - related to space flight applications - Thread 2
Post by: spacenut on 03/31/2015 02:03 pm
Do Hall thrusters use Xenon or can they use a more common fuel since Xenon is rare.  Massive tonnage to Mars or the Moon over time will deplete a lot of Xenon.  I was thinking something like hydrogen, oxygen, or some other more common element for mass use in space. 
Title: Re: EM Drive Developments - related to space flight applications - Thread 2
Post by: aero on 03/31/2015 05:08 pm
Dr. Rodal,
Two questions:

You have the math for the dimensions of the cavities as a function of the resonant frequencies. Have you attempted to cast the dimensions in terms of the wavelength of the drive frequency, in closed form? That would be interesting to me in formulating models in Meep since I currently am modelling the geometry using wavelength as my base unit of length. Of course, maybe I am only asking for a simple division which I can easily do for each individual mode.

In discussing the Poynting vector you rely on the ideal boundary conditions being zero at the cavity walls. To what extent do the shear forces resulting from copper being "not ideal" modify these assumptions? 0%, .001%, 1%, 10%? And how large might those left over shear forces be?
Title: Re: EM Drive Developments - related to space flight applications - Thread 2
Post by: Stormbringer on 03/31/2015 05:12 pm
Do Hall thrusters use Xenon or can they use a more common fuel since Xenon is rare.  Massive tonnage to Mars or the Moon over time will deplete a lot of Xenon.  I was thinking something like hydrogen, oxygen, or some other more common element for mass use in space.
See the Electric Thruster thread. There are other options. the ELF thruster in particular can use anything. But that is where you will find them :) The answer though is yes. Ion thrusters can use stuff other than xenon.
Title: Re: EM Drive Developments - related to space flight applications - Thread 2
Post by: Rodal on 03/31/2015 05:16 pm
...Have you attempted to cast the dimensions in terms of the wavelength of the drive frequency, in closed form? ...
Something for me to add to "things to do when I have a chance"   :)
Title: Re: EM Drive Developments - related to space flight applications - Thread 2
Post by: Rodal on 03/31/2015 05:24 pm
...
In discussing the Poynting vector you rely on the ideal boundary conditions being zero at the cavity walls. To what extent do the shear forces resulting from copper being "not ideal" modify these assumptions? 0%, .001%, 1%, 10%? And how large might those left over shear forces be?
There are many boundary conditions.  I presume you are referring here to the electric permittivity of copper, effectively assuming copper's permittivity to be Infinite  so that the Electric Field E tangent to the copper surface is zero (to satisfy continuity of that vector component of E ).  It looks to me the effect of impurities should be extremely small, much smaller than 0.001%, see:

http://large.stanford.edu/courses/2007/ap272/peng1/

http://physics.stackexchange.com/questions/93542/dielectric-constant-or-permittivity-of-metals

https://www.researchgate.net/post/Why_is_the_dielectric_constant_for_metals_infinity

Continuity of the vector component of E tangent to the surface must be satisfied (see  http://en.wikipedia.org/wiki/Interface_conditions_for_electromagnetic_fields  ), so that if one were to pick a finite value for the permittivity of copper, it is straightforward to quantify the effect of assuming it to be Infinite, and one then readily arrives at the conclusion that the effect is extremely small (closer to 0 than to 0.001%).
Title: Re: EM Drive Developments - related to space flight applications - Thread 2
Post by: aero on 03/31/2015 06:25 pm
OK, so that means that by modelling the cavity as perfect metal, I'm not missing by very much. That is, perfect metal is a good assumption for copper at the frequencies under consideration. I wonder how much higher frequency I can use before that assumption starts to break down. I know that copper becomes transparent in the mid THz range, but I think it maintains its character up to  low THz frequencies. (1 or 2 THz).

I'm looking at the option of modelling the cavity at much higher frequencies, relying on the linearity of Maxwell's equations to relate to 2 GHz. The reason is that Meep runs much faster at higher frequencies where the dimensions measured in wavelength are smaller. Its kind of like trading spacial resolution for time resolution but they are way out of balance at 2 GHz for these cavities. What is the opinion on that idea?
Title: Re: EM Drive Developments - related to space flight applications - Thread 2
Post by: Mulletron on 04/01/2015 12:03 am
....

So my most likely candidate for unloaded testing is 2413.5mhz. I have no clue what mode is being excited here.

....

Yes you have been given a strong clue in this thread  :) ( http://forum.nasaspaceflight.com/index.php?topic=36313.msg1352878#msg1352878 ) as to what mode is being excited here:

Mode        Frequency (GHz) [Exact sltn.]     Frequency (GHz) [COMSOL FEA]       Poynting Vector

Cyl. TE012     2.20244                                      2.1794                                                0
Cyl.TM311   2.45835                                      2.4068                                                 Towards Small Base
Cyl. TM212    2.49342                                      2.4575                                                 ~ 0


Clearly this frequency ( 2.4135 GHz) falls right in the range of the calculated frequency for mode Cyl. TM311, if the dimensions of your truncated cone are within 1% of the assumed dimensions (Big Diameter=11 inches, Small Diameter=6.25 inches and Axial Length=9 inches).


It is the best mode to excite (in that frequency range) with a cavity lacking a polymer dielectric, because this mode shape (TM311) has a clear Poynting vector.  The other modes have zero Poynting vectors in the longitudinal direction of the truncated cone.

As to why the measured frequency (if correct) is closer to the Finite Element Analysis (FEA) solution, here are possible reasons:

1) Internal dimensions of truncated cone may be 1% larger than the assumed internal dimensions in the exact solution analysis (Big Diameter=11 inches, Small Diameter=6.25 inches and Axial Length=9 inches).

2) Flatness of the big base and the small base.  The exact solution assumes spherical section surfaces for the bases while the Finite Element solution assumes them to be perfectly flat. This can be shown to make a small difference, and its sign (increasing or decreasing the frequency) depends on the electromagnetic mode shape.

3) Internal Damping: damping decreases the frequency of the damped solution as compared to the undamped solution.  The exact solution assumes infinite Q (undamped conditions). 

I attach again the predicted heating profile (at the big base) for this mode (TM311) from the exact solution

Thanks for the support in digging up that data from the thread. Now that I've had time to look, I can see those on pages 25 & 26 of Frustum modes overview 2A.pdf graciously provided by Mr. Paul March here:
http://forum.nasaspaceflight.com/index.php?topic=36313.msg1333246#msg1333246

Also note that the Comsol plot on page 25 looks exactly like what you provided.

As to the few mhz difference in the first one, TM311(the TM212 one is very very close), as you alluded to, there are many reasons and hidden variables that go into this.

For example, when messing around with the frustum hooked up to the SNA, it was refreshing to see how I could change the resonant frequencies at will just my applying pressure to the large end (raising the resonant freq) and then it would return to steady state when the large end rebounded. I knew I could do that, but it was neat to see it in action.

The VSWR of TM212 was all around bad, around 5.3 or so. I think I can improve it by shortening the probe. It isn't on my list of things to do unless a reason comes up later for it.

The VSWR of TM311 isn't great, but it'll do. I was able to get it down to 1.4 by really torquing down on the cable but it would go back to ~2.

There's lots of quirks I've discovered, such as just the weight of the test cable applying pressure to the frustum walls slightly changes the measured results.

The data I've provided was reproduced after a tear down and build up of the setup to ensure the same results kept happening.

Another source of minor differences is this: So last night, I tried a test fit of the 6.25"x1" HDPE discs I made. They are as close to exactly 6.25" wide as my eyes can see. So the small end of this frustum is a hare less than 6.25" It is hard to tell exactly, but I think I'm narrow by about 1/32" of an inch. So they don't fit. Looking back at the zoomed in ruler shots, I think I can see it on the "width small end right side ruler" photo:
http://forum.nasaspaceflight.com/index.php?topic=36313.msg1345818#msg1345818

To correct for this, I'm going to have to mill them down by a small amount still to be determined. For now I can't mount them anyway because all the Nylon bolts I have are just not suitable for these big honkin chunks of plastic. Long bolts are on the way.

On the bright side of things, I was able to test the cables, connectors and the isolator I purchased and they all work up to specs. My old power splitter with port isolation still works. The isolator in particular has a VSWR of 1.03 which is good. The forward power loss was .3dB and the reverse loss is 20dB as advertized.

Title: Re: EM Drive Developments - related to space flight applications - Thread 2
Post by: Rodal on 04/01/2015 12:59 am
....

For example, when messing around with the frustum hooked up to the SNA, it was refreshing to see how I could change the resonant frequencies at will just my applying pressure to the large end (raising the resonant freq) and then it would return to steady state when the large end rebounded. I knew I could do that, but it was neat to see it in action.

The VSWR of TM212 was all around bad, around 5.3 or so. I think I can improve it by shortening the probe. It isn't on my list of things to do unless a reason comes up later for it.

The VSWR of TM311 isn't great, but it'll do. I was able to get it down to 1.4 by really torquing down on the cable but it would go back to ~2.

There's lots of quirks I've discovered, such as just the weight of the test cable applying pressure to the frustum walls slightly changes the measured results.

....

@Mulletron, thanks for disclosing that information: as far as I know you are the first one reporting it, as I have not seen Shawer, Juan Yang or NASA Eagleworks disclose the above information.

Could you please provide more information on the movement of the natural frequency: when applying pressure to the large end, roughly  how much did the natural frequency change? roughly from what natural frequency (without outside pressure on the big end) to what natural frequency (by pushing the big end towards the inside)?
Did you push the big end at its center? Roughly speaking how much was the displacement? would you say that it was very small, of the order of the thickness or less than the thickness of the big base plate?

Thanks  :)
Title: Re: EM Drive Developments - related to space flight applications - Thread 2
Post by: Mulletron on 04/01/2015 01:09 am
....

For example, when messing around with the frustum hooked up to the SNA, it was refreshing to see how I could change the resonant frequencies at will just my applying pressure to the large end (raising the resonant freq) and then it would return to steady state when the large end rebounded. I knew I could do that, but it was neat to see it in action.

The VSWR of TM212 was all around bad, around 5.3 or so. I think I can improve it by shortening the probe. It isn't on my list of things to do unless a reason comes up later for it.

The VSWR of TM311 isn't great, but it'll do. I was able to get it down to 1.4 by really torquing down on the cable but it would go back to ~2.

There's lots of quirks I've discovered, such as just the weight of the test cable applying pressure to the frustum walls slightly changes the measured results.

....

@Mulletron, thanks for disclosing that information: as far as I know you are the first one reporting it, as I have not seen Shawer, Juan Yang or NASA Eagleworks disclose the above information.

Could you please provide more information on the movement of the natural frequency: when applying pressure to the large end, roughly  how much did the natural frequency change? roughly from what natural frequency (without outside pressure on the big end) to what natural frequency (by pushing the big end towards the inside)?
Did you push the big end at its center? Roughly speaking how much was the displacement? would you say that it was very small, of the order of the thickness or less than the thickness of the big base plate?

Thanks  :)

Push on your LCD screen hard enough for the color to change, about that much pressure in the center of the large end. It takes deliberate force to do it. Like squeezing a shaken up Coke bottle. The change was very slight. From memory, it was only about a 1-2mhz. The SNA screen updates every 200ms so you can see it in real time. You can see the whole plot shift to the right slightly.

Another weird quirk I remember from the 700-2700mhz sweeps I did several days ago, is that you can see spikes at other frequencies on the spectrum analyzer (attached to sense port of frustum), which are other than the current center frequency being generated by the sweeper. You can't see/notice them happening unless the frequency is sweeping, otherwise they blend in. So I guess there is some mixing going on inside the cavity. Those harmonics weren't produced by the sweeper. I checked. Anyway, the reason I thought that was neat is because I remembered reading this * a few days ago when I posted it here:
http://forum.nasaspaceflight.com/index.php?topic=36313.msg1348074#msg1348074
* https://www.vahala.caltech.edu/Research/Nonlinear

I very specifically remember being spanned on the spec anny from 700 to 2700, and when the sweeper would start over, at around 800mhz sweeper frequency, I'd see harmonics at the other end of the sweep being chased.
During this, you can even see a spike at the end:
https://drive.google.com/folderview?id=0B4PCfHCM1KYoZWphS29nSDZkZVE&amp;usp=sharing&tid=0B4PCfHCM1KYoTXhSUTd5ZDN2WnM

I was sweeping from 800-2600, and spanned from 700-2700, and there are spikes outside my sweep that max hold picked up.

So I guess that means that there are n higher order modes inside these cavities happening too.
Title: Re: EM Drive Developments - related to space flight applications - Thread 2
Post by: RotoSequence on 04/01/2015 05:07 am
If I'm understanding all of these efforts correctly (please correct me if I don't!), in layman's terms, an EM drive is a conductive cavity in which most radio frequencies (RF) can propagate freely, and come out of the other end with a minimal loss of energy. But, at certain frequencies, the cavity will resonate, containing and amplifying the energy of the RF signal. For certain cavity geometries at specific frequencies, the shape (or energy?) of the resonating photons will be pointed in one direction. This directionality seems to be important for making an EM drive work. Conventional wisdom says that the energy should dissipate as heat, but instead, the energy seems to be taking the form of net thrust.
Title: Re: EM Drive Developments - related to space flight applications - Thread 2
Post by: zen-in on 04/01/2015 05:34 am
If I'm understanding all of these efforts correctly (please correct me if I don't!), in layman's terms, an EM drive is a conductive cavity in which most radio frequencies (RF) can propagate freely, and come out of the other end with a minimal loss of energy. But, at certain frequencies, the cavity will resonate, containing and amplifying the energy of the RF signal. For certain cavity geometries at specific frequencies, the shape (or energy?) of the resonating photons will be pointed in one direction. This directionality seems to be important for making an EM drive work. Conventional wisdom says that the energy should dissipate as heat, but instead, the energy seems to be taking the form of net thrust.

If that is an accurate summary of the graphs, simulations and related discussion then I will have to disagree.   A long time ago an RF engineer friend explained to me that cavities, filters and LC circuits are never dissipative.   The power either goes through them or is reflected.   If an RF signal with 20 MHz of bandwidth at 2085 MHz is sent through a 5 pole cavity filter with 5 MHz passband a large fraction of the RF power is simply reflected back to the amplifier.   It is for this reason that isolators (circulators with a 50 Ohm load on one port) are used between the amplifier and a filter.    The green on black graphs shown above, unless I am mistaken are S12 plots.   Most of the power is transmitted through the cavity.   More power is reflected at frequencies where there are dips in the S12 plot.  This is where the reflection coefficient (SWR) is higher.   Inside the cavity the Poynting vector is directed from the input port to the output port.   Outside the cavity the Poynting vector is inside the dielectric of the coax; pointing away from the PA.    An interesting experiment would be to decrease the length of the coax from the PA to the cavity by 2-3 cm.   This will change the position of the dips in the S12 plot.  Any reflected power, or return wave as it's sometimes called, will be dissipated as heat inside the amplifier.
Title: Re: EM Drive Developments - related to space flight applications - Thread 2
Post by: jmossman on 04/01/2015 06:02 am
I very specifically remember being spanned on the spec anny from 700 to 2700, and when the sweeper would start over, at around 800mhz sweeper frequency, I'd see harmonics at the other end of the sweep being chased.
During this, you can even see a spike at the end:
https://drive.google.com/folderview?id=0B4PCfHCM1KYoZWphS29nSDZkZVE&amp;usp=sharing&tid=0B4PCfHCM1KYoTXhSUTd5ZDN2WnM

I was sweeping from 800-2600, and spanned from 700-2700, and there are spikes outside my sweep that max hold picked up.

So I guess that means that there are n higher order modes inside these cavities happening too.

Would anyone care to speculate as to how much effect spherical faces/endcaps for the small and large ends (instead of flat plates) would have on the these observed interactions?
(i.e.  "truncated spherical cone" versus as-tested "conical frustum")
Title: Re: EM Drive Developments - related to space flight applications - Thread 2
Post by: RotoSequence on 04/01/2015 06:08 am
If I'm understanding all of these efforts correctly (please correct me if I don't!), in layman's terms, an EM drive is a conductive cavity in which most radio frequencies (RF) can propagate freely, and come out of the other end with a minimal loss of energy. But, at certain frequencies, the cavity will resonate, containing and amplifying the energy of the RF signal. For certain cavity geometries at specific frequencies, the shape (or energy?) of the resonating photons will be pointed in one direction. This directionality seems to be important for making an EM drive work. Conventional wisdom says that the energy should dissipate as heat, but instead, the energy seems to be taking the form of net thrust.

If that is an accurate summary of the graphs, simulations and related discussion then I will have to disagree.   A long time ago an RF engineer friend explained to me that cavities, filters and LC circuits are never dissipative.   The power either goes through them or is reflected.   If an RF signal with 20 MHz of bandwidth at 2085 MHz is sent through a 5 pole cavity filter with 5 MHz passband a large fraction of the RF power is simply reflected back to the amplifier.   It is for this reason that isolators (circulators with a 50 Ohm load on one port) are used between the amplifier and a filter.    The green on black graphs shown above, unless I am mistaken are S12 plots.   Most of the power is transmitted through the cavity.   More power is reflected at frequencies where there are dips in the S12 plot.  This is where the reflection coefficient (SWR) is higher.   Inside the cavity the Poynting vector is directed from the input port to the output port.   Outside the cavity the Poynting vector is inside the dielectric of the coax; pointing away from the PA.    An interesting experiment would be to decrease the length of the coax from the PA to the cavity by 2-3 cm.   This will change the position of the dips in the S12 plot.  Any reflected power, or return wave as it's sometimes called, will be dissipated as heat inside the amplifier.

I am not an RF engineer, and I must defer to the experts. I have no idea what's happening to the RF power inside of the cavity.
Title: Re: EM Drive Developments - related to space flight applications - Thread 2
Post by: Notsosureofit on 04/01/2015 10:52 am
Real (as opposed to theoretical) tuned circuits always have resistive losses and some times nonlinear effects.
Title: Re: EM Drive Developments - related to space flight applications - Thread 2
Post by: Rodal on 04/01/2015 11:24 am
If I'm understanding all of these efforts correctly (please correct me if I don't!), in layman's terms, an EM drive is a conductive cavity in which most radio frequencies (RF) can propagate freely, and come out of the other end with a minimal loss of energy. But, at certain frequencies, the cavity will resonate, containing and amplifying the energy of the RF signal. For certain cavity geometries at specific frequencies, the shape (or energy?) of the resonating photons will be pointed in one direction. This directionality seems to be important for making an EM drive work. Conventional wisdom says that the energy should dissipate as heat, but instead, the energy seems to be taking the form of net thrust.

If that is an accurate summary of the graphs, simulations and related discussion then I will have to disagree.   A long time ago an RF engineer friend explained to me that cavities, filters and LC circuits are never dissipative.   The power either goes through them or is reflected.  ....

Real (as opposed to theoretical) tuned circuits always have resistive losses and some times nonlinear effects.

"That cavities, filters and LC circuits are never dissipative" is either an over-simplification or a reference to something else: exotic superconductivity (true only for certain materials at very low temperatures).   Even near absolute zero, a real sample of a normal conductor shows some resistance.

One thing is to state that dissipative losses are negligible in comparison to something and another one to flatly state that there is never any dissipation.  :)

Non-negligible dissipated heat from induction of the magnetic field producing eddy currents in the metal has been documented by NASA Eagleworks using an IR thermal camera image of the exterior surface of the big base of the truncated cone, (for mode shape Cyl. TM212.  [Perhaps under a partial vacuum ? -I don't recall-] ). 

(http://forum.nasaspaceflight.com/index.php?action=dlattach;topic=36313.0;attach=635194;image)

(http://forum.nasaspaceflight.com/index.php?action=dlattach;topic=36313.0;attach=635195;image)

Also Prof. Juan Yang's reported temperature vs. time measurements with embedded thermocouples throughout their EM Drive cavity (without a polymer dielectric insert) under atmospheric conditions, that, curiously, show the highest temperature at the center of the small base (trace #1), followed, at a significantly lower temperature by the temperature at the periphery of the big base (trace #5).

(http://forum.nasaspaceflight.com/index.php?action=dlattach;topic=29276.0;attach=622845;image)

(http://forum.nasaspaceflight.com/index.php?action=dlattach;topic=29276.0;attach=655009;image)


Also, tan delta out-of-phase losses in these cavities are not zero: the reported experiments show that the tan delta values of the materials used in these cavities is consistent with real materials experiencing out-of-phase dissipation (therefore one must use the complex form of the physical properties and not neglect the imaginary part if one is interested in assessing the finite value of Q, for example).

Stating that the cavity has no dissipation whatsoever is tantamount to stating that the cavity has an infinite Q, which for purposes of examining the EM Drive would lead to an oversimplification, as it would not allow to assess what is precisely going on at resonance.
Title: Re: EM Drive Developments - related to space flight applications - Thread 2
Post by: zen-in on 04/01/2015 02:00 pm
Real (as opposed to theoretical) tuned circuits always have resistive losses and some times nonlinear effects.

Resistive losses are very small in a high Q filter or cavity.   That is what is meant when it is said that filters, cavities, etc are not dissipative.  Power is either transmitted through a filter or is reflected.  A very small fraction is dissipated due to Ohmic losses.
Title: Re: EM Drive Developments - related to space flight applications - Thread 2
Post by: Mulletron on 04/01/2015 09:33 pm
Built a torsion head. Can adjust for height and twist. *Height adjustment is for safely entering and exiting the Galinstan, and for lining up the center of masses, for the Cavendish experiment.

Twist is for finding/adjusting zero at equilibrium, to zero the laser reflected on screen at theta zero.

Math and methodology for Cavendish:
https://www.youtube.com/watch?v=4JGgYjJhGEE

Coulomb measured charge in similar way:
https://www.youtube.com/watch?v=FYSTGX-F1GM

Starting at 32:30:
https://www.youtube.com/watch?v=wp6BQalV__g

Man, this is turning out to be a very daunting undertaking indeed.

Rest:
https://drive.google.com/folderview?id=0B4PCfHCM1KYoTl90eDBuMklOeTg&usp=sharing&tid=0B4PCfHCM1KYoTXhSUTd5ZDN2WnM

*edit

*An improved clinch knot will be tied to a rigid body which sits in the detent, in the rigid channel I created in the top HDPE part, and will fall through the hollow body (without touching the sides) to the balance and DUT.

(http://www.netknots.com/files/9113/2980/7293/improved-clinch-knot.jpg)
Title: Re: EM Drive Developments - related to space flight applications - Thread 2
Post by: Stormbringer on 04/02/2015 06:03 am
i know this one has a big giggle factor but Dave Pares has updated his website again with more experimental results. It is a species of EM drive if real.

http://www.paresspacewarpresearch.org/Projet_Space_Warp/Experiment_5.htm

Title: Re: EM Drive Developments - related to space flight applications - Thread 2
Post by: ThinkerX on 04/03/2015 12:03 am
Quote
know this one has a big giggle factor but Dave Pares has updated his website again with more experimental results. It is a species of EM drive if real.

http://www.paresspacewarpresearch.org/Projet_Space_Warp/Experiment_5.htm

hmmm...worth including with the other experimental EM Drive projects or not?

Maybe this guys efforts warrant a brief look by Notsosureofit or Doctor Rodal?
Title: Re: EM Drive Developments - related to space flight applications - Thread 2
Post by: Notsosureofit on 04/03/2015 12:44 am
Looks atmospheric so far but only time will tell ....
Title: Re: EM Drive Developments - related to space flight applications - Thread 2
Post by: zen-in on 04/03/2015 03:59 am
i know this one has a big giggle factor but Dave Pares has updated his website again with more experimental results. It is a species of EM drive if real.

http://www.paresspacewarpresearch.org/Projet_Space_Warp/Experiment_5.htm

Whatever he's doing it looks original.   The PVC pipes with wires through them may be the solution.
Title: Re: EM Drive Developments - related to space flight applications - Thread 2
Post by: Rodal on 04/03/2015 05:29 pm
BIG CORRECTION

It has bothered me that, if the  Poynting vector  (http://en.wikipedia.org/wiki/Poynting_vector)(ExB) would be a quadratic function of the harmonic function so that it never changes sign and therefore does not change orientation with time, even for an AC field, as for example discussed in http://en.wikipedia.org/wiki/Poynting_vector#Time-averaged_Poynting_vector, then the operation of the EM Drive would not be a subject of so much controversy.

For the Poynting vector to vary like (Cos[ ω t + phaseAngle])^2 one needs that the E and B fields to be in phase with each other, as shown in the following image for example:

(http://www.blazelabs.com/pics/empropagation2.gif)

But then it dawned on me, that the E and B fields cannot be in-phase with each other (therefore the above image is only true for a travelling wave and it is inappropriate for an EM Drive cavity which instead has standing waves), because Maxwell's equation states that they must 90 degrees out of phase with each other:


One of Maxwell's equations (http://en.wikipedia.org/wiki/Faraday%27s_law_of_induction#Maxwell.E2.80.93Faraday_equation) states that:

Curl E = - d B / dt



so, for example if the magnetic field B varies as Cos[ ω t], then the electric field must vary as its time derivative:
- d(Cos[ ω t])/dt =  ω Sin[ ω t] , and therefore the Poynting vector ExB should vary as
Sin[ ω t] Cos[ ω t]  = Sin[ 2 ω t] /2, which oscillates at twice the frequency of the electromagnetic fields and has a time average value of zero.

Since the Poynting vector has a time average of zero, there cannot be any net energy flow out of the EM Drive.

This is due to the fact that the waves inside the EM Drive are standing waves.  Therefore the Poynting vector is just describing how energy is transferred between the electric and magnetic fields.

Also this means that there cannot be momentum outflow either, due to the Poynting vector, if the electromagnetic fields are harmonic functions of time.

Imagine, for discussion's sake, that it could indeed be possible that virtual electron-positron pairs would materialize out of the Quantum Vacuum, and that when such a pair materializes the Poynting vector is pointing towards the big base of the truncated cone EM Drive.  Then the electron-positron pair would be transported by the Poynting vector field towards the big base of the truncated cone, and shortly during that transport the electron-positron would cease to exist, returning back to the vacuum.  Then (as shown by Einstein himself in a though-model he proposed a long time ago concerning light particles being transported within a friction-less railroad car) the truncated cone would experience a recoil -simultaneous with the transport of the electron-positron pair-, which would result in a net force towards the small base of the truncated cone.  If the Poynting vector would always be pointing towards the big base, this would function as proposed by Dr. White.

Unfortunately, the standing waves within an EM Drive cavity are such that the E and B  fields must be 90 degrees out of phase with each other (due to Maxwell's equations), and this dictates that the Poynting vector is changing direction at a frequency twice as high as the frequency of the electromagnetic fields.  Therefore, if electron-positron pairs would materialize such as in the thought-model discussed above resulting in a recoil of the EM Drive towards the small base, it would occur just as often that electron-positron pairs would be transported in the completely opposite direction and the EM Drive would experience a force in the opposite direction.  Therefore what would be expected (out of the Quantum Vacuum model) is to have forces in the EM Drive pointing towards the small base just as often as having forces pointed in the opposite direction towards the big base, and this would result in no net transport of the EM Drive over a period of such oscillations.

I will need to correct some of my previous postings concerning the Poynting vector for the EM Drive: for a cavity like the EM Drive, the Poynting vector oscillates with time as  Sin[ 2 ω t] /2: therefore the time average of the Poynting vector must be zero.   

Title: Re: EM Drive Developments - related to space flight applications - Thread 2
Post by: aero on 04/03/2015 06:57 pm
That's all good, (I think) but how does the big end vs. small end affect the strength of the Poynting vector  and hence magnitude of the force? Likely not at all on average? But the strength varies continuously so for small dx in positive direction it increases in strength but for same small dx in negative direction it decreases in strength. (Not sure I have the signs right.) But momentum is not the same even for the same electron/positron pair.

I am postulating that the strength of the fields in the axial direction is related to the geometry of the cavity in the axial direction. Or perhaps even the mode of the resonance. It seems unlikely that the strength is constant from one end to the other.
Title: Re: EM Drive Developments - related to space flight applications - Thread 2
Post by: Rodal on 04/03/2015 07:08 pm
That's all good, (I think) but how does the big end vs. small end affect the strength of the Poynting vector  and hence magnitude of the force? Likely not at all on average? But the strength varies continuously so for small dx in positive direction it increases in strength but for same small dx in negative direction it decreases in strength. (Not sure I have the signs right.) But momentum is not the same even for the same electron/positron pair.

I am postulating that the strength of the fields in the axial direction is related to the geometry of the cavity in the axial direction. Or perhaps even the mode of the resonance. It seems unlikely that the strength is constant from one end to the other.
If there are classical harmonic standing waves (resulting in classical resonance due to Maxwell's equations leading to a high Q) then the Poynting vector varies like Sin[ 2 ω t] /2 at twice the frequency ω of the electromagnetic fields and will average zero over a complete period of time (or multiple periods of time).

If you are postulating non-harmonic non-standing waves, we need you to  formally state and post your non-classical equations, to be able to discuss them.
Title: Re: EM Drive Developments - related to space flight applications - Thread 2
Post by: Rodal on 04/03/2015 08:04 pm
That the electric and magnetic fields must be out of phase by 90 degrees, and therefore the Poynting vector oscillates at twice their frequency and averages zero over a period, really follows from Maxwell-Faraday's law.

http://en.wikipedia.org/wiki/Faraday%27s_law_of_induction#Faraday.27s_law_and_relativity

" Although Faraday's law does not apply to all situations, the Maxwell–Faraday equation and Lorentz force law are always correct and can always be used directly"  (Richard Phillips Feynman, Leighton R B & Sands M L (2006). The Feynman Lectures on Physics. San Francisco: Pearson/Addison-Wesley. Vol. II, pp. 17-2. ISBN 0-8053-9049-9.)
Title: Re: EM Drive Developments - related to space flight applications - Thread 2
Post by: Rodal on 04/03/2015 08:33 pm
I found the following paper  http://www-ssc.igpp.ucla.edu/personnel/russell/papers/skip_ed/node4.html, ( http://onlinelibrary.wiley.com/doi/10.1029/98JA02101/pdf  ) for example, that confirms the simple result I discussed above, for standing waves:

Quote
First, we may obtain some information from the simultaneous Poynting vectors as shown in Figures 5 and 6. If we consider a transverse wave causing field line oscillations, the Poynting vectors behave very differently depending on whether the wave is traveling or standing. Figure 9 is a diagram of the Poynting vectors for the two different schemes. Even though the wave amplitudes for both conditions are set to be the same and the magnitude of Poynting vector oscillations is consequently the same, the traveling wave propagates energy, while the standing wave produces no net energy flux. The Poynting vectors in Figures 5 and 6 more resemble the traveling wave pattern. Thus for the Pc3-4 wave activities in our observations the traveling wave component is stronger. We may also estimate the resonant condition by examining the phase difference between dE and dB [e.g., Singer et al., 1982]. If the phase difference is 90, the wave is standing and a resonant condition is reached.


   TRAVELLING WAVE                                                   STANDING WAVE (EM Drive)

  Poynting Vector time average is (+1/2)                       Poynting Vector time average is zero

  (Cos[ ω t])^2  =( 1+Cos[ 2 ω t]) /2                          Sin[ ω t] Cos[ ω t]  = Sin[ 2 ω t] /2

(http://www-ssc.igpp.ucla.edu/personnel/russell/papers/skip_ed/fig9.gif)

So it is as simply as this: to transfer energy or momentum from virtual particles in the Quantum Vacuum, as proposed by Dr. White, a traveling wave would be needed, but then one would have no resonance, and no Q.

If one has a cavity EM Drive then resonance can take place, and hence a high Q, but that precludes the possibility of transferring energy or momentum, according to Maxwell's equations.
Title: Re: EM Drive Developments - related to space flight applications - Thread 2
Post by: Mulletron on 04/03/2015 09:55 pm
I had the lightweight spreader bar part of the balance flying overnight on the Spiderwire reviewed below. I was surprised that the bar did find stable equilibrium on the very thin .011" diameter line. I figured it would sit there and oscillate all over the place.

I got started on the laser screen/camera setup.

Spent the day attaching hardware and stress testing the balance to make sure it will be safe to fly the frustum. The all up weight of the frustum +balance and everything else required for it to run is ~7lbs.

I have all this stuff weighed down to the gram btw but I haven't posted the details of it yet. I'm writing the weights of all the items on each item with a sharpie in grams with the precision of .1 gram.

I was shooting for a consistent safety factor of at least 80% of breaking strength to ensure no accidents. I used Spiderwire .011" 10lb test monofilament. This stuff broke at >10lbs all up weight two times, which is good. It does stretch quite a bit. I expected that and it really isn't a problem.

But I had two breaks at ~8 pounds. One break was with a Improved Clinch Knot, the other with the Palomar knot. It can certainly hold up the experiment, but that is too close for comfort for me, so it is out. That inconsistency in the break has me worried to use it. Honestly now that I know how much weight I'm trying to fly, I need to up the tensile strength anyway.

So I spent the day hanging and then breaking my balance.

I don't want to ride the razor's edge of possibly having a dented frustum, so I'm going overkill on tensile strength while maintaining the same diameter.

I had one slip (too many) with the Improved Clinch Knot, so I'm going to go with the double Palomar, which is a must for this new line I'm getting, and it's good that this line has it's own dedicated tying method:
http://www.netknots.com/fishing_knots/nanofil-knot

I have some Berkley Nanofil .01" 17lb test on the way.
http://www.amazon.com/Berkley-Nanofil-Uni-Filament-010-Inch-Diameter/dp/B009UCPF2C
This is about as good as it gets without going to a thicker line or a braid. I don't want to resort to a braid because I don't think a braided line is suitable for a torsion experiment.

I'm trying to use very thin line for now until I get proof that there is a force being generated or not. I only have 2 watts of power to play with. What happens next will inform whether a thicker line or a metallic wire is warranted.

Well anyway, this is a minor setback. I'm still going to fly the thing using some very strong braided Nylon line (yellow stuff in the photo coming from the spreader bar) in place of the mono torsion line, for now. If it moves (which it seems crazy that it would) there won't be any reliable force measurements.

I hooked up all the electronics and the amp to the frustum and let it run overnight and nothing failed or burst into flames, so that's good. Sadly it didn't hover or even wiggle.

https://drive.google.com/folderview?id=0B4PCfHCM1KYoTl90eDBuMklOeTg&amp;usp=sharing&tid=0B4PCfHCM1KYoTXhSUTd5ZDN2WnM
Title: Re: EM Drive Developments - related to space flight applications - Thread 2
Post by: Rodal on 04/03/2015 11:32 pm
....
I hooked up all the electronics and the amp to the frustum and let it run overnight and nothing failed or burst into flames, so that's good. Sadly it didn't hover or even wiggle.

https://drive.google.com/folderview?id=0B4PCfHCM1KYoTl90eDBuMklOeTg&amp;usp=sharing&tid=0B4PCfHCM1KYoTXhSUTd5ZDN2WnM
All the above refers to the truncated cone without the HDPE polymer dielectric segment inside it, correct?
Title: Re: EM Drive Developments - related to space flight applications - Thread 2
Post by: Mulletron on 04/03/2015 11:42 pm
....
I hooked up all the electronics and the amp to the frustum and let it run overnight and nothing failed or burst into flames, so that's good. Sadly it didn't hover or even wiggle.

https://drive.google.com/folderview?id=0B4PCfHCM1KYoTl90eDBuMklOeTg&amp;usp=sharing&tid=0B4PCfHCM1KYoTXhSUTd5ZDN2WnM
All the above refers to the truncated cone without the HDPE polymer dielectric segment inside it, correct?

Yep. Empty. It wasn't on the balance. Just sitting on the floor.
Title: Re: EM Drive Developments - related to space flight applications - Thread 2
Post by: RotoSequence on 04/04/2015 12:33 am
....
I hooked up all the electronics and the amp to the frustum and let it run overnight and nothing failed or burst into flames, so that's good. Sadly it didn't hover or even wiggle.

https://drive.google.com/folderview?id=0B4PCfHCM1KYoTl90eDBuMklOeTg&amp;usp=sharing&tid=0B4PCfHCM1KYoTXhSUTd5ZDN2WnM
All the above refers to the truncated cone without the HDPE polymer dielectric segment inside it, correct?

Yep. Empty. It wasn't on the balance. Just sitting on the floor.

The entire frustum was on the floor, or just the HDPE disk? Sorry, I'm a little confused about what test you conducted.  :-[
Title: Re: EM Drive Developments - related to space flight applications - Thread 2
Post by: Mulletron on 04/04/2015 12:44 am
....
I hooked up all the electronics and the amp to the frustum and let it run overnight and nothing failed or burst into flames, so that's good. Sadly it didn't hover or even wiggle.

https://drive.google.com/folderview?id=0B4PCfHCM1KYoTl90eDBuMklOeTg&amp;usp=sharing&tid=0B4PCfHCM1KYoTXhSUTd5ZDN2WnM
All the above refers to the truncated cone without the HDPE polymer dielectric segment inside it, correct?

Yep. Empty. It wasn't on the balance. Just sitting on the floor.

The entire frustum was on the floor, or just the HDPE disk? Sorry, I'm a little confused about what test you conducted.  :-[

The HDPE disc wasn't part of this. All the test was just hooking everything up to make sure it didn't overheat or fail. Just turned it all on for the first time.
Title: Re: EM Drive Developments - related to space flight applications - Thread 2
Post by: mlindner on 04/04/2015 01:08 am
It appears my previous post got deleted.

So I'll ask again. Has this psudoscience nonsense been found to be false yet? Have any proper scientists tested this contraption so that they can show that it doesn't work?
Title: Re: EM Drive Developments - related to space flight applications - Thread 2
Post by: Notsosureofit on 04/04/2015 01:12 am
It appears my previous post got deleted.

So I'll ask again. Has this psudoscience nonsense been found to be false yet? Have any proper scientists tested this contraption so that they can show that it doesn't work?

There has as yet been no nullifying experiment.  We keep at it though !
Title: Re: EM Drive Developments - related to space flight applications - Thread 2
Post by: QuantumG on 04/04/2015 01:15 am
Have any proper scientists tested this contraption so that they can show that it doesn't work?

The real scientists are too busy doing grade school experiments (http://laughingsquid.com/a-feather-and-a-bowling-ball-dropped-together-inside-the-worlds-largest-vacuum-chamber/) to test something as unimportant as this.  :P
Title: Re: EM Drive Developments - related to space flight applications - Thread 2
Post by: jmossman on 04/04/2015 01:42 am
I found the following paper  http://www-ssc.igpp.ucla.edu/personnel/russell/papers/skip_ed/node4.html, ( http://onlinelibrary.wiley.com/doi/10.1029/98JA02101/pdf  ) for example, that confirms the simple result I discussed above, for standing waves:

Quote
First, we may obtain some information from the simultaneous Poynting vectors as shown in Figures 5 and 6. If we consider a transverse wave causing field line oscillations, the Poynting vectors behave very differently depending on whether the wave is traveling or standing. Figure 9 is a diagram of the Poynting vectors for the two different schemes. Even though the wave amplitudes for both conditions are set to be the same and the magnitude of Poynting vector oscillations is consequently the same, the traveling wave propagates energy, while the standing wave produces no net energy flux. The Poynting vectors in Figures 5 and 6 more resemble the traveling wave pattern. Thus for the Pc3-4 wave activities in our observations the traveling wave component is stronger. We may also estimate the resonant condition by examining the phase difference between dE and dB [e.g., Singer et al., 1982]. If the phase difference is 90, the wave is standing and a resonant condition is reached.


   TRAVELLING WAVE                                                   STANDING WAVE (EM Drive)

  Poynting Vector time average is (+1/2)                       Poynting Vector time average is zero

  (Cos[ ω t])^2  =( 1+Cos[ 2 ω t]) /2                          Sin[ ω t] Cos[ ω t]  = Sin[ 2 ω t] /2

(http://www-ssc.igpp.ucla.edu/personnel/russell/papers/skip_ed/fig9.gif)

So it is as simply as this: to transfer energy or momentum from virtual particles in the Quantum Vacuum, as proposed by Dr. White, a traveling wave would be needed, but then one would have no resonance, and no Q.

If one has a cavity EM Drive then resonance can take place, and hence a high Q, but that precludes the possibility of transferring energy or momentum, according to Maxwell's equations.

Dr. Rodal, I am not an RF engineer and haven't performed math with Maxwell's equations many years.  My following thoughts may have completely overlooked a fundamental issue, so please take them with the appropriate degree of skeptism.  :)

Doesn't the above description (regarding the mathematical basis for the standing wave, and resulting zero time-average Poynting vector) require an ideal resonator with no losses?

http://www.physicsclassroom.com/class/waves/Lesson-4/Traveling-Waves-vs-Standing-Waves
Quote from: web link=http://www.physicsclassroom.com/class/waves/Lesson-4/Traveling-Waves-vs-Standing-Waves
When the proper frequency is used, the interference of the incident wave and the reflected wave occur in such a manner that there are specific points along the medium that appear to be standing still. Because the observed wave pattern is characterized by points that appear to be standing still, the pattern is often called a standing wave pattern...  These points vibrate back and forth from a positive displacement to a negative displacement; the vibrations occur at regular time intervals such that the motion of the medium is regular and repeating.

In the excited modes demonstrated by the IR thermal imagery, the frustum doesn't appear to be behaving like an ideal resonator cavity.  I would suspect that the time average reflected power from the "hot" end of the frustum is lower than the average power of the incident wave.  Doesn't that imply that the average energy of the Dr. White's QV collisions from the reflected wave will be lower than the average the energy of the QV collisions from the incident wave?  While the delta in power between incident and reflected might be close enough for many to consider EM drive as a "standing wave" within the frustum resonator, wouldn't a non-zero average be more accurate?  (i.e. a very low power traveling wave?)

Phrased a little differently, doesn't the frustum "hot" end look more like a "resistive" [1] load, thereby invalidating the simplistic ideal resonator and allowing for a non-zero time average Poynting vector?

[1]  a better word than "resistive" escapes me;  the induced currents are "draining" energy from the cavity and acting as a load

Best regards,
James
Title: Re: EM Drive Developments - related to space flight applications - Thread 2
Post by: aero on 04/04/2015 01:46 am
I found the following paper  http://www-ssc.igpp.ucla.edu/personnel/russell/papers/skip_ed/node4.html, ( http://onlinelibrary.wiley.com/doi/10.1029/98JA02101/pdf  ) for example, that confirms the simple result I discussed above, for standing waves:

Quote
First, we may obtain some information from the simultaneous Poynting vectors as shown in Figures 5 and 6. If we consider a transverse wave causing field line oscillations, the Poynting vectors behave very differently depending on whether the wave is traveling or standing. Figure 9 is a diagram of the Poynting vectors for the two different schemes. Even though the wave amplitudes for both conditions are set to be the same and the magnitude of Poynting vector oscillations is consequently the same, the traveling wave propagates energy, while the standing wave produces no net energy flux. The Poynting vectors in Figures 5 and 6 more resemble the traveling wave pattern. Thus for the Pc3-4 wave activities in our observations the traveling wave component is stronger. We may also estimate the resonant condition by examining the phase difference between dE and dB [e.g., Singer et al., 1982]. If the phase difference is 90, the wave is standing and a resonant condition is reached.


   TRAVELLING WAVE                                                   STANDING WAVE (EM Drive)

  Poynting Vector time average is (+1/2)                       Poynting Vector time average is zero

  (Cos[ ω t])^2  =( 1+Cos[ 2 ω t]) /2                          Sin[ ω t] Cos[ ω t]  = Sin[ 2 ω t] /2

(http://www-ssc.igpp.ucla.edu/personnel/russell/papers/skip_ed/fig9.gif)

So it is as simply as this: to transfer energy or momentum from virtual particles in the Quantum Vacuum, as proposed by Dr. White, a traveling wave would be needed, but then one would have no resonance, and no Q.

If one has a cavity EM Drive then resonance can take place, and hence a high Q, but that precludes the possibility of transferring energy or momentum, according to Maxwell's equations.

I'm not sure that I am in 100% agreement with your result but my computational results do show that any force created without a QV model is far to small to be considered the answer to the cause of the measured experimental thrusts.

My current thoughts still hinge on the evanescent waves through the gaps in the structure as being key to the operation of this device. That is, evanescent waves are unidirectional, they do not average to zero but do collapse to zero at some distance from the cavity. This distance is considerably greater than one third or even one full wavelength.

My current thought is that perhaps, as the evanescent waves pass through the gaps, they apply force to the virtual particles in one direction only and then collapse as we know they do. However, the virtual particles have already dissipated back into the QV so only the momentum of the cavity remains.

I hope to run some cases testing this in the near future but a Mathematica model would be much more desirable, and telling, than a Meep test result. Fortunately, Meep does provide a QV model in 2-D - it was prepared in order to measure Casimir forces but should be useful for our situation, too. Unfortunately, only 1-D and 2-D models were implemented. Once I figure out how to run the model, and what it means, I will be ready to attempt some results.

I would be very happy to see you and Mathematica get the jump on me and provide some estimates before I can do so.
Title: Re: EM Drive Developments - related to space flight applications - Thread 2
Post by: ThinkerX on 04/04/2015 01:47 am
So...then, how is this device generating thrust if the laws of thermodynamics rule out the electromagnetic approach?

Plus, how is it that the predictions from the now invalid approach are...at least in the same ballpark as the reported experimental results? 

Aero's suggestion might be the only way to salvage this. 
Title: Re: EM Drive Developments - related to space flight applications - Thread 2
Post by: Mulletron on 04/04/2015 02:01 am
It appears my previous post got deleted.

So I'll ask again. Has this psudoscience nonsense been found to be false yet? Have any proper scientists tested this contraption so that they can show that it doesn't work?

I'm sure some "proper scientists" will come round for a look eventually. In the meantime, history has a lot to teach us about these sorts of things:

http://amasci.com/weird/vindac.html
http://www.lifehack.org/articles/lifestyle/6-world-changing-ideas-that-were-originally-rejected.html
http://www.cracked.com/article_18822_5-famous-scientists-dismissed-as-morons-in-their-time.html

Quote
...so that they can show that it doesn't work

Just curious, what makes you so certain that it doesn't work? I mean, I have no idea if it works or not. The only certain thing I know here is that I don't know for certain if it works or doesn't.

There is a growing body of evidence which suggest that it does work. It hasn't been proven by anybody that it doesn't work.


Title: Re: EM Drive Developments - related to space flight applications - Thread 2
Post by: Rodal on 04/04/2015 03:11 am
...
Dr. Rodal, I am not an RF engineer and haven't performed math with Maxwell's equations many years.  My following thoughts may have completely overlooked a fundamental issue, so please take them with the appropriate degree of skeptism.  :)

Doesn't the above description (regarding the mathematical basis for the standing wave, and resulting zero time-average Poynting vector) require an ideal resonator with no losses?

...
Hi James,

No, there is no requirement to have an ideal resonator with no losses, meaning an infinite Q.

The Finite Element Analysis (FEA) using COMSOL performed by NASA took into account tan delta losses to compute a finite Q.  Still, COMSOL FEA solves the standard Maxwell equations, and it contains no esoteric physics whatsoever.

One can also consider a non-ideal resonator using an exact solution.  One must use complex variables.  The losses responsible for a finite Q are due to the imaginary part, which is responsible for the material property "loss tangent" or "tan delta". 

See for example:  http://web.mit.edu/22.09/ClassHandouts/Charged%20Particle%20Accel/CHAP12.PDF

Maxwell's equation mandating that the Curl of the electric field E must equal the negative of the time derivative of the magnetic field B still must be satisfied.  See Equation 12.34 in the above link.



The electric permittivity describes the interaction of a material with an electric field E and is a complex quantity.

The real part of permittivity is a measure of how much energy from an external electric field is stored in a material. The imaginary part of permittivity is called the loss factor and is a measure of how dissipative or lossy a material is to an external electric field. The imaginary part of permittivity is always greater than zero and is usually much smaller than the real part. The loss factor includes the effects of both dielectric loss and conductivity.

Similarly, real materials have a magnetic susceptibility which is a complex quantity.   The complex permeability consists of a real part (µ') that represents the energy storage term and an imaginary part (µ'') that represents the energy loss term.

An analysis taking into account the complex (real and imaginary parts) of these physical properties does not change Maxwell's equations.
Title: Re: EM Drive Developments - related to space flight applications - Thread 2
Post by: jmossman on 04/04/2015 03:53 am
...
Dr. Rodal, I am not an RF engineer and haven't performed math with Maxwell's equations many years.  My following thoughts may have completely overlooked a fundamental issue, so please take them with the appropriate degree of skeptism.  :)

Doesn't the above description (regarding the mathematical basis for the standing wave, and resulting zero time-average Poynting vector) require an ideal resonator with no losses?

...
Hi James,

No, there is no requirement to have an ideal resonator with no losses, meaning an infinite Q.

The Finite Element Analysis (FEA) using COMSOL performed by NASA took into account tan delta losses to compute a finite Q.  Still, COMSOL FEA solves the standard Maxwell equations, and it contains no esoteric physics whatsoever.

One can also consider a non-ideal resonator using an exact solution.  One must use complex variables.  The losses responsible for a finite Q are due to the imaginary part, which is responsible for the material property "loss tangent" or "tan delta". 

See for example:  http://web.mit.edu/22.09/ClassHandouts/Charged%20Particle%20Accel/CHAP12.PDF

Maxwell's equation mandating that the Curl of the electric field E must equal the negative of the time derivative of the magnetic field B still must be satisfied.  See Equation 12.34 in the above link.



The electric permittivity describes the interaction of a material with an electric field E and is a complex quantity.

The real part of permittivity is a measure of how much energy from an external electric field is stored in a material. The imaginary part of permittivity is called the loss factor and is a measure of how dissipative or lossy a material is to an external electric field. The imaginary part of permittivity is always greater than zero and is usually much smaller than the real part. The loss factor includes the effects of both dielectric loss and conductivity.

Similarly, real materials have a magnetic susceptibility which is a complex quantity. 

An analysis taking into account the complex (real and imaginary parts) of these physical properties does not change Maxwell's equations.

Which part of Maxwell's equations account for the observed thermal losses due to the induced currents in the frustum's large base?  Thermal losses are coming from the field within the cavity, so I'm not sure how the idealized concept of a standing wave (i.e. perfect reflection at the cavity boundary) can be satisfied and still account for the thermal energy loss.

I'm not trying to challenge Maxwell's equations, but I don't see the entire system being modeled.  Once the losses in the frustum are accounted for, my (perhaps overly simplistic) mental math suggests that the energy of the reflected wave won't be exactly equal to the energy of the incident wave.  Doesn't every reflection reduce the total energy, with the maximum energy reduction occurring on the face with the largest induced currents (which also generate thermal heat losses)?  Over time wouldn't this incident-vs-reflected energy delta accumulate and erode the ideal assumption behind the perfect standing wave?  (and therefore provide a mathematical explanation for a small non-zero time average Poynting vector?)

Regards,
James
Title: Re: EM Drive Developments - related to space flight applications - Thread 2
Post by: Rodal on 04/04/2015 04:16 am
...
Dr. Rodal, I am not an RF engineer and haven't performed math with Maxwell's equations many years.  My following thoughts may have completely overlooked a fundamental issue, so please take them with the appropriate degree of skeptism.  :)

Doesn't the above description (regarding the mathematical basis for the standing wave, and resulting zero time-average Poynting vector) require an ideal resonator with no losses?

...
Hi James,

No, there is no requirement to have an ideal resonator with no losses, meaning an infinite Q.

The Finite Element Analysis (FEA) using COMSOL performed by NASA took into account tan delta losses to compute a finite Q.  Still, COMSOL FEA solves the standard Maxwell equations, and it contains no esoteric physics whatsoever.

One can also consider a non-ideal resonator using an exact solution.  One must use complex variables.  The losses responsible for a finite Q are due to the imaginary part, which is responsible for the material property "loss tangent" or "tan delta". 

See for example:  http://web.mit.edu/22.09/ClassHandouts/Charged%20Particle%20Accel/CHAP12.PDF

Maxwell's equation mandating that the Curl of the electric field E must equal the negative of the time derivative of the magnetic field B still must be satisfied.  See Equation 12.34 in the above link.



The electric permittivity describes the interaction of a material with an electric field E and is a complex quantity.

The real part of permittivity is a measure of how much energy from an external electric field is stored in a material. The imaginary part of permittivity is called the loss factor and is a measure of how dissipative or lossy a material is to an external electric field. The imaginary part of permittivity is always greater than zero and is usually much smaller than the real part. The loss factor includes the effects of both dielectric loss and conductivity.

Similarly, real materials have a magnetic susceptibility which is a complex quantity. 

An analysis taking into account the complex (real and imaginary parts) of these physical properties does not change Maxwell's equations.

Which part of Maxwell's equations account for the observed thermal losses due to the induced currents in the frustum's large base?
In order to solve differential equations one must provide boundary conditions.  The thermal losses due to the induced eddy-currects in the copper are a result of the imaginary part of the material properties appearing in the solution of the boundary conditions to solve Maxwell's differential equations.

That is how COMSOL's Finite Element Analysis provided the solution for the thermal losses and the predicted temperature for NASA's EM Drive truncated cone.

The frequencies obtained by COMSOL's FEA were obtained by solving an eigenvalue problem.

Damping is responsible for the finite amplitude of the response, but the presence of damping does not preclude standing waves, waves which have fixed nodes and anti-nodes.  The boundary conditions due to the end plates do not disappear due to heat production.  For example, one of the boundary conditions is that the tangent electric field must be continuous at the material interface.  This boundary condition (and therefore the node produced in that component of the electric field) does not cease to exist due to heat generation.  That only affects the amplitude.  The only way to remove that boundary condition would be to remove the end plate, and if you would do that, the EM Drive would no longer be a completely enclosed cavity.
Title: Re: EM Drive Developments - related to space flight applications - Thread 2
Post by: Mulletron on 04/04/2015 04:44 am
BIG CORRECTION

It has bothered me that, if the  Poynting vector  (http://en.wikipedia.org/wiki/Poynting_vector)(ExB) would be a quadratic function of the harmonic function so that it never changes sign and therefore does not change orientation with time, even for an AC field, as for example discussed in http://en.wikipedia.org/wiki/Poynting_vector#Time-averaged_Poynting_vector, then the operation of the EM Drive would not be a subject of so much controversy.

For the Poynting vector to vary like (Cos[ ω t + phaseAngle])^2 one needs that the E and B fields to be in phase with each other, as shown in the following image for example:

(http://www.blazelabs.com/pics/empropagation2.gif)

But then it dawned on me, that the E and B fields cannot be in-phase with each other (therefore the above image is only true for a travelling wave and it is inappropriate for an EM Drive cavity which instead has standing waves), because Maxwell's equation states that they must 90 degrees out of phase with each other:


One of Maxwell's equations (http://en.wikipedia.org/wiki/Faraday%27s_law_of_induction#Maxwell.E2.80.93Faraday_equation) states that:

Curl E = - d B / dt



so, for example if the magnetic field B varies as Cos[ ω t], then the electric field must vary as its time derivative:
- d(Cos[ ω t])/dt =  ω Sin[ ω t] , and therefore the Poynting vector ExB should vary as
Sin[ ω t] Cos[ ω t]  = Sin[ 2 ω t] /2, which oscillates at twice the frequency of the electromagnetic fields and has a time average value of zero.

Since the Poynting vector has a time average of zero, there cannot be any net energy flow out of the EM Drive.

This is due to the fact that the waves inside the EM Drive are standing waves.  Therefore the Poynting vector is just describing how energy is transferred between the electric and magnetic fields.

Also this means that there cannot be momentum outflow either, due to the Poynting vector, if the electromagnetic fields are harmonic functions of time.

Imagine, for discussion's sake, that it could indeed be possible that virtual electron-positron pairs would materialize out of the Quantum Vacuum, and that when such a pair materializes the Poynting vector is pointing towards the big base of the truncated cone EM Drive.  Then the electron-positron pair would be transported by the Poynting vector field towards the big base of the truncated cone, and shortly during that transport the electron-positron would cease to exist, returning back to the vacuum.  Then (as shown by Einstein himself in a though-model he proposed a long time ago concerning light particles being transported within a friction-less railroad car) the truncated cone would experience a recoil -simultaneous with the transport of the electron-positron pair-, which would result in a net force towards the small base of the truncated cone.  If the Poynting vector would always be pointing towards the big base, this would function as proposed by Dr. White.

Unfortunately, the standing waves within an EM Drive cavity are such that the E and B  fields must be 90 degrees out of phase with each other (due to Maxwell's equations), and this dictates that the Poynting vector is changing direction at a frequency twice as high as the frequency of the electromagnetic fields.  Therefore, if electron-positron pairs would materialize such as in the thought-model discussed above resulting in a recoil of the EM Drive towards the small base, it would occur just as often that electron-positron pairs would be transported in the completely opposite direction and the EM Drive would experience a force in the opposite direction.  Therefore what would be expected (out of the Quantum Vacuum model) is to have forces in the EM Drive pointing towards the small base just as often as having forces pointed in the opposite direction towards the big base, and this would result in no net transport of the EM Drive over a period of such oscillations.

I will need to correct some of my previous postings concerning the Poynting vector for the EM Drive: for a cavity like the EM Drive, the Poynting vector oscillates with time as  Sin[ 2 ω t] /2: therefore the time average of the Poynting vector must be zero.   

This article in Wikipedia does not apply to a resonating cavity like the EM Drive:

http://en.wikipedia.org/wiki/Poynting_vector#Time-averaged_Poynting_vector

because it does not obey Maxwell's equation  Curl E = - d B / dt which must be obeyed for a resonating cavity.

So now that you're officially abandoning reliance on the Poynting vector, what do you think of those other ideas like spectral non-reciprocity due to broken symmetry? That's the reason why I asked your opinion of how you thought the magnetic fields might play into this here:
http://forum.nasaspaceflight.com/index.php?topic=36313.msg1352873#msg1352873
http://forum.nasaspaceflight.com/index.php?topic=36313.msg1352987#msg1352987

It's been shown over and over again that the electric and magnetic fields time average to zero, and now the Poynting vector too (thanks :) ), so it is clear that we're witnessing "other than usual" symmetry conditions at work.


Title: Re: EM Drive Developments - related to space flight applications - Thread 2
Post by: Rodal on 04/04/2015 04:58 am
...

My current thoughts still hinge on the evanescent waves through the gaps in the structure as being key to the operation of this device. That is, evanescent waves are unidirectional, they do not average to zero but do collapse to zero at some distance from the cavity. This distance is considerably greater than one third or even one full wavelength.

My current thought is that perhaps, as the evanescent waves pass through the gaps, they apply force to the virtual particles in one direction only and then collapse as we know they do. However, the virtual particles have already dissipated back into the QV so only the momentum of the cavity remains.

I hope to run some cases testing this in the near future but a Mathematica model would be much more desirable, and telling, than a Meep test result. Fortunately, Meep does provide a QV model in 2-D - it was prepared in order to measure Casimir forces but should be useful for our situation, too. Unfortunately, only 1-D and 2-D models were implemented. Once I figure out how to run the model, and what it means, I will be ready to attempt some results.



To have confidence on numerical results one should start by comparing the results with known solutions for (at least) simpler cases.  (Particularly for a numerical method, like the Finite Difference Method used in MEEP, that as we have discussed before one can not be assured to converge to a solution? )

 For example, I compared my exact solutions, and they were within 1% or so of COMSOL FEA results for NASA's truncated cone and their experimental results.

From what I recall, your MEEP calculations were very far away from the exact solutions for the cylindrical cavity case and very far away from the COMSOL FEA results for the truncated cone.  Also, the plots for the electromagnetic fields you posted did not look right. But that was some time ago when you were using a 2-D model, and I don't recall seeing recent postings of your MEEP solutions.

Have you now obtained MEEP solutions for the truncated cone and the cylindrical cavity that are in good agreement with known 3D solutions that give you such strong confidence on MEEP's calculations for much more complicated cases like the ones you are discussing above?

Title: Re: EM Drive Developments - related to space flight applications - Thread 2
Post by: jmossman on 04/04/2015 05:54 am
...
Dr. Rodal, I am not an RF engineer and haven't performed math with Maxwell's equations many years.  My following thoughts may have completely overlooked a fundamental issue, so please take them with the appropriate degree of skeptism.  :)

Doesn't the above description (regarding the mathematical basis for the standing wave, and resulting zero time-average Poynting vector) require an ideal resonator with no losses?

...
Hi James,

No, there is no requirement to have an ideal resonator with no losses, meaning an infinite Q.

The Finite Element Analysis (FEA) using COMSOL performed by NASA took into account tan delta losses to compute a finite Q.  Still, COMSOL FEA solves the standard Maxwell equations, and it contains no esoteric physics whatsoever.

One can also consider a non-ideal resonator using an exact solution.  One must use complex variables.  The losses responsible for a finite Q are due to the imaginary part, which is responsible for the material property "loss tangent" or "tan delta". 

See for example:  http://web.mit.edu/22.09/ClassHandouts/Charged%20Particle%20Accel/CHAP12.PDF

Maxwell's equation mandating that the Curl of the electric field E must equal the negative of the time derivative of the magnetic field B still must be satisfied.  See Equation 12.34 in the above link.



The electric permittivity describes the interaction of a material with an electric field E and is a complex quantity.

The real part of permittivity is a measure of how much energy from an external electric field is stored in a material. The imaginary part of permittivity is called the loss factor and is a measure of how dissipative or lossy a material is to an external electric field. The imaginary part of permittivity is always greater than zero and is usually much smaller than the real part. The loss factor includes the effects of both dielectric loss and conductivity.

Similarly, real materials have a magnetic susceptibility which is a complex quantity. 

An analysis taking into account the complex (real and imaginary parts) of these physical properties does not change Maxwell's equations.

Which part of Maxwell's equations account for the observed thermal losses due to the induced currents in the frustum's large base?
In order to solve differential equations one must provide boundary conditions.  The thermal losses due to the induced eddy-currects in the copper are a result of the imaginary part of the material properties appearing in the solution of the boundary conditions to solve Maxwell's differential equations.

That is how COMSOL's Finite Element Analysis provided the solution for the thermal losses and the predicted temperature for NASA's EM Drive truncated cone.

The frequencies obtained by COMSOL's FEA were obtained by solving an eigenvalue problem.

Damping is responsible for the finite amplitude of the response, but the presence of damping does not preclude standing waves, waves which have fixed nodes and anti-nodes.  The boundary conditions due to the end plates do not disappear due to heat production.  For example, one of the boundary conditions is that the tangent electric field must be continuous at the material interface.  This boundary condition (and therefore the node produced in that component of the electric field) does not cease to exist due to heat generation.  That only affects the amplitude.  The only way to remove that boundary condition would be to remove the end plate, and if you would do that, the EM Drive would no longer be a completely enclosed cavity.

You accurately and completely answered the questions I asked.  Unfortunately, I didn't ask my questions very well.  :)

http://forum.nasaspaceflight.com/index.php?topic=36313.msg1354235#msg1354235

A constant-amplitude standing wave does indeed result in a zero time-average Poynting vector.  However, I am questioning your conclusion that a constant-amplitude standing wave accurately represents a real resonator cavity such as the as-tested EM drive frustum.  Instead, I would expect a decaying amplitude standing wave to be a more accurate model/plot (as would be derived from a full solution to Maxwell's equations with proper boundary conditions such as non-zero resistance, etc).

Once a time-decaying standing wave is used for computation of a time-average Poynting vector, I'm having trouble seeing how the incident and reflected energy can perfectly cancel and become zero.  I'll readily admit I may be oversimplifying and/or missing a fundamental concept;  it's been a long time since I actually computed time constants for resonant cavities using Maxwell's equations and non-zero resistances.

Phrased a bit differently, I believe only excited modes with current/thermal losses in the base plates will significantly weight the direction of the time-average Poynting vector.  Each pair of incident/reflected waves would have a larger energy loss at the base plate with the excited E field (and therefore excited currents) than the energy loss at the opposing base plate.  For modes with near-zero E fields at the base plate boundaries, each incident/reflected wave pair would have a near equal energy delta regardless of which base plate they came in contact with;  the resulting time-averaged direction would be random and magnitude limited by the energy lost in the very first reflection (randomly either the large or small base, with a magnitude very close to zero).

I view this Poynting vector discussion to be completely independent of whether Dr. White's QV interactions, or some other classical physics can explain the EM drive anomalous thrust.  Just wanted to chime in on a what appeared to be the use of a constant-amplitude standing wave to describe a real-world system.  Your earlier observation of a non-zero time averaged Poynting vector seemed like a reasonable statement given that only excited modes with current/thermal losses in the base plates would quickly diverge from the simplified constant-amplitude standing wave model. 

Regards,
James
Title: Re: EM Drive Developments - related to space flight applications - Thread 2
Post by: mlindner on 04/04/2015 07:38 am
It appears my previous post got deleted.

So I'll ask again. Has this psudoscience nonsense been found to be false yet? Have any proper scientists tested this contraption so that they can show that it doesn't work?

I'm sure some "proper scientists" will come round for a look eventually. In the meantime, history has a lot to teach us about these sorts of things:

http://amasci.com/weird/vindac.html
http://www.lifehack.org/articles/lifestyle/6-world-changing-ideas-that-were-originally-rejected.html
http://www.cracked.com/article_18822_5-famous-scientists-dismissed-as-morons-in-their-time.html

Every previous world changing idea didn't try to violate a founding principal of all physics, namely CoE and CoM.

Quote
Quote
...so that they can show that it doesn't work

Just curious, what makes you so certain that it doesn't work? I mean, I have no idea if it works or not. The only certain thing I know here is that I don't know for certain if it works or doesn't.

There is a growing body of evidence which suggest that it does work. It hasn't been proven by anybody that it doesn't work.

I'm still waiting for an actual test of the operation of this craft. There haven't been any non-faulty experiments done yet that actually show it producing any thrust. Namely it must be tested in a vacuum. I don't try to merge philosophy and science and currently this "EM Drive" is purely in the realm of philosophy with no actual data yet. Thus I dismiss it just like the people claiming they made an anti-gravity drive in their garage.

It's rather insulting that this forum topic even exists here.
Title: Re: EM Drive Developments - related to space flight applications - Thread 2
Post by: Star One on 04/04/2015 08:42 am
The people who do decide if a thread/topic exists or not on here seem happy for it too. I suggest we leave it at that.
Title: Re: EM Drive Developments - related to space flight applications - Thread 2
Post by: Galactic Penguin SST on 04/04/2015 10:22 am
It appears my previous post got deleted.

So I'll ask again. Has this psudoscience nonsense been found to be false yet? Have any proper scientists tested this contraption so that they can show that it doesn't work?

I'm sure some "proper scientists" will come round for a look eventually. In the meantime, history has a lot to teach us about these sorts of things:

http://amasci.com/weird/vindac.html
http://www.lifehack.org/articles/lifestyle/6-world-changing-ideas-that-were-originally-rejected.html
http://www.cracked.com/article_18822_5-famous-scientists-dismissed-as-morons-in-their-time.html

Every previous world changing idea didn't try to violate a founding principal of all physics, namely CoE and CoM.

Quote
Quote
...so that they can show that it doesn't work

Just curious, what makes you so certain that it doesn't work? I mean, I have no idea if it works or not. The only certain thing I know here is that I don't know for certain if it works or doesn't.

There is a growing body of evidence which suggest that it does work. It hasn't been proven by anybody that it doesn't work.

I'm still waiting for an actual test of the operation of this craft. There haven't been any non-faulty experiments done yet that actually show it producing any thrust. Namely it must be tested in a vacuum. I don't try to merge philosophy and science and currently this "EM Drive" is purely in the realm of philosophy with no actual data yet. Thus I dismiss it just like the people claiming they made an anti-gravity drive in their garage.

It's rather insulting that this forum topic even exists here.

The main thing is that these guys here are completely skeptical of the original "theory" and are trying to do their own experiments - from what I read here it seems that they were skeptical of the "experimental results" as well but they want to do an independent experiment as well to settle the case, whether there is some effect that could be used in spaceflight or, if I read correctly, more probably if there is some other thing in action that cannot be used as a thruster.

At least, that is what I understand from my own undergraduate level physics anyway (some of the math is beyond my level, but at least I knew what a Bessel function is  ;)). So I don't see why such a thread here is "insulting" as long as those guys aren't hopping to a conclusion that this "EM drive" works with completely wrong physics explanations?

Title: Re: EM Drive Developments - related to space flight applications - Thread 2
Post by: Mulletron on 04/04/2015 04:40 pm
It appears my previous post got deleted.

So I'll ask again. Has this psudoscience nonsense been found to be false yet? Have any proper scientists tested this contraption so that they can show that it doesn't work?

I'm sure some "proper scientists" will come round for a look eventually. In the meantime, history has a lot to teach us about these sorts of things:

http://amasci.com/weird/vindac.html
http://www.lifehack.org/articles/lifestyle/6-world-changing-ideas-that-were-originally-rejected.html
http://www.cracked.com/article_18822_5-famous-scientists-dismissed-as-morons-in-their-time.html

Every previous world changing idea didn't try to violate a founding principal of all physics, namely CoE and CoM.

Quote
Quote
...so that they can show that it doesn't work

Just curious, what makes you so certain that it doesn't work? I mean, I have no idea if it works or not. The only certain thing I know here is that I don't know for certain if it works or doesn't.

There is a growing body of evidence which suggest that it does work. It hasn't been proven by anybody that it doesn't work.

I'm still waiting for an actual test of the operation of this craft. There haven't been any non-faulty experiments done yet that actually show it producing any thrust. Namely it must be tested in a vacuum. I don't try to merge philosophy and science and currently this "EM Drive" is purely in the realm of philosophy with no actual data yet. Thus I dismiss it just like the people claiming they made an anti-gravity drive in their garage.

It's rather insulting that this forum topic even exists here.

Well I can't fault you for being skeptical, but I do urge you to review the pages of this thread. Things start getting rather interesting around page 20 or so. Many of your concerns have been addressed here, such as the vacuum testing, which has been completed with data provided, and reported here by an engineer at Eagleworks.
http://forum.nasaspaceflight.com/index.php?topic=36313.msg1326608#msg1326608

Also interesting:
http://forum.nasaspaceflight.com/index.php?topic=36313.msg1333835#msg1333835

And, in my humble opinion, there has been plenty of existing, published in reputable journals...science uncovered, which can shed light on how the EMdrive can thrust, without violating any conservation laws.
http://forum.nasaspaceflight.com/index.php?topic=36313.msg1330846#msg1330846
http://forum.nasaspaceflight.com/index.php?topic=36313.msg1333392#msg1333392

Also @Notsosureofit has developed his own very plausible ideas about how EMdrive can thrust, using good old established science. @Rodal, @Aero, and @Frobnicat have all provided exhaustive data and calculations both for and against the reports of measured thrust at Eagleworks and NWPU China.

We're openly hostile to pseudoscience here, but at the same time, we must be mindful that in order to get to the bottom of this mystery, we have to be willing to step outside of our comfort zones.

From a practical standpoint, I find it extremely unlikely that the only means mankind will ever have to propel spacecraft through the vacuum, is to carry along stores of fuel and shoot propellant out the back side.

If EMdrives aren't the answer, the lessons learned from this may lead to the answer. Once field propulsion is a reality, we can consider the solar system to be unlocked.

Now that is worth a look.
Title: Re: EM Drive Developments - related to space flight applications - Thread 2
Post by: Star One on 04/04/2015 05:11 pm
Also how many times in the history of science have things been declared to be nonsense by the majority only later turning out to be correct and the status quo wrong.

By the way aren't Eagleworks approaching the end of the period of time when they have to produce the results for this party testing?

Title: Re: EM Drive Developments - related to space flight applications - Thread 2
Post by: MichaelBlackbourn on 04/04/2015 05:17 pm
Great post Mulletron about the why of doing this.

I'm hopeful something like EMdrive or WoodwardME ends up being possible.

Both are a little like the Polywell, 10-20 million and you could settle the issue conclusively. The payoff on any of these if they come up legit would be immense. Even better if you can pair a polywell with an EMdrive.... Then we're in business with just a pocket full of boron and hydrogen.

I wonder what the math is on pushing a 3m polywell chamber with pb+j He conversion equipment and an EMdrive... Wait, maybe you could just bypass the EMdrive with a polywell and just fire the He out one side... Hmm. Either way... We need to investigate these possibilities... The rocket equation is too brutal.
Title: Re: EM Drive Developments - related to space flight applications - Thread 2
Post by: Star One on 04/04/2015 05:18 pm
Problem is no one is going to give that kind of money unless they are proved to work at the level they are now.
Title: Re: EM Drive Developments - related to space flight applications - Thread 2
Post by: MichaelBlackbourn on 04/04/2015 05:32 pm
Problem is no one is going to give that kind of money unless they are proved to work at the level they are now.

I would guess that if not now, then in 10-15 years someone like spacex might get into a bit more speculative research once they have their mars plans in action. Let's hope.
Title: Re: EM Drive Developments - related to space flight applications - Thread 2
Post by: Notsosureofit on 04/04/2015 06:21 pm
FYI

Still fussing in short bursts to find an integral version of the dispersion speculation.


@RODAL

One thing that seems to keep coming through is that the step change of dielectric in the mid-waveguide (cylindrical) case has twice the thrust of a linear change, all else being equal.

Got to run but more on this later.

Title: Re: EM Drive Developments - related to space flight applications - Thread 2
Post by: Rodal on 04/04/2015 08:15 pm

You accurately and completely answered the questions I asked.  Unfortunately, I didn't ask my questions very well.  :)

http://forum.nasaspaceflight.com/index.php?topic=36313.msg1354235#msg1354235

A constant-amplitude standing wave does indeed result in a zero time-average Poynting vector.  However, I am questioning your conclusion that a constant-amplitude standing wave accurately represents a real resonator cavity such as the as-tested EM drive frustum.  Instead, I would expect a decaying amplitude standing wave to be a more accurate model/plot (as would be derived from a full solution to Maxwell's equations with proper boundary conditions such as non-zero resistance, etc).

Once a time-decaying standing wave is used for computation of a time-average Poynting vector, I'm having trouble seeing how the incident and reflected energy can perfectly cancel and become zero.  I'll readily admit I may be oversimplifying and/or missing a fundamental concept;  it's been a long time since I actually computed time constants for resonant cavities using Maxwell's equations and non-zero resistances.

Phrased a bit differently, I believe only excited modes with current/thermal losses in the base plates will significantly weight the direction of the time-average Poynting vector.  Each pair of incident/reflected waves would have a larger energy loss at the base plate with the excited E field (and therefore excited currents) than the energy loss at the opposing base plate.  For modes with near-zero E fields at the base plate boundaries, each incident/reflected wave pair would have a near equal energy delta regardless of which base plate they came in contact with;  the resulting time-averaged direction would be random and magnitude limited by the energy lost in the very first reflection (randomly either the large or small base, with a magnitude very close to zero).

I view this Poynting vector discussion to be completely independent of whether Dr. White's QV interactions, or some other classical physics can explain the EM drive anomalous thrust.  Just wanted to chime in on a what appeared to be the use of a constant-amplitude standing wave to describe a real-world system.  Your earlier observation of a non-zero time averaged Poynting vector seemed like a reasonable statement given that only excited modes with current/thermal losses in the base plates would quickly diverge from the simplified constant-amplitude standing wave model. 

Regards,
James
James, thank you for the interesting, thought-provoking discussion.  :)

Let's calculate some numbers to estimate what we are discussing.

The electromagnetic fields transition from the air or vacuum medium (where they are out of phase by 90 degrees) to the copper over an extremely small distance: a boundary layer. The skin depth for copper at 2 GHz is 1.48 micrometers = 58.2 microinches .  When showing the Poynting vector field distribution this distance is infinitesimal compared to the rest of the cavity.  In this very small distance inside the copper (1.48 micrometers ) the electromagnetic fields in the copper are out of phase by approximately 45 degrees (due to the high conductivity of copper).  For a transverse magnetic (TM) mode, the only electromagnetic field component that is continuous across the vacuum/copper interface is the electric field component tangent to the copper surface.

More interestingly (for this thread's discussion due to the significance that the NASA experimenters have placed on the dielectric being responsible for providing the measured thrust) is what happens in the High Density PolyEthylene (HDPE) dielectric polymer insert.  Because the dimensions of the dielectric are not negligible compared to the EM Drive's dimension, and the dielectric is not modeled as just a boundary condition.

The loss tangent of HDPE is reported to be

tan delta = 0.0004

Therefore the intrinsic impedance angle is

intrinsic impedance angle =(ArcTan[0.0004])/2

Therefore, inside the HDPE dielectric the electric and magnetic fields, instead of being out of phase by 90 degrees (as they are in the air or vacuum medium), will be out of phase by:

90 - (180/Pi) (ArcTan[0.0004])/2 = 89.98854084470854

This phase angle (89.9885 degrees)  will show practically no visual difference with 90 degrees at the resolution of the following image :

(http://www-ssc.igpp.ucla.edu/personnel/russell/papers/skip_ed/fig9.gif)

The Poynting vector inside the HDPE dielectric, instead of having a zero time average, will have the following factor multiplying ExB/mu :

Cos[(Pi/180) (90 - (180/Pi) (ArcTan[0.0004])/2)]/2= 0.00009999999400006368

So, inside the HDPE polymer dielectric the Poynting vector has this small magnitude over a period (or multiples thereof).

So, the extent of this approximation, for the HDPE dielectric is about 0.01 % (which is negligible in comparison to several other approximations).


Now, let's examine what this (very small intensity Poynting vector time average) means, concerning the discussion in this EM Drive.

If one were to posit that the EM Drive's thrust is due to the very small magnitude of the time average of the Poynting vector due to these thermal losses (in the HDPE dielectric or in the copper):

1) It would mean that there should be more thrust with lower Q.  This is the complete opposite of what the experimenters like Shawyer claims (Shawyer claims that the higher the Q, the greater the thrust).  Notice that

Tan [loss angle] = Tan[ 2 impedance angle ]= 1/Q

2) All the equations proposed so far (by Shawyer, McCulloch and @Notsosureofit) have predicted thrust proportional to Q.  This is the complete opposite of what such a Poynting vector would predict (it would predict thrust proportional to 1/Q instead), because

Tan delta= 1/Q

measured Q        effective tan delta

7320                  1.366* 10 ^(-4)
22000                4.545* 10 ^(-5)
10^6                  10^(-7)

3) It would mean that experimenters like Shawyer and Fetta are in the completely wrong track pursuing superconducting EM Drives, as superconducting EM Drives would lead to practically no thrust (the opposite of what they claim) because superconducting EM Drives would display practically no heat losses and hence zero time average Poynting vector.

4) Considering the HDPE dielectric acting as a sink (energy flowing from the EM Drive towards the HDPE where the energy is dissipated internally in the dielectric polymer due to its tandelta and hence irretrievably lost instead of being reflected), the Poynting vector would be directed towards the HDPE dielectric, that is towards the small base, and hence the EM Drive should experience a recoil force and acceleration towards the big base.  This is the opposite direction force found in NASA's experiments with the dielectric.  (Recall that NASA Eagleworks found no thrust force with mode TE012 without a HDPE dielectric and that with the HDPE dielectric inserted at the small base they found a force and acceleration directed towards the small base.)

Title: Re: EM Drive Developments - related to space flight applications - Thread 2
Post by: jmossman on 04/04/2015 10:02 pm
I realize this is dredging up an old discussion, but wanted to suggest an additional factor that may have been overlooked in the Chinese/Shawyer/Eagleworks comparison:  the losses from the antenna within the cavities.

Folks:

In the meantime, lets ask why 60 watts of relatively harmonic free sine-wave RF power at the 1,937.118 MHz AKA the TM212 resonant frequency in this copper frustum cavity, can only generate a paltry ~60uN, whereas the Chinese claimed to have produce 160,000uN using just ~150 watts of 2,450 MHz RF signals from a magnetron?  The magnetron RF signal source that is anything but a pure sine-wave generator, that instead has a modulated FM bandwidth of at least +/-30 MHz that is also concurrently amplitude modulated (AM) with thermal electron noise. 


Taking a critical look at this question, and knowing that the spectral shape of a magnetron looks like (see below) compared to a CW spike. It seems evident that a CW spike isn't the best waveform to use if you want to maximize thrust. Dollars to donuts says the Chinese are making full use of the available bandwidth of their resonant cavity by using that noisy magnetron. Magnetrons have lots of phase noise too. You can't easily use them on phased array radars because of that for example.
...
I agree with Mulletron that the answer to Paul March's question is that it is much more effective to have a distributed power spectral density than the power concentrated at a single frequency spike.  When the natural frequency changes in an unpredictable manner, it is much more effective to have a distributed power spectral density of excitation (it is the power spectral density ( http://en.wikipedia.org/wiki/Spectral_density#Power_spectral_density ) over the spectrum of changing natural frequencies that matters).
...
This is evident from the very low Q's reported by NASA (7K to 22K) compared with the Chinese, who report a Q=117K
Quote from: Juan Yang
the resonant frequency and quality factor of the independent microwave resonator system are 2.44895 GHz and 117495.08 respectively
...
NASA's reported Q for the vacuum experiment is a meager Q = 6726, which is 17 times smaller than the Chinese reported Q = 117495.

Zen-in nicely summarized the effect of an antenna within a resonant cavity:
...
A long time ago an RF engineer friend explained to me that cavities, filters and LC circuits are never dissipative.   The power either goes through them or is reflected.   If an RF signal with 20 MHz of bandwidth at 2085 MHz is sent through a 5 pole cavity filter with 5 MHz passband a large fraction of the RF power is simply reflected back to the amplifier.   

In the diagrams I have seen of both the Chinese and latest Shawyer designs, I see an effective antenna source which is significantly more efficient at injecting energy into the cavity than extracting energy out of the cavity.   In contrast, I see a loop antenna within the Eagleworks design which does not have nearly the same effective beam shape, and therefore couples/extracts energy from the cavity far more than the Chinese/Shawyer designs.

I've attached some incredibly crude annotated diagrams trying to show a hypothetical path of a reflected wave.  I've taken huge artistic license and the diagrams are no where close to be scientifically accurate (reflection angles, etc).  However, the use of the magnetron has more than just a wide bandwith;  the waveguide port feeding the cavity will look like a very directional antenna and will not couple/extract energy efficiently that is traveling perpendicular to the waveguide port.  Once a resonance is excited within the cavity/frustum, a majority of the energy will be in the axle direction which is perpendicular to the injection waveguide port.

This contrasts sharply with the loop antenna within the Eagleworks cavity.  While I don't know the exact antenna beam pattern/shape, I suspect a dipole isn't a bad approximation;  this means that the loop antenna couples/injects and couples/extracts equally well in the parallel direction to the conical walls.  While a majority of the resonant energy is between the small and big plates, some energy will be parallel to the conical walls where the loop antenna can couple/extract energy which gets fed back to the amplifier.

If the above reasoning is sound (and perhaps it is not;  I am not an RF engineer and haven't dealt with these concepts in many years), then the frustum when excited by a magnetron/waveguide perpendicular to the conical major axis can build to energy levels higher than the Eagleworks loop antenna design (using the same input energy).  From my layperson's perspective, the magnetron/waveguide approach would also seem to introduce a time constant into mix as a cavity "charge time" (and discharge time).

Best Regards,
James
Title: Re: EM Drive Developments - related to space flight applications - Thread 2
Post by: Mulletron on 04/04/2015 11:27 pm
From the letter linked to below:

Quote
Note here that we are choosing to work with the
momentum density associated with the canonical energy
momentum tensor rather than the Poynting vector; the
latter is expected to integrate to zero [9]

Quote
Further,
they should be arranged so that the resulting tetrahe-
dron (with the four particles placed at the vertices) has
no parity symmetry so that the vacuum photons get to
see a chiral structure. If these conditions are fulfilled
then a non-zero momentum develops which scales as the
fourteenth inverse power of the length scale of the tetra-
hedron
.

http://arxiv.org/pdf/1011.4376.pdf
http://iopscience.iop.org/0295-5075/93/4/41002/fulltext/epl_93_4_41002.html

Don't think I've linked to this one before.

Quote
Structure of polyethylene consists of directed tetrahedral configuration of sp3 hybridized carbon bonds along repeat structure with no free valence electrons.
http://plastics.tamu.edu/class-resources/semi-conducting_polymers

Title: Re: EM Drive Developments - related to space flight applications - Thread 2
Post by: Rodal on 04/05/2015 01:58 am
From the letter linked to below:

Quote
Note here that we are choosing to work with the
momentum density associated with the canonical energy
momentum tensor rather than the Poynting vector; the
latter is expected to integrate to zero [9]

Quote
Further,
they should be arranged so that the resulting tetrahe-
dron (with the four particles placed at the vertices) has
no parity symmetry so that the vacuum photons get to
see a chiral structure. If these conditions are fulfilled
then a non-zero momentum develops which scales as the
fourteenth inverse power of the length scale of the tetra-
hedron
.

http://arxiv.org/pdf/1011.4376.pdf
http://iopscience.iop.org/0295-5075/93/4/41002/fulltext/epl_93_4_41002.html

Don't think I've linked to this one before.

Quote
Structure of polyethylene consists of directed tetrahedral configuration of sp3 hybridized carbon bonds along repeat structure with no free valence electrons.
http://plastics.tamu.edu/class-resources/semi-conducting_polymers

Thank you for bringing attention to this paper.  Easier to read because it is shorter than their usual papers !.  I noticed that they used Mathematica as well to calculate their functions.  I am adding calculating the momentum density associated with the canonical energy momentum tensor to my "list of things to do when I have a chance."  At first glance it seems to me that unless one takes into account an anisotropic HDPE dielectric I am not going to find anything new, because I expect all the terms of the divergence of the canonical energy momentum tensor (for  the EM Drive with an isotropic HDPE dielectric section) to average zero over a period (or multiples thereof).

I also notice that they state the non-trivial contribution is a 4th order term in a perturbation analysis of the nonlinear problem of  mapping between object profile and scattered field: http://en.wikipedia.org/wiki/Lippmann%E2%80%93Schwinger_equation
http://www.ece.neu.edu/faculty/devaney/ppt/stanford.ppt
 
!

Quote
It is shown that the first non-trivial contribution of the momentum transfer to the object from the radiation field occurs at fourth order in the Born series.
The fourth order terms are usually neglected in most treatises !  Actually, sometimes the second order Born terms are taken into account (for example in http://en.wikipedia.org/wiki/Grazing-incidence_small-angle_scattering  ), but I don't know of applications that have used the 3rd order term and much less the 4th order term. 

So even if the authors are correct, it is no wonder that this is stuff that is usually neglected.
.  According to the authors it only appears when there is

1) an anisotropic (chiral) medium
and
2) taking into account 4th order terms in a perturbation analysis




Let's look again at the energy-momentum tensor:



(http://upload.wikimedia.org/math/9/0/6/9067342a3c3e13deacfc7cded6b5da36.png)

(http://upload.wikimedia.org/wikipedia/commons/thumb/f/fe/StressEnergyTensor_contravariant.svg/805px-StressEnergyTensor_contravariant.svg.png)

I have already calculated the divergence of the Poynting vector for the EM Drive (without any HDPE dielectric insert), that is the divergence of the momentum density components Sx/c =T01 , Sy/c =T02 , Sz/c =T03 and will post the images when I have a chance and have checked my results.

EDIT 1: yes, the paper itself states:

Quote
One should also note that for purely dielectric scatterers, the momentum is expected to vanish as the electromagnetic momentum density coincides with the Poynting vector.

So perhaps the only experimenters that have found something along these lines is NASA Eagleworks due to Paul March trying the PTFE and HDPE dielectric polymer inserts, and without them there is really no thrust (one of the very interesting things disclosed by Paul March is that Neoprene Rubber gave negligible thrust force). This would mean that the experimental forces measured by Shawyer and Juan Yang in China for EM Drives (using much higher power input) without thermoplastic inserts maybe just thermal effect artifacts.

EDIT 2: It the authors are correct and one needs to take into account nonlinearity + anisotropy to get momentum from the quantum vacuum, this is much more complicated stuff than what Dr. White or Dr. Woodward have been considering.  It will be much more difficult to prove or disprove ....There are non-uniqueness issues associated with an inverse nonlinear problem, and many other complications...There is no unique canonical correction to the energy-stress tensor, there are several and it is not clear which one is right for the problem....

Title: Re: EM Drive Developments - related to space flight applications - Thread 2
Post by: ThinkerX on 04/05/2015 05:16 am
Quote
So perhaps the only experimenters that have found something along these lines is NASA Eagleworks due to Paul March trying the PTFE and HDPE dielectric polymer inserts, and without them there is really no thrust (one of the very interesting things disclosed by Paul March is that Neoprene Rubber gave negligible thrust force). This would mean that the experimental forces measured by Shawyer and Juan Yang in China for EM Drives (using much higher power input) without thermoplastic inserts maybe just thermal effect artifacts.

EDIT 2: It the authors are correct and one needs to take into account nonlinearity + anisotropy to get momentum from the quantum vacuum, this is much more complicated stuff than what Dr. White or Dr. Woodward have been considering.  It will be much more difficult to prove or disprove ....There are non-uniqueness issues associated with an inverse nonlinear problem, and many other complications...There is no unique canonical correction to the energy-stress tensor, there are several and it is not clear which one is right for the problem....

So is this a brick wall or a convoluted maze for the EM Drive?  Are these thermoplastic inserts suitable long term use in the environment created by the EM Drive?  (thinking of the amount of heat this device produces)  Is there someway they could be modified to increase thrust?
Title: Re: EM Drive Developments - related to space flight applications - Thread 2
Post by: Star-Drive on 04/05/2015 05:34 am
Dr. Rodal:

I think that the reason why the Neoprean rubber didn't generate any detectable thrust with the ~30W of available power was that it's carbon loaded which increases its RF loss tangent from that of HDPE or PTFE (~0.0004) up to 0.02 which really loads down the net E-field generated in the cavity for a given input power.  And since we think that the thrust effect we are observing is proportional to E-field squared (E^2) that could make a huge difference in performance.

Now please note that I concur with your analysis on the E&M Poynting vector time averaging to zero in the frustums.  For example when the Cannae pillbox cavity's input RF power feed is well Z-matched with a VSWR= ~1.00, the RF Poynting power flow into the cavity is always towards the large OD pancake section of the cavity.  If one envisions the quantum vacuum (Q-V) as a semi-virtual electrical plasma as Dr. White does, that would imply that the Poynting power flow vector would entrain the Q-V plasma and send it on its way toward the pillbox end of the cavity and then out of the cavity, the back-reaction on the cavity should be in the opposite direction towards the RF feed end of the Cannae test article, but the observed thrust vector is opposite to that surmise, i.e. toward the shorter RF sense antenna end of the cavity per the attached slide. 

Now Per the newly authored Q-V plasma simulation code that Dr. White just finished, the equal bidirectional Q-V plasma flow for the Cannae cavity comes from the high-Q pancake section with a Q of ~9,000 for the un-slotted version of the cavity.  However due to the high E-field region created n the throat of the RF feed, this cylindrically shaped high E-field volume acts as an obstruction to the Q-V plasma flow.  This E-field obstruction created in the PTFE cylinder then accelerates the Q-V plasma around it in a Bernoulli like effect that accelerates the Q-V plasma flow coming from the main pillbox cavity.  This unbalanced and accelerated Q-V plasma flow that goes away from the large pill box cavity in the direction of the RF input section is what generates the NET thrust in our model.   

Next, using this new Q-V plasma simulation tool that utilizes the instantaneous E&M fields from COMSOL for one complete RF cycle in 5 degree increments as its input file, we are now seeing why we need the PTFE or HDPE dielectrics in the frustum while using near pure sine wave power levels below ~100W in the ~2.0 GHz frequency range to generate detectable thrust, and why Shawyer and the Chinese didn't while pumping 80W to 2,500W using magnetron RF sources.  We think the reasons are two fold. 

The first is that Shawyer and the Chinese both used magnetron RF sources for their experiments.  An RF source that generates large AM, FM and PM modulation of the carrier wave with typical FM modulation bandwidth on the order of at least +/-20 MHz.  (These time rate to change of energy modulations increase the Q-V density in our model.) 

The second reason we found running these 3D Q-V plasma simulations for the EMPTY copper frustum, was that increasing the input power tends to focus the Q-V plasma flow from near omnidirectional from the frustum at low powers, to a much more jet like beam at higher powers measured in kW to tens of kW-rf.  In fact the simulation for the 100W run predicted only ~50uN for our pure RF system with dielectric, while the 10kW run predicted a thrust level of ~6.0 Newton without a dielectric in the cavity.  And at 100kW-rf it was now up to ~1300 Newton, but the input power to thrust production nonlinearity was starting to taper off around 50kW.   Of course these Q-V plasma thrust predictions are based on the Q-V not being immutable and non-degradable, a feature we admit is not widely accepted by the mainstream physics community, at least at the moment. :)

Lastly, due to the above non-linear thrust scaling with input power predictions, we have started the build up of a 100W-to-1,200W waveguide magnetron RF power system that will drive one of our aluminum RF frustum cavities.  Initially the test rig will follow Shawyer's first generation test rig that used a tetter-totter balance system in air only to see if we can generate similar thrust levels that Shawyer reported using a hermetic sealed box, which were in the ~16 to 300 milli-Newton range dependent on the Q-Factor of the frustum.

BTW, the reason we included the "what-if" Eagleworks can make this thing work solar system trajectory section on our 2014 JPC paper was that we have to continually tell management the value proposition for why they should fund our research, much in the same way we have to convince Chris Bergin here at NSF we really will be talking about space applications for these Q-Thruster like devices, once we get our hands around the physics they are using.  However when we do, the solar system and beyond will be ours for the picking...

Best, Paul M.
Title: Re: EM Drive Developments - related to space flight applications - Thread 2
Post by: Stormbringer on 04/05/2015 07:46 am
My analysis: "Mind Blown...resetting."   ;D

Title: Re: EM Drive Developments - related to space flight applications - Thread 2
Post by: flux_capacitor on 04/05/2015 11:01 am
Moving to higher power magnetron-based experiments is VERY good news! :)

Lastly, like any busy lab, Eagleworks could always use extra funding to deal with its daily heart burns and required salaries to keep it going.  However we are currently a NASA sponsored facility, which sadly precludes being able to accept crowd sourcing or any other outside source of funding, unless it's through a commercial NASA Space Act Agreement that has to be approved up through NASA headquarters in Washington DC.

Paul, a contributor on Talk-Polywell forums, Carl White, asked this interesting question (http://www.talk-polywell.org/bb/viewtopic.php?f=10&t=2949&p=120754#p120743):
Quote
They can't accept dollars, but can they still accept donations of equipment? What about equipment "sold" to them for $1? Could provide some RF amplifiers for example.
So would it be possible to give you some contribution in kind? If so, what do you need?
Title: Re: EM Drive Developments - related to space flight applications - Thread 2
Post by: Paul451 on 04/05/2015 03:08 pm
Carl White, asked this interesting question (http://www.talk-polywell.org/bb/viewtopic.php?f=10&t=2949&p=120754#p120743):
Quote
They can't accept dollars, but can they still accept donations of equipment? What about equipment "sold" to them for $1? Could provide some RF amplifiers for example.
So would it be possible to give you some contribution in kind? If so, what do you need?

In general, government regs forbidding accepting gifts include "or value" precisely to avoid that loophole.¹ So equipment, whether donated or "sold" for $1, is still a donation of value. IIRC, even volunteering your own labour is forbidden unless specifically authorised.

It's the same with NASA's unused facilities. NASA can't donate time to a private company, even if the facility (or staff!) would otherwise go unused, because they are required to charge "full equivalent commercial rates" or similar wording. The only exemption is if NASA and the private user "exchange services of equivalent value" via an SAA. NASA has been interpreting "equivalent value" as the private user letting NASA researchers play with their toys, which Congress has cracked down on recently.

That said, an exemption for crowd-sourced donations makes sense. Similarly, a Patreon type system where amazing people sponsor their favourite research project via a small monthly amount. It may be something worth lobbying for. Even if it is just a pro-forma authorisation for agency officials to be able to use SAA's for specific crowd-sourcing efforts. Unfortunately, SAA's have been tightened even further by Congress.

¹ For example, if I pay the tuition fees for the children of a government official to go to a private-school/top-university, or give them a free car/house/cheap-loan, etc, it's still bribery even though there was no cash-in-hand.
Title: Re: EM Drive Developments - related to space flight applications - Thread 2
Post by: Star-Drive on 04/05/2015 07:27 pm
Carl White, asked this interesting question (http://www.talk-polywell.org/bb/viewtopic.php?f=10&t=2949&p=120754#p120743):
Quote
They can't accept dollars, but can they still accept donations of equipment? What about equipment "sold" to them for $1? Could provide some RF amplifiers for example.
So would it be possible to give you some contribution in kind? If so, what do you need?

In general, government regs forbidding accepting gifts include "or value" precisely to avoid that loophole.¹ So equipment, whether donated or "sold" for $1, is still a donation of value. IIRC, even volunteering your own labour is forbidden unless specifically authorised.

It's the same with NASA's unused facilities. NASA can't donate time to a private company, even if the facility (or staff!) would otherwise go unused, because they are required to charge "full equivalent commercial rates" or similar wording. The only exemption is if NASA and the private user "exchange services of equivalent value" via an SAA. NASA has been interpreting "equivalent value" as the private user letting NASA researchers play with their toys, which Congress has cracked down on recently.

That said, an exemption for crowd-sourced donations makes sense. Similarly, a Patreon type system where amazing people sponsor their favourite research project via a small monthly amount. It may be something worth lobbying for. Even if it is just a pro-forma authorisation for agency officials to be able to use SAA's for specific crowd-sourcing efforts. Unfortunately, SAA's have been tightened even further by Congress.

¹ For example, if I pay the tuition fees for the children of a government official to go to a private-school/top-university, or give them a free car/house/cheap-loan, etc, it's still bribery even though there was no cash-in-hand.

What Paul451 said.

All:

Just to make Chris happy, lets assume that the Eagleworks' current Q-V plasma code thrust predictions for the TM010 or TM011, 942 MHz resonant mode running at 100kW is correct for our copper frustum, see attached slide, and we then attached it to a commercial 100kW, 2.45 GHz magnetron, see second attached slide.  What you might get is a ~2,000 Newton thruster with an effective Isp of well over one million seconds dependent on the attributes of the selected power plant needed to drive it.  Now what could one do with that kind of thruster system for a human Mars or say Saturn trips if we specified the use of an LENR based 1.0 MWe reactor that needs very little radiation shielding.

Best, Paul M.
Title: Re: EM Drive Developments - related to space flight applications - Thread 2
Post by: flux_capacitor on 04/05/2015 07:59 pm
If a cubesat experiment in space cannot be done (due to insufficient funding for now), very high power experiments (10-100 kW) in ambient air are the only way to go, to prove the EmDrive does work as expected:

- Shawyer experiments in ambient air (as well as newer superconducting designs) are now kept under a shroud of secrecy. 

- Chinese mid-to-high power experimental results (~ 2000 W) in ambient air are on the contrary regularly published in academic journals, but are still uncertain because Pr. Juan Yang is not able to share additional data, and spurious causes like air currents are not ruled-out.

- Eagleworks ruled-out the possibility of ambient air currents, running the experiments in a hard vacuum. But because the setup needs to be compact and light enough to sit into the vacuum chamber and hang on the sensitive torsion pendulum (power source included), the amp had to be very weak (10 to 30 watts only) and moreover leaked/sparked in that lower pressure environment. Hence, sadly, less than a hundred micronewtons of thrust. Because of the tiny thrust signatures just above the seismic noise, the results published so far created more questions about other possible spurious causes (EM interaction with the torsion pendulum, the walls of the chamber…).

IMHO, only a very high power experiment producing a very high thrust (above a newton and more), and if possible even lift-off, where air flow circulation around the cavity could not account for the enormous thrust measured by equipment and observed with our own eyes, will settle the case. And will open the colonization of our solar system.
Title: Re: EM Drive Developments - related to space flight applications - Thread 2
Post by: Cinder on 04/05/2015 08:22 pm
LENR
Probably beyond the speculation tolerance of NSF :(  Does no proven power source (e.g. SEP, but even nuclear) make for a propulsion system that's competitive enough with existing and more conventional propulsion schemes, for short term applications?

Nuclear currently is, separately, a non-starter as well.  But opening up the solar system as conjectured with EM propulsion -- that would probably upset the public and industrial status quo enough to cause some reconsideration.. ?  EM merely needs to beat the alternatives:
we have to continually tell management the value proposition for why they should fund our research, [...] once we get our hands around the physics they are using.
  Utter revolution is accessory as a very short term goal; but probably inevitable anyway, if EM propulsion took off.
Title: Re: EM Drive Developments - related to space flight applications - Thread 2
Post by: tchernik on 04/05/2015 08:30 pm

IMHO, only a very high power experiment producing a very high thrust (above a newton and more), and if possible even lift-off, where air flow circulation around the cavity could not account for the enormous thrust measured by equipment and observed with our own eyes, will settle the case. And will open the colonization of our solar system.

Fully agree. Subtle pushes able to move a paper sheet in the vacuum of space in 0G are unlikely to attract any funding, because even if it works, it won't convince people that have already made their minds and are certain things like that can't be real.

While a self propelled chariot moving around on EM thrust alone would make some heads turn. And a flying demonstrator would be nearly irrefutable.
Title: Re: EM Drive Developments - related to space flight applications - Thread 2
Post by: Rodal on 04/05/2015 08:55 pm
Dr. Rodal:

I think that the reason why the Neoprean rubber didn't generate any detectable thrust with the ~30W of available power was that it's carbon loaded which increases its RF loss tangent from that of HDPE or PTFE (~0.0004) up to 0.02 which really loads down the net E-field generated in the cavity for a given input power.  And since we think that the thrust effect we are observing is proportional to E-field squared (E^2) that could make a huge difference in performance.

Now please note that I concur with your analysis on the E&M Poynting vector time averaging to zero in the frustums.  For example when the Cannae pillbox cavity's input RF power feed is well Z-matched with a VSWR= ~1.00, the RF Poynting power flow into the cavity is always towards the large OD pancake section of the cavity.  If one envisions the quantum vacuum (Q-V) as a semi-virtual electrical plasma as Dr. White does, that would imply that the Poynting power flow vector would entrain the Q-V plasma and send it on its way toward the pillbox end of the cavity and then out of the cavity, the back-reaction on the cavity should be in the opposite direction towards the RF feed end of the Cannae test article, but the observed thrust vector is opposite to that surmise, i.e. toward the shorter RF sense antenna end of the cavity per the attached slide. 

Now Per the newly authored Q-V plasma simulation code that Dr. White just finished, the equal bidirectional Q-V plasma flow for the Cannae cavity comes from the high-Q pancake section with a Q of ~9,000 for the un-slotted version of the cavity.  However due to the high E-field region created n the throat of the RF feed, this cylindrically shaped high E-field volume acts as an obstruction to the Q-V plasma flow.  This E-field obstruction created in the PTFE cylinder then accelerates the Q-V plasma around it in a Bernoulli like effect that accelerates the Q-V plasma flow coming from the main pillbox cavity.  This unbalanced and accelerated Q-V plasma flow that goes away from the large pill box cavity in the direction of the RF input section is what generates the NET thrust in our model.   

Next, using this new Q-V plasma simulation tool that utilizes the instantaneous E&M fields from COMSOL for one complete RF cycle in 5 degree increments as its input file, we are now seeing why we need the PTFE or HDPE dielectrics in the frustum while using near pure sine wave power levels below ~100W in the ~2.0 GHz frequency range to generate detectable thrust, and why Shawyer and the Chinese didn't while pumping 80W to 2,500W using magnetron RF sources.  We think the reasons are two fold. 

The first is that Shawyer and the Chinese both used magnetron RF sources for their experiments.  An RF source that generates large AM, FM and PM modulation of the carrier wave with typical FM modulation bandwidth on the order of at least +/-20 MHz.  (These time rate to change of energy modulations increase the Q-V density in our model.) 

The second reason we found running these 3D Q-V plasma simulations for the EMPTY copper frustum, was that increasing the input power tends to focus the Q-V plasma flow from near omnidirectional from the frustum at low powers, to a much more jet like beam at higher powers measured in kW to tens of kW-rf.  In fact the simulation for the 100W run predicted only ~50uN for our pure RF system with dielectric, while the 10kW run predicted a thrust level of ~6.0 Newton without a dielectric in the cavity.  And at 100kW-rf it was now up to ~1300 Newton, but the input power to thrust production nonlinearity was starting to taper off around 50kW.   Of course these Q-V plasma thrust predictions are based on the Q-V not being immutable and non-degradable, a feature we admit is not widely accepted by the mainstream physics community, at least at the moment. :)

Lastly, due to the above non-linear thrust scaling with input power predictions, we have started the build up of a 100W-to-1,200W waveguide magnetron RF power system that will drive one of our aluminum RF frustum cavities.  Initially the test rig will follow Shawyer's first generation test rig that used a tetter-totter balance system in air only to see if we can generate similar thrust levels that Shawyer reported using a hermetic sealed box, which were in the ~16 to 300 milli-Newton range dependent on the Q-Factor of the frustum.

BTW, the reason we included the "what-if" Eagleworks can make this thing work solar system trajectory section on our 2014 JPC paper was that we have to continually tell management the value proposition for why they should fund our research, much in the same way we have to convince Chris Bergin here at NSF we really will be talking about space applications for these Q-Thruster like devices, once we get our hands around the physics they are using.  However when we do, the solar system and beyond will be ours for the picking...

Best, Paul M.

Fantastic post, Paul.  We are so lucky that you are still working on this project and that you are reporting this information.  We hope that this project, as you outlined it, gets full funding.

I also love your scientific objectivity, for example, concerning that Dr. White's simulations do not appear to abide by the assumption that the Quantum Vacuum should be indestructible and immutable.  One reason that the mainstream physics community assumes the Quantum Vacuum is indestructible and immutable is because of the experimental observation that a fundamental particle like an electron (or a positron) has the same properties (e.g. mass, charge or spin), regardless of when or where the particle was created, whether now or in the early universe, through astrophysical processes or in a laboratory.  Another reason is that the Quantum Vacuum has what Albert Einstein in 1913 called "Nullpunktsenergie" (zero point energy): the lowest possible (time-averaged) energy that a quantum mechanical physical system may have.

Godspeed ahead :)
Title: Re: EM Drive Developments - related to space flight applications - Thread 2
Post by: Star One on 04/05/2015 09:02 pm

If a cubesat experiment in space cannot be done (due to insufficient funding for now), very high power experiments (10-100 kW) in ambient air are the only way to go, to prove the EmDrive does work as expected:

- Shawyer experiments in ambient air (as well as newer superconducting designs) are now kept under a shroud of secrecy. 

- Chinese mid-to-high power experimental results (~ 2000 W) in ambient air are on the contrary regularly published in academic journals, but are still uncertain because Pr. Juan Yang is not able to share additional data, and spurious causes like air currents are not ruled-out.

- Eagleworks ruled-out the possibility of ambient air currents, running the experiments in a hard vacuum. But because the setup needs to be compact and light enough to sit into the vacuum chamber and hang on the sensitive torsion pendulum (power source included), the amp had to be very weak (10 to 30 watts only) and moreover leaked/sparked in that lower pressure environment. Hence, sadly, less than a hundred micronewtons of thrust. Because of the tiny thrust signatures just above the seismic noise, the results published so far created more questions about other possible spurious causes (EM interaction with the torsion pendulum, the walls of the chamber…).

IMHO, only a very high power experiment producing a very high thrust (above a newton and more), and if possible even lift-off, where air flow circulation around the cavity could not account for the enormous thrust measured by equipment and observed with our own eyes, will settle the case. And will open the colonization of our solar system.

Just as an aside why have Shawyer's experiments dropped off the radar?
Title: Re: EM Drive Developments - related to space flight applications - Thread 2
Post by: flux_capacitor on 04/05/2015 10:12 pm
Just as an aside why have Shawyer's experiments dropped off the radar?

I don't know. I could think of two different possibilities:
- Either the latest superconducting tests are not positive at all, and contrary to Shawyer's expectation, the thrust does not dramatically improve with the Q factor of the cavity. The EmDrive, at least here on earth, would then be a dead-end.
- Or the exact opposite and Shawyer, who runs a private company (SPR Ltd) and filed various patents on the EmDrive, has now signed a contract with one or several big private aerospace companies for the development of new advanced propulsion systems based on the EmDrive technology, and does not want/is forbidden to disclose any information, maybe (more exactly surely if it's the case) because this information is protected under NDA.

All he accepted to answer me when I asked him about the precise dimensions of the Chinese frustum, is that for a number of years, SPR Ltd policy has been to provide worldwide support to universities and national research agencies (including NASA), for who they checked out their EmDrive cavity geometry using the design software SPR Ltd has developed in-house, because he claims commercial finite element software does not give accurate enough results for engineering purposes. But now we must understand the information that has been disclosed to him by the Chinese cannot be passed on to a third party.
Title: Re: EM Drive Developments - related to space flight applications - Thread 2
Post by: aero on 04/05/2015 11:12 pm
Quoting Dr. Rodal
Quote
To have confidence on numerical results one should start by comparing the results with known solutions for (at least) simpler cases.  (Particularly for a numerical method, like the Finite Difference Method used in MEEP, that as we have discussed before one can not be assured to converge to a solution? )

 For example, I compared my exact solutions, and they were within 1% or so of COMSOL FEA results for NASA's truncated cone and their experimental results.

Since we are interested in forces, Meep's ability to accurately calculate resonance frequency doesn't enter into the question. (I am provided with the cavity dimensions and drive frequencies, thereby avoiding that question.)

I have calculated a simple case to verify forces and achieved results within 1% of the known values. That is, Meep calculates the radiant force of a plane wave striking an absorbing surface in vacuum as -1.0006645812/c which is well within 1% of 1/c, the exact answer. Further, Meep calculates the radiant force of a plane wave striking a perfectly reflecting surface in vacuum as -2.0295743306/c which is within 1.5% of 2/c, the exact answer.

In these cases, the positive direction is from the detector to the source, so the minus sign is correct.



Title: Re: EM Drive Developments - related to space flight applications - Thread 2
Post by: Star One on 04/05/2015 11:34 pm
Just as an aside why have Shawyer's experiments dropped off the radar?

I don't know. I could think of two different possibilities:
- Either the latest superconducting tests are not positive at all, and contrary to Shawyer's expectation, the thrust does not dramatically improve with the Q factor of the cavity. The EmDrive, at least here on earth, would then be a dead-end.
- Or the exact opposite and Shawyer, who runs a private company (SPR Ltd) and filed various patents on the EmDrive, has now signed a contract with one or several big private aerospace companies for the development of new advanced propulsion systems based on the EmDrive technology, and does not want/is forbidden to disclose any information, maybe (more exactly surely if it's the case) because this information is protected under NDA.

All he accepted to answer me when I asked him about the precise dimensions of the Chinese frustum, is that for a number of years, SPR Ltd policy has been to provide worldwide support to universities and national research agencies (including NASA), for who they checked out their EmDrive cavity geometry using the design software SPR Ltd has developed in-house, because he claims commercial finite element software does not give accurate enough results for engineering purposes. But now we must understand the information that has been disclosed to him by the Chinese cannot be passed on to a third party.
Thanks for that informative response. I do wonder if we will hear anything from him at all then for the foreseeable future.:(
Title: Re: EM Drive Developments - related to space flight applications - Thread 2
Post by: Rodal on 04/06/2015 12:38 am
Quoting Dr. Rodal
Quote
To have confidence on numerical results one should start by comparing the results with known solutions for (at least) simpler cases.  (Particularly for a numerical method, like the Finite Difference Method used in MEEP, that as we have discussed before one can not be assured to converge to a solution? )

 For example, I compared my exact solutions, and they were within 1% or so of COMSOL FEA results for NASA's truncated cone and their experimental results.

Since we are interested in forces, Meep's ability to accurately calculate resonance frequency doesn't enter into the question. (I am provided with the cavity dimensions and drive frequencies, thereby avoiding that question.)

I have calculated a simple case to verify forces and achieved results within 1% of the known values. That is, Meep calculates the radiant force of a plane wave striking an absorbing surface in vacuum as -1.0006645812/c which is well within 1% of 1/c, the exact answer. Further, Meep calculates the radiant force of a plane wave striking a perfectly reflecting surface in vacuum as -2.0295743306/c which is within 1.5% of 2/c, the exact answer.

In these cases, the positive direction is from the detector to the source, so the minus sign is correct.

Modeling a plane wave striking an absorbing surface is a different problem than modeling the standing waves in a resonating cavity like the EM Drive.

I was referring to images like these ones, that you posted some time ago:

(http://forum.nasaspaceflight.com/index.php?action=dlattach;topic=36313.0;attach=705861;image)

The electromagnetic fields in that image don't look like the electromagnetic fields in the EM Drive, indicating that there was something wrong with your model of the EM Drive.  Since electromagnetic forces are due to the electromagnetic fields, if one doesn't get the electromagnetic fields correctly, then it stands to reason that the calculations of electromagnetic forces in the EM Drive cannot be correct either.

The electromagnetic field surface distributions should look like this for the frequency being tested under partial vacuum by NASA Eagleworks since last December:

(http://forum.nasaspaceflight.com/index.php?action=dlattach;topic=36313.0;attach=635195;image)

(http://forum.nasaspaceflight.com/index.php?action=dlattach;topic=36313.0;attach=635196;image)
Title: Re: EM Drive Developments - related to space flight applications - Thread 2
Post by: aero on 04/06/2015 01:45 am
Yes, that one seems to be off but without looking at the evolution of the fields in time, one cannot determine how far off it is. You've seen my movie showing the fields in perfect resonance but not all of my runs are in precise resonance. When it is only off by a "small" amount, the fields appear to resonate for a few cycles then they collapse giving something like the image above. Then the resonance pattern re-forms. I speculate that it relates to the integral multiple of wavelengths (or half wavelengths) for the various mode indices, and the fact that I am driving the cavity (in cases where I save the field patterns) with an ideal sinusoid. Maybe I should introduce a little noise in the driving signal.

In any case, my focus is on the forces of evanescent waves and although I am sure that you know this, other readers may not so I should explain. Meep flux and force calculations in isolation don't necessarily give correct absolute values, rather the values are typically relative to something. When evaluating values from cavity models, I always normalize the answers to something else, usually 1/c. Because I normalize the force to Newtons/Watt the actual power in the cavity does not drive the solution. Of course it is important, even crucial when conducting experiments, but even the experimental data published is normalized to Newtons/Watt or mN/W or muN/W. So, modelling a plane wave striking an absorbing surface is quite meaningful. Both the flux and the force are reduced for lack of a quality factor (Q=1) but the ratio of the two is the same as always. The cavity contributes more than just a Q factor to the signal though, so it is necessary that I use the cavity particularly as it seems to aid in the creation of evanescent waves, which is my focus.
Title: Re: EM Drive Developments - related to space flight applications - Thread 2
Post by: aceshigh on 04/06/2015 02:41 am
Oh Paul... propellantless thrusters have already so many detractors and skeptics... and you mentioned a LERN power source for them? 


aside the LERN part, your posts are awesome as always.
Title: Re: EM Drive Developments - related to space flight applications - Thread 2
Post by: Star-Drive on 04/06/2015 03:59 am
Oh Paul... propellantless thrusters have already so many detractors and skeptics... and you mentioned a LERN power source for them? 


aside the LERN part, your posts are awesome as always.

Aces-High:

I don't work directly with LENR R&D, so my sources are all second hand on this topic by nature, but from what've I've been told by very mainstream sources in the field that cold fusion, AKA LENR, can not be and should not be discounted in the mid to long term.  Past that I won't mention it again in this venue.

In the meantime if we discount closed cycle nuclear fission for a space power source due to political issues, all that is left is H2 & CH4 fuel cells and/or solar photovoltaic arrays up to about 300kWe for sure, and maybe pushed up to 500kWe at Earth orbit, but no more for some time to come.   Lets hope that power range is sufficient for the needs of the first generation of exotic thrusters like the EM-Drive and its other brethren.

Best,  Paul M.
Title: Re: EM Drive Developments - related to space flight applications - Thread 2
Post by: Star-Drive on 04/06/2015 04:15 am
Dr. Rodal:

I think that the reason why the Neoprean rubber didn't generate any detectable thrust with the ~30W of available power was that it's carbon loaded which increases its RF loss tangent from that of HDPE or PTFE (~0.0004) up to 0.02 which really loads down the net E-field generated in the cavity for a given input power.  And since we think that the thrust effect we are observing is proportional to E-field squared (E^2) that could make a huge difference in performance.

Now please note that I concur with your analysis on the E&M Poynting vector time averaging to zero in the frustums.  For example when the Cannae pillbox cavity's input RF power feed is well Z-matched with a VSWR= ~1.00, the RF Poynting power flow into the cavity is always towards the large OD pancake section of the cavity.  If one envisions the quantum vacuum (Q-V) as a semi-virtual electrical plasma as Dr. White does, that would imply that the Poynting power flow vector would entrain the Q-V plasma and send it on its way toward the pillbox end of the cavity and then out of the cavity, the back-reaction on the cavity should be in the opposite direction towards the RF feed end of the Cannae test article, but the observed thrust vector is opposite to that surmise, i.e. toward the shorter RF sense antenna end of the cavity per the attached slide. 

Now Per the newly authored Q-V plasma simulation code that Dr. White just finished, the equal bidirectional Q-V plasma flow for the Cannae cavity comes from the high-Q pancake section with a Q of ~9,000 for the un-slotted version of the cavity.  However due to the high E-field region created n the throat of the RF feed, this cylindrically shaped high E-field volume acts as an obstruction to the Q-V plasma flow.  This E-field obstruction created in the PTFE cylinder then accelerates the Q-V plasma around it in a Bernoulli like effect that accelerates the Q-V plasma flow coming from the main pillbox cavity.  This unbalanced and accelerated Q-V plasma flow that goes away from the large pill box cavity in the direction of the RF input section is what generates the NET thrust in our model.   

Next, using this new Q-V plasma simulation tool that utilizes the instantaneous E&M fields from COMSOL for one complete RF cycle in 5 degree increments as its input file, we are now seeing why we need the PTFE or HDPE dielectrics in the frustum while using near pure sine wave power levels below ~100W in the ~2.0 GHz frequency range to generate detectable thrust, and why Shawyer and the Chinese didn't while pumping 80W to 2,500W using magnetron RF sources.  We think the reasons are two fold. 

The first is that Shawyer and the Chinese both used magnetron RF sources for their experiments.  An RF source that generates large AM, FM and PM modulation of the carrier wave with typical FM modulation bandwidth on the order of at least +/-20 MHz.  (These time rate to change of energy modulations increase the Q-V density in our model.) 

The second reason we found running these 3D Q-V plasma simulations for the EMPTY copper frustum, was that increasing the input power tends to focus the Q-V plasma flow from near omnidirectional from the frustum at low powers, to a much more jet like beam at higher powers measured in kW to tens of kW-rf.  In fact the simulation for the 100W run predicted only ~50uN for our pure RF system with dielectric, while the 10kW run predicted a thrust level of ~6.0 Newton without a dielectric in the cavity.  And at 100kW-rf it was now up to ~1300 Newton, but the input power to thrust production nonlinearity was starting to taper off around 50kW.   Of course these Q-V plasma thrust predictions are based on the Q-V not being immutable and non-degradable, a feature we admit is not widely accepted by the mainstream physics community, at least at the moment. :)

Lastly, due to the above non-linear thrust scaling with input power predictions, we have started the build up of a 100W-to-1,200W waveguide magnetron RF power system that will drive one of our aluminum RF frustum cavities.  Initially the test rig will follow Shawyer's first generation test rig that used a tetter-totter balance system in air only to see if we can generate similar thrust levels that Shawyer reported using a hermetic sealed box, which were in the ~16 to 300 milli-Newton range dependent on the Q-Factor of the frustum.

BTW, the reason we included the "what-if" Eagleworks can make this thing work solar system trajectory section on our 2014 JPC paper was that we have to continually tell management the value proposition for why they should fund our research, much in the same way we have to convince Chris Bergin here at NSF we really will be talking about space applications for these Q-Thruster like devices, once we get our hands around the physics they are using.  However when we do, the solar system and beyond will be ours for the picking...

Best, Paul M.

Fantastic post, Paul.  We are so lucky that you are still working on this project and that you are reporting this information.  We hope that this project, as you outlined it, gets full funding.

I also love your scientific objectivity, for example, concerning that Dr. White's simulations do not appear to abide by the assumption that the Quantum Vacuum should be indestructible and immutable.  One reason that the mainstream physics community assumes the Quantum Vacuum is indestructible and immutable is because of the experimental observation that a fundamental particle like an electron (or a positron) has the same properties (e.g. mass, charge or spin), regardless of when or where the particle was created, whether now or in the early universe, through astrophysical processes or in a laboratory.  Another reason is that the Quantum Vacuum has what Albert Einstein in 1913 called "Nullpunktsenergie" (zero point energy): the lowest possible (time-averaged) energy that a quantum mechanical physical system may have.

Godspeed ahead :)


Dr. Rodal:

The Eagleworks first paper on why Q-V longitudinal acoustical wave action rules the subatomic realm, and the epiphany that the electron is NOT unique, as a loner, or in "orbit" around a proton or protons, will be posted on the NASA servers within a few weeks or less.  And from this point on IMO, we are going to find the only "real" thing in this 5 or 6D brane universe is the quantum vacuum and the high and low "weather patterns" that manipulate it.  Meanwhile back to more mundane things, like making balky low power Q-Thrusters work well enough to get my next set of data points required by management.

Best,  Paul M.   
Title: Re: EM Drive Developments - related to space flight applications - Thread 2
Post by: aceshigh on 04/06/2015 05:18 am
Oh Paul... propellantless thrusters have already so many detractors and skeptics... and you mentioned a LERN power source for them? 


aside the LERN part, your posts are awesome as always.

Aces-High:

I don't work directly with LENR R&D, so my sources are all second hand on this topic by nature, but from what've I've been told by very mainstream sources in the field that cold fusion, AKA LENR, can not be and should not be discounted in the mid to long term.  Past that I won't mention it again in this venue.

In the meantime if we discount closed cycle nuclear fission for a space power source due to political issues, all that is left is H2 & CH4 fuel cells and/or solar photovoltaic arrays up to about 300kWe for sure, and maybe pushed up to 500kWe at Earth orbit, but no more for some time to come.   Lets hope that power range is sufficient for the needs of the first generation of exotic thrusters like the EM-Drive and its other brethren.

Best,  Paul M.

Paul, I do not discount LERN in the mid to long term (nor do I count on it, I am just skeptical and will wait). It´s just that you know that propellantless propulsion and other stuff is already seen a bit like fringe science, with many doubters and negative skeptics who will call the work you guys do at Eagleworks as crackpottery... IF you add LERN to the picture (in a powerpoint, or even here at NSF, since we know that Google finds it all :)), it will only create more problems for Eagleworks and it´s science.

So imho, best to not even mention LERN since it´s not even necessary to mention it. Better to use a hot fusion reactor in the examples... so many projects to choose from, some lightweight, etc. Shielding problems for spacecraft reactors, then are only a question of engineering, not of physics like LERN.

Just my two cents
Title: Re: EM Drive Developments - related to space flight applications - Thread 2
Post by: aceshigh on 04/06/2015 05:26 am
In the meantime if we discount closed cycle nuclear fission for a space power source due to political issues, all that is left is H2 & CH4 fuel cells and/or solar photovoltaic arrays up to about 300kWe for sure, and maybe pushed up to 500kWe at Earth orbit, but no more for some time to come.   Lets hope that power range is sufficient for the needs of the first generation of exotic thrusters like the EM-Drive and its other brethren.

Best,  Paul M.

if you are counting on LERN mid and long term... why not count on hot fusion at the mid and long term as a power source for EM-Drive? Polywell, General Fusion, Skunkworks, etc.

All are mid to long term, should be light and compact enough to be used on spacecraft, are more engineering problems than physics problems (I mean, we KNOW fusion happens, we know how to make it happen, we just are trying to win over problems like how to get more output power than input power and similar hurdles).

LERN on the other hand has not even be proved to be real yet nor there are valid and accepted theories for why it should happen.

Therefore, it would be much more elegant to include in any EM Thruster paper, hot compact fusion as a power source... more like a "make your bet on WHICH fusion proposal will work and have the specs we need"...
Title: Re: EM Drive Developments - related to space flight applications - Thread 2
Post by: DIYFAN on 04/06/2015 05:48 am
LENR
Probably beyond the speculation tolerance of NSF :(  ...

I think Paul's observation is apropos and quite relevant to space flight applications, and should be within the speculation tolerance of NSF.  NASA has a long history of researching LENR even to the point of experimentation and prototypes.  Multiple well-known universities in the U.S. currently have fully-funded LENR research programs.  A well-known Japanese university just funded a LENR research program along with some industry giants from that country.  Airbus has a keen interest and research program in LENR.  Boeing--same.  Rest assured, many if not most governmental and private entities with an interest in space flight have active LENR research programs.  All of which ironically makes LENR seem less speculative in a sense than the EM Drive itself.
Title: Re: EM Drive Developments - related to space flight applications - Thread 2
Post by: aceshigh on 04/06/2015 06:44 am
LENR
Probably beyond the speculation tolerance of NSF :(  ...

I think Paul's observation is apropos and quite relevant to space flight applications, and should be within the speculation tolerance of NSF.  NASA has a long history of researching LENR even to the point of experimentation and prototypes.  Multiple well-known universities in the U.S. currently have fully-funded LENR research programs.  A well-known Japanese university just funded a LENR research program along with some industry giants from that country.  Airbus has a keen interest and research program in LENR.  Boeing--same.  Rest assured, many if not most governmental and private entities with an interest in space flight have active LENR research programs.  All of which ironically makes LENR seem less speculative in a sense than the EM Drive itself.

both are somewhat considered fringe science, so associating one with the other makes it über-fringe, which can make it even more difficult to get funding, support, etc
Title: Re: EM Drive Developments - related to space flight applications - Thread 2
Post by: Stormbringer on 04/06/2015 09:30 am


if you are counting on LERN mid and long term... why not count on hot fusion at the mid and long term as a power source for EM-Drive? Polywell, General Fusion, Skunkworks, etc.

All are mid to long term, should be light and compact enough to be used on spacecraft, are more engineering problems than physics problems (I mean, we KNOW fusion happens, we know how to make it happen, we just are trying to win over problems like how to get more output power than input power and similar hurdles).

LERN on the other hand has not even be proved to be real yet nor there are valid and accepted theories for why it should happen.

Therefore, it would be much more elegant to include in any EM Thruster paper, hot compact fusion as a power source... more like a "make your bet on WHICH fusion proposal will work and have the specs we need"...
Midterm is 5 to ten years? (LM says they're going to knock it out in that timeframe.) That's not my definition of medium to long term. :) That's short term to me. Short term as in:  "OMG! We're late designing a hull to put it in already."
Title: Re: EM Drive Developments - related to space flight applications - Thread 2
Post by: Star-Drive on 04/06/2015 12:25 pm
Aces High:

The hot fusion candidates you mentioned all have one major problem.  That being that except for D-T fusion, no hot fusion scheme to date has ever demonstrated breakeven let alone enough energy gain over its input to make it a viable aerospace fusion reactor candidate.  (I believe that the UK D-T fusion reactor may have reached breakeven for a few minute run, but Lord, look at the neutron flux it generates, which is ~70% of a pure fission reactor for a given output power, a pure fission reactor that can be much smaller and lighter than any tokomak reactor can ever be.)  And after seeing what happened during the poly-well fusion reactor saga, I have great doubts as to the claims by any of the current crop of alternate fusion reactor concepts, L-M's optimistic claims about their fusion reactor design concept not withstanding. 

Best, Paul M.
Title: Re: EM Drive Developments - related to space flight applications - Thread 2
Post by: flux_capacitor on 04/06/2015 12:44 pm
Aces High:

The hot fusion candidates you mentioned all have one major problem.  That being that except for D-T fusion, no hot fusion scheme to date has ever demonstrated breakeven let alone enough energy gain over its input to make it a viable aerospace fusion reactor candidate.  (I believe that the UK D-T fusion reactor may have reached breakeven for a few minute run, but Lord, look at the neutron flux it generates, which is ~70% of a pure fission reactor for a given output power, a pure fission reactor that can be much smaller and lighter than any tokomak reactor can ever be.)  And after seeing what happened during the poly-well fusion reactor saga, I have great doubts as to the claims by any of the current crop of alternate fusion reactor concepts, L-M's optimistic claims about their fusion reactor design concept not withstanding. 

Best, Paul M.

You're mentioning the Joint European Torus (JET) tokamak. But there are other possible paths for controlled fusion other than tokamaks. Besides Polywell and Lockheed-Martin reactors you've just cited, maybe another scheme could reach breakeven "soon" among the aneutronic p-B11 Dense Plasma Focus fusion from LPP, the (also aneutronic) Colliding Beam Fusion Reactor from Tri Alpha Energy, the colliding FRC Fusion Engine from Helion Energy, the MTF compressor from General Fusion, the MagLIF z-pinch from Sandia National Labs… None of them are based on tokamaks. Whatever, finding an appropriate energy source for the spaceship before proving the EmDrive does work is an interesting thought experiment (and perhaps mandatory for management) but it is a bit putting the cart before the horse in my opinion.
Title: Re: EM Drive Developments - related to space flight applications - Thread 2
Post by: JasonAW3 on 04/06/2015 12:53 pm
Crazy, possibly stupid question;

     Has anyone ever considered a more direct approace to fusion?  Instead of trying to heat a Deuterium / Tritium mix, why not do a set directed streams of high velocity Tritium into a target point, using the equivelent of a particle accelerator?  There has been success at single atom streams at cryogenic tempertures for directing streams of atoms, but I've never read anywhere that anyone has tried this approach.
Title: Re: EM Drive Developments - related to space flight applications - Thread 2
Post by: QuantumG on 04/06/2015 01:01 pm
Start a new thread.. try to make it space relevant.
Title: Re: EM Drive Developments - related to space flight applications - Thread 2
Post by: flux_capacitor on 04/06/2015 01:02 pm
Crazy, possibly stupid question;

     Has anyone ever considered a more direct approace to fusion?  Instead of trying to heat a Deuterium / Tritium mix, why not do a set directed streams of high velocity Tritium into a target point, using the equivelent of a particle accelerator?  There has been success at single atom streams at cryogenic tempertures for directing streams of atoms, but I've never read anywhere that anyone has tried this approach.
It has been tried and it was successful. Even aneutronic fusion reactions have been achieved with lasers in 2005 (http://fire.pppl.gov/fusion_lasers_nature_082605.pdf). But how do you extract energy, and how much if you can? As Paul said, the problem is not producing fusion reactions, because we already do know how. It's extracting more power than what has been injected into the reactor to produce those fusion reactions. This is the concept of break-even applied to nuclear fusion. Wikipedia (http://en.wikipedia.org/wiki/Break-even) has a good explanation of the concept.

[EDIT]:
Start a new thread.. try to make it space relevant.
What QuantumG said. I'm sorry for being part of the digression. Let's focus on the EmDrive and its space-related flight applications.
Title: Re: EM Drive Developments - related to space flight applications - Thread 2
Post by: Rodal on 04/06/2015 01:10 pm
Modeling a plane wave striking an absorbing surface is a different problem than modeling the standing waves in a resonating cavity like the EM Drive.

I was referring to images like these ones, that you posted some time ago:

(http://forum.nasaspaceflight.com/index.php?action=dlattach;topic=36313.0;attach=705861;image)

The electromagnetic fields in that image don't look like the electromagnetic fields in the EM Drive, indicating that there was something wrong with your model of the EM Drive.  Since electromagnetic forces are due to the electromagnetic fields, if one doesn't get the electromagnetic fields correctly, then it stands to reason that the calculations of electromagnetic forces in the EM Drive cannot be correct either.

Wasn't that particular simulation modeling the EM Drive without openings, which in turn showed us how important the openings are to the overall operation of the drive (at least in the simulation)?
If a numerical model cannot get the electromagnetic fields correctly for the EM Drive in that simpler case, one cannot derive from such a model valid conclusions on the importance of "the openings are to the overall operation of the drive" or to the validity of forces that are based on derivatives of the fields.

One should not extrapolate that a two-dimensional (2D) model of the EM Drive that ignores its circular cross-section and three-dimensional field effects can be used to show valid conclusions on the importance of "the openings are to the overall operation of the drive" of the physical three-dimensional (3D) EM Drive.

A flat 2D model like this one, completely ignores for example the main electromagnetic field in the TM (transverse magnetic) 212 mode presently being tested by NASA Eagleworks.  The transverse magnetic field occurs out of the plane shown in the figure, in the azimuthal direction and it has 2 full wave patterns in that circumferential direction.  None of this is being modeled by this 2D Finite Difference simulation.

Title: Re: EM Drive Developments - related to space flight applications - Thread 2
Post by: Rodal on 04/06/2015 01:58 pm
Modeling a plane wave striking an absorbing surface is a different problem than modeling the standing waves in a resonating cavity like the EM Drive.

I was referring to images like these ones, that you posted some time ago:

(http://forum.nasaspaceflight.com/index.php?action=dlattach;topic=36313.0;attach=705861;image)

The electromagnetic fields in that image don't look like the electromagnetic fields in the EM Drive, indicating that there was something wrong with your model of the EM Drive.  Since electromagnetic forces are due to the electromagnetic fields, if one doesn't get the electromagnetic fields correctly, then it stands to reason that the calculations of electromagnetic forces in the EM Drive cannot be correct either.

Wasn't that particular simulation modeling the EM Drive without openings, which in turn showed us how important the openings are to the overall operation of the drive (at least in the simulation)?
If a numerical model cannot get the electromagnetic fields correctly for the EM Drive in that simpler case, one cannot derive from such a model valid conclusions on the importance of "the openings are to the overall operation of the drive" or to the validity of forces that are based on derivatives of the fields.

One should not extrapolate that a two-dimensional (2D) model of the EM Drive that ignores its circular cross-section and three-dimensional field effects can be used to show valid conclusions on the importance of "the openings are to the overall operation of the drive" of the physical three-dimensional (3D) EM Drive.

A flat 2D model like this one, completely ignores for example the main electromagnetic field in the TM (transverse magnetic) 212 mode presently being tested by NASA Eagleworks.  The transverse magnetic field occurs out of the plane shown in the figure, in the azimuthal direction and it has 2 full wave patterns in that circumferential direction.  None of this is being modeled by this 2D Finite Difference simulation.
I had asked (at a time that I didn't know that this was a 2D instead of a 3D model) whether the figure represented a component of (or the norm of) the electric or the magnetic field.

Do you have an intuition for what this 2D model represents?  For cyl. transverse magnetic (TM) modes the magnetic field is a directionless scalar.  For cyl. transverse electric (TE) modes the electric field is a directionless scalar.

In such a two dimensional model, where the magnetic field component direction and variation in the circumferential (azimuthal) direction is completely ignored,  what do you have for the magnetic fields?  The magnetic fields become just a scalar, the magnetic field is not longer a vector.   

In the real 3D EM Drive cavity the standing waves are such that the electric and magnetic fields are out of phase by 90 degrees, and therefore the energy is transferred from the electric field to the magnetic field back and forth.  What happens in this 2D model? The electric field energy from the electric vector field gets transferred into a scalar magnetic field ?

In 3D we have dot products and cross products. With a scalar magnetic field in 2D, there's no curl or divergence. We only have a gradient.  The "curl" of the electric field is a scalar in such a model.

The equation that says magnetic field lines form closed loops which never begin or end- the divergence of the magnetic field is always zero - (http://farside.ph.utexas.edu/teaching/302l/lectures/node78.html) vanishes. Hence magnetic monopoles exist in this 2D electromagnetic Flatland?
 

Can you show how the magnetic scalar field in this 2D model relates to the transverse magnetic field in the cyl. TM212 mode being tested at NASA?

How do the "two-dimensional forces" in such a 2D flatland relate to our 3D problem?



Some good-nature humor to make us think about the differences between flatland and our 3D space  :) :

(http://imgs.xkcd.com/comics/flatland.png)

(http://theuniversesolved.files.wordpress.com/2013/02/flatland2.gif)
Title: Re: EM Drive Developments - related to space flight applications - Thread 2
Post by: aceshigh on 04/06/2015 02:50 pm
What QuantumG said. I'm sorry for being part of the digression. Let's focus on the EmDrive and its space-related flight applications.

not sure it was a digression, while it´s related on how to power EMDrive in spaceflight related applications.


it´s just like VASIMR thread... the power source is an important part of the thread.

but I agree that it´s futile to discuss it, it´s putting the cart in front of the horse (or a better analogy... cart is spaceship, horse is EM Drive and the carrot is the power source.)

Title: Re: EM Drive Developments - related to space flight applications - Thread 2
Post by: spacenut on 04/06/2015 04:09 pm
Wouldn't argon be a better fuel for large EM drives, like for transfers to Mars with a lot of tonnage of equipment?  Say an in space only craft that would transfer cargo to Mars flyby, let the cargo have a chemical powered craft that would land equipment and the EM craft flys back to Earth say to L1 OR L2 to pick up more cargo?  Xenon is rare in comparison to argon.  If the craft were large enough, it could refuel at L1 with argon plus pick up or return a Mars cargo lander.  If the transfer time was fast enough, humans could also be transferred. 

All that being said, with todays technology, how large would the solar panels be to say transfer 100 tons of cargo to and from Mars?  Would it be better to go chemical all the way? 
Title: Re: EM Drive Developments - related to space flight applications - Thread 2
Post by: aero on 04/06/2015 04:42 pm
@Rodal -
Are you telling me that an effect that shows in 2D  may not show in 3D, or that an effect that exists in 3D may not show in 2D? Or both?

I would prefer to have solutions in 3D myself, but I'm not going to get them with this computer system, the meep EM thruster cavity model is to large. Its not only long run times, but also memory requirements and even hard disk storage. I had one run bomb out because I ran out of disk space while writing the output files at the end of the run.

I am forced to operate on the hope and expectation that I will be able to confirm an effect that shows in 2D by making a 3D run targeted to that effect. If I can't do that, then I can't, but I certainly can not explore the parameter space by making 3D runs. 
Title: Re: EM Drive Developments - related to space flight applications - Thread 2
Post by: Left Field on 04/06/2015 04:49 pm
Wouldn't argon be a better fuel for large EM drives, like for transfers to Mars with a lot of tonnage of equipment?  Say an in space only craft that would transfer cargo to Mars flyby, let the cargo have a chemical powered craft that would land equipment and the EM craft flys back to Earth say to L1 OR L2 to pick up more cargo?  Xenon is rare in comparison to argon.  If the craft were large enough, it could refuel at L1 with argon plus pick up or return a Mars cargo lander.  If the transfer time was fast enough, humans could also be transferred. 

All that being said, with todays technology, how large would the solar panels be to say transfer 100 tons of cargo to and from Mars?  Would it be better to go chemical all the way?

The point of this EmDrive is that it does not require a reaction mass, so no xenon nor any other gas would be used. In short, it is not an ion or a magnetoplasma drive. If you would like to learn more then here is the wiki page for starters:

http://en.wikipedia.org/wiki/EmDrive
Title: Re: EM Drive Developments - related to space flight applications - Thread 2
Post by: seggybop on 04/06/2015 04:57 pm
I would prefer to have solutions in 3D myself, but I'm not going to get them with this computer system, the meep EM thruster cavity model is to large. Its not only long run times, but also memory requirements and even hard disk storage. I had one run bomb out because I ran out of disk space while writing the output files at the end of the run

I'm familiar with these 2D vs 3D issues and I agree 3D is necessary.
I or others here with access to more substantial hardware can probably run said simulations for you, provided you can package up the software for us.
Title: Re: EM Drive Developments - related to space flight applications - Thread 2
Post by: birchoff on 04/06/2015 04:59 pm
Aces High:

The hot fusion candidates you mentioned all have one major problem.  That being that except for D-T fusion, no hot fusion scheme to date has ever demonstrated breakeven let alone enough energy gain over its input to make it a viable aerospace fusion reactor candidate.  (I believe that the UK D-T fusion reactor may have reached breakeven for a few minute run, but Lord, look at the neutron flux it generates, which is ~70% of a pure fission reactor for a given output power, a pure fission reactor that can be much smaller and lighter than any tokomak reactor can ever be.)  And after seeing what happened during the poly-well fusion reactor saga, I have great doubts as to the claims by any of the current crop of alternate fusion reactor concepts, L-M's optimistic claims about their fusion reactor design concept not withstanding. 

Best, Paul M.

You're mentioning the Joint European Torus (JET) tokamak. But there are other possible paths for controlled fusion other than tokamaks. Besides Polywell and Lockheed-Martin reactors you've just cited, maybe another scheme could reach breakeven "soon" among the aneutronic p-B11 Dense Plasma Focus fusion from LPP, the (also aneutronic) Colliding Beam Fusion Reactor from Tri Alpha Energy, the colliding FRC Fusion Engine from Helion Energy, the MTF compressor from General Fusion, the MagLIF z-pinch from Sandia National Labs… None of them are based on tokamaks. Whatever, finding an appropriate energy source for the spaceship before proving the EmDrive does work is an interesting thought experiment (and perhaps mandatory for management) but it is a bit putting the cart before the horse in my opinion.

Does anyone actually believe that if we had a working EmDrive with the predicted thrust capabilities that we would have a problem finding power for it?

From my perspective if we have a EmDrive with the predicted performance that Paul mentioned. Then I see the following happening.

1. Develop a hybrid power system that leverages existing fuelcell technology and solar cells. This opens up exploration and permanent bases on the moon, cis-lunar mars and venus. Hell if we can increase the efficiency even more then we could potentially get permanent access to the asteroid belt.

2. Once the commercial side effects of space expansion have taken root. There i no way in this universe that anyone will be able to prevent nuclear technology in whatever form it is economically available from being used in the 2nd generation ships. At that point we should have the power needed to explore our solar system and start sending "Fast" probes to our nearest neighbors.

Hell if the Emdrive provided the ability to do lift off from earth for the right amount of energy in. I could see a variant of the space elevator idea being built. Where we beam via lasers or microwaves the energy needed to feed the EmDrives till they get to Space with their cargo. They would then dock and offload cargo and passengers.

So from where I am standing, power will not be an issue if the EmDrive is proven to deliver usable thrust. We as a species will find a way.
Title: Re: EM Drive Developments - related to space flight applications - Thread 2
Post by: Rodal on 04/06/2015 05:30 pm
@Rodal -
Are you telling me that an effect that shows in 2D  may not show in 3D, or that an effect that exists in 3D may not show in 2D? Or both?

I would prefer to have solutions in 3D myself, but I'm not going to get them with this computer system, the meep EM thruster cavity model is to large. Its not only long run times, but also memory requirements and even hard disk storage. I had one run bomb out because I ran out of disk space while writing the output files at the end of the run.

I am forced to operate on the hope and expectation that I will be able to confirm an effect that shows in 2D by making a 3D run targeted to that effect. If I can't do that, then I can't, but I certainly can not explore the parameter space by making 3D runs.
The general answer to the first question is: "both".

For an example of a non-physical effect found in a flat 2D model, I gave the example that the 2D flat Maxwell model allows for magnetic monopoles which are not allowed in 3D Maxwell's equations.  The equation that says magnetic field lines form closed loops which never begin or end- the divergence of the magnetic field is always zero - (http://farside.ph.utexas.edu/teaching/302l/lectures/node78.html) vanishes in the flat 2D model.  Just as one cannot conclude, based on a 2D model that magnetic monopoles exist in our world, one should be similarly careful about the conclusions of a flat 2D model for a real EM Drive.

For an example of a physical effect in 3D that will not show in a 2D flat model, all we have to do is consider the mode presently tested by NASA Eagleworks: cyl. TM212.  The flat 2D model not only fails to model the loop that the magnetic field makes in the circumferential (azimuthal) direction but the 2D model cannot model the variation of the magnetic field in that direction (it cannot model for cyl. TM212 the harmonic motion shape of the magnetic field in the azimuthal direction).

This will affect the calculation of the momentum of the evanescent waves, and therefore affect the calculation of the force.

I was responding to @sghill, and warning other readers of the thread that may not be aware of the flat nature of your numerical model about the issues involved with a "flat" two dimensional numerical model and therefore not to jump to conclusions based on flat 2D model.
Title: Re: EM Drive Developments - related to space flight applications - Thread 2
Post by: tchernik on 04/06/2015 06:04 pm
Does anyone actually believe that if we had a working EmDrive with the predicted thrust capabilities that we would have a problem finding power for it?

....

So from where I am standing, power will not be an issue if the EmDrive is proven to deliver usable thrust. We as a species will find a way.

I fully agree. Fretting about the energy source is a bit too much at this stage, when conclusively proving the existence of the phenomenon has yet to happen.

If this becomes a proven scientific fact, with a few more conclusive replications, finding a power source for it is just an engineering problem. That is, one based on the application of already known scientific and technical principles. We know nukes work and we have them, we know fuel cells work and we have them, the same as solar photovoltaics, etc.

Gee, if Emdrives are proven to work as H. White and Paul M. expect, we would be soon putting gasoil engines/generators to power them, and this for replacing turbofans/propellers for flying in the low atmosphere!
Title: Re: EM Drive Developments - related to space flight applications - Thread 2
Post by: Star One on 04/06/2015 06:55 pm
Does anyone actually believe that if we had a working EmDrive with the predicted thrust capabilities that we would have a problem finding power for it?

....

So from where I am standing, power will not be an issue if the EmDrive is proven to deliver usable thrust. We as a species will find a way.

I fully agree. Fretting about the energy source is a bit too much at this stage, when conclusively proving the existence of the phenomenon has yet to happen.

If this becomes a proven scientific fact, with a few more conclusive replications, finding a power source for it is just an engineering problem. That is, one based on the application of already known scientific and technical principles. We know nukes work and we have them, we know fuel cells work and we have them, the same as solar photovoltaics, etc.

Gee, if Emdrives are proven to work as H. White and Paul M. expect, we would be soon putting gasoil engines/generators to power them, and this for replacing turbofans/propellers for flying in the low atmosphere!

I believe because of their extremely contentious nature & reputation to the majority in the scientific community it's not just a case of showing EM drives work but producing a rock solid case for why they work. Unfortunately there has been so many other claims of a similar nature that have been made over the years that have proved to be nonsense that it has severely muddied the waters leaving things such as the EM drive with incredibly high hurdles to clear before anyone gives them the time of day.
Title: Re: EM Drive Developments - related to space flight applications - Thread 2
Post by: Rodal on 04/06/2015 06:56 pm
I would prefer to have solutions in 3D myself, but I'm not going to get them with this computer system, the meep EM thruster cavity model is to large. Its not only long run times, but also memory requirements and even hard disk storage. I had one run bomb out because I ran out of disk space while writing the output files at the end of the run

I'm familiar with these 2D vs 3D issues and I agree 3D is necessary.
I or others here with access to more substantial hardware can probably run said simulations for you, provided you can package up the software for us.
Even in the 2D flat model the finite difference mesh is coarse enough that one can see a fractal nature of the solution in these images:

(http://forum.nasaspaceflight.com/index.php?action=dlattach;topic=36313.0;attach=705861;image)

(http://forum.nasaspaceflight.com/index.php?action=dlattach;topic=36313.0;attach=705863;image)

compare this fractal looking images of the finite difference solution model with the continuous, much smoother solution provided by the COMSOL Finite Element solution:

(http://forum.nasaspaceflight.com/index.php?action=dlattach;topic=36313.0;attach=635196;image)

The fractal nature of the electromagnetic fields inside the EM Drive look wrong.

This fractal nature is an artifact of the finite difference mesh discretization.  The electromagnetic fields are known to be continuous and not fractal.  The forces due to evanescent waves are primarily due to the gradient of the electromagnetic fields.   Due to the discontinuous fractal nature of the finite difference discretization coarse mesh, the gradient of the electromagnetic fields, and therefore the computed force is going to be significantly more inaccurate.  A 3D finite difference model would encounter severe mesh discretization issues.

If there are problems modeling the standing waves in the EM Drive by this finite difference model, just imagine the challenge to model evanescent waves with the numerical solution method. The evanescent wave is a small-scale feature which rapidly decays away from a material interface.

To compute the force, one needs to compute the gradient of the electromagnetic field.  Due to the exponential nature of the decay, significant numerical errors may occur when calculating the gradient, and hence when calculating the force.

It can be shown that the decay in the direction perpendicular to the interface is exponential and thus can lead to a very high gradient of the evanescent wave solution in that direction. In the direction along the interface, the evanescent solution oscillates.

(http://upload.wikimedia.org/wikipedia/commons/thumb/1/16/Electron_density_wave_-_plasmon_excitations.png/350px-Electron_density_wave_-_plasmon_excitations.png)

(http://qph.is.quoracdn.net/main-qimg-6238097c8770d0045c1beb352daa135b?convert_to_webp=true)

(http://www.tau.ac.il/~applphys/research_fews_evan_wave.jpg)

(http://worldstartech.com/uploads/articles/20131220163344_image_Refractive_index_of_thin,_aqueous_films_between_hydrophobic_surfaces_studied_using_evanescent_wave_atomic_force_microscopy-laser_diode_module.gif)

Thus, capturing in a numerical computation the evanescent wave may require a fine mesh along the interface, even when the wave number is low enough that a coarser mesh suffices for other purposes.
Title: Re: EM Drive Developments - related to space flight applications - Thread 2
Post by: birchoff on 04/06/2015 08:35 pm
Does anyone actually believe that if we had a working EmDrive with the predicted thrust capabilities that we would have a problem finding power for it?

....

So from where I am standing, power will not be an issue if the EmDrive is proven to deliver usable thrust. We as a species will find a way.

I fully agree. Fretting about the energy source is a bit too much at this stage, when conclusively proving the existence of the phenomenon has yet to happen.

If this becomes a proven scientific fact, with a few more conclusive replications, finding a power source for it is just an engineering problem. That is, one based on the application of already known scientific and technical principles. We know nukes work and we have them, we know fuel cells work and we have them, the same as solar photovoltaics, etc.

Gee, if Emdrives are proven to work as H. White and Paul M. expect, we would be soon putting gasoil engines/generators to power them, and this for replacing turbofans/propellers for flying in the low atmosphere!

I believe because of their extremely contentious nature & reputation to the majority in the scientific community it's not just a case of showing EM drives work but producing a rock solid case for why they work. Unfortunately there has been so many other claims of a similar nature that have been made over the years that have proved to be nonsense that it has severely muddied the waters leaving things such as the EM drive with incredibly high hurdles to clear before anyone gives them the time of day.

I somewhat agree with you. Mainly because I do not believe that a proven (not necessarily accepted) theory of how EmDrive's work is a show stopper. If someone gave us a blackbox space rated emdrive thruster were all we had to do was plug it in to an electrical energy source. I doubt any lay person or physicist would doubt that the drive works. Which means that the only reason we need an accepted theory is because it will increase the availability of funding. That said, if Eagleworks builds a replication kit that works every time I would argue that you can get the same increase in funding availability. So there are two avenues one can pursue; one can build a working, easily replicated experiment. And or they can work on developing a proven theory and work on getting it accepted.
Title: Re: EM Drive Developments - related to space flight applications - Thread 2
Post by: birchoff on 04/06/2015 08:37 pm
Given the figures the predicted thrust that paul march provided us. Anyone mind educating this lay person on how a single EmDrive thruster would perform vs our current rockets.
Title: Re: EM Drive Developments - related to space flight applications - Thread 2
Post by: Chrochne on 04/06/2015 08:55 pm
Does anyone actually believe that if we had a working EmDrive with the predicted thrust capabilities that we would have a problem finding power for it?

....

So from where I am standing, power will not be an issue if the EmDrive is proven to deliver usable thrust. We as a species will find a way.

I fully agree. Fretting about the energy source is a bit too much at this stage, when conclusively proving the existence of the phenomenon has yet to happen.

If this becomes a proven scientific fact, with a few more conclusive replications, finding a power source for it is just an engineering problem. That is, one based on the application of already known scientific and technical principles. We know nukes work and we have them, we know fuel cells work and we have them, the same as solar photovoltaics, etc.

Gee, if Emdrives are proven to work as H. White and Paul M. expect, we would be soon putting gasoil engines/generators to power them, and this for replacing turbofans/propellers for flying in the low atmosphere!


I fully agree with you. Especially on the last part. If it will do, what it looks like it may, we stand on the verge of unprecedented change to the world. And of course not only that. It is a dream for many to reach Europa (moon) and dig in. We could even make a base there.

Of course for us down here on Earth it would mean revolution in the transportation as Roger Shawyer predicted. It is even more far reaching than that. We would not be discussing this here, if not for this man. He was pushing for this for decades and the critics were just demolishing each move he done. I am so glad he did not give up.

Mr. Shawyer, if you are reading this I want to say thank you for not giving up.

What will results show I leave to brilliant minds of scientists and engineers here, but I am glad for this great debate. :)
Title: Re: EM Drive Developments - related to space flight applications - Thread 2
Post by: Abyss on 04/06/2015 09:17 pm
When doing EM simulations you can model the transient propagation of radiation as it interacts with your model, which can be valuable to see exactly how it reflects and interacts with various features (like a groove, slit, etc).  In this case you model an incoming pulse, composed of many frequencies.

Another common simulation is to do a frequency domain simulation in which you compute the steady state solution of inputting a single frequency into the model.  But you have to be careful in how you interpret these results, because they are non intuitive, e.g. they don't show the propagation of radiation.

I know you can perform both types of simulation using COMSOL and CST.

I have a few questions that perhaps the emdrive experts here can answer.  Why don't we use much smaller cavities? I believe they would be easier to machine, and experimentally work with.  This would require scaling the wavelength, but what is the problem with that?
Title: Re: EM Drive Developments - related to space flight applications - Thread 2
Post by: Star One on 04/06/2015 09:37 pm
One thing that has mystified me about this whole business is it not the fact that this effect has been known about for a number of years so why is it only in recent years that people have started looking into it?
Title: Re: EM Drive Developments - related to space flight applications - Thread 2
Post by: Abyss on 04/06/2015 09:46 pm
One thing that has mystified me about this whole business is it not the fact that this effect has been known about for a number of years so why is it only in recent years that people have started looking into it?

It's not clear, what effect are you referring to?
Title: Re: EM Drive Developments - related to space flight applications - Thread 2
Post by: Rodal on 04/06/2015 10:30 pm
When doing EM simulations you can model the transient propagation of radiation as it interacts with your model, which can be valuable to see exactly how it reflects and interacts with various features (like a groove, slit, etc).  In this case you model an incoming pulse, composed of many frequencies.

Another common simulation is to do a frequency domain simulation in which you compute the steady state solution of inputting a single frequency into the model.  But you have to be careful in how you interpret these results, because they are non intuitive, e.g. they don't show the propagation of radiation.

I know you can perform both types of simulation using COMSOL and CST.

I have a few questions that perhaps the emdrive experts here can answer.  Why don't we use much smaller cavities? I believe they would be easier to machine, and experimentally work with.  This would require scaling the wavelength, but what is the problem with that?
Welcome to the forum  :)

The smaller the cavity, the higher the natural frequencies.  Shawyer's and NASA Eagleworks first truncated cones with flat ends were literally "home made" from thin copper sheets.  Using a conventional magnetron used for microwave ovens (2.45 GHz) the size for Shawyer's (the first one to do these experiments) was mainly dictated by the frequency of the lowest natural frequencies. (One should take into account the cut-off frequency condition).

One of the main contributors to this thread (@Notsosureofit) has posted that he plans to run his own tests with a significantly smaller EM Drive geometry using a Gunn diode to excite the microwaves at a higher frequency (using the X band (8 to 12 GHz) ), as used in the older radar speed guns used by the highway patrolmen  :)
Title: Re: EM Drive Developments - related to space flight applications - Thread 2
Post by: aceshigh on 04/06/2015 10:47 pm
Does anyone actually believe that if we had a working EmDrive with the predicted thrust capabilities that we would have a problem finding power for it?

....

So from where I am standing, power will not be an issue if the EmDrive is proven to deliver usable thrust. We as a species will find a way.

I fully agree. Fretting about the energy source is a bit too much at this stage, when conclusively proving the existence of the phenomenon has yet to happen.

If this becomes a proven scientific fact, with a few more conclusive replications, finding a power source for it is just an engineering problem. That is, one based on the application of already known scientific and technical principles. We know nukes work and we have them, we know fuel cells work and we have them, the same as solar photovoltaics, etc.

Gee, if Emdrives are proven to work as H. White and Paul M. expect, we would be soon putting gasoil engines/generators to power them, and this for replacing turbofans/propellers for flying in the low atmosphere!

I believe because of their extremely contentious nature & reputation to the majority in the scientific community it's not just a case of showing EM drives work but producing a rock solid case for why they work. Unfortunately there has been so many other claims of a similar nature that have been made over the years that have proved to be nonsense that it has severely muddied the waters leaving things such as the EM drive with incredibly high hurdles to clear before anyone gives them the time of day.

which is why I replied to Paul on the issue of power source when he mentioned LERN.

the issue didn´t really started as a discussing about what´s the best power source... I don´t really care what will the power source, nor am I interested in discussing here which power producing method has the most potential, whatever.


the issue I had here was how the scientific community looks at the EM Drive and the Eagleworks Lab experiments,which ARE fringe, and how will they look at what is already considered fringe physics if powerpoint presentations include LERN stuff...

Am I that wrong in those fears?

Given the figures the predicted thrust that paul march provided us. Anyone mind educating this lay person on how a single EmDrive thruster would perform vs our current rockets.


well, it´s about 1250 kN is about two times the thrust of a Merlin 1D engine from the Falcon 9...

he also mentioned 1 million of ISP... I guess that´s like having the power of a Merlin 1D engine consuming the same fuel per second as an electric bycicle??


I GUESS a single one would be enough to counter gravity and take a Falcon 9 rocket to space. However, because of the Falcon 9 weight and the thrust equivalent of only 2 merlin engines, it would probably take longer to get to orbit (which as you know is a matter of speed, not height).

But considering the ISP of 1 million (so little fuel being consumed per second) it HAS THE TIME to accelerate to orbital speed in the vacuum, while a normal rocket doesn´t have it (fuel ends fast, if it hasnt reached orbital speed, it falls).

I guess you could use it at like 45 degrees angle... half of thrust to keep it in space vacuum while the other half is horizontal speed to gather orbital speed?


Once in orbit... a single motor with 1250 N of thrust and 1 million of ISP can do wonders. Anyone wants to calculate the acceleration possible with such motor and a ship the weight of a fully loaded Falcon 9? (after all, he asked how it compared to current rockets)

Title: Re: EM Drive Developments - related to space flight applications - Thread 2
Post by: tchernik on 04/06/2015 10:55 pm

which is why I replied to Paul on the issue of power source when he mentioned LERN.

the issue didn´t really started as a discussing about what´s the best power source... I don´t really care what will the power source, nor am I interested in discussing here which power producing method has the most potential, whatever.


the issue I had here was how the scientific community looks at the EM Drive and the Eagleworks Lab experiments,which ARE fringe, and how will they look at what is already considered fringe physics if powerpoint presentations include LERN stuff...

Am I that wrong in those fears?


IMHO, no, you are right. When trying to defend yourself and your ideas from strong criticism, don't become an even bigger target. Not even by making the innocent mistake of passingly referencing another controversial topic in your slides.

Focus on the pill you want your public to swallow. And not on any other.
Title: Re: EM Drive Developments - related to space flight applications - Thread 2
Post by: Star One on 04/06/2015 11:44 pm
One thing that has mystified me about this whole business is it not the fact that this effect has been known about for a number of years so why is it only in recent years that people have started looking into it?

It's not clear, what effect are you referring to?
Microwave communications on satellites wasn't it first noted.
Title: Re: EM Drive Developments - related to space flight applications - Thread 2
Post by: ThinkerX on 04/07/2015 12:43 am
Quote
Microwave communications on satellites wasn't it first noted.

From what I've read, yes.

Shawyer was investigating why certain satellites were burning through their fuel far faster than expected.  He concluded that microwave emissions from devices on board these satellites was responsible, and from there began looking into this as a means of thrust.   
Title: Re: EM Drive Developments - related to space flight applications - Thread 2
Post by: Rodal on 04/07/2015 01:15 am
One thing that has mystified me about this whole business is it not the fact that this effect has been known about for a number of years so why is it only in recent years that people have started looking into it?

It's not clear, what effect are you referring to?
Microwave communications on satellites wasn't it first noted.

Quote
Microwave communications on satellites wasn't it first noted.

From what I've read, yes.

Shawyer was investigating why certain satellites were burning through their fuel far faster than expected.  He concluded that microwave emissions from devices on board these satellites was responsible, and from there began looking into this as a means of thrust.   

The microwave communications are possible because microwaves are transmitted from (and to) the satellite.  The microwaves obviously are leaving  (or entering) the satellite.  If there is any propulsion resulting from communications from the satellite, it may not violate the laws of conservation of momentum and conservation of energy, because the microwave photons are leaving the sateliite.  It would work like a photon rocket for microwaves leaving the satellite (or like a solar sail for microwave photons hitting the satellite).  Scientists have no problems with the concept of a photon rocket (except that it is one the least useful forms of space propulsion: one can also use a military searchlight as a photon rocket).

Shawyer's EM Drive is completely different: it is a completely enclosed cavity.  The microwaves inside it are standing waves.  No microwaves leave the EM Drive.  No photons leave the EM Drive (certainly not on purpose). That's why scientists think that the EM Drive should not be able to have any propulsion: because it appears to violate conservation of momentum. 

The standing microwaves inside an EM Drive cannot be used for communication purposes because they stay inside the EM Drive: the metal acts like a Faraday cage.

So what is mystifying, if indeed Shaywer was inspired by this issue, is whether Shawyer did not understand the difference between a completely enclosed microwave cavity and a communications satellite transmitting microwaves or whether the story that he was inspired by this is not really accurate.

It would be like somebody saying that they invented a new form of space propulsion based on a flashlight inside a a completely sealed box with mirrors on every inside surface, and claiming that they were inspired by noticing that a satellite in space flashing a light for communications purposes, also acts like a photon rocket. 

Yes, a flashlight in space will act like a photon rocket, but if you put the flaslight inside a sealed box so that no light leaves the box, it is just a box sitting in space: it is no longer good for communications and it is no longer good for propulsion.  :)



(http://www.conniq.com/images/PHY_satellite.gif)

(http://www.wired.com/images_blogs/photos/uncategorized/2008/09/24/emdrive_2.jpg)

(http://www.bitrebels.com/wp-content/uploads/2013/03/emergency-flash-light-translator-2.jpg)

(http://www.projectrho.com/public_html/rocket/images/spaceageposter/photonDrive5.jpg)

(http://the-gadgeteer.com/wp-content/uploads/2011/08/fenix_td70-box.jpg)
Title: Re: EM Drive Developments - related to space flight applications - Thread 2
Post by: DIYFAN on 04/07/2015 01:17 am

which is why I replied to Paul on the issue of power source when he mentioned LERN.

the issue didn´t really started as a discussing about what´s the best power source... I don´t really care what will the power source, nor am I interested in discussing here which power producing method has the most potential, whatever.


the issue I had here was how the scientific community looks at the EM Drive and the Eagleworks Lab experiments,which ARE fringe, and how will they look at what is already considered fringe physics if powerpoint presentations include LERN stuff...

Am I that wrong in those fears?


IMHO, no, you are right. When trying to defend yourself and your ideas from strong criticism, don't become an even bigger target. Not even by making the innocent mistake of passingly referencing another controversial topic in your slides.

Focus on the pill you want your public to swallow. And not on any other.

Those who are hostile to EM Drive research will be the same ones that are hostile to LENR research.  Let's not kid ourselves.  There is a growing body of evidence that LENR is not only real, but commercially useful and viable.  There is likewise a growing body of evidence in support of the EM Drive, with multiple groups in multiple countries citing remarkable results.  In each case, further replications are paramount to rule out or otherwise nullify the purported effects.  I reject the idea that we should stay silent and not mention certain words or areas of research to avoid offending the sensibilities of certain individuals.  Nor is the general public the main target audience at this juncture.  The target audience right now are space and propulsion enthusiasts, technologists, engineers, and scientists.  I think that LENR can be openly mentioned and even discussed without fears of an ominous discrediting campaign.  That campaign has already run its course.   

The scientific method is of utmost importance in these situations--otherwise we risk falling into the tantalizing trap of refusing to gaze through the telescope when the answers are before us for the looking.  Dismissing phenomena out of hand is anathema to the scientific method.  The safer and more astute approach is to maintain a skeptical but open mind until the evidence bears it out or proves it wrong.  And in the meanwhile, fear, uncertainty, and doubt of discussing LENR and its possible implications for the EM Drive should be replaced with skepticism, open-mindedness, and a willingness to role up the sleeves and try things. 
Title: Re: EM Drive Developments - related to space flight applications - Thread 2
Post by: Cinder on 04/07/2015 02:02 am
But this forum topic is not about the scientific method.  The scientific method as applies to where to go next, beyond a working EM Drive, will be topical when we have a definitive EM Drive proof of concept.  We're not there yet, and so anything beyond getting to that step is speculation on top of speculation; no?

I reject the idea that we should stay silent and not mention certain words or areas of research to avoid offending the sensibilities of certain individuals. 
...
The safer and more astute approach is to maintain a skeptical but open mind until the evidence bears it out or proves it wrong.  And in the meanwhile, fear, uncertainty, and doubt of discussing LENR and its possible implications for the EM Drive should be replaced with skepticism, open-mindedness, and a willingness to role up the sleeves and try things.
All best discussed in its own thread, I reckon.  The problem is not the sensibilities of people.  It's keeping this forum in maximum shipshape; it's a functional matter, not fashion.
Title: Re: EM Drive Developments - related to space flight applications - Thread 2
Post by: ThinkerX on 04/07/2015 03:26 am
Quote
The microwave communications are possible because microwaves are transmitted from (and to) the satellite.  The microwaves obviously are leaving  (or entering) the satellite.  If there is any propulsion resulting from communications from the satellite, it may not violate the laws of conservation of momentum and conservation of energy, because the microwave photons are leaving the sateliite.  It would work like a photon rocket for microwaves leaving the satellite (or like a solar sail for microwave photons hitting the satellite).  Scientists have no problems with the concept of a photon rocket (except that it is one the least useful forms of space propulsion: one can also use a military searchlight as a photon rocket).

Shawyer's EM Drive is completely different: it is a completely enclosed cavity.  The microwaves inside it are standing waves.  No microwaves leave the EM Drive.  No photons leave the EM Drive (certainly not on purpose). That's why scientists think that the EM Drive should not be able to have any propulsion: because it appears to violate conservation of momentum. 

The standing microwaves inside an EM Drive cannot be used for communication purposes because they stay inside the EM Drive: the metal acts like a Faraday cage.

So what is mystifying, if indeed Shaywer was inspired by this issue, is whether Shawyer did not understand the difference between a completely enclosed microwave cavity and a communications satellite transmitting microwaves or whether the story that he was inspired by this is not really accurate.

It would be like somebody saying that they invented a new form of space propulsion based on a flashlight inside a a completely sealed box with mirrors on every inside surface, and claiming that they were inspired by noticing that a satellite in space flashing a light for communications purposes, also acts like a photon rocket. 

Yes, a flashlight in space will act like a photon rocket, but if you put the flaslight inside a sealed box so that no light leaves the box, it is just a box sitting in space: it is no longer good for communications and it is no longer good for propulsion

My guess - and its no more than that - is that Shawyer concluded the 'thrust' produced by these microwave emitters was significantly greater than that of a photon rocket effect. 

Despite a fair number of internet searches, I have yet to encounter any details on Shawyer's research into this.

That said, it seems like everybody looking into this is playing catch-up with Shawyer - witness 'Star Drives' last few posts, where to me it reads like they hope to copy a EM Drive model Shawyer was working with years ago.   I have found myself wondering a few times if Shawyer's theory is correct, but just so badly presented it gives the false appearance of being wrong. 
Title: Re: EM Drive Developments - related to space flight applications - Thread 2
Post by: Star-Drive on 04/07/2015 04:09 am
Does anyone actually believe that if we had a working EmDrive with the predicted thrust capabilities that we would have a problem finding power for it?

....

So from where I am standing, power will not be an issue if the EmDrive is proven to deliver usable thrust. We as a species will find a way.

I fully agree. Fretting about the energy source is a bit too much at this stage, when conclusively proving the existence of the phenomenon has yet to happen.

If this becomes a proven scientific fact, with a few more conclusive replications, finding a power source for it is just an engineering problem. That is, one based on the application of already known scientific and technical principles. We know nukes work and we have them, we know fuel cells work and we have them, the same as solar photovoltaics, etc.

Gee, if Emdrives are proven to work as H. White and Paul M. expect, we would be soon putting gasoil engines/generators to power them, and this for replacing turbofans/propellers for flying in the low atmosphere!

I believe because of their extremely contentious nature & reputation to the majority in the scientific community it's not just a case of showing EM drives work but producing a rock solid case for why they work. Unfortunately there has been so many other claims of a similar nature that have been made over the years that have proved to be nonsense that it has severely muddied the waters leaving things such as the EM drive with incredibly high hurdles to clear before anyone gives them the time of day.

which is why I replied to Paul on the issue of power source when he mentioned LERN.

the issue didn´t really started as a discussing about what´s the best power source... I don´t really care what will the power source, nor am I interested in discussing here which power producing method has the most potential, whatever.


the issue I had here was how the scientific community looks at the EM Drive and the Eagleworks Lab experiments,which ARE fringe, and how will they look at what is already considered fringe physics if powerpoint presentations include LERN stuff...

Am I that wrong in those fears?

Given the figures the predicted thrust that paul march provided us. Anyone mind educating this lay person on how a single EmDrive thruster would perform vs our current rockets.


well, it´s about 1250 kN is about two times the thrust of a Merlin 1D engine from the Falcon 9...

he also mentioned 1 million of ISP... I guess that´s like having the power of a Merlin 1D engine consuming the same fuel per second as an electric bycicle??


I GUESS a single one would be enough to counter gravity and take a Falcon 9 rocket to space. However, because of the Falcon 9 weight and the thrust equivalent of only 2 merlin engines, it would probably take longer to get to orbit (which as you know is a matter of speed, not height).

But considering the ISP of 1 million (so little fuel being consumed per second) it HAS THE TIME to accelerate to orbital speed in the vacuum, while a normal rocket doesn´t have it (fuel ends fast, if it hasnt reached orbital speed, it falls).

I guess you could use it at like 45 degrees angle... half of thrust to keep it in space vacuum while the other half is horizontal speed to gather orbital speed?


Once in orbit... a single motor with 1250 N of thrust and 1 million of ISP can do wonders. Anyone wants to calculate the acceleration possible with such motor and a ship the weight of a fully loaded Falcon 9? (after all, he asked how it compared to current rockets)



Aces-high:

"well, it´s about 1250 kN is about two times the thrust of a Merlin 1D engine from the Falcon 9..."

Let's not get carried away here!  Our COMSOL/Q-V plasma code simulation indicated ~1,250 Newton for 100kW RF input using a water cooled version of our current copper cavity, NOT 1,250 kN, which is three orders of magnitude larger than stated in my previous post.  I'm still trying to wrap my head around this 1,250 to 2,000 Newton figure...

Best, Paul M.

Title: Re: EM Drive Developments - related to space flight applications - Thread 2
Post by: ThinkerX on 04/07/2015 05:02 am
Hmmm...

Compare Star Drives statement:

Quote
Let's not get carried away here!  Our COMSOL/Q-V plasma code simulation indicated ~1,250 Newton for 100kW RF input using a water cooled version of our current copper cavity, NOT 1,250 kN, which is three orders of magnitude larger than stated in my previous post.  I'm still trying to wrap my head around this 1,250 to 2,000 Newton figure...

With the table at the bottom of this link:


http://www.paresspacewarpresearch.org/Projet_Space_Warp/Experiment_5.htm

Star Drive:    100 watts power = 1250 - 2000 newton's
Space Warp:  185 - 205 watts power = 767 - 2303 newton's

Pretty close.  Well, sort of...

Hmmm... maybe the relevant formula's / assumptions need independent vetting?   
Title: Re: EM Drive Developments - related to space flight applications - Thread 2
Post by: aceshigh on 04/07/2015 05:34 am
Aces-high:

"well, it´s about 1250 kN is about two times the thrust of a Merlin 1D engine from the Falcon 9..."

Let's not get carried away here!  Our COMSOL/Q-V plasma code simulation indicated ~1,250 Newton for 100kW RF input using a water cooled version of our current copper cavity, NOT 1,250 kN, which is three orders of magnitude larger than stated in my previous post.  I'm still trying to wrap my head around this 1,250 to 2,000 Newton figure...

Best, Paul M.

bah, just a 3 orders of magnitude difference!

oops  :-[ :-[

well Paul, you might want to answer Birchoff's question directly?

Quote from: Birchoff
Given the figures the predicted thrust that paul march provided us. Anyone mind educating this lay person on how a single EmDrive thruster would perform vs our current rockets.
Title: Re: EM Drive Developments - related to space flight applications - Thread 2
Post by: aceshigh on 04/07/2015 05:48 am
The problem is not the sensibilities of people.

the problem is keeping Eagleworks open and well funded so it can investigate EM to the maximum extent and obviously, as a NASA facility, with the main objective of spaceflight applications.

It´s the Eagleworks investigation that keeps this thread open, because it´s NASA and because we get info about it, unlike the investigations of Shawyer and in China.

DYIFAN is probably aware that funding is not exactly only about the merits of scientific method  :(


Paul March himself mentioned weeks ago that without good data he might be retireing this past March (well, I guess that since he didn´t that is good news too)
Title: Re: EM Drive Developments - related to space flight applications - Thread 2
Post by: Cinder on 04/07/2015 07:21 am
Anyway we're beating around the bush of something pretty much everyone understands... So let's let the thread get back on track. 

I do know that March & co already published one illustrated gist of the practical implications of a working EM Drive, but for the life of me I can't find it.  I thought it was in one of these NSF threads.  WRT Birchoff's question.
Title: Re: EM Drive Developments - related to space flight applications - Thread 2
Post by: mlindner on 04/07/2015 08:48 am
It appears my previous post got deleted.

So I'll ask again. Has this psudoscience nonsense been found to be false yet? Have any proper scientists tested this contraption so that they can show that it doesn't work?

I'm sure some "proper scientists" will come round for a look eventually. In the meantime, history has a lot to teach us about these sorts of things:

http://amasci.com/weird/vindac.html
http://www.lifehack.org/articles/lifestyle/6-world-changing-ideas-that-were-originally-rejected.html
http://www.cracked.com/article_18822_5-famous-scientists-dismissed-as-morons-in-their-time.html

Every previous world changing idea didn't try to violate a founding principal of all physics, namely CoE and CoM.

Quote
Quote
...so that they can show that it doesn't work

Just curious, what makes you so certain that it doesn't work? I mean, I have no idea if it works or not. The only certain thing I know here is that I don't know for certain if it works or doesn't.

There is a growing body of evidence which suggest that it does work. It hasn't been proven by anybody that it doesn't work.

I'm still waiting for an actual test of the operation of this craft. There haven't been any non-faulty experiments done yet that actually show it producing any thrust. Namely it must be tested in a vacuum. I don't try to merge philosophy and science and currently this "EM Drive" is purely in the realm of philosophy with no actual data yet. Thus I dismiss it just like the people claiming they made an anti-gravity drive in their garage.

It's rather insulting that this forum topic even exists here.

Well I can't fault you for being skeptical, but I do urge you to review the pages of this thread. Things start getting rather interesting around page 20 or so. Many of your concerns have been addressed here, such as the vacuum testing, which has been completed with data provided, and reported here by an engineer at Eagleworks.
http://forum.nasaspaceflight.com/index.php?topic=36313.msg1326608#msg1326608

Also interesting:
http://forum.nasaspaceflight.com/index.php?topic=36313.msg1333835#msg1333835

And, in my humble opinion, there has been plenty of existing, published in reputable journals...science uncovered, which can shed light on how the EMdrive can thrust, without violating any conservation laws.
http://forum.nasaspaceflight.com/index.php?topic=36313.msg1330846#msg1330846
http://forum.nasaspaceflight.com/index.php?topic=36313.msg1333392#msg1333392

Also @Notsosureofit has developed his own very plausible ideas about how EMdrive can thrust, using good old established science. @Rodal, @Aero, and @Frobnicat have all provided exhaustive data and calculations both for and against the reports of measured thrust at Eagleworks and NWPU China.

We're openly hostile to pseudoscience here, but at the same time, we must be mindful that in order to get to the bottom of this mystery, we have to be willing to step outside of our comfort zones.

From a practical standpoint, I find it extremely unlikely that the only means mankind will ever have to propel spacecraft through the vacuum, is to carry along stores of fuel and shoot propellant out the back side.

If EMdrives aren't the answer, the lessons learned from this may lead to the answer. Once field propulsion is a reality, we can consider the solar system to be unlocked.

Now that is worth a look.

While that's all very interesting. You can immediately rule out the drive's functionality because of the lack of reaction mass. CoM is clearly broken.

I only have a minor in physics so I honestly can't understand most of what's talked about here, but I know at the least you can't go about thrusting around the universe with no particles/energy leaving the engine without something like an Alcubierre drive. Even if you're pushing against quantum virtual particle pairs, virtual particle pairs have opposite signs so will be propelled in opposite directions thus canceling out any net force.

I have yet to year a simple explanation of how this drive works that doesn't dive into theoretical physics to explain its operation. It is true of everything in physics that simple laws lead to complex behavior. If the operation of the EmDrive or other various families of the drive cannot be explained in an abstract way within a couple sentences then it doesn't work.

People are being sold snake oil and I don't like it.
Title: Re: EM Drive Developments - related to space flight applications - Thread 2
Post by: RotoSequence on 04/07/2015 09:23 am
While that's all very interesting. You can immediately rule out the drive's functionality because of the lack of reaction mass. CoM is clearly broken.

I only have a minor in physics so I honestly can't understand most of what's talked about here, but I know at the least you can't go about thrusting around the universe with no particles/energy leaving the engine without something like an Alcubierre drive. Even if you're pushing against quantum virtual particle pairs, virtual particle pairs have opposite signs so will be propelled in opposite directions thus canceling out any net force.

I have yet to year a simple explanation of how this drive works that doesn't dive into theoretical physics to explain its operation. It is true of everything in physics that simple laws lead to complex behavior. If the operation of the EmDrive or other various families of the drive cannot be explained in an abstract way within a couple sentences then it doesn't work.

People are being sold snake oil and I don't like it.

"We don't know how, why, or if it works and known physics do not readily suffice, therefore it doesn't work and its proponents are perpetuating fraud" is jumping the gun a bit, don't you think?
Title: Re: EM Drive Developments - related to space flight applications - Thread 2
Post by: mlindner on 04/07/2015 10:03 am
While that's all very interesting. You can immediately rule out the drive's functionality because of the lack of reaction mass. CoM is clearly broken.

I only have a minor in physics so I honestly can't understand most of what's talked about here, but I know at the least you can't go about thrusting around the universe with no particles/energy leaving the engine without something like an Alcubierre drive. Even if you're pushing against quantum virtual particle pairs, virtual particle pairs have opposite signs so will be propelled in opposite directions thus canceling out any net force.

I have yet to year a simple explanation of how this drive works that doesn't dive into theoretical physics to explain its operation. It is true of everything in physics that simple laws lead to complex behavior. If the operation of the EmDrive or other various families of the drive cannot be explained in an abstract way within a couple sentences then it doesn't work.

People are being sold snake oil and I don't like it.

"We don't know how, why, or if it works and known physics do not readily suffice, therefore it doesn't work and its proponents are perpetuating fraud" is jumping the gun a bit, don't you think?

Tornados don't produces magical instruments. They produce junk. Luck doesn't make engineering.

The chances of someone stumbling upon some contraption that violates physics principles by throwing electronics parts out of his garage together is nil.
Title: Re: EM Drive Developments - related to space flight applications - Thread 2
Post by: QuantumG on 04/07/2015 10:09 am
Tornados don't produces magical instruments. They produce junk. Luck doesn't make engineering.

The chances of someone stumbling upon some contraption by throwing electronics parts out of his garage together is nil.

This Michael Faraday guy is just a shyster, he doesn't even know calculus!
Title: Re: EM Drive Developments - related to space flight applications - Thread 2
Post by: CW on 04/07/2015 11:34 am
How do rockets work? They push out hot gas - the rocket goes direction x, the gas direction -x .

How does the hypothesized Q-thruster work? It pushes out QV particle 'plasma' - the Q-thruster goes direction x, the QV particle 'plasma' direciton -x .

What's the difference? In case of a rocket, we can still 'see' the hot gas. In case of the Q-thruster, we cannot. Philosophically speaking, it does not matter if you use QV particle 'plasma' or 'real' particles. Both types can be considered a subset of what 'quantum vacuum' is - namely the superset of all that can possibly exist.

It's just interesting to think about CoM. I think that the most likely reaction would be that in direction of acceleration, the Q-thruster should produce a sort of 'suction effect', and a repelling effect on the opposite side. Reason being that when the virtual plasma particle pairs vanish again, the impeded momentum must be conserved still in some form. I can only imagine this as a unidirectional 'gravity'-like effect, or maybe an effect similar to a water jet engine's behaviour. Just that in this case the fabric of spacetime itself is being used instead of water. It seems clear to me that not only should a working Q-thruster propel itself, but also create clearly measurable local side-effects.

What do you guys think?
Title: Re: EM Drive Developments - related to space flight applications - Thread 2
Post by: Star One on 04/07/2015 11:36 am
While that's all very interesting. You can immediately rule out the drive's functionality because of the lack of reaction mass. CoM is clearly broken.

I only have a minor in physics so I honestly can't understand most of what's talked about here, but I know at the least you can't go about thrusting around the universe with no particles/energy leaving the engine without something like an Alcubierre drive. Even if you're pushing against quantum virtual particle pairs, virtual particle pairs have opposite signs so will be propelled in opposite directions thus canceling out any net force.

I have yet to year a simple explanation of how this drive works that doesn't dive into theoretical physics to explain its operation. It is true of everything in physics that simple laws lead to complex behavior. If the operation of the EmDrive or other various families of the drive cannot be explained in an abstract way within a couple sentences then it doesn't work.

People are being sold snake oil and I don't like it.

"We don't know how, why, or if it works and known physics do not readily suffice, therefore it doesn't work and its proponents are perpetuating fraud" is jumping the gun a bit, don't you think?

It would be pretty counterproductive if we took the OP's stance to a lot of quantum physics. The world of the quantum often seems to do its own thing and whether it and classical physics will ever be able to coexist happily seems from a layman's viewpoint very much up for debate. But unfortunately saying anything further would be thread drift.

@CW you mean something like a quantum wake?
Title: Re: EM Drive Developments - related to space flight applications - Thread 2
Post by: Rodal on 04/07/2015 12:14 pm
Quote
The microwave communications are possible because microwaves are transmitted from (and to) the satellite.  The microwaves obviously are leaving  (or entering) the satellite.  If there is any propulsion resulting from communications from the satellite, it may not violate the laws of conservation of momentum and conservation of energy, because the microwave photons are leaving the sateliite.  It would work like a photon rocket for microwaves leaving the satellite (or like a solar sail for microwave photons hitting the satellite).  Scientists have no problems with the concept of a photon rocket (except that it is one the least useful forms of space propulsion: one can also use a military searchlight as a photon rocket).

Shawyer's EM Drive is completely different: it is a completely enclosed cavity.  The microwaves inside it are standing waves.  No microwaves leave the EM Drive.  No photons leave the EM Drive (certainly not on purpose). That's why scientists think that the EM Drive should not be able to have any propulsion: because it appears to violate conservation of momentum. 

The standing microwaves inside an EM Drive cannot be used for communication purposes because they stay inside the EM Drive: the metal acts like a Faraday cage.

So what is mystifying, if indeed Shaywer was inspired by this issue, is whether Shawyer did not understand the difference between a completely enclosed microwave cavity and a communications satellite transmitting microwaves or whether the story that he was inspired by this is not really accurate.

It would be like somebody saying that they invented a new form of space propulsion based on a flashlight inside a a completely sealed box with mirrors on every inside surface, and claiming that they were inspired by noticing that a satellite in space flashing a light for communications purposes, also acts like a photon rocket. 

Yes, a flashlight in space will act like a photon rocket, but if you put the flaslight inside a sealed box so that no light leaves the box, it is just a box sitting in space: it is no longer good for communications and it is no longer good for propulsion

My guess - and its no more than that - is that Shawyer concluded the 'thrust' produced by these microwave emitters was significantly greater than that of a photon rocket effect. 

Despite a fair number of internet searches, I have yet to encounter any details on Shawyer's research into this.

....
Think of the analogy: somebody is inspired by noticing that a flashlight used for communications from space also acts as a photon rocket.  Or, as you propose, it actually has even more thrust than a photon rocket.
Does it make sense then that the person being "inspired" by this would put the flashlight inside a sealed box, such that the flashlight cannot longer work as a communication device or as a super photon rocket?

Being inspired by microwave communications satellites and reacting by enclosing it inside a Faraday cage where the microwaves cannot escape doesn't make any sense to me.

If he was inspired by the microwave communications satellites his first design should have looked like a satellite's microwave antenna:

(http://www.esa.int/var/esa/storage/images/esa_multimedia/images/2009/04/artemis/9519912-4-eng-GB/ARTEMIS_medium.jpg)

(http://media-1.web.britannica.com/eb-media/24/4624-004-0ED4DA55.jpg)

instead of a sealed box: a truncated cone without any openings:

(http://emdrive.com/images/feasibility3.jpg) 


No wonder then that you couldn't find anything supporting this story.  It is a contradictory story that hangs from a thread: that Roger Shawyer was a consultant to the Galileo project (Europe’s satnav system), but this "inspiration story" conceals the fact that Shawyer's EM Drive is not at all like a microwave communications antenna, since it is designed such that the microwaves canNOT escape the EM Drive, which is the opposite of the design of a microwave communications antenna.
Title: Re: EM Drive Developments - related to space flight applications - Thread 2
Post by: CW on 04/07/2015 12:17 pm
While that's all very interesting. You can immediately rule out the drive's functionality because of the lack of reaction mass. CoM is clearly broken.

I only have a minor in physics so I honestly can't understand most of what's talked about here, but I know at the least you can't go about thrusting around the universe with no particles/energy leaving the engine without something like an Alcubierre drive. Even if you're pushing against quantum virtual particle pairs, virtual particle pairs have opposite signs so will be propelled in opposite directions thus canceling out any net force.

I have yet to year a simple explanation of how this drive works that doesn't dive into theoretical physics to explain its operation. It is true of everything in physics that simple laws lead to complex behavior. If the operation of the EmDrive or other various families of the drive cannot be explained in an abstract way within a couple sentences then it doesn't work.

People are being sold snake oil and I don't like it.

"We don't know how, why, or if it works and known physics do not readily suffice, therefore it doesn't work and its proponents are perpetuating fraud" is jumping the gun a bit, don't you think?

It would be pretty counterproductive if we took the OP's stance to a lot of quantum physics. The world of the quantum often seems to do its own thing and whether it and classical physics will ever be able to coexist happily seems from a layman's viewpoint very much up for debate. But unfortunately saying anything further would be thread drift.

@CW you mean something like a quantum wake?

Pretty much, yes. Just that you would feel a sort of invisible 'spatial current' forming, that enters the Q-thruster in front and leaves the thruster behind it again, while the thruster accelerates.
Title: Re: EM Drive Developments - related to space flight applications - Thread 2
Post by: Star-Drive on 04/07/2015 12:41 pm
Aces-high:

"well, it´s about 1250 kN is about two times the thrust of a Merlin 1D engine from the Falcon 9..."

Let's not get carried away here!  Our COMSOL/Q-V plasma code simulation indicated ~1,250 Newton for 100kW RF input using a water cooled version of our current copper cavity, NOT 1,250 kN, which is three orders of magnitude larger than stated in my previous post.  I'm still trying to wrap my head around this 1,250 to 2,000 Newton figure...

Best, Paul M.

bah, just a 3 orders of magnitude difference!

oops  :-[ :-[

well Paul, you might want to answer Birchoff's question directly?

Quote from: Birchoff
Given the figures the predicted thrust that paul march provided us. Anyone mind educating this lay person on how a single EmDrive thruster would perform vs our current rockets.

Aces-high:

Well what's a few zeros between friends?! :)

In regards to Birchoff's question, (Anyone mind educating this lay person on how a single EmDrive thruster would perform vs our current rockets.), well, lets say the Eagleworks COMSOL / Q-V plasma code is really providing accurate thrust predictions in its current V1.1 status, remembering that even Microsoft took to V3.1 to get the first really working version of Windows to a point of use-ability, i.e. we only just started to experimentally verify this code,  what does ~1,300 Newton bring to the table as far as rocket applications are concerned? 

The EM-Drive applications all depends on the thrust to mass ratio of the Q-Thruster AND its supporting power supply, structure and payload, and for how long the power supply & its energy source in question can operate.  If its thrust to weight (T/W) ratio is less than 1-to-1, then this is a in space based propulsion system that first has to be placed into orbit by chemical rockets.  If its greater than 1-to-1 and it can operate for long enough, then this EM-Drive rocket  could place itself into LEO and perhaps beyond for that all depends on how long its batteries/reactor can keep supply power.

Best, Paul M.
Title: Re: EM Drive Developments - related to space flight applications - Thread 2
Post by: CW on 04/07/2015 12:51 pm
Aces-high:

Well what's a few zeros between friends?! :)

In regards to Birchoff's question, (Anyone mind educating this lay person on how a single EmDrive thruster would perform vs our current rockets.), well, lets say the Eagleworks COMSOL / Q-V plasma code is really providing accurate thrust predictions in its current V1.1 status, remembering that even Microsoft took to V3.1 to get the first really working version of Windows to a point of use-ability, i.e. we only just started to experimentally verify this code,  what does ~1,300 Newton bring to the table as far as rocket applications are concerned? 

The EM-Drive applications all depends on the thrust to mass ratio of the Q-Thruster AND its supporting power supply, structure and payload, and for how long the power supply & its energy source in question can operate.  If its thrust to weight (T/W) ratio is less than 1-to-1, then this is a in space based propulsion system that first has to be placed into orbit by chemical rockets.  If its greater than 1-to-1 and it can operate for long enough, then this EM-Drive rocket  could place itself into LEO and perhaps beyond for that all depends on how long its batteries/reactor can keep supply power.

Best, Paul M.

I think that you could get away with a ratio of less than 1-to-1, if you use the propulsive force to accelerate like an airplane. Difference being, an airplane needs a dense enough atmosphere to further accelerate and gain altitude (which becomes a problem as of certain altitudes), while an EM-drive on a plane-shaped body could still accelerate further and further in a thinning atmosphere to gain tangential momentum and eventually reach orbit.
Title: Re: EM Drive Developments - related to space flight applications - Thread 2
Post by: Star-Drive on 04/07/2015 12:54 pm
While that's all very interesting. You can immediately rule out the drive's functionality because of the lack of reaction mass. CoM is clearly broken.

I only have a minor in physics so I honestly can't understand most of what's talked about here, but I know at the least you can't go about thrusting around the universe with no particles/energy leaving the engine without something like an Alcubierre drive. Even if you're pushing against quantum virtual particle pairs, virtual particle pairs have opposite signs so will be propelled in opposite directions thus canceling out any net force.

I have yet to year a simple explanation of how this drive works that doesn't dive into theoretical physics to explain its operation. It is true of everything in physics that simple laws lead to complex behavior. If the operation of the EmDrive or other various families of the drive cannot be explained in an abstract way within a couple sentences then it doesn't work.

People are being sold snake oil and I don't like it.

"We don't know how, why, or if it works and known physics do not readily suffice, therefore it doesn't work and its proponents are perpetuating fraud" is jumping the gun a bit, don't you think?

Tornados don't produces magical instruments. They produce junk. Luck doesn't make engineering.

The chances of someone stumbling upon some contraption that violates physics principles by throwing electronics parts out of his garage together is nil.

"I  only have a minor in physics so I honestly can't understand most of what's talked about here, but I know at the least you can't go about thrusting around the universe with no particles/energy leaving the engine without something like an Alcubierre drive. Even if you're pushing against quantum virtual particle pairs, virtual particle pairs have opposite signs so will be propelled in opposite directions thus canceling out any net force."

In a neutral plasma there is an equal number of plus and minus electrical charges or ions that can react to applied E-fields and B-fields in various ways.  If there is only an electric field applied to the plasma volume then yes the positive charges will go one way and the negative charges will go in the opposite direction.  However if we apply a spatially crossed E-field and B-field across this volume, then we have a Lorentz force produced on the plasma ions that is at right angles to the applied E-field and B-field.  Then BOTH the positive and negative ions will be accelerated in the SAME direction, but with counter rotating twists AKA Gyro radius modifying their accelerated trajectories.  All of these EM-Drive like thruster utilize some form of this Lorentz force acceleration on some type of propellant, be it real as in a Hall thruster or semi-virtual.

Best, Paul M. 
Title: Re: EM Drive Developments - related to space flight applications - Thread 2
Post by: Star-Drive on 04/07/2015 01:01 pm
Aces-high:

Well what's a few zeros between friends?! :)

In regards to Birchoff's question, (Anyone mind educating this lay person on how a single EmDrive thruster would perform vs our current rockets.), well, lets say the Eagleworks COMSOL / Q-V plasma code is really providing accurate thrust predictions in its current V1.1 status, remembering that even Microsoft took to V3.1 to get the first really working version of Windows to a point of use-ability, i.e. we only just started to experimentally verify this code,  what does ~1,300 Newton bring to the table as far as rocket applications are concerned? 

The EM-Drive applications all depends on the thrust to mass ratio of the Q-Thruster AND its supporting power supply, structure and payload, and for how long the power supply & its energy source in question can operate.  If its thrust to weight (T/W) ratio is less than 1-to-1, then this is a in space based propulsion system that first has to be placed into orbit by chemical rockets.  If its greater than 1-to-1 and it can operate for long enough, then this EM-Drive rocket  could place itself into LEO and perhaps beyond for that all depends on how long its batteries/reactor can keep supply power.

Best, Paul M.

I think that you could get away with a ratio of less than 1-to-1, if you use the propulsive force to accelerate like an airplane. Difference being, an airplane needs a dense enough atmosphere to further accelerate and gain altitude (which becomes a problem as of certain altitudes), while an EM-drive on a plane-shaped body could still accelerate further and further in a thinning atmosphere to gain tangential momentum and eventually reach orbit.

CW:

Thanks much for pointing out this in between status of utilizing aerodynamic forces as a first stage that can be applied to exotic thrusters that are only producing ~0.1 N/kWe up to ~20 N/kWe where VTOL craft designs become viable. 

Best, Paul M.
Title: Re: EM Drive Developments - related to space flight applications - Thread 2
Post by: Star One on 04/07/2015 01:10 pm
Aces-high:

Well what's a few zeros between friends?! :)

In regards to Birchoff's question, (Anyone mind educating this lay person on how a single EmDrive thruster would perform vs our current rockets.), well, lets say the Eagleworks COMSOL / Q-V plasma code is really providing accurate thrust predictions in its current V1.1 status, remembering that even Microsoft took to V3.1 to get the first really working version of Windows to a point of use-ability, i.e. we only just started to experimentally verify this code,  what does ~1,300 Newton bring to the table as far as rocket applications are concerned? 

The EM-Drive applications all depends on the thrust to mass ratio of the Q-Thruster AND its supporting power supply, structure and payload, and for how long the power supply & its energy source in question can operate.  If its thrust to weight (T/W) ratio is less than 1-to-1, then this is a in space based propulsion system that first has to be placed into orbit by chemical rockets.  If its greater than 1-to-1 and it can operate for long enough, then this EM-Drive rocket  could place itself into LEO and perhaps beyond for that all depends on how long its batteries/reactor can keep supply power.

Best, Paul M.

I think that you could get away with a ratio of less than 1-to-1, if you use the propulsive force to accelerate like an airplane. Difference being, an airplane needs a dense enough atmosphere to further accelerate and gain altitude (which becomes a problem as of certain altitudes), while an EM-drive on a plane-shaped body could still accelerate further and further in a thinning atmosphere to gain tangential momentum and eventually reach orbit.

Would it be useful to have some kind of launcher on the ground to assist it off the ground rather like the catapult on an aircraft carrier or as they are now moving to in the navy on some carriers an electro magnetic launcher?
Title: Re: EM Drive Developments - related to space flight applications - Thread 2
Post by: tea monster on 04/07/2015 01:24 pm
NASA has tested one of these things and got a positive result. So That is why people are discussing it. Otherwise, it would be lumped in with the Dean Drives, Ancient Astronauts, UFOs and other bizzare pseudo science stories.

Nobody is quite sure ***how*** they got a positive result. It may yet turn out to be not what they think it is, or it might turn out to be something interesting nobody really knows yet.
Title: Re: EM Drive Developments - related to space flight applications - Thread 2
Post by: JPLeRouzic on 04/07/2015 02:08 pm
However if we apply a spatially crossed E-field and B-field across this volume, then we have a Lorentz force produced on the plasma ions that is at right angles to the applied E-field and B-field.  Then BOTH the positive and negative ions will be accelerated in the SAME direction,
Hello,

Thanks for your patience and answers which are always very clear and precise.

However I don't understand the two quoted sentences, at least in this context:
http://en.wikipedia.org/wiki/Lorentz_force#Force_on_a_current-carrying_wire
It seems to me that the second sentence contradicts the first sentence which looks correct to me.
Please could elaborate?

Jean-Pierre
Title: Re: EM Drive Developments - related to space flight applications - Thread 2
Post by: JasonAW3 on 04/07/2015 02:23 pm

While that's all very interesting. You can immediately rule out the drive's functionality because of the lack of reaction mass. CoM is clearly broken.

I only have a minor in physics so I honestly can't understand most of what's talked about here, but I know at the least you can't go about thrusting around the universe with no particles/energy leaving the engine without something like an Alcubierre drive. Even if you're pushing against quantum virtual particle pairs, virtual particle pairs have opposite signs so will be propelled in opposite directions thus canceling out any net force.

I have yet to year a simple explanation of how this drive works that doesn't dive into theoretical physics to explain its operation. It is true of everything in physics that simple laws lead to complex behavior. If the operation of the EmDrive or other various families of the drive cannot be explained in an abstract way within a couple sentences then it doesn't work.

People are being sold snake oil and I don't like it.

I'm not really certain that you can call this snake oil.

     Physics wise, energy is being generated, which consumes a form of fuel, be it nuclear, fossile or solar energy, energy is being generated.

     In theory, according to basic physics, it should be possible to convert one form of energy into another, such as kinetic energy being converted to heat energy as a vehicle is decellerated via air during reentry.

     So, in theory, it should be possible to convert generated electrical energy into kinetic energy, which, in normal circumstances, is typically done via the use of either motors or mass being expelled at a high velocity, typically using heat to produce a high amount of directionalized kinetic energy in said mass.  Thus, it is not beyond the realms of physics that it may be possible to convert electrical energy directly into kinetic energy.  There would, of course, be some loss or energy as heat, but, in theory, it should be possible.

     Whether or not this is what is happening here is unknown, but SOMETHING appears to be happening, (namely the imparting of thrust through non-normal means,) beyond the realm of "noise" in the experiment, thay as yet, cannot be fully explained.

     I think thay trying to figure out what is happening, and whether or not it is some artifact of the experiment or an actual force that can be utilized, goes well beyond "snake oil" directly into science.
Title: Re: EM Drive Developments - related to space flight applications - Thread 2
Post by: matthewpapa on 04/07/2015 03:39 pm
How do rockets work? They push out hot gas - the rocket goes direction x, the gas direction -x .

How does the hypothesized Q-thruster work? It pushes out QV particle 'plasma' - the Q-thruster goes direction x, the QV particle 'plasma' direciton -x .

What's the difference? In case of a rocket, we can still 'see' the hot gas. In case of the Q-thruster, we cannot. Philosophically speaking, it does not matter if you use QV particle 'plasma' or 'real' particles. Both types can be considered a subset of what 'quantum vacuum' is - namely the superset of all that can possibly exist.

It's just interesting to think about CoM. I think that the most likely reaction would be that in direction of acceleration, the Q-thruster should produce a sort of 'suction effect', and a repelling effect on the opposite side. Reason being that when the virtual plasma particle pairs vanish again, the impeded momentum must be conserved still in some form. I can only imagine this as a unidirectional 'gravity'-like effect, or maybe an effect similar to a water jet engine's behaviour. Just that in this case the fabric of spacetime itself is being used instead of water. It seems clear to me that not only should a working Q-thruster propel itself, but also create clearly measurable local side-effects.

What do you guys think?

Yes, this I think is a good way to explain it I think
Seems like we should be able to find some sort of QV wake. The energy has to be expended somewhere.

Would be interesting to see if
Power in = Simultaneous Dissipated Heat (measured)
You would of course have to run the experiment for a while to get to a steady state to make the heat measurements (J/S).
If these numbers match and we still get "thrust" we can afford to be suspicious.
But, if
Power in != Dissipated Heat
And we can deduce
Pin = Heat+ Thrust

Then we can be assured there is something interesting is going on

This may be hard to see with the current test article due to the low amounts of thrust measured. But it should eventually be testable with what Paul is currently building
Title: Re: EM Drive Developments - related to space flight applications - Thread 2
Post by: jmossman on 04/07/2015 03:48 pm
Quote
The microwave communications are possible because microwaves are transmitted from (and to) the satellite.  The microwaves obviously are leaving  (or entering) the satellite.  If there is any propulsion resulting from communications from the satellite, it may not violate the laws of conservation of momentum and conservation of energy, because the microwave photons are leaving the sateliite.  It would work like a photon rocket for microwaves leaving the satellite (or like a solar sail for microwave photons hitting the satellite).  Scientists have no problems with the concept of a photon rocket (except that it is one the least useful forms of space propulsion: one can also use a military searchlight as a photon rocket).

Shawyer's EM Drive is completely different: it is a completely enclosed cavity.  The microwaves inside it are standing waves.  No microwaves leave the EM Drive.  No photons leave the EM Drive (certainly not on purpose). That's why scientists think that the EM Drive should not be able to have any propulsion: because it appears to violate conservation of momentum. 

The standing microwaves inside an EM Drive cannot be used for communication purposes because they stay inside the EM Drive: the metal acts like a Faraday cage.

So what is mystifying, if indeed Shaywer was inspired by this issue, is whether Shawyer did not understand the difference between a completely enclosed microwave cavity and a communications satellite transmitting microwaves or whether the story that he was inspired by this is not really accurate.

It would be like somebody saying that they invented a new form of space propulsion based on a flashlight inside a a completely sealed box with mirrors on every inside surface, and claiming that they were inspired by noticing that a satellite in space flashing a light for communications purposes, also acts like a photon rocket. 

Yes, a flashlight in space will act like a photon rocket, but if you put the flaslight inside a sealed box so that no light leaves the box, it is just a box sitting in space: it is no longer good for communications and it is no longer good for propulsion

My guess - and its no more than that - is that Shawyer concluded the 'thrust' produced by these microwave emitters was significantly greater than that of a photon rocket effect. 

Despite a fair number of internet searches, I have yet to encounter any details on Shawyer's research into this.

....
Think of the analogy: somebody is inspired by noticing that a flashlight used for communications from space also acts as a photon rocket.  Or, as you propose, it actually has even more thrust than a photon rocket.
Does it make sense then that the person being "inspired" by this would put the flashlight inside a sealed box, such that the flashlight cannot longer work as a communication device or as a super photon rocket?

Being inspired by microwave communications satellites and reacting by enclosing it inside a Faraday cage where the microwaves cannot escape doesn't make any sense to me.

If he was inspired by the microwave communications satellites his first design should have looked like a satellite's microwave antenna:

(http://www.esa.int/var/esa/storage/images/esa_multimedia/images/2009/04/artemis/9519912-4-eng-GB/ARTEMIS_medium.jpg)

(http://media-1.web.britannica.com/eb-media/24/4624-004-0ED4DA55.jpg)

instead of a sealed box: a truncated cone without any openings:

(http://emdrive.com/images/feasibility3.jpg) 


No wonder then that you couldn't find anything supporting this story.  It is a contradictory story that hangs from a thread: that Roger Shawyer was a consultant to the Galileo project (Europe’s satnav system), but this "inspiration story" conceals the fact that Shawyer's EM Drive is not at all like a microwave communications antenna, since it is designed such that the microwaves canNOT escape the EM Drive, which is the opposite of the design of a microwave communications antenna.

What if the satellite(s) that were studied had some kind of internal microwave resonant cavity as part of a system to help amplify signals?  If the orientation of the internal cavity correlated strongly to the unexpected fuel burn direction.....

BTW I have no idea if this Shawyer background story has any truth or not...  just offering a (rather feeble) lifeline for the hypothetical Shawyer back story.   :P
Title: Re: EM Drive Developments - related to space flight applications - Thread 2
Post by: CW on 04/07/2015 04:14 pm

While that's all very interesting. You can immediately rule out the drive's functionality because of the lack of reaction mass. CoM is clearly broken.

I only have a minor in physics so I honestly can't understand most of what's talked about here, but I know at the least you can't go about thrusting around the universe with no particles/energy leaving the engine without something like an Alcubierre drive. Even if you're pushing against quantum virtual particle pairs, virtual particle pairs have opposite signs so will be propelled in opposite directions thus canceling out any net force.

I have yet to year a simple explanation of how this drive works that doesn't dive into theoretical physics to explain its operation. It is true of everything in physics that simple laws lead to complex behavior. If the operation of the EmDrive or other various families of the drive cannot be explained in an abstract way within a couple sentences then it doesn't work.

People are being sold snake oil and I don't like it.

I'm not really certain that you can call this snake oil.

     Physics wise, energy is being generated, which consumes a form of fuel, be it nuclear, fossile or solar energy, energy is being generated.

     In theory, according to basic physics, it should be possible to convert one form of energy into another, such as kinetic energy being converted to heat energy as a vehicle is decellerated via air during reentry.

     So, in theory, it should be possible to convert generated electrical energy into kinetic energy, which, in normal circumstances, is typically done via the use of either motors or mass being expelled at a high velocity, typically using heat to produce a high amount of directionalized kinetic energy in said mass.  Thus, it is not beyond the realms of physics that it may be possible to convert electrical energy directly into kinetic energy.  There would, of course, be some loss or energy as heat, but, in theory, it should be possible.

     Whether or not this is what is happening here is unknown, but SOMETHING appears to be happening, (namely the imparting of thrust through non-normal means,) beyond the realm of "noise" in the experiment, thay as yet, cannot be fully explained.

     I think thay trying to figure out what is happening, and whether or not it is some artifact of the experiment or an actual force that can be utilized, goes well beyond "snake oil" directly into science.

I 100% agree to the observation that physics does not forbid in any way a mechanism to directly convert electrical energy into kinetic energy. The problem is finding the mechanism. Experimentation will show whether or not this contraption can do the trick.
Title: Re: EM Drive Developments - related to space flight applications - Thread 2
Post by: kdhilliard on 04/07/2015 04:53 pm
In theory, according to basic physics, it should be possible to convert one form of energy into another, such as kinetic energy being converted to heat energy as a vehicle is decellerated via air during reentry.
Conservation of momentum is as fundamental a property as conservation of energy, and it is the former which many critics believe is violated by the EM drive.

~Kirk
Title: Re: EM Drive Developments - related to space flight applications - Thread 2
Post by: Rodal on 04/07/2015 05:04 pm
In theory, according to basic physics, it should be possible to convert one form of energy into another, such as kinetic energy being converted to heat energy as a vehicle is decellerated via air during reentry.
Conservation of momentum is as fundamental a property as conservation of energy, and it is the former which many critics believe is violated by the EM drive.

~Kirk

Conservation of momentum is satisfied in Maxwell's equations, Newtonian mechanics, special relativity, general relativity, quantum mechanics and quantum electrodynamics.

In relativity, momentum and energy are both tied up together in the energy-momentum tensor.
Title: Re: EM Drive Developments - related to space flight applications - Thread 2
Post by: Stormbringer on 04/07/2015 05:14 pm
From the peanut gallery again... apparently massive particles in groups (sort of a meta-material or meta particle) can become effectively massless? That's a neat trick if you could pull it off at will.

http://phys.org/news/2015-04-unparticles-path-superconductivity.html

Just how many of these special cases in the standard model are there? We have this and we have stable kaons in lambda baryons and I have read mirror neutrons or alice matter.

These special modes of behavior in regular matter might have something to do with unexpected physical effects like what we are discussing in the present thread. I realize it probably doesn't but it is at least a remote possibility.

Title: Re: EM Drive Developments - related to space flight applications - Thread 2
Post by: Rodal on 04/07/2015 05:23 pm
....

     In theory, according to basic physics, it should be possible to convert one form of energy into another, such as kinetic energy being converted to heat energy as a vehicle is decellerated via air during reentry.

     So, in theory, it should be possible to convert generated electrical energy into kinetic energy, which, in normal circumstances, is typically done via the use of either motors or mass being expelled at a high velocity, typically using heat to produce a high amount of directionalized kinetic energy in said mass.  Thus, it is not beyond the realms of physics that it may be possible to convert electrical energy directly into kinetic energy.  There would, of course, be some loss or energy as heat, but, in theory, it should be possible.

    ...


If electrical energy can be converted directly into spacecraft's momentum, without any matter or energy leaving the spacecraft then many problems appear.

One interesting problem has been repeatedly pointed out by @frobnicat in this thread.  Since the kinetic energy is:

K = (1/2) m v^2

The power needed to accelerate (a question appearing in this thread over the last few pages concerning a comparison with chemical rockets) is dK/dt,and since in this EM Drive spacecraft, the mass doesn't change, the only thing that changes is the velocity, therefore:

dK/dt = (1/2) m [2 v dv/dt] = m v a

So the power needed to accelerate is a function of not only the acceleration wanted, but also the speed at which you're currently traveling.  But, according to relativity, there is no absolute measurement of spacecraft speed, it depends on the observer. So, the needed power (to escape the surface of the Earth, etc.) depends on your frame of reference. 

For a conventional rocket, the delta V is related to the mass of the spacecraft decreasing, see http://en.wikipedia.org/wiki/Delta-v for example.  For the EM Drive there is no decrease of mass of the spacecraft (and if one adheres to the assumption that the Quantum Vacuum (QV) is immutable and indestructible, the QV being the zero point energy, the energy=mass*c^2 of the QV shouldn't change either).  So one has a conundrum.

So, no, "according to basic physics, it should" not be possible to directly convert electrical energy into a spacecraft's momentum change without any change in mass of the spacecraft (or the action of external forces).

There are several paradoxes  with the EM Drive related to this.

(http://figures.boundless.com/12999/full/figure-09-07-01a.jpe)
Title: Re: EM Drive Developments - related to space flight applications - Thread 2
Post by: Star One on 04/07/2015 05:30 pm

In theory, according to basic physics, it should be possible to convert one form of energy into another, such as kinetic energy being converted to heat energy as a vehicle is decellerated via air during reentry.
Conservation of momentum is as fundamental a property as conservation of energy, and it is the former which many critics believe is violated by the EM drive.

~Kirk

Conservation of momentum is satisfied in Maxwell's equations, Newtonian mechanics, special relativity, general relativity, quantum mechanics and quantum electrodynamics.

In relativity, momentum and energy are both tied up together in the energy-momentum tensor.

I wonder if somehow a post along these lines could be tagged to the start of the thread so this question would stop being repeated in it?
Title: Re: EM Drive Developments - related to space flight applications - Thread 2
Post by: birchoff on 04/07/2015 06:40 pm
@Star-Drive

So its now the middle of the first week of April. I was under the impression that Eagleworks needed to get GRC something to test by the end of March. Is that an accurate recollection of what your part of your research timeline was? If so, were you able to hit that deadline or was it pushed out?
Title: Re: EM Drive Developments - related to space flight applications - Thread 2
Post by: JasonAW3 on 04/07/2015 08:10 pm
....

     In theory, according to basic physics, it should be possible to convert one form of energy into another, such as kinetic energy being converted to heat energy as a vehicle is decellerated via air during reentry.

     So, in theory, it should be possible to convert generated electrical energy into kinetic energy, which, in normal circumstances, is typically done via the use of either motors or mass being expelled at a high velocity, typically using heat to produce a high amount of directionalized kinetic energy in said mass.  Thus, it is not beyond the realms of physics that it may be possible to convert electrical energy directly into kinetic energy.  There would, of course, be some loss or energy as heat, but, in theory, it should be possible.

    ...


If electrical energy can be converted directly into spacecraft's momentum, without any matter or energy leaving the spacecraft then many problems appear.

One interesting problem has been repeatedly pointed out by @frobnicat in this thread.  Since the kinetic energy is:

K = (1/2) m v^2

The power needed to accelerate (a question appearing in this thread over the last few pages concerning a comparison with chemical rockets) is dK/dt,and since in this EM Drive spacecraft, the mass doesn't change, the only thing that changes is the velocity, therefore:

dK/dt = (1/2) m [2 v dv/dt] = m v a

So the power needed to accelerate is a function of not only the acceleration wanted, but also the speed at which you're currently traveling.  But, according to relativity, there is no absolute measurement of spacecraft speed, it depends on the observer. So, the needed power (to escape the surface of the Earth, etc.) depends on your frame of reference. 

For a conventional rocket, the delta V is related to the mass of the spacecraft decreasing, see http://en.wikipedia.org/wiki/Delta-v for example.  For the EM Drive there is no decrease of mass of the spacecraft (and if one adheres to the assumption that the Quantum Vacuum (QV) is immutable and indestructible, the QV being the zero point energy, the energy=mass*c^2 of the QV shouldn't change either).  So one has a conundrum.

So, no, "according to basic physics, it should" not be possible to directly convert electrical energy into a spacecraft's momentum change without any change in mass of the spacecraft (or the action of external forces).

There are several paradoxes  with the EM Drive related to this.

(http://figures.boundless.com/12999/full/figure-09-07-01a.jpe)

Be it nuclear, fuel cell or even solar, as the source of electricity, mass is being expended in the form of electrons. (Solar is taking advantage of the Sun's own expendature of mass in the form of photons).  Whiloe the quantity of mass being expended is minute, it IS being expended.

     Unfortunately, the AMOUNT of mass being expended, even if it were being a total conversion to energy, does seem to be way out of proportion to the amount of kinetic energy being imparted, assuming the use of Newtonian Mass physics.

     Even considering a direct conversion of electrical to kinetic energy seems very much disproportionate.  Occam's Razor seems to apply in this case.  All things being equal, the simplest explaination seems the most likely.  Unfortuanately, most of the explainations of this phenomena seem to be of the complex and wonderful nature.

    I am wondering if anone has tried to place a force sensor against the side of the device that appeards to be producing the thrust to see if ther is some sort of repulsive force involved.  As I understand it, all the testing thus far have been with the device and arm that it's attached to.  If there is some force being produced, then an apparatus resting against the device should detect it.  If there is a kinetic energy being produced then the device pulling away from the apperatus should be detectible.  If there is a spacial distortion, this should be observable, but no real pressure would be detected by the apperatus.

     I have little doubt that someone's probably already done susch a test, but I haven't seen any data on this as yet.  (Or if I have, it's been cloaked in such higher math and physices that it left me completely baffled).
Title: Re: EM Drive Developments - related to space flight applications - Thread 2
Post by: Rodal on 04/07/2015 08:50 pm
....
Be it nuclear, fuel cell or even solar, as the source of electricity, mass is being expended in the form of electrons. (Solar is taking advantage of the Sun's own expendature of mass in the form of photons).  Whiloe the quantity of mass being expended is minute, it IS being expended.
   ...
Even if (for discussion's sake) mass would be converted into energy with a nuclear reaction E=mc^2 inside the spacecraft to provide the electricity for the EM Drive, that does not solve the conundrum: the issue is not "to expend energy", the issue is to satisfy conservation of momentum.  If no mass leaves the spacecraft, while kinetic energy is converted into a change in momentum of the spacecraft's center of mass, you still have the same conundrum and the same paradoxes I previously noted:

The needed power for the EM Drive (to escape the surface of the Earth, or anything else you want the spacecraft to do) depends on your frame of reference. 

As Paul March himself admitted, for their Quantum Vacuum explanation for the EM Drive to hold, they need to disrespect the mainstream physics assumption that the Quantum Vacuum is indestructible and immutable.

Bottom line: no, "according to basic physics, it should" not be possible to directly convert electrical energy into a spacecraft's momentum change without any change in mass of the spacecraft (or the action of external forces).  If the EM Drive were to work for space propulsion, it certainly would not be explainable in terms of mainstream physics where conservation of momentum is paramount, and the Quantum Vacuum is both indestructible and immutable.

(http://ecx.images-amazon.com/images/I/51dQFZfQT2L._SY344_BO1,204,203,200_.jpg)
Title: Re: EM Drive Developments - related to space flight applications - Thread 2
Post by: CW on 04/07/2015 09:06 pm
....
Be it nuclear, fuel cell or even solar, as the source of electricity, mass is being expended in the form of electrons. (Solar is taking advantage of the Sun's own expendature of mass in the form of photons).  Whiloe the quantity of mass being expended is minute, it IS being expended.
   ...
Even if (for discussion's sake) mass would be converted into energy with a nuclear reaction E=mc^2 inside the spacecraft to provide the electricity for the EM Drive, that does not solve the conundrum: the issue is not "to expend energy", the issue is to satisfy conservation of momentum.  If no mass leaves the spacecraft, while kinetic energy is converted into a change in momentum of the spacecraft's center of mass, you still have the same conundrum and the same paradoxes I previously noted:

The needed power for the EM Drive (to escape the surface of the Earth, or anything else you want the spacecraft to do) depends on your frame of reference. 

As Paul March himself admitted, for their Quantum Vacuum explanation for the EM Drive to hold, they need to disrespect the mainstream physics assumption that the Quantum Vacuum is indestructible and immutable.

Bottom line: no, "according to basic physics, it should" not be possible to directly convert electrical energy into a spacecraft's momentum change without any change in mass of the spacecraft (or the action of external forces).  If the EM Drive were to work for space propulsion, it certainly would not be explainable in terms of mainstream physics where conservation of momentum is paramount, and the Quantum Vacuum is both indestructible and immutable.

(http://www.projectrho.com/public_html/rocket/images/reactionlessdrive/sub03.jpg)

Just being curious: What, in your book, would be the most important theoretical consequences of the discovery of a different QV nature?
Title: Re: EM Drive Developments - related to space flight applications - Thread 2
Post by: Rodal on 04/07/2015 09:16 pm
....
Be it nuclear, fuel cell or even solar, as the source of electricity, mass is being expended in the form of electrons. (Solar is taking advantage of the Sun's own expendature of mass in the form of photons).  Whiloe the quantity of mass being expended is minute, it IS being expended.
   ...
Even if (for discussion's sake) mass would be converted into energy with a nuclear reaction E=mc^2 inside the spacecraft to provide the electricity for the EM Drive, that does not solve the conundrum: the issue is not "to expend energy", the issue is to satisfy conservation of momentum.  If no mass leaves the spacecraft, while kinetic energy is converted into a change in momentum of the spacecraft's center of mass, you still have the same conundrum and the same paradoxes I previously noted:

The needed power for the EM Drive (to escape the surface of the Earth, or anything else you want the spacecraft to do) depends on your frame of reference. 

As Paul March himself admitted, for their Quantum Vacuum explanation for the EM Drive to hold, they need to disrespect the mainstream physics assumption that the Quantum Vacuum is indestructible and immutable.

Bottom line: no, "according to basic physics, it should" not be possible to directly convert electrical energy into a spacecraft's momentum change without any change in mass of the spacecraft (or the action of external forces).  If the EM Drive were to work for space propulsion, it certainly would not be explainable in terms of mainstream physics where conservation of momentum is paramount, and the Quantum Vacuum is both indestructible and immutable.

(http://www.projectrho.com/public_html/rocket/images/reactionlessdrive/sub03.jpg)

Just being curious: What, in your book, would be the most important theoretical consequences of the discovery of a different QV nature?
If the QV is not immutable and indestructible, the theoretical and practical consequences would be so groundbreaking that they would make the 20th century's discoveries of atomic and nuclear energy pale in comparison.
Title: Re: EM Drive Developments - related to space flight applications - Thread 2
Post by: Star One on 04/07/2015 09:59 pm
....
Be it nuclear, fuel cell or even solar, as the source of electricity, mass is being expended in the form of electrons. (Solar is taking advantage of the Sun's own expendature of mass in the form of photons).  Whiloe the quantity of mass being expended is minute, it IS being expended.
   ...
Even if (for discussion's sake) mass would be converted into energy with a nuclear reaction E=mc^2 inside the spacecraft to provide the electricity for the EM Drive, that does not solve the conundrum: the issue is not "to expend energy", the issue is to satisfy conservation of momentum.  If no mass leaves the spacecraft, while kinetic energy is converted into a change in momentum of the spacecraft's center of mass, you still have the same conundrum and the same paradoxes I previously noted:

The needed power for the EM Drive (to escape the surface of the Earth, or anything else you want the spacecraft to do) depends on your frame of reference. 

As Paul March himself admitted, for their Quantum Vacuum explanation for the EM Drive to hold, they need to disrespect the mainstream physics assumption that the Quantum Vacuum is indestructible and immutable.

Bottom line: no, "according to basic physics, it should" not be possible to directly convert electrical energy into a spacecraft's momentum change without any change in mass of the spacecraft (or the action of external forces).  If the EM Drive were to work for space propulsion, it certainly would not be explainable in terms of mainstream physics where conservation of momentum is paramount, and the Quantum Vacuum is both indestructible and immutable.

(http://www.projectrho.com/public_html/rocket/images/reactionlessdrive/sub03.jpg)

Just being curious: What, in your book, would be the most important theoretical consequences of the discovery of a different QV nature?
If the QV is not immutable and indestructible, the theoretical and practical consequences would be so groundbreaking that they would make the 20th century's discoveries of atomic and nuclear energy pale in comparison.
Could you expand on that to help out a poor layperson to these matters, I hadn't realised that the QV being described in this way was such a fundamental cornerstone of physics as it stands?
Title: Re: EM Drive Developments - related to space flight applications - Thread 2
Post by: Rodal on 04/07/2015 10:25 pm
....
Just being curious: What, in your book, would be the most important theoretical consequences of the discovery of a different QV nature?
If the QV is not immutable and indestructible, the theoretical and practical consequences would be so groundbreaking that they would make the 20th century's discoveries of atomic and nuclear energy pale in comparison.
Could you expand on that to help out a poor layperson to these matters, I hadn't realised that the QV being described in this way was such a fundamental cornerstone of physics as it stands?
The mainstream physics community assumes the Quantum Vacuum is indestructible and immutable because of the experimental observation that a fundamental particle like an electron (or a positron) has the same properties (e.g. mass, charge or spin), regardless of when or where the particle was created, whether now or in the early universe, through astrophysical processes or in a laboratory.  Another reason is that the Quantum Vacuum has what Albert Einstein in 1913 called "Nullpunktsenergie" (zero point energy): the lowest possible (time-averaged) energy that a quantum mechanical physical system may have.

If the Quantum Vacuum is instead like a fluid (experiencing vortices and streamlines for example) as Paul March hinted (perhaps because our universe is part of a higher multidimensional multiverse as described by string theory and the QV was there before the Big Bang) and if it can be used for space propulsion, it may also be used for energy (as a loose analogy one can use the wind to sail the ocean as well as to produce energy with windmills) for peaceful as well as for destructive purposes... Of course, this is just science fiction at the moment, certainly as to the destructive purposes (as we cannot yet even control the weather for destructive purposes, although we have been able to use the wind for sailing and energy production for centuries).
Title: Re: EM Drive Developments - related to space flight applications - Thread 2
Post by: birchoff on 04/07/2015 10:43 pm
....
Just being curious: What, in your book, would be the most important theoretical consequences of the discovery of a different QV nature?
If the QV is not immutable and indestructible, the theoretical and practical consequences would be so groundbreaking that they would make the 20th century's discoveries of atomic and nuclear energy pale in comparison.
Could you expand on that to help out a poor layperson to these matters, I hadn't realised that the QV being described in this way was such a fundamental cornerstone of physics as it stands?
The mainstream physics community assumes the Quantum Vacuum is indestructible and immutable because of the experimental observation that a fundamental particle like an electron (or a positron) has the same properties (e.g. mass, charge or spin), regardless of when or where the particle was created, whether now or in the early universe, through astrophysical processes or in a laboratory.  Another reason is that the Quantum Vacuum has what Albert Einstein in 1913 called "Nullpunktsenergie" (zero point energy): the lowest possible (time-averaged) energy that a quantum mechanical physical system may have.

If the Quantum Vacuum is instead like a fluid (experiencing vortices and streamlines for example) as Paul March hinted (perhaps because our universe is part of a higher multidimensional multiverse as described by string theory and the QV was there before the Big Bang) and if it can be used for space propulsion, it may also be used for energy (as a loose analogy one can use the wind to sail the ocean as well as to produce energy with windmills) for peaceful as well as for destructive purposes... Of course, this is just science fiction at the moment, certainly as to the destructive purposes (as we cannot yet even control the weather for destructive purposes, although we have been able to use the wind for sailing and energy production for centuries).

Is there record of a previously carried out experiment where the properties of a fundamental particle were measured in an environment identical to that which exists inside of the tapered frustum at EagleWorks?
Title: Re: EM Drive Developments - related to space flight applications - Thread 2
Post by: Star One on 04/07/2015 10:46 pm

....
Just being curious: What, in your book, would be the most important theoretical consequences of the discovery of a different QV nature?
If the QV is not immutable and indestructible, the theoretical and practical consequences would be so groundbreaking that they would make the 20th century's discoveries of atomic and nuclear energy pale in comparison.
Could you expand on that to help out a poor layperson to these matters, I hadn't realised that the QV being described in this way was such a fundamental cornerstone of physics as it stands?
The mainstream physics community assumes the Quantum Vacuum is indestructible and immutable because of the experimental observation that a fundamental particle like an electron (or a positron) has the same properties (e.g. mass, charge or spin), regardless of when or where the particle was created, whether now or in the early universe, through astrophysical processes or in a laboratory.  Another reason is that the Quantum Vacuum has what Albert Einstein in 1913 called "Nullpunktsenergie" (zero point energy): the lowest possible (time-averaged) energy that a quantum mechanical physical system may have.

If the Quantum Vacuum is instead like a fluid (experiencing vortices and streamlines for example) as Paul March hinted (perhaps because our universe is part of a higher multidimensional multiverse as described by string theory and the QV was there before the Big Bang) and if it can be used for space propulsion, it may also be used for energy (as a loose analogy one can use the wind to sail the ocean as well as to produce energy with windmills) for peaceful as well as for destructive purposes... Of course, this is just science fiction at the moment, certainly as to the destructive purposes (as we cannot yet even control the weather for destructive purposes, although we have been able to use the wind for sailing and energy production for centuries).

Thank you for that clear explanation, is it something that could ever be proved either way experimentally? It sounds a little like the assumption that Gravity is the same everywhere in the universe which some are now disputing. But that's thread drift again.
Title: Re: EM Drive Developments - related to space flight applications - Thread 2
Post by: CW on 04/07/2015 10:46 pm
....
Just being curious: What, in your book, would be the most important theoretical consequences of the discovery of a different QV nature?
If the QV is not immutable and indestructible, the theoretical and practical consequences would be so groundbreaking that they would make the 20th century's discoveries of atomic and nuclear energy pale in comparison.
Could you expand on that to help out a poor layperson to these matters, I hadn't realised that the QV being described in this way was such a fundamental cornerstone of physics as it stands?
The mainstream physics community assumes the Quantum Vacuum is indestructible and immutable because of the experimental observation that a fundamental particle like an electron (or a positron) has the same properties (e.g. mass, charge or spin), regardless of when or where the particle was created, whether now or in the early universe, through astrophysical processes or in a laboratory.  Another reason is that the Quantum Vacuum has what Albert Einstein in 1913 called "Nullpunktsenergie" (zero point energy): the lowest possible (time-averaged) energy that a quantum mechanical physical system may have.

If the Quantum Vacuum is instead like a fluid (experiencing vortices and streamlines for example) as Paul March hinted (perhaps because our universe is part of a higher multidimensional multiverse as described by string theory and the QV was there before the Big Bang) and if it can be used for space propulsion, it may also be used for energy (as a loose analogy one can use the wind to sail the ocean as well as to produce energy with windmills) for peaceful as well as for destructive purposes... Of course, this is just science fiction at the moment, certainly as to the destructive purposes (as we cannot yet even control the weather for destructive purposes, although we have been able to use the wind for sailing and energy production for centuries).

OK.. if I understand this correctly, I think an example would then be a possibility to extract energy from gravitational fields, because one could then view gravitating bodies as a drain for this 'fluid' space and use the 'space current' for energy production? Sounds.. interesting, to say the least. I can see why this would have enormous implications.
Title: Re: EM Drive Developments - related to space flight applications - Thread 2
Post by: Rodal on 04/07/2015 11:24 pm

....
Just being curious: What, in your book, would be the most important theoretical consequences of the discovery of a different QV nature?
If the QV is not immutable and indestructible, the theoretical and practical consequences would be so groundbreaking that they would make the 20th century's discoveries of atomic and nuclear energy pale in comparison.
Could you expand on that to help out a poor layperson to these matters, I hadn't realised that the QV being described in this way was such a fundamental cornerstone of physics as it stands?
The mainstream physics community assumes the Quantum Vacuum is indestructible and immutable because of the experimental observation that a fundamental particle like an electron (or a positron) has the same properties (e.g. mass, charge or spin), regardless of when or where the particle was created, whether now or in the early universe, through astrophysical processes or in a laboratory.  Another reason is that the Quantum Vacuum has what Albert Einstein in 1913 called "Nullpunktsenergie" (zero point energy): the lowest possible (time-averaged) energy that a quantum mechanical physical system may have.

If the Quantum Vacuum is instead like a fluid (experiencing vortices and streamlines for example) as Paul March hinted (perhaps because our universe is part of a higher multidimensional multiverse as described by string theory and the QV was there before the Big Bang) and if it can be used for space propulsion, it may also be used for energy (as a loose analogy one can use the wind to sail the ocean as well as to produce energy with windmills) for peaceful as well as for destructive purposes... Of course, this is just science fiction at the moment, certainly as to the destructive purposes (as we cannot yet even control the weather for destructive purposes, although we have been able to use the wind for sailing and energy production for centuries).

Thank you for that clear explanation, is it something that could ever be proved either way experimentally? It sounds a little like the assumption that Gravity is the same everywhere in the universe which some are now disputing. But that's thread drift again.
The movie Interstellar counted with a team of leading, well respected, academic Physicists, led by Kip Thorne from CalTech as consultants.  Look at the end of Interstellar for what some leading physicists think that the future may bring (the part after the leading actor comes back from the black hole)...
Title: Re: EM Drive Developments - related to space flight applications - Thread 2
Post by: Rodal on 04/08/2015 02:09 am
Virtual photons travelling arbitrarily slower than light, may transmit information without transmitting energy, due to Casimir-type interaction in the Quantum Vacuum

http://phys.org/news/2015-03-photon-afterglow-transmit-transmitting-energy.html

Quote
we use the fact that, when real photons are emitted (and propagate at the speed of light), the photons leave a small afterglow of virtual photons that propagate slower than light. This afterglow does not carry energy (in contrast to real photons), but it does carry information about the event that generated the light. Receivers can 'tap' into that afterglow, spending energy to recover information about light that passed by a long time ago



http://journals.aps.org/prl/abstract/10.1103/PhysRevLett.114.110505


Quote
We show that it is possible to use a massless field in the vacuum to communicate in such a way that the signal travels arbitrarily slower than the speed of light and such that no energy is transmitted from the sender to the receiver. Instead, the receiver has to supply a signal-dependent amount of work to switch his detector on and off. This type of communication is related to Casimir-like interactions, and it is made possible by dimension—and curvature—dependent subtleties of Huygens’ principle.

Title: Re: EM Drive Developments - related to space flight applications - Thread 2
Post by: zen-in on 04/08/2015 02:49 am
....

     In theory, according to basic physics, it should be possible to convert one form of energy into another, such as kinetic energy being converted to heat energy as a vehicle is decellerated via air during reentry.
    ...


If electrical energy can be converted directly into spacecraft's momentum, without any matter or energy leaving the spacecraft then many problems appear.

One interesting problem has been repeatedly pointed out by @frobnicat in this thread.  Since the kinetic energy is:

K = (1/2) m v^2

The power needed to accelerate (a question appearing in this thread over the last few pages concerning a comparison with chemical rockets) is dK/dt,and since in this EM Drive spacecraft, the mass doesn't change, the only thing that changes is the velocity, therefore:

dK/dt = (1/2) m [2 v dv/dt] = m v a

So the power needed to accelerate is a function of not only the acceleration wanted, but also the speed at which you're currently traveling.  But, according to relativity, there is no absolute measurement of spacecraft speed, it depends on the observer. So, the needed power (to escape the surface of the Earth, etc.) depends on your frame of reference. 

So, no, "according to basic physics, it should" not be possible to directly convert electrical energy into a spacecraft's momentum change without any change in mass of the spacecraft (or the action of external forces).

There are several paradoxes  with the EM Drive related to this.


A possible exception to this restriction is the lorentz force.   Satellites use torque coils to dump angular momentum.   Space tethers, if they ever are shown to work, transfer momentum to the Earth when current flowing along the tether is normal to the Earth's magnetic field.   The return path is provided by ions.   In fact any current-carrying wire experiences a lorentz force with a direction that is the cross product of the current direction and the magnetic field direction.  The force is the same if the wire is stationary or is moving.  So, in the case of an electrodynamic tether the restriction:

dK/dt = (1/2) m [2 v dv/dt] = m v a

Doesn't apply.    How this applies to the EM-drive I can't even guess.
Title: Re: EM Drive Developments - related to space flight applications - Thread 2
Post by: Star-Drive on 04/08/2015 04:31 am
....
Be it nuclear, fuel cell or even solar, as the source of electricity, mass is being expended in the form of electrons. (Solar is taking advantage of the Sun's own expendature of mass in the form of photons).  Whiloe the quantity of mass being expended is minute, it IS being expended.
   ...
Even if (for discussion's sake) mass would be converted into energy with a nuclear reaction E=mc^2 inside the spacecraft to provide the electricity for the EM Drive, that does not solve the conundrum: the issue is not "to expend energy", the issue is to satisfy conservation of momentum.  If no mass leaves the spacecraft, while kinetic energy is converted into a change in momentum of the spacecraft's center of mass, you still have the same conundrum and the same paradoxes I previously noted:

The needed power for the EM Drive (to escape the surface of the Earth, or anything else you want the spacecraft to do) depends on your frame of reference. 

As Paul March himself admitted, for their Quantum Vacuum explanation for the EM Drive to hold, they need to disrespect the mainstream physics assumption that the Quantum Vacuum is indestructible and immutable.

Bottom line: no, "according to basic physics, it should" not be possible to directly convert electrical energy into a spacecraft's momentum change without any change in mass of the spacecraft (or the action of external forces).  If the EM Drive were to work for space propulsion, it certainly would not be explainable in terms of mainstream physics where conservation of momentum is paramount, and the Quantum Vacuum is both indestructible and immutable.

(http://www.projectrho.com/public_html/rocket/images/reactionlessdrive/sub03.jpg)

Just being curious: What, in your book, would be the most important theoretical consequences of the discovery of a different QV nature?
If the QV is not immutable and indestructible, the theoretical and practical consequences would be so groundbreaking that they would make the 20th century's discoveries of atomic and nuclear energy pale in comparison.

Dr. Rodal:

"If the QV is not immutable and indestructible,''')

We've not tired to make that a secret and in fact that is at the core of our Q-V conjecture and Q-V plasma code that Dr. White and Dr. Vera have written that produced the Q-Thruster thrust predictions that I posted earlier on this forum.  And this new paper is an expansion of Dr. White's STAIF-2007 conjecture and a partial rebuttal to the criticisms raised by the NASA Blue Ribbon panel's critique from last summer, an independent body of eight PhDs knowledge in the field that was created by NASA/JSC/EP management to vet Dr. White's QVF/MHD conjecture.  And yes, if the accumulated chemical and nuclear data keeps pointing us in the same direction as it and our own experimental data has so far, we will be able to transmit and receive momentum through the Q-V via Q-Thruster like device.  AND ultimately, be able harvest energy from the Q-V based cosmological gravitational field via various thermodynamic processes, at least in the far term.  So find attached the Abstract and Introduction of our "Dynamics of the Vacuum" paper that will be out on the NASA servers hopefully by the end of April.

Best, Paul M.

Title: Re: EM Drive Developments - related to space flight applications - Thread 2
Post by: dustinthewind on 04/08/2015 08:12 am
....
Just being curious: What, in your book, would be the most important theoretical consequences of the discovery of a different QV nature?
If the QV is not immutable and indestructible, the theoretical and practical consequences would be so groundbreaking that they would make the 20th century's discoveries of atomic and nuclear energy pale in comparison.
Could you expand on that to help out a poor layperson to these matters, I hadn't realised that the QV being described in this way was such a fundamental cornerstone of physics as it stands?
The mainstream physics community assumes the Quantum Vacuum is indestructible and immutable because of the experimental observation that a fundamental particle like an electron (or a positron) has the same properties (e.g. mass, charge or spin), regardless of when or where the particle was created, whether now or in the early universe, through astrophysical processes or in a laboratory.  Another reason is that the Quantum Vacuum has what Albert Einstein in 1913 called "Nullpunktsenergie" (zero point energy): the lowest possible (time-averaged) energy that a quantum mechanical physical system may have.

If the Quantum Vacuum is instead like a fluid (experiencing vortices and streamlines for example) as Paul March hinted (perhaps because our universe is part of a higher multidimensional multiverse as described by string theory and the QV was there before the Big Bang) and if it can be used for space propulsion, it may also be used for energy (as a loose analogy one can use the wind to sail the ocean as well as to produce energy with windmills) for peaceful as well as for destructive purposes... Of course, this is just science fiction at the moment, certainly as to the destructive purposes (as we cannot yet even control the weather for destructive purposes, although we have been able to use the wind for sailing and energy production for centuries).

I have been toying with a similar idea myself.  The idea was connected with the idea of space time as a super-fluid.  Our motion with respect to another bodies surrounding super-fluid gives the effects of time dilation and pancaking possibly.  I wanted to connect it with the idea that a ship resists acceleration because it is accelerating its local space though I have to admit this may be a bit far fetched. 

The idea extends to earths gravity and uses matter as if it is a hole in which the super-fluid flows into.  But then where does it come out?  - dark energy? -(don't know).  So the fluid flows into the earth with increasing velocity by 1/r^2 and drags us.  We observer the difference in velocity with respect to the space as the slowing of clocks also?  As we go into the earth gravity decreases as the fluid disappears or the fluid velocity drops to zero linearly. 

An initial thought I had looking at the equation was what if "v" is motion with respect to that fluid?  Seems iffy to me but the thought was what if it could push that fluid. 
dK/dt = (1/2) m [2 v dv/dt] = m v a
I know I hear a lot of people say not to use the 1/2 m*v^2=K for some matters and was thinking to taking a look at E=sqrt(p^2*c^2+m_o^2*c^4) http://hyperphysics.phy-astr.gsu.edu/hbase/relativ/releng.html and see what it would look like. 

Another line of thought was the magnets out of phase (I mentioned earlier) by 90 degrees and 1/4\lambda appear to push by manipulating space and time information.  The thought was I wonder if it is possible it could push space time because of that but I can't really say that idea would work for sure.  There does seem to be a balance of static electric and relativistic magnetic edit:{forces} that oppose each other. 

for 2.45ghz I get about 12.25cm spacing for 1/4 wavelength but used 3E8m/s.  I will have to find dimensions of the cavities with magnetic fields and try and identify current loops with spacing.  It would seem that if there were two current loops inside they should be 180 degrees out of phase because the 1st current loop would be a counter current of the 2nd current loop but maybe the conical cavity is changing that?  Of course then there are the TM modes but I'm not thinking of those at the moment. 

I think I have wanted to say space was a fluid after hearing how rotating black holes twist space into a vortex and thinking of earth gravity and acceleration in a space ship but I'm not convinced it is a fluid. 

Getting some sleep for now. 
Title: Re: EM Drive Developments - related to space flight applications - Thread 2
Post by: Left Field on 04/08/2015 08:31 am
@Star-Drive

This is all very exciting. I cannot wait to read it and then pretend I understand.

The very best of luck to you Paul and your colleagues, and no pressure, but:

https://www.youtube.com/watch?v=SmHeP9Sve48]
Title: Re: EM Drive Developments - related to space flight applications - Thread 2
Post by: WBY1984 on 04/08/2015 08:58 am
@Star-Drive

This is all very exciting. I cannot wait to read it and then pretend I understand.

The very best of luck to you Paul and your colleagues, and no pressure, but:


It's already past the point when the Eagleworks team should have found results or their funding gets pulled, and there has been no response to a contributor on this thread who asked a direct question about this point. I wouldn't get your hopes up.
Title: Re: EM Drive Developments - related to space flight applications - Thread 2
Post by: Star One on 04/08/2015 10:58 am
@Star-Drive

This is all very exciting. I cannot wait to read it and then pretend I understand.

The very best of luck to you Paul and your colleagues, and no pressure, but:


It's already past the point when the Eagleworks team should have found results or their funding gets pulled, and there has been no response to a contributor on this thread who asked a direct question about this point. I wouldn't get your hopes up.

I would like to think that if it had been pulled there would be some indication of this, but there hasn't been so far, therefore it's reasonable to assume that either it hasn't or it is in the process of being reviewed.
Title: Re: EM Drive Developments - related to space flight applications - Thread 2
Post by: Rodal on 04/08/2015 12:20 pm
....

     In theory, according to basic physics, it should be possible to convert one form of energy into another, such as kinetic energy being converted to heat energy as a vehicle is decellerated via air during reentry.
    ...


If electrical energy can be converted directly into spacecraft's momentum, without any matter or energy leaving the spacecraft then many problems appear.

One interesting problem has been repeatedly pointed out by @frobnicat in this thread.  Since the kinetic energy is:

K = (1/2) m v^2

The power needed to accelerate (a question appearing in this thread over the last few pages concerning a comparison with chemical rockets) is dK/dt,and since in this EM Drive spacecraft, the mass doesn't change, the only thing that changes is the velocity, therefore:

dK/dt = (1/2) m [2 v dv/dt] = m v a

So the power needed to accelerate is a function of not only the acceleration wanted, but also the speed at which you're currently traveling.  But, according to relativity, there is no absolute measurement of spacecraft speed, it depends on the observer. So, the needed power (to escape the surface of the Earth, etc.) depends on your frame of reference. 

So, no, "according to basic physics, it should" not be possible to directly convert electrical energy into a spacecraft's momentum change without any change in mass of the spacecraft (or the action of external forces).

There are several paradoxes  with the EM Drive related to this.


A possible exception to this restriction is the lorentz force.   Satellites use torque coils to dump angular momentum.   Space tethers, if they ever are shown to work, transfer momentum to the Earth when current flowing along the tether is normal to the Earth's magnetic field.   The return path is provided by ions.   In fact any current-carrying wire experiences a lorentz force with a direction that is the cross product of the current direction and the magnetic field direction.  The force is the same if the wire is stationary or is moving.  So, in the case of an electrodynamic tether the restriction:

dK/dt = (1/2) m [2 v dv/dt] = m v a

Doesn't apply.    How this applies to the EM-drive I can't even guess.

Notice that I had excluded magnetic tethers from these paradoxes:

Quote from: Rodal
"according to basic physics, it should" not be possible to directly convert electrical energy into a spacecraft's momentum change without any change in mass of the spacecraft (or the action of external forces).

Therefore, space tethers, had already been explicitly excluded under "the action of external forces".

Space tethers can only work through the action of the external magnetic force. One cannot get a space tether to work in space sufficiently away from the external magnetic field.  Space tethers only work sufficiently close to magnetic fields, for example close to the magnetic field of the Earth.

Quote from: zen-in
How this applies to the EM-drive I can't even guess. 

All that is needed is to consider whether the system is an open system or a closed system.

If there is no mass leaving the EM Drive and there are no external fields changing its momentum, it is a closed system.

If one considers the EM Drive as a closed system, as done for example by Shawyer in his "explanation"  then there are paradoxes in the EM Drive as I wrote, due to the fact that

dK/dt = (1/2) m [2 v dv/dt] = m v a

implies a frame-dependence because the speed "v" is frame dependent.

On the other hand if one explains the EM Drive as an open system (for example due to coupling of electricity with gravitation, or any other external field) then there is no paradox, just like there is no paradox for magnetic tethers.

Again, what scientists find perturbing about the EM Drive is conservation of momentum.  If the EM Drive is a closed system, then momentum is not conserved, which doesn't make scientific sense.

If the EM Drive is considered as an open system under the action of external forces, then momentum can be shown to be conserved and there is no paradox. Magnetic tethers work through the action of external magnetic forces, hence there is no issue with conservation of momentum.
Title: Re: EM Drive Developments - related to space flight applications - Thread 2
Post by: Star-Drive on 04/08/2015 12:47 pm
@Star-Drive

This is all very exciting. I cannot wait to read it and then pretend I understand.

The very best of luck to you Paul and your colleagues, and no pressure, but:


It's already past the point when the Eagleworks team should have found results or their funding gets pulled, and there has been no response to a contributor on this thread who asked a direct question about this point. I wouldn't get your hopes up.

Whoops, let that one slip by.  Just for the record I think due to team's other work on the theoretical side the Q-V, I was given a reprieve on my contract termination date until the end of September 2015.  Management still wants us to perform and IV&V at Glenn Research Center (GRC), but appears to be willing to wait a few months more until we can get our arms around increasing the current test setup's thrust up to a repeatable 100+uN force every time we apply power.  Right now its about every third time that I can find the "Just-So" conditions needed to evoke the thrust signature in the reversed thrust mode.

Best, Paul M.
Title: Re: EM Drive Developments - related to space flight applications - Thread 2
Post by: WBY1984 on 04/08/2015 01:22 pm
With repect, if it's not repeatable, I'd be concerned I wasn't in control of every element of my test setup. In which case who knows where such a tiny thrust signature could be coming from.
Title: Re: EM Drive Developments - related to space flight applications - Thread 2
Post by: birchoff on 04/08/2015 01:45 pm
@Star-Drive

This is all very exciting. I cannot wait to read it and then pretend I understand.

The very best of luck to you Paul and your colleagues, and no pressure, but:


It's already past the point when the Eagleworks team should have found results or their funding gets pulled, and there has been no response to a contributor on this thread who asked a direct question about this point. I wouldn't get your hopes up.

Whoops, let that one slip by.  Just for the record I think due to team's other work on the theoretical side the Q-V, I was given a reprieve on my contract termination date until the end of September 2015.  Management still wants us to perform and IV&V at Glenn Research Center (GRC), but appears to be willing to wait a few months more until we can get our arms around increasing the current test setup's thrust up to a repeatable 100+uN force every time we apply power.  Right now its about every third time that I can find the "Just-So" conditions needed to evoke the thrust signature in the reversed thrust mode.

Best, Paul M.

Is the inability to hit the "Just-So" conditions a side effect of some sort of degredation of either the cavity (as a result of thermal expansion or something similar) or the RF amplifier (failing while in vacuum)? If not can you share any ideas on what you think is impeding repeatability at the 100un level and above?
Title: Re: EM Drive Developments - related to space flight applications - Thread 2
Post by: Star One on 04/08/2015 02:15 pm
@Star-Drive

This is all very exciting. I cannot wait to read it and then pretend I understand.

The very best of luck to you Paul and your colleagues, and no pressure, but:


It's already past the point when the Eagleworks team should have found results or their funding gets pulled, and there has been no response to a contributor on this thread who asked a direct question about this point. I wouldn't get your hopes up.

Whoops, let that one slip by.  Just for the record I think due to team's other work on the theoretical side the Q-V, I was given a reprieve on my contract termination date until the end of September 2015.  Management still wants us to perform and IV&V at Glenn Research Center (GRC), but appears to be willing to wait a few months more until we can get our arms around increasing the current test setup's thrust up to a repeatable 100+uN force every time we apply power.  Right now its about every third time that I can find the "Just-So" conditions needed to evoke the thrust signature in the reversed thrust mode.

Best, Paul M.

Is the inability to hit the "Just-So" conditions a side effect of some sort of degredation of either the cavity (as a result of thermal expansion or something similar) or the RF amplifier (failing while in vacuum)? If not can you share any ideas on what you think is impeding repeatability at the 100un level and above?

Hasn't he already mentioned issues with failures in some test equipment after a certain amount of use especially the RF amplifier.
Title: Re: EM Drive Developments - related to space flight applications - Thread 2
Post by: JasonAW3 on 04/08/2015 02:18 pm
....
Be it nuclear, fuel cell or even solar, as the source of electricity, mass is being expended in the form of electrons. (Solar is taking advantage of the Sun's own expendature of mass in the form of photons).  Whiloe the quantity of mass being expended is minute, it IS being expended.
   ...
Even if (for discussion's sake) mass would be converted into energy with a nuclear reaction E=mc^2 inside the spacecraft to provide the electricity for the EM Drive, that does not solve the conundrum: the issue is not "to expend energy", the issue is to satisfy conservation of momentum.  If no mass leaves the spacecraft, while kinetic energy is converted into a change in momentum of the spacecraft's center of mass, you still have the same conundrum and the same paradoxes I previously noted:

The needed power for the EM Drive (to escape the surface of the Earth, or anything else you want the spacecraft to do) depends on your frame of reference. 

As Paul March himself admitted, for their Quantum Vacuum explanation for the EM Drive to hold, they need to disrespect the mainstream physics assumption that the Quantum Vacuum is indestructible and immutable.

Bottom line: no, "according to basic physics, it should" not be possible to directly convert electrical energy into a spacecraft's momentum change without any change in mass of the spacecraft (or the action of external forces).  If the EM Drive were to work for space propulsion, it certainly would not be explainable in terms of mainstream physics where conservation of momentum is paramount, and the Quantum Vacuum is both indestructible and immutable.

(http://ecx.images-amazon.com/images/I/51dQFZfQT2L._SY344_BO1,204,203,200_.jpg)

Alright, different angle.

Is the device creating a closed magnetic field?  It may be possible that it maybe interacting with the Earth's own magnetic field in such a way as to "borrow" energy and momentum from the Earth's own magnetic field.

     While very weak locally, it is pervasive and overall, has a tremendous amount of potentil energy that can be tapped into.  The explaination may be as simple as that.

     I take it the device has also been tested in a North / South and East / West orientations for potential motion induced be the Earth's own rotation?
Title: Re: EM Drive Developments - related to space flight applications - Thread 2
Post by: Rodal on 04/08/2015 05:28 pm
....
Be it nuclear, fuel cell or even solar, as the source of electricity, mass is being expended in the form of electrons. (Solar is taking advantage of the Sun's own expendature of mass in the form of photons).  Whiloe the quantity of mass being expended is minute, it IS being expended.
   ...
Even if (for discussion's sake) mass would be converted into energy with a nuclear reaction E=mc^2 inside the spacecraft to provide the electricity for the EM Drive, that does not solve the conundrum: the issue is not "to expend energy", the issue is to satisfy conservation of momentum.  If no mass leaves the spacecraft, while kinetic energy is converted into a change in momentum of the spacecraft's center of mass, you still have the same conundrum and the same paradoxes I previously noted:

The needed power for the EM Drive (to escape the surface of the Earth, or anything else you want the spacecraft to do) depends on your frame of reference. 

As Paul March himself admitted, for their Quantum Vacuum explanation for the EM Drive to hold, they need to disrespect the mainstream physics assumption that the Quantum Vacuum is indestructible and immutable.

Bottom line: no, "according to basic physics, it should" not be possible to directly convert electrical energy into a spacecraft's momentum change without any change in mass of the spacecraft (or the action of external forces).  If the EM Drive were to work for space propulsion, it certainly would not be explainable in terms of mainstream physics where conservation of momentum is paramount, and the Quantum Vacuum is both indestructible and immutable.

(http://www.projectrho.com/public_html/rocket/images/reactionlessdrive/sub03.jpg)

Just being curious: What, in your book, would be the most important theoretical consequences of the discovery of a different QV nature?
If the QV is not immutable and indestructible, the theoretical and practical consequences would be so groundbreaking that they would make the 20th century's discoveries of atomic and nuclear energy pale in comparison.

Dr. Rodal:

"If the QV is not immutable and indestructible,''')

We've not tired to make that a secret and in fact that is at the core of our Q-V conjecture and Q-V plasma code that Dr. White and Dr. Vera have written that produced the Q-Thruster thrust predictions that I posted earlier on this forum.  And this new paper is an expansion of Dr. White's STAIF-2007 conjecture and a partial rebuttal to the criticisms raised by the NASA Blue Ribbon panel's critique from last summer, an independent body of eight PhDs knowledge in the field that was created by NASA/JSC/EP management to vet Dr. White's QVF/MHD conjecture.  And yes, if the accumulated chemical and nuclear data keeps pointing us in the same direction as it and our own experimental data has so far, we will be able to transmit and receive momentum through the Q-V via Q-Thruster like device.  AND ultimately, be able harvest energy from the Q-V based cosmological gravitational field via various thermodynamic processes, at least in the far term.  So find attached the Abstract and Introduction of our "Dynamics of the Vacuum" paper that will be out on the NASA servers hopefully by the end of April.

Best, Paul M.

Paul,

Well, certainly the NASA Eagleworks team has very strong convictions and guts.  Pursuing the conjecture that the QV is not immutable and indestructible will run full front against physicists like Prof. John Baez and Dr. Sean Carroll that not only have commented that one cannot extract momentum from the Quantum Vacuum, but that extracting energy from the QV is impossible.   Baez, for example, wrote:


Quote from: Baez http://www.scientificamerican.com/article/follow-up-what-is-the-zer/
"One should not take this vacuum energy too literally, however, because the free-field theory is just a mathematical tool to help us understand what we are really interested in: the interacting theory. Only the interacting theory is supposed to correspond directly to reality. Because the vacuum state of the interacting theory is the state of least energy in reality, there is no way to extract the vacuum energy and use it for anything.

"It is a bit like this: say a bank found it more convenient (for some strange reason) to start counting at 1,000, so that even when you had no money in the bank, your account read $1,000. You might get excited and try to spend this $1,000, but the bank would say, 'Sorry, that $1,000 is just an artifact of how we do our bookkeeping: you're actually flat broke.'

"Similarly, one should not get one's hope up when people talk about vacuum energy. It is just how we do our bookkeeping in quantum field theory. There is much more to say about why we do our bookkeeping this funny way, but I will stop here.



Can you point out people in academia that also think that it may be possible that the Quantum Vacuum is not immutable and indestructible, (whether based on higher dimensional branes using string theory or based on other theories)?

Of course, I am aware that Albert Einstein wrote  ;)

Quote from: Albert Einstein
The one who follows the crowd will usually get no further than the crowd. The one who walks alone, is likely to find himself/herself in places no one has ever been

(http://fallingintowonderland.files.wordpress.com/2013/06/395772_3404170548765_1404926877_3325646_1948945115_n.jpg?w=634)
Title: Re: EM Drive Developments - related to space flight applications - Thread 2
Post by: Star-Drive on 04/09/2015 03:12 am
Dr. Rodal:

Sonny White formulated a compressible quantum vacuum conjecture that requires us to live in a portion of the universe that is immersed in a false vacuum that apparently has a ground or zero-energy level much smaller than science first assumed.  However what will drive this debate is experimental data first and foremost.  Experimental data like what just came out of the Eagleworks Lab's latest warp-field interferometer tests based on 27,000, 1.5 second long on/off data samples that indicates we have finally observed the first spacetime contraction effects that we are fairly confident are the real deal.  We again are looking for more possible false positives as well as ways of increasing the signal to noise ratio above its current ~2-to-3 sigma level, which I've already suggested several ways to do so to Dr. White.  However what is really interesting about these new test results is that the laser interferometer observed spacetime contractions are being developed in a TM010 RF resonant cavity that is driving ac E-field levels over 900kV/m at a 1.48 GHz rate.  A similar RF resonant system used to implement the EM-Drive and Q-thruster designs, for these spacetime contraction effects are paramount to the operation of both.

BTW, we have also started the build of our 1.2kW magnetron powered EM-Drive prototype in a tetter-totter balance system that is being built to replicate the thrust magnitudes of the Shawyer tests and the Chinese replication of same.  Estimated build time should in the 2 month time period with the limited manpower available.  A picture of the chaotic magnetron spectra that will be used on this system is attached along with the TE011 mode that will be driven.

Best,  Paul M.
Title: Re: EM Drive Developments - related to space flight applications - Thread 2
Post by: Stormbringer on 04/09/2015 03:40 am
this might be topical here:  http://phys.org/news/2015-04-electromagnetism-enable-antennas-chip.html

Since it couples electromagnetic radiation in dielectrics with quantum phenomenon.
Title: Re: EM Drive Developments - related to space flight applications - Thread 2
Post by: RotoSequence on 04/09/2015 07:36 am
this might be topical here:  http://phys.org/news/2015-04-electromagnetism-enable-antennas-chip.html

Since it couples electromagnetic radiation in dielectrics with quantum phenomenon.

I found the abstract at Physical Review Letters (http://journals.aps.org/prl/accepted/7107bY8fL5b1214fa0d663c0608801546967531a2):

Quote
We report our observation that radiation from a system of accelerating charges is possible only when there is explicit breaking of symmetry in the electric field in space within the spatial configuration of the radiating system. Under symmetry breaking, current within an enclosed area around the radiating structure is not conserved at a certain instant of time resulting in radiation in free space. Electromagnetic radiation from dielectric and piezoelectric material based resonators are discussed in this context. Finally, it is argued that symmetry of a resonator of any form can be explicitly broken to create a radiating antenna.

I haven't had any luck finding an online link to the paper itself.
Title: Re: EM Drive Developments - related to space flight applications - Thread 2
Post by: Star One on 04/09/2015 08:48 am
It wouldn't surprise me if it turned out to be on a paid for service somewhere, in my limited experience that seems to be the case when you can only locate abstracts.
Title: Re: EM Drive Developments - related to space flight applications - Thread 2
Post by: Flyby on 04/09/2015 11:55 am

BTW, we have also started the build of our 1.2kW magnetron powered EM-Drive prototype in a tetter-totter balance system that is being built to replicate the thrust magnitudes of the Shawyer tests and the Chinese replication of same.  Estimated build time should in the 2 month time period with the limited manpower available.  A picture of the chaotic magnetron spectra that will be used on this system is attached along with the TE011 mode that will be driven.

Best,  Paul M.

Exciting news to hear you're going for the (high) power output. That should help to finalize the debate if this phenomena is real or not...
A door to a new understanding of the world, or just yet another crackpot theory...
If this setup will generate a clear directional force then the controversy can shift towards the theoretical search on how it's  possible...

One of the things to keep in mind while building the new setup, is to have the ability to change the Q factor. I don't know if you can already factor it in now, or that it would be better to keep that for a later phase?
Shawyer's envisioned real world application rests on the notion that a dramatically increased Q will result in a substantial force increase. I've not yet seen any evidence for that...

thumbs up ! and let's hope for a positive result...
Title: Re: EM Drive Developments - related to space flight applications - Thread 2
Post by: Star-Drive on 04/09/2015 12:38 pm
this might be topical here:  http://phys.org/news/2015-04-electromagnetism-enable-antennas-chip.html

Since it couples electromagnetic radiation in dielectrics with quantum phenomenon.

Great find!  And it may provide another way to excite METs, EM-Drives and Q-Thrusters.

Dr. Rodal:

"Can you point out people in academia that also think that it may be possible that the Quantum Vacuum is not immutable and indestructible, (whether based on higher dimensional branes using string theory or based on other theories)?"

I have several papers at work that may satisfy your quest and I'll bring them home this evening to upload them to NSF.  In the meantime here are a couple of papers that are at least related to your question.
Title: Re: EM Drive Developments - related to space flight applications - Thread 2
Post by: Giovanni DS on 04/09/2015 12:42 pm
From the wikipedia article on magnetrons:

"In the post-war era the magnetron became less widely used in the radar role. This was due to the fact that the magnetron's output changes from pulse to pulse, both in frequency and phase."

"Phase is almost never preserved, which makes the magnetron difficult to use in phased array systems. Frequency also drifts pulse to pulse, a more difficult problem for a wider array of radar systems."

How would this phase instability affect this kind of system? the risk is to get incoherent data.

Title: Re: EM Drive Developments - related to space flight applications - Thread 2
Post by: Stormbringer on 04/09/2015 02:07 pm
I don't want to put a tangent on this discussion here but I have a question about something that could pertain to exotic drives of this nature. I posted my question in the "ansible" thread at http://forum.nasaspaceflight.com/index.php?topic=37002.msg1355745#msg1355745

The post is reply 38 in that thread. It's about the implicit negative mass that should have to be involved in turning positive mass particles into  zero mass meta-particles called unparticles in the literature. I would appreciate your thoughts (because there are some heavy hitters in this thread) but over there so as not to further disturb this thread.
Title: Re: EM Drive Developments - related to space flight applications - Thread 2
Post by: Rodal on 04/09/2015 03:55 pm
I don't want to put a tangent on this discussion here but I have a question about something that could pertain to exotic drives of this nature. I posted my question in the "ansible" thread at http://forum.nasaspaceflight.com/index.php?topic=37002.msg1355745#msg1355745

The post is reply 38 in that thread. It's about the implicit negative mass that should have to be involved in turning positive mass particles into  zero mass meta-particles called unparticles in the literature. I would appreciate your thoughts (because there are some heavy hitters in this thread) but over there so as not to further disturb this thread.

There is no mention of negative mass (implicit or explicit) in Georgi's presentations:

http://www.people.fas.harvard.edu/~hgeorgi/public-slides.pdf

http://www.people.fas.harvard.edu/~hgeorgi/utalkc.pdf

or in his papers:

http://arxiv.org/pdf/hep-ph/0703260v3.pdf

http://arxiv.org/pdf/0704.2457v2.pdf

In fact, I don't think that Georgi's theory has anything to do with negative mass.  (and I'm glad about that  ;) )

Quote from: Georgi
We know the missing mass is zero if there is a single invisible massless particle. what missing mass do we expect for two? In any given event, we don’t know, but if we repeat the process many times, the angle between the two invisible particle momenta will be distributed at random.  If there are three invisible massless particles, all three momenta have to line up exactly to get zero missing mass, even less likely to happen at random! So small missing mass is even more unlikely, and the number of events as a function of missing mass would increase even faster

Quote from: FERMILAB  http://www.fnal.gov/pub/today/archive/archive_2014/today14-10-23.html
In a particle detector, unparticles would look like particles with random masses. One unparticle decay might have many times the apparent mass of the next — the distribution would be broad.

Also keep in mind that this unparticle  stuff is still a theoretical conjecture and that such unparticle stuff has not been verified in experiments.

(http://inspirehep.net/record/779711/files/RHO0.png)
Title: Re: EM Drive Developments - related to space flight applications - Thread 2
Post by: Mulletron on 04/09/2015 08:07 pm
While that's all very interesting. You can immediately rule out the drive's functionality because of the lack of reaction mass. CoM is clearly broken.

I only have a minor in physics so I honestly can't understand most of what's talked about here, but I know at the least you can't go about thrusting around the universe with no particles/energy leaving the engine without something like an Alcubierre drive. Even if you're pushing against quantum virtual particle pairs, virtual particle pairs have opposite signs so will be propelled in opposite directions thus canceling out any net force.

I have yet to year a simple explanation of how this drive works that doesn't dive into theoretical physics to explain its operation. It is true of everything in physics that simple laws lead to complex behavior. If the operation of the EmDrive or other various families of the drive cannot be explained in an abstract way within a couple sentences then it doesn't work.

People are being sold snake oil and I don't like it.

"We don't know how, why, or if it works and known physics do not readily suffice, therefore it doesn't work and its proponents are perpetuating fraud" is jumping the gun a bit, don't you think?

Tornados don't produces magical instruments. They produce junk. Luck doesn't make engineering.

The chances of someone stumbling upon some contraption that violates physics principles by throwing electronics parts out of his garage together is nil.

"I  only have a minor in physics so I honestly can't understand most of what's talked about here, but I know at the least you can't go about thrusting around the universe with no particles/energy leaving the engine without something like an Alcubierre drive. Even if you're pushing against quantum virtual particle pairs, virtual particle pairs have opposite signs so will be propelled in opposite directions thus canceling out any net force."

In a neutral plasma there is an equal number of plus and minus electrical charges or ions that can react to applied E-fields and B-fields in various ways.  If there is only an electric field applied to the plasma volume then yes the positive charges will go one way and the negative charges will go in the opposite direction.  However if we apply a spatially crossed E-field and B-field across this volume, then we have a Lorentz force produced on the plasma ions that is at right angles to the applied E-field and B-field.  Then BOTH the positive and negative ions will be accelerated in the SAME direction, but with counter rotating twists AKA Gyro radius modifying their accelerated trajectories. All of these EM-Drive like thruster utilize some form of this Lorentz force acceleration on some type of propellant, be it real as in a Hall thruster or semi-virtual.

Best, Paul M.

Playing devil's advocate, I've spent two days researching this but I simply can't find any evidence which can support that virtual particles can be accelerated and used as propellant in this fashion.

Evidence from multiple sources suggests treating virtual particle pairs as a plasma to be accelerated by the Lorentz force is not possible.

In addition, the Lorentz invariance of the QV precludes this.

Every resource I can find tells me that virtual particles are not on mass shell, are not subject to the classical equations of motion, and can't be accelerated by the Lorentz force.

http://web.mit.edu/course/22/22.09/ClassHandouts/Charged%20Particle%20Accel/CHAP03.PDF
http://www.perimeterinstitute.ca/news/new-face-feynman-diagrams/deeper-dive-shell-and-shell
http://profmattstrassler.com/articles-and-posts/particle-physics-basics/virtual-particles-what-are-they/
http://cronodon.com/Atomic/QED.html
http://en.wikipedia.org/wiki/Equations_of_motion#Electrodynamics
http://en.wikipedia.org/wiki/On_shell_and_off_shell
https://universe-review.ca/R15-17-relativity01.htm








Title: Re: EM Drive Developments - related to space flight applications - Thread 2
Post by: Rodal on 04/09/2015 11:27 pm
Concerning the issue of Lorentz covariance,  Paul March has pointed towards multi-dimensional branes in string theory and multi-vacuum states.

In quantum gravity, there doesn't seem to be a single unique observer-independent notion of a quantum gravity vacuum state (examples: the Unruh effect and Hawking radiation), hence no Lorentz covariance. Perhaps in quantum gravity the whole family of states taken together are Poincare invariant or invariant under a higher group. 

Even if this would be the case, I don't understand how these (otherwise inaccessible extra dimensions of string theory) multi-vacuum states would be accessible with simple EM Drive experiments: they involve comparatively low energies (we are not dealing with a particle accelerator and much less with a black hole), they are conducted at room temperature, and they involve electromagnetic standing waves and therefore the Poynting vector is constantly switching sign at a frequency twice as high as the frequency of the electromagnetic fields in the EM Drive. 

Also I cannot understand where the direction of the force in Dr. White's computer program is coming from. Paul admitted that Dr. White's model has the force depending on the energy density (http://upload.wikimedia.org/math/8/e/d/8ede3e1f8a6d138bed238aa0b7d0434b.png) instead of the Poynting vector ExB.  This takes care of the harmonic time-average issue, because the energy density has a positive (never negative) time average.  But this energy density Too is the time-time component of the contravariant stress-energy tensor T

(http://upload.wikimedia.org/math/6/8/1/6818f05d140435c3561895c0e26d7a81.png)

Therefore this stress-energy tensor component   Too has no preferred direction in space. 

Where does the directionality in space come from in Dr. White's computer model ?



For example, since 4D=3D+time spacetime is difficult to conceptualize, imagine that the time coordinate is the "x" coordinate in the following graph and the space coordinates are y and z:

(http://upload.wikimedia.org/wikipedia/commons/6/67/Components_of_Stress_Tensor.jpg)

Then, the  Too component of the energy-stress tensor is equivalent to the σxx component in this image.  One can see that the  σxx component is pointed in the x direction and it has no directionality along y or z coordinates (which are perpendicular to it).  Similarly, the energy density  Too is pointed in the time direction: it has no directionality along any of the space coordinates.
Title: Re: EM Drive Developments - related to space flight applications - Thread 2
Post by: Star-Drive on 04/10/2015 03:38 am
Dr. Rodal:

As promised, find attached a few related papers from work.  As to the rest of your and Mulletron's concerns over the Eagleworks evolving theoretical musings on the EM-Drive propulsion topic, I leave you with Boyd Bushman's, (was senior scientist at LM/FW, now retired and passed-on), admonition to me when I first met him back in 2000 when discussing Jim Woodward's Mach-Effect work with Boyd's boss, "Follow the data, theory be dammed!"  We intend to do just that, no matter where it might take us.

Best,  Paul M.
Title: Re: EM Drive Developments - related to space flight applications - Thread 2
Post by: dustinthewind on 04/10/2015 05:18 am
I just thought this graphic might be interesting for some.  Hopefully it's correct... i think.  It just illustrates the resonating waves as traveling waves and sort of illustrates how they can alter from B max to E max or in-between. 

I found it interesting that doppler shift in the cavity, when it starts moving, should give the bouncing light a poynting vector indicating the light and cavity are in motion relative to the moving frame observer. 

Another thought is, what if the waves in the cavity are experiencing some change in velocity?  Why would this happen?  The thought goes back to the Casimir Force on two plates where radiation of wavelengths larger than a specific length can't exist between the plates.   I can't quite remember where but I thought I read that tunneling was thought to happen faster than light across the plates and one explanation was the dielectric constant of space between the plates had changed to give space an index of refraction smaller than that of free space and so the light jumped the gap faster than light.  So what if the cone shaped cavity could be slightly modifying the speed of light inside the cavity in a similar manner to the Casimir effect (narrow end as opposed to the larger end)? 

One exhaustive article on the matter, haven't read through it yet: http://arxiv.org/abs/physics/9911062
connected I think: http://cds.cern.ch/record/409429

Possibly connected and mentions ZPE thrust: http://utahspace.org/special/zero_point.html

and plenty more scattered on the webbs: http://www.utahspace.org/special/casimir-force-thrusters.html
Quote from site:"NASA Lewis Research Center Breakthrough Propulsion Physics Workshop (January 1999). Dr. Raymond Chio, in the Physics Dept. of the University of California-Berkley, has reported observation of a photon tunneling through a barrier at three times the speed of light."<-- "I believe faster-than-light tunneling times can be predicted and accurately controlled, without the need for a barrier, by both extending wave properties of a photon past its energy barrier, and using capacitive reactance phase shift correspondence, with related timing constants."

If so then maybe a change in light velocity would offset the phase of transformations from B to E inside the cavity? 
Title: Re: EM Drive Developments - related to space flight applications - Thread 2
Post by: sanman on 04/10/2015 07:19 am
Just reading an excerpt from this:

http://nextbigfuture.com/2015/04/magnetron-powered-em-drive-construction.html#more

Quote
Sonny White formulated a compressible quantum vacuum conjecture that requires us to live in a portion of the universe that is immersed in a false vacuum that apparently has a ground or zero-energy level much smaller than science first assumed. However what will drive this debate is experimental data first and foremost. Experimental data like what just came out of the Eagleworks Lab's latest warp-field interferometer tests based on 27,000, 1.5 second long on/off data samples that indicates we have finally observed the first spacetime contraction effects that we are fairly confident are the real deal. We again are looking for more possible false positives as well as ways of increasing the signal to noise ratio above its current ~2-to-3 sigma level, which I've already suggested several ways to do so to Dr. White. However what is really interesting about these new test results is that the laser interferometer observed spacetime contractions are being developed in a TM010 RF resonant cavity that is driving ac E-field levels over 900kV/m at a 1.48 GHz rate. A similar RF resonant system used to implement the EM-Drive and Q-thruster designs, for these spacetime contraction effects are paramount to the operation of both.

So is Dr White claiming that his interferometer is detecting a path-length difference resulting from an applied voltage on a capacitor ring?
Title: Re: EM Drive Developments - related to space flight applications - Thread 2
Post by: Rodal on 04/10/2015 12:14 pm
Dr. Rodal:

As promised, find attached a few related papers from work.  As to the rest of your and Mulletron's concerns over the Eagleworks evolving theoretical musings on the EM-Drive propulsion topic, I leave you with Boyd Bushman's, (was senior scientist at LM/FW, now retired and passed-on), admonition to me when I first met him back in 2000 when discussing Jim Woodward's Mach-Effect work with Boyd's boss, "Follow the data, theory be dammed!"  We intend to do just that, no matter where it might take us.

Best,  Paul M.

Paul,

Thanks so much for taking the time to dig and post these papers.  The new paper by Bush (2015) from MIT, showing that one can model quantum statistics hydrodynamically,  is outstanding, very clearly written.  :)

Also see this 2012 article from Hajdukovic from CERN, who envisions the Quantum Vacuum as a fluid where the virtual particle-antiparticle pairs in the quantum vacuum may be considered as gravitational dipoles:

"Quantum Vacuum and Virtual Gravitational Dipoles: The solution to the Dark Energy Problem?"

http://arxiv.org/ftp/arxiv/papers/1201/1201.4594.pdf

http://www.sciencedirect.com/science/article/pii/S2212686414000077

and this 2014 paper:

"Virtual gravitational dipoles as alternative to cosmic inflation in the primordial Universe"

https://hal.archives-ouvertes.fr/hal-00965289v1/document

and this 2014 easy-to-follow presentation:

http://sigrav.na.infn.it/wp-content/uploads/2015/03/Dragan.pdf


Wiki article on antimatter gravitation:  http://en.wikipedia.org/wiki/Antimatter_gravity_measurement

Physics society:  http://physics.aps.org/synopsis-for/10.1103/PhysRevLett.112.121102

CERN's antimatter gravitation experiment (ALPHA)  http://alpha.web.cern.ch/node/248

Quote from: CERN
Based on our data, we can exclude the possibility that the gravitiational mass of antihydrogen is more than 110 times its inertial mass, or that it falls upwards with a gravitational mass more than 65 times its inertial mass.

Our results far from settle the question of antimatter gravity. But they open the way towards higher-precision measurements in the future, using the same technique, but more, and colder trapped antihydrogen atoms, and a better understanding of the systematic effects in our apparatus.

Tests at CERN
— The ALPHA Collaboration; A.E. Charman. Description and first
application of a new technique to measure the gravitational mass of
antihydrogen, Nat. Commun. 4 (2013) 1785.
http://dx.doi.org/10.1038/ncomms2787
— A. Kellerbauer, Proposed antimatter gravity measurement with an
antihydrogen beam Nucl. Instrum. Meth. Phys. Res. B 266 (2008) 351–
356
— P. Perez, Y. Sacquin, The GBAR experiment: gravitational behaviour of
antihy- drogen at rest Class. Quantum Gravity 29 (2012) 184008

(http://www.mpi-hd.mpg.de/kellerbauer/en/projects/anti-gravity.png)

(http://www.berkeley.edu/news2/2013/04/weighingantimatter350.jpg)

https://www.youtube.com/watch?v=46NanUtEIhk
Title: Re: EM Drive Developments - related to space flight applications - Thread 2
Post by: Star One on 04/10/2015 03:02 pm
Talking of CERN will the LHC be able to help with any of this? I know they are using it to see if they can prove if super-symmetry exists.
Title: Re: EM Drive Developments - related to space flight applications - Thread 2
Post by: Rodal on 04/10/2015 03:25 pm
Talking of CERN will the LHC be able to help with any of this? I know they are using it to see if they can prove if super-symmetry exists.

No, this is not directly related to the LHC (except that they get the antiprotons from a particle accelerator). The CERN ALPHA experiment uses instead a magnetic trap  (http://en.wikipedia.org/wiki/Magnetic_trap_%28atoms%29), a Penning trap ( http://en.wikipedia.org/wiki/Penning_trap  )  and an anhilitation detector (Silicon Vertex Detector http://en.wikipedia.org/wiki/Particle_detector ).  In the Penning trap  charged plasmas of positron, antiproton and electrons are confined in a superposition of magnetic and electric fields. The Plasmas are axially confined in the Penning trap by quadratic electric potentials. Charged particles in the trap travel in the direction of the magnetic field lines and cannot escape outwards, or perpendicular to the field. The potentials in CERN's ALPHA experiment are produced by the application of DC voltages to a stack of hollow cylindrical electrodes.

(This is in direct contrast with the EM Drive because it uses DC voltages, in contrast  with the use of GigaHertz AC electromagnetic standing waves at room temperature in the EM Drive  !!!! ).

In the trap, positron and electron particles are subjected to the Lorenz force. The Lorenz force deflects particles perpendicular to the magnetic field, causing them to spiral around magnetic field lines.

The ALPHA trap electrodes are mounted inside of a vacuum chamber surrounded by a liquid helium cryostat ( http://en.wikipedia.org/wiki/Cryostat ).

Notes: Because antihydrogen is neutral, it cannot be held in a traditional Penning trap.  Therefore the ALPHA experiment uses a non-traditional Penning trap.

ALPHA derives its positrons from a radioactive beta-decay source containing an isotope of sodium, Na-22 (http://en.wikipedia.org/wiki/Positron_emission)

Positrons implanted into solid material typically have a lifetime less than one nanosecond, a thousand millionth of a second. However, during that brief time most will slow down by a variety of energy loss processes to reach kinetic energies close to those characteristic of the temperature of the solid. This process is termed moderation, as the positron’s kinetic energy is lowered, or moderated. Whilst most of the positrons penetrate deep into the bulk of the material and annihilate there, about 1% stop close enough to the surface that they can diffuse back to it before they annihilate. Incredibly, most of the positrons which reach the surface are emitted into vacuum at low energy, and can be readily formed into a beam and transported, typically using magnetic guiding fields. ALPHA uses a solid film of condensed neon as its moderator; this is one of the most efficient positron moderators.

(http://image.slidesharecdn.com/positronenablingtechnology-140905031032-phpapp01/95/positron-enablingtechnology-25-638.jpg?cb=1409904651)

(http://image.slidesharecdn.com/positronenablingtechnology-140905031032-phpapp01/95/positron-enablingtechnology-24-638.jpg?cb=1409904651)

(http://www2.mpq.mpg.de/~haensch/antihydrogen/penning_trap.gif)

(http://alpha.web.cern.ch/sites/alpha.web.cern.ch/files/trapandwells.png)
Title: Re: EM Drive Developments - related to space flight applications - Thread 2
Post by: Rodal on 04/10/2015 07:31 pm
I should also note that there are a number of prestigious scientists that wholly disagree with John Baez's assertion that virtual particles are just a bookkeeping device and that they are not real ( http://forum.nasaspaceflight.com/index.php?topic=36313.msg1355580#msg1355580 ) .  For example Gordon Kane, director of the Michigan Center for Theoretical Physics at the University of Michigan at Ann Arbor, also writing in Scientific American (as did John Baez with that comment) who wrote:

Quote from: Gordon Kane, director of the Michigan Center for Theoretical Physics at the University of Michigan at Ann Arbor
Virtual particles are indeed real particles...But while the virtual particles are briefly part of our world they can interact with other particles, and that leads to a number of tests of the quantum-mechanical predictions about virtual particles. The first test was understood in the late 1940s. In a hydrogen atom an electron and a proton are bound together by photons (the quanta of the electromagnetic field). Every photon will spend some time as a virtual electron plus its antiparticle, the virtual positron, since this is allowed by quantum mechanics as described above. The hydrogen atom has two energy levels that coincidentally seem to have the same energy. But when the atom is in one of those levels it interacts differently with the virtual electron and positron than when it is in the other, so their energies are shifted a tiny bit because of those interactions. That shift was measured by Willis Lamb and the Lamb shift was born, for which a Nobel Prize was eventually awarded....
Another very good test some readers may want to look up, which we do not have space to describe here, is the Casimir effect, where forces between metal plates in empty space are modified by the presence of virtual particles.

Thus virtual particles are indeed real and have observable effects that physicists have devised ways of measuring.



http://www.scientificamerican.com/article/are-virtual-particles-rea/

and it has been predicted since the 1930's that a very strong electric field would transform virtual particles into real ones that we can observe (by separating the particle and antiparticle, so as to prevent annihilation of each other).  Lasers in the future may make this real:

http://www.newscientist.com/article/dn19327-lasers-could-make-virtual-particles-real.html#.VSgmJfmjNpV

and this has already been done with photons:

http://phys.org/news/2011-11-scientists-vacuum.html

The assumption that the Quantum Vacuum is indestructible and immutable demands that virtual particles are exactly the same as real particles because of the experimental observation that a fundamental particle like an electron (or a positron) has the same properties (e.g. mass, charge or spin), regardless of when or where the particle was created, whether now or in the early universe, through astrophysical processes or in a laboratory.

The photons created by the Chalmers scientist, Christopher Wilson and his co-workers, who succeeded in getting photons to leave their virtual state and become real photons, are indistinguishable from real photons now and here, or photons from the early universe arriving to us.  Particles like photons, electrons, positrons, protons and antiprotons don't come with tags reading "virtual" or "real", they don't have hair, or any distinguishing features from each other.



What I have a problem seeing is how the very low power electromagnetic fields in the EM Drive operated at room temperature, and the fact that the electromagnetic fields in the EM Drive are standing waves whose Poynting vector is changing direction at a frequency twice as high as the operating frequency, can result in thrust from the vacuum acting like a positron/electron plasma.  And I also don't see how Dr. White's computer code can get a directional thrust from the energy density, which, as I discussed is a scalar component with no direction in space ( http://forum.nasaspaceflight.com/index.php?topic=36313.msg1355995#msg1355995 ).
Title: Re: EM Drive Developments - related to space flight applications - Thread 2
Post by: Stormbringer on 04/10/2015 09:27 pm
excuse me but isn't the laser thing is an accomplished fact now? A few years ago there was a couple of related articles on a desktop accelerator that generated electrons and positrons then separated them with magnets.

http://phys.org/news/2013-06-physicists-tabletop-antimatter-gun.html

http://phys.org/news/2013-06-particle-tabletop-chapter-science.html


here is one of them and then there were two more. the third talks about (the rapidly approaching future ability)  using colliding laser beams to create matter out of photons.

http://phys.org/news/2014-05-scientists-year-quest.html

http://www.newscientist.com/article/dn19327-lasers-could-make-virtual-particles-real.html
Title: Re: EM Drive Developments - related to space flight applications - Thread 2
Post by: Rodal on 04/10/2015 10:05 pm
excuse me but isn't the laser thing is an accomplished fact now? A few years ago there was a couple of related articles on a desktop accelerator that generated electrons and positrons then separated them with magnets.

http://phys.org/news/2013-06-physicists-tabletop-antimatter-gun.html

http://phys.org/news/2013-06-particle-tabletop-chapter-science.html


here is one of them and then there were two more. the third talks about (the rapidly approaching future ability)  using colliding laser beams to create matter out of photons.

http://phys.org/news/2014-05-scientists-year-quest.html

http://www.newscientist.com/article/dn19327-lasers-could-make-virtual-particles-real.html
Good to hear that, if making real positrons out of "virtual" positrons has already been accomplished, so much the better for my point that virtual particles are indistinguishable from real particles.  They don't have tags or any other features identifying them as "virtual", they are exactly the same as any other real particle except for their ephemeral life if (as they must do unless one intervenes) annihilate each other  :)
Title: Re: EM Drive Developments - related to space flight applications - Thread 2
Post by: aero on 04/10/2015 10:38 pm
excuse me but isn't the laser thing is an accomplished fact now? A few years ago there was a couple of related articles on a desktop accelerator that generated electrons and positrons then separated them with magnets.

http://phys.org/news/2013-06-physicists-tabletop-antimatter-gun.html

http://phys.org/news/2013-06-particle-tabletop-chapter-science.html


here is one of them and then there were two more. the third talks about (the rapidly approaching future ability)  using colliding laser beams to create matter out of photons.

http://phys.org/news/2014-05-scientists-year-quest.html

http://www.newscientist.com/article/dn19327-lasers-could-make-virtual-particles-real.html

Very interesting. And I found another paper interesting, speaking of symmetry breaking in dielectric materials. I'm not sure how it relates to EM thrusters but it does focus on a couple of keys that we have discussed in detail.

http://phys.org/news/2015-04-electromagnetism-enable-antennas-chip.html (http://phys.org/news/2015-04-electromagnetism-enable-antennas-chip.html)
Title: Re: EM Drive Developments - related to space flight applications - Thread 2
Post by: CW on 04/11/2015 06:19 am
There's one thing that has bothered me about virtual particle pairs popping in and out of existence:

When besaid pairs, for instance, pop into existence in a gravitational field like the Earth's, they do exist for a minute time span. During that time span, they are 'real' due to borrowed energy from the quantum vacuum. But real particles with mass experience an acceleration in a gravitational field and thus gain momentum. There are two cases to consider, of which only one should be correct:

1) The pair's particles react to gravity the same way and experience a momentum gain in the same direction. When they pop out of existence again, the gained momentum 'magically' disappears with the pair and CoM is broken. Momentum must not simply 'disappear' (in lack for a better word). I read somewhere that CoM is not applicable to virtual particles (can't find the link), but still this doesn't seem right.
2) The anti-particle gains a momentum opposite to the particle's momentum due to repulsive forces on an anti-particle within a gravitational field, and so the force and momentum vectors add up to Zero at any instance in time. CoM is maintained.

What do you think?
Title: Re: EM Drive Developments - related to space flight applications - Thread 2
Post by: Rodal on 04/11/2015 01:06 pm
Dr. Rodal:

As promised, find attached a few related papers from work.  As to the rest of your and Mulletron's concerns over the Eagleworks evolving theoretical musings on the EM-Drive propulsion topic, I leave you with Boyd Bushman's, (was senior scientist at LM/FW, now retired and passed-on), admonition to me when I first met him back in 2000 when discussing Jim Woodward's Mach-Effect work with Boyd's boss, "Follow the data, theory be dammed!"  We intend to do just that, no matter where it might take us.

Best,  Paul M.

Paul,

Thanks so much for taking the time to dig and post these papers.  The new paper by Bush (2015) from MIT, showing that one can model quantum statistics hydrodynamically,  is outstanding, very clearly written.  :)

...

Great YouTube video by Harris and Bush at MIT.

Watch that droplet ride its own guiding pilot wave, "magically" hovering over the fluid (can you describe what's responsible for its motion ? : instability due to nonlinear standing waves):

https://www.youtube.com/watch?v=nmC0ygr08tE

The great (7th duc) de Broglie, Noble Prize in Physics, recognized 90 years after his Solvay presentation: a triumph of creative imagination

(http://www.nobelprize.org/nobel_prizes/physics/laureates/1929/broglie.jpg)
Title: Re: EM Drive Developments - related to space flight applications - Thread 2
Post by: rfmwguy on 04/11/2015 02:41 pm
Tried to search for answer, could not find...my apologies if already discussed:

Has there been an accurate mass comparison on the dielectric (PTFE et al) prior to and after RF excitation?
Title: Re: EM Drive Developments - related to space flight applications - Thread 2
Post by: Rodal on 04/11/2015 03:00 pm
Tried to search for answer, could not find...my apologies if already discussed:

Has there been an accurate mass comparison on the dielectric (PTFE et al) prior to and after RF excitation?
Welcome to the forum. 

I don't recall seeing such a comparison reported yet.

What motivates your question? pyrolysis decomposition in nitrogen atmosphere determined by ThermoGravimetricAnalysis of high density polyethylene (HDPE, the main material used as a dielectric) starts at around 700 deg K (800 deg F): http://w3.gazi.edu.tr/~ikayacan/Part%20I.pdf

I expect that decomposition should be enhanced by oxygen, so it would start at around 700 deg F in an oxygen atmosphere.
Title: Re: EM Drive Developments - related to space flight applications - Thread 2
Post by: rfmwguy on 04/11/2015 03:30 pm
Thanks for the welcome. I've been around high power RF for many years and have seen low temp PTFE issues at relatively low temps. Specifically, changes in capacitance, yeilding center frequency drift in tchebychev bandpass filters using PTFE tape and discs. Outgassing in hermetically sealed tubes were noticed. Could be totally unrelated but...maybe not.
Title: Re: EM Drive Developments - related to space flight applications - Thread 2
Post by: Star-Drive on 04/11/2015 03:42 pm
Dr. Rodal:

As promised, find attached a few related papers from work.  As to the rest of your and Mulletron's concerns over the Eagleworks evolving theoretical musings on the EM-Drive propulsion topic, I leave you with Boyd Bushman's, (was senior scientist at LM/FW, now retired and passed-on), admonition to me when I first met him back in 2000 when discussing Jim Woodward's Mach-Effect work with Boyd's boss, "Follow the data, theory be dammed!"  We intend to do just that, no matter where it might take us.

Best,  Paul M.

Paul,

Thanks so much for taking the time to dig and post these papers.  The new paper by Bush (2015) from MIT, showing that one can model quantum statistics hydrodynamically,  is outstanding, very clearly written.  :)

...

Great YouTube video by Harris and Bush at MIT.

Watch that droplet ride its own guiding pilot wave, "magically" hovering over the fluid (can you describe what's responsible for its motion ? : instability due to nonlinear standing waves):

https://www.youtube.com/watch?v=nmC0ygr08tE

The great (7th duc) de Broglie, Noble Prize in Physics, recognized 90 years after his Solvay presentation: a triumph of creative imagination

(http://www.nobelprize.org/nobel_prizes/physics/laureates/1929/broglie.jpg)

Dr. Rodal:

Quote from Einstein:  "As I have said so many times, God doesn't play dice with the world."

In other words, de Broglie got it right with his pilot-wave conjecture and QM really IS deterministic.  However that brings us back to what the universe's pilot-wave media actually is, and the Eagleworks position is that the Quantum Vacuum AKA Dirac Sea is that medium.  A medium that can make ALL subatomic particles for ANY length of time locally, and a media that can convey BOTH longitudinal and transverse wave action.  However if the Casimir effect has anything to say about this topic, and we think it does, then longitudinal Q-V waves have to fall off at not 1/r^2 as transverse waves do in E&M, but at 1/r^4.   That is why we think we have to live in a 5 spatial plus one time dimension universe, so we can explain the Casimir effect's 1/r^4  fall off, since it takes n+1 dimensions to account for this 1/r^4 Casimir effect scaling.

Best,  Paul M.
Title: Re: EM Drive Developments - related to space flight applications - Thread 2
Post by: Rodal on 04/11/2015 04:40 pm
Thanks for the welcome. I've been around high power RF for many years and have seen low temp PTFE issues at relatively low temps. Specifically, changes in capacitance, yeilding center frequency drift in tchebychev bandpass filters using PTFE tape and discs. Outgassing in hermetically sealed tubes were noticed. Could be totally unrelated but...maybe not.
Agreed, outgassing (due to microwave heating of water vapor previously trapped in the HDPE or PTFE polymer dielectric) would be something to watch out for in a vacuum environment at significantly lower temperatures (near 200 deg F) than pyrolysis (>700 deg F). 

(http://www.aac-research.at/images/Bilder-AAC/outgassing-curve.jpg)
Title: Re: EM Drive Developments - related to space flight applications - Thread 2
Post by: Rodal on 04/11/2015 06:13 pm
Thanks for the welcome. I've been around high power RF for many years and have seen low temp PTFE issues at relatively low temps. Specifically, changes in capacitance, yeilding center frequency drift in tchebychev bandpass filters using PTFE tape and discs. Outgassing in hermetically sealed tubes were noticed. Could be totally unrelated but...maybe not.
Agreed, outgassing (due to microwave heating of water vapor previously trapped in the HDPE or PTFE polymer dielectric) would be something to watch out for in a vacuum environment at significantly lower temperatures (near 200 deg F) than pyrolysis (>700 deg F). 



This Lawrence Livermore Lab report on outgassing of water vapor from HDPE is pertinent:

Vacuum Outgassing of High Density Polyethylene
L. N. Dinh*, J. Sze, M. A. Schildbach, S. C. Chinn, R. S. Maxwell, P. Raboin, W. McLean II
Lawrence Livermore National Laboratory, Livermore, Ca, USA

https://e-reports-ext.llnl.gov/pdf/364291.pdf


It concludes that outgassing of H2O from HDPE can be significantly reduced by vacuum baking at 368 degres K
(203 deg F) for a few hours prior to device assembly.
Title: Re: EM Drive Developments - related to space flight applications - Thread 2
Post by: Rodal on 04/11/2015 08:05 pm
Dr. Rodal:

As promised, find attached a few related papers from work.  As to the rest of your and Mulletron's concerns over the Eagleworks evolving theoretical musings on the EM-Drive propulsion topic, I leave you with Boyd Bushman's, (was senior scientist at LM/FW, now retired and passed-on), admonition to me when I first met him back in 2000 when discussing Jim Woodward's Mach-Effect work with Boyd's boss, "Follow the data, theory be dammed!"  We intend to do just that, no matter where it might take us.

Best,  Paul M.

Paul,

Thanks so much for taking the time to dig and post these papers.  The new paper by Bush (2015) from MIT, showing that one can model quantum statistics hydrodynamically,  is outstanding, very clearly written.  :)

...

Great YouTube video by Harris and Bush at MIT.

Watch that droplet ride its own guiding pilot wave, "magically" hovering over the fluid (can you describe what's responsible for its motion ? : instability due to nonlinear standing waves):

https://www.youtube.com/watch?v=nmC0ygr08tE
...
Note that @Notsosureofit mentioned that nonlinearity in the dielectric could be important and that some of the papers posted by @Mulletron rely on both nonlinearity and anisotropy of the dielectric to explain thrust force as an interaction with the quantum vacuum.

This would leave open to question whether Eagleworks will be able to measure thrust force (and hence reproduce the UK and Chinese results) in the upcoming test without a dielectric, using a magnetron with much higher power inputs...
Title: Re: EM Drive Developments - related to space flight applications - Thread 2
Post by: dustinthewind on 04/12/2015 08:21 am
There's one thing that has bothered me about virtual particle pairs popping in and out of existence:

When besaid pairs, for instance, pop into existence in a gravitational field like the Earth's, they do exist for a minute time span. During that time span, they are 'real' due to borrowed energy from the quantum vacuum. But real particles with mass experience an acceleration in a gravitational field and thus gain momentum. There are two cases to consider, of which only one should be correct:

1) The pair's particles react to gravity the same way and experience a momentum gain in the same direction. When they pop out of existence again, the gained momentum 'magically' disappears with the pair and CoM is broken. Momentum must not simply 'disappear' (in lack for a better word). I read somewhere that CoM is not applicable to virtual particles (can't find the link), but still this doesn't seem right.

Quote from: DustinTheWind

Interesting line of thought.  What if when imaginary particles become real we measure them in gravity having downward momentum, they accelerate, then go back to imaginary.  Later we do the same experiment lower in the gravity field and they become real but this time they have greater momentum, accelerate, then go imaginary again.  We might then suspect the quantum vacuum was already in motion and accelerating before they came into existence.  Maybe the rate they pop in and out of existence determines the drag they have on normal matter? 

In relation to the cavity then what if the particles become real, we give them a shove, then they become non-real and are allowed to exit the cavity by way of the quantum vacuum.  Could that momentum then be imparted to the vacuum?  Would we measure time space waves from it? 


2) The anti-particle gains a momentum opposite to the particle's momentum due to repulsive forces on an anti-particle within a gravitational field, and so the force and momentum vectors add up to Zero at any instance in time. CoM is maintained.

What do you think?

Quote from: DustinTheWind
Matter that was repelled from gravity would remind me of negative energy density.  I think that would be the stuff like the Alcubierre warp bubble where it shows negative energy density on one side and positive on the other to make the ship go forward by warp.  Maybe gravity propulsion?  Might be related to this, link: http://en.wikipedia.org/wiki/Negative_mass, "Runaway motion"  top paragraph, "Although no particles are known to have negative mass..."  towards the bottom, "Hence Bondi pointed out ..."<-- is he talking about the warp bubble effect?

Under this link: http://en.wikipedia.org/wiki/Negative_mass under "Gravitational interaction of antimatter" it states, "The overwhelming consensus among physicists is that antimatter has positive mass and should be affected by gravity just like normal matter. Direct experiments on neutral antihydrogen have not detected any difference between the gravitational interaction of antimatter, compared to normal matter.[19]"

As far as I knew I thought imaginary particles were matter and anti-matter.  One big mystery is where did all the anti-mater in the universe go.  I guess I can't say that they have measured the gravitational attraction of anti-matter yet that I know of though wiki suggests in section, "Gravitational interaction of antimatter" the "Bubble chamber experiments" as an arrow that suggests they have positive mass though don't ask me how conclusive that is. 

Title: Re: EM Drive Developments - related to space flight applications - Thread 2
Post by: CW on 04/12/2015 01:35 pm
@dustinthewind:

Hmm.. so if we go by http://en.wikipedia.org/wiki/Negative_mass (http://en.wikipedia.org/wiki/Negative_mass) as referred by you and if we assume that the 'overwhelming consensus' is that antimatter shows the same gravitational behavior as does matter, then my case 1) should be correct and CoM be broken for matter-antimatter pairs popping in and out of existence within gravitational fields. Either this is true, or CoM does not apply for those pairs. Hmm..

About creating an ExB = S Poynting vector.. wouldn't it be much easier to do this if one takes a circular electromagnet with iron core that has a relatively big air gap, and puts a highly charged plate capacitor on opposite sides of the air gap? The resulting space should be 'filled' with Poynting vectors ExB that can interact with the theorized QV plasma, which is supposed to accelerate the QV plasma and in turn accelerate the contraption.

Does that make sense, or am I missing something relevant concerning how a QV thruster should work?


EDIT: I made a drawing of a contraption I call 'Poynting vector field generator' and attached it to this message. If there's something wrong with the drawing, please tell.

EDIT: Sorry for the mix of wire frame and solid view. I was a bit lazy to tinker with the drawing program for dotted lines to indicate hidden edges ;-) .
Title: Re: EM Drive Developments - related to space flight applications - Thread 2
Post by: zen-in on 04/12/2015 04:30 pm
@dustinthewind:

About creating an ExB = S Poynting vector.. wouldn't it be much easier to do this if one takes a circular electromagnet with iron core that has a relatively big air gap, and puts a highly charged plate capacitor on opposite sides of the air gap? The resulting space should be 'filled' with Poynting vectors ExB that can interact with the theorized QV plasma, which is supposed to accelerate the QV plasma and in turn accelerate the contraption.

Does that make sense, or am I missing something relevant concerning how a QV thruster should work?


EDIT: I made a drawing of a contraption I call 'Poynting vector field generator' and attached it to this message. If there's something wrong with the drawing, please tell.

EDIT: Sorry for the mix of wire frame and solid view. I was a bit lazy to tinker with the drawing program for dotted lines to indicate hidden edges ;-) .

A Poynting vector is not a field.   In your drawing the Poynting vector is dispersed in all directions because the energy flow is entirely due to thermal loss. (DC case)   If there are AC drives to the capacitor and gap magnet then some energy is transferred to near field RF radiation.   Nothing exotic happens here.  The em fields just combine.
Title: Re: EM Drive Developments - related to space flight applications - Thread 2
Post by: CW on 04/12/2015 05:01 pm
@dustinthewind:

About creating an ExB = S Poynting vector.. wouldn't it be much easier to do this if one takes a circular electromagnet with iron core that has a relatively big air gap, and puts a highly charged plate capacitor on opposite sides of the air gap? The resulting space should be 'filled' with Poynting vectors ExB that can interact with the theorized QV plasma, which is supposed to accelerate the QV plasma and in turn accelerate the contraption.

Does that make sense, or am I missing something relevant concerning how a QV thruster should work?


EDIT: I made a drawing of a contraption I call 'Poynting vector field generator' and attached it to this message. If there's something wrong with the drawing, please tell.

EDIT: Sorry for the mix of wire frame and solid view. I was a bit lazy to tinker with the drawing program for dotted lines to indicate hidden edges ;-) .

A Poynting vector is not a field.   In your drawing the Poynting vector is dispersed in all directions because the energy flow is entirely due to thermal loss. (DC case)   If there are AC drives to the capacitor and gap magnet then some energy is transferred to near field RF radiation.   Nothing exotic happens here.  The em fields just combine.

Of course a single Poynting vector is not a field. I meant that a spatial volume as depicted, that is filled with an E-field and a perpendicular B-field, formally creates a E&B perpendicular Poynting vector (vector = absolute value and direction) in each spacial point of that volume. So what you say is that this spatial region is then still no field? Since the discussion was about Q-thrusters that are supposed to use a virtual QV plasma as 'propellant', I thought that it is exactly the point that the Poynting vector is to be used to impart impulse on this virtual QV plasma and get an opposite reaction that drives the thruster forward. So, some Poynting energy flow should happen in this theory that is greater than Zero.

Best regards
Title: Re: EM Drive Developments - related to space flight applications - Thread 2
Post by: flux_capacitor on 04/12/2015 05:47 pm
@dustinthewind:

Hmm.. so if we go by http://en.wikipedia.org/wiki/Negative_mass (http://en.wikipedia.org/wiki/Negative_mass) as referred by you and if we assume that the 'overwhelming consensus' is that antimatter shows the same gravitational behavior as does matter, then my case 1) should be correct and CoM be broken for matter-antimatter pairs popping in and out of existence within gravitational fields. Either this is true, or CoM does not apply for those pairs. Hmm..

About antimatter and negative mass: antiparticles are expected to fall in a gravity field exactly like ordinary matter, because both have a positive energy, hence positive mass m = E / c^2

If antimatter had a negative energy -E, the encounter between a particle ant its antiparticle would result in nothing, since E - E = 0
This is not the case. All their mass is converted into energy in this reaction. That's why antimatter could be considered as a powerful source for space propulsion, or bombs.

Do you know CPT symmetry? C-symmetry is for charge conjugation. P-symmetry is for parity transformation, and T-symmetry is for time reversal.

When the charge C of a particle is inverted (C-symmetry), it becomes its antiparticle. That's the classical antimatter as discovered by Paul Dirac.

But Richard Feynman envisioned another nonclassical type of antiparticles, PT-symmetric of ordinary matter.

It has been shown with dynamic group theory (coadjoint action of Lorentz and complete Poincaré groups on their moment space) that T-symmetry is the same as E-symmetry (Souriau, J.M. (1997), Structure of Dynamical Systems (http://www.amazon.com/Structure-Dynamical-Systems-Symplectic-Mathematics/dp/0817636951), Birkhäuser Boston Editions, ISBN 0817636951 - See Chapter III - part 14, page 189 "Inversions of space and time"). When you reverse the arrow of time of a particle, it means you get a particle with a negative energy, and a negative mass if it has one.

In this regard, Feynman's antimatter (PT-symmetry) would not fall in Earth gravitational field. But we have not observed that kind of special antiparticles yet. This would be "negative energy antimatter". By the way, CPT-symmetric particles would be negative energy matter, with negative mass. And Feynman's PT-symmetric particles would just be "the antiparticles of those CPT-symmetric particles", their charge-conjugated version).
Title: Re: EM Drive Developments - related to space flight applications - Thread 2
Post by: flux_capacitor on 04/12/2015 06:03 pm
A Poynting vector is not a field.   In your drawing the Poynting vector is dispersed in all directions because the energy flow is entirely due to thermal loss. (DC case)   If there are AC drives to the capacitor and gap magnet then some energy is transferred to near field RF radiation.   Nothing exotic happens here.  The em fields just combine.

I don't get your point.

1. CW's drawing does not show a Poynting vector dispersed in all directions, but a very directional vector going from the left to the right in the picture.

2. That Poynting vector S = ExB actually gives a Lorentz force F = q(E + vxB) when electric charges are able to flow within the electric field. If the sum of those electric charges in movement are able to create an electric current in a continuous charge distribution, i.e. with a current density J due to the charge density ρ, the Lorentz Force is then a "3D volumetric force", accelerating all electric charges in the same direction whatever their sign, and also the neutral atoms in the plasma with them, through collisions. This is the basis of magnetohydrodynamics applied to propulsion (MHD accelerators), sometimes called magnetoplasmadynamic (MPD) thrusters when the accelerating fluid is a ionized gas.

Really I'm not sure if Harold White's idea of pushing upon virtual particles from the quantum vacuum with MHD (Lorentz forces) as if it was a real plasma is the answer. But I second CW's question about a setup like an MPD thruster, with crossed E and B fields. Applying an orthogonal magnetic field with an electromagnet would considerably enhance the propulsive Lorentz force. If I remember correctly, White's first Q-thruster (The DC version of Woodward's first Mach-Lorentz Thruster or MLT, that ran on AC) was based on that concept but was a failure -except while being powered on and off (so the DC current would transiently become AC, as per Woodward). White's Quantum Vacuum Fluctuation (QVF) conjecture does not need AC currents, only DC, contrary to Woodward's Mach Effect (ME) conjecture. Maybe Paul could clarify all this.
Title: Re: EM Drive Developments - related to space flight applications - Thread 2
Post by: Stormbringer on 04/12/2015 06:34 pm
Feynmans's species of antiparticle might be alice matter AKA Mirror matter? If so it is theoretically possible to get some. :)

http://phys.org/news/2012-06-neutrons-parallel-world.html

http://www.technologyreview.com/view/426676/how-neutrons-might-escape-into-another-universe/



Title: Re: EM Drive Developments - related to space flight applications - Thread 2
Post by: CW on 04/12/2015 06:54 pm
A Poynting vector is not a field.   In your drawing the Poynting vector is dispersed in all directions because the energy flow is entirely due to thermal loss. (DC case)   If there are AC drives to the capacitor and gap magnet then some energy is transferred to near field RF radiation.   Nothing exotic happens here.  The em fields just combine.

I don't get your point.

1. CW's drawing does not show a Poynting vector dispersed in all directions, but a very directional vector going from the left to the right in the picture.

2. That Poynting vector S = ExB actually gives a Lorentz force F = q(E + vxB) when electric charges are able to flow within the electric field. If the sum of those electric charges in movement are able to create an electric current in a continuous charge distribution, i.e. with a current density J due to the charge density ρ, the Lorentz Force is then a "3D volumetric force", accelerating all electric charges in the same direction whatever their sign, and also the neutral atoms in the plasma with them, through collisions. This is the basis of magnetohydrodynamics applied to propulsion (MHD accelerators), sometimes called magnetoplasmadynamic (MPD) thrusters when the accelerating fluid is a ionized gas.

Really I'm not sure if Harold White's idea of pushing upon virtual particles from the quantum vacuum with MHD (Lorentz forces) as if it was a real plasma is the answer. But I second CW's question about a setup like an MPD thruster, with crossed E and B fields. Applying an orthogonal magnetic field with an electromagnet would considerably enhance the propulsive Lorentz force. If I remember correctly, White's first Q-thruster (The DC version of Woodward's first Mach-Lorentz Thruster or MLT, that ran on AC) was based on that concept but was a failure -except while being powered on and off (so the DC current would transiently become AC, as per Woodward). White's Quantum Vacuum Fluctuation (QVF) conjecture does not need AC currents, only DC, contrary to Woodward's Mach Effect (ME) conjecture. Maybe Paul could clarify all this.

I came up with this drawing because I had the impression that Dr. White's conjecture needs a strong Poynting vector field to do what it's supposed to do. And so I was wondering why they are shooting EM waves of varying center frequency and bandwidth into a closed hollow conductor and hope (rather: pray) that S = ExB becomes more or less unidirectional and can interact with the hypothesized virtual QV plasma in the desired way. I'm an electrical engineer (with some extra physics on the side..) by profession and so I sought a simpler and IMO more robust setup to create Poynting vectors. Oh well.. what do I know :) .
Title: Re: EM Drive Developments - related to space flight applications - Thread 2
Post by: flux_capacitor on 04/12/2015 07:09 pm
Feynmans's species of antiparticle might be alice matter AKA Mirror matter? If so it is theoretically possible to get some. :)

http://phys.org/news/2012-06-neutrons-parallel-world.html

http://www.technologyreview.com/view/426676/how-neutrons-might-escape-into-another-universe/

Exactly. As the experiments pointed out a probable feature of those T-symmetric "mirror particles" is the strangest: they would not follow our 3+1 dimensional spacetime. Because of the T-symmetry, those particles (negative energy photons and negative mass particles) would follow their own geodesics within their own metric, in a spacetime having an opposite arrow of time. Hence they could not be seen, nor be touched. Maybe they could interact (anti)gravitationally with our own matter. In this case they would be the perfect candidate for the invisible dark matter.

If you're interested in this topic about negative mass, not really (?) related to the EmDrive, here are attached to this message two full papers of a very recent work done after connecting A. Sakharov, R. Feynman, JM Souriau and H. Bondi ideas:
- Negative mass hypothesis in cosmology and the nature of dark energy (20 September 2014), Astrophysics and Space Science, 354:2106, doi 10.1007/s10509-014-2106-5 (http://link.springer.com/article/10.1007%2Fs10509-014-2106-5)
- Cosmological bimetric model with interacting positive and negative masses and two different speeds of light, in agreement with the observed acceleration of the Universe (10 November 2014), Modern Physics Letters A 29:1450182, doi 10.1142/S021773231450182X (http://www.worldscientific.com/doi/abs/10.1142/S021773231450182X)

Finally, count also this third VERY interesting paper by the same author about a natural process of mass inversion, alternate solution to the classical blackhole/wormhole, but still classical GR:
- Cancellation of the central singularity of the Schwarzschild solution with natural mass inversion process (21 March 2015), Modern Physics Letters A, 30:1550051, doi 10.1142/S0217732315500510 (http://www.worldscientific.com/doi/abs/10.1142/S0217732315500510)
Title: Re: EM Drive Developments - related to space flight applications - Thread 2
Post by: Stormbringer on 04/12/2015 07:22 pm
I am not going to pretend like I have a real firm grip on the math or even the theory but...

The experiments in the articles I linked (and in other related articles I didn't) seem to me like the interaction of the fields in the em cavity might just accidentally create the condition needed for particle transition and possibly would create an imbalance in the system as particles appeared or disappeared from the mirror sector.

     The primary ( external ) requirement in one of the experiments was a very weak mirror magnetic field which would be provided naturally by the earth itself via trapped mirror matter particles mixed in the normal matter of the earth and collected by sweeping space (inelastic collisions and gravitational interaction) as solar and galactic orbital motions occur over geological and cosmic time scales.

EDIT:  On consideration this might not bode well for this to work in space unless a mirror magnetic source was deployed with the device. But that might be one way to test the hypothesis. Of course; you could just look for the loss or gain of particles in the cavity or near the cavity.

EDIT:  The sea of mirror particles would work just as well as quantum plasma i think. It Would perform the same function.
Title: Re: EM Drive Developments - related to space flight applications - Thread 2
Post by: Stormbringer on 04/12/2015 08:21 pm
I don't think running the experiment in air would have much effect on signal. As far as I know most interferometry experiments are in open air. The laser doesn't care much at short ranges.
Title: Re: EM Drive Developments - related to space flight applications - Thread 2
Post by: aceshigh on 04/13/2015 02:11 am
all these theories of the virtual particles having mass or not (I guess they have, otherwise wouldn´t be used for propulsion)... wouldn´t the total mass of virtual particles be much greater than the mass of even dark energy + dark matter together, and thus cause a collapse of the universe?
Title: Re: EM Drive Developments - related to space flight applications - Thread 2
Post by: Stormbringer on 04/13/2015 02:51 am
Well the mass of particles is a sum of the mass energy equivalence of all their properties which turns out to leave a slight overabundance of positive mass.

The negative bare mass is assumed in order to get rid of infinities and divergences (infinite electrical and gravitational self energy) and it is large; In some theories it is infinite; but that is theoretical.

 In practice the negative mass is manageable in the same way that the virtual particle world is supposed to be infinite but in practice is extremely low energy. Dr Woodward said something about this. In practical terms ZPE is tiny because generally only the very lowest energies are ever manifest from it. This is because though there are all possible mass energy equivalents it the vast probabilities cause only the lower powered virtual stuff to be "commonly" present. The lower energy possibilities swamp the larger ones. I hope I expressed that properly. if I didn't; it's my fault and not my sources.

On negative mass side of the equation defining particle mass there is a factor of C^4 while on the positive side on the right side of the equals sign there is the standard C^2 term. So even if the negative mass (assuming it can be made manifest) is limited it would still be quite large; easily a Jupiter mass worth. Perhaps Dr Woodward is being overly optimistic but his grasp of the math, theory and history of the evolution of the standard model is formidable. I would not bet against him. 
Title: Re: EM Drive Developments - related to space flight applications - Thread 2
Post by: dustinthewind on 04/13/2015 02:55 am
all these theories of the virtual particles having mass or not (I guess they have, otherwise wouldn´t be used for propulsion)... wouldn´t the total mass of virtual particles be much greater than the mass of even dark energy + dark matter together, and thus cause a collapse of the universe?

Interesting point.  If I remember correctly the vacuum is seething with waves of energy but statistically those waves interfere with each other destructively so that there appears to be nothing.  However there is still chance that energy at some points in the randomness will suddenly constructively interfere and become real.  Hope I'm not mistaken here. 

Maybe considering time slows near matter then with time passing by faster in space more particles pop into existence farther from the gravity source then near which might suggest a pressure but as stated earlier the Casimir force is supposed to drop off by 1/r^4 rather than 1/r^2 so ...  maybe there is some way to reconcile that and maybe not. 

While energy has its equivalence to mass and one might then suppose energy could induce gravity.  There is a way out of the gravity problem you mention.  Consider the universe to be infinite.  Gravity inside of a shell goes to zero if you go through the math and is why earth gravity drops to zero in the center of the earth.  (Also why the hollow earth ideas are wrong , eg. no gravity inside the hollow earth.) By the speed of light our universe we perceive I think is centered on us and so we are in the center of concentric shells of the quantum sea.  If this quantum sea is even density in general then gravity should cancel out. 
Title: Re: EM Drive Developments - related to space flight applications - Thread 2
Post by: zen-in on 04/13/2015 04:07 am
A Poynting vector is not a field.   In your drawing the Poynting vector is dispersed in all directions because the energy flow is entirely due to thermal loss. (DC case)   If there are AC drives to the capacitor and gap magnet then some energy is transferred to near field RF radiation.   Nothing exotic happens here.  The em fields just combine.

I don't get your point.

1. CW's drawing does not show a Poynting vector dispersed in all directions, but a very directional vector going from the left to the right in the picture.

2. That Poynting vector S = ExB actually gives a Lorentz force F = q(E + vxB) when electric charges are able to flow within the electric field. If the sum of those electric charges in movement are able to create an electric current in a continuous charge distribution, i.e. with a current density J due to the charge density ρ, the Lorentz Force is then a "3D volumetric force", accelerating all electric charges in the same direction whatever their sign, and also the neutral atoms in the plasma with them, through collisions. This is the basis of magnetohydrodynamics applied to propulsion (MHD accelerators), sometimes called magnetoplasmadynamic (MPD) thrusters when the accelerating fluid is a ionized gas.


I think you might be mixing a lot of things together here and jumping to conclusions.   If you believe this gap magnet and capacitor can generate reactionless thrust you should build it and test it out for yourself.   I could give you 20-30 more interesting ideas I have played with in the last 15 years when you get done with that one.  Just because something looks "cool" doesn't mean it has any significance.
Title: Re: EM Drive Developments - related to space flight applications - Thread 2
Post by: aero on 04/13/2015 04:23 am
Since you all have brought it up, I was thinking about that question earlier today so I did a web search. Turns out that there are 5 different theories that could potentially be used to estimate the energy mass of the QV. General Relativity and Quantum Theory are two that disagree by over 100 orders of magnitude. One of the theories can't be applied and the other two disagree wildly. The estimate that has some observational support comes from GR from which the answer is, Energy density, rho =  7x10-27 kg/m3.

The observational support comes from the expanding universe being caused by the pressure of the QV where rho = 3 P.
The 3 comes from the fact of 3 spacial dimensions. Then it gets complicated ...

As I recall, my search key was  - mass energy of quantum vacuum - .
Title: Re: EM Drive Developments - related to space flight applications - Thread 2
Post by: MichaelBlackbourn on 04/13/2015 04:49 am
I have to thank StarDrive and others. I hadn't heard of pilot waves and the idea of a return to a deterministic universe with the potential to leverage the 'medium' that the waves travel through is very interesting. The fact that Einstein preferred it and that all objections to it have been found lacking is outstanding. I also like how the measurement problem becomes just a Normal problem, not something intrinsic to the universe. Great reading.

The oil droplet experiments are excellent.

Best of luck finding a way to push against this 'sea'. I hope EMdrive or Woodward gives us something other than tossing atoms overboard for spaceflight.

Imagine if EMdrive is shown to be repeatable then there will be a change in the preferred QM interpretation.
Title: Re: EM Drive Developments - related to space flight applications - Thread 2
Post by: CW on 04/13/2015 06:04 am
A Poynting vector is not a field.   In your drawing the Poynting vector is dispersed in all directions because the energy flow is entirely due to thermal loss. (DC case)   If there are AC drives to the capacitor and gap magnet then some energy is transferred to near field RF radiation.   Nothing exotic happens here.  The em fields just combine.

I don't get your point.

1. CW's drawing does not show a Poynting vector dispersed in all directions, but a very directional vector going from the left to the right in the picture.

2. That Poynting vector S = ExB actually gives a Lorentz force F = q(E + vxB) when electric charges are able to flow within the electric field. If the sum of those electric charges in movement are able to create an electric current in a continuous charge distribution, i.e. with a current density J due to the charge density ρ, the Lorentz Force is then a "3D volumetric force", accelerating all electric charges in the same direction whatever their sign, and also the neutral atoms in the plasma with them, through collisions. This is the basis of magnetohydrodynamics applied to propulsion (MHD accelerators), sometimes called magnetoplasmadynamic (MPD) thrusters when the accelerating fluid is a ionized gas.


I think you might be mixing a lot of things together here and jumping to conclusions.   If you believe this gap magnet and capacitor can generate reactionless thrust you should build it and test it out for yourself.   I could give you 20-30 more interesting ideas I have played with in the last 15 years when you get done with that one.  Just because something looks "cool" doesn't mean it has any significance.

Hmm.. I think you're somewhat misunderstanding my intention. First of all, I don't believe that this contraption does anything special. It is simply a practical model of how to most efficiently and simply create a volume of space that also contains unidirectional Poynting vectors. Unidirectional Poynting vectors seem to be the key to Dr. White's conjecture.. at least to me. Maybe I'm wrong on this. Also, Dr. White's conjecture is no reactionless drive, because he predicts an interaction with a 'virtual QV plasma' that can be used similarly as in 'real' MHD thrusters. Although I agree that apparent 'coolness' of something doesn't imply any kind of significance, this was really not the point.

Best regards
Title: Re: EM Drive Developments - related to space flight applications - Thread 2
Post by: Star-Drive on 04/13/2015 12:45 pm
Folks:

From our Q-V plasma code studies, it is becoming clear that the Poynting power flow vector is only one of several parameters that has to be used in determining the magnitude and direction of the Q-V plasma flow and its resulting reaction thrust in the QVH/MHD based thrusters such as the EM-drives.  In fact the Cannae pillbox cavity produces thrust in the opposite direction than what the net Poynting vector would lead one to expect.  We think, though have not confirmed yet, that this is due to the hydrodynamics of the Q-V plasma and how it interacts with high ac E-field volumes that may be present in the operational thruster, such as the high ac E-fields developed in the small OD of the Cannae Teflon Z-matching tube located in its RF input section.  I'm re-appending a Rice University paper on the hydrodynamics of the vacuum for your reference and study.  You will find that a Q-V plasma acts quite a bit like a water based fluid, but with some startling differences as well, since it does salute MHD rules as well AND the still curious rules of the Quantum world.

Best, Paul M.

PS: From astrophysics it appears that the Quantum-Vacuum (Q-V) has an average density of 9.1x10^-27 kg/m^3 in the observable universe, but it can be much higher near electric charges such as the hydrogen proton.  In fact our new Eagleworks paper shows that the Q-V plasma density should increase at the 1/r^4 distance relationship demonstrated by the Casimir effect.  In fact due to this 1/r^4 scaling factor, as one approaches the classical radius of the proton, the Q-V density goes up to nuclear matter density.  To me that implies that ALL subatomic particles are just various types of vortices of Q-V...
Title: Re: EM Drive Developments - related to space flight applications - Thread 2
Post by: Stormbringer on 04/13/2015 08:28 pm
Stephen Hawking sings Monty Python's Galaxy song:

http://www.thedailybeast.com/articles/2015/04/13/stephen-hawking-covers-monty-python-s-galaxy-song.html
Title: Re: EM Drive Developments - related to space flight applications - Thread 2
Post by: Rodal on 04/13/2015 10:36 pm
...I'm re-appending a Rice University paper on the hydrodynamics of the vacuum for your reference and study.  You will find that a Q-V plasma acts quite a bit like a water based fluid, but with some startling differences as well, since it does salute MHD rules as well AND the still curious rules of the Quantum world.

Best, Paul M.

...
Hi Paul, looking at equation (4.22) in the attachment it appears that Stevenson's hydrodynamic model for the vacuum ignores electromagnetism and shear stresses: it has all components of Poynting's vector equal to zero and all shear stress components equal to zero.  The only non-zero component of Maxwell's stress tensor in Stevenson's formulation is pressure.  Thus, Stevenson models the vacuum as a fluid without any viscosity, incapable of supporting any shear stress, a superfluid.  He only considers the energy-density and the pressure as variables.

He doesn't appear (at least in the attached paper) to have developed his theory yet to consider any magnetohydrodynamics.  Also his theory is 3D+time standard relativity, without considering any extra space dimensions.

Does he have other papers where he deals with magnetohydrodynamics?


(http://upload.wikimedia.org/wikipedia/commons/thumb/f/fe/StressEnergyTensor_contravariant.svg/805px-StressEnergyTensor_contravariant.svg.png)




This paper discusses the electromagnetic (EM) field interacting with the quantum vacuum, where the vacuum is described as a superfluid:

http://arxiv.org/pdf/1204.6380v5.pdf

Interaction between the electromagnetic field and the quantum vacuum is shown to endow (charged) particles with internal structure and spatial extent.


Title: Re: EM Drive Developments - related to space flight applications - Thread 2
Post by: Mulletron on 04/14/2015 02:57 am
Just wanted to share the good news! Put a check in the box for diametric drives.

http://www.nature.com/nphys/journal/v9/n12/full/nphys2777.html?WT.ec_id=NPHYS-201312
http://phys.org/news/2013-10-optical-diametric.html
http://www.creol.ucf.edu/Research/Publications/7155.pdf

Other stuff I saved to share:
http://revolution-green.com/optics-breakthrough-demonstrates-new-behaviors-physics/
http://www.researchgate.net/publication/262451086_Paritytime-symmetric_whispering-gallery_microcavities
http://arxiv.org/ftp/arxiv/papers/1308/1308.4564.pdf


Hold onto your hats for this one....see the double torus in figure 1. Using that example, linked to below, applied to the whispering gallery research above, can the internal dimensions of the Emdrive be topologically morphed into the double torus (linked by a fiber) configuration from above and still conserve the same properties? I can do the transformation in my head and it kinda makes sense, and taking the continuously changing diameter longitudinally, where is the "balanced gain and loss?"
http://www.mit.edu/~linglu/pdf/2014_Lu_NatPhoton.pdf

Other stuff that I read over and is applicable:
http://arxiv.org/abs/0707.4407 (about the Cr₂O₃)
http://arxiv.org/abs/0903.1261 "
http://www-dft.ts.infn.it/~resta/gtse/axion.pdf (slide 11), and axions


http://arxiv.org/abs/0710.2219 (net force zero from virtual EM, nontrivial from real EM)


Way far out stuff, still applicable:
http://arxiv.org/abs/1408.1636
http://arxiv.org/ftp/arxiv/papers/1009/1009.5663.pdf
http://citeseerx.ist.psu.edu/viewdoc/download?doi=10.1.1.10.1027&rep=rep1&type=pdf

Managing breakthrough research, nice insider view:
http://gcep.stanford.edu/pdfs/lh-ivzYPrcfEnjOxV0q59g/7_11_millis_breakthrough.pdf

PT symmetry and nonreciprocal dynamics of linear and angular optical momentum.
http://arxiv.org/ftp/arxiv/papers/1409/1409.7031.pdf

The ESA had a study called Ariadna 04/1201 and a contract called PHOTONIMPULS ANR-09-BLAN-0088-01 to investigate Feigel's claims, derive a Lorentz invariant (and correct) description of the Feigel process, and figure out if the QV can be used for propulsion:
http://www.esa.int/gsp/ACT/ariadna/projects/ari_study_04-1201.html
http://www.esa.int/gsp/ACT/doc/ARI/ARI%20Study%20Report/ACT-RPT-PHY-ARI-041201-Koln_Feigel.pdf
http://www.esa.int/gsp/ACT/doc/ARI/ARI%20Study%20Report/ACT-RPT-PHY-ARI-041201-Grenoble_Feigel.pdf










Title: Re: EM Drive Developments - related to space flight applications - Thread 2
Post by: dustinthewind on 04/14/2015 08:48 am

http://citeseerx.ist.psu.edu/viewdoc/download?doi=10.1.1.10.1027&rep=rep1&type=pdf


bottom of page 8 "The phase displacements are exactly those that are required to produce constructive interference in the region behind the EGM Array and destructive interference in the region ahead of the array. They..."

Thanks Mulletron. I recognize the concept but I didn't know about this paper.  I am sure it is directly related to the idea I had posted in the other thread http://forum.nasaspaceflight.com/index.php?topic=36911.0 .  I find it interesting in the paper it is suggested relating it to the zero point field and possibly gravity.  It is also mentioned negative energy density and the resulting radiation projected. 

I'll have to take a closer look at some of the others.

and the post here:
I still think we can violate newtons 3rd law in a way but in another way it is not violated because the propulsion device projects radiation out one end.  This looks like radiation propulsion but by sticking a dielectric between the two current loops we can change the speed of light making the two current loops closer or lowering the frequency needed while also getting near field effects?  What this does for the radiation projected I'm not exactly sure but I would assume it should intensify.  This is assuming none of the current loops have constant current but are both changing in time and out of phase pi/2 (see figure EM Propulsion 2.png). 

I guess the idea was if there was something similar going on inside the radiation cavity...

The graphic posted before that is associated with the concept in the paper.
Title: Re: EM Drive Developments - related to space flight applications - Thread 2
Post by: flux_capacitor on 04/14/2015 02:03 pm
Just wanted to share the good news! Put a check in the box for diametric drives.

http://www.nature.com/nphys/journal/v9/n12/full/nphys2777.html?WT.ec_id=NPHYS-201312
http://phys.org/news/2013-10-optical-diametric.html
http://www.creol.ucf.edu/Research/Publications/7155.pdf

About this diametric drive stuff:

Those researchers cite Marc G. Millis (1997) but don't even bother to cite physicist Joaquin Mazdak Luttinger who was first to imagine that strange behavior in 1951, nor Herman Bondi who gave it its official name in 1957 in the first peer-reviewed paper (http://journals.aps.org/rmp/abstract/10.1103/RevModPhys.29.423) on the subject: the runaway motion. They even appear to don't know the designation by the way. I cite those pioneers only to show this is an old paradox known in GR for a long time, not a recent discovery.

Except that the "runaway effect" presupposes that the couple "positive mass + negative mass" would accelerate together indefinitely, which is an unphysical effect that ruled-out the possibility of the natural existence of negative mass in our universe. It presupposes that negative mass repels both other negative masses and positive masses.

Recent studies shows on the contrary that positive mass attracts positive mass, negative mass attracts negative mass, but positive mass and negative mass repel each other. See http://en.wikipedia.org/wiki/Negative mass#Runaway motion (http://en.wikipedia.org/wiki/Negative_mass#Runaway_motion)

In this context, no diametric drive.

Next, in the experiments you linked, how real massless photons in optical fibers can create an "antigravity effect" like a hypothetical negative mass is beyond me. I think their effect may be real but very, very different than the physical effect of a true effective negative mass matter, if we were able to produce some.



The ESA had a study called Ariadna 04/1201 and a contract called PHOTONIMPULS ANR-09-BLAN-0088-01 to investigate Feigel's claims, derive a Lorentz invariant (and correct) description of the Feigel process, and figure out if the QV can be used for propulsion:
http://www.esa.int/gsp/ACT/ariadna/projects/ari_study_04-1201.html
http://www.esa.int/gsp/ACT/doc/ARI/ARI%20Study%20Report/ACT-RPT-PHY-ARI-041201-Koln_Feigel.pdf
http://www.esa.int/gsp/ACT/doc/ARI/ARI%20Study%20Report/ACT-RPT-PHY-ARI-041201-Grenoble_Feigel.pdf

About the ARIADNA study #04/1201 from the ESA, evaluating the anomalous Feigel Process for the extraction of momentum from a vacuum, I quote the conclusions of the final report (http://www.esa.int/gsp/ACT/doc/ARI/ARI%20Study%20Report/ACT-RPT-PHY-ARI-041201-Koln_Feigel.pdf) (emphasis by me):
Quote
On the basis of our study, we come to the following conclusions: The derivation of the generally covariant relativistic constitutive relations for a moving magnetoelectric medium, together with the subsequent analysis of the vacuum waves travelling through the sample of a finite size shows that the magnetoelectric body will not move, despite the presence of a certain asummetry between the left- and right-moving waves in the matter. However, this only refers to the case of waves due to vacuum fluctuation.
For the real waves falling symmetrically from the two sides on a magnetoelectric body, we expect a nontrivial effect of the Feigel type. Thus, we cannot confirm the possibility of "extracting momentum from nothing".

So this study pretty much invalidated the anomalous Feigel effect when considering the QFV conjecture, aka "extraction of momentum from the virtual photons of the quantum vacuum fluctuations". It showed however that an anomalous Feigel effect could be obtained with real photons. So for a thruster, does the photons (microwaves ?) generator (magnetron ?) has to be decoupled (exterior) from the thruster or could it be part of it as in an EmDrive?
Title: Re: EM Drive Developments - related to space flight applications - Thread 2
Post by: Rodal on 04/14/2015 02:52 pm
Paul March has nicely summarized funding and future of the EM Drive project at NASA, a few hours ago in the Advanced Propulsion thread of LinkedIn, as follows:

Quote
"It is a prediction, but experimental results are needed. VASIMR drive finally got those funds at NASA, I would like to see something similar on EM-Drive"

So would we, but don't hold your breath. In the meantime we have enough funding for the rest of the NASA fiscal year to keep building up our 0.12-to-1.2kW, WR-340 waveguide based EM-Drive magnetron system on a teeter-totter balance system using a earlier aluminum frustum as our test article. The build of that experiment should be completed by the end of June using just civil servant labor and the existing hardware on hand. Then we get to see if Shawyer's and the Chinese's reported EM-Drive results are the real deal, or not.
Title: Re: EM Drive Developments - related to space flight applications - Thread 2
Post by: Notsosureofit on 04/14/2015 02:55 pm
@ Mulletron
Great find!

Separating the two photon directions allows them to control their interaction (equivalent of the cavity taper and/or dielectric).  In the EMdrive the two directions are intimately coupled within the cavity.  If you fold figure 1 in half (vertical fold line) you have something like a tapered microwave cavity.   I don't see the need to invoke negative mass (they did say effectively) as the same description looks like the "self accelerating" particle papers.

The question is "is the nonlinearity a required condition ?".
Title: Re: EM Drive Developments - related to space flight applications - Thread 2
Post by: Star One on 04/14/2015 03:03 pm

Paul March has nicely summarized funding and future of the EM Drive project at NASA, a few hours ago in the Advanced Propulsion thread of LinkedIn, as follows:

Quote
"It is a prediction, but experimental results are needed. VASIMR drive finally got those funds at NASA, I would like to see something similar on EM-Drive"

So would we, but don't hold your breath. In the meantime we have enough funding for the rest of the NASA fiscal year to keep building up our 0.12-to-1.2kW, WR-340 waveguide based EM-Drive magnetron system on a teeter-totter balance system using a earlier aluminum frustum as our test article. The build of that experiment should be completed by the end of June using just civil servant labor and the existing hardware on hand. Then we get to see if Shawyer's and the Chinese's reported EM-Drive results are the real deal, or not.

Thanks for that. But you know what they say   extraordinary claims require extraordinary evidence. I'm sure if this is ever proven too a sufficient degree they'll have no problem obtaining funding.
Title: Re: EM Drive Developments - related to space flight applications - Thread 2
Post by: Rodal on 04/14/2015 03:15 pm
@ Mulletron
Great find!

Separating the two photon directions allows them to control their interaction (equivalent of the cavity taper and/or dielectric).  In the EMdrive the two directions are intimately coupled within the cavity.  If you fold figure 1 in half (vertical fold line) you have something like a tapered microwave cavity.   I don't see the need to invoke negative mass (they did say effectively) as the same description looks like the "self accelerating" particle papers.

The question is "is the nonlinearity a required condition ?".

They (http://www.creol.ucf.edu/Research/Publications/7155.pdf) invoke the Kerr effect.  For the EM Drive, it would be the AC Kerr effect ( http://en.wikipedia.org/wiki/Kerr_effect#AC_Kerr_effect ) resulting in self-focusing.  The dielectric material would need to have a significant second-order nonlinear refractive index.  Does HDPE have a significant second-order nonlinear refractive index ?

Without a dielectric anisotropic insert in the EM Drive, it appears that there would be no nonlinearity, and therefore there would be no measurable "thrust" due to self-focusing due to this nonlinear effect. 

In the truncated cone EM Drive without a dielectric anisotropic polymer one would still have the "focusing" due to the geometry, but isn't this (conical geometry focusing effect) a linear effect and therefore self-cancelling for linear harmonic standing waves in the closed cavity?

Assuming that the vacuum itself has a negligible second-order nonlinear refractive index, it will be interesting to see whether Eagleworks reproduces Shawyer's and the Chinese's experiments without a polymer dielectric insert.
Title: Re: EM Drive Developments - related to space flight applications - Thread 2
Post by: Notsosureofit on 04/14/2015 04:05 pm
There are still the various thin film coatings (oxides, etc.) that naturally form on the inside of the cavity walls.

Added:  If nothing else, it looks like the force (acceleration) is in the right direction (toward the small end).

If I'm reading correctly, they did not (?) observe defocusing, and they would still get an effect if the negative mass was zero. (?)
Title: Re: EM Drive Developments - related to space flight applications - Thread 2
Post by: Stormbringer on 04/14/2015 04:58 pm





Thanks for that. But you know what they say   extraordinary claims require extraordinary evidence. I'm sure if this is ever proven too a sufficient degree they'll have no problem obtaining funding.

Sometimes something extraordinary is also pretty subtle. :) Why can't even modest results that show something new be extraordinary? Perhaps extraordinary is in the intellectual flexibility of the beholder. :)

When does a skeptic's evaluation turn to "that's extraordinary?" And is that a sound benchmark to evaluate the merit?

Title: Re: EM Drive Developments - related to space flight applications - Thread 2
Post by: Rodal on 04/14/2015 05:56 pm
@ Mulletron
Great find!

Separating the two photon directions allows them to control their interaction (equivalent of the cavity taper and/or dielectric).  In the EMdrive the two directions are intimately coupled within the cavity.  If you fold figure 1 in half (vertical fold line) you have something like a tapered microwave cavity.   I don't see the need to invoke negative mass (they did say effectively) as the same description looks like the "self accelerating" particle papers.

The question is "is the nonlinearity a required condition ?".
....

Added:  If nothing else, it looks like the force (acceleration) is in the right direction (toward the small end).

If I'm reading correctly, they did not (?) observe defocusing, and they would still get an effect if the negative mass was zero. (?)

You are entirely correct that they are referring to the effective mass (http://en.wikipedia.org/wiki/Effective_mass_%28solid-state_physics%29 ) behaving as if it would be negative because of regions of inverted curvature in the dispersion relation:

Quote
quasiparticles such as electrons and holes in solid-state
crystals, massspring systems or collective excitations such as
BoseEinstein condensates in lattices may exhibit a dispersion
relation with regions of inverted curvature where the effective mass
is negative.  Similarly, in photonic guiding structures, the effective
photon mass can be positive or negative depending on the sign of
the associated group velocity dispersion



Quote
This is directly analogous to a relativistic particle whose mass seems to increase during the course
of acceleration and therefore cannot exceed the velocity of light. The
hyperbolic trajectory of a constantly accelerated relativistic particle
viewed from an inertial reference frame coincides well with the
motion of our optical diametric drive (dashed white line overlaid to
Fig. 4a,b; see Supplementary Methods), thus proving the ongoing
action of the propulsion mechanism.




Concerning whether they observed defocusing, they state:

Quote
In contrast, when the same Gaussian
excitation excites the lower band (where the effective mass is
negative), the nonlinearity reverses its action and induces strong
nonlinear defocusing effects (Fig. 3b).
In our set-up, optical diametric drive acceleration is realized
by allowing the self-trapped wave packet of Fig. 3a to nonlinearly
interact with the defocusing beam shown in Fig. 3b. While the
positive-mass soliton is attracted by the negative-mass beam, the
latter is constantly repelled. As a result, the positive-mass beam will
permanently pursue its negative-mass counterpart while the latter
one tries to escape.
Title: Re: EM Drive Developments - related to space flight applications - Thread 2
Post by: Star One on 04/14/2015 06:05 pm





Thanks for that. But you know what they say   extraordinary claims require extraordinary evidence. I'm sure if this is ever proven too a sufficient degree they'll have no problem obtaining funding.

Sometimes something extraordinary is also pretty subtle. :) Why can't even modest results that show something new be extraordinary? Perhaps extraordinary is in the intellectual flexibility of the beholder. :)

When does a skeptic's evaluation turn to "that's extraordinary?" And is that a sound benchmark to evaluate the merit?

Problem is you're trying to convince the scientific & engineering majority and that's a heck of a lot of people you need to satisfy of the validity of your results.
Title: Re: EM Drive Developments - related to space flight applications - Thread 2
Post by: Mulletron on 04/14/2015 08:58 pm

The ESA had a study called Ariadna 04/1201 and a contract called PHOTONIMPULS ANR-09-BLAN-0088-01 to investigate Feigel's claims, derive a Lorentz invariant (and correct) description of the Feigel process, and figure out if the QV can be used for propulsion:
http://www.esa.int/gsp/ACT/ariadna/projects/ari_study_04-1201.html
http://www.esa.int/gsp/ACT/doc/ARI/ARI%20Study%20Report/ACT-RPT-PHY-ARI-041201-Koln_Feigel.pdf
http://www.esa.int/gsp/ACT/doc/ARI/ARI%20Study%20Report/ACT-RPT-PHY-ARI-041201-Grenoble_Feigel.pdf

About the ARIADNA study #04/1201 from the ESA, evaluating the anomalous Feigel Process for the extraction of momentum from a vacuum, I quote the conclusions of the final report (http://www.esa.int/gsp/ACT/doc/ARI/ARI%20Study%20Report/ACT-RPT-PHY-ARI-041201-Koln_Feigel.pdf) (emphasis by me):
Quote
On the basis of our study, we come to the following conclusions: The derivation of the generally covariant relativistic constitutive relations for a moving magnetoelectric medium, together with the subsequent analysis of the vacuum waves travelling through the sample of a finite size shows that the magnetoelectric body will not move, despite the presence of a certain asummetry between the left- and right-moving waves in the matter. However, this only refers to the case of waves due to vacuum fluctuation.
For the real waves falling symmetrically from the two sides on a magnetoelectric body, we expect a nontrivial effect of the Feigel type. Thus, we cannot confirm the possibility of "extracting momentum from nothing".

So this study pretty much invalidated the anomalous Feigel effect when considering the QFV conjecture, aka "extraction of momentum from the virtual photons of the quantum vacuum fluctuations". It showed however that an anomalous Feigel effect could be obtained with real photons. So for a thruster, does the photons (microwaves ?) generator (magnetron ?) has to be decoupled (exterior) from the thruster or could it be part of it as in an EmDrive?

So the ARIADNA study #04/1201 from the ESA in my view lends credibility to the researchers  (which they don't need anyway) and validity to the momentum from the QV approach (referencing van Tiggelen et al) to explaining Emdrive I've remarked on, in that the ESA was willing to throw money at the problem and put mainstream science on contract to figure it out. There's a back story to all this.

I realize that van Tiggelen may not even know what Emdrive is and would likely not want association with it.

The fact is though, that this copper can isn't expelling any reaction mass yet folks are measuring thrust from it. It is crystal clear that in order for this to happen, symmetry must be broken somewhere.
Trust me, I didn't stumble onto this because of the name of the paper. I was looking for broken symmetry (parity at the time) and found that it had already been done:
http://forum.nasaspaceflight.com/index.php?topic=29276.msg1269010#msg1269010

The Feigel paper, and the work that was inspired by it later on by others, (which is a progression of research spanning a decade) is why I feel this research is important and likely a good candidate to explain the anomalous thrust. 

We're aware that the Feigel effect is falsified*. The work that came later from it is what is important. Note that this story begins in 2004. There is a timeline which one must follow in order to make sense of available information. Here's the timeline:

1) Feigel comes along and says you can gain momentum from the QV. http://journals.aps.org/prl/abstract/10.1103/PhysRevLett.92.020404
http://arxiv.org/abs/physics/0304100

2)The ESA did a study to explore and reformulate the "broken" Feigel derivation in reference 8 here:
http://www.esa.int/gsp/ACT/ariadna/projects/ari_study_04-1201.html
http://journals.aps.org/prl/abstract/10.1103/PhysRevLett.92.020404
Takeaway: Others came along and said, um no you can't and here's what's wrong with your derivation. But they didn't walk away from the problem, as I will show below.

3) Two years later from 1), the say, ok well you can gain momentum from the QV in a Casimir type geometry. **
http://journals.aps.org/prl/abstract/10.1103/PhysRevLett.96.130402
Quote
A recent publication [Phys. Rev. Lett. 92, 020404 (2004)] raises the possibility of momentum transfer from zero-point quantum fluctuations to matter, controlled by applied electric and magnetic fields. We present a Lorentz-invariant description using field-theoretical regularization techniques. We find no momentum transfer for homogeneous media, but predict a very small transfer for a Casimir-type geometry.

4)So from there the prospect of gaining momentum from the QV using magnetoelectric matter was dependent on a "Casimir-type geometry." This is remarked about here*, **:
http://forum.nasaspaceflight.com/index.php?topic=36313.msg1335771#msg1335771
Several years later, others go on to say, that yes you can gain momentum from the QV and non-reciprocity is required, and here's how you can achieve that using a fully quantum approach (as opposed to Feigel's classical approach). That is all the PT symmetry breaking stuff from earlier posts.

5*) Later, other researchers predicted NO momentum transfer from the QV using the Feigel derivation.
http://arxiv.org/abs/0710.2219
http://arxiv.org/pdf/1304.3338.pdf

**On what is magnetoelectric and what isn't and Casimir geometries:
So magnetoelectric properties of matter can either be intrinsic or induced:

http://en.wikipedia.org/wiki/Magnetoelectric_effect (Cr2O3 et al)
https://hal.archives-ouvertes.fr/hal-00551421v1/document (nitrogen induced)
http://arxiv.org/abs/1101.0712 (nitrogen induced)

So from the research above on magnetoelectrics, you need a Casimir geometry:
From what I gather, there are 3 potential areas inside the emdrive which can fit into this metric.
1) For the unloaded cavity (no dielectric) the gaps where the end plates meet the frustum. Two Casimir plates in a ring, one large, one small.
2) For the loaded cavity, the above, but add in the gap between the copper and HDPE as well as the gap between the two HDPE discs.
3) More difficult to justify in my opinion, is the whole cavity technically is a Casimir cavity, albeit a gigantic one.

So it seems like a good idea to sandwich some Cr2O3 in between the two HDPE discs and/or between the HDPE and the copper end plate. That's why I got a baggie of the stuff to eventually try out. I have some .5 micron in powder form and .3 micron in paste form.

Title: Re: EM Drive Developments - related to space flight applications - Thread 2
Post by: Mulletron on 04/14/2015 10:01 pm
@ Mulletron
Great find!

Separating the two photon directions allows them to control their interaction (equivalent of the cavity taper and/or dielectric).  In the EMdrive the two directions are intimately coupled within the cavity.  If you fold figure 1 in half (vertical fold line) you have something like a tapered microwave cavity.   I don't see the need to invoke negative mass (they did say effectively) as the same description looks like the "self accelerating" particle papers.

The question is "is the nonlinearity a required condition ?".

I remember reading somewhere that nonlinearity is required. I remember @Rodal saying so too. I'm trying to find the references.*

I think I accidentally found evidence of nonlinearity when I saw frequency mixing reported here:
http://forum.nasaspaceflight.com/index.php?topic=36313.msg1353384#msg1353384

http://en.wikipedia.org/wiki/Nonlinear_optics#Nonlinear_optical_processes
https://www.vahala.caltech.edu/Research/Nonlinear

I didn't know what was going on at the time but after the fact it may have been one of these:
Second harmonic generation (SHG), or frequency doubling, generation of light with a doubled frequency (half the wavelength), two photons are destroyed creating a single photon at two times the frequency.
Third harmonic generation (THG), generation of light with a tripled frequency (one-third the wavelength), three photons are destroyed creating a single photon at three times the frequency.

Which means that I have to go back and do it again and try and measure the frequency difference.


*Here's one reference, but this is new. There are others I remember too.
http://revolution-green.com/optics-breakthrough-demonstrates-new-behaviors-physics/
Quote
“PT-symmetry breaking alone is not sufficient to have nonreciprocal response; operation in the nonlinear regime is also necessary. In the linear regime, light transmission is always reciprocal regardless of whether PT-symmetry is broken or not,”

Quote
....
While the formula I've been using is based on satisfying General Relativity, it does not tell us anything about the mechanism of momentum conservation.. PT asymmetry, as Mulletron mentions, is a viable candidate, and nonlinear frequency effects could (in theory) satisfy the requirement.
Can anybody present quantitative experimentally-measured data showing significant PT asymmetry or nonlinear frequency effects for a bulk High Density Polyethylene (purchased commercially from McMaster Carr, if my memory serves me correctly ?) used as the dielectric by NASA Eagleworks in their tests ?

That could be a Rosetta Stone...
Does it have to be nonlinearity of the dielectric? Why not nonlinearity of the cavity itself?
Title: Re: EM Drive Developments - related to space flight applications - Thread 2
Post by: Notsosureofit on 04/14/2015 11:17 pm
Well the cavity walls may have thin films of nonlinear material on them, but I'm not entirely convinced that nonlinear behavior is absolutely necessary.  The asymmetric behavior of the photon timelines  may well be the fundamental factor.
Title: Re: EM Drive Developments - related to space flight applications - Thread 2
Post by: Rodal on 04/14/2015 11:56 pm
Well the cavity walls may have thin films of nonlinear material on them, but I'm not entirely convinced that nonlinear behavior is absolutely necessary.  The asymmetric behavior of the photon timelines  may well be the fundamental factor.
I can understand the asymmetry arising from nonlinearity.   This paper (a really outstanding contribution by @Mulletron, who really deserves the strongest thank you for posting it)

          Optical diametric drive acceleration through action–reaction symmetry breaking
           Martin Wimmer, Alois Regensburger, Christoph Bersch, Mohammad-Ali Miri, Sascha Batz,Georgy Onishchukov,                Demetrios N. Christodoulides and Ulf Peschel

http://www.creol.ucf.edu/Research/Publications/7155.pdf

shows it to arise from a nonlinear (second order) term in a perturbation expansion.  @Mulletron had previously posted another paper showing the effect arising from a fourth order term in a perturbation expansion:

            Casimir momentum of magneto-chiral matter
            J. Babington and B. A. van Tiggelen

http://iopscience.iop.org/0295-5075/93/4/41002/fulltext/epl_93_4_41002.html

http://forum.nasaspaceflight.com/index.php?topic=36313.msg1354556#msg1354556

On the other hand, if there is no nonlinear behavior in a dielectric insert and there is no nonlinear material in the cavity inner walls, electromagnetic fields can be described by linear Maxwell's equations, and hence we have harmonic electromagnetic fields and standing waves inside the cavity, the Poynting vector will oscillate at a frequency twice as high as the electromagnetic field frequency, and hence if it points towards the small base during a half-period (of the Poynting vector frequency), it will point in the opposite direction towards the big base, during the next half-period.  The behavior seems reversible and hence self-cancelling if it can be described by linear Maxwell's equations.
Title: Re: EM Drive Developments - related to space flight applications - Thread 2
Post by: Notsosureofit on 04/15/2015 12:15 am
Yes, the question is, if an accelerating frame of reference is intrinsically nonlinear, (for photons along that axis) what transformation can maintain that relationship (ie. potential) in the rest frame.  Maxwell's Equations (the tautological relationship between the field and particle descriptions)  just won't do it by themselves.  Different higher spacial GR scenarios will theoretically generate a gravitational current, but I haven't seen anything convincing.
Title: Re: EM Drive Developments - related to space flight applications - Thread 2
Post by: Mulletron on 04/15/2015 01:46 am
Some other ways to induce non-reciprocity:
Any odd vector under time reversal.

http://ieeexplore.ieee.org/xpl/articleDetails.jsp?reload=true&arnumber=6715442
Quote
A structure can be non-reciprocal when biased with a vector that is odd under time reversal, i.e. the magnetic field, the current, the linear momentum and the angular momentum [J. D. Jackson, Classical Electrodynamics, John Wiley & Sons, Inc., 1999].

http://www.nature.com/ncomms/2013/130902/ncomms3407/abs/ncomms3407.html?message-global=remove
Quote
The Onsager–Casimir principle states that any odd vector under time reversal, such as electric current and linear momentum, can also produce a non-reciprocal response.


https://community.apan.org/cfs-file.ashx/__key/telligent-evolution-components-attachments/13-7784-00-00-00-13-16-14/Alu.pdf

Found this again:
http://arxiv.org/pdf/1211.0530.pdf

Title: Re: EM Drive Developments - related to space flight applications - Thread 2
Post by: CW on 04/15/2015 05:42 am
[snip]


[snip]

So from the research above on magnetoelectrics, you need a Casimir geometry:
From what I gather, there are 3 potential areas inside the emdrive which can fit into this metric.
1) For the unloaded cavity (no dielectric) the gaps where the end plates meet the frustum. Two Casimir plates in a ring, one large, one small.
2) For the loaded cavity, the above, but add in the gap between the copper and HDPE as well as the gap between the two HDPE discs.
3) More difficult to justify in my opinion, is the whole cavity technically is a Casimir cavity, albeit a gigantic one.

So it seems like a good idea to sandwich some Cr2O3 in between the two HDPE discs and/or between the HDPE and the copper end plate. That's why I got a baggie of the stuff to eventually try out. I have some .5 micron in powder form and .3 micron in paste form.

Regarding 3): Is it thinkable that the EM-drive is a kind of large 'inverse' Casimir cavity? In the 'normal' Casimir cavity, there is a reduced virtual photon spectrum, compared to outside the cavity. However, one might argue that inside the comparably giant EM-drive cavity, there is an increased real photon spectrum for that specific spatial volume, compared to the outside of the cavity and hence a similar effect arises as would be the case for a 'normal' Casimir cavity. Does that make sense?

What do you think?
Title: Re: EM Drive Developments - related to space flight applications - Thread 2
Post by: Stormbringer on 04/15/2015 06:02 am
Random silly question:

Casimir effects manifest in boundary areas between two closely spaced objects. I know you thinking in terms of solid materials with microscopic separations. But what about the gaps between lobes of RF in those graphics of the various modes? aren't they boundary interfaces in their own right?

EDIT: Isn't a requirement for casimir set ups spacing of incredibly small dimensions; not just any gap will do. considerable calibration of the gap is a part of experiments on Casimir force and effects.

EDIT: caveat: i recently read an article that had to do with amplification of vacuum energy. but Casimir force normally drops off extremely quickly with distance between plates or spheres.
Title: Re: EM Drive Developments - related to space flight applications - Thread 2
Post by: dustinthewind on 04/15/2015 09:02 am

http://www.creol.ucf.edu/Research/Publications/7155.pdf


I would dare say this is the same concept as the phase modulation of the electro-magnetic field for propulsion concept also.  I get the impression their negative mass laser is just a laser at a different phase from the other laser so that the effect it induces does negative work. 

"Photons excited in the lower band have a negative effective mass, whereas for thosein the upper branch this same quantity is positive."  and " The phase relation between the two loops is appropriately set by external phase modulation..."  page 787 last quarter

The idea being the 1st wire puts off a signal and light (electric field parallel to the current).  The -dB/dt light signal approaches the 2nd wire.  If the 2nd wire moves its current the same as the signal from the 1st wire then it does positive work against the light and is attracted (positive beam).  If the 2nd wires current moves the opposite as the current appears (light) to be in the 1st wire negative work is done and the wire is repulsed moving the device in the direction of negative work.   

The difference being were dealing with lasers and instead of microwaves, visible light.  Instead of wires I am guessing were dealing with currents in dielectrics or mediums. 

negative work-energy on one side, positive work-energy on the other and a force from one to the other.  Again with implications of light being induced to travel in the direction of constructive interference.

(by the way do the resulting photons from constructive interference merge? (frequency doubling)  I have been wondering if they do considering how the electric field of light from constructive interference overlaps)  I know in some non-linear crystals it can be done with lasers but at a very specific angle.  I would guess in the case of the propulsion if it it is a small effect, if at all, but it could lead to high energy radiation if it builds up, maybe.  The mention of frequency mixing sounds like it for some reason. 

Title: Re: EM Drive Developments - related to space flight applications - Thread 2
Post by: Mulletron on 04/15/2015 10:43 am
[snip]


[snip]

So from the research above on magnetoelectrics, you need a Casimir geometry:
From what I gather, there are 3 potential areas inside the emdrive which can fit into this metric.

[snip]
3) More difficult to justify in my opinion, is the whole cavity technically is a Casimir cavity, albeit a gigantic one.


Regarding 3): Is it thinkable that the EM-drive is a kind of large 'inverse' Casimir cavity? In the 'normal' Casimir cavity, there is a reduced virtual photon spectrum, compared to outside the cavity. However, one might argue that inside the comparably giant EM-drive cavity, there is an increased real photon spectrum for that specific spatial volume, compared to the outside of the cavity and hence a similar effect arises as would be the case for a 'normal' Casimir cavity. Does that make sense?

What do you think?

It makes sense but the frustum cavity is so huge compared to a Casimir cavity. The Casimir force doesn't dominate in such a spacious regime.
Title: Re: EM Drive Developments - related to space flight applications - Thread 2
Post by: CW on 04/15/2015 11:41 am
[snip]


[snip]

So from the research above on magnetoelectrics, you need a Casimir geometry:
From what I gather, there are 3 potential areas inside the emdrive which can fit into this metric.

[snip]
3) More difficult to justify in my opinion, is the whole cavity technically is a Casimir cavity, albeit a gigantic one.


Regarding 3): Is it thinkable that the EM-drive is a kind of large 'inverse' Casimir cavity? In the 'normal' Casimir cavity, there is a reduced virtual photon spectrum, compared to outside the cavity. However, one might argue that inside the comparably giant EM-drive cavity, there is an increased real photon spectrum for that specific spatial volume, compared to the outside of the cavity and hence a similar effect arises as would be the case for a 'normal' Casimir cavity. Does that make sense?

What do you think?

It makes sense but the frustum cavity is so huge compared to a Casimir cavity. The Casimir force doesn't dominate in such a spacious regime.

Hmm.. I seem to remember that Paul March wrote somewhere in this thread that the Casimir regime shows a 1/r^4 dependency (due to 5 proposed spatial dimensions). I'm just curious whether there is a possibility of existence for a kind of 'pseudo' (or what have you) Casimir effect that shows up in our 3+1 dimensional space for 'real photons' instead of the proposed 5+1 dimensional space for 'virtual photons'. Following the logic of 'n+1 dimensions needed', could there be a Casimir-like regime that shows a 1/r^2 dependency for our 3D-space? Just wondering :) .
Title: Re: EM Drive Developments - related to space flight applications - Thread 2
Post by: Notsosureofit on 04/15/2015 02:25 pm
FYI:  one of many...about nonlinear surface, etc.

http://www.lajpe.org/sep12/14_LAJPE_687_Gouri_Sankar_preprint_corr_f.pdf
Title: Re: EM Drive Developments - related to space flight applications - Thread 2
Post by: dustinthewind on 04/15/2015 05:16 pm
Something that makes me think that time delay phase modulation EM propulsion is going on inside the cavity, [diametric propulsion if you want to call it that] is I remember the big plate being fairly hot like positive work was going on there and the narrow end of the cavity looked fairly cool in comparison (negative work?).  If so then propulsion being toward the side doing negative work [the narrow end]. 
Title: Re: EM Drive Developments - related to space flight applications - Thread 2
Post by: Rodal on 04/15/2015 05:34 pm
Something that makes me think that time delay phase modulation EM propulsion is going on inside the cavity, [diametric propulsion if you want to call it that] is I remember the big plate being fairly hot like positive work was going on there and the narrow end of the cavity looked fairly cool in comparison (negative work?).  If so then propulsion being toward the side doing negative work [the narrow end].
The small base of NASA's Eagleworks EM Drive truncated cone is insulated by the (2.13 inches) thick polymer HDPE while the large base of the truncated cone is not insulated and it is directly exposed to induction heating.

(http://forum.nasaspaceflight.com/index.php?action=dlattach;topic=36313.0;attach=636341;image)

On the other hand, Prof. Juan Yang in China did not use dielectric inserts, and hence neither end of their truncated cone EM Drive was insulated.  The temperature at the center of the small end (thermocouple #1) in the Chinese experiment rose much more than the temperature at the center of the big end (thermocouple #6), actually the small end experienced the highest overall temperature in the Chinese EM Drive experiments.


(http://forum.nasaspaceflight.com/index.php?action=dlattach;topic=29276.0;attach=622853)
(http://forum.nasaspaceflight.com/index.php?action=dlattach;topic=29276.0;attach=655010)
Title: Re: EM Drive Developments - related to space flight applications - Thread 2
Post by: MazonDel on 04/15/2015 08:25 pm
Greetings again everyone,

A thought I just had, given that the frustum cavity is a truncated cone and the HDPE is a cylinder, does anyone have any idea how the RF might be effected if the HDPE was converted into a truncated cone such that it's depth into the frustum is the same (2.13 inches), but it actually takes up the entire width with a relatively tight fit?
Title: Re: EM Drive Developments - related to space flight applications - Thread 2
Post by: rfmwguy on 04/15/2015 08:44 pm
Thanks for the welcome. I've been around high power RF for many years and have seen low temp PTFE issues at relatively low temps. Specifically, changes in capacitance, yeilding center frequency drift in tchebychev bandpass filters using PTFE tape and discs. Outgassing in hermetically sealed tubes were noticed. Could be totally unrelated but...maybe not.
Agreed, outgassing (due to microwave heating of water vapor previously trapped in the HDPE or PTFE polymer dielectric) would be something to watch out for in a vacuum environment at significantly lower temperatures (near 200 deg F) than pyrolysis (>700 deg F). 



This Lawrence Livermore Lab report on outgassing of water vapor from HDPE is pertinent:

Vacuum Outgassing of High Density Polyethylene
L. N. Dinh*, J. Sze, M. A. Schildbach, S. C. Chinn, R. S. Maxwell, P. Raboin, W. McLean II
Lawrence Livermore National Laboratory, Livermore, Ca, USA

https://e-reports-ext.llnl.gov/pdf/364291.pdf


It concludes that outgassing of H2O from HDPE can be significantly reduced by vacuum baking at 368 degres K
(203 deg F) for a few hours prior to device assembly.

Well put...brings back memories of PTFE baking at 150C overnight. We also noticed other contaminants such as machine oil (circular dielectrics formed with a lathe), oil from parts handler's skin, etc. Might be even more important to clean and shock the dielectrics for vacuum testing.
Title: Re: EM Drive Developments - related to space flight applications - Thread 2
Post by: dustinthewind on 04/17/2015 06:43 am
Something that makes me think that time delay phase modulation EM propulsion is going on inside the cavity, [diametric propulsion if you want to call it that] is I remember the big plate being fairly hot like positive work was going on there and the narrow end of the cavity looked fairly cool in comparison (negative work?).  If so then propulsion being toward the side doing negative work [the narrow end].
The small base of NASA's Eagleworks EM Drive truncated cone is insulated by the (2.13 inches) thick polymer HDPE while the large base of the truncated cone is not insulated and it is directly exposed to induction heating.

(http://forum.nasaspaceflight.com/index.php?action=dlattach;topic=36313.0;attach=636341;image)

On the other hand, Prof. Juan Yang in China did not use dielectric inserts, and hence neither end of their truncated cone EM Drive was insulated.  The temperature at the center of the small end (thermocouple #1) in the Chinese experiment rose much more than the temperature at the center of the big end (thermocouple #6), actually the small end experienced the highest overall temperature in the Chinese EM Drive experiments.


(http://forum.nasaspaceflight.com/index.php?action=dlattach;topic=29276.0;attach=622853)
(http://forum.nasaspaceflight.com/index.php?action=dlattach;topic=29276.0;attach=655010)

Thanks for pointing out the difference between the two drives.  I notice they both get the same force direction.  I am now wondering about the over all thermal energy at the large end compared to the sidewalls and small end together and if they differ overall.  The Chinese EM Drive small end may be more hot but it's also a small area compared to the large end.   I'll have to look up if the Chinese version is more or less efficient (N/Watt) than the one with the dielectric?  This makes me wonder if more negative work would be going on with the dielectric installed making it more efficient.
Title: Re: EM Drive Developments - related to space flight applications - Thread 2
Post by: Rodal on 04/17/2015 02:21 pm

We should discuss these news, that NASA Eagleworks

finally observed the first spacetime contraction effects that we are fairly confident are the real deal...the laser interferometer observed spacetime contractions are being developed in a TM010 RF resonant cavity that is driving ac E-field levels over 900kV/m at a 1.48 GHz rate. 
Title: Re: EM Drive Developments - related to space flight applications - Thread 2
Post by: Star One on 04/17/2015 02:50 pm
Just how significant is that to physics in general and is it something that has been theorised to happen?
Title: Re: EM Drive Developments - related to space flight applications - Thread 2
Post by: Rodal on 04/17/2015 02:57 pm
Just how significant is that to physics in general and is it something that has been theorised to happen?

See:

http://en.wikipedia.org/wiki/Warp-field_experiments

http://en.wikipedia.org/wiki/White%E2%80%93Juday_warp-field_interferometer

http://science.nasa.gov/science-news/science-at-nasa/2011/04may_epic/

http://en.wikipedia.org/wiki/Gravity_Probe_B

http://science.nasa.gov/science-news/science-at-nasa/2004/19apr_gravitomagnetism/

EDIT: If my memory is correct, I think that NASA Eagleworks is aware of the issue with air refraction addressed in this paper by Lee and Cleaver at Baylor University:

http://arxiv.org/ftp/arxiv/papers/1407/1407.7772.pdf

http://www.quora.com/Can-a-large-energy-field-other-than-gravity-warp-spacetime


Title: Re: EM Drive Developments - related to space flight applications - Thread 2
Post by: aceshigh on 04/17/2015 06:45 pm

We should discuss these news, that NASA Eagleworks

finally observed the first spacetime contraction effects that we are fairly confident are the real deal...the laser interferometer observed spacetime contractions are being developed in a TM010 RF resonant cavity that is driving ac E-field levels over 900kV/m at a 1.48 GHz rate. 


would that be a subject to ME Drive thread?

even if the underlying principle is related (quantum vaccum, etc), still, both drives would work very differently... so the warp drive needs it´s own thread, imho.
Title: Re: EM Drive Developments - related to space flight applications - Thread 2
Post by: Rodal on 04/17/2015 07:29 pm

We should discuss these news, that NASA Eagleworks

finally observed the first spacetime contraction effects that we are fairly confident are the real deal...the laser interferometer observed spacetime contractions are being developed in a TM010 RF resonant cavity that is driving ac E-field levels over 900kV/m at a 1.48 GHz rate. 

would that be a subject to ME Drive thread?

even if the underlying principle is related (quantum vaccum, etc), still, both drives would work very differently... so the warp drive needs it´s own thread, imho.

Yes, definitely a subject for this thread: "EM Drive Developments", insofar as NASA Eagleworks reports to have laser interferometer readings (which they attribute to spacetime contractions after several years of trying) by using an EM Drive (pillbox shape, short cylinder, I think, given the reported frequency and mode shape) in TM010 resonance mode to produce the measurements.

The reason why it belongs in this thread has nothing to do with the reason you state <<the underlying principle is related (quantum vaccum, etc),>>, but instead it has everything to do with the fact that to obtain the measurements they had to use an EM Drive !

It is very significant that it is only an EM Drive (rather than the other electromagnetic devices, including capacitors, that were previously tried)  that finally produced a laser interferometer signal that they are confident to report as such.

And concerning your statement that <<both drives would work very differently... so the warp drive needs it´s own thread>> that's incorrect: the EM Drive in the laser interferometer experiment didn't work "very differently", it worked in a transverse magnetic resonant mode at 1.4 GHz frequency, so you should clarify what you mean by working very differently from the EM Drives that have been discussed in this thread.

Discussions about laser interferometer experiments with other types of devices (other than the EM Drive) belong elsewhere.
Title: Re: EM Drive Developments - related to space flight applications - Thread 2
Post by: Chrochne on 04/17/2015 07:53 pm

We should discuss these news, that NASA Eagleworks

finally observed the first spacetime contraction effects that we are fairly confident are the real deal...the laser interferometer observed spacetime contractions are being developed in a TM010 RF resonant cavity that is driving ac E-field levels over 900kV/m at a 1.48 GHz rate. 

would that be a subject to ME Drive thread?

even if the underlying principle is related (quantum vaccum, etc), still, both drives would work very differently... so the warp drive needs it´s own thread, imho.

Yes, definitely a subject for this thread: "EM Drive Developments", insofar as NASA Eagleworks reports to have measured spacetime contractions (after several years of trying) by using an EM Drive (pillbox shape, short cylinder, I think, given the reported frequency and mode shape) in TM010 resonance mode to produce the measurements.

The reason why it belongs in this thread has nothing to do with the reason you state <<the underlying principle is related (quantum vaccum, etc),>>, but instead it has everything to do with the fact that to obtain the measurements they had to use an EM Drive !

It is very significant that it is only an EM Drive (rather than the other drives that were previously tried)  that finally produced a laser interferometer signal that they are confident to report as such.

Discussions about laser interferometer readigns with other types of drives (other than the EM Drive) belong in other, separate threads.

Very interesting Dr. Rodal. I noticed that as well. It seems that EmDrive technology is opening doors to the new levels never before explored. And it is very interesting to see that happen "live" trough posts of Mr. Paul March.  I was wondering how you guys would react to this. Lately, it is like reading a sci-fi book except it is really happening in Eagleworks laboratory :)
Title: Re: EM Drive Developments - related to space flight applications - Thread 2
Post by: tchernik on 04/17/2015 07:54 pm

We should discuss these news, that NASA Eagleworks

finally observed the first spacetime contraction effects that we are fairly confident are the real deal...the laser interferometer observed spacetime contractions are being developed in a TM010 RF resonant cavity that is driving ac E-field levels over 900kV/m at a 1.48 GHz rate. 

would that be a subject to ME Drive thread?

even if the underlying principle is related (quantum vaccum, etc), still, both drives would work very differently... so the warp drive needs it´s own thread, imho.

Yes, definitely a subject for this thread: "EM Drive Developments", insofar as NASA Eagleworks reports to have measured spacetime contractions (after several years of trying) by using an EM Drive (pillbox shape, short cylinder, I think, given the reported frequency and mode shape) in TM010 resonance mode to produce the measurements.

The reason why it belongs in this thread has nothing to do with the reason you state <<the underlying principle is related (quantum vaccum, etc),>>, but instead it has everything to do with the fact that to obtain the measurements they had to use an EM Drive !

It is very significant that it is only an EM Drive (rather than the other drives that were previously tried)  that finally produced a laser interferometer signal that they are confident to report as such.

Discussions about laser interferometer readigns with other types of drives (other than the EM Drive) belong in other, separate threads.

This is a point that really stood out for me, among the latest salvo of updates by Paul M.

It could represent a way to falsify the assertions about something really weird and novel going on inside these drives. And probably, becoming even an explanation of the observed results (e.g. by making the measured forces gravitic or space-warp related).

As a totally not-expert in these subjects, I can't say if the topic is pertinent or not to this topic. So, what's the usual approach in the forum? Do our kind hosts prefer to take such tangential -yet relevant- topics outside their parent thread? I'm also assuming the topic is still relevant to space flight, of course.
Title: Re: EM Drive Developments - related to space flight applications - Thread 2
Post by: Rodal on 04/17/2015 07:58 pm

We should discuss these news, that NASA Eagleworks

finally observed the first spacetime contraction effects that we are fairly confident are the real deal...the laser interferometer observed spacetime contractions are being developed in a TM010 RF resonant cavity that is driving ac E-field levels over 900kV/m at a 1.48 GHz rate. 

would that be a subject to ME Drive thread?

even if the underlying principle is related (quantum vaccum, etc), still, both drives would work very differently... so the warp drive needs it´s own thread, imho.

Yes, definitely a subject for this thread: "EM Drive Developments", insofar as NASA Eagleworks reports to have measured spacetime contractions (after several years of trying) by using an EM Drive (pillbox shape, short cylinder, I think, given the reported frequency and mode shape) in TM010 resonance mode to produce the measurements.

The reason why it belongs in this thread has nothing to do with the reason you state <<the underlying principle is related (quantum vaccum, etc),>>, but instead it has everything to do with the fact that to obtain the measurements they had to use an EM Drive !

It is very significant that it is only an EM Drive (rather than the other drives that were previously tried)  that finally produced a laser interferometer signal that they are confident to report as such.

Discussions about laser interferometer readigns with other types of drives (other than the EM Drive) belong in other, separate threads.

This is a point that really stood out for me, among the latest salvo of updates by Paul M.

It could represent a way to falsify the assertions about something really weird and novel going on inside these drives. And probably, becoming even an explanation of the observed results (e.g. by making the measured forces gravitic or space-warp related).

As a totally not-expert in this subjects, I can't say if the topic is pertinent or not to this topic. So, what's the usual approach in the forum? Do our kind hosts prefer to take such tangential -yet relevant- topics outside their parent thread? I'm also assuming the topic is still relevant to space flight, of course.
The poster asking for a change of thread may have innocently assumed that the laser interferometer tests were performed with a similar set-up as several years ago (that resulted in null measurements): the poster may not have understood that the recent experiments that for the first time are being reported as giving confidence in these laser interferometer measurements were performed with an EM Drive.
Title: Re: EM Drive Developments - related to space flight applications - Thread 2
Post by: Star One on 04/17/2015 08:11 pm
How close are we to establishing if there is genuine and useful effect with these drives or are we still a long way from a categorical answer. From my perspective it appears progress has been made but it seems like there's still a long, long way to go partly due to limited funding. I hate to say this but I suspect that outside of Eagle Works until they produce strong positive cast iron results backed by watertight peer reviewed theoretical papers you're not going to get others trying to replicate this work and it will continue to be perceived as fringe science.
Title: Re: EM Drive Developments - related to space flight applications - Thread 2
Post by: ResonantSpace on 04/17/2015 09:38 pm

Birchoff:

"Is it possible to run the Frustum in a null configuration? If so, is that in the plans before the next report is published?"

Yes and yes.  In fact it was one of the requests made by the blue ribbon panel of PhDs that NASA/EP hired to review the Eagleworks Lab's theoretical and experimental work last summer.  Even if will take a new mounting arrangement to get it accomplished. 

Overall though the blue ribbon panel's experimentalists appeared to be pleased with our previous and upcoming lab work.  However they ripped into Sonny's QVF/MHD conjecture because it relies on the quantum vacuum being mutable and engineer-able whereas the current physics mainstream thinks that the quantum vacuum is an immutable ground energy state of the universe that can-NOT be used to convey energy or momentum as proposed by Dr. White.   However they brushed aside Sonny's QVF based derivation of the Bohr hydrogen atom electron radius as a "mathematical coincidence" and didn't have a word to say what the Casimir effect and other quantum vacuum phenomenon were caused by, that can only occur only if the QV is mutable and can convey energy and momentum.   So Sonny and Jerry Vera took it upon themselves last fall to increase this mathematical coincidence from one to more than 47 times as they explored the QV created atomic electron shell radii for atoms up to atomic number 7 all based on the QV being the root cause for all of it including the origins of the electron and all other subatomic particles.   

Wow. First off, I would like to thank you, Paul for keeping us all updated on the research and experiments you are all are doing over there. This is both fascinating in terms of technological applications but also reaches into the fundamentals of our understanding of physics. There is a great deal of evidence both in theory and in experiment showing that space is not empty as it is thought to be as probably most of us here know.

I am particularly interested in what you have said that I highlighted in red. Is there more information you can share about this or is it all unpublished and kept under wraps thus far? I would be thrilled to hear more insight or elaboration on this subject!

Thank you again for the work you are doing over there!
Title: Re: EM Drive Developments - related to space flight applications - Thread 2
Post by: dustinthewind on 04/18/2015 02:42 am

We should discuss these news, that NASA Eagleworks

finally observed the first spacetime contraction effects that we are fairly confident are the real deal...the laser interferometer observed spacetime contractions are being developed in a TM010 RF resonant cavity that is driving ac E-field levels over 900kV/m at a 1.48 GHz rate. 

I do think this is an important subject.  I believe it is important because I don't believe we have ever detected space/time waves as of yet, have we?  Isn't that the whole reason they built LIGO? (i think stands for "laser interferometry gravity-wave observatory."  If this is true and no other observations have been made then this is a major find. 

I did have my suspicions of it however because of a few reasons.  The idea of pushing against something by modulating information faster than space time can keep up to violate newtons law for conservation of momentum seems to imply pushing against something.  Most people want to say it is just light but in the back of my mind it was nagging me if we could be pushing against space time.  That is there is distance between currents and it takes time for that information to travel thus pushing against space time.  It was only speculation in the back of my mind.

I also read an experimental paper not long ago that observed something like a gravity pulse.  I'll link it here: http://arxiv.org/abs/physics/0108005 .  They claimed they observed a gravity pulse when discharging a superconducting capacitor that would move a pendulum through shielding.  It is a bit long to read through. 

This guy here: http://citeseerx.ist.psu.edu/viewdoc/download?doi=10.1.1.10.1027&rep=rep1&type=pdf also claims the propulsion is greater than just emission of light, and suggested to possibly connected to Zero Point Energy or Gravity "The Casimir effect is made possible by the fact that the EM frequency modes used cannot pass through the conducting boundaries. A simple thought experiment shows that if there is any connection
between the EM ZPF and gravity, it must be at frequency modes that are not reflected by conducting surfaces. For example, a heavy object placed inside a closed metal box does not experience any appreciable
loss of weight. Therefore, the connection to gravitational fields should be looked for at EM frequency modes
that pass through all known materials, such as the sub-atomic frequency spectra"

This paper also suggests possibly more propulsion than just the light: http://arxiv.org/abs/1502.06288 "Therefore, as it is shown in Ref. [15], the propulsive force can result from the near- field and=or
the far-fi eld (radiative) mode, that is, it is not always the radiative mode of propulsion that
can be useful under the point of view of practical engineering"

Of course I only suspect what I'm talking about is related to what you dealing with (the cavity). 

One concern of mine is if they put the interferometer mirror too close to the cavity and the mirror was conductive.  The imaginary part of the magnetic field (I may be wrong about this) may penetrate the cavity wall and induce a current in the mirror.  This alternating current in the mirror would push against the current in the cavity and possibly give a false signal?  Does that sound plausible?  I don't yet know the design of this interferometry experiment.   

I had an idea one day it would be interesting to set up that capacitor experiment that gave the gravity pulse and see if LIGO or some other gravity observation unit could detect a space time wave from it. 
Title: Re: EM Drive Developments - related to space flight applications - Thread 2
Post by: aceshigh on 04/18/2015 05:02 am
Dr Rodal, I assume the laser interferometer detecting space-time contractions inside an ME Drive is subject to this thread. I agree.

What I think would be subject to another thread would be a WARP DRIVE, which was the reason Dr White built the laser interferometer.

It´s true the message I quoted said nothing about "warp drive", but I thought warp drives might be implied and end up being discussed, because they were the origin of the interferometer experiments.


if we try to understand ME Drive using these results of space warping being detected inside the cone, yes, it´s totally related to this thread.

if people decide to talk about superluminal travel by warping space, and how to even match the results inside the cone and how to extrapolate that to an alcubierre drive, than that´s a matter to another thread, right?
Title: Re: EM Drive Developments - related to space flight applications - Thread 2
Post by: Stormbringer on 04/18/2015 05:21 am
I really hope for progress on the interferometry experiments but i have reservations about the whole thing:


The mass of even tiny things like individual elementary particles warps space a tiny bit as per Einstein's description of gravity. So with sufficient sensitivity any interferometer measurement should detect a warp whether there is one from a device being turned on or not.

The problem is the warp effect expected is so small that I cannot be sure that the effect detected isn't due to the natural space curvature due to the mass of the device or particles in the device rather than an artificially enduced warp.

I guess i just don't understand the difference in magnitude between any natural signal and the expected artificially induced one.

If you have the beam positioned perfectly to detect a signal in the artificial regime how far to the left or right would the laser have to transit to find the natural curvature region?

I think it would be unmanageably tiny. I am not an expert so i am just going by intuition mostly. I hope that someone can reassure me i am wrong; because I really want it to be that simple to alter space-time curvature.

 
Title: Re: EM Drive Developments - related to space flight applications - Thread 2
Post by: CW on 04/18/2015 09:48 am
On a philosophical side note, I think that this whole endeavour is eventually related to 'spacetime engineering'. All existing particles or matter could be viewed as a locally 'degenerated QV', because QV is the superset of all that can potentially physically exist. In the exact place and time where a particle exists, other particles cannot exist - because that specific 'coordinate' in potentiality is already taken. The physical potentiality in that particle's place is less than in the places where no particles exist. I hence get the impression that gravity is actually a reflection or measure of how 'defective' QV is. Not without reason (to give an extreme example) we call black holes 'holes'.

Also, I have a feeling that Heisenberg's Uncertainty Principle might be applicable to those defects in QV, so that they are actually 'smeared' and give an observable 'QV defect gradient', that leads to particles 'attracting' each other. We call that 'gravity'. Rather than attracting, though, I tend to think that gravity is a 'pushing effect' exerted on particles by QV potentiality gradients. Maybe it makes some sense to try and think about the EM-drive and similar devices in this way, as it might open up more mental ways to find the correct answer.

Going back to spacetime engineering:

As matter and particles are a subset of QV and hence a part of QV itself, by building stuff we basically bring together 'defects in QV potentiality' in a very specific manner and geometric placement - I think it is correct to say that we are engineering spacetime itself by doing this. Hence, what Eagleworks for instance is doing here, is figuring out a spacetime configuration (in shape of properly placed matter and energy) that creates a behavior of spacetime, which is most likely not even observable in most of the practically infinite amount of spacetime configurations (in shape of properly placed matter and energy) that 'naturally' or statistically occur in the (at least current) rest-universe.

Let's see how far this can go and if the Eagleworks boys can cause spacetime to do some funny tricks for us.
Title: Re: EM Drive Developments - related to space flight applications - Thread 2
Post by: Rodal on 04/18/2015 05:27 pm

We should discuss these news, that NASA Eagleworks

finally observed the first spacetime contraction effects that we are fairly confident are the real deal...the laser interferometer observed spacetime contractions are being developed in a TM010 RF resonant cavity that is driving ac E-field levels over 900kV/m at a 1.48 GHz rate. 

would that be a subject to ME Drive thread?

even if the underlying principle is related (quantum vaccum, etc), still, both drives would work very differently... so the warp drive needs it´s own thread, imho.

Yes, definitely a subject for this thread: "EM Drive Developments", insofar as NASA Eagleworks reports to have laser interferometer readings (which they attribute to spacetime contractions after several years of trying) by using an EM Drive (pillbox shape, short cylinder, I think, given the reported frequency and mode shape) in TM010 resonance mode to produce the measurements.

The reason why it belongs in this thread has nothing to do with the reason you state <<the underlying principle is related (quantum vaccum, etc),>>, but instead it has everything to do with the fact that to obtain the measurements they had to use an EM Drive !

It is very significant that it is only an EM Drive (rather than the other electromagnetic devices, including capacitors, that were previously tried)  that finally produced a laser interferometer signal that they are confident to report as such.

And concerning your statement that <<both drives would work very differently... so the warp drive needs it´s own thread>> that's incorrect: the EM Drive in the laser interferometer experiment didn't work "very differently", it worked in a transverse magnetic resonant mode at 1.4 GHz frequency, so you should clarify what you mean by working very differently from the EM Drives that have been discussed in this thread.

Discussions about laser interferometer experiments with other types of devices (other than the EM Drive) belong elsewhere.
How do I infer that the EM Drive that was used in the first experiments reported to give laser interferometer readings of spacetime contractions must have been a pillbox-shaped, short cylinder ?

Based on the information given by Paul March:

Quote
TM010 RF resonant cavity ...at a 1.48 GHz rate.

As I posted previously in this thread, a truncated cone cannot have a TM010 mode because the electromagnetic fields in a truncated cone cannot be constant in the longitudinal direction of the cone (due to the geometry of the cone producing a focusing, attenuating effect in the longitudinal direction).  Therefore when somebody describes a resonant cavity having a TM010 mode it must be a cylindrical cavity.  (Or, if the analyst is describing the mode of a truncated cone analogous to a cylindrical cavity, the truncated cone must be such that the big and small bases have approximately the same diameter, because otherwise this approximate mode would be cut-off in a truncated cone).

Using the well-known equation for the frequency of a cylindrical cavity (see for example  http://en.wikipedia.org/wiki/Microwave_cavity#Cylindrical_cavity  ) , the frequency for a TM mode is:

fmnp =( c/(2 Pi  Sqrt[μr εr]) ) Sqrt[(Xmn/R)^2+(p Pi / L)^2]

for TM010 we have m=0, n=1, p=0 and therefore:

D = 2 R = ( c/(Pi  Sqrt[μr εr]) ) X01/f010

and since the speed of light in air is ( http://en.wikipedia.org/wiki/Speed_of_light ):

 c/(Sqrt[μr εr]) = 299700000 m/s

and the Bessel Zero for m=0, n=1 is (see  http://wwwal.kuicr.kyoto-u.ac.jp/www/accelerator/a4/besselroot.htmlx ):

X01 = 2.40482555769577

and the frequency, according to Paul March (see http://forum.nasaspaceflight.com/index.php?topic=36313.msg1355764#msg1355764 ) was:

f010 = 1.48*10^9 Hz

We conclude that the diameter of the cylindrical resonant cavity used in these experiments was:

D = (299700000 m/s)*2.40482555769577 / (Pi * 1.48*10^9 1/s) = 0.1550 m = 6.103 inches

Hence we conclude that the RF resonant cavity used for the first successful experiments reported to give laser interferometer readings of spacetime contractions must have been a cylinder having 6.103 inches diameter.

How do I infer that the cylinder must have been short, "pillbox shaped" with a length equal or smaller than its diameter ?  Because the geometry of the experimental setup previously disclosed by NASA Eagleworks does not appear to be big enough to accommodate a significantly longer cylinder:

http://upload.wikimedia.org/wikipedia/commons/9/9b/White-Juday_Warp_Field_Interferometer_Experiment.png

Perhaps Paul March could confirm what were the geometrical dimensions of the EM Drive used in these experiments,  and whether or not it had a dielectric insert and if so what type of dielectric was used.
Title: Re: EM Drive Developments - related to space flight applications - Thread 2
Post by: dustinthewind on 04/18/2015 07:35 pm

We should discuss these news, that NASA Eagleworks

finally observed the first spacetime contraction effects that we are fairly confident are the real deal...the laser interferometer observed spacetime contractions are being developed in a TM010 RF resonant cavity that is driving ac E-field levels over 900kV/m at a 1.48 GHz rate. 

would that be a subject to ME Drive thread?

even if the underlying principle is related (quantum vaccum, etc), still, both drives would work very differently... so the warp drive needs it´s own thread, imho.

Yes, definitely a subject for this thread: "EM Drive Developments", insofar as NASA Eagleworks reports to have laser interferometer readings (which they attribute to spacetime contractions after several years of trying) by using an EM Drive (pillbox shape, short cylinder, I think, given the reported frequency and mode shape) in TM010 resonance mode to produce the measurements.

The reason why it belongs in this thread has nothing to do with the reason you state <<the underlying principle is related (quantum vaccum, etc),>>, but instead it has everything to do with the fact that to obtain the measurements they had to use an EM Drive !

It is very significant that it is only an EM Drive (rather than the other electromagnetic devices, including capacitors, that were previously tried)  that finally produced a laser interferometer signal that they are confident to report as such.

And concerning your statement that <<both drives would work very differently... so the warp drive needs it´s own thread>> that's incorrect: the EM Drive in the laser interferometer experiment didn't work "very differently", it worked in a transverse magnetic resonant mode at 1.4 GHz frequency, so you should clarify what you mean by working very differently from the EM Drives that have been discussed in this thread.

Discussions about laser interferometer experiments with other types of devices (other than the EM Drive) belong elsewhere.
How do I infer that the EM Drive that was used in the first experiments reported to give laser interferometer readings of spacetime contractions must have been a pillbox-shaped, short cylinder ?

Based on the information given by Paul March:

Quote
TM010 RF resonant cavity ...at a 1.48 GHz rate.

As I posted previously in this thread, a truncated cone cannot have a TM010 mode because the electromagnetic fields in a truncated cone cannot be constant in the longitudinal direction of the cone (due to the geometry of the cone producing a focusing, attenuating effect in the longitudinal direction).  Therefore when somebody describes a resonant cavity having a TM010 mode it must be a cylindrical cavity.  (Or, if the analyst is describing the mode of a truncated cone analogous to a cylindrical cavity, the truncated cone must be such that the big and small bases have approximately the same diameter, because otherwise this approximate mode would be cut-off in a truncated cone).

Using the well-known equation for the frequency of a cylindrical cavity (see for example  http://en.wikipedia.org/wiki/Microwave_cavity#Cylindrical_cavity  ) , the frequency for a TM mode is:

fmnp =( c/(2 Pi  Sqrt[μr εr]) ) Sqrt[(Xmn/R)^2+(p Pi / L)^2]

for TM010 we have m=0, n=1, p=0 and therefore:

D = 2 R = ( c/(Pi  Sqrt[μr εr]) ) X01/f010

and since the speed of light in air is:

 c/(Sqrt[μr εr]) = 299705000 m/s

and the Bessel Zero 01 is:

X01 = 2.40482555769577

and the frequency, according to Paul March was:

f010 = 1.48*10^9 Hz

We conclude that the diameter of the cylindrical resonant cavity used in these experiments was:

D = (299705000 m/s)*2.40482555769577 / (Pi * 1.48*10^9 1/s) = 0.1550 m = 6.103 inches

Hence we conclude that the RF resonant cavity used for the first successful experiments reported to give laser interferometer readings of spacetime contractions must have been a cylinder having 6.103 inches diameter.

How do I infer that the cylinder must have been short, "pillbox shaped" with a length equal or smaller than its diameter ?  Because the geometry of the experimental setup previously disclosed by NASA Eagleworks does not appear to be big enough to accommodate a significantly longer cylinder:

http://upload.wikimedia.org/wikipedia/commons/9/9b/White-Juday_Warp_Field_Interferometer_Experiment.png

Perhaps Paul March could confirm what were the geometrical dimensions of the EM Drive used in these experiments,  and whether or not it had a dielectric insert and if so what type of dielectric was used.

I have to admit I have my doubts what I think is going on, is what is going on in these cavities, having only one radiation input I can't be sure.  If I were going to test them for what I am talking about I would take two cylindrical cavities with the radiation input of one cavity able to be phase shifted and amplified.  Put the two cavities flat plates next to each other so that the imaginary magnetic field (non radiating [decaying]) overlaps.  The separation would be about 1/4 lambda separation in air for the frequency chosen.  My guess is the signal of one would bleed into the other cavity which would seem to push them to be matched up in phase and not perfectly out of phase %pi/2.  They are supposed to be out of phase %pi/2 so you increase the phase and amplitude of the cavity that is working against the other till it seems they are properly out of phase 1/4 lambda with matching amplitude.  You might install a current sensor on each cavity to make sure you know the exact current phase and amplitude.  Maybe then its possible to stack the cavities on top of each other one after the other all being off in phase 0, %pi/2, %pi, 3%pi/2 ect.  I would possibly throw a dielectric between the two cavities which would change their separation thickness depending on the change in speed of light between them making the two cavities closer.  That's how I would do it if I had the ability. 
Title: Re: EM Drive Developments - related to space flight applications - Thread 2
Post by: Star-Drive on 04/19/2015 05:08 am

We should discuss these news, that NASA Eagleworks

finally observed the first spacetime contraction effects that we are fairly confident are the real deal...the laser interferometer observed spacetime contractions are being developed in a TM010 RF resonant cavity that is driving ac E-field levels over 900kV/m at a 1.48 GHz rate. 

would that be a subject to ME Drive thread?

even if the underlying principle is related (quantum vaccum, etc), still, both drives would work very differently... so the warp drive needs it´s own thread, imho.

Yes, definitely a subject for this thread: "EM Drive Developments", insofar as NASA Eagleworks reports to have laser interferometer readings (which they attribute to spacetime contractions after several years of trying) by using an EM Drive (pillbox shape, short cylinder, I think, given the reported frequency and mode shape) in TM010 resonance mode to produce the measurements.

The reason why it belongs in this thread has nothing to do with the reason you state <<the underlying principle is related (quantum vaccum, etc),>>, but instead it has everything to do with the fact that to obtain the measurements they had to use an EM Drive !

It is very significant that it is only an EM Drive (rather than the other electromagnetic devices, including capacitors, that were previously tried)  that finally produced a laser interferometer signal that they are confident to report as such.

And concerning your statement that <<both drives would work very differently... so the warp drive needs it´s own thread>> that's incorrect: the EM Drive in the laser interferometer experiment didn't work "very differently", it worked in a transverse magnetic resonant mode at 1.4 GHz frequency, so you should clarify what you mean by working very differently from the EM Drives that have been discussed in this thread.

Discussions about laser interferometer experiments with other types of devices (other than the EM Drive) belong elsewhere.
How do I infer that the EM Drive that was used in the first experiments reported to give laser interferometer readings of spacetime contractions must have been a pillbox-shaped, short cylinder ?

Based on the information given by Paul March:

Quote
TM010 RF resonant cavity ...at a 1.48 GHz rate.

As I posted previously in this thread, a truncated cone cannot have a TM010 mode because the electromagnetic fields in a truncated cone cannot be constant in the longitudinal direction of the cone (due to the geometry of the cone producing a focusing, attenuating effect in the longitudinal direction).  Therefore when somebody describes a resonant cavity having a TM010 mode it must be a cylindrical cavity.  (Or, if the analyst is describing the mode of a truncated cone analogous to a cylindrical cavity, the truncated cone must be such that the big and small bases have approximately the same diameter, because otherwise this approximate mode would be cut-off in a truncated cone).

Using the well-known equation for the frequency of a cylindrical cavity (see for example  http://en.wikipedia.org/wiki/Microwave_cavity#Cylindrical_cavity  ) , the frequency for a TM mode is:

fmnp =( c/(2 Pi  Sqrt[μr εr]) ) Sqrt[(Xmn/R)^2+(p Pi / L)^2]

for TM010 we have m=0, n=1, p=0 and therefore:

D = 2 R = ( c/(Pi  Sqrt[μr εr]) ) X01/f010

and since the speed of light in air is ( http://en.wikipedia.org/wiki/Speed_of_light ):

 c/(Sqrt[μr εr]) = 299700000 m/s

and the Bessel Zero for m=0, n=1 is (see  http://wwwal.kuicr.kyoto-u.ac.jp/www/accelerator/a4/besselroot.htmlx ):

X01 = 2.40482555769577

and the frequency, according to Paul March (see http://forum.nasaspaceflight.com/index.php?topic=36313.msg1355764#msg1355764 ) was:

f010 = 1.48*10^9 Hz

We conclude that the diameter of the cylindrical resonant cavity used in these experiments was:

D = (299700000 m/s)*2.40482555769577 / (Pi * 1.48*10^9 1/s) = 0.1550 m = 6.103 inches

Hence we conclude that the RF resonant cavity used for the first successful experiments reported to give laser interferometer readings of spacetime contractions must have been a cylinder having 6.103 inches diameter.

How do I infer that the cylinder must have been short, "pillbox shaped" with a length equal or smaller than its diameter ?  Because the geometry of the experimental setup previously disclosed by NASA Eagleworks does not appear to be big enough to accommodate a significantly longer cylinder:

http://upload.wikimedia.org/wikipedia/commons/9/9b/White-Juday_Warp_Field_Interferometer_Experiment.png

Perhaps Paul March could confirm what were the geometrical dimensions of the EM Drive used in these experiments,  and whether or not it had a dielectric insert and if so what type of dielectric was used.

Dr. Rodal:

The dimension for the 1.48 GHz TM010 pillbox cylindrical cavity used in the warp-field interferometer test are in the second presentation I sent you privately earlier this week that was authored by Frank Davies.  I'm appending the pertinent interior dimensions data from same below.

Height:   2.845”  (72.26 mm)
Diameter: 6.103”  (155.02 mm)
Material: Aluminum 6061-T4 
Conductivity: 2.265E+07

Next for ACES-High, a clarifying statement on the Q-Thrusters aka EM drives verses the Q-V based warp-drives.  They are one and the same thing except the warp drive requires a different more toroid-like topology, with much, much higher E-field intensities and perhaps somewhat higher operating frequencies as well.  So the Eagleworks lab considers our Q-thruster research a necessary pathway to the design of an operational warp drive.  The question then becomes how much energy are we going to have to switch and at what operating frequency needed to warp space-time into an Alcubierre drive like warp-bubble that is big enough to encompass our ship and that can generate a contraction in space-time that is equivalent to a 10X, 100X or even a 1,000X increase in the at rest vacuum speed of light?   

Lastly, the Eagleworks Lab's next paper on the Q-V entitled "Dynamics of the Vacuum" will be out on the NASA NTRS internet servers just any day now.  I've already provided this forum a one page abstract and introduction for this paper, but I need to agree with those that are saying that in the end analysis, the seat of all mater and space is nothing more than waves and various vortices AKA elementary particles in the Q-V.  And we also think from our ongoing work that gravity is an emergent phenomenon that is nothing more than a Q-V flow field between other Q-V entities.  So when the EM-Drive creates a thrust like phenomenon, what is really happening is that the EM-drive configuration is just setting up these Q-V flows via magneto-Hydro-Dynamics (MHD) like rules that translate into our 4D universe as space-time distortions or differential gravity gradients surrounding the drive. 

And my parting comment tonight is from the Star Trek NTG Universe: "Engage"

Best, Paul M.
Title: Re: EM Drive Developments - related to space flight applications - Thread 2
Post by: aceshigh on 04/19/2015 06:04 am
Paul, if the warp effect was only detected inside the cone topology, why is that and how Dr Sonny plans to "transplant" the effect to a torus like topology?

also, if the effect are one and the same, you mean the little thrust being detected on the ME Drive is actually caused by a small warping of space?
Title: Re: EM Drive Developments - related to space flight applications - Thread 2
Post by: CW on 04/19/2015 06:42 am

[..]

Lastly, the Eagleworks Lab's next paper on the Q-V entitled "Dynamics of the Vacuum" will be out on the NASA NTRS internet servers just any day now.  I've already provided this forum a one page abstract and introduction for this paper, but I need to agree with those that are saying that in the end analysis, the seat of all mater and space is nothing more than waves and various vortices AKA elementary particles in the Q-V.  And we also think from our ongoing work that gravity is an emergent phenomenon that is nothing more than a Q-V flow field between other Q-V entities.  So when the EM-Drive creates a thrust like phenomenon, what is really happening is that the EM-drive configuration is just setting up these Q-V flows via magneto-Hydro-Dynamics (MHD) like rules that translate into our 4D universe as space-time distortions or differential gravity gradients surrounding the drive. 

And my parting comment tonight is from the Star Trek NTG Universe: "Engage"

Best, Paul M.

When you speak of Q-V flows, can we imagine this phenomenon similar to a quantum wake that could be felt in front of the drive as a suction effect, and behind the drive as a pushing effect? Is it comparable to what a water jet engine produces in water?

Best regards
CW
Title: Re: EM Drive Developments - related to space flight applications - Thread 2
Post by: Rodal on 04/19/2015 09:13 am
CLARIFICATIONS




Paul, if the warp effect was only detected inside the cone topology, why is that ...

NOT A CONE, BUT A CYLINDER

1) as previously remarked in previous posts (TM010 modes cannot take place in a cone), and now confirmed by Paul March, the EM Drive used in the interferometer tests is not a cone.  It is a cylinder.  A cylinder that has its length equal to only about 1/2 its diameter.  It looks like a metallic round pillbox rather than looking like a cone.

(http://images.containerstore.com/catalogimages/107389/RoundPillBoxClear_xl.jpg)




Paul, if the warp effect was only detected inside ....

NOT INSIDE, BUT OUTSIDE ?

2) NASA's experiment is looking at changes of path length of the laser's beam path.  The EM Drive's are resonant closed cavities made of conducting metals (like copper or aluminum), with microwave fields inside them.  Hence, it appears to me, that the interferometer measurements are made outside (rather than inside) these closed metal cavities.

Of course, the laser and the mirror are not inside the round-pillbox-shaped EM Drive, but outside it.

Assuming that no optical windows were cut through the EM Drive, it appears to me therefore that this experimental set-up , is such that the round-pillbox-shaped EM Drive's is located so that its central axis is perpendicular to the line of sight between the laser and the reference mirror, out of the laser's line of sight.  And such that the line of sight between the laser and the mirror are optically uninterrupted by the EM Drive's closed metal cavity.


It would be helpful if Paul March could confirm this understanding.

EDIT: Paul March confirmed the opposite: that they did cutout holes through the pillbox resonant cavity for the laser to go through on its path from the second mirror (see post by Paul March below)






A minor point, only made because of the recurring typo ?  :)

you mean the little thrust being detected on the ME Drive
.. inside an ME Drive is subject to this thread. I agree.....
if we try to understand ME Drive ..
...
would that be a subject to ME Drive thread?..

NOT ME DRIVE, BUT EM DRIVE

3) We are discussing the EM Drive, where "EM" stands for "ElectroMagnetic", referring to the microwave electromagnetic fields inside these resonant cavities.
Title: Re: EM Drive Developments - related to space flight applications - Thread 2
Post by: MichaelBlackbourn on 04/19/2015 02:38 pm
Yah, the EM and ME thing is important. ME generally refers to mach effect (Woodward)   mass fluctuation thrusters.
Title: Re: EM Drive Developments - related to space flight applications - Thread 2
Post by: eischei on 04/19/2015 03:26 pm
Just thought a little down the road:

Resonant chambers are used to induce plasma discharges in an decreased pressure environment but discharges are also possible as parasitic phenomenons in highpower waveguides and called arcs.

Would it be possible to use an other dielectric medium than air? Because otherwise you maybe would have to cool the inner gas, or have to used a filled / completely evacuated chamber. And although the latter shouldn't be a problem in space it would be more convinient to have a device that isn't dependent on spaceconditions.

If for example aluminum nitride could be used, there could be also the advantage of the high heatconductivity and thermal stability of the whole system as well as the low thermal elongation factor. The surface could be coated with any metal that is best for the electrical conductivity by PVD.

Regards,

Sepp
Title: Re: EM Drive Developments - related to space flight applications - Thread 2
Post by: Star-Drive on 04/19/2015 04:44 pm
Dr. Rodal:

I missed your question last night on whether the warp-field interferometer cylindrical cavity had any dielectric inside of it.  The answer is no it does not, except for the nanometers thick aluminum oxide coating that bare aluminum develops as soon as it is exposed to the oxygen in the air.

Next you asked about whether there where optical windows cut into the center of the cylindrical resonant cavity end caps or not.  Well, yes there has to be optical window holes for the 633nm laser light to pass through the 7.23cm gap between the endplates of the aluminum cylindrical cavity.  We also added two, three inch long, 0.50" OD by 0.25" (6.35mm) ID threaded aluminum tubes to the resonant cavity endplates, see attached picture, that function as two RF chokes that keep the 1.48 GHz RF from leaking into the lab area.  So the laser light passes through these RF choke tubes and the cylindrical cavity where the peak ac E-field of 900kV/m is present along the entire 7.23 cm long laser path while in the resonant cavity and an exponentially reducing E-field in the RF chokes since these are cylindrical waveguides well into their cutoff mode since the RF wavelength at 1.48 GHz is 202.7mm. 

BTW, we are going to add optical borosilicate telescope grade flat windows to the ends of the RF chokes when we get around to pulling a vacuum in this 1.48 GHz resonant cavity.

Next a clarification.  We used a cylindrical cavity for the warp-field interferometer instead of a frustum shape because we didn't want to create a force with this unit, but instead we needed just a large densification of the Q-V along the active path length of the laser beam while it was traversing the resonant cavity's centerline volume.  And this is the main difference between the Q-thruster and a warp-drive.  In Dr. White's warp-field conjecture you first have to have an Q-Thruster derived acceleration vector to work on and then you engage the a toroidal warp-field around your accelerating vehicle that then multiples the initial Q-Thruster provided velocity vector by the selected warp-factor.   Thus if you have an initial velocity of say 0.01c towards Alpha Centauri with a warp factor of 1,000, your effective velocity becomes 10c while the warp-drive is engaged.

Best, Paul M.
Title: Re: EM Drive Developments - related to space flight applications - Thread 2
Post by: CW on 04/19/2015 05:10 pm
Dr. Rodal:

I missed your question last night on whether the warp-field interferometer cylindrical cavity had any dielectric inside of it.  The answer is no it does not, except for the nanometers thick aluminum oxide coating that bare aluminum develops as soon as it is exposed to the oxygen in the air.

Next you asked about whether there where optical windows cut into the center of the cylindrical resonant cavity end caps or not.  Well, yes there has to be optical window holes for the 633nm laser light to pass through the 7.23cm gap between the endplates of the aluminum cylindrical cavity.  We also added two, three inch long, 0.50" OD by 0.25" (6.35mm) ID threaded aluminum tubes to the resonant cavity endplates, see attached picture, that function as two RF chokes that keep the 1.48 GHz RF from leaking into the lab area.  So the laser light passes through these RF choke tubes and the cylindrical cavity where the peak ac E-field of 900kV/m is present along the entire 7.23 cm long laser path while in the resonant cavity and an exponentially reducing E-field in the RF chokes since these are cylindrical waveguides well into their cutoff mode since the RF wavelength at 1.48 GHz is 202.7mm. 

BTW, we are going to add optical borosilicate telescope grade flat windows to the ends of the RF chokes when we get around to pulling a vacuum in this 1.48 GHz resonant cavity.

Next a clarification.  We used a cylindrical cavity for the warp-field interferometer instead of a frustum shape because we didn't want to create a force with this unit, but instead we needed just a large densification of the Q-V along the active path length of the laser beam while it was traversing the resonant cavity's centerline volume.  And this is the main difference between the Q-thruster and a warp-drive.  In Dr. White's warp-field conjecture you first have to have an Q-Thruster derived acceleration vector to work on and then you engage the a toroidal warp-field around your accelerating vehicle that then multiples the initial Q-Thruster provided velocity vector by the selected warp-factor.   Thus if you have an initial velocity of say 0.01c towards Alpha Centauri with a warp factor of 1,000, your effective velocity becomes 10c while the warp-drive is engaged.

Best, Paul M.

Could a warp field be employed in Earth's atmosphere, for instance for an airplane? Or would the warp create problems in this gaseous environment?
Title: Re: EM Drive Developments - related to space flight applications - Thread 2
Post by: Rodal on 04/19/2015 06:26 pm
....
Next you asked about whether there where optical windows cut into the center of the cylindrical resonant cavity end caps or not.  Well, yes there has to be optical window holes for the 633nm laser light to pass through the 7.23cm gap between the endplates of the aluminum cylindrical cavity.  We also added two, three inch long, 0.50" OD by 0.25" (6.35mm) ID threaded aluminum tubes to the resonant cavity endplates, see attached picture, that function as two RF chokes that keep the 1.48 GHz RF from leaking into the lab area.  So the laser light passes through these RF choke tubes and the cylindrical cavity where the peak ac E-field of 900kV/m is present along the entire 7.23 cm long laser path while in the resonant cavity and an exponentially reducing E-field in the RF chokes since these are cylindrical waveguides well into their cutoff mode since the RF wavelength at 1.48 GHz is 202.7mm. 

....
Best, Paul M.

Thanks, great information and clarification.

Just curious as to whether the COMSOL FEA analyst included the optical window cutout holes for the 633nm laser light to pass through the 7.23cm gap between the endplates of the aluminum cylindrical cavity, into the Finite Element Mesh in his FEA analysis?

It is comforting (see my post http://forum.nasaspaceflight.com/index.php?topic=36313.msg1361552#msg1361552 ) that there is absolutely no difference between the natural frequency calculated by the COMSOL FEA analyst and the frequency I obtained from the exact solution based on a resonant cavity with no holes whatsoever. This is to be expected (that the holes make no difference) because the size of the cutout holes only affects much shorter wavelengths than the resonant wavelength (202.5 mm = 7.97 inches) associated with the natural frequency for TM010 of 1.48 GHz.  Or, equivalently, this can be stated as saying that the holes only affect much higher natural frequencies than 1.48 GHz.


Readers: notice emphasis on the following:

Quote
the laser light passes through these RF choke tubes and the cylindrical cavity where the peak ac E-field of 900kV/m is present along the entire 7.23 cm long laser path while in the resonant cavity and an exponentially reducing E-field in the RF chokes since these are cylindrical waveguides well into their cutoff mode since the RF wavelength at 1.48 GHz is 202.7mm. 


PS: the insignificant discrepancy on the calculated wavelength (202.7 mm vs 202.5 mm) is due to the fact that I used the speed of light in air and it appears that Paul calculated it based on the speed of light in a vacuum

Thanks once again !
Title: Re: EM Drive Developments - related to space flight applications - Thread 2
Post by: aceshigh on 04/19/2015 06:34 pm
Yah, the EM and ME thing is important. ME generally refers to mach effect (Woodward)   mass fluctuation thrusters.

sorry, I know that and got confused because I had just read a topic somewhere else that I thought it was about EM and it was ME, then ME was stuck in my head when I made the post.
Title: Re: EM Drive Developments - related to space flight applications - Thread 2
Post by: Mulletron on 04/19/2015 08:05 pm
FYI:  one of many...about nonlinear surface, etc.

http://www.lajpe.org/sep12/14_LAJPE_687_Gouri_Sankar_preprint_corr_f.pdf

Great find and possibly a significant lead. Here's more info to support the NLO properties of cuprous oxide Cu2O. And all you have to do to get cuprous oxide is to just have copper sitting in air. The frustum I'm playing around with has certainly darkened since I got it.
http://www.materialsviews.com/cuprous-oxide-a-new-super-material/
http://en.wikipedia.org/wiki/Copper_oxide

I know a super easy way to test this. IF I get any thrust from the unloaded cavity, what I can do to test whether cuprous oxide plays a part in it is to simply clean the cavity. If you boil copper in a salt water/vinegar solution, it'll remove the oxidation. The unloaded cavity shouldn't thrust if the above about cuprous oxide is true.
http://www.wikihow.com/Clean-Copper

Anyway, I have a pretty full plate right now trying to integrate all the electronics and the frustum safely on the balance.
https://drive.google.com/folderview?id=0B4PCfHCM1KYoTl90eDBuMklOeTg&amp;usp=sharing&tid=0B4PCfHCM1KYoTXhSUTd5ZDN2WnM

When that is done, I am absolutely going back for a repeat of the frequency mixing I saw evidence of and discussed here:
http://forum.nasaspaceflight.com/index.php?topic=36313.msg1353384#msg1353384
and here: http://forum.nasaspaceflight.com/index.php?topic=36313.msg1358851#msg1358851
and do some quantitative measurements.



Title: Re: EM Drive Developments - related to space flight applications - Thread 2
Post by: Mulletron on 04/19/2015 08:08 pm
Dr. Rodal:

I missed your question last night on whether the warp-field interferometer cylindrical cavity had any dielectric inside of it.  The answer is no it does not, except for the nanometers thick aluminum oxide coating that bare aluminum develops as soon as it is exposed to the oxygen in the air.

Next you asked about whether there where optical windows cut into the center of the cylindrical resonant cavity end caps or not.  Well, yes there has to be optical window holes for the 633nm laser light to pass through the 7.23cm gap between the endplates of the aluminum cylindrical cavity.  We also added two, three inch long, 0.50" OD by 0.25" (6.35mm) ID threaded aluminum tubes to the resonant cavity endplates, see attached picture, that function as two RF chokes that keep the 1.48 GHz RF from leaking into the lab area.  So the laser light passes through these RF choke tubes and the cylindrical cavity where the peak ac E-field of 900kV/m is present along the entire 7.23 cm long laser path while in the resonant cavity and an exponentially reducing E-field in the RF chokes since these are cylindrical waveguides well into their cutoff mode since the RF wavelength at 1.48 GHz is 202.7mm. 

BTW, we are going to add optical borosilicate telescope grade flat windows to the ends of the RF chokes when we get around to pulling a vacuum in this 1.48 GHz resonant cavity.

Next a clarification.  We used a cylindrical cavity for the warp-field interferometer instead of a frustum shape because we didn't want to create a force with this unit, but instead we needed just a large densification of the Q-V along the active path length of the laser beam while it was traversing the resonant cavity's centerline volume.  And this is the main difference between the Q-thruster and a warp-drive.  In Dr. White's warp-field conjecture you first have to have an Q-Thruster derived acceleration vector to work on and then you engage the a toroidal warp-field around your accelerating vehicle that then multiples the initial Q-Thruster provided velocity vector by the selected warp-factor.   Thus if you have an initial velocity of say 0.01c towards Alpha Centauri with a warp factor of 1,000, your effective velocity becomes 10c while the warp-drive is engaged.

Best, Paul M.

Paul March, have you seen this?

http://arxiv.org/abs/1101.0712
http://arxiv.org/abs/1101.1174
http://www2.cnrs.fr/en/1859.htm
http://phys.org/news/2011-05-when-the-speed-of-light.html
Title: Re: EM Drive Developments - related to space flight applications - Thread 2
Post by: Rodal on 04/19/2015 08:14 pm
Dr. Rodal:

I missed your question last night on whether the warp-field interferometer cylindrical cavity had any dielectric inside of it.  The answer is no it does not, except for the nanometers thick aluminum oxide coating that bare aluminum develops as soon as it is exposed to the oxygen in the air.

Next you asked about whether there where optical windows cut into the center of the cylindrical resonant cavity end caps or not.  Well, yes there has to be optical window holes for the 633nm laser light to pass through the 7.23cm gap between the endplates of the aluminum cylindrical cavity.  We also added two, three inch long, 0.50" OD by 0.25" (6.35mm) ID threaded aluminum tubes to the resonant cavity endplates, see attached picture, that function as two RF chokes that keep the 1.48 GHz RF from leaking into the lab area.  So the laser light passes through these RF choke tubes and the cylindrical cavity where the peak ac E-field of 900kV/m is present along the entire 7.23 cm long laser path while in the resonant cavity and an exponentially reducing E-field in the RF chokes since these are cylindrical waveguides well into their cutoff mode since the RF wavelength at 1.48 GHz is 202.7mm. 

BTW, we are going to add optical borosilicate telescope grade flat windows to the ends of the RF chokes when we get around to pulling a vacuum in this 1.48 GHz resonant cavity.

Next a clarification.  We used a cylindrical cavity for the warp-field interferometer instead of a frustum shape because we didn't want to create a force with this unit, but instead we needed just a large densification of the Q-V along the active path length of the laser beam while it was traversing the resonant cavity's centerline volume.  And this is the main difference between the Q-thruster and a warp-drive.  In Dr. White's warp-field conjecture you first have to have an Q-Thruster derived acceleration vector to work on and then you engage the a toroidal warp-field around your accelerating vehicle that then multiples the initial Q-Thruster provided velocity vector by the selected warp-factor.   Thus if you have an initial velocity of say 0.01c towards Alpha Centauri with a warp factor of 1,000, your effective velocity becomes 10c while the warp-drive is engaged.

Best, Paul M.

Paul,

Thanks once again for posting this great information !

QUESTION 1: Is it correct to assume that the assessment of the interferometer path-length-change measurements was accomplished by looking at the Power Spectral Density at an anomalous frequency high enough away from the pink noise area (system 1/f noise, quantum 1/f noise etc.), and so clearly distinguishable from system noise occurring at frequencies close to zero?

QUESTION 2: If so, did the observed anomalous peak in the Power Spectral Density occur at a frequency in accordance with the time taken to energize  and de-energize?

QUESTION 3: Did you plot three dimensional plots to look for power peak distribution distributions looking like ring-shaped circular-waves, corresponding to path length changes associated with such frequency (in question2) ?

(http://i.stack.imgur.com/fWFL6.jpg)

QUESTION 4: Did you conduct additional tests to confirm repeatibility of the measurements?

QUESTION 5: One would expect such ring-waves to display some statistical distribution, therefore using measures of central tendency like different truncated mean measures ( http://en.wikipedia.org/wiki/Truncated_mean ) of the multidimensional power spectral density data may be particularly helpful in assessing the data (at least I have found so in assessing massive data for different problems that also involve 1/f noise)

QUESTION 6: Has NASA Eagleworks addressed the issue with air refraction raised in this paper by Lee and Cleaver from Baylor University?:

http://arxiv.org/ftp/arxiv/papers/1407/1407.7772.pdf

In particular, has NASA Eagleworks assessed the likelihood of the path-length-change measurements being the result of transient air heating ?
Title: Re: EM Drive Developments - related to space flight applications - Thread 2
Post by: Mulletron on 04/19/2015 08:23 pm
@Notsosureofit @DIYFAN, others....are you guys building experiments?
Title: Re: EM Drive Developments - related to space flight applications - Thread 2
Post by: Rodal on 04/19/2015 08:57 pm
...

http://arxiv.org/abs/1101.0712
http://arxiv.org/abs/1101.1174
http://www2.cnrs.fr/en/1859.htm
http://phys.org/news/2011-05-when-the-speed-of-light.html

Good papers.  Thanks for posting them
Title: Re: EM Drive Developments - related to space flight applications - Thread 2
Post by: Star-Drive on 04/19/2015 09:43 pm
Dr. Rodal:

I missed your question last night on whether the warp-field interferometer cylindrical cavity had any dielectric inside of it.  The answer is no it does not, except for the nanometers thick aluminum oxide coating that bare aluminum develops as soon as it is exposed to the oxygen in the air.

Next you asked about whether there where optical windows cut into the center of the cylindrical resonant cavity end caps or not.  Well, yes there has to be optical window holes for the 633nm laser light to pass through the 7.23cm gap between the endplates of the aluminum cylindrical cavity.  We also added two, three inch long, 0.50" OD by 0.25" (6.35mm) ID threaded aluminum tubes to the resonant cavity endplates, see attached picture, that function as two RF chokes that keep the 1.48 GHz RF from leaking into the lab area.  So the laser light passes through these RF choke tubes and the cylindrical cavity where the peak ac E-field of 900kV/m is present along the entire 7.23 cm long laser path while in the resonant cavity and an exponentially reducing E-field in the RF chokes since these are cylindrical waveguides well into their cutoff mode since the RF wavelength at 1.48 GHz is 202.7mm. 

BTW, we are going to add optical borosilicate telescope grade flat windows to the ends of the RF chokes when we get around to pulling a vacuum in this 1.48 GHz resonant cavity.

Next a clarification.  We used a cylindrical cavity for the warp-field interferometer instead of a frustum shape because we didn't want to create a force with this unit, but instead we needed just a large densification of the Q-V along the active path length of the laser beam while it was traversing the resonant cavity's centerline volume.  And this is the main difference between the Q-thruster and a warp-drive.  In Dr. White's warp-field conjecture you first have to have an Q-Thruster derived acceleration vector to work on and then you engage the a toroidal warp-field around your accelerating vehicle that then multiples the initial Q-Thruster provided velocity vector by the selected warp-factor.   Thus if you have an initial velocity of say 0.01c towards Alpha Centauri with a warp factor of 1,000, your effective velocity becomes 10c while the warp-drive is engaged.

Best, Paul M.

Paul,

Thanks once again for posting this great information !

QUESTION 1: Is it correct to assume that the assessment of the interferometer path-length-change measurements was accomplished by looking at the Power Spectral Density at an anomalous frequency high enough away from the pink noise area (system 1/f noise, quantum 1/f noise etc.), and so clearly distinguishable from system noise occurring at frequencies close to zero?

QUESTION 2: If so, did the observed anomalous peak in the Power Spectral Density occur at a frequency in accordance with the time taken to energize  and de-energize?

QUESTION 3: Did you plot three dimensional plots to look for power peak distribution distributions looking like ring-shaped circular-waves, corresponding to path length changes associated with such frequency (in question2) ?

(http://i.stack.imgur.com/fWFL6.jpg)

QUESTION 4: Did you conduct additional tests to confirm repeatibility of the measurements?

QUESTION 5: One would expect such ring-waves to display some statistical distribution, therefore using measures of central tendency like different truncated mean measures ( http://en.wikipedia.org/wiki/Truncated_mean ) of the multidimensional power spectral density data may be particularly helpful in assessing the data (at least I have found so in assessing massive data for different problems that also involve 1/f noise)

QUESTION 6: Has NASA Eagleworks addressed the issue with air refraction raised in this paper by Lee and Cleaver from Baylor University?:

http://arxiv.org/ftp/arxiv/papers/1407/1407.7772.pdf

In particular, has NASA Eagleworks assessed the likelihood of the path-length-change measurements being the result of transient air heating ?

Dr. Rodal:

QUESTION 1: Is it correct to assume that the assessment of the interferometer path-length-change measurements was accomplished by looking at the Power Spectral Density at an anomalous frequency high enough away from the pink noise area (system 1/f noise, quantum 1/f noise etc.), and so clearly distinguishable from system noise occurring at frequencies close to zero?

Yes it is for its around 0.660 seconds

QUESTION 2: If so, did the observed anomalous peak in the Power Spectral Density occur at a frequency in accordance with the time taken to energize  and de-energize?

Yes, the on/off cycle time was around 1.5 seconds with some uncertainty due to Windows 7.0 time outs.  Need a real time operating system (RTOto clear that problem, a RTOS system we don't have.

QUESTION 3: Did you plot three dimensional plots to look for power peak distribution distributions looking like ring-shaped circular-waves, corresponding to path length changes associated with such frequency (in question2) ?

(http://i.stack.imgur.com/fWFL6.jpg)

Yes, see attached picture.

QUESTION 4: Did you conduct additional tests to confirm repeatibility of the measurements?

Yes Michael Rollins performed four additional 27,000 on/off data sets under the same 30W RF drive condition and obtained similar test results for all five cases.  Mind you at 20W RF input there was only a hint of the space-time compression effect visible above the noise platform.   

QUESTION 5: One would expect such ring-waves to display some statistical distribution, therefore using measures of central tendency like different truncated mean measures ( http://en.wikipedia.org/wiki/Truncated_mean ) of the multidimensional power spectral density data may be particularly helpful in assessing the data (at least I have found so in assessing massive data for different problems that also involve 1/f noise)

I will point that out to Dr. White tomorrow.

QUESTION 6: Has NASA Eagleworks addressed the issue with air refraction raised in this paper by Lee and Cleaver from Baylor University?:

http://arxiv.org/ftp/arxiv/papers/1407/1407.7772.pdf

In particular, has NASA Eagleworks assessed the likelihood of the path-length-change measurements being the result of transient air heating ?

See Dr. White's preliminary assessment of that issue in the attached slide.  Ultimately though we will be running the warp-field resonant cavity with a vacuum contained in its active volume to get rid of all possibilities of air heating problems.

Best, Paul M.
Title: Re: EM Drive Developments - related to space flight applications - Thread 2
Post by: Rodal on 04/19/2015 09:53 pm
Dr. Rodal:

I missed your question last night on whether the warp-field interferometer cylindrical cavity had any dielectric inside of it.  The answer is no it does not, except for the nanometers thick aluminum oxide coating that bare aluminum develops as soon as it is exposed to the oxygen in the air.

Next you asked about whether there where optical windows cut into the center of the cylindrical resonant cavity end caps or not.  Well, yes there has to be optical window holes for the 633nm laser light to pass through the 7.23cm gap between the endplates of the aluminum cylindrical cavity.  We also added two, three inch long, 0.50" OD by 0.25" (6.35mm) ID threaded aluminum tubes to the resonant cavity endplates, see attached picture, that function as two RF chokes that keep the 1.48 GHz RF from leaking into the lab area.  So the laser light passes through these RF choke tubes and the cylindrical cavity where the peak ac E-field of 900kV/m is present along the entire 7.23 cm long laser path while in the resonant cavity and an exponentially reducing E-field in the RF chokes since these are cylindrical waveguides well into their cutoff mode since the RF wavelength at 1.48 GHz is 202.7mm. 

BTW, we are going to add optical borosilicate telescope grade flat windows to the ends of the RF chokes when we get around to pulling a vacuum in this 1.48 GHz resonant cavity.

Next a clarification.  We used a cylindrical cavity for the warp-field interferometer instead of a frustum shape because we didn't want to create a force with this unit, but instead we needed just a large densification of the Q-V along the active path length of the laser beam while it was traversing the resonant cavity's centerline volume.  And this is the main difference between the Q-thruster and a warp-drive.  In Dr. White's warp-field conjecture you first have to have an Q-Thruster derived acceleration vector to work on and then you engage the a toroidal warp-field around your accelerating vehicle that then multiples the initial Q-Thruster provided velocity vector by the selected warp-factor.   Thus if you have an initial velocity of say 0.01c towards Alpha Centauri with a warp factor of 1,000, your effective velocity becomes 10c while the warp-drive is engaged.

Best, Paul M.

Paul,

Thanks once again for posting this great information !

QUESTION 1: Is it correct to assume that the assessment of the interferometer path-length-change measurements was accomplished by looking at the Power Spectral Density at an anomalous frequency high enough away from the pink noise area (system 1/f noise, quantum 1/f noise etc.), and so clearly distinguishable from system noise occurring at frequencies close to zero?

QUESTION 2: If so, did the observed anomalous peak in the Power Spectral Density occur at a frequency in accordance with the time taken to energize  and de-energize?

QUESTION 3: Did you plot three dimensional plots to look for power peak distribution distributions looking like ring-shaped circular-waves, corresponding to path length changes associated with such frequency (in question2) ?

(http://i.stack.imgur.com/fWFL6.jpg)

QUESTION 4: Did you conduct additional tests to confirm repeatibility of the measurements?

QUESTION 5: One would expect such ring-waves to display some statistical distribution, therefore using measures of central tendency like different truncated mean measures ( http://en.wikipedia.org/wiki/Truncated_mean ) of the multidimensional power spectral density data may be particularly helpful in assessing the data (at least I have found so in assessing massive data for different problems that also involve 1/f noise)

QUESTION 6: Has NASA Eagleworks addressed the issue with air refraction raised in this paper by Lee and Cleaver from Baylor University?:

http://arxiv.org/ftp/arxiv/papers/1407/1407.7772.pdf

In particular, has NASA Eagleworks assessed the likelihood of the path-length-change measurements being the result of transient air heating ?

Dr. Rodal:

QUESTION 1: Is it correct to assume that the assessment of the interferometer path-length-change measurements was accomplished by looking at the Power Spectral Density at an anomalous frequency high enough away from the pink noise area (system 1/f noise, quantum 1/f noise etc.), and so clearly distinguishable from system noise occurring at frequencies close to zero?

Yes it is for its around 0.660 seconds

QUESTION 2: If so, did the observed anomalous peak in the Power Spectral Density occur at a frequency in accordance with the time taken to energize  and de-energize?

Yes, the on/off cycle time was around 1.5 seconds with some uncertainty due to Windows 7.0 time outs.  Need a real time operating system (RTOto clear that problem, a RTOS system we don't have.

QUESTION 3: Did you plot three dimensional plots to look for power peak distribution distributions looking like ring-shaped circular-waves, corresponding to path length changes associated with such frequency (in question2) ?

(http://i.stack.imgur.com/fWFL6.jpg)

Yes, see attached picture.

QUESTION 4: Did you conduct additional tests to confirm repeatibility of the measurements?

Yes Michael Rollins performed four additional 27,000 on/off data sets under the same 30W RF drive condition and obtained similar test results for all five cases.  Mind you at 20W RF input there was only a hint of the space-time compression effect visible above the noise platform.   

QUESTION 5: One would expect such ring-waves to display some statistical distribution, therefore using measures of central tendency like different truncated mean measures ( http://en.wikipedia.org/wiki/Truncated_mean ) of the multidimensional power spectral density data may be particularly helpful in assessing the data (at least I have found so in assessing massive data for different problems that also involve 1/f noise)

I will point that out to Dr. White tomorrow.

QUESTION 6: Has NASA Eagleworks addressed the issue with air refraction raised in this paper by Lee and Cleaver from Baylor University?:

http://arxiv.org/ftp/arxiv/papers/1407/1407.7772.pdf

In particular, has NASA Eagleworks assessed the likelihood of the path-length-change measurements being the result of transient air heating ?

See Dr. White's preliminary assessment of that issue in the attached slide.  Ultimately though we will be running the warp-field resonant cavity with a vacuum contained in its active volume to get rid of all possibilities of air heating problems.

Best, Paul M.

Thanks.

It is undeniable that NASA Eagleworks does a very professional job !

Outstanding answers !
Title: Re: EM Drive Developments - related to space flight applications - Thread 2
Post by: Star-Drive on 04/19/2015 10:00 pm
Dr. Rodal:

I missed your question last night on whether the warp-field interferometer cylindrical cavity had any dielectric inside of it.  The answer is no it does not, except for the nanometers thick aluminum oxide coating that bare aluminum develops as soon as it is exposed to the oxygen in the air.

Next you asked about whether there where optical windows cut into the center of the cylindrical resonant cavity end caps or not.  Well, yes there has to be optical window holes for the 633nm laser light to pass through the 7.23cm gap between the endplates of the aluminum cylindrical cavity.  We also added two, three inch long, 0.50" OD by 0.25" (6.35mm) ID threaded aluminum tubes to the resonant cavity endplates, see attached picture, that function as two RF chokes that keep the 1.48 GHz RF from leaking into the lab area.  So the laser light passes through these RF choke tubes and the cylindrical cavity where the peak ac E-field of 900kV/m is present along the entire 7.23 cm long laser path while in the resonant cavity and an exponentially reducing E-field in the RF chokes since these are cylindrical waveguides well into their cutoff mode since the RF wavelength at 1.48 GHz is 202.7mm. 

BTW, we are going to add optical borosilicate telescope grade flat windows to the ends of the RF chokes when we get around to pulling a vacuum in this 1.48 GHz resonant cavity.

Next a clarification.  We used a cylindrical cavity for the warp-field interferometer instead of a frustum shape because we didn't want to create a force with this unit, but instead we needed just a large densification of the Q-V along the active path length of the laser beam while it was traversing the resonant cavity's centerline volume.  And this is the main difference between the Q-thruster and a warp-drive.  In Dr. White's warp-field conjecture you first have to have an Q-Thruster derived acceleration vector to work on and then you engage the a toroidal warp-field around your accelerating vehicle that then multiples the initial Q-Thruster provided velocity vector by the selected warp-factor.   Thus if you have an initial velocity of say 0.01c towards Alpha Centauri with a warp factor of 1,000, your effective velocity becomes 10c while the warp-drive is engaged.

Best, Paul M.

Paul March, have you seen this?

http://arxiv.org/abs/1101.0712
http://arxiv.org/abs/1101.1174
http://www2.cnrs.fr/en/1859.htm
http://phys.org/news/2011-05-when-the-speed-of-light.html

Mulletron:

No I hadn't but thanks for the pointers.  So what to you think an asymmetric difference of ~1x10^-18 m/s in velocity of light bring to the table?  That the vacuum can be differentially polarized by applied E and B-fields in a volume, in this case dc E&M fields??

Best, Paul M.

Title: Re: EM Drive Developments - related to space flight applications - Thread 2
Post by: Mulletron on 04/19/2015 10:15 pm


Mulletron:

No I hadn't but thanks for the pointers.  So what to you think an asymmetric difference of ~1x10^-18 m/s in velocity of light bring to the table?  That the vacuum can be differentially polarized by applied E and B-fields in a volume, in this case dc E&M fields??

Best, Paul M.

Their results seem to support what you guys are reporting from your open air experiments, which is a win, but I don't think we can call this length contraction (even though it might look like it) for sure until the same results are in repeated in vacuum.
Title: Re: EM Drive Developments - related to space flight applications - Thread 2
Post by: aceshigh on 04/20/2015 01:22 am
@Paul March: since the warp drive is to be considered then as ontopic in this thread, how exactly does Dr. White theories deal with time-travel to the past in superluminal speeds? I guess that would be a major point of any space travel related applications of a warp drive.

I remember that in Starship Congress 2013 (at which Dr. White made the first talk on Day 3 – Interstellar Future (50 years +) | Saturday August 17th, 2013 - Sonny White, “Warp Field Physics: an Update” ), the talk just after Dr. White's, was by Dr Eric Davis, that was quite mind bending, where he talks about tipping the light cone in Warp Drives (I guess that would be space-time engineering just like the Warp Drive itself?) so inside it's light cone the ship is not travelling to the past and thus there is no worry of causality violations.

Is that a view that Dr White (or yourself) agree with? Or are you sure a Warp Drive will certainly result in travel to the past?

To anyone wanting to see Dr Eric Davis talk, here is the video of the full Day 3... Dr Eric Davis talk starts at 58:00.
https://www.youtube.com/watch?v=ucyBMB_PWr8

I guess it´s on-topic because it´s intimally related to the Warp Drive by Dr White.

the topic is "light cone gymnastics"  (really) ;D

Title: Re: EM Drive Developments - related to space flight applications - Thread 2
Post by: Mulletron on 04/20/2015 02:24 am
@Paul March: since the warp drive is to be considered then as ontopic in this thread, how exactly does Dr. White theories deal with time-travel to the past in superluminal speeds? I guess that would be a major point of any space travel related applications of a warp drive.



Well as far as I'm concerned, as long as the warp interferometry experiments feature an RF resonant cavity (see pic below), and data from those experiments is giving us insight about what is happening inside the Emdrive (anisotropies in the speed of light for example), it is certainly on topic here. If they report those same anisotropies in vacuum, warp drive is born.

(http://forum.nasaspaceflight.com/index.php?action=dlattach;topic=36313.0;attach=825512;image)
Title: Re: EM Drive Developments - related to space flight applications - Thread 2
Post by: aceshigh on 04/20/2015 02:43 am
I know Mulletron. What I want to know is exactly how Dr White theory deals with superluminal speeds, which most physicists say leads to time-travel to the past and all the paradoxes that surface from that.


According to the video I showed from Dr Davis, superluminal speeds WITHOUT time-travel to the past are possible, if the light cone is tilted from 0 to 90 degrees only...


I think this question is related to spaceflight applications exactly because time travel IS an issue at relativistic velocities (to the future) and superluminal velocities (to the past, but not according to Dr Davis)

This question is probably more related to the spaceflight applications of a warp drive than the pure theoretical issues of how EM and Warp Drives work on quantum level, since the first is a result of spaceflight application while the second (which is being discussed in this thread) is not.
Title: Re: EM Drive Developments - related to space flight applications - Thread 2
Post by: tchernik on 04/20/2015 03:25 am
Nice & interesting update, as always.

I'm not sure why, but somehow I think a repeatable interference pattern in a vacuum coming from a resonating cavity would attract more scientific attention than a micro-Newton thruster, as things stand now.

Maybe because micro-thrust can be attributed to experimental error, while a clear interference pattern of the expected characteristics would be harder to explain, and could make others replicate the experiment.

Or maybe because it has already received some attention (and refutation) in some paper cited here. Having a counter-refutation could make many people turn their eyes to this.

A strong thrust on the higher power Emdrive on other hand...

Yep, it really seems things are approaching the point of breaking through or breaking down. Hopefully the former.
Title: Re: EM Drive Developments - related to space flight applications - Thread 2
Post by: Left Field on 04/20/2015 07:03 am
I know Mulletron. What I want to know is exactly how Dr White theory deals with superluminal speeds, which most physicists say leads to time-travel to the past and all the paradoxes that surface from that.


According to the video I showed from Dr Davis, superluminal speeds WITHOUT time-travel to the past are possible, if the light cone is tilted from 0 to 90 degrees only...


I think this question is related to spaceflight applications exactly because time travel IS an issue at relativistic velocities (to the future) and superluminal velocities (to the past, but not according to Dr Davis)

This question is probably more related to the spaceflight applications of a warp drive than the pure theoretical issues of how EM and Warp Drives work on quantum level, since the first is a result of spaceflight application while the second (which is being discussed in this thread) is not.

Don't forget that the ship is not really moving at relativistic speeds: space is.  Consequently, you could take a trip to Alpha Centauri in 2 days (or less with more power... who knows?), turn your ship around and observe the Earth as it was four years ago (as light has taken four years to get there - slow coach!). You could then observe Alpha Centauri as it is "now", and how people on the Earth will see it in four years.

With this type of technology,  it would be possible to predict when locally past events are going to be observable from the point of view of the Earth (or any other point that the light from such events had not yet reached). For example, a ship 1 light-day out from the Earth in the right place could witness a supernova before the Earth does and then be able to return to the Earth almost instantly and tell astronomers about the incoming light wave so that they could prepare to observe it.

Proviso: I am not an expert in time travel and I also have doubts about Dr Who.
Title: Re: EM Drive Developments - related to space flight applications - Thread 2
Post by: Star-Drive on 04/20/2015 12:39 pm
I know Mulletron. What I want to know is exactly how Dr White theory deals with superluminal speeds, which most physicists say leads to time-travel to the past and all the paradoxes that surface from that.


According to the video I showed from Dr Davis, superluminal speeds WITHOUT time-travel to the past are possible, if the light cone is tilted from 0 to 90 degrees only...


I think this question is related to spaceflight applications exactly because time travel IS an issue at relativistic velocities (to the future) and superluminal velocities (to the past, but not according to Dr Davis)

This question is probably more related to the spaceflight applications of a warp drive than the pure theoretical issues of how EM and Warp Drives work on quantum level, since the first is a result of spaceflight application while the second (which is being discussed in this thread) is not.

Aceshigh:

To be honest, I've not heard Dr. White talk or write about the time travel aspects of his warp-field conjecture.  I think that may be because Sonny is fairly conservative on some topics in this business, especially since his NASA management is even more super conservative in regards to anything that smacks of Sci-Fi like time travel.  Yea I know, its a tribute to Sonny's salesmanship that he has convinced his NASA/EP managers to even allow his current low cost investigations into Q-Thrusters and warp-drives based on his Q-V conjecture, when their normal response is to consider even delving into LOX/CH4 chemical rocket technology as daring...

Best,  Paul M.
Title: Re: EM Drive Developments - related to space flight applications - Thread 2
Post by: Rodal on 04/20/2015 12:44 pm
http://en.wikipedia.org/wiki/Novikov_self-consistency_principle



http://www.scottaaronson.com/papers/ctc.pdf

Closed Timelike Curves Make Quantum and Classical Computing Equivalent (2009)
Scott Aaronson, MIT and  John Watrous, University of Waterloo

Title: Re: EM Drive Developments - related to space flight applications - Thread 2
Post by: aceshigh on 04/20/2015 01:44 pm
well, my guess is that CTCs are impossible and any superluminal travel (including warp drives) are impossible unless they somehow avoid travelling to the past. (I really hate the notion of timetravel to the past, although science is not based on one's preferences)

In other words, I guess the universe is probably consistent in a way that the bending of space-time geometry  is impossible if it involves time travel to the past, unless you someway engineer this bending of space-time geometry in someway similar to what I understand Dr Eric Davis talked in the video above...


@LeftField: as far as I understand, having already questioned physicists I know about it, travelling to the past if you go FTL is independent of the ship not moving inside it´s own space-time (warp drive or wormholes) does not means you WILL travel back in time, but also means nothing stops you from doing so, causing all sort of causality violations.





Title: Re: EM Drive Developments - related to space flight applications - Thread 2
Post by: birchoff on 04/20/2015 02:18 pm
well, my guess is that CTCs are impossible and any superluminal travel (including warp drives) are impossible unless they somehow avoid travelling to the past. (I really hate the notion of timetravel to the past, although science is not based on one's preferences)

In other words, I guess the universe is probably consistent in a way that the bending of space-time geometry  is impossible if it involves time travel to the past, unless you someway engineer this bending of space-time geometry in someway similar to what I understand Dr Eric Davis talked in the video above...


@LeftField: as far as I understand, having already questioned physicists I know about it, travelling to the past if you go FTL is independent of the ship not moving inside it´s own space-time (warp drive or wormholes) does not means you WILL travel back in time, but also means nothing stops you from doing so, causing all sort of causality violations.

Seems a bit premature to be worried about these issues, when we don't even have a proven and accepted theory of how the devices EagleWorks are researching will work. So from my perspective I would have to say these concerns do not matter, because at this point in the research we "don't know what we don't know". Nature could have really simple solutions for all these concerns, but in the best case we have a device that seems to show that we can manipulate the vacuum into changing the length of the path light travels. We have no clue if the effect we are seeing is actually what we think it is, until further experiments are done. So the real question for me is, other than re running the test under vacuum. What other tests can we run to further enhance our understanding of what is going on.
Title: Re: EM Drive Developments - related to space flight applications - Thread 2
Post by: Rodal on 04/20/2015 03:01 pm
...
Seems a bit premature to be worried about these issues, when we don't even have a proven and accepted theory of how the devices EagleWorks are researching will work. So from my perspective I would have to say these concerns do not matter, because at this point in the research we "don't know what we don't know". Nature could have really simple solutions for all these concerns, but in the best case we have a device that seems to show that we can manipulate the vacuum into changing the length of the path light travels. We have no clue if the effect we are seeing is actually what we think it is, until further experiments are done. So the real question for me is, other than re running the test under vacuum. What other tests can we run to further enhance our understanding of what is going on.
Excellent conclusion, @Birchoff.  Concerning your question "other than re running the test under vacuum. What other tests can we run to further enhance our understanding of what is going on" in reference to the interferometer path length measurements, in addition to what they are doing and planning to do, they could also consider:

 running the tests in different pure gas environments ( in addition  to the existing ambient air tests, and in addition to the planned partial vacuum tests).

(This would further put to bed the issue of whether the path length change measurement could possibly be due to refraction, of particular gases.)

EDIT: conducting the tests under certain pure inert gases may be something that they can do first, more readily, faster, than doing these interferometer path length measurements in a partial vacuum.
Title: Re: EM Drive Developments - related to space flight applications - Thread 2
Post by: Rodal on 04/20/2015 03:05 pm
...

QUESTION 6: Has NASA Eagleworks addressed the issue with air refraction raised in this paper by Lee and Cleaver from Baylor University?:

http://arxiv.org/ftp/arxiv/papers/1407/1407.7772.pdf

In particular, has NASA Eagleworks assessed the likelihood of the path-length-change measurements being the result of transient air heating ?

See Dr. White's preliminary assessment of that issue in the attached slide.  Ultimately though we will be running the warp-field resonant cavity with a vacuum contained in its active volume to get rid of all possibilities of air heating problems.

Best, Paul M.

Thanks Paul for your excellent answers.

Concerning the likelihood of the path-length-change measurements being the result of transient air heating, were you able to monitor the transient temperature inside the cavity with embedded thermocouples (or otherwise its external temperature with an infrared thermal camera)?

If you did monitor the transient temperature, could you make that data/plots available to the public in this forum?

Thanks
Title: Re: EM Drive Developments - related to space flight applications - Thread 2
Post by: tchernik on 04/20/2015 06:05 pm
I know Mulletron. What I want to know is exactly how Dr White theory deals with superluminal speeds, which most physicists say leads to time-travel to the past and all the paradoxes that surface from that.


According to the video I showed from Dr Davis, superluminal speeds WITHOUT time-travel to the past are possible, if the light cone is tilted from 0 to 90 degrees only...


I think this question is related to spaceflight applications exactly because time travel IS an issue at relativistic velocities (to the future) and superluminal velocities (to the past, but not according to Dr Davis)

This question is probably more related to the spaceflight applications of a warp drive than the pure theoretical issues of how EM and Warp Drives work on quantum level, since the first is a result of spaceflight application while the second (which is being discussed in this thread) is not.

Don't forget that the ship is not really moving at relativistic speeds: space is.  Consequently, you could take a trip to Alpha Centauri in 2 days (or less with more power... who knows?), turn your ship around and observe the Earth as it was four years ago (as light has taken four years to get there - slow coach!). You could then observe Alpha Centauri as it is "now", and how people on the Earth will see it in four years.

With this type of technology,  it would be possible to predict when locally past events are going to be observable from the point of view of the Earth (or any other point that the light from such events had not yet reached). For example, a ship 1 light-day out from the Earth in the right place could witness a supernova before the Earth does and then be able to return to the Earth almost instantly and tell astronomers about the incoming light wave so that they could prepare to observe it.

Proviso: I am not an expert in time travel and I also have doubts about Dr Who.

This is pretty much what I take from Dr. Eric Davis presentation: not all kinds of FTL travel due to space time distortions (e.g. warp drive) result in the light cones becoming inverted. A lot of them result in just allowing the light cone of a traveler to be slightly "sideways" (e.g. any tilting less than 45 degrees avoids "instantaneous" travel or time travel), allowing the traveler to reach the classically forbidden regions very fast but without paradoxes.

In that way, you will be reaching "past" far away events from the point of view of your point of origin (being there before the light of those events reaches your point of origin), but not the past at your original light cone. As you say, if warp drives exist you could travel in a few days to Alpha Centauri, see what's going on there "in the past" from Earth's point of view (which is just due to you being traveling in a tilted light cone on a trajectory that takes you outside of the classically allowed in your light cone) and be back to report whatever you saw, but always strictly after you left.

If warp drives exist in practice, I won't be surprised if these tilted light cones can't actually bend enough to allow instantaneous or backwards time travel, but they might allow very fast, but finite travel speeds, by tilting the light cone less than 45 degrees...

But I digress: please dear Eagle Works team, continue the very valuable work of proving your points with experimental measurements, for everyone to see and replicate, and then we could emote and get excited about having impulse drives and warp drives. One step at a time.

And I also excuse myself with our kind hosts: I promise this is the last post where I'll discuss unverified claims and hypothesis.

Edit:
I re-read my comment and I think I noticed a mistake: assuming the maximum tilt of the light cone should be less than 45 degrees is wrong. Because that implies that you and your ship could be potentially moving at less than c with respect to your own warp field (inside the allowed region of the tilted light cone), when all evidence suggest you would be stationary inside of it in order to be under its influence!

Under this interpretation, a relatively stationary ship with a warp field tilting its light cone 45 degrees would be travelling exactly at c...

In general, all ships inside a warp field would follow a trajectory that is identical to that of their surrounding field, so the need of having warp fields with light cones with a tilt less than 90 degrees. So aceshigh was right.
Title: Re: EM Drive Developments - related to space flight applications - Thread 2
Post by: Stormbringer on 04/20/2015 07:55 pm
On the subject of further tests: maybe get together with some quantum communications people and devise a test of CTLs or time communications effects. Really the interferometer does this (in it's own way) anyway. But perhaps a test to deliberately check it a different way might have a different S/N ratio? Naturally the easier tests that were suggested by Dr Rodal and others could come first.
Title: Re: EM Drive Developments - related to space flight applications - Thread 2
Post by: dustinthewind on 04/21/2015 02:59 am
I thought I would post a graphic I made of a light cone.  If I am correct for normal warp the cone just flattens suggesting that the observed mote/traveler could move some maximum distance from their original location at some later time.   

I would imagine a tilted warp cone might happen where space is swirling around a rotating black hole and drags objects around it.  If that space reaches light speed or above then the space moving away looks like an event horizon while the space moving towards us is blue-shifted in spectrum.  If the space is moving away at less than c then it should be red-shifted.  In that case if one sits still in moving space then they are moving so the axis is tilted.  (I guess if our space is expanding this suggest we might have tilted light cones?) I don't know that if the central axis is tilted beyond 45 degrees that the light cone would necessarily cross the plane.  I would think light would appear to move at 2*c, 2=m, from an outside observer in one direction and not move at all the other direction m=0 (space is moving against it and it gets nowhere).  That would suggest some distortion of our light cone but that it's not crossing the plane where the slope m = infinity. 

I would think it would require infinite energy to get to warp infinity.  Hopefully I'm not too far off here. 
Title: Re: EM Drive Developments - related to space flight applications - Thread 2
Post by: Star-Drive on 04/21/2015 03:18 am
...

QUESTION 6: Has NASA Eagleworks addressed the issue with air refraction raised in this paper by Lee and Cleaver from Baylor University?:

http://arxiv.org/ftp/arxiv/papers/1407/1407.7772.pdf

In particular, has NASA Eagleworks assessed the likelihood of the path-length-change measurements being the result of transient air heating ?

See Dr. White's preliminary assessment of that issue in the attached slide.  Ultimately though we will be running the warp-field resonant cavity with a vacuum contained in its active volume to get rid of all possibilities of air heating problems.

Best, Paul M.

Thanks Paul for your excellent answers.

Concerning the likelihood of the path-length-change measurements being the result of transient air heating, were you able to monitor the transient temperature inside the cavity with embedded thermocouples (or otherwise its external temperature with an infrared thermal camera)?

If you did monitor the transient temperature, could you make that data/plots available to the public in this forum?

Thanks

Dr. Rodal:

I'm attaching three slides that provide a bit more insight into the data sets that were taken over two weekends in the Warp-field interferometer tests.  In the last you will note the temp rise of the resonant cavity over its first 70 minutes of on/off cycling.  After determining this thermal rise time constant with our IR camera, we let the test article warm up for approximately one hour before starting our data acquisition of 27,000 to 30,000 ~1.5 second on/off cycles with each on/off cycle resulting in a CCD camera picture with a few examples shown in the attached FFTs of Imager slide.  I've also appended a slide with the FFT noise present at the 2/3 second time point.

"If you did monitor the transient temperature, could you make that data/plots available to the public in this forum?"

As noted above we did not have a fast response thermocouple in the resonant cavity to monitor fast air temp changes, but the outside IR camera data indicates that the internal air temp probably stabilized around a mean aluminum case temperature of ~88F after about 90 minutes of on/off cycling operation.  We also found that after the cylindrical aluminum cavity reached its running temp, we did not have to continue adjusting the cavity RF tuning to maintain its 30W of input power at 1.48 GHz.

BTW, the Eagleworks 6061 aluminum cylindrical cavity used in this experiment has machined 0.25" thick endplate walls and cylinder, so it has proven to be very stable once it has reached its run temp.  I.e., there is over 2.5kg of thermal mass in this cavity design that reduces fast temp shifts.

Best, Paul M.

Correction:

I said Previously in this post:

"After determining this thermal rise time constant with our IR camera, we let the test article warm up for approximately one hour before starting our data acquisition of 27,000 to 30,000 ~1.5 second on/off cycles with each on/off cycle resulting in a CCD camera picture with a few examples shown in the attached FFTs of Imager slide."

This is what I get when I try to translate another persons work.  By that I mean after talking with Dr. White this morning, I realized I made an error in my above summary of the interferometer data acquisition process.  That being the 27,000-to-30,000 on/off cycles do NOT create an x-y "puddle plot" for every on/off cycle.  Instead all 27,000+ on/off samples are averaged over four active points on the ~1280 x 960 CCD pixel array and then averaged to make the associated FFT frequency plot from 0-to-5 Hz.  Sonny is now in the process of coming up with an average and peak data analysis technique that will use ALL the CCDE pixels instead of just four per frame.  Oh yes, and develop an algorithm that doesn't take 100 years to process on his laptop.  :)

Best, Paul M.
Title: Re: EM Drive Developments - related to space flight applications - Thread 2
Post by: Star-Drive on 04/21/2015 12:09 pm
I thought I would post a graphic I made of a light cone.  If I am correct for normal warp the cone just flattens suggesting that the observed mote/traveler could move some maximum distance from their original location at some later time.   

I would imagine a tilted warp cone might happen where space is swirling around a rotating black hole and drags objects around it.  If that space reaches light speed or above then the space moving away looks like an event horizon while the space moving towards us is blue-shifted in spectrum.  If the space is moving away at less than c then it should be red-shifted.  In that case if one sits still in moving space then they are moving so the axis is tilted.  (I guess if our space is expanding this suggest we might have tilted light cones?) I don't know that if the central axis is tilted beyond 45 degrees that the light cone would necessarily cross the plane.  I would think light would appear to move at 2*c, 2=m, from an outside observer in one direction and not move at all the other direction m=0 (space is moving against it and it gets nowhere).  That would suggest some distortion of our light cone but that it's not crossing the plane where the slope m = infinity. 

I would think it would require infinite energy to get to warp infinity.  Hopefully I'm not too far off here. 

When considering the use of EM-drives in the making of warp-drives the attached two papers might be of interest.

In the meantime, back to figuring out how to reliably drive an EM-drive...

Edit:  You might also like to read Sonny's Warp-field Mechanics 101 and 102 articles.
http://ntrs.nasa.gov/archive/nasa/casi.ntrs.nasa.gov/20130011213.pdf

Best, Paul M.
Title: Re: EM Drive Developments - related to space flight applications - Thread 2
Post by: Rodal on 04/21/2015 01:58 pm
Is the Cannae drive effort still ongoing ? Any news on Cannae?

Cannae's website ( http://cannae.com/ ) gives today an Internal Server Error.

Cannae's twitter account ( https://twitter.com/cannaeqdrive ) last twit was dated May 12 2014, practically a year ago.

Last video uploaded by Cannae to their Vimeo page was one year ago:

https://vimeo.com/cannae/videos

The wayback machine cannot longer display previous entries like this one (quoted on the wikipedia EM Drive article):

http://web.archive.org/web/20121102082714/http://www.cannae.com/proof-of-concept/experimental-results

due to the Robots exclusion standard.

One cannot get much information from the Bloomberg page on Cannae LLC:

http://www.bloomberg.com/research/stocks/private/snapshot.asp?privcapId=134038401

or in Manta:

http://www.manta.com/c/mwfq0td/cannae-llc

Also, I thought that at one time there was a Wikipedia page for Cannae's drive: either my memory is incorrect or the Wikipedia Cannae drive page has been removed or if it exists, I can't find it.  The EM Drive and Quantum vacuum plasma thruster Wikipedia pages remain (albeit both containing strong Wikipedia warnings about their quality).

Title: Re: EM Drive Developments - related to space flight applications - Thread 2
Post by: Rodal on 04/21/2015 06:25 pm
....
BTW, we are going to add optical borosilicate telescope grade flat windows to the ends of the RF chokes when we get around to pulling a vacuum in this 1.48 GHz resonant cavity.
....
Best, Paul M.

Issues like this (and several other practical difficulties) associated with doing this experiment in a partial vacuum, lead me to suggest to do different experiments of path length measurement, in the interim, in different inert gas environments, each having different refraction properties. 

Doing the same path length measurements, but this time in different inert gas environments may serve to put to bed the issue of the interferomenter measurements being due to refraction.

http://en.wikipedia.org/wiki/Kerr_effect

http://www.sciencedirect.com/science/article/pii/S0030401814003794

attachment:

Measurement of pressure dependent nonlinear refractive index of inert gases
Á. Börzsönyi,1 Z. Heiner,1,2 A.P. Kovács,1 M. P. Kalashnikov3 and K. Osvay1,*

Title: Re: EM Drive Developments - related to space flight applications - Thread 2
Post by: Chris Bergin on 04/21/2015 11:46 pm
So we're over 300,000 views on this thread now. Sure, a SpaceX thread - for example - can do that sort of level and more, but for an advanced topic thread this is rare.

The item of interest is a huge amount of NASA IPs have been hitting this thread over the past number of days. I know we already have a lot of NASA on here, but my experience says there's been a memo or such that has pointed NASA folk to this thread. If any of you wish to let me know, hit me up on PM. Previous experience of that sort of thing is where they add a "and a cool item being discussed on X site is...." at the end of a ViTS or "8th Floor" style memo. Would be nice to see if it's been linked as such.

(EDIT: Found out it was a NASA pointer brief, but no specific reference other than a link about "ongoing EM Drive conversations").

And yes, we are still working an article on this. Long process as the subject matter is pretty hard to translate to the wide audience of the news site.

EDIT: *Lots* of people turning up today, mainly social media like Reddit subs, Twitter, Google, the whole works, even Star Trek fans (I kid you not!) - all at the same time. I can see the referrals on our server stats, so no need to let me know (lots of PMs about it today, but nice to hear from new folk :) ).
Title: Re: EM Drive Developments - related to space flight applications - Thread 2
Post by: Star-Drive on 04/22/2015 04:20 am
....
BTW, we are going to add optical borosilicate telescope grade flat windows to the ends of the RF chokes when we get around to pulling a vacuum in this 1.48 GHz resonant cavity.
....
Best, Paul M.

Issues like this (and several other practical difficulties) associated with doing this experiment in a partial vacuum, lead me to suggest to do different experiments of path length measurement, in the interim, in different inert gas environments, each having different refraction properties. 

Doing the same path length measurements, but this time in different inert gas environments may serve to put to bed the issue of the interferomenter measurements being due to refraction.

http://en.wikipedia.org/wiki/Kerr_effect

http://www.sciencedirect.com/science/article/pii/S0030401814003794

attachment:

Measurement of pressure dependent nonlinear refractive index of inert gases
Á. Börzsönyi,1 Z. Heiner,1,2 A.P. Kovács,1 M. P. Kalashnikov3 and K. Osvay1,*


Dr. Rodal:

Thanks for the nonlinear refractive index of inert gases paper.  I'll pass it along to Dr. White and Dr. Rollins, but I'm sure they have already committed themselves to drawing at least a 1x10^-3 Torr vacuum in the 1.48 GHz cylindrical cavity for their next test series as soon as the parts for same show up.

On anther related front, today at our Eagleworks staff meeting we kicked around the idea of using a high resolution piezoelectric actuator to calibrate the magnitude of the space-time contractions that the 1.48 GHz resonant cavity might be generating.  As is we only have a qualitative measure of the magnitude of the path length contractions using the current interferometer setup.  However if we added the aforementioned piezo-actuator with a repeatable displacement resolution of at least a nanometer (nm) to one of the interferometer mirror mounts, we could turn off the resonant cavity and emulate its space-time contractions using the piezo driven mirror with the same timed on/off cycles using first, say a known +/-1.0 nm oscillation of the attached mirror, and then see what this piezo driven displacement magnitude drives in the FFT response of interferometer.  Once we know how much ac E-field in the cavity it takes to drive x.x  space-time contractions in the cavity, we can then start to populate the curve required to determine the degree of non-linearity there is in this forcing function and whether it matches Dr. White's Q-V conjecture predictions.

Best,  Paul M.
Title: Re: EM Drive Developments - related to space flight applications - Thread 2
Post by: zen-in on 04/22/2015 05:32 am
In the late 70's I worked for a company that made FTIR spectrometers.   They use small aperture interferometers for a laser and white light source to locate the position of the moving mirror and to trigger the A/D sampling.   A laser interferogram is a sine wave while the white light interferogram is a sinc function with a prominent peak.   The trick to achieving stable sampling is to position the sinc peak midway between two laser interferogram peaks so that any thermal drift does not result in a sample point jump.   The best long term stability I was able to achieve with a Digilab interferometer was +/- 40 nM shift in the laser peak wrt to the white light peak over a 24 Hr. period.   That interferometer was used on the KAO.   However during the test it was mounted in a covered and temperature controlled optical table.   The mirror mounts were specially designed to dampen vibrations.   With an air bearing Michelson interferometer (one mirror moving) the precision can be monitored.   The very small phase shifts ( a few nM) measured by the Eagleworks team as the magnitude of the space-time contractions are more than an order of magnitude below the best positional accuracy a well designed interferometer is capable of.    To validate this measurement I would suggest using an FTIR spectrometer in a dual sampling mode and with the laser as the source.   An additional reference would be the smaller laser interferometer used for A/D sampling.
Title: Re: EM Drive Developments - related to space flight applications - Thread 2
Post by: Rodal on 04/22/2015 12:15 pm
In the late 70's I worked for a company that made FTIR spectrometers.   They use small aperture interferometers for a laser and white light source to locate the position of the moving mirror and to trigger the A/D sampling. ...  The best long term stability I was able to achieve with a Digilab interferometer was +/- 40 nM shift in the laser peak wrt to the white light peak over a 24 Hr. period.   That interferometer was used on the KAO.   However during the test it was mounted in a covered and temperature controlled optical table.   The mirror mounts were specially designed to dampen vibrations.   With an air bearing Michelson interferometer (one mirror moving) the precision can be monitored.   The very small phase shifts ( a few nM) measured by the Eagleworks team as the magnitude of the space-time contractions are more than an order of magnitude below the best positional accuracy a well designed interferometer is capable of.   ...

There are several problems requiring measurement of changes in the optical path length with nanometer precision, for example:  A)  microelectromechanical system microdevices, from microscanners to micromotors, B) nanometer walk (piconewton force) in biological micromotors, C) the determination of the mechanical transfer function of low-mass cantilevers for data storage devices, and D) accurate determination of the transfer function for the pendular suspension used for the mirrors in gravitational wave interferometers.

State-of-the-art interferometers sensitive enough to measure displacements below 1 Hz with a RMS error of approximately 1 nanometer were reported 15 years ago.
Title: Re: EM Drive Developments - related to space flight applications - Thread 2
Post by: mvpel on 04/22/2015 12:45 pm
QUESTION 2: If so, did the observed anomalous peak in the Power Spectral Density occur at a frequency in accordance with the time taken to energize  and de-energize?

Yes, the on/off cycle time was around 1.5 seconds with some uncertainty due to Windows 7.0 time outs.  Need a real time operating system (RTOto clear that problem, a RTOS system we don't have.

We use Red Hat Enterprise MRG Realtime for our application, and the CentOS project provides a free rebuild version of the Red Hat "kernel-rt" package set which you can install. This site http://dev.centos.org/~z00dax/mrg/ has RPMs for it, but seems a little out of date - the latest RHEL kernel-rt is 3.10.58 - but the free one would probably suit your purposes. It can be a little finicky about hardware sometimes, so you'd want to take that into consideration.
Title: Re: EM Drive Developments - related to space flight applications - Thread 2
Post by: Rodal on 04/22/2015 04:04 pm
QUESTION 2: If so, did the observed anomalous peak in the Power Spectral Density occur at a frequency in accordance with the time taken to energize  and de-energize?

Yes, the on/off cycle time was around 1.5 seconds with some uncertainty due to Windows 7.0 time outs.  Need a real time operating system (RTOto clear that problem, a RTOS system we don't have.

We use Red Hat Enterprise MRG Realtime for our application, and the CentOS project provides a free rebuild version of the Red Hat "kernel-rt" package set which you can install. This site http://dev.centos.org/~z00dax/mrg/ has RPMs for it, but seems a little out of date - the latest RHEL kernel-rt is 3.10.58 - but the free one would probably suit your purposes. It can be a little finicky about hardware sometimes, so you'd want to take that into consideration.

The latest real time offering from Red Hat is here:
https://access.redhat.com/products/red-hat-enterprise-linux/realtime

The Red Hat realtime kernel is designed for environments which require low latency and more predictability than a generic OS. Apparently, financial companies (banks / hedge funds) use Red Hat realtime kernels (sometimes with their own modifications) for their trading purposes (perhaps high frequency trading, but obviously they keep their purposes confidential). 

Is a device driver for the instrument also involved in the interferometer tests ?

If a device driver for the instrument is also involved one may be stuck with whatever RTOS the instrument maker recommends (unless it is some generic interface). Also, embedded RTOS are used by various telecommunication companies.

(Any credit for this information goes to D.W. and any errors in it belong exclusively on my shoulders).
Title: Re: EM Drive Developments - related to space flight applications - Thread 2
Post by: bprager on 04/22/2015 06:08 pm
QUESTION 2: If so, did the observed anomalous peak in the Power Spectral Density occur at a frequency in accordance with the time taken to energize  and de-energize?

Yes, the on/off cycle time was around 1.5 seconds with some uncertainty due to Windows 7.0 time outs.  Need a real time operating system (RTOto clear that problem, a RTOS system we don't have.

We use Red Hat Enterprise MRG Realtime for our application, and the CentOS project provides a free rebuild version of the Red Hat "kernel-rt" package set which you can install. This site http://dev.centos.org/~z00dax/mrg/ has RPMs for it, but seems a little out of date - the latest RHEL kernel-rt is 3.10.58 - but the free one would probably suit your purposes. It can be a little finicky about hardware sometimes, so you'd want to take that into consideration.

There is also a free versions of RTOS available: http://www.freertos.org/
Title: Re: EM Drive Developments - related to space flight applications - Thread 2
Post by: Giovanni DS on 04/22/2015 07:00 pm
Oh there is also another free one, I know the guy maintaining it... very well: http://www.chibios.org
Title: Re: EM Drive Developments - related to space flight applications - Thread 2
Post by: cfs on 04/23/2015 01:59 am
Ubuntu RTOS hasn't been mentioned yet: https://wiki.ubuntu.com/RealTime (https://wiki.ubuntu.com/RealTime)
As actively maintained as RHEL, though with different package management and (slightly) different options in terms of drivers and features.

RHEL/CentOS is typically the choice for servers as they tend to focus more on dependability rather than pushing out the latest features.
Title: Re: EM Drive Developments - related to space flight applications - Thread 2
Post by: Mulletron on 04/23/2015 02:07 am
I never really got interested in Warp drive because I considered that to be a whole other can of worms (aka not even close to being the same thing as Emdrive), not to mention the whole idea just seemed way way out there.

So I asked myself, why is Eagleworks treating their Q-thruster like it is a warp experiment? This question cropped up before when the video below surfaced a few months ago. Specifically the question and answer at 56:40.
https://www.youtube.com/watch?v=Wokn7crjBbA


So I filed that one away for later. I didn't really get it. Boy was I in for a surprise. Check out this quote:

Quote
An actual laboratory demonstration of a metamaterial warp drive space time would require a non-reciprocal bi-anisotropic metamaterial, in which both spatial and time reversal symmetries are broken.
http://arxiv.org/abs/1009.5663
http://beforeitsnews.com/science-and-technology/2011/08/updated-paper-httparxiv-orgftparxivpapers10091009-5663-pdf-984797.html

There's the old PT symmetry thing all over again! The same old themes (magnetoelectrics, broken PT symmetries) which were uncovered while trying to figure out Emdrives... are there for warp drive too. As far as I can tell, the only difference between the two is whether the spacetime distortion is small and inside the ship or large and around the ship.

It is becoming clear that we need to think of the QV as a polarizable dielectric medium. Just like others have said. Even the mainstream says the QV is a dielectric. So while it is undeniable that GR and SR are correct as always, there is ample evidence which supports the notion that the classical spacetime we all know and love (flat and curved spacetime) is emergent from a more fundamental quantum chaos underneath, and that chaos can be engineered.

@Aceshigh posted a video a few days ago of the 2013 Icarus Interstellar conference day 3.
http://forum.nasaspaceflight.com/index.php?topic=36313.msg1362005#msg1362005

At the 1:31:00 mark there is a presentation by Dr. Hal Puthoff. I've always been a bit skeptical of his claims. I've read his work and took note of what he was saying but I had no evidence to suggest he might be onto something. A quick Google search reveals that he gets a bad rap from many, so that turned me away in search of my own ideas on how things work. Whether he is fully correct or not, I'm going to pay closer attention to what he has to say from now on.

Some papers we've discussed before on here, some of which were referenced in the Starship Conference presentation mentioned above:
http://arxiv.org/abs/1302.6165
http://arxiv.org/abs/1005.0131
http://arxiv.org/abs/1204.2184

This is all going to fall on experiment. The team I mentioned before who observed non-reciprocity of the velocity of light, http://arxiv.org/abs/1101.0712 predict the same for the vacuum. So if a team finds the same effect in vacuum, we might be in business.

Quote
Our long term goal is to search for
the magneto-electric non-reciprocity of quantum vacuum
[21, 28], which is approximately 7 × 108 times smaller
than what we have measured. Its detection would require
fields as high as B = 15 T and E = 20 MV/m, a better
cavity with a finesse of 200 000 and a noise level corre-
sponding to the shot-noise level with an injected laser
power near 50 mW. All these performances have already
been achieved separately, but bringing them together is
obviously a very ambitious challenge.







Title: Re: EM Drive Developments - related to space flight applications - Thread 2
Post by: dustinthewind on 04/23/2015 02:19 am
I found it interesting that in the video posted by aceshigh, starship congress the engineer concept of warp, youtube time:(1:44:17), -->the object grows brighter and more energetic and reduces in mass.  <--my paraphrasing.  The concept presented was reducing the space time dielectric constant and even shows the Alcubierre bubble. 

Maybe it is just me but this appears to contrast with the concept in the paper posted by Star-Drive.  Paper : * White & Davis_STAIF_conference_2.pdf [link: http://forum.nasaspaceflight.com/index.php?action=dlattach;topic=36313.0;attach=825790 ]

quote from paper: "Hence, an Earthbound coordinate observer will most likely see a dimming effect when viewing the ALIF as its spacetime expansion boost increases. Similarly, the ALIF will see his/her surrounding universe grow dimmer and dimmer as photons emitted on-brane cannot reach the now off-brane ALIF."

Question 1: So are we looking at two different concepts of warp (separate concepts) or am I am getting confused? 
Later edit: I later realized the engineer in the video may be talking about an object existing in the negative energy density space (glows brightly) where as in the paper the Alcubierre bubble (the ship dims) the ship resides inside the flat space time but is surrounded by a gradient of space density. 

In the back of my mind the concept I presented of two resonating cavities 1/4 [/lambda] out of phase and one doing negative work while the other positive work makes me question if it could end up also pumping space time or casmir vaccuum energy.  When I look at the ring around the ship [in the presentations] and the gradient in the Alcubierre bubble it makes me think of the ring as being a space time pump so I do wonder at the possibility of a relationship if it works.  I suspect something similar to two or more currents out of phase in space and time in the cavity but remain unsure.  It would just be impressive enough if we could beat, by a large factor, the best ion engines out there but some times one idea lead will lead to others. 

Question 2: One more question I have is the space time contractions they are observing in the cavity.  Is it constant and scales with power as the device is powered up?  Another way of asking this is if the fringes shift with power and then shift back as power is reduced.  Or is it that they are observing pulsating space time where the fringes are osculating maybe around the microwave frequency?

Edit later: thanks Mulltron for the video.  It may have answered  my question.  It looks like from the video they are talking about a constant space time contraction in the experiment detected rather than pulsations at a microwave frequency. 

@Paul March: since the warp drive is to be considered then as ontopic in this thread, how exactly does Dr. White theories deal with time-travel to the past in superluminal speeds? I guess that would be a major point of any space travel related applications of a warp drive.

I remember that in Starship Congress 2013 (at which Dr. White made the first talk on Day 3 – Interstellar Future (50 years +) | Saturday August 17th, 2013 - Sonny White, “Warp Field Physics: an Update” ), the talk just after Dr. White's, was by Dr Eric Davis, that was quite mind bending, where he talks about tipping the light cone in Warp Drives (I guess that would be space-time engineering just like the Warp Drive itself?) so inside it's light cone the ship is not travelling to the past and thus there is no worry of causality violations.

Is that a view that Dr White (or yourself) agree with? Or are you sure a Warp Drive will certainly result in travel to the past?

To anyone wanting to see Dr Eric Davis talk, here is the video of the full Day 3... Dr Eric Davis talk starts at 58:00.
https://www.youtube.com/watch?v=ucyBMB_PWr8

I guess it´s on-topic because it´s intimally related to the Warp Drive by Dr White.

the topic is "light cone gymnastics"  (really) ;D

other quote

I thought I would post a graphic I made of a light cone.  If I am correct for normal warp the cone just flattens suggesting that the observed mote/traveler could move some maximum distance from their original location at some later time.   

I would imagine a tilted warp cone might happen where space is swirling around a rotating black hole and drags objects around it.  If that space reaches light speed or above then the space moving away looks like an event horizon while the space moving towards us is blue-shifted in spectrum.  If the space is moving away at less than c then it should be red-shifted.  In that case if one sits still in moving space then they are moving so the axis is tilted.  (I guess if our space is expanding this suggest we might have tilted light cones?) I don't know that if the central axis is tilted beyond 45 degrees that the light cone would necessarily cross the plane.  I would think light would appear to move at 2*c, 2=m, from an outside observer in one direction and not move at all the other direction m=0 (space is moving against it and it gets nowhere).  That would suggest some distortion of our light cone but that it's not crossing the plane where the slope m = infinity. 

I would think it would require infinite energy to get to warp infinity.  Hopefully I'm not too far off here. 

When considering the use of EM-drives in the making of warp-drives the attached two papers might be of interest.

In the meantime, back to figuring out how to reliably drive an EM-drive...

Edit:  You might also like to read Sonny's Warp-field Mechanics 101 and 102 articles.
http://ntrs.nasa.gov/archive/nasa/casi.ntrs.nasa.gov/20130011213.pdf

Best, Paul M.

other


...  If I were going to test them for what I am talking about I would take two cylindrical cavities with the radiation input of one cavity able to be phase shifted and amplified.  Put the two cavities flat plates next to each other so that the imaginary magnetic field (non radiating [decaying]) overlaps.  The separation would be about 1/4 lambda separation in air for the frequency chosen.  My guess is the signal of one would bleed into the other cavity which would seem to push them to be matched up in phase and not perfectly out of phase %pi/2.  They are supposed to be out of phase %pi/2 so you increase the phase and amplitude of the cavity that is working against the other till it seems they are properly out of phase 1/4 lambda with matching amplitude.  You might install a current sensor on each cavity to make sure you know the exact current phase and amplitude.  Maybe then its possible to stack the cavities on top of each other one after the other all being off in phase 0, %pi/2, %pi, 3%pi/2 ect...
Title: Re: EM Drive Developments - related to space flight applications - Thread 2
Post by: eagleon on 04/23/2015 02:30 am
https://en.wikipedia.org/wiki/Air_vortex_cannon (https://en.wikipedia.org/wiki/Air_vortex_cannon)
https://en.wikipedia.org/wiki/Vortex_ring (https://en.wikipedia.org/wiki/Vortex_ring)
Just a crazy idea... but your apparatus looks like a toy I had when I was a kid. Toroidal vortex, with a propagating energy pressure gradient dipping into the negative at the top maybe? I'm sorry if I sound like a kook, but it'd be too great if I was on to something.
Title: Re: EM Drive Developments - related to space flight applications - Thread 2
Post by: Stormbringer on 04/23/2015 03:00 am
thinking that some devices present at least transient negative energy is not ko0ky k0oky. It's accepted physics.

That photonic diametric drive thing makes use of it. Squeezed light (i think.) But in a circular ring there would be more of it present more often. they used two of them stacked on top of one another. In fact it looks a little like that diagram people have been posting here lately.

Title: Re: EM Drive Developments - related to space flight applications - Thread 2
Post by: Mulletron on 04/23/2015 03:14 am
From Cuban researchers. Both deal with QV interaction with magnetic fields:

So this makes 4 teams I've found who predict momentum transfer from the QV.

Quote
Our results show that quantum vacuum in a constant magnetic field
may exert pressures, either positive or negative, which means a transfer of momentum
from vacuum to real particles or macroscopic bodies

The links I got these from are down most of the time, so I had to upload them.
ftp://www.astro.iag.usp.br/pub/dalpino/Proceedings/CD-ROM/014-Rodriguez_Quertz.pdf
http://www.if.ufrgs.br/hadrons/HRojas1.pdf



Title: Re: EM Drive Developments - related to space flight applications - Thread 2
Post by: Star-Drive on 04/23/2015 04:26 am
I never really got interested in Warp drive because I considered that to be a whole other can of worms (aka not even close to being the same thing as Emdrive), not to mention the whole idea just seemed way way out there.

So I asked myself, why is Eagleworks treating their Q-thruster like it is a warp experiment? This question cropped up before when the video below surfaced a few months ago. Specifically the question and answer at 56:40.
https://www.youtube.com/watch?v=Wokn7crjBbA


So I filed that one away for later. I didn't really get it. Boy was I in for a surprise. Check out this quote:

Quote
An actual laboratory demonstration of a metamaterial warp drive space time would require a non-reciprocal bi-anisotropic metamaterial, in which both spatial and time reversal symmetries are broken.
http://arxiv.org/abs/1009.5663
http://beforeitsnews.com/science-and-technology/2011/08/updated-paper-httparxiv-orgftparxivpapers10091009-5663-pdf-984797.html

There's the old PT symmetry thing all over again! The same old themes (magnetoelectrics, broken PT symmetries) which were uncovered while trying to figure out Emdrives... are there for warp drive too. As far as I can tell, the only difference between the two is whether the spacetime distortion is small and inside the ship or large and around the ship.

It is becoming clear that we need to think of the QV as a polarizable dielectric medium. Just like others have said. Even the mainstream says the QV is a dielectric. So while it is undeniable that GR and SR are correct as always, there is ample evidence which supports the notion that the classical spacetime we all know and love (flat and curved spacetime) is emergent from a more fundamental quantum chaos underneath, and that chaos can be engineered.

@Aceshigh posted a video a few days ago of the 2013 Icarus Interstellar conference day 3.
http://forum.nasaspaceflight.com/index.php?topic=36313.msg1362005#msg1362005

At the 1:31:00 mark there is a presentation by Dr. Hal Puthoff. I've always been a bit skeptical of his claims. I've read his work and took note of what he was saying but I had no evidence to suggest he might be onto something. A quick Google search reveals that he gets a bad rap from many, so that turned me away in search of my own ideas on how things work. Whether he is fully correct or not, I'm going to pay closer attention to what he has to say from now on.

Some papers we've discussed before on here, some of which were referenced in the Starship Conference presentation mentioned above:
http://arxiv.org/abs/1302.6165
http://arxiv.org/abs/1005.0131
http://arxiv.org/abs/1204.2184

This is all going to fall on experiment. The team I mentioned before who observed non-reciprocity of the velocity of light, http://arxiv.org/abs/1101.0712 predict the same for the vacuum. So if a team finds the same effect in vacuum, we might be in business.

Quote
Our long term goal is to search for
the magneto-electric non-reciprocity of quantum vacuum
[21, 28], which is approximately 7 × 10^8 times smaller
than what we have measured. Its detection would require
fields as high as B = 15 T and E = 20 MV/m, a better
cavity with a finesse of 200,000 and a noise level corre-
sponding to the shot-noise level with an injected laser
power near 50 mW. All these performances have already
been achieved separately, but bringing them together is
obviously a very ambitious challenge.


Mulletron:

Are the above E and B-field conditions just for the dc case?  If they are, their absolute magnitudes can be greatly reduced for the same vacuum compression if they use repetitive and very fast rise time ac signals instead of dc per Dr. White's Q-V conjecture.  That is the core truth behind what we are trying to do for both the Q-Thrusters and warp-drives, for the magnitude of the space-time bending effect or polarization of the quantum vacuum per Hal Puthoff is the product of the absolute magnitude of the applied E&M fields, AND their time rate of change of energy density phi with both dphi/dt and dphi^2/dt^2 terms driving the magnitude of the vacuum compression effect.

Best,  Paul M.   
Title: Re: EM Drive Developments - related to space flight applications - Thread 2
Post by: Mulletron on 04/23/2015 05:29 am
They report using permanent magnets for B and a sinusoidally driven HVPS for E.


Recent discussion wrt connecting EM to warp reminded me of this. His youtube channel is amazing. I wish we had him here in this thread.

http://youtu.be/8HazOEqeae8
Title: Re: EM Drive Developments - related to space flight applications - Thread 2
Post by: dustinthewind on 04/23/2015 06:06 am
They report using permanent magnets for B and a sinusoidally driven HVPS for E.


Recent discussion wrt connecting EM to warp reminded me of this. His youtube channel is amazing. I wish we had him here in this thread.

http://youtu.be/8HazOEqeae8

This is the same person doing the video I posted in the thread, "http://forum.nasaspaceflight.com/index.php?topic=36911.0".  It was this video here, https://www.youtube.com/watch?t=13&v=ZqC3AVcuFaE .  I messaged him over youtube and he responded.  You could message him and invite him to the thread I suppose. 
Title: Re: EM Drive Developments - related to space flight applications - Thread 2
Post by: Rodal on 04/23/2015 01:23 pm
They report using permanent magnets for B and a sinusoidally driven HVPS for E.


Recent discussion wrt connecting EM to warp reminded me of this. His youtube channel is amazing. I wish we had him here in this thread.

http://youtu.be/8HazOEqeae8

Is the following just "defined"

β f = - h /(1 - h )

without addressing what are the conditions for this "definition" to hold, and what is the validity of this "definition" ?

and then predictive conclusions are extracted from plugging in this "definition" into the exact solution?
Title: Re: EM Drive Developments - related to space flight applications - Thread 2
Post by: Stormbringer on 04/23/2015 04:08 pm
Has this been posted here yet?

http://www.emdrive.com/

There are links and such. Some of them might be new.
Title: Re: EM Drive Developments - related to space flight applications - Thread 2
Post by: Star One on 04/23/2015 04:55 pm

Has this been posted here yet?

http://www.emdrive.com/

There are links and such. Some of them might be new.

Are you referring to this?

Quote
The EmDrive

A New Concept in Spacecraft Propulsion

Satellite Propulsion Research Ltd (SPR Ltd) a small UK based company, has demonstrated a remarkable new space propulsion technology. The company has successfully tested both an experimental thruster and a demonstrator engine which use patented microwave technology to convert electrical energy directly into thrust. No propellant is used in the conversion process. Thrust is produced by the amplification of the radiation pressure of an electromagnetic wave propagated through a resonant waveguide assembly.
Title: Re: EM Drive Developments - related to space flight applications - Thread 2
Post by: JasonAW3 on 04/23/2015 04:59 pm
So it looks like my conjecture that the device is creating some sort of spatial warp field is likely?

Well, color me surprised!  I used Occam's Razor for that conjecture.  All the other explanations seemed too complex, so, I figured that the simplest explanation was a warping of space.  The conditions seemed to follow some of what I read about Einstein’s speculations about space warps and something I'd read about gravity and magnetism likely being related.  (Wish I could remember where I read that though).
Title: Re: EM Drive Developments - related to space flight applications - Thread 2
Post by: Rodal on 04/23/2015 05:10 pm
Has this been posted here yet?

http://www.emdrive.com/

There are links and such. Some of them might be new.
Not in that Wikipedia article, instead poster  "Quantanew" posted the interferometer information from the NSF EM Drive thread (including the images from Paul March) in this Wikipedia article:

http://en.wikipedia.org/wiki/White%E2%80%93Juday_warp-field_interferometer
Title: Re: EM Drive Developments - related to space flight applications - Thread 2
Post by: Stormbringer on 04/23/2015 05:26 pm
Just the page and the links in general; nothing specific. I was just wondering if it had been posted here yet.
Title: Re: EM Drive Developments - related to space flight applications - Thread 2
Post by: Star One on 04/23/2015 05:48 pm

Just the page and the links in general; nothing specific. I was just wondering if it had been posted here yet.

I've known of the website but are those particular contents new then?
Title: Re: EM Drive Developments - related to space flight applications - Thread 2
Post by: tchernik on 04/23/2015 06:27 pm

Just the page and the links in general; nothing specific. I was just wondering if it had been posted here yet.

I've known of the website but are those particular contents new then?

AFAIK, no. The links on the front page are ordered chronologically from newer at the top to older at the bottom, and the last update was on January of this year.
Title: Re: EM Drive Developments - related to space flight applications - Thread 2
Post by: Star One on 04/23/2015 06:40 pm


Just the page and the links in general; nothing specific. I was just wondering if it had been posted here yet.

I've known of the website but are those particular contents new then?

AFAIK, no. The links on the front page are ordered chronologically from newer at the top to older at the bottom, and the last update was on January of this year.

Thanks. Wasn't sure if it had all just been updated with announcements from various  dates.
Title: Re: EM Drive Developments - related to space flight applications - Thread 2
Post by: Rodal on 04/23/2015 06:41 pm
Concerning "warp-drive", please notice that the following was placed today on NASA Glenn's website ( http://www.nasa.gov/centers/glenn/technology/warp/warp.html ):

Bold added for emphasis:
Quote
Is Warp Drive Real?
Ever since the sound barrier was broken, people have turned their attention to how we can break the light speed barrier.  But “Warp Drive” or any other term for faster-than-light travel still remains at the level of speculation.

The bulk of scientific knowledge concludes that it’s impossible, especially when considering Einstein’s Theory of Relativity. There are certainly some credible concepts in scientific literature, however it’s too soon to know if they are viable.

Science fiction writers have given us many images of interstellar travel, but traveling at the speed of light is simply imaginary at present.

In the meantime, science moves forward.  And while NASA is not pursuing interstellar flight, scientists here continue to advance ion propulsion for missions to deep space and beyond using solar electric power. This form of propulsion is the fastest and most efficient to date.

There are many “absurd” theories that have become reality over the years of scientific research.  But for the near future, warp drive remains a dream.

If you would like to know more about the theories of interstellar flight, you should visit the Tau Zero Foundation.  Marc Millis, a former NASA Glenn physicist, founded the organization to consider revolutionary advancements in propulsion.

Past articles of warp drive found at this location have been archived.

Nancy Smith Kilkenny, SGT Inc.
NASA's Glenn Research Center

Last Updated: April 23, 2015
Editor: NASA Administrator

also notice that the original NASA Glenn webpages on

Alcubierre’s "Warp Drive"

and on

Worm Hole transportation

at http://www.nasa.gov/centers/glenn/technology/warp/warpstat_prt.htm

have been deleted and they have now been re-directed to the above message
Title: Re: EM Drive Developments - related to space flight applications - Thread 2
Post by: tchernik on 04/23/2015 06:50 pm
Concerning "warp-drive", please notice that the following was placed today on NASA Glenn's website ( http://www.nasa.gov/centers/glenn/technology/warp/warp.html ):

Bold added for emphasis:
Quote
Is Warp Drive Real?
Ever since the sound barrier was broken, people have turned their attention to how we can break the light speed barrier.  But “Warp Drive” or any other term for faster-than-light travel still remains at the level of speculation.

The bulk of scientific knowledge concludes that it’s impossible, especially when considering Einstein’s Theory of Relativity. There are certainly some credible concepts in scientific literature, however it’s too soon to know if they are viable.

Science fiction writers have given us many images of interstellar travel, but traveling at the speed of light is simply imaginary at present.

In the meantime, science moves forward.  And while NASA is not pursuing interstellar flight, scientists here continue to advance ion propulsion for missions to deep space and beyond using solar electric power. This form of propulsion is the fastest and most efficient to date.

There are many “absurd” theories that have become reality over the years of scientific research.  But for the near future, warp drive remains a dream.

If you would like to know more about the theories of interstellar flight, you should visit the Tau Zero Foundation.  Marc Millis, a former NASA Glenn physicist, founded the organization to consider revolutionary advancements in propulsion.

Past articles of warp drive found at this location have been archived.

Nancy Smith Kilkenny, SGT Inc.
NASA's Glenn Research Center

Last Updated: April 23, 2015
Editor: NASA Administrator

It's possible the recent spike of attention this discussion has brought to the topic in social networks (like reddit), has prompted some people to ask directly about it at NASA's official communication channels.

And they probably have to make a statement. If this statement you refer is the official position within NASA or just the public one at Glenn's, it's not yet clear (for me, at least).

But it's true such attention can make other parts of the institution pay more attention to these experiments. Which can be good or bad, depending on the level of tech readiness of the experiments happening at Eagle Works.
Title: Re: EM Drive Developments - related to space flight applications - Thread 2
Post by: Star One on 04/23/2015 06:51 pm
So it looks like any interest in warp drive has been for now shelved. But then warp drive isn't the EM drive & I imagine it's possible that people were getting the two mixed up and this is an act of clarification.
Title: Re: EM Drive Developments - related to space flight applications - Thread 2
Post by: Rodal on 04/23/2015 06:59 pm
So it looks like any interest in warp drive has been for now shelved. But then warp drive isn't the EM drive & I imagine it's possible that people were getting the two mixed up and this is an act of clarification.
Your statements are of course your personal view.  I regard the first statement as unwarranted speculation, as the announcement from NASA Glenn just pertains their own center's work on space propulsion, and also because NASA Eagleworks is not conducting engineering of an actual warp-drive vehicle but is instead conducting R&D interferometer tests for the purposes discussed in previous posts.

I regard Glenn's statement as a necessary sanitary statement to answer science-fiction fans that may be unaware of the difference between R&D and actual aerospace-engineering, and therefore may have completely unrealistic short-term expectations.
Title: Re: EM Drive Developments - related to space flight applications - Thread 2
Post by: tchernik on 04/23/2015 07:24 pm
Completely agree. I see the current state of research as similar to the state of atomic theory and radioactivity research in the XIX-th century: they started seeing some weird phenomena and started to advance some theories to explain them, without achieving full acceptance of it yet, but gathering evidence.

That is, far from any practical application yet.

But given the enthusiasm this creates  in some sector of the public, they can immediately make the conceptual jump and  think NASA is working on an Star-Trek-like propulsion engine or some such.
Title: Re: EM Drive Developments - related to space flight applications - Thread 2
Post by: Star One on 04/23/2015 07:36 pm
So it looks like any interest in warp drive has been for now shelved. But then warp drive isn't the EM drive & I imagine it's possible that people were getting the two mixed up and this is an act of clarification.
Your statements are of course your personal view.  I regard the first statement as unwarranted speculation, as the announcement from NASA Glenn just pertains their own center's work on space propulsion, and also because NASA Eagleworks is not conducting engineering of an actual warp-drive vehicle but is instead conducting R&D interferometer tests for the purposes discussed in previous posts.

I regard Glenn's statement as a necessary sanitary statement to answer science-fiction fans that may be unaware of the difference between R&D and actual aerospace-engineering, and therefore may have completely unrealistic short-term expectations.

I thought it was obvious that it's only my own very limited interpretation. I did manage to imply in what I did say that this and warp drive were not one & the same thing which is could be concern with the wider Internet.
Title: Re: EM Drive Developments - related to space flight applications - Thread 2
Post by: aceshigh on 04/23/2015 07:48 pm
So it looks like my conjecture that the device is creating some sort of spacial warp field is likely?

IF it´s creating a sort of warp of space-time... would it be safe to use on Earth when we increase it´s thrust by several orders of magnitude? Unless the warp distortion can be kept small but strong (think of a black hole) inside the engines...
Title: Re: EM Drive Developments - related to space flight applications - Thread 2
Post by: Rodal on 04/23/2015 08:03 pm
So it looks like my conjecture that the device is creating some sort of spacial warp field is likely?

IF it´s creating a sort of warp of space-time... would it be safe to use on Earth when we increase it´s thrust by several orders of magnitude? Unless the warp distortion can be kept small but strong (think of a black hole) inside the engines...
Whether it is creating a distortion of the space fabric of spacetime remains to be proven (hopefully they carry on their proposed experiments in vacuum to put to bed the issue of air refraction).  But just to entertain your speculative question, a back-of-the-envelope analysis quickly shows that the initial distortion of spacetime would be local and insignificant compared to the distortion of spatime produced by the Earth (which is immense in comparison).  There is no issues of safety related to "black holes".  No.  (And there are no issues of black-hole safety at CERN either).

EDIT: Remember that Dr. White (based on extra-3D-dimensional brane theory) showed that Viking-like masses are needed for his warp concept (Viking-like masses are negligible in comparison with the mass of the Earth).

Let's say that instead it takes a Jupiter-like mas-energy for the warp-drive to work, and that a future civilization can achieve that (a thousand years from now, at which point one has to wonder what is the need of humans in an interstellar spaceship if humans are still around instead of cyborgs  :) ) .  Then yes, any distortion of spacetime produced by a mass-energy comparable to (or exceeding) the mass of the Earth would be problematic to have near the Earth, but then there is a simple answer to that which is to have the warp-drive engaged at that energy far away from the Earth.

I hope that Interstellar producers followed Prof. Kip Thorne advise when they decided to place the wormhole for the Interstellar film all the way near Saturn. (And I hope that Prof. Thorne calculated the effect on Saturn's orbit and that it did not affect the Solar System  :)   )

If the location of the wormhole in Interstellar was not Prof. Thorne's advise, then you would have to engage your warp-drive's full power beyond Neptune, outside Kuipers belt.
Title: Re: EM Drive Developments - related to space flight applications - Thread 2
Post by: Mulletron on 04/23/2015 08:07 pm
They report using permanent magnets for B and a sinusoidally driven HVPS for E.


Recent discussion wrt connecting EM to warp reminded me of this. His youtube channel is amazing. I wish we had him here in this thread.



Is the following just "defined"

β f = - h /(1 - h )

without addressing what are the conditions for this "definition" to hold, and what is the validity of this "definition" ?

and then predictive conclusions are extracted from plugging in this "definition" into the exact solution?

I hopped on his youtube channel and asked him. At the 40 second mark he explains what is going on but not to the detail you're looking for.
Title: Re: EM Drive Developments - related to space flight applications - Thread 2
Post by: Rodal on 04/23/2015 08:12 pm
They report using permanent magnets for B and a sinusoidally driven HVPS for E.


Recent discussion wrt connecting EM to warp reminded me of this. His youtube channel is amazing. I wish we had him here in this thread.




Is the following just "defined"

β f = - h /(1 - h )

without addressing what are the conditions for this "definition" to hold, and what is the validity of this "definition" ?

and then predictive conclusions are extracted from plugging in this "definition" into the exact solution?

I hopped on his youtube channel and asked him. At the 40 second mark he explains what is going on but not to the detail you're looking for.

Thank you for going through the trouble and time to inquire this on his channel.

No, I find that what is said on the video at 40 sec is not a proper justification for the replacement of variables.  There are countless contrived examples in Mathematics and Physics were one can make such substitutions and show that they are unwarranted.  For example demonstrations that 1+1 does not equal 2.

If by making this (so far) unjustified substitution one finds exactly the same formula as Alcubierre's metric, then nothing new is added by the substitution.  On the other hand if by performing the substitution it leads one  to conclude something new, then any such new conclusion is unwarranted unless one can formally justify the substitution.
Title: Re: EM Drive Developments - related to space flight applications - Thread 2
Post by: aceshigh on 04/23/2015 08:37 pm
So it looks like my conjecture that the device is creating some sort of spacial warp field is likely?

IF it´s creating a sort of warp of space-time... would it be safe to use on Earth when we increase it´s thrust by several orders of magnitude? Unless the warp distortion can be kept small but strong (think of a black hole) inside the engines...
Whether it is creating a distortion of the space fabric of spacetime remains to be proven (hopefully they carry on their proposed experiments in vacuum to put to bed the issue of air refraction).  But just to entertain your speculative question, a back-of-the-envelope analysis quickly shows that the initial distortion of spacetime would be local and insignificant compared to the distortion of spatime produced by the Earth (which is immense in comparison).

even if the EM Drive thrust was increased to big levels?

Is it possible to calculate at which level of EM Drive thrust the local distortion of space time would be dangerous?


Quote
  There is no issues of safety related to "black holes".  No.  (And there are no issues of black-hole safety at CERN either).

well, I was not thinking in black holes being created (And the micro ones possibly created by CERN we know should evaporate in nanoseconds)

I mentioned black holes only in the sense of strong space-time warping in a small volume...

In that same sense, I would guess that despite Earth having a much stronger effect warping space time due to it´s mass, it´s spread over a large volume.

In other words... what I am thinking here is the difference between the distortion caused by a spoon of neutron star matter, weighing the same as the Everest, and Earth itself.

Earth itself has much more gravity, but it´s spread over a really big volume... while a spoon of neutron star matter equivalent to Mount Everest would condense all that space-time distortion over such a small volume...



The above was just an analogy for trying to make myself clear.

Now, Paul March talked about increasing thrust to 600 N. Let´s imagine a future EM Drive with a thrust equivalent to a chemical rocket... 600 thousand N. Would that create a significant space-time distortion?
Title: Re: EM Drive Developments - related to space flight applications - Thread 2
Post by: tchernik on 04/23/2015 08:48 pm
...

even if the EM Drive thrust was increased to big levels?

Is it possible to calculate at which level of EM Drive thrust the local distortion of space time would be dangerous?

...

I mentioned black holes only in the sense of strong space-time warping in a small volume...

In that same sense, I would guess that despite Earth having a much stronger effect warping space time due to it´s mass, it´s spread over a large volume.

In other words... what I am thinking here is the difference between the distortion caused by a spoon of neutron star matter, weighing the same as the Everest, and Earth itself.

Earth itself has much more gravity, but it´s spread over a really big volume... while a spoon of neutron star matter equivalent to Mount Everest would condense all that space-time distortion over such a small volume...



If you pardon my meddling: this is a very interesting question. I imagine one of the upcoming tasks of the research effort is precisely to characterize the field strength, its relation to input power and other parameters, and its changes with distance to the source, besides proving the field existence, of course.

If this field's strength is proportional to the input energy and it falls strongly with the distance to the device, then it may be safe to use  here on Earth. If it isn't, well, that would be interesting too (and probably also making it dangerous).
Title: Re: EM Drive Developments - related to space flight applications - Thread 2
Post by: Rodal on 04/23/2015 08:56 pm
....

Now, Paul March talked about increasing thrust to 600 N. Let´s imagine a future EM Drive with a thrust equivalent to a chemical rocket... 600 thousand N. Would that create a significant space-time distortion?

Look at my answer above, on what I wrote after "EDIT".

I would worry much more about particles in space being “swept up” into the warp bubble and focused into regions before and behind the ship. When your warp-driven spaceship decelerates, these particles will be released in  outbursts. The forward-facing particles can be very energetic — enough to destroy anyone at the destination directly in front of you.  So, if you are planning to go on a warp-drive beware that gamma ray and high energy particles released from your spaceship may destroy any beings on the location you are trying to reach  :)

Wormholes are more benign (if you can traverse them  :)
Title: Re: EM Drive Developments - related to space flight applications - Thread 2
Post by: CW on 04/23/2015 08:58 pm
I think the real point is that normal matter is pretty ineffective at producing a space warp, so you'd need e.g. a Jupiter mass equivalent for a decent space warp. If there were a highly effective synthetic way of inducing space warps, then that's exactly what the doctor ordered. I'm curious whether the higher-power regime of new test articles will produce any effects that correlate to the predicted dramatic thrust increase.
Title: Re: EM Drive Developments - related to space flight applications - Thread 2
Post by: CW on 04/23/2015 09:03 pm

[snip]

Look at my answer above, on what I wrote after "EDIT".

I would worry much more about particles in space being “swept up” into the warp bubble and focused into regions before and behind the ship. When your warp-driven spaceship decelerates, these particles will be released in  outbursts. The forward-facing particles can be very energetic — enough to destroy anyone at the destination directly in front of you.  So, if you are planning to go on a warp-drive beware that gamma ray and high energy particles may destroy any beings on the location you are trying to reach  :)

Wormholes are more benign (if you can traverse them  :)

Easy solution: Divide your travel way into hops with very short warp deactivation phases to get rid of unwanted particle collection.
Title: Re: EM Drive Developments - related to space flight applications - Thread 2
Post by: JasonAW3 on 04/23/2015 09:05 pm
So it looks like my conjecture that the device is creating some sort of spacial warp field is likely?

IF it´s creating a sort of warp of space-time... would it be safe to use on Earth when we increase it´s thrust by several orders of magnitude? Unless the warp distortion can be kept small but strong (think of a black hole) inside the engines...

At this point, all that the warp seems to be doing is an optical distortion in addition to causing motion in the test article.  However, the optical distortion is so small at this point, that they're not even sure that it is real, or an artifact of the instrumentation.  Even if it is an actual warp, the level of distortion and size are so small as to be effectively inconsequential.
Title: Re: EM Drive Developments - related to space flight applications - Thread 2
Post by: JasonAW3 on 04/23/2015 09:08 pm

[snip]

Look at my answer above, on what I wrote after "EDIT".

I would worry much more about particles in space being “swept up” into the warp bubble and focused into regions before and behind the ship. When your warp-driven spaceship decelerates, these particles will be released in  outbursts. The forward-facing particles can be very energetic — enough to destroy anyone at the destination directly in front of you.  So, if you are planning to go on a warp-drive beware that gamma ray and high energy particles may destroy any beings on the location you are trying to reach  :)

Wormholes are more benign (if you can traverse them  :)

Easy solution: Divide your travel way into hops with very short warp deactivation phases to get rid of unwanted particle collection.

I somehow suspect that the issue would be self resolving, as I suspect the particles and built up debris would flow around the warp in much the same way as the wake of a ship is formed.  It would tend to wrap around the craft and be left behind, much the same way that water wraps around and is left behind a submarine.
Title: Re: EM Drive Developments - related to space flight applications - Thread 2
Post by: Rodal on 04/23/2015 09:55 pm

[snip]

Look at my answer above, on what I wrote after "EDIT".

I would worry much more about particles in space being “swept up” into the warp bubble and focused into regions before and behind the ship. When your warp-driven spaceship decelerates, these particles will be released in  outbursts. The forward-facing particles can be very energetic — enough to destroy anyone at the destination directly in front of you.  So, if you are planning to go on a warp-drive beware that gamma ray and high energy particles may destroy any beings on the location you are trying to reach  :)

Wormholes are more benign (if you can traverse them  :)

Easy solution: Divide your travel way into hops with very short warp deactivation phases to get rid of unwanted particle collection.

I somehow suspect thatthey issue would be self resolving as I suspect teh particles and built up debris would flow around the warp in much the same way as the wake of a ship is formed.  It would tend to wrap around the craft and be left behind, much the same way that water wraps around and is left behind a submarine.
That's incorrect  :)  .  The effect has been modeled at the University of Sidney.  See this:

http://sydney.edu.au/news/science/397.html?newsstoryid=8790

http://arxiv.org/pdf/1202.5708

Quote
It would tend to wrap around the craft and be left behind, much the same way that water wraps around and is left behind a submarine.

That is an incorrect model for the warpdrive.  Water flows as an incompressible fluid around a submarine (*).  (Actually even air can be modeled, for practical purposes, to flow as an incompressible viscous fluid around aircraft traveling below the speed of sound).

On the contrary, the whole point about the Alcubierre warp drive is that space is compressed ahead of the spaceship and expanded behind it.  Not at all like an incompressible fluid.

_____

(*) there is possible cavitation (boiling of water as a result of pressure reduction rather than heat addition) in the propeller blades of course
Title: Re: EM Drive Developments - related to space flight applications - Thread 2
Post by: tchernik on 04/23/2015 10:32 pm

[snip]

Look at my answer above, on what I wrote after "EDIT".

I would worry much more about particles in space being “swept up” into the warp bubble and focused into regions before and behind the ship. When your warp-driven spaceship decelerates, these particles will be released in  outbursts. The forward-facing particles can be very energetic — enough to destroy anyone at the destination directly in front of you.  So, if you are planning to go on a warp-drive beware that gamma ray and high energy particles may destroy any beings on the location you are trying to reach  :)

Wormholes are more benign (if you can traverse them  :)

Easy solution: Divide your travel way into hops with very short warp deactivation phases to get rid of unwanted particle collection.

I wonder if this "broom" effect is actually a blessing in disguise, the kind that would make warp drive travel actually feasible (if it exists).

One of the biggest concerns of relativistic ships is the effect of dust and particles upon the traveling ship. The faster you go, the nastier the effect of these particles upon your ship. At some speeds blue shifting converts mere radio wave photons into gamma rays hitting you all the time. Up to the point of probably limiting the speed reachable by any ship, given the fact that even the weak CMB radiation would start heating up and melting the ship after certain fraction of c is reached.

 If the warp drive "focuses" the dirt and particles in the front of the ship, releasing them upon the field's deactivation, then that's maybe a solution to the problem of high energy impacts. And probably of dust up to certain grain size.
Title: Re: EM Drive Developments - related to space flight applications - Thread 2
Post by: CW on 04/23/2015 10:33 pm
I was just wondering.. there seems to be a lot of focus on dielectrics playing a big role in the hypothesized workings of such a drive. Taken literally, aren't also the subatomic particles like protons etc, which exist as quark combinations, somewhat comparable to dielectrics? Or speaking in general terms: If the QV can be seen as a dielectric (maybe the mother of all dielectrics), and if all kinds of existing 'real' particles are a subset of the QV, doesn't any kind of particle then implicitly have a special kind of dielectric behavior? And could this dielectric behaviour within the QV determine what we call mass?
Title: Re: EM Drive Developments - related to space flight applications - Thread 2
Post by: Rodal on 04/23/2015 10:37 pm

[snip]

Look at my answer above, on what I wrote after "EDIT".

I would worry much more about particles in space being “swept up” into the warp bubble and focused into regions before and behind the ship. When your warp-driven spaceship decelerates, these particles will be released in  outbursts. The forward-facing particles can be very energetic — enough to destroy anyone at the destination directly in front of you.  So, if you are planning to go on a warp-drive beware that gamma ray and high energy particles may destroy any beings on the location you are trying to reach  :)

Wormholes are more benign (if you can traverse them  :)

Easy solution: Divide your travel way into hops with very short warp deactivation phases to get rid of unwanted particle collection.

I wonder if this "broom" effect is actually a blessing in disguise, the kind that would make warp drive travel actually feasible (if it exists).

One of the biggest concerns of relativistic ships is the effect of dust and particles upon the traveling ship. The faster you go, the nastier the effect of these particles upon your ship. At some speeds blue shifting converts mere radio wave photons into gamma rays hitting you all the time. Up to the point of probably limiting the speed reachable by any ship, given the fact that even the weak CMB radiation would start heating up and melting the ship after certain fraction of c is reached.

 If the warp drive "focuses" the dirt and particles in the front of the ship, releasing them upon the field's deactivation, then that's maybe a solution to the problem of high energy impacts. And probably of dust up to certain grain size.

Interesting observation.  More like a vacuum cleaner effect than a broom (remember the film Spaceballs ?)  :)

(https://readneckreview.files.wordpress.com/2014/09/spaceballs_large_15.jpg)

A lot depends on how far ahead of the spearhead of the spacecraft is the nadir of the space contraction ahead of the spaceship.

You could send an e-mail to the guys at Sidney and ask them ==> even if they didn't model that already, they should be able to give you the best answer based on their model  :)
Title: Re: EM Drive Developments - related to space flight applications - Thread 2
Post by: Oberon_Command on 04/23/2015 11:08 pm
If the warp drive "focuses" the dirt and particles in the front of the ship, releasing them upon the field's deactivation, then that's maybe a solution to the problem of high energy impacts. And probably of dust up to certain grain size.

Maybe that's the solution to the drag problems associated with the Bussard ramjet, too, assuming you could somehow pull usable material from the "focus" in front of the starship.
Title: Re: EM Drive Developments - related to space flight applications - Thread 2
Post by: Mulletron on 04/23/2015 11:55 pm
So other than what is described in the links below, is there any proof that plane waves can travel slower or faster than c in vacuum?

http://www.sciencemag.org/content/347/6224/857
http://arxiv.org/abs/1411.3987
Title: Re: EM Drive Developments - related to space flight applications - Thread 2
Post by: Rodal on 04/24/2015 12:06 am
So other than what is described in the links below, is there any proof that plane waves can travel slower or faster than c in vacuum?

http://www.sciencemag.org/content/347/6224/857
http://arxiv.org/abs/1411.3987

A true plane wave is a theoretical construct, it is unbounded and hence it requires an infinite amount of energy.
Ditto for true Bessel beams (amplitude is described by a Bessel function of the first kind) .   Bessel beams can be approximated by focusing a Gaussian beam.  The article states that only plane waves are invariant traveling exactly at c.   Since plane waves are a theoretical construct, this effectively states that real waves are affected.  But it may not have any appreciable effect on Astronomy (see bottom of my post), but it may affect the EM Drive)

Quote
Our work highlights that, even in free space, the invariance of the speed of light only applies to plane waves

the delay in free space is very small though:

Quote
we show a reduction in the group velocity of photons in both a Bessel beam and photons in a focused Gaussian beam. In both cases, the delay is several micrometers over a propagation distance of ~1 meter




The following is known to be true:

Quote
....The speed of light in free space propagation is a fundamental quantity. It holds a pivotal role in the foundations of relativity and field theory, as well as in technological applications such as time-of-flight measurements, and radio and satellite communication. It has previously been experimentally established that single photons travel at the group velocity

That's correct.

This is what is new:
Quote
We have now shown that transverse structuring of the photon results in a decrease in the group velocity along the axis of propagation. The effect can be derived from a simple geometric argument, which is also supported by a rigorous calculation of the harmonic average of the group velocity. Beyond light, the effect observed will have applications to any wave theory, including sound waves and, potentially, gravitational waves

Hence pertinent to the EM Drive.  But again the relative difference is only ~ 10^(-6)

EDIT: on the other hand, the effect is biggest,when the diameter of the optics used is large and the distances are short, so it is unlikely to have any impact in astronomy, but it may have an impact on the EM Drive
Title: Re: EM Drive Developments - related to space flight applications - Thread 2
Post by: FutureStormtrooper on 04/24/2015 01:16 am

[snip]

Look at my answer above, on what I wrote after "EDIT".

I would worry much more about particles in space being “swept up” into the warp bubble and focused into regions before and behind the ship. When your warp-driven spaceship decelerates, these particles will be released in  outbursts. The forward-facing particles can be very energetic — enough to destroy anyone at the destination directly in front of you.  So, if you are planning to go on a warp-drive beware that gamma ray and high energy particles may destroy any beings on the location you are trying to reach  :)

Wormholes are more benign (if you can traverse them  :)

Easy solution: Divide your travel way into hops with very short warp deactivation phases to get rid of unwanted particle collection.

Or perhaps devise a way to siphon the particles off from within the bubble.

Speaking of which, something I've been wondering: what exactly would happen to any matter that were pushed from within the bubble out into the area of altered density space-time, say, a rod of some kind. What sort of forces/effects would the matter in the rod undergo?
Title: Re: EM Drive Developments - related to space flight applications - Thread 2
Post by: Stormbringer on 04/24/2015 01:31 am
comments:

The alcubierre metric requires a highly contorted space time topology. The versions that require less exotic energy were initially justified by having apertures smaller than the craft. In fact alcubierre's immediate successors had their apertures at microscopic dimensions.

The region inside the bubble is flat or nearly flat. Nothing inside the interior region is moving any faster than the speed it had before entering the entire warp region. The warp acceleration is in the skin region of the bubble. Things exterior to the bubble and it's skin are traveling at their normal space speed as well. Stuff that passes into the skin are given a the warp speed multiplier only while in the skin.

According to Dr white things that encounter the warp bubble shell are only accelerated for the duration of the transit through the skin region, not inside the bubble, not outside of the bubble. The description of the status of the ship  is no different to what happens to stuff entering the bubble while it is in route.

Since apparently stuff can pass into the bubble from outside there should be no accumulation of massive particles or photons. Also there should be no real space carry over of extra velocity. So even if there were an accumulation of matter or energy it would not be ammo for a relativistic rail gun effect.

Stuff in space can only go as straight as the local curvature allows. all stuff in space follows a curved path (geodetic.) Space around the warp bubble including the skin especially is really curved.  It seems to me that at stuff could not find the front end of the bubble let alone collect there; let alone fry stuff at the departure point or destination.

Apart from Dr Whites answer to the people that proposed the gamma ray bath hazard critique my own thoughts are most stuff would be diverted around the bubble either by the geodetic phenomenon or by plain old optical rules. the interfaces between regions would act as a lens for several reasons.

On another topic you can title lab safety: the energy Dr white is liable to have available to curve space is less than the energy in a big fire cracker. So lets say milligrams to a gram of TNT equivalents. In short any destructive effects from warping space at the level he is capable of in the near to medium term is comparable to an office prank.  :)  In other words; the minor curvature distortions he can provoke are not world threatening. They aren't even building threatening. They are on the level of slightly annoying the internal OSHA and fire safety additional duty guys.



Title: Re: EM Drive Developments - related to space flight applications - Thread 2
Post by: Rodal on 04/24/2015 01:48 am
On another topic you can title lab safety: the energy Dr white is liable to have available to curve space is less than the energy in a big fire cracker. So lets say milligrams to a gram of TNT equivalents. In short any destructive effects from warping space at the level he is capable of in the near to medium term is comparable to an office prank.  :)  In other words; the minor curvature distortions he can provoke are not world threatening. They aren't even building threatening. They are on the level of slightly annoying the internal OSHA and fire safety additional duty guys.

I would be surprised if anybody would disagree with that statement.  I think we all agree on that.





...
there should be no accumulation of massive particles or photons. also there should be no real space carry over of extra velocity. So even if there were an accumulation of matter or energy it would not be ammo for a relativistic rail gun effect. ... It seems to me that at stuff could not find the front end of the bubble let alone collect there; let alone fry stuff at the departure point or destination.

If you are referring to accumulation on the spacecraft itself.  The University of Sidney authors stated:

Quote
These results suggest that any ship using an Alcubierre warp drive carrying people would need shielding
to protect them from potential dangerously blueshifted particles during the journey

Shielding can be provided for blueshifted particles, so OK.




However, if you are referring to accumulation in the region of contracting space ahead of the spacecraft, those statements are in direct contradiction with the research conclusions at University of Sidney:

Quote
and any people at the destination would be gamma ray and high energy particle blasted into oblivion due to the extreme blueshifts for P+ region particles.

http://sydney.edu.au/news/science/397.html?newsstoryid=8790

http://arxiv.org/pdf/1202.5708

 if you are referring to accumulation in the region of contracting space ahead of the spacecraft, can you point out what assumptions made, or methods used, in the University of Sidney study are incorrect, in your view, that you arrive at a completely opposite conclusion?
Title: Re: EM Drive Developments - related to space flight applications - Thread 2
Post by: Stormbringer on 04/24/2015 01:54 am
The assumptions I posted are not mine but those of Dr White when he answered critics that brought forward the particle acceleration and accumulation objections to his theory. So I am not qualified to defend them. But Mr March certainly is. I just brought them up because it appeared the thread participants were unaware of them. :)

I am sure he will address this and this should be very interesting for all involved. :)
Title: Re: EM Drive Developments - related to space flight applications - Thread 2
Post by: Rodal on 04/24/2015 01:56 am
The assumptions I posted are not mine but those of Dr White when he answered critics that brought forward the particle acceleration and accumulation objections to his theory. So I am Not qualified to defend them. But Mr March certainly is. I just brought them up because it appeared the thread participants were unaware of them. :)

I am sure he will address this and this should be very interesting for all involved. :)

Is there a paper where Dr. White made those statements, or are they in a video you can link to?

I would like to see what he actually wrote or said on that matter.

EDIT: What I recall is Dr. White answering the objection to particles hitting the spacecraft at great speed, in which case his answer was correct, because high energy particles accumulate in the region of space that contracts in front of the spacecraft, and they never reach the spacecraft itself. 

Actually (hat tip to tchernik for the great insight), the research at University of Sidney supports Dr. White's answer, because  gamma ray and high energy particles don't hit the spacecraft.   The problem is just providing shielding against blueshifted particles.

I don't recall Dr. White addressing the different question of particles accumulating in front (not on the spacecraft itself) and  gamma ray and high energy particles released away from the spacecraft when the spacecraft decelerates, and "blasting beings into oblivion" ahead at their destination, as described by the University of Sidney research.


Thanks
Title: Re: EM Drive Developments - related to space flight applications - Thread 2
Post by: Stormbringer on 04/24/2015 01:58 am
The assumptions I posted are not mine but those of Dr White when he answered critics that brought forward the particle acceleration and accumulation objections to his theory. So I am Not qualified to defend them. But Mr March certainly is. I just brought them up because it appeared the thread participants were unaware of them. :)

I am sure he will address this and this should be very interesting for all involved. :)

Is there a paper where Dr. White made those statements, or are they in a video you can link to?

I would like to see what he actually wrote or said on that matter.

Thanks
Yes. I saw it in a video of one of his presentations, -I think it was a couple of years ago. If I can find it I will certainly provide a link :)

EDIT:  Going through the videos now. I'm half way through the first video.
Title: Re: EM Drive Developments - related to space flight applications - Thread 2
Post by: aceshigh on 04/24/2015 03:45 am
The assumptions I posted are not mine but those of Dr White when he answered critics that brought forward the particle acceleration and accumulation objections to his theory. So I am Not qualified to defend them. But Mr March certainly is. I just brought them up because it appeared the thread participants were unaware of them. :)

I am sure he will address this and this should be very interesting for all involved. :)

Is there a paper where Dr. White made those statements, or are they in a video you can link to?

I would like to see what he actually wrote or said on that matter.

Thanks
Yes. I saw it in a video of one of his presentations, -I think it was a couple of years ago. If I can find it I will certainly provide a link :)

EDIT:  Going through the videos now. I'm half way through the first video.

what videos? The one I posted or other ones?


@Dr Rodal: is the release of particles concentrated and focused or would they spread? If so, would it be a matter of turning off the warp field at a safe enough distance from an inhabited planet?

obviously, you might want to turn off the warp field the closest possible to a "target".
Title: Re: EM Drive Developments - related to space flight applications - Thread 2
Post by: zen-in on 04/24/2015 04:08 am
I would like to see more credible evidence this magnetic capacitive device generates a warp field; if that is what has been claimed.    The White-Juday Warp Field Interferometer pictures I have seen is simply some Edmund Scientific mirrors and beamsplitter mounted on flimsy stands on an open optical table.    The fringes imaged by the camera will move up or down one fringe when the path difference changes by +/- 632.8/2 nM.  There are all kinds of things that will make that happen with this imprecise setup.   Radiative heating of a mirrors surface from an object that is just a few degrees hotter, vibrations, and electrical noise are a few of the possible causes.   This is like the em-drive experiments:  No attempts have been made to rule out alternative explanations.   For example what happens when the DUT is placed outside the laser beam?    Does the "warp field" displacement still occur.  How sensitive is the interferometer to nearby objects that are being heated?    Most optical measurements are done with a dual beam setup, to cancel out drift and other common mode effects.   The measurement accuracy of White-Juday Warp Field Interferometer is questionable because there is no absolute positional reference like the white light interferogram used in FTIR instruments.    So when fringes move was it just one fringe or was it several hundred?   How is the aliasing effect of the camera factored into this?
Title: Re: EM Drive Developments - related to space flight applications - Thread 2
Post by: Stormbringer on 04/24/2015 04:09 am



what videos? The one I posted or ot
her ones?




I started with the Icarus  sponsored International Starship Congress symposium of 2013. it is not in that one. i am proceeding through the videos as fast as i can with my feeble attention span. The next one is over an hour long. I am ten minutes through that one and taking a break from it at the moment.

https://www.youtube.com/watch?v=Wokn7crjBbA
Title: Re: EM Drive Developments - related to space flight applications - Thread 2
Post by: aceshigh on 04/24/2015 05:41 am
it seems some people are getting excited by the new Star Wars trailers  ;)

remember however that Star Wars uses "hyperspace", not warp drives (in the movies however, they only use the term "lightspeed")

I will be really glad if the EM Drive proves to work, no warp drive yet for a few centuries.

Plenty of stuff to do in the Solar System before we have the need to venture forth.
Title: Re: EM Drive Developments - related to space flight applications - Thread 2
Post by: ThinkerX on 04/24/2015 06:14 am
Quote
Plenty of stuff to do in the Solar System before we have the need to venture forth.

Well, even the sub-light version of the EM drive opens up the solar systems within a few dozen light-years to exploration and colonization, if there's anyplace worth visiting.
Title: Re: EM Drive Developments - related to space flight applications - Thread 2
Post by: Stormbringer on 04/24/2015 06:31 am
the 4 newton version of the QVPT according to calculations using the copernicus software can send a probe to alpha proxima in 29 years of flight time; including acceleration and deacceleration time.

EDIT finished a second video. nothing yet. going on to a third video.
Title: Re: EM Drive Developments - related to space flight applications - Thread 2
Post by: ThinkerX on 04/24/2015 06:59 am
Quote
the 4 newton version of the QVPT according to calculations using the copernicus software can send a probe to alpha proxima in 29 years of flight time; including acceleration and deacceleration time.

I remember reading that.

But, going from some of the near exponential charts and graphs 'Star Drive' posted here some pages back, I suspect that projection is overly conservative.  I suspect two or three decades travel time would get the star ship out to twenty or thirty light years if those new numbers are anywhere near right. 

And even if I'm off on that...well, with missions past Alpha Centauri, the starship would keep accelerating to the journey's midpoint, which means that a trip to Tau Ceti would take roughly the same amount of time as the Alpha Centauri voyage - at least for the crew.  Relativistic effects starting to kick in with longer acceleration.
Title: Re: EM Drive Developments - related to space flight applications - Thread 2
Post by: dustinthewind on 04/24/2015 07:01 am
In regards to the death blast released when the ship stops due to collapse of the gravity well in front of the ship.  It was suggested a good idea it to make a pre-top before arriving.  So if anyone was using this then would we see what would appear to be a gamma ray burst?  I wonder if we could recognize it from other types of gamma ray bursts. 

I am guessing the gravity well in front of the ship pulls it along while the negative gravity well behind the ship pushes it.  I could imagine light trying to escape a gravity well getting red shifted and a light falling into a gravity well being blue shifted I suppose.  Making the gravity well disappear suddenly I suppose the particles or light no longer need to escape the gravity well and remain up-shifted (seems odd)?  I'm not sure why the ship itself wouldn't be blasted as the particles that had their potential energy converted to kinetic suddenly go every direction as the gravity well disappears but I suppose if so as previously stated there is shielding. 

So what kind of warp do they think they might be observing in the chamber?  Is it a positive gravity signature or a negative signature?  How would they replicate a negative gravity well?  Does it osculate between negative and positive?  Or is it just positive to null and positive space time waves? 

Maybe I would be correct in guessing there appears to be a shortening of distance for the laser which is similar to the heating lowering the density of air which is why they would worry about the air heating in the cavity.  I know as previously stated the cavity retains the heat so testing on, off, on, may eliminate the problem as the cavity remains hot.  Shortening of distance makes it look as though it is negative energy density I would guess.  But inside just a regular cylindrical cavity is what really baffles me.  If the field permeates and even effects the cavity could they detect the change in its atomic bonds?  Would they become more energetic and change the cavity characteristics?  Would it speed up a very accurate clock and could that be a test method?  Would the mass of the object change?
Title: Re: EM Drive Developments - related to space flight applications - Thread 2
Post by: Stormbringer on 04/24/2015 07:21 am
Assuming gamma ray bursts can be associated with exit from warp then the magnitude should be considerably smaller than known cosmic sources but still pretty bright. They should appear at semi-regular intervals near stars (probably just outside any planetary system) and close to each other but not quite at the same locations. This variability should make ruling out natural sources fairly easy.

I suppose if one is given that the blue shifted accumulated photon thing is real then you could calculate the amount of energy involved in a given warp traverse between star system A and star system B for a given mass of space ship. you could then determine if a gamma ray event would be detectable with current instruments and at what range. then you could watch likely stars within that range with the appropriate instrument.
Title: Re: EM Drive Developments - related to space flight applications - Thread 2
Post by: CW on 04/24/2015 07:31 am
Another question regarding a possible 'quantum wake', caused by the drive's operation:

In water for instance, we can observe longer lasting turbulences which are caused by propeller engines. I'm aware that the hypothesized MHD-like behavior of the QV is only roughly comparable to a fluid, but I'm still curious whether there would theoretically remain some sort of detectable QV 'space turbulences' when a QV-driven spaceship passes a location.
Title: Re: EM Drive Developments - related to space flight applications - Thread 2
Post by: Stormbringer on 04/24/2015 08:32 am
Partial success. I found a unfortunately trimmed edit of a presentation where he started to explain it. But the answer was truncated by the end of the clip.

https://www.youtube.com/watch?v=vKTgNCGhq9Y

starts at 53:46 seconds and ends before Dr White has finished his answer.  However it is probable that better clips of this presentation exist. I'll look for one tomorrow.



Title: Re: EM Drive Developments - related to space flight applications - Thread 2
Post by: CW on 04/24/2015 09:13 am
That's a good point to make: As the hypothesized warp region is dynamically created by the dPhi/dt dynamics, there is no constant warp. Hence, the 'boost factor' is only the time averaged or integrated value of the warp. Larger amounts of particles should hence not be able to accumulate. The real operation of the drive would be much more akin to many small 'boost' periods per second, like small warp jumps. Is that correct?
Title: Re: EM Drive Developments - related to space flight applications - Thread 2
Post by: niihelium on 04/24/2015 10:54 am

When considering the use of EM-drives in the making of warp-drives the attached two papers might be of interest.

In the meantime, back to figuring out how to reliably drive an EM-drive...

Edit:  You might also like to read Sonny's Warp-field Mechanics 101 and 102 articles.
http://ntrs.nasa.gov/archive/nasa/casi.ntrs.nasa.gov/20130011213.pdf

Best, Paul M.

Paul M,

Thanks a lot. This papers very interesting, but little old. And what about latest papers/presentations which dated 04/04/2015, 04/09/2015 and 04/15/2015. I've seen only few pages from them and it would be very interesting to read full version. Can you share full document, please. In attachment are this pages I've seen.

Best regards,
Nikita Unkovsky
Title: Re: EM Drive Developments - related to space flight applications - Thread 2
Post by: Rodal on 04/24/2015 12:07 pm
Partial success. I found a unfortunately trimmed edit of a presentation where he started to explain it. But the answer was truncated by the end of the clip.

https://www.youtube.com/watch?v=vKTgNCGhq9Y

starts at 53:46 seconds and ends before Dr White has finished his answer.  However it is probable that better clips of this presentation exist. I'll look for one tomorrow.

Great job at finding this video.  You were correct indeed that this question had been asked.  Unfortunately, the video is clipped off before one can hear Dr. White's full answer.

Caveats:

1) Does the video and therefore the question and answer predate the publishing of the paper by University of Sidney ?, Did Dr. White have an opportunity to read the University of Sidney's paper at the time that he answered this question?

http://sydney.edu.au/news/science/397.html?newsstoryid=8790

http://arxiv.org/pdf/1202.5708

If Dr. White didn't have an opportunity to read the University of Sidney's paper, we cannot really expect Dr. White to give an adequate answer to a paper he hadn't read, as I imagine Dr. White would have to conduct his own computations to arrive at a different answer than the University of Sidney researchers.

2) The University of Sidney researchers are not simply making an argument, they have conducted detailed numerical computations for the field accumulation of different types of particles and published it in a respected peer-reviewed journal:  Physical Review D published by the American Physical Society dealing with  particle physics, field theory, gravitation, and cosmology.

I invite the readers to actually read the paper and comment on the paper's computations and graphs:

http://arxiv.org/pdf/1202.5708
Title: Re: EM Drive Developments - related to space flight applications - Thread 2
Post by: Rodal on 04/24/2015 12:46 pm
That's a good point to make: As the hypothesized warp region is dynamically created by the dPhi/dt dynamics, there is no constant warp. Hence, the 'boost factor' is only the time averaged or integrated value of the warp. Larger amounts of particles should hence not be able to accumulate. The real operation of the drive would be much more akin to many small 'boost' periods per second, like small warp jumps. Is that correct?
The paper by the University of Sidney researchers carefully examined (with numerical computations) the paths of null and massive particles with a range of initial velocities from -c to c interacting with the warp bubble travelling at a range of globally subluminal and superluminal velocities on both constant and variable velocity paths. 

The dynamics of the warp bubble are nonlinear, hence numerical computations are needed in order to find out what happens to the particles. 

Quote from: Brendan McMonigal
Even for very short journeys the energy released is so large that you would completely obliterate anything in front of you,

It is not clear to me whether Dr. White had an opportunity to review the University of Sidney's work when he answered that question.

Has anyone else (other than the University of Sidney researchers) performed computations and arrived at a different answer?  It would be interesting if other numerical computations have been performed during the last 3 years.
Title: Re: EM Drive Developments - related to space flight applications - Thread 2
Post by: Stormbringer on 04/24/2015 01:05 pm
Dr Rodal:

It was my impression that Dr White made his defense after the critics which probably included the Sidney critques came out; But that is just my memory of it which could be faulty. I am not sure this video i found is even the one my memory is from because i remember him going into not mathematical but verbal detail as to why particle acceleration and photon blue shift were not issues with his warp drive. but it doe give a partial defense.

So here is my question to the forum:  Does anyone recognize  the venue that video was taken from so I can refine my search terms for google? We need a better clip than what i found.
Title: Re: EM Drive Developments - related to space flight applications - Thread 2
Post by: aceshigh on 04/24/2015 02:08 pm
Dr Rodal:

It was my impression that Dr White made his defense after the critics which probably included the Sidney critques came out; But that is just my memory of it which could be faulty. I am not sure this video i found is even the one my memory is from because i remember him going into not mathematical but verbal detail as to why particle acceleration and photon blue shift were not issues with his warp drive. but it doe give a partial defense.

So here is my question to the forum:  Does anyone recognize  the venue that video was taken from so I can refine my search terms for google? We need a better clip than what i found.

here, it´s about 4 minutes longer. You can jump to 54:00
https://www.youtube.com/watch?v=9M8yht_ofHc
Title: Re: EM Drive Developments - related to space flight applications - Thread 2
Post by: Rodal on 04/24/2015 02:34 pm
Dr Rodal:

It was my impression that Dr White made his defense after the critics which probably included the Sidney critques came out; But that is just my memory of it which could be faulty. I am not sure this video i found is even the one my memory is from because i remember him going into not mathematical but verbal detail as to why particle acceleration and photon blue shift were not issues with his warp drive. but it doe give a partial defense.

So here is my question to the forum:  Does anyone recognize  the venue that video was taken from so I can refine my search terms for google? We need a better clip than what i found.

here, it´s about 4 minutes longer. You can jump to 54:00
https://www.youtube.com/watch?v=9M8yht_ofHc

Thank you @aceshigh for taking the time to find this clip that provides the full answer, as well as the time and place of the presentation.  From Dr. White's answer, it appears to me (it is always difficult to know since the speaker had only a small amount time to answer) that Dr. White is addressing the papers that were written prior to the University of Sidney's work.  It appears that what is being addressed are the interactions examined by Clark et al. and the brief discussion by  Pfenning and Ford in the Journal Classical and Quantum Gravity, discussing only the interaction between a warp bubble travelling at constant velocity and Eulerian matter, that is matter stationary in the rest frame of the origin/destination of the ship. 

M. J. Pfenning and L. H. Ford, Classical and Quantum Gravity 14, 1743 (1997)

C. Clark, W. A. Hiscock, and S. L. Larson, Classical and Quantum Gravity 16, 3965 (1999)

The questioner did not mention the University of Sidney's research in his question, nor did the questioner have a follow-up question.  Thus it is perfectly reasonable to suppose that Dr. White may have thought that the question was referring to the interactions examined by Clark and briefly by Pfenning and Ford in the late 1990's.

In contrast, the University of Sidney's team appears to be the only one that has provided a detailed analysis of the interactions of null and massive particles with an Alcubierre warp bubble at both constant and variable velocity, via the calculation of representative geodesics through Alcubierre spacetime.  They numerically calculated in detail, the paths of particles with a range of initial velocities from -c to c interacting with the warp bubble travelling at a range of globally subluminal and superluminal velocities on both constant and variable velocity paths.   The dynamics of the warp bubble are nonlinear, hence numerical computations are needed in order to find out what happens to the particles. 



As an aside, that I bet we all agree on, I don't this issue as a critique from the audience and I don't see Dr. White being in a defensive position.  R&D scientists goal is always to find the truth, whatever that is. I see it as an interesting question, and I see Dr. White earnestly answering the question to the best of his knowledge at the time.

I agree with Stormbringer that it would be very interesting to hear Dr. White's opinion on this matter today, and whether he or other researchers have conducted calculations on this matter.
Title: Re: EM Drive Developments - related to space flight applications - Thread 2
Post by: MazonDel on 04/24/2015 02:37 pm
As far as the issue with the warp drive gamma burst, from my understanding of the topic, the amount of build up has to do with both the average amount of particulate matter in your way (random gasses, dust motes, even possibly small rocks) and the length of time in warp. (Speed might also factor into this, but I cannot remember to be honest.) So a boost through 10,000 miles of space past lunar orbit might not have a whole lot of gamma burst to it despite the density provided from solar winds. But a boost of the full 4.2-ish light years from Alpha Centauri COULD be quite the lethal light show.

As such, it seems like one of ways we'd handle this in the early days is just a matter of protocol. Any approaches to a star system end with the arriving ship coming out of the main leg of their warp fairly off in the distance to harmlessly shove off the burst. The advised distance for this would be an 'outer warp limit'. Until more information is known about the new star system, and the nature of the gamma burst itself, it might also be advisable to attempt to come in pointed to some degree off the target systems ecliptic. Following a short period of observation (verifying your position mostly), one could make a few shorter hops as they come in, to try and keep the burst minimized. Finally, it would seem that unless the 'inner warp limit' around the destination planet was quite large, the advisable thing to do is for the arriving ship to actually come out of warp having driven PAST the planet it is attempting to reach. This ensures that regardless of gamma burst, Earth and its satellites are not in any sort of peril. Speaking of the satellites, my own recommendation for the inner warp limit would be a distance a bit beyond the current useful orbits for satellites, just to ensure we don't bombard our infrastructure with gamma bursts unnecessarily.

In the early days of warp travel, this is probably quite sufficient to deal with any issues. Though as more and more warp craft are built, some amount of effort would need to be expended to deal with orbital traffic control to ensure that an arriving ship doesn't accidentally blast a recently arrived ship.

Chances are decent we'd set up some sort of scheduled window per the ships flight plan such that once you arrive at the outer warp limit, there is a radio ready to chat with you to let you know what flight path to take in and at what time intervals.

Just my thoughts.
Title: Re: EM Drive Developments - related to space flight applications - Thread 2
Post by: tchernik on 04/24/2015 03:15 pm
As far as the issue with the warp drive gamma burst, from my understanding of the topic, the amount of build up has to do with both the average amount of particulate matter in your way (random gasses, dust motes, even possibly small rocks) and the length of time in warp. (Speed might also factor into this, but I cannot remember to be honest.) So a boost through 10,000 miles of space past lunar orbit might not have a whole lot of gamma burst to it despite the density provided from solar winds. But a boost of the full 4.2-ish light years from Alpha Centauri COULD be quite the lethal light show.

As such, it seems like one of ways we'd handle this in the early days is just a matter of protocol. Any approaches to a star system end with the arriving ship coming out of the main leg of their warp fairly off in the distance to harmlessly shove off the burst. The advised distance for this would be an 'outer warp limit'. Until more information is known about the new star system, and the nature of the gamma burst itself, it might also be advisable to attempt to come in pointed to some degree off the target systems ecliptic. Following a short period of observation (verifying your position mostly), one could make a few shorter hops as they come in, to try and keep the burst minimized. Finally, it would seem that unless the 'inner warp limit' around the destination planet was quite large, the advisable thing to do is for the arriving ship to actually come out of warp having driven PAST the planet it is attempting to reach. This ensures that regardless of gamma burst, Earth and its satellites are not in any sort of peril. Speaking of the satellites, my own recommendation for the inner warp limit would be a distance a bit beyond the current useful orbits for satellites, just to ensure we don't bombard our infrastructure with gamma bursts unnecessarily.

In the early days of warp travel, this is probably quite sufficient to deal with any issues. Though as more and more warp craft are built, some amount of effort would need to be expended to deal with orbital traffic control to ensure that an arriving ship doesn't accidentally blast a recently arrived ship.

Chances are decent we'd set up some sort of scheduled window per the ships flight plan such that once you arrive at the outer warp limit, there is a radio ready to chat with you to let you know what flight path to take in and at what time intervals.

Just my thoughts.

While I really like the Jerry Pournelle's book-like rules for interstellar travel you describe (it allows interstellar  travel to be full of interplanetary sight-seeing and romantic adventures while traveling), I think it's too early to know if such thing would be necessary.

Some people have pointed out the warp drive source, assuming it is real, may not create a permanent warp field as long as the drive is in operation, as we often assume, but rather it could produce an oscillating one.

In such a case, the 'boost' factor can simply be an average of the oscillating warp field instantaneous boost, and the 'vacuum cleaner' effect would be very limited, because the incoming particles and dust could gather only for very short periods and they would be purged shortly thereafter, when the field becomes low enough in intensity (or briefly goes to zero).

There's still a long way before we know the actual properties of this presumed phenomenon, but we can speculate in the meantime, of course.
Title: Re: EM Drive Developments - related to space flight applications - Thread 2
Post by: JasonAW3 on 04/24/2015 03:28 pm
So it looks like any interest in warp drive has been for now shelved. But then warp drive isn't the EM drive & I imagine it's possible that people were getting the two mixed up and this is an act of clarification.
Your statements are of course your personal view.  I regard the first statement as unwarranted speculation, as the announcement from NASA Glenn just pertains their own center's work on space propulsion, and also because NASA Eagleworks is not conducting engineering of an actual warp-drive vehicle but is instead conducting R&D interferometer tests for the purposes discussed in previous posts.

I regard Glenn's statement as a necessary sanitary statement to answer science-fiction fans that may be unaware of the difference between R&D and actual aerospace-engineering, and therefore may have completely unrealistic short-term expectations.

Personally, I'm awaiting an unequivocal "Chicago Pile" moment, instead of near misses and uncertain results.
Title: Re: EM Drive Developments - related to space flight applications - Thread 2
Post by: Rodal on 04/24/2015 03:37 pm
So it looks like any interest in warp drive has been for now shelved. But then warp drive isn't the EM drive & I imagine it's possible that people were getting the two mixed up and this is an act of clarification.
Your statements are of course your personal view.  I regard the first statement as unwarranted speculation, as the announcement from NASA Glenn just pertains their own center's work on space propulsion, and also because NASA Eagleworks is not conducting engineering of an actual warp-drive vehicle but is instead conducting R&D interferometer tests for the purposes discussed in previous posts.

I regard Glenn's statement as a necessary sanitary statement to answer science-fiction fans that may be unaware of the difference between R&D and actual aerospace-engineering, and therefore may have completely unrealistic short-term expectations.

Personally, I'm awaiting an unequivocal "Chicago Pile" moment, instead of near misses and uncertain results.
The Chicago pile (the world's first artificial nuclear reactor) had no radiation shielding and no cooling system of any kind.  Enrico Fermi described the apparatus as "a crude pile of black bricks and wooden timbers." It was made of a large amount of graphite and uranium, with "control rods" of cadmium, indium, and silver. The Atomic Energy Commission later noted, that the real "gamble" was conducting "a possibly catastrophic experiment in one of the most densely populated areas of the nation!"

Thus, Eagleworks probably should adopt a better analogy than the "Chicago Pile moment"   :)

Perhaps we should help them find a more politically acceptable analogy  :)
Title: Re: EM Drive Developments - related to space flight applications - Thread 2
Post by: tchernik on 04/24/2015 03:44 pm
So it looks like any interest in warp drive has been for now shelved. But then warp drive isn't the EM drive & I imagine it's possible that people were getting the two mixed up and this is an act of clarification.
Your statements are of course your personal view.  I regard the first statement as unwarranted speculation, as the announcement from NASA Glenn just pertains their own center's work on space propulsion, and also because NASA Eagleworks is not conducting engineering of an actual warp-drive vehicle but is instead conducting R&D interferometer tests for the purposes discussed in previous posts.

I regard Glenn's statement as a necessary sanitary statement to answer science-fiction fans that may be unaware of the difference between R&D and actual aerospace-engineering, and therefore may have completely unrealistic short-term expectations.

Personally, I'm awaiting an unequivocal "Chicago Pile" moment, instead of near misses and uncertain results.

For me, it is clear that 'Chicago Pile' moment could come from the spectroscopy studies of the potential warp drive. That is an extremely hard to dismiss bit of physical evidence, given the right experimental conditions and successful replications are there.

This interferometry setup actually strikes me as very similar to the Michelson-Morley experiment, in terms of physical simplicity, hard to ignore results and far reaching implications...

Even if we couldn't use it for making any drive within a century, that bit of evidence is the one that could establish the existence of the phenomenon once and for all.

Edit: I'm aware the White-Juday interferometer experiment was meant to be a 'Chicago Pile' moment since the very beginning, I just want to point out the accidental and serendipitous nature of this new variant of that experiment using Emdrive-like resonating cavities.
Title: Re: EM Drive Developments - related to space flight applications - Thread 2
Post by: cfs on 04/24/2015 04:03 pm
Speaking of establishing phenomena - does anyone have a good reference on how using an oscillating EM field is supposed to induce spacetime contractions?

I've been doing a little background reading, specifically:
http://ntrs.nasa.gov/archive/nasa/casi.ntrs.nasa.gov/20110015936.pdf (http://ntrs.nasa.gov/archive/nasa/casi.ntrs.nasa.gov/20110015936.pdf)
http://www.earthtech.org/publications/davis_STAIF_conference_2.pdf (http://www.earthtech.org/publications/davis_STAIF_conference_2.pdf)
http://members.shaw.ca/mike.anderton/WarpDrive.pdf (http://members.shaw.ca/mike.anderton/WarpDrive.pdf)
http://link.springer.com/article/10.1023%2FA%3A1026299010288 (http://link.springer.com/article/10.1023%2FA%3A1026299010288)
http://www.gravityresearchfoundation.org/pdf/awarded/2004/loup_santos.pdf (http://www.gravityresearchfoundation.org/pdf/awarded/2004/loup_santos.pdf)

I saw this plot :
Is it just to push the peak past the 1/f noise? (http://forum.nasaspaceflight.com/index.php?action=dlattach;topic=36313.0;attach=825749;image[/url)
Title: Re: EM Drive Developments - related to space flight applications - Thread 2
Post by: Stormbringer on 04/24/2015 04:13 pm
I know that Dr white argued in one Q&A session at the end of one of his presentations that the negative energy requirement is satisfied in the QV so he didn't need any real world negative energy to make his micro warp experiment:  but i cannot help thinking that a warp made of real negative energy would be stronger and easier to detect?

I'm thinking about that ring laser thing (from the optical diametric drive research) again where the wave form is formed in such a way that there are always transient negative energy regions in the waveform.

Since the waveform itself travels around in a circle and is effectively infinite in length as opposed to running a few whatever units to a few feet in a linear set up (and there are two of them stacked so as to potentially interact with each other)...

I'm thinking that the ring set up would have a lot more negative stuff with which to produce a stronger space distortion than hoping some is available in the randomness of the vacuum. Would be a better test article... something with a sigma of "all you critics can get out of my face."
Title: Re: EM Drive Developments - related to space flight applications - Thread 2
Post by: CW on 04/24/2015 04:21 pm
So it looks like any interest in warp drive has been for now shelved. But then warp drive isn't the EM drive & I imagine it's possible that people were getting the two mixed up and this is an act of clarification.
Your statements are of course your personal view.  I regard the first statement as unwarranted speculation, as the announcement from NASA Glenn just pertains their own center's work on space propulsion, and also because NASA Eagleworks is not conducting engineering of an actual warp-drive vehicle but is instead conducting R&D interferometer tests for the purposes discussed in previous posts.

I regard Glenn's statement as a necessary sanitary statement to answer science-fiction fans that may be unaware of the difference between R&D and actual aerospace-engineering, and therefore may have completely unrealistic short-term expectations.

Personally, I'm awaiting an unequivocal "Chicago Pile" moment, instead of near misses and uncertain results.
The Chicago pile (the world's first artificial nuclear reactor) had no radiation shielding and no cooling system of any kind.  Enrico Fermi described the apparatus as "a crude pile of black bricks and wooden timbers." It was made of a large amount of graphite and uranium, with "control rods" of cadmium, indium, and silver. The Atomic Energy Commission later noted, that the real "gamble" was conducting "a possibly catastrophic experiment in one of the most densely populated areas of the nation!"

Thus, Eagleworks probably should adopt a better analogy than the "Chicago Pile moment"   :)

Perhaps we should help them find a more politically acceptable analogy  :)

Pile of Awesome moment? ;)
Title: Re: EM Drive Developments - related to space flight applications - Thread 2
Post by: Rodal on 04/24/2015 05:24 pm
...

I'm thinking about that ring laser thing (from the optical diametric drive research) again where the wave form is formed in such a way that there are always transient negative energy regions in the waveform.

...
What ring laser thing possessing " always transient negative energy regions in the waveform" are you referring to?

Link please  :)
Title: Re: EM Drive Developments - related to space flight applications - Thread 2
Post by: Stormbringer on 04/24/2015 06:27 pm
...

I'm thinking about that ring laser thing (from the optical diametric drive research) again where the wave form is formed in such a way that there are always transient negative energy regions in the waveform.

...
What ring laser thing possessing " always transient negative energy regions in the waveform" are you referring to?

Link please  :)

http://phys.org/news/2013-10-optical-diametric.html

http://www.creol.ucf.edu/Research/Publications/7155.pdf

http://www.nature.com/nphys/journal/v9/n12/full/nphys2777.html?WT.ec_id=NPHYS-201312

"Always transient"  refers to the way squeezed light makes negative energy. It's always for a tiny fraction of the over all cycle and the positive side always dwarfs it. But in a circular track... there should be a tiny bit present at all times and in all parts of the track. Or maybe not as tiny as it would be in a finite length course and there are two circular stacked circular tracks in those articles. Maybe the two fields can constructively reinforce the negative components or destructively interfere with the positive portion selectively. i don't know much about the physics. I'm just "using the force."

EDIT:  So here is a question I have about squeezed light. what would happen if a portion of the circular track was made to slow light down? I have read about certain research slowing light down under the SOL in a vacuum. You have two rings... one of the trackways could have unimpeded squeezed light and the other had at least a portion with a go slow zone. Could the tracks be juxtaposed in such a way that the negative regions of the waveform were close enough to sum together? And could there be a tidal pile up of negative energy caused by the go slow zone?

Title: Re: EM Drive Developments - related to space flight applications - Thread 2
Post by: Star-Drive on 04/24/2015 06:48 pm

When considering the use of EM-drives in the making of warp-drives the attached two papers might be of interest.

In the meantime, back to figuring out how to reliably drive an EM-drive...

Edit:  You might also like to read Sonny's Warp-field Mechanics 101 and 102 articles.
http://ntrs.nasa.gov/archive/nasa/casi.ntrs.nasa.gov/20130011213.pdf

Best, Paul M.

Paul M,

Thanks a lot. This papers very interesting, but little old. And what about latest papers/presentations which dated 04/04/2015, 04/09/2015 and 04/15/2015. I've seen only few pages from them and it would be very interesting to read full version. Can you share full document, please. In attachment are this pages I've seen.

Best regards,
Nikita Unkovsky

Nikita:

Dr. White's latest Warp Field Interferometer (WFI) results are just in PowerPoint file format at the moment, for they are just used for keeping the Eagleworks' technical and management teams aware of the latest developments.  So if you want a copy of same you need to send a note to Dr. White asking for it.  In the meantime Dr. White cautioned me yesterday that I need to be more careful in declaring we've observed the first lab based space-time warp signal and rather say we have observed another non-negative results in regards to the current still in-air WFI tests, even though they are the best signals we've seen to date.  It appears that whenever we talk about warp-drives in our work in a positive way, the general populace and the press reads way too much into our technical disclosures and progress.

Next find attached Sonny's latest WFI data set number-1 analysis that utilizes all 28.5k period samples instead of just a very few arbitrarily selected pixel addresses we used before so as to minimize the compute times.  And I'm glad that Dr. White was an ex-game programmer in his youth because his programming expertise was really needed for this problem to be able to analysis this large set in less than the 100 years his initial cut at is indicated.  In fact it is now running in less than 4-to-6 hours on a Windows 7.0 PC with an Intel i5 in it.  Be that as it may, you will note that the assumed in-air space-time compression signal is still there with now a much improved signal to noise ratio than the previous example I provided.

Best,  Paul M.
Title: Re: EM Drive Developments - related to space flight applications - Thread 2
Post by: Rodal on 04/24/2015 07:23 pm

When considering the use of EM-drives in the making of warp-drives the attached two papers might be of interest.

In the meantime, back to figuring out how to reliably drive an EM-drive...

Edit:  You might also like to read Sonny's Warp-field Mechanics 101 and 102 articles.
http://ntrs.nasa.gov/archive/nasa/casi.ntrs.nasa.gov/20130011213.pdf

Best, Paul M.

Paul M,

Thanks a lot. This papers very interesting, but little old. And what about latest papers/presentations which dated 04/04/2015, 04/09/2015 and 04/15/2015. I've seen only few pages from them and it would be very interesting to read full version. Can you share full document, please. In attachment are this pages I've seen.

Best regards,
Nikita Unkovsky

Nikita:

Dr. White's latest Warp Field Interferometer (WFI) results are just in PowerPoint file format at the moment, for they are just used for keeping the Eagleworks' technical and management teams aware of the latest developments.  So if you want a copy of same you need to send a note to Dr. White asking for it.  In the meantime Dr. White cautioned me yesterday that I need to be more careful in declaring we've observed the first lab based space-time warp signal and rather say we have observed another non-negative results in regards to the current still in-air WFI tests, even though they are the best signals we've seen to date.  It appears that whenever we talk about warp-drives in our work in a positive way, the general populace and the press reads way too much into our technical disclosures and progress.

Next find attached Sonny's latest WFI data set number-1 analysis that utilizes all 28.5k period samples instead of just a very few arbitrarily selected pixel addresses we used before so as to minimize the compute times.  And I'm glad that Dr. White was an ex-game programmer in his youth because his programming expertise was really needed for this problem to be able to analysis this large set in less than the 100 years his initial cut at is indicated.  In fact it is now running in less than 4-to-6 hours on a Windows 7.0 PC with an Intel i5 in it.  Be that as it may, you will note that the assumed in-air space-time compression signal is still there with now a much improved signal to noise ratio than the previous example I provided.

Best,  Paul M.

1) Labeling correction

Should read:

On/Off Sample Period Frames T = 1.53 sec
f = 1/T = 0.654 Hz

2) When plotting the FFT Spectral Power Density, I only plot it up to the Nyquist frequency (in this case 5 Hz), because, as pointed out in the figure, everything at a higher frequency than the the Nyquist frequency has aliasing

(http://forum.nasaspaceflight.com/index.php?action=dlattach;topic=36313.0;attach=826245;image)
Title: Re: EM Drive Developments - related to space flight applications - Thread 2
Post by: RotoSequence on 04/24/2015 07:34 pm
Paul M,

Are there any prepared graphics of null runs with the warp interferometry experiment?
Title: Re: EM Drive Developments - related to space flight applications - Thread 2
Post by: Rodal on 04/24/2015 07:57 pm
...

I'm thinking about that ring laser thing (from the optical diametric drive research) again where the wave form is formed in such a way that there are always transient negative energy regions in the waveform.

...
What ring laser thing possessing " always transient negative energy regions in the waveform" are you referring to?

Link please  :)

http://phys.org/news/2013-10-optical-diametric.html

http://www.creol.ucf.edu/Research/Publications/7155.pdf

http://www.nature.com/nphys/journal/v9/n12/full/nphys2777.html?WT.ec_id=NPHYS-201312

"Always transient"  refers to the way squeezed light makes negative energy. It's always for a tiny fraction of the over all cycle and the positive side always dwarfs it. But in a circular track... there should be a tiny bit present at all times and in all parts of the track. Or maybe not as tiny as it would be in a finite length course and there are two circular stacked circular tracks in those articles. Maybe the two fields can constructively reinforce the negative components or destructively interfere with the positive portion selectively. i don't know much about the physics. I'm just "using the force."

EDIT:  So here is a question I have about squeezed light. what would happen if a portion of the circular track was made to slow light down? I have read about certain research slowing light down under the SOL in a vacuum. You have two rings... one of the trackways could have unimpeded squeezed light and the other had at least a portion with a go slow zone. Could the tracks be juxtaposed in such a way that the negative regions of the waveform were close enough to sum together? And could there be a tidal pile up of negative energy caused by the go slow zone?
Thank you for providing the links and refreshing my mind.

(Hat tip to @NoSoSureOfIt for first pointing this out  :) ), a truncated cone EM Drive really is roughly analogous to 1/2 of this "optical diametric drive". 

So what NASA Eagleworks could try is doing further experiments having the laser go through:

Experiment A) a truncated cone EM Drive



Experiment B) two truncated cone EM Drive's mated together such that their small diameter bases are conjoined, forming an hourglass shape

instead of the pillbox-shaped EM Drive resonant cavity
Title: Re: EM Drive Developments - related to space flight applications - Thread 2
Post by: zen-in on 04/24/2015 08:36 pm
Speaking of establishing phenomena - does anyone have a good reference on how using an oscillating EM field is supposed to induce spacetime contractions?

I've been doing a little background reading, specifically:
http://ntrs.nasa.gov/archive/nasa/casi.ntrs.nasa.gov/20110015936.pdf (http://ntrs.nasa.gov/archive/nasa/casi.ntrs.nasa.gov/20110015936.pdf)
http://www.earthtech.org/publications/davis_STAIF_conference_2.pdf (http://www.earthtech.org/publications/davis_STAIF_conference_2.pdf)
http://members.shaw.ca/mike.anderton/WarpDrive.pdf (http://members.shaw.ca/mike.anderton/WarpDrive.pdf)
http://link.springer.com/article/10.1023%2FA%3A1026299010288 (http://link.springer.com/article/10.1023%2FA%3A1026299010288)
http://www.gravityresearchfoundation.org/pdf/awarded/2004/loup_santos.pdf (http://www.gravityresearchfoundation.org/pdf/awarded/2004/loup_santos.pdf)

I saw this plot :
Is it just to push the peak past the 1/f noise?
 (http://forum.nasaspaceflight.com/index.php?action=dlattach;topic=36313.0;attach=825749;image[/url)

There is really nothing here.   I have worked with much more precise interferometers and know that laser fringes are inherently noisy.   Any thermal gradiant, vibration, or EM energy pulses will  show up as a phase shift.   That is one of the biggest problems in achieving high resolution spectroscopy with FTIR.   Those interferometers are built to a much higher precision than the White Juday experiment and with accurate mirror alignment a narrow band monochromatic interference pattern will cover half the optical aperture.   This is what is shown in the diagram from White's paper, reproduced below. (first image)   The 3-D plot is just a 3-D rendering of an interference pattern.    The Z-scale has made up numbers and no dimension.  In actual fact the difference in path length indicated by this interference pattern is 633/4 nM and is just what normal interference patterns look like.   Applying a 3-D toolkit to an interference pattern image does not prove there is a warp field.   It is just a flashy powerpoint slide.
Title: Re: EM Drive Developments - related to space flight applications - Thread 2
Post by: Raj2014 on 04/24/2015 09:12 pm
What is the latest news on the EM drive. I have looked on Google. I have just read on another website that NASA may have accidentally created a warp field? Is this true. 
Title: Re: EM Drive Developments - related to space flight applications - Thread 2
Post by: UneducatedNitwit on 04/24/2015 09:32 pm
What is the latest news on the EM drive. I have looked on Google. I have just read on another website that NASA may have accidentally created a warp field? Is this true.

I believe the current status of public skepticism should be:

There is insufficient evidence to support the idea that it might be a warp in spacetime, in the unlikely scenario that it is we will have to wait at least 6 months for any supporting evidence, and this is gaining traction with SciFi hopefuls because it is being discussed by authoritative figures (NASA employees on this forum).
Title: Re: EM Drive Developments - related to space flight applications - Thread 2
Post by: Rodal on 04/24/2015 10:02 pm
....
Personally, I'm awaiting an unequivocal "Chicago Pile" moment, instead of near misses and uncertain results.

....
, in the unlikely scenario that it is we will have to wait at least 6 months for any supporting evidence, and this is gaining traction with SciFi hopefuls because it is being discussed by authoritative figures (NASA employees on this forum).

long-term R&D work is like an out-of-the-money call option:  the probability of success is small and conversely, the payoff is huge.

Regarding timing, it also works like an out-of-the-money call option: by the time you'll hear "an unequivocal yes, it is a success" you will be among the last to know.  This applies to R&D on biotech drugs, R&D on Defense projects,  R&D on high-tech, etc.

So resign yourself to the truth:  nobody will-ring a bell letting you know about the "pile moment" (certainly this was not done in WWII for the Chicago pile, we didn't announce to the enemy that we had launched onto the first steps leading to the A-Bomb). 

The likelihood of any particular long-term R&D project having a practical payoff is tiny, if there is a payoff it will be huge, and you will be the last to know  :)



Let's break this down:

The likelihood of any particular long-term R&D project
==> the tests at NASA Eagleworks are the first steps in a long-term R&D project.  Nobody ever though that this was a project where we were going to put these EM Drives into a spaceship next week and into a warp-drive next month. 

The likelihood of ...having a practical payoff is small,  if there is a payoff it will be huge,
==> this is well known and indisputable.  One doesn't need to be an engineer or scientist to know this. Any investor knows this

and you will be the last to know
==> this is well understood in Wall Street and in Main Street.  We didn't announce during WWII that the Chicago Pile success had launched us onto the first steps leading to the A-Bomb.  Biotech companies announce success in a biotech drug only after it has passed Phase 3 tests and approval from the FDA, by that time you are the last one to know.  Ditto in High Tech R&D projects.
Title: Re: EM Drive Developments - related to space flight applications - Thread 2
Post by: Star One on 04/24/2015 10:08 pm
....
Personally, I'm awaiting an unequivocal "Chicago Pile" moment, instead of near misses and uncertain results.

....
, in the unlikely scenario that it is we will have to wait at least 6 months for any supporting evidence, and this is gaining traction with SciFi hopefuls because it is being discussed by authoritative figures (NASA employees on this forum).

long-term R&D work is like an out-of-the-money call option:  the probability of success is small and conversely, the payoff is huge.

Regarding timing, it also works like an out-of-the-money call option: by the time you'll hear "an unequivocal yes, it is a success" you will be among the last to know.  This applies to R&D on biotech drugs, R&D on Defense projects,  R&D on high-tech, etc.

So resign yourself to the truth:  nobody will-ring a bell letting you know about the "pile moment" (certainly this was not done in WWII for the Chicago pile, we didn't announce to the enemy that we had launched onto the first steps leading to the A-Bomb). 

The likelihood of any particular long-term R&D project having a practical payoff is small, if there is a payoff it will be huge, and you will be the last to know  :)

Some might say it's a bit late in the day for people to be putting forward the line that there is nothing to see here please move along now. But that's going OT again.
Title: Re: EM Drive Developments - related to space flight applications - Thread 2
Post by: Star One on 04/24/2015 10:18 pm

...
long-term R&D work is like an out-of-the-money call option:  the probability of success is small and conversely, the payoff is huge.

Regarding timing, it also works like an out-of-the-money call option: by the time you'll hear "an unequivocal yes, it is a success" you will be among the last to know.  This applies to R&D on biotech drugs, R&D on Defense projects,  R&D on high-tech, etc.

So resign yourself to the truth:  nobody will-ring a bell letting you know about the "pile moment" (certainly this was not done in WWII for the Chicago pile, we didn't announce to the enemy that we had launched onto the first steps leading to the A-Bomb). 

The likelihood of any particular long-term R&D project having a practical payoff is small, if there is a payoff it will be huge, and you will be the last to know  :)

Bit late in the day for people to be putting forward the line that there is nothing to see here please move along now.

?
Who put "forward the line that there is nothing to see here please move along " ?
?

I meant in the sense that it's a bit late in the day to be attempting to cool expectations as regards warp drives at this time, as mention of such has already attracted the attention of the wider realm.
Title: Re: EM Drive Developments - related to space flight applications - Thread 2
Post by: Star One on 04/24/2015 10:24 pm

...
long-term R&D work is like an out-of-the-money call option:  the probability of success is small and conversely, the payoff is huge.

Regarding timing, it also works like an out-of-the-money call option: by the time you'll hear "an unequivocal yes, it is a success" you will be among the last to know.  This applies to R&D on biotech drugs, R&D on Defense projects,  R&D on high-tech, etc.

So resign yourself to the truth:  nobody will-ring a bell letting you know about the "pile moment" (certainly this was not done in WWII for the Chicago pile, we didn't announce to the enemy that we had launched onto the first steps leading to the A-Bomb). 

The likelihood of any particular long-term R&D project having a practical payoff is small, if there is a payoff it will be huge, and you will be the last to know  :)

Bit late in the day for people to be putting forward the line that there is nothing to see here please move along now.

?
Who put "forward the line that there is nothing to see here please move along " ?
?

I meant it's a bit late in the day to be attempting to cool expectations as regards warp drives even if unfounded in reality as mention of such has already attracted the attention of the wider realm.
Read again.  Who said "unfounded in reality " ????

I don't see what is wrong in taking this cautious stance at this time in the process of research.

But I have modified my wording in this post in light of your comments to better reflect the point that I was trying to put across.
Title: Re: EM Drive Developments - related to space flight applications - Thread 2
Post by: yg1968 on 04/25/2015 12:30 am
Here is an article referencing this thread:

http://mysteriousuniverse.org/2015/04/nasa-may-have-accidentally-developed-a-warp-drive/

https://twitter.com/KedarPavgi/status/591674468172435456
Title: Re: EM Drive Developments - related to space flight applications - Thread 2
Post by: Stormbringer on 04/25/2015 12:30 am
There is nothing wrong with regular people getting excited by this. Just as there is nothing wrong with non jurors and so forth forming an opinion on a court case.

research that stirs the public imagination is good for science itself and the budgets thereof.  ;D
Title: Re: EM Drive Developments - related to space flight applications - Thread 2
Post by: aceshigh on 04/25/2015 12:57 am
There is nothing wrong with regular people getting excited by this.

as opposed to what? Are we all NASA engineers or PHDs? In fact, non PhDs, get off!  ;D

"Dr Koothrappali, Dr Hofstadter, Dr Cooper, MISTER Wolowitz"

"I have a Masters Degree"

"Who doesn´t?"

https://www.youtube.com/watch?v=gdi3FIrsHgo

I guess I count myself among the "regular" people... well, space enthusiast?
Title: Re: EM Drive Developments - related to space flight applications - Thread 2
Post by: aceshigh on 04/25/2015 12:59 am
NASA developed a war-drive accidentally ????  ::)

was that an intended pun because of the possibility of destroying a small civilization just by turning off the warp field while pointing the ship at the planet?  :D
Title: Re: EM Drive Developments - related to space flight applications - Thread 2
Post by: Stormbringer on 04/25/2015 01:04 am
yeah i was just on my way to post that article. (and a facepalm.)

Facepalm >>>>>> http://i.stack.imgur.com/jiFfM.jpg

and i wanted to credit notsureof it on the optical diametric drive articles but did not find his post when briefly looking for it. so i knew it was either he or Mulletronn that posted it.
Title: Re: EM Drive Developments - related to space flight applications - Thread 2
Post by: aceshigh on 04/25/2015 01:08 am
NASA developed a war-drive accidentally ????  ::)

was that an intended pun because of the possibility of destroying a small civilization just by turning off the warp field while pointing the ship at the planet?  :D

The article states:

Quote
Another surprise is that the discovery was accidental, as this comment attests.

"Seems to have been an accidental connection. They were wondering where this “thrust” might be coming from. One scientists proposed that maybe it’s a warp of the spacetime foam, which is causing the thrust."

I was talking about you writing WAR-drive instead of warP-drive...
Title: Re: EM Drive Developments - related to space flight applications - Thread 2
Post by: Rodal on 04/25/2015 01:15 am
Ooops my bad, an unintended typo, an accidental pun  :-[
Title: Re: EM Drive Developments - related to space flight applications - Thread 2
Post by: dustinthewind on 04/25/2015 01:59 am
I am really thinking if we have a warped space field in the vicinity of the cavity that a very accurate clock would be a good choice of measuring it.  I am guessing a clock can measure a small change but it would accumulate over time where as I don't see the laser measurement accumulating over time. 

On the other hand it might be hard to stick the clock inside the cavity but maybe it doesn't have to be inside and just near by. 

If the issue is we have an osculating space time between positive and negative then maybe we could still accumulate a distortion in measured time by using a "phase lock amplifier" or "lock-in amplifier" http://en.wikipedia.org/wiki/Lock-in_amplifier

http://www.sciencedaily.com/releases/2013/10/131002103036.htm
Title: Re: EM Drive Developments - related to space flight applications - Thread 2
Post by: UneducatedNitwit on 04/25/2015 02:43 am
There is nothing wrong with regular people getting excited by this.

as opposed to what? Are we all NASA engineers or PHDs? In fact, non PhDs, get off!  ;D

As an uneducated nitwit who studies math and physics for sport, I am deeply frustrated by the idea that people without specialized training should be excited by anything which isn't in a textbook.
Title: Re: EM Drive Developments - related to space flight applications - Thread 2
Post by: Rodal on 04/25/2015 03:08 am
There is nothing wrong with regular people getting excited by this.

as opposed to what? Are we all NASA engineers or PHDs? In fact, non PhDs, get off!  ;D

As an uneducated nitwit who studies math and physics for sport, I am deeply frustrated by the idea that people without specialized training should be excited by anything which isn't in a textbook.
Please be assured that @Stormbringer didn't mean anything of the kind.  I have seen most of @Stormbringer's posts: a most considerate, polite, respectful, helpful and professional poster.  The written word, particularly when typing fast, some times can convey extra meanings completely unintended by the person who wrote them in good faith.

The comment by @AcesHigh was intented for comic release, as you can see from his grin  ;D , conveying humor.   
Title: Re: EM Drive Developments - related to space flight applications - Thread 2
Post by: Stormbringer on 04/25/2015 04:10 am
Yes. I only meant that people not accustomed to the academic rigor, experimental and peer reviews protocol nor educated in the physics or the math can get excited by this sort of thing and that's good, in my opinion. Not that credentialed professional people were not normal people. Perhaps I should have said something like "laypeople" though that doesn't sound appropriate.

And as dense as I can be; I hadn't even noticed that anyone had been offended. I thought the replies were all in humor. :)
Title: Re: EM Drive Developments - related to space flight applications - Thread 2
Post by: CW on 04/25/2015 06:53 am
I think that many underestimate the lengthy process that is R&D. It is often confused with prototyping, and thus the expectations are off. I for one am very thankful for showing the R&D process so openly and sharing the approach of how to possibly find a workable solution to a mindblowing prospect. Whatever comes out of it - knowledge and understanding will improve. I think that scientists should only follow the data and not be mindlessly bound by preconclusions of the past. If Max Planck had listened to his professor (he said something akin to "Why do you want to study physics? Except for some little things, everything that could be researched, was already researched."), when he wanted to study physics, he wouldn't have studied physics. Let's think about the consequences for a bit.. .

So, guys.. please continue to share the R&D process so openly as you have done.
8)
Title: Re: EM Drive Developments - related to space flight applications - Thread 2
Post by: Star One on 04/25/2015 07:07 am

There is nothing wrong with regular people getting excited by this. Just as there is nothing wrong with non jurors and so forth forming an opinion on a court case.

research that stirs the public imagination is good for science itself and the budgets thereof.  ;D

I suppose the danger it runs is of overly raising expectations at this stage in the process. That's all I really meant not that people shouldn't take an interest in it.
Title: Re: EM Drive Developments - related to space flight applications - Thread 2
Post by: Mulletron on 04/25/2015 10:13 am
Recent events have really captured the imaginations of many of us here and across the net, but we must maintain a skeptical eye and remain calm and objective. Plenty of plausible mechanisms have been presented here which can explain any perceived indications of spatial contraction due to the presence of atmosphere.

We can celebrate (and debate the finer points of warp travel) when results are successful in vacuum and replicated by others.

After some thought on the subject, I'm going to operate as if there is nothing to this until proven otherwise.

Now back to proven/accepted science for this guy.

The effect is predicted and still needs to be discovered and proven.

For now I'm treating this warp stuff as a zinger which I will store away if needed later.

I got kinda excited about this too but I realized it was largely based on little tangible, which caused me to lose focus. My intention for this post is to try and get the thread back on track, which is to explore whether Emdrive is real and if it can be used for spaceflight.
Title: Re: EM Drive Developments - related to space flight applications - Thread 2
Post by: Star One on 04/25/2015 10:44 am
For me zen-in's last post was very informative on these recent observations & something that's really worth keeping in mind.
Title: Re: EM Drive Developments - related to space flight applications - Thread 2
Post by: Rodal on 04/25/2015 12:43 pm
We ought to remember a famous quote written hundreds of years ago:

Quote
it ought to be remembered that there is nothing more difficult to take in hand, more perilous to conduct, or more uncertain in its success, than to take the lead in the introduction of a new order of things. Because the innovator has for enemies all those who have done well under the old conditions, and lukewarm defenders in those who may do well under the new. This coolness arises partly from fear of the opponents, who have the laws on their side, and partly from the incredulity of men, who do not readily believe in new things until they have had a long experience of them. Thus it happens that whenever those who are hostile have the opportunity to attack they do it like partisans, whilst the others defend lukewarmly

and thus avoid behaving like partisans, avoid getting excited and avoid emotional responses. 

Be cool  8)

Let's continue to use the scientific method.
Title: Re: EM Drive Developments - related to space flight applications - Thread 2
Post by: lele on 04/25/2015 02:06 pm

So, guys.. please continue to share the R&D process so openly as you have done.
8)

On this subject, please also report, as much as possible, negative results, especially since it's still probable that there is a mundane explanation for all the strange results obtained so far.
Reporting negative results is something scientists agree is not done enough, but it's easier on this forum since we have the advantage that there is no editorial board or reviewers.

Title: Re: EM Drive Developments - related to space flight applications - Thread 2
Post by: Star One on 04/25/2015 02:26 pm


So, guys.. please continue to share the R&D process so openly as you have done.
8)

On this subject, please also report, as much as possible, negative results, especially since it's still probable that there is a mundane explanation for all the strange results obtained so far.
Reporting negative results is something scientists agree is not done enough, but it's easier on this forum since we have the advantage that there is no editorial board or reviewers.

Very true as regards the under-reporting of negative results in general, read more than a few articles complaining about this.
Title: Re: EM Drive Developments - related to space flight applications - Thread 2
Post by: Star-Drive on 04/25/2015 03:43 pm
Paul M,

Are there any prepared graphics of null runs with the warp interferometry experiment?

Other than the 20W version of these WFI runs which barely had a signal present at ~0.654 Hz, no we haven't, but we will be performing one in the near future when our very part time Dr. Rollins from the science directorate can perform it on another "free" weekend of his.

Dr. Rodal:

Labeling; "T" not f for period, check.

All:

How does one report "negative" results when at times the acquired data just doesn't make sense?  I've been plowing through literally hundreds of copper frustum tests over the last year with various RF tuning configurations and finding that some appear to generate nothing but spurious thermal like results as demonstrated by their positive and then negative going gradual exponential rise and fall times, others that show a very prompt signal at RF turn-on and turn-off comparable to the electrostatic force calibrator on/off slopes, overlaid with the above thermal signatures and some that fall in between.  The only real way I can make sure the "thrust" plots I've been generating are real thrust signatures is to first check for a prompt signal during the first ~5 seconds before thermal effects take hold and then going into reverse thrust mode where the thrust signature opposes the thermal signature to the point it goes negative like the one I've already appended but repeat it here.  Any other testing approaches to cull out these blasted thermal signatures would be appreciated. 

BTW, when the RF is first turned on we literally have an RF induction heating system that immediately starts warming the copper cavity walls, especially at the large OD end of the frustum.  How long does that RF induced thermal heating take to start moving the copper frustum and to what degree?  Looks like another COMSOL problem that will take into account the specific heat of all the frustum components, then profiles the resulting differential temp rise of cone that then generates a frustum expansion rate that will then have to feed into a model of the torque pendulum's deflection sensitivity to off axis loading.  Yuck!   

Best, Paul M.
Title: Re: EM Drive Developments - related to space flight applications - Thread 2
Post by: Stormbringer on 04/25/2015 03:49 pm
Just as an aside; I'm sure it hasn't escaped the attention of most here that I am an uncredentialed layperson that usually goes cross eyed just looking at the math part of this. I handle most of this at a purely verbal level. And I am more willing than most here (at least temporarily) to dip into the speculative, the fringe and the Ko0ky side for bits that might have something to them. So it's not as if i am telling such people to get out. I'd be hoisting myself by my own belt loops and escorting my own self out if that were the case.

Seriously; I have had science especially anything to do with space, astronomy or cosmology as a hobby since grade school. I challenged out on enough credits to almost get a bachelor's degree in science topics and in fact got a 2 year degree in science by taking proctored tests while in the military with UNY Regents. I only have 3 actual traditional University courses out of 104 total ACE accredited credit hours. That's it. That's the extent of my academic credentials. Other than that I was a radar operator, org level radar repairman and technician over 20 years so i understand RF systems in that operational and technical niche.


 
Title: Re: EM Drive Developments - related to space flight applications - Thread 2
Post by: Rodal on 04/25/2015 04:04 pm
Just as an aside; I'm sure it hasn't escaped the attention of most here that I am an uncredentialed layperson that usually goes cross eyed just looking at the math part of this. I handle most of this at a purely verbal level. And I am more willing than most here (at least temporarily) to dip into the speculative, the fringe and the Ko0ky side for bits that might have something to them. So it's not as if i am telling such people to get out. I'd be hoisting myself by my own belt loops and escorting my own self out if that were the case.

...

I admire your honesty and humility, a great example to the rest of the forum.

Quote from: William Penn
Sense shines with a double luster when it is set in humility. An able yet humble man is a jewel worth a kingdom.

Title: Re: EM Drive Developments - related to space flight applications - Thread 2
Post by: Rodal on 04/25/2015 04:22 pm
...

BTW, when the RF is first turned on we literally have an RF induction heating system that immediately starts warming the copper cavity walls, especially at the large OD end of the frustum.  How long does that RF induced thermal heating take to start moving the copper frustum and to what degree?  ..
The induction heating through the cavity occurs at the speed of light, therefore it is practically instantaneous.  The induction heating decays exponentially in the copper thickness.  The skin depth of induction heating is minute: for copper at 2 GHz it is 1.46 micrometers.  So the only practical issue involving time is due to thermal conduction through the copper, which is governed by the thermal diffusivity of copper and the thickness of copper.  It is best discussed in terms of the dimensionless Fourier number. I have conducted a number of calculations based on this earlier on the thread and wrote a paper on thermal buckling for the cavity that has some representative numbers (less than a second to move the copper a fraction of its thickness).  The bending due to the thermal gradient through the thickness is signifcantly slower, on the order of the test duration.  @frobnicat and me had a number of discussions and calculations on this also earlier on the thread.
Title: Re: EM Drive Developments - related to space flight applications - Thread 2
Post by: Rodal on 04/25/2015 04:34 pm
...

How does one report "negative" results when at times the acquired data just doesn't make sense? ...
Forget it.  You are doing enough reporting interesting results to this thread, for which we are thankful.
Asking you to report all your negative results to a forum is unrealistic.

Ask those who ask you to report negative results whether they know what is the proportion of papers reporting experimental negative results in the most respected peer-reviewed physics journals.

Reporting negative results is a valid concern for non-exact sciences, as Biology used to be (now not so much with Biotechnology which is based on scientific calculations), and certainly necessary for Medicine and required for social "sciences" that need to rely on statistics because of the lack of physical laws and mathematical modeling for those disciplines.  In Medicine, a Nobel Prize was awarded to the inventor and practitioner of lobotomies (reporting of negative results would have been obviously important in this case ! ) and in the "dismal" social science (Economics)  Nobel prizes have been awarded simultaneously to people holding opposing views, so certainly in those disciplines negative results are most important.  The history of Nobel Prizes in Physics is pristine, in comparison, thanks to physical laws (universal laws like conservation of momentum).
Title: Re: EM Drive Developments - related to space flight applications - Thread 2
Post by: lele on 04/25/2015 05:54 pm
...

How does one report "negative" results when at times the acquired data just doesn't make sense? ...
Forget it.  You are doing enough reporting interesting results to this thread, for which we are thankful.
Asking you to report all your negative results to a forum is unrealistic.


Of course I don't want to tell Mr March how to do his job, and I think everyone here really appreciate that 1) people at JSC are working on that subject, 2) Mr March is reporting results here.

I know it's not possible to put the results of every test here.
What I meant is that, for example, Mr March said earlier that numerical computations predicted a 1250N force for a 100kW input. If experiments don't give the same results, I think most people here would like to know it (sorry if it's obvious).
Once again, people who are actually doing experiments or theoretical computations don't owe anything to random forum readers like myself (especially if said readers don't pay their taxes in the US).

The more is known about the experiments (such as the protocol for choosing "real" thrust plots Mr March described in his last post) the more people here can help.

Ask those who ask you to report negative results whether they know what is the proportion of papers reporting negative results in the most respected peer-reviewed scientific journals.

Well, that's another subject, but I think it would probably be better for science if this proportion were higher.
Title: Re: EM Drive Developments - related to space flight applications - Thread 2
Post by: zen-in on 04/25/2015 06:01 pm
...

How does one report "negative" results when at times the acquired data just doesn't make sense? ...
Forget it.  You are doing enough reporting interesting results to this thread, for which we are thankful.
Asking you to report all your negative results to a forum is unrealistic.

I agree with this.  Negative results don't need to be reported if a protocol has been developed that produces consistent reproducible results.   No one cares about Edisons' many failed experiments while developing an electric light.   In patent language this is called "reduction to practice".
Title: Re: EM Drive Developments - related to space flight applications - Thread 2
Post by: Rodal on 04/25/2015 06:05 pm
EXCELLENT points by @lele and @zen-in on the need for full disclosure of the experimental protocol and by @zen-in on "reduction to practice" and reproducible results.  Consistent reproducible results at Ealeworks as wells as at independent research centers will certainly be needed (as we have been discussing regarding NASA Glenn, JPL or at John Hopkins).
Title: Re: EM Drive Developments - related to space flight applications - Thread 2
Post by: CW on 04/25/2015 06:41 pm
...

How does one report "negative" results when at times the acquired data just doesn't make sense? ...
Forget it.  You are doing enough reporting interesting results to this thread, for which we are thankful.
Asking you to report all your negative results to a forum is unrealistic.

I agree with this.  Negative results don't need to be reported if a protocol has been developed that produces consistent reproducible results.   No one cares about Edisons' many failed experiments while developing an electric light.   In patent language this is called "reduction to practice".

I like that quote from Edison: "I have not failed. I've just found 10,000 ways that won't work." (http://www.brainyquote.com/quotes/quotes/t/thomasaed132683.html (http://www.brainyquote.com/quotes/quotes/t/thomasaed132683.html)) .

Documenting non-working setups is also vital (and explaining what went wrong, in best case), so that no-one repeats avoidable mistakes. This could be especially important for replication efforts. In case there appear unequivocal results, creating a detailed construction manual with bringup tests etc would make a lot of sense.
;)
Title: Re: EM Drive Developments - related to space flight applications - Thread 2
Post by: Rodal on 04/25/2015 07:33 pm
Yes we all agree that documenting every detail is always better.  But we have to be realistic and not insist with undue requests from the only person conducting this research that is openly discussing it (certainly Shawyer's team and Prof. Yang's team members have never directly disclosed any data or answered any questions whatsoever in this forum).

For the EM Drive we have a resonant electromagnetic cavity which according to classical physics (encompassing the theory of General Relativity as well as linear Quantum Mechanics), should not accelerate in outer space (under the action of no external forces or no external fields, assuming that the Quantum Vacuum is indestructible and immutable, involving no CP parity violation) because of the universal law of conservation of momentum.  (*)

A billion tests that resulted in the EM Drive producing no acceleration will hold no attention from the scientific community, since such results are fully expected.

A reproducible, incontrovertible test showing self-acceleration (and certainly one showing levitation of the test article for anybody to see) will hold much more attention from the scientific community than a billion tests showing no acceleration.

Once you get that reproducible, consistent, incontrovertible result,  one can proceed to discuss the kind of engineering development that Edison engaged in, and where negative results of variations from the  reproducible, consistent, incontrovertible result are valuable in optimizing the device.

At the moment we are still seeking that reproducible, consistent, incontrovertible result.

__________________________

(*) we are not dealing here with an inexact science.  This is not like Medicine, where a drug can be effective with 70% of the population, ineffective with 29% of the population and produce severe negative effects with 1% of the population.  Conservation of momentum applies in physics for 100% of all cases bar none.  Ditto for conservation of energy.  These are universal laws, just like everybody here is subject to gravitation and we don't have a certain percentage of the population that is unaffected by gravity.
Title: Re: EM Drive Developments - related to space flight applications - Thread 2
Post by: NovaSilisko on 04/25/2015 10:03 pm
And once again I continue to have my dislike of the "popular science" media reenforced.

I can't really articulate how pleased I am that there's nothing even close to the typical "pff science has been wrong before, they'll find a way around that dumb conservation of energy stuff" handwavium reaction around here. I applaud everyone doing the hard work here for being rigorous and scientific about it.
Title: Re: EM Drive Developments - related to space flight applications - Thread 2
Post by: toocoolforschool on 04/26/2015 03:01 am
couldn't this energy source be used for something other than space travel as well? i mean its generating thrust thrust can power a lot of things.  on another note consider the speed at which faster than light is and you have to consider the possibilities of path disruption say a stray celestial body getting in the way of the craft.  I mean warp speed technically is still theoretical but what could be done to direct a safe path at speeds beyond that of light.  I mean we could potential be talking about a space highway of some sort here.  Its all theory but this discovery has me wanting to be a part of this stuff.....  either way awesome find I wish I understood all the details but I'm definitely intrigued enough to go back to school and get into a bachelors program.   
Title: Re: EM Drive Developments - related to space flight applications - Thread 2
Post by: toocoolforschool on 04/26/2015 03:06 am
another thing if its giving off heat doesn't that mean that some of the energy is being wasted?  ;) 
Title: Re: EM Drive Developments - related to space flight applications - Thread 2
Post by: toocoolforschool on 04/26/2015 03:08 am
not sure how it all works but could insulation maybe be of some help... i apologize if I sound like a idiot but I will definitely do some research.
Title: Re: EM Drive Developments - related to space flight applications - Thread 2
Post by: Stormbringer on 04/26/2015 03:17 am
Talking about warp is a bit premature because- questions of if it will work aside; we have nothing that can power it and we have insufficient negative mass energy equivalence to do anything more than wiggle a laser beam interference pattern so slightly that no one wants to say that it is actually happening for fear of running afoul of peer review. Or enough to sort of maybe wiggle a hanging tissue paper on a windy day.

But that aside; precautions for impact avoidance/mitigation at low relativistic speed should suffice. Not that we have that either, yet. The way warp works there is no added velocity inside the bubble but just in the boundary region between normal space and the isolated interior flat space.
Title: Re: EM Drive Developments - related to space flight applications - Thread 2
Post by: Stormbringer on 04/26/2015 04:28 am
Terahertz source for miniaturized EM frustrum array chips?

http://phys.org/news/2015-04-broadband-terahertz-microplasma-air.html

It uses a laser to create a plasma that emits broadband terahertz spectrum waves and it looks small -like spark plug sized small. It also looks like it requires no cryogenic cooling?
Title: Re: EM Drive Developments - related to space flight applications - Thread 2
Post by: Star-Drive on 04/26/2015 04:44 am
Yes we all agree that documenting every detail is always better.  But we have to be realistic and not insist with undue requests from the only person conducting this research that is openly discussing it (certainly Shawyer's team and Prof. Yang's team members have never directly disclosed any data or answered any questions whatsoever in this forum).

For the EM Drive we have a resonant electromagnetic cavity which according to classical physics (encompassing the theory of General Relativity as well as linear Quantum Mechanics), should not accelerate in outer space (under the action of no external forces or no external fields, assuming that the Quantum Vacuum is indestructible and immutable, involving no CP parity violation) because of the universal law of conservation of momentum.  (*)

A billion tests that resulted in the EM Drive producing no acceleration will hold no attention from the scientific community, since such results are fully expected.

A reproducible, incontrovertible test showing self-acceleration (and certainly one showing levitation of the test article for anybody to see) will hold much more attention from the scientific community than a billion tests showing no acceleration.

Once you get that reproducible, consistent, incontrovertible result,  one can proceed to discuss the kind of engineering development that Edison engaged in, and where negative results of variations from the  reproducible, consistent, incontrovertible result are valuable in optimizing the device.

At the moment we are still seeking that reproducible, consistent, incontrovertible result.

__________________________

(*) we are not dealing here with an inexact science.  This is not like Medicine, where a drug can be effective with 70% of the population, ineffective with 29% of the population and produce severe negative effects with 1% of the population.  Conservation of momentum applies in physics for 100% of all cases bar none.  Ditto for conservation of energy.  These are universal laws, just like everybody here is subject to gravitation and we don't have a certain percentage of the population that is unaffected by gravity.

Dr. Rodal:

Agreed, all known laws of physics must be observed, but the fun part is trying to take unusual phenomenon like that demonstrated by the EM-drive and show how they DO observe these laws, but in new and unexpected ways.  IMO Woodward's and these Q-V based drives are interacting with the cosmological gravitational field via the proposed Q-V interactions we are now trying to get a handle on.  And of course that leads us to another of your above comments and how we can reduce this to practice :

"At the moment we are still seeking that reproducible, consistent, incontrovertible result."

Reproducibility of test results is what is still lacking in our current copper frustum testing.   So I'm now trying to develop the right combination of test ingredients for these copper frustums when using RF power levels of ~80W or less, since it appears from simulation that higher power, in the tens of kW, can overcome a lot of low power problems, but alas that solution does not come cheap. 

IMO it now appears that what the missing key ingredient to producing consistent thrust on the order of tens or even hundreds of micro-Newton on cue at power levels of less than 100W is how much harmonic content is present in the RF signal.  Oh yes and the magnitude of same injected into the RF resonant system relative to the magnitude of the near pure sine-wave RF signal.  A very low harmonic signal that our voltage controlled oscillator (VCO) is producing.  In other words higher amplitude and wider bandwidth harmonic content appears to produce higher thrust levels for a given RF input power and Q-factor product. 

BTW, this harmonic thrust generation dependency is predicted by both Woodward's and White's respective conjectures with the addition of the need for proper phasing between all these fundamental and AM & FM modulation signals.  In our current experimental setup however, our harmonic generators have all been generated by happenstance and at low levels that are induced by the interactions between our PLL circuit and the RF 2- & 3-stub tuners + coax runs to the frustum used to RF Z-match the amplifier to the cavity that appears to be generating extra resonances in the RF amplifier chain that are then fed back into the VCO and RF amplifier via conducted and radiated emissions.  I’m pretty sure that this is why we observed such great power to thrust efficiencies with the TE012 and one of the TM211 resonant tests when our RF amplifier was dying from internal corona discharges.  A condition that is choked full of not so low level sideband modulation that was ignored at the time because we were not monitoring for that condition.  This extra harmonic content was also true for the Cannae results as well. 

So now it looks like we really need to be able to inject harmonic signals in a controlled manner into the frustum cavity that will modulate the fundamental resonant frequency established by the VCO.  A setup that is able to explore this harmonic content venue with additional on-purpose modulator and monitoring equipment that will of course require more time and money… 

Best, Paul M.
Title: Re: EM Drive Developments - related to space flight applications - Thread 2
Post by: Peter Svancarek on 04/26/2015 08:21 am
Maybe stupid question... but why didn't NASA send small scale demonstrator to space?  It doesn't have to be heavy (resonance chamber doesn't have to be rated for outside pressure), it doesn't have to have even energy source (it could be plugged into ISS energy grid).
Many stupid toys made it to ISS so I think something small, maybe of 1 kg weight could be sent. It would function, or not. But it would be the PROOF of usefulness.
Title: Re: EM Drive Developments - related to space flight applications - Thread 2
Post by: RotoSequence on 04/26/2015 11:34 am
It's possible my eyes glassed over at some point, so I apologize if this has already been answered. I thought the preferred next step for Eagleworks testing was a high power, broad frequency test series, but it sounds like this plan has been shelved. What's changed, and/or has been learned?
Title: Re: EM Drive Developments - related to space flight applications - Thread 2
Post by: Notsosureofit on 04/26/2015 12:19 pm
Harmonic content will excite higher modes.  X's will be (much?) larger ...

Picture shows only modulation

(No keyboard here)
Title: Re: EM Drive Developments - related to space flight applications - Thread 2
Post by: Rodal on 04/26/2015 02:31 pm
It's possible my eyes glassed over at some point, so I apologize if this has already been answered. I thought the preferred next step for Eagleworks testing was a high power, broad frequency test series, but it sounds like this plan has been shelved. What's changed, and/or has been learned?
The next step remains the same: higher power and modulation.   What is being discussed is what is the optimal way to achieve the desired modulation in a controlled , optimal manner, for example:

(Bold added for emphasis)

inject harmonic signals in a controlled manner into the frustum cavity that will modulate the fundamental resonant frequency established by the VCO.  A setup that is able to explore this harmonic content venue with additional on-purpose modulator and monitoring equipment that will of course require more time and money.

instead of using an old-fashioned magnetron (which have been around since the 1920's, but suffer from the magnetron's output changing from pulse to pulse, both in frequency and phase)

Since they had already reported that they were working on experiments using a magnetron (to try to reproduce the results of Shawyer in the UK and of Prof. Yang in China), my understanding is that nothing has been shelved, but what is being presently discussed is to perform first the magnetron experiments as originally planned and to simultaneously plan for this more sophisticated approach involving injecting harmonic signals in a controlled manner (which would involve getting more money and time, with the possible payoff to optimize the output of the EM Drive, to surpass the performance achieved in the UK and in China).  Sounds like logical R&D program planning to me.
Title: Re: EM Drive Developments - related to space flight applications - Thread 2
Post by: Star-Drive on 04/26/2015 02:51 pm
Harmonic content will excite higher modes.  X's will be (much?) larger ...

Picture shows only modulation

(No keyboard here)

"X's will be (much?) larger ..."

In Dr. White's (edit) mutable Quantum-Vacuum (Q-V) conjecture, since the Q-V compression effects scale with at least the time rate of change of the E&M energy-density phi (dPhi/dt) and dphi^2/dt^2 in the resonant cavity, the input power to thrust generation scaling should be VERY non-linear until the frustum design in question is at least generating a fully collimated Q-V beam, see previous 1kW to 100kW Q-V jet simulation work. However this non-linear thrust scaling with input power may not stop at just the fully collimated Q-V beam stage, provided there are higher than 4D dimensional interactions going on that could be extracting energy from the universe's cosmological gravitational field via several proposed thermodynamic based energy harvesting cycles that this Q-V technology could open up to us.

"Picture shows only modulation"

Good point, but at the moment MHz modulations of the RF carrier is all that we can perform since it would take another set of Phased Locked Loop (PLL) controlled VCOs and 100W microwave RF amplifiers to drive AND phase control the upper harmonics of the 1,937.2 MHz carrier signal with those being 3,874.4 MHz and 5,811.6 MHz at a minimum.  And at ~$6,500 each for vacuum compatible RF amps, that's not going to happen until we've already proven this concept to NASA management.  Once again the Chicken and egg problem. 

So at the moment we have to rely on larger amplitude MHz modulations of the 1,937.2 MHz carrier needed to generate reliable ~100uN signals, (the current sideband levels are approx. -50 dB down from the carrier).  And/or we fall back on the Eagleworks' in-development 1.2 kW magnetron & waveguide teeter-totter experiment that had better replicate Roger Shawyer's first generation EM-Drive's thrust generation capabilities of at least ~16 milli-Newton.  We are still looking for experimental first light date for this magnetron experiment as occurring by the end of June.  Hmmm, that should also be about the same time window that we can pull a vacuum in the Warp-field Interferometer test article as well.

Best, Paul M.
Title: Re: EM Drive Developments - related to space flight applications - Thread 2
Post by: Star-Drive on 04/26/2015 03:05 pm
Maybe stupid question... but why didn't NASA send small scale demonstrator to space?  It doesn't have to be heavy (resonance chamber doesn't have to be rated for outside pressure), it doesn't have to have even energy source (it could be plugged into ISS energy grid).
Many stupid toys made it to ISS so I think something small, maybe of 1 kg weight could be sent. It would function, or not. But it would be the PROOF of usefulness.

Answer: Risk  NASA management is a very conservative lot and they don't want to fly something that might not work.

Best, Paul M.
Title: Re: EM Drive Developments - related to space flight applications - Thread 2
Post by: CW on 04/26/2015 03:28 pm
It's about reputation. Even 0.1% risk of becoming an international laughing stock in case of experimental failure of this kind of revoltionary claim is too high. It's a prudent stance. Better crank up the (milli)Newtons beforehand ;) .
Title: Re: EM Drive Developments - related to space flight applications - Thread 2
Post by: Peter Svancarek on 04/26/2015 05:54 pm
It's about reputation. Even 0.1% risk of becoming an international laughing stock in case of experimental failure of this kind of revolutionary claim is too high. It's a prudent stance. Better crank up the (milli)Newtons beforehand ;) .

They may do it secretly. All they need is to send the experiment aboard launch vehicle of some probe as sub satellite... Alternatively they may use US air force x37 spaceplane. I think it is much better and more decisive to have real space test than anything possible here in gravity well. If it doesn't function even if it functioned in vacuum chamber, then earthside test were improperly set... Or your experiment needs to be adjusted to zero gravity.
China could have thought of something similar with their EM Drive experiment. Secrecy is easy :) If it functions then you tell the tale and take gratulations, if it doesn't then you tell no one.

I remember quite imposant tests of mechanic inertial engines ... mounted on boat, on pendulum etc but everything functioned through vibrations... It wouldn't function in zero gravity.
Title: Re: EM Drive Developments - related to space flight applications - Thread 2
Post by: Chris Bergin on 04/26/2015 06:01 pm
It's about reputation. Even 0.1% risk of becoming an international laughing stock in case of experimental failure of this kind of revolutionary claim is too high. It's a prudent stance. Better crank up the (milli)Newtons beforehand ;) .

They may do it secretly. All they need is to send the experiment aboard launch vehicle of some probe as sub satellite... Alternatively they may use US air force x37 spaceplane. I think it is much better and more decisive to have real space test than anything possible here in gravity well. If it doesn't function even if it functioned in vacuum chamber, then earthside test were improperly set... Or your experiment needs to be adjusted to zero gravity.
China could have thought of something similar with their EM Drive experiment. Secrecy is easy :) If it functions then you tell the tale and take gratulations, if it doesn't then you tell no one.

I remember quite imposant tests of mechanic inertial engines ... mounted on boat, on pendulum etc but everything functioned through vibrations... It wouldn't function in zero gravity.

I wouldn't get too deep into this. Even the Chinese birds aren't that secret.....(their secrecy is mostly about when they are launching). X-37B payloads are "secret" but nothing crazy. NROL is pretty secret, but people well-versed in those birds have a general idea what they do.

Nothing NASA does is all that secret, it just seems that way to outsiders as they aren't seeing enough "NASA" in the mainstream media and that's versus the conspiracy sites that claim rocks on Mars are actually proof of alien dishwashers ;)

If anything related to this goes into space in the future, I'll bet it won't be secret.
Title: Re: EM Drive Developments - related to space flight applications - Thread 2
Post by: Star One on 04/26/2015 06:06 pm
It's about reputation. Even 0.1% risk of becoming an international laughing stock in case of experimental failure of this kind of revolutionary claim is too high. It's a prudent stance. Better crank up the (milli)Newtons beforehand ;) .

They may do it secretly. All they need is to send the experiment aboard launch vehicle of some probe as sub satellite... Alternatively they may use US air force x37 spaceplane. I think it is much better and more decisive to have real space test than anything possible here in gravity well. If it doesn't function even if it functioned in vacuum chamber, then earthside test were improperly set... Or your experiment needs to be adjusted to zero gravity.
China could have thought of something similar with their EM Drive experiment. Secrecy is easy :) If it functions then you tell the tale and take gratulations, if it doesn't then you tell no one.

I remember quite imposant tests of mechanic inertial engines ... mounted on boat, on pendulum etc but everything functioned through vibrations... It wouldn't function in zero gravity.

The X-37B would be a good choice actually for a testbed, it is after all specifically designed for experimental returnable payloads. Also from what the Air Force have revealed about OTV-4 it is by coincidence testing an experimental propulsion system for the AFRL & also doing materials testing for NASA.
Title: Re: EM Drive Developments - related to space flight applications - Thread 2
Post by: sanman on 04/26/2015 07:03 pm
Can't you just test it in 2D by floating an apparatus in a pool of water or something? (suitably protecting it from the water to avoid any electrical hazards, of course)

Or what about hanging it like a pendulum, and then turning the power on and off to gradually make the pendulum swing from side to side?

Or what about using some sensitive torsion balance thing like Dr Stephen Lamoreaux did for his Casimir measurement experiment?
Title: Re: EM Drive Developments - related to space flight applications - Thread 2
Post by: Star One on 04/26/2015 07:05 pm

Can't you just test it in 2D by floating an apparatus in a pool of water or something? (suitably protecting it from the water to avoid any electrical hazards, of course)

Space is going to be gold standard test environment as far as I can see.
Title: Re: EM Drive Developments - related to space flight applications - Thread 2
Post by: sanman on 04/26/2015 07:09 pm
Oops, I just edited to add couple of other suggestions.

Well, space is expensive - maybe you can get a Kickstarter like Planetary Resources did, and send it up on a SpaceX flight.

But getting some kind of proof here on Earth would definitely spark massive interest towards testing it "in the wild" of space.
Title: Re: EM Drive Developments - related to space flight applications - Thread 2
Post by: Rodal on 04/26/2015 07:17 pm
People:

1) The present force measured at NASA Eagleworks for the EM Drive is only about 0.00001 pound force = 0.00005 Newtons.

2) NASA Eagleworks has been using a sensitive torsional pendulum.

3) The X-37B orbit is too low to verify propulsion by solar sailing with LightSail A, because atmospherical drag prevents solar sailing in the X-37 B orbit.  Only verification of deployment of the LightSail A solar sail is possible at the X-37B orbit.  It would act more like a drag sail in the very thin upper atmosphere around Earth: it would slow the solar sail and make it lose altitude.

4) What would be the financial or experimental justification to return to Earth an EM Drive? If none, that's another reason why deployment from the X-37B would not make financial or experimental sense.
Title: Re: EM Drive Developments - related to space flight applications - Thread 2
Post by: sanman on 04/26/2015 07:20 pm
Okay, points taken on 2,3,4 - but for point 1, why wouldn't the pendulum approach work?

Just keep cycling the power on and off until you gradually build up some large oscillation, and the thing is visibly moving from side-to-side in a significant way.
Title: Re: EM Drive Developments - related to space flight applications - Thread 2
Post by: Rodal on 04/26/2015 07:25 pm
Okay, points taken on 2,3,4 - but for point 1, why wouldn't the pendulum approach work?

Just keep cycling the power on and off until you gradually build up some large oscillation, and the thing is visibly moving from side-to-side in a significant way.
To do that you need a repeatable test with enough precision.  As you can see from the previous messages they are trying to address repeatability by increasing the power and modulation.
Title: Re: EM Drive Developments - related to space flight applications - Thread 2
Post by: Acryte on 04/26/2015 07:29 pm
Harmonic content will excite higher modes.  X's will be (much?) larger ...

Picture shows only modulation

(No keyboard here)

"X's will be (much?) larger ..."

In Dr. White's (edit) mutable Quantum-Vacuum (Q-V) conjecture, since the Q-V compression effects scale with at least the time rate of change of the E&M energy-density phi (dPhi/dt) and dphi^2/dt^2 in the resonant cavity, the input power to thrust generation scaling should be VERY non-linear until the frustum design in question is at least generating a fully collimated Q-V beam, see previous 1kW to 100kW Q-V jet simulation work. However this non-linear thrust scaling with input power may not stop at just the fully collimated Q-V beam stage, provided there are higher than 4D dimensional interactions going on that could be extracting energy from the universe's cosmological gravitational field via several proposed thermodynamic based energy harvesting cycles that this Q-V technology could open up to us.

"Picture shows only modulation"

Good point, but at the moment MHz modulations of the RF carrier is all that we can perform since it would take another set of Phased Locked Loop (PLL) controlled VCOs and 100W microwave RF amplifiers to drive AND phase control the upper harmonics of the 1,937.2 MHz carrier signal with those being 3,874.4 MHz and 5,811.6 MHz at a minimum.  And at ~$6,500 each for vacuum compatible RF amps, that's not going to happen until we've already proven this concept to NASA management.  Once again the Chicken and egg problem. 

So at the moment we have to rely on larger amplitude MHz modulations of the 1,937.2 MHz carrier needed to generate reliable ~100uN signals, (the current sideband levels are approx. -50 dB down from the carrier).  And/or we fall back on the Eagleworks' in-development 1.2 kW magnetron & waveguide teeter-totter experiment that had better replicate Roger Shawyer's first generation EM-Drive's thrust generation capabilities of at least ~16 milli-Newton.  We are still looking for experimental first light date for this magnetron experiment as occurring by the end of June.  Hmmm, that should also be about the same time window that we can pull a vacuum in the Warp-field Interferometer test article as well.

Best, Paul M.

Although any sum below $100-400k could be readily obtained through something simple like a kickstarter for means of R&D (due to prior popularization of the concepts amongst sci-fi faithfuls), I'm assuming that there is a strong aversion to such methods because contributions made from the general public are likely to come with pressure to produce a tangible end-product and not simply confirm or disprove a scientific hypothesis? Judging by how lightly speculation and confirmation is being handled by Dr. White and colleagues I'm assuming the development of expectation and the pressures of it are the primary reason such avenues are unfavorable even if they could fast-track development... Basically, the last thing they want is to become the latest cold fusion incarnation. Is this an accurate assessment?
Title: Re: EM Drive Developments - related to space flight applications - Thread 2
Post by: sanman on 04/26/2015 07:39 pm
To do that you need a repeatable test with enough precision.  As you can see from the previous messages they are trying to address repeatability by increasing the power and modulation.

Hmm, just regarding the precision issue: I realize that my suggestion is more akin to a garage-tinkerer's approach rather than rigorous experimental determination. But I suggested it because the primary issue seems to be about demonstrating that this phenomenon/effect even exists at all, rather than what its exact quantifiable parameters are.

When I push a kid on a swingset, each of my pushes is not an exact repetition of the previous ones, and yet in spite of this the swinging motion will eventually build up, even if I only have the strength of a mouse and the kid has the mass of a SaturnV.

If there really is a signal here, won't enough simple additive repetition eventually make it visible over the noise -- ie. make the kid on the swingset move -- even if each push isn't identical?
Title: Re: EM Drive Developments - related to space flight applications - Thread 2
Post by: sanman on 04/26/2015 07:53 pm
Although any sum below $100-400k could be readily obtained through something simple like a kickstarter for means of R&D (due to prior popularization of the concepts amongst sci-fi faithfuls), I'm assuming that there is a strong aversion to such methods because contributions made from the general public are likely to come with pressure to produce a tangible end-product and not simply confirm or disprove a scientific hypothesis? Judging by how lightly speculation and confirmation is being handled by Dr. White and colleagues I'm assuming the development of expectation and the pressures of it are the primary reason such avenues are unfavorable even if they could fast-track development... Basically, the last thing they want is to become the latest cold fusion incarnation. Is this an accurate assessment?

Kickstarter doesn't work that way -- in exchange for receiving the donations, all you have to do is deliver these side-rewards (eg. the promised T-shirt, or mousepad, or coffee mug, etc)
Yes, people are mainly donating because they want to support you in finding the Holy Grail, but that's not what you're legally bound to deliver.

But maybe you should open-source the experimentation, like the Polywell people. More efforts in parallel can mean more progress.
Title: Re: EM Drive Developments - related to space flight applications - Thread 2
Post by: ThinkerX on 04/26/2015 08:21 pm
Quote
Although any sum below $100-400k could be readily obtained through something simple like a kickstarter for means of R&D (due to prior popularization of the concepts amongst sci-fi faithfuls), I'm assuming that there is a strong aversion to such methods because contributions made from the general public are likely to come with pressure to produce a tangible end-product and not simply confirm or disprove a scientific hypothesis? Judging by how lightly speculation and confirmation is being handled by Dr. White and colleagues I'm assuming the development of expectation and the pressures of it are the primary reason such avenues are unfavorable even if they could fast-track development... Basically, the last thing they want is to become the latest cold fusion incarnation. Is this an accurate assessment?

This sort of thing has been suggested more than once in this thread.  Apparently, the Eagleworks team cannot accept any outside money or equipment.
Title: Re: EM Drive Developments - related to space flight applications - Thread 2
Post by: Peter Svancarek on 04/26/2015 09:20 pm
Can't you just test it in 2D by floating an apparatus in a pool of water or something? (suitably protecting it from the water to avoid any electrical hazards, of course)

Or what about hanging it like a pendulum, and then turning the power on and off to gradually make the pendulum swing from side to side?

Or what about using some sensitive torsion balance thing like Dr Stephen Lamoreaux did for his Casimir measurement experiment?

Pendulum is not good for experiment. Vibrations can induce movement. I saw that with so called mechanical inertial engines . The same vibrations can move a boat. I would be also careful to believe in torsion balance.

But what about submerged case with thruster in it? That case would have to have spherical shape to eliminate possibility of by vibration induced movement. The case should be thermally isolated with at least polyurethane (aerogel would be better) so thermally induced flow would be reduced.

EM Drive is not solar sail, it could be small and it would show results even on low orbit.  Also it is not something very conspicuous if you don't want it look as such. You can mask it by solar collectors... NASA is public, I agree. I don't believe in conspiratorial theories much. I'm sure there are some things which are secret, but I'm sure there are not such hare-brained reasons as some people are thinking. It all goes whether you need for it to be secret because you are afraid of public. It is better to know whether that thruster is feasible or it is not. All those experiments here on earth are maybe more expensive than one try in zero G.  1kg experiment wouldn't be that costly to get on the orbit... I believe it is about 10.000$ per kg.

Just a thought... What about persuading Elon Musk for this experiment?
Title: Re: EM Drive Developments - related to space flight applications - Thread 2
Post by: Rodal on 04/26/2015 10:04 pm
Kurt Zeller and Brian Kraft at California Polytechnic State University in San Luis Obispo, CA, just published this (April 13, 2015) in the AIAA:

Investigation and Analysis of Anomalous Electromagnetic Propulsion Devices
Kurt Zeller and Brian Kraft
http://www.slideshare.net/KurtZeller/investigation-and-analysis-of-anomalous-electromagnetic-propulsion-devices-41315-46946953

Notice that their last reference (#8) is a very recent post (April 5, 2015) by Paul March in this NASA SpaceFlight Forum, and a lot of the material they discussed can be found in previous pages of the EM Drive threads at NASA SpaceFlight Forum.
Title: Re: EM Drive Developments - related to space flight applications - Thread 2
Post by: Rodal on 04/26/2015 10:31 pm
Elon Musk is aware that his first two initials are identical with the first two initials of "EM Drive"   ;)  , and has been aware for several years about the EM Drive work in the UK, China and the US, as well as being aware of Dr. White's research on warping spacetime.  In an older interview, Musk laughed and basically said that there have been breakthroughs in the field, presumably referring to NASA’s recent work, but such technology "isn’t on SpaceX’s immediate roadmap".
Title: Re: EM Drive Developments - related to space flight applications - Thread 2
Post by: Star One on 04/26/2015 10:34 pm

Elon Musk is aware that his first two initials are identical with the first two initials of "EM Drive"   ;)  , and has been aware for several years about the EM Drive work in the UK, China and the US, as well as being aware of Dr. White's research on warping spacetime.  In an older interview, Musk laughed and basically said that there have been breakthroughs in the field, presumably referring to NASA’s recent work, but such technology "isn’t on SpaceX’s immediate roadmap".

I noticed Space X were the conference sponsors on one of those videos featuring Dr White.
Title: Re: EM Drive Developments - related to space flight applications - Thread 2
Post by: sanman on 04/26/2015 11:22 pm
What kind of earth-side experiment can be done to very tangibly and visibly demonstrate that the effect exists?

Because even if you do an experiment in LEO, then people will claim "oh, it's probably just some magnetic field from the equipment interacting with the Earth's magnetic field"

Isn't it better to try and create an experimental result that people can see with their own eyes and can be captured on video?

When Low-Energy-Nuclear-Reactions fell into controversy, there were still outfits like US Naval Research Laboratories who were willing to keep poking at the problem. Even the more recent Polywell Fusion results were done there. Unfortunately, massless propulsion doesn't seem to have a compelling military application the way that compact energy does.

Would some farsighted group like DARPA ever be interested in this research?

Title: Re: EM Drive Developments - related to space flight applications - Thread 2
Post by: aceshigh on 04/27/2015 02:53 am

Elon Musk is aware that his first two initials are identical with the first two initials of "EM Drive"   ;)  , and has been aware for several years about the EM Drive work in the UK, China and the US, as well as being aware of Dr. White's research on warping spacetime.  In an older interview, Musk laughed and basically said that there have been breakthroughs in the field, presumably referring to NASA’s recent work, but such technology "isn’t on SpaceX’s immediate roadmap".

I noticed Space X were the conference sponsors on one of those videos featuring Dr White.

which is quite different from having warp drive on SpaceX roadmap. Or at least, on their PUBLIC roadmap. :)
Title: Re: EM Drive Developments - related to space flight applications - Thread 2
Post by: Star-Drive on 04/27/2015 03:42 am
Okay, points taken on 2,3,4 - but for point 1, why wouldn't the pendulum approach work?

Just keep cycling the power on and off until you gradually build up some large oscillation, and the thing is visibly moving from side-to-side in a significant way.
To do that you need a repeatable test with enough precision.  As you can see from the previous messages they are trying to address repeatability by increasing the power and modulation.

Dr. Rodal:

You also need to add dc parametric amplification to the above list of possible EM-Drive improvements.  Applying large dc E-fields and/or B-fields in a complimentary way to a particular frustum ac resonance like the TM011 could also greatly increase the thrust generation potential of these devices.

Best,  Paul M.
Title: Re: EM Drive Developments - related to space flight applications - Thread 2
Post by: birchoff on 04/27/2015 04:01 am
I wonder if in light of the range of parameters that need to be evaluated if it wouldnt be beneficial to pull togther a set of experiment descriptions that outside replicators could attempt?
Title: Re: EM Drive Developments - related to space flight applications - Thread 2
Post by: R.W. Keyes on 04/27/2015 04:14 am
Some questions and ideas.

Eagleworks can't accept donations, I think I read. Is that universally true, or just true from random people? If we were to establish "Space Flight Research Foundation", a NPO, which would buy and lend Eagleworks equipment like these vacuum-capable RF amplifiers, would that somehow fit in the rules?

Regarding amplifers: Magnetrons are cheap and dirty, microwave-oven equipment. But am I correct in saying that someone, Perhaps Paul March, said that the reason why the Chinese and Shawyer designs work, in spite of having no dielectric, is that their messy RF eliminated the need for one? Can someone explain how this might be?

My understanding is that there is increased efficiency by the use of a dielectric and a cleaner, more stable sine wave, such as that produced by a PLL, and amplified by the use of a linear amplifier using a device such as a TWT (Travelling Wave Tube), but the problem is that the capacitors used are not vacuum-proof, as they leak electrolyte at vacuum/low pressure. I don't understand why the amplifier itself, or at least the section containing the capacitors, can't be kept in a pressurized compartment.

Please don't think these are leading questions, I don't claim to be a professional. But am an interested hobbyist. I have it in my mind to produce a Shawyer-like first-gen EmDrive (i.e. without the piezoelectric compensator) constructed out of Magnesium diboride and cooled with liquid helium (yes, higher-temp superconductors would allow me to use liquid nitrogen, but coating the inside of the frustum with such a material seems to be more difficult), and drive it with a 20KW Magnetron. Yes, I know, a crude attempt, which is why I am learning more before I go about this project. My thought is that by using such a high power, I would not need very accurate or precise thrust measurement, as it should be pretty obvious if it's working, and of course I wouldn't be doing it in a vacuum.

I think Eagleworks is doing a great job, and I eagerly await the results of further tests.
Title: Re: EM Drive Developments - related to space flight applications - Thread 2
Post by: HMXHMX on 04/27/2015 05:20 am
Some questions and ideas.

Eagleworks can't accept donations, I think I read. Is that universally true, or just true from random people? If we were to establish "Space Flight Research Foundation", a NPO, which would buy and lend Eagleworks equipment like these vacuum-capable RF amplifiers, would that somehow fit in the rules?

Regarding amplifers: Magnetrons are cheap and dirty, microwave-oven equipment. But am I correct in saying that someone, Perhaps Paul March, said that the reason why the Chinese and Shawyer designs work, in spite of having no dielectric, is that their messy RF eliminated the need for one? Can someone explain how this might be?

My understanding is that there is increased efficiency by the use of a dielectric and a cleaner, more stable sine wave, such as that produced by a PLL, and amplified by the use of a linear amplifier using a device such as a TWT (Travelling Wave Tube), but the problem is that the capacitors used are not vacuum-proof, as they leak electrolyte at vacuum/low pressure. I don't understand why the amplifier itself, or at least the section containing the capacitors, can't be kept in a pressurized compartment.

Please don't think these are leading questions, I don't claim to be a professional. But am an interested hobbyist. I have it in my mind to produce a Shawyer-like first-gen EmDrive (i.e. without the piezoelectric compensator) constructed out of Magnesium diboride and cooled with liquid helium (yes, higher-temp superconductors would allow me to use liquid nitrogen, but coating the inside of the frustum with such a material seems to be more difficult), and drive it with a 20KW Magnetron. Yes, I know, a crude attempt, which is why I am learning more before I go about this project. My thought is that by using such a high power, I would not need very accurate or precise thrust measurement, as it should be pretty obvious if it's working, and of course I wouldn't be doing it in a vacuum.

I think Eagleworks is doing a great job, and I eagerly await the results of further tests.

Anything to do with NASA and non-gov't money is always tricky so I'll take a swing at commenting on the first part of your inquiry. 

NASA can't take money via Kickstarters and such; that's pretty much a given.  But NASA can cooperate with both for-profit and non-profit corporate organizations, provided the necessary contractual arrangements are made.  Usually these involve a so-called "Space Act' agreement.  I've been involved with several, and they can be fairly straightforward but time-consuming to negotiate.

A bit of history might also be enlightening.  Paul will have to remind me on the dates, but before he and Sonny established Eagleworks at NASA JSC, the two of us and myself actually set it up as a small R&D company, which I funded for about a year.  Unfortunately, I couldn't keep up the support required, and after Sonny was awarded his PhD, thankfully JSC found funding and facilities for them to use to keep the dream alive.  Meanwhile, I became President and a Trustee of the Space Studies Institute (www.ssi.org) and we established a fund to support "Exotic Propulsion" which is named (appropriately but uninspiringly*) the "Exotic Propulsion Initiative.  It is possible to donate to the fund at our website.  SSI is a 501(c)3 non-profit and thus contributions are tax-deductiable.  Targeted contributions go almost 100% to the named projects since we have very modest overhead costs (we don't pay salaries to our volunteer staff, for example).

SSI is currently supporting the work of Prof. Woodward, but it has always been our intent to expand the base of researchers as resources permit.  At the moment about the best we can do is to buy equipment and fund the occasional student intern, rather than pay for principal investigator labor, but that could be enough to help out more than one lab, JSC Eagleworks included.  So no need to set up another organization to help – we are here and willing to be involved if the need can be articulated.

Now back to your regularly scheduled programming...er, discussion...which has been very enjoyable to follow!

(*I think I made that word up.)
Title: Re: EM Drive Developments - related to space flight applications - Thread 2
Post by: Star-Drive on 04/27/2015 12:36 pm
Some questions and ideas.

Eagleworks can't accept donations, I think I read. Is that universally true, or just true from random people? If we were to establish "Space Flight Research Foundation", a NPO, which would buy and lend Eagleworks equipment like these vacuum-capable RF amplifiers, would that somehow fit in the rules?

Regarding amplifers: Magnetrons are cheap and dirty, microwave-oven equipment. But am I correct in saying that someone, Perhaps Paul March, said that the reason why the Chinese and Shawyer designs work, in spite of having no dielectric, is that their messy RF eliminated the need for one? Can someone explain how this might be?

My understanding is that there is increased efficiency by the use of a dielectric and a cleaner, more stable sine wave, such as that produced by a PLL, and amplified by the use of a linear amplifier using a device such as a TWT (Travelling Wave Tube), but the problem is that the capacitors used are not vacuum-proof, as they leak electrolyte at vacuum/low pressure. I don't understand why the amplifier itself, or at least the section containing the capacitors, can't be kept in a pressurized compartment.

Please don't think these are leading questions, I don't claim to be a professional. But am an interested hobbyist. I have it in my mind to produce a Shawyer-like first-gen EmDrive (i.e. without the piezoelectric compensator) constructed out of Magnesium diboride and cooled with liquid helium (yes, higher-temp superconductors would allow me to use liquid nitrogen, but coating the inside of the frustum with such a material seems to be more difficult), and drive it with a 20KW Magnetron. Yes, I know, a crude attempt, which is why I am learning more before I go about this project. My thought is that by using such a high power, I would not need very accurate or precise thrust measurement, as it should be pretty obvious if it's working, and of course I wouldn't be doing it in a vacuum.

I think Eagleworks is doing a great job, and I eagerly await the results of further tests.

R.W. Keys:

"Regarding amplifiers: Magnetrons are cheap and dirty, microwave-oven equipment. But am I correct in saying that someone, Perhaps Paul March, said that the reason why the Chinese and Shawyer designs work, in spite of having no dielectric, is that their messy RF eliminated the need for one?  Can someone explain how this might be?"

To tell you how one could create thrust without a dielectric all depends on what physical model is actually at work in producing the observed thrust effect in these frustum EM-Drives.  As far as I know there are only two robust conjectures that can explain how these "propellant-less" EM-Drives work. 

The first and earliest conjecture is Dr. Woodward's Mach-Effect conjecture that assumes that the thruster directly interacts with cosmological gravitational field via local E&M induced transient inertial mass fluctuations in a dielectric predicted by special and general relativity (SRT & GRT), without stating what the cosmological gravitational field actually is at its lowest level, other than stating it is created by all the mass/energy in the causally connected universe.  Some would say that this rules out the M-E in regards to the cause of the dielectric-less frustums generating thrust, but are any of them really dielectric free?  Please remember that our copper frustum has a baked on silicone PCB anti-oxidation ~0.001" thick coating on its interior surfaces to keep the copper surfaces from oxidizing and thus lowering its Q-factor over time.  Is that enough dielectric to keep the M-E in the running?  TBD but perhaps it is at high enough power levels and/or modulation techniques.

The second conjecture by Dr. White that might explain these results also posits that these EM-Drives interact with the cosmological gravitational field to generate thrust via SRT & GRT based interactions, but goes one step further in stating that the cosmological gravitational field IS the Quantum-Vacuum (Q-V) pressure field that's also created by all the mass & energy in the causally connected universe.  This is a subtle difference I know, but in doing so one can now treat gravity as an emergent force that can be manipulated with the application of local E&M fields using plasma physics' Magneto-Hydro-Dynamic (MHD) rules since the Q-V is consider to be a neutral electrical plasma made primarily from the transient e/p pairs in the Dirac sea.  It also allows one to think of transient mass fluctuations as actually being density fluctuations in the Q-V that can be generated hydrodynamically via Bernoulli pressure effects, etc.  Just as in a wing with air flowing over it can create lift via the Bernoulli effect, just by the SHAPE of the wing surfaces, a frustum can generate Q-V plasma flows under E&M excitations, AKA thrust, just by the shape of its topology with no need for dielectrics.

"I don't understand why the amplifier itself, or at least the section containing the capacitors, can't be kept in a pressurized compartment."

We can do that and where about ready to do so when we thought we had found an off the shelf "hermetically" sealed RF amplifier that didn't need an extra pressure sealed box around it.  However we found that EMPower's hermitically sealed boxes really weren't, but since they didn't use electrolytics caps in their design, all we had to do was vent this RF amp's interior to the vacuum and make sure that we didn't try to run the RF amp while in the corona discharge pressure region during depressing and pressurizing the vacuum chamber.

Best, Paul M.
Title: Re: EM Drive Developments - related to space flight applications - Thread 2
Post by: frobnicat on 04/27/2015 03:44 pm
From this post (http://forum.nasaspaceflight.com/index.php?topic=36313.msg1364270#msg1364270), how do we interpret the "RF Dissipated Power" in the central caption of this slide ?

(http://forum.nasaspaceflight.com/index.php?action=dlattach;topic=36313.0;attach=826537;image)
bigger (http://forum.nasaspaceflight.com/index.php?action=dlattach;topic=36313.0;attach=826536;image)

Is it the (DC) power input to RF amplifier ?  Or the difference of RF power input to frustum minus reflected power back to amplifier ? What is measured exactly ? What is horizontal scale, is the != 0 part 16.5s long like the power-on period of the thrust chart ? Why this particular profile with initial spike and 2 plateaus ? Why don't we see a corresponding 3x magnitude "step" on the thrust chart at half the excitation time ?

I'm not sure how to interpret the next one either :
(http://forum.nasaspaceflight.com/index.php?action=dlattach;topic=36313.0;attach=826539;image)
bigger (http://forum.nasaspaceflight.com/index.php?action=dlattach;topic=36313.0;attach=826538;image)

Is it to say that when the spectrum is broader the initial slope on thrust chart is steeper ? On most other charts ~10s of power-on is enough to reach a plateau (kind of), why isn't it the case here ? Seems to me this is magnitude of plateau vs spectrum width that would be relevant, not slope, too much noise on top of those transients. For instance, the decay after power-off looks like having a significantly longer time constant with the richer spectrum, is it relevant ?
Title: Re: EM Drive Developments - related to space flight applications - Thread 2
Post by: D_Dom on 04/27/2015 03:52 pm
Meanwhile, I became President and a Trustee of the Space Studies Institute (www.ssi.org) and we established a fund to support "Exotic Propulsion" which is named (appropriately but uninspiringly*) the "Exotic Propulsion Initiative.  It is possible to donate to the fund at our website.  SSI is a 501(c)3 non-profit

Personally I have always found your leadership to be inspiring, thanks for establishing Exotic Propulsion Initiative!
Title: Re: EM Drive Developments - related to space flight applications - Thread 2
Post by: Rodal on 04/27/2015 04:03 pm
From this post (http://forum.nasaspaceflight.com/index.php?topic=36313.msg1364270#msg1364270), how do we interpret the "RF Dissipated Power" in the central caption of this slide ?
...

WELCOME BACK @frobnicat !!!

We missed you !  :)
Title: Re: EM Drive Developments - related to space flight applications - Thread 2
Post by: frobnicat on 04/27/2015 04:58 pm
@ Paul March,

Quite a number of concerns that have been raised here about the frustum assembly (PCB caps and nylon screws...) could be dismissed with the Cannae test article. But there appear to have no report of test of Cannae in vacuum. A Cannae test in vacuum would have the advantage of answering both concerns of frustum assembly specifics and of air interactions, the test article exists and can be run in the same conditions with only the ambient pressure parameter different (from 2014 campaign results) making comparisons easier : do you have or plan to test Cannae in vacuum ?

Also, I for one, would really like to see a few recorded charts for "failed" thrust pulses, the ones that look "in between" (and their conditions of appearance), and most importantly at least one (preferably a few) of the charts showing "no significant thrust" without dielectrics.

...
How does one report "negative" results when at times the acquired data just doesn't make sense?  I've been plowing through literally hundreds of copper frustum tests over the last year with various RF tuning configurations and finding that some appear to generate nothing but spurious thermal like results as demonstrated by their positive and then negative going gradual exponential rise and fall times, others that show a very prompt signal at RF turn-on and turn-off comparable to the electrostatic force calibrator on/off slopes, overlaid with the above thermal signatures and some that fall in between.  The only real way I can make sure the "thrust" plots I've been generating are real thrust signatures is to first check for a prompt signal during the first ~5 seconds before thermal effects take hold and then going into reverse thrust mode where the thrust signature opposes the thermal signature to the point it goes negative like the one I've already appended but repeat it here.  Any other testing approaches to cull out these blasted thermal signatures would be appreciated. 

Running the horizontal pendulum strictly horizontal would help, at least for the displacement of centres of mass thermal effects. I understand a slight tilt of the vertical axis is helpful (needed) to stabilize to a confortable rest position, and that there is in the present configuration no way to compensate for the rest position (in rotation around vertical) of the flexure bearings as they appear to be fixed in angular position (both on the fixed part and on the pendulum arm).

Wouldn't it be relatively straightforward to add 1 or 2 tension springs between pendulum arm and fixed part, near axis of rotation (weak apparent added stiffness), with a position tuning system (basically pulling more or less one way or the other), to settle the problem of the rest position without resorting to tilting the whole platform : that has the very unfortunate consequence of making the system sensitive to displacements of CoMs, I mean not only as recoil effects but as sustained signals. A strict horizontal setup would eliminate the sustained component of CoMs shifts, making it way more convincing to interpret long lasting plateaus as thrust, as the only remaining CoMs shifts effects would be second order (can't last for long).

Quote
BTW, when the RF is first turned on we literally have an RF induction heating system that immediately starts warming the copper cavity walls, especially at the large OD end of the frustum.  How long does that RF induced thermal heating take to start moving the copper frustum and to what degree?  Looks like another COMSOL problem that will take into account the specific heat of all the frustum components, then profiles the resulting differential temp rise of cone that then generates a frustum expansion rate that will then have to feed into a model of the torque pendulum's deflection sensitivity to off axis loading.  Yuck!   

Best, Paul M.

Not only that but also the probable glass transitions of the nylon screws and the subsequent possible buckling of PCB end plates and/or change of shape relative to their natural "warp" and this partial release of constraints... Yes, this is not pretty but this needs to be done. But the Cannae in vacuum seems both experimentally cleaner and easier to model.

And, sorry if I sound insistent, but the vertical scale variability + apparent inconsistency of stiffness (wrt known parameters) needs really to be addressed and settled one day or another, especially since we see contradictory data when the balance is used in reverse (180° turn of test article that should yield same deflection magnitudes to the left or to the right, assuming same thrusts magnitudes). A central aspect of the pendulum appears to be not properly characterised. Having to rely only on calibration pulses and proportionality is not satisfactory, given the unusual physics involved and the general scepticism, everything must fit. If one were told that a scale had 10cm long arms, but dynamics (oscillation periods) showed it was rather 1m long arms, the fact that this wont change the end result (equal weight on the plates...) is not satisfactory, there would still be a 1 order of magnitude unexplained aspect at the heart the experiment.

(http://upload.wikimedia.org/wikipedia/commons/2/24/Eo-scale_of_justice.gif)

Thanks for the regular feedbacks and open stance.
Title: Re: EM Drive Developments - related to space flight applications - Thread 2
Post by: frobnicat on 04/27/2015 05:06 pm
From this post (http://forum.nasaspaceflight.com/index.php?topic=36313.msg1364270#msg1364270), how do we interpret the "RF Dissipated Power" in the central caption of this slide ?
...

WELCOME BACK @frobnicat !!!

We missed you !  :)

Thanks dr Rodal ! This is an attempt at a come back but not sure I will be able to contribute much in the coming months... hopefully there will be more data to answer the aspects I won't have time to tackle by modelling. Still enjoy reading, keep it alive !
Title: Re: EM Drive Developments - related to space flight applications - Thread 2
Post by: aceshigh on 04/27/2015 06:26 pm

A bit of history might also be enlightening.  Paul will have to remind me on the dates, but before he and Sonny established Eagleworks at NASA JSC, the two of us and myself actually set it up as a small R&D company, which I funded for about a year.  Unfortunately, I couldn't keep up the support required, and after Sonny was awarded his PhD, thankfully JSC found funding and facilities for them to use to keep the dream alive.  Meanwhile, I became President and a Trustee of the Space Studies Institute (www.ssi.org) and we established a fund to support "Exotic Propulsion" which is named (appropriately but uninspiringly*) the "Exotic Propulsion Initiative.  It is possible to donate to the fund at our website.  SSI is a 501(c)3 non-profit and thus contributions are tax-deductiable.  Targeted contributions go almost 100% to the named projects since we have very modest overhead costs (we don't pay salaries to our volunteer staff, for example).

SSI is currently supporting the work of Prof. Woodward, but it has always been our intent to expand the base of researchers as resources permit.  At the moment about the best we can do is to buy equipment and fund the occasional student intern, rather than pay for principal investigator labor, but that could be enough to help out more than one lab, JSC Eagleworks included.  So no need to set up another organization to help – we are here and willing to be involved if the need can be articulated.

Now back to your regularly scheduled programming...er, discussion...which has been very enjoyable to follow!

(*I think I made that word up.)

I am sorry I don´t know by name more people there, but I was surprised and happy to see Prof. Freeman Dyson on the board of trustees of an organization funding ME research and possibly if everything goes right, also EM research.

(Paul March says both are possibly two sides of the same coin, so it cracks me up ME and EM are mirrored acronyms!  ;D)
Title: Re: EM Drive Developments - related to space flight applications - Thread 2
Post by: randyhowk on 04/27/2015 06:55 pm
Thank you so much for this! I asked this Question almost a year ago, and have been wondering about it ever since the microwave version of the EM drive came out years ago.
http://physics.stackexchange.com/questions/129799/wavelength-and-frustum-shaped-resonant-cavity
Title: Re: EM Drive Developments - related to space flight applications - Thread 2
Post by: Rodal on 04/27/2015 07:16 pm
Thank you so much for this! I asked this Question almost a year ago, and have been wondering about it ever since the microwave version of the EM drive came out years ago.
http://physics.stackexchange.com/questions/129799/wavelength-and-frustum-shaped-resonant-cavity
The answers in http://physics.stackexchange.com/questions/129799/wavelength-and-frustum-shaped-resonant-cavity are incorrect regarding your question of a closed truncated cone resonating with standing electromagnetic waves. (Obviously electromagnetic resonance must obey Maxwell's equations, and it does not correspond to acoustic resonance).

Exact solutions can, and have been obtained, starting by the great Russian/American electrical engineer/scientist Schelkunoff in the late 1930's http://en.wikipedia.org/wiki/Sergei_Alexander_Schelkunoff who was at Bell Labs.

Here is a picture of Schelkunoff studying waveguide transmission in the early 1930’s (take a gander at the conical shape of the waveguides being examined by Schelkunoff in this picture  ;) ):
(http://www.ieeecincinnati.org/wp-content/uploads/2010/03/2010_04-fig-1.JPG)
(Reprinted from P. C. Mahon, Mission Communications: The Story of Bell Laboratories, 1975.)

There are two eigenvalue problems in the exact solution for the truncated cone.  One eigenvalue problem contains associated Legendre functions and the other eigenvalue problem contains spherical Bessel functions. The eigenvalue problems for the exact solution of the truncated cone need, therefore, to be solved numerically .  See this for example, regarding solutions for modes that are constant in the transverse direction:

http://gregegan.customer.netspace.net.au/SCIENCE/Cavity/Cavity.html

I have obtained an exact solution for arbitrarily changing electromagnetic fields in the transverse direction that compares excellently with NASA's results using the Finite Element program COMSOL.  I have published several results earlier in the thread, including the Poynting vector (Poynting vector results which I have not seen elsewhere in the literature, either numerically or by exact solution).

The important thing for the truncated cone (besides obvious feature like modes are progressively cutoff depending on the cone angle) is that the truncated cone presents features like attenuation and focusing in the longitudinal direction that are completely absent in the cylindrical resonant cavity.

See attached below the enclosed Power Point file for the Poynting vector fields for the electromagnetic mode (similar to TM212 of cylindrical cavities) without a dielectric polymer inserted in the cavity.
Title: Re: EM Drive Developments - related to space flight applications - Thread 2
Post by: Rodal on 04/27/2015 08:13 pm

A bit of history might also be enlightening.  Paul will have to remind me on the dates, but before he and Sonny established Eagleworks at NASA JSC, the two of us and myself actually set it up as a small R&D company, which I funded for about a year.  Unfortunately, I couldn't keep up the support required, and after Sonny was awarded his PhD, thankfully JSC found funding and facilities for them to use to keep the dream alive.  Meanwhile, I became President and a Trustee of the Space Studies Institute (www.ssi.org) and we established a fund to support "Exotic Propulsion" which is named (appropriately but uninspiringly*) the "Exotic Propulsion Initiative.  It is possible to donate to the fund at our website.  SSI is a 501(c)3 non-profit and thus contributions are tax-deductiable.  Targeted contributions go almost 100% to the named projects since we have very modest overhead costs (we don't pay salaries to our volunteer staff, for example).

SSI is currently supporting the work of Prof. Woodward, but it has always been our intent to expand the base of researchers as resources permit.  At the moment about the best we can do is to buy equipment and fund the occasional student intern, rather than pay for principal investigator labor, but that could be enough to help out more than one lab, JSC Eagleworks included.  So no need to set up another organization to help – we are here and willing to be involved if the need can be articulated.

Now back to your regularly scheduled programming...er, discussion...which has been very enjoyable to follow!

(*I think I made that word up.)

I am sorry I don´t know by name more people there, but I was surprised and happy to see Prof. Freeman Dyson on the board of trustees of an organization funding ME research and possibly if everything goes right, also EM research.

(Paul March says both are possibly two sides of the same coin, so it cracks me up ME and EM are mirrored acronyms!  ;D)

Yes, Prof. Freeman Dyson used to be the President of the Space Studies Institute, he was a member of the Board of Trustees as of 2013.   The Space Studies Institute was founded by Gerard K. O'Neill . O'Neill (diagnosed with terminal leukemia) directed his Space Studies Institute to continue his efforts "until people are living and working in space". After G. O'Neill's death, management of SSI was passed to his son Roger O'Neill and Prof. Freeman Dyson.

Dyson is now 91 years old, here is a younger picture of him  ;) , and also a picture of a Project Orion spaceship and of G. K. O'Neill's with MIT students building the first mass driver prototype (I remember him when he was a Visiting Professor during the 1976–77 academic year at MIT -great times  :) -):

(http://www.towntopics.com/wordpress/wp-content/uploads/2013/09/page32.jpg)

(https://encrypted-tbn1.gstatic.com/images?q=tbn:ANd9GcQbmUOF2sYoWYNj5OG29GuHcQ9RT9gehZK4ZFH7wHmlaeNxpUyX)

(http://www.nss.org/settlement/nasa/spaceresvol2/images/electrofig16a.GIF)

(http://upload.wikimedia.org/wikipedia/commons/e/e4/ONEILLMASS.GIF)

(https://spacefrontier.org/wp-content/uploads/2012/04/HighFrontier1stEd.jpeg)
Title: Re: EM Drive Developments - related to space flight applications - Thread 2
Post by: Rodal on 04/27/2015 10:53 pm
....I became President and a Trustee of the Space Studies Institute (www.ssi.org) and we established a fund to support "Exotic Propulsion" which is named ....SSI is currently supporting the work of Prof. Woodward, but it has always been our intent to expand the base of researchers as resources permit.  ...

Mr. Hudson,

Thanks so much for that great post.  It would be interesting to know whether Prof. Freeman Dyson knows about Prof. Woodward's work (*) on the Mach Effect and/or about Dr. White's work on a mutable Quantum Vacuum, and if he does, what does Prof. Dyson think about those conjectures.  I have tried to find out in the literature and I don't see anything that Dyson has published in this regard.

I did see that Dyson wrote that he believed that NASA's big and visible projects, the Shuttle and the Space Station, will in the future "appear as quaint and misguided gargantuan ventures in the wrong direction, the von Hindenbergs and Titanics of the late 20th century" and that manned spaceflight will use new launch technologies that will make chemical rockets seem absurdly inefficient.

_______________
(*) I cannot assume necessary agreement, since for example, Prof. Freeman Dyson wrote in his paper "Pilgrims, Saints, and Spacemen" that he disagreed with G.K. O'Neill on the economic viability of some of O'Neill's concepts.  This although they were great partners, as proven by the fact that O'Neill asked Freeman Dyson to carry on the responsibilities of the Space Studies Institute.
Title: Re: EM Drive Developments - related to space flight applications - Thread 2
Post by: HMXHMX on 04/27/2015 11:30 pm
....I became President and a Trustee of the Space Studies Institute (www.ssi.org) and we established a fund to support "Exotic Propulsion" which is named ....SSI is currently supporting the work of Prof. Woodward, but it has always been our intent to expand the base of researchers as resources permit.  ...

Mr. Hudson,

Thanks so much for that great post.  It would be interesting to know whether Prof. Freeman Dyson knows about Prof. Woodward's work (*) on the Mach Effect and/or about Dr. White's work on a mutable and degradable Quantum Vacuum, and if he does, what does Prof. Dyson think about those conjectures.  I have tried to find out in the literature and I don't see anything that Dyson has published in this regard.

I did see that Dyson wrote that he believed that NASA's big and visible projects, the Shuttle and the Space Station, will in the future "appear as quaint and misguided gargantuan ventures in the wrong direction, the von Hindenbergs and Titanics of the late 20th century" and that manned spaceflight will use new launch technologies that will make chemical rockets seem absurdly inefficient.

_______________
(*) I cannot assume necessary agreement, since for example, Prof. Freeman Dyson wrote in his paper "Pilgrims, Saints, and Spacemen" that he disagreed with G.K. O'Neill on the economic viability of some of O'Neill's concepts.  This although they were great partners, as proven by the fact that O'Neill asked Freeman Dyson to carry on the responsibilities of the Space Studies Institute.

I've not discussed our support of Prof. Woodward's work with Prof. Dyson, since it is such a tiny part of our activities.  So no endorsement is expressed or implied.  :)  I do know he knows about Dr. Sonny White's efforts.

I've known Freeman for probably close onto 40 years and I highly respect him, and generally find it politic never to disagree with him, as I'm likely to be on the wrong side of that argument.  I generally agree that some of Gerry's concepts and strategies were flawed.  (Bog knows that same can be said of many of my ideas...)  But both Freeman and SSI like innovation, and are willing to follow data wherever it leads, so I've felt comfortable in starting up the initiative on my own recognizance, as it were.  If we don't try and get to the bottom of this sort of thing, who else will? is the position I'm taking.

The main SSI effort has to remain focused on our G-Lab concept, which I believe to be crucial to the question of whether or not we can permanently live, work and reproduce off Earth.  But when dedicated and directed gifts are made to SSI for the EPI, I'll see that they get to the researchers doing real work.  In the meantime, I welcome the conversation that this forum has encouraged, and compliment all of the participants on their impressive efforts to understand a fascinating phenomena. 
Title: Re: EM Drive Developments - related to space flight applications - Thread 2
Post by: zen-in on 04/28/2015 04:11 am

SSI is currently supporting the work of Prof. Woodward, but it has always been our intent to expand the base of researchers as resources permit.  At the moment about the best we can do is to buy equipment and fund the occasional student intern, rather than pay for principal investigator labor, but that could be enough to help out more than one lab, JSC Eagleworks included.  So no need to set up another organization to help – we are here and willing to be involved if the need can be articulated.

Now back to your regularly scheduled programming...er, discussion...which has been very enjoyable to follow!

(*I think I made that word up.)

I am sorry I don´t know by name more people there, but I was surprised and happy to see Prof. Freeman Dyson on the board of trustees of an organization funding ME research and possibly if everything goes right, also EM research.

(Paul March says both are possibly two sides of the same coin, so it cracks me up ME and EM are mirrored acronyms!  ;D)

Yes, Prof. Freeman Dyson used to be the President of the Space Studies Institute, he was a member of the Board of Trustees as of 2013.   The Space Studies Institute was founded by Gerard K. O'Neill . O'Neill (diagnosed with terminal leukemia) directed his Space Studies Institute to continue his efforts "until people are living and working in space". After G. O'Neill's death, management of SSI was passed to his son Roger O'Neill and Prof. Freeman Dyson.

Dyson is now 91 years old, here is a younger picture of him  ;) , and also a picture of a Project Orion spaceship and of G. K. O'Neill's with MIT students building the first mass driver prototype (I remember him when he was a Visiting Professor during the 1976–77 academic year at MIT -great times  :) -):

(http://www.nss.org/settlement/nasa/spaceresvol2/images/electrofig16a.GIF)

(http://upload.wikimedia.org/wikipedia/commons/e/e4/ONEILLMASS.GIF)
[/quote]

I saw a demo of that mass driver during the MIT open house.  (1979 I think)   Very impressive!    The person demonstrating it - may have been Gerard K. O'Neill himself -   started charging the capacitor banks then decided to open them to show the audience.   For safety reasons he shorted the hot side with a big shepard's crook style grounding rod.   There was a very loud report and bright flash as an inch of the rod got vaporized!   That got everyone's attention.  The Bitter Lab did a lot of interesting work.   Too bad they lost their federal funding; but that's politics.
Title: Re: EM Drive Developments - related to space flight applications - Thread 2
Post by: asciencesimpleton on 04/28/2015 04:58 am
Hello all, as one of the commenters earlier in this thread suggested, I am one of the many Reddit users that has found amazing fascination in this thread. While I have no scientific opinions to add to this conversation, and while 85% of the conversations I've read through here have done well to send me into a swath of migraines and feelings of utter intellectual inferiority, I have one things to say. Thank you. Though I may not understand much of what is being tossed back and forth here, what I do understand, has done so much to increase my excitement and astonishment of the science that is happening. Please, keep this conversation going. I look forward to reading (what I can) and keeping up with what you are doing, hypothesizing, and proofing with all subjects related to this EM drive.

- a science simpleton
Title: Re: EM Drive Developments - related to space flight applications - Thread 2
Post by: ThinkerX on 04/28/2015 05:24 am
Quote
Hello all, as one of the commenters earlier in this thread suggested, I am one of the many Reddit users that has found amazing fascination in this thread. While I have no scientific opinions to add to this conversation, and while 85% of the conversations I've read through here have done well to send me into a swath of migraines and feelings of utter intellectual inferiority, I have one things to say. Thank you. Though I may not understand much of what is being tossed back and forth here, what I do understand, has done so much to increase my excitement and astonishment of the science that is happening. Please, keep this conversation going. I look forward to reading (what I can) and keeping up with what you are doing, hypothesizing, and proofing with all subjects related to this EM drive.

WELCOME ABOARD! :)

Don't be so quick to dismiss yourself here.  I have read both of these threads from the beginning.  Over that time, a rather large array of knowledge and skill sets has been called upon.  For example:  Not that long ago, an artist played a key role in determining the dimensions for one of Shawyer's devices - and precise dimensions are crucial here.  You may not know the science, but you may know something else of value.

And on occasion, even ideas tossed forth from the peanut gallery are found to have a bit of merit. 





Title: Re: EM Drive Developments - related to space flight applications - Thread 2
Post by: Peter Svancarek on 04/28/2015 08:48 am
And, sorry if I sound insistent, but the vertical scale variability + apparent inconsistency of stiffness (wrt known parameters) needs really to be addressed and settled one day or another, especially since we see contradictory data when the balance is used in reverse (180° turn of test article that should yield same deflection magnitudes to the left or to the right, assuming same thrusts magnitudes). A central aspect of the pendulum appears to be not properly characterised. Having to rely only on calibration pulses and proportionality is not satisfactory, given the unusual physics involved and the general scepticism, everything must fit. If one were told that a scale had 10cm long arms, but dynamics (oscillation periods) showed it was rather 1m long arms, the fact that this wont change the end result (equal weight on the plates...) is not satisfactory, there would still be a 1 order of magnitude unexplained aspect at the heart the experiment.

(http://upload.wikimedia.org/wikipedia/commons/2/24/Eo-scale_of_justice.gif)

Thanks for the regular feedbacks and open stance.

Lets this be warning how could be earthside test deceiving. https://www.youtube.com/watch?v=QJcbe8P5900 (https://www.youtube.com/watch?v=QJcbe8P5900) balance test is at the end of video. Of course none of this would function in zeroG.
Title: Re: EM Drive Developments - related to space flight applications - Thread 2
Post by: Rodal on 04/28/2015 12:20 pm
....

Lets this be warning how could be earthside test deceiving. https://www.youtube.com/watch?v=QJcbe8P5900 (https://www.youtube.com/watch?v=QJcbe8P5900) balance test is at the end of video. Of course none of this would function in zeroG.
All of the examples shown in the YouTube video attached above involve:

1) Stick-Slip friction
and
2) Mechanical Vibration

The motion has been understood for a century.  Its explanation is addressed in vibration courses (for example, in the old classic books by Prof. Den Hartog, predating WWII): it is due to the nonlinearity of the stick-slip friction relationship.  It is the same reason why screws (that are not spring-loaded or adhered with thread-locking adhesive) come loose under vibratory motion in moving vehicles, for example.

It cannot result in self-levitation of an object next to the surface of the Earth, under any gravitational force and it cannot result in unidirectional self-acceleration of an object in outer space either.  (*)

These examples do not show that one must perform EM Drive (or other sensitive) experiments in outer Space. 

Cavendish, for example, measured the force of gravity between small spherical masses in a rudimentary laboratory more than 200 years ago ( http://en.wikipedia.org/wiki/Cavendish_experiment ).  Thus Cavendish produced accurate values for the universal gravitational constant G, using a torsional balance, without having to wait to perform expensive measurements in Space 200 years later.

(http://milesmathis.com/cav2.jpg)

If the purpose of the examples is instead to warn about stick-slip friction and vibration in the EM Drive experiments performed at NASA Eagleworks, then it would be good to follow up by pointing out what evidence is there of mechanical vibration and stick-slip frictional motion in the NASA Eagleworks experiments.

___________________

(*) And of course, the International Space Station and near-Earth satellites are also under near the same gravitational force as the gravitational force present at the surface of the Earth, the main difference is that the ISS and satellites are in free-falling orbits.  Performing an experiment inside the ISS is identical to performing an experiment inside a free falling elevator, near the surface of the Earth, the difference is the practical duration of the experiment.
Title: Re: EM Drive Developments - related to space flight applications - Thread 2
Post by: bprager on 04/28/2015 01:57 pm
I have a probably rather naive question:

As far as I understand, we are trying to get the  microwaves resonate within the copper device, similar to blowing a pipe by more or less accurately finding the resonance frequency with our emitters, which are generating scattered waves. Would we get more refined or different results, if we could MASER's instead?
Title: Re: EM Drive Developments - related to space flight applications - Thread 2
Post by: FutureStormtrooper on 04/28/2015 02:05 pm
(*) And of course, the International Space Station and near-Earth satellites are also under near the same gravitational force as the gravitational force present at the surface of the Earth, the main difference is that the ISS and satellites are in free-falling orbits.  Performing an experiment inside the ISS is identical to performing an experiment inside a free falling elevator, near the surface of the Earth, the difference is the practical duration of the experiment.

An alternative would be to perform the experiment on one of NASA's Reduced Gravity Program aircraft (aka the vomit comet, one of the best nicknames I've come across.) The difficult remains, however, that getting space aboard for an experiment would likely require more confirmation from their existing experiments.
Title: Re: EM Drive Developments - related to space flight applications - Thread 2
Post by: Rodal on 04/28/2015 02:20 pm
(*) And of course, the International Space Station and near-Earth satellites are also under near the same gravitational force as the gravitational force present at the surface of the Earth, the main difference is that the ISS and satellites are in free-falling orbits.  Performing an experiment inside the ISS is identical to performing an experiment inside a free falling elevator, near the surface of the Earth, the difference is the practical duration of the experiment.

An alternative would be to perform the experiment on one of NASA's Reduced Gravity Program aircraft (aka the vomit comet, one of the best nicknames I've come across.) The difficult remains, however, that getting space aboard for an experiment would likely require more confirmation from their existing experiments.

Cavendish,measured the force of gravity between small spherical masses in a rudimentary laboratory more than 200 years ago ( http://en.wikipedia.org/wiki/Cavendish_experiment ).  Thus Cavendish produced accurate values for the universal gravitational constant G, using a torsional balance, without having to wait to perform expensive measurements in Space, or in aircraft performing a parabolic maneuver, 200 years later.

NASA, MIT, CalTech, Stanford, Princeton, etc. have been routinely testing space propulsion drives (including ion thrusters), very accurately, here on Earth-laboratories for decades.
Title: Re: EM Drive Developments - related to space flight applications - Thread 2
Post by: Chris Bergin on 04/28/2015 02:43 pm
Hi clever people :)

 So, we'll have our article on the EM Drive - expertly and patiently built by Dr. Rodal and subedited by my assistant editor Chris Gebhardt - published on Wednesday. We'll link in this thread and also have a standalone thread (the latter allowing people to get up to speed - an entry level thread as such, before braving this 104 page, 470,000 viewed thread).

 I've been following this thread the best I can and I have to say that while I'm proud I know my way around a RS-25 engine, this thread has proven I probably didn't pay as much attention in class as I should have! The encouraging thing is, while NASA has become somewhat "social media fluffy" and has lost a lot some of its focus of late, there's obviously less publicized areas, such as Eagleworks, that are the true essence of Bruce Willis' line in Armageddon, where he claims "You're NASA, you're geniuses. I bet you've got a room of people just thinking *stuff* up, and people backing them up!" :)
Title: Re: EM Drive Developments - related to space flight applications - Thread 2
Post by: Left Field on 04/28/2015 03:03 pm
Hi clever people :)

 So, we'll have our article on the EM Drive - expertly and patiently built by Dr. Rodal and subedited by my assistant editor Chris Gebhardt - published on Wednesday.

==snip==

Great news! Well let me be the first on here to thank Dr Rodal for writing the article and also for his astounding contributions to this thread, which have kept everything scientifically grounded.
Title: Re: EM Drive Developments - related to space flight applications - Thread 2
Post by: Star-Drive on 04/28/2015 04:44 pm
From this post (http://forum.nasaspaceflight.com/index.php?topic=36313.msg1364270#msg1364270), how do we interpret the "RF Dissipated Power" in the central caption of this slide ?

(http://forum.nasaspaceflight.com/index.php?action=dlattach;topic=36313.0;attach=826537;image)
bigger (http://forum.nasaspaceflight.com/index.php?action=dlattach;topic=36313.0;attach=826536;image)

Is it the (DC) power input to RF amplifier ?  Or the difference of RF power input to frustum minus reflected power back to amplifier ? What is measured exactly ? What is horizontal scale, is the != 0 part 16.5s long like the power-on period of the thrust chart ? Why this particular profile with initial spike and 2 plateaus ? Why don't we see a corresponding 3x magnitude "step" on the thrust chart at half the excitation time ?

I'm not sure how to interpret the next one either :
(http://forum.nasaspaceflight.com/index.php?action=dlattach;topic=36313.0;attach=826539;image)
bigger (http://forum.nasaspaceflight.com/index.php?action=dlattach;topic=36313.0;attach=826538;image)

Is it to say that when the spectrum is broader the initial slope on thrust chart is steeper ? On most other charts ~10s of power-on is enough to reach a plateau (kind of), why isn't it the case here ? Seems to me this is magnitude of plateau vs spectrum width that would be relevant, not slope, too much noise on top of those transients. For instance, the decay after power-off looks like having a significantly longer time constant with the richer spectrum, is it relevant ?


"How do we interpret the "RF Dissipated Power" in the central caption of this slide ? Is it the (DC) power input to RF amplifier ?  Or the difference of RF power input to frustum minus reflected power back to amplifier ? "

Simple, it's the forward RF power being dissipated in the RF load AKA resonant cavity, minus the reflected power being dissipated back in the RF amplifier's 50 ohm isolator load.  We measure it with two Mini-Circuit -50dBm to +20dBm RF power modules attached to the forward and reflected ports on a -40dB down bidirectional coupler.   

"What is horizontal scale, is the != 0 part 16.5s long like the power-on period of the thrust chart ?

The LabView RF power meter graphics has a scrolling data logging output with time on the horizontal x-axis and RF power level on the y-axis.  Since a full frame of the time axis takes about 3.5 minutes to scroll from right to left, there are no time units appended. 

"Why this particular profile with initial spike and 2 plateaus ?"

Because I was manually controlling the VCO frequency to maintain the minimum VSWR, which at times is a bit jerky, being that my control servo loops are not as good as they used to be.  :)

"Why don't we see a corresponding 3x magnitude "step" on the thrust chart at half the excitation time ?"

Overcoming the Inertia of the torque pendulum load which is close to 10 kg.

"I'm not sure how to interpret the next (two slides) either :
Is it to say that when the spectrum is broader the initial slope on thrust chart is steeper ?

Yes.  It appears that more thrust is generated with wider (FM) and larger (AM) signal sideband modulations.  We have not quantified this observation yet, so I can't given you numeric relationship between thrust and modulation levels yet.

Best, Paul M.
Title: Re: EM Drive Developments - related to space flight applications - Thread 2
Post by: Dmytry on 04/28/2015 05:09 pm
I have a few questions.

Why is all research so focussed on the powered thrust, rather than on the increasing opposite-direction acceleration which continues even after you turn the power off, which is visible in the graphs? The latter is far more amazing: it means that after you charge your drive for a few seconds, it just takes off, with an increasing acceleration, in the opposite direction.

The other question is: is there any work towards creating a battery powered, timer operated test article which can actually be enclosed in a sealed box with permalloy shielding? This would not only eliminate the interactions between the test article and the cavity, but would also permit running your RF amplifier at the ambient pressure.
Title: Re: EM Drive Developments - related to space flight applications - Thread 2
Post by: Rodal on 04/28/2015 05:38 pm
I have a probably rather naive question:

As far as I understand, we are trying to get the  microwaves resonate within the copper device, similar to blowing a pipe by more or less accurately finding the resonance frequency with our emitters, which are generating scattered waves. Would we get more refined or different results, if we could MASER's instead?
Using MASER technology is an interesting idea.

There are several issues here for you to think about.  One issue is that the EM Drive experimental results are not yet consistent, repeatable enough and that there is no consensus on an operating theory for the origin of the measurements.

If you adopt Dr. White's mutable, degradable QV conjecture then the results of his latest computational analysis,  is that "more thrust is generated with wider (FM) and larger (AM) signal sideband modulations".  That seems to be the opposite of what one would get from a MASER, which is coherent radiation.

@Mulletron, @NotSoSureOfIt, and others, is there an operating theory for the EM Drive that would support the use of a MASER ?

Also, the MASER would necessitate the introduction of a gas (unless you are thinking of a solid state maser), and the complexity of a MASER would be much higher, hence needing substantially more money and time than what NASA Eagleworks has had up to now (please consider that the truncated EM Drives where made at Paul March's living room by himself, as I understand).  Therefore substantial justification would be needed to use a maser.

(http://upload.wikimedia.org/wikipedia/commons/thumb/c/c9/Hydrogen_maser.gif/260px-Hydrogen_maser.gif)
Title: Re: EM Drive Developments - related to space flight applications - Thread 2
Post by: Mulletron on 04/28/2015 05:40 pm

Please remember that our copper frustum has a baked on silicone PCB anti-oxidation ~0.001" thick coating on its interior surfaces to keep the copper surfaces from oxidizing and thus lowering its Q-factor over time.

This fact didn't get much discussion here. Considering that Eagleworks reported no thrust from an unloaded cavity and the reports that Shawyer abandoned using dielectrics and the Chinese made no mention of using dielectrics, and reported thrust; this deserves discussion. Is there any evidence which support any other team using such coatings?

Still thinking about our nonlinear surface discussion from the other day.
http://forum.nasaspaceflight.com/index.php?topic=36313.1800
http://forum.nasaspaceflight.com/index.php?topic=36313.msg1361900#msg1361900

Edit: What I'm getting at here is that Shawyer/Chinese report thrust from unloaded cavities (no dielectric inserts) yet Eagleworks reports no thrust from an unloaded cavity. Eagleworks has the silicone PCB anti-oxidation ~0.001" thick coating, and I bet dollars to donuts the others don't.

All three cavities have air present as the dielectric, which has been shown by experiment to exhibit magnetoelectric nonreciprocity under crossed electric and magnetic fields*, **, but as was pointed out by Dr. Rodal, that isn't enough, nonlinearity is also required. The coating might be the culprit.

* http://forum.nasaspaceflight.com/index.php?topic=36313.msg1330846#msg1330846
** And researchers predict momentum transfer from the Quantum Vacuum under that circumstance.


Edit for more info:
Turns out that the first two oxides of copper have a k of 18.1.
http://www.clippercontrols.com/pages/Dielectric-Constant-Values.html
Title: Re: EM Drive Developments - related to space flight applications - Thread 2
Post by: Dmytry on 04/28/2015 05:58 pm
What kind of earth-side experiment can be done to very tangibly and visibly demonstrate that the effect exists?
One where the test apparatus is timer operated, battery powered and entirely enclosed in a permalloy-shielded, hermetically sealed container, and where there are no on-going (and changing) zero drifts of greater magnitude than the claimed forces, would demonstrate it far more tangibly than what has been done so far.

It would also allow to run the RF amplifier at an ambient pressure, and it would allow to use a far simpler, cheaper experimental set up. The exact lab set up of Henry Cavendish worked without any vacuum, down to 0.2 micronewtons or less, AFAIK.

You see, this is what bothers me the most. The operator turning the knob can be easily replaced with an Arduino and a handful of lines of code. The in-vacuum testing is very expensive in comparison, and only eliminates one conventional-physics force out of a dozen forces acting between the test article and the chamber walls.

At the very minimum, you need a test set up such that conventionally-interacting devices (such as a computer fan, a servo moving a weight around, a wire loop under current, a sharp point at high voltage) would not be able to generate enormously larger forces than your test article.

The set up must be such that you can't whip up in an afternoon a "drive" that would pass the test by doing something perfectly normal with the laws of physics as we know them. The only way to come close to accomplishing this is to completely enclose the test article.
Title: Re: EM Drive Developments - related to space flight applications - Thread 2
Post by: Rodal on 04/28/2015 06:09 pm
...Why is all research so focussed on the powered thrust, rather than on the increasing opposite-direction acceleration ...
We need to clarify this point.  My understanding is that NASA Eagleworks calls "thrust" the force occurring in the same direction as the acceleration.

Now, what does "thrust force" mean in this context? For NASA Eagleworks it means a vector proportional to the measured displacement, that can be converted to a rotational angle of the torsional balance, and hence to a torque (knowing the torsional stiffness of the torsional balance).  Or simply, force=displacement*stiffness . Like in a spring, for example. Now, this is a tautology, as the "thrust force" is entirely based on the displacement, and hence it is not surprising that the displacement should be in the same direction as the acceleration (the second order derivative of the displacement with respect to time). (And yes, forces are never directly measured, they are an intuitive and theoretical concept, what we do is measure displacements and obtain forces if we know the stiffness, or we measure accelerations and we obtain forces using F=m*a)

I am aware that Roger Shawyer has written that the thrust force and the displacement in his measurements occur in different directions, and that the EM Drive must be already in motion (or something to that effect), but frankly what he wrote in this regard is not comprehensible to me (and from what I recall was not comprehensible to @frobnicat either).  Apparently it was not comprehensible to other people either, as his latest report http://www.emdrive.com/EmDriveForceMeasurement.pdf addresses this (I find his latest report just as difficult to reconcile).

In the chart below, he even refers to a "force direction" which can be in the same direction as what he calls "thust" or what he calls "reaction" occurring in the opposite direction to what he calls "thrust".

For his Demonstrator engine, he claims (in the chart below) that he measured "forces" in opposite directions for this same device.  (It this was due to placing a dielectric at opposite ends or due to running the device in different mode shapes at different frequencies, I don't know.)

Title: Re: EM Drive Developments - related to space flight applications - Thread 2
Post by: frobnicat on 04/28/2015 06:58 pm
From this post (http://forum.nasaspaceflight.com/index.php?topic=36313.msg1364270#msg1364270), how do we interpret the "RF Dissipated Power" in the central caption of this slide ?

(http://forum.nasaspaceflight.com/index.php?action=dlattach;topic=36313.0;attach=826537;image)
bigger (http://forum.nasaspaceflight.com/index.php?action=dlattach;topic=36313.0;attach=826536;image)
...

"How do we interpret the "RF Dissipated Power" in the central caption of this slide ? Is it the (DC) power input to RF amplifier ?  Or the difference of RF power input to frustum minus reflected power back to amplifier ? "

Simple, it's the forward RF power being dissipated in the RF load AKA resonant cavity, minus the reflected power being dissipated back in the RF amplifier's 50 ohm isolator load.  We measure it with two Mini-Circuit -50dBm to +20dBm RF power modules attached to the forward and reflected ports on a -40dB down bidirectional coupler.   

Very clear.

Quote
"What is horizontal scale, is the != 0 part 16.5s long like the power-on period of the thrust chart ?

The LabView RF power meter graphics has a scrolling data logging output with time on the horizontal x-axis and RF power level on the y-axis.  Since a full frame of the time axis takes about 3.5 minutes to scroll from right to left, there are no time units appended. 

We are all very eager to have fresh data and it may be contradicting the need to detail all parameters and legend every possible axis, given the time it takes for crystal clear communication. I'm still struggling to fit the time axis of the power meter chart with the thrust chart. If its 210s from left unit 146030 to right unit 146229 (actually 1s per unit ?)  it would make the whole power-on run a minute long or so, from the power meter chart.

Can you confirm the time mapping is correct as shown overlaid in red in attached picture below ?

Quote
"Why this particular profile with initial spike and 2 plateaus ?"

Because I was manually controlling the VCO frequency to maintain the minimum VSWR, which at times is a bit jerky, being that my control servo loops are not as good as they used to be.  :)

"Why don't we see a corresponding 3x magnitude "step" on the thrust chart at half the excitation time ?"

Overcoming the Inertia of the torque pendulum load which is close to 10 kg.

Thing is, the pendulum arm with its 10kg is usually not that shy of steep responses (on µm scale) from steep excitations, typically below 3s time constant. So this is strange that we don't see a corresponding step in thrust to the step from plateau to plateau of power (each during almost 8s). Why change of excitation power would deserve more "inertial smoothing" than on/off power steps ?

Also this thrust chart happens to be very clean, with low noise. There is a slight inflection at ~2/3 of excitation time but it's quite clear there's nothing as dramatic as the usual responses to excitation steps. Such a weak response when going from 1W to 3W compared to when going to or from 0W would indicate strong non linearities... 

Or that what counts is not the net dissipated power in frustum (forward minus bounced back) but only forward power (regardless of what is bounced back). How could that be ?
Title: Re: EM Drive Developments - related to space flight applications - Thread 2
Post by: Rodal on 04/28/2015 07:10 pm
Hi clever people :)

 So, we'll have our article on the EM Drive - expertly and patiently built by Dr. Rodal and subedited by my assistant editor Chris Gebhardt - published on Wednesday. We'll link in this thread and also have a standalone thread (the latter allowing people to get up to speed - an entry level thread as such, before braving this 104 page, 470,000 viewed thread).

 I've been following this thread the best I can and I have to say that while I'm proud I know my way around a RS-25 engine, this thread has proven I probably didn't pay as much attention in class as I should have! The encouraging thing is, while NASA has become somewhat "social media fluffy" and has lost a lot some of its focus of late, there's obviously less publicized areas, such as Eagleworks, that are the true essence of Bruce Willis' line in Armageddon, where he claims "You're NASA, you're geniuses. I bet you've got a room of people just thinking *stuff* up, and people backing them up!" :)

It was soon apparent, when we started to write this article, that the amount of work done by different people in this thread was so immense that it was not possible to discuss it in the article and to do justice to their work.   So, unfortunately, there was not enough space to deal for example with the voluminous literature uncovered by @Mulletron, or to properly discuss the force equation, dependent on mode shape, derived by @NotSoSureOfIt, or to properly discuss the numerical simulations using MEEP by @aero, or the exact solutions I obtained.  Nor was there space in the article to discuss the experimental program independently being pursued by @Mulletron, or the excellent engineering discussions by @frobnicat and @zen-in of the EM Drive measurements as an experimental artifact, or my paper discussing thermal buckling, or countless other important contributions.  In order to attempt to make the paper comprehensible to a wider audience we needed to focus on the work at NASA Eagleworks with only a succinct reference to the work in the UK and China.  The emphasis was to try to write an objective review of NASA Eagleworks research. 

I want to thank everybody that helped in the article, to Chris Bergin for a huge amount of patient work in editing the paper to make it comprehensible to a wider audience, to @Mulletron for starting the ball rolling with an excellent draft, to @sghill for always being helpful and for motivating us to continue, until the end, to tell the story, and to @Rotosequence and @D_Dom for their always cheerful and helpful editing.  To them goes any  credit, and any criticism for the paper falls on my shoulders.
Title: Re: EM Drive Developments - related to space flight applications - Thread 2
Post by: Dmytry on 04/28/2015 07:39 pm
...Why is all research so focussed on the powered thrust, rather than on the increasing opposite-direction acceleration ...
We need to clarify this point.  My understanding is that NASA Eagleworks calls "thrust" the force occurring in the same direction as the acceleration.

Now, what does "thrust force" mean in this context? For NASA Eagleworks it means the measured displacement, that can be converted to a rotational angle of the torsional balance, and hence to a torque (knowing the torsional stiffness of the torsional balance).  Or simply, force=displacement*stiffness . Like in a spring, for example. Now, this is a tautology, as the "thrust force" is entirely based on the displacement, and hence it is not surprising that the displacement should be in the same direction as the acceleration (the second order derivative of the displacement with respect to time). (And yes, forces are never directly measured, they are an intuitive and theoretical concept, what we do is measure displacements and obtain forces if we know the stiffness, or we measure accelerations and we obtain forces using F=m*a)

I am aware that Roger Shawyer has written that the thrust force and the displacement in his measurements occur in different directions, and that the EM Drive must be already in motion (or something to that effect), but frankly what he wrote in this regard is not comprehensible to me (and from what I recall was not comprehensible to @frobnicat either).  Apparently it was not comprehensible to other people either, as his latest report http://www.emdrive.com/EmDriveForceMeasurement.pdf addresses this (I find his latest report just as difficult to reconcile).

In the chart below, he even refers to a "force direction" which can be in the same direction as what he calls "thust" or what he calls "reaction" occurring in the opposite direction to what he calls "thrust".

For his Demonstrator engine, he claims (in the chart below) that he measured "forces" in opposite directions for this same device.  (It this was due to placing a dielectric at opposite ends or due to running the device in different mode shapes at different frequencies, I don't know.)
What I was referring to is how in most of your graphs, after you turn your test article off, the graph shows a rising force acting in the opposite direction to the force you got when the device was turned on.

edit: to clarify, this:
(http://i.imgur.com/e365fIc.jpg)
Title: Re: EM Drive Developments - related to space flight applications - Thread 2
Post by: zen-in on 04/28/2015 07:55 pm

Thing is, the pendulum arm with its 10kg is usually not that shy of steep responses (on µm scale) from steep excitations, typically below 3s time constant. So this is strange that we don't see a corresponding step in thrust to the step from plateau to plateau of power (each during almost 8s). Why change of excitation power would deserve more "inertial smoothing" than on/off power steps ?

Also this thrust chart happens to be very clean, with low noise. There is a slight inflection at ~2/3 of excitation time but it's quite clear there's nothing as dramatic as the usual responses to excitation steps. Such a weak response when going from 1W to 3W compared to when going to or from 0W would indicate strong non linearities... 

Or that what counts is not the net dissipated power in frustum (forward minus bounced back) but only forward power (regardless of what is bounced back). How could that be ?

If the RF on time is the same as the duration of the thrust waveform there should be a second step response in the thrust waveform.   No one expects the response to be linear but it should be consistent.
Title: Re: EM Drive Developments - related to space flight applications - Thread 2
Post by: cskinfly on 04/28/2015 09:13 pm
Where might one find the plans for the em-drive and the experiment to replicate the experiment?
Title: Re: EM Drive Developments - related to space flight applications - Thread 2
Post by: Rodal on 04/28/2015 09:41 pm

What I was referring to is how in most of your graphs, after you turn your test article off, the graph shows a rising force acting in the opposite direction to the force you got when the device was turned on.

edit: to clarify, this:
(http://i.imgur.com/e365fIc.jpg)
You are referring to a drifting baseline for the measured displacement.

It is addressed (succintly) in the "Anomalous..." report text in reference to thermal effects, and in multiple posts of @Star-Drive, @frobnicat and @zen-in in the EM Drive threads 1 and 2.
Title: Re: EM Drive Developments - related to space flight applications - Thread 2
Post by: aceshigh on 04/28/2015 10:18 pm
and well, IGN has posted a video and an article, linking to this forum... and claiming NASA is one step closer to a warp drive and that Paul March has found that some of the laser beams fired inside the EM Drive Chamber were travelling faster than light.
http://www.ign.com/articles/2015/04/28/nasa-may-have-invented-a-warp-drive

 ::)

so far, the 1700 comments in a few hours are divided between the ones who say FTL is impossible, the ones who think a Warp Drive will be available in a few months, the ones trolling making fun and a few guys saying humanity should not venture into the stars in search for aliens, because all we need is god (really)...


what I wonder is how this big public awareness may influence NASA... pressure to keep funding EM/Warp, or pressure from conservatives (scientifically) that dismiss propellantless drives and FTL as nonsense without even looking at the data, for NASA to cut the funds to Eagleworks?
Title: Re: EM Drive Developments - related to space flight applications - Thread 2
Post by: RotoSequence on 04/28/2015 10:24 pm
This looks pertinent:

When mediated by superconductivity, light pushes matter million times more (http://www.aka.fi/en-GB/A/Academy-of-Finland/Media-services/Releases/When-mediated-by-superconductivity-light-pushes-matter-million-times-more/)

Quote
When a mirror reflects light, it experiences a slight push. This radiation pressure can be increased considerably with the help of a small superconducting island. This was revealed by the joint research done in the Aalto University and the Universities of Jyväskylä and Oulu. The finding paves a way for the studies of mechanical oscillations at the level of a single photon, the quantum of light. The results of the research were published in Nature Communications in April.

http://www.nature.com/ncomms/2015/150427/ncomms7981/full/ncomms7981.html

Quote from: Abstract
Coupling electromagnetic waves in a cavity and mechanical vibrations via the radiation pressure of photons is a promising platform for investigations of quantum–mechanical properties of motion. A drawback is that the effect of one photon tends to be tiny, and hence one of the pressing challenges is to substantially increase the interaction strength. A novel scenario is to introduce into the setup a quantum two-level system (qubit), which, besides strengthening the coupling, allows for rich physics via strongly enhanced nonlinearities. Here we present a design of cavity optomechanics in the microwave frequency regime involving a Josephson junction qubit. We demonstrate boosting of the radiation–pressure interaction by six orders of magnitude, allowing to approach the strong coupling regime. We observe nonlinear phenomena at single-photon energies, such as an enhanced damping attributed to the qubit. This work opens up nonlinear cavity optomechanics as a plausible tool for the study of quantum properties of motion.

In summary, non-linear behaviors of the quantum system in a microwave transmitter driven, superconducting resonating cavity is increasing the mechanical effect of radiation pressure by four to six orders of magnitude.
Title: Re: EM Drive Developments - related to space flight applications - Thread 2
Post by: frobnicat on 04/28/2015 10:32 pm
...
What I was referring to is how in most of your graphs, after you turn your test article off, the graph shows a rising force acting in the opposite direction to the force you got when the device was turned on.

edit: to clarify, this:
(http://i.imgur.com/e365fIc.jpg)

Thanks for clarification.

The near systematic downward slope after power-on runs on the present Eagleworks setup reported charts have been explained by Paul March in "Thread 1" as being due to heating of flexure bearings by the rf amplifier radiator (situated on the pendulum arm assembly, just behind those flexure bearings). From this post (http://forum.nasaspaceflight.com/index.php?topic=29276.msg1254143#msg1254143) :
Quote from: Star-Drive
We found that this slope change after the test article and RF amplifer were turned on for 10-to-20 seconds was apprently due to IR radiation from the amplifier's heatsink that is mounted on the back side of the torque penlulum on an 8" square platform was affecting the top C-flex bearing more than the lower one.  We tried aluminum shielding the top bearing assembly from the heatsink IR source and managed to reverse the metioned thermal slope in the thrust plots, but after shielding the bottom one we could reduce it but still coundn't completely get rid of this thremal drift artifact.  Currently we are just living with it.

Then later in present thread, after reporting (as seen on later pictures of setup) some mitigation of the problem (IR insulation) Paul March made mention (don't have the link at hand) of heating of same flexure bearings by return currents from pendulum arm (rotating assembly) to ground (fixed assembly).

In both cases, what would appear as increasing thrust "in the opposite direction" would be in fact drift of rest position as the flexure bearings heat. The fact that the effect lasts (and increases) for long after power off would be due to thermal inertia and lag of thermal conduction between hot source and flexure bearings. That makes sense for IR heating from RF amp radiators, IMO less with return currents as they would directly bulk heat the supporting spring steel...

Later on, my understanding is that team at Eagleworks was trying to explain those slopes still as thermal, but rather from expansion of test articles parts, and not exclusively as flexure bearings rest position drifts (not clear if this is still considered by them an important component or not). Also it is not clear if they are only considering that as "recoil effects" ( apparent thrust from  m d²position/dt² of some part during thermal transients). Or if they are thinking hard about or heard my petition that, since the horizontal pendulum is not strictly horizontal after all (or rather, that the rotation axis is not strictly vertical), and that the centre of mass of rotating assembly is not on said rotation axis (contrary to a Cavendish balance by construction), then there is a "direct drive" (not second order) between position of centre of mass and rest equilibrium of balance, ie. vertical_scale_on_charts(t)=arm_displacement(t)= lambda*position_centre_of_mass(t) + mu*thrust(t)
with lambda still to be determined (because inconsistencies in reported parameters/results makes it a 1 order of magnitude unknown).

The absence of second order derivative wrt time of CoMs position influence makes possible for a slowly moving CoM to induce a sustained (and even increasing) vertical measure on charts. I have not heard back of my remark that, given the inclination of axis toward the rear part of the experiment (as seen from front of vacuum chamber) and the position of rotating assembly's CoM behind the axis, the orientation of said influence is contrary to what appears on the published slides when considering only recoil effects : a displacement to the left of some part of the test article (relative to fixation on pendulum arm) would induce a corresponding proportional shift in position to the left of pendulum arms front part (where displacement is measured). This is of course only valid within the given information (tilted toward back + rotating assembly CoM behind axis) collected so far.

More problematic than those long lasting slow drifts in rest position after power on/off, is the fact that there seem to exist a continuum of situations in between slowly evolving charts (in response to excitation steps) and steep responses that serve as proof of real thrusts on the argument of their steepness (sorry for the poor wording). For instance, the rise and decay of the chart below (obtained in vacuum) show a very different time constant to reach and leave the "thrust" plateau than with the electrostatic calibration pulses with their clean square force(t) profile. The smoothed rise could be explained by change of tuning (effective received power) of the cavity, needing a "warm-up". But the smoothed decay is really a showdown. Off is off, there is no "warm-down" time to speak of for EM radiation to disappear (on the order of µs, given size and Q factor), and no electromagnetic/quantum theory could explain such a delay unless the vacuum is as heavy and viscous as oil or water. This lag in the decay, most visible in this particular chart, was already noticed by other contributors (sorry, can't remember).

(http://forum.nasaspaceflight.com/index.php?action=dlattach;topic=36313.0;attach=636345;image)
bigger (http://forum.nasaspaceflight.com/index.php?action=dlattach;topic=36313.0;attach=634625;image)
from this post (http://forum.nasaspaceflight.com/index.php?topic=36313.msg1326997#msg1326997)
Title: Re: EM Drive Developments - related to space flight applications - Thread 2
Post by: Dmytry on 04/28/2015 10:39 pm

What I was referring to is how in most of your graphs, after you turn your test article off, the graph shows a rising force acting in the opposite direction to the force you got when the device was turned on.

edit: to clarify, this:
(http://i.imgur.com/e365fIc.jpg)
You are referring to a drifting baseline for the measured displacement.

It is addressed (succintly) in the "Anomalous..." report text in reference to thermal effects, and in multiple posts of @Star-Drive, @frobnicat and @zen-in in the EM Drive threads 1 and 2.
Well, I have to admit I was being rather cheeky. To me, this "drift" (which did still exist in the vacuum) is a definite indication that there are experimental errors that are larger than the measured force.

This is what typical real world null results look like: they do not look like a literal zero, they look like a force which, no matter what you do, is of the same order of magnitude as other experimental errors. Sawyer and the Chinese do worse experiments, they have larger forces; you do better experiments and you have smaller forces; if you generate enough buzz and manage to annoy a modern experimental physicist remotely approaching Cavendish calibre to try to replicate it, he'll, no doubt, measure fractions of a micronewton.

As such the experiment lacks the most critical part: ability to distinguish between forces arising from the known physics, and some novel physics.

edit: for the sake of clarity, you is the general you not you personally, of course.
Title: Re: EM Drive Developments - related to space flight applications - Thread 2
Post by: Dmytry on 04/28/2015 11:07 pm

More problematic than those long lasting slow drifts in rest position after power on/off, is the fact that there seem to exist a continuum of situations in between slowly evolving charts (in response to excitation steps) and steep responses that serve as proof of real thrusts on the argument of their steepness (sorry for the poor wording). For instance, the rise and decay of the chart below (obtained in vacuum) show a very different time constant to reach and leave the "thrust" plateau than with the electrostatic calibration pulses with their clean square force(t) profile. The smoothed rise could be explained by change of tuning (effective received power) of the cavity, needing a "warm-up". But the smoothed decay is really a showdown. Off is off, there is no "warm-down" time to speak of for EM radiation to disappear (on the order of µs, given size and Q factor), and no electromagnetic/quantum theory could explain such a delay unless the vacuum is as heavy and viscous as oil or water. This lag in the decay, most visible in this particular chart, was already noticed by other contributors (sorry, can't remember).

(http://forum.nasaspaceflight.com/index.php?action=dlattach;topic=36313.0;attach=636345;image)
bigger (http://forum.nasaspaceflight.com/index.php?action=dlattach;topic=36313.0;attach=634625;image)
from this post (http://forum.nasaspaceflight.com/index.php?topic=36313.msg1326997#msg1326997)
Nice find, I think this one is utterly damning for any hope that the experimental set up is working correctly.

Keep in mind by the way that the time constant for a small thermal mass sinking into a large thermal mass can be similar to the time constant of the "drive", and would at one explain both the slow drift and the fast drift.

What bothers me the most about this research is that there is a perfectly simple and rather cheap way to screen out most of such effects. The test article needs to be powered off a battery, run from a timer, and be enclosed in a conductive box with permalloy shielding; hanging off a very very Cavendish style torsion pendulum (which is fairly insensitive to shifts in CoM as well, by the way).

The vacuum, stiff springs, damping assembly and electrostatic calibration, you can get by without any of those things. You could shine a laser off a little mirror to measure the acceleration directly, and even let the apparatus integrate the momentum for you.

 But a box that shields the apparatus from the test article, you can't do without! This is of paramount importance.

What ever manual frequency tweaking they do could be done with a few dozen lines of code in a microcontroller.
Title: Re: EM Drive Developments - related to space flight applications - Thread 2
Post by: Carl G on 04/29/2015 12:29 am

Well, I have to admit I was being rather cheeky.

I think you need to clarify who you are, if you're aiming to join this thread 104 pages in with your insight. Others have provided qualifications to their comments, it would be less "cheeky" if you did likewise.
Title: Re: EM Drive Developments - related to space flight applications - Thread 2
Post by: Rodal on 04/29/2015 12:48 am
...
What bothers me the most about this research is that there is a perfectly simple and rather cheap way to screen out most of such effects. The test article needs to be powered off a battery, run from a timer, and be enclosed in a conductive box with permalloy shielding; hanging off a very very Cavendish style torsion pendulum (which is fairly insensitive to shifts in CoM as well, by the way).

...

This has been discussed earlier in the thread (including shielding).  We also discussed an experiment hanging from a Cavendish style torsional pendulum, using a laser and a mirror, powering off a battery, done by another team to examine whether another type of self-enclosed electromagnetic drive was an artifact or a genuine propulsive effect.  That study was published years ago (2009) in the 45th AIAA/ASME/SAE/ASEE Joint Propulsion Conference proceedings:

http://enu.kz/repository/2009/AIAA-2009-5070.pdf

Perhaps @Star-Drive may be willing to comment further on these issues regarding the experiments at NASA Eagleworks.


Title: Re: EM Drive Developments - related to space flight applications - Thread 2
Post by: tchernik on 04/29/2015 02:19 am
I suggest a "required reading" link for new visitors.

Problem is: it would probably be 100+ pages long, though. More if we take the other thread into account.   ;D
Title: Re: EM Drive Developments - related to space flight applications - Thread 2
Post by: cuddihy on 04/29/2015 03:38 am
Maybe a "common objections" thread makes more sense as the article should provide background information, but objections and Eagleworks answers could be usefully pulled together to simplify reading through threads 1&2
Title: Re: EM Drive Developments - related to space flight applications - Thread 2
Post by: Dmytry on 04/29/2015 04:18 am
Well, one shouldn't even see a drift the size of the measured force, when it's not even sub-micronewton range.

If one is heating metal, one is probably heating it unevenly, and it bends as long as heat is applied (the hotter material expands more), with a fairly short time constant (because the temperatures will equalize quickly once the heat source is off). Then the heat gets slowly conducted to another structural element, and that structural element happens to bend in a way that influences the experiment in the other direction.

How much each structural element will bend will depend to how tight the screws are tightened, what stresses are already in the metal, and so on and so forth; it's essentially impossible to account for.
Title: Re: EM Drive Developments - related to space flight applications - Thread 2
Post by: Rodal on 04/29/2015 12:41 pm
Well, one shouldn't even see a drift the size of the measured force, when it's not even sub-micronewton range.

If one is heating metal, one is probably heating it unevenly, and it bends as long as heat is applied (the hotter material expands more), with a fairly short time constant (because the temperatures will equalize quickly once the heat source is off). Then the heat gets slowly conducted to another structural element, and that structural element happens to bend in a way that influences the experiment in the other direction.

How much each structural element will bend will depend to how tight the screws are tightened, what stresses are already in the metal, and so on and so forth; it's essentially impossible to account for.
Heat transfer, thermal expansion, thermal stress analysis for uncoupled and for coupled problems, in static and dynamic problems of complicated geometry and materials can be analyzed and modeled with Finite Element analysis (NASTRAN, ANSYS, ABAQUS, ADINA, COMSOL, etc.), just like rockets, spacecrafts and military and commercial aircraft have been analyzed for decades.

Here is an exact solution of the thermal buckling problem for the EM Drive's truncated cone base:

https://www.researchgate.net/publication/268804028_NASA%27S_MICROWAVE_PROPELLANT-LESS_THRUSTER_ANOMALOUS_RESULTS_CONSIDERATION_OF_A_THERMO-MECHANICAL_EFFECT

The issue for analyzing thermal structural effects (and other effects) for the EM Drive project at NASA Eagleworks is not a matter of present ability to be analyzed but is, instead, a matter of scarcity of project resources (money, time, and personnel) to analyze them. For example, NASA's EM Drive truncated cone was made by Paul March himself in Paul March's wife dining room. NO TAXPAYER's tax $$$ involved in its construction.

http://forum.nasaspaceflight.com/index.php?topic=36313.msg1327937#msg1327937

(http://forum.nasaspaceflight.com/index.php?action=dlattach;topic=36313.0;attach=635221;image)

Engineers and scientists interested in this project, (like @frobnicat, @notsosureofit, @aero, myself, etc.) have contributed their time as well to model different aspects of this experiment to try to distinguish any artifacts from real effects.   Earlier on I contributed results of a fully-coupled nonlinear analysis of the torque pendulum used in the experiment, as well as Autocorrelation, and Power Spectral Density analysis of the data.

Concerning the issue of shielding, @aero has contributed a MEEP analysis of evanescent waves leaking from the EM Drive, and possibly interacting with the stainless steeel vacuum chamber.  Subsequently, Paul March conducted an experiment with the EM Drive outside the stainless steel chamber that may have nullified that explanation.

@Mulletron has contributed his own resources and time to run his own EM Drive experiment from a hanging torsional pendulum.

Paul March has contributed detailed information about the experimental set-up, and if you are interested, you can also contribute any engineering, preferably quantitative, theoretical, numerical or experimental analysis you may be able to contribute.
Title: Re: EM Drive Developments - related to space flight applications - Thread 2
Post by: JasonAW3 on 04/29/2015 02:29 pm
The issue for analyzing thermal structural effects (and other effects) for the EM Drive project at NASA Eagleworks is not a matter of present ability to be analyzed but is, instead, a matter of scarcity of project resources (money, time, and personnel) to analyze them. For example, NASA's EM Drive truncated cone was made by Paul March himself in Paul March's wife dining room. NO TAXPAYER's tax $$$ involved in its construction.


This may seem dumb, but why not set up a Kickstarter for this?  Whole lot dumber items have gotten funded this way, so I think that helping to fund the development of the EM drive, ESPECIALLY if it proves to help further physics knowledge, would likely go over quite well.

     When you go to do this, make sure you keep it in layman's terms when you explain what the funding is for.  I think a LOT of people would be happy to have their names attached to this if it actually proves to be a usable drive of some sort!  If not, well, we just proved another way that the EM drive didn't work, doesn't mean that it can't work though.
Title: Re: EM Drive Developments - related to space flight applications - Thread 2
Post by: MazonDel on 04/29/2015 02:37 pm
Pretty much because of the rules for how the NASA Eagleworks team must work prevents it. They cannot accept donations of money, time, or resources. Any services provided by a company must be an equal value exchange. So, we cannot make a company, buy them a million dollar research device, and sell it to them for $1 or let them use it for anything less than competitive rates for a similar device.

Now you COULD try to set this up for a personal project rather than helping the Eagleworks team directly. But of course, you'd better have a decent proposal and such written up.
Title: Re: EM Drive Developments - related to space flight applications - Thread 2
Post by: Rodal on 04/29/2015 02:46 pm
....
This may seem dumb, but why not set up a Kickstarter for this?  ..

See this great post:  http://forum.nasaspaceflight.com/index.php?topic=36313.msg1364627#msg1364627

by the present President (as well as a Trustee) of the renowned Space Studies Institute (www.ssi.org) which was originally founded by G.K. O'Neill
Title: Re: EM Drive Developments - related to space flight applications - Thread 2
Post by: cfs on 04/29/2015 03:58 pm
....
This may seem dumb, but why not set up a Kickstarter for this?  ..

See this great post:  http://forum.nasaspaceflight.com/index.php?topic=36313.msg1364627#msg1364627

by the present President (as well as a Trustee) of the renowned Space Studies Institute (www.ssi.org) which was originally founded by G.K. O'Neill

So if I'm reading this correctly, what we need to do is set up a Kickstarter to donate money to ISS's exotic propulsion funds, which can then in turn be given to Eagleworks  ;D
Title: Re: EM Drive Developments - related to space flight applications - Thread 2
Post by: aceshigh on 04/29/2015 04:46 pm
....
This may seem dumb, but why not set up a Kickstarter for this?  ..

See this great post:  http://forum.nasaspaceflight.com/index.php?topic=36313.msg1364627#msg1364627

by the present President (as well as a Trustee) of the renowned Space Studies Institute (www.ssi.org) which was originally founded by G.K. O'Neill

So if I'm reading this correctly, what we need to do is set up a Kickstarter to donate money to ISS's exotic propulsion funds, which can then in turn be given to Eagleworks  ;D

I am not sure, but I guess that they would not be able to accept donations/kickstarters to a particular project. I guess they  can accept money but the distribution of the total sum is decided by them?


which would maybe somehow prevent the exact thing that makes NASA not accept donations/kickstarters.
Title: Re: EM Drive Developments - related to space flight applications - Thread 2
Post by: Jared on 04/29/2015 05:13 pm
Speaking of NASA, I wonder why they have just archived all previous articles about hypothetical advanced propulsion systems on their official site ...
Title: Re: EM Drive Developments - related to space flight applications - Thread 2
Post by: Star One on 04/29/2015 05:16 pm

Speaking of NASA, I wonder why they have just archived all previous articles about hypothetical advanced propulsion systems on their official site ...

I believe that's just the warp drive related articles.
Title: Re: EM Drive Developments - related to space flight applications - Thread 2
Post by: Rodal on 04/29/2015 05:29 pm
....
This may seem dumb, but why not set up a Kickstarter for this?  ..

See this great post:  http://forum.nasaspaceflight.com/index.php?topic=36313.msg1364627#msg1364627

by the present President (as well as a Trustee) of the renowned Space Studies Institute (www.ssi.org) which was originally founded by G.K. O'Neill

So if I'm reading this correctly, what we need to do is set up a Kickstarter to donate money to ISS's exotic propulsion funds, which can then in turn be given to Eagleworks  ;D

I am not sure, but I guess that they would not be able to accept donations/kickstarters to a particular project. I guess they  can accept money but the distribution of the total sum is decided by them?


which would maybe somehow prevent the exact thing that makes NASA not accept donations/kickstarters.






Quote from:  HMXHMX  http://forum.nasaspaceflight.com/index.php?topic=36313.msg1364627#msg1364627
we established a fund to support "Exotic Propulsion" which is named (appropriately but uninspiringly*) the "Exotic Propulsion Initiative.  It is possible to donate to the fund at our website.  SSI is a 501(c)3 non-profit and thus contributions are tax-deductiable. Targeted contributions go almost 100% to the named projects since we have very modest overhead costs (we don't pay salaries to our volunteer staff, for example).

unless HMXHMX cares to post further answers, it is advisable that anybody having further questions on SSI donations directly addresses them to the Contact listed at www.ssi.org

Space Studies Institute
16922 Airport Blvd.
#24
Mojave, CA 93501


PHONE (661) 750-2774

EMAIL [email protected]


and further inquires about destination of funds for targeted tax-deductible contributions, particularly if they have an interest in targeting a specific effort with their funds.
Title: Re: EM Drive Developments - related to space flight applications - Thread 2
Post by: Jared on 04/29/2015 05:52 pm

Speaking of NASA, I wonder why they have just archived all previous articles about hypothetical advanced propulsion systems on their official site ...

I believe that's just the warp drive related articles.

Thanks, I think you're right! I wonder why, though. I must say I am going to miss that nice little chart displaying the progress of warp science / technology from conjecture to application. :)
Title: Re: EM Drive Developments - related to space flight applications - Thread 2
Post by: Chris Bergin on 04/29/2015 06:01 pm
Here we go!

Over two months of work went into this article, involving the methodical and patient work of several people involved with the massive EM Drive thread - with the goal of creating an accurate and understandable (it's still high level science) article to reflect the interest in this development.

There have been several articles - all feeding off our forum thread on this - but this is something we feel will be more of a full news overview article.

Article:
http://www.nasaspaceflight.com/2015/04/evaluating-nasas-futuristic-em-drive/

Written by José Rodal, Ph.D, Jeremiah Mullikin and Noel Munson - subedited by Chris Gebhardt. Thanks also Don Domonoske and member RotoSequence for the work in the special area where we built this article. We would obviously like to thank Paul March at Eagleworks for his interesting interactions on the thread, and the numerous other high level folk who may not be too obvious behind their usernames (standard for a forum - if only you knew some of the people behind usernames on the SpaceX and such sections!)

Dedicated entry level thread for the article:
http://forum.nasaspaceflight.com/index.php?topic=37438.0
Title: Re: EM Drive Developments - related to space flight applications - Thread 2
Post by: JasonAW3 on 04/29/2015 07:55 pm
I would like to congratulate everyone involved in this project!

     As was suggested before, I think we have our "Chicago Pile" moment here and hopefully further development will be funded and can proceed expeditiously!

     On a side note, that 0.65Hz optical frequency shift strikes me as a VERY interesting result and should be examined MUCH more closely as the implications are interesting as well.
Title: Re: EM Drive Developments - related to space flight applications - Thread 2
Post by: MazonDel on 04/29/2015 08:03 pm
Congrats to Dr Rodal and the others!
Title: Re: EM Drive Developments - related to space flight applications - Thread 2
Post by: JasonAW3 on 04/29/2015 09:02 pm
Dr. Rodal,

    The 0.65Hz frequency shift of light with the new experiment is very interesting, but it was pointed out to me that air has a refraction index of 1.  I take it the refraction ndex of the air involved was taken into account for this interferometer test, or that it was conducted in a vacume?
Title: Re: EM Drive Developments - related to space flight applications - Thread 2
Post by: Rodal on 04/29/2015 09:10 pm
Dr. Rodal,

    The 0.65Hz frequency shift of light with the new experiment is very interesting, but it was pointed out to me that air has a refraction index of 1.  I take it the refraction ndex of the air involved was taken into account for this interferometer test, or that it was conducted in a vacume?

1) Air does not have a refractive index of 1.  The refractive index of air is approximately 1.000293.  As you know air is composed of several gases (mostly Nitrogen 78% mole fraction and Oxygen 21%, Argon almost 1%, etc.)

The issue involving air refraction was raised in this paper by Lee and Cleaver from Baylor University:

http://arxiv.org/ftp/arxiv/papers/1407/1407.7772.pdf

See the above paper for background.

 
2) The 0.65 Hz frequency you are referring to is not a "frequency shift of light".
Period of energizing/de-energizing: T=0.75s+0.75s=1.5s, hence frequency for energizing/de-energizing cycle should be f=1/T=0.66 Hz which is close to the frequecy of 0.65Hz appearing in the Power Spectrum

3) Quoting from the article:

Quote
Over 27,000 cycles of data (each 1.5 sec cycle energizing the system for 0.75 sec and de-energizing it for 0.75 sec) were averaged to obtain a power spectrum that revealed a signal frequency of 0.65 Hz with amplitude clearly above system noise.  Four additional tests were successfully conducted that demonstrated repeatability.

One possible explanation for the optical path length change is that it is due to refraction of the air.  The NASA team examined this possibility and concluded that it is not likely that the measured change is due to transient air heating because the experiment’s visibility threshold is forty times larger than the calculated effect from air considering atmospheric heating.

4) Concerning this experiment please refer to previous posts by @Star-Drive, myself, @zen-in and others

5) Concerning your last question, please refer to the ending sentence in the article:

Quote
Encouraged by these results, NASA Eagleworks plans to next conduct these interferometer tests in a vacuum.
Title: Re: EM Drive Developments - related to space flight applications - Thread 2
Post by: JasonAW3 on 04/29/2015 09:22 pm
Dr. Rodal,

    The 0.65Hz frequency shift of light with the new experiment is very interesting, but it was pointed out to me that air has a refraction index of 1.  I take it the refraction ndex of the air involved was taken into account for this interferometer test, or that it was conducted in a vacume?

1) Air does not have a refractive index of 1.  The refractive index of air is approximately 1.000293.  As you know air is composed of several gases:

Pure Gas Name   Symbol   Mole fraction
Nitrogen   N2   0.78084
Oxygen   O2   0.209476
Argon   Ar   0.00934
Carbon Dioxide   CO2   0.000314
Neon   Ne   0.00001818
Methane   CH4   0.000002
Helium   He   0.00000524
Krypton   Kr   0.00000114
Hydrogen   H2   0.0000005
Xenon   Xe   0.000000087

The issue involving air refraction is dealt was raised in this paper by Lee and Cleaver from Baylor University:

http://arxiv.org/ftp/arxiv/papers/1407/1407.7772.pdf

See the above paper for background.

 
2) The 0.65 Hz frequency you are referring to is not a "frequency shift of light".
Period of energizing/de-energizing: T=0.75s+0.75s=1.5s, f=1/T=0.66 Hz ~ 0.65Hz

3) Quoting from the article:

Quote
Over 27,000 cycles of data (each 1.5 sec cycle energizing the system for 0.75 sec and de-energizing it for 0.75 sec) were averaged to obtain a power spectrum that revealed a signal frequency of 0.65 Hz with amplitude clearly above system noise.  Four additional tests were successfully conducted that demonstrated repeatability.

One possible explanation for the optical path length change is that it is due to refraction of the air.  The NASA team examined this possibility and concluded that it is not likely that the measured change is due to transient air heating because the experiment’s visibility threshold is forty times larger than the calculated effect from air considering atmospheric heating.

4) Concerning this experiment please refer to previous posts by Star-Drive, myself, @zen-in and others

Dr. Rodal,

     No insult was intended or directed, as I said, this waas a point made by someone else to me, and I pretty much figured that this had been taken into account, otherwise it would not have been above backgroound "noise".

     Again, congratulations on what appears to be a most auspicious achievement!
Title: Re: EM Drive Developments - related to space flight applications - Thread 2
Post by: Rodal on 04/29/2015 09:27 pm
...
     No insult was intended or directed, as I said, this waas a point made by someone else to me, and I pretty much figured that this had been taken into account, otherwise it would not have been above backgroound "noise".

     Again, congratulations on what appears to be a most auspicious achievement!
No insult was taken  8)  .

If it appears that way, due to my cold, abrupt, way to answer the question, with references and quotations, it is due to the fact that I feel compelled to address this technically, carefully, and factually, because several other people are reading this as well, and it is important to avoid misinterpretations. 

Particularly in view of the shocking "news" that have been posted in fringe sites referring to this thread, talking about NASA finding by accident how to warp space and secretly working on a Star-Trek Enterprise.  :)

Actually, I thank you for your question, as it gives me the opportunity to clarify these matters.

Concerning whether a signal is or is not above "noise", that is not a trivial assessment, as randomness is ultimately undefinable.  What appears random to one observer may not appear random to another. An encrypted message is  a good example of this fact.

More, finer, experiments are needed.

Quote from: Ralph Waldo Emerson
All life is an experiment. The more experiments you make the better.

Title: Re: EM Drive Developments - related to space flight applications - Thread 2
Post by: LasJayhawk on 04/29/2015 09:55 pm
I am somewhat curious as to why AM modulation would increase the drives output. Is it possible that the modulation scheme is increasing the average power output, or is something else in play.

I'm wondering what it would do with pulsed RF at a high rep rate?
Title: Re: EM Drive Developments - related to space flight applications - Thread 2
Post by: Rodal on 04/29/2015 10:19 pm
I am somewhat curious as to why AM modulation would increase the drives output. Is it possible that the modulation scheme is increasing the average power output, or is something else in play.

I'm wondering what it would do with pulsed RF at a high rep rate?

That amplitude, frequency and phase modulation of the carrier wave results in greater thrust force is a prediction from Dr. White's computer code, and not yet an experimentally proven fact.  What we know in this regard is that the experimenters in the UK and China claim to have measured greater thrust force using a magnetron (for whatever reason) and that a magnetron performs amplitude, frequency and phase modulation of the carrier wave. We should exercise caution as neither the UK, nor the Chinese teams have been as forthcoming with data as NASA Eagleworks.  Those experiments (using a magnetron) remain to be replicated at NASA Eagleworks.

Concerning what a pulsed RF at a high rep rate would do, I expect that is something that Dr. White should be able to input in his code and give you an answer, but again whether such an answer from his computer code would be correct remains to be confirmed.

My understanding is that NASA Eagleworks is planning to replicate this June the experiments in the UK and China using a magnetron.
Title: Re: EM Drive Developments - related to space flight applications - Thread 2
Post by: JackFlash on 04/29/2015 10:29 pm
My apologies for the post I just removed; it was up only for a few seconds, and was an (accidental) reply to a very old post.

Thanks :)
Title: Re: EM Drive Developments - related to space flight applications - Thread 2
Post by: JasonAW3 on 04/29/2015 11:39 pm
...
     No insult was intended or directed, as I said, this waas a point made by someone else to me, and I pretty much figured that this had been taken into account, otherwise it would not have been above backgroound "noise".

     Again, congratulations on what appears to be a most auspicious achievement!
No insult was taken  8)  .

If it appears that way, due to my cold, abrupt, way to answer the question, with references and quotations, it is due to the fact that I feel compelled to address this technically, carefully, and factually, because several other people are reading this as well, and it is important to avoid misinterpretations. 

Particularly in view of the shocking "news" that have been posted in fringe sites referring to this thread, talking about NASA finding by accident how to warp space and secretly working on a Star-Trek Enterprise.  :)

Actually, I thank you for your question, as it gives me the opportunity to clarify these matters.

Concerning whether a signal is or is not above "noise", that is not a trivial assessment, as randomness is ultimately undefinable.  What appears random to one observer may not appear random to another. An encrypted message is  a good example of this fact.

More, finer, experiments are needed.

Quote from: Ralph Waldo Emerson
All life is an experiment. The more experiments you make the better.

Thanks,  I was hoping I hadn't stepped beyond the bounds of propriety.

Out of curiosity, what is the approximate ratio of watts to newtons, or millinewtons as the case may be, that you've been able to produce on the average?
Title: Re: EM Drive Developments - related to space flight applications - Thread 2
Post by: Rodal on 04/30/2015 01:55 am
...
     No insult was intended or directed, as I said, this waas a point made by someone else to me, and I pretty much figured that this had been taken into account, otherwise it would not have been above backgroound "noise".

     Again, congratulations on what appears to be a most auspicious achievement!
No insult was taken  8)  .

If it appears that way, due to my cold, abrupt, way to answer the question, with references and quotations, it is due to the fact that I feel compelled to address this technically, carefully, and factually, because several other people are reading this as well, and it is important to avoid misinterpretations. 

Particularly in view of the shocking "news" that have been posted in fringe sites referring to this thread, talking about NASA finding by accident how to warp space and secretly working on a Star-Trek Enterprise.  :)

Actually, I thank you for your question, as it gives me the opportunity to clarify these matters.

Concerning whether a signal is or is not above "noise", that is not a trivial assessment, as randomness is ultimately undefinable.  What appears random to one observer may not appear random to another. An encrypted message is  a good example of this fact.

More, finer, experiments are needed.

Quote from: Ralph Waldo Emerson
All life is an experiment. The more experiments you make the better.

Thanks,  I was hoping I hadn't stepped beyond the bounds of propriety.

Out of curiosity, what is the approximate ratio of watts to newtons, or millinewtons as the case may be, that you've been able to produce on the average?

It is not possible at this point to discuss "an average" because the experimental results are not consistent enough, and because there are not enough statistical experimental samples to perform meaningful statistics.

Therefore, let's point out instead the experimental range.

The complete range of experimental results for electromagnetic drives that appear to break the law of conservation of momentum is (I had to go back to look at page 1 of this thread for this  :) ):


MAXIMUM: 1 Newton/kiloWatt for Prof. Yang's experiments in China with an EM Drive microwave non-superconducting cavity and also for Cannae's Superconducting EM Drive microwave cavity

to
   
MINIMUM:  0.00001176  Newton/kiloWatt  for Fearn, Zachar, Woodward & Wanser's Woodward-Mach-Effect device experiment

As you can see, this is a large range: almost 10^5

I mention Fearn, Zachar, Woodward & Wanser's for completness and because Paul March wrote in one of his latest  posts that he still considers the Mach-Effect as a possible explanation for the EM Drive -based on the PCB silicone surface coating-, but I would like not to get this thread derailed: any arguments for ME go in this thread: http://forum.nasaspaceflight.com/index.php?topic=31037.0
while EM remains in this thread  :)
Title: Re: EM Drive Developments - related to space flight applications - Thread 2
Post by: MATTBLAK on 04/30/2015 03:28 am
I remain firmly skeptical.  And it's only a matter of time before the mainstream media starts calling it a 'warp drive'. As it is; mainstream media keeps harping on about VASMR every flipping time a manned Mars mission gets talked about :(
Title: Re: EM Drive Developments - related to space flight applications - Thread 2
Post by: UneducatedNitwit on 04/30/2015 03:29 am
Sad this site is indulging in quackery. Oh well.

The entire point of this, the other thread, the conversation, and the independent testing is the application of scientific rigor to this device in order to falsify the associated claims and develop the theory.

Is that quackery?
Title: Re: EM Drive Developments - related to space flight applications - Thread 2
Post by: HMXHMX on 04/30/2015 03:32 am
....
This may seem dumb, but why not set up a Kickstarter for this?  ..

See this great post:  http://forum.nasaspaceflight.com/index.php?topic=36313.msg1364627#msg1364627

by the present President (as well as a Trustee) of the renowned Space Studies Institute (www.ssi.org) which was originally founded by G.K. O'Neill

So if I'm reading this correctly, what we need to do is set up a Kickstarter to donate money to ISS's exotic propulsion funds, which can then in turn be given to Eagleworks  ;D

I am not sure, but I guess that they would not be able to accept donations/kickstarters to a particular project. I guess they  can accept money but the distribution of the total sum is decided by them?


which would maybe somehow prevent the exact thing that makes NASA not accept donations/kickstarters.






Quote from:  HMXHMX  http://forum.nasaspaceflight.com/index.php?topic=36313.msg1364627#msg1364627
we established a fund to support "Exotic Propulsion" which is named (appropriately but uninspiringly*) the "Exotic Propulsion Initiative.  It is possible to donate to the fund at our website.  SSI is a 501(c)3 non-profit and thus contributions are tax-deductiable. Targeted contributions go almost 100% to the named projects since we have very modest overhead costs (we don't pay salaries to our volunteer staff, for example).

unless HMXHMX cares to post further answers, it is advisable that anybody having further questions on SSI donations directly addresses them to the Contact listed at www.ssi.org

Space Studies Institute
16922 Airport Blvd.
#24
Mojave, CA 93501


PHONE (661) 750-2774

EMAIL [email protected]


and further inquires about destination of funds for targeted tax-deductible contributions, particularly if they have an interest in targeting a specific effort with their funds.

I can also be reached at "gary" (as above, replace admin with gary).  We can accept targeted donations for our Exotic Propulsion Initiative.  But any funds go into the fund for all "exotic propulsion" – not only one project. 
Title: Re: EM Drive Developments - related to space flight applications - Thread 2
Post by: Rodal on 04/30/2015 03:38 am
Latest paper by Dr. White, on http://ntrs.nasa.gov/ , exploring the idea of the quantum vacuum not being an absolute immutable and nondegradable state, and examining  some rami cations of the quantum vacuum being able to support non-trivial spatial variations in density.  They claim that their "considerations showed no predictions that were contrary to observation, and in fact duplicated predictions for energy states associated with the primary quantum number."

http://hdl.handle.net/2060/20150006842

Dynamics of the Vacuum and Casimir Analogs to the Hydrogen Atom
Harold White, Jerry Vera,y Paul Bailey,z Paul March,x Tim Lawrence,{ Andre Sylvester, and David Brady
NASA Johnson Space Center
2101 NASA Parkway, Houston, TX 77058
(Dated: April 2, 2015)

Publication Date:   Apr 02, 2015
Document ID:   
20150006842 (Acquired Apr 28, 2015)
Subject Category:   PHYSICS OF ELEMENTARY PARTICLES AND FIELDS; ATOMIC AND MOLECULAR PHYSICS; NUMERICAL ANALYSIS; COMPUTER PROGRAMMING AND SOFTWARE
Report/Patent Number:   JSC-CN-33080
Document Type:   Technical Report
Financial Sponsor:   NASA Johnson Space Center; Houston, TX, United States
Organization Source:   NASA Johnson Space Center; Houston, TX, United States
Description:   9p; In English

Title: Re: EM Drive Developments - related to space flight applications - Thread 2
Post by: Star-Drive on 04/30/2015 03:59 am
I am somewhat curious as to why AM modulation would increase the drives output. Is it possible that the modulation scheme is increasing the average power output, or is something else in play.

I'm wondering what it would do with pulsed RF at a high rep rate?

That amplitude, frequency and phase modulation of the carrier wave results in greater thrust force is a prediction from Dr. White's computer code, and not yet an experimentally proven fact.  What we know in this regard is that the experimenters in the UK and China claim to have measured greater thrust force using a magnetron (for whatever reason) and that a magnetron performs amplitude, frequency and phase modulation of the carrier wave. We should exercise caution as neither the UK, nor the Chinese teams have been as forthcoming with data as NASA Eagleworks.  Those experiments (using a magnetron) remain to be replicated at NASA Eagleworks.

Concerning what a pulsed RF at a high rep rate would do, I expect that is something that Dr. White should be able to input in his code and give you an answer, but again whether such an answer from his computer code would be correct remains to be confirmed.

My understanding is that NASA Eagleworks is planning to replicate this June the experiments in the UK and China using a magnetron.


Dr. Rodal:

"That amplitude, frequency and phase modulation of the carrier wave results in greater thrust force is a prediction from Dr. White's computer code, and not yet an experimentally proven fact."

I think I may have verified today the need for large time rate of change of the resonant circuit phase changes as the RF amplifier driven 1,937.088 MHz, +/- ~25kHz sine wave oscillates back and forth through the resonance frequency of the frustum cavity.  Through a methodical tuning campaign using our triple stub Z-matching tuner and 2 feet of RG-8 coax as the main transmission line to the frustum, I marched the Smith Chart solution circle around its impedance space while checking the thrust output for each over a dozen stub tuner configurations.   Only those tuning solutions that maximized the phase change through resonance over the smallest frequency span generated the largest thrust signatures and in fact it overcame its lower Q-factors that those solutions provided.  In fact a running solution that yielded Q-factor solutions as high as 7,500 were out performed by two or even three to one in thrust output by tuning solutions that had half these peak Q-factors, but maximized the resonant phase change per kHz.  And yes, the input power was maintained at around 50W for all tests.  More data later this week as I continue this investigation.

BTW, our Eagleworks Dynamics of the Quantum Vacuum paper has finally been published on the NASA/NTRS server.  You can find it here: 

http://tinyurl.com/mw64rsn

Best, Paul M.
Title: Re: EM Drive Developments - related to space flight applications - Thread 2
Post by: Mulletron on 04/30/2015 04:37 am
http://www.slideshare.net/mobile/KurtZeller/investigation-and-analysis-of-anomalous-electromagnetic-propulsion-devices-41315-46946953

Ran into an author of the above on Reddit and found out his team at California Polytechnic State University is planning a cylinder type experiment with dielectric inserts. :-)

https://www.reddit.com/r/EmDrive/comments/346sw8/big_expository_article_to_be_published_wednesday/cqsv8jl
Title: Re: EM Drive Developments - related to space flight applications - Thread 2
Post by: dustinthewind on 04/30/2015 04:56 am
So other than what is described in the links below, is there any proof that plane waves can travel slower or faster than c in vacuum?

http://www.sciencemag.org/content/347/6224/857
http://arxiv.org/abs/1411.3987

This may or may not apply to what your asking but I found them interesting. 

http://arxiv.org/ftp/physics/papers/9911/9911062.pdf "THE LIGHT VELOCITY CASIMIR EFFECT", "Does the Velocity of Light in a Vacuum Increase When Propagating Between the Casimir Plates?" which suggest the speed of light may increase between the plates. 

I also found it interesting the same authors had a paper on the idea of the quantum vacuum falling near the earth to account for time dilation and gravity.  http://arxiv.org/ftp/physics/papers/9902/9902029.pdf  "DOES THE QUANTUM VACUUM FALL NEAR THE EARTH?"

This paper above is what I suggested before that a sort of fluid was flowing into the earth and its resistance is where we get gravity from.  It is also suggested motion of the fluid also gives time slowing effects (relativity).  This is my first time seeing this paper but I am excited to find what I was thinking about.  This idea of fluid time space I suspect may be linked to any propellantless drive that works (if we are pushing off it).  It may also be linked to how large rotating black holes drag space time into a vortex around them.  It also links the accelerating force felt in an accelerating rocket to the accelerating force of gravity. 
Title: Re: EM Drive Developments - related to space flight applications - Thread 2
Post by: Mulletron on 04/30/2015 05:22 am
http://en.m.wikipedia.org/wiki/Scharnhorst_effect

Still very much not proven.
Title: Re: EM Drive Developments - related to space flight applications - Thread 2
Post by: dustinthewind on 04/30/2015 05:30 am
http://en.m.wikipedia.org/wiki/Scharnhorst_effect

Still very much not proven.

I don't think this proves it but it looks experimental.

http://cds.cern.ch/record/370620 from (https://scholar.google.com/scholar?cluster=1710259071594791986&hl=en&as_sdt=0,48)

Making the vacuum devoid of energy, (wavelengths restricted from existing between the 2 plates) I would assume might change the vacuum dielectric constant to lower than one.  Having an abundance of energy, giving a dielectric greater than one.  Maybe similar to how light might slow down near large gravitational objects, though when measured inside the field it still appears, because of shrunken rulers, that light velocity is still the same. 

Another paper starting on page 14 that suggest super-luminal tunneling (http://arxiv.org/pdf/1008.1640v2.pdf).

This one seems to deal with it specifically (http://arxiv.org/abs/0901.0508). 

one dealing with single photons (http://arxiv.org/abs/0708.3889)

appears connected (http://www.emis.ams.org/journals/SIGMA/2014/005/sigma14-005.pdf) (discusses moving faster than light)

Thanks for pointing out where the idea came from for the super luminal propagation between plates.  Hmm, I suppose from the link you gave it suggests the effect is not measurable (change in velocity) which contrasts with the experimental claim of having measured super-luminal velocity.  Interesting. 
Title: Re: EM Drive Developments - related to space flight applications - Thread 2
Post by: Nine_thermidor on 04/30/2015 06:52 am
I've been following this since New Scientist got in hot water for publishing an article on Shawyer's work back in 2006. It's exciting to see it has not been yet falsified. The debate and contributions in this thread remind me of the SpaceX video fixing effort; NSF is transcending the traditional role of forums and becoming some kind of collaborative knowledge creation nexus.

Preamble over, I have a suggestion (and hopefully this makes it clear why this comment is here rather than in the article thread): There are lots of new posters who are bringing up problems which have already been addressed up thread. One of the problems of the linear thread style discussion is that it makes it difficult for people to see what has been done. The new article is a wonderful overview of the work, but I wonder if there's room for a page/article/site which lists possible sources of error/standard criticisms of the physical impossibility (maybe each as a linkable heading), and describes clearly what has been done to eliminate them from the system or accurately measure errors, and to theoretically address apparent contradictions to physical laws. I have in mind something like Grist's 'how to talk to a climate sceptic' (http://grist.org/series/skeptics/) where many common talking point issues with AGW are listed, and debunked.

It could be useful if this list were editable like a wiki, so that when someone comes with a complaint already addressed, they can be directed to read that page in general, or linked to the explanation of their issue in particular, but also, if the suggestion is new, it can be easily added as an 'as yet un-eliminated issue' and can await further data.
Title: Re: EM Drive Developments - related to space flight applications - Thread 2
Post by: dustinthewind on 04/30/2015 07:24 am
I've been following this since New Scientist got in hot water for publishing an article on Shawyer's work back in 2006. It's exciting to see it has not been yet falsified. The debate and contributions in this thread remind me of the SpaceX video fixing effort; NSF is transcending the traditional role of forums and becoming some kind of collaborative knowledge creation nexus.

Preamble over, I have a suggestion (and hopefully this makes it clear why this comment is here rather than in the article thread): There are lots of new posters who are bringing up problems which have already been addressed up thread. One of the problems of the linear thread style discussion is that it makes it difficult for people to see what has been done. The new article is a wonderful overview of the work, but I wonder if there's room for a page/article/site which lists possible sources of error/standard criticisms of the physical impossibility (maybe each as a linkable heading), and describes clearly what has been done to eliminate them from the system or accurately measure errors, and to theoretically address apparent contradictions to physical laws. I have in mind something like Grist's 'how to talk to a climate sceptic' (http://grist.org/series/skeptics/) where many common talking point issues with AGW are listed, and debunked.

It could be useful if this list were editable like a wiki, so that when someone comes with a complaint already addressed, they can be directed to read that page in general, or linked to the explanation of their issue in particular, but also, if the suggestion is new, it can be easily added as an 'as yet un-eliminated issue' and can await further data.

Maybe a flow diagram that can connect comments with in the thread.  When you click a large thread the diagram is available.  click a box and it takes you into another diagram of multiple comments that have been linked to that particular part of the flow diagram.  It might make it easier to navigate and connect far spaced comments. 
Title: Re: EM Drive Developments - related to space flight applications - Thread 2
Post by: Flyby on 04/30/2015 09:15 am
I'm aware that my technical skills are most likely inferior to most of the ppl working at Eagleworks, so I've been pondering if it would be wise to bring up the following issue, but ah, why not...

About the high powered Shawyer-like rotating demonstrator that Eagleworks is building to be ready by june:

One of the main criticism about the non-vacuum demonstrator that Shawyer build, to showoff one of his early EMdrive workings, was that - according to critics - some kind of hot air jet was leaking from non-welded micro openings at the large base of the frustum.

I understand that Eagleworks will also not operate their version in vacuum, as its purpose is to illustrate an effective motion generated by the EM drive. Consequently they will most likely encounter the same remarks...

Countering that "hot air jet" criticism is fairly easy : just build an open enclosure that, in case of a leakage jet , would generate an opposing reaction force to the direction the frustum is supposed to go.

IF the setup turns clockwise, then some sort of hot jet is indeed in play,
however...
if it rotates counterclockwise then clearly, the EMdrive is for real...

In all honesty, a simple well taped cardboard enclosure would be enough, just to redirect the supposedly reactionary "exhaust" forces... Of course , testing with and without enclosure is a must, for reference.

This upcoming test in June will be a real "make or brake" event. No more endless discussions about measurement procedures and potential measurement errors.

 I'm sooo in anticipation modus now.. All it needs to do is...TURN in the right direction.  :-X
Title: Re: EM Drive Developments - related to space flight applications - Thread 2
Post by: KelvinZero on 04/30/2015 09:22 am
can anyone explain this point better?

from http://emdrive.com/faq.html
Q. Why does the thrust decrease as the spacecraft velocity along the thrust vector increases?
A. As the spacecraft accelerates along the thrust vector, energy is lost by the engine and gained as additional kinetic energy by the spacecraft. This energy can be defined as the thrust multiplied by the distance through which the thrust acts. For a given acceleration period, the higher the mean velocity, the longer the distance travelled, hence the higher the energy lost by the engine.
This loss of stored energy from the resonant cavity leads to a reduction in Q and hence a reduction of thrust.

The key question is why can't you just turn the machine off and start again from a new reference frame, giving you a traditional interpretation of a propellentless drive and free energy.
Title: Re: EM Drive Developments - related to space flight applications - Thread 2
Post by: CW on 04/30/2015 09:49 am
can anyone explain this point better?

from http://emdrive.com/faq.html
Q. Why does the thrust decrease as the spacecraft velocity along the thrust vector increases?
A. As the spacecraft accelerates along the thrust vector, energy is lost by the engine and gained as additional kinetic energy by the spacecraft. This energy can be defined as the thrust multiplied by the distance through which the thrust acts. For a given acceleration period, the higher the mean velocity, the longer the distance travelled, hence the higher the energy lost by the engine.
This loss of stored energy from the resonant cavity leads to a reduction in Q and hence a reduction of thrust.

The key question is why can't you just turn the machine off and start again from a new reference frame, giving you a traditional interpretation of a propellentless drive and free energy.

Reading that Q/A, ascribing a 'book-keeping' ability to such a device in regards to reference frames sounds unphysical to me. I can't do anything with that. I tend to believe that Mr. Shawyer might have accidentally found something mind-blowing.. but his explanations just don't add up. It might work for, uh, different reasons than he assumes.
;)
Title: Re: EM Drive Developments - related to space flight applications - Thread 2
Post by: Star One on 04/30/2015 10:15 am

can anyone explain this point better?

from http://emdrive.com/faq.html
Q. Why does the thrust decrease as the spacecraft velocity along the thrust vector increases?
A. As the spacecraft accelerates along the thrust vector, energy is lost by the engine and gained as additional kinetic energy by the spacecraft. This energy can be defined as the thrust multiplied by the distance through which the thrust acts. For a given acceleration period, the higher the mean velocity, the longer the distance travelled, hence the higher the energy lost by the engine.
This loss of stored energy from the resonant cavity leads to a reduction in Q and hence a reduction of thrust.

The key question is why can't you just turn the machine off and start again from a new reference frame, giving you a traditional interpretation of a propellentless drive and free energy.

Reading that Q/A, ascribing a 'book-keeping' ability to such a device in regards to reference frames sounds unphysical to me. I can't do anything with that. I tend to believe that Mr. Shawyer might have accidentally found something mind-blowing.. but his explanations just don't add up. It might work for, uh, different reasons than he assumes.
;)

Well you might ask how the electrons in the double slit experiment know they are being observed and therefore act differently.
Title: Re: EM Drive Developments - related to space flight applications - Thread 2
Post by: qraal on 04/30/2015 10:43 am
can anyone explain this point better?

from http://emdrive.com/faq.html
Q. Why does the thrust decrease as the spacecraft velocity along the thrust vector increases?
A. As the spacecraft accelerates along the thrust vector, energy is lost by the engine and gained as additional kinetic energy by the spacecraft. This energy can be defined as the thrust multiplied by the distance through which the thrust acts. For a given acceleration period, the higher the mean velocity, the longer the distance travelled, hence the higher the energy lost by the engine.
This loss of stored energy from the resonant cavity leads to a reduction in Q and hence a reduction of thrust.

The key question is why can't you just turn the machine off and start again from a new reference frame, giving you a traditional interpretation of a propellentless drive and free energy.

Contra the usual popularisations, Special Relativity doesn't remove the possibility of a preferred reference frame for the Universe, it just makes it very hard to observe. An alternative, being explored, is Lorentzian relativity, which experimentally gives much the same results as SR, but an absolute frame is preserved. This makes 'accounting' for one object's energy relative to another quite straightforward. These thrusters might provide one means of experimentally determining which version of Relativity is correct.
Title: Re: EM Drive Developments - related to space flight applications - Thread 2
Post by: KelvinZero on 04/30/2015 11:19 am
Contra the usual popularisations, Special Relativity doesn't remove the possibility of a preferred reference frame for the Universe, it just makes it very hard to observe. An alternative, being explored, is Lorentzian relativity, which experimentally gives much the same results as SR, but an absolute frame is preserved. This makes 'accounting' for one object's energy relative to another quite straightforward. These thrusters might provide one means of experimentally determining which version of Relativity is correct.
That is one branch of that tree I mentioned: it works wrt to some specific frame of reference. In this case it is probably not at rest when turned on and would have a tendency to rush off in one direction, and generate power rather than expend it. For example apparently there is something called the comoving cosmic rest frame that earth is moving at about 400km/s relative to. http://en.wikipedia.org/wiki/Cosmic_microwave_background#Features

We probably shouldn't speculate here though. I would prefer to hear the official answer.
Title: Re: EM Drive Developments - related to space flight applications - Thread 2
Post by: Star-Drive on 04/30/2015 11:57 am
I'm aware that my technical skills are most likely inferior to most of the ppl working at Eagleworks, so I've been pondering if it would be wise to bring up the following issue, but ah, why not...

About the high powered Shawyer-like rotating demonstrator that Eagleworks is building to be ready by june:

One of the main criticism about the non-vacuum demonstrator that Shawyer build, to showoff one of his early EMdrive workings, was that - according to critics - some kind of hot air jet was leaking from non-welded micro openings at the large base of the frustum.

I understand that Eagleworks will also not operate their version in vacuum, as its purpose is to illustrate an effective motion generated by the EM drive. Consequently they will most likely encounter the same remarks...

Countering that "hot air jet" criticism is fairly easy : just build an open enclosure that, in case of a leakage jet , would generate an opposing reaction force to the direction the frustum is supposed to go.

IF the setup turns clockwise, then some sort of hot jet is indeed in play,
however...
if it rotates counterclockwise then clearly, the EMdrive is for real...

In all honesty, a simple well taped cardboard enclosure would be enough, just to redirect the supposedly reactionary "exhaust" forces... Of course , testing with and without enclosure is a must, for reference.

This upcoming test in June will be a real "make or brake" event. No more endless discussions about measurement procedures and potential measurement errors.

 I'm sooo in anticipation modus now.. All it needs to do is...TURN in the right direction.  :-X

A bit of clarification:  The Eagleworks crew hope to have the 1.2kW magnetron teeter-totter test-rig up and running by the END of June or perhaps now the first couple of weeks of July 2015.  At the moment we are in the middle component procurement & fabrication, and that task is taking longer than expected due to mill breakdowns in the NASA fabrication shop and the usual supply vendor's taking their time to fill and ship orders.

Best, Paul M.
Title: Re: EM Drive Developments - related to space flight applications - Thread 2
Post by: Mulletron on 04/30/2015 01:22 pm
I am somewhat curious as to why AM modulation would increase the drives output. Is it possible that the modulation scheme is increasing the average power output, or is something else in play.

I'm wondering what it would do with pulsed RF at a high rep rate?

That amplitude, frequency and phase modulation of the carrier wave results in greater thrust force is a prediction from Dr. White's computer code, and not yet an experimentally proven fact.  What we know in this regard is that the experimenters in the UK and China claim to have measured greater thrust force using a magnetron (for whatever reason) and that a magnetron performs amplitude, frequency and phase modulation of the carrier wave. We should exercise caution as neither the UK, nor the Chinese teams have been as forthcoming with data as NASA Eagleworks.  Those experiments (using a magnetron) remain to be replicated at NASA Eagleworks.

Concerning what a pulsed RF at a high rep rate would do, I expect that is something that Dr. White should be able to input in his code and give you an answer, but again whether such an answer from his computer code would be correct remains to be confirmed.

My understanding is that NASA Eagleworks is planning to replicate this June the experiments in the UK and China using a magnetron.


Dr. Rodal:

"That amplitude, frequency and phase modulation of the carrier wave results in greater thrust force is a prediction from Dr. White's computer code, and not yet an experimentally proven fact."

I think I may have verified today the need for large time rate of change of the resonant circuit phase changes as the RF amplifier driven 1,937.088 MHz, +/- ~25kHz sine wave oscillates back and forth through the resonance frequency of the frustum cavity.  Through a methodical tuning campaign using our triple stub Z-matching tuner and 2 feet of RG-8 coax as the main transmission line to the frustum, I marched the Smith Chart solution circle around its impedance space while checking the thrust output for each over a dozen stub tuner configurations.   Only those tuning solutions that maximized the phase change through resonance over the smallest frequency span generated the largest thrust signatures and in fact it overcame its lower Q-factors that those solutions provided.  In fact a running solution that yielded Q-factor solutions as high as 7,500 were out performed by two or even three to one in thrust output by tuning solutions that had half these peak Q-factors, but maximized the resonant phase change per kHz.  And yes, the input power was maintained at around 50W for all tests.  More data later this week as I continue this investigation.

BTW, our Eagleworks Dynamics of the Quantum Vacuum paper has finally been published on the NASA/NTRS server.  You can find it here: 

http://tinyurl.com/mw64rsn

Best, Paul M.


In reference to the item highlighted in blue above about phase shifting. It helps if you highlight the word phase in those two posts linked to below. So that's multiple sources supporting the importance of phase shifting and now a possible experimental verification.

http://forum.nasaspaceflight.com/index.php?topic=36313.msg1331792#msg1331792
http://forum.nasaspaceflight.com/index.php?topic=36313.msg1331993#msg1331993

Folks:

In the meantime, lets ask why 60 watts of relatively harmonic free sine-wave RF power at the 1,937.118 MHz AKA the TM212 resonant frequency in this copper frustum cavity, can only generate a paltry ~60uN, whereas the Chinese claimed to have produce 160,000uN using just ~150 watts of 2,450 MHz RF signals from a magnetron?  The magnetron RF signal source that is anything but a pure sine-wave generator, that instead has a modulated FM bandwidth of at least +/-30 MHz that is also concurrently amplitude modulated (AM) with thermal electron noise. 


Taking a critical look at this question, and knowing that the spectral shape of a magnetron looks like (see below) compared to a CW spike. It seems evident that a CW spike isn't the best waveform to use if you want to maximize thrust. Dollars to donuts says the Chinese are making full use of the available bandwidth of their resonant cavity by using that noisy magnetron. Magnetrons have lots of phase noise too. You can't easily use them on phased array radars because of that for example.

Now to put this idea to test, Q: What is the bandwidth of the resonant cavity and what is the 90 percent power bandwidth of the signal you are driving it with? What kind of sig gen are you using? Can it do FM? Can you do any advanced waveforms like a PSK waveform? Do you have a way to produce wideband noise or a spread spectrum carrier for your testing? Can you do any waveforms like at the bottom?

Also during researching other possible theories which could explain Emdrive we found ample literature stating that molecules acquire a kinetic momentum during the switching of the magnetic field as a result of its interaction with the vacuum field. If correct, that may well be a very significant lead. So that raises the question, how does one increase the switching rate? What about phase shifting? http://en.wikipedia.org/wiki/Phase-shift_keying

Phase shifting seems important.
https://www.viasat.com/files/assets/web/datasheets/EBEM_MD-1366_043_web.pdf
One of these driving your amp would be helpful. They go up to 2ghz.
Title: Re: EM Drive Developments - related to space flight applications - Thread 2
Post by: cfs on 04/30/2015 01:31 pm
http://en.m.wikipedia.org/wiki/Scharnhorst_effect

Still very much not proven.

I don't think this proves it but it looks experimental.

http://cds.cern.ch/record/370620 from (https://scholar.google.com/scholar?cluster=1710259071594791986&hl=en&as_sdt=0,48)

Making the vacuum devoid of energy, (wavelengths restricted from existing between the 2 plates) I would assume might change the vacuum dielectric constant to lower than one.  Having an abundance of energy, giving a dielectric greater than one.  Maybe similar to how light might slow down near large gravitational objects, though when measured inside the field it still appears, because of shrunken rulers, that light velocity is still the same. 

Another paper starting on page 14 that suggest super-luminal tunneling (http://arxiv.org/pdf/1008.1640v2.pdf).

This one seems to deal with it specifically (http://arxiv.org/abs/0901.0508). 

one dealing with single photons (http://arxiv.org/abs/0708.3889)

appears connected (http://www.emis.ams.org/journals/SIGMA/2014/005/sigma14-005.pdf) (discusses moving faster than light)

Thanks for pointing out where the idea came from for the super luminal propagation between plates.  Hmm, I suppose from the link you gave it suggests the effect is not measurable (change in velocity) which contrasts with the experimental claim of having measured super-luminal velocity.  Interesting.

These are all looking at the group velocity, which is not necessarily the same as the signal propagation velocity. 
Title: Re: EM Drive Developments - related to space flight applications - Thread 2
Post by: Dmytry on 04/30/2015 01:54 pm
Well, one shouldn't even see a drift the size of the measured force, when it's not even sub-micronewton range.

If one is heating metal, one is probably heating it unevenly, and it bends as long as heat is applied (the hotter material expands more), with a fairly short time constant (because the temperatures will equalize quickly once the heat source is off). Then the heat gets slowly conducted to another structural element, and that structural element happens to bend in a way that influences the experiment in the other direction.

How much each structural element will bend will depend to how tight the screws are tightened, what stresses are already in the metal, and so on and so forth; it's essentially impossible to account for.
Heat transfer, thermal expansion, thermal stress analysis for uncoupled and for coupled problems, in static and dynamic problems of complicated geometry and materials can be analyzed and modeled with Finite Element analysis (NASTRAN, ANSYS, ABAQUS, ADINA, COMSOL, etc.), just like rockets, spacecrafts and military and commercial aircraft have been analyzed for decades.
Not to this level of precision and not without knowing material properties.

Quote
Here is an exact solution of the thermal buckling problem for the EM Drive's truncated cone base:

https://www.researchgate.net/publication/268804028_NASA%27S_MICROWAVE_PROPELLANT-LESS_THRUSTER_ANOMALOUS_RESULTS_CONSIDERATION_OF_A_THERMO-MECHANICAL_EFFECT

The issue for analyzing thermal structural effects (and other effects) for the EM Drive project at NASA Eagleworks is not a matter of present ability to be analyzed but is, instead, a matter of scarcity of project resources (money, time, and personnel) to analyze them. For example, NASA's EM Drive truncated cone was made by Paul March himself in Paul March's wife dining room. NO TAXPAYER's tax $$$ involved in its construction.
Now if he also made the vacuum chamber in his wife's dining room, and took apart his microwave oven for the RF source, and been testing everything in his basement, there wouldn't be a problem here.

Quote
http://forum.nasaspaceflight.com/index.php?topic=36313.msg1327937#msg1327937

(http://forum.nasaspaceflight.com/index.php?action=dlattach;topic=36313.0;attach=635221;image)

Engineers and scientists interested in this project, (like @frobnicat, @notsosureofit, @aero, myself, etc.) have contributed their time as well to model different aspects of this experiment to try to distinguish any artifacts from real effects.   Earlier on I contributed results of a fully-coupled nonlinear analysis of the torque pendulum used in the experiment, as well as Autocorrelation, and Power Spectral Density analysis of the data.

Concerning the issue of shielding, @aero has contributed a MEEP analysis of evanescent waves leaking from the EM Drive, and possibly interacting with the stainless steeel vacuum chamber.  Subsequently, Paul March conducted an experiment with the EM Drive outside the stainless steel chamber that may have nullified that explanation.

@Mulletron has contributed his own resources and time to run his own EM Drive experiment from a hanging torsional pendulum.

Paul March has contributed detailed information about the experimental set-up, and if you are interested, you can also contribute any engineering, preferably quantitative, theoretical, numerical or experimental analysis you may be able to contribute.
Sorry, I would trust a shielding box much more than I would trust your calculations (when you are apparently unable to even get rid of the "thermal drift"). The pendulum arms are mechanically complex and made of materials that were never characterized with a necessary degree of precision. The cavity is made in a dining room, as you say. edit: and the springs that the pendulum is suspended on are getting heated up, no? Possibly by an electric current? Seriously?

When it comes to energy (rather than momentum), there's always some people who got 1000W coming into a device, it makes 1010W heat, and you can plainly see that they aren't making the measurements to a necessary accuracy as to claim excess heat - they got various noise larger than their signal. Then they insist that they accounted for all the inaccuracies involving in the measurement of either.
Title: Re: EM Drive Developments - related to space flight applications - Thread 2
Post by: sghill on 04/30/2015 02:10 pm
Lastly, the Eagleworks Lab's next paper on the Q-V entitled "Dynamics of the Vacuum" will be out on the NASA NTRS internet servers just any day now....

Sorry if I missed it, but did this paper get released?  If so, where?
Title: Re: EM Drive Developments - related to space flight applications - Thread 2
Post by: Rodal on 04/30/2015 02:24 pm
"Dmytry" please stop quoting me  ( https://forum.nasaspaceflight.com/index.php?topic=36313.msg1366843#msg1366843 ) , and simultaneously using "you" incorrectly:

you are apparently unable to even get rid of the "thermal drift"

What "thermal drift" am I unable to get rid of?

The first time that you used the "you" incorrectly ( https://forum.nasaspaceflight.com/index.php?topic=36313.msg1365922#msg1365922 ) you edited your post by stating that you were referring to a "generic you".

Now you insist on mixing me with others.  When posting your criticisms you should not use "you" if you are referring to somebody else.  Why do you insist on using a generic "you"?  Do you mean "you people in the EM Drive thread"? "you people in the NSF forum"? "you people interested in NASA and outer space"? "you Americans"? Who are you intentionally and falsely trying to group together as the object of your criticism by insisting on using a "generic you"?

You ("Dmytry") use only words to mount your criticisms, directed to a group by using a "generic you" without defining what group you are intending to group together.  Engineers and scientists use mathematics and analysis instead, and they support their statements by linked peer-reviewed references.
Title: Re: EM Drive Developments - related to space flight applications - Thread 2
Post by: marshallC on 04/30/2015 02:29 pm
My apologies if this has already been answered - I've looked back into the thread somewhat but may have missed it. Will the results of this next test be observable to a layman - that is, some kind of  movement or the like? If so, would it be allowable under the rules of the Eagleworks team to stream it, possibly on something as simple as a twitch.tv setup? I know I'd watch it :)
Title: Re: EM Drive Developments - related to space flight applications - Thread 2
Post by: Star One on 04/30/2015 02:31 pm

Lastly, the Eagleworks Lab's next paper on the Q-V entitled "Dynamics of the Vacuum" will be out on the NASA NTRS internet servers just any day now....

Sorry if I missed it, but did this paper get released?  If so, where?

Look up thread for one of Dr March's recent post as its linked to in that.
Title: Re: EM Drive Developments - related to space flight applications - Thread 2
Post by: Lampyridae on 04/30/2015 02:32 pm
@Marshall C

Not at these thrust levels, I'm afraid. It's like a 10kg engine producing enough energy to lift a sheet of paper.  Perhaps you could see the physical movement of a very sensitive balance but nothing as dramatic as the ion wind devices known as "lifters". First prize would indeed be a battery-powered device levitating into the conference hall.
Title: Re: EM Drive Developments - related to space flight applications - Thread 2
Post by: SH on 04/30/2015 03:06 pm
Hello, I have been following Dr. White's publications with interest since he revived the Alcubierre concept in 2003.  I have a few comments and questions below.

--- On the EM drive for propulsion ---

(1) It has been argued that if the EM drive works, it would violate conservation of momentum and thus also conservation of energy.  This is not true.  Rather, if the EM drive is real, it means that the standard assumption that the quantum virtual plasma (QVP) is immutable would be false.

(2) Clarifying how momentum would be conserved:  Dr. White's theory is that the drive exchanges momentum with the QVP. Some people have said that this still does not conserve momentum because the virtual particles cannot disappear   after having exchanged momentum as this would violate conservation of momentum.  This is true -- if momentum is   transferred to a virtual particle, that virtual particle must transfer the momentum to a different virtual particle   before it can disappear.  Thus, the momentum of the ship would be propagated through the quantum virtual plasma as a wave until it reaches classical objects that can absorb the momentum permanently.  Think of the QVP as water being acted on by the propeller of a boat, with the exception that when the wake of the boat hits the shore it is absorbed by
  the shore instead of reflected back into the water (ie, there is no 'surf')

(3) Evidence in support of the QVP being mutable.  a) the force measurements of the EM-drive, b) the Casimir effect, c) as explained here (http://nextbigfuture.com/2015/02/more-emdrive-experiment-information.html), apparently Dr. White was able to show that the electron shell radii of all atoms up to atomic number 7 can be predicted based on the asumption that QVP is mutable.  I haven't read the details of that and would be curious to read where this is published if anyone knows.  d) A generic property of inflationary cosmology (as written about by Hawking, Alan Guth, Hartle, Turok, Pasachoff, Filippenko, Stenger, Vilenkin and others) is that the universe began from a small quantum fluctuation from the ground state, as stated by Vilenkin "small amount of energy was contained in that [initial] curvature, somewhat like the energy stored  in  a  strung  bow.   This  ostensible  violation  of  energy conservation is allowed by the Heisenberg uncertainty principle for sufficiently small time intervals.  The bubble then inflated exponentially and the universe grew by many orders of magnitude in a tiny fraction of a second".  Thus, it seems that inflationary cosmology is founded on a principle of mutable QVP as well.

(4) As discussed here (http://www.nasaspaceflight.com/2015/04/evaluating-nasas-futuristic-em-drive/), "The mainstream physics community assumes the Quantum Vacuum is indestructible and immutable because of the experimental observation that a fundamental particle like an electron (or a positron) has the same properties (e.g. mass, charge or spin), regardless of when or where the particle was created, whether now or in the early universe, through astrophysical processes or in a laboratory."  My question to Dr. White's team is, if your theory is correct and the QVP is mutable, then how would this be predicted to change the mass, charge, or spin of any classical particles that happened to be in the wake of an EMdrive thruster?

(5) Follow up question: if it would change the mass, charge, or spin of classical particles then it seems this would change the stable configuration of molecules or atoms and could result in spontaneous chemical reactions or cold fusion or fission, and these reactions could potentially be exploited to help generate power needed to run the EM drive. Thoughts?

(5.5) If the EmDrive works out, then it seems a small Throium reactor might be the ideal power source for long term missions: https://www.youtube.com/watch?v=N2vzotsvvkw (https://www.youtube.com/watch?v=N2vzotsvvkw)

-- On the EM drive as a method of warping spacetime --

(6) In "The Alcubierre Warp Drive in Higher Dimensional Spacetime", White and Davis (2006) theorized that, under the Chung-Freese model they predicted any torus of positive energy density would give rise to slight negative energy density in its core due to classical energy in 3+1 dimensions being shifted "off brain" into the unobservable higher dimensions.  They proposed an experiment to test this by constructing a charged capacitor ring.  My question: under the mass-energy equivalence, wouldn't you expect to get much more effect by using a rapidly spinning torus made of lead?

(7) If it were true that any torus of positive energy density contributes to a "boost" factor inside the torus, then it must be to an incredibly small amount, or else people would have noticed by pure chance that objects inside torus tend to move faster, and nobody has noticed this.  However, we have noticed that large heavy toruses require more fuel to propel.  Thus it seems that the theory of a positive energy density torus giving rise to a net boost in thrust must be impossible.

( 8 ) I feel like the equivalence between "boost" and "off brain bulk" in this paper was entirely speculative and unsupported by any real sound argument.

(9) In "Experimental Concepts for Generating Negative Energy in the Laboratory" Davis and Puthoff (2006) showed that negative energy density was producible in the lab using high energy lasers and other methods, and this would not require the more radical assumptions of extra dimensions in the Chung-Freese model.  Why weren't these methods explored?

(10) Supposing that you were able to engineer the required negative energy density around the craft in such a way as to produce a warp bubble.  The spacetime curvature inside might be flat, but not around the region of negative energy density, thus it seems that the warping of spacetime would necessarily obliterate any toroidal design used to hold the negative energy in place.
Title: Re: EM Drive Developments - related to space flight applications - Thread 2
Post by: Rodal on 04/30/2015 03:23 pm
Lastly, the Eagleworks Lab's next paper on the Q-V entitled "Dynamics of the Vacuum" will be out on the NASA NTRS internet servers just any day now....

Sorry if I missed it, but did this paper get released?  If so, where?


See:  https://forum.nasaspaceflight.com/index.php?topic=36313.msg1366622#msg1366622

This is the direct link:  http://hdl.handle.net/2060/20150006842
Title: Re: EM Drive Developments - related to space flight applications - Thread 2
Post by: LasJayhawk on 04/30/2015 04:11 pm
Star-drive: is your FM deviation noise or are you modulating the signal, with a sine wave perhaps?

SH: as to number 2 try this. The QVP is the water, the frustum is the boat, and the RF signal is the oar. Right now we are just flailing away at the water and getting little thrust for the energy expended. But Star-Drive is starting to learn how to use his new oar, as he refines his technique the paddling will get easier.
Title: Re: EM Drive Developments - related to space flight applications - Thread 2
Post by: Notsosureofit on 04/30/2015 05:11 pm
I am somewhat curious as to why AM modulation would increase the drives output. Is it possible that the modulation scheme is increasing the average power output, or is something else in play.

I'm wondering what it would do with pulsed RF at a high rep rate?

That amplitude, frequency and phase modulation of the carrier wave results in greater thrust force is a prediction from Dr. White's computer code, and not yet an experimentally proven fact.  What we know in this regard is that the experimenters in the UK and China claim to have measured greater thrust force using a magnetron (for whatever reason) and that a magnetron performs amplitude, frequency and phase modulation of the carrier wave. We should exercise caution as neither the UK, nor the Chinese teams have been as forthcoming with data as NASA Eagleworks.  Those experiments (using a magnetron) remain to be replicated at NASA Eagleworks.

Concerning what a pulsed RF at a high rep rate would do, I expect that is something that Dr. White should be able to input in his code and give you an answer, but again whether such an answer from his computer code would be correct remains to be confirmed.

My understanding is that NASA Eagleworks is planning to replicate this June the experiments in the UK and China using a magnetron.


Dr. Rodal:

"That amplitude, frequency and phase modulation of the carrier wave results in greater thrust force is a prediction from Dr. White's computer code, and not yet an experimentally proven fact."

I think I may have verified today the need for large time rate of change of the resonant circuit phase changes as the RF amplifier driven 1,937.088 MHz, +/- ~25kHz sine wave oscillates back and forth through the resonance frequency of the frustum cavity.  Through a methodical tuning campaign using our triple stub Z-matching tuner and 2 feet of RG-8 coax as the main transmission line to the frustum, I marched the Smith Chart solution circle around its impedance space while checking the thrust output for each over a dozen stub tuner configurations.   Only those tuning solutions that maximized the phase change through resonance over the smallest frequency span generated the largest thrust signatures and in fact it overcame its lower Q-factors that those solutions provided.  In fact a running solution that yielded Q-factor solutions as high as 7,500 were out performed by two or even three to one in thrust output by tuning solutions that had half these peak Q-factors, but maximized the resonant phase change per kHz.  And yes, the input power was maintained at around 50W for all tests.  More data later this week as I continue this investigation.

BTW, our Eagleworks Dynamics of the Quantum Vacuum paper has finally been published on the NASA/NTRS server.  You can find it here: 

http://tinyurl.com/mw64rsn

Best, Paul M.

Yes, that is essentially "Q Multiplication".
Title: Re: EM Drive Developments - related to space flight applications - Thread 2
Post by: Space Ghost 1962 on 04/30/2015 05:27 pm
I've examined his simulation/model. There are other ways to interpret the physics behind it.

Model can match results and yet the interpretation of "why" it works be completely different than described.

In particular, the equations remind me of particle accelerator(SSC) and mirror machine (fusion) simulations I've had to debug in the 70's - 80's. Found some surprises in Lorenz effects there reconciling Biot Svart.

I think there's something here, and I think his simulation partially matches it. But I've been through his explanation and it doesn't jive with particle/other physics results in recent years. For one, the Higgs would have to have a considerably different appearance than we are currently seeing.

The mathematics being used here is highly subjective to many different interpretations.

Suggest he found something else that's interesting than he think he's found.

Suggest also both the extreme enthusiasm and the extreme reductionism be avoided by advocates/pessimists.
Title: Re: EM Drive Developments - related to space flight applications - Thread 2
Post by: Mulletron on 04/30/2015 05:44 pm
I was given permission to share these communications. Mr. Shawyer also graciously attached a basic theory presentation which I put in the Google Drive to pass along.

His last communication is especially telling. Perhaps if folks can remain respectful (even if you disagree with his theory), maybe we can get him to speak publicly again.

While the jury is still out whether or not this will be useful for spaceflight (at least here in the USA), there is obviously an urgent need to give this serious academic attention.

https://drive.google.com/folderview?id=0B4PCfHCM1KYoTXhSUTd5ZDN2WnM&usp=sharing
Title: Re: EM Drive Developments - related to space flight applications - Thread 2
Post by: Space Ghost 1962 on 04/30/2015 06:42 pm
Thank you for this. Having a lively discussion with others about this right now.

Suggest those interested annotate his "EMdrive basic theory" ppt and place it where it can be seen by others (respectfully).

Please keep in mind ensemble dynamics when considering it, like with any waveguide/magnetron/klystron and not field theory or simple E/B field evaluations - this will avoid nonsensical "challenges" thus better s/n ratio for "crowd sourced" contributions.

Please also thank the author for a simple case to focus on here.

add:
Hint - group and phase velocity are not kept separate. This confuses the presentation and allows ambiguity to creep into the discussion. Strongly suggest that the presentation delineate the difference because otherwise different groups criticizing will assert different combinations as a result, and everyone will talk by each other in an unfortunate way. Also, near field and far field effects are quite different. "There be tigers here".
Title: Re: EM Drive Developments - related to space flight applications - Thread 2
Post by: tchernik on 04/30/2015 06:46 pm
I empathize with Mr. Shawyer: it's not easy to carry the burden of proving something to a mostly hostile crowd. Especially on the Internet, where people feeling anonymous can be particularly vicious in their criticism.

If the Emdrive is confirmed to work, he would have all the merit for finding this phenomenon and making it repeatable enough for convincing others to do their own replications.

But it's my understanding that his theory of how the Emdrive works has been torn to shreds by several people. Probably not in front of him, but certainly online and in the media (with Greg Egan being a very proactive critic).

This is completely independent of the empiric problem of the Emdrive working (or not). The theory can be wrong and the device still work.

The same for Dr. White's QV approach. Nevertheless, if the results follow his present forecasts of a certain amount of force per certain amount of input power, that could help reinforce the idea that his theory might be onto something. And it can help reinforce  the assertion that the thrust is real and usable.
Title: Re: EM Drive Developments - related to space flight applications - Thread 2
Post by: Rodal on 04/30/2015 06:54 pm
For information, here are the links to:

Roger Shawyer's theory paper:

http://www.emdrive.com/theorypaper9-4.pdf

and

Greg Egan's analysis:

http://gregegan.customer.netspace.net.au/SCIENCE/Cavity/Cavity.html

Basically, this is the issue: the solution for Maxwell's equation in a truncated cone cavity is comprised of standing waves, with the electromagnetic field displaying a harmonic motion every period.

Only travelling waves, in open cavities, display a directional flux.  Standing waves in closed cavities, do not, because for standing waves  the Poyinting vector is pointing towards the small base of the truncated cone during half a period, and it is pointing towards the opposite direction, towards the big base during the next half period of the Poynting vector frequency.

Therefore, the solution of Maxwell's equation shows that the effects are self-cancelling during a whole period in a closed cavity.

No unidirectional thrust of the EM Drive can take place based on Maxwell's equations, which are also valid in special relativity. 

If experiments shows a directional thrust, then appeal must be made to a different theoretical explanation (other than linear Maxwell's equations having a simple harmonic solution).

Concerning "microwave cavity engineering" this is a truly excellent article on microwave cavity design, from CERN, highly recommended:

http://cas.web.cern.ch/cas/Germany2009/Lectures/PDF-Web/Jensen.pdf
Title: Re: EM Drive Developments - related to space flight applications - Thread 2
Post by: zero123 on 04/30/2015 07:06 pm
I know Mulletron. What I want to know is exactly how Dr White theory deals with superluminal speeds, which most physicists say leads to time-travel to the past and all the paradoxes that surface from that.


According to the video I showed from Dr Davis, superluminal speeds WITHOUT time-travel to the past are possible, if the light cone is tilted from 0 to 90 degrees only...


I think this question is related to spaceflight applications exactly because time travel IS an issue at relativistic velocities (to the future) and superluminal velocities (to the past, but not according to Dr Davis)

This question is probably more related to the spaceflight applications of a warp drive than the pure theoretical issues of how EM and Warp Drives work on quantum level, since the first is a result of spaceflight application while the second (which is being discussed in this thread) is not.

Don't forget that the ship is not really moving at relativistic speeds: space is.  Consequently, you could take a trip to Alpha Centauri in 2 days (or less with more power... who knows?), turn your ship around and observe the Earth as it was four years ago (as light has taken four years to get there - slow coach!). You could then observe Alpha Centauri as it is "now", and how people on the Earth will see it in four years.

With this type of technology,  it would be possible to predict when locally past events are going to be observable from the point of view of the Earth (or any other point that the light from such events had not yet reached). For example, a ship 1 light-day out from the Earth in the right place could witness a supernova before the Earth does and then be able to return to the Earth almost instantly and tell astronomers about the incoming light wave so that they could prepare to observe it.

Proviso: I am not an expert in time travel and I also have doubts about Dr Who.

(apologies if this is has already been discussed)

The question of whether/how warp drives can be used to form CTCs and time travel has been explored in this paper:

http://exvacuo.free.fr/div/Sciences/Dossiers/Time/A%20E%20Everett%20-%20Warp%20drive%20and%20causality%20-%20prd950914.pdf

The conclusion (as I understand it) is that yes, you can form a CTC if you use two separate bubbles. You travel from A to B in one warp bubble, then from B to C using a standard subluminal method and then from C back to A using another warp bubble. Then you can arrive back at A before you left for B.
Title: Re: EM Drive Developments - related to space flight applications - Thread 2
Post by: Mulletron on 04/30/2015 07:07 pm
And what about the experimental observation of non-reciprocity in room temperature and pressure nitrogen (largest portion of air) that was reported? That is also unbalanced momentum.
Title: Re: EM Drive Developments - related to space flight applications - Thread 2
Post by: Rodal on 04/30/2015 07:14 pm
And what about the experimental observation of non-reciprocity in room temperature and pressure nitrogen (largest portion of air) that was reported? That is also unbalanced momentum.
From my recollection of reading your prior links, those involved nonlinear polarization, hence not addressed by linear, isotropic Maxwell's equations.  One of the papers addressed a 2nd order term and another addressed a 4th order term in a nonlinear perturbation analysis.

Clearly, a nonlinear differential equation does not have a simple harmonic solution.
Title: Re: EM Drive Developments - related to space flight applications - Thread 2
Post by: Mulletron on 04/30/2015 07:25 pm
And what about the experimental observation of non-reciprocity in room temperature and pressure nitrogen (largest portion of air) that was reported? That is also unbalanced momentum.
From my recollection of reading your prior links, those involved nonlinear polarization, hence not addressed by linear, isotropic Maxwell's equations.  One of the papers addressed a 2nd order term and another addressed a 4th order term in a nonlinear perturbation analysis.

Clearly, a nonlinear differential equation does not have a simple harmonic solution.

http://forum.nasaspaceflight.com/index.php?topic=36313.msg1361903#msg1361903

Edit: Added screenshot noting no dependence on polarization.
Title: Re: EM Drive Developments - related to space flight applications - Thread 2
Post by: Rodal on 04/30/2015 07:36 pm
And what about the experimental observation of non-reciprocity in room temperature and pressure nitrogen (largest portion of air) that was reported? That is also unbalanced momentum.
From my recollection of reading your prior links, those involved nonlinear polarization, hence not addressed by linear, isotropic Maxwell's equations.  One of the papers addressed a 2nd order term and another addressed a 4th order term in a nonlinear perturbation analysis.

Clearly, a nonlinear differential equation does not have a simple harmonic solution.

http://forum.nasaspaceflight.com/index.php?topic=36313.msg1361903#msg1361903

Yes, that also confirms what I stated: 

Quote
bilinear magneto-electric effects correspond to a coupling beyond the electric dipole approximation.

Bi-linearity is nonlinearity.  Many nonlinearities are modeled as piecewise linear, for example the elastic-plastic deformation of a solid, hysteresis in continuum mechanics, or hysteresis in magnetic materials, etc.

Also note: 

coupling beyond the electric dipole approximation

and

directional anisotropies

_____

Perturbation analyses are local (perturbation series) approximations to a nonlinearity, while piecewise-linear (bi-linear, and higher order) approximations are global approximations to a nonlinearity.

____

Concerning your focusing on "polar", yes it is an induced anisotropy.  What matters (concerning the validity of the linear isotropic Maxwell's equation) is the anisotropy (not whether it is initial anisotropy or induced anisotropy), Maxwell's isotropic equations do not take into account any induced anisotropy.

This happens also in continuous mechanics: as an imperfect analogy if you take an isotropic piece of certain metals and you stretch it to nonlinear permanent deformations, you will produce anisotropic alignment of the material, which will now have different properties in different directions.  If one wants to analyze the anisotropic deformation of a material that is initially isotropic, one cannot do it using isotropic elasticity based on just using elasticity equations with just Young's modulus and Poisson's ratio.


Title: Re: EM Drive Developments - related to space flight applications - Thread 2
Post by: Mulletron on 04/30/2015 08:05 pm
It is clear that we are operating under other than usual symmetry conditions and that Maxwell's equations as we were trained to use them do not apply.
Title: Re: EM Drive Developments - related to space flight applications - Thread 2
Post by: Rodal on 04/30/2015 08:13 pm
R.S.'s analysis does not take into account either initial or induced (directional) anisotropy, it does not take into account nonlinear anisotropic effects, or higher order effects.  It does not take into account coupling. 

It needs to take something "extra and relevant" into account beyond Maxwell's equations and special relativity to explain acceleration of closed microwave cavities.
Title: Re: EM Drive Developments - related to space flight applications - Thread 2
Post by: Rodal on 04/30/2015 08:21 pm
Rather than debate theory, here is a suggestion: why don't we discuss with Roger Shawyer solely the experiments?

The experiments are extremely interesting.  He states that NASA Eagleworks is behind his state of the art.  I would find it fascinating if Shawyer could inform to us the progress of the superconducting EM Drive with curved bases.  Anything he can provide on the experiments (including the Demonstrator Engine, the only EM Drive for which there is YouTube video showing its movement for anybody to see) would be very valuable.

He doesn't need to give any confidential information.  We just need more description of what he already publicized. For example, when he wrote that he measured thrust for the Demonstrator Engine in opposite directions: force direction in both the "thrust" and the "reaction" directions.  Could he please explain this?
Title: Re: EM Drive Developments - related to space flight applications - Thread 2
Post by: squid on 04/30/2015 08:55 pm
It is clear that we are operating under other than usual symmetry conditions and that Maxwell's equations as we were trained to use them do not apply.

What are you talking about? Maxwell's equations apply to EM radiation unequivocally. Especially in these low power regimes there is no chance of observing any perturbations due to higher order effects from field theory.

Look, I want to believe in a "reactionless" drive as much as the next SF junkie, but this does not pass the smell test. Consider:
e
0. The proposed explanations violate: conservation of energy, conservation of momentum, and the Lorentz transformations. These are some of the best-tested physical arguments we have today, and yet this "drive" violates them by huge orders of magnitude. Is it really plausible that 100+ years of experiments have failed to notice a comparatively huge effect? RF cavities are not a poorly understood system. I used to work with superconducting RF cavities for particle accelerators with Q > 1 million supporting fields of > 10 MV/m. I can guarantee you we would have noticed if power was disappearing into "thrust", or if the damn things were starting to levitate.

1. Even if we are pushing against the "quantum vacuum" this does not make sense, as any such vacuumo  must be charge neutral and so we would be pushing in opposite directions on electrons and positrons. Not to mention the accelerated positrons would smash into the surrounding cavity, producing copious, easily-observable gamma rays.

1a. Even ignoring this objection, to promote particles from "virtual" to real (as in Hawking radiation...) you have to provide the particles with their mass-energy. 511 keV per electron/positron. Does this make sense?

Assume 100% of the energy delivered to the cavity goes into making virtual particles real: 100 W / (mass of electron * c^2) = 1e15 electrons / s. Assume the particles are instantaneously acclerated to the speed of light (a pretty generous assumption). Then F = (1e15 electron/s)*(mass of electron)*(speed of light) = 2e-7 newtons. Much smaller than what we observe. The explanation does not hold up to scrutiny.

2. The "quantum vacuum"/Casimir effect should not be given more emphasis than is appropriate. It is a calculational tool. For example, one can explain the casimir effect solely through the van der Waals forces between two neighboring conductors, without handwaving explanations about virtual particles: http://physics.stackexchange.com/questions/11544/vander-waals-and-casimir-forces

3. The quantum vacuum publication referenced earlier is really, really bad. It starts with the Bohr formula for hydrogen (thus neglecting any fine structure, etc... effects), and takes a "radius" (which as we know from QM should not be interpreted literally...) to get a volume and from that some kind of density. This has no physical meaning. Then a function 1/r^4 is fitted to the values -- with no justification! This is then compared to the casimir force for cavities separated by this radius... and the values are  pretty far off. But it looks like a factor of 1/3! So some further handwaving about general relativity which also has a factor of 1/3 in one equation! Then there's a whole word salad about solving hydrogen atom wavefunctions with COMSOL [???], which is ridiculous since any undergraduate physics students can solve them with pen and paper...

This papers seems explicitly designed to use lots of fancy terminology and equations to look impressive to anyone with no background in physics, while saying nothing at all of substance. It does not even rise to the level of coherence.

In conclusion, this whole affair appears to be the work of someone who has convinced himself his theory is right and is on a fishing expedition for evidence that supports it. The experimental design is poor (camera pictures of LabView windows? unable to find an RF amplifier to deliver more than 100W of power? inability to measure forces that would be measurable in the 1800's?), the past 100+ years of physical experiments contradict the experiments, and there is no coherent underlying theory.

This is not science, it is cargo cult science.
Title: Re: EM Drive Developments - related to space flight applications - Thread 2
Post by: Dmytry on 04/30/2015 09:13 pm
It is clear that we are operating under other than usual symmetry conditions and that Maxwell's equations as we were trained to use them do not apply.

What are you talking about? Maxwell's equations apply to EM radiation unequivocally. Especially in these low power regimes there is no chance of observing any perturbations due to higher order effects from field theory.

Look, I want to believe in a "reactionless" drive as much as the next SF junkie, but this does not pass the smell test. Consider:
e
0. The proposed explanations violate: conservation of energy, conservation of momentum, and the Lorentz transformations. These are some of the best-tested physical arguments we have today, and yet this "drive" violates them by huge orders of magnitude. Is it really plausible that 100+ years of experiments have failed to notice a comparatively huge effect? RF cavities are not a poorly understood system. I used to work with superconducting RF cavities for particle accelerators with Q > 1 million supporting fields of > 10 MV/m. I can guarantee you we would have noticed if power was disappearing into "thrust", or if the damn things were starting to levitate.

1. Even if we are pushing against the "quantum vacuum" this does not make sense, as any such vacuumo  must be charge neutral and so we would be pushing in opposite directions on electrons and positrons. Not to mention the accelerated positrons would smash into the surrounding cavity, producing copious, easily-observable gamma rays.

1a. Even ignoring this objection, to promote particles from "virtual" to real (as in Hawking radiation...) you have to provide the particles with their mass-energy. 511 keV per electron/positron. Does this make sense?

Assume 100% of the energy delivered to the cavity goes into making virtual particles real: 100 W / (mass of electron * c^2) = 1e15 electrons / s. Assume the particles are instantaneously acclerated to the speed of light (a pretty generous assumption). Then F = (1e15 electron/s)*(mass of electron)*(speed of light) = 2e-7 newtons. Much smaller than what we observe. The explanation does not hold up to scrutiny.

2. The "quantum vacuum"/Casimir effect should not be given more emphasis than is appropriate. It is a calculational tool. For example, one can explain the casimir effect solely through the van der Waals forces between two neighboring conductors, without handwaving explanations about virtual particles: http://physics.stackexchange.com/questions/11544/vander-waals-and-casimir-forces

3. The quantum vacuum publication referenced earlier is really, really bad. It starts with the Bohr formula for hydrogen (thus neglecting any fine structure, etc... effects), and takes a "radius" (which as we know from QM should not be interpreted literally...) to get a volume and from that some kind of density. This has no physical meaning. Then a function 1/r^4 is fitted to the values -- with no justification! This is then compared to the casimir force for cavities separated by this radius... and the values are  pretty far off. But it looks like a factor of 1/3! So some further handwaving about general relativity which also has a factor of 1/3 in one equation! Then there's a whole word salad about solving hydrogen atom wavefunctions with COMSOL [???], which is ridiculous since any undergraduate physics students can solve them with pen and paper...

This papers seems explicitly designed to use lots of fancy terminology and equations to look impressive to anyone with no background in physics, while saying nothing at all of substance. It does not even rise to the level of coherence.

In conclusion, this whole affair appears to be the work of someone who has convinced himself his theory is right and is on a fishing expedition for evidence that supports it. The experimental design is poor (camera pictures of LabView windows? unable to find an RF amplifier to deliver more than 100W of power? inability to measure forces that would be measurable in the 1800's?), the past 100+ years of physical experiments contradict the experiments, and there is no coherent underlying theory.

This is not science, it is cargo cult science.
It'd be great also if there was a list of graphs organized by resonance mode (rather than an entirely data-free article). What I see so far is that in vacuum all responses are slower...
Title: Re: EM Drive Developments - related to space flight applications - Thread 2
Post by: Mulletron on 04/30/2015 09:18 pm
It is clear that we are operating under other than usual symmetry conditions and that Maxwell's equations as we were trained to use them do not apply.

What are you talking about? Maxwell's equations apply to EM radiation unequivocally. Especially in these low power regimes there is no chance of observing any perturbations due to higher order effects from field theory.

Look, I want to believe in a "reactionless" drive as much as the next SF junkie, but this does not pass the smell test. Consider:
e
0. The proposed explanations violate: conservation of energy, conservation of momentum, and the Lorentz transformations. These are some of the best-tested physical arguments we have today, and yet this "drive" violates them by huge orders of magnitude. Is it really plausible that 100+ years of experiments have failed to notice a comparatively huge effect? RF cavities are not a poorly understood system. I used to work with superconducting RF cavities for particle accelerators with Q > 1 million supporting fields of > 10 MV/m. I can guarantee you we would have noticed if power was disappearing into "thrust", or if the damn things were starting to levitate.

1. Even if we are pushing against the "quantum vacuum" this does not make sense, as any such vacuumo  must be charge neutral and so we would be pushing in opposite directions on electrons and positrons. Not to mention the accelerated positrons would smash into the surrounding cavity, producing copious, easily-observable gamma rays.

1a. Even ignoring this objection, to promote particles from "virtual" to real (as in Hawking radiation...) you have to provide the particles with their mass-energy. 511 keV per electron/positron. Does this make sense?

Assume 100% of the energy delivered to the cavity goes into making virtual particles real: 100 W / (mass of electron * c^2) = 1e15 electrons / s. Assume the particles are instantaneously acclerated to the speed of light (a pretty generous assumption). Then F = (1e15 electron/s)*(mass of electron)*(speed of light) = 2e-7 newtons. Much smaller than what we observe. The explanation does not hold up to scrutiny.

2. The "quantum vacuum"/Casimir effect should not be given more emphasis than is appropriate. It is a calculational tool. For example, one can explain the casimir effect solely through the van der Waals forces between two neighboring conductors, without handwaving explanations about virtual particles: http://physics.stackexchange.com/questions/11544/vander-waals-and-casimir-forces

3. The quantum vacuum publication referenced earlier is really, really bad. It starts with the Bohr formula for hydrogen (thus neglecting any fine structure, etc... effects), and takes a "radius" (which as we know from QM should not be interpreted literally...) to get a volume and from that some kind of density. This has no physical meaning. Then a function 1/r^4 is fitted to the values -- with no justification! This is then compared to the casimir force for cavities separated by this radius... and the values are  pretty far off. But it looks like a factor of 1/3! So some further handwaving about general relativity which also has a factor of 1/3 in one equation! Then there's a whole word salad about solving hydrogen atom wavefunctions with COMSOL [???], which is ridiculous since any undergraduate physics students can solve them with pen and paper...

This papers seems explicitly designed to use lots of fancy terminology and equations to look impressive to anyone with no background in physics, while saying nothing at all of substance. It does not even rise to the level of coherence.

In conclusion, this whole affair appears to be the work of someone who has convinced himself his theory is right and is on a fishing expedition for evidence that supports it. The experimental design is poor (camera pictures of LabView windows? unable to find an RF amplifier to deliver more than 100W of power? inability to measure forces that would be measurable in the 1800's?), the past 100+ years of physical experiments contradict the experiments, and there is no coherent underlying theory.

This is not science, it is cargo cult science.

I'm going to wallop you in the morning. :D Hope you know what you're attacking me for (what I've been advocating which isn't QVPT BTW) You might want to make sure you have your facts straight. This is your head start.
Title: Re: EM Drive Developments - related to space flight applications - Thread 2
Post by: Cinder on 04/30/2015 09:21 pm
Plenty of dissent in this topic's history.  Just stay polite and pertinent.
Title: Re: EM Drive Developments - related to space flight applications - Thread 2
Post by: sanman on 04/30/2015 09:29 pm
Article on Shawyer/EM/Cannae drive in NBF:

http://nextbigfuture.com/2015/04/emdrive-roger-shawyer-believes-midterm.html



Gee, this looks like one of those cryo-cooled overclocked PC rigs:

(http://3.bp.blogspot.com/-YKuK6EyuQL8/VUJzCb6W8NI/AAAAAAAA9co/spEUDkAO1QI/s1600/emdriveengine.jpg)

So does this mean that all PC-overclocking dreamer-nerds everywhere could create their own EM-drive at home?

Just like with Polywell or other small-scale attempts at fusion energy, perhaps a multitude of parallel efforts could yield more advances at a faster pace.


I still don't understand why major laboratories can't be enlisted to verify or disprove the phenomenon.
Title: Re: EM Drive Developments - related to space flight applications - Thread 2
Post by: squid on 04/30/2015 09:32 pm

I'm going to wallop you in the morning. :D Hope you know what you're attacking me for (what I've been advocating which isn't QVPT BTW) You might want to make sure you have your facts straight. This is your head start.

Please don't misconstrue my post as a personal attack. I was asking what you mean by "other than usual" symmetry conditions. Maxwell's equations operate regardless of the frame of reference we are in, so they are valid in all situations.

I have been lurking on this thread for a few weeks, so I feel I am somewhat conversant with who has what explanation. The bulk of my comment was on Harold White's "explanation" of the EM drive.

In any case, a copper cone moves a bit when we dump 100 W RF into it. This is not surprising. Claiming you have some totally unknown (macroscopic!!) force operating, when such forces have been searched for and ruled out to high precision (http://en.wikipedia.org/wiki/Fifth_force) is surprising. Claiming that this is proof of a reactionless drive, space-time warping, and quantum vacuum weirdness is just ridiculous.

Title: Re: EM Drive Developments - related to space flight applications - Thread 2
Post by: Stormbringer on 04/30/2015 09:39 pm
On the theory that the other thread was supposed to be the one that is kept pristine and this one is for (ahem) discussion I have decided to post this here in case it is pertinent to the discussion. My rationale is it has to do with the QV and particle weirdness therein.

Over the last several years there have been at least a couple of articles on emergent monopoles in condensed matter/ solid state physics. And apparently I missed one earlier article on non solid state/condensed matter Quantum physics (referenced in this article.) But I saw the one published today and it does not appear to be a forced or emergent species of monopole but a quantum plasma variant of yet another "sort of" monopole.

They do not say it is one of the fundamental monopole species from various theories but at the same time it is not a fake one made by pinning flux lines in solid state objects.

http://phys.org/news/2015-04-physicists-quantum-mechanical-monopoles.html

some of the statements in the article seem to pertain to the level of existence Dr White and some others are postulating are behind the EM drive and the QVPT.
Title: Re: EM Drive Developments - related to space flight applications - Thread 2
Post by: tchernik on 04/30/2015 09:41 pm
This is not science, it is cargo cult science.

If it can be replicated, it's not.

Besides I don't see anyone here arguing for accepting NASA Eagle Works', NWPU's and Roger Shawyer's results on faith alone.

We are well past the point of being able to dismiss this with a simple snarky remark and a good laugh. I want proof and data now, in order to believe any conclusive assertion about this.
Title: Re: EM Drive Developments - related to space flight applications - Thread 2
Post by: Dmytry on 04/30/2015 09:41 pm
I still don't understand why major laboratories can't be enlisted to verify or disprove the phenomenon.
Because it is obvious pseudoscience and is treated as such (unlike e.g. early cold fusion, which didn't look anywhere near as silly and resulted in many replication attempts worldwide).
Title: Re: EM Drive Developments - related to space flight applications - Thread 2
Post by: Stormbringer on 04/30/2015 09:51 pm
So pseudo-science consists of careful experiments, peer review, critiques and replication attempts and redesigning test articles to get higher Sigma confidence? Who knew!?

It may turn out that Dr White and others are mistaken. However that does not make it pseudo-science.
Title: Re: EM Drive Developments - related to space flight applications - Thread 2
Post by: tchernik on 04/30/2015 09:53 pm
I still don't understand why major laboratories can't be enlisted to verify or disprove the phenomenon.
Because it is obvious pseudoscience and is treated as such (unlike e.g. early cold fusion, which didn't look anywhere near as silly and resulted in many replication attempts worldwide).

Pseudoscience in what sense?

Is it based on falsifiable assertions or not? As far as I know, yes. Either it provides thrust or it doesn't. "Eppur si muove" at its best.

Can it be replicated? Sure, there's nothing magical with the setup. Any willing and able engineer could make one (and several around here are).

Does it work? We don't know for certain yet. There are hints of the answer being yes, but more replications are in order.

Should we ban and discourage any experiments on this obviously silly idea, only because it goes against our models and theories? Definitely not.
Title: Re: EM Drive Developments - related to space flight applications - Thread 2
Post by: RotoSequence on 04/30/2015 10:05 pm
I still don't understand why major laboratories can't be enlisted to verify or disprove the phenomenon.
Because it is obvious pseudoscience and is treated as such (unlike e.g. early cold fusion, which didn't look anywhere near as silly and resulted in many replication attempts worldwide).

Naked disdain on the grounds of established theory, without citation of related experiments or numerical analysis, is unhelpful and unproductive.
Title: Re: EM Drive Developments - related to space flight applications - Thread 2
Post by: CW on 04/30/2015 10:14 pm
I still don't understand why major laboratories can't be enlisted to verify or disprove the phenomenon.
Because it is obvious pseudoscience and is treated as such (unlike e.g. early cold fusion, which didn't look anywhere near as silly and resulted in many replication attempts worldwide).

Naked disdain on the grounds of established theory, without citation of related experiments or numerical analysis, is unhelpful and unproductive.

Also known as trolling.
Title: Re: EM Drive Developments - related to space flight applications - Thread 2
Post by: squid on 04/30/2015 10:14 pm
I still don't understand why major laboratories can't be enlisted to verify or disprove the phenomenon.
Because it is obvious pseudoscience and is treated as such (unlike e.g. early cold fusion, which didn't look anywhere near as silly and resulted in many replication attempts worldwide).

Pseudoscience in what sense?

Is it based on falsifiable assertions or not? As far as I know, yes. Either it provides thrust or it doesn't. "Eppur si muove" at its best.

Can it be replicated? Sure, there's nothing magical with the setup. Any willing and able engineer could make one (and several around here are).

Does it work? We don't know for certain yet. There are hints of the answer being yes, but more replications are in order.

Should we ban and discourage any experiments on this obviously silly idea, only because it goes against our models and theories? Definitely not.

Nobody is arguing for the ban or discouragement of these experiments. By all means, let's tinker. It's fun.

BUT: It's cargo cult science (my preferred term) in the sense that it is a simple, crude experiment that contradicts decades of far more well executed experiments. We go from a small displacement of a metal body when subjected to an RF drive to wild claims about spacetime distortion. It looks scientific from the outside, but does not withstand detailed scrutiny.

When they are running the RF cavities in the LHC at CERN (far larger, superconducting, far higher power), why don't they try to tear themselves off their mountings?

Why do microwave electronics (radars, cell phones, etc...) not behave differently as the earth rotates around the sun and moves through some hypothetical background of quantum vacuum?

The EM drive theories rely on coupling of electromagnetic fields to something else. Why has this coupling never been observed before? Why can QED be accurate to 20+ decimal places if energy is disappearing into the quantum vacuum?

Astrophysical processes create electromagnetic fields orders of magnitude larger than what could conceivably made in the laboratory. Why don't we see any interaction in these vastly larger fields, or any "spacetime bending"?

If you have a new physical theory which predicts some phenomenon, it should reduce to previously known physical theories, and be able to explain existing experiments and observations. If I believe that the EM drive is actually providing some nonclassical, unexplained thrust, I must throw out 100+ years of physics experiment and theory. I choose not to do that, which is why I do not believe further experimentation is warranted. Others may choose differently, but then they should ask themselves why they are so eager to disregard such a large body of established science.
Title: Re: EM Drive Developments - related to space flight applications - Thread 2
Post by: NovaSilisko on 04/30/2015 10:24 pm
So, an opinion here - I think this thread should be kept as the place where all the theorycrafting and experimental work is specifically discussed by (by and large) the people involved. The strong skepticism and debate over the practicality/realism of the technology and etc ought to be taken perhaps to the article thread, to keep this one cleaner and more usable/readable.
Title: Re: EM Drive Developments - related to space flight applications - Thread 2
Post by: virtualighter on 04/30/2015 10:28 pm
It doesn't have anything to do with quantum mechanics, it has to do with relativity.  So refutations based on quantum mechanics are irrelevant.

Read the paper.  The paper shows that there will be a force.  Yes, the content of the paper is hard to keep in your head, but it's in there. 

http://www.emdrive.com/theorypaper9-4.pdf

As I understand it, the speed of light, being constant no matter what, essentially creates a separate reference frame from what is going on inside the cavity.  The propulsive force is generated because of differences between the two reference frames.  Going out on a metaphoric limb, it's like the EM waves in the cavity get squeezed against the speed of light, and it doesn't budge, so the cavity has to.

But it doesn't matter how it is put into words.  The reality is not in the narrative metaphors.  The reality is in the physics equations, the math.  The paper uses well-established equations from physics, and provides sources. 

Yes, it's mind boggling.  However, it's not that complicated or mysterious in terms of the equations and their sources.   It's undergrad physics.

Read the paper, understand it, and try to refute what it says.

If you can't understand the paper, which is not easy unless you DO happen to have undergrad physics (which I can't fully claim) well, that's a separate problem, which perhaps should have its own thread.  I think there's a chance it could be chunked out into something intuitively graspable.
Title: Re: EM Drive Developments - related to space flight applications - Thread 2
Post by: KelvinZero on 04/30/2015 10:30 pm
So, an opinion here - I think this thread should be kept as the place where all the theorycrafting and experimental work is specifically discussed by (by and large) the people involved. The strong skepticism and debate over the practicality/realism of the technology and etc ought to be taken perhaps to the article thread, to keep this one cleaner and more usable/readable.
It would be a very good idea for the moderators to set up some sort of rules like that.

Additionally, I created a thread at one point that was specifically to answer often repeated questions such as "Why constant thrust for constant power will eventually return more energy than you put in". (And I learnt something too, this has an exception: the light drive :) )

Ah here it is.
http://forum.nasaspaceflight.com/index.php?topic=35332
The plan was for people to put essay-like answers in here and then reference that particular post. Thus the information never need get lost even if the thread sprawls. Someone can always re-edit their original essay answer if discussion raises an issue.
Title: Re: EM Drive Developments - related to space flight applications - Thread 2
Post by: RonM on 04/30/2015 10:44 pm
If you have a new physical theory which predicts some phenomenon, it should reduce to previously known physical theories, and be able to explain existing experiments and observations. If I believe that the EM drive is actually providing some nonclassical, unexplained thrust, I must throw out 100+ years of physics experiment and theory. I choose not to do that, which is why I do not believe further experimentation is warranted. Others may choose differently, but then they should ask themselves why they are so eager to disregard such a large body of established science.

Forget all these new theories for the moment. Concentrate on the experiment.

If all there is to these experiments is experimental error, then we can forget the whole thing.

If there is anomalous thrust from these experiments, then theorists can try to figure out how it works.

Some very intelligent and highly trained people here have been trying to show these results are experimental error. They are still talking about it because they can't explain away the results yet.
Title: Re: EM Drive Developments - related to space flight applications - Thread 2
Post by: zen-in on 04/30/2015 10:55 pm
It is clear that we are operating under other than usual symmetry conditions and that Maxwell's equations as we were trained to use them do not apply.

In conclusion, this whole affair appears to be the work of someone who has convinced himself his theory is right and is on a fishing expedition for evidence that supports it. The experimental design is poor (camera pictures of LabView windows? unable to find an RF amplifier to deliver more than 100W of power? inability to measure forces that would be measurable in the 1800's?), the past 100+ years of physical experiments contradict the experiments, and there is no coherent underlying theory.

This is not science, it is cargo cult science.

Calling it cargo cult science is a bit too harsh.   I reserve that description for individuals who have Bizarro World concepts of science, physics, etc.   This is not the case here.   What is missing are controlled experiments that would validate any conventional explanations for the anomalous thrust.   These researchers are over-reaching.  They are too eager to show just positive results.     I am 99% certain there is nothing here.   To explain the 1%:  There remains a possibility that Shawyer has discovered something and the Eagleworks team are just not able to replicate what he has done.   To their credit the Eagleworks team have provided a lot of experimental results in the last year.   Shawyer has not done this.   What I have seen in both the Eaglework's thrust graphs and the  few of Shawyer's that I have seen is a lack of consistency in the step response of these devices.   The calibration force (a capacitor on the Eagleworks setup) produces a driving function that can be considered to be two step functions.   Anything else that drives this torque pendulum with a step function should cause the same system response.   That is fundamental physics any mechanical or electrical engineer would agree with.   But we don't see this happening when the RF is switched on.   We also see an effect that looks like there is stored momentum.   How is this happening?  There is something else causing the "anomalous thrust".   Until the effects of that other cause are nulled out any actual warp-drive or em-drive thrust has not been observed because a priori it must have the same system response as the calibration drive.

Title: Re: EM Drive Developments - related to space flight applications - Thread 2
Post by: Zach Swena on 04/30/2015 11:08 pm
If you have a new physical theory which predicts some phenomenon, it should reduce to previously known physical theories, and be able to explain existing experiments and observations. If I believe that the EM drive is actually providing some nonclassical, unexplained thrust, I must throw out 100+ years of physics experiment and theory. I choose not to do that, which is why I do not believe further experimentation is warranted. Others may choose differently, but then they should ask themselves why they are so eager to disregard such a large body of established science.

Forget all these new theories for the moment. Concentrate on the experiment.

If all there is to these experiments is experimental error, then we can forget the whole thing.

If there is anomalous thrust from these experiments, then theorists can try to figure out how it works.

Some very intelligent and highly trained people here have been trying to show these results are experimental error. They are still talking about it because they can't explain away the results yet.

As an engineer, I am extremely skeptical of law breaking stuff like this, but the statement above is where it is at.  You don't have to throw out all of the old body of science when new as of yet unexplained phenomenon like this come up.  The proper role of science now is to figure out if these experiments are flawed in some way or if this points out one of the many flaws in our understanding of physics today. 

If you view scientific knowledge as a static group of absolutes, then your understanding of science is extremely flawed.  It is healthy to have faith in the existence of an absolute reality, but we shouldn't be so vain to think that we have a full understanding of such that can't be subject to revision when we see more of the picture.

Something like this, if true will most likely just result in a few additional coefficients or terms in our equations.  If it does pan out, that would be awesome.  I suspect it is just a side effect of not controlling for enough variables in the experiment.
Title: Re: EM Drive Developments - related to space flight applications - Thread 2
Post by: ragingrei on 04/30/2015 11:15 pm
Not a physicist, but is it really that unusual or unprofessional for a researcher to tinker with unexplained phenomena?

They haven't even made any official claims yet about why they think they're seeing what they are; they're just sharing screenshots of their observations and ideas about them on a forum the way they might with colleagues.

If they publish a paper and it turns out to be complete garbage, then sure, but right now it seems like they're still at the scratching-their-heads phase as much as anyone else is -- albeit scratching with a little more vigor and excitement than many.
Title: Re: EM Drive Developments - related to space flight applications - Thread 2
Post by: Star One on 04/30/2015 11:15 pm
It is clear that we are operating under other than usual symmetry conditions and that Maxwell's equations as we were trained to use them do not apply.

In conclusion, this whole affair appears to be the work of someone who has convinced himself his theory is right and is on a fishing expedition for evidence that supports it. The experimental design is poor (camera pictures of LabView windows? unable to find an RF amplifier to deliver more than 100W of power? inability to measure forces that would be measurable in the 1800's?), the past 100+ years of physical experiments contradict the experiments, and there is no coherent underlying theory.

This is not science, it is cargo cult science.

Calling it cargo cult science is a bit too harsh.   I reserve that description for individuals who have Bizarro World concepts of science, physics, etc.   This is not the case here.   What is missing are controlled experiments that would validate any conventional explanations for the anomalous thrust.   These researchers are over-reaching.  They are too eager to show just positive results.     I am 99% certain there is nothing here.   To explain the 1%:  There remains a possibility that Shawyer has discovered something and the Eagleworks team are just not able to replicate what he has done.   To their credit the Eagleworks team have provided a lot of experimental results in the last year.   Shawyer has not done this.   What I have seen in both the Eaglework's thrust graphs and the  few of Shawyer's that I have seen is a lack of consistency in the step response of these devices.   The calibration force (a capacitor on the Eagleworks setup) produces a driving function that can be considered to be two step functions.   Anything else that drives this torque pendulum with a step function should cause the same system response.   That is fundamental physics any mechanical or electrical engineer would agree with.   But we don't see this happening when the RF is switched on.   We also see an effect that looks like there is stored momentum.   How is this happening?  There is something else causing the "anomalous thrust".   Until the effects of that other cause are nulled out any actual warp-drive or em-drive thrust has not been observed because a priori it must have the same system response as the calibration drive.

Well then let's hear your suggestions for how these concerns can be dealt with then? After all that's what this thread is all about.:)
Title: Re: EM Drive Developments - related to space flight applications - Thread 2
Post by: FutureStormtrooper on 04/30/2015 11:32 pm
While I do not share Squid's absolute certainty that all the experimental data is a fluke, and that the theories are bunk, there is one question of his that I would be very interested in an explanation being made for. Namely, how is that this phenomenon hasn't been discovered and documented in centuries past?

What is so unique about the experimental setups of Eagleworks, Yang, and Shawyer compared to the presumably millions of other microwave cavities which have been experimented on in the past? Is it actual physical differences in these setups, is it that the forces produced are so small that nobody noticed beforehand, is it something else?

I think that this is a very good question, and I'd be interested in the enlightened answers of the denizens of this thread   :)
Title: Re: EM Drive Developments - related to space flight applications - Thread 2
Post by: zen-in on 04/30/2015 11:34 pm
It is clear that we are operating under other than usual symmetry conditions and that Maxwell's equations as we were trained to use them do not apply.

In conclusion, this whole affair appears to be the work of someone who has convinced himself his theory is right and is on a fishing expedition for evidence that supports it. The experimental design is poor (camera pictures of LabView windows? unable to find an RF amplifier to deliver more than 100W of power? inability to measure forces that would be measurable in the 1800's?), the past 100+ years of physical experiments contradict the experiments, and there is no coherent underlying theory.

This is not science, it is cargo cult science.

Calling it cargo cult science is a bit too harsh.   I reserve that description for individuals who have Bizarro World concepts of science, physics, etc.   This is not the case here.   What is missing are controlled experiments that would validate any conventional explanations for the anomalous thrust.   These researchers are over-reaching.  They are too eager to show just positive results.     I am 99% certain there is nothing here.   To explain the 1%:  There remains a possibility that Shawyer has discovered something and the Eagleworks team are just not able to replicate what he has done.   To their credit the Eagleworks team have provided a lot of experimental results in the last year.   Shawyer has not done this.   What I have seen in both the Eaglework's thrust graphs and the  few of Shawyer's that I have seen is a lack of consistency in the step response of these devices.   The calibration force (a capacitor on the Eagleworks setup) produces a driving function that can be considered to be two step functions.   Anything else that drives this torque pendulum with a step function should cause the same system response.   That is fundamental physics any mechanical or electrical engineer would agree with.   But we don't see this happening when the RF is switched on.   We also see an effect that looks like there is stored momentum.   How is this happening?  There is something else causing the "anomalous thrust".   Until the effects of that other cause are nulled out any actual warp-drive or em-drive thrust has not been observed because a priori it must have the same system response as the calibration drive.

Well then let's hear your suggestions for how these concerns can be dealt with then? After all that's what this thread is all about.:)

My first suggestion was to replace the RF feed with a suitably sized power resistor.  What happens when the same amount of power is delivered to the interior of the cavity, but with no RF?   Use the same test protocol (is there a protocol?).  Move the power resistor around.  Clamp it to different parts of the cavity (magnets on the outside will do this) and plot the response to a thermal drive with the same power level.   I made this suggestion over 6 months ago.

Of course this type of test is difficult to perform because anytime something is added or removed from the em-drive the balance has to be adjusted again.   And there has been a lot of discussion about how this torque pendulum works and whether it is contributing to alleged erroneous results.    So another way to do this test is to use focused  heat lamps or direct the RF energy at the outside of the cone using feedhorns.
Title: Re: EM Drive Developments - related to space flight applications - Thread 2
Post by: MADscientist on 04/30/2015 11:57 pm
Just (mostly) a lurker here, and by far NOT anything other than an "armchair scientist" (lol), but it would seem to me that if you are having trouble telling whether there really is "thrust" from this thing or not, presumably because that thrust is either so small it is hard to detect reliably, or because it is drowned out by "background noise" or whatever, then perhaps you need to "get closer" to it to "see" it better. In my line of work I have to "see" very small differences in "things". I do this using magnification. Now, magnification merely allows me to get my eyeball "closer" to that which I am looking at. Another way I  COULD accomplish the same thing would to be to make the "thing" larger (which is what magnification sort of does). Obviously, you can't get closer to this phenomena you are trying to examine and verify, but you COULD make it larger. Why not increase the effect's size? Pump MORE initial energy into it? Stop pussyfooting around with it and take this baby past Warp11!

Personally, I am MORE interested in the reporting about the experiment(s) having suggestively produced a (manipulable?) space-warp bubble. I think that holds more promise.
Title: Re: EM Drive Developments - related to space flight applications - Thread 2
Post by: Nemo on 05/01/2015 12:13 am
If new non-classical physics models are required to explain the EM Drive thrust as mentioned above by Squid we would have to apply any new physics models to astrophysical observations. Quoting Squids question
Quote
Why don't we see any interaction in these vastly larger fields, or any "spacetime bending"?
. Assuming that the EM Drive thrust is eventually confirmed could it be that this same effect is responsible for Dark Energy and the expanding universe?
Title: Re: EM Drive Developments - related to space flight applications - Thread 2
Post by: virtualighter on 05/01/2015 12:13 am
Well then let's hear your suggestions for how these concerns can be dealt with then? After all that's what this thread is all about.:)

See my post 4/30 10:43 PM by virtualighter

All squid's concerns are based on the assumption that the effect is via quantum mechanics.  This paper explains that it's not QM, it's relativity.  Refute this paper, then there will be something to discuss. 

http://www.emdrive.com/theorypaper9-4.pdf


Title: Re: EM Drive Developments - related to space flight applications - Thread 2
Post by: JasonAW3 on 05/01/2015 12:22 am

Well then let's hear your suggestions for how these concerns can be dealt with then? After all that's what this thread is all about.:)

Well, for one thing, it might be an idea to take a less cynical approach to this situation, and go back and reread the the data in the threads concerning the EM drive.  The results have been reproduced numerous times in the UK and China. Variations of the output compared to input seems related to experiment set up, but all have had similar results, all of them producing some measure of detectable thrust.  By varying the input voltage and frequency of the RF feed, they have varied the amount of thrust.

Something IS happening, something that is reproducible with the same setups.  There had been no attempts to withhold data, if anything, we've been overwhelmed with data and math.

Yes,  there are those who have their own theories as to what is happening, and for all we know, all of those theories could be wrong.  We don't know yet.  Thrust IS being produced, and has done so both in atmosphere and in vacume, so air, as a factor, has been effectively eliminated.

Science requires both an open mind and a degree of skepticism.  Science is experimentation, hypothisizing, theorizing and observation.  Not in that order, but let's be honest. Science is messy and we don't know all there is to know.

More than once, experiments have produced results completely outside of accepted theories.  Sometimes it's because the experiment was set up wrong, sometimes its because of observer bias, and sometimes we discover something new that we never expected.  The core here is repeatability of the results by other experimenters using the same sort of set ups.

This has been accomplished.  Thrust has been produced by all of these set ups.  How its being produced? That seems to be the major sticking point.  Nobody's really sure.  That's why we do science to these experiments to figure out why something does what it does.  Even when starts out with a theory that turns out to be wrong, sometimes you get legitmate results for completely different reasons.  This may be what is happening.

Me,  I'm excited about the results so far, as it once again proves a smart man knows a lot of things, but a wise man knows how much he really doesn't know.
Title: Re: EM Drive Developments - related to space flight applications - Thread 2
Post by: lasoi on 05/01/2015 12:33 am
A lot of the skepticism of the results thus far has been centered around the support setups for the drive.  Seems like a lot of time and energy could be saved if someone could convince Mr. Musk to tote one up into orbit and turn it on.
Title: Re: EM Drive Developments - related to space flight applications - Thread 2
Post by: JasonAW3 on 05/01/2015 12:40 am
A lot of the skepticism of the results thus far has been centered around the support setups of the drive.  Seems like a lot of time and energy could be saved if someone could convince Mr. Musk to tote one up into orbit and turn it on.

I suspect that that all be one of the next experiments.  I also suspect that performance may be far better than expected the further it is from Earth.
Title: Re: EM Drive Developments - related to space flight applications - Thread 2
Post by: ragingrei on 05/01/2015 01:00 am
A lot of the skepticism of the results thus far has been centered around the support setups of the drive.  Seems like a lot of time and energy could be saved if someone could convince Mr. Musk to tote one up into orbit and turn it on.

I suspect that that all be one of the next experiments.  I also suspect that performance may be far better than expected the further it is from Earth.

Could you explain for a layman why they couldn't simply test it on the ground with the power cranked up or with multiple drives in concert, in order to drown out any other effects? It would probably be less expensive than launching a satellite, wouldn't it?
Title: Re: EM Drive Developments - related to space flight applications - Thread 2
Post by: LasJayhawk on 05/01/2015 01:08 am
The "Edison Effect" predates the discovery of the electron by 15 years. Manipulating something that you didn't know existed or wasn't fully understood would appear to be doable.

Does Eagleworks have a motto? If not may I suggest something Robert Heinlein once wrote:
"Always listen to experts. They'll tell you what can't be done, and why. Then do it."
Title: Re: EM Drive Developments - related to space flight applications - Thread 2
Post by: JasonAW3 on 05/01/2015 01:20 am
There's a lot more experimentation that needs to be done before sending something like this into space.  However: it's been suggested that this could be put together in a cubs at format, although I am a bit skeptical about that, as it would still require a significant amount mof test equipment to observewhat happens in orbit.
Title: Re: EM Drive Developments - related to space flight applications - Thread 2
Post by: JasonAW3 on 05/01/2015 01:26 am
The "Edison Effect" predates the discovery of the electron by 15 years. Manipulating something that you didn't know existed or wasn't fully understood would appear to be doable.

Does Eagleworks have a motto? If not may I suggest something Robert Heinlein once wrote:
"Always listen to experts. They'll tell you what can't be done, and why. Then do it."

Always a good quote.  This whole situation kind of reminds me of "Waldo and Magic, Inc."

There was another author who'd written a story about a student who'd built a cold fusion bomb.  He did this as the result of a question in his college physics class.  "Explain why cold fusion is impossible."  Sometimes the answer is all in how you ask the quesdtion.
Title: Re: EM Drive Developments - related to space flight applications - Thread 2
Post by: laika on 05/01/2015 01:53 am
I haven't read this entire thread, maybe it has already been covered. I was disappointed by the complexity and even crudity of the Chinese test setup, and also the poor spectral output of their microwave generator, all of this tending to add noise to their results.
I would be interested in the return loss of the cavity, and what load it presents to the microwave generator during acceleration. Presumably the microwave power is turned into kinetic energy in the device, so what effect does loaded Q have on thrust generation in the device?
Title: Re: EM Drive Developments - related to space flight applications - Thread 2
Post by: squid on 05/01/2015 02:18 am
It doesn't have anything to do with quantum mechanics, it has to do with relativity.  So refutations based on quantum mechanics are irrelevant.

Read the paper.  The paper shows that there will be a force.  Yes, the content of the paper is hard to keep in your head, but it's in there. 

http://www.emdrive.com/theorypaper9-4.pdf

As I understand it, the speed of light, being constant no matter what, essentially creates a separate reference frame from what is going on inside the cavity.  The propulsive force is generated because of differences between the two reference frames.  Going out on a metaphoric limb, it's like the EM waves in the cavity get squeezed against the speed of light, and it doesn't budge, so the cavity has to.

But it doesn't matter how it is put into words.  The reality is not in the narrative metaphors.  The reality is in the physics equations, the math.  The paper uses well-established equations from physics, and provides sources. 


First off, maybe I was a little harsh in calling in cargo cult science. So sorry about that. I will suspend value judgements and concentrate on the scientific aspect.

 I don't dispute that something is moving, what I dispute is that there are any unknown physics occurring. Experimentation without some kind of underlying theory doesn't tell us much... we need theoretical guidance to explain measurements, otherwise they do not fit in a conceptual framework. Classical physics provides us with such a conceptual framework.

I have read the paper you cite, and I find it lacking. Here is an easy refutation. Equation (7) only considers the reaction on the two end plates of the resonator, and calculates the force based on the difference between the two. But of course, there will be a reaction on the cylindrical wall of the cavity, which will exactly cancel out this extra force.

Here is a general proof that, with the equations of classical electrodynamics there can be no thrust on a cavity of ANY SHAPE. Let me remind you that Maxwell's equations are perfectly Lorentz invariant, and so obey the lays of Special Relativity:

Let us assume a source-free cavity with perfectly conducting walls. The fields in the cavity will be harmonic as a function of time, as required by solutions to the Helmholtz equation (see Rodal's excellent mathematical analysis of the conical cavity upthread for an example).

The rate of change in time of momentum (density) of the system (mechanical components and EM fields) is given by:

dp/dt = div T, where T is the Maxwell stress-energy tensor.

div T = epsilon0 ( E x curl E + c2 B x curl B - (div E)E - c2(div E)E )

Per Maxwell's equations and the absence of free charges, div E = 0, div B = 0, and using the equations for the curl of the magnetic and electric fields,

div T = - epsilon0 d( E x B)/dt = -(1/c^2)dS/dt, where S in the poyting vector.

Integrating this from t to t+2pi/omega (ie one frequency cycle), we get -(1/c^2) S(t+2pi/omega) - S(t). Since the fields are harmonic, this equals 0. There is no change in momentum on the cavity, and no net force.

Again, Rodal has calculated all this explicitly for the case of a conical cavity.

To be clear, using the equations of classical physics (including Special Relativity through Maxwell's equations), there can be no force on a closed cavity of ANY shape
Title: Re: EM Drive Developments - related to space flight applications - Thread 2
Post by: squid on 05/01/2015 02:28 am
Furthermore, I think there has been some confusion from where I am coming from. I am merely trying to point out that new experiments must be consistent with past experiments when they overlap. No one has addressed my main concern:

Why have we not seen this effect before?

Please try to understand:

IF the EM drive is working
THEN the EM drive must be coupling electromagnetic energy to something

Then why has this coupling never been observed before? This is not a small effect. There are many many test cavities that operate in particle accelerators, as antenna feedhorns, as resonators in oscillators... if this effect were real, it would affect all of these. Why has this not been the case?

We are not working in a realm of physics that is out there... not subatomic particles, not astronomical observations of dark matter/dark energy, not trying to build a fusion device or a quantum computer. This is the kind of physics we need to know 100% to be able to design microwave antennas for cell phone towers.
Title: Re: EM Drive Developments - related to space flight applications - Thread 2
Post by: RotoSequence on 05/01/2015 02:39 am
Furthermore, I think there has been some confusion from where I am coming from. I am merely trying to point out that new experiments must be consistent with past experiments when they overlap. No one has addressed my main concern:

Why have we not seen this effect before?

Please try to understand:

IF the EM drive is working
THEN the EM drive must be coupling electromagnetic energy to something

Then why has this coupling never been observed before? This is not a small effect. There are many many test cavities that operate in particle accelerators, as antenna feedhorns, as resonators in oscillators... if this effect were real, it would affect all of these. Why has this not been the case?

We are not working in a realm of physics that is out there... not subatomic particles, not astronomical observations of dark matter/dark energy, not trying to build a fusion device or a quantum computer. This is the kind of physics we need to know 100% to be able to design microwave antennas for cell phone towers.

I'm going to preface this by admitting straight off the bat that I don't actually know why it wouldn't have been noticed before. However, the functionality of these EM drives is dependent on resonance, which requires the broadcast energy to be absorbed, or at the very least contained, by the cavity. For the purposes of transmission, resonating cavities are a characteristic of inefficiency and loss to design against, no?
Title: Re: EM Drive Developments - related to space flight applications - Thread 2
Post by: Hexadecibel on 05/01/2015 02:48 am
I'm not a engineer, or an academic,  but like those new users before me I too possess an armchair and access to basic physics books. This qualifies me as well to come here and make snarky remarks and say it can't be done.


Checkmate scientists.


But seriously. No one here is touting any sort of discovery. Science is happening, and if it can not be reproduced than it will sort itself out. You would think professionals of such caliber would have better things to do than stoop to trolling.
Title: Re: EM Drive Developments - related to space flight applications - Thread 2
Post by: JordanLeDoux on 05/01/2015 02:58 am
If you have a new physical theory which predicts some phenomenon, it should reduce to previously known physical theories, and be able to explain existing experiments and observations. If I believe that the EM drive is actually providing some nonclassical, unexplained thrust, I must throw out 100+ years of physics experiment and theory. I choose not to do that, which is why I do not believe further experimentation is warranted. Others may choose differently, but then they should ask themselves why they are so eager to disregard such a large body of established science.

I think Shawyer is completely wrong, but the device might very well produce thrust.

This: http://physicsfromtheedge.blogspot.co.uk/2014/01/mihsc-101.html

Is a theory of inertia that predicts the effects of dark matter and dark energy, and one of its only testable claims with today's technology would be something like the EmDrive.
Title: Re: EM Drive Developments - related to space flight applications - Thread 2
Post by: squid on 05/01/2015 03:01 am

I'm going to preface this by admitting straight off the bat that I don't actually know why it wouldn't have been noticed before. However, the functionality of these EM drives is dependent on resonance, which requires the broadcast energy to be absorbed, or at the very least contained, by the cavity. For the purposes of transmission, resonating cavities are a characteristic of inefficiency and loss to design against, no?

For transmission, sure, but antennas and the like are by definition resonant structures. Most microwave sources include a resonant cavity. Waveguides are resonant structures. Most concerning for the present work are superconducting RF cavities in particle accelerators (see the attached picture). I have personally worked with cavities similar to those in the picture. They are superconducting at 4K being made out of niobium, and typically support >10 MV/m of electric field, with quality factors of several 10's of million. Any loss of energy to an external medium would have been readily apparent, and we would not be able to run the LHC if there was some unknown effect affecting these sorts of cavities. Not to mention... those walls are actually fairly thin! If we believe the EM drive thrust claims... they would have buckled under the strain and torn from their mountings.

Please believe me, I am not trolling. I am a scientist who has been in love with space and NASA since my first visit to KSC as a kid some 20 years ago. I got to see the shuttle land! It was amazing! I am seriously concerned that these "results" will damage the credibility of the organization and making it even harder for people who are legitimately trying to advance science to secure funding [which is damn hard already...].
Title: Re: EM Drive Developments - related to space flight applications - Thread 2
Post by: sanman on 05/01/2015 03:03 am
So what are the leading contenders among established physical phenomena which could account for some of the observations/results from the experimental apparatus?

Could these results be due to a mere photon rocket - ie. due to mere photon emissions?
Could they be due to the EM fields interacting with the rest of the apparatus in ways that would throw off the measurements?

Could the results be due to some mechanical effect of electric current passing through wires, etc?

List the possible legitimate effects that could be contributing to these anomalous results, and then figure out ways to modify the rig to eliminate or otherwise correct for them.
Title: Re: EM Drive Developments - related to space flight applications - Thread 2
Post by: squid on 05/01/2015 03:16 am
So what are the leading contenders among established physical phenomena which could account for some of the observations/results from the experimental apparatus?

Could these results be due to a mere photon rocket - ie. due to mere photon emissions?
Could they be due to the EM fields interacting with the rest of the apparatus in ways that would throw off the measurements?

Could the results be due to some mechanical effect of electric current passing through wires, etc?

List the possible legitimate effects that could be contributing to these anomalous results, and then figure out ways to modify the rig to eliminate or otherwise correct for them.

In my personal opinion, in order of likleyhood

1. Buckling of the structure due to thermal or electromagnetic stresses
2. Lorentz forces from the current being carried to the device
3. Noise in electronics (there should be an error budget, and steps should be taken to minimize crosstalk, which is HIGHLY NONTRIVIAL when dealing with large RF powers). Signal to noise of (eyeballing) ~10 is pretty bad for such a huge claim.
4. Magnetic interaction between cavity and vacuum chamber wall

How to eliminate almost all of these (and for the life of me I can't understand why this hasn't been done, as it's pretty obvious) -- and I should credit Dymytry for the suggestion upthread:

1. Place the ENTIRE apparatus in a thermally shielded, electrically shielded hermetic enclosure. Doesn't need to be a vacuum, doesn't need to be anything fancy. A few hours in a machine shop with some aluminum plate is all you need for this. Mu-metal (to screen magnetic fields) would be best, though, with perhaps some polystyrene insulation on the outside. This would include a battery powered RF amplifier. Doesn't need to be anything fancy -- switch it on with a timer after you seal up, for example.

2. Hang the entire apparatus on a Cavendish-style torsion balance.

IF it moves:

3. Replicate the experiment with a 50 ohm load instead of a cavity.

IF it moves:

4. Replicate the experiment with cavities made of different metal, or filled with dielectric... this should reduce the e-field (lower the Q) and show less thrust.

Then I will be much more convinced that something interesting is going on.
Title: Re: EM Drive Developments - related to space flight applications - Thread 2
Post by: WarpTech on 05/01/2015 03:44 am
Go back and read the entire thread, you'll find they already did most of what you suggested. Their Rig has been tested and has not been falsified.

There is another explanation that has been around for many decades, it is that variations in the gravitational field are indistinguishable from variations in the refractive index of the vacuum. When you refer to the Lorentz invariance of Maxwell's equations, this does not hold in a vacuum where epsilon0 and mu0 are variables. In General Relativity we refer to the metric components, g^uv. These can and are interpreted as components of a variable refractive index, in the Polarizable Vacuum Model. Primarily refined by Hal Puthoff, a few papers of my own and many others have contributed to it. There are many, many papers on this available.

In this case, the interior energy density is not symmetrical, so the refractive index has a gradient. It was said in a previous post that the speed of light inside is different than it is outside. This is the correct interpretation, however it must include a gradient in the refractive index, as it passes through the structure itself, to cause motion. The gradient in the refractive index "is" a gravitational field. That is what the warp drive requires.

In a separate experiment, they may have shown that the speed of light inside the chamber varies. The amount is varies would only be noticed "IF" you were looking for it. Most resonant systems are "tuned" to eliminate such affects in manufacturing. In such a small cavity, I'm not confident it can be measured.


 

Title: Re: EM Drive Developments - related to space flight applications - Thread 2
Post by: cuddihy on 05/01/2015 04:01 am
No, no one has enclosed the entire apparatus and power suppply in a faraday cage and checked that...but that's not a trivial test to put together, either,  even if it should be easier than a vacuum rig.

Maybe you do that with additional funding if the new magnetron setup works?
Title: Re: EM Drive Developments - related to space flight applications - Thread 2
Post by: Hexadecibel on 05/01/2015 04:27 am
The mere fact that this saga has continued for 15 years without a definitive experiment such as suggested by squid leads one to smell a rat! Could it all just be due to poor experimental method?

Granted. But perhaps maybe its been dragged on for 15 years because no one would take it seriously. Consider how sad it would be if that were the case. This is certainly the first I've heard of it.

Eagleworks isn't full of crackpots. If they discover it was experimental error then that will be the end of it. Hand waving this away because you do not agree with their observations wont kill it, but the science they're doing now will.
Title: Re: EM Drive Developments - related to space flight applications - Thread 2
Post by: George Turner on 05/01/2015 04:30 am

How to eliminate almost all of these (and for the life of me I can't understand why this hasn't been done, as it's pretty obvious) -- and I should credit Dymytry for the suggestion upthread:

1. Place the ENTIRE apparatus in a thermally shielded, electrically shielded hermetic enclosure. Doesn't need to be a vacuum, doesn't need to be anything fancy. A few hours in a machine shop with some aluminum plate is all you need for this. Mu-metal (to screen magnetic fields) would be best, though, with perhaps some polystyrene insulation on the outside. This would include a battery powered RF amplifier. Doesn't need to be anything fancy -- switch it on with a timer after you seal up, for example.

2. Hang the entire apparatus on a Cavendish-style torsion balance.

IF it moves:

3. Replicate the experiment with a 50 ohm load instead of a cavity.

IF it moves:

4. Replicate the experiment with cavities made of different metal, or filled with dielectric... this should reduce the e-field (lower the Q) and show less thrust.

Then I will be much more convinced that something interesting is going on.

Many months ago I noted that if electrons are flowing, it's almost impossible not to accidentally build an electric motor or actuator, however weak, unless everything in the circuit is bolted or glued down.  Also, almost any material with a band gap can form a crappy little diode and create a DC current and corresponding magnetic field.  Going back to Newton, if the unit is being pushed, something else is getting pushed, too.

So imagine the test unit is the rotor, and the goal is to make the stator reveal itself.  My suggestion was to take every bit of test equipment in the room, from computers, cables, tables, chairs, and the Faraday cage, and suspend them all from the ceiling with strings, so that their pendular period of oscillation is some interval T.  Then cycle the test unit on and off with period T for an hour and see which pieces of equipment are swinging.  Those would be the stators.
Title: Re: EM Drive Developments - related to space flight applications - Thread 2
Post by: matthewpapa on 05/01/2015 04:57 am
Go back and read the entire thread, you'll find they already did most of what you suggested. Their Rig has been tested and has not been falsified.

There is another explanation that has been around for many decades, it is that variations in the gravitational field are indistinguishable from variations in the refractive index of the vacuum. When you refer to the Lorentz invariance of Maxwell's equations, this does not hold in a vacuum where epsilon0 and mu0 are variables. In General Relativity we refer to the metric components, g^uv. These can and are interpreted as components of a variable refractive index, in the Polarizable Vacuum Model. Primarily refined by Hal Puthoff, a few papers of my own and many others have contributed to it. There are many, many papers on this available.

In this case, the interior energy density is not symmetrical, so the refractive index has a gradient. It was said in a previous post that the speed of light inside is different than it is outside. This is the correct interpretation, however it must include a gradient in the refractive index, as it passes through the structure itself, to cause motion. The gradient in the refractive index "is" a gravitational field. That is what the warp drive requires.

In a separate experiment, they may have shown that the speed of light inside the chamber varies. The amount is varies would only be noticed "IF" you were looking for it. Most resonant systems are "tuned" to eliminate such affects in manufacturing. In such a small cavity, I'm not confident it can be measured.

Yes. Go back and read the thread. They already subbed the test article for a power resistor.

This was a very productive thread. Now full of trolls like the last one.
If your not going to provide constructive discussion leave. Many of us have been following this thread for months now and and not had any issues.

As others have said if this is fake no big deal. But why not give the experiments a chance to prove itself? I've never understood how scientists can be so petty in shooting someone down who makes a claim they think is impossible.

Now back to the applied spaceflight applications for emdrive before this thread is locked like the last one...
Title: Re: EM Drive Developments - related to space flight applications - Thread 2
Post by: Lumina on 05/01/2015 05:12 am
Do EagleWorks have plans to send up a CubeSat with a prototype drive? If thrust is being produced in the lab, what is stopping the experimenters from producing this thrust in a CubeSat? A CubeSat carrying zero propellant and changing its orbital period up and down for a year or so without performance degradation would be conclusive proof that we are dealing with something real.
Title: Re: EM Drive Developments - related to space flight applications - Thread 2
Post by: laika on 05/01/2015 05:18 am
The mere fact that this saga has continued for 15 years without a definitive experiment such as suggested by squid leads one to smell a rat! Could it all just be due to poor experimental method?

Granted. But perhaps maybe its been dragged on for 15 years because no one would take it seriously. Consider how sad it would be if that were the case. This is certainly the first I've heard of it.

Eagleworks isn't full of crackpots. If they discover it was experimental error then that will be the end of it. Hand waving this away because you do not agree with their observations wont kill it, but the science they're doing now will.
ote]


Hopefully Eagleworks will resolve it, but they haven't yet resolved to everyone's satisfaction whether or not there IS any thrust, and yet they are forging ahead hypothesizing warp effects related to zero point energy!
Even their test setup is only intended to produce micro-Newton thrust levels, extremely difficult to measure requiring complex test equipment all a potential source of measurement error.
The UK and Chinese tests apparently produced thrusts you almost could feel! So with accurate, well thought out measurements, an unambiguous result could be produced. Then would be the time to determine what is causing it.



Title: Re: EM Drive Developments - related to space flight applications - Thread 2
Post by: Hexadecibel on 05/01/2015 05:23 am
The mere fact that this saga has continued for 15 years without a definitive experiment such as suggested by squid leads one to smell a rat! Could it all just be due to poor experimental method?

Granted. But perhaps maybe its been dragged on for 15 years because no one would take it seriously. Consider how sad it would be if that were the case. This is certainly the first I've heard of it.

Eagleworks isn't full of crackpots. If they discover it was experimental error then that will be the end of it. Hand waving this away because you do not agree with their observations wont kill it, but the science they're doing now will.
ote]


Hopefully Eagleworks will resolve it, but they haven't yet resolved to everyone's satisfaction whether or not there IS any thrust, and yet they are forging ahead hypothesizing warp effects related to zero point energy!
Even their test setup is only intended to produce micro-Newton thrust levels, extremely difficult to measure requiring complex test equipment all a potential source of measurement error.
The UK and Chinese tests apparently produced thrusts you almost could feel! So with accurate, well thought out measurements, an unambiguous result could be produced. Then would be the time to determine what is causing it.






Then remain skeptical until you are satisfied.  Hypothesizing is part of the process, its not a declaration of fact.
Title: Re: EM Drive Developments - related to space flight applications - Thread 2
Post by: Star One on 05/01/2015 06:37 am
As I suggested in the other thread it seems like this is a topic crying out for a FAQ judging by the repeated questions?
Title: Re: EM Drive Developments - related to space flight applications - Thread 2
Post by: JordanLeDoux on 05/01/2015 07:20 am
I haven't been following these threads closely until recently, but I'm really curious if anyone here has examined whether or not Modified inertia from Hubble scale Casimir effects (MiHsC), which is a theory I just came across today, makes any sense at all. I never got far enough in math to really evaluate this level of physics on my own, but the "crackpot" alarms in my head didn't sound as I was reading about it.

The basics of it are, any object moving to the right will create an event horizon somewhere to left beyond which information cannot be observed. Like other event horizons, this will result in radiation (similar to Hawking radiation) called Unruh radiation. The wavelengths for this radiation are at normal accelerations on the order of light years.

But if you have something like a tube with light inside and reflective surfaces, the photons (because of their speed) will generate Unruh wavelength that are the exact resonant frequency of the tube.

In a uniform tube, this does nothing, but in a cone shaped tube, it would bias the direction of force toward the narrow end.

Again, this isn't my theory, it is proposed by a physicist at Plymouth U in the UK, but it seemed... reasonable.

The theory evidently also has the nice benefit of explaining the effects of dark matter and dark energy without any special tuning, and it explains how inertia works in general from what I was reading.

Does any of that make sense or sound plausible?

EDIT: I ask mainly because a device like the EmDrive is one of the only testable predictions that you could make with this theory given the technology we have now.
Title: Re: EM Drive Developments - related to space flight applications - Thread 2
Post by: Mulletron on 05/01/2015 09:57 am
It is clear that we are operating under other than usual symmetry conditions and that Maxwell's equations as we were trained to use them do not apply.

What are you talking about? Maxwell's equations apply to EM radiation unequivocally. Especially in these low power regimes there is no chance of observing any perturbations due to higher order effects from field theory.

Look, I want to believe in a "reactionless" drive as much as the next SF junkie, but this does not pass the smell test. Consider:
e
0. The proposed explanations violate: conservation of energy, conservation of momentum, and the Lorentz transformations. These are some of the best-tested physical arguments we have today, and yet this "drive" violates them by huge orders of magnitude. Is it really plausible that 100+ years of experiments have failed to notice a comparatively huge effect? RF cavities are not a poorly understood system. I used to work with superconducting RF cavities for particle accelerators with Q > 1 million supporting fields of > 10 MV/m. I can guarantee you we would have noticed if power was disappearing into "thrust", or if the damn things were starting to levitate.

1. Even if we are pushing against the "quantum vacuum" this does not make sense, as any such vacuumo  must be charge neutral and so we would be pushing in opposite directions on electrons and positrons. Not to mention the accelerated positrons would smash into the surrounding cavity, producing copious, easily-observable gamma rays.

1a. Even ignoring this objection, to promote particles from "virtual" to real (as in Hawking radiation...) you have to provide the particles with their mass-energy. 511 keV per electron/positron. Does this make sense?

Assume 100% of the energy delivered to the cavity goes into making virtual particles real: 100 W / (mass of electron * c^2) = 1e15 electrons / s. Assume the particles are instantaneously acclerated to the speed of light (a pretty generous assumption). Then F = (1e15 electron/s)*(mass of electron)*(speed of light) = 2e-7 newtons. Much smaller than what we observe. The explanation does not hold up to scrutiny.

2. The "quantum vacuum"/Casimir effect should not be given more emphasis than is appropriate. It is a calculational tool. For example, one can explain the casimir effect solely through the van der Waals forces between two neighboring conductors, without handwaving explanations about virtual particles: http://physics.stackexchange.com/questions/11544/vander-waals-and-casimir-forces

3. The quantum vacuum publication referenced earlier is really, really bad. It starts with the Bohr formula for hydrogen (thus neglecting any fine structure, etc... effects), and takes a "radius" (which as we know from QM should not be interpreted literally...) to get a volume and from that some kind of density. This has no physical meaning. Then a function 1/r^4 is fitted to the values -- with no justification! This is then compared to the casimir force for cavities separated by this radius... and the values are  pretty far off. But it looks like a factor of 1/3! So some further handwaving about general relativity which also has a factor of 1/3 in one equation! Then there's a whole word salad about solving hydrogen atom wavefunctions with COMSOL [???], which is ridiculous since any undergraduate physics students can solve them with pen and paper...

This papers seems explicitly designed to use lots of fancy terminology and equations to look impressive to anyone with no background in physics, while saying nothing at all of substance. It does not even rise to the level of coherence.

In conclusion, this whole affair appears to be the work of someone who has convinced himself his theory is right and is on a fishing expedition for evidence that supports it. The experimental design is poor (camera pictures of LabView windows? unable to find an RF amplifier to deliver more than 100W of power? inability to measure forces that would be measurable in the 1800's?), the past 100+ years of physical experiments contradict the experiments, and there is no coherent underlying theory.

This is not science, it is cargo cult science.

I really didn't like being quoted alongside items 0-4 as that implies that I advocate such "proposed explanations", which you went on to associate me with cult science. I can tell you that I (and other regulars here) am on record as rejecting all known proposed explanations presented thus far.

From the top:

-What I mean is that any attempt to formulate a theory of where the anomalous thrust is coming from which is based on classical electrodynamics will fail. A fully quantum approach is required. What I mean by "other than usual symmetry conditions" is that based off what I've been reading (aka not my own original research), simultaneous breaking of P & T symmetries is required. Long story short, this leads to a nonzero vacuum momentum density. Want to know more? Read my posts. If you can help put this approach to bed, and replace it with a better one, then do it. If not, help out.

-There is no such thing as a reactionless drive. Just because something isn't belching fire out the back side doesn't make it reactionless. Our job is to find the reaction or find the source of experimental error that everyone is apparently making and has gone undetected.

-0 As I just said, there is NO accepted explanation to date. This means that if the experimentalist are to be believed (multiple parties reporting anomalous thrust) there is a huge opportunity for a research group to earn a nice shiny award for a correct theory backed up with experiment......if this thing really works. It seems more and more likely that it does work.

This is not a "a comparatively huge effect" otherwise we wouldn't need a low torque torsion pendulum or similar to measure it.

-1, 1a Yeah I know. You're both preaching to the choir and barking up the wrong tree. I'm not a card carrying QVPT theory believer. Eagleworks is doing great work. I acknowledge that. At the same time, I think for myself and remain respectful.
http://forum.nasaspaceflight.com/index.php?topic=36313.msg1355944#msg1355944

2. I'm aware of the ongoing controversy you are describing. Doesn't mean you are correct any more than anyone else. The QED vacuum is accepted as fact. Virtual particles aren't only a calculation tool, they are also disturbances in a field. They aren't ONLY relegated to the vacuum either.
http://profmattstrassler.com/articles-and-posts/particle-physics-basics/virtual-particles-what-are-they/

3. I feel ya. And it is not my problem.

As to the rest. You have voiced your concerns. Notice that I am not trying to call you out on any of them. I don't see a reason to be hostile to Eagleworks over their theory or experimental setup though. Now what are your recommendations? Do you have a way forward?

We are very lucky to have a representative from Eagleworks commenting and providing data here. I applaud their experimental work. How about we not scare them off. This is a unique opportunity for people to get together and try to figure out if these copper cans can be put to good use in space.
Title: Re: EM Drive Developments - related to space flight applications - Thread 2
Post by: Star One on 05/01/2015 10:42 am
A question I've been wanting to ask for a while if this drive was on a craft of some type and you stood behind it would you feel any force acting upon you, if you held your hand up would you feel a force pushing upon it, would you note any physical effect at all or any kind of EM effects.
Title: Re: EM Drive Developments - related to space flight applications - Thread 2
Post by: Carl G on 05/01/2015 11:57 am
I'm to tidy up the thread as there's some people who have joined this thread and should be using the entry level thread as opening posts - here: https://forum.nasaspaceflight.com/index.php?topic=37438.0

I will move as many of posts as I can into that thread.

We're also attracting armwavers. Those posts should be reported for moderators to deal with. Insulting posts will be removed, so there's no point responding to them. That is a forum rule.
Title: Re: EM Drive Developments - related to space flight applications - Thread 2
Post by: SH on 05/01/2015 12:35 pm
I tried asking some of these questions before but they were quickly drowned out by senseless bickering, so I'm going to try asking again.

(0) Supposing that you were able to engineer the required negative energy density around the craft in such a way as to produce an Alcubierre style warp bubble.  The spacetime curvature in the CENTER but the edges of the bubble itself would be highly warped, and yet you need to have engineered some structure to hold the negative energy density ring in place.  Thus it seems that the warping of spacetime would necessarily destroy any toroidal structure used to hold the negative energy in place, making it impossible to maintain such a bubble.  Am I missing something here?

(1) The Eagleworks team reportedly made a simulation to predict thrust levels based on the assumption that the vacuum energy of empty space behaves like a plasma which is mutable as suggested by Dr. White.  Then, virtual particles would be able to store and propagate momentum as a wave from one virtual particle to the next until it reached a non-virtual particle to finally absorb the momentum.  Thus the EmDrive would leave a "wake" and this is exactly how White described it.  However there seems to be a problem with this explanation, because with the EmDrive being a resonant cavity, any such QVP wave would need to be initially generated INSIDE the closed cavity, which means the first matter this virtual particle wave would interact with would be the walls of the cavity itself, which would absorb the opposite momentum and thus cancel out any net thrust.  What additional assumptions were made in the simulation to make this not cancel out?

(http://www.nasaspaceflight.com/wp-content/uploads/2015/04/2015-04-19-010503-350x259.jpg)

(2) In "The Alcubierre Warp Drive in Higher Dimensional Spacetime", White and Davis (2006) theorized that, under the Chung-Freese model they predicted any torus of positive energy density would give rise to slight negative energy density in its core due to classical energy in 3+1 dimensions being shifted "off brain" into the unobservable higher dimensions.  They proposed an experiment to test this by constructing a charged capacitor ring.  However, under the mass-energy equivalence, a rapidly spinning torus made of lead would have orders of magnitude greater positive energy density -- so why were they using capacitors in their experiment?

(3) If it were true that any torus of positive energy density contributes to a "boost" factor inside the torus, then it must be to an incredibly small amount, or else people would have noticed by pure chance that objects inside torus tend to move faster, and nobody has noticed this.  However, we have noticed that large heavy toruses require more fuel to propel.  Thus it seems that this theory of a positive energy density torus giving rise to a net boost in thrust must be impossible.

(4) In "Experimental Concepts for Generating Negative Energy in the Laboratory" Davis and Puthoff (2006) showed that negative energy density was producible in the lab using high energy lasers and other methods, and this would not require the more radical assumptions of extra dimensions in the Chung-Freese model.  Why weren't these methods explored?
 
(5) On Rodal's writeup he mentioned that skeptics were hesitant to accept the idea of a mutable QVP because "The mainstream physics community assumes the Quantum Vacuum is indestructible and immutable because of the experimental observation that a fundamental particle like an electron (or a positron) has the same properties (e.g. mass, charge or spin), regardless of when or where the particle was created".  However, if the QVP was storing momentum then any non-virtual particles exposed to it would merely absorb this momentum, which would restore the QVP to its usual ground state without changing any more fundamental properties.  Thus this criticism seems to be moot.
Title: Re: EM Drive Developments - related to space flight applications - Thread 2
Post by: Notsosureofit on 05/01/2015 12:40 pm
Go back and read the entire thread, you'll find they already did most of what you suggested. Their Rig has been tested and has not been falsified.

There is another explanation that has been around for many decades, it is that variations in the gravitational field are indistinguishable from variations in the refractive index of the vacuum. When you refer to the Lorentz invariance of Maxwell's equations, this does not hold in a vacuum where epsilon0 and mu0 are variables. In General Relativity we refer to the metric components, g^uv. These can and are interpreted as components of a variable refractive index, in the Polarizable Vacuum Model. Primarily refined by Hal Puthoff, a few papers of my own and many others have contributed to it. There are many, many papers on this available.

In this case, the interior energy density is not symmetrical, so the refractive index has a gradient. It was said in a previous post that the speed of light inside is different than it is outside. This is the correct interpretation, however it must include a gradient in the refractive index, as it passes through the structure itself, to cause motion. The gradient in the refractive index "is" a gravitational field. That is what the warp drive requires.

In a separate experiment, they may have shown that the speed of light inside the chamber varies. The amount is varies would only be noticed "IF" you were looking for it. Most resonant systems are "tuned" to eliminate such affects in manufacturing. In such a small cavity, I'm not confident it can be measured.

Yes, the simple formula I put up is based on a similar type of assumption.  Could you reference your papers ? (publicly or privately)

Thanks

Edit:  Many decades is correct, I'm 40 years out of date trying to catch up on any changes.
Title: Re: EM Drive Developments - related to space flight applications - Thread 2
Post by: flux_capacitor on 05/01/2015 01:05 pm
Most microwave sources include a resonant cavity. Waveguides are resonant structures. Most concerning for the present work are superconducting RF cavities in particle accelerators (see the attached picture). I have personally worked with cavities similar to those in the picture. They are superconducting at 4K being made out of niobium, and typically support >10 MV/m of electric field, with quality factors of several 10's of million. Any loss of energy to an external medium would have been readily apparent, and we would not be able to run the LHC if there was some unknown effect affecting these sorts of cavities. Not to mention... those walls are actually fairly thin! If we believe the EM drive thrust claims... they would have buckled under the strain and torn from their mountings.

Those resonant cavities (as pictured) do not seem asymmetric to me. Di you ever work with a superconducting truncated conical (frustum) microwave resonant cavity before?
Title: Re: EM Drive Developments - related to space flight applications - Thread 2
Post by: Dmytry on 05/01/2015 01:15 pm
They aren't perfectly symmetric. And even if they were always perfectly symmetric due to the existence of some conspiracy, that would merely balance out the claimed forces, still subjecting the cavity to (far greater than otherwise expected) stresses.

edit: not to mention that any unexpected coupling between EM and anything else would throw off the cavity's parameters by a far more easily measurable amount than the thrust.
Title: Re: EM Drive Developments - related to space flight applications - Thread 2
Post by: CW on 05/01/2015 01:18 pm
I tried asking some of these questions before but they were quickly drowned out by senseless bickering, so I'm going to try asking again.

(0) Supposing that you were able to engineer the required negative energy density around the craft in such a way as to produce an Alcubierre style warp bubble.  The spacetime curvature in the CENTER but the edges of the bubble itself would be highly warped, and yet you need to have engineered some structure to hold the negative energy density ring in place.  Thus it seems that the warping of spacetime would necessarily destroy any toroidal structure used to hold the negative energy in place, making it impossible to maintain such a bubble.  Am I missing something here?

(1) The Eagleworks team reportedly made a simulation to predict thrust levels based on the assumption that the vacuum energy of empty space behaves like a plasma which is mutable as suggested by Dr. White.  Then, virtual particles would be able to store and propagate momentum as a wave from one virtual particle to the next until it reached a non-virtual particle to finally absorb the momentum.  Thus the EmDrive would leave a "wake" and this is exactly how White described it.  However there seems to be a problem with this explanation, because with the EmDrive being a resonant cavity, any such QVP wave would need to be initially generated INSIDE the closed cavity, which means the first matter this virtual particle wave would interact with would be the walls of the cavity itself, which would absorb the opposite momentum and thus cancel out any net thrust.  What additional assumptions were made in the simulation to make this not cancel out?

(http://www.nasaspaceflight.com/wp-content/uploads/2015/04/2015-04-19-010503-350x259.jpg)

(2) In "The Alcubierre Warp Drive in Higher Dimensional Spacetime", White and Davis (2006) theorized that, under the Chung-Freese model they predicted any torus of positive energy density would give rise to slight negative energy density in its core due to classical energy in 3+1 dimensions being shifted "off brain" into the unobservable higher dimensions.  They proposed an experiment to test this by constructing a charged capacitor ring.  However, under the mass-energy equivalence, a rapidly spinning torus made of lead would have orders of magnitude greater positive energy density -- so why were they using capacitors in their experiment?

(3) If it were true that any torus of positive energy density contributes to a "boost" factor inside the torus, then it must be to an incredibly small amount, or else people would have noticed by pure chance that objects inside torus tend to move faster, and nobody has noticed this.  However, we have noticed that large heavy toruses require more fuel to propel.  Thus it seems that this theory of a positive energy density torus giving rise to a net boost in thrust must be impossible.

(4) In "Experimental Concepts for Generating Negative Energy in the Laboratory" Davis and Puthoff (2006) showed that negative energy density was producible in the lab using high energy lasers and other methods, and this would not require the more radical assumptions of extra dimensions in the Chung-Freese model.  Why weren't these methods explored?
 
(5) On Rodal's writeup he mentioned that skeptics were hesitant to accept the idea of a mutable QVP because "The mainstream physics community assumes the Quantum Vacuum is indestructible and immutable because of the experimental observation that a fundamental particle like an electron (or a positron) has the same properties (e.g. mass, charge or spin), regardless of when or where the particle was created".  However, if the QVP was storing momentum then any non-virtual particles exposed to it would merely absorb this momentum, which would restore the QVP to its usual ground state without changing any more fundamental properties.  Thus this criticism seems to be moot.

I think that are some great questions :) .

Regarding 1):
Virtual particle pairs only exist for an extremely short time and then vanish again. One question I already asked elsewhere is: If virtual particle pairs gained momentum and they afterwards 'disappeared' again, where would the momentum go?
Title: Re: EM Drive Developments - related to space flight applications - Thread 2
Post by: PaulF on 05/01/2015 01:43 pm
@ CW:

In response to your own question, where does the momentum go?

Well, I think I have a proposition. To be completely honest I am very very interested in astronomy and physics in general, but I am not a mathematician by a long shot. I am the philosopher-type astronomer, and I like to think that my ideas, however weird or stupid they may seem to some, can give others handles to ascertain the true nature of this truely remarkable discovery.

WHat I was thinking (and I now assume QV is useable and potentially limitless as far as energy contents go) is that the QV is NOT a static medium (virtual particles are made and destroyed continuously by the QV) but more like an energy buffer. Is it possible QV is extradimensional? It seems to me that CoM and CoE are both not broken, if the QV acts like an energy bank, you lend energy from it, and the energy and momentum are poured back in to the QV when the virtual particles, including their momentum, annihilate eachother and are consumed by the QV again. This way the QV retains it's energy and momentum (on average). Indeed like wind and a sail. I also agree on the statement of some that there will be non-linear acceleration when converting electrical energy directly to kinetic energy, there's no such thing as truely free energy. Even with a potential energy well like QV we have to put energy in to get energy out. If the efficiency of that method ever exceeds 1:1 I will be surprised.
Another thought of mine was: Could Back holes be a funnel to the QV? And that this way the energy that was released in the (assumed) Big Bang, is absorbed back into the QV? Could this in any way be connected to the spontaneous creation of virtual particles?
Third pondering: If the QV is not indestructible/immutable, is it possible that the QV is responsible for dark energy? Someone else in this thread proposed  that the concentration of energy in the QV could be dependent on how much mass is in the vicinity, etc. I see a link in these two postulations. If Dark energy is amplified by the presence of matter that could explain a few things.

Do not blast me, these are just propositions. Please confirm or debunk me, whatever you like. Just don't go overboard :)
Title: Re: EM Drive Developments - related to space flight applications - Thread 2
Post by: cfs on 05/01/2015 01:51 pm
Speaking of virtual particles, have there been any attempts to quantify the radiation spewing out of the apparatus?  Something like harmonics of 0.511MeV would be a dead giveaway to solving how this thing works (though I highly doubt it is this simple)...
Title: Re: EM Drive Developments - related to space flight applications - Thread 2
Post by: Mulletron on 05/01/2015 02:27 pm
A message I posted before, with additions and edits from @Rodal.

I am happy that we are finally getting some new talent here.

First, if you find that your post got deleted, it is likely because your post contained inflammatory, disrespectful language, or otherwise didn't have any value proposition. Dissent is fine, try to back it up with a reference or something.  Don't use any name-calling or disrespectful language ;)

If your post doesn't bring something useful or valuable to the table, put it in the other thread.
http://forum.nasaspaceflight.com/index.php?topic=37438.0

With that said, we welcome (and are actively recruiting) new people. We're actually trying to figure out a difficult problem here. This isn't a fan club.

Hostile posts from less-moderated or un-moderated threads are bleeding over here. Keep insulting posts out of here please.

If you are feeling emotional, try a different approach besides handwaving and quitting. If you are unhappy with some conjecture, dig up some science to explain it another way, and tell us about what you learned!

Don't like the experimental setup? Offer CONSTRUCTIVE criticism with a reference or construct one on your own.

I don't want to see this thread derailed ONCE AGAIN by outside agitators.
http://forum.nasaspaceflight.com/index.php?topic=29276.3640

Now back to work.
Title: Re: EM Drive Developments - related to space flight applications - Thread 2
Post by: SH on 05/01/2015 02:32 pm
Regarding 1):
Virtual particle pairs only exist for an extremely short time and then vanish again. One question I already asked elsewhere is: If virtual particle pairs gained momentum and they afterwards 'disappeared' again, where would the momentum go?

I just answered your question in the post you replied to...let me try explaining it again...under White's theory, if a virtual particle absorbs the momentum, then it must transfer the momentum to a different particle before it disappears.  In the vacuum of empty space, that means it propagates from one virtual particle to the next creating a wave, until the wave reaches a non-virtual particle to absorb the momentum.
Title: Re: EM Drive Developments - related to space flight applications - Thread 2
Post by: Rodal on 05/01/2015 02:55 pm
Adding to @Mulletron's guidelines:

Some specific examples (among many -sorry to not mention all  :-[  -) of previous constructive, skeptical criticism on the test set-up are the numerous posts of @frobnicat and @zen-in.  They are objective, logical, analytical, systematic, methodical, and they follow the scientific and engineering method without resorting to abusive language. 

Abusive or insulting language is not part of the scientific method. 

The pen is mightier than the sword, but logical, objective arguments are much more powerful than abusive words.
Title: Re: EM Drive Developments - related to space flight applications - Thread 2
Post by: MikeMcCulloch on 05/01/2015 03:02 pm
I haven't been following these threads closely until recently, but I'm really curious if anyone here has examined whether or not Modified inertia from Hubble scale Casimir effects (MiHsC), which is a theory I just came across today, makes any sense at all. I never got far enough in math to really evaluate this level of physics on my own, but the "crackpot" alarms in my head didn't sound as I was reading about it.

The basics of it are, any object moving to the right will create an event horizon somewhere to left beyond which information cannot be observed. Like other event horizons, this will result in radiation (similar to Hawking radiation) called Unruh radiation. The wavelengths for this radiation are at normal accelerations on the order of light years.

But if you have something like a tube with light inside and reflective surfaces, the photons (because of their speed) will generate Unruh wavelength that are the exact resonant frequency of the tube.

In a uniform tube, this does nothing, but in a cone shaped tube, it would bias the direction of force toward the narrow end.

Again, this isn't my theory, it is proposed by a physicist at Plymouth U in the UK, but it seemed... reasonable.

The theory evidently also has the nice benefit of explaining the effects of dark matter and dark energy without any special tuning, and it explains how inertia works in general from what I was reading.

Does any of that make sense or sound plausible?

EDIT: I ask mainly because a device like the EmDrive is one of the only testable predictions that you could make with this theory given the technology we have now.

Hi Jordan,

Good summary. I have tested MiHsC on the emdrive & the results are encouraging / not conclusive, see my paper:

http://www.ptep-online.com/index_files/2015/PP-40-15.PDF

Best wishes,
Mike
Title: Re: EM Drive Developments - related to space flight applications - Thread 2
Post by: SH on 05/01/2015 03:08 pm
Adding to @Mulletron's guidelines:

Excellent examples of constructive, skeptical criticism on the test set-up are the numerous posts of @frobnicat and @zen-in.  They are objective, logical, analytical, systematic, methodical, and they follow the scientific and engineering method without resorting to abusive language...

Rodal,
Back on the subject of logical and constructive discussion...would you mind responding to the points and questions I just raised above?
Title: Re: EM Drive Developments - related to space flight applications - Thread 2
Post by: Rodal on 05/01/2015 03:13 pm
Adding to @Mulletron's guidelines:

Excellent examples of constructive, skeptical criticism on the test set-up are the numerous posts of @frobnicat and @zen-in.  They are objective, logical, analytical, systematic, methodical, and they follow the scientific and engineering method without resorting to abusive language...

Rodal,
Back on the subject of logical and constructive discussion...would you mind responding to the points and questions I just raised above?

I need some time to give you a thoughtful answer to your thoughtful questions   :)
Title: Re: EM Drive Developments - related to space flight applications - Thread 2
Post by: cfs on 05/01/2015 03:16 pm
Speaking of virtual particles, have there been any attempts to quantify the radiation spewing out of the apparatus?  Something like harmonics of 0.511MeV would be a dead giveaway to solving how this thing works (though I highly doubt it is this simple)...

Seriously though, if an array of PMTs or APDs is set up to capture key parts of the solid angle of radiation and a flux of 0.511MeV gammas is detected manly spewing out in the direction of thrust this would provide support for Dr. White's theory.  If such radiation is found, one could use simulations (e.g. Geant4) of the apparatus to see exactly how things mesh with different theories.

I'm thinking to first order this could just be 6 PMTs placed on +/- xyz axes, though with enough PMTs you could capture a significant portion of the solid angle which could be used to track the momentum of the generated particles.  This would really give us a clear picture of what is happening here.
Title: Re: EM Drive Developments - related to space flight applications - Thread 2
Post by: LasJayhawk on 05/01/2015 03:25 pm
On the test set up. My understanding from what star-drive posted, that the power was about 50 watts, and there is a total of 16' or "RG8" coax between the amplifier and the frustum.
Q1 Was the power measured at the frustum with a directional coupler and power meter, or is that the rated power of the amp?
Q2 what's the lay line information on the cable ( manufacturer and number )?
Title: Re: EM Drive Developments - related to space flight applications - Thread 2
Post by: JoeOfTex on 05/01/2015 03:41 pm
I am assuming the microwaves path is something like this?

(http://i.imgur.com/nA1U9X6.png)

Where forces F0 through F16 represent the force of a single microwave at different time intervals.

If the directional force happens at F2, and the opposing force happens from F3 to F16, this still obeys Newtons third law.  Isn't the opposing force just being damped like a spring over time? 

If M1, M2, ... Mn represent different microwaves, and each Mi is following a similar course, wouldn't this suggest that the directional force will remain positive at a constant rate until the device is shut off causing all forces to cancel?

 
Title: Re: EM Drive Developments - related to space flight applications - Thread 2
Post by: Rodal on 05/01/2015 03:53 pm
I am assuming the microwaves path is something like this?

(http://i.imgur.com/nA1U9X6.png)

Where forces F0 through F16 represent the force of a single microwave at different time intervals.

If the directional force happens at F2, and the opposing force happens from F3 to F16, this still obeys Newtons third law.  Isn't the opposing force just being damped like a spring over time? 

If M1, M2, ... Mn represent different microwaves, and each Mi is following a similar course, wouldn't this suggest that the directional force will remain positive at a constant rate until the device is shut off causing all forces to cancel?

 

Not according to Maxwell's linear, isotropic equations.

The small base of the EM Drive is not open.  It is a closed cavity.  As such, the waves inside it are not travelling waves, but standing waves.   See this:  http://en.wikipedia.org/wiki/Standing_wave

The energy flux is pointed towards one end during half a (Poynting vector) period and it is pointed towards the opposite end during the next half-period.  Hence the net energy flux over a whole period is completely self-cancelling.

No net directional thrust for a microwave closed cavity can be explained just using Maxwell's linear, isotropic equations.  If the measurements are not an experimental artifact, another explanation is needed.

If the small base would be open, then it would be an inefficient photon rocket, with the microwave photons escaping at the small base end.  Less efficient for space propulsion than using a flashlight or a military searchlight as a means of propulsion.

Even for a perfectly collimated photon rocket, the thrust/powerInput of such a photon rocket is orders of magnitude less than what is claimed for these experiments.
Title: Re: EM Drive Developments - related to space flight applications - Thread 2
Post by: flux_capacitor on 05/01/2015 03:59 pm
Next, using this new Q-V plasma simulation tool that utilizes the instantaneous E&M fields from COMSOL for one complete RF cycle in 5 degree increments as its input file, we are now seeing why we need the PTFE or HDPE dielectrics in the frustum while using near pure sine wave power levels below ~100W in the ~2.0 GHz frequency range to generate detectable thrust, and why Shawyer and the Chinese didn't while pumping 80W to 2,500W using magnetron RF sources.  We think the reasons are two fold. 

The first is that Shawyer and the Chinese both used magnetron RF sources for their experiments.  An RF source that generates large AM, FM and PM modulation of the carrier wave with typical FM modulation bandwidth on the order of at least +/-20 MHz.  (These time rate to change of energy modulations increase the Q-V density in our model.) 

The second reason we found running these 3D Q-V plasma simulations for the EMPTY copper frustum, was that increasing the input power tends to focus the Q-V plasma flow from near omnidirectional from the frustum at low powers, to a much more jet like beam at higher powers measured in kW to tens of kW-rf.  In fact the simulation for the 100W run predicted only ~50uN for our pure RF system with dielectric, while the 10kW run predicted a thrust level of ~6.0 Newton without a dielectric in the cavity.  And at 100kW-rf it was now up to ~1300 Newton, but the input power to thrust production nonlinearity was starting to taper off around 50kW.   Of course these Q-V plasma thrust predictions are based on the Q-V not being immutable and non-degradable, a feature we admit is not widely accepted by the mainstream physics community, at least at the moment. :)

@Paul (and Dr. Rodal who knows that matter for having written the breaking news on NSF homepage):
Can you please clarify two things about that recent predictive simulation:

1- How (i.e. what is the physical cause) does the dielectric enhance and control (hence is mandatory) that almost omnidirectional flow of virtual particles at low power, into a detectable thrust, as per Dr. White's QVF conjecture?

2- Why the model transforms that omnidirectional cloud of QV particles at low powers, into a "jet like beam" at higher powers? (progressively or in a non-linear way BTW?)

Maybe I missed some important messages in the thread, but since there is more and more reactions to your posts and to the NSF news article in the media and twitter, and since Eagleworks hasn't published anything yet about the EmDrive vacuum tests, the RF resonant warp experiment, and the newest simulations, I think those claims (let's start with the low power dielectric and high power spike modeling of QV flow) deserve more explanation.
Title: Re: EM Drive Developments - related to space flight applications - Thread 2
Post by: Rodal on 05/01/2015 04:18 pm
Next, using this new Q-V plasma simulation tool that utilizes the instantaneous E&M fields from COMSOL for one complete RF cycle in 5 degree increments as its input file, we are now seeing why we need the PTFE or HDPE dielectrics in the frustum while using near pure sine wave power levels below ~100W in the ~2.0 GHz frequency range to generate detectable thrust, and why Shawyer and the Chinese didn't while pumping 80W to 2,500W using magnetron RF sources.  We think the reasons are two fold. 

The first is that Shawyer and the Chinese both used magnetron RF sources for their experiments.  An RF source that generates large AM, FM and PM modulation of the carrier wave with typical FM modulation bandwidth on the order of at least +/-20 MHz.  (These time rate to change of energy modulations increase the Q-V density in our model.) 

The second reason we found running these 3D Q-V plasma simulations for the EMPTY copper frustum, was that increasing the input power tends to focus the Q-V plasma flow from near omnidirectional from the frustum at low powers, to a much more jet like beam at higher powers measured in kW to tens of kW-rf.  In fact the simulation for the 100W run predicted only ~50uN for our pure RF system with dielectric, while the 10kW run predicted a thrust level of ~6.0 Newton without a dielectric in the cavity.  And at 100kW-rf it was now up to ~1300 Newton, but the input power to thrust production nonlinearity was starting to taper off around 50kW.   Of course these Q-V plasma thrust predictions are based on the Q-V not being immutable and non-degradable, a feature we admit is not widely accepted by the mainstream physics community, at least at the moment. :)

@Paul (and Dr. Rodal who knows that matter for having written the breaking news on NSF homepage):
Can you please clarify two things about that recent predictive simulation:

1- How (i.e. what is the physical cause) does the dielectric enhance and control (hence is mandatory) that almost omnidirectional flow of virtual particles at low power, into a detectable thrust, as per Dr. White's QVF conjecture?

2- Why the model transforms that omnidirectional cloud of QV particles at low powers, into a "jet like beam" at higher powers? (progressively or in a non-linear way BTW?)

Maybe I missed some important messages in the thread, but since there is more and more reactions to your posts and to the NSF news article in the media and twitter, and since Eagleworks hasn't published anything yet about the EmDrive vacuum tests, the RF resonant warp experiment, and the newest simulations, I think those claims (let's start with the low power dielectric and high power spike modeling of QV flow) deserve more explanation.

As discussed in the article, these are reported results from the computer model of Dr. White.  Whenever there is a numerical computer analysis result reported, the validity of the results is entirely dependent on 1) the validity of the physical model underlying the numerical model, 2) the validity of the numerical implementation, 3) the validity of the computer programming ("bugs" and any errors of any kind) and 4) the validity of the input data.  Even if the computer simulation is valid (under the previously discussed considerations), the results of a complicated numerical analysis may not be readily understood. For example, entirely valid results from numerical simulations of nonlinear dynamic impact and explosive blasts on aerospace structures are very difficult to understand without careful consideration  (ideally, difficult to understand computer predictions should always be backed up by experiments).  Examples abound, think for example of how possible impact of foam was mishandled and misunderstood by NASA on a Space Shuttle disaster.

Experimental testing is needed to verify these computer analysis predictions.  Reportedly this will occur in June or July of this year.
Title: Re: EM Drive Developments - related to space flight applications - Thread 2
Post by: MazonDel on 05/01/2015 04:27 pm
(2) In "The Alcubierre Warp Drive in Higher Dimensional Spacetime", White and Davis (2006) theorized that, under the Chung-Freese model they predicted any torus of positive energy density would give rise to slight negative energy density in its core due to classical energy in 3+1 dimensions being shifted "off brain" into the unobservable higher dimensions.  They proposed an experiment to test this by constructing a charged capacitor ring.  However, under the mass-energy equivalence, a rapidly spinning torus made of lead would have orders of magnitude greater positive energy density -- so why were they using capacitors in their experiment?

If this is the experiments I am thinking of, I believe the point of the capacitor vs the lead torus was that they would be more easily able to adjust it. On state vs Off state, or even partially charged states. Less physical poking at the rig to adjust the variable in question. Plus, it might be a bit safer than rapidly spinning the lead torus (though I wouldn't know how fast they'd have to spin it, so I could be wrong on that point).
Title: Re: EM Drive Developments - related to space flight applications - Thread 2
Post by: WarpTech on 05/01/2015 04:27 pm
Go back and read the entire thread, you'll find they already did most of what you suggested. Their Rig has been tested and has not been falsified.

There is another explanation that has been around for many decades, it is that variations in the gravitational field are indistinguishable from variations in the refractive index of the vacuum. When you refer to the Lorentz invariance of Maxwell's equations, this does not hold in a vacuum where epsilon0 and mu0 are variables. In General Relativity we refer to the metric components, g^uv. These can and are interpreted as components of a variable refractive index, in the Polarizable Vacuum Model. Primarily refined by Hal Puthoff, a few papers of my own and many others have contributed to it. There are many, many papers on this available.

In this case, the interior energy density is not symmetrical, so the refractive index has a gradient. It was said in a previous post that the speed of light inside is different than it is outside. This is the correct interpretation, however it must include a gradient in the refractive index, as it passes through the structure itself, to cause motion. The gradient in the refractive index "is" a gravitational field. That is what the warp drive requires.

In a separate experiment, they may have shown that the speed of light inside the chamber varies. The amount is varies would only be noticed "IF" you were looking for it. Most resonant systems are "tuned" to eliminate such affects in manufacturing. In such a small cavity, I'm not confident it can be measured.

Yes, the simple formula I put up is based on a similar type of assumption.  Could you reference your papers ? (publicly or privately)

Thanks

Edit:  Many decades is correct, I'm 40 years out of date trying to catch up on any changes.


No problem, most of the stuff is on Research Gate and Vixra.

https://www.researchgate.net/publication/223130116_Advanced_Space_Propulsion_Based_on_Vacuum_%28Spacetime_Metric%29_Engineering

https://www.dropbox.com/s/tyi7ja597c7pax7/Desiato_EM_warp_drive_1-15-15_v18.pdf?dl=0

https://www.researchgate.net/profile/Todd_Desiato

https://www.researchgate.net/profile/Harold_Puthoff

Also, look at the references therein.

Title: Re: EM Drive Developments - related to space flight applications - Thread 2
Post by: aero on 05/01/2015 04:29 pm
As many here know, I have been pursuing time-domain modelling (FDTD, using Meep from MIT) of the EM thruster with the thought that thrust may be generated by evanescent waves escaping through gaps in the construction.

My conclusion is "No." While evanescent waves do appear to consistently generate on the order of 10% greater thrust than an ideal photon rocket, that is orders of magnitude below the experimentally measured forces. Recall that the ideal photon rocket uses an ideal plane wave as the drive and the evanescent waves escaping via the EM thruster gaps are anything but ideal plane waves so slightly greater forces are reasonable.

Still using FDTD, I am considering a model of the QV but as I don't do original mathematical physics, I must rely on published results. I did find this paper, published in February this year that addresses QV models in the time domain. I don't think it has been referenced here as yet. Prog. Theor. Exp. Phys. 2015, 023A01 (11 pages)
DOI: 10.1093/ptep/ptu170  Radiation reaction in quantum vacuum

http://ptep.oxfordjournals.org/content/2015/2/023A01.full.pdf (http://ptep.oxfordjournals.org/content/2015/2/023A01.full.pdf)

I can just barely read the math, but I need a Drude-Lorenz model of equation 33 in order to make any headway.

First, does this paper provide anything new? (Obviously I think it does.)
Second, does anyone want to provide a Drude-Lorenz model of the QV?

And I agree with Dr. Rodel, results from a numerical model must be very carefully evaluated as it could easily mean nothing. Results from two completely different numerical approaches to the same problem can also mean nothing, but if they were in good agreement that would be something to consider seriously.
Title: Re: EM Drive Developments - related to space flight applications - Thread 2
Post by: Thutmose on 05/01/2015 04:37 pm
Speaking of virtual particles, have there been any attempts to quantify the radiation spewing out of the apparatus?  Something like harmonics of 0.511MeV would be a dead giveaway to solving how this thing works (though I highly doubt it is this simple)...

The problem with this method of thrust, is that at the low power of the device tested, it would only result in about 1% or so of the measured thrust.

Assuming you have a 50w power supply (P), and 1% of that energy goes into pair production Rp, then 50% of the remainder goes into accelerated the produced pairs (K), then the impulse (I) of the device over one second can be calculated as follows:
m = Rp * P / c^2
I = ( 2 * K * m + (K/c)^2 ) ^ 0.5

where c is the speed of light.

the relativistic momentum is used as the particles end up being imparted with a few MeV/c^2 of momentum each, which seems reasonable for a single step in a RF acceleration cavity.

which results in 4.13µNs of impulse, or over 1 second, that same value of thrust.
the thrust increases as pair production rate decreases, but not significantly once less than 1% (10% the thrust is 3.75µN).

one interesting note if pair production is the factor causing thrust, assuming the power supply mentioned here https://web.archive.org/web/20041022135357/http://www.spacetransportation.com/ast/presentations/7b_vandy.pdf is feasable, and 50MWe can be produced at 1kW/kg, then such a device would accelerate at 8g

If something is horribly wrong with my maths, please correct me.
Title: Re: EM Drive Developments - related to space flight applications - Thread 2
Post by: Faustus on 05/01/2015 04:43 pm
Hooray for EM Drives!!!  We are finally catching up to the U.S.S. Enterprise!!!
Title: Re: EM Drive Developments - related to space flight applications - Thread 2
Post by: DoeJoe on 05/01/2015 04:46 pm
Let us not forget:

Kepler’s laws gave us a beautiful and powerfully predictive description of the Solar System: planets orbiting in ellipses with the Sun at its center. Newton’s Universal Theory of Gravity gave us the physics that explained Kepler’s laws but also showed that they weren’t perfect. Centuries later, Einstein’s General Relativity gave us our modern picture of gravity, showing that Newton’s gravity isn’t quite right. Science progresses by overthrowing old theories and showing that they are wrong… only, very often, the term “wrong” is over-simplistic, hiding subtleties in how science really progresses.

First presented in Second Life on June 6, 2008.


There is infinitely more that we do not know, than we know.

DJ
Title: Re: EM Drive Developments - related to space flight applications - Thread 2
Post by: Robotbeat on 05/01/2015 04:50 pm
Also, if you use the wrong solution method or too large of a timestep (or too small, sometimes!) but the equations are right, you can easily have a simulation which does not conserve energy.

Heck, if you've ever written an orbital mechanics simulation using the simple Euler's method, you often end up with your planets flung out to the stars or changing in orbits unless you use a really, really small step size (but then you get into problems with rounding!).

tl;dr: Computer simulations are no way to prove this thing works. It's easy to glitch them (by accident) into screwing up the physics. And, of course, they are no better than their underlying assumptions (and often are worse...).
Title: Re: EM Drive Developments - related to space flight applications - Thread 2
Post by: wes_wilson on 05/01/2015 04:59 pm
Regarding 1):
Virtual particle pairs only exist for an extremely short time and then vanish again. One question I already asked elsewhere is: If virtual particle pairs gained momentum and they afterwards 'disappeared' again, where would the momentum go?

I just answered your question in the post you replied to...let me try explaining it again...under White's theory, if a virtual particle absorbs the momentum, then it must transfer the momentum to a different particle before it disappears.  In the vacuum of empty space, that means it propagates from one virtual particle to the next creating a wave, until the wave reaches a non-virtual particle to absorb the momentum.

Or maybe they don't disappear again at all?  Hawking radiation is one known example where the addition of energy to a virtual particle pair can promote them into real particles; and that concept's been pretty rigorously analyzed.  Maybe that's what's happening here: the energy is promoting virtual particles into real particles and the drive is exchanging momentum between the spacecraft and real particle pairs.

Title: Re: EM Drive Developments - related to space flight applications - Thread 2
Post by: cfs on 05/01/2015 05:10 pm
Also, if you use the wrong solution method or too large of a timestep (or too small, sometimes!) but the equations are right, you can easily have a simulation which does not conserve energy.

Heck, if you've ever written an orbital mechanics simulation using the simple Euler's method, you often end up with your planets flung out to the stars or changing in orbits unless you use a really, really small step size (but then you get into problems with rounding!).

tl;dr: Computer simulations are no way to prove this thing works. It's easy to glitch them (by accident) into screwing up the physics. And, of course, they are no better than their underlying assumptions (and often are worse...).

Which is exactly why one needs to use mature simulation packages which have hammered out most of the important physics and weeded out most of the bugs.   ;D

When I first started writing simulations as an undergrad I remember myself thinking how much easier and quicker it would be to write simulations for detectors on my own rather than learning something as monstrously large as Geant4.  But then after taking a computational physics class where we actually got to design these basic simulations and had to worry about things such as weather or not double precision alone is enough to handle calculating derivatives in such small time steps it gave me full appreciation of how well things like Geant4 are actually able to handle the underlying physics. 

Provided one uses a more sophisticated algorithm or package which can handle these small time steps it provides an excellent way to simulate important physics -- especially in cases where it is impossible to solve the system analytically.
Title: Re: EM Drive Developments - related to space flight applications - Thread 2
Post by: mlintin on 05/01/2015 05:18 pm
Light has momentum and therefore interacts with gravitational fields--the directional force of the gravity blueshifts the light as it travels towards the center of mass. This change in energy will result in an increase in momentum as the energy is increasing at an exponential rate to the increase in speed of light (the relationship is non-linear based on the redshifting equation).  What's being disproven here isn't the Law of Conservation of Momentum, but rather the consistent speed of light in a vacuum.  It's actually a theory of mine, so this is pretty neat news. I mean, obviously it's unlikely to be true.  But I've found more and more support for this (It also explains gravitational redshifting as well as special relativity concepts).   I wonder if they oriented the engine directly opposite to the direction of the gravitational force if the thrust would change, Or if they took the engine out into space into a place with no gravitational field to see what happens.  I'm fascinated by this, despite how unlikely it is to be true.
Title: Re: EM Drive Developments - related to space flight applications - Thread 2
Post by: flux_capacitor on 05/01/2015 05:23 pm
Regarding 1):
Virtual particle pairs only exist for an extremely short time and then vanish again. One question I already asked elsewhere is: If virtual particle pairs gained momentum and they afterwards 'disappeared' again, where would the momentum go?

I just answered your question in the post you replied to...let me try explaining it again...under White's theory, if a virtual particle absorbs the momentum, then it must transfer the momentum to a different particle before it disappears.  In the vacuum of empty space, that means it propagates from one virtual particle to the next creating a wave, until the wave reaches a non-virtual particle to absorb the momentum.

Or maybe they don't disappear again at all?  Hawking radiation is one known example where the addition of energy to a virtual particle pair can promote them into real particles; and that concept's been pretty rigorously analyzed.  Maybe that's what's happening here: the energy is promoting virtual particles into real particles and the drive is exchanging momentum between the spacecraft and real particle pairs.
That's also a problem. If virtual particles do appear, gain momentum then disappear, momentum conservation seems to be violated. If the virtual particles are promoted as real particles as you suggest and don't disappear, they gain momentum and conservation is preserved, but since they don't escape the cavity (which is hermetically closed) all the momentum sums to zero and there is no thrust, i.e. no EmDrive. [EDIT: it's even worse than that since in a pair production, one particle is an antiparticle, see SH answer just below]
Title: Re: EM Drive Developments - related to space flight applications - Thread 2
Post by: SH on 05/01/2015 05:30 pm
Or maybe they don't disappear again at all?  Hawking radiation is one known example where the addition of energy to a virtual particle pair can promote them into real particles; and that concept's been pretty rigorously analyzed.  Maybe that's what's happening here: the energy is promoting virtual particles into real particles and the drive is exchanging momentum between the spacecraft and real particle pairs.

Vacuum fluctuations produce virtual particle-antiparticle pairs.  When real antimatter and matter come into contact they annihilate each other and produce gamma rays.   thus if the virtual particle pair were promoted to "real" particles then they would still annihilate each other and this would be observable as large amounts of gamma radiation.  Moreover, the presence of this gamma radiation (from nothingness) would violate conservation of energy.

Hawking radiation is only present at the event horizon of a black hole because, in order for it to occur, one of the virtual particles in the pair must be sucked into the black hole while the other managed to escape.  This essentially prevents the one that escaped from disappearing back into the vacuum, and energy is conserved because the positive energy of the escaped particle is cancelled out by the negative energy of the particle that got sucked into the black hole, which effectively reduces the black hole's externally measurable mass.

Title: Re: EM Drive Developments - related to space flight applications - Thread 2
Post by: Hexadecibel on 05/01/2015 05:37 pm
Regarding 1):
Virtual particle pairs only exist for an extremely short time and then vanish again. One question I already asked elsewhere is: If virtual particle pairs gained momentum and they afterwards 'disappeared' again, where would the momentum go?

I just answered your question in the post you replied to...let me try explaining it again...under White's theory, if a virtual particle absorbs the momentum, then it must transfer the momentum to a different particle before it disappears.  In the vacuum of empty space, that means it propagates from one virtual particle to the next creating a wave, until the wave reaches a non-virtual particle to absorb the momentum.

Or maybe they don't disappear again at all?  Hawking radiation is one known example where the addition of energy to a virtual particle pair can promote them into real particles; and that concept's been pretty rigorously analyzed.  Maybe that's what's happening here: the energy is promoting virtual particles into real particles and the drive is exchanging momentum between the spacecraft and real particle pairs.
That's also a problem. If virtual particles do appear, gain momentum then disappear, momentum conservation seems to be violated. If the virtual particles are promoted as real particles as you suggest and don't disappear, they gain momentum and conservation is preserved, but since they don't escape the cavity (which is hermetically closed) all the momentum sums to zero and there is no thrust, i.e. no EmDrive. [EDIT: it's even worse than that since in a pair production, one particle is an antiparticle, see SH answer just below]

I'm no physicist and this probably way off base and completely wrong... but could one explanation be that the device is somehow causing the virtual particles to tunnel?
Title: Re: EM Drive Developments - related to space flight applications - Thread 2
Post by: flux_capacitor on 05/01/2015 05:40 pm
Hawking radiation is only present at the event horizon of a black hole because, in order for it to occur, one of the virtual particles in the pair must be sucked into the black hole while the other managed to escape.  This essentially prevents the one that escaped from disappearing back into the vacuum, and energy is conserved because the positive energy of the escaped particle is cancelled out by the negative energy of the particle that got sucked into the black hole, which effectively reduces the black hole's externally measurable mass.

What you've just said assumes that antiparticles have a negative energy, which has not been proven, and it would rather be the opposite as the various papers and experiments on that subject seems to tell us that antimatter has a positive mass. Thus positive energy.

Antimatter (as per Dirac) is C-symmetry. But charge conjugation does not reverse energy. T-symmetry does. Feynman imagined another type of antiparticles, with PT-Symmetry. Those beasts would have a negative energy, and negative mass (if they have one). But they have never been observed.
Title: Re: EM Drive Developments - related to space flight applications - Thread 2
Post by: squid on 05/01/2015 05:46 pm
Let us not forget:

Kepler’s laws gave us a beautiful and powerfully predictive description of the Solar System: planets orbiting in ellipses with the Sun at its center. Newton’s Universal Theory of Gravity gave us the physics that explained Kepler’s laws but also showed that they weren’t perfect. Centuries later, Einstein’s General Relativity gave us our modern picture of gravity, showing that Newton’s gravity isn’t quite right. Science progresses by overthrowing old theories and showing that they are wrong… only, very often, the term “wrong” is over-simplistic, hiding subtleties in how science really progresses.

First presented in Second Life on June 6, 2008.


There is infinitely more that we do not know, than we know.

DJ

Ah yes, but the issue is that General Relativity reduces to Newtonian gravity in the appropriate limit. Special relativity recovers newtonian mechanics at low speeds. Quantum mechanics recovers classical dynamics in the limit of large quantum number (*). QFT reduces to regular QM at low energies. And so on.

Here is a list of experiments which CANNOT be right, or would give different results if the EM drive really is coupling energy electromagnetically to something else. Others should feel free to add to the list...

1. The Michelson-Morely experiment would not give a null result
2. Atomic transition lines would be at the wrong frequencies
3. Superconducting qubits would show a reduced T1 time due to extra losses in superconducting cavities
4. Accelerator RF cavities would not work as designed, and the LHC would be unable to operate
5. The predicted standard model couplings for subatomic partices would be off
6. W and Z bosons would have different masses
6. Microwave electronics would work differently as the earth rotates around the sun
7. RF waveguide frequencies would not be as predicted
9. Stability of nuclear particles would be changed
10. Solar emission lines would be different

I remind people that the cavity is in some sense a red herring -- it just supports some electromagnetic field, which we must then assume acts on something else to generate a reaction force. You cannot make a theory which only applies in the case of a particular cavity geometry... if momentum is being anomalously generated by a particular configuration of electric and magnetic fields, all fields must create some momentum as Maxwell's equations are linear

(*) okay, for the experts, this is not quite right but you get the idea...
Title: Re: EM Drive Developments - related to space flight applications - Thread 2
Post by: squid on 05/01/2015 05:52 pm
Hawking radiation is only present at the event horizon of a black hole because, in order for it to occur, one of the virtual particles in the pair must be sucked into the black hole while the other managed to escape.  This essentially prevents the one that escaped from disappearing back into the vacuum, and energy is conserved because the positive energy of the escaped particle is cancelled out by the negative energy of the particle that got sucked into the black hole, which effectively reduces the black hole's externally measurable mass.

What you've just said assumes that antiparticles have a negative energy, which has not been proven, and it would rather be the opposite as the various papers and experiments on that subject seems to tell us that antimatter has a positive mass. Thus positive energy.

Antimatter (as per Dirac) is C-symmetry. But charge conjugation does not reverse energy. T-symmetry does. Feynman imagined another type of antiparticles, with PT-Symmetry. Those beasts would have a negative energy, and negative mass (if they have one). But they have never been observed.

This is a common misconception. The antiparticle does not have "negative energy". What is going on is simply that the gravitational binding energy of the particle is greater than its rest mass. To an outside observer this looks like negative energy, so to balance the books the other particle gains positive energy and the black hole mass is reduced. However, to an observer inside the event horizon, the antiparticle has positive energy.

See for example the comments by researchers (I am not a specialist in GR) in this article: http://arstechnica.com/science/2015/03/01/completely-implausible-a-controversial-paper-exists-but-so-do-black-holes/

And again, please remember that all discussion about virtual particles should view them first and foremost as calculation tools, not literally particles popping into and out of existence all the time. The fundamental quantity in QFT is always the field, and "virtual particles" refers to fluctuations in the expectation value of the field. To promote them to actual physical particles is to take Feynman diagrams a little too literally...

Title: Re: EM Drive Developments - related to space flight applications - Thread 2
Post by: flux_capacitor on 05/01/2015 05:56 pm
I admit this Hawking radiation thingy and its bald or hairy black hole evaporating, as well as generally speaking the QVF/zero-point energy stuff, is over my head. I like plain vanilla GR more (even including Mach's principle ;) ) Whatever, thanks for the link.
Title: Re: EM Drive Developments - related to space flight applications - Thread 2
Post by: SH on 05/01/2015 06:00 pm
if momentum is being anomalously generated by a particular configuration of electric and magnetic fields, all fields must create some momentum as Maxwell's equations are linear[/b]

This is untrue.  EM fields could be used to produce a momentum indirectly by some other interaction that is nonlinear, non-monotonic, or non-differentiable.  For a simple counter example of your claim, consider an electric car.  A large amount of electricity on the motor causes it to spin and this is translated into linear momentum through the wheels...but if you reduce the amount of power, at some point the motor still stop rotating entirely and there will be zero force on the car.
Title: Re: EM Drive Developments - related to space flight applications - Thread 2
Post by: Rodal on 05/01/2015 06:11 pm
Hello, I have been following Dr. White's publications with interest since he revived the Alcubierre concept in 2003.  I have a few comments and questions below.

...

(3) Evidence in support of the QVP being mutable.  a) the force measurements of the EM-drive, b) the Casimir effect, c) as explained here (http://nextbigfuture.com/2015/02/more-emdrive-experiment-information.html), apparently Dr. White was able to show that the electron shell radii of all atoms up to atomic number 7 can be predicted based on the asumption that QVP is mutable.  I haven't read the details of that and would be curious to read where this is published if anyone knows.  d) A generic property of inflationary cosmology (as written about by Hawking, Alan Guth, Hartle, Turok, Pasachoff, Filippenko, Stenger, Vilenkin and others) is that the universe began from a small quantum fluctuation from the ground state, as stated by Vilenkin "small amount of energy was contained in that [initial] curvature, somewhat like the energy stored  in  a  strung  bow.   This  ostensible  violation  of  energy conservation is allowed by the Heisenberg uncertainty principle for sufficiently small time intervals.  The bubble then inflated exponentially and the universe grew by many orders of magnitude in a tiny fraction of a second".  Thus, it seems that inflationary cosmology is founded on a principle of mutable QVP as well.

...


Several of these questions can only be properly addressed by Dr. White and his colleagues since they best understand their theory. Addressing the EM Drive, concerning question 3) (posted above), the "evidence" although interesting, is not conclusive:

a) the force measurements of the EM-drive,

They are inconsistent.  For example, with the EM Drive re-oriented, rotated by 180 degrees so that it points in the opposite direction, the measurements differ significantly.  Need to have consistent measurements replicated at several labs (NASA Glenn, JPL, John Hopkins have been mentioned) for the evidence to be conclusive. Also, even if the data would be consistent, and the EM Drive can be used for space propulsion, that would not be conclusive evidence that its operation is due to a degradable and mutable quantum vacuum as there are several other explanations being explored.



b) the Casimir effect,

does not need to be explained by a degradable quantum vacuum.  The Casimir effect can alternatively be explained by van der Waal forces without any reference to the quantum vacuum, just based on the fine structure constant (see original publications by the great Nobel Prize winner Julian Schwinger and by Prof. Jaffe -MIT Physics department- for example)



c) Dr. White was able to show that the electron shell radii of all atoms up to atomic number 7 can be predicted based on the asumption that QVP is mutable.

Not conclusive evidence.  The prediction is approximate.  The famous astronomer Eddington (one of the first to successfully confirm General Relativity) believed in the significance of certain numbers having a physical significance that turned out to be unjustified.  Eddington argued that the value of the fine-structure constant α , could be obtained by pure deduction. He related α to the Eddington number, which was his estimate of the number of protons in the universe. This led him in 1929 to conjecture that α was exactly 1/137.



d) A generic property of inflationary cosmology (as written about by Hawking, Alan Guth, Hartle, Turok, Pasachoff, Filippenko, Stenger, Vilenkin and others) is that the universe began from a small quantum fluctuation from the ground state, as stated by Vilenkin "small amount of energy was contained in that [initial] curvature, somewhat like the energy stored  in  a  strung  bow.   This  ostensible  violation  of  energy conservation is allowed by the Heisenberg uncertainty principle for sufficiently small time intervals.  The bubble then inflated exponentially and the universe grew by many orders of magnitude in a tiny fraction of a second".  Thus, it seems that inflationary cosmology is founded on a principle of mutable QVP as well.

Not conclusive evidence because of its extremely low likelihood (based on quantum probability) of such an organized  (rather than random)  small quantum fluctuation from the ground state occurring.  In other words, the mutability and degradability of the quantum vacuum may be possible but extremely unlikely.




Conclusion: consistent experimental results need to be replicated at independent laboratories and a theoretical explanation needs to be embraced by the scientific community.
Title: Re: EM Drive Developments - related to space flight applications - Thread 2
Post by: CW on 05/01/2015 06:15 pm
Regarding 1):
Virtual particle pairs only exist for an extremely short time and then vanish again. One question I already asked elsewhere is: If virtual particle pairs gained momentum and they afterwards 'disappeared' again, where would the momentum go?

I just answered your question in the post you replied to...let me try explaining it again...under White's theory, if a virtual particle absorbs the momentum, then it must transfer the momentum to a different particle before it disappears.  In the vacuum of empty space, that means it propagates from one virtual particle to the next creating a wave, until the wave reaches a non-virtual particle to absorb the momentum.

Or maybe they don't disappear again at all?  Hawking radiation is one known example where the addition of energy to a virtual particle pair can promote them into real particles; and that concept's been pretty rigorously analyzed.  Maybe that's what's happening here: the energy is promoting virtual particles into real particles and the drive is exchanging momentum between the spacecraft and real particle pairs.
That's also a problem. If virtual particles do appear, gain momentum then disappear, momentum conservation seems to be violated. If the virtual particles are promoted as real particles as you suggest and don't disappear, they gain momentum and conservation is preserved, but since they don't escape the cavity (which is hermetically closed) all the momentum sums to zero and there is no thrust, i.e. no EmDrive.

I see your point. On the other hand, the crystal lattice of the metal cone is more like a sieve from the viewpoint of electrons/positrons, right?
Title: Re: EM Drive Developments - related to space flight applications - Thread 2
Post by: jknuble on 05/01/2015 06:33 pm
Hi Everyone,

I am an RF engineer in the Microwave Instrument Technology Branch at NASA Goddard Space Flight Center.  I had seen a few articles here and there about the EM drive and today it caught my eye on IO9.com.  While I have only spent the last hour or so reviewing what has been done to eliminated external factors to explain the phenomenon I would like to offer my two-cents.  If what I'm suggesting as an explanation has already been eliminated, I apologize.

Have you considered the effects of breakdown, and in particular multipaction and corona generation?  Multipaction breakdown events are known phenomenon on the RF radar and communication systems community.  Essentially, at high RF powers you see an effect similar to arcing within your components.  This arcing can occur between conductors and dielectrics or even between conductors in vacuum.  Sharp edges such as welds and fasteners - particularly in a cavity resonator such as this - can cause these events.  This result is damage to the interior conductor and particle generation (even in metal-only situations) as material is "burned."  In this case, the metal walls and / or contaminants of your cavity would serve as the propellant.  Corona / plasma can then develop from this particle release and exacerbate the situation.

Better descriptions can be found here : 

http://en.wikipedia.org/wiki/Multipactor_effect

http://www.microwaves101.com/encyclopedias/multipaction 

As a real world example, I am the lead engineer for the Radiometer Front End on the recently launched Soil Moisture Active Passive (SMAP) Mission. (http://smap.jpl.nasa.gov/)  We had a terrible problem with multipaction in our RF diplexer which was a cavity resonator - similar to your setup.  These devices are essentially three-port band-pass filters whose resonant frequencies are set by the physical dimensions of the (mostly) empty cavity.  Picture an empty aluminum box about 10'' x 5'' x 5''.  We saw damaging breakdown events beginning around 350W at 1.2GHz which is the nominal operating point of our radar.  It took several design iterations and many months to totally eliminate various sources of breakdown including sharp edges, gas trapped by resonator pucks, tuning screws, etc.   While your cavity and ours arn't exactly the same one could say the situations are quite similar.  The NASA Eagleworks system operated at 935MHz at (?)W, Roger Shawyer 2.45GHz at 850W, and Dr Yang at (?)MHz at 2.5KW (apologize if these missing values have been published, I didn't immediately see them).   

In a nutshell, at these power levels I would be surprised if your systems were not multipacting to some degree as designing a cavity that does not have breakdown at these levels takes a good deal of expertise on the nuances of the issue.  So as multipaction events are particle generators these could produce the force you are seeing.  What order of magnitude force we would see I havn't the foggiest.  But if I were an independent reviewer of your technology I would first ask that you prove this cannot be explained by multipaction.  Or show that even if multipaction were occuring the magnitude of the forces involved cannot be explained. These events can be observed by monitoring the RF power level passing through, or in your case reflecting, from a system.  An ideal setup would be to add an RF coupler between your magnetron and the cavity and observe the return loss into the system as power is slowly ramped up.  You will see a reflected power loss as the energy is converted into the events described.  A further test would be to have your resonator opened and carefully inspected by an expert as burn marks and other evidence can be detected optically.   

Good luck, I can pass you some names off-line if that is of interest.   If you havn't already, it would be useful to consult a high-power RF engineer, not necessarily and EM physicist (sorry guys!  :) ).  As stated, I am not an expert on this phenomenon but if there are further questions I can perhaps pass them along.

-Joseph Knuble

(Also, I hope I'm wrong!)
Title: Re: EM Drive Developments - related to space flight applications - Thread 2
Post by: Peter Svancarek on 05/01/2015 06:39 pm
Is this right? This looks to me like bad science. How could thrust fall as speed increases? I also looked to "scientific" paper on emdrive page and there is stated that at about 0.7c thrust reverses?

http://emdrive.com/sciencemissions.html (http://emdrive.com/sciencemissions.html)
 The 15 year thrust period is based on the cathode life of the magnetron. This will be the same technology as that used in space qualified TWTA’s, which are currently specified for 15 years continuous operation.

If the 700 W (dc) engine was used as primary propulsion for a 50 kg science probe, a velocity increment of 5.6 km/sec would be achieved in the first year for a thrust of 88 mN. Thereafter, due to the effects of equation 2, the thrust falls as the velocity increases, until after 15 years, the thrust would be 16 mN at a terminal velocity approaching 30 km/sec.


Title: Re: EM Drive Developments - related to space flight applications - Thread 2
Post by: Rodal on 05/01/2015 06:40 pm
Hi Everyone,

I am an RF engineer in the Microwave Instrument Technology Branch at NASA Goddard Space Flight Center.  I had seen a few articles here and there about the EM drive and today it caught my eye on IO9.com.  While I have only spent the last hour or so reviewing what has been done to eliminated external factors to explain the phenomenon I would like to offer my two-cents.  If what I'm suggesting as an explanation has already been eliminated, I apologize.

Have you considered the effects of breakdown, and in particular multipaction and corona generation?  Multipaction breakdown events are known phenomenon on the RF radar and communication systems community.  Essentially, at high RF powers you see an effect similar to arcing within your components.  This arcing can occur between conductors and dielectrics or even between conductors in vacuum.  Sharp edges such as welds and fasteners - particularly in a cavity resonator such as this - can cause these events.  This result is damage to the interior conductor and particle generation (even in metal-only situations) as material is "burned."  In this case, the metal walls and / or contaminants of your cavity would serve as the propellant.  Corona / plasma can then develop from this particle release and exacerbate the situation.

Better descriptions can be found here : 

http://en.wikipedia.org/wiki/Multipactor_effect

http://www.microwaves101.com/encyclopedias/multipaction 

As a real world example, I am the lead engineer for the Radiometer Front End on the recently launched Soil Moisture Active Passive (SMAP) Mission. (http://smap.jpl.nasa.gov/)  We had a terrible problem with multipaction in our RF diplexer which was a cavity resonator - similar to your setup.  These devices are essentially three-port band-pass filters whose resonant frequencies are set by the physical dimensions of the (mostly) empty cavity.  Picture an empty aluminum box about 10'' x 5'' x 5''.  We saw damaging breakdown events beginning around 350W at 1.2GHz which is the nominal operating point of our radar.  It took several design iterations and many months to totally eliminate various sources of breakdown including sharp edges, gas trapped by resonator pucks, tuning screws, etc.   While your cavity and ours arn't exactly the same one could say the situations are quite similar.  The NASA Eagleworks system operated at 935MHz at (?)W, Roger Shawyer 2.45GHz at 850W, and Dr Yang at (?)MHz at 2.5KW (apologize if these missing values have been published, I didn't immediately see them).   

In a nutshell, at these power levels I would be surprised if your systems were not multipacting to some degree as designing a cavity that does not have breakdown at these levels takes a good deal of expertise on the nuances of the issue.  So as multipaction events are particle generators these could produce the force you are seeing.  What order of magnitude force we would see I havn't the foggiest.  But if I were an independent reviewer of your technology I would first ask that you prove this cannot be explained by multipaction.  Or show that even if multipaction were occuring the magnitude of the forces involved cannot be explained. These events can be observed by monitoring the RF power level passing through, or in your case reflecting, from a system.  An ideal setup would be to add an RF coupler between your magnetron and the cavity and observe the return loss into the system as power is slowly ramped up.  You will see a reflected power loss as the energy is converted into the events described.  A further test would be to have your resonator opened and carefully inspected by an expert as burn marks and other evidence can be detected optically.   

Good luck, I can pass you some names off-line if that is of interest.   If you havn't already, it would be useful to consult a high-power RF engineer, not necessarily and EM physicist (sorry guys!  :) ).  As stated, I am not an expert on this phenomenon but if there are further questions I can perhaps pass them along.

-Joseph Knuble

(Also, I hope I'm wrong!)

Welcome to the thread  :)  We hope you are here to stay  ;)

Please correct me if I'm wrong, but my understanding of this effect is that the multipactor effect occurs "when electrons accelerated by radio-frequency (RF) fields are self-sustained in a vacuum (or near vacuum)"

Quote from:  http://www.microwaves101.com/encyclopedias/multipaction   
The existence of multipaction is dependent on the following four conditions being met:

The mean free path of the electrons should be (much) greater then the spacing between the opposing surfaces, which is normally only the case in good vacuum and without any further obstruction in the way (no other di-electricum).
The average number of electrons released is greater than one which is dependent on the secondary electron yield of the surface, which in turn is dependent on the field strength (RF power) between the surfaces.
The time taken by the electron to travel from the surface from which it was released to the surface it impacts with, is to be an integer multiple of one half of the RF period (resonance).
The availability of free electrons to start of the release of secondary electrons.
(In space, free electrons are released from the surfaces by high energy particles, while during on-ground testing they are provided by a radioactive source (strontium 90) or an electron gun)

The counter-argument to this is that most of the EM Drive experiments in the US, UK and China have been conducted in ambient conditions (not in a vacuum).

The only experiments being conducted in a partial vacuum have been those conducted in the US at NASA Eagleworks since the end of December 2014.
Title: Re: EM Drive Developments - related to space flight applications - Thread 2
Post by: Rodal on 05/01/2015 06:43 pm
Is this right? This looks to me like bad science. How could thrust fall as speed increases? I also looked to "scientific" paper on emdrive page and there is stated that at about 0.7c thrust reverses?

http://emdrive.com/sciencemissions.html (http://emdrive.com/sciencemissions.html)
...

There are several theoretical problems with the emdrive.com theories, as they have been addressed in prior pages of this thread.
Title: Re: EM Drive Developments - related to space flight applications - Thread 2
Post by: LasJayhawk on 05/01/2015 06:53 pm
Hi Everyone,

I am an RF engineer in the Microwave Instrument Technology Branch at NASA Goddard Space Flight Center.  I had seen a few articles here and there about the EM drive and today it caught my eye on IO9.com.  While I have only spent the last hour or so reviewing what has been done to eliminated external factors to explain the phenomenon I would like to offer my two-cents.  If what I'm suggesting as an explanation has already been eliminated, I apologize.

Have you considered the effects of breakdown, and in particular multipaction and corona generation?  Multipaction breakdown events are known phenomenon on the RF radar and communication systems community.  Essentially, at high RF powers you see an effect similar to arcing within your components.  This arcing can occur between conductors and dielectrics or even between conductors in vacuum.  Sharp edges such as welds and fasteners - particularly in a cavity resonator such as this - can cause these events.  This result is damage to the interior conductor and particle generation (even in metal-only situations) as material is "burned."  In this case, the metal walls and / or contaminants of your cavity would serve as the propellant.  Corona / plasma can then develop from this particle release and exacerbate the situation.

Better descriptions can be found here : 

http://en.wikipedia.org/wiki/Multipactor_effect

http://www.microwaves101.com/encyclopedias/multipaction 

As a real world example, I am the lead engineer for the Radiometer Front End on the recently launched Soil Moisture Active Passive (SMAP) Mission. (http://smap.jpl.nasa.gov/)  We had a terrible problem with multipaction in our RF diplexer which was a cavity resonator - similar to your setup.  These devices are essentially three-port band-pass filters whose resonant frequencies are set by the physical dimensions of the (mostly) empty cavity.  Picture an empty aluminum box about 10'' x 5'' x 5''.  We saw damaging breakdown events beginning around 350W at 1.2GHz which is the nominal operating point of our radar.  It took several design iterations and many months to totally eliminate various sources of breakdown including sharp edges, gas trapped by resonator pucks, tuning screws, etc.   While your cavity and ours arn't exactly the same one could say the situations are quite similar.  The NASA Eagleworks system operated at 935MHz at (?)W, Roger Shawyer 2.45GHz at 850W, and Dr Yang at (?)MHz at 2.5KW (apologize if these missing values have been published, I didn't immediately see them).   

In a nutshell, at these power levels I would be surprised if your systems were not multipacting to some degree as designing a cavity that does not have breakdown at these levels takes a good deal of expertise on the nuances of the issue.  So as multipaction events are particle generators these could produce the force you are seeing.  What order of magnitude force we would see I havn't the foggiest.  But if I were an independent reviewer of your technology I would first ask that you prove this cannot be explained by multipaction.  Or show that even if multipaction were occuring the magnitude of the forces involved cannot be explained. These events can be observed by monitoring the RF power level passing through, or in your case reflecting, from a system.  An ideal setup would be to add an RF coupler between your magnetron and the cavity and observe the return loss into the system as power is slowly ramped up.  You will see a reflected power loss as the energy is converted into the events described.  A further test would be to have your resonator opened and carefully inspected by an expert as burn marks and other evidence can be detected optically.   

Good luck, I can pass you some names off-line if that is of interest.   If you havn't already, it would be useful to consult a high-power RF engineer, not necessarily and EM physicist (sorry guys!  :) ).  As stated, I am not an expert on this phenomenon but if there are further questions I can perhaps pass them along.

-Joseph Knuble

(Also, I hope I'm wrong!)
My understanding is the current setup is a little over 1.9 GHz @ 50 watts.

From what I can find out online, the end caps of the cavity are single sided FR4, but I couldn't find out if they were baked and post coated or not. Do you think they could be getting some out gassing or other effects from the FR4? Would glass Teflon be a better choice of dielectric?
Title: Re: EM Drive Developments - related to space flight applications - Thread 2
Post by: jknuble on 05/01/2015 06:53 pm
Good luck, I can pass you some names off-line if that is of interest.   If you havn't already, it would be useful to consult a high-power RF engineer, not necessarily and EM physicist (sorry guys!  :) ).  As stated, I am not an expert on this phenomenon but if there are further questions I can perhaps pass them along.

-Joseph Knuble

(Also, I hope I'm wrong!)

Welcome to the thread  :)  We hope you are here to stay  ;)

Please correct me if I'm wrong, but my understanding of this effect is that the multipactor effect occurs "when electrons accelerated by radio-frequency (RF) fields are self-sustained in a vacuum (or near vacuum)"

The counter-argument to this is that most of the EM Drive experiments have been conducted in ambient conditions (not in a vacuum).

I may have overly used the word "multipaction" as that is what primarily plagued us.  Multipaction is one type of "high-power breakdown" and you can see different types of breakdown in air or vacuum.  Testing a system in one or the other can have differing results.  There could also be plasma generation for instance.  The "better" sources I have provided may be overly academic.

-JK
Title: Re: EM Drive Developments - related to space flight applications - Thread 2
Post by: Andy USA on 05/02/2015 12:05 am
Ok guys, so a lot of new people into this thread, but this thread is mainly for the development of the EM Drive. We knew this would happen, so we have a new "Entry Level" thread for opening questions and general questions.

I've moved the last few pages of new members asking questions into that thread, so if you posted here and can't see it, don't worry, it's in this thread.

http://forum.nasaspaceflight.com/index.php?topic=37438.0

Posting this notice as some sites are linking to this thread and not the section or the article, so people are thinking this is the only thread on this.

Remember to use the above link and allow this thread to continue with the Eaglework folk and others updating progress.
Title: Re: EM Drive Developments - related to space flight applications - Thread 2
Post by: R.W. Keyes on 05/02/2015 01:55 am
Ok guys, so a lot of new people into this thread, but this thread is mainly for the development of the EM Drive. We knew this would happen, so we have a new "Entry Level" thread for opening questions and general questions.

I've moved the last few pages of new members asking questions into that thread, so if you posted here and can't see it, don't worry, it's in this thread.

http://forum.nasaspaceflight.com/index.php?topic=37438.0

Posting this notice as some sites are linking to this thread and not the section or the article, so people are thinking this is the only thread on this.

Remember to use the above link and allow this thread to continue with the Eaglework folk and others updating progress.

I think we need a F.A.Q.
Title: Re: EM Drive Developments - related to space flight applications - Thread 2
Post by: Rodal on 05/02/2015 02:19 am
Ok guys, so a lot of new people into this thread, but this thread is mainly for the development of the EM Drive. We knew this would happen, so we have a new "Entry Level" thread for opening questions and general questions.

I've moved the last few pages of new members asking questions into that thread, so if you posted here and can't see it, don't worry, it's in this thread.

http://forum.nasaspaceflight.com/index.php?topic=37438.0

Posting this notice as some sites are linking to this thread and not the section or the article, so people are thinking this is the only thread on this.

Remember to use the above link and allow this thread to continue with the Eaglework folk and others updating progress.

I think we need a F.A.Q.

Yes, if the EM Drive thread would be about well-understood science and/or engineering, an FAQ section would be great.

We all want answers, that's why we are here.

Unfortunately there is substantial amount of disagreement as to the measurements and even more disagreement about the theories.

Who is going to be in charge of answering the questions?  Are (maybe innocently incorrect or innocently misleading) answers provided by forum volunteers for an unsettled, controversial, Research project like the EM Drive more useful than not having an FAQ section?

How can a FAQ section for unsettled Research in a forum be objectively handled for scientific/engineering accuracy?
Who will give an authoritative answer in a forum?
How and how often are answers going to be updated?
How is misinformation going to be prevented? 
How is agreement on what is the correct answer to a question going to be reached?

Title: Re: EM Drive Developments - related to space flight applications - Thread 2
Post by: ppnl on 05/02/2015 02:57 am
Ok guys, so a lot of new people into this thread, but this thread is mainly for the development of the EM Drive. We knew this would happen, so we have a new "Entry Level" thread for opening questions and general questions.

I've moved the last few pages of new members asking questions into that thread, so if you posted here and can't see it, don't worry, it's in this thread.

http://forum.nasaspaceflight.com/index.php?topic=37438.0

Posting this notice as some sites are linking to this thread and not the section or the article, so people are thinking this is the only thread on this.

Remember to use the above link and allow this thread to continue with the Eaglework folk and others updating progress.

I think we need a F.A.Q.

Yes, if the EM Drive thread would be about well-understood science and/or engineering, an FAQ section would be great.

We all want answers, that's why we are here.

Unfortunately there is substantial amount of disagreement as to the measurements and even more disagreement about the theories.

Who is going to be in charge of answering the questions?  Are (maybe innocently incorrect or innocently misleading) answers provided by forum volunteers for an unsettled, controversial, Research project like the EM Drive more useful than not having an FAQ section?

How can a FAQ section for unsettled Research in a forum be objectively handled for scientific/engineering accuracy?
Who will give an authoritative answer in a forum?
How is misinformation going to be prevented? 
How is agreement on what is the correct answer to a question going to be reached?

Maybe you should elect the most hardcore and knowledgeable proponent and the most hardcore and knowledgeable critic and produce two answers to each question.
Title: Re: EM Drive Developments - related to space flight applications - Thread 2
Post by: Rodal on 05/02/2015 03:04 am
Ok guys, so a lot of new people into this thread, but this thread is mainly for the development of the EM Drive. We knew this would happen, so we have a new "Entry Level" thread for opening questions and general questions.

I've moved the last few pages of new members asking questions into that thread, so if you posted here and can't see it, don't worry, it's in this thread.

http://forum.nasaspaceflight.com/index.php?topic=37438.0

Posting this notice as some sites are linking to this thread and not the section or the article, so people are thinking this is the only thread on this.

Remember to use the above link and allow this thread to continue with the Eaglework folk and others updating progress.

I think we need a F.A.Q.

Yes, if the EM Drive thread would be about well-understood science and/or engineering, an FAQ section would be great.

We all want answers, that's why we are here.

Unfortunately there is substantial amount of disagreement as to the measurements and even more disagreement about the theories.

Who is going to be in charge of answering the questions?  Are (maybe innocently incorrect or innocently misleading) answers provided by forum volunteers for an unsettled, controversial, Research project like the EM Drive more useful than not having an FAQ section?

How can a FAQ section for unsettled Research in a forum be objectively handled for scientific/engineering accuracy?
Who will give an authoritative answer in a forum?
How is misinformation going to be prevented? 
How is agreement on what is the correct answer to a question going to be reached?

Maybe you should elect the most hardcore and knowledgeable proponent and the most hardcore and knowledgeable critic and produce two answers to each question.
Even assuming that an FAQ with opposing answers (*) to the questions would be helpful, having somebody choose who is " the most hardcore and knowledgeable" would result in a certain amount of arbitrariness, and  the main problem is keeping such FAQ updated with correct information, because it often turns out to be the case that  " the most knowledgeable" doesn't have the time to update the FAQ section and often doesn't volunteer, and those who readily volunteer are not " the most knowledgeable"  :(

_________
(*) Therefore the possibilities are that either 1) one of the answers is wrong or,  2) both answers are wrong.  (Since they are opposing answers, they cannot both be true). In the first case, one of the answers is misleading and it confuses the reader, and in the second case there is no point to the FAQ section to provide those answers as both are wrong.  In my opinion it is always better to give no answer than to give two answers, at least one of which we know (in the case of opposing answers) must be an incorrect answer. 
Title: Re: EM Drive Developments - related to space flight applications - Thread 2
Post by: SH on 05/02/2015 04:06 am
We don't need a FAQ.  The collection of manuscripts and reports written by White's team and everyone else, as well as the Wikipedia article and other writeups all serve as a FAQ.  Perhaps what we should have, though, is a comprehensive listing of all the various sources of information that have been published or reported, as well as unbiased accountings including the wiki article and Rhodal's summary.

Also, it would certainly help if some people bothered to update the Wikipedia article...as it is in many ways incorrect, out of date, and lacking a lot of the known references and possible explanations.  All of this would help provide context to incoming scientists wishing to look at this issue.
Title: Re: EM Drive Developments - related to space flight applications - Thread 2
Post by: R.W. Keyes on 05/02/2015 04:30 am
The Wikipedia article, last time I looked at it, seemed to consist mostly of quotes of famous scientists saying it was a fraud. I don't know if I'd want to wander into the political storm of wikipedia in this.

Perhaps many questions can't be take care of with a FAQ. But perhaps some can. Perhaps some questions can be answered with "there's no widely accepted answer, but these people have proposed the following, and these other people have found fault", without getting to deep into the finer points of the disagreement.

But perhaps instead of a FAQ, we need a wiki. I don't know how many people share my evaluation of Wikipedia as a political battleground, though. I've only created one wikipedia entry (Botryococcus braunii), and done my best to keep it accurate and organized, though perhaps it is now out of date.

The need I am trying to fill, is a way to satisfy the many newcomers here who haven't gone through the voluminous and sometimes difficult to follow threads. I am not wedded to any particular approach, I merely want to do my part to help out, whatever that may be.
Title: Re: EM Drive Developments - related to space flight applications - Thread 2
Post by: R.W. Keyes on 05/02/2015 04:49 am
This is a question for Eagleworks people:
It seems as though you are short of resources of various types. I propose an online petition, utilizing the "we the people" facility of whitehouse.gov, to ask for more resources to be given to NASA, earmarked for Eagleworks, for the projects of the EM-drive/ME-drive/Q-thruster, and its application to the Alcubierre drive. If you could come up with a proposal, perhaps more focused than I've indicated, this would be the meat of the petition.

I don't mean to over the heads of the NASA Administration, and force them to take money away from other programs for this one. This would be additional, federal money. In my experience, being given more resources than you can use raises expectations beyond what is achievable. Most often, you can't get a project done twice as fast by spending twice as much money on it.

If Eagleworks thinks this is a bad approach to obtaining additional resources, please let me know why.

With great respect,
Robert W. Keyes
Title: Re: EM Drive Developments - related to space flight applications - Thread 2
Post by: NovaSilisko on 05/02/2015 05:05 am
additional, federal money.

If that's the case, I don't see it materializing, even if it's a pittance...
Title: Re: EM Drive Developments - related to space flight applications - Thread 2
Post by: SH on 05/02/2015 05:22 am
If Eagleworks thinks this is a bad approach to obtaining additional resources, please let me know why.

Honestly, it would reflect poorly on the Eagleworks team if they did respond with a shopping list of resources...because it would give the impression that NASA wasn't sufficiently supplying them to do their job, and that they were trying to violate the chain of command so to speak.

I imagine that there have already been some tense conversations between Dr. White and NASA leadership.  Most likely, NASA leadership is really not liking all the publicity about this right about now...because NASA knows (or assumes) that this is most likely just a fluke, and that when this finally does get refuted, it's going to reflect very poorly on NASA for having gone on so long, with all the news articles saying "NASA made a warp drive" and "NASA doesn't believe in conservation of energy anymore," and that's going to make it very easy for politicians who don't like NASA to cut their funding even more.

I'm sure that they would rather move Eagleworks into a secret team, but it's too late for that now...so if this EmDrive is invalidated, don't be surprised if the Eagleworks team gets the axe.
Title: Re: EM Drive Developments - related to space flight applications - Thread 2
Post by: Prunesquallor on 05/02/2015 05:27 am
This is a question for Eagleworks people:
It seems as though you are short of resources of various types. I propose an online petition, utilizing the "we the people" facility of whitehouse.gov, to ask for more resources to be given to NASA, earmarked for Eagleworks, for the projects of the EM-drive/ME-drive/Q-thruster, and its application to the Alcubierre drive. If you could come up with a proposal, perhaps more focused than I've indicated, this would be the meat of the petition.

I don't mean to over the heads of the NASA Administration, and force them to take money away from other programs for this one. This would be additional, federal money. In my experience, being given more resources than you can use raises expectations beyond what is achievable. Most often, you can't get a project done twice as fast by spending twice as much money on it.

If Eagleworks thinks this is a bad approach to obtaining additional resources, please let me know why.

With great respect,
Robert W. Keyes

I do not represent EagleWorks, but...

The Administration is the wrong place to do this.  The President can request a budget, but he does not allocate NASA's money. Congress does. What you're proposing is indeed an earmark and it would only be seen as valuable to Texas politicians.  That's fine - it is done all the time, but the Texas senators and the JSC congressional district representatives are the ones you want to convince.

And you don't want to ask for "new money".  There isn't any.  This is such a pittance it is the rounding error from NASA's big programs.
Title: Re: EM Drive Developments - related to space flight applications - Thread 2
Post by: R.W. Keyes on 05/02/2015 08:27 am
These seem to be depressive, defeatist, Kafkaesque responses. I am going to wait to see what people who actually work at Eagleworks say. $25M (just picking a reasonable but arbitrary amount here) might to a lot of money Eagleworks, but for the federal government, that's pin money. Even in a word where governments are overrun by corruption, pettiness, and mismanagement, that doesn't mean that all parts are, all the time.
Title: Re: EM Drive Developments - related to space flight applications - Thread 2
Post by: dustinthewind on 05/02/2015 08:36 am
I am assuming the microwaves path is something like this?

(http://i.imgur.com/nA1U9X6.png)

Where forces F0 through F16 represent the force of a single microwave at different time intervals.

If the directional force happens at F2, and the opposing force happens from F3 to F16, this still obeys Newtons third law.  Isn't the opposing force just being damped like a spring over time? 

If M1, M2, ... Mn represent different microwaves, and each Mi is following a similar course, wouldn't this suggest that the directional force will remain positive at a constant rate until the device is shut off causing all forces to cancel?

 

Not according to Maxwell's linear, isotropic equations.

The small base of the EM Drive is not open.  It is a closed cavity.  As such, the waves inside it are not travelling waves, but standing waves.   See this:  http://en.wikipedia.org/wiki/Standing_wave

The energy flux is pointed towards one end during half a (Poynting vector) period and it is pointed towards the opposite end during the next half-period.  Hence the net energy flux over a whole period is completely self-cancelling.

No net directional thrust for a microwave closed cavity can be explained just using Maxwell's linear, isotropic equations.  If the measurements are not an experimental artifact, another explanation is needed.

If the small base would be open, then it would be an inefficient photon rocket, with the microwave photons escaping at the small base end.  Less efficient for space propulsion than using a flashlight or a military searchlight as a means of propulsion.

Even for a perfectly collimated photon rocket, the thrust/powerInput of such a photon rocket is orders of magnitude less than what is claimed for these experiments.

With all respect, and I am not sure this is much of an issue to point out but I think the model of the waves in the cavity as standing waves may be a bit over-simplified. 

1. I think one can look at standing waves as super-imposed traveling waves of opposite direction but this part is likely less significant. 

2.  Maybe of more significance is the ac power in power lines can be modeled as standing waves but if no one is using power.  When power starts being consumed the standing waves begin to travel towards the object consuming the power.  The moving bulges of magnetic/electric field can be thought of as transporting power from the power station to the consumer.  There should be some traveling of the standing waves bulges from the power supply towards areas of heat loss in the cavity I would assume.  I can't say the power dissipated into heat loss is significant but it does seem to buck the perfect standing wave view for me a bit.  I do understand on the other hand that it may be a good approximation. 
Title: Re: EM Drive Developments - related to space flight applications - Thread 2
Post by: Mulletron on 05/02/2015 09:41 am
I am assuming the microwaves path is something like this?

(http://i.imgur.com/nA1U9X6.png)

Where forces F0 through F16 represent the force of a single microwave at different time intervals.

If the directional force happens at F2, and the opposing force happens from F3 to F16, this still obeys Newtons third law.  Isn't the opposing force just being damped like a spring over time? 

If M1, M2, ... Mn represent different microwaves, and each Mi is following a similar course, wouldn't this suggest that the directional force will remain positive at a constant rate until the device is shut off causing all forces to cancel?

 

Not according to Maxwell's linear, isotropic equations.

The small base of the EM Drive is not open.  It is a closed cavity.  As such, the waves inside it are not travelling waves, but standing waves.   See this:  http://en.wikipedia.org/wiki/Standing_wave

The energy flux is pointed towards one end during half a (Poynting vector) period and it is pointed towards the opposite end during the next half-period.  Hence the net energy flux over a whole period is completely self-cancelling.

No net directional thrust for a microwave closed cavity can be explained just using Maxwell's linear, isotropic equations.  If the measurements are not an experimental artifact, another explanation is needed.

If the small base would be open, then it would be an inefficient photon rocket, with the microwave photons escaping at the small base end.  Less efficient for space propulsion than using a flashlight or a military searchlight as a means of propulsion.

Even for a perfectly collimated photon rocket, the thrust/powerInput of such a photon rocket is orders of magnitude less than what is claimed for these experiments.

With all respect, and I am not sure this is much of an issue to point out but I think the model of the waves in the cavity as standing waves may be a bit over-simplified. 

1. I think one can look at standing waves as super-imposed traveling waves of opposite direction but this part is likely less significant. 

2.  Maybe of more significance is the ac power in power lines can be modeled as standing waves but if no one is using power.  When power starts being consumed the standing waves begin to travel towards the object consuming the power.  The moving bulges of magnetic/electric field can be thought of as transporting power from the power station to the consumer.  There should be some traveling of the standing waves bulges from the power supply towards areas of heat loss in the cavity I would assume.  I can't say the power dissipated into heat loss is significant but it does seem to buck the perfect standing wave view for me a bit.  I do understand on the other hand that it may be a good approximation.

In ref to the image above, there is wealth of information buried within the earlier pages of this thread. A more accurate representation of what it looks like inside the cavity is available here:
http://forum.nasaspaceflight.com/index.php?topic=36313.msg1333246#msg1333246
Paul March was kind enough to attach the Frustrum modes overview 2A.pdf which has all the mode shapes and characteristics of their test article.

So things are a bit more complicated than photons bouncing around like marbles in a can. For example, I know that I can only excite TM212 and TM311 (thanks @Rodal for modeling this) with my little setup at home.
http://forum.nasaspaceflight.com/index.php?topic=36313.msg1353372#msg1353372
http://forum.nasaspaceflight.com/index.php?topic=36313.msg1352878#msg1352878

Paul March has shown clearly in his many posts that there is a clear correlation between mode shape and magnitude and direction of thrust. This is where input from RF Engineers would be extremely valuable.

Just food for thought, it is worth going back to page 1 and commenting on the stuff starting there. That way the conversation can keep building on ideas.

(http://forum.nasaspaceflight.com/index.php?action=dlattach;topic=36313.0;attach=820102;image)
(http://forum.nasaspaceflight.com/index.php?action=dlattach;topic=36313.0;attach=820104;image)
(http://forum.nasaspaceflight.com/index.php?action=dlattach;topic=36313.0;attach=796989;image)
Title: Re: EM Drive Developments - related to space flight applications - Thread 2
Post by: Stoney on 05/02/2015 10:04 am
Have dielectrophoretic (and perhaps magnetophoretic?) effects been ruled out? It seems to me that you have a lot of the ingredients (high voltage, RF, non-uniform field) for a dielectrophoretic force to appear on any surrounding bulk material that may behave like a dielectric at these frequencies...

Also, how is the thrust being measured? (I apologise as this has probably been mentioned earlier but I'm struggling to find it in >100 pages of discussion)
Title: Re: EM Drive Developments - related to space flight applications - Thread 2
Post by: Mulletron on 05/02/2015 10:35 am
Dr. Rodal:

I missed your question last night on whether the warp-field interferometer cylindrical cavity had any dielectric inside of it.  The answer is no it does not, except for the nanometers thick aluminum oxide coating that bare aluminum develops as soon as it is exposed to the oxygen in the air.

Next you asked about whether there where optical windows cut into the center of the cylindrical resonant cavity end caps or not.  Well, yes there has to be optical window holes for the 633nm laser light to pass through the 7.23cm gap between the endplates of the aluminum cylindrical cavity.  We also added two, three inch long, 0.50" OD by 0.25" (6.35mm) ID threaded aluminum tubes to the resonant cavity endplates, see attached picture, that function as two RF chokes that keep the 1.48 GHz RF from leaking into the lab area.  So the laser light passes through these RF choke tubes and the cylindrical cavity where the peak ac E-field of 900kV/m is present along the entire 7.23 cm long laser path while in the resonant cavity and an exponentially reducing E-field in the RF chokes since these are cylindrical waveguides well into their cutoff mode since the RF wavelength at 1.48 GHz is 202.7mm. 

BTW, we are going to add optical borosilicate telescope grade flat windows to the ends of the RF chokes when we get around to pulling a vacuum in this 1.48 GHz resonant cavity.

Next a clarification.  We used a cylindrical cavity for the warp-field interferometer instead of a frustum shape because we didn't want to create a force with this unit, but instead we needed just a large densification of the Q-V along the active path length of the laser beam while it was traversing the resonant cavity's centerline volume.  And this is the main difference between the Q-thruster and a warp-drive.  In Dr. White's warp-field conjecture you first have to have an Q-Thruster derived acceleration vector to work on and then you engage the a toroidal warp-field around your accelerating vehicle that then multiples the initial Q-Thruster provided velocity vector by the selected warp-factor.   Thus if you have an initial velocity of say 0.01c towards Alpha Centauri with a warp factor of 1,000, your effective velocity becomes 10c while the warp-drive is engaged.

Best, Paul M.

Paul March, have you seen this?

http://arxiv.org/abs/1101.0712
http://arxiv.org/abs/1101.1174
http://www2.cnrs.fr/en/1859.htm
http://phys.org/news/2011-05-when-the-speed-of-light.html



Mulletron:

No I hadn't but thanks for the pointers.  So what to you think an asymmetric difference of ~1x10^-18 m/s in velocity of light bring to the table?  That the vacuum can be differentially polarized by applied E and B-fields in a volume, in this case dc E&M fields??

Best, Paul M.

Their results seem to support what you guys are reporting from your open air experiments, which is a win, but I don't think we can call this length contraction (even though it might look like it) for sure until the same results are in repeated in vacuum.

Might be of interest to somebody. Another researcher predicting the possibility. Still yet to be verified as far as I know. The references are good leads too.

From all the research I've found, the Europeans are way ahead of the USA on this in so many ways.

https://www.ostfalia.de/export/sites/default/de/pws/turtur/FundE/English/Licht_in_elmagn_Feldern_engl.pdf
Title: Re: EM Drive Developments - related to space flight applications - Thread 2
Post by: GregoryNemitz on 05/02/2015 11:43 am
"Dean Drive" mechanics can offer an insight to EM Drive function.

The Dean Drive is a mechanical device proposed and experimented in the 1960s.  It could never work as hoped because it would need to operate at frequencies that will tear a mechanical device apart.

Essentially, it requires force waves in solid objects that get out of phase with each other due to one path being longer and the other path being shorter.  When energy is put into the end of a solid object to move it, the object will not begin to move until the force wave reaches the long end of the object and it is reflected back.  The Dean Drive proposed a system when the direct path and the reflected path were different lengths so an out of phase relationship would occur and the object would move as a system, reactionlessly.

With the EM Drive the electromagnetic energy from the microwaves act upon the inner surface of the cone like a solar sail.  The reflected energy is pushing on the surfaces.

Since it is a cone, towards the center is the long path and towards the edges is the short path.  This creates an imbalance in the push on the system, just like the Dean Drive was proposed to work.  Except it is working at gigahertz frequencies, far above what will tear a mechanical Dean Drive apart.
Title: Re: EM Drive Developments - related to space flight applications - Thread 2
Post by: Prunesquallor on 05/02/2015 11:55 am
These seem to be depressive, defeatist, Kafkaesque responses. I am going to wait to see what people who actually work at Eagleworks say. $25M (just picking a reasonable but arbitrary amount here) might to a lot of money Eagleworks, but for the federal government, that's pin money. Even in a word where governments are overrun by corruption, pettiness, and mismanagement, that doesn't mean that all parts are, all the time.

Maybe, but the risk of unintended consequences is high, IMHO. Given the type of publicity this effort garners, would the following headline be unrealistic?:
"President asks Congress to fund Warp Drive development".
I expect the response could be unfortunate.
Title: Re: EM Drive Developments - related to space flight applications - Thread 2
Post by: Mulletron on 05/02/2015 12:27 pm
"Dean Drive" mechanics can offer an insight to EM Drive function.

-snip-


http://www.npl.washington.edu/AV/altvw83.html

Quote
Later more detailed studies showed that the Dean Drive developed no net time- averaged force and that Newton's 3rd Law remained intact.
Title: Re: EM Drive Developments - related to space flight applications - Thread 2
Post by: Jim on 05/02/2015 02:11 pm
This is a question for Eagleworks people:
It seems as though you are short of resources of various types. I propose an online petition, utilizing the "we the people" facility of whitehouse.gov, to ask for more resources to be given to NASA, earmarked for Eagleworks, for the projects of the EM-drive/ME-drive/Q-thruster, and its application to the Alcubierre drive. If you could come up with a proposal, perhaps more focused than I've indicated, this would be the meat of the petition.

I don't mean to over the heads of the NASA Administration, and force them to take money away from other programs for this one. This would be additional, federal money. In my experience, being given more resources than you can use raises expectations beyond what is achievable. Most often, you can't get a project done twice as fast by spending twice as much money on it.


That is completely the wrong way to go at it.   Why should they be singled out?  what says they are worthy of it?  If anything, it should be a competition for the resources. 
Title: Re: EM Drive Developments - related to space flight applications - Thread 2
Post by: rgreen on 05/02/2015 03:28 pm
Following on what jknuble said about the multipactor-like effect as a possible cause of thrust. http://en.wikipedia.org/wiki/Multipactor_effect  I can't help but wonder about what's going on with the copper surface of the frustum. A quick back of the envelope (well, python) calculation shows that there's certainly enough energy in these devices to somehow atomize a small amount of copper , and propel them with enough momentum to produce a small amount of thrust.

For example, a 30 watt emdrive where 0.001% of the energy went towards atomization and 1% went toward addtional momentum of the particles... You'd have a device with 91uN thrust, propelling 1.4ng of copper a second at 65500m/s.

I can think of 3 ways to debunk this. 1) perhaps that amount of particles going that fast would be noticeable with the naked eye, so this isn't really a valid explanation. 2) stick a detector behind the thruster (are they ionized?). 3) SEM of the surface compared to scraps from the same batch of copper not used in the thrustum.
Title: Re: EM Drive Developments - related to space flight applications - Thread 2
Post by: Mulletron on 05/02/2015 03:51 pm
Just want to run this by the group.

I am a believer that thrust doesn't scale ONLY with Q. We even can see that in the data. See the original Nasa paper.
http://www.libertariannews.org/wp-content/uploads/2014/07/AnomalousThrustProductionFromanRFTestDevice-BradyEtAl.pdf

I think I have a good idea for once. I think the "Where is the balanced gain and loss?" thing from the other day is addressed by creating an unstable cavity, aka not high Q, not low Q either.

The balanced gain and loss stuff came up here. 4th-6th links from top.
http://forum.nasaspaceflight.com/index.php?topic=36313.msg1357829#msg1357829

I think the trick is to get energy in, put it to work a few thousand times (doing all that quantum wizardry I posted papers on :) ), then let it go as heat, which will inevitably happen as photons are red shifted and fall out of resonance. High Q is a baddie. Low Q is a baddie.

What's the point of having all that accumulated energy sitting in there static, doing nothing?

We need this thing to ride the razor's edge between gain and loss.

Also, what made this kinda click with me is what Mr. Shawyer said below. The Cullen paper he mentioned is shared here:
https://drive.google.com/folderview?id=0B4PCfHCM1KYoTXhSUTd5ZDN2WnM&amp;usp=sharing

So if this passes the smell test, how is the next question. Seems like not having the dielectric covering the entire small end (vs just a small patch) might be a good thing to try. I'm sure there's a ton of ways to do this.

There's a lot we can learn from that whispering-gallery research cited.
http://revolution-green.com/optics-breakthrough-demonstrates-new-behaviors-physics/
http://www.researchgate.net/publication/262451086_Paritytime-symmetric_whispering-gallery_microcavities
http://arxiv.org/ftp/arxiv/papers/1308/1308.4564.pdf

I'm openly brainstorming here. Would like some feedback.
Title: Re: EM Drive Developments - related to space flight applications - Thread 2
Post by: inquisitive-j on 05/02/2015 04:02 pm
I am new to this forum and hesitant to post since my expertise is not in a relevant field of science, but I am excited about this research and have read all pages of comments since page 80 though most were admittedly over my head. Please forgive me if my ignorance shows too much in comments to come. But right now I'm just curious about this article.

I received an article last night via google alert that said that the interferometer test, not the thrust tests, had now been replicated in a vacuum. Is this true? I've seen no mention of it here though perhaps I just missed it. Here is the link: http://www.escapistmagazine.com/news/view/140670-NASA-Confirms-The-EMDrive-Warp-Field-Still-Generates-Works-In-A-Vacuum.
Title: Re: EM Drive Developments - related to space flight applications - Thread 2
Post by: richardcampos on 05/02/2015 04:10 pm
Let's say this all works as claimed by Shawyer, and there is indeed scability and improved performance in high Qs.

Theoretically, what would be the smallest size possible for a Q-thrust device?

Rick
Title: Re: EM Drive Developments - related to space flight applications - Thread 2
Post by: Stormbringer on 05/02/2015 05:02 pm
the limit would probably be the soft to medium X-ray range? because the smaller the frustrum the higher the frequency required to obtain resonance.
Title: Re: EM Drive Developments - related to space flight applications - Thread 2
Post by: richardcampos on 05/02/2015 05:17 pm
So we're talking about something on the intracellular level...
Title: Re: EM Drive Developments - related to space flight applications - Thread 2
Post by: Stormbringer on 05/02/2015 05:28 pm
I know there also has to be room inside the frustrum to establish various resonant modes and that x rays begin to heat metal particularly the higher energy ones. I do not know the actual minimal dimensions needed to establish the resonant modes or that won't be melted or compromised by the x rays so I am just going to use the force and say you probably cannot go down to IC chip level though because of the one or both of the above issues.
Title: Re: EM Drive Developments - related to space flight applications - Thread 2
Post by: Thutmose on 05/02/2015 05:30 pm
Let's say this all works as claimed by Shawyer, and there is indeed scability and improved performance in high Qs.

Theoretically, what would be the smallest size possible for a Q-thrust device?

Rick

the physical size should be directly related to the wavelength of the microwaves used, so it can theoretically be scaled down as small as you want, until you start hitting engineering limits of precisely manufacturing the correctly shaped cavity, and generating high power microwaves of very high frequencies
Title: Re: EM Drive Developments - related to space flight applications - Thread 2
Post by: LasJayhawk on 05/02/2015 05:56 pm
Star-Drive, is your current test setup the same as shown in figure 17 of the AIAA paper from July last year, but with the RG-8 cable? If so, you might have issues with the cable. Most high quality RG-8 has a  manufacturers minimum bend radius of 4" and can develop some "entertaining" behavior in short order if bent beyond that. You might want to check the cable in its current shape on a network analyzer to verify performance.

If the cable is degrading, you might want to look at something like Gore phaseflex cable in that spot. It would give you the flexibility of the RG-142 with the RF performance of the RG-8, and better repeatablity to boot.
Title: Re: EM Drive Developments - related to space flight applications - Thread 2
Post by: Dmytry on 05/02/2015 06:01 pm
Star-Drive, is your current test setup the same as shown in figure 17 of the AIAA paper from July last year, but with the RG-8 cable? If so, you might have issues with the cable. Most high quality RG-8 has a  manufacturers minimum bend radius of 4" and can develop some "entertaining" behavior in short order if bent beyond that. You might want to check the cable in its current shape on a network analyzer to verify performance.

If the cable is degrading, you might want to look at something like Gore phaseflex cable in that spot. It would give you the flexibility of the RG-142 with the RF performance of the RG-8, and better repeatablity to boot.
A very high frequency microwave resonant cavity is commonly known as a "laser".
Title: Re: EM Drive Developments - related to space flight applications - Thread 2
Post by: Dmytry on 05/02/2015 06:02 pm
Following on what jknuble said about the multipactor-like effect as a possible cause of thrust. http://en.wikipedia.org/wiki/Multipactor_effect  I can't help but wonder about what's going on with the copper surface of the frustum. A quick back of the envelope (well, python) calculation shows that there's certainly enough energy in these devices to somehow atomize a small amount of copper , and propel them with enough momentum to produce a small amount of thrust.

For example, a 30 watt emdrive where 0.001% of the energy went towards atomization and 1% went toward addtional momentum of the particles... You'd have a device with 91uN thrust, propelling 1.4ng of copper a second at 65500m/s.

I can think of 3 ways to debunk this. 1) perhaps that amount of particles going that fast would be noticeable with the naked eye, so this isn't really a valid explanation. 2) stick a detector behind the thruster (are they ionized?). 3) SEM of the surface compared to scraps from the same batch of copper not used in the thrustum.
4: enclose the device (complete with the microwave source and, preferably, batteries, provided the pendulum is not too sensitive to shifts in CoM) in a sealed box. Then you can even run your microwave cavity in pressurized SF4 if that helps.
Title: Re: EM Drive Developments - related to space flight applications - Thread 2
Post by: Rodal on 05/02/2015 06:34 pm
This thread is for EM Drive DEVELOPMENT, related to space flight applications.

THERE ARE A NUMBER OF INQUIRING POSTS HERE LATELY that instead belong on this thread:

http://forum.nasaspaceflight.com/index.php?topic=37438.280



Title: Re: EM Drive Developments - related to space flight applications - Thread 2
Post by: squid on 05/02/2015 07:09 pm
Just want to run this by the group.

I am a believer that thrust doesn't scale ONLY with Q. We even can see that in the data. See the original Nasa paper.
http://www.libertariannews.org/wp-content/uploads/2014/07/AnomalousThrustProductionFromanRFTestDevice-BradyEtAl.pdf

I think I have a good idea for once. I think the "Where is the balanced gain and loss?" thing from the other day is addressed by creating an unstable cavity, aka not high Q, not low Q either.

The balanced gain and loss stuff came up here. 4th-6th links from top.
http://forum.nasaspaceflight.com/index.php?topic=36313.msg1357829#msg1357829

I think the trick is to get energy in, put it to work a few thousand times (doing all that quantum wizardry I posted papers on :) ), then let it go as heat, which will inevitably happen as photons are red shifted and fall out of resonance. High Q is a baddie. Low Q is a baddie.

What's the point of having all that accumulated energy sitting in there static, doing nothing?

We need this thing to ride the razor's edge between gain and loss.

Also, what made this kinda click with me is what Mr. Shawyer said below. The Cullen paper he mentioned is shared here:
https://drive.google.com/folderview?id=0B4PCfHCM1KYoTXhSUTd5ZDN2WnM&amp;usp=sharing

So if this passes the smell test, how is the next question. Seems like not having the dielectric covering the entire small end (vs just a small patch) might be a good thing to try. I'm sure there's a ton of ways to do this.

There's a lot we can learn from that whispering-gallery research cited.
http://revolution-green.com/optics-breakthrough-demonstrates-new-behaviors-physics/
http://www.researchgate.net/publication/262451086_Paritytime-symmetric_whispering-gallery_microcavities
http://arxiv.org/ftp/arxiv/papers/1308/1308.4564.pdf

I'm openly brainstorming here. Would like some feedback.

Shawyer's explanation does not pass the smell test, and is not adressed by the Cullen paper you linked. Look for example at figure 5. There is a movable piston at the end of the waveguide T-junction, which is subject to radiation pressure. The piston will exert a (Newton's 3rd law) reaction force, and so momentum is conserved.

The EM drive is a fully enclosed cavity. The radiation inside will reflect off the walls and create some strain in the copper, but the net force integrated by the surface (given by the integral of the Poynting vector) has a time average of 0, as has been demonstrated mathematically many many times.

It is absolutely true that one can view standing waves as linear superpositions of traveling waves. This is just a different way of saying that Maxwell's equations are linear. Rodal's analysis is true whether one thinks of the fields as standing or superpositions of traveling waves. There is nothing besides the stress-energy tensor in the classical theory of electromagnetism.

To be clear: there is NO explanation for any increase in momentum of the drive to be found in classical theory (including Special Relativity through Maxwell's equations).

If there is an actual effect, then it must be caused by the coupling of electromagnetic fields to some other heretofore unobserved field. Even if such a coupling could be made in a way that is Lorentz invariant, it should have been detectable very easily at particle colliders. So again I ask:

If there is some effect here, why has it not been observed in far more precise experiments that probe the exact same physics?





Title: Re: EM Drive Developments - related to space flight applications - Thread 2
Post by: zellerium on 05/02/2015 07:18 pm
Hey everyone!


My name is Kurt Zeller, my colleague Bran Kraft and I are undergraduate Aerospace Engineering students at California Polytechnic State University, San Luis Obispo. (authors of this review: http://www.slideshare.net/KurtZeller/investigation-and-analysis-of-anomalous-electromagnetic-propulsion-devices-41315-46946953 )

We are currently designing an experiment to test a resonant cavity. We have made considerable progress on our own and would like to share our methodology with the forum to gain insight and suggestions before we start purchasing materials. Our primary objective is to demonstrate thrust using a counterbalanced lever. Our secondary objective is to quantify the thrust and make changes to the geometry of the dielectric.

We have recently received funding for a copper cavity, PTFE plate, and aluminum beam. This aluminum beam will have a sharp fulcrum welded to the bottom and the ends will be recessed lower than the fulcrum to put the center of mass closer to the balance point. We anticipate the thrust will be larger than the fulcrum's friction but this is still a concern in our experiment. An additional concern is the tension force from the cables that supply power. We plan to fix these cables above the fulcrum with enough slack to negate this issue. We acknowledge the difficulties in accurately obtaining thrust measurements but are mainly attempting to demonstrate thrust before quantifying it. 

The cavity will be made of C10100 Copper alloy tube with two C11000 copper end plates. A symmetric shape was chosen to minimize complications and cost as well as provide a different cavity geometry for comparison. A PTFE plate will machined into discs of varying thickness that fit adequately at the end of the cylinder. We have access to a VNA from our EE department and will use it to determine the resonant frequencies.

We are still awaiting approval for our latest proposal to the Cal Poly committee where we requested $3,500 to rent a 2-3 GHz signal generator, a 50W amplifier, and a spectrum analyzer. We are also planning to implement a matched load to absorb reflected power but are still working with professors to design it.   

Because these devices are incredibly expensive, we have been researching how to engineer a microwave oven magnetron into the power source. Unfortunately, the relatively constant frequency will limit the number of resonant modes that can be excited unless our cavity length is adjustable. Furthermore, thermal expansion may be a much greater issue at higher power and resonance may be more difficult to maintain.

In the event that we are approved funding for the rental equipment we will most likely attempt both setups.
We are happy to address questions or concerns and we look forward to any suggestions you may have. We understand Eagleworks is planning a similar experiment and we are hoping to gain more insight into their power delivery system.

We would like to thank all the contributors to this forum who have been a great help and inspiration for us throughout this project.
Title: Re: EM Drive Developments - related to space flight applications - Thread 2
Post by: cfs on 05/02/2015 07:19 pm
Just want to run this by the group.

I am a believer that thrust doesn't scale ONLY with Q. We even can see that in the data. See the original Nasa paper.
http://www.libertariannews.org/wp-content/uploads/2014/07/AnomalousThrustProductionFromanRFTestDevice-BradyEtAl.pdf

I think I have a good idea for once. I think the "Where is the balanced gain and loss?" thing from the other day is addressed by creating an unstable cavity, aka not high Q, not low Q either.

The balanced gain and loss stuff came up here. 4th-6th links from top.
http://forum.nasaspaceflight.com/index.php?topic=36313.msg1357829#msg1357829

I think the trick is to get energy in, put it to work a few thousand times (doing all that quantum wizardry I posted papers on :) ), then let it go as heat, which will inevitably happen as photons are red shifted and fall out of resonance. High Q is a baddie. Low Q is a baddie.

What's the point of having all that accumulated energy sitting in there static, doing nothing?

We need this thing to ride the razor's edge between gain and loss.

Also, what made this kinda click with me is what Mr. Shawyer said below. The Cullen paper he mentioned is shared here:
https://drive.google.com/folderview?id=0B4PCfHCM1KYoTXhSUTd5ZDN2WnM&amp;usp=sharing

So if this passes the smell test, how is the next question. Seems like not having the dielectric covering the entire small end (vs just a small patch) might be a good thing to try. I'm sure there's a ton of ways to do this.

There's a lot we can learn from that whispering-gallery research cited.
http://revolution-green.com/optics-breakthrough-demonstrates-new-behaviors-physics/
http://www.researchgate.net/publication/262451086_Paritytime-symmetric_whispering-gallery_microcavities
http://arxiv.org/ftp/arxiv/papers/1308/1308.4564.pdf

I'm openly brainstorming here. Would like some feedback.

Shawyer's explanation does not pass the smell test, and is not adressed by the Cullen paper you linked. Look for example at figure 5. There is a movable piston at the end of the waveguide T-junction, which is subject to radiation pressure. The piston will exert a (Newton's 3rd law) reaction force, and so momentum is conserved.

The EM drive is a fully enclosed cavity. The radiation inside will reflect off the walls and create some strain in the copper, but the net force integrated by the surface (given by the integral of the Poynting vector) has a time average of 0, as has been demonstrated mathematically many many times.

It is absolutely true that one can view standing waves as linear superpositions of traveling waves. This is just a different way of saying that Maxwell's equations are linear. Rodal's analysis is true whether one thinks of the fields as standing or superpositions of traveling waves. There is nothing besides the stress-energy tensor in the classical theory of electromagnetism.

To be clear: there is NO explanation for any increase in momentum of the drive to be found in classical theory (including Special Relativity through Maxwell's equations).

If there is an actual effect, then it must be caused by the coupling of electromagnetic fields to some other heretofore unobserved field. Even if such a coupling could be made in a way that is Lorentz invariant, it should have been detectable very easily at particle colliders. So again I ask:

If there is some effect here, why has it not been observed in far more precise experiments that probe the exact same physics?

This is why I'm saying there should be more of an effort to actually quantify the radiation emanating from the EMDrive.  If there really are e+/e- pairs popping out of the vacuum then we should be able to see them with appropriate detection mechanism (e.g. if NASA is like a standard physics department they should have some PMTs and scintillation crystals lying around somewhere).
Title: Re: EM Drive Developments - related to space flight applications - Thread 2
Post by: Mulletron on 05/02/2015 08:04 pm
Just want to run this by the group.

I am a believer that thrust doesn't scale ONLY with Q. We even can see that in the data. See the original Nasa paper.
http://www.libertariannews.org/wp-content/uploads/2014/07/AnomalousThrustProductionFromanRFTestDevice-BradyEtAl.pdf

I think I have a good idea for once. I think the "Where is the balanced gain and loss?" thing from the other day is addressed by creating an unstable cavity, aka not high Q, not low Q either.

The balanced gain and loss stuff came up here. 4th-6th links from top.
http://forum.nasaspaceflight.com/index.php?topic=36313.msg1357829#msg1357829

I think the trick is to get energy in, put it to work a few thousand times (doing all that quantum wizardry I posted papers on :) ), then let it go as heat, which will inevitably happen as photons are red shifted and fall out of resonance. High Q is a baddie. Low Q is a baddie.

What's the point of having all that accumulated energy sitting in there static, doing nothing?

We need this thing to ride the razor's edge between gain and loss.

Also, what made this kinda click with me is what Mr. Shawyer said below. The Cullen paper he mentioned is shared here:
https://drive.google.com/folderview?id=0B4PCfHCM1KYoTXhSUTd5ZDN2WnM&amp;usp=sharing

So if this passes the smell test, how is the next question. Seems like not having the dielectric covering the entire small end (vs just a small patch) might be a good thing to try. I'm sure there's a ton of ways to do this.

There's a lot we can learn from that whispering-gallery research cited.
http://revolution-green.com/optics-breakthrough-demonstrates-new-behaviors-physics/
http://www.researchgate.net/publication/262451086_Paritytime-symmetric_whispering-gallery_microcavities
http://arxiv.org/ftp/arxiv/papers/1308/1308.4564.pdf

I'm openly brainstorming here. Would like some feedback.

Shawyer's explanation does not pass the smell test, and is not adressed by the Cullen paper you linked. Look for example at figure 5. There is a movable piston at the end of the waveguide T-junction, which is subject to radiation pressure. The piston will exert a (Newton's 3rd law) reaction force, and so momentum is conserved.

The EM drive is a fully enclosed cavity. The radiation inside will reflect off the walls and create some strain in the copper, but the net force integrated by the surface (given by the integral of the Poynting vector) has a time average of 0, as has been demonstrated mathematically many many times.

It is absolutely true that one can view standing waves as linear superpositions of traveling waves. This is just a different way of saying that Maxwell's equations are linear. Rodal's analysis is true whether one thinks of the fields as standing or superpositions of traveling waves. There is nothing besides the stress-energy tensor in the classical theory of electromagnetism.

To be clear: there is NO explanation for any increase in momentum of the drive to be found in classical theory (including Special Relativity through Maxwell's equations).

If there is an actual effect, then it must be caused by the coupling of electromagnetic fields to some other heretofore unobserved field. Even if such a coupling could be made in a way that is Lorentz invariant, it should have been detectable very easily at particle colliders. So again I ask:

If there is some effect here, why has it not been observed in far more precise experiments that probe the exact same physics?

Thank you for that thoughtful response and yes I agree with you that there is no way to explain this with classical theory:
http://forum.nasaspaceflight.com/index.php?topic=36313.msg1367355#msg1367355
Quoting myself:
Quote
What I mean is that any attempt to formulate a theory of where the anomalous thrust is coming from which is based on classical electrodynamics will fail. A fully quantum approach is required. What I mean by "other than usual symmetry conditions" is that based off what I've been reading (aka not my own original research), simultaneous breaking of P & T symmetries is required.

On the flipside, the Q-thruster theory out there doesn't resonate  ;) with me either. That's why I've been trying to find other science out there that is accepted and makes sense. Do you think that is a worthy endeavor?
Title: Re: EM Drive Developments - related to space flight applications - Thread 2
Post by: dustinthewind on 05/02/2015 08:23 pm
Following on what jknuble said about the multipactor-like effect as a possible cause of thrust. http://en.wikipedia.org/wiki/Multipactor_effect  I can't help but wonder about what's going on with the copper surface of the frustum. A quick back of the envelope (well, python) calculation shows that there's certainly enough energy in these devices to somehow atomize a small amount of copper , and propel them with enough momentum to produce a small amount of thrust.

For example, a 30 watt emdrive where 0.001% of the energy went towards atomization and 1% went toward addtional momentum of the particles... You'd have a device with 91uN thrust, propelling 1.4ng of copper a second at 65500m/s.

I can think of 3 ways to debunk this. 1) perhaps that amount of particles going that fast would be noticeable with the naked eye, so this isn't really a valid explanation. 2) stick a detector behind the thruster (are they ionized?). 3) SEM of the surface compared to scraps from the same batch of copper not used in the thrustum.

Just how would we get a net-thrust from a closed cavity with atomization.  Even if atoms are being ioniozed inside the cavity I don't see how that could result in a net thrust.  Atomization results in immediate thrust but then that creates impact on the other side of the cavity canceling out the propulsion. 
Title: Re: EM Drive Developments - related to space flight applications - Thread 2
Post by: dustinthewind on 05/02/2015 08:47 pm

Not according to Maxwell's linear, isotropic equations.

The small base of the EM Drive is not open.  It is a closed cavity.  As such, the waves inside it are not travelling waves, but standing waves.   See this:  http://en.wikipedia.org/wiki/Standing_wave

The energy flux is pointed towards one end during half a (Poynting vector) period and it is pointed towards the opposite end during the next half-period.  Hence the net energy flux over a whole period is completely self-cancelling.

No net directional thrust for a microwave closed cavity can be explained just using Maxwell's linear, isotropic equations.  If the measurements are not an experimental artifact, another explanation is needed.

If the small base would be open, then it would be an inefficient photon rocket, with the microwave photons escaping at the small base end.  Less efficient for space propulsion than using a flashlight or a military searchlight as a means of propulsion.

Even for a perfectly collimated photon rocket, the thrust/powerInput of such a photon rocket is orders of magnitude less than what is claimed for these experiments.

With all respect, and I am not sure this is much of an issue to point out but I think the model of the waves in the cavity as standing waves may be a bit over-simplified. 

1. I think one can look at standing waves as super-imposed traveling waves of opposite direction but this part is likely less significant. 

2.  Maybe of more significance is the ac power in power lines can be modeled as standing waves but if no one is using power.  When power starts being consumed the standing waves begin to travel towards the object consuming the power.  The moving bulges of magnetic/electric field can be thought of as transporting power from the power station to the consumer.  There should be some traveling of the standing waves bulges from the power supply towards areas of heat loss in the cavity I would assume.  I can't say the power dissipated into heat loss is significant but it does seem to buck the perfect standing wave view for me a bit.  I do understand on the other hand that it may be a good approximation.

In ref to the image above, there is wealth of information buried within the earlier pages of this thread. A more accurate representation of what it looks like inside the cavity is available here:
http://forum.nasaspaceflight.com/index.php?topic=36313.msg1333246#msg1333246
Paul March was kind enough to attach the Frustrum modes overview 2A.pdf which has all the mode shapes and characteristics of their test article.

So things are a bit more complicated than photons bouncing around like marbles in a can. For example, I know that I can only excite TM212 and TM311 (thanks @Rodal for modeling this) with my little setup at home.
http://forum.nasaspaceflight.com/index.php?topic=36313.msg1353372#msg1353372
http://forum.nasaspaceflight.com/index.php?topic=36313.msg1352878#msg1352878

Paul March has shown clearly in his many posts that there is a clear correlation between mode shape and magnitude and direction of thrust. This is where input from RF Engineers would be extremely valuable.

Just food for thought, it is worth going back to page 1 and commenting on the stuff starting there. That way the conversation can keep building on ideas.

(http://forum.nasaspaceflight.com/index.php?action=dlattach;topic=36313.0;attach=820102;image)
(http://forum.nasaspaceflight.com/index.php?action=dlattach;topic=36313.0;attach=820104;image)
(http://forum.nasaspaceflight.com/index.php?action=dlattach;topic=36313.0;attach=796989;image)

In response to the bold and underlined statements above I was just pointing out that I think we may be losing information by the idea of the standing wave model with out considering the power losses (due heating of the cavity and any propulsion) and the transport of energy by [E^2+B^2] http://hyperphysics.phy-astr.gsu.edu/hbase/electric/engfie.html where the stading wave bulges tend to move towards the areas of consumed energy (heating + sparks + propulsion +ect).  I was paralleling it to energy consumption in power lines.  There might be some hidden phase relation in here lurking that could suggest em-phase propulsion but that's just speculation. 

I agree in the line of  thinking of what is going on in the cavity as more of a wave model than thinking of them as marbles bouncing inside the cavity.  I admit there might be some parallel to the marbles but I don't see it. 

It is interesting to think that as the wave peaks travel towards the small end of the cavity they are being squeezed but if the wave peaks are traveling towards the larger end they are experiencing expansion.  It looks like as a result we see the increased B field near the tight end of the cavity and small B field near the big end.  I almost want to think of this squeezing as a form of propulsion in the form of resistance of the traveling of the semi-standing waves as they transport energy to areas of heat loss. 
Title: Re: EM Drive Developments - related to space flight applications - Thread 2
Post by: Rodal on 05/02/2015 09:23 pm
...

In response to the bold and underlined statements above I was just pointing out that I think we may be losing information by the idea of the standing wave model with out considering the power losses (due heating of the cavity and any propulsion) and the transport of energy by [E^2+B^2] http://hyperphysics.phy-astr.gsu.edu/hbase/electric/engfie.html where the stading wave bulges tend to move towards the areas of consumed energy (heating + sparks + propulsion +ect).  I was paralleling it to energy consumption in power lines.  There might be some hidden phase relation in here lurking that could suggest em-phase propulsion but that's just speculation. 

I agree in the line of  thinking of what is going on in the cavity as more of a wave model than thinking of them as marbles bouncing inside the cavity.  I admit there might be some parallel to the marbles but I don't see it. 

It is interesting to think that as the wave peaks travel towards the small end of the cavity they are being squeezed but if the wave peaks are traveling towards the larger end they are experiencing expansion.  It looks like as a result we see the increased B field near the tight end of the cavity and small B field near the big end.  I almost want to think of this squeezing as a form of propulsion in the form of resistance of the traveling of the semi-standing waves as they transport energy to areas of heat loss.

Can you please answer, according to your conjecture,when the thrust force is  measured towards the small end of the truncated cone:

1) Are there any particles being emitted, according to your conjecture, out of the cavity in the axial direction towards the big base ?

[If nothing is being emitted, then there cannot be any propulsion, because it would violate conservation of momentum.  If nothing is being emitted, please skip the next question]

2) what  particle is being emitted, out of the cavity, in the axial direction towards the big base ?

[for example, if you conjecture that photons are being emitted, even if that conjecture would be true, it would be contradicted by what is claimed, because what is claimed is a thrust thousands of times better than the thrust of a perfectly collimated photon rocket]

So, if you are attempting to explain the claimed thrust just based on Maxwell's equations, I still don't understand your conjecture. If I misunderstood something, please correct me.  Thanks.
Title: Re: EM Drive Developments - related to space flight applications - Thread 2
Post by: Notsosureofit on 05/02/2015 09:24 pm
Just want to run this by the group.

I am a believer that thrust doesn't scale ONLY with Q. We even can see that in the data. See the original Nasa paper.
http://www.libertariannews.org/wp-content/uploads/2014/07/AnomalousThrustProductionFromanRFTestDevice-BradyEtAl.pdf

I think I have a good idea for once. I think the "Where is the balanced gain and loss?" thing from the other day is addressed by creating an unstable cavity, aka not high Q, not low Q either.

The balanced gain and loss stuff came up here. 4th-6th links from top.
http://forum.nasaspaceflight.com/index.php?topic=36313.msg1357829#msg1357829

I think the trick is to get energy in, put it to work a few thousand times (doing all that quantum wizardry I posted papers on :) ), then let it go as heat, which will inevitably happen as photons are red shifted and fall out of resonance. High Q is a baddie. Low Q is a baddie.

What's the point of having all that accumulated energy sitting in there static, doing nothing?

We need this thing to ride the razor's edge between gain and loss.

Also, what made this kinda click with me is what Mr. Shawyer said below. The Cullen paper he mentioned is shared here:
https://drive.google.com/folderview?id=0B4PCfHCM1KYoTXhSUTd5ZDN2WnM&amp;usp=sharing

So if this passes the smell test, how is the next question. Seems like not having the dielectric covering the entire small end (vs just a small patch) might be a good thing to try. I'm sure there's a ton of ways to do this.

There's a lot we can learn from that whispering-gallery research cited.
http://revolution-green.com/optics-breakthrough-demonstrates-new-behaviors-physics/
http://www.researchgate.net/publication/262451086_Paritytime-symmetric_whispering-gallery_microcavities
http://arxiv.org/ftp/arxiv/papers/1308/1308.4564.pdf

I'm openly brainstorming here. Would like some feedback.

Shawyer's explanation does not pass the smell test, and is not adressed by the Cullen paper you linked. Look for example at figure 5. There is a movable piston at the end of the waveguide T-junction, which is subject to radiation pressure. The piston will exert a (Newton's 3rd law) reaction force, and so momentum is conserved.

The EM drive is a fully enclosed cavity. The radiation inside will reflect off the walls and create some strain in the copper, but the net force integrated by the surface (given by the integral of the Poynting vector) has a time average of 0, as has been demonstrated mathematically many many times.

It is absolutely true that one can view standing waves as linear superpositions of traveling waves. This is just a different way of saying that Maxwell's equations are linear. Rodal's analysis is true whether one thinks of the fields as standing or superpositions of traveling waves. There is nothing besides the stress-energy tensor in the classical theory of electromagnetism.

To be clear: there is NO explanation for any increase in momentum of the drive to be found in classical theory (including Special Relativity through Maxwell's equations).

If there is an actual effect, then it must be caused by the coupling of electromagnetic fields to some other heretofore unobserved field. Even if such a coupling could be made in a way that is Lorentz invariant, it should have been detectable very easily at particle colliders. So again I ask:

If there is some effect here, why has it not been observed in far more precise experiments that probe the exact same physics?

Hmmm...  See:  http://newsoffice.mit.edu/2015/self-accelerating-particles-0120

Edit:  The self acceleration of photons, on the other hand, is old news.
         http://www.nature.com/ncomms/2014/141030/ncomms6189/abs/ncomms6189.html

Title: Re: EM Drive Developments - related to space flight applications - Thread 2
Post by: PaulF on 05/02/2015 09:47 pm
Hello again all,

The layman in physics would like to offer an analogy to ponder, nullify or use as an example..If it can assist you in getting a bit closer to the solution, I am thrilled I could help. If not, I am thrilled I got the opportunity to propose my idea to people who really know what they are talking about :D

Q: Would it be useful to suggest to also consider comparing the process in the cavity to standing sound waves? They can exist as the result of reflected self-interference but I know that contructive bulges can pack a hell of a punch. And especially what triggered this thought: the notion that the waves may be squeezed and stretched in the small and wide end of the cavity. I know matter waves are different from EM waves as they have duality, but still.

Another thought I had about CoE. It's probably already been explained in this context before but I probably didn't understand it :P I assume when you talk about CoE, that as much power is pumped into the device, as is released from the device in the various forms of energy and matter? Closed as it is, power is flowing THROUGH the device, which means the above must be measured to be certain as much power comes out as goes in. If there is a measurable difference and all errors have been nulled (and assuming the energy content in the cavity remains constant on average), would that indicate energy is being injected into the QV? If you have the time and patience, can someone clarify or correct me on this? Thank you so much.

P.S. I just had another bit of imagination: Remember the Cake Walk at the fair? Two vertical sets of steps that move up and down in a sine type movement, and moving 180 degrees out of phase of each other. Now in the beginning you try to get up but if you step on the steps at the wrong moment of the phase, you end up stepping on the same steps over and over. But if you change your cyclic movement and do it 180 degrees out of phase, you can ride the steps up and you would use hardly any energy yourself (the electromotors would draw a bit more current to provide the kinetic energy to lift me up every step).

With far-fetched (how appropriate!) deducing you could conclude that it is possible to ride wave bulges with the correct frequency , amplitude, and phase shift... Could this idea be adapted to suit EM waves? Is this any food for thought for anyone of you brilliant guys? ;)

With regards,

Paul F
Title: Re: EM Drive Developments - related to space flight applications - Thread 2
Post by: Nathan on 05/02/2015 10:01 pm
My take:
There is a potential gradient formed within the cavity causing one end of the device to fall toward the other. The energy density at each end would differ due to the geometry leaving one side with a slightly higher mass-energy density than the other, creating the gradient.
Nathan Rogers
Title: Re: EM Drive Developments - related to space flight applications - Thread 2
Post by: inquisitive-j on 05/02/2015 10:28 pm
If light is passing through the EM drive faster than the speed of light as people are suggesting from warp field inferometer test results, wouldn't that imply that that area had a refractive index less than 1? And if that's the case, isn't that something that can be easily tested?
Title: Re: EM Drive Developments - related to space flight applications - Thread 2
Post by: frobnicat on 05/02/2015 10:40 pm
Hey everyone!


My name is Kurt Zeller, my colleague Bran Kraft and I are undergraduate Aerospace Engineering students at California Polytechnic State University, San Luis Obispo. (authors of this review: http://www.slideshare.net/KurtZeller/investigation-and-analysis-of-anomalous-electromagnetic-propulsion-devices-41315-46946953 )

We are currently designing an experiment to test a resonant cavity. We have made considerable progress on our own and would like to share our methodology with the forum to gain insight and suggestions before we start purchasing materials. Our primary objective is to demonstrate thrust using a counterbalanced lever. Our secondary objective is to quantify the thrust and make changes to the geometry of the dielectric.

We have recently received funding for a copper cavity, PTFE plate, and aluminum beam. This aluminum beam will have a sharp fulcrum welded to the bottom and the ends will be recessed lower than the fulcrum to put the center of mass closer to the balance point. We anticipate the thrust will be larger than the fulcrum's friction but this is still a concern in our experiment. An additional concern is the tension force from the cables that supply power. We plan to fix these cables above the fulcrum with enough slack to negate this issue. We acknowledge the difficulties in accurately obtaining thrust measurements but are mainly attempting to demonstrate thrust before quantifying it. 

The cavity will be made of C10100 Copper alloy tube with two C11000 copper end plates. A symmetric shape was chosen to minimize complications and cost as well as provide a different cavity geometry for comparison. A PTFE plate will machined into discs of varying thickness that fit adequately at the end of the cylinder. We have access to a VNA from our EE department and will use it to determine the resonant frequencies.

We are still awaiting approval for our latest proposal to the Cal Poly committee where we requested $3,500 to rent a 2-3 GHz signal generator, a 50W amplifier, and a spectrum analyzer. We are also planning to implement a matched load to absorb reflected power but are still working with professors to design it.   

Because these devices are incredibly expensive, we have been researching how to engineer a microwave oven magnetron into the power source. Unfortunately, the relatively constant frequency will limit the number of resonant modes that can be excited unless our cavity length is adjustable. Furthermore, thermal expansion may be a much greater issue at higher power and resonance may be more difficult to maintain.

In the event that we are approved funding for the rental equipment we will most likely attempt both setups.
We are happy to address questions or concerns and we look forward to any suggestions you may have. We understand Eagleworks is planning a similar experiment and we are hoping to gain more insight into their power delivery system.

We would like to thank all the contributors to this forum who have been a great help and inspiration for us throughout this project.

Hello and welcome.

Maybe a (simplified) schematic of your projected set up would help people here (including those less fluent in English) visualize your future experiment and have suggestions. By "sharp fulcrum" do you mean a conical piece with "point contact" on support that allows for rotation around vertical axis, or a prismatic edge with linear contact on support, that rotates horizontally like in a teeter totter balance ? What roughly would be the dimensions and weight of the moving assembly, what choice of material for fulcrum and support where it rests ? Risk with dry contacts at µN levels is stiction... why not go hanging Cavendish torsion pendulum ? There seems to be a rare consensus that it would be best.

edit : also be aware that you are pumping some heat in a contained volume, either make you cavity completely hermetic or leave a large opening (1cm² or so, with a conductive mesh to keep µwaves from leaking) between inside and outside. Either will prevent a potential pressure build up to "jet", but the worse would be an uncontrolled thin slit letting the heated air leak outside but not enough to equalize pressure "instantly". And while I'm at it, beware of thermal dilation centre of mass shifts (depending on the type of balance...).

For powering the system, why not go with batteries to avoid problem with cables from fixed part to moving assembly ? Lithium batteries at ~100Wh/kg are readily available at modest costs. A pack sized for 50W output for half an hour would add significant but not enormous weight to the moving assembly and would avoid a whole lot of potential spurious signals, and make your experiment better than Eagleworks' one (so far) on that aspect. A lot of people have suggested a fully self enclosed system (thermally, electrically, and magnetically if possible) on the balance as a required condition for better credibility.

One of the very few (only ?) reported null result (down to sensitivity) with experiment on propellantless device (not em drive type...) was obtained with a fully self contained test article :
Null Findings on Electromagnetic Inertia Thruster
Experiments using a Torsion Pendulum by Brito, Marini, Galian 2009 (http://enu.kz/repository/2009/AIAA-2009-5070.pdf)

Knowing how to create and see an EM thrust signature, if there is such a thing, is nice. Designing and conducting the experiment to see no thrust signature if there is none is more scientifically valuable IMHO. And at this stage there appear to be more risks in false positives than in false negatives.
Title: Re: EM Drive Developments - related to space flight applications - Thread 2
Post by: Dmytry on 05/02/2015 10:57 pm
One of the very few (only ?) reported null result (down to sensitivity) with experiment on propellantless device (not em drive type...) was obtained with a fully self contained test article :
Null Findings on Electromagnetic Inertia Thruster
Experiments using a Torsion Pendulum by Brito, Marini, Galian 2009 (http://enu.kz/repository/2009/AIAA-2009-5070.pdf)

Knowing how to create and see an EM thrust signature, if there is such a thing, is nice. Designing and conducting the experiment to see no thrust signature if there is none is more scientifically valuable IMHO. And at this stage there appear to be more risks in false positives than in false negatives.
Now that was a great experimental setup. Appropriate $ spending for how credible the claims were, yet potentially very sensitive, repeatable, and accurate.

Regarding enclosing it, the first test is to at least just enclose it pneumatically, i.e. so it wouldn't be blowing ions. Albeit enclosing it electrically may be necessary to avoid it messing up the pendulum. Another good thing about the hanging pendulum that is not too stiff, is that heating of the wire it's hanging from is not likely to result in a huge zero shift.

(The fact of just how cheap a good set up can be is also why any requests for extra funding only bring to mind an image of a certain generally very disrespected Italian scientist who had been doing otherwise straightforward calorimetry using a very expensive thermal imaging camera.)

And it's more in line with the topic of the thread as well, because to use it in space you need it to be self contained. Also it helps a little if you have more money left for space stuff.
Title: Re: EM Drive Developments - related to space flight applications - Thread 2
Post by: Robotbeat on 05/02/2015 11:46 pm
But if there's no thrust when the drive is enclosed and if the thrust is diminished, though still present, in a vacuum (when there is no air to push against), doesn't that suggest that the cavity is spewing particles somehow?
Right, put kilowatts of power into something, it will heat up and start outgassing something fierce. But also do a lot of other things.

2.5kW is a LOT of power. If it's all essentially absorbed by the cone, then it gets converted into heat. 2.5kW is like the power output of an oven or like 40 incandescent light bulbs. Especially in a vacuum, you're going to heat up your device to hundreds of degrees. If this gets transferred to the thrust-measuring arm at all (which it likely would), you're going to change the resistance of your strain gauge, thus giving you a false thrust reading.


Remember that strain gauges are just copper wire, and so acts just as well as a kind of thermometer. A temperature change (unless super, super carefully canceled out--almost impossible to do if there's a large temperature gradient on the gauge itself) will show up as a change in resistance, just like force will.
Title: Re: EM Drive Developments - related to space flight applications - Thread 2
Post by: Rodal on 05/03/2015 12:02 am
Just want to run this by the group.

I am a believer that thrust doesn't scale ONLY with Q. We even can see that in the data. See the original Nasa paper.
http://www.libertariannews.org/wp-content/uploads/2014/07/AnomalousThrustProductionFromanRFTestDevice-BradyEtAl.pdf

I think I have a good idea for once. I think the "Where is the balanced gain and loss?" thing from the other day is addressed by creating an unstable cavity, aka not high Q, not low Q either.

The balanced gain and loss stuff came up here. 4th-6th links from top.
http://forum.nasaspaceflight.com/index.php?topic=36313.msg1357829#msg1357829

I think the trick is to get energy in, put it to work a few thousand times (doing all that quantum wizardry I posted papers on :) ), then let it go as heat, which will inevitably happen as photons are red shifted and fall out of resonance. High Q is a baddie. Low Q is a baddie.

What's the point of having all that accumulated energy sitting in there static, doing nothing?

We need this thing to ride the razor's edge between gain and loss.

Also, what made this kinda click with me is what Mr. Shawyer said below. The Cullen paper he mentioned is shared here:
https://drive.google.com/folderview?id=0B4PCfHCM1KYoTXhSUTd5ZDN2WnM&amp;usp=sharing

So if this passes the smell test, how is the next question. Seems like not having the dielectric covering the entire small end (vs just a small patch) might be a good thing to try. I'm sure there's a ton of ways to do this.

There's a lot we can learn from that whispering-gallery research cited.
http://revolution-green.com/optics-breakthrough-demonstrates-new-behaviors-physics/
http://www.researchgate.net/publication/262451086_Paritytime-symmetric_whispering-gallery_microcavities
http://arxiv.org/ftp/arxiv/papers/1308/1308.4564.pdf

I'm openly brainstorming here. Would like some feedback.

Shawyer's explanation does not pass the smell test, and is not adressed by the Cullen paper you linked. Look for example at figure 5. There is a movable piston at the end of the waveguide T-junction, which is subject to radiation pressure. The piston will exert a (Newton's 3rd law) reaction force, and so momentum is conserved.

The EM drive is a fully enclosed cavity. The radiation inside will reflect off the walls and create some strain in the copper, but the net force integrated by the surface (given by the integral of the Poynting vector) has a time average of 0, as has been demonstrated mathematically many many times.

It is absolutely true that one can view standing waves as linear superpositions of traveling waves. This is just a different way of saying that Maxwell's equations are linear. Rodal's analysis is true whether one thinks of the fields as standing or superpositions of traveling waves. There is nothing besides the stress-energy tensor in the classical theory of electromagnetism.

To be clear: there is NO explanation for any increase in momentum of the drive to be found in classical theory (including Special Relativity through Maxwell's equations).

If there is an actual effect, then it must be caused by the coupling of electromagnetic fields to some other heretofore unobserved field. Even if such a coupling could be made in a way that is Lorentz invariant, it should have been detectable very easily at particle colliders. So again I ask:

If there is some effect here, why has it not been observed in far more precise experiments that probe the exact same physics?

Hmmm...  See:  http://newsoffice.mit.edu/2015/self-accelerating-particles-0120

Edit:  The self acceleration of photons, on the other hand, is old news.
         http://www.nature.com/ncomms/2014/141030/ncomms6189/abs/ncomms6189.html

(Kudos to old-timers  8) ), this has been predicted since at least 36 years ago, going back to 1979, when Berry and Balazs proposed that the Schrodinger equation describing a free particle can exhibit a nonspreading Airy wave packet solution with the ability to freely accelerate even in the absence of any external potential (in other words, with no force acting).  What is relatively new (last 10 years) are the experiments, confirming their theoretical prediction.

See page 266 of this: 

https://michaelberryphysics.files.wordpress.com/2013/07/berry078.pdf
M.V. Berry, N.L. Balazs, Am. J. Phys. 47, 264 (1979)



PS: Balazs was at State University in Stony Brook at the time (kudos to MIT's James Simons, who I think was chairman of the math department at Stony Brook University at the time (am I correct ?)

Nandor Balázs was assistant to Schroedinger at the Dublin Institute for Advanced Studies, and a one year assistant to Einstein at the Institute for Advanced Study in Princeton.  He also had close working collaboration with  Paul Dirac , Chandrasekhar, and Eugene Wigner.
Title: Re: EM Drive Developments - related to space flight applications - Thread 2
Post by: Dmytry on 05/03/2015 12:45 am
Speaking of the power levels, there was 55 uN in air from 2.6 watts, TE012. Now that's something that's easy and safe to build with a couple high frequency transistors and assorted passives. Can obtain feedback with another antenna in the cavity so that it is always resonating. Albeit you'd need a quite expensive oscilloscope.
Title: Re: EM Drive Developments - related to space flight applications - Thread 2
Post by: Rodal on 05/03/2015 12:45 am
But if there's no thrust when the drive is enclosed and if the thrust is diminished, though still present, in a vacuum (when there is no air to push against), doesn't that suggest that the cavity is spewing particles somehow?
Right, put kilowatts of power into something, it will heat up and start outgassing something fierce. But also do a lot of other things.

2.5kW is a LOT of power. If it's all essentially absorbed by the cone, then it gets converted into heat. 2.5kW is like the power output of an oven or like 40 incandescent light bulbs. Especially in a vacuum, you're going to heat up your device to hundreds of degrees. If this gets transferred to the thrust-measuring arm at all (which it likely would), you're going to change the resistance of your strain gauge, thus giving you a false thrust reading.


Remember that strain gauges are just copper wire, and so acts just as well as a kind of thermometer. A temperature change (unless super, super carefully canceled out--almost impossible to do if there's a large temperature gradient on the gauge itself) will show up as a change in resistance, just like force will.

NASA Eagleworks reported a thrust force measurement with only 2.6 watts input in ambient air conditions (that's almost 1,000 times less power than quoted above):

lengths in meter
rfFrequency in 1/second (microwave frequency during test)
power in watts
force in milliNewtons
force per PowerInput in milliNewtons/kW

rfFrequency = 1.8804*10^9;
power = 2.6
Q = 22000
measured force = 0.05541
measured ForcePerPowerInput = 21.31
Force/PowerInput of a Photon Rocket =0.003337
measured ForcePerPowerInput to the one of a photon rocket =6,386.7
Title: Re: EM Drive Developments - related to space flight applications - Thread 2
Post by: Dmytry on 05/03/2015 12:47 am
Keep in mind also that such results make me think the rig is not working right. Power levels vary widely, so does the Q-factor (and thus the actual EM field inside the cavity), thrust on the other hand somehow always stays within the range of drifts.
Title: Re: EM Drive Developments - related to space flight applications - Thread 2
Post by: Robotbeat on 05/03/2015 01:01 am
Also, if you're talking about microNewtons in the ambient air... That's absurd. The HVAC system would easily produce stronger forces on the experimental system. Heck, I could easily see it producing milliNewtons, not just microNewtons. I've had this very problem while teaching physics lab to electrical engineering students.

And even if it's enclosed, in order to reliably measure in the microNewtons, you need careful vibration isolation.

In order for me to be anywhere near convinced that there's the slightest possibility they're on to something, I'd have to have a very thorough examination of the experimental setup, or have someone else used to such highly sensitive measurements.
Title: Re: EM Drive Developments - related to space flight applications - Thread 2
Post by: Rodal on 05/03/2015 01:04 am
The original report of Brady et.al. "Anomalous report..."
The only experimental report written so far by NASA Eagleworks on the EM Drive
Title: Re: EM Drive Developments - related to space flight applications - Thread 2
Post by: Rodal on 05/03/2015 01:06 am
But seriously what is this complaining about Engineers ?

And what is this complaining about the units?  The article you misquoted has the units next to the numbers

Quote from: http://www.nasaspaceflight.com/2015/04/evaluating-nasas-futuristic-em-drive/
However, in 2010, Prof. Juan Yang in China began publishing about her research into EM Drive technology, culminating in her 2012 paper reporting higher input power (2.5kW)

What's wrong with the units in 2.5kW  ???
Title: Re: EM Drive Developments - related to space flight applications - Thread 2
Post by: Mulletron on 05/03/2015 04:45 am
My take:
There is a potential gradient formed within the cavity causing one end of the device to fall toward the other. The energy density at each end would differ due to the geometry leaving one side with a slightly higher mass-energy density than the other, creating the gradient.
Nathan Rogers

First, thanks for stimulating discussion.  :)

Yes there is. It is especially evident in TE012. This anisotropy alone isn't enough to explain the thrust.
http://forum.nasaspaceflight.com/index.php?topic=36313.1440
Where is the coupling?

What I have quoted above is theorized to lead to lots of exciting phenomenon like anisotropy of the vacuum itself yet proof remains elusive. It is very much predicted by multiple independent parties.

When you say "fall" though, that throws very controversial gravity research into the mix. What experimental evidence do we have to support the PV model by Puthoff et al?

It is very tempting to try and connect anisotropy of light in matter....(proven)to anisotropy of light in vacuum  (not proven), and then further try and tie that to gravity itself (not proven). It may very well be that way. But where is the proof? Right now there is none.

I looked back through the thread and we've discussed Dr. White's dependence on energy density rather than the Poynting vector.
http://forum.nasaspaceflight.com/index.php?topic=36313.msg1355995#msg1355995

To my knowledge, we haven't produced anything showing energy densities, like what Egan presented here:
http://gregegan.customer.netspace.net.au/SCIENCE/Cavity/Cavity.html

There's so much info buried in the thread that I might have lost it.

The thrust dependence on energy density theme is reoccurring. I'm not so sure that is the best approach to go with ONLY dependence on energy density. I think we need energy density and a defined Poynting vector.

It doesn't appear as if the original design intent was centered on the that idea:
http://www.emdrive.com/theorypaper9-4.pdf
http://emdrive.com/principle.html
..............Everything is about group velocity at Emdrive.com

A nice writeup here about the discrepancy: http://en.wikipedia.org/wiki/EmDrive#Theory

Dependence on the Poynting vector has more or less been abandoned here, mostly due to very good calculations by Dr. Rodal. I've been trying to sway conversation away from it.But I started thinking that the design of EMdrive is just way off the mark when it comes to the Poynting vector.

Especially when Shawyer himself is telling me the thrust is coming from the traveling wave:
http://forum.nasaspaceflight.com/index.php?topic=36313.msg1368066#msg1368066

The only hope I see left for the Poynting vector would be unknowns which may introduce a Poynting vector such as uneven heat loss. Would a more detailed analysis uncover a non zero Poynting vector?

Based on my post from yesterday:
http://forum.nasaspaceflight.com/index.php?topic=36313.msg1368066#msg1368066
I'm reconsidering throwing out the Poynting vector. Also see Rodal's post I linked to below which I rediscovered while writing this post.

It seems like the EMdrive as it is currently incarnated is needs a slight change in design. We want to keep the features which make it an energy storage device, but we need to introduce a very pronounced Poynting vector. A very high Q just kills that Poynting vector. Just as @Rodal said too.

This probably explains how Emdrives work, experimenters are telling us they work....but they barely work.

How can we get very high Q and get a nice Poynting vector at the same time?
That doesn't make sense to me, but that is what we need.
http://forum.nasaspaceflight.com/index.php?topic=36313.msg1354235#msg1354235

I believe that slight changes are needed in the basic design of Emdrive if it is ever going to be useful for spaceflight. The biggest factor I see is the addition of a better dielectric or magnetoelectric material.
http://arxiv.org/abs/1211.0530
Title: Re: EM Drive Developments - related to space flight applications - Thread 2
Post by: Mulletron on 05/03/2015 07:17 am

You accurately and completely answered the questions I asked.  Unfortunately, I didn't ask my questions very well.  :)

http://forum.nasaspaceflight.com/index.php?topic=36313.msg1354235#msg1354235

A constant-amplitude standing wave does indeed result in a zero time-average Poynting vector.  However, I am questioning your conclusion that a constant-amplitude standing wave accurately represents a real resonator cavity such as the as-tested EM drive frustum.  Instead, I would expect a decaying amplitude standing wave to be a more accurate model/plot (as would be derived from a full solution to Maxwell's equations with proper boundary conditions such as non-zero resistance, etc).

Once a time-decaying standing wave is used for computation of a time-average Poynting vector, I'm having trouble seeing how the incident and reflected energy can perfectly cancel and become zero.  I'll readily admit I may be oversimplifying and/or missing a fundamental concept;  it's been a long time since I actually computed time constants for resonant cavities using Maxwell's equations and non-zero resistances.

Phrased a bit differently, I believe only excited modes with current/thermal losses in the base plates will significantly weight the direction of the time-average Poynting vector.  Each pair of incident/reflected waves would have a larger energy loss at the base plate with the excited E field (and therefore excited currents) than the energy loss at the opposing base plate.  For modes with near-zero E fields at the base plate boundaries, each incident/reflected wave pair would have a near equal energy delta regardless of which base plate they came in contact with;  the resulting time-averaged direction would be random and magnitude limited by the energy lost in the very first reflection (randomly either the large or small base, with a magnitude very close to zero).

I view this Poynting vector discussion to be completely independent of whether Dr. White's QV interactions, or some other classical physics can explain the EM drive anomalous thrust.  Just wanted to chime in on a what appeared to be the use of a constant-amplitude standing wave to describe a real-world system.  Your earlier observation of a non-zero time averaged Poynting vector seemed like a reasonable statement given that only excited modes with current/thermal losses in the base plates would quickly diverge from the simplified constant-amplitude standing wave model. 

Regards,
James
James, thank you for the interesting, thought-provoking discussion.  :)

Let's calculate some numbers to estimate what we are discussing.

The electromagnetic fields transition from the air or vacuum medium (where they are out of phase by 90 degrees) to the copper over an extremely small distance: a boundary layer. The skin depth for copper at 2 GHz is 1.48 micrometers = 58.2 microinches .  When showing the Poynting vector field distribution this distance is infinitesimal compared to the rest of the cavity.  In this very small distance inside the copper (1.48 micrometers ) the electromagnetic fields in the copper are out of phase by approximately 45 degrees (due to the high conductivity of copper).  For a transverse magnetic (TM) mode, the only electromagnetic field component that is continuous across the vacuum/copper interface is the electric field component tangent to the copper surface.

More interestingly (for this thread's discussion due to the significance that the NASA experimenters have placed on the dielectric being responsible for providing the measured thrust) is what happens in the High Density PolyEthylene (HDPE) dielectric polymer insert.  Because the dimensions of the dielectric are not negligible compared to the EM Drive's dimension, and the dielectric is not modeled as just a boundary condition.

The loss tangent of HDPE is reported to be

tan delta = 0.0004

Therefore the intrinsic impedance angle is

intrinsic impedance angle =(ArcTan[0.0004])/2

Therefore, inside the HDPE dielectric the electric and magnetic fields, instead of being out of phase by 90 degrees (as they are in the air or vacuum medium), will be out of phase by:

90 - (ArcTan[0.0004])/2 = 89.9998 degrees

This phase angle (89.9998 degrees)  will show practically no visual difference with 90 degrees at the resolution of the following image :

(http://www-ssc.igpp.ucla.edu/personnel/russell/papers/skip_ed/fig9.gif)

The Poynting vector inside the HDPE dielectric, instead of having a zero time average, will have the following factor multiplying ExB/mu :

Cos[90 - (ArcTan[0.0004])/2]/2 = 0.0000999999940000637

So, inside the HDPE polymer dielectric the Poynting vector has this small magnitude over a period (or multiples thereof).

So, the extent of this approximation, for the HDPE dielectric is about 0.01 % (which is negligible in comparison to several other approximations).


Now, let's examine what this (very small intensity Poynting vector time average) means, concerning the discussion in this EM Drive.

If one were to posit that the EM Drive's thrust is due to the very small magnitude of the time average of the Poynting vector due to these thermal losses (in the HDPE dielectric or in the copper):

1) It would mean that there should be more thrust with lower Q.  This is the complete opposite of what the experimenters like Shawyer claims (Shawyer claims that the higher the Q, the greater the thrust).  Notice that

Tan [loss angle] = Tan[ 2 impedance angle ]= 1/Q

2) All the equations proposed so far (by Shawyer, McCulloch and @Notsosureofit) have predicted thrust proportional to Q.  This is the complete opposite of what such a Poynting vector would predict (it would predict thrust proportional to 1/Q instead), because

Tan delta= 1/Q

measured Q        effective tan delta

7320                  1.366* 10 ^(-4)
22000                4.545* 10 ^(-5)
10^6                  10^(-7)

3) It would mean that experimenters like Shawyer and Fetta are in the completely wrong track pursuing superconducting EM Drives, as superconducting EM Drives would lead to practically no thrust (the opposite of what they claim) because superconducting EM Drives would display practically no heat losses and hence zero time average Poynting vector.

4) Considering the HDPE dielectric acting as a sink (energy flowing from the EM Drive towards the HDPE where the energy is dissipated internally in the dielectric polymer due to its tandelta and hence irretrievably lost instead of being reflected), the Poynting vector would be directed towards the HDPE dielectric, that is towards the small base, and hence the EM Drive should experience a recoil force and acceleration towards the big base.  This is the opposite direction force found in NASA's experiments with the dielectric.  (Recall that NASA Eagleworks found no thrust force with mode TE012 without a HDPE dielectric and that with the HDPE dielectric inserted at the small base they found a force and acceleration directed towards the small base.)

Dang! How did this slip through the cracks? Read the whole thing. See #3! This has been bubbling up from all the discussion, up until now.

@Rodal, we need you more than ever to help quantify this missing Poynting vector that everyone says isn't there in an unloaded cavity, and absolutely is not there when calculated using standard classical methods.

James is right, there is NO WAY there is a perfect standing wave in there.
http://forum.nasaspaceflight.com/index.php?topic=36313.msg1354319#msg1354319

I see a way forward here:

1) Find the tiny missing Poynting vector. TE012 is a candidate. See if it is consistent with the anomalous thrust.
2) If that works, find a way to enhance the Poynting vector.
3) Plug in a Casimir momentum theory that is consistent with established scientific principles.
4) Test, rinse, repeat.

#1) and #3) should be enough to stop the handwaving.
#2) and #4) to make it work better and to test ideas.

IRT to this and many posts about TE012, the Poynting vector is self canceling without the dielectric.
http://forum.nasaspaceflight.com/index.php?topic=36313.msg1352885#msg1352885

Quote
I showed the fact that boundary conditions for the transverse electric (TE) electromagnetic mode shapes all Poynting vector components vanish at the walls of the EM Drive and the fact that for mode shape TE012 without the dielectric the Poynting vector is self-cancelling. This is consistent with NASA Eagleworks results and supports NASA Eagleworks preference for the transverse magnetic (TM) modes over the transverse electric (TE) modes.

I can't remember if the Poynting vector was self canceling with the dielectric or not. I'm pretty sure it was not self canceling WITH the dielectric. Is that accurate?

Keep in mind, NASA reported no thrust at TE012 without dielectric, and good thrust with dielectric.
Title: Re: EM Drive Developments - related to space flight applications - Thread 2
Post by: CW on 05/03/2015 08:24 am
One question, regarding Mr. Shawyer's understanding of what's going on in his EM-drive:

Let's pretend we are a photon. We are traveling at c. Due to Lorentz-contraction of space in our traveling direction, how does the universe look like for us, a photon? I think that the universe actually appears as a sort of surface in our traveling direction, due to the maximum Lorentz contraction of space in front of us.

The question is: For a photon, moving at c, how does the inner geometry of an EM-drive frustum really look like? Could there be an apparent nonlinearity, from the viewpoint of a photon, within an EM-drive device, that we in our everyday, non-Lorentz contracted world don't perceive? Is there someone on the forum who can do the math that projects how the maximum-Lorentz contracted universe looks like for a photon, and apply this to an EM-drive environment?

Best,
CW
Title: Re: EM Drive Developments - related to space flight applications - Thread 2
Post by: Mulletron on 05/03/2015 10:00 am
Excellent work posting the Shawyer emails.

Question on your test rig. You are aware, according to Shawyer, a non moving EMDrive will not generate any thrust and that it must move to enable thrust to be indirectly measured? Are you planning to use a rotary test?

http://www.emdrive.com/EmDriveForceMeasurement.pdf

Additionally his test with a dielectric, resulted in reduced thrust generation.

http://www.emdrive.com/iac2014presentation.pdf - page 2

Wish you all the best of luck. Would be interested to replicate your device.

That is the missing Poynting vector. See above in bold. Think about Doppler shift inside a resonant cavity when it moves, then photons fall out of resonance, and go evanescent. That energy is lost to work/heat.

The energy flowing back into the cavity is the missing Poynting vector.

This is common to all resonant modes.

From the http://www.emdrive.com/EmDriveForceMeasurement.pdf
Quote
Because the thruster is at rest, no force will be measured on the load cell.
i.e. F = T-R = 0
It therefore appears that a force measurement can only be made in a dynamic environment, ideally by allowing the thruster to accelerate, measuring that acceleration, and then calculating the thrust from T = -Ma.

How is the Eagleworks pendulum satisfying that requirement? That it must be moving in order to get a measurement? All this noise.....see screenshot.

Reminds me of the old light in a box discussion we had.
http://usersguidetotheuniverse.com/?p=2865 (also where the math is)
http://forum.nasaspaceflight.com/index.php?topic=36313.msg1334277#msg1334277
http://forum.nasaspaceflight.com/index.php?topic=36313.msg1334560#msg1334560


Please review page 2
http://www.emdrive.com/iac2014presentation.pdf

Note the use of a dielectric resulted in weaker thrust and in the opposite direction, line 3, to a non dielectric EMDrive, line 4, 5 & 6.

This suggest EW may have found a new way to gen thrust, via a dielectric, that appears to work when the test device is fixed/stationary and in the opposite direction to a classic EMDrive.

Interesting.
TheTraveler keeps sending me PMs which are good stuff. He needs to post in open forum. He's essentially saying the dielectric section also serves to "bootstrap" this initial acceleration requirement.

Probably should have listened more closely to what Shawyer was saying. Knowing what we know now, this is an opportunity to improve these devices by looking at how to improve energy flow through the cavity with different dielectric and magnetoelectric materials, or some other way......

http://arxiv.org/abs/1211.0530 A whole bunch of candidate materials.

This has some interesting ramifications too. An Emdrive just sitting there on the bench is subject to a constant 1g proper acceleration.
Title: Re: EM Drive Developments - related to space flight applications - Thread 2
Post by: PaulF on 05/03/2015 11:36 am
One question, regarding Mr. Shawyer's understanding of what's going on in his EM-drive:

Let's pretend we are a photon. We are traveling at c. Due to Lorentz-contraction of space in our traveling direction, how does the universe look like for us, a photon? I think that the universe actually appears as a sort of surface in our traveling direction, due to the maximum Lorentz contraction of space in front of us.

The question is: For a photon, moving at c, how does the inner geometry of an EM-drive frustum really look like? Could there be an apparent nonlinearity, from the viewpoint of a photon, within an EM-drive device, that we in our everyday, non-Lorentz contracted world don't perceive? Is there someone on the forum who can do the math that projects how the maximum-Lorentz contracted universe looks like for a photon, and apply this to an EM-drive environment?

Best,
CW
OK, I am going to take a (partial)shot at this. I am a complete idiot as far as maths go, but I did remember some of it from school. I hope this can give you a handle at least, so please be gentle...

at light speed the universe in front of you would seem to me indeed like a flat surface. A projection of the universe on a wall from your viewpoint (edit- I realize, because photons experience no time, it would just look like a single point "directly in front" I realize this is also not accurate because the photon would experience itself within that same point. But for argument's sake lets assume we can take a step back. More importantly, a photon and the universe would seem to be 0-dimensional from that viewpoint) Am I completely wrong in stating that in case of the photon, you could apply formulas that are relevant to non-relativistic viewers, but differentiated twice? remove two dimensions from one, remove two dimensions from the other idea?

Deducing from that I posit that as speed increases our 3 dimensional space-time is reduced to 0 dimensions at at the point where light speed is reached. Which would also be another confirmation that faster than light travel is not possible. With the exception of warping space-time.
Title: Re: EM Drive Developments - related to space flight applications - Thread 2
Post by: TheTraveller on 05/03/2015 11:54 am
Here Shawyer compares 7 devices, some classic EMDrives, some with just a dielectic & the superconducting LN cooled Cannae drive.

It is important to note the direction of generated thrust as dielectics apparently generate thrust in the opposite direction to that of EMDrives.

I do note the EW test of a Cannae like device was not tested at cryo temp nor had superconducting interior lining and had an added dielectic. So was not a test of a true Cannae device.

From Shawyer, EW should not expect to see any thrust from an EMDrive like test device when tested fixed, not allowed to accelerate, without added dielectic. Which is what was observed. Static thrust was not measured, with the EW EMDrive like device until a dielectic was added.

Appears EW have discovered a new way to gen thrust in a fixed device. Use a dielectric. From Shawyer device summary, line 1, it appears all that may be needed is a short section of resonate pipe with a dielectic stuck in one end. IE EW Cannae test device minus the cavity.

However as the dielectric thrust is weaker than the EMDrive thrust and in the opposite thrust direction to the classic EMDrive thrust direction, putting a dielectric into an accelerating EMDrive may reduce the overall delivered thrust and effective cavity Q as would be seen by the classic EMDrive operational mode.

Is Shawyer giving away spoilers?
Title: Re: EM Drive Developments - related to space flight applications - Thread 2
Post by: Mulletron on 05/03/2015 12:05 pm
Never really thought about this before, but is light in a cavity resonator gravitationally redshifted?

http://en.wikipedia.org/wiki/Gravitational_redshift
Title: Re: EM Drive Developments - related to space flight applications - Thread 2
Post by: PaulF on 05/03/2015 12:12 pm
Never really thought about this before, but is light in a cavity resonator gravitationally redshifted?

http://en.wikipedia.org/wiki/Gravitational_redshift
Do you mean natural (i.e. earth) or artificial gravity that could be generated by space-time warping or other quantum voodoo?
Title: Re: EM Drive Developments - related to space flight applications - Thread 2
Post by: Mulletron on 05/03/2015 12:24 pm
Never really thought about this before, but is light in a cavity resonator gravitationally redshifted?

http://en.wikipedia.org/wiki/Gravitational_redshift
Do you mean natural (i.e. earth) or artificial gravity that could be generated by space-time warping or other quantum voodoo?

I mean, if you had an Emdrive sitting the table (large diameter down) here on Earth, and it was energized by feeding RF into a slot/probe located at the large diameter. Would an observer at the small end notice a red shift?
Title: Re: EM Drive Developments - related to space flight applications - Thread 2
Post by: Mulletron on 05/03/2015 12:32 pm
https://twitter.com/elonmusk/status/594756342641922048

But why in the world did @ElonMusk link to an article from August 2014? That is soooo last year.
Title: Re: EM Drive Developments - related to space flight applications - Thread 2
Post by: PaulF on 05/03/2015 12:36 pm
Never really thought about this before, but is light in a cavity resonator gravitationally redshifted?

http://en.wikipedia.org/wiki/Gravitational_redshift
Do you mean natural (i.e. earth) or artificial gravity that could be generated by space-time warping or other quantum voodoo?

I mean, if you had an Emdrive sitting the table (large diameter down) here on Earth, and it was energized by feeding RF into a slot/probe located at the large diameter. Would an observer at the small end notice a red shift?
Would seem to me that if the thrust produced is horizontally, gravity can't have effect on it, besides bending it a teeny weeny bit. Time dilation at beginning and endpoint are same (same distance from gravity well), so in effect 0.

Vertically you could measure it, but the redshift from earth's gravity is already extremely difficult to measure, but if you could measure it precisely, you could then subtract that value from the redshift value measured in the cavity.
Title: Re: EM Drive Developments - related to space flight applications - Thread 2
Post by: Mulletron on 05/03/2015 12:39 pm
Never really thought about this before, but is light in a cavity resonator gravitationally redshifted?

http://en.wikipedia.org/wiki/Gravitational_redshift
Do you mean natural (i.e. earth) or artificial gravity that could be generated by space-time warping or other quantum voodoo?

I mean, if you had an Emdrive sitting the table (large diameter down) here on Earth, and it was energized by feeding RF into a slot/probe located at the large diameter. Would an observer at the small end notice a red shift?
Would seem to me that if the thrust produced is horizontally, gravity can't have effect on it, besides bending it a teeny weeny bit. Time dilation at beginning and endpoint are same (same distance from gravity well), so in effect 0.

Vertically you could measure it, but the redshift from earth's gravity is already extremely difficult to measure, but if you could you could then detract that from the redshift value measured in the cavity.

Quote
so in effect 0.

So nonzero? Say outside the bandwidth of a very narrow bandwidth cavity?
Title: Re: EM Drive Developments - related to space flight applications - Thread 2
Post by: PaulF on 05/03/2015 12:40 pm
Never really thought about this before, but is light in a cavity resonator gravitationally redshifted?

http://en.wikipedia.org/wiki/Gravitational_redshift
Do you mean natural (i.e. earth) or artificial gravity that could be generated by space-time warping or other quantum voodoo?

I mean, if you had an Emdrive sitting the table (large diameter down) here on Earth, and it was energized by feeding RF into a slot/probe located at the large diameter. Would an observer at the small end notice a red shift?
Would seem to me that if the thrust produced is horizontally, gravity can't have effect on it, besides bending it a teeny weeny bit. Time dilation at beginning and endpoint are same (same distance from gravity well), so in effect 0.

Vertically you could measure it, but the redshift from earth's gravity is already extremely difficult to measure, but if you could you could then detract that from the redshift value measured in the cavity.

Quote
so in effect 0.

So nonzero? Say outside the bandwidth of a very narrow bandwidth cavity?
To be completely honest, I don't know. But that's how I would theorize it. Please realize I am in no way a scientist or physicist. I am a simple IT technician that loves to gobble up threads like this :)

 It seems to me that if a warp bubble is actually created in the cavity, you would definitely measure a non-zero net result in all thrust directions. Maybe this way you can prove or disprove a warp bubble?
Title: Re: EM Drive Developments - related to space flight applications - Thread 2
Post by: Rodal on 05/03/2015 12:43 pm
Here Shawyer compares 7 devices, some classic EMDrives, some with just a dielectic & the superconducting LN cooled Cannae drive.

It is important to note the direction of generated thrust as dielectics apparently generate thrust in the opposite direction to that of EMDrives.

I do note the EW test of a Cannae like device was not tested at cryo temp nor had superconducting interior lining and had an added dielectic. So was not a test of a true Cannae device.

From Shawyer, EW should not expect to see any thrust from an EMDrive like test device when tested fixed, not allowed to accelerate, without added dielectic. Which is what was observed. Static thrust was not measured, with the EW EMDrive like device until a dielectic was added.

Appears EW have discovered a new way to gen thrust in a fixed device. Use a dielectric. From Shawyer device summary, line 1, it appears all that may be needed is a short section of resonate pipe with a dielectic stuck in one end. IE EW Cannae test device minus the cavity.

However as the dielectric thrust is weaker than the EMDrive thrust and in the opposite thrust direction to the classic EMDrive thrust direction, putting a dielectric into an accelerating EMDrive may reduce the overall delivered thrust and effective cavity Q as would be seen by the classic EMDrive operational mode.

Is Shawyer giving away spoilers?

(http://forum.nasaspaceflight.com/index.php?action=dlattach;topic=36313.0;attach=828231;image)

Thank you for your post, as it gives me the opportunity to ask again a couple of questions that are unanswered so far.  Perhaps you can answer them:

1) Shawyer reported in the above graph, that his Demonstrator engine is the only EM Drive so far that has shown forces in BOTH directions, towards the small end as well as towards the large end.

1a) What does Shawyer mean by this? Does he mean that the Demonstrator engine displayed, and he measured, forces simultaneously in both directions? But as the forces in both directions are almost equal, that would mean practically no net force.  That doesn't seem to make sense as the Demonstrator engine is the only one that had a force large enough that it could move the whole assembly as shown in a video:

https://www.youtube.com/watch?v=57q3_aRiUXs

1b) Did Shawyer measure a force towards the big end in some tests and a force towards the small end in other tests? If so, what made the difference between the tests? Is the change random, unpredictable and unexplained? (Hopefully not). If the change in force direction can be deliberately controlled, how was Shawyer able to change the force direction for the Demonstrator engine?  Did he have to move something? (like the location of the dielectric)? Was there a dielectric in the Demonstrator engine? Or did he just accomplish this change of force direction by changing the exciting frequency (and thereby changing the mode shape)?



2) Shawyer defines the measured "thrust force" in the opposite direction to the direction of motion of the EM Drive. This is completely the opposite of NASA Eagleworks that defines the thrust force in the same direction as the motion of the EM Drive.  Can you reconcile and make sense of these two opposite, contradictory definitions and measurements?

Thanks
Title: Re: EM Drive Developments - related to space flight applications - Thread 2
Post by: Rodal on 05/03/2015 12:57 pm
https://twitter.com/elonmusk/status/594756342641922048

But why in the world did @ElonMusk link to an article from August 2014? That is soooo last year.
I don't use twitter, but perhaps you (or somebody else in the forum) does.  If such is the case, can you (or somebody else) twitter him back a message informing him that this article contains much newer information ? :

http://www.nasaspaceflight.com/2015/04/evaluating-nasas-futuristic-em-drive/
Title: Re: EM Drive Developments - related to space flight applications - Thread 2
Post by: Mulletron on 05/03/2015 12:59 pm
Seems like these things need an initial push to get going. Once they're moving, they stay moving as long as they're powered.

The SPR Demonstrator is the odd ball. Is it possible that one is different in some way that it will go in whatever direction you push it?

I'm hopeful we can get Mr. Shawyer to participate here IF folks are civil.
Title: Re: EM Drive Developments - related to space flight applications - Thread 2
Post by: Mulletron on 05/03/2015 01:02 pm
https://twitter.com/elonmusk/status/594756342641922048

But why in the world did @ElonMusk link to an article from August 2014? That is soooo last year.
I don't use twitter, but perhaps you (or somebody else in the forum) does.  If such is the case, can you (or somebody else) twitter him back a message informing him that this article contains much newer information ? :

http://www.nasaspaceflight.com/2015/04/evaluating-nasas-futuristic-em-drive/

Done. Enjoying the lively discussion over there too.
Title: Re: EM Drive Developments - related to space flight applications - Thread 2
Post by: Rodal on 05/03/2015 01:09 pm
...
I'm hopeful we can get Mr. Shawyer to participate here IF folks are civil.
I would not bet on that.  The uncivil level of attack displayed by several new posters in this thread during the last few days has been such that I would not be surprised if Star-Drive decides not to post any further Eagleworks test information here.  These uncivil attacks are launched by anonymous posters that hide behind monickers without ever revealing their real names.  I very much doubt that these uncivil posters would dare to express themselves this way, face-to-face in a public presentation, as in an AIAA meeting, for example.  I very much doubt that they would express themselves that way in writing if they would do it under their real name.  It is certainly conduct never seen at professional meetings or in academia.
Title: Re: EM Drive Developments - related to space flight applications - Thread 2
Post by: Mulletron on 05/03/2015 01:13 pm
We can safely bet that the excellent moderators here will take good care of Mr. Shawyer so that he can explain how this thing works in his own words.
Title: Re: EM Drive Developments - related to space flight applications - Thread 2
Post by: Dmytry on 05/03/2015 01:14 pm
Speaking of 2.6 watt result, this is what the data looks like:

http://i.imgur.com/McG6ZPd.png

Huge "drift". The only thing that stays consistent across the power levels, resonant modes, and quality factor, is that the force is comparable to "drifts".

If the drift is due to heating the flat spring that the pendulum is suspended on, there will be multitude of temperature differential effects as well, with different time constants (and a time constant for a thermal effect in a small piece of metal, sinking heat into other metal, absolutely can be a fraction of a second).

By the way note that even though the turn off looks as steep as the calibration pulse - to the eye - it clearly isn't (much less undershoot. Seems like the magnetic effect dies out instantly and makes the thrust curve look steeper than it is).

There's that ball bearing motor, spins pretty damn fast off thermal effects, supposedly, and puzzled the scientists for ages (with all sorts of novel physics explanations because some physicists just didn't feel that thermal effects can be fast).

By the way, GHz-frequency vibrations are a thing. It is possible (albeit still unlikely) to discover a new, entirely classical effect here.
Title: Re: EM Drive Developments - related to space flight applications - Thread 2
Post by: TheTraveller on 05/03/2015 01:21 pm
Here Shawyer compares 7 devices, some classic EMDrives, some with just a dielectic & the superconducting LN cooled Cannae drive.

It is important to note the direction of generated thrust as dielectrics apparently generate thrust in the opposite direction to that of EMDrives.

I do note the EW test of a Cannae like device was not tested at cryo temp nor had superconducting interior lining and had an added dielectic. So was not a test of a true Cannae device.

From Shawyer, EW should not expect to see any thrust from an EMDrive like test device when tested fixed, not allowed to accelerate, without added dielectic. Which is what was observed. Static thrust was not measured, with the EW EMDrive like device until a dielectic was added.

Appears EW have discovered a new way to gen thrust in a fixed device. Use a dielectric. From Shawyer device summary, line 1, it appears all that may be needed is a short section of resonate pipe with a dielectic stuck in one end. IE EW Cannae test device minus the cavity.

However as the dielectric thrust is weaker than the EMDrive thrust and in the opposite thrust direction to the classic EMDrive thrust direction, putting a dielectric into an accelerating EMDrive may reduce the overall delivered thrust and effective cavity Q as would be seen by the classic EMDrive operational mode.

Is Shawyer giving away spoilers?

(http://forum.nasaspaceflight.com/index.php?action=dlattach;topic=36313.0;attach=828231;image)

Thank you for your post, as it gives me the opportunity to ask again a couple of questions that are unanswered so far.  Perhaps you can answer them:

1) Shawyer reported in the above graph, that his Demonstrator engine is the only EM Drive so far that has shown forces in BOTH directions, towards the small end as well as towards the large end.

1a) What does Shawyer mean by this? Does he mean that the Demonstrator engine displayed, and he measured, forces simultaneously in both directions? But as the forces in both directions are almost equal, that would mean practically no net force.  That doesn't seem to make sense as the Demonstrator engine is the only one that had a force large enough that it could move the whole assembly as shown in a video:

https://www.youtube.com/watch?v=57q3_aRiUXs

1b) Did Shawyer measure a force towards the big end in some tests and a force towards the small end in other tests? If so, what made the difference between the tests? Is the change random, unpredictable and unexplained? (Hopefully not). If the change in force direction can be deliberately controlled, how was Shawyer able to change the force direction for the Demonstrator engine?  Did he have to move something? (like the location of the dielectric)? Was there a dielectric in the Demonstrator engine? Or did he just accomplish this change of force direction by changing the exciting frequency (and thereby changing the mode shape)?



2) Shawyer defines the measured "thrust force" in the opposite direction to the direction of motion of the EM Drive. This is completely the opposite of NASA Eagleworks that defines the thrust force in the same direction as the motion of the EM Drive.  Can you reconcile and make sense of these two opposite, contradictory definitions and measurements?

Thanks
The device you pictured is the 1st Dynamic Test unit (data apparently not in the summary):
http://emdrive.com/dynamictests.html

His portable Demonstrator device is here:
http://emdrive.com/demonstratorengine.html
Why he shows thrust in both direction is unknown to me.
I assume he can control the Demonstrator device in such a way to shift the thrust direction. It does have the ability to dynamically control cavity length.

Likewise his Flight Thruster is here:
http://emdrive.com/flightprogramme.html

The Shawyer EMDrive thrust direction is consistent in one mode of the line 4 device (Demonstrator), in line 5 (Chinese device), 6 (Flight Thruster).

The 3 dielectric devices and the Cannae cryo cooled, superconducting device generate thrust in the same direction and opposite to the classic EMDrive.

Additionally from the EW tests, apparently the addition of a dielectric to their EMDrive like device and the Cannae like device allows weak thrust to be generated without needing the device to accelerate.

Would be interesting to see EW chop off the Cannae test device and test it again with just the RF feed pipe, with of course the dielectric. Could be a very simple and quick test device to build and if it can generate thrust without moving (as it seem to do in the EMDrive/Cannae like devices), so much the better.

I really like the Shawyer Demonstrator device (attached). As an engineer I can appreciate designing, building and testing it plus the hours and money involved. It is a serious and professionally built device. It is not a toy but a real working thruster that can be taken anywhere to have additional tests done.

Maybe if EW asked Shawyer nicely, they could test it? At least then they have an established test data base and working device to work from.
Title: Re: EM Drive Developments - related to space flight applications - Thread 2
Post by: Rodal on 05/03/2015 01:24 pm
Actually, the level of what the experimenters call "thermal drift" for the 2.6 watt experiment is significantly less than for other, higher power experimental results, as evident per the Brady et.al "Anomalous ..." results.
Title: Re: EM Drive Developments - related to space flight applications - Thread 2
Post by: TheTraveller on 05/03/2015 01:25 pm
Might be useful to listen to what Shawyer has to say:
http://www.emdrive.com/interview.html

Interesting that around 2006, the UK gov transferred all his tech to Boeing. 3:30 in the 1st interview video. Would not think that would happen if there was nothing of interest to the US/Boeing.
Title: Re: EM Drive Developments - related to space flight applications - Thread 2
Post by: Rodal on 05/03/2015 01:30 pm
Might be useful to listen to what Shawyer has to say:
http://www.emdrive.com/interview.html
I listened to those some time ago, and my recollection is that he does not answer this question in that inverview, that still remains unanswered:

How did Shawyer achieve thrust in opposite directions for the same Demonstrator engine?

Title: Re: EM Drive Developments - related to space flight applications - Thread 2
Post by: TheTraveller on 05/03/2015 01:38 pm
Might be useful to listen to what Shawyer has to say:
http://www.emdrive.com/interview.html
I listened to those some time ago, and my recollection is that he does not answer this question in that inverview, that still remains unanswered:

How did Shawyer achieve thrust in opposite directions for the same Demonstrator engine?
We need to ask Shawyer as maybe a typo.

Maybe Mulletron can ask him as he has email contact and the data is in the public domain?
Title: Re: EM Drive Developments - related to space flight applications - Thread 2
Post by: TheTraveller on 05/03/2015 01:40 pm
We can safely bet that the excellent moderators here will take good care of Mr. Shawyer so that he can explain how this thing works in his own words.
Please make that happen.

Imagine EW testing his portable EMDrive demonstrator. That would make my day.
http://emdrive.com/demonstratorengine.html
Q of 45,000 is impressive.
Title: Re: EM Drive Developments - related to space flight applications - Thread 2
Post by: Rodal on 05/03/2015 01:52 pm
Might be useful to listen to what Shawyer has to say:
http://www.emdrive.com/interview.html
I listened to those some time ago, and my recollection is that he does not answer this question in that inverview, that still remains unanswered:

How did Shawyer achieve thrust in opposite directions for the same Demonstrator engine?
We need to ask Shawyer as maybe a typo.

Maybe Mulletron can ask him as he has email contact and the data is in the public domain?

I don't think that we can dismiss this as a typo (and therefore I agree with you that it should be pursued with a question to Shawyer) because in a previous discussion in this thread I had addressed this question to Star-Drive, who answered that as far as he knew there were two ways to change the direction of thrust of EM Drives: a) to change the location of the dielectric polymer insert and b) to change the mode shape.  Star-Drive posted his experimental evidence for a) and wrote that he had no experimental evidence for b) which (to my mind) means that this is information he may know from other experiments.
Title: Re: EM Drive Developments - related to space flight applications - Thread 2
Post by: TheTraveller on 05/03/2015 02:12 pm
Might be useful to listen to what Shawyer has to say:
http://www.emdrive.com/interview.html
I listened to those some time ago, and my recollection is that he does not answer this question in that inverview, that still remains unanswered:

How did Shawyer achieve thrust in opposite directions for the same Demonstrator engine?
We need to ask Shawyer as maybe a typo.

Maybe Mulletron can ask him as he has email contact and the data is in the public domain?

I don't think that we can dismiss this as a typo (and therefore I agree with you that it should be pursued with a question to Shawyer) because in a previous discussion in this thread I had addressed this question to Star-Drive, who answered that as far as he knew there were two ways to change the direction of thrust of EM Drives: a) to change the location of the dielectric polymer insert and b) to change the mode shape.  Star-Drive posted his experimental evidence for a) and wrote that he had no experimental evidence for b) which (to my mind) means that this is information he may know from other experiments.
We know from the 2014 summary that there was no dielectric in the Demonstrator. From the SPR site data on the Demonstrator Engine we know it can alter it's cavity length to assist tuning but that should not alter thrust direction.

What now seems clear is dielectrics can generate thrust in a static device and in the opposite direction to a classic EMDrive. That should tell us more about what is going on as in classic Shawyer EMDrive it must move to gen thrust and with just a dielectric in a tuned pipe (Cannae test device minus the cavity) we should get thrust without movement. Very different effects going on here.

This matches with what Shawyer said in his email that dielectrics, in a classic EMDrive, increase losses and reduce Q. Dielectric generated thrust, in the opposite direction, would sure do that.
Title: Re: EM Drive Developments - related to space flight applications - Thread 2
Post by: Notsosureofit on 05/03/2015 02:13 pm
So far, no mode change can reverse direction w/o dielectric in the cavity.

Still working on it, but the reflection from the dielectric surface and the dielectric properties vs frequency are the relevant parameters.
Title: Re: EM Drive Developments - related to space flight applications - Thread 2
Post by: TheTraveller on 05/03/2015 02:28 pm
90 seconds of thrust at 326mN/kW over a power input range of 150w to 450W is not something that is hard to test for.
SPR say they built & qualified this Flight Thruster for Boeing.
http://www.emdrive.com/flightprogramme.html

So where is it? Why don't EW have it to test?
Title: Re: EM Drive Developments - related to space flight applications - Thread 2
Post by: Rodal on 05/03/2015 02:33 pm
90 seconds of thrust at 326mN/kW over a power input range of 150w to 450W is not something that is hard to test for.
SPR say they built & qualified this Flight Thruster for Boeing.
http://www.emdrive.com/flightprogramme.html

So where is it? Why don't EW have it to test?

My recollection is that it was earlier reported in these threads that Boeing decided to discontinue any further work on it.  Do you have any further (or different) information ?

Concerning Eagleworks working with Shawyer, Eagleworks is already under severe budget constraints, for NASA to enter into an R&D agreement with a foreign company pursuing intellectual property rights is not a trivial endeavor as it would require IP negotiations and agreement from NASA's IP department, so that is fully understandable.
Title: Re: EM Drive Developments - related to space flight applications - Thread 2
Post by: Consultant on 05/03/2015 02:38 pm
Hi,

I have two questions:

1. if the pendulum that measures the force is made of metal, is it properly shielded from electro magnetism?
2. why, in the second run there was much less thrust? (this was an experiment where more measures were taken against confounding variables, so does the drop in thrust indicate any confounding variable to have played a major role in the thrust measured in the first experment?
91.2 µN at 17 W of input power
50 µN with 50 W of input power
thanks,
Siem
Title: Re: EM Drive Developments - related to space flight applications - Thread 2
Post by: Rodal on 05/03/2015 02:41 pm
...Nobody here is addressing the frame problem either.
I am not addressing the frame problem at this point in time because I think it is very premature to deal with future applications (for which the frame problem I agree is indeed important to address) when we are still discussing the experiments and the working theory.

Can you address the frame problem for self-accelerating particles (shown to be a valid solution of Schrodinger's equation, for at least 36 years) ?

 http://newsoffice.mit.edu/2015/self-accelerating-particles-0120

http://www.nature.com/ncomms/2014/141030/ncomms6189/abs/ncomms6189.html
Title: Re: EM Drive Developments - related to space flight applications - Thread 2
Post by: TheTraveller on 05/03/2015 02:43 pm
90 seconds of thrust at 326mN/kW over a power input range of 150w to 450W is not something that is hard to test for.
SPR say they built & qualified this Flight Thruster for Boeing.
http://www.emdrive.com/flightprogramme.html

So where is it? Why don't EW have it to test?

My recollection is that it was earlier reported in these threads that Boeing decided to discontinue any further work on it.  Do you have any further (or different) information ?

Concerning Eagleworks working with Shawyer, Eagleworks is already under severe budget constraints, for NASA to enter into an R&D agreement with a foreign company pursuing intellectual property rights is not a trivial endeavor as it would require IP negotiations and agreement from NASA's IP department, so that is fully understandable.
Yes I know of the Boeing reports. As Shawyer said in his vid, "It Went Dark".

Boeing/DARPA provided a SFE (Serrano Force Effect) device to EW to test (lower left attached image). Which they did and EW reported thrust. Seems fairly easy for Boeing to send a SPR Flight Thruster to EW to test?

Interesting EW shows the SPR Flight Thruster and calls it "High Fidelity". So why not test it?
Title: Re: EM Drive Developments - related to space flight applications - Thread 2
Post by: Rodal on 05/03/2015 02:46 pm
... Seems fairly easy for Boeing to send a SPR Flight Thruster to EW to test?
No, NASA cannot just "accept" a SPR Flight Thruster to test just like that, when SPR is a foreign company pursuing intellectual property rights.  Even Ford Motor Company will not accept any letter from any inventor disclosing an invention and much less accept to test in their R&D department a rig from a company pursuing IP rights.  It would need to involve an IP agreement.
Title: Re: EM Drive Developments - related to space flight applications - Thread 2
Post by: Stormbringer on 05/03/2015 02:48 pm
Hi,

I have two questions:

1. if the pendulum that measures the force is made of metal, is it properly shielded from electro magnetism?
2. why, in the second run there was much less thrust? (this was an experiment where more measures were taken against confounding variables, so does the drop in thrust indicate any confounding variable to have played a major role in the thrust measured in the first experment?
91.2 µN at 17 W of input power
50 µN with 50 W of input power
thanks,
Siem
For item one i can help. It is not a pendulum. It is a torsion balance. Great pains have been taken to eliminate all sources of error including electromagnetic related effects or at least quantify them and deduct them from the data as much as possible. You cannot be sure though that there isn't some factor you have not thought of at work. But with all the scrutiny this is getting and has got you can be sure that the probability of some unknown but mundane source of error is pretty low.
Title: Re: EM Drive Developments - related to space flight applications - Thread 2
Post by: TheTraveller on 05/03/2015 02:54 pm
... Seems fairly easy for Boeing to send a SPR Flight Thruster to EW to test?
No, NASA cannot just "accept" a SPR Flight Thruster to test just like that, when SPR is a foreign company pursuing intellectual property rights.  Even Ford Motor Company will not accept any letter from any inventor disclosing an invention and much less accept to test in their R&D department a rig from a company pursuing IP rights.  It would involve an IP agreement.
Except all the SPR tech was acquired by Boeing. So you are saying that if Boeing wanted to ask EW to test a Flight Thruster they had in their possession and was their paid for property, they could not do it? Surely Boeing can use other test labs to verify claimed performance? I mean Boeing did acquire all the SPR tech. Doubt they would do that without being sure it worked. Which implies it was tested and probably by several labs.

Note that EW calls the SPR Flight Thruster a "High Fidelity Test Article". Must mean someone has tested it and verified it works as claimed?
Title: Re: EM Drive Developments - related to space flight applications - Thread 2
Post by: Rodal on 05/03/2015 02:59 pm
... Seems fairly easy for Boeing to send a SPR Flight Thruster to EW to test?
No, NASA cannot just "accept" a SPR Flight Thruster to test just like that, when SPR is a foreign company pursuing intellectual property rights.  Even Ford Motor Company will not accept any letter from any inventor disclosing an invention and much less accept to test in their R&D department a rig from a company pursuing IP rights.  It would involve an IP agreement.
....
Yes, based on my experience being in charge of R&D departments in private companies and managing Intellectual Property negotiations and litigation, I would expect that for NASA to test devices from companies pursuing intellectual property rights it would involve a negotiation of an IP agreement.
Title: Re: EM Drive Developments - related to space flight applications - Thread 2
Post by: TheTraveller on 05/03/2015 03:08 pm
... Seems fairly easy for Boeing to send a SPR Flight Thruster to EW to test?
No, NASA cannot just "accept" a SPR Flight Thruster to test just like that, when SPR is a foreign company pursuing intellectual property rights.  Even Ford Motor Company will not accept any letter from any inventor disclosing an invention and much less accept to test in their R&D department a rig from a company pursuing IP rights.  It would involve an IP agreement.
....
Yes, based on my experience being in charge of R&D departments in private companies and managing Intellectual Property negotiations and litigation, I would expect that for NASA to test devices from companies pursuing intellectual property rights it would involve a negotiation of an IP agreement.
The SPR/Boeing deal was done and dusted in 2010.
Boeing sent the SFE device to EW in 2013.
Doubt Shawyer could object to Boeing sending it to EW to test?
Point being Boeing has a SPR Flight Certified EMDrive thruster in their possession. EW call it a "High Fidelity Test Article". EW has pictures of it. Yet they mess around with mosquito thrust EMDrive clones with embedded dielectrics, trying to get thrust in a static test rig.

BTW I have talked to Serrano and know the history of that device and test run. It produced torque and not linear thrust. He has a newer linear thrust device that will be tested very soon.
Title: Re: EM Drive Developments - related to space flight applications - Thread 2
Post by: TheTraveller on 05/03/2015 03:12 pm

Yes, based on my experience being in charge of R&D departments in private companies and managing Intellectual Property negotiations and litigation, I would expect that for NASA to test devices from companies pursuing intellectual property rights it would involve a negotiation of an IP agreement.
Are you saying SPR is currently pursuing Boeing for intellectual property rights? My info says the deal was done and dusted many years ago and Shawyer has moved on.

What do you know?
Title: Re: EM Drive Developments - related to space flight applications - Thread 2
Post by: ppnl on 05/03/2015 03:16 pm
...Nobody here is addressing the frame problem either.
I am not addressing the frame problem at this point in time because I think it is very premature to deal with future applications (for which the frame problem I agree is indeed important to address) when we are still discussing the experiments and the working theory.

Can you address the frame problem for self-accelerating particles (shown to be a valid solution of Schrodinger's equation, for at least 36 years) ?

 http://newsoffice.mit.edu/2015/self-accelerating-particles-0120

http://www.nature.com/ncomms/2014/141030/ncomms6189/abs/ncomms6189.html

You cannot address the theory without addressing the frame problem.

And no I can't address the frame problem of the self accelerating particle. At least not from a verbal description. But I bet dollars to donuts that they can. If they can't their work is nonsense.

Actually I doubt there is a frame problem involved.
Title: Re: EM Drive Developments - related to space flight applications - Thread 2
Post by: Rodal on 05/03/2015 03:36 pm
...Nobody here is addressing the frame problem either.
I am not addressing the frame problem at this point in time because I think it is very premature to deal with future applications (for which the frame problem I agree is indeed important to address) when we are still discussing the experiments and the working theory.

Can you address the frame problem for self-accelerating particles (shown to be a valid solution of Schrodinger's equation, for at least 36 years) ?

 http://newsoffice.mit.edu/2015/self-accelerating-particles-0120

http://www.nature.com/ncomms/2014/141030/ncomms6189/abs/ncomms6189.html

You cannot address the theory without addressing the frame problem.

And no I can't address the frame problem of the self accelerating particle. At least not from a verbal description. But I bet dollars to donuts that they can. If they can't their work is nonsense.

Actually I doubt there is a frame problem involved.

The center of gravity for self-accelerating particles cannot be defined, because the Airy function is not square integrable: it corresponds to an infinite number of particles, just like the plane wave and other wave functions in scattering theory.

Both T-invariance and P-invariance are broken in this Schrodinger solution system, it does not admit a Lagrangian formulation.
Title: Re: EM Drive Developments - related to space flight applications - Thread 2
Post by: ppnl on 05/03/2015 03:42 pm
...Nobody here is addressing the frame problem either.
I am not addressing the frame problem at this point in time because I think it is very premature to deal with future applications (for which the frame problem I agree is indeed important to address) when we are still discussing the experiments and the working theory.

Can you address the frame problem for self-accelerating particles (shown to be a valid solution of Schrodinger's equation, for at least 36 years) ?

 http://newsoffice.mit.edu/2015/self-accelerating-particles-0120

http://www.nature.com/ncomms/2014/141030/ncomms6189/abs/ncomms6189.html

You cannot address the theory without addressing the frame problem.

And no I can't address the frame problem of the self accelerating particle. At least not from a verbal description. But I bet dollars to donuts that they can. If they can't their work is nonsense.

Actually I doubt there is a frame problem involved.

The center of gravity for self-accelerating particles cannot be defined, because the Airy function is not square integrable: it corresponds to an infinite number of particles, just like the plane wave and other wave functions in scattering theory.

Great, but unless it behaves differently depending on what frame of reference it is in it isn't a frame problem. For example if it works differently on Mars than on Earth because of the different orbital velocity.
Title: Re: EM Drive Developments - related to space flight applications - Thread 2
Post by: RonM on 05/03/2015 03:43 pm
...Nobody here is addressing the frame problem either.
I am not addressing the frame problem at this point in time because I think it is very premature to deal with future applications (for which the frame problem I agree is indeed important to address) when we are still discussing the experiments and the working theory.

Can you address the frame problem for self-accelerating particles (shown to be a valid solution of Schrodinger's equation, for at least 36 years) ?

 http://newsoffice.mit.edu/2015/self-accelerating-particles-0120

http://www.nature.com/ncomms/2014/141030/ncomms6189/abs/ncomms6189.html

You cannot address the theory without addressing the frame problem.

And no I can't address the frame problem of the self accelerating particle. At least not from a verbal description. But I bet dollars to donuts that they can. If they can't their work is nonsense.

Actually I doubt there is a frame problem involved.

Yes, the frame problem is very important, probably indicating the theories on how this works are incorrect.

However, we are getting the cart before the horse. This theory controversy goes away if more precise, totally isolated from the environment, experiments are connected and no anomalous force is found.

If there really is a force being produced, I'm confident that a theory can be developed that does not throw the past hundred years of physics out the window because current theory is extremely successful in describing the universe. Current theory is not complete, but we know it is not wrong.
Title: Re: EM Drive Developments - related to space flight applications - Thread 2
Post by: Rodal on 05/03/2015 03:47 pm
...Nobody here is addressing the frame problem either.
I am not addressing the frame problem at this point in time because I think it is very premature to deal with future applications (for which the frame problem I agree is indeed important to address) when we are still discussing the experiments and the working theory.

Can you address the frame problem for self-accelerating particles (shown to be a valid solution of Schrodinger's equation, for at least 36 years) ?

 http://newsoffice.mit.edu/2015/self-accelerating-particles-0120

http://www.nature.com/ncomms/2014/141030/ncomms6189/abs/ncomms6189.html

You cannot address the theory without addressing the frame problem.

And no I can't address the frame problem of the self accelerating particle. At least not from a verbal description. But I bet dollars to donuts that they can. If they can't their work is nonsense.

Actually I doubt there is a frame problem involved.

The center of gravity for self-accelerating particles cannot be defined, because the Airy function is not square integrable: it corresponds to an infinite number of particles, just like the plane wave and other wave functions in scattering theory.

Great, but unless it behaves differently depending on what frame of reference it is in it isn't a frame problem. For example if it works differently on Mars than on Earth because of the different orbital velocity.
OK we fully agree on that.  But why is it necessary that if the EM Drive were to "work" as space propulsion in some restricted sense, that it necessarily would involve a frame problem? For example, the EM Drive could be given experimental force measurements just due to outgassing (which also would work in space -for a reduced amount of time  :) ), which is a perfectly classical explanation.  There are many other theories besides Dr. White's and Shawyer's.
Title: Re: EM Drive Developments - related to space flight applications - Thread 2
Post by: vide on 05/03/2015 04:14 pm
I'm sorry if this question is covered somewhere else, but is there a list of recent force plots that were posted on this forum?
Title: Re: EM Drive Developments - related to space flight applications - Thread 2
Post by: ppnl on 05/03/2015 04:18 pm
...Nobody here is addressing the frame problem either.
I am not addressing the frame problem at this point in time because I think it is very premature to deal with future applications (for which the frame problem I agree is indeed important to address) when we are still discussing the experiments and the working theory.

Can you address the frame problem for self-accelerating particles (shown to be a valid solution of Schrodinger's equation, for at least 36 years) ?

 http://newsoffice.mit.edu/2015/self-accelerating-particles-0120

http://www.nature.com/ncomms/2014/141030/ncomms6189/abs/ncomms6189.html

You cannot address the theory without addressing the frame problem.

And no I can't address the frame problem of the self accelerating particle. At least not from a verbal description. But I bet dollars to donuts that they can. If they can't their work is nonsense.

Actually I doubt there is a frame problem involved.

The center of gravity for self-accelerating particles cannot be defined, because the Airy function is not square integrable: it corresponds to an infinite number of particles, just like the plane wave and other wave functions in scattering theory.

Great, but unless it behaves differently depending on what frame of reference it is in it isn't a frame problem. For example if it works differently on Mars than on Earth because of the different orbital velocity.
OK we fully agree on that.  But why is it necessary that if the EM Drive were to "work" as space propulsion in some restricted sense, that it necessarily would involve a frame problem? For example, the EM Drive could be given experimental force measurements just due to outgassing (which also would work in space -for a reduced amount of time  :) ), which is a perfectly classical explanation.  There are many other theories besides Dr. White's and Shawyer's.

Well if it works by outgassing then it is just a rocket and isn't very interesting.

If it works by reacting against something like the quantum vacuum you have to either violate relativity or COE.

You can make something up like it reacts against the local gravitational gradient. But that's just pure invention. That's a science fiction plot not a scientific theory. There are thousands of ways I could make something up ,to explain what I want to believe but thats doing science backwards.

Shawyer developed a theory and built a device to test it. If his theory is nonsense then the first conclusion we should reach is that his test results are nonsense. Either that or he is the luckiest person in the world. If he cannot see and address the violation of Galilean relativity then I wouldn't trust him to test a light bulb.
Title: Re: EM Drive Developments - related to space flight applications - Thread 2
Post by: vide on 05/03/2015 04:26 pm
It could react against the surrounding environment in a way that is not too sensitive to what the environment is.

I had a book of historical experiments... I don't have it any more but to my recollection, almost every important historical experiment had provided measurements with high degree of precision and repeatability, which helped to avoid the situation of many competing theories.
Title: Re: EM Drive Developments - related to space flight applications - Thread 2
Post by: KittyMoo on 05/03/2015 04:27 pm
Quote
Shawyer developed a theory and built a device to test it. If his theory is nonsense then the first conclusion we should reach is that his test results are nonsense. Either that or he is the luckiest person in the world. If he cannot see and address the violation of Galilean relativity then I wouldn't trust him to test a light bulb.
This is wrong in so many different ways I don't know where to start...
Title: Re: EM Drive Developments - related to space flight applications - Thread 2
Post by: Rodal on 05/03/2015 04:28 pm
...If his theory is nonsense then the first conclusion we should reach is that his test results are nonsense. Either that or he is the luckiest person in the world. ..
Science is full of experimental results that were found by accident, without meeting the theoretical expectations of the experimenter.  Do I need to name them? Probably not, I'm sure you know about them too.

You write about luck.  One can state that it is "unlikely" in some intuitive sense, but I haven't seen a Bayesian analysis of what is the probability (do you know of any such Bayesian analysis ?), and hence your assessment of what is the likelihood may be subjective (most humans cannot correctly distinguish what is random from what is not, and they can't assess what events are Gaussian or fat-tailed, as for example most professional traders incorrectly think that the probability of a random event like a crash in the stock market is much less likely than it really is, because they use Gaussian distributions when the real probability distribution is fat-tailed and because they incorrectly assume constant volatility)

In any case, the probability has to be based on at least three independent testing centers: the reason why this is being discussed in this forum, and I'm still here is not just because of Shawyer's experimental claims, but it is mainly due to NASA's Dr. White's experimental report and also due to Prof. Yang's experiments in China. 

Title: Re: EM Drive Developments - related to space flight applications - Thread 2
Post by: TheTraveller on 05/03/2015 04:29 pm
Might be useful to listen to what Shawyer has to say:
http://www.emdrive.com/interview.html
I listened to those some time ago, and my recollection is that he does not answer this question in that inverview, that still remains unanswered:

How did Shawyer achieve thrust in opposite directions for the same Demonstrator engine?
We need to ask Shawyer as maybe a typo.

Maybe Mulletron can ask him as he has email contact and the data is in the public domain?
Shawyer shows how the EM Drive can act as a Motor or Generator depending on the direction of movement relative to the EM Drive: http://www.emdrive.com/IAC13paper17254.v2.pdf

This may be where Shawyers 2 measurements come from. Either the EM Drive acting in Motor mode, increasing load kinetic energy or in Generator mode, decreading load kinetic energy.

As Shawyers says, an EM Drive in Generator mode would decelerate the spacecraft.

The results shown in line 4 are the forces one would measure if you say pushed the EM Drive in one direction or the other. Push it one way & it accelerates away from you, now being in Motor mode. Push it the other way and it fights your push, now being in Generator mode.
Title: Re: EM Drive Developments - related to space flight applications - Thread 2
Post by: TheTraveller on 05/03/2015 04:33 pm
I'm sorry if this question is covered somewhere else, but is there a list of recent force plots that were posted on this forum?
These?
Title: Re: EM Drive Developments - related to space flight applications - Thread 2
Post by: SH on 05/03/2015 04:33 pm
If it works by reacting against something like the quantum vacuum you have to either violate relativity or COE.

Interacting with the quantum vacuum doesn't require violating COE or COM, but it still wouldn't generate any thrust without making additional assumptions that are even harder to swallow, as I discuss here:

In order for White's proposed explanation to make any sense at all, you need to assume 3 things: ...
Title: Re: EM Drive Developments - related to space flight applications - Thread 2
Post by: vide on 05/03/2015 04:41 pm
...If his theory is nonsense then the first conclusion we should reach is that his test results are nonsense. Either that or he is the luckiest person in the world. ..
Science is full of experimental results that were found by accident, without meeting the theoretical expectations of the experimenter.  Do I need to name them? Probably not, I'm sure you know about them too.

You can state that it is "unlikely" in some sense, but I haven't seen a Bayesian analysis of what is the probability (do you know of any such Bayesian analysis ?).

In any case, whatever the probability has to be based on at least three independent testing centers: the reason why this is being discussed in this forum, and I'm still here is not just because of Shawyer's experimental claims, but it is mainly due to NASA's Dr. White's report and also due to Prof. Yang's experiments in China.
Is there a quantitative agreement between reports in respect of certain variables?
Title: Re: EM Drive Developments - related to space flight applications - Thread 2
Post by: Flyby on 05/03/2015 04:51 pm
...
I'm hopeful we can get Mr. Shawyer to participate here IF folks are civil.
I would not bet on that.  The uncivil level of attack displayed by several new posters in this thread during the last few days has been such that I would not be surprised if Star-Drive decides not to post any further Eagleworks test information here.  These uncivil attacks are launched by anonymous posters that hide behind monickers without ever revealing their real names.  I very much doubt that these uncivil posters would dare to express themselves this way, face-to-face in a public presentation, as in an AIAA meeting, for example.  I very much doubt that they would express themselves that way in writing if they would do it under their real name.  It is certainly conduct never seen at professional meetings or in academia.

I must say I'm rather perplexed and puzzled by the sudden hostility that some of the newer participants demonstrate...
There is absolutely nothing wrong with being skeptic but i thought that curiosity was one of the key characteristics of being a scientist/researcher/engineer.
There is no better way to make progress then to have a thesis and antithesis collide in a civil manner.

I really do not understand what can be obtained or achieved by aggressively attacking people or their ideas. If you aim to disprove a theory then all you need to do is mass enough evidence that their theory is flawed.

Personally I find it still too early trying to come up with theories until the effect has been validated or not.
For me the most compelling evidence so far is still the 2007 rotating Demonstrator video by R Shawyer.
All we can do now is wait for Eagleworks to duplicate that test (and improve on some of the possible setup flaws, like hot jet exhaust nullification).

If the test fails, then the credibility of the device will get a serious hit...
If the test succeeds, it will most likely be a turning point in the research (and funding) of the EM drive.

So, instead of shooting lead at each other, why don't we just all relax and be supportive to the Eagleworks team so they can finalize that crucial test by July?

Pro or contra, you'll have your answer by July...

That said, it was to foresee that giving more publicity to the research through the NASA publication article, would attract some of the most aggressive opinionated people inhere.
On the positive side however, it also attracted some very much needed new participants that have clearly high level qualifications... (be them pro or contra, it doesn't matter)

I suppose it is up to the mods to weed out the offensive ones...

As for mr Shawyer, i think it is already obvious that he will not engage into the discussion here, partially because of the engagements he already has with other parties, as he explained in that private conversation, partially because he had his share of abusive language in the past....


....eagerly anticipating the next , high power test from Eagleworks... 8)
Title: Re: EM Drive Developments - related to space flight applications - Thread 2
Post by: frobnicat on 05/03/2015 04:53 pm
I'm sorry if this question is covered somewhere else, but is there a list of recent force plots that were posted on this forum?

Those are all the force plots I'm aware of concerning the frustum test article at Eagleworks

Source file name or figure in 2014 Brady report :
1/ Page 15, Fig 19 top
2/ Page 15, Fig 19 middle
3/ Page 15, Fig 19 bottom
4/ Page 17, Fig 21 top
5/ Page 17, Fig 21 bottom
6/ Page 18, Fig 22 Eagleworks Lab Conical Resonant Cavity Test Article Data-1, TE012 Mode_03-06-2014.jpg
7/ July 01, 2014 Copper Frustum PTFE_901.93MHz TM010 Thrust Signal in-Air.jpg
8/ March 2015 Copper Frustum 1,937.15MHz_50W_Forward_in-Air_Foam-Board Encl_Data Run.jpg
9/ Frustrum 2 energy and pressure Plus Lab Data_Polycarbonate-1_May 07, 2014.jpg
10/ Copper Kettle Data-Runs_TM212_12-12-2014A_5x10-6 Torr_50W.jpg
11/ Jan 16 2015_ Copper Frustum 1,937.188MHz in-5x10-4 Torr_35W_Reversed_Data Run-1B.jpg

Of those only the last 2 are results in vacuum

There is also
12/ Jan 2015 Copper Frustum 1,937.15MHz in-Air_50W_Forward & Reversed_Data Runs.jpg
but I'm unsure the forward data is different from one of the previous forward plots

The file names are the ones as downloaded from the attached pictures from Paul March contributions to the thread, they should show up when searching directly for them with goggles.
I'm looking for a way to tabulate all the corresponding data together with a way to consult the plots either thumbnailed or full size, if anyone has time and patience it would be a great contribution to gather and sum up all the "raw" data together with link to sources when parameters have been explained through posts.

WIP (badly presented and too early) : attached is a first attempt to spreadsheet the data with some very rough eyeballed values... Last 2 columns are derived values : thrust/power and thrust/power/(vertical scale in µN/µm) times a factor to have same mean as thrust/power. Overall, thrust/power µN/W has more standard deviation (5.63) than the one with µm/W units (2.84). Sorry, at this stage this is more just like numerology.

Title: Re: EM Drive Developments - related to space flight applications - Thread 2
Post by: Rodal on 05/03/2015 04:55 pm
...

Shawyer wrote (to @Mulletron, in the communication where Shawyer kindly scanned Cullen's paper  http://forum.nasaspaceflight.com/index.php?topic=36313.msg1368066#msg1368066 ) about why the posters in this thread had such a hard time understanding his EM Drive theoretical model.  Given this reaction, it appears that Shawyer knows of scientists that understand his model.  Which leads to this question: do you know of scientists that agree with, or further elaborate on Shaywer's model?  It would be interesting to read any such supporting papers that agree with Shawyer's model, as they may contain alternative explanations of his theoretical model.  If so, can you reference or link to such papers supporting his theoretical model?

Concerning Cullens' paper, I have been reading it, and I will post my present understanding.
Title: Re: EM Drive Developments - related to space flight applications - Thread 2
Post by: Mulletron on 05/03/2015 05:18 pm
If it works by reacting against something like the quantum vacuum you have to either violate relativity or COE.

Interacting with the quantum vacuum doesn't require violating COE or COM, but it still wouldn't generate any thrust without making additional assumptions that are even harder to swallow, as I discuss here:

In order for White's proposed explanation to make any sense at all, you need to assume 3 things: ...

There's other perfectly sane ways of interacting with the QV which have been posted literally a thousand times in THIS forum over and over again since October.
Title: Re: EM Drive Developments - related to space flight applications - Thread 2
Post by: Mulletron on 05/03/2015 05:23 pm
...
I'm hopeful we can get Mr. Shawyer to participate here IF folks are civil.
I would not bet on that.  The uncivil level of attack displayed by several new posters in this thread during the last few days has been such that I would not be surprised if Star-Drive decides not to post any further Eagleworks test information here.  These uncivil attacks are launched by anonymous posters that hide behind monickers without ever revealing their real names.  I very much doubt that these uncivil posters would dare to express themselves this way, face-to-face in a public presentation, as in an AIAA meeting, for example.  I very much doubt that they would express themselves that way in writing if they would do it under their real name.  It is certainly conduct never seen at professional meetings or in academia.

I must say I'm rather perplexed and puzzled by the sudden hostility that some of the newer participants demonstrate...
There is absolutely nothing wrong with being skeptic but i thought that curiosity was one of the key characteristics of being a scientist/researcher/engineer.
There is no better way to make progress then to have a thesis and antithesis collide in a civil manner.

I really do not understand what can be obtained or achieved by aggressively attacking people or their ideas. If you aim to disprove a theory then all you need to do is mass enough evidence that their theory is flawed.

Personally I find it still too early trying to come up with theories until the effect has been validated or not.
For me the most compelling evidence so far is still the 2007 rotating Demonstrator video by R Shawyer.
All we can do now is wait for Eagleworks to duplicate that test (and improve on some of the possible setup flaws, like hot jet exhaust nullification).

If the test fails, then the credibility of the device will get a serious hit...
If the test succeeds, it will most likely be a turning point in the research (and funding) of the EM drive.

So, instead of shooting lead at each other, why don't we just all relax and be supportive to the Eagleworks team so they can finalize that crucial test by July?

Pro or contra, you'll have your answer by July...

That said, it was to foresee that giving more publicity to the research through the NASA publication article, would attract some of the most aggressive opinionated people inhere.
On the positive side however, it also attracted some very much needed new participants that have clearly high level qualifications... (be them pro or contra, it doesn't matter)

I suppose it is up to the mods to weed out the offensive ones...

As for mr Shawyer, i think it is already obvious that he will not engage into the discussion here, partially because of the engagements he already has with other parties, as he explained in that private conversation, partially because he had his share of abusive language in the past....


....eagerly anticipating the next , high power test from Eagleworks... 8)

I think we need to ignore the handwaving when it comes by and make sure we put to full use all the info that Shawyer has provided us, with a focus on the engineering aspects and operational details. We shouldn't get hung up on his theory.

Quoting @TheTraveller:
Quote
Shawyer has been doing this for longer than anybody. My eng gut says follow Shaywer's lead as he has already gone down many dead ends. Why try to reinvent the wheel and repeat his failures?
Title: Re: EM Drive Developments - related to space flight applications - Thread 2
Post by: CW on 05/03/2015 05:39 pm
...
I'm hopeful we can get Mr. Shawyer to participate here IF folks are civil.
I would not bet on that.  The uncivil level of attack displayed by several new posters in this thread during the last few days has been such that I would not be surprised if Star-Drive decides not to post any further Eagleworks test information here.  These uncivil attacks are launched by anonymous posters that hide behind monickers without ever revealing their real names.  I very much doubt that these uncivil posters would dare to express themselves this way, face-to-face in a public presentation, as in an AIAA meeting, for example.  I very much doubt that they would express themselves that way in writing if they would do it under their real name.  It is certainly conduct never seen at professional meetings or in academia.

I must say I'm rather perplexed and puzzled by the sudden hostility that some of the newer participants demonstrate...
There is absolutely nothing wrong with being skeptic but i thought that curiosity was one of the key characteristics of being a scientist/researcher/engineer.
There is no better way to make progress then to have a thesis and antithesis collide in a civil manner.

I really do not understand what can be obtained or achieved by aggressively attacking people or their ideas. If you aim to disprove a theory then all you need to do is mass enough evidence that their theory is flawed.

Personally I find it still too early trying to come up with theories until the effect has been validated or not.
For me the most compelling evidence so far is still the 2007 rotating Demonstrator video by R Shawyer.
All we can do now is wait for Eagleworks to duplicate that test (and improve on some of the possible setup flaws, like hot jet exhaust nullification).

If the test fails, then the credibility of the device will get a serious hit...
If the test succeeds, it will most likely be a turning point in the research (and funding) of the EM drive.

So, instead of shooting lead at each other, why don't we just all relax and be supportive to the Eagleworks team so they can finalize that crucial test by July?

Pro or contra, you'll have your answer by July...

That said, it was to foresee that giving more publicity to the research through the NASA publication article, would attract some of the most aggressive opinionated people inhere.
On the positive side however, it also attracted some very much needed new participants that have clearly high level qualifications... (be them pro or contra, it doesn't matter)

I suppose it is up to the mods to weed out the offensive ones...

As for mr Shawyer, i think it is already obvious that he will not engage into the discussion here, partially because of the engagements he already has with other parties, as he explained in that private conversation, partially because he had his share of abusive language in the past....


....eagerly anticipating the next , high power test from Eagleworks... 8)

I think I can see where this aggressive stance comes from. It's rooted in how the human mind works. Each individual's world foundation is eventually based on belief. As a scientist, the state-of-the-art is that foundation. For some, even a tiny chance for this belief to prove not right, is (on a subconscious level) such a threatening thing, that this fear expresses itself as a possibly inadequate reaction to what's presented or happening. An individual's belief is the foundation of the world he/she is standing on, which everything else is based on. If that were to be removed, then in their mind it feels like the ground has vanished and they are about to hit a black hole's event horizon.. it would make their life, that they lived so far and full of conviction, a delusion. Not many can cope with this feeling well, in my experience.

On the one hand, I see the importance to also be a caretaker of what was proven to work. It's very valuable. And still - I see humans more as explorers than as caretakers. Religious structures are caretaking facilities. The progress of the scientific world view is not that we can now wirelessly download and watch cat videos in 4K on our smartphones. The progress is that we are not just caretakers anymore - we are explorers. We are adventurers of the mind. Let's try to take care of what is useful and works for us - but let's just see this as stepping stones to further heights of human insight into the wonders of nature.

My $0.02
-CW
Title: Re: EM Drive Developments - related to space flight applications - Thread 2
Post by: dustinthewind on 05/03/2015 05:42 pm

Not according to Maxwell's linear, isotropic equations.

The small base of the EM Drive is not open.  It is a closed cavity.  As such, the waves inside it are not travelling waves, but standing waves.   See this:  http://en.wikipedia.org/wiki/Standing_wave

The energy flux is pointed towards one end during half a (Poynting vector) period and it is pointed towards the opposite end during the next half-period.  Hence the net energy flux over a whole period is completely self-cancelling.

...



...the ac power in power lines can be modeled as standing waves but if no one is using power.  When power starts being consumed the standing waves begin to travel towards the object consuming the power.  The moving bulges of magnetic/electric field can be thought of as transporting power from the power station to the consumer.  There should be some traveling of the standing waves bulges from the power supply towards areas of heat loss in the cavity I would assume.  I can't say the power dissipated into heat loss is significant but it does seem to buck the perfect standing wave view for me a bit...

In ref to the image above, there is wealth of information buried within the earlier pages of this thread. A more accurate representation of what it looks like inside the cavity is available here:
http://forum.nasaspaceflight.com/index.php?topic=36313.msg1333246#msg1333246
Paul March was kind enough to attach the Frustrum modes overview 2A.pdf which has all the mode shapes and characteristics of their test article.

So things are a bit more complicated than photons bouncing around like marbles in a can. For example, I know that I can only excite TM212 and TM311 (thanks @Rodal for modeling this) with my little setup at home.
http://forum.nasaspaceflight.com/index.php?topic=36313.msg1353372#msg1353372
http://forum.nasaspaceflight.com/index.php?topic=36313.msg1352878#msg1352878

Paul March has shown clearly in his many posts that there is a clear correlation between mode shape and magnitude and direction of thrust. This is where input from RF Engineers would be extremely valuable.

Just food for thought, it is worth going back to page 1 and commenting on the stuff starting there. That way the conversation can keep building on ideas.

(http://forum.nasaspaceflight.com/index.php?action=dlattach;topic=36313.0;attach=820102;image)
(http://forum.nasaspaceflight.com/index.php?action=dlattach;topic=36313.0;attach=820104;image)
(http://forum.nasaspaceflight.com/index.php?action=dlattach;topic=36313.0;attach=796989;image)

...I think we may be losing information by the idea of the standing wave model with out considering the power losses (due heating of the cavity and any propulsion) and the transport of energy by [E^2+B^2] http://hyperphysics.phy-astr.gsu.edu/hbase/electric/engfie.html where the stading wave bulges tend to move towards the areas of consumed energy (heating + sparks + propulsion +ect).  I was paralleling it to energy consumption in power lines...

I agree in the line of  thinking of what is going on in the cavity as more of a wave model than thinking of them as marbles bouncing inside the cavity... 

It is interesting to note that as the wave modes travel towards the small end of the cavity they are being squeezed but if the wave peaks are traveling towards the larger end they are experiencing expansion.  It looks like as a result we see the increased B field near the tight end of the cavity and small B field near the big end.  I almost want to think of this squeezing as a form of propulsion in the form of resistance of the traveling of the semi-standing waves as they transport energy to areas of heat loss.


Can you please answer, according to your conjecture,when the thrust force is  measured towards the small end of the truncated cone:

1) Are there any particles being emitted, according to your conjecture, out of the cavity in the axial direction towards the big base ?

[If nothing is being emitted, then there cannot be any propulsion, because it would violate conservation of momentum.  If nothing is being emitted, please skip the next question]

2) what  particle is being emitted, out of the cavity, in the axial direction towards the big base ?

[for example, if you conjecture that photons are being emitted, even if that conjecture would be true, it would be contradicted by what is claimed, because what is claimed is a thrust thousands of times better than the thrust of a perfectly collimated photon rocket]

So, if you are attempting to explain the claimed thrust just based on Maxwell's equations, I still don't understand your conjecture. If I misunderstood something, please correct me.  Thanks.

I have no hypothesis on how we wouuld be pushing out particles from the cavity.  I was only speculating that we have traveling modes and not standing modes in the cavity.  As a result I noticed that the traveling modes almost appear to be pushing on the sides of the can (cavity) as they travel.  I definitly don't think simple light propulsion would produce enough thrust but the modes pushing on the side of the can don't strike me as light propulsion.  Maybe I am wrong about that though.  I will attach an image which may help others see what I am seeing.  I am not saying it is what is going on but just throwing it out there as something interesting that might be going on. 
Title: Re: EM Drive Developments - related to space flight applications - Thread 2
Post by: Rodal on 05/03/2015 05:47 pm
(http://forum.nasaspaceflight.com/index.php?action=dlattach;topic=36313.0;attach=828276;image)
1) In your image, you have a T-Junction from a waveguide into the EM Drive truncated cone.
Although Prof. Yang may used such construction (for at least part of her tests), to my knowledge, NASA Eagleworks does not have any such T-Junction between the NASA truncated cone and a waveguide.

If I am incorrect, I would appreciate being corrected.

If I am correct, I don't understand the rationale that would support travelling waves in a completely enclosed truncated cone, as a travelling wave will not satisfy the boundary conditions necessary to solve Maxwell's equations for the tests performed by NASA Eagleworks.

2) Thermal losses in a vacuum (unless there is outgassing, etc.) cannot support the claimed thrust forces, due to the previously addressed issue (what particles are being emitted, and what is their momentum).
Title: Re: EM Drive Developments - related to space flight applications - Thread 2
Post by: dustinthewind on 05/03/2015 05:54 pm
(http://forum.nasaspaceflight.com/index.php?action=dlattach;topic=36313.0;attach=828276;image)
In your image, you have a T-Junction from a waveguide into the EM Drive truncated cone.
Although Prof. Yang used such construction (for at least part of her tests), to my knowledge, NASA Eagleworks does not have any such T-Junction between the NASA truncated cone and a waveguide.

If I am incorrect, I would appreciate being corrected.

If I am correct, I don't understand the rationale that would support travelling waves in a completely enclosed truncated cone, as a travelling wave will not satisfy the boundary conditions necessary to solve Maxwell's equations for the tests performed by NASA Eagleworks.

Dosn't a standing wave assume 100% power reflection?  Such as a powerline with no one consuming power.  I thought the moving magnetic field modes were a symbol or illustration of power transport to a location (thermal loss).  Maybe I am mistaken? 

I don't think the T-junction is a critical part of the idea and can be eliminated.  As long as there is some form of power input. 
Title: Re: EM Drive Developments - related to space flight applications - Thread 2
Post by: Rodal on 05/03/2015 05:58 pm
(http://forum.nasaspaceflight.com/index.php?action=dlattach;topic=36313.0;attach=828276;image)
In your image, you have a T-Junction from a waveguide into the EM Drive truncated cone.
Although Prof. Yang used such construction (for at least part of her tests), to my knowledge, NASA Eagleworks does not have any such T-Junction between the NASA truncated cone and a waveguide.

If I am incorrect, I would appreciate being corrected.

If I am correct, I don't understand the rationale that would support travelling waves in a completely enclosed truncated cone, as a travelling wave will not satisfy the boundary conditions necessary to solve Maxwell's equations for the tests performed by NASA Eagleworks.

Dosn't a standing wave assume 100% power reflection?  Such as a powerline with no one consuming power.  I thought the moving magnetic field modes were a symbol or illustration of power transport to a location (thermal loss).  Maybe I am mistaken?
The issue of power dissipation due to the skin effect is fully addressed in the COMSOL Fnite Element analyses, which predicted the measured Q's.  The COMSOL Finite Element analysis fully respects conservation of momentum and conservation of energy.

There can only be travelling waves if there is a net momentum flux.  But such momentum is prevented by the previous arguments based on conservation of momentum (unless suitable emission of particles can support the measured thrust).

Thermal losses in a vacuum (unless there is outgassing, etc.) cannot support the claimed thrust forces, due to the previously addressed issue (what particles are being emitted, and what is their momentum).

All the above is true under linear Maxwell's equations and special relativity.  Otherwise one would have to argue for breaking of P T parity, nonlinear anisotropic effects, coupling interaction with outside fields, etc.
Title: Re: EM Drive Developments - related to space flight applications - Thread 2
Post by: ppnl on 05/03/2015 06:12 pm
...
I'm hopeful we can get Mr. Shawyer to participate here IF folks are civil.
I would not bet on that.  The uncivil level of attack displayed by several new posters in this thread during the last few days has been such that I would not be surprised if Star-Drive decides not to post any further Eagleworks test information here.  These uncivil attacks are launched by anonymous posters that hide behind monickers without ever revealing their real names.  I very much doubt that these uncivil posters would dare to express themselves this way, face-to-face in a public presentation, as in an AIAA meeting, for example.  I very much doubt that they would express themselves that way in writing if they would do it under their real name.  It is certainly conduct never seen at professional meetings or in academia.

I must say I'm rather perplexed and puzzled by the sudden hostility that some of the newer participants demonstrate...
There is absolutely nothing wrong with being skeptic but i thought that curiosity was one of the key characteristics of being a scientist/researcher/engineer.
There is no better way to make progress then to have a thesis and antithesis collide in a civil manner.

I really do not understand what can be obtained or achieved by aggressively attacking people or their ideas. If you aim to disprove a theory then all you need to do is mass enough evidence that their theory is flawed.

Personally I find it still too early trying to come up with theories until the effect has been validated or not.
For me the most compelling evidence so far is still the 2007 rotating Demonstrator video by R Shawyer.
All we can do now is wait for Eagleworks to duplicate that test (and improve on some of the possible setup flaws, like hot jet exhaust nullification).

If the test fails, then the credibility of the device will get a serious hit...
If the test succeeds, it will most likely be a turning point in the research (and funding) of the EM drive.

So, instead of shooting lead at each other, why don't we just all relax and be supportive to the Eagleworks team so they can finalize that crucial test by July?

Pro or contra, you'll have your answer by July...

That said, it was to foresee that giving more publicity to the research through the NASA publication article, would attract some of the most aggressive opinionated people inhere.
On the positive side however, it also attracted some very much needed new participants that have clearly high level qualifications... (be them pro or contra, it doesn't matter)

I suppose it is up to the mods to weed out the offensive ones...

As for mr Shawyer, i think it is already obvious that he will not engage into the discussion here, partially because of the engagements he already has with other parties, as he explained in that private conversation, partially because he had his share of abusive language in the past....


....eagerly anticipating the next , high power test from Eagleworks... 8)

The problem is I predict that Eagleworks will succeed and still nobody will be impressed. Remember cold fusion? Excess heat beyond chemistry... replicated... 10x energy input... 100x energy input... the thing produced so much heat that it melted down in the middle of the night.. heat in palladium... nickle... thin films... neutrons... maybe it isn't fusion but something else... zero point energy...

Yet year after year nobody was producing a commercial product, viable theory or convincing demo. For some the only explanation was a conspiracy. Anyone remember the "hot fusion (sorry, need to stop here for a second and just say that I have to use stupid words to get my point across. I know that means I must have a weak argument, but that's why I use bad words)." preventing research into cold fusion in order to protect their programs? I bet Jed Rothwell is still chasing cold fusion. I do know that ICCF-19 was held last month.

Emdrive has all the same properties. An extraordinary claim much more so than cold fusion in fact. A theory that makes no sense. People making up new theories to fit bad experiments. Other people day dreaming about how we can build a real spaceship now. Arguments over how to obtain funding. Free bubble up and rainbow stew.

I'm sorry. I don't mean to be rude, unpleasant or confrontational but sometimes reality is unpleasant. In time most will give up on the EMdrive. Some will stay and probably drift into conspiracy theories. A trickle of new results will continue to excite a younger crowd. As a result the EM drive will never succeed and never ever ever go away.

Talk to me in ten years and see if I'm not right. I hope I'm wrong.

Title: Re: EM Drive Developments - related to space flight applications - Thread 2
Post by: Mulletron on 05/03/2015 06:18 pm

The problem is I predict that Eagleworks will succeed and still nobody will be impressed. Remember cold fusion? Excess heat beyond chemistry... replicated... 10x energy input... 100x energy input... the thing produced so much heat that it melted down in the middle of the night.. heat in palladium... nickle... thin films... neutrons... maybe it isn't fusion but something else... zero point energy...

Yet year after year nobody was producing a commercial product, viable theory or convincing demo. For some the only explanation was a conspiracy. Anyone remember the "hot fusion (sorry, need to stop here for a second and just say that I have to use stupid words to get my point across. I know that means I must have a weak argument, but that's why I use bad words)." preventing research into cold fusion in order to protect their programs? I bet Jed Rothwell is still chasing cold fusion. I do know that ICCF-19 was held last month.

Emdrive has all the same properties. An extraordinary claim much more so than cold fusion in fact. A theory that makes no sense. People making up new theories to fit bad experiments. Other people day dreaming about how we can build a real spaceship now. Arguments over how to obtain funding. Free bubble up and rainbow stew.

I'm sorry. I don't mean to be rude, unpleasant or confrontational but sometimes reality is unpleasant. In time most will give up on the EMdrive. Some will stay and probably drift into conspiracy theories. A trickle of new results will continue to excite a younger crowd. As a result the EM drive will never succeed and never ever ever go away.

Talk to me in ten years and see if I'm not right. I hope I'm wrong.

@ppnl we're here to solve problems, not handwave and accept the world will never find a better way. If you don't have any solutions, you're in the wrong place.
Title: Re: EM Drive Developments - related to space flight applications - Thread 2
Post by: TheTraveller on 05/03/2015 06:27 pm
Here is a thought experiment that goes into a EM Drive physical action that doesn't yet seem to be discussed.

Imagine an EM Drive mounted on horizontally free to spin rotary arm, small and large end plates vertical, at right angles to the rotary arm. No microwave energy in the cavity.

A push by your finger on either the small or large end plates will induce rotation in opposite directions.

Now stop the EM Drive and fill the cavity full of microwave energy.

Note it does not move, no thrust in either direction is generated.

Now push on the small end plate with your finger and note it refuses to move, feels like the rotary arm is fixed in place, unless your applied force exceeds that the EM Drive can generate in Generator mode. Note the load on the microwave Generator drops as does current drawn from the power supply. Measuring the cavity temperature should see it increase as the force from your finger, applied to the small end plate, is converted into increased stored cavity microwave energy, which as it can't flow back to the electrical energy source must eventually turn into heat.

Stop applying force to the small end plate.

Now apply force to the larger end plate. Note the EM Drive quickly moves away from your finger and power supply current increases.

While the last operation may be familiar, the 1st may be a new area for discussion.

All these device characterists are explained by Shawyer.
Title: Re: EM Drive Developments - related to space flight applications - Thread 2
Post by: meberbs on 05/03/2015 06:42 pm
I was surprised to see that the experiment results have not confirmed a null result yet. While I hope they do turn up a useful propulsion system, my money is still on a null result.

While this thread seems to have produced useful discussion, it seems that it is still being cluttered with references to Shawyer's theory. I have a physics background up through intermediate quantum mechanics, with just a touch of particle physics. I would like to answer some of the questions regarding Shawyer's theory so the discussion can move on to theories that are at least plausible such as the White's QV model.

Quote
Shawyer developed a theory and built a device to test it. If his theory is nonsense then the first conclusion we should reach is that his test results are nonsense. Either that or he is the luckiest person in the world. If he cannot see and address the violation of Galilean relativity then I wouldn't trust him to test a light bulb.
This is wrong in so many different ways I don't know where to start...

By "this is wrong" are you referring to the emdrive theory paper by Shawyer? A partial list of things he demonstrates he does not understand in that paper include:

-the principle of relativity (the foundation of special relativity)
-how to apply velocity transforms in special relativity
-how to do a force balance (he ignores the slanted walls)
-the definition of an open vs closed system

I don't see how a person who fails at basic physics in this way could actually design an accurate experiment or correctly calculate the resulting forces.

Note that ppnl's post used the word "nonsense" not "wrong". Plenty of respectable physicists come up with wrong theories. In this case the theory is complete nonsense and demonstrates a lack of understanding*. In this case the experiment results cannot be trusted, hence the other labs attempting to replicate the results.

If anyone needs clarification on what exactly is wrong with Shawyer's paper, let me know, so we can get this out of the way.

*A less charitable assumption would be that this is deliberate, but "Never ascribe to malice that which is adequately explained by incompetence."
Title: Re: EM Drive Developments - related to space flight applications - Thread 2
Post by: Mulletron on 05/03/2015 06:57 pm
I was surprised to see that the experiment results have not confirmed a null result yet. While I hope they do turn up a useful propulsion system, my money is still on a null result.

While this thread seems to have produced useful discussion, it seems that it is still being cluttered with references to Shawyer's theory. I have a physics background up through intermediate quantum mechanics, with just a touch of particle physics. I would like to answer some of the questions regarding Shawyer's theory so the discussion can move on to theories that are at least plausible such as the White's QV model.

Quote
Shawyer developed a theory and built a device to test it. If his theory is nonsense then the first conclusion we should reach is that his test results are nonsense. Either that or he is the luckiest person in the world. If he cannot see and address the violation of Galilean relativity then I wouldn't trust him to test a light bulb.
This is wrong in so many different ways I don't know where to start...

By "this is wrong" are you referring to the emdrive theory paper by Shawyer? A partial list of things he demonstrates he does not understand in that paper include:

-the principle of relativity (the foundation of special relativity)
-how to apply velocity transforms in special relativity
-how to do a force balance (he ignores the slanted walls)
-the definition of an open vs closed system

I don't see how a person who fails at basic physics in this way could actually design an accurate experiment or correctly calculate the resulting forces.

Note that ppnl's post used the word "nonsense" not "wrong". Plenty of respectable physicists come up with wrong theories. In this case the theory is complete nonsense and demonstrates a lack of understanding*. In this case the experiment results cannot be trusted, hence the other labs attempting to replicate the results.

If anyone needs clarification on what exactly is wrong with Shawyer's paper, let me know, so we can get this out of the way.

*A less charitable assumption would be that this is deliberate, but "Never ascribe to malice that which is adequately explained by incompetence."

Thank you for the well put together post. I think you are being a bit harsh with your words toward Shawyer. Citing previous information reported here....luckily I have an excellent memory.

Shawyer is an Aerospace Engineer. Not a physicist. What I believe is that he made the device, and then the theory came later.

The issues with his theory are old news. That is why I just said we should not get hung up on his theory. There is NO accepted theory of EMdrive. Welcome to the group. We need people with your training.
Title: Re: EM Drive Developments - related to space flight applications - Thread 2
Post by: KittyMoo on 05/03/2015 07:01 pm
Quote
By "this is wrong" are you referring to the emdrive theory paper by Shawyer?
No. I am referring to the complete lack of understanding of the scientific principle by ppnl.
Title: Re: EM Drive Developments - related to space flight applications - Thread 2
Post by: frobnicat on 05/03/2015 07:30 pm
Never really thought about this before, but is light in a cavity resonator gravitationally redshifted?

http://en.wikipedia.org/wiki/Gravitational_redshift
Do you mean natural (i.e. earth) or artificial gravity that could be generated by space-time warping or other quantum voodoo?

I mean, if you had an Emdrive sitting the table (large diameter down) here on Earth, and it was energized by feeding RF into a slot/probe located at the large diameter. Would an observer at the small end notice a red shift?
Would seem to me that if the thrust produced is horizontally, gravity can't have effect on it, besides bending it a teeny weeny bit. Time dilation at beginning and endpoint are same (same distance from gravity well), so in effect 0.

Vertically you could measure it, but the redshift from earth's gravity is already extremely difficult to measure, but if you could you could then detract that from the redshift value measured in the cavity.

Quote
so in effect 0.

So nonzero? Say outside the bandwidth of a very narrow bandwidth cavity?

For weak gravitational field the frequency ratio between top and bottom is ft/fb = (1 + Rs/2rt - Rs/2rb) where Rs is Schwarzschild radius, rt and rb distance from centre of body (earth centre). From there (http://fr.wikipedia.org/wiki/D%C3%A9calage_d%27Einstein#Fr.C3.A9quence_propre_et_fr.C3.A9quence_observ.C3.A9e), sorry this is French wikipedia, I don't find a convenient English resource for the same formula.

For Earth Rs is about 9mm (http://en.wikipedia.org/wiki/Schwarzschild_radius#Parameters), so lets say we have 0.3m altitude difference at earth surface (6.371e6m) => ft/fb = 1 - 3.3e-17
This is one part in 3e16 redshift in frequency.

Likewise any Doppler effect affecting the relative wavelengths (momentums) of photons between forward and backward plates of an accelerating frustum would indeed induce a non 0 net force : this force would always be opposite to the acceleration (ie. never a thrust) and in fact could be interpreted as the inertia of the mass equivalent of energy bouncing back and forth in the cavity (whatever its shape). The time constant of a photon in a Q=10000 about 0.3m across frustum would be like 10µs, at 100W pumped into the frustum there is then on the order of 1e-3 J EM energy content at any given time, that is equivalent to 1.1e-20 kg of mass, or an apparent added "force of inertia" of 1.1e-19N for a spacecraft accelerating at 1g, or equivalently an added weight of 1.1e-19N vertically for a resting frustum on earth.

In summary, within classical frameworks, yes there can be non 0 net force of EM radiation in an accelerating cavity, but this will be vanishingly small forces, and always opposite to acceleration (aka "inertia").
Is it correct ?
Title: Re: EM Drive Developments - related to space flight applications - Thread 2
Post by: Andy USA on 05/03/2015 07:30 pm
Ok guys, so a lot of new people into this thread, but this thread is mainly for the development of the EM Drive. We knew this would happen, so we have a new "Entry Level" thread for opening questions and general questions.

I've moved the last few pages of new members asking questions into that thread, so if you posted here and can't see it, don't worry, it's in this thread.

http://forum.nasaspaceflight.com/index.php?topic=37438.0

Posting this notice as some sites are linking to this thread and not the section or the article, so people are thinking this is the only thread on this.

Remember to use the above link and allow this thread to continue with the Eaglework folk and others updating progress.

A reminder, as we still have people jumping in and perhaps missing key parts or interrupting a thread covering developments with questions better placed in the other thread.

I would also remind people to ensure their posts are respectful and useful, not noise or insulting. Insulting posts show a weak position and are subject to removal if they lower the tone of this forum (new people may not be aware how this forum is a civil and friendly place. We do not shy from removing people and posts who think they can jump in feet first and be rude).

Otherwise this is a good thread and all opinions are welcome.
Title: Re: EM Drive Developments - related to space flight applications - Thread 2
Post by: TheTraveller on 05/03/2015 07:31 pm
I was surprised to see that the experiment results have not confirmed a null result yet. While I hope they do turn up a useful propulsion system, my money is still on a null result.

While this thread seems to have produced useful discussion, it seems that it is still being cluttered with references to Shawyer's theory. I have a physics background up through intermediate quantum mechanics, with just a touch of particle physics. I would like to answer some of the questions regarding Shawyer's theory so the discussion can move on to theories that are at least plausible such as the White's QV model.

Quote
Shawyer developed a theory and built a device to test it. If his theory is nonsense then the first conclusion we should reach is that his test results are nonsense. Either that or he is the luckiest person in the world. If he cannot see and address the violation of Galilean relativity then I wouldn't trust him to test a light bulb.
This is wrong in so many different ways I don't know where to start...

By "this is wrong" are you referring to the emdrive theory paper by Shawyer? A partial list of things he demonstrates he does not understand in that paper include:

-the principle of relativity (the foundation of special relativity)
-how to apply velocity transforms in special relativity
-how to do a force balance (he ignores the slanted walls)
-the definition of an open vs closed system

I don't see how a person who fails at basic physics in this way could actually design an accurate experiment or correctly calculate the resulting forces.

Note that ppnl's post used the word "nonsense" not "wrong". Plenty of respectable physicists come up with wrong theories. In this case the theory is complete nonsense and demonstrates a lack of understanding*. In this case the experiment results cannot be trusted, hence the other labs attempting to replicate the results.

If anyone needs clarification on what exactly is wrong with Shawyer's paper, let me know, so we can get this out of the way.

*A less charitable assumption would be that this is deliberate, but "Never ascribe to malice that which is adequately explained by incompetence."
It would seem Yang Juan uses a very similar explanation in her recent EM Drive paper. In case you have not had an opportunity to read it, here is the link: http://www.emdrive.com/yang-juan-paper-2012.pdf
Title: Re: EM Drive Developments - related to space flight applications - Thread 2
Post by: Rodal on 05/03/2015 07:41 pm
Objections and technical criticism of theories and experiments are encouraged but they should be addressed as in technical journals.  Technical theories are best addressed with technical arguments, mathematics and references.

Any name calling just serves to weaken any such argument, if newcomers like to employ abusive language in addition to or rather than technical arguments, mathematics or references, they do not belong in this forum.
Title: Re: EM Drive Developments - related to space flight applications - Thread 2
Post by: SH on 05/03/2015 07:50 pm
I really like the Shawyer Demonstrator device (attached). As an engineer I can appreciate designing, building and testing it plus the hours and money involved. It is a serious and professionally built device. It is not a toy but a real working thruster that can be taken anywhere to have additional tests done.

Maybe if EW asked Shawyer nicely, they could test it? At least then they have an established test data base and working device to work from.

Why doesn't Sawyer just bring it to Glenn Research Center to test there?  They already offered to test the device if it can produce more than 100 micro-newton, and Shawyer's device is purportedly well above that.
Title: Re: EM Drive Developments - related to space flight applications - Thread 2
Post by: SH on 05/03/2015 07:56 pm
There's other perfectly sane ways of interacting with the QV which have been posted literally a thousand times in THIS forum over and over again since October.

I have been reading these threads quite diligently lately but I am not sure what you are referring to.  A link would be appreciated!
Title: Re: EM Drive Developments - related to space flight applications - Thread 2
Post by: Star One on 05/03/2015 08:00 pm

I really like the Shawyer Demonstrator device (attached). As an engineer I can appreciate designing, building and testing it plus the hours and money involved. It is a serious and professionally built device. It is not a toy but a real working thruster that can be taken anywhere to have additional tests done.

Maybe if EW asked Shawyer nicely, they could test it? At least then they have an established test data base and working device to work from.

Why doesn't Sawyer just bring it to Glenn Research Center to test there?  They already offered to test the device if it can produce more than 100 micro-newton, and Shawyer's device is purportedly well above that.

Is that device someone up thread said belongs to Boeing now?
Title: Re: EM Drive Developments - related to space flight applications - Thread 2
Post by: TheTraveller on 05/03/2015 08:03 pm
I really like the Shawyer Demonstrator device (attached). As an engineer I can appreciate designing, building and testing it plus the hours and money involved. It is a serious and professionally built device. It is not a toy but a real working thruster that can be taken anywhere to have additional tests done.

Maybe if EW asked Shawyer nicely, they could test it? At least then they have an established test data base and working device to work from.

Why doesn't Sawyer just bring it to Glenn Research Center to test there?  They already offered to test the device if it can produce more than 100 micro-newton, and Shawyer's device is purportedly well above that.
I also proposed EW to test either the SPR Demonstrator device or the SPR Flight Thruster (which Boeing should have sitting on a shelf) which it seems is considered a "High Fidelity Test Article"
Title: Re: EM Drive Developments - related to space flight applications - Thread 2
Post by: Star One on 05/03/2015 08:05 pm

I really like the Shawyer Demonstrator device (attached). As an engineer I can appreciate designing, building and testing it plus the hours and money involved. It is a serious and professionally built device. It is not a toy but a real working thruster that can be taken anywhere to have additional tests done.

Maybe if EW asked Shawyer nicely, they could test it? At least then they have an established test data base and working device to work from.

Why doesn't Sawyer just bring it to Glenn Research Center to test there?  They already offered to test the device if it can produce more than 100 micro-newton, and Shawyer's device is purportedly well above that.
I also proposed EW to test either the SPF Demonstrator device or the SPF Flight Thruster (which Boeing should have sitting on a shelf) which it seems is considered a "High Fidelity Test Article"

Good luck with that being as from what was posted up thread it has been implied that's gone dark as they say.
Title: Re: EM Drive Developments - related to space flight applications - Thread 2
Post by: TheTraveller on 05/03/2015 08:06 pm

I really like the Shawyer Demonstrator device (attached). As an engineer I can appreciate designing, building and testing it plus the hours and money involved. It is a serious and professionally built device. It is not a toy but a real working thruster that can be taken anywhere to have additional tests done.

Maybe if EW asked Shawyer nicely, they could test it? At least then they have an established test data base and working device to work from.

Why doesn't Sawyer just bring it to Glenn Research Center to test there?  They already offered to test the device if it can produce more than 100 micro-newton, and Shawyer's device is purportedly well above that.

Is that device someone up thread said belongs to Boeing now?
This belongs to Boeing: http://www.emdrive.com/flightprogramme.html

And maybe this as Boeign bought all the SPR EM Drive IP in 2010: http://emdrive.com/demonstratorengine.html
Title: Re: EM Drive Developments - related to space flight applications - Thread 2
Post by: KittyMoo on 05/03/2015 08:08 pm
We will have to wait until a B3 floats out of the hanger then? ;)
Title: Re: EM Drive Developments - related to space flight applications - Thread 2
Post by: Notsosureofit on 05/03/2015 08:12 pm
Never really thought about this before, but is light in a cavity resonator gravitationally redshifted?

http://en.wikipedia.org/wiki/Gravitational_redshift
Do you mean natural (i.e. earth) or artificial gravity that could be generated by space-time warping or other quantum voodoo?

I mean, if you had an Emdrive sitting the table (large diameter down) here on Earth, and it was energized by feeding RF into a slot/probe located at the large diameter. Would an observer at the small end notice a red shift?
Would seem to me that if the thrust produced is horizontally, gravity can't have effect on it, besides bending it a teeny weeny bit. Time dilation at beginning and endpoint are same (same distance from gravity well), so in effect 0.

Vertically you could measure it, but the redshift from earth's gravity is already extremely difficult to measure, but if you could you could then detract that from the redshift value measured in the cavity.

Quote
so in effect 0.

So nonzero? Say outside the bandwidth of a very narrow bandwidth cavity?

For weak gravitational field the frequency ratio between top and bottom is ft/fb = (1 + Rs/2rt - Rs/2rb) where Rs is Schwarzschild radius, rt and rb distance from centre of body (earth centre). From there (http://fr.wikipedia.org/wiki/D%C3%A9calage_d%27Einstein#Fr.C3.A9quence_propre_et_fr.C3.A9quence_observ.C3.A9e), sorry this is French wikipedia, I don't find a convenient English resource for the same formula.

For Earth Rs is about 9mm (http://en.wikipedia.org/wiki/Schwarzschild_radius#Parameters), so lets say we have 0.3m altitude difference at earth surface (6.371e6m) => ft/fb = 1 - 3.3e-17
This is one part in 3e16 redshift in frequency.

Likewise any Doppler effect affecting the relative wavelengths (momentums) of photons between forward and backward plates of an accelerating frustum would indeed induce a non 0 net force : this force would always be opposite to the acceleration (ie. never a thrust) and in fact could be interpreted as the inertia of the mass equivalent of energy bouncing back and forth in the cavity (whatever its shape). The time constant of a photon in a Q=10000 about 0.3m across frustum would be like 10µs, at 100W pumped into the frustum there is then on the order of 1e-3 J EM energy content at any given time, that is equivalent to 1.1e-20 kg of mass, or an apparent added "force of inertia" of 1.1e-19N for a spacecraft accelerating at 1g, or equivalently an added weight of 1.1e-19N vertically for a resting frustum on earth.

In summary, within classical frameworks, yes there can be non 0 net force of EM radiation in an accelerating cavity, but this will be vanishingly small forces, and always opposite to acceleration (aka "inertia").
Is it correct ?

Close, those forces are similar to the observed forces.  Perhaps @Rodal has the page reference back to my calculation which is what I beleive you are stating.

Edit:  I think this was it (have to check)

"
The proposition that dispersion caused by an accelerating frame of reference implied an accelerating frame of reference caused by a dispersive cavity resonator. (to 1st order using massless, perfectly conducting cavity)


Starting with the expressions for the frequency of an RF cavity:

f = (c/(2*Pi))*((X/R)^2+((p*Pi)/L)^2)^.5

For TM modes, X = X[sub m,n] = the n-th zero of the m-th Bessel function.
[1,1]=3.83, [0,1]=2.40, [0,2]=5.52 [1,2]=7.02, [2,1]=5.14, [2,2]=8.42, [1,3]=10.17, etc.

and for TE modes, X = X'[subm,n] = the n-th zero of the derivative of the m-th Bessel function.
[0,1]=3.83, [1,1]=1.84, [2,1]=3.05, [0,2]=7.02, [1,2]=5.33, [1,3]=8.54, [0,3]=10.17, [2,2]=6.71, etc.

Rotate the dispersion relation of the cavity into doppler frame to get the Doppler shifts, that is to say, look at the dispersion curve intersections of constant wave number instead of constant frequency.

df = (1/(2*f))*(c/(2*Pi))^2*X^2*((1/Rs^2)-(1/Rb^2))

and from there the expression for the acceleration g from:

g = (c^2/L)*(df/f) such that:

g = (c^2/(2*L*f^2))*(c/(2*Pi))^2*X^2*((1/Rs^2)-(1/Rb^2))

Using the "weight" of the photon in the accelerated frame from:

"W" = (h*f/c^2)*g =>  "W" = T = (h/L)*df

gives thrust per photon:

T = (h/(2*L*f))*(c/(2*pi))^2*X^2*((1/Rs^2)-(1/Rb^2))

If the number of photons is (P/hf)*(Q/2*pi) then:

NT = P*Q*(1/(4*pi*L*f^3))*(c/(2*pi))^2*X^2*((1/Rs^2)-(1/Rb^2))"
Title: Re: EM Drive Developments - related to space flight applications - Thread 2
Post by: TheTraveller on 05/03/2015 08:14 pm

I really like the Shawyer Demonstrator device (attached). As an engineer I can appreciate designing, building and testing it plus the hours and money involved. It is a serious and professionally built device. It is not a toy but a real working thruster that can be taken anywhere to have additional tests done.

Maybe if EW asked Shawyer nicely, they could test it? At least then they have an established test data base and working device to work from.

Why doesn't Sawyer just bring it to Glenn Research Center to test there?  They already offered to test the device if it can produce more than 100 micro-newton, and Shawyer's device is purportedly well above that.
I also proposed EW to test either the SPF Demonstrator device or the SPF Flight Thruster (which Boeing should have sitting on a shelf) which it seems is considered a "High Fidelity Test Article"

Good luck with that being as from what was posted up thread it has been implied that's gone dark as they say.
Well was not that dark as EW displayed the attached.
Lower right is the Flight Thruster Boeing acquired from SPF (Shawyers company). Note it is rated as a "High Fidelity Test Article". Guess that means it works well and is highly reliable in the test results generated.
Title: Re: EM Drive Developments - related to space flight applications - Thread 2
Post by: Flyby on 05/03/2015 08:21 pm
I really like the Shawyer Demonstrator device (attached). As an engineer I can appreciate designing, building and testing it plus the hours and money involved. It is a serious and professionally built device. It is not a toy but a real working thruster that can be taken anywhere to have additional tests done.

Maybe if EW asked Shawyer nicely, they could test it? At least then they have an established test data base and working device to work from.

Why doesn't Sawyer just bring it to Glenn Research Center to test there?  They already offered to test the device if it can produce more than 100 micro-newton, and Shawyer's device is purportedly well above that.
I also proposed EW to test either the SPF Demonstrator device or the SPF Flight Thruster (which Boeing should have sitting on a shelf) which it seems is considered a "High Fidelity Test Article"

There are merits to rebuilding a device instead of simply retesting the same device.

By building a new device, according similar specs and testing it, you can actually validate the principle behind the 2 devices (Shawyer's and EW's) if they produce similar results.
If they contradict each other, you'll need additional testing, of course..

By simply retesting the Shawyer's device you could potentially duplicate the same flaw.  Just the measurement setup would be different....
Title: Re: EM Drive Developments - related to space flight applications - Thread 2
Post by: SH on 05/03/2015 08:26 pm
This belongs to Boeing: http://www.emdrive.com/flightprogramme.html

And maybe this as Boeign bought all the SPR EM Drive IP in 2010: http://emdrive.com/demonstratorengine.html

Boeing's Phantom Works, which has previously explored exotic forms of space propulsion, was said to be looking into it some years ago. Such work has evidently ceased. “Phantom Works is not working with Mr. Shawyer,” a Boeing representative says, adding that the company is no longer pursuing this avenue [as of Nov 5, 2012].

http://aviationweek.com/awin/propellentless-space-propulsion-research-continues
Title: Re: EM Drive Developments - related to space flight applications - Thread 2
Post by: TheTraveller on 05/03/2015 08:27 pm
I really like the Shawyer Demonstrator device (attached). As an engineer I can appreciate designing, building and testing it plus the hours and money involved. It is a serious and professionally built device. It is not a toy but a real working thruster that can be taken anywhere to have additional tests done.

Maybe if EW asked Shawyer nicely, they could test it? At least then they have an established test data base and working device to work from.

Why doesn't Sawyer just bring it to Glenn Research Center to test there?  They already offered to test the device if it can produce more than 100 micro-newton, and Shawyer's device is purportedly well above that.
I also proposed EW to test either the SPF Demonstrator device or the SPF Flight Thruster (which Boeing should have sitting on a shelf) which it seems is considered a "High Fidelity Test Article"

There are merits to rebuilding a device instead of simply retesting the same device.

By building a new device, according similar specs and testing it, you can actually validate the principle behind the 2 devices (Shawyer's and EW's) if they produce similar results.
If they contradict each other, you'll need additional testing, of course..

By simply retesting the Shawyer's device you could potentially duplicate the same flaw.  Just the measurement setup would be different....
Agree building your own device is good. So is validation of Shawyer EM Drive thrust. Established a base line. Establishes once and forever that it works as claimed. Then start to work out why.
Title: Re: EM Drive Developments - related to space flight applications - Thread 2
Post by: TheTraveller on 05/03/2015 08:32 pm
This belongs to Boeing: http://www.emdrive.com/flightprogramme.html

And maybe this as Boeign bought all the SPR EM Drive IP in 2010: http://emdrive.com/demonstratorengine.html

Boeing's Phantom Works, which has previously explored exotic forms of space propulsion, was said to be looking into it some years ago. Such work has evidently ceased. “Phantom Works is not working with Mr. Shawyer,” a Boeing representative says, adding that the company is no longer pursuing this avenue [as of Nov 5, 2012].

http://aviationweek.com/awin/propellentless-space-propulsion-research-continues
Great. So Boeing should have no issues sending the Demonstrator and Flight Thruster EM Drives over to EW to test?

Boeing has already sent over at least one other propellantless thruster to EW for them to test.

Interesting article which fails to mention, in 2010, Boeing acquired all the SPR IP through a licensing deal setup by the British government. Listen to Shawyer talk about the deal from 3:30 in the 1st interview video: http://www.emdrive.com/interview.html
He claims it went dark.
Title: Re: EM Drive Developments - related to space flight applications - Thread 2
Post by: Flyby on 05/03/2015 08:32 pm
Well was not that dark as EW displayed the attached.
Lower right is the Flight Thruster Boeing acquired from SPF (Shawyers company). Note it is rated as a "High Fidelity Test Article". Guess that means it works well and is highly reliable in the test results generated.

Not sure where I got this from, (been reading it somewhere...) but i thought the Flight Thruster was made out of ALU to have a better heat dissipation capacity (thicker walls also), so it could be tested for a longer period without fast Q degradation. The copper frustum devices are tested in short burst due to heat degrading performance  problems.
This ALU one could probably "work" for an extended period.. hence the naming of "High fidelity" ? In all honesty, I'm guessing here, but it does make sense, no?
Title: Re: EM Drive Developments - related to space flight applications - Thread 2
Post by: SH on 05/03/2015 08:35 pm
By simply retesting the Shawyer's device you could potentially duplicate the same flaw.  Just the measurement setup would be different....

It's easier to reverse engineer a working alien technology if you have an example that you can perform experiments on, than it is to re-engineer that technology when it violates what you think you know about physics!

The eagleworks drive is orders of magnitude lower thrust and cannot even be independently validated due to the low thrust levels.  We don't even know if eagleworks will be capable of producing a higher thrust version, because their plans are based on a hypothetical understanding of how it works which has been highly criticized.

We don't know if Shawyer's device actually produces the thrust levels that he claims.  The fact that Boeing purchased the technology, then decided not to pursue it, is evidence that after their independent testing of his device, they must have concluded it to be useless (although I suppose it is possible that they are lying, and that it was a strategic statement designed to make everyone think it was pseudo-science so that they could have more time to develop it in secret).
Title: Re: EM Drive Developments - related to space flight applications - Thread 2
Post by: SH on 05/03/2015 08:49 pm
The fact that Boeing purchased the technology, then decided not to pursue it, is evidence that after their independent testing of his device, they must have concluded it to be useless (although I suppose it is possible that they are lying, and that it was a strategic statement designed to make everyone think it was pseudo-science so that they could have more time to develop it in secret).

Phantom Works often works on classified projects so that last conspiracy theory might not actually be too far fetched, especially considering how little they had to say about why they are no longer working with Shawyer.  If anyone has contact with Shawyer, it might be interesting "experiment" to ask Shawyer how things are going with Phantom Works, to see if his response sounds suspiciously like someone who's been silenced under contract to not talk about it.
Title: Re: EM Drive Developments - related to space flight applications - Thread 2
Post by: dustinthewind on 05/03/2015 08:51 pm
(http://forum.nasaspaceflight.com/index.php?action=dlattach;topic=36313.0;attach=828276;image)
In your image, you have a T-Junction from a waveguide into the EM Drive truncated cone.
Although Prof. Yang used such construction (for at least part of her tests), to my knowledge, NASA Eagleworks does not have any such T-Junction between the NASA truncated cone and a waveguide.

If I am incorrect, I would appreciate being corrected.

If I am correct, I don't understand the rationale that would support travelling waves in a completely enclosed truncated cone, as a travelling wave will not satisfy the boundary conditions necessary to solve Maxwell's equations for the tests performed by NASA Eagleworks.

Dosn't a standing wave assume 100% power reflection?  Such as a powerline with no one consuming power.  I thought the moving magnetic field modes were a symbol or illustration of power transport to a location (thermal loss).  Maybe I am mistaken?
The issue of power dissipation due to the skin effect is fully addressed in the COMSOL Fnite Element analyses, which predicted the measured Q's.  The COMSOL Finite Element analysis fully respects conservation of momentum and conservation of energy.

There can only be travelling waves if there is a net momentum flux.  But such momentum is prevented by the previous arguments based on conservation of momentum (unless suitable emission of particles can support the measured thrust).

Thermal losses in a vacuum (unless there is outgassing, etc.) cannot support the claimed thrust forces, due to the previously addressed issue (what particles are being emitted, and what is their momentum).

All the above is true under linear Maxwell's equations and special relativity.  Otherwise one would have to argue for breaking of P T parity, nonlinear anisotropic effects, coupling interaction with outside fields, etc.

I am just addressing that there should be standing waves that appear to move.  It comes from when you have an initial wave from the power source.  There is also a reflected wave that leaves with a smaller amplitude.  I did the plots in wxmaxima. 
c:1;
w:1;
k:1;
A:1;
B:.5;
f(x,t):=A*sin(k*x-w*t);
g(x,t):=B*sin(k*x+w*t);
for a:0 thru %pi step %pi/10 do wxplot2d(f(x,a)+g(x,a),[x,0,2*%pi],[y,-1.5,1.5]);

So then the traveling modes appear to carry power to the area of power loss.  I am not saying it explains the thrust but maybe it is important to keep in mind.
Title: Re: EM Drive Developments - related to space flight applications - Thread 2
Post by: TheTraveller on 05/03/2015 08:58 pm
The fact that Boeing purchased the technology, then decided not to pursue it, is evidence that after their independent testing of his device, they must have concluded it to be useless (although I suppose it is possible that they are lying, and that it was a strategic statement designed to make everyone think it was pseudo-science so that they could have more time to develop it in secret).

Phantom Works often works on classified projects so that last conspiracy theory might not actually be too far fetched, especially considering how little they had to say about why they are no longer working with Shawyer.  If anyone has contact with Shawyer, it might be interesting "experiment" to ask Shawyer how things are going with Phantom Works, to see if his response sounds suspiciously like someone who's been silenced under contract to not talk about it.
In his recent interview video Shawyer discusses the Boeing deal at 3:30, 1st video. http://www.emdrive.com/interview.html
Says it went dark / out of the public domain.
Title: Re: EM Drive Developments - related to space flight applications - Thread 2
Post by: KittyMoo on 05/03/2015 09:12 pm
Look.
It seems that the Sawyer work has gone 'dark' in one way or another.
Lets leave that where it ended. (Seemed to end?)
EW is openly carrying on experiments for us all to see. AND telling us what they plan in the very near future.
I suggest we analyse and discuss EW experiments and not Sawyers for obvious reasons.
If Mr. Sawyer would talk to us then maybe all would change. Who knows?
Title: Re: EM Drive Developments - related to space flight applications - Thread 2
Post by: Rodal on 05/03/2015 09:17 pm
He may have just meant that Boeing is not communicating with him as to the reasons why the project was terminated.

If a project really goes dark (with the military meaning of that term) and one has or is involved in that project, one doesn't answer that "the project went dark", one instead either doesn't answer or just says that one doesn't know, or more often, one cleverly deflects the question.  The last thing one wants to do is to attract attention to a dark military project.

The first rule of Fight Club is you don't talk about the Fight Club.   ;)
The second rule is...
Title: Re: EM Drive Developments - related to space flight applications - Thread 2
Post by: ThinkerX on 05/03/2015 09:18 pm
Ok...watched Shawyer's video.

Clears up something I'd been wondering about and posted about here a time or two before:

1) his original concept came about 40 years ago as part of a cold war era missile development program.  He and his team were encouraged to consider radical ideas, an apparently (his version) of the core concept stems from this period.

2) about a decade later, while designing military satellites, he began looking into this again, and started contemplating commercial possibilities.  His superiors were less than impressed, so Shawyer parted ways and founded his own company. 

I like the 'turntable test.'  Seems like something the Eagleworks team should shoot for.  But...does this system of measuring thrust have any flaws that might skew the results?
Title: Re: EM Drive Developments - related to space flight applications - Thread 2
Post by: zen-in on 05/03/2015 09:20 pm
By simply retesting the Shawyer's device you could potentially duplicate the same flaw.  Just the measurement setup would be different....

It's easier to reverse engineer a working alien technology if you have an example that you can perform experiments on, than it is to re-engineer that technology when it violates what you think you know about physics!

The eagleworks drive is orders of magnitude lower thrust and cannot even be independently validated due to the low thrust levels.  We don't even know if eagleworks will be capable of producing a higher thrust version, because their plans are based on a hypothetical understanding of how it works which has been highly criticized.

We don't know if Shawyer's device actually produces the thrust levels that he claims.  The fact that Boeing purchased the technology, then decided not to pursue it, is evidence that after their independent testing of his device, they must have concluded it to be useless (although I suppose it is possible that they are lying, and that it was a strategic statement designed to make everyone think it was pseudo-science so that they could have more time to develop it in secret).

The moderators may have to start up a new thread devoted to em-drive conspiracy theories; or better yet just delete all such posts. :)
Title: Re: EM Drive Developments - related to space flight applications - Thread 2
Post by: A_M_Swallow on 05/03/2015 09:23 pm
{snip}
We don't know if Shawyer's device actually produces the thrust levels that he claims.  The fact that Boeing purchased the technology, then decided not to pursue it, is evidence that after their independent testing of his device, they must have concluded it to be useless (although I suppose it is possible that they are lying, and that it was a strategic statement designed to make everyone think it was pseudo-science so that they could have more time to develop it in secret).

Or Boeing did not get a government grant to develop the thruster so they cancelled the project.
Title: Re: EM Drive Developments - related to space flight applications - Thread 2
Post by: PaulF on 05/03/2015 09:33 pm
By simply retesting the Shawyer's device you could potentially duplicate the same flaw.  Just the measurement setup would be different....

It's easier to reverse engineer a working alien technology if you have an example that you can perform experiments on, than it is to re-engineer that technology when it violates what you think you know about physics!

The eagleworks drive is orders of magnitude lower thrust and cannot even be independently validated due to the low thrust levels.  We don't even know if eagleworks will be capable of producing a higher thrust version, because their plans are based on a hypothetical understanding of how it works which has been highly criticized.

We don't know if Shawyer's device actually produces the thrust levels that he claims.  The fact that Boeing purchased the technology, then decided not to pursue it, is evidence that after their independent testing of his device, they must have concluded it to be useless (although I suppose it is possible that they are lying, and that it was a strategic statement designed to make everyone think it was pseudo-science so that they could have more time to develop it in secret).
Even without conspiracy theories, that last line sounds very plausible to me. We all know Boeing is in bed with the U.S. government (OK one then :P)
Title: Re: EM Drive Developments - related to space flight applications - Thread 2
Post by: ThinkerX on 05/03/2015 09:33 pm
Ok, I'm trying to keep this straight, because I don't think we've seen the complete list posted here recently

1) Shawyer - how many different EM drives did he build?  Three?  Seven?

    And because he talks about it in his videos, how many 1st generation, and how many second generation?  He says the 'first generation' devices went to Boeing, but how many?

2) Chinese - how many different EM Drive devices?

3) Eagleworks team - just one device, or more than that?

then, so far as I am aware, a number of people in this thread are building EM drives of their own:

1) Mulletron - still under construction

2) Notsosureofit - started?

3) DIY Fan - no word in a while

Seem to remember at least one other.
Title: Re: EM Drive Developments - related to space flight applications - Thread 2
Post by: PaulF on 05/03/2015 09:35 pm
{snip}
We don't know if Shawyer's device actually produces the thrust levels that he claims.  The fact that Boeing purchased the technology, then decided not to pursue it, is evidence that after their independent testing of his device, they must have concluded it to be useless (although I suppose it is possible that they are lying, and that it was a strategic statement designed to make everyone think it was pseudo-science so that they could have more time to develop it in secret).

Or Boeing did not get a government grant to develop the thruster so they cancelled the project.
If Boeing found the tech to be viable I don't think the rejection of federal budget would stop them pumping their own money into it. This may be tracable through bookkeeping records that are made public.
Title: Re: EM Drive Developments - related to space flight applications - Thread 2
Post by: TheTraveller on 05/03/2015 09:35 pm
By simply retesting the Shawyer's device you could potentially duplicate the same flaw.  Just the measurement setup would be different....

It's easier to reverse engineer a working alien technology if you have an example that you can perform experiments on, than it is to re-engineer that technology when it violates what you think you know about physics!

The eagleworks drive is orders of magnitude lower thrust and cannot even be independently validated due to the low thrust levels.  We don't even know if eagleworks will be capable of producing a higher thrust version, because their plans are based on a hypothetical understanding of how it works which has been highly criticized.

We don't know if Shawyer's device actually produces the thrust levels that he claims.  The fact that Boeing purchased the technology, then decided not to pursue it, is evidence that after their independent testing of his device, they must have concluded it to be useless (although I suppose it is possible that they are lying, and that it was a strategic statement designed to make everyone think it was pseudo-science so that they could have more time to develop it in secret).

The moderators may have to start up a new thread devoted to em-drive conspiracy theories; or better yet just delete all such posts. :)
Shawyer made a public statement. Specifically about the EM Drive tech Boeing licensed. There is no speculation about what he said.
Title: Re: EM Drive Developments - related to space flight applications - Thread 2
Post by: Flyby on 05/03/2015 10:01 pm
transcript from the video (timeframe 4:10):

"...in fact... we actually transfered all our design and test data to Boeing.
It is noticable that any subsequent programs have not been acknowledged in the public domain..."

It doesn't really say it has been terminated by Boeing, just that no info is being released from their side.
It could either mean it has gone "dark" (militarized), or that it is shelved (cancelled) or put on ice (on hold)... yours to pick..
Title: Re: EM Drive Developments - related to space flight applications - Thread 2
Post by: Stormbringer on 05/03/2015 10:18 pm
So if they didn't shelve it this may be the intermittent windshield wiper of exotic propulsion. :)
Title: Re: EM Drive Developments - related to space flight applications - Thread 2
Post by: TheTraveller on 05/03/2015 10:22 pm
transcript from the video (timeframe 4:10):

"...in fact... we actually transfered all our design and test data to Boeing.
It is noticable that any subsequent programs have not been acknowledged in the public domain..."

It doesn't really say it has been terminated by Boeing, just that no info is being released from their side.
It could either mean it has gone "dark" (militarized), or that it is shelved (cancelled) or put on ice (on hold)... yours to pick..
And yet EW or some other body labels the SPR Flight Thruster that Boeing licensed (lower right in attachment) a "High Fidelity Test Article". Don't think that label, on a NASA publication, came from SPR or Shawyer or would be put on a non functional device that did not produce reliable thrust.
Title: Re: EM Drive Developments - related to space flight applications - Thread 2
Post by: KittyMoo on 05/03/2015 10:28 pm
"High Fidelity Test Article" is just a name/label... Copied from Shawyer's literature.
It could have been named "1000N/kW Test Article"
Would that have made any difference without independent testing?
Title: Re: EM Drive Developments - related to space flight applications - Thread 2
Post by: ppnl on 05/03/2015 10:41 pm

The problem is I predict that Eagleworks will succeed and still nobody will be impressed. Remember cold fusion? Excess heat beyond chemistry... replicated... 10x energy input... 100x energy input... the thing produced so much heat that it melted down in the middle of the night.. heat in palladium... nickle... thin films... neutrons... maybe it isn't fusion but something else... zero point energy...

Yet year after year nobody was producing a commercial product, viable theory or convincing demo. For some the only explanation was a conspiracy. Anyone remember the "hot fusion (sorry, need to stop here for a second and just say that I have to use stupid words to get my point across. I know that means I must have a weak argument, but that's why I use bad words)." preventing research into cold fusion in order to protect their programs? I bet Jed Rothwell is still chasing cold fusion. I do know that ICCF-19 was held last month.

Emdrive has all the same properties. An extraordinary claim much more so than cold fusion in fact. A theory that makes no sense. People making up new theories to fit bad experiments. Other people day dreaming about how we can build a real spaceship now. Arguments over how to obtain funding. Free bubble up and rainbow stew.

I'm sorry. I don't mean to be rude, unpleasant or confrontational but sometimes reality is unpleasant. In time most will give up on the EMdrive. Some will stay and probably drift into conspiracy theories. A trickle of new results will continue to excite a younger crowd. As a result the EM drive will never succeed and never ever ever go away.

Talk to me in ten years and see if I'm not right. I hope I'm wrong.

@ppnl we're here to solve problems, not handwave and accept the world will never find a better way. If you don't have any solutions, you're in the wrong place.

I'm not hand-waving. I'm pointing out that that Shawyer is.

He claims that it obeys COM and maybe it does. Maybe it reacts against the quantum vacuum, the dark matter field or matter in a parallel universe. But Shawyer never makes any such claim. In the end we have a device accelerating in one direction and nothing appears to go in the other direction. Until Shawyer can say what is being pushed in the other direction he cannot know that it obeys COM.

Shawyer claims that it obeys COE. He does this by clumsily equating energy transferred to the device to a calculation of the change in kinetic energy. But since kinetic energy is frame dependent he creates a preferred frame in violation of relativity. Now if he had done this on purpose it would be one thing. But he never mentions the frame roblem. It is just a side effect of his hand-waving. Maybe the thing does obey COE but until Sawyer shows where the preferred frame he cannot know this.

It is an extraordinary level of hand-waving.

And I am not saying things will not get better. My first computer operated at 4.77 mhz, had 32 kilobytes of memory and cost 3000 dollars. I now have alienware with 16 gigabytes of memory and a terabyte hard-drive for less than 2000. Don't talk to me about how things have changed. 
Title: Re: EM Drive Developments - related to space flight applications - Thread 2
Post by: SH on 05/03/2015 10:43 pm
I like the 'turntable test.'  Seems like something the Eagleworks team should shoot for.  But...does this system of measuring thrust have any flaws that might skew the results?

Correct me if I'm wrong, but It appears to me that this apparatus is merely a measure of total displacement, not force, and it is only your assumption that the displacement is due to a consistent net force.  Any unconstrained vibrating object restricted to planar motion is likely to form a random walk across the plane, but when you restrict the motion to 1 dimension (clockwise or counterclockwise), as was done in this turntable, then the displacement will monotonically go either clockwise or counterclockwise, because a complete reversal of direction would require overcoming the existing momentum.  In other words, I think the motion in this video could be explained by mere vibrations that have no net force, with a direction of motion that is restricted by the conditions at initialization.
Title: Re: EM Drive Developments - related to space flight applications - Thread 2
Post by: Star One on 05/03/2015 10:51 pm
This belongs to Boeing: http://www.emdrive.com/flightprogramme.html

And maybe this as Boeign bought all the SPR EM Drive IP in 2010: http://emdrive.com/demonstratorengine.html

Boeing's Phantom Works, which has previously explored exotic forms of space propulsion, was said to be looking into it some years ago. Such work has evidently ceased. “Phantom Works is not working with Mr. Shawyer,” a Boeing representative says, adding that the company is no longer pursuing this avenue [as of Nov 5, 2012].

http://aviationweek.com/awin/propellentless-space-propulsion-research-continues

In this case that either means precisely what it says or it means the complete opposite which version you believe your mileage may vary with and not something I'll speculate on.
Title: Re: EM Drive Developments - related to space flight applications - Thread 2
Post by: KittyMoo on 05/03/2015 10:53 pm
Measurement of force from the various EM devices seem to go to great lengths to eliminate gravity as a force (excepting maybe some of the Cannae experiments.)
Is it plausible that the local gravity vector is involved in the anomalous thrusts?
Eg the difference between the experiments is how 'level' they are wrt the local gravity vector.
Would an 'up-down' weight measuring setup be a good idea as a compliment to existing measurements and eliminate some possible experimental errors?
Title: Re: EM Drive Developments - related to space flight applications - Thread 2
Post by: Rodal on 05/03/2015 11:05 pm
I like the 'turntable test.'  Seems like something the Eagleworks team should shoot for.  But...does this system of measuring thrust have any flaws that might skew the results?

Correct me if I'm wrong, but It appears to me that this apparatus is merely a measure of total displacement, not force, and it is only your assumption that the displacement is due to a consistent net force.  Any unconstrained vibrating object restricted to planar motion is likely to form a random walk across the plane, but when you restrict the motion to 1 dimension (clockwise or counterclockwise), as was done in this turntable, then the displacement will monotonically go either clockwise or counterclockwise, because a complete reversal of direction would require overcoming the existing momentum.  In other words, I think the motion in this video could be explained by mere vibrations that have no net force, with a direction of motion that is restricted by the conditions at initialization.

Very interesting suggestion.  But I would have expected random vibrations to produce random walk motion in one direction with a ratchet form of stick-slip friction or a bearing acting with a ratchet-like action, as found in molecular motors


F. Ju¨licher, A. Ajdari, J. Prost, Rev. Mod. Phys. 69 (1997) 1269.

R. Lipowsky, in: J. Freund, T. Po¨ schel (Eds.), Stochastic Processes in Physics, Chemistry and Biology, Springer, Berlin, 2000, pp. 21–31.

R.D. Astumian, P. Ha¨ nggi, Physics Today 55 (11) (2002) 33.

The ratchet-like action would explain why it wants to move in only one direction (a factor TheTraveller has pointed out). (Without the ratchet-like action, just with stick slip friction it would initially move in either direction, depending on initial conditions)

Also, a displacement vs. time plot should be able to reveal the random vibration (performing Autocorrelation and Power Spectral Density via FFT). 

I performed Autocorrelation and Power Spectral Density analysis of the NASA Eagleworks data (of course only to the extent  allowed by the Nyquist frequency of the digitization from a picture, so not able to explore high frequencies).  The NASA Eagleworks data was fully explained by the natural frequency of the torsional (horizontal) pendulum and did not reveal random vibration (to the restricted extent previously explained).

There is not enough data available from Shawyer or Yang to perform a similar analysis, unfortunately.
Title: Re: EM Drive Developments - related to space flight applications - Thread 2
Post by: lasoi on 05/03/2015 11:13 pm
Assuming the cavity isn't somehow spewing particles, how else could completely enclosing the test setup nullify the measured thrust?  Admittedly I know very little about these things and prefer to leave things to the experts (occasionally I can't resist the temptation to interject, mindful of my ignorance).  But assuming the null test is valid, any theory to account for the thrust will have to account for this test, it would seem.  Just seems like the null test should be getting more consideration.  Pardon my ignorance if this is a stupid comment.

http://enu.kz/repository/2009/AIAA-2009-5070.pdf

Edit: if anything you'd think the null test would nullify those theories suggesting that the anomalous thrust is due to some vibrational or heat transfer to the support device for the frustum.  I'm assuming that the support device was enclosed too.  Please correct me if I'm wrong about that.
Title: Re: EM Drive Developments - related to space flight applications - Thread 2
Post by: Eye_one on 05/03/2015 11:15 pm
I posted yesterday in this thread http://forum.nasaspaceflight.com/index.php?topic=37438.300 (http://forum.nasaspaceflight.com/index.php?topic=37438.300) hoping to get a reply to a question on if this device producing gravity I realize now the question I posted was vague and the way I comprised the post might look childish so I'll try to expand on it.

The original post was as follows.
Quote
I apologize in advance my understanding is likely no where near where it should be but, there are no stupid questions only stupid people so prove me stupid.

Is it possible this device is condensing spacetime at one side and expanding it at the other creating a gravitational flow to one side?

This could explain some things like why when more power is put in the force becomes more directional or why the force changes depending on its orientation to the Earth's gravitational field.

Maybe somebody should place an atomic clock in the force it is producing.

I would like to correct a mistake in my original question before I start. When I said expanding spacetime at one side that is wrong it would simply be less compressed than the other side.

As I said in my original post my understanding of physics is not where it should be so it should be easy to prove this wrong for most of you and if you take 5 minutes to do so I will be extremely grateful.

I'll explain how I think this might be happening. If there is a denser concentration of microwaves in one side of the chamber compared to the other and these groups of microwaves are manipulating spacetime it would create a gravitational flow.

Basically I'm asking if it's possible this device is producing force by passing gravitons between groups of microwaves?

Yes I realize this probably sounds like crazy pseudo-science so I apologize in advance if you think this wasted your time.
Title: Re: EM Drive Developments - related to space flight applications - Thread 2
Post by: SH on 05/03/2015 11:55 pm
Very interesting suggestion.  But I would have expected random vibrations to produce random walk motion in one direction with a ratchet form of stick-slip friction or a bearing acting with a ratchet-like action, as found in molecular motors...
The ratchet-like action would explain why it wants to move in only one direction (a factor TheTraveller has pointed out). (Without the ratchet-like action, just with stick slip friction it would initially move in either direction, depending on initial conditions)

The direction of motion could be controlled with a ratchet, but that would be pretty difficult to conceal and so would be an unlikely way to cheat.

More likely would be to have a very slight inclination, or to set it up so there is slightly more friction on one side than the other, thus controlling the initial otherwise random direction of movement. 
Title: Re: EM Drive Developments - related to space flight applications - Thread 2
Post by: Rodal on 05/04/2015 12:27 am
Very interesting suggestion.  But I would have expected random vibrations to produce random walk motion in one direction with a ratchet form of stick-slip friction or a bearing acting with a ratchet-like action, as found in molecular motors...
The ratchet-like action would explain why it wants to move in only one direction (a factor TheTraveller has pointed out). (Without the ratchet-like action, just with stick slip friction it would initially move in either direction, depending on initial conditions)

The direction of motion could be controlled with a ratchet, but that would be pretty difficult to conceal and so would be an unlikely way to cheat.

More likely would be to have a very slight inclination, or to set it up so there is slightly more friction on one side than the other, thus controlling the initial otherwise random direction of movement.
I didn't mean that an actual ratchet was used to conceal the motion of course, instead I meant that something in the system is naturally acting as a ratchet, that's why I gave the example with references about molecular motors whose motion work as a ratchet random walk.   

https://www.youtube.com/watch?v=FsJJrP7hBoA

https://www.youtube.com/watch?v=-7AQVbrmzFw

The references I gave in my prior post actually deal with random walks that have a preferred direction, called in academia ratchet motion.

A biased random walk is another type of possible random walk.  But due to biased stick-slip it would be a ratchet random walk.

 One would have to examine the system, for anything that unintentionally acts like a ratchet.

just a few examples of a ratchet mechanism due to stick slip friction:


https://www.youtube.com/watch?v=JWLXmY0QzP8

https://www.youtube.com/watch?v=aCN-HEBsdYM

https://www.youtube.com/watch?v=78l17ntJeqo
Title: Re: EM Drive Developments - related to space flight applications - Thread 2
Post by: SH on 05/04/2015 12:55 am
One would have to examine the system, for anything that unintentionally acts like a ratchet.

Interesting.  However, this would only be necessary in order to figure out exactly what was going on, if other methods of measuring the force were negative.

The real test that should be done is measuring the force using different methods, because if the experiment is somehow interacting with one type of measurement apparatus, it is unlikely to be interacting with others.  If the force is real, then any method of measuring the force should produce the same result.

Perhaps we have been thinking about this problem the wrong way, trying to think of ways that a force would be generated, rather than trying to think of ways that the measurement apparatus used by EW might possibly be biased by the experiment.
Title: Re: EM Drive Developments - related to space flight applications - Thread 2
Post by: Rodal on 05/04/2015 12:59 am
One would have to examine the system, for anything that unintentionally acts like a ratchet.

Interesting.  However, this would only be necessary in order to figure out exactly what was going on, if other methods of measuring the force were negative.

The real test that should be done is measuring the force using different methods, because if the experiment is somehow interacting with one type of measurement apparatus, it is unlikely to be interacting with others.  If the force is real, then any method of measuring the force should produce the same result.

Perhaps we have been thinking about this problem the wrong way, trying to think of ways that a force would be generated, rather than trying to think of ways that the measurement apparatus used by EW might possibly be biased by the experiment.

Well, that was one my earlier thoughts of course (probably @frobnicat remembers  :) )

I thought there could be chaotic motion resulting from nonlinear coupling in the pendulum setup.  I actually modeled with Mathematica the nonlinear coupled equations of motion and I found certain conditions leading to chaotic motion (I posted them in earlier pages of the first thread).  @frobnicat warned me that was perhaps too elaborate a thought: apparently he was right as the data from NASA Eagleworks that I analyzed did not display any chaotic motion that I could pick up from my FFT analysis.

But I didn't model the whole intricate system.  I only modeled what Paul March was able to give me details for (in the first thread Paul March also had to stop posting due to abusive rude behavior from other posters  :(  so the number of questions I could ask him were limited  ) 
Title: Re: EM Drive Developments - related to space flight applications - Thread 2
Post by: ThinkerX on 05/04/2015 01:13 am
Quote
The real test that should be done is measuring the force using different methods, because if the experiment is somehow interacting with one type of measurement apparatus, it is unlikely to be interacting with others.  If the force is real, then any method of measuring the force should produce the same result.

Except, in a sense, we already have that.  Shawyer tested one of his devices (and maybe others???) on the turntable; while Eagleworks uses a very different means for their device.  Should Eagleworks somehow produce a device that generates 'thrust' using both detection instruments, then that goes a long ways towards verifying this effect...especially if both are done in a vacuum.  Hmmm...would the turntable device even fit in a vacuum chamber?
Title: Re: EM Drive Developments - related to space flight applications - Thread 2
Post by: aminordisaster on 05/04/2015 01:23 am
I posted yesterday in this thread http://forum.nasaspaceflight.com/index.php?topic=37438.300 (http://forum.nasaspaceflight.com/index.php?topic=37438.300) hoping to get a reply to a question on if this device producing gravity I realize now the question I posted was vague and the way I comprised the post might look childish so I'll try to expand on it.

The original post was as follows.
Quote
I apologize in advance my understanding is likely no where near where it should be but, there are no stupid questions only stupid people so prove me stupid.

Is it possible this device is condensing spacetime at one side and expanding it at the other creating a gravitational flow to one side?

This could explain some things like why when more power is put in the force becomes more directional or why the force changes depending on its orientation to the Earth's gravitational field.

Maybe somebody should place an atomic clock in the force it is producing.

I would like to correct a mistake in my original question before I start. When I said expanding spacetime at one side that is wrong it would simply be less compressed than the other side.

As I said in my original post my understanding of physics is not where it should be so it should be easy to prove this wrong for most of you and if you take 5 minutes to do so I will be extremely grateful.

I'll explain how I think this might be happening. If there is a denser concentration of microwaves in one side of the chamber compared to the other and these groups of microwaves are manipulating spacetime it would create a gravitational flow.

Basically I'm asking if it's possible this device is producing force by passing gravitons between groups of microwaves?

Yes I realize this probably sounds like crazy pseudo-science so I apologize in advance if you think this wasted your time.

Simply put, photons carry the electromagnetic force of which microwaves are a part of. Photons have no mass therefor do not manipulate spacetime, only travel through it. The term "denser" can not apply to a massless particle.

The theoretical graviton is similar to the photon in that it is massless and it carries the gravitational force. Any mechanism for the absorption or emission of gravitons hasn't made much sense.

If this interests you, may i suggest "The Theory of Almost Everything" by Robert Derter. It will introduce you to some basic principles you will need to know.

Everyone knew nothing before they knew something!
Title: Re: EM Drive Developments - related to space flight applications - Thread 2
Post by: SH on 05/04/2015 01:34 am
It is very sad when people come to a forum hiding under monickers behaving in an abusive way which they would never dare do face to face or using their real names, and prevent communication with researchers, oh well

I don't blame people for being close-minded about something that appears to violate the known laws of physics.  From religion to pseudo science to mystics, mind readers, astrology, a person is constantly being bombarded by contradictory beliefs of others, and in general I think it is important to build up an established basis of truths and be close-minded to any idea that violates those truths. 

I also don't blame people for being frustrated at the media for sensationalizing articles that appear to violate the known laws of physics, and being frustrated at how quickly they see other people will reject established laws of physics, and I can see how they would be upset at the authors for misleading people into believing something they know is false, just as I am sometimes upset to see a psychic mislead my girlfriend into thinking she has received a message from her dead grandmother.  It's in effect preying upon the less knowledgeable and filling their heads with jelly beans, and contributing to a culture that does not respect science.

Therefore, I can understand how that would cause a lot of upset people to come here in an angry state to vent about how all this is garbage...

I personally am quite intrigued by this EM drive, and I am allowing myself on this rare occasion to indulge in the act of fantasizing about the implications if it is proven to be real....but it does not bother me to see these other more close-minded people.

I think it is simply important to have a space that is separated, where those who choose to entertain the idea can do so without continuous distraction from those who want to vent their frustration.
Title: Re: EM Drive Developments - related to space flight applications - Thread 2
Post by: Notsosureofit on 05/04/2015 01:39 am
I posted yesterday in this thread http://forum.nasaspaceflight.com/index.php?topic=37438.300 (http://forum.nasaspaceflight.com/index.php?topic=37438.300) hoping to get a reply to a question on if this device producing gravity I realize now the question I posted was vague and the way I comprised the post might look childish so I'll try to expand on it.

The original post was as follows.
Quote
I apologize in advance my understanding is likely no where near where it should be but, there are no stupid questions only stupid people so prove me stupid.

Is it possible this device is condensing spacetime at one side and expanding it at the other creating a gravitational flow to one side?

This could explain some things like why when more power is put in the force becomes more directional or why the force changes depending on its orientation to the Earth's gravitational field.

Maybe somebody should place an atomic clock in the force it is producing.

I would like to correct a mistake in my original question before I start. When I said expanding spacetime at one side that is wrong it would simply be less compressed than the other side.

As I said in my original post my understanding of physics is not where it should be so it should be easy to prove this wrong for most of you and if you take 5 minutes to do so I will be extremely grateful.

I'll explain how I think this might be happening. If there is a denser concentration of microwaves in one side of the chamber compared to the other and these groups of microwaves are manipulating spacetime it would create a gravitational flow.

Basically I'm asking if it's possible this device is producing force by passing gravitons between groups of microwaves?

Yes I realize this probably sounds like crazy pseudo-science so I apologize in advance if you think this wasted your time.

Simply put, photons carry the electromagnetic force of which microwaves are a part of. Photons have no mass therefor do not manipulate spacetime, only travel through it. The term "denser" can not apply to a massless particle.

The theoretical graviton is similar to the photon in that it is massless and it carries the gravitational force. Any mechanism for the absorption or emission of gravitons hasn't made much sense.

If this interests you, may i suggest "The Theory of Almost Everything" by Robert Derter. It will introduce you to some basic principles you will need to know.

Everyone knew nothing before they knew something!

Confined photons in particular do have an energy density and affect spacetime as does any other (ie the cavity when filled w/ photons is heavier than when empty)
Title: Re: EM Drive Developments - related to space flight applications - Thread 2
Post by: tchernik on 05/04/2015 01:45 am
I just wish we could remain on topic and talk about Emdrive experimental results, instead of the umpteenth attempt of refutation from new comers, that add nothing to the same umpteenth+1 reasons already brought and discussed here.

I'm not hostile to criticism, but the arguments of violation of conservation momentum, conservation of energy and relativity are well known. Just read the thread history people.
Title: Re: EM Drive Developments - related to space flight applications - Thread 2
Post by: KelvinZero on 05/04/2015 01:53 am
I think it is simply important to have a space that is separated, where those who choose to entertain the idea can do so without continuous distraction from those who want to vent their frustration.
Yes and the moderators have made some attempts.

Suggestion: perhaps this thread could be temporarily moved to a harder to find location or renamed or a new thread (3) created. (this last one would completely solve the apparent direct linking, but any EM thread that pops to the top of the advanced topics forum will get hits from newcomers who don't read the fineprint)

It isnt my place to suggest something to the moderators though. This is a suggestion to people who post regularly here, to think about suggesting to the moderators in whichever variation suits you.
Title: Re: EM Drive Developments - related to space flight applications - Thread 2
Post by: inquisitive-j on 05/04/2015 02:04 am
I just wish we could remain on topic and talk about Emdrive experimental results, instead of the umpteenth attempt of refutation from new comers, that add nothing to the same umpteenth+1 reasons already brought and discussed here.

I'm not hostile to criticism, but the arguments of violation of conservation momentum, conservation of energy and relativity are well known. Just read the thread history people.

I agree completely. It's well established that it "shouldn't" work. The question is if it does work. I really wish people would leave the naysaying and the conspiracy theories at the door. It's useful to point out things that might create the appearance of thrust and to suggest a way to control for those things. It's not helpful to say that it can't work so stop experimenting. I also think that constructing theories to explain a thrust we aren't yet sure exists is a bit premature to say the least.

 I think we should concentrate on getting higher quality data. From there we can construct or refute theories.
Title: Re: EM Drive Developments - related to space flight applications - Thread 2
Post by: TheTraveller on 05/04/2015 03:11 am
"High Fidelity Test Article" is just a name/label... Copied from Shawyer's literature.
It could have been named "1000N/kW Test Article"
Would that have made any difference without independent testing?
Interesting.
Can't find where Shawyer gives it that label. His web page on the Flight Thruster never mentions it.
http://www.emdrive.com/flightprogramme.html
Please share your information.
Thanks.
Title: Re: EM Drive Developments - related to space flight applications - Thread 2
Post by: NovaSilisko on 05/04/2015 03:26 am
It's useful to point out things that might create the appearance of thrust and to suggest a way to control for those things. It's not helpful to say that it can't work so stop experimenting.

Entirely agreed. I'm still a skeptic but I still want to see it tested thoroughly and robustly, just giving up at it solves nothing. I want to see each and every possible source of thrust eliminated until either it stops producing thrust, or a genuine mystery force remains.
Title: Re: EM Drive Developments - related to space flight applications - Thread 2
Post by: Rodal on 05/04/2015 03:37 am
I posted yesterday in this thread http://forum.nasaspaceflight.com/index.php?topic=37438.300 (http://forum.nasaspaceflight.com/index.php?topic=37438.300) hoping to get a reply to a question on if this device producing gravity I realize now the question I posted was vague and the way I comprised the post might look childish so I'll try to expand on it.

The original post was as follows.
Quote
I apologize in advance my understanding is likely no where near where it should be but, there are no stupid questions only stupid people so prove me stupid.

Is it possible this device is condensing spacetime at one side and expanding it at the other creating a gravitational flow to one side?

This could explain some things like why when more power is put in the force becomes more directional or why the force changes depending on its orientation to the Earth's gravitational field.

Maybe somebody should place an atomic clock in the force it is producing.

I would like to correct a mistake in my original question before I start. When I said expanding spacetime at one side that is wrong it would simply be less compressed than the other side.

As I said in my original post my understanding of physics is not where it should be so it should be easy to prove this wrong for most of you and if you take 5 minutes to do so I will be extremely grateful.

I'll explain how I think this might be happening. If there is a denser concentration of microwaves in one side of the chamber compared to the other and these groups of microwaves are manipulating spacetime it would create a gravitational flow.

Basically I'm asking if it's possible this device is producing force by passing gravitons between groups of microwaves?

Yes I realize this probably sounds like crazy pseudo-science so I apologize in advance if you think this wasted your time.

Simply put, photons carry the electromagnetic force of which microwaves are a part of. Photons have no mass therefor do not manipulate spacetime, only travel through it. The term "denser" can not apply to a massless particle.

The theoretical graviton is similar to the photon in that it is massless and it carries the gravitational force. Any mechanism for the absorption or emission of gravitons hasn't made much sense.

If this interests you, may i suggest "The Theory of Almost Everything" by Robert Derter. It will introduce you to some basic principles you will need to know.

Everyone knew nothing before they knew something!

Confined photons in particular do have an energy density and affect spacetime as does any other (ie the cavity when filled w/ photons is heavier than when empty)

http://hyperphysics.phy-astr.gsu.edu/hbase/quantum/phodens.html

http://hyperphysics.phy-astr.gsu.edu/hbase/quantum/rayj.html#c1

https://cas.web.cern.ch/cas/Denmark-2010/Lectures/Wolski-2.pdf
Title: Re: EM Drive Developments - related to space flight applications - Thread 2
Post by: TheTraveller on 05/04/2015 03:37 am
I just wish we could remain on topic and talk about Emdrive experimental results, instead of the umpteenth attempt of refutation from new comers, that add nothing to the same umpteenth+1 reasons already brought and discussed here.

I'm not hostile to criticism, but the arguments of violation of conservation momentum, conservation of energy and relativity are well known. Just read the thread history people.
Sounds good to me.

1st thing for EWs to do is to realise, using their existing thrust measurement setup is that they will never measure thrust as long as the EM Drive is restrained from movement and not put into Motor or Generator mode by the application of an outside initial force.

They need to apply a small force to the big end to see acceleration (Motor mode) or a force to the small end to measure resistance (Generator mode).

Shawyer has also made it clear there is no need to put a dielectric inside the cavity & stated that doing so will reduce cavity Q (which will reduce thrust) and increase losses.

Seems to me that if anyone wishes to test an EM Drive, they should follow what Shawyer has said in what to avoid inside the cavity and how to put the drive into either Motor mode or Generator mode.

Matters not if there is agreement or not with Shawyer's theory.
http://www.emdrive.com/theorypaper9-4.pdf

His testing instructions are very clear.
http://www.emdrive.com/EmDriveForceMeasurement.pdf

I would speculate that an internal dielectric, placed on the small end wall, will generate a thrust, without initial movement as if a thrust had been placed on the small end plate. The direction of this dielectric generated force will not put the EM Drive into the desired accelerate / Motor mode but instead put it into the resistance / Generator mode, meaning the EM Drive will try to resist the thrust generated by the dielectric.

This seems clear from my reading of the data & emails Shawyer has provided.

If I was doing the tests, I would setup to measure the resistance / Generator mode as all you need to do is to apply a force to the small end and measure if there is any resistance to that force generated by the EM Drive. Seems easier to do than indirectly measuring acceleration via Shawyers suggested method as attached.
Title: Re: EM Drive Developments - related to space flight applications - Thread 2
Post by: Stormbringer on 05/04/2015 04:07 am

Shawyer has also made it clear there is no need to put a dielectric inside the cavity & stated that doing so will reduce cavity Q (which will reduce thrust) and increase losses.

Seems to me that if anyone wishes to test an EM Drive, they should follow what Shawyer has said in what to avoid inside the cavity and how to put the drive into either Motor mode or Generator mode.

{snipped to keep the focus on my target}
His testing instructions are very clear.
http://www.emdrive.com/EmDriveForceMeasurement.pdf

{snipped to keep focus on my target}

This seems clear from my reading of the data & emails Shawyer has provided.


This point coupled with another poster's post (vulture4 at post 323 in the other thread http://forum.nasaspaceflight.com/index.php?topic=37438.msg1368687#msg1368687 ) revealing how projects like these are funded or not clarifies something that has troubled me for years. Something I have found inexplicable now makes more sense.

NASA also attempted to replicate Podkletnov's anomalous gravity effect in a spinning superconductor. They failed to replicate the effect and vindicate Podkletnov. Subsequently Podkletnov claimed and the engineers involved apparently verified that they could not fabricate a test article identical to his specification nor could they replicate the rotational velocity that he specified with materials they had on hand.

Obviously with these deficiencies extant they could not replicate but neither could they really nullify his work. When I read of Podkletnov's objections I was astonished that NASA, In my mind the most formidable scientific and technically adept organization on earth would choose either to ignore vital technical details or else be unable to provide the necessary resources for a project they chose to take on like that.

Now I am sadder and wiser. :(
Title: Re: EM Drive Developments - related to space flight applications - Thread 2
Post by: SethR on 05/04/2015 04:19 am
Could a gravitational "Meissner effect" explain what we see here?

If such a gravitational anomaly could exist, it might explain Shawyer's early experiments?
Title: Re: EM Drive Developments - related to space flight applications - Thread 2
Post by: ppnl on 05/04/2015 04:43 am
I just wish we could remain on topic and talk about Emdrive experimental results, instead of the umpteenth attempt of refutation from new comers, that add nothing to the same umpteenth+1 reasons already brought and discussed here.

I'm not hostile to criticism, but the arguments of violation of conservation momentum, conservation of energy and relativity are well known. Just read the thread history people.

Well by the thread title this isn't the place to discuss experimental results. This is the thread to discuss space flight applications. That pretty much demands a discussion of COE, COM, frame dependence and such. If it violates COE can we make it power itself? And if we create frame dependence do we need to launch with the earths orbit or against? These are application relevant questions.

I certainly have no problem with people discussing the experimental setup but really who is off topic here?
Title: Re: EM Drive Developments - related to space flight applications - Thread 2
Post by: TheTraveller on 05/04/2015 04:52 am
Very interesting suggestion.  But I would have expected random vibrations to produce random walk motion in one direction with a ratchet form of stick-slip friction or a bearing acting with a ratchet-like action, as found in molecular motors...
The ratchet-like action would explain why it wants to move in only one direction (a factor TheTraveller has pointed out). (Without the ratchet-like action, just with stick slip friction it would initially move in either direction, depending on initial conditions)

The direction of motion could be controlled with a ratchet, but that would be pretty difficult to conceal and so would be an unlikely way to cheat.

More likely would be to have a very slight inclination, or to set it up so there is slightly more friction on one side than the other, thus controlling the initial otherwise random direction of movement.
I didn't mean that an actual ratchet was used to conceal the motion of course, instead I meant that something in the system is naturally acting as a ratchet, that's why I gave the example with references about molecular motors whose motion work as a ratchet random walk.   

https://www.youtube.com/watch?v=FsJJrP7hBoA

https://www.youtube.com/watch?v=-7AQVbrmzFw

The references I gave in my prior post actually deal with random walks that have a preferred direction, called in academia ratchet motion.

A biased random walk is another type of possible random walk.  But due to biased stick-slip it would be a ratchet random walk.

 One would have to examine the system, for anything that unintentionally acts like a ratchet.

just a few examples of a ratchet mechanism due to stick slip friction:


https://www.youtube.com/watch?v=JWLXmY0QzP8

https://www.youtube.com/watch?v=aCN-HEBsdYM

https://www.youtube.com/watch?v=78l17ntJeqo
Yes there is a Rachet mode but internal to the EM Drive. As Shawyer has explained, a EM Drive will not of itself move. It needs an unbalancing of the cavity forces. Push it one way and it resists as if it had infinite mass. Push it the other way and it moves as if it had no mass. This action forms a natural ratchet.

In the design of Shawyers space plane it lifts a 300t vehicle to GEO altitude but at 0.05g. I suggest the low g is due to the EM Drive lift engine working in ratchet mode, where random upward forces fractionally lift a weightless vehicle via motor mode, while random downward forces are resisted by generator mode. Thus the EM Drive lift motors will cause the 300t mass to act like it has 0 mass and using random activated motor / generator mode very slowly climbs from sea level to GEO altitude.

What this also says is a EM Drive, for space flight, needs an auxiliary engine, Hall thruster would work ok, to ensure the EM Drive cavity was always in Motor mode. As the EM Drive will make the vehicle virtually massless, in relation to the Hall thruster, the required auxilary thrust would be very low.

This operarional characteristics may be why Boeing lost interest. As either be happy with random ratched mode acceleration of 0.05g or you need an auxilary thruster to continually place the cavity in motor mode.

So maybe rename the EM Drive to EM Mass Neutralisation Device.
Title: Re: EM Drive Developments - related to space flight applications - Thread 2
Post by: Stormbringer on 05/04/2015 05:01 am
if what  you say is true there is no way Boeing or anyone else involved with any form of propulsion but especially aviation or space flight would lose interest. any reduced mass is astoundingly useful. Even if the thing only reduced mass by 1 percent it would revolutionize everything. Nothing would ever be the same again. Automobiles, trains, planes, rockets; everything.
Title: Re: EM Drive Developments - related to space flight applications - Thread 2
Post by: meberbs on 05/04/2015 05:48 am
It would seem Yang Juan uses a very similar explanation in her recent EM Drive paper. In case you have not had an opportunity to read it, here is the link: http://www.emdrive.com/yang-juan-paper-2012.pdf

I hadn't read through that paper before. While they make the incorrect claim that classical EM can explain the thrust, they don't include enough detail in the theory to see where they went wrong. Their statements of the EM creating a radiation pressure, and that any force would be calculated by using surface integrals is basically correct. If this calculation was correctly done with Maxwell's equations, the time-averaged force would be 0.

Most of the paper is on the experiment and setup. I haven't gone through all of their calculation details, but it looks like they are trying to account for various sources of error.

Quote from: Yang Juan
Based on the installation orientation of Figure 2, when the thruster is working normally, if the electromagnetic coil 3 is working, the net thrust is from the microwave resonator large end to the small end; if the electromagnetic coil 4 is working, the net thrust is from the microwave resonator small end to the large end.   

I may be misunderstanding this part, since it seems to have been translated from Chinese. It seems like they are saying that by using the different coils, the direction of the force changes. The coils are external to the cavity based on the diagram. Unless they mean that the coils are EM based sensors to detect the motion, but that would be a weird choice in this case since leaked EM radiation could throw it off. If they are active devices, then it seems like an alternative possible source for the force ... turning on an electromagnet near a piece of metal.

Does anyone know what those coils actually are for? They don't seem to be mentioned later.

I haven't read the entire thread, but I feel like this paper was probably discussed at some point, if so could someone point me to where that was? I found nothing from a search. They provide enough structural details that an assessment of how well they controlled for biases in the experiment design could be made. I don't want to rehash that if it has already been thoroughly considered.
Title: Re: EM Drive Developments - related to space flight applications - Thread 2
Post by: PushHigher on 05/04/2015 06:09 am
For the last week or so, I've been reading through this thread and trying to grapple with the physics involved using my limited understanding of relativity.  As a child I used to love reading about Einstein and his thought experiments.  I decided to try it and came up with the following diagram of my thought process.  Take it with a grain of salt.  It's likely wrong, full of spelling mistakes, bad scientific assumptions etc.

One of the points I feel is lacking in this thread is how we should try to optimize this effect by increasing the EM Drive's efficiency. Shawyer eludes to this in his recent conversations.  If we could increase the effect significantly it becomes much easier to prove/disprove.  Further more, increasing efficiency would be key for space-travel  so the diagram also includes some thoughts I had on the subject.

Feedback welcome.


Title: Re: EM Drive Developments - related to space flight applications - Thread 2
Post by: TheTraveller on 05/04/2015 06:10 am
if what  you say is true there is no way Boeing or anyone else involved with any form of propulsion but especially aviation or space flight would lose interest. any reduced mass is astoundingly useful. Even if the thing only reduced mass by 1 percent it would revolutionize everything. Nothing would ever be the same again. Automobiles, trains, planes, rockets; everything.
Is not what I say. Is what Shawyer explains in his how to measure the forces pdf and in his explanation of how his space plane would do a 0.05g vertical lift.

As another example of EM Drive ratchet mode operation, assume we had a EM Drive with a motor & generator mode maximum force generation of 9.8 Newtons and the device had a mass of 0.5kg.

Now support it 1 mtr off the ground via say a small pedestal table, oriented such that the force of gravity would put the EM Drive into Generator / force resistance mode.

Next switch it on.

Now remove the pedestal support table.

Observe it is hovering as the downward 0.5kg weight is opposed by generator mode, which has a max ability to resist 9.8 Newtons of force or 1kg of mass at the Earth's surface.

Note that if you push down with your finger, it resists your pressure and feels as if the pedestal table were still there.

If you push up, from under the hovering EM Drive, it moves up and appears to be massless / weightless.

Next start placing additional 50g masses on the top of the hovering EMDrive.

After placing 10 of these (0.5kg) on the top, again now push down and observe the EM Drive moves down as if massless because your additional finger pressure exceeds the EM Drive generator mode max force generation capability. Pushing up achieves the same thing.

Now place the 11th 50g mass on the top of the EM Drive and observe it fall to the ground as if it's power had been cut off.

These thought experiment results are supported by what Shawyer has written but not in this way.

I trust this shows how the EM Drive is unlike anything humanity has experienced before. So please do not move forward thinking it is like a propellantless Hall thruster or rocket motor that generates thrust when the cavity is filled with microwave energy.
Title: Re: EM Drive Developments - related to space flight applications - Thread 2
Post by: Eye_one on 05/04/2015 06:18 am


Simply put, photons carry the electromagnetic force of which microwaves are a part of. Photons have no mass therefor do not manipulate spacetime, only travel through it. The term "denser" can not apply to a massless particle.

The theoretical graviton is similar to the photon in that it is massless and it carries the gravitational force. Any mechanism for the absorption or emission of gravitons hasn't made much sense.

If this interests you, may i suggest "The Theory of Almost Everything" by Robert Derter. It will introduce you to some basic principles you will need to know.

Everyone knew nothing before they knew something!

Photons effect spacetime just like every other particle. Hence why a box full of light weighs more than a box full of nothing.

If the term denser doesn't describe more particles condensing to one side versus the other how should I describe it?

I'm a bit confused by the part on the graviton are you saying the mechanism I described doesn't make sense or no description makes sense?

I'll look into that book I haven't been to the public library in far longer than I care to admit and I have a few other books I'd like to read.

Sorry to everyone for going a bit of the topic of the use of this device in space travel but, I believe finding out exactly what it's doing is probably the most important aspect of how it can be applied in a spacecraft.
Title: Re: EM Drive Developments - related to space flight applications - Thread 2
Post by: frobnicat on 05/04/2015 09:09 am
if what  you say is true there is no way Boeing or anyone else involved with any form of propulsion but especially aviation or space flight would lose interest. any reduced mass is astoundingly useful. Even if the thing only reduced mass by 1 percent it would revolutionize everything. Nothing would ever be the same again. Automobiles, trains, planes, rockets; everything.
Is not what I say. Is what Shawyer explains in his how to measure the forces pdf and in his explanation of how his space plane would do a 0.05g vertical lift.

As another example of EM Drive ratchet mode operation, assume we had a EM Drive with a motor & generator mode maximum force generation of 9.8 Newtons and the device had a mass of 0.5kg.

Now support it 1 mtr off the ground via say a small pedestal table, oriented such that the force of gravity would put the EM Drive into Generator / force resistance mode.

Next switch it on.

Now remove the pedestal support table.

Observe it is hovering as the downward 0.5kg weight is opposed by generator mode, which has a max ability to resist 9.8 Newtons of force or 1kg of mass at the Earth's surface.


So if I read you well and equivalence principle holds, having a rocket in deep space accelerating, by conventional mean, at 1g, a floor that is orthogonal to this acceleration, a pedestal resting on this floor, a powered EM drive resting on this pedestal in same configuration, remove the pedestal and one will observe the EM drive hovering above that floor (that is still accelerating at 1g). Meaning we now have a powered EM drive not needing to be "pushed" to accelerate at 1g (there is no longer any interaction between rocket and device).

Stop the conventional thrust of the rocket : the rocket will stop accelerating and proceed as an inertial mass at constant velocity (relative to whatever inertial frame). Let the device escape from an open front bay : it will continue to accelerate at 1g since it was no longer interacting with the rocket when the change in acceleration of the rocket occurred (no interaction => whatever change in rocket trajectory ignored). We now have a "conventionally" accelerating (thrusting) EM drive needing no added force.

So why bother with a Hall thruster ? Just put behind a big dumb powder booster that makes your EM drive accelerate at 1g for a fraction of a second the time it takes for the EM drive to "record" that acceleration as a "starting point". BTW, same argument above could be made if acceleration was 0.5g instead of 1g : this means we now have to add a new intrinsic variable to a moving object. What physical mechanism explains this memory effect of "initial acceleration" into an ongoing acceleration of given magnitude ?

Quote
...
I trust this shows how the EM Drive is unlike anything humanity has experienced before. So please do not move forward thinking it is like a propellantless Hall thruster or rocket motor that generates thrust when the cavity is filled with microwave energy.

Don't underestimate the aptitude of intelligent people to integrate counter intuitive formal systems when they show internal consistency.
Title: Re: EM Drive Developments - related to space flight applications - Thread 2
Post by: TheTraveller on 05/04/2015 10:08 am
if what  you say is true there is no way Boeing or anyone else involved with any form of propulsion but especially aviation or space flight would lose interest. any reduced mass is astoundingly useful. Even if the thing only reduced mass by 1 percent it would revolutionize everything. Nothing would ever be the same again. Automobiles, trains, planes, rockets; everything.
Is not what I say. Is what Shawyer explains in his how to measure the forces pdf and in his explanation of how his space plane would do a 0.05g vertical lift.

As another example of EM Drive ratchet mode operation, assume we had a EM Drive with a motor & generator mode maximum force generation of 9.8 Newtons and the device had a mass of 0.5kg.

Now support it 1 mtr off the ground via say a small pedestal table, oriented such that the force of gravity would put the EM Drive into Generator / force resistance mode.

Next switch it on.

Now remove the pedestal support table.

Observe it is hovering as the downward 0.5kg weight is opposed by generator mode, which has a max ability to resist 9.8 Newtons of force or 1kg of mass at the Earth's surface.


So if I read you well and equivalence principle holds, having a rocket in deep space accelerating, by conventional mean, at 1g, a floor that is orthogonal to this acceleration, a pedestal resting on this floor, a powered EM drive resting on this pedestal in same configuration, remove the pedestal and one will observe the EM drive hovering above that floor (that is still accelerating at 1g). Meaning we now have a powered EM drive not needing to be "pushed" to accelerate at 1g (there is no longer any interaction between rocket and device).

Stop the conventional thrust of the rocket : the rocket will stop accelerating and proceed as an inertial mass at constant velocity (relative to whatever inertial frame). Let the device escape from an open front bay : it will continue to accelerate at 1g since it was no longer interacting with the rocket when the change in acceleration of the rocket occurred (no interaction => whatever change in rocket trajectory ignored). We now have a "conventionally" accelerating (thrusting) EM drive needing no added force.

So why bother with a Hall thruster ? Just put behind a big dumb powder booster that makes your EM drive accelerate at 1g for a fraction of a second the time it takes for the EM drive to "record" that acceleration as a "starting point". BTW, same argument above could be made if acceleration was 0.5g instead of 1g : this means we now have to add a new intrinsic variable to a moving object. What physical mechanism explains this memory effect of "initial acceleration" into an ongoing acceleration of given magnitude ?

Quote
...
I trust this shows how the EM Drive is unlike anything humanity has experienced before. So please do not move forward thinking it is like a propellantless Hall thruster or rocket motor that generates thrust when the cavity is filled with microwave energy.

Don't underestimate the aptitude of intelligent people to integrate counter intuitive formal systems when they show internal consistency.
Your example is interesting.

Here Shawyer says the measurements for the Flight Thruster where done with the Em Drive oriented up and down.
http://www.emdrive.com/flightprogramme.html
So following your excellent example, the EM Drive should do as you suggest. Continue to generate thrust at 1g, once removed from the Earth reference frame.

According to what Shawyer says in his "How to measure thrust" pdf and his space plane pdf, what you suggest should happen. The hovering EM Drive in a 1g accelerating rocket (frame of reference) should continue to hover in a 1g accelerating frame of reference, once exiting the bay doors. Assuming HAL opened them on command.

BTW it is not my EM Drive. I just read the data Shawyer has presented and worked out how such as device would work if placed in front of me and I had to work out how to test it. Which is something I have had to do many times.

Let the testing begin.
Title: Re: EM Drive Developments - related to space flight applications - Thread 2
Post by: Mulletron on 05/04/2015 10:24 am
Mr. Shawyer passed this along to share. See attachment. I didn't see it on his website. Maybe I missed it.

Looks like it goes along with the IAC-14 presentation here:
http://www.emdrive.com/iac2014presentation.pdf

The conference, page 133:
http://www.iafastro.org/wp-content/uploads/2014/09/IAC-2014-Final-Programme.pdf

He didn't give permission to share the email text this time. If he ever emails back, I'll ask for permission.

We're all very fortunate to have the inventor of EmDrive contributing to the discussion.
Title: Re: EM Drive Developments - related to space flight applications - Thread 2
Post by: Mulletron on 05/04/2015 10:44 am
There's other perfectly sane ways of interacting with the QV which have been posted literally a thousand times in THIS forum over and over again since October.

I have been reading these threads quite diligently lately but I am not sure what you are referring to.  A link would be appreciated!

Yes, "lately" is the operative word. I've stopped posting the same stuff in hopes newcomers will start at page 1 and make their own conclusions. Forums are great for discussion happening here and now, but horrible for knowledge management. People just hopping in the discussion now are in the same situation as we were back at the start:
http://forum.nasaspaceflight.com/index.php?topic=29276.0
I'm pretty clueless now, but I was super really clueless back then.

Speaking of KM, is there anyone out there that has what it takes and is willing to volunteer and set up a wiki or something?
http://en.wikipedia.org/wiki/MediaWiki

I've set up a MediaWiki server before and it wasn't too bad. That is a good platform. I'm simply stretched too thin right now to try it again. My previous attempt at KM (just a simple Google Doc, which was pretty bad didn't catch on so I deleted it:
https://drive.google.com/folderview?id=0B4PCfHCM1KYoTXhSUTd5ZDN2WnM&usp=sharing



Title: Re: EM Drive Developments - related to space flight applications - Thread 2
Post by: TheTraveller on 05/04/2015 10:54 am
Mr. Shawyer passed this along to share. See attachment. I didn't see it on his website. Maybe I missed it.

Looks like it goes along with the IAC-14 presentation here:
http://www.emdrive.com/iac2014presentation.pdf

The conference, page 133:
http://www.iafastro.org/wp-content/uploads/2014/09/IAC-2014-Final-Programme.pdf

He didn't give permission to share the email text this time. If he ever emails back, I'll ask for permission.

We're all very fortunate to have the inventor of EmDrive contributing to the discussion.
Please thank Roger Shawyer for this information and thank you for sharing.

Seems Shawyer has thrown down a gauntlet.
Let the games begin.
Title: Re: EM Drive Developments - related to space flight applications - Thread 2
Post by: Rodal on 05/04/2015 11:15 am
Mr. Shawyer passed this along to share. See attachment. I didn't see it on his website. Maybe I missed it.

Looks like it goes along with the IAC-14 presentation here:
http://www.emdrive.com/iac2014presentation.pdf

The conference, page 133:
http://www.iafastro.org/wp-content/uploads/2014/09/IAC-2014-Final-Programme.pdf

He didn't give permission to share the email text this time. If he ever emails back, I'll ask for permission.

We're all very fortunate to have the inventor of EmDrive contributing to the discussion.
Please thank Roger Shawyer for this information and thank you for sharing.

Seems Shawyer has thrown down a gauntlet.
Let the games begin.

Concerning the phrase “Shawyer has thrown down a gauntlet” , my understanding is that it means to challenge or confront someone.  Please explain further:

Who is being challenged by Shawyer? 
What is the challenge?
Title: Re: EM Drive Developments - related to space flight applications - Thread 2
Post by: RotoSequence on 05/04/2015 11:18 am
Concerning the phrase “Shawyer has thrown down a gauntlet” , my understanding is that it means to challenge or confront someone.  Please explain further:

Who is being challenged by Shawyer? 
What is the challenge?

I think the challenge is to catch up to the state of Shawyer's own research, so that it can be surpassed.  :)
Title: Re: EM Drive Developments - related to space flight applications - Thread 2
Post by: Rodal on 05/04/2015 11:23 am
Last year's R. Shawyer's conference presentation slides had been already posted in this thread.  I had not seen the final version of the presentation paper (the attachment in @Mulletron's post) but, although it is nice to have for reference, I did not see something there that we had not discussed or reviewed before (including the latest superconducting design by Shawyer which we have discussed multiple times), as well as his project studies for aerospace.

That conference was 6 months ago. 

There are several things that are NOT new, and that have been discussed for several months in these threads, for example:

* that R. Shawyer no longer uses any dielectric inserts in his EM Drive
* that the latest design of R. Shawyer is superconducting with Doppler compensation, and large cone angle and spherical ends



As an example of what can happen in 6 months of R&D, during the last 6 months, NASA Eagleworks reported:

1) The first time that any organization has conducted EM Drive tests in a vacuum
2) A positive signal in their interferometer tests, using an EM Drive pillbox shape as the test item.

QUESTIONS:

Is there any update on what is the progress with Shawyer's superconducting EM Drive?

Does he report an experimental Q?

Does he report any experimental measurements during the last 6 months?

Does anybody have an explanation why Shawyer reports measurements of force in opposing directions for the Demonstrator engine and ONLY for this engine (it cannot be a typo, since it is repeated in the final version of the conference paper).

If I have missed something new, that had not been reviewed previously, I would appreciate if somebody could point it out. 

If there nothing new that can be pointed out, there is no need to reply.

Thanks.
Title: Re: EM Drive Developments - related to space flight applications - Thread 2
Post by: SeeShells on 05/04/2015 01:34 pm
Wonderful breakthrough in many ways.
I was wondering has anyone given the thought to putting several of the EM drives in series?
Title: Re: EM Drive Developments - related to space flight applications - Thread 2
Post by: rgreen on 05/04/2015 01:40 pm
Following on what jknuble said about the multipactor-like effect as a possible cause of thrust. http://en.wikipedia.org/wiki/Multipactor_effect  I can't help but wonder about what's going on with the copper surface of the frustum. A quick back of the envelope (well, python) calculation shows that there's certainly enough energy in these devices to somehow atomize a small amount of copper , and propel them with enough momentum to produce a small amount of thrust.

For example, a 30 watt emdrive where 0.001% of the energy went towards atomization and 1% went toward addtional momentum of the particles... You'd have a device with 91uN thrust, propelling 1.4ng of copper a second at 65500m/s.

I can think of 3 ways to debunk this. 1) perhaps that amount of particles going that fast would be noticeable with the naked eye, so this isn't really a valid explanation. 2) stick a detector behind the thruster (are they ionized?). 3) SEM of the surface compared to scraps from the same batch of copper not used in the thrustum.

Just how would we get a net-thrust from a closed cavity with atomization.  Even if atoms are being ioniozed inside the cavity I don't see how that could result in a net thrust.  Atomization results in immediate thrust but then that creates impact on the other side of the cavity canceling out the propulsion.
I'm talking about signs of atomization on the *outside* surface of the frustum. Although it would probably be happening in the inside as well. The copper atoms would be the propellent. I'm not sure what the mechanism would be, but it's obviously more than a thermal effect, and the whole reversal in phase/thrust would be difficult to explain. It seems more simple than QV or relativity models, but it's still probably interesting physics.

It might also explain the interferometer results.
Title: Re: EM Drive Developments - related to space flight applications - Thread 2
Post by: SeeShells on 05/04/2015 01:45 pm
I know I'm new here but I've been in engineering for almost 50 years. The EM drive seems to parallel so many things I've seen in electronics and embrace harmonics and it got me thinking how it would compare to things like a YAGI antenna for gain buy linking them in series. Would you get a Q gain in thrust?
Title: Re: EM Drive Developments - related to space flight applications - Thread 2
Post by: TheTraveller on 05/04/2015 01:45 pm
Last year's R. Shawyer's conference presentation slides had been already posted in this thread.  I had not seen the final version of the presentation paper (the attachment in @Mulletron's post) but, although it is nice to have for reference, I did not see something there that we had not discussed or reviewed before (including the latest superconducting design by Shawyer which we have discussed multiple times), as well as his project studies for aerospace.

That conference was 6 months ago. 

There are several things that are NOT new, and that have been discussed for several months in these threads, for example:

* that R. Shawyer no longer uses any dielectric inserts in his EM Drive
* that the latest design of R. Shawyer is superconducting with Doppler compensation, and large cone angle and spherical ends



As an example of what can happen in 6 months of R&D, during the last 6 months, NASA Eagleworks reported:

1) The first time that any organization has conducted EM Drive tests in a vacuum
2) A positive signal in their interferometer tests, using an EM Drive pillbox shape as the test item.

QUESTIONS:

Is there any update on what is the progress with Shawyer's superconducting EM Drive?

Does he report an experimental Q?

Does he report any experimental measurements during the last 6 months?

Does anybody have an explanation why Shawyer reports measurements of force in opposing directions for the Demonstrator engine and ONLY for this engine (it cannot be a typo, since it is repeated in the final version of the conference paper).

If I have missed something new, that had not been reviewed previously, I would appreciate if somebody could point it out. 

If there nothing new that can be pointed out, there is no need to reply.

Thanks.
Roger that.
Good to know all the data that can be mined from past presentations and postings has been mined, discussed, sorted and filed away. Guess we wait for new data from EW's test of their new build, which hopefully follows Shawyers test protocol and elimination of dielectrics recommendations, at least initially.
Title: Re: EM Drive Developments - related to space flight applications - Thread 2
Post by: Rodal on 05/04/2015 01:55 pm
The challenge (not yet met) is to explain the experimental measurements as an experimental artifact or as something useful for space propulsion, with an explanation that can be embraced without objections.

Therefore, there is plenty of new observations on experimental reports that can still be made to explain the measurements.
Title: Re: EM Drive Developments - related to space flight applications - Thread 2
Post by: Mulletron on 05/04/2015 02:00 pm

Roger that.
Good to know all the data that can be mined from past presentations and postings has been mined, discussed, sorted and filed away. Guess we wait for new data from EW's test of their new build, which hopefully follows Shawyers test protocol and elimination of dielectrics recommendations, at least initially.

No, you have already shown that we need to revisit what Mr. Shawyer has been reporting. This forum has not given the necessary attention to most of it. Also look back up in the thread where @meberbs was talking about the Chinese using coils. I don't ever remember talking about that.
http://forum.nasaspaceflight.com/index.php?topic=36313.msg1368813#msg1368813

I about fainted when I read that because I'm convinced this thing works primarily off the magnetic field component inside the cavity.


I know from experiment that a magnetic field goes right though the thin copper from one side to the other. So that is trivial to pull off. Hopefully Eagleworks accounted for the magnetic field going the other way..inside to out.

@TheTraveller, keep bring in fresh insight. You're rocking this thread.
Title: Re: EM Drive Developments - related to space flight applications - Thread 2
Post by: Rodal on 05/04/2015 02:00 pm
Wonderful breakthrough in many ways.
I was wondering has anyone given the thought to putting several of the EM drives in series?
Yes, it has been discussed earlier in these threads, and it would enable testing at NASA Glenn for example.  NASA Eagleworks does not have the budget to do it.

I know I'm new here but I've been in engineering for almost 50 years. The EM drive seems to parallel so many things I've seen in electronics and embrace harmonics and it got me thinking how it would compare to things like a YAGI antenna for gain buy linking them in series. Would you get a Q gain in thrust?

The honest answer is that it is unknown, as the theories explaining the thrust meet with objections that are not yet convincingly answered and the experimental results are inconsistent.  NASA Eagleworks is actively working on further experiments with the goal to achieve consistency in their experimental results.
Title: Re: EM Drive Developments - related to space flight applications - Thread 2
Post by: TheTraveller on 05/04/2015 02:14 pm

Roger that.
Good to know all the data that can be mined from past presentations and postings has been mined, discussed, sorted and filed away. Guess we wait for new data from EW's test of their new build, which hopefully follows Shawyers test protocol and elimination of dielectrics recommendations, at least initially.

No, you have already shown that we need to revisit what Mr. Shawyer has been reporting. This forum has not given the necessary attention to most of it. Also look back up in the thread where @meberbs was talking about the Chinese using coils. I don't ever remember talking about that.

I about fainted when I read that because I'm convinced this thing works primarily off the magnetic field component inside the cavity.
http://forum.nasaspaceflight.com/index.php?topic=36313.msg1368813#msg1368813

I know from experiment that a magnetic field goes right though the thin copper from one side to the other. So that is trivial to pull off. Hopefully Eagleworks accounted for the magnetic field going the other way..inside to out.

@TheTraveller, keep bring in fresh insight. You're rocking this thread.
What I read is, the Chinese test rig has 2 coils, electromagnets, one of which is activated, once the test article moves from plum, to force the test article back to plum. Coils are calibrated as to the restorative force they generate per current input. Depending on which way the test article moves, either coil 3 or coil 4 is activated to restore the test article to plum and the current drawn determines the force generated by the test article.

Chinese thrust measurement system attached.
Title: Re: EM Drive Developments - related to space flight applications - Thread 2
Post by: Mulletron on 05/04/2015 02:23 pm

Hopefully Eagleworks accounted for the magnetic field going the other way..inside to out.


Actually they did. Right here:
http://forum.nasaspaceflight.com/index.php?topic=36313.msg1346066#msg1346066

Quoting Paul March:
Quote
Lastly find attach a slide with the results of this week's test that demonstrate that the copper frustum still generates a thrust signature when it is not in the stainless steel vacuum chamber walls.  And as you will note the forward thrust signature is similar in magnitude for the same 50W case in-air in the vacuum chamber, so I think we can start to put to bed the idea that standard E&M evanescent waves interactions with the vacuum chamber walls are the cause of these thrust signatures.

I do have to admit that those thrust signatures are barely above the noise floor. You can't deny that.

(http://forum.nasaspaceflight.com/index.php?action=dlattach;topic=36313.0;attach=814653;image)
Title: Re: EM Drive Developments - related to space flight applications - Thread 2
Post by: PushHigher on 05/04/2015 02:27 pm
Further thoughts that I think correlate to the TheTraveller's statements.

I was thinking about my post from last night and was pondering why, if pushed in the "forward" direction the drive begins and continues to accelerate.  If you imagine the drive creating two gravity wells, small/deep at rear and large/shallow at front.  Once the drive is pushed, the gravity wells would follow the drive HOWEVER the reaction is subject to the speed of light.  That momentary delay might allow for the drive to slip into the forward gravity well's wake and begin accelerating (essentially falling).

Just food for thought...
Title: Re: EM Drive Developments - related to space flight applications - Thread 2
Post by: Mulletron on 05/04/2015 02:30 pm
Further thoughts that I think correlate to the TheTraveller's statements.

I was thinking about my post from last night and was pondering why, if pushed in the "forward" direction the drive begins and continues to accelerate.  If you imagine the drive creating two gravity wells, small/deep at rear and large/shallow at front.  Once the drive is pushed, the gravity wells would follow the drive HOWEVER the reaction is subject to the speed of light.  That momentary delay might allow for the drive to slip into the forward gravity well's wake and begin accelerating (essentially falling).

Just food for thought...

Or something much more mundane:
http://usersguidetotheuniverse.com/?p=2865

A resonant cavity can ONLY support certain frequencies. If those frequencies change, say due to red or blue shift, they won't/can't resonate, and are lost. That resonant cavity that was once saturated, now has a "hole" which can be filled by more incoming radiation, said another way energy flowing back in. The transient Poynting vector. In layman's terms.

The missing Poynting vector is due to us never considering an accelerating cavity. Shawyer says it has to move first before you observe a force. He isn't just saying that. He probably observed that.

That sounds exactly like ME too. Same freaking thing. Two sides of the same coin.

ME=Classical description
EM=Quantum description

See that bottom pic. Those "dips" are the only frequencies that will exist within that range of my cavity. If they get shifted up or down (like by if I pick up the cavity and shake the crap out of it), they're history.

(http://usersguidetotheuniverse.com/wp-content/uploads/2013/03/box_photons_moving.png)
Title: Re: EM Drive Developments - related to space flight applications - Thread 2
Post by: PushHigher on 05/04/2015 02:51 pm
Mulletron -

Fantastic, I think we are saying the same thing using different analogies because in my non-scientific mind - aligned photons and gravity wells are the same thing.  From the blog post you reference it seems that once there is forward motion, the EM radiation has more force in the forward direction. 

I wonder if, as the drive accelerates forward and as the photons bounce back and forth between the rear and front that - from the perspective of a single photon (or the microwave) the frustum chamber looks like a perfect cylinder.  If this is the case, it seems that the frustum chamber would require less slope to get started and more slope to accelerate further. 

I wonder if a frustum chamber that could dynamically change the angle/slope of the walls in response to momentary acceleration might remove the theoretical speed limit.
Title: Re: EM Drive Developments - related to space flight applications - Thread 2
Post by: Notsosureofit on 05/04/2015 02:56 pm

Roger that.
Good to know all the data that can be mined from past presentations and postings has been mined, discussed, sorted and filed away. Guess we wait for new data from EW's test of their new build, which hopefully follows Shawyers test protocol and elimination of dielectrics recommendations, at least initially.

No, you have already shown that we need to revisit what Mr. Shawyer has been reporting. This forum has not given the necessary attention to most of it. Also look back up in the thread where @meberbs was talking about the Chinese using coils. I don't ever remember talking about that.

I about fainted when I read that because I'm convinced this thing works primarily off the magnetic field component inside the cavity.
http://forum.nasaspaceflight.com/index.php?topic=36313.msg1368813#msg1368813

I know from experiment that a magnetic field goes right though the thin copper from one side to the other. So that is trivial to pull off. Hopefully Eagleworks accounted for the magnetic field going the other way..inside to out.

@TheTraveller, keep bring in fresh insight. You're rocking this thread.
What I read is, the Chinese test rig has 2 coils, electromagnets, one of which is activated, once the test article moves from plum, to force the test article back to plum. Coils are calibrated as to the restorative force they generate per current input. Depending on which way the test article moves, either coil 3 or coil 4 is activated to restore the test article to plum and the current drawn determines the force generated by the test article.

Chinese thrust measurement system attached.


Yes, this is the way we built balances in the past.  The coil provides a balancing force so that the position is static and the force is calibrated against the coil current.
Title: Re: EM Drive Developments - related to space flight applications - Thread 2
Post by: Rodal on 05/04/2015 02:58 pm
Further thoughts that I think correlate to the TheTraveller's statements.

I was thinking about my post from last night and was pondering why, if pushed in the "forward" direction the drive begins and continues to accelerate.  If you imagine the drive creating two gravity wells, small/deep at rear and large/shallow at front.  Once the drive is pushed, the gravity wells would follow the drive HOWEVER the reaction is subject to the speed of light.  That momentary delay might allow for the drive to slip into the forward gravity well's wake and begin accelerating (essentially falling).

Just food for thought...

Or something much more mundane:
http://usersguidetotheuniverse.com/?p=2865

A resonant cavity can ONLY support certain frequencies. If those frequencies change, say due to red or blue shift, they won't/can't resonate, and are lost. That resonant cavity that was once saturated, now has a "hole" which can be filled by more incoming radiation, said another way energy flowing back in. The transient Poynting vector. In layman's terms.

The missing Poynting vector is due to us never considering an accelerating cavity. Shawyer says it has to move first before you observe a force. He isn't just saying that. He probably observed that.

That sounds exactly like ME too. Same freaking thing. Two sides of the same coin.

ME=Classical description
EM=Quantum description

See that bottom pic. Those "dips" are the only frequencies that will exist within that range of my cavity. If they get shifted up or down (like by if I pick up the cavity and shake the crap out of it), they're history.

(http://usersguidetotheuniverse.com/wp-content/uploads/2013/03/box_photons_moving.png)
But this is NOT what was measured at NASA Eagleworks.  Nothing needed to be pushed and nothing was pushed at NASA Eagleworks.

Now, two different experiments and mechanisms are being discussed  (#2 as treated by Mulletron and TheTraveller):

1) NASA Eagleworks.  Stationary test item.  Thrust force in same direction as movement.

2) SHAWYER.  Need moving item to measure force ?.  Thrust force in opposite direction to movement. (All kind of experimental issues here, and a lack of experimental force-vs.-time data to analyze).

Since they are different, I wonder whether we should split threads into NASA Eagleworks and a separate thead for the others.  This would prevent confusion.
Title: Re: EM Drive Developments - related to space flight applications - Thread 2
Post by: tchernik on 05/04/2015 02:58 pm

Or something much more mundane:
http://usersguidetotheuniverse.com/?p=2865

A resonant cavity can ONLY support certain frequencies. If those frequencies change, say due to red or blue shift, they won't/can't resonate, and are lost. That resonant cavity that was once saturated, now has a "hole" which can be filled by more incoming radiation, said another way energy flowing back in. The transient Poynting vector. In layman's terms.

The missing Poynting vector is due to us never considering an accelerating cavity. Shawyer says it has to move first before you observe a force. He isn't just saying that. He probably observed that.

That sounds exactly like ME too. Same freaking thing. Two sides of the same coin.

ME=Classical description
EM=Quantum description

See that bottom pic. Those "dips" are the only frequencies that will exist within that range of my cavity. If they get shifted up or down (like by if I pick up the cavity and shake the crap out of it), they're history.

(http://usersguidetotheuniverse.com/wp-content/uploads/2013/03/box_photons_moving.png)

Going into the speculation side and maybe a silly error: this observation could also match with EW's potential observation of a warpdrive signature.

If I remember well, they have mentioned something called 'boost' that seems to be a speed multiplier.

What if we are actually seeing that, a initial speed from an imparted momentum, just slightly 'boosted'?
Title: Re: EM Drive Developments - related to space flight applications - Thread 2
Post by: Mulletron on 05/04/2015 02:59 pm

But this is NOT what was measured at NASA Eagleworks.  Nothing needed to be pushed and nothing was pushed at NASA Eagleworks.

Now, two different experiments and mechanisms are being discussed  (#2 as treated by Mulletron and TheTraveller):

1) NASA Eagleworks.  Stationary test item.  Thrust force in same direction as movement.

2) SHAWYER.  Need moving item to measure force ?.  Thrust force in opposite direction to movement. (All kind of experimental issues here, and a lack of experimental force-vs.-time data to analyze).

Since they are different, I wonder whether we should split threads into NASA Eagleworks and a separate thead for the others.  This would prevent confusion.


No we're NOT going to split threads. We're going to resolve the disparity. @TheTraveller already figured it out yesterday. Maybe you missed it.....Those dielectrics are the bootstrap to get it going. The other, as mentioned by me is the constant seismic noise. The dielectrics give the preferred direction by their helicity, which is the source of the non-reciprocity as described by the transfer of linear momentum from the QV papers.

What really bothers me about Eagleworks is this quote:
Quote
We performed some very early evaluations without the dielectric resonator (TE012 mode at 2168 MHz, with power levels up to ~30 watts) and measured no significant net thrust.
http://www.libertariannews.org/wp-content/uploads/2014/07/AnomalousThrustProductionFromanRFTestDevice-BradyEtAl.pdf

Significant...........that probably screwed up our reasoning for all these months. No significant thrust says there was thrust, it was just insignificant..........but there WAS thrust.

An unloaded cavity appears to be metastable. You have to give it a push in a particular direction. Look at the Shawyer Demonstrator, it goes both ways. The ones with dielectric, go one direction.

There's still some inconsistencies with this that need figured out.

(http://forum.nasaspaceflight.com/index.php?action=dlattach;topic=36313.0;attach=828264;image)

@Star-Drive, we need this to be addressed.

From talking with Mr. Shawyer, I get it why high Q=higher thrust. High Q also equals a narrow bandwidth cavity....aka easy to perturb. Design one with a bandwidth narrow enough to be perturbed by the redshift caused by Earth's 1g acceleration, you get unassisted levitation.
Title: Re: EM Drive Developments - related to space flight applications - Thread 2
Post by: PushHigher on 05/04/2015 03:04 pm
Quote
But this is NOT what was measured at NASA Eagleworks.  Nothing needed to be pushed and nothing was pushed at NASA Eagleworks.

We are on a rotating sphere, could that be the push?
Title: Re: EM Drive Developments - related to space flight applications - Thread 2
Post by: Rodal on 05/04/2015 03:11 pm
Quote
But this is NOT what was measured at NASA Eagleworks.  Nothing needed to be pushed and nothing was pushed at NASA Eagleworks.

We are on a rotating sphere, could that be the push?
That (rotation of the Earth) does not make any difference regarding a change from initial conditions for an unrestrained drive.  What Mulletron and TheTraveller are proposing is:

d2x/dt2 (t=0) = 0

d2x/dt2  (t>0&t<?) >0

Ditto for gravity, and any other field force, unless one postulates that you have the EM Drive restrained and suddenly you unrestrain it (easy to do for gravity, difficult to do for the Earth's rotation)  at t=0+, but this is not what is being reported in experiments.

_________
PS: Seismic noise is not a satisfactory explanation: it is random (would require a ratchet mechanism and a defense for what is the ratchet mechanism and the magnitude would still be random)
Title: Re: EM Drive Developments - related to space flight applications - Thread 2
Post by: Stormbringer on 05/04/2015 03:27 pm

Or something much more mundane:
http://usersguidetotheuniverse.com/?p=2865

A resonant cavity can ONLY support certain frequencies. If those frequencies change, say due to red or blue shift, they won't/can't resonate, and are lost. That resonant cavity that was once saturated, now has a "hole" which can be filled by more incoming radiation, said another way energy flowing back in. The transient Poynting vector. In layman's terms.

The missing Poynting vector is due to us never considering an accelerating cavity. Shawyer says it has to move first before you observe a force. He isn't just saying that. He probably observed that.

That sounds exactly like ME too. Same freaking thing. Two sides of the same coin.

ME=Classical description
EM=Quantum description

See that bottom pic. Those "dips" are the only frequencies that will exist within that range of my cavity. If they get shifted up or down (like by if I pick up the cavity and shake the crap out of it), they're history.

(http://usersguidetotheuniverse.com/wp-content/uploads/2013/03/box_photons_moving.png)

Going into the speculation side and maybe a silly error: this observation could also match with EW's potential observation of a warpdrive signature.

If I remember well, they have mentioned something called 'boost' that seems to be a speed multiplier.

What if we are actually seeing that, a initial speed from an imparted momentum, just slightly 'boosted'?
I think just recently (maybe within 15 to 20 pages ago someone said that Dr White considers the EM drive to be a case of warp drive because he thinks the underlying principles of operation are dipping into the same source. My memory is really vague but if I remember right you are not too far off the mark. at least on one branch of consideration here.
Title: Re: EM Drive Developments - related to space flight applications - Thread 2
Post by: Mulletron on 05/04/2015 03:29 pm
I think just recently (maybe within 15 05 20 pages ago Someone Said that Dr White considers the EM drive to be a case of warp drive because he thinks the underlying principles of operation are dipping into the same source. My memory is really vague but if I remember right you are not too far off the mark. at least on one branch of consideration here.

Yeah and we've established that Eagleworks methodology is driven by the Dr. White QVPT conjecture, so no wonder why the test protocol isn't matching what the inventor is telling us.

Ning Li all over again.

Read for yourself about what happened:
https://www.google.it/search?q=ning+li&ie=utf-8&oe=utf-8&gws_rd=cr&ei=0XIsVfriEojqaPP5gIAJ#q=%22ning+li%22+%22fell+apart%22

*No I'm not going to remove the Ning Li reference. We should not fall prey to lessons already learned. Gut check moments are important. Reporting the facts, even if they are unpopular is important.
Providing forceful backup is important.
https://www.google.com/search?q=forceful+backup&ie=utf-8&oe=utf-8

Quote
In order for an organization to be Operationally Disciplined (doing the right thing, the right way, every time to achieve Operational Excellence), it takes a commitment from all employees not only to themselves, but also to one another. The concept of Forceful Watch Team Backup is rooted in everyone’s understanding that they are part of something larger than themselves, everyone relies on one another and there is a level of seriousness to the jobs they do. With that as a foundation, employees are driven to back up one another to ensure that everyone is doing the right thing, the right way, every time. And if a fellow employee has overlooked something or is not behaving in an Operationally Disciplined manner, other employees have the courage to step in and help resolve the issue. Employees are actively looking for what might be wrong in each other’s areas and expect others to do the same in return. It is everyone’s responsibility to ensure the company succeeds-if one person fails, everyone fails.
Title: Re: EM Drive Developments - related to space flight applications - Thread 2
Post by: PaulF on 05/04/2015 03:56 pm
If a drive needs an impulse to get thrust production, how small would that impulse need to be to get things going? is there a threshold value beyond which a push will make a difference, or not? Maybe that could explain why it needs an impulse to start up, and that current theories pertaining random particle movement imply impulses that fall below the threshold value? Maybe someone can answer that question.

Furthermore, If it needs a push to put things in motion, wouldn't that action create the needed reference frame to explain this the classical way? And that before the push(with the EM drive activated) a new explanation is needed? (saying it this way because i cannot describe it).

Physical interaction may be a better term if that threshold value turns out to be 0.

I am truly sorry if this has all been discussed/dismissed before. delete this post if necessary.
Title: Re: EM Drive Developments - related to space flight applications - Thread 2
Post by: saucyjack on 05/04/2015 04:03 pm
Speaking of KM, is there anyone out there that has what it takes and is willing to volunteer and set up a wiki or something?
http://en.wikipedia.org/wiki/MediaWiki

I've set up a MediaWiki server before and it wasn't too bad. That is a good platform. I'm simply stretched too thin right now to try it again. My previous attempt at KM (just a simple Google Doc, which was pretty bad didn't catch on so I deleted it:
https://drive.google.com/folderview?id=0B4PCfHCM1KYoTXhSUTd5ZDN2WnM&usp=sharing

@Mulletron et al., I am happy to assist in this.  I agree, this forum is excellent for reviewing the latest contributions by all the excellent minds here, but newer folks are finding it difficult to get a handle on how we got here over the past hundred pages.  As a result some of the same questions are being re-asked repeatedly.  Ideally it might become the FAQ section that was discussed previously (which @Rodal noted some valid concerns regarding).  Worth a try, at least.

To this end, I have set up a MediaWiki server at http://emdrive.echothis.com and will start by the adding the relevant links in today.  However, I'm just a (non-practicing) mechanical engineer, not a physicist so my role in this would be limited to setting up the organization and linking to the relevant content already posted by people far more knowledgeable than I.  Anyone who has been tracking the forum and would like to join in this endeavor, welcome!
Title: Re: EM Drive Developments - related to space flight applications - Thread 2
Post by: D_Dom on 05/04/2015 04:07 pm

A resonant cavity can ONLY support certain frequencies. If those frequencies change, say due to red or blue shift, they won't/can't resonate, and are lost. That resonant cavity that was once saturated, now has a "hole" which can be filled by more incoming radiation, said another way energy flowing back in. The transient Poynting vector. In layman's terms.

The missing Poynting vector is due to us never considering an accelerating cavity. Shawyer says it has to move first before you observe a force. He isn't just saying that. He probably observed that.
...
See that bottom pic. Those "dips" are the only frequencies that will exist within that range of my cavity. If they get shifted up or down (like by if I pick up the cavity and shake the crap out of it), they're history.


Trying to understand the implication here. If your frustum were mounted on a shaker table can you speculate the most interesting excitation frequencies? Keeping in mind your test setup would a sub-woofer be adequate?
Title: Re: EM Drive Developments - related to space flight applications - Thread 2
Post by: TheTraveller on 05/04/2015 04:13 pm
If a cavity needs an impulse, how small would that impulse need to be to get things going? is there a threshold value beyond which a push will make a difference, or not? Maybe that could explain why it needs an impulse to start up, and that current theories pertaining random particle movement imply impulses that fall below the threshold value? Maybe someone can answer that question.

Furthermore, If it needs a push to put things in motion, wouldn't that action create the needed reference frame to explain this the classical way? And that before the push(with the EM drive activated) a new explanation is needed? (saying it this way because i cannot describe it).

I am truly sorry if this has all been discussed/dismissed before. delete this post if necessary.
My understanding, based on Shawyer data, is the EM Drive has 2 modes that will cause a external force, in opposite directions, to be generated. One he calls Motor mode and the other Generator mode.

If you push against the EM Drive such that Generator mode is activated, it will resist your push and not move. If you push it in the other direction, Motor mode is activated and it will move away from your push.

I then see any random event causing Generator to be activated resulting in no backward movement, while random events which activate Motor mode will move the EM Drive forward with no applied external force. This may be a small effect and use of an long term external force may cause higher acceleration rates.

In Generator mode, the backward push kinetic energy is converted into higher cavity energy and finally heat and in Motor mode the forward push drains cavity energy and converts it into forward kinetic energy, while drawing replacement from the microwave generator, which draws energy from the primary energy supply. Thus vehicle gained kinetic energy is that drawn from the primary energy supply minus losses. COE conserved.

Shawyer proposed Generator could be used to decelerate the vehicle at no energy cost to the primary energy source. Could be used to stop additional acceleration as the vehicle enters a gravity well.

This is my opinion based on what I read from what Shawyer has stated. Time will tell if both, one or the other or neither are correct.
Title: Re: EM Drive Developments - related to space flight applications - Thread 2
Post by: Rodal on 05/04/2015 04:15 pm
If a drive needs an impulse to get thrust production, how small would that impulse need to be to get things going? is there a threshold value beyond which a push will make a difference, or not? Maybe that could explain why it needs an impulse to start up, and that current theories pertaining random particle movement imply impulses that fall below the threshold value? Maybe someone can answer that question.

Furthermore, If it needs a push to put things in motion, wouldn't that action create the needed reference frame to explain this the classical way? And that before the push(with the EM drive activated) a new explanation is needed? (saying it this way because i cannot describe it).

Physical interaction may be a better term if that threshold value turns out to be 0.

I am truly sorry if this has all been discussed/dismissed before. delete this post if necessary.

I fully agree with you.  Neither the threshold impulse is defined, nor is it supported by anything.   :)
Title: Re: EM Drive Developments - related to space flight applications - Thread 2
Post by: TheTraveller on 05/04/2015 04:18 pm
If a drive needs an impulse to get thrust production, how small would that impulse need to be to get things going? is there a threshold value beyond which a push will make a difference, or not? Maybe that could explain why it needs an impulse to start up, and that current theories pertaining random particle movement imply impulses that fall below the threshold value? Maybe someone can answer that question.

Furthermore, If it needs a push to put things in motion, wouldn't that action create the needed reference frame to explain this the classical way? And that before the push(with the EM drive activated) a new explanation is needed? (saying it this way because i cannot describe it).

Physical interaction may be a better term if that threshold value turns out to be 0.

I am truly sorry if this has all been discussed/dismissed before. delete this post if necessary.

I fully agree with you.  Neither the threshold impulse is defined, nor is it supported by anything.   :)
How small a movement will it take for the resonate cavity energy waves to become unbalanced? 1um? Larger? Smaller? Function of wavelength versus phase distortion?

At 3.86 GHz, as used in the Flight Thruster, the full wave length is 77.9 mm.
Title: Re: EM Drive Developments - related to space flight applications - Thread 2
Post by: Rodal on 05/04/2015 04:25 pm
I had high hopes for this thread to keep on an analytical basis, away from opinions about "methodology driven R&D managers", and comparison with the misfortune of disassociated persons.

Wait, this happened before.

Perhaps this too shall pass?
Title: Re: EM Drive Developments - related to space flight applications - Thread 2
Post by: Mulletron on 05/04/2015 04:36 pm
If a cavity needs an impulse, how small would that impulse need to be to get things going? is there a threshold value beyond which a push will make a difference, or not? Maybe that could explain why it needs an impulse to start up, and that current theories pertaining random particle movement imply impulses that fall below the threshold value? Maybe someone can answer that question.

Furthermore, If it needs a push to put things in motion, wouldn't that action create the needed reference frame to explain this the classical way? And that before the push(with the EM drive activated) a new explanation is needed? (saying it this way because i cannot describe it).

I am truly sorry if this has all been discussed/dismissed before. delete this post if necessary.
My understanding, based on Shawyer data, is the EM Drive has 2 modes that will cause a external force, in opposite directions, to be generated. One he calls Motor mode and the other Generator mode.

If you push against the EM Drive such that Generator mode is activated, it will resist your push and not move. If you push it in the other direction, Motor mode is activated and it will move away from your push.

I then see any random event causing Generator to be activated resulting in no backward movement, while random events which activate Motor mode will move the EM Drive forward with no applied external force. This may be a small effect and use of an long term external force may cause higher acceleration rates.

In Generator mode, the backward push kinetic energy is converted into higher cavity energy and finally heat and in Motor mode the forward push drains cavity energy and converts it into forward kinetic energy, while drawing replacement from the microwave generator, which draws energy from the primary energy supply. Thus vehicle gained kinetic energy is that drawn from the primary energy supply minus losses. COE conserved.

Shawyer proposed Generator could be used to decelerate the vehicle at no energy cost to the primary energy source. Could be used to stop additional acceleration as the vehicle enters a gravity well.

This is my opinion based on what I read from what Shawyer has stated. Time will tell if both, one or the other or neither are correct.

That makes sense. I see we differ on a minor point too. If we can get confirmation from Mr. Shawyer on this point, it would do wonders for everybody.

Would @Chris Bergin be willing to set up a private thread just for talking with Mr. Shawyer, undisturbed by outside agitators?

http://emdrive.com/
Contact [email protected]
Title: Re: EM Drive Developments - related to space flight applications - Thread 2
Post by: Mulletron on 05/04/2015 04:41 pm
Speaking of KM, is there anyone out there that has what it takes and is willing to volunteer and set up a wiki or something?
http://en.wikipedia.org/wiki/MediaWiki

I've set up a MediaWiki server before and it wasn't too bad. That is a good platform. I'm simply stretched too thin right now to try it again. My previous attempt at KM (just a simple Google Doc, which was pretty bad didn't catch on so I deleted it:
https://drive.google.com/folderview?id=0B4PCfHCM1KYoTXhSUTd5ZDN2WnM&usp=sharing

@Mulletron et al., I am happy to assist in this.  I agree, this forum is excellent for reviewing the latest contributions by all the excellent minds here, but newer folks are finding it difficult to get a handle on how we got here over the past hundred pages.  As a result some of the same questions are being re-asked repeatedly.  Ideally it might become the FAQ section that was discussed previously (which @Rodal noted some valid concerns regarding).  Worth a try, at least.

To this end, I have set up a MediaWiki server at http://emdrive.echothis.com and will start by the adding the relevant links in today.  However, I'm just a (non-practicing) mechanical engineer, not a physicist so my role in this would be limited to setting up the organization and linking to the relevant content already posted by people far more knowledgeable than I.  Anyone who has been tracking the forum and would like to join in this endeavor, welcome!

Seriously thank you bro. That is some serious gettin' stuff done.
Title: Re: EM Drive Developments - related to space flight applications - Thread 2
Post by: Mulletron on 05/04/2015 04:44 pm

A resonant cavity can ONLY support certain frequencies. If those frequencies change, say due to red or blue shift, they won't/can't resonate, and are lost. That resonant cavity that was once saturated, now has a "hole" which can be filled by more incoming radiation, said another way energy flowing back in. The transient Poynting vector. In layman's terms.

The missing Poynting vector is due to us never considering an accelerating cavity. Shawyer says it has to move first before you observe a force. He isn't just saying that. He probably observed that.
...
See that bottom pic. Those "dips" are the only frequencies that will exist within that range of my cavity. If they get shifted up or down (like by if I pick up the cavity and shake the crap out of it), they're history.


Trying to understand the implication here. If your frustum were mounted on a shaker table can you speculate the most interesting excitation frequencies? Keeping in mind your test setup would a sub-woofer be adequate?

If it were on a shaker table, I wouldn't know what to expect. Guess that depends on how narrow the bandwidth of the cavity is. That is @Rodal quality math.
Title: Re: EM Drive Developments - related to space flight applications - Thread 2
Post by: lasoi on 05/04/2015 04:50 pm
Speaking of KM, is there anyone out there that has what it takes and is willing to volunteer and set up a wiki or something?
http://en.wikipedia.org/wiki/MediaWiki

I've set up a MediaWiki server before and it wasn't too bad. That is a good platform. I'm simply stretched too thin right now to try it again. My previous attempt at KM (just a simple Google Doc, which was pretty bad didn't catch on so I deleted it:
https://drive.google.com/folderview?id=0B4PCfHCM1KYoTXhSUTd5ZDN2WnM&usp=sharing

@Mulletron et al., I am happy to assist in this.  I agree, this forum is excellent for reviewing the latest contributions by all the excellent minds here, but newer folks are finding it difficult to get a handle on how we got here over the past hundred pages.  As a result some of the same questions are being re-asked repeatedly.  Ideally it might become the FAQ section that was discussed previously (which @Rodal noted some valid concerns regarding).  Worth a try, at least.

To this end, I have set up a MediaWiki server at http://emdrive.echothis.com and will start by the adding the relevant links in today.  However, I'm just a (non-practicing) mechanical engineer, not a physicist so my role in this would be limited to setting up the organization and linking to the relevant content already posted by people far more knowledgeable than I.  Anyone who has been tracking the forum and would like to join in this endeavor, welcome!

Seriously thank you bro. That is some serious gettin' stuff done.
A FAQ page would be super helpful and is sorely needed.  But it won't make much difference if it isn't easily accessible.  Ideally there'd be a link to it at the top of every page on this forum- or better yet, appended to every post that anybody makes.
Title: Re: EM Drive Developments - related to space flight applications - Thread 2
Post by: TheTraveller on 05/04/2015 04:51 pm

A resonant cavity can ONLY support certain frequencies. If those frequencies change, say due to red or blue shift, they won't/can't resonate, and are lost. That resonant cavity that was once saturated, now has a "hole" which can be filled by more incoming radiation, said another way energy flowing back in. The transient Poynting vector. In layman's terms.

The missing Poynting vector is due to us never considering an accelerating cavity. Shawyer says it has to move first before you observe a force. He isn't just saying that. He probably observed that.
...
See that bottom pic. Those "dips" are the only frequencies that will exist within that range of my cavity. If they get shifted up or down (like by if I pick up the cavity and shake the crap out of it), they're history.


Trying to understand the implication here. If your frustum were mounted on a shaker table can you speculate the most interesting excitation frequencies? Keeping in mind your test setup would a sub-woofer be adequate?

If it were on a shaker table, I wouldn't know what to expect. Guess that depends on how narrow the bandwidth of the cavity is. That is @Rodal quality math.
Chinese did report their cavity bandwidth data:
http://www.emdrive.com/yang-juan-paper-2012.pdf
Title: Re: EM Drive Developments - related to space flight applications - Thread 2
Post by: Chris Bergin on 05/04/2015 04:57 pm
Speaking of KM, is there anyone out there that has what it takes and is willing to volunteer and set up a wiki or something?
http://en.wikipedia.org/wiki/MediaWiki

I've set up a MediaWiki server before and it wasn't too bad. That is a good platform. I'm simply stretched too thin right now to try it again. My previous attempt at KM (just a simple Google Doc, which was pretty bad didn't catch on so I deleted it:
https://drive.google.com/folderview?id=0B4PCfHCM1KYoTXhSUTd5ZDN2WnM&usp=sharing

@Mulletron et al., I am happy to assist in this.  I agree, this forum is excellent for reviewing the latest contributions by all the excellent minds here, but newer folks are finding it difficult to get a handle on how we got here over the past hundred pages.  As a result some of the same questions are being re-asked repeatedly.  Ideally it might become the FAQ section that was discussed previously (which @Rodal noted some valid concerns regarding).  Worth a try, at least.

To this end, I have set up a MediaWiki server at http://emdrive.echothis.com and will start by the adding the relevant links in today.  However, I'm just a (non-practicing) mechanical engineer, not a physicist so my role in this would be limited to setting up the organization and linking to the relevant content already posted by people far more knowledgeable than I.  Anyone who has been tracking the forum and would like to join in this endeavor, welcome!

Welcome to the site's forum. That is a very good idea (as we can't really do a wiki style page here).

Top work, good new member :)



Would @Chris Bergin be willing to set up a private thread just for talking with Mr. Shawyer, undisturbed by outside agitators?

http://emdrive.com/
Contact [email protected]

We could create a standalone Q&A thread, like this one?

http://forum.nasaspaceflight.com/index.php?topic=37295.0

(Notice the format is different to a normal thread).
Title: Re: EM Drive Developments - related to space flight applications - Thread 2
Post by: Mulletron on 05/04/2015 05:20 pm
Speaking of KM, is there anyone out there that has what it takes and is willing to volunteer and set up a wiki or something?
http://en.wikipedia.org/wiki/MediaWiki

I've set up a MediaWiki server before and it wasn't too bad. That is a good platform. I'm simply stretched too thin right now to try it again. My previous attempt at KM (just a simple Google Doc, which was pretty bad didn't catch on so I deleted it:
https://drive.google.com/folderview?id=0B4PCfHCM1KYoTXhSUTd5ZDN2WnM&usp=sharing

@Mulletron et al., I am happy to assist in this.  I agree, this forum is excellent for reviewing the latest contributions by all the excellent minds here, but newer folks are finding it difficult to get a handle on how we got here over the past hundred pages.  As a result some of the same questions are being re-asked repeatedly.  Ideally it might become the FAQ section that was discussed previously (which @Rodal noted some valid concerns regarding).  Worth a try, at least.

To this end, I have set up a MediaWiki server at http://emdrive.echothis.com and will start by the adding the relevant links in today.  However, I'm just a (non-practicing) mechanical engineer, not a physicist so my role in this would be limited to setting up the organization and linking to the relevant content already posted by people far more knowledgeable than I.  Anyone who has been tracking the forum and would like to join in this endeavor, welcome!

Welcome to the site's forum. That is a very good idea (as we can't really do a wiki style page here).

Top work, good new member :)



Would @Chris Bergin be willing to set up a private thread just for talking with Mr. Shawyer, undisturbed by outside agitators?

http://emdrive.com/
Contact [email protected]

We could create a standalone Q&A thread, like this one?

http://forum.nasaspaceflight.com/index.php?topic=37295.0

(Notice the format is different to a normal thread).

Okay good, we're getting organized. That kind of venue is perfect. We have a place to collaborate and a KM solution. I am hopeful that Mr. Shawyer will come speak with us, and we will provide a professional venue for a productive Q&A.

Thank you @Chris Bergin.
Title: Re: EM Drive Developments - related to space flight applications - Thread 2
Post by: TheTraveller on 05/04/2015 05:24 pm
Would seem from Shawyers latest presentation, recently shared by Mulletron, that flat end plates are out and convex / concave end plates are the new standard to reduce cavity losses and boost cavity Q / thrust generation per kW of cavity stored microwave energy.

While Shawyer did share this cavity variation applied to a superconducting cavity, why it was used and what the curves are based on is new information. Would seem to make very good sense to incorporate it into non superconducting cavities.
Title: Re: EM Drive Developments - related to space flight applications - Thread 2
Post by: Mulletron on 05/04/2015 05:27 pm
Speaking of KM, is there anyone out there that has what it takes and is willing to volunteer and set up a wiki or something?
http://en.wikipedia.org/wiki/MediaWiki

I've set up a MediaWiki server before and it wasn't too bad. That is a good platform. I'm simply stretched too thin right now to try it again. My previous attempt at KM (just a simple Google Doc, which was pretty bad didn't catch on so I deleted it:
https://drive.google.com/folderview?id=0B4PCfHCM1KYoTXhSUTd5ZDN2WnM&usp=sharing

@Mulletron et al., I am happy to assist in this.  I agree, this forum is excellent for reviewing the latest contributions by all the excellent minds here, but newer folks are finding it difficult to get a handle on how we got here over the past hundred pages.  As a result some of the same questions are being re-asked repeatedly.  Ideally it might become the FAQ section that was discussed previously (which @Rodal noted some valid concerns regarding).  Worth a try, at least.

To this end, I have set up a MediaWiki server at http://emdrive.echothis.com and will start by the adding the relevant links in today.  However, I'm just a (non-practicing) mechanical engineer, not a physicist so my role in this would be limited to setting up the organization and linking to the relevant content already posted by people far more knowledgeable than I.  Anyone who has been tracking the forum and would like to join in this endeavor, welcome!

Seriously thank you bro. That is some serious gettin' stuff done.
A FAQ page would be super helpful and is sorely needed.  But it won't make much difference if it isn't easily accessible.  Ideally there'd be a link to it at the top of every page on this forum- or better yet, appended to every post that anybody makes.

The newly created Emdrive wiki http://emdrive.echothis.com/index.php/Main_Page can fulfill the FAQ requirement. Thanks to @saucyjack for your hard work! :)
Title: Re: EM Drive Developments - related to space flight applications - Thread 2
Post by: Rodal on 05/04/2015 05:27 pm
Would seem from Shawyers latest presentation, recently shared by Mulletron, that flat end plates are out and convex / concave end plates are the new standard to reduce cavity losses and boost cavity Q / thrust generation per kW of cavity stored microwave energy.

While Shawyer did share this cavity variation applied to a superconducting cavity, why it was used and what the curves are based on is new information. Would seem to make very good sense to incorporate it into non superconducting cavities.

NOT NEW information.

Discussed already much earlier in the thread.  I pointed out the reason.  Please take a look at the ends of the truncated cone in my exact solutions and also in Greg Egan's.

The reason for the spherical ends is because the standing waves in a truncated cone cavity are spherical waves.

So, that:

*the standing wave in a truncated cone are spherical waves has been known since the 1930's as per Shelkunoff's. 

*that Shawyer is using spherical ends for his superconducting EM Drive has been known and discussed in this thread multiple times since Oct 2014
Title: Re: EM Drive Developments - related to space flight applications - Thread 2
Post by: TheTraveller on 05/04/2015 05:45 pm
Would seem from Shawyers latest presentation, recently shared by Mulletron, that flat end plates are out and convex / concave end plates are the new standard to reduce cavity losses and boost cavity Q / thrust generation per kW of cavity stored microwave energy.

While Shawyer did share this cavity variation applied to a superconducting cavity, why it was used and what the curves are based on is new information. Would seem to make very good sense to incorporate it into non superconducting cavities.

NOT NEW information at all.

Discussed already much earlier in the thread.  I pointed out the reason.  Please take a look at the ends of the truncated cone in my exact solutions and also in Greg Egan's.

The reason for the spherical ends is because the standing waves in a truncated cone cavity are spherical waves (this is known since the 1930's as per Shelkunoff's analysis)
Is it not new information from Shawyer?

I plan to build and test a Shawyer EM Drive with his new convex / concave end plates. Will take my design lead 100% from Shawyer at 1st as he is the only source that has built multiple EM Drives, measured generated significant force / thrust and shared enough real / practical data to allow replication of his many years of blood, sweat and tears.

The theory talk here is good background but not focused enough to be something an engineer can use to built a successful working EM Drive, at least not in the 1st instance. I believe in not reinventing the wheel, until I have several working wheels to do further development from.

I also plan to duplicate Shawyers vertical force measurement system as used in his Flight Thruster qualification. To me that is a very KISS solution.

I'm not doing this for any monetary gain. My plans will be open sourced. My gut just wants to know how this works. Not in a theory sense but in a practical now do you use this device to achieve a desired end goal as it apparently has very unusual operational modes.

Would welcome any suggestions as I have read others here have started down this pathway.
Title: Re: EM Drive Developments - related to space flight applications - Thread 2
Post by: TheTraveller on 05/04/2015 05:50 pm
Would seem from Shawyers latest presentation, recently shared by Mulletron, that flat end plates are out and convex / concave end plates are the new standard to reduce cavity losses and boost cavity Q / thrust generation per kW of cavity stored microwave energy.

While Shawyer did share this cavity variation applied to a superconducting cavity, why it was used and what the curves are based on is new information. Would seem to make very good sense to incorporate it into non superconducting cavities.

NOT NEW information at all.

Discussed already much earlier in the thread.  I pointed out the reason.  Please take a look at the ends of the truncated cone in my exact solutions and also in Greg Egan's.

The reason for the spherical ends is because the standing waves in a truncated cone cavity are spherical waves (this is known since the 1930's as per Shelkunoff's analysis)
Is it not new information from Shawyer?

I plan to build and test a Shawyer EM Drive with his new convex / concave end plates. Will take my design lead 100% from Shawyer at 1st as he is the only source that has built multiple EM Drives, measured generated significant force / thrust and shared enough real / practical data to allow replication of his many years of blood, sweat and tears.

The theory talk here is good background but not focused enough to be something an engineer can use to built a successful working EM Drive, at least not in the 1st instance. I believe in not reinventing the wheel, until I have several working wheels to do further development from.

I also plan to duplicate Shawyers vertical force measurement system as used in his Flight Thruster qualification. To me that is a very KISS solution.

Would welcome any suggestions as I have read others here have started down this pathway.

No, the information you posted from Shawyer was not new information on this thread.

The information you posted on Shawyer's including Shawyer's presentation (which dates 6 months ago) has been discussed multiple times in this thread.

One of the discussions was with an European Architect/Designer.  We also discussed paraboloid cavities, for example.
We are talking about the same presentation that Mullerton just received from Shawyer and posted a day ago? The attached information was known 6 months ago?
Title: Re: EM Drive Developments - related to space flight applications - Thread 2
Post by: TheTraveller on 05/04/2015 05:57 pm
...
We are talking about the same presentation that Mullerton just received from Shawyer and posted a day ago? The attached information was known 6 months ago?
yes.
Link?
Title: Re: EM Drive Developments - related to space flight applications - Thread 2
Post by: Mulletron on 05/04/2015 06:11 pm
Would seem from Shawyers latest presentation, recently shared by Mulletron, that flat end plates are out and convex / concave end plates are the new standard to reduce cavity losses and boost cavity Q / thrust generation per kW of cavity stored microwave energy.

While Shawyer did share this cavity variation applied to a superconducting cavity, why it was used and what the curves are based on is new information. Would seem to make very good sense to incorporate it into non superconducting cavities.

NOT NEW information at all.

Discussed already much earlier in the thread.  I pointed out the reason.  Please take a look at the ends of the truncated cone in my exact solutions and also in Greg Egan's.

The reason for the spherical ends is because the standing waves in a truncated cone cavity are spherical waves (this is known since the 1930's as per Shelkunoff's analysis)
Is it not new information from Shawyer?

I plan to build and test a Shawyer EM Drive with his new convex / concave end plates. Will take my design lead 100% from Shawyer at 1st as he is the only source that has built multiple EM Drives, measured generated significant force / thrust and shared enough real / practical data to allow replication of his many years of blood, sweat and tears.

The theory talk here is good background but not focused enough to be something an engineer can use to built a successful working EM Drive, at least not in the 1st instance. I believe in not reinventing the wheel, until I have several working wheels to do further development from.

I also plan to duplicate Shawyers vertical force measurement system as used in his Flight Thruster qualification. To me that is a very KISS solution.

Would welcome any suggestions as I have read others here have started down this pathway.

No, the information you posted from Shawyer was not new information on this thread.

The information you posted on Shawyer's including Shawyer's presentation (which dates 6 months ago) has been discussed multiple times in this thread.

One of the discussions was with an European Architect/Designer.  We also discussed paraboloid cavities, for example.
We are talking about the same presentation that Mullerton just received from Shawyer and posted a day ago? The attached information was known 6 months ago?

Never discussed before:
Quote
The cavity comprises a small convex end plate, a
truncated conical side wall section, and a large
convex end plate. The end plate Radii R1 and R2
are selected such that R2-R1 = L1 where L1 is the
length of the side wall.
This geometry ensures that
the EM wavefront propagates between the end plates
with every point on the wavefront travelling along a
radius line of length L1, centred at point O. This
constant path length over the wavefront ensures that
phase distortion over the very large number of
reflections within a high Q cavity, is minimised, and
the value of Q that is achieved in practice
approaches the theoretical maximum. Note that this
configuration ensures that there is no orthogonal
component of the guide velocity reflected from the
side wall, thus ensuring a zero side wall force
component in the axial plane.

Source IAC-14 paper:
http://tinyurl.com/ofl4527
Title: Re: EM Drive Developments - related to space flight applications - Thread 2
Post by: Rodal on 05/04/2015 06:30 pm
Would seem from Shawyers latest presentation, recently shared by Mulletron, that flat end plates are out and convex / concave end plates are the new standard to reduce cavity losses and boost cavity Q / thrust generation per kW of cavity stored microwave energy.

While Shawyer did share this cavity variation applied to a superconducting cavity, why it was used and what the curves are based on is new information. Would seem to make very good sense to incorporate it into non superconducting cavities.

NOT NEW information at all.

Discussed already much earlier in the thread.  I pointed out the reason.  Please take a look at the ends of the truncated cone in my exact solutions and also in Greg Egan's.

The reason for the spherical ends is because the standing waves in a truncated cone cavity are spherical waves (this is known since the 1930's as per Shelkunoff's analysis)
Is it not new information from Shawyer?

I plan to build and test a Shawyer EM Drive with his new convex / concave end plates. Will take my design lead 100% from Shawyer at 1st as he is the only source that has built multiple EM Drives, measured generated significant force / thrust and shared enough real / practical data to allow replication of his many years of blood, sweat and tears.

The theory talk here is good background but not focused enough to be something an engineer can use to built a successful working EM Drive, at least not in the 1st instance. I believe in not reinventing the wheel, until I have several working wheels to do further development from.

I also plan to duplicate Shawyers vertical force measurement system as used in his Flight Thruster qualification. To me that is a very KISS solution.

Would welcome any suggestions as I have read others here have started down this pathway.

No, the information you posted from Shawyer was not new information on this thread.

The information you posted on Shawyer's including Shawyer's presentation (which dates 6 months ago) has been discussed multiple times in this thread.

One of the discussions was with an European Architect/Designer.  We also discussed paraboloid cavities, for example.
We are talking about the same presentation that Mullerton just received from Shawyer and posted a day ago? The attached information was known 6 months ago?

Never discussed before:
Quote
The cavity comprises a small convex end plate, a
truncated conical side wall section, and a large
convex end plate. The end plate Radii R1 and R2
are selected such that R2-R1 = L1 where L1 is the
length of the side wall.
This geometry ensures that
the EM wavefront propagates between the end plates
with every point on the wavefront travelling along a
radius line of length L1, centred at point O. This
constant path length over the wavefront ensures that
phase distortion over the very large number of
reflections within a high Q cavity, is minimised, and
the value of Q that is achieved in practice
approaches the theoretical maximum. Note that this
configuration ensures that there is no orthogonal
component of the guide velocity reflected from the
side wall, thus ensuring a zero side wall force
component in the axial plane.

Source IAC-14 paper:
http://tinyurl.com/ofl4527


The statement that R2-R1=L1 is not new information, it is a definition, a necessary tautology.

The length L1 must be equal to R2-R1, so no new information of any kind is given.



The dimensionless ratios for Shawyer's superconducting drive were estimated, for example:

http://forum.nasaspaceflight.com/index.php?topic=36313.msg1335023#msg1335023

Including the most important parameter: the cone angle

It would be appreciated if Roger Shawyer could provide the actual dimensions for R1, R2 and L1 for the superconducting design, since while the dimensionless ratios can be accurately estimated from a diagram, the actual dimensions cannot (as known from painstaking efforts by aero,  Lasoi, Flyby, Fornaro and other people in this thread).

@aero reports in previous posts that he asked Shawyer for the dimensions of Shawyer's EM Drives used in his reported experiments and received the answer that the small base was chosen on the basis of the cutoff frequency (and no further ado  :( ).  Therefore dimensions for the Flight Thruster, Experimental, Demonstrator and other Shawyer's experiments would be helpful to @notsosureofit, @aero, McCulloch and others in order to compare with the predicting formulas.

_____________

PS: Thanks once again to Paul March for patiently and willingly providing all geometrical data he was asked for regarding the NASA experiments.  We are lacking the actual geometrical dimensions for Shawyer's experiments and for the Chinese experiments.
Title: Re: EM Drive Developments - related to space flight applications - Thread 2
Post by: zellerium on 05/04/2015 07:05 pm
I have not yet spent enough time working with the proposed models, thus I'll stay out of the theory debate.

Instead, I am focusing on the difference in parameters of each experiment--what does the thrust depend on? EW saw a clear dependence on a dielectric, but Shawyer, NWPU and Fetta observed high efficiencies without one. But why does Shawyer mention the "dielectric-filled section" in one of his papers and not reference it in later experiments? http://www.emdrive.com/theorypaper9-4.pdf

Has NWPU tried their magnetron cavity using a dielectric? I believe I read a comment referencing a 2014 paper by Yang but I have not been able to find it.
Did EW test without a dielectric in a vacuum?

We don't have the money to build a sensitive torsion pendulum, thus my team is building a teeter-totter. It seems any spurious effects present with a teeter-totter would also be present on a Cavendish balance, but correct me if I'm wrong. I admit connection to an external power supply is going to be an issue, but most likely a consistent issue. High power batteries are not in our budget at the moment.

Instead of eliminating all extraneous possibilities, we plan to change the parameters and observe the change in thrust. We start by changing the dielectric parameters: 4 inches thick to .5 inch thick discs. Next we change dielectric location, and maybe even shape while we're at it. We can also test upward and downward orientations to observe a change due to the gravity vector. I would be very surprised if we saw identical thrust from all of these setups, hopefully we can single out the best and then figure out why it is the best.

We have designed a cylindrical cavity to simplify the problem. Solutions to resonant freq are much easier and more intuitive in a cylinder, and hitting resonance (when we adjust the dielectric) with a constant frequency (from a microwave oven magnetron) will be possible by adjusting the height of the cylinder. We will know this week whether we will receive funding for a variable frequency/power setup which will make things much easier as far as resonance goes. But I am worried that small powers (50 W) won't produce enough thrust to be seen on the teeter-totter, and a high power amp is too expensive.

   
I think this entire discussion should be moved to reddit.com/r/emdrive. The comment structure will allow the most informed individuals in this thread to quickly rise to the top. Replies to certain comments will be separated and various topics can be explored in separate threads. Just a more efficient system in my opinion...


Title: Re: EM Drive Developments - related to space flight applications - Thread 2
Post by: zen-in on 05/04/2015 07:21 pm


Chinese did report their cavity bandwidth data:
http://www.emdrive.com/yang-juan-paper-2012.pdf

Interesting.   Earlier in the first thread the discussion centered around cavity Q and the need for a high Q.   This cavity only has a Q = 1531.
Title: Re: EM Drive Developments - related to space flight applications - Thread 2
Post by: zellerium on 05/04/2015 07:41 pm


Chinese did report their cavity bandwidth data:
http://www.emdrive.com/yang-juan-paper-2012.pdf

Interesting.   Earlier in the first thread the discussion centered around cavity Q and the need for a high Q.   This cavity only has a Q = 1531.

The 2013 paper from NWPU: "It was found that the thruster
cavity made by copper and resonating on the equivalent TE011
mode has a quality factor 320400 and generates total net EM
thrust 411 mN for 1000 W 2.45 GHz incident microwave." 
http://iopscience.iop.org/1674-1056/22/5/050301
Title: Re: EM Drive Developments - related to space flight applications - Thread 2
Post by: Rodal on 05/04/2015 07:46 pm


Chinese did report their cavity bandwidth data:
http://www.emdrive.com/yang-juan-paper-2012.pdf

Interesting.   Earlier in the first thread the discussion centered around cavity Q and the need for a high Q.   This cavity only has a Q = 1531.

The 2013 paper from NWPU: "It was found that the thruster
cavity made by copper and resonating on the equivalent TE011
mode has a quality factor 320400 and generates total net EM
thrust 411 mN for 1000 W 2.45 GHz incident microwave." 
http://iopscience.iop.org/1674-1056/22/5/050301
The Q's reported in the tables 1 and 2 of http://www.emdrive.com/NWPU2010paper.pdf  and in the above quote are much higher than the calculated Q based on the bandwidth. 

Zen-In's calculated Q from the bandwidth is correct: Q = f/bandwidthf (by definition)

Can anybody make sense of the discrepancy?
Title: Re: EM Drive Developments - related to space flight applications - Thread 2
Post by: Rodal on 05/04/2015 07:56 pm
I repost here this post which may be of interest to people here as well:

So this made me think about the time I took a 3 port circulator apart. For those that are not familiar with circulators, the work like this:
Put power in port 1 and it comes out port 2, put it in 2 and it comes out 3, put it in 3 and it comes out 1. All with out much loss. But if you try to go backwards, say 3 to 2,  you loose 99% of the power.

Cool little device. So when I take it apart all it is is a flat triangle of copper, 2 triangle shaped pieces of ferrite, and a magnet.

If you don't know the math behind it, is looks at first blush as "silly" as the emdrive. No way could it do that. But it does. This thing may well work, we just don't know the math.

I'm an Engineer and I've studied the Polarizable Vacuum Model of General Relativity. What it would say is the following;

As a waveguide, the group velocity is something like;

v_g = c x sqrt(1 - (c/2d*f)^2)

Where, c is the usual speed of light, d is the diameter of the cylinder, and f is the frequency of the microwave excitation.

c/2d = fc,  is the Low cut-off frequency of the waveguide.

The refractive index depends on the Low cut-off frequency as a function of the diameter,

K = 1/sqrt( 1 - (fc/f)^2)

For f >> fc, K~1. But for frequencies in the band fc1 < f <~ fc2, K is much larger.

There is a strong gradient in the refractive index from one end of the cone to the other. This "mimics" gravity, as interpreted in the PV Model.

Therefore, we can assume there is a "gravitational" gradient in the microwave band refractive index, along the length of the cone. At one end they have diameter d1, and at the other end they have diameter d2, and d1 > d2. Below fc1, the mode frequencies exponentially decay to zero. Just like the Casimir effect.

Here is how it conserves momentum;

In the PV Model, momentum transforms as,

p => p*sqrt(K)

In a resonant cavity, p is the SUM of all the photons “in phase", minus the losses of the cavity. 

However, as photons “fall” from the large end toward the small end, they gain momentum, which is passed on to the cone when they are reflected from the small end. The photon then loses momentum as it travels back to the large end, where it imparts “less” momentum to the large end. The result is a NET propulsion in the direction of the small end. In other words, the photons are blue-shifted falling forward, and red-shifted going backwards, due to the gradient in the refractive index. It is literally gravitational red & blue shift, according to the PV Model.

The interesting thing is, the refractive index in the waveguide does not depend on the power of the microwaves, or the energy density. It is simply a matter of the geometry and frequency band relative to the cut-off. What matters more, is having enough resonant momentum stored to make the effect noticeable.

That’s IMHO as an engineer of course. Any comments?

See PV Model: https://www.researchgate.net/publication/223130116_Advanced_Space
_Propulsion_Based_on_Vacuum_%28Spacetime_Metric%29_Engineering

Todd D.

In what sense does this conserve momentum?

Treat the device as a black box. I don't know or care what is happening inside it. At time T0 it has no momentum. Turn it on and let it accelerate so that it has some velocity and so momentum at time T1. Unless you can point to something with the same amount of momentum going in the other direction then by definition you have violated conservation of momentum. What happens inside the box simple does not affect that fact.

When you drop an object and it falls to the ground. Relative to you, it gained momentum from the gravitational field. It did not expel any propellant to fall. The gravitational field is simply a gradient in the refractive index of the vacuum surrounding the Earth. If you can explain conservation of momentum for falling objects in a gravitational field, then you have your answer.

(Edit) In other words, if the cavity were not tapered, then you have equal momentum inside traveling left and right. At T0, it will go nowhere. However, because it is tapered such that you have a gradient in the refractive index, then "just like gravity", photons will be blue shifted moving into higher K, and red shifted moving into lower K, because momentum,

p => p*sqrt(K)

THIS is a violation of conservation of momentum. Therefore, the cavity must move to conserve momentum, as it tries to establish equilibrium with it's own internal stress.


Regards,
Todd D.

What would be the equation for the acceleration or the force, given the geometrical dimensions of the truncated cone, the Q, the input power, the frequency, and any other variables? Do you have a closed-form solution that could be compared to actual experimental results and also compared with the equation of Shawyer, and also to the equation of McCulloch ?

Regards,

JR

Not yet... I'm just now coming to grips with this myself. My light-bulb went off when I realized if the frequency of the microwaves is very close to the cut-off frequencies, then the speed of light will have a very large gradient inside the Frustum. Relative to the "traveling" waves (photons) attempting to move at the speed of light from end to end. When they approach the small end, their wavelength is squeezed by the reduced group velocity. Momentum depends on wavelength;

p = h/lambda

wavelength depends on velocity, and v_g is a variable inside the frustum.

That is where the momentum is coming from. Inside the Frustum, relative to the traveling waves you have an accelerated reference frame, into which you are injecting photons that are affected by this manufactured "gravitational" field, that must be compensated for by moving the Frustum.

I'll see what I can come up with for a formal equation, but I've got a day job. As for @ppnl, you will never get a Newtonian-type COM equation out of this. The two frames are the Frustum, and the frame of the moving photons inside it. The acceleration is caused by the geometry of the waveguide or a variable refractive index, i.e. the GR or PV Interpretation lead to the same result.

Todd D.
Title: Re: EM Drive Developments - related to space flight applications - Thread 2
Post by: deltaMass on 05/04/2015 08:09 pm
Although most of what's being discussed here concerns the static force produced by an EmDrive, I would like to look ahead to the dynamics in free space. In this regard, and needing no recourse to relativity for low velocities, all types of propellantless propulsion device are created equal; i.e. we model them with constant input power which results in a constant thrust, which in turn produces a constant acceleration. A little high school physics and a dash of algebra (see below) should suffice to convince you that, for any and all such devices, there exists a break-even velocity, above which more energy has been produced than has been consumed. As the device continues to further accelerate, a continuous source of free power is available. Thus we have not only perpetual motion, but free energy to boot.

It is worthwhile to quantify the value of this break-even velocity, which turns out to be a very simple expression. The relevant equations we use (symbols having their usual meaning) are:
k := F/P in Newton/Watt
v = a t = (F/m) t
Ein = P t
Eout = 0.5 m v2

From the first 2 equations we get
t = (m v) / (k P)

At breakeven, Ein= Eout and v := v0
or
2 P m v0 / (k P) = mv02
so
v0 = 2/k

Armed with this expression, the performance of any propellantless propulsion device may be examined, if its 'k' value is known.
Title: Re: EM Drive Developments - related to space flight applications - Thread 2
Post by: UneducatedNitwit on 05/04/2015 08:11 pm
Would @Chris Bergin be willing to set up a private thread just for talking with Mr. Shawyer, undisturbed by outside agitators?

We could create a standalone Q&A thread, like this one?

http://forum.nasaspaceflight.com/index.php?topic=37295.0

(Notice the format is different to a normal thread).

Please do this.  Although I haven't seen the moderated posts, I realize from what @Rodal has said recently that perhaps not everyone who came here from forums and link aggregators has been polite or even spent very long reading your discussion.  That's incredibly disappointing, but I guess from what I have seen on Reddit.com/r/physics it should be expected every time there is a new development.  There are very many people who are angered by the idea of taking this experiment, your discussion, and the credentials of the people involved here seriously.  They don't realize that this discussion has already covered all the bases.

It took me nearly a week to skim through the entirety of both threads.  You can't expect most people coming from other websites to dedicate their time that way before posting.  However, I think we should be given the opportunity to read what is written without participating unless we have something of value to say.

Title: Re: EM Drive Developments - related to space flight applications - Thread 2
Post by: Flyby on 05/04/2015 08:17 pm
Did anyone notice the appearance of piezoelectric elements in the updated 2nd generation drawing?
Apparently, a computerized system to adjust the walls of the supercooled frustum, in order to compensate for the Doppler effect...

I thought that is new, compared to previous drawings of EMdrive2.0 ?

When reading that new text i do start to wonder if this is a description of an already build item, or a description of an item to build?
Title: Re: EM Drive Developments - related to space flight applications - Thread 2
Post by: MazonDel on 05/04/2015 08:24 pm
Speaking of KM, is there anyone out there that has what it takes and is willing to volunteer and set up a wiki or something?
http://en.wikipedia.org/wiki/MediaWiki

I've set up a MediaWiki server before and it wasn't too bad. That is a good platform. I'm simply stretched too thin right now to try it again. My previous attempt at KM (just a simple Google Doc, which was pretty bad didn't catch on so I deleted it:
https://drive.google.com/folderview?id=0B4PCfHCM1KYoTXhSUTd5ZDN2WnM&usp=sharing

@Mulletron et al., I am happy to assist in this.  I agree, this forum is excellent for reviewing the latest contributions by all the excellent minds here, but newer folks are finding it difficult to get a handle on how we got here over the past hundred pages.  As a result some of the same questions are being re-asked repeatedly.  Ideally it might become the FAQ section that was discussed previously (which @Rodal noted some valid concerns regarding).  Worth a try, at least.

To this end, I have set up a MediaWiki server at http://emdrive.echothis.com and will start by the adding the relevant links in today.  However, I'm just a (non-practicing) mechanical engineer, not a physicist so my role in this would be limited to setting up the organization and linking to the relevant content already posted by people far more knowledgeable than I.  Anyone who has been tracking the forum and would like to join in this endeavor, welcome!

I would be most happy to join this effort!

Quite amusingly, I only ever look at updates on this thread during the week (worktime simulations are...boring) and this weekend I had decided that what seemed like the best thing I could do to help out the efforts here (since I'm not particularly knowledgeable with this sort of physics) would be to set up the wiki. When I showed up this morning I saw that I had a rather astounding 13+ pages of forum thread to catch up on! (Usually it seems to be about 4-5).

So when I read Mulletron's suggestion my thoughts were "Ah ha! I'm on the case! But I should finish catching up first." and then I saw you had started one up.

My provisional plan had been to slowly work through from page 1 again creating topics for things as they came up and beginning to organize them as the groupings became obvious.

A few groupings I had identified earlier:
Possible Error Sources of Thrust: Things like thermal buckling, atmospheric effects, etc would get pages of their own and this grouping would just allow for easy perusing of these topics. It might be wise to organize them into some sort of 'Busted', 'Under Investigation', 'Other' groupings. Of course new information can elevate one of the topics out of the busted area or descend it as necessary.

Current Theories: Relatively simple, listing out Shawyer's, White's, the Chinese Teams, Mulletron's, etc theories on what is going on.

Thrust Modes and Results: This one probably just ends up being a table of its own, but is probably particularly specific to the Eaglework's team.

Status of DIY Efforts: Lets Mulletron, Notsosureofit, DIYFan, and others that are replicating this provide easy to reference status updates and other things (future plans, etc).

What I was envisioning was to attempt to use the wiki-source system to provide pointers from information to the forum posts concerning them.

Thoughts?
Title: Re: EM Drive Developments - related to space flight applications - Thread 2
Post by: Notsosureofit on 05/04/2015 08:33 pm
I repost here this post which may be of interest to people here as well:

So this made me think about the time I took a 3 port circulator apart. For those that are not familiar with circulators, the work like this:
Put power in port 1 and it comes out port 2, put it in 2 and it comes out 3, put it in 3 and it comes out 1. All with out much loss. But if you try to go backwards, say 3 to 2,  you loose 99% of the power.

Cool little device. So when I take it apart all it is is a flat triangle of copper, 2 triangle shaped pieces of ferrite, and a magnet.

If you don't know the math behind it, is looks at first blush as "silly" as the emdrive. No way could it do that. But it does. This thing may well work, we just don't know the math.

I'm an Engineer and I've studied the Polarizable Vacuum Model of General Relativity. What it would say is the following;

As a waveguide, the group velocity is something like;

v_g = c x sqrt(1 - (c/2d*f)^2)

Where, c is the usual speed of light, d is the diameter of the cylinder, and f is the frequency of the microwave excitation.

c/2d = fc,  is the Low cut-off frequency of the waveguide.

The refractive index depends on the Low cut-off frequency as a function of the diameter,

K = 1/sqrt( 1 - (fc/f)^2)

For f >> fc, K~1. But for frequencies in the band fc1 < f <~ fc2, K is much larger.

There is a strong gradient in the refractive index from one end of the cone to the other. This "mimics" gravity, as interpreted in the PV Model.

Therefore, we can assume there is a "gravitational" gradient in the microwave band refractive index, along the length of the cone. At one end they have diameter d1, and at the other end they have diameter d2, and d1 > d2. Below fc1, the mode frequencies exponentially decay to zero. Just like the Casimir effect.

Here is how it conserves momentum;

In the PV Model, momentum transforms as,

p => p*sqrt(K)

In a resonant cavity, p is the SUM of all the photons “in phase", minus the losses of the cavity. 

However, as photons “fall” from the large end toward the small end, they gain momentum, which is passed on to the cone when they are reflected from the small end. The photon then loses momentum as it travels back to the large end, where it imparts “less” momentum to the large end. The result is a NET propulsion in the direction of the small end. In other words, the photons are blue-shifted falling forward, and red-shifted going backwards, due to the gradient in the refractive index. It is literally gravitational red & blue shift, according to the PV Model.

The interesting thing is, the refractive index in the waveguide does not depend on the power of the microwaves, or the energy density. It is simply a matter of the geometry and frequency band relative to the cut-off. What matters more, is having enough resonant momentum stored to make the effect noticeable.

That’s IMHO as an engineer of course. Any comments?

See PV Model: https://www.researchgate.net/publication/223130116_Advanced_Space
_Propulsion_Based_on_Vacuum_%28Spacetime_Metric%29_Engineering

Todd D.

In what sense does this conserve momentum?

Treat the device as a black box. I don't know or care what is happening inside it. At time T0 it has no momentum. Turn it on and let it accelerate so that it has some velocity and so momentum at time T1. Unless you can point to something with the same amount of momentum going in the other direction then by definition you have violated conservation of momentum. What happens inside the box simple does not affect that fact.

When you drop an object and it falls to the ground. Relative to you, it gained momentum from the gravitational field. It did not expel any propellant to fall. The gravitational field is simply a gradient in the refractive index of the vacuum surrounding the Earth. If you can explain conservation of momentum for falling objects in a gravitational field, then you have your answer.

(Edit) In other words, if the cavity were not tapered, then you have equal momentum inside traveling left and right. At T0, it will go nowhere. However, because it is tapered such that you have a gradient in the refractive index, then "just like gravity", photons will be blue shifted moving into higher K, and red shifted moving into lower K, because momentum,

p => p*sqrt(K)

THIS is a violation of conservation of momentum. Therefore, the cavity must move to conserve momentum, as it tries to establish equilibrium with it's own internal stress.


Regards,
Todd D.

What would be the equation for the acceleration or the force, given the geometrical dimensions of the truncated cone, the Q, the input power, the frequency, and any other variables? Do you have a closed-form solution that could be compared to actual experimental results and also compared with the equation of Shawyer, and also to the equation of McCulloch ?

Regards,

JR

Not yet... I'm just now coming to grips with this myself. My light-bulb went off when I realized if the frequency of the microwaves is very close to the cut-off frequencies, then the speed of light will have a very large gradient inside the Frustum. Relative to the "traveling" waves (photons) attempting to move at the speed of light from end to end. When they approach the small end, their wavelength is squeezed by the reduced group velocity. Momentum depends on wavelength;

p = h/lambda

wavelength depends on velocity, and v_g is a variable inside the frustum.

That is where the momentum is coming from. Inside the Frustum, relative to the traveling waves you have an accelerated reference frame, into which you are injecting photons that are affected by this manufactured "gravitational" field, that must be compensated for by moving the Frustum.

I'll see what I can come up with for a formal equation, but I've got a day job. As for @ppnl, you will never get a Newtonian-type COM equation out of this. The two frames are the Frustum, and the frame of the moving photons inside it. The acceleration is caused by the geometry of the waveguide or a variable refractive index, i.e. the GR or PV Interpretation lead to the same result.

Todd D.

Yep, right on target.  Accelerated frames:  GR likes to use time dilation, PV likes variable speed of light (ie refractive index)  They both should be able to get the formula.  The key point is that force needs accelerated frames.  Inertial frames won't do the job.  Also the force will diminish w/ the actual acceleration and get to a balance depending on the total mass of the system compared to the "weight" of the photons.
Title: Re: EM Drive Developments - related to space flight applications - Thread 2
Post by: Rodal on 05/04/2015 08:40 pm
....
We have designed a cylindrical cavity to simplify the problem. Solutions to resonant freq are much easier and more intuitive in a cylinder, and hitting resonance (when we adjust the dielectric) with a constant frequency (from a microwave oven magnetron) will be possible by adjusting the height of the cylinder. ....
But a cylindrical cavity displays no attenuation and no focusing in the axial direction (unlike the truncated cone).  The experimenters in the US, UK and China are using truncated cones. 

If you obtain no net thrust for the cylindrical cavity, will that have been worth the effort you went through (instead of testing a truncated cone, preferably with the same geometry dimensions as NASA's or one of Shaywer's ? )

Concerning obtaining the frequency and mode shape, does your University give you access to a Finite Element analysis package like COMSOL, or ANSYS Multiphysics, etc. so that you could then obtain a numerical solution for any arbitrary geometry and inserted dielectric ?
Title: Re: EM Drive Developments - related to space flight applications - Thread 2
Post by: zellerium on 05/04/2015 09:33 pm
....
We have designed a cylindrical cavity to simplify the problem. Solutions to resonant freq are much easier and more intuitive in a cylinder, and hitting resonance (when we adjust the dielectric) with a constant frequency (from a microwave oven magnetron) will be possible by adjusting the height of the cylinder. ....
But a cylindrical cavity displays no attenuation and no focusing in the axial direction (unlike the truncated cone).  The experimenters in the US, UK and China are using truncated cones. 

If you obtain no net thrust for the cylindrical cavity, will that have been worth the effort you went through (instead of testing a truncated cone, preferably with the same geometry dimensions as NASA's or one of Shaywer's ? )

Concerning obtaining the frequency and mode shape, does your University give you access to a Finite Element analysis package like COMSOL, or ANSYS Multiphysics, etc. so that you could then obtain a numerical solution for any arbitrary geometry and inserted dielectric ?

Dr. Rodal:

I was under the impression that a symmetric shape should produce a net thrust, based on the null Cannae test article at EW. The only asymmetry there was the dielectric slug in the end, correct?

I don't believe we have access to COMSOL, but I will look into another method.

We have yet to purchase materials and may be able to test a truncated shape as well as a symmetric shape with the same copper tube and end plates.
Title: Re: EM Drive Developments - related to space flight applications - Thread 2
Post by: MazonDel on 05/04/2015 09:37 pm
Zellerium, I am uncertain about if your University allows for this sort of thing or not, but for the purposes of the nascent Wiki under development, would you be willing to provide (later once you have them) some Bills of Materials as well as instructions documenting how you assembled and set up your rig?

I imagine that if you were to provide almost an Instructables-level documentation on how your team does what it does, that it would also better allow for people here to make suggestions on possible tests to look into or to help overcome problems you may run into.
Title: Re: EM Drive Developments - related to space flight applications - Thread 2
Post by: lasoi on 05/04/2015 10:09 pm
Speaking of KM, is there anyone out there that has what it takes and is willing to volunteer and set up a wiki or something?
http://en.wikipedia.org/wiki/MediaWiki

I've set up a MediaWiki server before and it wasn't too bad. That is a good platform. I'm simply stretched too thin right now to try it again. My previous attempt at KM (just a simple Google Doc, which was pretty bad didn't catch on so I deleted it:
https://drive.google.com/folderview?id=0B4PCfHCM1KYoTXhSUTd5ZDN2WnM&usp=sharing

@Mulletron et al., I am happy to assist in this.  I agree, this forum is excellent for reviewing the latest contributions by all the excellent minds here, but newer folks are finding it difficult to get a handle on how we got here over the past hundred pages.  As a result some of the same questions are being re-asked repeatedly.  Ideally it might become the FAQ section that was discussed previously (which @Rodal noted some valid concerns regarding).  Worth a try, at least.

To this end, I have set up a MediaWiki server at http://emdrive.echothis.com and will start by the adding the relevant links in today.  However, I'm just a (non-practicing) mechanical engineer, not a physicist so my role in this would be limited to setting up the organization and linking to the relevant content already posted by people far more knowledgeable than I.  Anyone who has been tracking the forum and would like to join in this endeavor, welcome!

I would be most happy to join this effort!

Quite amusingly, I only ever look at updates on this thread during the week (worktime simulations are...boring) and this weekend I had decided that what seemed like the best thing I could do to help out the efforts here (since I'm not particularly knowledgeable with this sort of physics) would be to set up the wiki. When I showed up this morning I saw that I had a rather astounding 13+ pages of forum thread to catch up on! (Usually it seems to be about 4-5).

So when I read Mulletron's suggestion my thoughts were "Ah ha! I'm on the case! But I should finish catching up first." and then I saw you had started one up.

My provisional plan had been to slowly work through from page 1 again creating topics for things as they came up and beginning to organize them as the groupings became obvious.

A few groupings I had identified earlier:
Possible Error Sources of Thrust: Things like thermal buckling, atmospheric effects, etc would get pages of their own and this grouping would just allow for easy perusing of these topics. It might be wise to organize them into some sort of 'Busted', 'Under Investigation', 'Other' groupings. Of course new information can elevate one of the topics out of the busted area or descend it as necessary.

Current Theories: Relatively simple, listing out Shawyer's, White's, the Chinese Teams, Mulletron's, etc theories on what is going on.

Thrust Modes and Results: This one probably just ends up being a table of its own, but is probably particularly specific to the Eaglework's team.

Status of DIY Efforts: Lets Mulletron, Notsosureofit, DIYFan, and others that are replicating this provide easy to reference status updates and other things (future plans, etc).

What I was envisioning was to attempt to use the wiki-source system to provide pointers from information to the forum posts concerning them.

Thoughts?

I just visited the page.  Great work saucyjack and MazonDel.  Will there be a section for Null tests of the drives as well?  That would be a great addition IMO- maybe to the "Building" section.  Here's the only null test I'm aware of:
http://enu.kz/repository/2009/AIAA-2009-5070.pdf
though Dr. Rodal and other posters might be able to provide more information as far as that goes.

I also think a link at the top or bottom of the most current page of this forum would be very helpful.  People coming to this forum for the first time will continue to broach covered topics if they aren't aware of the wiki link.
Title: Re: EM Drive Developments - related to space flight applications - Thread 2
Post by: Rodal on 05/04/2015 10:11 pm
....
We have designed a cylindrical cavity to simplify the problem. Solutions to resonant freq are much easier and more intuitive in a cylinder, and hitting resonance (when we adjust the dielectric) with a constant frequency (from a microwave oven magnetron) will be possible by adjusting the height of the cylinder. ....
But a cylindrical cavity displays no attenuation and no focusing in the axial direction (unlike the truncated cone).  The experimenters in the US, UK and China are using truncated cones. 

If you obtain no net thrust for the cylindrical cavity, will that have been worth the effort you went through (instead of testing a truncated cone, preferably with the same geometry dimensions as NASA's or one of Shaywer's ? )

Concerning obtaining the frequency and mode shape, does your University give you access to a Finite Element analysis package like COMSOL, or ANSYS Multiphysics, etc. so that you could then obtain a numerical solution for any arbitrary geometry and inserted dielectric ?

Dr. Rodal:

I was under the impression that a symmetric shape should produce a net thrust, based on the null Cannae test article at EW. The only asymmetry there was the dielectric slug in the end, correct?

I don't believe we have access to COMSOL, but I will look into another method.

We have yet to purchase materials and may be able to test a truncated shape as well as a symmetric shape with the same copper tube and end plates.

1) At the moment we have a situation where there is only one research location worldwide that has provided research data to analyze (force vs. time curves, geometrical dimensions, testing parameters, testing equipment details, etc. etc.).  This is the only location that has also tested a truncated cone in a vacuum.  This location is NASA Eagleworks, of course.  NASA did not do any testing with the Cannae device in vacuum, to my knowledge.

2) There is (some of them are Ph.D. thesis) research data on this type of measurements going back for more than 50 years, that indicates the problem with thermal convection currents, but there is a documented way around it that apparently neither NASA, nor Shawyer, nor Yang in China have utilized.  More on this lately if you don't have a vacuum chamber and are planning to test in ambient pressure.  Ditto for Mulletron and everybody else trying to do experiments in ambient pressure: there is a better way that has not yet been discussed in this thread.

3) If you don't have access to COMSOL, can you inquire whether you have access to ANSYS-Multiphysics [which is just as capable for these purposes, really] or another FEA package for multiphysics ?
Title: Re: EM Drive Developments - related to space flight applications - Thread 2
Post by: TheTraveller on 05/04/2015 10:18 pm
Instead, I am focusing on the difference in parameters of each experiment--what does the thrust depend on? EW saw a clear dependence on a dielectric, but Shawyer, NWPU and Fetta observed high efficiencies without one. But why does Shawyer mention the "dielectric-filled section" in one of his papers and not reference it in later experiments? http://www.emdrive.com/theorypaper9-4.pdf

Has NWPU tried their magnetron cavity using a dielectric? I believe I read a comment referencing a 2014 paper by Yang but I have not been able to find it.
Did EW test without a dielectric in a vacuum?
Early on Shawyer did use a dielectric but gave it up saying he found it reduced Q and increased losses. Mullerton shared this email he recently received from Shawyer where the non use of a dielectric is stated.
Title: Re: EM Drive Developments - related to space flight applications - Thread 2
Post by: zellerium on 05/04/2015 11:25 pm
Zellerium, I am uncertain about if your University allows for this sort of thing or not, but for the purposes of the nascent Wiki under development, would you be willing to provide (later once you have them) some Bills of Materials as well as instructions documenting how you assembled and set up your rig?

MazonDel:
I am also unfamiliar with our school's policy on the matter, but I will check to make sure it isn't an issue.
I'll finish the schematics for the cavity and counterbalanced teeter-totter soon and post them here.

Dr. Rodal:
Do you think the thrust signiture observed from the null Cannae drive could be due to air currents?
I assume the frustum yielded similar thrusts while in a vacuum and in air, which shows how much the air is effecting thrust.  Unless the shape of the Cannae drive led to a significant air current, it seems a symmetric cavity should produce thrust.

Although we may be able to find the resonant frequencies for the frustum using software, changing parameters like the dielectric thickness will change the frequency required. And if we have to use a microwave magnetron then we have to change the frustum's size which is more challenging, but still possible.
However, if we recieve enough money for the variable frequency test a tapered cavity would be a much better option.

I just downloaded EM Pro recently, but I only have experience with 3-D design software (Creo) and Matlab. We may have licenses available for Multiphysics, I'll look into it.

TheTraveler:
Thank you, I didn't see that email.
Perhaps we are overestimating the effect of the dielectric and our experiment is headed in the wrong direction... On the other hand, could cavity geometry and signal manipulation be the only factors determining efficiency?
Title: Re: EM Drive Developments - related to space flight applications - Thread 2
Post by: TheTraveller on 05/04/2015 11:27 pm
if what  you say is true there is no way Boeing or anyone else involved with any form of propulsion but especially aviation or space flight would lose interest. any reduced mass is astoundingly useful. Even if the thing only reduced mass by 1 percent it would revolutionize everything. Nothing would ever be the same again. Automobiles, trains, planes, rockets; everything.
Is not what I say. Is what Shawyer explains in his how to measure the forces pdf and in his explanation of how his space plane would do a 0.05g vertical lift.

As another example of EM Drive ratchet mode operation, assume we had a EM Drive with a motor & generator mode maximum force generation of 9.8 Newtons and the device had a mass of 0.5kg.

Now support it 1 mtr off the ground via say a small pedestal table, oriented such that the force of gravity would put the EM Drive into Generator / force resistance mode.

Next switch it on.

Now remove the pedestal support table.

Observe it is hovering as the downward 0.5kg weight is opposed by generator mode, which has a max ability to resist 9.8 Newtons of force or 1kg of mass at the Earth's surface.


So if I read you well and equivalence principle holds, having a rocket in deep space accelerating, by conventional mean, at 1g, a floor that is orthogonal to this acceleration, a pedestal resting on this floor, a powered EM drive resting on this pedestal in same configuration, remove the pedestal and one will observe the EM drive hovering above that floor (that is still accelerating at 1g). Meaning we now have a powered EM drive not needing to be "pushed" to accelerate at 1g (there is no longer any interaction between rocket and device).

Stop the conventional thrust of the rocket : the rocket will stop accelerating and proceed as an inertial mass at constant velocity (relative to whatever inertial frame). Let the device escape from an open front bay : it will continue to accelerate at 1g since it was no longer interacting with the rocket when the change in acceleration of the rocket occurred (no interaction => whatever change in rocket trajectory ignored). We now have a "conventionally" accelerating (thrusting) EM drive needing no added force.

So why bother with a Hall thruster ? Just put behind a big dumb powder booster that makes your EM drive accelerate at 1g for a fraction of a second the time it takes for the EM drive to "record" that acceleration as a "starting point". BTW, same argument above could be made if acceleration was 0.5g instead of 1g : this means we now have to add a new intrinsic variable to a moving object. What physical mechanism explains this memory effect of "initial acceleration" into an ongoing acceleration of given magnitude ?

Quote
...
I trust this shows how the EM Drive is unlike anything humanity has experienced before. So please do not move forward thinking it is like a propellantless Hall thruster or rocket motor that generates thrust when the cavity is filled with microwave energy.

Don't underestimate the aptitude of intelligent people to integrate counter intuitive formal systems when they show internal consistency.
Anyway after some sleep and morning coffee, I can now answer your question.

As I see your thought experiment, the hovering EM Drive in the 1g accelerating ship is actually accelerating at 1g, just not in reference to the ship accelerating at 1g. So if it can hover in the 1g accelerating reference frame of the ship, when the ships stops accelerating, the EM Drive will of course continue to accelerate at 1g.

Which BTW it really can't do as energy demand to increasing device kinetic energy conversion is going up with the square of the velocity. As the EM Drive has a max microwave energy capacity in the cavity to draw upon for conversion into device kinetic energy and same for the microwave source and same for the primary energy supply, once the accelerating kinetic demand exceeds that of the cavity / microwave source / primary energy source, the EM Drive's acceleration will stop as there is not enough energy generation capacity to meet the increasing velocity / kinetic energy needs.

Shawyer has covered this.

Can't get blood from a stone or more kinetic energy conversion than the EM Drive's system components will deliver.
Title: Re: EM Drive Developments - related to space flight applications - Thread 2
Post by: dustinthewind on 05/04/2015 11:27 pm
I thought this might be an alternative way of measuring small forces.  Maybe it could be exploited to make a working model if some one thought it was easier to make. 
Title: Re: EM Drive Developments - related to space flight applications - Thread 2
Post by: frobnicat on 05/04/2015 11:27 pm

.../...

For weak gravitational field the frequency ratio between top and bottom is ft/fb = (1 + Rs/2rt - Rs/2rb) where Rs is Schwarzschild radius, rt and rb distance from centre of body (earth centre). From there (http://fr.wikipedia.org/wiki/D%C3%A9calage_d%27Einstein#Fr.C3.A9quence_propre_et_fr.C3.A9quence_observ.C3.A9e), sorry this is French wikipedia, I don't find a convenient English resource for the same formula.

For Earth Rs is about 9mm (http://en.wikipedia.org/wiki/Schwarzschild_radius#Parameters), so lets say we have 0.3m altitude difference at earth surface (6.371e6m) => ft/fb = 1 - 3.3e-17
This is one part in 3e16 redshift in frequency.

Likewise any Doppler effect affecting the relative wavelengths (momentums) of photons between forward and backward plates of an accelerating frustum would indeed induce a non 0 net force : this force would always be opposite to the acceleration (ie. never a thrust) and in fact could be interpreted as the inertia of the mass equivalent of energy bouncing back and forth in the cavity (whatever its shape). The time constant of a photon in a Q=10000 about 0.3m across frustum would be like 10µs, at 100W pumped into the frustum there is then on the order of 1e-3 J EM energy content at any given time, that is equivalent to 1.1e-20 kg of mass, or an apparent added "force of inertia" of 1.1e-19N for a spacecraft accelerating at 1g, or equivalently an added weight of 1.1e-19N vertically for a resting frustum on earth.

In summary, within classical frameworks, yes there can be non 0 net force of EM radiation in an accelerating cavity, but this will be vanishingly small forces, and always opposite to acceleration (aka "inertia").
Is it correct ?

Close, those forces are similar to the observed forces.  Perhaps @Rodal has the page reference back to my calculation which is what I beleive you are stating.

What do you mean by "similar to the observed forces" ? Quantitatively what my very crude estimations of effect of acceleration on EM net force imbalance are 14 orders of magnitude below observed forces at EW, and that is for 1g acceleration, which is far from acceleration of the frustum (horizontal wise), even taking into account recorded noise.

I should really not put my feet in that but : if Doppler effects are to be given importance ( to the point of wanting to compensate for them ) and related influence of acceleration of device on resonance, then this should be quantitatively assessed. A 30cm journey from one end to the other takes a photon 1ns. In such a short time a 1g accelerating frustum would have added or subtracted only v=1e-8 m/s to the velocity seen by the photon on the "last bounce" (this is very crude !). What is the Doppler effect of such a tiny velocity difference ?
f1/f0 = 1 +or- v/c that would amount to a ratio of 1 +or- 3.3e-17, in agreement with above value for gravitational redshift (equivalently, for the same 1g and 30cm distance). I fail to understand how even a superconducting cavity with a Q of a billion and a super narrow bandwidth would care about such a tiny Doppler shift...

And the net force imbalance induced by this Doppler shift, for 100W pumped into frustum at Q=10000 would be 1.1e-19N, at Q=1e9 would be 1.1e-14N ... (and opposite to acceleration). Basically this is just the "inertial force" of the equivalent EM mass multiplied by acceleration...

Beyond this crude photon model I must admit I don't get the more sophisticated equations, I know this was explained quite a bit. Notsosureofit would you care plugging the same parameters in your formulas and derive values, what a ~30cm frustum would experience as net EM force imbalance due to acceleration at 1g, given a feed of 100W at Q=10000 ? What is wrong with a crude photon model that don't show dependency on frequency that would be correct with your model where I see some f^3 but no acceleration ?

I have : F = - acc * P*Q*L/c^3
F net EM radiation imbalance due to acceleration of frustum
acc acceleration in m/s²
P input power in W
L length of cavity in m
c speed of light

This is not the levels of "thrust" (well, this is not a thrust at all !) experimentally recorded, but is it physically sound ?

Quote
Edit:  I think this was it (have to check)

"
The proposition that dispersion caused by an accelerating frame of reference implied an accelerating frame of reference caused by a dispersive cavity resonator. (to 1st order using massless, perfectly conducting cavity)


Starting with the expressions for the frequency of an RF cavity:

f = (c/(2*Pi))*((X/R)^2+((p*Pi)/L)^2)^.5

For TM modes, X = X[sub m,n] = the n-th zero of the m-th Bessel function.
[1,1]=3.83, [0,1]=2.40, [0,2]=5.52 [1,2]=7.02, [2,1]=5.14, [2,2]=8.42, [1,3]=10.17, etc.

and for TE modes, X = X'[subm,n] = the n-th zero of the derivative of the m-th Bessel function.
[0,1]=3.83, [1,1]=1.84, [2,1]=3.05, [0,2]=7.02, [1,2]=5.33, [1,3]=8.54, [0,3]=10.17, [2,2]=6.71, etc.

Rotate the dispersion relation of the cavity into doppler frame to get the Doppler shifts, that is to say, look at the dispersion curve intersections of constant wave number instead of constant frequency.

df = (1/(2*f))*(c/(2*Pi))^2*X^2*((1/Rs^2)-(1/Rb^2))

and from there the expression for the acceleration g from:

g = (c^2/L)*(df/f) such that:

g = (c^2/(2*L*f^2))*(c/(2*Pi))^2*X^2*((1/Rs^2)-(1/Rb^2))

Using the "weight" of the photon in the accelerated frame from:

"W" = (h*f/c^2)*g =>  "W" = T = (h/L)*df

gives thrust per photon:

T = (h/(2*L*f))*(c/(2*pi))^2*X^2*((1/Rs^2)-(1/Rb^2))

If the number of photons is (P/hf)*(Q/2*pi) then:

NT = P*Q*(1/(4*pi*L*f^3))*(c/(2*pi))^2*X^2*((1/Rs^2)-(1/Rb^2))"
Title: Re: EM Drive Developments - related to space flight applications - Thread 2
Post by: QuantumG on 05/04/2015 11:32 pm
The easiest way to measure small forces is to make them bigger.
Title: Re: EM Drive Developments - related to space flight applications - Thread 2
Post by: Rodal on 05/04/2015 11:36 pm
Zellerium, I am uncertain about if your University allows for this sort of thing or not, but for the purposes of the nascent Wiki under development, would you be willing to provide (later once you have them) some Bills of Materials as well as instructions documenting how you assembled and set up your rig?

...
Dr. Rodal:
Do you think the thrust signiture observed from the null Cannae drive could be due to air currents?
I assume the frustum yielded similar thrusts while in a vacuum and in air, which shows how much the air is effecting thrust.  Unless the shape of the Cannae drive led to a significant air current, it seems a symmetric cavity should produce thrust.

Although we may be able to find the resonant frequencies for the frustum using software, changing parameters like the dielectric thickness will change the frequency required. And if we have to use a microwave magnetron then we have to change the frustum's size which is more challenging, but still possible.
However, if we recieve enough money for the variable frequency test a tapered cavity would be a much better option.

I just downloaded EM Pro recently, but I only have experience with 3-D design software (Creo) and Matlab. We may have licenses available for Multiphysics, I'll look into it.

...

Thermal convection forces have plagued radiation pressure measurements since the time of Maxwell, for about 139 years and counting.  There is a rich history showing this.

The shape of the Cannae device maximizes the effect of thermal convection effects.

Please notice that the thrust/PowerInput obtained by NASA Eagleworks in a vacuum is only a fraction of the one that they measured in air, which shows quantitatively the huge problem with conducting tests under ambient conditions like Shawyer in the UK and Yang in China have done.  But, again, if you don't have a vacuum chamber, there are proven ways to minimize this effect that neither Shawyer nor Yang appear to have utilized.

I advise against performing any Finite Element Analysis calculations unless you have an analyst available that has taken a University course in Finite Element Analysis and has practical experience with such Finite Element packages.  If you need to perform FEA you must add to your team such an analyst.

EDIT: If you use the same geometrical dimensions and materials used by NASA, couldn't you use NASA's COMSOL calculations (available in this thread) to assess the frequencies and mode shapes ? (and thus avoid the need to perform any numerical calculations on your own)

Best regards,
Title: Re: EM Drive Developments - related to space flight applications - Thread 2
Post by: TheTraveller on 05/04/2015 11:43 pm
Fellow EM Drive replicators.

Shawyers 1st EM Drive and test rig looks like something fairly simple to duplicate:
http://emdrive.com/feasibilitystudy.html
While sophistication is nice, to start off maybe better to go for KISS? Build what has been shown to work in both the drive and the measurement system. Is what my replication efforts will be following, with no new wheels invested until I have a few that work as Shawyer predicts.

One thing to note, the EM Drive was tested in a vertical orientation, pointed up, then down and the 2 thrust values averaged.
Title: Re: EM Drive Developments - related to space flight applications - Thread 2
Post by: Notsosureofit on 05/04/2015 11:44 pm
I thought this might be an alternative way of measuring small forces.  Maybe it could be exploited to make a working model if some one thought it was easier to make.

We used to use a differential ac capacitor to measure displacement.
Title: Re: EM Drive Developments - related to space flight applications - Thread 2
Post by: apoc2021 on 05/04/2015 11:55 pm

Speaking of KM, is there anyone out there that has what it takes and is willing to volunteer and set up a wiki or something?
http://en.wikipedia.org/wiki/MediaWiki

I've set up a MediaWiki server before and it wasn't too bad. That is a good platform. I'm simply stretched too thin right now to try it again.

I've lurked for the whole thread, but actually did create a media wiki server for this! Its located at http://tracket.co.

I've always wanted to create a distributed, public project management platform to help organize and promote collaboration in global academic and research efforts. Given the acceleration in knowledge generation, I felt that we could increase efficiency through more productive collaboration.

Edit: now that I've finished catching up with the last few pages, I see that a community member has already taken care of this. This wiki will remain online (as it has unused for a couple months) but feel free to ignore as you see fit.
Title: Re: EM Drive Developments - related to space flight applications - Thread 2
Post by: Notsosureofit on 05/05/2015 12:08 am

.../...

For weak gravitational field the frequency ratio between top and bottom is ft/fb = (1 + Rs/2rt - Rs/2rb) where Rs is Schwarzschild radius, rt and rb distance from centre of body (earth centre). From there (http://fr.wikipedia.org/wiki/D%C3%A9calage_d%27Einstein#Fr.C3.A9quence_propre_et_fr.C3.A9quence_observ.C3.A9e), sorry this is French wikipedia, I don't find a convenient English resource for the same formula.

For Earth Rs is about 9mm (http://en.wikipedia.org/wiki/Schwarzschild_radius#Parameters), so lets say we have 0.3m altitude difference at earth surface (6.371e6m) => ft/fb = 1 - 3.3e-17
This is one part in 3e16 redshift in frequency.

Likewise any Doppler effect affecting the relative wavelengths (momentums) of photons between forward and backward plates of an accelerating frustum would indeed induce a non 0 net force : this force would always be opposite to the acceleration (ie. never a thrust) and in fact could be interpreted as the inertia of the mass equivalent of energy bouncing back and forth in the cavity (whatever its shape). The time constant of a photon in a Q=10000 about 0.3m across frustum would be like 10µs, at 100W pumped into the frustum there is then on the order of 1e-3 J EM energy content at any given time, that is equivalent to 1.1e-20 kg of mass, or an apparent added "force of inertia" of 1.1e-19N for a spacecraft accelerating at 1g, or equivalently an added weight of 1.1e-19N vertically for a resting frustum on earth.

In summary, within classical frameworks, yes there can be non 0 net force of EM radiation in an accelerating cavity, but this will be vanishingly small forces, and always opposite to acceleration (aka "inertia").
Is it correct ?

Close, those forces are similar to the observed forces.  Perhaps @Rodal has the page reference back to my calculation which is what I beleive you are stating.

What do you mean by "similar to the observed forces" ? Quantitatively what my very crude estimations of effect of acceleration on EM net force imbalance are 14 orders of magnitude below observed forces at EW, and that is for 1g acceleration, which is far from acceleration of the frustum (horizontal wise), even taking into account recorded noise.

I should really not put my feet in that but : if Doppler effects are to be given importance ( to the point of wanting to compensate for them ) and related influence of acceleration of device on resonance, then this should be quantitatively assessed. A 30cm journey from one end to the other takes a photon 1ns. In such a short time a 1g accelerating frustum would have added or subtracted only v=1e-8 m/s to the velocity seen by the photon on the "last bounce" (this is very crude !). What is the Doppler effect of such a tiny velocity difference ?
f1/f0 = 1 +or- v/c that would amount to a ratio of 1 +or- 3.3e-17, in agreement with above value for gravitational redshift (equivalently, for the same 1g and 30cm distance). I fail to understand how even a superconducting cavity with a Q of a billion and a super narrow bandwidth would care about such a tiny Doppler shift...

And the net force imbalance induced by this Doppler shift, for 100W pumped into frustum at Q=10000 would be 1.1e-19N, at Q=1e9 would be 1.1e-14N ... (and opposite to acceleration). Basically this is just the "inertial force" of the equivalent EM mass multiplied by acceleration...

Beyond this crude photon model I must admit I don't get the more sophisticated equations, I know this was explained quite a bit. Notsosureofit would you care plugging the same parameters in your formulas and derive values, what a ~30cm frustum would experience as net EM force imbalance due to acceleration at 1g, given a feed of 100W at Q=10000 ? What is wrong with a crude photon model that don't show dependency on frequency that would be correct with your model where I see some f^3 but no acceleration ?

I have : F = - acc * P*Q*L/c^3
F net EM radiation imbalance due to acceleration of frustum
acc acceleration in m/s²
P input power in W
L length of cavity in m
c speed of light

This is not the levels of "thrust" (well, this is not a thrust at all !) experimentally recorded, but is it physically sound ?

Quote
Edit:  I think this was it (have to check)

"
The proposition that dispersion caused by an accelerating frame of reference implied an accelerating frame of reference caused by a dispersive cavity resonator. (to 1st order using massless, perfectly conducting cavity)


Starting with the expressions for the frequency of an RF cavity:

f = (c/(2*Pi))*((X/R)^2+((p*Pi)/L)^2)^.5

For TM modes, X = X[sub m,n] = the n-th zero of the m-th Bessel function.
[1,1]=3.83, [0,1]=2.40, [0,2]=5.52 [1,2]=7.02, [2,1]=5.14, [2,2]=8.42, [1,3]=10.17, etc.

and for TE modes, X = X'[subm,n] = the n-th zero of the derivative of the m-th Bessel function.
[0,1]=3.83, [1,1]=1.84, [2,1]=3.05, [0,2]=7.02, [1,2]=5.33, [1,3]=8.54, [0,3]=10.17, [2,2]=6.71, etc.

Rotate the dispersion relation of the cavity into doppler frame to get the Doppler shifts, that is to say, look at the dispersion curve intersections of constant wave number instead of constant frequency.

df = (1/(2*f))*(c/(2*Pi))^2*X^2*((1/Rs^2)-(1/Rb^2))

and from there the expression for the acceleration g from:

g = (c^2/L)*(df/f) such that:

g = (c^2/(2*L*f^2))*(c/(2*Pi))^2*X^2*((1/Rs^2)-(1/Rb^2))

Using the "weight" of the photon in the accelerated frame from:

"W" = (h*f/c^2)*g =>  "W" = T = (h/L)*df

gives thrust per photon:

T = (h/(2*L*f))*(c/(2*pi))^2*X^2*((1/Rs^2)-(1/Rb^2))

If the number of photons is (P/hf)*(Q/2*pi) then:

NT = P*Q*(1/(4*pi*L*f^3))*(c/(2*pi))^2*X^2*((1/Rs^2)-(1/Rb^2))"

My Bad !  I misinterpreted what you were saying.  You are correct the change in a 1g field is quite small for these purposes.

The "g" in the formulas is the amount of acceleration needed to make the tapered cavity "look like" a cylindrical one, ie. compensated.  The force (in the cavity frame) then becomes the equivalent of the "weight" of the photons in that accelerated frame.

The system acceleration that results is then the force divided by the entire system mass.

Hope that helps.
Title: Re: EM Drive Developments - related to space flight applications - Thread 2
Post by: Rodal on 05/05/2015 12:28 am
I thought this might be an alternative way of measuring small forces.  Maybe it could be exploited to make a working model if some one thought it was easier to make.
what is the advantage of this method over a hanging torsional pendulum or over a horizontal torsional balance ?

Thanks
Title: Re: EM Drive Developments - related to space flight applications - Thread 2
Post by: vide on 05/05/2015 12:28 am
The easiest way to measure small forces is to make them bigger.
Much smaller forces have been measured much more accurately for centuries.

Measurement is not the issue; non measurement of classical forces is.
Title: Re: EM Drive Developments - related to space flight applications - Thread 2
Post by: PushHigher on 05/05/2015 12:41 am
Quote
The "g" in the formulas is the amount of acceleration needed to make the tapered cavity "look like" a cylindrical one, ie. compensated

Isn't this the speed limit?  When the frustum warps into the shape of a perfect cylinder (from the perspective of a photon moving toward the large end)? 

If so - it seems that early on in the acceleration, one would want a shallow taper and as the acceleration limit is approached - increasing the taper (restoring the original frustum shape) would allow for further acceleration.  I might have these mixed.

So whoever is building high efficiency version of this thing - you may want to design a wall that can angle, similarly to a jet engine cowl. 

http://www.vehiclehi.com/thumbnails/detail/20121102/engines%20planes%20jet%
20aircraft%20afterburner%201920x1080%20wallpaper_www.vehiclehi.com_16.jpg

I'm not saying it would be easy.   :)
Title: Re: EM Drive Developments - related to space flight applications - Thread 2
Post by: zellerium on 05/05/2015 12:56 am
Thermal convection forces have plagued radiation pressure measurements since the time of Maxwell, for about 139 years and counting.  There is a rich history showing this.

The shape of the Cannae device maximizes the effect of thermal convection effects.

Please notice that the thrust/PowerInput obtained by NASA Eagleworks in a vacuum is only a fraction of the one that they measured in air, which shows quantitatively the huge problem with conducting tests under ambient conditions like Shawyer in the UK and Yang in China have done.  But, again, if you don't have a vacuum chamber, there are proven ways to minimize this effect that neither Shawyer nor Yang appear to have utilized.

I strongly advise against performing any Finite Element Analysis calculations unless you have an analyst available that has taken University courses in Finite Element Analysis and has practical experience with such Finite Element packages.  If you want to perform FEA you must add to your team such an analyst.

EDIT: If you use the same geometrical dimensions and materials used by NASA, couldn't you use NASA's COMSOL calculations (available in this thread) to assess the frequencies and mode shapes ? (and thus avoid the need to perform any numerical calculations on your own)

Best regards,


Oh! I hadn't found the data from the vacuum test, I assume it is buried in the forum somewhere?

If there isn't strong evidence that a symmetric cavity will work, we will build a frustum.
We could use EW's dimensions if we recieve funding for the variable frequency equipment. However, if we have to use a microwave magnetron we will have to do the FEA. I might be able find someone willing to help.

We don't have a vacuum chamber large enough so we will look into minimizing the effects of convection.

Thank you for the response!

Title: Re: EM Drive Developments - related to space flight applications - Thread 2
Post by: Mulletron on 05/05/2015 01:14 am
Want to make sure it isn't forgotten that:

Shawyer said to use narrow band source for cavity with shaped ends.

Wideband is for cavity with flat ends.

Thus, Eagleworks is using the wrong type of signal source.

Can't say we didn't tell them.
Title: Re: EM Drive Developments - related to space flight applications - Thread 2
Post by: WarpTech on 05/05/2015 01:54 am
Thermal convection forces have plagued radiation pressure measurements since the time of Maxwell, for about 139 years and counting.  There is a rich history showing this.

The shape of the Cannae device maximizes the effect of thermal convection effects.

Please notice that the thrust/PowerInput obtained by NASA Eagleworks in a vacuum is only a fraction of the one that they measured in air, which shows quantitatively the huge problem with conducting tests under ambient conditions like Shawyer in the UK and Yang in China have done.  But, again, if you don't have a vacuum chamber, there are proven ways to minimize this effect that neither Shawyer nor Yang appear to have utilized.

I strongly advise against performing any Finite Element Analysis calculations unless you have an analyst available that has taken University courses in Finite Element Analysis and has practical experience with such Finite Element packages.  If you want to perform FEA you must add to your team such an analyst.

EDIT: If you use the same geometrical dimensions and materials used by NASA, couldn't you use NASA's COMSOL calculations (available in this thread) to assess the frequencies and mode shapes ? (and thus avoid the need to perform any numerical calculations on your own)

Best regards,


Oh! I hadn't found the data from the vacuum test, I assume it is buried in the forum somewhere?

If there isn't strong evidence that a symmetric cavity will work, we will build a frustum.
We could use EW's dimensions if we recieve funding for the variable frequency equipment. However, if we have to use a microwave magnetron we will have to do the FEA. I might be able find someone willing to help.

We don't have a vacuum chamber large enough so we will look into minimizing the affects of convection.

Thank you for the response!

This is just my opinion. It is highly doubtful that a symmetric cavity will work "on the same principle" as a frustum. Of course, if the momentum is absorbed into deforming the dielectric, it isn't increasing the kinetic energy of the cylinder in that direction. So that sort of asymmetry can cause a slight walk, but I do not think that is how the frustum works.

The frustum is a waveguide that has a strong gradient in the group velocity, near the cut-off modes, relative to the microwave photons moving inside it. This gradient is mathematically equivalent to a gravitational field and appears to be such from the frame and perspective of the moving photons.

Gravity, interpreted as a variable refractive index K, is a very wide bandwidth effect. The resonant frequencies of all particles, atomic and sub-atomic, are in equilibrium with the local ZPF. As they resonate while immersed in a  variable refractive index, they "fall" toward higher values of K. The EM Drive is the same effect occuring over a very narrow bandwidth, close to the cut-off modes of the frustum where the acceleration of the group velocity is the greatest. The label on the outside should say, Gravity Inside!.

Best Regards,
Todd D.



Title: Re: EM Drive Developments - related to space flight applications - Thread 2
Post by: Rodal on 05/05/2015 02:07 am
....
The frustum is a waveguide that has a strong gradient in the group velocity, near the cut-off modes, relative to the microwave photons moving inside it....
As the initial photons travel towards one of the ends, and it hits the end-plate, yes it functions as a travelling  wave in a waveguide.  However, once the wave hits the end it gets reflected (almost perfectly since the skin depth is extremely small compared to the wavelength and hence the losses are negligible).  At that point we have standing waves.  The high Q of the cavity is a result of these standing waves producing resonance.  So instead of a

waveguide with travelling waves having a non-zero Poynting vector transmitting energy from one end to the other,

what we have upon reflection is a

closed cavity with standing waves having a self-cancelling zero mean value (over a period) Poynting vector and there is no transmission of energy

(http://www.acs.psu.edu/drussell/Demos/superposition/standing.gif)




So, the question is, do you see your mechanism as resulting into an acceleration vs. time that gives constant acceleration at constant power input, for ever and ever (which involves an energy paradox)

or

do you see your mechanism as just resulting into a one-time short-impulse or Dirac delta-function spike in acceleration only (due to the initial photons hitting the end) upon energizing the cavity ?

Regards,

JR
Title: Re: EM Drive Developments - related to space flight applications - Thread 2
Post by: MrVibrating on 05/05/2015 03:10 am
@Rodal

Presumably calorimetry would show an increased output workload when there's a mechanical displacement, compared to the static case, and likewise an accompanying increase in load drawn from the power supply.

With a potential classical symmetry break on the cards wrt N3, testing the energy symmetry should be of top priority..  cos if the virtual photon flux is donating momentum, it's also donating energy...

Title: Re: EM Drive Developments - related to space flight applications - Thread 2
Post by: WarpTech on 05/05/2015 03:40 am
....
The frustum is a waveguide that has a strong gradient in the group velocity, near the cut-off modes, relative to the microwave photons moving inside it....
As the initial photons travel towards one of the ends, and it hits the end-plate, yes it functions as a travelling  wave in a waveguide.  However, once the wave hits the end it gets reflected (almost perfectly since the skin depth is extremely small compared to the wavelength and hence the losses are negligible).  At that point we have standing waves.  The high Q of the cavity is a result of these standing waves producing resonance.  So instead of a

waveguide with travelling waves having a non-zero Poynting vector transmitting energy from one end to the other,

what we have upon reflection is a

closed cavity with standing waves having a self-cancelling zero mean value (over a period) Poynting vector and there is no transmission of energy

(http://www.acs.psu.edu/drussell/Demos/superposition/standing.gif)




So, the question is, do you see your mechanism as resulting into an acceleration vs. time that gives constant acceleration at constant power input, for ever and ever (which involves an energy paradox)

or

do you see your mechanism as just resulting into a one-time short-impulse or Dirac delta-function spike in acceleration only (due to the initial photons hitting the end) upon energizing the cavity ?

Regards,

JR



Thank you for the interesting questions, Dr. Rodal.

I do not think it will have constant acceleration for constant power input over time, because the matter of the frustum and it's source will experience relativistic effects, on mass, time and length. It takes more than a frustum to make an actual warp drive.

For now, I'm not certain that having the resonance occur inside the frustum is necessary. Resonance could occur in an exterior chamber, like pumping a "laser", something to pump the input power. Then inject a tuned coherent pulse of limited bandwidth into the frustum near it's cut-off modes. The objective being, to use the variable refractive index to amplify the momentum toward the small end. By this I mean, the momentum transforms due to the refractive index,

p => p*sqrt(c/vg)

The group velocity depends on location in the frustum and is lowest at the small end. So momentum is amplified toward the small end. The energy is not lost through resistive copper losses, it is absorbed through momentum transfer of the exponentially decaying waves that have been squeezed beyond their cut-off diameter in the waveguide. For these waves, the speed of light has come to a halt and they cannot propagate, so their momentum must be absorbed by the frustum. Where else can it go? They have crossed the event horizon, where c -> 0, the momentum can't escape.

The resonant modes are probably not so close to the cut-off and contribute very little, if anything. There is still a gradient in v group, but a much, much smaller one. I would consider this a different design, one that optimizes Q and very high energy storage over thrust, but the result will work for the same reason.

You also asked me for some equations, graphs and such. I've just started researching here and found an enormous body of information I did not know about. So... it may be a while. :) Any questions you may have on the PV Model and my quantum electrodynamic interpretation of it, I'm happy to assist.

Best Regards,
Todd D.

Title: Re: EM Drive Developments - related to space flight applications - Thread 2
Post by: tchernik on 05/05/2015 03:53 am

...

I do not think it will have constant acceleration for constant power input over time, because the matter of the frustum and it's source will experience relativistic effects, on mass, time and length. It takes more than a frustum to make an actual warp drive.

...

Question: under your interpretation, the Emdrive would have a maximum speed always less than c, because acceleration would eventually decrease until maybe becoming zero due to relativistic effects on the frustum at some speed. Is that correct?

If so, what speed are we talking about? something close to c or much lower?

This topic of the Emdrive's maximum speed and the potentially diminishing acceleration is something that has appeared repeatedly in the discussions, without a clear answer yet because there is no experimental data backing it or disproving it yet.

This notion has also been rebuffed by some people, because assuming a "maximum speed" also assumes a privileged reference frame, which is a big no no in current theories.

I feel there could be a GR explanation, related to the fact that we do have an absolute speed limit: the speed of light, which is the same on all reference frames, including that of the microwaves inside the frustum.
Title: Re: EM Drive Developments - related to space flight applications - Thread 2
Post by: WarpTech on 05/05/2015 05:52 am

...

I do not think it will have constant acceleration for constant power input over time, because the matter of the frustum and it's source will experience relativistic effects, on mass, time and length. It takes more than a frustum to make an actual warp drive.

...

Question: under your interpretation, the Emdrive would have a maximum speed always less than c, because acceleration would eventually decrease until maybe becoming zero due to relativistic effects on the frustum at some speed. Is that correct?

If so, what speed are we talking about? something close to c or much lower?

This topic of the Emdrive's maximum speed and the potentially diminishing acceleration is something that has appeared repeatedly in the discussions, without a clear answer yet because there is no experimental data backing it or disproving it yet.

This notion has also been rebuffed by some people, because assuming a "maximum speed" also assumes a privileged reference frame, which is a big no no in current theories.

I feel there could be a GR explanation, related to the fact that we do have an absolute speed limit: the speed of light, which is the same on all reference frames, including that of the microwaves inside the frustum.

Thank you. I can give you my opinion, but without the context of my latest warp drive paper, I don't know if it will make much sense to anyone. I put a link to that paper back around page 110, I think.

First off, what you said, "...the speed of light, which is the same on all reference frames, including that of the microwaves inside the frustum."

This is not true in EM, GR or the PV Model. The speed of light is only constant in "inertial" reference frames which are not constrained by matter. The frame of reference inside the frustum is non-inertial because the geometry of the waveguide causes a gradient in the speed of light "across a narrow bandwidth" of frequencies, by definition.

I believe the EM drive mimics gravity over a narrow bandwidth, inside the cavity. Anything outside the cavity is still subject to the usual relativistic effects. So it's just a thruster, but a very unique one.

The speed limit is interesting. I'm sure it will be less than c in vacuum, perhaps it would be c/K where K is the largest refractive index, at the small end. That being said...

In the PV Model, the speed of light is NOT constant. The metric coefficients of GR are interpreted as components of a variable refractive index. In regards to the notion of a preferred frame, it is "defined" not absolute. If you and I define the speed of light at the surface of the Earth to be c = 1, then in the PV model, light traveling past at higher altitude is traveling faster, c > 1 because we defined it inside a gravity well. So in the PV Model, we choose to define the refractive index K = 1, "locally" for convenience, but the vacuum is not immutable and neither are the rulers and clocks we use to measure c.

So when I say the speed limit is c/K, I'm implying that outside the EM Drive, K=1 everywhere. If you move it up or down in a gravity well, then K must be scaled to the local value. Relatively speaking, for planets, stars, black holes, K > 1. To build a warp drive that can overcome relativistic effects and go FTL relative to c "locally", then we need to inflate a warp bubble where K < 1. The EM Drive does not do this, but it's on the right track, IMO anyway.

Best Regards,
Todd D.
Title: Re: EM Drive Developments - related to space flight applications - Thread 2
Post by: Flyby on 05/05/2015 07:09 am
Want to make sure it isn't forgotten that:

Shawyer said to use narrow band source for cavity with shaped ends.

Wideband is for cavity with flat ends.

Thus, Eagleworks is using the wrong type of signal source.

Can't say we didn't tell them.

Indeed when reading the new posts this morning, that quote from Shawyer also got my attention, but I see you already spotted it....

The Flight Demonstrator is the alu frustum I've been looking at to get the dimensions. As top and bottom plate are indeed bolted on, and those plates are unusually thick, it would not be too hard to mill them into convex and concave surfaces.

So basically, the idea of shaped endplates has been developed first in the Flight Demonstrator and not for the EMdrive2.0.

Consequently, we should see the EMdrive 2.0, not so much as a testbed, but the gathering of different research aspects into 1 design.  EMdrive2.0 is not so much a theoretical design or paper on a yet to build device, as i first thought but the post-design reconstruction drawings an effective engineered device.

There is a big difference between drawings of a project device and drawings of en executed device.
The value of the drawing (as far as information goes) in the last case is considerably higher....
Title: Re: EM Drive Developments - related to space flight applications - Thread 2
Post by: StrongGR on 05/05/2015 07:31 am
About the possible interpretation of this effect, if confirmed, I would like to point out that a plane wave implies a modified metric in general relativity. This has been presented in Misner, Thorne, Wheeler, Gravitation  at section 35.11 page 961. You can also find a Wikipedia entry (https://en.wikipedia.org/wiki/Monochromatic_electromagnetic_plane_wave) describing it. For the article used in the measurements the situation is more involved as the frustum has not just a single mode but, in principle, each one of these can be seen as propagating in a modified metric. The smaller the input power the smaller the effect. It is my conviction that a full understanding could be achieved with a proper treatment using general relativity. What I have found in literature is overlooking any analysis of the interaction between microwaves and space-time. The effect is miniscule in any case but the interferometer devised at Eagleworks seems well equipped to unveil it.

Title: Re: EM Drive Developments - related to space flight applications - Thread 2
Post by: deuteragenie on 05/05/2015 08:00 am
Quote
Concerning obtaining the frequency and mode shape, does your University give you access to a Finite Element analysis package like COMSOL, or ANSYS Multiphysics, etc. so that you could then obtain a numerical solution for any arbitrary geometry and inserted dielectric ?

OpenFoam (http://www.openfoam.com/) and Elmer (https://www.csc.fi/web/elmer) are free.

For those of us who are not in a position to built a test device, it would be fantastic to play around with a simulation. Cherry on the cake: make it available on github, and link it from the created-a-moment-ago wiki.

Oh, I forgot: if the 3D model would be made with, say, openSCAD (http://www.openscad.org/) then we could parametrize it easily and 3D print it as a shelf model to look at while waiting for the FE simulations to complete...
Title: Re: EM Drive Developments - related to space flight applications - Thread 2
Post by: dustinthewind on 05/05/2015 08:20 am
I thought this might be an alternative way of measuring small forces.  Maybe it could be exploited to make a working model if some one thought it was easier to make.
what is the advantage of this method over a hanging torsional pendulum or over a horizontal torsional balance ?

Thanks

Notsureofit seems to know from the quote. 

We used to use a differential ac capacitor to measure displacement.

He might have more experience with it than I.  I would guess you can increase the surface of the capacitor to increase your sensitivity.  A lock-in amplifier can eliminate noise and amplifiers can further amplify the signal from the capacitor.  It is an alternative to other ways of measuring.  I'm wondering if there might be a way to tune the rate of natural osculation of the system by applying a small offset DC voltage (of the AC wave to used to measure capacitance) as a way of tuning.  There might be some give and take compared to other methods.  I haven't ever personally used one. 
Title: Re: EM Drive Developments - related to space flight applications - Thread 2
Post by: PaulF on 05/05/2015 09:02 am

...

I do not think it will have constant acceleration for constant power input over time, because the matter of the frustum and it's source will experience relativistic effects, on mass, time and length. It takes more than a frustum to make an actual warp drive.

...
Question: under your interpretation, the Emdrive would have a maximum speed always less than c, because acceleration would eventually decrease until maybe becoming zero due to relativistic effects on the frustum at some speed. Is that correct?

If so, what speed are we talking about? something close to c or much lower?

This topic of the Emdrive's maximum speed and the potentially diminishing acceleration is something that has appeared repeatedly in the discussions, without a clear answer yet because there is no experimental data backing it or disproving it yet.

This notion has also been rebuffed by some people, because assuming a "maximum speed" also assumes a privileged reference frame, which is a big no no in current theories.

I feel there could be a GR explanation, related to the fact that we do have an absolute speed limit: the speed of light, which is the same on all reference frames, including that of the microwaves inside the frustum.
The reason some people mentioned reduced acceleration at higher velocities, is with classic newtonian thrust, kinetic energy is squared to the speed. EM Drive tech is by some assumed to convert electrical energy directly to kinetic energy (or at least using a different mechanism than action/reaction), which is a linear process. By that statement alone, you can deduce that with constant power input, and not taking into account relativistic effects, the acceleration will reduce, but it will not do so asymptotically. It's a square root. Of course it will force itself into an asymptote because of eventual relativistic effects (For the distant observer). From traveller's PoV, you can still accellerate indefinitely and the traveller's perceived speed can still go to infinity. Relativity is still in tact as a whole.
Title: Re: EM Drive Developments - related to space flight applications - Thread 2
Post by: Notsosureofit on 05/05/2015 10:55 am
The limit is on acceleration not speed.
Title: Re: EM Drive Developments - related to space flight applications - Thread 2
Post by: Notsosureofit on 05/05/2015 11:07 am
I thought this might be an alternative way of measuring small forces.  Maybe it could be exploited to make a working model if some one thought it was easier to make.
what is the advantage of this method over a hanging torsional pendulum or over a horizontal torsional balance ?

Thanks

Notsureofit seems to know from the quote. 

We used to use a differential ac capacitor to measure displacement.

He might have more experience with it than I.  I would guess you can increase the surface of the capacitor to increase your sensitivity.  A lock-in amplifier can eliminate noise and amplifiers can further amplify the signal from the capacitor.  It is an alternative to other ways of measuring.  I'm wondering if there might be a way to tune the rate of natural osculation of the system by applying a small offset DC voltage (of the AC wave to used to measure capacitance) as a way of tuning.  There might be some give and take compared to other methods.  I haven't ever personally used one.

That was an MIT gizmo, a balanced system of a differential transformer and 4 capacitor plates into which one placed a grounded vane.  Because it was an AC bridge circuit it easily measured microinches, and that wasn't pushing it at all.

Edit: It would not be too hard to modify this into a unit that would both measure displacement and supply a restoring force.
Title: Re: EM Drive Developments - related to space flight applications - Thread 2
Post by: Rodal on 05/05/2015 12:47 pm
About the possible interpretation of this effect, if confirmed, I would like to point out that a plane wave implies a modified metric in general relativity. This has been presented in Misner, Thorne, Wheeler, Gravitation  at section 35.11 page 961. You can also find a Wikipedia entry (https://en.wikipedia.org/wiki/Monochromatic_electromagnetic_plane_wave) describing it. For the article used in the measurements the situation is more involved as the frustum has not just a single mode but, in principle, each one of these can be seen as propagating in a modified metric. The smaller the input power the smaller the effect. It is my conviction that a full understanding could be achieved with a proper treatment using general relativity. What I have found in literature is overlooking any analysis of the interaction between microwaves and space-time. The effect is miniscule in any case but the interferometer devised at Eagleworks seems well equipped to unveil it.

Thanks for the reference and the excellent comment.

This analysis seems very difficult to perform.  The plane wave is a mathematical idealization: it is not possible in practice to have a true plane wave; only a wave of infinite extent will propagate as a plane wave.   Far away from an antenna, it may be a good approximation, but I think you will agree, not inside the small and finite truncated cone (fustrum)  used for the EM Drive which is a closed cavity with practically perfectly reflecting walls.

The plane wave solution is one of the few exact solutions in general relativity  ( https://en.wikipedia.org/wiki/Category:Exact_solutions_in_general_relativity  ).  I doubt whether it is possible to obtain an exact solution for waves in the truncated cone using general relativity.  Even using just Maxwell's equations, the solution for the truncated cone involves two separate eigenvalue problems that need to be solved numerically (one eigenvalue problem in terms of associated Legendre functions and another eigenvalue problem in terms of spherical Bessel functions).

How is one to proceed?

I see your point, that each mode propagates in a modified metric, but how do we analyze this effect inside the geometry of the truncated cone? And like you state, the effect should be miniscule, so I don't understand how this effect will produce a thrust force measurement (for the same power input) that is thousands of times greater than the one of a perfectly collimated photon rocket (letting all the photons escape out of the drive perfectly parallel to each other).

The posts by Todd (  http://forum.nasaspaceflight.com/index.php?action=profile;u=47405 , click on  "Show posts" at the left of the screen  ) are of interest.
Also, @notsosureofit's formulation presented previously in this thread (http://forum.nasaspaceflight.com/index.php?action=profile;u=45732  , click on  "Show posts" at the left of the screen  )

Please let us know if you can advance further into this analysis.

(https://upload.wikimedia.org/wikipedia/en/thumb/3/30/Pw_nevac_monochrom_expansion.gif/300px-Pw_nevac_monochrom_expansion.gif)

Transverse expansion (negative to the right, positive to the left) tensor component  (Electromagnetic plane wave in general relativity)
Title: Re: EM Drive Developments - related to space flight applications - Thread 2
Post by: TheTraveller on 05/05/2015 01:11 pm
Here is my KISS EMDrive test system rough draught.

The EM Drive will sit on top of but not connected to one end of a balance beam. On the other will be an adjustable counter balance mass. Very low stiction bearings will be used.

The EM Drive end of the balance beam will sit on top of but not connected to a digital load cell with a 0.01g resolution / 0.5kg max and be connected via USB to a laptop running data logger software. Would like more resolution, will see how the budget goes.

The counter balance will be adjusted to produce a down force on the load cell of 0.25kg when the EM Drive is unpowered so to bias the load cell into the middle of it's range.

The frequency and power adjustable RF source will be connected to the EM Drive by a free floating length of coax with SWR matching capability and to the laptop via USB connector. Control of frequency and power will be via software on the laptop.

Fairly long power pulses of of upto 2 minutes will be applied to the EM Drive as per Shawyer's 1st test protocol. http://emdrive.com/feasibilitystudy.html

The idea is to keep this KISS and stay as close to the 1st Shawyer test setup that also used vertical orientation of the EM Drive.

Based on achieving 10mN/kW (~1gf/kW) performance, the desired 0.1gf (10x load cell resolution) will need the application of 100W of RF power.

Desire is to use common 2.4GHz narrow band WiFi based signal generators which can be smoothly varied in frequency and power output to find optimal cavity frequency and energy loading.

As this is a narrow band RF signal, ideally the end caps should be spherical to eliminate end plate variable phase change and to get a much better cavity Q. But being a realist and KISS engineer, who hates to reinvent the wheel, will start with simpler flat plates and will follow the excellent work of Mullerton.

Comments most welcome
Title: Re: EM Drive Developments - related to space flight applications - Thread 2
Post by: Giovanni DS on 05/05/2015 01:30 pm
TheTraveler, the RF generator/amplifier/etc could be placed on the scale as part of the counter weight, no floating RF cables this way. Power supply could come up from the center.

There is also need for a down pointing arm from center with a weight for stability.

This setup should not be affected by buckling.
Title: Re: EM Drive Developments - related to space flight applications - Thread 2
Post by: TheTraveller on 05/05/2015 01:45 pm
TheTraveler, the RF generator/amplifier/etc could be placed on the scale as part of the counter weight, no floating RF cables this way. Power supply could come up from the center.

There is also need for a down pointing arm from center with a weight for stability.

This setup should not be affected by buckling.
RF gen & wide band RF amp need pwr and USB connections. Probably better to feed a single & thin RF cable up through a hole in the teeter totter centre and then to left side EM Drive but leave the electronics on the bench.

The balance beam will be at least 200mm wide (bit wider than EM Drive external support rods), with through axle & 2 side bearings, so should be no stability issues. Design like kids teeter totter. Simple. KISS.

Roger no thermal buckling or CG movement issues.

The 4 EM Drive external rod supports will sit on one end on the Teeter Totter. Sorry girls I need your Teeter Totter.
Title: Re: EM Drive Developments - related to space flight applications - Thread 2
Post by: PaulF on 05/05/2015 01:59 pm
The limit is on acceleration not speed.
Yes but only when observed from a reference frame. Traveller's accelleration (from travellers PoV) reduces as a square root and that is not asymptotically (the limit you stated). If it were, speed from the traveller's frame of reference could never grow toward infinity, which it can occording to GR.
Title: Re: EM Drive Developments - related to space flight applications - Thread 2
Post by: TheTraveller on 05/05/2015 02:07 pm
TheTraveler, the RF generator/amplifier/etc could be placed on the scale as part of the counter weight, no floating RF cables this way. Power supply could come up from the center.

There is also need for a down pointing arm from center with a weight for stability.

This setup should not be affected by buckling.
RF gen & wide band RF amp need pwr and USB connections. Probably better to feed a single & thin RF cable up through a hole in the teeter totter centre and then to left side EM Drive but leave the electronics on the bench.

The balance beam will be at least 200mm wide (bit wider than EM Drive external support rods), with through axle & 2 side bearings, so should be no stability issues. Design like kids teeter totter. Simple. KISS.

Roger no thermal buckling or CG movement issues.

The 4 EM Drive external rod supports will sit on one end on the Teeter Totter. Sorry girls I need your Teeter Totter.
Potential USB narrow band RF signal generator / spectrum analyser with frequency adjustable in 1kHz steps and adjustable power output.

With 2 of these units I can setup 1 to slowly gen RF across a frequency range while using the other to do a spectrum scan to see what is happening inside the cavity.

http://www.ebay.com/itm/138M-4-4G-USB-SMA-signal-source-signal-generator-simple-spectrum-analyzer/121372341978

0.01g resolution digital scale with USB:
http://www.ebay.com/itm/500g-x-0-01g-High-Precision-Digital-Scale-SF-400D2-Counting-w-USB-Wall-Adapter-/381198858937

Any comments?
Title: Re: EM Drive Developments - related to space flight applications - Thread 2
Post by: Rodal on 05/05/2015 02:08 pm
....
Thank you for the interesting questions, Dr. Rodal.

I do not think it will have constant acceleration for constant power input over time, because the matter of the frustum and it's source will experience relativistic effects, on mass, time and length. It takes more than a frustum to make an actual warp drive.

For now, I'm not certain that having the resonance occur inside the frustum is necessary. Resonance could occur in an exterior chamber, like pumping a "laser", something to pump the input power. Then inject a tuned coherent pulse of limited bandwidth into the frustum near it's cut-off modes. The objective being, to use the variable refractive index to amplify the momentum toward the small end. By this I mean, the momentum transforms due to the refractive index,

p => p*sqrt(c/vg)

The group velocity depends on location in the frustum and is lowest at the small end. So momentum is amplified toward the small end. The energy is not lost through resistive copper losses, it is absorbed through momentum transfer of the exponentially decaying waves that have been squeezed beyond their cut-off diameter in the waveguide. For these waves, the speed of light has come to a halt and they cannot propagate, so their momentum must be absorbed by the frustum. Where else can it go? They have crossed the event horizon, where c -> 0, the momentum can't escape.

The resonant modes are probably not so close to the cut-off and contribute very little, if anything. There is still a gradient in v group, but a much, much smaller one. I would consider this a different design, one that optimizes Q and very high energy storage over thrust, but the result will work for the same reason.

You also asked me for some equations, graphs and such. I've just started researching here and found an enormous body of information I did not know about. So... it may be a while. :) Any questions you may have on the PV Model and my quantum electrodynamic interpretation of it, I'm happy to assist.

Best Regards,
Todd D.

Todd,

It is exciting to read your explanation about exponentially decaying waves that have been squeezed beyond their cut-off diameter in the waveguide.  You write very clearly.

It's great that you found your way into this thread  :)

Please see the following reference (https://www.osapublishing.org/oe/fulltext.cfm?uri=oe-17-1-34&id=175583, click "Get PDF" to download the paper for free):

It is shown that all modes run continuously from travelling waves through a transition to an evanescent  (exponentially decaying) wave region and the value of the attenuation increases as they approach the cone vertex.

A strict distinction between pure travelling waves and pure evanescent  (exponentially decaying) waves cannot be achieved for conical waveguide.

One mode after the other reaches cutoff in the tapered hollow metallic waveguide as they approach the cone vertex.

Unfortunately, this analysis is for an open waveguide, not for a closed cavity, but the fact that a strict distinction between pure travelling waves and pure evanescent waves cannot be achieved for a conical waveguide, also has implications for modes approaching cutoff in the truncated cone cavity. 

ADDENDUM:

One thing that has not been explored is whether these truncated cones are being prematurely ended towards the cone vertex.  The tested designs are almost cylindrical.

Roger Shawyer has progressively (but very slowly with time) increased the cone angle of his truncated cones, culminating in the superconducting design he unveiled last October 2014.  NASA Eagleworks and Yang in China have truncated cone designs that look like earlier Shawyer designs, with smaller cone angles.


For reference. the tangent of the cone's half angle thetaw and the cone's half angle thetaw, in ascending order, for the following cases are:

(Notice how Shawyer progressively increased the cone's half-angle, with time, in his experimental designs, by a factor of 7 in the tangent of the half-angle)

Example (and geometry)                    { Tan[thetaw],thetaw (degrees) }

Shawyer Experimental                        {0.104019,   5.93851}
Shawyer Fligth Thruster                      {0.19086,   10.8055}
Shawyer Demo                                   {0.219054, 12.3557}
NASA Eagleworks frustum                   {0.263889, 14.7827}
Egan's example                                  {0.36397 ,  20}
Prof. Juan Yang  (2014)                      {0.4538,     24.4 }
Shawyer Superconducting 2014          {0.7002,     35}


(http://gregegan.customer.netspace.net.au/SCIENCE/Cavity/CavityShape.gif)



The people looking at running experiments here are (understandably) also looking at earlier designs with small cone angles and prematurely terminated before reaching the vertex.  The group from a university was looking at running experiments with perfectly cylindrical geometry.

Given the latest write-up by Shawyer (concerning his choice of R1 being too large) it appears that what you are bringing up concerning modes near cutoff has not yet been appreciated or explored, as the researchers are not focusing on general wave solutions in the complex plane containing simultaneously both real and imaginary components.

Best regards,   :)



PS: @aero has valiantly attempted to run a full analysis, of general waves, containing simultaneously both real and imaginary components, using MEEP.  Unfortunately MEEP is finite difference code and hence he has only been able to run 2-D simulations (due to computer time limitations).  It is known that this problem (truncated cone) is 3-D, as a 2-D analysis cannot simulate very important features of the geometry.  The MEEP solutions have also been very difficult to interpret, because of the lack of suitable post-processing software to explore the solutions.

@Mulletron envisioned a completely conical EM Drive, terminating at the vertex (at the beginning of the thread when we were all trying to explore all possibilities with an open mind).  A perfect cone may not be the best solution because in a perfect cone ALL modes are cut-off and hence there will not be any resonance, but in reality it is impossible to have a perfect cone, since the vertex will always terminate with a finite dimension (not a point).  I explored some time ago some geometries, and it looks like there is plenty of room to explore truncated cones that terminate at different distances from the cone vertex.
Title: Re: EM Drive Developments - related to space flight applications - Thread 2
Post by: Notsosureofit on 05/05/2015 02:28 pm
The limit is on acceleration not speed.
Yes but only when observed from a reference frame. Traveller's accelleration (from travellers PoV) reduces as a square root and that is not asymptotically (the limit you stated). If it were, speed from the traveller's frame of reference could never grow toward infinity, which it can occording to GR.

The limit on acceleration in the cavity frame of reference will always be less than that given in the formula if the cavity (and power system) has finite mass.  The "g" there is indicative of what a self-accelerating wave packet of those dimensions would see if there was a zero-mass cavity.

Note: That @##$% typo is still in the last set of formula!

Change to:  N = (P/hf)*(Q/(2*pi*f))   
Title: Re: EM Drive Developments - related to space flight applications - Thread 2
Post by: Flyby on 05/05/2015 03:05 pm
while realizing my knowledge of electromagnetism falls short compared to level that is discussed here, i do have a question about that interesting idea on momentum transfer of the waves :

-with the law on conservation of energy in the back of my head -
 
How can the momentum transfer of a wave be bigger then the energy contained in a photon, as seen in a pure photon rocket ? I believe calculations showed the forces observed in the frustum are many times (100? 1000?) greater then what a photon rocket would be able to produce...

Due to the duality of microwave being a photon particle and a wave at the same time, shouldn't the energy contained in a wave/particle be the same?

Is it because for a photon rocket only a small portion of that energy is used for kinetic motion, while in the momentum transfer a greater part of the energy is transferred? ???


Title: Re: EM Drive Developments - related to space flight applications - Thread 2
Post by: Rodal on 05/05/2015 04:07 pm
while realizing my knowledge of electromagnetism falls short compared to level that is discussed here, i do have a question about that interesting idea on momentum transfer of the waves :

-with the law on conservation of energy in the back of my head -
 
How can the momentum transfer of a wave be bigger then the energy contained in a photon, as seen in a pure photon rocket ? I believe calculations showed the forces observed in the frustum are many times (100? 1000?) greater then what a photon rocket would be able to produce...

Due to the duality of microwave being a photon particle and a wave at the same time, shouldn't the energy contained in a wave/particle be the same?

Is it because for a photon rocket only a small portion of that energy is used for kinetic motion, while in the momentum transfer a greater part of the energy is transferred? ???
Concerning Todd's formulation, I attach below his reply that was posted in another thread, that may also be of help concerning the above question:

COM=conservation of momentum
GR=General Relativity

His answer has to do with the "gravitational field" inside the cavity, which can have different levels of energy :
(in one post he suggested that the experimenters should post a label on the frustum reading "Gravitational Field Inside")

<snip>

Not yet... I'm just now coming to grips with this myself. My light-bulb went off when I realized if the frequency of the microwaves is very close to the cut-off frequencies, then the speed of light will have a very large gradient inside the Frustum. Relative to the "traveling" waves (photons) attempting to move at the speed of light from end to end. When they approach the small end, their wavelength is squeezed by the reduced group velocity. Momentum depends on wavelength;

p = h/lambda

wavelength depends on velocity, and v_g is a variable inside the frustum.

That is where the momentum is coming from. Inside the Frustum, relative to the traveling waves you have an accelerated reference frame, into which you are injecting photons that are affected by this manufactured "gravitational" field, that must be compensated for by moving the Frustum.

I'll see what I can come up with for a formal equation, but I've got a day job. As for @ppnl, you will never get a Newtonian-type COM equation out of this. The two frames are the Frustum, and the frame of the moving photons inside it. The acceleration is caused by the geometry of the waveguide or a variable refractive index, i.e. the GR or PV Interpretation lead to the same result.

Todd D.

Well I certainly agree that You will never get Newtonian-type COM equation out of this. That's what makes it a violation of COM. Hard and simple. You are free to develop a theory that does not conserve momentum but you should call it what it is.

And I don't care what frames are inside the thing. Frames of reference are mathematical fictions. They don't exist. I should not need two frames of reference but only one and it is chosen only for convenience not truth. Any frame should do. Again you are free to develop a theory with a preferred frame that is real but you need to know that that is what you are doing and tell people that that is what you are doing.

If you insist on using Newtonian mechanics, then you will never understand COM in terms of General Relativity. I have not formulated a "new" theory, I'm using GR correctly. If you learn how to do COM in GR, then you would have no trouble seeing that this does indeed conserve momentum. The fact that you "don't care what is inside" is what is preventing you from learning. The "gravitational" field effect of a variable speed of light, acting on the photons inside the Frustum is what makes it move. If you neglect that it has a gravitational field inside it, then you neglect the very essence of how it works and why momentum is conserved. If you want to neglect GR and "believe" COM is violated, then that is your prerogative.

As for why it was not discovered already, I'm kicking myself in the a** for not thinking of this setup 10 years ago when I realized we can mimic gravity over a limited bandwidth with much less energy than over the full bandwidth of all light and matter waves. When my colleague and I wrote our EGM III paper, we had a resonant cavity like this in mind, but we didn't consider the taper.

Best Regards,
Todd D.
Title: Re: EM Drive Developments - related to space flight applications - Thread 2
Post by: TheTraveller on 05/05/2015 04:15 pm
Apologies to all. I thought I had read all the older forum pages. Seems I missed quite a few. Just about finished.

What I did find and confirmed via a search is that Shawyer's latest revelation, in an email to Mullerton, was that the Flight Thruster end plates WERE SHAPED seems to have missed making the headlines.

I did read there was quite a bit of speculation during the time the Flight Thruster dimensions were being worked out as to whether the end plates were flat or shaped.

So how we have it from Shawyer himself, the Flight Thruster was designed to have a high Q (narrow bandwidth), use a narrow band RF generator and have shaped end plates. Seems we finally know for sure why the end plates were so thick.
Title: Re: EM Drive Developments - related to space flight applications - Thread 2
Post by: zen-in on 05/05/2015 04:20 pm
Here is my KISS EMDrive test system rough draught.

The EM Drive will sit on top of but not connected to one end of a balance beam. On the other will be an adjustable counter balance mass. Very low stiction bearings will be used.

The EM Drive end of the balance beam will sit on top of but not connected to a digital load cell with a 0.01g resolution / 0.5kg max and be connected via USB to a laptop running data logger software. Would like more resolution, will see how the budget goes.

The counter balance will be adjusted to produce a down force on the load cell of 0.25kg when the EM Drive is unpowered so to bias the load cell into the middle of it's range.

The frequency and power adjustable RF source will be connected to the EM Drive by a free floating length of coax with SWR matching capability and to the laptop via USB connector. Control of frequency and power will be via software on the laptop.

Fairly long power pulses of of upto 2 minutes will be applied to the EM Drive as per Shawyer's 1st test protocol. http://emdrive.com/feasibilitystudy.html

The idea is to keep this KISS and stay as close to the 1st Shawyer test setup that also used vertical orientation of the EM Drive.

Based on achieving 10mN/kW (~1gf/kW) performance, the desired 0.1gf (10x load cell resolution) will need the application of 100W of RF power.

Desire is to use common 2.4GHz narrow band WiFi based signal generators which can be smoothly varied in frequency and power output to find optimal cavity frequency and energy loading.

As this is a narrow band RF signal, ideally the end caps should be spherical to eliminate end plate variable phase change and to get a much better cavity Q. But being a realist and KISS engineer, who hates to reinvent the wheel, will start with simpler flat plates and will follow the excellent work of Mullerton.

Comments most welcome

One possible problem I see with your proposed experiment is the coiled coax.   There will always be a significant error force from that and there is no simple way of cancelling it out.    Maybe a telescoping waveguide feed would solve that problem.   The load cell will also be in jeopordy.  Too much imbalance will crush it.   
Title: Re: EM Drive Developments - related to space flight applications - Thread 2
Post by: CW on 05/05/2015 04:29 pm
I was wondering about the following:

The tapered frustum's sole purpose is to create a standing EM wave of specific frequency. As far as I remember, a standing wave could also be interpreted as a confined particle in quantum mechanics. Is there any possibilty that the standing waves within the cavity resemble dynamically created exotic matter that shows weird behavior when interacting with the 'normal' matter that the EM-drive is made of?

Title: Re: EM Drive Developments - related to space flight applications - Thread 2
Post by: Stormbringer on 05/05/2015 04:39 pm
This is at least similar to some claims made by Dr white. However I am not sure I have properly understood what he meant when he said these things are doing essentially the same thing the QVPT does and (I think?) maybe he said the QVPT and the Warp Interferometry test article got it's negative energy from the vacuum. Anyhow it may be that your supposition aligns with Dr White but i am not completely sure I understand it.
Title: Re: EM Drive Developments - related to space flight applications - Thread 2
Post by: Flyby on 05/05/2015 04:48 pm
Apologies to all. I thought I had read all the older forum pages. Seems I missed quite a few. Just about finished.

What I did find and confirmed via a search is that Shawyer's latest revelation, in an email to Mullerton, was that the Flight Thruster end plates WERE SHAPED seems to have missed making the headlines.

I did read there was quite a bit of speculation during the time the Flight Thruster dimensions were being worked out as to whether the end plates were flat or shaped.

So how we have it from Shawyer himself, the Flight Thruster was designed to have a high Q (narrow bandwidth), use a narrow band RF generator and have shaped end plates. Seems we finally know for sure why the end plates were so thick.

euhmm... came to the same conclusion...just didn't add the pretty pictures... ;D
http://forum.nasaspaceflight.com/index.php?topic=36313.msg1369324#msg1369324
Title: Re: EM Drive Developments - related to space flight applications - Thread 2
Post by: RotoSequence on 05/05/2015 04:51 pm
Apologies to all. I thought I had read all the older forum pages. Seems I missed quite a few. Just about finished.

What I did find and confirmed via a search is that Shawyer's latest revelation, in an email to Mullerton, was that the Flight Thruster end plates WERE SHAPED seems to have missed making the headlines.

I did read there was quite a bit of speculation during the time the Flight Thruster dimensions were being worked out as to whether the end plates were flat or shaped.

So how we have it from Shawyer himself, the Flight Thruster was designed to have a high Q (narrow bandwidth), use a narrow band RF generator and have shaped end plates. Seems we finally know for sure why the end plates were so thick.

euhmm... came to the same conclusion...just didn't add the pretty pictures... ;D
http://forum.nasaspaceflight.com/index.php?topic=36313.msg1369324#msg1369324

You certainly did! You also ended up with a bottom-of-the-page post, which are fairly easy to miss; that never helps.  :)
Title: Re: EM Drive Developments - related to space flight applications - Thread 2
Post by: TheTraveller on 05/05/2015 04:57 pm
Here is my KISS EMDrive test system rough draught.

The EM Drive will sit on top of but not connected to one end of a balance beam. On the other will be an adjustable counter balance mass. Very low stiction bearings will be used.

The EM Drive end of the balance beam will sit on top of but not connected to a digital load cell with a 0.01g resolution / 0.5kg max and be connected via USB to a laptop running data logger software. Would like more resolution, will see how the budget goes.

The counter balance will be adjusted to produce a down force on the load cell of 0.25kg when the EM Drive is unpowered so to bias the load cell into the middle of it's range.

The frequency and power adjustable RF source will be connected to the EM Drive by a free floating length of coax with SWR matching capability and to the laptop via USB connector. Control of frequency and power will be via software on the laptop.

Fairly long power pulses of of upto 2 minutes will be applied to the EM Drive as per Shawyer's 1st test protocol. http://emdrive.com/feasibilitystudy.html

The idea is to keep this KISS and stay as close to the 1st Shawyer test setup that also used vertical orientation of the EM Drive.

Based on achieving 10mN/kW (~1gf/kW) performance, the desired 0.1gf (10x load cell resolution) will need the application of 100W of RF power.

Desire is to use common 2.4GHz narrow band WiFi based signal generators which can be smoothly varied in frequency and power output to find optimal cavity frequency and energy loading.

As this is a narrow band RF signal, ideally the end caps should be spherical to eliminate end plate variable phase change and to get a much better cavity Q. But being a realist and KISS engineer, who hates to reinvent the wheel, will start with simpler flat plates and will follow the excellent work of Mullerton.

Comments most welcome

One possible problem I see with your proposed experiment is the coiled coax.   There will always be a significant error force from that and there is no simple way of cancelling it out.    Maybe a telescoping waveguide feed would solve that problem.   The load cell will also be in jeopordy.  Too much imbalance will crush it.
Thanks for the feedback.
Most appreciated.

RF coax layout has been modified. Feed will come up through a small hole at the balance point of the Teeter Totter like balance beam system.

Load cell concerns are valid. Plan to use a block under the EM Drive side of the balance system until it is closely balanced, then remove the block, allowing the EM Drive side of the system to contact the load cell and then adjust the balance weight to get a 0.25kg reading on the load cell or it is operating in the centre of it's range and the data logger can record any resultant increase or decrease in EM Drive weight.

RF frequency, power usage of the RF amplified, internal spectral scan from an internal sense antenna, RF power output, cavity top, middle and bottom temperature and load cell deviation += 0.25kg will be recorded with a data logger.

Will also be movement limits on the balance beam teeter totter.
Title: Re: EM Drive Developments - related to space flight applications - Thread 2
Post by: TheTraveller on 05/05/2015 05:01 pm
Apologies to all. I thought I had read all the older forum pages. Seems I missed quite a few. Just about finished.

What I did find and confirmed via a search is that Shawyer's latest revelation, in an email to Mullerton, was that the Flight Thruster end plates WERE SHAPED seems to have missed making the headlines.

I did read there was quite a bit of speculation during the time the Flight Thruster dimensions were being worked out as to whether the end plates were flat or shaped.

So how we have it from Shawyer himself, the Flight Thruster was designed to have a high Q (narrow bandwidth), use a narrow band RF generator and have shaped end plates. Seems we finally know for sure why the end plates were so thick.

euhmm... came to the same conclusion...just didn't add the pretty pictures... ;D
http://forum.nasaspaceflight.com/index.php?topic=36313.msg1369324#msg1369324

You certainly did! You also ended up with a bottom-of-the-page post, which are fairly easy to miss; that never helps.  :)
Good news is the shaped end plates are no longer speculation nor conjecture. The guy who made the Flight Thruster has told us they are shaped.

So WELL DONE. Seems the speculation / conjecture was SPOT ON.
Title: Re: EM Drive Developments - related to space flight applications - Thread 2
Post by: ThereIWas3 on 05/05/2015 05:10 pm
How much power are you planning on putting through that coax?  The thicker it gets, the stiffer it gets, which would mess up any delicate movements of the balance beam.  My experience is with amateur radio, and there 50 watts is the most you would try to put through something like RG-59 (looks like cable-TV cable).   RG-8, which can handle a kilowatt, is about 1 cm in diameter and not very flexible at all.

Power losses in the cable of course turn into heat...
Title: Re: EM Drive Developments - related to space flight applications - Thread 2
Post by: Mulletron on 05/05/2015 05:21 pm
I am afraid that Star-Drive...aka Paul March may not come back. We had such a productive collaboration. That was open source science happening right before the World's eyes.

We'll reach the stars someday, and you're getting us there...one data point at a time.

Title: Re: EM Drive Developments - related to space flight applications - Thread 2
Post by: Rodal on 05/05/2015 05:23 pm


Chinese did report their cavity bandwidth data:
http://www.emdrive.com/yang-juan-paper-2012.pdf

Interesting.   Earlier in the first thread the discussion centered around cavity Q and the need for a high Q.   This cavity only has a Q = 1531.

The 2013 paper from NWPU: "It was found that the thruster
cavity made by copper and resonating on the equivalent TE011
mode has a quality factor 320400 and generates total net EM
thrust 411 mN for 1000 W 2.45 GHz incident microwave." 
http://iopscience.iop.org/1674-1056/22/5/050301
The Q's reported in the tables 1 and 2 of http://www.emdrive.com/NWPU2010paper.pdf  and in the above quote are much higher than the calculated Q based on the bandwidth. 

Zen-In's calculated Q from the bandwidth is correct: Q = f/bandwidthf (by definition)

Can anybody make sense of the discrepancy?

Zen-In's calculated Q from the bandwidth is correct: Q = f/bandwidthf (by definition), but the Q's reported in the tables 1 and 2 of http://www.emdrive.com/NWPU2010paper.pdf  and in the above quotes are much higher than the calculated Q based on the bandwidth.

Paul March has addressed  and explained this as follows: Chinese (Prof. Yang) calculated loaded Q factors are much higher than the Q's reported by Shawyer and by NASA' Eagleworks because of the unorthodox way that the Chinese calculate their loaded Q factors.  Instead of using the S11 zero dB reference plane to measure their -3dB down bandwidths from, as is done elsewhere, the Chinese use the most negative dB S11 value located at the resonance frequency and measure up 3dB toward the S11 zero  dB plane.  Therefore, of course, the bandwidth figures used by the Chinese in this unorthodox calculation are going to be ridiculously small which yields correspondingly artificially large values of the calculated Q-factor.  Prof. Yang's most recent paper containing the embedded thermocouple temperature measurements spell this out very clearly.


BOTTOM LINE: We have to communicate in a common language.  When discussing Q's for Prof. Yang's experiments we cannot use Prof. Yang's calculated Q's as they are artificially high due to her unorthodox calculation.  Instead, the procedure followed by Zen-In should be followed, which gives the real Q (which is actually lower than the Q's measured by NASA Eagleworks or by Shawyer).
Title: Re: EM Drive Developments - related to space flight applications - Thread 2
Post by: CW on 05/05/2015 05:25 pm
BTW, I think Mr. Shawyer claimed that there was no dielectric inserted into his cavity at all because it was not needed for its functioning. Technically though, that's not possible with materials like copper and aluminum. They instantly oxidize during manufacture of the cavity. So, the standing EM waves will encounter a dielectric on any metallic surface.
Title: Re: EM Drive Developments - related to space flight applications - Thread 2
Post by: Rodal on 05/05/2015 05:29 pm
I am afraid that Star-Drive...aka Paul March isn't coming back. We had such a productive collaboration. That was open source science happening right before the World's eyes.

We'll reach the stars someday, and you're getting us there...one data point at a time.

Have you heard something to that effect ?  I have heard it said that NASA has asked them to not discuss NASA work in any public forum, because of what was written in the media (NASA warping space by accident, NASA working on a Star-Trek spaceship, etc.) . 
Title: Re: EM Drive Developments - related to space flight applications - Thread 2
Post by: Stormbringer on 05/05/2015 05:29 pm
I am afraid that Star-Drive...aka Paul March may not come back. We had such a productive collaboration. That was open source science happening right before the World's eyes.

We'll reach the stars someday, and you're getting us there...one data point at a time.
Maybe someone can tell him we've settled down here.  Did the tumult from a few of the new thread participants because of the new publicity discourage him?
Title: Re: EM Drive Developments - related to space flight applications - Thread 2
Post by: Star One on 05/05/2015 05:34 pm
I am afraid that Star-Drive...aka Paul March may not come back. We had such a productive collaboration. That was open source science happening right before the World's eyes.

We'll reach the stars someday, and you're getting us there...one data point at a time.

I was afraid this might happen. Which is a crying shame but perhaps was inevitable once this hit big with the net & attracted the attention of certain denizens of its realm. I really do pray this doesn't fatally impede research into this subject.
Title: Re: EM Drive Developments - related to space flight applications - Thread 2
Post by: Flyby on 05/05/2015 05:34 pm
I am afraid that Star-Drive...aka Paul March isn't coming back. We had such a productive collaboration. That was open source science happening right before the World's eyes.

We'll reach the stars someday, and you're getting us there...one data point at a time.

Is that your personal fear or an unofficial confirmation?

From a personal point of view, that is what I feared the most when I saw how the summary article got mangled , twisted and tortured on some of the more populist science sites.

NASA has a reputation to guard (of performing reliable science and research) and the assumptions made in those other populist articles really corner NASA into a position that -I can  imagine- wouldn't be too pleasing for those at the top of the organization...
If I were in their shoes, i wouldn't be very happy either with the "NASA discovers StarTrek like warp-drive" silliness...
And that the finger waving reaches the lab staff wouldn't surprise me at all.
All these things draw an amazing amount of unwanted FLAK...

Publicity cuts both ways... always...

I really hope P.March reports back to us at certain point with an update on their high power microwave test. Cause that test is going to be soooooo important...
Title: Re: EM Drive Developments - related to space flight applications - Thread 2
Post by: Mulletron on 05/05/2015 05:42 pm
Changes are happening on http://www.emdrive.com/

Mr. Shawyer is speaking to us and we can offer him a forum here if he so desires.

This is new:
NWPU 2010 unpublished test results
http://www.emdrive.com/NWPU2010testresults.pdf
Title: Re: EM Drive Developments - related to space flight applications - Thread 2
Post by: zen-in on 05/05/2015 05:50 pm
I am afraid that Star-Drive...aka Paul March may not come back. We had such a productive collaboration. That was open source science happening right before the World's eyes.

We'll reach the stars someday, and you're getting us there...one data point at a time.

That's to bad.  I hope it is only temporary.   Paul's honesty and commitment to answering our questions was very helpful.    It is not  an enviable task to be doing research in a high risk / high yield area and also fielding questions from the general public.   He deserves a break from all that.
Title: Re: EM Drive Developments - related to space flight applications - Thread 2
Post by: Mulletron on 05/05/2015 05:51 pm
BTW, I think Mr. Shawyer claimed that there was no dielectric inserted into his cavity at all because it was not needed for its functioning. Technically though, that's not possible with materials like copper and aluminum. They instantly oxidize during manufacture of the cavity. So, the standing EM waves will encounter a dielectric on any metallic surface.

Yes and air is also a dielectric and it has been shown experimentally that plain old air exhibits magnetoelectric non-reciprocity under similar conditions.

https://www.google.com/search?q=non+reciprocity+casimir+momentum&ie=utf-8&oe=utf-8
Title: Re: EM Drive Developments - related to space flight applications - Thread 2
Post by: Mulletron on 05/05/2015 05:54 pm
I am afraid that Star-Drive...aka Paul March isn't coming back. We had such a productive collaboration. That was open source science happening right before the World's eyes.

We'll reach the stars someday, and you're getting us there...one data point at a time.

Have you heard something to that effect ?  I have heard it said that NASA has asked them to not discuss NASA work in any public forum, because of what was written in the media (NASA warping space by accident, NASA working on a Star-Trek spaceship, etc.) .

I haven't heard anything like that but it makes perfect sense. I see where Star-Drive is logging into this site but he isn't commenting anymore.  :(
Title: Re: EM Drive Developments - related to space flight applications - Thread 2
Post by: Flyby on 05/05/2015 06:04 pm
while realizing my knowledge of electromagnetism falls short compared to level that is discussed here, i do have a question about that interesting idea on momentum transfer of the waves :

-with the law on conservation of energy in the back of my head -
 
How can the momentum transfer of a wave be bigger then the energy contained in a photon, as seen in a pure photon rocket ? I believe calculations showed the forces observed in the frustum are many times (100? 1000?) greater then what a photon rocket would be able to produce...

Due to the duality of microwave being a photon particle and a wave at the same time, shouldn't the energy contained in a wave/particle be the same?

Is it because for a photon rocket only a small portion of that energy is used for kinetic motion, while in the momentum transfer a greater part of the energy is transferred? ???
Concerning Todd's formulation, I attach below his reply that was posted in another thread, that may also be of help concerning the above question:

COM=conservation of momentum
GR=General Relativity

His answer has to do with the "gravitational field" inside the cavity, which can have different levels of energy :
(in one post he suggested that the experimenters should post a label on the frustum reading "Gravitational Field Inside")
I'm trying to wrap my head around this, so let's see if i got this right...(in simple wording)

Due to the shape of the frustum, energy is gradually compressed going from the large base to the small base. (energy density differs along the longitudinal axis).
 This energy increase also causes a denser gravitational field, effectively slowing down electromagnetic waves, who transfer part of their momentum energy to the frustum...
I can more or less see how "gravitational drag" can cause a forward motion for wave moving towards the small end... but what about the returning waves?

Do they accelerate again when the gravitational field inside the frustum decreases (going from small to large base) , hence nullifying the forward momentum..

or

do they keep their lower energy state on the way back, hence slowly dying out to zero after X bounces?

or.. did i completely miss the point? :'(
Title: Re: EM Drive Developments - related to space flight applications - Thread 2
Post by: aceshigh on 05/05/2015 06:05 pm
I wonder if, while not possible to post directly here anymore, Paul March would still be allowed to indirectly communicate with this open-source science experiment, by answering questions/talking with a specific person FROM the forum (like Dr Rodal maybe).


in some way that he keeps helping us (and people from here helping him with suggestions, etc) BUT when he answers something, that cannot be directly attributed to NASA?
Title: Re: EM Drive Developments - related to space flight applications - Thread 2
Post by: CW on 05/05/2015 06:07 pm
I was wondering about the following:

The tapered frustum's sole purpose is to create a standing EM wave of specific frequency. As far as I remember, a standing wave could also be interpreted as a confined particle in quantum mechanics. Is there any possibilty that the standing waves within the cavity resemble dynamically created exotic matter that shows weird behavior when interacting with the 'normal' matter that the EM-drive is made of?

This is at least similar to some claims made by Dr white. However I am not sure I have properly understood what he meant when he said these things are doing essentially the same thing the QVPT does and (I think?) maybe he said the QVPT and the Warp Interferometry test article got it's negative energy from the vacuum. Anyhow it may be that your supposition aligns with Dr White but i am not completely sure I understand it.

Who does?
;)

Anyway.. I can't help now but picture the confined, standing EM wave as being equivalent to a giant exotic particle that is being squeezed into a metal can made of ordinary matter.

One thing about the dielectric in the cavity also strikes me as similar to what the standing wave of an electron would experience, if the atomic orbital of an electron were squeezed or compressed, so that the current wavelength of the electron does not really fit onto the orbital anymore. Question: What happens to the electron, when the orbital is compressed? Does the wavelength shorten, what would actually happen?
Title: Re: EM Drive Developments - related to space flight applications - Thread 2
Post by: sanman on 05/05/2015 06:23 pm
From what I remember, originally there was the Alcubierre "Warp Drive" concept proposed by Mexican physicist Miguel Alcubierre, which required a jupiter-sized mass of dark matter. Then Dr White came up with a refinement of the geometry, so that the amount of dark matter was much less (someone compared it to the mass of the Voyager space probe.) Then Dr White announced his intention to test whether a "space warp" was possible, by using a Michelson interferometer to try to detect a path-length difference when a voltage was applied to a capacitor ring.

So that stuff was all purely related to Alcubierre "Warp Drive". And of course news media were widely reporting that NASA was researching how to develop a Warp Drive, and the news stories were of course showing pictures of the starship Enterprise from Star Trek. But Shawyer's EMdrive was nowhere in that picture, and was an altogether separate and much less reported story on its own.

Then when the Eagleworks lab got involved in experimental testing to verify the Shawyer EMdrive concept (or Cannae drive, whatever) that's when suddenly the 2 stories began to merge, and speculation was put forth that the EMdrive was somehow a "Q-thruster" and then further that it was a "Warp Drive". Since the actual mechanism - if any - is unknown, what it is seems to depend on who's doing the speculating.
Title: Re: EM Drive Developments - related to space flight applications - Thread 2
Post by: Stormbringer on 05/05/2015 06:34 pm
From what I remember, originally there was the Alcubierre "Warp Drive" concept proposed by Mexican physicist Miguel Alcubierre, which required a jupiter-sized mass of dark matter. Then Dr White came up with a refinement of the geometry, so that the amount of dark matter was much less (someone compared it to the mass of the Voyager space probe.) Then Dr White announced his intention to test whether a "space warp" was possible, by using a Michelson interferometer to try to detect a path-length difference when a voltage was applied to a capacitor ring.

So that stuff was all purely related to Alcubierre "Warp Drive". And of course news media were widely reporting that NASA was researching how to develop a Warp Drive, and the news stories were of course showing pictures of the starship Enterprise from Star Trek. But Shawyer's EMdrive was nowhere in that picture, and was an altogether separate and much less reported story on its own.

Then when the Eagleworks lab got involved in experimental testing to verify the Shawyer EMdrive concept (or Cannae drive, whatever) that's when suddenly the 2 stories began to merge, and speculation was put forth that the EMdrive was somehow a "Q-thruster" and then further that it was a "Warp Drive". Since the actual mechanism - if any - is unknown, what it is seems to depend on who's doing the speculating.
Maybe there is some of that going on out there. But what i am recalling is from this thread. and I think it may have been Mr March or someone directly in a conversation with Mr March in this thread. Again my memory may be faulty on who but not where. It happened here. :) ...or maybe i am insane. :)
Title: Re: EM Drive Developments - related to space flight applications - Thread 2
Post by: TheTraveller on 05/05/2015 06:49 pm
How much power are you planning on putting through that coax?  The thicker it gets, the stiffer it gets, which would mess up any delicate movements of the balance beam.  My experience is with amateur radio, and there 50 watts is the most you would try to put through something like RG-59 (looks like cable-TV cable).   RG-8, which can handle a kilowatt, is about 1 cm in diameter and not very flexible at all.

Power losses in the cable of course turn into heat...
Thanks for the feedback.

Did a few years as a ham. You are right. Cable heat losses need to be considered. Will be very small. Max power will be 100W.

What I plan to do is not a lot different to how Shawyer tests is cryo EM Drive variant as attached. Note the scale under the cryo EM Drive and the other above left?

He has been doing this since 2003 or before. Why reinvent the wheel? Follow the path he has made.
Title: Re: EM Drive Developments - related to space flight applications - Thread 2
Post by: Rodal on 05/05/2015 06:54 pm
I am afraid that Star-Drive...aka Paul March isn't coming back. We had such a productive collaboration. That was open source science happening right before the World's eyes.

We'll reach the stars someday, and you're getting us there...one data point at a time.

There was a time, when a German/US engineer working on ICBM's at the US Army in the early 1950's, at the height of the cold war with the Soviet Union, was allowed to publish a fantastic vision of space exploration first in Collier's magazine and then to appear on TV's Disney's hour to discuss a trip to Mars with a flotilla of spaceships, this at a time before Sputnik, when such a trip to Mars seemed impossible:


https://www.youtube.com/watch?v=MEwOy4UDqC0

https://www.youtube.com/watch?v=Zjs3nBfyIwM

https://www.youtube.com/watch?v=3wIXZsbjIxA

Those programs on Disney inspired a generation that motivated them to go into math, physics, and engineering, and there was no backlash from the US Army.

Amazing how times have changed that now discussing future space applications (even if they are impractical like the flotilla of spaceships to Mars was) is met with such backlash at the one organization that should be inspiring young people to go into mathematics, physics and engineering (instead of being condemned to short trips to near-Earth orbit gazing at our own planet).   

:(


Besides, NASA's present Administrator had (previous to Dr.White's papers on warp-drive) made presentations stating that

We Can Only Survive If We Are A Multi-Planet Species

https://www.youtube.com/watch?v=9K33iXUMqMI
Title: Re: EM Drive Developments - related to space flight applications - Thread 2
Post by: CW on 05/05/2015 07:01 pm
Somehow reminds me of the movie Interstellar, when one of the 'bright' guys says something in the vein of "The world doesn't need any more engineers, it needs farmers". Yeeaahh.. right, 'Einstein'. Well.. in the end it is clear that all who thought like that, perished in the story. Which is, uh, all of mankind except for a couple 1000's of (probably pretty smart) people.

EDIT: Actually.. the movie implies that, because only very smart people made it into space and procreate there, their descendants should also be quite a bit smarter in general than the old generations on Earth. An evolutionary jump, if you will, caused by extinction pressure.
;)
Title: Re: EM Drive Developments - related to space flight applications - Thread 2
Post by: Flyby on 05/05/2015 07:25 pm
.....

Amazing how times have changed that now discussing future space applications (even if they are impractical like the flotilla of spaceships to Mars was) is met with such backlash at the one organization that should be inspiring young people to go into mathematics, physics and engineering (instead of being condemned to short trips to near-Earth orbit gazing at our own planet).   

:(

......


I agree with you for a 110%, but i can't help noticing that our current world is rougher, harsher and more aggressive then the 50 or 60's. The way we communicate with each other has become way more aggressive and opinionated/polarized.

I've just read the infamous Forbes article ppl were referring to in the other (article) topic.........sigh... :-\
Blindly aggressive, polarized and opinionated.. I suppose that ridiculing others makes someone a top article writer these days?

Things like this get under my skin...
why not give hard working ppl (Eagleworks & R.Shawyer) give a chance to chase a dream? If it doesn't pan out, no real harm done...
Title: Re: EM Drive Developments - related to space flight applications - Thread 2
Post by: aceshigh on 05/05/2015 07:28 pm
EDIT: Actually.. the movie implies that, because only very smart people made it into space and procreate there, their descendants should also be quite a bit smarter in general than the old generations on Earth. An evolutionary jump, if you will, caused by extinction pressure.
;)

which goes against current fact: the 10% smarter people on Earth seems to procreate much more slowly than the other 90%.


maybe this trend is older than we think and is the reason why we are still stuck to Earth.
Title: Re: EM Drive Developments - related to space flight applications - Thread 2
Post by: WarpTech on 05/05/2015 07:33 pm
while realizing my knowledge of electromagnetism falls short compared to level that is discussed here, i do have a question about that interesting idea on momentum transfer of the waves :

-with the law on conservation of energy in the back of my head -
 
How can the momentum transfer of a wave be bigger then the energy contained in a photon, as seen in a pure photon rocket ? I believe calculations showed the forces observed in the frustum are many times (100? 1000?) greater then what a photon rocket would be able to produce...

Due to the duality of microwave being a photon particle and a wave at the same time, shouldn't the energy contained in a wave/particle be the same?

Is it because for a photon rocket only a small portion of that energy is used for kinetic motion, while in the momentum transfer a greater part of the energy is transferred? ???
Concerning Todd's formulation, I attach below his reply that was posted in another thread, that may also be of help concerning the above question:

COM=conservation of momentum
GR=General Relativity

His answer has to do with the "gravitational field" inside the cavity, which can have different levels of energy :
(in one post he suggested that the experimenters should post a label on the frustum reading "Gravitational Field Inside")
I'm trying to wrap my head around this, so let's see if i got this right...(in simple wording)

Due to the shape of the frustum, energy is gradually compressed going from the large base to the small base. (energy density differs along the longitudinal axis).
 This energy increase also causes a denser gravitational field, effectively slowing down electromagnetic waves, who transfer part of their momentum energy to the frustum...
I can more or less see how "gravitational drag" can cause a forward motion for wave moving towards the small end... but what about the returning waves?

Do they accelerate again when the gravitational field inside the frustum decreases (going from small to large base) , hence nullifying the forward momentum..

or

do they keep their lower energy state on the way back, hence slowly dying out to zero after X bounces?

or.. did i completely miss the point? :'(

I too am still trying to wrap my head around it all, but to answer some of the questions here, I think it is important to understand the relationship of the Energy-momentum 4-vector and how it transforms in a gravitational field. I'm sure we are all aware of the relationship for transverse EM waves where,

E^2 = p^2 * c^2

This can be re-written simply

p = E/c

In free space, no-one would argue with this. However, in the wave guide, as in a gravitational field, c is replaced by a group velocity, then roughly speaking,

p ~ E/v_g

The group velocity is a variable inside the tapered waveguide. So momentum is "amplified" toward the small end, relative to the "conserved" energy input. Momentum is conserved at the expense of a velocity gradient and the motion of the frustum. The gain in momentum is inversely proportional to the reduction in wave velocity.

If we had a photon rocket in a vacuum where the speed of light, c/K << c, it would have a very large thrust! It is the value of c in vacuum that makes it so weak.

Inside the waveguide, you do not need to worry about energy density as you do in GR. It is simply the geometry that is controlling the group velocity, not the energy density. I should be more careful with my words. It is the same principle as "gravity", it "mimics" gravity over a narrow bandwidth, but if you try to apply GR and calculate space-time curvature, you will have a difficult time justifying it. It's not space-time that's curved, it's the taper of the frustum.

Waves that are reflected and are not cut-off by the taper do not contribute much, if anything. They are however, red-shifted when moving backwards, toward the higher v_g end. What I am currently trying to ascertain is whether the wavelengths of these reflected waves bouncing back and fourth, decay over time to longer wavelengths that are then cut-off and absorbed. If that's true, then obviously, having a high Q resonant cavity will amplify the effect enormously. I have a lot of research to do. :)

I"m also wondering if anyone would like to discuss my warp drive technology and how ZPF gravity works, I can start another thread, so as not to clutter the EM Drive thread with other stuff. It would be great if more engineers understood it as I do.

Best Regards,
Todd D.
Title: Re: EM Drive Developments - related to space flight applications - Thread 2
Post by: zen-in on 05/05/2015 07:50 pm
How much power are you planning on putting through that coax?  The thicker it gets, the stiffer it gets, which would mess up any delicate movements of the balance beam.  My experience is with amateur radio, and there 50 watts is the most you would try to put through something like RG-59 (looks like cable-TV cable).   RG-8, which can handle a kilowatt, is about 1 cm in diameter and not very flexible at all.

Power losses in the cable of course turn into heat...
Thanks for the feedback.

Did a few years as a ham. You are right. Cable heat losses need to be considered. Will be very small. Max power will be 100W.

What I plan to do is not a lot different to how Shawyer tests is cryo EM Drive variant as attached. Note the scale under the cryo EM Drive and the other above left?

He has been doing this since 2003 or before. Why reinvent the wheel? Follow the path he has made.

Eagleworks mounted their PA on the balance beam.   The frequency source could also be custom made and mounted on the balance beam.   That would eliminate the coax problem.
Interesting photograph; I have not seen it before.   The experiment is nicely setup with very good shielding for air currents.   But I don't see how it can be called cryogenic.   It looks like the Nitrogen dewar is just supplying dry Nitrogen.   The apparatus inside the clear boxes and sitting on a postage scale is not a dewar.   Unless he has found a supplier of room temperature superconductors it is not a superconductor test.  Superconducting cavities have to be cooled with liquid Helium and to keep the lHe from boiling off in a flash the lHe dewar has to be surrounded by a liquid Nitrogen dewar.   A high vacuum has to separate the dewars from everything and high IR reflective surfaces and other exotic contrivances are needed.   Otherwise the cryogens boil off and the whole thing turns into a missile.   High temperature superconductors are easier to work with because they only require liquid Nitrogen.  The second picture shows my crufty rotating magnetic field setup.   The white thing is a styrofoam container filled with liquid Nitrogen and the dut (device under test).   
Title: Re: EM Drive Developments - related to space flight applications - Thread 2
Post by: PaulF on 05/05/2015 08:05 pm
BTW, I think Mr. Shawyer claimed that there was no dielectric inserted into his cavity at all because it was not needed for its functioning. Technically though, that's not possible with materials like copper and aluminum. They instantly oxidize during manufacture of the cavity. So, the standing EM waves will encounter a dielectric on any metallic surface.
Would a copper or aluminium (british :) ) alloy, one that does not oxidize easily, do the trick or would that never provide enough reflectivity for the microwaves?

Because what you are stating is that even mr. Shawyer never had a thruster without dielectric. And thus measurements without dielectric do not exist...
Title: Re: EM Drive Developments - related to space flight applications - Thread 2
Post by: CW on 05/05/2015 08:07 pm
EDIT: Actually.. the movie implies that, because only very smart people made it into space and procreate there, their descendants should also be quite a bit smarter in general than the old generations on Earth. An evolutionary jump, if you will, caused by extinction pressure.
;)

which goes against current fact: the 10% smarter people on Earth seems to procreate much more slowly than the other 90%.


maybe this trend is older than we think and is the reason why we are still stuck to Earth.

True. However, schools and universities serve as a filtering mechanism. There is a clear tendency for highly educated and intelligent people to marry each other and procreate. Rarely will you find a couple with a highly incompatible intellectual profile. Hence, over time, our modern societies create an pre-filtered work evironment that literally promotes genetic reinforcement of certain properties, for instance greater-than-average intelligence, within their specific metiers. I remember a TEDtalks presentation that stressed that aspect of our modern world (sorry, no link right now). Pretty interesting stuff :) .
Title: Re: EM Drive Developments - related to space flight applications - Thread 2
Post by: Flyby on 05/05/2015 08:11 pm
to Todd D.

Thank you for replying in such an elaborated way!
 
As you've noticed by my wording, I do not have a scientific background and most certainly will need to reread your reply several times.
On top of that,  English is only my 3rd language so assessment of what's being said is obviously a slow process on it self.
But, I must say that,  since a few months, this EMdrive topic forced me to study some aspects of science and math I've never seen before.
All in all, not a bad thing... :P

It is on rare moments like these, that i have a tiny tiny regret for not having taken the path of a nuclear science engineer, but architecture instead...a difficult choice i made decades ago...
Title: Re: EM Drive Developments - related to space flight applications - Thread 2
Post by: Rodal on 05/05/2015 08:13 pm
Yesterday I was too busy with other work, and I gave the benefit of the doubt to posters that insisted that there was new information supplied by R. Shawyer.

Upon further inspection, it is immediately evident from looking at Shawyer's paper that this is not new information.

The way that R. Shawyer defines R2 and R1 and L1, it necessarily follows that L1 must be such that L1=R2-R1

This is a tautology, and as such it cannot be described as "new information", any more than supplying another paper stating that R2=R1+L1 could claimed to be new information, and then supplying another paper stating that R1=R2-L1 could claimed to be new information, or supplying another paper defining a new variable R3=R1+R2 is new information, or supplying another paper defining a new variable R4=Sqrt[R1^2+R2^2].  One can construct endless definitions, definitions of new variables don't provide any new information.

New information would be to supply the dimensional value of R1 and R2 (from which L1 necessarily follows and it would be unnecessary, superfluous and bad engineering drawing practice to also give).


http://forum.nasaspaceflight.com/index.php?topic=36313.msg1369066#msg1369066
Title: Re: EM Drive Developments - related to space flight applications - Thread 2
Post by: TheTraveller on 05/05/2015 08:13 pm
Eagleworks mounted their PA on the balance beam.   The frequency source could also be custom made and mounted on the balance beam.   That would eliminate the coax problem.
Interesting photograph; I have not seen it before.   The experiment is nicely setup with very good shielding for air currents.   But I don't see how it can be called cryogenic.   It looks like the Nitrogen dewar is just supplying dry Nitrogen.   The apparatus inside the clear boxes and sitting on a postage scale is not a dewar.   Unless he has found a supplier of room temperature superconductors it is not a superconductor test.  Superconducting cavities have to be cooled with liquid Helium and to keep the lHe from boiling off in a flash the lHe dewar has to be surrounded by a liquid Nitrogen dewar.   A high vacuum has to separate the dewars from everything and high IR reflective surfaces and other exotic contrivances are needed.   Otherwise the cryogens boil off and the whole thing turns into a missile.   High temperature superconductors are easier to work with because they only require liquid Nitrogen.  The second picture shows my crufty rotating magnetic field setup.   The white thing is a styrofoam container filled with liquid Nitrogen and the dut (device under test).
Here is what is inside the shiny metal cylinder. An EM Drive with a superconducting inner surface.

According to Shawyer it is designed to be cooled by liquid Nitrogen and it's super-conducting surfaces are formed from YBCO thin films on sapphire substrates.
Title: Re: EM Drive Developments - related to space flight applications - Thread 2
Post by: tea monster on 05/05/2015 08:18 pm
The Disney/Von Braun films of the 50's were based on solid, mathematically provable science and physics. Apart from building the ships, all the concepts were based on solid, proven rules of nature.

The EM drive is nowhere near that stage. None of the physics of how it works are proven and its operation as described seems to violate the conservation of energy.

It could well be that we are on the cusp of a new wonderful age of innovation. Or we may be looking at another Dean drive. Until we know for certain, nobody is going to be producing films for Disney.

If it is proven to work as described (and we should know soon), then there will be plenty of excitement, magazine articles and documentaries.
Title: Re: EM Drive Developments - related to space flight applications - Thread 2
Post by: StrongGR on 05/05/2015 08:22 pm
As far as I can tell, I am not the only one to have thought to a general relativity effect here. This paper (http://arxiv.org/abs/1302.5690) uses a modified version of Einstein equations to explain what is going on in a conical resonant cavity without violating momentum conservation. The point is that the author uses a weak perturbation approximation and I do not know if this is fully justified.
Title: Re: EM Drive Developments - related to space flight applications - Thread 2
Post by: FutureStormtrooper on 05/05/2015 08:24 pm
Maybe someone can tell him we've settled down here.  Did the tumult from a few of the new thread participants because of the new publicity discourage him?

I sent him an Email at his nasa.gov address letting him know essentially that last night. Didn't hear back, but time will tell.
Title: Re: EM Drive Developments - related to space flight applications - Thread 2
Post by: Rodal on 05/05/2015 08:35 pm
As far as I can tell, I am not the only one to have thought to a general relativity effect here. This paper (http://arxiv.org/abs/1302.5690) uses a modified version of Einstein equations to explain what is going on in a conical resonant cavity without violating momentum conservation. The point is that the author uses a weak perturbation approximation and I do not know if this is fully justified.

I had read that paper. Unfortunately, as the author of the paper states:

Quote
The weakest part of the theory seems to be that there is no clear way of preventing large gravitational effects due to the magnetic field of the Earth, as predicted by Eq. (17)

Therefore the author himself admits that his model is contradicted by predicting large gravitational effects due to the Earth's magnetic field that are contrary to all experimental evidence.
Title: Re: EM Drive Developments - related to space flight applications - Thread 2
Post by: sanman on 05/05/2015 08:43 pm
From what I remember, originally there was the Alcubierre "Warp Drive" concept proposed by Mexican physicist Miguel Alcubierre, which required a jupiter-sized mass of dark matter. Then Dr White came up with a refinement of the geometry, so that the amount of dark matter was much less (someone compared it to the mass of the Voyager space probe.) Then Dr White announced his intention to test whether a "space warp" was possible, by using a Michelson interferometer to try to detect a path-length difference when a voltage was applied to a capacitor ring.

So that stuff was all purely related to Alcubierre "Warp Drive". And of course news media were widely reporting that NASA was researching how to develop a Warp Drive, and the news stories were of course showing pictures of the starship Enterprise from Star Trek. But Shawyer's EMdrive was nowhere in that picture, and was an altogether separate and much less reported story on its own.

Then when the Eagleworks lab got involved in experimental testing to verify the Shawyer EMdrive concept (or Cannae drive, whatever) that's when suddenly the 2 stories began to merge, and speculation was put forth that the EMdrive was somehow a "Q-thruster" and then further that it was a "Warp Drive". Since the actual mechanism - if any - is unknown, what it is seems to depend on who's doing the speculating.
Maybe there is some of that going on out there. But what i am recalling is from this thread. and I think it may have been Mr March or someone directly in a conversation with Mr March in this thread. Again my memory may be faulty on who but not where. It happened here. :) ...or maybe i am insane. :)

And besides that, Paul March was initially involved with research on Prof Woodward's "Mach Effect"  theory, which is again something altogether different from Alcubierre warp drive and EMdrive. Mach Effect as of yet cannot be claimed to violate the established laws of physics, and is supposedly a prerequisite or necessary consequence of Einstein's Relativity.

But so the Woodward Mach Effect experiments were about oscillating masses, weren't they? They were not the same type of apparatus as the EMdrive.

So when I see people mentioning warp drive, Woodward's theory, and EMdrive all in the same thread, it kind of looks like the lines are being blurred here. These things originally started out as completely distinct from each other - they were 3 separate things - and now it looks like they're all being mashed together in the thread discussions. Perhaps it's because the same researchers at Eagleworks have been involved in investigating all 3 things.
Title: Re: EM Drive Developments - related to space flight applications - Thread 2
Post by: Rodal on 05/05/2015 08:51 pm
From what I remember, originally there was the Alcubierre "Warp Drive" concept proposed by Mexican physicist Miguel Alcubierre, which required a jupiter-sized mass of dark matter. Then Dr White came up with a refinement of the geometry, so that the amount of dark matter was much less (someone compared it to the mass of the Voyager space probe.) Then Dr White announced his intention to test whether a "space warp" was possible, by using a Michelson interferometer to try to detect a path-length difference when a voltage was applied to a capacitor ring.

So that stuff was all purely related to Alcubierre "Warp Drive". And of course news media were widely reporting that NASA was researching how to develop a Warp Drive, and the news stories were of course showing pictures of the starship Enterprise from Star Trek. But Shawyer's EMdrive was nowhere in that picture, and was an altogether separate and much less reported story on its own.

Then when the Eagleworks lab got involved in experimental testing to verify the Shawyer EMdrive concept (or Cannae drive, whatever) that's when suddenly the 2 stories began to merge, and speculation was put forth that the EMdrive was somehow a "Q-thruster" and then further that it was a "Warp Drive". Since the actual mechanism - if any - is unknown, what it is seems to depend on who's doing the speculating.
Maybe there is some of that going on out there. But what i am recalling is from this thread. and I think it may have been Mr March or someone directly in a conversation with Mr March in this thread. Again my memory may be faulty on who but not where. It happened here. :) ...or maybe i am insane. :)

And besides that, Paul March was initially involved with research on Prof Woodward's "Mach Effect"  theory, which is again something altogether different from Alcubierre warp drive and EMdrive. Mach Effect as of yet cannot be claimed to violate the established laws of physics, and is supposedly a prerequisite or necessary consequence of Einstein's Relativity.

But so the Woodward Mach Effect experiments were about oscillating masses, weren't they? They were not the same type of apparatus as the EMdrive.

So when I see people mentioning warp drive, Woodward's theory, and EMdrive all in the same thread, it kind of looks like the lines are being blurred here. These things originally started out as completely distinct from each other - they were 3 separate things - and now it looks like they're all being mashed together in the thread discussions. Perhaps it's because the same researchers at Eagleworks have been involved in investigating all 3 things.
Or perhaps they are connected as there are countless examples I can give in physics where what was supposed to be earlier theories (thought to be very different at the time) they turned out to be incomplete parts of a bigger theory encompassing them.  Many examples: Schrodinger wave equation and Heissenberg's matrix formulation,   Five consistent versions of string theory were developed until it was realized in the mid-1990s that they were different limits of a conjectured single 11-dimensional theory now known as M-theory. etc etc   I can go on and on...

(Of course, I know that Schrodinger, Heisenberg, and Witten are on a complete different scale to the level of the present "theories" , but you get the point of what Paul March refers to when he states that QV and ME may be different sides of the same coin)
Title: Re: EM Drive Developments - related to space flight applications - Thread 2
Post by: zen-in on 05/05/2015 08:52 pm
Eagleworks mounted their PA on the balance beam.   The frequency source could also be custom made and mounted on the balance beam.   That would eliminate the coax problem.
Interesting photograph; I have not seen it before.   The experiment is nicely setup with very good shielding for air currents.   But I don't see how it can be called cryogenic.   It looks like the Nitrogen dewar is just supplying dry Nitrogen.   The apparatus inside the clear boxes and sitting on a postage scale is not a dewar.   Unless he has found a supplier of room temperature superconductors it is not a superconductor test.  Superconducting cavities have to be cooled with liquid Helium and to keep the lHe from boiling off in a flash the lHe dewar has to be surrounded by a liquid Nitrogen dewar.   A high vacuum has to separate the dewars from everything and high IR reflective surfaces and other exotic contrivances are needed.   Otherwise the cryogens boil off and the whole thing turns into a missile.   High temperature superconductors are easier to work with because they only require liquid Nitrogen.  The second picture shows my crufty rotating magnetic field setup.   The white thing is a styrofoam container filled with liquid Nitrogen and the dut (device under test).
Here is what is inside the shiny metal cylinder. An EM Drive with a superconducting inner surface.

According to Shawyer it is designed to be cooled by liquid Nitrogen and it's super-conducting surfaces are formed from YBCO thin films on sapphire substrates.

OK I see how that works now; just a liquid Nitrogen dewar with the dut suspended in it.   Very similar to my setup.   Nitrogen boil-off will affect the lower scale but not the one that supports the dut so much.   I would imagine there is a lot of boil-off during a test.     YBCO thin films have high AC losses; typically above 60 Hz.   No doubt these are specially designed to minimize those lossses.   
Title: Re: EM Drive Developments - related to space flight applications - Thread 2
Post by: Stormbringer on 05/05/2015 09:02 pm
From what I remember, originally there was the Alcubierre "Warp Drive" concept proposed by Mexican physicist Miguel Alcubierre, which required a jupiter-sized mass of dark matter. Then Dr White came up with a refinement of the geometry, so that the amount of dark matter was much less (someone compared it to the mass of the Voyager space probe.) Then Dr White announced his intention to test whether a "space warp" was possible, by using a Michelson interferometer to try to detect a path-length difference when a voltage was applied to a capacitor ring.

So that stuff was all purely related to Alcubierre "Warp Drive". And of course news media were widely reporting that NASA was researching how to develop a Warp Drive, and the news stories were of course showing pictures of the starship Enterprise from Star Trek. But Shawyer's EMdrive was nowhere in that picture, and was an altogether separate and much less reported story on its own.

Then when the Eagleworks lab got involved in experimental testing to verify the Shawyer EMdrive concept (or Cannae drive, whatever) that's when suddenly the 2 stories began to merge, and speculation was put forth that the EMdrive was somehow a "Q-thruster" and then further that it was a "Warp Drive". Since the actual mechanism - if any - is unknown, what it is seems to depend on who's doing the speculating.
Maybe there is some of that going on out there. But what i am recalling is from this thread. and I think it may have been Mr March or someone directly in a conversation with Mr March in this thread. Again my memory may be faulty on who but not where. It happened here. :) ...or maybe i am insane. :)

And besides that, Paul March was initially involved with research on Prof Woodward's "Mach Effect"  theory, which is again something altogether different from Alcubierre warp drive and EMdrive. Mach Effect as of yet cannot be claimed to violate the established laws of physics, and is supposedly a prerequisite or necessary consequence of Einstein's Relativity.

But so the Woodward Mach Effect experiments were about oscillating masses, weren't they? They were not the same type of apparatus as the EMdrive.

So when I see people mentioning warp drive, Woodward's theory, and EMdrive all in the same thread, it kind of looks like the lines are being blurred here. These things originally started out as completely distinct from each other - they were 3 separate things - and now it looks like they're all being mashed together in the thread discussions. Perhaps it's because the same researchers at Eagleworks have been involved in investigating all 3 things.
Having had time to think about it. the person said one is just an "introverted warp" drive. compared to the WI article which is an extroverted warp drive. so maybe it's searchable...

EDIT:  No; no luck. But maybe that jarred someone else's memory?
Title: Re: EM Drive Developments - related to space flight applications - Thread 2
Post by: TheTraveller on 05/05/2015 09:08 pm
Eagleworks mounted their PA on the balance beam.   The frequency source could also be custom made and mounted on the balance beam.   That would eliminate the coax problem.
Interesting photograph; I have not seen it before.   The experiment is nicely setup with very good shielding for air currents.   But I don't see how it can be called cryogenic.   It looks like the Nitrogen dewar is just supplying dry Nitrogen.   The apparatus inside the clear boxes and sitting on a postage scale is not a dewar.   Unless he has found a supplier of room temperature superconductors it is not a superconductor test.  Superconducting cavities have to be cooled with liquid Helium and to keep the lHe from boiling off in a flash the lHe dewar has to be surrounded by a liquid Nitrogen dewar.   A high vacuum has to separate the dewars from everything and high IR reflective surfaces and other exotic contrivances are needed.   Otherwise the cryogens boil off and the whole thing turns into a missile.   High temperature superconductors are easier to work with because they only require liquid Nitrogen.  The second picture shows my crufty rotating magnetic field setup.   The white thing is a styrofoam container filled with liquid Nitrogen and the dut (device under test).
Here is what is inside the shiny metal cylinder. An EM Drive with a superconducting inner surface.

According to Shawyer it is designed to be cooled by liquid Nitrogen and it's super-conducting surfaces are formed from YBCO thin films on sapphire substrates.

OK I see how that works now; just a liquid Nitrogen dewar with the dut suspended in it.   Very similar to my setup.   Nitrogen boil-off will affect the lower scale but not the one that supports the dut so much.   I would imagine there is a lot of boil-off during a test.     YBCO thin films have high AC losses; typically above 60 Hz.   No doubt these are specially designed to minimize those lossses.
Shawyer reports on that unit:

Quote
Small signal testing at 77 deg K confirmed the design, with a Q of 6.8x106 being measured

An EM Drive with a Q of 6,800,000! Massive!
Title: Re: EM Drive Developments - related to space flight applications - Thread 2
Post by: Rodal on 05/05/2015 09:36 pm
Eagleworks mounted their PA on the balance beam.   The frequency source could also be custom made and mounted on the balance beam.   That would eliminate the coax problem.
Interesting photograph; I have not seen it before.   The experiment is nicely setup with very good shielding for air currents.   But I don't see how it can be called cryogenic.   It looks like the Nitrogen dewar is just supplying dry Nitrogen.   The apparatus inside the clear boxes and sitting on a postage scale is not a dewar.   Unless he has found a supplier of room temperature superconductors it is not a superconductor test.  Superconducting cavities have to be cooled with liquid Helium and to keep the lHe from boiling off in a flash the lHe dewar has to be surrounded by a liquid Nitrogen dewar.   A high vacuum has to separate the dewars from everything and high IR reflective surfaces and other exotic contrivances are needed.   Otherwise the cryogens boil off and the whole thing turns into a missile.   High temperature superconductors are easier to work with because they only require liquid Nitrogen.  The second picture shows my crufty rotating magnetic field setup.   The white thing is a styrofoam container filled with liquid Nitrogen and the dut (device under test).
Here is what is inside the shiny metal cylinder. An EM Drive with a superconducting inner surface.

According to Shawyer it is designed to be cooled by liquid Nitrogen and it's super-conducting surfaces are formed from YBCO thin films on sapphire substrates.

OK I see how that works now; just a liquid Nitrogen dewar with the dut suspended in it.   Very similar to my setup.   Nitrogen boil-off will affect the lower scale but not the one that supports the dut so much.   I would imagine there is a lot of boil-off during a test.     YBCO thin films have high AC losses; typically above 60 Hz.   No doubt these are specially designed to minimize those lossses.
Shawyer reports on that unit:

Quote
Small signal testing at 77 deg K confirmed the design, with a Q of 6.8x106 being measured

An EM Drive with a Q of 6,800,000! Massive!


what is "small signal testing" ?  I imagine that it means just testing the cavity for resonance, to measure the Q but it is not referring to a small force measurement, is that right ?
Title: Re: EM Drive Developments - related to space flight applications - Thread 2
Post by: Flyby on 05/05/2015 09:42 pm

Shawyer reports on that unit:

Quote
Small signal testing at 77 deg K confirmed the design, with a Q of 6.8x106 being measured

An EM Drive with a Q of 6,800,000! Massive!

and no trace on how much force it generated?   :'(
because that is what we all really want to know: does a higher Q effectively increase the thrust factor dramatically ?
So far we've only seen claims and projections on how great it will be.

A clear result would be so much nicer...
It would give better idea whether it effectively works and if it does, how well the EMdrive is scalable...

Iow,(IF it works) is it only good enough for orbital corrections or do we have prospect on interplanetary travel in the coming 10 or 20 years?
Title: Re: EM Drive Developments - related to space flight applications - Thread 2
Post by: R.W. Keyes on 05/05/2015 10:04 pm
How much power are you planning on putting through that coax?  The thicker it gets, the stiffer it gets, which would mess up any delicate movements of the balance beam.  My experience is with amateur radio, and there 50 watts is the most you would try to put through something like RG-59 (looks like cable-TV cable).   RG-8, which can handle a kilowatt, is about 1 cm in diameter and not very flexible at all.

Power losses in the cable of course turn into heat...
Thanks for the feedback.

Did a few years as a ham. You are right. Cable heat losses need to be considered. Will be very small. Max power will be 100W.

Me as well. I am still licensed but not very active.
CQ CQ de N1YRK   

I agree coax does get stiff, but some moreso than others. at these frequencies, they'd probably be using LMR400, which is still pretty stuff, but I recall it being less so than RG8.  I was one of the principles of a small company which made 2.4 ghz antennas in the early 2000s.
Title: Re: EM Drive Developments - related to space flight applications - Thread 2
Post by: JonnyDamnnox on 05/05/2015 11:19 pm
Hi guys, on the theoretical side, has someone looked into this:


http://arxiv.org/abs/hep-th/0508246
Using strong external fields to modify locally a space-time
Title: Re: EM Drive Developments - related to space flight applications - Thread 2
Post by: ThereIWas3 on 05/05/2015 11:32 pm
The cool thing about all this that, while the theories may need the PhDs and the GR stuff, the experiments can be done by hundreds of basement hacker types.  (Maybe not the superconductive part).  There may be a Zephram Cochrane out there somewhere yet who will crack this.
Title: Re: EM Drive Developments - related to space flight applications - Thread 2
Post by: TheTraveller on 05/05/2015 11:32 pm
Traveller

I strongly suggest you do not place your "mass displacement measurement device" underneath the drive unit.  If you invert your thinking and make the opposite end of the fulcrum lever "heavier" such that it places a small but measurable  force on the scale, you can data log the hopefully rising-value differences on activation. The reasoning behind this is that your scales "might" be impacted by a grav-wave event originating from the em-drive, creating a chaotic temporal event stream.

You may also need to consider the geometric shape of the end caps.

I suggest you have a browse though data on  Schmidt-Cassegrain telescopes for the optical path folding models employed and their direct relevance to the end cap shapes. Consider the possibility for the existence of a focal-point inside the actual tube assembly "that has not been mentioned in the literature", and where this point may occur relative to the end caps. Also consider the ability to extend the tubes physical length with simple mechanical sliding action end cap(s). These factors could impact the deliberate creation of a specific point-of-intersection and as such an area of modified intensity relative to the internal space of the unit. The energy imbalances.. could be interesting.
Or
Consider end cap shapes as a geometric means of creating a parallel-path guide for simple reflection/refraction in a laser-like mode
Interesting suggestions for your 1st post. Most appreciated. Welcome to the forum.

I once designed and built Schmidt-Cassegrain telescopes so I understand fully your, light bulb illuminating, suggestions.

Ever spent any time in the UK?
Title: Re: EM Drive Developments - related to space flight applications - Thread 2
Post by: JonnyDamnnox on 05/05/2015 11:33 pm
I have another question, is there any official statement that NASA is working on warp field experiments? Just for the Wikipedia article on the topic.
Title: Re: EM Drive Developments - related to space flight applications - Thread 2
Post by: Rodal on 05/06/2015 01:29 am
REVIEW OF A.L. Cullen's "Absolute Power Measurement at Microwave Frequencies" IN REFERENCE TO EM DRIVE

First of all I would like to thank Roger Shawyer, C.Eng MIEE. SPR Ltd., for having made available to us Cullen's paper published in Feb., 1952, which is very difficult to obtain otherwise.  This paper has been referenced by Mr. Shawyer in a number of his papers and presentations on the theory and practice of the EM Drive operation, and thus we were always curious as to its content.

As Cullen himself writes, the purpose of Cullen's paper was to describe the techniques adopted for his Ph.D. thesis experiments (at the University of London) and to explain their theoretical basis.

It is apparent, upon reading Cullen's paper, that:

1) No truncated cone (fustrum of a cone, as in Shawyer's EM Drive) of any kind was used in Cullen's experiments or ever discussed in Cullen's paper from a theoretical viewpoint. Cullen used constant (through their length) cross-section waveguides.  One waveguide had a rectangular cross-section and another waveguide had a circular cross-section.

2) No summation of forces for microwaves inside a closed cavity truncated cone (fustrum of a cone, as in Shawyer's EM Drive) are ever discussed in Cullen's paper.

3) No Einstein’s law of addition of velocities or relativistic frames for the beam and the waveguide are used or discussed in Cullen's paper.

4) The cylindrical waveguide used in Cullen's experiment for radiation pressure measurement was not a closed-cavity with all internal surfaces reflective (as in the EM Drive) but was instead a cylindrical waveguide with transparent glass as one of its ends (transparent glass which deliberately allowed the microwave beam to enter the cylindrical waveguide and also allowed the reflected waves to escape the cylindrical waveguide so that only travelling waves would hit the other end of the waveguide for pressure measurements).

5) The expressions that Cullen uses to calculate the pressure were first derived by Maxwell (the diagonal component of Maxwell's stress tensor).  Cullen uses the more intuitive, physical derivation of J.J. Thompson.  In any case, Cullen's radiation pressure expression is fully compatible with the expressions used by Greg Egan http://gregegan.customer.netspace.net.au/SCIENCE/Cavity/Cavity.html, where Greg Egan restates the known conclusion (from Maxwell's equations) that any closed resonating cavity has zero net force due to radiation.  (Cullen correctly shows the non-zero pressure on a cylindrical waveguide that has only one reflecting end - the other is "open" because it is made of transparent glass). 

6) Cullen points out (in section 2.1 of his paper) that the power supplied to the load can be obtained by integrating the time-average Poynting vector, and that the force on the reflecting end of the circumferential waveguide is proportional to the supplied power.  This is completely correct.  It follows therefore that, since the time-average Poynting vector for a closed cavity is known to be zero, then the total radiation force in a closed cavity is zero, as per Cullen, which is in agreement with the known result restated by Greg Egan.

SUMMARY: I could not find anything in Cullen's paper supporting that a closed cavity resonating at microwave frequencies, like the EM Drive, can produce a net thrust force (or be used for any kind of propulsion).   

Cullen's Ph.D. thesis is based on the classical theory of electromagnetism, by Maxwell, and hence wholly consistent with the known result that Maxwell's equations show no net thrust for any closed cavity, with all internal surfaces reflective, and having any arbitrary shape.




Cullen's article has a number of useful observations:

A) It was impossible to obtain a stable baseline, even on a relatively short-term basis of a minute's duration.  This continual drifting of the baseline was found to be due to air convection currents set up by small and changing temperature gradients within the microwave waveguides.  The remedy was to reduce the air resistance of the reflecting end plate so that the convection currents would have no appreciable effect.  The reflecting end plate was replaced by a system of concentric wire rings (shown on Fig. 12 of Cullen's paper).  The rings acted as an almost perfect reflector of the electromagnetic waves but at the same time had a small effective cross-section to air currents.  NASA, Shawyer, Yang, and other EM Drive researchers would be well advised to experiment with replacing the end plates of the EM Drive with this system of concentric rings, in order to address the problem of air convection currents that has plagued radiation pressure experiments in ambient conditions ever since Maxwell 140 years ago.

B) It is very interesting that the deflection vs. time curves for the microwave radiation pressure experiments look  similar to the deflection vs. time curves for the tests performed at NASA Eagleworks, except for the drifting baseline present in the NASA tests.

C) Cullen used a rectangular waveguide to direct the radiation. To prevent subsequent radiation from the reflector the waveguide was terminated with a matched load, such that there was no reflection of the incident wave.  A T-junction was used for transmission to a piston (a cylindrical waveguide with one end made of transparent glass and the other end capped by a reflector), without reflection into the main guide.

D) Standing wave measurements were separately performed with a closed cavity and movable (micrometer) end plate to assess the purity of mode TM01 in the waveguides, to make sure that resonance at other modes (mainly TM21) was not significant.


_______________________

Attachment: ABSOLUTE POWER MEASUREMENT AT MICROWAVE FREQUENCIES
By A. L. CULLEN, Ph.D., B.Sc.(Eng.), Associate Member.
(published February, 1952.)
Title: Re: EM Drive Developments - related to space flight applications - Thread 2
Post by: ThinkerX on 05/06/2015 01:30 am
Quote
I have another question, is there any official statement that NASA is working on warp field experiments? Just for the Wikipedia article on the topic.

Eagleworks, NOT NASA proper.  And:

Quote
Mr March told us recently back in the thread that they had a small positive signal on the warp interferometry front.

Emphasis on the small, and possibly a false positive (refraction from atmosphere was mentioned).  Lots more tests under differing conditions badly needed.
Title: Re: EM Drive Developments - related to space flight applications - Thread 2
Post by: Carl G on 05/06/2015 01:48 am
I have another question, is there any official statement that NASA is working on warp field experiments? Just for the Wikipedia article on the topic.
Mr March told us recently back in the thread that they had a small positive signal on the warp interferometry front.

Which isn't an official NASA statement, as anyone will obviously know, which is why Stormbringer didn't emphasis that. To be clear for those who need it, the answer is no.
Title: Re: EM Drive Developments - related to space flight applications - Thread 2
Post by: PushHigher on 05/06/2015 02:02 am
Here is my KISS EMDrive test system rough draught.

TheTraveller - best wishes with your endeavor.  While pondering the shape of the device from here:

http://1.bp.blogspot.com/-IUOMkncS1sM/VFLSUw2HeoI/AAAAAAAA3yE/Zl3iaKJyrX0/s1600/emdrivedata2.png (http://1.bp.blogspot.com/-IUOMkncS1sM/VFLSUw2HeoI/AAAAAAAA3yE/Zl3iaKJyrX0/s1600/emdrivedata2.png)

I feel the drive is optimized to align the EM vertically down the chamber.  Any force measuring apparatus that requires the device to turn might reduce efficiency.  I would suggest using a pulley.

Wesley Crusher wishes this  ;)
Title: Re: EM Drive Developments - related to space flight applications - Thread 2
Post by: deltaMass on 05/06/2015 02:17 am
Although most of what's being discussed here concerns the static force produced by an EmDrive, I would like to look ahead to the dynamics in free space. In this regard, and needing no recourse to relativity for low velocities, all types of propellantless propulsion device are created equal; i.e. we model them with constant input power which results in a constant thrust, which in turn produces a constant acceleration. A little high school physics and a dash of algebra (see below) should suffice to convince you that, for any and all such devices, there exists a break-even velocity, above which more energy has been produced than has been consumed. As the device continues to further accelerate, a continuous source of free power is available. Thus we have not only perpetual motion, but free energy to boot.

It is worthwhile to quantify the value of this break-even velocity, which turns out to be a very simple expression. The relevant equations we use (symbols having their usual meaning) are:
k := F/P in Newton/Watt
v = a t = (F/m) t
Ein = P t
Eout = 0.5 m v2

From the first 2 equations we get
t = (m v) / (k P)

At breakeven, Ein= Eout and v := v0
or
2 P m v0 / (k P) = mv02
so
v0 = 2/k

Armed with this expression, the performance of any propellantless propulsion device may be examined, if its 'k' value is known.
This is an adjunct to my overunity analysis for all propellantless propulsion craft when allowed to move subrelativistically in free space. This time we'll use the mundane expressions for cars on roads, rockets and all the familiar things that do not employ floobie dust in their operation. Let's look at an ideal car on a flat road using ideal tyres. The constants of the motion this time are only P (engine power) and m (all up mass). Now we have a velocity-dependent thrust F and variable velocity and acceleration. Now our Newtons/Watt coefficient 'k' also varies with velocity. We only need two equations to sort this all out and to demonstrate its mundanity.

1. F = k P = m a, so a = (P/m) k
2. P = F v = P k v, so k = 1/v
so again 1. a = dv/dt = (P/m) (1/v)

Solving this by integration (with the limit v=0 @t=0) we get
P t = 0.5 m v2

which is the same as saying
Ein = Eout

No overunity here. Utterly mundane.
It is the crucial expression P = F v which makes everything come out OK.

This is what Shawyer uses.
But EmDrive is not a tyre and spacetime is not a road.
It's in fact eventually (v > 2/k) overunity,  because F remains constant at all v.
And that's in turn because if F knew about v then a preferred frame of reference exists, and SR is violated.
Title: Re: EM Drive Developments - related to space flight applications - Thread 2
Post by: sanman on 05/06/2015 02:52 am
Or perhaps they are connected as there are countless examples I can give in physics where what was supposed to be earlier theories (thought to be very different at the time) they turned out to be incomplete parts of a bigger theory encompassing them.  Many examples: Schrodinger wave equation and Heissenberg's matrix formulation, etc etc

But Rodal,

Prof Alcubierre's theory doesn't really involve any displacement in the conventional sense, and only says that spacetime can be warped using the exotic Dark Matter (whose existence has not been proven or disproven yet). The Alcubierre-based spaceship only sits still, while space itself does all the moving/warping. So Alcubierre's concept is able to go FTL, since there are supposedly no limits on warping space as there is with conventional displacement of matter. It doesn't relate to conservation of momentum at all, since it doesn't even refer to impulse or acceleration - so I don't see how it connects to the other 2 things.

Warp Drive is a purely FTL thing and doesn't claim to have anything to do with conventional displacement or momentum as we know it. And neither Mach Effect nor EMdrive claim to be able to go FTL, even though they are claiming a novel mechanism for displacement.

Prof Woodward's Mach Effect theory of transient mass fluctuations says that if you change the internal energy of an accelerating mass, then you are effectively altering its mass property in a way that creates an asymmetric imbalance in the action-reaction force, and can thus achieve net movement without ejecting any propellant in the conventional way. Do this over and over again, using a mechanically oscillating system where the charge oscillates in phase, and you get continuous movement. He is also saying that no system is a truly closed system, since it will always be interacting with every other mass in the universe. That seems to also imply that you cannot truly apply Conservation of Momentum unless you're talking about the universe as a whole.

Shawyer's EMdrive seems to claim that asymmetric radiation pressure is possible in an asymmetrically-shaped cavity where one end acts as a waveguide for the trapped radiation. The EMdrive is supposed to be similar to a photon-rocket, but it's a photon-rocket fitted with an expander nozzle at the end, in the form of a waveguide.

Title: Re: EM Drive Developments - related to space flight applications - Thread 2
Post by: sanman on 05/06/2015 02:54 am
I wanted to keep this in its own separate post.
For a clear and detailed explanation of EMdrive's propulsion mechanism, I will quote to you from Goatguy who hangs out on NextBigFuture:

http://nextbigfuture.com/2015/05/emdrive-possible-warp-or-hyperspace.html#more

Quote from: GoatGuy
Quote from: sanman
So if it's the standing wave inside the resonant cavity which is losing energy, then how is that energy being converted into momentum? If the standing wave is pushing on the cavity in a net direction, then how is that standing wave creating an asymmetric radiation pressure inside the cavity? How does the asymmetric shape of the cavity result in asymmetric radiation pressure? Shouldn't radiation pressure inside a cavity always be symmetric?


GoatGuy in reply to sanman:

…then how is the energy being converted to momentum… is one of the fundamental questions which physicists are arguing over. The notion is that the asymmetry of the cone as a waveguide for RF energy realizes a net force away from the small end as the RF wave stream propagates outward and out the end.

This is my gross simplification, but it has a large analogy to help visualize the principle(s) involved: the classic chemical rocket engine.

If one thinks of a chemical rocket engine combustion chamber as NOT a combustion chamber, but merely a round sphere full of high pressure gas, if the sphere is unperforated, obviously there is no gas escape, and no net thrust. No matter how much pressure is in the theoretical sphere. Now, poke a hole in it. The entrained gas will escape. The speed with which it escapes depends on its viscosity, the size of the hole, its density and the speed of sound in the gas, and the interior pressure. The thrust can be either measured or calculated with relative ease. Yet, this is not the highest thrust that the pierced sphere can produce.

Higher thrust can be produced by having an expansion nozzle on the other side of the hole. How's this? Simple enough: the exhaust stream is still pressurized, and will expand in 3 dimensions, so its cross sectional area is also increasing. This too is an outward radial pressure. If not captured (from a simple hole), then it is lost thrust/force. If the expanding gas though impinges on the exhaust cone as it expands outward, the outward expansion is vectored “downward”. Change of momentum vector results in more thrust.

Same gas, more thrust.

(This is one of the chief reasons there are shock diamonds in jet and transparent rocket thrust.)

_______

Now, take the analogy back to the EM-thruster regime. And a very brief review of electromagnetic principles. The first is, that when one reflects an electromagnetic wave, there will be momentum transfer orthogonal to the net moment vector change. Fancy wording, but what it means is this. IF you reflect an electromagnetic wave 180°, or exactly backwards at 100% efficiency (no loss), the reflection will produce a force F which equal to 2E/c. (E = energy, c is speed of light). 1 joule of reflected energy will produce a force of 6.67×10⁻⁹ newton-seconds.

(Note that this applies to light itself. Hence why giant reflectors of sunlight in space might make useful force generators someday. The light is “free”)

Now, imagine instead that we are reflecting the EMR (electromagnetic radiation) only 90°, at a right angle. The moment change will be the same, but the vector isn't ½(180°) but ½(90°) = 45°. The force will depend on the grazing angle, but just suffice to say that we'd realize something closer to E/c instead of 2E/c.

(Again in the solar-sail idea, if one were to fold the solar sail at right angles, and present the corner to the Sun, obviously one side would reflect sunlight away at right angle, and the other side would reflect the other way. The outward force would cancel. But there'd still be a net push. Only ½ that of the reflected backward geometry.)

OK, enough EMF theory. The point is clear: reflected EMR generates a force which depends on the angle of reflectance and the power of the EMR being reflected.
You might want to re-read that!

The principle of the EM-Thruster is that it is a cone of (at this point) constant angle. Looks like about 30° to me or so. Like the rocket-engine, the "high pressure gas" or intense EMR is injected at the small end. Being standing waves in a resonant structure, it bounces around a lot, making its way outward after many reflections. Every time it reflects, it imparts a bit of force against the angled surface of the cone.

This results (as the theory goes) in an amplification of imparted momentum-vector change force. The purveyors of the idea go on to theorize that the amplification should depend on the number of times the EMR wave reflects before exit (or diminishment from lossy-ohmic resistance by the copper metal each reflection). This is actually a rather reasonable idea. The trick though is to keep the RF energy entrained in that cone for as many reflections as is possible while also maximizing the cone's 'exhaust angle'. (i.e. make it a loooooong pencil of a cone, and the angle is so small that there would be very little net propulsive force. But you'd get zillions of reflections. Make it quite wide, and you get good force from each reflected wavelet, but they won't hang out in the cone very long. So, not much net force.)

ANYTHING that I've heard so far seems to rest upon the above idea. Which is fine, because its solid physics. It even explains why someone got the thought of putting a dielectric back there at the concentrated end, to try to keep the EMR in-tube longer. More reflections. Keeping the energy in-tube.

But where the principle goes “crazy” is in supposing that the Q of a EMR reflective cone -and-dielectric-system- can be engineered up to the giga Q level or higher (through superconduction). Obviously, not for an open-ended system. So far (and the images are surprisingly hard to find), I believe that the cone is not open ended, but is capped with a flat reflector. Again, to entrain the EMR in the cone for as many reflections as possible. (and the only way to get Q above about 100 or so for 30° angle cones).

So, we must wait and see. Tho' my electromagnetic theory is pretty solid, I really haven't spent significant time analyzing the train-of-physics that is concocted to support the idea that thrust is dependent on power² (which it should not be!), or of EMR frequency² (which it should not be!).

Lastly, there is a kind of “esthetic solution” to the problem of the high-output devices violating conservation of energy above W/F (watts of power per newtons of force) m/s (meters per second velocity)… sort of. The first is, when the device accelerates, the reflecting EMR inside the cone is bouncing off surfaces which are also accelerating; though infinitesimal, each reflected wave will lose energy, lowering (widening) its wavelength. Since by its nature, resonant systems have a sharper-and-sharper frequency of resonance as Q increases, the wave-system will no longer be in resonance, and won't optimally reflect endlessly. This is helpful in solving the free-energy-device problem, though not complete.

The actual conservation of energy solution comes from noting that in any reflected momentum system, one cannot derive more momentum energy from the energy reflected, than the reflected energy itself.

Sounds circular? It is. But it also works. Bounce a fast-moving glass ball bearing back and forth between large glass reflective surfaces, and it will bound perhaps thousands of times (losing near-all of its kinetic energy to sound (tick-tick-tick…) and to heat). Put the reflectors though on perfect bearings (so that they might accelerate). The ball bearing will only now bounce a few dozen times. If one calcultes how much kinetic energy is imparted to the moving reflector wall, its easy to show that no more kinetic energy than what was within the bearing's frame of reference is imparted to the accelerating reflector. In a perfectly coupled system, the energy transfer would be 100%. The ball would come to rest, and the moving reflector would have all its original kinetic energy added to whatever Ek it had to begin with.

This then looks (to me) like the same system. The EM-Drive concept is bouncing the EMR around, transforming its electromagnetic energy into kinetic thrust energy, with some limit, which is very probably asymptotic to the input energy.

Which would be just fine. Exactly what electric motors do all the time. Transforming electromagnetic energy to kinetic energy. The trick is to find a way to "do this" and also limit the transfer of kinetic energy to that imposed by the basic rocket equation. (Which limits imparted kinetic energy in the vehicle to 1/2 the kinetic energy of the exhaust, integrated over the flight of the vessel.)

GoatGuy
Title: Re: EM Drive Developments - related to space flight applications - Thread 2
Post by: deltaMass on 05/06/2015 03:04 am
Quote from: GoatGuy
Now, take the analogy back to the EM-thruster regime. And a very brief review of electromagnetic principles. The first is, that when one reflects an electromagnetic wave, there will be momentum transfer orthogonal to the net moment vector change. Fancy wording, but what it means is this. IF you reflect an electromagnetic wave 180°, or exactly backwards at 100% efficiency (no loss), the reflection will produce a force F which equal to 2E/c. (E = energy, c is speed of light). 1 joule of reflected energy will produce a force of 6.67×10⁻⁹ newton-seconds.
If you want to quote people who explain physics to you, I'd caution you to first check that they understand some physics. Because what is written there is nonsense. It's not even dimensionally correct.

Please begin all over again, this time using  F = P/c
Title: Re: EM Drive Developments - related to space flight applications - Thread 2
Post by: sanman on 05/06/2015 03:15 am
I hear you, but the main salient thing for me is that the EMdrive is supposed to be a photon rocket - the resonant cavity is not a closed chamber but is actually a chamber with a hole in it (the waveguide end), just as the combustion chamber of a conventional rocket has a hole in it.

So photons are shooting out one end of the EMdrive, just as photons would shoot out one end of a photon rocket, just as propellant shoots out the end of a conventional chemical rocket. What differentiates EMdrive from a traditional photon rocket, is that the photons have been drained of a lot more momentum due to bouncing around in the waveguide first before exiting. This is analogous to the way an expansion nozzle and bell allow the propellant exhaust particles to bounce around more and transfer more of their momentum to the rocket before exiting.

So maybe it's best to start with the picture of a photon rocket in your head, since a photon rocket doesn't violate Conservation of Momentum.
Title: Re: EM Drive Developments - related to space flight applications - Thread 2
Post by: deltaMass on 05/06/2015 03:25 am
If I pretend that the EmDrive is a photon rocket, for which the thrust F = P/c, then the value of k in N/W = 1/c = 3.3 10-9 N/W. That is at least 3 orders smaller than the experimentally determined values of k that we've seen. And yet you wish to model the EmDrive as a diluted form (in some sense I do not understand) of a photon rocket? That would seem to imply a k value lower than 1/c, but in fact we see a higher one.

I can't bring myself to agree.

And about this "exhaust hole" of yours: this implies that the placement of the hole (or the waveguide power feed, you assert) must align with the resultant thrust vector. But experiment shows that this is not the case.

Again, I cannot agree with you.
Title: Re: EM Drive Developments - related to space flight applications - Thread 2
Post by: sanman on 05/06/2015 03:45 am
Alright, but I want to ask about theory first: in principle, is it possible to have a classical photon rocket fitted with an optical equivalent of the expander nozzle and bell? Can a waveguide be used as an optical equivalent of expander nozzle and bell, allowing photons to be drained of more momentum than they otherwise would when exiting a photon rocket? Because that part doesn't seem like it's violating Conservation of Momentum or any established physical laws.

So if a chemical rocket with nozzle and bell is more efficient than a chemical rocket without nozzle and bell, then imagine a device which is analogously trying to be more efficient than a classical photon rocket in a similar way. Is that idea by itself a violation of any established mainstream physics?

If it is not, then how to build such a device?
Title: Re: EM Drive Developments - related to space flight applications - Thread 2
Post by: deltaMass on 05/06/2015 03:49 am
The power input P dictates the available thrust. If the steady state thrust exceeds P/c, then you're violating conservation law.

But if I were you I'd disavow myself of the photon rocket model entirely, for the reasons I've given. It's a dead end.
Title: Re: EM Drive Developments - related to space flight applications - Thread 2
Post by: ppnl on 05/06/2015 03:59 am
As far as I can tell, I am not the only one to have thought to a general relativity effect here. This paper (http://arxiv.org/abs/1302.5690) uses a modified version of Einstein equations to explain what is going on in a conical resonant cavity without violating momentum conservation. The point is that the author uses a weak perturbation approximation and I do not know if this is fully justified.

Yeah...

No.

The blasted thing goes faster and that is an increase in momentum. Unless you show where the opposite momentum is then it violates conservation of momentum. It can't get any simpler than that.

Title: Re: EM Drive Developments - related to space flight applications - Thread 2
Post by: ThereIWas3 on 05/06/2015 04:02 am
Rather than using stiff coax to bring the microwaves onto the balance beam and to the chamber, it might be easier to actually mount the RF generator on the balance beam and then only bring line power to it, through a lightweight piece of stranded, unshielded, zip-cord (no thicker than 12 or 14 gauge).  That will be a lot more flexible than any kind of shielded cable.

You might also beam the power to the apparatus, through open air, at right angles to the pivot.  A feedhorn at the fulcrum picks up the power and connects to the waveguide leading to the frustrum.  No wires at all. The air gap could be just a few millimeters.  The trick is in avoiding reflections from the impedance discontinuity and cooking your magnetron.

One of my most enjoyable college courses, back when the world was young, was the 3rd section of Electromagnetic Field Theory, taught by Dr John Kraus (https://en.wikipedia.org/wiki/John_D._Kraus) himself.  He made it not only interesting, but fun, using a Socratic teaching method.  It was really cool having one of the leading authorities on antenna design teaching a course on antenna and waveguide design to us undergraduates.
Title: Re: EM Drive Developments - related to space flight applications - Thread 2
Post by: deltaMass on 05/06/2015 04:09 am
As far as my experience with exotic propulsion devices goes (20 years), the best way of all to construct a debunk-free apparatus to unambiguously demonstrate thrust is to make the whole thing completely self-contained.

That means battery power and the whole kit and kaboodle sealed in a box.
Accept no substitutes.
Title: Re: EM Drive Developments - related to space flight applications - Thread 2
Post by: sanman on 05/06/2015 04:10 am
The blasted thing goes faster and that is an increase in momentum. Unless you show where the opposite momentum is then it violates conservation of momentum. It can't get any simpler than that.

Well, if EMdrive is a photon rocket, then the opposite momentum is in the photons that are leaving. Except that photons with lower momentum are just photons with lower energy. So if you want to blame something for violating Conservation of Momentum, then why not blame the lousy photons for becoming fainter instead of behaving more intuitively like particles of matter do?

Just quoting from EMdrive's site:

http://www.emdrive.com/

Quote
Satellite Propulsion Research Ltd (SPR Ltd) a small UK based company, has demonstrated a remarkable new space propulsion technology. The company has successfully tested both an experimental thruster and a demonstrator engine which use patented microwave technology to convert electrical energy directly into thrust. No propellant is used in the conversion process. Thrust is produced by the amplification of the radiation pressure of an electromagnetic wave propagated through a resonant waveguide assembly.
Title: Re: EM Drive Developments - related to space flight applications - Thread 2
Post by: PushHigher on 05/06/2015 04:11 am
Warp Drive is a purely FTL thing and doesn't claim to have anything to do with conventional displacement or momentum as we know it. And neither Mach Effect nor EMdrive claim to be able to go FTL, even though they are claiming a novel mechanism for displacement.

Regarding FTL - can I beg to differ?

Everything in the universe warps space-time.  A rocket at rest warps space-time.  An accelerating rocket warps it more.

IMHO - warping space-time to infinity still brings us to the speed of light because the speed of gravity (the warp) is limited to c.  The EM Drive has nothing to do with FTL and I personally don't think FTL exists; in my mind the speed of light is the speed of reality.

http://en.wikipedia.org/wiki/Speed_of_gravity (http://en.wikipedia.org/wiki/Speed_of_gravity)
Title: Re: EM Drive Developments - related to space flight applications - Thread 2
Post by: ppnl on 05/06/2015 04:13 am
I hear you, but the main salient thing for me is that the EMdrive is supposed to be a photon rocket - the resonant cavity is not a closed chamber but is actually a chamber with a hole in it (the waveguide end), just as the combustion chamber of a conventional rocket has a hole in it.

So photons are shooting out one end of the EMdrive, just as photons would shoot out one end of a photon rocket, just as propellant shoots out the end of a conventional chemical rocket. What differentiates EMdrive from a traditional photon rocket, is that the photons have been drained of a lot more momentum due to bouncing around in the waveguide first before exiting. This is analogous to the way an expansion nozzle and bell allow the propellant exhaust particles to bounce around more and transfer more of their momentum to the rocket before exiting.

So maybe it's best to start with the picture of a photon rocket in your head, since a photon rocket doesn't violate Conservation of Momentum.

The thing is a photon rocket would require a huge number of photons that would probably burn down your lab. The power requirements would be monstrous. The only way to make a "practical" photon rocket is if you had a supply of antimatter. And it would be entirely useless for tooling around the solar system. It would only be used for interstellar travel.

Mix a great deal more matter in to reduce the energy of the exhaust and you could do interplanetary.
Title: Re: EM Drive Developments - related to space flight applications - Thread 2
Post by: sanman on 05/06/2015 04:15 am
Warp Drive is a purely FTL thing and doesn't claim to have anything to do with conventional displacement or momentum as we know it. And neither Mach Effect nor EMdrive claim to be able to go FTL, even though they are claiming a novel mechanism for displacement.

Regarding FTL - can I beg to differ?

Everything in the universe warps space-time.  A rocket at rest warps space-time.  An accelerating rocket warps it more.

IMHO - warping space-time to infinity still brings us to the speed of light because the speed of gravity (the warp) is limited to c.  The EM Drive has nothing to do with FTL and I personally don't think FTL exists; in my mind the speed of light is the speed of reality.

http://en.wikipedia.org/wiki/Speed_of_gravity (http://en.wikipedia.org/wiki/Speed_of_gravity)

But what I said was that EMdrive has nothing to do with Alcubierre Warp Drive or its proposed FTL. That was the point I was trying to make. On that, we seem to agree.
Title: Re: EM Drive Developments - related to space flight applications - Thread 2
Post by: sanman on 05/06/2015 04:27 am
So the crux of the issue for me, is that photons and matter particles behave differently, and are Apples & Oranges which cannot be judged by the same standards:

When you fully drain the momentum of a particle of propellant/matter (wrt your reference frame), then its velocity drops all the way down to stationary - but when you fully drain a photon of its momentum (wrt your reference frame), then it drops out of existence (ie. fades to nothing), because it loses energy rather than losing velocity.

So:

Photon Propellant != Matter Propellant

Matter is persistent, and you can see that exhaust hanging around after it's left the rocket.
Photons are not persistent, so they can disappear and leave you scratching your head about who/what was pushing your rocket to begin with.

Maybe the experimenters need to vary the angles of that waveguide, so that you have a series of cases where you tap greater and greater amounts of energy from the photons. The steeper and longer the waveguide, the more energy you're draining (transferring to your rocket/apparatus as momentum). The shallower the waveguide, the less energy you're draining (transferring to your rocket/apparatus as momentum).

Shouldn't the experimenters be trying to measure the energy of the photons that leave the resonant cavity via the waveguide exit? Wouldn't establishing some sort of correlation there be useful for proof of principle??
Title: Re: EM Drive Developments - related to space flight applications - Thread 2
Post by: warpmebro on 05/06/2015 04:34 am
Seems to me like too many people are worried about "laws" being "violated" instead of figuring out the new science behind this. Half the discussion here is about "this can't happen because of this "law" we came up with." Quit trying to fit within the "current model" of physics. Please, let me explain.

Throughout history, these "laws" have constantly changed. The earth is square, the earth is the center of the universe, (not the greatest examples but you get the idea) whatever the hell other dumb idea we can think of. On and on. And in each "era" they thought they knew it all. There's a few people here who know that we DON'T know it all, and these "laws" aren't really "laws" at all, it's ideas we've created to that fit our CURRENT UNDERSTANDING of the universe. Well guess what? We really don't know shit about the universe, honestly, and you should expect everything you know to change. That's the way it's been throughout history. It's a simple concept. Expect the unexpected, right?

We have an opportunity before us that nobody in history as been privileged with...worldwide collaboration via the internet, by the greatest minds of our age. Let's take advantage of it.

Maybe I'm just spitting out nonsense, and maybe I'm out of line, but I know one thing for sure...putting my ideas into writing isn't my strong-point, so take it for what it's worth.

The only physics I've studied is caveman physics, so this "idea" of mine is all I can "contribute", but I hope it can spark an idea or two, or a different way of thinking about all of this.
Title: Re: EM Drive Developments - related to space flight applications - Thread 2
Post by: PushHigher on 05/06/2015 04:41 am
So the crux of the issue for me, is that photons and matter particles behave differently, and are Apples & Oranges which cannot be judged by the same standards:

When you fully drain the momentum of a particle of propellant/matter (wrt your reference frame), then its velocity drops all the way down to stationary - but when you fully drain a photon of its momentum (wrt your reference frame), then it drops out of existence (ie. fades to nothing), because it loses energy rather than losing velocity.

So:

Photon Propellant != Matter Propellant

Matter is persistent, and you can see that exhaust hanging around after it's left the rocket.
Photons are not persistent, so they can disappear and leave you scratching your head about who/what was pushing your rocket to begin with.

Agreed.  If EM is just vibrations of space-time then EM is the (very small) waves of the ocean and mass is the boat.  The enclosed frustum with concave/convex ends bottles this up.

The reason I use the gravity gradient analogy so much is because if you imagine a wave pool that has a narrow and wide end - the waves would be stronger toward the narrow end and weaker toward the wide end.
Title: Re: EM Drive Developments - related to space flight applications - Thread 2
Post by: WarpTech on 05/06/2015 05:12 am
....

Todd,

It is exciting to read your explanation about exponentially decaying waves that have been squeezed beyond their cut-off diameter in the waveguide.  You write very clearly.

It's great that you found your way into this thread  :)

Please see the following reference (https://www.osapublishing.org/oe/fulltext.cfm?uri=oe-17-1-34&id=175583, click "Get PDF" to download the paper for free):

It is shown that all modes run continuously from travelling waves through a transition to an evanescent  (exponentially decaying) wave region and the value of the attenuation increases as they approach the cone vertex.

A strict distinction between pure travelling waves and pure evanescent  (exponentially decaying) waves cannot be achieved for conical waveguide.

One mode after the other reaches cutoff in the tapered hollow metallic waveguide as they approach the cone vertex.

Unfortunately, this analysis is for an open waveguide, not for a closed cavity, but the fact that a strict distinction between pure travelling waves and pure evanescent waves cannot be achieved for a conical waveguide, also has implications for modes approaching cutoff in the truncated cone cavity. 

ADDENDUM:

One thing that has not been explored is whether these truncated cones are being prematurely ended towards the cone vertex.  The tested designs are almost cylindrical.

Roger Shawyer has progressively (but very slowly with time) increased the cone angle of his truncated cones, culminating in the superconducting design he unveiled last October 2014.  NASA Eagleworks and Yang in China have truncated cone designs that look like earlier Shawyer designs, with smaller cone angles.


For reference. the tangent of the cone's half angle thetaw and the cone's half angle thetaw, in ascending order, for the following cases are:

(Notice how Shawyer progressively increased the cone's half-angle, with time, in his experimental designs, by a factor of 7 in the tangent of the half-angle)

Example (and geometry)                    { Tan[thetaw],thetaw (degrees) }

Shawyer Experimental                        {0.104019,   5.93851}
Shawyer Fligth Thruster                      {0.19086,   10.8055}
Shawyer Demo                                   {0.219054, 12.3557}
NASA Eagleworks frustum                   {0.263889, 14.7827}
Egan's example                                  {0.36397 ,  20}
Prof. Juan Yang  (2014)                      {0.4538,     24.4 }
Shawyer Superconducting 2014          {0.7002,     35}


(http://gregegan.customer.netspace.net.au/SCIENCE/Cavity/CavityShape.gif)



The people looking at running experiments here are (understandably) also looking at earlier designs with small cone angles and prematurely terminated before reaching the vertex.  The group from a university was looking at running experiments with perfectly cylindrical geometry.

Given the latest write-up by Shawyer (concerning his choice of R1 being too large) it appears that what you are bringing up concerning modes near cutoff has not yet been appreciated or explored, as the researchers are not focusing on general wave solutions in the complex plane containing simultaneously both real and imaginary components.

Best regards,   :)



PS: @aero has valiantly attempted to run a full analysis, of general waves, containing simultaneously both real and imaginary components, using MEEP.  Unfortunately MEEP is finite difference code and hence he has only been able to run 2-D simulations (due to computer time limitations).  It is known that this problem (truncated cone) is 3-D, as a 2-D analysis cannot simulate very important features of the geometry.  The MEEP solutions have also been very difficult to interpret, because of the lack of suitable post-processing software to explore the solutions.

@Mulletron envisioned a completely conical EM Drive, terminating at the vertex (at the beginning of the thread when we were all trying to explore all possibilities with an open mind).  A perfect cone may not be the best solution because in a perfect cone ALL modes are cut-off and hence there will not be any resonance, but in reality it is impossible to have a perfect cone, since the vertex will always terminate with a finite dimension (not a point).  I explored some time ago some geometries, and it looks like there is plenty of room to explore truncated cones that terminate at different distances from the cone vertex.

Thank you Dr. Rodal for a very informative post!

I've been studying the reference you provided, to Zeng and Fan. You cannot imagine how coincidental it is, but sometimes the universe works that way. Their equations 8 thru 11, are simple enough to understand without too much difficulty. These are "effectively" the same equations that govern gravity in the Engineering model of GR I work with, which is based on the PV Model. Gravity, as a refractive index, appears as the Damping function that governs the attenuation of the wave functions, and the ZPF acts as the Driving function that keeps it all afloat at "our" relative vacuum energy level. The two are in equilibrium, in what QED calls the fluctuation-dissipation relationship, and gravity is the asymmetry between the two that occurs wherever you have matter that filters the modes. It's pretty simple and intuitive to understand, but nobody seems to get it.

You asked about the truncated cones. From an engineering perspective, if it is not truncated it will have a difficult time resonating at any mode. The convex-concave end plates would seem to be necessary to maximize energy storage as spherical harmonics. So then, what modes do we want to attenuate? That would depend on what modes we can inject that will sustain resonance. It won't resonate when the angle is increased too much, but if we have attenuation factor equations from this paper, then I believe it can be modeled.

Getting back to the paper, based on their graphs for attenuation, it would seem a small angle is preferred. A large angle approximates a flat plate. Anything greater than pi/6 is not much better than bouncing photons off of a flat plate. However, for theta = pi/24, the attenuation is very high at much shorter wavelengths, and very high at longer wavelengths. It needs to strike a balance between energy storage and thrust at the modes available to us.

Again, gravity acts on the wave functions through the metric, transforming the (E,p) 4-vector. The metric is a refractive index. The effect on the wave function is equivalent to a Damping factor, in the damped harmonic oscillator equation. I see the attenuation factor in their plots as "similar" to that effect, acting on the microwaves in the cavity near the cut-offs. As the waves are attenuated, their momentum is absorbed as wave velocity goes to zero, just like light falling into a black hole. The result is propulsion. The bonus is that in such a space-time where the speed of light is variable, momentum conservation is dependent on the group velocity. It's not Newtonian anymore, because velocity is not a constant.

The light is being squeezed by the slowing of the group velocity, and since Energy is conserved, momentum must increase to compensate for reduced wave velocity. Another way to look at it is, photons in the waveguide "gain" an "effective mass". 

I see a lot of people arguing over photon rockets, despite the evidence that the thrust is orders of magnitude larger. No rocket nozzle is going to change that as long as the speed of light is considered to be constant, even if it captured all the energy from all the reflections produced. In order to get the thrust values they are seeing you must consider the reduction in wave velocity inside the waveguide, and that attenuation is asymmetrical, just like it is in a gravitational field.

Thanks again, for some very interesting new information.

Best Regards,
Todd D.





Title: Re: EM Drive Developments - related to space flight applications - Thread 2
Post by: ppnl on 05/06/2015 05:28 am
So the crux of the issue for me, is that photons and matter particles behave differently, and are Apples & Oranges which cannot be judged by the same standards:

When you fully drain the momentum of a particle of propellant/matter (wrt your reference frame), then its velocity drops all the way down to stationary - but when you fully drain a photon of its momentum (wrt your reference frame), then it drops out of existence (ie. fades to nothing), because it loses energy rather than losing velocity.

So:

Photon Propellant != Matter Propellant

Matter is persistent, and you can see that exhaust hanging around after it's left the rocket.
Photons are not persistent, so they can disappear and leave you scratching your head about who/what was pushing your rocket to begin with.

Maybe the experimenters need to vary the angles of that waveguide, so that you have a series of cases where you tap greater and greater amounts of energy from the photons. The steeper and longer the waveguide, the more energy you're draining (transferring to your rocket/apparatus as momentum). The shallower the waveguide, the less energy you're draining (transferring to your rocket/apparatus as momentum).

Shouldn't the experimenters be trying to measure the energy of the photons that leave the resonant cavity via the waveguide exit? Wouldn't establishing some sort of correlation there be useful for proof of principle??

What you are describing is just red shift. I think. It is very confused.

In an ordinary rocket the propellent has no momentum with respect to the rockets reference frame. You burn it and give it a huge momentum with respect to the rockets reference frame. In reaction the rocket gets a huge momentum in the other direction. In the original reference frame they both have huge momentum gain in opposit directions. That is how conservation of momentum works.

A photon rocket is not much different. A photon has some amount of momentum despite being massless. As the photon rocket accelerated away you would see the photons slowly red shifting. But you could not be "scratching your head about who/what was pushing your rocket" until it reached the speed of light which would be never. Instead you would be scratching your head wondering what vaporized the asteroid behind you.  Hint: the photons didn't just disappear. A photon rocket has an exhaust that is hotter and more destructive than any other kind of rocket giving the same thrust.
Title: Re: EM Drive Developments - related to space flight applications - Thread 2
Post by: ipcalit on 05/06/2015 05:37 am
Someone posted the theory of a warp bubble happening around the device as a means for propulsion. I recall seeing some interferometry setup with the half of a laser beam going through the device...

Did anybody test splitting the beam and sending each half very close (0.1mm) to the front and back of the device while keeping the device stationary in hard vacuum? This should be easy to do with the existing setup, just move some mirrors around.
Title: Re: EM Drive Developments - related to space flight applications - Thread 2
Post by: sanman on 05/06/2015 06:08 am
What you are describing is just red shift. I think. It is very confused.

In an ordinary rocket the propellent has no momentum with respect to the rockets reference frame. You burn it and give it a huge momentum with respect to the rockets reference frame. In reaction the rocket gets a huge momentum in the other direction. In the original reference frame they both have huge momentum gain in opposit directions. That is how conservation of momentum works.

A photon rocket is not much different. A photon has some amount of momentum despite being massless. As the photon rocket accelerated away you would see the photons slowly red shifting. But you could not be "scratching your head about who/what was pushing your rocket" until it reached the speed of light which would be never. Instead you would be scratching your head wondering what vaporized the asteroid behind you.  Hint: the photons didn't just disappear. A photon rocket has an exhaust that is hotter and more destructive than any other kind of rocket giving the same thrust.

But I'm not talking about light that the rocket-rider would see, I'm talking about light that the guy back on the launchpad would see. The launchpad guy would see the photon exhaust as red-shifted for a different reason, which is that the photons would have transferred some of their energy (momentum) to the rocket - ie. those photons are coming out of the rocket with lower energy than what they had when they were originally created inside the cavity.

That's not really a red-shift.
ie. energy transfer to a 3rd party (Mr Rocket) is not the same as a Relativistic Doppler effect.
Title: Re: EM Drive Developments - related to space flight applications - Thread 2
Post by: PushHigher on 05/06/2015 06:23 am
I tried to search for this but I think the thread search is broken because I was only seeing one result incorrectly.

Has anyone given any thought to using a a tapered electromagnet against a cylinder versus using a frustum?  Would this allow for similar attenuation while having the added benefit of being variable and tunable?
Title: Re: EM Drive Developments - related to space flight applications - Thread 2
Post by: sanman on 05/06/2015 06:23 am
If I pretend that the EmDrive is a photon rocket, for which the thrust F = P/c, then the value of k in N/W = 1/c = 3.3 10-9 N/W. That is at least 3 orders smaller than the experimentally determined values of k that we've seen. And yet you wish to model the EmDrive as a diluted form (in some sense I do not understand) of a photon rocket? That would seem to imply a k value lower than 1/c, but in fact we see a higher one.

I can't bring myself to agree.

And about this "exhaust hole" of yours: this implies that the placement of the hole (or the waveguide power feed, you assert) must align with the resultant thrust vector. But experiment shows that this is not the case.

Again, I cannot agree with you.

But the useful angle for the waveguide is the inverse of what would be the useful angle on a bell.

Light hitting the waveguide is producing momentum transfer in the same (parallel) direction as the photon emissions.
Exhaust from matter propellant is producing momentum transfer in the opposite (anti-parallel) direction as the exhaust stream.

It's because the mechanism of momentum transfer is different for a waveguide vs a bell.

Title: Re: EM Drive Developments - related to space flight applications - Thread 2
Post by: ppnl on 05/06/2015 06:25 am
What you are describing is just red shift. I think. It is very confused.

In an ordinary rocket the propellent has no momentum with respect to the rockets reference frame. You burn it and give it a huge momentum with respect to the rockets reference frame. In reaction the rocket gets a huge momentum in the other direction. In the original reference frame they both have huge momentum gain in opposit directions. That is how conservation of momentum works.

A photon rocket is not much different. A photon has some amount of momentum despite being massless. As the photon rocket accelerated away you would see the photons slowly red shifting. But you could not be "scratching your head about who/what was pushing your rocket" until it reached the speed of light which would be never. Instead you would be scratching your head wondering what vaporized the asteroid behind you.  Hint: the photons didn't just disappear. A photon rocket has an exhaust that is hotter and more destructive than any other kind of rocket giving the same thrust.

But I'm not talking about light that the rocket-rider would see, I'm talking about light that the guy back on the launchpad would see. The launchpad guy would see the photon exhaust as red-shifted for a different reason, which is that the photons would have transferred some of their energy (momentum) to the rocket - ie. those photons are coming out of the rocket with lower energy than what they had when they were originally created inside the cavity. That's not really a red-shift.

That's what I'm talking about. The guy on the launch pad would see ever more redshifted light as the rocket accelerated away. And the loss of the momentum of the redshifted photons would exactly match the gain in momentum of the photon rocket. If you did not see those photons then you are violating conservation of momentum. The redshifted photons are as necessary for the photon rocket as the drifting hot gasses of the regular rocket is to it. They are what conserves momentum.

Title: Re: EM Drive Developments - related to space flight applications - Thread 2
Post by: sanman on 05/06/2015 07:11 am
Hmm, so let me maybe replace the photon rocket idea with a solar sail.

So you've got a solar sail that is moving because photons are hitting it. But what if your photon source isn't some external sun/star, but is actually sitting onboard the ship attached to the solar sail? Then it sounds like you can't generate any net thrust, since whatever momentum your photons transfer to the sail is offset by the momentum that was lost when the photons popped out of your onboard emitter.

But suppose your sail could experience multiple collisions with each photon that came out of the emitter? So that's the waveguide that's allowing this to happen. And each of those collisions is transferring some momentum. So the sum total of all momentum that a photon can transfer to the waveguide can't exceed the original momentum of the photon when you first produced/emitted it. So net thrust is zero because of the anti-thrust from producing the original photon.

Hmm, so now it doesn't work again...

I better go ask GoatGuy.  :P
Title: Re: EM Drive Developments - related to space flight applications - Thread 2
Post by: ppnl on 05/06/2015 07:39 am
Hmm, so let me maybe replace the photon rocket idea with a solar sail.

So you've got a solar sail that is moving because photons are hitting it. But what if your photon source isn't some external sun/star, but is actually sitting onboard the ship attached to the solar sail? Then it sounds like you can't generate any net thrust, since whatever momentum your photons transfer to the sail is offset by the momentum that was lost when the photons popped out of your onboard emitter.

But suppose your sail could experience multiple collisions with each photon that came out of the emitter? So that's the waveguide that's allowing this to happen. And each of those collisions is transferring some momentum. So the sum total of all momentum that a photon can transfer to the waveguide can't exceed the original momentum of the photon when you first produced/emitted it. So net thrust is zero because of the anti-thrust from producing the original photon.

Hmm, so now it doesn't work again...

I better go ask GoatGuy.  :P

You get thrust when the photon is produced. With a solar sail or mirror you can change the direction of that thrust. Nothing will cancel out as long as you let the photon go into deep space.

Multiple reflections of a photon help nothing. You get as much thrust from that photon as you can ever hope to get simply by letting it go on its merry way. If you screw with it you are likely to get less thrust or none at all.

Let the light shine through.
Title: Re: EM Drive Developments - related to space flight applications - Thread 2
Post by: StrongGR on 05/06/2015 07:42 am
As far as I can tell, I am not the only one to have thought to a general relativity effect here. This paper (http://arxiv.org/abs/1302.5690) uses a modified version of Einstein equations to explain what is going on in a conical resonant cavity without violating momentum conservation. The point is that the author uses a weak perturbation approximation and I do not know if this is fully justified.

I had read that paper. Unfortunately, as the author of the paper states:

Quote
The weakest part of the theory seems to be that there is no clear way of preventing large gravitational effects due to the magnetic field of the Earth, as predicted by Eq. (17)

Therefore the author himself admits that his model is contradicted by predicting large gravitational effects due to the Earth's magnetic field that are contrary to all experimental evidence.

Yes, this is a drawback of that paper. What I am trying to understand is the reason why, beaming a laser through a EM Drive, there appears such an odd behavior with respect to the transit time, if confirmed. The only explanation that comes to mind is that some kind of warping is happening inside the device. This is not so strange as just an e.m. plane wave warps space-time even if by a very tiny (unmeasurable?) quantity. Now, it would be interesting to evaluate the full behavior of the frustum with respect to such laser beam propagation inside the cavity taking into account the Q factor, the frequency and the input power yielding the field intensity. This could be large enough to produce a meaningful and accessible number to the experimental setup conceived by Eagleworks.
Title: Re: EM Drive Developments - related to space flight applications - Thread 2
Post by: Spaniard on 05/06/2015 08:09 am
Multiple reflections of a photon help nothing.
Yeah. Because most of us are completely sure that newtons law are correct, the only way that EMDrive and similar devices could work is transferring momentum. What, where and how are the questions.

It must be something different to photons and it must exists before, because push photons will mean less pushing for this energy.
White talk about virtual particles. I thing that the form could be more exotic. Something like a partially stable perturbation of quantum vacuum. Something like a vortex in a fluid but in quantum form, like a field not locally binded to a particle.

Whatever it was, it must carry energy by itself so push it carries more momentum that pure photons.
If a photon is like a fish in water, these unknown "momentum carriers" must be like walls in water. It must push vacuum energy, so it carries more momentum in the same way as you push more mass for the same energy.
Title: Re: EM Drive Developments - related to space flight applications - Thread 2
Post by: sanman on 05/06/2015 08:26 am
Just looking at this EMdrive paper on force measurement:

http://www.emdrive.com/EmDriveForceMeasurement.pdf

Quote
The most important point to be made, is that to measure force, the cavity must
experience acceleration. In a fully restrained cavity, thrust and reaction force
cancel out.

Why is that?

Quote
This situation is unique to a propellantless thruster such as EmDrive and analogies
with conventional devices are pointless.

Why is that?

Quote
It therefore appears that a force measurement can only be made in a dynamic
environment, ideally by allowing the thruster to accelerate, measuring that
acceleration, and then calculating the thrust from T = -Ma

Why can force only be measured when it accelerates?
Is he saying that if there's no motion, there's no thrust?
Title: Re: EM Drive Developments - related to space flight applications - Thread 2
Post by: CW on 05/06/2015 08:37 am
Multiple reflections of a photon help nothing.
Yeah. Because most of us are completely sure that newtons law are correct, the only way that EMDrive and similar devices could work is transferring momentum. What, where and how are the questions.

It must be something different to photons and it must exists before, because push photons will mean less pushing for this energy.
White talk about virtual particles. I thing that the form could be more exotic. Something like a partially stable perturbation of quantum vacuum. Something like a vortex in a fluid but in quantum form, like a field not locally binded to a particle.

Whatever it was, it must carry energy by itself so push it carries more momentum that pure photons.
If a photon is like a fish in water, these unknown "momentum carriers" must be like walls in water. It must push vacuum energy, so it carries more momentum in the same way as you push more mass for the same energy.

I previously raised the question whether or not the standing EM wave within the cavity could be interpreted - as per quantum mechanics - as a confined (and possibly exotic) particle, due to the dualism of wave and particle. Perhaps some of our more prominent physicists could say something about this possibility? Is it correct to assume that this standing wave can be interpreted as a confined particle? Could we perhaps view a resonating EM wave cavity as a means to dynamically 'synthesize' or evoke 'exotic matter' ?
Title: Re: EM Drive Developments - related to space flight applications - Thread 2
Post by: Spaniard on 05/06/2015 09:08 am
Could we perhaps view a resonating EM wave cavity as a means to dynamically 'synthesize' or evoke 'exotic matter' ?
I think that the term "momentum carrier" is a good name for something that we really don't know.
Particles? Fields? Something more exotic?
A photon is easy to detect because interact with electrons. How could detect something with unknown properties that I suppose interact weakly to be undetected until now?
Title: Re: EM Drive Developments - related to space flight applications - Thread 2
Post by: sanman on 05/06/2015 09:22 am
Multiple reflections of a photon help nothing.
Yeah. Because most of us are completely sure that newtons law are correct, the only way that EMDrive and similar devices could work is transferring momentum. What, where and how are the questions.

It must be something different to photons and it must exists before, because push photons will mean less pushing for this energy.


The waveguide seems to be able to extract momentum from the photon in a way that can't be done with regular fermionic exhaust particles. The photon's energy and momentum are tied to its wavelength and are entirely accessible/exploitable through this, whereas that's not really true for fermionic exhaust particles, whose energy and momentum are tied to both velocity and intrinsic mass.

Since the conical waveguide is axisymmetric about the axis of thrust/acceleration/motion, it is able to bleed/suck energy (momentum) off the photon, in a way that can't occur when colliding with the other side of the cavity where there is no waveguide. Is this creating an inelastic collision for the photon on the waveguide side of the cavity only?
Title: Re: EM Drive Developments - related to space flight applications - Thread 2
Post by: TheTraveller on 05/06/2015 09:49 am
After rereading all the Shawyer papers and the web site pages several times I discovered Shawyer had defined the TC of his drive as TC = Q / (Fr 2 Pi).

http://www.emdrive.com/Toulouse2010paper01.doc

Example was Fr = 4GHz, Q = 50,000, TC = 2us.
So the time to effectively fully charge = TC x 5 = 10us.

For Shawyer's 1st system he had Fr = 2.45GHz, Q = 5,900, TC = 0.38us or 1.9us to fully charge.

I see this as saying a really high Q EM Drive could take a really long time to charge the cavity.

For his 1st superconducting cavity with Fr = 3.85GHz, Q = 6,800,000 = TC 280us = 1.4ms to fully charge.

Is probably why his space plane uses 8 cavities that are pulse cycled and phased apart as their Q could be 1,000 times higher with cavity charge times of over a second. If not smoothed out, could be a bumpy ride.

Pulse phasing discussed here:
http://s000.tinyupload.com/index.php?file_id=05426775513544255924
Title: Re: EM Drive Developments - related to space flight applications - Thread 2
Post by: RareSaturn on 05/06/2015 10:28 am
Hmm, so let me maybe replace the photon rocket idea with a solar sail.

So you've got a solar sail that is moving because photons are hitting it. But what if your photon source isn't some external sun/star, but is actually sitting onboard the ship attached to the solar sail? Then it sounds like you can't generate any net thrust, since whatever momentum your photons transfer to the sail is offset by the momentum that was lost when the photons popped out of your onboard emitter.

But suppose your sail could experience multiple collisions with each photon that came out of the emitter? So that's the waveguide that's allowing this to happen. And each of those collisions is transferring some momentum. So the sum total of all momentum that a photon can transfer to the waveguide can't exceed the original momentum of the photon when you first produced/emitted it. So net thrust is zero because of the anti-thrust from producing the original photon.

Hmm, so now it doesn't work again...

I better go ask GoatGuy.  :P

As far as I know, and I'm no expert, a solar sail can have the light source on board, simply because the photons that hit the sail impart their energey/momentum, and the ones that do not hit the sail, don't.  I once thought up a glowworm powered spacecraft working on the same principal.  A colony of glowworms lives in a conical structure that is highly reflective on the inner surface, except for the large end at the back which is transparent.  All the photons that hit the surface are reflected (so produce momentum), the ones that shoot out the back window are not so there is a gain in momentum forwards.  If only the suckers were immortal and didn't need feeding...
Title: Re: EM Drive Developments - related to space flight applications - Thread 2
Post by: Spaniard on 05/06/2015 10:44 am
Pulse phasing discussed here:
A interesting idea. That the EMDrive could be something like a "momentum capacitator", in form of photons inside the cavity. That must be accounted accurately, because is a good source of errors.
Momentum could be "transfered" in time. A lot of photons are accumulated inside and when the walls of EMDrive change the temperature, the light reflect more than before, making do absorbed at different rates on different walls of the EMDrive making the net momentum accumulated in the photons.
But if the photons are accumulated on some time and released on less time and measured in this smaller time window, the force could be accounted greater than really are.

This hypotesis would require a "charging time" on the EMDrive before the measurement.
Title: Re: EM Drive Developments - related to space flight applications - Thread 2
Post by: TheTraveller on 05/06/2015 10:53 am
Pulse phasing discussed here:
http://s000.tinyupload.com/index.php?file_id=05426775513544255924
A interesting idea. That the EMDrive could be something like a "momentum capacitator", in form of photons inside the cavity. That must be accounted accurately, because is a good source of errors.
Momentum could be "transfered" in time. A lot of photons are accumulated inside and when the walls of EMDrive change the temperature, the light reflect more than before, making do absorbed at different rates on different walls of the EMDrive making the net momentum accumulated in the photons.
But if the photons are accumulated on some time and released on less time and measured in this smaller time window, the force could be accounted greater than really are.

This hypotesis would require a "charging time" on the EMDrive before the measurement.

Charging time is per the cavity TC x 5 (see attached), which is frequency and Q dependent as per:

Charge time = 5 x TC.
TC = Q / (Fr 2 Pi)

BTW this is not an EM Drive specific resonate circuit TC calc.

So maybe a small delay between saying "Engage" and the ship starting to accelerate, even with a 2 x 4 straight cavity EM Drive engine? Maybe the new V8 like space engine?
Title: Re: EM Drive Developments - related to space flight applications - Thread 2
Post by: rrb6699 on 05/06/2015 11:12 am
hello all.  I have made a couple reply posts thus far and just found this site.  I'm very excited and fascinated about EM.  I hope to make some small contribution as I come "up to speed" on this subject.

if interested, please check my other posts for replies so I don't have to duplicate them here.  nice work on the discovery.  I have several questions forthcoming that will influence my posts that I reserve for now so as to not speak before I understand better.

regards!

Ray
Title: Re: EM Drive Developments - related to space flight applications - Thread 2
Post by: KelvinZero on 05/06/2015 11:15 am
So you've got a solar sail that is moving because photons are hitting it. But what if your photon source isn't some external sun/star, but is actually sitting onboard the ship attached to the solar sail? Then it sounds like you can't generate any net thrust, since whatever momentum your photons transfer to the sail is offset by the momentum that was lost when the photons popped out of your onboard emitter.
If the sail is reflective then it delivers twice the momentum of when the photon was initially released. Think of it like a baseball. When someone throws it and another catches it, the actions cancel. This is like absorbing the photon. If the catcher throws it back again as well (reflection) this is like a whole new throw. Luckily due to conservation of momentum we need not worry about the details, we can just look at before and after.

(edit: some might have misread this to say the reflection doubles effectiveness over just emitting a photon. I was trying to explain why emitting a photon and then reflecting it does not cancel to zero, but is instead just like emitting a photon in the other direction)
Title: Re: EM Drive Developments - related to space flight applications - Thread 2
Post by: TheTraveller on 05/06/2015 11:33 am
So you've got a solar sail that is moving because photons are hitting it. But what if your photon source isn't some external sun/star, but is actually sitting onboard the ship attached to the solar sail? Then it sounds like you can't generate any net thrust, since whatever momentum your photons transfer to the sail is offset by the momentum that was lost when the photons popped out of your onboard emitter.
If the sail is reflective then it delivers twice the momentum of when the photon was initially released. Think of it like a baseball. When someone throws it and another catches it, the actions cancel. This is like absorbing the photon. If the catcher throws it back again as well (reflection) this is like a whole new throw. Luckily due to conservation of momentum we need not worry about the details, we can just look at before and after.
The ball is not caught and thrown back. Instead it bounces off the catcher's glove, transferring to it the momentum gained from the 1st thrower. So no overall momentum gain.
Title: Re: EM Drive Developments - related to space flight applications - Thread 2
Post by: deltaMass on 05/06/2015 12:31 pm
I find these descriptions all rather imprecise. Let's do this with a laser first.
We begin with zero momentum. The laser emits a photon bunch of momentum +p forwards towards the sail, and the laser recoils with -p. Momentum conserved because +p + (-p) = 0, which is was originally. Now the photon bunch hits the sail with +p and rebounds (100% reflectivity) with -p. So the bunch experiences a momentum change of -2p = -p - (+p) and so the sail experiences an equal and opposite change of +2p forwards. But shortly before the attached laser had received -p, so the sail+laser combination gets a net +p of forward momentum. The bunch is travelling backwards with -p. Total momentum is conserved.

So yes, Victoria, you can blow into your own sail.

But notice we could have obtained the same result with the laser simply pointing backwards, acting like a photon rocket. +p for laser, -p for photons, as before. But we could have ditched the sail in this case, thus reducing the all-up mass and getting more acceleration. So a sail is a dumb idea when you have a laser.

Now repeat with an isotropic radiator like a glowing ball of light. This time a reflector is definitely needed, otherwise nothing moves, there being no preferred direction. The best reflector arrangement is to place the globe at the focus of a paraboloid mirror. Once again we get forward momentum for globe+reflector, as we still have an exhaust. But a little thought will show that it's not as efficient as the laser, because some energy is lost to the sides.
Title: Re: EM Drive Developments - related to space flight applications - Thread 2
Post by: Fugudaddy on 05/06/2015 01:31 pm
Okay, so this is a totally utterly, fascinating, and amazing place to be right now. Thank you all for the amazing science that's being done right here in front of our eyes. If this thing does actually pan out into something 'real', then being here, now, at the edge of this discovery is a heady place to be.

IANAS (I am not a scientist), the advanced maths are too hard for my brain. But I do have a good head for the theory of all of this. I've followed along with this whole thread but hadn't seen any discussion on a particular aspect (the Quantum Vacuum) that was in the NSF article.

So as I understand the EM drive:
EM radiation is pumped into a particularly shaped container which causes reflections and resonance as the waves bounce around. Something in the way those waves bounce causes particles to become more energized in one direction over another direction, resulting in, basically, thrust.

Yeah, utterly simplified.

So there seems to be some debate about a few things that seemed obvious to me, but hey, let's see how good my understanding really is.

There is no way that this could be a 'free energy' system, simply because the container is not absolute. The waves lose energy either through the vessel or through momentum exchange with the walls/interior particles, so more energy is always required to be 'pumped' in from the outside. Correct?

There is also no way that this could violate FTL, so far as I understand it, since as the container itself approaches C, it is no longer quite the same 'closed system' as it was before. I would guess as the whole system approaches C, the way the EM radiation bounces changes enough that the effects that are being generated would end up changing enough to stop doing whatever it is they're doing now.

So here's a theory, such as it is. The EM waves are 'cut' and 'squeezed' by the shape of the vessel. This (appears) to have some effect on gravity/space time (hence the confusion regarding 'warp' drives), as the concentration/focus of the EM waves is increased with each subsequent 'bounce' in the chamber.

The image I had in mind was of a jet, flying low over water. The disturbance of the airflow causes a specific shape (a 'rooster tail') to form behind the jet as the water particles are 'lifted' from the surface by the energy of the air.

Could there be some quantum mechanism that's doing something similar- the compressed waves somehow 'perturbing' the Quantum Vacuum and pushing (real or virtual) particles into a higher energy state, (preserving the momentum law), which then transfers upwards eventually resulting in particles that are in the slightly more excited state in the direction of the 'pull' of the compressed wave travelling by.

Either way, this is a fascinating place to be. Thank you for all the science :D
 
Title: Re: EM Drive Developments - related to space flight applications - Thread 2
Post by: KelvinZero on 05/06/2015 01:48 pm
So here's a theory, such as it is. The EM waves are 'cut' and 'squeezed' by the shape of the vessel.
It does look a bit like a juicer ;)
Title: Re: EM Drive Developments - related to space flight applications - Thread 2
Post by: Rodal on 05/06/2015 02:38 pm
Regarding the main controller parameter for the EM Drive

It is my understanding, from what Paul March wrote, that the main controlling parameter in determining the thrust generation performance of the EM-Drive is the rate of phase modulation of the RF signal that is injected into the resonant cavity. 

This requires an FM modulated signal of around 100 kHz deviation that dithers back and forth around the resonant cavity's resonant frequency as fast as possible. 

One has to feed the cavity through a 2 foot long RG-8 cable with Type-N connectors with the other end of this coax connected to a 3-Stub tuner that is used to set the 50 ohm Smith Chart Z-matching circle to an impedance solution that matches the 50 ohm load AND generates the narrowest +/-90 degree capacitive to inductive reactive phase change bandwidth in the cavity.  This phase change bandwidth needs to be 10 kHz or less and preferable less than 4 kHz. 

This information is useful to correctly and consistently tune an EM-drive prototype for maximum thrust.

If anybody has a different understanding of what he wrote, it would be useful to know.
Title: Re: EM Drive Developments - related to space flight applications - Thread 2
Post by: TheTraveller on 05/06/2015 02:54 pm
Regarding the main controller parameter for the EM Drive

It is my understanding, from what Paul March wrote, that the main controlling parameter in determining the thrust generation performance of the EM-Drive is the rate of phase modulation of the RF signal that is injected into the resonant cavity. 

This requires an FM modulated signal of around 100 kHz deviation that dithers back and forth around the resonant cavity's resonant frequency as fast as possible. 

One has to feed the cavity through a 2 foot long RG-8 cable with Type-N connectors with the other end of this coax connected to a 3-Stub tuner that is used to set the 50 ohm Smith Chart Z-matching circle to an impedance solution that matches the 50 ohm load AND generates the narrowest +/-90 degree capacitive to inductive reactive phase change bandwidth in the cavity.  This phase change bandwidth needs to be 10 kHz or less and preferable less than 4 kHz. 

This information is useful to correctly and consistently tune an EM-drive prototype for maximum thrust.

If anybody has a different understanding of what he wrote, it would be useful to know.
Thought Paul's latest pathway was to duplicate the 1st Shawyer teeter-totter test rig and blast away with a wide band magnetron and waveguide to feed the cavity. Hope they add the ability to manually adjust / tune the cavity length as Shawyer did before he went to an active feedback loop to lock the narrow band RF to the cavity resonance in the curved end plates Flight Thruster.
Title: Re: EM Drive Developments - related to space flight applications - Thread 2
Post by: Notsosureofit on 05/06/2015 02:59 pm
My understanding was that he could set the match to obtain the maximum phase change across the resonance line.   This would give him the maximum effective Q.  The FM dither would be a means of following the resonance.  Back when, I used a double balanced modulator to find the 2 half-power points as the most accurate way to identify the center frequency. (but I didn't need a carrier there, I was measuring linewidth) AM or FM can be used the same way and in these cases you have power at the center of the resonance.
Title: Re: EM Drive Developments - related to space flight applications - Thread 2
Post by: TheTraveller on 05/06/2015 03:08 pm
My understanding was that he could set the match to obtain the maximum phase change across the resonance line.   This would give him the maximum effective Q.  The FM dither would be a means of following the resonance.  Back when, I used a double balanced modulator to find the 2 half-power points as the most accurate way to identify the center frequency. (but I didn't need a carrier there, I was measuring linewidth) AM or FM can be used the same way and in these cases you have power at the center of the resonance.
Was this with or without a dielectric inside the cavity?

If EW do as they have said and replicate Shawyer & the Chinese, then they will not be using dielectrics.
http://forum.nasaspaceflight.com/index.php?topic=36313.msg1355764#msg1355764
Title: Re: EM Drive Developments - related to space flight applications - Thread 2
Post by: zlspradlin on 05/06/2015 03:41 pm
Like most here I'm fascinated and intrigued. However, I'm terrible at this type of math and better at making things. The device seems kind of simple on the surface, but I must assume I'm missing something (many things).

Have any science hobbyists built their own machine yet? I can't imagine that only three people have done this so far, and I'm working with a peer to design our own and build it this summer.

I'm still reading and re-reading this thread, and I'm not too many pages into it. So if someone has posted their experiments I'm sure I'll run across them sooner or later, but if anyone knows of any build logs or hobbyists making a drive I'd love to see what they did and how it worked.

I do have a question for those who may know, what do you think is the biggest hurdle to building a functioning EM Drive?
Title: Re: EM Drive Developments - related to space flight applications - Thread 2
Post by: bprager on 05/06/2015 03:56 pm
Like most here I'm fascinated and intrigued. However, I'm terrible at this type of math and better at making things. The device seems kind of simple on the surface, but I must assume I'm missing something (many things).

Have any science hobbyists built their own machine yet? I can't imagine that only three people have done this so far, and I'm working with a peer to design our own and build it this summer.

I'm still reading and re-reading this thread, and I'm not too many pages into it. So if someone has posted their experiments I'm sure I'll run across them sooner or later, but if anyone knows of any build logs or hobbyists making a drive I'd love to see what they did and how it worked.

I do have a question for those who may know, what do you think is the biggest hurdle to building a functioning EM Drive?
How about some open source design? openscad (http://www.openscad.org/) has been mentioned before. We could put the STL files on the wiki (http://emdrive.echothis.com/index.php/Main_Page).
Title: Re: EM Drive Developments - related to space flight applications - Thread 2
Post by: TheTraveller on 05/06/2015 04:01 pm
Like most here I'm fascinated and intrigued. However, I'm terrible at this type of math and better at making things. The device seems kind of simple on the surface, but I must assume I'm missing something (many things).

Have any science hobbyists built their own machine yet? I can't imagine that only three people have done this so far, and I'm working with a peer to design our own and build it this summer.

I'm still reading and re-reading this thread, and I'm not too many pages into it. So if someone has posted their experiments I'm sure I'll run across them sooner or later, but if anyone knows of any build logs or hobbyists making a drive I'd love to see what they did and how it worked.

I do have a question for those who may know, what do you think is the biggest hurdle to building a functioning EM Drive?
DO NOT reinvent the wheel.

Follow Shawyer as close as you can.

Plenty of clues but you need to do a lot of reading.

Checkout Mulletron's device & photos:
https://drive.google.com/folderview?id=0B4PCfHCM1KYoTXhSUTd5ZDN2WnM&usp=sharing

Mulletron has links where you can buy a cavity the same size as used by EW. PM him for details.

Read all of Mulletron & Star-Drives posts and attachments:
http://forum.nasaspaceflight.com/index.php?action=profile;area=showposts;u=45378
http://forum.nasaspaceflight.com/index.php?action=profile;area=showposts;u=2074

Be a KISS replicator (Keep It Simple Stupid)

Read the Wiki:
http://emdrive.echothis.com/index.php/Main_Page

I'm also in the early planning stage of replicating the 1st Shawyer test rig and cavity. Will put up a log, photos & drawings as I progress.

Good luck
Title: Re: EM Drive Developments - related to space flight applications - Thread 2
Post by: LasJayhawk on 05/06/2015 04:03 pm
...

I do have a question for those who may know, what do you think is the biggest hurdle to building a functioning EM Drive?
Funding.  :(
Title: Re: EM Drive Developments - related to space flight applications - Thread 2
Post by: TheTraveller on 05/06/2015 04:05 pm
Like most here I'm fascinated and intrigued. However, I'm terrible at this type of math and better at making things. The device seems kind of simple on the surface, but I must assume I'm missing something (many things).

Have any science hobbyists built their own machine yet? I can't imagine that only three people have done this so far, and I'm working with a peer to design our own and build it this summer.

I'm still reading and re-reading this thread, and I'm not too many pages into it. So if someone has posted their experiments I'm sure I'll run across them sooner or later, but if anyone knows of any build logs or hobbyists making a drive I'd love to see what they did and how it worked.

I do have a question for those who may know, what do you think is the biggest hurdle to building a functioning EM Drive?
How about some open source design? openscad (http://www.openscad.org/) has been mentioned before. We could put the STL files on the wiki (http://emdrive.echothis.com/index.php/Main_Page).
Could add Mulletron's GDrive:
https://drive.google.com/folderview?id=0B4PCfHCM1KYoTXhSUTd5ZDN2WnM&usp=sharing
Cavity specs there plus he can point you to a cavity source/supplier if you wish to buy ready made. His photos of his progress are excellent.
Title: Re: EM Drive Developments - related to space flight applications - Thread 2
Post by: TheTraveller on 05/06/2015 04:08 pm
...

I do have a question for those who may know, what do you think is the biggest hurdle to building a functioning EM Drive?
Funding.  :(
My budget is $500. Should be enough but I have a good workshop & cabinet load of electronics. Plus a lot of self build & many hours of blood, sweat and tears.
Title: Re: EM Drive Developments - related to space flight applications - Thread 2
Post by: Rodal on 05/06/2015 04:11 pm
...
DO NOT reinvent the wheel.

Follow Shawyer as close as you can.

Plenty of clues but you need to do a lot of reading.

....

You are writing that in reference to Shaywer's experiments, I presume, but a number of Shawyer's prescriptions are tied to his theoretical model.  I read the paper that Shawyer sent to Mulletron, to support Shawyer's theoretical model.

I wrote a review of this paper here:  http://forum.nasaspaceflight.com/index.php?topic=36313.msg1369861#msg1369861

Showing that there is nothing in that paper (by Cullen) supporting Shawyer's theoretical model.  On the contrary, it follows the same Maxwell's equations and laws followed by Greg Egan http://gregegan.customer.netspace.net.au/SCIENCE/Cavity/Cavity.html  who concludes that the EM Drive should have no thrust force.

Title: Re: EM Drive Developments - related to space flight applications - Thread 2
Post by: zlspradlin on 05/06/2015 04:23 pm
Awesome, such fantastic responses so quickly! Thanks for the great links TheTraveller.

As for budget, I think that I might be ok in that regard, I'm afforded a few resources that a lot of hobbyists aren't, and thanks to my wife I have a great network of makers & craftsmen of all types from fine artists to industrial designers.

And I will absolutely provide STLs and anything else we create during this venture to the community. And of course I'll provide many updates whenever we get started.

I don't want to get too terribly off topic, but I just had a thought... will microwaves work with the boundary layer effect? Could a microwave rotate a Tesla Turbine?
Title: Re: EM Drive Developments - related to space flight applications - Thread 2
Post by: TheTraveller on 05/06/2015 04:24 pm
...
DO NOT reinvent the wheel.

Follow Shawyer as close as you can.

Plenty of clues but you need to do a lot of reading.

....

You are writing that in reference to Shaywer's experiments, I presume, but a number of Shawyer's prescriptions are tied to his theoretical model.  I read the paper that Shawyer sent to Mulletron, to support Shawyer's theoretical model.

I wrote a review of this paper here:  http://forum.nasaspaceflight.com/index.php?topic=36313.msg1369861#msg1369861

Showing that there is nothing in that paper (by Cullen) supporting Shawyer's theoretical model.  On the contrary, it follows the same Maxwell's equations and laws followed by Greg Egan http://gregegan.customer.netspace.net.au/SCIENCE/Cavity/Cavity.html  who concludes that the EM Drive should have no thrust force.

Did Shawyer send the paper by Cullen by mistake, and he meant to send another paper instead to support his theory?

Do you know of any paper supporting Shawyer's theoretical model ?
I'm an engineer that designs, builds, commissions & teaches others how to maintain what I designed and built, while being very willing to pick up a tool bag and get grease up to my arm pits to get things back working and into service.

I look at tests of stuff that are claimed to work. If they pass my gut test, I may used them to guide my replication and test process. I try to limit reinventing the wheel.

Everything I read from Shawyer about how the devices interact with / work in the physical world, supports his many & various claims & statements & builds a strong model that they do indeed work.

To settle the matter if the EM Drive works or not, for me, demands that I do a replication, as close to the work Shawyer has done as possible. Which is what I'm planning to do.
Title: Re: EM Drive Developments - related to space flight applications - Thread 2
Post by: Rodal on 05/06/2015 04:34 pm
...
DO NOT reinvent the wheel.

Follow Shawyer as close as you can.

Plenty of clues but you need to do a lot of reading.

....

You are writing that in reference to Shaywer's experiments, I presume, but a number of Shawyer's prescriptions are tied to his theoretical model.  I read the paper that Shawyer sent to Mulletron, to support Shawyer's theoretical model.

I wrote a review of this paper here:  http://forum.nasaspaceflight.com/index.php?topic=36313.msg1369861#msg1369861

Showing that there is nothing in that paper (by Cullen) supporting Shawyer's theoretical model.  On the contrary, it follows the same Maxwell's equations and laws followed by Greg Egan http://gregegan.customer.netspace.net.au/SCIENCE/Cavity/Cavity.html  who concludes that the EM Drive should have no thrust force.
I'm an engineer that designs, builds, commissions & teaches others how to maintain what I designed and built, while being very willing to pick up a tool bag and get grease up to my arm pits to get things back working and into service.

I look at tests of stuff that are claimed to work. If they pass my gut test, I may used them to guide my replication and test process. I try to limit reinventing the wheel.

Everything I read from Shawyer about how the devices interact with / work in the physical world, supports his many & various claims & statements & builds a strong model that they do indeed work.

To settle the matter if the EM Drive works or not, for me, demands that I do a replication, as close to the work Shawyer has done as possible. Which is what I'm planning to do.

OK, here is one practical thing that researchers can adopt, based on Cullen's paper (and 140 years since Maxwell, with people trying to perform experimental measurements of radiation pressure, which are plagued by air convection currents), to avoid getting false-positives:

It was impossible to obtain a stable baseline, even on a relatively short-term basis of a minute's duration.  This continual drifting of the baseline was found to be due to air convection currents set up by small and changing temperature gradients within the microwave waveguides.  The remedy was to reduce the air resistance of the reflecting end plate so that the convection currents would have no appreciable effect.  The reflecting end plate was replaced by a system of concentric wire rings (shown on Fig. 12 of Cullen's paper).  The rings acted as an almost perfect reflector of the electromagnetic waves but at the same time had a small effective cross-section to air currents.  NASA, Shawyer, Yang, and other EM Drive researchers would be well advised to experiment with replacing the end plates of the EM Drive with this system of concentric rings, in order to address the problem of air convection currents that has plagued radiation pressure experiments in ambient conditions ever since Maxwell 140 years ago.
Title: Re: EM Drive Developments - related to space flight applications - Thread 2
Post by: PaulF on 05/06/2015 04:35 pm
Question to all:

Has quantum tunneling been incorporated in any calculations? I read once that at least some photons will tunnel through the medium of the cavity walls. Hope this sparks something in someone's mind.
Title: Re: EM Drive Developments - related to space flight applications - Thread 2
Post by: TheTraveller on 05/06/2015 05:33 pm
OK, here is one practical thing that researchers can adopt, based on Cullen's paper (and 140 years since Maxwell, with people trying to perform experimental measurements of radiation pressure, which are plagued by air convection currents), to avoid getting false data:

It was impossible to obtain a stable baseline, even on a relatively short-term basis of a minute's duration.  This continual drifting of the baseline was found to be due to air convection currents set up by small and changing temperature gradients within the microwave waveguides.  The remedy was to reduce the air resistance of the reflecting end plate so that the convection currents would have no appreciable effect.  The reflecting end plate was replaced by a system of concentric wire rings (shown on Fig. 12 of Cullen's paper).  The rings acted as an almost perfect reflector of the electromagnetic waves but at the same time had a small effective cross-section to air currents.  NASA, Shawyer, Yang, and other EM Drive researchers would be well advised to experiment with replacing the end plates of the EM Drive with this system of concentric rings, in order to address the problem of air convection currents that has plagued radiation pressure experiments in ambient conditions ever since Maxwell 140 years ago.
Thanks for the feedback.

My replication plan is to build a Teeter-Totter balance system as Shawyer used in his 1st Feasibility device tests.

Cavity will be a copper replication of the narrow band Flight Thruster, with spherical concave/convex end plates.

RF frequency control will be via multiple stub antenna internal sense feedbacks loops (as Shawyer used with his Flight Thruster, 1 RF generator not 2), multiple on device / ambient temp sensors and RF power amp current, will all data logged including variations in the reference 250g preset downward force.

If after several months and much consulting with other replicators, there is not the slightest indication of thrust from any of the replicators, then time to consider other pathways.

If significant thrust is detected, a transparent box will be built around the test device and balance rig and the test protocols repeated. If there is still significant thrust detected, discussion will proceed to obtain, at my cost, independent verification. I do believe EarthTech and other such test labs may be interested.

I feel there is enough info now available to do a series of independent tests and draw a line in the sand. Either it works or not. If the independent lab results shows thrust, it is time to "Engage" and grow well beyond this dirt ball.

Of course, assuming it works, someone needs to figure out how it works.

Full plans and BOM (Bill of Material & suppliers) will be progressively be made available over the internet.
Title: Re: EM Drive Developments - related to space flight applications - Thread 2
Post by: zlspradlin on 05/06/2015 05:56 pm
The concentric rings at the end plates has sparked an idea for me. Perhaps there is a "perfect" shape for these waves to create the most thrust, couldn't this be manipulated with a holographic film? Could you not pass microwaves through a hologram to reshape the waves into the most efficient form? I suppose we need to know more about how this thing actually works before we can know the best methods/shapes. This doesn't address the air currents, but could address reflecting the waves in a more efficient way.

Holograms are not inaccessible, I've made plenty of them and even took a course on lasers & holography in college. The cost of a microwave hologram, however, I've no idea.
Title: Re: EM Drive Developments - related to space flight applications - Thread 2
Post by: Rodal on 05/06/2015 06:19 pm
.....

Please see the following reference (https://www.osapublishing.org/oe/fulltext.cfm?uri=oe-17-1-34&id=175583, click "Get PDF" to download the paper for free):

It is shown that all modes run continuously from travelling waves through a transition to an evanescent  (exponentially decaying) wave region and the value of the attenuation increases as they approach the cone vertex.

A strict distinction between pure travelling waves and pure evanescent  (exponentially decaying) waves cannot be achieved for conical waveguide.

One mode after the other reaches cutoff in the tapered hollow metallic waveguide as they approach the cone vertex.

.....

Thank you Dr. Rodal for a very informative post!

I've been studying the reference you provided, to Zeng and Fan. You cannot imagine how coincidental it is, but sometimes the universe works that way. Their equations 8 thru 11, are simple enough to understand without too much difficulty. These are "effectively" the same equations that govern gravity in the Engineering model of GR I work with, which is based on the PV Model. Gravity, as a refractive index, appears as the Damping function that governs the attenuation of the wave functions, and the ZPF acts as the Driving function that keeps it all afloat at "our" relative vacuum energy level. The two are in equilibrium, in what QED calls the fluctuation-dissipation relationship, and gravity is the asymmetry between the two that occurs wherever you have matter that filters the modes. It's pretty simple and intuitive to understand, but nobody seems to get it.

You asked about the truncated cones. From an engineering perspective, if it is not truncated it will have a difficult time resonating at any mode. The convex-concave end plates would seem to be necessary to maximize energy storage as spherical harmonics. So then, what modes do we want to attenuate? That would depend on what modes we can inject that will sustain resonance. It won't resonate when the angle is increased too much, but if we have attenuation factor equations from this paper, then I believe it can be modeled.

Getting back to the paper, based on their graphs for attenuation, it would seem a small angle is preferred. A large angle approximates a flat plate. Anything greater than pi/6 is not much better than bouncing photons off of a flat plate. However, for theta = pi/24, the attenuation is very high at much shorter wavelengths, and very high at longer wavelengths. It needs to strike a balance between energy storage and thrust at the modes available to us.

Again, gravity acts on the wave functions through the metric, transforming the (E,p) 4-vector. The metric is a refractive index. The effect on the wave function is equivalent to a Damping factor, in the damped harmonic oscillator equation. I see the attenuation factor in their plots as "similar" to that effect, acting on the microwaves in the cavity near the cut-offs. As the waves are attenuated, their momentum is absorbed as wave velocity goes to zero, just like light falling into a black hole. The result is propulsion. The bonus is that in such a space-time where the speed of light is variable, momentum conservation is dependent on the group velocity. It's not Newtonian anymore, because velocity is not a constant.

The light is being squeezed by the slowing of the group velocity, and since Energy is conserved, momentum must increase to compensate for reduced wave velocity. Another way to look at it is, photons in the waveguide "gain" an "effective mass". 

I see a lot of people arguing over photon rockets, despite the evidence that the thrust is orders of magnitude larger. No rocket nozzle is going to change that as long as the speed of light is considered to be constant, even if it captured all the energy from all the reflections produced. In order to get the thrust values they are seeing you must consider the reduction in wave velocity inside the waveguide, and that attenuation is asymmetrical, just like it is in a gravitational field.

Thanks again, for some very interesting new information.

Best Regards,
Todd D.

in the following quotation

Quote
Getting back to the paper, based on their graphs for attenuation, it would seem a small angle is preferred. A large angle approximates a flat plate. Anything greater than pi/6 is not much better than bouncing photons off of a flat plate. However, for theta = pi/24, the attenuation is very high at much shorter wavelengths, and very high at longer wavelengths. It needs to strike a balance between energy storage and thrust at the modes available to us.

Concerning Fig. 2 in the reference,  the attenuation is a very nonlinear function of kr.  For large kr (kr>20) the attenuation is small and it is also independent of the cone angle.

For small values of kr, the attenuation rapidly increases.  As you state,  for theta = pi/24, the attenuation is very high at much shorter wavelengths, and very high at longer wavelengths. 

Unfortunately, Pi/24 is the smallest angle for which they calculate the eigenvalues: there is no data shown for cone angles smaller than Pi/24.

Pi/24 = 7.5 degrees

Now, examining the following cone angles for the experiments:

Example (and geometry)                    { Tan[thetaw],thetaw (degrees) }

Shawyer Experimental                        {0.104019,   5.93851}
Shawyer Fligth Thruster                      {0.19086,   10.8055}
Shawyer Demo                                   {0.219054, 12.3557}
NASA Eagleworks frustum                   {0.263889, 14.7827}
Egan's example                                  {0.36397 ,  20}
Prof. Juan Yang  (2014)                      {0.4538,     24.4 }
Shawyer Superconducting 2014          {0.7002,     35}

It would appear that the optimum geometries were the Shawyer Experimental  and the Shawyer Fligth Thruster designs, that bracket that value (7.5 degrees) for the cone angle.

The next cone angle, examined by the authors was Pi/12 = 15 degrees, which is very close to the angle of NASA's  truncated cone.  This would show that NASA's truncated cone, Prof. Yang's and the new Superconducting design by Shawyer are non-optimal, and that Shawyer has actually been increasing the cone angle too much, if your theory would explain the measured thrust.




Concerning the statement

Quote
I see the attenuation factor in their plots as "similar" to that effect, acting on the microwaves in the cavity near the cut-offs. As the waves are attenuated, their momentum is absorbed as wave velocity goes to zero, just like light falling into a black hole. The result is propulsion.


Concerning light falling into a Black Hole, have you seen McCulloch's explanation, based on Unruh radiation ?


http://www.ptep-online.com/index_files/2015/PP-40-15.PDF

http://physicsfromtheedge.blogspot.com/2015/04/light-in-box-emdrive.html

http://physicsfromtheedge.blogspot.com/2015/03/one-wave-approximation-of-mihsc.html

http://physicsfromtheedge.blogspot.com/2015/02/mihsc-vs-emdrive-data-3d.html

http://physicsfromtheedge.blogspot.com/2014/10/mihsc-vs-emdrive-updated-table.html


Title: Re: EM Drive Developments - related to space flight applications - Thread 2
Post by: zen-in on 05/06/2015 06:33 pm
Wired has a piece today on the em-drive.   Sorry I can't quote the title; that would be too disruptive.   

http://www.wired.com/2015/05/nasa-warp-drive-yeah-still-poppycock/
Title: Re: EM Drive Developments - related to space flight applications - Thread 2
Post by: WarpTech on 05/06/2015 06:33 pm
...
DO NOT reinvent the wheel.

Follow Shawyer as close as you can.

Plenty of clues but you need to do a lot of reading.

....

You are writing that in reference to Shaywer's experiments, I presume, but a number of Shawyer's prescriptions are tied to his theoretical model.  I read the paper that Shawyer sent to Mulletron, to support Shawyer's theoretical model.

I wrote a review of this paper here:  http://forum.nasaspaceflight.com/index.php?topic=36313.msg1369861#msg1369861

Showing that there is nothing in that paper (by Cullen) supporting Shawyer's theoretical model.  On the contrary, it follows the same Maxwell's equations and laws followed by Greg Egan http://gregegan.customer.netspace.net.au/SCIENCE/Cavity/Cavity.html  who concludes that the EM Drive should have no thrust force.

The conclusions of Egan are correct when only considering radiation pressure at wavelength's that are short compared to the cut-off modes. They are incorrect because he did not take into consideration the variable speed of light inside the waveguide for wavelengths close to the cut-off modes. He used eps0 and mu0 as the permittivity and permeability of free vacuum in all his calculations of energy density and force. That is an error!

The space inside the waveguide is not free vacuum, it is constrained by the waveguide. Near the cut-ff modes, his calculations are invalidated because the speed of light is not the same throughout the cavity.

Best Regards,
Todd D.
Title: Re: EM Drive Developments - related to space flight applications - Thread 2
Post by: Rodal on 05/06/2015 06:44 pm
...
DO NOT reinvent the wheel.

Follow Shawyer as close as you can.

Plenty of clues but you need to do a lot of reading.

....

You are writing that in reference to Shaywer's experiments, I presume, but a number of Shawyer's prescriptions are tied to his theoretical model.  I read the paper that Shawyer sent to Mulletron, to support Shawyer's theoretical model.

I wrote a review of this paper here:  http://forum.nasaspaceflight.com/index.php?topic=36313.msg1369861#msg1369861

Showing that there is nothing in that paper (by Cullen) supporting Shawyer's theoretical model.  On the contrary, it follows the same Maxwell's equations and laws followed by Greg Egan http://gregegan.customer.netspace.net.au/SCIENCE/Cavity/Cavity.html  who concludes that the EM Drive should have no thrust force.

The conclusions of Egan are correct when only considering radiation pressure at wavelength's that are short compared to the cut-off modes. They are incorrect because he did not take into consideration the variable speed of light inside the waveguide for wavelengths close to the cut-off modes. He used eps0 and mu0 as the permittivity and permeability of free vacuum in all his calculations of energy density and force. That is an error!

The space inside the waveguide is not free vacuum, it is constrained by the waveguide. Near the cut-ff modes, his calculations are invalidated because the speed of light is not the same throughout the cavity.

Best Regards,
Todd D.

There is a better way to state it.  There is nothing mathematically incorrect in Greg Egan's analysis.  Egan's analysis is a restatement of well-known exact solutions to the truncated cone (going back to the 1930's).
The validity of these solutions have been confirmed during the last 80 years again and again, and they are confirmed again by the COMSOL FEA analysis and the frequencies and mode shapes measured at NASA Eagleworks.

As an example, the mathematical solution for a plane wave is exact.  One may say that anyone using a plane wave solution is not accurately representing nature because there cannot be plane waves in nature (since they extend to infinity).  Similarly, one can state that Greg Egan is using an approximation that is valid to calculate frequencies and mode shapes but perhaps not for calculating microNewton level thrust forces.

The correct statement is that any solution solely based on Maxwell's equations (like Greg Egan's analysis) predicts no thrust, and that therefore the measurements at NASA Eagleworks are due to something else not addressed by Maxwell's equations.

Todd, your explanation uses General Relativity and the Quantum Vacuum, which are explicitly not addresed by Greg Egan.  His solution is still mathematically exact (solution of Maxwell's equations), it may just not be representing the actual physical tests. Either because the tests are an artifact or because they represent some form of propulsion that may be explained by your model or other alternative models.

On the other hand what is mathematically incorrect would be to state that a solution solely based on Maxwell's equation and special relativity (without invoking GR, or the QV, or something else) can predict a thrust in a closed cavity: that is plainly mathematically incorrect.  Something else is needed besides Maxwell's equations and special relativity.

I hope yours (or another theory) succeeds in explaining the measurements as something that can be used for space propulsion, or that it is an artifact.  But the experimental measurements cannot be explained solely based on Maxwell's equations and special relativity.

 :)

Quote from: Von Neumann
The sciences do not try to explain, they hardly even try to interpret, they mainly make models. By a model is meant a mathematical construct which, with the addition of certain verbal interpretations, describes observed phenomena. The justification of such a mathematical construct is solely and precisely that it is expected to work.

(https://upload.wikimedia.org/wikipedia/commons/5/5e/JohnvonNeumann-LosAlamos.gif)
Title: Re: EM Drive Developments - related to space flight applications - Thread 2
Post by: TheTraveller on 05/06/2015 07:18 pm
...
DO NOT reinvent the wheel.

Follow Shawyer as close as you can.

Plenty of clues but you need to do a lot of reading.

....

You are writing that in reference to Shaywer's experiments, I presume, but a number of Shawyer's prescriptions are tied to his theoretical model.  I read the paper that Shawyer sent to Mulletron, to support Shawyer's theoretical model.

I wrote a review of this paper here:  http://forum.nasaspaceflight.com/index.php?topic=36313.msg1369861#msg1369861

Showing that there is nothing in that paper (by Cullen) supporting Shawyer's theoretical model.  On the contrary, it follows the same Maxwell's equations and laws followed by Greg Egan http://gregegan.customer.netspace.net.au/SCIENCE/Cavity/Cavity.html  who concludes that the EM Drive should have no thrust force.
I did read your review that there should be no thrust. But there seems to be thrust, of a level and direction which agrees with Shawyer's theory. Which is why the Shawyer and Chinese thrust claims need to be experimentally verified or not.

I believe there is enough data in the public domain to experimentally replicate their test setups, cavity designs and RF generation / feed methods, starting with the RF narrow band, spherical end plate Flight Thruster, feed via coax, which I plan to replicate in copper and if necessary in Alumininum.

EW is said to be replicating another cavity design, which I believe is Alumininum based, has flat end plates and is planning to blast it with wide band magnetron RF via a waveguide. Is that correct?
Title: Re: EM Drive Developments - related to space flight applications - Thread 2
Post by: Rodal on 05/06/2015 07:24 pm
...
DO NOT reinvent the wheel.

Follow Shawyer as close as you can.

Plenty of clues but you need to do a lot of reading.

....

You are writing that in reference to Shaywer's experiments, I presume, but a number of Shawyer's prescriptions are tied to his theoretical model.  I read the paper that Shawyer sent to Mulletron, to support Shawyer's theoretical model.

I wrote a review of this paper here:  http://forum.nasaspaceflight.com/index.php?topic=36313.msg1369861#msg1369861

Showing that there is nothing in that paper (by Cullen) supporting Shawyer's theoretical model.  On the contrary, it follows the same Maxwell's equations and laws followed by Greg Egan http://gregegan.customer.netspace.net.au/SCIENCE/Cavity/Cavity.html  who concludes that the EM Drive should have no thrust force.
I did read your review that there should be no thrust. But there seems to be thrust, of a level and direction which agrees with Shawyer's theory. Which is why the Shawyer and Chinese thrust claims need to be experimentally verified or not.

I believe there is enough data in the public domain to experimentally replicate their test setups, cavity designs and RF generation / feed methods, starting with the RF narrow band, spherical end plate Flight Thruster, feed via coax, which I plan to replicate in copper and if necessary in Alumininum.

...

No, my review never states that there should be no thrust in the experimental results. 

My review instead states (and shows, carefully, point by point) that the reference given by Shawyer to support his theoretical model, does NOT support his model at all.

Therefore I do not understand why Shawyer references Cullen's paper.

As a reductio ad absurdum, it would be almost like Shawyer referencing Greg Egan, and sending Greg Egan's analysis as support for Shawyer's theoretical explanation.
Title: Re: EM Drive Developments - related to space flight applications - Thread 2
Post by: Star One on 05/06/2015 07:33 pm
Out of interest have any of Eagleworks results appeared on arXiv?
Title: Re: EM Drive Developments - related to space flight applications - Thread 2
Post by: Rodal on 05/06/2015 07:34 pm
Out of interest have any of Eagleworks results appeared on arXiv?
I saw that in the article.  I didn't bother to check because anyone can publish anything on arXiv:

ArXiv is now moderated (it wasn't at the beginning).

ArXiv has this disclaimer:  "The moderators are not referees and do not provide detailed feedback on submissions."

ArXiv is not a refereed, peer reviewed process.

What an academic would inquire is about the number of times that an article has been cited in peer reviewed journals.

In any case, how many peer-reviewed papers did Edison publsh ?
Did Edison publish in ArXiv ?
Title: Re: EM Drive Developments - related to space flight applications - Thread 2
Post by: SeeShells on 05/06/2015 07:44 pm
I may be just an engineer but over the 40 years of my career I've plenty of unexplained things in designs. Anyone taken into consideration the thermal expansion coefficient of the EM case during your pulsed phase? I've been reading for days and there is so much material I might have missed it. Sorry if I did.
Title: Re: EM Drive Developments - related to space flight applications - Thread 2
Post by: Rodal on 05/06/2015 07:45 pm
I may be just an engineer but over the 40 years of my career I've plenty of unexplained things in designs. Anyone taken into consideration the thermal expansion coefficient of the EM case during your pulsed phase? I've been reading for days and there is so much material I might have missed it. Sorry if I did.

Yes, see this analysis by one of the people in this forum:

https://www.researchgate.net/publication/268804028_NASA%27S_MICROWAVE_PROPELLANT-LESS_THRUSTER_ANOMALOUS_RESULTS_CONSIDERATION_OF_A_THERMO-MECHANICAL_EFFECT
Title: Re: EM Drive Developments - related to space flight applications - Thread 2
Post by: WarpTech on 05/06/2015 07:45 pm

<snip>
The conclusions of Egan are correct when only considering radiation pressure at wavelength's that are short compared to the cut-off modes. They are incorrect because he did not take into consideration the variable speed of light inside the waveguide for wavelengths close to the cut-off modes. He used eps0 and mu0 as the permittivity and permeability of free vacuum in all his calculations of energy density and force. That is an error!

The space inside the waveguide is not free vacuum, it is constrained by the waveguide. Near the cut-ff modes, his calculations are invalidated because the speed of light is not the same throughout the cavity.

Best Regards,
Todd D.

...The correct statement is that any solution solely based on Maxwell's equations (like Greg Egan's analysis) predicts no thrust, and that therefore the measurements at NASA Eagleworks are due to something else not addressed by Maxwell's equations.

Todd, your explanation uses General Relativity and the Quantum Vacuum, which are explicitly not addresed by Greg Egan.  His solution is still mathematically exact (solution of Maxwell's equations), it may just not be representing the actual physical tests. Either because the tests are an artifact or because they represent some form of propulsion that may be explained by your model or other alternative models.

On the other hand what is mathematically incorrect would be to state that a solution solely based on Maxwell's equation and special relativity (without invoking GR, or the QV, or something else) can predict a thrust in a closed cavity: that is plainly mathematically incorrect.  Something else is needed besides Maxwell's equations and special relativity.

I hope yours (or another theory) succeeds in explaining the measurements as something that can be used for space propulsion, or that it is an artifact.  But the experimental measurements cannot be explained solely based on Maxwell's equations and special relativity.

 :)

...

Agreed. I believe they would be addressed by Maxwell's questions in curved space-time perfectly well, if a metric were included to account for the variable speed of light, at certain wavelengths.

I do not think there is any "new" physics here to be discovered. It's just new understanding and interpretation;

1. We "know" from Shawyer and textbook physics that the speed of light inside a waveguide depends on the wavelength of the microwaves, and the diameter of the waveguide.

2. We "know" from the PV Model interpretation of GR and observations of gravitational lensing, that a gravitational field is a variable refractive index where the speed of light is not a constant.

3. Therefore, the logical next step is to write Egan's formulation in a generally covariant form and apply a frequency dependent space-time metric, to model the variable refractive index of the tapered waveguide at specific frequency modes.

Personally, I'm not very good at writing generally covariant equations correctly, but I'll give it a shot of there are no GR'ist here who could do this for us. To me, it's simply Shawyer's equation;

Let J1 and J2 be two separate 4-current densities
Let F1 and F2 be the Maxwell field tensor for each.
The force density;

f = J1*F2 - J2*F1 = 0  in flat space-time

It does not equal zero in curved space-time because there is a metric in each product, that has a different value in each location. Shawyer is right in this regard, but I'm no better at the covariant equations than he is.

Thank you!

Best Regards,
Todd
Title: Re: EM Drive Developments - related to space flight applications - Thread 2
Post by: TheTraveller on 05/06/2015 07:47 pm
...
DO NOT reinvent the wheel.

Follow Shawyer as close as you can.

Plenty of clues but you need to do a lot of reading.

....

You are writing that in reference to Shaywer's experiments, I presume, but a number of Shawyer's prescriptions are tied to his theoretical model.  I read the paper that Shawyer sent to Mulletron, to support Shawyer's theoretical model.

I wrote a review of this paper here:  http://forum.nasaspaceflight.com/index.php?topic=36313.msg1369861#msg1369861

Showing that there is nothing in that paper (by Cullen) supporting Shawyer's theoretical model.  On the contrary, it follows the same Maxwell's equations and laws followed by Greg Egan http://gregegan.customer.netspace.net.au/SCIENCE/Cavity/Cavity.html  who concludes that the EM Drive should have no thrust force.
I did read your review that there should be no thrust. But there seems to be thrust, of a level and direction which agrees with Shawyer's theory. Which is why the Shawyer and Chinese thrust claims need to be experimentally verified or not.

I believe there is enough data in the public domain to experimentally replicate their test setups, cavity designs and RF generation / feed methods, starting with the RF narrow band, spherical end plate Flight Thruster, feed via coax, which I plan to replicate in copper and if necessary in Alumininum.

...

No, my review never states that there should be no thrust in the experimental results. 

My review instead states (and shows, carefully, point by point) that the reference given by Shawyer to support his theoretical model, does NOT support his model at all.

Therefore I do not understand why Shawyer references Cullen's paper.

As a reductio ad absurdum, it would be almost like Shawyer referencing Greg Egan, and sending Greg Egan's analysis as support for Shawyer's theoretical explanation.
With respect, your theory.

Shawyer has his & says the EM Drive gens thrust as per his theory based calcs.

For me I will build what Shawyer claims works. I'm fairly certain I have the engineering / experimentalists skills to make it work, if it can work. If it produces thrust, you and anyone else will be welcome to visit and test it yourself. Might even add Shawyer, Paul March & Dr. White to the invite list. Would be interesting watching the 4 of you poke and prod the test unit, arguing the strengths and weaknesses of each others theories.
Title: Re: EM Drive Developments - related to space flight applications - Thread 2
Post by: Rodal on 05/06/2015 07:54 pm
...
Let F1 and F2 be the Maxwell field tensor for each.
The force density;

f = J1*F2 - J2*F1 = 0  in flat space-time

It does not equal zero in curved space-time because there is a metric in each product, that has a different value in each location. Shawyer is right in this regard, but I'm no better at the covariant equations than he is.

Thank you!

Best Regards,
Todd

Regarding the force summation by Shawyer, what happened to the force vector components on the lateral, conical  surfaces?  Why is it that only  the forces on the end plates (the bases of the truncated cone) are being addressed?

If one includes the forces on the lateral conical surfaces everything sums to zero (solely using Maxwell's equations)
Title: Re: EM Drive Developments - related to space flight applications - Thread 2
Post by: TheTraveller on 05/06/2015 07:55 pm
I may be just an engineer but over the 40 years of my career I've plenty of unexplained things in designs. Anyone taken into consideration the thermal expansion coefficient of the EM case during your pulsed phase? I've been reading for days and there is so much material I might have missed it. Sorry if I did.
Ditto on the engineering years and seeing strange things. Been there, seen that.

Shawyers Teeter-Totter test system, with the cavity oriented vertically and supported on 4 external rods, virtually eliminates effects from thermal expansion and COG movement. Is what I will build. Rough sketch here. Like Shawyers 1st test unit sitting on one end of a wide Teeter Totter balance system.

RF coax will feed up through a hole in the centre of the Teeter Totter instead of the method shown on the sketch. 0.5kg scale with 0.001g sensitivity will be on the other end, not directly under the EM Drive. Want to remove any electronics from near the EM Drive.
Title: Re: EM Drive Developments - related to space flight applications - Thread 2
Post by: Mulletron on 05/06/2015 07:58 pm
Enjoyed reading the comments from Davis and Millis. More replication attempts to come. The race has begun.  :)
Title: Re: EM Drive Developments - related to space flight applications - Thread 2
Post by: TheTraveller on 05/06/2015 08:02 pm
Enjoyed reading the comments from Davis and Millis. More replication attempts to come. The race has begun.  :)
If we hang together and openly share info, we can make this happen.
Title: Re: EM Drive Developments - related to space flight applications - Thread 2
Post by: Rodal on 05/06/2015 08:03 pm
...
It does not equal zero in curved space-time because there is a metric in each product, that has a different value in each location. Shawyer is right in this regard, but I'm no better at the covariant equations than he is.

...
I think we (people reading this) can write the covariant equations. The equations would be nonlinear, and therefore an exact solution would prove impossible to obtain for the truncated cone. How would you proceed ?  via a perturbation analysis (if so in terms of what dimensionless parameter?) or via a numerical solution?

EDIT :1)  I think the motivation would increase if there would be an answer to the energy paradox (what restricts the energy paradox for constant acceleration at constant power ?)

and this also needs to be addressed:

2) Regarding the force summation by Shawyer, what happened to the force vector components on the lateral, conical  surfaces?  Why is it that only  the forces on the end plates (the bases of the truncated cone) are being addressed? If one includes the forces on the lateral conical surfaces everything sums to zero (solely using Maxwell's equations)
Title: Re: EM Drive Developments - related to space flight applications - Thread 2
Post by: StrongGR on 05/06/2015 08:19 pm
...
It does not equal zero in curved space-time because there is a metric in each product, that has a different value in each location. Shawyer is right in this regard, but I'm no better at the covariant equations than he is.

...
I think we (people reading this) can write the covariant equations. The equations would be nonlinear, and therefore an exact solution would prove impossible to obtain for the truncated cone. How would you proceed ?  via a perturbation analysis (if so in terms of what dimensionless parameter?) or via a numerical solution?

EDT : I think the motivation would increase if there would be an answer to the energy paradox (what restricts the energy paradox for constant acceleration at constant power ?)

The non-linear set of Einstein-Maxwell equations has exact solutions for plane waves. You can apply it to a schematic of a laser beam transiting in a cavity having the form of a cube without difficulty. A plane wave always deforms space-time and a cavity has the advantage that the energy of the field depends also on the Q factor that can increase it by orders of magnitude. This means that an explanation for the question of the behaviour of the laser beam inside the cavity can be easily at hand. For the other question, the thrust, as a physicist I keep on being rather sceptical because I tried to move my car by hitting the windscreen with punches and nothing happened.

I will post here the solution for a very simple set-up of a cube cavity maintaining a single mode and show the way the laser beam propagates inside it. This resonant cavity seems to be very good for engineering of space-time rather than else.
Title: Re: EM Drive Developments - related to space flight applications - Thread 2
Post by: Rodal on 05/06/2015 08:27 pm
...
It does not equal zero in curved space-time because there is a metric in each product, that has a different value in each location. Shawyer is right in this regard, but I'm no better at the covariant equations than he is.

...
I think we (people reading this) can write the covariant equations. The equations would be nonlinear, and therefore an exact solution would prove impossible to obtain for the truncated cone. How would you proceed ?  via a perturbation analysis (if so in terms of what dimensionless parameter?) or via a numerical solution?

EDT : I think the motivation would increase if there would be an answer to the energy paradox (what restricts the energy paradox for constant acceleration at constant power ?)

The non-linear set of Einstein-Maxwell equations has exact solutions for plane waves. You can apply it to a schematic of a laser beam transiting in a cavity having the form of a cube without difficulty. A plane wave always deforms space-time and a cavity has the advantage that the energy of the field depends also on the Q factor that can increase it by orders of magnitude. This means that an explanation for the question of the behaviour of the laser beam inside the cavity can be easily at hand. For the other question, the thrust, as a physicist I keep on being rather sceptical because I tried to move my car by hitting the windscreen with punches and nothing happened.

I will post here the solution for a very simple set-up of a cube cavity maintaining a single mode and show the way the laser beam propagates inside it. This resonant cavity seems to be very good for engineering of space-time rather than else.
OK, I agree with what you state above.

However, what these experimenters claim, is that without a dielectric polymer insert, the thrust is only produced by a conical cavity (a truncated cone)  .  They claim that a cavity with constant cross-section will produce no thrust (unless it includes a dielectric section inside it).

So what what would be great to have  :)


1)  the solution for a very simple set-up of a constant cross-section cavity (containing an insert of a polymer dielectric and the rest of the cavity void) maintaining a single mode and show the way the laser beam propagates inside it

or

2)  the solution for a truncated cone

__________

#1 (constant cross-section cavity containing a polymer dielectric in part of the cavity) would be most useful to the group in California and to one of our members (@notsosureofit) who has been looking at such a solution and also was planning testing using a Gunn diode cavity
Title: Re: EM Drive Developments - related to space flight applications - Thread 2
Post by: Star One on 05/06/2015 08:31 pm
I've noticed that there is a number of people now attempting to do their own builds, as has mentioned elsewhere, is there any danger to themselves in this? By the way I'm not attempting to cast aspersions on anyone's technical abilities by asking this I'm just genuinely wondering?
Title: Re: EM Drive Developments - related to space flight applications - Thread 2
Post by: sghill on 05/06/2015 08:39 pm
I will post here the solution for a very simple set-up of a cube cavity maintaining a single mode and show the way the laser beam propagates inside it. This resonant cavity seems to be very good for engineering of space-time rather than else.

May I respectfully ask that you also include a visualization of the solution if possible?  I'd very much like to see this.
Title: Re: EM Drive Developments - related to space flight applications - Thread 2
Post by: D_Dom on 05/06/2015 08:41 pm
RF energies can cause severe burns.
http://rfsafetysolutions.com/RF%20Radiation%20Pages/Biological_Effects.html
"Perhaps the most vulnerable organs are the eyes. The eyes have virtually no blood flow that can provide cooling from other parts of the body, and their dimensions make them very good antennas at microwave frequencies."
Title: Re: EM Drive Developments - related to space flight applications - Thread 2
Post by: Rodal on 05/06/2015 08:49 pm
I will post here the solution for a very simple set-up of a cube cavity maintaining a single mode and show the way the laser beam propagates inside it. This resonant cavity seems to be very good for engineering of space-time rather than else.

May I respectfully ask that you also include a visualization of the solution if possible?  I'd very much like to see this.
StrongGR  may be talking about a closed-form solution for that case (without the dielectric insert), therefore a mathematical formula.  Not necessarily including plots, as they are necessary for numerical solutions. 

Given the closed-form solution (for which we would be most thankful  :) ) then all of us could make plots using our own software, for any numerical values we are interested in.

That's why closed-form solutions rock :)
Title: Re: EM Drive Developments - related to space flight applications - Thread 2
Post by: aceshigh on 05/06/2015 08:49 pm
Enjoyed reading the comments from Davis and Millis. More replication attempts to come. The race has begun.  :)

are you saying that Marc Millis and Eric Davis are posting directly here at NSF forum?? Under what nicknames?

if so, I wonder if Dr Davis could clear out the doubts we had regarding the light cone issue from his talk at Icarus Interstellar Starship Conference (2013), at page 93 of this thread.
Title: Re: EM Drive Developments - related to space flight applications - Thread 2
Post by: WarpTech on 05/06/2015 09:05 pm
...
It does not equal zero in curved space-time because there is a metric in each product, that has a different value in each location. Shawyer is right in this regard, but I'm no better at the covariant equations than he is.

...
I think we (people reading this) can write the covariant equations. The equations would be nonlinear, and therefore an exact solution would prove impossible to obtain for the truncated cone. How would you proceed ?  via a perturbation analysis (if so in terms of what dimensionless parameter?) or via a numerical solution?

EDIT :1)  I think the motivation would increase if there would be an answer to the energy paradox (what restricts the energy paradox for constant acceleration at constant power ?)

and this also needs to be addressed:

2) Regarding the force summation by Shawyer, what happened to the force vector components on the lateral, conical  surfaces?  Why is it that only  the forces on the end plates (the bases of the truncated cone) are being addressed? If one includes the forces on the lateral conical surfaces everything sums to zero (solely using Maxwell's equations)

This is at the limit of my mathematical abilities, but I would perturb the refractive index as the dimensionless parameter. It is the same as a perturbation of the metric coefficients, as a small deviation from Minkowski's metric, eta^uv.

g^uv = eta^uv + h^uv

The h^uv represent the variations of c/K in the waveguide, and will need to be experimentally determined over specific frequencies and bandwidths to find out what those components "really" are. In this case, the non-linearity will appear in the B-H and D-E curves of the waveguide, but a linear approximation of a metric, the way it is done for gravitational waves, should suffice if the cone angle is small.

Regarding the energy paradox.

Power = Force * Velocity
Acceleration = Force / Mass = Power / Momentum

Therefore, as the momentum increases, the acceleration decreases for a constant power input. That's without relativistic effects. Why is this a paradox? You've mentioned this a few times, but I guess I've missed something.

Regarding Shawyer's analysis. I do not agree with the formulation, it is a layman's approximation. What I agree with is that the difference in group velocity "is" the reason it works. He's right about that. His formulation is not the generally covariant one it should be, as I described. Again, using Maxwell's equations in a curved space-time with a frequency dependent metric.

Best Regards,
Todd D.

Title: Re: EM Drive Developments - related to space flight applications - Thread 2
Post by: deltaMass on 05/06/2015 09:05 pm
I may be just an engineer but over the 40 years of my career I've plenty of unexplained things in designs. Anyone taken into consideration the thermal expansion coefficient of the EM case during your pulsed phase? I've been reading for days and there is so much material I might have missed it. Sorry if I did.

Yes, see this analysis by one of the people in this forum:

https://www.researchgate.net/publication/268804028_NASA%27S_MICROWAVE_PROPELLANT-LESS_THRUSTER_ANOMALOUS_RESULTS_CONSIDERATION_OF_A_THERMO-MECHANICAL_EFFECT
That's an excellent analysis - Kudos. I would not be surprised if you were a small numerical factor in error (as I'm sure neither would you be) and as such you're pointing up the possibility of all the measured thrust being attributable to thermally-induced movement.

And of course, the measurement of a reverse thrust does not gainsay the thermal explanation, since everything will be thermally perturbed in the opposite direction when the cavity is mounted 180o to its default mounting orientation.

Mr. March counters this with the fact that the measured thrust onset is as prompt as that of the calibration pulse. Yet you are showing quite prompt thermal onsets. How do you reconcile these two points of view?
Title: Re: EM Drive Developments - related to space flight applications - Thread 2
Post by: deltaMass on 05/06/2015 09:10 pm
Regarding the energy paradox.

Power = Force * Velocity
Acceleration = Force / Mass = Power / Momentum

Therefore, as the momentum increases, the acceleration decreases for a constant power input. That's without relativistic effects. Why is this a paradox? You've mentioned this a few times, but I guess I've missed something.

Best Regards,
Todd D.

No.  You are conflating input power and output power here. Please see my analysis.
http://forum.nasaspaceflight.com/index.php?topic=36313.msg1369875#msg1369875
The EmDrive is not a tyre and spacetime is not the road upon which it rides.

If you propose that an EmDrive accelerating in free space exhibits thrust which depends somehow on its velocity, then what you propose violates special relativity.
Title: Re: EM Drive Developments - related to space flight applications - Thread 2
Post by: Rodal on 05/06/2015 09:13 pm
I may be just an engineer but over the 40 years of my career I've plenty of unexplained things in designs. Anyone taken into consideration the thermal expansion coefficient of the EM case during your pulsed phase? I've been reading for days and there is so much material I might have missed it. Sorry if I did.

Yes, see this analysis by one of the people in this forum:

https://www.researchgate.net/publication/268804028_NASA%27S_MICROWAVE_PROPELLANT-LESS_THRUSTER_ANOMALOUS_RESULTS_CONSIDERATION_OF_A_THERMO-MECHANICAL_EFFECT
That's an excellent analysis - Kudos. I would not be surprised if you were a small numerical factor in error (as I'm sure neither would you be) and as such you're pointing up the possibility of all the measured thrust being attributable to thermally-induced movement.

And of course, the measurement of a reverse thrust does not gainsay the thermal explanation, since everything will be thermally perturbed in the opposite direction when the cavity is mounted 180o to its default mounting orientation.

Mr. March counters this with the fact that the measured thrust onset is as prompt as that of the calibration pulse. Yet you are showing quite prompt thermal onsets. How do you reconcile these two points of view?

Thermal buckling is extremely dependent on initial imperfections, thus it is highly unlikely that it can be the only explanation for multiple tests at multiple locations with multiple specimens.

Therefore something else is at play here.

Pressure measurements due to radiation have been plagued with air convection currents for 140 years.
None of the experimenters have used Cullen's set-up in ambient conditions: nobody has used concentric rings (see this  http://forum.nasaspaceflight.com/index.php?topic=36313.msg1370479#msg1370479 )

NASA Eagleworks is the only one that has conducted tests in a vacuum.  Their results could be due to outgassing from the fiberglass-reinforced-epoxy end-plate.

Therefore I highly advise that they should replace the end plates with concentric rings to rule out false positives, as done by Cullen in 1951, the first person to accurately measure microwave radiation pressure from microwaves.
Title: Re: EM Drive Developments - related to space flight applications - Thread 2
Post by: deltaMass on 05/06/2015 09:17 pm
Rodal: " Their results could be due to outgassing from FRP4"

This is some sort of fibreglass?
Title: Re: EM Drive Developments - related to space flight applications - Thread 2
Post by: RotoSequence on 05/06/2015 09:19 pm
Rodal: " Their results could be due to outgassing from FRP4"

This is some sort of fibreglass?

The Eagleworks EM Drive backplate is copper coated, fiberglass reinforced polymer of the type used to make printed circuit boards.
Title: Re: EM Drive Developments - related to space flight applications - Thread 2
Post by: Rodal on 05/06/2015 09:20 pm
Rodal: " Their results could be due to outgassing from FRP4"

This is some sort of fibreglass?

Bilayer: an external thick layer of Fiberglass-reinforced-epoxy end-plate with an extremely thin layer of copper deposited on the inner surface of the truncated cone end plate
Title: Re: EM Drive Developments - related to space flight applications - Thread 2
Post by: deltaMass on 05/06/2015 09:21 pm
I have not seen descriptions of EagleWorks' vacuum outgassing/bake-out protocols.
Title: Re: EM Drive Developments - related to space flight applications - Thread 2
Post by: TheTraveller on 05/06/2015 09:22 pm
I may be just an engineer but over the 40 years of my career I've plenty of unexplained things in designs. Anyone taken into consideration the thermal expansion coefficient of the EM case during your pulsed phase? I've been reading for days and there is so much material I might have missed it. Sorry if I did.

Yes, see this analysis by one of the people in this forum:

https://www.researchgate.net/publication/268804028_NASA%27S_MICROWAVE_PROPELLANT-LESS_THRUSTER_ANOMALOUS_RESULTS_CONSIDERATION_OF_A_THERMO-MECHANICAL_EFFECT
That's an excellent analysis - Kudos. I would not be surprised if you were a small numerical factor in error (as I'm sure neither would you be) and as such you're pointing up the possibility of all the measured thrust being attributable to thermally-induced movement.

And of course, the measurement of a reverse thrust does not gainsay the thermal explanation, since everything will be thermally perturbed in the opposite direction when the cavity is mounted 180o to its default mounting orientation.

Mr. March counters this with the fact that the measured thrust onset is as prompt as that of the calibration pulse. Yet you are showing quite prompt thermal onsets. How do you reconcile these two points of view?

Thermal buckling is extremely dependent on initial imperfections, thus it is highly unlikely that it can be the only explanation for multiple tests at multiple locations with multiple specimens.

Therefore something else is at play here.

Pressure measurements due to radiation have been plagued with air convection currents for 140 years.
None of the experimenters have used the proper test set-up in ambient conditions: nobody has used concentric rings (see this  http://forum.nasaspaceflight.com/index.php?topic=36313.msg1370479#msg1370479 )

Only NASA Eagleworks is the only one that has conducted tests in a vacuum.  Their results could be due to outgassing from the fiberglass-reinforced-epoxy end-plate.

Therefore I highly advise that they should replace the end plates with concentric rings to rule out false positives, as done by Cullen in 1951, the first person to accurately measure microwave radiation pressure from microwaves.
Would be interested in testing that idea. AFTER either no thrust or thrust is found.

"Follow the Data, Ignore the Theories".
Title: Re: EM Drive Developments - related to space flight applications - Thread 2
Post by: Rodal on 05/06/2015 09:25 pm
That's not a theory!

That was the first accurate, successful experimental measurement of microwave pressure, and Cullen learnt that by first trying a solid end plate (which resulted in false-positive due to air currents), it was his PhD thesis   :)

Cullen's Ph.D. thesis is an experiment to measure radiation pressure from microwaves.

Just like Michelson's was an experiment, and Cavendish was an experiment, and so on and on.


Title: Re: EM Drive Developments - related to space flight applications - Thread 2
Post by: PaulF on 05/06/2015 09:28 pm
Question to all:

Has quantum tunneling been incorporated in any calculations? I read once that at least some photons will tunnel through the medium of the cavity walls. Hope this sparks something in someone's mind.
Yes, we considered it.  It would become an inefficient photon rocket, as only a few of the photons would tunnel and the beam would not be perfectly collimated.  Hence it would not explain a claimed thrust/PoweInput thousands of times greater than a perfectly collimated photon rocket.
But what if... lets assume the theory of the thrusting of virtual particles. What if they also were able to tunnel and end up outside the cavity before they do whatever they normally do?
Title: Re: EM Drive Developments - related to space flight applications - Thread 2
Post by: SeeShells on 05/06/2015 09:44 pm
I may be just an engineer but over the 40 years of my career I've plenty of unexplained things in designs. Anyone taken into consideration the thermal expansion coefficient of the EM case during your pulsed phase? I've been reading for days and there is so much material I might have missed it. Sorry if I did.

Yes, see this analysis by one of the people in this forum:

https://www.researchgate.net/publication/268804028_NASA%27S_MICROWAVE_PROPELLANT-LESS_THRUSTER_ANOMALOUS_RESULTS_CONSIDERATION_OF_A_THERMO-MECHANICAL_EFFECT
That's an excellent analysis - Kudos. I would not be surprised if you were a small numerical factor in error (as I'm sure neither would you be) and as such you're pointing up the possibility of all the measured thrust being attributable to thermally-induced movement.

And of course, the measurement of a reverse thrust does not gainsay the thermal explanation, since everything will be thermally perturbed in the opposite direction when the cavity is mounted 180o to its default mounting orientation.

Mr. March counters this with the fact that the measured thrust onset is as prompt as that of the calibration pulse. Yet you are showing quite prompt thermal onsets. How do you reconcile these two points of view?

I'm slowly digging through the nice thermal expansion write-up, nice work. I was wondering if you have an IR camera and pics to back your calculations, as I'd like to see the thermal increases not only on the endcap but on the sides of the EM chamber. Thanks Guys!
 
Title: Re: EM Drive Developments - related to space flight applications - Thread 2
Post by: Rodal on 05/06/2015 10:33 pm
I may be just an engineer but over the 40 years of my career I've plenty of unexplained things in designs. Anyone taken into consideration the thermal expansion coefficient of the EM case during your pulsed phase? I've been reading for days and there is so much material I might have missed it. Sorry if I did.

Yes, see this analysis by one of the people in this forum:

https://www.researchgate.net/publication/268804028_NASA%27S_MICROWAVE_PROPELLANT-LESS_THRUSTER_ANOMALOUS_RESULTS_CONSIDERATION_OF_A_THERMO-MECHANICAL_EFFECT
That's an excellent analysis - Kudos. I would not be surprised if you were a small numerical factor in error (as I'm sure neither would you be) and as such you're pointing up the possibility of all the measured thrust being attributable to thermally-induced movement.

And of course, the measurement of a reverse thrust does not gainsay the thermal explanation, since everything will be thermally perturbed in the opposite direction when the cavity is mounted 180o to its default mounting orientation.

Mr. March counters this with the fact that the measured thrust onset is as prompt as that of the calibration pulse. Yet you are showing quite prompt thermal onsets. How do you reconcile these two points of view?

I'm slowly digging through the nice thermal expansion write-up, nice work. I was wondering if you have an IR camera and pics to back your calculations, as I'd like to see the thermal increases not only on the endcap but on the sides of the EM chamber. Thanks Guys!

These are the thermal measurements vs. thermal and electromagnetic field analysis (COMSOL FEA) for NASA Eagleworks for the experiments in a partial vacuum  ;) :

http://forum.nasaspaceflight.com/index.php?action=dlattach;topic=36313.0;attach=634723

The thermal distribution is clearly due to induction heating from the magnetic field, as expected.

My exact solution (elsewhere in other posts) for the magnetic field, for the electric field, confirms that the numerical solution using COMSOL FEA is within 1% of the exact solution, they used a good finite element mesh discretization and the numerical solution is practically converged to the exact result.

Also Prof. Juan Yang's reported temperature vs. time measurements with embedded thermocouples throughout their EM Drive cavity (without a polymer dielectric insert) under atmospheric conditions, that, curiously, show the highest temperature at the center of the small base (trace #1), followed, at a significantly lower temperature by the temperature at the periphery of the big base (trace #5).

(http://forum.nasaspaceflight.com/index.php?action=dlattach;topic=29276.0;attach=622845;image)

(http://forum.nasaspaceflight.com/index.php?action=dlattach;topic=29276.0;attach=655009;image)

Notice how much higher temperature is present in the Chinese (Yang) experiments.
Shawyer has not reported graphical information on thermal measurements for his experiments, to my knowledge.
I suspect same problem for Shawyer's experiments: huge thermal effects for Yang and Shawyer.

Also, tan delta out-of-phase losses in these cavities are not zero,particularly in NASA's dielectric polymer insert: the reported experiments show that the tan delta values of the materials used in these cavities is consistent with real materials experiencing out-of-phase dissipation (therefore one must use the complex form of the physical properties and not neglect the imaginary part if one is interested in assessing the finite value of Q, for example).

Title: Re: EM Drive Developments - related to space flight applications - Thread 2
Post by: WarpTech on 05/06/2015 11:05 pm
Regarding the energy paradox.

Power = Force * Velocity
Acceleration = Force / Mass = Power / Momentum

Therefore, as the momentum increases, the acceleration decreases for a constant power input. That's without relativistic effects. Why is this a paradox? You've mentioned this a few times, but I guess I've missed something.

Best Regards,
Todd D.

No.  You are conflating input power and output power here. Please see my analysis.
http://forum.nasaspaceflight.com/index.php?topic=36313.msg1369875#msg1369875
The EmDrive is not a tyre and spacetime is not the road upon which it rides.

If you propose that an EmDrive accelerating in free space exhibits thrust which depends somehow on its velocity, then what you propose violates special relativity.

Special Relativity does not apply to accelerating reference frames, which is what is being described here. Accelerated reference frames "are" preferred frames because they can be distinguished from one another, unlike inertial frames.

If F = P*k

Then the work done is the integral;

W = integral[Pin*k*v]*dt   from t=0 to t2.

Pout = dW/dt = Pin*k*v, Since Pin = Pout, k = 1/v.

The Power in will equal the Power out, the Work done will be the integration of Force * distance. Acceleration is not constant with constant Power input. I have a hunch that there is a hidden assumption in your derivation that acceleration is constant, so you get over-unity results. This needs to be considered like a Power Transformer. Power in = Power out.

That's my 2 cents worth.

Todd D.

Title: Re: EM Drive Developments - related to space flight applications - Thread 2
Post by: deltaMass on 05/06/2015 11:48 pm
Regarding the energy paradox.

Power = Force * Velocity
Acceleration = Force / Mass = Power / Momentum

Therefore, as the momentum increases, the acceleration decreases for a constant power input. That's without relativistic effects. Why is this a paradox? You've mentioned this a few times, but I guess I've missed something.

Best Regards,
Todd D.

Quote from: deltaMass
No.  You are conflating input power and output power here. Please see my analysis.
http://forum.nasaspaceflight.com/index.php?topic=36313.msg1369875#msg1369875
The EmDrive is not a tyre and spacetime is not the road upon which it rides.

If you propose that an EmDrive accelerating in free space exhibits thrust which depends somehow on its velocity, then what you propose violates special relativity.

Quote from: WarpTech
Special Relativity does not apply to accelerating reference frames, which is what is being described here. Accelerated reference frames "are" preferred frames because they can be distinguished from one another, unlike inertial frames.
If F = P*k
 

There is no "if" about it. Note my use of a colon to indicate definition.
I define k := F/P   in Newtons/Watt

Quote from: WarpTech
Then the work done is the integral;
W = integral[Pin*k*v]*dt   from t=0 to t2.
Pout = dW/dt = Pin*k*v
Since Pin = Pout, k = 1/v

Same mistake as before. You are starting with the implicit assumption that Pin = Pout.
It is no surprise that you conclude the same.

Quote from: WarpTech
The Power in will equal the Power out, the Work done will be the integration of Force * distance. Acceleration is not constant with constant Power input.
I have a hunch that there is a hidden assumption in your derivation that acceleration is constant, so you get over-unity results.

Hidden? No. It's clearly stated in my preamble that acceleration will be constant.
More to the point, if you think there's a flaw in my maths, I'd like to hear specifics.

Quote from: WarpTech
This needs to be considered like a Power Transformer. Power in = Power out.
That's my 2 cents worth.

Todd D.
Title: Re: EM Drive Developments - related to space flight applications - Thread 2
Post by: Rodal on 05/06/2015 11:52 pm
Regarding the energy paradox.

Power = Force * Velocity
Acceleration = Force / Mass = Power / Momentum

Therefore, as the momentum increases, the acceleration decreases for a constant power input. That's without relativistic effects. Why is this a paradox? You've mentioned this a few times, but I guess I've missed something.

Best Regards,
Todd D.

No.  You are conflating input power and output power here. Please see my analysis.
http://forum.nasaspaceflight.com/index.php?topic=36313.msg1369875#msg1369875
The EmDrive is not a tyre and spacetime is not the road upon which it rides.

If you propose that an EmDrive accelerating in free space exhibits thrust which depends somehow on its velocity, then what you propose violates special relativity.

Special Relativity does not apply to accelerating reference frames, which is what is being described here. Accelerated reference frames "are" preferred frames because they can be distinguished from one another, unlike inertial frames.

If F = P*k

Then the work done is the integral;

W = integral[Pin*k*v]*dt   from t=0 to t2.

Pout = dW/dt = Pin*k*v, Since Pin = Pout, k = 1/v.

The Power in will equal the Power out, the Work done will be the integration of Force * distance. Acceleration is not constant with constant Power input. I have a hunch that there is a hidden assumption in your derivation that acceleration is constant, so you get over-unity results. This needs to be considered like a Power Transformer. Power in = Power out.

That's my 2 cents worth.

Todd D.

Thanks !

Since the quick trip scenarios presented by Dr. White in his papers (that were paraphrased in the NSF article) assumed a constant acceleration under constant power, it looks like that there are two alternatives here:


1) At speeds v <<c; F = P/v => m a = P/v => a = P/(m*v) therefore at constant power the acceleration is inversely proportional to the speed.  Equivalently for constant acceleration the power needs to be increased proportional to the speed.  Therefore the real performance of the EM Drive (for interplanetary missions or for going to Alpha Centauri) is much, much inferior to the calculations in White papers, and White with Joosten, etc. and they need to revisited.

or

2) as Paul March proposes, White's QV theory is the other side of the same coin as Woodward's Mach Effect,

https://en.wikipedia.org/wiki/Woodward_effect#Quantum_mechanics

for which they assume constant acceleration at constant power, and the energy paradox has to be formulated in terms of the inertial mass of Woodward instead.  (All kinds of questions here remaining to be answered: is Woodward conjecture a real effect? is the EM Drive "thrust" due to Woodward Mach Effect? is there an energy paradox for Woodward's effect ?)
Title: Re: EM Drive Developments - related to space flight applications - Thread 2
Post by: SeeShells on 05/07/2015 12:00 am
I may be just an engineer but over the 40 years of my career I've plenty of unexplained things in designs. Anyone taken into consideration the thermal expansion coefficient of the EM case during your pulsed phase? I've been reading for days and there is so much material I might have missed it. Sorry if I did.

Yes, see this analysis by one of the people in this forum:

https://www.researchgate.net/publication/268804028_NASA%27S_MICROWAVE_PROPELLANT-LESS_THRUSTER_ANOMALOUS_RESULTS_CONSIDERATION_OF_A_THERMO-MECHANICAL_EFFECT
That's an excellent analysis - Kudos. I would not be surprised if you were a small numerical factor in error (as I'm sure neither would you be) and as such you're pointing up the possibility of all the measured thrust being attributable to thermally-induced movement.

And of course, the measurement of a reverse thrust does not gainsay the thermal explanation, since everything will be thermally perturbed in the opposite direction when the cavity is mounted 180o to its default mounting orientation.

Mr. March counters this with the fact that the measured thrust onset is as prompt as that of the calibration pulse. Yet you are showing quite prompt thermal onsets. How do you reconcile these two points of view?

I'm slowly digging through the nice thermal expansion write-up, nice work. I was wondering if you have an IR camera and pics to back your calculations, as I'd like to see the thermal increases not only on the endcap but on the sides of the EM chamber. Thanks Guys!

These are the thermal measurements vs. thermal and electromagnetic field analysis (COMSOL FEA) for NASA Eagleworks for the experiments in a partial vacuum  ;) :

http://forum.nasaspaceflight.com/index.php?action=dlattach;topic=36313.0;attach=634723

The thermal distribution is clearly due to induction heating from the magnetic field, as expected.

My exact solution (elsewhere in other posts) for the magnetic field, for the electric field, confirms that the numerical solution using COMSOL FEA is within 1% of the exact solution, they used a good finite element mesh discretization and the numerical solution is practically converged to the exact result.

Also Prof. Juan Yang's reported temperature vs. time measurements with embedded thermocouples throughout their EM Drive cavity (without a polymer dielectric insert) under atmospheric conditions, that, curiously, show the highest temperature at the center of the small base (trace #1), followed, at a significantly lower temperature by the temperature at the periphery of the big base (trace #5).

(http://forum.nasaspaceflight.com/index.php?action=dlattach;topic=29276.0;attach=622845;image)

(http://forum.nasaspaceflight.com/index.php?action=dlattach;topic=29276.0;attach=655009;image)

Notice how much higher temperature is present in the Chinese (Yang) experiments.
Shawyer has not reported graphical information on thermal measurements for his experiments, to my knowledge.
I suspect same problem for Shawyer's experiments: huge thermal effects for Yang and Shawyer.

Also, tan delta out-of-phase losses in these cavities are not zero,particularly in NASA's dielectric polymer insert: the reported experiments show that the tan delta values of the materials used in these cavities is consistent with real materials experiencing out-of-phase dissipation (therefore one must use the complex form of the physical properties and not neglect the imaginary part if one is interested in assessing the finite value of Q, for example).

Thank You for taking your time to answer me! I've got a little to digest here, but there is more than enough.
Is there any reason no one has used a simple copper mesh for the EM chamber?
Title: Re: EM Drive Developments - related to space flight applications - Thread 2
Post by: deltaMass on 05/07/2015 12:00 am
It's good that at least one person understands what I'm on about. :-\

Consider an EmDrive in free space and accelerating. We switch it off temporarily and let it coast at speed v relative to the inertial frame in which it began its acceleration. When we switch it back on, are we going to assert that somehow the thrust F  knows what speed it's going and adjusts the thrust like F = P/v?

I assert again that this kind of thinking requires a preferred frame, and thus violates SR.
Title: Re: EM Drive Developments - related to space flight applications - Thread 2
Post by: Rodal on 05/07/2015 12:10 am
....
Thank You for taking your time to answer me! I've got a little to digest here, but there is more than enough.
Is there any reason no one has used a simple copper mesh for the EM chamber?

Excellent question.  I think an answer is found by looking at the last pages: although there are a couple of posts detailing that Cullen (the first person to successfully measure the microwave radiation pressure in his PhD experiments) solved the thermal convection problem by using a mesh, there are countless replies here saying "Theories are theories.  I will do exactly the same thing as Shawyer. "  :)  In other words: some experimenters may be initially doing what Shawyer did.  Shawyer did not use a mesh.

I bet that Paul  March would try the mesh and report on his results (just like he did the test with the unit outside the vacuum chamber to eliminate the issue of evanescent wave coupling once we discussed it in this forum)
Title: Re: EM Drive Developments - related to space flight applications - Thread 2
Post by: Rodal on 05/07/2015 12:13 am
It's good that at least one person understands what I'm on about. :-\

Consider an EmDrive in free space and accelerating. We switch it off temporarily and let it coast at speed v relative to the inertial frame in which it began its acceleration. When we switch it back on, are we going to assert that somehow the thrust F  knows what speed it's going and adjusts the thrust like F = P/v?

I assert again that this kind of thinking requires a preferred frame, and thus violates SR.

Or you could also give the example of EM Drive ship 1 (which started from a different place) being overtaken by EM Drive ship 2 ...

EDIT:

See White and Joosten, Appendix :

http://ntrs.nasa.gov/archive/nasa/casi.ntrs.nasa.gov/20140013174.pdf
Title: Re: EM Drive Developments - related to space flight applications - Thread 2
Post by: deltaMass on 05/07/2015 12:18 am
LOL. I am not sure whether you're agreeing with me or not, but ship #2 is going to do that anyway  8)

There are people in various comment threads citing Langmuir's definitions of pathological science, and specifically that as time goes on, the effect becomes smaller and smaller. But this is not the case here; indeed, since Mr. March has gained some extra control of the phasing profile, they are now north of 100 uN and breaking their own records.

So it's never a good time to launch a starship? - stuff and nonsense!
Title: Re: EM Drive Developments - related to space flight applications - Thread 2
Post by: Rodal on 05/07/2015 12:20 am
EM Drive ships seem to  "know" what to do and when to do it in order not to produce a paradox:


Quote from: Joosten and White
This condition occurs at a change in time of  which equates to a. When this
situation occurs, in order to ensure that the input energy is
equal to the change in kinetic energy, the thrust to power
performance will have to decrease over time
. This scenario
has an analog in the terrestrial realm when considering
a turbine aircraft flight profile. At takeoff, the turbine
aircraft has a very high thrust to power (hundreds of
N/kW), but at cruise altitude, the thrust to power performance
is much lower (1-10 N/kW). The following graph
shows the curve with some highlighted data points for
consideration.

I don't understand this.  Is there an entropy-like emergent 2nd law that prevents the paradox from occurring?
(Just like it is extremely improbable for an egg smashed on the floor to spontaneously reconstitute itself)

Or something like Hawking's law to prevent time-travel-to-the-past paradoxes? https://en.wikipedia.org/wiki/Chronology_protection_conjecture

How about the scenario proposed by @frobnicat of using the EM Drives as generators? and create a paradox on top of a paradox?

http://forum.nasaspaceflight.com/index.php?topic=37438.msg1370818#msg1370818
Title: Re: EM Drive Developments - related to space flight applications - Thread 2
Post by: deltaMass on 05/07/2015 12:28 am
I must reject the analysis by J & W in Appendix A, but thanks for the link.

I have already mentioned in my preamble that not only is any propellantless propulsion craft capable of perpetuum mobile operation, but that free energy is available on top of that to boot.

This causes many people to break out in hives, or to resort to chewing their towels.  ::)

We can of course test to see this at work. We use a rotary configuration and when the tangential velocity exceeds 2/k, we switch from external power to a coaxially mounted generator. And then begin charging people for the juice. Outrageous!  8)
Title: Re: EM Drive Developments - related to space flight applications - Thread 2
Post by: Rodal on 05/07/2015 12:33 am
I must reject the analysis by J & W in Appendix A, but thanks for the link.

I have already mentioned in my preamble that not only is any propellantless propulsion craft capable of perpetuum mobile operation, but that free energy is available on top of that to boot.

This causes many people to break out in hives, or to resort to chewing their towels.  ::)

Well, when somebody like Hawkings proposes chronology protection to prevent time travel:
https://en.wikipedia.org/wiki/Chronology_protection_conjecture

it is invested with the imprimatura of somebody serious. So White and Woodward deserve the same attention and respect.

How do you address Woodward's conjecture ?  (he claims that it is perfectly compatible with GR)
Title: Re: EM Drive Developments - related to space flight applications - Thread 2
Post by: deltaMass on 05/07/2015 12:35 am
Having been raised in the sixties, I am kinda allergic to imprimatura imprimaturs. A pox on them, say I.
Title: Re: EM Drive Developments - related to space flight applications - Thread 2
Post by: PushHigher on 05/07/2015 01:15 am
It's good that at least one person understands what I'm on about. :-\

Consider an EmDrive in free space and accelerating. We switch it off temporarily and let it coast at speed v relative to the inertial frame in which it began its acceleration. When we switch it back on, are we going to assert that somehow the thrust F  knows what speed it's going and adjusts the thrust like F = P/v?

I assert again that this kind of thinking requires a preferred frame, and thus violates SR.

deltaMass, can you explain this further? 

With my limited SR understanding - the statement "x knows what speed it's going and adjusts the thrust" seems perfectly acceptable to me (minus the word "thrust") but I realize I'm likely missing some key points.  If you have two star systems and a rocket sets out for the other at the speed of light (99.999 etc) from each star system toward each other.  Is the speed of the two rockets approaching each other 2c?  No.  Does that mean the two dropped down to .5c? No.  Are they "aware" of each other?  Kinda.  If they decide to burn their thrusters a max does their approach speed increase? No.

I realize you probably already understand this or maybe my understanding is wrong - so what am I missing? 

Thank you for your help!
 

Title: Re: EM Drive Developments - related to space flight applications - Thread 2
Post by: deltaMass on 05/07/2015 01:28 am
It's good that at least one person understands what I'm on about. :-\

Consider an EmDrive in free space and accelerating. We switch it off temporarily and let it coast at speed v relative to the inertial frame in which it began its acceleration. When we switch it back on, are we going to assert that somehow the thrust F  knows what speed it's going and adjusts the thrust like F = P/v?

I assert again that this kind of thinking requires a preferred frame, and thus violates SR.

deltaMass, can you explain this further? 

With my limited SR understanding - the statement "x knows what speed it's going and adjusts the thrust" seems perfectly acceptable to me (minus the word "thrust") but I realize I'm likely missing some key points.  If you have two star systems and a rocket sets out for the other at the speed of light (99.999 etc) from each star system toward each other.  Is the speed of the two rockets approaching each other 2c?  No.  Does that mean the two dropped down to .5c? No.  Are they "aware" of each other?  Kinda.  If they decide to burn their thrusters a max does their approach speed increase? No.

I realize you probably already understand this or maybe my understanding is wrong - so what am I missing? 

Thank you for your help!

I don't mean to muddy the waters of my maths by including fully relativistic calculations (as for example is done in Appendix B of that paper, quite correctly I think). I specifically stated that the derivation is strictly subrelativistic. Indeed, one can entertain a real life rotary device that gets to v = 2/k and hits breakeven, provided that we can achieve a k-value about 50x higher than what is currently achieved, reportedly, by Yuan. That velocity is about the speed of sound in air. So, no SR is needed to address my maths, nor to build a machine to test it.

My mention of SR is simply in order to highlight a core principle of Einstein's thinking about space and time; to whit, there is no preferred inertial frame, such that physics there is different to physics in another one.

So one has a choice here - reject Noether or reject Einstein. It's a tough call.
Unless of course propellantless propulsion sui generis is a canard.
Title: Re: EM Drive Developments - related to space flight applications - Thread 2
Post by: Notsosureofit on 05/07/2015 01:39 am
I must reject the analysis by J & W in Appendix A, but thanks for the link.

I have already mentioned in my preamble that not only is any propellantless propulsion craft capable of perpetuum mobile operation, but that free energy is available on top of that to boot.

This causes many people to break out in hives, or to resort to chewing their towels.  ::)

We can of course test to see this at work. We use a rotary configuration and when the tangential velocity exceeds 2/k, we switch from external power to a coaxially mounted generator. And then begin charging people for the juice. Outrageous!  8)

I would think the free energy speculation is a bit premature.  The "frame of reference problem" could inhibit any constrained mechanism, not that such can be calculated until the momentum question is resolved.
Title: Re: EM Drive Developments - related to space flight applications - Thread 2
Post by: WarpTech on 05/07/2015 01:39 am
It's good that at least one person understands what I'm on about. :-\

Consider an EmDrive in free space and accelerating. We switch it off temporarily and let it coast at speed v relative to the inertial frame in which it began its acceleration. When we switch it back on, are we going to assert that somehow the thrust F  knows what speed it's going and adjusts the thrust like F = P/v?

I assert again that this kind of thinking requires a preferred frame, and thus violates SR.

First, there are two different points here. You seem to want to use this example to invalidate SR. I can help you with that. SR is only valid in an inertial reference frame. However, the quantum vacuum permits a continuum of inertial reference frames, each with a different value of the speed of light, depending on the local gravitational field. These are called "local tangent planes" and on them we "define" c = 1, but if we compare them, we find that c in one frame, is c/K at another altitude in the field. So yes, it does violate the usual idea of SR because it is traveling at "v" relative to it's preferred rest frame. But it's rest frame is not absolute! It is relative to the gravitational field "vacuum energy" at the location it started from.

In your analysis, you did not specify any losses. Therefore, Ein = Eout at all times, t, not just when it gets to "break even". I think I found the error in your analysis. You said,

v = a*t = (F/m)*t

This statement "assumes" acceleration is a constant. It is not a constant, so this must be replaced by the integral.

v(t) = integral[a(t)]*dt

When put it in integral form, we end up with the same Work integral I presented earlier. Your value of;

Eout = .5*m*v0^2

is assuming acceleration is constant from 0 to v0. That is the where the assumption is made. The integral I presented (from Wikipedia) is integrating "Force x Distance", where Power in = Power out. THAT is the correct way to do the calculation. There is no paradox here if you understand the quantum vacuum correctly.

Todd D.
Title: Re: EM Drive Developments - related to space flight applications - Thread 2
Post by: deltaMass on 05/07/2015 01:49 am
Quote from: WarpTech
I think I found the error in your analysis. You said,
v = a*t = (F/m)*t
This statement "assumes" acceleration is a constant. It is not a constant, so...
It is constant if and only if F is a constant of the motion.  Which, as I have argued with recourse to SR, it indeed is.

Did you find any other mistakes?
Title: Re: EM Drive Developments - related to space flight applications - Thread 2
Post by: deltaMass on 05/07/2015 01:58 am
I must reject the analysis by J & W in Appendix A, but thanks for the link.

I have already mentioned in my preamble that not only is any propellantless propulsion craft capable of perpetuum mobile operation, but that free energy is available on top of that to boot.

This causes many people to break out in hives, or to resort to chewing their towels.  ::)

We can of course test to see this at work. We use a rotary configuration and when the tangential velocity exceeds 2/k, we switch from external power to a coaxially mounted generator. And then begin charging people for the juice. Outrageous!  8)

I would think the free energy speculation is a bit premature.  The "frame of reference problem" could inhibit any constrained mechanism, not that such can be calculated until the momentum question is resolved.
To your first sentence:
If by that you mean the engineering issues, then I couldn't agree more, and have already estimated here the performance gap which exists. Feel free to run your own numbers; I reckon currently we're about a factor of 50 down on breakeven.
But if by that you mean the physics, I must needs take issue. Have you looked at string theory lately?  8)

I'm sorry, but I don't really understand your second sentence. Perhaps you could say it another way?
Title: Re: EM Drive Developments - related to space flight applications - Thread 2
Post by: WarpTech on 05/07/2015 02:13 am
Quote from: WarpTech
I think I found the error in your analysis. You said,
v = a*t = (F/m)*t
This statement "assumes" acceleration is a constant. It is not a constant, so...
It is constant if and only if F is a constant of the motion.  Which, as I have argued with recourse to SR, it indeed is.

Did you find any other mistakes?

No, and I hear you. I believe energy is Force x Distance and Power in = Power out, and energy is conserved. I also believe SR is an "approximation" to a more accurate theory that includes the relative energy of the local quantum vacuum. That is how my model works, because that is how the Math in GR and QED tells us it should work.

Todd D.
Title: Re: EM Drive Developments - related to space flight applications - Thread 2
Post by: Notsosureofit on 05/07/2015 02:14 am
I must reject the analysis by J & W in Appendix A, but thanks for the link.

I have already mentioned in my preamble that not only is any propellantless propulsion craft capable of perpetuum mobile operation, but that free energy is available on top of that to boot.

This causes many people to break out in hives, or to resort to chewing their towels.  ::)

We can of course test to see this at work. We use a rotary configuration and when the tangential velocity exceeds 2/k, we switch from external power to a coaxially mounted generator. And then begin charging people for the juice. Outrageous!  8)

I would think the free energy speculation is a bit premature.  The "frame of reference problem" could inhibit any constrained mechanism, not that such can be calculated until the momentum question is resolved.
To your first sentence:
If by that you mean the engineering issues, then I couldn't agree more, and have already estimated here the performance gap which exists. Feel free to run your own numbers; I reckon currently we're about a factor of 50 down on breakeven.
But if by that you mean the physics, I must needs take issue. Have you looked at string theory lately?  8)

I'm sorry, but I don't really understand your second sentence. Perhaps you could say it another way?

Admittedly,  I'm sitting here rereading my Einstein after an absence of 50 years so take me w/ a pound of salt until I catch up.

The acceleration in the frame of a free cavity system can be a "constant" added to the background gravitational gradient (or variation in the index of refraction if you prefer)  So GR.  "Free Energy" if you want to call it that as you accelerate away in non-local fashion.

So far, of course, the experiments have been about (nearly) static forces which involve little or no work.

Once the cavity is constrained into some fixture that ties it to a fixed frame of reference we become dependent on the type of mechanism (and it's GR behavior) which is responsible for the change in momentum of the cavity.   White's mechanism, (I'm not particularly fond of it)  for example, will not yield free energy.  Some unknown version might, but I don't see it as likely yet.

Title: Re: EM Drive Developments - related to space flight applications - Thread 2
Post by: LasJayhawk on 05/07/2015 02:18 am
Health and safety, another concern.

You will be dealing with voltages that could be lethal. Making sure your frustum case is at ground, and not several thousand volts above it, would be advised.
Title: Re: EM Drive Developments - related to space flight applications - Thread 2
Post by: deltaMass on 05/07/2015 02:19 am
Quote from: WarpTech
I think I found the error in your analysis. You said,
v = a*t = (F/m)*t
This statement "assumes" acceleration is a constant. It is not a constant, so...
It is constant if and only if F is a constant of the motion.  Which, as I have argued with recourse to SR, it indeed is.

Did you find any other mistakes?

No, and I hear you. I believe energy is Force x Distance and Power in = Power out, and energy is conserved. I also believe SR is an "approximation" to a more accurate theory that includes the relative energy of the local quantum vacuum. That is how my model works, because that is how the Math in GR and QED tells us it should work.

Todd D.
My mention of SR is simply in order to highlight a core principle of Einstein's thinking about space and time; to whit, there is no preferred inertial frame, such that physics there is different to physics in another one.

Are you really saying that you reject this?
Title: Re: EM Drive Developments - related to space flight applications - Thread 2
Post by: Einstein79 on 05/07/2015 02:48 am
The reason for the confusion over the violation of classical physics is because this system has nothing to do with classical physics. Moreover, the “thrust” that is being calculated is not thrust at all but space moving the drive from one position to another which can merely be related to thrust but is not, per se, thrust. The controlling factor here is, of course, the resonant frequency. If you match the resonant frequency that space uses to “hold” the object you will develop a “cavity” that the “object will move towards”. The reason why the device cannot be “pushed off of” for conservation of momentum to hold true is because space is already pushing on it satisfying the law.

A couple of postulates to keep in mind that will help with these experiments are:
1. Space creates light.
2. Space itself is a resonating chamber.
Title: Re: EM Drive Developments - related to space flight applications - Thread 2
Post by: deltaMass on 05/07/2015 02:50 am
Quote from: Notsosureofit
Once the cavity is constrained into some fixture that ties it to a fixed frame of reference we become dependent on the type of mechanism (and it's GR behavior) which is responsible for the change in momentum of the cavity.
Indeed. I was limiting my discussion to a device freely moving under its own power in free space. Clearly, bolting it down to a lab vac chamber does not satisfy that criterion.

I trust you appreciate how I've "black boxed" propellantless propulsion devices - any and all of them. What I've said doesn't depend on what kind of device it is, nor upon any particular pet theory used to explain its "propellantlessness".
Title: Re: EM Drive Developments - related to space flight applications - Thread 2
Post by: deltaMass on 05/07/2015 02:52 am
The reason for the confusion over the violation of classical physics is because this system has nothing to do with classical physics. Moreover, the “thrust” that is being calculated is not thrust at all but space moving the drive from one position to another which can merely be related to thrust but is not, per se, thrust. The controlling factor here is, of course, the resonant frequency. If you match the resonant frequency that space uses to “hold” the object you will develop a “cavity” that the “object will move towards”. The reason why the device cannot be “pushed off of” for conservation of momentum to hold true is because space is already pushing on it satisfying the law.

A couple of postulates to keep in mind that will help with these experiments are:
1. Space creates light.
2. Space itself is a resonating chamber.
Interesting! Would you then be prepared to write down the equations of motion so that we can play with them?
Title: Re: EM Drive Developments - related to space flight applications - Thread 2
Post by: WarpTech on 05/07/2015 03:21 am
Quote from: WarpTech
I think I found the error in your analysis. You said,
v = a*t = (F/m)*t
This statement "assumes" acceleration is a constant. It is not a constant, so...
It is constant if and only if F is a constant of the motion.  Which, as I have argued with recourse to SR, it indeed is.

Did you find any other mistakes?

No, and I hear you. I believe energy is Force x Distance and Power in = Power out, and energy is conserved. I also believe SR is an "approximation" to a more accurate theory that includes the relative energy of the local quantum vacuum. That is how my model works, because that is how the Math in GR and QED tells us it should work.

Todd D.
My mention of SR is simply in order to highlight a core principle of Einstein's thinking about space and time; to whit, there is no preferred inertial frame, such that physics there is different to physics in another one.

Are you really saying that you reject this?

As I said, it is not applicable. "Special" Relativity is the special case that ONLY applies when we have a Minkowski metric, such as;

ds^2 = -(c*dt)^2 + (dx^2 + dy^2 + dz^2)

When we compare 2 inertial frames using Lorentz Transformations, both frames have the same Minkowski metric signature because they are indistinguishable.

However, consider the metric,

ds^2 = -(1/K)*(c*dt)^2 + K*(dx^2 + dy^2 + dz^2)

where, K is a "constant" different from 1, over all coordinates. This metric is just as Flat as Minkowski's. However, it has been scaled to a different maximum speed-limit, c/K rather than c. THIS is no longer SR! It is a general case, so we must use General Relativity. In GR, the speed of light is not constant in accelerated reference frames, and when you accelerate from one inertial frame to another, they are NOT equal, just as changing altitude in a gravitational field can be modeled as a variable Refractive Index where two adjacent tangent planes are NOT equal when you compare them. Rulers and clocks are scaled by acceleration.

When matter is accelerated, length contracts and time dilates, this is a scale transformation. When the thrust is turned off and the coasting rocket finds itself is at rest relative to some distant planet. It's length does not spring back to "normal" and the clock does not speed up. They remain in this relative state until thrust is reversed and they return to the same vacuum energy state they started from. For accelerating matter, the vacuum's refractive index increases until wave velocity goes to zero and time stops. A true warp drive, modifies the vacuum to flatten the local light-cone, reducing the refractive index to compensate for this effect, thereby raising the speed limit.

Best Regards,
Todd


Title: Re: EM Drive Developments - related to space flight applications - Thread 2
Post by: deltaMass on 05/07/2015 03:31 am
I'm sorry, but I can only entertain a further discussion if we restrict ourselves to severely subrelativistic (i.e. slow) scenarios - since that is a constraint I've imposed upon myself for the purposes of the most elementary possible discussion of the dynamics.

In that framework then, and assuming (to first order, of course) a flat spacetime, do you now agree with Einstein's assertion about physics in inertial frames?
Title: Re: EM Drive Developments - related to space flight applications - Thread 2
Post by: WarpTech on 05/07/2015 03:41 am
Quote from: Notsosureofit
Once the cavity is constrained into some fixture that ties it to a fixed frame of reference we become dependent on the type of mechanism (and it's GR behavior) which is responsible for the change in momentum of the cavity.
Indeed. I was limiting my discussion to a device freely moving under its own power in free space. Clearly, bolting it down to a lab vac chamber does not satisfy that criterion.

I trust you appreciate how I've "black boxed" propellantless propulsion devices - any and all of them. What I've said doesn't depend on what kind of device it is, nor upon any particular pet theory used to explain its "propellantlessness".

Hypothetically, if there were a black box with a gravitational field "inside", i.e., it has a NET acceleration vector along the X axis, pointing toward the blue side of the box..., (which is opposite the red side of the box)  but has no discernible gravitational field "outside" of the box other than what a normal box of that mass would have. Let's say that inside the box there is all the equipment and energy storage, necessary to generate this field. Nothing comes in or goes out, but the energy stored inside it (battery) is being dissipated without being expelled.

What sort of motion would YOU expect to see?

1. Will it move forward with the blue side leading?

2. Will it move forward with the red side leading?

3. Will it not move at all because nothing is coming out?

Keep in mind, by definition, it has a NET acceleration vector inside along the X axis.

Todd

Title: Re: EM Drive Developments - related to space flight applications - Thread 2
Post by: dustinthewind on 05/07/2015 03:46 am
Quote from: WarpTech
I think I found the error in your analysis. You said,
v = a*t = (F/m)*t
This statement "assumes" acceleration is a constant. It is not a constant, so...
It is constant if and only if F is a constant of the motion.  Which, as I have argued with recourse to SR, it indeed is.

Did you find any other mistakes?

No, and I hear you. I believe energy is Force x Distance and Power in = Power out, and energy is conserved. I also believe SR is an "approximation" to a more accurate theory that includes the relative energy of the local quantum vacuum. That is how my model works, because that is how the Math in GR and QED tells us it should work.

Todd D.
My mention of SR is simply in order to highlight a core principle of Einstein's thinking about space and time; to whit, there is no preferred inertial frame, such that physics there is different to physics in another one.

Are you really saying that you reject this?

What about with respect to the "Cosmic Microwave Background Radiation" CMBR?  Couldn't we say the universe has this as an absolute frame? 

https://scholar.google.com/scholar?cluster=16121186223305818545&hl=en&as_sdt=0,48

Lets say there is a God view looking above far from all gravitational fields observing the universe and stationary with respect to its boundaries.  (It should have a finite size if it had a beginning and an expansion.)  Light falling into gravity fields I would think should slow down, (considering the limit when light reaches an event horizon).  From inside a gravity field I should think light should still appear to be going c due the shrinking of the ruller.  This could give the appearance of the index of refraction (gravitational lensing).  Two objects traveling towards each other @ .6c still appear from the God view to be approaching at 1.2c though I suppose the two observers both have their (space/time) warped so it appears to them they are not approaching each other at 1.2c but rather v<c.  I mean sure time/space screws our perceptions all up but why not have an absolute frame of the universe or CMB where either we are moving with respect to it or we arent?  Or am I missing something. 

http://arxiv.org/abs/physics/0306196

Title: Re: EM Drive Developments - related to space flight applications - Thread 2
Post by: WarpTech on 05/07/2015 03:53 am
I'm sorry, but I can only entertain a further discussion if we restrict ourselves to severely subrelativistic (i.e. slow) scenarios - since that is a constraint I've imposed upon myself for the purposes of the most elementary possible discussion of the dynamics.

In that framework then, and assuming (to first order, of course) a flat spacetime, do you now agree with Einstein's assertion about physics in inertial frames?

Okay then. If we are restricting ourselves to Newtonian mechanics, then we have a paradox that cannot be resolved. It leads to either an over-unity device or a preferred reference frame, and momentum is not conserved.

However, if we honestly want to resolve the paradox and conserve momentum, then we must use General Relativity to solve the problem, regardless of how fast it is going.

I'm sorry, that's just the way it is. There is no Newtonian resolution for this argument.

Thank you.
Todd
Title: Re: EM Drive Developments - related to space flight applications - Thread 2
Post by: PushHigher on 05/07/2015 03:57 am
When matter is accelerated, length contracts and time dilates, this is a scale transformation. When the thrust is turned off and the coasting rocket finds itself is at rest relative to some distant planet. It's length does not spring back to "normal" and the clock does not speed up. They remain in this relative state until thrust is reversed and they return to the same vacuum energy state they started from.

WarpTech,

From my understanding the length contraction and time dialation (warping of space-time) is based on velocity.  From the perspective of point A, if a rocket zooms past it at constant velocity it will display the warped effect.  Lets say Planet B already matches the direction and velocity of the rocket and the rocket slows down relative to planet A.  Planet B would see the time dilation and length contraction start to change and that change would stop when deceleration of the rocket stops relative to planet A (at the same time - it is also accelerating away from planet B).

It is my personal feeling, and not something I've read nor can quantify - that acceleration has a sort of friction against space-time because it needs to bend it.  The EM Drive (if it works) bypasses this by spending it's energy on bending space-time rather than accelerating matter.  It is in essence falling.  I believe you are proving this but I may be wrong.

The only way I can explain this away is that the EM Drive creates two gravity wells with the same energy level on both sides.  The rear one is tightly focused and deep and the front one is broad and weak.  Both are centered within the frustum near the ends.  The reason there is motion is due to the larger/weaker gravity well extending beyond the bounds of the frustum more prominently in the front than the rear.  The rear well would be spending most of it's energy tugging at the walls of the frustum.  I cannot prove any of this so please consider it food for thought.

I love reading your posts, please keep up the great work!
Title: Re: EM Drive Developments - related to space flight applications - Thread 2
Post by: WarpTech on 05/07/2015 04:02 am
Quote from: WarpTech
I think I found the error in your analysis. You said,
v = a*t = (F/m)*t
This statement "assumes" acceleration is a constant. It is not a constant, so...
It is constant if and only if F is a constant of the motion.  Which, as I have argued with recourse to SR, it indeed is.

Did you find any other mistakes?

No, and I hear you. I believe energy is Force x Distance and Power in = Power out, and energy is conserved. I also believe SR is an "approximation" to a more accurate theory that includes the relative energy of the local quantum vacuum. That is how my model works, because that is how the Math in GR and QED tells us it should work.

Todd D.
My mention of SR is simply in order to highlight a core principle of Einstein's thinking about space and time; to whit, there is no preferred inertial frame, such that physics there is different to physics in another one.

Are you really saying that you reject this?

What about with respect to the "Cosmic Microwave Background Radiation" CMBR?  Couldn't we say the universe has this as an absolute frame? 

https://scholar.google.com/scholar?cluster=16121186223305818545&hl=en&as_sdt=0,48

Lets say there is a God view looking above far from all gravitational fields observing the universe and stationary with respect to its boundaries.  (It should have a finite size if it had a beginning and an expansion.)  Light falling into gravity fields I would think should slow down, (considering the limit when light reaches an event horizon).  From inside a gravity field I should think light should still appear to be going c due the shrinking of the ruller.  This could give the appearance of the index of refraction (gravitational lensing).  Two objects traveling towards each other @ .6c still appear from the God view to be approaching at 1.2c though I suppose the two observers both have their (space/time) warped so it appears to them they are not approaching each other at 1.2c but rather v<c.  I mean sure time/space screws our perceptions all up but why not have an absolute frame of the universe or CMB where either we are moving with respect to it or we arent?  Or am I missing something.

Congratulations! You have just "correctly" described the Polarizable Vacuum Model of General Relativity. However, we do not need the CMBR. We simply define an observer "at infinity" to have a refractive index, K=1. From that perspective, he can observe the relative value of K, for all gravitational fields, where K > 1. If he sees something moving FTL, he will assign that space with a relative value of, K < 1.

It is still not a preferred frame, because K can be "defined" as 1 anywhere, and all observations are relative to that definition. Mark Millis has said that the CMBR is a preferred frame. IMO, it doesn't make the Math any easier, it just confuses people. As an engineer, I see all things as relative and take everything with a grain of salt. :)

Todd
Title: Re: EM Drive Developments - related to space flight applications - Thread 2
Post by: MrVibrating on 05/07/2015 04:04 am
@deltaMass

Glad you've broached the issue of net efficiency - i mentioned it previously but it sank like a lead baloon...

Thought experiment:

Suppose we sat our EM drive in an accurate calorimeter.  Switched on but restrained stationary, all of the input energy goes to heat via Joule's 2nd law.

If however we allow the drive to accelerate, then this further raises the temperature inside our perfectly insulated box.   Yet there's no causal reason why this should reduce the resistive losses - IOW this KE gain doesn't subtract from those baseline losses - for a given input energy they remain constant...

It costs nothing, in principle, to merely render a field.  Only if that field's force then performs work does that work present as a load upon the input energy.

Calorimetry of the stationary vs accelerating drive could prove extremely interesting...

My hunch is that, as it is the virtual photonsphere that mediates the EM interaction, then it is likewise responsible for the KE gain, essentially converting ambient momentum into mechanical energy.  The same conclusion would apply even if the reaction matter were virtual electrons / protons as has been postulated, since it's still virtual photons carrying the forces between them, and the drive.

Getting an N3 violation out of an energy symmetry break is seemingly impossible - the latter doesn't even speak to the former's problem.  But conversely, getting free energy from an N3 break is trivial, as you note.  Equally, this applies to the EM equivalent of N3 - CEMF / BEMF...  if you can spin a rotor without incurring counter EMF or back EMF, you gain 'free' energy...

Ultimately such asymmetric systems aren't thermodynamically closed at all, hence it's not really 'free'... rather they're in open circuit with the vacuum energy, sinking or sourcing energy to or from it, depending on the direction of the asymmetry.

Incidentally, if any of this does bear out, the EM drive presents a novel set of risks in that, while anchored to Earth, it can be used to affect the planet's KE (however infinitesimaly) - unless a pair of them were set in opposition... at sufficiently high powers though we could make significant alterations, given enough time.   The other risk pertains to perturbing, even very slightly, the balance of thermodynamic and vacuum energies and thus any constants that may be the result of delicate a homeostasis between them, such as the Higgs or alpha, as outlined in this article from PhyOrg a couple of years back:

http://phys.org/news/2013-12-collapse-universe-closer.html

Scaremongering no doubt, but raises the comic spectacle of the future dregs of humanity winging it across the cosmos Battlestar Galactica style, desperately trying to outrun the big rip their own drive systems are accelerating...   

But i digress.  Calorimetry.  That's the thing.
Title: Re: EM Drive Developments - related to space flight applications - Thread 2
Post by: WarpTech on 05/07/2015 04:10 am
When matter is accelerated, length contracts and time dilates, this is a scale transformation. When the thrust is turned off and the coasting rocket finds itself is at rest relative to some distant planet. It's length does not spring back to "normal" and the clock does not speed up. They remain in this relative state until thrust is reversed and they return to the same vacuum energy state they started from.

WarpTech,

From my understanding the length contraction and time dialation (warping of space-time) is based on velocity.  From the perspective of point A, if a rocket zooms past it at constant velocity it will display the warped effect.  Lets say Planet B already matches the direction and velocity of the rocket and the rocket slows down relative to planet A.  Planet B would see the time dilation and length contraction start to change and that change would stop when deceleration of the rocket stops relative to planet A (at the same time - it is also accelerating away from planet B).

It is my personal feeling, and not something I've read nor can quantify - that acceleration has a sort of friction against space-time because it needs to bend it.  The EM Drive (if it works) bypasses this by spending it's energy on bending space-time rather than accelerating matter.  It is in essence falling.  I believe you are proving this but I may be wrong.

The only way I can explain this away is that the EM Drive creates two gravity wells with the same energy level on both sides.  The rear one is tightly focused and deep and the front one is broad and weak.  Both are centered within the frustum near the ends.  The reason there is motion is due to the larger/weaker gravity well extending beyond the bounds of the frustum more prominently in the front than the rear.  The rear well would be spending most of it's energy tugging at the walls of the frustum.  I cannot prove any of this so please consider it food for thought.

I love reading your posts, please keep up the great work!

Sort of.. A black hole is a gravity well. When light falls into a black hole, wave velocity goes to zero and the energy cannot escape. The momentum is absorbed by the black hole.

Inside the frustum, toward the large end is a reflector. Toward the small end, the taper is a high-pass filter. As waves are attenuated, they shift toward longer wavelengths and are cut-off by the dimensions of the waveguide. The cut-off is analogous to a black hole. The wave velocity goes to zero and that end of the frustum absorbs the momentum because it can't escape. There is only 1 gravity well inside it. That is in the direction of the lowest group velocity.

Thank you!
Todd
Title: Re: EM Drive Developments - related to space flight applications - Thread 2
Post by: deltaMass on 05/07/2015 04:16 am
I'm sorry, but I can only entertain a further discussion if we restrict ourselves to severely subrelativistic (i.e. slow) scenarios - since that is a constraint I've imposed upon myself for the purposes of the most elementary possible discussion of the dynamics.

In that framework then, and assuming (to first order, of course) a flat spacetime, do you now agree with Einstein's assertion about physics in inertial frames?

Okay then. If we are restricting ourselves to Newtonian mechanics, then we have a paradox that cannot be resolved. It leads to either an over-unity device or a preferred reference frame, and momentum is not conserved.

However, if we honestly want to resolve the paradox and conserve momentum, then we must use General Relativity to solve the problem, regardless of how fast it is going.

I'm sorry, that's just the way it is. There is no Newtonian resolution for this argument.

Thank you.
Todd
Excellent. Then we are in complete agreement. Phew. Let's let this sink in for everyone else.



If we are restricting ourselves to Newtonian mechanics, then we have a paradox that cannot be resolved. It leads to either an over-unity device or a preferred reference frame, and momentum is not conserved.



I know you're keen to wander off into PV, GR and QCD. I am content to stay with Newton and Galileo for the nonce,  "unambitiously" trolling around the solar system without carrying mondo tanks of fuel. Appreciable fractions of the speed of light can wait until later. This I say because this is how we begin to actually use a device like the EmDrive. We don't immediately leap off on an interstellar adventure (although I would welcome that too!).

I have suggested a resolution of the paradox - experiment. I think that's in the best scientific tradition.
I've also suggested the sort of apparatus we'd need to test the issue.
Title: Re: EM Drive Developments - related to space flight applications - Thread 2
Post by: LasJayhawk on 05/07/2015 04:26 am
Quote from: WarpTech
I think I found the error in your analysis. You said,
v = a*t = (F/m)*t
This statement "assumes" acceleration is a constant. It is not a constant, so...
It is constant if and only if F is a constant of the motion.  Which, as I have argued with recourse to SR, it indeed is.

Did you find any other mistakes?

No, and I hear you. I believe energy is Force x Distance and Power in = Power out, and energy is conserved. I also believe SR is an "approximation" to a more accurate theory that includes the relative energy of the local quantum vacuum. That is how my model works, because that is how the Math in GR and QED tells us it should work.

Todd D.
My mention of SR is simply in order to highlight a core principle of Einstein's thinking about space and time; to whit, there is no preferred inertial frame, such that physics there is different to physics in another one.

Are you really saying that you reject this?

What about with respect to the "Cosmic Microwave Background Radiation" CMBR?  Couldn't we say the universe has this as an absolute frame? 

https://scholar.google.com/scholar?cluster=16121186223305818545&hl=en&as_sdt=0,48

Lets say there is a God view looking above far from all gravitational fields observing the universe and stationary with respect to its boundaries.  (It should have a finite size if it had a beginning and an expansion.)  Light falling into gravity fields I would think should slow down, (considering the limit when light reaches an event horizon).  From inside a gravity field I should think light should still appear to be going c due the shrinking of the ruller.  This could give the appearance of the index of refraction (gravitational lensing).  Two objects traveling towards each other @ .6c still appear from the God view to be approaching at 1.2c though I suppose the two observers both have their (space/time) warped so it appears to them they are not approaching each other at 1.2c but rather v<c.  I mean sure time/space screws our perceptions all up but why not have an absolute frame of the universe or CMB where either we are moving with respect to it or we arent?  Or am I missing something. 

http://arxiv.org/abs/physics/0306196
I brought this up in the other thread. In my mind the CMBR is our zero point for energy. Anything below it would look like negative energy from our frame of reference. Oddly enough, only the tests that had an odd harmonic close to 160200 MHz worked. I just believe that there is a "bingo frequency" that will make this thing go.

But no I can't provide the math. I've lost my slip-stick and walmart doesn't have log paper. :(
Title: Re: EM Drive Developments - related to space flight applications - Thread 2
Post by: PushHigher on 05/07/2015 04:32 am
Inside the frustum, toward the large end is a reflector. Toward the small end, the taper is a high-pass filter. As waves are attenuated, they shift toward longer wavelengths and are cut-off by the dimensions of the waveguide. The cut-off is analogous to a black hole. The wave velocity goes to zero and that end of the frustum absorbs the momentum because it can't escape. There is only 1 gravity well inside it. That is in the direction of the lowest group velocity.

What perplexes me is that the latest improvements to Shawyer's design have been to optimize reflectivity.  Which to me seems that the goal is to accumulate the energy within the frustum rather than trying to absorb the energy by capturing momentum.  In your opinion - does one exclude the other?
Title: Re: EM Drive Developments - related to space flight applications - Thread 2
Post by: WarpTech on 05/07/2015 04:56 am
Inside the frustum, toward the large end is a reflector. Toward the small end, the taper is a high-pass filter. As waves are attenuated, they shift toward longer wavelengths and are cut-off by the dimensions of the waveguide. The cut-off is analogous to a black hole. The wave velocity goes to zero and that end of the frustum absorbs the momentum because it can't escape. There is only 1 gravity well inside it. That is in the direction of the lowest group velocity.

What perplexes me is that the latest improvements to Shawyer's design have been to optimize reflectivity.  Which to me seems that the goal is to accumulate the energy within the frustum rather than trying to absorb the energy by capturing momentum.  In your opinion - does one exclude the other?

I haven't read up on everything Shawyer has done yet. I have a lot of catching up to do. However, a narrow taper  that is more cylindrical will have a higher Q and resonate well, but it will have very little attenuation to provide thrust. Too much taper and it will be difficult to sustain resonance. Ideally, a balance needs to be engineered between how much energy can be stored in resonant modes, and how much needs to be attenuated to provide thrust. The higher the Q, the higher the reserve so increasing reflectivity reduces losses to heat. He's trying to maximize the energy stored, which is correct so that the percentage of attenuated modes will go up too.

Todd

Title: Re: EM Drive Developments - related to space flight applications - Thread 2
Post by: deltaMass on 05/07/2015 05:51 am
Consider the following thought experiment which takes place in free space (insignificant fields, flat spacetime). We have two propellantless propulsion craft A, B. They are identical in that a given input power P produces the same static thrust F in both of them, and their masses are the same too.

We let them both fly off on parallel courses in the same direction while both consume input power P. When A reaches a speed of 1 m/s, we switch it off and let it coast. Later, when B reaches 10 m/s, we also switch it off. Now both are coasting at their respective speeds. We switch them both back on at roughly the same time at the same input power level P. We measure the new accelerations of each and from these (knowing their masses) we deduce the thrusts acting via F = m a.

As discussed by WarpTech, Rodal and me, one of two things can happen:

1.  Energy is conserved, but there exists a preferred rest frame.
Because F = P/v, we must expect a 10:1 ratio between the two thrusts due to the different velocities.

2. Overunity will eventually obtain at v >= 2/k. There exists no preferred frame of reference.
Because F = k P and both k, P are constant, we must expect the two thrusts to be identical.

You choose how you expect the experiment to go.

Here's my thinking.
Both craft are coasting inertially at extremely low speed. Both craft, when switched back on, should exert the same thrust because their physics is identical. They have no way of knowing their speed because nothing here is remotely relativistic. There is no relativity to come to the rescue at 10 metres per second in a field-free flat spacetime. The gamma factor is 1.0000000000001 or suchlike. F = P/v is a ridiculous equation in this context. Therefore I vote for #2.

Wanna buy some Free Energy?  8)

Title: Re: EM Drive Developments - related to space flight applications - Thread 2
Post by: WarpTech on 05/07/2015 05:56 am
It's good that at least one person understands what I'm on about. :-\

Consider an EmDrive in free space and accelerating. We switch it off temporarily and let it coast at speed v relative to the inertial frame in which it began its acceleration. When we switch it back on, are we going to assert that somehow the thrust F  knows what speed it's going and adjusts the thrust like F = P/v?

I assert again that this kind of thinking requires a preferred frame, and thus violates SR.

Or you could also give the example of EM Drive ship 1 (which started from a different place) being overtaken by EM Drive ship 2 ...

EDIT:

See White and Joosten, Appendix :

http://ntrs.nasa.gov/archive/nasa/casi.ntrs.nasa.gov/20140013174.pdf

I just read this... Oh my!

deltaMass, I owe you an apology. I thought this was your derivation. I did not realize they published it that way. It was an assumption right from the start that acceleration is constant for constant input power. A "what if?" scenario. Fine, that's how the game is played if you want to get funded.

Thank you.
Todd

Title: Re: EM Drive Developments - related to space flight applications - Thread 2
Post by: deltaMass on 05/07/2015 06:12 am
@WarpTech:
No problem! You are a worthy debater indeed and it sharpens both of us up to kick the can around like this (pun intended  ;D).

In case you didn't see my response to the doc - I don't accept the content of Appendix A for reasons that by now should be crystal clear to you.

Title: Re: EM Drive Developments - related to space flight applications - Thread 2
Post by: TheTraveller on 05/07/2015 06:25 am
Interesting way to do an experiment:
https://www.youtube.com/watch?v=7yNWWvl5zpg
Stream it live via You Tube and include real time data logging.

This is about MFMP's replication of the Parkhomov LENR results, so sure OFF TOPIC, but how they are doing their testing is not: https://www.facebook.com/MartinFleischmannMemorialProject

When I get my test rig up and working, will do this. Others may also like to consider going live with their EM Drive experiments. BTW using this media channel, many can join the conversation using audio and/or textual comments, so it becomes a 2 way event. All recorded, so there forever.
Title: Re: EM Drive Developments - related to space flight applications - Thread 2
Post by: deltaMass on 05/07/2015 06:32 am
Quote from: WarpTech
Hypothetically, if there were a black box with a gravitational field "inside", i.e., it has a NET acceleration vector along the X axis, pointing toward the blue side of the box..., (which is opposite the red side of the box)  but has no discernible gravitational field "outside" of the box other than what a normal box of that mass would have. Let's say that inside the box there is all the equipment and energy storage, necessary to generate this field. Nothing comes in or goes out, but the energy stored inside it (battery) is being dissipated without being expelled.

What sort of motion would YOU expect to see?

1. Will it move forward with the blue side leading?

2. Will it move forward with the red side leading?

3. Will it not move at all because nothing is coming out?

Keep in mind, by definition, it has a NET acceleration vector inside along the X axis.

Todd
I must confess that I don't understand this system. It seems to suggest that a gravitational field can be generated at will de novo and also shielded. I wouldn't know where to start with something like that.
Title: Re: EM Drive Developments - related to space flight applications - Thread 2
Post by: StrongGR on 05/07/2015 06:46 am
Hi guys, on the theoretical side, has someone looked into this:


http://arxiv.org/abs/hep-th/0508246
Using strong external fields to modify locally a space-time

I wrote this paper. It went published in IJMPD thanks to the Editor Jorge Pullin. The example given there can well represent a traveling microwave with an electric field in the dipole approximation. Something of interest to discuss here. This approach could be also used for the device discussed here. I am evaluating this.
Title: Re: EM Drive Developments - related to space flight applications - Thread 2
Post by: TheTraveller on 05/07/2015 07:02 am
If my EM Drive experiment generates significant thrust, it will be time to move onto the next experiment which will answer the COE question.

Plan is to build a rotary test device that drives a DC generator and it a variable load. Then can vary the load on the DC generator and observe changes in the energy delivered by the power supply to the RF amp versus energy consumed by the load. All data logged. Will draw a line in the sand about if an EM Drive obeys COE while it delivers energy to it's load.

While you guys debate theory, which is good, I've engaged the process to turn my initial test plans into reality.

My goal is to give you some solid experimental data on which to move forward.
Title: Re: EM Drive Developments - related to space flight applications - Thread 2
Post by: deltaMass on 05/07/2015 07:07 am
If my EM Drive experiment generates significant thrust, it will be time to move onto the next experiment which will answer the COE question.

Plan is to build a rotary test device that drives a DC generator and it a variable load. Then can vary the load on the DC generator and observe changes in the energy delivered by the power supply to the RF amp versus energy consumed by the load. All data logged. Will draw a line in the sand about if an EM Drive obeys COE while it delivers energy to it's load.
It is going to be a mechanical challenge! I wish you all the best though.

And you are quite correct that such a rotary device can answer the CofE question without going all the way up to breakeven. It is a rotary version of the thought experiment I wrote about above. The principle of operation can be tested at quite low speeds. For some stupid brainfarty reason I had imagined that one had to go right up to breakeven speed.

If you believe Appendix A (and I hope you don't) then you do have to get up to breakeven and beyond, because of the strange knee function that is proposed there. It can't be like that.
Title: Re: EM Drive Developments - related to space flight applications - Thread 2
Post by: TheTraveller on 05/07/2015 07:29 am
If my EM Drive experiment generates significant thrust, it will be time to move onto the next experiment which will answer the COE question.

Plan is to build a rotary test device that drives a DC generator and it a variable load. Then can vary the load on the DC generator and observe changes in the energy delivered by the power supply to the RF amp versus energy consumed by the load. All data logged. Will draw a line in the sand about if an EM Drive obeys COE while it delivers energy to it's load.
It is going to be a mechanical challenge! I wish you all the best though.

And you are quite correct that such a rotary device can answer the CofE question without going all the way up to breakeven. It is a rotary version of the thought experiment I wrote about above. The principle of operation can be tested at quite low speeds. For some stupid brainfarty reason I had imagined that one had to go right up to breakeven speed.

If you believe Appendix A (and I hope you don't) then you do have to get up to breakeven and beyond, because of the strange knee function that is proposed there. It can't be like that.
Rotary test rig is not that hard or complex to build. Will be self powered with recharge Lithium batts as the counter balance and 2 way WiFi data links. No wires to worry about. So many really nice & inexpensive tools today.

Oh have added a Faraday Cage around the EM Drive.

What I believe is "Follow the Data, Ignore the Theory".
Title: Re: EM Drive Developments - related to space flight applications - Thread 2
Post by: deltaMass on 05/07/2015 07:31 am
The main attraction would be to get an EmDrive to accelerate a wheel from rest, period.
That would be a Red Letter Day indeed - irrespective of the CofE issue.
That would be headline news.

Because of the coax RF feed I assume you'll mount the RF amp and any associated electronics next to the cavity?
Title: Re: EM Drive Developments - related to space flight applications - Thread 2
Post by: TheTraveller on 05/07/2015 07:39 am
The main attraction would be to get an EmDrive to accelerate a wheel from rest, period.
That would be a Red Letter Day indeed - irrespective of the CofE issue.
That would be headline news.

Because of the coax RF feed I assume you'll mount the RF amp and any associated electronics next to the cavity?
Already been done.
http://emdrive.com/dynamictests.html
Watch the videos.

RF amp will be mounted next to the cavity but outside the Faraday Cage. If it leaks too much RF, will put it in it's own Faraday Cage.

Note this comment.
Title: Re: EM Drive Developments - related to space flight applications - Thread 2
Post by: RotoSequence on 05/07/2015 07:44 am
The main attraction would be to get an EmDrive to accelerate a wheel from rest, period.
That would be a Red Letter Day indeed - irrespective of the CofE issue.
That would be headline news.

Because of the coax RF feed I assume you'll mount the RF amp and any associated electronics next to the cavity?
Already been done.
http://emdrive.com/dynamictests.html
Watch the videos.

RF amp will be mounted next to the cavity but outside the Faraday Cage. If it leaks too much RF, will put it in it's own Faraday Cage.

The videos are nice, but until the mechanisms are replicated by an independent third party, it doesn't count for beans. Third party replication and verification is what makes the scientific method such a powerful tool. If the replication experiments are successful, then it'll be time to shout it from the rooftops.  ;D
Title: Re: EM Drive Developments - related to space flight applications - Thread 2
Post by: deltaMass on 05/07/2015 07:45 am
Already been done.
http://emdrive.com/dynamictests.html
Watch the videos.
Not really. Large air bearings will give you their own thrust with the slightest imperfection or obstruction. A wheel is much cleaner and more believable.
Title: Re: EM Drive Developments - related to space flight applications - Thread 2
Post by: TheTraveller on 05/07/2015 07:51 am
Already been done.
http://emdrive.com/dynamictests.html
Watch the videos.
Not really. Large air bearings will give you their own thrust with the slightest imperfection or obstruction. A wheel is much cleaner and believable.
Point is Shawyer did a rotary test, did measurements & they confirmed COE observed.

But YES it needs to be replicated and verified. Which is what I intend to do once the 1st static test phase is completed with enough thrust to overcome rotary test rig frictional losses.

All live over a YouTube stream.
Title: Re: EM Drive Developments - related to space flight applications - Thread 2
Post by: TheTraveller on 05/07/2015 07:57 am
The main attraction would be to get an EmDrive to accelerate a wheel from rest, period.
That would be a Red Letter Day indeed - irrespective of the CofE issue.
That would be headline news.

Because of the coax RF feed I assume you'll mount the RF amp and any associated electronics next to the cavity?
Already been done.
http://emdrive.com/dynamictests.html
Watch the videos.

RF amp will be mounted next to the cavity but outside the Faraday Cage. If it leaks too much RF, will put it in it's own Faraday Cage.

The videos are nice, but until the mechanisms are replicated by an independent third party, it doesn't count for beans. Third party replication and verification is what makes the scientific method such a powerful tool. If the replication experiments are successful, then it'll be time to shout it from the rooftops.  ;D
My tests will be streamed live via YouTube & recorded for all to watch over and over.

If it works, the Twitter verse will do the shouting.
Title: Re: EM Drive Developments - related to space flight applications - Thread 2
Post by: deltaMass on 05/07/2015 07:59 am
I disagree that they confirmed CofE observed. What was demonstrated was constant angular velocity at constant thrust. That represents zero acceleration. That does not correspond to either a constant acceleration model (non-CofE) nor to a decreasing acceleration model (CofE). What it shows is that an equilibrium was reached between the thrust and the friction at a particular angular velocity.

Friction is not our friend because it muddies the waters. It needs to be modelled and it dissipates power to boot. Basically, it complicates the analysis. Nevertheless, one is going to have to deal with it in the data analysis.
Title: Re: EM Drive Developments - related to space flight applications - Thread 2
Post by: TheTraveller on 05/07/2015 08:08 am
I disagree that they confirmed CofE observed. What was demonstrated was constant angular velocity at constant thrust. That represents zero acceleration. That does not correspond to either a constant acceleration model (non-CofE) nor to a decreasing acceleration model (CofE). What it shows is that an equilibrium was reached between the thrust and the friction at a particular angular velocity.

Friction is not our friend because it muddies the waters. It needs to be modelled and it dissipates power to boot. Basically, it complicates the analysis. Nevertheless, one is going to have to deal with it in the data analysis.
Which is why I designed in an ability to be able to vary the mechanical load on the rotary test rig and observe changes in the amount of energy needed (increased RF power) to maintain a constant rate of angular motion. I plan to use magnetic bearings.

From the statuc test data, should have good power to thrust curves, which will then be useful in the dynamic tests.
Title: Re: EM Drive Developments - related to space flight applications - Thread 2
Post by: WBY1984 on 05/07/2015 08:10 am
Quote
If it works, the Twitter verse will do the shouting.

Really? As far as I can see, all you'll have proven is that a hot metal cone mounted on a turntable can create enough convection currents to move it around a bit  :-\
Title: Re: EM Drive Developments - related to space flight applications - Thread 2
Post by: deltaMass on 05/07/2015 08:14 am
Heh. Even when it's inside a sealed box?
Title: Re: EM Drive Developments - related to space flight applications - Thread 2
Post by: TheTraveller on 05/07/2015 08:20 am
Quote
If it works, the Twitter verse will do the shouting.

Really? As far as I can see, all you'll have proven is that a hot metal cone mounted on a turntable can create enough convection currents to move it around a bit  :-\
Good comment.

Will add an IR camera video feed so we can see what the heat is doing. Could also put the EM Drive inside a 25mm thick foam box.

Are you OK with those changes eliminating any heat anomaly?

This dynamic test will only be done once the static tests show significant thrust and there is a input power to thrust curve available. This rotary test is not designed to measure thrust but to show how the power consumed by the EM Drive varies as rotary load varies, which to the EM Drive would be like a performance curve of power versus accelerative load mass.
Title: Re: EM Drive Developments - related to space flight applications - Thread 2
Post by: WBY1984 on 05/07/2015 08:29 am
Ensuring that the device is encased and no heat is getting out sounds very sensible, yes.  :)
Title: Re: EM Drive Developments - related to space flight applications - Thread 2
Post by: TheTraveller on 05/07/2015 08:32 am
Ensuring that the device is encased and no heat is getting out sounds very sensible, yes.  :)
Well heat always gets out. Insulation only slows down the rate of heat transfer.
Title: Re: EM Drive Developments - related to space flight applications - Thread 2
Post by: deltaMass on 05/07/2015 08:47 am
@TheTraveller:
What sort of static thrust are you shooting for? Corresponding to what RF power?
I ask because you do need enough to get over at least any stiction.
Title: Re: EM Drive Developments - related to space flight applications - Thread 2
Post by: PaulF on 05/07/2015 08:57 am
Quote from: WarpTech
I think I found the error in your analysis. You said,
v = a*t = (F/m)*t
This statement "assumes" acceleration is a constant. It is not a constant, so...
It is constant if and only if F is a constant of the motion.  Which, as I have argued with recourse to SR, it indeed is.

Did you find any other mistakes?

No, and I hear you. I believe energy is Force x Distance and Power in = Power out, and energy is conserved. I also believe SR is an "approximation" to a more accurate theory that includes the relative energy of the local quantum vacuum. That is how my model works, because that is how the Math in GR and QED tells us it should work.

Todd D.
My mention of SR is simply in order to highlight a core principle of Einstein's thinking about space and time; to whit, there is no preferred inertial frame, such that physics there is different to physics in another one.

Are you really saying that you reject this?

What about with respect to the "Cosmic Microwave Background Radiation" CMBR?  Couldn't we say the universe has this as an absolute frame? 

https://scholar.google.com/scholar?cluster=16121186223305818545&hl=en&as_sdt=0,48

Lets say there is a God view looking above far from all gravitational fields observing the universe and stationary with respect to its boundaries.  (It should have a finite size if it had a beginning and an expansion.)  Light falling into gravity fields I would think should slow down, (considering the limit when light reaches an event horizon).  From inside a gravity field I should think light should still appear to be going c due the shrinking of the ruller.  This could give the appearance of the index of refraction (gravitational lensing).  Two objects traveling towards each other @ .6c still appear from the God view to be approaching at 1.2c though I suppose the two observers both have their (space/time) warped so it appears to them they are not approaching each other at 1.2c but rather v<c.  I mean sure time/space screws our perceptions all up but why not have an absolute frame of the universe or CMB where either we are moving with respect to it or we arent?  Or am I missing something.

Congratulations! You have just "correctly" described the Polarizable Vacuum Model of General Relativity. However, we do not need the CMBR. We simply define an observer "at infinity" to have a refractive index, K=1. From that perspective, he can observe the relative value of K, for all gravitational fields, where K > 1. If he sees something moving FTL, he will assign that space with a relative value of, K < 1.

It is still not a preferred frame, because K can be "defined" as 1 anywhere, and all observations are relative to that definition. Mark Millis has said that the CMBR is a preferred frame. IMO, it doesn't make the Math any easier, it just confuses people. As an engineer, I see all things as relative and take everything with a grain of salt. :)

Todd
I am not a math magician either, but when I read this, I picture "God" viewing it all from a higher spatial dimension, because that is the only way to properly overview 3D spacetime and be completele free of relativistic effects (example: we can oversee everything drawn on a sheet of paper which is one dimension lower than us, while a "flatlander" who lives in the paper cannot). That way "God" could create preferred frames at will. Or am I now talking BS?

I am sorry if this does not contribute to the discussion. Trying to wrap my head around a lot of this.

By the way, I am agnostic.
Title: Re: EM Drive Developments - related to space flight applications - Thread 2
Post by: Star One on 05/07/2015 09:03 am

Quote
If it works, the Twitter verse will do the shouting.

Really? As far as I can see, all you'll have proven is that a hot metal cone mounted on a turntable can create enough convection currents to move it around a bit  :-\
Good comment.

Will add an IR camera video feed so we can see what the heat is doing. Could also put the EM Drive inside a 25mm thick foam box.

Are you OK with those changes eliminating any heat anomaly?

This dynamic test will only be done once the static tests show significant thrust and there is a input power to thrust curve available. This rotary test is not designed to measure thrust but to show how the power consumed by the EM Drive varies as rotary load varies, which to the EM Drive would be like a performance curve of power versus accelerative load mass.

Can I just add I applaud your proactivity in all of this.:)
Title: Re: EM Drive Developments - related to space flight applications - Thread 2
Post by: Flyby on 05/07/2015 09:37 am
Heh. Even when it's inside a sealed box?

I would not place it in a sealed box, because that will always bring up the question of what if it wasn't perfectly sealed and a air-jet leak helped it propel in the right direct....

Place the device in a box that has one open side and place it so that the open edge is on the same side of the small plate. If any serious thermal effect of comes of the emdrive the open_on_one_side_box will act as a "rocket engine" rotating the device in the opposite direction of what we think the EMdrive will do (movement towards the small endplate )

IF the EM drive still moves in the small endplate direction, then it simply CAN NOT  be any thermal effect, air jet or convection effect.

No need to have a high standard proof that your box is indeed completely sealed as the open box will redirect any forces generated by convection, hot air jets, etc into the opposite way.
Title: Re: EM Drive Developments - related to space flight applications - Thread 2
Post by: deltaMass on 05/07/2015 09:53 am
But it would suck if the thermal efflux completely masked the thrust
Title: Re: EM Drive Developments - related to space flight applications - Thread 2
Post by: Flyby on 05/07/2015 10:14 am
agreed..
but then again, I do not know what power TheTraveler plans on using, but if the thrust is so weak.. why bother talking about devices that lift 1 ton?

 What is needed is a clear , irrefutable force that can not be explained by any known side effect, performed by several other labs or home builders.

The whole issue with the EagleWorks last tests is just the difficulty the keep net force results separated from background noise. It leads to endless bickering about measurement methods, noise reduction systems, flawed setups, false readings, etc...

That's why they're building a high power test of ±1Kw or so, hopefully by July...

We really need a brute force approach to get scientific/engineering interest and above all... credibility...

If, in the end, it turns out to be nothing, then we've all wasted some time on this... no real harm done, and we had some fun with the theoretical speculations. (although I'm more a front row spectator , then a participant)
If however this would turn out to be something, then space exploration will never be the same...
Title: Re: EM Drive Developments - related to space flight applications - Thread 2
Post by: deltaMass on 05/07/2015 10:19 am
The Shawyer video shows constant input power producing constant thrust working against (presumably) friction, resulting in constant velocity.  Thus the output power is constant. But is this velocity limit really due to friction, or due to something more fundamental?
Title: Re: EM Drive Developments - related to space flight applications - Thread 2
Post by: deltaMass on 05/07/2015 10:21 am
That's why they're building a high power test of ±1Kw or so, hopefully by July...
I'm fairly sure it's +1 KW  :P
Title: Re: EM Drive Developments - related to space flight applications - Thread 2
Post by: Star One on 05/07/2015 10:34 am

That's why they're building a high power test of ±1Kw or so, hopefully by July...
I'm fairly sure it's +1 KW  :P

1.2 KW.
Title: Re: EM Drive Developments - related to space flight applications - Thread 2
Post by: deltaMass on 05/07/2015 10:38 am

That's why they're building a high power test of ±1Kw or so, hopefully by July...
I'm fairly sure it's +1 KW  :P

1.2 KW.
Yes, my infrequent attempts at humour generally end up that way
Title: Re: EM Drive Developments - related to space flight applications - Thread 2
Post by: Star One on 05/07/2015 10:41 am


That's why they're building a high power test of ±1Kw or so, hopefully by July...
I'm fairly sure it's +1 KW  :P

1.2 KW.
Yes, my infrequent attempts at humour generally end up that way

As long as we hear the results that's all that matters.
Title: Re: EM Drive Developments - related to space flight applications - Thread 2
Post by: deltaMass on 05/07/2015 10:49 am
It might occur to some to do a speculative build at a frequency other than that of a microwave oven magnetron (~125 mm wavelength) in order to get a smaller cavity which is easier to make and move around. What might that frequency range be? Well, they make magnetrons up to almost 100 GHz (3 mm wavelength) so there's a wide range to choose from. Cost is a factor also. The scopes, chiefly.
Title: Re: EM Drive Developments - related to space flight applications - Thread 2
Post by: CW on 05/07/2015 10:54 am
I think that the solution is really simple, if it's about demonstrating a thrust effect that is many times larger than measurement precision: Crank up the RF power. A lot. There is really no two ways about it. Please don't even try to play with a power level that even a 9V-battery can put out.. . Personally, I'd play in a power regime of about 1kW (actually I do, but that's another story..) . 1KW is a level that can easily be handled by readily available parts and off-the-shelf electronics, but is still not excessive.
Title: Re: EM Drive Developments - related to space flight applications - Thread 2
Post by: deltaMass on 05/07/2015 11:03 am
I think that the solution is really simple, if it's about demonstrating a thrust effect that is many times larger than measurement precision: Crank up the RF power. A lot. There is really no two ways about it. Please don't even try to play with a power level that even a 9V-battery can put out.. . Personally, I'd play in a power regime of about 1kW (actually I do, but that's another story..) . 1KW is a level that can easily be handled by readily available parts and off-the-shelf electronics, but is still not excessive.
Which battery types would you recommend for 1 KW operation?
Title: Re: EM Drive Developments - related to space flight applications - Thread 2
Post by: Flyby on 05/07/2015 11:06 am
Although i have a slight inclination towards believe there is indeed an effect, simply because of shawyer's video of his dynamic test, because of the test performed in china and because the Eagleworks results, I still feel very uncomfortable about the silence Shawyer has on his first generation super cooled device.

If I would have irrefutable and impressive results with a supercooled EMdrive, I would not hesitate to make it known to the world. I'm sure a lot of scientific and financial interest would come my way... but I'm not Shawyer... :)
still.. the question remains.. why the silence?
Title: Re: EM Drive Developments - related to space flight applications - Thread 2
Post by: TheTraveller on 05/07/2015 11:07 am
Ensuring that the device is encased and no heat is getting out sounds very sensible, yes.  :)
Plan is to replicate the Flight Thruster using 100Ws of RF during the static tests. At 150W, Shawyer got around 40mN or 4gf of thrust, which would give me around 25mN or 2.5gf. As my setup will be much lighter than Shawyers rotary system and will be using magnetic bearings, 2.5gf should be more than enough thrust to run load versus power consumed tests and generate a descent curve to show COE is obeyed or not.
Title: Re: EM Drive Developments - related to space flight applications - Thread 2
Post by: TheTraveller on 05/07/2015 11:14 am


That's why they're building a high power test of ±1Kw or so, hopefully by July...
I'm fairly sure it's +1 KW  :P

1.2 KW.
Yes, my infrequent attempts at humour generally end up that way

As long as we hear the results that's all that matters.
My tests will be streamed, in real time, via YouTube & recorded. Will setup a web site with the stream links, run by run measurement results, plans, BOM, sources etc.
Title: Re: EM Drive Developments - related to space flight applications - Thread 2
Post by: TheTraveller on 05/07/2015 11:18 am
Although i have a slight inclination towards believe there is indeed an effect, simply because of shawyer's video of his dynamic test, because of the test performed in china and because the Eagleworks results, I still feel very uncomfortable about the silence Shawyer has on his first generation super cooled device.

If I would have irrefutable and impressive results with a supercooled EMdrive, I would not hesitate to make it known to the world. I'm sure a lot of scientific and financial interest would come my way... but I'm not Shawyer... :)
still.. the question remains.. why the silence?
NDA, money, under contract, testing issues?

Shawyer did email Mullerton
Title: Re: EM Drive Developments - related to space flight applications - Thread 2
Post by: TheTraveller on 05/07/2015 11:27 am
I think that the solution is really simple, if it's about demonstrating a thrust effect that is many times larger than measurement precision: Crank up the RF power. A lot. There is really no two ways about it. Please don't even try to play with a power level that even a 9V-battery can put out.. . Personally, I'd play in a power regime of about 1kW (actually I do, but that's another story..) . 1KW is a level that can easily be handled by readily available parts and off-the-shelf electronics, but is still not excessive.
As a one time ham, I'm not keen to crank up 1kW or more of power as there will be other issues to deal with at those power levels. Even at my max target of 100W, I suspect there may be issues to deal with.
Title: Re: EM Drive Developments - related to space flight applications - Thread 2
Post by: deuteragenie on 05/07/2015 12:49 pm
Quote

For the other question, the thrust, as a physicist I keep on being rather sceptical because I tried to move my car by hitting the windscreen with punches and nothing happened.


As an electrician I point my flashlight at the windscreen and the car moves. 
My knuckles are in perfect condition.
Title: Re: EM Drive Developments - related to space flight applications - Thread 2
Post by: deltaMass on 05/07/2015 01:04 pm
It's good that at least one person understands what I'm on about. :-\

Consider an EmDrive in free space and accelerating. We switch it off temporarily and let it coast at speed v relative to the inertial frame in which it began its acceleration. When we switch it back on, are we going to assert that somehow the thrust F  knows what speed it's going and adjusts the thrust like F = P/v?

I assert again that this kind of thinking requires a preferred frame, and thus violates SR.

Or you could also give the example of EM Drive ship 1 (which started from a different place) being overtaken by EM Drive ship 2 ...

EDIT:

See White and Joosten, Appendix :

http://ntrs.nasa.gov/archive/nasa/casi.ntrs.nasa.gov/20140013174.pdf

I just read this... Oh my!

deltaMass, I owe you an apology. I thought this was your derivation. I did not realize they published it that way. It was an assumption right from the start that acceleration is constant for constant input power. A "what if?" scenario. Fine, that's how the game is played if you want to get funded.

Thank you.
Todd

It really is a terrible piece of work. There is in fact not one, but are two bloopers. The first one is what we've been discussing, and the way he solves the problem by imprimatur, and then shows a graph absent the knee he describes. The second one is his attempt to show a "paradox" for the conventional operation of observing kinetic energy changes in different inertial frames. He neglects the extra term (m V dv) (V = relative frame velocity, dv = delta-v of object) which must be added in.
Title: Re: EM Drive Developments - related to space flight applications - Thread 2
Post by: Rodal on 05/07/2015 01:13 pm
Given the explosive number of posts in this thread, since the NSF article, how about if we divide the thread as follows

1) One thread for people that believe "EM Drive, Follow the Data, Ignore the Theory" (some posters have adhered to this principle of investigation, as of late)

and

2) another thread for those that rather continue with "EM Drive, Analyze experiments and Analyze theory" which was the original focus of threads EMDrive 1 and 2.





It will result in easier searching and less clutter. 
The forum moderator may have to close this thread and start another one pretty soon, as this 2nd EM Drive thread is already approaching 140 pages, as threads cannot have an unlimited number of pages.

This is an excellent time to make this decision.


It will also be more in the spirit of this forum which is to keep threads focused on topic, as approach #1 above (which disregards any theory) is in conflict with approach #2  :)


Title: Re: EM Drive Developments - related to space flight applications - Thread 2
Post by: deltaMass on 05/07/2015 01:13 pm
Quote from: TheTraveller
Plan is to replicate the Flight Thruster using 100Ws of RF during the static tests. At 150W, Shawyer got around 40mN or 4gf of thrust, which would give me around 25mN or 2.5gf. As my setup will be much lighter than Shawyers rotary system and will be using magnetic bearings, 2.5gf should be more than enough thrust to run load versus power consumed tests and generate a descent curve to show COE is obeyed or not.
I imagine the chief source of friction will be the generator bearings. Or can these also be magnetic?
Title: Re: EM Drive Developments - related to space flight applications - Thread 2
Post by: StrongGR on 05/07/2015 01:20 pm
I will post here the solution for a very simple set-up of a cube cavity maintaining a single mode and show the way the laser beam propagates inside it. This resonant cavity seems to be very good for engineering of space-time rather than else.

May I respectfully ask that you also include a visualization of the solution if possible?  I'd very much like to see this.
StrongGR  may be talking about a closed-form solution for that case (without the dielectric insert), therefore a mathematical formula.  Not necessarily including plots, as they are necessary for numerical solutions. 

Given the closed-form solution (for which we would be most thankful  :) ) then all of us could make plots using our own software, for any numerical values we are interested in.

That's why closed-form solutions rock :)

Correct, I am providing some closed form equations for a simplified case that should describe correctly what observed recently at Eagleworks. As said before, I cannot find an explanation for thrust in the framework of general relativity. In the aforementioned Minotti's paper it is shown that one needs to modify the theory to account for it. Minotti's paper can be helpful to discuss the full problem and this will be work for the (very) near future. The point that I would like to understand is if the linearized Einstein theory could be enough. Probably so but my analysis for the simplified problem makes me think that a cavity can yield more for a laser propagating inside.

Eagleworks' results about the laser and the cavity are exciting because could pave the way both to table-top experiments in general relativity and space-time engineering as the technology to manage electromagnetic fields is well acquired.

Feel free to comment on this first draft.
Title: Re: EM Drive Developments - related to space flight applications - Thread 2
Post by: WarpTech on 05/07/2015 01:24 pm
Quote from: WarpTech
Hypothetically, if there were a black box with a gravitational field "inside", i.e., it has a NET acceleration vector along the X axis, pointing toward the blue side of the box..., (which is opposite the red side of the box)  but has no discernible gravitational field "outside" of the box other than what a normal box of that mass would have. Let's say that inside the box there is all the equipment and energy storage, necessary to generate this field. Nothing comes in or goes out, but the energy stored inside it (battery) is being dissipated without being expelled.

What sort of motion would YOU expect to see?

1. Will it move forward with the blue side leading?

2. Will it move forward with the red side leading?

3. Will it not move at all because nothing is coming out?

Keep in mind, by definition, it has a NET acceleration vector inside along the X axis.

Todd
I must confess that I don't understand this system. It seems to suggest that a gravitational field can be generated at will de novo and also shielded. I wouldn't know where to start with something like that.

Exactly my point! The black box test is inadequate for such a device. A more "open minded" test is required to get to the true nature of such a device. Otherwise, rampant discussions of violations of COM will pursue.

Todd
Title: Re: EM Drive Developments - related to space flight applications - Thread 2
Post by: JasonAW3 on 05/07/2015 01:32 pm
Given the explosive number of posts in this thread, since the NSF article, how about if we divide the thread as follows

1) One thread for people that believe "EM Drive, Follow the Data, Ignore the Theory" (some posters have adhered to this principle of investigation, as of late)

and

2) another thread for those that rather continue with "EM Drive, Analyze data and Analyze theory" which was the original focus of threads EMDrive 1 and 2.





It will result in easier searching and less clutter. 
The forum moderator may have to close this thread and start another one pretty soon, as this 2nd EM Drive thread is already approaching 140 pages, as threads cannot have an unlimited number of pages.

This is an excellent time to make this decision.


It will also be more in the spirit of this forum which is to keep threads focused on topic, as approach #1 above (which disregards any theory) is in conflict with approach #2  :)

Doctor,
     May be I am wrong in this approach, but, perhaps all the data SHOULD be examined first and the actual "How" this came about be theorized about later?  As I understand it, most science is done by observation and then theorizing on how something happened.

     At present, it appeares you have some sort of motion being created by the various units.  By varying the conditions of the experiments, be it power, frequency or even device configuration, the results appear to vary in a predictible fashion.  Perhaps examination of what variables seem to do what outputs you are getting, may provide insights into what is actually happening?  (Yeah, kind of self evident in retrospect)

       I figure that, once we see what all is actually happening with the device, a more coherent theory can be constructed to explain WHY these things are happening.
Title: Re: EM Drive Developments - related to space flight applications - Thread 2
Post by: JasonAW3 on 05/07/2015 01:35 pm

That's why they're building a high power test of ±1Kw or so, hopefully by July...
I'm fairly sure it's +1 KW  :P

1.2 KW.
Yes, my infrequent attempts at humour generally end up that way

As long as it's not "1.21 Jiggawatts!"
Title: Re: EM Drive Developments - related to space flight applications - Thread 2
Post by: deltaMass on 05/07/2015 01:51 pm

That's why they're building a high power test of ±1Kw or so, hopefully by July...
I'm fairly sure it's +1 KW  :P

1.2 KW.
Yes, my infrequent attempts at humour generally end up that way

As long as it's not "1.21 Jiggawatts!"
It's bad form to explain a joke, but I was implying that I hoped it wasn't -1 kW!
Title: Re: EM Drive Developments - related to space flight applications - Thread 2
Post by: Mulletron on 05/07/2015 01:53 pm
Given the explosive number of posts in this thread, since the NSF article, how about if we divide the thread as follows

1) One thread for people that believe "EM Drive, Follow the Data, Ignore the Theory" (some posters have adhered to this principle of investigation, as of late)

and

2) another thread for those that rather continue with "EM Drive, Analyze experiments and Analyze theory" which was the original focus of threads EMDrive 1 and 2.





It will result in easier searching and less clutter. 
The forum moderator may have to close this thread and start another one pretty soon, as this 2nd EM Drive thread is already approaching 140 pages, as threads cannot have an unlimited number of pages.

This is an excellent time to make this decision.


It will also be more in the spirit of this forum which is to keep threads focused on topic, as approach #1 above (which disregards any theory) is in conflict with approach #2  :)

That is no reason to split the thread. If we don't stick together, and share data and insight....this effort will be severely hampered. One team, one fight.
Title: Re: EM Drive Developments - related to space flight applications - Thread 2
Post by: Mulletron on 05/07/2015 01:59 pm
There is NO accepted theory. It is up to us (or someone else) to find the truth which will lead to falsifying EMdrive........or proving EMdrive and developing a working accepted theory.

We need to keep the best and brightest minds together on this problem if we're ever to succeed. The last thing we need, is a wedge being driven right through the effort.
Title: Re: EM Drive Developments - related to space flight applications - Thread 2
Post by: Cinder on 05/07/2015 02:10 pm
The point there being, AIUI, that you can't develop those compelling theories under an "ignore the theory" MO; and that a thread excluding that approach would necessarily cause friction: minor friction in individuals exercising that discipline and major friction for moderators exercising it for those who won't.
Title: Re: EM Drive Developments - related to space flight applications - Thread 2
Post by: Rodal on 05/07/2015 02:18 pm
 "There is NO accepted theory" doesn't justify "Follow the Data. Ignore the theory".  On the contrary.

Although I have been personally involved in experiments all my life,  I am not familiar with anybody following that approach (Ignore any theories) at Universities or private R&D companies. 

I think it may be an exaggeration to overemphasize a point, as we all tend to do.  Nobody conducts completely random experiments in the hopes of trying to find a new physical result by pure lack.  Unexpected results many times turn out, that later have to be explained, but that is different from conducting completely random experiments on purpose. 

There is nothing wrong with "Follow the data".   The wrong part is "Ignore the theory" if by ignoring the theory means abandoning all human knowledge.

I think that what was probably meant should have been "Ignore a particular theory that is contradicted by the data"

All fundamental R&D at leading institutions deals with problems in which there is "no accepted theory" of course. That is what fundamental research means.

If there would be a single, universally accepted theory, with experimental verification at multiple research locations, then it would not be fundamental research  :).

CERN does not use the approach "Follow the Data. Ignore the theory".  :)
Title: Re: EM Drive Developments - related to space flight applications - Thread 2
Post by: Mulletron on 05/07/2015 02:18 pm
The point I'm making is this. You see these men sitting at the table together in this video? They're all on the same team....trying to figure out how to pull off interstellar flight. For the good of all of us. Now they're duking it out in an interview in Wired. They (like us) should all be working together, pooling resources, combined knowledge and experience. Instead a rift has formed, which will likely kill progress.

I do not want that to happen to this thread, which mas made pretty darn good progress so far. Not bad for a once unknown internet forum. Now back to work.

https://www.youtube.com/watch?v=ucyBMB_PWr8
Title: Re: EM Drive Developments - related to space flight applications - Thread 2
Post by: Flyby on 05/07/2015 02:22 pm
Dr. Rodal,
with all respect , but I do tend to follow Mulletron in this.
Due to the absence of THE theory, there are several theories and solutions being discussed at once. Agreed , that causes some confusion, certainly for a layman like me, but at the same time, it makes it interesting and easier to compare the ideas.

Is your irritation not more due to the fact that these interesting theoretical discussions are crosscut by the pragmatic talk of those ppl who are build test setups?

If there would be any split up needed then, i would suggest to split the engineering part of the home builders apart from the pure theoretical/mathematical discussions.
But in all honesty, that would be regrettable, because it would make cross-fertilization harder (fe, like the radial wire mesh you've found in those theoretical papers)
Title: Re: EM Drive Developments - related to space flight applications - Thread 2
Post by: lasoi on 05/07/2015 02:26 pm
I don't think anyone's ignoring the theory (or should be).  It's just that data is a lot easier to produce than theory, and a couple of the builders have been particularly active lately.

I'd like to hear more about WarpTech's theory about momentum transfer once the resonant waves reach their break-off limit.  He compares it to an event horizon, though perhaps not in the physical sense.  I'm not clear on that or exactly how the transfer would occur.  Also, why, if there is an event due to the physical constraints of the waveguide, would it necessarily be a momentum transfer rather than a heat transfer?

Thank you.
Title: Re: EM Drive Developments - related to space flight applications - Thread 2
Post by: Iulian Berca on 05/07/2015 02:36 pm
Hello, I`m new in this forum.

I read almost all the papers that can be found on Emdrive.com website and a part of this thread, and think is not so difficult to replicate the EMdrive.

I have electrical engineering studies, i build some brushless motor for electric cars and scooters,  motor controllers (inverters), lifters in the past, and other thinks.

As i noticed, Juan Yang professor from Xian Northwestern University China, had the best trust results.

I have some questions about building the emdrive.

I can not find the exact dimensions for her drive. Maybe you can help me with them.

1. It is possible to use a microwave oven magnetron and  assume that i will still produce trust?
If yes :
2. Can i just make a hole to the cone and put the antenna of the magnetron inside like in the attachment.
The wave guide has other purpose than feed microwaves to the resonance chamber ?
I will make the cone length to be adjustable (+/- 2cm) to be able to tune the of the resonator cavity for around 2,45Ghz frequency.

The power will be adjustable from ~ 200W- 800W . I`m not sure if i also need to decrease the current to the filament.
I will build myself the voltage regulator or i will just use an AutoTransformer to adjust the input power.
The cone material will be cooper 0.6m thikness.

I will try with different materials on the both ends of the cone. ( bare Cooper, HDPP, PCB, metaglas, and any other materials you may suggest).
I will post all the construction progress and the results, after i receive all the materials, in a couple of days.
(http://)
Iulian


Title: Re: EM Drive Developments - related to space flight applications - Thread 2
Post by: TheTraveller on 05/07/2015 02:42 pm
"Follow the data, theory be dammed!"

As stated by Paul March:
http://forum.nasaspaceflight.com/index.php?topic=36313.msg1356062#msg1356062

Quote
Dr. Rodal:

As promised, find attached a few related papers from work.  As to the rest of your and Mulletron's concerns over the Eagleworks evolving theoretical musings on the EM-Drive propulsion topic, I leave you with Boyd Bushman's, (was senior scientist at LM/FW, now retired and passed-on), admonition to me when I first met him back in 2000 when discussing Jim Woodward's Mach-Effect work with Boyd's boss, "Follow the data, theory be dammed!"  We intend to do just that, no matter where it might take us.

Best,  Paul M.
Title: Re: EM Drive Developments - related to space flight applications - Thread 2
Post by: D_Dom on 05/07/2015 02:46 pm
Feeling incapable of contributing much to the theoretical analysis I feel competent in building a device and providing accurate data. I have learned to think carefully before experiment/measurement so that I understand what is being measured, what the data is expected to reveal and why it is relevant. My point is both disciplined theoretical analysis and experimental data are useful.
Title: Re: EM Drive Developments - related to space flight applications - Thread 2
Post by: Blaine on 05/07/2015 02:49 pm
I'm new here and I just thought I would post this video for you all.  Its a very VERY sloppy experimental setup of something like the what people on this forum are talking about.  The interesting thing here is the man in the video doesn't use end-plates and its quite a bit slimmer than the EM Drive.  Here is the video:

https://www.youtube.com/watch?v=vcaOKX7Ko7w

What are some thoughts about the video posted?
Title: Re: EM Drive Developments - related to space flight applications - Thread 2
Post by: Rodal on 05/07/2015 02:51 pm
I'm new here and I just thought I would post this video for you all.  Its a very VERY sloppy experimental setup of something like the what people on this forum are talking about.  The interesting thing here is the man in the video doesn't use end-plates and its quite a bit slimmer than the EM Drive.  Here is the video:

youtube.com/watch?v=vcaOKX7Ko7w

What are some thoughts about the video posted?

Rough translation (From original in Russian):

Quote
Created Shawyer  (EM Drive) engine is very easy and simple in its design . It provides the necessary thrust " by the oscillation of the microwaves inside the vacuum container ."
http: //hi-news.ru/technology/v-nasa-i ...
I decided that the system should not be closed



It is a waveguide with one end open.  The Russian author points out (later in his Russian text) that he thinks that Shawyer, and others are wrong in using a closed cavity.

The reference (Cullen) given by Shawyer to support his theory also used in his experimental measurements of pressure, a cavity with one end open (with a transparent glass)


It is known that a microwave waveguide having one end open will display directional thrust, as the microwave photons escape the waveguide.  The problem with the EM Drive is that it is a closed cavity, hence it cannot be explained solely based on Maxwell's equations. Something else is needed: General Relativity, QV,  something else.

A waveguide with one end open will behave as a very inefficient photon rocket: thousands of times less thrust per power input than what is claimed in the EM Drive experiments.
Title: Re: EM Drive Developments - related to space flight applications - Thread 2
Post by: TheTraveller on 05/07/2015 03:05 pm
Feeling incapable of contributing much to the theoretical analysis I feel competent in building a device and providing accurate data. I have learned to think carefully before experiment/measurement so that I understand what is being measured, what the data is expected to reveal and why it is relevant. My point is both disciplined theoretical analysis and experimental data are useful.
Glad to get another builder on board.

We all have different skill sets and look at the world through different eyes. Thus we ALL bring to the table, different skills, experience, risk tolerance, outlooks and desires, which when combined synergistically, will allow test and theory to go forward TOGETHER, based on respect for each others ability to contribute and think outside the square.

One thing I learned very early on in life:
"None of us is as smart as ALL of us".
Title: Re: EM Drive Developments - related to space flight applications - Thread 2
Post by: Rodal on 05/07/2015 03:08 pm
Now, that's something we can all agree with  :)
Title: Re: EM Drive Developments - related to space flight applications - Thread 2
Post by: jknuble on 05/07/2015 03:27 pm
Following on what jknuble said about the multipactor-like effect as a possible cause of thrust. http://en.wikipedia.org/wiki/Multipactor_effect  I can't help but wonder about what's going on with the copper surface of the frustum. A quick back of the envelope (well, python) calculation shows that there's certainly enough energy in these devices to somehow atomize a small amount of copper , and propel them with enough momentum to produce a small amount of thrust.

For example, a 30 watt emdrive where 0.001% of the energy went towards atomization and 1% went toward addtional momentum of the particles... You'd have a device with 91uN thrust, propelling 1.4ng of copper a second at 65500m/s.

I can think of 3 ways to debunk this. 1) perhaps that amount of particles going that fast would be noticeable with the naked eye, so this isn't really a valid explanation. 2) stick a detector behind the thruster (are they ionized?). 3) SEM of the surface compared to scraps from the same batch of copper not used in the thrustum.

Just how would we get a net-thrust from a closed cavity with atomization.  Even if atoms are being ioniozed inside the cavity I don't see how that could result in a net thrust.  Atomization results in immediate thrust but then that creates impact on the other side of the cavity canceling out the propulsion.
I'm talking about signs of atomization on the *outside* surface of the frustum. Although it would probably be happening in the inside as well. The copper atoms would be the propellent. I'm not sure what the mechanism would be, but it's obviously more than a thermal effect, and the whole reversal in phase/thrust would be difficult to explain. It seems more simple than QV or relativity models, but it's still probably interesting physics.

It might also explain the interferometer results.


Hi Everyone,

Just checking in and reading through some comments.  Thanks for taking a look at the potential forces involved, rgreen. I believe a force from atomized particles on the interior could have similar - or stronger - effect than being generated on the exterior even if your design is properly vented in vacuum. Especially if the cavity was not designed to be hermetic but is not intentionally vented.  Think of a balloon with a pin-hole in it.  I did some searching and couldn't find the word "hermetic" or "vented" in the paper or discussions but perhaps the design is vented.  If it is designed to be hermetic I would suggest publishing the results of a fine and gross leak test as a hermetic design of this size with braised glass seals for the launch is non-trivial.  With a small leak you could have built a simple thruster.  The sensitivity of that leak test would need to be appropriate to catch a particle stream that equates to millionths-of-a-pound (more tangible units than "micronewtons" to me) of thrust.  This effect could occur in our out of vacuum. This is all speculation and hand-waving though as I don't know all the details of your assembly.  While I can't say the exact mechanism that would cause a force to occur, I think it is possible this force is present (sound familiar? :) ).   

Regardless, putting on my "NASA Independent Reviewer" hat, I would say that if you are claiming to have developed a technology which can provide propulsion without a propellant, you have a burden of proof to show that you are not self-generating your propellant due to RF energy interacting with the materials in your setup.  These materials could include your metals, coatings, adhesives, dielectrics and / or contaminants. I don't believe there is sufficient evidence from the test points over a range of power levels and in and out of vacuum and across test teams (US, China, Britain) to confirm or refute this as I can imagine situations where particle generating effects would occur in any case due to out-gassing, breakdown, corona, multipaction etc.  I am not involved in your effort at all so please forgive my intrusion but from an RF engineering perspective it would be great to see a paragraph in a paper someday explaining how you tested for these effects and can now dismiss them and that you havn't inadvertently built a conventional thruster or ion drive.  I saw an earlier post suggesting a wiki be developed which would include potential sources of error and this topic could be filed there.

I believe a good test would involve at least following (again, my apologies if these were done in part or seem obvious.. I am also repeating myself a bit here from earlier posts):

1 - Add sufficient vent holes to the RF cavity.  Ensure the hole diameter is small enough such that their wave-guide effects do not effect the S11 of the system in the range you're operating in.  It seems you do have an ability to do 3D e-mag simulations but here is a tool you can use to verify the venting does not have a significant electrical impact: http://multipactor.esa.int/features.html. Unless you have specifically designed the system to be hermetic with glass seals etc, it will slowly leak and confuse the test and possibly be the source of thrust.  Hopefully you are not currently relying on your antenna launch or fasteners on the cavity caps for venting.  Testing under vacuum is actually the best way to resolve this issue and is the "relevant space environment" (i.e. high TRL!) so hermetic designs should be avoided as this helps your case anyways.  The proposed physics don't require a gas to be present, correct?  To be clear, whether or not you are currently vented does not confirm or refute anything as I can envision scenarios where a force would be generated even with a properly vented design in normal atmosphere from the momentum imparted by the particles at the focused atomization point (millionths-of-a-pound!)... so this is just a first step.

2 - Ensure you perform an un-powered elevated temperature bake-out of the DUT and support electronics at 50 to 60C for least 24 hours under vacuum.  This is generally what we do for RF space-flight hardware as that is what is required to get most of the typical volatiles out of a system.  It looks like this was done for the chamber but the DUT does not appear to have been heated.  This needs to include any support electronics - in fact, move all but the bare minimum of electronics outside the chamber.  Completion of most out-gassing should be observed not with your vacuum pump monitor but with a TQCM in real-time (http://en.wikipedia.org/wiki/Quartz_crystal_microbalance).  Note- this bake-out is a pretty crucial step to eliminate the possibility of simple out-gassing being the culprit for this phenomenon.  Out-gassing can be a non-linear behavior in some materials.  Some materials out-gas at a rate proportional to their temperature.  Others will not begin out-gassing until they reach a certain temperature.  So the experimenter should perform an un-powered bake-out over the range of temperatures the *powered* system should expect to see.  So if you are dissipating 2.6W in your cavity a thermal analysis should be done to see what temperature the system (especially the launch) would get to in vacuum.  Alternatively just instrument it properly and run a test first.  A bake-out will need to be properly done prior to every test as re-exposure to atmosphere will re-coat the DUT with contaminants and your dielectric will absorb moisture.  A mistake we made on the Aquarius mission (http://aquarius.nasa.gov/) was not performing a high temperature bake-out with our heaters once we reached orbit.  Consequently it took weeks for the system to stabilize as the dielectrics out-gassed moisture and we could observe the dielectric constant of RF boards drifting.  In short, ensure the un-powered temperatures reach and exceed the temperatures caused by RF heating when operational at the test temperature.

[General note on the 2.6W case as it is often used for an argument against the causes I have listed:  Some have noted the 2.6W test is "low" but that is a huge amount of power for parts to handle in my field (RF radiometry). For example, consider that few of the passive parts sold by these folks can handle more than a watt or 10s of watts unless they are specially engineered for high power applications: http://www.minicircuits.com/  Consider what is actually occuring in the part when you reach the maximum power level and could that effect generate enough particles to create millionths-of-a-pound of pressure.  Also, in many components the limiting factor is the launch design.  Returning to your test setup, your PTFE insert will certainly heat and out-gas due due to a portion of the 2.6W of RF power being dissipated within it.  I note the slow time-constant on the plot in Figure 22 looks an awful lot like what you would see from a thermal effect, particularly as thrust slowly decays after RF power is removed:  http://arc.aiaa.org/doi/pdf/10.2514/6.2014-4029.  Also I interpret the setup as 2.6W being dissipated in chamber but with 28W incident from the power amps with the related electric field levels of 28W in your system.  This is my understanding based on this statement in Part B of http://arc.aiaa.org/doi/pdf/10.2514/6.2014-4029: " In this test configuration, the VNA system indicated a quality factor of ~7320, and the difference of power forward and power reflected as reported by the power meters was indicated to be ~16.92 watts as a result of manual tuning to maximize the power difference. " So for purposes of thermal effects, 2.6W is correct.  But for breakdown effects, the electric field strength associated with 28W is what should be considered, simulated and evaluated against breakdown.]

3 - Once the system is stabilized, I would turn on the RF power transmitting into an RF coaxial short rather than a load, perhaps with an RF switch. A full reflection which is a worst-case in your system could cause issues on your source and induce some out-gassing or breakdown from that device.  Ensure the TQCM does not detect anything. This will let you know if your setup would cause a false-negative for the test I'm proposing. Generally, ensure you have brought on an experienced materials or contamination engineer to set this test up for you. Also this would serve as further proof there are not a conducted or radiated interference issue.  The paper notes the DC currents from the power amps do effect the measurement to a degree so this case is more similar to what the amps are actually driving just to be thorough. 

4 - Turn on your system in the normal test setup and observe the TQCM.  If particles are detected you may have found the culprit as these may be the result of out-gassing, breakdown, corona, multipaction, etc.  These effects all generate particles and correspond to different power levels and environments. This step can be tricky but I've seen it work well when experienced people are involved.  When you have pulled a vacuum, small particle generation detection is more reliant on Brownian motion etc. to cause particles to exit the vent holes and get to the detector so some care is required here.  As I mentioned in an earlier comment this was the method used to verify that multipaction was occuring within the SMAP diplexers and not a purely reactive effect.

5- Allow the system to run for at least a week to ensure the force does not dissipate or change due to a propellant being expended.  Also, if possible, increase the sampling rate of your force sensor to be faster than what one would see from thermal effects.  I'm not at all familiar with the proposed physics to explain the phenomenon but generally a purely RF cause should result in "instantaneous" force.  Faster than milliseconds should be enough to discount thermal effects although not all the sources of error I have mentioned are thermal.   

6 - (Mentioned this previously) - Open the cavity and have someone experienced inspect the materials for any evidence of breakdown.  A detailed inspection and cleaning should be done before the unit is sealed if that is possible.

So hopefully some of this is helpful.  Someone else may be able to devise a more compelling test for the hypothesis. In summary, there are a variety of particle generating effects of high RF power including out-gassing, corona breakdown, multipaction, plasmas, etc.  Each can occur at different power levels, some will occur in vacuum, some not.  It is possible each of the teams is seeing one or more of these effects depending on these variables during any given test.   The above statements are general and also apply to the tests done by the Chinese and British teams and any future test.  So while "multipaction" may not be the current culprit with the setup at 28W it could be as testing is done at higher power levels.  I have focused much of the above discussion on the small 28W case in the referenced paper but KW of RF energy are known to cause somewhat violent events including (but not always) magic smoke and burned components.  So generating a newton of force during these events seems conceivable.

I dug around online for resources related to high power breakdown at RF frequencies and couldn't find much.  Here are a few things but these may a bit too basic.

http://en.wikipedia.org/wiki/Electrical_breakdown

http://www.microwaves101.com/encyclopedias/power-handling

Helpful tool:

http://multipactor.esa.int/

I hope this post wasn't too rambling or too full of misconceptions regarding the tests you have performed.  I don't envy you folks trying to make progress on this complex issue via a public internet forum. Good luck, and again, I hope I'm wrong!

-Joseph Knuble,
NASA GSFC Code 555
Microwave Instrument Technology Branch

Final note for any students here who are curious about RF: This is far below the power levels I believe you have operated at but for future consideration at the 100W to KW level note the acoustic (i.e. pressure) effects of this demonstration which uses a 2.4GHz magnetron in a closed cavity with a contaminant: https://www.youtube.com/watch?v=RwTjsRt0Fzo  Depending on the dynamics involved, teams which have tested at high power could be seeing the effects of a similar unstable vibration (think of your vibrating cell-phone skittering across the table.)
Title: Re: EM Drive Developments - related to space flight applications - Thread 2
Post by: CW on 05/07/2015 03:34 pm
I think that the solution is really simple, if it's about demonstrating a thrust effect that is many times larger than measurement precision: Crank up the RF power. A lot. There is really no two ways about it. Please don't even try to play with a power level that even a 9V-battery can put out.. . Personally, I'd play in a power regime of about 1kW (actually I do, but that's another story..) . 1KW is a level that can easily be handled by readily available parts and off-the-shelf electronics, but is still not excessive.
Which battery types would you recommend for 1 KW operation?

I personally use LiFePO4 batteries, as they are inherently safer than alternatives. For instance, I have one (about 1kg) with 8400mAh capacity and 30C continuous discharge capability (252 amps). You can extract ~1kW for a couple minutes, which should be enough for measurements. Recharging is also fast.
Title: Re: EM Drive Developments - related to space flight applications - Thread 2
Post by: SeeShells on 05/07/2015 03:41 pm
Now, that's something we can all agree with  :)

I second that statement! You know I have some old wire mesh (old front door screen) to build a cone shaped can, connect some wires to an old and discarded microwave, power it and Bingo, Warp Drive! It's not that simple and what if I, by the shot in the dark make something that does work better than anything out there? I've accomplished very little as the other part of the equation of why, isn't there. Theory, Design, Engineering and Testing all go hand in hand. One aspect may leapfrog another during a process.
I have no solid clue why this EM drive works, sure there are some great theories (some of them have "warped" my little brain) out there. I have no doubt a answer will make itself known by Developing Theories, Designing, Building, Testing in synergy.
Title: Re: EM Drive Developments - related to space flight applications - Thread 2
Post by: Rodal on 05/07/2015 03:46 pm
I will post here the solution for a very simple set-up of a cube cavity maintaining a single mode and show the way the laser beam propagates inside it. This resonant cavity seems to be very good for engineering of space-time rather than else.

May I respectfully ask that you also include a visualization of the solution if possible?  I'd very much like to see this.
StrongGR  may be talking about a closed-form solution for that case (without the dielectric insert), therefore a mathematical formula.  Not necessarily including plots, as they are necessary for numerical solutions. 

Given the closed-form solution (for which we would be most thankful  :) ) then all of us could make plots using our own software, for any numerical values we are interested in.

That's why closed-form solutions rock :)

Correct, I am providing some closed form equations for a simplified case that should describe correctly what observed recently at Eagleworks. As said before, I cannot find an explanation for thrust in the framework of general relativity. In the aforementioned Minotti's paper it is shown that one needs to modify the theory to account for it. Minotti's paper can be helpful to discuss the full problem and this will be work for the (very) near future. The point that I would like to understand is if the linearized Einstein theory could be enough. Probably so but my analysis for the simplified problem makes me think that a cavity can yield more for a laser propagating inside.

Eagleworks' results about the laser and the cavity are exciting because could pave the way both to table-top experiments in general relativity and space-time engineering as the technology to manage electromagnetic fields is well acquired.

Feel free to comment on this first draft.

What an original contribution !

this is very interesting:

Quote from: Marco Frasca
One sees that there is an additional component to the laser field exiting the cavity that interacts with the mode inside. This can have terms with the frequency shifted and is a purely gravitational effect.

This result is extremely interesting. 

Quote from: Marco Frasca
I have shown how a plane wave could produce a gravitational effect inside a cavity that could be observed using a propagating laser beam inside it. The effect could be unveiled using an interferometer or observing the components of the laser field outside the cavity. Components with a shifted frequency, due to the modes inside the cavity, should be seen. This could explain recent results at Eagleworks with a resonator having the form of a truncated cone. A local warp of the geometry due to the electromagnetic field pumped inside the cavity could be a satisfactory explanation

This justifies Dr. White's attempts to measure these effects with an interferometer

(http://forum.nasaspaceflight.com/index.php?action=dlattach;topic=37438.0;attach=827718;image)

I'm looking forward to your attempt at dealing with a truncated cone geometry (difficult geometry to analyze)
Title: Re: EM Drive Developments - related to space flight applications - Thread 2
Post by: TheTraveller on 05/07/2015 03:51 pm
I hope this post wasn't too rambling or too full of misconceptions regarding the tests you have performed.  I don't envy you folks trying to make progress on this complex issue via a public internet forum. Good luck, and again, I hope I'm wrong!

-Joseph Knuble,
NASA GSFC Code 555
Microwave Instrument Technology Branch

Final note for any students here who are curious about RF: This is far below the power levels I believe you have operated at but for future consideration at the 100W to KW level note the acoustic (i.e. pressure) effects of this demonstration which uses a 2.4GHz magnetron in a closed cavity with a contaminant: https://www.youtube.com/watch?v=RwTjsRt0Fzo  Depending on the dynamics involved, teams which have tested at high power could be seeing the effects of a similar unstable vibration (think of your vibrating cell-phone skittering across the table.)
Appreciate your post. Food for thought.

Do have concerns about gaseous venting from inside my heated cavity, Plan to have a very fine hole in the centre of the small and large end plates, which can be block if so desired. As in both ends, any venting should counter balance. Being able to block or not either or both will generate additional experimental data sets.

Seeing that Shawyer used vertical cavity orientation and balanced Teeter Totter beams says to me he knew of the issues and took steps to avoid them. Which encourages me his thrust data is real and I can replicate it.

BTW I'm not using a magnetron as my RF source but instead using a variable frequency narrow band RF generator (1kHz steps) and a solid state RF amplifier, so no vibrational noise generation issue. Biggest noise generator would be the fan on my laptop.
Title: Re: EM Drive Developments - related to space flight applications - Thread 2
Post by: WarpTech on 05/07/2015 03:57 pm
The point I'm making is this. You see these men sitting at the table together in this video? They're all on the same team....trying to figure out how to pull off interstellar flight. For the good of all of us. Now they're duking it out in an interview in Wired. They (like us) should all be working together, pooling resources, combined knowledge and experience. Instead a rift has formed, which will likely kill progress.

I do not want that to happen to this thread, which mas made pretty darn good progress so far. Not bad for a once unknown internet forum. Now back to work.

https://www.youtube.com/watch?v=ucyBMB_PWr8


Starship Congress 2015 was just announced. I hope to be there this time!

https://www.eventbrite.com/e/starship-congress-2015-interstellar-hackathon-tickets-16813758404?aff=efblike&fb_ref=Default

Title: Re: EM Drive Developments - related to space flight applications - Thread 2
Post by: TheTraveller on 05/07/2015 04:25 pm
Quote from: TheTraveller
Plan is to replicate the Flight Thruster using 100Ws of RF during the static tests. At 150W, Shawyer got around 40mN or 4gf of thrust, which would give me around 25mN or 2.5gf. As my setup will be much lighter than Shawyers rotary system and will be using magnetic bearings, 2.5gf should be more than enough thrust to run load versus power consumed tests and generate a descent curve to show COE is obeyed or not.
I imagine the chief source of friction will be the generator bearings. Or can these also be magnetic?
Generator frictional losses will be significant. Scrap that idea.

Thought is now to add mass to both sides of the rotary test rig, so to incrementally increase the acceleratory mass load on the EM Drive, which should have the same effect as putting increased loads on the system.

I'm also thinking about a counter balanced weight pulley system which keeps the test cavity in the same orientation as the static test rig and allows the test article to move up or down from thrust generation. More KISS than a fully rotary test rig.

Pity I can't use the "knife edge" bearings of the Teeter Totter balance system.

This is interesting, yes I know it still has one point of frictional contact but I'm sure I could get it to work for the rotary test rig and for the pulley based test rig. Stiction would be VERY low:

https://www.youtube.com/watch?v=_vzzbxMIITE
Title: Re: EM Drive Developments - related to space flight applications - Thread 2
Post by: saucyjack on 05/07/2015 04:52 pm
All-
The EM Drive wiki project (http://emdrive.echothis.com/) is starting to take shape - thanks in particular to @MazonDel who has begun adding and organizing content, including links back to the relevant posts and attachments on this forum and elsewhere.  It's still very early, so any assistance is greatly appreciated!  The site is run on MediaWiki (the same software used by Wikipedia) so it's quite easy to jump in and start contributing.

For those building their own test articles, I invite you to update this page (http://emdrive.echothis.com/Building) with your plans and links to any relevant photos, diagrams, videos, etc.  We also have sections to list out the various theory proposals (http://emdrive.echothis.com/Theory), possible error sources (http://emdrive.echothis.com/Possible_Error_Sources) and a placeholder FAQ (http://emdrive.echothis.com/Frequently_Asked_Questions).

@Chris Bergin, feel free to link to this from the forums, when and if you think it would be of value to your visitors (particularly the newer ones).

-Rolf
Title: Re: EM Drive Developments - related to space flight applications - Thread 2
Post by: PaulF on 05/07/2015 04:56 pm
Although i have a slight inclination towards believe there is indeed an effect, simply because of shawyer's video of his dynamic test, because of the test performed in china and because the Eagleworks results, I still feel very uncomfortable about the silence Shawyer has on his first generation super cooled device.

If I would have irrefutable and impressive results with a supercooled EMdrive, I would not hesitate to make it known to the world. I'm sure a lot of scientific and financial interest would come my way... but I'm not Shawyer... :)
still.. the question remains.. why the silence?
NDA, money, under contract, testing issues?

Shawyer did email Mullerton
It is entirely possible, and plausible, that Shawyer had to sell some of his knowledge to keep going and that now he has to keep his mouth shut about any progress made under the new owner.

Of course, this is pure speculation at this point.
Title: Re: EM Drive Developments - related to space flight applications - Thread 2
Post by: Rodal on 05/07/2015 04:58 pm
...

Correct, I am providing some closed form equations for a simplified case that should describe correctly what observed recently at Eagleworks. As said before, I cannot find an explanation for thrust in the framework of general relativity. In the aforementioned Minotti's paper it is shown that one needs to modify the theory to account for it. Minotti's paper can be helpful to discuss the full problem and this will be work for the (very) near future. The point that I would like to understand is if the linearized Einstein theory could be enough. Probably so but my analysis for the simplified problem makes me think that a cavity can yield more for a laser propagating inside.

Eagleworks' results about the laser and the cavity are exciting because could pave the way both to table-top experiments in general relativity and space-time engineering as the technology to manage electromagnetic fields is well acquired.

Feel free to comment on this first draft.

I would like to clarify for the general audience that

Quote
Eagleworks' results about the laser and the cavity are exciting

 this is a reference to NASA's experiments with an interferometer, where the laser beam goes through small portholes on a pillbox shaped cylindrical  EM Drive cavity.   

(http://forum.nasaspaceflight.com/index.php?action=dlattach;topic=37438.0;attach=827718;image)

It is my understanding that NASA has not measured any thrust forces from this experiment involving a pillbox shaped EM Drive cavity.

NASA expected no significant thrust either, because the pillbox-shaped cavity is entirely symmetric in the longitudinal direction (parallel to the laser beam) and because the cavity did not contain any polymer dielectric insert in the experiments (as so far performed).
Title: Re: EM Drive Developments - related to space flight applications - Thread 2
Post by: aero on 05/07/2015 05:25 pm
Quote
... and because the cavity did not contain any polymer dielectric insert.

But if desired, couldn't a donut shaped dielectric be used? That is, just drill a small hole in the center of the dielectric for the laser beam to pass through?
Title: Re: EM Drive Developments - related to space flight applications - Thread 2
Post by: Rodal on 05/07/2015 05:37 pm
All-
The EM Drive wiki project (http://emdrive.echothis.com/) is starting to take shape - thanks in particular to @MazonDel who has begun adding and organizing content, including links back to the relevant posts and attachments on this forum and elsewhere.  It's still very early, so any assistance is greatly appreciated!  The site is run on MediaWiki (the same software used by Wikipedia) so it's quite easy to jump in and start contributing.

For those building their own test articles, I invite you to update this page (http://emdrive.echothis.com/Building) with your plans and links to any relevant photos, diagrams, videos, etc.  We also have sections to list out the various theory proposals (http://emdrive.echothis.com/Theory), possible error sources (http://emdrive.echothis.com/Possible_Error_Sources) and a placeholder FAQ (http://emdrive.echothis.com/Frequently_Asked_Questions).

@Chris Bergin, feel free to link to this from the forums, when and if you think it would be of value to your visitors (particularly the newer ones).

-Rolf

EDIT: The following has already been done !  Thanks so much to Rolf




Rolf,

Thank you for this very noble effort  :)

I have a suggestion, based on the experience of our NSF article and Star-Drive's posts in the forum being misquoted and misattributed in several publications in the media, and the negative effect this has had on the NASA Eagleworks team.

Could you please have an announcement up-front on the EM Drive wiki project for this (*) un-moderated effort, that any poster is free to write whatever she/he wants regarding her/his own work or posts regarding the EM Drive.  But that other people's work (posts in this forum and attachments) should only be linked or posted verbatim (but never paraphrased), in order to prevent the problems we have witnessed due to the false, misleading reports in the media about NASA Eagleworks.

This is particularly important regarding the work of posters that have either used their own names (rather than monickers) or whose real name has become generally known otherwise.

Thank you :)

_________
(*) my understanding
Title: Re: EM Drive Developments - related to space flight applications - Thread 2
Post by: StrongGR on 05/07/2015 05:44 pm
I will post here the solution for a very simple set-up of a cube cavity maintaining a single mode and show the way the laser beam propagates inside it. This resonant cavity seems to be very good for engineering of space-time rather than else.

May I respectfully ask that you also include a visualization of the solution if possible?  I'd very much like to see this.
StrongGR  may be talking about a closed-form solution for that case (without the dielectric insert), therefore a mathematical formula.  Not necessarily including plots, as they are necessary for numerical solutions. 

Given the closed-form solution (for which we would be most thankful  :) ) then all of us could make plots using our own software, for any numerical values we are interested in.

That's why closed-form solutions rock :)

Correct, I am providing some closed form equations for a simplified case that should describe correctly what observed recently at Eagleworks. As said before, I cannot find an explanation for thrust in the framework of general relativity. In the aforementioned Minotti's paper it is shown that one needs to modify the theory to account for it. Minotti's paper can be helpful to discuss the full problem and this will be work for the (very) near future. The point that I would like to understand is if the linearized Einstein theory could be enough. Probably so but my analysis for the simplified problem makes me think that a cavity can yield more for a laser propagating inside.

Eagleworks' results about the laser and the cavity are exciting because could pave the way both to table-top experiments in general relativity and space-time engineering as the technology to manage electromagnetic fields is well acquired.

Feel free to comment on this first draft.

What an original contribution !

this is very interesting:

Quote from: Marco Frasca
One sees that there is an additional component to the laser field exiting the cavity that interacts with the mode inside. This can have terms with the frequency shifted and is a purely gravitational effect.

This result is extremely interesting. 

Quote from: Marco Frasca
I have shown how a plane wave could produce a gravitational effect inside a cavity that could be observed using a propagating laser beam inside it. The effect could be unveiled using an interferometer or observing the components of the laser field outside the cavity. Components with a shifted frequency, due to the modes inside the cavity, should be seen. This could explain recent results at Eagleworks with a resonator having the form of a truncated cone. A local warp of the geometry due to the electromagnetic field pumped inside the cavity could be a satisfactory explanation

This justifies Dr. White's attempts to measure these effects with an interferometer

(http://forum.nasaspaceflight.com/index.php?action=dlattach;topic=37438.0;attach=827718;image)

I'm looking forward to your attempt at dealing with a truncated cone geometry (difficult geometry to analyze)

Thanks a lot! Indeed my idea was to justify White's results that appear already striking from a physicist's standpoint. I am working on the frustum case and I hope to update the paper soon.
Title: Re: EM Drive Developments - related to space flight applications - Thread 2
Post by: R.W. Keyes on 05/07/2015 06:17 pm
I think that the solution is really simple, if it's about demonstrating a thrust effect that is many times larger than measurement precision: Crank up the RF power. A lot. There is really no two ways about it. Please don't even try to play with a power level that even a 9V-battery can put out.. . Personally, I'd play in a power regime of about 1kW (actually I do, but that's another story..) . 1KW is a level that can easily be handled by readily available parts and off-the-shelf electronics, but is still not excessive.
Which battery types would you recommend for 1 KW operation?

I personally use LiFePO4 batteries, as they are inherently safer than alternatives. For instance, I have one (about 1kg) with 8400mAh capacity and 30C continuous discharge capability (252 amps). You can extract ~1kW for a couple minutes, which should be enough for measurements. Recharging is also fast.

LiFePO4 is a good battery tech..better, safer than others out there though not quite as much power/mass. For short tests, I think that whatever can be effectively used is good. For actual vehicles, where safety and reliability is required, I'd want LiFePO4.

Regarding your earlier comments, about just using more power to get results: I think we're going to see two different methodologies here,  the first being what Eagleworks is doing, which is carefully thought out, precision science, with work on theory to explain and therefore maximize efficiency. The other approach is the hacker approach, which is more of a mechanical and pragmatic approach, of doing things like applying more power or using different materials based upon hunches...that the proof will be in the pudding, so to speak. The problems arise when trying to unify the approaches - without some really fantastic results (such as a flying car), no one is going to let a hacker near the precision and certified labs to prove their claims, and the naysayers will have a field-day with "outlandish claims". Much of the guff EMdrive etc have received so far is because of insufficient theory and tests. I don't have a PhD in physics, and even if I did, if I wasn't prominent in the field I'd see no reason why Eagleworks or other reputable places would waste their time with me or the hundred other 'crackpots' in serious tests.

The exception to this may be if a program were implemented for inexpensive, quick, formal tests. An example which passed these tests would then be eligible for further testing and scrutiny. I think that such a program would be the best interface between the 'hackers' and the 'scientists'.

I've been looking at surplus 20kw radar magnetrons on ebay, and the availability of 3D printed silicons carbide base on which to plate my Magnesium diboride superconductor, and also at cryocoolers. It's all very complex and expensive, and with the cryo, somewhat dangerous. I am most definitely in the realm of a 'hacker' in this endeavor (and have been so in other fields for a number of years), so I know what to expect (or at least I think I do).

Title: Re: EM Drive Developments - related to space flight applications - Thread 2
Post by: R.W. Keyes on 05/07/2015 06:46 pm
Now, that's something we can all agree with  :)

I second that statement! You know I have some old wire mesh (old front door screen) to build a cone shaped can, connect some wires to an old and discarded microwave, power it and Bingo, Warp Drive! It's not that simple and what if I, by the shot in the dark make something that does work better than anything out there? I've accomplished very little as the other part of the equation of why, isn't there. Theory, Design, Engineering and Testing all go hand in hand. One aspect may leapfrog another during a process.
I have no solid clue why this EM drive works, sure there are some great theories (some of them have "warped" my little brain) out there. I have no doubt a answer will make itself known by Developing Theories, Designing, Building, Testing in synergy.

We'd be like soldiers in the army of infinite monkeys.

Except we're not infinite, not soldiers, and not monkeys. Let's see how good we can be. I have this idea of a community of enthusiast experimenters, sharing and sometimes arguing about what works and how we've measured it, with meetups and tests, like radio amateurs of old. We would devour the papers of Shayer, Eagleworks, and other professional scientists....but not present ourselves as such. There may be some people who can be good conduits between the two communities.

I don't know if others on this thread are familiar with the software development essay "The Cathedral and the Bazaar". It might be an interesting read to find similarities. I feel that software, and Em Drive, need both the Cathedral and the Bazaar.
Title: Re: EM Drive Developments - related to space flight applications - Thread 2
Post by: TheTraveller on 05/07/2015 06:57 pm
I think that the solution is really simple, if it's about demonstrating a thrust effect that is many times larger than measurement precision: Crank up the RF power. A lot. There is really no two ways about it. Please don't even try to play with a power level that even a 9V-battery can put out.. . Personally, I'd play in a power regime of about 1kW (actually I do, but that's another story..) . 1KW is a level that can easily be handled by readily available parts and off-the-shelf electronics, but is still not excessive.
Which battery types would you recommend for 1 KW operation?

I personally use LiFePO4 batteries, as they are inherently safer than alternatives. For instance, I have one (about 1kg) with 8400mAh capacity and 30C continuous discharge capability (252 amps). You can extract ~1kW for a couple minutes, which should be enough for measurements. Recharging is also fast.

LiFePO4 is a good battery tech..better, safer than others out there though not quite as much power/mass. For short tests, I think that whatever can be effectively used is good. For actual vehicles, where safety and reliability is required, I'd want LiFePO4.

Regarding your earlier comments, about just using more power to get results: I think we're going to see two different methodologies here,  the first being what Eagleworks is doing, which is carefully thought out, precision science, with work on theory to explain and therefore maximize efficiency. The other approach is the hacker approach, which is more of a mechanical and pragmatic approach, of doing things like applying more power or using different materials based upon hunches...that the proof will be in the pudding, so to speak. The problems arise when trying to unify the approaches - without some really fantastic results (such as a flying car), no one is going to let a hacker near the precision and certified labs to prove their claims, and the naysayers will have a field-day with "outlandish claims". Much of the guff EMdrive etc have received so far is because of insufficient theory and tests. I don't have a PhD in physics, and even if I did, if I wasn't prominent in the field I'd see no reason why Eagleworks or other reputable places would waste their time with me or the hundred other 'crackpots' in serious tests.

The exception to this may be if a program were implemented for inexpensive, quick, formal tests. An example which passed these tests would then be eligible for further testing and scrutiny. I think that such a program would be the best interface between the 'hackers' and the 'scientists'.

I've been looking at surplus 20kw radar magnetrons on ebay, and the availability of 3D printed silicons carbide base on which to plate my Magnesium diboride superconductor, and also at cryocoolers. It's all very complex and expensive, and with the cryo, somewhat dangerous. I am most definitely in the realm of a 'hacker' in this endeavor (and have been so in other fields for a number of years), so I know what to expect (or at least I think I do).
My engineer's gut tells me the best device for "Replicators" would be the Flight Thruster as it uses narrow band RF, which can be generated by a programmable RF generator, with an auxiliary RF amplifier and the RF energy feed into the cavity by standard RF connectors and coax. This gives good control over both frequency and power, which is not really available with a magnetron based RF generator, plus stops the need to add waveguides into the build.

I'm doing as close as possible to Shawyer replication as I can as I feel the closer I stay to Shawyer, the higher the chance of success. WHEN that works I may start trying stuff outside what Shawyer has shared.
Title: Re: EM Drive Developments - related to space flight applications - Thread 2
Post by: PaulF on 05/07/2015 07:03 pm
I think that the solution is really simple, if it's about demonstrating a thrust effect that is many times larger than measurement precision: Crank up the RF power. A lot. There is really no two ways about it. Please don't even try to play with a power level that even a 9V-battery can put out.. . Personally, I'd play in a power regime of about 1kW (actually I do, but that's another story..) . 1KW is a level that can easily be handled by readily available parts and off-the-shelf electronics, but is still not excessive.
Which battery types would you recommend for 1 KW operation?

I personally use LiFePO4 batteries, as they are inherently safer than alternatives. For instance, I have one (about 1kg) with 8400mAh capacity and 30C continuous discharge capability (252 amps). You can extract ~1kW for a couple minutes, which should be enough for measurements. Recharging is also fast.

LiFePO4 is a good battery tech..better, safer than others out there though not quite as much power/mass. For short tests, I think that whatever can be effectively used is good. For actual vehicles, where safety and reliability is required, I'd want LiFePO4.

Regarding your earlier comments, about just using more power to get results: I think we're going to see two different methodologies here,  the first being what Eagleworks is doing, which is carefully thought out, precision science, with work on theory to explain and therefore maximize efficiency. The other approach is the hacker approach, which is more of a mechanical and pragmatic approach, of doing things like applying more power or using different materials based upon hunches...that the proof will be in the pudding, so to speak. The problems arise when trying to unify the approaches - without some really fantastic results (such as a flying car), no one is going to let a hacker near the precision and certified labs to prove their claims, and the naysayers will have a field-day with "outlandish claims". Much of the guff EMdrive etc have received so far is because of insufficient theory and tests. I don't have a PhD in physics, and even if I did, if I wasn't prominent in the field I'd see no reason why Eagleworks or other reputable places would waste their time with me or the hundred other 'crackpots' in serious tests.

The exception to this may be if a program were implemented for inexpensive, quick, formal tests. An example which passed these tests would then be eligible for further testing and scrutiny. I think that such a program would be the best interface between the 'hackers' and the 'scientists'.

I've been looking at surplus 20kw radar magnetrons on ebay, and the availability of 3D printed silicons carbide base on which to plate my Magnesium diboride superconductor, and also at cryocoolers. It's all very complex and expensive, and with the cryo, somewhat dangerous. I am most definitely in the realm of a 'hacker' in this endeavor (and have been so in other fields for a number of years), so I know what to expect (or at least I think I do).
That just jogged my memory. Check out www.zooniverse.org

This project has been going for years and involves participants yound and old from all over the world classifying galaxies from the Sloan Digital Sky Survey. Very effective. I think their method could be adapted to suit EM drive theoretical and possible practical development and have the masses participate. BOINC could be another way, getting CPU time donated for simulations if anyone has the need for them within the project.
Title: Re: EM Drive Developments - related to space flight applications - Thread 2
Post by: deuteragenie on 05/07/2015 07:07 pm
I think that the solution is really simple, if it's about demonstrating a thrust effect that is many times larger than measurement precision: Crank up the RF power. A lot. There is really no two ways about it. Please don't even try to play with a power level that even a 9V-battery can put out.. . Personally, I'd play in a power regime of about 1kW (actually I do, but that's another story..) . 1KW is a level that can easily be handled by readily available parts and off-the-shelf electronics, but is still not excessive.

How about using Tesla batteries for a few seconds (>300 KW?) and send the damn thing at the other side of the galaxy? Would that form a convincing test result?
Title: Re: EM Drive Developments - related to space flight applications - Thread 2
Post by: PaulF on 05/07/2015 07:09 pm
I think that the solution is really simple, if it's about demonstrating a thrust effect that is many times larger than measurement precision: Crank up the RF power. A lot. There is really no two ways about it. Please don't even try to play with a power level that even a 9V-battery can put out.. . Personally, I'd play in a power regime of about 1kW (actually I do, but that's another story..) . 1KW is a level that can easily be handled by readily available parts and off-the-shelf electronics, but is still not excessive.

How about using Tesla batteries for a few seconds (>300 KW?) and send the damn thing at the other side of the galaxy? Would that form a convincing test result?
From what I distill from some of the information I have read, coronal discharges were already plaguing some of the teams at power between 100 and 1000 Watts. a 300KW burst would probably fry everything we can throw at it and would require serious re-design of the components and use of materials, if at all possible.
Title: Re: EM Drive Developments - related to space flight applications - Thread 2
Post by: frobnicat on 05/07/2015 07:09 pm
I must reject the analysis by J & W in Appendix A, but thanks for the link.

I have already mentioned in my preamble that not only is any propellantless propulsion craft capable of perpetuum mobile operation, but that free energy is available on top of that to boot.

This causes many people to break out in hives, or to resort to chewing their towels.  ::)

We can of course test to see this at work. We use a rotary configuration and when the tangential velocity exceeds 2/k, we switch from external power to a coaxially mounted generator. And then begin charging people for the juice. Outrageous!  8)

Added emphasis. If it can be of any comfort to you, I understand both your arguments and humour (so far). I only disagree on a minor point : while as stated in later posts (http://forum.nasaspaceflight.com/index.php?topic=36313.msg1370960#msg1370960) it could be shown experimentally much below 2/k (say, at 1% of that velocity) that the system phenomenology is on the way to reach break-even (assuming no unphysical knee in the thrust/power=function_of(velocity) equation), the system would potentially exhibit breakeven not at 2/k but starting at 1/k velocity tangential velocity.

Hope this is not perceived as self-promotional cross-posting but since you appear not to contribute in the "feature article thread" maybe you haven't seen my little drawing I posted there. That might be helpful :
(http://forum.nasaspaceflight.com/index.php?action=dlattach;topic=29276.0;attach=619550;image)
Em drives mounted on a rotor turning at 2000m/s tangential velocity (not easy but this is the kind of tangential velocity attained in some energy storage flywheels...). Em drives consuming 1kW microwave, radiating some (all ?) of this power as heat and thrusting at 1N : 1N/kW is in the ballpark of what has been experimentally claimed already. This gives 2000m/s * 1N = 2kW mechanical power to the shaft of the rotor. 2kW mechanical power at the shaft are converted to 1800W DC current by a generator (and 200W radiated as heat). Of this 1800W DC electrical power, 250W are diverted for any use we like. To keep it in line with the topic I put it to good use to power another Em drive but really we are free to use those 250W for whatever (creating mass for instance). The power splitter is not 100% efficient, it radiates 50W of power. 1800-250-50 = 1500W to feed the RF amplifier. The RF amplifier wastes (radiates) 500W as heat and pumps 1000W of clean microwave back into the Em drives on the rotor.

The process needs an initial investment in energy (to make rotor move at 2000m/s tangential velocity) but then this is a free energy generator for all practical purpose. If small variations in efficiency make the rotor lose a bit of velocity, just divert a little more power to the RF amplifier : this is just a regulation problem, there is ample margins to adjust and stabilize around the optimal operating point.
...


In the following post, Side note 1 (http://forum.nasaspaceflight.com/index.php?topic=37438.msg1370818#msg1370818), I explain why I think the right metric for the velocity above which there is energy issue (ie apparent COE breaking, or more factually, possible breakeven) is not 2/k but a more stringent (more easily accessible) 1/k limit. I do use twice 1/k as the tangential velocity for my quantitative argument above, but only to accommodate for both inefficiencies in the feedback cycle and leave a significant amount of "free juice". Both inefficiencies and diverted power could in principle be reduced to an arbitrary epsilon, making the breakeven reachable in principle (if not in practice) starting at any epsilon above 1/k tangential speed.

Not that it makes much difference for the argument, since tangential velocity can be arbitrarily raised in principle (if not in practice) unless the required breakeven velocity is c (case for the photon rocket, but not the proposed em drives that have a higher k hence breakeven velocity < c).

I suggest we call 1/k the breakeven velocity (Vbe ?) and discuss the possibility of such breakeven rather than speaking of "free energy" (which I try to avoid) or "apparent free energy" (which I used to leave open the fact that it may be possible and still respect COE, provided a hidden energy source is involved). But "free energy" is so much connoted that no proponent (of short trips to Saturn by using constant thrust/power) want to admit that their claimed phenomenology (constant thrust/power) requires "apparent free energy".

This conducts proponents (of short trips to Saturn with Em drives of constant thrust/power well above .1N/kW) to incoherent statements and inconsistent positions where we can see "thrust (at constant power) will therefore decrease as velocity increase" in one paragraph about how well the Em drive respects COE, and "at 10N/kW Saturn could be reached in a few weeks" in the next paragraph about how good Em drive would be at opening fast deep space transportation.

So, proponents of Em drive, will the theoretical breakeven above 1/k velocity (or practical breakeven above 2/k or 3/k to leave margin for innefficiencies in the feedback cycle, and k better than 1N/kW for a velocity compatible with known materials), will this breakeven be
1/ Impossible ?  => Where and how exactly does it fail ?
2/ Possible ? => What and where is the hidden field/flow that is "harvested" for apparent surplus energy ?


Side note : proponents of "this is all different of what we know, this must take GR into account...", please give quantitative arguments about the level of space-time curvature needed to yield micronewtons or otherwise be significant relative to values obtained with Newtonian mechanics in the low velocity and weak curvature regime. And the corresponding amount of local mass-energy to reach such curvature in usual GR. The total energy content of EM radiation bouncing around in a 30cm Q=10000 cavity fed at 100W is on the order of .001 J, this will bend space-time, but a lot less than the mass of a microbe...

Title: Re: EM Drive Developments - related to space flight applications - Thread 2
Post by: PaulF on 05/07/2015 07:11 pm
I think that the solution is really simple, if it's about demonstrating a thrust effect that is many times larger than measurement precision: Crank up the RF power. A lot. There is really no two ways about it. Please don't even try to play with a power level that even a 9V-battery can put out.. . Personally, I'd play in a power regime of about 1kW (actually I do, but that's another story..) . 1KW is a level that can easily be handled by readily available parts and off-the-shelf electronics, but is still not excessive.
Which battery types would you recommend for 1 KW operation?

I personally use LiFePO4 batteries, as they are inherently safer than alternatives. For instance, I have one (about 1kg) with 8400mAh capacity and 30C continuous discharge capability (252 amps). You can extract ~1kW for a couple minutes, which should be enough for measurements. Recharging is also fast.

LiFePO4 is a good battery tech..better, safer than others out there though not quite as much power/mass. For short tests, I think that whatever can be effectively used is good. For actual vehicles, where safety and reliability is required, I'd want LiFePO4.

Regarding your earlier comments, about just using more power to get results: I think we're going to see two different methodologies here,  the first being what Eagleworks is doing, which is carefully thought out, precision science, with work on theory to explain and therefore maximize efficiency. The other approach is the hacker approach, which is more of a mechanical and pragmatic approach, of doing things like applying more power or using different materials based upon hunches...that the proof will be in the pudding, so to speak. The problems arise when trying to unify the approaches - without some really fantastic results (such as a flying car), no one is going to let a hacker near the precision and certified labs to prove their claims, and the naysayers will have a field-day with "outlandish claims". Much of the guff EMdrive etc have received so far is because of insufficient theory and tests. I don't have a PhD in physics, and even if I did, if I wasn't prominent in the field I'd see no reason why Eagleworks or other reputable places would waste their time with me or the hundred other 'crackpots' in serious tests.

The exception to this may be if a program were implemented for inexpensive, quick, formal tests. An example which passed these tests would then be eligible for further testing and scrutiny. I think that such a program would be the best interface between the 'hackers' and the 'scientists'.

I've been looking at surplus 20kw radar magnetrons on ebay, and the availability of 3D printed silicons carbide base on which to plate my Magnesium diboride superconductor, and also at cryocoolers. It's all very complex and expensive, and with the cryo, somewhat dangerous. I am most definitely in the realm of a 'hacker' in this endeavor (and have been so in other fields for a number of years), so I know what to expect (or at least I think I do).
My engineer's gut tells me the best device for "Replicators" would be the Flight Thruster as it uses narrow band RF, which can be generated by a programmable RF generator, with an auxiliary RF amplifier and the RF energy feed into the cavity by standard RF connectors and coax. This gives good control over both frequency and power, which is not really available with a magnetron based RF generator, plus stops the need to add waveguides into the build.

I'm doing as close as possible to Shawyer replication as I can as I feel the closer I stay to Shawyer, the higher the chance of success. WHEN that works I may start trying stuff outside what Shawyer has shared.
I may be asking a stupid question, but has anyone ever tried Pulse Width Modulation with the micro waves? To see how that effects forces within the cavity? Or has that mathematically been ruled out? Or?
Title: Re: EM Drive Developments - related to space flight applications - Thread 2
Post by: deuteragenie on 05/07/2015 07:19 pm
I think that the solution is really simple, if it's about demonstrating a thrust effect that is many times larger than measurement precision: Crank up the RF power. A lot. There is really no two ways about it. Please don't even try to play with a power level that even a 9V-battery can put out.. . Personally, I'd play in a power regime of about 1kW (actually I do, but that's another story..) . 1KW is a level that can easily be handled by readily available parts and off-the-shelf electronics, but is still not excessive.

How about using Tesla batteries for a few seconds (>300 KW?) and send the damn thing at the other side of the galaxy? Would that form a convincing test result?
From what I distill from some of the information I have read, coronal discharges were already plaguing some of the teams at power between 100 and 1000 Watts. a 300KW burst would probably fry everything we can throw at it and would require serious re-design of the components and use of materials, if at all possible.

Interesting.  Corona discharges could prevent the device to scale then (assuming it works).
Title: Re: EM Drive Developments - related to space flight applications - Thread 2
Post by: Rodal on 05/07/2015 07:46 pm
...
My engineer's gut tells me the best device for "Replicators" would be the Flight Thruster as it uses narrow band RF, which can be generated by a programmable RF generator, with an auxiliary RF amplifier and the RF energy feed into the cavity by standard RF connectors and coax. This gives good control over both frequency and power, which is not really available with a magnetron based RF generator, plus stops the need to add waveguides into the build.
...

Another advantage to replicate the Flight Thruster, is that the Flight Thruster is one of only two designs that are close to what appears to be the optimum cone angle (7.5 degrees).  Only the Shawyer Experimental and Shawyer's Flight Thruster are close to this optimum geometry:

http://forum.nasaspaceflight.com/index.php?topic=36313.msg1370559#msg1370559
Title: Re: EM Drive Developments - related to space flight applications - Thread 2
Post by: CharCamOlson on 05/07/2015 07:46 pm
I am not an engineer, a scientist, or an inventor. I am just an english major who writes science fiction and thinks this is really cool. So, that said, is there any chance that the guys at Eagleworks are ever going to have a Gofundme? Is there any way for all the people watching this project who don't have technical skills to kick in a little money to fund those who DO?

Just asking. Seems crowdfunding might help get around some of the "big organizations won't fund our research" problem.
Title: Re: EM Drive Developments - related to space flight applications - Thread 2
Post by: zen-in on 05/07/2015 07:51 pm

I've been looking at surplus 20kw radar magnetrons on ebay, and the availability of 3D printed silicons carbide base on which to plate my Magnesium diboride superconductor, and also at cryocoolers. It's all very complex and expensive, and with the cryo, somewhat dangerous. I am most definitely in the realm of a 'hacker' in this endeavor (and have been so in other fields for a number of years), so I know what to expect (or at least I think I do).

Radar magnetrons operate in pulsed mode.    The output power during the pulse may be 20 kW but the duration is very short and so the average power output is very low.   Any high power CW magnetron is very dangerous to play with when it is removed from the equipment it was designed for.    The RF discharges can cause serious injury and temporary blindness.
Title: Re: EM Drive Developments - related to space flight applications - Thread 2
Post by: TheTraveller on 05/07/2015 07:54 pm
...
My engineer's gut tells me the best device for "Replicators" would be the Flight Thruster as it uses narrow band RF, which can be generated by a programmable RF generator, with an auxiliary RF amplifier and the RF energy feed into the cavity by standard RF connectors and coax. This gives good control over both frequency and power, which is not really available with a magnetron based RF generator, plus stops the need to add waveguides into the build.
...

Another advantage to replicate the Flight Thruster, is that the Flight Thruster is one of only two designs that are close to what appears to be the optimum cone angle (7.5 degrees).  Only the Shawyer Experimental and Shawyer's Flight Thruster are close to this optimum geometry:

http://forum.nasaspaceflight.com/index.php?topic=36313.msg1370559#msg1370559
Saw the conic data. Nicely done.

Did you or anyone else ever write an excel spreadsheet to calc Shawyers Design Factor? If so pls link it or if not please consider doing it as your skills there are much better than mine.
Title: Re: EM Drive Developments - related to space flight applications - Thread 2
Post by: Rodal on 05/07/2015 07:57 pm
I am not an engineer, a scientist, or an inventor. I am just an english major who writes science fiction and thinks this is really cool. So, that said, is there any chance that the guys at Eagleworks are ever going to have a Gofundme? Is there any way for all the people watching this project who don't have technical skills to kick in a little money to fund those who DO?

Just asking. Seems crowdfunding might help get around some of the "big organizations won't fund our research" problem.

No crowdfunding allowed for NASA. 
Please see this as an alternative:  https://forum.nasaspaceflight.com/index.php?topic=36313.msg1366620#msg1366620
Title: Re: EM Drive Developments - related to space flight applications - Thread 2
Post by: aceshigh on 05/07/2015 07:58 pm
The point I'm making is this. You see these men sitting at the table together in this video? They're all on the same team....trying to figure out how to pull off interstellar flight. For the good of all of us. Now they're duking it out in an interview in Wired. They (like us) should all be working together, pooling resources, combined knowledge and experience. Instead a rift has formed, which will likely kill progress.

it seems to me Wired totally distorted what Davis and Millis said. Most comments also note that.
Title: Re: EM Drive Developments - related to space flight applications - Thread 2
Post by: Rodal on 05/07/2015 07:59 pm
....

Did you or anyone else ever write an excel spreadsheet to calc Shawyers Design Factor? If so pls link it or if not please consider doing it as your skills there are much better than mine.
Yes I have calculated it, but it is a Mathematica program, not an Excel spreadsheet.  I posted (earlier in the thread) comparisons of the measurements vs. predictions using Shawyer's and McCulloch's formulas.

You may want to PM @aero to ask whether he did it with Excel (if my memory is correct @aero also calculated Shawyer's Design Factor, as I recall having exchanges in this forum with him).

And of course, when running your program, you will first check your results vs. Shawyer's published Design Factor results, etc., to make sure that your program is correct.
Title: Re: EM Drive Developments - related to space flight applications - Thread 2
Post by: Rodal on 05/07/2015 08:01 pm
The point I'm making is this. You see these men sitting at the table together in this video? They're all on the same team....trying to figure out how to pull off interstellar flight. For the good of all of us. Now they're duking it out in an interview in Wired. They (like us) should all be working together, pooling resources, combined knowledge and experience. Instead a rift has formed, which will likely kill progress.

it seems to me Wired totally distorted what Davis and Millis said. Most comments also note that.
That's why everybody would be well-advised never to talk to reporters, unless they can be assured in writing by the reporter that nothing will get published without their prior approval   ;)
Title: Re: EM Drive Developments - related to space flight applications - Thread 2
Post by: TheTraveller on 05/07/2015 08:10 pm

I've been looking at surplus 20kw radar magnetrons on ebay, and the availability of 3D printed silicons carbide base on which to plate my Magnesium diboride superconductor, and also at cryocoolers. It's all very complex and expensive, and with the cryo, somewhat dangerous. I am most definitely in the realm of a 'hacker' in this endeavor (and have been so in other fields for a number of years), so I know what to expect (or at least I think I do).

Radar magnetrons operate in pulsed mode.    The output power during the pulse may be 20 kW but the duration is very short and so the average power output is very low.   Any high power CW magnetron is very dangerous to play with when it is removed from the equipment it was designed for.    The RF discharges can cause serious injury and temporary blindness.
Yup.

Was one of the reasons I went with a programmable narrow band RF generator and separate solid state RF power amplifier, all connected together with coax & twist lock / screw on connectors. Plus this was what Shawyer did for his Flight Thruster. Except he used a Travelling Wave Tube RF amplifier as he had to interface with existing space rated gear.

Bottom line, keep it KISS and follow what has been done before.

In his latest superconducting tests, note the use of coax and a programmable RF generator.

Should add Shawyer's development path is not to use massive RF power, but to increase the cavity Q, via superconducting cavity and thus to cause a massive increase in thrust, with no need to increase cavity RF power input.

So higher cavity Q and lower power is expected to give a better result than using lower cavity Q and higher power. Which is why I'm replicating the Flight Thruster and designing to use spherical concave / conver end plates so as to get the highest cavity Q I can.
Title: Re: EM Drive Developments - related to space flight applications - Thread 2
Post by: MrVibrating on 05/07/2015 08:18 pm
@TheTraveller

I know the guy with the magnetic bearing in that vid you posted - a bloke called Craigy, now works out of a private R&D lab in east London.  We were both members of Steorn's SKDB (aka "The Spudclub") and what you see in the video is a rough draught of one of Steorn's low-friction bearings, a design that came to be known as "nero zero" for its low friction (not to be confused with their patented Earnshaw-defying "Zero-F" bearings which are entirely non-contact passive bearings).

The basic design for a nero zero bearing is pretty much what you see there, although can be improved by using a diametrically-polarised NdFeB disc magnet on the bottom of the vertical shaft, levitating above a toroidal NdFeB - ideally of N42 grade or better if possible.  The top of the shaft tapers to a needle, resting in a jewel cup.  The jewel is affixed to the underside of a micrometer head, so that the ride height of the whole shaft + disc magnet assembly can be adjusted over the ring magnet stator; this allows accurate tuning of stability vs friction.  I actually still have one here lying around...   we used to test them in terms of their wind-down period, and a well set up rig weighing a just a couple of hundred grams can easily achieve wind-downs of 10 minutes from 1,000 rpm.

The fully Zero-F bearings (no contact) are slightly more complex, using a mixture of paramegnetic and ferromagnetic materials - hence why they're not truly Earnshaw-exceptions, although they achieve the same end - but the nero-zero articles are good enough for measuring in the nJ range..  We used to use laser tachos, establish the MoI and baseline loss rates then calculate magnetic interaction efficiencies from there...  You can find everything cheap on eBay, and even get magnets made to your own specs for very little..
Title: Re: EM Drive Developments - related to space flight applications - Thread 2
Post by: R.W. Keyes on 05/07/2015 08:27 pm
I think that the solution is really simple, if it's about demonstrating a thrust effect that is many times larger than measurement precision: Crank up the RF power. A lot. There is really no two ways about it. Please don't even try to play with a power level that even a 9V-battery can put out.. . Personally, I'd play in a power regime of about 1kW (actually I do, but that's another story..) . 1KW is a level that can easily be handled by readily available parts and off-the-shelf electronics, but is still not excessive.

How about using Tesla batteries for a few seconds (>300 KW?) and send the damn thing at the other side of the galaxy? Would that form a convincing test result?
From what I distill from some of the information I have read, coronal discharges were already plaguing some of the teams at power between 100 and 1000 Watts. a 300KW burst would probably fry everything we can throw at it and would require serious re-design of the components and use of materials, if at all possible.
The problems with coronal discharges were with amplifiers. Magnetrons aren't affected by this, as they have no capacitors.
Tesla's batteries are unobtanium for the next year, it appears, so I am not even evaluating them at this stage.
Title: Re: EM Drive Developments - related to space flight applications - Thread 2
Post by: R.W. Keyes on 05/07/2015 08:34 pm

I've been looking at surplus 20kw radar magnetrons on ebay, and the availability of 3D printed silicons carbide base on which to plate my Magnesium diboride superconductor, and also at cryocoolers. It's all very complex and expensive, and with the cryo, somewhat dangerous. I am most definitely in the realm of a 'hacker' in this endeavor (and have been so in other fields for a number of years), so I know what to expect (or at least I think I do).

Radar magnetrons operate in pulsed mode.    The output power during the pulse may be 20 kW but the duration is very short and so the average power output is very low.   Any high power CW magnetron is very dangerous to play with when it is removed from the equipment it was designed for.    The RF discharges can cause serious injury and temporary blindness.

I hadn't known about their pulsed nature, but that makes sense. This doesn't make them useless, but does limit the application. I am thinking I should start with low power magnetrons. I understand that RF at this wavelength and power are dangerous, and I would certainly take much caution in its use. But certainly, lower power is going to give some safety margin.

You can actually be killed by microwaves. I have heard stories about Navy accidents...
Title: Re: EM Drive Developments - related to space flight applications - Thread 2
Post by: R.W. Keyes on 05/07/2015 08:44 pm
I think that the solution is really simple, if it's about demonstrating a thrust effect that is many times larger than measurement precision: Crank up the RF power. A lot. There is really no two ways about it. Please don't even try to play with a power level that even a 9V-battery can put out.. . Personally, I'd play in a power regime of about 1kW (actually I do, but that's another story..) . 1KW is a level that can easily be handled by readily available parts and off-the-shelf electronics, but is still not excessive.
Which battery types would you recommend for 1 KW operation?

I personally use LiFePO4 batteries, as they are inherently safer than alternatives. For instance, I have one (about 1kg) with 8400mAh capacity and 30C continuous discharge capability (252 amps). You can extract ~1kW for a couple minutes, which should be enough for measurements. Recharging is also fast.

LiFePO4 is a good battery tech..better, safer than others out there though not quite as much power/mass. For short tests, I think that whatever can be effectively used is good. For actual vehicles, where safety and reliability is required, I'd want LiFePO4.

Regarding your earlier comments, about just using more power to get results: I think we're going to see two different methodologies here,  the first being what Eagleworks is doing, which is carefully thought out, precision science, with work on theory to explain and therefore maximize efficiency. The other approach is the hacker approach, which is more of a mechanical and pragmatic approach, of doing things like applying more power or using different materials based upon hunches...that the proof will be in the pudding, so to speak. The problems arise when trying to unify the approaches - without some really fantastic results (such as a flying car), no one is going to let a hacker near the precision and certified labs to prove their claims, and the naysayers will have a field-day with "outlandish claims". Much of the guff EMdrive etc have received so far is because of insufficient theory and tests. I don't have a PhD in physics, and even if I did, if I wasn't prominent in the field I'd see no reason why Eagleworks or other reputable places would waste their time with me or the hundred other 'crackpots' in serious tests.

The exception to this may be if a program were implemented for inexpensive, quick, formal tests. An example which passed these tests would then be eligible for further testing and scrutiny. I think that such a program would be the best interface between the 'hackers' and the 'scientists'.

I've been looking at surplus 20kw radar magnetrons on ebay, and the availability of 3D printed silicons carbide base on which to plate my Magnesium diboride superconductor, and also at cryocoolers. It's all very complex and expensive, and with the cryo, somewhat dangerous. I am most definitely in the realm of a 'hacker' in this endeavor (and have been so in other fields for a number of years), so I know what to expect (or at least I think I do).
My engineer's gut tells me the best device for "Replicators" would be the Flight Thruster as it uses narrow band RF, which can be generated by a programmable RF generator, with an auxiliary RF amplifier and the RF energy feed into the cavity by standard RF connectors and coax. This gives good control over both frequency and power, which is not really available with a magnetron based RF generator, plus stops the need to add waveguides into the build.

I'm doing as close as possible to Shawyer replication as I can as I feel the closer I stay to Shawyer, the higher the chance of success. WHEN that works I may start trying stuff outside what Shawyer has shared.
The nice thing about magnetrons is you get a lot of power without having to use an amplifier, and hence capacitors, which don't behave well in vacuum. Of course, if you're not worried about tests in a vacuum, then something less of an RF sledgehammer, some RF signal generator with more finesse, has advantages. But, amplification becomes expensive. TWTs (Travelling Wave Tubes) are more expensive that Magnetrons. I intend to used both approaches, but will probably start with the Magnetron, in an attempt to replicate Shawyer's work, before I do anything else.
Title: Re: EM Drive Developments - related to space flight applications - Thread 2
Post by: TheTraveller on 05/07/2015 08:46 pm
....

Did you or anyone else ever write an excel spreadsheet to calc Shawyers Design Factor? If so pls link it or if not please consider doing it as your skills there are much better than mine.
Yes I have calculated it, but it is a Mathematica program, not an Excel spreadsheet.  I posted (earlier in the thread) comparisons of the measurements vs. predictions using Shawyer's and McCulloch's formulas.

You may want to PM @aero to ask whether he did it with Excel (if my memory is correct @aero also calculated Shawyer's Design Factor, as I recall having exchanges in this forum with him).

And of course, when running your program, you will first check your results vs. Shawyer's published Design Factor results, etc., to make sure that your program is correct.
Mathematica looks interesting but maybe later as I suspect there would be a learning curve.

Is this still your Design Factor equation?

Thanks for your assistance. Most appreciated.
Title: Re: EM Drive Developments - related to space flight applications - Thread 2
Post by: R.W. Keyes on 05/07/2015 08:49 pm
I wonder if some of us who are closeby each other would want to meet up to discuss the concepts, and perhaps team up to build. I am currently in Pittsfield, Massachusetts, so it easy for me to drive to Albany, Springfield, Worcester, or Hartford to meet. Boston is a bit more of a stretch. I'll be on vacation in Ireland in the beginning of June, so I may be able to meet people there.

I presently own an industrial building of moderate size (4000sf+attached 2200sf house) which I have considered selling, but perhaps it would make an adequate workshop for emdrive experiments. It needs some work, however.

EDIT I also like the idea of sharing compute power, and computer resources in general. I have a moderately fast Linux machine with 24GB of ram that I could share, in addition to a bunch of older machines (but is the electric consumption worth the CPU power they could provide?) I may also be able to get a COMSOL license (very uncertain at this point), but I have no idea how to run it.
Title: Re: EM Drive Developments - related to space flight applications - Thread 2
Post by: zen-in on 05/07/2015 08:52 pm

The problems with coronal discharges were with amplifiers. Magnetrons aren't affected by this, as they have no capacitors.
Tesla's batteries are unobtanium for the next year, it appears, so I am not even evaluating them at this stage.

RF arcing occurs with any kind of RF amplifier if the load is not matched.   It has nothing to do with capacitors.    When the amplifier experiences a high SWR power is reflected back to the amplifier.   This creates a standing wave on the feedline.    The high voltages present on the feedline can destroy a solid state amplifier.    Tube amplifers are more resistant to this kind of damage and often provide a visual indication; but internal arcing can destroy them as well.    The main problem is that the cavity's high Q (several 100k or k depending on how it is measured ;)) means it is next to impossible to tune the drive to resonance.   So most of the power is reflected.    This is especially a problem in a vacuum.
Title: Re: EM Drive Developments - related to space flight applications - Thread 2
Post by: TheTraveller on 05/07/2015 08:57 pm
@TheTraveller

I know the guy with the magnetic bearing in that vid you posted - a bloke called Craigy, now works out of a private R&D lab in east London.  We were both members of Steorn's SKDB (aka "The Spudclub") and what you see in the video is a rough draught of one of Steorn's low-friction bearings, a design that came to be known as "nero zero" for its low friction (not to be confused with their patented Earnshaw-defying "Zero-F" bearings which are entirely non-contact passive bearings).

The basic design for a nero zero bearing is pretty much what you see there, although can be improved by using a diametrically-polarised NdFeB disc magnet on the bottom of the vertical shaft, levitating above a toroidal NdFeB - ideally of N42 grade or better if possible.  The top of the shaft tapers to a needle, resting in a jewel cup.  The jewel is affixed to the underside of a micrometer head, so that the ride height of the whole shaft + disc magnet assembly can be adjusted over the ring magnet stator; this allows accurate tuning of stability vs friction.  I actually still have one here lying around...   we used to test them in terms of their wind-down period, and a well set up rig weighing a just a couple of hundred grams can easily achieve wind-downs of 10 minutes from 1,000 rpm.

The fully Zero-F bearings (no contact) are slightly more complex, using a mixture of paramegnetic and ferromagnetic materials - hence why they're not truly Earnshaw-exceptions, although they achieve the same end - but the nero-zero articles are good enough for measuring in the nJ range..  We used to use laser tachos, establish the MoI and baseline loss rates then calculate magnetic interaction efficiencies from there...  You can find everything cheap on eBay, and even get magnets made to your own specs for very little..
Thanks for that info.

Will do a bit of research and yes EarnShaw is like Murphy. Party spoilers.
Title: Re: EM Drive Developments - related to space flight applications - Thread 2
Post by: TheTraveller on 05/07/2015 09:12 pm

The problems with coronal discharges were with amplifiers. Magnetrons aren't affected by this, as they have no capacitors.
Tesla's batteries are unobtanium for the next year, it appears, so I am not even evaluating them at this stage.

RF arcing occurs with any kind of RF amplifier if the load is not matched.   It has nothing to do with capacitors.    When the amplifier experiences a high SWR power is reflected back to the amplifier.   This creates a standing wave on the feedline.    The high voltages present on the feedline can destroy a solid state amplifier.    Tube amplifers are more resistant to this kind of damage and often provide a visual indication; but internal arcing can destroy them as well.    The main problem is that the cavity's high Q (several 100k or k depending on how it is measured ;)) means it is next to impossible to tune the drive to resonance.   So most of the power is reflected.
Which is why using a programmable RF generator that also allows output level to be varied, allows use of low power to lock the RF frequency to the cavity and then slowly increase power as you continue to seek cavity lock.

In Shawyers 1st 2 units, the Feasibility and the Demonstrator devices, the cavity was physically tuned to the RF signal, a magnetron. You can listen to Shawyer saying the initial part of the rotary Demonstrator test produced no thrust until the magnetron LOCKED to the cavity. I suspect what really happened was the stepper motor at the small end of the cavity, mechanically tuned the cavity to the magnetron frequency and then thrust happened.

Look at the gears and stepper motors fitted to the small end of the Demonstrator EM Drive. Very clever mechanics but not needed if you use a narrow band programmable RF source and use spherical end plates.

Shawyer learned much from his 1st 2 EM Drives and applied it all to the Flight Thruster. No physical cavity tuning needed as he used a programmable RF generator which feeds a RF amplifier. Plus he uses a sensor in the big end to provide feedback for his frequency control loop to ensure the cavity Rf frequency is always at the right frequency to dump the max RF energy into the cavity to generate the max thrust.

as an engineer, I look at the development history and my guts says, good job Mr Shawyer.
Title: Re: EM Drive Developments - related to space flight applications - Thread 2
Post by: Rodal on 05/07/2015 09:58 pm
....

Did you or anyone else ever write an excel spreadsheet to calc Shawyers Design Factor? If so pls link it or if not please consider doing it as your skills there are much better than mine.
Yes I have calculated it, but it is a Mathematica program, not an Excel spreadsheet.  I posted (earlier in the thread) comparisons of the measurements vs. predictions using Shawyer's and McCulloch's formulas.

You may want to PM @aero to ask whether he did it with Excel (if my memory is correct @aero also calculated Shawyer's Design Factor, as I recall having exchanges in this forum with him).

And of course, when running your program, you will first check your results vs. Shawyer's published Design Factor results, etc., to make sure that your program is correct.
Mathematica looks interesting but maybe later as I suspect there would be a learning curve.

Is this still your Design Factor equation?

Thanks for your assistance. Most appreciated.

(https://forum.nasaspaceflight.com/index.php?action=dlattach;topic=36313.0;attach=829379;image)


I recognize that equation is my Mathematica-writing, but I need a link to the message where I posted it, in order to remember the context.  Too long ago  :)
Title: Re: EM Drive Developments - related to space flight applications - Thread 2
Post by: deltaMass on 05/07/2015 10:00 pm
@TheTraveller
A small, slow-speed generator with magnetic bearings is I think what you need. I have looked but so far had no luck. One can find them in profusion, however, for direct drive power generation from a wind power machine. These are of course massive and expensive.
Title: Re: EM Drive Developments - related to space flight applications - Thread 2
Post by: WarpTech on 05/07/2015 10:06 pm
Latest paper by Dr. White, on http://ntrs.nasa.gov/ , exploring the idea of the quantum vacuum not being an absolute immutable and nondegradable state, and examining  some rami cations of the quantum vacuum being able to support non-trivial spatial variations in density.  They claim that their "considerations showed no predictions that were contrary to observation, and in fact duplicated predictions for energy states associated with the primary quantum number."

http://hdl.handle.net/2060/20150006842

Dynamics of the Vacuum and Casimir Analogs to the Hydrogen Atom
Harold White, Jerry Vera,y Paul Bailey,z Paul March,x Tim Lawrence,{ Andre Sylvester, and David Brady
NASA Johnson Space Center
2101 NASA Parkway, Houston, TX 77058
(Dated: April 2, 2015)

Publication Date:   Apr 02, 2015
Document ID:   
20150006842 (Acquired Apr 28, 2015)
Subject Category:   PHYSICS OF ELEMENTARY PARTICLES AND FIELDS; ATOMIC AND MOLECULAR PHYSICS; NUMERICAL ANALYSIS; COMPUTER PROGRAMMING AND SOFTWARE
Report/Patent Number:   JSC-CN-33080
Document Type:   Technical Report
Financial Sponsor:   NASA Johnson Space Center; Houston, TX, United States
Organization Source:   NASA Johnson Space Center; Houston, TX, United States
Description:   9p; In English

There is an error in the integration of equation 11. Apparently their "simple enough" integration was not simple enough!

Title: Re: EM Drive Developments - related to space flight applications - Thread 2
Post by: zen-in on 05/07/2015 10:12 pm
Which is why using a programmable RF generator that also allows output level to be varied, allows use of low power to lock the RF frequency to the cavity and then slowly increase power as you continue to seek cavity lock.

In Shawyers 1st 2 units, the Feasibility and the Demonstrator devices, the cavity was physically tuned to the RF signal, a magnetron. You can listen to Shawyer saying the initial part of the rotary Demonstrator test produced no thrust until the magnetron LOCKED to the cavity. I suspect what really happened was the stepper motor at the small end of the cavity, mechanically tuned the cavity to the magnetron frequency and then thrust happened.

Look at the gears and stepper motors fitted to the small end of the Demonstrator EM Drive. Very clever mechanics but not needed if you use a narrow band programmable RF source and use spherical end plates.

Shawyer learned much from his 1st 2 EM Drives and applied it all to the Flight Thruster. No physical cavity tuning needed as he used a programmable RF generator which feeds a RF amplifier. Plus he uses a sensor in the big end to provide feedback for his frequency control loop to ensure the cavity Rf frequency is always at the right frequency to dump the max RF energy into the cavity to generate the max thrust.

as an engineer, I look at the development history and my guts says, good job Mr Shawyer.

It makes for an interesting theory as to why Shawyer's results were so much better than EW.  Earlier in thread 1 I stated a better method would be to just make a cavity oscillator out of the whole thing.    It's a lot easier to make an RF circuit oscillate at the resonant frequency than it is to "lock" the frequency.    What does he mean by "lock" anyway?   And what kind of sensor tells him the generator is at the right frequency?    It really looks more like a cavity oscillator that is constrained to a narrow frequency band.    If the blocks labelled FGU were bandpass filters followed by amplifiers it would be a cavity oscillator.    That might be what he is doing but there is still nothing that explains how fundamental laws of physics are violated.
Title: Re: EM Drive Developments - related to space flight applications - Thread 2
Post by: deltaMass on 05/07/2015 10:33 pm
@frobnicat
Quote
Added emphasis. If it can be of any comfort to you, I understand both your arguments and humour (so far).
Gratifying because I'm sure many don't. We do seem to think along the same lines. On a practical note, I think the next step is to predict how best to distinguish the "trash Noether" or "trash Einstein" models. This comes down to doing what TheTraveller describes when comparing input and output power using some sort of rotary device. We will have a differential equation that includes velocity-dependent friction.  We will need to devise a good metric so as to be able to best differentiate between the two scenarios based on the Pout vs. Pin data. I am handwaving here because I have not yet drilled down on this.

Here in a nutshell are my "position papers"
http://forum.nasaspaceflight.com/index.php?topic=36313.msg1369875#msg1369875
http://forum.nasaspaceflight.com/index.php?topic=36313.msg1370943#msg1370943
Title: Re: EM Drive Developments - related to space flight applications - Thread 2
Post by: WarpTech on 05/07/2015 10:37 pm
I know Shawyer and EW have tried a dielectric in the frustum. Are there any specifications for that dielectric? Material properties? Absorption properties at microwave frequencies?

I was looking at Pyramid Absorbers for microwaves, they can attenuate up to -55dB. A high power microwave source, pumped through a diode into such an absorber, seems to me should have a higher probability of thrust than the EM Drive and relatively simple to construct.


Todd D.
Title: Re: EM Drive Developments - related to space flight applications - Thread 2
Post by: PaulF on 05/07/2015 10:38 pm
I think that the solution is really simple, if it's about demonstrating a thrust effect that is many times larger than measurement precision: Crank up the RF power. A lot. There is really no two ways about it. Please don't even try to play with a power level that even a 9V-battery can put out.. . Personally, I'd play in a power regime of about 1kW (actually I do, but that's another story..) . 1KW is a level that can easily be handled by readily available parts and off-the-shelf electronics, but is still not excessive.

How about using Tesla batteries for a few seconds (>300 KW?) and send the damn thing at the other side of the galaxy? Would that form a convincing test result?
From what I distill from some of the information I have read, coronal discharges were already plaguing some of the teams at power between 100 and 1000 Watts. a 300KW burst would probably fry everything we can throw at it and would require serious re-design of the components and use of materials, if at all possible.
The problems with coronal discharges were with amplifiers. Magnetrons aren't affected by this, as they have no capacitors.
Tesla's batteries are unobtanium for the next year, it appears, so I am not even evaluating them at this stage.
Another way to get high power from a small battery pack is to use non-rechargeable liquid-electrolyte lithium batteries. They have a hell of a capacity per kilogram. Put a good few of those in series and parallel and you can probably power up to 2.5 kW for a short time. Disadvantage is that they are non-rechargeable and not exactly cheap.. But they could be useful for short duration high power tests to check linearity or whatever you science guys want for comparison with lower power tests.
Title: Re: EM Drive Developments - related to space flight applications - Thread 2
Post by: ThereIWas3 on 05/07/2015 10:47 pm
Using low temps and low power is definitely the way to go during this "how does it work?" phase.  Much safer.  But keep your eyes away from it and have a microwave leak detector at hand.

The high-power engineering can be done after the basic principles involved are understood.
Title: Re: EM Drive Developments - related to space flight applications - Thread 2
Post by: ArthurEKing on 05/07/2015 10:57 pm
     Okay, well I'm a first time poster, I'm not a physics major, or mathematician, and I'm expecting people to shoot down what I'm going to say anyways, but hopefully someone will pay it a little mind.  That said, I think I may have an idea about this that may just answer many of the problems that most people have with the still-experimental results that Dr. White is posting here.

     I will start by saying that I really don't know what math exactly would be involved, and if anyone DOES know, and is willing to either do it, or help me do it, then feel free!  Also I'm going to start my theory making only 3 assumptions.  They are as follows.

Assumption 1) That the results Dr. White is posting are correct, and unbiased, that he has followed scientific protocols within the limits of his abilities, and that he has drawn conclusions from empirical evidence as is the scientific method.

Assumption 2) That the suspected warp-bubble like effects are NOT a malfunction of scientific instrumentation, and are not being either misrepresented, or misinterpreted.

Assumption 3)  The laws of physics are in fact NOT being broken.

     Now  assumption 3 is of course the big one that everyone has a problem with, but if you hear me out I believe you may be able to come to an understanding about it.

     You see the law of conservation of momentum, easily stated, is that every action has an equal and opposite reaction.  Of course, being a law, it is inviolable, and anything that does not conform to it cannot exist, and has to be immediately discarded as junk science, right?  And of course the EMDrive DOES seem to break this law, and therefore it has to be fake, or some kind of misunderstanding.

     But what if it ISN'T breaking that law?  What if instead we include ANOTHER law of physics.

     The law of conservation of energy.

     You see, energy cannot be created, or destroyed, it can only change forms.  So if we are not seeing an equal and opposite reaction in kinetic energy, than perhaps the energy has been converted to some other form, and THIS is in fact the reason why the EMDrive actually works, as it is effectively converting kinetic energy from one direction into some other form of energy, and ergo there is a net thrust differential.

     But what form would this energy take?  Waste heat is always a good culprit, as entropy states.  But that doesn't seem to be the whole store, so perhaps could there be a different answer?

     Dr. White has theorized that the EMDrive is in some way interacting with the quantuum vacuum (and other scientists have theorized the same).  IF it is in fact interacting with the quantuum vacuum in some manner, than perhaps this may give us the solution to the warp-field being detected. 

     The quantuum vacuum is often stated as being the lowest possible energy-state of the universe, or in another way of looking at it, a state of 0 energy, which is of course why you can't extract thrust from it, or interact with it, because there's no energy to be extracted.

     However, if you DID in fact do so, you would in essence be creating a negative energy sum.  which when added to the thrust of the EMDrive, would equate to a 0-sum, and thus an equal, and opposite, reaction.  Just that the reaction would not be in a form that we would recognize as momentum, ergo, it appears to us as net thrust.

     If that is the case, this would satisfy some of the requirements of Alcubierre's theorem, allowing for the production of warp-bubbles, which in turn would explain the interference pattern that has been detected.

     Again, this is all just conjecture, and perhaps my understanding of physics is horribly flawed, or someone will do the math involved, and just disprove what I postulated.  However, if it's accurate, it would in essence explain all of the results we have experienced, and effectively wrap everything up in a neat little package that actually makes sense, and does NOT defy the laws of physics.

     Now as for HOW it is converting that energy, THAT I have no idea, but again, this is just something I came up with today.  Feel free to poke holes in it.

Title: Re: EM Drive Developments - related to space flight applications - Thread 2
Post by: Rodal on 05/07/2015 11:07 pm
I know Shawyer and EW have tried a dielectric in the frustum. Are there any specifications for that dielectric? Material properties? Absorption properties at microwave frequencies?
I was looking at Pyramid Absorbers for microwaves, they can attenuate up to -55dB. A high power microwave source, pumped through a diode into such an absorber, seems to me should have a higher probability of thrust than the EM Drive and relatively simple to construct.


Todd D.

NASA Eagleworks has used a High Density Polyethylene dielectric insert for most of their experiments.  It was obtained from McMaster Carr.  It came from a supplier of extruded HDPE rods.

http://www.mcmaster.com/#standard-plastic-rods/=w0bzy0

under Rigid HDPE Polyethylene, apparently they no longer supply the 6.25" dimension.  Biggest diameter they list is 6.00 inches:  Rigid HDPE Polyethylene Rod, 6" Diameter

Tan delta or the dielectric constant of the HDPE were not measured at NASA Eagleworks.  I understand from prior communications in this thread that NASA Eagleworks does not have a dielectrometer at their lab.  So they used commonly known values for HDPE in their calculations.

(http://forum.nasaspaceflight.com/index.php?action=dlattach;topic=36313.0;attach=636341;image)

Bigger image (click here to download):
http://forum.nasaspaceflight.com/index.php?action=dlattach;topic=36313.0;attach=634621
............................................

DIELECTRIC STRENGTH

Under Performance of Plastics it directs you to a .pdf  that lists

450-1,800  Dielectric Strength, volts/0.001inch

that is 17.72 million V/m (on the low end) that's a little lower than the lowest value listed here: 

http://hypertextbook.com/facts/2009/CherryXu.shtml

but the same minimum listed here:

http://www.rfcafe.com/references/electrical/dielectric-constants-strengths.htm

************
The only attenuation parameter that was discussed was the attenuation dielectric loss tangent, but in other contexts:

DIELECTRIC LOSS TANGENT

Paul March, during some discussions quoted

Quote
RF loss tangent from that of HDPE  (~0.0004)



Interestingly (for this thread's discussion due to the significance that the NASA experimenters have placed on the dielectric being responsible for providing the measured thrust) is what happens in the High Density PolyEthylene (HDPE) dielectric polymer insert.  Because the dimensions of the dielectric are not negligible compared to the EM Drive's dimension, and the dielectric is not modeled as just a boundary condition.

The loss tangent of HDPE is reported to be

tan delta = 0.0004

Therefore the intrinsic impedance angle is

intrinsic impedance angle =(ArcTan[0.0004])/2

Therefore, inside the HDPE dielectric the electric and magnetic fields, instead of being out of phase by 90 degrees (as they are in the air or vacuum medium), will be out of phase by:

90 - (180/Pi) (ArcTan[0.0004])/2 = 89.98854084470854 degrees

EDIT: Thanks to @ace for pointing out the originally missing conversion factor from radians to degrees

This phase angle (89.9885 degrees)  will show practically no visual difference with 90 degrees at the resolution of the following image :

(http://www-ssc.igpp.ucla.edu/personnel/russell/papers/skip_ed/fig9.gif)

TRAVELLING WAVE POYNTING VECTOR                                           STANDING WAVE POYNTING VECTOR
OPEN WAVEGUIDE                                                                          CLOSED CAVITY

The Poynting vector inside the HDPE dielectric, instead of having a zero time average, will have the following factor multiplying ExB/mu :

Cos[(Pi/180) (90 - (180/Pi) (ArcTan[0.0004])/2)]/2= 0.00009999999400006368

So, inside the HDPE polymer dielectric the Poynting vector has this small magnitude over a period (or multiples thereof).

So, the extent of this approximation, for the HDPE dielectric is about 0.01 % (which is negligible in comparison to several other approximations).


Now, let's examine what this (very small intensity Poynting vector time average) means, concerning the discussion in this EM Drive.

If one were to posit that the EM Drive's thrust is due to the very small magnitude of the time average of the Poynting vector due to these thermal losses (in the HDPE dielectric or in the copper):

1) It would mean that there should be more thrust with lower Q.  This is the complete opposite of what the experimenters like Shawyer claims (Shawyer claims that the higher the Q, the greater the thrust).  Notice that

Tan [loss angle] = Tan[ 2 impedance angle ]= 1/Q

2) All the equations proposed so far (by Shawyer, McCulloch and @Notsosureofit) have predicted thrust proportional to Q.  This is the complete opposite of what such a Poynting vector would predict (it would predict thrust proportional to 1/Q instead), because

Tan delta= 1/Q

measured Q        effective tan delta

7320                  1.366* 10 ^(-4)
22000                4.545* 10 ^(-5)
10^6                  10^(-7)

3) It would mean that experimenters like Shawyer and Fetta are in the completely wrong track pursuing superconducting EM Drives, as superconducting EM Drives would lead to practically no thrust (the opposite of what they claim) because superconducting EM Drives would display practically no heat losses and hence zero time average Poynting vector.

4) Considering the HDPE dielectric acting as a sink (energy flowing from the EM Drive towards the HDPE where the energy is dissipated internally in the dielectric polymer due to its tandelta and hence irretrievably lost instead of being reflected), the Poynting vector would be directed towards the HDPE dielectric, that is towards the small base, and hence the EM Drive should experience a recoil force and acceleration towards the big base.  This is the opposite direction force found in NASA's experiments with the dielectric.  (Recall that NASA Eagleworks found no thrust force with mode TE012 without a HDPE dielectric and that with the HDPE dielectric inserted at the small base they found a force and acceleration directed towards the small base.)

Title: Re: EM Drive Developments - related to space flight applications - Thread 2
Post by: PaulF on 05/07/2015 11:15 pm
     Okay, well I'm a first time poster, I'm not a physics major, or mathematician, and I'm expecting people to shoot down what I'm going to say anyways, but hopefully someone will pay it a little mind.  That said, I think I may have an idea about this that may just answer many of the problems that most people have with the still-experimental results that Dr. White is posting here.

     I will start by saying that I really don't know what math exactly would be involved, and if anyone DOES know, and is willing to either do it, or help me do it, then feel free!  Also I'm going to start my theory making only 3 assumptions.  They are as follows.

Assumption 1) That the results Dr. White is posting are correct, and unbiased, that he has followed scientific protocols within the limits of his abilities, and that he has drawn conclusions from empirical evidence as is the scientific method.

Assumption 2) That the suspected warp-bubble like effects are NOT a malfunction of scientific instrumentation, and are not being either misrepresented, or misinterpreted.

Assumption 3)  The laws of physics are in fact NOT being broken.

     Now  assumption 3 is of course the big one that everyone has a problem with, but if you hear me out I believe you may be able to come to an understanding about it.

     You see the law of conservation of momentum, easily stated, is that every action has an equal and opposite reaction.  Of course, being a law, it is inviolable, and anything that does not conform to it cannot exist, and has to be immediately discarded as junk science, right?  And of course the EMDrive DOES seem to break this law, and therefore it has to be fake, or some kind of misunderstanding.

     But what if it ISN'T breaking that law?  What if instead we include ANOTHER law of physics.

     The law of conservation of energy.

     You see, energy cannot be created, or destroyed, it can only change forms.  So if we are not seeing an equal and opposite reaction in kinetic energy, than perhaps the energy has been converted to some other form, and THIS is in fact the reason why the EMDrive actually works, as it is effectively converting kinetic energy from one direction into some other form of energy, and ergo there is a net thrust differential.

     But what form would this energy take?  Waste heat is always a good culprit, as entropy states.  But that doesn't seem to be the whole store, so perhaps could there be a different answer?

     Dr. White has theorized that the EMDrive is in some way interacting with the quantuum vacuum (and other scientists have theorized the same).  IF it is in fact interacting with the quantuum vacuum in some manner, than perhaps this may give us the solution to the warp-field being detected. 

     The quantuum vacuum is often stated as being the lowest possible energy-state of the universe, or in another way of looking at it, a state of 0 energy, which is of course why you can't extract thrust from it, or interact with it, because there's no energy to be extracted.

     However, if you DID in fact do so, you would in essence be creating a negative energy sum.  which when added to the thrust of the EMDrive, would equate to a 0-sum, and thus an equal, and opposite, reaction.  Just that the reaction would not be in a form that we would recognize as momentum, ergo, it appears to us as net thrust.

     If that is the case, this would satisfy some of the requirements of Alcubierre's theorem, allowing for the production of warp-bubbles, which in turn would explain the interference pattern that has been detected.

     Again, this is all just conjecture, and perhaps my understanding of physics is horribly flawed, or someone will do the math involved, and just disprove what I postulated.  However, if it's accurate, it would in essence explain all of the results we have experienced, and effectively wrap everything up in a neat little package that actually makes sense, and does NOT defy the laws of physics.

     Now as for HOW it is converting that energy, THAT I have no idea, but again, this is just something I came up with today.  Feel free to poke holes in it.
As far as the QV goes, and also assuming all the thing you just assumed (also my preference) the QV could be considered point-like when you actually tap into it. A singularity. Only difference is this singularity is the same singularity where ever in the universes spacetime you poke a hole. No matter where you are, poke a hole in spacetime and you are tapping into the exact same singularity. Not a singularity like a black hole as we know it, but one that is the result of a higher dimensional Quantum Vacuum. In that higher dimension, the QV could be anything, We cant know that. Some accepted theories use more than our 3+1 spacetime dimensions. Having no dimensions to us means that it also has no preferred reference frame, solving that part of GR. What it manifests itself as could be in fact like the earth is for our electricity grid. It doesn't matter where on earth you are you can use it as a return for your power line.
At this moment, it just costs a whole load of power to punch through to our "QV-earth".

Continuing on that matter, it could be possible that the energy that black holes suck in is energy that in fact is just being returned to the QV, where it presumably came from. Big bang anyone?

Could it be that by opening a "hole" to the QV , that the effect of negative energy is leaking through to our dimensions causing the, by some asumed, gradient? I say negative energy because on our side we have only positive energy. It sounds reasonable if one posits that the big bang burped all it's energy into two realities; positive in one, and negative in the other (the QV), hell for all that matters the QV could be nothing else than the negative-energy mirror of our universe. By the way that could also explain dark energy. I know I don't sound scientific by bantering all this, they are just thoughts to ponder. Maybe one day someone will get an inception for something truely wonderful from this.
Title: Re: EM Drive Developments - related to space flight applications - Thread 2
Post by: LasJayhawk on 05/08/2015 12:25 am
Does anyone know on the EW tests, what the total forward and reflected power was ( not the net )?

iOS auto correct strikes again...
Title: Re: EM Drive Developments - related to space flight applications - Thread 2
Post by: TheTraveller on 05/08/2015 01:03 am
Using low temps and low power is definitely the way to go during this "how does it work?" phase.  Much safer.  But keep your eyes away from it and have a microwave leak detector at hand.

The high-power engineering can be done after the basic principles involved are understood.
2.45GHz is home microwave oven frequency. Most run around 600W. I'm sure forum menber have seen what 600W of microwave can do in regard to plasma creation and other pryotechinque tricks in a microwave oven.

Using a 1kW magnetron could do serious damage to people & equipment around a EM Drive if any microwave energy escaped. So please use microwave leak detectors and limit time close to the cavity.

Shawyer has experienced this. He made this comment about leaking microwaves interfering with his video camera when filming the rotary test rig.

Is why I'll build a Faraday Cage around my test unit so to stop microwave leaks interfering with my test and control system.
Title: Re: EM Drive Developments - related to space flight applications - Thread 2
Post by: deltaMass on 05/08/2015 01:34 am
@TheTraveller
A small, slow-speed generator with magnetic bearings is I think what you need. I have looked but so far had no luck. One can find them in profusion, however, for direct drive power generation from a wind power machine. These are of course massive and expensive.
http://www.alibaba.com/product-detail/2014-promotional-bldc-motor-magnetic-bearing_1803241008.html
might fit the bill, using good quality gearing so the gennie runs fast
Title: Re: EM Drive Developments - related to space flight applications - Thread 2
Post by: PaulF on 05/08/2015 01:48 am

I've been looking at surplus 20kw radar magnetrons on ebay, and the availability of 3D printed silicons carbide base on which to plate my Magnesium diboride superconductor, and also at cryocoolers. It's all very complex and expensive, and with the cryo, somewhat dangerous. I am most definitely in the realm of a 'hacker' in this endeavor (and have been so in other fields for a number of years), so I know what to expect (or at least I think I do).

Radar magnetrons operate in pulsed mode.    The output power during the pulse may be 20 kW but the duration is very short and so the average power output is very low.   Any high power CW magnetron is very dangerous to play with when it is removed from the equipment it was designed for.    The RF discharges can cause serious injury and temporary blindness.
But..if those pulses have enough duration to make measurements..one could potentially see what happens at those powers without scaling the dangers with it so much.
Title: Re: EM Drive Developments - related to space flight applications - Thread 2
Post by: WarpTech on 05/08/2015 02:03 am
Thanks a lot! Indeed my idea was to justify White's results that appear already striking from a physicist's standpoint. I am working on the frustum case and I hope to update the paper soon.

Nice work Marco! I just finished reading through it a few times. Thank you for sharing.

Todd


Title: Re: EM Drive Developments - related to space flight applications - Thread 2
Post by: maciejzi on 05/08/2015 02:07 am
Has anyone compared the thrust of a closed setup with one side open or two sides open setup?

As in this amateur video: https://www.youtube.com/watch?v=vcaOKX7Ko7w
Title: Re: EM Drive Developments - related to space flight applications - Thread 2
Post by: TheTraveller on 05/08/2015 02:13 am
@TheTraveller
A small, slow-speed generator with magnetic bearings is I think what you need. I have looked but so far had no luck. One can find them in profusion, however, for direct drive power generation from a wind power machine. These are of course massive and expensive.
http://www.alibaba.com/product-detail/2014-promotional-bldc-motor-magnetic-bearing_1803241008.html
might fit the bill, using good quality gearing so the gennie runs fast
Looks interesting.
Will check it out for sure.
Thanks.
Title: Re: EM Drive Developments - related to space flight applications - Thread 2
Post by: Bubs on 05/08/2015 02:23 am
Can someone explain why the EMdrive experiment isn't run inside Helmholtz Coils (to cancel Earth magnetic field)?

The propulsion could be easily explained by the interaction between the large DC currents used to operate the magnetron/RF power amplifier with Earth’s magnetic field by way of the Lorentz force. In other words, a homopolar motor. This is an experiment that any child can do with a battery and a piece of wire:

http://en.wikipedia.org/wiki/Homopolar_motor

To do a proper measurement, the Earth's magnetic field should be canceled around the experiment using this setup:

http://en.wikipedia.org/wiki/Helmholtz_coil
Title: Re: EM Drive Developments - related to space flight applications - Thread 2
Post by: deltaMass on 05/08/2015 02:27 am
Can someone explain why the EMdrive experiment isn't run inside Helmholtz Coils (to cancel Earth magnetic field)?

The propulsion could be easily explained by the interaction between the large DC currents used to operate the magnetron/RF power amplifier with Earth’s magnetic field by way of the Lorentz force. In other words, a homopolar motor. This is an experiment that any child can do with a battery and a piece of wire:

http://en.wikipedia.org/wiki/Homopolar_motor

To do a proper measurement, the Earth's magnetic field should be canceled around the experiment using this setup:

http://en.wikipedia.org/wiki/Helmholtz_coil
It's all AC past the tie-off points, so there's no net current for this effect.
Title: Re: EM Drive Developments - related to space flight applications - Thread 2
Post by: Bubs on 05/08/2015 02:35 am
There is still a shared ground connection between the source and the device-under-test, so it's not fully balanced AC. You can never be sure about residual ground currents, ground loops etc. It can easily cause this residual force, and explain the large variation between various experimental setups.

It would be a wise precaution to run this experiment inside Helmholtz coils to cancel possible interactions with Earth's magnetic field.
Title: Re: EM Drive Developments - related to space flight applications - Thread 2
Post by: Rodal on 05/08/2015 02:37 am
Can someone explain why the EMdrive experiment isn't run inside Helmholtz Coils (to cancel Earth magnetic field)?

The propulsion could be easily explained by the interaction between the large DC currents used to operate the magnetron/RF power amplifier with Earth’s magnetic field by way of the Lorentz force. In other words, a homopolar motor. This is an experiment that any child can do with a battery and a piece of wire:

http://en.wikipedia.org/wiki/Homopolar_motor

To do a proper measurement, the Earth's magnetic field should be canceled around the experiment using this setup:

http://en.wikipedia.org/wiki/Helmholtz_coil

Assuming one believes Shawyer's experimental reports, wouldn't that interaction with the Earth's negative field be negated by the fact that Shawyer claims to have measured similar thrust/PowerInput for the EM Drive pointing small base UP (vertical), small base DOWN (vertical), small base to the RIGHT (horizontal)  ?

(since the Lorentz force vector due to the Earth's magnetic field would have a definite direction, and therefore the measured force should vary upon direction)
Title: Re: EM Drive Developments - related to space flight applications - Thread 2
Post by: Bubs on 05/08/2015 02:41 am
Can someone explain why the EMdrive experiment isn't run inside Helmholtz Coils (to cancel Earth magnetic field)?

The propulsion could be easily explained by the interaction between the large DC currents used to operate the magnetron/RF power amplifier with Earth’s magnetic field by way of the Lorentz force. In other words, a homopolar motor. This is an experiment that any child can do with a battery and a piece of wire:

http://en.wikipedia.org/wiki/Homopolar_motor

To do a proper measurement, the Earth's magnetic field should be canceled around the experiment using this setup:

http://en.wikipedia.org/wiki/Helmholtz_coil

Assuming one believes Shawyer's experimental reports, wouldn't that interaction with the Earth's negative field be negated by the fact that Shawyer claims to have measured similar thrust/PowerInput for the EM Drive pointing small base UP (vertical), small base DOWN (vertical), small base to the RIGHT (horizontal)  ?

I referring to NASA's experimental setup, not Shawyer's. (I consider Shawyer's setup to be too sloppy - rotating platform, with laptop? with fans? and rotating hard-disk? Extremely unprofessional).

In NASA's setup, there is still a shared ground connection between the source and the device-under-test, so it's not fully balanced AC. You can never be sure about residual ground currents, ground loops etc. It can easily cause this residual force, and explain the large variation between various experimental setups.

It would be a wise precaution to run this experiment inside Helmholtz coils to cancel possible interactions with Earth's magnetic field.
Title: Re: EM Drive Developments - related to space flight applications - Thread 2
Post by: zen-in on 05/08/2015 02:44 am
There is still a shared ground connection between the source and the device-under-test, so it's not fully balanced AC. You can never be sure about residual ground currents, ground loops etc. It can easily cause this residual force, and explain the large variation between various experimental setups.

It would be a wise precaution to run this experiment inside Helmholtz coils to cancel possible interactions with Earth's magnetic field.

The EW apparatus has one or two large NIB magnets attached to the torque pendulum; part of an eddy current brake.    This has been one source of error discussed in their Aug. 2014 paper.
Title: Re: EM Drive Developments - related to space flight applications - Thread 2
Post by: Rodal on 05/08/2015 02:48 am
...
I referring to NASA's experimental setup, not Shawyer's. (I consider Shawyer's setup to be too sloppy - rotating platform, with laptop? with fans? and rotating hard-disk? Extremely unprofessional).

In NASA's setup, there is still a shared ground connection between the source and the device-under-test, so it's not fully balanced AC. You can never be sure about residual ground currents, ground loops etc. It can easily cause this residual force, and explain the large variation between various experimental setups.

It would be a wise precaution to run this experiment inside Helmholtz coils to cancel possible interactions with Earth's magnetic field.

Shawyer also had this non-rotating setup (if I recall correctly he had the EM Drive vertically in the UP and DOWN positions):

(http://forum.nasaspaceflight.com/index.php?action=dlattach;topic=36313.0;attach=829227;image)

Shawyer's website SPR states:

Quote
The maximum thrust, measured using a precision balance was 16mN for an input power of 850W, which is very close to the thrust of 16.6mN predicted from equation 1.

The thrust could be varied from zero to maximum by varying the input power, or by varying the resonant frequency of the thruster. Considerable efforts were made to test for possible thermal and electromagnetic spurious effects. The primary method was to carry out all tests in both nominal and inverted orientations, and to take the mean of the results. The thruster was also sealed into a hermetic enclosure to eliminate buoyancy effects of the cooling air. Three different types of test rig were used, two using 1 mg resolution balances in a counterbalance test rig and one using a 100 mg resolution balance in a direct measurement of thruster weight.

Comparison of the rates of increase of thrust for the different spring constants, using pulsed input power, gave a clear proof that the thrust was produced by momentum transfer and was not due to any “undefined” spurious effect.

The total test programme encompassed 450 test runs of periods up to 50 seconds, using 5 different magnetrons.



 If you are aware of Shawyer's non-rotating force measurements, did you find it to be similarly

Quote from: Bubs
Extremely unprofessional
?
Title: Re: EM Drive Developments - related to space flight applications - Thread 2
Post by: LasJayhawk on 05/08/2015 02:54 am
Dr. Rodel. What would happen if the signal with the phase shifted E&H fields ran into a non-shifted signal with the same amplitude and the E field 180 degrees out? There is a lot of RF bouncing in there after all. Seems all that would be left is a bit of the H field.

Would that leave me with a magnetic monopole? ( now wouldn't that be funny...)
Title: Re: EM Drive Developments - related to space flight applications - Thread 2
Post by: deltaMass on 05/08/2015 02:58 am
There is still a shared ground connection between the source and the device-under-test, so it's not fully balanced AC. You can never be sure about residual ground currents, ground loops etc. It can easily cause this residual force, and explain the large variation between various experimental setups.

It would be a wise precaution to run this experiment inside Helmholtz coils to cancel possible interactions with Earth's magnetic field.
I ran the numbers and was surprised how little current it takes. I assumed it ran in the shield of the RF feed coax and took length to be 0.5m. That yields 4 Amps of ground loop current necessary to produce 100 uN in the Earth's field.

So yeah - except that reversing the test article pretty much reverses the thrust and does NOT reverse your Lorentz force. So I think it's unnecessary.
Title: Re: EM Drive Developments - related to space flight applications - Thread 2
Post by: WarpTech on 05/08/2015 02:59 am
...
4) Considering the HDPE dielectric acting as a sink (energy flowing from the EM Drive towards the HDPE where the energy is dissipated internally in the dielectric polymer due to its tandelta and hence irretrievably lost instead of being reflected), the Poynting vector would be directed towards the HDPE dielectric, that is towards the small base, and hence the EM Drive should experience a recoil force and acceleration towards the big base.  This is the opposite direction force found in NASA's experiments with the dielectric.  (Recall that NASA Eagleworks found no thrust force with mode TE012 without a HDPE dielectric and that with the HDPE dielectric inserted at the small base they found a force and acceleration directed towards the small base.)

Thank you Dr. Rodal for a very informative post!

This last statement is what I was looking for. The drive works precisely because there is no internal polarizable medium pushing the other way. I was thinking about a straight cylinder with an absorber at one end and a reflector at the other end, but it's still no better than a photon rocket even with a high Q.

Todd




Title: Re: EM Drive Developments - related to space flight applications - Thread 2
Post by: TheTraveller on 05/08/2015 03:05 am
Can someone explain why the EMdrive experiment isn't run inside Helmholtz Coils (to cancel Earth magnetic field)?

The propulsion could be easily explained by the interaction between the large DC currents used to operate the magnetron/RF power amplifier with Earth’s magnetic field by way of the Lorentz force. In other words, a homopolar motor. This is an experiment that any child can do with a battery and a piece of wire:

http://en.wikipedia.org/wiki/Homopolar_motor

To do a proper measurement, the Earth's magnetic field should be canceled around the experiment using this setup:

http://en.wikipedia.org/wiki/Helmholtz_coil

Assuming one believes Shawyer's experimental reports, wouldn't that interaction with the Earth's negative field be negated by the fact that Shawyer claims to have measured similar thrust/PowerInput for the EM Drive pointing small base UP (vertical), small base DOWN (vertical), small base to the RIGHT (horizontal)  ?

I referring to NASA's experimental setup, not Shawyer's. (I consider Shawyer's setup to be too sloppy - rotating platform, with laptop? with fans? and rotating hard-disk? Extremely unprofessional).

In NASA's setup, there is still a shared ground connection between the source and the device-under-test, so it's not fully balanced AC. You can never be sure about residual ground currents, ground loops etc. It can easily cause this residual force, and explain the large variation between various experimental setups.

It would be a wise precaution to run this experiment inside Helmholtz coils to cancel possible interactions with Earth's magnetic field.
EW is actually replicating Shawyers Teeter Totter balance beam for their next round.

The Teeter Totter balance beam setup (which has the EM Drive mounted vertically, which eliminates thernal buckling effects EW experienced) was used for the 1st test EM Drive and for the 3rd EM Drive, the Flight Thruster. The rotary rig was used to test the 2nd EM Drive as was the Teeter Totter rig.

Shawyers superconduction test rig is a variation of the Teeter Totter test rigs.

That EW are changing their measurement system to Shawyers Teeter Totter balance beam system should tell you something.
Title: Re: EM Drive Developments - related to space flight applications - Thread 2
Post by: deltaMass on 05/08/2015 03:05 am
Quote from: Rodal
Quote from: Bubs
Extremely unprofessional
?
He's referring to spurious angular momentum from various rotating components of the test equipment possibly coupling into the angular momentum of the platform on the air bearing.
Title: Re: EM Drive Developments - related to space flight applications - Thread 2
Post by: WarpTech on 05/08/2015 03:06 am
Using low temps and low power is definitely the way to go during this "how does it work?" phase.  Much safer.  But keep your eyes away from it and have a microwave leak detector at hand.

The high-power engineering can be done after the basic principles involved are understood.
2.45GHz is home microwave oven frequency. Most run around 600W. I'm sure forum menber have seen what 600W of microwave can do in regard to plasma creation and other pryotechinque tricks in a microwave oven.

Using a 1kW magnetron could do serious damage to people & equipment around a EM Drive if any microwave energy escaped. So please use microwave leak detectors and limit time close to the cavity.

Shawyer has experienced this. He made this comment about leaking microwaves interfering with his video camera when filming the rotary test rig.

Is why I'll build a Faraday Cage around my test unit so to stop microwave leaks interfering with my test and control system.

From what I read today elsewhere, I gather that you should also cover the walls of your lab in microwave absorbers, so if any does leak out, it isn't reflected back.

Todd
Title: Re: EM Drive Developments - related to space flight applications - Thread 2
Post by: deltaMass on 05/08/2015 03:08 am
Why would you care? - photon rocket effects are 3 orders down
Title: Re: EM Drive Developments - related to space flight applications - Thread 2
Post by: Bubs on 05/08/2015 03:11 am
There is still a shared ground connection between the source and the device-under-test, so it's not fully balanced AC. You can never be sure about residual ground currents, ground loops etc. It can easily cause this residual force, and explain the large variation between various experimental setups.

It would be a wise precaution to run this experiment inside Helmholtz coils to cancel possible interactions with Earth's magnetic field.
I ran the numbers and was surprised how little current it takes. I assumed it ran in the shield of the RF feed coax and took length to be 0.5m. That yields 4 Amps of ground loop current necessary to produce 100 uN in the Earth's field.

So yeah - except that reversing the test article pretty much reverses the thrust and does NOT reverse your Lorentz force. So I think it's unnecessary.

NASA's RF PA runs around 10Amps, so double that number to 200uN. That's the same order of magnitude (or more) as the measured EMdrive trust, so it would be wise to completely cancel this potential magnetic interaction. Regarding reversing the test article - it depends exactly how the 'reversing' is done - how are the wires re-routed? Could they be re-routed in a different orientation? It can be easily done "wrong", wrt. to earth's magnetic field. It's better to avoid those questions entirely by running inside Helmholtz coils...
Title: Re: EM Drive Developments - related to space flight applications - Thread 2
Post by: deltaMass on 05/08/2015 03:18 am
No and no. That 10 Amps is main current, not residual ground loop current . Reversing the test article leaves the coax feed exiting in the same place, so also no.
Title: Re: EM Drive Developments - related to space flight applications - Thread 2
Post by: Bubs on 05/08/2015 03:24 am
No and no. That 10 Amps is main current, not residual ground loop current . Reversing the test article leaves the coax feed exiting in the same place, so also no.

DC loops are notorious in RF power systems. You may think that the 10 Amps are returning via your carefully routed 20 AWG ground wire, while in fact the majority of the current may be returning via the 0000 AWG metallic truss. There is no ground isolation or balanced-DC design in either experiments, so expect the worse.
Title: Re: EM Drive Developments - related to space flight applications - Thread 2
Post by: deltaMass on 05/08/2015 04:22 am
Such a shame it's so difficult to simply put it in space and check. It would have saved Woodward 20 years and I don't know how many years on this already.

I speak as someone who watched Apollo 11 on the moon.
Title: Re: EM Drive Developments - related to space flight applications - Thread 2
Post by: WarpTech on 05/08/2015 05:51 am
Consider the following EM Drive design:

1st component: A symmetrical resonator cavity that is optimized to store energy at high Q, and plays no role in thrust. Here, we want to store the lowest order mode that the waveguide can sustain because it has the slowest wave velocity inside the waveguide. By targeting the slowest wave velocity, we are maximizing the amount of stored electromagnetic momentum for a given amount electromagnetic energy.

p = E/v, where  v << c inside the waveguide.
E = P*t, energy is power in x time (assuming a lossless cavity)

2nd component: We need a frustum that matches the diameter of the resonant chamber "at the small end" and slowly expands to a TBD length and diameter. As the waveguide expands, the wavelength will be increasing proportional to the increasing wave velocity. We want it to expand to near it's free space value.

3rd component: We need a partial reflector, between the two, so that we can have resonance in one chamber, and thrust in the other. Like a laser.

Principle of operation:

When the energy stored in the resonant chamber is released into the frustum, it expands due to the increased wave velocity to a longer wavelength before it is reflected. It leaves the "nozzle" with momentum p1, but when it arrives at the reflector, it has momentum p2 << p1, because the wave velocity has increased "significantly".

The reflected waves then travel back into the frustum only to find that due to their increased wavelength, now they don't fit anymore, so they are attenuated. Again, absorbing the momentum into the frustum as heat and kinetic energy in the "forward" direction.

To put it quantum mechanically:

N photons are injected at momentum p1 = N*h/lambda_1

N photons are reflected at the far end with momentum p2 = N*h/lambda_2

The wavelengths are not equal. Momentum is conserved because the system moves forward. The red-shifted photons are mimicking a gravitational field. They are trying to escape a gravity well, only to be reflected back in and fall back through the event horizon as wave velocity goes to zero. (Sorry I love that analogy!)

Todd Desiato



Title: Re: EM Drive Developments - related to space flight applications - Thread 2
Post by: FieldEffect on 05/08/2015 08:19 am
Im seeing an overlap between this kind of work and work done by T.T.Brown...
anyone care to briefly comment on it?

(Incase your unfamiliar with T.T. Brown, he discovered Biefeld-Brown effect, but also discovered another force that he insisted was NOT biefeld-brown effect (ion wind), also he believed the dielectric played a primary role in the force, and that a reaction force existed on "all solid material bodies making up the physical environment")

He makes some very peculiar statements in his 1929 article, which lead me to believe he was an honest experimenter (ofcourse he explains in terms of what people knew in the 1920's)
the 1929 article & gravitator patent: http://www.rexresearch.com/gravitor/gravitor.htm#patent

This patent strikes me the most (attached): US3187206 (1965),
He has a Half-Wave Radiator, tapered dielectric member etc...

please tell me what you think because im seeing a fair bit of crossover.

I seriously think there is something to be gleaned from TTBrown's work.

Regards.

PS: Im thoroughly enjoying the thread, wish i was as up to speed as you lot.  8)
Title: Re: EM Drive Developments - related to space flight applications - Thread 2
Post by: deuteragenie on 05/08/2015 08:29 am
Latest paper by Dr. White, on http://ntrs.nasa.gov/ , exploring the idea of the quantum vacuum not being an absolute immutable and nondegradable state, and examining  some rami cations of the quantum vacuum being able to support non-trivial spatial variations in density.  They claim that their "considerations showed no predictions that were contrary to observation, and in fact duplicated predictions for energy states associated with the primary quantum number."

http://hdl.handle.net/2060/20150006842

Dynamics of the Vacuum and Casimir Analogs to the Hydrogen Atom
Harold White, Jerry Vera,y Paul Bailey,z Paul March,x Tim Lawrence,{ Andre Sylvester, and David Brady
NASA Johnson Space Center
2101 NASA Parkway, Houston, TX 77058
(Dated: April 2, 2015)

Publication Date:   Apr 02, 2015
Document ID:   
20150006842 (Acquired Apr 28, 2015)
Subject Category:   PHYSICS OF ELEMENTARY PARTICLES AND FIELDS; ATOMIC AND MOLECULAR PHYSICS; NUMERICAL ANALYSIS; COMPUTER PROGRAMMING AND SOFTWARE
Report/Patent Number:   JSC-CN-33080
Document Type:   Technical Report
Financial Sponsor:   NASA Johnson Space Center; Houston, TX, United States
Organization Source:   NASA Johnson Space Center; Houston, TX, United States
Description:   9p; In English

There is an error in the integration of equation 11. Apparently their "simple enough" integration was not simple enough!

Indeed.  Should be something like 4 pi A0 C^2 (Rn - R0)
Title: Re: EM Drive Developments - related to space flight applications - Thread 2
Post by: deuteragenie on 05/08/2015 08:34 am
Quote
Rodal:
Interestingly (for this thread's discussion due to the significance that the NASA experimenters have placed on the dielectric being responsible for providing the measured thrust) is what happens in the High Density PolyEthylene (HDPE) dielectric polymer insert.  Because the dimensions of the dielectric are not negligible compared to the EM Drive's dimension, and the dielectric is not modeled as just a boundary condition.

The loss tangent of HDPE is reported to be

tan delta = 0.0004

Therefore the intrinsic impedance angle is

intrinsic impedance angle =(ArcTan[0.0004])/2

Therefore, inside the HDPE dielectric the electric and magnetic fields, instead of being out of phase by 90 degrees (as they are in the air or vacuum medium), will be out of phase by:

90 - (ArcTan[0.0004])/2 = 89.9998 degrees

This phase angle (89.9998 degrees)  will show practically no visual difference with 90 degrees at the resolution of the following image :

Mr Rodal can you advise me on the value .0004 in your work above,  is that in degrees or radians or just a figure that one simply takes the arctan of.  Im trying to use the "bc" program in linux command line to get a grip on the overall topic, and bc reports results in radians, hence the requirement to do a 180/pi conversion.

echo "scale=20; 90 - (a(0.0004)/2)*(180/(4*a(1)))" | bc -l
89.98854084470853845905

Thanks
arc

Last three digits are incorrect I believe.
Here is the correct value:

89.98854084470853845862664694223578691171775499331997712909452 ...

Title: Re: EM Drive Developments - related to space flight applications - Thread 2
Post by: deuteragenie on 05/08/2015 09:11 am
Im seeing an overlap between this kind of work and work done by T.T.Brown...
anyone care to briefly comment on it?

(Incase your unfamiliar with T.T. Brown, he discovered Biefeld-Brown effect, but also discovered another force that he insisted was NOT biefeld-brown effect (ion wind), also he believed the dielectric played a primary role in the force, and that a reaction force existed on "all solid material bodies making up the physical environment")

He makes some very peculiar statements in his 1929 article, which lead me to believe he was an honest experimenter (ofcourse he explains in terms of what people knew in the 1920's)
the 1929 article & gravitator patent: http://www.rexresearch.com/gravitor/gravitor.htm#patent

This patent strikes me the most (attached): US3187206 (1965),
He has a Half-Wave Radiator, tapered dielectric member etc...

please tell me what you think because im seeing a fair bit of crossover.

I seriously think there is something to be gleaned from TTBrown's work.

Regards.

PS: Im thoroughly enjoying the thread, wish i was as up to speed as you lot.  8)

Asymmetrical Capacitors for Propulsion : http://ntrs.nasa.gov/archive/nasa/casi.ntrs.nasa.gov/20040171929.pdf

Conclusions: "... In spite of decades of speculation about possible new physical principles being responsible for the thrust produced by ACTs and lifters, we find no evidence to support such a conclusion. On the contrary, we find that their operation is fully explained by a very simple theory that uses only
electrostatic forces and the transfer of momentum by multiple collisions
. ".

My understanding is that for emDrive, the new tests were successfully performed in a (close to) vacuum.
Title: Re: EM Drive Developments - related to space flight applications - Thread 2
Post by: FieldEffect on 05/08/2015 09:52 am
Im seeing an overlap between this kind of work and work done by T.T.Brown...
anyone care to briefly comment on it?

(Incase your unfamiliar with T.T. Brown, he discovered Biefeld-Brown effect, but also discovered another force that he insisted was NOT biefeld-brown effect (ion wind), also he believed the dielectric played a primary role in the force, and that a reaction force existed on "all solid material bodies making up the physical environment")

He makes some very peculiar statements in his 1929 article, which lead me to believe he was an honest experimenter (ofcourse he explains in terms of what people knew in the 1920's)
the 1929 article & gravitator patent: http://www.rexresearch.com/gravitor/gravitor.htm#patent

This patent strikes me the most (attached): US3187206 (1965),
He has a Half-Wave Radiator, tapered dielectric member etc...

please tell me what you think because im seeing a fair bit of crossover.

I seriously think there is something to be gleaned from TTBrown's work.

Regards.

PS: Im thoroughly enjoying the thread, wish i was as up to speed as you lot.  8)

Asymmetrical Capacitors for Propulsion : http://ntrs.nasa.gov/archive/nasa/casi.ntrs.nasa.gov/20040171929.pdf

Conclusions: "... In spite of decades of speculation about possible new physical principles being responsible for the thrust produced by ACTs and lifters, we find no evidence to support such a conclusion. On the contrary, we find that their operation is fully explained by a very simple theory that uses only
electrostatic forces and the transfer of momentum by multiple collisions
. ".

My understanding is that for emDrive, the new tests were successfully performed in a (close to) vacuum.

I agree with you for the most part, but i also believe im seeing too many coincidences.
It was assumed by conspirators that the asymmetrical capacitor and hence a non-uniform electric field would produce anti-gravity, obviously that's false. TTBrown believed the force was from the dielectric.
 
If the mechanism described by marco frasca is correct,
isnt it possible that TTBrown failed to recognise he had actually built a dielectric resonator, and with the very high voltages used, achieved "gross intensity of the electromagnetic energy" (as Marco put it) and then mistakenly put it down to the very high voltages he was using?

wouldnt the AC source emit microwaves at the small electrode (that he advised be half-the wavelength in diameter)? TTBrown mentions things in the 1965 patent like including Lead-Oxide in the dielectric to increase the effect.

also his "Gravitator" (big heavy capacitor/condenser hung like a pendulum) he describes as recieving an impulse when switched on, and would slowly return to its original rest position, even though the potential was maintained.
Would the initial charging of the gravitator at high voltage emit a high-power radio signal that would resonate throughout the dielectric insulators? inducing the effect marco wrote about?

(Im not arguing, I wanna know what you guys think).
Title: Re: EM Drive Developments - related to space flight applications - Thread 2
Post by: Rodal on 05/08/2015 10:14 am
Quote
Rodal:
Interestingly (for this thread's discussion due to the significance that the NASA experimenters have placed on the dielectric being responsible for providing the measured thrust) is what happens in the High Density PolyEthylene (HDPE) dielectric polymer insert.  Because the dimensions of the dielectric are not negligible compared to the EM Drive's dimension, and the dielectric is not modeled as just a boundary condition.

The loss tangent of HDPE is reported to be

tan delta = 0.0004

Therefore the intrinsic impedance angle is

intrinsic impedance angle =(ArcTan[0.0004])/2

Therefore, inside the HDPE dielectric the electric and magnetic fields, instead of being out of phase by 90 degrees (as they are in the air or vacuum medium), will be out of phase by:

90 - (ArcTan[0.0004])/2 = 89.9998 degrees

This phase angle (89.9998 degrees)  will show practically no visual difference with 90 degrees at the resolution of the following image :

Mr Rodal can you advise me on the value .0004 in your work above,  is that in degrees or radians or just a figure that one simply takes the arctan of.  Im trying to use the "bc" program in linux command line to get a grip on the overall topic, and bc reports results in radians, hence the requirement to do a 180/pi conversion.

echo "scale=20; 90 - (a(0.0004)/2)*(180/(4*a(1)))" | bc -l
89.98854084470853845905

Thanks
arc

In:

tan delta = 0.0004

delta is an angle (whose dimension can be expressed in degrees or in radians, but I have not used delta, so the dimensions of delta did not matter).  It is called the loss angle (see:  https://en.wikipedia.org/wiki/Dielectric_loss   http://www.microwaves101.com/encyclopedias/transmission-line-loss ).  Both the loss angle delta and tan delta characterize a material property.

tan delta is commonly used nomenclature used for the Tangent of the angle delta ( Tangent[delta] ), as such tan delta is a dimensionless number

The "intrinsic impedance angle" on the other hand is an angle, and as such one needs to carefully decide whether one wants to express it in radians or degrees or any other unit as long as one consistently uses it

In the example I gave above, you are correct, it was missing the conversion factor, it should have read

90 - (180/Pi) (ArcTan[0.0004])/2 = 89.98854084470854

Thank you for finding this error :)  I have corrected the original

The important final numerical result (0.00009999999400006368) for the magnitude of the mean (over a period) of the Poyinting vector was not affected:

Cos[(Pi/180) (90 - (180/Pi) (ArcTan[0.0004])/2)]/2= 0.00009999999400006368

Inside the HDPE polymer dielectric the Poynting vector has this small magnitude factor over a period (or multiples thereof).



Title: Re: EM Drive Developments - related to space flight applications - Thread 2
Post by: Rodal on 05/08/2015 11:31 am
Consider the following EM Drive design:

1st component: A symmetrical resonator cavity that is optimized to store energy at high Q, and plays no role in thrust. Here, we want to store the lowest order mode that the waveguide can sustain because it has the slowest wave velocity inside the waveguide. By targeting the slowest wave velocity, we are maximizing the amount of stored electromagnetic momentum for a given amount electromagnetic energy.

p = E/v, where  v << c inside the waveguide.
E = P*t, energy is power in x time (assuming a lossless cavity)

2nd component: We need a frustum that matches the diameter of the resonant chamber "at the small end" and slowly expands to a TBD length and diameter. As the waveguide expands, the wavelength will be increasing proportional to the increasing wave velocity. We want it to expand to near it's free space value.

3rd component: We need a partial reflector, between the two, so that we can have resonance in one chamber, and thrust in the other. Like a laser.

Principle of operation:

When the energy stored in the resonant chamber is released into the frustum, it expands due to the increased wave velocity to a longer wavelength before it is reflected. It leaves the "nozzle" with momentum p1, but when it arrives at the reflector, it has momentum p2 << p1, because the wave velocity has increased "significantly".

The reflected waves then travel back into the frustum only to find that due to their increased wavelength, now they don't fit anymore, so they are attenuated. Again, absorbing the momentum into the frustum as heat and kinetic energy in the "forward" direction.

....

Excellent.  This makes to me more engineering sense than the closed cavity design pioneered by Shawyer and imitated in the US and Chinese laboratories.

(Also, let's think of a maser )

So it looks to me that if the UK/US/Chinese measurements are real (*), the design you sketched above should produce more thrust than the present designs.

___
(*) if they are real, I would like to summarize the present understanding regarding the energy paradox.  I thought you had reached a common conclusion but @frobnicat raised apparently new points that apparently still need to be addressed ?

....

To put it quantum mechanically:

N photons are injected at momentum p1 = N*h/lambda_1

N photons are reflected at the far end with momentum p2 = N*h/lambda_2

The wavelengths are not equal. Momentum is conserved because the system moves forward. The red-shifted photons are mimicking a gravitational field. They are trying to escape a gravity well, only to be reflected back in and fall back through the event horizon as wave velocity goes to zero. (Sorry I love that analogy!)

Todd Desiato

Where would the Q factor enter into consideration?

(Without the Q into the thrust force equation, we still have an inefficient photon rocket)
Title: Re: EM Drive Developments - related to space flight applications - Thread 2
Post by: Rodal on 05/08/2015 11:40 am
Such a shame it's so difficult to simply put it in space and check. It would have saved Woodward 20 years and I don't know how many years on this already.

I speak as someone who watched Apollo 11 on the moon.
Do you think is difficult to put in space?

(if so why?, because of engineering difficulty, because of expense, or because of the will of any organization to take a risk on a very controversial item that may not work)
Title: Re: EM Drive Developments - related to space flight applications - Thread 2
Post by: deltaMass on 05/08/2015 11:49 am
Clearly it's not easy.  EagleWorks would have their rig up there in a heartbeat if it was easy.
Title: Re: EM Drive Developments - related to space flight applications - Thread 2
Post by: Notsosureofit on 05/08/2015 12:29 pm
Consider the following EM Drive design:

1st component: A symmetrical resonator cavity that is optimized to store energy at high Q, and plays no role in thrust. Here, we want to store the lowest order mode that the waveguide can sustain because it has the slowest wave velocity inside the waveguide. By targeting the slowest wave velocity, we are maximizing the amount of stored electromagnetic momentum for a given amount electromagnetic energy.

p = E/v, where  v << c inside the waveguide.
E = P*t, energy is power in x time (assuming a lossless cavity)

2nd component: We need a frustum that matches the diameter of the resonant chamber "at the small end" and slowly expands to a TBD length and diameter. As the waveguide expands, the wavelength will be increasing proportional to the increasing wave velocity. We want it to expand to near it's free space value.

3rd component: We need a partial reflector, between the two, so that we can have resonance in one chamber, and thrust in the other. Like a laser.

Principle of operation:

When the energy stored in the resonant chamber is released into the frustum, it expands due to the increased wave velocity to a longer wavelength before it is reflected. It leaves the "nozzle" with momentum p1, but when it arrives at the reflector, it has momentum p2 << p1, because the wave velocity has increased "significantly".

The reflected waves then travel back into the frustum only to find that due to their increased wavelength, now they don't fit anymore, so they are attenuated. Again, absorbing the momentum into the frustum as heat and kinetic energy in the "forward" direction.

To put it quantum mechanically:

N photons are injected at momentum p1 = N*h/lambda_1

N photons are reflected at the far end with momentum p2 = N*h/lambda_2

The wavelengths are not equal. Momentum is conserved because the system moves forward. The red-shifted photons are mimicking a gravitational field. They are trying to escape a gravity well, only to be reflected back in and fall back through the event horizon as wave velocity goes to zero. (Sorry I love that analogy!)

Todd Desiato

Excellent.  This makes to me much more engineering/scientific sense than the closed cavity design pioneered by Shawyer and imitated in the US and Chinese laboratories.

So it looks to me that either the UK/US/Chinese measurements are an artifact, or if they are real (*), the design you sketched above should produce more thrust than the present designs.

___
(*) if they are real, I would like to summarize the present understanding regarding the energy paradox.  I thought you had reached a common conclusion but @frobnicat raised apparently new points that apparently still need to be addressed ?


Mmmm.....   Maybe I'm missing something here?  Do I need a diagram of these coupled cavities?

The statement " The reflected waves then travel back into the frustum only to find that due to their increased wavelength, now they don't fit anymore, so they are attenuated. Again, absorbing the momentum into the frustum as heat and kinetic energy in the "forward" direction. "  makes no sense.  As they travel back into the frustrum the velocity decreases again.   Even if they were "attenuated" w/o momentum transfer you would still have, at best, a self blown photon sail ????

Todd:  If you take my GR calculation and redo it in "index of refraction" terms you should get the same force result.

Ultimately, what needs to be shown here is that the "covariant force vector equal to zero" on a photon in the accelerated frame is the same vector in the cavity rest frame (ie. can be transformed to) such that the force on a photon in the rest frame is a result of the (velocity) dispersion due to the shape of the cavity boundary conditions.  That transformation would show that the effect is to be expected under General Relativity.
Title: Re: EM Drive Developments - related to space flight applications - Thread 2
Post by: Rodal on 05/08/2015 12:36 pm
Yes, I added:

Where would the Q factor enter into consideration?

(Without the Q into the thrust force equation, we still have an inefficient photon rocket)

[and when one enters the Q factor for a closed cavity, I yet have to embrace an explanation for a closed cavity where I can fully see conservation of momentum and conservation of energy satisfied.  For example, Shawyer only takes into account the forces perpendicular to the bases and neglects the counterbalancing forces from the conical surface which result in zero net thrust force]

Ditto for the GR calculation: they are one-dimensional longitudinal approximations, where the index of refraction changes in the longitudinal direction: but the truncated cone is 3-D and has counterbalancing forces due to the forces on the conical surface which cancel the thrust

If I'm wrong, I would like someone to show what happens with the forces on the conical surface (which are at an angle to the longitudinal direction)

I need a free-body diagram showing the force vectors on all the copper surfaces for the 3-D problem force summation  :)

Title: Re: EM Drive Developments - related to space flight applications - Thread 2
Post by: Notsosureofit on 05/08/2015 01:13 pm
Yes, I added:

Where would the Q factor enter into consideration?

(Without the Q into the thrust force equation, we still have an inefficient photon rocket)

[and when one enters the Q factor for a closed cavity, I yet have to embrace an explanation for a closed cavity where I can fully see conservation of momentum and conservation of energy satisfied.  For example, Shawyer only takes into account the forces perpendicular to the bases and neglects the counterbalancing forces from the conical surface which result in zero net thrust force]

Ditto for the GR calculation: they are one-dimensional longitudinal approximations, where the index of refraction changes in the longitudinal direction: but the truncated cone is 3-D and has counterbalancing forces due to the forces on the conical surface which cancel the thrust

If I'm wrong, I would like someone to show what happens with the forces on the conical surface (which are at an angle to the longitudinal direction)

I need a free-body diagram showing the force vectors on all the copper surfaces for the 3-D problem force summation  :)

The Q only enters in the case of a gravitational interaction through its role in calculating the number of photons (Total mass/energy) in the cavity.  If it was a Newtonian momentum interaction only the Power enters.  (I get zero when I try Newtonian for the reasons you have mentioned)

Note: I should probably use "Classical" instead of "Newtonian" but I think you would get the idea that we are looking at the 4-volume as an invariant rather than an integration over the surface of the boundary conditions of classical momentum exchange. That can apply as the summation of the (false gravitational?, ie. frame-dependent) forces on the individual photons to give the resultant force on the cavity.
Title: Re: EM Drive Developments - related to space flight applications - Thread 2
Post by: Rodal on 05/08/2015 01:19 pm
Yes, I added:

Where would the Q factor enter into consideration?

(Without the Q into the thrust force equation, we still have an inefficient photon rocket)

[and when one enters the Q factor for a closed cavity, I yet have to embrace an explanation for a closed cavity where I can fully see conservation of momentum and conservation of energy satisfied.  For example, Shawyer only takes into account the forces perpendicular to the bases and neglects the counterbalancing forces from the conical surface which result in zero net thrust force]

Ditto for the GR calculation: they are one-dimensional longitudinal approximations, where the index of refraction changes in the longitudinal direction: but the truncated cone is 3-D and has counterbalancing forces due to the forces on the conical surface which cancel the thrust

If I'm wrong, I would like someone to show what happens with the forces on the conical surface (which are at an angle to the longitudinal direction)

I need a free-body diagram showing the force vectors on all the copper surfaces for the 3-D problem force summation  :)

The Q only enters in the case of a gravitational interaction through its role in calculating the number of photons (Total mass/energy) in the cavity.  If it was a Newtonian momentum interaction only the Power enters.  (I get zero when I try Newtonian)
Another issue:  my understanding of your derivation is that it follows from the expression for the cylindrical cavity (cone angle = zero): hence it is a constant cross-section longitudinal approximation to the cone, where the index of refraction changes in the longitudinal direction: but the truncated cone is 3-D and has counterbalancing forces due to the forces on the conical surface which cancel the thrust

It seems that the forces on the conical surfaces of the cone are being ignored, and if one takes into account the forces on the cone surface, the "thrust" would be nullified: no thrust. 

Hence the only possible thrust is with a dielectric (either in the cylinder or in the truncated cone).

Is that correct ?
Title: Re: EM Drive Developments - related to space flight applications - Thread 2
Post by: maciejzi on 05/08/2015 01:20 pm
As I understand there are 2 possible setups: closed (tested) and open (one amateur video on youtube with kitchen microwave magnetron).
Regardless of the closed/open structure, shouldn't the cavity have form a reversed parabolic shape?
With the microwave injecting module on its side or in the middle, the reversed parabolic shape will send more microwaves in the required direction.

(http://s7.postimg.org/d9kdplu63/resonatingspeaker.jpg)

In my opinion the engine resembles a very stiff speaker calculated for resonance frequency of 2,45MHz. The stiffer the material the better it resonates in this frequency. It's not only the Q that matters. That may be the reason the Cannae engine has lower efficiency - it is made of thinner metal that is less stiff.


I propose 4 options, as in the picture (A,B,C,D).
Title: Re: EM Drive Developments - related to space flight applications - Thread 2
Post by: deuteragenie on 05/08/2015 01:35 pm
Like this ?

http://upload.wikimedia.org/wikipedia/commons/thumb/9/93/Horn_loudspeaker_animation.gif/250px-Horn_loudspeaker_animation.gif
Title: Re: EM Drive Developments - related to space flight applications - Thread 2
Post by: maciejzi on 05/08/2015 01:37 pm

Like this ?

http://upload.wikimedia.org/wikipedia/commons/thumb/9/93/Horn_loudspeaker_animation.gif/250px-Horn_loudspeaker_animation.gif

Exactly, but calculated for the MW frequency, so it has to be very stiff. Actually, it has to have the exact stiffness of this frequency to send the maximum waves in the required direction of thrust.


The stiffness of the membrane (the drive cavity) has to be selected according to the medium it operates in. Different for air, different for cosmic vacuum.
Title: Re: EM Drive Developments - related to space flight applications - Thread 2
Post by: Rodal on 05/08/2015 01:40 pm
As I understand there are 2 possible setups: closed (tested) and open (one amateur video on youtube with kitchen microwave magnetron).
Regardless of the closed/open structure, shouldn't the cavity have form a reversed parabolic shape?
With the microwave injecting module on its side or in the middle, the reversed parabolic shape will send more microwaves in the required direction.

(http://s7.postimg.org/d9kdplu63/resonatingspeaker.jpg)

In my opinion the engine resembles a very stiff speaker calculated for resonance frequency of 2,45MHz. The stiffer the material the better it resonates in this frequency. It's not only the Q that matters. That may be the reason the Cannae engine has lower efficiency - it is made of thinner metal that is less stiff.


I propose 4 options, as in the picture (A,B,C,D).
The problem is that I can hear music from an open speaker (a "waveguide") but I cannot hear anything out of a speaker that is a closed cavity, such that all internal surfaces are perfectly reflective (the closed cavity of the EM Drive).

And if you open the big base of the EM Drive cavity, so that it becomes a speaker, then it is a very inefficient photon rocket, and therefore the claims of Shawyer don't make sense because they are thousands of time greater thrust per input power than a photon rocket  :'(
Title: Re: EM Drive Developments - related to space flight applications - Thread 2
Post by: Notsosureofit on 05/08/2015 01:45 pm
Yes, I added:

Where would the Q factor enter into consideration?

(Without the Q into the thrust force equation, we still have an inefficient photon rocket)

[and when one enters the Q factor for a closed cavity, I yet have to embrace an explanation for a closed cavity where I can fully see conservation of momentum and conservation of energy satisfied.  For example, Shawyer only takes into account the forces perpendicular to the bases and neglects the counterbalancing forces from the conical surface which result in zero net thrust force]

Ditto for the GR calculation: they are one-dimensional longitudinal approximations, where the index of refraction changes in the longitudinal direction: but the truncated cone is 3-D and has counterbalancing forces due to the forces on the conical surface which cancel the thrust

If I'm wrong, I would like someone to show what happens with the forces on the conical surface (which are at an angle to the longitudinal direction)

I need a free-body diagram showing the force vectors on all the copper surfaces for the 3-D problem force summation  :)

The Q only enters in the case of a gravitational interaction through its role in calculating the number of photons (Total mass/energy) in the cavity.  If it was a Newtonian momentum interaction only the Power enters.  (I get zero when I try Newtonian)
Another issue:  my understanding of your derivation is that it follows from the expression for the cylindrical cavity (cone angle = zero): hence it is a constant cross-section longitudinal approximation to the cone, where the index of refraction changes in the longitudinal direction: but the truncated cone is 3-D and has counterbalancing forces due to the forces on the conical surface which cancel the thrust

It seems that the forces on the conical surfaces of the cone are being ignored, and if one takes into account the forces on the cone surface, the "thrust" would be nullified: no thrust. 

Hence the only possible thrust is with a dielectric (either in the cylinder or in the truncated cone).

Is that correct ?

You are too fast for my old fingers...

Ultimately, what needs to be shown here is that the "covariant force vector equal to zero" on a photon in the accelerated frame is the same vector in the cavity rest frame (ie. can be transformed to) such that the force on a photon in the rest frame is a result of the (velocity) dispersion due to the shape of the cavity boundary conditions.  That transformation would show that the effect is to be expected under General Relativity.

The Q only enters in the case of a gravitational interaction through its role in calculating the number of photons (Total mass/energy) in the cavity.  If it was a Newtonian momentum interaction only the Power enters.  (I get zero when I try Newtonian for the reasons you have mentioned)

Note: I should probably use "Classical" instead of "Newtonian" but I think you would get the idea that we are looking at the 4-volume as an invariant rather than an integration over the surface of the boundary conditions of classical momentum exchange. That can apply as the summation of the (false gravitational?, ie. frame-dependent) forces on the individual photons to give the resultant force on the cavity.
Title: Re: EM Drive Developments - related to space flight applications - Thread 2
Post by: Rodal on 05/08/2015 01:53 pm
....

You are too fast for my old fingers...

Ultimately, what needs to be shown here is that the "covariant force vector equal to zero" on a photon in the accelerated frame is the same vector in the cavity rest frame (ie. can be transformed to) such that the force on a photon in the rest frame is a result of the (velocity) dispersion due to the shape of the cavity boundary conditions.  That transformation would show that the effect is to be expected under General Relativity.

The Q only enters in the case of a gravitational interaction through its role in calculating the number of photons (Total mass/energy) in the cavity.  If it was a Newtonian momentum interaction only the Power enters.  (I get zero when I try Newtonian for the reasons you have mentioned)

Note: I should probably use "Classical" instead of "Newtonian" but I think you would get the idea that we are looking at the 4-volume as an invariant rather than an integration over the surface of the boundary conditions of classical momentum exchange. That can apply as the summation of the (false gravitational?, ie. frame-dependent) forces on the individual photons to give the resultant force on the cavity.

I very much appreciate the explanation, unfortunately, I may not be able to understand this until I see the explicit mathematical formula for the 4-volume invariant, particularly the derivation of that invariant  :)

Unfortunately, I cannot do it intuitively as you can.

Unfortunately I only have an intuition for linear differential equation Newtonian problems, very little intuition for nonlinear Newtonian problems and no intuition for nonlinear general relativity problems :(
Title: Re: EM Drive Developments - related to space flight applications - Thread 2
Post by: Notsosureofit on 05/08/2015 01:56 pm
....

You are too fast for my old fingers...

Ultimately, what needs to be shown here is that the "covariant force vector equal to zero" on a photon in the accelerated frame is the same vector in the cavity rest frame (ie. can be transformed to) such that the force on a photon in the rest frame is a result of the (velocity) dispersion due to the shape of the cavity boundary conditions.  That transformation would show that the effect is to be expected under General Relativity.

The Q only enters in the case of a gravitational interaction through its role in calculating the number of photons (Total mass/energy) in the cavity.  If it was a Newtonian momentum interaction only the Power enters.  (I get zero when I try Newtonian for the reasons you have mentioned)

Note: I should probably use "Classical" instead of "Newtonian" but I think you would get the idea that we are looking at the 4-volume as an invariant rather than an integration over the surface of the boundary conditions of classical momentum exchange. That can apply as the summation of the (false gravitational?, ie. frame-dependent) forces on the individual photons to give the resultant force on the cavity.

I very much appreciate the explanation, unfortunately, I may not be able to understand this until I see the explicit mathematical formula for the 4-volume invariant, particularly the derivation of that invariant  :)

Unfortunately, I cannot do it intuitively as you can.

That would, of course, constitute a proof.
Title: Re: EM Drive Developments - related to space flight applications - Thread 2
Post by: maciejzi on 05/08/2015 01:57 pm

The problem is that I can hear music from an open speaker (a "waveguide") but I cannot hear anything out of a speaker that is a closed cavity, such that all internal surfaces are perfectly reflective (the closed cavity of the EM Drive).

And if you open the big base of the EM Drive cavity, so that it becomes a speaker, then it is a very inefficient photon rocket, and therefore the claims of Shawyer don't make sense because they are thousands of time greater thrust per input power than a photon rocket  :'(


All right. Than maybe the magnetic field of the microwaves resonating inside the cavity "pushes" against the magnetic field of Earth. Anyway, the reversed parabolic (speaker-like) shape may send more waves to the wide end.

Whatever the reason it moves, all I am saying is that it has to be tested. Not only for different Q, but also different shapes, different MW injection positions and different stiffness of the 'emiting' structure.

Title: Re: EM Drive Developments - related to space flight applications - Thread 2
Post by: Rodal on 05/08/2015 02:08 pm
...Than maybe the magnetic field of the microwaves resonating inside the cavity "pushes" against the magnetic field of Earth. ...
If you believe Shawyer's experiments to not be artifacts, the effect of the Earth's magnetic field was nullified by the fact that Shawyer claims to have  tested the EM Drive UP, DOWN, and HORIZONTALLY oriented, measuring the same thrust/PowerInput  :(
Title: Re: EM Drive Developments - related to space flight applications - Thread 2
Post by: WarpTech on 05/08/2015 02:16 pm
....
And if you open the big base of the EM Drive cavity, so that it becomes a speaker, then it is a very inefficient photon rocket, and therefore the claims of Shawyer don't make sense because they are thousands of time greater thrust per input power than a photon rocket  :'(

I need to study @notsosureofit's GR calculation, but I think the point that is being missed here is, the resonant cavity acts as an EM Momentum amplifier. The Q factor comes in not "Just" as the number of photons stored, but also;  In GR as in PV, as the energy density inside the cavity increases, the wave velocity decreases. As velocity decreases, momentum increases. That "effective mass" of the photons in the cavity kicks in stronger.

p ~ E/v  and in the cavity v << c

This makes it a not-so "inefficient" photon rocket!

In the EM Drive design I proposed, we want the Q in the resonant cavity to be very high, but we do not want a high Q in the frustum. We want the backward-moving waves to expand, and the reflected waves moving forward to be attenuated. So we want a low Q, high attenuation in the frustum.

Todd
Title: Re: EM Drive Developments - related to space flight applications - Thread 2
Post by: Rodal on 05/08/2015 02:28 pm
....
And if you open the big base of the EM Drive cavity, so that it becomes a speaker, then it is a very inefficient photon rocket, and therefore the claims of Shawyer don't make sense because they are thousands of time greater thrust per input power than a photon rocket  :'(

I need to study @notsosureofit's GR calculation, but I think the point that is being missed here is, the resonant cavity acts as an EM Momentum amplifier. The Q factor comes in not "Just" as the number of photons stored, but also;  In GR as in PV, as the energy density inside the cavity increases, the wave velocity decreases. As velocity decreases, momentum increases. That "effective mass" of the photons in the cavity kicks in stronger.

p ~ E/v  and in the cavity v << c

This makes it a not-so "inefficient" photon rocket!

In the EM Drive design I proposed, we want the Q in the resonant cavity to be very high, but we do not want a high Q in the frustum. We want the backward-moving waves to expand, and the reflected waves moving forward to be attenuated. So we want a low Q, high attenuation in the frustum.

Todd

I see the need for:

*low Q
*high attenuation

(This maybe confirmed by the Chinese: Prof. Yang measured the highest thrust, and the highest thrust/PowerInput, using a Q~1500 (*) much lower than the other researchers)

in the truncated cone, but this is the complete opposite of what is claimed by Shawyer.

Actually Shawyer is proceeding in the completely opposite direction:

1) a superconducting truncated cone with Q>10^6 and

2) Shawyer threw away the dielectric, hence NO attenuation from the dielectric.

3) Also Shawyer increased the cone angle from 10 degrees to 35 degrees, which decreased the geometrical attenuation by a large factor.

How do you reconcile Shawyer going in the completely opposite direction ????

****On the other hand, we have not seen Shawyer flying around with his superconducting  EM Drive, and he hasn't reported any results in 6 months.  Companies are eager to publicize major positive breakthroughs as soon as possible, particularly when owning Intellectual Property, as patents suffer from exponentially decaying value with time, like long options, so you want to exercise their value as soon as possible ****

____________

(*) Readers just looking at the translation from the Chinese Paper: notice that the Q in the tables are computed by a very unorthodox method.  The Q of the Chinese experiments, calculated in the same way as in the West is very low
Title: Re: EM Drive Developments - related to space flight applications - Thread 2
Post by: deuteragenie on 05/08/2015 02:32 pm

The problem is that I can hear music from an open speaker (a "waveguide") but I cannot hear anything out of a speaker that is a closed cavity, such that all internal surfaces are perfectly reflective (the closed cavity of the EM Drive).

And if you open the big base of the EM Drive cavity, so that it becomes a speaker, then it is a very inefficient photon rocket, and therefore the claims of Shawyer don't make sense because they are thousands of time greater thrust per input power than a photon rocket  :'(


All right. Than maybe the magnetic field of the microwaves resonating inside the cavity "pushes" against the magnetic field of Earth. Anyway, the reversed parabolic (speaker-like) shape may send more waves to the wide end.

Whatever the reason it moves, all I am saying is that it has to be tested. Not only for different Q, but also different shapes, different MW injection positions and different stiffness of the 'emiting' structure.

Chapter 9 of Acoustics: Sound Fields and Transducers By Leo L. Beranek, Tim J. Mellow is dedicated to properties of Horn loudspeakers and studies different shapes (hyperbolic, conical, parabolic... finite / infinite) and provides many nice formulas that could stimulate your thinking. 
Title: Re: EM Drive Developments - related to space flight applications - Thread 2
Post by: Rodal on 05/08/2015 02:53 pm

The problem is that I can hear music from an open speaker (a "waveguide") but I cannot hear anything out of a speaker that is a closed cavity, such that all internal surfaces are perfectly reflective (the closed cavity of the EM Drive).

And if you open the big base of the EM Drive cavity, so that it becomes a speaker, then it is a very inefficient photon rocket, and therefore the claims of Shawyer don't make sense because they are thousands of time greater thrust per input power than a photon rocket  :'(


All right. Than maybe the magnetic field of the microwaves resonating inside the cavity "pushes" against the magnetic field of Earth. Anyway, the reversed parabolic (speaker-like) shape may send more waves to the wide end.

Whatever the reason it moves, all I am saying is that it has to be tested. Not only for different Q, but also different shapes, different MW injection positions and different stiffness of the 'emiting' structure.

Chapter 9 of Acoustics: Sound Fields and Transducers By Leo L. Beranek, Tim J. Mellow is dedicated to properties of Horn loudspeakers and studies different shapes (hyperbolic, conical, parabolic... finite / infinite) and provides many nice formulas that could stimulate your thinking.
May also be useful to look at how microwaves actually propagate in a cone, and different other shapes, as in the literature of radar, according to Maxwell's equations instead of acoustic waves.

See this for paraboloid cavities for example : http://forum.nasaspaceflight.com/index.php?topic=36313.msg1334659#msg1334659
Title: Re: EM Drive Developments - related to space flight applications - Thread 2
Post by: Einstein79 on 05/08/2015 03:42 pm
The reason for the confusion over the violation of classical physics is because this system has nothing to do with classical physics. Moreover, the “thrust” that is being calculated is not thrust at all but space moving the drive from one position to another which can merely be related to thrust but is not, per se, thrust. The controlling factor here is, of course, the resonant frequency. If you match the resonant frequency that space uses to “hold” the object you will develop a “cavity” that the “object will move towards”. The reason why the device cannot be “pushed off of” for conservation of momentum to hold true is because space is already pushing on it satisfying the law.

A couple of postulates to keep in mind that will help with these experiments are:
1. Space creates light.
2. Space itself is a resonating chamber.
Interesting! Would you then be prepared to write down the equations of motion so that we can play with them?

The equations of motion do not exist from an inertial reference frame. We must assume that the object is not moving and that space is moving around and through the object. I am trying to develop the Hamiltonian for space but having difficulty because we have always assumed space to be a virtual plasma and it is not virtual at all, but real.
Title: Re: EM Drive Developments - related to space flight applications - Thread 2
Post by: maciejzi on 05/08/2015 03:44 pm
....
And if you open the big base of the EM Drive cavity, so that it becomes a speaker, then it is a very inefficient photon rocket, and therefore the claims of Shawyer don't make sense because they are thousands of time greater thrust per input power than a photon rocket  :'(

I need to study @notsosureofit's GR calculation, but I think the point that is being missed here is, the resonant cavity acts as an EM Momentum amplifier. The Q factor comes in not "Just" as the number of photons stored, but also;  In GR as in PV, as the energy density inside the cavity increases, the wave velocity decreases. As velocity decreases, momentum increases. That "effective mass" of the photons in the cavity kicks in stronger.

p ~ E/v  and in the cavity v << c

This makes it a not-so "inefficient" photon rocket!

What if the photon rocket utilized the same construction with a cavity that would make the light move slower?

http://news.harvard.edu/gazette/1999/02.18/light.html

As it is possible to slow the light, then the lower its speed the bigger its thrust.
That's one thing. The other is that the thrust is multiplied by the number of bounccs.
That applies to MW waves in the resonating cavity. Hence - the bigger the number of bounces, the bigger the thrust.
You can actually amplify the photon rocket thrust as well.
See here:
(http://s22.postimg.org/xv4nrzxch/Photon_Thrust_Amplification.jpg)
Source: Wiki, http://upload.wikimedia.org/wikipedia/commons/8/8e/Photon-Thrust-Amplification.jpg

Bend this light path with optical fiber and you receive a powerful thruster.
Like this:

(http://s3.postimg.org/8ajq5b2g3/bentfiber.jpg)


A very simple idea... nobody ever checked it yet, I suppose. I think this setup may be worth checking.
Please consider the fact that it is not c in the nominator, but the actual speed of light in the optical fiber. The slower the speed of the light in this apparatus, the greater the thrust!
The EmDrive works based on similar principle.
Title: Re: EM Drive Developments - related to space flight applications - Thread 2
Post by: Einstein79 on 05/08/2015 03:51 pm
The reason for the confusion over the violation of classical physics is because this system has nothing to do with classical physics. Moreover, the “thrust” that is being calculated is not thrust at all but space moving the drive from one position to another which can merely be related to thrust but is not, per se, thrust. The controlling factor here is, of course, the resonant frequency. If you match the resonant frequency that space uses to “hold” the object you will develop a “cavity” that the “object will move towards”. The reason why the device cannot be “pushed off of” for conservation of momentum to hold true is because space is already pushing on it satisfying the law.

A couple of postulates to keep in mind that will help with these experiments are:
1. Space creates light.
2. Space itself is a resonating chamber.
Interesting! Would you then be prepared to write down the equations of motion so that we can play with them?

The Mexican hat potential is a good place to start, I think, but not sure yet. It seems reasonable because it might coalesce with symmetry breaking. 
Title: Re: EM Drive Developments - related to space flight applications - Thread 2
Post by: LasJayhawk on 05/08/2015 04:06 pm
The reason for the confusion over the violation of classical physics is because this system has nothing to do with classical physics. Moreover, the “thrust” that is being calculated is not thrust at all but space moving the drive from one position to another which can merely be related to thrust but is not, per se, thrust. The controlling factor here is, of course, the resonant frequency. If you match the resonant frequency that space uses to “hold” the object you will develop a “cavity” that the “object will move towards”. The reason why the device cannot be “pushed off of” for conservation of momentum to hold true is because space is already pushing on it satisfying the law.

A couple of postulates to keep in mind that will help with these experiments are:
1. Space creates light.
2. Space itself is a resonating chamber.
Interesting! Would you then be prepared to write down the equations of motion so that we can play with them?

The Mexican hat potential is a good place to start, I think, but not sure yet. It seems reasonable because it might coalesce with symmetry breaking.

You and Mulletron seem to be going in the same direction, you might want to bounce ideas off each other...
Title: Re: EM Drive Developments - related to space flight applications - Thread 2
Post by: WarpTech on 05/08/2015 04:21 pm
....
And if you open the big base of the EM Drive cavity, so that it becomes a speaker, then it is a very inefficient photon rocket, and therefore the claims of Shawyer don't make sense because they are thousands of time greater thrust per input power than a photon rocket  :'(

I need to study @notsosureofit's GR calculation, but I think the point that is being missed here is, the resonant cavity acts as an EM Momentum amplifier. The Q factor comes in not "Just" as the number of photons stored, but also;  In GR as in PV, as the energy density inside the cavity increases, the wave velocity decreases. As velocity decreases, momentum increases. That "effective mass" of the photons in the cavity kicks in stronger.

p ~ E/v  and in the cavity v << c

This makes it a not-so "inefficient" photon rocket!

In the EM Drive design I proposed, we want the Q in the resonant cavity to be very high, but we do not want a high Q in the frustum. We want the backward-moving waves to expand, and the reflected waves moving forward to be attenuated. So we want a low Q, high attenuation in the frustum.

Todd

I see the need for:

*low Q
*high attenuation

(This maybe confirmed by the Chinese: Prof. Yang measured the highest thrust, and the highest thrust/PowerInput, using a Q~1500 (*) much lower than the other researchers)

in the truncated cone, but this is the complete opposite of what is claimed by Shawyer.

Actually Shawyer is proceeding in the completely opposite direction:

1) a superconducting truncated cone with Q>10^6 and

2) Shawyer threw away the dielectric, hence NO attenuation from the dielectric.

3) Also Shawyer increased the cone angle from 10 degrees to 35 degrees, which decreased the geometrical attenuation by a large factor.

How do you reconcile Shawyer going in the completely opposite direction ????

****On the other hand, we have not seen Shawyer flying around with his superconducting  EM Drive, and he hasn't reported any results in 6 months.  Companies are eager to publicize major positive breakthroughs as soon as possible, particularly when owning Intellectual Property, as patents suffer from exponentially decaying value with time, like long options, so you want to exercise their value as soon as possible ****

____________

(*) Readers just looking at the translation from the Chinese Paper: notice that the Q in the tables are computed by a very unorthodox method.  The Q of the Chinese experiments, calculated in the same way as in the West is very low

I'm still studying "everything" from Shawyer, but so far I am not sure he really understands the principle of operation. His force calculation is insightful but it does not explain the "extra" momentum.

FYI: Marco's 1st draft paper went out on an email today from Paul M. and was distributed to about 35 individuals, including myself to my surprise. A swath of replies went back and forth, including a copy and paste of one of our posts here, between Dr. Rodal and myself. The comments were not constructive, as both Jack S. and Eric D. discarded the notion of gravity in the frustum, because GR would require overcoming the G/c^4 factor.

What these Dr's are missing is that we are not effecting the full spectrum from long wavelength RF to high energy quarks with what is happening in the frustum. It is only mimicking what gravity does in a very narrow bandwidth of the EM spectrum. The factor G/c^4 is only applicable for gravity that affects the FULL bandwidth of all energy and particles. G/c^4 comes from the integral over all modes in the field. We are not affecting all modes, just a few in a relatively low energy regime.

That's IMO of course.

Todd
Title: Re: EM Drive Developments - related to space flight applications - Thread 2
Post by: Rodal on 05/08/2015 05:00 pm
....

FYI: Marco's 1st draft paper went out on an email today from Paul M. and was distributed to about 35 individuals, including myself to my surprise. A swath of replies went back and forth, including a copy and paste of one of our posts here, between Dr. Rodal and myself. The comments were not constructive, as both Jack S. and Eric D. discarded the notion of gravity in the frustum, because GR would require overcoming the G/c^4 factor.

What these Dr's are missing is that we are not effecting the full spectrum from long wavelength RF to high energy quarks with what is happening in the frustum. It is only mimicking what gravity does in a very narrow bandwidth of the EM spectrum. The factor G/c^4 is only applicable for gravity that affects the FULL bandwidth of all energy and particles. G/c^4 comes from the integral over all modes in the field. We are not affecting all modes, just a few in a relatively low energy regime.

That's IMO of course.

Todd

I don't know who Jack S. and Eric D. are. If the letter is in reference to forum posts, I'm glad I had posted this some time ago:

http://forum.nasaspaceflight.com/index.php?topic=36313.msg1371241#msg1371241

Quote from: Rodal
this is a reference to NASA's experiments with an interferometer, where the laser beam goes through small portholes on a pillbox shaped cylindrical  EM Drive cavity.   

It is my understanding that NASA has not measured any thrust forces from this experiment involving a pillbox shaped EM Drive cavity.

NASA expected no significant thrust either, because the pillbox-shaped cavity is entirely symmetric in the longitudinal direction (parallel to the laser beam) and because the cavity did not contain any polymer dielectric insert in the experiments (as so far performed).

So if Jack S. and Eric D.  (whoever they are) are discussing "the notion of gravity in the frustum" I already pointed out that the Interferometer tests were not conducted with a frustum but instead with a pillbox shaped cylindrical geometry. As to the size of the spacetime distortion everybody is in agreement that one expects it to be extremely small.  That's the relevance of Marco Frasca's papers: and why Frasca's papers are so interesting, regarding the appropriate coupling.

QUESTION: In any case what are these people (Jack S. and Eric D. ) so excited about that they are writing about it?  What's the harm with Dr. White trying to find out?

As Frasca wrote in his webpage:

Quote
I would like to remember that White is not using exotic matter at all. Rather, he is working with strong RF fields to try to develop a warp bubble. This was stated here even if implicitly. Finally, an EmDrive device has been properly described here. Using strong external fields to modify locally a space-time has been described here.

If this will be confirmed in the next few months, it will represent a major breakthrough in experimental general relativity since Eddington  confirmed the bending of light near the sun. Applications would follow if this idea will appear scalable but it will be a shocking result anyway. We look forward to hear from White very soon.

Marco Frasca (2005). Strong coupling expansion for general relativity Int.J.Mod.Phys.D15:1373-1386,2006 arXiv: hep-th/0508246v3
Title: Re: EM Drive Developments - related to space flight applications - Thread 2
Post by: WarpTech on 05/08/2015 05:09 pm

What if the photon rocket utilized the same construction with a cavity that would make the light move slower?

http://news.harvard.edu/gazette/1999/02.18/light.html

As it is possible to slow the light, then the lower its speed the bigger its thrust.
That's one thing. The other is that the thrust is multiplied by the number of bounccs.
That applies to MW waves in the resonating cavity. Hence - the bigger the number of bounces, the bigger the thrust.
You can actually amplify the photon rocket thrust as well.

I think a longer frustum will work better than multiple bounces, because the frustum should not be used to resonate, it should be used to attenuate and minimize heating. You can't derive more momentum from multiple bounces, than can be acquired by 100% attenuation of the wave after 1 bounce. The key is to minimize it's momentum at the first bounce by maximizing the wave's velocity at the far end.

Todd
Title: Re: EM Drive Developments - related to space flight applications - Thread 2
Post by: frobnicat on 05/08/2015 05:27 pm
...
In my opinion the engine resembles a very stiff speaker calculated for resonance frequency of 2,45MHz. The stiffer the material the better it resonates in this frequency. It's not only the Q that matters. That may be the reason the Cannae engine has lower efficiency - it is made of thinner metal that is less stiff.
...


The problem is that I can hear music from an open speaker (a "waveguide") but I cannot hear anything out of a speaker that is a closed cavity, such that all internal surfaces are perfectly reflective (the closed cavity of the EM Drive).

And if you open the big base of the EM Drive cavity, so that it becomes a speaker, then it is a very inefficient photon rocket, and therefore the claims of Shawyer don't make sense because they are thousands of time greater thrust per input power than a photon rocket  :'(


All right. Than maybe the magnetic field of the microwaves resonating inside the cavity "pushes" against the magnetic field of Earth. Anyway, the reversed parabolic (speaker-like) shape may send more waves to the wide end.

Whatever the reason it moves, all I am saying is that it has to be tested. Not only for different Q, but also different shapes, different MW injection positions and different stiffness of the 'emiting' structure.

Enough stiffness is important for the stability of cavity shape against deformations that might occur in a fraction of second, but there is no way a macroscopic "bell" can have acoustic modes (of significant amplitude) at 1GHz and above ! The time constants (between microwaves and possible acoustic modes) are many orders of magnitude apart.
Title: Re: EM Drive Developments - related to space flight applications - Thread 2
Post by: StrongGR on 05/08/2015 05:43 pm
....

FYI: Marco's 1st draft paper went out on an email today from Paul M. and was distributed to about 35 individuals, including myself to my surprise. A swath of replies went back and forth, including a copy and paste of one of our posts here, between Dr. Rodal and myself. The comments were not constructive, as both Jack S. and Eric D. discarded the notion of gravity in the frustum, because GR would require overcoming the G/c^4 factor.

What these Dr's are missing is that we are not effecting the full spectrum from long wavelength RF to high energy quarks with what is happening in the frustum. It is only mimicking what gravity does in a very narrow bandwidth of the EM spectrum. The factor G/c^4 is only applicable for gravity that affects the FULL bandwidth of all energy and particles. G/c^4 comes from the integral over all modes in the field. We are not affecting all modes, just a few in a relatively low energy regime.

That's IMO of course.

Todd

I don't know who Jack S. and Eric D. are. If the letter is in reference to forum posts, I'm glad I had posted this some time ago:

http://forum.nasaspaceflight.com/index.php?topic=36313.msg1371241#msg1371241

Quote from: Rodal
this is a reference to NASA's experiments with an interferometer, where the laser beam goes through small portholes on a pillbox shaped cylindrical  EM Drive cavity.   

It is my understanding that NASA has not measured any thrust forces from this experiment involving a pillbox shaped EM Drive cavity.

NASA expected no significant thrust either, because the pillbox-shaped cavity is entirely symmetric in the longitudinal direction (parallel to the laser beam) and because the cavity did not contain any polymer dielectric insert in the experiments (as so far performed).

So if Jack S. and Eric D.  (whoever they are) are discussing "the notion of gravity in the frustum" I already pointed out that the Interferometer tests were not conducted with a frustum but instead with a pillbox shaped cylindrical geometry. As to the size of the spacetime distortion everybody is in agreement that one expects it to be extremely small.  That's the relevance of Marco Frasca's papers: and why Frasca's papers are so interesting, regarding the appropriate coupling.

QUESTION: In any case what are these people (Jack S. and Eric D. ) so excited about that they are writing about it?  What's the harm with Dr. White trying to find out?

As Frasca wrote in his webpage:

Quote
I would like to remember that White is not using exotic matter at all. Rather, he is working with strong RF fields to try to develop a warp bubble. This was stated here even if implicitly. Finally, an EmDrive device has been properly described here. Using strong external fields to modify locally a space-time has been described here.

If this will be confirmed in the next few months, it will represent a major breakthrough in experimental general relativity since Eddington  confirmed the bending of light near the sun. Applications would follow if this idea will appear scalable but it will be a shocking result anyway. We look forward to hear from White very soon.

Marco Frasca (2005). Strong coupling expansion for general relativity Int.J.Mod.Phys.D15:1373-1386,2006 arXiv: hep-th/0508246v3

Jose,

That is exactly the point about interference experiments. It was gravity that provides an explanation for White's interference experiments. For the frustum, I am working on it and the difficulty is to explain the net force that some people claims to have measured. With an interplay between electromagnetic field, cavity and space-time one should account for this but momentum conservation precludes it. Indeed, if exotic matter would be involved we would be done as, in this case, there is a violation of energy conditions in general relativity and nobody would complain. This is the root of the rejection of the work of EM Drive by the scientific community and there does not seem to be any mechanism inside such a cavity to provide exotic matter to put it at work. The other possibility is that general relativity should be corrected in some way by something else. So far I am sceptical about this because experiments supporting it are overwhelming while nothing is seen on the contrary. On the other side, White's measurement proves unequivocally that the space-time effect is there for a cavity.
Title: Re: EM Drive Developments - related to space flight applications - Thread 2
Post by: frobnicat on 05/08/2015 06:07 pm
....
And if you open the big base of the EM Drive cavity, so that it becomes a speaker, then it is a very inefficient photon rocket, and therefore the claims of Shawyer don't make sense because they are thousands of time greater thrust per input power than a photon rocket  :'(

I need to study @notsosureofit's GR calculation, but I think the point that is being missed here is, the resonant cavity acts as an EM Momentum amplifier. The Q factor comes in not "Just" as the number of photons stored, but also;  In GR as in PV, as the energy density inside the cavity increases, the wave velocity decreases. As velocity decreases, momentum increases. That "effective mass" of the photons in the cavity kicks in stronger.

p ~ E/v  and in the cavity v << c

This makes it a not-so "inefficient" photon rocket!

In the EM Drive design I proposed, we want the Q in the resonant cavity to be very high, but we do not want a high Q in the frustum. We want the backward-moving waves to expand, and the reflected waves moving forward to be attenuated. So we want a low Q, high attenuation in the frustum.

Todd

Sorry if this was already stated but, what is quantitatively the energy density inside the cavity (with a given Q of say, 10000) ? And how much quantitatively the wave velocity would decrease due to this energy density ? According to strict and uncontroversial interpretation of GR ?
Title: Re: EM Drive Developments - related to space flight applications - Thread 2
Post by: frobnicat on 05/08/2015 06:26 pm
....
And if you open the big base of the EM Drive cavity, so that it becomes a speaker, then it is a very inefficient photon rocket, and therefore the claims of Shawyer don't make sense because they are thousands of time greater thrust per input power than a photon rocket  :'(

I need to study @notsosureofit's GR calculation, but I think the point that is being missed here is, the resonant cavity acts as an EM Momentum amplifier. The Q factor comes in not "Just" as the number of photons stored, but also;  In GR as in PV, as the energy density inside the cavity increases, the wave velocity decreases. As velocity decreases, momentum increases. That "effective mass" of the photons in the cavity kicks in stronger.

p ~ E/v  and in the cavity v << c

This makes it a not-so "inefficient" photon rocket!

What if the photon rocket utilized the same construction with a cavity that would make the light move slower?

http://news.harvard.edu/gazette/1999/02.18/light.html

As it is possible to slow the light, then the lower its speed the bigger its thrust.
That's one thing. The other is that the thrust is multiplied by the number of bounccs.
That applies to MW waves in the resonating cavity. Hence - the bigger the number of bounces, the bigger the thrust.
You can actually amplify the photon rocket thrust as well.
See here:
(http://s22.postimg.org/xv4nrzxch/Photon_Thrust_Amplification.jpg)
Source: Wiki, http://upload.wikimedia.org/wikipedia/commons/8/8e/Photon-Thrust-Amplification.jpg

Note that it is not any more useful than a spring pushing between two points : this is just a way to beam action(force)/reaction(opposite force). In principle this could be used to build a "pillar of light" between ground and a craft rising to orbit (albeit the engineering seems too hard).

Quote
Bend this light path with optical fiber and you receive a powerful thruster.
Like this:

(http://s3.postimg.org/8ajq5b2g3/bentfiber.jpg)


A very simple idea... nobody ever checked it yet, I suppose. I think this setup may be worth checking.
Please consider the fact that it is not c in the nominator, but the actual speed of light in the optical fiber. The slower the speed of the light in this apparatus, the greater the thrust!
The EmDrive works based on similar principle.

I'm sorry but to "bend" the light 180° around like that is equivalent in terms of momentum exchange to just bouncing it : in your above drawing the fibre bundle would receive 2F to the right from this momentum exchange. The total force on an isolated rigid craft would still be 0, as in the first drawing.
Title: Re: EM Drive Developments - related to space flight applications - Thread 2
Post by: Notsosureofit on 05/08/2015 06:28 pm
....

You are too fast for my old fingers...

Ultimately, what needs to be shown here is that the "covariant force vector equal to zero" on a photon in the accelerated frame is the same vector in the cavity rest frame (ie. can be transformed to) such that the force on a photon in the rest frame is a result of the (velocity) dispersion due to the shape of the cavity boundary conditions.  That transformation would show that the effect is to be expected under General Relativity.

The Q only enters in the case of a gravitational interaction through its role in calculating the number of photons (Total mass/energy) in the cavity.  If it was a Newtonian momentum interaction only the Power enters.  (I get zero when I try Newtonian for the reasons you have mentioned)

Note: I should probably use "Classical" instead of "Newtonian" but I think you would get the idea that we are looking at the 4-volume as an invariant rather than an integration over the surface of the boundary conditions of classical momentum exchange. That can apply as the summation of the (false gravitational?, ie. frame-dependent) forces on the individual photons to give the resultant force on the cavity.

I very much appreciate the explanation, unfortunately, I may not be able to understand this until I see the explicit mathematical formula for the 4-volume invariant, particularly the derivation of that invariant  :)

Unfortunately, I cannot do it intuitively as you can.

That would, of course, constitute a proof.

Well, OK, the obvious place to start is w/ the Einstein field equation G^jk = 8pT^jk.

  In the accelerated frame of reference w/ the acceleration, g = (c^2/(2*L*f^2))*(c/(2*Pi))

^2*X^2*((1/Rs^2)-(1/Rb^2)) [for this waveguide-like approximation].  We can set the stress–

energy tensor T^jk to a plane wave solution since the assumption here is that the acceleration

is such as to balance out the dispersion found in the rest frame.

     The internet pulls up the Wikipedia entry:

 https://en.wikipedia.org/wiki/Monochromatic_electromagnetic_plane_wave

 [which has already been referenced by @StrongGR] and

 http://en.wikipedia.org/wiki/Talk%3AMonochromatic_electromagnetic_plane_wave

 which is new [?] by Chris Hillman, which points to;

 http://en.wikipedia.org/wiki/Pp-wave_spacetime.

Edit: I used to have
 Penrose, Roger (1965). "A remarkable property of plane waves in general relativity". Rev. Mod. Phys. 37: 215–220. Bibcode:1965RvMP...37..215P. doi:10.1103/RevModPhys.37.215
around here somewhere, I'll look.

Title: Re: EM Drive Developments - related to space flight applications - Thread 2
Post by: WarpTech on 05/08/2015 06:29 pm
....
And if you open the big base of the EM Drive cavity, so that it becomes a speaker, then it is a very inefficient photon rocket, and therefore the claims of Shawyer don't make sense because they are thousands of time greater thrust per input power than a photon rocket  :'(

I need to study @notsosureofit's GR calculation, but I think the point that is being missed here is, the resonant cavity acts as an EM Momentum amplifier. The Q factor comes in not "Just" as the number of photons stored, but also;  In GR as in PV, as the energy density inside the cavity increases, the wave velocity decreases. As velocity decreases, momentum increases. That "effective mass" of the photons in the cavity kicks in stronger.

p ~ E/v  and in the cavity v << c

This makes it a not-so "inefficient" photon rocket!

In the EM Drive design I proposed, we want the Q in the resonant cavity to be very high, but we do not want a high Q in the frustum. We want the backward-moving waves to expand, and the reflected waves moving forward to be attenuated. So we want a low Q, high attenuation in the frustum.

Todd

Sorry if this was already stated but, what is quantitatively the energy density inside the cavity (with a given Q of say, 10000) ? And how much quantitatively the wave velocity would decrease due to this energy density ? According to strict and uncontroversial interpretation of GR ?

My questions as well. Working on it... Like I said, I have a day job and not enough hours in a day. Even if there were no amplification from GR effects, it still amplifies the momentum simply due to the constraints of the cavity forcing a reduced wave velocity.

Todd

Title: Re: EM Drive Developments - related to space flight applications - Thread 2
Post by: TheTraveller on 05/08/2015 07:11 pm
....

Did you or anyone else ever write an excel spreadsheet to calc Shawyers Design Factor? If so pls link it or if not please consider doing it as your skills there are much better than mine.
Yes I have calculated it, but it is a Mathematica program, not an Excel spreadsheet.  I posted (earlier in the thread) comparisons of the measurements vs. predictions using Shawyer's and McCulloch's formulas.

You may want to PM @aero to ask whether he did it with Excel (if my memory is correct @aero also calculated Shawyer's Design Factor, as I recall having exchanges in this forum with him).

And of course, when running your program, you will first check your results vs. Shawyer's published Design Factor results, etc., to make sure that your program is correct.
Mathematica looks interesting but maybe later as I suspect there would be a learning curve.

Is this still your Design Factor equation?

Thanks for your assistance. Most appreciated.

(https://forum.nasaspaceflight.com/index.php?action=dlattach;topic=36313.0;attach=829379;image)


I recognize that equation is my Mathematica-writing, but I need a link to the message where I posted it, in order to remember the context.  Too long ago  :)
Apologies for the delay. Requested link here:
http://forum.nasaspaceflight.com/index.php?topic=29276.msg1276053#msg1276053

Aero did reply but his Design Factor equation did not include Rf

Quote
I did, but the Excel file is not suitable for distribution, having all of my work scattered throughout a 500 line file with little or no embedded explanations. That is, it's un-usable even for me, without considerable time discovering what I intended to do. For example, here is my design factor, Df, equations. I think the variables are:
Lo - length
Lg1-diameter
Lg2-diameter
Df = 0.844   Df=S_o*Lo((1/Lg1) - (1/lg2))  where S_o = (1-(Lo^2/(Lg1*Lg2)))^-1         
Combining   Df = Lo*( 1 - (Lo^2/(Lg1*Lg2)))^-1 *((1/Lg1)-(1/Lg2))     

So before turning either your or Aero's equation to Excel and posting it to the Wiki for all replicators to use, wpuld you please give some feedback on which is correct?

From my trying to follow the Shawyer DF equations, it seems your equation which uses Rf imay be correct?

Would appreciate a set of data variables that I can use to verify my excel equation gives the same results as your Mathematica equation.

As always, thanks for the assistance.
Appreciated.
Title: Re: EM Drive Developments - related to space flight applications - Thread 2
Post by: Blaine on 05/08/2015 07:36 pm
Sorry if this has nothing to due with anything, but just a random thought popped into my head.

http://www.usna.edu/Users/oceano/raylee/RainbowBridge/RB_images/Fig8_7.jpg

To see how this works, we first note that supernumerary bows are not caused by interference between two light waves. Instead, two different portions of the same light wave interfere. In Fig. 8-7, we once again show a circular slice through a raindrop, much as we did in Fig. 6-5. Now, rather than parallel light rays entering the drop, a series of wave ridges and troughs (Fig. 8-7’s vertical lines) advances toward the drop as a front of parallel waves. Think of the parallel lines as representing the wave fronts of parallel rays of sunlight. Rays, which are always locally perpendicular to their corresponding waves, show the waves’ direction of travel.

Feel free to delete if this is stupid or misleading.  Again, its just a random thought whilst thinking about waves in relation to each other.
Title: Re: EM Drive Developments - related to space flight applications - Thread 2
Post by: aero on 05/08/2015 07:55 pm
Well, since we're on random thoughts, I had one, too.

As I have posted on several occasions, there exist a number of papers reporting experiments which seem to show  evanescent wave superluminal velocity and evanescent wave superluminal momentum. That idea is not well accepted in the general physics community yet the experiments do show something out of the ordinary. Does anyone suppose that the experimental results that are explained by superluminal velocity/momentum could be also, perhaps better, explained by Dr. White's quantum vacuum concepts?

And regarding Shawyer's design factor, I did program Dr. Rodel's equation and did match his numbers but it was some time ago and is now lost in the fog of the past.
Title: Re: EM Drive Developments - related to space flight applications - Thread 2
Post by: Notsosureofit on 05/08/2015 08:05 pm


Edit: I used to have
 Penrose, Roger (1965). "A remarkable property of plane waves in general relativity". Rev. Mod. Phys. 37: 215–220. Bibcode:1965RvMP...37..215P. doi:10.1103/RevModPhys.37.215
around here somewhere, I'll look.

Amazing !  Found it after all these years.  I have to re-read and re-understand it of course, but check Fig. 1. "The sandwich wave" where you have a section of curved space-time sandwiched between flat.  That alone is closer to this situation than anything I've seen so far.

The previous article, Rosen, Joe. "Embedding of Various Relativistic Riemannian Spaces in Pseudo-Euclidean Spaces" contains tables of transforms.  That could prove invaluable !
 
Title: Re: EM Drive Developments - related to space flight applications - Thread 2
Post by: TheTraveller on 05/08/2015 08:14 pm
And regarding Shawyer's design factor, I did program Dr. Rodel's equation and did match his numbers but it was some time ago and is now lost in the fog of the past.
Is ok. Will recreate the Design Factor excel spreadsheet and post to the WiKi for all to use. Will never be lost again.

I trust Dr. Rodal will provide a few data sets so I can verify the results? PLEASE.
Title: Re: EM Drive Developments - related to space flight applications - Thread 2
Post by: rfmwguy on 05/08/2015 08:15 pm
Its great to see continuing discussions on this emdrive forum. There are Theorists and there are Observers. I tend to fall into the latter category and appreciate all those willing to build their own test articles, sharing plans & results. Onwards and upwards...
Title: Re: EM Drive Developments - related to space flight applications - Thread 2
Post by: Rodal on 05/08/2015 08:15 pm


Edit: I used to have
 Penrose, Roger (1965). "A remarkable property of plane waves in general relativity". Rev. Mod. Phys. 37: 215–220. Bibcode:1965RvMP...37..215P. doi:10.1103/RevModPhys.37.215
around here somewhere, I'll look.

Amazing !  Found it after all these years.  I have to re-read and re-understand it of course, but check Fig. 1. "The sandwich wave" where you have a section of curved space-time sandwiched between flat.  That alone is closer to this situation than anything I've seen so far.

The previous article, Rosen, Joe. "Embedding of Various Relativistic Riemannian Spaces in Pseudo-Euclidean Spaces" contains tables of transforms.  That could prove invaluable !

An earlier paper, this time by Bondi, on sandwich waves and plane gravitational waves, that, unlike the other ones, has a PDF with a link   :):

http://www.itp.kit.edu/~schreck/general_relativity_seminar/Gravitational_waves_in_general_relativity_exact_plane_waves.pdf

Gravitational waves in general relativity
III. Exact plane waves
BY H. BONDI* AND F. A. E. PIRANIt
King's College, London
AND T. ROBINSON
Lately of University College of Wales, Aberystwyth
(Communicated by W H. McCrea, F.R.S.-Received 18 October 1958)


Plane gravitational waves are here defined to be non-flat solutions of Einstein's empty spacetime
field equations which admit as much symmetry as do plane electromagnetic waves,
namely, a 5-parameter group of motions. A general plane-wave metric is written down and
the properties of plane wave space-times are studied in detail. In particular, their characterization
as 'plane' is justified further by the construction of 'sandwich waves' bounded on both
sides by (null) hyperplanes in flat space-time. It is shown that the passing of a sandwich wave
produces a relative acceleration in free test particles, and inferred from this that such waves
transport energy.
Title: Re: EM Drive Developments - related to space flight applications - Thread 2
Post by: Econocritic on 05/08/2015 08:18 pm
Consider the following EM Drive design:

1st component: A symmetrical resonator cavity that is optimized to store energy at high Q, and plays no role in thrust. Here, we want to store the lowest order mode that the waveguide can sustain because it has the slowest wave velocity inside the waveguide. By targeting the slowest wave velocity, we are maximizing the amount of stored electromagnetic momentum for a given amount electromagnetic energy.

p = E/v, where  v << c inside the waveguide.
E = P*t, energy is power in x time (assuming a lossless cavity)

2nd component: We need a frustum that matches the diameter of the resonant chamber "at the small end" and slowly expands to a TBD length and diameter. As the waveguide expands, the wavelength will be increasing proportional to the increasing wave velocity. We want it to expand to near it's free space value.

...

Is this design analogous to the "Modified Aluminum Cavity for TE011 Mode Resonance" previously presented by Paul March?  Note, there appears to be two chambers separated by a 1/16" plate (aluminum?).  As an avid follower of this thread I have not noticed any comments about the Eagleworks dual chamber design.  However, I could be misinterpreting the attached image. 

This has been a truly fascinating discussion which has never ceased capturing my attention. I feel fortunate to passively witness.  Thank you all, especially the replicators.

-Matthew Trimble       
Title: Re: EM Drive Developments - related to space flight applications - Thread 2
Post by: TheTraveller on 05/08/2015 08:18 pm
Its great to see continuing discussions on this emdrive forum. There are Theorists and there are Observers. I tend to fall into the latter category and appreciate all those willing to build their own test articles, sharing plans & results. Onwards and upwards...
There are also Replicators / Builders here ;)
Title: Re: EM Drive Developments - related to space flight applications - Thread 2
Post by: Rodal on 05/08/2015 08:24 pm
Consider the following EM Drive design:

1st component: A symmetrical resonator cavity that is optimized to store energy at high Q, and plays no role in thrust. Here, we want to store the lowest order mode that the waveguide can sustain because it has the slowest wave velocity inside the waveguide. By targeting the slowest wave velocity, we are maximizing the amount of stored electromagnetic momentum for a given amount electromagnetic energy.

p = E/v, where  v << c inside the waveguide.
E = P*t, energy is power in x time (assuming a lossless cavity)

2nd component: We need a frustum that matches the diameter of the resonant chamber "at the small end" and slowly expands to a TBD length and diameter. As the waveguide expands, the wavelength will be increasing proportional to the increasing wave velocity. We want it to expand to near it's free space value.

...

Is this design analogous to the "Modified Aluminum Cavity for TE011 Mode Resonance" previously presented by Paul March?  Note, there appears to be two chambers separated by a 1/16" plate (aluminum?).  As an avid follower of this thread I have not noticed any comments about this dual chamber design.  However, I could be misinterpreting the attached image. 

This has been a truly fascinating discussion which has never ceased capturing my attention. I feel fortunate to passively witness.  Thank you all, especially the replicators.

-Matthew Trimble       

Thank you for pointing out that important fact, that had escaped my attention.  Much appreciated  :)
Title: Re: EM Drive Developments - related to space flight applications - Thread 2
Post by: TheTraveller on 05/08/2015 08:24 pm
Consider the following EM Drive design:

1st component: A symmetrical resonator cavity that is optimized to store energy at high Q, and plays no role in thrust. Here, we want to store the lowest order mode that the waveguide can sustain because it has the slowest wave velocity inside the waveguide. By targeting the slowest wave velocity, we are maximizing the amount of stored electromagnetic momentum for a given amount electromagnetic energy.

p = E/v, where  v << c inside the waveguide.
E = P*t, energy is power in x time (assuming a lossless cavity)

2nd component: We need a frustum that matches the diameter of the resonant chamber "at the small end" and slowly expands to a TBD length and diameter. As the waveguide expands, the wavelength will be increasing proportional to the increasing wave velocity. We want it to expand to near it's free space value.

...

Is this design analogous to the "Modified Aluminum Cavity for TE011 Mode Resonance" previously presented by Paul March?  Note, there appears to be two chambers separated by a 1/16" aluminum plate.  As an avid follower of this thread I have not noticed any comments about this dual chamber design.  However, I could be misinterpreting the attached image. 

This has been a truly fascinating discussion which has never ceased capturing my attention. I feel fortunate to passively witness.  Thank you all, especially the replicators.

-Matthew Trimble       
According to Paul March, this is the cavity EW will be using to do a more formal replication of Shawyer and the Chinese using Shawyers Teeter Totter balance beam, which I assume means operation in a vertical orientation.

Trust EW will remove the partition in their alum cavity?
Title: Re: EM Drive Developments - related to space flight applications - Thread 2
Post by: maciejzi on 05/08/2015 08:43 pm

Quote
Bend this light path with optical fiber and you receive a powerful thruster.
Like this:

(http://s3.postimg.org/8ajq5b2g3/bentfiber.jpg)


A very simple idea... nobody ever checked it yet, I suppose. I think this setup may be worth checking.
Please consider the fact that it is not c in the nominator, but the actual speed of light in the optical fiber. The slower the speed of the light in this apparatus, the greater the thrust!
The EmDrive works based on similar principle.

I'm sorry but to "bend" the light 180° around like that is equivalent in terms of momentum exchange to just bouncing it : in your above drawing the fibre bundle would receive 2F to the right from this momentum exchange. The total force on an isolated rigid craft would still be 0, as in the first drawing.

Photons are in a different frame of reference. The above system should move to the left.

The above system as well as EmDrive is like a cavity with an externally attached pendulum. The pendulum's ball (represents light wave or microwave) bounces in it. And moves the light/MW cavity to the side where it bounces the most.

Title: Re: EM Drive Developments - related to space flight applications - Thread 2
Post by: deltaMass on 05/08/2015 09:03 pm
Only Shawyer talks about "the frame of reference of a photon".
http://physics.stackexchange.com/questions/68600/frame-of-reference-of-the-photon
Speaking personally, I wouldn't. And especially if it implies that bent optical fibres constitute a propellantless propulsion device.

For which, may I remind, you must either abandon Noether or you must abandon Einstein(*)

(*) Special principle of relativity: If a system of coordinates K is chosen so that, in relation to it, physical laws hold good in their simplest form, the same laws hold good in relation to any other system of coordinates K' moving in uniform translation relatively to K.
    —Albert Einstein: The foundation of the general theory of relativity, Section A, §1

Oh, and the total force on the mirrors (assume attached to one another via a vertical truss) is 4 P/c, where P is the power of the beam. By symmetry, that's the total force integrated over the bend. Net force = 0.




Title: Re: EM Drive Developments - related to space flight applications - Thread 2
Post by: frobnicat on 05/08/2015 10:54 pm
...

Photons are in a different frame of reference. The above system should move to the left.

The above system as well as EmDrive is like a cavity with an externally attached pendulum. The pendulum's ball (represents light wave or microwave) bounces in it. And moves the light/MW cavity to the side where it bounces the most.

It will not "move" to the left (or rather you should say "thrust to the left", or equivalently "accelerate to the left" as velocity is always relative while acceleration can be defined in absolute, relative to an inertial test particle). Anyway, it will not, as deltaMass says.

If playing around with curved trajectories of momentum carriers were a way to cheat like that on momentum conservation, it would be long ago mechanical devices such as Dean drives would have been shown to work reliably to provide propulsion, er, action without reaction. Just imagine a perfect ball, attached to a string to a pivot on a plate, the ball is launched parallel to the plate with given velocity. After a quarter circle trajectory it bounces on the plate and make half circle to bounce again (at point opposite from pivot), half circle up, half circle down and so on. This is similar to your photon bouncing back and forth through your optical fibre bundle. When the ball does its half circle trajectory, the centrifugal force pull on the string. While the bounces give impulse to the left, the pull of the string gives impulse to the right. 0 net average.

Likewise imagine you hold a half pipe and someone throws a bowling bowl through it so that it is exiting back to the launcher, wouldn't you experience quite a recoil ? This would be the same recoil, but opposite (to the right) that the sum of the recoil the launcher experienced first when launching, then when catching back the returning bowl (both to the left) . If each you and the launcher are standing on your own skateboard, after the experiment both will have same momentum but opposite direction (same speed but opposite, assuming same weights). If  you and the launcher are standing on the same skateboard, after the experiment not only the skateboard has no velocity (assuming starting from rest) but it is also in same position as before. This is conservation of momentum, and has been experimentally shown with great precision in quite a variety of contexts, and has strong theoretical backing with Noether's theorem relating it to translation invariance of space (basically if reality behaves a certain way at location A, then it behaves the same way at location B, all things and fields being equal there are no special places of reality).
Title: Re: EM Drive Developments - related to space flight applications - Thread 2
Post by: maciejzi on 05/08/2015 11:02 pm
For the same reason the MW cavity shouldn't exert any directional force. But it does. And if so, then probably/maybe a similarly built light cavity with large mirrors on one side and bent optical fiber or small mirror on the other side should produce thrust as well.

The number of light bounces (mirror reflectance) would then represent the photonic thrust multiplication factor similar to Q in EmDrive thrust formula devised by Shawyer.
---

Another issue: why not use Peltier element to cool the superconducting cavity instead of liquid gas?
The vehicle will need electric power anyway, so why add heavy liquid hydrogen reservoir?
The additional electric energy for Peltier cooling shouldn't require as much mass and space as the hydrogen.
It may be economical even for satellites, because the large increase in Q and resulting smaller required input power and smaller engine size would justify additional Peltier module.

Please kindly excuse me if this is an incorrect way of thinking for some reason I am not aware of.
Title: Re: EM Drive Developments - related to space flight applications - Thread 2
Post by: deltaMass on 05/08/2015 11:21 pm
Interesting that breaking CofM means that translational invariance dies, and breaking Einstein's SR core principle means that inertial frame invariance dies. Interesting because you may notice that one is essentially the time derivative of the other.  And further that breaking CofE means that time invariance dies.

Perhaps that's a deep observation, but I'm not smart enough to know what it means. All I know is that, for a propellantless propulsor, we are forced to choose between killing off one or the other. If we got creative we could kill off both, maybe!

I'm sure my physics friends down the pub here in Cupertino would simply shrug and say that, since momentum appears not be conserved in the first place, there's ample room for odd statements to be correspondingly made about energy conservation (and I have made them here on this forum).

Perhaps there's an exit route out of this bind via playing off conservation of momentum against conservation of energy. Just a wild thought.
Title: Re: EM Drive Developments - related to space flight applications - Thread 2
Post by: maciejzi on 05/08/2015 11:23 pm
I must admit though, that unfortunately the EmDrive in its principles resembles the Dean drive...
 :-[
Title: Re: EM Drive Developments - related to space flight applications - Thread 2
Post by: deltaMass on 05/08/2015 11:27 pm
Spacetime has stiction? Say it ain't so!  ::)
Title: Re: EM Drive Developments - related to space flight applications - Thread 2
Post by: frobnicat on 05/08/2015 11:36 pm
For the same reason the MW cavity shouldn't exert any directional force.

No, it shouldn't. Why the high degree of scepticism. Don't fool yourself, the ratio of people believing this is possible to those believing this is impossible in this thread is not representative of the ratio in the general population of engineers and scientists at large. Sometimes a scientific minority is right and the general consensus is wrong. But often, the consensus is right. Contrary to what we see in movies, the ratio of maverick scientist (in fundamental science) that win at the end is low.

Quote
But it does.

It appears to do, in a few labs. A lot of "3 sigmas" experimental signals (probability of .001 that result is due to a fluke rather than a real effect) in fundamental science prove to be flukes after all, or lose cable, or wrong interpretation, or "look elsewhere effect" (that is, if you look hard enough for something in a big enough variety of ways, you will find it, even if it doesn't exist).

Quote
And if so, then probably/maybe a similarly built light cavity with large mirrors on one side and bent optical fiber or small mirror on the other side should produce thrust as well.

If so, then a lot of things become possible, like pulling on one own shoes to fly above the ground. Einstein protects us ! But looks to me bouncing back vs bending 180° would still make no fundamental difference.

Quote
The number of light bounces (mirror reflectance) would then represent the photonic thrust multiplication factor similar to Q in EmDrive thrust formula devised by Shawyer.

mm

Quote
---

Another issue: why not use Peltier element to cool the superconducting cavity instead of liquid gas?
The vehicle will need electric power anyway, so why add heavy liquid hydrogen reservoir?
The additional electric energy for Peltier cooling shouldn't require as much mass and space as the hydrogen.
It may be economical even for satellites, because the large increase in Q and resulting smaller required input power and smaller engine size would justify additional Peltier module.

Please kindly excuse me if this is an incorrect way of thinking for some reason I am not aware of.

Because Peltier effect can't quite reach cryogenic temperatures, for some thermodynamic reason I forgot. Maybe it changed with recent progress on the subject ? Please inquire : what are the limits of low temp. with Peltier effect, now and tomorrow ? Isn't the Peltier effect still quite low in efficiency ?
Title: Re: EM Drive Developments - related to space flight applications - Thread 2
Post by: maciejzi on 05/08/2015 11:45 pm
Because Peltier effect can't quite reach cryogenic temperatures, for some thermodynamic reason I forgot. Maybe it changed with recent progress on the subject ? Please inquire : what are the limits of low temp. with Peltier effect, now and tomorrow ? Isn't the Peltier effect still quite low in efficiency ?

The Peltier limits are below 100K, which is enough for high-temparature superconductors. The efficiency in such temperature is low, but probably that could be economically viable because of much smaller MW input power and smaller superconducting engine size.

That said, I must sadly admit, that this engine won't probably move in free space. It may move on air cushion though, as in the experiments conducted so far on Earth.

Anyway, if so, then it is at least good for new generation of rotorless helicopters, here on Earth.

Title: Re: EM Drive Developments - related to space flight applications - Thread 2
Post by: frobnicat on 05/08/2015 11:51 pm
Spacetime has stiction? Say it ain't so!  ::)

Quantified spacetime ? But this would likely need to go down to Planck scales... and a bunch of top theorists to avoid random musing drift.
Title: Re: EM Drive Developments - related to space flight applications - Thread 2
Post by: SeeShells on 05/09/2015 12:00 am
I was wondering in your testing did you use a smoke stick also called a smoke pencil to check for air flows around the EM Drive in ambient air conditions?
Title: Re: EM Drive Developments - related to space flight applications - Thread 2
Post by: frobnicat on 05/09/2015 12:42 am
Interesting that breaking CofM means that translational invariance dies, and breaking Einstein's SR core principle means that inertial frame invariance dies. Interesting because you may notice that one is essentially the time derivative of the other.  And further that breaking CofE means that time invariance dies.

Perhaps that's a deep observation, but I'm not smart enough to know what it means. All I know is that, for a propellantless propulsor, we are forced to choose between killing off one or the other. If we got creative we could kill off both, maybe!

I'm sure my physics friends down the pub here in Cupertino would simply shrug and say that, since momentum appears not be conserved in the first place, there's ample room for odd statements to be correspondingly made about energy conservation (and I have made them here on this forum).

Perhaps there's an exit route out of this bind via playing off conservation of momentum against conservation of energy. Just a wild thought.

Killing CoM is killing CoE if SR still stands.
Both leading theories about propellantless devices, White's and Woodward's, can pretend to kill none.
White mutable vacuum -> extract both momentum and energy from what is there locally
Woodward Mach effect -> extract both momentum and energy from what is there globally
But none admit that this gives, in effect, a practically limitless energy generator sitting in a box sitting in deep space, even while this could be argued on potentially sane conservation basis (zenergy comes from somewhere...)

I also have my theory that can pretend to kill no principle and yet allows for a device at 1N/kW, it is not strictly propellantless, it emits particles, namely tachyons, allowing it to give more apparent energy than expanded, as tachyons can be seen as energetic debt. Basically it is sending away at FTL speed the energetic debt of the apparent local CoE breaking so that the accountancy is kept in balance overall. This is my "white paper" on the topic. (http://forum.nasaspaceflight.com/index.php?topic=29276.msg1275281#msg1275281) It is somehow reminiscent of the advanced/retarded waves of Woodward, so I probably can't pretend at complete anteriority on that one, maybe just a minor alternate way of stating the same things.

While they don't like to discuss the theories in terms of locally generable energy from fear of association with "free energy looneys", I'm sure both contenders are convinced that their 1N/kW is stable (not frame/history dependant), and know that they must apparently break CoE, that they can be used as generators of ultra cheap (almost free) energy. What makes me think like that is
1/ It makes a whole lot more sense (bringing back no apparent CoE after the 1N/kW fact is untenable)
2/ We have never seen a single time any mention of check of dependency of experimental thrust relative to sidereal time or other "absolute frame" relative to which it may vary (if we are to save apparent CoE at the expanse of SR)

Appears the only compulsory casualty is apparent CoE, not necessarily CoE in itself if one can find a suitable positive energy source (or a negative energy sink) and a mechanism to couple with such source (or sink to get rid of debt).

How the (few) other theories out there deal with the apparent CoE demise ?
Title: Re: EM Drive Developments - related to space flight applications - Thread 2
Post by: zellerium on 05/09/2015 12:48 am
Hey everyone,

Based on the advice from the forum we have reformulated our experimental design . We have been given permission to use one of the vacuum chambers at Cal Poly which can pull about 1mT using a mechanical pump. By mounting everything on a pendulum we can measure a displacement using a reflected laser. I am not experienced with laser measurements and would greatly appreciate any advice for improving this measurement. With a simple setup it seems we will be able to see 100 mN easily, hopefully this is within reason at powers ~1200 W.   

We have also been considering using aluminum for the frustum to minimize the mass of the pendulum. This would coincide with Todd's theory that a lower Q in the frustum is more desirable. We could easily implement a symmetric copper resonant cavity to between the magnetron and the frsutum:

1st component: A symmetrical resonator cavity that is optimized to store energy at high Q, and plays no role in thrust. Here, we want to store the lowest order mode that the waveguide can sustain because it has the slowest wave velocity inside the waveguide.

I did some basic solutions for a resonant rectangular chamber and found that to isolate the TE 011 mode at a frequency of 2.4503 GHz we would need a box 7.068 x 7.068 x 12.24 cm. Would we want to match the end of this box to the small end of the frustum?

What should the interface look like?
Perhaps we should keep our experiment simple and introduce this complication later...

Also, does anyone know the dimensions of the aluminum cavity being constructed at Eagleworks?  I believe the picture posted is missing a dimension required to solve for the others, or maybe my geometry is lacking.
 
Is this design analogous to the "Modified Aluminum Cavity for TE011 Mode Resonance" previously presented by Paul March?  Note, there appears to be two chambers separated by a 1/16" plate (aluminum?).  As an avid follower of this thread I have not noticed any comments about this dual chamber design.  However, I could be misinterpreting the attached image. 

This has been a truly fascinating discussion which has never ceased capturing my attention. I feel fortunate to passively witness.  Thank you all, especially the replicators.

-Matthew Trimble       

Hopefully we can use EW's dimensions and not have to worry about FEA. 

The electrical connections into the chamber will allow the circuit to be kept outside and connected with slack to a magnetron on the pendulum.   I am concerned that the magnetron will overheat quickly due to a lack of convection cooling, but maybe we can run shorter tests to compensate.

Any thoughts?


Glad to see everyone still asking questions and presenting ideas! It is quite an amazing discussion and I look forward to contributing some data.

Kurt Zeller

Title: Re: EM Drive Developments - related to space flight applications - Thread 2
Post by: Rodal on 05/09/2015 01:09 am
....

Did you or anyone else ever write an excel spreadsheet to calc Shawyers Design Factor? If so pls link it or if not please consider doing it as your skills there are much better than mine.
Yes I have calculated it, but it is a Mathematica program, not an Excel spreadsheet.  I posted (earlier in the thread) comparisons of the measurements vs. predictions using Shawyer's and McCulloch's formulas.

You may want to PM @aero to ask whether he did it with Excel (if my memory is correct @aero also calculated Shawyer's Design Factor, as I recall having exchanges in this forum with him).

And of course, when running your program, you will first check your results vs. Shawyer's published Design Factor results, etc., to make sure that your program is correct.
Mathematica looks interesting but maybe later as I suspect there would be a learning curve.

Is this still your Design Factor equation?

Thanks for your assistance. Most appreciated.

(https://forum.nasaspaceflight.com/index.php?action=dlattach;topic=36313.0;attach=829379;image)


I recognize that equation is my Mathematica-writing, but I need a link to the message where I posted it, in order to remember the context.  Too long ago  :)
Apologies for the delay. Requested link here:
http://forum.nasaspaceflight.com/index.php?topic=29276.msg1276053#msg1276053

Aero did reply but his Design Factor equation did not include Rf

Quote
I did, but the Excel file is not suitable for distribution, having all of my work scattered throughout a 500 line file with little or no embedded explanations. That is, it's un-usable even for me, without considerable time discovering what I intended to do. For example, here is my design factor, Df, equations. I think the variables are:
Lo - length
Lg1-diameter
Lg2-diameter
Df = 0.844   Df=S_o*Lo((1/Lg1) - (1/lg2))  where S_o = (1-(Lo^2/(Lg1*Lg2)))^-1         
Combining   Df = Lo*( 1 - (Lo^2/(Lg1*Lg2)))^-1 *((1/Lg1)-(1/Lg2))     

So before turning either your or Aero's equation to Excel and posting it to the Wiki for all replicators to use, wpuld you please give some feedback on which is correct?

From my trying to follow the Shawyer DF equations, it seems your equation which uses Rf imay be correct?

Would appreciate a set of data variables that I can use to verify my excel equation gives the same results as your Mathematica equation.

As always, thanks for the assistance.
Appreciated.
Recall that Shawyer defines the Thrust force as:

ThrustForce = 2 * InputPower * Q * DesignFactor / SpeedOfLightInMedium


I checked my formula for Shawyer's Design Factor  (Eq. 12 of http://www.emdrive.com/theorypaper9-4.pdf ) (http://forum.nasaspaceflight.com/index.php?action=dlattach;topic=36313.0;attach=829378) and it is correct.
(The nomenclature could be better: speedOfLightInMedium instead of c,  cavityLength instead of demoCavityLength, frequency instead of rfFrequency etc.)

(Note: Shawyer to my knowledge never wrote his Design Factor formula this way, instead he wrote it as a lengthy series of operations.   My formula is identical to his, except that it involves fewer steps to calculate, since I eliminated unnecessary calculations)


To make things easier for you I quickly wrote the calculation on an Excel spreadsheet that I attach below.

You can try different inputs and check the results vs. the calculations I posted here:

http://forum.nasaspaceflight.com/index.php?topic=29276.msg1276053#msg1276053
Title: Re: EM Drive Developments - related to space flight applications - Thread 2
Post by: WarpTech on 05/09/2015 01:14 am
Hey everyone,

Based on the advice from the forum we have reformulated our experimental design . We have been given permission to use one of the vacuum chambers at Cal Poly which can pull about 1mT using a mechanical pump. By mounting everything on a pendulum we can measure a displacement using a reflected laser. I am not experienced with laser measurements and would greatly appreciate any advice for improving this measurement. With a simple setup it seems we will be able to see 100 mN easily, hopefully this is within reason at powers ~1200 W.   

We have also been considering using aluminum for the frustum to minimize the mass of the pendulum. This would coincide with Todd's theory that a lower Q in the frustum is more desirable. We could easily implement a symmetric copper resonant cavity to between the magnetron and the frsutum:

1st component: A symmetrical resonator cavity that is optimized to store energy at high Q, and plays no role in thrust. Here, we want to store the lowest order mode that the waveguide can sustain because it has the slowest wave velocity inside the waveguide.

I did some basic solutions for a resonant rectangular chamber and found that to isolate the TE 011 mode at a frequency of 2.4503 GHz we would need a box 7.068 x 7.068 x 12.24 cm. Would we want to match the end of this box to the small end of the frustum?

What should the interface look like?
Perhaps we should keep our experiment simple and introduce this complication later...

Also, does anyone know the dimensions of the aluminum cavity being constructed at Eagleworks?  I believe the picture posted is missing a dimension required to solve for the others, or maybe my geometry is lacking.
 
Is this design analogous to the "Modified Aluminum Cavity for TE011 Mode Resonance" previously presented by Paul March?  Note, there appears to be two chambers separated by a 1/16" plate (aluminum?).  As an avid follower of this thread I have not noticed any comments about this dual chamber design.  However, I could be misinterpreting the attached image. 

This has been a truly fascinating discussion which has never ceased capturing my attention. I feel fortunate to passively witness.  Thank you all, especially the replicators.

-Matthew Trimble       

Hopefully we can use EW's dimensions and not have to worry about FEA. 

The electrical connections into the chamber will allow the circuit to be kept outside and connected with slack to a magnetron on the pendulum.   I am concerned that the magnetron will overheat quickly due to a lack of convection cooling, but maybe we can run shorter tests to compensate.

Any thoughts?


Glad to see everyone still asking questions and presenting ideas! It is quite an amazing discussion and I look forward to contributing some data.

Kurt Zeller

Kurt,

I wish I knew the type of interface to use. I am not an expert in microwave components, but I'm researching as time permits. Also, at 2.4503 GHz, it's going to be pretty large. Did you calculate the wave velocity for that mode?

Todd
Title: Re: EM Drive Developments - related to space flight applications - Thread 2
Post by: Rodal on 05/09/2015 01:37 am
I was wondering in your testing did you use a smoke stick also called a smoke pencil to check for air flows around the EM Drive in ambient air conditions?
Although a Google search reveals that this has been brought up before, for example here by @aero: http://forum.nasaspaceflight.com/index.php?topic=29276.msg1275034#msg1275034 there is no entry I could find in the threads as to whether any of the research groups actually used smoke to check air flow.   Perhaps others can try separate searches either using their memories or using Google (anything but the poorly functioning "Search" button between "Unread Topics" and "Profile"  ;)  )
Title: Re: EM Drive Developments - related to space flight applications - Thread 2
Post by: Rodal on 05/09/2015 02:03 am


Edit: I used to have
 Penrose, Roger (1965). "A remarkable property of plane waves in general relativity". Rev. Mod. Phys. 37: 215–220. Bibcode:1965RvMP...37..215P. doi:10.1103/RevModPhys.37.215
around here somewhere, I'll look.

Amazing !  Found it after all these years.  I have to re-read and re-understand it of course, but check Fig. 1. "The sandwich wave" where you have a section of curved space-time sandwiched between flat.  That alone is closer to this situation than anything I've seen so far.

The previous article, Rosen, Joe. "Embedding of Various Relativistic Riemannian Spaces in Pseudo-Euclidean Spaces" contains tables of transforms.  That could prove invaluable !

An earlier paper, this time by Bondi, on sandwich waves and plane gravitational waves, that, unlike the other ones, has a PDF with a link   :):

http://www.itp.kit.edu/~schreck/general_relativity_seminar/Gravitational_waves_in_general_relativity_exact_plane_waves.pdf

Gravitational waves in general relativity
III. Exact plane waves
BY H. BONDI* AND F. A. E. PIRANIt
King's College, London
AND T. ROBINSON
Lately of University College of Wales, Aberystwyth
(Communicated by W H. McCrea, F.R.S.-Received 18 October 1958)


Plane gravitational waves are here defined to be non-flat solutions of Einstein's empty spacetime
field equations which admit as much symmetry as do plane electromagnetic waves,
namely, a 5-parameter group of motions. A general plane-wave metric is written down and
the properties of plane wave space-times are studied in detail. In particular, their characterization
as 'plane' is justified further by the construction of 'sandwich waves' bounded on both
sides by (null) hyperplanes in flat space-time. It is shown that the passing of a sandwich wave
produces a relative acceleration in free test particles, and inferred from this that such waves
transport energy.

Bondi wrote the following regarding the possibility to extract energy from plane gravitational waves (Bondi points out that he had brought this up in 1957):

Quote
As previously pointed out by one of us (Bondi I957), this relative acceleration and
consequent relative velocity prove that gravitational waves transport energy, since
it is in principle possible, utilizing this effect, to construct a device which will extract
energy from a wave. The simplest such device consists of a stiff rod (the rod need not
be rigid in the technical sense, and the difficulties surrounding the consideration of
rigid bodies in relativity theory are not relevant here) and a bead which slides on the
rod with some friction. If the rod lies in a suitable direction transverse to the
direction of wave propagation, and if the bead is at rest relative to the rod at a
position well displaced from the rod's centre of mass, the passing of the wave will
result in some relative motion of the rod and the bead, for in the first approximation
the bead and the mass centre of the rod will each move on a geodesic. This relative
motion will generate heat, and thus locally available energy may be extracted from
the wave.

In these considerations, the effect of the device on the wave has been neglected.
This is a test device-a device constructed out of test particles. Consequently, such
considerations cannot be used to calculate the total amount of available energy in
the wave.
[snip]
...............................................................................
It is clear from the relative acceleration acquired by test particles, as described in sections 3 and 4, that energy
is transferred to test particles by a plane wave, but this does not enable us to make
quantitative assertions about energy transport in general. The present fluid state
of the theory of the energy pseudo-tensor would not appear to justify a discussion of
energy transport in terms of this concept.

 
Title: Re: EM Drive Developments - related to space flight applications - Thread 2
Post by: SeeShells on 05/09/2015 02:05 am
I was wondering in your testing did you use a smoke stick also called a smoke pencil to check for air flows around the EM Drive in ambient air conditions?
Although a Google search reveals that this has been brought up before, for example here by @aero: http://forum.nasaspaceflight.com/index.php?topic=29276.msg1275034#msg1275034 there is no entry I could find in the threads as to whether any of the research groups actually used smoke to check air flow.   Perhaps others can try separate searches either using their memories or using Google (anything but the poorly functioning "Search" button between "Unread Topics" and "Profile"  ;)  )
Thank you Rodal for replying, I also couldn't find where this simple test was performed. Maybe it could be something tested at a later date?
Title: Re: EM Drive Developments - related to space flight applications - Thread 2
Post by: zen-in on 05/09/2015 02:18 am
Because Peltier effect can't quite reach cryogenic temperatures, for some thermodynamic reason I forgot. Maybe it changed with recent progress on the subject ? Please inquire : what are the limits of low temp. with Peltier effect, now and tomorrow ? Isn't the Peltier effect still quite low in efficiency ?

The Peltier limits are below 100K, which is enough for high-temparature superconductors. The efficiency in such temperature is low, but probably that could be economically viable because of much smaller MW input power and smaller superconducting engine size.

That said, I must sadly admit, that this engine won't probably move in free space. It may move on air cushion though, as in the experiments conducted so far on Earth.

Anyway, if so, then it is at least good for new generation of rotorless helicopters, here on Earth.

Stirling coolers are used where cryogenic temperatures are required.   They are more efficient than Peltier devices.   I have used very compact Stirling cooled IR detectors.   Several companies make them,   It takes about 1 Min. to reduce the temperature of the detector to 95 K, using 1 Watt.    However the thermal mass is very tiny.    NASA has been investigating Stirling coolers for liquifying rocket fuels (H2, O2) in space and for space telescope applications.
Title: Re: EM Drive Developments - related to space flight applications - Thread 2
Post by: MrVibrating on 05/09/2015 02:19 am
Interesting that breaking CofM means that translational invariance dies, and breaking Einstein's SR core principle means that inertial frame invariance dies. Interesting because you may notice that one is essentially the time derivative of the other.  And further that breaking CofE means that time invariance dies.

Perhaps that's a deep observation, but I'm not smart enough to know what it means. All I know is that, for a propellantless propulsor, we are forced to choose between killing off one or the other. If we got creative we could kill off both, maybe!

I'm sure my physics friends down the pub here in Cupertino would simply shrug and say that, since momentum appears not be conserved in the first place, there's ample room for odd statements to be correspondingly made about energy conservation (and I have made them here on this forum).

Perhaps there's an exit route out of this bind via playing off conservation of momentum against conservation of energy. Just a wild thought.

The conundrum may be resolved in two words:

Noether's theorem. 

In defining how symmetries are enforced, by extension it reveals where they are not.

Specifically, time-dependent (ie. temporally variant) interactions are, by definition, non-conservative.  Although we more commonly encounter such asymmetries in dissipative systems, there are, within electromagnetism at least, non-dissipative non-conservative systems.

And before anyone protests, this isn't half as controversial as it may seem - any electrodynamics textbook will include a section on non-conservative, temporally variant EM interactions.  By definition, CoE does not and cannot be applied to them.  See Rutherford's first paper ca 1886 on magnetic entropy viscosity (Sv)... in such a delayed response, the time-dependent rise of B to a given H means input and output FxD integrals can be non-equitable, if their mechanical displacements are varied to sub- and super-Sv speeds respectively.

Example: take two permanent magnets, at least one of which has appreciable Sv.  Allow them to attract together before B can reach Bmax, obtaining our output FxD integral, then let B peak before separating them against this now-higher force, requiring an appropriately-greater input work integral due to the higher force over the same distance.... we've input more work than the interaction has output!  Where'd the energy go?  Not dissipated to heat (the magnetocaloric profiles are almost identical, and incidental since net change in B up vs down is equal for both integrals - ie. Sv isn't a direct heating mechanism).  Rather, the answer's right there in the setup - the missing energy was spent entirely on displacement against a higher magnetic force.  Or, looking at it from the alternative perspective, squandered away by not harvesting it in the first place during the initial delayed-response output displacement.

We can repeat this interaction forever, dumping the same amount of mechanical energy into the vacuum (via the virtual photon exchanges mediating the force) each cycle.  Calorimetry will show a continuing loss...

And yes, this asymmetry IS reversible, but now things really are getting controversial...

The way to look at it is that time-dependent classical symmetry breaks aren't so much Noether exceptions, as ultimate validations of the principle - an inevitable consequence of passive temporally-variable field densities and their corresponding force magnitudes in non-dissipative systems.  The term 'directional vacuum coupler' has already been coined for other RF apparatus, which is a shame as it'd be most apposite here with this unwieldy nomenclature, but i'm no good at neologisms so it is what it is...  "temporally-asymmetric non-dissipative systems".  These are fundamentally open thermodyanic systems, due to the time-dependence factor, and it is the vacuum that they are open to.

What would make headlines for me would be doing it with momentum...  i've been trying for years without success.  Mass and momentum always seem to be mediated at C and there's no obvious mechanical equivalents of magnetic properties like coercivity, remanance or permeability etc.  I've seen tantalising suggestions, yet nothing concrete, all but concluding that breaking CoE is almost trivial compared to CoM.   There's still one or two pots on the boil though, so maybe the EM drive is also a contender.. Might the frustum geometry introduce some kind of temporal variance to the EM interaction between alternate ends of the bell?

Whatever's happening in there, Noether has to have something to say about it....
Title: Re: EM Drive Developments - related to space flight applications - Thread 2
Post by: zen-in on 05/09/2015 05:40 am
I was wondering in your testing did you use a smoke stick also called a smoke pencil to check for air flows around the EM Drive in ambient air conditions?

That is a very good idea.  I don't think it has been suggested before.   There was some discussion on convective air flow and the possiblity it may explain the anomalous force measured by EW early in thread 1.   However interest in that explanation has dissipated and Dr. Rodal's analysis of thermal-mechanical effects as a conventional explanation has replaced it.
Title: Re: EM Drive Developments - related to space flight applications - Thread 2
Post by: maciejzi on 05/09/2015 07:49 am
This engine won't get us to stars. The reason is not the shape of the frutrum or cavity or frutrum material, but the ineffective process of converting mass into electricity.

E=mc^2

For MW energy to take us to stars we need to efficiently convert the mass in the ship to MW and then efficiently send it in the direction opposite to the direction the ship is going to.

Let's say 10% of the ship's  mass is propellent. We want the ship to achieve, say, 0,5 c and get to Alpha Centauri in, say, 15 years.

Hence, it is possible to calculate the required propelling energy.

Hence, it is possible to calculate the required ship's mass to energy conversion efficiency.

As far as I know it is currently much lower than the requred level.

Hence, we currently cannot create as much microwaves (or any other energy) to achieve required acceleration consistently throughout the entire journey.


The EmDrive CAN move in vacuum OR in zero gravity. It CANNOT move in vacum AND zero gravity.


Thrust readouts achieved on Earth (non-zero gravity) are incorrect because it pushes against the air cusion in air bearing with very small force but high MW frequency. That is why it moves so quickly compared to low input power.

In free space (vacuum AND zero gravity) it probably has thrust in order of nanonewtons or even smaller. I don't think its shape is efficient in sending MW in one direction.
Title: Re: EM Drive Developments - related to space flight applications - Thread 2
Post by: jmossman on 05/09/2015 07:56 am
Interesting that breaking CofM means that translational invariance dies, and breaking Einstein's SR core principle means that inertial frame invariance dies. Interesting because you may notice that one is essentially the time derivative of the other.  And further that breaking CofE means that time invariance dies.

Perhaps that's a deep observation, but I'm not smart enough to know what it means. All I know is that, for a propellantless propulsor, we are forced to choose between killing off one or the other. If we got creative we could kill off both, maybe!

I'm sure my physics friends down the pub here in Cupertino would simply shrug and say that, since momentum appears not be conserved in the first place, there's ample room for odd statements to be correspondingly made about energy conservation (and I have made them here on this forum).

Perhaps there's an exit route out of this bind via playing off conservation of momentum against conservation of energy. Just a wild thought.

The conundrum may be resolved in two words:

Noether's theorem. 

In defining how symmetries are enforced, by extension it reveals where they are not.

Specifically, time-dependent (ie. temporally variant) interactions are, by definition, non-conservative.  Although we more commonly encounter such asymmetries in dissipative systems, there are, within electromagnetism at least, non-dissipative non-conservative systems.

And before anyone protests, this isn't half as controversial as it may seem - any electrodynamics textbook will include a section on non-conservative, temporally variant EM interactions.  By definition, CoE does not and cannot be applied to them.  See Rutherford's first paper ca 1886 on magnetic entropy viscosity (Sv)... in such a delayed response, the time-dependent rise of B to a given H means input and output FxD integrals can be non-equitable, if their mechanical displacements are varied to sub- and super-Sv speeds respectively.

Example: take two permanent magnets, at least one of which has appreciable Sv.  Allow them to attract together before B can reach Bmax, obtaining our output FxD integral, then let B peak before separating them against this now-higher force, requiring an appropriately-greater input work integral due to the higher force over the same distance.... we've input more work than the interaction has output!  Where'd the energy go?  Not dissipated to heat (the magnetocaloric profiles are almost identical, and incidental since net change in B up vs down is equal for both integrals - ie. Sv isn't a direct heating mechanism).  Rather, the answer's right there in the setup - the missing energy was spent entirely on displacement against a higher magnetic force.  Or, looking at it from the alternative perspective, squandered away by not harvesting it in the first place during the initial delayed-response output displacement.

We can repeat this interaction forever, dumping the same amount of mechanical energy into the vacuum (via the virtual photon exchanges mediating the force) each cycle.  Calorimetry will show a continuing loss...

I hesitate to interject, but I do have a quick question on the topic of calorimetry associated with the EM drive.

When a resonant cavity (such as Shawyer's frustum) is oriented with it's major resonant axis oriented parallel with a gravitation field, the resulting Doppler red shift experienced by any resonating microwaves will slowly shift their frequency away from the resonant frequency (Fr) of the cavity.  As the frequency of a trapped microwave approaches the cutoff frequency of the resonant cavity, one would expect it to be attenuated faster.

Would the faster attenuation due to the microwave frequency drifting away from the cavity's Fr be completely measured as additional thermal losses in the copper/cavity walls?  What about the energy reduction associated with the Doppler red shift?

Another forum member (@Frobnicat, IIRC) ran some quick numbers to illustrate the miniscule gravitationally induced red shift, but I've been curious whether attenuation near a resonant cavity's cutoff frequency would be expected to manifest as purely a thermal loss phenomena.  There has been the occasional use of the phrase "event horizon" to describe the resonant cavity's cutoff frequency, which invoked a mental image of a possible non-thermal loss mechanism (although probably an error of my own doing due to my very limited understanding of GR).

Thanks,
James
Title: Re: EM Drive Developments - related to space flight applications - Thread 2
Post by: Mulletron on 05/09/2015 09:14 am
http://journals.aps.org/prl/abstract/10.1103/PhysRevLett.110.194301

Observation of Displacement Momentum in Normal and Chiral Dielectrics
G. L. J. A. Rikken and B. A. van Tiggelen
Phys. Rev. Lett. 110, 194301 – Published 10 May 2013

Would be nice if this were open access. Can't learn much from reading abstracts.

Hopefully in the future, open access to science becomes the norm.
Title: Re: EM Drive Developments - related to space flight applications - Thread 2
Post by: R.W. Keyes on 05/09/2015 10:12 am
Because Peltier effect can't quite reach cryogenic temperatures, for some thermodynamic reason I forgot. Maybe it changed with recent progress on the subject ? Please inquire : what are the limits of low temp. with Peltier effect, now and tomorrow ? Isn't the Peltier effect still quite low in efficiency ?

The Peltier limits are below 100K, which is enough for high-temparature superconductors. The efficiency in such temperature is low, but probably that could be economically viable because of much smaller MW input power and smaller superconducting engine size.

That said, I must sadly admit, that this engine won't probably move in free space. It may move on air cushion though, as in the experiments conducted so far on Earth.

Anyway, if so, then it is at least good for new generation of rotorless helicopters, here on Earth.

Stirling coolers are used where cryogenic temperatures are required.   They are more efficient than Peltier devices.   I have used very compact Stirling cooled IR detectors.   Several companies make them,   It takes about 1 Min. to reduce the temperature of the detector to 95 K, using 1 Watt.    However the thermal mass is very tiny.    NASA has been investigating Stirling coolers for liquifying rocket fuels (H2, O2) in space and for space telescope applications.

More efficient are coolers that utilize the Giant Magnetocaloric Effect (GME), the so-called 'solid state' refrigerators. In this device, a mass of one of several substances exhibiting the effect is placed into a magnetic field, where it heats up. This is allowed to cool to ambient temperature, and then moved into the area to cooled, and then the magnetic field removed, upon which the mass cools, and circulating coolant then creates thermal equilibrium in that system.

The problem with GME coolers it that each substance known the exhibit the GME does so over a narrow temperature range. These ranges vary, so it is possible to cascade stages with differing GME materials, but this adds complexity. I am of the believe that using a cryosystem technology with a greater range of temperatures should be used for the initial cooling, and then a single-stage GME is used to keep the temperature within its operating range.
Title: Re: EM Drive Developments - related to space flight applications - Thread 2
Post by: CW on 05/09/2015 11:03 am
One idea just popped into my mind:

As far as I know (and please correct me if I'm wrong!), any kind of particle decays after some more or less long time (that should include photons? Although that seems to be a controversy, from what I read so far).
Let's then say we do have an EM-drive that apparently draws momentum out of nothing. Now let the EM-drive inertially move forward with its 'magically' gained momentum from nothing for an eternity via inertia, without it being active for the whole time. When, at one point in time, all atoms of the former EM-drive start to decay (and in consequence all possible 'offspring' particles of those decays, after some other more or less long time).. I think that the following question arises:

Where does the gained momentum from nothing go to, when in the end all involved particles, that had more momentum than our current understanding permits, decay into nothing?

I think that logically the answer is in the question already. Momentum is simply taken from nothing, and inevitably returns to nothing (because that's just how physics works). The additional momentum is only temporarily borrowed from nothing (even if this means an eternity from our point of view) and added as a delta to the previous momentum of the particles. You just have to give it enough time for the momentum-return to happen - because it will happen automatically.. and there's no way around it. We're just too short-lived to observe it happen eventually.
;D
Title: Re: EM Drive Developments - related to space flight applications - Thread 2
Post by: SeeShells on 05/09/2015 11:40 am
I was wondering in your testing did you use a smoke stick also called a smoke pencil to check for air flows around the EM Drive in ambient air conditions?

That is a very good idea.  I don't think it has been suggested before.   There was some discussion on convective air flow and the possiblity it may explain the anomalous force measured by EW early in thread 1.   However interest in that explanation has dissipated and Dr. Rodal's analysis of thermal-mechanical effects as a conventional explanation has replaced it.
Thank you and I did review of the very nice workup by Dr. Rodal after he pointed it out.  It's not only heat convection I was thinking of, but any aberrations in air movement other than the expected thermal currents from the EM device.
Title: Re: EM Drive Developments - related to space flight applications - Thread 2
Post by: Rodal on 05/09/2015 02:52 pm
I was wondering in your testing did you use a smoke stick also called a smoke pencil to check for air flows around the EM Drive in ambient air conditions?

That is a very good idea.  I don't think it has been suggested before.   There was some discussion on convective air flow and the possiblity it may explain the anomalous force measured by EW early in thread 1.   However interest in that explanation has dissipated and Dr. Rodal's analysis of thermal-mechanical effects as a conventional explanation has replaced it.
Thank you and I did review of the very nice workup by Dr. Rodal after he pointed it out.  It's not only heat convection I was thinking of, but any aberrations in air movement other than the expected thermal currents from the EM device.

Quote from:  Jorge Ruiz de Santayana "George Santayana"
Those who cannot remember the past are condemned to repeat it.

1873 Maxwell derives equations showing that radiation will give rise to stresses on a surface due to the electromagnetic energy density (Maxwells' stress tensor).
 
1876 Bartoli attempts to measure the radiation pressure on a reflecting surface experimentally but he is unable to overcome disturbing effects due to the heating of the reflecting surface which gives rise to convection currents of air and to the radiometer effect ( https://en.wikipedia.org/wiki/Crookes_radiometer#Explanations_for_the_force_on_the_vanes ), which are collectively described by Bartoli and other scientists of the period, as "gas action."
 
1900 Lebedew, using light waves is the first person in history to succeed in eliminating these unwanted artifact effects by performing the experiments in a partial vacuum.  He used a torsion balance and a highly reflecting mirror in his measurements.

1902 Nichols and Hull made a thorough investigation of the unwanted artifact effects collectively called "gas action" and accurately establish the accuracy of Maxwell's predicted stress tensor.  Nichols and Hull also performed experiments in a partial vacuum using a torsion balance and a highly reflecting mirror in their measurements.

1949 Carrara and Lombardini qualitatively demonstrate the existence of radiation pressure at microwave frequencies to the correct order of magnitude, but no quantitative results are obtained.  They employ a free wave method, which involves a very difficult refraction problem due to air, which precludes a quantitative assessment.

1950 Cullen is the first person to accurately measure radiation pressure at microwave frequencies.  Cullen uses power ranging from 10 to 50 watts in his microwave pressure measurements, at a wavelength of 10 cm, measuring 6.77 microNewtons/kW.  Cullen used a torsion balance and a highly reflecting mirror in his measurements.  Cullen performed his experiments in ambient air conditions.  It was impossible to obtain a stable baseline, even on a relatively short-term basis of a minute's duration.  This continual drifting of the baseline was found to be due to air convection currents set up by small and changing temperature gradients within the microwave waveguides.  The remedy was to reduce the air resistance of the reflecting end plate so that the convection currents would have no appreciable effect.  The reflecting end plate was replaced by a system of concentric wire rings (shown on Fig. 12 of Cullen's paper).  The rings acted as an almost perfect reflector of the electromagnetic waves but at the same time had a small effective cross-section to air currents.  NASA, Shawyer, Yang, and other EM Drive researchers would be well advised to experiment with replacing the end plates of the EM Drive with this system of concentric rings, in order to address the problem of air convection currents that has plagued radiation pressure experiments in ambient conditions ever since Maxwell 140 years ago.  Even in a partial vacuum, if one uses for example bilayer plates of copper/glass-fiber-reinforced epoxy with the reinforced polymer on the external surface, there is the possibility of outgassing in a vacuum producing a false positive.  The use of a mesh precludes this problem both in ambient air conditions and in a vacuum.

(http://forum.nasaspaceflight.com/index.php?action=dlattach;topic=36313.0;attach=829135;image)

Attachment: ABSOLUTE POWER MEASUREMENT AT MICROWAVE FREQUENCIES
By A. L. CULLEN, Ph.D., B.Sc.(Eng.), Associate Member.
(published February, 1952.)

http://forum.nasaspaceflight.com/index.php?action=dlattach;topic=36313.0;attach=828862
Title: Re: EM Drive Developments - related to space flight applications - Thread 2
Post by: Rodal on 05/09/2015 04:35 pm
Quote from:  Hackaday.io EM Drive project
In the first step, we will replicate the original EM-Drive thruster.
It will be driven by a high power RF source (magnetron) which is easily available.
The second step will be a miniaturization by using higher frequencies.
To achieve this, a numerical simulation of the waves inside the cone frustum must be made to obtain the optimal geometry for the cone.
A high frequency generator in the 20-30 GHz range has to be built with the power of a few watts

https://hackaday.io/project/5596-em-drive
Title: Re: EM Drive Developments - related to space flight applications - Thread 2
Post by: jmossman on 05/09/2015 05:33 pm
I was wondering in your testing did you use a smoke stick also called a smoke pencil to check for air flows around the EM Drive in ambient air conditions?

That is a very good idea.  I don't think it has been suggested before.   There was some discussion on convective air flow and the possiblity it may explain the anomalous force measured by EW early in thread 1.   However interest in that explanation has dissipated and Dr. Rodal's analysis of thermal-mechanical effects as a conventional explanation has replaced it.
Thank you and I did review of the very nice workup by Dr. Rodal after he pointed it out.  It's not only heat convection I was thinking of, but any aberrations in air movement other than the expected thermal currents from the EM device.

Quote from:  Jorge Ruiz de Santayana "George Santayana"
Those who cannot remember the past are condemned to repeat it.

1873 Maxwell derives equations showing that radiation will give rise to stresses on a surface due to the electromagnetic energy density (Maxwells' stress tensor).
 
1876 Bartoli attempts to measure the radiation pressure on a reflecting surface experimentally but he is unable to overcome disturbing effects due to the heating of the reflecting surface which gives rise to convection currents of air and to the radiometer effect ( https://en.wikipedia.org/wiki/Crookes_radiometer#Explanations_for_the_force_on_the_vanes ), which are collectively described by Bartoli and other scientists of the period, as "gas action."
 
1900 Lebedew, using light waves is the first person in history to succeed in eliminating these unwanted artifact effects by performing the experiments in a partial vacuum.  He used a torsion balance and a highly reflecting mirror in his measurements.

1902 Nichols and Hull made a thorough investigation of the unwanted artifact effects collectively called "gas action" and accurately establish the accuracy of Maxwell's predicted stress tensor.  Nichols and Hull also performed experiments in a partial vacuum using a torsion balance and a highly reflecting mirror in their measurements.

1949 Carrara and Lombardini qualitatively demonstrate the existence of radiation pressure at microwave frequencies to the correct order of magnitude, but no quantitative results are obtained.  They employ a free wave method, which involves a very difficult refraction problem due to air, which precludes a quantitative assessment.

1950 Cullen is the first person to accurately measure radiation pressure at microwave frequencies.  Cullen uses power ranging from 10 to 50 watts in his microwave pressure measurements, at a wavelength of 10 cm, measuring 6.77 microNewtons/kW.  Cullen used a torsion balance and a highly reflecting mirror in his measurements.  Cullen performed his experiments in ambient air conditions.  It was impossible to obtain a stable baseline, even on a relatively short-term basis of a minute's duration.  This continual drifting of the baseline was found to be due to air convection currents set up by small and changing temperature gradients within the microwave waveguides.  The remedy was to reduce the air resistance of the reflecting end plate so that the convection currents would have no appreciable effect.  The reflecting end plate was replaced by a system of concentric wire rings (shown on Fig. 12 of Cullen's paper).  The rings acted as an almost perfect reflector of the electromagnetic waves but at the same time had a small effective cross-section to air currents.  NASA, Shawyer, Yang, and other EM Drive researchers would be well advised to experiment with replacing the end plates of the EM Drive with this system of concentric rings, in order to address the problem of air convection currents that has plagued radiation pressure experiments in ambient conditions ever since Maxwell 140 years ago.  Even in a partial vacuum, if one uses for example bilayer plates of copper/glass-fiber-reinforced epoxy with the reinforced polymer on the external surface, there is the possibility of outgassing in a vacuum producing a false positive.  The use of a mesh precludes this problem both in ambient air conditions and in a vacuum.

(http://forum.nasaspaceflight.com/index.php?action=dlattach;topic=36313.0;attach=829135;image)

Attachment: ABSOLUTE POWER MEASUREMENT AT MICROWAVE FREQUENCIES
By A. L. CULLEN, Ph.D., B.Sc.(Eng.), Associate Member.
(published February, 1952.)

http://forum.nasaspaceflight.com/index.php?action=dlattach;topic=36313.0;attach=828862

Dr. Rodal,

How much of effect would the use of a ring (or mesh) have on ohmic losses in the endplate for some of the excited modes tested at EW?  Any hunches for the effect on the cavity Q?  (i.e. aren't the losses going to be larger for the ring/mesh versus a solid sheet of copper?)

For in-air testing, the use of a ring/mesh makes perfect sense.  I'm just curious as to the predicted effect on Q  (perhaps neglible?).  Engineering is almost always a series of tradeoffs;  minimizing a known noise source during delicate force measurements would seem like a much more important design parameter than maximizing Q at this stage.

Thanks,
James
Title: Re: EM Drive Developments - related to space flight applications - Thread 2
Post by: Rodal on 05/09/2015 05:54 pm
....

Dr. Rodal,

How much of effect would the use of a ring (or mesh) have on ohmic losses in the endplate for some of the excited modes tested at EW?  Any hunches for the effect on the cavity Q?  (i.e. aren't the losses going to be larger for the ring/mesh versus a solid sheet of copper?)

For in-air testing, the use of a ring/mesh makes perfect sense.  I'm just curious as to the predicted effect on Q  (perhaps neglible?).  Engineering is almost always a series of tradeoffs;  minimizing a known noise source during delicate force measurements would seem like a much more important design parameter than maximizing Q at this stage.

Thanks,
James
Fair point.  A detailed discussion of the effect of the ring mesh is not trivial.  I refer you to Cullen's discussion of the effect on the pressure measurement (not taking into account Q, because Cullen tested an open waveguide) in Cullen's paper attached in my post above.  The effect of a ring mesh has been further understood during the past 65 years, thanks to great advances on numerical calculations. 

Given the fact that the highest measured thrust forces, and the highest measured thrust force/InputPower were obtained with the lowest Q reported (Q~1500 see my notes concerning this) by Prof. Yang in China, while high Q force measurements at NASA Eagleworks have resulted in much lower forces and force/InputPower, a proportional relationship between force and Q remains to be experimentally corroborated (it is actually negated by the Chinese experiments).

Furthermore, the theoretical considerations advanced by Todd in these pages put more emphasis on the attenuation (which may be supported by NASA Eagleworks findings that they only measured thrust forces with an insert polymer dielectric and no forces without it). 

Finally, it is not clear that the portion of the Q due to reflection of the low frequencies (larger wavelength) of the resonant microwave spectrum would be impaired by the mesh. 

My microwave oven has a conductive metal mesh on the inside of the transparent glass, so that the microwaves are reflected,  to prevent microwaves from escaping the microwave.

(http://makeitorfixit.com/wp-content/uploads/2014/01/MicrowaveDoor.jpg.jpg)

BOTTOM LINE: I would suggest for experimenters to try 4 different kinds of ends:

1) A solid reflecting end (copper or aluminum)

2) A conductive wire mesh

3) A transparent glass (transparent to microwaves)

4) An open end 


And compare the results.  Such tests would be very valuable both for scientific and engineering purposes to understand what is being measured.
Title: Re: EM Drive Developments - related to space flight applications - Thread 2
Post by: jmossman on 05/09/2015 07:15 pm
....

Dr. Rodal,

How much of effect would the use of a ring (or mesh) have on ohmic losses in the endplate for some of the excited modes tested at EW?  Any hunches for the effect on the cavity Q?  (i.e. aren't the losses going to be larger for the ring/mesh versus a solid sheet of copper?)

For in-air testing, the use of a ring/mesh makes perfect sense.  I'm just curious as to the predicted effect on Q  (perhaps neglible?).  Engineering is almost always a series of tradeoffs;  minimizing a known noise source during delicate force measurements would seem like a much more important design parameter than maximizing Q at this stage.

Thanks,
James
Fair point.  A detailed discussion of the effect of the ring mesh is not trivial.  I refer you to Cullen's discussion of the effect on the pressure measurement (not taking into account Q, because Cullen tested an open waveguide) in Cullen's paper attached in my post above.  The effect of a ring mesh has been further understood during the past 65 years, thanks to great advances on numerical calculations. 

Given the fact that the highest measured thrust forces, and the highest measured thrust force/InputPower were obtained with the lowest Q reported (Q~1500 see my notes concerning this) by Prof. Yang in China, while high Q force measurements at NASA Eagleworks have resulted in much lower forces and force/InputPower, a proportional relationship between force and Q remains to be experimentally corroborated (it is actually negated by the Chinese experiments).

Furthermore, the theoretical considerations advanced by Todd in these pages put more emphasis on the attenuation (which may be supported by NASA Eagleworks findings that they only measured thrust forces with an insert polymer dielectric and no forces without it). 

Finally, it is not clear that the portion of the Q due to reflection of the low frequencies (larger wavelength) of the resonant microwave spectrum would be impaired by the mesh. 

My microwave oven has a conductive metal mesh on the inside of the transparent glass, so that the microwaves are reflected,  to prevent microwaves from escaping the microwave.

(http://makeitorfixit.com/wp-content/uploads/2014/01/MicrowaveDoor.jpg.jpg)

BOTTOM LINE: I would suggest for experimenters to try 4 different kinds of ends:

1) A solid reflecting end (copper or aluminum)

2) A ring-wired mesh as used by Cullen

3) A transparent glass (transparent to microwaves)

4) An open end 


And compare the results.  Such tests would be very valuable both for scientific and engineering purposes to understand what is being measured.

Your recommendations make sense to me.  RLC (resistive, inductive, capacitive) mesh analysis of power grids within microchips is similarly non-trivial (correlating the numerical values of the model elements to a given fabrication technology being the most difficult).

Probably also worth reiterating that Q of a microwave cavity can be influenced by "losses" of various types (resistive losses, dielectric losses, geometry, antenna coupling, etc).  Not to mention the lingering unanswered question of whether the EM drive has a potentially novel "loss" mechanism to help explain the anomalous thrust.  :)

If EM drive anomalous thrust turns out not to be an experimental artifact, I suspect teasing out the different "losses" currently being lumped into Q measurement will eventually be important.  However, lowering the noise-floor (via use of a grid/mesh endcap) to ensure repeatable "thrust" measurement seems like a much higher priority right now for any potential replication attempt.

Thanks,
James
Title: Re: EM Drive Developments - related to space flight applications - Thread 2
Post by: deuteragenie on 05/09/2015 07:37 pm
Quote
...
1) A solid reflecting end (copper or aluminum)
2) A conductive wire mesh
3) A transparent glass (transparent to microwaves)
4) An open end 
And compare the results.  Such tests would be very valuable both for scientific and engineering purposes to understand what is being measured.

Apart for the conductive wire mesh it would seem to me that Shawyer likely tested the other ways (why would you use something when you can use nothing? etc.) and found out that the best results were with a solid reflecting end.  BTW as we do not have a theory, we do not know if "reflective" is the right property to look after. I would be interested to know what happens when the thickness of the "reflector" is increased / decreased. I would be equally interested to know what happens when the "reflector" is replaced by non reflective crystal.
Title: Re: EM Drive Developments - related to space flight applications - Thread 2
Post by: RotoSequence on 05/09/2015 08:54 pm
Quote
...
1) A solid reflecting end (copper or aluminum)
2) A conductive wire mesh
3) A transparent glass (transparent to microwaves)
4) An open end 
And compare the results.  Such tests would be very valuable both for scientific and engineering purposes to understand what is being measured.

Apart for the conductive wire mesh it would seem to me that Shawyer likely tested the other ways (why would you use something when you can use nothing? etc.) and found out that the best results were with a solid reflecting end.

We won't be doing ourselves any favors by making assumptions about Shawyer's undisclosed body of work.
Title: Re: EM Drive Developments - related to space flight applications - Thread 2
Post by: LasJayhawk on 05/09/2015 09:06 pm
I keep looking at this and thinking it's not right.

(http://i1365.photobucket.com/albums/r745/LASJayhawk/image.jpg1_zpszvfxogt9.jpg) (http://s1365.photobucket.com/user/LASJayhawk/media/image.jpg1_zpszvfxogt9.jpg.html)

It looks to symmetrical.

COMSOL's website says the RF module can do far field calculations, and the antenna placement would appear to put at least part of the frustum in the near field. It looks by eyeball that the software is only looking at the far field being reflecting off the small end of the frustum.

Or am I missing something?
Title: Re: EM Drive Developments - related to space flight applications - Thread 2
Post by: RotoSequence on 05/09/2015 09:19 pm
I keep looking at this and thinking it's not right.

It looks to symmetrical.

COMSOL's website says the RF module can do far field calculations, and the antenna placement would appear to put at least part of the frustum in the near field. It looks by eyeball that the software is only looking at the far field being reflecting off the small end of the frustum.

Or am I missing something?

I don't have the link handy, but infrared imagery of the Eagleworks large diameter end plate did verify the thermal behavior of resonance modes predicted in COMSOL analysis.
Title: Re: EM Drive Developments - related to space flight applications - Thread 2
Post by: Rodal on 05/09/2015 09:33 pm
I keep looking at this and thinking it's not right.

(http://i1365.photobucket.com/albums/r745/LASJayhawk/image.jpg1_zpszvfxogt9.jpg) (http://s1365.photobucket.com/user/LASJayhawk/media/image.jpg1_zpszvfxogt9.jpg.html)

It looks to symmetrical.

COMSOL's website says the RF module can do far field calculations, and the antenna placement would appear to put at least part of the frustum in the near field. It looks by eyeball that the software is only looking at the far field being reflecting off the small end of the frustum.

Or am I missing something?

This COMSOL Finite Element Analysis solution is a numerical solution of Maxwell's differential equations, taking into account the boundary conditions, including the losses responsible for the finite Q.  What you are looking at is the steady state of the electromagnetic fields.

The steady state electromagnetic field solution are standing waves.  Although the initial condition is not symmetrical, due to travelling waves, this is a very-short-lived transient, as the solution soon reaches a (practically) symmetric steady state. 

This has been shown by @aero with a very interesting movie based on a 2D solution of the truncated cone as a flat trapezium using MEEP which is a Finite Difference code (free from MIT alumni) that performs the full transient solution.  It was neat to see how the (practically) symmetric steady state was soon reached starting from an unsymmetric initial condition.

Furthermore, as pointed out by RotoSequence, the steady state solution for the magnetic field and a COMSOL thermal analysis was corroborated by temperature measurements using an infrared thermal camera, which verifies that the heating is due to induction heating from the magnetic field.

I obtained an exact solution for the symmetric steady state (which of course does not take into account the initial unsymmetric transient) and it fully verifies NASA's COMSOL FEA steady state solution for the electromagnetic field: the natural frequency is within 1% of the exact solution and the mode shapes are extremely close (I posted the exact solution comparison some time ago).

If one is interested in near-field far-field, transient, fully complex solution (including initial travelling waves morphing into standing waves, as well as evanescent waves) then one has to resort to a time-marching solution as with MEEP finite difference approach or a [FEA in space/FD in time] solution that imposes a finite element discretization in space and a finite difference time discretization.   (Such a transient, 3D solution containing evanescent waves is extremely time consuming.)
Title: Re: EM Drive Developments - related to space flight applications - Thread 2
Post by: deltaMass on 05/09/2015 09:54 pm
@Rodal
Have you posted your exact solution code here?
I have my Mathematica fired up and ready to go!
Title: Re: EM Drive Developments - related to space flight applications - Thread 2
Post by: Rodal on 05/09/2015 09:58 pm
@Rodal
Have you posted your exact solution code here?
I have my Mathematica fired up and ready to go!
No I only posted the graphic results and numerical comparisons.  I also posted the output graphics for the Poynting vector (I have not seen anyone else do that, including Greg Egan).  Greg Egan posted a solution for constant in the transverse (aximuthal) direction modes, but arbitrary variation in the other directions . Greg Egan's solution will not address the Cyl TM212 being shown above, but it does address other modes like Cyl TE012 that have been discussed, see: http://gregegan.customer.netspace.net.au/SCIENCE/Cavity/Cavity.html

I write "Cyl" because there is no convention on how to denominate the truncated cone mode quantum numbers, and what NASA Eagleworks has done is use the same convention as the closest mode shape for the cylinder with uniform cross-section.
Title: Re: EM Drive Developments - related to space flight applications - Thread 2
Post by: deltaMass on 05/09/2015 10:00 pm
@Rodal
Have you posted your exact solution code here?
I have my Mathematica fired up and ready to go!
No I only posted the graphic results.  Greg Egan posted a solution for constant in the transverse (aximuthal) direction modes, but arbitrary variation in the other directions . Greg Egan's solution will not address the Cyl TM212 being shown above, but it does address other modes like TE012 that have been discussed, see: http://gregegan.customer.netspace.net.au/SCIENCE/Cavity/Cavity.html
Can you possibly post your Mathematica notebook?

What I'm looking for is a frequency prediction of various modes as I vary the dimensions and frustum angle.
Title: Re: EM Drive Developments - related to space flight applications - Thread 2
Post by: deltaMass on 05/09/2015 10:09 pm
And while on the subject of simulation: Guido Fetta of Cannae tells me that his COMSOL(?unsure) sim predicts a nonzero net Lorentz force for his device. Now, we all learned that no closed system of currents can produce such a net force. There's a paradox. He insists that there are no significant cumulative rounding errors.

Anyone have insight into this?
Title: Re: EM Drive Developments - related to space flight applications - Thread 2
Post by: Rodal on 05/09/2015 10:45 pm
And while on the subject of simulation: Guido Fetta of Cannae tells me that his COMSOL(?unsure) sim predicts a nonzero net Lorentz force for his device. Now, we all learned that no closed system of currents can produce such a net force. There's a paradox. He insists that there are no significant cumulative rounding errors.

Anyone have insight into this?

There cannot be a paradox.

Let's remember that the COMSOL FEA solution is the steady state solution showing the spatial distribution of the field.  Remember that the steady state solution of Maxwell's differential equations can be accomplished by separation of variables.

 The harmonic (time varying) part of the field is assumed. So, for example, the Magnetic Field shown on the COMSOL output is the spatial distribution of the magnetic field.  Now, what is shown as a maximum and what is shown as a minimum is arbitrary, since depending at what time one arbitrarily chooses to display the magnetic field, as the magnetic field varies with time like a harmonic function.

Similarly, the Poynting vector is a harmonic function of time, and this is, as you point out, well known in the literature, with a frequency which is twice the frequency of the magnetic and the electric field.

Although the spatial distribution of the Poynting vector is non-zero at arbitrary points in time, over a whole cycle the Poynting vector (and the Lorentz force) for a cavity sums up to exactly zero, just like the mean of the magnetic and electric fields is also zero.  COMSOL is an excellent package.

The Poynting vector solution of Maxwell's equations points towards the Big Base half of the time, and points towards the Small Base half of the time.

COMSOL will not tell that to the analyst obtaining a steady solution where the harmonic function of time is implicit. 
It is recommended that COMSOL and any other FEA  packages (ANSYS MultiPhysics, etc., ABAQUS, ADINA, NASTRAN) should be run by experienced FEA analysts, to prevent errors.  (Ditto for FD, control volume , and any other numerical packages).
Title: Re: EM Drive Developments - related to space flight applications - Thread 2
Post by: bprager on 05/09/2015 10:52 pm
...
BOTTOM LINE: I would suggest for experimenters to try 4 different kinds of ends:

1) A solid reflecting end (copper or aluminum)

2) A conductive wire mesh

3) A transparent glass (transparent to microwaves)

4) An open end 


And compare the results.  Such tests would be very valuable both for scientific and engineering purposes to understand what is being measured.

For completeness, should we also consider:

1a) shaped end plates with narrow RF modulation and

1b) plane end plates with wide modulation?
Title: Re: EM Drive Developments - related to space flight applications - Thread 2
Post by: deltaMass on 05/09/2015 10:55 pm
And while on the subject of simulation: Guido Fetta of Cannae tells me that his COMSOL(?unsure) sim predicts a nonzero net Lorentz force for his device. Now, we all learned that no closed system of currents can produce such a net force. There's a paradox. He insists that there are no significant cumulative rounding errors.

Anyone have insight into this?
Yes.  Let's remember that the COMSOL FEA solution is the steady state solution showing the spatial distribution of the field.  The harmonic (time varying) part of the field is assumed. So, for example, the Magnetic Field shown on the COMSOL output is the spatial distribution of the magnetic field.  Now, what is shown as a maximum and what is shown as a minimum is arbitrary, since depending at what time one arbitrarily chooses to display the magnetic field, as the magnetic field varies with time like a harmonic function.

Similarly, the Poynting vector is a harmonic function of time, as you point out, well known in the literature, with a frequency which is twice the frequency of the magnetic and the electric field.

Although the spatial distribution of the Poynting vector is non-zero at arbitrary points in time, over a whole cycle it sums up to exactly zero, just like the mean of the magnetic and electric fields is also zero.

The Poynting vector solution of Maxwell's equations points towards the Big Base half of the time, and points towards the Small Base half of the time.

COMSOL will not tell that to the analyst.  The  FEA analyst is supposed to know that.

It is recommended that COMSOL and any other packages (ANSYS MultiPhysics, etc., ABAQUS, ADINA, NASTRAN) should be run by experience FEA analysts.
Do you know for a fact that Fetta made this elementary blunder, or are you just hypothesising?
Title: Re: EM Drive Developments - related to space flight applications - Thread 2
Post by: Rodal on 05/09/2015 11:03 pm
...
Do you know for a fact that Fetta made this elementary blunder, or are you just hypothesising?
I don't know the gentleman, and I have no idea as to what he may or may not have done, may have said or may not have said.

I was only and innocently answering your question based solely on the basis of my knowledge and experience of FEA and COMSOL and the fact that the correct solution of Maxwell's equations for a cavity of any shape precludes a non-zero force over a whole period.

This follows logically from the fact that a Finite Element solution performs a finite element discretization in space, not a finite element discretization in time.  For a steady state solution, the harmonic variation in time is assumed and not displayed.  For a transient solution different packages use different methods, usually a fiinite difference discretization in time for example (as the time variation is one-dimensional).

COMSOL is an excellent Finite Element package.

If you know the gentleman you may ask him (if interested) exactly what he did and what was the time variation he obtained for the calculated force, and if the variation is not harmonic, how did he obtain a non-harmonic variation using COMSOL.  Obviously, I can't answer for him.
Title: Re: EM Drive Developments - related to space flight applications - Thread 2
Post by: deltaMass on 05/09/2015 11:14 pm
Thanks. And we are in violent agreement about the consequences of applying Maxwell.
Title: Re: EM Drive Developments - related to space flight applications - Thread 2
Post by: LasJayhawk on 05/09/2015 11:42 pm
Thanks. And we are in violent agreement about the consequences of applying Maxwell.

How about his friend, the demon?

If I pick my waveleghth, design my cavity, and antenna placement, then adjust the phase of the signal such that one end is in a node and the other is riding a crest, both all the time all the time, I would have an unbalance between the two ends???
Title: Re: EM Drive Developments - related to space flight applications - Thread 2
Post by: deltaMass on 05/10/2015 12:14 am
Thanks. And we are in violent agreement about the consequences of applying Maxwell.

How about his friend, the demon?

If I pick my waveleghth, design my cavity, and antenna placement, then adjust the phase of the signal such that one end is in a node and the other is riding a crest, both all the time all the time, I would have an unbalance between the two ends???
Yes, but not the sort that is supposed to matter according to Maxwell.

I like the idea of an antenna to avoid the potential measurement artifacts of hooking up a coaxial cable feed. Unfortunately, the practicality of beaming sufficient far field power to an antenna is not great. And even if this sufficed, there'd be RF flying everywhere messing things up.

Really and truly, this experiment cries out for a space test.
Title: Re: EM Drive Developments - related to space flight applications - Thread 2
Post by: Rodal on 05/10/2015 12:21 am
....

Really and truly, this experiment cries out for a space test.
There are several people that think the same way (that it cries out for a space test), while there are several other ones that think it would be premature.  Since you made a powerful argument for the energy paradox, it would be useful if you could list all the reasons (and what and how should be tested) why a space test should be the next step, as a powerful argument in that direction may help to push the ball rolling...upwards  :)
Title: Re: EM Drive Developments - related to space flight applications - Thread 2
Post by: PaulF on 05/10/2015 12:59 am
....

Really and truly, this experiment cries out for a space test.
There are several people that think the same way (that it cries out for a space test), while there are several other ones that think it would be premature.  Since you made a powerful argument for the energy paradox, it would be useful if you could list all the reasons (and what and how should be tested) why a space test should be the next step, as a powerful argument in that direction may help to push the ball rolling...upwards  :)
If enough people here are solidary, we could petition directly to Elon Musk for a ride on a dragon slated for the ISS, and see what he has to say. As collateral we could offer him all the research that has been done and collected by the fine gentlemen and ladies here. That is, IF the device actually works in space. But Elon Musk likes taking chances, that's what made him a billionaire.
Title: Re: EM Drive Developments - related to space flight applications - Thread 2
Post by: Notsosureofit on 05/10/2015 01:01 am
FYI   Arghhhh!

Wasted this rare free day chasing constant acceleration transforms til I remembered my own hypothesis is based on negative feedback of the acceleration.  (shows what can happen once the bit is in the mouth)

Time for a hot tub...
Title: Re: EM Drive Developments - related to space flight applications - Thread 2
Post by: deltaMass on 05/10/2015 01:19 am
....

Really and truly, this experiment cries out for a space test.
There are several people that think the same way (that it cries out for a space test), while there are several other ones that think it would be premature.  Since you made a powerful argument for the energy paradox, it would be useful if you could list all the reasons (and what and how should be tested) why a space test should be the next step, as a powerful argument in that direction may help to push the ball rolling...upwards  :)
I am happy that you found my position on the energy paradox powerful. But speaking of premature, some further mulling and reading - ongoing - tells me that there may well be a third possibility for the behaviour of the thrust over time. If this "third thrust scenario" turns out to be the correct one, then the EmDrive may not be useful for any kind of propulsion, and would at best remain as an interesting test framework for various physics variants.

A third thrust scenario
Again we assume that the measured thrust is actual. We note its two characteristics:
a) It is measured as a static force.
b) It remains constant as long as input power is applied to this static configuration.
'a)' implies that we can only theorise as to its dynamic behaviour - i.e. when the EmDrive moves over time.

I take together two further pieces of information:
1. Shawyer's video of the moving EmDrive
2. Mike McCulloch's MiHsC theory of operation.
Now of course, there are a variety of interpretations possible for both of these. For example, that the video is flawed because all the angular momentum is being supplied by imperfections in the air bearing. Or that Mike's theory is nonsense because it violates GR. And so forth. So here I have to decide what to assume so as to justify this 3rd scenario. So here I choose:
1. The vid shows that an impulse is produced which results in a real and constant momentum of the EmDrive.
2. The theory predicts the same thing - constant forward EmDrive momentum.

And so this third thrust scenario is this:
As soon as the EmDrive is free to move, a definite and constant momentum is established and its thrust falls to zero.

So far, so good. But given this third thrust scenario is the correct one, what then is expected to happen when we switch off the power?
Title: Re: EM Drive Developments - related to space flight applications - Thread 2
Post by: Rodal on 05/10/2015 01:34 am
...
And so this third thrust scenario is this:
As soon as the EmDrive is free to move, a definite and constant momentum is established and its thrust falls to zero.

So far, so good. But given this third thrust scenario is the correct one, what then is expected to happen when we switch off the power?
How do you reconcile this with the NASA Tests showing an impulsive force, in the same direction as the movement, which once it reaches the knee of the uprise, after ~2 sec it either stays fairly constant or continues increasing at a much smaller rate?.  None of the NASA tests showed a Dirac delta function type of response. None of the NASA tests showed the force decreasing to zero once the EM Drive started to move, on the contrary, the force stayed constant or it increased.
Title: Re: EM Drive Developments - related to space flight applications - Thread 2
Post by: deltaMass on 05/10/2015 01:36 am
I'll attempt to answer my own question, using the same set of assumptions as before. Momentum is a signed quantity, and we have to respect the symmetry of the situation. Therefore, when the power is switched off, an equal and opposite momentum is newly established, resulting in the EmDrive coming to rest.
Title: Re: EM Drive Developments - related to space flight applications - Thread 2
Post by: Rodal on 05/10/2015 01:37 am
FYI   Arghhhh!

Wasted this rare free day chasing constant acceleration transforms til I remembered my own hypothesis is based on negative feedback of the acceleration.  (shows what can happen once the bit is in the mouth)

Time for a hot tub...

“Einstein box” Gedanken experiment first proposed by Balazs

N. L. Balazs, “The energy-momentum tensor of the electromagnetic field inside matter,” Phys. Rev. 91, 408-411
(1953).

for the system’s center of mass to be in the same place in both experiments, it is necessary for the slab in the latter case to have shifted to the right. The difference between the free-space momentum of the pulse and its electromagnetic (or Abraham) momentum is thus transferred to the slab in the form of mechanical momentum, pM, causing the slab’s eventual displacement in a manner consistent with the demands of the Einstein box experiment.

http://bit.ly/1DZl2z6

Resolution of the Abraham-Minkowski Controversy
Masud Mansuripur
College of Optical Sciences, The University of Arizona, Tucson, Arizona 85721
[Published in Optics Communications 283, 1997-2005 (2010)]

The dielectric insert may be acting as the slab.  That produces a solid movement towards the small end according to Balazs, which is what NASA observes.  However, the small end of the EM Drive is not open, so doesn't the wave reflect on the small end's copper surface and therefore enters the dielectric again now heading in the direction towards the big end which produces a movement towards the big base? back and forth?

(http://forum.nasaspaceflight.com/index.php?action=dlattach;topic=36313.0;attach=635478;image)
Title: Re: EM Drive Developments - related to space flight applications - Thread 2
Post by: deltaMass on 05/10/2015 01:39 am
...
And so this third thrust scenario is this:
As soon as the EmDrive is free to move, a definite and constant momentum is established and its thrust falls to zero.

So far, so good. But given this third thrust scenario is the correct one, what then is expected to happen when we switch off the power?
How do you reconcile this with the NASA Tests showing an impulsive force, in the same direction as the movement, which once it reaches the knee of the uprise, after ~2 sec it either stays fairly constant or continues increasing at a much smaller rate?.  None of the NASA tests showed a Dirac delta function type of response. None of the NASA tests showed the force decreasing to zero once the EM Drive started to move, on the contrary, the force stayed constant or it increased.
I'm not aware of any NASA tests on a moving EmDrive. Refs?
As for the static force profile, the small residual increase appears to have a slope that matches, to within experimental accuracy, the prevailing thermal drift.
Title: Re: EM Drive Developments - related to space flight applications - Thread 2
Post by: Rodal on 05/10/2015 01:43 am
..
I'm not aware of any NASA tests on a moving EmDrive. Refs?
As for the static force profile, the small residual increase appears to have a slope that matches, to within experimental accuracy, the prevailing thermal drift.

NASA is measuring a displacement of the EM Drive vs. time, which means that the EM Drive is moving.  This is true for all the NASA experiments of the truncated cone EM Drive.

The motion is a rigid body rotation of the EM Drive around the center of the torque pendulum, and the force on the EM Drive is reacted by the torsional stiffness of the torque pendulum.

References:

1) Anomalous Thrust Production from an RF Test Device Measured on a Low-Thrust Torsion Pendulum (Brady et al)
AIAA 2014-4029

2) NSF EM Drive threads 1 and 2
Title: Re: EM Drive Developments - related to space flight applications - Thread 2
Post by: deltaMass on 05/10/2015 02:28 am
Briefly moving, yes, but constrained from free dynamics - which is what I'm attempting to address in all three thrust scenarios. If the torque balance shows a rectangular pulse (which it largely does, up to thermal drift), this indicates the prompt onset of a constant force. But this arrangement by no means guarantees that the appropriate momentum, per the third thrust scenario, has been established. In fact it guarantees that it has not, since as soon as the torque balances the thrust, the balance ceases to turn and the momentum is zero. And we're back to a situation tantamount to the EmDrive pushing against an immovable wall - i.e. fully static.

Recall the third thrust scenario says that the thrust only falls to zero after the appropriate momentum is established.
Title: Re: EM Drive Developments - related to space flight applications - Thread 2
Post by: Rodal on 05/10/2015 02:37 am
Briefly moving, yes, but constrained from free dynamics - which is what I'm attempting to address in all three thrust scenarios. If the torque balance shows a rectangular pulse (which it largely does, up to thermal drift), this indicates the prompt onset of a constant force. But this arrangement by no means guarantees that the appropriate momentum, per the third thrust scenario, has been established. In fact it guarantees that it has not, since as soon as the torque balances the thrust, the balance ceases to turn and the momentum is zero. And we're back to a situation tantamount to the EmDrive pushing against an immovable wall - i.e. fully static.

Recall the third thrust scenario says that the thrust only falls to zero after the appropriate momentum is established.
free dynamics ?

Both Shawyer's rotating EM Drive (in his video) and NASA's EM Drive are under the action of the Earth's gravity, a force which is balanced by the support in both cases.
Both encounter an opposing force: the stiffness of the torsional pendulum in NASA's case, and the friction of the air bearing in Shawyer's case.


How do you define what freely moves and what doesn't freely move?
Is there a velocity threshold? Does it have a preferred frame of reference?
Is there an acceleration threshold?
Is there a threshold for what the opposing, resisting force should be ? (torsional stiffness of the pendulum or frictional resistance of the bearing)
Title: Re: EM Drive Developments - related to space flight applications - Thread 2
Post by: deltaMass on 05/10/2015 02:51 am
I really don't know where you're going with that lot. All I am saying is that the torque balance does not allow a constant momentum of the EmDrive to be established and retained.

And in that sense its dynamics are not free.
Title: Re: EM Drive Developments - related to space flight applications - Thread 2
Post by: Notsosureofit on 05/10/2015 02:54 am
FYI   Arghhhh!

Wasted this rare free day chasing constant acceleration transforms til I remembered my own hypothesis is based on negative feedback of the acceleration.  (shows what can happen once the bit is in the mouth)

Time for a hot tub...

“Einstein box” Gedanken experiment first proposed by Balazs

N. L. Balazs, “The energy-momentum tensor of the electromagnetic field inside matter,” Phys. Rev. 91, 408-411
(1953).

for the system’s center of mass to be in the same place in both experiments, it is necessary for the slab in the latter case to have shifted to the right. The difference between the free-space momentum of the pulse and its electromagnetic (or Abraham) momentum is thus transferred to the slab in the form of mechanical momentum, pM, causing the slab’s eventual displacement in a manner consistent with the demands of the Einstein box experiment.

http://bit.ly/1DZl2z6

Resolution of the Abraham-Minkowski Controversy
Masud Mansuripur
College of Optical Sciences, The University of Arizona, Tucson, Arizona 85721
[Published in Optics Communications 283, 1997-2005 (2010)]

The dielectric insert may be acting as the slab.  That produces a solid movement towards the small end according to Balazs, which is what NASA observes.  However, the small end of the EM Drive is not open, so doesn't the wave reflect on the small end's copper surface and therefore enters the dielectric again now heading in the direction towards the big end which produces a movement towards the big base? back and forth?

(http://forum.nasaspaceflight.com/index.php?action=dlattach;topic=36313.0;attach=635478;image)

Yes, I am aware of those arguments, and as you can see, there is no internal arrangement of dielectric and/or absorbers that will not integrate to zero w/ those forces.

The only common thing I see between the photon directions is that they both see the gradient.  So, is the photon smart enough to distinguish between the cavity induced gradient and an acceleration induced one ?

Maybe, maybe not, still an open question.
Title: Re: EM Drive Developments - related to space flight applications - Thread 2
Post by: zellerium on 05/10/2015 05:16 am

Kurt,

I wish I knew the type of interface to use. I am not an expert in microwave components, but I'm researching as time permits. Also, at 2.4503 GHz, it's going to be pretty large. Did you calculate the wave velocity for that mode?

Todd

Todd:

Apologies for the late response, school has been taking quite a lot of time...

The group velocity for a box width of 7.07 cm would be 1.5 E8 m/s. Unfortunately I am not very experienced with antennas either and don't know how to store energy in the cavity while simultaneously injecting it into the frustum. I'll keep researching and discuss with a few professors to gain some more insight.


Zellerium

May I suggest using a curved mirror as it results in greater relational movement of the laser dot. You may find them in some hardware shops? or some womens makeup departments according to my daughter.

At a minimum you will need two strings, one for front, one for rear of the unit. This will minimise sideways /rotational translation which would mask effects.  Keep each string separate and attach each to the roof.

If you still get sideways torque/momentum due to induced mechanical events derived from the switching on/off of the magnetron etc then you may require 4 strings, one on each "corner" of the unit.


Arc:

A curved mirror sounds like a good solution. I will have to do some calculations to determine what curvature is appropriate.
 
As for balancing the pendulum, two strings will probably be sufficient. I imagine hanging the frame will be a challenge, but luckily we will have plenty of time to figure out the little details once we start constructing. We will start construction mid June and will be experimenting until September. We have no other obligations and I have heard no one else needs the vacuum chamber over the summer.   

Does anyone have any recommendations for the thickness of sheet metal to buy?
Clearly we want something structurally sound that won't warp very much when heated, but we also want minimal mass on the pendulum.
I think 1/16 inch will be a good balance. I am planning to buy enough metal to make a copper and aluminum frustum and hopefully have enough copper leftover to try the separate symmetric cavity connected to the aluminum frustum. I'll also buy some PTFE and maybe HDPE as well.

Kurt Zeller
Title: Re: EM Drive Developments - related to space flight applications - Thread 2
Post by: WarpTech on 05/10/2015 07:10 am
....

Really and truly, this experiment cries out for a space test.
There are several people that think the same way (that it cries out for a space test), while there are several other ones that think it would be premature.  Since you made a powerful argument for the energy paradox, it would be useful if you could list all the reasons (and what and how should be tested) why a space test should be the next step, as a powerful argument in that direction may help to push the ball rolling...upwards  :)
I am happy that you found my position on the energy paradox powerful. But speaking of premature, some further mulling and reading - ongoing - tells me that there may well be a third possibility for the behaviour of the thrust over time. If this "third thrust scenario" turns out to be the correct one, then the EmDrive may not be useful for any kind of propulsion, and would at best remain as an interesting test framework for various physics variants.

A third thrust scenario
Again we assume that the measured thrust is actual. We note its two characteristics:
a) It is measured as a static force.
b) It remains constant as long as input power is applied to this static configuration.
'a)' implies that we can only theorise as to its dynamic behaviour - i.e. when the EmDrive moves over time.

I take together two further pieces of information:
1. Shawyer's video of the moving EmDrive
2. Mike McCulloch's MiHsC theory of operation.
Now of course, there are a variety of interpretations possible for both of these. For example, that the video is flawed because all the angular momentum is being supplied by imperfections in the air bearing. Or that Mike's theory is nonsense because it violates GR. And so forth. So here I have to decide what to assume so as to justify this 3rd scenario. So here I choose:
1. The vid shows that an impulse is produced which results in a real and constant momentum of the EmDrive.
2. The theory predicts the same thing - constant forward EmDrive momentum.

And so this third thrust scenario is this:
As soon as the EmDrive is free to move, a definite and constant momentum is established and its thrust falls to zero.

So far, so good. But given this third thrust scenario is the correct one, what then is expected to happen when we switch off the power?

Have you tried this one yet?

P = F*v = m*a*v

P/a = m*v = constant

m*a = - v*dm/dt

Lose mass at the same rate at which the remaining mass gains velocity, then the ratio of acceleration to power will remain constant, while there is mass remaining to lose. Total Energy can never go over unity because by definition, the momentum is constant and it's losing mass. Any paradox here?

Todd D.
Title: Re: EM Drive Developments - related to space flight applications - Thread 2
Post by: KittyMoo on 05/10/2015 07:19 am
Yes. It is not a propellant-less thruster anymore.
Title: Re: EM Drive Developments - related to space flight applications - Thread 2
Post by: WarpTech on 05/10/2015 07:32 am
Yes. It is not a propellant-less thruster anymore.

Capacitors and batteries lose mass when they are discharged.
Title: Re: EM Drive Developments - related to space flight applications - Thread 2
Post by: Stormbringer on 05/10/2015 08:33 am
hey; these table top experiments are getting out of hand:  http://phys.org/news/2015-05-dark-energy-theories-table.html 
Title: Re: EM Drive Developments - related to space flight applications - Thread 2
Post by: Mulletron on 05/10/2015 11:14 am
I keep looking at this and thinking it's not right.

(http://i1365.photobucket.com/albums/r745/LASJayhawk/image.jpg1_zpszvfxogt9.jpg) (http://s1365.photobucket.com/user/LASJayhawk/media/image.jpg1_zpszvfxogt9.jpg.html)

It looks to symmetrical.

COMSOL's website says the RF module can do far field calculations, and the antenna placement would appear to put at least part of the frustum in the near field. It looks by eyeball that the software is only looking at the far field being reflecting off the small end of the frustum.

Or am I missing something?

This COMSOL Finite Element Analysis solution is a numerical solution of Maxwell's differential equations, taking into account the boundary conditions, including the losses responsible for the finite Q.  What you are looking at is the steady state of the electromagnetic fields.

The steady state electromagnetic field solution are standing waves.  Although the initial condition is not symmetrical, due to travelling waves, this is a very-short-lived transient, as the solution soon reaches a (practically) symmetric steady state. 

This has been shown by @aero with a very interesting movie based on a 2D solution of the truncated cone as a flat trapezium using MEEP which is a Finite Difference code (free from MIT alumni) that performs the full transient solution.  It was neat to see how the (practically) symmetric steady state was soon reached starting from an unsymmetric initial condition.

Furthermore, as pointed out by RotoSequence, the steady state solution for the magnetic field and a COMSOL thermal analysis was corroborated by temperature measurements using an infrared thermal camera, which verifies that the heating is due to induction heating from the magnetic field.

I obtained an exact solution for the symmetric steady state (which of course does not take into account the initial unsymmetric transient) and it fully verifies NASA's COMSOL FEA steady state solution for the electromagnetic field: the natural frequency is within 1% of the exact solution and the mode shapes are extremely close (I posted the exact solution comparison some time ago).

If one is interested in near-field far-field, transient, fully complex solution (including initial travelling waves morphing into standing waves, as well as evanescent waves) then one has to resort to a time-marching solution as with MEEP finite difference approach or a [FEA in space/FD in time] solution that imposes a finite element discretization in space and a finite difference time discretization.   (Such a transient, 3D solution containing evanescent waves is extremely time consuming.)

Highlighted above in red. This very short lived transient is the key to everything. The aspect of this that we need to find ways to exploit further.

And while on the subject of simulation: Guido Fetta of Cannae tells me that his COMSOL(?unsure) sim predicts a nonzero net Lorentz force for his device. Now, we all learned that no closed system of currents can produce such a net force. There's a paradox. He insists that there are no significant cumulative rounding errors.

Anyone have insight into this?

There cannot be a paradox.

Let's remember that the COMSOL FEA solution is the steady state solution showing the spatial distribution of the field.  Remember that the steady state solution of Maxwell's differential equations can be accomplished by separation of variables.

 The harmonic (time varying) part of the field is assumed. So, for example, the Magnetic Field shown on the COMSOL output is the spatial distribution of the magnetic field.  Now, what is shown as a maximum and what is shown as a minimum is arbitrary, since depending at what time one arbitrarily chooses to display the magnetic field, as the magnetic field varies with time like a harmonic function.

Similarly, the Poynting vector is a harmonic function of time, and this is, as you point out, well known in the literature, with a frequency which is twice the frequency of the magnetic and the electric field.

Although the spatial distribution of the Poynting vector is non-zero at arbitrary points in time, over a whole cycle the Poynting vector (and the Lorentz force) for a cavity sums up to exactly zero, just like the mean of the magnetic and electric fields is also zero.  COMSOL is an excellent package.

The Poynting vector solution of Maxwell's equations points towards the Big Base half of the time, and points towards the Small Base half of the time.

COMSOL will not tell that to the analyst obtaining a steady solution where the harmonic function of time is implicit. 
It is recommended that COMSOL and any other FEA  packages (ANSYS MultiPhysics, etc., ABAQUS, ADINA, NASTRAN) should be run by experienced FEA analysts, to prevent errors.  (Ditto for FD, control volume , and any other numerical packages).

@deltaMass, you're on the right track. We've shown that the utility of COMSOL in figuring out this problem is limited as COMSOL is considering the standing wave and isn't considering the traveling wave. It gives no consideration to what is happening over extended time (over multiple full cycles) or to what is happening to a resonant cavity under dynamic operation, for example while under acceleration. COMSOL provides no insight as to what is happening when the resonant cavity is being excited by FM or other sources of phase noise. Building off the comments about COMSOL's ability to simulate mode shapes..that was proven to be accurate by Paul March's thermal shots here:

http://forum.nasaspaceflight.com/index.php?topic=36313.msg1327406#msg1327406
See Thermal IR comparison.pdf

Quoting Rodal and source is Star-drive comments:
Source:http://forum.nasaspaceflight.com/index.php?topic=36313.msg1370374#msg1370374
Quote
It is my understanding, from what Paul March wrote, that the main controlling parameter in determining the thrust generation performance of the EM-Drive is the rate of phase modulation of the RF signal that is injected into the resonant cavity. 

This requires an FM modulated signal of around 100 kHz deviation that dithers back and forth around the resonant cavity's resonant frequency as fast as possible. 

That main controlling parameter should not be taken lightly.

See Mr. Shawyer's comments in the screenshot about how the standing wave is constructed by addition of a traveling wave which is IN PHASE with the standing wave. Those of us who regularly work with electronics know about constructive and destructive interference, and this mechanism applied here is what puts energy into and takes energy out of the standing wave within the cavity.

If I present a signal at the loop probe (or dipole or waveguide slot) which is in phase with the standing wave, energy will couple into the cavity. If I present a signal at the loop probe (or dipole or waveguide slot) which is out of phase with the standing wave, energy will couple out of the cavity. This works both ways. The way Paul March mentioned, and what Shawyer mentions as motor/generator mode.

We can create the effects by the above method that mimic what a photon would experience in an accelerating box. This box is a quantum harmonic oscillator which can only resonate at precise discrete frequencies. This all reminds me of @Notsosureofit's AFR thrust equation.

http://usersguidetotheuniverse.com/?p=2865
http://physics.stackexchange.com/questions/78119/what-are-good-examples-to-demonstrate-einsteins-mass-energy-relation (scroll down)
I'm linking to these because they contain useful math. Rodal likes math. ;)

Quote
COMSOL is an excellent package.
Sure, but I wouldn't use a wrench in place of a screwdriver either. The utility of COMSOL should not be extended past where it is useful. COMSOL has no usefulness beyond the "knowns." The unknowns can't be simulated. See Paul March & Mr. Shawyer's comments in this post, in screenshots and quotes.

Let's not forget that Mr. Shawyer (the inventor of EMdrive) is telling us to NOT get hung up on the standing wave, see screenshot:

http://forum.nasaspaceflight.com/index.php?topic=36313.msg1368066#msg1368066

The standing waves inside the frustum are simply a large potential energy store. Those standing waves still have to be put to use to get work out of them. From what Shawyer and Paul March have told us, there are two known ways......So far we've uncovered two ways; 1) Introducing phase noise into the excitation signal *, 2) Accelerating the cavity **. Both of these play off the same theme, which is that a resonant cavity will only resonate at a narrow frequency range within its bandwidth. If energy within that cavity is shifted out of that narrow bandwidth, it will no longer resonate, thus an energy deficit to be filled by the traveling wave.

*Theorized before, now proven by Paul March:
http://forum.nasaspaceflight.com/index.php?topic=36313.msg1366823#msg1366823
http://forum.nasaspaceflight.com/index.php?topic=36313.msg1331792#msg1331792
**Red shift/Blue shift changes the phase. Also see Shawyer's comments here: http://www.emdrive.com/EmDriveForceMeasurement.pdf and here: see motor/generator mode comments http://www.emdrive.com/IAC13paper17254.v2.pdf
***Possible 3 is I've been toying around with the idea of using phase shift keying as a means to modulate the excitation signal, this approach is similar to what Paul March was talking about using FM, but using PSK keeps the signal on resonant frequency peak, while varying the phase. So this keeps the frequency the same, but flips the phase, so it might not work. I know from experience that a PSK signal as viewed on a spectrum analyzer is a distribution above and below CF. I envision varying the symbol rate as a means to control thrust. I believe the directionality from both Paul's and my PSK idea comes from two sources, 1) The helicity of the polymer chains, 2) non-reciprocity within the cavity, which I've posted papers on. For now I acknowledge there is no proof of any non-reciprocity or PT symmetry breaking (either alone or as a pair) happening within Emdrive, yet I expect it will be coming.
****Possible 4, excite EMdrive with two signals on separate cables. I know these cavities can support multiple modes at ones, so why not exploit that, see where it goes?
https://goo.gl/nosWnF

Title: Re: EM Drive Developments - related to space flight applications - Thread 2
Post by: Mulletron on 05/10/2015 11:57 am
Update on the replication attempt.

I have succeeded in making a safe stable balance (finally), but I am still absolutely bombing in my attempts to get power to the balance. The slip ring approach has its faults and I'm going to have to find some sort of flexible power pickup or abandon the whole slip ring idea altogether. If I stick with this approach, I'll never get anywhere near the sensitivity of Cavendish.

If I go to flying a battery and use DC-DC converters (http://www.amazon.com/dp/B00JUFJ1GA?psc=1) to power the electronics, any future of high power testing using this setup will be dashed. Not to mention I'll have to go to a much stronger torsion wire or a Dyneema braid (as opposed to the solid Dyneema line I'm using now) to hold everything up. As I know, engineering is a bunch of tradeoffs. 

https://goo.gl/Q3jGN1

Useful references I found:
http://www.alta-space.com/uploads/file/publications/feep/049-dAgostino.pdf
http://photonicassociates.com/ISBEP4-2.pdf
Title: Re: EM Drive Developments - related to space flight applications - Thread 2
Post by: Notsosureofit on 05/10/2015 12:05 pm
Update on the replication attempt.

I have succeeded in making a safe stable balance (finally), but I am still absolutely bombing in my attempts to get power to the balance. The slip ring approach has its faults and I'm going to have to find some sort of flexible power pickup or abandon the whole slip ring idea altogether. If I stick with this approach, I'll never get anywhere near the sensitivity of Cavendish.

If I go to flying a battery and use DC-DC converters (http://www.amazon.com/dp/B00JUFJ1GA?psc=1) to power the electronics, any future of high power testing using this setup will be dashed. As I know, engineering is a bunch of tradeoffs. 

https://goo.gl/Q3jGN1

Yes, flying battery is good but power limited.  Good for vacuum.   We do this already w/ alkaline sealed to run dc motors in vacuo
Title: Re: EM Drive Developments - related to space flight applications - Thread 2
Post by: frobnicat on 05/10/2015 01:31 pm
Update on the replication attempt.

I have succeeded in making a safe stable balance (finally), but I am still absolutely bombing in my attempts to get power to the balance. The slip ring approach has its faults and I'm going to have to find some sort of flexible power pickup or abandon the whole slip ring idea altogether. If I stick with this approach, I'll never get anywhere near the sensitivity of Cavendish.

If I go to flying a battery and use DC-DC converters (http://www.amazon.com/dp/B00JUFJ1GA?psc=1) to power the electronics, any future of high power testing using this setup will be dashed. Not to mention I'll have to go to a much stronger torsion wire or a Dyneema braid (as opposed to the solid Dyneema line I'm using now) to hold everything up. As I know, engineering is a bunch of tradeoffs. 

https://goo.gl/Q3jGN1

Useful references I found:
http://www.alta-space.com/uploads/file/publications/feep/049-dAgostino.pdf
http://photonicassociates.com/ISBEP4-2.pdf

Great news, hopefully all those replication attempts by individuals or small teams will shed light on relevant parameters, and you are on the forefront. Can't you you go battery without DC/DC, what are the DC requirement of your RF amplifier ?
Title: Re: EM Drive Developments - related to space flight applications - Thread 2
Post by: frobnicat on 05/10/2015 01:38 pm
Yes. It is not a propellant-less thruster anymore.

Capacitors and batteries lose mass when they are discharged.

Yes, the mass equivalent of the energy delta. That is very very small, has potential chemical energy mass even ever been weighted experimentally ? Also this very very small mass would, eventually, just been transferred as heat in the cavity's walls. A hot brick is heavier than a cold brick, by the same amount a battery loses mass to heat it. Ultimately this mass_energy is radiated away as IR, either isotropically (0 net momentum) or collimated (net momentum no better than photon rocket). Just saying, from conventional interpretations of conventional frameworks.
Title: Re: EM Drive Developments - related to space flight applications - Thread 2
Post by: SeeShells on 05/10/2015 02:03 pm
Following on what jknuble said about the multipactor-like effect as a possible cause of thrust. http://en.wikipedia.org/wiki/Multipactor_effect  I can't help but wonder about what's going on with the copper surface of the frustum. A quick back of the envelope (well, python) calculation shows that there's certainly enough energy in these devices to somehow atomize a small amount of copper , and propel them with enough momentum to produce a small amount of thrust.

For example, a 30 watt emdrive where 0.001% of the energy went towards atomization and 1% went toward addtional momentum of the particles... You'd have a device with 91uN thrust, propelling 1.4ng of copper a second at 65500m/s.

I can think of 3 ways to debunk this. 1) perhaps that amount of particles going that fast would be noticeable with the naked eye, so this isn't really a valid explanation. 2) stick a detector behind the thruster (are they ionized?). 3) SEM of the surface compared to scraps from the same batch of copper not used in the thrustum.

Just how would we get a net-thrust from a closed cavity with atomization.  Even if atoms are being ioniozed inside the cavity I don't see how that could result in a net thrust.  Atomization results in immediate thrust but then that creates impact on the other side of the cavity canceling out the propulsion.
For the last month I've been digging through the mountains of well thought out (some not so well thought out) material, tests and ideas of why this EM Drive does what it does. It's a beautiful conundrum. For over 40 years as an engineer I've built/designed/tested computers, electronics, electro-mechanical, semiconductor machines, imaging systems for the Super Conductor Super Collider , the list is long.
Not a single thing I built defied the laws of physics or the formulas of the trade. . . Maxwell, Ohms law, etc. If something didn't work for some weird reason, it still followed the basic laws and formulas when it ended up.
It doesn't really matter to me what is happening inside of the EM Chamber it must follow the principals of physics and conservation of energy and momentum is one of them. If I have a Air Tank pressurized with 200psi of air and a audio speaker inside that can blast at 100 watts any frequency range no matter what mixture of sound or what mixture of harmonics I crank, the tank will not move, but put a hole in one end and stand back. The second law of thermodynamics states that the entropy of an isolated system never decreases and the EM Chamber is an isolated enclosed system, we think.  If we are getting thrust that, thrust must be acting outside the chamber in some form. This is why I asked the simple question if smoke was used in the tests, it wasn't to detect thermal air currents but to see if it was moving away from any thrust from the EM Chamber. Smoke is small .5 to 2 um and might be be directly effected. If not then look for other forms of accelerated energy, providing thrust emanating out of the EM Chamber.
Frobicat said "Appears the only compulsory casualty is apparent CoE, not necessarily CoE in itself if one can find a suitable positive energy source (or a negative energy sink) and a mechanism to couple with such source (or sink to get rid of debt).
How the (few) other theories out there deal with the apparent CoE demise" ?

Just my 2 cents on this Rocky Mountain Snowy Morning. FWIW A hot tub in the snow is a great place to simply think. ;)
Michelle
Title: Re: EM Drive Developments - related to space flight applications - Thread 2
Post by: zen-in on 05/10/2015 02:16 pm
Yes. It is not a propellant-less thruster anymore.

Capacitors and batteries lose mass when they are discharged.

Capacitors can have interesting aspects
213310.pdf (http://infinitas.co/r.d.d/project/framework/docs/em-quantum/nasa213310.pdf)

I've never heard of that before.   If you had a fully charged capacitor that was wired in series with a switch and resistor where would the mass be lost when the switch was closed?  Note: The paper quoted above does not provide any proof of the claim.    The "Mach Guitar" experiment was inconclusive.
Title: Re: EM Drive Developments - related to space flight applications - Thread 2
Post by: zen-in on 05/10/2015 02:30 pm
Update on the replication attempt.

I have succeeded in making a safe stable balance (finally), but I am still absolutely bombing in my attempts to get power to the balance. The slip ring approach has its faults and I'm going to have to find some sort of flexible power pickup or abandon the whole slip ring idea altogether. If I stick with this approach, I'll never get anywhere near the sensitivity of Cavendish.

If I go to flying a battery and use DC-DC converters (http://www.amazon.com/dp/B00JUFJ1GA?psc=1) to power the electronics, any future of high power testing using this setup will be dashed. Not to mention I'll have to go to a much stronger torsion wire or a Dyneema braid (as opposed to the solid Dyneema line I'm using now) to hold everything up. As I know, engineering is a bunch of tradeoffs. 

https://goo.gl/Q3jGN1

Useful references I found:
http://www.alta-space.com/uploads/file/publications/feep/049-dAgostino.pdf
http://photonicassociates.com/ISBEP4-2.pdf

There must be some way of coupling the RF power to the cavity using feed horns or near-field antennas.    That would eliminate the need to weigh down the balance with a PA, batteries or achive a workable cummutator.    There may not be much gain at the frequency of interest but all you want to do is to transmit a fraction of the power.   Maybe 2 identical collinear dipoles would work.   The recieve dipole, mounted on the outside of the cavity, would connect to the internal loop used to drive the cavity.   If you have access to a network analyzer you could optimize the match and maximize the return loss.
Title: Re: EM Drive Developments - related to space flight applications - Thread 2
Post by: frobnicat on 05/10/2015 03:04 pm
...

There must be some way of coupling the RF power to the cavity using feed horns or near-field antennas.    That would eliminate the need to weigh down the balance with a PA, batteries or achive a workable cummutator.    There may not be much gain at the frequency of interest but all you want to do is to transmit a fraction of the power.   Maybe 2 identical collinear dipoles would work.   The recieve dipole, mounted on the outside of the cavity, would connect to the internal loop used to drive the cavity.   If you have access to a network analyzer you could optimize the match and maximize the return loss.

Beaming RF power directly is tempting for the reasons you say, but wouldn't that introduce a momentum coupling between the fixed dipole and the one on the balance ? Even assuming a cylindrical wave guide coaxial with rotation axis, the E and B fields would have a definite transverse favorite direction (there are experts here, please correct me if wrong), the fields from fixed parts and geometry of the cavity would not be symmetrical under rotation of the cavity, no rotation invariance => potential energy variation with rotation => torque. Can that be (classically) quantitatively estimated ? Maybe it's much lower torque coupling than energy transfer alternatives (thin wire or rotating wet contacts for DC to RF amp.) so still interesting option...

This could allow for a very light freely rotating setup (consider magnetic bearings instead of just point contact on jewel cup, or just a floater on a cup full of liquid, goal is lowest stiction as possible, not necessarily ultra-low dynamic viscosity as net thrust can always be integrated as cumulative displacement) :
Title: Re: EM Drive Developments - related to space flight applications - Thread 2
Post by: HypergolicSunrise on 05/10/2015 03:07 pm
Not a single thing I built defied the laws of physics or the formulas of the trade. . . Maxwell, Ohms law, etc. If something didn't work for some weird reason, it still followed the basic laws and formulas when it ended up.
It doesn't really matter to me what is happening inside of the EM Chamber it must follow the principals of physics and conservation of energy and momentum is one of them. If I have a Air Tank pressurized with 200psi of air and a audio speaker inside that can blast at 100 watts any frequency range no matter what mixture of sound or what mixture of harmonics I crank, the tank will not move, but put a hole in one end and stand back. The second law of thermodynamics states that the entropy of an isolated system never decreases and the EM Chamber is an isolated enclosed system, we think.  If we are getting thrust that, thrust must be acting outside the chamber in some form. This is why I asked the simple question if smoke was used in the tests, it wasn't to detect thermal air currents but to see if it was moving away from any thrust from the EM Chamber. Smoke is small .5 to 2 um and might be be directly effected. If not then look for other forms of accelerated energy, providing thrust emanating out of the EM Chamber.

I too would like to see a smoke test.  I can't see conservation of momentum being violated.  It just goes against everything we know both empirically and theoretically.  I think that even in the off chance that the EmDrive is not experimental error, conservation of momentum will still hold albeit in a more subtle manner than the classical analysis would expect.

You gave the illustrative example of a closed container with different traveling and standing waves of different frequencies and amplitudes bouncing around inside.  There is a very neat quantum mechanical reason that such a container is not truly closed.  Even in an infinite potential well, the wave function can extend outside the walls of the well, leading to effects such as tunneling.  Another great example of the wavefunction extending beyond barriers that appears to be somewhat related to the possible effect seen here is the Aharanov Bohm effect: http://en.wikipedia.org/wiki/Aharonov%E2%80%93Bohm_effect (http://en.wikipedia.org/wiki/Aharonov%E2%80%93Bohm_effect).  This is due to the wavefunction of a particle outside of a container extending past the barrier of the container and interacting with the EM field on the inside of the container.

Now I leave this paper to ruminate upon:
http://arxiv.org/abs/0708.0681 (http://arxiv.org/abs/0708.0681)
Perhaps the EmDrive is acting as an evanescent mode photon rocket where momentum is carried away outside the cavity via this mechanism.
Title: Re: EM Drive Developments - related to space flight applications - Thread 2
Post by: Rodal on 05/10/2015 03:25 pm
...
EDIT: Please click on the above link to see the above quotation which is not reproduced in full in order to save bandwidth

A) We should make an effort to properly characterize Finite Element solution packages like COMSOL that are able to obtain solutions to a large number of physical problems.  COMSOL is not just a "wrench" but a whole tool box of solutions.

 
the utility of COMSOL in figuring out this problem is limited as COMSOL is considering the standing wave and isn't considering the traveling wave. It gives no consideration to what is happening over extended time (over multiple full cycles) or to what is happening to a resonant cavity under dynamic operation, for example while under acceleration. COMSOL provides no insight as to what is happening when the resonant cavity is being excited by FM or other sources of phase noise.


In this case, "COMSOL" is being used as shorthand for "an eigenvalue solution to Maxwell's equations in a cavity."  The fact that the COMSOL numerical solution provides an accurate solution for the natural frequencies and mode shapes of a cavity is not questionable.

Let's remember why NASA Eagleworks conducted these particular analyses:

1) To calculate the natural frequencies of the truncated cone cavity, so that they know at what frequency they can excite resonance at a high Q.

2) To calculate the Q for the different mode shapes.

3) To calculate the different mode shapes (whether they are transverse magnetic or transverse electric modes) and the field distributions for different modes.

4) To calculate the resulting temperature from induction heating by the magnetic field (those calculations compare very well with the temperatures measured by a thermal camera).

NASA Eagleworks did not use the COMSOL Finite Element analysis to calculate any thrust forces.


Of course, it is well known by the NASA Eagleworks team that any claimed interaction with the Quantum Vacuum does not have any appreciable effect on the above calculated quantities (which are solely based on Maxwell's equations).  NASA Eagleworks has not reported any force calculations (based on Maxwell's stress tensor) or Poynting vector calculations based on Maxwell's equations.  They have stated that they are aware that such force calculations would give zero net thrust. 

Dr. White's calculations for the thrust are based on a separate code, which reportedly he wrote based on his Quantum Vacuum conjecture.  I have not seen the actual formulation for this code, or anything addressing what his code is based on, so I will not comment any further on his attempt.

Returning to COMSOL's capabilities, of course it is well known that one can also conduct a transient solution to the problem using COMSOL.  It is well known that a transient solution to Maxwell's equations cannot result in a solution that will explain the claims of the EM Drive being able to be used for space propulsion.

NASA conducted an eigenvalue problem solution of Maxwell's equations, in order to obtain the natural frequencies and mode shapes for the cavity.  This makes eminent sense.  Concerning FM modulation, COMSOL can also be used to obtain a solution (if so desired) for an arbitrary spectrum as an input (and the variation of the spectrum vs. time).  None of this is going to change the natural frequencies of the cavity, nor is it going to explain (just based on Maxwell's equations) how an EM Drive can be used for space propulsion.

No solution, eigensolution or transient solution, solely based on Maxwell's equations, can explain a violation of conservation of momentum, as the equations being solved by COMSOL (in this particular analysis: Maxwell's equations) fully satisfy conservation of momentum.



B) I am surprised at the statements that Shawyer makes in http://forum.nasaspaceflight.com/index.php?action=dlattach;topic=36313.0;attach=829636  , particularly his claim that

Quote from: Shawyer
the standing wave is constructed by the addition, in phase, of a continuous travelling wave.  This travelling wave results in the forces that are produced on the end plates which are orthogonal to the group velocity vector of the wave.  In a cavity with the correct radius spherical end plates there is no force on the side walls due to the travelling wave, because the walls are parallel to the group velocity vector

This claim that there are no forces on the side walls is contradicted by a calculation of Maxwell's stress tensor, particularly under classical mechanics, as explicitly shown by Greg Egan  (http://gregegan.customer.netspace.net.au/SCIENCE/Cavity/Cavity.html ) for example for such a truncated cone with spherical ends.  Shawyer (to my knowledge) has never adequately answered Greg Egans's proof.



C) I am also surprised by Shawyer invoking Cullen

Quote from: Shawyer
The forces produced by travelling waves in a waveguide were first investigated theoretically  and experimentally by Prof. Alex Cullen back in the 1950s. His analysis is as true today as it was when he first did the work
 

Never mind that Cullen was not the first to investigate theoretically the forces produced by travelling waves in a waveguide (as Cullen himself points out that he is using the much older intuitive derivation by Sir J.J.Thompson, which is well-known to be fully consistent with the derivation due to Maxwell himself).  Cullen was the first, in 1950, to experimentally measure quantitatively the radiation pressure at microwave frequencies (for light frequencies, it had been done back in 1900 by Lebedew, as it is well known). 

The analysis and experiments performed by Cullen were based on waveguides with constant cross-section.  In such waveguides Maxwell's stress tensor components on the side walls are self-cancelling due to the fact that the waveguide has constant cross-section.  This is completely inapplicable to a truncated cone or any such geometry where the cross-section is not constant.  For a geometry with variable cross-section Maxwell's stress tensor components are not self-cancelling (as follows trivially from vector analysis) and hence must be taken into account when calculating the net thrust under classical mechanics and special relativity, which unfortunately Shawyer has failed to do.




SUMMARY: COMSOL Finite Element Analysis can accurately solve a huge range of classical, nonlinear, anisotropic, coupled, transient, and physical problems based on classical mechanics.  All COMSOL solutions satisfy the universal laws of conservation of momentum and conservation of energy.

The issue of conservation of momentum and energy in the EM Drive has not yet been formally and satisfactorily addressed.  Valiant attempts are being conducted in this forum to explain the EM Drive thrust (if it is not an artifact) on the basis of General Relativity or other physical solutions that lie outside the range of classical mechanics addressed by COMSOL. 

Understandably at this early stage of analysis, these attempts at solutions remain to be formally proven and remain to be shown to quantitatively predict EM Drive measurements (even within engineering accuracy, and much less to scientific accuracy).
Title: Re: EM Drive Developments - related to space flight applications - Thread 2
Post by: Left Field on 05/10/2015 03:36 pm
I was thinking about superposition and whether mass and energy are conserved when two waves cancel. If that were not the case then perhaps there could be an imbalance in the frustrum that causes there to be an asymmetric photon pressure buildup on each face of the frustrum. I found this article in my travels that seems to suggest this but it is behind a paywall (doubtless I would not understand it anyway):

http://www.opticsinfobase.org/josaa/abstract.cfm?uri=josaa-27-11-2468

Oblique superposition of two elliptically polarized lightwaves using geometric algebra: is energy–momentum conserved? Michelle Wynne C. Sze, Quirino M. Sugon, Jr., and Daniel J. McNamara JOSA A, Vol. 27, Issue 11, pp. 2468-2479 (2010)

We added the two elliptically polarized waves and computed the energy–momentum density of their sum. We showed that energy and momentum are not generally conserved, except when the two waves are moving in opposite directions. We also showed that the momentum of the superposition has an extra component perpendicular to the propagation directions of both waves. But when we took the time-average of the energy and momentum of the superposition, we found that the time-average energy and momentum could also be conserved if both waves are circularly polarized but with opposite handedness, regardless of the directions of the two waves. The non-conservation of energy and momentum of the superposition of two elliptically polarized plane waves is not due to the form of the plane waves themselves, but rather to the accepted definitions of the electromagnetic energy and momentum. Perhaps we may need to modify these definitions in order to preserve the energy–momentum conservation. In our computations, we restricted ourselves to the superposition of two waves with the same frequency.

... Just a thought from someone with an interest in Physics.

Be gentle :)
Title: Re: EM Drive Developments - related to space flight applications - Thread 2
Post by: Rodal on 05/10/2015 04:03 pm
.. There is a very neat quantum mechanical reason that such a container is not truly closed.  Even in an infinite potential well, the wave function can extend outside the walls of the well, leading to effects such as tunneling.  Another great example of the wavefunction extending beyond barriers that appears to be somewhat related to the possible effect seen here is the Aharanov Bohm effect: http://en.wikipedia.org/wiki/Aharonov%E2%80%93Bohm_effect (http://en.wikipedia.org/wiki/Aharonov%E2%80%93Bohm_effect).  This is due to the wavefunction of a particle outside of a container extending past the barrier of the container and interacting with the EM field on the inside of the container.
...
A warm welcome to the forum and thank you for a great post.   :)

I think we all agree with you regarding the above statement. 

One of the people in this forum (@aero) has pursued such an explanation with numerical calculations using MEEP (a freely available computer code using the Finite Difference numerical discretization approach).  Unfortunately, due to computer memory and computer time available, @aero's calculations have been restricted to a 2-D formulation modeling the 3D truncated cone as a flat 2-D trapezium (which obviously cannot appropriately model the 3D problem).

The problem is that the EM Drive researchers claim thrust/InputPower measurements that are thousands of times greater than the one of a perfectly collimated photon rocket.

It would be most interesting to this forum if you could lead us to an explanation that could explain how the EM Drive can result in thrust/InputPower measurements that are thousands of times greater than the one of a perfectly collimated photon rocket.

Do you see any such explanation ?
Title: Re: EM Drive Developments - related to space flight applications - Thread 2
Post by: SeeShells on 05/10/2015 04:20 pm
You gave the illustrative example of a closed container with different traveling and standing waves of different frequencies and amplitudes bouncing around inside.  There is a very neat quantum mechanical reason that such a container is not truly closed.  Even in an infinite potential well, the wave function can extend outside the walls of the well, leading to effects such as tunneling.  Another great example of the wavefunction extending beyond barriers that appears to be somewhat related to the possible effect seen here is the Aharanov Bohm effect: http://en.wikipedia.org/wiki/Aharonov%E2%80%93Bohm_effect (http://en.wikipedia.org/wiki/Aharonov%E2%80%93Bohm_effect).  This is due to the wavefunction of a particle outside of a container extending past the barrier of the container and interacting with the EM field on the inside of the container.

Now I leave this paper to ruminate upon:
http://arxiv.org/abs/0708.0681 (http://arxiv.org/abs/0708.0681)
Perhaps the EmDrive is acting as an evanescent mode photon rocket where momentum is carried away outside the cavity via this mechanism.
[/quote]

Thank you, more hot tub thinking.
Nice paper on tunneling and it does make sense on how the waves with quantum mechanical tunneling cross the plane of incidence (Goos-Hänchen shift). (Back in school studying Tunneling diodes we couldn't understand how they worked).  Should I say that a tunnel (quantum or not) is a hole in the air tank or EM Device? What comprises the pressure and how do we detect it, that's the big question isn't it? 
Title: Re: EM Drive Developments - related to space flight applications - Thread 2
Post by: CW on 05/10/2015 04:24 pm
As far as I understand, everything in physics is a process. That logically includes all kinds of conservation mechanisms.

Question is: Are processes like CoM and CoE finite or infinite? If we can't even tell whether or not these processes are finite or not, this whole discussion is perhaps all moot. If these processes are finite, I think it would mean that all apparent momentum and energy return to the QV anyways.
I think it's become clear for everyone that within Maxwell's linear framework no thrust whatsoever is possible from within such a closed cavity. Key will be the experimental data of Eagleworks around summertime to show whether or not expected improved 'thrust' signatures can be obtained in a methodical and controlled manner.
What also might happen is that some random day Mr. Shawyer's device makes it to the public in form of a powerful in-your-face demonstrator that leaves all people dumb-struck. Same for the 'Cannae' device. For now, I think it's best to wait for results from Eagleworks this summer. They have had the best visibility so far.
Title: Re: EM Drive Developments - related to space flight applications - Thread 2
Post by: Mulletron on 05/10/2015 04:28 pm
http://forum.nasaspaceflight.com/index.php?topic=36313.msg1326608#msg1326608

I believe the series of successful vacuum tests negates the need for a smoke stick test. Comments?
Title: Re: EM Drive Developments - related to space flight applications - Thread 2
Post by: Mulletron on 05/10/2015 04:31 pm
Also I want to make sure this is corrected:

I was recently quoted above, yet I do not recall making any such statements which consisted of only 4 periods.... nor did I abruptly end my sentence with the word "shapes."

I will remove this post when I am properly quoted in full or the offending post is removed.

....

A) We should make an effort to properly characterize Finite Element solution packages like COMSOL that are able to obtain solutions to a large number of physical problems.  COMSOL is not just a "wrench" but a whole tool box of solutions.

 
the utility of COMSOL in figuring out this problem is limited as COMSOL is considering the standing wave and isn't considering the traveling wave. It gives no consideration to what is happening over extended time (over multiple full cycles) or to what is happening to a resonant cavity under dynamic operation, for example while under acceleration. COMSOL provides no insight as to what is happening when the resonant cavity is being excited by FM or other sources of phase noise. Building off the comments about COMSOL's ability to simulate mode shapes
Title: Re: EM Drive Developments - related to space flight applications - Thread 2
Post by: Flyby on 05/10/2015 04:38 pm
Update on the replication attempt.

I have succeeded in making a safe stable balance (finally), but I am still absolutely bombing in my attempts to get power to the balance. The slip ring approach has its faults and I'm going to have to find some sort of flexible power pickup or abandon the whole slip ring idea altogether. If I stick with this approach, I'll never get anywhere near the sensitivity of Cavendish.

If I go to flying a battery and use DC-DC converters (http://www.amazon.com/dp/B00JUFJ1GA?psc=1) to power the electronics, any future of high power testing using this setup will be dashed. Not to mention I'll have to go to a much stronger torsion wire or a Dyneema braid (as opposed to the solid Dyneema line I'm using now) to hold everything up. As I know, engineering is a bunch of tradeoffs. 

https://goo.gl/Q3jGN1

Useful references I found:
http://www.alta-space.com/uploads/file/publications/feep/049-dAgostino.pdf
http://photonicassociates.com/ISBEP4-2.pdf

Just a wild idea, but have you considered this form of powering :
http://en.wikipedia.org/wiki/Wireless_power

basically, contact-less induction would give you frictionless movement (except for the bearing friction and air resistance) and provide power. I'm however not sure what the electromagnetic implications would be on any surrounding devices...(fe, does it act as a magnetic brake?)

Secondly, not sure what would be needed to feed 1Kw of power. It might be not so practical all together...

Although.... I've seen street based induction plates for loading batteries of electric cars.

well.. it's just a wild idea flare... :)
Title: Re: EM Drive Developments - related to space flight applications - Thread 2
Post by: deltaMass on 05/10/2015 04:49 pm
Well, although my first two thrust scenarios (trash Noether, trash Einstein) generated a fair amount of discussion, my third thrust scenario went down like a lead balloon, despite it being based on an actual video (Shawyer) and an actual theory (McCullough). Could this be because it forecast so little propulsive utility?   ???

I do hope that's not the reason, else objectivity is being lost here.

I might be wrong about this third scenario too. I am uneasy about the final consideration re. what happens when the power is removed. I see a possibility of exploitation of an agglomerative thrust effect via some sort of pulsed operation. But currently it's just a twinkle in the eye. Or that I fell asleep in a hot tub Friday night and have not been quite right since :)
Title: Re: EM Drive Developments - related to space flight applications - Thread 2
Post by: Rodal on 05/10/2015 04:50 pm
The only tests reported for an EM Drive in a partial vacuum, have, to my knowledge, been conducted with a bi-layer copper/glass-fiber-reinforced-epoxy base plate with the reinforced epoxy exposed to the vacuum.

Partial vacuum tests do not address the issue of outgassing from the reinforced-epoxy exposed to the vacuum. 

Therefore, it would be wise to address the possible issue of outgassing in such experiments.
Title: Re: EM Drive Developments - related to space flight applications - Thread 2
Post by: Notsosureofit on 05/10/2015 04:50 pm
Not a single thing I built defied the laws of physics or the formulas of the trade. . . Maxwell, Ohms law, etc. If something didn't work for some weird reason, it still followed the basic laws and formulas when it ended up.
It doesn't really matter to me what is happening inside of the EM Chamber it must follow the principals of physics and conservation of energy and momentum is one of them. If I have a Air Tank pressurized with 200psi of air and a audio speaker inside that can blast at 100 watts any frequency range no matter what mixture of sound or what mixture of harmonics I crank, the tank will not move, but put a hole in one end and stand back. The second law of thermodynamics states that the entropy of an isolated system never decreases and the EM Chamber is an isolated enclosed system, we think.  If we are getting thrust that, thrust must be acting outside the chamber in some form. This is why I asked the simple question if smoke was used in the tests, it wasn't to detect thermal air currents but to see if it was moving away from any thrust from the EM Chamber. Smoke is small .5 to 2 um and might be be directly effected. If not then look for other forms of accelerated energy, providing thrust emanating out of the EM Chamber.

I too would like to see a smoke test.  I can't see conservation of momentum being violated.  It just goes against everything we know both empirically and theoretically.  I think that even in the off chance that the EmDrive is not experimental error, conservation of momentum will still hold albeit in a more subtle manner than the classical analysis would expect.

You gave the illustrative example of a closed container with different traveling and standing waves of different frequencies and amplitudes bouncing around inside.  There is a very neat quantum mechanical reason that such a container is not truly closed.  Even in an infinite potential well, the wave function can extend outside the walls of the well, leading to effects such as tunneling.  Another great example of the wavefunction extending beyond barriers that appears to be somewhat related to the possible effect seen here is the Aharanov Bohm effect: http://en.wikipedia.org/wiki/Aharonov%E2%80%93Bohm_effect (http://en.wikipedia.org/wiki/Aharonov%E2%80%93Bohm_effect).  This is due to the wavefunction of a particle outside of a container extending past the barrier of the container and interacting with the EM field on the inside of the container.

Now I leave this paper to ruminate upon:
http://arxiv.org/abs/0708.0681 (http://arxiv.org/abs/0708.0681)
Perhaps the EmDrive is acting as an evanescent mode photon rocket where momentum is carried away outside the cavity via this mechanism.

I can remember experiments (if not the references) of time delay measurements of sub-cutoff frequency waveguide transmission.  They were loudly poo-pooed at the time as claiming FTL information.
Title: Re: EM Drive Developments - related to space flight applications - Thread 2
Post by: SeeShells on 05/10/2015 05:03 pm
http://forum.nasaspaceflight.com/index.php?topic=36313.msg1326608#msg1326608

I believe the series of successful vacuum tests negates the need for a smoke stick test. Comments?

It's not only smoke, please I don't mean to upset you or anyone here, the smoke is more a analogy. The idea behind it is something is making it out of the EM device, unless you think the laws of CoE and CoM are being violated and I honestly don't. The question arises no matter how many equations I drudge through or papers I read. I want to say what tests this summer are you planning to do to try to see what form of energy/mass is causing the thrust? Maybe a sheet of carbon nano-fibers  would work  http://static-content.springer.com/lookinside/art%3A10.1186%2F1556-276X-8-60/000.png
 
Title: Re: EM Drive Developments - related to space flight applications - Thread 2
Post by: lasoi on 05/10/2015 05:09 pm
The smoke test is still a good idea.  If the thrust proved to be due to experimental artifact, it could be due to multiple artifacts working in conjunction.  Wasn't the thrust quite a bit smaller in the vacuum tests?
Title: Re: EM Drive Developments - related to space flight applications - Thread 2
Post by: Mulletron on 05/10/2015 05:10 pm
Update on the replication attempt.

I have succeeded in making a safe stable balance (finally), but I am still absolutely bombing in my attempts to get power to the balance. The slip ring approach has its faults and I'm going to have to find some sort of flexible power pickup or abandon the whole slip ring idea altogether. If I stick with this approach, I'll never get anywhere near the sensitivity of Cavendish.

If I go to flying a battery and use DC-DC converters (http://www.amazon.com/dp/B00JUFJ1GA?psc=1) to power the electronics, any future of high power testing using this setup will be dashed. Not to mention I'll have to go to a much stronger torsion wire or a Dyneema braid (as opposed to the solid Dyneema line I'm using now) to hold everything up. As I know, engineering is a bunch of tradeoffs. 

https://goo.gl/Q3jGN1

Useful references I found:
http://www.alta-space.com/uploads/file/publications/feep/049-dAgostino.pdf
http://photonicassociates.com/ISBEP4-2.pdf

Great news, hopefully all those replication attempts by individuals or small teams will shed light on relevant parameters, and you are on the forefront. Can't you you go battery without DC/DC, what are the DC requirement of your RF amplifier ?

I need 6VDC 2A and 5VDC 2A. I guess this is a good time to explain my reasoning here. I am a firm believer that this EMdrive technology (if it is even viable) will never get off the ground if it isn't introduced and validated in a very public way.

One of my original goals was to encourage replication attempts by not only academia (which has resources) but also to make it accessible to your average Joe. In keeping with the second goal, I have decided to go with Plain Jane COTS wifi gear and see if I can make something work with that.

I'm well aware of how stupid that sounds.

I know that the frustum design that I currently have was originally driven by a high power (and dangerous to most people) magnetron. The frequency range of your average microwave oven magnetron and wifi are the same. I verified the frustum will resonate within this frequency range using a spectrum analyzer and a SNA. Mine works on wifi channel 1 and 10. Given that Eagleworks was able to observe thrust with only 2.6 watts for one of their tests*, I think this is an acceptable risk to take. Besides, it is just money right? So I am literally driving the frustum with the RF from the wifi camera (used to observe and is riding on the experiment) and a 2watt amp. I can scale this up to 20 watts using other amps if needed. Amps are cheap and plug and play.

* http://www.libertariannews.org/wp-content/uploads/2014/07/AnomalousThrustProductionFromanRFTestDevice-BradyEtAl.pdf

I think the frequency hopping aspect of the waveform might end up doing me in though.

So this is a gamble. I'm accepting the risk of not getting a successful replication attempt in hopes that if I do, I will have built a ready-made solution for mister tinkerer to easily observe anomalous thrust at home.

If all that fails, I'll just shove in a magnetron. I deal with voltages at work (big fry your eyeballs out radars) that literally will and do make your hair stand up on end as soon as you open the cabinet, so I'm comfortable with and respect high voltages.

Potential tinkerers should know, that that big capacitor next to the magnetron inside their microwave oven, WILL KILL THEM if they get in between it and ground. USE A SHORTING PROBE!

(http://electriciantraining.tpub.com/14188/img/14188_30_1.jpg)
Title: Re: EM Drive Developments - related to space flight applications - Thread 2
Post by: deltaMass on 05/10/2015 05:11 pm
As I've mentioned, I am a strong proponent of a completely stand-alone system when it comes to measuring purported thrust from exotic devices (propellantless thrusters). The simple act of coupling to the stationary lab frame be it via coax, RF, Galinstan contacts or whichever, brings with it the possibility of artifacts masquerading as measured thrust. The whole thing has to be in a sealed box. And even then we are not out of the woods. Thermal power will tend to "balloon" the box volume and generate a buoyancy signal if the box is in air, aligned with local gravity's direction. Charges may accumulate on the box and generate phantom forces. Magnetic fields from outside, and/or from inside, may penetrate the box and also generate phantom forces. Only in space can all these phantoms be banished.

In space we need no balance; nor do we need any kind of force sensor. All we need is a measurement system to log the position of the DUT relative to its original null geodesic. This immediately kills two birds with one stone. We get a readout of initial "static" thrust from computing the initial acceleration, and, assuming actual off-geodesic motion subsequently ensues, we get the much sought-after free space dynamics.

Position measurement implies an interaction. Were LIDAR to be used, we have to correct our measured positions by taking into account the "photon sail" effect. It will be small, but it may yet be of a comparable magnitude to the actual thrust we detect. Then there's solar wind because we're not going outside the heliosphere (yet). And there is a direct photon flux from the sun. All these need to be addressed as sources of error.
Title: Re: EM Drive Developments - related to space flight applications - Thread 2
Post by: Mulletron on 05/10/2015 05:17 pm
Update on the replication attempt.

I have succeeded in making a safe stable balance (finally), but I am still absolutely bombing in my attempts to get power to the balance. The slip ring approach has its faults and I'm going to have to find some sort of flexible power pickup or abandon the whole slip ring idea altogether. If I stick with this approach, I'll never get anywhere near the sensitivity of Cavendish.

If I go to flying a battery and use DC-DC converters (http://www.amazon.com/dp/B00JUFJ1GA?psc=1) to power the electronics, any future of high power testing using this setup will be dashed. Not to mention I'll have to go to a much stronger torsion wire or a Dyneema braid (as opposed to the solid Dyneema line I'm using now) to hold everything up. As I know, engineering is a bunch of tradeoffs. 

https://goo.gl/Q3jGN1

Useful references I found:
http://www.alta-space.com/uploads/file/publications/feep/049-dAgostino.pdf
http://photonicassociates.com/ISBEP4-2.pdf

Just a wild idea, but have you considered this form of powering :
http://en.wikipedia.org/wiki/Wireless_power

basically, contact-less induction would give you frictionless movement (except for the bearing friction and air resistance) and provide power. I'm however not sure what the electromagnetic implications would be on any surrounding devices...(fe, does it act as a magnetic brake?)

Secondly, not sure what would be needed to feed 1Kw of power. It might be not so practical all together...

Although.... I've seen street based induction plates for loading batteries of electric cars.

well.. it's just a wild idea flare... :)

I did consider wireless power but not enough wattage:
http://forum.nasaspaceflight.com/index.php?topic=36313.msg1326813#msg1326813

See product at bottom link in above post.
Title: Re: EM Drive Developments - related to space flight applications - Thread 2
Post by: SeeShells on 05/10/2015 05:18 pm

I can remember experiments (if not the references) of time delay measurements of sub-cutoff frequency waveguide transmission.  They were loudly poo-pooed at the time as claiming FTL information.
:) In the 80s I remember being poo-pooed by a physics professor when I told him the the junction speeds in an IC or even transistor was the same as sound. Acoustic speed in the IC substrate. It's true, do the numbers for amorphous silicon or GAS. I like to keep an open mind and if something new presents itself in the violation of some of our ideas of how things work, I will jump on the bandwagon.
Title: Re: EM Drive Developments - related to space flight applications - Thread 2
Post by: dustinthewind on 05/10/2015 05:21 pm
Another paper that suggest we can get forces comparable to magnetic motors from (time/space phase em propulsion with near field effects further described here -->http://forum.nasaspaceflight.com/index.php?topic=36911.0).  Here is the paper below and a quote from page 9.

Exotic Matter and Propulsion within Maxwell’s Equations
Todd J. Desiato1
, Riccardo C. Storti2
November 6, 2003 v1"
http://citeseerx.ist.psu.edu/viewdoc/download?doi=10.1.1.10.1027&rep=rep1&type=pdf

Quote: "Note that equation (4) is not the equation typically considered for EM propulsion, that is force = power c/ . This is a very week force because there is no Lorentz force coupled to the radiation emitter. For example, it’s like using a flashlight for EM propulsion.

The EGM Array produces EM propulsion that is derived from the exertion Lorentz forces on the 4-currents, not simply by the exhaust of EM radiation. This is the same principle that moves electric motors, in which the current carrying conductors are coupled to magnetic flux linkages. [16] Gradients in the flux exert Lorentz forces on the conductors to turn the motor. Reciprocity between the forces acting on each source is suppressed by engineering considerations such as proper phase control and by purposeful design.

For example, compare the thrust produced by a 1 watt flashlight to the torque produced by a 1 watt electric motor. The coupling of the 4-currents to the EM field to produce Lorentz forces does mechanical work that a radiated EM field alone cannot do.

The EGM Array is comparable to a linear electric motor and may be described as a linear Rotor with a holographic Stator. [10,16] Holographic referring to the time-varying superposition of EM fields surrounding the EGM Array, from which the Lorentz forces emerge."

This paper appears to be in agreement with the paper listed below.

DEF: The Physical Basis of Electromagnetic Propulsion
Mario J. Pinheiro
http://arxiv.org/abs/1502.06288

Quote: page 4 "As is exposed in textbooks (see, e.g., Ref. [3]), from Eqs. 8- 9 we can obtain the electric and magnetic elds, showing terms that vary as 1=r (radiation eld), 1=r2 (induction terms), and 1=r3 (electrostatic eld terms). Therefore, as it is shown in Ref. [15], the propulsive force can result from the near- eld and=or the far- eld (radiative) mode, that is, it is not always the radiative mode of propulsion that can be useful under the point of view of practical engineering."

One worry is that the propulsion would be that of a photon rocket.  However this should not be the case.  We have the potential to see much larger forces with the correct engineering.  It is my suspicion that this may be what is also occurring in the EM drive but due to the obscurity of what is actually happening inside it can't yet be confirmed.  Some suggestions [I might need to look up where I saw this in this thread] of the imaginary fields ([near field, exponentially decaying] associated with currents in the frustrum) being out of phase with the light at one end of the frustum make me think we might be dealing with this phenomena.

As stated in the top quoted article and others, there is the possibility of connections to this type of propulsion and manipulating space and time [zpf] [casimir energy density] ect with EM fields.   

How Electrodynamics with Statistical Mechanics
Can Imply Gravitation
Cynthia Kolb Whitney
Editor, Galilean Electrodynamics
http://worldnpa.org/abstracts/abstracts_7185.pdf

Archimedes’ Principle and Gravitational Levitation
Charles T. Ridgely
Fullerton, CA
http://ridgely.ws/inertia/Ridgely-Archimedes.pdf
Title: Re: EM Drive Developments - related to space flight applications - Thread 2
Post by: Mulletron on 05/10/2015 05:31 pm
Update on the replication attempt.

I have succeeded in making a safe stable balance (finally), but I am still absolutely bombing in my attempts to get power to the balance. The slip ring approach has its faults and I'm going to have to find some sort of flexible power pickup or abandon the whole slip ring idea altogether. If I stick with this approach, I'll never get anywhere near the sensitivity of Cavendish.

If I go to flying a battery and use DC-DC converters (http://www.amazon.com/dp/B00JUFJ1GA?psc=1) to power the electronics, any future of high power testing using this setup will be dashed. Not to mention I'll have to go to a much stronger torsion wire or a Dyneema braid (as opposed to the solid Dyneema line I'm using now) to hold everything up. As I know, engineering is a bunch of tradeoffs. 

https://goo.gl/Q3jGN1

Useful references I found:
http://www.alta-space.com/uploads/file/publications/feep/049-dAgostino.pdf
http://photonicassociates.com/ISBEP4-2.pdf

There must be some way of coupling the RF power to the cavity using feed horns or near-field antennas.    That would eliminate the need to weigh down the balance with a PA, batteries or achive a workable cummutator.    There may not be much gain at the frequency of interest but all you want to do is to transmit a fraction of the power.   Maybe 2 identical collinear dipoles would work.   The recieve dipole, mounted on the outside of the cavity, would connect to the internal loop used to drive the cavity.   If you have access to a network analyzer you could optimize the match and maximize the return loss.

The Rf source can just ride the balance, but the issue is providing electrical power to the gear. The feed horns you're speaking of are a thing though, they're called standard gain horns and also reference antennas *. I finally decided they were too expensive and a huge pain to use. Very low loss though.

* https://goo.gl/o2mwZH
http://www.nardamicrowave.com/east/index.php?m=Products&e=list&categoryId=195
http://www.pasternack.com/standard-gain-horn-antennas-category.aspx
Title: Re: EM Drive Developments - related to space flight applications - Thread 2
Post by: LasJayhawk on 05/10/2015 05:33 pm


I need 6VDC 2A and 5VDC 2A. I guess this is a good time to explain my reasoning here. I am a firm believer that this EMdrive technology (if it is even viable) will never get off the ground if it isn't introduced and validated in a very public way.

One of my original goals was to encourage replication attempts by not only academia (which has resources) but also to make it accessible to your average Joe. In keeping with the second goal, I have decided to go with Plain Jane COTS wifi gear and see if I can make something work with that.

I'm well aware of how stupid that sounds.

I know that the frustum design that I currently have was originally driven by a high power (and dangerous to most people) magnetron. The frequency range of your average microwave oven magnetron and wifi are the same. I verified the frustum will resonate within this frequency range using a spectrum analyzer and a SNA. Mine works on wifi channel 1 and 10. Given that Eagleworks was able to observe thrust with only 2.6 watts for one of their tests*, I think this is an acceptable risk to take. Besides, it is just money right? So I am literally driving the frustum with the RF from the wifi camera (used to observe and is riding on the experiment) and a 2watt amp. I can scale this up to 20 watts using other amps if needed. Amps are cheap and plug and play.

* http://www.libertariannews.org/wp-content/uploads/2014/07/AnomalousThrustProductionFromanRFTestDevice-BradyEtAl.pdf

I think the frequency hopping aspect of the waveform might end up doing me in though.

So this is a gamble. I'm accepting the risk of not getting a successful replication attempt in hopes that if I do, I will have built a ready-made solution for mister tinkerer to easily observe anomalous thrust at home.

If all that fails, I'll just shove in a magnetron. I deal with voltages at work (big fry your eyeballs out radars) that literally will and do make your hair stand up on end as soon as you open the cabinet, so I'm comfortable with them too.

Some of the super fine strand the RC model guys use has a high ampacity, I think the 28 gauge is rated about 5 amps @20 degrees C rise. For the tests I would worry more about the voltage drop than the heating. Me I would feed it with 12 volts @ 2A and use linear regulators to reduce my wire count by 1.


Wifi signals? Who knows at this point, it might like the hopping.  :)
Title: Re: EM Drive Developments - related to space flight applications - Thread 2
Post by: deltaMass on 05/10/2015 05:39 pm
Continuing with the "Test In Space" theme.

We lack cheap space access for unmanned cargo. We have no railgun running from the coast of Ecuador up into the Andes to the east, and we have no Skylon/SABRE SSTO yet.  So we must pay many thousands of dollars per launched kilogram rather than what could be only tens of dollars.

What we do have is Cubesat and SpaceX. The problem is that the devices under consideration here won't fit even into the largest Cubesat. So let's talk miniaturisation.

What we have is photons in an asymmetric cavity. So let's use light instead of microwaves. I'll stop there for now.
Title: Re: EM Drive Developments - related to space flight applications - Thread 2
Post by: LasJayhawk on 05/10/2015 05:42 pm

I can remember experiments (if not the references) of time delay measurements of sub-cutoff frequency waveguide transmission.  They were loudly poo-pooed at the time as claiming FTL information.
:) In the 80s I remember being poo-pooed by a physics professor when I told him the the junction speeds in an IC or even transistor was the same as sound. Acoustic speed in the IC substrate. It's true, do the numbers for amorphous silicon or GAS. I like to keep an open mind and if something new presents itself in the violation of some of our ideas of how things work, I will jump on the bandwagon.

I remember talking to an engineer at IFR in the mid 80's and telling him I wanted a blue LED. He told me the physics were to hard, and the only market would be a high beam indicator for cars. When I told him I wanted a blue LED so I could make a color video display he just laughed and said " That will NEVER happen. "  ::)
Title: Re: EM Drive Developments - related to space flight applications - Thread 2
Post by: deltaMass on 05/10/2015 05:48 pm
I would like advice on lifetime estimates of the power within the cavity - what is the expected half life? Does a simple expression exist involving the Q-factor? I remind that Q can be defined as the angular frequency multiplied by the ratio of (stored energy) / (rate of energy dissipation).
Title: Re: EM Drive Developments - related to space flight applications - Thread 2
Post by: Rodal on 05/10/2015 05:49 pm
As I've mentioned, I am a strong proponent of a completely stand-alone system when it comes to measuring purported thrust from exotic devices (propellantless thrusters). The simple act of coupling to the stationary lab frame be it via coax, RF, Galinstan contacts or whichever, brings with it the possibility of artifacts masquerading as measured thrust. The whole thing has to be in a sealed box. And even then we are not out of the woods. Thermal power will tend to "balloon" the box volume and generate a buoyancy signal if the box is in air, aligned with local gravity's direction. Charges may accumulate on the box and generate phantom forces. Magnetic fields from outside, and/or from inside, may penetrate the box and also generate phantom forces. Only in space can all these phantoms be banished.

In space we need no balance; nor do we need any kind of force sensor. All we need is a measurement system to log the position of the DUT relative to its original null geodesic. This immediately kills two birds with one stone. We get a readout of initial "static" thrust from computing the initial acceleration, and, assuming actual off-geodesic motion subsequently ensues, we get the much sought-after free space dynamics.

Position measurement implies an interaction. Were LIDAR to be used, we have to correct our measured positions by taking into account the "photon sail" effect. It will be small, but it may yet be of a comparable magnitude to the actual thrust we detect. Then there's solar wind because we're not going outside the heliosphere (yet). And there is a direct photon flux from the sun. All these need to be addressed as sources of error.

Thank you for taking the time to write the reasons to fly the EM Drive in space.

Now, can we go in further detail with this?

The question is at what minimum altitude would this make sense. 

In LEO there are drag and magnetic field effects, so in LEO the EM Drive would not be really under the action of no forces.

What kind of orbit do you think it would be necessary to achieve to properly test an EM Drive in orbit, given that the only measurements we have for the EM Drive in a vacuum are the recently (2015) experimental measurements from NASA Eagleworks?

(http://forum.nasaspaceflight.com/index.php?action=dlattach;topic=36313.0;attach=635484;image)
Title: Re: EM Drive Developments - related to space flight applications - Thread 2
Post by: Mulletron on 05/10/2015 05:56 pm
The smoke test is still a good idea.  If the thrust proved to be due to experimental artifact, it could be due to multiple artifacts working in conjunction.  Wasn't the thrust quite a bit smaller in the vacuum tests?

Quote
Wasn't the thrust quite a bit smaller in the vacuum tests?
It sure was, you can't deny that:
http://forum.nasaspaceflight.com/index.php?topic=36313.msg1326608#msg1326608

I note that Paul March admitted his frustum is vented to allow air to escape:
http://forum.nasaspaceflight.com/index.php?topic=36313.msg1327937#msg1327937

My comments on that:
http://forum.nasaspaceflight.com/index.php?topic=36313.msg1330846#msg1330846
-see re-pressurize the cavity.....based off of 1)


Title: Re: EM Drive Developments - related to space flight applications - Thread 2
Post by: deltaMass on 05/10/2015 06:04 pm
I would like advice on lifetime estimates of the power within the cavity - what is the expected half life? Does a simple expression exist involving the Q-factor? I remind that Q can be defined as the angular frequency multiplied by the ratio of (stored energy) / (rate of energy dissipation).
A simplistic analysis produces an expression for half-life that is independent of the stored energy, viz:

T(half) = Q / (2*w)

with w being the angular frequency. Plugging in EW's approx Q ~= 6,000, f ~= 2.4 GHz. we get
T(half) ~= 200 nanoseconds.
This corresponds to a refresh rate of ~5 MHz.

What's the mark:space ratio? Well, this does depend on the cavity energy we require.
Do we know the cavity energy?

ETA There's method in my madness. I am working up to a "decoupled" measurement protocol.
Title: Re: EM Drive Developments - related to space flight applications - Thread 2
Post by: dustinthewind on 05/10/2015 06:04 pm
On the note about a vacuum test.  What about a high altitude balloon launch?  Quote from site: http://www.arhab.org/

"Near space is that region of the atmosphere above 60,000 feet but below the accepted altitude of space, 328,000 feet. These altitudes make near space far more like earth orbit than the surface of the earth. Air pressure in near space reaches 99% of a vacuum or better. Air temperatures drop to a low of -60 degrees F or colder. Cosmic radiation is over 100 times greater than at sea level. Near space is located within the ozone layer and therefore is an environment of increased damaging ultraviolet radiation. Near space is reached by helium or hydrogen-filled weather balloons. Since it is far less expensive to send payloads into near space than earth orbit, organizations like NASA will send new designs into near space first, as a test."

I guess the question is at what altitude do wind turbulence become a non-factor.  Maybe it is possible to even measure that and then subtract it from measurements inside a container on the balloon. 
Title: Re: EM Drive Developments - related to space flight applications - Thread 2
Post by: deltaMass on 05/10/2015 06:10 pm
I believe there's some meat on that bone. The idea here is to fly two geometrically identical test articles of equal mass close together, and only power one of them using wireless commands. Then the metric of interest is their difference. This helps to get rid of a lot of the guff you mentioned.
Title: Re: EM Drive Developments - related to space flight applications - Thread 2
Post by: Mulletron on 05/10/2015 06:14 pm
Continuing with the "Test In Space" theme.

We lack cheap space access for unmanned cargo. We have no railgun running from the coast of Ecuador up into the Andes to the east, and we have no Skylon/SABRE SSTO yet.  So we must pay many thousands of dollars per launched kilogram rather than what could be only tens of dollars.

What we do have is Cubesat and SpaceX. The problem is that the devices under consideration here won't fit even into the largest Cubesat. So let's talk miniaturisation.

What we have is photons in an asymmetric cavity. So let's use light instead of microwaves. I'll stop there for now.

I like where you're going with this...........and I want to talk miniaturization too. @Notsosureofit is working with 10ghz gunn diodes so he might be able to help.

These might be able to be put to use. These (scroll down to pic) are common in schools and easy to get:
http://demoweb.physics.ucla.edu/content/experiment-2-microwave-optics

Judging from reading (and really reading) the latest tweets from @ElonMusk, I feel that one of these Emdrive tests will find a ride on a rocket real soon.  :)

http://forum.nasaspaceflight.com/index.php?topic=36313.msg1368379#msg1368379

Note there is also a (very much new) favorable Io9 article released on April 30th,
(A day after this: http://www.nasaspaceflight.com/2015/04/evaluating-nasas-futuristic-em-drive/)
that wasn't linked to in the tweet. What was linked to was an old negative article from August last year.

**** http://www.pasco.com/prodCatalog/WA/WA-9314_microwave-optics-basic-system/
Title: Re: EM Drive Developments - related to space flight applications - Thread 2
Post by: dustinthewind on 05/10/2015 06:30 pm
Continuing with the "Test In Space" theme.

We lack cheap space access for unmanned cargo. We have no railgun running from the coast of Ecuador up into the Andes to the east, and we have no Skylon/SABRE SSTO yet.  So we must pay many thousands of dollars per launched kilogram rather than what could be only tens of dollars.

What we do have is Cubesat and SpaceX. The problem is that the devices under consideration here won't fit even into the largest Cubesat. So let's talk miniaturisation.

What we have is photons in an asymmetric cavity. So let's use light instead of microwaves. I'll stop there for now.

I like where you're going with this...........and I want to talk miniaturization too. @Notsosureofit is working with 10ghz gunn diodes so he might be able to help.

Judging from reading (and really reading) the latest tweets from @ElonMusk, I feel that one of these Emdrive tests will find a ride on a rocket real soon.  :)

http://forum.nasaspaceflight.com/index.php?topic=36313.msg1368379#msg1368379

Note there is also a (very much new) favorable Io9 article released on April 30th,
(A day after this: http://www.nasaspaceflight.com/2015/04/evaluating-nasas-futuristic-em-drive/)
that wasn't linked to in the tweet. What was linked to was an old negative article from August last year.

Interesting idea.  If for instance we went from 1cm wavelength to infrared at 100um then that would be a factor of 100 in wavelength.  A 1m object would then be 1cm in length?  Maybe we could use glass which reflects infrared?  Not too familiar with injecting infrared signals. 
Title: Re: EM Drive Developments - related to space flight applications - Thread 2
Post by: jmossman on 05/10/2015 06:39 pm
I know that the frustum design that I currently have was originally driven by a high power (and dangerous to most people) magnetron. The frequency range of your average microwave oven magnetron and wifi are the same. I verified the frustum will resonate within this frequency range using a spectrum analyzer and a SNA. Mine works on wifi channel 1 and 10. Given that Eagleworks was able to observe thrust with only 2.6 watts for one of their tests*, I think this is an acceptable risk to take. Besides, it is just money right? So I am literally driving the frustum with the RF from the wifi camera (used to observe and is riding on the experiment) and a 2watt amp. I can scale this up to 20 watts using other amps if needed. Amps are cheap and plug and play.

* http://www.libertariannews.org/wp-content/uploads/2014/07/AnomalousThrustProductionFromanRFTestDevice-BradyEtAl.pdf

I think the frequency hopping aspect of the waveform might end up doing me in though.

So this is a gamble. I'm accepting the risk of not getting a successful replication attempt in hopes that if I do, I will have built a ready-made solution for mister tinkerer to easily observe anomalous thrust at home.

If all that fails, I'll just shove in a magnetron.

If one were willing to deal with a USB cable originating from off of the balance beam, another possibility might be to try one of the cheap USB programmable generators identified by @TheTraveller (IIRC)...  but you might need to brush up on your Mandarin.    :P

http://www.ebay.com/itm/NEW-138MHz-4-4GHz-USB-SMA-Source-Signal-Generator-Simple-Spectrum-Analyzer-/171776507875?pt=LH_DefaultDomain_0&hash=item27fead7fe3

Probably too much to hope for that the generator could merely be connected to an external USB hub, "configured" via a laptop, and then have the external USB hub disconnected from the laptop....  Perhaps turning the amp output on/off (remotely?) would then provide the necessary experimental control?

Wireless USB extenders do exist, but based on my quick read of their reviews, most device USB drivers don't seem to work correctly when tunneled through a "wireless extender"...
Title: Re: EM Drive Developments - related to space flight applications - Thread 2
Post by: deltaMass on 05/10/2015 06:43 pm
The takeaway is that the entire system - battery, laser and cavity - could easily fit into the palm of your hand. The hard part is the cavity. Let's look at that a little bit.

If you make your cavity of dimensions a large number of wavelengths, then what you end up with is a very frequency-selective (i.e. narrow-band) system. The higher the multiple, the smaller the bandwidth. So it's a good idea to stay with one or two lambda at most, and also because that's what the replication experiments are doing.

But that requires a very precise dimensioning of the cavity geometry. We are talking nanometre accuracy and therefore nano-assembly techniques. Although I possess most of Drexler's books and follow the nano field as a casual observer, I am no expert. Over to you guys.

p.s. Re. Gunn diodes - way back in 1967 I was up at 18 GHz with one, while at Marconi Space & Defense, Stanmore.
Title: Re: EM Drive Developments - related to space flight applications - Thread 2
Post by: StrongGR on 05/10/2015 07:00 pm
Not a single thing I built defied the laws of physics or the formulas of the trade. . . Maxwell, Ohms law, etc. If something didn't work for some weird reason, it still followed the basic laws and formulas when it ended up.
It doesn't really matter to me what is happening inside of the EM Chamber it must follow the principals of physics and conservation of energy and momentum is one of them. If I have a Air Tank pressurized with 200psi of air and a audio speaker inside that can blast at 100 watts any frequency range no matter what mixture of sound or what mixture of harmonics I crank, the tank will not move, but put a hole in one end and stand back. The second law of thermodynamics states that the entropy of an isolated system never decreases and the EM Chamber is an isolated enclosed system, we think.  If we are getting thrust that, thrust must be acting outside the chamber in some form. This is why I asked the simple question if smoke was used in the tests, it wasn't to detect thermal air currents but to see if it was moving away from any thrust from the EM Chamber. Smoke is small .5 to 2 um and might be be directly effected. If not then look for other forms of accelerated energy, providing thrust emanating out of the EM Chamber.

I too would like to see a smoke test.  I can't see conservation of momentum being violated.  It just goes against everything we know both empirically and theoretically.  I think that even in the off chance that the EmDrive is not experimental error, conservation of momentum will still hold albeit in a more subtle manner than the classical analysis would expect.

You gave the illustrative example of a closed container with different traveling and standing waves of different frequencies and amplitudes bouncing around inside.  There is a very neat quantum mechanical reason that such a container is not truly closed.  Even in an infinite potential well, the wave function can extend outside the walls of the well, leading to effects such as tunneling.  Another great example of the wavefunction extending beyond barriers that appears to be somewhat related to the possible effect seen here is the Aharanov Bohm effect: http://en.wikipedia.org/wiki/Aharonov%E2%80%93Bohm_effect (http://en.wikipedia.org/wiki/Aharonov%E2%80%93Bohm_effect).  This is due to the wavefunction of a particle outside of a container extending past the barrier of the container and interacting with the EM field on the inside of the container.

Now I leave this paper to ruminate upon:
http://arxiv.org/abs/0708.0681 (http://arxiv.org/abs/0708.0681)
Perhaps the EmDrive is acting as an evanescent mode photon rocket where momentum is carried away outside the cavity via this mechanism.

I agree with your point. No violation of momentum whatsoever and we are left, if the effect should be confirmed, with some kind of violation of cherished laws of physics. Not so exciting due also to the particular article in play completely understandable by ordinary physics.

As I will post in a few days, general relativity cannot come to rescue in this case. I would like to evaluate the presence of a hypothetical source of energy-momentum from scalar fields that, since sixties, are ubiquitous in physics. No much to be excited so far except for White's interference experiments that could open up some new avenues in experimental general relativity in a lab.
Title: Re: EM Drive Developments - related to space flight applications - Thread 2
Post by: apoc2021 on 05/10/2015 07:24 pm
The reason for the confusion over the violation of classical physics is because this system has nothing to do with classical physics. Moreover, the “thrust” that is being calculated is not thrust at all but space moving the drive from one position to another which can merely be related to thrust but is not, per se, thrust. The controlling factor here is, of course, the resonant frequency. If you match the resonant frequency that space uses to “hold” the object you will develop a “cavity” that the “object will move towards”. The reason why the device cannot be “pushed off of” for conservation of momentum to hold true is because space is already pushing on it satisfying the law.

A couple of postulates to keep in mind that will help with these experiments are:
1. Space creates light.
2. Space itself is a resonating chamber.
Interesting! Would you then be prepared to write down the equations of motion so that we can play with them?

The equations of motion do not exist from an inertial reference frame. We must assume that the object is not moving and that space is moving around and through the object. I am trying to develop the Hamiltonian for space but having difficulty because we have always assumed space to be a virtual plasma and it is not virtual at all, but real.

Bumping this because of its elegance.
Title: Re: EM Drive Developments - related to space flight applications - Thread 2
Post by: tchernik on 05/10/2015 07:42 pm
The reason for the confusion over the violation of classical physics is because this system has nothing to do with classical physics. Moreover, the “thrust” that is being calculated is not thrust at all but space moving the drive from one position to another which can merely be related to thrust but is not, per se, thrust. The controlling factor here is, of course, the resonant frequency. If you match the resonant frequency that space uses to “hold” the object you will develop a “cavity” that the “object will move towards”. The reason why the device cannot be “pushed off of” for conservation of momentum to hold true is because space is already pushing on it satisfying the law.

A couple of postulates to keep in mind that will help with these experiments are:
1. Space creates light.
2. Space itself is a resonating chamber.
Interesting! Would you then be prepared to write down the equations of motion so that we can play with them?

The equations of motion do not exist from an inertial reference frame. We must assume that the object is not moving and that space is moving around and through the object. I am trying to develop the Hamiltonian for space but having difficulty because we have always assumed space to be a virtual plasma and it is not virtual at all, but real.

Bumping this because of its elegance.

I guess we will know if this is true relatively soon, if EW's interferometer experiments confirm there is a space warp field associated with the microwave resonating cavities, especially those that do provide "thrust".
Title: Re: EM Drive Developments - related to space flight applications - Thread 2
Post by: deltaMass on 05/10/2015 07:50 pm
I'm asking about the energy in the cavity (for reasons that may be clearer later). Here's a really dumb way to calculate it. This is so ugly, I hope someone shoots it down. It goes like this:

We know the input power, so it remains only to estimate the time to "fill" the cavity.  We can think of the Q factor as the number of bounces inside the cavity (!). Taking Q=6,000 and L = 0.25 m, we get t = 5 us.
Thus for an input power of 50 W, the cavity energy is 2.5*10-4 Joules.

Pretty bad hunh?

Another way to calculate it is to take the cavity volume and the average E-field from the simulation, and then the cavity energy is 0.5*epsilon0*E2*V Joules.
Title: Re: EM Drive Developments - related to space flight applications - Thread 2
Post by: TheTraveller on 05/10/2015 08:02 pm
I'm asking about the energy in the cavity (for reasons that may be clearer later). Here's a really dumb way to calculate it. This is so ugly, I hope someone shoots it down. It goes like this:

We know the input power, so it remains only to estimate the time to "fill" the cavity.  We can think of the Q factor as the number of bounces inside the cavity (!). Taking Q=6,000 and L = 0.25 m, we get t = 5 us.
Thus for an input power of 50 W, the cavity energy is 2.5*10-4 Joules.

Pretty bad hunh?

Another way to calculate it is to take the cavity volume and the average E-field from the simulation, and then the cavity energy is 0.5*epsilon0*E2*V Joules.
Resonate circuit has a time constant. 5 time constants to fill.
TC = Q / (2 Pi Fr).
Shawyer did comment on this and gave example.
Title: Re: EM Drive Developments - related to space flight applications - Thread 2
Post by: deltaMass on 05/10/2015 08:09 pm
I'm asking about the energy in the cavity (for reasons that may be clearer later). Here's a really dumb way to calculate it. This is so ugly, I hope someone shoots it down. It goes like this:

We know the input power, so it remains only to estimate the time to "fill" the cavity.  We can think of the Q factor as the number of bounces inside the cavity (!). Taking Q=6,000 and L = 0.25 m, we get t = 5 us.
Thus for an input power of 50 W, the cavity energy is 2.5*10-4 Joules.

Pretty bad hunh?

Another way to calculate it is to take the cavity volume and the average E-field from the simulation, and then the cavity energy is 0.5*epsilon0*E2*V Joules.
Resonate circuit has a time constant. 5 time constants to fill.
TC = Q / (2 Pi Fr).
Shawyer did comment on this and gave example.
t(fill) = 5*Q/w then. With (Q=6000, f = 2.4 GHz) , t(fill) = 2 us.
Boy I was close. Go figger.

So we now know the cavity energy is about 10-4 Joules.
Right?
Title: Re: EM Drive Developments - related to space flight applications - Thread 2
Post by: deltaMass on 05/10/2015 08:16 pm
So about 0,2 us to empty to half full and then about 1 us to fill up again. Somehow these should be equal, so let's say around a 1 MHz refresh rate to half-empty and refill.

So if we make measurements at times for which the power input is disconnected, we have discovered something new about the system, have we not?

Title: Re: EM Drive Developments - related to space flight applications - Thread 2
Post by: WarpTech on 05/10/2015 08:27 pm
@ RODAL

Just got a minute but from your p expression;

If L1/c1 = L2/c2

del f = (1/2*f)*((c1*c2)/(L1*L2))*b^2*((1/dD1^2)-(1/dD2^2))

might be a solution ??

Got to check the thinking later.

Night !

I find your previous expression

del f = ( f/(2*c^2)) * (c1^2-c2^2)

more physically appealing, since it goes to zero for equal dielectric constants, regardless or their dielectric length,

while on the other hand

del f = (1/2*f)*((c1*c2)/(L1*L2))*b^2*((1/dD1^2)-(1/dD2^2))

goes to zero for equal dielectric lengths, regardless of their dielectric constants.

The previous expression is only valid approximation for a "uniformly varying dielectric".  There is no L1 and L2 in that case.

What do you think might maximize the second expression ? (valid only for L1/c1 = L2/c2 )

I was discussing this last night and we made some interesting observations. In a variable dielectric, like in the frustum, when the waves are accelerating to a higher group velocity, they are losing momentum. This momentum is lost to the material "in the direction of the wave". It is similar to frame dragging. The wave is losing energy trying to drag the waveguide or the dielectric with it.

After the wave is reflected, it again tries to drag the dielectric or frustum with it, and this time it meets more resistance. It becomes an evanescent wave and decays faster.

I do not believe a small end cap is needed and the frustum should taper all the way down to the wave guide feeding it. The reflected waves cannot reach the small plate. That's what the thermal images show as well. Most of the energy I think should be trapped at the big end.

Todd D.

Title: Re: EM Drive Developments - related to space flight applications - Thread 2
Post by: WarpTech on 05/10/2015 08:36 pm
The reason for the confusion over the violation of classical physics is because this system has nothing to do with classical physics. Moreover, the “thrust” that is being calculated is not thrust at all but space moving the drive from one position to another which can merely be related to thrust but is not, per se, thrust. The controlling factor here is, of course, the resonant frequency. If you match the resonant frequency that space uses to “hold” the object you will develop a “cavity” that the “object will move towards”. The reason why the device cannot be “pushed off of” for conservation of momentum to hold true is because space is already pushing on it satisfying the law.

A couple of postulates to keep in mind that will help with these experiments are:
1. Space creates light.
2. Space itself is a resonating chamber.
Interesting! Would you then be prepared to write down the equations of motion so that we can play with them?

The equations of motion do not exist from an inertial reference frame. We must assume that the object is not moving and that space is moving around and through the object. I am trying to develop the Hamiltonian for space but having difficulty because we have always assumed space to be a virtual plasma and it is not virtual at all, but real.

Bumping this because of its elegance.

See the equations of motion for the Refractive Index, K.

https://www.dropbox.com/s/m3ztc8e5tv921z1/PV%20Approach.pdf?dl=0

Todd


Title: Re: EM Drive Developments - related to space flight applications - Thread 2
Post by: Flyby on 05/10/2015 08:44 pm
Continuing with the "Test In Space" theme.

We lack cheap space access for unmanned cargo. We have no railgun running from the coast of Ecuador up into the Andes to the east, and we have no Skylon/SABRE SSTO yet.  So we must pay many thousands of dollars per launched kilogram rather than what could be only tens of dollars.

What we do have is Cubesat and SpaceX. The problem is that the devices under consideration here won't fit even into the largest Cubesat. So let's talk miniaturisation.

What we have is photons in an asymmetric cavity. So let's use light instead of microwaves. I'll stop there for now.

The bitter irony is that we need expensive space access to (hopefully) validate a device that will gives us cheap space access... :-\
Although I think the EMdrive needs  either to gain more credibility first (or fail completely) before thinking of sending a test sample in orbit...
There is a lot more to do and test here on earth first, no?
Title: Re: EM Drive Developments - related to space flight applications - Thread 2
Post by: deltaMass on 05/10/2015 08:48 pm
Sure, but I'm just pointing out what the lack of cheap and readily available space access costs us in terms of either proving or disproving new propulsion theories. It would have saved Woodward 20 years of mucking about, for instance. It will cost this endeavour years also.
Title: Re: EM Drive Developments - related to space flight applications - Thread 2
Post by: PaulF on 05/10/2015 08:57 pm
Sure, but I'm just pointing out what the lack of cheap and readily available space access costs us in terms of either proving or disproving new propulsion theories. It would have saved Woodward 20 years of mucking about, for instance. It will cost this endeavour years also.
And I proposed a petition to Elon Musk / SpaceX. Nobody replied, is it such a bad idea?
Title: Re: EM Drive Developments - related to space flight applications - Thread 2
Post by: deltaMass on 05/10/2015 09:12 pm
A kickstarter would pay for a cubesat launch. It therefore seems logical to be confident about thrust from a miniature device in the lab, and then kickstart it for a cubesat deployment. Is there a better, faster route?

http://www.satmagazine.com/story.php?number=602922274
$50K min I think

Then there's NASA
http://www.nasa.gov/directorates/heo/home/CubeSats_initiative.html
and EW gets a free ride?
There's no cost info here.
Title: Re: EM Drive Developments - related to space flight applications - Thread 2
Post by: ThinkerX on 05/10/2015 09:40 pm
Ok, time for me to humiliate myself here yet again (I have been doing so in these threads since last year at regular intervals).

Something I have thought about before and am thinking about once again is thermal effects.  Virtually every engineer or theorist to jump into these EM Drive threads has cited thermal artefacts as a possible solution for EM Drive thrust.  Doctor Rodal's first posts in the original thread dealt with this.   Back then, the reasoning was:

1) thermal effects could mimic significant thrust in ambient air conditions;

2) those same thermal effects would still produce thrust, albeit far smaller thrust, in a vacuum on earth;

3) but in space, the thermal effects, would, at best result in rotary motion (not propelling the device).

Thus far, we have multiple experiments producing significant thrust in ambient air. (in accord with point 1)  The one series of known vacuum tests produced far less thrust (in accord with point 2).  Now, nobody has yet shot an EM Drive device into space...but there is an interesting comparable case:

http://en.wikipedia.org/wiki/Pioneer_anomaly

Quote
Various explanations, both of spacecraft behavior and of gravitation itself, were proposed to explain the anomaly. Over the period 1998–2012, one particular explanation became accepted. The spacecraft, which are surrounded by an ultra-high vacuum and are each powered by a radioisotope thermoelectric generator (RTG), can shed heat only via thermal radiation. If, due to the design of the spacecraft, more heat is emitted in a particular direction—what is known as a radiative anisotropy—then the spacecraft would exhibit a small acceleration in the direction opposite that of the excess emitted radiation due to radiation pressure. Because this force is due to the recoil of thermal photons, it is also called the thermal recoil force. If the excess radiation and attendant radiation pressure were pointed in a general direction opposite the Sun, the spacecrafts’ velocity away from the Sun would be decelerating at a greater rate than could be explained by previously recognized forces, such as gravity and trace friction, due to the interplanetary medium (imperfect vacuum).

By 2012 several papers by different groups, all reanalyzing the thermal radiation pressure forces inherent in the spacecraft, showed that a careful accounting of this could explain the entire anomaly, and thus the cause was mundane and did not point to any new phenomena or need for a different physical paradigm.[2][3] The most detailed analysis to date, by some of the original investigators, explicitly looks at two methods of estimating thermal forces, then states "We find no statistically significant difference between the two estimates and conclude that once the thermal recoil force is properly accounted for, no anomalous acceleration remains

A known example of 'accidental thermal effects' 'propelling' spacecraft.

Wild speculation 1:

Consensus is there is a lot of energy being pumped into these frustums.  Witness the funky colored cones posted in this thread now and again, showing a concentration of energy/heat at the 'big end.'  So...could the EM Drive be some sort of focused or amplified thermal drive - a vastly 'improved' version of the forces behind the 'pioneer anomaly?'  This sort of leads to

Wild speculation 2:

If this is some sort of thermal drive, then perhaps...thrust remains constant only for so long as the temperature increases?  So, if the temperature plateau's, then thrust declines.  This might provide a solution of sorts to the Conservation of Energy issue.

If this is the case, then the EM drive might make for a useful orbital thruster, and perhaps power interplanetary probes...but would it be adequate for interstellar propulsion?


Title: Re: EM Drive Developments - related to space flight applications - Thread 2
Post by: deltaMass on 05/10/2015 09:49 pm
If the power supply, RF gear and cavity were in a box in space, you wouldn't need to speculate. You would simply need to observe whether the box went off-geodesic (after having removed solar origins of residual thrust)

Well, thermal origins also (like Pioneer)
Title: Re: EM Drive Developments - related to space flight applications - Thread 2
Post by: A_M_Swallow on 05/10/2015 10:12 pm
{snip}
Wild speculation 2:

If this is some sort of thermal drive, then perhaps...thrust remains constant only for so long as the temperature increases?  So, if the temperature plateau's, then thrust declines.  This might provide a solution of sorts to the Conservation of Energy issue.

If this is the case, then the EM drive might make for a useful orbital thruster, and perhaps power interplanetary probes...but would it be adequate for interstellar propulsion?



An interstellar probe would have to be powered by a nuclear reactor. Currently the cooling radiators for nuclear reactors are flat but curving metal, to produce the equivalent of a nozzle, is not hard.
Title: Re: EM Drive Developments - related to space flight applications - Thread 2
Post by: zellerium on 05/10/2015 11:23 pm

Something I have thought about before and am thinking about once again is thermal effects.  Virtually every engineer or theorist to jump into these EM Drive threads has cited thermal artefacts as a possible solution for EM Drive thrust.  Doctor Rodal's first posts in the original thread dealt with this.   Back then, the reasoning was:

1) thermal effects could mimic significant thrust in ambient air conditions;

2) those same thermal effects would still produce thrust, albeit far smaller thrust, in a vacuum on earth;


I did some simple calculations to determine how fast the gas particles in the vacuum chamber would need to be going to produce a 50 uN force.

Pressure in the chamber = 5 E-5 Torr which corresponds to ~10^17 molecules of air. Density of air in the chamber = 10^17/(6.022*E23 molecules/mol) * .02896 kg/mol = 4.8 E-9 kg/m^3

Assuming a steady force, constant velocity across the area of the frustum, and constant density, the Reynolds Transport theorem simplifies to F = rho*v^2*A

Net pressure on the large end of the frustum [Area .25*pi*(.2794m)^2 ] = 8.42 E-4 Pa

Therefore the velocity required to produce this pressure force is sqrt( 8.42 E-4/(4.8 E-9) ) = 419 m/s.
Using the expression for the thermal kinetic energy 1/2 mv^2 = 3/2 kBT, and that a molecule of air has a mass of 4.81 E-26 kg

I found that the temperature of the gas would need to be 203 K or -70 C.

So if large end of the cavity could create that magnitude of temperature differential could the entire thrust be due to the kinetic energy of the gas?
Title: Re: EM Drive Developments - related to space flight applications - Thread 2
Post by: frobnicat on 05/10/2015 11:55 pm
@ RODAL

Just got a minute but from your p expression;

If L1/c1 = L2/c2

del f = (1/2*f)*((c1*c2)/(L1*L2))*b^2*((1/dD1^2)-(1/dD2^2))

might be a solution ??

Got to check the thinking later.

Night !

I find your previous expression

del f = ( f/(2*c^2)) * (c1^2-c2^2)

more physically appealing, since it goes to zero for equal dielectric constants, regardless or their dielectric length,

while on the other hand

del f = (1/2*f)*((c1*c2)/(L1*L2))*b^2*((1/dD1^2)-(1/dD2^2))

goes to zero for equal dielectric lengths, regardless of their dielectric constants.

The previous expression is only valid approximation for a "uniformly varying dielectric".  There is no L1 and L2 in that case.

What do you think might maximize the second expression ? (valid only for L1/c1 = L2/c2 )

I was discussing this last night and we made some interesting observations. In a variable dielectric, like in the frustum, when the waves are accelerating to a higher group velocity, they are losing momentum. This momentum is lost to the material "in the direction of the wave". It is similar to frame dragging. The wave is losing energy trying to drag the waveguide or the dielectric with it.

After the wave is reflected, it again tries to drag the dielectric or frustum with it, and this time it meets more resistance. It becomes an evanescent wave and decays faster.

I do not believe a small end cap is needed and the frustum should taper all the way down to the wave guide feeding it. The reflected waves cannot reach the small plate. That's what the thermal images show as well. Most of the energy I think should be trapped at the big end.

Todd D.

Looking for a mechanical analogy :
Let's play with a bended pipe and a ball rolling in it. The pipe can constrain the ball to various path, it can rise or fall, at various steepness. Height of the pipe at a given location defines gravitational potential energy of the ball there. The ball is launched with a given velocity, and then turns around the pipe if it is a closed circuit, or goes back and forth if the two ends of the pipe are high enough, should make no difference.

Assuming no friction, the ball goes-on forever. When rising the ball loses kinetic energy, slows, and imparts momentum to the pipe. When on the return path (different part of pipe if circuit path or same part of pipe if going back and forth), the same delta height will make ball regain same kinetic energy as lost when rising, accelerate, and imparts momentum again. When taking curves, ball also imparts momentum on pipe. Integrating all those momentum exchanges on a cycle yields 0 net momentum. Not depending on path details.

Assuming a closed circuit path and friction (dry, viscous, magnetic... whatever dissipative interaction), including parts with low friction (forth) and parts with high friction (back) and arbitrary height profile (potential well whatever). After a number of cycles the ball will come to rest. Integrating all the momentum exchanges of ball on pipe (changes of height, curves, friction) will yield a total momentum equal to the initial momentum of the ball when launched. Not depending on path details and what parts are more or less dissipative.

I know a photon is not a ball but my question is, in "Newtonian layman's terms" how does the line of thinking you are developing making that analogy not valid, i.e. imply apparent deviation from conservation of momentum ?
Title: Re: EM Drive Developments - related to space flight applications - Thread 2
Post by: deltaMass on 05/11/2015 12:05 am
@zellerium:
70oC temperature difference across the copper? But it's an excellent conductor of heat. I can't imagine that.
Title: Re: EM Drive Developments - related to space flight applications - Thread 2
Post by: Rodal on 05/11/2015 12:16 am
@zellerium:
70oC temperature difference across the copper? But it's an excellent conductor of heat. I can't imagine that.
NASA's Eagleworks base is made of glass-fiber reinforced epoxy with an extremely thin layer (deposited) of copper

on the outside: 0.063 inch thick FR4 printed circuit board and ~35 microns thick of Cu on the inside

Thermal conductivity, through-plane   0.29 W/(m·K)

Title: Re: EM Drive Developments - related to space flight applications - Thread 2
Post by: RareSaturn on 05/11/2015 12:43 am
Continuing with the "Test In Space" theme.

We lack cheap space access for unmanned cargo. We have no railgun running from the coast of Ecuador up into the Andes to the east, and we have no Skylon/SABRE SSTO yet.  So we must pay many thousands of dollars per launched kilogram rather than what could be only tens of dollars.

What we do have is Cubesat and SpaceX. The problem is that the devices under consideration here won't fit even into the largest Cubesat. So let's talk miniaturisation.

What we have is photons in an asymmetric cavity. So let's use light instead of microwaves. I'll stop there for now.

I have been advocating a scaled down version in a Cubesat.  I'm sure the cavity can be 3D printed, out of copper or any other metal. 
Title: Re: EM Drive Developments - related to space flight applications - Thread 2
Post by: Rodal on 05/11/2015 12:47 am
I know Shawyer and EW have tried a dielectric in the frustum. Are there any specifications for that dielectric? Material properties? Absorption properties at microwave frequencies?

I was looking at Pyramid Absorbers for microwaves, they can attenuate up to -55dB. A high power microwave source, pumped through a diode into such an absorber, seems to me should have a higher probability of thrust than the EM Drive and relatively simple to construct.


Todd D.

I found this document with the sourcing and the material properties assumed (based on the literature) for the High Density PolyEthylene used as a dielectric at NASA Eagleworks

http://forum.nasaspaceflight.com/index.php?action=dlattach;topic=36313.0;attach=635018
Title: Re: EM Drive Developments - related to space flight applications - Thread 2
Post by: deltaMass on 05/11/2015 01:06 am
@zellerium:
70oC temperature difference across the copper? But it's an excellent conductor of heat. I can't imagine that.
The base is made of glass-fiber reinforced epoxy with an extremely thin layer (deposited) of copper

on the outside: 0.063 inch thick FR4 printed circuit board and ~35 microns thick of Cu on the inside

Thermal conductivity, through-plane   0.29 W/(m·K)
A rough calc says that the Cu thickness has to be less than 44 microns. Looks like it's possible. But I calculated heat flow around the copper, not through-plane. Someone should recheck both.
P = 50 W, dT = 70oK
Title: Re: EM Drive Developments - related to space flight applications - Thread 2
Post by: Rodal on 05/11/2015 01:07 am
@zellerium:
70oC temperature difference across the copper? But it's an excellent conductor of heat. I can't imagine that.
The base is made of glass-fiber reinforced epoxy with an extremely thin layer (deposited) of copper

on the outside: 0.063 inch thick FR4 printed circuit board and ~35 microns thick of Cu on the inside

Thermal conductivity, through-plane   0.29 W/(m·K)
A rough calc says that the Cu thickness has to be less than 44 microns. Looks like it's possible. But I calculated heat flow around the copper, not through-plane. Someone should recheck both.
P = 50 W, dT = 70oK



Prof. Juan Yang's reported temperature vs. time measurements with embedded thermocouples throughout their EM Drive cavity (without a polymer dielectric insert) under atmospheric conditions, that, curiously, show the highest temperature at the center of the small base (trace #1), followed, at a significantly lower temperature by the temperature at the periphery of the big base (trace #5).

Take a look at the temperatures measured by the thermocouple Trace #1

(http://forum.nasaspaceflight.com/index.php?action=dlattach;topic=29276.0;attach=622845;image)

(http://forum.nasaspaceflight.com/index.php?action=dlattach;topic=29276.0;attach=655009;image)
Title: Re: EM Drive Developments - related to space flight applications - Thread 2
Post by: deltaMass on 05/11/2015 01:10 am
Yabut he' she's ackling at kilowatts. EW - not so much
Title: Re: EM Drive Developments - related to space flight applications - Thread 2
Post by: WarpTech on 05/11/2015 01:26 am
@ RODAL

Just got a minute but from your p expression;

If L1/c1 = L2/c2

del f = (1/2*f)*((c1*c2)/(L1*L2))*b^2*((1/dD1^2)-(1/dD2^2))

might be a solution ??

Got to check the thinking later.

Night !

I find your previous expression

del f = ( f/(2*c^2)) * (c1^2-c2^2)

more physically appealing, since it goes to zero for equal dielectric constants, regardless or their dielectric length,

while on the other hand

del f = (1/2*f)*((c1*c2)/(L1*L2))*b^2*((1/dD1^2)-(1/dD2^2))

goes to zero for equal dielectric lengths, regardless of their dielectric constants.

The previous expression is only valid approximation for a "uniformly varying dielectric".  There is no L1 and L2 in that case.

What do you think might maximize the second expression ? (valid only for L1/c1 = L2/c2 )

I was discussing this last night and we made some interesting observations. In a variable dielectric, like in the frustum, when the waves are accelerating to a higher group velocity, they are losing momentum. This momentum is lost to the material "in the direction of the wave". It is similar to frame dragging. The wave is losing energy trying to drag the waveguide or the dielectric with it.

After the wave is reflected, it again tries to drag the dielectric or frustum with it, and this time it meets more resistance. It becomes an evanescent wave and decays faster.

I do not believe a small end cap is needed and the frustum should taper all the way down to the wave guide feeding it. The reflected waves cannot reach the small plate. That's what the thermal images show as well. Most of the energy I think should be trapped at the big end.

Todd D.

Looking for a mechanical analogy :
Let's play with a bended pipe and a ball rolling in it. The pipe can constrain the ball to various path, it can rise or fall, at various steepness. Height of the pipe at a given location defines gravitational potential energy of the ball there. The ball is launched with a given velocity, and then turns around the pipe if it is a closed circuit, or goes back and forth if the two ends of the pipe are high enough, should make no difference.

Assuming no friction, the ball goes-on forever. When rising the ball loses kinetic energy, slows, and imparts momentum to the pipe. When on the return path (different part of pipe if circuit path or same part of pipe if going back and forth), the same delta height will make ball regain same kinetic energy as lost when rising, accelerate, and imparts momentum again. When taking curves, ball also imparts momentum on pipe. Integrating all those momentum exchanges on a cycle yields 0 net momentum. Not depending on path details.

Assuming a closed circuit path and friction (dry, viscous, magnetic... whatever dissipative interaction), including parts with low friction (forth) and parts with high friction (back) and arbitrary height profile (potential well whatever). After a number of cycles the ball will come to rest. Integrating all the momentum exchanges of ball on pipe (changes of height, curves, friction) will yield a total momentum equal to the initial momentum of the ball when launched. Not depending on path details and what parts are more or less dissipative.

I know a photon is not a ball but my question is, in "Newtonian layman's terms" how does the line of thinking you are developing making that analogy not valid, i.e. imply apparent deviation from conservation of momentum ?

The ball (photon) doesn't fall back down the well. There is nothing to give it back enough energy to do so. It dissipates in multiple reflections between the walls and the big end. They are not getting more out than they put in, so it does not violate conservation of energy. They are simply getting more NET momentum on one direction than in the other direction because there is more dissipation and attenuation in one direction than there is in the other. Dissipative systems are typically "not" conservative, loses prevent a true equal measure from occuring in both directions.

Todd D.

Title: Re: EM Drive Developments - related to space flight applications - Thread 2
Post by: Rodal on 05/11/2015 01:31 am
...

Looking for a mechanical analogy :
Let's play with a bended pipe and a ball rolling in it. The pipe can constrain the ball to various path, it can rise or fall, at various steepness. Height of the pipe at a given location defines gravitational potential energy of the ball there. The ball is launched with a given velocity, and then turns around the pipe if it is a closed circuit, or goes back and forth if the two ends of the pipe are high enough, should make no difference.

Assuming no friction, the ball goes-on forever. When rising the ball loses kinetic energy, slows, and imparts momentum to the pipe. When on the return path (different part of pipe if circuit path or same part of pipe if going back and forth), the same delta height will make ball regain same kinetic energy as lost when rising, accelerate, and imparts momentum again. When taking curves, ball also imparts momentum on pipe. Integrating all those momentum exchanges on a cycle yields 0 net momentum. Not depending on path details.

Assuming a closed circuit path and friction (dry, viscous, magnetic... whatever dissipative interaction), including parts with low friction (forth) and parts with high friction (back) and arbitrary height profile (potential well whatever). After a number of cycles the ball will come to rest. Integrating all the momentum exchanges of ball on pipe (changes of height, curves, friction) will yield a total momentum equal to the initial momentum of the ball when launched. Not depending on path details and what parts are more or less dissipative.

I know a photon is not a ball but my question is, in "Newtonian layman's terms" how does the line of thinking you are developing making that analogy not valid, i.e. imply apparent deviation from conservation of momentum ?

The ball (photon) doesn't fall back down the well. There is nothing to give it back enough energy to do so. It dissipates in multiple reflections between the walls and the big end. They are not getting more out than they put in, so it does not violate conservation of energy. They are simply getting more NET momentum on one direction than in the other direction because there is more dissipation and attenuation in one direction than there is in the other. Dissipative systems are typically "not" conservative, loses prevent a true equal measure from occuring in both directions.

Todd D.
Friction is a non-conservative force of course.  The energy that is not conserved turns into heat.

Follower forces are also non-conservatve (leading to very interesting instability problems).

Flutter instability --> follower force ---> energy harvesting

https://www.youtube.com/watch?v=bBVnhpYKzh4

Water coming out of a flexible hose: is  a follower force

https://www.youtube.com/watch?v=KMxyy5NrZ-o

and here we have flutter induced by friction (something that the great Mechanician Koiter thought was impossible)

https://www.youtube.com/watch?v=RTyHSjMVcB4

Title: Re: EM Drive Developments - related to space flight applications - Thread 2
Post by: deltaMass on 05/11/2015 04:16 am
@arc:
Quote
Do higher em frequencies deliver increased net effect?
Excellent question. To which as yet we have no answers. This might be taken to imply that we should find out :)

But but but...there is no guarantee that these forces in any way combine to produce a larger force. And that's static I mean. I'm even more doubtful about the free space dynamics.

Statically we care about k, or N/W
Dynamically we don't know what we care about yet  8)
Title: Re: EM Drive Developments - related to space flight applications - Thread 2
Post by: deltaMass on 05/11/2015 06:22 am
It seems to me, having noodled some algebra, that all 4 combinations of
- energy is conserved/not conserved
- momentum is conserved/not conserved
are algebraically possible.

Ignoring Ms. Noether then, there is no reason why not.

The relevance here is that although we seem to have a violation of CofM, we can still maintain CofE
Or vice versa
Title: Re: EM Drive Developments - related to space flight applications - Thread 2
Post by: CW on 05/11/2015 10:05 am
Do we really understand the nature of CoE and CoM correctly? I'm curious why nobody else seems to wonder.. . or maybe it's just me.

;)
Title: Re: EM Drive Developments - related to space flight applications - Thread 2
Post by: deltaMass on 05/11/2015 10:28 am
Our understanding of conservation is very deep, and Emmie Noether discovered it. She found that for every symmetry or invariance there exists a corresponding conservation law. That's a rather remarkable statement!

For momentum it's the symmetry of  space. For energy it's the symmetry of time. It's actually more mathematically complex than that, since it involves differentials of the Lagrangian.
http://en.wikipedia.org/wiki/Noether%27s_theorem

Susskind has some public lectures about all the gory details, on YouTube.
Title: Re: EM Drive Developments - related to space flight applications - Thread 2
Post by: frobnicat on 05/11/2015 10:44 am
...
I know a photon is not a ball but my question is, in "Newtonian layman's terms" how does the line of thinking you are developing making that analogy not valid, i.e. imply apparent deviation from conservation of momentum ?

The ball (photon) doesn't fall back down the well. There is nothing to give it back enough energy to do so. It dissipates in multiple reflections between the walls and the big end. They are not getting more out than they put in, so it does not violate conservation of energy. They are simply getting more NET momentum on one direction than in the other direction because there is more dissipation and attenuation in one direction than there is in the other. Dissipative systems are typically "not" conservative, loses prevent a true equal measure from occuring in both directions.

Todd D.

Classically, a dissipative system is conservative for both momentum and energy, it's just that for energy there is a (irreversible) conversion to a degraded form of energy, but there is no such thing as a mysterious part of total energy that would simply vanish. Even if not always convenient, an open system can be seen as part of a bigger closed system, and short of that the deltas total energy and total momentum of an open system can still be accounted, at least in principle, as integrated fluxes exchanged between open system and an outside.

My Newtonian ball of momentum pb can encounter an arbitrarily varying Force Fcb(t) of container on ball (vectors in bold). And dpb/dt=Fcb=-Fbc=-dpc/dt. That is instant conservation of momentum, and obviously integrating on successive instants just yields delta_pb=-delta_pc or  delta_pb+delta_pc=0, conservation of momentum on any time interval whatever the shape of varying Force Fcb(t). Where and how quantitatively your system is showing an apparent breaking of CoM at an "instantaneous scale" dt ? Short of that, details of trajectory is just, ahem, arm waving for propulsion purpose (aka Dean drive).
Title: Re: EM Drive Developments - related to space flight applications - Thread 2
Post by: deltaMass on 05/11/2015 10:54 am
Just an FYI that you have buttons to do superscripts and subscripts up there - sup and sub.
e.g. pb2
Title: Re: EM Drive Developments - related to space flight applications - Thread 2
Post by: Rodal on 05/11/2015 11:29 am
Do higher em frequencies deliver increased net effect?
Excellent question. To which as yet we have no answers. This might be taken to imply that we should find out :)

But but but...there is no guarantee that these forces in any way combine to produce a larger force. And that's static I mean. I'm even more doubtful about the free space dynamics.

Statically we care about k, or N/W
Dynamically we don't know what we care about yet  8)
We have three tentative answers.  They are the mathematical formulas based on their respective theoretical models:

Quote from: John von Neumann
The sciences do not try to explain, they hardly even try to interpret, they mainly make models. By a model is meant a mathematical construct which, with the addition of certain verbal interpretations, describes observed phenomena. The justification of such a mathematical construct is solely and precisely that it is expected to work.

If a theory does not have a mathematical formula predicting variables, it doesn't qualify as a physical theory.

The predictive formulas of Shawyer and of McCulloch both are inversely proportional to the frequency.  So, on the contrary, both Shawyer and McCulloch predict that the higher the frequency the smaller the thrust force.

The formulas of Shawyer and of McCulloch have no dependence on mode shape.

@Notsosureofit's formula is the only formula that shows dependence on mode shape (through the Bessel zeros, Xmn or X'mn) and hence it has a more nuanced, subtle dependence on frequency. 

As the Bessel zeros Xmn (for TM modes) and X'mn (for TE modes) increase with frequency, @Notsosureofit's thrust force increases with frequency for mode shapes with (both) high m and n (circular and radial) quantum numbers (because Xmn and X'mn increase with frequency at a faster rate than the frequency itself)

 @Notsosureofit's thrust force decreases with frequency for mode shapes having (both) low m and n (circular and radial) quantum numbers (but in this case some of these low m and n mode shapes may be cut off at higher frequencies).


Testing experiments have not yet been performed at a large enough range of frequencies to discriminate whether any of these formulas is correct with respect to frequency variation.

In this context, performing experiments at higher microwave frequencies, perhaps using a Gunn Diode would be most helpful.
Title: Re: EM Drive Developments - related to space flight applications - Thread 2
Post by: CW on 05/11/2015 11:31 am
Our understanding of conservation is very deep, and Emmie Noether discovered it. She found that for every symmetry or invariance there exists a corresponding conservation law. For momentum it's the symmetry of  space. For energy it's the symmetry of time. It's actually more mathematically complex than that, since it involves differentials of the Lagrangian.
http://en.wikipedia.org/wiki/Noether%27s_theorem

Susskind has some public lectures about all the gory details, on YouTube.

That's the point I want to make. We're working with a model that has proven to be correct so far, for a century. Everybody is so used to applying this powerful insight, that perception and thought patterns are fixated on it. What, for instance, if the symmetry of space is (temporarily) being changed by a device like an EM-drive? I think that such a thing might be at least in some way outside of Noether's theorem. It might all depend on whether or not the QV is immutable and indestructible. What do you think?

BR,
-CW
Title: Re: EM Drive Developments - related to space flight applications - Thread 2
Post by: SeeShells on 05/11/2015 11:39 am
Quote
SeeShells:
The second law of thermodynamics states that the entropy of an isolated system never decreases and the EM Chamber is an isolated enclosed system, we think.  If we are getting thrust that, thrust must be acting outside the chamber in some form. This is why I asked the simple question if smoke was used in the tests, it wasn't to detect thermal air currents but to see if it was moving away from any thrust from the EM Chamber. Smoke is small .5 to 2 um and might be be directly effected. If not then look for other forms of accelerated energy, providing thrust emanating out of the EM Chamber.

I agree with your thought train.

I have a feeling that we are so busy looking at the big paw print in the dirt that we don't see the bear sneaking up behind us.

Is it really the reflection/refraction of microwave photons that are the core issue here?.
Or is it the expanded energy shell they carry... that any high voltage source might carry?

It is obvious that the thermal state of the test device will be impacted.

Is the gravitational mass of the test unit "altered" between power up/power down?.
Is the "inertia" of the test device impacted?.
Is the Virtual mass of the test unit altering?, the reactionary energy based mass attachment to space-time. {Mach principle}

Does the chamber emit external acoustic waves? ( well beyond the human hearing range)
Nice rundown of most everything that can change the energy state of the Em Chamber at rest. I believe (unless new physics is discovered here, then the whole ballgame changes) whatever is happening it's going to be interesting. This rundown was the site that got me thinking.
http://hyperphysics.phy-astr.gsu.edu/hbase/quantum/phodens.html#c1
Photon Energy Density

Title: Re: EM Drive Developments - related to space flight applications - Thread 2
Post by: deltaMass on 05/11/2015 11:49 am
@arc:
Quote
Do higher em frequencies deliver increased net effect?
Excellent question. To which as yet we have no answers. This might be taken to imply that we should find out :)

But but but...there is no guarantee that these forces in any way combine to produce a larger force. And that's static I mean. I'm even more doubtful about the free space dynamics.

Statically we care about k, or N/W
Dynamically we don't know what we care about yet  8)
We have three tentative answers.  They are the mathematical formulas based on their respective theoretical models:

Quote from: John von Neumann
The sciences do not try to explain, they hardly even try to interpret, they mainly make models. By a model is meant a mathematical construct which, with the addition of certain verbal interpretations, describes observed phenomena. The justification of such a mathematical construct is solely and precisely that it is expected to work.

If a theory does not have a mathematical formula predicting variables, it doesn't qualify as a physical theory.

The predictive formulas of Shawyer and of McCulloch both are inversely proportional to the frequency.  So, on the contrary, both Shawyer and McCulloch predict that the higher the frequency the smaller the thrust force.

The formulas of Shawyer and of McCulloch have no dependence on mode shape.

@Notsosureofit's formula is the only formula that shows dependence on mode shape (through the Bessel zeros, Xmn or X'mn) and hence it has a more nuanced, subtle dependence on frequency. 

As the Bessel zeros Xmn (for TM modes) and X'mn (for TE modes) increase with frequency, @Notsosureofit's thrust force increases with frequency for mode shapes with (both) high m and n (circular and radial) quantum numbers (because Xmn and X'mn increase with frequency at a faster rate than the frequency itself)

 @Notsosureofit's thrust force decreases with frequency for mode shapes having (both) low m and n (circular and radial) quantum numbers (but in this case some of these low m and n mode shapes may be cut off at higher frequencies).

That's a nice enumeration. But what of Sonny White's PV derivative theory - called QVF I believe?
Title: Re: EM Drive Developments - related to space flight applications - Thread 2
Post by: deltaMass on 05/11/2015 11:55 am
Our understanding of conservation is very deep, and Emmie Noether discovered it. She found that for every symmetry or invariance there exists a corresponding conservation law. For momentum it's the symmetry of  space. For energy it's the symmetry of time. It's actually more mathematically complex than that, since it involves differentials of the Lagrangian.
http://en.wikipedia.org/wiki/Noether%27s_theorem

Susskind has some public lectures about all the gory details, on YouTube.

That's the point I want to make. We're working with a model that has proven to be correct so far, for a century. Everybody is so used to applying this powerful insight, that perception and thought patterns are fixated on it. What, for instance, if the symmetry of space is (temporarily) being changed by a device like an EM-drive? I think that such a thing might be at least in some way outside of Noether's theorem. It might all depend on whether or not the QV is immutable and indestructible. What do you think?

BR,
-CW
I have a real problem believing that an oddly-shaped cavity is going to change the workings of spacetime such that translational invariance is affected. But it's a creative suggestion nonetheless. On the other hand, if the thrust is truly not a measurement artifact, I have no explanation for it.
Title: Re: EM Drive Developments - related to space flight applications - Thread 2
Post by: Rodal on 05/11/2015 11:58 am
...
That's a nice enumeration. But what of Sonny White's PV derivative theory - called QVF I believe?
I have never seen an explicit mathematical expression for Dr. White's QVF thrust force in any of his papers.

Lately a new computer program of Dr. White was described and an image posted, described as displaying dependence on FM, AM and PM modulation, but the mathematical expressions on what the program was based were not released to the public.

I have read his papers (NOTE TO ALL READERS: please no need to bring up papers that talk about the QVF, with respect to this message, if they do not contain a mathematical expression explicitly showing a dependence of thrust force with frequency  ;)   )
Title: Re: EM Drive Developments - related to space flight applications - Thread 2
Post by: marshallC on 05/11/2015 12:01 pm

Interesting concept, I like it. 
10 stacks of "cavities" 100 units high arranged either side by side (flat/wall) or in a circular/tubular form. The tube would allow you to feed the energy up the middle to each drive unit.. 1000 micro-thrusters in Series-Parallel
Sounds like a 3D printing job. Glass would not stand that heat for very long... perhaps Silver?. Or Titanium with polished silver coating

Each cavity could use the wall of its neighbor for mechanical rigidity and the gaps resulting from conic shaped chambers would allow a coolant to be pumped around the cavities. Drive the units in cyclic pumped mode to minimise thermal overloading.

In this setup, would we be able to use one light source that's split up between all of the parallel units, or would we have individual light sources for each stack?  If I understand correctly, the force observed is far more than even a perfect photon thruster, so perhaps the potential to lose energy or get conflicting thrust while redirecting photos from one source into each individual frustum might not be significant compared to the minituarization it could allow compared to having multiple sources.

Title: Re: EM Drive Developments - related to space flight applications - Thread 2
Post by: MrVibrating on 05/11/2015 12:13 pm
Our understanding of conservation is very deep, and Emmie Noether discovered it. She found that for every symmetry or invariance there exists a corresponding conservation law. For momentum it's the symmetry of  space. For energy it's the symmetry of time. It's actually more mathematically complex than that, since it involves differentials of the Lagrangian.
http://en.wikipedia.org/wiki/Noether%27s_theorem

Susskind has some public lectures about all the gory details, on YouTube.

That's the point I want to make. We're working with a model that has proven to be correct so far, for a century. Everybody is so used to applying this powerful insight, that perception and thought patterns are fixated on it. What, for instance, if the symmetry of space is (temporarily) being changed by a device like an EM-drive? I think that such a thing might be at least in some way outside of Noether's theorem. It might all depend on whether or not the QV is immutable and indestructible. What do you think?

BR,
-CW

It's not so much symmetry of space (or time) that Noether's theorem concerns, but rather the fields within and the interactions possible with their forces.  So for example an Alcubierre drive doesn't transgress the principle - the physics within the bubble remaining entirely consistent with it.  Rather than viewing a symmetry break as a Noether exception, it's more fruitful to consider how it works within the framework. 

As Frobnicat alluded  a few posts back, any apparent violation merely indicates the system is open, albeit unexpectedly so.  One possible way of negotiating around the constrainsts is to play off two different interactions against each other, wherein both interactions are fully conservative in their own right, but the exchange of energy between them is subject to time-dependent variations in one or other, resulting in an open system from the interplay between two closed ones.  I gave an example a few pages back of such a non-dissipative non-conservative EM exchange, however in reality it's dissipatiting energy into the vacuum.

If the EM drive is doing something similar, it would instead be drawing energy from the vacuum, so Noether, CoE and CoM would still all be satisfied...

ETA:  or rather, momentum from the vacuum - Shawyer states clearly that his drive fully conforms to CoE, so draws onboard power equal to any work done.
Title: Re: EM Drive Developments - related to space flight applications - Thread 2
Post by: Notsosureofit on 05/11/2015 12:20 pm
Do higher em frequencies deliver increased net effect?
Excellent question. To which as yet we have no answers. This might be taken to imply that we should find out :)

But but but...there is no guarantee that these forces in any way combine to produce a larger force. And that's static I mean. I'm even more doubtful about the free space dynamics.

Statically we care about k, or N/W
Dynamically we don't know what we care about yet  8)
We have three tentative answers.  They are the mathematical formulas based on their respective theoretical models:

Quote from: John von Neumann
The sciences do not try to explain, they hardly even try to interpret, they mainly make models. By a model is meant a mathematical construct which, with the addition of certain verbal interpretations, describes observed phenomena. The justification of such a mathematical construct is solely and precisely that it is expected to work.

If a theory does not have a mathematical formula predicting variables, it doesn't qualify as a physical theory.

The predictive formulas of Shawyer and of McCulloch both are inversely proportional to the frequency.  So, on the contrary, both Shawyer and McCulloch predict that the higher the frequency the smaller the thrust force.

The formulas of Shawyer and of McCulloch have no dependence on mode shape.

@Notsosureofit's formula is the only formula that shows dependence on mode shape (through the Bessel zeros, Xmn or X'mn) and hence it has a more nuanced, subtle dependence on frequency. 

As the Bessel zeros Xmn (for TM modes) and X'mn (for TE modes) increase with frequency, @Notsosureofit's thrust force increases with frequency for mode shapes with (both) high m and n (circular and radial) quantum numbers (because Xmn and X'mn increase with frequency at a faster rate than the frequency itself)

 @Notsosureofit's thrust force decreases with frequency for mode shapes having (both) low m and n (circular and radial) quantum numbers (but in this case some of these low m and n mode shapes may be cut off at higher frequencies).


Testing experiments have not yet been performed at a large enough range of frequencies to discriminate whether any of these formulas is correct with respect to frequency variation.

In this context, performing experiments at higher microwave frequencies, perhaps using a Gunn Diode would be most helpful.


Really hadn't thought about it, but at large X, isn't f prop X ?  and NT prop X^2/f^3  ??
Title: Re: EM Drive Developments - related to space flight applications - Thread 2
Post by: Rodal on 05/11/2015 12:38 pm
...
Really hadn't thought about it, but at large X, isn't f prop X ?  and NT prop X^2/f^3  ??
My recollection (from running numerical examples at the time at which you were planning to run experiments with a Gunn diode) is that at large Xmn, with both m~n simultaneously highest, Xmn increases higher than f, so that 

Xmn (f) ~ f ^y  where y>1

Need y>3/2 in order for thrust force to increase with frequency.

My recollection is that the thrust force predictions using your formula for the Gunn Diode frequency were much higher than the predictions of Shawyer and McCulloch.

EDIT: Also the calculations for Shawyer's Flight Thruster (which I recall was run at higher frequency: 3.85GHz, twice the frequency of NASA's EM Drive tests)

Is my memory correct ?  (too bad that we don't have a good search function to look for things like that  :(  )
Title: Re: EM Drive Developments - related to space flight applications - Thread 2
Post by: deltaMass on 05/11/2015 01:03 pm
It occurs to me to ask a history question about all this. What on Earth occasioned Shawyer in the first place to carefully measure thrust on this odd-shaped cavity?
Title: Re: EM Drive Developments - related to space flight applications - Thread 2
Post by: TheTraveller on 05/11/2015 01:24 pm
It occurs to me to ask a history question about all this. What on Earth occasioned Shawyer in the first place to carefully measure thrust on this odd-shaped cavity?
Did read Shawyer, back in his sat days, noticed unexpected sat movements when certain on board microwave systems were powered on which could not be explained via conventional expectations.

He did investigations & came up with his theory of what was happening. As I undersrand it, his employers were not interested, he resigned and set up SPR.

Here SPR claims seven reviews got positive results:

Quote
The team claims to have undergone seven independent reviews from experts at BAE Systems, EADS Astrium, Siemens and the IEE. The DTI has awarded the company £125,000 to develop a prototype engine as part of a three-year, £250,000 programme.

Read more: http://www.theengineer.co.uk/news/defying-gravity/266633.article#ixzz3Zq1ARpVP
Title: Re: EM Drive Developments - related to space flight applications - Thread 2
Post by: Rodal on 05/11/2015 01:25 pm
It occurs to me to ask a history question about all this. What on Earth occasioned Shawyer in the first place to carefully measure thrust on this odd-shaped cavity?
Isn't the answer given by the statements Shawyer makes in http://forum.nasaspaceflight.com/index.php?action=dlattach;topic=36313.0;attach=829636 ? , particularly his claim that

Quote from: Shawyer
In a cavity with the correct radius spherical end plates there is no force on the side walls due to the travelling wave, because the walls are parallel to the group velocity vector

This claim that there are no forces on the side walls is contradicted by a calculation of Maxwell's stress tensor, particularly under classical electromagnetism, as explicitly shown by Greg Egan  (http://gregegan.customer.netspace.net.au/SCIENCE/Cavity/Cavity.html ) for example for such a truncated cone with spherical ends.  Shawyer (to my knowledge) has never adequately answered Greg Egans's proof.

However, if one believes (as Shawyer does) that there are no forces on the conical side walls, then Shawyer's construction immediately follows.

NOTE: To my knowledge, no closed cavity truncated cones (as in Shawyer's EM Drive) are used for microwave communication satellites.
Title: Re: EM Drive Developments - related to space flight applications - Thread 2
Post by: Notsosureofit on 05/11/2015 01:31 pm
...
Really hadn't thought about it, but at large X, isn't f prop X ?  and NT prop X^2/f^3  ??
My recollection (from running numerical examples at the time at which you were planning to run experiments with a Gunn diode) is that at large Xmn, with both m~n simultaneously highest, Xmn increases higher than f, so that 

Xmn (f) ~ f ^y  where y>1

Need y>3/2 in order for thrust force to increase with frequency.

My recollection is that the thrust force predictions using your formula for the Gunn Diode frequency were much higher than the predictions of Shawyer and McCulloch.

EDIT: Also the calculations for Shawyer's Flight Thruster (which I recall was run at higher frequency: 3.85GHz, twice the frequency of NASA's EM Drive tests)

Is my memory correct ?  (too bad that we don't have a good search function to look for things like that  :(  )

Can't remember either, that may have been before I did the units check and noticed the 1/f error (typo on post-it notes) in N (number of photons).

I should go back and set up a graphing program for these formulas.  Cavity dimensions scale as 1/f just to add to the mess.   

I never did get back to the cylindrical w/ dielectric case.   Except for a uniform change, most dielectric surfaces need to include a reflection and you need an iterative program to solve them numerically.  (wrote some of these for optical filter companies in the dim dark past)   It would be nice to have an integral solution that could be optimized.

Just a note:  CoM from spacial symmetry is thought to be local from present cosmology, or so I believe, really haven't checked.

Title: Re: EM Drive Developments - related to space flight applications - Thread 2
Post by: rfmwguy on 05/11/2015 01:38 pm
Bad humor alert: If someone comes up with a formula that makes the ideal frustum look like Die Glocke, I'm outta here. ;)
Title: Re: EM Drive Developments - related to space flight applications - Thread 2
Post by: WarpTech on 05/11/2015 02:19 pm
It occurs to me to ask a history question about all this. What on Earth occasioned Shawyer in the first place to carefully measure thrust on this odd-shaped cavity?
Isn't the answer given by the statements Shawyer makes in http://forum.nasaspaceflight.com/index.php?action=dlattach;topic=36313.0;attach=829636 ? , particularly his claim that

Quote from: Shawyer
In a cavity with the correct radius spherical end plates there is no force on the side walls due to the travelling wave, because the walls are parallel to the group velocity vector

This claim that there are no forces on the side walls is contradicted by a calculation of Maxwell's stress tensor, particularly under classical electromagnetism, as explicitly shown by Greg Egan  (http://gregegan.customer.netspace.net.au/SCIENCE/Cavity/Cavity.html ) for example for such a truncated cone with spherical ends.  Shawyer (to my knowledge) has never adequately answered Greg Egans's proof.

However, if one believes (as Shawyer does) that there are no forces on the conical side walls, then Shawyer's construction immediately follows.

NOTE: To my knowledge, no closed cavity truncated cones (as in Shawyer's EM Drive) are used for microwave communication satellites.


Correct. The group velocity is limited precisely because, there is EM drag on the walls of the waveguide.

Title: Re: EM Drive Developments - related to space flight applications - Thread 2
Post by: Rodal on 05/11/2015 02:51 pm
...
Quote
The team claims to have undergone seven independent reviews from experts at BAE Systems, EADS Astrium, Siemens and the IEE. The DTI has awarded the company £125,000 to develop a prototype engine as part of a three-year, £250,000 programme.

Read more: http://www.theengineer.co.uk/news/defying-gravity/266633.article#ixzz3Zq1ARpVP

See how Shawyer's explanation is quoted (Bold and Color added for emphasis) in that article, particularly the claim that the electromagnetic wave is travelling at a speed of only 0.1 c at the small end of the EM Drive: !

Quote
At the wider end of the cone the wave travels at the speed of light, while at the other the wave travels at one tenth of that speed, due to the geometry of the waveguide.

This creates higher radiation pressure at the wider end of the waveguide because the rate of change of momentum of the waves is different. Newton’s second law defines force as the rate of change of momentum.

Shawyer explained that if these forces were the result of a working fluid, there would merely be a mechanical strain in the waveguide walls. But as the working fluid is replaced by an electromagnetic wave at close to the speed of light, Newtonian mechanics are replaced with the special theory of relativity.

’The electromagnetic wave is going at very high velocities, so you have to apply two different reference planes,’ he said. ’It can no longer be considered a closed system. As soon as you approach the speed of light the wave can be considered completely independent of the waveguide.’ Shawyer compared the engine to a laser gyroscope, which also relies on Einstein’s laws, where attitude information is obtained from an apparently closed system.

In a ring laser gyroscope, two counter-rotating beams are channeled to a photo detector. If the spaceship is not rotating, the beams remain in phase. If rotation is occurring, one beam continuously changes phase with respect to the other. A diode translates that moving interference pattern into digital pulses, each pulse representing an angle of rotation. The rate at which the pulses are produced is also a measure of the rate of rotation.

(https://upload.wikimedia.org/wikipedia/en/c/c8/Ring_laser_interferometer.png)

How can the principle of operation of the ring laser gyroscope be compared to the principle of operation of EM Drive thrust, escapes me...
Title: Re: EM Drive Developments - related to space flight applications - Thread 2
Post by: Notsosureofit on 05/11/2015 02:57 pm
Just another idle thought while trying to jog my memory:

If evanescent waves turn out to be a coupling mechanism (to the universe by virtue of FTL propagation ???) then high-Q dielectric resonators have a big advantage over metallic chambers.  Such coupling was used in cold-war radars w/ spherical resonators for stability reasons.  (very low coupling constant, etc.  somewhere I probably still have a few Russian ones in a drawer)

As long as I'm rambling far afield:  Any thoughts on how to incorporate the super-regeneration effect into the system.
 That would mean pulsed (quenched) oscillation which, if anything, might detect interfering back ground signals. ???

Title: Re: EM Drive Developments - related to space flight applications - Thread 2
Post by: Blaine on 05/11/2015 02:58 pm
It's unclear from these forum posts if the prototype propulsion system actually generated any thrust during the recent tests, said Ethan Siegel, a physics and astronomy professor at Lewis & Clark College in Portland, Oregon
(http://www.space.com/29363-impossible-em-drive-space-engine-nasa.html)

Basically a new article that I like for the most part as its not too optimistic, but not too pessimistic either.  However the part that I quoted above is clearly mistaken.  As thrust was generated and is shown on this site and the Nasa test results.
Title: Re: EM Drive Developments - related to space flight applications - Thread 2
Post by: Flyby on 05/11/2015 03:54 pm
I'm wondering, in the context of dr. rodal's remark on Shawyer's theory, where he (Shawyer) poses that forces are to be observed on the ends only, what would happen if one or both ends were to be replace by Cullen type ring reflector or mesh?
Would one still observe forces of the same magnitude?

From what I understood, waves would still be bouncing yet the surface upon which a force can be applied, or a momentum be transferred to would be drastically reduced...

Somewhere , somehow, if this device really works, there must be a momentum transfer onto the frustum....

If it is uncertain what role the end plates play, why try dielectric materials, or materials with a high magnetic permeability (this subject got completely lost in the current discussion?) on the side walls of the frustum to see if any drag effect is in effect?
Title: Re: EM Drive Developments - related to space flight applications - Thread 2
Post by: Einstein79 on 05/11/2015 04:05 pm
The reason for the confusion over the violation of classical physics is because this system has nothing to do with classical physics. Moreover, the “thrust” that is being calculated is not thrust at all but space moving the drive from one position to another which can merely be related to thrust but is not, per se, thrust. The controlling factor here is, of course, the resonant frequency. If you match the resonant frequency that space uses to “hold” the object you will develop a “cavity” that the “object will move towards”. The reason why the device cannot be “pushed off of” for conservation of momentum to hold true is because space is already pushing on it satisfying the law.

A couple of postulates to keep in mind that will help with these experiments are:
1. Space creates light.
2. Space itself is a resonating chamber.
Interesting! Would you then be prepared to write down the equations of motion so that we can play with them?

The Mexican hat potential is a good place to start, I think, but not sure yet. It seems reasonable because it might coalesce with symmetry breaking.

You and Mulletron seem to be going in the same direction, you might want to bounce ideas off each other...

YES please! If there are any others trying to tackle this problem this way we certainly need to put our heads together. I think we are being overly literal when speaking of the shape of the resonating cavity. The article that Dr. Rodal quoted stated that Dr. White was creating a warp bubble is on point because this "warp bubble" should be the resonating cavity itself in order to be in agreement with general relativity. Therefore, it does not matter what shape the EM drive is so long as it creates the fields necessary to generate the warp bubble/resonating chamber we need.
Title: Re: EM Drive Developments - related to space flight applications - Thread 2
Post by: Einstein79 on 05/11/2015 04:19 pm
I'm wondering, in the context of dr. rodal's remark on Shawyer's theory, where he (Shawyer) poses that forces are to be observed on the ends only, what would happen if one or both ends were to be replace by Cullen type ring reflector or mesh?
Would one still observe forces of the same magnitude?

From what I understood, waves would still be bouncing yet the surface upon which a force can be applied, or a momentum be transferred to would be drastically reduced...

Somewhere , somehow, if this device really works, there must be a momentum transfer onto the frustum....

If it is uncertain what role the end plates play, why try dielectric materials, or materials with a high magnetic permeability (this subject got completely lost in the current discussion?) on the side walls of the frustum to see if any drag effect is in effect?

This device does really work and there need not be a momentum transfer onto the frustum because the momentum transfer is between the warp bubble and space-time itself. Remember, Dr. White's intention is to create a warp bubble where everything inside this warp bubble (the drive) are unperturbed.
Title: Re: EM Drive Developments - related to space flight applications - Thread 2
Post by: seeingstars on 05/11/2015 05:11 pm
There was discussion early in the thread of the claimed (once-claimed?  still-claimed?  not sure) mechanism of ExB drift on e+/e- pairs driving the thrust.  This was later muddied a bit by some of March's comments that thrust is not related to the Poynting vector direction after all, but I am still trying to wrap my head around the idea of whether a net ExB drift in a cavity is ever a possibility.  I don't immediately see how you could get a nonzero ExB drift integrated over an RF cycle. 

Statements I think are true:
1) E and B fields in a waveguide or cavity will be mutually perpendicular at all points
2) They will oscillate like sines in time (with more complicated spatial structure, for sure)
3) The ExB drift velocity magnitude will likewise oscillate in time
4) The net thrust produced by causing e+/e- pairs to ExB drift will be proportional to the (double) integral of the cross-product ExB over an RF cycle and over the cavity volume
5) This integral will go identically to zero because you can separate the time and space integrals, and the time integral will go to zero

Mathematically my Jackson-fu is rusty but I think you avoid sin^2 integrals (which would be nonzero) because E and B are perpendicular or out of phase, so in any vector direction you're really integrating a sin(t)*cos(t) term which is equivalent to sin(2t).

The above is a micro view.  From a macro view, the ExB drift is in the same direction as the Poynting vector or radiation pressure, so again I'd expect in classical E&M with conservation of momentum that this would integrate to zero over the interior walls of the cavity.  The only niggle I have there is you're pumping energy into the cavity from a source port, so that energy must flow somewhere to dissipate; perhaps this gives a net energy flow and thus net ExB drift direction.  I don't see where dielectrics come into the picture at all, I'm afraid. 

The point of all this is that I could accept a funny structure in classical E&M that produces net ExB if I could wrap my head around how that's possible, and then say "maybe" to pushing on funny e+/e- pairs pending clear experimental evidence.  But I can't see how you make the cavity to do the pushing in the first place. 
Title: Re: EM Drive Developments - related to space flight applications - Thread 2
Post by: Mulletron on 05/11/2015 05:30 pm
We really need to take a stand and not entertain further discussion of "pushing on virtual particles" or referring to the QV as a plasma. The overwhelming scientific consensus is that the quantum vacuum doesn't work this way.
Title: Re: EM Drive Developments - related to space flight applications - Thread 2
Post by: Rodal on 05/11/2015 05:42 pm
We really need to take a stand and not entertain further discussion of "pushing on virtual particles" or referring to the QV as a plasma. The overwhelming scientific consensus is that the quantum vacuum doesn't work this way.

There is also overwhelming consensus that:

1) Conservation of Momentum is a universal principle
2) Conservation of Energy is a universal principle
3) Maxwell's stress tensor components on the lateral surfaces of a conical cavity are not zero (resonating with electromagnetic waves). (There are forces on the conical walls of the EM Drive that cannot be ignored).
4) The principle of operation of the ring laser gyroscope does not justify EM Drive thrust,
5) Electromagnetic waves are not travelling at a speed of only 0.1 c at the small end of the EM Drive
6) Finite Element Analysis based on variational, virtual energy principles (like COMSOL, ANSYS, ABAQUS, etc.) should not predict a net Lorentz force or a Poynting vector for the EM Drive over a whole period or multiples thereof, because FEA satisfies conservation of momentum.
7) etc. etc.

I adhere and support all six of the above.


Why should we not entertain further discussion of any of these topics?

Having to support and defend the above-mentioned arguments and many more, based on technical arguments, experiments and math, (instead of name calling) keeps our brains young and creative.

Hopefully such discussions will lead us to find the truth, and if we don't get there, at least they are fun to discuss  :)
Title: Re: EM Drive Developments - related to space flight applications - Thread 2
Post by: Mulletron on 05/11/2015 06:12 pm
Good. We are in violent agreement to self regulate this thread and keep pseudoscience out. Otherwise, this effort is lost.
Title: Re: EM Drive Developments - related to space flight applications - Thread 2
Post by: tchernik on 05/11/2015 06:29 pm
I wholeheartedly agree with the intention of the smart people around here, helping the rest of us to filter the science from the chaff.

Nevertheless, I assume there is going to be a break point: either the Emdrive proves to deliver some anomalous thrust or it doesn't.

If it does, such anomaly would maybe go against some of the scientific principles stated here (as very pointedly remarked by frobnicat et al).

But such proof by counterexample would need to be really overwhelming and repeatable in order to be accepted. Is that correct?
Title: Re: EM Drive Developments - related to space flight applications - Thread 2
Post by: RonM on 05/11/2015 06:54 pm
Good. We are in violent agreement to self regulate this thread and keep pseudoscience out. Otherwise, this effort is lost.

Perhaps it is time to start a new thread with stricter rules clearly listed in the OP. Point and move other comments to the Feature Article thread. I think that would be fair to everyone.

Anyway, this thread is getting pretty long.

Everyone, keep up the good work. This has been a fascinating and educational thread.
Title: Re: EM Drive Developments - related to space flight applications - Thread 2
Post by: WarpTech on 05/11/2015 07:02 pm
...
I know a photon is not a ball but my question is, in "Newtonian layman's terms" how does the line of thinking you are developing making that analogy not valid, i.e. imply apparent deviation from conservation of momentum ?

The ball (photon) doesn't fall back down the well. There is nothing to give it back enough energy to do so. It dissipates in multiple reflections between the walls and the big end. They are not getting more out than they put in, so it does not violate conservation of energy. They are simply getting more NET momentum on one direction than in the other direction because there is more dissipation and attenuation in one direction than there is in the other. Dissipative systems are typically "not" conservative, loses prevent a true equal measure from occuring in both directions.

Todd D.

Classically, a dissipative system is conservative for both momentum and energy, it's just that for energy there is a (irreversible) conversion to a degraded form of energy, but there is no such thing as a mysterious part of total energy that would simply vanish. Even if not always convenient, an open system can be seen as part of a bigger closed system, and short of that the deltas total energy and total momentum of an open system can still be accounted, at least in principle, as integrated fluxes exchanged between open system and an outside.

My Newtonian ball of momentum pb can encounter an arbitrarily varying Force Fcb(t) of container on ball (vectors in bold). And dpb/dt=Fcb=-Fbc=-dpc/dt. That is instant conservation of momentum, and obviously integrating on successive instants just yields delta_pb=-delta_pc or  delta_pb+delta_pc=0, conservation of momentum on any time interval whatever the shape of varying Force Fcb(t). Where and how quantitatively your system is showing an apparent breaking of CoM at an "instantaneous scale" dt ? Short of that, details of trajectory is just, ahem, arm waving for propulsion purpose (aka Dean drive).

At the "instantaneous scale"  there are collisions between photons and atoms where momentum is transferred and it generates heat but not thrust. When the photons are injected their momentum is p1 and energy is E1. It can only conserve NET momentum if there is a 50/50 probability that momentum is absorbed in each direction, without generating ANY heat at all. As soon as things start getting hot, the probability is not 50/50 anymore, then some of the momentum is not being absorbed as thrust, but rather to heat up the metal. Therefore, the NET momentum in either direction will depend on the difference in the dissipation and attenuation, in each direction.

Todd D.
Title: Re: EM Drive Developments - related to space flight applications - Thread 2
Post by: Rodal on 05/11/2015 07:09 pm
...

At the "instantaneous scale"  there are collisions between photons and atoms where momentum is transferred and it generates heat but not thrust. When the photons are injected their momentum is p1 and energy is E1. It can only conserve NET momentum if there is a 50/50 probability that momentum is absorbed in each direction, without generating ANY heat at all. As soon as things start getting hot, the probability is not 50/50 anymore, then some of the momentum is not being absorbed as thrust, but rather to heat up the metal. Therefore, the NET momentum in either direction will depend on the difference in the dissipation and attenuation, in each direction.

Todd D.
Yes, the problem is, however, how would this, quantitatively, result in a more efficient (thrust/PowerInput) propellant-less drive than a perfectly-collimated photon rocket.

Another fascinating observation that @frobnicat made early on, is that for photons (whether tunneling, dissipation or another mechanism) to end up producing a more efficient drive than a photon rocket, the photons would have to escape the EM Drive as tachyons, superluminally.
Title: Re: EM Drive Developments - related to space flight applications - Thread 2
Post by: SeeShells on 05/11/2015 07:09 pm
I know I'm new here but I've been in engineering for almost 50 years. The EM drive seems to parallel so many things I've seen in electronics and embrace harmonics and it got me thinking how it would compare to things like a YAGI antenna for gain buy linking them in series. Would you get a Q gain in thrust?
https://www.youtube.com/watch?v=Wokn7crjBbA
Great thought!
I'm digging on this idea right now but more than Q it seems like the phasing has some profound effects as well. Not a lot of data there, but what little I've gleaned it looks like something I want to pursue. I've been refreshing my old vacuum tube/crystal radio school education so I can wrap my brain around it. I have some ideas but they are not ready to throw out here quite yet.
I found a little blurb in a presentation where Dr. White (Sonny) briefly mentioned one of the tests they were thinking of was to put another passive EM device behind the active one to see if they could discern a pattern. To me this sounded like an YAGI and a great idea. Stack them up like a YAGI I thought, phase them to increase thrust.
Title: Re: EM Drive Developments - related to space flight applications - Thread 2
Post by: Einstein79 on 05/11/2015 07:13 pm
Good. We are in violent agreement to self regulate this thread and keep pseudoscience out. Otherwise, this effort is lost.

This endeavour is by definition "pseudoscience". I.e. it defies all our logic and principles that we have previously and whole heartedly embraced. Space is an actual object and performs work on everything we observe. The "vacuum" does not exist and this experiment is proof of this statement. We must question everything that we have learned and realize that the truth may lie in "pseudoscience". This experiment reveals that we lack understanding of our physical reality and the "laws" we blindly accept as truth are evidence of this misunderstanding because if they were completely accurrate then we would have already solved the problem.     
Title: Re: EM Drive Developments - related to space flight applications - Thread 2
Post by: Rodal on 05/11/2015 07:17 pm
I know I'm new here but I've been in engineering for almost 50 years. The EM drive seems to parallel so many things I've seen in electronics and embrace harmonics and it got me thinking how it would compare to things like a YAGI antenna for gain buy linking them in series. Would you get a Q gain in thrust?
https://www.youtube.com/watch?v=Wokn7crjBbA
Great thought!
I'm digging on this idea right now but more than Q it seems like the phasing has some profound effects as well. Not a lot of data there, but what little I've gleaned it looks like something I want to pursue. I've been refreshing my old vacuum tube/crystal radio school education so I can wrap my brain around it. I have some ideas but they are not ready to throw out here quite yet.
I found a little blurb in a presentation where Dr. White (Sonny) briefly mentioned one of the tests they were thinking of was to put another passive EM device behind the active one to see if they could discern a pattern. To me this sounded like an YAGI and a great idea. Stack them up like a YAGI I thought, phase them to increase thrust.
1) Interesting how that presentation was just a few months ago. That was presented by the Chief Scientist NASA Ames.  How things have changed, given the present official reaction by NASA and NASA Glenn regarding the efforts at NASA Eagleworks.  (The "applications" section of the NSF article that generated so much controversy are old papers by Dr. White that he had published months and years ago).

2) Unfortunately the tiny budget at NASA Eagleworks has prevented them from conducting the proposal to put another passive EM device behind the active one.  Star-Drive posted some time ago that they didn't have another EM Drive to conduct the test  :( 
Title: Re: EM Drive Developments - related to space flight applications - Thread 2
Post by: WarpTech on 05/11/2015 07:31 pm
...

At the "instantaneous scale"  there are collisions between photons and atoms where momentum is transferred and it generates heat but not thrust. When the photons are injected their momentum is p1 and energy is E1. It can only conserve NET momentum if there is a 50/50 probability that momentum is absorbed in each direction, without generating ANY heat at all. As soon as things start getting hot, the probability is not 50/50 anymore, then some of the momentum is not being absorbed as thrust, but rather to heat up the metal. Therefore, the NET momentum in either direction will depend on the difference in the dissipation and attenuation, in each direction.

Todd D.
Yes, the problem is, however, how would this, quantitatively, result in a more efficient (thrust/PowerInput) propellant-less drive than a perfectly-collimated photon rocket.

Another fascinating observation that @frobnicat made early on, is that for photons (whether tunneling, dissipation or another mechanism) to end up producing a more efficient drive than a photon rocket, the photons would have to escape the EM Drive as tachyons, superluminally.

Where is the calculation for a photon rocket comparison? When calculating the photon rocket thrust was the input power used, or the input power multiplied by the Q? I don't think I've seen that yet. I seriously need to start keeping a spreadsheet of data, tests, results and discussions. It's too much to keep track of in my head.
Title: Re: EM Drive Developments - related to space flight applications - Thread 2
Post by: SeeShells on 05/11/2015 07:33 pm

1) Interesting how that presentation was just a few months ago. That was presented by the Chief Scientist NASA Ames.  How things have changed, given the present official reaction by NASA and NASA Glenn regarding the efforts at NASA Eagleworks.  (The "applications" section of the NSF article that generated so much controversy are old papers by Dr. White that he had published months and years ago).

2) Unfortunately the tiny budget at NASA Eagleworks has prevented them from conducting the proposal to put another passive EM device behind the active one.  Star-Drive posted some time ago that they didn't have another EM Drive to conduct the test  :(
My intuition gave me patents (have more than a few) but the science gave me understanding. My intuition says that you'll find a solution, funding can happen and greater public exposure will help. On this point I will write my congressman and anyone else who can help push, cajole or twist an arm and I would urge anyone else to do. I have no doubt something is there and that something needs good science to resolve it. The investment is not big, but the ROI is the stars (well, maybe the local neighborhood). ;)
Title: Re: EM Drive Developments - related to space flight applications - Thread 2
Post by: Rodal on 05/11/2015 07:36 pm
...
Where is the calculation for a photon rocket comparison? When calculating the photon rocket thrust was the input power used, or the input power multiplied by the Q? I don't think I've seen that yet. I seriously need to start keeping a spreadsheet of data, tests, results and discussions. It's too much to keep track of in my head.

See:

1) http://forum.nasaspaceflight.com/index.php?topic=36313.msg1302455#msg1302455

2) http://forum.nasaspaceflight.com/index.php?topic=29276.msg1276053#msg1276053

Title: Re: EM Drive Developments - related to space flight applications - Thread 2
Post by: zlspradlin on 05/11/2015 07:48 pm
I'm new here and I just thought I would post this video for you all.  Its a very VERY sloppy experimental setup of something like the what people on this forum are talking about.  The interesting thing here is the man in the video doesn't use end-plates and its quite a bit slimmer than the EM Drive.  Here is the video:

youtube.com/watch?v=vcaOKX7Ko7w

What are some thoughts about the video posted?

Rough translation (From original in Russian):

Quote
Created Shawyer  (EM Drive) engine is very easy and simple in its design . It provides the necessary thrust " by the oscillation of the microwaves inside the vacuum container ."
http: //hi-news.ru/technology/v-nasa-i ...
I decided that the system should not be closed



It is a waveguide with one end open.  The Russian author points out (later in his Russian text) that he thinks that Shawyer, and others are wrong in using a closed cavity.

The reference (Cullen) given by Shawyer to support his theory also used in his experimental measurements of pressure, a cavity with one end open (with a transparent glass)


It is known that a microwave waveguide having one end open will display directional thrust, as the microwave photons escape the waveguide.  The problem with the EM Drive is that it is a closed cavity, hence it cannot be explained solely based on Maxwell's equations. Something else is needed: General Relativity, QV,  something else.

A waveguide with one end open will behave as a very inefficient photon rocket: thousands of times less thrust per power input than what is claimed in the EM Drive experiments.

Would it be possible to amplify the waves in the cavity, trying to create a high pressure scenario that perhaps could be an efficient "photon" rocket? Can these waves be pressurized? Is this pointless in speculating at this point in time?

I had a thought over the weekend and saw that at least one other poster had posted about CMB (cosmic microwave background) but that discussion quickly died and I'm not sure why. Seeing as CMB is basically homogenous and permeates in all directions couldn't this help explain the thrust? Perhaps "warping" (for lack of a better word) around the CMB, or the CMB around the drive?
Title: Re: EM Drive Developments - related to space flight applications - Thread 2
Post by: RotoSequence on 05/11/2015 07:49 pm
Good. We are in violent agreement to self regulate this thread and keep pseudoscience out. Otherwise, this effort is lost.

This endeavour is by definition "pseudoscience". I.e. it defies all our logic and principles that we have previously and whole heartedly embraced. Space is an actual object and performs work on everything we observe. The "vacuum" does not exist and this experiment is proof of this statement. We must question everything that we have learned and realize that the truth may lie in "pseudoscience". This experiment reveals that we lack understanding of our physical reality and the "laws" we blindly accept as truth are evidence of this misunderstanding because if they were completely accurrate then we would have already solved the problem.     

This is a strange definition of pseudoscience, and seems somewhat distant from its actual meaning. Pseudoscience involves false, or otherwise inaccurate claims of adherence to the scientific method.
Title: Re: EM Drive Developments - related to space flight applications - Thread 2
Post by: Flyby on 05/11/2015 07:51 pm
I'm wondering, in the context of dr. rodal's remark on Shawyer's theory, where he (Shawyer) poses that forces are to be observed on the ends only, what would happen if one or both ends were to be replace by Cullen type ring reflector or mesh?
Would one still observe forces of the same magnitude?

From what I understood, waves would still be bouncing yet the surface upon which a force can be applied, or a momentum be transferred to would be drastically reduced...

Somewhere , somehow, if this device really works, there must be a momentum transfer onto the frustum....

If it is uncertain what role the end plates play, why try dielectric materials, or materials with a high magnetic permeability (this subject got completely lost in the current discussion?) on the side walls of the frustum to see if any drag effect is in effect?

This device does really work and there need not be a momentum transfer onto the frustum because the momentum transfer is between the warp bubble and space-time itself. Remember, Dr. White's intention is to create a warp bubble where everything inside this warp bubble (the drive) are unperturbed.

may i bring in a few small but significant corrections?

The device does not" really work", but it SEEMS to work...
Perception is a different thing then certitude..

There are precedences of test setups that appear to be working, yet have not been able to defend themselves very well again other possible solutions. The proof of its working is still fragmentary and most certainly needs further testing to validate it is actually functioning.
I'm not a "believer" or "non-believer" but it is about gathering enough irrefutable evidence to give credibility to the device.
I do not want to see a smoking gun, i want to hear and see the gunshot... (hence my anticipation for the upcoming July test at EW)

As for the warp bubble, it is just - and nothing more - one of the possible mathematical constructs/theories that try to give a theoretical framework for the observations.
And theories can be debates endlessly as long there is no hard experimental data.
In the end it is experimental data that defines the preferred theory that gives a satisfactory explanation...



On a personal level, being thought only old school newtonian physics, I do have some reluctance to accept quantum mechanics, but i wont exclude it as a possibility...

As it is now, my preference goes to Todd's theoretical model, simply because that's something i can still understand and it makes sense to me, as layman. Not sure if it is THE right theory, but at this stage we're in , that doesn't matter much...(yet)
Title: Re: EM Drive Developments - related to space flight applications - Thread 2
Post by: Rodal on 05/11/2015 08:10 pm
...
Would it be possible to amplify the waves in the cavity, trying to create a high pressure scenario that perhaps could be an efficient "photon" rocket? Can these waves be pressurized?
The waves are electromagnetic waves with photons at a microwave frequency.  They can be Maser [ https://en.wikipedia.org/wiki/Maser ] which is very expensive -beyond the reach of the NASA researchers' budget- and whose goal would be unclear, given the present understanding.



...Totally random thought but what if y = e
e is not random  :)  (e is like an encrypted message: its digits are in a random sequence, but it contains a huge amount of information, if one has the key  :) )

This is not random either, it is beautiful:  (https://upload.wikimedia.org/math/f/8/9/f897005615c391e14cd50112cda44665.png)



...As it is now, my preference goes to Todd's theoretical model, simply because that's something i can still understand and it makes sense to me, as layman. Not sure if it is THE right theory, but at this stage we're in , that doesn't matter much...(yet)
We are very lucky to have Todd and all other newcomers join our forum.  It is delightful to see how clearly Todd writes about GR, and the flood of new ideas and experience being shared in this thread.  All the pain and suffering resulting from the commingling of the NSF article with the fringe articles  was well worth it if as a result of it we received a flood of new blood with fresh eyes and fresh brains on this matter  :)
Title: Re: EM Drive Developments - related to space flight applications - Thread 2
Post by: zlspradlin on 05/11/2015 08:27 pm
Thanks for the response, and thank to all of you who even bother reading my posts. I'm so privileged to be here talking with minds like yours. I really do want to keep crack-pot and "pseudoscience" to a minimum, but I'm afraid I've not been classically trained. But I do have a passion & (very) open mind.

As for the Maser, what about this discovery: http://www.nature.com/news/microwave-laser-fulfills-60-years-of-promise-1.11199

Could NASA not handle something like this? I wish I could give NASA the DEA's budget... this is so frustrating.

Quote
The laser light excited the pentacene molecules to an energy level known as a metastable state. Then a microwave passing through the crystal triggered the molecules to relax, releasing a cascade of microwaves of the same wavelength.

It was the same principle as an optical laser. "The signal that came out of it was huge," says Oxborrow, about a hundred million times as powerful as an existing maser. Alone in his lab, "I swore a lot and walked around the corridor about five times talking to myself".

Seems like the goal, one of them anyway, could be to reduce the input energy? Also, presuming this drive is working as it seems that it does, and the fact that we don't understand it very well or at all, this could be one of those "Hobbyist/Hacker Hunches" (which is all I have to contribute at this time, sorry!).
Title: Re: EM Drive Developments - related to space flight applications - Thread 2
Post by: deltaMass on 05/11/2015 08:29 pm
I wholeheartedly agree with the intention of the smart people around here, helping the rest of us to filter the science from the chaff.

Nevertheless, I assume there is going to be a break point: either the Emdrive proves to deliver some anomalous thrust or it doesn't.

If it does, such anomaly would maybe go against some of the scientific principles stated here (as very pointedly remarked by frobnicat et al).

But such proof by counterexample would need to be really overwhelming and repeatable in order to be accepted. Is that correct?
You can call me Al  ;D
Title: Re: EM Drive Developments - related to space flight applications - Thread 2
Post by: Rodal on 05/11/2015 08:32 pm
...
Really hadn't thought about it, but at large X, isn't f prop X ?  and NT prop X^2/f^3  ??
My recollection (from running numerical examples at the time at which you were planning to run experiments with a Gunn diode) is that at large Xmn, with both m~n simultaneously highest, Xmn increases higher than f, so that 

Xmn (f) ~ f ^y  where y>1

Need y>3/2 in order for thrust force to increase with frequency.

My recollection is that the thrust force predictions using your formula for the Gunn Diode frequency were much higher than the predictions of Shawyer and McCulloch.

EDIT: Also the calculations for Shawyer's Flight Thruster (which I recall was run at higher frequency: 3.85GHz, twice the frequency of NASA's EM Drive tests)

Is my memory correct ?  (too bad that we don't have a good search function to look for things like that  :(  )

Can't remember either, that may have been before I did the units check and noticed the 1/f error (typo on post-it notes) in N (number of photons).

I should go back and set up a graphing program for these formulas.  Cavity dimensions scale as 1/f just to add to the mess.   

I never did get back to the cylindrical w/ dielectric case.   Except for a uniform change, most dielectric surfaces need to include a reflection and you need an iterative program to solve them numerically.  (wrote some of these for optical filter companies in the dim dark past)   It would be nice to have an integral solution that could be optimized.

Just a note:  CoM from spacial symmetry is thought to be local from present cosmology, or so I believe, really haven't checked.

It turns out that, on the mean, all three formulas (Shawyer, McCulloch and Notsosureofit) predict decreasing thrust force with increasing frequency

Notice the subtle effect in Notsosureofit's formula (the only formula that contains the mode shapes) showing that at low frequencies, some of the modes have an increasing force with higher frequency, but this happens over a short range of frequency.

Notsosureofit (not at his office presently) kindly sent me an Excel file, whose image I'm attaching below (i "beautified" it a bit -which is subjective-, and I gave him credit on the image's title, if there are any errors they rest on my shoulders, the credit goes to Notsosureofit)

attachment:  Excel file from Notsosureofit / Image mangled by Rodal  ;)
Title: Re: EM Drive Developments - related to space flight applications - Thread 2
Post by: Notsosureofit on 05/11/2015 08:50 pm
Thanks for the response, and thank to all of you who even bother reading my posts. I'm so privileged to be here talking with minds like yours. I really do want to keep crack-pot and "pseudoscience" to a minimum, but I'm afraid I've not been classically trained. But I do have a passion & (very) open mind.

As for the Maser, what about this discovery: http://www.nature.com/news/microwave-laser-fulfills-60-years-of-promise-1.11199

Could NASA not handle something like this? I wish I could give NASA the DEA's budget... this is so frustrating.

Quote
The laser light excited the pentacene molecules to an energy level known as a metastable state. Then a microwave passing through the crystal triggered the molecules to relax, releasing a cascade of microwaves of the same wavelength.

It was the same principle as an optical laser. "The signal that came out of it was huge," says Oxborrow, about a hundred million times as powerful as an existing maser. Alone in his lab, "I swore a lot and walked around the corridor about five times talking to myself".

Seems like the goal, one of them anyway, could be to reduce the input energy? Also, presuming this drive is working as it seems that it does, and the fact that we don't understand it very well or at all, this could be one of those "Hobbyist/Hacker Hunches" (which is all I have to contribute at this time, sorry!).

If it works as a maser then it must have a reasonably high index at those frequencies (or it was in some sort of cavity)  Too bad it wasn't frustrum shaped.

OK so it was inside a tuned cavity.
Title: Re: EM Drive Developments - related to space flight applications - Thread 2
Post by: deltaMass on 05/11/2015 09:06 pm
Where is the calculation for a photon rocket comparison? When calculating the photon rocket thrust was the input power used, or the input power multiplied by the Q? I don't think I've seen that yet. I seriously need to start keeping a spreadsheet of data, tests, results and discussions. It's too much to keep track of in my head.
Rodal, I appreciate the synopses you've posted in order to answer this, and the comparative work by NotSoSureOfIt also. It seems that, to within an order or so (and there's nothing especially accurate happening with the modelling of results yet), the thrust can be modelled as Q*(photon rocket).

Is this an acceptable handwave? And if it is, it says something important.
Title: Re: EM Drive Developments - related to space flight applications - Thread 2
Post by: zlspradlin on 05/11/2015 09:11 pm
If it works as a maser then it must have a reasonably high index at those frequencies (or it was in some sort of cavity)  Too bad it wasn't frustrum shaped.

OK so it was inside a tuned cavity.

The tuned cavity being a cube, correct? Like the attached image? Would feeding the output of the maser into a frustrum change Shawyer's findings? Would it have an effect on the perceived thrust at all? Would it be better to make the maser in a frustrum rather than feed the output into the resonant cavity? Is there any reason this should not be further investigated in regards to an EM Drive?
Title: Re: EM Drive Developments - related to space flight applications - Thread 2
Post by: frobnicat on 05/11/2015 09:56 pm
...

At the "instantaneous scale"  there are collisions between photons and atoms where momentum is transferred and it generates heat but not thrust. When the photons are injected their momentum is p1 and energy is E1. It can only conserve NET momentum if there is a 50/50 probability that momentum is absorbed in each direction, without generating ANY heat at all. As soon as things start getting hot, the probability is not 50/50 anymore, then some of the momentum is not being absorbed as thrust, but rather to heat up the metal. Therefore, the NET momentum in either direction will depend on the difference in the dissipation and attenuation, in each direction.

Todd D.
Yes, the problem is, however, how would this, quantitatively, result in a more efficient (thrust/PowerInput) propellant-less drive than a perfectly-collimated photon rocket.

Another fascinating observation that @frobnicat made early on, is that for photons (whether tunneling, dissipation or another mechanism) to end up producing a more efficient drive than a photon rocket, the photons would have to escape the EM Drive as tachyons, superluminally.

Where is the calculation for a photon rocket comparison? When calculating the photon rocket thrust was the input power used, or the input power multiplied by the Q? I don't think I've seen that yet. I seriously need to start keeping a spreadsheet of data, tests, results and discussions. It's too much to keep track of in my head.

There is, in classical interpretation of SR, no reason to multiply the thrust of a photon rocket by Q as the number of time a quantum of momentum carrier bounced around before leaving the rocket for good and living its own trajectory (by definition : as a real particle) is irrelevant. Only the momentum "lost" by a leaving particle in a direction is a "gained" momentum in the opposite direction for a spacecraft.

For a classical SR interpretation of what is possible for a deep space craft assuming no field/bath/mutable_vacuum to swallow and/or push on, see second half of this fascinating :D post ( 3/ ... ) (http://forum.nasaspaceflight.com/index.php?topic=36313.msg1315118#msg1315118)

And the other way around, if a cavity is receiving a flux of particles, there is no way (classically) it can recover more nor less momentum than the initial amount entering the boundary, no matter the path and number and modality of interactions of particles with said cavity, if they are to end their lives within. And not only that would make the emitting system to transfer momentum with no more efficiency than using the equivalent of a photon rocket thrust to blow on a sail  (unless it is sending tachyons), but if the sender of the flux is attached to the receiving cavity then the net momentum delta of the whole process is 0 (assuming nothing leaves).

Only between two different systems exchanging photons back and forth many times a transfer of momentum more efficient (by a factor Q) than the photon rocket limit can occur. This could be used to "beam" a force from system A to system B. But again, if system A and B are rigidly linked within a same craft, this is just a static force within the craft, so the Q factor won't amplify nothing useful. EM internal pressure inside a resonating cavity is proportional to Q but does not help to create net imbalance. And as soon as photons leave (if they leave), there can be net imbalance, but Q is not an amplifying factor of efficiency, there are not more photons leaving (for a given energy input) because they bounced many times before leaving.

I've scratched my head for 15 minutes trying to understand the points made in your previous post about asymmetric dissipation but this is not making sense for me so far, even trying hard to think "out of the box". Will try to address and ask for clarifications when time permits.

edit: blowing on a reflective sail a flux of photon actually allows the emitter to "transmit a force", for a given power, twice that of a photon rocket thrust (best case). Still irrelevant if emitter is rigidly linked to sail : perfectly absorbing sail -> 0 net thrust, perfectly reflecting and collimating sail -> thrust/power=1/c
Title: Re: EM Drive Developments - related to space flight applications - Thread 2
Post by: deltaMass on 05/11/2015 10:03 pm
@frobnicat:
I totally agree that "Q*(photon rocket thrust)" is a nonsense piece of physics on the face of it, for the same reasons you elaborate. However, I was just pointing out that it simply seems to be a good experimental thrust predictor.

Which, of course, is completely weird if you take the thrust measurements as largely unflawed. And no, I don't know what it points to.
Title: Re: EM Drive Developments - related to space flight applications - Thread 2
Post by: Rodal on 05/11/2015 10:10 pm
If it works as a maser then it must have a reasonably high index at those frequencies (or it was in some sort of cavity)  Too bad it wasn't frustrum shaped.

OK so it was inside a tuned cavity.

The tuned cavity being a cube, correct? Like the attached image? Would feeding the output of the maser into a frustrum change Shawyer's findings? Would it have an effect on the perceived thrust at all? Would it be better to make the maser in a frustrum rather than feed the output into the resonant cavity?
All good questions that cannot be answered with the present state of knowledge.  EM Drive researchers have operated with very tiny budgets.

Is there any reason this should not be further investigated in regards to an EM Drive?
Yes, the reason is that NASA Eagleworks doesn't even have the money to test another EM Drive piggybacked in series, the only one they have was made in the living-room of one of the researchers.  A MASER costs hundreds of thousands of dollars, which is way out of their  tiny budget.
Title: Re: EM Drive Developments - related to space flight applications - Thread 2
Post by: WarpTech on 05/11/2015 10:17 pm
...

At the "instantaneous scale"  there are collisions between photons and atoms where momentum is transferred and it generates heat but not thrust. When the photons are injected their momentum is p1 and energy is E1. It can only conserve NET momentum if there is a 50/50 probability that momentum is absorbed in each direction, without generating ANY heat at all. As soon as things start getting hot, the probability is not 50/50 anymore, then some of the momentum is not being absorbed as thrust, but rather to heat up the metal. Therefore, the NET momentum in either direction will depend on the difference in the dissipation and attenuation, in each direction.

Todd D.
Yes, the problem is, however, how would this, quantitatively, result in a more efficient (thrust/PowerInput) propellant-less drive than a perfectly-collimated photon rocket.

Another fascinating observation that @frobnicat made early on, is that for photons (whether tunneling, dissipation or another mechanism) to end up producing a more efficient drive than a photon rocket, the photons would have to escape the EM Drive as tachyons, superluminally.

Where is the calculation for a photon rocket comparison? When calculating the photon rocket thrust was the input power used, or the input power multiplied by the Q? I don't think I've seen that yet. I seriously need to start keeping a spreadsheet of data, tests, results and discussions. It's too much to keep track of in my head.

There is, in classical interpretation of SR, no reason to multiply the thrust of a photon rocket by Q as the number of time a quantum of momentum carrier bounced around before leaving the rocket for good and living its own trajectory (by definition : as a real particle) is irrelevant. Only the momentum "lost" by a leaving particle in a direction is a "gained" momentum in the opposite direction for a spacecraft.

For a classical SR interpretation of what is possible for a deep space craft assuming no field/bath/mutable_vacuum to swallow and/or push on, see second half of this fascinating :D post ( 3/ ... ) (http://forum.nasaspaceflight.com/index.php?topic=36313.msg1315118#msg1315118)

And the other way around, if a cavity is receiving a flux of particles, there is no way (classically) it can recover more nor less momentum than the initial amount entering the boundary, no matter the path and number and modality of interactions of particles with said cavity, if they are to end their lives within. And not only that would make the emitting system to transfer momentum with no more efficiency than using the equivalent of a photon rocket thrust to blow on a sail  (unless it is sending tachyons), but if the sender of the flux is attached to the receiving cavity then the net momentum delta of the whole process is 0 (assuming nothing leaves).

Only between two different systems exchanging photons back and forth many times a transfer of momentum more efficient (by a factor Q) than the photon rocket limit can occur. This could be used to "beam" a force from system A to system B. But again, if system A and B are rigidly linked within a same craft, this is just a static force within the craft, so the Q factor won't amplify nothing useful. EM internal pressure inside a resonating cavity is proportional to Q but does not help to create net imbalance. And as soon as photons leave (if they leave), there can be net imbalance, but Q is not an amplifying factor of efficiency, there are not more photons leaving (for a given energy input) because they bounced many times before leaving.

I've scratched my head for 15 minutes trying to understand the points made in your previous post about asymmetric dissipation but this is not making sense for me so far, even trying hard to think "out of the box". Will try to address and ask for clarifications when time permits.

edit: blowing on a reflective sail a flux of photon actually allows the emitter to "transmit a force", for a given power, twice that of a photon rocket thrust (best case). Still irrelevant if emitter is rigidly linked to sail : perfectly absorbing sail -> 0 net thrust, perfectly reflecting and collimating sail -> thrust/power=1/c

In steady-state, I would agree with you, but it's not a steady state system. It stores up a lot of photons slowly, then they collectively exert a thrust while depleting the energy stored. Shawyer said that acceleration causes the Q to decrease, it causes the stored energy to decrease. When that is happening, there is more energy and momentum being used for Thrust than there is being input by the source. So just because you have 100W input, does not mean the maximum thrust is based on 100W. The maximum thrust will depend on the total momentum bouncing back and forth at any given time. Accelerating, depletes that energy more quickly than it can be restored by the source.

Frankly, if you had a source of microwaves equal to Q*100W, that photon rocket would probably be more efficient than a frustum at exerting thrust because there are fewer losses.

Todd

Title: Re: EM Drive Developments - related to space flight applications - Thread 2
Post by: Rodal on 05/11/2015 10:31 pm
...
Frankly, if you had a source of microwaves equal to Q*100W, that photon rocket would probably be more efficient than a frustum at exerting thrust because there are fewer losses.

Todd

Shawyer/PhtnRckt = 2 * Q * designFactor
McCulloch/PhtnRckt = Q * ((cavityLength/smallDiameter) - (cavityLength/bigDiameter))

hence

Shawyer/(Q *PhtnRckt) = 2 * designFactor
McCulloch/( Q * PhtnRckt) =  ((cavityLength/smallDiameter) - (cavityLength/bigDiameter))



The predictive formulas therefore satisfy the "Todd conjecture"

(EmDriveThrustForce/(Q*EmDrivePowerInput) /(PhotonRocketThrust/ (Q*PhotonRocketPowerInput)  < 1

equivalently

(EmDriveThrustForce/(EmDrivePowerInput) /(PhotonRocketThrust/ (PhotonRocketPowerInput)  < 1

if and only if this condition is met:

Shawyer:   designFactor < 0.5
McCulloch:   ((cavityLength/smallDiameter) - (cavityLength/bigDiameter)) < 1


[notice that for smallDiameter approaching zero, as the frustum becomes a cone, the condition is not met]

(At first, rapid glance, the experimental results, seem to satisfy Todd's conjecture,although I have not checked them in detail)


Example:

Shawyer Experimental

Design Factor = 1.23205

does not satisfy condition designFactor< 1/2

but the experimental force is 1/2.5625 of what Shawyer's formula predicts, so the experiments do satisfy it:

(1/2.5625)/(1/2) = 0.78 which is less than 1, hence the experiment satisfies Todd's conjecture

See:

1) http://forum.nasaspaceflight.com/index.php?topic=36313.msg1302455#msg1302455

2) http://forum.nasaspaceflight.com/index.php?topic=29276.msg1276053#msg1276053

________

NOTE: we never got the exact geometry of the experiments from Shawyer (it is not given in his papers), so it was estimated to the best of the abilities of this group
Title: Re: EM Drive Developments - related to space flight applications - Thread 2
Post by: frobnicat on 05/11/2015 10:56 pm
@frobnicat:
I totally agree that "Q*(photon rocket thrust)" is a nonsense piece of physics on the face of it, for the same reasons you elaborate. However, I was just pointing out that it simply seems to be a good experimental thrust predictor.

Which, of course, is completely weird if you take the thrust measurements as largely unflawed. And no, I don't know what it points to.
As you calculated, the internal energy is proportional to Q.  The internal energy is dissipated into heat. So that again points toward a thermal effect.  Vacuum someone says?  Let's look at outgassing from the heated FR4

There seems to be confusion here. No question internal energy is proportional to Q, for a given net power input. But this net power input is not clear (at least to me, still quite clueless when it comes to microwave engineering). There is power going from RF amplifier to cavity, there is power bouncing back to RF amplifier (and maybe some part bouncing forward again ?). What is dissipated where ?

My understanding is that, by definition, stating a net power input (to the cavity) of 50W means that 50W are dissipated in the walls of the cavity. So from this, assuming constant net power input, thermal effects should not be dependant on Q, no ? An effect directly dependant on Q would be EM pressure inside, but unless running Q of a lot more than a million, that is low enough to have negligible impact upon mechanical rigidity of cavity... What other effects would have internal energy (dependant on Q) but not dissipated power (not directly dependant on Q) ?

Or is it to say that, at fixed DC power input to RF amplifier, net power input is so strongly dependant on Q ? There is the bandwidth that changes, the ability to excite within the bandwidth... The dummy load tests at EW are pretty convincing, that on a dummy load the thrust chart is flat. The dummy load bounces no power back to RF amplifier (?). Could it be that higher Q means less net input power to cavity (at fixed DC power input to RF amplifier), more bounced back, and thermal effects are to be investigated in the dissipation of this bounced back power at RF amplifier ?

Sorry, this was my time for random musings. But a clarification on power (net vs forward only) vs internal energy seems on order. Are all the data in N/W talking about net power or just "output of RF amp" ?
Title: Re: EM Drive Developments - related to space flight applications - Thread 2
Post by: Rodal on 05/11/2015 11:03 pm
...There seems to be confusion here. No question internal energy is proportional to Q, for a given net power input. But this net power input is not clear...
My bad.  I had not thought it through. You are right, Q is actually the contrary of energy losses.   
I deleted the message as it was a non-sequitur.
Thanks for catching the error.  :)
Title: Re: EM Drive Developments - related to space flight applications - Thread 2
Post by: deltaMass on 05/11/2015 11:45 pm
If perfect impedance matching is achieved between amplifier output and cavity input port, then, yes, 50W of amplifier RF power results in 50W being dissipated within the cavity (overwhelmingly as Joule heating of the cavity's conducting walls). Such perfect impedance matching "makes the cavity input port look like" a pure resistor to the amplifier. So that's 50W of heat.

In practise perfect matching is not achieved, but we can get close. The match is quantified by the VSWR and can be directly measured by a VSWR meter. When the match is imperfect, a certain percentage of the forward power is reflected at the cavity port and sent back up the feed.

I don't recall seeing any data on EW's VSWR
Title: Re: EM Drive Developments - related to space flight applications - Thread 2
Post by: deltaMass on 05/11/2015 11:59 pm
Cartoon break.

Woodward had his Angry Flower.
The Quantum Vacuum has xkcd
http://xkcd.com/1486/
Title: Re: EM Drive Developments - related to space flight applications - Thread 2
Post by: Rodal on 05/12/2015 12:23 am
...
I should go back and set up a graphing program for these formulas.  Cavity dimensions scale as 1/f just to add to the mess.   
...
Great graphing program !

I calculated the natural frequencies (NASA truncated cone Brady et.al cavity) for a cylinder with the Geometrical Mean of the Base Diameters (NOT the natural frequencies of the true truncated cone, which are based on Legendre Associated functions and Spherical Bessel functions.  I calculated the frequencies based on Cylindrical Bessel functions for consistency with your method, without a dielectric). I obtained, up to 5 GHz (mode shapes not shown are cut-off):

{{{"TE", 1, 1, 2}, 1.55384*10^9}, {{"TM", 0, 1, 2},
  1.70454*10^9}, {{"TE", 2, 1, 2}, 1.90601*10^9}, {{"TE", 1, 1, 3},
  2.13611*10^9}, {{"TM", 0, 1, 3}, 2.24811*10^9}, {{"TE", 2, 1, 3},
  2.40446*10^9}, {{"TM", 1, 1, 3}, 2.62296*10^9}, {{"TE", 0, 1, 3},
  2.62296*10^9}, {{"TE", 3, 1, 3}, 2.73659*10^9}, {{"TE", 1, 1, 4},
  2.75153*10^9}, {{"TM", 0, 1, 4}, 2.83936*10^9}, {{"TE", 2, 1, 4},
  2.96469*10^9}, {{"TM", 2, 1, 3}, 3.04616*10^9}, {{"TM", 1, 1, 4},
  3.1445*10^9}, {{"TE", 0, 1, 4}, 3.1445*10^9}, {{"TE", 3, 1, 4},
  3.23989*10^9}, {{"TE", 1, 1, 5}, 3.38206*10^9}, {{"TM", 0, 1, 5},
  3.45389*10^9}, {{"TM", 2, 1, 4}, 3.50529*10^9}, {{"TE", 2, 1, 5},
  3.55764*10^9}, {{"TE", 4, 1, 4}, 3.56052*10^9}, {{"TE", 1, 2, 4},
  3.56478*10^9}, {{"TM", 0, 2, 4}, 3.62321*10^9}, {{"TM", 1, 1, 5},
  3.70881*10^9}, {{"TE", 0, 1, 5}, 3.70881*10^9}, {{"TE", 3, 1, 5},
  3.79002*10^9}, {{"TM", 3, 1, 4}, 3.90228*10^9}, {{"TE", 5, 1, 4},
  3.91419*10^9}, {{"TE", 2, 2, 4}, 4.01286*10^9}, {{"TM", 2, 1, 5},
  4.01926*10^9}, {{"TE", 1, 1, 6}, 4.02059*10^9}, {{"TE", 4, 1, 5},
  4.06751*10^9}, {{"TE", 1, 2, 5}, 4.07124*10^9}, {{"TM", 0, 1, 6},
  4.0812*10^9}, {{"TM", 0, 2, 5}, 4.1225*10^9}, {{"TE", 2, 1, 6},
  4.16937*10^9}, {{"TM", 1, 1, 6}, 4.29908*10^9}, {{"TE", 0, 1, 6},
  4.29908*10^9}, {{"TE", 3, 1, 6}, 4.36934*10^9}, {{"TM", 3, 1, 5},
  4.3698*10^9}, {{"TE", 5, 1, 5}, 4.38044*10^9}, {{"TE", 2, 2, 5},
  4.46883*10^9}, {{"TM", 1, 2, 5}, 4.56527*10^9}, {{"TE", 0, 2, 5},
  4.56527*10^9}, {{"TM", 2, 1, 6}, 4.5696*10^9}, {{"TE", 4, 1, 6},
  4.6121*10^9}, {{"TE", 1, 2, 6}, 4.61539*10^9}, {{"TM", 0, 2, 6},
  4.66067*10^9}, {{"TE", 1, 1, 7}, 4.66384*10^9}, {{"TM", 0, 1, 7},
  4.71619*10^9}, {{"TE", 6, 1, 5}, 4.72106*10^9}, {{"TM", 4, 1, 5},
  4.74952*10^9}, {{"TE", 2, 1, 7}, 4.79269*10^9}, {{"TM", 3, 1, 6},
  4.88078*10^9}, {{"TE", 5, 1, 6}, 4.8903*10^9}, {{"TE", 3, 2, 5},
  4.89129*10^9}, {{"TM", 1, 1, 7}, 4.90595*10^9}, {{"TE", 0, 1, 7},
  4.90595*10^9}, {{"TE", 3, 1, 7}, 4.96763*10^9}, {{"TE", 2, 2, 6},
  4.96963*10^9}}

Perhaps in case you want to calculate the remaining missing m,n modes and populate the full scatter plot to see what it looks like ?

Best site for the Bessel Zeros (to within 15 digits):

http://wwwal.kuicr.kyoto-u.ac.jp/www/accelerator/a4/besselroot.htmlx


Thanks
Title: Re: EM Drive Developments - related to space flight applications - Thread 2
Post by: deltaMass on 05/12/2015 12:37 am
As yet I haven't dipped my toe into the doubtless tasty maths of frustum mode frequencies as a function of frustum dimensions. It would seem best to use the most successful mode from an experimental POV. This would be the so-called TE2012 of EW? or something else? I'm talking about max thrust, which is really the only interesting metric at this stage.
Title: Re: EM Drive Developments - related to space flight applications - Thread 2
Post by: Rodal on 05/12/2015 12:51 am
As yet I haven't dipped my toe into the doubtless tasty maths of frustum mode frequencies as a function of frustum dimensions. It would seem best to use the most successful mode from an experimental POV. This would be the so-called TE2012 of EW? or something else? I'm talking about max thrust, which is really the only interesting metric at this stage.
Just like everything else to do with the EM Drive very tortuous and controversial topic.

I agree that according to the Brady et.al. results the TE012 mode had the highest Thrust/InputPower but they are using TM212 in the vacuum tests because they found TE012 too hard to get reproducible results.

The EM Drive is the tar-baby in Uncle Remus (used in the sense of The Oxford English Dictionary defining the "tar baby" as "a difficult problem which is only aggravated by attempts to solve it")

Quote
The more that Br'er Rabbit fights the Tar-Baby, the more entangled he becomes

And the TE012, TM212 designation is based on the cylindrical shape analogue closest to what the fields look like, as the designation for the truncated cone is not standardized, and the quantum numbers for the truncated cone are different
Title: Re: EM Drive Developments - related to space flight applications - Thread 2
Post by: deltaMass on 05/12/2015 01:28 am
I appreciate that there's no standard nomenclature for the frustum. What about Jang's mode? Shawyer's mode? EW actually gets the smallest N/W value of the three.
Title: Re: EM Drive Developments - related to space flight applications - Thread 2
Post by: LasJayhawk on 05/12/2015 01:49 am
I'm gonna go deep here.

Thinking about the standing waves, and the way the nodes move back and forth, remind me of something. A millipede on walkabout. Now the millipede can't tell you why he moves, he just knows that he does.

There may be a variable we don't understand in the equations, ( pure freaking magic, PFM) that will balance the force equations out, or maybe it's all measurement error. Who knows. But in any event, the value of thrashing this about cannot be understated.

To those who don't understand, I point you to here: http://www.cnet.com/news/this-is-what-sunset-looks-like-on-mars/#ftag=YHF65cbda0

NASA has given me eyes so that I can see the sunset on another world. Perhaps we can give the next generation the chance to see it in person, and perhaps the star won't be our own.

I'm going to go back to crying now...
Title: Re: EM Drive Developments - related to space flight applications - Thread 2
Post by: Rodal on 05/12/2015 01:58 am
I appreciate that there's no standard nomenclature for the frustum. What about Jang's mode? Shawyer's mode? EW actually gets the smallest N/W value of the three.
Shawyer's modes here: http://forum.nasaspaceflight.com/index.php?action=dlattach;topic=36313.0;attach=634741

NASA EagleWorks VNAS21  http://forum.nasaspaceflight.com/index.php?action=dlattach;topic=36313.0;attach=778914

NASA EagleWorks S21 and Surface Integral of Energy Density http://forum.nasaspaceflight.com/index.php?action=dlattach;topic=36313.0;attach=778912

Yang measured thrust levels http://forum.nasaspaceflight.com/index.php?action=dlattach;topic=36313.0;attach=782186

Yang's 2013 paper http://forum.nasaspaceflight.com/index.php?action=dlattach;topic=36313.0;attach=785783

Yang's mode shapes and frequencies cannot be computed because Yang didn't provide any geometrical data


NASA EagleWorks Truncated Cone modes NO DIELECTRIC up to 2.5 GHz :  attached
Title: Re: EM Drive Developments - related to space flight applications - Thread 2
Post by: WarpTech on 05/12/2015 02:24 am
...
Frankly, if you had a source of microwaves equal to Q*100W, that photon rocket would probably be more efficient than a frustum at exerting thrust because there are fewer losses.

Todd

Shawyer/PhtnRckt = 2 * Q * designFactor
McCulloch/PhtnRckt = Q * ((cavityLength/smallDiameter) - (cavityLength/bigDiameter))

hence

Shawyer/(Q *PhtnRckt) = 2 * designFactor
McCulloch/( Q * PhtnRckt) =  ((cavityLength/smallDiameter) - (cavityLength/bigDiameter))



The predictive formulas therefore satisfy the "Todd conjecture"

(EmDriveThrustForce/(Q*EmDrivePowerInput) /(PhotonRocketThrust/ (Q*PhotonRocketPowerInput)  < 1

equivalently

(EmDriveThrustForce/(EmDrivePowerInput) /(PhotonRocketThrust/ (PhotonRocketPowerInput)  < 1

if and only if this condition is met:

Shawyer:   designFactor < 0.5
McCulloch:   ((cavityLength/smallDiameter) - (cavityLength/bigDiameter)) < 1


[notice that for smallDiameter approaching zero, as the frustum becomes a cone, the condition is not met]

(At first, rapid glance, the experimental results, seem to satisfy Todd's conjecture,although I have not checked them in detail)


Example:

Shawyer Experimental

Design Factor = 1.23205

does not satisfy condition designFactor< 1/2

but the experimental force is 1/2.5625 of what Shawyer's formula predicts, so the experiments do satisfy it:

(1/2.5625)/(1/2) = 0.78 which is less than 1, hence the experiment satisfies Todd's conjecture

See:

1) http://forum.nasaspaceflight.com/index.php?topic=36313.msg1302455#msg1302455

2) http://forum.nasaspaceflight.com/index.php?topic=29276.msg1276053#msg1276053

________

NOTE: we never got the exact geometry of the experiments from Shawyer (it is not given in his papers), so it was estimated to the best of the abilities of this group

Wow! Thank you for going through that analysis. The "Todd conjecture" huh? I call it the "Desiato effect" and it usually happens sooner or later, when I'm around.  ;)

I will take a closer look at this as I'm still doing research, but so far, I see the frustum as an asymmetrical attenuator. Therefore we would get more efficient thrust out of a photon rocket of equal power, Q*P. It mimics gravity in that it has a variable refractive index, but it mimic's it too well and exhibits the analog of frame dragging. The variable refractive index is the result of the force between the wave and the waveguide, not mass or energy density.

With a low power source, this would make an impulse engine. Input energy until the Q power is high enough to get a bit of thrust, then let it dissipate. Then do it again, repeatedly.

Frustum + Spark-gap? Any Tesla coil fans out there?

EDIT: I checked out the link above. None of the experiments to date have outperformed a photon rocket of output power Q*Pin. There is one data point at the high end of "Shawyer's Demo" where this did not hold.

A photon rocket at a F/P = 1/c is 3.33 uN/kW. A "Q" of 300,000 would produce 1N/kW at the top of it's exponential decay cycle, as that energy is dissipated as thrust. Then it needs to be recharged again.

The Cannae SC experiment was almost as good as a photon rocket, with a ratio of 285,500 < Q they claim was 1.1*10^7

Shawyer's Experiment had the best efficiency of 5640 < Q of 5900

and all the Brady experiments had a ratio to the photon rocket that was less than Q.

Look at it this way. Given a momentary burst photon rocket of thrust Q*P, directed into a closed frustum, it gains back 80% of the forward momentum through reflections and only loses 20% as heat and opposing forces. :)

Todd




 

Title: Re: EM Drive Developments - related to space flight applications - Thread 2
Post by: Rodal on 05/12/2015 02:57 am
EM Drive:

(http://www.emagine.com/assets/sites/2/marktwain-featured.jpg)
Title: Re: EM Drive Developments - related to space flight applications - Thread 2
Post by: deltaMass on 05/12/2015 03:17 am
If I go along with this disturbingly nasty model, then an EmDrive is kinda like a Dean Drive, jerking and scraping its way across spacetime.
Title: Re: EM Drive Developments - related to space flight applications - Thread 2
Post by: Stormbringer on 05/12/2015 03:25 am
dean drive arguments will be effectively ruled out in the upcoming vertical tests if they still have thrust in that configuration right?

Also both Dr White and Mr March are very familiar with Woodward's work and one thing Dr Woodward did in his book was trash/debunk dean drive arguments for his own set up. Because of this I doubt Dean drive artifacts might be ignored in their analysis and precautions.
Title: Re: EM Drive Developments - related to space flight applications - Thread 2
Post by: Einstein79 on 05/12/2015 03:43 am
Good. We are in violent agreement to self regulate this thread and keep pseudoscience out. Otherwise, this effort is lost.

This endeavour is by definition "pseudoscience". I.e. it defies all our logic and principles that we have previously and whole heartedly embraced. Space is an actual object and performs work on everything we observe. The "vacuum" does not exist and this experiment is proof of this statement. We must question everything that we have learned and realize that the truth may lie in "pseudoscience". This experiment reveals that we lack understanding of our physical reality and the "laws" we blindly accept as truth are evidence of this misunderstanding because if they were completely accurrate then we would have already solved the problem.     

This is a strange definition of pseudoscience, and seems somewhat distant from its actual meaning. Pseudoscience involves false, or otherwise inaccurate claims of adherence to the scientific method.

Well then it is my mistake that I did not completely explain what I meant. What I meant was that the scientific method is not very helpful when we do not even have the science available. We should expect to obtain the answer completely by accident which would reveal the "new physics" much like thermodynamics was not developed until after it was physically discovered.
Title: Re: EM Drive Developments - related to space flight applications - Thread 2
Post by: deltaMass on 05/12/2015 03:59 am
@Einstein79:
I've noticed some commonality between your qualitative descriptions of a theory, and the idea of Mike McCullough. Coincidence or not, you should hook up
http://physicsfromtheedge.blogspot.com
Title: Re: EM Drive Developments - related to space flight applications - Thread 2
Post by: deltaMass on 05/12/2015 04:12 am
I've read most of the links kindly provided about modes and energy but can find nothing against which to compare my cavity stored energy estimate of around 10-4 Joules for 50 W input.
Title: Re: EM Drive Developments - related to space flight applications - Thread 2
Post by: Einstein79 on 05/12/2015 04:56 am
@Einstein79:
I've noticed some commonality between your qualitative descriptions of a theory, and the idea of Mike McCullough. Coincidence or not, you should hook up
http://physicsfromtheedge.blogspot.com

A coincidence indeed. I am reading his work now. He has a very interesting and unique approach but from what I read so far, he is still ignoring that space is an object. This "Unrah radiation" he uses is still based on virtual particles and not space itself and is only "created" (or seen) by moving objects. However, I particularly like his idea of a macroscopic Casimir effect because that certainly is a space-time anomaly.

If I were to use his ideas I would only use this "Unrah radiation" to describe the doppler shift of the space-time wave function as a result of the object moving. An object does not need to be moving in order for it to "feel space" and/or be perturbed by it. The Casimir effect is an excellent example of this. As far as the allowable nodes he wrote about, he kind of contradicted himself by stating that only certain wavelengths would be allowed between the object and the Rindler horizon because he assumed that the speed limit for light is what limited the transfer of information but then later suggests that the speed of light can vary under this approach. I find this contradictory because if the speed of light can vary then nothing would limit information exchange thereby allowing all possible wavelengths. There also is the phenomenon of quantum entanglement. In this situation, the transfer of information is not limited by light at all but is instantaneously transmitted directly through the space-time mechanism whether the particles are moving or not.

Thanks for finding that, I will keep reading to see what I can use.     
Title: Re: EM Drive Developments - related to space flight applications - Thread 2
Post by: dustinthewind on 05/12/2015 05:03 am
I've read most of the links kindly provided about modes and energy but can find nothing against which to compare my cavity stored energy estimate of around 10-4 Joules for 50 W input.

Not sure I would be correct in assuming this but I was using this link http://en.wikipedia.org/wiki/Q_factor . 

They give the resonant cavity Q = 2*pi*f*E/P where f is cavity frequency I would guess, E is the power stored and P = 50 watts in your case?  I would wan't to solve for E but it seems wrong to exclude f so I keep f and E together to get Watts.  I solve for f*E=Q*P/(2*pi) which I would assume f*E is in Watts inside the cavity as Q is dimensionless.  Assuming Q is 1000 then 7957.7W is inside the cavity with 50W flowing out. 

If I divide by f @ 30E9Hz I get 2.6525823848649226*10^-7 Joules assuming 1cm wavelength @ c=3E8. 

Maybe I'm doing something wrong though. 
Title: Re: EM Drive Developments - related to space flight applications - Thread 2
Post by: deltaMass on 05/12/2015 05:10 am
Thanks; I feel stupid. E = P Q/w = 2.10-5J (Q=6000, f=2.4 GHz, P=50W). Close enough.
Title: Re: EM Drive Developments - related to space flight applications - Thread 2
Post by: TheTraveller on 05/12/2015 07:14 am
Just finished a Design Factor frequency scanner as attached.

The spreadsheet scans the physical cavity parameters over a frequency range of 1GHz to 10GHz in 100kHz steps and plots the resultant Df using Dr Rodal's Df equation and the applied frequency. Also included are Thrust and Df calculators derived from Shawyers Thrust equation T = 2DfPoQ/c

The frequency for the highest Df found is displayed as well as the 2x & 3x harmonics of that frequency. From this data it appears Shawyer operates his cavities at either the 2x or 3x subharmonic of his Rf generator frequency. This is probably due to the availability of the frequency source.

Please throw rocks as I need this to be correct before designing my test cavity's parameters to be as close to those of the Flight Thruster and it's 3.85GHz design/operational frequency.

You can alter the cavity physical parameters in c2, c3 & c4 and watch the resultant change in freq to obtain max Df at either your prime or 2x or 3x harmonic frequency. Once you get close, you can adjust the start frequency and the step increment to get finer resolution. The scanner has 10,000 steps.

Attached updated version with a few more features, including the ability to select c velocity as in vacuum or atmosphere.
Title: Re: EM Drive Developments - related to space flight applications - Thread 2
Post by: deltaMass on 05/12/2015 07:33 am
@Einstein79:
I've noticed some commonality between your qualitative descriptions of a theory, and the idea of Mike McCullough. Coincidence or not, you should hook up
http://physicsfromtheedge.blogspot.com

A coincidence indeed. I am reading his work now. He has a very interesting and unique approach but from what I read so far, he is still ignoring that space is an object. This "Unrah radiation" he uses is still based on virtual particles and not space itself and is only "created" (or seen) by moving objects. However, I particularly like his idea of a macroscopic Casimir effect because that certainly is a space-time anomaly.

If I were to use his ideas I would only use this "Unrah radiation" to describe the doppler shift of the space-time wave function as a result of the object moving. An object does not need to be moving in order for it to "feel space" and/or be perturbed by it. The Casimir effect is an excellent example of this. As far as the allowable nodes he wrote about, he kind of contradicted himself by stating that only certain wavelengths would be allowed between the object and the Rindler horizon because he assumed that the speed limit for light is what limited the transfer of information but then later suggests that the speed of light can vary under this approach. I find this contradictory because if the speed of light can vary then nothing would limit information exchange thereby allowing all possible wavelengths. There also is the phenomenon of quantum entanglement. In this situation, the transfer of information is not limited by light at all but is instantaneously transmitted directly through the space-time mechanism whether the particles are moving or not.

Thanks for finding that, I will keep reading to see what I can use.     
McCullough has a paper "Newtonian gravitation from the uncertainty principle" which is very creative, but cannot possibly be right. Sitting next to me here is a Mettler mechanical balance that weighs down to half a Planck mass (10 ug). He asserts that anything smaller than mP does not gravitate.

ADDED McCullough replies on his blog "You're right. I have had second thoughts about that paper. There's also some circular reasoning in it after eq 8. However, I still think the idea is sound (gravity from uncertainty) and I can now do a better job of the derivation. "
Title: Re: EM Drive Developments - related to space flight applications - Thread 2
Post by: boznz on 05/12/2015 08:45 am
A Quick search of the threads did not show me the answer so I was wondering if different gasses, pressures, mixes, even vacuum have been tried inside the sealed waveguide/chamber?

Also would such tests be worth doing due to the fact atoms have different frequency/absorption characteristics ?

- Feel free to delete the post if its a dumb/repeated question.
Title: Re: EM Drive Developments - related to space flight applications - Thread 2
Post by: Devilstower on 05/12/2015 09:47 am
I know it's simple, but I have to ask. Is the torsion measurement equipment sensitive to shifts in the balance of the mounted drive? In short, if the system were in fact acting as a sort of Maxwell's Daemon on the gases interior to the system, could this be perceived as "thrust?"
Title: Re: EM Drive Developments - related to space flight applications - Thread 2
Post by: Iulian Berca on 05/12/2015 11:47 am
Hi, Yesterday i received and tested a magnetron from a microwave oven.
 Today i received the cooper and in a couple of hours i will expect to finish the frustum, will post the results in my website, where i started to post the progress of my work live: http://www.masinaelectrica.com/emdrive-independent-test/. If i do not encounter big problems i should test the drive in 3-4 hours.

Iulian
Title: Re: EM Drive Developments - related to space flight applications - Thread 2
Post by: Rodal on 05/12/2015 12:04 pm
Just finished a Design Factor frequency scanner as attached.

The spreadsheet scans the physical cavity parameters over a frequency range of 1GHz to 10GHz in 100kHz steps and plots the resultant Df using Dr Rodal's Df equation and the applied frequency. Also included are Thrust and Df calculators derived from Shawyers Thrust equation T = 2DfPoQ/c

The frequency for the highest Df found is displayed as well as the 2x & 3x harmonics of that frequency. From this data it appears Shawyer operates his cavities at either the 2x or 3x subharmonic of his Rf generator frequency. This is probably due to the availability of the frequency source.

Please throw rocks as I need this to be correct before designing my test cavity's parameters to be as close to those of the Flight Thruster and it's 3.85GHz design/operational frequency.

You can alter the cavity physical parameters in c2, c3 & c4 and watch the resultant change in freq to obtain max Df at either your prime or 2x or 3x harmonic frequency. Once you get close, you can adjust the start frequency and the step increment to get finer resolution. The scanner has 10,000 steps.

Attached updated version with a few more features, including the ability to select c velocity as in vacuum or atmosphere.
I immediately noticed in your spreadsheet the following issues:


1) On top of where it says

Shawyer Flight Thruster data:         
Q = 60,000, Power In 440W, Thrust 0.17 N (170mN)         

it says:

20452.053   Design Factor

Which is an extremely high number for a Design factor.  Such a high number cannot correspond to a physically valued case.  See Todd's prescription constraining such numbers  http://forum.nasaspaceflight.com/index.php?topic=36313.msg1373025#msg1373025



2) I used the original Design Factor spreadsheet I posted with the following input data (I suppose this is the data for the Flight Thruster ? since it has the corresponding frequency):

input   unit   value
big diameter   m   0.2440
small diameter   m   0.1400
cavity length   m   0.1640
frequency   Hz   3.8500E+09

and I obtain with the spreadsheet I posted:


output      
Shawyer Design Factor (air)           0.277137
Shawyer Design Factor (vacuum)   0.277244


these Design Factor numbers, calculated by the original spreadsheet I posted, make sense for a physical situation and are much smaller than the number you have in your spreadsheet of 20452.053 (which is more than 70,000 times larger) which is too large to make physical sense.

See Todd's prescription constraining such numbers  http://forum.nasaspaceflight.com/index.php?topic=36313.msg1373025#msg1373025



3) I also notice that you have a long list of negative values of the Design Factor.  For example this negative Design Factor appears next to my original format (underneath my original calculation for the Design Factor in Air -which is now erased-)

Shawyer Design Factor (vacuum)      -1.645

Such a negative value is non-physical (a design factor value for a physically-valued case should not be negative)

See Todd's prescription constraining such numbers  http://forum.nasaspaceflight.com/index.php?topic=36313.msg1373025#msg1373025



References:

The original spreadsheet I posted:   http://forum.nasaspaceflight.com/index.php?action=dlattach;topic=36313.0;attach=829477


My message where the original spreadsheet appeared:  http://forum.nasaspaceflight.com/index.php?topic=36313.msg1371917#msg1371917

The spreadsheet posted by TheTraveller in which I noticed these issues:  http://forum.nasaspaceflight.com/index.php?action=dlattach;topic=36313.0;attach=829740

EDIT: Shawyer's Design Factor formula has a singularity (due to the expression in the denominator) that leads to negative values under certain non-physical conditions.  Since the negative values (and the values approaching infinity at Shawyer's singularity) are non-physical I would not display them in a spreadsheet because they may lead to confusion.





Title: Re: EM Drive Developments - related to space flight applications - Thread 2
Post by: TheTraveller on 05/12/2015 01:01 pm
Just finished a Design Factor frequency scanner as attached.

The spreadsheet scans the physical cavity parameters over a frequency range of 1GHz to 10GHz in 100kHz steps and plots the resultant Df using Dr Rodal's Df equation and the applied frequency. Also included are Thrust and Df calculators derived from Shawyers Thrust equation T = 2DfPoQ/c

The frequency for the highest Df found is displayed as well as the 2x & 3x harmonics of that frequency. From this data it appears Shawyer operates his cavities at either the 2x or 3x subharmonic of his Rf generator frequency. This is probably due to the availability of the frequency source.

Please throw rocks as I need this to be correct before designing my test cavity's parameters to be as close to those of the Flight Thruster and it's 3.85GHz design/operational frequency.

You can alter the cavity physical parameters in c2, c3 & c4 and watch the resultant change in freq to obtain max Df at either your prime or 2x or 3x harmonic frequency. Once you get close, you can adjust the start frequency and the step increment to get finer resolution. The scanner has 10,000 steps.

Attached updated version with a few more features, including the ability to select c velocity as in vacuum or atmosphere.
I immediately noticed in your spreadsheet the following issues:


1) On top of where it says

Shawyer Flight Thruster data:         
Q = 60,000, Power In 440W, Thrust 0.17 N (170mN)         

it says:

20452.053   Design Factor

Which is an extremely high number for a Design factor.  Such a high number cannot correspond to a physically valued case.  See Todd's prescription constraining such numbers  http://forum.nasaspaceflight.com/index.php?topic=36313.msg1373025#msg1373025



2) I used the original Design Factor spreadsheet I posted with the following input data (I suppose this is the data for the Flight Thruster ? since it has the corresponding frequency):

input   unit   value
big diameter   m   0.2440
small diameter   m   0.1400
cavity length   m   0.1640
frequency   Hz   3.8500E+09

and I obtain with the spreadsheet I posted:


output      
Shawyer Design Factor (air)           0.277137
Shawyer Design Factor (vacuum)   0.277244


these Design Factor numbers, calculated by the original spreadsheet I posted, make sense for a physical situation and are much smaller than the number you have in your spreadsheet of 20452.053 (which is more than 70,000 times larger) which is too large to make physical sense.

See Todd's prescription constraining such numbers  http://forum.nasaspaceflight.com/index.php?topic=36313.msg1373025#msg1373025



3) I also notice that you have a long list of negative values of the Design Factor.  For example this negative Design Factor appears next to my original format (underneath my original calculation for the Design Factor in Air -which is now erased-)

Shawyer Design Factor (vacuum)      -1.645

Such a negative value is non-physical (a design factor value for a physically-valued case should not be negative)

See Todd's prescription constraining such numbers  http://forum.nasaspaceflight.com/index.php?topic=36313.msg1373025#msg1373025



References:

The original spreadsheet I posted:   http://forum.nasaspaceflight.com/index.php?action=dlattach;topic=36313.0;attach=829477


My message where the original spreadsheet appeared:  http://forum.nasaspaceflight.com/index.php?topic=36313.msg1371917#msg1371917
Your / Shawyer's Df equation has a frequency input, which if set to the Rf source frequency indicated, gives the Df value you indicated. My spreadsheet and yours agree.

Being an engineer I asked myself, what if the Rf source frequency was altered to find which frequency generates the best Df? So I created the ability for the Df to be calculated for a wide range of frequencies, the resultant Df calculated/plotted and highest Df (as per the resolution of the incremental freq steps) obtained.

Have attached the latest version which shows the standard Df calc per cavity set of dimensions, and the best frequency and the resultant Df.

There are now 3 cavity data sets, the standard Df calc at the indicated frequency, the highest Df and the frequency that generated that Df.

By doing this series of calculations, to me it seems clear Shawyer operates his cavities at either the subharmonic 1/2 or 1/3 of best Df versus the Rf source.

It may be that by supplying the cavity input Rf at 2x or 3x the optimal Df frequency, the Travelling Wave Shawyer speaks of is generated inside the cavity?

Maybe you might care to simulate a cavity optionally Df resonate at 1/2 the applied Rf frequency? Might be interesting.

BTW negative Dfs are generated if the applied frequency is lower than the optimal cavity Df frequency and positive if above. Simple to confirm.

With this version you can select vac or air c.
Title: Re: EM Drive Developments - related to space flight applications - Thread 2
Post by: frobnicat on 05/12/2015 01:07 pm
I know it's simple, but I have to ask. Is the torsion measurement equipment sensitive to shifts in the balance of the mounted drive?

Specifically on EagleWorks torsion pendulum as it was used so far (all recent EW results) the measures are indeed sensitive to horizontal shifts in centre of mass orthogonal to the pendulum's arm. There is a sensitivity as recoil effect (second order derivative wrt time) that is acknowledged by everyone but that can't possibly explain sustained (for 45s) deviations of the arm from rest position, as observed, because it is either short lived (transient with much smaller time constant) or would require shifts of 100s of grams for many centimetres with very specific acceleration profile (constant acceleration while running, and much slower deceleration and acceleration back to initial position). Roughly would need a =  Thrust/mass and total displacement in first acceleration phase alone of x = .5 a t² = .5 t² Thrust/mass
=> mass*x = .5*Thrust*t² = 50e-6(N)*45(s)² = roughly 50 g m : 50 grams moving 1 m or 500g moving 10cm... that would hardly get unnoticed !

There is also sensitivity to horizontal shifts in centre of mass orthogonal to the pendulum's arm that is linear and is directly proportional to shift (not its second derivative wrt time) and that could possibly explain sustained deviations of the arm from rest position without implying any movement (during the sustained phase). This is due to the fact that the axis of rotation of pendulum is not strictly vertical, therefore making it equivalent to a hanging pendulum in reduced gravity g*sin(theta) where theta is deviation from strict verticality. I'm not saying it is reducing gravity ! It's just that the equations are equivalent... and only with theta=0 makes the "equivalent hanging pendulum component" to cancel (and would make the experimental results strictly identical if performed in 0g or not). And a hanging pendulum (or pendulum with some component equivalent to hanging pendulum) is linearly sensitive to shifts in centre of mass.

Unfortunately the data published so far and the answers given by Paul March on this specific aspect are contradicting and make hard to pinpoint exactly the proportionality constant. To within 1 or 2 orders of magnitude my guesstimates were that 100 grams moving a few or few tens of µm could account for the balance rest position to deviate for a µm (give or take) and be interpreted as sustained thrusts above 30µN

Quote
In short, if the system were in fact acting as a sort of Maxwell's Daemon on the gases interior to the system, could this be perceived as "thrust?"

What mechanism could cause and sustain a density gradient (i.e. pressure gradient) of the gas inside cavity ? Gas pressure delta packs a lot of punch, this tends to equalize pretty quickly within a cavity... unless you can come with a precise mechanism that would allow (rough) quantitative estimate of a possible microwave powered gas pressure gradient build-up in the cavity, it's not possible to know if it would be enough to be significant (through the poorly characterized sensitivity of balance to centre of mass shifts).

In any case, this would not impact the (very few) apparently non null thrust data points in vacuum (for which the cavity was also evacuated)...
Title: Re: EM Drive Developments - related to space flight applications - Thread 2
Post by: TheTraveller on 05/12/2015 01:16 pm
...
Your / Shawyer's Df equation has a frequency input, which if set to the Rf source frequency indicated, gives the Df value you indicated. My spreadsheet and yours agree.

Being an engineer I asked myself, what if the Rf source frequency was altered to find which frequency generates the best Df? So I created the ability for the Df to be calculated for a wide range of frequencies, the resultant Df calculated/plotted and highest Df (as per the resolution of the incremental freq steps) obtained.

Have attached the latest version which shows the standard Df calc per cavity set of dimensions, and the best frequency and the resultant Df.

There are now 3 cavity data sets, the standard Df calc at the indicated frequency, the highest Df and the frequency that generated that Df.

By doing this series of calculations, to me it seems clear Shawyer operates his cavities at either the subharmonic 1/2 or 1/3 of best Df versus the Rf source.

It may be that by operating the cavity at 2x or 3x the optimal Df frequency, the Travelling Wave Shawyer speaks of is generated inside the cavity?

Maybe you might care to simulate a cavity optionally Df resonate at 1/2 the applied Rf frequency? Might be interesting.

BTW negative Dfs are generated if the applied frequency is lower than the optimal cavity Df frequency and positive if above. Simple to confirm.

1) Shawyer's Design Factor formula has a singularity (due to the expression in the denominator) that leads to negative values under certain non-physical conditions.  Since the negative values (and the values approaching infinity at Shawyer's singularity) are non-physical I would not display them in a spreadsheet because they may lead to confusion.

2) What is your explanation for the Flight Thruster comparison? That value you have (more than 70,000 times higher than what my spreadsheet calculates) doesn't make sense.  It appears you have an error in that calculation



2) I used the original Design Factor spreadsheet I posted with the following input data (I suppose this is the data for the Flight Thruster ? since it has the corresponding frequency):

input   unit   value
big diameter   m   0.2440
small diameter   m   0.1400
cavity length   m   0.1640
frequency   Hz   3.8500E+09

and I obtain with the spreadsheet I posted:


output      
Shawyer Design Factor (air)           0.277137
Shawyer Design Factor (vacuum)   0.277244


these Design Factor numbers, calculated by the original spreadsheet I posted, make sense for a physical situation and are much smaller than the number you have in your spreadsheet of 20452.053 (which is more than 70,000 times larger) which is too large to make physical sense.

See Todd's prescription constraining such numbers  http://forum.nasaspaceflight.com/index.php?topic=36313.msg1373025#msg1373025


For the fixed frequency model our Dfs agree. However that frequency is not the optimal frequency to generate the highest Df for the cavity model.

Negative Dfs occur if the Rf frequency is BELOW that of the optimal Df frequency and positive if above.

If you take your spreadsheet and adjust the frequency, you will see the Df change. Set it to the max Df frequency values shown in my spreadsheet and you will see the recorded Dfs.

All my spreadsheet does is to apply your excel Df equation to 10,000 different frequencies and then displays the frequency that is found to have the highest Df.

What it says is the Rf freq Shawyer uses is 2x or 3x that of the optimal Df frequency. So he runs his cavities at a higher frequency that the optimal Df frequency which just happen to be 2x or 3x the best Df cavity value. Don't think that is coincidence the optimal Df freq is a harmonic of the driven Rf freq. Was probably done to obscure that is going on inside the cavity.
Title: Re: EM Drive Developments - related to space flight applications - Thread 2
Post by: TheTraveller on 05/12/2015 01:33 pm
..

Again:

1) I would not display non-physical negative values in a graph (in my Mathematica code, I have an IF condiitional statement to deal with them).  That's a personal choice

2) You have a big error in your calculation of 20452.053 design factor that you are not addressing in any of your replies.  I haven't seen a reply acknowledging your error.

Use this data:

Big dia: 0.2797m
Small dia: 0.1588m
Length: 0.2286m
Frequency: 2.45GHz
Df: 0.469

Now alter frequency to 1.262GHz, which is very close to the 1/2 subharmonic of 2.45GHz.

What is your Df?
Title: Re: EM Drive Developments - related to space flight applications - Thread 2
Post by: Rodal on 05/12/2015 01:41 pm
..

Again:

1) I would not display non-physical negative values in a graph (in my Mathematica code, I have an IF condiitional statement to deal with them).  That's a personal choice

2) You have a big error in your calculation of 20452.053 design factor that you are not addressing in any of your replies.  I haven't seen a reply acknowledging your error.

Use this data:

Big dia: 0.2797m
Small dia: 0.1588m
Length: 0.2286m
Frequency: 1.262GHz which is very close to the 1/2 subharmonic of 2.45GHz.

What is your Df?

Well those dimensions are different from the dimensions that appear next to "Flight Thruster" in your original spreadsheet and the frequency is way too low, therefore I do get these non-physical values

Shawyer Design Factor (air)      20452.052793
Shawyer Design Factor (vacuum)      -1329.537911

When solving a physical problem I like to display values corresponding to physically valid cases.
I would not display non-physical values.  But again that is a matter of choice. So there is no need to argue about it any further, now that I understand that you are displaying non-physical values.

Thank you :)
Title: Re: EM Drive Developments - related to space flight applications - Thread 2
Post by: TheTraveller on 05/12/2015 02:03 pm
..

Again:

1) I would not display non-physical negative values in a graph (in my Mathematica code, I have an IF condiitional statement to deal with them).  That's a personal choice

2) You have a big error in your calculation of 20452.053 design factor that you are not addressing in any of your replies.  I haven't seen a reply acknowledging your error.

Use this data:

Big dia: 0.2797m
Small dia: 0.1588m
Length: 0.2286m
Frequency: 1.262GHz which is very close to the 1/2 subharmonic of 2.45GHz.

What is your Df?

Well those dimensions are different from the dimensions that appear next to "Flight Thruster" in your original spreadsheet and the frequency is way too low, therefore I do get these non-physical values

Shawyer Design Factor (air)      20452.052793
Shawyer Design Factor (vacuum)      -1329.537911

I don't understand why you are displaying that non-physical case.
When solving a physical problem I like to display values corresponding to physically valid cases.
I would not display non-physical values.  But again that is a matter of choice. So there is no need to argue about it any further, now that I understand that you are displaying non-physical values.

Thank you :)
These are the facts as I see them:

1) The Df alters with the Rf frequency and cavity configuration.

2) The value of the cavity Df effects the generated Thrust. T = 2 Df Po Q / c. Therefore Rf frequency effects Thrust.

3) There is a Rf frequency for every cavity configuration that will generate an infinite Df.

4) Rf frequencies below that optimal frequency generate negative Dfs.

5) Rf frequencies above that optimal frequency generate positive Dfs.

6) All tuned / resonant circuits have a peak resonate frequency.

7) High Q tuned / resonant circuits have a very sharp energy stored versus frequency curve.


Speculation:

1)  The plots for Df versus frequency sure like what I would expect from a tuned circuit that has a very high Q being subjected to a spectrum sweep.

1) Shawyer appears to operate his Flight Thruster at an Rf input frequency of 3x the optimal cavity Df frequency.

2) I don't think that is coincidence.
Title: Re: EM Drive Developments - related to space flight applications - Thread 2
Post by: TheTraveller on 05/12/2015 02:05 pm
..

Again:

1) I would not display non-physical negative values in a graph (in my Mathematica code, I have an IF condiitional statement to deal with them).  That's a personal choice

2) You have a big error in your calculation of 20452.053 design factor that you are not addressing in any of your replies.  I haven't seen a reply acknowledging your error.

Use this data:

Big dia: 0.2797m
Small dia: 0.1588m
Length: 0.2286m
Frequency: 1.262GHz which is very close to the 1/2 subharmonic of 2.45GHz.

What is your Df?

Well those dimensions are different from the dimensions that appear next to "Flight Thruster" in your original spreadsheet and the frequency is way too low, therefore I do get these non-physical values

Shawyer Design Factor (air)      20452.052793
Shawyer Design Factor (vacuum)      -1329.537911

When solving a physical problem I like to display values corresponding to physically valid cases.
I would not display non-physical values.  But again that is a matter of choice. So there is no need to argue about it any further, now that I understand that you are displaying non-physical values.

Thank you :)
Waiting for your answer:

Use this data:

Big dia: 0.2797m
Small dia: 0.1588m
Length: 0.2286m
Frequency: 2.45GHz
Df: 0.469

Now alter frequency to 1.262GHz, which is very close to the 1/2 subharmonic of 2.45GHz.

What is your Df?
Title: Re: EM Drive Developments - related to space flight applications - Thread 2
Post by: Rodal on 05/12/2015 02:14 pm
...
These are the facts as I see them:

1) The Df alters with the Rf frequency and cavity configuration.

2) The value of the cavity Df effects the generated Thrust. T = 2 Df Po Q / c. Therefore Rf frequency effects Thrust.

3) There is a Rf frequency for every cavity configuration that will generate an infinite Df.

4) Rf frequencies below that optimal frequency generate negative Dfs.

5) Rf frequencies above that optimal frequency generate positive Dfs.

6) All tuned / resonant circuits have a peak resonate frequency.

7) High Q tuned / resonant circuits have a very sharp energy stored versus frequency curve.


Speculation:

1)  The plots for Df versus frequency sure like what I would expect from a tuned circuit that has a very high Q being subjected to a spectrum sweep.

1) Shawyer appears to operate his Flight Thruster at an Rf input frequency of 3x the optimal cavity Df frequency.

2) I don't think that is coincidence.

The Design Factor is an approximate equation that Shawyer presented but that he did not (to my knowledge) mathematically formally derived.  It has been criticized by a number of physicists and engineers.

Shawyer's Design Factor mathematical formula contains a singularity for finite values of the geometrical dimensions and finite values of frequency.  The infinite values at the singularity and the negative values are obviously non-physical: they correspond to values that cannot take place in our physical world.

Hence instead of writing (for example):

3) There is a Rf frequency for every cavity configuration that will generate an infinite Df.

That may be misinterpreted as if it would be physically possible for this to occur, and that Shawyer's approximation is an accurate model for reality over the whole range of input variables, I would instead write that there is a finite frequency at which Shawyer's expression blows up due to a singularity in Shawyer's approximaton.

As an analogy, McCulloch's expression blows up for the small diameter approaching zero.  I brought this up to the attention of McCulloch and McCulloch readily admits that this is due to an approximation he made in his 1-D formula.  Obviously, a pointy cone would not behave the way that McCulloch's formula would predict: that limit is a non-physical situation outside the approximation limits of McCulloch.

Similarly it maybe that Shawyer's expression is an engineering approximation to certain unknown ranges of dimensions and frequencies.  Clearly, very large values of Shawyer's Design Factor much exceeding 1/2 and any negative values are non-physical (as brought up by Todd, separately).

It may also very well be that the singularity in Shawyer's Design Factor is due to the cut-off condition.  If that is the case, how accurate it is for engineering purposes and for what range of input values, remains to be assessed.
Title: Re: EM Drive Developments - related to space flight applications - Thread 2
Post by: WarpTech on 05/12/2015 02:16 pm
Hi, Yesterday i received and tested a magnetron from a microwave oven.
 Today i received the cooper and in a couple of hours i will expect to finish the frustum, will post the results in my website, where i started to post the progress of my work live: http://www.masinaelectrica.com/emdrive-independent-test/. If i do not encounter big problems i should test the drive in 3-4 hours.

Iulian

From the thermal and A/m plots from EW, most of the resonance is happening at the big end. I would not put the magnetron in that space because the input there will probably perturb the waves. Shawyer put the input near the small end. I would put it "at" the small end, depending on wave polarization. The walls should do most of the reflecting, not the small end.

Todd D.

Title: Re: EM Drive Developments - related to space flight applications - Thread 2
Post by: TheTraveller on 05/12/2015 02:52 pm
...
These are the facts as I see them:

1) The Df alters with the Rf frequency and cavity configuration.

2) The value of the cavity Df effects the generated Thrust. T = 2 Df Po Q / c. Therefore Rf frequency effects Thrust.

3) There is a Rf frequency for every cavity configuration that will generate an infinite Df.

4) Rf frequencies below that optimal frequency generate negative Dfs.

5) Rf frequencies above that optimal frequency generate positive Dfs.

6) All tuned / resonant circuits have a peak resonate frequency.

7) High Q tuned / resonant circuits have a very sharp energy stored versus frequency curve.


Speculation:

1)  The plots for Df versus frequency sure like what I would expect from a tuned circuit that has a very high Q being subjected to a spectrum sweep.

1) Shawyer appears to operate his Flight Thruster at an Rf input frequency of 3x the optimal cavity Df frequency.

2) I don't think that is coincidence.

The Design Factor is an approximate equation that Shawyer presented but that he never mathematically formally derived.  It has been criticized by a large number of physicists and engineers.

Shawyer's Design Factor mathematical formula contains a singularity for finite values of the geometrical dimensions and finite values of frequency.  The infinite values at the singularity and the negative values are obviously non-physical: they correspond to values that cannot take place in our physical world.

Hence instead of writing (for example):

3) There is a Rf frequency for every cavity configuration that will generate an infinite Df.

That may be misinterpreted as if it would be physically possible for this to occur, and that Shawyer's approximation is an accurate model for reality over the whole range of input variables, I would instead write that there is a finite frequency at which Shawyer's expression blows up due to a singularity in Shawyer's approximaton.

As an analogy, McCulloch's expression blows up for the small diameter approaching zero.  I brought this up to the attention of McCulloch and McCulloch readily admits that this is due to an approximation he made in his 1-D formula.  Obviously, a pointy cone would not behave the way that McCulloch's formula would predict: that limit is a non-physical situation outside the approximation limits of McCulloch.

Similarly it maybe that Shawyer's expression is an engineering approximation to certain unknown ranges of dimensions and frequencies.  Clearly, very large values of Shawyer's Design Factor much exceeding 1/2 and any negative values are non-physical (as brought up by Todd, separately).

It may also very well be that the singularity in Shawyer's Design Factor is due to the cut-off condition.  If that is the case, how accurate it is for engineering purposes and for what range of input values, remains to be assessed.
I know infinite values are impossible. Likewise negative Dfs. That was not the point.

The point is the Df equation, applied to a variable frequency, shows there is an ideal frequency that will generate the best cavity Df. Driving the cavity with some chosen frequency, like 2.45GHZ will probably NOT make anything happen. Like trying to drive a tuned LC circuit with a frequency far away from it's resonate frequency. Waste of time. Likewise driving the cavity with the calculated best Df frequency will probably do the same thing. No thrust in the real world.

What the exercise shows is that the Rf generator driving the cavity should be operating 2x or 3x the best Df cavity frequency and that Rf frequency generating system must be able to vary the driving frequency so to search for the best frequency in the real world. The spreadsheet give me a starting place and an understand the cavity best Df frequency should be 1/2 or 1/3 the applied Rf frequency.

To me as an engineer starting a replication of the Flight Thruster and associated variable Rf generation system, it is very new and valuable information. This is all related to real world engineering (building actual hardware) to give the best chance of generating thrust.

To assisting theory development, well it may not be of much value.
Title: Re: EM Drive Developments - related to space flight applications - Thread 2
Post by: TheTraveller on 05/12/2015 02:54 pm
Hi, Yesterday i received and tested a magnetron from a microwave oven.
 Today i received the cooper and in a couple of hours i will expect to finish the frustum, will post the results in my website, where i started to post the progress of my work live: http://www.masinaelectrica.com/emdrive-independent-test/. If i do not encounter big problems i should test the drive in 3-4 hours.

Iulian

From the thermal and A/m plots from EW, most of the resonance is happening at the big end. I would not put the magnetron in that space because the input there will probably perturb the waves. Shawyer put the input near the small end. I would put it "at" the small end, depending on wave polarization. The walls should do most of the reflecting, not the small end.

Todd D.
Yup.

Small end Rf feed.
Title: Re: EM Drive Developments - related to space flight applications - Thread 2
Post by: Rodal on 05/12/2015 03:24 pm
...
I know infinite values are impossible. Likewise negative Dfs. That was not the point.

The point is the Df equation, applied to a variable frequency, shows there is an ideal frequency that will generate the best cavity Df. Driving the cavity with some chosen frequency, like 2.45GHZ will probably NOT make anything happen. Like trying to drive a tuned LC circuit with a frequency far away from it's resonate frequency. Waste of time. Likewise driving the cavity with the calculated best Df frequency will probably do the same thing. No thrust in the real world.

What the exercise shows is that the Rf generator driving the cavity should be operating 2x or 3x the best Df cavity frequency and that Rf frequency generating system must be able to vary the driving frequency so to search for the best frequency in the real world. The spreadsheet give me a starting place and an understand the cavity best Df frequency should be 1/2 or 1/3 the applied Rf frequency.

To me as an engineer starting a replication of the Flight Thruster and associated variable Rf generation system, it is very new and valuable information. This is all related to real world engineering (building actual hardware) to give the best chance of generating thrust.

To assisting theory development, well it may not be of much value.

As to why Shawyer is using a particular excitation frequency I suggest that you use your spreadsheet to check the above vs. the calculated natural frequencies and mode shapes here:

http://forum.nasaspaceflight.com/index.php?action=dlattach;topic=36313.0;attach=634741

(see in the slide the drawing insert detail containing the two dimensions (the diameter of the big base and the length of the truncated cone) given by Shawyer while the third dimension (the diameter of the small base) of the truncated cone is parametrized on the horizontal axis (ranging from a pointy cone to a cylinder) to ascertain its correct value)

Shawyer (to my knowledge) has never provided all three geometrical dimensions of the truncated cone.  Hence one of the dimensions (the diameter of the small base) has to be estimated (obtained from the inverse expression for the Design Factor -when Shawyer has provided the Design Factor, parametrized as per the attached file, or estimated from images, as done by @aero and others).  Since Shaywer has not provided all three dimensions, there is uncertainty as to what he actually did and why

@aero had correspondence with Shawyer asking for the dimension of the small diameter, to my recollection Shawyer cryptically answered "small base diameter chosen on the basis of the cut-off frequency" (hence still unknown how close to the cutoff wavelength was Shawyer's chosen small base diameter).
Title: Re: EM Drive Developments - related to space flight applications - Thread 2
Post by: Jared on 05/12/2015 03:53 pm

Shawyer (to my knowledge) has never provided all three geometrical dimensions of the truncated cone.  Hence one of the dimensions (the diameter of the small base) has to be estimated (as per the attached file, parametrized, or estimated from images, as done by @aero and others).  Since Shaywer has not provided all three dimensions, there is uncertainty as to what he actually did and why

@aero had correspondence with Shawyer asking for the dimension of the small diameter, to my knowledge Shawyer cryptically answered "small base diameter chosen on the basis of the cut-off frequency" (hence still unknown how close to the cutoff wavelength was the small base diameter.

Dr. Rodal, keeping track of these developments, it always seems to me that you and the other active participants here are in a position of "archaeologists", trying to retrace the somewhat mysterious and obscure steps by Shawyer. Maybe I missed the relevant information somewhere down the thread, but shouldn't it be easy to acquire the needed information if Shawyer has a working drive AND supposedly has already been through a lot of what is being now recreated / retraced here (again)? It's not that he has disappeared off the planet (or has he? ;) )...
Title: Re: EM Drive Developments - related to space flight applications - Thread 2
Post by: Rodal on 05/12/2015 04:17 pm
[...
Dr. Rodal, keeping track of these developments, it always seems to me that you and the other active participants here are in a position of "archaeologists", trying to retrace the somewhat mysterious and obscure steps by Shawyer. Maybe I missed the relevant information somewhere down the thread, but shouldn't it be easy to acquire the needed information if Shawyer has a working drive AND supposedly has already been through a lot of what is being now recreated / retraced here (again)? It's not that he has disappeared off the planet (or has he? ;) )...

Personally, more than an "archaeologist" I feel like somebody dealing with the tar-baby in Uncle Remus: The more that Br'er Rabbit fights the Tar-Baby, the more entangled he becomes.  :)

(http://i.ytimg.com/vi/u9svyIqwbOg/hqdefault.jpg)

As noted @aero (hat tip to him) already made a valiant effort to obtain the geometrical data from Shawyer.  It has already been pointed out that people in present e-mail communication with Shawyer as well as others doing do-it-yourself can similarly attempt to obtain such data directly from him, if so interested.
Title: Re: EM Drive Developments - related to space flight applications - Thread 2
Post by: TheTraveller on 05/12/2015 04:34 pm
...
I know infinite values are impossible. Likewise negative Dfs. That was not the point.

The point is the Df equation, applied to a variable frequency, shows there is an ideal frequency that will generate the best cavity Df. Driving the cavity with some chosen frequency, like 2.45GHZ will probably NOT make anything happen. Like trying to drive a tuned LC circuit with a frequency far away from it's resonate frequency. Waste of time. Likewise driving the cavity with the calculated best Df frequency will probably do the same thing. No thrust in the real world.

What the exercise shows is that the Rf generator driving the cavity should be operating 2x or 3x the best Df cavity frequency and that Rf frequency generating system must be able to vary the driving frequency so to search for the best frequency in the real world. The spreadsheet give me a starting place and an understand the cavity best Df frequency should be 1/2 or 1/3 the applied Rf frequency.

To me as an engineer starting a replication of the Flight Thruster and associated variable Rf generation system, it is very new and valuable information. This is all related to real world engineering (building actual hardware) to give the best chance of generating thrust.

To assisting theory development, well it may not be of much value.

As to why Shawyer is using a particular excitation frequency I suggest that you use your spreadsheet to check the above vs. the calculated natural frequencies and mode shapes here:

http://forum.nasaspaceflight.com/index.php?action=dlattach;topic=36313.0;attach=634741

(see in the slide the drawing insert detail containing the two dimensions (the diameter of the big base and the length of the truncated cone) given by Shawyer while the third dimension (the diameter of the small base) of the truncated cone is parametrized on the horizontal axis (ranging from a pointy cone to a cylinder) to ascertain its correct value)

Shawyer (to my knowledge) has never provided all three geometrical dimensions of the truncated cone.  Hence one of the dimensions (the diameter of the small base) has to be estimated (obtained from the inverse expression for the Design Factor -when Shawyer has provided the Design Factor, parametrized as per the attached file, or estimated from images, as done by @aero and others).  Since Shaywer has not provided all three dimensions, there is uncertainty as to what he actually did and why

@aero had correspondence with Shawyer asking for the dimension of the small diameter, to my recollection Shawyer cryptically answered "small base diameter chosen on the basis of the cut-off frequency" (hence still unknown how close to the cutoff wavelength was Shawyer's chosen small base diameter).
Thanks for that data.

Applying that cavity data to my SS I get a small diameter of 155.6mm (Df at 2.4GHZ = 0.500) to give a best cavity Df at 1.2GHz and a 2x harmonic at 2.4GHz Rf input frequency.

What did the other guys get at small end = 151.3mm diameter? Would be interested to know.

Skip that. My diameter of .1556 or 0.0778 is just about spot on the vertical blue dashed line and hits the 2.4GHz as operating at 2x the best Df frequency.

I'm happy with that.
Title: Re: EM Drive Developments - related to space flight applications - Thread 2
Post by: LasJayhawk on 05/12/2015 04:34 pm
[...
Dr. Rodal, keeping track of these developments, it always seems to me that you and the other active participants here are in a position of "archaeologists", trying to retrace the somewhat mysterious and obscure steps by Shawyer. Maybe I missed the relevant information somewhere down the thread, but shouldn't it be easy to acquire the needed information if Shawyer has a working drive AND supposedly has already been through a lot of what is being now recreated / retraced here (again)? It's not that he has disappeared off the planet (or has he? ;) )...

Personally, more than an "archaeologist" I feel like somebody dealing with the tar-baby in Uncle Remus: The more that Br'er Rabbit fights the Tar-Baby, the more entangled he becomes.  :)

(http://i.ytimg.com/vi/u9svyIqwbOg/hqdefault.jpg)

As noted @aero (hat tip to him) already made a valiant effort to obtain the geometrical data from Shawyer.  It has already been pointed out that people in present e-mail communication with Shawyer as well as others doing do-it-yourself can similarly attempt to obtain such data directly from him, if so interested.

So you've become entangled with Dr. White's Q drive. There is a name for that.  ;D
Title: Re: EM Drive Developments - related to space flight applications - Thread 2
Post by: Jared on 05/12/2015 04:53 pm
[...
Dr. Rodal, keeping track of these developments, it always seems to me that you and the other active participants here are in a position of "archaeologists", trying to retrace the somewhat mysterious and obscure steps by Shawyer. Maybe I missed the relevant information somewhere down the thread, but shouldn't it be easy to acquire the needed information if Shawyer has a working drive AND supposedly has already been through a lot of what is being now recreated / retraced here (again)? It's not that he has disappeared off the planet (or has he? ;) )...

Personally, more than an "archaeologist" I feel like somebody dealing with the tar-baby in Uncle Remus: The more that Br'er Rabbit fights the Tar-Baby, the more entangled he becomes.  :)

(http://i.ytimg.com/vi/u9svyIqwbOg/hqdefault.jpg)


Haha, thanks for this analogy. :) So let's hope all mysteries will be solved before anyone gets thrown into the Briar Patch... (pun so very much intended ;) ).
Title: Re: EM Drive Developments - related to space flight applications - Thread 2
Post by: Rodal on 05/12/2015 05:10 pm
[...
Dr. Rodal, keeping track of these developments, it always seems to me that you and the other active participants here are in a position of "archaeologists", trying to retrace the somewhat mysterious and obscure steps by Shawyer. Maybe I missed the relevant information somewhere down the thread, but shouldn't it be easy to acquire the needed information if Shawyer has a working drive AND supposedly has already been through a lot of what is being now recreated / retraced here (again)? It's not that he has disappeared off the planet (or has he? ;) )...

Personally, more than an "archaeologist" I feel like somebody dealing with the tar-baby in Uncle Remus: The more that Br'er Rabbit fights the Tar-Baby, the more entangled he becomes.  :)

(http://i.ytimg.com/vi/u9svyIqwbOg/hqdefault.jpg)


Haha, thanks for this analogy. :) So let's hope all mysteries will be solved before anyone gets thrown into the Briar Patch... (pun so very much intended ;) ).

1) I have not read it in a long long time   :), but as I recall Br'er Rabbit wanted to trick whoever caught him into throwing him into the Briar Patch, which is where he really wanted to be

2) I recall this better: the Ba'ku planet (in Star Trek's Briar Patch) is effectively a fountain of youth, so not a bad place to be either  :)
Title: Re: EM Drive Developments - related to space flight applications - Thread 2
Post by: Jared on 05/12/2015 05:26 pm

1) I have not read it in a long long time   :), but as I recall Br'er Rabbit wanted to trick whoever caught him into throwing him into the Briar Patch, which is where he really wanted to be

2) I recall this better: the Ba'ku planet (in Star Trek's Briar Patch) is effectively a fountain of youth, so not a bad place to be either  :)

1) I stand corrected! :)

2) True, but as far as I know neither Warp nor Impulse engines seem to work there, so I am not sure if that's the place where you'd like to be. ;)
Title: Re: EM Drive Developments - related to space flight applications - Thread 2
Post by: Rodal on 05/12/2015 05:43 pm
...
...
...
...
...

Having dealt with such technical subjects as Br'er Rabbit, the Tar-Baby, the Briar Patch, and the Ba'ku planet, we embark now into the more mundane world of Shawyer's experimental data.  Please observe the attached image of Shawyer's Demonstrator Engine Data.  Please notice the huge difference between the onset of the Power signal trace (vs. time) and the Down and Up thrust force signals vs. time.  Note the time (from onset to reaching the first plateau or knee of the curve) is about ~20 sec.
Recall that for NASA Eagleworks tests this delay is ~2 sec (about 10 times less). 



QUESTION 1: What is responsible for the huge time delay in Shaywer's force signal traces?  For NASA Eagleworks this delay represents about ~4*10^9 electromagnetic wave cycles.  For Shawyer this represents ~33*10^9 cycles. 

Clearly, this huge number of cycles has nothing to do with the Quantum Vacuum, or the speed of light, or the time required for a resonant cavity to reach steady state in standing waves. 
Is this time delay (20 sec) due to a time delay associated with a Q-multiplier effect ?
Is the time delay mainly due to the phase shift settling with time ? (due to Shawyer working with a Q multiplier setup having a Q=X times multiplier with feedback, having a phase shift narrow in frequency).  (hat tip to @Notsosureofit for the explanatory hint, any misinterpretation of which is mine).



QUESTION 2: What is responsible for the time delay in Shaywer's force signal traces after the power is turned off? 



Note: quotes used to call attention to this message in case they have quotes linked to e-mail message forum notifications.

Reference:  http://www.emdrive.com/IAC-08-C4-4-7.pdf
Title: Re: EM Drive Developments - related to space flight applications - Thread 2
Post by: LasJayhawk on 05/12/2015 06:11 pm
Water cooled Maggie and radiator out of a Buick ? Sure looks like thermal effects from the cooling system to me...
Title: Re: EM Drive Developments - related to space flight applications - Thread 2
Post by: TheTraveller on 05/12/2015 06:28 pm
...
...
...
...
...

Having dealt with such technical subjects as Br'er Rabbit, the Tar-Baby, the Briar Patch, and the Ba'ku planet, we embark now into the more mundane world of Shawyer's experimental data.  Please observe the attached image of Shawyer's Demonstrator Engine Data.  Please notice the huge difference between the onset of the Power signal trace (vs. time) and the Down and Up thrust force signals vs. time.  Note the time (from onset to reaching the first plateau or knee of the curve) is about ~20 sec.
Recall that for NASA Eagleworks tests this delay is ~2 sec (about 10 times less). 



QUESTION 1: What is responsible for the huge time delay in Shaywer's force signal traces?  For NASA Eagleworks this delay represents about ~4*10^9 electromagnetic wave cycles.  For Shawyer this represents ~33*10^9 cycles. 

Clearly, this huge number of cycles has nothing to do with the Quantum Vacuum, or the speed of light, or the time required for a resonant cavity to reach steady state in standing waves. 
Is this time delay (20 sec) due to a time delay associated with a Q-multiplier effect ?
Is the time delay mainly due to the phase shift settling with time ? (due to Shawyer working with a Q multiplier setup having a Q=X times multiplier with feedback, having a phase shift narrow in frequency).  (hat tip to @Notsosureofit for the explanatory hint, any misinterpretation of which is mine).



QUESTION 2: What is responsible for the time delay in Shaywer's force signal traces after the power is turned off? 



Note: quotes used to call attention to this message in case they have quotes linked to e-mail message forum notifications.

Reference:  http://www.emdrive.com/IAC-08-C4-4-7.pdf
The Demonstrator EM Drive has a stepper motor outside the small end of the cavity. It physically adjust a cavity parameter (maybe length), via the gears and stepper motor, to obtain internal resonance with the applied Rf frequency. This takes time.

Shawyer has already explained why there was movement after the power was removed.

I really don't appreciate you quoting me and others with nothing but "....."
Title: Re: EM Drive Developments - related to space flight applications - Thread 2
Post by: Rodal on 05/12/2015 07:25 pm
From the thermal and A/m plots from EW, most of the resonance is happening at the big end. I would not put the magnetron in that space because the input there will probably perturb the waves. Shawyer put the input near the small end. I would put it "at" the small end, depending on wave polarization. The walls should do most of the reflecting, not the small end.

Todd D.

Please take a gander at this Demo Drive by Shawyer: http://forum.nasaspaceflight.com/index.php?action=dlattach;topic=36313.0;attach=829850

It seems to have the feed near the big diameter end.
Title: Re: EM Drive Developments - related to space flight applications - Thread 2
Post by: rfmwguy on 05/12/2015 07:48 pm
From the thermal and A/m plots from EW, most of the resonance is happening at the big end. I would not put the magnetron in that space because the input there will probably perturb the waves. Shawyer put the input near the small end. I would put it "at" the small end, depending on wave polarization. The walls should do most of the reflecting, not the small end.

Todd D.

Please take a gander at this Demo Drive by Shawyer: http://forum.nasaspaceflight.com/index.php?action=dlattach;topic=36313.0;attach=829850

It seems to have the feed near the big diameter end.

Good call doctor. I find the stub coupler/feed at the larger diameter of the cavity interesting...thinking this was  the end with the highest return loss/standing wave at 2.4 GHz...perhaps not.

The stepper motor appears to be adjusting the length of a tuning stub centered in the cavity on the narrow end, probably something like this: http://i.stack.imgur.com/Vfdxq.png Its and old tried and true methodology.

Regardless, this tuning stub is simply a matching element which can be fixed (non-adjustable) once a center frequency is set and a tuning stub length measurement can be made. Normally, this tuning stub is adjusted for best S parameter match/bandwidth: http://www.antenna-theory.com/definitions/sparameters.php
Title: Re: EM Drive Developments - related to space flight applications - Thread 2
Post by: Rodal on 05/12/2015 07:59 pm
From the thermal and A/m plots from EW, most of the resonance is happening at the big end. I would not put the magnetron in that space because the input there will probably perturb the waves. Shawyer put the input near the small end. I would put it "at" the small end, depending on wave polarization. The walls should do most of the reflecting, not the small end.

Todd D.

Please take a gander at this Demo Drive by Shawyer: http://forum.nasaspaceflight.com/index.php?action=dlattach;topic=36313.0;attach=829850

It seems to have the feed near the big diameter end.

Good call doctor. I find the stub coupler/feed at the larger diameter of the cavity interesting...thinking this was  the end with the highest return loss/standing wave at 2.4 GHz...perhaps not.

The stepper motor appears to be adjusting the length of a tuning stub centered in the cavity on the narrow end, probably something like this: http://i.stack.imgur.com/Vfdxq.png Its and old tried and true methodology.

Regardless, this tuning stub is simply a matching element which can be fixed (non-adjustable) once a center frequency is set and a tuning stub length measurement can be made. Normally, this tuning stub is adjusted for best S parameter match/bandwidth: http://www.antenna-theory.com/definitions/sparameters.php

Thank you.  I agree with you   :)
Title: Re: EM Drive Developments - related to space flight applications - Thread 2
Post by: Rodal on 05/12/2015 08:04 pm
...
...

Todd and Notsosureofit:

I found a textbook that is in Google Books that has some formulas (including calculating the Q) for (slabs and also for coaxial) dielectric inserts in a cylindrical cavity.  See this:

The Google URL is huge, I hope this URL shortener works:

start at page 111 on this link  http://bit.ly/1FiKoz6


Propagation, Scattering and Dissipation of Electromagnetic Waves
 By A. S. Ilʹinskiĭ, A. Ya Slepyan

Series: IEEE Electromagnetic Waves Series (Book 36)
Publisher: The Institution of Engineering and Technology; First Edition edition (December 2, 1993)
ISBN-10: 0863412831
ISBN-13: 978-0863412837

Hopefully this can help you further in analyzing the thrust of a cylindrical EM Drive with a dielectric insert  :)

Title: Re: EM Drive Developments - related to space flight applications - Thread 2
Post by: deltaMass on 05/12/2015 08:16 pm
Back a ways there existed in woo land something called a "Slepian Drive". Same guy?
https://groups.yahoo.com/neo/groups/Stardrive1/conversations/topics/3910
Title: Re: EM Drive Developments - related to space flight applications - Thread 2
Post by: rfmwguy on 05/12/2015 08:25 pm
Back a ways there existed in woo land something called a "Slepian Drive". Same guy?
https://groups.yahoo.com/neo/groups/Stardrive1/conversations/topics/3910

Nice absrtact...bottom line:

"We conclude that a new breed of propulsion scientists is required as they will need to be cross disciplined into a variety of the physical sciences to include: electromagnetism, spacetime and string-brane theory to but name only a few."
Title: Re: EM Drive Developments - related to space flight applications - Thread 2
Post by: Cinder on 05/12/2015 08:25 pm
That page requires Yahoo login.
Title: Re: EM Drive Developments - related to space flight applications - Thread 2
Post by: rfmwguy on 05/12/2015 08:30 pm
Slepian abstract http://www.earthtech.org/publications/Robertson-Murad-Davis_ECM_49_3.pdf
Title: Re: EM Drive Developments - related to space flight applications - Thread 2
Post by: WarpTech on 05/12/2015 08:43 pm
From the thermal and A/m plots from EW, most of the resonance is happening at the big end. I would not put the magnetron in that space because the input there will probably perturb the waves. Shawyer put the input near the small end. I would put it "at" the small end, depending on wave polarization. The walls should do most of the reflecting, not the small end.

Todd D.

Please take a gander at this Demo Drive by Shawyer: http://forum.nasaspaceflight.com/index.php?action=dlattach;topic=36313.0;attach=829850

It seems to have the feed near the big diameter end.

Good call doctor. I find the stub coupler/feed at the larger diameter of the cavity interesting...thinking this was  the end with the highest return loss/standing wave at 2.4 GHz...perhaps not.

The stepper motor appears to be adjusting the length of a tuning stub centered in the cavity on the narrow end, probably something like this: http://i.stack.imgur.com/Vfdxq.png Its and old tried and true methodology.

Regardless, this tuning stub is simply a matching element which can be fixed (non-adjustable) once a center frequency is set and a tuning stub length measurement can be made. Normally, this tuning stub is adjusted for best S parameter match/bandwidth: http://www.antenna-theory.com/definitions/sparameters.php

Thank you.  I agree with you   :)

If that stub extends inside the frustum, that would explain why he put the input at the big end. I still say, it should be at the small end to avoid perturbing the harmonics.

Todd
Title: Re: EM Drive Developments - related to space flight applications - Thread 2
Post by: deltaMass on 05/12/2015 09:04 pm
There's still the issue of drifting off tune with temperature
Title: Re: EM Drive Developments - related to space flight applications - Thread 2
Post by: Mulletron on 05/12/2015 09:17 pm
Early on in this thread, there was a lot of discussion of potential spaceflight applications which could be enabled by emdrives if the tech were to be viable. Here is an opportunity to roll some of that discussion into the design of the "challenge criteria" for an XPRIZE.

http://forum.nasaspaceflight.com/index.php?topic=37563.0

http://www.xprize.org/about/what-is-an-xprize
Title: Re: EM Drive Developments - related to space flight applications - Thread 2
Post by: deltaMass on 05/12/2015 09:18 pm
Early on in this thread, there was a lot of discussion of potential spaceflight applications which could be enabled by emdrives if the tech were to be viable. Here is an opportunity to roll some of that discussion into the design of the "challenge criteria" for an XPRIZE.

http://forum.nasaspaceflight.com/index.php?topic=37563.0

http://www.xprize.org/about/what-is-an-xprize
I don't think you can begin to design any sort of mission until you know the equation of motion in free space.
Title: Re: EM Drive Developments - related to space flight applications - Thread 2
Post by: LasJayhawk on 05/12/2015 09:19 pm
There's still the issue of drifting off tune with temperature

I think Harold's lapsed patent on a resolver tuned vco could be adapted to keep the oscillator tuned to the cavity instead of the obverse.

https://www.google.com/patents/US4636747?dq=harold+selim&hl=en&sa=X&ei=oWxSVb6lHZGsogS5voHYAQ&ved=0CCoQ6AEwAg
Title: Re: EM Drive Developments - related to space flight applications - Thread 2
Post by: TheTraveller on 05/12/2015 09:20 pm
From the thermal and A/m plots from EW, most of the resonance is happening at the big end. I would not put the magnetron in that space because the input there will probably perturb the waves. Shawyer put the input near the small end. I would put it "at" the small end, depending on wave polarization. The walls should do most of the reflecting, not the small end.

Todd D.

Please take a gander at this Demo Drive by Shawyer: http://forum.nasaspaceflight.com/index.php?action=dlattach;topic=36313.0;attach=829850

It seems to have the feed near the big diameter end.
The EM Drive Demonstrator was feed via a waveguide from a magnetron Rf source, not via coax. You can see the magnetron & waveguide in these 2 shots. Assume that connector is for feedback sense.
Title: Re: EM Drive Developments - related to space flight applications - Thread 2
Post by: zen-in on 05/12/2015 09:21 pm

QUESTION 1: What is responsible for the huge time delay in Shaywer's force signal traces?  For NASA Eagleworks this delay represents about ~4*10^9 electromagnetic wave cycles.  For Shawyer this represents ~33*10^9 cycles. 

Clearly, this huge number of cycles has nothing to do with the Quantum Vacuum, or the speed of light, or the time required for a resonant cavity to reach steady state in standing waves. 
Is this time delay (20 sec) due to a time delay associated with a Q-multiplier effect ?
Is the time delay mainly due to the phase shift settling with time ? (due to Shawyer working with a Q multiplier setup having a Q=X times multiplier with feedback, having a phase shift narrow in frequency).  (hat tip to @Notsosureofit for the explanatory hint, any misinterpretation of which is mine).



QUESTION 2: What is responsible for the time delay in Shaywer's force signal traces after the power is turned off? 



Note: quotes used to call attention to this message in case they have quotes linked to e-mail message forum notifications.

Reference:  http://www.emdrive.com/IAC-08-C4-4-7.pdf
(Ques 1):
The initial delay has been blamed on the "tune-up" of the cavity.   There are claimed to be stepper motors for adjusting the interior dimensions of the cavity.   As these get adjusted and the Q increases the thrust increases.   That's what has been quoted in this forum by others; from one of Shawyer's papers.

(Ques 2):
There is no explanation based on Shawyer's theory for the thrust to continue after the RF drive has been switched off.   For that to occur the cavity has to store energy and release it as kinetic energy.   An inductor stores electrical energy and will release that electrical energy when the (DC) current flow is stopped.   But there is no equivalent electrical effect that would explain why Shawyer's cavity stores RF energy and then releases it as kinetic energy, in spite of it's high Q.   Or why there is a time delay.   A likely explanation is the slosh of coolant.   No doubt the coolant pump generates a measurable torque.    His apparatus is susceptable to spurious torque.   Any rotating machinery (motor, pump, etc)  mounted on a turntable with it's rotational axis parallel to the turntable's will start the turntable moving when it is turned on.    Even if the pump motor is mounted at right angles to the bearing axis the fluid mass flow could still cause rotation.  When the rotating machinery is switched off the fluid mass flow stops and the rotation of the turntable will stop.   That is just the conservation of angular momentum.    Maybe Shawyer has factored this error torque from the coolant pump into his graphs.
Title: Re: EM Drive Developments - related to space flight applications - Thread 2
Post by: TheTraveller on 05/12/2015 09:22 pm
There's still the issue of drifting off tune with temperature
Is why the Rf frequency is adjusted via a feedback loop as used in the Flight Thruster.
Title: Re: EM Drive Developments - related to space flight applications - Thread 2
Post by: Mulletron on 05/12/2015 09:31 pm
Early on in this thread, there was a lot of discussion of potential spaceflight applications which could be enabled by emdrives if the tech were to be viable. Here is an opportunity to roll some of that discussion into the design of the "challenge criteria" for an XPRIZE.

http://forum.nasaspaceflight.com/index.php?topic=37563.0

http://www.xprize.org/about/what-is-an-xprize
I don't think you can begin to design any sort of mission until you know the equation of motion in free space.

Well before we start designing missions, we could design some simple performance objectives first. A good place to start? Prove these thrusters work as advertised.
Title: Re: EM Drive Developments - related to space flight applications - Thread 2
Post by: WarpTech on 05/12/2015 09:39 pm
There's still the issue of drifting off tune with temperature

... and acceleration. That is why I believe now that it should be pulsed, not steady state operation.

Todd
Title: Re: EM Drive Developments - related to space flight applications - Thread 2
Post by: Rodal on 05/12/2015 09:44 pm
From the thermal and A/m plots from EW, most of the resonance is happening at the big end. I would not put the magnetron in that space because the input there will probably perturb the waves. Shawyer put the input near the small end. I would put it "at" the small end, depending on wave polarization. The walls should do most of the reflecting, not the small end.

Todd D.

Please take a gander at this Demo Drive by Shawyer: http://forum.nasaspaceflight.com/index.php?action=dlattach;topic=36313.0;attach=829850

It seems to have the feed near the big diameter end.

Good call doctor. I find the stub coupler/feed at the larger diameter of the cavity interesting...thinking this was  the end with the highest return loss/standing wave at 2.4 GHz...perhaps not.

The stepper motor appears to be adjusting the length of a tuning stub centered in the cavity on the narrow end, probably something like this: http://i.stack.imgur.com/Vfdxq.png Its and old tried and true methodology.

Regardless, this tuning stub is simply a matching element which can be fixed (non-adjustable) once a center frequency is set and a tuning stub length measurement can be made. Normally, this tuning stub is adjusted for best S parameter match/bandwidth: http://www.antenna-theory.com/definitions/sparameters.php

Thank you.  I agree with you   :)

If that stub extends inside the frustum, that would explain why he put the input at the big end. I still say, it should be at the small end to avoid perturbing the harmonics.

Todd

Somebody else pointed out that Shawyer's Demonstrator was feed via a waveguide instead of via coax.  Does that make any difference to your point that feeding should preferentially occur (if possible) at the small end to avoid perturbing harmonics? In other words do you think that feeding with a waveguide avoids perturbing harmonics and therefore if one feeds with a waveguide (fed upstream from a magnetron) you could just as well feed the waveguide at the big end ?
Title: Re: EM Drive Developments - related to space flight applications - Thread 2
Post by: Rodal on 05/12/2015 09:51 pm
There's still the issue of drifting off tune with temperature

... and acceleration. That is why I believe now that it should be pulsed, not steady state operation.

Todd
That also makes sense to me. 

Separately, but interestingly the Serrano Field Effect Boeing Darpa device tested by Dr. White displayed the highest thrust/InputPower of any device, yet it only showed very short time impulses (like Dirac Delta Functions) instead of steady state operation (although to me its principle of operation is very different from the EM Drive, Dr. White classified this device also as a Q-thuster).

This is the text for Boeing/DARPA in slide 40 of Dr. White's presentation (http://ntrs.nasa.gov/archive/nasa/casi.ntrs.nasa.gov/20140000851.pdf ):

<<SFE Test Article at JSC

In 2013, Boeing/DARPA sent Eagleworks Lab an SFE test article for testing and evaluation

Evaluation of the test article in and out of a Faraday Shield performed from Feb through June 2013.

• There is a consistent transient thrust at device turn-on and turn-off that is consistent with Qthruster physics
• The magnitude of the thrust scaled approximately with the cube of the input voltage (20-110uN).
• The magnitude of the thrust is dependent on the AC content of the turn-on and turn-off pulse
• Specific force of transient thrust was in the ~1- 20 N/kW range.

~20-110 uN Thrust Pulses
Specific Force ~1-20N/kW>>

NASA Eagleworks also provided this information in a 2013 Newsletter, which is available in the Internet from this link:  https://xa.yimg.com/kq/groups/86787010/513081407/name/Eagleworks+Newsletter+2013.pdf

that reads:

<<NASA/Boeing/SFE Campaign: Boeing/DARPA sent Eagleworks Lab an SFE test article for testing and
evaluation. The guest thruster was evaluated in numerous test configurations using varying degrees of
Faraday shielding and vacuum conditions. Observations show that there is a consistent transient thrust
at device turn-on and turn-off that is consistent with Q-thruster physics. The magnitude of the thrust
scaled approximately with the cube of the input voltage (20-110uN). The magnitude of the thrust is
dependent on the AC content of the turn-on and turn-off pulse. Thrust to power of transient thrust was
in the ~1-20 N/kW range


Yes, that's twenty Newtons per kiloWatt on the upper range

Notice that the SFE Boeing Darpa Thrust/InputPower is 50 times greater than Shawyer's Flight Thruster shown on the slide to the right of it
Title: Re: EM Drive Developments - related to space flight applications - Thread 2
Post by: PaulF on 05/12/2015 09:55 pm
There's still the issue of drifting off tune with temperature

... and acceleration. That is why I believe now that it should be pulsed, not steady state operation.

Todd
That also makes sense to me. 

Separately, but interestingly the Serrano field effect device tested by Dr. White displayed the highest thrust/InputPower of any device, yet it only showed very short time impulses (like Dirac Delta Functions) instead of steady state operation (although to me its principle of operation is very different from the EM Drive, Dr. White classified this device also as a Q-thuster).
Forgive me for interrupting, but with pulsing, do you mean Pulse Width Modulation or PWM?
Title: Re: EM Drive Developments - related to space flight applications - Thread 2
Post by: TheTraveller on 05/12/2015 09:56 pm
From the thermal and A/m plots from EW, most of the resonance is happening at the big end. I would not put the magnetron in that space because the input there will probably perturb the waves. Shawyer put the input near the small end. I would put it "at" the small end, depending on wave polarization. The walls should do most of the reflecting, not the small end.

Todd D.

Please take a gander at this Demo Drive by Shawyer: http://forum.nasaspaceflight.com/index.php?action=dlattach;topic=36313.0;attach=829850

It seems to have the feed near the big diameter end.

Good call doctor. I find the stub coupler/feed at the larger diameter of the cavity interesting...thinking this was  the end with the highest return loss/standing wave at 2.4 GHz...perhaps not.

The stepper motor appears to be adjusting the length of a tuning stub centered in the cavity on the narrow end, probably something like this: http://i.stack.imgur.com/Vfdxq.png Its and old tried and true methodology.

Regardless, this tuning stub is simply a matching element which can be fixed (non-adjustable) once a center frequency is set and a tuning stub length measurement can be made. Normally, this tuning stub is adjusted for best S parameter match/bandwidth: http://www.antenna-theory.com/definitions/sparameters.php

Thank you.  I agree with you   :)

If that stub extends inside the frustum, that would explain why he put the input at the big end. I still say, it should be at the small end to avoid perturbing the harmonics.

Todd

Somebody else pointed out that Shawyer's Demonstrator was feed via a waveguide instead of via coax.  Does that make any difference to your point that feeding should preferentially occur (if possible) at the small end to avoid perturbing harmonics? In other words do you think that feeding with a waveguide avoids perturbing harmonics and therefore if one feeds with a waveguide (fed upstream from a magnetron) you could just as well feed the waveguide at the big end ?
Shawyer feed it with a magnetron as that was what was available and what he used on the 1st test of principal EM Drive (attached) which was fed via a waveguide from the middle. The coax connector is for sense. You can see the waveguide behind the EmDrive.

I suspect due to the tuning latency issues, in the Flight Thruster he changed to a fixed cavity and using Rf from a narrow band variable frequency Rf generator that fed a TWTA and was quickly adjusted from the E field sensor installed in the big end of the cavity.
Title: Re: EM Drive Developments - related to space flight applications - Thread 2
Post by: TheTraveller on 05/12/2015 10:00 pm
There's still the issue of drifting off tune with temperature

... and acceleration. That is why I believe now that it should be pulsed, not steady state operation.

Todd
That also makes sense to me. 

Separately, but interestingly the Serrano Field Effect Boeing Darpa device tested by Dr. White displayed the highest thrust/InputPower of any device, yet it only showed very short time impulses (like Dirac Delta Functions) instead of steady state operation (although to me its principle of operation is very different from the EM Drive, Dr. White classified this device also as a Q-thuster).

This is the text for Boeing/DARPA in slide 40 of Dr. White's presentation (http://ntrs.nasa.gov/archive/nasa/casi.ntrs.nasa.gov/20140000851.pdf ):

<<SFE Test Article at JSC

In 2013, Boeing/DARPA sent Eagleworks Lab an SFE test article for testing and evaluation

Evaluation of the test article in and out of a Faraday Shield performed from Feb through June 2013.

• There is a consistent transient thrust at device turn-on and turn-off that is consistent with Qthruster physics
• The magnitude of the thrust scaled approximately with the cube of the input voltage (20-110uN).
• The magnitude of the thrust is dependent on the AC content of the turn-on and turn-off pulse
• Specific force of transient thrust was in the ~1- 20 N/kW range.

~20-110 uN Thrust Pulses
Specific Force ~1-20N/kW>>

NASA Eagleworks also provided this information in a 2013 Newsletter, which is available in the Internet from this link:  https://xa.yimg.com/kq/groups/86787010/513081407/name/Eagleworks+Newsletter+2013.pdf

that reads:

<<NASA/Boeing/SFE Campaign: Boeing/DARPA sent Eagleworks Lab an SFE test article for testing and
evaluation. The guest thruster was evaluated in numerous test configurations using varying degrees of
Faraday shielding and vacuum conditions. Observations show that there is a consistent transient thrust
at device turn-on and turn-off that is consistent with Q-thruster physics. The magnitude of the thrust
scaled approximately with the cube of the input voltage (20-110uN). The magnitude of the thrust is
dependent on the AC content of the turn-on and turn-off pulse. Thrust to power of transient thrust was
in the ~1-20 N/kW range


Yes, that's twenty Newtons per kiloWatt on the upper range
Have spoken with Hector Serrano. The SFE device EW tested produced torque pulses, not linear thrust. He is currently testing a linear version.
Title: Re: EM Drive Developments - related to space flight applications - Thread 2
Post by: rfmwguy on 05/12/2015 10:01 pm
good comments re tuning; fc (ctr freq) or mechanical (cavity). Thermal coeffecient of copper aside, another tuning requirement comes to mind...in a vacuum, the dielectric constant will change in the frustum, albeit small, air and vacuum differ http://hyperphysics.phy-astr.gsu.edu/hbase/tables/diel.html

I did not find thread discussion abt whether the frustum cavity was sealed during recent vac test...apologies if I overlooked one
Title: Re: EM Drive Developments - related to space flight applications - Thread 2
Post by: Jim Beichler on 05/12/2015 10:01 pm
As far as I can tell, this propulsion system works by utilizing Dark Energy. That is according to my version of Einstein's unified theory which actually puts mach's Principle into a workable theoretical model. See the section on predictions in "The Einstein unified field theory completed: A direct challenge to the basic assumptions, theories and direction of modern and post-modern physics"
http://tinyurl.com/o2e62jb
Title: Re: EM Drive Developments - related to space flight applications - Thread 2
Post by: jmossman on 05/12/2015 10:12 pm
...
I know infinite values are impossible. Likewise negative Dfs. That was not the point.

The point is the Df equation, applied to a variable frequency, shows there is an ideal frequency that will generate the best cavity Df. Driving the cavity with some chosen frequency, like 2.45GHZ will probably NOT make anything happen. Like trying to drive a tuned LC circuit with a frequency far away from it's resonate frequency. Waste of time. Likewise driving the cavity with the calculated best Df frequency will probably do the same thing. No thrust in the real world.

What the exercise shows is that the Rf generator driving the cavity should be operating 2x or 3x the best Df cavity frequency and that Rf frequency generating system must be able to vary the driving frequency so to search for the best frequency in the real world. The spreadsheet give me a starting place and an understand the cavity best Df frequency should be 1/2 or 1/3 the applied Rf frequency.

To me as an engineer starting a replication of the Flight Thruster and associated variable Rf generation system, it is very new and valuable information. This is all related to real world engineering (building actual hardware) to give the best chance of generating thrust.

To assisting theory development, well it may not be of much value.

As to why Shawyer is using a particular excitation frequency I suggest that you use your spreadsheet to check the above vs. the calculated natural frequencies and mode shapes here:

http://forum.nasaspaceflight.com/index.php?action=dlattach;topic=36313.0;attach=634741

(see in the slide the drawing insert detail containing the two dimensions (the diameter of the big base and the length of the truncated cone) given by Shawyer while the third dimension (the diameter of the small base) of the truncated cone is parametrized on the horizontal axis (ranging from a pointy cone to a cylinder) to ascertain its correct value)

Shawyer (to my knowledge) has never provided all three geometrical dimensions of the truncated cone.  Hence one of the dimensions (the diameter of the small base) has to be estimated (obtained from the inverse expression for the Design Factor -when Shawyer has provided the Design Factor, parametrized as per the attached file, or estimated from images, as done by @aero and others).  Since Shaywer has not provided all three dimensions, there is uncertainty as to what he actually did and why

@aero had correspondence with Shawyer asking for the dimension of the small diameter, to my recollection Shawyer cryptically answered "small base diameter chosen on the basis of the cut-off frequency" (hence still unknown how close to the cutoff wavelength was Shawyer's chosen small base diameter).

Be gentle as the following is highly speculative, but I’ve attempted to be “self-consistent” with several of the observed behaviors and characteristics of the Shawyer, Yang, and CW experiments.  In many ways the following is recirculating an idea already proposed within this forum, but putting a slightly different spin on it.

What if the standing wave within the frustum is necessary but not sufficient for thrust generation?

@TheTraveller’s recent observations about Shawyer’s design equation seem to be consistent with the resonant mode (i.e. standing wave) not being the primary source of thrust.  If the frustum is designed for more than just resonant mode characteristics, this could explain why Shawyer seems to be progressively increasing his frustum angle to much larger values than seem to be beneficial for a resonant cavity.  Perhaps the best empirical confirmation that resonance alone being insufficient for thrust involves a basic sanity check: 

Why has no one noticed superconducting resonant cavities floating or ripping themselves apart due to supposed EM drive thrust effects? 
Answer:  no EM drive thrust possible within a symmetrical resonant cavity


As postulated much earlier within this Forum's thread by others, the “standing wave necessary but not sufficient” conjecture would also help explain the disparity in thrust between the EW tests (a near pure standing wave at the cavity’s resonant frequency) versus the “dirty” magnetron sources in both Shawyer and Yang tests.

A follow-on to this line of reasoning would be to suspect that most (if not all) of EW’s vacuum-tested 40micronewton “thrust” is actually due to a combination of thermal effects altering center-of-mass and possible outgassing from the FR4 and/or dielectric insert.  (Although perhaps the non-resonant/off-frequency portions of the EW energy allows for marginal thrust, along with the possibility that the dielectric attenuation helps provide a gradient that somehow manifests as “thrust”)

Shawyer has described peculiar generator and motor modes for the EM drive.  While I still can’t wrap my head around the “motor” mode, the “generator” mode seems a bit less of a conceptual jump since we know the standing wave within the frustum is a source of stored energy.  If we start accelerating the frustum then there will be a red shift for the photons striking the end plate moving “away” from the rest frame;  this is analogous to a gravity induced red shift.  If the photons are close enough to the frustum cut-off frequency, the red shift will induce attenuation that is greater than the vanilla ohmic losses in a resonant cavity at rest.  How the attenuation results in a “thrust” is beyond me…  but the number of photons near the cut-off frequency will get larger based upon the cavity Q.  The larger the Q, the larger the number of photons that can be red-shifted into attenuation in response to acceleration (and presumably providing “thrust” in an attempt to counter said acceleration when in "generator" mode).  A similar thought experiment applies to the opposite end plate moving "towards" the rest frame, which should introduce a blue shift.  How does blue or red shifting correlate to "thrust"? Or is such an observation merely another Red Herring?  I have no idea...

I leave with this final thought:  what if the standing wave provided a “structure” within the cavity against which attenuated waves could interact?  I’ve attached a very crude Photoshopped image as an illustration….  I realize this conjecture should probably be best left to science fiction literature than this forum…. 

Perhaps the forum readership should just consider this post as another comedic break.   :P

Thanks,
James
Title: Re: EM Drive Developments - related to space flight applications - Thread 2
Post by: Razel.Korr on 05/12/2015 10:26 pm
Obligatory Disclosure - I am not a trained professional in any way. I have been lurking this thread for the last couple months and I am entirely incapable of formulating my ideas mathematically.  However -

How can the principle of operation of the ring laser gyroscope be compared to the principle of operation of EM Drive thrust, escapes me...

This seems important. The Gyroscope uses lasers  sent in different directions and then their varying travel times due to rotation to create the interference. The EMD [supposedly?] uses the difference between speeds at two ends of a wave to generate an effect. Both items [EMD and LRG] use propagating waves being affected by differences in their velocities to create their results. Would that I could dig through the equations for a jumping off point.
Title: Re: EM Drive Developments - related to space flight applications - Thread 2
Post by: deltaMass on 05/12/2015 10:41 pm
Serrano's 0.02 N/W means breakeven speed is only 100 m/s = 2/k
Title: Re: EM Drive Developments - related to space flight applications - Thread 2
Post by: TheTraveller on 05/12/2015 10:48 pm
Serrano's 0.02 N/W means breakeven speed is only 100 m/s = 2/k
According to Serrano, the device EW tested only produces rotary torque forces, not linear force. So could spin a spacecraft around it's CG but can't alter its velocity.
Title: Re: EM Drive Developments - related to space flight applications - Thread 2
Post by: WarpTech on 05/12/2015 10:50 pm
There's still the issue of drifting off tune with temperature

... and acceleration. That is why I believe now that it should be pulsed, not steady state operation.

Todd
That also makes sense to me. 

Separately, but interestingly the Serrano field effect device tested by Dr. White displayed the highest thrust/InputPower of any device, yet it only showed very short time impulses (like Dirac Delta Functions) instead of steady state operation (although to me its principle of operation is very different from the EM Drive, Dr. White classified this device also as a Q-thuster).
Forgive me for interrupting, but with pulsing, do you mean Pulse Width Modulation or PWM?

Yes. Ramp it up to peak thrust and let it exponentially decay back to zero... and Repeat. Since it can't spend more than the input power supplies and remain in steady state operation, and we already know that the input power will only provide uN of thrust. Higher thrust cannot be sustained at steady state operation. Higher thrust levels can only be momentary, and will quickly decay to nothing when output exceeds input.

Todd
Title: Re: EM Drive Developments - related to space flight applications - Thread 2
Post by: WarpTech on 05/12/2015 11:06 pm
From the thermal and A/m plots from EW, most of the resonance is happening at the big end. I would not put the magnetron in that space because the input there will probably perturb the waves. Shawyer put the input near the small end. I would put it "at" the small end, depending on wave polarization. The walls should do most of the reflecting, not the small end.

Todd D.

Please take a gander at this Demo Drive by Shawyer: http://forum.nasaspaceflight.com/index.php?action=dlattach;topic=36313.0;attach=829850

It seems to have the feed near the big diameter end.

Good call doctor. I find the stub coupler/feed at the larger diameter of the cavity interesting...thinking this was  the end with the highest return loss/standing wave at 2.4 GHz...perhaps not.

The stepper motor appears to be adjusting the length of a tuning stub centered in the cavity on the narrow end, probably something like this: http://i.stack.imgur.com/Vfdxq.png Its and old tried and true methodology.

Regardless, this tuning stub is simply a matching element which can be fixed (non-adjustable) once a center frequency is set and a tuning stub length measurement can be made. Normally, this tuning stub is adjusted for best S parameter match/bandwidth: http://www.antenna-theory.com/definitions/sparameters.php

Thank you.  I agree with you   :)

If that stub extends inside the frustum, that would explain why he put the input at the big end. I still say, it should be at the small end to avoid perturbing the harmonics.

Todd

Somebody else pointed out that Shawyer's Demonstrator was feed via a waveguide instead of via coax.  Does that make any difference to your point that feeding should preferentially occur (if possible) at the small end to avoid perturbing harmonics? In other words do you think that feeding with a waveguide avoids perturbing harmonics and therefore if one feeds with a waveguide (fed upstream from a magnetron) you could just as well feed the waveguide at the big end ?

If the "Todd Conjecture" is correct, then we want the input momentum facing toward the large end. If the small end were made the same size as the waveguide feeder, reflected waves should not be able to travel back up the magnetron. All the reflections should be off the large end and the walls, and there is nothing to perturb the resonance. Injecting in the side or big end is likely to disrupt the resonance that we want to amplify.

The only difference between a waveguide feeder and a coax from what I can see, is the bandwidth of the signal. A magnetron has more energy over a wider bandwidth.

Todd
Title: Re: EM Drive Developments - related to space flight applications - Thread 2
Post by: PaulF on 05/12/2015 11:34 pm
There's still the issue of drifting off tune with temperature

... and acceleration. That is why I believe now that it should be pulsed, not steady state operation.

Todd
That also makes sense to me. 

Separately, but interestingly the Serrano field effect device tested by Dr. White displayed the highest thrust/InputPower of any device, yet it only showed very short time impulses (like Dirac Delta Functions) instead of steady state operation (although to me its principle of operation is very different from the EM Drive, Dr. White classified this device also as a Q-thuster).
Forgive me for interrupting, but with pulsing, do you mean Pulse Width Modulation or PWM?

Yes. Ramp it up to peak thrust and let it exponentially decay back to zero... and Repeat. Since it can't spend more than the input power supplies and remain in steady state operation, and we already know that the input power will only provide uN of thrust. Higher thrust cannot be sustained at steady state operation. Higher thrust levels can only be momentary, and will quickly decay to nothing when output exceeds input.

Todd
Funny. I suggested PWM like 20-25 pages ago...  ::)

But to stay on topic, if PWM is used with microwaves, to my knowledge the wavelength of the microwaves would be altered by the short pulses, I forget the name of the effect, but it averages out to the wavelength with the energy of the full cycle (duty+null effort) (or so I presume). Is this a problem or a boon? This is where my knowledge reaches it's limits. I am presuming using low frequency duty cycles would negate the effect.
Title: Re: EM Drive Developments - related to space flight applications - Thread 2
Post by: frobnicat on 05/13/2015 12:14 am
...
The larger the Q, the larger the number of photons that can be red-shifted into attenuation in response to acceleration (and presumably providing “thrust” in an attempt to counter said acceleration when in "generator" mode).  A similar thought experiment applies to the opposite end plate moving "towards" the rest frame, which should introduce a blue shift.  How does blue or red shifting correlate to "thrust"? Or is such an observation merely another Red Herring?  I have no idea...
...

About that there is a simple conventional idea : the "thrust" yielded by blue or red shifting of contained photons due to acceleration of container (relative to some inertial frame) is always opposite to acceleration, doesn't depend on the shape of the container, is proportional to mass equivalent E/c² of contained energy E (that is indeed proportional to Q, for a given power input), and is given by  Fphotons/container=-(E/c²)*a (vectors in bold).

Conventional justification : to accelerate the container of mass mc implies also accelerating the mass equivalent of its content (photons) mp=E/c², one must provide an exterior force on container Fext/container=(mc+mp)*a. Insisting that what is to be accelerated is only the mass of container makes an artificial pseudo-force term to appear on the left :
Fext/container+Fphotons/container=mc*a with Fphotons/container=-mp*a

Hence it appears that this "thrust" is not a thrust as it will always subtract from an exterior force that tries to make the container accelerate. Actually this is just the opposing "inertial force" of the mass equivalent of energy content. Put more simply, and leaving things at their place in the dynamic equation, it's just that contained photons (fields, whatever) add mass to the system (container+contained).

My rough and clumsy estimates with a cavity of Q=10000 and input power of 100W was giving energy content of 10-3 J. Later (probably better) calculation by deltaMass (http://forum.nasaspaceflight.com/index.php?topic=36313.msg1373072#msg1373072) would give more on the order of 10-4 J (for same Q=10000 @ 100W around 2GHz). Anyway, will take 10-3 J as an upper bound of contained EM energy at a given time within frustum tested at Eagleworks. This is equivalent to a mass on the order of 10-20 kg. Before it played a role at the µN level, opposite from thrust moreover, would mean accelerations (of the container) on the order of 1013 gees !

@WarpTech or other proponents of "dependence of the effect on acceleration", please run the numbers. What would be the acceleration of frustum needed to really drift out of bandwidth for Q around 10000 or otherwise reach a magnitude significant to behaviour of waves inside ?
Title: Re: EM Drive Developments - related to space flight applications - Thread 2
Post by: Corlock Striker on 05/13/2015 12:43 am
Why has no one noticed superconducting resonant cavities floating or ripping themselves apart due to supposed EM drive thrust effects? 
Answer:  no EM drive thrust possible within a symmetrical resonant cavity


I've been mostly lurking through the thread for the past few days, and I can't claim to be any sort of an expert on any of this.  I can only just barely follow what you all are discussing.  However, there's been a lot of talk about the potential for thermal artifacts to be causing the thrust, and jmossman made mention of super conductors in the post I quoted above.  I came across this article earlier today that I thought might be of interest to you all, dealing with a team from the Max Planck Institute creating a super conductor at room temperature for a fraction of a nanosecond in a ceramic material that had a small amount of copper in it.  Perhaps somehow the EM drive is achieving a similar effect?

http://www.sciencealert.com/physicists-achieve-superconductivity-at-room-temperature?utm_source=Article&utm_medium=Website&utm_campaign=InArticleReadMore

I just thought that might be something that would be useful to all of you.  Good luck in cracking this.
Title: Re: EM Drive Developments - related to space flight applications - Thread 2
Post by: rrb6699 on 05/13/2015 12:51 am
is there any way to recapture the "thrust" out of
 the main part of the stream and recycle it (so to say) so it could be reflected back into the chamber so it can be used more efficiently?

rr
Title: Re: EM Drive Developments - related to space flight applications - Thread 2
Post by: Rodal on 05/13/2015 12:55 am
@ RODAL

Just got a minute but from your p expression;

If L1/c1 = L2/c2

del f = (1/2*f)*((c1*c2)/(L1*L2))*b^2*((1/dD1^2)-(1/dD2^2))

might be a solution ??

Got to check the thinking later.

Night !

I find your previous expression

del f = ( f/(2*c^2)) * (c1^2-c2^2)

more physically appealing, since it goes to zero for equal dielectric constants, regardless or their dielectric length,

while on the other hand

del f = (1/2*f)*((c1*c2)/(L1*L2))*b^2*((1/dD1^2)-(1/dD2^2))

goes to zero for equal dielectric lengths, regardless of their dielectric constants.

The previous expression is only valid approximation for a "uniformly varying dielectric".  There is no L1 and L2 in that case.

What do you think might maximize the second expression ? (valid only for L1/c1 = L2/c2 )

I was discussing this last night and we made some interesting observations. In a variable dielectric, like in the frustum, when the waves are accelerating to a higher group velocity, they are losing momentum. This momentum is lost to the material "in the direction of the wave". It is similar to frame dragging. The wave is losing energy trying to drag the waveguide or the dielectric with it.

After the wave is reflected, it again tries to drag the dielectric or frustum with it, and this time it meets more resistance. It becomes an evanescent wave and decays faster.

I do not believe a small end cap is needed and the frustum should taper all the way down to the wave guide feeding it. The reflected waves cannot reach the small plate. That's what the thermal images show as well. Most of the energy I think should be trapped at the big end.

Todd D.

Looking for a mechanical analogy :
Let's play with a bended pipe and a ball rolling in it. The pipe can constrain the ball to various path, it can rise or fall, at various steepness. Height of the pipe at a given location defines gravitational potential energy of the ball there. The ball is launched with a given velocity, and then turns around the pipe if it is a closed circuit, or goes back and forth if the two ends of the pipe are high enough, should make no difference.

Assuming no friction, the ball goes-on forever. When rising the ball loses kinetic energy, slows, and imparts momentum to the pipe. When on the return path (different part of pipe if circuit path or same part of pipe if going back and forth), the same delta height will make ball regain same kinetic energy as lost when rising, accelerate, and imparts momentum again. When taking curves, ball also imparts momentum on pipe. Integrating all those momentum exchanges on a cycle yields 0 net momentum. Not depending on path details.

Assuming a closed circuit path and friction (dry, viscous, magnetic... whatever dissipative interaction), including parts with low friction (forth) and parts with high friction (back) and arbitrary height profile (potential well whatever). After a number of cycles the ball will come to rest. Integrating all the momentum exchanges of ball on pipe (changes of height, curves, friction) will yield a total momentum equal to the initial momentum of the ball when launched. Not depending on path details and what parts are more or less dissipative.

I know a photon is not a ball but my question is, in "Newtonian layman's terms" how does the line of thinking you are developing making that analogy not valid, i.e. imply apparent deviation from conservation of momentum ?

The ball (photon) doesn't fall back down the well. There is nothing to give it back enough energy to do so. It dissipates in multiple reflections between the walls and the big end. They are not getting more out than they put in, so it does not violate conservation of energy. They are simply getting more NET momentum on one direction than in the other direction because there is more dissipation and attenuation in one direction than there is in the other. Dissipative systems are typically "not" conservative, loses prevent a true equal measure from occuring in both directions.

Todd D.

Come to think of it, it is not particularly surprising that a gradient is more easily interpreted in the comoving frame than in covariant form: dissipative phenomena are by nature alien to covariance.

They are associated with the production of entropy, they have a thermodynamical arrow of time.

It has taken some time for me to understand why Notsosureofit was using this approach but I'm slowly getting there   ;)

Title: Re: EM Drive Developments - related to space flight applications - Thread 2
Post by: WarpTech on 05/13/2015 01:05 am
...

@WarpTech or other proponents of "dependence of the effect on acceleration", please run the numbers. What would be the acceleration of frustum needed to really drift out of bandwidth for Q around 10000 or otherwise reach a magnitude significant to behaviour of waves inside ?

I read it in Shawyer's recent paper, he simulated it on a computer. I don't have that type of software. I've been wishing I did for decades. Since the system moves by attenuation and dissipation, the photons lose momentum and are red-shifted, while the frustum either gains momentum or gains heat from them.

You're correct, that the momentum it gains from the photons is in the direction of the photons, but results from the difference in the attenuation in each direction. The photons lose more momentum moving inward than moving outward, because they become evanescent waves. They do not increase their energy, except what they can take away from the frustum.

So I see it like ringing a bell. It is the exponential decay from a higher energy state that is giving the thrust. Attempting to make the Q very high to sustain resonance requires reducing the losses, but it is the losses that give it thrust. So... Shawyer increases the angle to make it more like a pill box. Anything over pi/6 is very close to a pill box. Then it should have a higher Q, but it should also have less efficient use of it.

If a photon rocket is: F/P = 1/c
and the Frustum is: F/P ~ Q/c x pulse width
Design efficiency should then target: (F*c)/(P*Q) = 1 but in practice < 1

This would imply maximizing thrust with a lower value of Q, i.e., we do not want to maximize Q, we want to maximize asymmetry in the attenuation, which is what I'm working on at the moment.

Best Regards,
Todd

 
Title: Re: EM Drive Developments - related to space flight applications - Thread 2
Post by: Notsosureofit on 05/13/2015 01:10 am
@ RODAL

Just got a minute but from your p expression;

If L1/c1 = L2/c2

del f = (1/2*f)*((c1*c2)/(L1*L2))*b^2*((1/dD1^2)-(1/dD2^2))

might be a solution ??

Got to check the thinking later.

Night !

I find your previous expression

del f = ( f/(2*c^2)) * (c1^2-c2^2)

more physically appealing, since it goes to zero for equal dielectric constants, regardless or their dielectric length,

while on the other hand

del f = (1/2*f)*((c1*c2)/(L1*L2))*b^2*((1/dD1^2)-(1/dD2^2))

goes to zero for equal dielectric lengths, regardless of their dielectric constants.

The previous expression is only valid approximation for a "uniformly varying dielectric".  There is no L1 and L2 in that case.

What do you think might maximize the second expression ? (valid only for L1/c1 = L2/c2 )

I was discussing this last night and we made some interesting observations. In a variable dielectric, like in the frustum, when the waves are accelerating to a higher group velocity, they are losing momentum. This momentum is lost to the material "in the direction of the wave". It is similar to frame dragging. The wave is losing energy trying to drag the waveguide or the dielectric with it.

After the wave is reflected, it again tries to drag the dielectric or frustum with it, and this time it meets more resistance. It becomes an evanescent wave and decays faster.

I do not believe a small end cap is needed and the frustum should taper all the way down to the wave guide feeding it. The reflected waves cannot reach the small plate. That's what the thermal images show as well. Most of the energy I think should be trapped at the big end.

Todd D.

Looking for a mechanical analogy :
Let's play with a bended pipe and a ball rolling in it. The pipe can constrain the ball to various path, it can rise or fall, at various steepness. Height of the pipe at a given location defines gravitational potential energy of the ball there. The ball is launched with a given velocity, and then turns around the pipe if it is a closed circuit, or goes back and forth if the two ends of the pipe are high enough, should make no difference.

Assuming no friction, the ball goes-on forever. When rising the ball loses kinetic energy, slows, and imparts momentum to the pipe. When on the return path (different part of pipe if circuit path or same part of pipe if going back and forth), the same delta height will make ball regain same kinetic energy as lost when rising, accelerate, and imparts momentum again. When taking curves, ball also imparts momentum on pipe. Integrating all those momentum exchanges on a cycle yields 0 net momentum. Not depending on path details.

Assuming a closed circuit path and friction (dry, viscous, magnetic... whatever dissipative interaction), including parts with low friction (forth) and parts with high friction (back) and arbitrary height profile (potential well whatever). After a number of cycles the ball will come to rest. Integrating all the momentum exchanges of ball on pipe (changes of height, curves, friction) will yield a total momentum equal to the initial momentum of the ball when launched. Not depending on path details and what parts are more or less dissipative.

I know a photon is not a ball but my question is, in "Newtonian layman's terms" how does the line of thinking you are developing making that analogy not valid, i.e. imply apparent deviation from conservation of momentum ?

The ball (photon) doesn't fall back down the well. There is nothing to give it back enough energy to do so. It dissipates in multiple reflections between the walls and the big end. They are not getting more out than they put in, so it does not violate conservation of energy. They are simply getting more NET momentum on one direction than in the other direction because there is more dissipation and attenuation in one direction than there is in the other. Dissipative systems are typically "not" conservative, loses prevent a true equal measure from occuring in both directions.

Todd D.

Come to think of it, it is not particularly surprising that a gradient is more easily interpreted in the comoving frame than in covariant form: dissipative phenomena are by nature alien to covariance.

They are associated with the production of entropy, they have a thermodynamical arrow of time.

That does suggest that one might be able to calculate the change in entropy between the "strained" photon distribution in the rest frame vs a "balanced" distribution in the accelerated frame.  A change in entropy can be related to a force (waves at seashore, shock fronts, etc)

I can vaguely remember that there were some theorems about that developed in studying the magnetosphere in the 70's.  I believe it was J. McGuire (or MacQuire? ) who worked w/ Van-Alan and Caravillano on that stuff ???

These are the right guys..a place to start:
Carovillano, R.L., and J.J. McGuire, Magnetic energy relationships
in the magnetosphere, in Physics of the Magnetosphere, edited
by R.L. Carovillano, J.F. McClay, and H.R. Radoski, pp. 290-300,
D. Reidel, Norwell, Mass., 1968

This the sort of thing:
http://qap2.onlinelibrary.wiley.com/doi/10.1029/RG011i002p00289/pdf

but, it doesn't seem to be immediately useful....seems there might be more about entropy in and about the theory of those shock fronts.... ?

Maybe Eq. 62 generalizing A1 and following ?  (62 looks worth remembering for future ref.)

Well, it was worth a shot.  The entropy calculation looks worth doing.  A difference would indicate an  irreversible process connected to the development of a force due to a photon distribution change of state.
Title: Re: EM Drive Developments - related to space flight applications - Thread 2
Post by: rfmwguy on 05/13/2015 02:13 am
...

@WarpTech or other proponents of "dependence of the effect on acceleration", please run the numbers. What would be the acceleration of frustum needed to really drift out of bandwidth for Q around 10000 or otherwise reach a magnitude significant to behaviour of waves inside ?

I read it in Shawyer's recent paper, he simulated it on a computer. I don't have that type of software. I've been wishing I did for decades. Since the system moves by attenuation and dissipation, the photons lose momentum and are red-shifted, while the frustum either gains momentum or gains heat from them.

You're correct, that the momentum it gains from the photons is in the direction of the photons, but results from the difference in the attenuation in each direction. The photons lose more momentum moving inward than moving outward, because they become evanescent waves. They do not increase their energy, except what they can take away from the frustum.

So I see it like ringing a bell. It is the exponential decay from a higher energy state that is giving the thrust. Attempting to make the Q very high to sustain resonance requires reducing the losses, but it is the losses that give it thrust. So... Shawyer increases the angle to make it more like a pill box. Anything over pi/6 is very close to a pill box. Then it should have a higher Q, but it should also have less efficient use of it.

If a photon rocket is: F/P = 1/c
and the Frustum is: F/P ~ Q/c x pulse width
Design efficiency should then target: (F*c)/(P*Q) = 1 but in practice < 1

This would imply maximizing thrust with a lower value of Q, i.e., we do not want to maximize Q, we want to maximize asymmetry in the attenuation, which is what I'm working on at the moment.

Best Regards,
Todd

Barely able to keep pace Todd...its a good thing. maximizing asymmetry in attenuation different from absorption like the stuff I used to work with?

http://www.westernrubber.com/products/himag-microwave-absorbers/himag-cavity-resonance-absorbers/
Title: Re: EM Drive Developments - related to space flight applications - Thread 2
Post by: SeeShells on 05/13/2015 02:33 am
...

Best Regards,
Todd

Barely able to keep pace Todd...its a good thing. maximizing asymmetry in attenuation different from absorption like the stuff I used to work with?

http://www.westernrubber.com/products/himag-microwave-absorbers/himag-cavity-resonance-absorbers/

It is getting pretty hot and heavy in here and I'm not sure I am keeping up either. Love it though.
I've been mulling around the ideas of harmonics and wondered if anyone has considered injecting 2 RF sources into the cavity,
One set and the other variable in frequency? I've been slowly working my way through this but like I said it's been slow. I welcome and inputs and thoughts.
Title: Re: EM Drive Developments - related to space flight applications - Thread 2
Post by: rfmwguy on 05/13/2015 02:41 am
...

Best Regards,
Todd

Barely able to keep pace Todd...its a good thing. maximizing asymmetry in attenuation different from absorption like the stuff I used to work with?

http://www.westernrubber.com/products/himag-microwave-absorbers/himag-cavity-resonance-absorbers/

It is getting pretty hot and heavy in here and I'm not sure I am keeping up either. Love it though.
I've been mulling around the ideas of harmonics and wondered if anyone has considered injecting 2 RF sources into the cavity,
One set and the other variable in frequency? I've been slowly working my way through this but like I said it's been slow. I welcome and inputs and thoughts.

From what I gather a magnetron source is full of harmonics and subharmonics...one called it "dirty" which is a good visual... spectrum-wise.
Title: Re: EM Drive Developments - related to space flight applications - Thread 2
Post by: LasJayhawk on 05/13/2015 02:48 am
...

Best Regards,
Todd

Barely able to keep pace Todd...its a good thing. maximizing asymmetry in attenuation different from absorption like the stuff I used to work with?

http://www.westernrubber.com/products/himag-microwave-absorbers/himag-cavity-resonance-absorbers/

It is getting pretty hot and heavy in here and I'm not sure I am keeping up either. Love it though.
I've been mulling around the ideas of harmonics and wondered if anyone has considered injecting 2 RF sources into the cavity,
One set and the other variable in frequency? I've been slowly working my way through this but like I said it's been slow. I welcome and inputs and thoughts.

I've been wondering about that, and also it looks like this has a high reflected, so feeding it with a 4 port circulator and running the reflected back into antenna 2, and it's reflected into antenna 3.
Title: Re: EM Drive Developments - related to space flight applications - Thread 2
Post by: deltaMass on 05/13/2015 03:06 am
A comment from http://www.masinaelectrica.com/emdrive-independent-test/ 
Quote
I was hoping to build a mini EmDrive using modified intruder alarm (22 GHz) modules as these can be tuned by simply adjusting the voltage and physical dimensions of the Gunn diodes resonant cavity.
On reflection, not so hot. Only about 200 mW from these things.
Title: Re: EM Drive Developments - related to space flight applications - Thread 2
Post by: WarpTech on 05/13/2015 03:18 am
...

@WarpTech or other proponents of "dependence of the effect on acceleration", please run the numbers. What would be the acceleration of frustum needed to really drift out of bandwidth for Q around 10000 or otherwise reach a magnitude significant to behaviour of waves inside ?

I read it in Shawyer's recent paper, he simulated it on a computer. I don't have that type of software. I've been wishing I did for decades. Since the system moves by attenuation and dissipation, the photons lose momentum and are red-shifted, while the frustum either gains momentum or gains heat from them.

You're correct, that the momentum it gains from the photons is in the direction of the photons, but results from the difference in the attenuation in each direction. The photons lose more momentum moving inward than moving outward, because they become evanescent waves. They do not increase their energy, except what they can take away from the frustum.

So I see it like ringing a bell. It is the exponential decay from a higher energy state that is giving the thrust. Attempting to make the Q very high to sustain resonance requires reducing the losses, but it is the losses that give it thrust. So... Shawyer increases the angle to make it more like a pill box. Anything over pi/6 is very close to a pill box. Then it should have a higher Q, but it should also have less efficient use of it.

If a photon rocket is: F/P = 1/c
and the Frustum is: F/P ~ Q/c x pulse width
Design efficiency should then target: (F*c)/(P*Q) = 1 but in practice < 1

This would imply maximizing thrust with a lower value of Q, i.e., we do not want to maximize Q, we want to maximize asymmetry in the attenuation, which is what I'm working on at the moment.

Best Regards,
Todd

Barely able to keep pace Todd...its a good thing. maximizing asymmetry in attenuation different from absorption like the stuff I used to work with?

http://www.westernrubber.com/products/himag-microwave-absorbers/himag-cavity-resonance-absorbers/

Well, that would appear to "fit" my conjecture and equations nicely. I would assume, if the frustum walls where partially lined with this stuff except for the big end plate, which needs to be highly reflective. Then even more losses would be in the forward direction. The design would need to strike a balance between reflection and absorption. Perhaps only line the frustum walls near the big end, but leave most of the walls reflective to allow a relatively high Q?

IMO, this might do the trick simply because if more momentum is absorbed here, it's less absorbed somewhere else, and it can take the heat. One thing I didn't see on their data sheet is; what is the difference in absorption as a function of angle?

I'll caveat this by adding, it could also very well just add to the heat generation and not add to the momentum transfer. I think it is worth looking into further.

Todd D.

 

Title: Re: EM Drive Developments - related to space flight applications - Thread 2
Post by: Notsosureofit on 05/13/2015 03:24 am
Entropy is keeping me out of bed....I'm too old for this (etc. etc.)

Force is dependent on the rate of change of entropy, (need to get to a thermo book tomorrow), so depends on Power.  Degree of change depends on ratio of order to disorder, so Q as representing the difference between a highly monochromatic frequency distribution and the Boltzman distribution of the dissipated power as heat IN AN ASYMMETRICAL FORM DEPENDENT ON THE CAVITY SHAPE AND THE MODE !

Good Night ! (I hope ??)
Title: Re: EM Drive Developments - related to space flight applications - Thread 2
Post by: WarpTech on 05/13/2015 05:42 am
Entropy is keeping me out of bed....I'm too old for this (etc. etc.)

Force is dependent on the rate of change of entropy, (need to get to a thermo book tomorrow), so depends on Power.  Degree of change depends on ratio of order to disorder, so Q as representing the difference between a highly monochromatic frequency distribution and the Boltzman distribution of the dissipated power as heat IN AN ASYMMETRICAL FORM DEPENDENT ON THE CAVITY SHAPE AND THE MODE !

Good Night ! (I hope ??)

Oh I get it! EM Drive = Entropy Maximization Drive !  ;D

Good night!
Title: Re: EM Drive Developments - related to space flight applications - Thread 2
Post by: SeeShells on 05/13/2015 06:04 am
...

Best Regards,
Todd

Barely able to keep pace Todd...its a good thing. maximizing asymmetry in attenuation different from absorption like the stuff I used to work with?

http://www.westernrubber.com/products/himag-microwave-absorbers/himag-cavity-resonance-absorbers/

It is getting pretty hot and heavy in here and I'm not sure I am keeping up either. Love it though.
I've been mulling around the ideas of harmonics and wondered if anyone has considered injecting 2 RF sources into the cavity,
One set and the other variable in frequency? I've been slowly working my way through this but like I said it's been slow. I welcome and inputs and thoughts.

From what I gather a magnetron source is full of harmonics and subharmonics...one called it "dirty" which is a good visual... spectrum-wise.
I'm not looking at that dirty, more a controlled dirty in the harmonics and sub-harmonics. I remember looking at the spectrum of a magnetron years ago and was appalled at the wild mixture spewing out of it.   I don't think a magnetron will do unless a notch filters are used. Still working my way through it, but time for bed.
Title: Re: EM Drive Developments - related to space flight applications - Thread 2
Post by: TheTraveller on 05/13/2015 08:28 am
Email received from Roger Shawyer in regard to a question to verify if Dr. Rodal's Df excel equation is correct.

Shawyer also cautions, very strongly, on the dangers of working with high power microwave systems.
Title: Re: EM Drive Developments - related to space flight applications - Thread 2
Post by: deltaMass on 05/13/2015 09:50 am
The mapping Shawyer defines can be modelled by
DF = 1 - c-d
where d = difference in diameters >=0
and c is some constant >=1; c=e for example
Title: Re: EM Drive Developments - related to space flight applications - Thread 2
Post by: frobnicat on 05/13/2015 10:10 am
...

Best Regards,
Todd

Barely able to keep pace Todd...its a good thing. maximizing asymmetry in attenuation different from absorption like the stuff I used to work with?

http://www.westernrubber.com/products/himag-microwave-absorbers/himag-cavity-resonance-absorbers/

It is getting pretty hot and heavy in here and I'm not sure I am keeping up either. Love it though.
I've been mulling around the ideas of harmonics and wondered if anyone has considered injecting 2 RF sources into the cavity,
One set and the other variable in frequency? I've been slowly working my way through this but like I said it's been slow. I welcome and inputs and thoughts.

Maybe first try to state what kind of nonlinear coupling effects could be at play to make a difference ? Linear system => result(wave1+wave2)=result(wave1)+result(wave2) so doing wave1+wave2 would bring nothing new, qualitatively, without a mechanism to explain result(single wave)!=0 in the first place...

Side note : no harm but you are skipping some quote closing, that mixes up citations.
Title: Re: EM Drive Developments - related to space flight applications - Thread 2
Post by: deltaMass on 05/13/2015 11:31 am
I don't know if anyone's tried to minimise the all-up mass budget, but with batteries I can't see it going much below 50 Kg. There should be no rotating components like fans in the equipment. There will be heat but try to arrange to exhaust it orthogonal to the expected motion. Perhaps use a Teflon-on-Teflon channel for motion. It must be carefully levelled. Air must be still (smoke sticks!). Stiction may be an issue. Then let's compare EW to Juan, assuming 50 Kg all-up mass, assuming (at least initially) constant acceleration:
F=100 uN: a =  2*10-6 m/s2; t(1 cm) = 100 secs; t(10 cm) = 316 secs
F = 0.7  N: a =  0.014 m/s2; t(1 cm) = 1.2 secs;  t(10 cm) = 3.8 secs

Title: Re: EM Drive Developments - related to space flight applications - Thread 2
Post by: Notsosureofit on 05/13/2015 12:55 pm
Entropy is keeping me out of bed....I'm too old for this (etc. etc.)

Force is dependent on the rate of change of entropy, (need to get to a thermo book tomorrow), so depends on Power.  Degree of change depends on ratio of order to disorder, so Q as representing the difference between a highly monochromatic frequency distribution and the Boltzman distribution of the dissipated power as heat IN AN ASYMMETRICAL FORM DEPENDENT ON THE CAVITY SHAPE AND THE MODE !

Good Night ! (I hope ??)

Oh I get it! EM Drive = Entropy Maximization Drive !  ;D

Good night!

Well, yes, in that sense.  The entropy is one more condition, like the Equivalence Principle, that the EM drive has to satisfy.  The interesting point is that the entropy change can be related to a force vector.  Think back to the semi-permeable membrane example in Thermodynamics 1.
Title: Re: EM Drive Developments - related to space flight applications - Thread 2
Post by: Rodal on 05/13/2015 12:57 pm
Email received from Roger Shawyer in regard to a question to verify if Dr. Rodal's Df excel equation is correct.
...
http://forum.nasaspaceflight.com/index.php?action=dlattach;topic=36313.0;attach=829894

The equation that you refer to was derived by me as an honest effort to elucidate variables, "reading tea leaves", that Shawyer had not explicitly defined (originally with @aero and others to compare numerical predictions with experimental measurements). 

(http://3.bp.blogspot.com/-WXBrwiuxZRs/U92ibnqM-NI/AAAAAAAAbac/TzEbn2EqJvc/s1600/Tea-Leaves-In-Cup.gif)

You repeatedly asked me for Shaywer's Design Factor equation.  Obviously, if you had been given the Design Factor equation by Shawyer, or if this expression was obvious to you from reading Shawyer's writings, there was no point in asking me for Shaywer's Design Factor (I hope and fully expect that in that case you would have made Shawyer's Design Factor available to the forum). 

What was expected from Shawyer, the author of the equation, is to produce his equation: to answer "this is my equation:..." (defining the Design Factor and its variables, and not resorting to references where the variables are not explicitly defined).  Instead, his response, relayed through you as a messenger, communicating behind a curtain, is an unconstructive response: Shawyer's Design Factor equation is never provided.  Certainly not provided to this forum (was his equation provided instead to you in private?). Not a response one would expect in an Engineering or a Scientific journal, where the editors expect disclosure (and hence a definition) of what is being discussed (in this case what is Shawyer's Design Factor equation ?) before asserting that somebody's interpretation is wrong. 

This response stands in direct contrast to the direct, constructve responses of others: Paul March (NASA) who consistently provided geometrical dimensions, defined variables, and provided honest answers to honest questions.  Same for Prof. McCulloch who answered questions directly (not using messengers) both in this forum and in his blog, instead of keeping obscure what are his equations.

I look forward to your providing to this forum what is Shawyer's Design Factor equation (and then we can have a discussion of my effort to ascertain Shawyer's Design Factor, vis-à-vis Shawyer's previous papers that I had to rely on).

I also look forward to Shawyer providing the big and small diameters and the length of the truncated cones used in his  experiments, so that this forum can verify the claims being made in his papers regarding the validity of his Design Factor (instead of having to rely on assessing dimensions from photographs, or resorting to parametrization of variables over a whole range, as NASA had to do to guess Shawyer's dimensions).
Title: Re: EM Drive Developments - related to space flight applications - Thread 2
Post by: RegMaster on 05/13/2015 01:18 pm
Hi, I'm new in this forum. I'm from Spain, and my english is not perfect.
I have been reading this "infinite" thread several days, but here are too many technical concepts that are in some cases dificult to understand when I read it (due lenguage translation limitations).

I was thinking very much about this EM Drive technology, their physics principies and other many advanced physics concepts.

IMHO, EM Drive would be related to this:
http://en.wikipedia.org/wiki/Ronald_Mallett#Time_travel_research

Where microwaves are too photons like láser, but in other frequencies... I think that truncated cone works as a type of "well", but in a very inneficient form. I think that the reflectivity is not the main problem, but the angle of microwaves injected in the chamber and their relation between diameter and microwave frequency....

(http://www.mathpages.com/home/kmath443/kmath443_files/image001.gif)

You can see that this path, with microwaves reflected in a shape of "well" can generate a "spiral" resonance o s-wave inside the chamber, raising the efficient of EM Drive joint it with Mallett article.

Is this posible?
sorry for my english.
Title: Re: EM Drive Developments - related to space flight applications - Thread 2
Post by: Rodal on 05/13/2015 01:51 pm
The mapping Shawyer defines can be modelled by
DF = 1 - c-d
where d = difference in diameters >=0
and c is some constant >=1; c=e for example
Yes, what you state would be a possible (one of several) valid interpretation(s) of Shawyer's latest message relayed here: http://forum.nasaspaceflight.com/index.php?action=dlattach;topic=36313.0;attach=829894. 

However, that's not the mapping defined in Shawyer's previous published papers.
Title: Re: EM Drive Developments - related to space flight applications - Thread 2
Post by: Rodal on 05/13/2015 02:01 pm
Entropy is keeping me out of bed....I'm too old for this (etc. etc.)

Force is dependent on the rate of change of entropy, (need to get to a thermo book tomorrow), so depends on Power.  Degree of change depends on ratio of order to disorder, so Q as representing the difference between a highly monochromatic frequency distribution and the Boltzman distribution of the dissipated power as heat IN AN ASYMMETRICAL FORM DEPENDENT ON THE CAVITY SHAPE AND THE MODE !

Good Night ! (I hope ??)

Oh I get it! EM Drive = Entropy Maximization Drive !  ;D

Good night!

Well, yes, in that sense.  The entropy is one more condition, like the Equivalence Principle, that the EM drive has to satisfy.  The interesting point is that the entropy change can be related to a force vector.  Think back to the semi-permeable membrane example in Thermodynamics 1.

It is also interesting because the 2nd law of Thermodynamics is an emergent law: it is not intrinsically (*) present in Einstein's General Relativity, and not intrinsically present in Quantum Mechanics.  Yet a lot of effort has been devoted here and elsewhere (**) in analyzing the EM Drive (including energy paradoxes) without taking into account entropy.

___________
(*) Of course there are multiple papers considering entropy as an add-on to Einstein's GR, e.g. entropy considerations in Black Holes, etc.
(**) A general statement, as Shawyer's, Prof. Yang's and  Dr. White's considerations (or those of their critics, to the extent I can recall) have not explicitly considered the 2nd Law of thermodynamics in their published papers.

Quote from: Sir Arthur Stanley Eddington,
The law that entropy always increases holds, I think, the supreme position among the laws of Nature. If someone points out to you that your pet theory of the universe is in disagreement with Maxwell's equations — then so much the worse for Maxwell's equations. If it is found to be contradicted by observation — well, these experimentalists do bungle things sometimes. But if your theory is found to be against the second law of thermodynamics I can give you no hope; there is nothing for it but to collapse in deepest humiliation.
Title: Re: EM Drive Developments - related to space flight applications - Thread 2
Post by: rrb6699 on 05/13/2015 02:11 pm

I have a belief concept of the universe and dark energy. perhaps I don't know the term, but I believe since everything from the atom to the solar system to galaxies and beyond follow a specific atom-like pattern, AND because we cannot identify the smallest particle or the largest "particle", then everything taps into the same place. since there can be no smallest particle, otherwise it has to be comprised of something smaller, then it must be a passage to or through to negative or dark energy. I believe we are about to tap into this energy and it will be the key to everything including distant travel in short times, but, it may not be like we imagine in science fiction. it may be we can pass through at an atomic level. I believe space travel as a whole body may be possible this way, but, will be limited distance until we find the key to the universe at the atomic level. but possibly both will be discovered simultaneously if we can just recognize it.   Just a thought of many roaming around my mind.
Title: Re: EM Drive Developments - related to space flight applications - Thread 2
Post by: WarpTech on 05/13/2015 02:37 pm
...
...

Todd and Notsosureofit:

I found a textbook that is in Google Books that has some formulas (including calculating the Q) for (slabs and also for coaxial) dielectric inserts in a cylindrical cavity.  See this:

The Google URL is huge, I hope this URL shortener works:

start at page 111 on this link  http://bit.ly/1FiKoz6


Propagation, Scattering and Dissipation of Electromagnetic Waves
 By A. S. Ilʹinskiĭ, A. Ya Slepyan

Series: IEEE Electromagnetic Waves Series (Book 36)
Publisher: The Institution of Engineering and Technology; First Edition edition (December 2, 1993)
ISBN-10: 0863412831
ISBN-13: 978-0863412837

Hopefully this can help you further in analyzing the thrust of a cylindrical EM Drive with a dielectric insert  :)

That is a really good book! I wish I owned it. The previous sections before pg. 111 cover modes in a cylinder, sphere and cone. Very good read!

I also found this one last night, regarding attenuation in a lossy waveguide. It's elementary, and I like that! :)

http://www.radioeng.cz/fulltexts/2011/11_02_472_478.pdf

Todd

Title: Re: EM Drive Developments - related to space flight applications - Thread 2
Post by: Rodal on 05/13/2015 02:43 pm
...

Best Regards,
Todd

Barely able to keep pace Todd...its a good thing. maximizing asymmetry in attenuation different from absorption like the stuff I used to work with?

http://www.westernrubber.com/products/himag-microwave-absorbers/himag-cavity-resonance-absorbers/

It is getting pretty hot and heavy in here and I'm not sure I am keeping up either. Love it though.
I've been mulling around the ideas of harmonics and wondered if anyone has considered injecting 2 RF sources into the cavity,
One set and the other variable in frequency? I've been slowly working my way through this but like I said it's been slow. I welcome and inputs and thoughts.


Maybe first try to state what kind of nonlinear coupling effects could be at play to make a difference ? Linear system => result(wave1+wave2)=result(wave1)+result(wave2) so doing wave1+wave2 would bring nothing new, qualitatively, without a mechanism to explain result(single wave)!=0 in the first place...


Kudos to SeeShells !!!!!  (do we need more hot and heavy weather to keep producing these great suggestions   :) )

This from this paper recently brought up by Todd (hat tip to WarpTech), which confirms the validity of SeeShells suggestion:

http://www.radioeng.cz/fulltexts/2011/11_02_472_478.pdf

Attenuation in Rectangular Waveguides with Finite Conductivity Walls
Kim Ho YEAP, Choy Yoong THAM, Ghassan YASSIN, Kee Choon YEONG
RADIOENGINEERING, VOL. 20, NO. 2, JUNE 2011


Quote from: Kim Ho YEAP, Choy Yoong THAM, Ghassan YASSIN, Kee Choon YEONG
An important consequence of this
work is the demonstration that the loss computed for degenerate
modes propagating simultaneously is not simply
additive.
In other words, the combined loss of two co-existing
modes is higher than adding the losses of two modes
propagating independently. This can be explained by the
mode coupling effects, which is significant when the phase
constants of two propagating modes are different yet very
close. 
Title: Re: EM Drive Developments - related to space flight applications - Thread 2
Post by: Notsosureofit on 05/13/2015 02:53 pm
Quote
An important consequence of this
work is the demonstration that the loss computed for degenerate
modes propagating simultaneously is not simply
additive. In other words, the combined loss of two co-existing
modes is higher than adding the losses of two modes
propagating independently. This can be explained by the
mode coupling effects, which is significant when the phase
constants of two propagating modes are different yet very
close.

Yes, this is the condition for a resonant cavity in an accelerating frame, and a limit on EM type drives.
Title: Re: EM Drive Developments - related to space flight applications - Thread 2
Post by: TheTraveller on 05/13/2015 03:01 pm
Email received from Roger Shawyer in regard to a question to verify if Dr. Rodal's Df excel equation is correct.
...
http://forum.nasaspaceflight.com/index.php?action=dlattach;topic=36313.0;attach=829894

The equation that you refer to was derived by me as an honest effort to elucidate variables, "reading tea leaves", that Shawyer had not explicitly defined (originally with @aero and others to compare numerical predictions with experimental measurements). 

(http://3.bp.blogspot.com/-WXBrwiuxZRs/U92ibnqM-NI/AAAAAAAAbac/TzEbn2EqJvc/s1600/Tea-Leaves-In-Cup.gif)

You repeatedly asked me for Shaywer's Design Factor equation.  Obviously, if you had been given the Design Factor equation by Shawyer, or if this expression was obvious to you from reading Shawyer's writings, there was no point in asking me for Shaywer's Design Factor (I hope and fully expect that in that case you would have made Shawyer's Design Factor available to the forum). 

What was expected from Shawyer, the author of the equation, is to produce his equation: to answer "this is my equation:..." (defining the Design Factor and its variables, and not resorting to references where the variables are not explicitly defined).  Instead, his response, relayed through you as a messenger, communicating behind a curtain, is an unconstructive response: Shawyer's Design Factor equation is never provided.  Certainly not provided to this forum (was his equation provided instead to you in private?). Not a response one would expect in an Engineering or a Scientific journal, where the editors expect disclosure (and hence a definition) of what is being discussed (in this case what is Shawyer's Design Factor equation ?) before asserting that somebody's interpretation is wrong. 

This response stands in direct contrast to the direct, constructve responses of others: Paul March (NASA) who consistently provided geometrical dimensions, defined variables, and provided honest answers to honest questions.  Same for Prof. McCulloch who answered questions directly (not using messengers) both in this forum and in his blog, instead of keeping obscure what are his equations.

I look forward to your providing to this forum what is Shawyer's Design Factor equation (and then we can have a discussion of my effort to ascertain Shawyer's Design Factor, vis-à-vis Shawyer's previous papers that I had to rely on).

I also look forward to Shawyer providing the big and small diameters and the length of the truncated cones used in his  experiments, so that this forum can verify the claims being made in his papers regarding the validity of his Design Factor (instead of having to rely on assessing dimensions from photographs, or resorting to parametrization of variables over a whole range, as NASA had to do to guess Shawyer's dimensions).

Here is my latest email to Roger Shawyer, where I asked him to please provide me his Df equation in either Excel format or to modify your shared Df equation:

Quote
Hi Roger,

I have posted your email to the forum and agree working with high power anything can be dangerous. Your warning is very timely as others are buying magnetrons and screwing them to homemade cavities.

My approach is to use a programmable Rf generator (with programmable output level), connected to a max 20W Rf amp and SLOWLY increase the power applied as I vary the Rf frequency to parametrise the cavities characteristics. My cavity and Rf amplifier will be inside a tight Alum mesh 6 sided Faraday Cage (FC) to ensure no energy gets out to cause any problems. Even with the FC in place, I don't plan on getting any closer than 2 meters when it is powered on. Like you I have seen what high power and high voltage in high Q circuits can do.

As I really don't like working with bad data nor reinventing the wheel, would you please send me your Df equation in either excel or just as a corrected formula in the form below.

This would really help me understand the dynamics occurring inside a cavity as the applied Rf versus the Df derived from that frequency and cavity dimensions vary.

If I understand you correctly, no matter what the frequency nor cavity dimensions, the calculated Df should always be in the range 0 - 1 even if the small end diameter is below the cutoff frequency? In the Rodal version, if I set a really large big diameter, a really large length, small diameter to 0.299705m, the Df does = 1.0000 at 1GHz but will go above 1 if the small end diameter is less than the frequency wavelength (less than 0.299705m at 1GHz). Should that not happen?

Cl = cavity length
Cb = cavity big end diameter
Cs = cavity small end diameter
f = applied frequency
c = light speed in selected medium

Df =    2c * (Cl^2) * SQRT(4 + (c / (Cl * f))^2) * f * (Cb - Cs) /
          (4 * Cb * Cs * (Cl * f)^2 + (c^2) * (Cb * Cs - (2 * Cl)^2))

My intention has been made clear, to openly share everything I learn and experience during my journey to replicate the Shawyer Teeter-Totter balance beam test rig and the Flight Thruster.

When given a new equation for the EM Drive, I will write an Excel spreadsheet to allow all variables to be varied, observe the results and try to learn about how this device works. Which is what I did with your equation, and discovered there appears to be a optional frequency that gives the highest Df for a fixed set of cavity dimensions.

Further to what I learned from Shawyer, I have verified your equation does conditionally match what Shawyer claims for his Df.

Big diameter matches small diameter, Df = 0. Frequency and length variation have no effect.

Very large big diameter AND very long length, Df = 1, when small diameter matches frequency wavelength. When small diameter is larger than wavelength Df < 1, when small diameter is less than wavelength Df > 1. I note Shawyer did not mention than cavity length would have an effect on Df at this boundary condition but it does.

I'm not here to score points nor take sides. I will follow the data to where it leads me, will safely replicate the Flight Thruster and openly share that journey. I do appreciate your assistance in providing your Df equation and further thoughts.

Peace.
Title: Re: EM Drive Developments - related to space flight applications - Thread 2
Post by: Rodal on 05/13/2015 03:17 pm
...
My intention has been made clear, to openly share everything I learn and experience during my journey to replicate the Shawyer Teeter-Totter balance beam test rig and the Flight Thruster.

When given a new equation for the EM Drive, I will write an Excel spreadsheet to allow all variables to be varied, observe the results and try to learn about how this device works. Which is what I did with your equation, and discovered there appears to be a optional frequency that gives the highest Df for a fixed set of cavity dimensions.

Further to what I learned from Shawyer, I have verified your equation does conditionally match what Shawyer claims for his Df.

Big diameter matches small diameter, Df = 0. Frequency and length variation have no effect.

Very large big diameter AND very long length, Df = 1, when small diameter matches frequency wavelength. When small diameter is larger than wavelength Df < 1, when small diameter is less than wavelength Df > 1. I note Shawyer did not mention than cavity length would have an effect on Df at this boundary condition but it does.

I'm not here to score points nor take sides. I will follow the data to where it leads me, will safely replicate the Flight Thruster and openly share that journey. I do appreciate your assistance in providing your Df equation and further thoughts.

Peace.
A) Since you are appreciative of my efforts in "reading the tea leaves" from Shawyer's papers  :) ,  as a helpful, and further elucidating point, one of the critical variables that Shawyer did not explicitly define in his Design Factor equation variables in his papers is the cut-off frequency.  As previously discussed in the forum (in my exchanges with @aero). The equation I provided you takes:

cutOffWavelength = 2*cavityLength

This is where the cavity length comes from in the expression I provided you.

I can also provide you with an equation with any other definition for the cutOffWavelength you may prefer, for example based on the small diameter's or the big diameter's dimension.  However in that case, you will be faced with two conundrums:

1) are you assuming that the longest length (suitably multiplied by the appropriate factor) is the cavityLength or the big diameter?  (if both could occur then two expressions for the cutOffWavelength need to be computed and an IF statement to decide on the correct cutOffWavelength)

2) the correct cutOffWavelength for a truncated cone depends on the exact solution functions  (*): spherical Bessel function and associated Legendre functions eigenvalue solutions, which would result in an equation that you would not be able to model with Excel.  Based on my reading of Shawyer's papers it is apparent that Shawyer's solution is not an exact solution based on spherical Bessel functions and associated Legendre functions but is instead based on elementary functions used as an approximation over an undefined range.



B) I can also provide with a closed-form equation (that you can calculate quickly in Excel) that gives the frequency at which the Design Factor blows up and goes to infinity (instead of having to numerically approximate this frequency).  Please let me know whether this would be useful to you.



(*) for a perfect cylinder, there is only one eigenvalue problem, which is readily solved in terms of the cylindrical Bessel function zeros Xmn and  X'mn, and closed-form expressions can be given for the cutoff frequency in terms of given mode shapes, see http://en.wikipedia.org/wiki/Cutoff_frequency#Waveguides
Title: Re: EM Drive Developments - related to space flight applications - Thread 2
Post by: rrb6699 on 05/13/2015 03:23 pm
wouldn't a logical place to start be at 180 Degrees
out-of-phase then vary it from there to see if a sweet spot exists?
rr
Title: Re: EM Drive Developments - related to space flight applications - Thread 2
Post by: TheTraveller on 05/13/2015 03:46 pm
...
My intention has been made clear, to openly share everything I learn and experience during my journey to replicate the Shawyer Teeter-Totter balance beam test rig and the Flight Thruster.

When given a new equation for the EM Drive, I will write an Excel spreadsheet to allow all variables to be varied, observe the results and try to learn about how this device works. Which is what I did with your equation, and discovered there appears to be a optional frequency that gives the highest Df for a fixed set of cavity dimensions.

Further to what I learned from Shawyer, I have verified your equation does conditionally match what Shawyer claims for his Df.

Big diameter matches small diameter, Df = 0. Frequency and length variation have no effect.

Very large big diameter AND very long length, Df = 1, when small diameter matches frequency wavelength. When small diameter is larger than wavelength Df < 1, when small diameter is less than wavelength Df > 1. I note Shawyer did not mention than cavity length would have an effect on Df at this boundary condition but it does.

I'm not here to score points nor take sides. I will follow the data to where it leads me, will safely replicate the Flight Thruster and openly share that journey. I do appreciate your assistance in providing your Df equation and further thoughts.

Peace.
A) Since you are appreciative of my efforts in "reading the tea leaves" from Shawyer's papers  :) ,  as a helpful, and further elucidating point, one of the critical variables that Shawyer did not explicitly define in his Design Factor equation variables in his papers is the cut-off frequency.  As previously discussed in the forum (in my exchanges with @aero). The equation I provided you takes:

cutOffWavelength = 2*cavityLength

This is where the cavity length comes from in the expression I provided you.

I can also provide you with an equation with any other definition for the cutOffWavelength you may prefer, for example based on the small diameter's or the big diameter's dimension.  However in that case, you will be faced with two conundrums:

1) are you assuming that the longest length (suitably multiplied by the appropriate factor) is the cavityLength or the big diameter?  (if both could occur then two expressions for the cutOffWavelength need to be computed and an IF statement to decide on the correct cutOffWavelength)

2) the correct cutOffWavelength for a truncated cone depends on the exact solution functions  (*): spherical Bessel function and associated Legendre functions eigenvalue solutions, which would result in an equation that you would not be able to model with Excel.  Based on my reading of Shawyer's papers it is apparent that Shawyer's solution is not an exact solution based on spherical Bessel functions and associated Legendre functions but is instead based on elementary functions used as an approximation over an undefined range.



B) I can also provide with a closed-form equation (that you can calculate quickly in Excel) that gives the frequency at which the Design Factor blows up and goes to infinity (instead of having to numerically approximate this frequency).  Please let me know whether this would be useful to you.



(*) for a perfect cylinder, there is only one eigenvalue problem, which is readily solved in terms of the cylindrical Bessel function zeros Xmn and  X'mn, and closed-form expressions can be given for the cutoff frequency in terms of given mode shapes, see http://en.wikipedia.org/wiki/Cutoff_frequency#Waveguides

Appreciate the reply. Lets wait to see if Shawyer decided to help us stop needing to read tea leaves.

This may be useful information:

Quote
As a start to understanding the microwave engineering needed to design a successful EmDrive cavity I recommend:

Microwave Engineering Passive Circuits

Peter A. Rizzi

Prentice Hall 1988

ISBN 0-13-586702-9 025

This will enable you to calculate the guide wavelengths for your proposed cavity geometry, resonant frequency and mode. You can then develop a numerical model to integrate incremental guide wavelengths to arrive at an accurate set of dimensions.

If you use a commercial  finite element software package as a design aid, make sure it can cope with close to cut-off conditions. Most are hopelessly inaccurate and will even give an answer at dimensions below cut-off.

Good luck

Roger Shawyer

Just received this from Roger:

Quote

Hi

The derivation of my Design Factor is given in: The EmDrive - A New Satellite Propulsion Technology. This paper was presented at the “2nd Conference on Disruptive Technology in Space Activities” in Toulouse 2010, which I have attached.

As you can see the equation contains the guide wavelength terms for both ends of the cavity. The solution of these terms depends on waveguide type, dimensions, frequency and mode and can be found in any good microwave engineering text book such as the one I recommended to you.

As you will discover the solutions to the guide wavelength terms are very non-linear, which makes the accurate design of a tapered cavity particularly difficult. At SPR Ltd we have developed our own software to give a 2D high resolution numerical solution to the problem. This proprietary design software has been validated for a number of different cavities by ourselves, as well as different research groups operating under commercial agreements with SPR Ltd.

To answer your particular question about operating below cut-off, in these conditions the guide wavelength goes to infinity. As you can see from my equation, this is one reason why the value of the design factor is constrained between 0 and 1.

Hope this helps, and feel free to share this on the thread.

Best regards

Roger.

So we need to design for Df slightly < 1.0.
Title: Re: EM Drive Developments - related to space flight applications - Thread 2
Post by: SeeShells on 05/13/2015 03:50 pm
...

Best Regards,
Todd

Barely able to keep pace Todd...its a good thing. maximizing asymmetry in attenuation different from absorption like the stuff I used to work with?

http://www.westernrubber.com/products/himag-microwave-absorbers/himag-cavity-resonance-absorbers/

It is getting pretty hot and heavy in here and I'm not sure I am keeping up either. Love it though.
I've been mulling around the ideas of harmonics and wondered if anyone has considered injecting 2 RF sources into the cavity,
One set and the other variable in frequency? I've been slowly working my way through this but like I said it's been slow. I welcome and inputs and thoughts.


Maybe first try to state what kind of nonlinear coupling effects could be at play to make a difference ? Linear system => result(wave1+wave2)=result(wave1)+result(wave2) so doing wave1+wave2 would bring nothing new, qualitatively, without a mechanism to explain result(single wave)!=0 in the first place...


Kudos to SeeShells !!!!!  (do we need more hot and heavy weather to keep producing these great suggestions   :) )

This from this paper recently brought up by Todd (hat tip to WarpTech), which confirms the validity of SeeShells suggestion:

http://www.radioeng.cz/fulltexts/2011/11_02_472_478.pdf

Attenuation in Rectangular Waveguides with Finite Conductivity Walls
Kim Ho YEAP, Choy Yoong THAM, Ghassan YASSIN, Kee Choon YEONG
RADIOENGINEERING, VOL. 20, NO. 2, JUNE 2011


Quote from: Kim Ho YEAP, Choy Yoong THAM, Ghassan YASSIN, Kee Choon YEONG
An important consequence of this
work is the demonstration that the loss computed for degenerate
modes propagating simultaneously is not simply
additive.
In other words, the combined loss of two co-existing
modes is higher than adding the losses of two modes
propagating independently. This can be explained by the
mode coupling effects, which is significant when the phase
constants of two propagating modes are different yet very
close. 
Thanks. You guys are good!
All I was doing was sitting and relaxing in my hot tub (ok ok I had a cold one going too) imagining and visualizing the 2 waves in the EM container and seeing the mixing of the waves. I saw the additive factors and the losses and it looked like it might work well, but when added in the idea that to really work I needed to include what Mulletron threw at me (big kudos to him) the PTFE material you used and the properties of it to attenuate the signals I decided it was to much info and needed more time or another cold one. ;)
Title: Re: EM Drive Developments - related to space flight applications - Thread 2
Post by: Rodal on 05/13/2015 03:54 pm
...
This may be useful information:

Quote
As a start to understanding the microwave engineering needed to design a successful EmDrive cavity I recommend:

Microwave Engineering Passive Circuits

Peter A. Rizzi

Prentice Hall 1988

ISBN 0-13-586702-9 025

This will enable you to calculate the guide wavelengths for your proposed cavity geometry, resonant frequency and mode. You can then develop a numerical model to integrate incremental guide wavelengths to arrive at an accurate set of dimensions.

If you use a commercial  finite element software package as a design aid, make sure it can cope with close to cut-off conditions. Most are hopelessly inaccurate and will even give an answer at dimensions below cut-off.

Good luck

Roger Shawyer
Again, Shawyer does not answer directly: he does not provide his equation for the Design Factor, and neither does he define the variables thereof.

Quote
What was expected from Shawyer, the author of the equation, is to produce his equation: to answer "this is my equation:..." (defining the Design Factor and its variables, and not resorting to references where the variables are not explicitly defined).

As I pointed out in my prior message http://forum.nasaspaceflight.com/index.php?topic=36313.msg1373671#msg1373671  the main issue is a definition for the cut-off frequency in Shawyer's design factor.  Shawyer does not give you an equation for the cutoff-frequency either. 

As Shawyer signs in his message:  "Good Luck" 

(Good luck, I suppose in ever finding out what is an explicit equation for Shawyer's Design Factor that you can program in Excel  :) )


Title: Re: EM Drive Developments - related to space flight applications - Thread 2
Post by: SeeShells on 05/13/2015 04:25 pm
...

Best Regards,
Todd

Barely able to keep pace Todd...its a good thing. maximizing asymmetry in attenuation different from absorption like the stuff I used to work with?

http://www.westernrubber.com/products/himag-microwave-absorbers/himag-cavity-resonance-absorbers/

It is getting pretty hot and heavy in here and I'm not sure I am keeping up either. Love it though.
I've been mulling around the ideas of harmonics and wondered if anyone has considered injecting 2 RF sources into the cavity,
One set and the other variable in frequency? I've been slowly working my way through this but like I said it's been slow. I welcome and inputs and thoughts.


Maybe first try to state what kind of nonlinear coupling effects could be at play to make a difference ? Linear system => result(wave1+wave2)=result(wave1)+result(wave2) so doing wave1+wave2 would bring nothing new, qualitatively, without a mechanism to explain result(single wave)!=0 in the first place...


Kudos to SeeShells !!!!!  (do we need more hot and heavy weather to keep producing these great suggestions   :) )

This from this paper recently brought up by Todd (hat tip to WarpTech), which confirms the validity of SeeShells suggestion:

http://www.radioeng.cz/fulltexts/2011/11_02_472_478.pdf

Attenuation in Rectangular Waveguides with Finite Conductivity Walls
Kim Ho YEAP, Choy Yoong THAM, Ghassan YASSIN, Kee Choon YEONG
RADIOENGINEERING, VOL. 20, NO. 2, JUNE 2011


Quote from: Kim Ho YEAP, Choy Yoong THAM, Ghassan YASSIN, Kee Choon YEONG
An important consequence of this
work is the demonstration that the loss computed for degenerate
modes propagating simultaneously is not simply
additive.
In other words, the combined loss of two co-existing
modes is higher than adding the losses of two modes
propagating independently. This can be explained by the
mode coupling effects, which is significant when the phase
constants of two propagating modes are different yet very
close. 
Thanks. You guys are good!
All I was doing was sitting and relaxing in my hot tub (ok ok I had a cold one going too) imagining and visualizing the 2 waves in the EM container and seeing the mixing of the waves. I saw the additive factors and the losses and it looked like it might work well, but when added in the idea that to really work I needed to include what Mulletron threw at me (big kudos to him) the PTFE material you used and the properties of it to attenuate the signals I decided it was to much info and needed more time or another cold one. ;)
I've got to be somewhere so I just quickly scanned the paper and it looks very good. I dig into it latter.  Big Thanks Rodal!
Title: Re: EM Drive Developments - related to space flight applications - Thread 2
Post by: zellerium on 05/13/2015 04:29 pm
Zellerium

In relation to your proposed replication I remember originally mentioning a curved mirror. I was doing some microcontroller work today and now think this is not what you need for this particular experiment.  The curved mirror is just a visual display device, as you will be aware it just accentuates the reflected angle of the laser, good for viewing movement but that is about all at this particular level of usage.

What would be good is an ordinary plane/flat mirror, but DONT mount it on top of the replication.
If it is on top then due to the 2 attached strings to the roof, the device will move forward BUT also due to it hanging at the bottom of a "circle" the device will move slightly upwards. This results in the mirror moving both forward and up with the effect that the lasers beam will not correctly traverse the line you want.

Place the mirror on the rear of the replication such that the laser is pointed horizontally (not vertically) at the mirror. It will strike the mirror and reflect onto the rear wall. eg mirror at 45 degree to the replication body with the mirrored side facing towards you (and outer edge back at 45) looking horizontally at the experiment. The expected line of laser traversal is horizontal.

This allows the mirror to move forwards and up as before but the upwards movement wont impact the movement/accuracy of the laser as much as before.

Ideally if you could obtain (or borrow) some light triggered timers/counters, and arrange them with equal spacing "straight" along the projected path of the beam (push the unit gently by hand initially to observe the actual path). The timers are connected to a single-trigger-unit that starts ALL the timers Simultaneously as the beam crosses the initial "start" LDR switch., then the beams path will activate each timers LDR (Light Dependent Resistor or Light Dependent Diode) to stop each timer as the beam crosses each detectors face. From there you have counts of time, difference between timer units (acceleration or constant momentum?), average velocity etc.

Arc:

Thank you for taking the time to think about our setup. I agree that a vertical mirror will be a better choice, I hadn't considered the difference between the x and y displacements. Shouldn't be a problem to implement.

As for the light dependent diode, I like the idea! We have been talking to a computer engineer who wants to shoot the laser directly into a camera to measure the change in its position. He can write a program that finds a baseline and hopefully will be able to detect very small changes in position, and perhaps also time its trajectory to detemine momentum. But this momentum measurement might be limited by the fps of the camera?

Looking at the force plots from EW tells me this force is relatively constant (so long as our power supply is) and the pendulum will settle at a different equilibrium position fairly quickly.

What do you think of the camera idea?

Kurt Zeller


Title: Re: EM Drive Developments - related to space flight applications - Thread 2
Post by: jordan.greenhall on 05/13/2015 04:32 pm
Hi everyone, this is a notification that there is a new thread here http://forum.nasaspaceflight.com/index.php?topic=37563.0 (http://forum.nasaspaceflight.com/index.php?topic=37563.0) specifically focused on an X-prize for the EM Drive. If you are interested in helping out, please come and join us.
Title: Re: EM Drive Developments - related to space flight applications - Thread 2
Post by: TheTraveller on 05/13/2015 04:38 pm
...
This may be useful information:

Quote
As a start to understanding the microwave engineering needed to design a successful EmDrive cavity I recommend:

Microwave Engineering Passive Circuits

Peter A. Rizzi

Prentice Hall 1988

ISBN 0-13-586702-9 025

This will enable you to calculate the guide wavelengths for your proposed cavity geometry, resonant frequency and mode. You can then develop a numerical model to integrate incremental guide wavelengths to arrive at an accurate set of dimensions.

If you use a commercial  finite element software package as a design aid, make sure it can cope with close to cut-off conditions. Most are hopelessly inaccurate and will even give an answer at dimensions below cut-off.

Good luck

Roger Shawyer
Again, Shawyer does not answer directly: he does not provide his equation for the Design Factor, and neither does he define the variables thereof.

Quote
What was expected from Shawyer, the author of the equation, is to produce his equation: to answer "this is my equation:..." (defining the Design Factor and its variables, and not resorting to references where the variables are not explicitly defined).

As I pointed out in my prior message http://forum.nasaspaceflight.com/index.php?topic=36313.msg1373671#msg1373671  the main issue is a definition for the cut-off frequency in Shawyer's design factor.  Shawyer does not give you an equation for the cutoff-frequency either. 

As Shawyer signs in his message:  "Good Luck" 

(Good luck, I suppose in ever finding out what is an explicit equation for Shawyer's Design Factor that you can program in Excel  :) )



As to Shawyer's response:

Quote
At SPR Ltd we have developed our own software to give a 2D high resolution numerical solution to the problem. This proprietary design software has been validated for a number of different cavities by ourselves, as well as different research groups operating under commercial agreements with SPR Ltd. 

That is not an answer to your question, which was what is the formula for the Design Factor.  As a counterexample, I have a full 3D exact solution to the truncated cone for arbitrary mode shapes, in Mathematica, that is proprietary to me. That is useful to me, and of no use to anybody else (it can only be of some use to the extent that I release particular numerical solutions and images, but it is of no help to you in programming the Design Factor in Excel)   ???  .

I appreciate SPR is in business and maybe giving away too much info is not in their best interest. I asked but did not receive all of what I asked for. I'm appreciative of the info Shawyer has shared. I'll follow his bread crumbs and see what I can come up with.

I have started to learn microwave waveguide theory and application. Lots of info on the net. I learn quick. From my read of Shawyers equations, just with an hour of microwave waveguide research, I'm starting to see where is he going and why.

End result is I will have a version of the SPR Flight Thruster, variable Rf frequency generator and variable power amp working on a knife edge Teeter Totter test rig. I expect it will teach me a lot and fill in any holes.
Title: Re: EM Drive Developments - related to space flight applications - Thread 2
Post by: TheTraveller on 05/13/2015 04:49 pm
Hi everyone, this is a notification that there is a new thread here http://forum.nasaspaceflight.com/index.php?topic=37563.0 (http://forum.nasaspaceflight.com/index.php?topic=37563.0) specifically focused on an X-prize for the EM Drive. If you are interested in helping out, please come and join us.

My comment:
http://forum.nasaspaceflight.com/index.php?topic=37563.msg1373714#msg1373714
Title: Re: EM Drive Developments - related to space flight applications - Thread 2
Post by: Rodal on 05/13/2015 04:54 pm
....

My comment:
http://forum.nasaspaceflight.com/index.php?topic=37563.msg1373714#msg1373714
I sincerely applaud your altruism shown in your comment.  Thank you for your generosity. 
 :)
Title: Re: EM Drive Developments - related to space flight applications - Thread 2
Post by: zen-in on 05/13/2015 04:55 pm
Zellerium

In relation to your proposed replication I remember originally mentioning a curved mirror. I was doing some microcontroller work today and now think this is not what you need for this particular experiment.  The curved mirror is just a visual display device, as you will be aware it just accentuates the reflected angle of the laser, good for viewing movement but that is about all at this particular level of usage.

What would be good is an ordinary plane/flat mirror, but DONT mount it on top of the replication.
If it is on top then due to the 2 attached strings to the roof, the device will move forward BUT also due to it hanging at the bottom of a "circle" the device will move slightly upwards. This results in the mirror moving both forward and up with the effect that the lasers beam will not correctly traverse the line you want.

Place the mirror on the rear of the replication such that the laser is pointed horizontally (not vertically) at the mirror. It will strike the mirror and reflect onto the rear wall. eg mirror at 45 degree to the replication body with the mirrored side facing towards you (and outer edge back at 45) looking horizontally at the experiment. The expected line of laser traversal is horizontal.

This allows the mirror to move forwards and up as before but the upwards movement wont impact the movement/accuracy of the laser as much as before.

Ideally if you could obtain (or borrow) some light triggered timers/counters, and arrange them with equal spacing "straight" along the projected path of the beam (push the unit gently by hand initially to observe the actual path). The timers are connected to a single-trigger-unit that starts ALL the timers Simultaneously as the beam crosses the initial "start" LDR switch., then the beams path will activate each timers LDR (Light Dependent Resistor or Light Dependent Diode) to stop each timer as the beam crosses each detectors face. From there you have counts of time, difference between timer units (acceleration or constant momentum?), average velocity etc.

There was a lot of discussion about the EW measurement methods on pages 62 - 70 of this thread.   The actual displacement of the TP when RF power is applied is very miniscule.   The illustration below shows a calibration pulse next to the distance measurement scale.   The end of the TP has moved just 1 micrometer.    Earlier in this thread I did a rough calculation of a laser - mirror displacement measurement setup based on this small displacement:

A problem with this experiment is the extremely small displacements that indicate a thrust.   A displacement of 4 micrometers has the TP beam move through just 1.7 arcSec. of rotation.   If a laser beam was reflected off the LDS moving mirror and someone was 1 km away they would see the reflected dot move just a few mm.   The LDS is just as sensitive to angular changes of the mirror.   An experiment of this type requires repeatable, consistent results with a signal level far above what is currently seen to provide proof of this proposed theory of its operation.
Title: Re: EM Drive Developments - related to space flight applications - Thread 2
Post by: TheTraveller on 05/13/2015 05:11 pm
....

My comment:
http://forum.nasaspaceflight.com/index.php?topic=37563.msg1373714#msg1373714
I sincerely applaud your altruism shown in your comment.  Thank you for your generosity. 
 :)

This is me and how I work & live.

Been around too long. Have seen what greed does to people. I'm interested in solving the issue of does it work or not, with no theory wheel barrow to push. That is why I will follow the data and why I did what I did with your Df equation. It was an example of chasing the data to see what it says and where it leads me.

I'm rolling back the years and working through Shawyers Df and other supporting equation, while teaching myself what happens to microwaves when constrained in waveguides of various shapes and contours.
Title: Re: EM Drive Developments - related to space flight applications - Thread 2
Post by: zellerium on 05/13/2015 05:20 pm


There was a lot of discussion about the EW measurement methods on pages 62 - 70 of this thread.   The actual displacement of the TP when RF power is applied is very miniscule.   The illustration below shows a calibration pulse next to the distance measurement scale.   The end of the TP has moved just 1 micrometer.    Earlier in this thread I did a rough calculation of a laser - mirror displacement measurement setup based on this small displacement:

A problem with this experiment is the extremely small displacements that indicate a thrust.   A displacement of 4 micrometers has the TP beam move through just 1.7 arcSec. of rotation.   If a laser beam was reflected off the LDS moving mirror and someone was 1 km away they would see the reflected dot move just a few mm.   The LDS is just as sensitive to angular changes of the mirror.   An experiment of this type requires repeatable, consistent results with a signal level far above what is currently seen to provide proof of this proposed theory of its operation.

Is that displacement assuming we will get EW's thrust values?
If we use a microwave oven magnetron putting out 1 kW I hope we get much higher thrust values than EW who used only 17 ~ 50 W.
I see your concern, but without a low thrust torsion pendulum it seems near impossible to measure 20~100 micoNewtons 
Title: Re: EM Drive Developments - related to space flight applications - Thread 2
Post by: Rodal on 05/13/2015 05:29 pm
....

There was a lot of discussion about the EW measurement methods on pages 62 - 70 of this thread.   The actual displacement of the TP when RF power is applied is very miniscule.   The illustration below shows a calibration pulse next to the distance measurement scale.   The end of the TP has moved just 1 micrometer.    Earlier in this thread I did a rough calculation of a laser - mirror displacement measurement setup based on this small displacement:

A problem with this experiment is the extremely small displacements that indicate a thrust.   A displacement of 4 micrometers has the TP beam move through just 1.7 arcSec. of rotation.   If a laser beam was reflected off the LDS moving mirror and someone was 1 km away they would see the reflected dot move just a few mm.   The LDS is just as sensitive to angular changes of the mirror.   An experiment of this type requires repeatable, consistent results with a signal level far above what is currently seen to provide proof of this proposed theory of its operation.
What would be an acceptable displacement level for NASA EW to consistently measure with their set-up, in your view?

20 micrometers ?

Can you give us your guesstimate for an acceptable displacement threshold ?  Thanks
Title: Re: EM Drive Developments - related to space flight applications - Thread 2
Post by: Jared on 05/13/2015 05:35 pm
....

My comment:
http://forum.nasaspaceflight.com/index.php?topic=37563.msg1373714#msg1373714
I sincerely applaud your altruism shown in your comment.  Thank you for your generosity. 
 :)

This is me and how I work & live.

Been around too long. Have seen what greed does to people. I'm interested in solving the issue of does it work or not, with no theory wheel barrow to push. That is why I will follow the data and why I did what I did with your Df equation. It was an example of chasing the data to see what it says and where it leads me.

I'm rolling back the years and working through Shawyers Df and other supporting equation, while teaching myself what happens to microwaves when constrained in waveguides of various shapes and contours.

What a great statement. For weeks now, reading this thread has been my favorite intellectual pastime (can't help you with any of the maths though, as I majored in linguistics and political science). ;)
Title: Re: EM Drive Developments - related to space flight applications - Thread 2
Post by: zen-in on 05/13/2015 05:36 pm


There was a lot of discussion about the EW measurement methods on pages 62 - 70 of this thread.   The actual displacement of the TP when RF power is applied is very miniscule.   The illustration below shows a calibration pulse next to the distance measurement scale.   The end of the TP has moved just 1 micrometer.    Earlier in this thread I did a rough calculation of a laser - mirror displacement measurement setup based on this small displacement:

A problem with this experiment is the extremely small displacements that indicate a thrust.   A displacement of 4 micrometers has the TP beam move through just 1.7 arcSec. of rotation.   If a laser beam was reflected off the LDS moving mirror and someone was 1 km away they would see the reflected dot move just a few mm.   The LDS is just as sensitive to angular changes of the mirror.   An experiment of this type requires repeatable, consistent results with a signal level far above what is currently seen to provide proof of this proposed theory of its operation.

Is that displacement assuming we will get EW's thrust values?
If we use a microwave oven magnetron putting out 1 kW I hope we get much higher thrust values than EW who used only 17 ~ 50 W.
I see your concern, but without a low thrust torsion pendulum it seems near impossible to measure 20~100 micoNewtons

I don't dispute the need for a low thrust torsion pendulum.  I'm just stating that a reflected laser beam can't provide the required resolution.    The measurement sensor EW used could measure small displacements with more precision than a reflected laser beam.    Microwave Oven magnetrons are hazardous and potentially lethal devices when removed from an oven.   For FCC compliance and safety reasons it should be enclosed by a 6-side Faraday cage with a power interlock.   Even that may not be enough to satisfy the FCC if interference is reported.
Title: Re: EM Drive Developments - related to space flight applications - Thread 2
Post by: Notsosureofit on 05/13/2015 05:37 pm


There was a lot of discussion about the EW measurement methods on pages 62 - 70 of this thread.   The actual displacement of the TP when RF power is applied is very miniscule.   The illustration below shows a calibration pulse next to the distance measurement scale.   The end of the TP has moved just 1 micrometer.    Earlier in this thread I did a rough calculation of a laser - mirror displacement measurement setup based on this small displacement:

A problem with this experiment is the extremely small displacements that indicate a thrust.   A displacement of 4 micrometers has the TP beam move through just 1.7 arcSec. of rotation.   If a laser beam was reflected off the LDS moving mirror and someone was 1 km away they would see the reflected dot move just a few mm.   The LDS is just as sensitive to angular changes of the mirror.   An experiment of this type requires repeatable, consistent results with a signal level far above what is currently seen to provide proof of this proposed theory of its operation.

Is that displacement assuming we will get EW's thrust values?
If we use a microwave oven magnetron putting out 1 kW I hope we get much higher thrust values than EW who used only 17 ~ 50 W.
I see your concern, but without a low thrust torsion pendulum it seems near impossible to measure 20~100 micoNewtons

Cavendish pendulum.
Title: Re: EM Drive Developments - related to space flight applications - Thread 2
Post by: TheTraveller on 05/13/2015 05:39 pm


There was a lot of discussion about the EW measurement methods on pages 62 - 70 of this thread.   The actual displacement of the TP when RF power is applied is very miniscule.   The illustration below shows a calibration pulse next to the distance measurement scale.   The end of the TP has moved just 1 micrometer.    Earlier in this thread I did a rough calculation of a laser - mirror displacement measurement setup based on this small displacement:

A problem with this experiment is the extremely small displacements that indicate a thrust.   A displacement of 4 micrometers has the TP beam move through just 1.7 arcSec. of rotation.   If a laser beam was reflected off the LDS moving mirror and someone was 1 km away they would see the reflected dot move just a few mm.   The LDS is just as sensitive to angular changes of the mirror.   An experiment of this type requires repeatable, consistent results with a signal level far above what is currently seen to provide proof of this proposed theory of its operation.

Is that displacement assuming we will get EW's thrust values?
If we use a microwave oven magnetron putting out 1 kW I hope we get much higher thrust values than EW who used only 17 ~ 50 W.
I see your concern, but without a low thrust torsion pendulum it seems near impossible to measure 20~100 micoNewtons

If I might ask. What are your cavity dimensions to obtain cavity resonance at 2.45GHz? For the EW copper cavity dimensions, 2.45GHZ is a long way from anything that will enable the magnetron to fill the cavity with a lot of microwave energy. If you don't get cavity resonance with the magnetron frequency, do you expect to get thrust?
Title: Re: EM Drive Developments - related to space flight applications - Thread 2
Post by: zen-in on 05/13/2015 05:47 pm
....

There was a lot of discussion about the EW measurement methods on pages 62 - 70 of this thread.   The actual displacement of the TP when RF power is applied is very miniscule.   The illustration below shows a calibration pulse next to the distance measurement scale.   The end of the TP has moved just 1 micrometer.    Earlier in this thread I did a rough calculation of a laser - mirror displacement measurement setup based on this small displacement:

A problem with this experiment is the extremely small displacements that indicate a thrust.   A displacement of 4 micrometers has the TP beam move through just 1.7 arcSec. of rotation.   If a laser beam was reflected off the LDS moving mirror and someone was 1 km away they would see the reflected dot move just a few mm.   The LDS is just as sensitive to angular changes of the mirror.   An experiment of this type requires repeatable, consistent results with a signal level far above what is currently seen to provide proof of this proposed theory of its operation.
What would be an acceptable displacement level for NASA EW to consistently measure with their set-up, in your view?

20 micrometers ?

Can you give us your guesstimate for an acceptable displacement threshold ?  Thanks
First it needs to be established that a displacement is produced when the EW device has RF power applied to it.   Tilting and changes of the CoM due to thermal-mechanical effects could also produce the same readings.    The step response of the EW TP when high voltage DC calibration pulses are applied is very different from what is observed when RF power is applied.  We know the calibration pulses produce a displacement.     If the step response due to applied RF looked the same then that would be more proof the TP underwent a displacement when RF power was applied.    Conversely if a thermal step drive was applied to the cavity (with a DC power resistor, heating pads, or focused IR energy) and that step response had the same characteristics as the RF response there would be more evidence in the other direction.
Title: Re: EM Drive Developments - related to space flight applications - Thread 2
Post by: frobnicat on 05/13/2015 06:02 pm


There was a lot of discussion about the EW measurement methods on pages 62 - 70 of this thread.   The actual displacement of the TP when RF power is applied is very miniscule.   The illustration below shows a calibration pulse next to the distance measurement scale.   The end of the TP has moved just 1 micrometer.    Earlier in this thread I did a rough calculation of a laser - mirror displacement measurement setup based on this small displacement:

A problem with this experiment is the extremely small displacements that indicate a thrust.   A displacement of 4 micrometers has the TP beam move through just 1.7 arcSec. of rotation.   If a laser beam was reflected off the LDS moving mirror and someone was 1 km away they would see the reflected dot move just a few mm.   The LDS is just as sensitive to angular changes of the mirror.   An experiment of this type requires repeatable, consistent results with a signal level far above what is currently seen to provide proof of this proposed theory of its operation.

Is that displacement assuming we will get EW's thrust values?
If we use a microwave oven magnetron putting out 1 kW I hope we get much higher thrust values than EW who used only 17 ~ 50 W.
I see your concern, but without a low thrust torsion pendulum it seems near impossible to measure 20~100 micoNewtons

Cavendish pendulum.

Mmm, yes, for having "probed" that specific aspect of the pendulum system used at EagleWorks, their design is quite stiff : produces small deviation wrt forces. The forces are small and this relatively high stiffness (due to flexure bearings as pivots) doesn't help. Rambling again : the exact apparent stiffness (in µN/µm at end of arm) at EW is a poorly characterised aspect of the experiments, taking the calibration pulses as a reliable starting point gives between 9µN/µm to 40µN/µm across the various released charts, and all are in contradiction with both flexure bearings ratings and harmonic oscillation period (when visible, on underdamped plots). Appears the measures on the vertical scale of plots show much higher stiffness than it should be (too low displacement readings ?).

Anyway, there is no reason not to go with a much lower stiffness system, or no stiffness at all (no position restoring force) and record mm or cm displacements (linear or angular) as thrust accelerates the mass (as in Shawyer).
Title: Re: EM Drive Developments - related to space flight applications - Thread 2
Post by: TheTraveller on 05/13/2015 06:05 pm
Email received from Roger Shawyer in regard to a question to verify if Dr. Rodal's Df excel equation is correct.

Shawyer also cautions, very strongly, on the dangers of working with high power microwave systems.
I obtained the following expression for the frequency at which my interpretation of Shawyer's Design Factor blows up.

frequencySingularity =cMedium* Sqrt[ (4 cavityLength^2 -  bigDiameter smallDiameter)]     
                                                     /(2*cavityLength*Sqrt[bigDiameter*smallDiameter] )

This can be simplified as follows:

Defining:

cutOffWavelength = 2* cavityLength   (this was assumed, and built-in in my equation, other expressions for cutOffWavelength  can be used alternatively)

geometricAverageDiameter = Sqrt[bigDiameter*smallDiameter]

then

frequencySingularity = cMedium* Sqrt[ (cutOffWavelength^2 - geometricAverageDiameter^2)])
                                                  /(cutOffWavelength * geometricAverageDiameter)

Thanks. Appreciated. Will add yours to the Shawyer equations my old brain is working through. Making progress but what happened to the skills I had during my Uni days? Too many engineering calculators now days. But then they do reduce human calculator / slide rule errors and hence risk.

Question? What if cavity length is not the biggest cavity dimension? Should not the biggest cavity dimension be used, as I have read in the microwave waveguide theory stuff I'm researching?
Title: Re: EM Drive Developments - related to space flight applications - Thread 2
Post by: TheTraveller on 05/13/2015 06:12 pm
...
Question? What if cavity length is not the biggest cavity dimension? Should not the biggest cavity dimension be used, as I have read in the microwave waveguide theory stuff I'm researching?
Roughly correct (the diameter and the length have different factors), would you like me to give you an equation for the Design Factor in that case?

Yes please. :)

Would also like to understand the different factors & how they are derived.
Title: Re: EM Drive Developments - related to space flight applications - Thread 2
Post by: Rodal on 05/13/2015 06:16 pm
...

Mmm, yes, for having "probed" that specific aspect of the pendulum system used at EagleWorks, their design is quite stiff : produces small deviation wrt forces. The forces are small and this relatively high stiffness (due to flexure bearings as pivots) doesn't help. Rambling again : the exact apparent stiffness (in µN/µm at end of arm) at EW is a poorly characterised aspect of the experiments, taking the calibration pulses as a reliable starting point gives between 9µN/µm to 40µN/µm across the various released charts, and all are in contradiction with both flexure bearings ratings and harmonic oscillation period (when visible, on underdamped plots). Appears the measures on the vertical scale of plots show much higher stiffness than it should be (too low displacement readings ?).

Anyway, there is no reason not to go with a much lower stiffness system, or no stiffness at all (no position restoring force) and record mm or cm displacements (linear or angular) as thrust accelerates the mass (as in Shawyer).

What measured force at NASA Eagleworks would be high enough to give confidence in the force, in your opinion?

Or there would still be a problem in your opinion with higher measured forces and higher measured displacements, as long as the stiffness remains in that range?

For example would a measured force of 200 µN and a measured displacement of 20 µm giving 10 µN/µm stiffness still lack confidence in your opinion ?

(200 µN is twice the minimum of NASA Glenn's threshold force 100 µN stipulated for measuring the EM Drive)
Title: Re: EM Drive Developments - related to space flight applications - Thread 2
Post by: TheTraveller on 05/13/2015 06:22 pm
Question? What if cavity length is not the biggest cavity dimension? Should not the biggest cavity dimension be used, as I have read in the microwave waveguide theory stuff I'm researching?

And weren't there earlier murmurings that a domed end-plate should be used on the large end of the frustum instead of a flat-plate?  Wouldn't you need to measure length to the center of the dome, and not the edge?

From what I can see, Shawyer assumes the end plates are spherical and the length between them, measured relative to a line from the vertex of the cone are the same at all points.

Which means cavities with flat end plates will not achieve the calculated Df nor have a narrow resonant frequency band not have in phase end plate reflections. Is why if you desire to have a high Q cavity, curved end plates are required.

Which is what will happen with my Flight Thruster replication, which will have a variable frequency Rf generator to hunt for and find the best operational frequency.
Title: Re: EM Drive Developments - related to space flight applications - Thread 2
Post by: zellerium on 05/13/2015 06:46 pm


I don't dispute the need for a low thrust torsion pendulum.  I'm just stating that a reflected laser beam can't provide the required resolution.    The measurement sensor EW used could measure small displacements with more precision than a reflected laser beam.    Microwave Oven magnetrons are hazardous and potentially lethal devices when removed from an oven.   For FCC compliance and safety reasons it should be enclosed by a 6-side Faraday cage with a power interlock.   Even that may not be enough to satisfy the FCC if interference is reported.

I agree the reflected laser beam cannot provide the resolution required using low power (~50W) which is why we will be using high power (.5 ~1 kW). I fully understand the risks of working with high power sources and will be taking the necessary safety precautions.   

Notsosureofit:
I have thought about using a cavendish pendulum but the diameter of the vacuum chamber is only 18.5 inches, so it will be a bit more difficult. Our current apparatus will (probably) be able to be transformed into a small cavendish if necessary, but I anticipate we will get thrust values close to Shawyer or NWPU because we are using magnetron, thus we might not need one. 

TheTraveler:
The dimensions for resonance near 2.45 GHz have been posted on Mulletron's drive along with his VNA plots. We plan to use his dimensions and incorporate a constant cross section cylinder (~3 inches tall)  connected to the larger end of the frustum with a movable plate inside. This will allow us to adjust the effective height of the cavity until our VNA tells us we have the dimensions necessary for resonance. We also plan to use an intermedate symmetric resonant cavity to convert the magnetron signal to a coaxial cable so that we can sample the signal input to the frustum to determine frequency and power. This will allow the magnetron to stay outside the vacuum chamber so that it can be air cooled as well as reduces the weight of our pendulum.

One thing I am worried about using the intermediate cavity is we are reducing the amount of noise in our signal and isolating the frequency. It seems that the phase modulation has led to an increased efficiency, so I have been considering splitting the coax signal to two different ports on the frustum for equal power signals at different phases. However this is a recent idea I have yet to research and would only be something we try after the simple setup.


 
Title: Re: EM Drive Developments - related to space flight applications - Thread 2
Post by: TheTraveller on 05/13/2015 07:05 pm
Looks like it may get very interesting:

http://forum.nasaspaceflight.com/index.php?topic=37563.msg1373799#msg1373799
Title: Re: EM Drive Developments - related to space flight applications - Thread 2
Post by: Iulian Berca on 05/13/2015 07:07 pm
Today i did the first test with the Emdrive (microwave oven magnetron and cooper frustum)
The setup (magnetron and frusum) was suspended in a pendulum.
I applied power for 40 Seconds with no visible thrust. Tomorrow will will try again with the magnetron on the small side. You have any suggestion for what should be the distance from the small side?
After this i will adjust the power to the filament of magnetron and the frequency.
To fine adjust the frequency i thought i can put 2 coils over the magnetron magnets to modify the magnetic field.
My website;
http://www.masinaelectrica.com/emdrive-independent-test/

https://www.youtube.com/watch?v=2-cgVoLUJ8w
Title: Re: EM Drive Developments - related to space flight applications - Thread 2
Post by: TheTraveller on 05/13/2015 07:10 pm
And weren't there earlier murmurings that a domed end-plate should be used on the large end of the frustum instead of a flat-plate?  Wouldn't you need to measure length to the center of the dome, and not the edge?

From what I can see, Shawyer assumes the end plates are spherical and the length between them, measured relative to a line from the vertex of the cone are the same at all points.

Which means cavities with flat end plates will not achieve the calculated Df nor have a narrow resonant frequency band not have in phase end plate reflections. Is why if you desire to have a high Q cavity, curved end plates are required.

Which is what will happen with my Flight Thruster replication, which will have a variable frequency Rf generator to hunt for and find the best operational frequency.

Thank you for the response.

I guess that anecdotally lends credence to my previously-stated suspicion (from God-knows how many months back :) ) that the bottom ring on this thruster is a painted rubber gasket that acts as a footer for the domed bottom-plate it is protecting (even though the visible top plate is flat).  The cracked paint on it is a dead-giveaway that it's a spongy material like rubber or silicon- perfect for a footer.

The Flight Thruster has a quoted Q of between 50,000 and 60,000. IMO, to achieve that, both end plates must be spherical, which helps to explain the external thick end caps, which may also enclose pressure seals.
Title: Re: EM Drive Developments - related to space flight applications - Thread 2
Post by: Mulletron on 05/13/2015 07:10 pm
As an added bonus, sticking with the dimensions provided by Eagleworks allows folks to use their COMSOL plots. While this cavity design might not be the optimal shape/size for max thrust, Paul March has provided potential replicators with an arsenal of very useful data on this thread applicable to those particular dimensions.

Quality useful info like this:
http://forum.nasaspaceflight.com/index.php?topic=36313.msg1327467#msg1327467
http://forum.nasaspaceflight.com/index.php?topic=36313.msg1333246#msg1333246
Title: Re: EM Drive Developments - related to space flight applications - Thread 2
Post by: Rodal on 05/13/2015 07:16 pm
Today i did the first test with the Emdrive (microwave oven magnetron and cooper frustum)
The setup (magnetron and frusum) was suspended in a pendulum.
I applied power for 40 Seconds with no visible thrust. Tomorrow will will try again with the magnetron on the small side. You have any suggestion for what should be the distance from the small side?
After this i will adjust the power to the filament of magnetron and the frequency.
To fine adjust the frequency i thought i can put 2 coils over the magnetron magnets to modify the magnetic field.
My website;
http://www.masinaelectrica.com/emdrive-independent-test/


VIVA LA ROMANIA  !!!!      :)

(http://www.universdecopil.ro/images/stories//animate/tricolor_imagini_animate/drapelul%20romaniei%20gif.gif)
Title: Re: EM Drive Developments - related to space flight applications - Thread 2
Post by: thebigcat on 05/13/2015 07:23 pm
I just want to throw in here that if any of you fine gentlemen need any software written I'd be more than happy to help out. I have a background in image processing and visualization, as well as machine learning, so I could also potentially help with data analysis (Have some data you want to visualize in 3D?)

I haven't caught up with the math yet, but if I can help out with any coding I'd be more than happy to.

Feel free to email ([email protected]).
Title: Re: EM Drive Developments - related to space flight applications - Thread 2
Post by: Jared on 05/13/2015 07:23 pm
Great! :) Good luck with your tests and take care!
Title: Re: EM Drive Developments - related to space flight applications - Thread 2
Post by: TheTraveller on 05/13/2015 07:24 pm
As an added bonus, sticking with the dimensions provided by Eagleworks allows folks to use their COMSOL plots. While this cavity design might not be the optimal shape/size for max thrust, Paul March has provided potential replicators with an arsenal of very useful data on this thread applicable to those particular dimensions.

Quality useful info like this:
http://forum.nasaspaceflight.com/index.php?topic=36313.msg1327467#msg1327467
http://forum.nasaspaceflight.com/index.php?topic=36313.msg1333246#msg1333246

As I understand it, your cavity is same as that one? Correct?

Did you find the same 2,445GHz cavity resonance as Paul did? If not what resonance frequency did you find?
Title: Re: EM Drive Developments - related to space flight applications - Thread 2
Post by: TheTraveller on 05/13/2015 07:29 pm
Today i did the first test with the Emdrive (microwave oven magnetron and cooper frustum)
The setup (magnetron and frusum) was suspended in a pendulum.
I applied power for 40 Seconds with no visible thrust. Tomorrow will will try again with the magnetron on the small side. You have any suggestion for what should be the distance from the small side?
After this i will adjust the power to the filament of magnetron and the frequency.
To fine adjust the frequency i thought i can put 2 coils over the magnetron magnets to modify the magnetic field.
My website;
http://www.masinaelectrica.com/emdrive-independent-test/

https://www.youtube.com/watch?v=2-cgVoLUJ8w

Well done.

Nicely rolled cavity walls.

Have you tried to calibrate your pendulum test rig by using a small spring scale to see how much force is needed to pull the cavity forward (toward the big end) say 1mm?

Doing this will give you some info on how much force you will need to generate to see some movement.
Title: Re: EM Drive Developments - related to space flight applications - Thread 2
Post by: Mulletron on 05/13/2015 08:03 pm
As an added bonus, sticking with the dimensions provided by Eagleworks allows folks to use their COMSOL plots. While this cavity design might not be the optimal shape/size for max thrust, Paul March has provided potential replicators with an arsenal of very useful data on this thread applicable to those particular dimensions.

Quality useful info like this:
http://forum.nasaspaceflight.com/index.php?topic=36313.msg1327467#msg1327467
http://forum.nasaspaceflight.com/index.php?topic=36313.msg1333246#msg1333246

As I understand it, your cavity is same as that one? Correct?

Did you find the same 2,445GHz cavity resonance as Paul did? If not what resonance frequency did you find?

Got different freqs. One is close. Found another at lower freq. Cavity dimensions are same to within about 1mm. See results here: https://goo.gl/YJN8y9  See VSWR plots.

You'll never get exactly what Eagleworks measured due to minor differences. Even torque on the rf connector makes a slight difference. I'm using a different probe too.

Congratulations to Romania too!
Title: Re: EM Drive Developments - related to space flight applications - Thread 2
Post by: zen-in on 05/13/2015 08:22 pm
Today i did the first test with the Emdrive (microwave oven magnetron and cooper frustum)
The setup (magnetron and frusum) was suspended in a pendulum.
I applied power for 40 Seconds with no visible thrust. Tomorrow will will try again with the magnetron on the small side. You have any suggestion for what should be the distance from the small side?
After this i will adjust the power to the filament of magnetron and the frequency.
To fine adjust the frequency i thought i can put 2 coils over the magnetron magnets to modify the magnetic field.
My website;
http://www.masinaelectrica.com/emdrive-independent-test/


You could modify the cavity so that it's resonant frequency can be adjusted.   I don't know how easy it would be for you to do this but you could add an interior plate on the small end that can be moved.   A couple of copper tubes soldered to the outside of the small end would act as guides for plastic rods connected to the interior plate.   Then just move the plate a little bit at a time between tests.   Eventually you will reach a resonance point.   BTW how hot does the cavity get?   It doesn't seem like the magnetron is overheating at all.   Maybe it is operating at a low power.
Title: Re: EM Drive Developments - related to space flight applications - Thread 2
Post by: SeeShells on 05/13/2015 08:28 pm
It seems that the phase modulation has led to an increased efficiency, so I have been considering splitting the coax signal to two different ports on the frustum for equal power signals at different phases. However this is a recent idea I have yet to research and would only be something we try after the simple setup.

Yes, great idea! Do you think it's a good idea to see if you can also provide phase shifting on the second injected signal?

I'm still working out this but the preliminary scan into the papers forwarded by Rodal it looks very good but other verification is needed as my math is rusty and my eraser is getting worn.
 
http://forum.nasaspaceflight.com/index.php?topic=36313.msg1373661#msg1373661

This from this paper recently brought up by Todd (hat tip to WarpTech), which confirms the validity of SeeShells suggestion:

http://www.radioeng.cz/fulltexts/2011/11_02_472_478.pdf

Attenuation in Rectangular Waveguides with Finite Conductivity Walls
Kim Ho YEAP, Choy Yoong THAM, Ghassan YASSIN, Kee Choon YEONG
RADIOENGINEERING, VOL. 20, NO. 2, JUNE 2011


Quote from: Kim Ho YEAP, Choy Yoong THAM, Ghassan YASSIN, Kee Choon YEONG
An important consequence of this
work is the demonstration that the loss computed for degenerate
modes propagating simultaneously is not simply
additive. In other words, the combined loss of two co-existing
modes is higher than adding the losses of two modes
propagating independently. This can be explained by the
mode coupling effects, which is significant when the phase
constants of two propagating modes are different yet very
close. 
Title: Re: EM Drive Developments - related to space flight applications - Thread 2
Post by: TheTraveller on 05/13/2015 08:38 pm
For a microwave newbie, for this site interesting:

http://www.microwaves101.com/encyclopedias/waveguide-mathematics#guide

Nice info on Guide Wavelength, which is something Shawyer mentions and uses in many of his equation but never gives the Guide Wavelength equation, which is as attached. Knowing the cutoff frequency for both ends, can now calc Lambda g1 (big end) and Lambda g2 (small end), then feed that into all his other equations.

BTW found this circular waveguide cutoff frequency equation at another useful site.
http://www.radio-electronics.com/info/antennas/waveguide/cutoff-frequency.php

Will be interesting to see how this all hangs together as I put together my version of the Df equation and see how it works compared to Dr. Rodal's.

Process would seem to be:

1) Calc the cutoff frequency for both ends based on their circumference.

2) Using the cutoff frequency for each end, calc the waveguide wavelength at each end using the upper equation of the two.

Then you have Lambda g1 (big end) and Lambda g2 (small end) that you can use to calc Df and other variables.

I will attempt to construct a spreadsheet that calcs all the variables in the Shawyer Theory pages. Will be interesting to see how it all hangs together.
Title: Re: EM Drive Developments - related to space flight applications - Thread 2
Post by: TheTraveller on 05/13/2015 08:39 pm
Today i did the first test with the Emdrive (microwave oven magnetron and cooper frustum)
The setup (magnetron and frusum) was suspended in a pendulum.
I applied power for 40 Seconds with no visible thrust. Tomorrow will will try again with the magnetron on the small side. You have any suggestion for what should be the distance from the small side?
After this i will adjust the power to the filament of magnetron and the frequency.
To fine adjust the frequency i thought i can put 2 coils over the magnetron magnets to modify the magnetic field.
My website;
http://www.masinaelectrica.com/emdrive-independent-test/


You could modify the cavity so that it's resonant frequency can be adjusted.   I don't know how easy it would be for you to do this but you could add an interior plate on the small end that can be moved.   A couple of copper tubes soldered to the outside of the small end would act as guides for plastic rods connected to the interior plate.   Then just move the plate a little bit at a time between tests.   Eventually you will reach a resonance point.   BTW how hot does the cavity get?   It doesn't seem like the magnetron is overheating at all.   Maybe it is operating at a low power.

Can be done / tuned like this:

Images thanks to Mulletron.

The moving internal plate is probably what Shawyer did in his EM Drive Demonstrator with the long cylindrical back end.
Title: Re: EM Drive Developments - related to space flight applications - Thread 2
Post by: deuteragenie on 05/13/2015 08:57 pm
Any resemblance with existing or future device(s) is purely coincidental.
Made with MEEP.  Does not use the correct frequencies, material, etc. but looks cute.
Title: Re: EM Drive Developments - related to space flight applications - Thread 2
Post by: deltaMass on 05/13/2015 09:08 pm
An HT Mettler balance is a work of art. Fully mechanical and gets you ten micrograms resolution which is 0.1 uN. Can't be fooled by e/m.
Title: Re: EM Drive Developments - related to space flight applications - Thread 2
Post by: Rodal on 05/13/2015 09:41 pm
Any resemblance with existing or future device(s) is purely coincidental.
Made with MEEP.  Does not use the correct frequencies, material, etc. but looks cute.

(http://forum.nasaspaceflight.com/index.php?action=dlattach;topic=36313.0;attach=829948;image)


Is that a 2-Dimensional model with MEEP?
Maxwell's equations in a flat 2-D surface?
modeling the truncated cone as a FLAT trapezium ?

Is the magnetic field (for TM modes)  a point scalar (only able to have + or - sign but the direction is always perpendicular to the surface) instead of being a vector in the azimuthal (circumferential) direction  ?

What are we seeing out of the EM Drive? evanescent wave field?

If the answers are yes, do you have enough memory to run a 3D model instead?

Thanks

Title: Re: EM Drive Developments - related to space flight applications - Thread 2
Post by: aero on 05/13/2015 10:00 pm
Any resemblance with existing or future device(s) is purely coincidental.
Made with MEEP.  Does not use the correct frequencies, material, etc. but looks cute.

You might want to increase your skin thickness or Meep resolution. Looks kind of like a Gaussian source with some of the shorter wavelengths bouncing around and longer wavelengths stepping over the boundary (numerically)?

If you are using perfect metal for the skin, then the thickness won't change the result but will help to avoid the model numerically spanning the skin. If you are modelling copper in the GHz range, please tell me about your model as I have struggled for 6 months trying to find a Drude model for copper at 2 GHz.
Title: Re: EM Drive Developments - related to space flight applications - Thread 2
Post by: frobnicat on 05/13/2015 11:04 pm
...

Mmm, yes, for having "probed" that specific aspect of the pendulum system used at EagleWorks, their design is quite stiff : produces small deviation wrt forces. The forces are small and this relatively high stiffness (due to flexure bearings as pivots) doesn't help. Rambling again : the exact apparent stiffness (in µN/µm at end of arm) at EW is a poorly characterised aspect of the experiments, taking the calibration pulses as a reliable starting point gives between 9µN/µm to 40µN/µm across the various released charts, and all are in contradiction with both flexure bearings ratings and harmonic oscillation period (when visible, on underdamped plots). Appears the measures on the vertical scale of plots show much higher stiffness than it should be (too low displacement readings ?).

Anyway, there is no reason not to go with a much lower stiffness system, or no stiffness at all (no position restoring force) and record mm or cm displacements (linear or angular) as thrust accelerates the mass (as in Shawyer).

What measured force at NASA Eagleworks would be high enough to give confidence in the force, in your opinion?

Or there would still be a problem in your opinion with higher measured forces and higher measured displacements, as long as the stiffness remains in that range?

For example would a measured force of 200 µN and a measured displacement of 20 µm giving 10 µN/µm stiffness still lack confidence in your opinion ?

(200 µN is twice the minimum of NASA Glenn's threshold force 100 µN stipulated for measuring the EM Drive)

Concerning the stiffness specifically, my remark that a relatively high stiffness (like the one at EW) "doesn't help" was for measuring displacements in the context of DIY replication attempts. Less stiffness, i.e. more displacement for a given thrust, seems more easy to measure (but at the price of longer time constants).

This is all going a frantic pace... not sure my opinion will matter in a few month but as you can guess me and a lot of people sceptics of the results are waiting for a better characterised/uncharacterised ratio rather than necessarily reaching a given absolute thrust magnitude or displacement threshold. For instance, if experiments with huge particle colliders can reliably claim fundamental discoveries from signals that are sometimes deviations less than 1% of background is because the Standard Model background is so well understood and precisely characterised (by so many people) in the context of the beams and detectors. That and a lot of data and exhaustively recorded and documented parameters to dig and slice into.

Obviously, having an experiment with a better SN ratio from the ground up is always better than recording at bad SN and then subtracting the noise or background effects, because in the latter case validity of discovery rests on the proper characterisation of such background.

Come to think of it, in the unlikely (ahem) event that there is no EM drive effect after all. Then where all those (false) positives would come from ? Necessarily from a methodology that might try to diminish backgrounds that clearly don't look like what is expected (we are trying to reduce the drift thermal effect...) but that will keep the backgrounds that look like what is expected and confounding them with signal : because removing the dielectrics slabs make thrust disappear we keep running with dielectric slabs. And those slabs are fixed with nylons screws that sometimes melt but that appear to do the job otherwise. In the hypothesis that there is no EM drive effect, experiments showing positives will always depend on strange recipes, dark corners, and overall lack of consistency and reliability. Like one shot performances that are too hard to reproduce and experiment with, or thrust that wont show same magnitude when turning test article 180°...

What I fear is that the new experiment at kW range at EW makes more thrust but no better characterised/uncharacterised ratio, that the inconsistencies or poorly characterised aspects of the experiments at 16W and 50W get forgotten or never quite elucidated, and the new experiment introduces new poorly characterised aspects, and so on.

So, what would count as a true positive IMO ? Absence of dark corners, either much better SN ratio (preferably) or much much better justified and recorded and tested background, stability against a range of parameters and phenomenological predictability wrt such parameters, no man in the feedback loop, 100s plots with more parameter studies (like the one on power), stationary or pulsed stationary operation long enough to reach new thermal equilibrium. This takes time, patience, rigoristic methodology, and enthusiasm to discover nothing if there is nothing. Not necessarily millions of $ or kW.
Title: Re: EM Drive Developments - related to space flight applications - Thread 2
Post by: TheTraveller on 05/13/2015 11:13 pm
After going over Shawyers equations and referencing several microwave info sites, I have come up with what I think is the Shawyer Df equation as attached.

It appears to be different to Dr. Rodal's Df equation as there are no references, that I know of, to cavity length and focuses on the wavelengths (Lambda g1 & Lambda g2) at the 2 end plates.

While Shawyer doesn't give equations for lambda g1 or Lambda g2 as used in his Df equation, I did found the attached reference equations which uses the circular waveguide cutoff frequency to calc end plate wavelength and the other equation to calc cutoff frequency from the circular diameter at each end plate.

Here I note Shawyer says the small end (g2) should operate just above cutoff and the big end (g1) as close to Lambda 0 as possible, while having the thinness cone angle as possible.

Trust this makes some sense. My bed time. Please throw small rocks so I can sleep and dream of microwaves flowing through tapered circular waveguides.
Title: Re: EM Drive Developments - related to space flight applications - Thread 2
Post by: deltaMass on 05/13/2015 11:47 pm
Quote from: TheTraveller
It appears to be different to Dr. Rodal's Df equation as there are no references, that I know of, to cavity length and focuses on the wavelengths (Lambda g1 & Lambda g2) at the 2 end plates.
How's that again? There is only one wavelength in the cavity - that of the RF.
Title: Re: EM Drive Developments - related to space flight applications - Thread 2
Post by: frobnicat on 05/13/2015 11:50 pm
http://www.radioeng.cz/fulltexts/2011/11_02_472_478.pdf

Attenuation in Rectangular Waveguides with Finite Conductivity Walls
Kim Ho YEAP, Choy Yoong THAM, Ghassan YASSIN, Kee Choon YEONG
RADIOENGINEERING, VOL. 20, NO. 2, JUNE 2011


Quote from: Kim Ho YEAP, Choy Yoong THAM, Ghassan YASSIN, Kee Choon YEONG
An important consequence of this
work is the demonstration that the loss computed for degenerate
modes propagating simultaneously is not simply
additive.
In other words, the combined loss of two co-existing
modes is higher than adding the losses of two modes
propagating independently. This can be explained by the
mode coupling effects, which is significant when the phase
constants of two propagating modes are different yet very
close. 

Nice gold nugget. Not quite understanding yet what mechanism is at the core of the non linearity... is it the ohmic losses in the skin effect ?

Could it allow a net DC current component in the walls from a mix of AC waves ? I'm thinking of a possible coupling with exterior B field (geomagnetic field, or nearby powerful magnetic damping system at EW) : one sine skin current would always average to 0 net Lorentz force, but could sum of sine excitations get a non null net Lorentz force, locally, from a static B field present in vicinity ?
Title: Re: EM Drive Developments - related to space flight applications - Thread 2
Post by: TheTraveller on 05/13/2015 11:53 pm
Quote from: TheTraveller
It appears to be different to Dr. Rodal's Df equation as there are no references, that I know of, to cavity length and focuses on the wavelengths (Lambda g1 & Lambda g2) at the 2 end plates.
How's that again? There is only one wavelength in the cavity - that of the RF.
Cavity wavelength varies depending on the cavity dimensions.

http://www.microwaves101.com/encyclopedias/waveguide-mathematics
Title: Re: EM Drive Developments - related to space flight applications - Thread 2
Post by: flux_capacitor on 05/13/2015 11:56 pm
Quote from: TheTraveller
It appears to be different to Dr. Rodal's Df equation as there are no references, that I know of, to cavity length and focuses on the wavelengths (Lambda g1 & Lambda g2) at the 2 end plates.
How's that again? There is only one wavelength in the cavity - that of the RF.
Cavity wavelength varies depending on the cavity dimensions.

http://www.microwaves101.com/encyclopedias/waveguide-mathematics

Indeed. The increasing confinement of a narrowing waveguide (convergent) produces a widening wavelength and a decrease of the group velocity. Conversely, a widening waveguide (divergent) produces a narrowing wavelength and an increase of the group velocity.
Title: Re: EM Drive Developments - related to space flight applications - Thread 2
Post by: TheTraveller on 05/14/2015 12:01 am
Quote from: TheTraveller
It appears to be different to Dr. Rodal's Df equation as there are no references, that I know of, to cavity length and focuses on the wavelengths (Lambda g1 & Lambda g2) at the 2 end plates.
How's that again? There is only one wavelength in the cavity - that of the RF.
Cavity wavelength varies depending on the cavity dimensions.

http://www.microwaves101.com/encyclopedias/waveguide-mathematics

Indeed. The increasing confinement of a narrowing waveguide (convergent) produces a widening wavelength and a decrease of the group velocity. Conversely, a widening waveguide (divergent) produces a narrowing wavelength and an increase of the group velocity.

The different wavelengths at the big & small ends, due to the different diameters, are what drives Shawyers Df equation.

In fact they are what cause the end plate force differentials that drive the EM Drive.
Title: Re: EM Drive Developments - related to space flight applications - Thread 2
Post by: TheTraveller on 05/14/2015 12:30 am
Quote from: TheTraveller
It appears to be different to Dr. Rodal's Df equation as there are no references, that I know of, to cavity length and focuses on the wavelengths (Lambda g1 & Lambda g2) at the 2 end plates.
How's that again? There is only one wavelength in the cavity - that of the RF.
Cavity wavelength varies depending on the cavity dimensions.

http://www.microwaves101.com/encyclopedias/waveguide-mathematics

Indeed. The increasing confinement of a narrowing waveguide (convergent) produces a widening wavelength and a decrease of the group velocity. Conversely, a widening waveguide (divergent) produces a narrowing wavelength and an increase of the group velocity.

The different wavelengths at the big & small ends, due to the different diameters, are what drives Shawyers Df equation.

In fact they are what cause the end plate force differentials that drive the EM Drive.

Notice that the expression you have posted above, as defined by Shawyer blows up (goes to infinity) for the denominator going to zero.  This occurs for

lambda0 = Sqrt[lambdag1*lambdag2]
or equivalently

cutOffWavelength = Sqrt[lambdag1*lambdag2]



same condition I have above in my message. 

_________________________________________
Disclaimer: I don't agree with the description above (since Shawyer's uses a lot of unstated engineering approximations whose validity neither he or anyone else has proven) .  I would instead write: according to Shawyer "they are what cause the end plate force differentials that drive the EM Drive."

Will ask Shawyer how SPR calculates big and small end wavelengths Lambda g1 and g2. If he shares that info, then we will have eliminated one unknown. I dislike guessing and dislike even more to reinvent the wheel especially when it involves making actual hardware. Playing mind games or with excel is different. There we can play a bit. But building hardware is serious business. I will do what ever is necessary to reduce uncertainty about operational parameters before the cavity build starts.

Don't understand your Lambda0 equation, which I use as c wavelength in reference medium, our case air.

Each end has it's own Cutoff frequency / wavelength as per these 2 industry equations.
Title: Re: EM Drive Developments - related to space flight applications - Thread 2
Post by: Rodal on 05/14/2015 12:40 am
...
Will ask Shawyer how SPR calculates big and small end wavelengths Lambda g1 and g2. If he shares that info, then we will have eliminated one unknown. I dislike guessing and dislike even more to reinvent the wheel especially when it involves making actual hardware. Playing mind games or with excel is different. There we can play a bit. But building hardware is serious business. I will do what ever is necessary to reduce uncertainty about operational parameters before the cavity build starts.
Writing technical papers is just as much serious business. 

I also dislike guessing.  Readers shouldn't have to guess what an author means when the author submits a "theory paper."

If you have to ask Shawyer how he defined something in his papers is admitting that in your view his papers are insufficiently clear for you. 

One thing for sure: Shawyer never stated in his papers how he defines the cut-off frequency. If the cut-off frequency is to be based on the truncated cone cut-off frequency (and not an approximation) one then has to run a a numerical solution (an eigenvalue problem) to obtain the cut-off frequency.


That's a small part of the reason why the engineering/scientific community has issues with his "theory paper".  The biggest part is that he makes a large range of assumptions that are not clearly stated and are not clearly supported (certainly his reference to Cullen's paper does not support his Design Factor, Cullen's paper has nothing to do with cavities having different diameter ends).
Title: Re: EM Drive Developments - related to space flight applications - Thread 2
Post by: TheTraveller on 05/14/2015 12:50 am
...
Will ask Shawyer how SPR calculates big and small end wavelengths Lambda g1 and g2. If he shares that info, then we will have eliminated one unknown. I dislike guessing and dislike even more to reinvent the wheel especially when it involves making actual hardware. Playing mind games or with excel is different. There we can play a bit. But building hardware is serious business. I will do what ever is necessary to reduce uncertainty about operational parameters before the cavity build starts.
Writing technical papers is just as much serious business. 

I also dislike guessing.  Readers shouldn't have to guess what an author means when the author submits a "theory paper."

If you have to ask Shawyer how he defined something in his papers is admitting that in your view his papers are insufficiently clear for you. 

One thing for sure: Shawyer never stated in his papers how he defines the cut-off frequency. If the cut-off frequency is to be based on the truncated cone cut-off frequency (and not an approximation) one then has to run a a numerical solution (an eigenvalue problem) to obtain the cut-off frequency.


That's a small part of the reason why the engineering/scientific community has issues with his "theory paper".  The biggest part is that he makes a large range of assumptions that are not clearly stated and are not clearly supported (certainly his reference to Cullen's paper does not support his Design Factor).

Shawyer pointed me to a reference book that he said had the equations I needed. I Google to find them. The microwave industry has those equations and I suggest Shawyer wrote his theory paper based on someone skilled in the microwave industry knowing how to use microwave industry standard equations to obtain Lambda g1 & g2. Didnt take me long to find them, so not secret squirrel info.

Anyway my bed time.
Title: Re: EM Drive Developments - related to space flight applications - Thread 2
Post by: aero on 05/14/2015 01:15 am
Doesn't that mean that it blows up for a cylindrical cavity? Yes. And for what other conditions does it blow up?

Lambda02 = Lambdag1 * Lambdag2 : I forget which one is the big end but it blows up whenever
Lambdag-small = Lambda02/Lambdag-big

I agree that the papers would be far more enlightening if the likelihood of such an occurrence were explained. And so would this forum for that matter.
Title: Re: EM Drive Developments - related to space flight applications - Thread 2
Post by: deltaMass on 05/14/2015 01:20 am
Can we at least assume that lambda1 <= lambda0 <= lambda2 ?
Title: Re: EM Drive Developments - related to space flight applications - Thread 2
Post by: deltaMass on 05/14/2015 01:28 am
was that a "yes"?
Title: Re: EM Drive Developments - related to space flight applications - Thread 2
Post by: TheTraveller on 05/14/2015 02:05 am
Doesn't that mean that it blows up for a cylindrical cavity? Yes. And for what other conditions does it blow up?

Lambda02 = Lambdag1 * Lambdag2 : I forget which one is the big end but it blows up whenever
Lambdag-small = Lambda02/Lambdag-big

I agree that the papers would be far more enlightening if the likelihood of such an occurrence were explained. And so would this forum for that matter.

Lambda0 = free air
Lambda1 = big end
Lambda2 = small end.

As defined in the attachment dominant circular waveguide cutoff wavelength is 1.7 x diameter. So Lambda1 (big end) is > Lambda2 (small end).
Title: Re: EM Drive Developments - related to space flight applications - Thread 2
Post by: deltaMass on 05/14/2015 02:15 am
First people are saying that the big end has a smaller lambda than the small end, and then other people say the opposite. Can't be bothered assembling the quotes.

In any case, if lambda1 < lambda2 (whichever ends they represent), I am still asking if the following is true

lambda1 <= lambda0 <= lambda2 ???

Title: Re: EM Drive Developments - related to space flight applications - Thread 2
Post by: Rodal on 05/14/2015 02:20 am
First people are saying that the big end has a smaller lambda than the small end, and then other people say the opposite. Can't be bothered assembling the quotes.

In any case, if lambda1 < lambda2 (whichever ends they represent), I am still asking if the following is true

lambda1 <= lambda0 <= lambda2 ???
If Shawyer's papers would clearly define the variables, you wouldn't need to ask, would you  ;)  :

Shawyer's technical paper (IAC- 08 – C4.4.7):
http://www.emdrive.com/IAC-08-C4-4-7.pdf

Shawyer's theory paper:
http://www.emdrive.com/theorypaper9-4.pdf
Title: Re: EM Drive Developments - related to space flight applications - Thread 2
Post by: deltaMass on 05/14/2015 02:35 am
I'm only asking so as to help out with the maths. It doesn't mean I believe a word about its physicality.

For example: if the expression I'm trying to get verified is true, then you can get your blow-up condition. Other conditional relations produce different results (x0 < x1 < x2 or x1 < x2 < x0: x=lambda).

Then there's also the case that it blows up exactly when the numerator is also zero. As you know, 0/0 is an indeterminate quantity.

I find a nicer way to write Shawyer's Df is like this (again using x=lambda)

Df = x0*(x2 - x1) / (x1*x2 - x02)

You'll notice that Df=0 when x1 or x2 -> infinity (proof available on request but it's dead simple)
Someone said that Df varied between 0 and 1, and should =1 when x1 or x2 -> inf. It doesn't.

And obviously Df=0 when x1 = x2. So it can be zero for two separate reasons.
Doh!
Title: Re: EM Drive Developments - related to space flight applications - Thread 2
Post by: deltaMass on 05/14/2015 03:17 am
I won't bother reading all that. From equation (5) of the IAC paper it's clear that any of the wavelengths in the waveguide are greater than the free space value. Or lambda1 > lambda0 and lambda2 > lambda0
Therefore your blow-up condition is impossible.
Title: Re: EM Drive Developments - related to space flight applications - Thread 2
Post by: TheTraveller on 05/14/2015 03:18 am
....
As defined in the attachment dominant circular waveguide cutoff wavelength is 1.7 x diameter....

 1.7 x diameter...

Yes of course, but what diameter? A cylinder has only one diameter. It has constant diameter.

Shawyer's EM Drive is not a cylinder with constant diameter.  Shawyer's EM Drive is a truncated cone.

A truncated cone has a variable diameter.  The diameters are different at each end.

Shawyer's reference (Cullen) does not deal with truncated cones.

What diameter ?

Look at what I derived in http://forum.nasaspaceflight.com/index.php?topic=36313.msg1373898#msg1373898:

...choosing the cutOffWavelength  to be TE110,

Look at where the factor of 1.7 comes from: it is Pi divided by X'11

cOW = gmD *(Pi/1.84118378134065) 
        = gmD *1.7062895542683174

1.7 times the Geometric Mean of the Diameters

cOW = cutOffWavelength

gmD =  Sqrt[bD * sD]

bD = big end diameter (m)
sD= small end diameter (m)

Shawyer only talks about the 2 ends of his cone and the wavelength of the Rf input.

Lambda0 is the wavelength of the input Rf signal in air or vacuum.

Lambda1 is the dominant cutoff wavelength for the bigger end of the cone.

Lambda2 is the dominant cutoff wavelength for the smaller end of the cone.

The focus on what is happening at each end of his cone cavity as the waves bounce off and transfer momentum, based on their individual group velocity, which varies as the dominant cutoff wavelength varies at each end. Higher group velocity at the big end and lower group velocity at the smaller end, delivering higher momentum xfer at the big end and lower momentum xfer at the smaller end.
Title: Re: EM Drive Developments - related to space flight applications - Thread 2
Post by: Rodal on 05/14/2015 03:21 am
I'm only asking so as to help out with the maths. It doesn't mean I believe a word about its physicality.

For example: if the expression I'm trying to get verified is true, then you can get your blow-up condition. Other conditional relations produce different results (x0 < x1 < x2 or x1 < x2 < x0: x=lambda).

Then there's also the case that it blows up exactly when the numerator is also zero. As you know, 0/0 is an indeterminate quantity.

I find a nicer way to write Shawyer's Df is like this (again using x=lambda)

Df = x0*(x2 - x1) / (x1*x2 - x02)

You'll notice that Df=0 when x1 or x2 -> infinity (proof available on request but it's dead simple)
Someone said that Df varied between 0 and 1, and should =1 when x1 or x2 -> inf. It doesn't.

And obviously Df=0 when x1 = x2. So it can be zero for two separate reasons.
Doh!

Don't you have your Mathematica up and running ?

 <<Df=0 when x1 or x2 -> infinity (proof available on request but it's dead simple) >> ??

Wrong.  (You need AND) See:



In[16]:= Limit[x0*(x2 - x1)/(x1*x2 - x0^2), x1 -> Infinity]

Out[16]= -(x0/x2)

In[17]:= Limit[x0*(x2 - x1)/(x1*x2 - x0^2), x2 -> Infinity]

Out[17]= x0/x1

In[18]:= Limit[x0*(x2 - x1)/(x1*x2 - x0^2), x0 -> Infinity]

Out[18]= 0

In[19]:= Limit[x0*(x2 - x1)/(x1*x2 - x0^2), x1 -> 0]

Out[19]= -(x2/x0)

In[20]:= Limit[x0*(x2 - x1)/(x1*x2 - x0^2), x2 -> 0]

Out[20]= x1/x0

In[21]:= Limit[x0*(x2 - x1)/(x1*x2 - x0^2), x0 -> 0]

Out[21]= 0

In[22]:= Limit[Limit[x0*(x2 - x1)/(x1*x2 - x0^2), x1 -> Infinity],
 x2 -> Infinity]

Out[22]= 0

In[23]:= Limit[Limit[x0*(x2 - x1)/(x1*x2 - x0^2), x2 -> Infinity],
 x1 -> Infinity]

Out[23]= 0

In[24]:= Limit[Limit[x0*(x2 - x1)/(x1*x2 - x0^2), x1 -> 0],
 x2 -> Infinity]

Out[24]= -\[Infinity]/x0

In[25]:= Limit[Limit[x0*(x2 - x1)/(x1*x2 - x0^2), x1 -> Infinity],
 x2 -> 0]

Out[25]= x0 (-\[Infinity])

In[26]:= Limit[Limit[x0*(x2 - x1)/(x1*x2 - x0^2), x1 -> 0], x2 -> 0]

Out[26]= 0

In[27]:= Limit[Limit[x0*(x2 - x1)/(x1*x2 - x0^2), x2 -> 0], x1 -> 0]

Out[27]= 0
Title: Re: EM Drive Developments - related to space flight applications - Thread 2
Post by: deltaMass on 05/14/2015 03:28 am
You missed my post above. In order for the group velocity in the waveguide to never exceed c (and it never does!) it follows from eqn(5) that all lambda's in the waveguide are greater than the lambda in free space.
Therefore 1 - (lambda02/(lambda1*lambda2)) is always positive definite.

Therefore blow-up is impossible
Title: Re: EM Drive Developments - related to space flight applications - Thread 2
Post by: Rodal on 05/14/2015 03:37 am
You missed my post above. In order for the group velocity in the waveguide to never exceed c (and it never does!) it follows from eqn(5) that all lambda's in the waveguide are greater than the lambda in free space.
Therefore 1 - (lambda02/(lambda1*lambda2)) is always positive definite.

Therefore blow-up is impossible
I agree with you that the lambda in free space should be smaller than in the waveguide in principle. 

I will carefully double check tomorrow his definitions and my derivation
Title: Re: EM Drive Developments - related to space flight applications - Thread 2
Post by: TheTraveller on 05/14/2015 03:52 am
You missed my post above. In order for the group velocity in the waveguide to never exceed c (and it never does!) it follows from eqn(5) that all lambda's in the waveguide are greater than the lambda in free space.
Therefore 1 - (lambda02/(lambda1*lambda2)) is always positive definite.

Therefore blow-up is impossible
I agree with you in principle.  I have to double check where he puts the cutoff wavelength...

All wavelengths inside the conic cavity are bigger than outside and the resultant group velocities are less than the velocity outside.

The guide wavelength and resultant group velocity constantly varies, driven by the constantly varying diameter of the conic section the wave is passing through.

Also the edges of the wave fronts are at right angles to the cone sides due to being spherical wave fronts as if they originated from and are returning to the vertex of the cone.

If however the conic cavity end plates are flat and not spherically matching the spherical wave fronts bouncing off them, well you may be pushing s##t up hill before things are working well inside the cavity.

I'm actually amazed a flat ended conic cavity can generate thrust.
Title: Re: EM Drive Developments - related to space flight applications - Thread 2
Post by: Rodal on 05/14/2015 04:05 am
You missed my post above. In order for the group velocity in the waveguide to never exceed c (and it never does!) it follows from eqn(5) that all lambda's in the waveguide are greater than the lambda in free space.
Therefore 1 - (lambda02/(lambda1*lambda2)) is always positive definite.

Therefore blow-up is impossible
I agree with you in principle.  I have to double check where he puts the cutoff wavelength...



All wavelengths inside the conic cavity are bigger than outside and the resultant group velocities are less than the velocity outside.

The guide wavelength and resultant group velocity constantly varies, driven by the constantly varying diameter of the conic section the wave is passing through.

Also the edges of the wave fronts are at right angles to the cone sides due to being spherical wave fronts as if they originated from and are returning to the vertex of the cone.

If however the conic cavity end plates are flat and not spherically matching the spherical wave fronts bouncing off them, well you may be pushing s##t up hill before things are working well inside the cavity.

So Shawyer does not have a cut-off wavelength appearing anywhere on his Design Factor ?

If so, the Design Factor should be trivial, much easier to calculate...
Title: Re: EM Drive Developments - related to space flight applications - Thread 2
Post by: TheTraveller on 05/14/2015 04:14 am
You missed my post above. In order for the group velocity in the waveguide to never exceed c (and it never does!) it follows from eqn(5) that all lambda's in the waveguide are greater than the lambda in free space.
Therefore 1 - (lambda02/(lambda1*lambda2)) is always positive definite.

Therefore blow-up is impossible
I agree with you in principle.  I have to double check where he puts the cutoff wavelength...



All wavelengths inside the conic cavity are bigger than outside and the resultant group velocities are less than the velocity outside.

The guide wavelength and resultant group velocity constantly varies, driven by the constantly varying diameter of the conic section the wave is passing through.

Also the edges of the wave fronts are at right angles to the cone sides due to being spherical wave fronts as if they originated from and are returning to the vertex of the cone.

If however the conic cavity end plates are flat and not spherically matching the spherical wave fronts bouncing off them, well you may be pushing s##t up hill before things are working well inside the cavity.

So Shawyer does not have a cut-off wavelength appearing anywhere on his Design Factor ?

He uses what the industry uses. Guide wavelength as in the attached. It is related to cutoff wavelength as per the attached equation
Title: Re: EM Drive Developments - related to space flight applications - Thread 2
Post by: deltaMass on 05/14/2015 04:16 am
Df = 0 when lambda1 = lambda2
Df = 1 when min(lambda1, lambda2) = lambda0  (free space condition)

So it looks like the right Df
Title: Re: EM Drive Developments - related to space flight applications - Thread 2
Post by: Rodal on 05/14/2015 04:17 am
You missed my post above. In order for the group velocity in the waveguide to never exceed c (and it never does!) it follows from eqn(5) that all lambda's in the waveguide are greater than the lambda in free space.
Therefore 1 - (lambda02/(lambda1*lambda2)) is always positive definite.

Therefore blow-up is impossible

Back to basics.  How does Shawyer define

lambda0 ? Does he define  lambda0 = loFreeSpaceWavelength = c/rfFrequency  ?

How does Shawyer define lambda 1 and lambda 2 ?

He does define lambda 1 not equal to lambda2, so they are different, we know that
Title: Re: EM Drive Developments - related to space flight applications - Thread 2
Post by: Rodal on 05/14/2015 04:21 am
You missed my post above. In order for the group velocity in the waveguide to never exceed c (and it never does!) it follows from eqn(5) that all lambda's in the waveguide are greater than the lambda in free space.
Therefore 1 - (lambda02/(lambda1*lambda2)) is always positive definite.

Therefore blow-up is impossible
I agree with you in principle.  I have to double check where he puts the cutoff wavelength...



All wavelengths inside the conic cavity are bigger than outside and the resultant group velocities are less than the velocity outside.

The guide wavelength and resultant group velocity constantly varies, driven by the constantly varying diameter of the conic section the wave is passing through.

Also the edges of the wave fronts are at right angles to the cone sides due to being spherical wave fronts as if they originated from and are returning to the vertex of the cone.

If however the conic cavity end plates are flat and not spherically matching the spherical wave fronts bouncing off them, well you may be pushing s##t up hill before things are working well inside the cavity.

So Shawyer does not have a cut-off wavelength appearing anywhere on his Design Factor ?

He uses what the industry uses. Guide wavelength as in the attached. It is related to cutoff wavelength as per the attached equation

No, he doesn't use the standard definition because according to that formula there is only one waveguide wavelength and he is defining two of them.

Same question that deltaMass was asking

How does Shawyer define lambdag 1 and lambdag 2 ?

He does define lambdag1 not equal to lambdag2, so they are different, we know that

what is the difference?

In the formula you show the only things that appear are the free space wavelength and the cut off wavelength

(http://forum.nasaspaceflight.com/index.php?action=dlattach;topic=36313.0;attach=829971;image)

But there is only one free space wavelength

There is only one cutoff wavelength

SO WHAT IS THE DIFFERENCE between lambdag1 and lambdag2
Title: Re: EM Drive Developments - related to space flight applications - Thread 2
Post by: TheTraveller on 05/14/2015 04:24 am
Df = 0 when lambda1 = lambda2
Df = 1 when min(lambda1, lambda2) = lambda0  (free space condition)

So it looks like the right Df

Shawyers definitions for Df = 0 and Df = 1 are attached.

Your Df = 0 assumption matches Shawyer but for Df = 1, small end plate lambda2 must be at cutoff and big end plate lambda1 close to lambda0.
Title: Re: EM Drive Developments - related to space flight applications - Thread 2
Post by: deltaMass on 05/14/2015 04:25 am
Rodal, you seem confused. Lambda0 is the free space wavelength = c/f. Equation (5) tells you the rest.
Title: Re: EM Drive Developments - related to space flight applications - Thread 2
Post by: deltaMass on 05/14/2015 04:26 am
The Traveller: I know, and what I said doesn't contradict you, except that cut-off isn't necessary.
Title: Re: EM Drive Developments - related to space flight applications - Thread 2
Post by: Rodal on 05/14/2015 04:28 am
Rodal, you seem confused. Lambda0 is the free space wavelength = c/f. Equation (5) tells you the rest.

that's the same definition of free space wavelength I gave you

What is  lambdag 1 and lambdag 2 ?

Please write them down in your message

Thanks
Title: Re: EM Drive Developments - related to space flight applications - Thread 2
Post by: deltaMass on 05/14/2015 04:32 am
They are what TheTraveller described and what's in eqn (5). To whit:
vgN / c  =  lambda0 /  lambdagN
for N=1,2,3,4,5,6...whatever

...assuming the relative permeability and relative permittivity both =1
Title: Re: EM Drive Developments - related to space flight applications - Thread 2
Post by: TheTraveller on 05/14/2015 04:34 am
You missed my post above. In order for the group velocity in the waveguide to never exceed c (and it never does!) it follows from eqn(5) that all lambda's in the waveguide are greater than the lambda in free space.
Therefore 1 - (lambda02/(lambda1*lambda2)) is always positive definite.

Therefore blow-up is impossible
I agree with you in principle.  I have to double check where he puts the cutoff wavelength...



All wavelengths inside the conic cavity are bigger than outside and the resultant group velocities are less than the velocity outside.

The guide wavelength and resultant group velocity constantly varies, driven by the constantly varying diameter of the conic section the wave is passing through.

Also the edges of the wave fronts are at right angles to the cone sides due to being spherical wave fronts as if they originated from and are returning to the vertex of the cone.

If however the conic cavity end plates are flat and not spherically matching the spherical wave fronts bouncing off them, well you may be pushing s##t up hill before things are working well inside the cavity.

So Shawyer does not have a cut-off wavelength appearing anywhere on his Design Factor ?

He uses what the industry uses. Guide wavelength as in the attached. It is related to cutoff wavelength as per the attached equation

No, he doesn't use the standard definition because according to that formula there is only one waveguide wavelength and he is defining two of them.

Same question that deltaMass was asking

How does Shawyer define lambdag 1 and lambdag 2 ?

He does define lambdag1 not equal to lambdag2, so they are different, we know that

what is the difference?

In the formula you show the only things that appear are the free space wavelength and the cut off wavelength

But there is only one free space wavelength

There is only one cutoff wavelength

SO WHAT IS THE DIFFERENCE between lambdag1 and lambdag2

The cutoff wavelength is defined for EACH end, based on it's diameter * 1.71. See attachment 2.

Lambda1 = guide wavelength equation from attachment 1 below using big end cutoff wavelength.

Lambda2 = guide wavelength equation from attachment 1 below using small end cutoff wavelength.
Title: Re: EM Drive Developments - related to space flight applications - Thread 2
Post by: Rodal on 05/14/2015 04:37 am
....
The cutoff wavelength is defined for EACH end, based on it's diameter * 1.71. See attachment 2.

Lambda1 = guide wavelength equation from attachment 1 below using big end cutoff wavelength.

Lambda2 = guide wavelength equation from attachment 1 below using small end cutoff wavelength.

OK

Thanks for the answer

Very clear answer

That's NOT what deltaMass had written  (there is no N=1,2,3,4, in equation 5)

That's exactly the same factor of 1.7    I derived, based on TE110

Now I'm going to sleep.
Title: Re: EM Drive Developments - related to space flight applications - Thread 2
Post by: deltaMass on 05/14/2015 04:39 am
Notice that the game may change if sqrt(mur epsr) < 1. Usually it's ~=1, or a bit more , and that's the assumption I'm making with my inequalities games
Title: Re: EM Drive Developments - related to space flight applications - Thread 2
Post by: TheTraveller on 05/14/2015 04:40 am
The Traveller: I know, and what I said doesn't contradict you, except that cut-off isn't necessary.

Shawyer always says the small end cutoff wavelength should be just above small end cutoff for best thrust generation. The higher the Df, the higher the resultant thrust from T = 2 Df Po Q / c

So the small end operating at just above cutoff is important to achieve a Df close to 1.
Title: Re: EM Drive Developments - related to space flight applications - Thread 2
Post by: deltaMass on 05/14/2015 04:40 am
I'm sorry, but I can't make it any simpler.
N=1,2 is the case when you evaluate something at either end.
But the equation holds for any position in between.
Title: Re: EM Drive Developments - related to space flight applications - Thread 2
Post by: TheTraveller on 05/14/2015 04:41 am
....
The cutoff wavelength is defined for EACH end, based on it's diameter * 1.71. See attachment 2.

Lambda1 = guide wavelength equation from attachment 1 below using big end cutoff wavelength.

Lambda2 = guide wavelength equation from attachment 1 below using small end cutoff wavelength.

OK

Thanks for the answer

Very clear answer

That's NOT what deltaMass had written  (there is no N=1,2,3,4, in equation 5)

That's exactly the same factor of 1.7    I derived, based on TE110

Now I'm going to sleep.

Did notice you came up with 1.71.
Nice work.
Now we both go to sleep.
Title: Re: EM Drive Developments - related to space flight applications - Thread 2
Post by: deltaMass on 05/14/2015 04:55 am
The Traveller: I know, and what I said doesn't contradict you, except that cut-off isn't necessary.

Shawyer always says the small end cutoff wavelength should be just above small end cutoff for best thrust generation. The higher the Df, the higher the resultant thrust from T = 2 Df Po Q / c

So the small end operating at just above cutoff is important to achieve a Df close to 1.

Not if you take that Df at face value, no. Let's do some high school algebra. I use x to mean lambda, and I can't be bothered using suffices throughout.

We start with the relation we know to be true: x2 >= x1 >= x0.
We now quantify this with these definitions:
x2 := a*x1  (a >= 1)
x1 := b*x0 (b >= 1)
The original Df expression:
Df = x0*(x2 - x1) / (x1*x2 - x02)
becomes, using the above definitions:

Df = b*(a - 1) / (a*b2 - 1)

What's Df when x1 = x0?
This is tantamount to setting b=1. Doing that, we get
Df = (a-1) / (a-1) = 1.   QED.
i.e. it doesn't matter what the value of 'a' might be; i.e. it doesn't matter about cutoff.
Title: Re: EM Drive Developments - related to space flight applications - Thread 2
Post by: deltaMass on 05/14/2015 05:25 am
I ought to mention that Shawyer does not specify how to compute vg or lambdag. What he does do is specify the functional relation between them - i.e. v(lambda) or lambda(v) - and that's all.

Anyone know how to compute vg without using  lambdag?
Anyone know how to compute lambdag without using  vg?
Title: Re: EM Drive Developments - related to space flight applications - Thread 2
Post by: TheTraveller on 05/14/2015 05:33 am
The Traveller: I know, and what I said doesn't contradict you, except that cut-off isn't necessary.

Shawyer always says the small end cutoff wavelength should be just above small end cutoff for best thrust generation. The higher the Df, the higher the resultant thrust from T = 2 Df Po Q / c

So the small end operating at just above cutoff is important to achieve a Df close to 1.

Not if you take that Df at face value, no. Let's do some high school algebra. I use x to mean lambda, and I can't be bothered using suffices throughout.

We start with the relation we know to be true: x2 >= x1 >= x0.
We now quantify this with these definitions:
x2 := a*x1  (a >= 1)
x1 := b*x0 (b >= 1)
The original Df expression:
Df = x0*(x2 - x1) / (x1*x2 - x02)
becomes, using the above definitions:

Df = b*(a - 1) / (a*b2 - 1)

What's Df when x1 = x0?
This is tantamount to setting b=1. Doing that, we get
Df = (a-1) / (a-1) = 1.   QED.
i.e. it doesn't matter what the value of 'a' might be; i.e. it doesn't matter about cutoff.

Interesting.

I feel what Shawyer is saying is to start a cavity design at the small end. Once you get the small end cut off and guide wavelengths matched to the desired driving Rf wavelength as 1/2x or 1x or 2x, you then switch to the big end and again set the big end diameter to match the driving wavelength. Then adjust length to match the small and big end wavelengths as multiples or subs of each other.

Bit too late for my fuzzy brain. Can see how all this interaction can be folded into an interactive cavity design spreadsheet.
Title: Re: EM Drive Developments - related to space flight applications - Thread 2
Post by: TheTraveller on 05/14/2015 05:46 am
I ought to mention that Shawyer does not specify how to compute vg or lambdag. What he does do is specify the functional relation between them - i.e. v(lambda) or lambda(v) - and that's all.

Anyone know how to compute vg without using  lambdag?
Anyone know how to compute lambdag without using  vg?

Both equations can be found on the net. Standard microwave stuff. Have posted them earlier:
http://www.microwaves101.com/encyclopedias/waveguide-mathematics

Lambda g1 or g2 is guide wavelength for big end (1) or small end (2).

vg1 or vg2 or group velocity 1 or 2 is derived from / determined by guide wavelength for lambda g1/g2.

Shawyer calls lambda g1 and lambda g2 "guide wavelengths" as they are microwave industry definations that I found in less than 5 minutes of Googling.

As far as I can see on microwave industry web sites, vg or group velocity is calculated based on guide wavelength or Shawyers lambda g1 or g2.
Title: Re: EM Drive Developments - related to space flight applications - Thread 2
Post by: deltaMass on 05/14/2015 06:17 am
Shawyer's Df says that you want the big end lambda (the smallest lambda ) as close to free space lambda as possible. Of course, he doesn't recommend opening up the big end.

Plays havoc with the Q :)
Title: Re: EM Drive Developments - related to space flight applications - Thread 2
Post by: zellerium on 05/14/2015 06:59 am
I've been trying to wrap my brain around why a difference in phase results in a better thrust.  Also, why couldn't EW obtain a thrust without a dielectric?

I think we can all agree that in order for a net thrust, the momentum delivered to the larger end plate is smaller than that delivered to the smaller end plate. So where did the momentum go?

Can momentum be delivered and removed from an orbiting electron?

Take a simple two dimensional case with two atoms, one on the small end, one on the large end, each with their own electron orbiting at a the same angular frequency. If a force is applied to both of them, one in the direction of revolution and the other opposite, one of the forces would slow down the electron and the other would speed it up. The sped up electron requires a larger force to keep it tied to the nucleus and we have a net thrust.

Perhaps this could help explain a couple things:

The dielectric is composed of different elements, thus the electrons are orbiting at a different angular velocity. Using a constant frequency with different elements gives a certain degree of difference in the phase at which momentum is delivered to the electrons.

Shawyer observed more losses with a dielectric because a magnetron outputs a signal at many phases and somehow 'matches' the orbital tendency of the electrons.
I imagine the magnetic component of the wave could be contributing to an alignment of electrons which could amplify this miniscule effect.

Any thoughts?
Title: Re: EM Drive Developments - related to space flight applications - Thread 2
Post by: deltaMass on 05/14/2015 07:22 am
If this is supposed to be a phenomenon based on waves in a can, then surely it applies to any waves. Perhaps someone should look at plain old water waves in a can.

Cum aquis ad sideros
Title: Re: EM Drive Developments - related to space flight applications - Thread 2
Post by: deltaMass on 05/14/2015 10:40 am
Quote from: Rodal
Wrong.  (You need AND)
Thanks! - you are quite correct. But no need for Mathematica.
Title: Re: EM Drive Developments - related to space flight applications - Thread 2
Post by: deuteragenie on 05/14/2015 10:47 am
Any resemblance with existing or future device(s) is purely coincidental.
Made with MEEP.  Does not use the correct frequencies, material, etc. but looks cute.

Is that a 2-Dimensional model with MEEP?
Maxwell's equations in a flat 2-D surface?
modeling the truncated cone as a FLAT trapezium ?

Is the magnetic field (for TM modes)  a point scalar (only able to have + or - sign but the direction is always perpendicular to the surface) instead of being a vector in the azimuthal (circumferential) direction  ?

What are we seeing out of the EM Drive? evanescent wave field?

If the answers are yes, do you have enough memory to run a 3D model instead?

Thanks

I tried my best to warn that this was art and not science.
Anyway, the MEEP ctl file is attached.

Modelling the truncated cone as a FLAT trapezium ? Rather modelling a slice of of the 3D cone on a 2D lattice I guess.
Maxwell's equations in a flat 2-D surface? That is my understanding on how MEEP works.
Is the magnetic field (for TM modes)  a point scalar (only able to have + or - sign but the direction is always perpendicular to the surface) instead of being a vector in the azimuthal (circumferential) direction  ? Ez field is shown.  Producing Hz is straightforward.
What are we seeing out of the EM Drive? evanescent wave field? You'll understand when you look at the attached file :)
It took about 2 minutes to generate in 2D and should be fairly easy to move it to 3D, although I am not too sure which tools are available to visualize the 3D output or to slice it.
Title: Re: EM Drive Developments - related to space flight applications - Thread 2
Post by: Mulletron on 05/14/2015 10:51 am
Published 12 May 2015
This represents the current state of the art.

http://iopscience.iop.org/0953-8984/27/21/210301/article
http://iopscience.iop.org/0953-8984/27/21 (TOC)

I'm pretty excited about what could come out of applying these materials too...maybe a way forward:
http://www.lap.physik.uni-erlangen.de/lap/?page=research_krstic_chiral&language=en

Title: Re: EM Drive Developments - related to space flight applications - Thread 2
Post by: deuteragenie on 05/14/2015 10:59 am
Any resemblance with existing or future device(s) is purely coincidental.
Made with MEEP.  Does not use the correct frequencies, material, etc. but looks cute.

You might want to increase your skin thickness or Meep resolution. Looks kind of like a Gaussian source with some of the shorter wavelengths bouncing around and longer wavelengths stepping over the boundary (numerically)?

Yes, I noticed that and tried to improve the resolution with not much success so far.  I'll keep playing.

If you are using perfect metal for the skin, then the thickness won't change the result but will help to avoid the model numerically spanning the skin. If you are modelling copper in the GHz range, please tell me about your model as I have struggled for 6 months trying to find a Drude model for copper at 2 GHz.

I tried to change the boundary to copper and the inside to air and then to vacuum, with the definition for copper provided here:
http://www.fzu.cz/~dominecf/meep/data/meep-metals.pdf

Unfortunately, I was not able to produce anything appealing so far with that approach (either I get completely white or dark results). It also did not work when I tried to increase the frequency too much. I need to study physics first then I'll be able to understand what I am supposed to enter as parameters...

Attached is the .ctl file (renamed to .txt because the forum does not allow .ctl to be uploaded).
Title: Re: EM Drive Developments - related to space flight applications - Thread 2
Post by: TheTraveller on 05/14/2015 11:55 am
Have modified my Shawyer Df calculator and best Df scanner as per the derived Shawyer Df equation, using cutoff wavelength and guide wavelength as per microwave industry supplied equations. I assume Shawyer did not supply these equations in his papers as they are equations that should be known to microwave industry individuals skilled in the art. Anyway they are now in the public record.

The scanner still sweeps the frequency range 0Hz to 10GHz but reports the frequency that generates a Df as close to 1 as possible but not over.

The attached results are very interesting as the frequency needed to get the Df to just below 1 is very close to the Rf driving frequency used to generate Lambda0 or free wavelength in the selected medium.

While I'm still testing the spreadsheet, which meets both of Shawyers boundary conditions, the results for my Flight Thruster design are looking to be very close to what I could build. Bit of dimension tweaking should get the Df 1 frequency to the 3.85GHz Shawyer used.

Will post the spreadsheet after a bit more testing.
Title: Re: EM Drive Developments - related to space flight applications - Thread 2
Post by: Rodal on 05/14/2015 12:18 pm
Have modified my Shawyer Df calculator and best Df scanner as per the derived Shawyer Df equation, using cutoff wavelength and guide wavelength as per microwave industry supplied equations. I assume Shawyer did not supply these equations in his papers as they are equations that should be known to microwave industry individuals skilled in the art. Anyway they are now in the public record.

The scanner still sweeps the frequency range 0Hz to 10GHz but reports the frequency that generates a Df as close to 1 as possible but not over.

The attached results are very interesting as the frequency needed to get the Df to just below 1 is very close to the Rf driving frequency used to generate Lambda0 or free wavelength in the selected medium.

While I'm still testing the spreadsheet, which meets both of Shawyers boundary conditions, the results for my Flight Thruster design are looking to be very close to what I could build. Bit of dimension tweaking should get the Df 1 frequency to the 3.85GHz Shawyer used.

Will post the spreadsheet after a bit more testing.

This is what I get:

DESIGN FACTOR WITH CUT-OFF FREQUENCY CHOSEN AS TE110

Defining the following variables:

bD = big end diameter (m)
sD= small end diameter (m)
f = applied frequency (Hz)
cutOffW1= cutOffWavelength1 (m)
cutOffW2= cutOffWavelength2 (m)
assuming the cutOffWavelengths in the truncated cone to be the lowest possible mode in a cylindrical cavity, TE110, the constant "cst" is given by Pi divided by the 1,1 zero of the derivative of the cylindrical Bessel function:
cst= Pi /X'11
     =Pi/1.84118378134065
     =1.7062895542683174
cM = light speed in selected medium (m/s)
     = 299705000 (m/s) (speed of light in air)
     = 299792458 (m/s) (speed of light in vacuum)



lambda0 = cM /f;

cutOffW1 = bD*cst;

cutOffW2 = sD*cst;

lGuide1 = lambda0/Sqrt[1 - (lambda0/cutOffW1)^2];

lGuide2 = lambda0/Sqrt[1 - (lambda0/cutOffW2)^2];

soCorrectionFactor = 1/(1 - ((lambda0^2)/(lGuide1*lGuide2)));

multiplier = (lambda0/lGuide1) - (lambda0/lGuide2);

designFactor = soCorrectionFactor*multiplier;

designFactor =
           (bD^2 - sD^2)/( (bD^2)*Sqrt[1 - (cM/(bD*cst*f))^2] + (sD ^2)*Sqrt[1 - (cM/(cst*f*sD))^2] )

The Design Factor has a singularity (due to the cut-off frequencies) starting at the cut-off frequency associated with the small diameter

Singularity frequency= cM/(cst*sD)                        

When I input the same values you have, I get practically (*) the same output for the Design Factor as you do

EXAMPLE Ref 2

Using speed of light in air

INPUT:

bD = 0.2797; sD = 0.1588; f = 2.45*10^9; cM = 299705000; cst = 1.7062895542683174;

OUTPUT:

Design Factor = 0.540306

singularity frequency = 1.10609*10^9 Hz due to the cut-off frequency associated with the small diameter



(*) small difference is due to the fact that I use 1.7062895542683174 instead of the approximate value of 1.71 you use
Title: Re: EM Drive Developments - related to space flight applications - Thread 2
Post by: SeeShells on 05/14/2015 12:26 pm
Published 12 May 2015
This represents the current state of the art.

http://iopscience.iop.org/0953-8984/27/21/210301/article
http://iopscience.iop.org/0953-8984/27/21 (TOC)

I'm pretty excited about what could come out of applying these materials too...maybe a way forward:
http://www.lap.physik.uni-erlangen.de/lap/?page=research_krstic_chiral&language=en

Excited? I'm more than excited.  I think I've reached my limits of current understanding and applying these effects to the EM device. I need more time to read and digest because every article seems to have a little something that fits in the overall scheme of why. Mullutron I think you have a tiger by the tale here.
Like this one.
Transfer of linear momentum from the quantum vacuum to a magnetochiral molecule
M Donaire1, B A van Tiggelen2 and G L J A Rikken3
Show affiliations

M Donaire et al 2015 J. Phys.: Condens. Matter 27 214002. doi:10.1088/0953-8984/27/21/214002
Received 14 April 2014, accepted for publication 2 July 2014. Published 12 May 2015.
© 2015 IOP Publishing Ltd

Abstract
In a recent publication [1] we have shown using a QED approach that, in the presence of a magnetic field, the quantum vacuum coupled to a chiral molecule provides a kinetic momentum directed along the magnetic field. Here we explain the physical mechanisms which operate in the transfer of momentum from the vacuum to the molecule. We show that the variation of the molecular kinetic energy originates from the magnetic energy associated with the vacuum correction to the magnetization of the molecule. We carry out a semiclassical calculation of the vacuum momentum and compare the result with the QED calculation.
Title: Re: EM Drive Developments - related to space flight applications - Thread 2
Post by: Rodal on 05/14/2015 12:40 pm
Have modified my Shawyer Df calculator and best Df scanner as per the derived Shawyer Df equation, using cutoff wavelength and guide wavelength as per microwave industry supplied equations. I assume Shawyer did not supply these equations in his papers as they are equations that should be known to microwave industry individuals skilled in the art. Anyway they are now in the public record.

The scanner still sweeps the frequency range 0Hz to 10GHz but reports the frequency that generates a Df as close to 1 as possible but not over.

The attached results are very interesting as the frequency needed to get the Df to just below 1 is very close to the Rf driving frequency used to generate Lambda0 or free wavelength in the selected medium.

While I'm still testing the spreadsheet, which meets both of Shawyers boundary conditions, the results for my Flight Thruster design are looking to be very close to what I could build. Bit of dimension tweaking should get the Df 1 frequency to the 3.85GHz Shawyer used.

Will post the spreadsheet after a bit more testing.

designFactor =
           (bD^2 - sD^2)/( (bD^2)*Sqrt[1 - (cM/(bD*cst*f))^2)] + (sD ^2)*Sqrt[1 - (cM/(cst*f*sD))^2)] )

The Design Factor has a singularity (due to the cut-off frequencies) starting at the cut-off frequency associated with the small diameter

Singularity frequency= cM/(cst*sD)     

LIMITS

1) Limit[DesignFactor, sD -> bD] = 0

2) Limit[DesignFactor, bD -> Infinity] =1

3) Limit[DesignFactor, sD -> 0] = 1/Sqrt[1 - (cM/(bD * cst * f ))^2]

4) Limit[DesignFactor, f -> Infinity]  = (bD^2 - sD^2)/(bD^2 + sD^2)

5) Limit[DesignFactor, f -> 0] =0
Title: Re: EM Drive Developments - related to space flight applications - Thread 2
Post by: SeeShells on 05/14/2015 12:48 pm
Published 12 May 2015
This represents the current state of the art.

http://iopscience.iop.org/0953-8984/27/21/210301/article
http://iopscience.iop.org/0953-8984/27/21 (TOC)

I'm pretty excited about what could come out of applying these materials too...maybe a way forward:
http://www.lap.physik.uni-erlangen.de/lap/?page=research_krstic_chiral&language=en

Excited? I'm more than excited.  I think I've reached my limits of current understanding and applying these effects to the EM device. I need more time to read and digest because every article seems to have a little something that fits in the overall scheme of why. Mullutron I think you have a tiger by the tale here.
Like this one.
Transfer of linear momentum from the quantum vacuum to a magnetochiral molecule
M Donaire1, B A van Tiggelen2 and G L J A Rikken3
Show affiliations

M Donaire et al 2015 J. Phys.: Condens. Matter 27 214002. doi:10.1088/0953-8984/27/21/214002
Received 14 April 2014, accepted for publication 2 July 2014. Published 12 May 2015.
© 2015 IOP Publishing Ltd

Abstract
In a recent publication [1] we have shown using a QED approach that, in the presence of a magnetic field, the quantum vacuum coupled to a chiral molecule provides a kinetic momentum directed along the magnetic field. Here we explain the physical mechanisms which operate in the transfer of momentum from the vacuum to the molecule. We show that the variation of the molecular kinetic energy originates from the magnetic energy associated with the vacuum correction to the magnetization of the molecule. We carry out a semiclassical calculation of the vacuum momentum and compare the result with the QED calculation.
another good publication.
http://arxiv.org/pdf/1404.5990v2.pdf
Title: Re: EM Drive Developments - related to space flight applications - Thread 2
Post by: Notsosureofit on 05/14/2015 12:54 pm
Post-sleep (?) rambling thoughts on entropy:

It has always bothered me that (to me) it sounds like Shawyer's theory should make the frustrum go in the other direction, not that I spend any time thinking about it, but anyway;

Very roughly, it seemed to me like entropy change of a photon gas would give an isotropic force from the surface integral of the entropy change times the temperature divided by the work that was done (by the pressure expanding the surface)

But what if the temperature was inside the integral.  A symmetrical cavity would still have no net force as long as the surface temperature change was symmetrical.

Of course, EW and @RODAL's calculations show the big end hotter.

But, if the radiation temperature of the photons is much LESS than the ambient, raising the big end temperature gives a NEGATIVE force in that direction when the entropy increases.   Sort of in keeping w/ the 1p on the big end 2p on the small end thoughts.

These are just sleep (or non-sleep) induced images so use a 10lb salt bag.  (havn't found my old thermo text)  Including the surface integral of entropy change of photons opens a BIG can of worms, but it might explain EW not getting thrust w/o dielectric if the cavity surface has uneven characteristics (ie circuit boards on the ends, ferinstance)

As stated above, 10lb bag of salt.

Title: Re: EM Drive Developments - related to space flight applications - Thread 2
Post by: Rodal on 05/14/2015 12:58 pm
... I assume Shawyer did not supply these equations in his papers as they are equations that should be known to microwave industry individuals skilled in the art. Anyway they are now in the public record....
I think you have done an outstanding job in discovering what Shawyer may have meant.

You are too humble  :) in saying that these equations may be known in the art.

No, Shawyer's equations are NOT known to microwave industry individuals skilled in the art, because the overwhelming majority of microwave industry professionals do NOT use truncated cone cavities. 

Shawyer makes a particular choice for lambdag1 and lambdag2 which is not found in the literature.
He essentially models the truncated cone as two disjointed different cylinders: a cylinder corresponding to the big end and a cylinder corresponding to the small end.  My intuition would have been to model the truncated cone instead as a single cylinder having a diameter equal to the Geometric Mean diameter, as done for example by @Notsosureofit for his model. (That's what I thought Shawyer had done and that's one reason why my interpretation of Shawyer's Design Factor was apparently not what he meant).

Shawyer's choice of modeling the truncated cone as two different cylinders with lambdag1 and lambdag2 may be a stroke of intuitive genius, or it may be flawed, that remains to be shown by your (and others) replications.
But his choice of lambdag1 and lambdag2 is anything but obvious to me (*), it is never made explicit in his writings, and its validity remains to be proven  :)

____________
(*) Modeling a truncated cone as two disjointed cylinders is not only not obvious but it presents big theoretical problems in the limit as the small diameter goes to zero as essentially Shawyer models the pointy cone as a cylinder and a line.  Again it maybe a great engineering approximation (or not), it remains to be proven.
Title: Re: EM Drive Developments - related to space flight applications - Thread 2
Post by: Mulletron on 05/14/2015 01:03 pm
Today i did the first test with the Emdrive (microwave oven magnetron and cooper frustum)
The setup (magnetron and frusum) was suspended in a pendulum.
I applied power for 40 Seconds with no visible thrust. Tomorrow will will try again with the magnetron on the small side. You have any suggestion for what should be the distance from the small side?
After this i will adjust the power to the filament of magnetron and the frequency.
To fine adjust the frequency i thought i can put 2 coils over the magnetron magnets to modify the magnetic field.
My website;
http://www.masinaelectrica.com/emdrive-independent-test/

https://www.youtube.com/watch?v=2-cgVoLUJ8w

Hey great job BTW. While I was out driving I was thinking of you and had a couple ideas. If you're willing would you/could you try:

1) Set the whole apparatus in motion, like a swing or a pendulum and measure the period while it is turned off. Then measure the period again while turned on. See if the period changes.
2) If you have a way of measuring this, could you see if it is any easier or harder to displace the frustum from when it is turned off and then turned on. Any difference?

Keep it up! We need folks like you.
Title: Re: EM Drive Developments - related to space flight applications - Thread 2
Post by: Rodal on 05/14/2015 01:07 pm
...

Of course, EW and @RODAL's calculations show the big end hotter.

...

Great post  !!!

My calculations only assume Maxwell's linear equations.  EW has a dielectric inside and they measured very small forces and very low temperatures.

Keep thinking and challenging all assumptions:

Because the Chinese (Prof. Yang's) measurements with embedded thermocouples show the OPPOSITE of NASA Eagleworks, in agreement with your expectation: the small end is much hotter

Prof. Juan Yang's reported temperature vs. time measurements with embedded thermocouples throughout their EM Drive cavity (without a polymer dielectric insert) under atmospheric conditions, that, curiously, show the highest temperature at the center of the small base (trace #1), followed, at a significantly lower temperature by the temperature at the periphery of the big base (trace #5).

Take a look at the temperatures measured by the thermocouple Trace #1

(http://forum.nasaspaceflight.com/index.php?action=dlattach;topic=29276.0;attach=622845;image)

(http://forum.nasaspaceflight.com/index.php?action=dlattach;topic=29276.0;attach=655009;image)
Title: Re: EM Drive Developments - related to space flight applications - Thread 2
Post by: Notsosureofit on 05/14/2015 01:23 pm
...

Of course, EW and @RODAL's calculations show the big end hotter.

...

Great post  !!!

My calculations only assume Maxwell's linear equations.  EW has a dielectric inside and they measured very small forces and very low temperatures.

Keep thinking and challenging all assumptions:

Because the Chinese (Prof. Yang's) measurements with embedded thermocouples show the OPPOSITE of NASA Eagleworks, in agreement with your expectation: the small end is much hotter

Prof. Juan Yang's reported temperature vs. time measurements with embedded thermocouples throughout their EM Drive cavity (without a polymer dielectric insert) under atmospheric conditions, that, curiously, show the highest temperature at the center of the small base (trace #1), followed, at a significantly lower temperature by the temperature at the periphery of the big base (trace #5).

Take a look at the temperatures measured by the thermocouple Trace #1

(http://forum.nasaspaceflight.com/index.php?action=dlattach;topic=29276.0;attach=622845;image)

(http://forum.nasaspaceflight.com/index.php?action=dlattach;topic=29276.0;attach=655009;image)

I'm not sure what that would mean at this point.

Anyway, 2GHz is much less than the cosmic background temp. which is ~160GHz peak.

For the statistical mechanics among us, Q comes in w/ the partition function of the almost monochromatic photon spectra.
Title: Re: EM Drive Developments - related to space flight applications - Thread 2
Post by: deltaMass on 05/14/2015 01:31 pm
@Rodal: You say that you get a blow-up with your Df formula, and yet
a) you agree that both lambdag1,2 are greater than lambda0 (because vg < c)
b) you agree that, this being the case, it's mathematically impossible to get blow-up (denominator zero)

Clarify please?
Title: Re: EM Drive Developments - related to space flight applications - Thread 2
Post by: Rodal on 05/14/2015 01:41 pm
@Rodal: You say that you get a blow-up with your Df formula, and yet
a) you agree that both lambdag1,2 are greater than lambda0 (because vg < c)
b) you agree that, this being the case, it's mathematically impossible to get blow-up (denominator zero)

Clarify please?
You are thinking correctly, as a physical situation, as if lambda0 and lambdag1,2 are wavelengths occurring at a particular point in time and space. 

However, the mathematical relationship between  lambda0 and lambdag1,2, as defined above, is only physically correct for frequencies above the cutoff frequency.

For frequencies below the cutoff , the Design Factor blows up, at the value I gave in my post.

You can verify that this blow up only occurs at frequencies below the cut off .

The cutoff is chosen at the lowest natural frequency.

There are no standing waves in a cavity below the lowest natural frequency.

EDIT: More technically correct is to state that the Design Factor has a singularity: it becomes non-real (Imaginary) at that frequency.

The Design Factor has a singularity (due to the cut-off frequencies) starting at the cut-off frequency associated with the small diameter

Singularity frequency= cM/(cst*sD) 
Title: Re: EM Drive Developments - related to space flight applications - Thread 2
Post by: Chris Bergin on 05/14/2015 02:12 pm
Looks like this has all calmed down after the mass media binge.

Housekeeping plans:

We'll eventually look to start a Thread 3 - based on the timing of the next update of interest. That'll be up to the main contributors to decide that point as this thread is loooooooooooong and now coming up to 900,000 views (yikes).

Thread 3 will be constructed with an opening post that will be a quick look overview (article, previous threads, wiki and so on).
Title: Re: EM Drive Developments - related to space flight applications - Thread 2
Post by: rfcavity on 05/14/2015 02:26 pm
Have modified my Shawyer Df calculator and best Df scanner as per the derived Shawyer Df equation, using cutoff wavelength and guide wavelength as per microwave industry supplied equations. I assume Shawyer did not supply these equations in his papers as they are equations that should be known to microwave industry individuals skilled in the art. Anyway they are now in the public record.

The scanner still sweeps the frequency range 0Hz to 10GHz but reports the frequency that generates a Df as close to 1 as possible but not over.

The attached results are very interesting as the frequency needed to get the Df to just below 1 is very close to the Rf driving frequency used to generate Lambda0 or free wavelength in the selected medium.

While I'm still testing the spreadsheet, which meets both of Shawyers boundary conditions, the results for my Flight Thruster design are looking to be very close to what I could build. Bit of dimension tweaking should get the Df 1 frequency to the 3.85GHz Shawyer used.

Will post the spreadsheet after a bit more testing.

I've built and tested many microwave cavities over many years.

You're guided wavelength equation is wrong, because this is for a rectangular wave guide (i.e., not even a rectangular cavity)

You need to derive mode of frequency yourself (unless there is a paper somewhere) for a circular tapered cavity. There is no other way around it. I would start with Balanis - Advanced Engineering Electromagnetics as he derives a few examples for other topologies. Right now everything you are doing is wrong because you don't understand the physics. I would study that book from front to cover if I were you.


Also, to the guy operating the microwave magnetron outside of the microwave: STOP
At best you are violating the laws of your local government's regulatory committee for the electromagnetic spectrum. At worst you will damage your body. At this frequency, the damage is somewhat insidious. Due to low water content of your skin, you don't feel the heat, but internal nerve endings can be damaged so that chronic phantom pain can appear. Sometimes days after exposure. Please STOP otherwise you will inevitably be reported to your government.

Title: Re: EM Drive Developments - related to space flight applications - Thread 2
Post by: Rodal on 05/14/2015 02:40 pm
...

You're guided wavelength equation is wrong, because this is for a rectangular wave guide (i.e., not even a rectangular cavity)

...
You are correct: Shawyer uses a number of references for open waveguides instead of closed cavities.  For example, his main reference is Cullen's Ph.D. thesis published in the early '50s.  When reading Cullen's paper I was surprised to find out that Cullen had used an OPEN waveguide (it had a transparent glass at one end, transparent to microwaves  on purpose) to experimentally measure the pressure on the other end (where waves are reflected) of an open cylindrical waveguide.  Not a truncated cone closed cavity. 

Shawyer essentially models the closed truncated cone cavity as being two disjointed open waveguides.  Having brought this up a number of times, only results in Shawyer providing a number of references and texts that don't support his approximation.  His approximation is not obvious (at least to me), it is unsupported, and basically remains to be proven.
Title: Re: EM Drive Developments - related to space flight applications - Thread 2
Post by: Rodal on 05/14/2015 02:45 pm
...

You're guided wavelength equation is wrong, because this is for a rectangular wave guide (i.e., not even a rectangular cavity)

...
Yes, see this:  http://forum.nasaspaceflight.com/index.php?topic=36313.msg1369861#msg1369861
Title: Re: EM Drive Developments - related to space flight applications - Thread 2
Post by: deltaMass on 05/14/2015 02:49 pm
The anti-theoretical, heuristic, hands-on practical approach is to build a cavity and sweep through a likely frequency range while feeding it. Monitoring the match with a simple AC voltmeter will show dips, which will correspond to the various modes. Pick a few, use them in experiments. Viola :)
Title: Re: EM Drive Developments - related to space flight applications - Thread 2
Post by: SeeShells on 05/14/2015 03:18 pm
I've been trying to wrap my brain around why a difference in phase results in a better thrust.  Also, why couldn't EW obtain a thrust without a dielectric?

I think we can all agree that in order for a net thrust, the momentum delivered to the larger end plate is smaller than that delivered to the smaller end plate. So where did the momentum go?

Can momentum be delivered and removed from an orbiting electron?

Take a simple two dimensional case with two atoms, one on the small end, one on the large end, each with their own electron orbiting at a the same angular frequency. If a force is applied to both of them, one in the direction of revolution and the other opposite, one of the forces would slow down the electron and the other would speed it up. The sped up electron requires a larger force to keep it tied to the nucleus and we have a net thrust.

Perhaps this could help explain a couple things:

The dielectric is composed of different elements, thus the electrons are orbiting at a different angular velocity. Using a constant frequency with different elements gives a certain degree of difference in the phase at which momentum is delivered to the electrons.

Shawyer observed more losses with a dielectric because a magnetron outputs a signal at many phases and somehow 'matches' the orbital tendency of the electrons.
I imagine the magnetic component of the wave could be contributing to an alignment of electrons which could amplify this miniscule effect.

Any thoughts?
I'm digging through this paper right now and it seems to offer an answer to your question. I'll re-read it several times, as it's looking to be the Tar Baby in the Brier Patch for me.
http://arxiv.org/pdf/1011.4376.pdf
Title: Re: EM Drive Developments - related to space flight applications - Thread 2
Post by: DIYFAN on 05/14/2015 03:26 pm
... Right now everything you are doing is wrong because you don't understand the physics. ...

Given that he is at least one person among billions on this planet who is attempting a replication of a mystifying device, much of what he is doing is right. 

Also, to the guy operating the microwave magnetron outside of the microwave: STOP
At best you are violating the laws of your local government's regulatory committee for the electromagnetic spectrum. At worst you will damage your body. At this frequency, the damage is somewhat insidious. Due to low water content of your skin, you don't feel the heat, but internal nerve endings can be damaged so that chronic phantom pain can appear. Sometimes days after exposure. Please STOP otherwise you will inevitably be reported to your government.

While I agree with your safety suggestions, I think your tone and delivery are a bit over the top.  Microwaves are easily contained.
Title: Re: EM Drive Developments - related to space flight applications - Thread 2
Post by: Rodal on 05/14/2015 03:30 pm
...

I'm digging through this paper right now and it seems to offer an answer to your question. I'll re-read it several times, as it's looking to be the Tar Baby in the Brier Patch for me.
http://arxiv.org/pdf/1011.4376.pdf
I find it disheartening how much smaller is the effect explored in this "Tar Baby in the Brier Patch" paper and van Tiggelen's other papers, compared to what is claimed by the EM Drive researchers (particularly what is claimed by Shawyer and Prof. Yang regarding measured forces), and the fact that Shawyer and Prof. Yang do not use any dielectric polymer insert in their tests.
Title: Re: EM Drive Developments - related to space flight applications - Thread 2
Post by: deltaMass on 05/14/2015 03:38 pm
I had a look through the paper and am surprised the effect exists at all. Can someone explain where the opposite momentum goes?
Title: Re: EM Drive Developments - related to space flight applications - Thread 2
Post by: Rodal on 05/14/2015 03:56 pm
I had a look through the paper and am surprised the effect exists at all. Can someone explain where the opposite momentum goes?
I'm surprised that you are not asking: if indeed one can obtain this momentum (however little teeny tiny) by interacting with the QV, does that mean that one can obtain energy (however little teeny tiny) from the QV, and therefore there is a paradox ?  ;)
Title: Re: EM Drive Developments - related to space flight applications - Thread 2
Post by: TheTraveller on 05/14/2015 04:03 pm
One unanswered question I had, as I put my engineers build spreadsheet together, was what effect does cavity length have on operation as the Shawyer Df equation does not use cavity length?

Then I found the attached and it all fell into place.

Then added the ability to calc the length resonance wavelength / frequency and guess what? The Flight Thruster specs, as worked on on this thread, say the length is resonate at close to 2x the wavelength of the applied 3.85GHz Rf signal. Did a bit of length tweaking and how have the length resonate at 1/2 the applied Rf frequency or 2x the applied wavelength.

Nice when a plan comes together.
Title: Re: EM Drive Developments - related to space flight applications - Thread 2
Post by: Notsosureofit on 05/14/2015 04:17 pm
I had a look through the paper and am surprised the effect exists at all. Can someone explain where the opposite momentum goes?

Transferred to the source of the external magnetic field
Title: Re: EM Drive Developments - related to space flight applications - Thread 2
Post by: SeeShells on 05/14/2015 04:21 pm
...
I can feel your pain, understand your concern and yes it concerns me too.
I'm digging through this paper right now and it seems to offer an answer. I'll re-read it several times, as it's looking to be the Tar Baby in the Brier Patch for me.
http://arxiv.org/pdf/1011.4376.pdf
I find it disheartening how much smaller is the effect explored in this "Tar Baby in the Brier Patch" paper and van Tiggelen's other papers, compared to what is claimed by the EM Drive researchers (particularly what is claimed by Shawyer and Prof. Yang regarding measured forces), and the fact that Shawyer and Prof. Yang do not use any dielectric polymer insert in their tests.
I understand the concern, I really do and it worries me too. There might be more than one way. I'm looking for the connection(s) and commonality in all.

It's hard to glean information from the other tests, I see what's going on right now in setting the cavity sizes and selecting correct harmonics (good detective work BTW) from the lack of information. Your tests were the only one where I feel confident that you used and reported a Dielectric Polymer with a solid yea/nay, it works, it doesn't. On a side note in my business of building Semiconductor machines (sold it and retired in 08 btw) I received sheet metals (copper included) with very thin coatings of plastic sheeting that needed to be pealed off, did I get it all before using, did some adhere, bonding to the surface affecting the tests? I'm not sure as that info isn't there.

This paper looks like it may offer out a way to the issues of violation of CoE and CoM which is a severe no no. As to the difference between the tests we simply have to do some more detective work. I think the answers are there.
Title: Re: EM Drive Developments - related to space flight applications - Thread 2
Post by: TheTraveller on 05/14/2015 04:24 pm
Have modified my Shawyer Df calculator and best Df scanner as per the derived Shawyer Df equation, using cutoff wavelength and guide wavelength as per microwave industry supplied equations. I assume Shawyer did not supply these equations in his papers as they are equations that should be known to microwave industry individuals skilled in the art. Anyway they are now in the public record.

The scanner still sweeps the frequency range 0Hz to 10GHz but reports the frequency that generates a Df as close to 1 as possible but not over.

The attached results are very interesting as the frequency needed to get the Df to just below 1 is very close to the Rf driving frequency used to generate Lambda0 or free wavelength in the selected medium.

While I'm still testing the spreadsheet, which meets both of Shawyers boundary conditions, the results for my Flight Thruster design are looking to be very close to what I could build. Bit of dimension tweaking should get the Df 1 frequency to the 3.85GHz Shawyer used.

Will post the spreadsheet after a bit more testing.

I've built and tested many microwave cavities over many years.

You're guided wavelength equation is wrong, because this is for a rectangular wave guide (i.e., not even a rectangular cavity)

You need to derive mode of frequency yourself (unless there is a paper somewhere) for a circular tapered cavity. There is no other way around it. I would start with Balanis - Advanced Engineering Electromagnetics as he derives a few examples for other topologies. Right now everything you are doing is wrong because you don't understand the physics. I would study that book from front to cover if I were you.

The Guide Wavelength equation uses a circular cutoff wavelength as it's basis. That cutoff wavelength used is for the end conditions, just before reflection. It only focuses on that happens at each end.

A rectangular waveguide will have a different cutoff wavelength and hence a different Guide Wavelength.

As an ex ham, I see this as 2 semi connected resonate elements of an antenna. Each element has it's own unique operational characteristics as do each of the ends.

The length between the 2 ends is tuned to be at resonance of some sub, prime or harmonic of the Rf driving frequency as Shawyer says in the attachment.

My EM Drive design calculator says that for the Flight Thruster dimensions worked out on this thread, the length is very close to resonance at 2x the 3.85GHz wavelength and likewise the Df = 1 condition also occurs at close to the Rf driving frequency. I don't think this is a random event.

While I respect you may not agree with Shawyers or this analysis, I suggest that the Df =1 and length resonance results supporting operation at 3.85GHZ has added some degree of validity to the equation and calc process. At least for me.
Title: Re: EM Drive Developments - related to space flight applications - Thread 2
Post by: rfcavity on 05/14/2015 04:26 pm
... Right now everything you are doing is wrong because you don't understand the physics. ...

Given that he is at least one person among billions on this planet who is attempting a replication of a mystifying device, much of what he is doing is right. 

Also, to the guy operating the microwave magnetron outside of the microwave: STOP
At best you are violating the laws of your local government's regulatory committee for the electromagnetic spectrum. At worst you will damage your body. At this frequency, the damage is somewhat insidious. Due to low water content of your skin, you don't feel the heat, but internal nerve endings can be damaged so that chronic phantom pain can appear. Sometimes days after exposure. Please STOP otherwise you will inevitably be reported to your government.

While I agree with your safety suggestions, I think your tone and delivery are a bit over the top.  Microwaves are easily contained.

If microwaves were easily contained there wouldn't be a multi billion dollar industry for the testing and consultation of microwave containment.

Seriously, its a big problem. I was amused by this before but when people start using high powered dirty sources to corrupt bands of the spectrum I get angry. I've processed too many Earth Observing land surface experiments that were corrupted and unusable outside of Canada and the US due to unlicensed device usage. It is perplexing that a forum dedicated to spaceflight would encourage such bad form.
Title: Re: EM Drive Developments - related to space flight applications - Thread 2
Post by: rfcavity on 05/14/2015 04:30 pm
Have modified my Shawyer Df calculator and best Df scanner as per the derived Shawyer Df equation, using cutoff wavelength and guide wavelength as per microwave industry supplied equations. I assume Shawyer did not supply these equations in his papers as they are equations that should be known to microwave industry individuals skilled in the art. Anyway they are now in the public record.

The scanner still sweeps the frequency range 0Hz to 10GHz but reports the frequency that generates a Df as close to 1 as possible but not over.

The attached results are very interesting as the frequency needed to get the Df to just below 1 is very close to the Rf driving frequency used to generate Lambda0 or free wavelength in the selected medium.

While I'm still testing the spreadsheet, which meets both of Shawyers boundary conditions, the results for my Flight Thruster design are looking to be very close to what I could build. Bit of dimension tweaking should get the Df 1 frequency to the 3.85GHz Shawyer used.

Will post the spreadsheet after a bit more testing.

I've built and tested many microwave cavities over many years.

You're guided wavelength equation is wrong, because this is for a rectangular wave guide (i.e., not even a rectangular cavity)

You need to derive mode of frequency yourself (unless there is a paper somewhere) for a circular tapered cavity. There is no other way around it. I would start with Balanis - Advanced Engineering Electromagnetics as he derives a few examples for other topologies. Right now everything you are doing is wrong because you don't understand the physics. I would study that book from front to cover if I were you.

The Guide Wavelength equation uses a circular cutoff wavelength as it's basis. That cutoff wavelength used is for the end conditions, just before reflection. It only focuses on that happens at each end.

A rectangular waveguide will have a different cutoff wavelength and hence a different Guide Wavelength.

As an ex ham, I see this as 2 semi connected resonate elements of an antenna. Each element has it's own unique operational characteristics as do each of the ends.

The length between the 2 ends is tuned to be at resonance of some sub, prime or harmonic of the Rf driving frequency as Shawyer says in the attachment.

My EM Drive design calculator says that for the Flight Thruster dimensions worked out on this thread, the length is very close to resonance at 2x the 3.85GHz wavelength and likewise the Df = 1 condition also occurs at close to the Rf driving frequency. I don't think this is a random event.

While I respect you may not agree with Shawyers or this analysis, I suggest that the Df =1 and length resonance results supporting operation at 3.85GHZ has added some degree of validity to the equation and calc process. At least for me.

The length of the waveguide from the magnetron is chosen to reduce back reflected power from the cavity to protect itself. The waveguide feeding from the magnetron into the cavity is a separate entity. So its hard to really understand what you are talking about. Cut off frequency and mode shapes for different cavity shapes have been described long ago and proven again and again, so I'm not sure what this has to do with Sawyer and his (strange) calculations.
Title: Re: EM Drive Developments - related to space flight applications - Thread 2
Post by: Mulletron on 05/14/2015 04:39 pm
...
I can feel your pain, understand your concern and yes it concerns me too.
I'm digging through this paper right now and it seems to offer an answer. I'll re-read it several times, as it's looking to be the Tar Baby in the Brier Patch for me.
http://arxiv.org/pdf/1011.4376.pdf
I find it disheartening how much smaller is the effect explored in this "Tar Baby in the Brier Patch" paper and van Tiggelen's other papers, compared to what is claimed by the EM Drive researchers (particularly what is claimed by Shawyer and Prof. Yang regarding measured forces), and the fact that Shawyer and Prof. Yang do not use any dielectric polymer insert in their tests.
I understand the concern, I really do and it worries me too. There might be more than one way. I'm looking for the connection(s) and commonality in all.

It's hard to glean information from the other tests, I see what's going on right now in setting the cavity sizes and selecting correct harmonics (good detective work BTW) from the lack of information. Your tests were the only one where I feel confident that you used and reported a Dielectric Polymer with a solid yea/nay, it works, it doesn't. On a side note in my business of building Semiconductor machines (sold it and retired in 08 btw) I received sheet metals (copper included) with very thin coatings of plastic sheeting that needed to be pealed off, did I get it all before using, did some adhere, bonding to the surface affecting the tests? I'm not sure as that info isn't there.

This paper looks like it may offer out a way to the issues of violation of CoE and CoM which is a severe no no. As to the difference between the tests we simply have to do some more detective work. I think the answers are there.

Don't forget about the nonreciprocity of Nitrogen papers.
http://arxiv.org/abs/1101.0712
http://arxiv.org/abs/1101.1174
http://www2.cnrs.fr/en/1859.htm
http://phys.org/news/2011-05-when-the-speed-of-light.html
http://forum.nasaspaceflight.com/index.php?topic=36313.msg1330846#msg1330846

Imagine the implications of having the speed of light depend on its direction in a medium.


Quote
I've been trying to wrap my brain around why a difference in phase results in a better thrust.  Also, why couldn't EW obtain a thrust without a dielectric?

I think we can all agree that in order for a net thrust, the momentum delivered to the larger end plate is smaller than that delivered to the smaller end plate. So where did the momentum go?

Can momentum be delivered and removed from an orbiting electron?

Take a simple two dimensional case with two atoms, one on the small end, one on the large end, each with their own electron orbiting at a the same angular frequency. If a force is applied to both of them, one in the direction of revolution and the other opposite, one of the forces would slow down the electron and the other would speed it up. The sped up electron requires a larger force to keep it tied to the nucleus and we have a net thrust.

Perhaps this could help explain a couple things:

The dielectric is composed of different elements, thus the electrons are orbiting at a different angular velocity. Using a constant frequency with different elements gives a certain degree of difference in the phase at which momentum is delivered to the electrons.

Shawyer observed more losses with a dielectric because a magnetron outputs a signal at many phases and somehow 'matches' the orbital tendency of the electrons.
I imagine the magnetic component of the wave could be contributing to an alignment of electrons which could amplify this miniscule effect.

Any thoughts?
I'm digging through this paper right now and it seems to offer an answer to your question. I'll re-read it several times, as it's looking to be the Tar Baby in the Brier Patch for me.
http://arxiv.org/pdf/1011.4376.pdf

Exact words from Eagleworks about obtaining thrust without a dielectric:
Quote
We performed some very early evaluations without the dielectric resonator (TE012 mode at 2168 MHz, with power levels up to ~30 watts) and measured no significant net thrust.
http://www.libertariannews.org/wp-content/uploads/2014/07/AnomalousThrustProductionFromanRFTestDevice-BradyEtAl.pdf
Title: Re: EM Drive Developments - related to space flight applications - Thread 2
Post by: LasJayhawk on 05/14/2015 04:53 pm
... Right now everything you are doing is wrong because you don't understand the physics. ...

Given that he is at least one person among billions on this planet who is attempting a replication of a mystifying device, much of what he is doing is right. 

Also, to the guy operating the microwave magnetron outside of the microwave: STOP
At best you are violating the laws of your local government's regulatory committee for the electromagnetic spectrum. At worst you will damage your body. At this frequency, the damage is somewhat insidious. Due to low water content of your skin, you don't feel the heat, but internal nerve endings can be damaged so that chronic phantom pain can appear. Sometimes days after exposure. Please STOP otherwise you will inevitably be reported to your government.

While I agree with your safety suggestions, I think your tone and delivery are a bit over the top.  Microwaves are easily contained.

If microwaves were easily contained there wouldn't be a multi billion dollar industry for the testing and consultation of microwave containment.

Seriously, its a big problem. I was amused by this before but when people start using high powered dirty sources to corrupt bands of the spectrum I get angry. I've processed too many Earth Observing land surface experiments that were corrupted and unusable outside of Canada and the US due to unlicensed device usage. It is perplexing that a forum dedicated to spaceflight would encourage such bad form.

Like the 17 years it took Parkes to find out it was a microwave oven causing the perytons they were getting in the radio telescope? http://abcnews.go.com/Technology/microwave-oven-stumped-astronomers-17-years/story?id=30822415

And I don't see how you will get repeatablity with a nasty old maggie, unless this is a brute force effect. A Klystron based source would be a better option IMHO.
Title: Re: EM Drive Developments - related to space flight applications - Thread 2
Post by: Rodal on 05/14/2015 05:00 pm
...Don't forget about the nonreciprocity of Nitrogen papers.
http://arxiv.org/abs/1101.0712
http://arxiv.org/abs/1101.1174
http://www2.cnrs.fr/en/1859.htm
http://phys.org/news/2011-05-when-the-speed-of-light.html
http://forum.nasaspaceflight.com/index.php?topic=36313.msg1330846#msg1330846

Imagine the implications of having the speed of light depend on its direction in a medium....
Isn't the magnitude of this effect (nonreciprocity of Nitrogen papers), regarding a thrust force even smaller than the magnitude for the chiral polymer ?

Therefore, isn't this unable to explain what is claimed by Shawyer and Yang? (numerically speaking, as those effects appear to be orders of magnitude smaller than what is being claimed by the researchers)
Title: Re: EM Drive Developments - related to space flight applications - Thread 2
Post by: Mulletron on 05/14/2015 05:03 pm
... Right now everything you are doing is wrong because you don't understand the physics. ...

Given that he is at least one person among billions on this planet who is attempting a replication of a mystifying device, much of what he is doing is right. 

Also, to the guy operating the microwave magnetron outside of the microwave: STOP
At best you are violating the laws of your local government's regulatory committee for the electromagnetic spectrum. At worst you will damage your body. At this frequency, the damage is somewhat insidious. Due to low water content of your skin, you don't feel the heat, but internal nerve endings can be damaged so that chronic phantom pain can appear. Sometimes days after exposure. Please STOP otherwise you will inevitably be reported to your government.

While I agree with your safety suggestions, I think your tone and delivery are a bit over the top.  Microwaves are easily contained.

If microwaves were easily contained there wouldn't be a multi billion dollar industry for the testing and consultation of microwave containment.

Seriously, its a big problem. I was amused by this before but when people start using high powered dirty sources to corrupt bands of the spectrum I get angry. I've processed too many Earth Observing land surface experiments that were corrupted and unusable outside of Canada and the US due to unlicensed device usage. It is perplexing that a forum dedicated to spaceflight would encourage such bad form.

Like the 17 years it took Parkes to find out it was a microwave oven causing the perytons they were getting in the radio telescope? http://abcnews.go.com/Technology/microwave-oven-stumped-astronomers-17-years/story?id=30822415

And I don't see how you will get repeatablity with a nasty old maggie, unless this is a brute force effect. A Klystron based source would be a better option IMHO.

The guy in Romania has the magnetron antenna clearly placed inside the frustum. And yes it is a brute force effort and will likely burn out his source if he pushes it. Luckily they're a dime a dozen. He's in Romania, so we have no idea what sort of regulatory environment they have there. Even in the US, nobody is going to notice noise in the ISM band for 40 seconds. Those in the US might not want to post on the internet what they're doing with magnetrons without consulting the FCC first.

The spoils go to those who are bold enough to try. The trick is to accept and manage risk appropriately. If you aren't willing to take risk, you'll never even get the chance to fail.
Title: Re: EM Drive Developments - related to space flight applications - Thread 2
Post by: TheTraveller on 05/14/2015 05:08 pm
The length of the waveguide from the magnetron is chosen to reduce back reflected power from the cavity to protect itself. The waveguide feeding from the magnetron into the cavity is a separate entity. So its hard to really understand what you are talking about. Cut off frequency and mode shapes for different cavity shapes have been described long ago and proven again and again, so I'm not sure what this has to do with Sawyer and his (strange) calculations.

Yes of course and as Shawyer says the:

Microwave energy is fed from a magnetron,
.. via a tuned feed,
.... to a tapered waveguide,
...... whose overall length
........ gives resonance
.......... at the operating frequency
............ of the magnetron.

1) microwave feed from magnetron to tapered waveguide / cavity is tuned for best feed.

2) overall length (up and down) of the tapered waveguide / cavity is adjusted so it resonates with the applied Rf or the applied Rf frequency is altered to achieve resonance between the 2 end plates.

This is what Shawyer was doing physically in his 1st and 2nd EM Drives. Note tuning mechanism on top of the 1st test device and extensive stepper motor length tuning system on end of the 2nd device.

Last attachment shows what I believe the left side tuning system was doing inside the fixed diameter cylindrical section.

The Rf signal must achieve length resonance for the EM Drive to deliver thrust. No resonance, to thrust. I also believe the DF should get very close to 1 at the same applied Rf frequency.

So 3 dimensions to tune to get optimal thrust production. Get Df close to 1 at Rf frequency via small & big end diameter adjustments and get cavity length resonance to the applied Rf frequency via adjustment of the spacing between the end plates. Get one or both wrong and there will be no thrust.
Title: Re: EM Drive Developments - related to space flight applications - Thread 2
Post by: SeeShells on 05/14/2015 05:09 pm
...
I can feel your pain, understand your concern and yes it concerns me too.
I'm digging through this paper right now and it seems to offer an answer. I'll re-read it several times, as it's looking to be the Tar Baby in the Brier Patch for me.
http://arxiv.org/pdf/1011.4376.pdf
I find it disheartening how much smaller is the effect explored in this "Tar Baby in the Brier Patch" paper and van Tiggelen's other papers, compared to what is claimed by the EM Drive researchers (particularly what is claimed by Shawyer and Prof. Yang regarding measured forces), and the fact that Shawyer and Prof. Yang do not use any dielectric polymer insert in their tests.
I understand the concern, I really do and it worries me too. There might be more than one way. I'm looking for the connection(s) and commonality in all.

It's hard to glean information from the other tests, I see what's going on right now in setting the cavity sizes and selecting correct harmonics (good detective work BTW) from the lack of information. Your tests were the only one where I feel confident that you used and reported a Dielectric Polymer with a solid yea/nay, it works, it doesn't. On a side note in my business of building Semiconductor machines (sold it and retired in 08 btw) I received sheet metals (copper included) with very thin coatings of plastic sheeting that needed to be pealed off, did I get it all before using, did some adhere, bonding to the surface affecting the tests? I'm not sure as that info isn't there.

This paper looks like it may offer out a way to the issues of violation of CoE and CoM which is a severe no no. As to the difference between the tests we simply have to do some more detective work. I think the answers are there.

Don't forget about the nonreciprocity of Nitrogen papers.
http://arxiv.org/abs/1101.0712
http://arxiv.org/abs/1101.1174
http://www2.cnrs.fr/en/1859.htm
http://phys.org/news/2011-05-when-the-speed-of-light.html
http://forum.nasaspaceflight.com/index.php?topic=36313.msg1330846#msg1330846

Imagine the implications of having the speed of light depend on its direction in a medium.


Quote
I've been trying to wrap my brain around why a difference in phase results in a better thrust.  Also, why couldn't EW obtain a thrust without a dielectric?

I think we can all agree that in order for a net thrust, the momentum delivered to the larger end plate is smaller than that delivered to the smaller end plate. So where did the momentum go?

Can momentum be delivered and removed from an orbiting electron?

Take a simple two dimensional case with two atoms, one on the small end, one on the large end, each with their own electron orbiting at a the same angular frequency. If a force is applied to both of them, one in the direction of revolution and the other opposite, one of the forces would slow down the electron and the other would speed it up. The sped up electron requires a larger force to keep it tied to the nucleus and we have a net thrust.

Perhaps this could help explain a couple things:

The dielectric is composed of different elements, thus the electrons are orbiting at a different angular velocity. Using a constant frequency with different elements gives a certain degree of difference in the phase at which momentum is delivered to the electrons.

Shawyer observed more losses with a dielectric because a magnetron outputs a signal at many phases and somehow 'matches' the orbital tendency of the electrons.
I imagine the magnetic component of the wave could be contributing to an alignment of electrons which could amplify this miniscule effect.

Any thoughts?
I'm digging through this paper right now and it seems to offer an answer to your question. I'll re-read it several times, as it's looking to be the Tar Baby in the Brier Patch for me.
http://arxiv.org/pdf/1011.4376.pdf

Exact words from Eagleworks about obtaining thrust without a dielectric:
Quote
We performed some very early evaluations without the dielectric resonator (TE012 mode at 2168 MHz, with power levels up to ~30 watts) and measured no significant net thrust.
http://www.libertariannews.org/wp-content/uploads/2014/07/AnomalousThrustProductionFromanRFTestDevice-BradyEtAl.pdf

Explain to me if I'm light in a medium that forced to go faster than the medium allows, what laws have I broken? Blame it on me sitting in my hot tub watching the waves interact in ways faster than they normally would just by going across the water. . .  and also needing a break.
Title: Re: EM Drive Developments - related to space flight applications - Thread 2
Post by: zellerium on 05/14/2015 05:11 pm


If microwaves were easily contained there wouldn't be a multi billion dollar industry for the testing and consultation of microwave containment.

Seriously, its a big problem. I was amused by this before but when people start using high powered dirty sources to corrupt bands of the spectrum I get angry. I've processed too many Earth Observing land surface experiments that were corrupted and unusable outside of Canada and the US due to unlicensed device usage. It is perplexing that a forum dedicated to spaceflight would encourage such bad form.

We plan to enclose our system in many layers of laminated microwave absorbing sheets:
http://www.lessemf.com/259.pdf

Title: Re: EM Drive Developments - related to space flight applications - Thread 2
Post by: phaseshift on 05/14/2015 05:33 pm
Today i did the first test with the Emdrive (microwave oven magnetron and cooper frustum)
The setup (magnetron and frusum) was suspended in a pendulum.
I applied power for 40 Seconds with no visible thrust. Tomorrow will will try again with the magnetron on the small side. You have any suggestion for what should be the distance from the small side?
After this i will adjust the power to the filament of magnetron and the frequency.
To fine adjust the frequency i thought i can put 2 coils over the magnetron magnets to modify the magnetic field.
My website;
http://www.masinaelectrica.com/emdrive-independent-test/

https://www.youtube.com/watch?v=2-cgVoLUJ8w

Can you turn the drive on and off rapidly so that it lengthens the oscillation of the pendulum, like a child's swing? It would have to be computer controlled.. The period might be less than the engine response time though. I'm thinking of running through the math on this.

Title: Re: EM Drive Developments - related to space flight applications - Thread 2
Post by: Rodal on 05/14/2015 05:49 pm
@Rodal: You say that you get a blow-up with your Df formula, and yet
a) you agree that both lambdag1,2 are greater than lambda0 (because vg < c)
b) you agree that, this being the case, it's mathematically impossible to get blow-up (denominator zero)

Clarify please?

This is a picture for the case

bD=0.2797 m;sD=0.1588 m;cMedium=in Air

which has a cut-off frequency associated with the small diameter of 1.10609 GHz

Notice that there is a singularity at 1.10609 GHz such that Shawyer's Design Factor doesn't have a Real value for frequencies below it.  Also notice the rise and steepening of the Design Factor curve as the cut-off frequency is approached

Title: Re: EM Drive Developments - related to space flight applications - Thread 2
Post by: StrongGR on 05/14/2015 06:00 pm
I am back with an updated draft after some terrible news around about NASA dismissing these researches. They should not as, otherwise, it could happen as with Galilei having his detractors even not trying to look in the telescope, just dismissing on faith.

I have analysed the case of the frustum and the results appear to be striking. One must admit that geometry comes to rescue not just general relativity. For this particular geometry the cavity can be made susceptible to gravitational effects if your choice of the two radii of the cavity is smart enough. This is something to be confirmed yet, just my theoretical result, but shocking anyway.

As usual, any comment is very welcome.
Title: Re: EM Drive Developments - related to space flight applications - Thread 2
Post by: Rodal on 05/14/2015 06:05 pm
Have modified my Shawyer Df calculator and best Df scanner as per the derived Shawyer Df equation, using cutoff wavelength and guide wavelength as per microwave industry supplied equations. I assume Shawyer did not supply these equations in his papers as they are equations that should be known to microwave industry individuals skilled in the art. Anyway they are now in the public record.

The scanner still sweeps the frequency range 0Hz to 10GHz but reports the frequency that generates a Df as close to 1 as possible but not over.

The attached results are very interesting as the frequency needed to get the Df to just below 1 is very close to the Rf driving frequency used to generate Lambda0 or free wavelength in the selected medium.

While I'm still testing the spreadsheet, which meets both of Shawyers boundary conditions, the results for my Flight Thruster design are looking to be very close to what I could build. Bit of dimension tweaking should get the Df 1 frequency to the 3.85GHz Shawyer used.

Will post the spreadsheet after a bit more testing.

This is a picture for the FLIGHT THRUSTER case

bD=0.2440 m ;sD=0.1450 m ;cMedium=299705000 m/s (Air)

which has a cut-off frequency associated with the small diameter of 1.21136 GHz

Notice that there is a singularity at 1.21136 GHz such that Shawyer's Design Factor doesn't have a Real value for frequencies below it.  Also notice the rise and steepening of the Design Factor curve as the cut-off frequency is approached.

QUESTION: If Shawyer thinks that his Design Factor steepening behavior near the cut-off frequency associated with the small diameter is correct, why didn't he test his Flight Thruster at a lower frequency, closer to 1.2 GHz instead of the higher frequency he chose of 3.782 GHz? Doesn't Shawyer want to maximize thrust force ?
Title: Re: EM Drive Developments - related to space flight applications - Thread 2
Post by: TheTraveller on 05/14/2015 06:07 pm
Here are the 1st cut of the Flight Thruster dimensions based on Df = just below 1.0 and length resonance.

I'm impressed the dimensions are VERY close to those calculated from the Flight Thruster picture.

The calculator lets me set the big and small end dimensions to get Df just below 1 at a calculated frequency close to the 3.85GHz design frequency. Then using the What If capability, the cavity length is auto adjusted to get resonance at the same frequency that generates a Df just below 1.

Attached are the Flight Thruster dimensions and resonate frequency that lock the 2 optimal frequencies together. While 3.78GHz is not 3.85GHz, it is close enough for my variable Rf frequency generator to lock on.

Time to start getting serious and design, build and test the Teeter Totter balance beam thrust measurement system and data capture & logging system.
Title: Re: EM Drive Developments - related to space flight applications - Thread 2
Post by: TheTraveller on 05/14/2015 06:09 pm
Have modified my Shawyer Df calculator and best Df scanner as per the derived Shawyer Df equation, using cutoff wavelength and guide wavelength as per microwave industry supplied equations. I assume Shawyer did not supply these equations in his papers as they are equations that should be known to microwave industry individuals skilled in the art. Anyway they are now in the public record.

The scanner still sweeps the frequency range 0Hz to 10GHz but reports the frequency that generates a Df as close to 1 as possible but not over.

The attached results are very interesting as the frequency needed to get the Df to just below 1 is very close to the Rf driving frequency used to generate Lambda0 or free wavelength in the selected medium.

While I'm still testing the spreadsheet, which meets both of Shawyers boundary conditions, the results for my Flight Thruster design are looking to be very close to what I could build. Bit of dimension tweaking should get the Df 1 frequency to the 3.85GHz Shawyer used.

Will post the spreadsheet after a bit more testing.

This is a picture for the FLIGHT THRUSTER case

bD=0.2440 m ;sD=0.1450 m ;cMedium=299705000 m/s (Air)

which has a cut-off frequency associated with the small diameter of 1.21136 GHz

Notice that there is a singularity at 1.21136 GHz such that Shawyer's Design Factor doesn't have a Real value for frequencies below it.  Also notice the rise and steepening of the Design Factor curve as the cut-off frequency is approached.

QUESTION: If Shawyer thinks that his Design Factor steepening behavior near the cut-off frequency associated with the small diameter is correct, why didn't he test his Flight Thruster at a lower frequency, closer to 1.2 GHz instead of the higher frequency he chose of 3.782 GHz? Doesn't Shawyer want to maximize thrust force ?

Shawyer has said Df = 1 is the max value to consider in the real world. Which I have done. Try these dimensions and frequency and see what Df you get?

Note both the DF = 1 frequency and cavity length resonance frequency are the same and that the cavity dimension are close to those workout on this forum. So the guys who did that work were close.
Title: Re: EM Drive Developments - related to space flight applications - Thread 2
Post by: Rodal on 05/14/2015 06:13 pm
Have modified my Shawyer Df calculator and best Df scanner as per the derived Shawyer Df equation, using cutoff wavelength and guide wavelength as per microwave industry supplied equations. I assume Shawyer did not supply these equations in his papers as they are equations that should be known to microwave industry individuals skilled in the art. Anyway they are now in the public record.

The scanner still sweeps the frequency range 0Hz to 10GHz but reports the frequency that generates a Df as close to 1 as possible but not over.

The attached results are very interesting as the frequency needed to get the Df to just below 1 is very close to the Rf driving frequency used to generate Lambda0 or free wavelength in the selected medium.

While I'm still testing the spreadsheet, which meets both of Shawyers boundary conditions, the results for my Flight Thruster design are looking to be very close to what I could build. Bit of dimension tweaking should get the Df 1 frequency to the 3.85GHz Shawyer used.

Will post the spreadsheet after a bit more testing.

This is a picture for the FLIGHT THRUSTER case

bD=0.2440 m ;sD=0.1450 m ;cMedium=299705000 m/s (Air)

which has a cut-off frequency associated with the small diameter of 1.21136 GHz

Notice that there is a singularity at 1.21136 GHz such that Shawyer's Design Factor doesn't have a Real value for frequencies below it.  Also notice the rise and steepening of the Design Factor curve as the cut-off frequency is approached.

QUESTION: If Shawyer thinks that his Design Factor steepening behavior near the cut-off frequency associated with the small diameter is correct, why didn't he test his Flight Thruster at a lower frequency, closer to 1.2 GHz instead of the higher frequency he chose of 3.782 GHz? Doesn't Shawyer want to maximize thrust force ?

Shawyer has said Df = 1 is the max value to consider in the real world. Which I have done. Try these dimensions and frequency and see what Df you get?

Note both the DF = 1 frequency and cavity length resonance frequency are the same and that the cavity dimension are close to those workout on this forum. So the guys who did that work were close.

(http://forum.nasaspaceflight.com/index.php?action=dlattach;topic=36313.0;attach=830141;image)

Sorry, the dimensions I see in that spreadsheet are the same dimensions I see in your prior post:

http://forum.nasaspaceflight.com/index.php?topic=36313.msg1374106#msg1374106

bD=0.2440 m ;sD=0.1450 m ;

which give a Design Factor below 0.5

Could you please send the dimensions that give DF=0.9999 in another post and double check them ? Thanks
Title: Re: EM Drive Developments - related to space flight applications - Thread 2
Post by: TheTraveller on 05/14/2015 06:16 pm
Have modified my Shawyer Df calculator and best Df scanner as per the derived Shawyer Df equation, using cutoff wavelength and guide wavelength as per microwave industry supplied equations. I assume Shawyer did not supply these equations in his papers as they are equations that should be known to microwave industry individuals skilled in the art. Anyway they are now in the public record.

The scanner still sweeps the frequency range 0Hz to 10GHz but reports the frequency that generates a Df as close to 1 as possible but not over.

The attached results are very interesting as the frequency needed to get the Df to just below 1 is very close to the Rf driving frequency used to generate Lambda0 or free wavelength in the selected medium.

While I'm still testing the spreadsheet, which meets both of Shawyers boundary conditions, the results for my Flight Thruster design are looking to be very close to what I could build. Bit of dimension tweaking should get the Df 1 frequency to the 3.85GHz Shawyer used.

Will post the spreadsheet after a bit more testing.

This is a picture for the FLIGHT THRUSTER case

bD=0.2440 m ;sD=0.1450 m ;cMedium=299705000 m/s (Air)

which has a cut-off frequency associated with the small diameter of 1.21136 GHz

Notice that there is a singularity at 1.21136 GHz such that Shawyer's Design Factor doesn't have a Real value for frequencies below it.  Also notice the rise and steepening of the Design Factor curve as the cut-off frequency is approached.

QUESTION: If Shawyer thinks that his Design Factor steepening behavior near the cut-off frequency associated with the small diameter is correct, why didn't he test his Flight Thruster at a lower frequency, closer to 1.2 GHz instead of the higher frequency he chose of 3.782 GHz? Doesn't Shawyer want to maximize thrust force ?

Shawyer has said Df = 1 is the max value to consider in the real world. Which I have done. Try these dimensions and frequency and see what Df you get?

Note both the DF = 1 frequency and cavity length resonance frequency are the same and that the cavity dimension are close to those workout on this forum. So the guys who did that work were close.

Sorry, the dimensions I see in that spreadsheet are the same dimensions I see in your prior post:

http://forum.nasaspaceflight.com/index.php?topic=36313.msg1374106#msg1374106

bD=0.2440 m ;sD=0.1450 m ;

which give a Design Factor below 0.5

Could you please send the dimensions in another post and double check them ? Thanks

Note the frequency is NOT 3.85GHz. There are THREE factors in the Df equation, Small End diameter, Big End diameter and FREQUENCY.

Check the Df at the quoted frequency and let me know the Df?
Title: Re: EM Drive Developments - related to space flight applications - Thread 2
Post by: Rodal on 05/14/2015 06:18 pm
...
Note the frequency is NOT 3.85GHz. There are THREE factors in the Df equation, Small End diameter, Big End diameter and FREQUENCY.

Look at this image:  http://forum.nasaspaceflight.com/index.php?action=dlattach;topic=36313.0;attach=830138

for those dimensions and look at the horizontal axis for frequency.  At that frequency the Design Factor is still below 0.5

Shawyer's design factor is practically unaffected by frequency except near the cut-off frequency for the small diameter.

Shawyer's Design Factor is highest at lowest frequency (near cut-off) and it decreases with frequency

See my equations:  http://forum.nasaspaceflight.com/index.php?topic=36313.msg1374110#msg1374110
Title: Re: EM Drive Developments - related to space flight applications - Thread 2
Post by: tchernik on 05/14/2015 06:22 pm
I am back with an updated draft after some terrible news around about NASA dismissing these researches. They should not as, otherwise, it could happen as with Galilei having his detractors even not trying to look in the telescope, just dismissing on faith.

I have analysed the case of the frustum and the results appear to be striking. One must admit that geometry comes to rescue not just general relativity. For this particular geometry the cavity can be made susceptible to gravitational effects if your choice of the two radii of the cavity is smart enough. This is something to be confirmed yet, just my theoretical result, but shocking anyway.

As usual, any comment is very welcome.

Which news, can you elaborate?
Title: Re: EM Drive Developments - related to space flight applications - Thread 2
Post by: Blaine on 05/14/2015 06:30 pm
Lucky that Nasa hasn't stopped the experiments.  It would be a more than a little upsetting if they stoped the progress Eagleworks has made.
Title: Re: EM Drive Developments - related to space flight applications - Thread 2
Post by: Rodal on 05/14/2015 06:32 pm
Lucky that Nasa hasn't stopped the experiments.  It would be a more than a little upsetting if they stoped the progress Eagle works has made.
NASA has surely not increased Eagleworks little tiny teeny budget and we haven't heard from Paul March since the NSF article...

They are downplaying it.  At NASA Glenn's site they are directing interested people to an ex-employee (Millis) and not saying anything about Eagleworks work... :(

Everybody interested should write to their congressmen
Title: Re: EM Drive Developments - related to space flight applications - Thread 2
Post by: Notsosureofit on 05/14/2015 06:35 pm
I am back with an updated draft after some terrible news around about NASA dismissing these researches. They should not as, otherwise, it could happen as with Galilei having his detractors even not trying to look in the telescope, just dismissing on faith.

I have analysed the case of the frustum and the results appear to be striking. One must admit that geometry comes to rescue not just general relativity. For this particular geometry the cavity can be made susceptible to gravitational effects if your choice of the two radii of the cavity is smart enough. This is something to be confirmed yet, just my theoretical result, but shocking anyway.

As usual, any comment is very welcome.

Yes, but am I correct as seeing the a and b terms in the frustum case as generating Doppler-shifted sidebands ? (the alternative being non-linear terms)

This would suggest that the EW interferometer should replace the "pillbox" cavity with a frustrum and analyze by including a Fabry-Perot into the system?   (might need a mode-locked laser as well.)
Title: Re: EM Drive Developments - related to space flight applications - Thread 2
Post by: SeeShells on 05/14/2015 06:39 pm
Lucky that Nasa hasn't stopped the experiments.  It would be a more than a little upsetting if they stoped the progress Eagle works has made.
NASA has surely not increased Eagleworks little tiny teeny budget and we haven't heard from Paul March since the NSF article...

They are downplaying it.  At NASA Glenn's site they are directing interested people to an ex-employee (Millis) and not saying anything about Eagleworks work... :(

Everybody interested should write to their congressmen
Yes! I have done a letter and also urge you to do so, oh also beg your friends and relatives too.
Title: Re: EM Drive Developments - related to space flight applications - Thread 2
Post by: StrongGR on 05/14/2015 07:04 pm
I am back with an updated draft after some terrible news around about NASA dismissing these researches. They should not as, otherwise, it could happen as with Galilei having his detractors even not trying to look in the telescope, just dismissing on faith.

I have analysed the case of the frustum and the results appear to be striking. One must admit that geometry comes to rescue not just general relativity. For this particular geometry the cavity can be made susceptible to gravitational effects if your choice of the two radii of the cavity is smart enough. This is something to be confirmed yet, just my theoretical result, but shocking anyway.

As usual, any comment is very welcome.

Which news, can you elaborate?

NASA reporting that there is no warpdrive spacecraft and downplaying Eagleworks:

See:

http://news.yahoo.com/no-warp-drive-nasa-downplays-impossible-em-drive-193528141.html

I tried to post this but the moderators removed my post.
Title: Re: EM Drive Developments - related to space flight applications - Thread 2
Post by: StrongGR on 05/14/2015 07:06 pm
I am back with an updated draft after some terrible news around about NASA dismissing these researches. They should not as, otherwise, it could happen as with Galilei having his detractors even not trying to look in the telescope, just dismissing on faith.

I have analysed the case of the frustum and the results appear to be striking. One must admit that geometry comes to rescue not just general relativity. For this particular geometry the cavity can be made susceptible to gravitational effects if your choice of the two radii of the cavity is smart enough. This is something to be confirmed yet, just my theoretical result, but shocking anyway.

As usual, any comment is very welcome.

Yes, but am I correct as seeing the a and b terms in the frustum case as generating Doppler-shifted sidebands ? (the alternative being non-linear terms)

This would suggest that the EW interferometer should replace the "pillbox" cavity with a frustrum and analyze by including a Fabry-Perot into the system?   (might need a mode-locked laser as well.)

I agree. This would make a tunable device to measure with.
Title: Re: EM Drive Developments - related to space flight applications - Thread 2
Post by: StrongGR on 05/14/2015 07:15 pm
Lucky that Nasa hasn't stopped the experiments.  It would be a more than a little upsetting if they stoped the progress Eagle works has made.
NASA has surely not increased Eagleworks little tiny teeny budget and we haven't heard from Paul March since the NSF article...

They are downplaying it.  At NASA Glenn's site they are directing interested people to an ex-employee (Millis) and not saying anything about Eagleworks work... :(

Everybody interested should write to their congressmen

NASA cannot increase spending for this with all the scientific community complaining and crying.
Title: Re: EM Drive Developments - related to space flight applications - Thread 2
Post by: Einstein79 on 05/14/2015 07:17 pm
Reading this:

Siegel said he is seeing claims of thrust happening just a few times over many tests, with a frequency that is "not inconsistent with random chance." Further, the thrust that was produced in these rare instances was apparently just above the margin of error for measurement, he added.

from this article:

http://news.yahoo.com/no-warp-drive-nasa-downplays-impossible-em-drive-193528141.html (http://news.yahoo.com/no-warp-drive-nasa-downplays-impossible-em-drive-193528141.html)

makes me think that maybe the drive is the wrong shape.

It has been stated by someone, not sure who, that the thrust was the result of the waves constructively interfering on the larger diameter and destructively interfering on the other, smaller, diameter. Also, the waves are allowed to travel freely in the cavity so that the photons have multiple paths. Because thrusts measured are alleged to simply be error and a "random chance" due to the multiple optical paths, then seems to me that this has now become an optimization problem.

Maybe we can change the dimensions to force coherence by making the total optical path shorter which would also decrease the temperature and make it more efficient. Maybe the "slits" for the device that were originally proposed should be placed before the source to split the beam so we can control the path length and will know the phase shift, similar to a Michelson or Mach-Zehnder interferometer.

Any thoughts?   

Title: Re: EM Drive Developments - related to space flight applications - Thread 2
Post by: TheTraveller on 05/14/2015 07:23 pm
...
Note the frequency is NOT 3.85GHz. There are THREE factors in the Df equation, Small End diameter, Big End diameter and FREQUENCY.

Look at this image:  http://forum.nasaspaceflight.com/index.php?action=dlattach;topic=36313.0;attach=830138

for those dimensions and look at the horizontal axis for frequency.  At that frequency the Design Factor is still below 0.5

Shawyer's design factor is practically unaffected by frequency except near the cut-off frequency for the small diameter.

Shawyer's Design Factor is highest at lowest frequency (near cut-off) and it decreases with frequency

See my equations:  http://forum.nasaspaceflight.com/index.php?topic=36313.msg1374110#msg1374110

Houston we have a problem.

Working it.

Ok sorted. Good to go.
Title: Re: EM Drive Developments - related to space flight applications - Thread 2
Post by: Rodal on 05/14/2015 07:33 pm
...
Note the frequency is NOT 3.85GHz. There are THREE factors in the Df equation, Small End diameter, Big End diameter and FREQUENCY.

Look at this image:  http://forum.nasaspaceflight.com/index.php?action=dlattach;topic=36313.0;attach=830138

for those dimensions and look at the horizontal axis for frequency.  At that frequency the Design Factor is still below 0.5

Shawyer's design factor is practically unaffected by frequency except near the cut-off frequency for the small diameter.

Shawyer's Design Factor is highest at lowest frequency (near cut-off) and it decreases with frequency

See my equations:  http://forum.nasaspaceflight.com/index.php?topic=36313.msg1374110#msg1374110

Houston we have a problem.

Working it.

Notice that the Design Factor of Shawyer approaches this value for high frequencies (it becomes practically independent of frequency)

Limit[DesignFactor, f -> Infinity]  = (bD^2 - sD^2)/(bD^2 + sD^2)

The highest value of the Design Factor is reached at frequencies just a little over the cut-off frequency for the small end:

Cut-Off frequency for small end= cM/(cst*sD)   

Equations: http://forum.nasaspaceflight.com/index.php?topic=36313.msg1374110#msg1374110

For more limits see this http://forum.nasaspaceflight.com/index.php?topic=36313.msg1374114#msg1374114

Example

bD=0.2440 m ;sD=0.1450 m ;cMedium=299705000 m/s (Air)

which has a cut-off frequency associated with the small diameter of 1.21136 GHz

For the Flight Thruster, the Design Factor approaches 0.478 at infinite frequency:

Limit[DesignFactor, f -> Infinity]  = (bD^2 - sD^2)/(bD^2 + sD^2) = 0.478
Title: Re: EM Drive Developments - related to space flight applications - Thread 2
Post by: TheTraveller on 05/14/2015 08:51 pm
Thanks Dr. Rodal for taking the time to point out my error. Fixed now. Get a Df curve like yours that peaks and then goes to zero as the Guide Frequency goes below CutOff Frequency for the small end.

Interesting interactions.

Keeping small end (g2) Guide Wavelength just above CutOff Wavelength, while operating at a Rf frequency which will generate a high Df, adjusting the length to obtain length resonance at the operation Rf frequency.

The interactions are making sense. My gut is not so much in knots. Starting to feel confident in being able to generate at least 1g of force. Calcs say more like 2g but then the cavity needs a Q of around 50,000, which using spherical end caps should help to make a reality.

Oh BTW obtaining length cavity resonance, which I believe is what is seen doing a spectrum sweep, may not generate any thrust as the small end guide wavelength may be bigger than the cutoff wavelength with the result that little or no significant energy reaches the small end to bounce back.

Shields up. Rock throwing time.
Title: Re: EM Drive Developments - related to space flight applications - Thread 2
Post by: tchernik on 05/14/2015 09:00 pm
Thanks Dr. Rodal for taking the time to point out my error. Fixed now. Get a Df curve like yours that peaks and then goes to zero as the Guide Frequency goes below CutOff Frequency for the small end.

Interesting interactions.

Keeping small end (g2) Guide Wavelength just above CutOff Wavelength, while operating at a Rf frequency which will generate a high Df, adjusting the length to obtain length resonance at the operation Rf frequency.

The interactions are making sense. My gut is not so much in knots. Starting to feel confident in being able to generate at least 1g of force. Calcs say more like 2g but then the cavity needs a Q of around 50,000, which using spherical end caps should help to make a reality.

Oh BTW obtaining length cavity resonance, which I believe is what is seen doing a spectrum sweep, may not generate any thrust as the small end guide wavelength may be bigger than the cutoff wavelength with the result that little or no significant energy reaches the small end to bounce back.

Shields up. Rock throwing time.

This is great. Best of luck.

My mechanic/EM engineering skills are nearly nil, but I'm still an engineer of a different sort and I can tell when people know their stuff. Keep us posted.
Title: Re: EM Drive Developments - related to space flight applications - Thread 2
Post by: Rodal on 05/14/2015 09:45 pm
Thanks Dr. Rodal for taking the time to point out my error. Fixed now. Get a Df curve like yours that peaks and then goes to zero as the Guide Frequency goes below CutOff Frequency for the small end.

Interesting interactions.

Keeping small end (g2) Guide Wavelength just above CutOff Wavelength, while operating at a Rf frequency which will generate a high Df, adjusting the length to obtain length resonance at the operation Rf frequency.

The interactions are making sense. My gut is not so much in knots. Starting to feel confident in being able to generate at least 1g of force. Calcs say more like 2g but then the cavity needs a Q of around 50,000, which using spherical end caps should help to make a reality.

Oh BTW obtaining length cavity resonance, which I believe is what is seen doing a spectrum sweep, may not generate any thrust as the small end guide wavelength may be bigger than the cutoff wavelength with the result that little or no significant energy reaches the small end to bounce back.

Shields up. Rock throwing time.

Congratulations on fixing it.

I see some differences with my plot using Mathematica.  One minor, immaterial difference is that Excel artificially brings the value of the Design Factor down to zero for frequencies below the cutoff frequency and it artificially draws a vertical line at the cutoff frequency.  Mathematica does not, because the Design Factor does not go down to zero below the cut-off frequency, actually its value becomes a Complex number below the cutoff frequency (since it involves the Square Root of a negative number), and the vertical line shouldn't be there.  In reality there is no Real value of the Design Factor below the cut-off frequency associated with the small diameter.  That's a quirk with Excel.

The other thing is that the curvature in your plot looks different than mine. 
Perhaps it is because my plot goes to 4 GHz and your plot goes to a frequency not as high ?
Perhaps it is because Excel does not plot the vertical part of the knee of the curve, since to do so would involve fine discretization of the curve near the cut-off point.   In any case, if not plotted to 4 GHz, it would be wise to double check it and see what whether they look more alike if you plot it up to 4 GHz.

Thanks.  Congratulations again to you persistent digging of Shawyer's literature to understand what he meant by the different variables, particularly lambdg1 and lambdag2.
Title: Re: EM Drive Developments - related to space flight applications - Thread 2
Post by: TheTraveller on 05/14/2015 10:33 pm
Thanks Dr. Rodal for taking the time to point out my error. Fixed now. Get a Df curve like yours that peaks and then goes to zero as the Guide Frequency goes below CutOff Frequency for the small end.

Interesting interactions.

Keeping small end (g2) Guide Wavelength just above CutOff Wavelength, while operating at a Rf frequency which will generate a high Df, adjusting the length to obtain length resonance at the operation Rf frequency.

The interactions are making sense. My gut is not so much in knots. Starting to feel confident in being able to generate at least 1g of force. Calcs say more like 2g but then the cavity needs a Q of around 50,000, which using spherical end caps should help to make a reality.

Oh BTW obtaining length cavity resonance, which I believe is what is seen doing a spectrum sweep, may not generate any thrust as the small end guide wavelength may be bigger than the cutoff wavelength with the result that little or no significant energy reaches the small end to bounce back.

Shields up. Rock throwing time.

Congratulations on fixing it.

I see some differences with my plot using Mathematica.  One minor, immaterial difference is that Excel artificially brings the value of the Design Factor down to zero for frequencies below the cutoff frequency and it artificially draws a vertical line at the cutoff frequency.  Mathematica does not, because the Design Factor does not go down to zero below the cut-off frequency, actually its value becomes a Complex number below the cutoff frequency (since it involves the Square Root of a negative number), and the vertical line shouldn't be there.  In reality there is no Real value of the Design Factor below the cut-off frequency associated with the small diameter.  That's a quirk with Excel.

The other thing is that the curvature in your plot looks different than mine. 
Perhaps it is because my plot goes to 4 GHz and your plot goes to a frequency not as high ?
Perhaps it is because Excel does not plot the vertical part of the knee of the curve, since to do so would involve fine discretization of the curve near the cut-off point.   In any case, if not plotted to 4 GHz, it would be wise to double check it and see what whether they look more alike if you plot it up to 4 GHz.

Thanks.  Congratulations again to you persistent digging of Shawyer's literature to understand what he meant by the different variables, particularly lambdg1 and lambdag2.

I test for the potential of a negative sqrt and if found set the value to zero, so yes for frequencies below cutoff, the Df is forced to zero.

It is interesting playing with Rf frequency & small diameter, which watching how cutoff frequency and guide frequency interplay. I found it is possible to get small end guide frequency to be just above small end cutoff frequency, which is a condition Shawyer says gives good thrust.

Then need to adjust big end diameter to get good Df and then adjust small end to big end spacing to get cavity length resonance.

As Shawyer said, this is not easy, which I believe, and why they developed custom in-house software to make finding optimal parameter mixes quicker and less hunt and peck or cut and try.

What all this tells my gut, is this is real as it all hangs together. Have seen no engineering Red Herrings. BUT and a BIG BUT, there is secret squirrel sauce involved and if you don't get the secret squirrel sauce right, there is no thrust. My job now is to work out the secret squirrel sauce, how to apply it, when and where.
Title: Re: EM Drive Developments - related to space flight applications - Thread 2
Post by: deltaMass on 05/14/2015 10:36 pm
If you happen to be using Excel, I have found the bog standard Solver plug-in very efficacious for problems involving optimisation with several variables. The trick to making it purr is intelligent setting of the constraints.
Title: Re: EM Drive Developments - related to space flight applications - Thread 2
Post by: frobnicat on 05/14/2015 11:01 pm
Today i did the first test with the Emdrive (microwave oven magnetron and cooper frustum)
The setup (magnetron and frusum) was suspended in a pendulum.
I applied power for 40 Seconds with no visible thrust. Tomorrow will will try again with the magnetron on the small side. You have any suggestion for what should be the distance from the small side?
After this i will adjust the power to the filament of magnetron and the frequency.
To fine adjust the frequency i thought i can put 2 coils over the magnetron magnets to modify the magnetic field.
My website;
http://www.masinaelectrica.com/emdrive-independent-test/

https://www.youtube.com/watch?v=2-cgVoLUJ8w

Well done.

Nicely rolled cavity walls.

Have you tried to calibrate your pendulum test rig by using a small spring scale to see how much force is needed to pull the cavity forward (toward the big end) say 1mm?

Doing this will give you some info on how much force you will need to generate to see some movement.

Maybe I should feel ashamed to propose the following calculation with all those heavy weight equations flying around, but since nobody is taking a bite at it :
a hanging swing pendulum like that has, for small deviations, a linear dependency between force (thrust) F and displacement d  F=(m*g/h)*d  where h is length of strings and m the mass of test article and g local gravity. As a first guess, with m=2kg (or more like 5kg ?) and h=2m that's in the ballpark of 10mN/mm (milliNewton per millimetre) or 10µN/µm. Quite remarkably similar to Eagleworks balance apparent stiffness, making this mechanical setup basically as sensitive (displacement wrt thrust wise). If a linear displacement sensor of µm resolution were used it could probe into µN effects, provided proper casing to isolate from drafts and good damping where strings are suspended.

Can you confirm :
- weight of system 2kg, more ?
- height of doorway, or rather length of strings about 2m ?
- graduations marks spacing about 1cm ?

The graduations marks on the video appear about 1cm apart, there is no obvious swing or displacement at power-on visible above, roughly eyeballing 1mm. That gives an upper order of magnitude bound for a thrust (if any) below 10mN/kW for this blazing fast experiment setup. Kudos, and stay safe.
Title: Re: EM Drive Developments - related to space flight applications - Thread 2
Post by: TheTraveller on 05/14/2015 11:02 pm
If you happen to be using Excel, I have found the bog standard Solver plug-in very efficacious for problems involving optimisation with several variables. The trick to making it purr is intelligent setting of the constraints.

Thanks. Will have a look.

Do use "What if Goal Seek" to calc cavity length to get resonance to a desired value (input Rf frequency). Works a charm.
Title: Re: EM Drive Developments - related to space flight applications - Thread 2
Post by: WarpTech on 05/14/2015 11:03 pm


If microwaves were easily contained there wouldn't be a multi billion dollar industry for the testing and consultation of microwave containment.

Seriously, its a big problem. I was amused by this before but when people start using high powered dirty sources to corrupt bands of the spectrum I get angry. I've processed too many Earth Observing land surface experiments that were corrupted and unusable outside of Canada and the US due to unlicensed device usage. It is perplexing that a forum dedicated to spaceflight would encourage such bad form.

We plan to enclose our system in many layers of laminated microwave absorbing sheets:
http://www.lessemf.com/259.pdf

Try putting this on the inside, on the conical wall at the big end, cover about 2 to 4 inches. This should amplify the amount of momentum absorbed in the "forward" direction, for waves that have already fallen out of resonance.

Todd D.
Title: Re: EM Drive Developments - related to space flight applications - Thread 2
Post by: TheTraveller on 05/14/2015 11:13 pm
Today i did the first test with the Emdrive (microwave oven magnetron and cooper frustum)
The setup (magnetron and frusum) was suspended in a pendulum.
I applied power for 40 Seconds with no visible thrust. Tomorrow will will try again with the magnetron on the small side. You have any suggestion for what should be the distance from the small side?
After this i will adjust the power to the filament of magnetron and the frequency.
To fine adjust the frequency i thought i can put 2 coils over the magnetron magnets to modify the magnetic field.
My website;
http://www.masinaelectrica.com/emdrive-independent-test/

https://www.youtube.com/watch?v=2-cgVoLUJ8w

Well done.

Nicely rolled cavity walls.

Have you tried to calibrate your pendulum test rig by using a small spring scale to see how much force is needed to pull the cavity forward (toward the big end) say 1mm?

Doing this will give you some info on how much force you will need to generate to see some movement.

Maybe I should feel ashamed to propose the following calculation with all those heavy weight equations flying around, but since nobody is taking a bite at it :
a hanging swing pendulum like that has, for small deviations, a linear dependency between force (thrust) F and displacement d  F=(m*g/h)*d  where h is length of strings and m the mass of test article and g local gravity. As a first guess, with m=2kg (or more like 5kg ?) and h=2m that's in the ballpark of 10mN/mm (milliNewton per millimetre) or 10µN/µm. Quite remarkably similar to Eagleworks balance apparent stiffness, making this mechanical setup basically as sensitive (displacement wrt thrust wise). If a linear displacement sensor of µm resolution were used it could probe into µN effects, provided proper casing to isolate from drafts and good damping where strings are suspended.

Can you confirm :
- weight of system 2kg, more ?
- height of doorway, or rather length of strings about 2m ?
- graduations marks spacing about 1cm ?

The graduations marks on the video appear about 1cm apart, there is no obvious swing or displacement at power-on visible above, roughly eyeballing 1mm. That gives an upper order of magnitude bound for a thrust (if any) below 10mN/kW for this blazing fast experiment setup. Kudos, and stay safe.

Would suggest 1gf / 10mN would be a really good result from this setup.

Based on his published frustum height dimension of 228.6mm, Rf cavity resonance is 1.311GHz or 2.622GHz, which is a bit high for his magnetron. 244.7mm will give resonance at 2.45GHz. Assuming there are no fudge factors to be applied to parallel plate microwave resonance.
Title: Re: EM Drive Developments - related to space flight applications - Thread 2
Post by: deltaMass on 05/15/2015 12:49 am
The RF feed is going to screw him up, sorry to say
Title: Re: EM Drive Developments - related to space flight applications - Thread 2
Post by: deltaMass on 05/15/2015 12:51 am
Speaking of safety, do glasses exist that are transparent to visible light and attenuate microwaves?
How about clothing? - chain mail mebbe? :)
(http://i.ytimg.com/vi/crX4E-dul4Y/hqdefault.jpg)
Title: Re: EM Drive Developments - related to space flight applications - Thread 2
Post by: Rodal on 05/15/2015 01:09 am
Recent photo of Shawyer standing next to table top EM Drive experiment:

click here for HD picture with details:

http://www.ibtimes.co.uk/emdrive-warp-drive-are-two-different-things-nasas-still-working-emdrive-1501268#slideshow/1436361

(http://d.ibtimes.co.uk/en/full/1436361/roger-shawyer-inventor-emdrive.jpg?w=350)
Title: Re: EM Drive Developments - related to space flight applications - Thread 2
Post by: phaseshift on 05/15/2015 01:26 am
Recent photo of Shawyer standing next to table top EM Drive experiment:

click here for HD picture with details:

http://www.ibtimes.co.uk/emdrive-warp-drive-are-two-different-things-nasas-still-working-emdrive-1501268#slideshow/1436361

(http://d.ibtimes.co.uk/en/full/1436361/roger-shawyer-inventor-emdrive.jpg?w=350)

I see two feeds
Title: Re: EM Drive Developments - related to space flight applications - Thread 2
Post by: WarpTech on 05/15/2015 01:31 am
Speaking of safety, do glasses exist that are transparent to visible light and attenuate microwaves?
How about clothing? - chain mail mebbe? :)
(http://i.ytimg.com/vi/crX4E-dul4Y/hqdefault.jpg)

LOL! A Chainmail body suit would work quite well I think, but only if it weighs the same as a duck. 
Title: Re: EM Drive Developments - related to space flight applications - Thread 2
Post by: Rodal on 05/15/2015 01:46 am
Recent photo of Shawyer standing next to table top EM Drive experiment:

click here for HD picture with details:

http://www.ibtimes.co.uk/emdrive-warp-drive-are-two-different-things-nasas-still-working-emdrive-1501268#slideshow/1436361

(http://d.ibtimes.co.uk/en/full/1436361/roger-shawyer-inventor-emdrive.jpg?w=350)

I see two feeds

One feed a couple of inches from the big end and the other feed a couple of inches from the small end
Title: Re: EM Drive Developments - related to space flight applications - Thread 2
Post by: SeeShells on 05/15/2015 01:51 am
Recent photo of Shawyer standing next to table top EM Drive experiment:

click here for HD picture with details:

http://www.ibtimes.co.uk/emdrive-warp-drive-are-two-different-things-nasas-still-working-emdrive-1501268#slideshow/1436361

(http://d.ibtimes.co.uk/en/full/1436361/roger-shawyer-inventor-emdrive.jpg?w=350)

I see two feeds
Interesting isn't it? We just talked about this a few pages ago.
Title: Re: EM Drive Developments - related to space flight applications - Thread 2
Post by: WarpTech on 05/15/2015 01:53 am
Today i did the first test with the Emdrive (microwave oven magnetron and cooper frustum)
The setup (magnetron and frusum) was suspended in a pendulum.
I applied power for 40 Seconds with no visible thrust. Tomorrow will will try again with the magnetron on the small side. You have any suggestion for what should be the distance from the small side?
After this i will adjust the power to the filament of magnetron and the frequency.
To fine adjust the frequency i thought i can put 2 coils over the magnetron magnets to modify the magnetic field.
My website;
http://www.masinaelectrica.com/emdrive-independent-test/

https://www.youtube.com/watch?v=2-cgVoLUJ8w

Well done.

Nicely rolled cavity walls.

Have you tried to calibrate your pendulum test rig by using a small spring scale to see how much force is needed to pull the cavity forward (toward the big end) say 1mm?

Doing this will give you some info on how much force you will need to generate to see some movement.

Maybe I should feel ashamed to propose the following calculation with all those heavy weight equations flying around, but since nobody is taking a bite at it :
a hanging swing pendulum like that has, for small deviations, a linear dependency between force (thrust) F and displacement d  F=(m*g/h)*d  where h is length of strings and m the mass of test article and g local gravity. As a first guess, with m=2kg (or more like 5kg ?) and h=2m that's in the ballpark of 10mN/mm (milliNewton per millimetre) or 10µN/µm. Quite remarkably similar to Eagleworks balance apparent stiffness, making this mechanical setup basically as sensitive (displacement wrt thrust wise). If a linear displacement sensor of µm resolution were used it could probe into µN effects, provided proper casing to isolate from drafts and good damping where strings are suspended.

Can you confirm :
- weight of system 2kg, more ?
- height of doorway, or rather length of strings about 2m ?
- graduations marks spacing about 1cm ?

The graduations marks on the video appear about 1cm apart, there is no obvious swing or displacement at power-on visible above, roughly eyeballing 1mm. That gives an upper order of magnitude bound for a thrust (if any) below 10mN/kW for this blazing fast experiment setup. Kudos, and stay safe.

Would suggest 1gf / 10mN would be a really good result from this setup.

Based on his published frustum height dimension of 228.6mm, Rf cavity resonance is 1.311GHz or 2.622GHz, which is a bit high for his magnetron. 244.7mm will give resonance at 2.45GHz. Assuming there are no fudge factors to be applied to parallel plate microwave resonance.

I think Shawyer is weighing it, which may be a better setup because the results can be easily quantified. 10 mN would affect its weight measurably, and if a balance were used, the accuracy could be very high.

Todd D.
Title: Re: EM Drive Developments - related to space flight applications - Thread 2
Post by: LasJayhawk on 05/15/2015 01:54 am
Recent photo of Shawyer standing next to table top EM Drive experiment:

click here for HD picture with details:

http://www.ibtimes.co.uk/emdrive-warp-drive-are-two-different-things-nasas-still-working-emdrive-1501268#slideshow/1436361

(http://d.ibtimes.co.uk/en/full/1436361/roger-shawyer-inventor-emdrive.jpg?w=350)

I see two feeds
one looks like a feed from the signal generator, the other looks like it is going to a power sensor.
Title: Re: EM Drive Developments - related to space flight applications - Thread 2
Post by: deltaMass on 05/15/2015 02:08 am
It looks like 3.5<something> GHz and I also see "FM". Hmm.
I think LasJayhawk is right about the topmost cable being power sensing
Title: Re: EM Drive Developments - related to space flight applications - Thread 2
Post by: LasJayhawk on 05/15/2015 02:11 am
There is an HP 848xx power sensor just to the left of the scales.
Title: Re: EM Drive Developments - related to space flight applications - Thread 2
Post by: phaseshift on 05/15/2015 02:13 am
It looks like 3.5<something> GHz and I also see "FM". Hmm.
I think LasJayhawk is right about the topmost cable being power sensing

He is using an RF generator very similar to this one, goes to 3.2GHz.

http://www.sglabs.it/en/product.php?s=hp-hewlett-packard-agilent-8648c&id=690

The D model goes a little higher.
Title: Re: EM Drive Developments - related to space flight applications - Thread 2
Post by: deltaMass on 05/15/2015 02:17 am
Maybe 3.250 GHz. And the "FM" might simply be "AM"
Title: Re: EM Drive Developments - related to space flight applications - Thread 2
Post by: LasJayhawk on 05/15/2015 02:27 am
Looks like 3650.3 MHz, 3KHz fm at a 1K rate. But the generators RF out is " off"
Title: Re: EM Drive Developments - related to space flight applications - Thread 2
Post by: phaseshift on 05/15/2015 02:31 am
Maybe 3.250 GHz. And the "FM" might simply be "AM"

I blew up the image and I'm pretty sure it says: 3850.36000MHz  FM 3.00kHz -40.0dBm
Title: Re: EM Drive Developments - related to space flight applications - Thread 2
Post by: zen-in on 05/15/2015 02:40 am
Recent photo of Shawyer standing next to table top EM Drive experiment:

click here for HD picture with details:

http://www.ibtimes.co.uk/emdrive-warp-drive-are-two-different-things-nasas-still-working-emdrive-1501268#slideshow/1436361

(http://d.ibtimes.co.uk/en/full/1436361/roger-shawyer-inventor-emdrive.jpg?w=350)

The large blue cross shaped device with the humungous waveguide dummy load is a C band directional coupler.   The type N connector on the rightmost arm would be connected with coax to more attenuators and then a power meter.   Beneath it and obscured by the Agilent  frequency generator is what looks like a C band waveguide switch, for directing the RF to the directional coupler or to the blue waveguide to BNC section with coax attached that is feeding the cavity.   It's possible the device behind the Agilent signal generator may be the business end of a C band TWT but there isn't much room there so this might just be a staged photo-op.
Title: Re: EM Drive Developments - related to space flight applications - Thread 2
Post by: deltaMass on 05/15/2015 02:43 am
An electronic balance isn't a terribly smart idea. But as has been said, this is probably a staged demo.
Title: Re: EM Drive Developments - related to space flight applications - Thread 2
Post by: SeeShells on 05/15/2015 03:23 am
Recent photo of Shawyer standing next to table top EM Drive experiment:

click here for HD picture with details:

http://www.ibtimes.co.uk/emdrive-warp-drive-are-two-different-things-nasas-still-working-emdrive-1501268#slideshow/1436361

(http://d.ibtimes.co.uk/en/full/1436361/roger-shawyer-inventor-emdrive.jpg?w=350)

The large blue cross shaped device with the humungous waveguide dummy load is a C band directional coupler.   The type N connector on the rightmost arm would be connected with coax to more attenuators and then a power meter.   Beneath it and obscured by the Agilent  frequency generator is what looks like a C band waveguide switch, for directing the RF to the directional coupler or to the blue waveguide to BNC section with coax attached that is feeding the cavity.   It's possible the device behind the Agilent signal generator may be the business end of a C band TWT but there isn't much room there so this might just be a staged photo-op.

It also looks like the end plates maybe flat.
Title: Re: EM Drive Developments - related to space flight applications - Thread 2
Post by: deltaMass on 05/15/2015 03:29 am
It looks smaller than EW's frustum, and the frequency therefore makes sense.
Title: Re: EM Drive Developments - related to space flight applications - Thread 2
Post by: Iulian Berca on 05/15/2015 03:30 am
Today i did the first test with the Emdrive (microwave oven magnetron and cooper frustum)
The setup (magnetron and frusum) was suspended in a pendulum.
I applied power for 40 Seconds with no visible thrust. Tomorrow will will try again with the magnetron on the small side. You have any suggestion for what should be the distance from the small side?
After this i will adjust the power to the filament of magnetron and the frequency.
To fine adjust the frequency i thought i can put 2 coils over the magnetron magnets to modify the magnetic field.
My website;
http://www.masinaelectrica.com/emdrive-independent-test/

https://www.youtube.com/watch?v=2-cgVoLUJ8w

Well done.

Nicely rolled cavity walls.

Have you tried to calibrate your pendulum test rig by using a small spring scale to see how much force is needed to pull the cavity forward (toward the big end) say 1mm?

Doing this will give you some info on how much force you will need to generate to see some movement.

Maybe I should feel ashamed to propose the following calculation with all those heavy weight equations flying around, but since nobody is taking a bite at it :
a hanging swing pendulum like that has, for small deviations, a linear dependency between force (thrust) F and displacement d  F=(m*g/h)*d  where h is length of strings and m the mass of test article and g local gravity. As a first guess, with m=2kg (or more like 5kg ?) and h=2m that's in the ballpark of 10mN/mm (milliNewton per millimetre) or 10µN/µm. Quite remarkably similar to Eagleworks balance apparent stiffness, making this mechanical setup basically as sensitive (displacement wrt thrust wise). If a linear displacement sensor of µm resolution were used it could probe into µN effects, provided proper casing to isolate from drafts and good damping where strings are suspended.

Can you confirm :
- weight of system 2kg, more ?
- height of doorway, or rather length of strings about 2m ?
- graduations marks spacing about 1cm ?

The graduations marks on the video appear about 1cm apart, there is no obvious swing or displacement at power-on visible above, roughly eyeballing 1mm. That gives an upper order of magnitude bound for a thrust (if any) below 10mN/kW for this blazing fast experiment setup. Kudos, and stay safe.

The distance between lines is 1cm, tonight i will measure the height and the weight.
If i blow air with my mouth on the frustum, i`m able to visible deviate the pendulum.
I did the same think to a smal scale and i recorded  ~ 1 grame of push
Title: Re: EM Drive Developments - related to space flight applications - Thread 2
Post by: TheTraveller on 05/15/2015 06:08 am
It is an unplated Flight Thruster. Large red feed at bottom is 3.85GHz Rf input from TWTA and small upper coax is E field sense to variable frequency Rf generator.

Dr. Rodal how did you learn of the article? Did Shawyer give you a heads up?

The data on the laptop, attached, is the Flight Thruster mN to Watts input curve that was published.
Title: Re: EM Drive Developments - related to space flight applications - Thread 2
Post by: TheTraveller on 05/15/2015 06:34 am
An electronic balance isn't a terribly smart idea. But as has been said, this is probably a staged demo.

Read what Shawyer has written about how he measures thrust using a balance beam and electronic scales. BTW that Flight Thruster would be an excellent Faraday Cage, so don't think much Rf is leaking out.

http://emdrive.com/feasibilitystudy.html He used the same test rig to record data from his Flight Thruster.

That same scale and HP freq generator is being used in the superconducting experiments.
Title: Re: EM Drive Developments - related to space flight applications - Thread 2
Post by: TheTraveller on 05/15/2015 06:40 am
An electronic balance isn't a terribly smart idea. But as has been said, this is probably a staged demo.

Would be at some event. Note the award in his hands and smile on his face. Probably around 2010 when he licensed his IP to Boeing via a deal arranged between the US and UK governments.
Title: Re: EM Drive Developments - related to space flight applications - Thread 2
Post by: TheTraveller on 05/15/2015 06:42 am
Looks like 3650.3 MHz, 3KHz fm at a 1K rate. But the generators RF out is " off"

Manual: http://www.naic.edu/~phil/hardware/synth/hp8648/agilent8648OperationAndServiceGuide.pdf

Display layout attached. Cleaned up the display best I could.

It may be that if the Rf output amplifier is disabled (as it is), the freq & modulation / deviation circuits still work and display what is on the output but now input coax. Which would make sense as the Rf is being fed into the Flight Thruster vis the bottom red coax, from the TWTA switched output.

All good info for me as I can now set 3.85GHz as a reference frequency and play with the 3 Flight Thruster dimensions to be optimal at 3.85GHz. So one variable MAYBE known.

From that can calc the height to get resonance between the curved end plates. Then 2 variables are locked down.

That will leave only the big and small end plate dimensions to work on to get best Df at 3.85GHz.

Title: Re: EM Drive Developments - related to space flight applications - Thread 2
Post by: Iulian Berca on 05/15/2015 07:31 am
Yesterday night i did a new test with the Magnetron moved to the small side (10cm from the small side). I patched the previous hole.
I also put a coil around one magnet in hope to change the frequency.
I ordered a frequency counter, so i will now exactly what is the frequency produced and the intensity.

No pendulum movement was observed. The duration of the test was ~40 sec.

In the future tests i will be able to observe any change in frequency by modifying the current in the coil
If this will not change the frequency i will modify the frustum to add a moving plate inside.

https://www.youtube.com/watch?v=UbStna1-XZU
Title: Re: EM Drive Developments - related to space flight applications - Thread 2
Post by: TheTraveller on 05/15/2015 07:36 am
Does the Flight Thruster have a slightly concave top and convex bottom? Would appear so from the gaps.

Enhanced the photo as much as I can for those wishing to try to extract dimensions as this photo is better that the original as it has no distortion.

If we can find the dimensions of the bottom Rf connector flange, we can set pixels per cm and start doing measurements.
Title: Re: EM Drive Developments - related to space flight applications - Thread 2
Post by: TheTraveller on 05/15/2015 07:39 am
Yesterday night i did a new test with the Magnetron moved to the small side (10cm from the small side). I patched the previous hole.
I also put a coil around one magnet in hope to change the frequency.
I ordered a frequency counter, so i will now exactly what is the frequency produced and the intensity.

No pendulum movement was observed. The duration of the test was ~40 sec.

In the future tests i will be able to observe any change in frequency by modifying the current in the coil
If this will not change the frequency i will modify the frustum to add a moving plate inside.

Moving a small internal end plate back and forth was what Shawyer seemed to do to get his cavity into length resonance.

As per attached.

Would suspect the movable small end plate disc was very near or at the right end of the cylinder as in the drawing.
Title: Re: EM Drive Developments - related to space flight applications - Thread 2
Post by: Iulian Berca on 05/15/2015 08:28 am
Yesterday night i did a new test with the Magnetron moved to the small side (10cm from the small side). I patched the previous hole.
I also put a coil around one magnet in hope to change the frequency.
I ordered a frequency counter, so i will now exactly what is the frequency produced and the intensity.

No pendulum movement was observed. The duration of the test was ~40 sec.

In the future tests i will be able to observe any change in frequency by modifying the current in the coil
If this will not change the frequency i will modify the frustum to add a moving plate inside.

Moving a small internal end plate back and forth was what Shawyer seemed to do to get his cavity into length resonance.

As per attached.

Would suspect the movable small end plate was very near or at the right end of the cylinder as in the drawing.

Yes this is the disk i`m refering. I read almos all this thread and first one, and almost all the papers related to em drive from www.emdrive.com
I bought this counter,i should receive it in a few days.


 https://www.youtube.com/watch?v=OUo76oEt1yM.


I want to know if i can change the magnetron frequency. It`s working up to 2,6Ghz and it also has signal strength indicator, so i can check for leaks.
Here you can buy from ebay.
http://www.ebay.com/itm/Black-Mini-Frequency-Counter-IBQ101-Handheld-LCD-Display-HoT-Counter-/171299666945?_trksid=p2054897.l4275

Title: Re: EM Drive Developments - related to space flight applications - Thread 2
Post by: TheTraveller on 05/15/2015 08:37 am
Yesterday night i did a new test with the Magnetron moved to the small side (10cm from the small side). I patched the previous hole.
I also put a coil around one magnet in hope to change the frequency.
I ordered a frequency counter, so i will now exactly what is the frequency produced and the intensity.

No pendulum movement was observed. The duration of the test was ~40 sec.

In the future tests i will be able to observe any change in frequency by modifying the current in the coil
If this will not change the frequency i will modify the frustum to add a moving plate inside.

Moving a small internal end plate back and forth was what Shawyer seemed to do to get his cavity into length resonance.

As per attached.

Would suspect the movable small end plate was very near or at the right end of the cylinder as in the drawing.

Yes this is the disk i`m refering. I read almos all this thread and first one, and almost all the papers related to em drive from www.emdrive.com
I bought this counter,i should receive it in a few days.


 https://www.youtube.com/watch?v=OUo76oEt1yM.


I want to know if i can change the magnetron frequency. It`s working up to 2,6Ghz and it also has signal strength indicator, so i can check for leaks.
Here you can buy from ebay.
http://www.ebay.com/itm/Black-Mini-Frequency-Counter-IBQ101-Handheld-LCD-Display-HoT-Counter-/171299666945?_trksid=p2054897.l4275

Shawyer said his video camera had sync issues because of the high E field the EM Drive generates. Said the rotary demo generated the equivalent of a 17MW E field being the Rf input energy times the cavity Q. 378W Rf * Q of 45,000 = 17MWs.

That is why I'm building an external Faraday Cage around my Flight Thruster replication and around the Rf amp. Not so much to protect against being microwave cooked but against interference with my data collection and logging system.
Title: Re: EM Drive Developments - related to space flight applications - Thread 2
Post by: redliox on 05/15/2015 08:42 am
So an EM drive is basically a magnetron that shoots out microwaves?  Possibly useful since it doesn't need fuel, but I just wonder what kind of power is required.  Any expectations yet?

I'd like to see how this would eventually compare with the ion drive.  That would be it's first competitor.
Title: Re: EM Drive Developments - related to space flight applications - Thread 2
Post by: TheTraveller on 05/15/2015 08:52 am
So an EM drive is basically a magnetron that shoots out microwaves?  Possibly useful since it doesn't need fuel, but I just wonder what kind of power is required.  Any expectations yet?

I'd like to see how this would eventually compare with the ion drive.  That would be it's first competitor.

Please read:
http://emdrive.com
http://www.nasaspaceflight.com/2015/04/evaluating-nasas-futuristic-em-drive/

No microwaves leave the Em Drive, other than small unintentional leakage. Thrust is produced INSIDE the sealed cavity.
Title: Re: EM Drive Developments - related to space flight applications - Thread 2
Post by: Iulian Berca on 05/15/2015 12:07 pm
Today i did the first test with the Emdrive (microwave oven magnetron and cooper frustum)
The setup (magnetron and frusum) was suspended in a pendulum.
I applied power for 40 Seconds with no visible thrust. Tomorrow will will try again with the magnetron on the small side. You have any suggestion for what should be the distance from the small side?
After this i will adjust the power to the filament of magnetron and the frequency.
To fine adjust the frequency i thought i can put 2 coils over the magnetron magnets to modify the magnetic field.
My website;
http://www.masinaelectrica.com/emdrive-independent-test/

https://www.youtube.com/watch?v=2-cgVoLUJ8w

Well done.

Nicely rolled cavity walls.

Have you tried to calibrate your pendulum test rig by using a small spring scale to see how much force is needed to pull the cavity forward (toward the big end) say 1mm?

Doing this will give you some info on how much force you will need to generate to see some movement.

Maybe I should feel ashamed to propose the following calculation with all those heavy weight equations flying around, but since nobody is taking a bite at it :
a hanging swing pendulum like that has, for small deviations, a linear dependency between force (thrust) F and displacement d  F=(m*g/h)*d  where h is length of strings and m the mass of test article and g local gravity. As a first guess, with m=2kg (or more like 5kg ?) and h=2m that's in the ballpark of 10mN/mm (milliNewton per millimetre) or 10µN/µm. Quite remarkably similar to Eagleworks balance apparent stiffness, making this mechanical setup basically as sensitive (displacement wrt thrust wise). If a linear displacement sensor of µm resolution were used it could probe into µN effects, provided proper casing to isolate from drafts and good damping where strings are suspended.

Can you confirm :
- weight of system 2kg, more ?
- height of doorway, or rather length of strings about 2m ?
- graduations marks spacing about 1cm ?

The graduations marks on the video appear about 1cm apart, there is no obvious swing or displacement at power-on visible above, roughly eyeballing 1mm. That gives an upper order of magnitude bound for a thrust (if any) below 10mN/kW for this blazing fast experiment setup. Kudos, and stay safe.

Would suggest 1gf / 10mN would be a really good result from this setup.

Based on his published frustum height dimension of 228.6mm, Rf cavity resonance is 1.311GHz or 2.622GHz, which is a bit high for his magnetron. 244.7mm will give resonance at 2.45GHz. Assuming there are no fudge factors to be applied to parallel plate microwave resonance.

I think Shawyer is weighing it, which may be a better setup because the results can be easily quantified. 10 mN would affect its weight measurably, and if a balance were used, the accuracy could be very high.

Todd D.

The weight of my setup is 5.9Kg and the height is 1.9 meters.
Title: Re: EM Drive Developments - related to space flight applications - Thread 2
Post by: Rodal on 05/15/2015 12:21 pm
Have modified my Shawyer Df calculator and best Df scanner as per the derived Shawyer Df equation, using cutoff wavelength and guide wavelength as per microwave industry supplied equations. I assume Shawyer did not supply these equations in his papers as they are equations that should be known to microwave industry individuals skilled in the art. Anyway they are now in the public record.

The scanner still sweeps the frequency range 0Hz to 10GHz but reports the frequency that generates a Df as close to 1 as possible but not over.

The attached results are very interesting as the frequency needed to get the Df to just below 1 is very close to the Rf driving frequency used to generate Lambda0 or free wavelength in the selected medium.

While I'm still testing the spreadsheet, which meets both of Shawyers boundary conditions, the results for my Flight Thruster design are looking to be very close to what I could build. Bit of dimension tweaking should get the Df 1 frequency to the 3.85GHz Shawyer used.

Will post the spreadsheet after a bit more testing.

This is a picture for the FLIGHT THRUSTER case

bD=0.2440 m ;sD=0.1450 m ;cMedium=299705000 m/s (Air)

which has a cut-off frequency associated with the small diameter of 1.21136 GHz

Notice that there is a singularity at 1.21136 GHz such that Shawyer's Design Factor doesn't have a Real value for frequencies below it.  Also notice the rise and steepening of the Design Factor curve as the cut-off frequency is approached.

QUESTION: If Shawyer thinks that his Design Factor steepening behavior near the cut-off frequency associated with the small diameter is correct, why didn't he test his Flight Thruster at a lower frequency, closer to 1.2 GHz instead of the higher frequency he chose of 3.782 GHz? Doesn't Shawyer want to maximize thrust force ?

3D Plot of Shawyer's Design Factor vs. frequency and vs. small diameter; for same big diameter as Flight Thruster, but with the small diameter ranging from zero to same size as big diameter.

Remember: according to Shawyer the Design Factor multiplies the Power Input and the Q. The higher the Design Factor, the higher the thrust of the EM Drive, the smaller the Design Factor, the smaller the thrust.

Observe that at high frequency, the Design Factor changes almost linearly with small diameter, such that the Design Factor goes to zero as the small diameter approaches the big diameter.

The Design Factor approaches 1 for the small diameter approaching zero.

As the small diameter approaches zero, the cut-off frequency clips the Design Factor, such that to be able to have a smaller small diameter one has to operate at higher frequency (in order to avoid cut-off).

A very nice feature of Shawyer's Design Factor (as opposed to McCulloch's formula) is that Shawyer's Design Factor incorporates the cut-off frequency and hence it prevents consideration of a pointy cone, as the cut-off prevents too small of a small diameter to be considered.

The highest value of the Design Factor is reached at frequencies just a little over the cut-off frequency for the small end:

Cut-Off frequency for small end= cM/(cst*sD)   

where

sD= small end diameter (m)
cst=1.7062895542683174
cM = light speed in selected medium (m/s)
     = 299705000 (m/s) (speed of light in air)
     = 299792458 (m/s) (speed of light in vacuum)

Observe that for sD -> 0  (for the small diameter approaching zero, a pointy cone), the cut-Off frequency for the small end goes to Infinity, hence according to Shawyer's Design Factor the EM Drive cannot have a pointy cone.  It must be truncated.

The Design Factor has little dependence on frequency, except near the cut-off frequency. The Design Factor of Shawyer asymptotically approaches this value for high frequencies (it becomes practically independent of frequency)

Limit[DesignFactor, f -> Infinity]  = (bD^2 - sD^2)/(bD^2 + sD^2)

where

bD = big end diameter (m)
sD= small end diameter (m)

Whether Shawyer's Design Factor is correct, remains to be proven.  For example, Shawyer's Design Factor predicts that the smaller the small diameter the better (hence larger cone angles, for constant frustum length), in contrast with Todd's conjecture that the highest attenuation the better (which leads to small cone angles ~7.5 degrees as the optimal design).



Reference: formula for Design Factor here:  http://forum.nasaspaceflight.com/index.php?topic=36313.msg1374110#msg1374110

designFactor =
           (bD^2 - sD^2)/( (bD^2)*Sqrt[1 - (cM/(bD*cst*f))^2] + (sD ^2)*Sqrt[1 - (cM/(cst*f*sD))^2] )

bD = big end diameter (m)
sD= small end diameter (m)
f = applied frequency (Hz)
cst=1.7062895542683174
cM = light speed in selected medium (m/s)
     = 299705000 (m/s) (speed of light in air)
     = 299792458 (m/s) (speed of light in vacuum)
Title: Re: EM Drive Developments - related to space flight applications - Thread 2
Post by: frobnicat on 05/15/2015 01:13 pm
... linear dependency between force (thrust) F and displacement d  F=(m*g/h)*d  where h is length of strings and m the mass of test article and g local gravity.
....

Would suggest 1gf / 10mN would be a really good result from this setup.

Based on his published frustum height dimension of 228.6mm, Rf cavity resonance is 1.311GHz or 2.622GHz, which is a bit high for his magnetron. 244.7mm will give resonance at 2.45GHz. Assuming there are no fudge factors to be applied to parallel plate microwave resonance.

I think Shawyer is weighing it, which may be a better setup because the results can be easily quantified. 10 mN would affect its weight measurably, and if a balance were used, the accuracy could be very high.

Todd D.

The weight of my setup is 5.9Kg and the height is 1.9 meters.

So the apparent stiffness is more like 30mN/mm : 3 grams force for 1mm visible deviation. If the cavity were experiencing thrust/power ratio similar to some of Shawyer or Yang et al this would be visible though (almost a cm at the Chinese record 288mN/kW).

The power cord setup is far from ideal, maybe you could lead it along one of the suspending wire, starting from the top fixation points, with as thin/supple as possible wires. Also it should be possible to record displacements down to 1/10mm (3mN = .3gf) with a thin needle flying close above millimetre graph paper and filming closer, in macro mode or through magnifying lens ?
Title: Re: EM Drive Developments - related to space flight applications - Thread 2
Post by: IRobot on 05/15/2015 01:44 pm
The power cord setup is far from ideal, maybe you could lead it along one of the suspending wire, starting from the top fixation points, with as thin/supple as possible wires. Also it should be possible to record displacements down to 1/10mm (3mN = .3gf) with a thin needle flying close above millimetre graph paper and filming closer, in macro mode or through magnifying lens ?
Point a fixed laser to a small slit on the device. Place a target on the other side of the room. The diffraction pattern should change with microscopic variations of position.

EDIT: the slit should probably be slightly tilted so the distance between the target and the slit changes with position.
Title: Re: EM Drive Developments - related to space flight applications - Thread 2
Post by: txdrive on 05/15/2015 02:12 pm
The Design Factor has little dependence on frequency, except near the cut-off frequency. The Design Factor of Shawyer asymptotically approaches this value for high frequencies (it becomes practically independent of frequency)
Do people in this thread even realize that this would boil down to a contradiction with radiation pressure measurements dating back over 100 years?
Title: Re: EM Drive Developments - related to space flight applications - Thread 2
Post by: Rodal on 05/15/2015 02:17 pm
The Design Factor has little dependence on frequency, except near the cut-off frequency. The Design Factor of Shawyer asymptotically approaches this value for high frequencies (it becomes practically independent of frequency)
Do people in this thread even realize that this would boil down to a contradiction with radiation pressure measurements dating back over 100 years?
Do you "even realize" that people in this thread already realized this, and this is addressed earlier in this thread, in a much more comprehensive and technical manner?  ;)

http://forum.nasaspaceflight.com/index.php?topic=36313.msg1372044#msg1372044

http://forum.nasaspaceflight.com/index.php?topic=36313.msg1369861#msg1369861

There are numerous posts, where Shawyer's formulation is criticized point by point, not just historically, but on a scientific basis.

The point of displaying Shawyer's Design Factor is not an endorsement of his formulation, but it is a scientific/technical way to further understand and comment on what it implies, to be able to compare it with experimental data and with physical laws.  Physics for hundreds of years has been a science whose language is mathematics instead of words.  How else is one going to be able to adequately assess Shawyer's formulation (or any other formulation) unless his mathematical equations are displayed, and analyzed?
Title: Re: EM Drive Developments - related to space flight applications - Thread 2
Post by: hhexo on 05/15/2015 02:29 pm
I have analysed the case of the frustum and the results appear to be striking. One must admit that geometry comes to rescue not just general relativity. For this particular geometry the cavity can be made susceptible to gravitational effects if your choice of the two radii of the cavity is smart enough. This is something to be confirmed yet, just my theoretical result, but shocking anyway.

I have read the draft and although I can't vouch for the maths (my skills are way below what is required - as soon as tensors are involved I have to give up) the concepts do make sense to me. This is most definitely interesting and it would be a great result if we could measure this gravitational effect with an interferometer setup.

I have a question: if there is a gravitational effect in the cavity, how is this effect distributed? Is it symmetric? And what is the entity of it, quantitatively?

I am reasoning that if there is an asymmetry in the gravitational effect and the cavity is filled with a perfect gas (or even just air), then in principle there will be a pressure gradient within the cavity causing a (very small?) average density gradient.
In absence of an external gravity field, this is irrelevant. And certainly it would not generate any thrust on its own...
However, if this density gradient is immersed in a gravity field (like Earth's) the apparatus will weigh more on one side than the other. This will generate a (very very small?) torque trying to twist the apparatus until the gradient aligns with the gravity field.

Could this torque create a displacement affecting the experiment in a way that can be confused with thrust? Could the experiment be susceptible to this particular condition? (I'm thinking that that pendulum-suspended EmDrive looks like it would be affected by differences in the weight distribution of the device)

One more reason to perform experiments in a vacuum, I guess...
Title: Re: EM Drive Developments - related to space flight applications - Thread 2
Post by: StrongGR on 05/15/2015 02:39 pm
I have analysed the case of the frustum and the results appear to be striking. One must admit that geometry comes to rescue not just general relativity. For this particular geometry the cavity can be made susceptible to gravitational effects if your choice of the two radii of the cavity is smart enough. This is something to be confirmed yet, just my theoretical result, but shocking anyway.

I have read the draft and although I can't vouch for the maths (my skills are way below what is required - as soon as tensors are involved I have to give up) the concepts do make sense to me. This is most definitely interesting and it would be a great result if we could measure this gravitational effect with an interferometer setup.

I have a question: if there is a gravitational effect in the cavity, how is this effect distributed? Is it symmetric? And what is the entity of it, quantitatively?

I am reasoning that if there is an asymmetry in the gravitational effect and the cavity is filled with a perfect gas (or even just air), then in principle there will be a pressure gradient within the cavity causing a (very small?) average density gradient.
In absence of an external gravity field, this is irrelevant. And certainly it would not generate any thrust on its own...
However, if this density gradient is immersed in a gravity field (like Earth's) the apparatus will weigh more on one side than the other. This will generate a (very very small?) torque trying to twist the apparatus until the gradient aligns with the gravity field.

Could this torque create a displacement affecting the experiment in a way that can be confused with thrust? Could the experiment be susceptible to this particular condition? (I'm thinking that that pendulum-suspended EmDrive looks like it would be affected by differences in the weight distribution of the device)

One more reason to perform experiments in a vacuum, I guess...

The main result of the paper is that the gravitational effects are proportional to the energy density of the electromagnetic field inside the cavity (roughly speaking). This means that the electromagnetic field inside the cavity determines the way the gravitational field is distributed. A change could be due to the proportionality factor L(x). Of course, there could be an effect due to the Earth gravitational field but I did not estimate it yet.
Title: Re: EM Drive Developments - related to space flight applications - Thread 2
Post by: TheTraveller on 05/15/2015 03:15 pm
QUESTION: If Shawyer thinks that his Design Factor steepening behavior near the cut-off frequency associated with the small diameter is correct, why didn't he test his Flight Thruster at a lower frequency, closer to 1.2 GHz instead of the higher frequency he chose of 3.782 GHz? Doesn't Shawyer want to maximize thrust force ?

Reading the history of the SPR Flight Thruster, it was a design/build project for Boeing. I suspect the 3.85GHz dual redundant variable Rf source and 300W TWTA are from some space rated project and Boeing wanted the Flight Thruster to be driven from it?

If so, the Flight Thruster may have been a design constrained by the clients selection of the input frequency and maybe max dimensions and mass?
Title: Re: EM Drive Developments - related to space flight applications - Thread 2
Post by: TheTraveller on 05/15/2015 03:49 pm
3D Plot of Shawyer's Design Factor vs. frequency and vs. small diameter; for same big diameter as Flight Thruster, but with the small diameter ranging from zero to same size as big diameter.

Remember: according to Shawyer the Design Factor multiplies the Power Input and the Q. The higher the Design Factor, the higher the thrust of the EM Drive, the smaller the Design Factor, the smaller the thrust.

Observe that at high frequency, the Design Factor changes almost linearly with small diameter, such that the Design Factor goes to zero as the small diameter approaches the big diameter.

The Design Factor approaches 1 for the small diameter approaching zero.

As the small diameter approaches zero, the cut-off frequency clips the Design Factor, such that to be able to have a smaller small diameter one has to operate at higher frequency (in order to avoid cut-off).

A very nice feature of Shawyer's Design Factor (as opposed to McCulloch's formula) is that Shawyer's Design Factor incorporates the cut-off frequency and hence it prevents consideration of a pointy cone, as the cut-off prevents too small of a small diameter to be considered.

The highest value of the Design Factor is reached at frequencies just a little over the cut-off frequency for the small end:

Cut-Off frequency for small end= cM/(cst*sD)   

where

sD= small end diameter (m)
cst=1.7062895542683174
cM = light speed in selected medium (m/s)
     = 299705000 (m/s) (speed of light in air)
     = 299792458 (m/s) (speed of light in vacuum)


The Design Factor has little dependence on frequency, except near the cut-off frequency. The Design Factor of Shawyer asymptotically approaches this value for high frequencies (it becomes practically independent of frequency)

Limit[DesignFactor, f -> Infinity]  = (bD^2 - sD^2)/(bD^2 + sD^2)

where

bD = big end diameter (m)
sD= small end diameter (m)

Whether Shawyer's Design Factor is correct, remains to be proven.  For example, Shawyer's Design Factor predicts that the smaller the small diameter the better (hence larger cone angles, for constant frustum length), in contrast with Todd's conjecture that the highest attenuation the better (which leads to small cone angles ~7.5 degrees as the optimal design).



Reference: formula for Design Factor here:  http://forum.nasaspaceflight.com/index.php?topic=36313.msg1374110#msg1374110

designFactor =
           (bD^2 - sD^2)/( (bD^2)*Sqrt[1 - (cM/(bD*cst*f))^2] + (sD ^2)*Sqrt[1 - (cM/(cst*f*sD))^2] )

bD = big end diameter (m)
sD= small end diameter (m)
f = applied frequency (Hz)
cst=1.7062895542683174
cM = light speed in selected medium (m/s)
     = 299705000 (m/s) (speed of light in air)
     = 299792458 (m/s) (speed of light in vacuum)

Very interesting plot. Was working Excel to do the same, but why reinvent the wheel? Nice data.

One element we have yet to nut out is external applied Rf cavity resonance between the 2 end plates. Is vital for this to happen, as without resonance to applied Rf frequency, there will be no thrust developed as Q will be very low.

Simple to calc wavelength 1/2 wavelength between and call it done but with constantly variable guide wavelength between the end plates, my gut says end plate resonance is not 1/2 of Lambda0 (free) as Lambda0 only exists outside the cavity, nor Lambda1 (big end) guide wavelength nor Lambda2 (small end) guide wavelength. Additionally guide wavelength varies continuously, at each point of diameter change, from one end of the cavity to the other.

So how to calc the external Rf frequency required to bring the cavity into end plate to end plate resonance and allow Q to grow very large?

Of course all this assumes any external waveguide / coax is tuned (SWR 1:1) to supply Rf energy with minimal loss.

It is important as Shawyer says in the 1st attachment.

In the 2nd attachment there is notes an interesting object built into the side of the cavity.

It is used as in the 3rd attachment to assist the cavity getting into resonance?

Title: Re: EM Drive Developments - related to space flight applications - Thread 2
Post by: hhexo on 05/15/2015 04:08 pm
The main result of the paper is that the gravitational effects are proportional to the energy density of the electromagnetic field inside the cavity (roughly speaking). This means that the electromagnetic field inside the cavity determines the way the gravitational field is distributed. A change could be due to the proportionality factor L(x). Of course, there could be an effect due to the Earth gravitational field but I did not estimate it yet.

So, if I understand correctly, L(x) is purely a function of the position "x vector" obtained by a defined volume integral, and is only dependent on the geometry of the cavity and the l0 (ell zero) constant which depends on U0 which depends on the source energy.

In Equation 37, given a defined geometry of the frustum, "a" is later determined to be a constant term, "b" is also a constant term, so the shape of the function depends on the logarithm and the square of "r vector".
However Equation 38 sets a condition by which "r vector" is linear to "z vector". Does this mean that "r vector" is a function of z, i.e. it varies with the position on the height axis of the frustum?
Am I understanding it correctly?

If that is the case, then the gravitational effect is stronger towards one plate than the other. Which leads to my concerns about a torque being present rather than a thrust. Presumably, in an experiment the frustum is suspended or supported in such a way that a static balance is achieved; however if a torque is present the balance is altered and this could potentially be confused for a thrust depending on how the mechanical parts of the experiment are set up.
(I've realised that there could be a torque even in vacuum because you can't remove the existence of the frustum itself, and its weight distribution could change with the gravitational effect)

But again, I'm not 100% sure I'm understanding this correctly. I'm just throwing ideas for the smarter people to think about. I hope this is useful.
Title: Re: EM Drive Developments - related to space flight applications - Thread 2
Post by: StrongGR on 05/15/2015 04:55 pm
The main result of the paper is that the gravitational effects are proportional to the energy density of the electromagnetic field inside the cavity (roughly speaking). This means that the electromagnetic field inside the cavity determines the way the gravitational field is distributed. A change could be due to the proportionality factor L(x). Of course, there could be an effect due to the Earth gravitational field but I did not estimate it yet.

So, if I understand correctly, L(x) is purely a function of the position "x vector" obtained by a defined volume integral, and is only dependent on the geometry of the cavity and the l0 (ell zero) constant which depends on U0 which depends on the source energy.

In Equation 37, given a defined geometry of the frustum, "a" is later determined to be a constant term, "b" is also a constant term, so the shape of the function depends on the logarithm and the square of "r vector".
However Equation 38 sets a condition by which "r vector" is linear to "z vector". Does this mean that "r vector" is a function of z, i.e. it varies with the position on the height axis of the frustum?
Am I understanding it correctly?

If that is the case, then the gravitational effect is stronger towards one plate than the other. Which leads to my concerns about a torque being present rather than a thrust. Presumably, in an experiment the frustum is suspended or supported in such a way that a static balance is achieved; however if a torque is present the balance is altered and this could potentially be confused for a thrust depending on how the mechanical parts of the experiment are set up.
(I've realised that there could be a torque even in vacuum because you can't remove the existence of the frustum itself, and its weight distribution could change with the gravitational effect)

But again, I'm not 100% sure I'm understanding this correctly. I'm just throwing ideas for the smarter people to think about. I hope this is useful.

Yes, I have to impose the condition that I am living in a frustum to solve my equation for L(x). So, you can express it both as function of r or z. Please, note that l0 (ell zero) is very large making some contributions less important than others. Besides, contributions from Earth gravitational field are damped by a factor (Schwarzschild radius)/(Earth radius) where the Schwarzschild radius of the Earth is 9 mm while the Earth radius is about 6000 km. This factor being 10^(-12) makes negligible the effect with respect to the behaviour of the electromagnetic field inside the cavity.
Title: Re: EM Drive Developments - related to space flight applications - Thread 2
Post by: TheTraveller on 05/15/2015 04:56 pm
Went back to investigate what I reported in post #233 http://forum.nasaspaceflight.com/index.php?topic=36313.msg1318683#msg1318683 about the RF and Microwave Toolbox app. I found that the app is reporting the correct solutions for TE and TM. The help file just had a typo. I verified it against the KWOK lectures http://www.engr.sjsu.edu/rkwok/EE172/Cavity_Resonator.pdf slide 16. KWOK and the APP match. So this works as a quick and easy way to find resonant modes! There really is an app for everything.

I remain unconvinced that calculating resonant modes for cylinders is a good approximation for conical frustums though.

The problem as I see it with the cylindrical resonate model to the (in progress) frustum resonate model is in the cylinder model the guide wavelength / frequency stays the same from one end to the other, while in the frustum, the guide wavelength / frequency continually varies as the diameter varies.

As example in your frustum big end = 0.2797m dia & small = 0.1588m dia and 2.45GHz applied Rf frequency.

Guide wavelengths / frequencies:
 big end = 0.12654m / 2.368,462,699Ghz
 small end = 0.13703m / 2.187,156,644GHz
 diff (big-small) = -0.010490m / 0.181,306,055GHz

Data attached.

These are guide wavelength / frequency and NOT cut off wavelength/frequencies. At 2.45GHz, both of your ends operate well above cutoff.

So for your frustum, operating at 2.45GHZ, there is a 181.3MHz difference in the guide frequencies at each end.

Trying to imagine what this would look like. Can easily see a cylindrical model with constant guide wavelength from end to end.

I may be wrong but as I see it cavity resonance occurs at the internal guide wavelength and not at the external applied Rf wavelength. We know the guide frequency at each end of the frustum, so how to get the applied Rf frequency that causes end plate to end plate resonance at 1/2 wave or other harmonic of some internal guide frequency?
Title: Re: EM Drive Developments - related to space flight applications - Thread 2
Post by: hhexo on 05/15/2015 05:25 pm
Besides, contributions from Earth gravitational field are damped by a factor (Schwarzschild radius)/(Earth radius) where the Schwarzschild radius of the Earth is 9 mm while the Earth radius is about 6000 km. This factor being 10^(-12) makes negligible the effect with respect to the behaviour of the electromagnetic field inside the cavity.

I see! So there may be an effect but the entity would be too small compared with the values in the experiments.

Thanks for your answers!
Title: Re: EM Drive Developments - related to space flight applications - Thread 2
Post by: LasJayhawk on 05/15/2015 05:42 pm
Does the Flight Thruster have a slightly concave top and convex bottom? Would appear so from the gaps.

Enhanced the photo as much as I can for those wishing to try to extract dimensions as this photo is better that the original as it has no distortion.

If we can find the dimensions of the bottom Rf connector flange, we can set pixels per cm and start doing measurements.

Most N connectors like that are 1" square, and the holes are .718" center line to center line.

Pixel away. :)
Title: Re: EM Drive Developments - related to space flight applications - Thread 2
Post by: Rodal on 05/15/2015 05:45 pm
Went back to investigate what I reported in post #233 http://forum.nasaspaceflight.com/index.php?topic=36313.msg1318683#msg1318683 about the RF and Microwave Toolbox app. I found that the app is reporting the correct solutions for TE and TM. The help file just had a typo. I verified it against the KWOK lectures http://www.engr.sjsu.edu/rkwok/EE172/Cavity_Resonator.pdf slide 16. KWOK and the APP match. So this works as a quick and easy way to find resonant modes! There really is an app for everything.

I remain unconvinced that calculating resonant modes for cylinders is a good approximation for conical frustums though.

The problem as I see it with the cylindrical resonate model to the (in progress) frustum resonate model is in the cylinder model the guide wavelength / frequency stays the same from one end to the other, while in the frustum, the guide wavelength / frequency continually varies as the diameter varies.

As example in your frustum big end = 0.2797m dia & small = 0.1588m dia and 2.45GHz applied Rf frequency.

Guide wavelengths / frequencies:
 big end = 0.12654m / 2.368,462,699Ghz
 small end = 0.13703m / 2.187,156,644GHz
 diff (big-small) = -0.010490m / 0.181,306,055GHz

Data attached.

These are guide wavelength / frequency and NOT cut off wavelength/frequencies. At 2.45GHz, both of your ends operate well above cutoff.

So for your frustum, operating at 2.45GHZ, there is a 181.3MHz difference in the guide frequencies at each end.

Trying to imagine what this would look like. Can easily see a cylindrical model with constant guide wavelength from end to end.

I may be wrong but as I see it cavity resonance occurs at the internal guide wavelength and not at the external applied Rf wavelength. We know the guide frequency at each end of the frustum, so how to get the applied Rf frequency that causes end plate to end plate resonance at 1/2 wave or other harmonic of some internal guide frequency?

You can get a reasonably close approximation to the exact expression for the natural frequencies of a truncated cone by using the exact expression for a cylindrical cavity (see:  http://en.wikipedia.org/wiki/Microwave_cavity#Cylindrical_cavity where c/Sqrt[μr εr ] is the speed of light in the medium.  You will need a table of zeros of cylindrical Bessel function and its derivatives Xmn and X'mn.  This is the best table I have found:  http://wwwal.kuicr.kyoto-u.ac.jp/www/accelerator/a4/besselroot.htmlx  )

and use the Root mean square (http://en.wikipedia.org/wiki/Root_mean_square)  diameter as a diameter in the expression for the cylindrical cavity:

Root Mean Square diameter = Sqrt[(1/2) (bigDiameter^2 + smallDiameter^2)]

The use of the Root Mean Square (RMS) diameter is in the same spirit as Shawyer's expression for the Design Factor http://forum.nasaspaceflight.com/index.php?topic=36313.msg1374110#msg1374110 (which in the limit for infinite frequency, contains the square of the Root Mean Square diameter in its denominator:

Limit[DesignFactor, f -> Infinity]  = (bigDiameter^2 - smallDiameter^2)/(bigDiameter^2 + smallDiameter^2)
                                                  = ((bigDiameter^2 - smallDiameter^2)/2)/(RMSdiameter^2)

), and gives results closer to the exact solution (see comparison below).



For example, for a truncated cone with the dimensions used by NASA Eagleworks,

Actual geometry
Large OD : 11.00 " (0.2794m),
Small OD: 6.25" (0.1588 m)
Length : 9.00 " (0.2286m)
speed of light in medium= c/Sqrt[μr εr] = 299705000 (m/s) (speed of light in air)

TM212 mode

truncated cone exact solution = 2.49342 GHz
cylindrical solution using RMS diameter = 2.52343 GHz
(difference with exact solution: 1.2% )
cylindrical solution using Mean diameter = 2.59234 GHz (difference with exact solution: 4.0% )
cylindrical solution using Geometric Mean diameter =2.67031 GHz (difference with exact solution: 7.1% )
Title: Re: EM Drive Developments - related to space flight applications - Thread 2
Post by: TheTraveller on 05/15/2015 06:08 pm
You can get a reasonably close approximation to the exact expression for the natural frequencies of a truncated cone

What is the "exact expression for the natural frequencies of a truncated cone"?

Why go with close estimates?
Title: Re: EM Drive Developments - related to space flight applications - Thread 2
Post by: TheTraveller on 05/15/2015 06:10 pm
Does the Flight Thruster have a slightly concave top and convex bottom? Would appear so from the gaps.

Enhanced the photo as much as I can for those wishing to try to extract dimensions as this photo is better that the original as it has no distortion.

If we can find the dimensions of the bottom Rf connector flange, we can set pixels per cm and start doing measurements.

Most N connectors like that are 1" square, and the holes are .718" center line to center line.

Pixel away. :)

Thanks. Have fine rotated to vertical / horizontal and lined up. Attached if anyone else wants to have a go.
Title: Re: EM Drive Developments - related to space flight applications - Thread 2
Post by: Rodal on 05/15/2015 06:14 pm
You can get a reasonably close approximation to the exact expression for the natural frequencies of a truncated cone

What is the "exact expression for the natural frequencies of a truncated cone"?

Why go with close estimates?
The exact expression for the solution to Maxwell's equations in a truncated cone involves the solution of two eigenvalue problems, one in terms of Legendre Associated functions and the other one in terms of spherical Bessel functions.  Take a gander at this http://gregegan.customer.netspace.net.au/SCIENCE/Cavity/Cavity.html for an exact expression (Egan only gives the exact solution for TE0np and TM0np modes, not valid for TM212 for example), so that you can see that it would be unwieldly to program it in Excel.

Minotti also gives the exact solution to Maxwell's equations in a truncated cone (section 4 of http://arxiv.org/pdf/1302.5690v3 ) quoting Greg Egan.  The exact solution goes back to the 1930's in papers by S.A. Schelkunoff of Bell Labs.

Here is a picture of Schelkunoff examining some conical cavities in the early 1930’s.
(Bad Joke coming: They seem to be tied down, maybe that's why Bell Labs didn't report any thrust from conical cavities  :) )
(Image from P. C. Mahon, Mission Communications: The Story of Bell Laboratories, 1975.)
(http://www.ieeecincinnati.org/wp-content/uploads/2010/03/2010_04-fig-1.JPG)
Title: Re: EM Drive Developments - related to space flight applications - Thread 2
Post by: deltaMass on 05/15/2015 07:13 pm
The competition
http://nextbigfuture.com/2015/05/photonic-laser-thruster-has-moved-one.html
Title: Re: EM Drive Developments - related to space flight applications - Thread 2
Post by: WarpTech on 05/15/2015 07:25 pm
The competition
http://nextbigfuture.com/2015/05/photonic-laser-thruster-has-moved-one.html

That's what I'm talking about! Something like this would be at least 20% more efficient than an EM Drive could ever be. IMO, no competition...

Todd D.
Title: Re: EM Drive Developments - related to space flight applications - Thread 2
Post by: Notsosureofit on 05/15/2015 07:30 pm
The competition
http://nextbigfuture.com/2015/05/photonic-laser-thruster-has-moved-one.html

Extremely rough but a cavity at 100KW and 400 Q should be ~14X that force...
Title: Re: EM Drive Developments - related to space flight applications - Thread 2
Post by: lasoi on 05/15/2015 07:34 pm
Does the Flight Thruster have a slightly concave top and convex bottom? Would appear so from the gaps.

Enhanced the photo as much as I can for those wishing to try to extract dimensions as this photo is better that the original as it has no distortion.

If we can find the dimensions of the bottom Rf connector flange, we can set pixels per cm and start doing measurements.

Most N connectors like that are 1" square, and the holes are .718" center line to center line.

Pixel away. :)

Thanks. Have fine rotated to vertical / horizontal and lined up. Attached if anyone else wants to have a go.
If you tell me specifically what measure you are trying to find, and what measures you know with regards to the photo, I might be able to help.
Title: Re: EM Drive Developments - related to space flight applications - Thread 2
Post by: aceshigh on 05/15/2015 07:38 pm
The competition
http://nextbigfuture.com/2015/05/photonic-laser-thruster-has-moved-one.html

That's what I'm talking about! Something like this would be at least 20% more efficient than an EM Drive could ever be. IMO, no competition...

Todd D.

how to stop the spaceship with this?
Title: Re: EM Drive Developments - related to space flight applications - Thread 2
Post by: Rodal on 05/15/2015 07:47 pm
The competition
http://nextbigfuture.com/2015/05/photonic-laser-thruster-has-moved-one.html

That's what I'm talking about! Something like this would be at least 20% more efficient than an EM Drive could ever be. IMO, no competition...

Todd D.

The concept of the EM Drive (if the EM Drive is ever possible) is self-contained while the Photonic Thruster is limited to the distance to the Resource Vehicle, at the present time:

Quote from: http://www.spaceref.com/news/viewpr.html?pid=45847
I can see future development that includes optical cavities that span many kilometers achieved with precise mirror alignment to enable maneuvering spacecraft many kilometers apart, and propellant-free propulsion of satellites in formations.

(http://www.photonics.com/images/Web/Articles/2013/10/8/thumbnail_55064.jpg)

Although Bae has written about the concept being used for Interstellar Travel:

http://ykbcorp.com/downloads/Bae_photon_propulsion_STAIF2_Paper_Circulation.pdf

using an "Interstellar Photonic Railway"
Title: Re: EM Drive Developments - related to space flight applications - Thread 2
Post by: deltaMass on 05/15/2015 07:55 pm
The competition
http://nextbigfuture.com/2015/05/photonic-laser-thruster-has-moved-one.html

That's what I'm talking about! Something like this would be at least 20% more efficient than an EM Drive could ever be. IMO, no competition...

Todd D.
How do you figure that?
Title: Re: EM Drive Developments - related to space flight applications - Thread 2
Post by: TheTraveller on 05/15/2015 07:57 pm
Does the Flight Thruster have a slightly concave top and convex bottom? Would appear so from the gaps.

Enhanced the photo as much as I can for those wishing to try to extract dimensions as this photo is better that the original as it has no distortion.

If we can find the dimensions of the bottom Rf connector flange, we can set pixels per cm and start doing measurements.

Most N connectors like that are 1" square, and the holes are .718" center line to center line.

Pixel away. :)

Thanks. Have fine rotated to vertical / horizontal and lined up. Attached if anyone else wants to have a go.
If you tell me specifically what measure you are trying to find, and what measures you know with regards to the photo, I might be able to help.

The size of the rectangular section the Flight Thruster is sitting on and the size of the Rf connector flange and mounting hole spacing should be knowable.

Assuming the build has the concave and convex spherical section machined in the top and bottom plates, the internal cavity height is then the distance from the underside lip of the top plate to the upper lip of the bottom plate.

Looking at the surface of the Flight Thruster, it looks either cast or machined but not rolled, so assume a 1mm thick wall. Should then be able to determine the internal diameter at the bottom and top. Like lifting off the end caps and measuring the inside top and bottom diameters.

Here is an enhanced, better resolution than the original, image of the Flight Thruster to work from. I found the image had to be rotated 1.5 deg CCW to bring the bottom alum beam into horizontal. I can do that for you if you need.

BTW thanks. Your efforts on working out the dimension from the 1st Flight Thruster photo were fantastic. This non distorted and higher resolution image, with more knowable dimensions, should be easier to work with.

Updated image with a better one.

Looking at the image again, there may be copper end plates that have the curve and those end plates are sandwiched between a upper and lower flange that is part of the Flight Thruster case and bolted on end caps that are also internally curved to allow the copper end plates curves to exits without being damaged.

I do however note the bottom end cap doesn't appear to be sitting flush on the bottom beam, maybe it has a convex curve on the bottom and likewise the top end cap appears to be concaved.

Have attached my effort at making up the Flight Thruster. Note it is rotated 1.5 deg CCW.
Title: Re: EM Drive Developments - related to space flight applications - Thread 2
Post by: deuteragenie on 05/15/2015 08:00 pm
For the fun ot it: the animated version of the MEEP pic I posted before.
Title: Re: EM Drive Developments - related to space flight applications - Thread 2
Post by: deltaMass on 05/15/2015 08:10 pm
Quote
The size of the rectangular section the Flight Thruster is sitting on and the size of the Rf connector flange and mounting hole spacing should be knowable.
The one "known" is the 1" N connector. Notice that it is slightly off-centre because 1) the top is more distant than the bottom (frustum angle) and 2) it does not lie exactly face-on to the image plane, but rather a little around the circumference to the left. Fortunately we can correct for both of these small changes fairly exactly.
1) The frustum angle is directly measurable
2) Since both circumferential edges are visible, the degree of rotation is directly computable.
Having converted the "one inch, modified" to pixels it's off to the races.
Title: Re: EM Drive Developments - related to space flight applications - Thread 2
Post by: rfmwguy on 05/15/2015 08:21 pm
Yesterday night i did a new test with the Magnetron moved to the small side (10cm from the small side). I patched the previous hole.
I also put a coil around one magnet in hope to change the frequency.
I ordered a frequency counter, so i will now exactly what is the frequency produced and the intensity.

No pendulum movement was observed. The duration of the test was ~40 sec.

In the future tests i will be able to observe any change in frequency by modifying the current in the coil
If this will not change the frequency i will modify the frustum to add a moving plate inside.

Moving a small internal end plate back and forth was what Shawyer seemed to do to get his cavity into length resonance.

As per attached.

Would suspect the movable small end plate was very near or at the right end of the cylinder as in the drawing.

Yes this is the disk i`m refering. I read almos all this thread and first one, and almost all the papers related to em drive from www.emdrive.com
I bought this counter,i should receive it in a few days.


 https://www.youtube.com/watch?v=OUo76oEt1yM.


I want to know if i can change the magnetron frequency. It`s working up to 2,6Ghz and it also has signal strength indicator, so i can check for leaks.
Here you can buy from ebay.
http://www.ebay.com/itm/Black-Mini-Frequency-Counter-IBQ101-Handheld-LCD-Display-HoT-Counter-/171299666945?_trksid=p2054897.l4275

Be cautious of these broadband Frequency counters. My pesonal experience is that they may register a spur frequency because it could be more sensitive in the UHF range as opposed to over 2 GHz. The only accurate way to analyze the Fc of the magnetron would be either a Spectrum Analyzer or filtered frequency counter. Magnetrons are loaded with many frequencies.
Title: Re: EM Drive Developments - related to space flight applications - Thread 2
Post by: lasoi on 05/15/2015 08:53 pm
Okay, I think I can give you the numbers you need.  But I'm not sure I understand you.  Are you saying you want the diameters of the end plates and the height of the cavity?  And if you want the height of the cavity, do you mean excluding the concavities in the endcaps since we don't know their depth?

Also, if the 1" N connector is there in front, where is the RF connector and mounting hole?  Is that the same as the N connector?  If so, where does it measure 1", specifically?

And are you saying you also want the width of the thin metal beam the entire thruster is sitting upon?

I don't mean to be obtuse, but I'm not an engineer and this stuff is largely foreign to me.

Title: Re: EM Drive Developments - related to space flight applications - Thread 2
Post by: SeeShells on 05/15/2015 08:58 pm
The search function aren't that super here euuuuu.

Why is the RF injected into the side of the EM Device. I know it's a basic question but can someone expand on it a little more?

Shells
Title: Re: EM Drive Developments - related to space flight applications - Thread 2
Post by: deltaMass on 05/15/2015 09:15 pm
About Bae's PLT:
The technique will come into its own when alignment over at least one million Km is feasible.

Now you'll notice that they were in a clean room (albeit in air) so that should give pause to anyone considering using this as a launch system for extremely light payloads directly from Earth's surface.  Nevertheless, a little noodling would not go amiss on this topic.

The available motive power is Q*P (Q=200, P=800 W in the video), so the force F = 2*Q*P/c (1.1 mN in the video), so the acceleration a = F/m (m=0.45 Kg, a = 2.5 mm/s2 in the video).

What would it take to get up to 1 gee for an Earth-based launch?
The acceleration needs to be increased by a factor of ~4000x.
Putting this all together we get
a = 2 Q P / (m c)
Leaving m alone for the moment, Bae states that Q could improve by a factor ~5x (200->1000).
Now we need 4000/5 = 800x improvement.
Using a 800 kW laser does that for us (1000x).
Alternatively we can use a lower mass and thus a lower power laser.
Title: Re: EM Drive Developments - related to space flight applications - Thread 2
Post by: deltaMass on 05/15/2015 09:22 pm
The search function aren't that super here euuuuu.
To be blunt, it totally sucks :(
Title: Re: EM Drive Developments - related to space flight applications - Thread 2
Post by: jmossman on 05/15/2015 09:32 pm
The search function aren't that super here euuuuu.

Why is the RF injected into the side of the EM Device. I know it's a basic question but can someone expand on it a little more?

Shells

No one else has chimed in, so I'll give this a shot.   ;)

I believe the choice of antenna position is predominantly a function of the antenna beam pattern, and the desire to couple maximum energy into the cavity.

A simple dipole antenna radiates/couples well in the perpendicular direction, so placing a dipole antenna perpendicular to the cavity wall would allow direct coupling into the dominant resonant direction (i.e.  between the concave/convex end plates).

(attached image from http://www.trevormarshall.com/byte_articles/byte1.htm (http://www.trevormarshall.com/byte_articles/byte1.htm))

I had proposed (many pages back) that the use of a waveguide to inject a magnetron's signal had the effect of a directional beam pattern that was much better at injecting energy than removing energy from the cavity. (since resonanting energy is dominantly between the end plates, a waveguide input roughly perpendicular to the walls would inject energy better than remove energy)  However, I'll readily admit my reasoning may be overly simplistic.

Thanks,
James
Title: Re: EM Drive Developments - related to space flight applications - Thread 2
Post by: Rodal on 05/15/2015 09:48 pm
About Bae's PLT:
The technique will come into its own when alignment over at least one million Km is feasible.

Now you'll notice that they were in a clean room (albeit in air) so that should give pause to anyone considering using this as a launch system for extremely light payloads directly from Earth's surface.  Nevertheless, a little noodling would not go amiss on this topic.

The available motive power is Q*P (Q=200, P=800 W in the video), so the force F = 2*Q*P/c (1.1 mN in the video), so the acceleration a = F/m (m=0.45 Kg, a = 2.5 mm/s2 in the video).

What would it take to get up to 1 gee for an Earth-based launch?
The acceleration needs to be increased by a factor of ~4000x.
Putting this all together we get
a = 2 Q P / (m c)
Leaving m alone for the moment, Bae states that Q could improve by a factor ~5x (200->1000).
Now we need 4000/5 = 800x improvement.
Using a 800 kW laser does that for us (1000x).
Alternatively we can use a lower mass and thus a lower power laser.

See:  http://ykbcorp.com/downloads/Bae_photon_propulsion_STAIF2_Paper_Circulation.pdf
Title: Re: EM Drive Developments - related to space flight applications - Thread 2
Post by: deltaMass on 05/15/2015 10:16 pm
Props to anyone who spots the error in my calculation
Title: Re: EM Drive Developments - related to space flight applications - Thread 2
Post by: phaseshift on 05/15/2015 11:59 pm
Does the Flight Thruster have a slightly concave top and convex bottom? Would appear so from the gaps.

Enhanced the photo as much as I can for those wishing to try to extract dimensions as this photo is better that the original as it has no distortion.

If we can find the dimensions of the bottom Rf connector flange, we can set pixels per cm and start doing measurements.

Most N connectors like that are 1" square, and the holes are .718" center line to center line.

Pixel away. :)

Thanks. Have fine rotated to vertical / horizontal and lined up. Attached if anyone else wants to have a go.

Using the enhanced photo I built a Google SketchUp model such that when overlayed with the image it matches.  Then I scaled the model so that the RF connector plate measured 1 inch along one side.  The resulting cone has dimensions of sD: 122.2mm, bD: 223.8mm, L: 153.98mm. Not sure the margin of error but the numbers should look fairly close I hope.
Title: Re: EM Drive Developments - related to space flight applications - Thread 2
Post by: zellerium on 05/16/2015 12:06 am
Random thought but;

1-  Could someone please try an em cavity with the bottom (large curved end) not electrically bonded to the sidewalls & top. ie place a circular insulating gasket between the contact point of the sidewall bottom edge and the actual bottom curved plate,  just the wall/plate boundary NOT covering the internal surface area of the curved plate.

What do you think will be different if the bottom plate is electrically insulated?

I think we could easily incorporate this into our design, we have planned to leave a small clearance between the bottom movable plate and the frustum. Our bottom plate may not be as curved as Shawyer's, but we'll be able to provide a small amount of curvature by tighting the screws to different lengths.
Title: Re: EM Drive Developments - related to space flight applications - Thread 2
Post by: deltaMass on 05/16/2015 12:15 am
Accurate to about 5% = 1-cos(18o). Could be better were you to use my observation about the distortion in the 1" dimensions.
Title: Re: EM Drive Developments - related to space flight applications - Thread 2
Post by: aero on 05/16/2015 12:21 am
I have this, but don't know what the small end chord, large end chord, or the perpendicular distance between the chords should be, in terms of wavelengths. Or the drive frequency for that matter.

What is your antenna like. A centered dipole (vertical in the images) works well, and the longer the better it seems to me.
Title: Re: EM Drive Developments - related to space flight applications - Thread 2
Post by: phaseshift on 05/16/2015 12:23 am
Accurate to about 5% = 1-cos(18o). Could be better were you to use my observation about the distortion in the 1" dimensions.

It's an actual 3D Model and the projection lines up to a pixel, so the 'distortion' should be accounted for.
Title: Re: EM Drive Developments - related to space flight applications - Thread 2
Post by: phaseshift on 05/16/2015 12:25 am
Random thought but;

1-  Could someone please try an em cavity with the bottom (large curved end) not electrically bonded to the sidewalls & top. ie place a circular insulating gasket between the contact point of the sidewall bottom edge and the actual bottom curved plate,  just the wall/plate boundary NOT covering the internal surface area of the curved plate.

What do you think will be different if the bottom plate is electrically insulated?

I think we could easily incorporate this into our design, we have planned to leave a small clearance between the bottom movable plate and the frustum. Our bottom plate may not be as curved as Shawyer's, but we'll be able to provide a small amount of curvature by tighting the screws to different lengths.

After building the 3D-Model I'm 'fairly certain' there is a rubber gasket between the end plates and cone.  It's 1/16 of an inch thick in the model which reflects what I see in the image.
Title: Re: EM Drive Developments - related to space flight applications - Thread 2
Post by: phaseshift on 05/16/2015 12:29 am
A little more detail - hidden edges can be seen in the xray rendering.
Title: Re: EM Drive Developments - related to space flight applications - Thread 2
Post by: Notsosureofit on 05/16/2015 12:34 am
Random thought but;

1-  Could someone please try an em cavity with the bottom (large curved end) not electrically bonded to the sidewalls & top. ie place a circular insulating gasket between the contact point of the sidewall bottom edge and the actual bottom curved plate,  just the wall/plate boundary NOT covering the internal surface area of the curved plate.

What do you think will be different if the bottom plate is electrically insulated?

I think we could easily incorporate this into our design, we have planned to leave a small clearance between the bottom movable plate and the frustum. Our bottom plate may not be as curved as Shawyer's, but we'll be able to provide a small amount of curvature by tighting the screws to different lengths.

After building the 3D-Model I'm 'fairly certain' there is a rubber gasket between the end plates and cone.  It's 1/16 of an inch thick in the model which reflects what I see in the image.

Typically, it would be a copper gasket w/ bevels on the plate edges, but rubber o-rings could be used.  In any event, they would be used to make it vacuum tight.  (the copper gaskets are ~1/16")
Title: Re: EM Drive Developments - related to space flight applications - Thread 2
Post by: Iulian Berca on 05/16/2015 12:38 am
The search function aren't that super here euuuuu.

Why is the RF injected into the side of the EM Device. I know it's a basic question but can someone expand on it a little more?

Shells

It was first question in my mind also. Why not on top or bottom ?
Title: Re: EM Drive Developments - related to space flight applications - Thread 2
Post by: Rodal on 05/16/2015 12:57 am
I am back with an updated draft after some terrible news around about NASA dismissing these researches. They should not as, otherwise, it could happen as with Galilei having his detractors even not trying to look in the telescope, just dismissing on faith.

I have analysed the case of the frustum and the results appear to be striking. One must admit that geometry comes to rescue not just general relativity. For this particular geometry the cavity can be made susceptible to gravitational effects if your choice of the two radii of the cavity is smart enough. This is something to be confirmed yet, just my theoretical result, but shocking anyway.

As usual, any comment is very welcome.
In page 8, equation 34 of http://forum.nasaspaceflight.com/index.php?action=dlattach;topic=36313.0;attach=830137,
for the integral on dr', should the limits, instead of

0 to ((r2-r1)/h) z' + r2

be

r1 to ((r2-r1)/h) z' + r1 ?
Title: Re: EM Drive Developments - related to space flight applications - Thread 2
Post by: Notsosureofit on 05/16/2015 01:02 am
I am back with an updated draft after some terrible news around about NASA dismissing these researches. They should not as, otherwise, it could happen as with Galilei having his detractors even not trying to look in the telescope, just dismissing on faith.

I have analysed the case of the frustum and the results appear to be striking. One must admit that geometry comes to rescue not just general relativity. For this particular geometry the cavity can be made susceptible to gravitational effects if your choice of the two radii of the cavity is smart enough. This is something to be confirmed yet, just my theoretical result, but shocking anyway.

As usual, any comment is very welcome.
In page 8, equation 34 of http://forum.nasaspaceflight.com/index.php?action=dlattach;topic=36313.0;attach=830137,
for the integral on dr', should the limits, instead of

0 to ((r2-r1)/h) z' + r2

be

r1 to ((r2-r1)/h) z' + r1 ?

Good Point !  But volume integral ?     0 to ...    ((r2-r1)/h) z' + r1 /l0  ?  (seems circular that way)

It's Eq. 18 that still bothers me a bit.  Invoking the Heaviside step function is OK, but I don't see the addl. components being detectable outside the cavity w/o a non-linear term.  Maybe I'm missing something ?

Title: Re: EM Drive Developments - related to space flight applications - Thread 2
Post by: LasJayhawk on 05/16/2015 01:08 am
The search function aren't that super here euuuuu.

Why is the RF injected into the side of the EM Device. I know it's a basic question but can someone expand on it a little more?

Shells
Beats me. And it looks like the placement would put part of the big end in the near near field and loading the bejesus out of the source.

I've also noted they use a simple loop antenna, but there seems to be no consideration to what we called at Collins the "look angle"
Title: Re: EM Drive Developments - related to space flight applications - Thread 2
Post by: deltaMass on 05/16/2015 01:14 am
Collins used to make top line ham receivers back in the day
Title: Re: EM Drive Developments - related to space flight applications - Thread 2
Post by: ThinkerX on 05/16/2015 01:17 am
Quote
The search function aren't that super here euuuuu.

Basically, you have to use a defined google search to find past items of interest on this thread. 
Title: Re: EM Drive Developments - related to space flight applications - Thread 2
Post by: txdrive on 05/16/2015 01:39 am
Have modified my Shawyer Df calculator and best Df scanner as per the derived Shawyer Df equation, using cutoff wavelength and guide wavelength as per microwave industry supplied equations. I assume Shawyer did not supply these equations in his papers as they are equations that should be known to microwave industry individuals skilled in the art. Anyway they are now in the public record.

The scanner still sweeps the frequency range 0Hz to 10GHz but reports the frequency that generates a Df as close to 1 as possible but not over.

The attached results are very interesting as the frequency needed to get the Df to just below 1 is very close to the Rf driving frequency used to generate Lambda0 or free wavelength in the selected medium.

While I'm still testing the spreadsheet, which meets both of Shawyers boundary conditions, the results for my Flight Thruster design are looking to be very close to what I could build. Bit of dimension tweaking should get the Df 1 frequency to the 3.85GHz Shawyer used.

Will post the spreadsheet after a bit more testing.

I've built and tested many microwave cavities over many years.

You're guided wavelength equation is wrong, because this is for a rectangular wave guide (i.e., not even a rectangular cavity)

You need to derive mode of frequency yourself (unless there is a paper somewhere) for a circular tapered cavity. There is no other way around it. I would start with Balanis - Advanced Engineering Electromagnetics as he derives a few examples for other topologies. Right now everything you are doing is wrong because you don't understand the physics. I would study that book from front to cover if I were you.


Also, to the guy operating the microwave magnetron outside of the microwave: STOP
At best you are violating the laws of your local government's regulatory committee for the electromagnetic spectrum. At worst you will damage your body. At this frequency, the damage is somewhat insidious. Due to low water content of your skin, you don't feel the heat, but internal nerve endings can be damaged so that chronic phantom pain can appear. Sometimes days after exposure. Please STOP otherwise you will inevitably be reported to your government.
The biggest danger, in my understanding, is cooking your eyes. The thermal conductivity is very low and there are no pain receptors.

This thread, the posts some people are making here, have a potential to actually, physically hurt some people, to cause an irreversible vision loss.

There had been a result of ~50uN from ~2.6 watts. This is a safe power level which can be obtained with low voltage components, and this is a force that should be measurable to sub 1% accuracy (in spite of the heating) with a torsion pendulum consisting of a rod suspended from the ceiling on a metal string, in still air, without requiring an external power feed.
Title: Re: EM Drive Developments - related to space flight applications - Thread 2
Post by: deltaMass on 05/16/2015 01:52 am
Yes. And the way to silence sceptics is to make measurements on a fully-boxed rig - including battery. All these trailing feeds generate pseudo-forces. That would be OK if thrust were Newton level - but it ain't, so it ain't.
Title: Re: EM Drive Developments - related to space flight applications - Thread 2
Post by: A_M_Swallow on 05/16/2015 02:42 am
Yes. And the way to silence sceptics is to make measurements on a fully-boxed rig - including battery. All these trailing feeds generate pseudo-forces. That would be OK if thrust were Newton level - but it ain't, so it ain't.

The Dynamic Tests of the EM Drive showed a slowly rotating table top of machinery. So sufficient force was being produced to move the equipment 2 metres in a few minutes.
Title: Re: EM Drive Developments - related to space flight applications - Thread 2
Post by: SeeShells on 05/16/2015 02:46 am
The search function aren't that super here euuuuu.

Why is the RF injected into the side of the EM Device. I know it's a basic question but can someone expand on it a little more?

Shells

No one else has chimed in, so I'll give this a shot.   ;)

I believe the choice of antenna position is predominantly a function of the antenna beam pattern, and the desire to couple maximum energy into the cavity.

A simple dipole antenna radiates/couples well in the perpendicular direction, so placing a dipole antenna perpendicular to the cavity wall would allow direct coupling into the dominant resonant direction (i.e.  between the concave/convex end plates).

(attached image from http://www.trevormarshall.com/byte_articles/byte1.htm (http://www.trevormarshall.com/byte_articles/byte1.htm))

I had proposed (many pages back) that the use of a waveguide to inject a magnetron's signal had the effect of a directional beam pattern that was much better at injecting energy than removing energy from the cavity. (since resonanting energy is dominantly between the end plates, a waveguide input roughly perpendicular to the walls would inject energy better than remove energy)  However, I'll readily admit my reasoning may be overly simplistic.

Thanks,
James
Thanks James!
Know the pattern well and that pattern is in free space, but it makes me wonder sitting in the EM funnel shaped chamber what kind of witches brew of harmonic and sub-harmonic patterns it would make. I've just downloaded some software and I'm going to try my hand at seeing what I end up with.
Title: Re: EM Drive Developments - related to space flight applications - Thread 2
Post by: Rodal on 05/16/2015 03:06 am
I am back with an updated draft after some terrible news around about NASA dismissing these researches. They should not as, otherwise, it could happen as with Galilei having his detractors even not trying to look in the telescope, just dismissing on faith.

I have analysed the case of the frustum and the results appear to be striking. One must admit that geometry comes to rescue not just general relativity. For this particular geometry the cavity can be made susceptible to gravitational effects if your choice of the two radii of the cavity is smart enough. This is something to be confirmed yet, just my theoretical result, but shocking anyway.

As usual, any comment is very welcome.
In page 8, equation 34 of http://forum.nasaspaceflight.com/index.php?action=dlattach;topic=36313.0;attach=830137,
for the integral on dr', should the limits, instead of

0 to ((r2-r1)/h) z' + r2

be

r1 to ((r2-r1)/h) z' + r1 ?

Good Point !  But volume integral ?     0 to ...    ((r2-r1)/h) z' + r1 /l0  ?  (seems circular that way)

It's Eq. 18 that still bothers me a bit.  Invoking the Heaviside step function is OK, but I don't see the addl. components being detectable outside the cavity w/o a non-linear term.  Maybe I'm missing something ?

1) Good point about Eq. 18: how do those harmonic components make it outside the cavity?  Do they go outside the cavity because of the small transparent portholes on the EM Drive through which the interferometer laser beam is going through?

2) If one wants to nondimensionalize the expression, it would have to be

r1/lo  to   ((r2-r1)/h) z'/l0 + r1 /l0

instead of

0   to  ((r2-r1)/h) z' + r1 /l0

and since

zbar= z/l0

zbar' = z'/l0

then

r1 /l0     to   (r2-r1)/h) zbar' + r1 /l0

but the last integral would have to be on drbar' instead of on dr'

and the first integral would have to be on dzbar' instead of on dz,

and the first integral would have to  be over

0 to h/lo

instead of

0 to h
Title: Re: EM Drive Developments - related to space flight applications - Thread 2
Post by: deltaMass on 05/16/2015 03:27 am
Yes. And the way to silence sceptics is to make measurements on a fully-boxed rig - including battery. All these trailing feeds generate pseudo-forces. That would be OK if thrust were Newton level - but it ain't, so it ain't.

The Dynamic Tests of the EM Drive showed a slowly rotating table top of machinery. So sufficient force was being produced to move the equipment 2 metres in a few minutes.
A constant force produces a constant acceleration, unless you are prepared to abandon Isaac-baby. Where was it? Coupled [sic] with which, the table contained several rotating devices like fans and a hard drive.
Title: Re: EM Drive Developments - related to space flight applications - Thread 2
Post by: Rodal on 05/16/2015 03:42 am
Yes. And the way to silence sceptics is to make measurements on a fully-boxed rig - including battery. All these trailing feeds generate pseudo-forces. That would be OK if thrust were Newton level - but it ain't, so it ain't.

The Dynamic Tests of the EM Drive showed a slowly rotating table top of machinery. So sufficient force was being produced to move the equipment 2 metres in a few minutes.
A constant force produces a constant acceleration, unless you are prepared to abandon Isaac-baby. Where was it? Coupled [sic] with which, the table contained several rotating devices like fans and a hard drive.

That's in free-space.

If there was a torsional kinetic friction associated with the bearing, then there was a torque due to that torsional kinetic friction, and therefore there was a force (times the radius) associated with that kinetic friction, under constant velocity.

The force due to kinetic friction could be roughly (pun intended) constant for dry friction at constant velocity, if the force being exerted was just enough to balance the kinetic friction. 

Even a fluid bearing will have a force due to the viscosity of the fluid.


Or it could have accelerated. 

Who knows?  Does anybody have a displacement vs. time trace for that test in the YouTube video?
Title: Re: EM Drive Developments - related to space flight applications - Thread 2
Post by: SeeShells on 05/16/2015 03:46 am
Yes. And the way to silence sceptics is to make measurements on a fully-boxed rig - including battery. All these trailing feeds generate pseudo-forces. That would be OK if thrust were Newton level - but it ain't, so it ain't.

The Dynamic Tests of the EM Drive showed a slowly rotating table top of machinery. So sufficient force was being produced to move the equipment 2 metres in a few minutes.
A constant force produces a constant acceleration, unless you are prepared to abandon Isaac-baby. Where was it? Coupled [sic] with which, the table contained several rotating devices like fans and a hard drive.
You're right it was with hard drives and fans, but weren't they running before they switched on the EM Drive? 
Title: Re: EM Drive Developments - related to space flight applications - Thread 2
Post by: WarpTech on 05/16/2015 03:47 am
The competition
http://nextbigfuture.com/2015/05/photonic-laser-thruster-has-moved-one.html

That's what I'm talking about! Something like this would be at least 20% more efficient than an EM Drive could ever be. IMO, no competition...

Todd D.
How do you figure that?

Because, IMO the EM Drive has no thrust without losses. Resonating modes do not exert any NET force. It is the attenuation of evanescent modes that provide whatever thrust can be had.
Title: Re: EM Drive Developments - related to space flight applications - Thread 2
Post by: SeeShells on 05/16/2015 03:56 am
Anyone seen this little tidbit?
http://phys.org/news/2015-05-newton-law-broken.html
Title: Re: EM Drive Developments - related to space flight applications - Thread 2
Post by: deltaMass on 05/16/2015 04:04 am
Anyone seen this little tidbit?
http://phys.org/news/2015-05-newton-law-broken.html
Sorry, but, as one commenter notes: "Egregious click-bait"
Title: Re: EM Drive Developments - related to space flight applications - Thread 2
Post by: deltaMass on 05/16/2015 04:06 am
Yes. And the way to silence sceptics is to make measurements on a fully-boxed rig - including battery. All these trailing feeds generate pseudo-forces. That would be OK if thrust were Newton level - but it ain't, so it ain't.

The Dynamic Tests of the EM Drive showed a slowly rotating table top of machinery. So sufficient force was being produced to move the equipment 2 metres in a few minutes.
A constant force produces a constant acceleration, unless you are prepared to abandon Isaac-baby. Where was it? Coupled [sic] with which, the table contained several rotating devices like fans and a hard drive.
You're right it was with hard drives and fans, but weren't they running before they switched on the EM Drive?
Excellent point! Unless of course they had clamped the table until the EmDrive was switched on :)
Title: Re: EM Drive Developments - related to space flight applications - Thread 2
Post by: Rodal on 05/16/2015 04:06 am
Anyone seen this little tidbit?
http://phys.org/news/2015-05-newton-law-broken.html
Non-equiibrium thermodynamics

I have to find my old copy of the book by S. R. De Groot, P. Mazur and other such books I have in boxes

and see whether we can derive a force for the EM Drive based on Onsager's relations  :)

Title: Re: EM Drive Developments - related to space flight applications - Thread 2
Post by: deltaMass on 05/16/2015 04:07 am
The competition
http://nextbigfuture.com/2015/05/photonic-laser-thruster-has-moved-one.html

That's what I'm talking about! Something like this would be at least 20% more efficient than an EM Drive could ever be. IMO, no competition...

Todd D.
How do you figure that?

Because, IMO the EM Drive has no thrust without losses. Resonating modes do not exert any NET force. It is the attenuation of evanescent modes that provide whatever thrust can be had.
I'm glad someone understands how the EmDrive works!!
Title: Re: EM Drive Developments - related to space flight applications - Thread 2
Post by: deltaMass on 05/16/2015 06:09 am
Reading that makes me feel that I've just taken a wrong turn and ended up on the film set of Fear And Loathing In Las Vegas.
Title: Re: EM Drive Developments - related to space flight applications - Thread 2
Post by: Mulletron on 05/16/2015 07:53 am
Anyone seen this little tidbit?
http://phys.org/news/2015-05-newton-law-broken.html
Sorry, but, as one commenter notes: "Egregious click-bait"

Awful lot of familiar concepts in there. Clickbait title is clickbait.
Title: Re: EM Drive Developments - related to space flight applications - Thread 2
Post by: Flyby on 05/16/2015 08:08 am
I'm glad someone understands how the EmDrive works!!

Does it? 
I'm still at the stage that i very much want to believe it, but I don't... :-X

At east, not until someone can produce additional proof it actually moves.
Let's hope that in 2 months time, the people at Eagleworks will be able to share their test results.
Or that any of the homebuilders inhere gets some amazing results to show.

Must say I'm impressed by the preparation and analytical work done by TheTraveller. Let's hope it yields some results...
Title: Re: EM Drive Developments - related to space flight applications - Thread 2
Post by: StrongGR on 05/16/2015 09:37 am
I am back with an updated draft after some terrible news around about NASA dismissing these researches. They should not as, otherwise, it could happen as with Galilei having his detractors even not trying to look in the telescope, just dismissing on faith.

I have analysed the case of the frustum and the results appear to be striking. One must admit that geometry comes to rescue not just general relativity. For this particular geometry the cavity can be made susceptible to gravitational effects if your choice of the two radii of the cavity is smart enough. This is something to be confirmed yet, just my theoretical result, but shocking anyway.

As usual, any comment is very welcome.
In page 8, equation 34 of http://forum.nasaspaceflight.com/index.php?action=dlattach;topic=36313.0;attach=830137,
for the integral on dr', should the limits, instead of

0 to ((r2-r1)/h) z' + r2

be

r1 to ((r2-r1)/h) z' + r1 ?

The choice of the integration limits was done taking into account that if the integrand is 1 I recover the volume of the frustum. However I do not evaluate this integral rather I solve the Poisson equation for L.
Title: Re: EM Drive Developments - related to space flight applications - Thread 2
Post by: StrongGR on 05/16/2015 09:46 am
I am back with an updated draft after some terrible news around about NASA dismissing these researches. They should not as, otherwise, it could happen as with Galilei having his detractors even not trying to look in the telescope, just dismissing on faith.

I have analysed the case of the frustum and the results appear to be striking. One must admit that geometry comes to rescue not just general relativity. For this particular geometry the cavity can be made susceptible to gravitational effects if your choice of the two radii of the cavity is smart enough. This is something to be confirmed yet, just my theoretical result, but shocking anyway.

As usual, any comment is very welcome.
In page 8, equation 34 of http://forum.nasaspaceflight.com/index.php?action=dlattach;topic=36313.0;attach=830137,
for the integral on dr', should the limits, instead of

0 to ((r2-r1)/h) z' + r2

be

r1 to ((r2-r1)/h) z' + r1 ?

Good Point !  But volume integral ?     0 to ...    ((r2-r1)/h) z' + r1 /l0  ?  (seems circular that way)

It's Eq. 18 that still bothers me a bit.  Invoking the Heaviside step function is OK, but I don't see the addl. components being detectable outside the cavity w/o a non-linear term.  Maybe I'm missing something ?

That's interesting. The effect of the modified geometry is to couple the resonant mode of the cavity with the external source that in this case is a laser beam. This enters into the correction factor L(v) (sorry for the unfortunate choice of the notation, this is not the same L I use below). I do a first order computation to uncover such a coupling.
Title: Re: EM Drive Developments - related to space flight applications - Thread 2
Post by: StrongGR on 05/16/2015 09:48 am
I am back with an updated draft after some terrible news around about NASA dismissing these researches. They should not as, otherwise, it could happen as with Galilei having his detractors even not trying to look in the telescope, just dismissing on faith.

I have analysed the case of the frustum and the results appear to be striking. One must admit that geometry comes to rescue not just general relativity. For this particular geometry the cavity can be made susceptible to gravitational effects if your choice of the two radii of the cavity is smart enough. This is something to be confirmed yet, just my theoretical result, but shocking anyway.

As usual, any comment is very welcome.
In page 8, equation 34 of http://forum.nasaspaceflight.com/index.php?action=dlattach;topic=36313.0;attach=830137,
for the integral on dr', should the limits, instead of

0 to ((r2-r1)/h) z' + r2

be

r1 to ((r2-r1)/h) z' + r1 ?

Good Point !  But volume integral ?     0 to ...    ((r2-r1)/h) z' + r1 /l0  ?  (seems circular that way)

It's Eq. 18 that still bothers me a bit.  Invoking the Heaviside step function is OK, but I don't see the addl. components being detectable outside the cavity w/o a non-linear term.  Maybe I'm missing something ?

1) Good point about Eq. 18: how do those harmonic components make it outside the cavity?  Do they go outside the cavity because of the small transparent portholes on the EM Drive through which the interferometer laser beam is going through?

2) If one wants to nondimensionalize the expression, it would have to be

r1/lo  to   ((r2-r1)/h) z'/l0 + r1 /l0

instead of

0   to  ((r2-r1)/h) z' + r1 /l0

and since

zbar= z/l0

zbar' = z'/l0

then

r1 /l0     to   (r2-r1)/h) zbar' + r1 /l0

but the last integral would have to be on drbar' instead of on dr'

and the first integral would have to be on dzbar' instead of on dz,

and the first integral would have to  be over

0 to h/lo

instead of

0 to h

That's right. All the quantity in the integral should be intended overlined. But this integral is never computed.
Title: Re: EM Drive Developments - related to space flight applications - Thread 2
Post by: Notsosureofit on 05/16/2015 01:19 pm
I am back with an updated draft after some terrible news around about NASA dismissing these researches. They should not as, otherwise, it could happen as with Galilei having his detractors even not trying to look in the telescope, just dismissing on faith.

I have analysed the case of the frustum and the results appear to be striking. One must admit that geometry comes to rescue not just general relativity. For this particular geometry the cavity can be made susceptible to gravitational effects if your choice of the two radii of the cavity is smart enough. This is something to be confirmed yet, just my theoretical result, but shocking anyway.

As usual, any comment is very welcome.
In page 8, equation 34 of http://forum.nasaspaceflight.com/index.php?action=dlattach;topic=36313.0;attach=830137,
for the integral on dr', should the limits, instead of

0 to ((r2-r1)/h) z' + r2

be

r1 to ((r2-r1)/h) z' + r1 ?

Good Point !  But volume integral ?     0 to ...    ((r2-r1)/h) z' + r1 /l0  ?  (seems circular that way)

It's Eq. 18 that still bothers me a bit.  Invoking the Heaviside step function is OK, but I don't see the addl. components being detectable outside the cavity w/o a non-linear term.  Maybe I'm missing something ?

That's interesting. The effect of the modified geometry is to couple the resonant mode of the cavity with the external source that in this case is a laser beam. This enters into the correction factor L(v) (sorry for the unfortunate choice of the notation, this is not the same L I use below). I do a first order computation to uncover such a coupling.

OK, I think I see it.  Everything is symmetrical to that point.  You need the geometry of the frustum to get a residual modulation.  (I'm getting there in 10min interludes !)
Title: Re: EM Drive Developments - related to space flight applications - Thread 2
Post by: Notsosureofit on 05/16/2015 01:23 pm
Anyone seen this little tidbit?
http://phys.org/news/2015-05-newton-law-broken.html
Non-equiibrium thermodynamics

I have to find my old copy of the book by S. R. De Groot, P. Mazur and other such books I have in boxes

and see whether we can derive a force for the EM Drive based on Onsager's relations  :)

Still looking for my copy as well !  (old cavity work was on transport properties)
Title: Re: EM Drive Developments - related to space flight applications - Thread 2
Post by: StrongGR on 05/16/2015 01:36 pm
Anyone seen this little tidbit?
http://phys.org/news/2015-05-newton-law-broken.html
Non-equiibrium thermodynamics

I have to find my old copy of the book by S. R. De Groot, P. Mazur and other such books I have in boxes

and see whether we can derive a force for the EM Drive based on Onsager's relations  :)

Still looking for my copy as well !  (old cavity work was on transport properties)

The system in that paper is not isolated from the environment. For EM Drive this is not true. You have to cope with a closed system and to understand why third law appears to be violated, if the effect is confirmed
Title: Re: EM Drive Developments - related to space flight applications - Thread 2
Post by: Notsosureofit on 05/16/2015 01:39 pm
Anyone seen this little tidbit?
http://phys.org/news/2015-05-newton-law-broken.html
Non-equiibrium thermodynamics

I have to find my old copy of the book by S. R. De Groot, P. Mazur and other such books I have in boxes

and see whether we can derive a force for the EM Drive based on Onsager's relations  :)

Still looking for my copy as well !  (old cavity work was on transport properties)

The system in that paper is not isolated from the environment. For EM Drive this is not true. You have to cope with a closed system and to understand why third law appears to be violated, if the effect is confirmed

Yup !

Do you remember if DeGroot mentioned photons ?
Title: Re: EM Drive Developments - related to space flight applications - Thread 2
Post by: Mulletron on 05/16/2015 01:52 pm
https://goo.gl/IFeT48  Some of it is missing. Hope it is enough to help.

Stuff I dug up that I'm studying that may be useful to others:
http://benasque.org/2011qfext/talks_contr/2019_Buhmann.pdf
http://library.naist.jp/dspace/bitstream/10061/9833/1/PhysRevLett.113.235501.pdf (very cool)
http://pf.is.mpg.de/assets/ph500305z.pdf
http://www.nature.com/srep/2013/130313/srep01444/full/srep01444.html
Title: Re: EM Drive Developments - related to space flight applications - Thread 2
Post by: StrongGR on 05/16/2015 01:55 pm
Anyone seen this little tidbit?
http://phys.org/news/2015-05-newton-law-broken.html
Non-equiibrium thermodynamics

I have to find my old copy of the book by S. R. De Groot, P. Mazur and other such books I have in boxes

and see whether we can derive a force for the EM Drive based on Onsager's relations  :)

Still looking for my copy as well !  (old cavity work was on transport properties)

The system in that paper is not isolated from the environment. For EM Drive this is not true. You have to cope with a closed system and to understand why third law appears to be violated, if the effect is confirmed

Yup !

Do you remember if DeGroot mentioned photons ?

That book is rather old. I just gave a cursory look and found nothing about photons. You should rather consider the case for electromagnetic radiation. What could make the thing resemble that of a non-isolated system is leakage of radiation out of the cavity. I have to guess that one has built it reducing such losses. The idea to look at gravitational effects, started by Minotti's paper, is because in this case there is not a boundary due to the cavity and system appears to be open (space-time is everywhere). So, the third law would be saved by the expulsion of gravitational momentum. The reason why people in the community of scientists did not consider the case is the smallness of the gravity with respect to all other effects and so, one should not expect it to account for such a measurement.
Title: Re: EM Drive Developments - related to space flight applications - Thread 2
Post by: Notsosureofit on 05/16/2015 02:12 pm
Anyone seen this little tidbit?
http://phys.org/news/2015-05-newton-law-broken.html
Non-equiibrium thermodynamics

I have to find my old copy of the book by S. R. De Groot, P. Mazur and other such books I have in boxes

and see whether we can derive a force for the EM Drive based on Onsager's relations  :)

Still looking for my copy as well !  (old cavity work was on transport properties)

The system in that paper is not isolated from the environment. For EM Drive this is not true. You have to cope with a closed system and to understand why third law appears to be violated, if the effect is confirmed

Yup !

Do you remember if DeGroot mentioned photons ?

That book is rather old. I just gave a cursory look and found nothing about photons. You should rather consider the case for electromagnetic radiation. What could make the thing resemble that of a non-isolated system is leakage of radiation out of the cavity. I have to guess that one has built it reducing such losses. The idea to look at gravitational effects, started by Minotti's paper, is because in this case there is not a boundary due to the cavity and system appears to be open (space-time is everywhere). So, the third law would be saved by the expulsion of gravitational momentum. The reason why people in the community of scientists did not consider the case is the smallness of the gravity with respect to all other effects and so, one should not expect it to account for such a measurement.

As you can see from my crude dispersion argument, the g value (of the photons) is much larger than one might naively expect.

"In the accelerated frame of reference w/ the acceleration, g = (c^2/(2*L*f^2))*(c/(2*Pi))^2*X^2*((1/Rs^2)-(1/Rb^2)) [for this waveguide-like approximation]."
Title: Re: EM Drive Developments - related to space flight applications - Thread 2
Post by: txdrive on 05/16/2015 02:27 pm
Yes. And the way to silence sceptics is to make measurements on a fully-boxed rig - including battery. All these trailing feeds generate pseudo-forces. That would be OK if thrust were Newton level - but it ain't, so it ain't.

The Dynamic Tests of the EM Drive showed a slowly rotating table top of machinery. So sufficient force was being produced to move the equipment 2 metres in a few minutes.
Or a sufficient shift in the centre of mass, for a non perfectly vertical axis.
Title: Re: EM Drive Developments - related to space flight applications - Thread 2
Post by: Rodal on 05/16/2015 02:29 pm
It is interesting that one of the things that Landsberg shows in 1979 is that the following is not longer true in a gravitational field:

"Thermal equilibrium is equivalent to having a system at uniform temperature throughout"
Title: Re: EM Drive Developments - related to space flight applications - Thread 2
Post by: Notsosureofit on 05/16/2015 02:37 pm
It is interesting that one of the things that Landsberg shows in 1979 is that the following is not longer true in a gravitational field:

"Thermal equilibrium is equivalent to having a system at uniform temperature throughout"

Right.  I have a copy of Kantor, but I find his presentation hard going.  He has an introduction ~ p.19.

F. W. Kantor "Information Mechanics", Wiley, 1977

(Heh!  Just found Sokolnikoff, noticed all the markup in in chap.4...was working on "Hamiltonian" radar then .... I need to read chap. 3 !  Opps maybe I did ...."Surfaces in space is marked up!  Hard gettin' old....)
Title: Re: EM Drive Developments - related to space flight applications - Thread 2
Post by: SeeShells on 05/16/2015 02:53 pm
https://goo.gl/IFeT48  Some of it is missing. Hope it is enough to help.

Stuff I dug up that I'm studying that may be useful to others:
http://benasque.org/2011qfext/talks_contr/2019_Buhmann.pdf
http://library.naist.jp/dspace/bitstream/10061/9833/1/PhysRevLett.113.235501.pdf (very cool)
http://pf.is.mpg.de/assets/ph500305z.pdf
http://www.nature.com/srep/2013/130313/srep01444/full/srep01444.html
Nice little treasure trove, good work! Quick reading of some of it seems to be what I've been looking for in one clump. Gads I love the internet and wished it was there when I was going to school. I'm not ready to throw the slab of idea and thought meat into the shark pool yet but thanks for the links!!! Kudos!
Title: Re: EM Drive Developments - related to space flight applications - Thread 2
Post by: Notsosureofit on 05/16/2015 03:32 pm
...
(Heh!  Just found Sokolnikoff, noticed all the markup in in chap.4...was working on "Hamiltonian" radar then .... I need to read chap. 3 !  Opps maybe I did ...."Surfaces in space is marked up!  Hard gettin' old....)
Ivan Sokolnikoff? Which book ? His Higher Math book, his Tensor Analysis book or his less known book with Rainich on the mathematics of relativity ?

Tensor Analysis, '64 edition
Title: Re: EM Drive Developments - related to space flight applications - Thread 2
Post by: Rodal on 05/16/2015 03:34 pm
...Gads I love the internet and wished it was there when I was going to school. I'm not ready to throw the slab of idea and thought meat into the shark pool yet but thanks for the links!!! Kudos!
Not sure about that...wonder how much individual thought process, creativity and imagination is overwhelmed by the huge amount of information available from the Internet nowadays.  Many scientists think that it may have the opposite effect on individual intelligence and thought process as the mind needs as much or more exercise (to produce connections of neurons, etc.) as human muscles.  With so many "answers" freely available at a click of one's fingers, there is much less exercise of one's brain  :)

Hopefully not a future of people thinking that Physics and Engineering is just a question of getting answers from a huge Internet cookbook of recipes and from black-box computer software instead of using mathematics and experiments to find out how Nature operates...
Title: Re: EM Drive Developments - related to space flight applications - Thread 2
Post by: Rodal on 05/16/2015 03:41 pm
...
(Heh!  Just found Sokolnikoff, noticed all the markup in in chap.4...was working on "Hamiltonian" radar then .... I need to read chap. 3 !  Opps maybe I did ...."Surfaces in space is marked up!  Hard gettin' old....)
Ivan Sokolnikoff? Which book ? His Higher Math book, his Tensor Analysis book or his less known book with Rainich on the mathematics of relativity ?

Tensor Analysis, '64 edition
I also like the classic (unabridged) Tensor Analysis book by Schouten ("Ricci-Calculus" the one over 500 pages long)  :)
Title: Re: EM Drive Developments - related to space flight applications - Thread 2
Post by: Notsosureofit on 05/16/2015 03:50 pm
...
(Heh!  Just found Sokolnikoff, noticed all the markup in in chap.4...was working on "Hamiltonian" radar then .... I need to read chap. 3 !  Opps maybe I did ...."Surfaces in space is marked up!  Hard gettin' old....)
Ivan Sokolnikoff? Which book ? His Higher Math book, his Tensor Analysis book or his less known book with Rainich on the mathematics of relativity ?

Tensor Analysis, '64 edition
I also like the classic (unabridged) Tensor Analysis book by Schouten ("Ricci-Calculus" the one over 500 pages long)  :)

http://m.fotoprikolov.net/148099-ricci-calculus-an-introduction-to-tensor-analysis-and-its-geometrical-applications-pdf-download.html ?

Site seems fishy ???
Title: Re: EM Drive Developments - related to space flight applications - Thread 2
Post by: DIYFAN on 05/16/2015 03:52 pm
...Gads I love the internet and wished it was there when I was going to school. I'm not ready to throw the slab of idea and thought meat into the shark pool yet but thanks for the links!!! Kudos!
Not sure about that...wonder how much individual thought process, creativity and imagination is overwhelmed by the huge amount of information available from the Internet nowadays.  Many scientists think that it may have the opposite effect on individual intelligence and thought process as the mind needs as much or more exercise (to produce connections of neurons, etc.) as human muscles.  With so many "answers" freely available at a click of one's fingers, there is much less exercise of one's brain  :)

I almost always find myself agreeing with you Dr. Rodal, but will respectfully and lightheartedly disagree with you on this.  I've heard others take the same position.  My experience and observation have been quite the opposite.  Those who I know who are Internet savvy, are aware of and participate in online forums, are willing to do deep dives into knowledge areas at a whim using the Internet, are some of the most highly informed and highly evolved people that I know. 

Some argue that the quality of information freely available is less than stellar.  Again, I would disagree with that notion as well.  Wikipedia, for example, has been found to be more accurate in recent years than most printed encyclopedias. 

Hooorrrrahh for the Internet, which enables open science pertaining to space flight and other topics, the quick dissemination of information, world-wide collaboration, the trembling of despots, and the dawning of a new age of freedom.

Notice the reference to space flight in my ranting.  :)
Title: Re: EM Drive Developments - related to space flight applications - Thread 2
Post by: Rodal on 05/16/2015 03:58 pm
...Gads I love the internet and wished it was there when I was going to school. I'm not ready to throw the slab of idea and thought meat into the shark pool yet but thanks for the links!!! Kudos!
Not sure about that...wonder how much individual thought process, creativity and imagination is overwhelmed by the huge amount of information available from the Internet nowadays.  Many scientists think that it may have the opposite effect on individual intelligence and thought process as the mind needs as much or more exercise (to produce connections of neurons, etc.) as human muscles.  With so many "answers" freely available at a click of one's fingers, there is much less exercise of one's brain  :)

I almost always find myself agreeing with you Dr. Rodal, but will respectfully and lightheartedly disagree with you on this.  I've heard others take the same position.  My experience and observation have been quite the opposite.  Those who I know who are Internet savvy, are aware of and participate in online forums, are willing to do deep dives into knowledge areas at a whim using the Internet, are some of the most highly informed and highly evolved people that I know. 

Some argue that the quality of information freely available is less than stellar.  Again, I would disagree with that notion as well.  Wikipedia, for example, has been found to be more accurate in recent years than most printed encyclopedias. 

Hooorrrrahh for the Internet, which enables open science pertaining to space flight and other topics, the quick dissemination of information, world-wide collaboration, the trembling of despots, and the dawning of a new age of freedom.

Notice the reference to space flight in my ranting.  :)

I actually agree, there is always negative and positive aspects to everything but the optimistic viewpoint has prevailed throughout history, and hopefully will continue that way  :)

World-wide collaboration in the Internet is a sure plus  :)

Title: Re: EM Drive Developments - related to space flight applications - Thread 2
Post by: SeeShells on 05/16/2015 04:05 pm
...Gads I love the internet and wished it was there when I was going to school. I'm not ready to throw the slab of idea and thought meat into the shark pool yet but thanks for the links!!! Kudos!
Not sure about that...wonder how much individual thought process, creativity and imagination is overwhelmed by the huge amount of information available from the Internet nowadays.  Many scientists think that it may have the opposite effect on individual intelligence and thought process as the mind needs as much or more exercise (to produce connections of neurons, etc.) as human muscles.  With so many "answers" freely available at a click of one's fingers, there is much less exercise of one's brain  :)
That's the truth!
When I read we might have attention spans less than a goldfish from the information load I had to agree also. I  try to back away from it, go sit in the hot tub, wash dishes, pet the cat, anything to keep on trying to shove more information in that I know will just get jumbled. I get older and those periods of intense concentration get less and less, oh, look a squirrel! Now I'm trying to do stuff like I did when in collage and honestly it's not working as well, I call it the Teflon brain syndrome. :o
One thing hasn't changed, flashes of insight, just by watching the stupid squirrel, and where in the heck they come from, I have no idea.
Title: Re: EM Drive Developments - related to space flight applications - Thread 2
Post by: StrongGR on 05/16/2015 04:12 pm
Anyone seen this little tidbit?
http://phys.org/news/2015-05-newton-law-broken.html
Non-equiibrium thermodynamics

I have to find my old copy of the book by S. R. De Groot, P. Mazur and other such books I have in boxes

and see whether we can derive a force for the EM Drive based on Onsager's relations  :)

Still looking for my copy as well !  (old cavity work was on transport properties)

The system in that paper is not isolated from the environment. For EM Drive this is not true. You have to cope with a closed system and to understand why third law appears to be violated, if the effect is confirmed

Yup !

Do you remember if DeGroot mentioned photons ?

That book is rather old. I just gave a cursory look and found nothing about photons. You should rather consider the case for electromagnetic radiation. What could make the thing resemble that of a non-isolated system is leakage of radiation out of the cavity. I have to guess that one has built it reducing such losses. The idea to look at gravitational effects, started by Minotti's paper, is because in this case there is not a boundary due to the cavity and system appears to be open (space-time is everywhere). So, the third law would be saved by the expulsion of gravitational momentum. The reason why people in the community of scientists did not consider the case is the smallness of the gravity with respect to all other effects and so, one should not expect it to account for such a measurement.

As you can see from my crude dispersion argument, the g value (of the photons) is much larger than one might naively expect.

"In the accelerated frame of reference w/ the acceleration, g = (c^2/(2*L*f^2))*(c/(2*Pi))^2*X^2*((1/Rs^2)-(1/Rb^2)) [for this waveguide-like approximation]."

I missed something. Did you write a paper about? Please, let me know as I have not followed this discussion from the start.
Title: Re: EM Drive Developments - related to space flight applications - Thread 2
Post by: StrongGR on 05/16/2015 04:15 pm
It is interesting that one of the things that Landsberg shows in 1979 is that the following is not longer true in a gravitational field:

"Thermal equilibrium is equivalent to having a system at uniform temperature throughout"

That is understandable as the system is open with respect to the gravitational field but this effect should be large enough otherwise the system will retain its equilibrium state. Generally this applies rather well to very large body.
Title: Re: EM Drive Developments - related to space flight applications - Thread 2
Post by: Notsosureofit on 05/16/2015 04:19 pm
Anyone seen this little tidbit?
http://phys.org/news/2015-05-newton-law-broken.html
Non-equiibrium thermodynamics

I have to find my old copy of the book by S. R. De Groot, P. Mazur and other such books I have in boxes

and see whether we can derive a force for the EM Drive based on Onsager's relations  :)

Still looking for my copy as well !  (old cavity work was on transport properties)

The system in that paper is not isolated from the environment. For EM Drive this is not true. You have to cope with a closed system and to understand why third law appears to be violated, if the effect is confirmed

Yup !

Do you remember if DeGroot mentioned photons ?

That book is rather old. I just gave a cursory look and found nothing about photons. You should rather consider the case for electromagnetic radiation. What could make the thing resemble that of a non-isolated system is leakage of radiation out of the cavity. I have to guess that one has built it reducing such losses. The idea to look at gravitational effects, started by Minotti's paper, is because in this case there is not a boundary due to the cavity and system appears to be open (space-time is everywhere). So, the third law would be saved by the expulsion of gravitational momentum. The reason why people in the community of scientists did not consider the case is the smallness of the gravity with respect to all other effects and so, one should not expect it to account for such a measurement.

As you can see from my crude dispersion argument, the g value (of the photons) is much larger than one might naively expect.

"In the accelerated frame of reference w/ the acceleration, g = (c^2/(2*L*f^2))*(c/(2*Pi))^2*X^2*((1/Rs^2)-(1/Rb^2)) [for this waveguide-like approximation]."

I missed something. Did you write a paper about? Please, let me know as I have not followed this discussion from the start.

FYI

Cleanup and detypo of the take on applying the Equivalence Principle.


The proposition that dispersion caused by an accelerating frame of reference implied an accelerating frame of reference caused by a dispersive cavity resonator. (to 1st order using massless, perfectly conducting cavity)


Starting with the expressions for the frequency of a cylindrical RF cavity:

f = (c/(2*Pi))*((X/R)^2+((p*Pi)/L)^2)^.5

For TM modes, X = X[sub m,n] = the n-th zero of the m-th Bessel function.
[1,1]=3.83, [0,1]=2.40, [0,2]=5.52 [1,2]=7.02, [2,1]=5.14, [2,2]=8.42, [1,3]=10.17, etc.

and for TE modes, X = X'[subm,n] = the n-th zero of the derivative of the m-th Bessel function.
[0,1]=3.83, [1,1]=1.84, [2,1]=3.05, [0,2]=7.02, [1,2]=5.33, [1,3]=8.54, [0,3]=10.17, [2,2]=6.71, etc.

Rotate the dispersion relation of the cavity into doppler frame to get the Doppler shifts, that is to say, look at the dispersion curve intersections of constant wave number instead of constant frequency.

delta(f) = (1/(2*f))*(c/(2*Pi))^2*X^2*((1/Rs^2)-(1/Rb^2))

and from there the expression for the acceleration g from:

g = (c^2/L)*(delta(f)/f) such that:

g = (c^2/(2*L*f^2))*(c/(2*Pi))^2*X^2*((1/Rs^2)-(1/Rb^2))

Using the "weight" of the photon in the accelerated frame from:

"W" = (h*f/c^2)*g =>  "W" = T = (h/L)*delta(f)

gives thrust per photon:

T = (h/(2*L*f))*(c/(2*pi))^2*X^2*((1/Rs^2)-(1/Rb^2))

If the number of photons is (P/hf)*(Q/2*pi*f) then:

NT = P*Q*(1/(4*pi*L*f^3))*(c/(2*pi))^2*X^2*((1/Rs^2)-(1/Rb^2))


This does fit (as far as I've gotten) the concept of a self-accelerating Dirac wavepacket (which does conserve momentum).
---------------------------------------------------------------------------------------------------------------------------
 Q is the effective Q due to phase shift etc.
Title: Re: EM Drive Developments - related to space flight applications - Thread 2
Post by: StrongGR on 05/16/2015 04:22 pm
...Gads I love the internet and wished it was there when I was going to school. I'm not ready to throw the slab of idea and thought meat into the shark pool yet but thanks for the links!!! Kudos!
Not sure about that...wonder how much individual thought process, creativity and imagination is overwhelmed by the huge amount of information available from the Internet nowadays.  Many scientists think that it may have the opposite effect on individual intelligence and thought process as the mind needs as much or more exercise (to produce connections of neurons, etc.) as human muscles.  With so many "answers" freely available at a click of one's fingers, there is much less exercise of one's brain  :)

Hopefully not a future of people thinking that Physics and Engineering is just a question of getting answers from a huge Internet cookbook of recipes and from black-box computer software instead of using mathematics and experiments to find out how Nature operates...

As an active researcher, internet appears essential for my work. Journals accept submissions online and can be read online. Arxiv is an internet site without which I cannot work at all. Extensive bibliographic researches can be easily accomplished and I need them to write down a paper. Besides, there are blogs, emails and forums like this. All together this made my life as a researcher really easy. Of course, it is often really difficult to tell what is good from what is bad but this is all what happens in everyday life and does not seem so difficult for well trained people.
Title: Re: EM Drive Developments - related to space flight applications - Thread 2
Post by: StrongGR on 05/16/2015 04:45 pm
Anyone seen this little tidbit?
http://phys.org/news/2015-05-newton-law-broken.html
Non-equiibrium thermodynamics

I have to find my old copy of the book by S. R. De Groot, P. Mazur and other such books I have in boxes

and see whether we can derive a force for the EM Drive based on Onsager's relations  :)

Still looking for my copy as well !  (old cavity work was on transport properties)

The system in that paper is not isolated from the environment. For EM Drive this is not true. You have to cope with a closed system and to understand why third law appears to be violated, if the effect is confirmed

Yup !

Do you remember if DeGroot mentioned photons ?

That book is rather old. I just gave a cursory look and found nothing about photons. You should rather consider the case for electromagnetic radiation. What could make the thing resemble that of a non-isolated system is leakage of radiation out of the cavity. I have to guess that one has built it reducing such losses. The idea to look at gravitational effects, started by Minotti's paper, is because in this case there is not a boundary due to the cavity and system appears to be open (space-time is everywhere). So, the third law would be saved by the expulsion of gravitational momentum. The reason why people in the community of scientists did not consider the case is the smallness of the gravity with respect to all other effects and so, one should not expect it to account for such a measurement.

As you can see from my crude dispersion argument, the g value (of the photons) is much larger than one might naively expect.

"In the accelerated frame of reference w/ the acceleration, g = (c^2/(2*L*f^2))*(c/(2*Pi))^2*X^2*((1/Rs^2)-(1/Rb^2)) [for this waveguide-like approximation]."

I missed something. Did you write a paper about? Please, let me know as I have not followed this discussion from the start.

FYI

Cleanup and detypo of the take on applying the Equivalence Principle.


The proposition that dispersion caused by an accelerating frame of reference implied an accelerating frame of reference caused by a dispersive cavity resonator. (to 1st order using massless, perfectly conducting cavity)


Starting with the expressions for the frequency of an RF cavity:

f = (c/(2*Pi))*((X/R)^2+((p*Pi)/L)^2)^.5

For TM modes, X = X[sub m,n] = the n-th zero of the m-th Bessel function.
[1,1]=3.83, [0,1]=2.40, [0,2]=5.52 [1,2]=7.02, [2,1]=5.14, [2,2]=8.42, [1,3]=10.17, etc.

and for TE modes, X = X'[subm,n] = the n-th zero of the derivative of the m-th Bessel function.
[0,1]=3.83, [1,1]=1.84, [2,1]=3.05, [0,2]=7.02, [1,2]=5.33, [1,3]=8.54, [0,3]=10.17, [2,2]=6.71, etc.

Rotate the dispersion relation of the cavity into doppler frame to get the Doppler shifts, that is to say, look at the dispersion curve intersections of constant wave number instead of constant frequency.

df = (1/(2*f))*(c/(2*Pi))^2*X^2*((1/Rs^2)-(1/Rb^2))

and from there the expression for the acceleration g from:

g = (c^2/L)*(df/f) such that:

g = (c^2/(2*L*f^2))*(c/(2*Pi))^2*X^2*((1/Rs^2)-(1/Rb^2))

Using the "weight" of the photon in the accelerated frame from:

"W" = (h*f/c^2)*g =>  "W" = T = (h/L)*df

gives thrust per photon:

T = (h/(2*L*f))*(c/(2*pi))^2*X^2*((1/Rs^2)-(1/Rb^2))

If the number of photons is (P/hf)*(Q/2*pi*f) then:

NT = P*Q*(1/(4*pi*L*f^3))*(c/(2*pi))^2*X^2*((1/Rs^2)-(1/Rb^2))


This does fit (as far as I've gotten) the concept of a self-accelerating Dirac wavepacket (which does conserve momentum).
---------------------------------------------------------------------------------------------------------------------------
Yes the search function is terrible

df is delta f not the differential,  Q is the effective Q due to phase shift etc.

I have put your text in a LaTeX format and enclose it here. But the question is: Where does your acceleration come from?
Title: Re: EM Drive Developments - related to space flight applications - Thread 2
Post by: Notsosureofit on 05/16/2015 04:57 pm
Anyone seen this little tidbit?
http://phys.org/news/2015-05-newton-law-broken.html
Non-equiibrium thermodynamics

I have to find my old copy of the book by S. R. De Groot, P. Mazur and other such books I have in boxes

and see whether we can derive a force for the EM Drive based on Onsager's relations  :)

Still looking for my copy as well !  (old cavity work was on transport properties)

The system in that paper is not isolated from the environment. For EM Drive this is not true. You have to cope with a closed system and to understand why third law appears to be violated, if the effect is confirmed

Yup !

Do you remember if DeGroot mentioned photons ?

That book is rather old. I just gave a cursory look and found nothing about photons. You should rather consider the case for electromagnetic radiation. What could make the thing resemble that of a non-isolated system is leakage of radiation out of the cavity. I have to guess that one has built it reducing such losses. The idea to look at gravitational effects, started by Minotti's paper, is because in this case there is not a boundary due to the cavity and system appears to be open (space-time is everywhere). So, the third law would be saved by the expulsion of gravitational momentum. The reason why people in the community of scientists did not consider the case is the smallness of the gravity with respect to all other effects and so, one should not expect it to account for such a measurement.

As you can see from my crude dispersion argument, the g value (of the photons) is much larger than one might naively expect.

"In the accelerated frame of reference w/ the acceleration, g = (c^2/(2*L*f^2))*(c/(2*Pi))^2*X^2*((1/Rs^2)-(1/Rb^2)) [for this waveguide-like approximation]."

I missed something. Did you write a paper about? Please, let me know as I have not followed this discussion from the start.

FYI

Cleanup and detypo of the take on applying the Equivalence Principle.


The proposition that dispersion caused by an accelerating frame of reference implied an accelerating frame of reference caused by a dispersive cavity resonator. (to 1st order using massless, perfectly conducting cavity)


Starting with the expressions for the frequency of an RF cavity:

f = (c/(2*Pi))*((X/R)^2+((p*Pi)/L)^2)^.5

For TM modes, X = X[sub m,n] = the n-th zero of the m-th Bessel function.
[1,1]=3.83, [0,1]=2.40, [0,2]=5.52 [1,2]=7.02, [2,1]=5.14, [2,2]=8.42, [1,3]=10.17, etc.

and for TE modes, X = X'[subm,n] = the n-th zero of the derivative of the m-th Bessel function.
[0,1]=3.83, [1,1]=1.84, [2,1]=3.05, [0,2]=7.02, [1,2]=5.33, [1,3]=8.54, [0,3]=10.17, [2,2]=6.71, etc.

Rotate the dispersion relation of the cavity into doppler frame to get the Doppler shifts, that is to say, look at the dispersion curve intersections of constant wave number instead of constant frequency.

df = (1/(2*f))*(c/(2*Pi))^2*X^2*((1/Rs^2)-(1/Rb^2))

and from there the expression for the acceleration g from:

g = (c^2/L)*(df/f) such that:

g = (c^2/(2*L*f^2))*(c/(2*Pi))^2*X^2*((1/Rs^2)-(1/Rb^2))

Using the "weight" of the photon in the accelerated frame from:

"W" = (h*f/c^2)*g =>  "W" = T = (h/L)*df

gives thrust per photon:

T = (h/(2*L*f))*(c/(2*pi))^2*X^2*((1/Rs^2)-(1/Rb^2))

If the number of photons is (P/hf)*(Q/2*pi*f) then:

NT = P*Q*(1/(4*pi*L*f^3))*(c/(2*pi))^2*X^2*((1/Rs^2)-(1/Rb^2))


This does fit (as far as I've gotten) the concept of a self-accelerating Dirac wavepacket (which does conserve momentum).
---------------------------------------------------------------------------------------------------------------------------
Yes the search function is terrible

df is delta f not the differential,  Q is the effective Q due to phase shift etc.

I have put your text in a LaTeX format and enclose it here. But the question is: Where does your acceleration come from?

The accelerated frame or reference in which the asymmetry of the cavity is balanced out.  That is as though the cavity was cylindrical in this case.  Notice you would probably need the coordinate changes to do this as a proper derivation.  As we said, that might have to constitute a (mathematical) proof.
Title: Re: EM Drive Developments - related to space flight applications - Thread 2
Post by: StrongGR on 05/16/2015 05:06 pm
Anyone seen this little tidbit?
http://phys.org/news/2015-05-newton-law-broken.html
Non-equiibrium thermodynamics

I have to find my old copy of the book by S. R. De Groot, P. Mazur and other such books I have in boxes

and see whether we can derive a force for the EM Drive based on Onsager's relations  :)

Still looking for my copy as well !  (old cavity work was on transport properties)

The system in that paper is not isolated from the environment. For EM Drive this is not true. You have to cope with a closed system and to understand why third law appears to be violated, if the effect is confirmed

Yup !

Do you remember if DeGroot mentioned photons ?

That book is rather old. I just gave a cursory look and found nothing about photons. You should rather consider the case for electromagnetic radiation. What could make the thing resemble that of a non-isolated system is leakage of radiation out of the cavity. I have to guess that one has built it reducing such losses. The idea to look at gravitational effects, started by Minotti's paper, is because in this case there is not a boundary due to the cavity and system appears to be open (space-time is everywhere). So, the third law would be saved by the expulsion of gravitational momentum. The reason why people in the community of scientists did not consider the case is the smallness of the gravity with respect to all other effects and so, one should not expect it to account for such a measurement.

As you can see from my crude dispersion argument, the g value (of the photons) is much larger than one might naively expect.

"In the accelerated frame of reference w/ the acceleration, g = (c^2/(2*L*f^2))*(c/(2*Pi))^2*X^2*((1/Rs^2)-(1/Rb^2)) [for this waveguide-like approximation]."

I missed something. Did you write a paper about? Please, let me know as I have not followed this discussion from the start.

FYI

Cleanup and detypo of the take on applying the Equivalence Principle.


The proposition that dispersion caused by an accelerating frame of reference implied an accelerating frame of reference caused by a dispersive cavity resonator. (to 1st order using massless, perfectly conducting cavity)


Starting with the expressions for the frequency of an RF cavity:

f = (c/(2*Pi))*((X/R)^2+((p*Pi)/L)^2)^.5

For TM modes, X = X[sub m,n] = the n-th zero of the m-th Bessel function.
[1,1]=3.83, [0,1]=2.40, [0,2]=5.52 [1,2]=7.02, [2,1]=5.14, [2,2]=8.42, [1,3]=10.17, etc.

and for TE modes, X = X'[subm,n] = the n-th zero of the derivative of the m-th Bessel function.
[0,1]=3.83, [1,1]=1.84, [2,1]=3.05, [0,2]=7.02, [1,2]=5.33, [1,3]=8.54, [0,3]=10.17, [2,2]=6.71, etc.

Rotate the dispersion relation of the cavity into doppler frame to get the Doppler shifts, that is to say, look at the dispersion curve intersections of constant wave number instead of constant frequency.

df = (1/(2*f))*(c/(2*Pi))^2*X^2*((1/Rs^2)-(1/Rb^2))

and from there the expression for the acceleration g from:

g = (c^2/L)*(df/f) such that:

g = (c^2/(2*L*f^2))*(c/(2*Pi))^2*X^2*((1/Rs^2)-(1/Rb^2))

Using the "weight" of the photon in the accelerated frame from:

"W" = (h*f/c^2)*g =>  "W" = T = (h/L)*df

gives thrust per photon:

T = (h/(2*L*f))*(c/(2*pi))^2*X^2*((1/Rs^2)-(1/Rb^2))

If the number of photons is (P/hf)*(Q/2*pi*f) then:

NT = P*Q*(1/(4*pi*L*f^3))*(c/(2*pi))^2*X^2*((1/Rs^2)-(1/Rb^2))


This does fit (as far as I've gotten) the concept of a self-accelerating Dirac wavepacket (which does conserve momentum).
---------------------------------------------------------------------------------------------------------------------------
Yes the search function is terrible

df is delta f not the differential,  Q is the effective Q due to phase shift etc.

I have put your text in a LaTeX format and enclose it here. But the question is: Where does your acceleration come from?

The accelerated frame or reference in which the asymmetry of the cavity is balanced out.  That is as though the cavity was cylindrical in this case.  Notice you would probably need the coordinate changes to do this as a proper derivation.  As we said, that might have to constitute a (mathematical) proof.

You have a cavity standing still. You claim there is a Doppler shift in the modes. What is the origin of this Doppler shift? I cannot see any way to get "accelerated" photons here as you claim they are.
Title: Re: EM Drive Developments - related to space flight applications - Thread 2
Post by: Notsosureofit on 05/16/2015 05:12 pm
Yes the search function is terrible

df is delta f not the differential,  Q is the effective Q due to phase shift etc.

I have put your text in a LaTeX format and enclose it here. But the question is: Where does your acceleration come from?
[/quote]

Thank you !  I'll look up LaTex (new to me)   How does one edit the pdf ?  ( df should be delta f, for example)

Its the (taking the extremes of the) equivalent wavelength change along the cavity axis of the resonant frequency in the rest frame.  It should really be the result of an integral over that length to get a more general case ( included dielectrics for example )
Title: Re: EM Drive Developments - related to space flight applications - Thread 2
Post by: StrongGR on 05/16/2015 05:18 pm
Yes the search function is terrible

df is delta f not the differential,  Q is the effective Q due to phase shift etc.

I have put your text in a LaTeX format and enclose it here. But the question is: Where does your acceleration come from?

Thank you !  I'll look up LaTex (new to me)   How does one edit the pdf ?  ( df should be delta f, for example)

Its the (taking the extremes of the) equivalent wavelength change along the cavity axis of the resonant frequency in the rest frame.  It should really be the result of an integral over that length to get a more general case ( included dielectrics for example )
[/quote]

You are welcome. But what is the moving frame? I see a cavity that is still. No way to get a frequency changed (gravity is an exception).
Title: Re: EM Drive Developments - related to space flight applications - Thread 2
Post by: Rodal on 05/16/2015 05:20 pm
...
I have put your text in a LaTeX format and enclose it here. But the question is: Where does your acceleration come from?
Watch out for errors on the transcription to LaTeX pdf http://forum.nasaspaceflight.com/index.php?action=dlattach;topic=36313.0;attach=830511

1) equation (1) reads

2c/Pi

instead of

c/(2*Pi)



2) Also there is an error in Eq. (5)

T=(h/c)*delta f

should read

T=(h/L)*delta f



3) "c" in Notsosureofit's post stands for

speed-of-light-in-medium = c/Sqrt[μr εr ]

where c is the speed of light in vacuum



4) Also this link may be helpful, giving a table of the Xmn and  X'mn to 15 digits accuracy


http://wwwal.kuicr.kyoto-u.ac.jp/www/accelerator/a4/besselroot.htmlx



Title: Re: EM Drive Developments - related to space flight applications - Thread 2
Post by: StrongGR on 05/16/2015 05:22 pm
Quote
Thank you !  I'll look up LaTex (new to me)   How does one edit the pdf ?  ( df should be delta f, for example)

Its the (taking the extremes of the) equivalent wavelength change along the cavity axis of the resonant frequency in the rest frame.  It should really be the result of an integral over that length to get a more general case ( included dielectrics for example )

I fixed that delta f.
Title: Re: EM Drive Developments - related to space flight applications - Thread 2
Post by: StrongGR on 05/16/2015 05:24 pm
...
I have put your text in a LaTeX format and enclose it here. But the question is: Where does your acceleration come from?
Watch out for errors on the transcription to LaTeX pdf.

For example, first equation (1) reads 2c/Pi instead of c/(2*Pi)

http://forum.nasaspaceflight.com/index.php?action=dlattach;topic=36313.0;attach=830511

Fixed. Thanks.
Title: Re: EM Drive Developments - related to space flight applications - Thread 2
Post by: Acryte on 05/16/2015 05:28 pm
Today i did the first test with the Emdrive (microwave oven magnetron and cooper frustum)
The setup (magnetron and frusum) was suspended in a pendulum.
I applied power for 40 Seconds with no visible thrust. Tomorrow will will try again with the magnetron on the small side. You have any suggestion for what should be the distance from the small side?
After this i will adjust the power to the filament of magnetron and the frequency.
To fine adjust the frequency i thought i can put 2 coils over the magnetron magnets to modify the magnetic field.
My website;
http://www.masinaelectrica.com/emdrive-independent-test/

https://www.youtube.com/watch?v=2-cgVoLUJ8w

Don't burn your house down. Your setup looks inadequately safe.
Title: Re: EM Drive Developments - related to space flight applications - Thread 2
Post by: aero on 05/16/2015 05:36 pm
...
I have put your text in a LaTeX format and enclose it here. But the question is: Where does your acceleration come from?
Watch out for errors on the transcription to LaTeX pdf.

For example, first equation (1) reads 2c/Pi instead of c/(2*Pi)

http://forum.nasaspaceflight.com/index.php?action=dlattach;topic=36313.0;attach=830511

Fixed. Thanks.

It would be helpful to mark revision number both inside the file and in the title. Maybe with a date inside.

And thanks, the LaTeX pdf is so much easier to read even if LaTeX can't be inserted directly into the text of this thread. ( I suppose it could be inserted or attached as an image? But that's not necessary.)
Title: Re: EM Drive Developments - related to space flight applications - Thread 2
Post by: StrongGR on 05/16/2015 05:42 pm
...
I have put your text in a LaTeX format and enclose it here. But the question is: Where does your acceleration come from?
Watch out for errors on the transcription to LaTeX pdf.

For example, first equation (1) reads 2c/Pi instead of c/(2*Pi)

http://forum.nasaspaceflight.com/index.php?action=dlattach;topic=36313.0;attach=830511

Fixed. Thanks.

It would be helpful to mark revision number both inside the file and in the title. Maybe with a date inside.

Yes, knowing LaTeX, all free from internet (editor included), it is possible to accomplish all the tasks you are asking for. I did this just because it is really difficult to read ascii formulas and I would like to understand the argument by Notsureofit. These forums do not foresee to use LaTeX and this can make it overall complicated.
Title: Re: EM Drive Developments - related to space flight applications - Thread 2
Post by: WarpTech on 05/16/2015 06:29 pm
Anyone seen this little tidbit?
http://phys.org/news/2015-05-newton-law-broken.html
Non-equiibrium thermodynamics

I have to find my old copy of the book by S. R. De Groot, P. Mazur and other such books I have in boxes

and see whether we can derive a force for the EM Drive based on Onsager's relations  :)

Still looking for my copy as well !  (old cavity work was on transport properties)

The system in that paper is not isolated from the environment. For EM Drive this is not true. You have to cope with a closed system and to understand why third law appears to be violated, if the effect is confirmed

Yup !

Do you remember if DeGroot mentioned photons ?

That book is rather old. I just gave a cursory look and found nothing about photons. You should rather consider the case for electromagnetic radiation. What could make the thing resemble that of a non-isolated system is leakage of radiation out of the cavity. I have to guess that one has built it reducing such losses. The idea to look at gravitational effects, started by Minotti's paper, is because in this case there is not a boundary due to the cavity and system appears to be open (space-time is everywhere). So, the third law would be saved by the expulsion of gravitational momentum. The reason why people in the community of scientists did not consider the case is the smallness of the gravity with respect to all other effects and so, one should not expect it to account for such a measurement.

As you can see from my crude dispersion argument, the g value (of the photons) is much larger than one might naively expect.

"In the accelerated frame of reference w/ the acceleration, g = (c^2/(2*L*f^2))*(c/(2*Pi))^2*X^2*((1/Rs^2)-(1/Rb^2)) [for this waveguide-like approximation]."

I missed something. Did you write a paper about? Please, let me know as I have not followed this discussion from the start.

FYI

Cleanup and detypo of the take on applying the Equivalence Principle.


The proposition that dispersion caused by an accelerating frame of reference implied an accelerating frame of reference caused by a dispersive cavity resonator. (to 1st order using massless, perfectly conducting cavity)


Starting with the expressions for the frequency of an RF cavity:

f = (c/(2*Pi))*((X/R)^2+((p*Pi)/L)^2)^.5

For TM modes, X = X[sub m,n] = the n-th zero of the m-th Bessel function.
[1,1]=3.83, [0,1]=2.40, [0,2]=5.52 [1,2]=7.02, [2,1]=5.14, [2,2]=8.42, [1,3]=10.17, etc.

and for TE modes, X = X'[subm,n] = the n-th zero of the derivative of the m-th Bessel function.
[0,1]=3.83, [1,1]=1.84, [2,1]=3.05, [0,2]=7.02, [1,2]=5.33, [1,3]=8.54, [0,3]=10.17, [2,2]=6.71, etc.

Rotate the dispersion relation of the cavity into doppler frame to get the Doppler shifts, that is to say, look at the dispersion curve intersections of constant wave number instead of constant frequency.

df = (1/(2*f))*(c/(2*Pi))^2*X^2*((1/Rs^2)-(1/Rb^2))

and from there the expression for the acceleration g from:

g = (c^2/L)*(df/f) such that:

g = (c^2/(2*L*f^2))*(c/(2*Pi))^2*X^2*((1/Rs^2)-(1/Rb^2))

Using the "weight" of the photon in the accelerated frame from:

"W" = (h*f/c^2)*g =>  "W" = T = (h/L)*df

gives thrust per photon:

T = (h/(2*L*f))*(c/(2*pi))^2*X^2*((1/Rs^2)-(1/Rb^2))

If the number of photons is (P/hf)*(Q/2*pi*f) then:

NT = P*Q*(1/(4*pi*L*f^3))*(c/(2*pi))^2*X^2*((1/Rs^2)-(1/Rb^2))


This does fit (as far as I've gotten) the concept of a self-accelerating Dirac wavepacket (which does conserve momentum).
---------------------------------------------------------------------------------------------------------------------------
Yes the search function is terrible

df is delta f not the differential,  Q is the effective Q due to phase shift etc.

I have put your text in a LaTeX format and enclose it here. But the question is: Where does your acceleration come from?

It comes from "work" done by the waves on the copper, i.e. EM induction, which slows down the wave and steals it's momentum. It mimics gravity in this regard, because photons lose energy going "up".

I'm still working on a slightly different version of this. My time has been severely limited this past week however.

Todd D.
Title: Re: EM Drive Developments - related to space flight applications - Thread 2
Post by: deltaMass on 05/16/2015 07:41 pm
Perhaps MathJax would enhance this forum? StackExchange uses it to great advantage.
Title: Re: EM Drive Developments - related to space flight applications - Thread 2
Post by: deuteragenie on 05/16/2015 08:01 pm
4) Also this link may be helpful, giving a table of the Xmn and  X'mn to 15 digits accuracy
http://wwwal.kuicr.kyoto-u.ac.jp/www/accelerator/a4/besselroot.htmlx

Try:

 Wolfram alpha  (https://www.wolframalpha.com/input/?i=besseljzero[1%2C1)
Title: Re: EM Drive Developments - related to space flight applications - Thread 2
Post by: deuteragenie on 05/16/2015 09:57 pm
4) Also this link may be helpful, giving a table of the Xmn and  X'mn to 15 digits accuracy
http://wwwal.kuicr.kyoto-u.ac.jp/www/accelerator/a4/besselroot.htmlx

Try:

 Wolfram alpha  (https://www.wolframalpha.com/input/?i=besseljzero[1%2C1)
Thanks.  That link ( https://www.wolframalpha.com/input/?i=besseljzero[1%2C1  ) only gives Xmn which are only useful for TM modes

Is there a way that Wolfram Alpha can give  X'mn which are needed for TE modes ?

I don't think so. But I believe you have Mathemagica:
http://www.me.rochester.edu/courses/ME201/webexamp/derbesszer.pdf - see ln67



Title: Re: EM Drive Developments - related to space flight applications - Thread 2
Post by: Rodal on 05/16/2015 10:33 pm
4) Also this link may be helpful, giving a table of the Xmn and  X'mn to 15 digits accuracy
http://wwwal.kuicr.kyoto-u.ac.jp/www/accelerator/a4/besselroot.htmlx

Try:

 Wolfram alpha  (https://www.wolframalpha.com/input/?i=besseljzero[1%2C1)
Thanks.  That link ( https://www.wolframalpha.com/input/?i=besseljzero[1%2C1  ) only gives Xmn which are only useful for TM modes

Is there a way that Wolfram Alpha can give  X'mn which are needed for TE modes ?

I don't think so. But I believe you have Mathemagica:
http://www.me.rochester.edu/courses/ME201/webexamp/derbesszer.pdf - see ln67
OK, yes, that's helpful to the people that have Mathematica.  They can program that function ( notice that X'mn is not a buil-in function in Mathematica yet).  To those that don't have Mathematica, that's the great usefulness of this link http://wwwal.kuicr.kyoto-u.ac.jp/www/accelerator/a4/besselroot.htmlx: it gives X'mn to 15 digits accuracy and it is accessible to everyone. 

The Kyoto (Japan) University link only goes to m=10, n=5 though, so if anyone knows of a link going to a higher quantum number (n > 5) , please post it, as it may be helpful to people just using Excel.  (In case anyone is interested in investigating super-high mode shapes for n > 5  :)
Title: Re: EM Drive Developments - related to space flight applications - Thread 2
Post by: Rodal on 05/17/2015 12:12 am
I am back with an updated draft after some terrible news around about NASA dismissing these researches. They should not as, otherwise, it could happen as with Galilei having his detractors even not trying to look in the telescope, just dismissing on faith.

I have analysed the case of the frustum and the results appear to be striking. One must admit that geometry comes to rescue not just general relativity. For this particular geometry the cavity can be made susceptible to gravitational effects if your choice of the two radii of the cavity is smart enough. This is something to be confirmed yet, just my theoretical result, but shocking anyway.

As usual, any comment is very welcome.

http://forum.nasaspaceflight.com/index.php?action=dlattach;topic=36313.0;attach=830137

Marco, your paper, after equation 39, has this interesting comment:

Quote from: Frasca
These equations appear rather interesting as, by a proper choice of parameters, one can make a gravitational effect more or less relevant in the physics of the problem. It is the case to say that geometry comes to rescue.


Just what this "proper choice of parameters" is, it is not spelled out in your paper, so I will follow my interpretation of your equation. 

It appears (see proof below) that in order to maximize the constants "a" and "b" in Equation 39, we want to have:

1) r1 as close as possible to r2
2) r2 as large as possible


Now, if this is correct, this is a rather peculiar, surprising geometry: it says that the axial length of the truncated cone should be close to zero while the radius should be as large as possible, in other words, the geometry should be as close to an almost perfect cylinder (small cone angle) with very short axial length, and with flat faces. In other words, this optimized geometry, according to Eq. 39 in your paper seems to be much closer to Cannae's device.
 And it is actually not far from the geometry presently used by Dr. White. It is certainly not the geometry of a cone

(http://1.bp.blogspot.com/-2MORMyXf97o/VU4f2f8MtII/AAAAAAAA90w/Oc5eLpuujA4/s400/cannae_driveNASA.jpg)

However  (see below), we also conclude that the maximum possible values of the constants b (and also a) are extremely small and hence they are negligible unless the EM Drive happens to be near a magnestar.

The "lo" constant is a really huge number (unless, as you state the EM Drive happens to be located next to the field of a magnetar, which is certainly not the case, I might add, since the closest magnetar SGR 1806-20 is located about 50,000 light-years away from Earth  ;) ). 



PROOF:

Taking (from Eq. 39 in your paper)

b = (r1^2 - r2^2)/(4 (lo^2) Log[r2/r1])

Assuming r1 =<r2

Define

r1bar = r1 / lo
r2bar = r2 / lo

Then

b = (r1bar^2 - r2bar^2)/(4 Log[r2bar/r1bar])

Define:

r1bar = r2bar/c  (where c >= 1 since r1bar =< r2bar )

bb = b / (r2bar^2)

then

bb = (1 - c^2)/(4 (c^2) Log[c])

Limit[bb, c -> 1] = -  0.5  (this corresponds to the maximum possible value of r1, r1 ~ r2 )

Limit[bb, c -> 1.5] = -  0.342542

Limit[bb, c -> 2] = - 0.270505

Limit[bb, c -> Infinity] = 0  (this corresponds to the minimum possible value of r1, r1 -> 0 )

Then it follows that the maximum absolute value of bb  (bb= - 0.5) occurs at c = 1 , at r1 ~ r2
And to maximize b we must maximize r2bar, and therefore maximize r2, since b =  bb (r2bar^2)= (- 1/2) (r2bar^2)

The proof for "a' is similar but it involves an extra step

Now, since the maximum (absolute magnitude) value of b is b = (- 1/2) (r2bar^2) and we know that
r2bar = r2 / lo
then the maximum value of b is

b = (- 1/2) ((r2 / lo)^2)

but since we know that lo is a huge number (unless the EM Drive is next to a magnestar) and feasible values of r2 are such that

r2 < lo

then it necessarily follows that the maximum possible value of b is much, much smaller than 1

b  << 1

we conclude that the maximum possible values of b (and also a) are extremely small and hence they are negligible unless the EM Drive happens to be near a magnetar http://en.wikipedia.org/wiki/Magnetar.

(http://www.dailygalaxy.com/my_weblog/images/2008/09/25/magnetarfig3.jpg)
Title: Re: EM Drive Developments - related to space flight applications - Thread 2
Post by: WarpTech on 05/17/2015 01:28 am
...
FYI

Cleanup and detypo of the take on applying the Equivalence Principle.


The proposition that dispersion caused by an accelerating frame of reference implied an accelerating frame of reference caused by a dispersive cavity resonator. (to 1st order using massless, perfectly conducting cavity)


Starting with the expressions for the frequency of an RF cavity:

f = (c/(2*Pi))*((X/R)^2+((p*Pi)/L)^2)^.5

....

Rotate the dispersion relation of the cavity into doppler frame to get the Doppler shifts, that is to say, look at the dispersion curve intersections of constant wave number instead of constant frequency.

df = (1/(2*f))*(c/(2*Pi))^2*X^2*((1/Rs^2)-(1/Rb^2))

...

...

It comes from "work" done by the waves on the copper, i.e. EM induction, which slows down the wave and steals it's momentum. It mimics gravity in this regard, because photons lose energy going "up".

I'm still working on a slightly different version of this. My time has been severely limited this past week however.

Todd D.

@Notsosureofit, I just realized why I was perplexed by df, though I understood what you meant I could not derive it. Now I think I see what you have is;

df = (f(Rs)^2 - f(Rb)^2/2f

What I would do is take the derivative wrt L, where R is a fn of L.

df = - (1/f) * (c/2pi)^2 * ((X^2 / R^3) * dR + ((p*pi)^2 / L^3) * dL)

What is going on here is something akin to the Relativistic energy equation:

p^2 = E^2 - m^2   (for c = 1)

The dispersion relation correlates to;

1/lambda_g^2 = 1/lambda_0^2 - 1/lambda_c^2

Where _g is for guide, _0 is for free space and _c is for cut-off wavelengths.

Taking the derivative, lambda_0 is a constant and that term goes to zero. It is then easy to see that the relationship between the guide wavelength and the cut-off are equal and opposite, and how it accelerates.

Standing waves have E and B out of phase 90 deg. These fields can do no work. They are there to store and accumulate energy, like an amplifier.

Where work is being done is when E < m and p is imaginary. That imaginary component adds a 90 degree phase shift, that aligns E & B so that work can be done on the frustum. For those Electrical Engineers out there, it adjusts the Power Factor from 0 to 1.

This occurs each time waves travel back toward the small end. Their momentum is absorbed. The force will be the rate, dp/dt.

p = sqrt(E^2 - m^2), which will be imaginary. We want to maximize this asymmetrically.

This is why my gut instinct tells me that trying to optimize Q and resonance in the same device required to generate attenuation is counter productive. The amplifier and the attenuator should be separate systems.

Todd D.
Title: Re: EM Drive Developments - related to space flight applications - Thread 2
Post by: Notsosureofit on 05/17/2015 01:56 am
Just got back (tired)

FYI

http://www.perimeterinstitute.ca/news/general-relativity-z
Title: Re: EM Drive Developments - related to space flight applications - Thread 2
Post by: dustinthewind on 05/17/2015 08:55 am
I have been meaning to ask.  If anyone gets the chance to test this out for me I would be fascinated to hear the results.  It's a dual cavity experiment.  I think it would require a klystron or twt with a continuous phase.  Cylindrical cavities should work.  They would need to have a gap between the two of them of 1/4 lambda wavelength with their phase out of alignment by 90 degrees or pi/2 radians.   

What should happen is the plate of cavity 1 should have circulating current stimulated by the radiation inside that builds up depending on the Q of the cavity.  Information is limited by the speed of light so that appearance of current should be on its way to the next cavity which has its current out of phase by 90 degrees.  When the information reaches cavity 2 the current in cavity 2 should appear to be moving with cavity 1 so it is attracted.  Cavity 1 on the other hand when the signal reaches it appears to have its current moving against cavity 2 and should be repulsed (similar to current in wires).  I would guess this effect should also scale with the Q of the cavity. 

It may be possible for a dielectric to slow the information transfer between the cavities and move them closer together and increase the effect.  I would guess some radiation may tunnel through the cavities into the other one and attenuation of one cavity may be required to keep them equal in amplitude and out of phase by 90 degrees. 

My guess is it should be similar to attraction between magnets but the force is in the same direction for both cavities.  Images attached for ease of understanding. 

Fig1 Simple.png key:
1. Forces
2. Current
3. Current
4. Appearance of current limited by the speed of light (bottom wire)
5. Appearance of current limited by the speed of light (top wire)

8, 9, 10, 11 are frames of current over time.  Frame 12 is a repeat of frame 8 and frame 13 is a repeat of 9.  (Only 4 frames needed).  The illustration of force between wires can parallel to the cavity plate I think. 

Thanks!
Title: Re: EM Drive Developments - related to space flight applications - Thread 2
Post by: StrongGR on 05/17/2015 10:34 am
I am back with an updated draft after some terrible news around about NASA dismissing these researches. They should not as, otherwise, it could happen as with Galilei having his detractors even not trying to look in the telescope, just dismissing on faith.

I have analysed the case of the frustum and the results appear to be striking. One must admit that geometry comes to rescue not just general relativity. For this particular geometry the cavity can be made susceptible to gravitational effects if your choice of the two radii of the cavity is smart enough. This is something to be confirmed yet, just my theoretical result, but shocking anyway.

As usual, any comment is very welcome.

http://forum.nasaspaceflight.com/index.php?action=dlattach;topic=36313.0;attach=830137

Marco, your paper, after equation 39, has this interesting comment:

Quote from: Frasca
These equations appear rather interesting as, by a proper choice of parameters, one can make a gravitational effect more or less relevant in the physics of the problem. It is the case to say that geometry comes to rescue.


Just what this "proper choice of parameters" is, it is not spelled out in your paper, so I will follow my interpretation of your equation. 

It appears (see proof below) that in order to maximize the constants "a" and "b" in Equation 39, we want to have:

1) r1 as close as possible to r2
2) r2 as large as possible


Now, if this is correct, this is a rather peculiar, surprising geometry: it says that the axial length of the truncated cone should be close to zero while the radius should be as large as possible, in other words, the geometry should be as close to an almost perfect cylinder (small cone angle) with very short axial length, and with flat faces. In other words, this optimized geometry, according to Eq. 39 in your paper seems to be much closer to Cannae's device.
 And it is actually not far from the geometry presently used by Dr. White. It is certainly not the geometry of a cone

(http://1.bp.blogspot.com/-2MORMyXf97o/VU4f2f8MtII/AAAAAAAA90w/Oc5eLpuujA4/s400/cannae_driveNASA.jpg)

However  (see below), we also conclude that the maximum possible values of the constants b (and also a) are extremely small and hence they are negligible unless the EM Drive happens to be near a magnestar.

The "lo" constant is a really huge number (unless, as you state the EM Drive happens to be located next to the field of a magnetar, which is certainly not the case, I might add, since the closest magnetar SGR 1806-20 is located about 50,000 light-years away from Earth  ;) ). 



PROOF:

Taking (from Eq. 39 in your paper)

b = (r1^2 - r2^2)/(4 (lo^2) Log[r2/r1])

Assuming r1 =<r2

Define

r1bar = r1 / lo
r2bar = r2 / lo

Then

b = (r1bar^2 - r2bar^2)/(4 Log[r2bar/r1bar])

Define:

r1bar = r2bar/c  (where c >= 1 since r1bar =< r2bar )

bb = b / (r2bar^2)

then

bb = (1 - c^2)/(4 (c^2) Log[c])

Limit[bb, c -> 1] = -  0.5  (this corresponds to the maximum possible value of r1, r1 ~ r2 )

Limit[bb, c -> 1.5] = -  0.342542

Limit[bb, c -> 2] = - 0.270505

Limit[bb, c -> Infinity] = 0  (this corresponds to the minimum possible value of r1, r1 -> 0 )

Then it follows that the maximum absolute value of bb  (bb= - 0.5) occurs at c = 1 , at r1 ~ r2
And to maximize b we must maximize r2bar, and therefore maximize r2, since b =  bb (r2bar^2)= (- 1/2) (r2bar^2)

The proof for "a' is similar but it involves an extra step

Now, since the maximum (absolute magnitude) value of b is b = (- 1/2) (r2bar^2) and we know that
r2bar = r2 / lo
then the maximum value of b is

b = (- 1/2) ((r2 / lo)^2)

but since we know that lo is a huge number (unless the EM Drive is next to a magnestar) and feasible values of r2 are such that

r2 < lo

then it necessarily follows that the maximum possible value of b is much, much smaller than 1

b  << 1

we conclude that the maximum possible values of b (and also a) are extremely small and hence they are negligible unless the EM Drive happens to be near a magnetar http://en.wikipedia.org/wiki/Magnetar.

(http://www.dailygalaxy.com/my_weblog/images/2008/09/25/magnetarfig3.jpg)

Dear Jose,

Thanks a lot for spending some time on my calculations. You tried with b but did you check for a? Also l0 contains the density of energy of the electromagnetic field (the constant U0 squared). In any case, I am aware, as my colleagues, that general relativity could not be enough. My aim is to explore this approach to evaluate the magnitude of the effects in play. If such an effect would be confirmed by independent measurements at other NASA labs, as stated in the news, it would mean that we have to cope with something real new. It would be a great result. Otherwise, Baez and others will have reasons to laugh.
Title: Re: EM Drive Developments - related to space flight applications - Thread 2
Post by: Rodal on 05/17/2015 10:48 am
...
Dear Jose,

Thanks a lot for spending some time on my calculations. You tried with b but did you check for a? Also l0 contains the density of energy of the electromagnetic field (the constant U0 squared). In any case, I am aware, as my colleagues, that general relativity could not be enough. My aim is to explore this approach to evaluate the magnitude of the effects in play. If such an effect would be confirmed by independent measurements at other NASA labs, as stated in the news, it would mean that we have to cope with something real new. It would be a great result. Otherwise, Baez and others will have reasons to laugh.
Dear Marco,

Yes, I also obtained a proof for "a" which is similar to the proof for "b".

1) I am not in my office now to reproduce my proof for "a"  (it is in another computer) but please let me know if you would like me to post the proof for "a" when I get back later today.



 I did not calculate the magnitude of "lo", I took  your word that it has a very large value unless one is close to a magnetar.

2) It would be helfpful if we could "put meat on the bone", could you please estimate a numerical value for "lo" (length in meters) so we can have a numerical idea of the small size of this effect ?

Thank for spending the time to publish your report, as the only way to understand the claims of the researchers is to conduct detailed analyses like yours.

Regards
Title: Re: EM Drive Developments - related to space flight applications - Thread 2
Post by: deuteragenie on 05/17/2015 11:47 am
4) Also this link may be helpful, giving a table of the Xmn and  X'mn to 15 digits accuracy
http://wwwal.kuicr.kyoto-u.ac.jp/www/accelerator/a4/besselroot.htmlx

Try:

 Wolfram alpha  (https://www.wolframalpha.com/input/?i=besseljzero[1%2C1)
Thanks.  That link ( https://www.wolframalpha.com/input/?i=besseljzero[1%2C1  ) only gives Xmn which are only useful for TM modes

Is there a way that Wolfram Alpha can give  X'mn which are needed for TE modes ?

I don't think so. But I believe you have Mathemagica:
http://www.me.rochester.edu/courses/ME201/webexamp/derbesszer.pdf - see ln67
OK, yes, that's helpful to the people that have Mathematica.  They can program that function ( notice that X'mn is not a buil-in function in Mathematica yet).  To those that don't have Mathematica, that's the great usefulness of this link http://wwwal.kuicr.kyoto-u.ac.jp/www/accelerator/a4/besselroot.htmlx: it gives X'mn to 15 digits accuracy and it is accessible to everyone. 

The Kyoto (Japan) University link only goes to m=10, n=5 though, so if anyone knows of a link going to a higher quantum number (n > 5) , please post it, as it may be helpful to people just using Excel.  (In case anyone is interested in investigating super-high mode shapes for n > 5  :)

You can also try to solve the derivate probably.  Something like this:
"Using secant method solve derivate(J11(x))=0 at x1=34 and x2=36" will return x  =  35.16671442739261
For m=11 and n= ... , which does not appear in the Kyoto table.  Looks like alpha will revert to machine precision instead of arbitrary precision.  This is crying for an updated Kyoto table :)
Title: Re: EM Drive Developments - related to space flight applications - Thread 2
Post by: TheTraveller on 05/17/2015 12:03 pm
http://www.ibtimes.co.uk/emdrive-warp-drive-are-two-different-things-nasas-still-working-emdrive-1501268

Interesting comments from Shawyer are attached:
Title: Re: EM Drive Developments - related to space flight applications - Thread 2
Post by: txdrive on 05/17/2015 12:36 pm
Assuming that EagleWorks can set up a magnetron powered experiment that is substantially more precise than Shawyer's, the force can be expected to be proportionally less. There's also a good chance that it would point in the opposite direction.

So it is imminent that Shawyer will need to disregard such results, so he started saying things like "Obviously I'm very happy for Nasa, they're having great fun, but it's not really real science," Shawyer told IBTimes UK in an exclusive interview." (from an older article (http://www.ibtimes.co.uk/nasa-validates-emdrive-roger-shawyer-says-aerospace-industry-needs-watch-out-1499141)).
Title: Re: EM Drive Developments - related to space flight applications - Thread 2
Post by: StrongGR on 05/17/2015 01:04 pm
http://www.ibtimes.co.uk/emdrive-warp-drive-are-two-different-things-nasas-still-working-emdrive-1501268

Interesting comments from Shawyer are attached:

Aside from such declarations, what I regret for is a dismissal a priori of an object really easy to test, with just few bucks. At the time of cold fusion scientific community was much more open minded. Important labs tried to repeat the experiment with the help of Fleishmann himself. Now, there are nor secrets neither patents at stake. It seems like we learnt to behave like Bellarmino at Galileo time where it was far too easy to look at the sky with the scope, but hard to admit to have been wrong and dismiss a lot of acquired "knowledge".
Title: Re: EM Drive Developments - related to space flight applications - Thread 2
Post by: Rodal on 05/17/2015 01:08 pm
I agree, this statement:

Quote from: ibtimes uk
Roger Shawyer, the British scientist who invented the highly controversial electromagnetic space propulsion technology called EmDrive, has said Nasa's work is encouraging but still far behind many private firms working on it already.

"Obviously I'm very happy for Nasa, they're having great fun, but it's not really real science," Shawyer told IBTimes UK in an exclusive interview.

"Obviously I'm very happy for Nasa, they're having great fun, but it's not really real science,"
Shawyer told IBTimes UK in an exclusive interview.

is a very strange quote (for Shawyer to be quoted in http://www.ibtimes.co.uk/nasa-validates-emdrive-roger-shawyer-says-aerospace-industry-needs-watch-out-1499141 as saying that "NASA is just having great fun, but the experiments at NASA are not really science" in Shawyer's opinion).

Which companies are the many private companies (besides SPR) working on EM Drive that are far ahead of NASA in the opinion of Mr. Shawyer ?

If, what NASA is doing is just "great fun" but "not real science" in Shawyer's opinion, in which peer-reviewed scientific journal have these EM Drive "real scientific experiments" that Shawyer is referring to, been documented?
Title: Re: EM Drive Developments - related to space flight applications - Thread 2
Post by: StrongGR on 05/17/2015 01:19 pm
I am back with an updated draft after some terrible news around about NASA dismissing these researches. They should not as, otherwise, it could happen as with Galilei having his detractors even not trying to look in the telescope, just dismissing on faith.

I have analysed the case of the frustum and the results appear to be striking. One must admit that geometry comes to rescue not just general relativity. For this particular geometry the cavity can be made susceptible to gravitational effects if your choice of the two radii of the cavity is smart enough. This is something to be confirmed yet, just my theoretical result, but shocking anyway.

As usual, any comment is very welcome.

http://forum.nasaspaceflight.com/index.php?action=dlattach;topic=36313.0;attach=830137

Marco, your paper, after equation 39, has this interesting comment:

Quote from: Frasca
These equations appear rather interesting as, by a proper choice of parameters, one can make a gravitational effect more or less relevant in the physics of the problem. It is the case to say that geometry comes to rescue.


Just what this "proper choice of parameters" is, it is not spelled out in your paper, so I will follow my interpretation of your equation. 

It appears (see proof below) that in order to maximize the constants "a" and "b" in Equation 39, we want to have:

1) r1 as close as possible to r2
2) r2 as large as possible


Now, if this is correct, this is a rather peculiar, surprising geometry: it says that the axial length of the truncated cone should be close to zero while the radius should be as large as possible, in other words, the geometry should be as close to an almost perfect cylinder (small cone angle) with very short axial length, and with flat faces. In other words, this optimized geometry, according to Eq. 39 in your paper seems to be much closer to Cannae's device.
 And it is actually not far from the geometry presently used by Dr. White. It is certainly not the geometry of a cone

(http://1.bp.blogspot.com/-2MORMyXf97o/VU4f2f8MtII/AAAAAAAA90w/Oc5eLpuujA4/s400/cannae_driveNASA.jpg)

However  (see below), we also conclude that the maximum possible values of the constants b (and also a) are extremely small and hence they are negligible unless the EM Drive happens to be near a magnestar.

The "lo" constant is a really huge number (unless, as you state the EM Drive happens to be located next to the field of a magnetar, which is certainly not the case, I might add, since the closest magnetar SGR 1806-20 is located about 50,000 light-years away from Earth  ;) ). 



PROOF:

Taking (from Eq. 39 in your paper)

b = (r1^2 - r2^2)/(4 (lo^2) Log[r2/r1])

Assuming r1 =<r2

Define

r1bar = r1 / lo
r2bar = r2 / lo

Then

b = (r1bar^2 - r2bar^2)/(4 Log[r2bar/r1bar])

Define:

r1bar = r2bar/c  (where c >= 1 since r1bar =< r2bar )

bb = b / (r2bar^2)

then

bb = (1 - c^2)/(4 (c^2) Log[c])

Limit[bb, c -> 1] = -  0.5  (this corresponds to the maximum possible value of r1, r1 ~ r2 )

Limit[bb, c -> 1.5] = -  0.342542

Limit[bb, c -> 2] = - 0.270505

Limit[bb, c -> Infinity] = 0  (this corresponds to the minimum possible value of r1, r1 -> 0 )

Then it follows that the maximum absolute value of bb  (bb= - 0.5) occurs at c = 1 , at r1 ~ r2
And to maximize b we must maximize r2bar, and therefore maximize r2, since b =  bb (r2bar^2)= (- 1/2) (r2bar^2)

The proof for "a' is similar but it involves an extra step

Now, since the maximum (absolute magnitude) value of b is b = (- 1/2) (r2bar^2) and we know that
r2bar = r2 / lo
then the maximum value of b is

b = (- 1/2) ((r2 / lo)^2)

but since we know that lo is a huge number (unless the EM Drive is next to a magnestar) and feasible values of r2 are such that

r2 < lo

then it necessarily follows that the maximum possible value of b is much, much smaller than 1

b  << 1

we conclude that the maximum possible values of b (and also a) are extremely small and hence they are negligible unless the EM Drive happens to be near a magnetar http://en.wikipedia.org/wiki/Magnetar.

(http://www.dailygalaxy.com/my_weblog/images/2008/09/25/magnetarfig3.jpg)

I have plotted with Maple the function bb(c)=(1-c^2)/(4*c^2*ln(c)) and I have found the figure I enclose. There is no maximum whatsoever. This curve has an asymptote at increasing c and another for c going to zero. This is what I meant saying that geometry comes to rescue.  I am currently working out all this with Mathematica and I will update my draft as soon as possible with realistic values. Of course, my take remains the same as yours, if the effect exists Einstein theory could not be enough.
Title: Re: EM Drive Developments - related to space flight applications - Thread 2
Post by: TheTraveller on 05/17/2015 01:23 pm
I agree, this statement:

Quote from: ibtimes uk
has said Nasa's work is encouraging but still far behind many private firms working on it already.

"Obviously I'm very happy for Nasa, they're having great fun, but it's not really real science,"
Shawyer told IBTimes UK in an exclusive interview.

is a very, very strange, not objective comment (for Shawyer to be quoted in http://www.ibtimes.co.uk/nasa-validates-emdrive-roger-shawyer-says-aerospace-industry-needs-watch-out-1499141 as saying that "NASA is just having great fun, but the experiments at NASA are not really science" in Shawyer's opinion)

Believe Shawyer was saying that EWs efforts, using EM Drive to search for a QV theory proof, was not about understanding the EM Drive nor testing it properly.

Will be very interesting to see what Shawyer and others post in 2015.
Title: Re: EM Drive Developments - related to space flight applications - Thread 2
Post by: Rodal on 05/17/2015 01:25 pm
....
I have plotted with Maple the function bb(c)=(1-c^2)/(4*c^2*ln(c)) and I have found the figure I enclose. There is no maximum whatsoever. This curve has an asymptote at increasing c and another for c going to zero. This is what I meant saying that geometry comes to rescue.  I am currently working out all this with Mathematica and I will update my draft as soon as possible with realistic values. Of course, my take remains the same as yours, if the effect exists Einstein theory could not be enough.
(http://forum.nasaspaceflight.com/index.php?action=dlattach;topic=36313.0;attach=830617;image)

Incorrect results follow from plotting variables over incorrect range of values of the variable.

The variable "c" valid range of values is from 1 to Infinity, instead of from 0 to 1 as you plotted above.

Take a gander at my proof  http://forum.nasaspaceflight.com/index.php?topic=36313.msg1375279#msg1375279.

I had explicitly stated:

r1bar = r2bar/c  (where c >= 1 since r1bar =< r2bar )


One has to assume that either r1 is smaller than r2 or that r2 is smaller than r1.

In your paper ( http://forum.nasaspaceflight.com/index.php?action=dlattach;topic=36313.0;attach=830137 ) you assumed that r1 is smaller than r2, both in the integral and also in the expression showing the term (r2-r1)/h, in your equations 34 and 38 of your paper.

in order for r1 to be equal or less than r2, c must be equal to or greater than 1.

Instead you have plotted c for values smaller than 1.

Your plot shows values for c in the wrong range. c is defined to have values from 1 to Infinity, instead of from 0 to 1 as you plotted.


The values of c smaller than 1 you plotted are in conflict with the assumption that r1 is smaller than r2 (which was explicitly defined). 

To be consistent with the assumption that r1 is smaller than r2, you should plot values of c equal to or greater than 1.
Title: Re: EM Drive Developments - related to space flight applications - Thread 2
Post by: txdrive on 05/17/2015 01:37 pm
http://www.ibtimes.co.uk/emdrive-warp-drive-are-two-different-things-nasas-still-working-emdrive-1501268

Interesting comments from Shawyer are attached:

Aside from such declarations, what I regret for is a dismissal a priori of an object really easy to test, with just few bucks.
It's not a-priori. It's a-posteriori (http://en.wikipedia.org/wiki/A_priori_and_a_posteriori). We didn't know a-priori that it does not work. We know from direct measurements that EM fields behave in a specific manner, down to parts per trillion. Momentum which is carried by an electromagnetic field can be measured directly as the field strength, with far greater precision. This knowledge puts a very low upper bound on the net momentum that EM fields can acquire and exchange with the cavity.

Hence the knowledge that his drive does not work is a-posteriori knowledge. If his force was in low nano or pico newtons range, it would have been an a-priori assumption.
Quote
At the time of cold fusion scientific community was much more open minded. Important labs tried to repeat the experiment with the help of Fleishmann himself.
Scientific community's open mindedness towards a claim depends mainly to the claim itself.
Quote
Now, there are nor secrets neither patents at stake. It seems like we learnt to behave like Bellarmino at Galileo time where it was far too easy to look at the sky with the scope, but hard to admit to have been wrong and dismiss a lot of acquired "knowledge".
How can you compare a person who received a grant from the British government to Galileo?
Title: Re: EM Drive Developments - related to space flight applications - Thread 2
Post by: TheTraveller on 05/17/2015 01:52 pm
We know from direct measurements that EM fields behave in a specific manner, down to parts per trillion. Momentum which is carried by an electromagnetic field can be measured directly as the field strength, with far greater precision. This knowledge puts a very low upper bound on the net momentum that EM fields can acquire and exchange with the cavity.

We also know that a high Q cavity of say 50,000 will do 50,000 bounces of the em field, each adding to the generated force. Shawyer's Force equation of (2 Po Df Q) / c clearly states Q is the way the EM Drive multiplies the very low force of one way to a much higher value.

Same effect used with laser thruster:
http://en.wikipedia.org/wiki/Photonic_laser_thruster
Title: Re: EM Drive Developments - related to space flight applications - Thread 2
Post by: StrongGR on 05/17/2015 01:53 pm
....
I have plotted with Maple the function bb(c)=(1-c^2)/(4*c^2*ln(c)) and I have found the figure I enclose. There is no maximum whatsoever. This curve has an asymptote at increasing c and another for c going to zero. This is what I meant saying that geometry comes to rescue.  I am currently working out all this with Mathematica and I will update my draft as soon as possible with realistic values. Of course, my take remains the same as yours, if the effect exists Einstein theory could not be enough.
(http://forum.nasaspaceflight.com/index.php?action=dlattach;topic=36313.0;attach=830617;image)

Take a gander at my proof  http://forum.nasaspaceflight.com/index.php?topic=36313.msg1375279#msg1375279.

I had explicitly stated:

r1bar = r2bar/c  (where c >= 1 since r1bar =< r2bar )


One has to assume that either r1 is smaller than r2 or that r2 is smaller than r1.

In your paper ( http://forum.nasaspaceflight.com/index.php?action=dlattach;topic=36313.0;attach=830137 ) you assumed that r1 is smaller than r2, both in the integral and also in the expression showing the term (r2-r1)/h, in your equations 34 and 38 of your paper.

in order for r1 to be equal or less than r2, c must be equal to or greater than 1.

Instead you have plotted c for values smaller than 1.

Your plot shows values for c in the wrong range. c is defined to have values from 1 to Infinity, instead of from 0 to 1 as you plotted.


The values of c smaller than 1 you plotted are in conflict with the assumption that r1 is smaller than r2 (which was explicitly defined). 

To be consistent with the assumption that r1 is smaller than r2, you should plot values of c equal to or greater than 1.

Your assumption is not needed as this function retains his sign independently on your choice. In any case, there are not extrema and this function runs to zero on the right and runs to minus infinity on the left. You can verify yourself that the function is symmetric under the exchange r1<->r2.
Title: Re: EM Drive Developments - related to space flight applications - Thread 2
Post by: StrongGR on 05/17/2015 01:56 pm
http://www.ibtimes.co.uk/emdrive-warp-drive-are-two-different-things-nasas-still-working-emdrive-1501268

Interesting comments from Shawyer are attached:

Aside from such declarations, what I regret for is a dismissal a priori of an object really easy to test, with just few bucks.
It's not a-priori. It's a-posteriori (http://en.wikipedia.org/wiki/A_priori_and_a_posteriori). We didn't know a-priori that it does not work. We know from direct measurements that EM fields behave in a specific manner, down to parts per trillion. Momentum which is carried by an electromagnetic field can be measured directly as the field strength, with far greater precision. This knowledge puts a very low upper bound on the net momentum that EM fields can acquire and exchange with the cavity.

Hence the knowledge that his drive does not work is a-posteriori knowledge. If his force was in low nano or pico newtons range, it would have been an a-priori assumption.
Quote
At the time of cold fusion scientific community was much more open minded. Important labs tried to repeat the experiment with the help of Fleishmann himself.
Scientific community's open mindedness towards a claim depends mainly to the claim itself.
Quote
Now, there are nor secrets neither patents at stake. It seems like we learnt to behave like Bellarmino at Galileo time where it was far too easy to look at the sky with the scope, but hard to admit to have been wrong and dismiss a lot of acquired "knowledge".
How can you compare a person who received a grant from the British government to Galileo?

I am not comparing this guy to Galileo, rather I consider that, in this case, a real small effort by a couple of serious labs can decide if he is a charlatan or not.
Title: Re: EM Drive Developments - related to space flight applications - Thread 2
Post by: Raj2014 on 05/17/2015 02:00 pm
Assuming the EM drive has passed peer review and has had many tests done to verify it works. What will be the next step?
Title: Re: EM Drive Developments - related to space flight applications - Thread 2
Post by: Rodal on 05/17/2015 02:03 pm
....
Your assumption is not needed as this function retains his sign independently on your choice. In any case, there are not extrema and this function runs to zero on the right and runs to minus infinity on the left. You can verify yourself that the function is symmetric under the exchange r1<->r2.
Sorry, but we cannot have a logical, mathematical discussion where you first write equations where r1 is assumed to be less than r2 (as Eq. 34 and 38 of your paper) and now you proceed to show a graph with a range of variables in conflict with YOUR assumption that r1 was smaller than r2.  That is inconsistent.  Incorrect results follow from inconsistencies in formulation.

I suggest that you:

1) make a plot of the geometry of the truncated cone you have in mind: clearly showing what you define to be r1, r2, h, and the z and r coordinates of your truncated cone.

2) explicitly show what is the optimal geometry of the truncated cone, as per your paper.

Further discussion without you clearly, and consistently, defining, and consistently using, your geometrical variables is moot and can lead nowhere, as your paper assumes r1 <r2 and now you show plots for r1 > r2, in strong conflict with your assumed geometry and equations

 :)
Title: Re: EM Drive Developments - related to space flight applications - Thread 2
Post by: Einstein79 on 05/17/2015 02:10 pm
Does the Flight Thruster have a slightly concave top and convex bottom? Would appear so from the gaps.

Enhanced the photo as much as I can for those wishing to try to extract dimensions as this photo is better that the original as it has no distortion.

If we can find the dimension<M

The big end most certainly should be convex and the small end concave, relative from the outside of course. The big end and small end radii should not be coincidence but offset having the small end radius much larger than the big end. In fact, it might be better for the small end to be flat. 
Title: Re: EM Drive Developments - related to space flight applications - Thread 2
Post by: StrongGR on 05/17/2015 02:12 pm
....
Your assumption is not needed as this function retains his sign independently on your choice. In any case, there are not extrema and this function runs to zero on the right and runs to minus infinity on the left. You can verify yourself that the function is symmetric under the exchange r1<->r2.
Sorry, but you cannot consistently first write equations where r1 is assumed to be less than r2 (as per your paper) and now proceed to show a graph in conflict with YOUR assumption that r1 was smaller than r2.  That is inconsistent.  Incorrect results follow from inconsistencies in formulation.

I suggest that you:

1) make a plot of the geometry of the truncated cone you have in mind: clearly showing what you define to be r1, r2, h, and the z and r coordinates

2) explicitly show what is the optimal geometry of the truncated cone, as per your paper.

Further discussion without you clearly defining your geometrical variables is moot and can lead nowhere, as your paper assumes r1 <r2 and now you show plots for r1 > r2, in strong conflict with your assumed geometry and equations

 :)

Jose,

I am not arguing against my paper. You know may take. I am questioning your proof based on a functional form that has the mathematical properties I have shown. I know by myself, and I stated it in the draft, that here we are working with really small values. As you can see, I have not attempted any computation of a force, if any, or thrust. I am thinking about this and I am trying to work with real values. But please note also that this is a function of three independent variables: r1, r2 and U0.
Title: Re: EM Drive Developments - related to space flight applications - Thread 2
Post by: Rodal on 05/17/2015 02:17 pm
....
Jose,

I am not arguing against my paper. You know may take. I am questioning your proof based on a functional form that has the mathematical properties I have shown. I know by myself, and I stated it in the draft, that here we are working with really small values. As you can see, I have not attempted any computation of a force, if any, or thrust. I am thinking about this and I am trying to work with real values. But please note also that this is a function of three independent variables: r1, r2 and U0.
Marco,

Please

1) make a plot (you can sketch the geometry by hand with pencil and paper, scan it and post the image  :) ) of the geometry of the truncated cone you have in mind: clearly showing what you define to be r1, r2, h, and the z and r coordinates

2) explicitly show what is the optimal geometry of the truncated cone, as per your paper.

and then we can continue  :)
Title: Re: EM Drive Developments - related to space flight applications - Thread 2
Post by: StrongGR on 05/17/2015 02:28 pm
....
Jose,

I am not arguing against my paper. You know may take. I am questioning your proof based on a functional form that has the mathematical properties I have shown. I know by myself, and I stated it in the draft, that here we are working with really small values. As you can see, I have not attempted any computation of a force, if any, or thrust. I am thinking about this and I am trying to work with real values. But please note also that this is a function of three independent variables: r1, r2 and U0.
Marco,

Please

1) make a plot (you can sketch the geometry by hand with pencil and paper, scan it and post the image  :) ) of the geometry of the truncated cone you have in mind: clearly showing what you define to be r1, r2, h, and the z and r coordinates

2) explicitly show what is the optimal geometry of the truncated cone, as per your paper.

and then we can continue  :)

I think you are completely off the target. What the heck we have to continue? You can choice r2>r1 and live happily with that or the other way around. Choose what you prefer and keep it. There is no claim to support here. It is also difficult for me to understand what you believe to have proven. There is no error there and you can do any computation you like with that. This is a real wasting of time.
Title: Re: EM Drive Developments - related to space flight applications - Thread 2
Post by: txdrive on 05/17/2015 02:33 pm
We know from direct measurements that EM fields behave in a specific manner, down to parts per trillion. Momentum which is carried by an electromagnetic field can be measured directly as the field strength, with far greater precision. This knowledge puts a very low upper bound on the net momentum that EM fields can acquire and exchange with the cavity.

We also know that a high Q cavity of say 50,000 will do 50,000 bounces of the em field, each adding to the generated force. Shawyer's Force equation of (2 Po Df Q) / c clearly states Q is the way the EM Drive multiplies the very low force of one way to a much higher value.

Same effect used with laser thruster:
http://en.wikipedia.org/wiki/Photonic_laser_thruster
2/c, which is 6.6 uN/kW , times 50 000 , is 0.33 N/kW . Multiply by the bound on EM exchanging momentum with something unknown, a very small force (nanoNewtons). edit: typo.
Title: Re: EM Drive Developments - related to space flight applications - Thread 2
Post by: Rodal on 05/17/2015 02:38 pm
...There is no claim to support here. It is also difficult for me to understand what you believe to have proven. There is no error there and you can do any computation you like with that. This is a real wasting of time.

Marco,

1) You wrote a very interesting paper, concluding that there is a geometry (I presume an optimal geometry) of a truncated cone that "comes to the rescue" which I assume it to mean that maximizes the General Relatvity effect in this cavity.

2) My proof, based on your paper shows that that optimal geometry is a cylinder with flat faces and negligible axial length.  The optimal geometry according to the equations in your paper being much closer to the Cannae cavity and the pillbox shape used by Dr. White than the geometry of a conical cavity.

3) What is the actual optimal geometry of the cavity to maximize the GR effect, according to your paper? I think that for you showing what is this optimal geometry would be great conclusion to your interesting paper.
Title: Re: EM Drive Developments - related to space flight applications - Thread 2
Post by: StrongGR on 05/17/2015 02:42 pm
....
Jose,

I am not arguing against my paper. You know may take. I am questioning your proof based on a functional form that has the mathematical properties I have shown. I know by myself, and I stated it in the draft, that here we are working with really small values. As you can see, I have not attempted any computation of a force, if any, or thrust. I am thinking about this and I am trying to work with real values. But please note also that this is a function of three independent variables: r1, r2 and U0.
Marco,

Please

1) make a plot (you can sketch the geometry by hand with pencil and paper, scan it and post the image  :) ) of the geometry of the truncated cone you have in mind: clearly showing what you define to be r1, r2, h, and the z and r coordinates

2) explicitly show what is the optimal geometry of the truncated cone, as per your paper.

and then we can continue  :)

I think you are completely off the target. What the heck we have to continue? You can choice r2>r1 and live happily with that or the other way around. Choose what you prefer and keep it. There is no claim to support here. It is also difficult for me to understand what you believe to have proven. There is no error there and you can do any computation you like with that. This is a real wasting of time.

Marco,

1) You wrote a very interesting paper, concluding that there is a geometry (I presume an optimal geometry) of a truncated cone that "comes to the rescue" which I assume it to mean that maximizes the General Relatvity effect in this cavity.

2) My proof, based on your paper shows that that optimal geometry is a cylinder with flat faces and negligible axial length.  The optimal geometry according to the equations in your paper being much closer to the Cannae cavity and the pillbox shape used by Dr. White than the geometry of a conical cavity.

3) What is the actual optimal geometry of the cavity to maximize the GR effect, according to your paper? I think that for you showing what is this optimal geometry would be great conclusion to your interesting paper.

But this is exactly what I am doing now to update my draft. Mathematica is running computing a realistic k with the values r1 and r2 taken from Minotti's paper. I need to estimate U0. In a few days I can give an answer to your question.
Title: Re: EM Drive Developments - related to space flight applications - Thread 2
Post by: Notsosureofit on 05/17/2015 02:47 pm
Algebra aside, I believe what Eq. 37 should represent is the modification of L() such the terms w/ a and b give the amplitude of the observable sidebands outside of a frustum cavity as opposed to a cylindrical one. (I'm not yet to being able to comment on the correct expressions for a or b)
Title: Re: EM Drive Developments - related to space flight applications - Thread 2
Post by: Rodal on 05/17/2015 02:49 pm
...
But this is exactly what I am doing now to update my draft. Mathematica is running computing a realistic k with the values r1 and r2 taken from Minotti's paper. I need to estimate U0. In a few days I can give an answer to your question.

Suggestion: to use the geometrical values used by Greg Egan  r1=2.5 cm, r2=10 cm, θw = 20° here: http://gregegan.customer.netspace.net.au/SCIENCE/Cavity/Cavity.html as Minotti quotes Egan's equations in Minotti's paper, and Egan shows explicit field solutions and the energy for those geometrical parameters.
Title: Re: EM Drive Developments - related to space flight applications - Thread 2
Post by: TheTraveller on 05/17/2015 02:53 pm
Does the Flight Thruster have a slightly concave top and convex bottom? Would appear so from the gaps.

Enhanced the photo as much as I can for those wishing to try to extract dimensions as this photo is better that the original as it has no distortion.

If we can find the dimension<M

The big end most certainly should be convex and the small end concave, relative from the outside of course. The big end and small end radii should not be coincidence but offset having the small end radius much larger than the big end. In fact, it might be better for the small end to be flat.

This is how Shawyer does it.

The end plate curve at each end should be the same radius as the curve of the em field as it expands outward. So not a flat wave front as some see but curved.

Which also means the curved wave front is always at right angles to the cone side walls as the wavelength increases as the cavity diameter increases, increasing the group velocity as well.

Shawyer did say the wave slides up and down the cavity walls and exerts no significant force on it. From this diagram, that seems to be the case. It does slide up and down the cavity walls, while always being at a right angle to it.

Then it hits one of the curved end plates and does a bounce back the other way.
Title: Re: EM Drive Developments - related to space flight applications - Thread 2
Post by: StrongGR on 05/17/2015 02:55 pm
...
But this is exactly what I am doing now to update my draft. Mathematica is running computing a realistic k with the values r1 and r2 taken from Minotti's paper. I need to estimate U0. In a few days I can give an answer to your question.

Suggestion: to use the geometrical values used by Greg Egan  r1=2.5 cm, r2=10 cm, θw = 20° here: http://gregegan.customer.netspace.net.au/SCIENCE/Cavity/Cavity.html as Minotti quotes Egan's equations in Minotti's paper, and Egan shows explicit field solutions and the energy for those geometrical parameters.

I am using Minotti's values r1=18 cm and r2=36 cm and imposing boundary conditions one of which is P_n(cos(theta_0))=0.
Title: Re: EM Drive Developments - related to space flight applications - Thread 2
Post by: HypergolicSunrise on 05/17/2015 02:59 pm
I just want to throw this up here:
http://www.nature.com/ncomms/2014/140306/ncomms4300/full/ncomms4300.html (http://www.nature.com/ncomms/2014/140306/ncomms4300/full/ncomms4300.html)
http://arxiv.org/ftp/arxiv/papers/1308/1308.0547.pdf (http://arxiv.org/ftp/arxiv/papers/1308/1308.0547.pdf)

I think if you're looking in classical physics for answers, you won't find them.
Title: Re: EM Drive Developments - related to space flight applications - Thread 2
Post by: TheTraveller on 05/17/2015 02:59 pm
We know from direct measurements that EM fields behave in a specific manner, down to parts per trillion. Momentum which is carried by an electromagnetic field can be measured directly as the field strength, with far greater precision. This knowledge puts a very low upper bound on the net momentum that EM fields can acquire and exchange with the cavity.

We also know that a high Q cavity of say 50,000 will do 50,000 bounces of the em field, each adding to the generated force. Shawyer's Force equation of (2 Po Df Q) / c clearly states Q is the way the EM Drive multiplies the very low force of one way to a much higher value.

Same effect used with laser thruster:
http://en.wikipedia.org/wiki/Photonic_laser_thruster
2/c, which is 6.6 uN/kW , times 50 000 , is 0.33 N/kW . Multiply by the bound on EM exchanging momentum with something unknown, a very small force (nanoNewtons). edit: typo.

T = (2 * 100W * 50,000 Q * Df 0.5) / c = 16.6mN
Title: Re: EM Drive Developments - related to space flight applications - Thread 2
Post by: Rodal on 05/17/2015 03:00 pm
....

Shawyer did say the wave slides up and down the cavity walls and exerts no significant force on it. From this diagram, that seems to be the case. It does slide up and down the cavity walls, while always being at a right angle to it.
...
Greg Egan first considered the spherical ends, as a response to Shawyer's article with flat faces in the New Scientist article.

See http://gregegan.customer.netspace.net.au/SCIENCE/Cavity/Cavity.html where Greg Egan shows that the stresses on the inner conical surfaces are not negligible: following Maxwell's equations they exactly balance out the stresses on the spherical faces to end with zero net force on the EM Drive.

Did Shawyer publish a paper answering in detail the objections raised by Greg Egan's ?
Title: Re: EM Drive Developments - related to space flight applications - Thread 2
Post by: StrongGR on 05/17/2015 03:07 pm
....

Shawyer did say the wave slides up and down the cavity walls and exerts no significant force on it. From this diagram, that seems to be the case. It does slide up and down the cavity walls, while always being at a right angle to it.
...
Greg Egan first considered the spherical ends, as a response to Shawyer's article with flat faces in the New Scientist article.

See http://gregegan.customer.netspace.net.au/SCIENCE/Cavity/Cavity.html where Greg Egan shows that the stresses on the inner conical surfaces are not negligible: following Maxwell's equations they exactly balance out the stresses on the spherical faces to end with zero net force on the EM Drive.

Did Shawyer publish a paper answering in detail the objections raised by Greg Egan's ?

I have that link constantly open and all the results, well almost all, are in the Minotti's paper. The point is that Shawyer never answered a single question put out by the community, being Egan or whoever else. Most of them think he is a charlatan.
Title: Re: EM Drive Developments - related to space flight applications - Thread 2
Post by: TheTraveller on 05/17/2015 03:12 pm
....

Shawyer did say the wave slides up and down the cavity walls and exerts no significant force on it. From this diagram, that seems to be the case. It does slide up and down the cavity walls, while always being at a right angle to it.
...
Greg Egan first considered the spherical ends, as a response to Shawyer's article with flat faces in the New Scientist article.

See http://gregegan.customer.netspace.net.au/SCIENCE/Cavity/Cavity.html where Greg Egan shows that the stresses on the inner conical surfaces are not negligible: following Maxwell's equations they exactly balance out the stresses on the spherical faces to end with zero net force on the EM Drive.

Did Shawyer publish a paper answering in detail the objections raised by Greg Egan's ?

How can an EM wave, acting at a right angle with the surface it is in contact with, exert any force on it? Did Egan use a flat wave front or a curved wave front at 90 deg to the side cavity wall for the interaction force?

Why would Shawyer object to what Egan wrote? Shawyer is not a theoretical guy, not in the business of theory point scoring or needs to publish or die. He is working from measured data from a device that produces thrust and the level of that thrust matches the equations he created to model it.

Shawyer published this measured thrust as did the Chinese and why they think it does what it does. Someone who publishes a paper claiming it can't work because of their theory, just might be politely ignored as they get on with business.

Title: Re: O09
Post by: TheTraveller on 05/17/2015 03:21 pm
Does the Flight Thruster have a slightly concave top and convex bottom? Would appear so from the gaps.

Enhanced the photo as much as I can for those wishing to try to extract dimensions as this photo is better that the original as it has no distortion.

If we can find the dimension<M

The big end most certainly should be convex and the small end concave, relative from the outside of course. The big end and small end radii should not be coincidence but offset having the small end radius much larger than the big end. In fact, it might be better for the small end to be flat.

If either end is flat, the bounce will introduce very significant phase distortion into the returning curved wave. For me it is hard to see that Shawyer ever used flat end plates INSIDE the cavity. As we never saw inside the cavity, what is to say he didn't use curved end plates inside and flat end covers outside? What he drew may not be what he built.
Title: Re: EM Drive Developments - related to space flight applications - Thread 2
Post by: Rodal on 05/17/2015 03:25 pm
...
Shawyer published this measured thrust as did the Chinese and why they think it does what it does. Someone who publishes a paper claiming it can't work because of their theory, just might be politely ignored as they get on with business.
Greg Egan did not claim  that the EM Drive cannot work because of a theory invented by Greg Egan. 

It is not his theory.  Greg Egan is showing again a well-known result.

Instead, Greg Egan showed an already known-proof that the stresses due to standing waves on the inner walls of a resonating cavity of any arbitrary shape whatsoever perfectly balance, and therefore there is zero net force in any direction, according to Maxwell's equations.

There are some ways out of this that have been discussed in this thread.  The curious thing to me is that Shawyer appears to be the only one still stating that the EM Drive can accelerate by itself just using Maxwell's equations and Special Relativity.
Title: Re: EM Drive Developments - related to space flight applications - Thread 2
Post by: StrongGR on 05/17/2015 03:31 pm
...
Shawyer published this measured thrust as did the Chinese and why they think it does what it does. Someone who publishes a paper claiming it can't work because of their theory, just might be politely ignored as they get on with business.
Greg Egan did not claim  that the EM Drive cannot work because of a theory invented by Greg Egan. 

It is not his theory.  Greg Egan is showing again a well-known result.

Instead, Greg Egan showed again, an already known-proof that the stresses due to standing waves on the inner walls of a resonating cavity of any arbitrary shape whatsoever perfectly balance, and therefore there is zero net force in any direction, according to Maxwell's equations.

Egan exploited the computations of what the community only stated: Momentum is not conserved and so this does not work. This is the reason why we are looking elsewhere than only Maxwell equations.
Title: Re: EM Drive Developments - related to space flight applications - Thread 2
Post by: TheTraveller on 05/17/2015 03:31 pm
...
Shawyer published this measured thrust as did the Chinese and why they think it does what it does. Someone who publishes a paper claiming it can't work because of their theory, just might be politely ignored as they get on with business.
Greg Egan did not claim  that the EM Drive cannot work because of a theory invented by Greg Egan.  Instead, Greg Egan showed again, a known-proof that the stresses on the inner walls of a resonating cavity of any arbitrary shape whatsoever (as long as it is closed) perfectly balance, and therefore there is zero net force in any direction, according to Maxwell's equations.

If you can, please answer my simple question:

How can a curved EM wave, as in the attachment, touching the cavity wall at right angles to the wall, cause any force to be generated on the wall?
Title: Re: EM Drive Developments - related to space flight applications - Thread 2
Post by: Rodal on 05/17/2015 03:34 pm
...
Shawyer published this measured thrust as did the Chinese and why they think it does what it does. Someone who publishes a paper claiming it can't work because of their theory, just might be politely ignored as they get on with business.
Greg Egan did not claim  that the EM Drive cannot work because of a theory invented by Greg Egan.  Instead, Greg Egan showed again, a known-proof that the stresses on the inner walls of a resonating cavity of any arbitrary shape whatsoever (as long as it is closed) perfectly balance, and therefore there is zero net force in any direction, according to Maxwell's equations.

If you can, please answer my simple question:

How can a curved EM wave, as in the attachment, touching the cavity wall at right angles to the wall, cause any force to be generated on the wall?

Because the solution of Maxwell's differential equations with the appropriate Boundary Conditions demand so.

An EM wave that causes no stresses on the conical inner surfaces does NOT satisfy the boundary conditions.
Title: Re: EM Drive Developments - related to space flight applications - Thread 2
Post by: StrongGR on 05/17/2015 03:34 pm
...
Shawyer published this measured thrust as did the Chinese and why they think it does what it does. Someone who publishes a paper claiming it can't work because of their theory, just might be politely ignored as they get on with business.
Greg Egan did not claim  that the EM Drive cannot work because of a theory invented by Greg Egan.  Instead, Greg Egan showed again, a known-proof that the stresses on the inner walls of a resonating cavity of any arbitrary shape whatsoever (as long as it is closed) perfectly balance, and therefore there is zero net force in any direction, according to Maxwell's equations.

If you can, please answer my simple question:

How can a curved EM wave, as in the attachment, touching the cavity wall at right angles to the wall, cause any force to be generated on the wall?

You cannot stare at a pictorial representation claiming something is there. Physics means mathematics and mathematics, through Maxwell equations, says no net momentum.
Title: Re: EM Drive Developments - related to space flight applications - Thread 2
Post by: TheTraveller on 05/17/2015 03:35 pm
...
Shawyer published this measured thrust as did the Chinese and why they think it does what it does. Someone who publishes a paper claiming it can't work because of their theory, just might be politely ignored as they get on with business.
Greg Egan did not claim  that the EM Drive cannot work because of a theory invented by Greg Egan. 

It is not his theory.  Greg Egan is showing again a well-known result.

Instead, Greg Egan showed again, an already known-proof that the stresses due to standing waves on the inner walls of a resonating cavity of any arbitrary shape whatsoever perfectly balance, and therefore there is zero net force in any direction, according to Maxwell's equations.

Egan exploited the computations of what the community only stated: Momentum is not conserved and so this does not work. This is the reason why we are looking elsewhere than only Maxwell equations.

What is your understanding of how an EM wave, acting at right angles to the cavity wall, can exert any force on it?

I assume the result would be zero, except for eddy current losses and as the cavity has a Q of 50,000, those losses are very small and are not involved in thrust generation.
Title: Re: EM Drive Developments - related to space flight applications - Thread 2
Post by: txdrive on 05/17/2015 03:43 pm
...
Shawyer published this measured thrust as did the Chinese and why they think it does what it does. Someone who publishes a paper claiming it can't work because of their theory, just might be politely ignored as they get on with business.
Greg Egan did not claim  that the EM Drive cannot work because of a theory invented by Greg Egan.  Instead, Greg Egan showed again, a known-proof that the stresses on the inner walls of a resonating cavity of any arbitrary shape whatsoever (as long as it is closed) perfectly balance, and therefore there is zero net force in any direction, according to Maxwell's equations.

If you can, please answer my simple question:

How can a curved EM wave, as in the attachment, touching the cavity wall at right angles to the wall, cause any force to be generated on the wall?
It's being reflected off the wall, why do you think it wouldn't generate a force orthogonally to the wall? I know it doesn't look like it's being reflected off the wall, but it has to be otherwise it would simply go through the wall.
Title: Re: EM Drive Developments - related to space flight applications - Thread 2
Post by: TheTraveller on 05/17/2015 03:44 pm
...
Shawyer published this measured thrust as did the Chinese and why they think it does what it does. Someone who publishes a paper claiming it can't work because of their theory, just might be politely ignored as they get on with business.
Greg Egan did not claim  that the EM Drive cannot work because of a theory invented by Greg Egan.  Instead, Greg Egan showed again, a known-proof that the stresses on the inner walls of a resonating cavity of any arbitrary shape whatsoever (as long as it is closed) perfectly balance, and therefore there is zero net force in any direction, according to Maxwell's equations.

If you can, please answer my simple question:

How can a curved EM wave, as in the attachment, touching the cavity wall at right angles to the wall, cause any force to be generated on the wall?

You cannot stare at a pictorial representation claiming something is there. Physics means mathematics and mathematics, through Maxwell equations, says no net momentum.

Are you saying the picture is wrong?

How? Where?

If you had a EM antenna at the vertex of the cone, EM waves of the same wavelength would radiate outward. When constrained in a frustum, the frustum guide wavelength will alter from the free wavelength as the diameter of the frustum alters and the wave edges will be at right angles to the cavity walls. Other than that the EM waves will radiate from small end to big end as if the cavity did not exist. That is until they hit the spherical big end plate, do an inphase bounce, head back toward the vertex, hit the small end plate, bounce and repeat the process until cavity eddy current losses eat up all the energy in the wave.
Title: Re: EM Drive Developments - related to space flight applications - Thread 2
Post by: SeeShells on 05/17/2015 03:44 pm
I just want to throw this up here:
http://www.nature.com/ncomms/2014/140306/ncomms4300/full/ncomms4300.html (http://www.nature.com/ncomms/2014/140306/ncomms4300/full/ncomms4300.html)
http://arxiv.org/ftp/arxiv/papers/1308/1308.0547.pdf (http://arxiv.org/ftp/arxiv/papers/1308/1308.0547.pdf)

I think if you're looking in classical physics for answers, you won't find them.

Yes! Running down that same path myself!
Title: Re: EM Drive Developments - related to space flight applications - Thread 2
Post by: StrongGR on 05/17/2015 03:45 pm
...
Shawyer published this measured thrust as did the Chinese and why they think it does what it does. Someone who publishes a paper claiming it can't work because of their theory, just might be politely ignored as they get on with business.
Greg Egan did not claim  that the EM Drive cannot work because of a theory invented by Greg Egan. 

It is not his theory.  Greg Egan is showing again a well-known result.

Instead, Greg Egan showed again, an already known-proof that the stresses due to standing waves on the inner walls of a resonating cavity of any arbitrary shape whatsoever perfectly balance, and therefore there is zero net force in any direction, according to Maxwell's equations.

Egan exploited the computations of what the community only stated: Momentum is not conserved and so this does not work. This is the reason why we are looking elsewhere than only Maxwell equations.

What is your understanding of how an EM wave, acting at right angles to the cavity wall, can exert any force on it?

I assume the result would be zero, except for eddy current losses and as the cavity has a Q of 50,000, those losses are very small and are not involved in thrust generation.

By the third principle, there is a movement of the cavity compensating the hit of the wave. It is the same that happens when you push on the steering wheel to move your car from inside. Your car stands still.
Title: Re: EM Drive Developments - related to space flight applications - Thread 2
Post by: phaseshift on 05/17/2015 03:54 pm
I'm working on a Ruby script that builds a Sketchup model of Sawyer's drive according to the equations and concepts being generated by this group. So as work proceeds a few questions have come up.

Most of these questions refer to the recently posted photo of Sawyer's drive.

1. What possible material is the large end plate made of?

It appears to be aluminum. One purpose of the o-ring is therefore to prevent galvanic corrosion. This also means the bolts are made of the same metal and they do appear to be the same color/luster and are seated in rubber grommets.

2. Why a different metal?

A possible answer (see question 4) is that it is far easier to put a curve in an aluminum blank (at least for me it is) than in copper (cheaper and lighter in weight as well).

3. What possible material is the small end plate made of?

It appears to be copper - yet has aluminum looking bolts also seated in grommets!

4. Do we know with fair certainty that the end plates have a spherical curve?

This might explain why the large end plate is so thick - it is much like a telescope mirror. It needs thickness to accommodate the curvature and provides rigidity.  The outside of the large plate appears to be flat to me and I see no reason for a curve there.

5. What are people's thoughts on the plates being silvered on the curved surfaces?
Title: Re: EM Drive Developments - related to space flight applications - Thread 2
Post by: TheTraveller on 05/17/2015 03:59 pm
...
Shawyer published this measured thrust as did the Chinese and why they think it does what it does. Someone who publishes a paper claiming it can't work because of their theory, just might be politely ignored as they get on with business.
Greg Egan did not claim  that the EM Drive cannot work because of a theory invented by Greg Egan.  Instead, Greg Egan showed again, a known-proof that the stresses on the inner walls of a resonating cavity of any arbitrary shape whatsoever (as long as it is closed) perfectly balance, and therefore there is zero net force in any direction, according to Maxwell's equations.

If you can, please answer my simple question:

How can a curved EM wave, as in the attachment, touching the cavity wall at right angles to the wall, cause any force to be generated on the wall?
It's being reflected off the wall, why do you think it wouldn't generate a force orthogonally to the wall? I know it doesn't look like it's being reflected off the wall, but it has to be otherwise it would simply go through the wall.

It is not reflection off the wall any more than it would in free space if the walls did not exist. All the altering frustum diameter does is to alter the guide wavelength and the group velocity based on the cutoff frequency. There is no side wall bounce involved here.

BTW this is not a laser with a very short wavelength compared to the frustum dimensions and guide wavelength. The wave front does not bounce from side wall to side wall as the guide wavelength is way too long to allow that to happen. This is a resonate cavity, with the Em wave bouncing back and forth between the end plates and altering guide wavelength, frequency and group velocity as the wave slides by each diameter change.

Additionally if the EM wave behaved like a very much shorted laser EM wave, bouncing from side wall to side wall, there would be no need for end plates curved as Shawyer shows nor any change in guide wavelength, not guide frequency nor group velocity as the frustum diameter varies. So the laser bouncing off the side walls image is wrong.

You can see the effective end plate wavelengths, frequencies and group velocities here
Title: Re: EM Drive Developments - related to space flight applications - Thread 2
Post by: deuteragenie on 05/17/2015 04:01 pm
Quote
By the third principle, there is a movement of the cavity compensating the hit of the wave. It is the same that happens when you push on the steering wheel to move your car from inside. Your car stands still.

Enlighten me: what happens if I point a flashlight to the car windshield?
Title: Re: EM Drive Developments - related to space flight applications - Thread 2
Post by: TheTraveller on 05/17/2015 04:20 pm
...
Shawyer published this measured thrust as did the Chinese and why they think it does what it does. Someone who publishes a paper claiming it can't work because of their theory, just might be politely ignored as they get on with business.
Greg Egan did not claim  that the EM Drive cannot work because of a theory invented by Greg Egan. 

It is not his theory.  Greg Egan is showing again a well-known result.

Instead, Greg Egan showed again, an already known-proof that the stresses due to standing waves on the inner walls of a resonating cavity of any arbitrary shape whatsoever perfectly balance, and therefore there is zero net force in any direction, according to Maxwell's equations.

Egan exploited the computations of what the community only stated: Momentum is not conserved and so this does not work. This is the reason why we are looking elsewhere than only Maxwell equations.

What is your understanding of how an EM wave, acting at right angles to the cavity wall, can exert any force on it?

I assume the result would be zero, except for eddy current losses and as the cavity has a Q of 50,000, those losses are very small and are not involved in thrust generation.

By the third principle, there is a movement of the cavity compensating the hit of the wave. It is the same that happens when you push on the steering wheel to move your car from inside. Your car stands still.

http://en.wikipedia.org/wiki/Radiation_pressure and as attached:

Radiant pressure has a cosine angle adjustment.

As the EM wave, at the wall boundary, is moving at 0 deg relative to the wall, the cosine loss angle is at max and no pressure is delivered to the side wall by the passing by EM wave. At the spherical end plate, the EM wave is moving at 90 deg relative to all the surface area of the end plate and delivers max pressure. Pressure delivered to the spherical end plates depends on the EM waves group velocity at impact / bounce.
Title: Re: EM Drive Developments - related to space flight applications - Thread 2
Post by: deltaMass on 05/17/2015 04:23 pm
For fear of stating the obvious:
With appropriately curved internal endplates, as TheTraveller has drawn, the Poynting vector is precisely parallel to the side walls at all locations along the side walls, at all times. Ergo zero thrust on the side walls anywhere, at any time. Another way this is expressed is in the maths of light sail thrust (see any of Geoff Landis's papers for example). The thrust goes as the cosine of the angle made by the Poynting vector with the surface normal. Thus in this case that angle is a right angle so the cosine is zero so the thrust is zero.
Title: Re: EM Drive Developments - related to space flight applications - Thread 2
Post by: phaseshift on 05/17/2015 04:35 pm
Scratch question #4 - doh - yes its certain. :)
Title: Re: EM Drive Developments - related to space flight applications - Thread 2
Post by: TheTraveller on 05/17/2015 04:35 pm
I'm working on a Ruby script that builds a Sketchup model of Sawyer's drive according to the equations and concepts being generated by this group. So as work proceeds a few questions have come up.

Most of these questions refer to the recently posted photo of Sawyer's drive.

1. What possible material is the large end plate made of?

It appears to be aluminum. One purpose of the o-ring is therefore to prevent galvanic corrosion. This also means the bolts are made of the same metal and they do appear to be the same color/luster and are seated in rubber grommets.

2. Why a different metal?

A possible answer (see question 4) is that it is far easier to put a curve in an aluminum blank (at least for me it is) than in copper (cheaper and lighter in weight as well).

3. What possible material is the small end plate made of?

It appears to be copper - yet has aluminum looking bolts also seated in grommets!

4. Do we know with fair certainty that the end plates have a spherical curve?

This might explain why the large end plate is so thick - it is much like a telescope mirror. It needs thickness to accommodate the curvature and provides rigidity.  The outside of the large plate appears to be flat to me and I see no reason for a curve there.

5. What are people's thoughts on the plates being silvered on the curved surfaces?

Assume copper spherical end caps.

Hard to see a Q of 50,000 to 60,000 with flat end plates.

End caps are not necessarily the frustum EM wave bounce end plates.

End caps may just be to provide pressure seal and sandwich end plates between cavity flange and the external end plates.

Also believe the bottom end cap is so think as it has to allow the big end curve to exist. Also note the big end cap is not sitting flush on the alum beam. It may have a convex curve or maybe other big end sense ports are down there as shown in the 2nd attachment.

Bolts are probably space grade stainless. Should be no H20 in a satellite so no galvanic action. I'm not a sat guy, so just my guestimation.

Once you know the frustum vertex you can work out the end plate radii from this:
Title: Re: EM Drive Developments - related to space flight applications - Thread 2
Post by: WarpTech on 05/17/2015 04:38 pm
...
Shawyer published this measured thrust as did the Chinese and why they think it does what it does. Someone who publishes a paper claiming it can't work because of their theory, just might be politely ignored as they get on with business.
Greg Egan did not claim  that the EM Drive cannot work because of a theory invented by Greg Egan.  Instead, Greg Egan showed again, a known-proof that the stresses on the inner walls of a resonating cavity of any arbitrary shape whatsoever (as long as it is closed) perfectly balance, and therefore there is zero net force in any direction, according to Maxwell's equations.

If you can, please answer my simple question:

How can a curved EM wave, as in the attachment, touching the cavity wall at right angles to the wall, cause any force to be generated on the wall?

If the walls are perfectly conducting, the NET will be zero, but they are not perfectly conducting and these are transverse waves. The electric field parallel to the wall may be zero but the field perpendicular is not. It attracts/repels electrons in the metal, driving currents. Also, magnetic permeability of copper is very low and conductivity is very high, so the rapidly changing magnetic field also drive currents. This is "work" being done on the walls of the cone.

Likewise, as work is done by the waves, the waves lose energy and decay to longer wavelengths. Giving up momentum to the Frustum, asymmetrically. As the waves are heading toward the small end, the guide wavelength eventually becomes imaginary and so the waves decay much faster. What Maxwell's equations and Greg Egan show is that the reflected waves that generate standing waves, play no part in the thrust, and they're right.

Todd
Title: Re: EM Drive Developments - related to space flight applications - Thread 2
Post by: phaseshift on 05/17/2015 04:47 pm
I'm working on a Ruby script that builds a Sketchup model of Sawyer's drive according to the equations and concepts being generated by this group. So as work proceeds a few questions have come up.

Most of these questions refer to the recently posted photo of Sawyer's drive.

1. What possible material is the large end plate made of?

It appears to be aluminum. One purpose of the o-ring is therefore to prevent galvanic corrosion. This also means the bolts are made of the same metal and they do appear to be the same color/luster and are seated in rubber grommets.

2. Why a different metal?

A possible answer (see question 4) is that it is far easier to put a curve in an aluminum blank (at least for me it is) than in copper (cheaper and lighter in weight as well).

3. What possible material is the small end plate made of?

It appears to be copper - yet has aluminum looking bolts also seated in grommets!

4. Do we know with fair certainty that the end plates have a spherical curve?

This might explain why the large end plate is so thick - it is much like a telescope mirror. It needs thickness to accommodate the curvature and provides rigidity.  The outside of the large plate appears to be flat to me and I see no reason for a curve there.

5. What are people's thoughts on the plates being silvered on the curved surfaces?

Assume copper spherical end caps.

Hard to see a Q of 50,000 to 60,000 with flat end plates.

End caps are not necessarily the frustum EM wave bounce end plates.

End caps may just be to provide pressure seal and sandwich end plates between cavity flange and the external end plates.

Also believe the bottom end cap is so think as it has to allow the big end curve to exist. Also note the big end cap is not sitting flush on the alum beam. It may have a convex curve or maybe other big end sense ports are down there as shown in the 2nd attachment.

Bolts are probably space grade stainless. Should be no H20 in a satellite so no galvanic action. I'm not a sat guy, so just my guestimation.

Once you know the frustum vertex you can work out the end plate radii from this:

Yep on the galvanic corrosion. 
Title: Re: EM Drive Developments - related to space flight applications - Thread 2
Post by: TheTraveller on 05/17/2015 04:50 pm
...
Shawyer published this measured thrust as did the Chinese and why they think it does what it does. Someone who publishes a paper claiming it can't work because of their theory, just might be politely ignored as they get on with business.
Greg Egan did not claim  that the EM Drive cannot work because of a theory invented by Greg Egan.  Instead, Greg Egan showed again, a known-proof that the stresses on the inner walls of a resonating cavity of any arbitrary shape whatsoever (as long as it is closed) perfectly balance, and therefore there is zero net force in any direction, according to Maxwell's equations.

If you can, please answer my simple question:

How can a curved EM wave, as in the attachment, touching the cavity wall at right angles to the wall, cause any force to be generated on the wall?

If the walls are perfectly conducting, the NET will be zero, but they are not perfectly conducting and these are transverse waves. The electric field parallel to the wall may be zero but the field perpendicular is not. It attracts/repels electrons in the metal, driving currents. Also, magnetic permeability of copper is very low and conductivity is very high, so the rapidly changing magnetic field also drive currents. This is "work" being done on the walls of the cone.

Likewise, as work is done by the waves, the waves lose energy and decay to longer wavelengths. Giving up momentum to the Frustum, asymmetrically. As the waves are heading toward the small end, the guide wavelength eventually becomes imaginary and so the waves decay much faster. What Maxwell's equations and Greg Egan show is that the reflected waves that generate standing waves, play no part in the thrust, and they're right.

Todd

Yes. Correct. Understand eddy current losses.

A Q of 50,000 say it takes 50,000 up and back cycles to drain off all the energy in the EM wave into wall eddy current losses and increased cavity wall temperature. The Chinese did measure and report on this as attached.

Due to the side wall cosine angle to the EM wave, there is no resultant radiation pressure on the side walls.
Title: Re: EM Drive Developments - related to space flight applications - Thread 2
Post by: TheTraveller on 05/17/2015 04:55 pm
Yep on the galvanic corrosion.

Galvanic corrosion occurs in space? Doesn't there needs to be an electrolyte between the different metals for it to happen? Thought spacecraft were ultra clean?
Title: Re: EM Drive Developments - related to space flight applications - Thread 2
Post by: deltaMass on 05/17/2015 04:58 pm
Yes. Correct. Understand eddy current losses.

A Q of 50,000 say it takes 50,000 up and back cycles to drain off all the energy in the EM wave into wall eddy current losses and increased cavity wall temperature. The Chinese did measure and report on this as attached.

Due to the side wall cosine angle to the EM wave, there is no resultant radiation pressure on the side walls.
Yes, that's a more complete description. 100% concur.

ETA Almost concur :)  Most losses occur at the end plates, I would imagine. Due to copper's high thermal conductivity, this heat gets conducted to the side walls too.
Title: Re: EM Drive Developments - related to space flight applications - Thread 2
Post by: phaseshift on 05/17/2015 04:59 pm
Yep on the galvanic corrosion.

Galvanic corrosion occurs in space? Doesn't there needs to be an electrolyte between the different metals for it to happen? Thought spacecraft were ultra clean?

Yep, as in I agree about there being none.
Title: Re: EM Drive Developments - related to space flight applications - Thread 2
Post by: txdrive on 05/17/2015 04:59 pm
...
Shawyer published this measured thrust as did the Chinese and why they think it does what it does. Someone who publishes a paper claiming it can't work because of their theory, just might be politely ignored as they get on with business.
Greg Egan did not claim  that the EM Drive cannot work because of a theory invented by Greg Egan.  Instead, Greg Egan showed again, a known-proof that the stresses on the inner walls of a resonating cavity of any arbitrary shape whatsoever (as long as it is closed) perfectly balance, and therefore there is zero net force in any direction, according to Maxwell's equations.

If you can, please answer my simple question:

How can a curved EM wave, as in the attachment, touching the cavity wall at right angles to the wall, cause any force to be generated on the wall?
It's being reflected off the wall, why do you think it wouldn't generate a force orthogonally to the wall? I know it doesn't look like it's being reflected off the wall, but it has to be otherwise it would simply go through the wall.

It is not reflection off the wall any more than it would in free space if the walls did not exist. All the altering frustum diameter does is to alter the guide wavelength and the group velocity based on the cutoff frequency. There is no side wall bounce involved here.
Your wave inside and outside the waveguide is merely a sum of what you have if the wall did not exist and what is radiated by the wall.

Quote
For fear of stating the obvious:
With appropriately curved internal endplates, as TheTraveller has drawn, the Poynting vector is precisely parallel to the side walls at all locations along the side walls, at all times. Ergo zero thrust on the side walls anywhere, at any time. Another way this is expressed is in the maths of light sail thrust (see any of Geoff Landis's papers for example). The thrust goes as the cosine of the angle made by the Poynting vector with the surface normal. Thus in this case that angle is a right angle so the cosine is zero so the thrust is zero.
At the surface of a perfectly reflective light sail, E is orthogonal to the sail and E x H is parallel to the sail, which merely means that there's nothing being absorbed, not that there's no force.

What works for the sail calculation is the Poynting vector without the sail reflecting the light back.

Without his walls, the wave will diffract out.
Title: Re: EM Drive Developments - related to space flight applications - Thread 2
Post by: phaseshift on 05/17/2015 05:01 pm
Yep on the galvanic corrosion.

Galvanic corrosion occurs in space? Doesn't there needs to be an electrolyte between the different metals for it to happen? Thought spacecraft were ultra clean?

Yep, as in I agree about there being none.

Though that might be the original reason. I think the drive is several years from heading to space and he would want to be able to disassemble/reassemble in the near term. 
Title: Re: EM Drive Developments - related to space flight applications - Thread 2
Post by: TheTraveller on 05/17/2015 05:08 pm
Yep on the galvanic corrosion.

Galvanic corrosion occurs in space? Doesn't there needs to be an electrolyte between the different metals for it to happen? Thought spacecraft were ultra clean?

Yep, as in I agree about there being none.

Though that might be the original reason. I think the drive is several years from heading to space and he would want to be able to disassemble/reassemble in the near term.

I assume this is the final product that SPR shipped to Boeing. Has been plated and I'm sure other issues addressed and fixed. Also suggest the photo was distorted to make it harder to work out the dimensions.

On a NASA EW slide, this unit was labeled a "High Fidelity Test Article".
Title: Re: EM Drive Developments - related to space flight applications - Thread 2
Post by: txdrive on 05/17/2015 05:13 pm
With regards to the forces on the side of the cavity, all I can say is, write the field equations for your fields (what is the H and E at a given point in space), then I can calculate the forces on the boundaries and/or show that your fields violate Maxwell's equations. When it is a picture I don't know what exactly you're talking about.
Title: Re: EM Drive Developments - related to space flight applications - Thread 2
Post by: phaseshift on 05/17/2015 05:20 pm
Yes, that version has a bunch of improvements over the copper version.

Building it in SketchUp really highlights issues that would come upduring construction. I have had this photo and really should have been going off of it. I am not trying to guess dimensions from the photos but instead are using the equations and dimensions in this forum. The script currently takes 3 parameters and generates the model from that.
Title: Re: EM Drive Developments - related to space flight applications - Thread 2
Post by: dustinthewind on 05/17/2015 05:25 pm
I have been meaning to ask.  If anyone gets the chance to test this out for me I would be fascinated to hear the results.  It's a dual cavity experiment.  I think it would require a klystron or twt with a continuous phase.  Cylindrical cavities should work.  They would need to have a gap between the two of them of 1/4 lambda wavelength with their phase out of alignment by 90 degrees or pi/2 radians.   

What should happen is the plate of cavity 1 should have circulating current stimulated by the radiation inside that builds up depending on the Q of the cavity.  Information is limited by the speed of light so that appearance of current should be on its way to the next cavity which has its current out of phase by 90 degrees.  When the information reaches cavity 2 the current in cavity 2 should appear to be moving with cavity 1 so it is attracted.  Cavity 1 on the other hand when the signal reaches it appears to have its current moving against cavity 2 and should be repulsed (similar to current in wires).  I would guess this effect should also scale with the Q of the cavity. 

It may be possible for a dielectric to slow the information transfer between the cavities and move them closer together and increase the effect.  I would guess some radiation may tunnel through the cavities into the other one and attenuation of one cavity may be required to keep them equal in amplitude and out of phase by 90 degrees. 

My guess is it should be similar to attraction between magnets but the force is in the same direction for both cavities.  Images attached for ease of understanding. 

Fig1 Simple.png key:
1. Forces
2. Current
3. Current
4. Appearance of current limited by the speed of light (bottom wire)
5. Appearance of current limited by the speed of light (top wire)

8, 9, 10, 11 are frames of current over time.  Frame 12 is a repeat of frame 8 and frame 13 is a repeat of 9.  (Only 4 frames needed).  The illustration of force between wires can parallel to the cavity plate I think. 

Thanks!

I would also be curious to hear if we think it won't work for some specific reason as a counter argument.  I'm open minded to counter arguments. 
Title: Re: EM Drive Developments - related to space flight applications - Thread 2
Post by: WarpTech on 05/17/2015 05:28 pm

If the walls are perfectly conducting, the NET will be zero, but they are not perfectly conducting and these are transverse waves. The electric field parallel to the wall may be zero but the field perpendicular is not. It attracts/repels electrons in the metal, driving currents. Also, magnetic permeability of copper is very low and conductivity is very high, so the rapidly changing magnetic field also drive currents. This is "work" being done on the walls of the cone.

Likewise, as work is done by the waves, the waves lose energy and decay to longer wavelengths. Giving up momentum to the Frustum, asymmetrically. As the waves are heading toward the small end, the guide wavelength eventually becomes imaginary and so the waves decay much faster. What Maxwell's equations and Greg Egan show is that the reflected waves that generate standing waves, play no part in the thrust, and they're right.

Todd

Yes. Correct. Understand eddy current losses.

A Q of 50,000 say it takes 50,000 up and back cycles to drain off all the energy in the EM wave into wall eddy current losses and increased cavity wall temperature. The Chinese did measure and report on this as attached.

Due to the side wall cosine angle to the EM wave, there is no resultant radiation pressure on the side walls.

I think the difference is, that it's not the radiation pressure of a transverse wave anymore. It's Lorentz force due to EM induction. When the waves become evanescent, the interaction becomes a near-field induction of eddy currents and a rapid loss of momentum. Where, in the other direction, waves traveling toward the large end do not see this evanescent wave effect. I'm concentrated on these evanescent waves because this is where the asymmetry is, in the attenuation.
Title: Re: EM Drive Developments - related to space flight applications - Thread 2
Post by: TheTraveller on 05/17/2015 05:35 pm
With regards to the forces on the side of the cavity, all I can say is, write the field equations for your fields (what is the H and E at a given point in space), then I can calculate the forces on the boundaries and/or show that your fields violate Maxwell's equations. When it is a picture I don't know what exactly you're talking about.

Last line say it all:

Quote
If the absorbing surface is planar at an angle a to the radiation source, the intensity across the surface will be reduced.

The effective radiation source is the vertex of the frustum. The frustum side wall are aligned to, pointing at the vertex. Therefore the radiant pressures on the side walls are zero.
Title: Re: EM Drive Developments - related to space flight applications - Thread 2
Post by: txdrive on 05/17/2015 05:52 pm
With regards to the forces on the side of the cavity, all I can say is, write the field equations for your fields (what is the H and E at a given point in space), then I can calculate the forces on the boundaries and/or show that your fields violate Maxwell's equations. When it is a picture I don't know what exactly you're talking about.

Last line say it all:

Quote
If the absorbing surface is planar at an angle a to the radiation source, the intensity across the surface will be reduced.

The effective radiation source is the vertex of the frustum. The frustum side wall are aligned to, pointing at the vertex. Therefore the radiant pressures on the side walls are zero.
You're ignoring diffraction. If the frustum side walls were not there, the radiation would leak out.

edit: write an equation of form E(position, time) and H(position, time) . Maybe it would be clearer for a cylinder with flat end pieces? You can't just take geometric optics approximation from Wikipedia and apply it to microwaves in a reasonable sized cavity... it won't be correct then.
Title: Re: EM Drive Developments - related to space flight applications - Thread 2
Post by: SeeShells on 05/17/2015 06:03 pm
...
Shawyer published this measured thrust as did the Chinese and why they think it does what it does. Someone who publishes a paper claiming it can't work because of their theory, just might be politely ignored as they get on with business.
Greg Egan did not claim  that the EM Drive cannot work because of a theory invented by Greg Egan. 

It is not his theory.  Greg Egan is showing again a well-known result.

Instead, Greg Egan showed again, an already known-proof that the stresses due to standing waves on the inner walls of a resonating cavity of any arbitrary shape whatsoever perfectly balance, and therefore there is zero net force in any direction, according to Maxwell's equations.

Egan exploited the computations of what the community only stated: Momentum is not conserved and so this does not work. This is the reason why we are looking elsewhere than only Maxwell equations.

What is your understanding of how an EM wave, acting at right angles to the cavity wall, can exert any force on it?

I assume the result would be zero, except for eddy current losses and as the cavity has a Q of 50,000, those losses are very small and are not involved in thrust generation.

By the third principle, there is a movement of the cavity compensating the hit of the wave. It is the same that happens when you push on the steering wheel to move your car from inside. Your car stands still.
Let's say your hitting the steering wheel very fast again and again and let's leave out it was someone who cut you off in traffic. Lets say each time you brought your hand back it gained energy in the action of the backward movement , it was from let's say a quantum/force which has no relation or attachment to the car but into the underlying quantum world (weird and neat stuff), then each time you would strike the wheel you would impart force and momentum to the car.  Off you go. Not so much with a flashlight on the windshield as light is a little different in it's construction and it would piss off the car in front of you but that's another story.
Shell
Title: Re: EM Drive Developments - related to space flight applications - Thread 2
Post by: txdrive on 05/17/2015 06:06 pm
...
Shawyer published this measured thrust as did the Chinese and why they think it does what it does. Someone who publishes a paper claiming it can't work because of their theory, just might be politely ignored as they get on with business.
Greg Egan did not claim  that the EM Drive cannot work because of a theory invented by Greg Egan. 

It is not his theory.  Greg Egan is showing again a well-known result.

Instead, Greg Egan showed again, an already known-proof that the stresses due to standing waves on the inner walls of a resonating cavity of any arbitrary shape whatsoever perfectly balance, and therefore there is zero net force in any direction, according to Maxwell's equations.

Egan exploited the computations of what the community only stated: Momentum is not conserved and so this does not work. This is the reason why we are looking elsewhere than only Maxwell equations.

What is your understanding of how an EM wave, acting at right angles to the cavity wall, can exert any force on it?

I assume the result would be zero, except for eddy current losses and as the cavity has a Q of 50,000, those losses are very small and are not involved in thrust generation.

By the third principle, there is a movement of the cavity compensating the hit of the wave. It is the same that happens when you push on the steering wheel to move your car from inside. Your car stands still.
Let's say your hitting the steering wheel very fast again and again and let's leave out it was someone who cut you off in traffic. Lets say each time you brought your hand back it gained energy in the action of the backward movement , it was from let's say a quantum/force which has no relation or attachment to the car but into the underlying quantum world (weird and neat stuff), then each time you would strike the wheel you would impart force and momentum to the car.  Off you go. Not so much with a flashlight on the windshield as light is a little different in it's construction and it would piss off the car in front of you but that's another story.
Shell
Then, unless this force and momentum is very small, the mechanics of motion of a hand through space would have to differ from what was observed in high precision experiments that did not involve cars.

edit: Suppose you computed the average force f that you'd obtain if you ignored the pressure of your back against the car seat - just the hand banging on the steering wheel.

If you had previously done a wide variety of highly precise parts-per-million experiments on the motion of the hand through the weird quantum world (never noticing any weird changes in energy or momentum, to a high precision), then you can very confidently exclude any forces much greater than a millionth of f.
Title: Re: EM Drive Developments - related to space flight applications - Thread 2
Post by: LasJayhawk on 05/17/2015 06:16 pm
Yep on the galvanic corrosion.

Galvanic corrosion occurs in space? Doesn't there needs to be an electrolyte between the different metals for it to happen? Thought spacecraft were ultra clean?

Yep, as in I agree about there being none.

Though that might be the original reason. I think the drive is several years from heading to space and he would want to be able to disassemble/reassemble in the near term.

I assume this is the final product that SPR shipped to Boeing. Has been plated and I'm sure other issues addressed and fixed. Also suggest the photo was distorted to make it harder to work out the dimensions.

On a NASA EW slide, this unit was labeled a "High Fidelity Test Article".


If I want to maximize the Q of a cavity I would silver plate the copper, and if I wanted maximum repeatablity I would gold flash the silver.

The cinder blocks in the wall should be 8" high. The rf connector with the cap on it looks to be a type SMA and not N, iirc the SMA flange connectors are about .5" square.
Title: Re: EM Drive Developments - related to space flight applications - Thread 2
Post by: TheTraveller on 05/17/2015 06:25 pm
With regards to the forces on the side of the cavity, all I can say is, write the field equations for your fields (what is the H and E at a given point in space), then I can calculate the forces on the boundaries and/or show that your fields violate Maxwell's equations. When it is a picture I don't know what exactly you're talking about.

Last line say it all:

Quote
If the absorbing surface is planar at an angle a to the radiation source, the intensity across the surface will be reduced.

The effective radiation source is the vertex of the frustum. The frustum side wall are aligned to, pointing at the vertex. Therefore the radiant pressures on the side walls are zero.
You're ignoring diffraction. If the frustum side walls were not there, the radiation would leak out.

edit: write an equation of form E(position, time) and H(position, time) . Maybe it would be clearer for a cylinder with flat end pieces? You can't just take geometric optics approximation from Wikipedia and apply it to microwaves in a reasonable sized cavity... it won't be correct then.

We were discussing radiation pressure generated from EM waves according to Maxwell's equations (attached). This pressure is apparently subject to cosine angle loss and thus when the EM wave moves along the frustum side walls and is not bouncing off the frustum spherical end plates, there is no Maxwell radiation pressure generated on the frustum side walls.

Or did I not understand what the cosine loss factor is for in the lower of the 2 equations?
Title: Re: EM Drive Developments - related to space flight applications - Thread 2
Post by: Flyby on 05/17/2015 06:27 pm
Yes, that version has a bunch of improvements over the copper version.

Building it in SketchUp really highlights issues that would come upduring construction. I have had this photo and really should have been going off of it. I am not trying to guess dimensions from the photos but instead are using the equations and dimensions in this forum. The script currently takes 3 parameters and generates the model from that.
As I showed about a hundred pages ago, building a 3D model based upon photographs requires adjustments to compensate the camera lens distortions. Unless you have the specific data of the camera and its lens, it will be impossible to accurately build a 3D model. At best you'll have an approximation.

Do not underestimate the distortion caused by targeting under the horizon line. They are important and increase when using a wide angle lens. When targeting under the horizon, the lens distortions  cause vertical lines to focus on a point below, making the top of the frustum appear larger to the bottom plate.

I did stop a further image analysis on the alu frustum because somebody found the exact dimensions of the frustum, as it appears to be an industrial manufactured piping part. It is readily available to everybody...There was no need anymore for an estimate on that frustum...

I'll try to dig it up... it's somewhere inhere, i suspect in the first 50 pages...

Title: Re: EM Drive Developments - related to space flight applications - Thread 2
Post by: TheTraveller on 05/17/2015 06:30 pm
Yep on the galvanic corrosion.

Galvanic corrosion occurs in space? Doesn't there needs to be an electrolyte between the different metals for it to happen? Thought spacecraft were ultra clean?

Yep, as in I agree about there being none.

Though that might be the original reason. I think the drive is several years from heading to space and he would want to be able to disassemble/reassemble in the near term.

I assume this is the final product that SPR shipped to Boeing. Has been plated and I'm sure other issues addressed and fixed. Also suggest the photo was distorted to make it harder to work out the dimensions.

On a NASA EW slide, this unit was labeled a "High Fidelity Test Article".


If I want to maximize the Q of a cavity I would silver plate the copper, and if I wanted maximum repeatablity I would gold flash the silver.

The cinder blocks in the wall should be 8" high. The rf connector with the cap on it looks to be a type SMA and not N, iirc the SMA flange connectors are about .5" square.

That small connector near the small end plate of the Flight Thruster may be for an e field sense feedback. Here is a view from the other side.

Note what looks like a cavity resonance tuner to the left of the red Rf input fed.
Title: Re: EM Drive Developments - related to space flight applications - Thread 2
Post by: txdrive on 05/17/2015 06:41 pm
With regards to the forces on the side of the cavity, all I can say is, write the field equations for your fields (what is the H and E at a given point in space), then I can calculate the forces on the boundaries and/or show that your fields violate Maxwell's equations. When it is a picture I don't know what exactly you're talking about.

Last line say it all:

Quote
If the absorbing surface is planar at an angle a to the radiation source, the intensity across the surface will be reduced.

The effective radiation source is the vertex of the frustum. The frustum side wall are aligned to, pointing at the vertex. Therefore the radiant pressures on the side walls are zero.
You're ignoring diffraction. If the frustum side walls were not there, the radiation would leak out.

edit: write an equation of form E(position, time) and H(position, time) . Maybe it would be clearer for a cylinder with flat end pieces? You can't just take geometric optics approximation from Wikipedia and apply it to microwaves in a reasonable sized cavity... it won't be correct then.

We were discussing radiation pressure generated from EM waves according to Maxwell's equations (attached). This pressure is apparently subject to cosine angle loss and thus when the EM wave moves along the frustum side walls and is not bouncing off the frustum spherical end plates, there is no Maxwell radiation pressure generated on the frustum side walls.

Or did I not understand what the cosine loss factor is for in the lower of the 2 equations?
It's a geometric optics approximation. Doesn't hold precisely.

For example suppose you got a beam of coherent light, 5mm across, 500nm wavelength, in space. It's not going all parallel to it's original direction, it's spreading, to about 1m across at 10 000m from the source (Makes a fuzzy "Airy disk" pattern).

edit: The Poynting vector on the sides of a perfect cylinder over this beam, is not parallel to the axis. It's this Poynting vector that matters.
Title: Re: EM Drive Developments - related to space flight applications - Thread 2
Post by: phaseshift on 05/17/2015 06:45 pm
Yes, that version has a bunch of improvements over the copper version.

Building it in SketchUp really highlights issues that would come upduring construction. I have had this photo and really should have been going off of it. I am not trying to guess dimensions from the photos but instead are using the equations and dimensions in this forum. The script currently takes 3 parameters and generates the model from that.
As I showed about a hundred pages ago, building a 3D model based upon photographs requires adjustments to compensate the camera lens distortions. Unless you have the specific data of the camera and its lens, it will be impossible to accurately build a 3D model. At best you'll have an approximation.

Do not underestimate the distortion caused by targeting under the horizon line. They are important and increase when using a wide angle lens. When targeting under the horizon, the lens distortions  cause vertical lines to focus on a point below, making the top of the frustum appear larger to the bottom plate.

I did stop a further image analysis on the alu frustum because somebody found the exact dimensions of the frustum, as it appears to be an industrial manufactured piping part. It is readily available to everybody...There was no need anymore for an estimate on that frustum...

I'll try to dig it up... it's somewhere inhere, i suspect in the first 50 pages...

I've mentioned a few times - I'm not getting the dimensions from the photos. :)
Title: Re: EM Drive Developments - related to space flight applications - Thread 2
Post by: Rodal on 05/17/2015 07:04 pm
...

Dear Jose,

Thanks a lot for spending some time on my calculations. You tried with b but did you check for a?...

http://forum.nasaspaceflight.com/index.php?action=dlattach;topic=36313.0;attach=830137

Here are the expressions including both a and b, to give a value to L

Assuming r1 =<r2

as per the geometry in the following image


(http://gregegan.customer.netspace.net.au/SCIENCE/Cavity/CavityShape.gif)

Taking (from Eq. 39 in your paper)

b = (r1^2 - r2^2)/(4 (lo^2) Log[r2/r1])

Define

r1bar = r1 / lo
r2bar = r2 / lo

Then

b = (r1bar^2 - r2bar^2)/(4 Log[r2bar/r1bar])

Define:

r1bar = r2bar/c  (where c >= 1 since r1bar =< r2bar )

bb = b / (r2bar^2)

then

bb = - cc/4

where

cc = (c^2 - 1)/( (c^2) Log[c])

and

b = - ( (r2bar^2) /4 ) *cc

Taking (from Eq. 39 in your paper)

a = (1/Log[r2/r1]) (1/(4*(lo^2)))*((r2^2)*Log[r1/lo] - (r1^2)*Log[r2/lo])

then

a = (1/Log[r2bar / r1bar]) (1/4)*((r2bar^2)*Log[r1bar] - (r1bar ^2)*Log[r2bar])

a = ( (r2bar^2) /4 ) * (Log[r2bar] *cc - 1 )

and we obtain

L = a + b Log[rbar] + (rbar^2) / 4

substituting the expressions for a and b

L = ( (r2bar^2)/4 ) * (Log[r2bar] *cc - 1 ) - ( (r2bar^2)/4) *cc * Log[rbar] + (rbar^2)/4

L = (r2bar^2)/4)*cc*( Log[r2bar] - Log[rbar]) - (r2bar^2 - rbar^2)/4

L = (r2bar^2)/4)*cc* Log[r2/r] - (r2bar^2 - rbar^2)/4

( since ( Log[r2bar] - Log[rbar]) = Log[r2bar/rbar] )



Recall that

rbar =< r2bar   (all material points must be at a radius vector r smaller than or equal to r2)

and that we want to maximize the following quantity:

L = (r2bar^2)/4)*cc* Log[r2/r] - (r2bar^2 - rbar^2)/4

Now, for r=r2,

L(r->r2) = (r2bar^2)/4)*cc* Log[r2/r2]  - (r2bar^2 - r2bar^2)/4
              =0

which is a minimum. While for r=r1,


L(r=r1) = (r2bar^2)/4)*cc* Log[r2/r1]  - (r2bar^2 - r1bar^2)/4
             =(r2bar^2)/4)*cc* Log[r2bar/r1bar]  - (r2bar^2 - r1bar^2)/4
             =(r2bar^2)/4)*cc* Log[c] - (r2bar^2 - r1bar^2)/4
             =(r2bar^2)/4)* (c^2 - 1)/(c^2) - (r2bar^2 - r1bar^2)/4                 

Now, we have the following limits:

For Limit[(c^2 - 1)/(c^2), c -> 1] = 0  (this corresponds to the maximum possible value of r1, r1 ~ r2 )

giving

L(r=r1, r1->r2)  = - (r2bar^2 - r1bar^2)/4 

While Limit[(c^2 - 1)/(c^2), c -> Infinity] = 1  (this corresponds to the minimum possible value of r1, r1 ~ 0 )

gives

L(r=r1, r1->0) = (r2bar^2)/4) (Log[r2/r1] - 1)  where Log[r2/r1] -> Infinity for r1 approaching zero

It is evident we also want to also maximize r2

Therefore in order to maximize L in Equation 39, we want to have:

1) r1 as close as possible to 0
2) r2 as large as possible


This is a long pointy cone. All the interesting action takes place near r=r1 for r1 -> 0

"The geometry comes to the rescue" as you said.

L(r=r2 ) = 0

L(r=r1  r1->0) =(r2bar^2)/4) (Log[r2/r1] -1)


As you said, the "lo" constant is a really huge number, unless the EM Drive happens to be near a magnestar.

We conclude that as Marco, said, the geometry comes to the rescue for L(r=r1  r1->0), a pointy cone
We have at r=r2 L=0, but at r=r1 for r1 approaching zero we have a small number (r2bar^2)/4) multiplying a large number (Log[r2/r1] -> Infinity  if r ~ 0 (a pointy cone))

We need to run some numerical results because Log[r2/r1] goes to Infinity very slowly for r1 approaching zero, so the size of L is very dependent on the magnitude of lo and the magnitude of r1.


Title: Re: EM Drive Developments - related to space flight applications - Thread 2
Post by: TheTraveller on 05/17/2015 07:20 pm
With regards to the forces on the side of the cavity, all I can say is, write the field equations for your fields (what is the H and E at a given point in space), then I can calculate the forces on the boundaries and/or show that your fields violate Maxwell's equations. When it is a picture I don't know what exactly you're talking about.

Last line say it all:

Quote
If the absorbing surface is planar at an angle a to the radiation source, the intensity across the surface will be reduced.

The effective radiation source is the vertex of the frustum. The frustum side wall are aligned to, pointing at the vertex. Therefore the radiant pressures on the side walls are zero.
You're ignoring diffraction. If the frustum side walls were not there, the radiation would leak out.

edit: write an equation of form E(position, time) and H(position, time) . Maybe it would be clearer for a cylinder with flat end pieces? You can't just take geometric optics approximation from Wikipedia and apply it to microwaves in a reasonable sized cavity... it won't be correct then.

We were discussing radiation pressure generated from EM waves according to Maxwell's equations (attached). This pressure is apparently subject to cosine angle loss and thus when the EM wave moves along the frustum side walls and is not bouncing off the frustum spherical end plates, there is no Maxwell radiation pressure generated on the frustum side walls.

Or did I not understand what the cosine loss factor is for in the lower of the 2 equations?
It's a geometric optics approximation. Doesn't hold precisely.

For example suppose you got a beam of coherent light, 5mm across, 500nm wavelength, in space. It's not going all parallel to it's original direction, it's spreading, to about 1m across at 10 000m from the source (Makes a fuzzy "Airy disk" pattern).

edit: The Poynting vector on the sides of a perfect cylinder over this beam, is not parallel to the axis. It's this Poynting vector that matters.

Built and used many optical telescopes, including a 12 inch, Schmidt Cassegrain. Diffraction and I got to be good enemies.

There is no natural spread for the beam inside the cavity. It is controlled by the guide and cutoff wavelengths determined by the cavity diameter it travels through and by bouncing off the spherical end plates, which as I see it orient the EM waves so they travel / slide along the side walls at a 0 bounce radiant cosine angle.

I do see that using spherical end plates and getting them highly aligned, pointing at the same vertex, is very necessary to getting an EM Drive to work correctly. If they are out of alignment even say 1 deg, the reflected wave will be not moving along the side wall properly, will partly bounce off it and hit the other end plate as a phase distorted mess.

Is like early days playing with Helium-Neon lasers with adjustable end mirrors. Bloody hard to get the mirrors properly aligned, even with confocal mirrors.
Title: Re: EM Drive Developments - related to space flight applications - Thread 2
Post by: Notsosureofit on 05/17/2015 07:44 pm
@RODAL

It seems (to me) that that is quite correct, but that does not mean that that is for max force since it neglects frequency.  I can see how that might be for max laser sideband generation w/o constraining the frequency. (ie very high radial modes)

I know I said I didn't think the cylindrical cavity should show sidebands outside of the cavity but I am rethinking that as the laser propagates in one direction and I'm used to thinking of symmetrical systems.

Title: Re: EM Drive Developments - related to space flight applications - Thread 2
Post by: StrongGR on 05/17/2015 07:54 pm
...

Dear Jose,

Thanks a lot for spending some time on my calculations. You tried with b but did you check for a?...

http://forum.nasaspaceflight.com/index.php?action=dlattach;topic=36313.0;attach=830137

Here are the expressions including both a and b, to give a value to L

Assuming r1 =<r2

as per the geometry in the following image


(http://gregegan.customer.netspace.net.au/SCIENCE/Cavity/CavityShape.gif)

Taking (from Eq. 39 in your paper)

b = (r1^2 - r2^2)/(4 (lo^2) Log[r2/r1])

Define

r1bar = r1 / lo
r2bar = r2 / lo

Then

b = (r1bar^2 - r2bar^2)/(4 Log[r2bar/r1bar])

Define:

r1bar = r2bar/c  (where c >= 1 since r1bar =< r2bar )

bb = b / (r2bar^2)

then

bb = - cc/4

where

cc = (c^2 - 1)/( (c^2) Log[c])

and

b = - ( (r2bar^2) /4 ) *cc

Taking (from Eq. 39 in your paper)

a = (1/Log[r2/r1]) (1/(4*(lo^2)))*((r2^2)*Log[r1/lo] - (r1^2)*Log[r2/lo])

then

a = (1/Log[r2bar / r1bar]) (1/4)*((r2bar^2)*Log[r1bar] - (r1bar ^2)*Log[r2bar])

a = ( (r2bar^2) /4 ) * (Log[r2bar] *cc - 1 )

and we obtain

L = a + b Log[rbar] + (rbar^2) / 4

substituting the expressions for a and b

L = ( (r2bar^2)/4 ) * (Log[r2bar] *cc - 1 ) - ( (r2bar^2)/4) *cc * Log[rbar] + (rbar^2)/4

L = (r2bar^2)/4)*cc*( Log[r2bar] - Log[rbar]) - (r2bar^2 - rbar^2)/4

Limit[cc, c -> 1] = 2  (this corresponds to the maximum possible value of r1, r1 ~ r2 )

Limit[cc, c -> Infinity] = 0  (this corresponds to the minimum possible value of r1, r1 -> 0 )


so for c -> 1,  r1 ~ r2

L = (r2bar^2)/2)( Log[r2bar] - Log[rbar]) - (r2bar^2 - rbar^2)/4

and for c -> Infinity, r1 -> 0

L = - (r2bar^2 - rbar^2)/4



Since

rbar =< r2bar   (all material points must be at a radius vector r smaller than or equal to r2)

We want to maximize the following quantity:

 ( (r2bar^2) /4 ) * (Log[r2bar] *cc

which means that we must have maximum cc

Maximum cc occurs for

Limit[cc, c -> 1] = 2  (this corresponds to the maximum possible value of r1, r1 ~ r2 )

Therefore, for a maximum L we want to have:

r1 as close as possible to r2

This says that the axial length of the truncated cone should be close to zero.In other words, this optimized geometry, according to Eq. 39 in your paper seems to be much closer to Cannae's device.
 And it is actually not far from the geometry presently used by Dr. White. It is certainly not the geometry of a pointy cone

(http://1.bp.blogspot.com/-2MORMyXf97o/VU4f2f8MtII/AAAAAAAA90w/Oc5eLpuujA4/s400/cannae_driveNASA.jpg)

As you said, the "lo" constant is a really huge number, unless the EM Drive happens to be near a magnestar.

This means that the variable  ((r2/lo)^2) is very small.

---->However, it get multiplied by Log[ r2/lo] which can be an even larger negative number<----

To be more specific, we need a number for lo, so that we can calculate the term  (r2bar^2)/2)( Log[r2bar] - Log[rbar]) in

L = (r2bar^2)/2)( Log[r2bar] - Log[rbar]) - (r2bar^2 - rbar^2)/4


(The above expression is for r1~r2, such that c~1 and cc~2)

 :)

NOTE: while the condition r1~r2 (r1 as close as possible to r2) still holds (since the smaller r1 with respect to r2 the smaller the term cc), notice that I removed a condition on r2 that I had previously stated.  This is because while to maximize the term (r2bar^2)/2) we also want r2 as large as possible, to maximize (-Log[r2bar]) one wants r2 as small as possible. So to be specific one needs a number for lo.



Dear Jose,

Thank you a lot for your effort to this question. Also thanks to @notsureofit for his help. You are right. I have checked this with Maple and optimized the function L(r) as a function of r1, r2 and U0 also. And yes, r1=r2 is the solution! I have other checks to do and my notebook just keeps on freezing for a computation of these but I hope to complete all in a few days.

This means that Harold White experiments could represent a great leap beyond in experimental general relativity. This was my initial hope. As a theoretical physicist please don't ask me about applications!

Give me a few days for further checks and I will update my draft, with due acknowledgements.

Regards.
Title: Re: EM Drive Developments - related to space flight applications - Thread 2
Post by: StrongGR on 05/17/2015 07:57 pm
@RODAL

It seems (to me) that that is quite correct, but that does not mean that that is for max force since it neglects frequency.  I can see how that might be for max laser sideband generation w/o constraining the frequency. (ie very high radial modes)
Thanks.  But where does the frequency enter into the equation for L? all I see is a, b, r2bar, r1bar, lo geometrical parameters.  It seems that the electromagnetic field Power only enters through the parameter (Uo)2 which is built inside the length "lo", so as we said, we need a number for "lo" to make any  more progress.

See Marco's Eq. 19, 20 and 28 in http://forum.nasaspaceflight.com/index.php?action=dlattach;topic=36313.0;attach=830137

It is the choice of the resonant mode that fixes l0 and this depends on frequency (indirectly). You should check Egan link for some numbers to put in.
Title: Re: EM Drive Developments - related to space flight applications - Thread 2
Post by: Rodal on 05/17/2015 08:03 pm
@RODAL

It seems (to me) that that is quite correct, but that does not mean that that is for max force since it neglects frequency.  I can see how that might be for max laser sideband generation w/o constraining the frequency. (ie very high radial modes)
Thanks.  But where does the frequency enter into the equation for L? all I see is a, b, r2bar, r1bar, lo geometrical parameters.  It seems that the electromagnetic field Power only enters through the parameter (Uo)2 which is built inside the length "lo", so as we said, we need a number for "lo" to make any  more progress.

See Marco's Eq. 19, 20 and 28 in http://forum.nasaspaceflight.com/index.php?action=dlattach;topic=36313.0;attach=830137

It is the choice of the resonant mode that fixes l0 and this depends on frequency (indirectly). You should check Egan link for some numbers to put in.

Yes, thank you.  I actually have a better exact, and more elegant solution than Egan's that unlike Egan's can calculate arbitrary field variation in the azimuthal direction (no I have not published it).  I have calculated higher modes like the one that Eagleworks is running with the frustum (TM212) and my solution is very close to the experiments and the Finite Element calculations.  Greg Egan only shows the solution for constant quantum number m corresponding to constant field in the azimuthal direction.

I think (?) that Notsosureofit's point is that while your paper has a dependence on Uo, it is a single variable, as you said your solution is a first order solution.  But I shouldn't be speaking for him  :)
Title: Re: EM Drive Developments - related to space flight applications - Thread 2
Post by: txdrive on 05/17/2015 08:13 pm
With regards to the forces on the side of the cavity, all I can say is, write the field equations for your fields (what is the H and E at a given point in space), then I can calculate the forces on the boundaries and/or show that your fields violate Maxwell's equations. When it is a picture I don't know what exactly you're talking about.

Last line say it all:

Quote
If the absorbing surface is planar at an angle a to the radiation source, the intensity across the surface will be reduced.

The effective radiation source is the vertex of the frustum. The frustum side wall are aligned to, pointing at the vertex. Therefore the radiant pressures on the side walls are zero.
You're ignoring diffraction. If the frustum side walls were not there, the radiation would leak out.

edit: write an equation of form E(position, time) and H(position, time) . Maybe it would be clearer for a cylinder with flat end pieces? You can't just take geometric optics approximation from Wikipedia and apply it to microwaves in a reasonable sized cavity... it won't be correct then.

We were discussing radiation pressure generated from EM waves according to Maxwell's equations (attached). This pressure is apparently subject to cosine angle loss and thus when the EM wave moves along the frustum side walls and is not bouncing off the frustum spherical end plates, there is no Maxwell radiation pressure generated on the frustum side walls.

Or did I not understand what the cosine loss factor is for in the lower of the 2 equations?
It's a geometric optics approximation. Doesn't hold precisely.

For example suppose you got a beam of coherent light, 5mm across, 500nm wavelength, in space. It's not going all parallel to it's original direction, it's spreading, to about 1m across at 10 000m from the source (Makes a fuzzy "Airy disk" pattern).

edit: The Poynting vector on the sides of a perfect cylinder over this beam, is not parallel to the axis. It's this Poynting vector that matters.

Built and used many optical telescopes, including a 12 inch, Schmidt Cassegrain. Diffraction and I got to be good enemies.

There is no natural spread for the beam inside the cavity. It is controlled by the guide and cutoff wavelengths determined by the cavity diameter it travels through
Which only happens thanks to the reflection off the sides.
Quote
and by bouncing off the spherical end plates, which as I see it orient the EM waves so they travel / slide along the side walls at a 0 bounce radiant cosine angle.
What this boils down to, is that if you actually specify the E and H, not with a picture but with equations (note that you can't have a spherically symmetrical wave with EM fields, because it doesn't work like a pressure wave), it will be possible to either show that 1: Maxwell's equations don't hold for E and H, or 2: there is a force on the walls.

By the way both Shawyer and White should do that, give their solution for the field inside the cavity - E and H fields as a function of time and position. White's field has to deviate from Maxwell's equations to net force on the cavity. By an amount that should be very easy to detect electrically (without measuring forces), even if the net force is very small. It is strange to be "weighting" electromagnetic waves when they can be measured more precisely with electronics. A test for an unexpected change in the momentum of electromagnetic waves is best done with a direct measurement of said waves.

If we used radiation pressure measurements to measure radiation, we would never have known that stars other than the Sun exist. We can literally detect trillions times smaller effects by measuring radiation directly than by measuring radiation pressure.
Title: Re: EM Drive Developments - related to space flight applications - Thread 2
Post by: ThinkerX on 05/17/2015 09:54 pm
Seems to me that the geometric / electric precision required to make a workable EM Drive is comparable with the degree of precision required in automotive mechanics.  There, if the cylinder is flawed or damaged, the engine either won't work or will work poorly.  (I have known a number of backyard mechanics who wrongly thought they could re-bore the cylinders on their engines to the required degree of precision).   
Title: Re: EM Drive Developments - related to space flight applications - Thread 2
Post by: Iulian Berca on 05/17/2015 11:20 pm
Test 03 Success. I have thrust.
I modified the setup and now i weight the frustum. the precision is much better.

https://youtu.be/Rbf7735o3hQ

I will make a modifications to be able to adjust the cavity length to achieve the resonance, so i should have more thrust then.
Title: Re: EM Drive Developments - related to space flight applications - Thread 2
Post by: RotoSequence on 05/18/2015 12:44 am
Test 03 Success. I have thrust.
I modified the setup and now i weight the frustum. the precision is much better.

I will make a modifications to be able to adjust the cavity length to achieve the resonance, so i should have more thrust then.

Interesting results, but since the apparatus has been changed and the orientation of the device altered, and there's no dielectric material in the cavity providing an extra experimental variable, wouldn't it be prudent to test for a thermal explanation of the thrust before modifying the device?
Title: Re: EM Drive Developments - related to space flight applications - Thread 2
Post by: davish on 05/18/2015 12:45 am
Test 03 Success. I have thrust.
I modified the setup and now i weight the frustum. the precision is much better.

https://youtu.be/Rbf7735o3hQ

I will make a modifications to be able to adjust the cavity length to achieve the resonance, so i should have more thrust then.

Do you have the weight of the setup, so that we can calculate the newtons of thrust from this?
Title: Re: EM Drive Developments - related to space flight applications - Thread 2
Post by: txdrive on 05/18/2015 12:47 am
Test 03 Success. I have thrust.
I modified the setup and now i weight the frustum. the precision is much better.

https://youtu.be/Rbf7735o3hQ

I will make a modifications to be able to adjust the cavity length to achieve the resonance, so i should have more thrust then.
Beware of buoyancy, thermal expansion of cables, magnetic forces on cables, vibration, etc etc. Also, most importantly, safety: you got high voltage and microwaves that can really damage your eyes (so don't be closer than a couple meters from it when it is on). Unless you get the same thrust upwards as downwards down to, say, 5% , with less than 5% of the thrust if you mount it sideways, without excessive fiddling and adjustments, I would just describe it as a deflection rather than thrust. Thrust is a hypothesis, and an experimenter should be impartial towards hypotheses. Essentially without tests sideways (so the weight change is zero) and upside down, it is less rigorous than soft sciences (which nowadays employ controls).
Title: Re: EM Drive Developments - related to space flight applications - Thread 2
Post by: TheTraveller on 05/18/2015 12:59 am
Test 03 Success. I have thrust.
I modified the setup and now i weight the frustum. the precision is much better.

https://youtu.be/Rbf7735o3hQ

I will make a modifications to be able to adjust the cavity length to achieve the resonance, so i should have more thrust then.

Do you have the weight of the setup, so that we can calculate the newtons of thrust from this?

-0.508gf is about 4.98 mN upward force.

Shawyers 1st device best result was 16mN at 850W input power.

http://emdrive.com/feasibilitystudy.html
Title: Re: EM Drive Developments - related to space flight applications - Thread 2
Post by: Rodal on 05/18/2015 01:05 am
....

Dear Jose,

Thank you a lot for your effort to this question. Also thanks to @notsureofit for his help. You are right. I have checked this with Maple and optimized the function L(r) as a function of r1, r2 and U0 also. And yes, r1=r2 is the solution! I have other checks to do and my notebook just keeps on freezing for a computation of these but I hope to complete all in a few days.

This means that Harold White experiments could represent a great leap beyond in experimental general relativity. This was my initial hope. As a theoretical physicist please don't ask me about applications!

Give me a few days for further checks and I will update my draft, with due acknowledgements.

Regards.

Dear Marco,

As you said, "the geometry comes to the rescue".  Yes it does !

One must consider in detail what happens at r=r1 for r1 approaching zero:  r1 -> 0 the Log[c] term gets removed and geometry comes to the rescue!


When I consider what happens at r=1 in detail, it turns out that the pointy cone, with r1 -> 0 is the key.

All the interesting action takes place near r=r1, for r1 -> 0

Of course, the pointy cone is an abstraction that we cannot obtain in practice.

I look forward to you running some numbers during the coming days   :) to further throw light on this.

Please see:  http://forum.nasaspaceflight.com/index.php?topic=36313.msg1375542#msg1375542
Title: Re: EM Drive Developments - related to space flight applications - Thread 2
Post by: phaseshift on 05/18/2015 01:09 am
It's awesome to see the above results. Kudos!

In the meantime the script that is being worked on now generates the large plate from a couple parameters - length of the untruncated cone and the diameter of the large end of the cone). 

The thickness is calculated by taking the depth of the concave surface and adding a couple millimeters.

In this run the untruncated cone is 357.2 mm in length and the diameter of the concave area is 223.8 mm with a depth of 17.9 mm.
Title: Re: EM Drive Developments - related to space flight applications - Thread 2
Post by: kdhilliard on 05/18/2015 01:17 am
Test 03 Success. I have thrust.

I'm new to this thread, but doesn't Shawyer's theory paper say that it is supposed to be Big-Endian?  That is, that a larger force is applied on the inner face of the big end than on the inner face of the small end, so that if it were flying free in space it would accelerate big end first?

Is Iulian's Little-Endian negative reading consistent with other theory or experimental results?

~Kirk
Title: Re: EM Drive Developments - related to space flight applications - Thread 2
Post by: Rodal on 05/18/2015 01:24 am
Test 03 Success. I have thrust.
I modified the setup and now i weight the frustum. the precision is much better.

https://youtu.be/Rbf7735o3hQ

I will make a modifications to be able to adjust the cavity length to achieve the resonance, so i should have more thrust then.

Congratulations !

In the best tradition of

(http://upload.wikimedia.org/wikipedia/commons/3/3b/Traian_vuia_flying_machine.jpg)

Traian Vuia Romanian inventor and aviation pioneer who designed, built and tested a tractor configuration monoplane. He was the first to demonstrate that a flying apparatus could rise into the air by running upon wheels upon an ordinary road.He is credited with a powered hop of 11 metres (36 feet) made on March 18, 1906 and he later claimed a powered hop of 24 metres (79 feet)

(http://www.aikirentacar.ro/images/RomaniaFlag.gif)
Title: Re: EM Drive Developments - related to space flight applications - Thread 2
Post by: TheTraveller on 05/18/2015 01:27 am
Test 03 Success. I have thrust.

I'm new to this thread, but doesn't Shawyer's theory paper say that it is supposed to be Big-Endian?  That is, that a larger force is applied on the inner face of the big end than on the inner face of the small end, so that if it were flying free in space it would accelerate big end first?

Is Iulian's Little-Endian negative reading consistent with other theory or experimental results?

~Kirk

Watch Shawyers dynamic test.
http://emdrive.com/dynamictests.html

Moves toward small end. So both tests generate device movement in the same direction. Toward the small end.
Title: Re: EM Drive Developments - related to space flight applications - Thread 2
Post by: demofsky on 05/18/2015 01:48 am
Test 03 Success. I have thrust.

I'm new to this thread, but doesn't Shawyer's theory paper say that it is supposed to be Big-Endian?  That is, that a larger force is applied on the inner face of the big end than on the inner face of the small end, so that if it were flying free in space it would accelerate big end first?

Is Iulian's Little-Endian negative reading consistent with other theory or experimental results?

~Kirk

Watch Shawyers dynamic test.
http://emdrive.com/dynamictests.html

Moves toward small end. So both tests generate device movement in the same direction. Toward the small end.

Also interesting, and just to remind folks, it appears that Iulian has the waveguide directly inside the fulstrum ending some what near the centre of the cavity.  Sawyer, et al all seemed to use an antenna of some type that was close to one side of the cavity.
Title: Re: EM Drive Developments - related to space flight applications - Thread 2
Post by: TheTraveller on 05/18/2015 01:54 am
Test 03 Success. I have thrust.

I'm new to this thread, but doesn't Shawyer's theory paper say that it is supposed to be Big-Endian?  That is, that a larger force is applied on the inner face of the big end than on the inner face of the small end, so that if it were flying free in space it would accelerate big end first?

Is Iulian's Little-Endian negative reading consistent with other theory or experimental results?

~Kirk

Watch Shawyers dynamic test.
http://emdrive.com/dynamictests.html

Moves toward small end. So both tests generate device movement in the same direction. Toward the small end.

Also interesting, and just to remind folks, it appears that Iulian has the waveguide directly inside the fulstrum ending some what near the centre of the cavity.  Sawyer, et al all seemed to use an antenna of some type that was close to one side of the cavity.

In the 1st EM Drive device, Shawyer fed the microwaves in via a short waveguide to around the centre of the frustum. You can see the wave guide at the rear of the frustum.

In the next Demonstration EM Drive, the magnetron generated microwaves were fed into the frustum at a point near the big end as can be seen in these 2 last images.

The 3rd device, the Flight Thruster, used a coax Rf feed at near the big end.

So it would seem near the big end feeding will work.
Title: Re: EM Drive Developments - related to space flight applications - Thread 2
Post by: Rodal on 05/18/2015 01:55 am
Test 03 Success. I have thrust.

I'm new to this thread, but doesn't Shawyer's theory paper say that it is supposed to be Big-Endian?  That is, that a larger force is applied on the inner face of the big end than on the inner face of the small end, so that if it were flying free in space it would accelerate big end first?

Is Iulian's Little-Endian negative reading consistent with other theory or experimental results?

~Kirk

1) The acceleration and displacement occurs towards the small end. That's what matters. What gets measured are displacements (and hence a force due to Force=stiffness*displacement) or acceleration (and hence Force=mass*acceleration).

2) Shawyer claims that this is a result of a recoil force (just as when you fire a gun) due to a thrust force towards the big end.  Shawyer's explanation for this "thrust" runs against what we know about Maxwell's equations because the EM Drive is a closed cavity and there are no particles escaping.  However, NASA Eagleworks does not claim this and there are other, more scientific theories to explain what may be happening (if the effect is not an artifact).
Title: Re: EM Drive Developments - related to space flight applications - Thread 2
Post by: demofsky on 05/18/2015 02:20 am
Test 03 Success. I have thrust.

I'm new to this thread, but doesn't Shawyer's theory paper say that it is supposed to be Big-Endian?  That is, that a larger force is applied on the inner face of the big end than on the inner face of the small end, so that if it were flying free in space it would accelerate big end first?

Is Iulian's Little-Endian negative reading consistent with other theory or experimental results?

~Kirk

Watch Shawyers dynamic test.
http://emdrive.com/dynamictests.html

Moves toward small end. So both tests generate device movement in the same direction. Toward the small end.

Also interesting, and just to remind folks, it appears that Iulian has the waveguide directly inside the fulstrum ending some what near the centre of the cavity.  Sawyer, et al all seemed to use an antenna of some type that was close to one side of the cavity.

In the 1st EM Drive device, Shawyer fed the microwaves in via a short waveguide to around the centre of the frustum. You can see the wave guide at the rear of the frustum.

In the next Demonstration EM Drive, the magnetron generated microwaves were fed into the frustum at a point near the big end as can be seen in these 2 last images.

The 3rd device, the Flight Thruster, used a coax Rf feed at near the big end.

So it would seem near the big end feeding will work.

Thanks for this.  :)   I never noticed the waveguide in the first device!  Do you know if the waveguides terminated at or near the interior surface of the fulstrum?
Title: Re: EM Drive Developments - related to space flight applications - Thread 2
Post by: aero on 05/18/2015 02:37 am
If any calculated results from Meep are relevant then note that Meep calculates the largest thrust, O(1.1/c), when the antenna is dead center in the cavity. That is a point source. A dipole source is centered and parallel to the base plates for electric stimulation, and parallel to the axis of symmetry for magnetic stimulation.

Moving the antenna away from center reduces the detected force/flux ratio.
Title: Re: EM Drive Developments - related to space flight applications - Thread 2
Post by: zellerium on 05/18/2015 02:55 am
Test 03 Success. I have thrust.
I modified the setup and now i weight the frustum. the precision is much better.

I will make a modifications to be able to adjust the cavity length to achieve the resonance, so i should have more thrust then.

Iulin Berca:

Congratulations on the success! I think this demonstration is fantastic. The fact that the peak thrust appears ~3 seconds after the power is turned on tells me it takes a little while for energy to 'build up' in the cavity. After the peak, the thrust slowly dies probably because the cavity walls are warping and you are losing resonance.

The adjustable end plate would certainly help to achieve a higher peak thrust, it just might take some trial and error. But I believe cavity will still heat up and warp and the thrust will taper off.

Maybe this thermal effect could be mitigated using some sort of heat sink?
Title: Re: EM Drive Developments - related to space flight applications - Thread 2
Post by: deltaMass on 05/18/2015 03:25 am
If I did the calculation right, a volume change of ~125 litres would be needed to account for this apparent weight change, due to thermal ballooning of a sealed volume. That's far too high a change in volume that could be reasonably expected, so it can't be the whole story.  Did I calculate this right?
air density = 4*10-4 Kg/m3
Title: Re: EM Drive Developments - related to space flight applications - Thread 2
Post by: WarpTech on 05/18/2015 03:40 am
Test 03 Success. I have thrust.
I modified the setup and now i weight the frustum. the precision is much better.

I will make a modifications to be able to adjust the cavity length to achieve the resonance, so i should have more thrust then.

Awesome! 4+ mN! Though you may want to wear a foil hat and some chainmail when you turn that on. Make yourself a Faraday cage around it, so you're safe before you do too many test. It will also eliminate the accumulation of static charge, considering that thing is not grounded. We just want you to be safe.
Title: Re: EM Drive Developments - related to space flight applications - Thread 2
Post by: aero on 05/18/2015 04:02 am
If I did the calculation right, a volume change of ~125 litres would be needed to account for this apparent weight change, due to thermal ballooning of a sealed volume. That's far too high a change in volume that could be reasonably expected, so it can't be the whole story.  Did I calculate this right?
air density = 4*10-4 Kg/m3

1.225 kg/m3  http://en.wikipedia.org/wiki/Density_of_air (http://en.wikipedia.org/wiki/Density_of_air)
Title: Re: EM Drive Developments - related to space flight applications - Thread 2
Post by: deltaMass on 05/18/2015 04:20 am
If I did the calculation right, a volume change of ~125 litres would be needed to account for this apparent weight change, due to thermal ballooning of a sealed volume. That's far too high a change in volume that could be reasonably expected, so it can't be the whole story.  Did I calculate this right?
air density = 4*10-4 Kg/m3

1.225 kg/m3  http://en.wikipedia.org/wiki/Density_of_air (http://en.wikipedia.org/wiki/Density_of_air)
Oops yes - it's deltaVolume = 400 cc that's needed to account for all the upthrust.
Still too big to reasonably expect.
Title: Re: EM Drive Developments - related to space flight applications - Thread 2
Post by: aero on 05/18/2015 04:26 am
If I did the calculation right, a volume change of ~125 litres would be needed to account for this apparent weight change, due to thermal ballooning of a sealed volume. That's far too high a change in volume that could be reasonably expected, so it can't be the whole story.  Did I calculate this right?
air density = 4*10-4 Kg/m3

1.225 kg/m3  http://en.wikipedia.org/wiki/Density_of_air (http://en.wikipedia.org/wiki/Density_of_air)
Oops yes - it's deltaVolume = 400 cc that's needed to account for all the upthrust.
Still too big to reasonably expect.
I don't think you want the density of air, rather the change in density of air with temperature. If the it is a sealed volume then the total air mass will be the same at any temperature. But I guess you are calculating buoyancy which is the density of the outside-the-cavity air times the change in volume of the cavity.

That is just one reason to turn the cavity upside down and re-run the test.
Title: Re: EM Drive Developments - related to space flight applications - Thread 2
Post by: deltaMass on 05/18/2015 04:27 am
Yes, for the buoyancy calculation you need the density of air.
For example, a coke can with an internal temperature rise of about 30oC will give an upthrust of about 0.1 mg-wt due to slight ballooning of its thin walls.

And I agree that turning it upside down is the first sensible control experiment.
Title: Re: EM Drive Developments - related to space flight applications - Thread 2
Post by: Iulian Berca on 05/18/2015 04:41 am
If I did the calculation right, a volume change of ~125 litres would be needed to account for this apparent weight change, due to thermal ballooning of a sealed volume. That's far too high a change in volume that could be reasonably expected, so it can't be the whole story.  Did I calculate this right?
air density = 4*10-4 Kg/m3

The volume is not completely air tide sealed. Air inside can escape around the hole were the magnetron antenna goes inside.I will try more tests with this setup. I noticed when the magnetron is completely cold the thrust is a even higher.The test was done with the magnetron already heated up. So if the magnetron  stays cool then thrust is even higher. Water cooling is a good solution but is not easy to make a good setup and without creating vibrations.
Title: Re: EM Drive Developments - related to space flight applications - Thread 2
Post by: demofsky on 05/18/2015 04:45 am
If any calculated results from Meep are relevant then note that Meep calculates the largest thrust, O(1.1/c), when the antenna is dead center in the cavity. That is a point source. A dipole source is centered and parallel to the base plates for electric stimulation, and parallel to the axis of symmetry for magnetic stimulation.

Moving the antenna away from center reduces the detected force/flux ratio.

Interesting.  For the fulstrum Eagleworks tested in a vacuum chamber, they used a loop antenna.  (I also have a vague memory of someone saying that Sawyer used a T antenna parallel to the major axis of the fulstrum.)

In any case I have to wonder if this is a major factor in the performance of these devices, reflecting your simulations.  Also the use of a magnetron seems to be important.


....

Be that as it may, the main reason that we went with the lower-Q TM modes was because they consistently produced higher thrust levels for a given input power than the TE modes.  I will grant you though that getting the most thrust out of a particular resonant mode depended very painfully on the size, placement and rotational orientation of the loop antenna in the frustum cavity.




Title: Re: EM Drive Developments - related to space flight applications - Thread 2
Post by: deltaMass on 05/18/2015 04:52 am
A 30oC temperature change of the air inside the (non-sealed) cavity suffices to account for all the thrust, based on dm = V d(rho):

rho @20oC ~= 1.2 Kg/m3
rho @50oC ~= 1.1 Kg/m3
so d(rho) ~= 0.1 Kg/m3
so taking V ~= 6 litres = 6*10-3 m3, we get

dm ~= 6*10-4 Kg = 0.6 gm

http://www.engineeringtoolbox.com/air-density-specific-weight-d_600.html

So again, the most important next test is to flip it upside down.
Title: Re: EM Drive Developments - related to space flight applications - Thread 2
Post by: Iulian Berca on 05/18/2015 05:03 am
Test 03 Success. I have thrust.
I modified the setup and now i weight the frustum. the precision is much better.

I will make a modifications to be able to adjust the cavity length to achieve the resonance, so i should have more thrust then.

Iulin Berca:

Congratulations on the success! I think this demonstration is fantastic. The fact that the peak thrust appears ~3 seconds after the power is turned on tells me it takes a little while for energy to 'build up' in the cavity. After the peak, the thrust slowly dies probably because the cavity walls are warping and you are losing resonance.

The adjustable end plate would certainly help to achieve a higher peak thrust, it just might take some trial and error. But I believe cavity will still heat up and warp and the thrust will taper off.

Maybe this thermal effect could be mitigated using some sort of heat sink?

I rather think the time difference between applying power and thrust is due to the time needed to heat up the filament (cathode) in the magnetron. After ~ 3.5 - 4 seconds you hear that buzz inside, exactly in that moment the thrust appears. After that the buzz drops a little bit and also the thrust. After work, i will turn the cavity upside down to see if i can have the opposite thrust.
Title: Re: EM Drive Developments - related to space flight applications - Thread 2
Post by: deltaMass on 05/18/2015 05:23 am
What's the cavity volume, roughly? (is 6 litres close?)
Title: Re: EM Drive Developments - related to space flight applications - Thread 2
Post by: CW on 05/18/2015 06:01 am
Test 03 Success. I have thrust.
I modified the setup and now i weight the frustum. the precision is much better.

I will make a modifications to be able to adjust the cavity length to achieve the resonance, so i should have more thrust then.

Iulin Berca:

Congratulations on the success! I think this demonstration is fantastic. The fact that the peak thrust appears ~3 seconds after the power is turned on tells me it takes a little while for energy to 'build up' in the cavity. After the peak, the thrust slowly dies probably because the cavity walls are warping and you are losing resonance.

The adjustable end plate would certainly help to achieve a higher peak thrust, it just might take some trial and error. But I believe cavity will still heat up and warp and the thrust will taper off.

Maybe this thermal effect could be mitigated using some sort of heat sink?

I rather think the time difference between applying power and thrust is due to the time needed to heat up the filament (cathode) in the magnetron. After ~ 3.5 - 4 seconds you hear that buzz inside, exactly in that moment the thrust appears. After that the buzz drops a little bit and also the thrust. After work, i will turn the cavity upside down to see if i can have the opposite thrust.

I agree. Take some good headphones, crank up the volume. Pretty much immediately when the buzz starts, the scale's display shoots up. Next step should be to try and reverse the orientation and repeat. If it's very close to the previous value, but with positive sign, buoyancy can be scratched IMHO. You also have to try and exclude the possibility for eletromagnetically induced mechanical forces on the setup during high-power operation, coming from the power wires leading to the magnetron.
Title: Re: EM Drive Developments - related to space flight applications - Thread 2
Post by: LasJayhawk on 05/18/2015 06:30 am
Test 03 Success. I have thrust.
I modified the setup and now i weight the frustum. the precision is much better.

I will make a modifications to be able to adjust the cavity length to achieve the resonance, so i should have more thrust then.

Iulin Berca:

Congratulations on the success! I think this demonstration is fantastic. The fact that the peak thrust appears ~3 seconds after the power is turned on tells me it takes a little while for energy to 'build up' in the cavity. After the peak, the thrust slowly dies probably because the cavity walls are warping and you are losing resonance.

The adjustable end plate would certainly help to achieve a higher peak thrust, it just might take some trial and error. But I believe cavity will still heat up and warp and the thrust will taper off.

Maybe this thermal effect could be mitigated using some sort of heat sink?

I rather think the time difference between applying power and thrust is due to the time needed to heat up the filament (cathode) in the magnetron. After ~ 3.5 - 4 seconds you hear that buzz inside, exactly in that moment the thrust appears. After that the buzz drops a little bit and also the thrust. After work, i will turn the cavity upside down to see if i can have the opposite thrust.
I agree on the filament heating time causing the delay. It is best if you can allow the filament to heat up before applying the anode voltage. Also note that the center frequency and spectrum of a magnetron changes with the temperature of the tube. You may be observing that effect as well.
Title: Re: EM Drive Developments - related to space flight applications - Thread 2
Post by: Left Field on 05/18/2015 06:43 am
Test 03 Success. I have thrust.
I modified the setup and now i weight the frustum. the precision is much better.

I will make a modifications to be able to adjust the cavity length to achieve the resonance, so i should have more thrust then.

Iulin Berca:

Congratulations on the success! I think this demonstration is fantastic. The fact that the peak thrust appears ~3 seconds after the power is turned on tells me it takes a little while for energy to 'build up' in the cavity. After the peak, the thrust slowly dies probably because the cavity walls are warping and you are losing resonance.

The adjustable end plate would certainly help to achieve a higher peak thrust, it just might take some trial and error. But I believe cavity will still heat up and warp and the thrust will taper off.

Maybe this thermal effect could be mitigated using some sort of heat sink?

I rather think the time difference between applying power and thrust is due to the time needed to heat up the filament (cathode) in the magnetron. After ~ 3.5 - 4 seconds you hear that buzz inside, exactly in that moment the thrust appears. After that the buzz drops a little bit and also the thrust. After work, i will turn the cavity upside down to see if i can have the opposite thrust.
Well done!

Would it also be prudent after a normal test run to slide the scale out from under the beam and energise the magnetron again to demonstrate that there are no interference effects at play on its electronics (thanks to the copper shield)?

BTW: Get yourself a pair of these glasses:

(http://www.bttfstore.com/back_to_the_future_part_2_1989_685x385.jpg)
Title: Re: EM Drive Developments - related to space flight applications - Thread 2
Post by: zen-in on 05/18/2015 06:43 am
Test 03 Success. I have thrust.
I modified the setup and now i weight the frustum. the precision is much better.

I will make a modifications to be able to adjust the cavity length to achieve the resonance, so i should have more thrust then.

Iulin Berca:

Congratulations on the success! I think this demonstration is fantastic. The fact that the peak thrust appears ~3 seconds after the power is turned on tells me it takes a little while for energy to 'build up' in the cavity. After the peak, the thrust slowly dies probably because the cavity walls are warping and you are losing resonance.

The adjustable end plate would certainly help to achieve a higher peak thrust, it just might take some trial and error. But I believe cavity will still heat up and warp and the thrust will taper off.

Maybe this thermal effect could be mitigated using some sort of heat sink?

I rather think the time difference between applying power and thrust is due to the time needed to heat up the filament (cathode) in the magnetron. After ~ 3.5 - 4 seconds you hear that buzz inside, exactly in that moment the thrust appears. After that the buzz drops a little bit and also the thrust. After work, i will turn the cavity upside down to see if i can have the opposite thrust.

Congratulations!   You have shown a lot of ingenuity.   One question I have however:   What happens when you move the electronic scale a few inches so that it is not under the metal boom supporting the cavity and you apply power to the magnetron?    Have you ruled out the possibility the RF energy is causing a false positive reading?  Another possible source of a false positive is the stiffening of the power wires when the magnetron is on.   This is due to magnetic forces between the conductors.    Those are possible sorces of error in your experiment.    I admire your ability to quickly build a workable test system.
Title: Re: EM Drive Developments - related to space flight applications - Thread 2
Post by: Iulian Berca on 05/18/2015 08:20 am
Test 03 Success. I have thrust.
I modified the setup and now i weight the frustum. the precision is much better.

I will make a modifications to be able to adjust the cavity length to achieve the resonance, so i should have more thrust then.

Iulin Berca:

Congratulations on the success! I think this demonstration is fantastic. The fact that the peak thrust appears ~3 seconds after the power is turned on tells me it takes a little while for energy to 'build up' in the cavity. After the peak, the thrust slowly dies probably because the cavity walls are warping and you are losing resonance.

The adjustable end plate would certainly help to achieve a higher peak thrust, it just might take some trial and error. But I believe cavity will still heat up and warp and the thrust will taper off.

Maybe this thermal effect could be mitigated using some sort of heat sink?

I rather think the time difference between applying power and thrust is due to the time needed to heat up the filament (cathode) in the magnetron. After ~ 3.5 - 4 seconds you hear that buzz inside, exactly in that moment the thrust appears. After that the buzz drops a little bit and also the thrust. After work, i will turn the cavity upside down to see if i can have the opposite thrust.

Congratulations!   You have shown a lot of ingenuity.   One question I have however:   What happens when you move the electronic scale a few inches so that it is not under the metal boom supporting the cavity and you apply power to the magnetron?    Have you ruled out the possibility the RF energy is causing a false positive reading?  Another possible source of a false positive is the stiffening of the power wires when the magnetron is on.   This is due to magnetic forces between the conductors.    Those are possible sorces of error in your experiment.    I admire your ability to quickly build a workable test system.

In the wires for the filament the voltage is AC so they can not have to much effect in only one direction, just some 50hz low  intensity vibrations.
The anode voltage is pulsed DC but the current is smaller ~ 0.2A.
I already tried the scale without weight and i do not have any disturbance in the  reading when i power on the magnetron.
I can use an external supply for the filament , in this way i hope to adjust the power and maybe the frequency. But is very dangerous because i will connect the cathode with 4Kv to my power supply. The capacitance in the transformer can send some hi voltage to the supply.
Title: Re: EM Drive Developments - related to space flight applications - Thread 2
Post by: TheTraveller on 05/18/2015 09:18 am
Test 03 Success. I have thrust.
I modified the setup and now i weight the frustum. the precision is much better.

I will make a modifications to be able to adjust the cavity length to achieve the resonance, so i should have more thrust then.

Iulin Berca:

Congratulations on the success! I think this demonstration is fantastic. The fact that the peak thrust appears ~3 seconds after the power is turned on tells me it takes a little while for energy to 'build up' in the cavity. After the peak, the thrust slowly dies probably because the cavity walls are warping and you are losing resonance.

The adjustable end plate would certainly help to achieve a higher peak thrust, it just might take some trial and error. But I believe cavity will still heat up and warp and the thrust will taper off.

Maybe this thermal effect could be mitigated using some sort of heat sink?

I rather think the time difference between applying power and thrust is due to the time needed to heat up the filament (cathode) in the magnetron. After ~ 3.5 - 4 seconds you hear that buzz inside, exactly in that moment the thrust appears. After that the buzz drops a little bit and also the thrust. After work, i will turn the cavity upside down to see if i can have the opposite thrust.

Congratulations!   You have shown a lot of ingenuity.   One question I have however:   What happens when you move the electronic scale a few inches so that it is not under the metal boom supporting the cavity and you apply power to the magnetron?    Have you ruled out the possibility the RF energy is causing a false positive reading?  Another possible source of a false positive is the stiffening of the power wires when the magnetron is on.   This is due to magnetic forces between the conductors.    Those are possible sorces of error in your experiment.    I admire your ability to quickly build a workable test system.

In the wires for the filament the voltage is AC so they can not have to much effect in only one direction, just some 50hz low  intensity vibrations.
The anode voltage is pulsed DC but the current is smaller ~ 5.3A.
I already tried the scale without weight and i do not have any disturbance in the  reading when i power on the magnetron.
I can use an external supply for the filament , in this way i hope to adjust the power and maybe the frequency. But is very dangerous because i will connect the cathode with 4Kv to my power supply. The capacitance in the transformer can send some hi voltage to the supply.

Big test is to invert cavity & see weight gain. Repeat small end up & then down say 10 times. Also need to leave pwr on for 30 sec or so.

Consider using a USB scale and record the data for reference.

To eliminate air heating / buoyancy as thrust source, maybe drill 6 x 1mm dia holes around both ends & middle of your frustum so as to vent the cavity in a way to stop heated air causing false positives. No Rf should leak out as hole size way below cutoff.

Could use 2 pwr transformers. 1 to supply heater and the other to supply the high voltage DC. Then can independently switch each off/on.
Title: Re: EM Drive Developments - related to space flight applications - Thread 2
Post by: deuteragenie on 05/18/2015 09:19 am
Dear Iulian,

Very nice.  If I followed your 3 experiments setup correctly, I believe that in the last experiment you changed both the location of the magnetron (moved from the big side to the small one) AND the way to measure "thrust". 

Would it be possible for you to repeat the experiment with the magnetron on the big side, but with your new way of measuring?

Please disregard if experiment 2 was performed with the new way of measuring.

Also, I suppose that starting the experiment with the cavity pre-heated would be helpful in minimizing the thermal effects.
Title: Re: EM Drive Developments - related to space flight applications - Thread 2
Post by: Flyby on 05/18/2015 10:13 am
 :o
Very exciting news to see another (apparent) positive result... - i use the word "apparent" because it is still not 100% validated, but it looks promising, for sure.

Can't wait for the other home-builders to join in and produce similar results.....

If we could have 3 positive results from independent testers and top that with the final results obtained from the Eagleworks research team (by July?), then i think a lot of the scientific community will turn heads and start wondering what's happening, instead of brushing it off as "impossible".

Gaining credibility is/was  the main goal...
Once that achieved, it will go fast... funding will unlock, researchers/engineers will be interested in participating...etc
Title: EM Drive Developments - related to space flight applications - Thread 2
Post by: Star One on 05/18/2015 10:24 am
It has been pointed out it causes a lot of RF interference during the test, could this in any way cause measuring issues especially using a digital balance?
Title: Re: EM Drive Developments - related to space flight applications - Thread 2
Post by: Flyby on 05/18/2015 10:38 am
Iulian answered that one (marked in red) :


In the wires for the filament the voltage is AC so they can not have to much effect in only one direction, just some 50hz low  intensity vibrations.
The anode voltage is pulsed DC but the current is smaller ~ 0.2A.
I already tried the scale without weight and i do not have any disturbance in the  reading when i power on the magnetron.
I can use an external supply for the filament , in this way i hope to adjust the power and maybe the frequency. But is very dangerous because i will connect the cathode with 4Kv to my power supply. The capacitance in the transformer can send some hi voltage to the supply.
Title: EM Drive Developments - related to space flight applications - Thread 2
Post by: Star One on 05/18/2015 11:29 am
Thanks I must have missed that.

If anything does come of this they better well shield the drives otherwise flying low enough you're going to risk knocking out everyone's electronic systems.
Title: Re: EM Drive Developments - related to space flight applications - Thread 2
Post by: txdrive on 05/18/2015 12:39 pm
If I did the calculation right, a volume change of ~125 litres would be needed to account for this apparent weight change, due to thermal ballooning of a sealed volume. That's far too high a change in volume that could be reasonably expected, so it can't be the whole story.  Did I calculate this right?
air density = 4*10-4 Kg/m3
1 litre weights about 1.2 grams, or 12 mN . Heated up by 10 degrees Kelvin it expands by about 1/30 , resulting in buoyancy of approximately 12/30 = 0.4 mN . Definitely enough to make an experiment inaccurate (4mN from 10 litres), albeit probably less than thermal and electromagnetic effects in cabling (which is basically impossible to estimate). edit: also, vibrations of something asymmetric in air should result in a force, which is also difficult to estimate.

The only thing he can do with cabling as it is, is see if it replicates turned upside down, with some accuracy (~5% for example), but gives a small result sideways, without excessive fiddling. But it probably won't even if the effect is there because of all the classical forces. To get any sort of conclusive validation you need some precision.
Title: Re: EM Drive Developments - related to space flight applications - Thread 2
Post by: TheTraveller on 05/18/2015 12:54 pm
If I did the calculation right, a volume change of ~125 litres would be needed to account for this apparent weight change, due to thermal ballooning of a sealed volume. That's far too high a change in volume that could be reasonably expected, so it can't be the whole story.  Did I calculate this right?
air density = 4*10-4 Kg/m3
1 litre weights about 1.2 grams, or 12 mN . Heated up by 10 degrees Kelvin it expands by about 1/30 , resulting in buoyancy of approximately 12/30 = 0.4 mN . Definitely enough to make an experiment inaccurate (4mN from 10 litres), albeit probably less than thermal and electromagnetic effects in cabling (which is basically impossible to estimate).

The only thing he can do with cabling as it is, is see if it replicates turned upside down, with some accuracy (~5% for example), without excessive fiddling. But it probably won't even if the effect is there because of all the classical forces.

Would you agree drilling 6 x 1 mm diameter equally spaced holes around the circumference at both ends of the frustum wall, say 5 mm away from the end plates, and the same around the frustum middle would eliminate any buoyancy or hot air jet false positives?
Title: Re: EM Drive Developments - related to space flight applications - Thread 2
Post by: Rodal on 05/18/2015 01:39 pm
If I did the calculation right, a volume change of ~125 litres would be needed to account for this apparent weight change, due to thermal ballooning of a sealed volume. That's far too high a change in volume that could be reasonably expected, so it can't be the whole story.  Did I calculate this right?
air density = 4*10-4 Kg/m3
1 litre weights about 1.2 grams, or 12 mN . Heated up by 10 degrees Kelvin it expands by about 1/30 , resulting in buoyancy of approximately 12/30 = 0.4 mN . Definitely enough to make an experiment inaccurate (4mN from 10 litres), albeit probably less than thermal and electromagnetic effects in cabling (which is basically impossible to estimate).

The only thing he can do with cabling as it is, is see if it replicates turned upside down, with some accuracy (~5% for example), without excessive fiddling. But it probably won't even if the effect is there because of all the classical forces.

Would you agree drilling 6 x 1 mm diameter equally spaced holes around the circumference at both ends of the frustum wall, say 5 mm away from the end plates, and the same around the frustum middle would eliminate any buoyancy or hot air jet false positives?

I wouldn't assume that IMHO, on the basis that the Ph.D. thesis that Shawyer uses as his only reference on radiation pressure measurements (Dr. Cullen's thesis) makes it clear that Cullen had to use a mesh as follows to eliminate air effects that have plagued these experiments since Maxwell's times:

(http://forum.nasaspaceflight.com/index.php?action=dlattach;topic=36313.0;attach=829247;image)

"drilling 6 x 1 mm diameter equally spaced holes around the circumference at both ends of the frustum wall, say 5 mm away from the end plates" may not be enough if the intent of the holes is to really eliminate a false positive.

It is puzzling to a researcher's mind that Shawyer uses Cullen as his only reference for radiation pressure from closed frustum cavities, when Cullen only used open waveguides with constant cross section in his pressure experiments, yet Shawyer apparently ignores Cullen's prescription on the need for a wired mesh to eliminate gas effects.
Title: Re: EM Drive Developments - related to space flight applications - Thread 2
Post by: TheTraveller on 05/18/2015 02:31 pm
If I did the calculation right, a volume change of ~125 litres would be needed to account for this apparent weight change, due to thermal ballooning of a sealed volume. That's far too high a change in volume that could be reasonably expected, so it can't be the whole story.  Did I calculate this right?
air density = 4*10-4 Kg/m3
1 litre weights about 1.2 grams, or 12 mN . Heated up by 10 degrees Kelvin it expands by about 1/30 , resulting in buoyancy of approximately 12/30 = 0.4 mN . Definitely enough to make an experiment inaccurate (4mN from 10 litres), albeit probably less than thermal and electromagnetic effects in cabling (which is basically impossible to estimate).

The only thing he can do with cabling as it is, is see if it replicates turned upside down, with some accuracy (~5% for example), without excessive fiddling. But it probably won't even if the effect is there because of all the classical forces.

Would you agree drilling 6 x 1 mm diameter equally spaced holes around the circumference at both ends of the frustum wall, say 5 mm away from the end plates, and the same around the frustum middle would eliminate any buoyancy or hot air jet false positives?

I wouldn't assume that IMHO, on the basis that the Ph.D. thesis that Shawyer uses as his only reference on radiation pressure measurements (Dr. Cullen's thesis) makes it clear that Cullen had to use a mesh as follows to eliminate air effects that have plagued these experiments since Maxwell's times:

(http://forum.nasaspaceflight.com/index.php?action=dlattach;topic=36313.0;attach=829247;image)

"drilling 6 x 1 mm diameter equally spaced holes around the circumference at both ends of the frustum wall, say 5 mm away from the end plates" would not be enough

It is puzzling to a researcher's mind that Shawyer uses Cullen as his only reference for radiation pressure from closed frustum cavities, when Cullen only used open waveguides with constant cross section in his pressure experiments, while Shawyer apparently ignores Cullen's prescription on the need for a wired mesh to eliminate gas effects.

18 holes, 1mm diameter = 14.2mm^2 of total air hole area to allow the internally heat air to not cause buoyancy or hot air gas jet false positives. Much simpler than the Cullen grid. As engineer, the rule is KISS.

Don't see how this relates to what EW, Shawyer and the Chinese did. None of those used the Cullen grid you are suggesting, so why bash Shawyer for not using it and not the others?

BTW in the Egan paper, could you please show me where he factors in the constantly changing cutoff wavelength, guide wavelength & group velocity as the EM wave bounces from end to end in a frustum with constantly varying diameter? Really can't follow how he calcs the resonate frequency for his frustum.
 
Title: Re: EM Drive Developments - related to space flight applications - Thread 2
Post by: Rodal on 05/18/2015 02:49 pm
.....Don't see how this relates to what EW, Shawyer and the Chinese did. None of those used the Cullen grid you are suggesting, so why bash Shawyer for not using it and not the others?...
As I said, because Shawyer is the only one that uses Cullen as his only reference on pressure radiation for the unorthodox pressure radiation calculations that Shawyer uses.  Neither Prof. Yang nor NASA Eagleworks use Cullen as a reference.  Pointing out the huge inconsistency in Shawyer using as his only reference a work that explicitly does NOT use cavities but instead uses open waveguides is not bashing somebody, it is pointing out a technical inconsistency that should bother any engineer.  Engineers care about accuracy.  Engineers that make bridges care about accuracy and aerospace engineers care even more about accuracy  (because of the weight constraints in aerospace engineering, the safety factor in aerospace engineering cannot be as high as in civil engineering, hence accuracy is paramount in aerospace engineering).  It is something that referees in peer-reviewed journals point out whenever engineering papers are reviewed for publication approval in a peer-reviewed journal. Ditto for Shawyer using Cullen as a reference for what Cullen does not do (measure pressure on cavities) and ignoring Cullen's prescription on what Cullen recommends to do (to use a mesh to get rid of the gas effect).

BTW in the Egan paper, could you please show me where he factors in the constantly changing cutoff wavelength, guide wavelength & group velocity as the EM wave bounces from end to end in a frustum with constantly varying diameter? Really can't follow how he calcs the resonate frequency for his frustum.
I can't help making it any more clear than what Greg Egan did.  Greg Egan's paper is extremely clear to me, as he uses textbook material.  Concerning the cut-off wavelength and associated cut-off frequencies, they are automatically built-in in the two eigenvalue problems that Egan addresses.

When one has an exact solution like Egan, one does not have to artificially impose a cut-off condition as a side-condition.  The reason why Shawyer has to impose the cut-off condition as a side-condition is because Shawyer's solution is obviously not an exact solution.  Shawyer's solution does not satisfy the Boundary Conditions of the problem as it has been pointed out in the forum previously.

EDIT: actually even a numerical solution, for example one using the Finite Element method (which is not exact of course, but it is instead based on variational principles), satisfies the cut-off conditions automatically, they are built-in in the eigenproblem (one does not have to artificially impose cut-off conditions on the FEA, the FEA eigensolution automatically cut-off modes).


The fact that Greg Egan's solution automatically includes the cut-off condition (and not as a side-condition) in the eigenvalue problem is evident in the examples given by Egan: notice that the frequency and mode shape associated with the quantum number p=0, constant in the longitudinal direction of the truncated cone, is automatically cut-off by the eigensolution shown by Egan.  The first natural frequency in the examples shown by Egan have p>0.

First TM mode (in Greg Egan's example):
(http://gregegan.customer.netspace.net.au/SCIENCE/Cavity/TM1.gif)

First TE mode (in Greg Egan's example):
(http://gregegan.customer.netspace.net.au/SCIENCE/Cavity/TE1.gif)
Title: Re: EM Drive Developments - related to space flight applications - Thread 2
Post by: TheTraveller on 05/18/2015 03:01 pm
I really have a problem with this Egan's explanation, attached below, which assumes all the wavelengths inside the cavity are the same length, which we know is not the case.

This theory of a simple resonance frequency, implying constant EM wavelength inside the cavity is just not correct.

This means his apparent lack of understanding of constantly varying frustum diameter, constantly altering cutoff wavelength, guide wavelength and group velocity, which are the reality, are thus not apparently modeled in his Shawyer debunking paper.

http://gregegan.customer.netspace.net.au/SCIENCE/Cavity/Simple.html
http://gregegan.customer.netspace.net.au/SCIENCE/Cavity/Cavity.html

Maybe the constantly varying wavelength inside the cavity is modeled in the full equations and ignored in his simple explanation. If so would someone please point out where they are modeled and how I can use Egan to calc the resonate frequency of my test frustum?

I desire this as a proof of the validity of the Egan calculations versus the resonate frequency I find inside my cavity. Effectively Egan has made a claim (which is his simple example is clearly wrong) and I intent to prove him right or wrong. Likewise Shawyer has made claims and I also intend to prove him right or wrong.
Title: Re: EM Drive Developments - related to space flight applications - Thread 2
Post by: TheTraveller on 05/18/2015 03:04 pm
.....Don't see how this relates to what EW, Shawyer and the Chinese did. None of those used the Cullen grid you are suggesting, so why bash Shawyer for not using it and not the others?...
As I said, because Shawyer is the only one that uses Cullen as his only reference on pressure radiation for the unorthodox pressure radiation calculations that Shawyer uses.  Neither Prof. Yang nor NASA Eagleworks use Cullen as a reference.  Pointing out the huge inconsistency in Shawyer using as his only reference a work that explicitly does NOT use cavities but instead uses open waveguides is not bashing somebody, it is pointing out a technical inconsistency that should bother any engineer.  Engineers care about accuracy.  Engineers that make bridges care about accuracy and aerospace engineers care even more about accuracy  (because of the weight constraints in aerospace engineering, the safety factor in aerospace engineering cannot be as high as in civil engineering, hence accuracy is paramount in aerospace engineering).  It is something that referees in peer-reviewed journals point out whenever engineering papers are reviewed for publication approval in a peer-reviewed journal. Ditto for Shawyer using Cullen as a reference for what Cullen does not do and ignoring Cullen's prescription on what Cullen recommends to do (to use a mesh).

BTW in the Egan paper, could you please show me where he factors in the constantly changing cutoff wavelength, guide wavelength & group velocity as the EM wave bounces from end to end in a frustum with constantly varying diameter? Really can't follow how he calcs the resonate frequency for his frustum.
I can't help making it any more clear than what Greg Egan did.  Greg Egan's paper is extremely clear to me, as he uses textbook material.  Concerning the cut-off wavelength and associated cut-off frequencies, they are automatically built-in in the two eigenvalue problems that Egan addresses.

When one has an exact solution like Egan, one does not have to artificially impose a cut-off condition as a side-condition.  The reason why Shawyer has to impose the cut-off condition as a side-condition is because Shawyer's solution is obviously not an exact solution.  Shawyer's solution does not satisfy the Boundary Conditions of the problem as it has been pointed out in the forum previously.

The fact that Greg Egan's solution automatically includes the cut-off condition (and not as a side-condition) in the eigenvalue problem is evident in the examples given by Egan: notice that the frequency and mode shape associated with the quantum number p=0, constant in the longitudinal direction of the truncated cone, is automatically cut-off by the eigensolution shown by Egan.  The first natural frequency in the examples shown by Egan have p>0.

And his exact solution to calc the resonant frequency of a frustum with spherical end plates is?
Title: Re: EM Drive Developments - related to space flight applications - Thread 2
Post by: Rodal on 05/18/2015 03:06 pm
...
And his exact solution to calc the resonant frequency of a frustum with spherical end plates is?
It is clearly given by Egan in here http://gregegan.customer.netspace.net.au/SCIENCE/Cavity/Cavity.html (for modes having the azimuthal quantum number m=0, but arbitrary n and p)

Egan even walks you through, step by step, examples of how to solve the eigenproblems for both the Legendre associated function and for the spherical Bessel function (there are textbooks that don't bother to walk the student through on how to solve the eigenproblem).
Title: Re: EM Drive Developments - related to space flight applications - Thread 2
Post by: TheTraveller on 05/18/2015 03:18 pm
...
And his exact solution to calc the resonant frequency of a frustum with spherical end plates is?
It is clearly given by Egan in here http://gregegan.customer.netspace.net.au/SCIENCE/Cavity/Cavity.html (for modes having the azimuthal quantum number m=0, but arbitrary n and p)

Egan even walks you through, step by step, examples of how to solve the eigenproblems for both the Legendre associated function and for the spherical Bessel function (there are textbooks that don't bother to walk the student through on how to solve the eigenproblem).

Working it.
Title: Re: EM Drive Developments - related to space flight applications - Thread 2
Post by: TheTraveller on 05/18/2015 03:40 pm
Looks like a photo of Shawyer's 1st knife edge balance beam test rig with the 1st EM Drive inside a sealed Faraday Cage.

http://www.shelleys.demon.co.uk/fdec02em.htm

Screws around the box seem to align up with the holes in the various mounting plates.
Title: Re: EM Drive Developments - related to space flight applications - Thread 2
Post by: Rodal on 05/18/2015 03:41 pm
I really have a problem with this Egan's explanation, attached below, which assumes all the wavelengths inside the cavity are the same length, which we know is not the case. ...
The solution shown in http://gregegan.customer.netspace.net.au/SCIENCE/Cavity/Cavity.html has an infinite number of eigenwavelengths.

I don't understand why one would assume that in a practical situation only one eigenwavelength (or equivalently only one eigenfrequency) would take place.  I don't think that Egan implies that.  On the contrary, Cullen (the reference used by Shawyer to justify his unorthodox calculations) explicitly shows how hard it was for him in his experimental Ph.D. thesis to excite the waveguide at just one eigenfrequency, as many eigenmodes are close to each other.  Cullen shows that it was practically impossible, the best he could do was to suppress the magnitude of other eigenfrequencies to a relatively low magnitude.

The reality is the contrary of what you apparently assume, Egan in  http://gregegan.customer.netspace.net.au/SCIENCE/Cavity/Cavity.html  discusses a solution to an eigenvalue problem having an infinite number of eigenwavelengths and in reality it is very difficult to have the cavity resonate at a single pure eigenwavelength, as many eigenmodes are close to each other.  This is particularly so when one uses a magnetron !
Title: Re: EM Drive Developments - related to space flight applications - Thread 2
Post by: tchernik on 05/18/2015 04:00 pm
Looks like a photo of Shawyer's 1st knife edge balance beam test rig with the 1st EM Drive inside a sealed Faraday Cage.

http://www.shelleys.demon.co.uk/fdec02em.htm

Screws around the box seem to align up with the holes in the various mounting plates.

I like this, because besides of eliminating hot air buoyancy as an explanation, by showing the device producing thrust in any direction, the logical next step for any DYI fan is to run the device fully enclosed in a Faraday cage, showing it producing thrust in any direction as well. Such setup would still be affordable for most DYI fans capable of building an Emdrive.

And from there, what's needed are vacuum tests. But those are significantly more expensive for a regular DYI fan, and would be easier done by people in universities and labs with facilities like vacuum chambers.
Title: Re: EM Drive Developments - related to space flight applications - Thread 2
Post by: TheTraveller on 05/18/2015 04:05 pm
I really have a problem with this Egan's explanation, attached below, which assumes all the wavelengths inside the cavity are the same length, which we know is not the case. ...
The solution shown in http://gregegan.customer.netspace.net.au/SCIENCE/Cavity/Cavity.html has an infinite number of eigenwavelengths.

I don't understand why one would assume that in a practical situation only one eigenwavelength (or equivalently only one eigenfrequency) would take place.  I don't think that Egan implies that.  On the contrary, Cullen (the reference used by Shawyer to justify his unorthodox calculations) explicitly shows how hard it was for him in his experimental Ph.D. thesis to excite the waveguide at just one eigenfrequency, as many eigenmodes are close to each other.  Cullen shows that it was practically impossible, the best he could do was to suppress the magnitude of other eigenfrequencies to a relatively low magnitude.

The reality is the contrary of what you apparently assume, Egan in  http://gregegan.customer.netspace.net.au/SCIENCE/Cavity/Cavity.html  discusses a solution to an eigenvalue problem having an infinite number of eigenwavelengths and in reality it is very difficult to have the cavity resonate at a single pure eigenwavelength, as many eigenmodes are close to each other.  This is particularly so when one uses a magnetron !

My point was simple.

There is not constant wavelength inside the cavity as Egan incorrectly shows there is.

Title: Re: EM Drive Developments - related to space flight applications - Thread 2
Post by: TheTraveller on 05/18/2015 04:12 pm
Looks like a photo of Shawyer's 1st knife edge balance beam test rig with the 1st EM Drive inside a sealed Faraday Cage.

http://www.shelleys.demon.co.uk/fdec02em.htm

Screws around the box seem to align up with the holes in the various mounting plates.

I like this, because besides of eliminating hot air buoyancy as an explanation, by showing the device producing thrust in any direction, the logical next step for any DYI fan is to run the device fully enclosed in a Faraday cage, showing it producing thrust in any direction as well. Such setup would still be affordable for most DYI fans capable of building an Emdrive.

And from there, what's needed are vacuum tests. But those are significantly more expensive for a regular DYI fan, and would be easier done by people in universities and labs with facilities like vacuum chambers.

Yes his box is easy to pick up and turn. No fiddly bits. All nicely secured so to reduce variables.This is basically what I'm building except my scale is under the right side of the balance beam, not under the left side Faraday Caged Em Drive and I'm using a programmable frequency and output level Rf generator & Rf amp instead of a magnetron and USB digital scale. That way I know what is going into the cavity.
Title: Re: EM Drive Developments - related to space flight applications - Thread 2
Post by: TheTraveller on 05/18/2015 04:16 pm
Interesting version of the C band Flight Thruster and dimensions.

www.slideshare.net/Stellvia/emdrive-presentation-at-space-08-conference-barbican-london-presentation
Title: Re: EM Drive Developments - related to space flight applications - Thread 2
Post by: rfmwguy on 05/18/2015 04:20 pm
If I did the calculation right, a volume change of ~125 litres would be needed to account for this apparent weight change, due to thermal ballooning of a sealed volume. That's far too high a change in volume that could be reasonably expected, so it can't be the whole story.  Did I calculate this right?
air density = 4*10-4 Kg/m3
1 litre weights about 1.2 grams, or 12 mN . Heated up by 10 degrees Kelvin it expands by about 1/30 , resulting in buoyancy of approximately 12/30 = 0.4 mN . Definitely enough to make an experiment inaccurate (4mN from 10 litres), albeit probably less than thermal and electromagnetic effects in cabling (which is basically impossible to estimate).

The only thing he can do with cabling as it is, is see if it replicates turned upside down, with some accuracy (~5% for example), without excessive fiddling. But it probably won't even if the effect is there because of all the classical forces.

Would you agree drilling 6 x 1 mm diameter equally spaced holes around the circumference at both ends of the frustum wall, say 5 mm away from the end plates, and the same around the frustum middle would eliminate any buoyancy or hot air jet false positives?

I wouldn't assume that IMHO, on the basis that the Ph.D. thesis that Shawyer uses as his only reference on radiation pressure measurements (Dr. Cullen's thesis) makes it clear that Cullen had to use a mesh as follows to eliminate air effects that have plagued these experiments since Maxwell's times:

(http://forum.nasaspaceflight.com/index.php?action=dlattach;topic=36313.0;attach=829247;image)

"drilling 6 x 1 mm diameter equally spaced holes around the circumference at both ends of the frustum wall, say 5 mm away from the end plates" would not be enough

It is puzzling to a researcher's mind that Shawyer uses Cullen as his only reference for radiation pressure from closed frustum cavities, when Cullen only used open waveguides with constant cross section in his pressure experiments, while Shawyer apparently ignores Cullen's prescription on the need for a wired mesh to eliminate gas effects.

18 holes, 1mm diameter = 14.2mm^2 of total air hole area to allow the internally heat air to not cause buoyancy or hot air gas jet false positives. Much simpler than the Cullen grid. As engineer, the rule is KISS.

Don't see how this relates to what EW, Shawyer and the Chinese did. None of those used the Cullen grid you are suggesting, so why bash Shawyer for not using it and not the others?

BTW in the Egan paper, could you please show me where he factors in the constantly changing cutoff wavelength, guide wavelength & group velocity as the EM wave bounces from end to end in a frustum with constantly varying diameter? Really can't follow how he calcs the resonate frequency for his frustum.

This grid might account for the unusual thermal signature of the large end base plate. Pardon my "warping" of the perspective to get it as close to a circle as time would permit.

This thermal pattern doesn't remind me of a concave surface pattern. It "could" be that the large base is copper-clad PC board with the "bulls eye" patern etched on the upper surface (connected to ground potential) and the bottom (outside) surface of the PC board completely copper. Perhaps this is the reason for the apparent thickness of the large base, a "suspended" pattern a few mm above the copper.

Just speculation...hope there is someone who can connect the dots. Original thermal image: https://d253pvgap36xx8.cloudfront.net/editor_uploads/1277/2015/05/06/NASA_emdrive2.jpg
Title: Re: EM Drive Developments - related to space flight applications - Thread 2
Post by: Rodal on 05/18/2015 04:20 pm
Interesting version of the C band Flight Thruster and dimensions.

www.slideshare.net/Stellvia/emdrive-presentation-at-space-08-conference-barbican-london-presentation
Never seen this before  ???

Not a truncated cone.

What happened to the spherical waves? 

Is there an explanation to its flat sides and the departure from the conical shape ?
Title: Re: EM Drive Developments - related to space flight applications - Thread 2
Post by: Rodal on 05/18/2015 04:27 pm
....

This grid might account for the unusual thermal signature of the large end base plate. Pardon my "warping" of the perspective to get it as close to a circle as time would permit.

This thermal pattern doesn't remind me of a concave surface pattern. It "could" be that the large base is copper-clad PC board with the "bulls eye" patern etched on the upper surface (connected to ground potential) and the bottom (outside) surface of the PC board completely copper. Perhaps this is the reason for the apparent thickness of the large base, a "suspended" pattern a few mm above the copper.

Just speculation...hope there is someone who can connect the dots. Original thermal image: https://d253pvgap36xx8.cloudfront.net/editor_uploads/1277/2015/05/06/NASA_emdrive2.jpg
Sorry, that:

https://d253pvgap36xx8.cloudfront.net/editor_uploads/1277/2015/05/06/NASA_emdrive2.jpg

is not a thermal image.  That is the result of the numerical solution using COMSOL FEA, with flat ends for the magnetic field in mode TM212.  It corresponds to the thermal losses because the thermal losses are due to induction heating from the magnetic field.

Here is the COMSOL FEA analysis of surface losses (to the left) and the thermal measurement (to the right)

(http://forum.nasaspaceflight.com/index.php?action=dlattach;topic=36313.0;attach=635194;image)
Title: Re: EM Drive Developments - related to space flight applications - Thread 2
Post by: TheTraveller on 05/18/2015 04:34 pm
Interesting version of the C band Flight Thruster and dimensions.

www.slideshare.net/Stellvia/emdrive-presentation-at-space-08-conference-barbican-london-presentation
Never seen this before  ???

Not a truncated cone.

What happened to the spherical waves? 

Is there an explanation to its flat sides and the departure from the conical shape ?

You did look at the superconducting device shape?

Shawyer likes to drop bread crumbs, to create a trail for those interested to follow.
Title: Re: EM Drive Developments - related to space flight applications - Thread 2
Post by: Rodal on 05/18/2015 04:40 pm
[...

Thanks Dr, in my world it was thermal. Interesting to note the interference pattern was divided into quadrants and the only image I could locate to explain it was the "diffuser". Soooo, guess no one did a thermal of the DUT?
For what it is worth, I obtained the same image (indistinguishable by eye) with my exact solution for the same mode shape TM212, using spherical ends.  It looks like it doesn't make a perceptible difference for these geometrical dimensions and for this mode TM212 magnetic field whether one has flat ends or spherical ends.

The pattern is due to the need to fulfill the boundary conditions and symmetry conditions for this higher mode.  Higher "m" modes have higher number of "cells" symmetrically distributed in the azimuthal "m" direction.
Title: Re: EM Drive Developments - related to space flight applications - Thread 2
Post by: TheTraveller on 05/18/2015 04:48 pm
...

Shawyer likes to drop bread crumbs, to create a trail for those interested to follow.
Bread crumbs?  I feel like a bird ... :)

Photo was from 2008 presentation.

Maybe it didn't produce enough thrust and Shawyer produced the final conical 2009 version?

I do note the Rf input is at the top and there is an E field sense output in the middleish of the flat side wall. Or maybe the other way around? Dimensions were given as was input power, which is lower than that published for the final Flight Thruster.
Title: Re: EM Drive Developments - related to space flight applications - Thread 2
Post by: txdrive on 05/18/2015 05:13 pm
Interesting version of the C band Flight Thruster and dimensions.

www.slideshare.net/Stellvia/emdrive-presentation-at-space-08-conference-barbican-london-presentation
Never seen this before  ???

Not a truncated cone.

What happened to the spherical waves? 

Is there an explanation to its flat sides and the departure from the conical shape ?
Why would it matter? The miscalculation of radiation pressure upon inclined surfaces works the same for a truncated pyramid.
Title: Re: EM Drive Developments - related to space flight applications - Thread 2
Post by: deltaMass on 05/18/2015 05:54 pm
The expectation for measured weight change for Iulian's device is about -0.5 gm-weight (becoming lighter), assuming a temperature rise of the air within the device of about 25oC, and assuming the device volume to be about 6 litres. The mechanism here is based simply on the change in weight of the air within the device. Because the device is not sealed, air can move in and out of it as its temperature changes.  The larger the device volume, the larger the expected measured weight change.

I've now calculated the volume to be ~37 litres, and so we need roughly only 1/6th of the previously calculated temperature rise to get the observed weight change - just a few degrees will do it.
Title: Re: EM Drive Developments - related to space flight applications - Thread 2
Post by: WarpTech on 05/18/2015 06:13 pm
The expectation for measured weight change for Iulian's device is about -0.5 gm-weight (becoming lighter), assuming a temperature rise of the air within the device of about 25oC, and assuming the device volume to be about 6 litres. The mechanism here is based simply on the change in weight of the air within the device. Because the device is not sealed, air can move in and out of it as its temperature changes.  The larger the device volume, the larger the expected measured weight change.

I've now calculated the volume to be ~37 litres, and so we need roughly only 1/6th of the previously calculated temperature rise to get the observed weight change - just a few degrees will do it.

I do not believe the air temp went up that fast. And, if it were just a change in air temperature, why would it drop as soon as it's switched off. The air inside can't cool that fast, and the thrust was repeatable with the switch.

Title: Re: EM Drive Developments - related to space flight applications - Thread 2
Post by: TheTraveller on 05/18/2015 06:13 pm
The expectation for measured weight change for Iulian's device is about -0.5 gm-weight (becoming lighter), assuming a temperature rise of the air within the device of about 25oC, and assuming the device volume to be about 6 litres. The mechanism here is based simply on the change in weight of the air within the device. Because the device is not sealed, air can move in and out of it as its temperature changes.  The larger the device volume, the larger the expected measured weight change.

A few points to consider:

1) Cavity is vented to atmo by 2 large holes in the side wall.

2) Expanding / warmer air will exit the cavity via the holes.

3) Thrust appears immediately the magnetron warms up. Heated air would not warm up immediately but do it slowly.

4) Thrust drops to zero immediately the power is off. With heated air, thrust drop would be slow.

The way the thrust reacted to the power switch, suggest to me it is not a heated air effect.

Looking at the solder job of the small end, I doubt it was air tight.
Title: Re: EM Drive Developments - related to space flight applications - Thread 2
Post by: deltaMass on 05/18/2015 06:20 pm
The characteristics of the heated air will depend significantly on its humidity too.
Title: Re: EM Drive Developments - related to space flight applications - Thread 2
Post by: TheTraveller on 05/18/2015 06:28 pm
The characteristics of the heated air will depend significantly on its humidity too.

Crank up the volume on the video, You can hear when the magnetron start to buzz and thrust appears immediately there after. When buzz stops, thrust stops. No delay.
Title: Re: EM Drive Developments - related to space flight applications - Thread 2
Post by: aero on 05/18/2015 06:29 pm
The expectation for measured weight change for Iulian's device is about -0.5 gm-weight (becoming lighter), assuming a temperature rise of the air within the device of about 25oC, and assuming the device volume to be about 6 litres. The mechanism here is based simply on the change in weight of the air within the device. Because the device is not sealed, air can move in and out of it as its temperature changes.  The larger the device volume, the larger the expected measured weight change.

I've now calculated the volume to be ~37 litres, and so we need roughly only 1/6th of the previously calculated temperature rise to get the observed weight change - just a few degrees will do it.

How on Earth did you come up with such a large volume? 37 litres is a cubic volume ~33.3 cm on a side. Even 6 litres is a cubic volume 18.17 cm on a side which is quite large.

Do we know the dimensions of the device?
Edit:
Don't bother, my bad. For an example, the Eagleworks Copper Kettle EM thruster dimensions    are 27.94 cm OD x 15.88 cm x 22.86cm high.
Equation for volume of a cone frustum is V =(pi*h/3)*(R^2 +Rr + r^2) which gives
 Volume = 8836.4908791301 cm^3 or 9 litres.
Edit - I changed the volume, I too had used diameters instead of radii.
Title: Re: EM Drive Developments - related to space flight applications - Thread 2
Post by: Blaine on 05/18/2015 06:31 pm
The characteristics of the heated air will depend significantly on its humidity too.

Crank up the volume on the video, You can hear when the magnetron start to buzz and thrust appears immediately there after. When buzz stops, thrust stops. No delay.

Both of you could be correct.  Although, I have to side with TheTraveller on this one.  It would have to be a pretty drastic, and immediate change in temperature which would not have likely happened.
Title: Re: EM Drive Developments - related to space flight applications - Thread 2
Post by: deltaMass on 05/18/2015 06:37 pm
The expectation for measured weight change for Iulian's device is about -0.5 gm-weight (becoming lighter), assuming a temperature rise of the air within the device of about 25oC, and assuming the device volume to be about 6 litres. The mechanism here is based simply on the change in weight of the air within the device. Because the device is not sealed, air can move in and out of it as its temperature changes.  The larger the device volume, the larger the expected measured weight change.

I've now calculated the volume to be ~37 litres, and so we need roughly only 1/6th of the previously calculated temperature rise to get the observed weight change - just a few degrees will do it.

How on Earth did you come up with such a large volume? 37 litres is a cubic volume ~33.3 cm on a side. Even 6 litres is a cubic volume 18.17 cm on a side which is quite large.

Do we know the dimensions of the device?
I miscalculated. I used diameter instead of radius. So divide that by 4 and we're back to about
9 litres . Here's the published dimensions:
http://www.masinaelectrica.com/wp-content/uploads/2015/05/EmDrive-1024x1024.jpg
Title: Re: EM Drive Developments - related to space flight applications - Thread 2
Post by: Rodal on 05/18/2015 06:48 pm
The characteristics of the heated air will depend significantly on its humidity too.
Yes, humid air will be heated practically instantaneously as microwave heating is not at all due to conduction or convection.  Microwave heating is dielectric heating, the water molecule is an electric dipole, hence the water molecules rotate to align themselves with the alternating electric field of the microwaves.  However, as pointed out by Warp-Tech, the tell tale sign is " why would it drop as soon as it's switched off" as the cooling has to proceed by convection (mostly) and conduction at the walls and that will be much slower than the experienced fast drop in measured force as soon as the electricity was turned off.  There is no dielectric cooling.
Title: Re: EM Drive Developments - related to space flight applications - Thread 2
Post by: deltaMass on 05/18/2015 06:52 pm
Not so sure about that. Air is driven out by the pressure differential. As soon as the magnetron is switched off, the driver for that pressure difference vanishes and colder air rushes back in.
Title: Re: EM Drive Developments - related to space flight applications - Thread 2
Post by: Rodal on 05/18/2015 06:54 pm
Not so sure about that. Air is driven out by the pressure differential. As soon as the magnetron is switched off, the driver for that pressure difference vanishes and colder air rushes back in.
Which has, for natural convection currents (that is what you propose) a significantly longer time constant.  (Relatively slow air speeds compared to forced convection).

Anyway, I hope he performs an experiment with the EM Drive upside down (and another one with the EM Drive sideways, pointing horizontal) and he let us know the results  :)
Title: Re: EM Drive Developments - related to space flight applications - Thread 2
Post by: Dortex on 05/18/2015 06:58 pm
The characteristics of the heated air will depend significantly on its humidity too.

Crank up the volume on the video, You can hear when the magnetron start to buzz and thrust appears immediately there after. When buzz stops, thrust stops. No delay.

I'd also like to point out that the thrust drops off the more it's run. If air were a significant contributor to the readings, you'd expect to see it go up (or rather, down) the more the device is run, no?
Title: Re: EM Drive Developments - related to space flight applications - Thread 2
Post by: TheTraveller on 05/18/2015 06:58 pm
Not so sure about that. Air is driven out by the pressure differential. As soon as the magnetron is switched off, the driver for that pressure difference vanishes and colder air rushes back in.

Sure the driver is gone immediately the power is off but not the heated air/moisture inside the cavity. That takes time to cool via conventional convection / heat xfer to the side walls. Listening to the maggie buzz on & off versus changes in the scale reading is important. Almost no apparent delay.
Title: Re: EM Drive Developments - related to space flight applications - Thread 2
Post by: TheTraveller on 05/18/2015 07:03 pm
The characteristics of the heated air will depend significantly on its humidity too.

Crank up the volume on the video, You can hear when the magnetron start to buzz and thrust appears immediately there after. When buzz stops, thrust stops. No delay.

I'd also like to point out that the thrust drops off the more it's run. If air were a significant contributor to the readings, you'd expect to see it go up (or rather, down) the more the device is run, no?

Lulian needs to do much longer runs. Those he has done so far are, IMO, way too short. Shawyer did runs of 50 to 90 seconds. But he needs things well matched, the cavity running at resonance, etc or can burn out maggies or other such issues may happen.

I really like this test setup. It tells me Shawyer did it properly.
Title: Re: EM Drive Developments - related to space flight applications - Thread 2
Post by: deltaMass on 05/18/2015 07:06 pm
The characteristics of the heated air will depend significantly on its humidity too.

Crank up the volume on the video, You can hear when the magnetron start to buzz and thrust appears immediately there after. When buzz stops, thrust stops. No delay.

I'd also like to point out that the thrust drops off the more it's run. If air were a significant contributor to the readings, you'd expect to see it go up (or rather, down) the more the device is run, no?
No, I think. Repeated runs occurring close in time to one another will tend to push up the average temperature. Therefore the change in temperature of the water vapour in the air, and of the air itself, will be that much less after several consecutive runs.
Title: Re: EM Drive Developments - related to space flight applications - Thread 2
Post by: Rodal on 05/18/2015 07:12 pm
The characteristics of the heated air will depend significantly on its humidity too.

Crank up the volume on the video, You can hear when the magnetron start to buzz and thrust appears immediately there after. When buzz stops, thrust stops. No delay.

I'd also like to point out that the thrust drops off the more it's run. If air were a significant contributor to the readings, you'd expect to see it go up (or rather, down) the more the device is run, no?

Very interesting observation  :)

This drop off can be explained by Todd's theory.  Actually, of all the theories proposed, (Shawyer's, McCulloch's, etc.) that only Todd's theory is able to explain the drop off.
Title: Re: EM Drive Developments - related to space flight applications - Thread 2
Post by: Dortex on 05/18/2015 07:14 pm
The characteristics of the heated air will depend significantly on its humidity too.

Crank up the volume on the video, You can hear when the magnetron start to buzz and thrust appears immediately there after. When buzz stops, thrust stops. No delay.

I'd also like to point out that the thrust drops off the more it's run. If air were a significant contributor to the readings, you'd expect to see it go up (or rather, down) the more the device is run, no?
No, I think. Repeated runs occurring close in time to one another will tend to push up the average temperature. Therefore the change in temperature of the water vapour in the air, and of the air itself, will be that much less after several consecutive runs.

Not sure if I'm dumb, or you're explaining it poorly. Probably the former. You mean to tell me buoyancy becomes less of an issue because the air around the chamber is getting hotter too?
Title: Re: EM Drive Developments - related to space flight applications - Thread 2
Post by: deltaMass on 05/18/2015 07:18 pm
No, it's me  :o
The air/water within the chamber will average to a higher temperature.
The ambient will stay ...ambient .

This isn't about buoyancy. It's about a simple change in the mass of air/water vapour within the cavity.
If this model is correct in accounting for the measured weight change, then we should see the same negative weight change when the cavity is flipped upside down.
Title: Re: EM Drive Developments - related to space flight applications - Thread 2
Post by: pogsquog on 05/18/2015 07:21 pm
Since the air is not significantly contained, the pressure will equalise very rapidly, i.e. air will move about rapidly, so no significant thrust from air after power off would be expected. Instead, after power off, one would expect cooling, and a sucking back of air, resulting in a smaller force of the opposite magnitude. I expect that the air being expelled during the powered phase will be proportional to the rate of change of temperature, i.e. it would fall asymtotically as the chamber heats to steady state. This can be modelled. Assuming chamber is mostly sealed except for large vent holes in side. with holes top and bottom one might get constant convection through the device, like a pulse jet.
Title: Re: EM Drive Developments - related to space flight applications - Thread 2
Post by: aero on 05/18/2015 07:25 pm
@Iulian Berca
I am happy to see your success and hope for many more. Congratulations!

But your video seems to show a lack of respect for the dangers of microwave radiation. As a leader in the DYI effort you must take it upon yourself to set an example of safe experimentation for all of those DYI'ers who will follow in your footsteps. Let me relate a story from the early days of Radar.

The story goes that a an airman, a radar tech in training learned of the risk of sterilization from the radiation. Wanting no children he thought that was a good idea. He stood next to the radar device and switched it on momentarily. Unfortunately for the tech, the surgeons had to amputate parts that he would have rather kept.

Lacking a complete Faraday cage, put a metal screen between yourself and the operating device. You can do everything of importance from behind the screen. Protect yourself because no one wants to hear that some DYI'er was injured by his experiment and when questioned, said, "Well, Iulian Berca did it that way so I thought if he could do it, so could I."
Title: Re: EM Drive Developments - related to space flight applications - Thread 2
Post by: deltaMass on 05/18/2015 07:28 pm
Since the air is not significantly contained, the pressure will equalise very rapidly, i.e. air will move about rapidly, so no significant thrust from air after power off would be expected. Instead, after power off, one would expect cooling, and a sucking back of air, resulting in a smaller force of the opposite magnitude. I expect that the air being expelled during the powered phase will be proportional to the rate of change of temperature, i.e. it would fall asymtotically as the chamber heats to steady state. This can be modelled. Assuming chamber is mostly sealed except for large vent holes in side. with holes top and bottom one might get constant convection through the device, like a pulse jet.
I think the details are interesting, but the main takeaway from the "varying air mass" model is that it is of a calculated magnitude that is able to fully account for the measured "thrust".
Title: Re: EM Drive Developments - related to space flight applications - Thread 2
Post by: SeeShells on 05/18/2015 07:28 pm
The characteristics of the heated air will depend significantly on its humidity too.

Crank up the volume on the video, You can hear when the magnetron start to buzz and thrust appears immediately there after. When buzz stops, thrust stops. No delay.

Both of you could be correct.  Although, I have to side with TheTraveller on this one.  It would have to be a pretty drastic, and immediate change in temperature which would not have likely happened.
Thinking we're seeing real thrust (not a heated air EM balloon). I watched the initial thrust degrading slowly during the test and I'm thinking this simply could be to the Microwave ionization of the air molecules inside of the Chamber causing the air to breakdown by avalanche ionization, in microwave propagation it can play a significant role. Changes in the Q, harmonics, wave patterns, due to these ionization effects could decrease the thrust like we are seeing. It takes a little time to build up an ionized plasma to the point it would start to de-tune the chamber and decay the thrust.
http://iopscience.iop.org/1009-0630/15/10/03


As soon as the power is turned off the EM scales return quickly to zero
Title: Re: EM Drive Developments - related to space flight applications - Thread 2
Post by: Dortex on 05/18/2015 07:30 pm
No, it's me  :o
The air/water within the chamber will average to a higher temperature.
The ambient will stay ...ambient .

This isn't about buoyancy. It's about a simple change in the mass of air/water vapour within the cavity.
If this model is correct in accounting for the measured weight change, then we should see the same negative weight change when the cavity is flipped upside down.

I'm no physicist, but this all sounds like a non-issue to me. He's drilled holes into the side of his device to let out warmer air and minimize their effects on the results. If, as you say, colder air rushes back in when the device is turned off, then his entire setup almost completely eliminates any issues relating to air temperature and humidity. He runs tests for short periods of time, keeping the inside as close to ambient as possible, and constantly refreshing the entire setup- eliminating whatever tiny artifact it generates in the first place.
Title: Re: EM Drive Developments - related to space flight applications - Thread 2
Post by: deltaMass on 05/18/2015 07:52 pm
When he makes a weight measurement, he is weighing the device plus whatever air/water vapour is inside it. Therefore when there is less air inside the device, the device will weigh less. It's as simple as that. Why should a higher temperature result in less air inside the device? - that's because the density of air depends on its temperature; it decreases with higher temperature. Since the device volume is constant and the density of air has dropped, there must be less air mass inside the device at higher temperature.
mass = density * volume.

And this is no "tiny artifact" - it's an effect on order negative half a gram, which turns out to be exactly what was measured.
Title: Re: EM Drive Developments - related to space flight applications - Thread 2
Post by: TheTraveller on 05/18/2015 08:06 pm
Here is the scale weight data, run by run, frame by frame as the numbers changed. Also find excel SS if you wish to play.

Sure looks like the thrust went up and down (weight went down and then up) VERY quickly as the maggie started to hum and stopped.

Can't see heated air reacting that quickly, especially at power off.

Thrust sags could be the maggie altering frequency as it got hit by the load or even dropping power. It is only a low cost microwave maggie after all.

Baseline seems to have increased but that may be hysterious in the measuring system.

I also noted a definite but small weight increase in the 1st 1/2 sec of the maggie starting to hum, then it shot straight up (weight dropping) very fast. You can see that effect in 4 of the 5 test sequences. Almost like an initial small but clearly seen, reverse thrust (downward) until the cavity settled down.
Title: Re: EM Drive Developments - related to space flight applications - Thread 2
Post by: SeeShells on 05/18/2015 08:12 pm
When he makes a weight measurement, he is weighing the device plus whatever air/water vapour is inside it. Therefore when there is less air inside the device, the device will weigh less. It's as simple as that. Why should a higher temperature result in less air inside the device? - that's because the density of air depends on its temperature; it decreases with higher temperature. Since the device volume is constant and the density of air has dropped, there must be less air mass inside the device at higher temperature.
mass = density * volume.

And this is no "tiny artifact" - it's an effect on order negative half a gram, which turns out to be exactly what was measured.
If that was the case I would like you to consider that the air takes time to evacuate the chamber and internally heat the air.  The scales would slowly increase instead the decrease like we see. This seem right?
Title: Re: EM Drive Developments - related to space flight applications - Thread 2
Post by: txdrive on 05/18/2015 08:20 pm
While buoyancy is probably not good for the experiment (keep in mind that you should want to actually measure something with some degree of accuracy here), the cables may be even worse. It's half a gram, all it takes is the cable bending a little due to  temperature difference across the materials and magnetics, so that half a gram more or less of the cable weight is supported by the scale. The bad thing about cables is that there's essentially no way to estimate anything, or at least it is far harder than to weight a self contained system, anyway.

Other issue is how the scale handles vibrations - it may or may not correctly average vibrations out.

It's not so much an issue of one force that is messing up the readings as an issue of a sum of many classical forces. One could get some very rough idea regarding the magnitude of that sum by turning the cavity 90 and 180 degrees, as some of the classical forces won't flip around.
Title: Re: EM Drive Developments - related to space flight applications - Thread 2
Post by: deltaMass on 05/18/2015 08:22 pm
When he makes a weight measurement, he is weighing the device plus whatever air/water vapour is inside it. Therefore when there is less air inside the device, the device will weigh less. It's as simple as that. Why should a higher temperature result in less air inside the device? - that's because the density of air depends on its temperature; it decreases with higher temperature. Since the device volume is constant and the density of air has dropped, there must be less air mass inside the device at higher temperature.
mass = density * volume.

And this is no "tiny artifact" - it's an effect on order negative half a gram, which turns out to be exactly what was measured.
If that was the case I would like you to consider that the air takes time to evacuate the chamber and internally heat the air.  The scales would slowly increase instead the decrease like we see. This seem right?
No, it doesn't seem right because I can't understand most of what you're saying.
Title: Re: EM Drive Developments - related to space flight applications - Thread 2
Post by: deltaMass on 05/18/2015 08:24 pm
@TheTraveller:
What are the units and semantics of your vertical scale?
Title: Re: EM Drive Developments - related to space flight applications - Thread 2
Post by: TheTraveller on 05/18/2015 08:26 pm
@TheTraveller:
What are the units and semantics of your vertical scale?

Units are what the scale displayed at every change of value. Time between measurement sort of constant (due to scale update frequency) but not time logged.

You can easily put the video in 1/4 time and freeze frame it after each number change of the scale to verify the data.
Title: Re: EM Drive Developments - related to space flight applications - Thread 2
Post by: txdrive on 05/18/2015 08:27 pm
So your thrust is in the opposite direction from EagleWorks, right?
Title: Re: EM Drive Developments - related to space flight applications - Thread 2
Post by: TheTraveller on 05/18/2015 08:30 pm
So your thrust is in opposite direction from EagleWorks, right?

Movement is as per Shawyer. From the big end toward the small end.
Watch the video to see which way it moves.
http://emdrive.com/dynamictests.html

Shawyer also pointed it out in the attachment that Reaction (physical movement of the EM Drive) is opposite to Thrust direction.
Title: Re: EM Drive Developments - related to space flight applications - Thread 2
Post by: txdrive on 05/18/2015 08:41 pm
So your thrust is in opposite direction from EagleWorks, right?

Movement is as per Shawyer. From the big end toward the small end.
Watch the video to see which way it moves.
http://emdrive.com/dynamictests.html
As per Shawyer's theory papers, it should be moving wide end forward.
Title: Re: EM Drive Developments - related to space flight applications - Thread 2
Post by: TheTraveller on 05/18/2015 08:43 pm
So your thrust is in opposite direction from EagleWorks, right?

Movement is as per Shawyer. From the big end toward the small end.
Watch the video to see which way it moves.
http://emdrive.com/dynamictests.html
As per Shawyer's theory papers, it should be moving wide end forward.

Reaction / EM Drive physical movement is in the opposite direction to Thrust direction. Should move toward small end. He has stated this many times.
Title: Re: EM Drive Developments - related to space flight applications - Thread 2
Post by: txdrive on 05/18/2015 08:45 pm
So your thrust is in opposite direction from EagleWorks, right?

Movement is as per Shawyer. From the big end toward the small end.
Watch the video to see which way it moves.
http://emdrive.com/dynamictests.html
As per Shawyer's theory papers, it should be moving wide end forward.

Reaction / EM Drive physical movement is in the opposite direction to Thrust direction. He has stated this many times.

http://www.emdrive.com/theorypaper9-4.pdf
Quote
The group velocity of the electromagnetic wave at the end plate of the larger
section is higher than the group velocity at the end plate of the smaller section. Thus
the radiation pressure at the larger end plate is higher that that at the smaller end plate.
The resulting force difference (F g1 -F g2 ) is multiplied by the Q of the resonant
assembly.

So, it would have to be mounted the larger plate forward if you want your ship to go forward. According to his "theory" anyway. No idea where his experiments are going, my guess is which ever ways vibration and shifts in the centre of mass take them.
Title: Re: EM Drive Developments - related to space flight applications - Thread 2
Post by: TheTraveller on 05/18/2015 08:52 pm
So your thrust is in opposite direction from EagleWorks, right?

Movement is as per Shawyer. From the big end toward the small end.
Watch the video to see which way it moves.
http://emdrive.com/dynamictests.html
As per Shawyer's theory papers, it should be moving wide end forward.

Reaction / EM Drive physical movement is in the opposite direction to Thrust direction. He has stated this many times.

http://www.emdrive.com/theorypaper9-4.pdf
Quote
The group velocity of the electromagnetic wave at the end plate of the larger
section is higher than the group velocity at the end plate of the smaller section. Thus
the radiation pressure at the larger end plate is higher that that at the smaller end plate.
The resulting force difference (F g1 -F g2 ) is multiplied by the Q of the resonant
assembly.

So, it would have to be mounted the larger plate forward if you want your ship to go forward.

Spacecraft move in the opposite direction to the thrust of the engine. EM Drive is no different.
Title: Re: EM Drive Developments - related to space flight applications - Thread 2
Post by: TheTraveller on 05/18/2015 08:57 pm
So your thrust is in opposite direction from EagleWorks, right?

Movement is as per Shawyer. From the big end toward the small end.
Watch the video to see which way it moves.
http://emdrive.com/dynamictests.html
As per Shawyer's theory papers, it should be moving wide end forward.

Reaction / EM Drive physical movement is in the opposite direction to Thrust direction. He has stated this many times.

http://www.emdrive.com/theorypaper9-4.pdf
Quote
The group velocity of the electromagnetic wave at the end plate of the larger
section is higher than the group velocity at the end plate of the smaller section. Thus
the radiation pressure at the larger end plate is higher that that at the smaller end plate.
The resulting force difference (F g1 -F g2 ) is multiplied by the Q of the resonant
assembly.

So, it would have to be mounted the larger plate forward if you want your ship to go forward. According to his "theory" anyway. No idea where his experiments are going, my guess is which ever ways vibration and shifts in the centre of mass take them.

Every rocket engine produces thrust in one direction and the craft moves in the opposite direction as a reaction to the thrust from the engine. What is different in the EM Drive is there is no need to expel hot gasses at high velocity to produce thrust.

With an EM Drive you point the small end of the frustum at where you want to go and apply power to the microwave generators. Thrust is generated toward the big end of the frustum and the craft accelerates in the opposite direction from the generated thrust.
Title: Re: EM Drive Developments - related to space flight applications - Thread 2
Post by: Blaine on 05/18/2015 09:16 pm
Iluian really needs to turn his device upside down.  People on the internet are getting antsy and so am I.
Title: Re: EM Drive Developments - related to space flight applications - Thread 2
Post by: txdrive on 05/18/2015 09:17 pm
So your thrust is in opposite direction from EagleWorks, right?

Movement is as per Shawyer. From the big end toward the small end.
Watch the video to see which way it moves.
http://emdrive.com/dynamictests.html
As per Shawyer's theory papers, it should be moving wide end forward.

Reaction / EM Drive physical movement is in the opposite direction to Thrust direction. He has stated this many times.

http://www.emdrive.com/theorypaper9-4.pdf
Quote
The group velocity of the electromagnetic wave at the end plate of the larger
section is higher than the group velocity at the end plate of the smaller section. Thus
the radiation pressure at the larger end plate is higher that that at the smaller end plate.
The resulting force difference (F g1 -F g2 ) is multiplied by the Q of the resonant
assembly.

So, it would have to be mounted the larger plate forward if you want your ship to go forward.

Spacecraft move in the opposite direction to the thrust of the engine. EM Drive is no different.
http://exploration.grc.nasa.gov/education/rocket/rktth1.html
Title: Re: EM Drive Developments - related to space flight applications - Thread 2
Post by: TheTraveller on 05/18/2015 09:18 pm
When he makes a weight measurement, he is weighing the device plus whatever air/water vapour is inside it. Therefore when there is less air inside the device, the device will weigh less. It's as simple as that. Why should a higher temperature result in less air inside the device? - that's because the density of air depends on its temperature; it decreases with higher temperature. Since the device volume is constant and the density of air has dropped, there must be less air mass inside the device at higher temperature.
mass = density * volume.

And this is no "tiny artifact" - it's an effect on order negative half a gram, which turns out to be exactly what was measured.

He is weighing the air/water vapour inside the cavity?

How is that happening?

Those gasses / molecules are not liquid / frozen solid on the bottom of the frustum. They are bouncing around, exerting the same pressure on all sides. If you heat the water vapour, it will just bounce around faster, transferring heat to the walls and end plates. Additionally there are 2 large holes in the side of the frustum walls. Any increase of pressure inside the frustum will equalize to outside pressure very quickly, like at the speed of the vibrating gas atoms / molecules. 

I will grant you that gasses trapped above the central side vent holes will reduce in pressure and will provide lift. But not gasses at or below to the hole closest to the small end.

So maybe calc the volume of air that would be trapped between the small end plate and the hole nearest the small end plate. Suggest it is a lot less air mass than you are using at present.

Did you actually look at the rate of change when the maggie drives microwaves into the cavity and not? It happens in one or 2 updates of the scale. VERY fast going UP and DOWN.

Heating the frustum will not increase nor decrease the weight measured by the scale due to the setup. Likewise vertical CG movement will not effect measured weight.
Title: Re: EM Drive Developments - related to space flight applications - Thread 2
Post by: TheTraveller on 05/18/2015 09:25 pm
So your thrust is in opposite direction from EagleWorks, right?

Movement is as per Shawyer. From the big end toward the small end.
Watch the video to see which way it moves.
http://emdrive.com/dynamictests.html
As per Shawyer's theory papers, it should be moving wide end forward.

Reaction / EM Drive physical movement is in the opposite direction to Thrust direction. He has stated this many times.

http://www.emdrive.com/theorypaper9-4.pdf
Quote
The group velocity of the electromagnetic wave at the end plate of the larger
section is higher than the group velocity at the end plate of the smaller section. Thus
the radiation pressure at the larger end plate is higher that that at the smaller end plate.
The resulting force difference (F g1 -F g2 ) is multiplied by the Q of the resonant
assembly.

So, it would have to be mounted the larger plate forward if you want your ship to go forward.

Spacecraft move in the opposite direction to the thrust of the engine. EM Drive is no different.
http://exploration.grc.nasa.gov/education/rocket/rktth1.html

Iulian EM Drive moves in the same direction as Shawyers Demo rotary table. Toward the small end.

Please read:
http://www.emdrive.com/EmDriveForceMeasurement.pdf

Shawyer shows in this diagram the direction of the Reaction, which moves the EM Drive.
Title: Re: EM Drive Developments - related to space flight applications - Thread 2
Post by: demofsky on 05/18/2015 09:28 pm
So should everyone be using perforated copper sheeting??  Admittedly, I can't even begin to imagine how this would affect the performance of the drive.  Do the various theories need a solid wall or as long as the perforations are sufficiently fine it should not matter??   :P

https://concordsheetmetal.com/store/perforated-copper/ (https://concordsheetmetal.com/store/perforated-copper/)
Title: Re: EM Drive Developments - related to space flight applications - Thread 2
Post by: TheTraveller on 05/18/2015 09:32 pm
So should everyone be using perforated copper sheeting??  Admittedly, I can't even begin to imagine how this would affect the performance of the drive.  Do the various theories need a solid wall or as long as the perforations are sufficiently fine it should not matter??   :P

https://concordsheetmetal.com/store/perforated-copper/ (https://concordsheetmetal.com/store/perforated-copper/)

Might be hard to roll into a smooth frustum.

I plan to drill 6 x 1mm diameter equally spaced holes 5mm in from each end of the frustum and the same around the central diameter to allow heated air to escape without causing any major issues.
Title: Re: EM Drive Developments - related to space flight applications - Thread 2
Post by: Rodal on 05/18/2015 09:36 pm
...

Spacecraft move in the opposite direction to the thrust of the engine. EM Drive is no different.

Propellant comes out of a rocket engine.  According to Shawyer nothing comes out of an EM Drive.

If nothing is coming out of the EM Drive, how can it have a thrust force in the opposite direction to its acceleration?

Has someone measured that thust force you are referring to ? If this thrust force of the EM Drive has been measured, how was the thrust force measured simultaneously with a measurement of the acceleration in the opposite direction  ?
 ???
Title: Re: EM Drive Developments - related to space flight applications - Thread 2
Post by: PaulF on 05/18/2015 09:38 pm
A couple of questions, out of curiosity:

1. Have any of the copper frustums been polished to a near perfect mirror on the inside? If not, why not? I can imagine that could have a positive effect on Q.

2. Has anyone picked up on the notion that the most efficient frustum may be a wide, short one? In other words, building one according to that model?

Thanks in advance. Paul F
Title: Re: EM Drive Developments - related to space flight applications - Thread 2
Post by: zellerium on 05/18/2015 09:38 pm
So should everyone be using perforated copper sheeting??  Admittedly, I can't even begin to imagine how this would affect the performance of the drive.  Do the various theories need a solid wall or as long as the perforations are sufficiently fine it should not matter??   :P

https://concordsheetmetal.com/store/perforated-copper/ (https://concordsheetmetal.com/store/perforated-copper/)

Might be hard to roll into a smooth frustum.

I plan to drill 6 x 1mm diameter holes 5mm in from each end of the frustum and the same around the central diameter to allow heated air to escape without causing any major issues.

It might be more difficult to roll a smooth frustum, but it would also cut down the cost.
Wouldn't thermal expansion be less of an issue with a perforated structure? It would allow much more convection cooling.
Title: Re: EM Drive Developments - related to space flight applications - Thread 2
Post by: Rodal on 05/18/2015 09:39 pm
So should everyone be using perforated copper sheeting??  Admittedly, I can't even begin to imagine how this would affect the performance of the drive.  Do the various theories need a solid wall or as long as the perforations are sufficiently fine it should not matter??   :P

https://concordsheetmetal.com/store/perforated-copper/ (https://concordsheetmetal.com/store/perforated-copper/)

It should not matter to a microwave oven.  Millions of microwave ovens with glass transparent to microwaves are covered with this perforated metal to prevent the microwaves from escaping the microwave open and hurting humans.

At least the small and big flat ends could be made of perforated metal.

The opening in the mesh is equivalent to a very small wavelength.  I haven't seen a paper supporting that such a small wavelength is responsible for the EM Drive thrust.

(http://makeitorfixit.com/wp-content/uploads/2014/01/MicrowaveDoor.jpg.jpg)
Title: Re: EM Drive Developments - related to space flight applications - Thread 2
Post by: TheTraveller on 05/18/2015 09:40 pm
...

Spacecraft move in the opposite direction to the thrust of the engine. EM Drive is no different.

Propellant comes out of a rocket engine.  According to Shawyer nothing comes out of an EM Drive.

If nothing is coming out of the EM Drive, how can it have a thrust force in the opposite direction to its acceleration?

Has someone measured that thust force you are referring to ? If this thrust force of the EM Drive has been measured, how was the thrust force measured simultaneously with a measurement of the acceleration in the opposite direction  ?
 ???

As you well know it was measured and reported here:

4th line. Measured on the rotary test rig as the Demonstrator EM Drive moved.
Title: Re: EM Drive Developments - related to space flight applications - Thread 2
Post by: demofsky on 05/18/2015 09:42 pm
So should everyone be using perforated copper sheeting??  Admittedly, I can't even begin to imagine how this would affect the performance of the drive.  Do the various theories need a solid wall or as long as the perforations are sufficiently fine it should not matter??   :P

https://concordsheetmetal.com/store/perforated-copper/ (https://concordsheetmetal.com/store/perforated-copper/)

Might be hard to roll into a smooth frustum.

I plan to drill 6 x 1mm diameter equally spaced holes 5mm in from each end of the frustum and the same around the central diameter to allow heated air to escape without causing any major issues.

Hm.  I can see folks being more comfortable with holes around the central diameter.  Holes at either end could result in debates over whether this was causing thrust towards one end or the other, biasing the results...
Title: Re: EM Drive Developments - related to space flight applications - Thread 2
Post by: Rodal on 05/18/2015 09:43 pm
...

Spacecraft move in the opposite direction to the thrust of the engine. EM Drive is no different.

Propellant comes out of a rocket engine.  According to Shawyer nothing comes out of an EM Drive.

If nothing is coming out of the EM Drive, how can it have a thrust force in the opposite direction to its acceleration?

Has someone measured that thust force you are referring to ? If this thrust force of the EM Drive has been measured, how was the thrust force measured simultaneously with a measurement of the acceleration in the opposite direction  ?
 ???

As you well know it was measured and reported here:
No, I don't know that.

I see a chart that Shawyer put together.  Nobody here could explain why the Demo Engine has it in both directions.

I never saw a paper detailing how such a thrust was measured simultaneously with measuring the acceleration in the opposite direction.
Title: Re: EM Drive Developments - related to space flight applications - Thread 2
Post by: TheTraveller on 05/18/2015 09:47 pm
So should everyone be using perforated copper sheeting??  Admittedly, I can't even begin to imagine how this would affect the performance of the drive.  Do the various theories need a solid wall or as long as the perforations are sufficiently fine it should not matter??   :P

https://concordsheetmetal.com/store/perforated-copper/ (https://concordsheetmetal.com/store/perforated-copper/)

Might be hard to roll into a smooth frustum.

I plan to drill 6 x 1mm diameter equally spaced holes 5mm in from each end of the frustum and the same around the central diameter to allow heated air to escape without causing any major issues.

Hm.  I can see folks being more comfortable with holes around the central diameter.  Holes at either end could result in debates over whether this was causing thrust towards one end or the other, biasing the results...

Holes would be in the side walls, not in the end plates. Hard to see how 6 x 1mm diameter holes even spaced around the ends of the frustum could generate significant thrust.

Easy to block and unblock during different test runs.
Title: Re: EM Drive Developments - related to space flight applications - Thread 2
Post by: demofsky on 05/18/2015 09:54 pm
So should everyone be using perforated copper sheeting??  Admittedly, I can't even begin to imagine how this would affect the performance of the drive.  Do the various theories need a solid wall or as long as the perforations are sufficiently fine it should not matter??   :P

https://concordsheetmetal.com/store/perforated-copper/ (https://concordsheetmetal.com/store/perforated-copper/)

It should not matter.  Millions of microwave ovens with glass transparent to microwaves are covered with this perforated metal to prevent the microwaves from escaping the microwave open and hurting humans.

At least the small and big flat ends could be made of perforated metal.

The opening in the mesh is equivalent to a very small wavelength.

(http://makeitorfixit.com/wp-content/uploads/2014/01/MicrowaveDoor.jpg.jpg)

That is my understanding as well.  But what about surface effects that may be happening?  Ultimately there is some type of coupling happening with the interior fulstrum surface or folks really are creating little warp drives. :)

Either this is purely a type of microwave phenomena as Sawyer, et al believe, in which case it really is all about Q or there is some other (secondary) effect that is causing thrust...
Title: Re: EM Drive Developments - related to space flight applications - Thread 2
Post by: tchernik on 05/18/2015 09:54 pm
...

Spacecraft move in the opposite direction to the thrust of the engine. EM Drive is no different.

Propellant comes out of a rocket engine.  According to Shawyer nothing comes out of an EM Drive.

If nothing is coming out of the EM Drive, how can it have a thrust force in the opposite direction to its acceleration?

Has someone measured that thust force you are referring to ? If this thrust force of the EM Drive has been measured, how was the thrust force measured simultaneously with a measurement of the acceleration in the opposite direction  ?
 ???

Yep. Force of what kind? can it be measured? felt? what medium carries it?

If we have no proper answer to this, just drop it.

Let us just content ourselves with proving if the device actually accelerates along a vector going from the big to the small end's direction when powered on, or not.

This point of the pseudo-force/reaction has really baffled me since I started reading about this, and it is very likely due to Roger Shawyer's own flaky explanation of his potential invention/discovery.
Title: Re: EM Drive Developments - related to space flight applications - Thread 2
Post by: demofsky on 05/18/2015 09:57 pm
So should everyone be using perforated copper sheeting??  Admittedly, I can't even begin to imagine how this would affect the performance of the drive.  Do the various theories need a solid wall or as long as the perforations are sufficiently fine it should not matter??   :P

https://concordsheetmetal.com/store/perforated-copper/ (https://concordsheetmetal.com/store/perforated-copper/)

Might be hard to roll into a smooth frustum.

I plan to drill 6 x 1mm diameter equally spaced holes 5mm in from each end of the frustum and the same around the central diameter to allow heated air to escape without causing any major issues.

Hm.  I can see folks being more comfortable with holes around the central diameter.  Holes at either end could result in debates over whether this was causing thrust towards one end or the other, biasing the results...

Holes would be in the side walls, not in the end plates. Hard to see how 6 x 1mm diameter holes even spaced around the ends of the frustum could generate significant thrust.

Easy to block and unblock during different test runs.

Yes, blocking and unblocking the holes should be sufficient. 
Title: Re: EM Drive Developments - related to space flight applications - Thread 2
Post by: TheTraveller on 05/18/2015 09:58 pm
...

Spacecraft move in the opposite direction to the thrust of the engine. EM Drive is no different.

Propellant comes out of a rocket engine.  According to Shawyer nothing comes out of an EM Drive.

If nothing is coming out of the EM Drive, how can it have a thrust force in the opposite direction to its acceleration?

Has someone measured that thust force you are referring to ? If this thrust force of the EM Drive has been measured, how was the thrust force measured simultaneously with a measurement of the acceleration in the opposite direction  ?
 ???

As you well know it was measured and reported here:
No, I don't know that.

I see a chart that Shawyer put together.  Nobody here could explain why the Demo Engine has it in both directions.

I never saw a paper detailing how such a thrust was measured simultaneously with measuring the acceleration in the opposite direction.

Shawyer is in business to sell licenses, not to give it all away so any DIY guy can replicate and go into business against SPR and it's clients.

There are holes in what he has put in the public domain. I doubt he has any intention of filling them in. For guys like me, working to replicate the EM Drive, we need to learn to follow the bread crumb trail he has left. For me, that trail is strong and delivering good intel.

I'm highly confident of replicating his Flight Thruster and getting close to his results.
Title: Re: EM Drive Developments - related to space flight applications - Thread 2
Post by: Rodal on 05/18/2015 10:19 pm
...

Shawyer is in business to sell licenses, not to give it all away so any DIY guy can replicate and go into business against SPR and it's clients.

There are holes in what he has put in the public domain. I doubt he has any intention of filling them in. For guys like me, working to replicate the EM Drive, we need to learn to follow the bread crumb trail he has left. For me, that trail is strong and delivering good intel.

I'm highly confident of replicating his Flight Thruster and getting close to his results.

Shawyer has patents granted on the  EM Drive.  One of the fundamental agreements and understanding under which patents are conferred by a state (or the European Union) to an inventor is that the inventor must disclose all the  "information material to patentability." See: Duty to disclose information material to patentability.

In intellectual property one always has to make a choice: trade secret (like the Coca Cola formula) or patent.

Under trade secret you run the risk that the secret may become public (by independent discovery), but as long as it is a trade secret it has no expiration.

A patent gives you a state-conferred monopoly, but it has an expiration date.  The state gets to make the patent information public in exchange for conferring the patent's monopoly to the inventor.

I'm sure that Shawyer would disagree with your statement if you are referring to information material to patentability protected by Shawyer's patents. 

On the other hand, if you are not referring to information material to patentability, then the information you are referring to "so any DIY guy can replicate and go into business" is not protected by Shawyer's patents.
Title: Re: EM Drive Developments - related to space flight applications - Thread 2
Post by: rfmwguy on 05/18/2015 10:20 pm
So should everyone be using perforated copper sheeting??  Admittedly, I can't even begin to imagine how this would affect the performance of the drive.  Do the various theories need a solid wall or as long as the perforations are sufficiently fine it should not matter??   :P

https://concordsheetmetal.com/store/perforated-copper/ (https://concordsheetmetal.com/store/perforated-copper/)

Good thoughts as the mesh would be a cost-effective solution for trials, however, its likely going to be a lower Q which theoretically inhibit power/effeciency. My old company used this exact material in an impedance matching network, while a competitor used solid aluminum. At the end of the day, the 50 ohm freq matching was extended abt 25% by using the more expensive solid cone. I'd suggest initial proof of performance testing on the mesh, moving up to silver-plated flash over copper.

Another topic I have not see discussed is intermodulation products caused by dissimilar metals, perhaps nickel( Magnetic)-plating of connectors and such. I'm not sure if IM products would adversly affect the trials, as the magnetron is inherently spraying out bits all over the spectrum.
Title: Re: EM Drive Developments - related to space flight applications - Thread 2
Post by: Rodal on 05/18/2015 10:28 pm
So should everyone be using perforated copper sheeting??  Admittedly, I can't even begin to imagine how this would affect the performance of the drive.  Do the various theories need a solid wall or as long as the perforations are sufficiently fine it should not matter??   :P

https://concordsheetmetal.com/store/perforated-copper/ (https://concordsheetmetal.com/store/perforated-copper/)

Good thoughts as the mesh would be a cost-effective solution for trials, however, its likely going to be a lower Q which theoretically inhibit power/effeciency. My old company used this exact material in an impedance matching network, while a competitor used solid aluminum. At the end of the day, the 50 ohm freq matching was extended abt 25% by using the more expensive solid cone. I'd suggest initial proof of performance testing on the mesh, moving up to silver-plated flash over copper.

Another topic I have not see discussed is intermodulation products caused by dissimilar metals, perhaps nickel( Magnetic)-plating of connectors and such. I'm not sure if IM products would adversly affect the trials, as the magnetron is inherently spraying out bits all over the spectrum.

OK all good points.  But it is not at all clear that one wants a high Q.

The highest thrust was achieved by Prof. Yang with an effective Q of only ~1500 (take into account that the reported Q's appear much larger because of the unorthodox way the Chinese report Q, if one uses the same method as in the West, Yang's Q was relatively low).

Todd's theory shows that one doesn't want a huge Q.

So using a mesh would be of research interest, to see what difference it makes., whether it makes no difference, or is worse or better
Title: Re: EM Drive Developments - related to space flight applications - Thread 2
Post by: deltaMass on 05/18/2015 10:56 pm
When Iulian flips it and if he finds the same -0.5 gm ballpark, I am confident that he will report it. Unlike some people, he seems to have no trouble in reporting null results. The mark of the true scientist.
Title: Re: EM Drive Developments - related to space flight applications - Thread 2
Post by: pogsquog on 05/18/2015 11:02 pm
Back of the envelope calculation for the 'hot air balloon' explanation, considering the pessimistic possibility that we just have a really heavy, electric powered Chinese sky lantern:

If we assume (worst case!!)
-a perfectly ventilated cavity
-that the air is heated primarily by the microwave, with a negligible impact from conduction/convection from the frustum to the air

then using convective airflow calculator:

temperature difference between inside and outside of cavity = 54 K
temperature of ambient = 273 + 20 K
cone surface area = 0.163 m^2
cone volume = 0.0063 m^3
mass of air displaced by heating air in cavity to +54 K ~= 1 gram
air flow rate (volume) = 0.0097 m^3 / s

i.e. it would take ~0.6 seconds for the hot air to leave the chamber. There will be a residual effect due to the heat of the cavity itself, with a much longer time period (cooling time of the metal, probably tens of seconds).

So, I guess this shows that it is not impossible to see a rapid time constant for thermal lift, if your frustum behaves quite like a sky lantern.

On the other hand, if we assume that the thrust is coming from a single hole, and is caused purely by thermal expansion of the air in the frustum, then we would expect to see a force like:

assume rate of change of air temperature is ~ 10 K per second, at first. Then thermal expansion will push ~ 3% of the air out of the cavity per second. If this is pushed through a hole of 5 mm, then the resulting force will be equivalent to something like 0.6 grams (falling over time as the system moves towards equilibrium), which is also in the ballpark of the measured force.
Title: Re: EM Drive Developments - related to space flight applications - Thread 2
Post by: WarpTech on 05/18/2015 11:18 pm
...

Spacecraft move in the opposite direction to the thrust of the engine. EM Drive is no different.

Propellant comes out of a rocket engine.  According to Shawyer nothing comes out of an EM Drive.

If nothing is coming out of the EM Drive, how can it have a thrust force in the opposite direction to its acceleration?

Has someone measured that thust force you are referring to ? If this thrust force of the EM Drive has been measured, how was the thrust force measured simultaneously with a measurement of the acceleration in the opposite direction  ?
 ???

As you well know it was measured and reported here:
No, I don't know that.

I see a chart that Shawyer put together.  Nobody here could explain why the Demo Engine has it in both directions.

I never saw a paper detailing how such a thrust was measured simultaneously with measuring the acceleration in the opposite direction.

Shawyer is in business to sell licenses, not to give it all away so any DIY guy can replicate and go into business against SPR and it's clients.

There are holes in what he has put in the public domain. I doubt he has any intention of filling them in. For guys like me, working to replicate the EM Drive, we need to learn to follow the bread crumb trail he has left. For me, that trail is strong and delivering good intel.

I'm highly confident of replicating his Flight Thruster and getting close to his results.

Shawyer has patents on the  EM Drive.  One of the fundamental agreements and understanding under which patents are conferred by a state (or the European Union) to an inventor is that the inventor must disclose all the  "information material to patentability." See: Duty to disclose information material to patentability.

In intellectual property one always has to make a choice: trade secret (like the Coca Cola formula) or patent.

Under trade secret you run the risk that the secret may become public (by independent discovery), but as long as it is a trade secret it has no expiration.

A patent gives you a state-conferred monopoly, but it has an expiration date.  The state gets to make the patent information public in exchange for conferring the patent's monopoly to the inventor.

I'm sure that Shawyer would disagree with your statement if you are referring to information material to patentability protected by Shawyer's patents. 

On the other hand, if you are not referring to information material to patentability, then the information you are referring to "so any DIY guy can replicate and go into business" is not protected by Shawyer's patents.

Has anyone tried to pull a copy of the patent from the patent office?

Title: Re: EM Drive Developments - related to space flight applications - Thread 2
Post by: Rodal on 05/18/2015 11:25 pm
[...
Has anyone tried to pull a copy of the patent from the patent office?
These are all the patent documents I know of, that have been posted in this thread.  All of them are UK patent applications.

EDIT: see http://forum.nasaspaceflight.com/index.php?topic=36313.msg1376186#msg1376186 for status of Shawyer's IP

Under justia, this is all that shows up (not an EM Drive patent):

http://patents.justia.com/inventor/roger-j-shawyer

Same under Google patents

and I couldn't find his EM Drive patents under USPTO search under inventor either
Title: Re: EM Drive Developments - related to space flight applications - Thread 2
Post by: SeeShells on 05/18/2015 11:45 pm
...

Spacecraft move in the opposite direction to the thrust of the engine. EM Drive is no different.

Propellant comes out of a rocket engine.  According to Shawyer nothing comes out of an EM Drive.

If nothing is coming out of the EM Drive, how can it have a thrust force in the opposite direction to its acceleration?

Has someone measured that thust force you are referring to ? If this thrust force of the EM Drive has been measured, how was the thrust force measured simultaneously with a measurement of the acceleration in the opposite direction  ?
 ???

As you well know it was measured and reported here:
No, I don't know that.

I see a chart that Shawyer put together.  Nobody here could explain why the Demo Engine has it in both directions.

I never saw a paper detailing how such a thrust was measured simultaneously with measuring the acceleration in the opposite direction.
I saw the chart and thought the same thing, it is a conundrum leaving me to just scratch my head perplexed.
Title: Re: EM Drive Developments - related to space flight applications - Thread 2
Post by: pogsquog on 05/18/2015 11:55 pm
Back of the envelope calculation for the 'hot air balloon' explanation, considering the pessimistic possibility that we just have a really heavy, electric powered Chinese sky lantern:

...

Thanks for running the numbers  :)

What is the basis for the air flow rate (volume) = 0.0097 m^3 / s ?  is it an assumption or is it an outcome of equations? if an outcome of equations, what where the equations  or theory used in the calculator?

I just plugged the numbers into a convection calculator:

http://www.engineeringtoolbox.com/convective-air-flow-d_1006.html

Clearly, this is an approximation, but I think it is sufficient for the purpose of showing that one cannot completely rule out a thermal (hot air balloon) effect for this particular result, as a rapid replacement of the air in the chamber is theoretically possible (assuming it is well ventilated), so it is possible to see a rapid loss of up-thrust after power-off.

Finding similar thrust in other orientations would rule this out, although the possibility of temporary thrust due to thermal expansion causing directed air flow, and sustained air flow due to jet effects would still exist; directed air flows of just a few percent of the frustum's volume per second are sufficient to cause this effect, which is easily possible when you are pumping a kilowatt of power into a small space.
Title: Re: EM Drive Developments - related to space flight applications - Thread 2
Post by: AnalogMan on 05/19/2015 12:03 am
[...
Has anyone tried to pull a copy of the patent from the patent office?
These are all the patent documents I know of, that have been posted in this thread.  All of them are UK patent applications.

I don't know whether any patents were actually conferred for the EM Drive.

Under justia, this is all that shows up (not an EM Drive patent):

http://patents.justia.com/inventor/roger-j-shawyer (http://patents.justia.com/inventor/roger-j-shawyer)

Same under Google patents

and I couldn't find his EM Drive patents under USPTO search under inventor either

Did you find any?

I edited my post to read:  patent (applications ?).  Thanks for questioning, as I didn't recall that all (apparently ?)  that has been posted here are just applications in the UK.

For the publication numbers listed in the above post:

GB2493361 High q microwave radiation thruster
Filed 6Feb11 - Awaiting first examination

GB2399601 High thrust microwave engine
Filed 13Mar03 - Granted 31Jan06 - last renewed 6Mar15

GB2334761 Microwave thruster for spacecraft
Filed 29Ap88 - Granted 21Mar00 - last renewed 1Apr15

GB2229865 Electrical propulsion unit for spacecraft
Filed 1Nov88 - Granted 5May93 - now ceased (not in force from 1Nov97)

https://www.gov.uk/search-for-patent (https://www.gov.uk/search-for-patent)
Title: Re: EM Drive Developments - related to space flight applications - Thread 2
Post by: tchernik on 05/19/2015 12:13 am
Back of the envelope calculation for the 'hot air balloon' explanation, considering the pessimistic possibility that we just have a really heavy, electric powered Chinese sky lantern:

...

Thanks for running the numbers  :)

What is the basis for the air flow rate (volume) = 0.0097 m^3 / s ?  is it an assumption or is it an outcome of equations? if an outcome of equations, what where the equations  or theory used in the calculator?

I just plugged the numbers into a convection calculator:

http://www.engineeringtoolbox.com/convective-air-flow-d_1006.html

Clearly, this is an approximation, but I think it is sufficient for the purpose of showing that one cannot completely rule out a thermal (hot air balloon) effect for this particular result, as a rapid replacement of the air in the chamber is theoretically possible (assuming it is well ventilated), so it is possible to see a rapid loss of up-thrust after power-off.

Finding similar thrust in other orientations would rule this out, although the possibility of temporary thrust due to thermal expansion causing directed air flow, and sustained air flow due to jet effects would still exist; directed air flows of just a few percent of the frustum's volume per second are sufficient to cause this effect, which is easily possible when you are pumping a kilowatt of power into a small space.

I fully support and thank people looking for alternative, more mundane explanations for this. Because all these objections will appear again and again, so better address them now.

The way out of this, as I see it, is for someone with an Emdrive (in this case, Iulian) to:

- Run the thrust tests with the device in all directions, and in this case upside-down, to see if the acceleration vector continues going to the same direction as before (from the big end towards the small end). This would rule out hot air buoyancy as an explanation for the thrust.

- Visibly inspect and/or check with a smoke test if there are holes or obvious exhaust plumes coming out from the device's big end, to rule out any unintended thermal "rocket" effect.

Or, alternatively, build and test a copper wire-mesh Emdrive, which would immediately keep all the frustum internal air at the same pressure as the outside (so no thermal expansion effects).

But I'm afraid we would be over-taxing the good will of Iulian with that last experiment. But such experiment would be very valuable indeed, without a doubt.
Title: Re: EM Drive Developments - related to space flight applications - Thread 2
Post by: Carl G on 05/19/2015 01:07 am
Please remember this is a long thread and this site's forum has a high signal to noise factor.

If you like a post, "like" it. Please only post if you have something useful to add to the thread, otherwise it will be deleted.

The reason is 99 percent of people (nearly a million reads) are watching this thread and those catching up don't need to have to search through posts that don't add anything. Thanks.
Title: Re: EM Drive Developments - related to space flight applications - Thread 2
Post by: TheTraveller on 05/19/2015 01:24 am
From the pdf's
Shawyer uses/used an internal "dielectric"

Shawyer drops a few hints:

1) The Flight Thruster and the Chinese units did not use dielectrics.

2) Dielectrics introduce additional losses and reduce Q.

3) The Flight Thruster used shaped end plates with a narrow band Rf source.

4) Flat end plates can be used with wideband magnetrons.
Title: Re: EM Drive Developments - related to space flight applications - Thread 2
Post by: Rodal on 05/19/2015 01:37 am
Please remember this is a long thread and this site's forum has a high signal to noise factor.
...
It would also be helpful if people wouldn't reproduce a whole quotation, but just the sentence that is being responded to.  Anyone interested in reading the whole original post can click on the quote to see the original post, so the whole post doesn't need to be posted again (many times the original post being quoted is just above the response).   The use of ellipsis (...) reduces bandwidth and makes a page cleaner. :)
Title: Re: EM Drive Developments - related to space flight applications - Thread 2
Post by: TheTraveller on 05/19/2015 01:40 am
OK all good points.  But it is not at all clear that one wants a high Q.

The highest thrust was achieved by Prof. Yang with an effective Q of only ~1500 (take into account that the reported Q's appear much larger because of the unorthodox way the Chinese report Q, if one uses the same method as in the West, Yang's Q was relatively low).

Todd's theory shows that one doesn't want a huge Q.

So using a mesh would be of research interest, to see what difference it makes., whether it makes no difference, or is worse or better

OK I'm scratching my head. Can't find where the Chinese state higher thrust needs lower Q. Maybe you can find it for me?

Can find it stated for the 4 modes examined, higher Q = higher thrust as attached:
http://www.emdrive.com/NWPU2010translation.pdf

Think I'll follow the data and go for high Q. ;)
Title: Re: EM Drive Developments - related to space flight applications - Thread 2
Post by: Rodal on 05/19/2015 01:44 am
...
OK I'm scratching my head. Can't find where the Chinese state higher thrust needs lower Q. Maybe you can find it for me?
...
What I stated was

...The highest thrust was achieved by Prof. Yang with an effective Q of only ~1500 (take into account that the reported Q's appear much larger because of the unorthodox way the Chinese report Q, if one uses the same method as in the West, Yang's Q was relatively low)...

Hat-tip to Star-Drive and Zen-In for first uncovering this fact about Prof. Yang's tests

____

TheTraveller: the Q's you posted are not calculated the same way as Shawyer and people in the West calculates them.   Apples and Oranges
Title: Re: EM Drive Developments - related to space flight applications - Thread 2
Post by: TheTraveller on 05/19/2015 01:54 am
...
OK I'm scratching my head. Can't find where the Chinese state higher thrust needs lower Q. Maybe you can find it for me?

Can find it stated for the 4 modes examined, higher Q = higher thrust as attached.
What I stated was that Prof. Yang has achieved the record highest thrust recorded for the EM Drive and that the Q's she conducted the experiments at (when calculated the same way as they are calculated in the West) are low Q~1500 when compared to Shawyer's Q (hat tip to Star-Drive and zen-in for first uncovering this)

I have read the 3 papers many times. Where did she state that? In what table? The 2010 paper I linked and the attached table makes it VERY clear the highest thrust came with the highest Q.

BTW I doubt you can get a good frustum Q measurement using a broadband microwave source as the frequency is all over the place and not at 2.45GHz. So the observed bandwidth would be as wide as a barn door because the wide band magnetron output is as wide as a barn door.

Shawyer does say that if using flat end plates, to use a magnetron because of the wide frequency output. So if you want to use a maggie as the Rf generator, use flat end plates and thus have lower Q than if you had shaped end plates and a narrow band Rf source. Nothing here says it is better to have lower Q to get higher power. It is about pragmatic engineering. Making do with what you have to get the best result, even if you know there is a better way.

I see now this was your assumption and not from the paper.

Again I state that if you read the 2010 paper, it is VERY clear the higher the Q, the higher the thrust. Nothing the Chinese nor Shawyer has presented goes against that.

As to how to get a frustum that has constantly varying internal wavelengths to resonate at each end plate, from a different applied Rf wavelength, well I'm working on that. Might be my secret squirrel secret sauce.
Title: Re: EM Drive Developments - related to space flight applications - Thread 2
Post by: Rodal on 05/19/2015 01:57 am
...
I have read the 3 papers many times. Where did she state that? In what table? The 2010 paper I linked and the attached table makes it VERY clear the highest thrust came with the highest Q.

BTW I doubt you can get a good frustum Q measurement using a broadband microwave source as the frequency is all over the place and not at 2.45GHz. So the observed bandwidth would be as wide as a barn door because the wide band magnetron output is as wide as a barn door.

I see now this was your assumption and not from the paper.

Again I state that if you read the 2010 paper, it is VERY clear the higher the Q, the higher the thrust. Nothing the Chinese nor Shawyer has presented goes against that.

Well it may help to read them again, and to recalculate the Q in Yang's tables the way Q is calculated in the West  :)

http://forum.nasaspaceflight.com/index.php?topic=36313.msg1369553#msg1369553

Particularly read Prof. Yang's most recent paper containing the embedded thermocouple temperature measurements, that spell this out very clearly.

Kudos to zen-in by computing the Q correctly, and to Star-Drive for first uncovering this fact.
Title: Re: EM Drive Developments - related to space flight applications - Thread 2
Post by: TheTraveller on 05/19/2015 02:06 am
...
I have read the 3 papers many times. Where did she state that? In what table? The 2010 paper I linked and the attached table makes it VERY clear the highest thrust came with the highest Q.

BTW I doubt you can get a good frustum Q measurement using a broadband microwave source as the frequency is all over the place and not at 2.45GHz. So the observed bandwidth would be as wide as a barn door because the wide band magnetron output is as wide as a barn door.

I see now this was your assumption and not from the paper.

Again I state that if you read the 2010 paper, it is VERY clear the higher the Q, the higher the thrust. Nothing the Chinese nor Shawyer has presented goes against that.

Well, read them again, and this time please figure out by yourself how Yang calculates the Q in her tables  :)

http://forum.nasaspaceflight.com/index.php?topic=36313.msg1369553#msg1369553

Particularly read, in Chinese, Prof. Yang's most recent paper containing the embedded thermocouple temperature measurements, that spell this out very clearly.

What does it matter how the Chinese measured their Q? Their data consistently shows the higher their Q, the higher the thrust. The relationship is clear.

Don't get hung up on how the numbers are calculated. I realise that is hard as you are a numbers guy. Look at the relationships between their Q values and their Thrust values. Is clear the higher the Q, the higher the thrust.
Title: Re: EM Drive Developments - related to space flight applications - Thread 2
Post by: Fugudaddy on 05/19/2015 02:10 am
Quick note on Iulian's test; this was his third attempt. Unless I'm missing something, wouldn't have any thermal effects have shown up on those earlier tests at least in some form or was his measurements not accurate enough to measure that small of an effect?

Not as quick note on the Internet. With all due respect to Dr. Rodal- his characterization of the Internet (http://forum.nasaspaceflight.com/index.php?topic=36313.msg1375139#msg1375139) is a bit inaccurate. Yes, there are plenty of distractions to be had, but to those with developed critical thinking skills, the Internet can be a fantastic tool.

In fact, the Internet was invented *for discussions just like this one*. Scientists getting together to share information and collaborate in new ways across distance and experience. This discussion has been an amazing example of social science application of this tool. Besides Dr. Rodal and the EW team's contributions, there's been several DIYers, mathematicians, skeptics, and scientists of all strips all tossing in ideas.

But then, out of that, there's been WarpTech's now known 'Todd Conjecture'. SeeShell's waveform expertise, seeing microwaves in hot tubs. Iulian's new much-debated video. Contributions from around the world. All the people working together here that have been pushing the literal boundaries of human understanding.

Having watched the Internet grow from it's ARPA/DARPA days in the 70's and 80's through to the ubiquitous tool that it is today, seeing exploration like this happening is an amazing experience; no matter *what* comes of this EM drive experimentation.

Yes, it's a bit like the Tower of Babel come to life, millions of voices all reaching out at once. But the Internet is a tool. And like any tool used in the hands of those with critical thinking skills, it becomes a powerful tool indeed.

Keep up the good work! This is an amazing time in human history to be doing science.
Title: Re: EM Drive Developments - related to space flight applications - Thread 2
Post by: Rodal on 05/19/2015 02:30 am
...Not as quick note on the Internet. With all due respect to Dr. Rodal- his characterization of the Internet (http://forum.nasaspaceflight.com/index.php?topic=36313.msg1375139#msg1375139) is a bit inaccurate....
Yes I corrected myself in a subsequent message (*):

http://forum.nasaspaceflight.com/index.php?topic=36313.msg1375149#msg1375149

where I say that I agree, with what DYFAN (and you) say about the Internet,

World-wide collaboration in the Internet is a sure plus  :)

______________
(*) I guess that when I wrote the first message I must have been feeling information overload  ;)
(http://blogs.ischool.utexas.edu/perspectives/files/2013/04/informationoverloadcartoon.jpg)
Title: Re: EM Drive Developments - related to space flight applications - Thread 2
Post by: WarpTech on 05/19/2015 02:33 am
I have read the 3 papers many times. Where did she state that? In what table? The 2010 paper I linked and the attached table makes it VERY clear the highest thrust came with the highest Q.

BTW I doubt you can get a good frustum Q measurement using a broadband microwave source as the frequency is all over the place and not at 2.45GHz. So the observed bandwidth would be as wide as a barn door because the wide band magnetron output is as wide as a barn door.

I see now this was your assumption and not from the paper.

Again I state that if you read the 2010 paper, it is VERY clear the higher the Q, the higher the thrust. Nothing the Chinese nor Shawyer has presented goes against that.

As to how to get a frustum that has constantly varying internal wavelengths to resonate at each end plate, from a different applied Rf wavelength, well I'm working on that. Might be my secret squirrel secret sauce.

Sorry for the confusion. I was the one that said that optimizing for a higher Q is counterproductive for generating thrust, not Yang. Here is my logic;

The only way the frustum can gain momentum from the EM waves inside is if those waves are attenuated on each cycle, asymmetrically. The Q is the energy stored/loss per cycle. So increasing Q by decreasing the loss per cycle, effectively it MUST reduce the amount of attenuation such that there is less thrust transferred to the frustum. Alternatively, if you increase Q by storing more energy, without altering the amount of power attenuated, then there is more energy in reserve to draw from. If the system used PWM, it could sustain a longer duty cycle.

So there are advantages to higher Q that can produce a higher thrust, but optimizing Q at the expense of reducing the attenuation, will lower the thrust, IMO.

I wonder what, if anything would result if there were a perfectly cylindrical 1/4 wave stub on one side of the source emitter and a frustum at 3/4 wavelength long on the other side of the emitter. The 1/4 wave stub side on the big end would resonate, and the frustum would attenuate. Engineered asymmetry with a much simpler resonant cavity.

Todd D.
Title: Re: EM Drive Developments - related to space flight applications - Thread 2
Post by: rfmwguy on 05/19/2015 02:39 am
Quick note on Iulian's test; this was his third attempt. Unless I'm missing something, wouldn't have any thermal effects have shown up on those earlier tests at least in some form or was his measurements not accurate enough to measure that small of an effect?

Not as quick note on the Internet. With all due respect to Dr. Rodal- his characterization of the Internet (http://forum.nasaspaceflight.com/index.php?topic=36313.msg1375139#msg1375139) is a bit inaccurate. Yes, there are plenty of distractions to be had, but to those with developed critical thinking skills, the Internet can be a fantastic tool.

In fact, the Internet was invented *for discussions just like this one*. Scientists getting together to share information and collaborate in new ways across distance and experience. This discussion has been an amazing example of social science application of this tool. Besides Dr. Rodal and the EW team's contributions, there's been several DIYers, mathematicians, skeptics, and scientists of all strips all tossing in ideas.

But then, out of that, there's been WarpTech's now known 'Todd Conjecture'. SeeShell's waveform expertise, seeing microwaves in hot tubs. Iulian's new much-debated video. Contributions from around the world. All the people working together here that have been pushing the literal boundaries of human understanding.

Having watched the Internet grow from it's ARPA/DARPA days in the 70's and 80's through to the ubiquitous tool that it is today, seeing exploration like this happening is an amazing experience; no matter *what* comes of this EM drive experimentation.

Yes, it's a bit like the Tower of Babel come to life, millions of voices all reaching out at once. But the Internet is a tool. And like any tool used in the hands of those with critical thinking skills, it becomes a powerful tool indeed.

Keep up the good work! This is an amazing time in human history to be doing science.

Well said, kudos...the "interwebs" is ideal for collaborative efforts, especially for those with an independent mind and a healthy measure of nonconformity. So often, public and private institutions adopt a play it safe mode and demotivate folks...perhaps the next Einstein is already posting here  ;)

Title: Re: EM Drive Developments - related to space flight applications - Thread 2
Post by: aero on 05/19/2015 02:51 am
The picture of the wire frame end plate of a frustum got me to thinking a little out of the box. If a frustum were constructed of screen wire, like a window screen - wire, not that plastic stuff, wouldn't it resonate, too? At very low power it could keep its shape but at higher power mightn't it measurably deform from the internal optical pressure? And since we know the strength of the screen wire and the optical pressure from plane waves, couldn't we determine whether or not there was excess force being generated?

Perhaps a more simple construction would be from a solid copper conic section and copper screens on the ends attached in such a way as to encourage the deformation.

My point being that so far, all of the experiments have been constructed to serve as a rocket engine, but no one has made an experiment to look at the forces directly. Might there not be a more suitable configuration to look at the cavity forces than the frustum? And if we knew for sure that strange forces existed wouldn't that help the effort?
Title: Re: EM Drive Developments - related to space flight applications - Thread 2
Post by: Rodal on 05/19/2015 02:59 am
The picture of the wire frame end plate of a frustum got me to thinking a little out of the box. If a frustum were constructed of screen wire, like a window screen - wire, not that plastic stuff, wouldn't it resonate, too? At very low power it could keep its shape but at higher power mightn't it measurably deform from the internal optical pressure? And since we know the strength of the screen wire and the optical pressure from plane waves, couldn't we determine whether or not there was excess force being generated?

Perhaps a more simple construction would be from a solid copper conic section and copper screens on the ends attached in such a way as to encourage the deformation.

My point being that so far, all of the experiments have been constructed to serve as a rocket engine, but no one has made an experiment to look at the forces directly. Might there not be a more suitable configuration to look at the cavity forces than the frustum? And if we knew for sure that strange forces existed wouldn't that help the effort?

That's a  great way to conduct research on optimization and to understand something complicated. It reminds me of how the first human powered plane design came about. The  Gossamer Condor beat the MIT design (that was based on analysis) by evolutionary refinement of the design based on a large number of tests.
Title: Re: EM Drive Developments - related to space flight applications - Thread 2
Post by: aero on 05/19/2015 03:28 am
The picture of the wire frame end plate of a frustum got me to thinking a little out of the box. If a frustum were constructed of screen wire, like a window screen - wire, not that plastic stuff, wouldn't it resonate, too? At very low power it could keep its shape but at higher power mightn't it measurably deform from the internal optical pressure? And since we know the strength of the screen wire and the optical pressure from plane waves, couldn't we determine whether or not there was excess force being generated?

Perhaps a more simple construction would be from a solid copper conic section and copper screens on the ends attached in such a way as to encourage the deformation.

My point being that so far, all of the experiments have been constructed to serve as a rocket engine, but no one has made an experiment to look at the forces directly. Might there not be a more suitable configuration to look at the cavity forces than the frustum? And if we knew for sure that strange forces existed wouldn't that help the effort?

That's a  great way to conduct research on optimization and to understand something complicated. It reminds me of how the first human powered plane design came about. The  Gossamer Condor beat the MIT design (that was based on analysis) by evolutionary refinement of the design based on a large number of tests.
Probably wouldn't need to use screen, just ultra thin copper bases. They should deform in one direction or the other. The direction of the deformation would say a lot about the force. If an untra thin copper sheet deformed differently than a copper screen, that might say that the effect is happening within the skin depth of the copper? One might even be able to string thin wires in one direction only, a screen with all of the y-direction wires removed. Then using the lower resonance modes only, the screen deformation might show a pattern of the forces. Might not, too.

Oh well, I won't be doing such experiments so perhaps that's enough for this train of thought.
Title: Re: EM Drive Developments - related to space flight applications - Thread 2
Post by: deltaMass on 05/19/2015 03:59 am
I'm betting that Iulian is going to measure the same -0.5 gm weight change after he's flipped it upside down, meaning he's measuring hot air. But I'm no big fan of throwing babies out with bathwaters. The next logical step would then be to put something (e.g. a big plastic bag) around the whole thing and make sure that it's a perfect seal. Then retest to see if there is any real thrust at all there.
Title: Re: EM Drive Developments - related to space flight applications - Thread 2
Post by: zen-in on 05/19/2015 04:08 am
I would put my money on electromagnetic effects of the HV wires connecting the capacitor to the magnetron.  When current starts moving in those wires they will repel each other.  That can resolve itself as an upward force on the cavity.   If this is true then any adjustment of the position of the wires will change the measured force.
Title: Re: EM Drive Developments - related to space flight applications - Thread 2
Post by: SeeShells on 05/19/2015 05:08 am
I wonder what, if anything would result if there were a perfectly cylindrical 1/4 wave stub on one side of the source emitter and a frustum at 3/4 wavelength long on the other side of the emitter. The 1/4 wave stub side on the big end would resonate, and the frustum would attenuate. Engineered asymmetry with a much simpler resonant cavity.

Todd D.
Is this the psychic blog? Great idea!  I had a similar thought and have been mulling it over on how to apply it still using the EM snow cone shape. I asked if anyone had thought of using 2 insertion sites into the cavity and the answer I got, it might be considered this next go around of tests.
I'm not sure a perfect cylinder would get the effects we're looking for. I think first is finding out what we are seeing that's causing this CoE and CoM abnormality, but it sure would be quite inexpensive to design, build and test so you could just to plug in and see. It would also give you some very solid baseline data from very well known formulas.
Title: Re: EM Drive Developments - related to space flight applications - Thread 2
Post by: dustinthewind on 05/19/2015 06:25 am
I wonder what, if anything would result if there were a perfectly cylindrical 1/4 wave stub on one side of the source emitter and a frustum at 3/4 wavelength long on the other side of the emitter. The 1/4 wave stub side on the big end would resonate, and the frustum would attenuate. Engineered asymmetry with a much simpler resonant cavity.

Todd D.
Is this the psychic blog? Great idea!  I had a similar thought and have been mulling it over on how to apply it still using the EM snow cone shape. I asked if anyone had thought of using 2 insertion sites into the cavity and the answer I got, it might be considered this next go around of tests.
I'm not sure a perfect cylinder would get the effects we're looking for. I think first is finding out what we are seeing that's causing this CoE and CoM abnormality, but it sure would be quite inexpensive to design, build and test so you could just to plug in and see. It would also give you some very solid baseline data from very well known formulas.

I get the impression the current in the 1/4 wave stub moves with the electric field of the incoming radiation for constructive interference and the 3/4 frustum I am guessing would be working against the radiation and be attenuated.  Is this similar to a directional antenna array?  I am not quite seeing the picture but I think I might understand the concept. 
Title: Re: EM Drive Developments - related to space flight applications - Thread 2
Post by: TheTraveller on 05/19/2015 08:46 am
I have read the 3 papers many times. Where did she state that? In what table? The 2010 paper I linked and the attached table makes it VERY clear the highest thrust came with the highest Q.

BTW I doubt you can get a good frustum Q measurement using a broadband microwave source as the frequency is all over the place and not at 2.45GHz. So the observed bandwidth would be as wide as a barn door because the wide band magnetron output is as wide as a barn door.

I see now this was your assumption and not from the paper.

Again I state that if you read the 2010 paper, it is VERY clear the higher the Q, the higher the thrust. Nothing the Chinese nor Shawyer has presented goes against that.

As to how to get a frustum that has constantly varying internal wavelengths to resonate at each end plate, from a different applied Rf wavelength, well I'm working on that. Might be my secret squirrel secret sauce.
The only way the frustum can gain momentum from the EM waves inside is if those waves are attenuated on each cycle, asymmetrically.

Which is how I understand Shawyer's CofE theory, which he claims to have experimentally verified.

Frustum movement (increased Kinetic energy of the frustum from stored cavity energy) causes the cavity to detune, increasing Q energy losses, dropping Q, dropping impedance, causing more microwave energy to enter the cavity, causing increased energy draw from the primary electrical source. Conserving CofE.

The frustum movement energy related Q energy loss is the energy that was converted into Kinetic.
Title: Re: EM Drive Developments - related to space flight applications - Thread 2
Post by: deltaMass on 05/19/2015 09:29 am
Quote
Frustum movement (increased Kinetic energy of the frustum from stored cavity energy) causes the cavity to detune, increasing Q energy losses,dropping Q, dropping impedance, causing more microwave energy to enter the cavity, causing increased energy draw from the primary electrical source. Conserving CofE.
I'm afraid electrical circuits don't work quite like that, although it must be said that it's indeed a valiant attempt at rationality. Typically, you see, anything that causes a source and a load to transition away from a matched state will result in less power being transferred from source to load, not more.
Title: Re: EM Drive Developments - related to space flight applications - Thread 2
Post by: dillonthomas on 05/19/2015 12:37 pm
Hello!

I've been reading this thread with enthusiasm for months now!  I had an idea/question that is bugging the heck out of me that I feel compelled to ask. I joined specifically to ask this question. Perhaps it will lend to the discussion.

Regarding the shape of the cavity. Would making the cavity the shape of a sphere provide any benefit Or maybe an oblong sphere, like an egg?  For some reason, I visualize having a controllable/shapeable interference pattern and waveguide  according to the strength and frequency of the microwaves while bouncing around inside a sphere. Or, perhaps, the question shows my ignorance of the subject.   :-[

I'm humbled and honored to be in the presence of all of you intellectual giants.

Dillon
Title: Re: EM Drive Developments - related to space flight applications - Thread 2
Post by: Rodal on 05/19/2015 01:01 pm
..
I was the one that said that optimizing for a higher Q is counterproductive for generating thrust, not Yang. Here is my logic;

The only way the frustum can gain momentum from the EM waves inside is if those waves are attenuated on each cycle, asymmetrically. The Q is the energy stored/loss per cycle. So increasing Q by decreasing the loss per cycle, effectively it MUST reduce the amount of attenuation such that there is less thrust transferred to the frustum. Alternatively, if you increase Q by storing more energy, without altering the amount of power attenuated, then there is more energy in reserve to draw from. If the system used PWM, it could sustain a longer duty cycle.

So there are advantages to higher Q that can produce a higher thrust, but optimizing Q at the expense of reducing the attenuation, will lower the thrust, IMO.
...
Excellent statement:  "The only way the frustum can gain momentum from the EM waves inside is if those waves are attenuated on each cycle, asymmetrically"

...

I wonder what, if anything would result if there were a perfectly cylindrical 1/4 wave stub on one side of the source emitter and a frustum at 3/4 wavelength long on the other side of the emitter. The 1/4 wave stub side on the big end would resonate, and the frustum would attenuate. Engineered asymmetry with a much simpler resonant cavity.

Todd D.

Resonant cavity for attenuating electromagnetic standing waves in an (open) waveguide.
Multiple frequencies of the standing wave may be attenuated by locating identical cavities at different antinodes along an (open) waveguide:

http://www.google.com/patents/WO1997038437A1?cl=en
Title: Re: EM Drive Developments - related to space flight applications - Thread 2
Post by: hhexo on 05/19/2015 01:07 pm
Probably wouldn't need to use screen, just ultra thin copper bases. They should deform in one direction or the other. The direction of the deformation would say a lot about the force. ...

Hm... interesting idea, but bear in mind that copper is very sensitive to thermal dilation.

If you made a very thin sheet of copper you'd have to find a way of cooling it down very effectively on the outside, otherwise it might be susceptible to a measurable deformation just due to the temperature increase when the system is on.
I guess that the additional forces due to the microwaves would contribute a tiny bit, and you might be able to separate the two effects, but it's a bit of an engineering challenge I think.

A screen is more easily cooled by air convecting through it, but it could possibly suffer from the same problem.
Title: Re: O09
Post by: Einstein79 on 05/19/2015 01:35 pm
Does the Flight Thruster have a slightly concave top and convex bottom? Would appear so from the gaps.

Enhanced the photo as much as I can for those wishing to try to extract dimensions as this photo is better that the original as it has no distortion.

If we can find the dimension<M

The big end most certainly should be convex and the small end concave, relative from the outside of course. The big end and small end radii should not be coincidence but offset having the small end radius much larger than the big end. In fact, it might be better for the small end to be flat.

If either end is flat, the bounce will introduce very significant phase distortion into the returning curved wave. For me it is hard to see that Shawyer ever used flat end plates INSIDE the cavity. As we never saw inside the cavity, what is to say he didn't use curved end plates inside and flat end covers outside? What he drew may not be what he built.

Sorry about that, did not think about the returning wave. The cavity that you posted with the same vertex for each end was my first thought anyway. Great job guys! I am trying to get things situated so I can help more.
Title: Re: EM Drive Developments - related to space flight applications - Thread 2
Post by: TheTraveller on 05/19/2015 01:59 pm
Quote
Frustum movement (increased Kinetic energy of the frustum from stored cavity energy) causes the cavity to detune, increasing Q energy losses,dropping Q, dropping impedance, causing more microwave energy to enter the cavity, causing increased energy draw from the primary electrical source. Conserving CofE.
I'm afraid electrical circuits don't work quite like that, although it must be said that it's indeed a valiant attempt at rationality. Typically, you see, anything that causes a source and a load to transition away from a matched state will result in less power being transferred from source to load, not more.

Energy will flow from the Rf input, into the frustum, until the 5 TCs required to fill it. Then the only additional energy input will be to replace Q losses or until it moves and frustum stored energy is converted into kinetic.

When the frustum moves, it is no longer resonant at the non moving frequency and a new resonant frequency results. This is why, in the Flight Thruster, the Rf input frequency tracks changes in the frustum resonant frequency.

With a magnetron Rf source, the broadband frequency output can handle the change in resonant frequency. For the higher tuned, high Q Flight Thruster, the narrow band Rf generator must track the resonant frequency change so as to be able to replace frustum energy to kinetic energy conversion / loss to the frustum.

In some way I see the operation of the frustum like a parallel LC circuit where circuit impedance is maxed at resonance and circuit current is at minimum.

Then if one of the resonant circuit elements changes, resonant frequency changes and effective circuit impedance is no longer at max nor current no longer at min.

A frustum at resonance is like a parallel tuned circuit:
http://www.electronics-tutorials.ws/accircuits/parallel-resonance.html

Max impedance and min current at resonance. As the frustum moves it is no longer at resonance, impedance drops and current increases.

One subject is how does Shawyer do impedance matching of the frustum to the microwave source? He seems to do it by, in the case of his superconducting frustum patent by adjusting the input slot size. With the Flight Thruster he has this as per the attached:
Title: Re: EM Drive Developments - related to space flight applications - Thread 2
Post by: txdrive on 05/19/2015 02:02 pm
Ultimately what Shawyer's and White's theories amount to, is that there is a radiation pressure imbalance on the inside of the cavity, resulting in a net force, which is a non small fraction of the total radiation pressure on the inside of the cavity (more than 1%). Shawyer says it is in accordance with Maxwell's equations, which is flat out wrong.

White says microwaves interact with the quantum vacuum in some manner, emitting some kind of a jet. People had been arguing about that, while forgetting that this does not alleviate the need for the radiation pressure imbalance, meaning that his E and H solution at the surface of the cavity differs from one obtained with Maxwell's equations, by an amount such that you're getting what, a net force of over 50 uN from 50W at a Q factor less than 7000 or so?

The total radiation pressure on the inside of the cavity is on the order of 50W*7000/c , about 2000 uN , so we're talking good ol Maxwell being off by at least 2.5% somewhere in the cavity. It's sort of like having a mechanical perpetuum mobile that needs it's levers to violate the lever equation by at least 2.5% .

edit: Even worse, microwave photons can't simply be absorbed in the interaction with "aether" (then the momentum imbalance will be equal to the power absorbed in this interaction divided by c exactly, with same performance as a regular photon drive except for all the losses in the cavity walls). They have to interact elastically, which means they must lose just a part of their energy at once, which means you would have new photons of a lower frequency, which would make it even more blatantly obvious for devices that sense microwaves.
Title: Re: EM Drive Developments - related to space flight applications - Thread 2
Post by: Mike-F on 05/19/2015 02:18 pm
Quote
Frustum movement (increased Kinetic energy of the frustum from stored cavity energy) causes the cavity to detune, increasing Q energy losses,dropping Q, dropping impedance, causing more microwave energy to enter the cavity, causing increased energy draw from the primary electrical source. Conserving CofE.
I'm afraid electrical circuits don't work quite like that, although it must be said that it's indeed a valiant attempt at rationality. Typically, you see, anything that causes a source and a load to transition away from a matched state will result in less power being transferred from source to load, not more.

So an easy way to test if the EMdrive works, would be to build a low power unit (1 watt or less). Then with the system powered and tuned move it! Any movement along  the thrust axis should have a corresponding effect on the cavity resonance. This may be far easier to detect and confirm than the  very small forces developed so far.

Mike
Title: Re: EM Drive Developments - related to space flight applications - Thread 2
Post by: Rodal on 05/19/2015 02:18 pm
The number of photons (N) inside a cavity is not conserved. A photon may collide with an electron on a wall, exciting it to a higher energy state, removing a photon. This electron may drop back to its lower level in a series of steps, each one of which releases an individual photon back into the cavity.  Although the sum of the energies of the emitted photons are the same as the absorbed photon, the number of emitted photons will vary.

The absorption (emission) of one photon might be accompanied by the emission (absorption) of more than one photon, as long as the frequency of the photons involved are such that the energy of the system remains constant.

N over volume, and the entropy density are proportional to the cube of the temperature of the photon gas, while the pressure and the energy density are proportional to the fourth power of its temperature. 

For black body radiation it can be shown that, as a result of this lack of constraint on the number of photons in the system, the chemical potential of the photons must be zero.

How does the Entropy/Information Bound Work?  http://arxiv.org/pdf/quant-ph/0404042

Decoherence via Dynamical Casimir Effect  http://arxiv.org/pdf/quant-ph/9910027

Thermodynamics of the Casimir effect  http://arxiv.org/pdf/quant-ph/9902074v1

Dynamical Casimir effect at finite temperature  http://arxiv.org/pdf/quant-ph/9906122
Title: Re: EM Drive Developments - related to space flight applications - Thread 2
Post by: TheTraveller on 05/19/2015 02:32 pm
Ultimately what Shawyer's and White's theories amount to, is that there is a radiation pressure imbalance on the inside of the cavity, resulting in a net force, which is a non small fraction of the total radiation pressure on the inside of the cavity (more than 1%). Shawyer says it is in accordance with Maxwell's equations, which is flat out wrong.

Where is Shawyer wrong?

1) Is the cutoff wavelength different at the small and big ends or not?

2) Is the guide wavelength different at the small and big ends or not?

3) Is the group velocity different at the small and big ends or not?

4) Is the bounce force different at the small and big ends or not?

5) Is the bounce force at the big end greater than at the small end or not?

6) Is there a bounce force on the side walls or not?

Just trying to understand where you believe Shawyer is wrong?
Title: Re: EM Drive Developments - related to space flight applications - Thread 2
Post by: txdrive on 05/19/2015 02:35 pm
Quote
Frustum movement (increased Kinetic energy of the frustum from stored cavity energy) causes the cavity to detune, increasing Q energy losses,dropping Q, dropping impedance, causing more microwave energy to enter the cavity, causing increased energy draw from the primary electrical source. Conserving CofE.
I'm afraid electrical circuits don't work quite like that, although it must be said that it's indeed a valiant attempt at rationality. Typically, you see, anything that causes a source and a load to transition away from a matched state will result in less power being transferred from source to load, not more.

So an easy way to test if the EMdrive works, would be to build a low power unit (1 watt or less). Then with the system powered and tuned move it! Any movement along  the thrust axis should have a corresponding effect on the cavity resonance. This may be far easier to detect and confirm than the  very small forces developed so far.

Mike
Earth is rotating and orbiting the sun, and there's all sorts of electronic devices where nothing was ever observed to vary with the time of day, down to very high precision.
Title: Re: EM Drive Developments - related to space flight applications - Thread 2
Post by: TheTraveller on 05/19/2015 02:37 pm
Quote
Frustum movement (increased Kinetic energy of the frustum from stored cavity energy) causes the cavity to detune, increasing Q energy losses,dropping Q, dropping impedance, causing more microwave energy to enter the cavity, causing increased energy draw from the primary electrical source. Conserving CofE.
I'm afraid electrical circuits don't work quite like that, although it must be said that it's indeed a valiant attempt at rationality. Typically, you see, anything that causes a source and a load to transition away from a matched state will result in less power being transferred from source to load, not more.

So an easy way to test if the EMdrive works, would be to build a low power unit (1 watt or less). Then with the system powered and tuned move it! Any movement along  the thrust axis should have a corresponding effect on the cavity resonance. This may be far easier to detect and confirm than the  very small forces developed so far.

Mike

Good idea! Thanks.

Don't need to power it as such. Just use a spectrum analyser to detect resonant frequency changes with acceleration.

Have added that to my list of things to test.
Title: Re: EM Drive Developments - related to space flight applications - Thread 2
Post by: TheTraveller on 05/19/2015 02:39 pm
Quote
Frustum movement (increased Kinetic energy of the frustum from stored cavity energy) causes the cavity to detune, increasing Q energy losses,dropping Q, dropping impedance, causing more microwave energy to enter the cavity, causing increased energy draw from the primary electrical source. Conserving CofE.
I'm afraid electrical circuits don't work quite like that, although it must be said that it's indeed a valiant attempt at rationality. Typically, you see, anything that causes a source and a load to transition away from a matched state will result in less power being transferred from source to load, not more.

So an easy way to test if the EMdrive works, would be to build a low power unit (1 watt or less). Then with the system powered and tuned move it! Any movement along  the thrust axis should have a corresponding effect on the cavity resonance. This may be far easier to detect and confirm than the  very small forces developed so far.

Mike
Earth is rotating and orbiting the sun, and there's all sorts of electronic devices where nothing was ever observed to vary with the time of day, down to very high precision.

Frustum would need to constantly accelerate to be able to see resonant frequency changes.
Title: Re: EM Drive Developments - related to space flight applications - Thread 2
Post by: txdrive on 05/19/2015 02:43 pm
Ultimately what Shawyer's and White's theories amount to, is that there is a radiation pressure imbalance on the inside of the cavity, resulting in a net force, which is a non small fraction of the total radiation pressure on the inside of the cavity (more than 1%). Shawyer says it is in accordance with Maxwell's equations, which is flat out wrong.

Where is Shawyer wrong?

1) Is the cutoff wavelength different at the small and big ends or not?

2) Is the guide wavelength different at the small and big ends or not?

3) Is the group velocity different at the small and big ends or not?

4) Is the bounce force different at the small and big ends or not?

5) Is the bounce force at the big end greater than at the small end or not?

6) Is there a bounce force on the side walls or not?

Just trying to understand where you believe Shawyer is wrong?
Well, firstly, Shawyer has greater radiation pressure upon the wide end, but his drive is pushing narrow end forwards.

Secondarily, yes, there is an interaction between the EM field and the walls, which results in a force on the side walls, equal to change in momentum of the EM field travelling down it, per time.
Title: Re: EM Drive Developments - related to space flight applications - Thread 2
Post by: TheTraveller on 05/19/2015 03:03 pm
Ultimately what Shawyer's and White's theories amount to, is that there is a radiation pressure imbalance on the inside of the cavity, resulting in a net force, which is a non small fraction of the total radiation pressure on the inside of the cavity (more than 1%). Shawyer says it is in accordance with Maxwell's equations, which is flat out wrong.

Where is Shawyer wrong?

1) Is the cutoff wavelength different at the small and big ends or not?

2) Is the guide wavelength different at the small and big ends or not?

3) Is the group velocity different at the small and big ends or not?

4) Is the bounce force different at the small and big ends or not?

5) Is the bounce force at the big end greater than at the small end or not?

6) Is there a bounce force on the side walls or not?

Just trying to understand where you believe Shawyer is wrong?
Well, firstly, Shawyer has greater radiation pressure upon the wide end, but his drive is pushing narrow end forwards.

Secondarily, yes, there is an interaction between the EM field and the walls, which results in a force on the side walls, equal to change in momentum of the EM field travelling down it, per time.

Maxwell has a cosine factor to adjust for waves that hit the bounce surface at an angle. Max force at the end plates and min force at the walls as attached.

CofM requires if the EM waves pushes more on the on the big end than the small end, the EM Drive frustum pushes back in the opposite direction of the imbalance. Momentum transfer is not one sided. If the EM Drive moved toward the big end, then CofE would be violated as only a one sided push.

That effect is very clear and confirmed in the Demonstrator rotary tests:
http://emdrive.com/dynamictests.html
where you can see the movement direction and that the unit is continually accelerating / speeding up over time.
Title: Re: EM Drive Developments - related to space flight applications - Thread 2
Post by: Notsosureofit on 05/19/2015 03:11 pm
The number of photons (N) inside a cavity is not conserved. A photon may collide with an electron on a wall, exciting it to a higher energy state, removing a photon. This electron may drop back to its lower level in a series of steps, each one of which releases an individual photon back into the cavity.  Although the sum of the energies of the emitted photons are the same as the absorbed photon, the number of emitted photons will vary.

The absorption (emission) of one photon might be accompanied by the emission (absorption) of more than one photon, as long as the frequency of the photons involved are such that the energy of the system remains constant.

N over volume, and the entropy density are proportional to the cube of the temperature of the photon gas, while the pressure and the energy density are proportional to the fourth power of its temperature. 

For black body radiation it can be shown that, as a result of this lack of constraint on the number of photons in the system, the chemical potential of the photons must be zero.

How does the Entropy/Information Bound Work?  http://arxiv.org/pdf/quant-ph/0404042

One point to keep in mind is that the change from 2GHz to ambient thermal radiation is an up-conversion, that is, it represents a negative entropy change component that must be more than made up somewhere else.
Title: Re: EM Drive Developments - related to space flight applications - Thread 2
Post by: TheTraveller on 05/19/2015 03:22 pm
Ah, found it. Dr. Rodal please note well the highlighted section in the 2nd attachment.

Here is how Shawyer measured his 2 force measurements on his Demonstrator EM Drive as per the 4th results line of the 1st attachment.

He used the rotary test rig and measured the acceleration and deceleration forces generated by the EM Drive. Did this in both directions. IE CW acceleration, then CW deceleration, then CCW acceleration, then CCW deceleration.

Clever boy our Roger.
Title: Re: EM Drive Developments - related to space flight applications - Thread 2
Post by: Rodal on 05/19/2015 03:33 pm
Ah, found it. Dr. Rodal please note well the highlighted section in the 2nd attachment.

Here is how Shawyer measured his 2 force measurements on his Demonstrator EM Drive as per the 4th results line of the 1st attachment.

He used the rotary test rig and measured the acceleration and deceleration forces generated by the EM Drive. Did this in both directions. IE CW acceleration, then CW deceleration, then CCW acceleration, then CCW deceleration.

Clever boy our Roger.
Thank you for the information.

My question was how were the two forces (what Shawyer calls "reaction" and "thrust") measured simultaneously (I had labored to use Italics first and then bold and blue to highlight "simultaneously").

From the description provided it looks like there has been no experiment measuring simultaneously the two forces (what Shawyer calls "reaction" and "thrust") that Shawyer claims to exist.

The "thrust" force is an explanation by Shawyer to explain conservation of momentum, to explain that Newton's third law of action and reaction is being obeyed. There is no experimental measurement of this "thrust" force simultaneously performed to verify its existence.

All that is being measured is displacement vs. time (or its second-order derivative with respect to time, whatever its sign) of the device

The problem with making measurements of acceleration (positive and negative) and then ascribing the results to the two forces are evident.
Title: Re: EM Drive Developments - related to space flight applications - Thread 2
Post by: TheTraveller on 05/19/2015 03:59 pm
Thank you for the information.

My question was how were the two forces (what Shawyer calls "reaction" and "thrust") measured simultaneously (I had labored to use Italics first and then bold and blue to highlight "simultaneously").

From the description provided it looks like there has been no experiment measuring simultaneously the two forces (what Shawyer calls "reaction" and "thrust") that (quoting you) "Clever boy our Roger" claims to have measured.

All that is being measured is displacement vs. time (or its second-order derivative with respect to time, whatever its sign) of the device

The problem with making measurements in acceleration and deceleration and then ascribing the results to the two forces are evident.

What Shawyer displayed are the forces measured on the Em Drive Demonstrator Engine as it accelerated for some time (gaining velocity / kinetic energy) and then decelerated (losing velocity / kinetic energy) for some time as the unit rotated in the same direction on the rotary turntable. Then the process was repeated with the EM  Drive rotating in the opposite direction. Is what Shawyer calls Motor & Generator mode.

Never read where Shawyer claimed to have measured them at the same time. Can you please point out where he claimed to have done that? If not why mention it being done simultaneously?

Anyway we now know how the 2 measurements were done and what they mean.

BTW in Generator mode, the reducing craft kinetic energy is turned into additional cavity energy. As you can't turn increased frustum microwave energy back into electrical energy, it will heat up the frustum, acting as does a car's break, converting unwanted kinetic energy into heat energy.

What the 2 measurements show are the ability of the Demonstrator Engine to either be a kinetic energy source during acceleration (motor mode) or a kinetic energy sink during deceleration (generator mode).

And yes, in my opinion, it is a clever test to show/prove Motor/Generator operation is real.

The bread crumb trail is there, just need to find them and follow them to where the bread crumbs / data leads you. ;)
Title: Re: EM Drive Developments - related to space flight applications - Thread 2
Post by: dustinthewind on 05/19/2015 04:51 pm
...there is an interaction between the EM field and the walls, which results in a force on the side walls, equal to change in momentum of the EM field travelling down it, per time.

I was thinking there should be forces on the side walls also.  I did an experiment once with an aluminum ring and a changing magnetic field and observed forces on it once that seemed to make it more obvious for me what should be going on in the side walls.  I don't think it explains why the drive would accelerate as I think Maxwell's equations would just give that the forces balance out for a static solution.  My guess is something asymmetric is going on in the drive with time and I suspect it to be a diametric drive where currents interacting with radiation are giving an effective negative mass for one side of the drive and an effective positive mass for currents interacting with the other side. 

Sort of like if you drive the current in the aluminum ring the opposite direction that the radiation would drive it then it attracted instead of repelled.  Then extend that to two aluminum rings and play with the phase in space and time. 
Title: Re: EM Drive Developments - related to space flight applications - Thread 2
Post by: bprager on 05/19/2015 05:16 pm
I was wondering if we would be able to get a faster turnaround with prototyping and maybe even cut costs if we could use 3D printing and e.g. conductive graphene filament like this one:

(http://cdn3.volusion.com/xptdz.fenaa/v/vspfiles/photos/GRPHN-175-2T.jpg).

We could share computer designs and use local 3D printing services. Would that work?
Title: Re: EM Drive Developments - related to space flight applications - Thread 2
Post by: xevioso on 05/19/2015 05:34 pm
Hello.  I've been reading this thread and Iulian's blog with great interest.  I'm just wondering if all of the calculations on possible thrust have only been done on truncated cones or cylinders... after Mr. Iulian retries his experiment inverting the frustrum to see if it's a metal hot air balloon, I'm curious what would happen if...one used a conical bore similar to the bell of a brass musical instrument.  As an experiment, has anyone tried these experiments using a shape similar to the bell end of a tuba, sousaphone, french horn, etc? It would probably be pretty easy to try this experiment using a shape similar to this... maybe with a different material? If this has been addressed (or if a dumb question) I'll remove my post...thanks!

(http://1.bp.blogspot.com/-ZielDMfkg38/Uytay8ovEtI/AAAAAAAAVjI/gJe4UhUkKm8/s1600/Bell+cut.jpg)
Title: Re: EM Drive Developments - related to space flight applications - Thread 2
Post by: phaseshift on 05/19/2015 05:35 pm
I was wondering if we would be able to get a faster turnaround with prototyping and maybe even cut costs if we could use 3D printing and e.g. conductive graphene filament like this one:


We could share computer designs and use local 3D printing services. Would that work?

That's what I've been planning on doing.  Makes construction a great deal simpler for some of the pieces. I'm getting the bottom plate printed in aluminum and I will drill and tap the bolt holes myself.  This makes it much easier to create the spherical concave surface.
Title: Re: EM Drive Developments - related to space flight applications - Thread 2
Post by: D_Dom on 05/19/2015 05:48 pm
Rated at 100 mA for 12 V it seems limited to non current carrying use with power levels under discussion. Website mentions EMI shielding as a potential application. High strength when compared with PLA and ABS could be a useful property in this case.

  Will have to think about how to compare this material with sheet copper or aluminum used in fabricating the cavities, might be helpful in making a set of rings as depicted in Rodals attachment upthread;
NASA, Shawyer, Yang, and other EM Drive researchers would be well advised to experiment with replacing the end plates of the EM Drive with this system of concentric rings, in order to address the problem of air convection currents that has plagued radiation pressure experiments in ambient conditions ever since Maxwell 140 years ago.  Even in a partial vacuum, if one uses for example bilayer plates of copper/glass-fiber-reinforced epoxy with the reinforced polymer on the external surface, there is the possibility of outgassing in a vacuum producing a false positive.  The use of a mesh precludes this problem both in ambient air conditions and in a vacuum.

Attachment: ABSOLUTE POWER MEASUREMENT AT MICROWAVE FREQUENCIES
By A. L. CULLEN, Ph.D., B.Sc.(Eng.), Associate Member.
(published February, 1952.)
http://forum.nasaspaceflight.com/index.php?action=dlattach;topic=36313.0;attach=828862
Title: Re: EM Drive Developments - related to space flight applications - Thread 2
Post by: LasJayhawk on 05/19/2015 05:50 pm
Quote
Frustum movement (increased Kinetic energy of the frustum from stored cavity energy) causes the cavity to detune, increasing Q energy losses,dropping Q, dropping impedance, causing more microwave energy to enter the cavity, causing increased energy draw from the primary electrical source. Conserving CofE.
I'm afraid electrical circuits don't work quite like that, although it must be said that it's indeed a valiant attempt at rationality. Typically, you see, anything that causes a source and a load to transition away from a matched state will result in less power being transferred from source to load, not more.

So an easy way to test if the EMdrive works, would be to build a low power unit (1 watt or less). Then with the system powered and tuned move it! Any movement along  the thrust axis should have a corresponding effect on the cavity resonance. This may be far easier to detect and confirm than the  very small forces developed so far.

Mike
Earth is rotating and orbiting the sun, and there's all sorts of electronic devices where nothing was ever observed to vary with the time of day, down to very high precision.

Frustum would need to constantly accelerate to be able to see resonant frequency changes.

We could pull it behind my 65 Pontiac Catalina. :) NASA has already proven that a 421 powered Pontiac will make things fly.

http://www.dfrc.nasa.gov/Gallery/Movie/M2-F1/HTML/EM-0020-02.html
Title: Re: EM Drive Developments - related to space flight applications - Thread 2
Post by: Rodal on 05/19/2015 05:53 pm
Hello.  I've been reading this thread and Iulian's blog with great interest.  I'm just wondering if all of the calculations on possible thrust have only been done on truncated cones or cylinders... after Mr. Iulian retries his experiment inverting the frustrum to see if it's a metal hot air balloon, I'm curious what would happen if...one used a conical bore similar to the bell of a brass musical instrument.  As an experiment, has anyone tried these experiments using a shape similar to the bell end of a tuba, sousaphone, french horn, etc? It would probably be pretty easy to try this experiment using a shape similar to this... maybe with a different material? If this has been addressed (or if a dumb question) I'll remove my post...thanks!

(http://1.bp.blogspot.com/-ZielDMfkg38/Uytay8ovEtI/AAAAAAAAVjI/gJe4UhUkKm8/s1600/Bell+cut.jpg)
Welcome to the thread   :).  Other shapes have been tried, for example, the Cannae drive:

(https://lh5.googleusercontent.com/-iuIRlN0RoQk/U93848CqEQI/AAAAAAAAzl8/YDUkix1Omos/cannae_drive.jpg)

see:  http://www.libertariannews.org/wp-content/uploads/2014/07/AnomalousThrustProductionFromanRFTestDevice-BradyEtAl.pdf

Please note that instead of air molecules vibrating as sound waves inside a musical instrument in the 10^3 Hz range, the EM Drive has photons oscillating as electromagnetic waves in the 10^9 Hz microwave frequency, hence the mode shapes are quite different than for musical instruments.
Title: Re: EM Drive Developments - related to space flight applications - Thread 2
Post by: squid on 05/19/2015 06:02 pm
Ultimately what Shawyer's and White's theories amount to, is that there is a radiation pressure imbalance on the inside of the cavity, resulting in a net force, which is a non small fraction of the total radiation pressure on the inside of the cavity (more than 1%). Shawyer says it is in accordance with Maxwell's equations, which is flat out wrong.

Where is Shawyer wrong?

1) Is the cutoff wavelength different at the small and big ends or not?

2) Is the guide wavelength different at the small and big ends or not?

3) Is the group velocity different at the small and big ends or not?

4) Is the bounce force different at the small and big ends or not?

5) Is the bounce force at the big end greater than at the small end or not?

6) Is there a bounce force on the side walls or not?

Just trying to understand where you believe Shawyer is wrong?

1-2-3) It really doesn't make sense to talk about "group velocity", "cutoff wavelength" for different "ends" in this case. A cavity is not a waveguide. Introducing end plates (whether curved or flat) changes the boundary conditions and thus the mode structure of the EM fields. There is just one cutoff wavelength for the entire cavity, much like an organ pipe has one "fundamental" tone.

Please understand that Egan's calculation of the fields is an exact solution of Maxwell's equations. Shawyer's talk about waveguides is at best a sloppy approximation.

4-5-6) There are forces on all surfaces of the cavity. Again referring to Egan's website, in the standard formulation of Maxwell's equations, for any closed cavity supporting time-harmonic fields, the integral of forces on the walls vanishes. To suggest Maxwell's equations say otherwise is just bad physics.

Generically, a question I asked many many pages ago, and that everyone has danced around: if the EM drive works, we must have a non-classical coupling of electromagnetic fields to something else. Why has this coupling never been observed before? What is special about a copper cone?
Title: Re: EM Drive Developments - related to space flight applications - Thread 2
Post by: squid on 05/19/2015 06:18 pm
Incidentally, here is my (modest) proposal for an experiment to prove or disprove if there is anything going on here:

1. Place a well-insulated (or otherwise thermally massive) EM drive on a linkage that causes it to move back and forth such that the cavity experiences sinusoidally varying acceleration.
2. Monitor the resonant frequency (with a VNA or spectrum analyzer).
3. See if there are any changes in center frequency with the same period as the acceleration.

Care should be taken to ensure that strain in cables or connectors does not induce spurious shifts in resonant frequencies, but by basically doing a form of lock-in measurement you can massively reduce any source of noise or interference.

This would be thousands of times more accurate than trying to measure minuscule forces and trying to rule out the classical explanations for them.
Title: Re: EM Drive Developments - related to space flight applications - Thread 2
Post by: txdrive on 05/19/2015 06:22 pm
Ultimately what Shawyer's and White's theories amount to, is that there is a radiation pressure imbalance on the inside of the cavity, resulting in a net force, which is a non small fraction of the total radiation pressure on the inside of the cavity (more than 1%). Shawyer says it is in accordance with Maxwell's equations, which is flat out wrong.

Where is Shawyer wrong?

1) Is the cutoff wavelength different at the small and big ends or not?

2) Is the guide wavelength different at the small and big ends or not?

3) Is the group velocity different at the small and big ends or not?

4) Is the bounce force different at the small and big ends or not?

5) Is the bounce force at the big end greater than at the small end or not?

6) Is there a bounce force on the side walls or not?

Just trying to understand where you believe Shawyer is wrong?

1-2-3) It really doesn't make sense to talk about "group velocity", "cutoff wavelength" for different "ends" in this case. A cavity is not a waveguide. Introducing end plates (whether curved or flat) changes the boundary conditions and thus the mode structure of the EM fields. There is just one cutoff wavelength for the entire cavity, much like an organ pipe has one "fundamental" tone.

Please understand that Egan's calculation of the fields is an exact solution of Maxwell's equations. Shawyer's talk about waveguides is at best a sloppy approximation.

4-5-6) There are forces on all surfaces of the cavity. Again referring to Egan's website, in the standard formulation of Maxwell's equations, for any closed cavity supporting time-harmonic fields, the integral of forces on the walls vanishes. To suggest Maxwell's equations say otherwise is just bad physics.

Generically, a question I asked many many pages ago, and that everyone has danced around: if the EM drive works, we must have a non-classical coupling of electromagnetic fields to something else. Why has this coupling never been observed before? What is special about a copper cone?
To clarify your point, electromagnetic fields can be measured directly, without measuring tiny forces. A page back I posted a rough calculation: to get >50uN from 50W pumped into a cavity with a Q of ~7000 , the microwaves at the surface of the cavity must deviate from the accepted solution by at least ~2.5% . (With Cannae drive giving similar results, the cavity shape is clearly not very important).

Needless to say we did successfully detect - electrically - effects trillions times weaker than that.
Title: Re: EM Drive Developments - related to space flight applications - Thread 2
Post by: TheTraveller on 05/19/2015 06:30 pm
Ultimately what Shawyer's and White's theories amount to, is that there is a radiation pressure imbalance on the inside of the cavity, resulting in a net force, which is a non small fraction of the total radiation pressure on the inside of the cavity (more than 1%). Shawyer says it is in accordance with Maxwell's equations, which is flat out wrong.

Where is Shawyer wrong?

1) Is the cutoff wavelength different at the small and big ends or not?

2) Is the guide wavelength different at the small and big ends or not?

3) Is the group velocity different at the small and big ends or not?

4) Is the bounce force different at the small and big ends or not?

5) Is the bounce force at the big end greater than at the small end or not?

6) Is there a bounce force on the side walls or not?

Just trying to understand where you believe Shawyer is wrong?

1-2-3) It really doesn't make sense to talk about "group velocity", "cutoff wavelength" for different "ends" in this case. A cavity is not a waveguide. Introducing end plates (whether curved or flat) changes the boundary conditions and thus the mode structure of the EM fields. There is just one cutoff wavelength for the entire cavity, much like an organ pipe has one "fundamental" tone.

Please understand that Egan's calculation of the fields is an exact solution of Maxwell's equations. Shawyer's talk about waveguides is at best a sloppy approximation.

4-5-6) There are forces on all surfaces of the cavity. Again referring to Egan's website, in the standard formulation of Maxwell's equations, for any closed cavity supporting time-harmonic fields, the integral of forces on the walls vanishes. To suggest Maxwell's equations say otherwise is just bad physics.

Generically, a question I asked many many pages ago, and that everyone has danced around: if the EM drive works, we must have a non-classical coupling of electromagnetic fields to something else. Why has this coupling never been observed before? What is special about a copper cone?

Both Shawyer & the Chinese claim their many physical devices produce thrust and the measurement of that thrust is in agreement with their theoretical calculations.

Both claim no new physics is needed and CofE / CofM are conserved.

Both use a different approach to calculate the thrust, yet come up with the same result.

I mean you say it can't work as they claim, yet it does and the measured thrust from many devices, calculated in different ways, measured in different ways, in different labs, in different countries all closely matched what their theory says the thrust should be.

With respect, just maybe your explanation / understanding of what is happening inside the frustum is not at the same level as Shawyer or the Chinese?
Title: Re: EM Drive Developments - related to space flight applications - Thread 2
Post by: Rodal on 05/19/2015 06:35 pm
The number of photons (N) inside a cavity is not conserved. A photon may collide with an electron on a wall, exciting it to a higher energy state, removing a photon. This electron may drop back to its lower level in a series of steps, each one of which releases an individual photon back into the cavity.  Although the sum of the energies of the emitted photons are the same as the absorbed photon, the number of emitted photons will vary.

The absorption (emission) of one photon might be accompanied by the emission (absorption) of more than one photon, as long as the frequency of the photons involved are such that the energy of the system remains constant.

N over volume, and the entropy density are proportional to the cube of the temperature of the photon gas, while the pressure and the energy density are proportional to the fourth power of its temperature. 

For black body radiation it can be shown that, as a result of this lack of constraint on the number of photons in the system, the chemical potential of the photons must be zero.

How does the Entropy/Information Bound Work?  http://arxiv.org/pdf/quant-ph/0404042

One point to keep in mind is that the change from 2GHz to ambient thermal radiation is an up-conversion, that is, it represents a negative entropy change component that must be more than made up somewhere else.

Quote
As an example we consider the Casimir effect with different temperatures between the plates (T) resp. outside of them (T′). For T′ < T the pressure of heat radiation can eventually compensate the Casimir force and the total pressure can vanish...If both T and T′ are fixed (isothermal case), this equilibrium has turned out unstable.
Thermodynamics of the Casimir effect  http://arxiv.org/pdf/quant-ph/9902074v1
Title: Re: EM Drive Developments - related to space flight applications - Thread 2
Post by: phaseshift on 05/19/2015 06:40 pm
A thought experiment:

Many physicists postulate that we live in a 3+1 deSitter space, and specifically I refer to Lisa Randall, Raman Sundrum http://arxiv.org/abs/hep-ph/9905221.  Briefly they claim an additional finite dimension between our brane (weak/Tev) and a gravity brane (strong) where gravity actually exists.  The distance between these branes is the reason that gravity is such a comparatively weak force.  The length of this 4th spacial dimension has an upper limit of somewhat less than 1 millimeter (otherwise we would have seen its effects long ago). The force of gravity on the gravity brane is expected to be 16 orders of magnitude greater than it is on our brane.

Is it possible that RF energy, specifically microwaves, is able to inflate this 4th dimension causing the gravity brane to be farther from our brane and thus weakening the gravitation force, effectively warping space, or warping it less than it usually is. Light passing through such a region would be blue shifted, or to put it another way gravitational redshifting would be lessened. I'll call this localized volume of warped space a warp field for my thought experiment.

Such an effect would not produce thrust. However, it would magnify thrust from another source (increased entropy).

To take this thought experiment a little farther.  IF the RF energy inflates this finite dimension then it is open to longer wavelength RF energy. The shorter wavelengths are in effect prying open the dimension and then by using longer wavelengths it can be further inflated.

Where in the frustrum is this possibly taking place? In the cavity proper itself, at the small end plate, large end plate, or perhaps at each end plate? If it is occurring at each end plate then there are two warp fields , each with possibly a different magnitude. How large are the fields and do they overlap? Can the presence of two warp fields with different magnitudes produce thrust?

My math skills are not adequate to pursue this, 1st year Calculus 35 years ago is a distant memory.
Title: Re: EM Drive Developments - related to space flight applications - Thread 2
Post by: txdrive on 05/19/2015 06:50 pm
Ultimately what Shawyer's and White's theories amount to, is that there is a radiation pressure imbalance on the inside of the cavity, resulting in a net force, which is a non small fraction of the total radiation pressure on the inside of the cavity (more than 1%). Shawyer says it is in accordance with Maxwell's equations, which is flat out wrong.

Where is Shawyer wrong?

1) Is the cutoff wavelength different at the small and big ends or not?

2) Is the guide wavelength different at the small and big ends or not?

3) Is the group velocity different at the small and big ends or not?

4) Is the bounce force different at the small and big ends or not?

5) Is the bounce force at the big end greater than at the small end or not?

6) Is there a bounce force on the side walls or not?

Just trying to understand where you believe Shawyer is wrong?
Well, firstly, Shawyer has greater radiation pressure upon the wide end, but his drive is pushing narrow end forwards.

Secondarily, yes, there is an interaction between the EM field and the walls, which results in a force on the side walls, equal to change in momentum of the EM field travelling down it, per time.

Maxwell has a cosine factor to adjust for waves that hit the bounce surface at an angle. Max force at the end plates and min force at the walls as attached.
Nope. Maxwell's equations are field equations and have nothing about waves hitting anything at an angle. They also work for a solenoid plugged into your household AC.
Quote

CofM requires if the EM waves pushes more on the on the big end than the small end, the EM Drive frustum pushes back in the opposite direction of the imbalance. Momentum transfer is not one sided. If the EM Drive moved toward the big end, then CofE would be violated as only a one sided push.

I think you're confused with regards to how forces and reaction works...

Let's say we got a real actual pressure imbalance, for example, by putting a flashlight inside the cavity, pointed at the wide end (which is not perfectly reflective). The flashlight itself is magnetically levitated from outside the cavity, so it stays in place. This will cause more radiation pressure upon the wide end, which will make it move wide end forward (with the same force as the force propelling the flashlight as a photon rocket, but in opposite direction). If held in place, the frustum will be applying two forces: one on the radiation, wide end to narrow end, other on the mounts, narrow end to the wide end (as when pushing wide end forward).

With regards to the alleged agreement between experiments and theory, EagleWorks did also report "agreement" somehow in spite of the fact that the theory predicts that you should have the same thrust when you turn your drive around 180 degrees, and they don't.
Title: Re: EM Drive Developments - related to space flight applications - Thread 2
Post by: squid on 05/19/2015 06:51 pm

Both Shawyer & the Chinese claim their many physical devices produce thrust and the measurement of that thrust is in agreement with their theoretical calculations. Both also claim no new physics is needed and CofE / CofM are conserved.

I mean you say it can't work as they claim, yet it does and the measured thrust from many devices, measured in different ways, in different labs, in different countries all closely matched what their theory says the thrust should be.

With respect, just maybe your explanation / understanding of what is happening inside the frustum is not at the same level as Shawyer or the Chinese?

Or perhaps they are all making basic mistakes in their sloppy experimental setups, pumping kW of microwave power into poorly shielded cavities and reporting thrusts near their error limits?

This is all standard physics, supported by a century of experiments all conducted with far more precision and rigor than anything published on the EM drive.

Incidentally, after doing a brief literature search, I have attached an experiment performed in the early 90's on a superconducting frustrum cavity, with a Q of at least 20,000. The paper is nice in that it gives explicit formulae for the EM fields in such a cavity. Their classical model fits the data perfectly. I might also note that they didn't see the thing shoot out of their dewar...
Title: Re: EM Drive Developments - related to space flight applications - Thread 2
Post by: SeeShells on 05/19/2015 06:53 pm

We could pull it behind my 65 Pontiac Catalina. :) NASA has already proven that a 421 powered Pontiac will make things fly.

http://www.dfrc.nasa.gov/Gallery/Movie/M2-F1/HTML/EM-0020-02.html
I thought my brain was being warped at the Super Conductor Super Collider, but this is a lot of fun. Love the pool of thoughts and ideas here it keeps me young and thinking. 
If you use solid copper sheeting for the body of the EM device and that seems to be because of it's thermal and electrical conductivity, couldn't you use a different extruded metal? Extruded Brass comes to mind, 90% copper and 10% tin. Or steel? Unless someone can provide me with as reason why I should just use copper sheeting. Yes, I'm thinking about building one too. I have a 2500 sq ft shop with all kind of toys for metal and electronics left over from my business. I'm not the kind to just throw it together and there is a lot of research yet to be done. And there will be shielding!
Title: Re: EM Drive Developments - related to space flight applications - Thread 2
Post by: TheTraveller on 05/19/2015 07:09 pm

We could pull it behind my 65 Pontiac Catalina. :) NASA has already proven that a 421 powered Pontiac will make things fly.

http://www.dfrc.nasa.gov/Gallery/Movie/M2-F1/HTML/EM-0020-02.html
I thought my brain was being warped at the Super Conductor Super Collider, but this is a lot of fun. Love the pool of thoughts and ideas here it keeps me young and thinking. 
If you use solid copper sheeting for the body of the EM device and that seems to be because of it's thermal and electrical conductivity, couldn't you use a different extruded metal? Extruded Brass comes to mind, 90% copper and 10% tin. Or steel? Unless someone can provide me with as reason why I should just use copper sheeting. Yes, I'm thinking about building one too. I have a 2500 sq ft shop with all kind of toys for metal and electronics left over from my business. I'm not the kind to just throw it together and there is a lot of research yet to be done. And there will be shielding!

Shawyer's 1st maggie powered test unit, Faraday Cage and the balance beam system. Not rocket science. Max thrust was 16mN at 850W input. Not much but with care, measurable.

http://emdrive.com/feasibilitystudy.html
Title: Re: EM Drive Developments - related to space flight applications - Thread 2
Post by: phaseshift on 05/19/2015 07:10 pm


If you use solid copper sheeting for the body of the EM device and that seems to be because of it's thermal and electrical conductivity, couldn't you use a different extruded metal? Extruded Brass comes to mind, 90% copper and 10% tin. Or steel? Unless someone can provide me with as reason why I should just use copper sheeting. Yes, I'm thinking about building one too.

I think weight is a consideration, so I've crossed off steel.  Also copper can not be anodized with silver, though there may be other techniques that can plate it.  Silver will raise the Q factor as I understand it.  A copper/aluminum alloy can be anodized with silver, as can aluminum itself.

For these reasons I've settled on aluminum until I find reason to pick something else.

Also 3D printing allows for additional weight savings, some of the parts can have internal cavities.  3D printing can be done at a resolution of 20 microns.
Title: Re: EM Drive Developments - related to space flight applications - Thread 2
Post by: TheTraveller on 05/19/2015 07:20 pm

Both Shawyer & the Chinese claim their many physical devices produce thrust and the measurement of that thrust is in agreement with their theoretical calculations. Both also claim no new physics is needed and CofE / CofM are conserved.

I mean you say it can't work as they claim, yet it does and the measured thrust from many devices, measured in different ways, in different labs, in different countries all closely matched what their theory says the thrust should be.

With respect, just maybe your explanation / understanding of what is happening inside the frustum is not at the same level as Shawyer or the Chinese?

Or perhaps they are all making basic mistakes in their sloppy experimental setups, pumping kW of microwave power into poorly shielded cavities and reporting thrusts near their error limits?

This is all standard physics, supported by a century of experiments all conducted with far more precision and rigor than anything published on the EM drive.

Incidentally, after doing a brief literature search, I have attached an experiment performed in the early 90's on a superconducting frustrum cavity, with a Q of at least 20,000. The paper is nice in that it gives explicit formulae for the EM fields in such a cavity. Their classical model fits the data perfectly. I might also note that they didn't see the thing shoot out of their dewar...

Guess you have not read the Chinese data:

http://www.emdrive.com/NWPU2010translation.pdf
http://www.emdrive.com/NWPU2010testresults.pdf
http://www.emdrive.com/yang-juan-paper-2012.pdf

The thrust measured was not the EagleWorks mosquito landing on your arm level. Maybe read the papers before claiming

Quote
Or perhaps they are all making basic mistakes in their sloppy experimental setups, pumping kW of microwave power into poorly shielded cavities and reporting thrusts near their error limits?

as a quick way to dismiss their results.
Title: Re: EM Drive Developments - related to space flight applications - Thread 2
Post by: SeeShells on 05/19/2015 07:22 pm


If you use solid copper sheeting for the body of the EM device and that seems to be because of it's thermal and electrical conductivity, couldn't you use a different extruded metal? Extruded Brass comes to mind, 90% copper and 10% tin. Or steel? Unless someone can provide me with as reason why I should just use copper sheeting. Yes, I'm thinking about building one too.

I think weight is a consideration, so I've crossed off steel.  Also copper can not be anodized with silver, though there may be other techniques that can plate it.  Silver will raise the Q factor as I understand it.  A copper/aluminum alloy can be anodized with silver, as can aluminum itself.

For these reasons I've settled on aluminum until I find reason to pick something else.

Also 3D printing allows for additional weight savings, some of the parts can have internal cavities.  3D printing can be done at a resolution of 20 microns.
20 microns is 0.0008 of an inch and quite impressive (I built and designed XYZand T semiconductor machines with an positional accuracy of .25 um across 300mm) Is the price dropped low enough vs the real advantages of just using digital micrometers and good machining practices? To me nothing is really set in stone in the cavity designs and it seems like every day some new revelation pops up. Has 3D printing (with metals or conductors) dropped that low? I at least want to built it somewhat modular and that can allow design changes.
Title: Re: EM Drive Developments - related to space flight applications - Thread 2
Post by: phaseshift on 05/19/2015 07:26 pm
A thought experiment:

Many physicists postulate that we live in a 3+1 deSitter space, and specifically I refer to Lisa Randall, Raman Sundrum http://arxiv.org/abs/hep-ph/9905221.  Briefly they claim an additional finite dimension between our brane (weak/Tev) and a gravity brane (strong) where gravity actually exists.  The distance between these branes is the reason that gravity is such a comparatively weak force.  The length of this 4th spacial dimension has an upper limit of somewhat less than 1 millimeter (otherwise we would have seen its effects long ago). The force of gravity on the gravity brane is expected to be 16 orders of magnitude greater than it is on our brane....

For whatever it's worth, a short note that Dr. White and Paul March both have invoked a (4+1) brane of spacetime in their explanation and papers.

Yes, I noticed that. Though I didn't follow why they did so, it seemed disconnected to me. I thought they referred to a 3+1 space, but if they used 4+1 then they included time. I'm more inclined to use 3+1 and set time aside.
Title: Re: EM Drive Developments - related to space flight applications - Thread 2
Post by: LasJayhawk on 05/19/2015 07:34 pm


If you use solid copper sheeting for the body of the EM device and that seems to be because of it's thermal and electrical conductivity, couldn't you use a different extruded metal? Extruded Brass comes to mind, 90% copper and 10% tin. Or steel? Unless someone can provide me with as reason why I should just use copper sheeting. Yes, I'm thinking about building one too.

I think weight is a consideration, so I've crossed off steel.  Also copper can not be anodized with silver, though there may be other techniques that can plate it.  Silver will raise the Q factor as I understand it.  A copper/aluminum alloy can be anodized with silver, as can aluminum itself.

For these reasons I've settled on aluminum until I find reason to pick something else.

Also 3D printing allows for additional weight savings, some of the parts can have internal cavities.  3D printing can be done at a resolution of 20 microns.

Both brass and copper take to electroplating with silver nicely and would provide a higher Q than bare copper.


Keep in mind that any metal can be plated with the right combination. A chrome car bumper was steel, plated with copper, plated with nickel, plated with chrome, the nickel won't plate  very well directly to steel...
Title: Re: EM Drive Developments - related to space flight applications - Thread 2
Post by: phaseshift on 05/19/2015 07:35 pm


If you use solid copper sheeting for the body of the EM device and that seems to be because of it's thermal and electrical conductivity, couldn't you use a different extruded metal? Extruded Brass comes to mind, 90% copper and 10% tin. Or steel? Unless someone can provide me with as reason why I should just use copper sheeting. Yes, I'm thinking about building one too.

I think weight is a consideration, so I've crossed off steel.  Also copper can not be anodized with silver, though there may be other techniques that can plate it.  Silver will raise the Q factor as I understand it.  A copper/aluminum alloy can be anodized with silver, as can aluminum itself.

For these reasons I've settled on aluminum until I find reason to pick something else.

Also 3D printing allows for additional weight savings, some of the parts can have internal cavities.  3D printing can be done at a resolution of 20 microns.
20 microns is 0.0008 of an inch and quite impressive (I built and designed XYZand T semiconductor machines with an positional accuracy of .25 um across 300mm) Is the price dropped low enough vs the real advantages of just using digital micrometers and good machining practices? To me nothing is really set in stone in the cavity designs and it seems like every day some new revelation pops up. Has 3D printing (with metals or conductors) dropped that low? I at least want to built it somewhat modular and that can allow design changes.

One off prices are still pretty high, though you can pick your resolution, lower res, lower cost. 20 microns is probably way more precise than needed.  The lower the res the shorter the print time and therefore the cost. The reason I'm looking at 3D printing is that I don't know how to machine a spherically concave surface. I've polished glass plates for telescope mirrors, but that's not helpful ;)  I'm not a machinist and it may actually be easy - but considering there is a local fablab I am happy to pay for a one off and not be bothered with having to figure out how to machine it.
Title: Re: EM Drive Developments - related to space flight applications - Thread 2
Post by: rfcavity on 05/19/2015 07:44 pm

Both Shawyer & the Chinese claim their many physical devices produce thrust and the measurement of that thrust is in agreement with their theoretical calculations. Both also claim no new physics is needed and CofE / CofM are conserved.

I mean you say it can't work as they claim, yet it does and the measured thrust from many devices, measured in different ways, in different labs, in different countries all closely matched what their theory says the thrust should be.

With respect, just maybe your explanation / understanding of what is happening inside the frustum is not at the same level as Shawyer or the Chinese?

Or perhaps they are all making basic mistakes in their sloppy experimental setups, pumping kW of microwave power into poorly shielded cavities and reporting thrusts near their error limits?

This is all standard physics, supported by a century of experiments all conducted with far more precision and rigor than anything published on the EM drive.

Incidentally, after doing a brief literature search, I have attached an experiment performed in the early 90's on a superconducting frustrum cavity, with a Q of at least 20,000. The paper is nice in that it gives explicit formulae for the EM fields in such a cavity. Their classical model fits the data perfectly. I might also note that they didn't see the thing shoot out of their dewar...

Guess you have not read the Chinese data:

http://www.emdrive.com/NWPU2010translation.pdf
http://www.emdrive.com/NWPU2010testresults.pdf
http://www.emdrive.com/yang-juan-paper-2012.pdf

The thrust measured was not the EagleWorks mosquito landing on your arm level. Maybe read the papers before claiming

Quote
Or perhaps they are all making basic mistakes in their sloppy experimental setups, pumping kW of microwave power into poorly shielded cavities and reporting thrusts near their error limits?

as a quick way to dismiss their results.

A.single paper is not a magic wand which causes all previous results to disappear. It cannot chan ge the tens of thousands of previous measure ments that have taken place in the last 50years. How do you refute cavities in pillbox shape that are used in GPS satellite atomic clocks? These have been characterized down to sub nanowatt levels, and no mystery power draw is observed and no thrust is observed in the GPS satellites (the location of which must be known very well.)
Title: Re: EM Drive Developments - related to space flight applications - Thread 2
Post by: deltaMass on 05/19/2015 07:48 pm
About building your own cavity - it would be helpful to know how this is supposed to scale with frequency. Since known physics predicts that the device doesn't work, that scaling law is not to hand. Otherwise, one might have a reason to build a light frequency device that fits in your hand. But one doesn't know. So, as TheTraveller points out, the only reasonable thing to do is a replication of an apparatus that is claimed to produce thrust. So either Shawyer's or Juan's.
Title: Re: EM Drive Developments - related to space flight applications - Thread 2
Post by: phaseshift on 05/19/2015 07:49 pm


If you use solid copper sheeting for the body of the EM device and that seems to be because of it's thermal and electrical conductivity, couldn't you use a different extruded metal? Extruded Brass comes to mind, 90% copper and 10% tin. Or steel? Unless someone can provide me with as reason why I should just use copper sheeting. Yes, I'm thinking about building one too.

I think weight is a consideration, so I've crossed off steel.  Also copper can not be anodized with silver, though there may be other techniques that can plate it.  Silver will raise the Q factor as I understand it.  A copper/aluminum alloy can be anodized with silver, as can aluminum itself.

For these reasons I've settled on aluminum until I find reason to pick something else.

Also 3D printing allows for additional weight savings, some of the parts can have internal cavities.  3D printing can be done at a resolution of 20 microns.

Both brass and copper take to electroplating with silver nicely and would provide a higher Q than bare copper.


Keep in mind that any metal can be plated with the right combination. A chrome car bumper was steel, plated with copper, plated with nickel, plated with chrome, the nickel won't plate  very well directly to steel...

Yes, electroplating is the way to go (there is so much to learn here) - aluminum still seems the best choice for its lower weight.  I see that copper can also be 3D printed: http://3dprint.com/59881/nasa-3d-prints-copper-rocket/.
Title: Re: EM Drive Developments - related to space flight applications - Thread 2
Post by: SeeShells on 05/19/2015 07:53 pm
A thought experiment:

Many physicists postulate that we live in a 3+1 deSitter space, and specifically I refer to Lisa Randall, Raman Sundrum http://arxiv.org/abs/hep-ph/9905221. 

I have Warped Passages by Lisa Randall, I need to get it out and re-read it, she is one of my heroes and if I believe (need to look again) she finds quasicrystals might have an underlying structure into other dimensions. Whoa! Ok need to sit in my hot tub and think some.
Title: Re: EM Drive Developments - related to space flight applications - Thread 2
Post by: phaseshift on 05/19/2015 08:04 pm
A thought experiment:

Many physicists postulate that we live in a 3+1 deSitter space, and specifically I refer to Lisa Randall, Raman Sundrum http://arxiv.org/abs/hep-ph/9905221. 

I have Warped Passages by Lisa Randall, I need to get it out and re-read it, she is one of my heroes and if I believe (need to look again) she finds quasicrystals might have an underlying structure into other dimensions. Whoa! Ok need to sit in my hot tub and think some.

I have all of her books (4 I think, or is it 3?).  In Warped Passages I swear I remember her mentioning the possibility of inflating the 4sd (4th spatial dimension) - though there was no known mechanism.  Just like there is no known physical mechanism for the  inflationary period of the Universe.
Title: Re: EM Drive Developments - related to space flight applications - Thread 2
Post by: TheTraveller on 05/19/2015 08:12 pm
Both of the SPR Flight Thrusters, build under contract with Boeing, may have been built from existing microwave horn antenna stock.

Compare the 1st attachment and this rectangular horn antenna:
http://www.pasternack.com/standard-gain-horn-waveguide-size-wr159-10-db-gain-sma-female-pe9860sf-10-p.aspx

Have also found circular horn antenna that may be close. Still looking. If I can find a stock cone horn antenna that closely fits the dimensions, we may be able to get the exact internal dimensions.
Title: Re: EM Drive Developments - related to space flight applications - Thread 2
Post by: LasJayhawk on 05/19/2015 08:15 pm


If you use solid copper sheeting for the body of the EM device and that seems to be because of it's thermal and electrical conductivity, couldn't you use a different extruded metal? Extruded Brass comes to mind, 90% copper and 10% tin. Or steel? Unless someone can provide me with as reason why I should just use copper sheeting. Yes, I'm thinking about building one too.

I think weight is a consideration, so I've crossed off steel.  Also copper can not be anodized with silver, though there may be other techniques that can plate it.  Silver will raise the Q factor as I understand it.  A copper/aluminum alloy can be anodized with silver, as can aluminum itself.

For these reasons I've settled on aluminum until I find reason to pick something else.

Also 3D printing allows for additional weight savings, some of the parts can have internal cavities.  3D printing can be done at a resolution of 20 microns.

Both brass and copper take to electroplating with silver nicely and would provide a higher Q than bare copper.


Keep in mind that any metal can be plated with the right combination. A chrome car bumper was steel, plated with copper, plated with nickel, plated with chrome, the nickel won't plate  very well directly to steel...

Yes, electroplating is the way to go (there is so much to learn here) - aluminum still seems the best choice for its lower weight.  I see that copper can also be 3D printed: http://3dprint.com/59881/nasa-3d-prints-copper-rocket/.

You can plate aluminum with copper or nickel then silver plate. I think there are some tricks (read extra steps) to get the first plate to stick to the aluminum base.
Title: Re: EM Drive Developments - related to space flight applications - Thread 2
Post by: Stormbringer on 05/19/2015 08:20 pm
How about extrapolating/interpolating downward by calculating dimensions from resonant frequencies of these test articles? Use that interpolation to select a smaller sized set of cavity dimensions and scale up the frequencies to get that thrust signal out of the "iffy" zone?
Title: Re: EM Drive Developments - related to space flight applications - Thread 2
Post by: deltaMass on 05/19/2015 08:25 pm
How about extrapolating/interpolating downward by calculating dimensions from resonant frequencies of these test articles? Use that interpolation to select a smaller sized set of cavity dimensions and scale up the frequencies to get that thrust signal out of the "iffy" zone?
The point I was making was that we don't know whether or not higher frequencies produce higher thrust. That's unless you happen to know.
Title: Re: EM Drive Developments - related to space flight applications - Thread 2
Post by: rthrfrd on 05/19/2015 08:29 pm
Hi all, lurker here,

Sadly I have no theoretical contribution to make, however as more people attempt to implement the device I hope I can help with the collaboration:

I've created a rough model of the device (with no idea of actual ratios/dimensions - if anyone has some I can quickly update) in a fairly new tool called Onshape (https://www.onshape.com/) (you can sign up for free):

https://cad.onshape.com/documents/17ece60a89f74af4b788d7c3/w/2f14c5d1fcd84a14ab67b3c2 (https://cad.onshape.com/documents/17ece60a89f74af4b788d7c3/w/2f14c5d1fcd84a14ab67b3c2)

Bit of background: It's an online, collaborative parametric modelling tool (think SolidWorks). The feature set naturally isn't comparable yet, but for the purposes of prototyping it's pretty intuitive, and free ;D The best thing about being web-based is anyone can fork (make their own version of) my document... and others can make versions of theirs... and so on, so my hope is it's a way that people (particularly those without access to this kind of tool) could refine engineering prototypes together.

For those of you not familiar: the 3D model is built from a series of 2D sketches and extrusions. If you constrain your dimensions correctly (as I've tried to do!) then you can quickly make changes to the dimensions to test different permutations of the design. If you've never used this kind of software before (I'm no expert), there are some nice video guides which can help you quickly get stuck in.

When you look to actually fabricate a prototype you can export to various common formats: https://cad.onshape.com/help/index.htm#cshid=export

I've just done a quick draft of the device itself - you could equally extend to model the whole experimental setup.

Disclaimer: I am in no way associated with Onshape! Just thought it was interesting compared to what's out there :)
Title: Re: EM Drive Developments - related to space flight applications - Thread 2
Post by: TheTraveller on 05/19/2015 08:37 pm

Both Shawyer & the Chinese claim their many physical devices produce thrust and the measurement of that thrust is in agreement with their theoretical calculations. Both also claim no new physics is needed and CofE / CofM are conserved.

I mean you say it can't work as they claim, yet it does and the measured thrust from many devices, measured in different ways, in different labs, in different countries all closely matched what their theory says the thrust should be.

With respect, just maybe your explanation / understanding of what is happening inside the frustum is not at the same level as Shawyer or the Chinese?

Or perhaps they are all making basic mistakes in their sloppy experimental setups, pumping kW of microwave power into poorly shielded cavities and reporting thrusts near their error limits?

This is all standard physics, supported by a century of experiments all conducted with far more precision and rigor than anything published on the EM drive.

Incidentally, after doing a brief literature search, I have attached an experiment performed in the early 90's on a superconducting frustrum cavity, with a Q of at least 20,000. The paper is nice in that it gives explicit formulae for the EM fields in such a cavity. Their classical model fits the data perfectly. I might also note that they didn't see the thing shoot out of their dewar...

Guess you have not read the Chinese data:

http://www.emdrive.com/NWPU2010translation.pdf
http://www.emdrive.com/NWPU2010testresults.pdf
http://www.emdrive.com/yang-juan-paper-2012.pdf

The thrust measured was not the EagleWorks mosquito landing on your arm level. Maybe read the papers before claiming

Quote
Or perhaps they are all making basic mistakes in their sloppy experimental setups, pumping kW of microwave power into poorly shielded cavities and reporting thrusts near their error limits?

as a quick way to dismiss their results.

A.single paper is not a magic wand which causes all previous results to disappear. It cannot chan ge the tens of thousands of previous measure ments that have taken place in the last 50years. How do you refute cavities in pillbox shape that are used in GPS satellite atomic clocks? These have been characterized down to sub nanowatt levels, and no mystery power draw is observed and no thrust is observed in the GPS satellites (the location of which must be known very well.)

I don't refute the past nor existing devices as you should not. However that does not say they are all that is possible or that we know all possible variations of the theories thus embodied.

There is not a single paper, there are many.

Before signing a license deal with SPR, involving both the US and UK governments, Boeing would have crawled all over the SPR, all their devices, test rigs and especially the Demonstrator Engine and its static and dynamic test rigs. As part of that license deal, SPR built, tested and shipped the Flight Thruster to Boeing. SPR's claims for the results of the Flight Thruster are well known to Boeing. Never heard Boeing claim the Flight Thruster did not meet contract conditions.

BTW on the EagleWorks slide showing the various SPR devices, the Flight Thruster is labelled as a "High Fidelity Test Article". As the slide is from NASA. I'm sure SPR did not write that on the slide.

Just maybe something is happening that is inside the existing theories, yet largely unrealised. Both the Chinese and SPR state no new physics is needed and both CofE and CofM are conserved. Their theory math supports their claims and as well the theory math predicts the thrust they measured in 3 different ways.
Title: Re: EM Drive Developments - related to space flight applications - Thread 2
Post by: TheTraveller on 05/19/2015 08:50 pm
How about extrapolating/interpolating downward by calculating dimensions from resonant frequencies of these test articles? Use that interpolation to select a smaller sized set of cavity dimensions and scale up the frequencies to get that thrust signal out of the "iffy" zone?
The point I was making was that we don't know whether or not higher frequencies produce higher thrust. That's unless you happen to know.

Shawyer states:

F = (2 * Df * Po * Q) / c (Po = applied power in watts)

Many factors at play. Df hits max when the small end operates at just above cutoff and then drops as frequency climbs. The applied Rf frequency needs to achieve resonance at some 1/2 wave of the cavity electrical length between the end plates. So to a large degree, the length of the frustum sets a number of 1/2 multiple input Rf frequencies.

Haven't seen anything that implies you will get more thrust with higher frequencies.

The attachment from Dr. Rodal is a good resource.
Title: Re: EM Drive Developments - related to space flight applications - Thread 2
Post by: TheTraveller on 05/19/2015 09:01 pm
How about extrapolating/interpolating downward by calculating dimensions from resonant frequencies of these test articles? Use that interpolation to select a smaller sized set of cavity dimensions and scale up the frequencies to get that thrust signal out of the "iffy" zone?

The only testers that had "Iffy" zone signals were EagleWorks and their devices fitted with dielectrics.

Shawyer did use dielectrics at one time, and has patents covering their use, but gave them up and advised others here to do likewise.

Shawyer's 1st device achieved 16mN at 850W, the Demonstrator Engine rotary test unit achieved 96mN at 334W and the Flight Thruster achieved 175mN at 430W. Well out of any "Iffy" zone. At 175 mN or 17.9 gf you could feel that with your hand if the power were turned off and on.
Title: Re: EM Drive Developments - related to space flight applications - Thread 2
Post by: deltaMass on 05/19/2015 09:24 pm
It should be pointed out that this Shawyer thrust equation
1) F = 2 Df Po Q / c
has been rejected by the physics community. Their version is
2) F = 0.

Let's however go with it. Assuming that we do a decent job with Df and so get Df = DfMax = 1. Then the equation predicts that you get 2*Q times the thrust you'd get from a photon rocket of the same power.
But since a cavity's Q = w E / P, where w=angular frequency, E=stored energy, P=input power, Shawyer's thrust equation becomes
3) F = (2 E/c) w
and thus thrust scales linearly with frequency, and linearly with the stored energy.

Let's imagine we have a little 150 mW laser diode attached to a resonant cavity, powered by a tiny battery.
The whole thing weighs probably about 50 gm (SWAG).
That's light enough and small enough to fit on a Mettler H20 balance, an inspired piece of Swiss mechanical engineering that reliably gets you 10 microgram-weight resolution, or 0.1 microNewtons.
Equation 1) with Df=1 predicts a thrust of Q/1000 microNewtons.
Thus the Mettler should detect thrust for Q > 100.
This is usually far exceeded by optical cavities, which can have Qs up in the millions.

So this experiment is expected to easily detect thrust from this little device.
Indeed, if Q > 5*108, it would lift off the bench when appropriately oriented.
If Shawyer is correct, that is.


Title: Re: EM Drive Developments - related to space flight applications - Thread 2
Post by: SeeShells on 05/19/2015 09:42 pm
A thought experiment:

Many physicists postulate that we live in a 3+1 deSitter space, and specifically I refer to Lisa Randall, Raman Sundrum http://arxiv.org/abs/hep-ph/9905221. 

I have Warped Passages by Lisa Randall, I need to get it out and re-read it, she is one of my heroes and if I believe (need to look again) she finds quasicrystals might have an underlying structure into other dimensions. Whoa! Ok need to sit in my hot tub and think some.

I have all of her books (4 I think, or is it 3?).  In Warped Passages I swear I remember her mentioning the possibility of inflating the 4sd (4th spatial dimension) - though there was no known mechanism.  Just like there is no known physical mechanism for the  inflationary period of the Universe.
Knocking on Heaven's Door and another about the Higgs.  Think it's 3.
It is somewhat light but meant for a specific audience, but she likes the string theory (I always liked it) and if you can write and make sense from our perspective another dimension without higher order math, my hat is off.
I keep on associating (several papers and books) again and again to magneto-chiral matter and how it interacts to the Casimir force and  momentum, there seems a wonderful link there. "I'M GIVIN' HER ALL SHE'S GOT, CAPTAIN!" Need more brains!
Title: Re: EM Drive Developments - related to space flight applications - Thread 2
Post by: TheTraveller on 05/19/2015 09:50 pm
It should be pointed out that this Shawyer thrust equation
1) F = 2 Df Po Q / c
has been rejected by the physics community. Their version is
2) F = 0.

Let's however go with it. Assuming that we do a decent job with Df and so get Df = DfMax = 1. Then the equation predicts that you get 2*Q times the thrust you'd get from a photon rocket of the same power.
But since a cavity's Q = w E / P, where w=angular frequency, E=stored energy, P=input power, Shawyer's thrust equation becomes
3) F = (2 E/c) w
and thus thrust scales linearly with frequency, and linearly with the stored energy.

Let's imagine we have a little 150 mW laser diode attached to a resonant cavity, powered by a tiny battery.
The whole thing weighs probably about 50 gm (SWAG).
That's light enough and small enough to fit on a Mettler H20 balance, an inspired piece of Swiss mechanical engineering that reliably gets you 10 microgram-weight resolution, or 0.1 microNewtons.
Equation 1) with Df=1 predicts a thrust of Q/1000 microNewtons.
Thus the Mettler should detect thrust for Q > 100.
This is usually far exceeded by optical cavities, which can have Qs up in the millions.

So this experiment is expected to easily detect thrust from this little device.
Indeed, if Q > 5*108, it would lift off the bench when appropriately oriented.
If Shawyer is correct, that is.

Of course frequency has an effect on thrust. Dumb statement for me to make as I only considered Df in a fixed frustum. Was thinking about my Flight Thruster design simulator, which did have a fixed Q of 50,000 at 3.85GHz but not anymore.

Thanks for that input. "Click" another piece of the puzzle drops into place.
Title: Re: EM Drive Developments - related to space flight applications - Thread 2
Post by: pogsquog on 05/19/2015 09:55 pm

Let's imagine we have a little 150 mW laser diode attached to a resonant cavity, powered by a tiny battery.
The whole thing weighs probably about 50 gm (SWAG).
...

So this experiment is expected to easily detect thrust from this little device.
Indeed, if Q > 5*108, it would lift off the bench when appropriately oriented.
If Shawyer is correct, that is.

I suspect it is quite difficult to build something with the required properties in laser wavelengths.
Title: Re: EM Drive Developments - related to space flight applications - Thread 2
Post by: WarpTech on 05/19/2015 09:56 pm
A thought experiment:

Many physicists postulate that we live in a 3+1 deSitter space, and specifically I refer to Lisa Randall, Raman Sundrum http://arxiv.org/abs/hep-ph/9905221.  Briefly they claim an additional finite dimension between our brane (weak/Tev) and a gravity brane (strong) where gravity actually exists.  The distance between these branes is the reason that gravity is such a comparatively weak force.  The length of this 4th spacial dimension has an upper limit of somewhat less than 1 millimeter (otherwise we would have seen its effects long ago). The force of gravity on the gravity brane is expected to be 16 orders of magnitude greater than it is on our brane....

For whatever it's worth, a short note that Dr. White and Paul March both have invoked a (4+1) brane of spacetime in their explanation and papers.

Yes, I noticed that. Though I didn't follow why they did so, it seemed disconnected to me. I thought they referred to a 3+1 space, but if they used 4+1 then they included time. I'm more inclined to use 3+1 and set time aside.

Gravity doesn't work the way they imagine and there is no evidence that higher dimensions actually exist, or that they have any influence on our 3 + 1 dimensions. However, gravity does present itself as a slight change in "scale" at the sub-atomic scale. Scale can be formulated as a 5th dimension that is dependent on the other 4, as in Kaluza-Klien theory, which has nothing to do with branes BTW. IMO, the whole of physics has been falling down a rabbit hole to nowhere, ever since String theory was created.

Todd
Title: Re: EM Drive Developments - related to space flight applications - Thread 2
Post by: txdrive on 05/19/2015 09:57 pm

Let's imagine we have a little 150 mW laser diode attached to a resonant cavity, powered by a tiny battery.
The whole thing weighs probably about 50 gm (SWAG).
...

So this experiment is expected to easily detect thrust from this little device.
Indeed, if Q > 5*108, it would lift off the bench when appropriately oriented.
If Shawyer is correct, that is.

I suspect it is quite difficult to build something with the required properties in laser wavelengths.
Make a laser diode of appropriate shape with no exit. edit: fibre lasers could be of interest as well. You can make tapered fibre.
Title: Re: EM Drive Developments - related to space flight applications - Thread 2
Post by: pogsquog on 05/19/2015 10:05 pm
Shawyer states that the cavity reflectors must be half a wavelength apart, which might be a bit fiddly at laser frequencies.
Title: Re: EM Drive Developments - related to space flight applications - Thread 2
Post by: Rodal on 05/19/2015 10:09 pm
...

Of course frequency has an effect on thrust. Dumb statement for me to make as I only considered Df in a fixed frustum. Was thinking about my Flight Thruster design simulator, which did have a fixed Q of 50,000 at 3.85GHz but not anymore.

Thanks for that input. "Click" another piece of the puzzle drops into place.

It should be pointed out that this Shawyer thrust equation
1) F = 2 Df Po Q / c
has been rejected by the physics community. Their version is
2) F = 0.

Let's however go with it. Assuming that we do a decent job with Df and so get Df = DfMax = 1. Then the equation predicts that you get 2*Q times the thrust you'd get from a photon rocket of the same power.
But since a cavity's Q = w E / P, where w=angular frequency, E=stored energy, P=input power, Shawyer's thrust equation becomes
3) F = (2 E/c) w
and thus thrust scales linearly with frequency, and linearly with the stored energy.

Let's imagine we have a little 150 mW laser diode attached to a resonant cavity, powered by a tiny battery.
The whole thing weighs probably about 50 gm (SWAG).
That's light enough and small enough to fit on a Mettler H20 balance, an inspired piece of Swiss mechanical engineering that reliably gets you 10 microgram-weight resolution, or 0.1 microNewtons.
Equation 1) with Df=1 predicts a thrust of Q/1000 microNewtons.
Thus the Mettler should detect thrust for Q > 100.
This is usually far exceeded by optical cavities, which can have Qs up in the millions.

So this experiment is expected to easily detect thrust from this little device.
Indeed, if Q > 5*108, it would lift off the bench when appropriately oriented.
If Shawyer is correct, that is.
To make F frequency (w) dependent you had to perform a variable substitution.
E is dependent on Q, so nothing is gained by that variable substitution.
You (and us) have a sense for what Q is, while we don't have a sense of what is E (without knowing Q).


There  is no advantage to use w*E instead of using Q*Po , as you yourself implicitly admitted when the time came  for you to estimate F (you calculated it in terms of Q*Po, instead of w*E)

You used:

F = 2 Po Q / c  for different Q values (Q=100 and Q=5*10^8)

which is not explicitly dependent on frequency (given Q),  all you had to use was Po, and Q: you never had to use the frequency.


Title: Re: EM Drive Developments - related to space flight applications - Thread 2
Post by: deltaMass on 05/19/2015 10:21 pm
Indeed you are correct, because eqn 3 suppresses the possible frequency dependence of E.

So let's go with thrust not being frequency dependent. That means that far higher thrust-to-weight ratios can be expected at higher frequencies.

And in that vein, I like the idea of a fully self-contained tapered fibre laser.
It would weigh next to nothing - the battery would be most of the weight.
Title: Re: EM Drive Developments - related to space flight applications - Thread 2
Post by: TheTraveller on 05/19/2015 10:26 pm
Shawyer states that the cavity reflectors must be half a wavelength apart, which might be a bit fiddly at laser frequencies.

Even tricker still as the internal guide wavelengths vary from end to end and are not at the applied Rf wavelength.

Have asked Shawyer how to calc frustum resonance wavelength as per applied Rf wavelength.
Title: Re: EM Drive Developments - related to space flight applications - Thread 2
Post by: Rodal on 05/19/2015 10:26 pm
Indeed you are correct, because eqn 3 suppresses the possible frequency dependence of E.

So let's go with thrust not being frequency dependent. That means that far higher thrust-to-weight ratios can be expected at higher frequencies.

And in that vein, I like the idea of a fully self-contained tapered fibre laser.
In any vibration problem one can express Q as 2*Pi*f*EnergyStored/PowerLoss (it can be a mechanical vibration).
The usual and practical way to asses the damping of vibration is to use Q (or the damping ratio = 1/(2Q), instead of using the Energy Stored.  That's why when it came time for you to come up with numbers you had to use Q*Po, instead of w*E
Title: Re: EM Drive Developments - related to space flight applications - Thread 2
Post by: TheTraveller on 05/19/2015 10:29 pm
Indeed you are correct, because eqn 3 suppresses the possible frequency dependence of E.

So let's go with thrust not being frequency dependent. That means that far higher thrust-to-weight ratios can be expected at higher frequencies.

And in that vein, I like the idea of a fully self-contained tapered fibre laser.
It would weigh next to nothing - the battery would be most of the weight.

Well it is frequency dependent as a set frustum Df is frequency dependent.
Title: Re: EM Drive Developments - related to space flight applications - Thread 2
Post by: Rodal on 05/19/2015 10:34 pm
Q is a primary dimensionless parameter that compares the exponential time constant τ for decay of an oscillating physical system's amplitude to its oscillation period. E (the energy stored) is not dimensionless, and it is not a primary parameter.
Title: Re: EM Drive Developments - related to space flight applications - Thread 2
Post by: deltaMass on 05/19/2015 10:40 pm
Of course in practice we have a P(w) dependence based on the fatness of your wallet. Doing it at 2 GHz means you can cheaply get kilowatts. Doing it at 22 GHz with a Gunn diode will get you only a few milliwatts.  And again, doing it at light frequencies also means milliwatts unless you want to spend mucho bucks.

All this is somewhat offset by higher Qs at higher frequencies.

But at the risk of labouring the point, much superior thrust-to-weight ratios can be had with lasers, where the weight is the all-up weight of everything including the batteries.
Title: Re: EM Drive Developments - related to space flight applications - Thread 2
Post by: WarpTech on 05/19/2015 10:56 pm
It should be pointed out that this Shawyer thrust equation
1) F = 2 Df Po Q / c
has been rejected by the physics community. Their version is
2) F = 0.

Let's however go with it. Assuming that we do a decent job with Df and so get Df = DfMax = 1. Then the equation predicts that you get 2*Q times the thrust you'd get from a photon rocket of the same power.
But since a cavity's Q = w E / P, where w=angular frequency, E=stored energy, P=input power, Shawyer's thrust equation becomes
3) F = (2 E/c) w
and thus thrust scales linearly with frequency, and linearly with the stored energy.

Let's imagine we have a little 150 mW laser diode attached to a resonant cavity, powered by a tiny battery.
The whole thing weighs probably about 50 gm (SWAG).
That's light enough and small enough to fit on a Mettler H20 balance, an inspired piece of Swiss mechanical engineering that reliably gets you 10 microgram-weight resolution, or 0.1 microNewtons.
Equation 1) with Df=1 predicts a thrust of Q/1000 microNewtons.
Thus the Mettler should detect thrust for Q > 100.
This is usually far exceeded by optical cavities, which can have Qs up in the millions.

So this experiment is expected to easily detect thrust from this little device.
Indeed, if Q > 5*108, it would lift off the bench when appropriately oriented.
If Shawyer is correct, that is.

More accurately, F = (2 Df Po Q / c) * D, where D is the Duty Cycle of the output. IF it were to put out thrust continuously at this value of F, CoE would be violated, because you are only putting in Po*t, not Q*Po*t. It takes time to store energy, and once it is stored, it can only deliver thrust for a limited amount of time before it needs to be recharged again. So there is a duty cycle associated with this thing that is being ignored.

Personally, I do not find Shawyer's equation to be that far off. I agree, his theory is flawed if you consider only perfectly conducting walls and group velocity. But, given asymmetrical losses, his assumption that F2 - F1 > 0 is exactly what it is. Two forces that are not exactly equal, opposing each other. The "how and why" are debatable, but the reality of it is not. Had someone done a Buckingham Pie Theory analysis of this, just based on input variables, the Max. potential thrust, 2*Q*P/c multiplied by an unknown Df based on the geometry, and whose value is to be experimentally determined, is exactly what you should get.

Todd D.

 
Title: Re: EM Drive Developments - related to space flight applications - Thread 2
Post by: TheTraveller on 05/19/2015 11:07 pm
It should be pointed out that this Shawyer thrust equation
1) F = 2 Df Po Q / c
has been rejected by the physics community. Their version is
2) F = 0.

Let's however go with it. Assuming that we do a decent job with Df and so get Df = DfMax = 1. Then the equation predicts that you get 2*Q times the thrust you'd get from a photon rocket of the same power.
But since a cavity's Q = w E / P, where w=angular frequency, E=stored energy, P=input power, Shawyer's thrust equation becomes
3) F = (2 E/c) w
and thus thrust scales linearly with frequency, and linearly with the stored energy.

Let's imagine we have a little 150 mW laser diode attached to a resonant cavity, powered by a tiny battery.
The whole thing weighs probably about 50 gm (SWAG).
That's light enough and small enough to fit on a Mettler H20 balance, an inspired piece of Swiss mechanical engineering that reliably gets you 10 microgram-weight resolution, or 0.1 microNewtons.
Equation 1) with Df=1 predicts a thrust of Q/1000 microNewtons.
Thus the Mettler should detect thrust for Q > 100.
This is usually far exceeded by optical cavities, which can have Qs up in the millions.

So this experiment is expected to easily detect thrust from this little device.
Indeed, if Q > 5*108, it would lift off the bench when appropriately oriented.
If Shawyer is correct, that is.

More accurately, F = (2 Df Po Q / c) * D, where D is the Duty Cycle of the output. IF it were to put out thrust continuously at this value of F, CoE would be violated, because you are only putting in Po*t, not Q*Po*t. It takes time to store energy, and once it is stored, it can only deliver thrust for a limited amount of time before it needs to be recharged again. So there is a duty cycle associated with this thing that is being ignored.

Personally, I do not find Shawyer's equation to be that far off. I agree, his theory is flawed if you consider only perfectly conducting walls and group velocity. But, given asymmetrical losses, his assumption that F2 - F1 > 0 is exactly what it is. Two forces that are not exactly equal, opposing each other. The "how and why" are debatable, but the reality of it is not. Had someone done a Buckingham Pie Theory analysis of this, just based on input variables, the Max. potential thrust, 2*Q*P/c multiplied by an unknown Df based on the geometry, and whose value is to be experimentally determined, is exactly what you should get.

Todd D.

It has not been ignored.

Sgawyer has discussed the TC and time to recharge the cavity energy lost to kinetic.

His superconducting space plane uses 8 EM Drives, arranged like 2 side by side 4 cylinder inline motors, driven in short pulses of less than 1 TC and phased apart to deliver continuous thrust.
Title: Re: EM Drive Developments - related to space flight applications - Thread 2
Post by: Rodal on 05/19/2015 11:10 pm
...Personally, I do not find Shawyer's equation to be that far off. I agree, his theory is flawed if you consider only perfectly conducting walls and group velocity. But, given asymmetrical losses, his assumption that F2 - F1 > 0 is exactly what it is. Two forces that are not exactly equal, opposing each other. The "how and why" are debatable, but the reality of it is not. Had someone done a Buckingham Pie Theory analysis of this, just based on input variables, the Max. potential thrust, 2*Q*P/c multiplied by an unknown Df based on the geometry, and whose value is to be experimentally determined, is exactly what you should get.
Todd D.
Right on !
Actually, there are three expressions, independently derived, with different physical models:

Shawyer
McCulloch
Notsosureofit

With similar dependence on P and Q

F = (P*Q / c ) * parameter

and of course, the expression P/c is in common with a photon rocket.

The three expressions just differ on the value of the parameter.  Concerning a  Buckingham Pie Theory Dimensionless analysis, this was done in EM Drive Thread 1, where frobnicat wrote a computer program to compare hundreds of equations to the experimental data, and it turns out that this form of expression (common to Shawyer, McCulloch and Notsosureofit) was a natural outcome of the analysis.

For the skeptic in me, P is directly related to heat, the surprising parameter is Q which is not related to heat.
Title: Re: EM Drive Developments - related to space flight applications - Thread 2
Post by: rfcavity on 05/19/2015 11:14 pm

Both Shawyer & the Chinese claim their many physical devices produce thrust and the measurement of that thrust is in agreement with their theoretical calculations. Both also claim no new physics is needed and CofE / CofM are conserved.

I mean you say it can't work as they claim, yet it does and the measured thrust from many devices, measured in different ways, in different labs, in different countries all closely matched what their theory says the thrust should be.

With respect, just maybe your explanation / understanding of what is happening inside the frustum is not at the same level as Shawyer or the Chinese?

Or perhaps they are all making basic mistakes in their sloppy experimental setups, pumping kW of microwave power into poorly shielded cavities and reporting thrusts near their error limits?

This is all standard physics, supported by a century of experiments all conducted with far more precision and rigor than anything published on the EM drive.

Incidentally, after doing a brief literature search, I have attached an experiment performed in the early 90's on a superconducting frustrum cavity, with a Q of at least 20,000. The paper is nice in that it gives explicit formulae for the EM fields in such a cavity. Their classical model fits the data perfectly. I might also note that they didn't see the thing shoot out of their dewar...

Guess you have not read the Chinese data:

http://www.emdrive.com/NWPU2010translation.pdf
http://www.emdrive.com/NWPU2010testresults.pdf
http://www.emdrive.com/yang-juan-paper-2012.pdf

The thrust measured was not the EagleWorks mosquito landing on your arm level. Maybe read the papers before claiming

Quote
Or perhaps they are all making basic mistakes in their sloppy experimental setups, pumping kW of microwave power into poorly shielded cavities and reporting thrusts near their error limits?

as a quick way to dismiss their results.

A.single paper is not a magic wand which causes all previous results to disappear. It cannot chan ge the tens of thousands of previous measure ments that have taken place in the last 50years. How do you refute cavities in pillbox shape that are used in GPS satellite atomic clocks? These have been characterized down to sub nanowatt levels, and no mystery power draw is observed and no thrust is observed in the GPS satellites (the location of which must be known very well.)

I don't refute the past nor existing devices as you should not. However that does not say they are all that is possible or that we know all possible variations of the theories thus embodied.

There is not a single paper, there are many.

Before signing a license deal with SPR, involving both the US and UK governments, Boeing would have crawled all over the SPR, all their devices, test rigs and especially the Demonstrator Engine and its static and dynamic test rigs. As part of that license deal, SPR built, tested and shipped the Flight Thruster to Boeing. SPR's claims for the results of the Flight Thruster are well known to Boeing. Never heard Boeing claim the Flight Thruster did not meet contract conditions.

BTW on the EagleWorks slide showing the various SPR devices, the Flight Thruster is labelled as a "High Fidelity Test Article". As the slide is from NASA. I'm sure SPR did not write that on the slide.

Just maybe something is happening that is inside the existing theories, yet largely unrealised. Both the Chinese and SPR state no new physics is needed and both CofE and CofM are conserved. Their theory math supports their claims and as well the theory math predicts the thrust they measured in 3 different ways.

It doesn't matter what Boeing or SPR does, they can't change physics.

Any portion of an atomic clock has been characterized to a degree many magnitudes greater than any of these test setups demonstrate. Just ask NIST. Unexpected results that have a divergence of the magnitude the papers you posted have claimed would have been identified during their development.

If you so strongly believe that these are happening, the timing on the GPS satellites have no correction for the effects observed from Eagleworks and the Chinese lab. Therefore, you should not use GPS receivers or trust any computer time that receives its time from NIST (basically most computers connected to the internet) until this 'new physics' is characterized. Because it would have a huge effect on those calculations.
Title: Re: EM Drive Developments - related to space flight applications - Thread 2
Post by: WarpTech on 05/19/2015 11:14 pm
...
It has not been ignored.

Sgawyer has discussed the TC and time to recharge the cavity energy lost to kinetic.

His superconducting space plane uses 8 EM Drives, arranged like 2 side by side 4 cylinder inline motors, driven in short pulses of less than 1 TC and phased apart to deliver continuous thrust.

If the thruster had 2 compartments, a cylinder, where resonance was easy to establish at high Q, and a long frustum designed for maximum attenuation connected at one end of the cylinder. Between the two, there is a "shutter" that can rapidly open and close. When closed, the cylinder resonates as a cylinder. When open, energy expands into the frustum chamber where it is attenuated. After the shutter closes again, the energy in frustum attenuates and energy in cylinder recharges.... repeat. I keep looking for ways to decouple the resonant amplifier from the attenuator.

Todd
Title: Re: EM Drive Developments - related to space flight applications - Thread 2
Post by: Notsosureofit on 05/19/2015 11:29 pm

Any portion of an atomic clock has been characterized to a degree many magnitudes greater than any of these test setups demonstrate. Just ask NIST. Unexpected results that have a divergence of the magnitude the papers you posted have claimed would have been identified during their development.

If you so strongly believe that these are happening, the timing on the GPS satellites have no correction for the effects observed from Eagleworks and the Chinese lab. Therefore, you should not use GPS receivers or trust any computer time that receives its time from NIST (basically most computers connected to the internet) until this 'new physics' is characterized. Because it would have a huge effect on those calculations.

The "Notsosureofit" formula mentioned above, was derived from, and is consistent with, the behavior of clocks in a gravitational field (ie accelerating frame of reference).  Whether the force is zero or not remains to be seen.

I suppose I should add that these dispersion effects are well known at optical wavelengths and have been used a anti-reflection surfaces both by varying the dielectric constant and by nano-sized conical resonators.  (probably the cone receptors in the eye as well but I'm no biologist)
Title: Re: EM Drive Developments - related to space flight applications - Thread 2
Post by: deltaMass on 05/19/2015 11:42 pm
And in other news, our Roumanian pal Iulian has gone silent.
https://www.youtube.com/watch?v=Rbf7735o3hQ
http://www.masinaelectrica.com/emdrive-independent-test/
Title: Re: EM Drive Developments - related to space flight applications - Thread 2
Post by: Rodal on 05/19/2015 11:48 pm
And in other news, our Roumanian pal Iulian has gone silent.
...
Iulian is a shining example to us all (myself included) as Iulian works silently and humbly and posts only when he has something new to report  :)

_____
Seriously speaking, I hope that Iulian is OK, and is getting the necessary, safe, protection
Title: Re: EM Drive Developments - related to space flight applications - Thread 2
Post by: snow on 05/20/2015 12:00 am
And in other news, our Roumanian pal Iulian has gone silent.
...
Iulian is a shining example to us all (myself included) as Iulian works silently and humbly and posts only when he has something new to report  :)

he could have hurt himself. he was using no protection and was asked to rotate the thing a few times.
Title: Re: EM Drive Developments - related to space flight applications - Thread 2
Post by: LasJayhawk on 05/20/2015 12:00 am
The cesium beam standards on the GPS satellites runs slow in space as compared to earth, and is corrected for.
Title: Re: EM Drive Developments - related to space flight applications - Thread 2
Post by: deltaMass on 05/20/2015 12:15 am
Which is to say that you've missed the point being made about GPS systems.
Title: Re: EM Drive Developments - related to space flight applications - Thread 2
Post by: deltaMass on 05/20/2015 12:19 am
And in other news, our Roumanian pal Iulian has gone silent.
...
Iulian is a shining example to us all (myself included) as Iulian works silently and humbly and posts only when he has something new to report  :)

he could have hurt himself. he was using no protection and was asked to rotate the thing a few times.
Just to disambiguate that, if I may - he was asked a few times to rotate it to an upside down orientation once.
Title: Re: EM Drive Developments - related to space flight applications - Thread 2
Post by: VAXHeadroom on 05/20/2015 12:31 am
I was wondering if we would be able to get a faster turnaround with prototyping and maybe even cut costs if we could use 3D printing and e.g. conductive graphene filament like this one:


We could share computer designs and use local 3D printing services. Would that work?

That's what I've been planning on doing.  Makes construction a great deal simpler for some of the pieces. I'm getting the bottom plate printed in aluminum and I will drill and tap the bolt holes myself.  This makes it much easier to create the spherical concave surface.

Probably more useful to use ABS or PLA and make forms over which to construct other cavities for testing.
 - been lurking for a few days and reading a LOT, you finally asked a question about which I have some expertise :)
Title: Re: EM Drive Developments - related to space flight applications - Thread 2
Post by: LasJayhawk on 05/20/2015 12:47 am
Which is to say that you've missed the point being made about GPS systems.

No, not really. We have to correct for relativistic effects, but still have a margin of error.

Fun fact: the GPS satellites all gave gamma ray detectors on them.
Title: Re: EM Drive Developments - related to space flight applications - Thread 2
Post by: deltaMass on 05/20/2015 01:01 am
Which is to say that you've missed the point being made about GPS systems.

No, not really. We have to correct for relativistic effects, but still have a margin of error.

Fun fact: the GPS satellites all gave gamma ray detectors on them.

I mean this

Quote from: rfcavity
It doesn't matter what Boeing or SPR does, they can't change physics.

Any portion of an atomic clock has been characterized to a degree many magnitudes greater than any of these test setups demonstrate. Just ask NIST. Unexpected results that have a divergence of the magnitude the papers you posted have claimed would have been identified during their development.

If you so strongly believe that these are happening, the timing on the GPS satellites have no correction for the effects observed from Eagleworks and the Chinese lab. Therefore, you should not use GPS receivers or trust any computer time that receives its time from NIST (basically most computers connected to the internet) until this 'new physics' is characterized. Because it would have a huge effect on those calculations.
Title: Re: EM Drive Developments - related to space flight applications - Thread 2
Post by: Notsosureofit on 05/20/2015 01:07 am
Which is to say that you've missed the point being made about GPS systems.

No, not really. We have to correct for relativistic effects, but still have a margin of error.

Fun fact: the GPS satellites all gave gamma ray detectors on them.


Another fun fact:  if you go to the Wikipedia page for GPS satellites and look at the equation for delta(1/gamma) you might see some similarity to spherical cavity ends.

http://en.wikipedia.org/wiki/Error_analysis_for_the_Global_Positioning_System#Relativity
Title: Re: EM Drive Developments - related to space flight applications - Thread 2
Post by: Rodal on 05/20/2015 01:28 am
http://en.wikipedia.org/wiki/Global_Positioning_System#Spherical_cones

(http://upload.wikimedia.org/math/8/7/2/872c33b94e70e0096214d7af764576b5.png)

The solution space [x, y, z, b] can be seen as a four-dimensional geometric space. In that case each of the equations describes a spherical cone, with the cusp located at the satellite, and the base a sphere around the satellite. The receiver is at the intersection of four or more of such cones.



(http://upload.wikimedia.org/wikipedia/commons/5/54/Spherical_cone_lines.png)


Stereographic projection of a spherical cone's generating lines (red), parallels (green) and hypermeridians (blue). Due to conformal property of Stereographic Projection, the curves intersect each other orthogonally (in the yellow points) as in 4D. All curves are circles or straight lines. The generatrices and parallels generates a 3D dual cone. The hypermeridians generates a set of concentric spheres.
Title: Re: EM Drive Developments - related to space flight applications - Thread 2
Post by: phaseshift on 05/20/2015 01:35 am
This is a 3D model of the "Shawyer Demo".  I built it as close as I can figure that it has to be and examining the several photographs that have been shared here.  The dimensions are from published values.

rfFrequency=2.45*10^9;
cavityLength=0.345;
bigDiameter=0.28;
smallDiameter= 0.128853

power =  421 to 1200
Q = 45000

(measured force = 102.30 milliNewtons only reported for  421 watts, 243 milliNewtons/kW )

measured ForcePerPowerInput = 80 to 243
Force/PowerInput of a Photon Rocket =0.003337
measured ForcePerPowerInput to the one of a photon rocket =23,980 to 72,830

While this is a crude SketchUp model if anyone wants the model I'm happy to share it. 
Title: Re: EM Drive Developments - related to space flight applications - Thread 2
Post by: phaseshift on 05/20/2015 01:39 am
Both end plates are reversible - one side is flat the other spherical - for testing.
Title: Re: EM Drive Developments - related to space flight applications - Thread 2
Post by: Rodal on 05/20/2015 02:16 am
This is a 3D model of the "Shawyer Demo".  I built it as close as I can figure that it has to be and examining the several photographs that have been shared here.  The dimensions are from published values.

rfFrequency=2.45*10^9;
cavityLength=0.345;
bigDiameter=0.28;
smallDiameter= 0.128853

power =  421 to 1200
Q = 45000

(measured force = 102.30 milliNewtons only reported for  421 watts, 243 milliNewtons/kW )

measured ForcePerPowerInput = 80 to 243
Force/PowerInput of a Photon Rocket =0.003337
measured ForcePerPowerInput to the one of a photon rocket =23,980 to 72,830

While this is a crude SketchUp model if anyone wants the model I'm happy to share it.

That is a really impressive job!

I have recalculated the small diameter, using Shawyer's paper http://www.emdrive.com/IAC-08-C4-4-7.pdf, see page 7, where Shawyer states

Quote from: Shawyer
The engine was built to operate at 2.45 GHz, with a design factor of 0.844 and has measured Q of 45,000 for an overall diameter of 280 mm.
(Unfortunately, Shawyer does not provide the small diameter or the cavity length in his paper)

I have used this information

bigDiameter = 0.28 m;
f = 2.45*10^9 Hz;
cst = 1.7062895542683174;
cM = 299705000 m/s (speed of light in air);
Design Factor = 0.844,

and inverted the equation for the Design Factor (see: http://forum.nasaspaceflight.com/index.php?topic=36313.msg1374110#msg1374110 ) to obtain the following correct dimension:

small diameter = 0.09613 m

Therefore, the dimensions should be corrected as follows

rfFrequency=2.45*10^9;
cavityLength=0.345; (ESTIMATED from Photographs)
bigDiameter=0.28 m; (provided by Shawyer)
smallDiameter=  0.09613 m; (obtained from the Design Factor, bigDiameter and frequency provided by Shawyer)


Title: Re: EM Drive Developments - related to space flight applications - Thread 2
Post by: phaseshift on 05/20/2015 02:33 am
This is a 3D model of the "Shawyer Demo".  I built it as close as I can figure that it has to be and examining the several photographs that have been shared here.  The dimensions are from published values.

rfFrequency=2.45*10^9;
cavityLength=0.345;
bigDiameter=0.28;
smallDiameter= 0.128853

power =  421 to 1200
Q = 45000

(measured force = 102.30 milliNewtons only reported for  421 watts, 243 milliNewtons/kW )

measured ForcePerPowerInput = 80 to 243
Force/PowerInput of a Photon Rocket =0.003337
measured ForcePerPowerInput to the one of a photon rocket =23,980 to 72,830

While this is a crude SketchUp model if anyone wants the model I'm happy to share it.

That is a really impressive job!

I have recalculated the small diameter, using Shawyer's paper http://www.emdrive.com/IAC-08-C4-4-7.pdf, see page 7, where Shawyer states

Quote from: Shawyer
The engine was built to operate at 2.45 GHz, with a design factor of 0.844 and has measured Q of 45,000 for an overall diameter of 280 mm.
(Unfortunately, Shawyer does not provide the small diameter or the cavity length in his paper)

I have used this information

bigDiameter = 0.28 m;
f = 2.45*10^9 Hz;
cst = 1.7062895542683174;
cM = 299705000 m/s (speed of light in air);
Design Factor = 0.844,

and inverted the equation for the Design Factor (see: http://forum.nasaspaceflight.com/index.php?topic=36313.msg1374110#msg1374110 ) to obtain the following correct dimension:

small diameter = 0.09613 m

Therefore, the dimensions should be corrected as follows

rfFrequency=2.45*10^9;
cavityLength=0.345; (ESTIMATED from Photographs)
bigDiameter=0.28 m; (provided by Shawyer)
smallDiameter=  0.09613 m; (obtained from the Design Factor, bigDiameter and frequency provided by Shawyer)


I believe the dimensions I used were published and copied here to the forum some time ago - I don't recall the source and I'm sure it would take awhile to find the original post - here are the dimensions as posted though (I had copied to a file)

Notice that the force per power input reported by  Fearn, Zachar, Woodward & Wanser is several orders of magnitude lower than the "EM drives".  Actually it is barely (3.5 times higher) more than the force per power input of a photon rocket:

reported measurement ForcePerPowerInput (milliNewtons/kW)

(* Cannae Superconducting *)             761.9 to 952.4
(* Shawyer Demo *)                               80 to 243
(* Shawyer Experimental *)                   18.82
(* Brady c TE mode *)                             21.31
(* Brady a TM mode*)                               5.396
(* Brady b TM mode*)                               3.000
(*Fearn, Zachar, Woodward & Wanser*) 0.01176


lengths in meter
rfFrequency in 1/second (microwave frequency during test)
power in watts
force in milliNewtons
force per PowerInput in milliNewtons/kW
c= 299705000 m/s (speed of light in air)
c= 299792458 m/s (speed of light in vacuum) (for Cannae Superconducting)
(the difference between c in air compared to c in vacuum is negligible)


Note: SmallDiameter for Shawyer's EM Drives obtained from his reported ShawyerDesignFactor .


Force/PowerInput of a Photon Rocket = 1 / c


(* Cannae Superconducting *)
rfFrequency = 1.047*10^9;
cavityLength = 0.01+0.004+0.006+0.01 = 0.03;
bigDiameter =(22.86-2*(0.00430)) = 0.220;
smallDiameter = bigDiameter-2*0.01=0.200;

power =  10.5
Q = 1.1*(10^7)

measured force = 8 to 10
measured ForcePerPowerInput = 761.9 to 952.4
Force/PowerInput of a Photon Rocket =0.003336
measured ForcePerPowerInput to the one of a photon rocket = 228,400 to 285,500

(* Shawyer Experimental *)
rfFrequency=2.45*10^9;
cavityLength=0.156;
bigDiameter=0.16;
smallDiameter=0.127546;

power =  850   
Q = 5900

measured force = 16
measured ForcePerPowerInput = 18.82
Force/PowerInput of a Photon Rocket =0.003337
measured ForcePerPowerInput to the one of a photon rocket =5,640

(* Shawyer Demo *)
rfFrequency=2.45*10^9;
cavityLength=0.345;
bigDiameter=0.28;
smallDiameter= 0.128853

power =  421 to 1200
Q = 45000

(measured force = 102.30 milliNewtons only reported for  421 watts, 243 milliNewtons/kW )

measured ForcePerPowerInput = 80 to 243
Force/PowerInput of a Photon Rocket =0.003337
measured ForcePerPowerInput to the one of a photon rocket =23,980 to 72,830

All Brady cases have the following dimensions:

cavityLength=0.332;
bigDiameter=0.397;
smallDiameter=0.244;

(* Brady a TM mode*)
rfFrequency=1.9326*10^9;

power =   16.9 
Q = 7320

measured force =  0.0912
measured ForcePerPowerInput = 5.396
Force/PowerInput of a Photon Rocket =0.003337
measured ForcePerPowerInput to the one of a photon rocket =1,617.2

(* Brady b TM mode*)   
rfFrequency=1.9367*10^9;

power = 16.7
Q =  18100

measured force = 0.0501
measured ForcePerPowerInput = 3.000
Force/PowerInput of a Photon Rocket =0.003337
measured ForcePerPowerInput to the one of a photon rocket =899.12

  (* Brady c  TE mode *)
rfFrequency = 1.8804*10^9;

power = 2.6
Q = 22000

measured force = 0.05541
measured ForcePerPowerInput = 21.31
Force/PowerInput of a Photon Rocket =0.003337
measured ForcePerPowerInput to the one of a photon rocket =6,386.7

(* Fearn, Zachar, Woodward & Wanser*)
rfFrequency = 39,300;

power =  170

measured force = 0.002
measured ForcePerPowerInput = 0.01176
Force/PowerInput of a Photon Rocket =0.003337
measured ForcePerPowerInput to the one of a photon rocket = 3.526

-------------
As it is my intention to duplicate Shawyer's model first I need to go with his dimensions - though now I want to verify where these dimensions came from.
Title: Re: EM Drive Developments - related to space flight applications - Thread 2
Post by: phaseshift on 05/20/2015 02:38 am
If I knew the dimensions of the Chinese device, or perhaps the Boeing device, I would just as soon use them.  Now that I have a script that can render any model from parameters I can play around for a while and see what I like. :)
Title: Re: EM Drive Developments - related to space flight applications - Thread 2
Post by: phaseshift on 05/20/2015 02:50 am
Ha, that table was from you - "Here is a comparison of reported measurements for EM Drives and for the latest report by Fearn, Zachar, Woodward & Wanser."
Title: Re: EM Drive Developments - related to space flight applications - Thread 2
Post by: phaseshift on 05/20/2015 03:12 am
I will rewrite the script so that it uses the design factor and frequency.  Currently it takes sD, bD, and length. It will be more useful to use frequency, bD, and length - calculating sD appropriately. 

I'm concerned that if the original dimensions of the Shawyer Demo device were done by eye - how is that the small plate was 3 centimeters off - that seems a huge discrepancy.
Title: Re: EM Drive Developments - related to space flight applications - Thread 2
Post by: aero on 05/20/2015 03:42 am
I was one of a few people trying to guestimate the small diameter of Shawyer's Demo. Yes, it was done by eye, or using tools attempting to match the profile of the cavity, which was by eye. For the error in small diameter you should fine a corresponding error in height as the taper is easy to match and the big diameter is given. These estimates were made before we had looked into the design factor so that approach wasn't considered. Neither did we have a good handle on cut-off or guide frequency. Time marches on, and you have somewhat better information now.

I suggest you go with the design factor calculation and adjust the height accordingly using the taper and the large and small diameters to calculate height. That is, of course if the numbers you derive will fit within the cavity as illustrated by the photographs. That is a simple sanity check. Others may have different and better justified opinions.
Title: Re: EM Drive Developments - related to space flight applications - Thread 2
Post by: phaseshift on 05/20/2015 04:06 am
I was one of a few people trying to guestimate the small diameter of Shawyer's Demo. Yes, it was done by eye, or using tools attempting to match the profile of the cavity, which was by eye. For the error in small diameter you should fine a corresponding error in height as the taper is easy to match and the big diameter is given. These estimates were made before we had looked into the design factor so that approach wasn't considered. Neither did we have a good handle on cut-off or guide frequency. Time marches on, and you have somewhat better information now.

I suggest you go with the design factor calculation and adjust the height accordingly using the taper and the large and small diameters to calculate height. That is, of course if the numbers you derive will fit within the cavity as illustrated by the photographs. That is a simple sanity check. Others may have different and better justified opinions.

Agreed. That makes the most sense to me.  As this is a work in progress it can always change. :)
Title: Re: EM Drive Developments - related to space flight applications - Thread 2
Post by: deltaMass on 05/20/2015 05:49 am
Asymmetry 101.
Throw a ball in the +x direction, perfect reflection from the front wall to hit the rear wall and be absorbed.
Throw: -p to floor through feet
Bounce: +2p to front wall
Absorb: -p to rear wall
Sum of momenta = 0

Conclusion: asymmetry doesn't make floobie dust.
Title: Re: EM Drive Developments - related to space flight applications - Thread 2
Post by: RotoSequence on 05/20/2015 05:55 am
Asymmetry 101.
Throw a ball in the +x direction, perfect reflection from the front wall to hit the rear wall and be absorbed.
Throw: -p to floor through feet
Bounce: +2p to front wall
Absorb: -p to rear wall
Sum of momenta = 0

Conclusion: asymmetry doesn't make floobie dust.

That's why much of the conversation in these threads has been dedicated to nonreciprocal behaviors. The outcomes get a little more complicated when you introduce (apparent) non-time-reversible mechanics.
Title: Re: EM Drive Developments - related to space flight applications - Thread 2
Post by: TheTraveller on 05/20/2015 08:33 am
This is a 3D model of the "Shawyer Demo".  I built it as close as I can figure that it has to be and examining the several photographs that have been shared here.  The dimensions are from published values.

rfFrequency=2.45*10^9;
cavityLength=0.345;
bigDiameter=0.28;
smallDiameter= 0.128853

power =  421 to 1200
Q = 45000

(measured force = 102.30 milliNewtons only reported for  421 watts, 243 milliNewtons/kW )

measured ForcePerPowerInput = 80 to 243
Force/PowerInput of a Photon Rocket =0.003337
measured ForcePerPowerInput to the one of a photon rocket =23,980 to 72,830

While this is a crude SketchUp model if anyone wants the model I'm happy to share it.

Nice job but the drawing looks like the Flight Thruster which ran at 3.85GHz.
http://www.emdrive.com/flightprogramme.html

Attached are photos of the Demonstrator. It is unknown if it had curved end caps.
http://emdrive.com/demonstratorengine.html
http://emdrive.com/dynamictests.html
Title: Re: EM Drive Developments - related to space flight applications - Thread 2
Post by: TheTraveller on 05/20/2015 09:01 am
I'm working on a method to calculate end plate to end plate resonance from the external Rf wavelength, based on a continually variable internal guide wavelength between the 2 end plates. Have had advise this is the correct way to do it.

Once that is worked out, it becomes possible to determine end plate spacing, knowing applied Rf frequency & Df.
Title: Re: EM Drive Developments - related to space flight applications - Thread 2
Post by: snow on 05/20/2015 09:19 am
can someone explain in normal people terms were is the recoil going from turning on the microwave?
shouldn't the metal box push on the microwave just as much as the microwave pushes on the metal box?

also, it gets really hot... could the recoil just be delayed?
Title: Re: EM Drive Developments - related to space flight applications - Thread 2
Post by: RotoSequence on 05/20/2015 09:33 am
can someone explain in normal people terms were is the recoil going from turning on the microwave?
shouldn't the metal box push on the microwave just as much as the microwave pushes on the metal box?

also, it gets really hot... could the recoil just be delayed?

Classical mechanics and known physics both say that the energy should take the form of waste heat and extraneous electromagnetic energy (in other words, the EM drive shouldn't be doing anything of interest). At this point, assuming the recoil force is real, there is no definitively known mechanism of action.
Title: Re: EM Drive Developments - related to space flight applications - Thread 2
Post by: pogsquog on 05/20/2015 09:40 am
can someone explain in normal people terms were is the recoil going from turning on the microwave?
shouldn't the metal box push on the microwave just as much as the microwave pushes on the metal box?

also, it gets really hot... could the recoil just be delayed?

Shawyer's explanation is that the waves bounce back and forth across the device.
The speed of the waves depends upon the shape of the container (the container is acting as a wave-guide, which reduces the propagation speed to be less than C). Shawyer asserts that the momentum change is thereby asymmetric, due to the differing effective impact speeds. Note that the wider physics community do not agree with this analysis, although I do not understand the details.
However, there are other theories, in particular Dr.White at NASA thinks that the EM fields from the standing waves are interacting with quantum 'virtual particles', effectively pushing off these. There are problems to do with special relativity and conservation of energy that would seem to make this unlikely unless you go and revive some very old theories of the aether and absolute reference frames, which are very marginal (special relativity has been well tested and aether theories abandoned as unproductive).
Another theory that allows breaking of local conservation of momentum and which _is consistent with observations  is Woodward 'Mach' effects. These relate to accelerating bodies which are changing in internal energy levels. The momentum is effectively transferred to all other bodies in the universe, at the speed of light. Attempts to produce or measure Mach effects have thus far failed, but there are reasonable theoretic reasons to believe they might be possible (they help to explain inertia and provide a preferred reference frame which special relativity lacks). I don't think anyone is explaining the EM drive in terms of this, however.
Title: Re: EM Drive Developments - related to space flight applications - Thread 2
Post by: OttO on 05/20/2015 09:41 am
Hello
Layman question:
Could not the frustum be an analogue to an optical black hole laser?

http://extremelight.eps.hw.ac.uk/publications/CQG-optical_BH_laser-Faccio(2012).pdf (http://extremelight.eps.hw.ac.uk/publications/CQG-optical_BH_laser-Faccio(2012).pdf)

http://relativity.livingreviews.org/Articles/lrr-2011-3/download/lrr-2011-3BW.pdf (http://relativity.livingreviews.org/Articles/lrr-2011-3/download/lrr-2011-3BW.pdf)
Title: Re: EM Drive Developments - related to space flight applications - Thread 2
Post by: Rodal on 05/20/2015 11:33 am
I have recalculated the small diameter, for Shawyer's EXPERIMENTAL THRUSTER using Shawyer's paper http://www.emdrive.com/IAC-08-C4-4-7.pdf, see page 6, where Shawyer states

Quote from: Shawyer
A 160 mm diameter experimental thruster, operating at 2.45 GHz was designed and built. (see fig 6) The design factor, calculated from as-built measurements of the thruster geometry
was 0.497. An unloaded Q of 5,900 was measured. [/b].
(Unfortunately, Shawyer does not provide the small diameter or the cavity length in his paper)

I have used this information

bigDiameter = 0.16 m;
f = 2.45*10^9 Hz;
cst = 1.7062895542683174;
cM = 299705000 m/s (speed of light in air);
Design Factor = 0.497,

and inverted the equation for the Design Factor (see: http://forum.nasaspaceflight.com/index.php?topic=36313.msg1374110#msg1374110 ) to obtain the following correct dimension:

small diameter = 0.1025 m SHAWYER's EXPERIMENTAL THRUSTER

(obtained from the Design Factor, bigDiameter and frequency provided by Shawyer)

I have updated the table in http://forum.nasaspaceflight.com/index.php?topic=36313.msg1302455#msg1302455 , originally dated 5 months ago, with his information.
Title: Re: EM Drive Developments - related to space flight applications - Thread 2
Post by: Star One on 05/20/2015 11:33 am

can someone explain in normal people terms were is the recoil going from turning on the microwave?
shouldn't the metal box push on the microwave just as much as the microwave pushes on the metal box?

also, it gets really hot... could the recoil just be delayed?

Shawyer's explanation is that the waves bounce back and forth across the device.
The speed of the waves depends upon the shape of the container (the container is acting as a wave-guide, which reduces the propagation speed to be less than C). Shawyer asserts that the momentum change is thereby asymmetric, due to the differing effective impact speeds. Note that the wider physics community do not agree with this analysis, although I do not understand the details.
However, there are other theories, in particular Dr.White at NASA thinks that the EM fields from the standing waves are interacting with quantum 'virtual particles', effectively pushing off these. There are problems to do with special relativity and conservation of energy that would seem to make this unlikely unless you go and revive some very old theories of the aether and absolute reference frames, which are very marginal (special relativity has been well tested and aether theories abandoned as unproductive).
Another theory that allows breaking of local conservation of momentum and which _is consistent with observations  is Woodward 'Mach' effects. These relate to accelerating bodies which are changing in internal energy levels. The momentum is effectively transferred to all other bodies in the universe, at the speed of light. Attempts to produce or measure Mach effects have thus far failed, but there are reasonable theoretic reasons to believe they might be possible (they help to explain inertia and provide a preferred reference frame which special relativity lacks). I don't think anyone is explaining the EM drive in terms of this, however.

To your last point I though they were, I first came across this whole business on Talkpolywell & last I looked that was the explanation being moved forward there by some posters.
Title: Re: EM Drive Developments - related to space flight applications - Thread 2
Post by: TheTraveller on 05/20/2015 11:51 am

I have recalculated the small diameter, for Shawyer's EXPERIMENTAL THRUSTER using Shawyer's paper http://www.emdrive.com/IAC-08-C4-4-7.pdf, see page 6, where Shawyer states

Quote from: Shawyer
A 160 mm diameter experimental thruster, operating at 2.45 GHz was designed and built. (see fig 6) The design factor, calculated from as-built measurements of the thruster geometry
was 0.497. An unloaded Q of 5,900 was measured. [/b].
(Unfortunately, Shawyer does not provide the small diameter or the cavity length in his paper)

I have used this information

bigDiameter = 0.16 m;
f = 2.45*10^9 Hz;
cst = 1.7062895542683174;
cM = 299705000 m/s (speed of light in air);
Design Factor = 0.497,

and inverted the equation for the Design Factor (see: http://forum.nasaspaceflight.com/index.php?topic=36313.msg1374110#msg1374110 ) to obtain the following correct dimension:

small diameter = 0.1025 m SHAWYER's EXPERIMENTAL THRUSTER

(obtained from the Design Factor, bigDiameter and frequency provided by Shawyer)

Nice idea. I got the same small end diameter using Excel goal seek, setting Df as goal and allowing small diameter to be varied until it hit the Df goal.
Title: Re: EM Drive Developments - related to space flight applications - Thread 2
Post by: StrongGR on 05/20/2015 12:13 pm
As promised I am posting the latest draft about general relativity and electromagnetic field. The relevant conclusion is that there is thrust. Thanks to the comments by Jose Rodal, it can be shown that this can be meaningful and the best geometry is that of the frustum tending to a cone. There is no violation of conservation law due to the presence of the gravity that can escape the device producing a reaction.

I post it here for your comments that are very welcome as usual. You can find the final equation at page 12 for your evaluations. Later on, I will post a version with a somewhat different presentation to arxiv.
Title: Re: EM Drive Developments - related to space flight applications - Thread 2
Post by: Rodal on 05/20/2015 12:14 pm
Ha, that table was from you - "Here is a comparison of reported measurements for EM Drives and for the latest report by Fearn, Zachar, Woodward & Wanser."

That table (http://forum.nasaspaceflight.com/index.php?topic=36313.msg1302455#msg1302455 ) was dated 12/14/2014, more than 5 months ago.

The table was the result of the best efforts of a team of people that have been in this thread from the beginning, working with this "EM Drive tar baby" where researchers report experimental measurements without giving the dimensions of the cavities used in the experiments .

Kudos to the people that did their best to guesstimate the dimensions of these cavities, which the researchers neglected to do in their reports.

Subsequent to that table, we owe thanks to Paul March of NASA Eagleworks for being the only researcher, worldwide, to have provided the dimensions of the cavity used in their reported experimental measurements.  I have updated the table with this information.

Shawyer has never provided all the dimensions for the cavities he used in his reported experiments (although he is in communication with people in this forum, and we have asked for these dimensions).

Shawyer only provided the large diameter for his Experimental and Demonstration thrusters.

As discussed in the last dozen pages of this thread, at the time (12/14/2014) that the table was reported we had interpreted Shawyer's Design Factor as including the cavity length.  We had interpreted Shawyer's Design Factor as modeling the truncated cone cavity as being a single continuous cavity (as it must be in our real world).  Thanks to the efforts of TheTraveller we learned that Shawyer has followed a very unorthodox procedure to derive his "DesignFactor" where he models the (real, continuous, single) truncated cone as being two (2) discontinuous, disjoint, cylindrical cavities, one having the big diameter and the other separate cavity having the small diameter. We have since then recomputed the Design Factor and have posted this interpretation of Shawyer's Design Factor (see: http://forum.nasaspaceflight.com/index.php?topic=36313.msg1374110#msg1374110 ).  We have also updated the dimension of the small diameter calculated on the basis of this Design Factor.

Notice that while our original interpretation of Shawyer's Design Factor took into account all vital dimensions: the small diameter, the big diameter and the cavity length (from which the truncated cone angle can be obtained), the latest interpretation of Shawyer's Design Factor, completely ignores the cavity length.  This interpretation of the Design Factor, worked out with TheTraveller shows that Shawyer calculates the thrust force on an EM Drive to be completely independent of the cavity length: it doesn't make a difference whether the cavity has zero length or whether the cavity has a length of 50,000 light years from here to the nearest magnetar.  It doesn't make a difference whether the truncated cone has a cone angle approaching zero (like a cylinder) or the truncated cone has a cone angle of 45 degrees.  While such a Design Factor, and hence such a formula for thrust force, that completely ignores the cavity length does not make physical sense to me (*), we have reproduced it, because if that is the formula that Shawyer used (however questionable it may be), that is the formula we need to unlock the geometrical dimension of the small diameter that Shawyer has not directly, explicitly provided.  Please note that Shawyer has not provided the length of the cavity either, which is consistent with his formula that ignores the cavity length. Therefore, notice that Shawyer's cavity lengths are being estimated and so anyone using them is forewarned to use them at their own peril.

_____________
(*) Obviously, since Shawyer's thrust force formula ignores the cavity length (and hence ignores the cone angle) it must be an approximation for an undisclosed range of cavity lengths (or alternatively, an approximation for an undisclosed range of cone angles). Since the initial cone angles of the experiments by Shawyer (the Experimental. Demonstration and Flight Thruster) involved small cone angles, according to the published photographs, it is apparent that Shawyer's thrust formula, and hence his Design Factor must be based on a small cone angle approximation.
Title: Re: EM Drive Developments - related to space flight applications - Thread 2
Post by: TheTraveller on 05/20/2015 12:50 pm
Notice that while our original interpretation of Shawyer's Design Factor took into account all vital dimensions: the small diameter, the big diameter and the cavity length (from which the truncated cone angle can be obtained), the latest interpretation of Shawyer's Design Factor, completely ignores the cavity length.  This interpretation of the Design Factor, worked out with TheTraveller shows that Shawyer calculates the thrust force on an EM Drive to be completely independent of the cavity length: it doesn't make a difference whether the cavity has zero length or whether the cavity has a length of 50,000 light years from here to the nearest magnetar.  It doesn't make a difference whether the truncated cone has a cone angle approaching zero (like a cylinder) or the truncated cone has a cone angle of 45 degrees.  While such a Design Factor, and hence such a formula for thrust force, that completely ignores the cavity length does not make physical sense to me, we have reproduced it, because if that is the formula that Shawyer used (however questionable it may be), that is the formula we need to unlock the geometrical dimension of the small diameter that Shawyer has not directly, explicitly provided.  Please note that Shawyer has not provided the length of the cavity either, which is consistent with his formula that ignores the cavity length. Therefore, notice that Shawyer's cavity lengths are being estimated and so anyone using them is forewarned to use them at their own peril.

It is not correct to state Shawyer ignores the length. He has stated many times that the applied Rf must cause frustum length resonance. Without this resonance there will be little thrust. Note he uses electrical length and not physical length. Spoilers? More bread crumbs for the trail?

I understand you focus on the equations but maybe spend some time reading his words as per the attached.

Has he described how to calc the physical end plate separation versus external Rf to obtain resonance? No he has not.

Is that a crime? No it is not.

SPR is in business to make money and giving away all their "Secret Squirrel Secret Sauce" may not be in their best interest.

If Shawyer does not fully disclose, that is his right and you have NO right to call him out for doing so.

Title: Re: EM Drive Developments - related to space flight applications - Thread 2
Post by: Rodal on 05/20/2015 12:55 pm
...
It is not correct to state Shawyer ignores the length. He has stated many times that the applied Rf must cause frustum length resonance. Without this resonance there will be little thrust.

I understand you focus on the equations but maybe spend some time reading his words as per the attached.

Has he described how to calc the end plate separation versus external Rf to obtain resonance? No he has not.

Is that a crime? No it is not.

SPR is in business to make money and giving away all their "Secret Squirrel Secret Sauce" may not be in their best interest.

If Shawyer does not fully disclose, that is his right and you have NO right to call him out for doing so.
Sir @TheTraveller, I was answering @phaseshift: I referred exclusively to Shawyer's mathematical equations, and the fact that the geometrical dimensions of the experiments where not reported in his papers.  It is expected that the geometrical dimensions should be disclosed in experimental papers, and it is pro forma in peer-review to argue mathematical equations in papers.

Sir, this thread is not a thread to deal with personalities, and much less is a thread to sponsor the commercial interests of private, for profit businesses.

Please try to adopt an objective, and skeptical attitude in analyzing the EM Drive technical subject and try not to personalize it.
Title: Re: EM Drive Developments - related to space flight applications - Thread 2
Post by: TheTraveller on 05/20/2015 12:59 pm
...
It is not correct to state Shawyer ignores the length. He has stated many times that the applied Rf must cause frustum length resonance. Without this resonance there will be little thrust.

I understand you focus on the equations but maybe spend some time reading his words as per the attached.

Has he described how to calc the end plate separation versus external Rf to obtain resonance? No he has not.

Is that a crime? No it is not.

SPR is in business to make money and giving away all their "Secret Squirrel Secret Sauce" may not be in their best interest.

If Shawyer does not fully disclose, that is his right and you have NO right to call him out for doing so.
Sir, I referred exclusively to Shawyer's mathematical equations, and the fact that the geometrical dimensions of the experiments where not reported in his papers.  It is expected that the geometrical dimensions should be disclosed in experimental papers, and it is pro forma in peer-review to argue mathematical equations in papers.

Sir, this thread is not a thread to deal with personalities, and much less is a thread to defend the business interests of personalities.

Please try to adopt an objective, and skeptical attitude in analyzing the EM Drive technical subject and try not to personalize it.

You made statements Shawyer ignores the length. That statement is not correct.

You made statements Shawyer has not fully disclosed data. That is his right.
Title: Re: EM Drive Developments - related to space flight applications - Thread 2
Post by: Rodal on 05/20/2015 01:01 pm
[...
You made statements Shawyer ignores the length. That statement is not correct.

You made statements Shawyer has not fully disclosed data. That is his right.
Incorrect.  Read again.  I stated that your interpretation of Shawyer's Design Factor ignores the cavity length.

I wrote: 
Quote from: Rodal http://forum.nasaspaceflight.com/index.php?topic=36313.msg1376840#msg1376840
the latest interpretation of Shawyer's Design Factor, completely ignores the cavity length.

Title: Re: EM Drive Developments - related to space flight applications - Thread 2
Post by: rfmwguy on 05/20/2015 01:17 pm
3D Plot of Shawyer's Design Factor vs. frequency and vs. small diameter; for same big diameter as Flight Thruster, but with the small diameter ranging from zero to same size as big diameter.

Remember: according to Shawyer the Design Factor multiplies the Power Input and the Q. The higher the Design Factor, the higher the thrust of the EM Drive, the smaller the Design Factor, the smaller the thrust.

Observe that at high frequency, the Design Factor changes almost linearly with small diameter, such that the Design Factor goes to zero as the small diameter approaches the big diameter.

The Design Factor approaches 1 for the small diameter approaching zero.

As the small diameter approaches zero, the cut-off frequency clips the Design Factor, such that to be able to have a smaller small diameter one has to operate at higher frequency (in order to avoid cut-off).

A very nice feature of Shawyer's Design Factor (as opposed to McCulloch's formula) is that Shawyer's Design Factor incorporates the cut-off frequency and hence it prevents consideration of a pointy cone, as the cut-off prevents too small of a small diameter to be considered.

The highest value of the Design Factor is reached at frequencies just a little over the cut-off frequency for the small end:

Cut-Off frequency for small end= cM/(cst*sD)   

where

sD= small end diameter (m)
cst=1.7062895542683174
cM = light speed in selected medium (m/s)
     = 299705000 (m/s) (speed of light in air)
     = 299792458 (m/s) (speed of light in vacuum)


The Design Factor has little dependence on frequency, except near the cut-off frequency. The Design Factor of Shawyer asymptotically approaches this value for high frequencies (it becomes practically independent of frequency)

Limit[DesignFactor, f -> Infinity]  = (bD^2 - sD^2)/(bD^2 + sD^2)

where

bD = big end diameter (m)
sD= small end diameter (m)

Whether Shawyer's Design Factor is correct, remains to be proven.  For example, Shawyer's Design Factor predicts that the smaller the small diameter the better (hence larger cone angles, for constant frustum length), in contrast with Todd's conjecture that the highest attenuation the better (which leads to small cone angles ~7.5 degrees as the optimal design).



Reference: formula for Design Factor here:  http://forum.nasaspaceflight.com/index.php?topic=36313.msg1374110#msg1374110

designFactor =
           (bD^2 - sD^2)/( (bD^2)*Sqrt[1 - (cM/(bD*cst*f))^2] + (sD ^2)*Sqrt[1 - (cM/(cst*f*sD))^2] )

bD = big end diameter (m)
sD= small end diameter (m)
f = applied frequency (Hz)
cst=1.7062895542683174
cM = light speed in selected medium (m/s)
     = 299705000 (m/s) (speed of light in air)
     = 299792458 (m/s) (speed of light in vacuum)

Very interesting plot. Was working Excel to do the same, but why reinvent the wheel? Nice data.

One element we have yet to nut out is external applied Rf cavity resonance between the 2 end plates. Is vital for this to happen, as without resonance to applied Rf frequency, there will be no thrust developed as Q will be very low.

Simple to calc wavelength 1/2 wavelength between and call it done but with constantly variable guide wavelength between the end plates, my gut says end plate resonance is not 1/2 of Lambda0 (free) as Lambda0 only exists outside the cavity, nor Lambda1 (big end) guide wavelength nor Lambda2 (small end) guide wavelength. Additionally guide wavelength varies continuously, at each point of diameter change, from one end of the cavity to the other.

So how to calc the external Rf frequency required to bring the cavity into end plate to end plate resonance and allow Q to grow very large?

Of course all this assumes any external waveguide / coax is tuned (SWR 1:1) to supply Rf energy with minimal loss.

It is important as Shawyer says in the 1st attachment.

In the 2nd attachment there is notes an interesting object built into the side of the cavity.

It is used as in the 3rd attachment to assist the cavity getting into resonance?

I would say the tuning stub might be used as an impedance (50 ohms) match to the input coupling loop or stub. Not sure what Shawyer used as a feeding element from the flange connector internally.

Speaking of which, trying to decide the connector type used for input on the above pic. Europeans are fond of the 7/16 DIN RF connector and the new 4.1-9.5 (Mini) DIN. Typically, the flange is about 1" square. I also thought it might be a TNC. Proves that pics are only so useful when replicating designs.

I think we can rule out N, BNC or SO-239 however ;D



Title: Re: EM Drive Developments - related to space flight applications - Thread 2
Post by: TheTraveller on 05/20/2015 01:17 pm
[...
You made statements Shawyer ignores the length. That statement is not correct.

You made statements Shawyer has not fully disclosed data. That is his right.
Incorrect.  Read again.  I stated that your interpretation of Shawyer's Design Factor ignores the cavity length.

I wrote:  Shawyer's Design Factor, completely ignores the cavity length.

Quote
While such a Design Factor, and hence such a formula for thrust force, that completely ignores the cavity length

T = (2 * Po * Df * Q) / c seems to ignore the requirement that the applied Rf must cause the frustum to operate in electrical length resonance. But in reality it is there.

Po does imply the frustum is operating at resonance as otherwise little energy would be transferred from the Rf generator to inside the frustum. Additionally the frustum load impedance must be matched to that of the Rf generator's output impedance or energy will be lost in the xfer.

Likewise for Q as using a Rf driving frequency that was not at frustum resonance would generate a low Q.

Which says both Shawyers Po and Q imply the frustum is impedance matched to the Rf generator and operating at length resonance with the driving Rf generator.
Title: Re: EM Drive Developments - related to space flight applications - Thread 2
Post by: Rodal on 05/20/2015 01:40 pm
...

T = (2 * Po * Df * Q) / c seems to ignore the requirement that the applied Rf must cause the frustum to operate in electrical length resonance. But in reality it is there.

Po does imply the frustum is operating at resonance as otherwise little energy would be transferred from the Rf generator to inside the frustum. Additionally the frustum load impedance must be matched to that of the Rf generator's output impedance or energy will be lost in the xfer.

Likewise for Q as using a Rf driving frequency that was not at frustum resonance would generate a low Q.

Which says both Shawyers Po and Q imply the frustum is impedance matched to the Rf generator and operating at length resonance with the driving Rf generator.
It is known that the criitcal parameter in the solution of a conical cavity is the cone angle.  It is obvious that Shawyer's Design Factor does not take into account the cone angle.  As I stated in the note to my post ( http://forum.nasaspaceflight.com/index.php?topic=36313.msg1376840#msg1376840 ), since the initial cone angles of the experiments by Shawyer (the Experimental. Demonstration and Flight Thruster) involved small cone angles, according to the published photographs, it is apparent that Shawyer's thrust formula, and hence his Design Factor must be based on a small cone angle approximation.

As Shawyer himself has stated, for the higher cone angle used in his Superconducting Design, he is using instead of  the aforementioned Design Factor that ignores the cone angle (or equivalently, the cavity length), a numerical solution that intends to take into account the higher cone angle geometry as well as other parameters.
Title: Re: EM Drive Developments - related to space flight applications - Thread 2
Post by: deltaMass on 05/20/2015 01:43 pm
As promised I am posting the latest draft about general relativity and electromagnetic field. The relevant conclusion is that there is thrust. Thanks to the comments by Jose Rodal, it can be shown that this can be meaningful and the best geometry is that of the frustum tending to a cone. There is no violation of conservation law due to the presence of the gravity that can escape the device producing a reaction.

I post it here for your comments that are very welcome as usual. You can find the final equation at page 12 for your evaluations. Later on, I will post a version with a somewhat different presentation to arxiv.
Thank you for an interesting paper. What I think would be helpful for anchoring the reader in reality would be a small worked example for the thrust prediction.
Title: Re: EM Drive Developments - related to space flight applications - Thread 2
Post by: TheTraveller on 05/20/2015 01:45 pm
...

T = (2 * Po * Df * Q) / c seems to ignore the requirement that the applied Rf must cause the frustum to operate in electrical length resonance. But in reality it is there.

Po does imply the frustum is operating at resonance as otherwise little energy would be transferred from the Rf generator to inside the frustum. Additionally the frustum load impedance must be matched to that of the Rf generator's output impedance or energy will be lost in the xfer.

Likewise for Q as using a Rf driving frequency that was not at frustum resonance would generate a low Q.

Which says both Shawyers Po and Q imply the frustum is impedance matched to the Rf generator and operating at length resonance with the driving Rf generator.
It is known that the criitcal parameter in the solution of a conical cavity is the cone angle.  It is obvious that Shawyer's Design Factor does not take into account the cone angle.  As I stated in the note to my original post, since the initial cone angles of the experiments by Shawyer (the Experimental. Demonstration and Flight Thruster) involved small cone angles, according to the published photographs, it is apparent that Shawyer's thrust formula, and hence his Design Factor must be based on a small cone angle approximation.

Change either or both end diameters and Slant angle changes as does Df.

What is needed is how to calc effective electrical wavelength when the guide wavelength is constantly changing as in the attachment of the assumed Flight Thruster dimensions.
Title: Re: EM Drive Developments - related to space flight applications - Thread 2
Post by: Flyby on 05/20/2015 01:50 pm
can someone explain in normal people terms were is the recoil going from turning on the microwave?
shouldn't the metal box push on the microwave just as much as the microwave pushes on the metal box?

also, it gets really hot... could the recoil just be delayed?

Shawyer's explanation is that the waves bounce back and forth across the device.
The speed of the waves depends upon the shape of the container (the container is acting as a wave-guide, which reduces the propagation speed to be less than C). Shawyer asserts that the momentum change is thereby asymmetric, due to the differing effective impact speeds. Note that the wider physics community do not agree with this analysis, although I do not understand the details.
However, there are other theories, in particular Dr.White at NASA thinks that the EM fields from the standing waves are interacting with quantum 'virtual particles', effectively pushing off these. There are problems to do with special relativity and conservation of energy that would seem to make this unlikely unless you go and revive some very old theories of the aether and absolute reference frames, which are very marginal (special relativity has been well tested and aether theories abandoned as unproductive).
Another theory that allows breaking of local conservation of momentum and which _is consistent with observations  is Woodward 'Mach' effects. These relate to accelerating bodies which are changing in internal energy levels. The momentum is effectively transferred to all other bodies in the universe, at the speed of light. Attempts to produce or measure Mach effects have thus far failed, but there are reasonable theoretic reasons to believe they might be possible (they help to explain inertia and provide a preferred reference frame which special relativity lacks). I don't think anyone is explaining the EM drive in terms of this, however.

My personal preference goes to the "Todd conjecture"...  :) (poke  poke)
This theory, developped by Warptech (aka Todd D) formulates that due to the shape of the cone, waves slow down towards the small end (compression?). As a consequence, the fall in speed is compensated by a change in frequency, causing them to fall out of resonance (between small en big plate). So they and up being attenuated or cut. The energy/momentum of that wave can't just vanish, but it is transferred into  the frustum walls...
dunno if I formulated it 100% correctly, but that's how I understood the idea.. more or less... sort of... :)

I like this idea because it gives a very comprehensible way to explain where the momentum of the frustum comes from. Something I really miss in all the other theories.
Quantum Vacuum field? dunno, but feels more like an artificial mathematical model that attempts to explain, then a valid theory to explain something real. QV has yet to be proven by experiments, btw...

However, it would also mean that the pursuit of a higher Q, to achieve higher performances is a dead end.
If Shawyer can prove with his supercooled rig that he is indeed getting considerable thrust improvements, then this theory goes down the drain...

So far, little news has been brought on his nitrogen cooled device.. tbh, that worries me a little. I'll admit, I've never believed Shawyer's linear extrapolations that promise 1ton of thrust at extremely high Q. I know very few devices that scale perfectly linear in power/thrust output.
Title: Re: EM Drive Developments - related to space flight applications - Thread 2
Post by: StrongGR on 05/20/2015 01:53 pm
As promised I am posting the latest draft about general relativity and electromagnetic field. The relevant conclusion is that there is thrust. Thanks to the comments by Jose Rodal, it can be shown that this can be meaningful and the best geometry is that of the frustum tending to a cone. There is no violation of conservation law due to the presence of the gravity that can escape the device producing a reaction.

I post it here for your comments that are very welcome as usual. You can find the final equation at page 12 for your evaluations. Later on, I will post a version with a somewhat different presentation to arxiv.
Thank you for an interesting paper. What I think would be helpful for anchoring the reader in reality would be a small worked example for the thrust prediction.

Thanks. Yes, you are right. It is the next step to fill with numbers and put some plots to see the orders of magnitude. I am not sure yet that I am able to account for a macroscopic effect.
Title: Re: EM Drive Developments - related to space flight applications - Thread 2
Post by: TheTraveller on 05/20/2015 01:55 pm
can someone explain in normal people terms were is the recoil going from turning on the microwave?
shouldn't the metal box push on the microwave just as much as the microwave pushes on the metal box?

also, it gets really hot... could the recoil just be delayed?

Shawyer's explanation is that the waves bounce back and forth across the device.
The speed of the waves depends upon the shape of the container (the container is acting as a wave-guide, which reduces the propagation speed to be less than C). Shawyer asserts that the momentum change is thereby asymmetric, due to the differing effective impact speeds. Note that the wider physics community do not agree with this analysis, although I do not understand the details.
However, there are other theories, in particular Dr.White at NASA thinks that the EM fields from the standing waves are interacting with quantum 'virtual particles', effectively pushing off these. There are problems to do with special relativity and conservation of energy that would seem to make this unlikely unless you go and revive some very old theories of the aether and absolute reference frames, which are very marginal (special relativity has been well tested and aether theories abandoned as unproductive).
Another theory that allows breaking of local conservation of momentum and which _is consistent with observations  is Woodward 'Mach' effects. These relate to accelerating bodies which are changing in internal energy levels. The momentum is effectively transferred to all other bodies in the universe, at the speed of light. Attempts to produce or measure Mach effects have thus far failed, but there are reasonable theoretic reasons to believe they might be possible (they help to explain inertia and provide a preferred reference frame which special relativity lacks). I don't think anyone is explaining the EM drive in terms of this, however.

My personal preference goes to the "Todd conjecture"...  :) (poke  poke)
This theory, developped by Warptech (aka Todd D) formulates that due to the shape of the cone, waves slow down towards the small end (compression?). As a consequence, the fall in speed is compensated by a change in frequency, causing them to fall out of resonance (between small en big plate). So they and up being attenuated or cut. The energy/momentum of that wave can't just vanish, but it is transferred into  the frustum walls...
dunno if I formulated it 100% correctly, but that's how I understood the idea.. more or less... sort of... :)

I like this idea because it gives a very comprehensible way to explain where the momentum of the frustum comes from. Something I really miss in all the other theories.
Quantum Vacuum field? dunno, but feels more like an artificial mathematical model that attempts to explain, then a valid theory to explain something real. QV has yet to be proven by experiments, btw...

However, it would also mean that the pursuit of a higher Q, to achieve higher performances is a dead end.
If Shawyer can prove with his supercooled rig that he is indeed getting considerable thrust improvements, then this theory goes down the drain...

So far, little news has been brought on his nitrogen cooled device.. tbh, that worries me a little. I'll admit, I've never believed Shawyer's linear extrapolations that promise 1ton of thrust. I know very few devices that scale perfectly linear in power/thrust output.

As I see it, if small end diameter is below cutoff diameter, there will be no in phase reflected wave, no Q, no thrust as the wave moving toward the small end plate never reached the end plate. But HEY I'm a wannabe Microwave engineer, in early rapid growth phase and still wearing "L" plates.

Shawyer says to operate the small end JUST ABOVE cutoff as that gives the biggest Df and still allows the waves to bounce back and forth, have a high Q and deliver high thrust.
Title: Re: EM Drive Developments - related to space flight applications - Thread 2
Post by: Notsosureofit on 05/20/2015 01:58 pm
As promised I am posting the latest draft about general relativity and electromagnetic field. The relevant conclusion is that there is thrust. Thanks to the comments by Jose Rodal, it can be shown that this can be meaningful and the best geometry is that of the frustum tending to a cone. There is no violation of conservation law due to the presence of the gravity that can escape the device producing a reaction.

I post it here for your comments that are very welcome as usual. You can find the final equation at page 12 for your evaluations. Later on, I will post a version with a somewhat different presentation to arxiv.
Thank you for an interesting paper. What I think would be helpful for anchoring the reader in reality would be a small worked example for the thrust prediction.

Thanks. Yes, you are right. It is the next step to fill with numbers and put some plots to see the orders of magnitude. I am not sure yet that I am able to account for a macroscopic effect.

It would be interesting to see which cylindrical cavity mode gave the greatest integrated field along a given laser beam path.   As far as detection goes, there has been great progress lately on separating out small sidebands from an intense laser line.  (i'll look into that)
Title: Re: EM Drive Developments - related to space flight applications - Thread 2
Post by: StrongGR on 05/20/2015 02:00 pm
As promised I am posting the latest draft about general relativity and electromagnetic field. The relevant conclusion is that there is thrust. Thanks to the comments by Jose Rodal, it can be shown that this can be meaningful and the best geometry is that of the frustum tending to a cone. There is no violation of conservation law due to the presence of the gravity that can escape the device producing a reaction.

I post it here for your comments that are very welcome as usual. You can find the final equation at page 12 for your evaluations. Later on, I will post a version with a somewhat different presentation to arxiv.
Thank you for an interesting paper. What I think would be helpful for anchoring the reader in reality would be a small worked example for the thrust prediction.

Thanks. Yes, you are right. It is the next step to fill with numbers and put some plots to see the orders of magnitude. I am not sure yet that I am able to account for a macroscopic effect.

It would be interesting to see which cylindrical cavity mode gave the greatest integrated field along a given laser beam path.   As far as detection goes, there has been great progress lately on separating out small sidebands from an intense laser line.  (i'll look into that)

Do you mean shooting a laser beam into a cylindrical cavity or into a frustum? 
Title: Re: EM Drive Developments - related to space flight applications - Thread 2
Post by: WarpTech on 05/20/2015 02:01 pm
Quote
Warptech
If the thruster had 2 compartments, a cylinder, where resonance was easy to establish at high Q, and a long frustum designed for maximum attenuation connected at one end of the cylinder. Between the two, there is a "shutter" that can rapidly open and close. When closed, the cylinder resonates as a cylinder. When open, energy expands into the frustum chamber where it is attenuated. After the shutter closes again, the energy in frustum attenuates and energy in cylinder recharges.... repeat. I keep looking for ways to decouple the resonant amplifier from the attenuator.

Firstly I need to ask what sort of timeframes you are looking at for connecting/ disconnecting cycle,  micro_sec, milli_sec, seconds?. 

How long do you think the coupling will need to be in place to create resonance inside the thruster cavity...{or are you thinking the resonance is not even needed in that chamber at all, just force fed from the attached cylinder in burst mode}.  I think I see where you are going with this but more info may help clarify the desired method, and help refine a model im working on for mechanical distribution of em waves.

I don't like posting my equations until I know they're right, but I believe I have shown that the force;

F ~ (alpha) * d(alpha)/dx

Where alpha is the attenuation "variable" of the waveguide. Alpha is larger for a small half-angle taper, so a long tapered pipe like a flagpole should be used for the attenuator. Alpha is variable in a frustum, it is not the same in both directions.

Also, the TC of alpha is Np/m, and it has very little effect over 1/2 a wavelength. Therefore, my thinking is that the resonant amplifier should be just a short cylinder to build up a high Q*P, then release that energy into a very long frustum pipe where all the momentum can be absorbed in the forward direction. Resonance is not needed, we want it to decay quickly, because faster decay is higher dp/dt = Force.

In reply to deltaMass, if you are only considering "reflection" then p = 0. But what happens when a wave is attenuated in a perfectly conducting circular waveguide? That energy is not lost as "heat" because there is no resistance to dissipate it.

Todd
Title: Re: EM Drive Developments - related to space flight applications - Thread 2
Post by: TheTraveller on 05/20/2015 02:06 pm
But what happens when a wave is attenuated in a perfectly conducting circular waveguide? That energy is not lost as "heat" because there is no resistance to dissipate it.

How will the wave attenuate (lose energy) if no wall losses?
Title: Re: EM Drive Developments - related to space flight applications - Thread 2
Post by: rfmwguy on 05/20/2015 02:08 pm
"Bessel function - magnetron - (SRF) superconducting RF cavity - Fermi Labs"

Sound interesting? I thought so: http://www.fnal.gov/pub/today/archive/archive_2014/today14-11-03.html

Should the emdrive become a reality...Fermi might be on the verge of helping to develop a massive, cost-effective RF source: http://www.fnal.gov/pub/today/archive/archive_2014/images/magnetron.jpg
Title: Re: EM Drive Developments - related to space flight applications - Thread 2
Post by: Notsosureofit on 05/20/2015 02:09 pm
As promised I am posting the latest draft about general relativity and electromagnetic field. The relevant conclusion is that there is thrust. Thanks to the comments by Jose Rodal, it can be shown that this can be meaningful and the best geometry is that of the frustum tending to a cone. There is no violation of conservation law due to the presence of the gravity that can escape the device producing a reaction.

I post it here for your comments that are very welcome as usual. You can find the final equation at page 12 for your evaluations. Later on, I will post a version with a somewhat different presentation to arxiv.
Thank you for an interesting paper. What I think would be helpful for anchoring the reader in reality would be a small worked example for the thrust prediction.

Thanks. Yes, you are right. It is the next step to fill with numbers and put some plots to see the orders of magnitude. I am not sure yet that I am able to account for a macroscopic effect.

It would be interesting to see which cylindrical cavity mode gave the greatest integrated field along a given laser beam path.   As far as detection goes, there has been great progress lately on separating out small sidebands from an intense laser line.  (i'll look into that)

Do you mean shooting a laser beam into a cylindrical cavity or into a frustum?

Just cylindrical at this point.
Title: Re: EM Drive Developments - related to space flight applications - Thread 2
Post by: StrongGR on 05/20/2015 02:20 pm
As promised I am posting the latest draft about general relativity and electromagnetic field. The relevant conclusion is that there is thrust. Thanks to the comments by Jose Rodal, it can be shown that this can be meaningful and the best geometry is that of the frustum tending to a cone. There is no violation of conservation law due to the presence of the gravity that can escape the device producing a reaction.

I post it here for your comments that are very welcome as usual. You can find the final equation at page 12 for your evaluations. Later on, I will post a version with a somewhat different presentation to arxiv.
Thank you for an interesting paper. What I think would be helpful for anchoring the reader in reality would be a small worked example for the thrust prediction.

Thanks. Yes, you are right. It is the next step to fill with numbers and put some plots to see the orders of magnitude. I am not sure yet that I am able to account for a macroscopic effect.

It would be interesting to see which cylindrical cavity mode gave the greatest integrated field along a given laser beam path.   As far as detection goes, there has been great progress lately on separating out small sidebands from an intense laser line.  (i'll look into that)

Do you mean shooting a laser beam into a cylindrical cavity or into a frustum?

Just cylindrical at this point.

That is in the first part of the paper, Sec. II, for a plane wave (a single mode in a box) propagating along z-axis. You can specialize this to the case of interest. The full computation, with the metric given in Sec. III, would only make things more complicated. The main result is that, due to coupling with gravity, there are two satellite frequencies beyond the main of the laser. The shift is exactly the frequency of the mode.
Title: Re: EM Drive Developments - related to space flight applications - Thread 2
Post by: TheTraveller on 05/20/2015 02:23 pm
"Bessel function - magnetron - (SRF) superconducting RF cavity - Fermi Labs"

Sound interesting? I thought so: http://www.fnal.gov/pub/today/archive/archive_2014/today14-11-03.html

Should the emdrive become a reality...Fermi might be on the verge of helping to develop a massive, cost-effective RF source: http://www.fnal.gov/pub/today/archive/archive_2014/images/magnetron.jpg

Gotta like this guy:

Quote
The reason I'm not retired is that I want to build this prototype," Pasquinelli said. "It's a solution to a real-world problem, and it will be a lot of fun to build the first one.
Title: Re: EM Drive Developments - related to space flight applications - Thread 2
Post by: deltaMass on 05/20/2015 02:40 pm
As mentioned, I'm interested in maximum thrust-to-weight ratio, and more than that, I'm interested in the smallest and lightest embodiment.

Because CubeSat.

The most interesting suggestion to date that meets these criteria has been the tapered fibre laser. It comes with a built-in cavity so to speak that's "sort of" the right asymmetrical shape (he said, handwaving wildly). To that end, here's a list of manufacturers/suppliers of these gizmos.
http://www.hanel-photonics.com/tapered_amplifier.html
Title: Re: EM Drive Developments - related to space flight applications - Thread 2
Post by: Rodal on 05/20/2015 02:40 pm
But what happens when a wave is attenuated in a perfectly conducting circular waveguide? That energy is not lost as "heat" because there is no resistance to dissipate it.

How will the wave attenuate (lose energy) if no wall losses?

Attenuation decreases the intensity of electromagnetic waves due to absorption or scattering of photons. 

The primary causes of attenuation in matter (for photon energies below 1 MeV) are:


the photoelectric effect ( http://en.wikipedia.org/wiki/Photoelectric_effect ) (it requires energies from a few electronvolts to over 1 MeV in elements with a high atomic number).  Early experiments in photoelectrcity were carried out with two copper plates, placed inside an evacuated glass bulb. One of the plates was illuminated, and the photoelectric current from it was measured.

and

Compton scattering ( http://en.wikipedia.org/wiki/Compton_scattering ) (this involves higher energies ~ 17keV)


For high photon energies (above 1 MeV ), pair production  ( http://en.wikipedia.org/wiki/Pair_production ) is also involved in attenuation of electromagnetic waves.  At high photon energies, it actually becomes the dominant mode of photon interaction with matter.
Title: Re: EM Drive Developments - related to space flight applications - Thread 2
Post by: ForgeMonkey on 05/20/2015 02:44 pm
Hi,

Here's another first post from someone who's been reading this thread for a long time.

I’ve been playing around with my own model of an EM drive for the last few days. It’s not as good as phaseshift’s version, (no scripted dimensions or curved end plates) but as I already have it in an almost finished state I thought it would be worth posting.

At the moment it’s still pretty crude; the main body is overcomplicated to make in its current configuration and the whole thing requires far too many bolts to assemble. It’s modelled roughly on the dimensions Mullentron is using for his build (posted at https://drive.google.com/folderview?id=0B4PCfHCM1KYoTXhSUTd5ZDN2WnM&usp=sharing#list).

I've posted the files (both original Solidworks .SLDPRT and printable .STLs) on Thingiverse for anyone that wants a copy (link: http://www.thingiverse.com/thing:838001). If I get time over the next few days/weeks I'm going to try and make it easier to build, break it down into more parts, tabulate the dimensions to make it easier to modify, and maybe add some proper drawings.

Disclaimer: I'm afraid I still need to work on my physics before I can make a meaningful contribution to the theoretical side of the discussion, so if I've made any mistakes in the model please point them out and I’ll do my best to fix them when I can.
Title: Re: EM Drive Developments - related to space flight applications - Thread 2
Post by: Rodal on 05/20/2015 02:47 pm
Hi,

Here's another first post from someone who's been reading this thread for a long time.
...
Welcome to the forum  :)
Title: Re: EM Drive Developments - related to space flight applications - Thread 2
Post by: hhexo on 05/20/2015 03:05 pm
As promised I am posting the latest draft about general relativity and electromagnetic field. The relevant conclusion is that there is thrust. Thanks to the comments by Jose Rodal, it can be shown that this can be meaningful and the best geometry is that of the frustum tending to a cone. There is no violation of conservation law due to the presence of the gravity that can escape the device producing a reaction.

I post it here for your comments that are very welcome as usual. You can find the final equation at page 12 for your evaluations. Later on, I will post a version with a somewhat different presentation to arxiv.
Thank you for an interesting paper. What I think would be helpful for anchoring the reader in reality would be a small worked example for the thrust prediction.

Thanks. Yes, you are right. It is the next step to fill with numbers and put some plots to see the orders of magnitude. I am not sure yet that I am able to account for a macroscopic effect.

*reads the paper*
... Wow. Ok, now the maths is definitely too much for a humble electronic engineer. :)

But I have to say, this work is impressive! I hope it can be peer-reviewed quickly and effectively, and then experimented with.

Even if the thrust was not macroscopic, I think this analysis would provide a link between EM and spacetime/gravity that could be interesting for science regardless. Maybe it would have other applications, or maybe it'll be another piece of the puzzle in our knowledge of the universe. The best result of course would be if this predicted the thrust measured for the EmDrive, but let's be calm, relaxed and skeptical about this for now.

Incidentally, since we had a "Yay for Romania!" moment in this thread for Iulian's work, may I (as an Italian expat) have a "Yay for Italy!" moment for Marco's work? :)
Title: Re: EM Drive Developments - related to space flight applications - Thread 2
Post by: The Amazing Catstronaut on 05/20/2015 03:09 pm


I've posted the files (both original Solidworks .SLDPRT and printable .STLs) on Thingiverse for anyone that wants a copy (link: http://www.thingiverse.com/thing:838001). If I get time over the next few days/weeks I'm going to try and make it easier to build, break it down into more parts, tabulate the dimensions to make it easier to modify, and maybe add some proper drawings.


Welcome to the forum! ;D It's great here!

Not to put words under your fingers, but it sounds like you're making something of your own template for EM drive home builds. As another none-expert interested in having a shot at it, I tip my proverbial hat to you.

Title: Re: EM Drive Developments - related to space flight applications - Thread 2
Post by: CW on 05/20/2015 03:10 pm
By the way.. where is Iulian? I hope he has not crippled or killed himself with the magnetron.. !
Title: Re: EM Drive Developments - related to space flight applications - Thread 2
Post by: Rodal on 05/20/2015 03:12 pm
...
...
Incidentally, since we had a "Yay for Romania!" moment in this thread for Iulian's work, may I (as an Italian expat) have a "Yay for Italy!" moment for Marco's work? :)
VIVA ITALIA

forza azzurri   :)

(http://digilander.libero.it/penelope.sm/bandiere/GlitterItalia.gif)

Fortuna i forti aiuta, e i timidi rifiuta.
Title: Re: EM Drive Developments - related to space flight applications - Thread 2
Post by: Star One on 05/20/2015 03:19 pm

By the way.. where is Iulian? I hope he has not crippled or killed himself with the magnetron.. !

Similar concerns about him have been raised elsewhere. Hope he's OK.
Title: Re: EM Drive Developments - related to space flight applications - Thread 2
Post by: Giovanni DS on 05/20/2015 03:22 pm

By the way.. where is Iulian? I hope he has not crippled or killed himself with the magnetron.. !

Similar concerns about him have been raised elsewhere. Hope he's OK.

Date Registered:
    05/07/2015 04:30 AM
Local Time:
    05/20/2015 03:21 PM
Last Active:
    Today at 05:05 PM

Apparently he is fine.
Title: Re: EM Drive Developments - related to space flight applications - Thread 2
Post by: Rodal on 05/20/2015 03:26 pm

By the way.. where is Iulian? I hope he has not crippled or killed himself with the magnetron.. !

Similar concerns about him have been raised elsewhere. Hope he's OK.
Besides the obvious dangers associated with microwaves, there is the non-obvious danger of Beryllium inhalation if it gets chipped and it becomes airborne.

https://www.youtube.com/watch?v=cGZsGN8PzU4

http://en.wikipedia.org/wiki/Beryllium_poisoning
Title: Re: EM Drive Developments - related to space flight applications - Thread 2
Post by: TheTraveller on 05/20/2015 03:28 pm
Moving onto frustum resonance and how to use Df to calculate end plate spacing.

We know each end has a unique guide wavelength.

If we assume the frustum is composed of 2 cylindrical sections, closed at one end and joined at the other each, with each of a length = it's guide wavelength, which says it is only what happens to guide wavelength at each end that matters to frustum resonance.

1st using lambda g1^2 (big end) / lambda g2 (small end) we get the wavelength of the external Rf that is required to generate the guide wavelengths at each end as per the Df equation. Then adjust either or both diameters to get the calculated effective guide wavelength to match the actual external wavelength.

Next knowing the guide wavelength of both ends, add them together as the target external Rf wavelength. Then adjust end plate spacing to achieve same physical spacing as the sum of the 2 end plate guide wavelengths.

Does this make sense? Seems to generate Flight Thruster dimensions close to what we have worked out.

Title: Re: EM Drive Developments - related to space flight applications - Thread 2
Post by: deltaMass on 05/20/2015 03:38 pm
I thought we had (x for lambda)
x02 = x1 x2
but here you say
x12 = x0 x2
which of course is not the same

where x0 is the input RF wavelength
Title: Re: EM Drive Developments - related to space flight applications - Thread 2
Post by: D_Dom on 05/20/2015 03:46 pm
Q: Will these equations hold for the 600 to 1500 nM wavelengths of the tapered fiber amplifiers?
Title: Re: EM Drive Developments - related to space flight applications - Thread 2
Post by: SeeShells on 05/20/2015 03:49 pm
As promised I am posting the latest draft about general relativity and electromagnetic field. The relevant conclusion is that there is thrust. Thanks to the comments by Jose Rodal, it can be shown that this can be meaningful and the best geometry is that of the frustum tending to a cone. There is no violation of conservation law due to the presence of the gravity that can escape the device producing a reaction.

I post it here for your comments that are very welcome as usual. You can find the final equation at page 12 for your evaluations. Later on, I will post a version with a somewhat different presentation to arxiv.
*Cherry picks the points and takes in the formulas . . . very slowly.
This is worth a hot tub print out and read. Kudos to you. My math is quite rusty but I still have enough stocked away to grasp what your inferring and so far it makes some good sense.

Reading it I had a question (more than one but..). And this is for everyone, why did Eagle Works observe no thrust in a EM device with no HDPE insert? Makes me wonder what effect achiral materials like this that can induce chirality would have with relativity and electromagnetic fields in your equations?

Thanks nice work!
Title: Re: EM Drive Developments - related to space flight applications - Thread 2
Post by: ForgeMonkey on 05/20/2015 03:53 pm


I've posted the files (both original Solidworks .SLDPRT and printable .STLs) on Thingiverse for anyone that wants a copy (link: http://www.thingiverse.com/thing:838001). If I get time over the next few days/weeks I'm going to try and make it easier to build, break it down into more parts, tabulate the dimensions to make it easier to modify, and maybe add some proper drawings.


Welcome to the forum! ;D It's great here!

Not to put words under your fingers, but it sounds like you're making something of your own template for EM drive home builds. As another none-expert interested in having a shot at it, I tip my proverbial hat to you.

Thanks for the proverbial hat-tip  :)

I’m interested in helping in whatever way I can. I’ve spent a lot of time over the last few years designing things in CAD (mostly, but not exclusively, Solidworks), so I thought the logical place to start would be a model of the device.

I doubt I’ll be able to pursue my own independent build, as a student I doubt I’ll have the time. But, I’m more than happy to draw up, or design, custom parts for home builders in what spare time I do have. I also operate a 3D printer, (an Ultimaker v1, nothing special) so if people want printed parts for their builds just drop me a line.

I wonder if it would be worth starting a list of “consulting” engineers and specialists alongside the list of builders at ( http://emdrive.echothis.com/Building )?
Title: Re: EM Drive Developments - related to space flight applications - Thread 2
Post by: TheTraveller on 05/20/2015 03:57 pm
I thought we had (x for lambda)
x02 = x1 x2
but here you say
x12 = x0 x2
which of course is not the same

where x0 is the input RF wavelength

The equation supplied seems to give the effective electrical length of the frustum for resonance at the external Rf.

BUT the physical end plate spacing may not be that needed to get resonance with the INTERNAL combined guide wavelengths.

Note here Shawyer says resonance at the electrical length.
Title: Re: EM Drive Developments - related to space flight applications - Thread 2
Post by: saucyjack on 05/20/2015 04:13 pm
That table (http://forum.nasaspaceflight.com/index.php?topic=36313.msg1302455#msg1302455 ) was dated 12/14/2014, more than 5 months ago.

The table was the result of the best efforts of a team of people that have been in this thread from the beginning, working with this "EM Drive tar baby" where researchers report experimental measurements without giving the dimensions of the cavities used in the experiments .

This data is some of the most-referenced in the entire thread.  To make it easier to read, I reformatted it into this page on the wiki (http://emdrive.echothis.com/Dimensions).  As the estimated dimensions and other parameters are revised, and tests run on new device variations, they can easily be updated there. 

In addition if there is other content that should be replicated there (I'm thinking of theory equations and derivations in particular), let me know - or just jump in yourself.  The wiki now supports MathJax (https://www.mathjax.org/) so the math there can be more readable than what we can do in a forum post.

Any other comments on how to make the wiki more useful, please PM me.  It's been averaging a few updates a day by various folks, so is starting to grow.

-Rolf
Title: Re: EM Drive Developments - related to space flight applications - Thread 2
Post by: Rodal on 05/20/2015 04:30 pm
That table (http://forum.nasaspaceflight.com/index.php?topic=36313.msg1302455#msg1302455 ) was dated 12/14/2014, more than 5 months ago.

The table was the result of the best efforts of a team of people that have been in this thread from the beginning, working with this "EM Drive tar baby" where researchers report experimental measurements without giving the dimensions of the cavities used in the experiments .

This data is some of the most-referenced in the entire thread.  To make it easier to read, I reformatted it into this page on the wiki (http://emdrive.echothis.com/Dimensions).  As the estimated dimensions and other parameters are revised, and tests run on new device variations, they can easily be updated there. 

In addition if there is other content that should be replicated there (I'm thinking of theory equations and derivations in particular), let me know - or just jump in yourself.  The wiki now supports MathJax (https://www.mathjax.org/) so the math there can be more readable than what we can do in a forum post.

Any other comments on how to make the wiki more useful, please PM me.  It's been averaging a few updates a day by various folks, so is starting to grow.

-Rolf

Thanks for doing all this work.  Very noble of you.

Please notice these minor corrections:

Fearn, Zachar, Woodward & Wanser:  not a microwave cavity, but is instead a piezoelectric MET thruster (Woodward Mach Effect)

Hence it doesn't have cavity dimensions:  0.2286   0.2794   0.15875

It is included because Prof. Woodward's device is also a propellant-less concept, and because Paul March (NASA) maintains that Prof. Woodward's Mach Effect theory might also be, in his opinion, an explanation for thrust for the EM Drive.



Cannae Superconducting should read:

measured ForcePerPowerInput = 761.9 to 952.4
Title: Re: EM Drive Developments - related to space flight applications - Thread 2
Post by: rfmwguy on 05/20/2015 04:58 pm

By the way.. where is Iulian? I hope he has not crippled or killed himself with the magnetron.. !

Similar concerns about him have been raised elsewhere. Hope he's OK.
Besides the obvious dangers associated with microwaves, there is the non-obvious danger of Beryllium inhalation if it gets chipped and it becomes airborne.

https://www.youtube.com/watch?v=cGZsGN8PzU4

http://en.wikipedia.org/wiki/Beryllium_poisoning

Well said. BeO is a common ceramic used in electronics. The grinding, sanding or drilling will make particulates airborne and if inhaled into lungs is dangerous. Normal handling or touching is OK...just do NOT try and modify the surface at all. This from my previous life working with microwave attenuators' substrates. GREAT thermal characteristics, however...
Title: Re: EM Drive Developments - related to space flight applications - Thread 2
Post by: Rodal on 05/20/2015 04:59 pm
...
Reading it I had a question (more than one but..). And this is for everyone, why did Eagle Works observe no thrust in a EM device with no HDPE insert? Makes me wonder what effect achiral materials like this that can induce chirality would have with relativity and electromagnetic fields in your equations?

Thanks nice work!
Excellent question that has been puzzling me for a long time as well.  To examine this, first let's recapitulate the statement in Brady et.al.' report:

Quote
There appears to be a clear dependency between thrust magnitude and the presence of some sort of dielectric RF resonator in the thrust chamber. The geometry, location, and material properties of this resonator must be evaluated using numerous COMSOL® iterations to arrive at a viable thruster solution. We performed some very early evaluations without the dielectric resonator (TE012 mode at 2168 MHz, with power levels up to ~30 watts) and measured no significant net thrust

My exact solution for the truncated cone, given the dimensions reported by Paul March and also used in NASA's COMSOL FEA analysis, for mode shape TE012, gives me a natural frequency of:

2.2024 GHz

So they operated at frequency of 2.168 GHz which is 1.59% away from the frequency given by the exact solution. Perhaps the reason was that that was the frequency given by the Finite Element analysis for mode TE012 (let's recall that the Finite Element solution converges from below to the correct eigensolution, and only for an infinite Finite Element mesh one can theoretically converge to the eigensolution).  So, they were looking in the right frequency range for TE012.

My recollection is that Paul March thought that the reason maybe that to generate thrust without a dielectric one needs to provide Amplitude, Frequency and Phase Modulation, and that at the time of the experiments detailed in the NASA report ( http://www.libertariannews.org/wp-content/uploads/2014/07/AnomalousThrustProductionFromanRFTestDevice-BradyEtAl.pdf ), they were not providing the needed AM, FM and PM modulation.  I was not able to find the exact quotation (I wonder why, given the fantastic search function we have at this site  ;) ).  I was only able to find this quotation from Paul March, in answer to this question:

Quote from: Star-Drive
(Shawyer and the Chinese used the magnetron excited TE012 mode in their frustum cavities without dielectrics being present.)
Title: Re: EM Drive Developments - related to space flight applications - Thread 2
Post by: Rodal on 05/20/2015 05:14 pm
That table (http://forum.nasaspaceflight.com/index.php?topic=36313.msg1302455#msg1302455 ) was dated 12/14/2014, more than 5 months ago.

The table was the result of the best efforts of a team of people that have been in this thread from the beginning, working with this "EM Drive tar baby" where researchers report experimental measurements without giving the dimensions of the cavities used in the experiments .

This data is some of the most-referenced in the entire thread.  To make it easier to read, I reformatted it into this page on the wiki (http://emdrive.echothis.com/Dimensions).  As the estimated dimensions and other parameters are revised, and tests run on new device variations, they can easily be updated there. 

In addition if there is other content that should be replicated there (I'm thinking of theory equations and derivations in particular), let me know - or just jump in yourself.  The wiki now supports MathJax (https://www.mathjax.org/) so the math there can be more readable than what we can do in a forum post.

Any other comments on how to make the wiki more useful, please PM me.  It's been averaging a few updates a day by various folks, so is starting to grow.

-Rolf

Also, if you could be so kind to do this for us, when you have a chance:

1) reads:          Brady c TM Mode
    should read:  Brady c TE Mode



2) If you have the space for another column, (perhaps by narrowing the column for the DesignFactor), could you please insert another column titled "Dielectric" that should read

For all the Brady cases:    HDPE

For all the other cases:    No dielectric


3) Instead of the simple title "Dimensions" for your comprehensive table, how about using instead:

"Experimental Data: Forces, Power, Frequency and Dimensions"


Title: Re: EM Drive Developments - related to space flight applications - Thread 2
Post by: StrongGR on 05/20/2015 05:15 pm
As promised I am posting the latest draft about general relativity and electromagnetic field. The relevant conclusion is that there is thrust. Thanks to the comments by Jose Rodal, it can be shown that this can be meaningful and the best geometry is that of the frustum tending to a cone. There is no violation of conservation law due to the presence of the gravity that can escape the device producing a reaction.

I post it here for your comments that are very welcome as usual. You can find the final equation at page 12 for your evaluations. Later on, I will post a version with a somewhat different presentation to arxiv.
*Cherry picks the points and takes in the formulas . . . very slowly.
This is worth a hot tub print out and read. Kudos to you. My math is quite rusty but I still have enough stocked away to grasp what your inferring and so far it makes some good sense.

Reading it I had a question (more than one but..). And this is for everyone, why did Eagle Works observe no thrust in a EM device with no HDPE insert? Makes me wonder what effect achiral materials like this that can induce chirality would have with relativity and electromagnetic fields in your equations?

Thanks nice work!

Thanks a lot. The reason why a material can change the behaviour is magnetic permeability. This can enhance the effect by several magnitude orders. They use low input power, if I am right, and so this could be a good way around to such a limitation.
Title: Re: EM Drive Developments - related to space flight applications - Thread 2
Post by: hhexo on 05/20/2015 05:16 pm
Thanks. Yes, you are right. It is the next step to fill with numbers and put some plots to see the orders of magnitude. I am not sure yet that I am able to account for a macroscopic effect.
...
Even if the thrust was not macroscopic, ...
...

Ok, I've run some back-of-the-envelope calculations in pure engineer style, i.e. just looking at the orders of magnitude.

I am handwaving constants as follows:

pi^2 = 10^1
G = 10^-11
c^4 = 81 * 10^32 = 10^34
mu0^2 = 16 * pi^2 * 10^-14 = 10^-12
h = 1 (this is the height of the frustum, right? Not Planck!)

and I get that the geometry related effects have to counter-balance a term which is 10^-32 * U0^4.

At this point we are left with a term (1 / ln2(r2/r1)) * (r2^4 * XXX - (r2^6 / r1^2) * YYY), where XXX and YYY are those complicated expressions for the fourth power of the magnetic field that I can't rewrite here. :) But, out of the two terms the second one is clearly dominant if r2 >> r1.

So we're left with 10^-32 * U0^4 * YYY * (r2^6 / r1^2) * (1 / ln2(r2/r1)).

I can't quite follow what the order of magnitude of U0 and YYY would be. Can anybody help me? Are they going to be significant powers of ten?

Anyway, if we just want to balance that 10^-32, we need something like r2/r1 = 10^5.5, for example:
r2 = 10^2.5 = several hundred meters
r1 = 10^-3 = one millimeter
given a frustum height of just one meter... Things get better if h is small, but not too much better.

Not very encouraging in terms of practicality. :( And if U0 or YYY are going to be negative powers of ten, it's going to get even worse.
Damn gravity being such a weak force! :)
Title: Re: EM Drive Developments - related to space flight applications - Thread 2
Post by: WarpTech on 05/20/2015 05:17 pm
But what happens when a wave is attenuated in a perfectly conducting circular waveguide? That energy is not lost as "heat" because there is no resistance to dissipate it.

How will the wave attenuate (lose energy) if no wall losses?

My understanding is that, if there are no resistive losses the energy will be stored inductively as circulating current flowing in the frustum. Where else can it go? Once there is current flowing in the copper, it will repel any change in magnetic flux attempting to damp it.
Title: Re: EM Drive Developments - related to space flight applications - Thread 2
Post by: RERT on 05/20/2015 05:19 pm
I've been reading for several days, I have a question regarding the Maxwell Equations simulations of the Frustrum.

I recently read a paper by Tuval and Yahalom (2013). They note that Maxwell's equations are conservative of momentum if the action of forces is taken to be instantaneous. However, if the forces are taken to be propagating at c they are not, in general, for particles. [The momentum of the fields is needed to make conservation of momentum work.]

So, the question is: do the simulations of the frustrum account for the retarded forces and fields on and created by the moving charges in the walls/end caps?

R.
Title: Re: EM Drive Developments - related to space flight applications - Thread 2
Post by: deltaMass on 05/20/2015 05:25 pm
It's a smart question but in a closed cavity like this, overall conservation is to be expected irrespective of whether the solution uses retarded or instantaneous wave solutions, I think.
Title: Re: EM Drive Developments - related to space flight applications - Thread 2
Post by: Rodal on 05/20/2015 05:30 pm
I've been reading for several days, I have a question regarding the Maxwell Equations simulations of the Frustrum.

I recently read a paper by Tuval and Yahalom (2013). They note that Maxwell's equations are conservative of momentum if the action of forces is taken to be instantaneous. However, if the forces are taken to be propagating at c they are not, in general, for particles. [The momentum of the fields is needed to make conservation of momentum work.]

So, the question is: do the simulations of the frustrum account for the retarded forces and fields on and created by the moving charges in the walls/end caps?

R.

No, the simulations (by COMSOL FEA and the exact solution of Greg Egan) does not consider the retarded fields.  It does not consider Jefimenko's equations for example.  The right hand side of Jefimenko's equatons involve a "retarded" time which reflects the "causality" of the expressions. In other words, the left side of each equation is actually "caused" by the right side, unlike the normal differential expressions for Maxwell's equations where both sides take place simultaneously.




Are you referring to this paper ?

Newton's Third Law in the Framework of Special Relativity 
Miron Tuval, Asher Yahalom
(Submitted on 26 Jan 2013) http://arxiv.org/abs/1302.2537
Title: Re: EM Drive Developments - related to space flight applications - Thread 2
Post by: StrongGR on 05/20/2015 05:32 pm
Thanks. Yes, you are right. It is the next step to fill with numbers and put some plots to see the orders of magnitude. I am not sure yet that I am able to account for a macroscopic effect.
...
Even if the thrust was not macroscopic, ...
...

Ok, I've run some back-of-the-envelope calculations in pure engineer style, i.e. just looking at the orders of magnitude.

I am handwaving constants as follows:

pi^2 = 10^1
G = 10^-11
c^4 = 81 * 10^32 = 10^34
mu0^2 = 16 * pi^2 * 10^-14 = 10^-12
h = 1 (this is the height of the frustum, right? Not Planck!)

and I get that the geometry related effects have to counter-balance a term which is 10^-32 * U0^4.

At this point we are left with a term (1 / ln2(r2/r1)) * (r2^4 * XXX - (r2^6 / r1^2) * YYY), where XXX and YYY are those complicated expressions for the fourth power of the magnetic field that I can't rewrite here. :) But, out of the two terms the second one is clearly dominant if r2 >> r1.

So we're left with 10^-32 * U0^4 * YYY * (r2^6 / r1^2) * (1 / ln2(r2/r1)).

I can't quite follow what the order of magnitude of U0 and YYY would be. Can anybody help me? Are they going to be significant powers of ten?

Anyway, if we just want to balance that 10^-32, we need something like r2/r1 = 10^5.5, for example:
r2 = 10^2.5 = several hundred meters
r1 = 10^-3 = one millimeter
given a frustum height of just one meter... Things get better if h is small, but not too much better.

Not very encouraging in terms of practicality. :( And if U0 or YYY are going to be negative powers of ten, it's going to get even worse.
Damn gravity being such a weak force! :)

You got it. A cone seems a better choice.
Title: Re: EM Drive Developments - related to space flight applications - Thread 2
Post by: hhexo on 05/20/2015 05:42 pm


Anyway, if we just want to balance that 10^-32, we need something like r2/r1 = 10^5.5, for example:
r2 = 10^2.5 = several hundred meters
r1 = 10^-3 = one millimeter
given a frustum height of just one meter...

Gah! Sorry, big glaring error there in my last passage. :)
Because it's r2^6 / r1^2, I can't just consider r2/r1 and scale the ratio.

r2=10^5.5
r1=1
is a correct example.
Title: Re: EM Drive Developments - related to space flight applications - Thread 2
Post by: StrongGR on 05/20/2015 05:46 pm


Anyway, if we just want to balance that 10^-32, we need something like r2/r1 = 10^5.5, for example:
r2 = 10^2.5 = several hundred meters
r1 = 10^-3 = one millimeter
given a frustum height of just one meter...

Gah! Sorry, big glaring error there in my last passage. :)
Because it's r2^6 / r1^2, I can't just consider r2/r1 and scale the ratio.

r2=10^5.5
r1=1
is a correct example.

It is my approximation: r2 increasingly large and r1 decreasing toward zero otherwise one should change the final formula. So, in your example it would be better to have r2=10 m and r1=10^-3 m and so on. This is a cone. Another way around, as suggested above, is to fill with some material the cavity. The formula goes like mur^-2 and mur much smaller than 1.
Title: Re: EM Drive Developments - related to space flight applications - Thread 2
Post by: deltaMass on 05/20/2015 05:56 pm
Yeah, and this is why some realistic numbers would help get a handle on this.
Title: Re: EM Drive Developments - related to space flight applications - Thread 2
Post by: SeeShells on 05/20/2015 06:03 pm
...
Reading it I had a question (more than one but..). And this is for everyone, why did Eagle Works observe no thrust in a EM device with no HDPE insert? Makes me wonder what effect achiral materials like this that can induce chirality would have with relativity and electromagnetic fields in your equations?

Thanks nice work!
Excellent question that has been puzzling me for a long time as well.  To examine this, first let's recapitulate the statement in Brady et.al.' report:

Quote
There appears to be a clear dependency between thrust magnitude and the presence of some sort of dielectric RF resonator in the thrust chamber. The geometry, location, and material properties of this resonator must be evaluated using numerous COMSOL® iterations to arrive at a viable thruster solution. We performed some very early evaluations without the dielectric resonator (TE012 mode at 2168 MHz, with power levels up to ~30 watts) and measured no significant net thrust

My exact solution for the truncated cone, given the dimensions reported by Paul March and also used in NASA's COMSOL FEA analysis, for mode shape TE012, gives me a natural frequency of:

2.2024 GHz

So they operated at frequency of 2.168 GHz which is 1.59% away from the frequency given by the exact solution. Perhaps the reason was that that was the frequency given by the Finite Element analysis for mode TE012 (let's recall that the Finite Element solution converges from below to the correct eigensolution, and only for an infinite Finite Element mesh one can theoretically converge to the eigensolution).  So, they were looking in the right frequency range for TE012.

My recollection is that Paul March thought that the reason maybe that to generate thrust without a dielectric one needs to provide Amplitude, Frequency and Phase Modulation, and that at the time of the experiments detailed in the NASA report ( http://www.libertariannews.org/wp-content/uploads/2014/07/AnomalousThrustProductionFromanRFTestDevice-BradyEtAl.pdf ), they were not providing the needed AM, FM and PM modulation.  I was not able to find the exact quotation (I wonder why, given the fantastic search function we have at this site  ;) ).  I was only able to find this quotation from Paul March, in answer to this question:

Quote from: Star-Drive
(Shawyer and the Chinese used the magnetron excited TE012 mode in their frustum cavities without dielectrics being present.)
Thanks for digging it out, I know you're more savvy with the search function than I am. I keep on coming back to this paper and this statement. Yes and I know the effects are very very small.
http://iopscience.iop.org/0295-5075/93/4/41002/fulltext/epl_93_4_41002.html
<An important point to appreciate here is that the permeability has two distinct contributions; a reciprocal part (μab(x, ω) = μba(x, ω)) which is just the standard vacuum contribution and a non-reciprocal part (μab(x, ω) ≠ μba(x, ω)) due to the external magnetic field interacting with the magnetic polarisability of the particles. The non-reciprocal piece breaks time-reversal invariance and results from the Faraday effect in magnetic media. Although the non-reciprocal parts arise from the point interaction where the external magnetic field couples to the scattering object, this is enough to require the correlation functions and thereby the Green tensors, to take a different form. Note here that we are choosing to work with the momentum density associated with the canonical energy momentum tensor rather than the Poynting vector; the latter is expected to integrate to zero [9] so a permeability is required to distinguish the two vectors. Using the expectation values of the noise currents at zero temperature, we find for the two-point function of the physical.

and

one usually calculates an energy or a force starting from the energy-momentum tensor of electromagnetism. Of course the measurable observable, that is the energy obtained by integration over all space, is but one component of a full tensor of observables. In particular, the time-space components are the associated momentum density of the fields which upon integration over all space gives the total momentum contained in the field [7]. In the case of a single parallel plate, it is precisely because the incident vacuum photons from the left and right tend to cancel that the plate does not develop a net momentum. This preserves the translational invariance of the vacuum. A natural question then to ask is how to find a system whereby a momentum can be displayed.>

I honestly think we have a "physical" system in the EM Drive if we use magneto-chiral matter (HDPE is a weak one in the way the carbon atoms are layered). REF: http://www.nature.com/nmat/journal/v7/n9/full/nmat2256.html
Title: Re: EM Drive Developments - related to space flight applications - Thread 2
Post by: Rodal on 05/20/2015 06:14 pm
...
I honestly think we have a "physical" system in the EM Drive if we use magneto-chiral matter (HDPE is a weak one in the way the carbon atoms are layered). REF: http://www.nature.com/nmat/journal/v7/n9/full/nmat2256.html
Having worked with polymers for decades, I agree that the extruded HDPE and PTFE (from McMaster Carr) that NASA Eagleworks used as dielectrics would be quite a stretch to be considered "strongly magneto-chiral", and be responsible for the measured thrust forces.  Also, (I have not read those papers in a long time, so please point out whether I'm wrong in my assessment, which I'll appreciate to know) my recollection is that they had to resort to a 4th order term in a perturbation series of the nonlinear problem to get the term that they rely on for the claimed effect.  My recollection is that they do not support what is the range of validity for that 4th order term to be significant, and it is also my recollection that the authors do not prove that the series in question (is it an asymptotic series ? ) is convergent and therefore whether the neglected higher order terms sum do not overwhelm the 4th order term they consider...

It looks like the magneto-chiral effect is very interesting for R&D but I have a difficulty embracing it as an explanation when the EM Drive's with the highest claimed thrust forces (Cannae superconducting. Shawyer and Prof Yang) did not use a polymer dielectric.
Title: Re: EM Drive Developments - related to space flight applications - Thread 2
Post by: deltaMass on 05/20/2015 06:14 pm
Quote
Note here that we are choosing to work with the momentum density associated with the canonical energy momentum tensor rather than the Poynting vector; the latter is expected to integrate to zero
Doesn't that imply zero net thrust?
Title: Re: EM Drive Developments - related to space flight applications - Thread 2
Post by: aero on 05/20/2015 06:19 pm
As promised I am posting the latest draft about general relativity and electromagnetic field. The relevant conclusion is that there is thrust. Thanks to the comments by Jose Rodal, it can be shown that this can be meaningful and the best geometry is that of the frustum tending to a cone. There is no violation of conservation law due to the presence of the gravity that can escape the device producing a reaction.

I post it here for your comments that are very welcome as usual. You can find the final equation at page 12 for your evaluations. Later on, I will post a version with a somewhat different presentation to arxiv.

I like you paper but have a request for a point of clarification. I got really confused when you went from equation 55 to equation 56. Why did you invert the c4/G term? (Actually I got confused way before that but I won't go there. :)
Title: Re: EM Drive Developments - related to space flight applications - Thread 2
Post by: Rodal on 05/20/2015 06:30 pm
Quote
Note here that we are choosing to work with the momentum density associated with the canonical energy momentum tensor rather than the Poynting vector; the latter is expected to integrate to zero
Doesn't that imply zero net thrust?
Certainly yes for a linear isotropic system, but apparently not for an anisotropic nonlinear system as the one they are considering.  They are considering a 4th order term.
Title: Re: EM Drive Developments - related to space flight applications - Thread 2
Post by: hhexo on 05/20/2015 06:44 pm
r2=10^5.5
r1=1
is a correct example.

It is my approximation: r2 increasingly large and r1 decreasing toward zero otherwise one should change the final formula. So, in your example it would be better to have r2=10 m and r1=10^-3 m and so on. This is a cone.

Yes. But the dependence on r2 is stronger than the one on r1, because r2 is to the sixth power whereas r1 is just squared.
So: if you keep r1 constant and vary r2, you don't need many orders of magnitude to have a reasonable effect; if you keep r2 constant and vary r1, you need to go down a lot of orders of magnitude.

If we have an r2 of one meter, r1 must be 10^-16, which is a tenth of a femtometer, which is less than the size of a proton. We cannot possibly manufacture such a cone and I think it would not work anyway because the resulting "small end plate" doesn't have enough electrons to behave like an ideal EM-reflecting plate. Although maybe it doesn't need to and it's just the cavity shape that matters.

What if we considered other shapes? We started with the truncated cone because of the EmDrive, but what if we considered a pillbox cavity (which is basically a rounded cone...)? All the equations would be different, for sure, but it might be worth exploring. Maybe in another thread. :)

Another way around, as suggested above, is to fill with some material the cavity. The formula goes like mur^-2 and mur much smaller than 1.

Yes, varying mu may be profitable too.
Title: Re: EM Drive Developments - related to space flight applications - Thread 2
Post by: SeeShells on 05/20/2015 06:52 pm
Quote
Note here that we are choosing to work with the momentum density associated with the canonical energy momentum tensor rather than the Poynting vector; the latter is expected to integrate to zero
Doesn't that imply zero net thrust?
Certainly yes for a linear isotropic system, but apparently not for an anisotropic nonlinear system as the one they are considering.  They are considering a 4th order term.
I see it!
Thank you to taking the time to answer and helping me by answering some questions that have a hard time going away. I guess I'm like a old dog with a bone, a good and bad thing. If you do any tests in the future on why that anomaly reared its head with the insertion of the HTPE I'd love to know what caused it. You would hate to see another anomaly pop up. You're dealing with a pretty big one right now ... thrust in a Can.
Title: Re: EM Drive Developments - related to space flight applications - Thread 2
Post by: Rodal on 05/20/2015 06:54 pm
r2=10^5.5
r1=1
is a correct example.

It is my approximation: r2 increasingly large and r1 decreasing toward zero otherwise one should change the final formula. So, in your example it would be better to have r2=10 m and r1=10^-3 m and so on. This is a cone.

...

What if we considered other shapes? We started with the truncated cone because of the EmDrive, but what if we considered a pillbox cavity (which is basically a rounded cone...)? All the equations would be different, for sure, but it might be worth exploring. Maybe in another thread. :)

...
Marco's present equations in his present papers are general enough to already include (in the limit for cone angle approaching zero) a cylindrical pillbox cavity as the one used by NASA in their actual experiments.  We (in a previous post) actually considered such a shape, and it is clearly inferior to a pointy cone.  The action takes place towards the apex of the cone.  The pointy conical geometry is important for this effect.  Notice that there is a singularity at a cone's vertex, we need some help like that from geometry, that I don't see in nice rounded shapes.  Considering a material with different permeability I think that a material discontinuity interface (providing a jump rather than a gradual variation) is also helpful in this regards :)
Title: Re: EM Drive Developments - related to space flight applications - Thread 2
Post by: squid on 05/20/2015 06:56 pm
Consider the typical SRF cavities described in this paper: http://arxiv.org/pdf/physics/0003011.pdf

Each cell in the stack of cavities could be thought of as being two cones arranged back to back: <><><><> (see Figure 5). We would then expect any forces to try and pull the two half-cavities apart.

These were operated at 1.3 GHz, 200 kW CW RF, with a measured Q of ~ 5e9.

Applying the quoted formula of F ~ Po*Q/c, we get a force of 3e6 N.

The cavities have a wall thickness of about 2 mm, and a major radius of ~100 mm, giving a strain of > 2000 MPa in the niobium. The yield strength of niobium is somewhere in the range of 80-150 MPa, depending on temper and annealing. And yet the cavities did not fly apart.

Anyone care to poke holes in this?
Title: Re: EM Drive Developments - related to space flight applications - Thread 2
Post by: phaseshift on 05/20/2015 07:10 pm
I'm incredibly appreciate of all the work that went into the dimensional estimates of the various drives but I'm having some difficulty understanding where the dimensional values came from regarding the attached image:

Shawyer reported a large plate diameter of .28m - so using that as a given.

Going straight off the image (not taking into account perspective) at the junction of the cone and the cylinder I get about .169m for the small plate diameter. This is a difference of 40mm with what was originally estimated and about 73mm from Dr. Rodal's calculation (96.13) of the small plate diameter.  This is an enormous difference and I can't believe it. I'm going to model this and lay it on top of the photo in perspective so I can find out what the numbers are closer to.

Also the cone length was estimated to be .345m - which is greater than the large plate diameter. Between which two points is this measurement for?





Title: Re: EM Drive Developments - related to space flight applications - Thread 2
Post by: Rodal on 05/20/2015 07:17 pm
I'm incredibly appreciate of all the work that went into the dimensional estimates of the various drives but I'm having some difficulty understanding where the dimensional values came from regarding the attached image:

Shawyer reported a large plate diameter of .28m - so using that as a given.

Going straight off the image (not taking into account perspective) at the junction of the cone and the cylinder I get about .169m for the small plate diameter. This is a difference of 40mm with what was originally estimated and about 73mm from Dr. Rodal's calculation (96.13) of the small plate diameter.  This is an enormous difference and I can't believe it. I'm going to model this and lay it on top of the photo in perspective so I can find out what the numbers are closer to.

Also the cone length was estimated to be .345m - which is larger than the plate diameter. Between which two points is this measurement for?
I can't answer that beyond hypotheticals like 1) Shawyer's paper may have a typo in the numbers (a small difference in the DesignFactor makes a big difference in the dimensions for example), 2) TheTraveller's latest interpretation of the DesignFactor may still not be Shawyer's DesignFactor (which I repeat was not explicitly defined by Shawyer in his papers), 3) the picture you are using may not represent what Shawyer meant by the Demonstration engine in his paper, 4) the "length" is arbitrary for this design because the location of the internal movable wall at the small end is unknown (I don't know whether the guesstimator used the maximum length for example or the average length or the minimum length) etc. etc. 
Title: Re: EM Drive Developments - related to space flight applications - Thread 2
Post by: deltaMass on 05/20/2015 07:22 pm
Consider the typical SRF cavities described in this paper: http://arxiv.org/pdf/physics/0003011.pdf

Each cell in the stack of cavities could be thought of as being two cones arranged back to back: <><><><> (see Figure 5). We would then expect any forces to try and pull the two half-cavities apart.

These were operated at 1.3 GHz, 200 kW CW RF, with a measured Q of ~ 5e9.

Applying the quoted formula of F ~ Po*Q/c, we get a force of 3e6 N.

The cavities have a wall thickness of about 2 mm, and a major radius of ~100 mm, giving a strain of > 2000 MPa in the niobium. The yield strength of niobium is somewhere in the range of 80-150 MPa, depending on temper and annealing. And yet the cavities did not fly apart.

Anyone care to poke holes in this?
It's worse than that because the cavities are niobium-coated copper; copper has a yield strength of 70 MPa.

It's these sorts of observations that keep people grounded in reality.

People will most likely attack your numbers  based on the dissimilarity between these cavities and the EmDrive ones.
The biggest difference I see is that these ones have curved walls. I don't imagine that makes any substantive difference.
Another difference might be the resonant mode choice. Do some modes not "generate thrust"?
Title: Re: EM Drive Developments - related to space flight applications - Thread 2
Post by: Rodal on 05/20/2015 07:23 pm
Consider the typical SRF cavities described in this paper: http://arxiv.org/pdf/physics/0003011.pdf

Each cell in the stack of cavities could be thought of as being two cones arranged back to back: <><><><> (see Figure 5). We would then expect any forces to try and pull the two half-cavities apart.

These were operated at 1.3 GHz, 200 kW CW RF, with a measured Q of ~ 5e9.

Applying the quoted formula of F ~ Po*Q/c, we get a force of 3e6 N.

The cavities have a wall thickness of about 2 mm, and a major radius of ~100 mm, giving a strain of > 2000 MPa in the niobium. The yield strength of niobium is somewhere in the range of 80-150 MPa, depending on temper and annealing. And yet the cavities did not fly apart.

Anyone care to poke holes in this?
Holes:

1) "Each cell in the stack of cavities could be thought of as being two cones arranged back to back: <><><><>"

The arrangement presupposed does not have an internal vertical plate at the location of the largest diameter.  It is a symmetrical arrangement without a wall at the biggest diameter.  The EM Drive's are asymmetric configurations.  WarpTech, StrongGR and Notsosureofit's theories for the EM Drive rely on asymmetry which is not present in the configuration you show. 



2) Extrapolating the equation  F ~ Po*Q/c to a Q of  5e9 is way beyond the range of any experimental measurement up to date.  The Q (measured the same way as in the West) of the highest measured EM Drive thrusts ever reported (experiments by Prof. Yang) are only ~10^3, and the Q's reported by NASA are ~10^3 to ~10^4.  Shawyer's Q was 5900 for the Experimental and 45000 for the Demo.  Thus, the experimental formula you are using was not based on any Q's less than 10^5. 

I had an exchange in the forum regarding this: TheTraveller favors using that formula in the Superconducting range (*).  I am on record stating that this does not make sense to me: there is no experimental data to make such an extrapolation (the thrust force of the experiment with Q~10^7 of the Cannae Superconducting was taken apart earlier in the thread -hat tip to zen-in- and it is very much in question: maximum force was less than 10 milliNewtons).  Shawyer has not reported (to our recollection) any thrust force measurements at Q's in the superconducting range.  The formulas advanced by Shawyer, McCulloch and Notsosureofit are also dependent on Q but they are not in a theoretical basis strong enough to warrant extrapolation beyond the range of experimental Q's either.

Using a Q ~10^9 is five orders of magnitude outside the range of an empirical formula based on experiments. 
Not a good thing to do (to extrapolate a formula much beyond the region where one has experimental points).

Just using your figures, but this time within the range of validity of the Q's experimental formula:

200MPa/10^5 = 2*10^-4 MPa

200 MPa/10^6 = 2*10^-5 MPa

both orders of magnitude smaller than 80-150 MPa.

NOTE: (*) If anything your example (when ignoring point #1 above) can be taken as a Reductio ad absurdum demonstrating that Todd (WarpTech) theory is correct (that Q shouldn't be too large)



Minor points:

3) " giving a strain of > 2000 MPa" . MegaPascals are a measure of stress and not strain.  Strain is a dimensionless quantity.
4) I won't go over into further detail on your stress analysis because it is not warranted, because the calculated force (see #2) is so many orders of magnitude outside of its experimental range.

Title: Re: EM Drive Developments - related to space flight applications - Thread 2
Post by: aero on 05/20/2015 07:25 pm
I'm incredibly appreciate of all the work that went into the dimensional estimates of the various drives but I'm having some difficulty understanding where the dimensional values came from regarding the attached image:

Shawyer reported a large plate diameter of .28m - so using that as a given.

Going straight off the image (not taking into account perspective) at the junction of the cone and the cylinder I get about .169m for the small plate diameter. This is a difference of 40mm with what was originally estimated and about 73mm from Dr. Rodal's calculation (96.13) of the small plate diameter.  This is an enormous difference and I can't believe it. I'm going to model this and lay it on top of the photo in perspective so I can find out what the numbers are closer to.

Also the cone length was estimated to be .345m - which is larger than the plate diameter. Between which two points is this measurement for?
That one was a challenge. Note the geared mechanism at the small end, end of the cylinder. It was considered that this mechanism drove a plunger that changed the length of the cavity in order to adjust the resonance frequency. Hence there needed to be some part of the diameter of the interior of the cylinder section devoted to supporting and guiding this plunger. How much was just a guess, but as I recall, I assumed that the actual small end of the resonant cavity was likely located about midway between the ends of the cylinder.

I think we have better information now so revisiting this estimate is warranted.
Title: Re: EM Drive Developments - related to space flight applications - Thread 2
Post by: StrongGR on 05/20/2015 07:42 pm
As promised I am posting the latest draft about general relativity and electromagnetic field. The relevant conclusion is that there is thrust. Thanks to the comments by Jose Rodal, it can be shown that this can be meaningful and the best geometry is that of the frustum tending to a cone. There is no violation of conservation law due to the presence of the gravity that can escape the device producing a reaction.

I post it here for your comments that are very welcome as usual. You can find the final equation at page 12 for your evaluations. Later on, I will post a version with a somewhat different presentation to arxiv.

I like you paper but have a request for a point of clarification. I got really confused when you went from equation 55 to equation 56. Why did you invert the c4/G term? (Actually I got confused way before that but I won't go there. :)

I have taken out a l0^-1 that so becomes l0^-2.
Title: Re: EM Drive Developments - related to space flight applications - Thread 2
Post by: StrongGR on 05/20/2015 07:48 pm
r2=10^5.5
r1=1
is a correct example.

It is my approximation: r2 increasingly large and r1 decreasing toward zero otherwise one should change the final formula. So, in your example it would be better to have r2=10 m and r1=10^-3 m and so on. This is a cone.

Yes. But the dependence on r2 is stronger than the one on r1, because r2 is to the sixth power whereas r1 is just squared.
So: if you keep r1 constant and vary r2, you don't need many orders of magnitude to have a reasonable effect; if you keep r2 constant and vary r1, you need to go down a lot of orders of magnitude.

If we have an r2 of one meter, r1 must be 10^-16, which is a tenth of a femtometer, which is less than the size of a proton. We cannot possibly manufacture such a cone and I think it would not work anyway because the resulting "small end plate" doesn't have enough electrons to behave like an ideal EM-reflecting plate. Although maybe it doesn't need to and it's just the cavity shape that matters.

What if we considered other shapes? We started with the truncated cone because of the EmDrive, but what if we considered a pillbox cavity (which is basically a rounded cone...)? All the equations would be different, for sure, but it might be worth exploring. Maybe in another thread. :)

Another way around, as suggested above, is to fill with some material the cavity. The formula goes like mur^-2 and mur much smaller than 1.

Yes, varying mu may be profitable too.

But then you have that U_0^4 that depends on Q^2 and P^2. As said before, I am not certain that I get a really macroscopic effect even if an interferometric device, sensible enough, can grant observation of the effect. I have to work out some numerics to see really what is going on. There is also the contribution coming from the square of the mode that can take the effect down.
Title: Re: EM Drive Developments - related to space flight applications - Thread 2
Post by: deltaMass on 05/20/2015 07:53 pm
You could always do a spot of partial differentiation to find the multidimensional optimum, if such exists.
Title: Re: EM Drive Developments - related to space flight applications - Thread 2
Post by: aero on 05/20/2015 08:07 pm
As promised I am posting the latest draft about general relativity and electromagnetic field. The relevant conclusion is that there is thrust. Thanks to the comments by Jose Rodal, it can be shown that this can be meaningful and the best geometry is that of the frustum tending to a cone. There is no violation of conservation law due to the presence of the gravity that can escape the device producing a reaction.

I post it here for your comments that are very welcome as usual. You can find the final equation at page 12 for your evaluations. Later on, I will post a version with a somewhat different presentation to arxiv.

I like you paper but have a request for a point of clarification. I got really confused when you went from equation 55 to equation 56. Why did you invert the c4/G term? (Actually I got confused way before that but I won't go there. :)

I have taken out a l0^-1 that so becomes l0^-2.

Sorry, but I still don't understand how that converts c4/G to (c4/G)-1.

I suspect that there is a transcription error somewhere, and that (c4/G)-1 was intended all along.
Title: Re: EM Drive Developments - related to space flight applications - Thread 2
Post by: ThereIWas3 on 05/20/2015 08:22 pm
A search of YouTube for the term "emdrive" turns up a bunch of interesting presentations, including this one (https://www.youtube.com/watch?v=Rbf7735o3hQ) by a "iulian" dated two days ago, showing his test setup and the measurement of thrust.
Title: Re: EM Drive Developments - related to space flight applications - Thread 2
Post by: StrongGR on 05/20/2015 08:40 pm
As promised I am posting the latest draft about general relativity and electromagnetic field. The relevant conclusion is that there is thrust. Thanks to the comments by Jose Rodal, it can be shown that this can be meaningful and the best geometry is that of the frustum tending to a cone. There is no violation of conservation law due to the presence of the gravity that can escape the device producing a reaction.

I post it here for your comments that are very welcome as usual. You can find the final equation at page 12 for your evaluations. Later on, I will post a version with a somewhat different presentation to arxiv.

I like you paper but have a request for a point of clarification. I got really confused when you went from equation 55 to equation 56. Why did you invert the c4/G term? (Actually I got confused way before that but I won't go there. :)

I have taken out a l0^-1 that so becomes l0^-2.

Sorry, but I still don't understand how that converts c4/G to (c4/G)-1.

I suspect that there is a transcription error somewhere, and that (c4/G)-1 was intended all along.

Thanks for pointing out this. I will check all tomorrow. Please, consider that a term c^4/G is coming out from the construction of the tensor in eq.(46). I have checked physical dimensions step by step by I'll redo it for sure.
Title: Re: EM Drive Developments - related to space flight applications - Thread 2
Post by: phaseshift on 05/20/2015 09:04 pm
I'm incredibly appreciate of all the work that went into the dimensional estimates of the various drives but I'm having some difficulty understanding where the dimensional values came from regarding the attached image:

Shawyer reported a large plate diameter of .28m - so using that as a given.

Going straight off the image (not taking into account perspective) at the junction of the cone and the cylinder I get about .169m for the small plate diameter. This is a difference of 40mm with what was originally estimated and about 73mm from Dr. Rodal's calculation (96.13) of the small plate diameter.  This is an enormous difference and I can't believe it. I'm going to model this and lay it on top of the photo in perspective so I can find out what the numbers are closer to.

Also the cone length was estimated to be .345m - which is greater than the large plate diameter. Between which two points is this measurement for?

I'm almost certain that the small plate diameter is close to .17m. This produces a design factor of .4853.  Where did you (Rodal) get the design factor of .844 that you used in calculating the small diameter of .09613m?
Title: Re: EM Drive Developments - related to space flight applications - Thread 2
Post by: Rodal on 05/20/2015 09:22 pm
Where did you (Rodal) get the design factor of .844 that you used in calculating the small diameter of .09613m?
Take a gander again  :) : I had meticulously answered that question in my message http://forum.nasaspaceflight.com/index.php?topic=36313.msg1376720#msg1376720 which you (phaseshift) quoted verbatim in your own message http://forum.nasaspaceflight.com/index.php?topic=36313.msg1376723#msg1376723
Title: Re: EM Drive Developments - related to space flight applications - Thread 2
Post by: phaseshift on 05/20/2015 09:38 pm
..Shawyer reported a large plate diameter of .28m - so using that as a given...
I'm almost certain that the small plate diameter is close to .17m. This produces a design factor of .5197. ...
No, it is even worse, using that value for the small diameter (0.17 m), the DesignFactor (calculated with this equation http://forum.nasaspaceflight.com/index.php?topic=36313.msg1374110#msg1374110 )  is even smaller:

smallDiameter = 0.17 m;
bigDiameter = 0.28 m;
f = 2.45*10^9 Hz;
cst = 1.7062895542683174;
cM = 299705000 m/s (speed of light in air);

results in the following DesignFactor

Design Factor = 0.4853 (instead of the 0.5197 value you quoted above)

Yes, I caught that just a minute ago.  its .4853
Title: Re: EM Drive Developments - related to space flight applications - Thread 2
Post by: phaseshift on 05/20/2015 09:49 pm
..Shawyer reported a large plate diameter of .28m - so using that as a given...
I'm almost certain that the small plate diameter is close to .17m. This produces a design factor of .5197. ...
No, it is even worse, using that value for the small diameter (0.17 m), the DesignFactor (calculated with this equation http://forum.nasaspaceflight.com/index.php?topic=36313.msg1374110#msg1374110 )  is even smaller:

smallDiameter = 0.17 m;
bigDiameter = 0.28 m;
f = 2.45*10^9 Hz;
cst = 1.7062895542683174;
cM = 299705000 m/s (speed of light in air);

results in the following DesignFactor

Design Factor = 0.4853 (instead of the 0.5197 value you quoted above)

Which is interesting given that his experimental thruster had a DF of .497.  So I don't know if he incorrectly reported .844 or the equation is wrong.  I can say for certain that the small plate diameter in the photo is not even close to 96mm - I couldn't be that much off with my eyes closed ;) lol
Title: Re: EM Drive Developments - related to space flight applications - Thread 2
Post by: phaseshift on 05/20/2015 09:58 pm
..Shawyer reported a large plate diameter of .28m - so using that as a given...
I'm almost certain that the small plate diameter is close to .17m. This produces a design factor of .5197. ...
No, it is even worse, using that value for the small diameter (0.17 m), the DesignFactor (calculated with this equation http://forum.nasaspaceflight.com/index.php?topic=36313.msg1374110#msg1374110 )  is even smaller:

smallDiameter = 0.17 m;
bigDiameter = 0.28 m;
f = 2.45*10^9 Hz;
cst = 1.7062895542683174;
cM = 299705000 m/s (speed of light in air);

results in the following DesignFactor

Design Factor = 0.4853 (instead of the 0.5197 value you quoted above)

Which is interesting given that his experimental thruster had a DF of .497.  So I don't know if he incorrectly reported .844 or the equation is wrong.  I can say for certain that the small plate diameter in the photo is not even close to 96mm - I couldn't be that much off with my eyes closed ;) lol
I can see people next stating that Shawyer just transposed the numbers: he meant to write

DesignFactor = 0.484 (which gives a sD = 0.17027 m)

but he wrote instead

DesignFactor =0.844   ;)

Yes, good possibility - I did the EXACT same thing about 10 minutes ago. :)
Title: Re: EM Drive Developments - related to space flight applications - Thread 2
Post by: Rodal on 05/20/2015 10:04 pm
...
I can see people next stating that Shawyer just transposed the numbers: he meant to write

DesignFactor = 0.484 (which gives a sD = 0.17027 m)

but he wrote instead

DesignFactor =0.844   ;)

Yes, good possibility - I did the EXACT same thing about 10 minutes ago. :)
I suggest that we wait for the person who has read most of Shawyer's papers (TheTraveller).  Perhaps TheTraveller can find another paper on the Demonstrator Engine by Shawyer besides the one I quoted, and check whether Shaywer quotes the same DesignFactor 0.844 or the more sensible number 0.484
Title: Re: EM Drive Developments - related to space flight applications - Thread 2
Post by: deltaMass on 05/20/2015 10:07 pm
I don't get it. Why would anyone use a Design Factor that wasn't (near as dammit) unity, since one has complete design freedom to do that?
Title: Re: EM Drive Developments - related to space flight applications - Thread 2
Post by: phaseshift on 05/20/2015 10:07 pm
I suggest that we wait for the person who has read most of Shawyer's papers (TheTraveller).  Perhaps TheTraveller can find another paper on the Demonstrator Engine by Shawyer besides the one I quoted, and check whether Shaywer quotes the same DesignFactor 0.844 or the more sensible number 0.484

This is my source: http://www.emdrive.com/IAC-08-C4-4-7.pdf
Title: Re: EM Drive Developments - related to space flight applications - Thread 2
Post by: Rodal on 05/20/2015 10:35 pm
I suggest that we wait for the person who has read most of Shawyer's papers (TheTraveller).  Perhaps TheTraveller can find another paper on the Demonstrator Engine by Shawyer besides the one I quoted, and check whether Shaywer quotes the same DesignFactor 0.844 or the more sensible number 0.484

This is my source: http://www.emdrive.com/IAC-08-C4-4-7.pdf
Meanwhile, if you have the time and find it worthwhile to do so, perhaps you could ascertain whether the following dimensions make sense

(* Shawyer Experimental *)
rfFrequency=2.45*10^9;
cavityLength=0.156;(estimated from photographs)
bigDiameter=0.16; (given by Shawyer)
smallDiameter=0.1025; (obtained from the Design Factor, bigDiameter and frequency provided by Shawyer)
Design Factor = 0.497;

Title: Re: EM Drive Developments - related to space flight applications - Thread 2
Post by: phaseshift on 05/20/2015 11:02 pm
I also think the cavity length needs to be adjusted on the Demonstrator Thruster to .187m.

My reasoning and measurements:

I believe Shawyer angled the cone to his expectation of where the small diameter plate will be.  When the device is turned on the plate is slightly back inside the cylinder and gradually moved forward until there is phase lock. The length of the cylinder, in part, is to accommodate the movement mechanism and the volume of the small plate.

In my recent model the cone length is exactly .187m (from the face of the small plate to the face of the large plate). Slightly longer than the .183m calculated from 3 * .123m / 2 - based on .2450Ghz. I think 4mm is ample distance to move the small plate to achieve phase lock as I described above.

Did I do the math right?

Title: Re: EM Drive Developments - related to space flight applications - Thread 2
Post by: hhexo on 05/21/2015 12:04 am
But then you have that U_0^4 that depends on Q^2 and P^2. As said before, I am not certain that I get a really macroscopic effect even if an interferometric device, sensible enough, can grant observation of the effect. I have to work out some numerics to see really what is going on. There is also the contribution coming from the square of the mode that can take the effect down.

Let's keep working this out.

Hm... from equation (22) I get that a volume integral of the square of the mode times U0/2mu0 is equal to QP/omega. Omega is the resonant angular frequency (which I think is the linear frequency divided by 2pi).
Q is about 10^4 (for the devices we've seen).
Let's say P is about 10^3 (1KW).
omega is about 10^9 (because gigahertz).
mu0 is still about 10^-6.

I don't know about that volume integral, but mu0*QP/omega is about 10^-8. :( Unless the volume integral is a substantially negative power of ten, this isn't helping much. Also, it looks like increasing the resonance frequency actually makes things worse (is that right?!).
U0^4 becomes about 10^-32 divided by the fourth power of that volume integral.

This effect is starting to become too small. :(
Still, these are just back-of-the-envelope ballpark calculations, and I can't figure out all those integrals as my maths isn't good enough. So, as you say, we need proper numerics.
I'm just not too hopeful. :(
Title: Re: EM Drive Developments - related to space flight applications - Thread 2
Post by: phaseshift on 05/21/2015 12:16 am
I suggest that we wait for the person who has read most of Shawyer's papers (TheTraveller).  Perhaps TheTraveller can find another paper on the Demonstrator Engine by Shawyer besides the one I quoted, and check whether Shaywer quotes the same DesignFactor 0.844 or the more sensible number 0.484

This is my source: http://www.emdrive.com/IAC-08-C4-4-7.pdf
Meanwhile, if you have the time and find it worthwhile to do so, perhaps you could ascertain whether the following dimensions make sense

(* Shawyer Experimental *)
rfFrequency=2.45*10^9;
cavityLength=0.156;(estimated from photographs)
bigDiameter=0.16; (given by Shawyer)
smallDiameter=0.1025; (obtained from the Design Factor, bigDiameter and frequency provided by Shawyer)
Design Factor = 0.497;

Almost an exact match!  small diameter was .100m and cavityLength was .155m.  2.5 millimeters off on the small diameter is easily within my margin of error.

This is also supports the correctness of the DF equation. :)
Title: Re: EM Drive Developments - related to space flight applications - Thread 2
Post by: aero on 05/21/2015 12:32 am
Regarding the Demo cavity dimensions - It would be very helpful if someone with knowledge and tools for image interpretation were to look at the Demo cavity as was done for the Flight thruster. In looking at the image again, it appears to me like there might be a cylinder section on both ends. The one on the small end is obvious but is there a short cylinder between the two flanges at the big end? And if so, why? Did Shawyer make his Demo thruster as two cylinders joined by a conic section?
Title: Re: EM Drive Developments - related to space flight applications - Thread 2
Post by: phaseshift on 05/21/2015 12:46 am
Regarding the Demo cavity dimensions - It would be very helpful if someone with knowledge and tools for image interpretation were to look at the Demo cavity as was done for the Flight thruster. In looking at the image again, it appears to me like there might be a cylinder section on both ends. The one on the small end is obvious but is there a short cylinder between the two flanges at the big end? And if so, why? Did Shawyer make his Demo thruster as two cylinders joined by a conic section?

aero,

I considered that when analyzing the dimensions - you don't even need image processing software - look at the reflection of the "workbench" on the cylinder, then on the cone, then on the next 'cylinder'.  It's rather obvious that it is a cylinder rather than more of the cone.  I believe given the dimensions (thickness of this area), and the possibility of a concave surface on the large plate that the larger cylinder contains the large plate.  For some reason, perhaps just because he was still figuring out construction methods, Shawyer did it this way rather than the fully external plate on the higher fidelity devices.
Title: Re: EM Drive Developments - related to space flight applications - Thread 2
Post by: aero on 05/21/2015 01:39 am
Regarding the Demo cavity dimensions - It would be very helpful if someone with knowledge and tools for image interpretation were to look at the Demo cavity as was done for the Flight thruster. In looking at the image again, it appears to me like there might be a cylinder section on both ends. The one on the small end is obvious but is there a short cylinder between the two flanges at the big end? And if so, why? Did Shawyer make his Demo thruster as two cylinders joined by a conic section?

aero,

I considered that when analyzing the dimensions - you don't even need image processing software - look at the reflection of the "workbench" on the cylinder, then on the cone, then on the next 'cylinder'.  It's rather obvious that it is a cylinder rather than more of the cone.  I believe given the dimensions (thickness of this area), and the possibility of a concave surface on the large plate that the larger cylinder contains the large plate.  For some reason, perhaps just because he was still figuring out construction methods, Shawyer did it this way rather than the fully external plate on the higher fidelity devices.

I think in the past we were enthralled by the geometry of the Eagleworks thruster cavity. Are you confirming that you think there are cylinders on both ends of the Demo frustum section? If so, then how long is, (what is the height of), the big end cylinder (estimate). Then what is the height of the frustum section? Lastly, what is your estimate of the length of the small cylinder? That is, what is the overall electrical length of the EM thruster "Demo"?

Now, can these pieces be related to the terms used in Shawyer's design factor equation?
Title: Re: EM Drive Developments - related to space flight applications - Thread 2
Post by: phaseshift on 05/21/2015 01:44 am
Regarding the Demo cavity dimensions - It would be very helpful if someone with knowledge and tools for image interpretation were to look at the Demo cavity as was done for the Flight thruster. In looking at the image again, it appears to me like there might be a cylinder section on both ends. The one on the small end is obvious but is there a short cylinder between the two flanges at the big end? And if so, why? Did Shawyer make his Demo thruster as two cylinders joined by a conic section?

aero,

I considered that when analyzing the dimensions - you don't even need image processing software - look at the reflection of the "workbench" on the cylinder, then on the cone, then on the next 'cylinder'.  It's rather obvious that it is a cylinder rather than more of the cone.  I believe given the dimensions (thickness of this area), and the possibility of a concave surface on the large plate that the larger cylinder contains the large plate.  For some reason, perhaps just because he was still figuring out construction methods, Shawyer did it this way rather than the fully external plate on the higher fidelity devices.

Are you confirming that you think there are cylinders on both ends of the frustum section? If so, then how long is, (what is the height of), the big end cylinder (estimate). Then what is the height of the frustum section? Lastly, what is your estimate of the length of the small cylinder? That is, what is the overall electrical length of the EM thruster "Demo"?

Now, can these pieces be related to the terms used in Shawyer's design factor equation?

Yes, I am saying that there are two cylinders.

See above for dimensions. :)  Those are the dimensions I got from the model - the thickness of the large plate is not terribly important - thick enough to contain a spherically concave surface if it is there.  The small plate is at the end of the cone.  Given the possibility of Shawyer mistyping his DF of .844 when it should have been .484 the numbers all work.
Title: Re: EM Drive Developments - related to space flight applications - Thread 2
Post by: aero on 05/21/2015 02:00 am
Quote
Yes, I am saying that there are two cylinders.

See above for dimensions. :)  Those are the dimensions I got from the model - the thickness of the large plate is not terribly important - thick enough to contain a spherically concave surface if it is there.  The small plate is at the end of the cone.  Given the possibility of Shawyer mistyping his DF of .844 when it should have been .484 the numbers all work.
Could you provide the link to the dimensions that you refer to? "See above" leaves a lot of room for me to choose the wrong post. And the length of the cylinder on the big end is kind of important if I were to model it with FDTD software (Meep), as is the length and radius of the small end cylinder and length of the frustum section. Not that I have any immediate plans to do so, but I might model it in 2D just for grins. It could be interesting to see what the wave forms might look like.
Title: Re: EM Drive Developments - related to space flight applications - Thread 2
Post by: phaseshift on 05/21/2015 02:11 am
Quote
Yes, I am saying that there are two cylinders.

See above for dimensions. :)  Those are the dimensions I got from the model - the thickness of the large plate is not terribly important - thick enough to contain a spherically concave surface if it is there.  The small plate is at the end of the cone.  Given the possibility of Shawyer mistyping his DF of .844 when it should have been .484 the numbers all work.
Could you provide the link to the dimensions that you refer to? "See above" leaves a lot of room for me to choose the wrong post. And the length of the cylinder on the big end is kind of important if I were to model it with FDTD software (Meep), as is the length and radius of the small end cylinder and length of the frustum section. Not that I have any immediate plans to do so, but I might model it in 2D just for grins. It could be interesting to see what the wave forms might look like.

My apologies, I thought the post was only a few before this one but it's actually spread out over the last couple pages.

Large diameter: .28m - given by Shawyer
Small diameter: .17m - calculated using .484 DF - measured to be .169m
Cone Length (surface to surface): .187m (measured)

This is why the lengths of the cylinders don't matter - the plate surfaces are at the ends of the cone section.  Though in the case of the small plate it probably starts about 5mm inside the cylinder and is moved forward until there is phase lock - it "should" be at the cone end. Either way the cylinders can be any size as long as they are large enough to contain whatever it is they contain - the large one by the way is "about" an inch thick which is also what I would calculate for the depression on a spherical surface. - I can determine that more precisely if you want but it will have to wait till tomorrow :)

Cheers

Title: Re: EM Drive Developments - related to space flight applications - Thread 2
Post by: Rodal on 05/21/2015 02:33 am
As promised I am posting the latest draft about general relativity and electromagnetic field. The relevant conclusion is that there is thrust. Thanks to the comments by Jose Rodal, it can be shown that this can be meaningful and the best geometry is that of the frustum tending to a cone. There is no violation of conservation law due to the presence of the gravity that can escape the device producing a reaction.

I post it here for your comments that are very welcome as usual. You can find the final equation at page 12 for your evaluations. Later on, I will post a version with a somewhat different presentation to arxiv.

I like you paper but have a request for a point of clarification. I got really confused when you went from equation 55 to equation 56. Why did you invert the c4/G term? (Actually I got confused way before that but I won't go there. :)

I have taken out a l0^-1 that so becomes l0^-2.

Sorry, but I still don't understand how that converts c4/G to (c4/G)-1.

I suspect that there is a transcription error somewhere, and that (c4/G)-1 was intended all along.

Thanks for pointing out this. I will check all tomorrow. Please, consider that a term c^4/G is coming out from the construction of the tensor in eq.(46). I have checked physical dimensions step by step by I'll redo it for sure.
I don't see any problem with the units the way they are in Eq. 55 and 56.

I see a units problem if you do what @aero is proposing.


Units of (G/c^4) from Eq. 23 is 1/Force,
hence (c^4)/G has units of Force

Units of L from Eq. 55 is  ((c^4)/G )/(length^2)
hence L has units of Force/(Length^2)
L has units of stress (force per surface area)

(Uo)2o has units of Energy per unit volume (Energy density)
(Marco made this clear in Eq. 22 for example)

Eq. 56 has units of (G/c^4) *(Length^2)*((Energy/Volume)^2)
hence (1/Force)*(Length^2)*(Force*Length/(Length^3))^2
hence (1/Force)*(Length^2)*(Force^2)/(Length^4)
hence Force/(Length^2) , which are the units of stress, which are the proper units of L from Eq. 55


Therefore the factor of G/(c^4) in Eq. 56 has the correct units, and it is the proper factor to use. 
Ditto for Eq. 55 you have to use (c^4)/G in order for L to have the units of stress.
There is no problem with the units in Eq. 55 or in Eq. 56.

On the other hand if you make the change proposed by @aero, the units get screwed up.



The answer to @aero's question,(Why did you invert the c4/G term?) is:
that term is due to the equality in Eq. 28 which you used to substitute for lo, which introduces the inverse square of  c4/G .  Also notice that that's the reason that the term (Pi)^2 appeared in Eq. 56. 

This makes perfect sense because that term is the proper term needed to have an equality hold, otherwise one cannot have an equal sign in Eq. 55 and Eq. 56


@aero: Look at Eq. 28 and Eq. 23.  You need to account for the equalities in Eq. 28 and Eq. 23 to follow what happened. You need to substitute the value of lo (from Eq. 28) into Eq. 55, so that Eq. 55 is no longer in terms of lo.

Then set r= r1 in Eq. 55 and take the limit of Eq. 55 for r1/r2 -> 0
Title: Re: EM Drive Developments - related to space flight applications - Thread 2
Post by: aero on 05/21/2015 03:02 am
I guess the step between Eqn. 55 and Eqn 56 is just to large for me.
Title: Re: EM Drive Developments - related to space flight applications - Thread 2
Post by: WarpTech on 05/21/2015 04:52 am
I guess the step between Eqn. 55 and Eqn 56 is just to large for me.

Marco loses me at the end there too. Difficult to follow it all the way through, damn index gymnastics makes my eyes glaze over. One thing to note however, equation 3;

alpha*w = sqrt(4pi*eps0*G) has units of Coulombs/kg.

alpha*w*E has units of acceleration, in the sense that this is a Lorentz force q*E/m, divided by mass. Some people believe this is a gravitational acceleration derived from the EM field, but I do not see it as such. It is many orders of magnitude larger than gravity would be. This is essentially the Planck charge / Planck mass

Todd
Title: Re: EM Drive Developments - related to space flight applications - Thread 2
Post by: WarpTech on 05/21/2015 05:12 am
Warptech

<snip>

Ok
If Im following your thought train correctly then, from my limited perspective on handling microwaves.

The generation side of the circuit requires cyclic refreshing to achieve resonance. {Unknown time-element at the moment}

Once resonance is achieved, {and not before} the energy is dumped into the "Load".

The operational functionality requires simplicity of operation.

The functionality requires the ability to alter running characteristics in realtime.

The length of the initial resonance chamber {ideally} needs to be automatically configurable {hence shawyers use of piezoelectric actuators inside his system}

The mechanical nature of the beam chopper requires simple operation but also an ability to tune in relation to time taken to achieve resonance compared with port-opening cycles. {potentially a software function to find the maximum thrust by adjusting resonance/port operation timing automatically, Microcontrollers are good for this, I use Arduino}

Thinking about the end product it may be easier engineering wise to have multiple attenuation chambers. This allows simplicity in design and beam chopper operation.

2 attenuators at 180, single beam splitter opening with weight adjusted disk to account for mass removed on one side.{less efficient model}

4 attenuation chambers equally spaced so any two are logically 180 degrees from each other, Both can be fed from the resonant chamber simultaneously via 2 opposed holes in the beam splitter. This also allows for slower rotation rate of the splitter as it has 2 holes not 1.

A variable speed rotary port opening  mechanism. The shape of the beam choppers pass-through-port determines the efficiency of the opening process, square, circle, ellipse, triangular

For shorter port opening times use 2 disks counter rotating with respect to each other. {or just smaller port openings}.

Personally I tend to favour 4 attenuators because 2 will be simultaneously active while the resonator recharges to pump the other 2 attenuators. {this is also because I have no idea how long it will take to attenuate the signals in relation to achieving resonance}

? any use or.. just junk?
Terrible drawing but you get the basic concept...

...

This is great arc! You understood what I meant exactly. I had this idea too (without the jockey shorts in the middle there), but I think it's better to use a single resonant cavity for each attenuator and an Iris diaphragm shutter. The advantage being, less distortion of the cylindrical waves as they enter the attenuator because the iris remains circular, and also the ability to use the iris as a high-pass filter for testing purposes.

Regarding the duty cycle, it will depend on the input power, the ability to store energy and the desired thrust. Just keep in mind, if you had the same amplifier setup without the frustum attenuator, it would be more efficient as a microwave photon rocket. I'm afraid this is why I'm not taking the EM drive so seriously anymore. Once I realized I can gain maximum efficiency by just letting the stored energy out as thrust, from a pulsed microwave cavity source of power Q*P, there's not much point in bottling it up in a frustum. The only advantage I see is that it can be used "safely", as opposed to cooking everything down-wind of the exhaust. In space, who cares?

Todd



Title: Re: EM Drive Developments - related to space flight applications - Thread 2
Post by: deltaMass on 05/21/2015 05:42 am
The task of the theorist, surely, is to figure out why the time-averaged thrust is about 1000x that of an equivalently-powered photon rocket in continuous mode. Isn't it?
Title: Re: EM Drive Developments - related to space flight applications - Thread 2
Post by: deltaMass on 05/21/2015 05:47 am
IMO the fastest way to put this to bed or elevate it to the status of a major new propulsion paradigm (let alone new physics) is to stuff it onto a CubeSat and test it in space. And the only way that's going to happen is using laser frequencies, else it won't physically fit.
Title: Re: EM Drive Developments - related to space flight applications - Thread 2
Post by: ThinkerX on 05/21/2015 06:14 am
Quote
Regarding the duty cycle, it will depend on the input power, the ability to store energy and the desired thrust. Just keep in mind, if you had the same amplifier setup without the frustum attenuator, it would be more efficient as a microwave photon rocket. I'm afraid this is why I'm not taking the EM drive so seriously anymore. Once I realized I can gain maximum efficiency by just letting the stored energy out as thrust, from a pulsed microwave cavity source of power Q*P, there's not much point in bottling it up in a frustum. The only advantage I see is that it can be used "safely", as opposed to cooking everything down-wind of the exhaust. In space, who cares?

One other possible 'sideways' advantage to the frustum route is this device will generate a lot of heat, which could be converted back into electrical energy with a bit of clever engineering.  Won't get anywhere near unity, but should cut down on the power bill.  Might not be near as much waste heat without the frustum.
 
Title: Re: EM Drive Developments - related to space flight applications - Thread 2
Post by: The Amazing Catstronaut on 05/21/2015 07:37 am
If or when power levels ramp up to "considerable" watts.. we might see unusual things emerge in the environment immediately around the drive. 


I appreciate that you don't have any hard numbers yet to support your conjecture, but what, roughly, do you consider to be "considerable watts"?
Title: Re: EM Drive Developments - related to space flight applications - Thread 2
Post by: StrongGR on 05/21/2015 08:02 am
But then you have that U_0^4 that depends on Q^2 and P^2. As said before, I am not certain that I get a really macroscopic effect even if an interferometric device, sensible enough, can grant observation of the effect. I have to work out some numerics to see really what is going on. There is also the contribution coming from the square of the mode that can take the effect down.

Let's keep working this out.

Hm... from equation (22) I get that a volume integral of the square of the mode times U0/2mu0 is equal to QP/omega. Omega is the resonant angular frequency (which I think is the linear frequency divided by 2pi).
Q is about 10^4 (for the devices we've seen).
Let's say P is about 10^3 (1KW).
omega is about 10^9 (because gigahertz).
mu0 is still about 10^-6.

I don't know about that volume integral, but mu0*QP/omega is about 10^-8. :( Unless the volume integral is a substantially negative power of ten, this isn't helping much. Also, it looks like increasing the resonance frequency actually makes things worse (is that right?!).
U0^4 becomes about 10^-32 divided by the fourth power of that volume integral.

This effect is starting to become too small. :(
Still, these are just back-of-the-envelope ballpark calculations, and I can't figure out all those integrals as my maths isn't good enough. So, as you say, we need proper numerics.
I'm just not too hopeful. :(

Just working! ;)
Title: Re: EM Drive Developments - related to space flight applications - Thread 2
Post by: Rodal on 05/21/2015 12:06 pm
This thread has gone over 1 million views !!!!


(http://i457.photobucket.com/albums/qq295/tugger29/Animated%20Gifs/anchormanhooray.gif)
(http://media.giphy.com/media/dM46pY2CMvWuc/giphy.gif)
Title: Re: EM Drive Developments - related to space flight applications - Thread 2
Post by: Notsosureofit on 05/21/2015 12:28 pm
I guess the step between Eqn. 55 and Eqn 56 is just to large for me.

Marco loses me at the end there too. Difficult to follow it all the way through, damn index gymnastics makes my eyes glaze over. One thing to note however, equation 3;

alpha*w = sqrt(4pi*eps0*G) has units of Coulombs/kg.

alpha*w*E has units of acceleration, in the sense that this is a Lorentz force q*E/m, divided by mass. Some people believe this is a gravitational acceleration derived from the EM field, but I do not see it as such. It is many orders of magnitude larger than gravity would be. This is essentially the Planck charge / Planck mass

Todd

Yes the apparent acceleration (of the photons in the chamber) is of the order of 10^15 m/s^2 in my calculation.  very large.
Title: Re: EM Drive Developments - related to space flight applications - Thread 2
Post by: Rodal on 05/21/2015 12:40 pm
I guess the step between Eqn. 55 and Eqn 56 is just to large for me.

Marco loses me at the end there too. Difficult to follow it all the way through, damn index gymnastics makes my eyes glaze over. One thing to note however, equation 3;

alpha*w = sqrt(4pi*eps0*G) has units of Coulombs/kg.

alpha*w*E has units of acceleration, in the sense that this is a Lorentz force q*E/m, divided by mass. Some people believe this is a gravitational acceleration derived from the EM field, but I do not see it as such. It is many orders of magnitude larger than gravity would be. This is essentially the Planck charge / Planck mass

Todd

Yes the apparent acceleration (of the photons in the chamber) is of the order of 10^15 m/s^2 in my calculation.  very large.
and let's recall that this apparent acceleration of the photons (of the order of 10^15 m/s^2)  in the cavity is invoked by Dr. McCulloch for his theory of Unruh radiation being responsible for the EM Drive's change in momentum  http://www.ptep-online.com/index_files/2015/PP-40-15.PDF
Title: Re: EM Drive Developments - related to space flight applications - Thread 2
Post by: TheTraveller on 05/21/2015 12:45 pm
Following Roger Shawyers kindly laid bread crumb trail, my EM Drive spreadsheet now can calc the effective internal guide wavelength and external Rf wavelength that will give resonance from end plate to end plate.

As an example for the Flight Thruster big and small end diameters as below, the required end plate to end plate spacing to achieve resonance with an external Rf of 3.85GHz is as below.

Alteration of either the big, small end or Rf frequency will now automatically generate a new Df and from that the end plate spacing needed to achieve resonance with the external Rf.

big diameter      m   0.2440000
small diameter   m   0.1450000
cavity length      m   0.1603484
rf frequency       Hz   3,850,000,000
Calculated Df      Df   0.49094
slant angle        Deg   28.8

1,000 point numerically integrated guide wavelength of the above example is: 0.0801741816

I will publish the spreadsheet but would 1st like to run / verify it against other frustum dimensions and what the calculated / measured resultant resonance was and in what mode.

With this spreadsheet if we know either end diameter, Rf frequency and Df, the other diameter and spacing can now be determined as all 3 dimensions and external Rf wavelength (4 variables) affect each other.

NEXT STEPS:

1) Determine the best way to inject the coax Rf into the Flight Thruster? Loop or Stub?

2) Determine the best location to inject the Rf into the Flight Thruster?

3) Determine the best way to impedance match the Flight Thruster to the impedance of the Rf generator so as to get optimal VSWR and energy delivery to inside the Flight Thruster?
Title: Re: EM Drive Developments - related to space flight applications - Thread 2
Post by: Voidman2015 on 05/21/2015 01:49 pm
Greetings,

I am currently playing catchup (page 151), so my apologies if this has already been talked about. My question is how is the acceleration witness from the emdrive different from the Pioneer anomaly? Full disclosure time; I am not a scientist, but I am a fan of physics.  Thank you for the great conversations on this forum.
http://en.m.wikipedia.org/wiki/Pioneer_anomaly

Title: Re: EM Drive Developments - related to space flight applications - Thread 2
Post by: Rodal on 05/21/2015 02:01 pm
Greetings,

I am currently playing catchup (page 151), so my apologies if this has already been talked about. My question is how is the acceleration witness from the emdrive different from the Pioneer anomaly? Full disclosure time; I am not a scientist, but I am a fan of physics.  Thank you for the great conversations on this forum.
http://en.m.wikipedia.org/wiki/Pioneer_anomaly
Hi and welcome,

The Pioneer anomaly effect (due to radiation of heat) is an extremely small acceleration of (8.74±1.33)×10^(−10) m/s^2, which is equivalent to slowly accelerating to a velocity of 1 kilometre per hour (0.6 mph) over a period of ten years.

Thus, it is ~thousands of times smaller than the acceleration researchers claim in the EM Drive experiments.
Title: Re: EM Drive Developments - related to space flight applications - Thread 2
Post by: Voidman2015 on 05/21/2015 02:13 pm
Rodal, thank you for the clarification. I am looking forward to reading more on this wonderful discussion. One quick stupid question: would not the shape of the cavity and the concentration of radiation in the cavity account for the higher acceleration reported? Or am I just way off?

Thank you again.
Title: Re: EM Drive Developments - related to space flight applications - Thread 2
Post by: OttO on 05/21/2015 02:40 pm
Stumbled on this one:

http://arxiv.org/abs/1503.06334 (http://arxiv.org/abs/1503.06334)

I think it can interest @Aero

Title: Re: EM Drive Developments - related to space flight applications - Thread 2
Post by: Rodal on 05/21/2015 02:52 pm
Stumbled on this one:

http://arxiv.org/abs/1503.06334 (http://arxiv.org/abs/1503.06334)

I think it can interest @Aero
Thanks.

For more background, see the following http://en.wikipedia.org/wiki/Faster-than-light#Faster_light_.28Casimir_vacuum_and_quantum_tunnelling.29 which discusses the work by  Guenter Nimtz,  author of the article

This is a quote from that section of the Wiki article:

Quote
They say they have conducted an experiment in which microwave photons—relatively low energy packets of light—travelled "instantaneously" between a pair of prisms that had been moved up to 3 ft (1 m) apart. Their experiment involved an optical phenomenon known as "evanescent modes", and they claim that since evanescent modes have an imaginary wave number, they represent a "mathematical analogy" to quantum tunnelling.[32] Nimtz has also claimed that "evanescent modes are not fully describable by the Maxwell equations and quantum mechanics have to be taken into consideration."[48] Other scientists such as Herbert G. Winful and Robert Helling have argued that in fact there is nothing quantum-mechanical about Nimtz's experiments, and that the results can be fully predicted by the equations of classical electromagnetism (Maxwell's equations).[49][50]

Nimtz told New Scientist magazine: "For the time being, this is the only violation of special relativity that I know of." However, other physicists say that this phenomenon does not allow information to be transmitted faster than light. Aephraim Steinberg, a quantum optics expert at the University of Toronto, Canada, uses the analogy of a train traveling from Chicago to New York, but dropping off train cars at each station along the way, so that the center of the ever shrinking main train moves forward at each stop; in this way, the speed of the center of the train exceeds the speed of any of the individual cars.[51]
Title: Re: EM Drive Developments - related to space flight applications - Thread 2
Post by: OttO on 05/21/2015 02:55 pm
I try to follow this thread from the beginning and I must thanks all the people posting here.
From my side the reality of the thrust is not the most important. What is important is the things I learned here.

I shall confess that the majority of the stuff is way off my poor head.

@rodal thank you for the link :-) I will try it...
You asked an interesting question a while ago, what is peculiar about copper frustum?
Well I think that the mirrors are interesting.

Would it be silly to think of a kind of Hawking radiation to "deflate" the frustum?
Title: Re: EM Drive Developments - related to space flight applications - Thread 2
Post by: Rodal on 05/21/2015 02:59 pm
...Would it be silly to think of a kind of Hawking radiation to "deflate" the frustum?
McCulloch's explanation ( http://www.ptep-online.com/index_files/2015/PP-40-15.PDF ) is essentially akin to Hawking radiation, as Unruh radiation is akin to Hawking radiation
Title: Re: EM Drive Developments - related to space flight applications - Thread 2
Post by: TheTraveller on 05/21/2015 03:24 pm
As a sanity check, what is the mode shape you predict for the above "resonance" calculation ?

Do you predict resonance at a Transverse Magnetic or Transverse Electric mode ?
what are the predicted mode shape quantum number m,n,p values ?

TMmnp  ?
TEmnp   ?

Thanks

Roger's bread crumb:

Quote
You need to develop a numerical model that calculates the guide wavelength, for the chosen mode, at discrete small increments along the cavity length and then integrate them into an effective wavelength for the whole cavity.

Which I followed and used the guide wavelengths, generated from the Df equation, at 1,000 equally spaced diameter points between and including the end plates.
Title: Re: EM Drive Developments - related to space flight applications - Thread 2
Post by: rfmwguy on 05/21/2015 03:25 pm
...Would it be silly to think of a kind of Hawking radiation to "deflate" the frustum?
McCulloch's explanation ( http://www.ptep-online.com/index_files/2015/PP-40-15.PDF ) is essentially akin to Hawking radiation, as Unruh radiation is akin to Hawking radiation

Thanks Doc... had not read this pdf. Helps unravel some of the theory for my practical, non-theoretical physics mind. I had actually been thinking about a non-frustum shape previously and there it was in this paper:

"The effect could be increased by increasing the degree of taper, for example using a pointed cone."

OK, I know what you're thinking...pointed cone...pointed rectangle...pyramid...So, perhaps I should add a Pyramid to my Die Glocke-shaped frustrum fears  ;)
Title: Re: EM Drive Developments - related to space flight applications - Thread 2
Post by: Rodal on 05/21/2015 03:25 pm
Rodal, thank you for the clarification. I am looking forward to reading more on this wonderful discussion. One quick stupid question: would not the shape of the cavity and the concentration of radiation in the cavity account for the higher acceleration reported? Or am I just way off?

Thank you again.
The Pioneer anomaly acceleration is towards the Sun, with the radiated heat (T1) occurring away from the Sun, at the back of the Pioneer probe facing the Sun.  The temperature (T2) around the EM Drive experiments (+68 F (+20 C)?) is much warmer than the background temperature (T2) at the back of the Pioneer probe (- 292 F (-180 C) around Saturn, for example).  The radiative heat transfer goes like the difference of the (absolute) temperatures to the fourth power:

(http://upload.wikimedia.org/math/9/c/a/9ca1177f3d75ec3bb4a98ab7ce668297.png)
Title: Re: EM Drive Developments - related to space flight applications - Thread 2
Post by: Rodal on 05/21/2015 03:35 pm
As a sanity check, what is the mode shape you predict for the above "resonance" calculation ?

Do you predict resonance at a Transverse Magnetic or Transverse Electric mode ?
what are the predicted mode shape quantum number m,n,p values ?

TMmnp  ?
TEmnp   ?

Thanks

Roger's bread crumb:

Quote
You need to develop a numerical model that calculates the guide wavelength, for the chosen mode, at discrete small increments along the cavity length and then integrate them into an effective wavelength for the whole cavity.

Which I followed and used the guide wavelengths, generated from the Df equation, at 1,000 equally spaced diameter points between and including the end plates.
In other words, I understand that you have developed a spreadsheet model predicting the correct geometry for resonance to occur at, but that you cannot predict whether the resonance occurs in a Transverse Magnetic mode, or a Transverse Electric mode, and that you cannot predict what is the resonance mode shape variation (m. n, p numberrs) in the longitudinal, transverse and azimuthal directions of the truncated cone.

Therefore we cannot check whether your ("Roger's bread crumb" ?) predicted resonance is correct.   :(
Title: Re: EM Drive Developments - related to space flight applications - Thread 2
Post by: hhexo on 05/21/2015 03:39 pm
McCulloch's explanation ( http://www.ptep-online.com/index_files/2015/PP-40-15.PDF ) is essentially akin to Hawking radiation, as Unruh radiation is akin to Hawking radiation

Hm... I read McCulloch's paper and it seems quite interesting, although the maths is again too much for me.
However, by following links and expanding the range of the bibliography I happened upon this: http://arxiv.org/abs/1105.4714
Yet again, too much maths, but it got me thinking.

The dynamic Casimir effect happens when an EM mirror moves at relativistic speed, and results in photon production in a narrow band.
Now, we know that the two plates in the EmDrive do not move. However, the plates are EM mirrors because of a "sea" of electron quantum states, which change when a wave hits the plate and is reflected. Could it be that the distribution of quantum states is somehow "oscillating" because of the resonance? Could this affect reflection in a "cycle"? In this case, we would have an "oscillating mirror" at each plate. There could be asymmetry factors (e.g. difference in plate size, or different "oscillation frequency") that cause an imbalance of dynamic Casimir effects between the two plates, leading to unbalanced photon production and net thrust.
(so, the net thrust from Maxwell's equations would be zero, but the high values for the oscillation of the EM field causing oscillations in the material's quantum states would create situations where Casimir effects are measurable)
(also, in the case of a dielectric the "mirror" might oscillate more?)

The first counter-argument to my own theory I can think of is that resonance is akin to a "standing wave" so there shouldn't be any oscillation at the boundary nodes. But reality might be more complex than that.
The second counter-argument to this hypothesis is that Casimir effects are minuscule and shouldn't be macroscopic. But I haven't even tried to run the numbers.

Also, I can't do the maths to do a proper derivation of any equation for this. :(

Please debunk me! :)
Title: Re: EM Drive Developments - related to space flight applications - Thread 2
Post by: WarpTech on 05/21/2015 04:06 pm
Warptech

Just had another look and your right.. jockey shorts.. laughing.

The iris idea is good.
Two counter rotating disks with circular holes could achieve the same thing at higher speed and minimal mechanical effort, better reliability.  My experience with sliding mechanical objects has usually not been that good over a reasonable time.... just saying.


I have been thinking about the cavity-resonance element of the em thruster.
I may be totally wrong but I strongly suspect there is far more than just em photons bouncing around inside a tin-can happening. The wave fronts momentum "could"? have a part to play in it but I am pursuing the trail of increased-energy-level confined to a small area, and the non-traditional events that may arise. My conjecture is that the Shawyer em-drive is in a very-mild way (at present power levels) interacting/distorting/impacting/modifying/???? space-time and hence its partner, Gravity, on a local level. If or when power levels ramp up to "considerable" watts.. we might see unusual things emerge in the environment immediately around the drive. 

The internal wavefronts inside the drive might somehow be helping to alternately create and then distort the shape and location of a G-modification?.

If the guys at NASA are reading this, are you able to up-the-watts, and test for G-modification not just inside but also around the device??.  {please santa}

I dont have any proof to support my conjecture....yet

I have a similar conjecture, but IMO it "mimics" gravity. The effect is not caused by a massive planet or high energy density however. It is caused by the attenuation of the wave due to the "effective mass" imposed on photons by the waveguide cut-off aperture. Go back and read some of my earlier posts, when I was working all this out for the 1st time.

http://forum.nasaspaceflight.com/index.php?topic=36313.msg1369287#msg1369287

Todd
Title: Re: EM Drive Developments - related to space flight applications - Thread 2
Post by: SeeShells on 05/21/2015 04:29 pm
As a sanity check, what is the mode shape you predict for the above "resonance" calculation ?

Do you predict resonance at a Transverse Magnetic or Transverse Electric mode ?
what are the predicted mode shape quantum number m,n,p values ?

TMmnp  ?
TEmnp   ?

Thanks

Roger's bread crumb:

Quote
You need to develop a numerical model that calculates the guide wavelength, for the chosen mode, at discrete small increments along the cavity length and then integrate them into an effective wavelength for the whole cavity.

Which I followed and used the guide wavelengths, generated from the Df equation, at 1,000 equally spaced diameter points between and including the end plates.
In other words, I understand that you have developed a spreadsheet model predicting the correct geometry for resonance to occur at, but that you cannot predict whether the resonance occurs in a Transverse Magnetic mode, or a Transverse Electric mode, and that you cannot predict what is the resonance mode shape variation (m. n, p numberrs) in the longitudinal, transverse and azimuthal directions of the truncated cone.

Therefore we cannot check whether your ("Roger's bread crumb" ?) predicted resonance is correct.   :(
Please doc this is just a simple question, but why would absolute dimensional numbers be so important as long as you're in the ballpark for resonance? I think every device I've seen has taken into account that either you mechanically tune the EM cavity or shift the insertion frequency to optimize the thrust.
I've even thought of using a airtight Conductive Elastic Fabric on the endplate and varying the air pressure the chamber to optimize the reflected EM wave.
Title: Re: EM Drive Developments - related to space flight applications - Thread 2
Post by: aero on 05/21/2015 04:48 pm
Quote
I've even thought of using a airtight Conductive Elastic Fabric on the endplate and varying the air pressure the chamber to optimize the reflected EM wave.

Now that's an interesting idea. Do you have data showing a variation in magnetic permeability of air at different pressures?
Title: Re: EM Drive Developments - related to space flight applications - Thread 2
Post by: Rodal on 05/21/2015 04:50 pm
...
Please doc this is just a simple question, but why would absolute dimensional numbers be so important as long as you're in the ballpark for resonance? I think every device I've seen has taken into account that either you mechanically tune the EM cavity or shift the insertion frequency to optimize the thrust.
I've even thought of using a airtight Conductive Elastic Fabric on the endplate and varying the air pressure the chamber to optimize the reflected EM wave.
Since whether the EM Drive "thrust" is an experimental artifact or whether it is something that can be used for space propulsion (and if so, what is its theoretical justification) is still unsettled, most issues, like whether it is good enough to be in the ballpark for resonance are very much a subject of debate.

If one assumes (as Shawyer appears to do) that the higher the Q the better, then it appears that one does want to be at the resonant peak, since as you know the higher the Q the narrower the bandwidth for such resonant peak. 

On the other hand, we have:

1) the beautiful theory being developed by Todd (WarpTech) that one may not want maximum Q
2) the experimental evidence from Prof. Yang achieving the highest thrust measured at Q's (when transformed such that they are calculated the same way as in the West) lower than Shawyer reported Q's
3) the experimental evidence from NASA Eagleworks showing that highest thrust is not one-to-one monotonically associated with highest Q
4) although Shawyer has been working on a superconducting EM Drive with a huge Q, no thrust values from such a device have been reported by Shawyer (who is in the process of being protected by his patent application for a superconducting EM Drive)
5) the superconducting Cannae drive achieved only milliNewtons and the test has been criticized earlier in the thread as being anomalous,  Cannae appeared to have shifted his attention to non-superconducting drives after that experiment (?)

Now, it is my understanding that TheTraveller is going through the trouble to calculate resonance following Shawyer's approach, based on achieving highest Q, which implies having to be at the resonant peak.

We also know from NASA Eagleworks that thrust is very much associated with particular mode shapes.  Notsosureofit's prediction formula is also dependent on mode shapes.

Thus, it appears that to understand the experimental reports one needs to assess what are the mode shapes involved.

Since I am skeptical of the derivation of TheTraveller's Shawye's resonance calculation (I am from Missouri: show me  :) ) the only way I have to verify it is by comparing the mode shapes to the exact solution (which would also be useful in its own right, as per NASA and Notsosureofit).   Since many mode shapes are bunched together at close frequencies, one cannot determine whether a natural frequency prediction is correct unless one assesses the mode shape prediction.
Title: Re: EM Drive Developments - related to space flight applications - Thread 2
Post by: aero on 05/21/2015 04:56 pm
Following Roger Shawyers kindly laid bread crumb trail, my EM Drive spreadsheet now can calc the effective internal guide wavelength and external Rf wavelength that will give resonance from end plate to end plate.

As an example for the Flight Thruster big and small end diameters as below, the required end plate to end plate spacing to achieve resonance with an external Rf of 3.85GHz is as below.

Alteration of either the big, small end or Rf frequency will now automatically generate a new Df and from that the end plate spacing needed to achieve resonance with the external Rf.

big diameter      m   0.2440000
small diameter   m   0.1450000
cavity length      m   0.1603484
rf frequency       Hz   3,850,000,000
Calculated Df      Df   0.49094
slant angle        Deg   28.8

1,000 point numerically integrated guide wavelength of the above example is: 0.0801741816

I will publish the spreadsheet but would 1st like to run / verify it against other frustum dimensions and what the calculated / measured resultant resonance was and in what mode.

With this spreadsheet if we know either end diameter, Rf frequency and Df, the other diameter and spacing can now be determined as all 3 dimensions and external Rf wavelength (4 variables) affect each other.

NEXT STEPS:

1) Determine the best way to inject the coax Rf into the Flight Thruster? Loop or Stub?

2) Determine the best location to inject the Rf into the Flight Thruster?

3) Determine the best way to impedance match the Flight Thruster to the impedance of the Rf generator so as to get optimal VSWR and energy delivery to inside the Flight Thruster?

Very interesting. Now I would like to see a comparison of cavity dimensions vs. drive frequency with the dimensions expressed in units of drive frequency wavelength. That is because it is very easy to model a the cavity using wavelength as the unit of length and it would be so nice to be able to conveniently change the drive frequency if I knew how the cavity dimensions changed with drive frequency wavelength.

Title: Re: EM Drive Developments - related to space flight applications - Thread 2
Post by: aero on 05/21/2015 05:00 pm
Greetings,

I am currently playing catchup (page 151), so my apologies if this has already been talked about. My question is how is the acceleration witness from the emdrive different from the Pioneer anomaly? Full disclosure time; I am not a scientist, but I am a fan of physics.  Thank you for the great conversations on this forum.
http://en.m.wikipedia.org/wiki/Pioneer_anomaly

Welcome - This thread is growing so fast that we're all playing catchup. It's almost a full time job just to keep up with the thread.
Title: Re: EM Drive Developments - related to space flight applications - Thread 2
Post by: TheTraveller on 05/21/2015 05:01 pm
As a sanity check, what is the mode shape you predict for the above "resonance" calculation ?

Do you predict resonance at a Transverse Magnetic or Transverse Electric mode ?
what are the predicted mode shape quantum number m,n,p values ?

TMmnp  ?
TEmnp   ?

Thanks

Roger's bread crumb:

Quote
You need to develop a numerical model that calculates the guide wavelength, for the chosen mode, at discrete small increments along the cavity length and then integrate them into an effective wavelength for the whole cavity.

Which I followed and used the guide wavelengths, generated from the Df equation, at 1,000 equally spaced diameter points between and including the end plates.
In other words, I understand that you have developed a spreadsheet model predicting the correct geometry for resonance to occur at, but that you cannot predict whether the resonance occurs in a Transverse Magnetic mode, or a Transverse Electric mode, and that you cannot predict what is the resonance mode shape variation (m. n, p numberrs) in the longitudinal, transverse and azimuthal directions of the truncated cone.

Therefore we cannot check whether your ("Roger's bread crumb" ?) predicted resonance is correct.

It is now SPR calcs length resonance and end plate separation.

The guide wavelengths used are based on the Df equation, which I assume uses the mode Shawyer does his designs with. Would seem the Mode he used and developed his DF around is another unanswered question.

That being said, it implies the resonance calc is as valid as his Df calc.

To be clear Shawyer doesn't give the mode his equations are based on. Must be a bit of IP he doesn't wish to share. Which I respect. Never the less we don't need to know the mode he uses, just the guide wavelength that is produced by his Df equation in that mode. From those numbers, the physical end plate separation can be determined, operating in what ever mode Shawyer developed his Df equation from.

While I agree it would be nice to know the mode his calcs use, it is not required to determine the physical dimensions, which as a replicator, is all I really need.

As your math skills are superior to mine, maybe you can reverse engineer the operational mode from the Df equation and generated values, while I get on with the physical replication.
Title: Re: EM Drive Developments - related to space flight applications - Thread 2
Post by: deltaMass on 05/21/2015 05:06 pm
@TheTraveller: Excuse my obtuseness, but don't you have the design freedom to arrange for Df to be as close to unity as you like? If there's one thing that Shawyer's work teaches, it's that maximising Df maximises thrust - therefore this seems to be worthwhile.

So what constraints forbid you designing for near-unity Df?
Title: Re: EM Drive Developments - related to space flight applications - Thread 2
Post by: rfmwguy on 05/21/2015 05:23 pm
For experimenters who might be considering a horn shaped frustum (easier to build for proof of concept), SETI has some interesting low-cost construction pics...note the RF feed pic from bottom on right:

http://www.setileague.org/photos/wghorn.htm
Title: Re: EM Drive Developments - related to space flight applications - Thread 2
Post by: SeeShells on 05/21/2015 05:25 pm
...
Please doc this is just a simple question, but why would absolute dimensional numbers be so important as long as you're in the ballpark for resonance? I think every device I've seen has taken into account that either you mechanically tune the EM cavity or shift the insertion frequency to optimize the thrust.
I've even thought of using a airtight Conductive Elastic Fabric on the endplate and varying the air pressure the chamber to optimize the reflected EM wave.


Since I am skeptical of the derivation of TheTraveller's Shawye's resonance calculation (I am from Missouri: show me  :) ) the only way I have to verify it is by comparing the mode shapes to the exact solution (which would also be useful in its own right, as per NASA and Notsosureofit).   Since many mode shapes are bunched together at close frequencies, one cannot determine whether a natural frequency prediction is correct unless one assesses the mode shape prediction.
I know I've been reviewing the mode work done by by Frank Davies of NASA and relating it to thrust and I'm observing some interesting correlations between the two. And it's not so much the Q.  :-X
Data, more data! 
For instance did you sweep the frequency from ~900mhz to ~3Ghz or just dial in a close TM frequency and then fine tune? If you did sweep do you have any data you could share?
Title: Re: EM Drive Developments - related to space flight applications - Thread 2
Post by: TheTraveller on 05/21/2015 05:29 pm
@TheTraveller: Excuse my obtuseness, but don't you have the design freedom to arrange for Df to be as close to unity as you like? If there's one thing that Shawyer's work teaches, it's that maximising Df maximises thrust - therefore this seems to be worthwhile.

So what constraints forbid you designing for near-unity Df?

Dr Rodal did this excellent bit of work, which shows near unity Dfs are possible.

Might help to explain why the Flight Thruster was designed to run at 3.85GHz. To my knowledge Shawyer has never quoted the Df of the Flight Thruster.
Title: Re: EM Drive Developments - related to space flight applications - Thread 2
Post by: Rodal on 05/21/2015 05:41 pm
...
For instance did you sweep the frequency from ~900mhz to ~3Ghz or just dial in a close TM frequency and then fine tune? If you did sweep do you have any data you could share?
Both Frank Davis at NASA (using Finite Element Analysis) and I (using my exact solution) have only performed an eigenvalue analysis problem, from where we obtain the natural frequencies and mode shapes.

The reported results of Frank Davis for the frequencies and mode shapes are within ~1% of my exact solution.  I think that Frank Davis is an excellent Finite Element analyst, as he got the finite element mesh just right.
The only think I had some questions with is that NASA decided to interpret the mode shapes of the truncated cone using the same numbering system as a perfect cylinder, and this only works for mode shapes that are analogous.  Earlier in the thread I have discussions with Paul March where I pointed out mode shapes in the truncated cone that are not analogous to a cylinder's mode shape.  This usually happens when a mode shape is close to being cut-off and there is a lot of attenuation.  Curiously, this is the condition that Shawyer is shooting for, hence I am skeptical of what TheTraveller is calculating based on simple wavelengths.

I have not performed (and Paul March has not reported such data either) a spectrum response analysis yet.

In a real cavity experiment it is very unlikely that one is going to excite a single mode shape.
Actually, as Notsosureofit has repeatedly pointed out, one may be better off having two equidistant close frequencies locked up at the half power points if one is pursuing the highest Q (which again it is not known whether that's what one really wants).

To answer these questions (and more) one has to perform a spectrum analysis (NASA can do that with COMSOL, I would have to use ANSYS or ADINA or write my own program with Mathematica).

Title: Re: EM Drive Developments - related to space flight applications - Thread 2
Post by: deltaMass on 05/21/2015 05:42 pm
Quote from: TheTraveller
Dr Rodal did this excellent bit of work, which shows near unity Dfs are possible.
I know that - and also I did the same derivations myself (upon which you commented). But that doesn't directly answer my question. You published some specs that showed you are designing to a small Df. Why?
Title: Re: EM Drive Developments - related to space flight applications - Thread 2
Post by: saucyjack on 05/21/2015 05:51 pm
We haven't heard TheTraveller's opinion.

Should we change the Experimental Spreadsheet assuming that Shawyer made a typo and the DesignFactor for the Demonstrator was 0.484 instead of 0.844 ?

Well, I was guessing he'd say yes, so I changed it already on http://emdrive.echothis.com/Experimental_Results.  Will of course switch it back if consensus says otherwise.
Title: Re: EM Drive Developments - related to space flight applications - Thread 2
Post by: Rodal on 05/21/2015 06:11 pm
...Reading it I had a question (more than one but..). And this is for everyone, why did Eagle Works observe no thrust in a EM device with no HDPE insert? Makes me wonder what effect achiral materials like this that can induce chirality would have with relativity and electromagnetic fields in your equations?

Thanks nice work!

Thanks a lot. The reason why a material can change the behaviour is magnetic permeability. This can enhance the effect by several magnitude orders. They use low input power, if I am right, and so this could be a good way around to such a limitation.

High Density PolyEthylene (HDPE) is a nonconductive polymer with no special magnetic properties, its relative magnetic permeability is very nearly unity, as is generally true of polymers. 

Hence we cannot explain the report that NASA observed thrust with a HDPE insert and observed no thrust in a EM device without the HDPE insert based on the relative magnetic permeability of HDPE (which is practically 1).

I think to explain this fact, and still be compatible with Marco's interesting paper, we must use the expression from Maxwell's theory for the speed of light:

1 / c2  =  μo εo = (magnetic permeabiltity) * (electric permittivity)

We can use this expression to convert magnetic permeability as follows:

o)2 = 1 / ( c4  (εo )2)

if we substitute this expression in the expression for the energy density parameter we obtain:

(Uo)4 / (μo)2  = (Uo)4  c4  (εo) 2

and therefore we can eliminate the factor of  c4 in Eq. 60 and get a more beautiful equation (with one less parameter) that depends explicitly on the electric permittivity as the following quantity gets transformed:

(Pi2 G / c4 ) ( (Uo)4  /(μo)2)  = Pi2 G  (Uo)4  (εo)2

In relativity, magnetic and electric fields are essentially different aspects of a unique electromagnetic field.  In the resonant cavity, when the magnetic field is maximum, the electric field is zero and vice-versa. The energy from the magnetic field goes into the electric field, back and forth.
Title: Re: EM Drive Developments - related to space flight applications - Thread 2
Post by: StrongGR on 05/21/2015 06:23 pm
...Reading it I had a question (more than one but..). And this is for everyone, why did Eagle Works observe no thrust in a EM device with no HDPE insert? Makes me wonder what effect achiral materials like this that can induce chirality would have with relativity and electromagnetic fields in your equations?

Thanks nice work!

Thanks a lot. The reason why a material can change the behaviour is magnetic permeability. This can enhance the effect by several magnitude orders. They use low input power, if I am right, and so this could be a good way around to such a limitation.

High Density PolyEthylene (HDPE) is a nonconductive polymer with no special magnetic properties, its relative magnetic permeability is very nearly unity, as is generally true of polymers. 

Hence we cannot explain the report that NASA observed thrust with a HDPE insert and observed no thrust in a EM device without the HDPE insert based on the relative magnetic permeability of HDPE (which is practically 1).

I think to explain this fact, and still be compatible with Marco's interesting paper, we must use the expression from Maxwell's theory for the speed of light:

1 / c2  =  μo εo = (magnetic permeabiltity) * (electric permittivity)

We can use this expression to convert magnetic permeability as follows:

o)2 = 1 / ( c4  (εo )2)

if we substitute this expression in the expression for the energy density parameter we obtain:

(Uo)4 / (μo)2  = (Uo)4  c4  εo 2

and therefore we can eliminate the factor of  c4 in Eq. 60 and get a more beautiful equation (with one less parameter) that depends explicitly on the electric permittivity as the following quantity gets transformed:

(Pi2 G / (  c4  ) )  Uo)4  / c4  =
(Pi2 G  Uo)4  (εo)2

That's fine and depends on the kind of material used as a dielectric. I was considering teflon with mu about 10^-6 but you were using HDPE and things can be quite different.

I am evaluating the thrust equation from general relativity and I can see that Q of cavity, the power P and choice of the mode, k, can alleviate the smallness of the factor G/c^4 as also the choice of a good dielectric. These enter with square. On the other side, this formula applies well for r2>>r1 and r1 even more smaller. Also h, the height of the frustum, appears to be relevant in this evaluation.
Title: Re: EM Drive Developments - related to space flight applications - Thread 2
Post by: TheTraveller on 05/21/2015 06:35 pm
Quote from: TheTraveller
Dr Rodal did this excellent bit of work, which shows near unity Dfs are possible.
I know that - and also I did the same derivations myself (upon which you commented). But that doesn't directly answer my question. You published some specs that showed you are designing to a small Df. Why?

My goal is to NOT reinvent the wheel 1st go and to build a Flight Thruster as close to the one Shawyer built. While Df is important, more so is Q, operating at resonance and impedance matching.
Title: Re: EM Drive Developments - related to space flight applications - Thread 2
Post by: TheTraveller on 05/21/2015 06:37 pm
We haven't heard TheTraveller's opinion.

Should we change the Experimental Spreadsheet assuming that Shawyer made a typo and the DesignFactor for the Demonstrator was 0.484 instead of 0.844 ?

Well, I was guessing he'd say yes, so I changed it already on http://emdrive.echothis.com/Experimental_Results.  Will of course switch it back if consensus says otherwise.

Could you also put also a (1) next to 0.484 and indicate in the note that Shawyer's reference has 0.844 but that this number gives a small diameter in conflict with the picture of the Demonstrator, therefore it is assumed there was typo transposition of the numbers ?

Would agree 0.844 seems a bit high for the Demonstrator. Will check it out.
Title: Re: EM Drive Developments - related to space flight applications - Thread 2
Post by: Reactionless on 05/21/2015 06:46 pm
Iulian is indeed alive and still (one assumes) possesses his eyeballs!

http://www.masinaelectrica.com/emdrive-independent-test/
Title: Re: EM Drive Developments - related to space flight applications - Thread 2
Post by: Rodal on 05/21/2015 06:48 pm
...
That's fine and depends on the kind of material used as a dielectric. I was considering teflon with mu about 10^-6 but you were using HDPE and things can be quite different...

Some information from NASA on their experiments with the truncated cone with the HDPE insert:

NASA Eagleworks has used a High Density Polyethylene dielectric insert for most of their experiments.  It was obtained from McMaster Carr.  It came from a supplier of extruded HDPE rods.

http://www.mcmaster.com/#standard-plastic-rods/=w0bzy0

under Rigid HDPE Polyethylene, apparently they no longer supply the 6.25" dimension.  Biggest diameter they list is 6.00 inches:  Rigid HDPE Polyethylene Rod, 6" Diameter

This site http://www.rfcafe.com/references/electrical/dielectric-constants-strengths.htm reports

HDPE to have a relative electric permittivity εr = 2.26 @ 3 GHz

Notice from this picture that the HDPE dielectric insert is located towards the apex of the cone, which is the location (r~r1) where the effect, discussed in Marco' paper, is strongest:

(http://forum.nasaspaceflight.com/index.php?action=dlattach;topic=36313.0;attach=636341;image)

Bigger image (click here to download):
http://forum.nasaspaceflight.com/index.php?action=dlattach;topic=36313.0;attach=634621
............................................

Title: Re: EM Drive Developments - related to space flight applications - Thread 2
Post by: Rodal on 05/21/2015 06:59 pm
Iulian is indeed alive and still (one assumes) possesses his eyeballs!

http://www.masinaelectrica.com/emdrive-independent-test/

Iulian reports:

"I flipped the cone in the original setup and i have the thrust downwards (scale goes positive). Unfortunately the thrust downwards is around 7 times smaller. difference on the scale is only 0.20 grams and is consistent with the power on and off . I will post the video on YouTube."

So, it appears that the true thrust should be

(Iulian convention:  positive reading going DOWN and a negative reading going UP)

TRUE EM DRIVE THRUST = (MeasurementDOWN - MeasurementUP) /2

which should subtract the lifting force due to the heated air (assuming that the lifting force is the same when pointed small diameter facing down and when the small diameter faces up)

For MeasurementDOWN = - MeasurementUP / 7; we get

TRUE EM DRIVE THRUST = MeasurementUP ( (1/7) + 1) /2 = 0.57 * MeasurementUP

It is interesting that the thrust of the EM Drive downward appears to be (a little)  greater than the lifting force of the heated air

Title: Re: EM Drive Developments - related to space flight applications - Thread 2
Post by: deltaMass on 05/21/2015 07:00 pm
We can figure out the true thrust now (T=thrust, a=air effect, in gm-wt)
T + a = -0.54
-T + a = +0.54/7
solving by eliminating ‘a’ gives
T = -0.31 gm-wt

so thrusting forward at the small end

a = -0.23 (upwards)
Title: Re: EM Drive Developments - related to space flight applications - Thread 2
Post by: SeeShells on 05/21/2015 07:04 pm
...Reading it I had a question (more than one but..). And this is for everyone, why did Eagle Works observe no thrust in a EM device with no HDPE insert? Makes me wonder what effect achiral materials like this that can induce chirality would have with relativity and electromagnetic fields in your equations?

Thanks nice work!

Thanks a lot. The reason why a material can change the behaviour is magnetic permeability. This can enhance the effect by several magnitude orders. They use low input power, if I am right, and so this could be a good way around to such a limitation.

High Density PolyEthylene (HDPE) is a nonconductive polymer with no special magnetic properties, its relative magnetic permeability is very nearly unity, as is generally true of polymers. 

Hence we cannot explain the report that NASA observed thrust with a HDPE insert and observed no thrust in a EM device without the HDPE insert based on the relative magnetic permeability of HDPE (which is practically 1).

I think to explain this fact, and still be compatible with Marco's interesting paper, we must use the expression from Maxwell's theory for the speed of light:

1 / c2  =  μo εo = (magnetic permeabiltity) * (electric permittivity)

We can use this expression to convert magnetic permeability as follows:

o)2 = 1 / ( c4  (εo )2)

if we substitute this expression in the expression for the energy density parameter we obtain:

(Uo)4 / (μo)2  = (Uo)4  c4  εo 2

and therefore we can eliminate the factor of  c4 in Eq. 60 and get a more beautiful equation (with one less parameter) that depends explicitly on the electric permittivity as the following quantity gets transformed:

(Pi2 G / (  c4  ) )  Uo)4  / c4  =
(Pi2 G  Uo)4  (εo)2

That's fine and depends on the kind of material used as a dielectric. I was considering teflon with mu about 10^-6 but you were using HDPE and things can be quite different.

(1) Magnetic susceptibilities of paramagnetic and diamagnetic materials... see anything interesting?
http://www.kayelaby.npl.co.uk/general_physics/2_6/2_6_6.html
Title: Re: EM Drive Developments - related to space flight applications - Thread 2
Post by: Lobo on 05/21/2015 07:06 pm
So I'm a newbie to this thread, and don't really understand the concept very well.  As I'll never have time to read through the previous 200+ pages (I've read through some of the more recent ones) I just had a couple of simple questions that could maybe catch me up a bit.  If Someone doesn't mind answering them although I'm sure they've been addressed at various points in this thread in the last 200 pages.

1)  I've read some articles online on this, including the recent article here.  I think I understand the general idea behind it.  There's a lot of skeptism about it about how it violates the laws of physics.  On the other side apparently the radiation is producing the force, so it's actually not a reactionless drive?
So are we at a point where the indications are a majority that it can work?...and be practically flown in the not too distant future?  After the recent Eagleworks testing?

2)  What would be the impulse of the thrust of such a drive?  I think Ion thrusters can get up to 5000s?  How would this compare?  Chris's article mentioned some very fast transit times, but no mention of what the actual impulse number is.

3)  Is it possible to generate enough power on a realistic spacecraft to power it so that it could actually take crews or robots to other planets in the solar system?  One criticism is that even if this worked, it would take so much power that it wouldn't be feasible.  But Chris's article didn't seem to indicated that the power requirements were unfeasible.   And that the thrust to power input ratio went down as power went up?  What's the view on that on the thread?

Anyway, pardon my simplistic questions.  Just kinda trying to get a summary of where this thread has covered in the past 200 pages, and if this is actually something we might see fly sometime?  Or is it highly theoretical like a "warp" drive? (which the Eagleworks had been working on)
Title: Re: EM Drive Developments - related to space flight applications - Thread 2
Post by: Reactionless on 05/21/2015 07:07 pm
We can figure out the true thrust now (T=thrust, a=air effect, in gm-wt)
T + a = -0.54
-T + a = +0.54/7
solving by eliminating ‘a’ gives
T = -0.31 gm-wt

so thrusting forward at the small end

Quote
the thrust downwards is around 7 times smaller. difference on the scale is only 0.20 grams

I'm confused.

From Iulian's quote, wouldn't Test 1 = 0.2*7=1.4g and Test 2 = 0.2 . Then the equation would be set up like this?

T=thrust
A=air

T+A=1.4g
T-A=0.2g
T=0.8g
A=0.6g
Title: Re: EM Drive Developments - related to space flight applications - Thread 2
Post by: Iulian Berca on 05/21/2015 07:07 pm
Hi,

When i saw the people start thinking i died i was concerned. I wanted to post when i have some results but now i`m forced me to post some unfinished work :)

I just closed the cone with new setup in place and in the next days i will be busy adjusting the distance and see if i have any improvement. The work will be slow because i need to make a lot of tests with different distance for each test i need to let the magnetron to cool down for at least 5 minutes.

Iulian
Title: Re: EM Drive Developments - related to space flight applications - Thread 2
Post by: tchernik on 05/21/2015 07:11 pm
Hi,

When i saw the people start thinking i died i was concerned. I wanted to post when i have some results but now i`m forced me to post some unfinished work :)

I just closed the cone with new setup in place and in the next days i will be busy adjusting the distance and see if i have any improvement. The work will be slow because i need to make a lot of tests with different distance for each test i need to let the magnetron to cool down for at least 5 minutes.

Iulian

Glad to hear you are OK.

Yes, you seem to be growing kind of a fan club on the Internet, with even rumors being spread on Twitter about your wellness. Happily, all false.

Title: Re: EM Drive Developments - related to space flight applications - Thread 2
Post by: Rodal on 05/21/2015 07:23 pm
...
(1) Magnetic susceptibilities of paramagnetic and diamagnetic materials... see anything interesting?
http://www.kayelaby.npl.co.uk/general_physics/2_6/2_6_6.html

mass susceptibility per kilogram, χ, at 20°C

Vacuum    .  .  .  .  0
   
Polyethylene   .  .  .+0.2 *10^(-8)  (paramagnetic)
   
Aluminium  .  .  .  . +0.82*10^(-8)  (paramagnetic)

Copper      .  .  .  .  −0.107*10^(-8) (diamagnetic)

All very small values compared to iron, cast iron and other magnetic materials

The value for Polyethylene confirms that the magnetic permeability of HDPE shoud be close to 1.

However, this is for the real part of the susceptibility.  I recall that Paul March said that what was most interesting were the imaginary components of the permittivity and the permeability.

Can anyone find the imaginary permittivity and imaginary permeability for HDPE ?

The imaginary parts act as absorption coefficients (more in the direction of attenuation pointed out by Todd)

Title: Re: EM Drive Developments - related to space flight applications - Thread 2
Post by: deltaMass on 05/21/2015 07:45 pm
We can figure out the true thrust now (T=thrust, a=air effect, in gm-wt)
T + a = -0.54
-T + a = +0.54/7
solving by eliminating ‘a’ gives
T = -0.31 gm-wt

so thrusting forward at the small end

Quote
the thrust downwards is around 7 times smaller. difference on the scale is only 0.20 grams

I'm confused.
From Iulian's quote, wouldn't Test 1 = 0.2*7=1.4g and Test 2 = 0.2 . Then the equation would be set up like this?
T=thrust
A=air

T+A=1.4g
T-A=0.2g
T=0.8g
A=0.6g
We are both wrong. He said two contradictory things
"I got 1/7th of the previous thrust, in the opposite direction" and
"I got 2 gm-wt thrust downwards"


Originally he got 0.54 gm-wt upwards, so 1/7th would be 0.08 - yet he says 0.2
I'll go with 0.2. T assumed towards small end.

T + A = 0.54 (up)
-T + A = -0.2 (down)

Solving:
T = 0.37 (thrusts from small end)
A = 0.17 (~50% of the thrust)
Title: Re: EM Drive Developments - related to space flight applications - Thread 2
Post by: TheTraveller on 05/21/2015 07:48 pm
The cutoff wavelength equation currently used for the Df equation is based on the dominant circular waveguide TE11 mode.

Shawyers frustum patent mentions the mode used is TM01 as per the attached. Means a different cutoff equation is required, which will alter the cutoff, guide and resonance wavelengths.

Will mod my spreadsheet and see how it changes things.

My bed time, so report back in 8 hours or so.
Title: Re: EM Drive Developments - related to space flight applications - Thread 2
Post by: Rodal on 05/21/2015 07:49 pm
We haven't heard TheTraveller's opinion.

Should we change the Experimental Spreadsheet assuming that Shawyer made a typo and the DesignFactor for the Demonstrator was 0.484 instead of 0.844 ?

Well, I was guessing he'd say yes, so I changed it already on http://emdrive.echothis.com/Experimental_Results.  Will of course switch it back if consensus says otherwise.

Could you also please change the length to  0.187m  ?.:

I also think the cavity length needs to be adjusted on the Demonstrator Thruster to .187m.

My reasoning and measurements:

I believe Shawyer angled the cone to his expectation of where the small diameter plate will be.  When the device is turned on the plate is slightly back inside the cylinder and gradually moved forward until there is phase lock. The length of the cylinder, in part, is to accommodate the movement mechanism and the volume of the small plate.

In my recent model the cone length is exactly .187m (from the face of the small plate to the face of the large plate). Slightly longer than the .183m calculated from 3 * .123m / 2 - based on .2450Ghz. I think 4mm is ample distance to move the small plate to achieve phase lock as I described above.

Did I do the math right?
Title: Re: EM Drive Developments - related to space flight applications - Thread 2
Post by: rfmwguy on 05/21/2015 07:58 pm
...
(1) Magnetic susceptibilities of paramagnetic and diamagnetic materials... see anything interesting?
http://www.kayelaby.npl.co.uk/general_physics/2_6/2_6_6.html

mass susceptibility per kilogram, χ, at 20°C

Vacuum    .  .  .  .  0
   
Polyethylene   .  .  .+0.2 *10^(-8)  (paramagnetic)
   
Aluminium  .  .  .  . +0.82*10^(-8)  (paramagnetic)

Copper      .  .  .  .  −0.107*10^(-8) (diamagnetic)

All very small values compared to iron, cast iron and other magnetic materials

The value for Polyethylene confirms that the magnetic permeability of HDPE shoud be close to 1.

However, this is for the real part of the susceptibility.  I recall that Paul March said that what was most interesting were the imaginary components of the permittivity and the permeability.

Can anyone find the imaginary permittivity and imaginary permeability for HDPE ?

The imaginary parts act as absorption coefficients (more in the direction of attenuation pointed out by Todd)

The use of HDPE rather than PTFE at MW freqs has been bothering me. Nowhere over 1 GHz have I heard of HDPE, which could indicate high moisture absorption or reflectability. Also, the melting point of HDPE is rather low compared to the 250 deg C of PTFE.

The comparison tables are here: http://www.vanderveerplastics.com/compare-materials.html?sel1=hdpe&sel2=teflon-ptfe-fep

If someone knows why HDPE (typically plastic milk jugs( were initially used, it would be interesting.

"High-density polyethylene (HDPE) or polyethylene high-density (PEHD) is a polyethylene thermoplastic made from petroleum. It is sometimes called "alkathene" or "polythene" when used for pipes.[1] With a high strength-to-density ratio, HDPE is used in the production of plastic bottles, corrosion-resistant piping, geomembranes, and plastic lumber. HDPE is commonly recycled, and has the number "2" as its resin identification code (formerly known as recycling symbol)." per http://en.wikipedia.org/wiki/High-density_polyethylene
Title: Re: EM Drive Developments - related to space flight applications - Thread 2
Post by: Rodal on 05/21/2015 08:00 pm
...Chris's article mentioned some very fast transit times, but no mention of what the actual impulse number is.

...Chris's article didn't seem to indicated that the power requirements were unfeasible.   And that the thrust to power input ratio went down as power went up? ...
Welcome to the thread.   Just a short answer and comment regarding the above points.  It was not Chris's article.  I was one of three authors.  The sections you describe paraphrased exactly what Dr. White had written in AIAA papers and what Paul March had written in his papers or at the forum.  There was not enough space to provide a thorough review of the theories in a form that could be understood to a general audience.  I did add to the article a number of objective counterarguments where appropriate as the fact that the EM Drive appears to violate conservation of momentum.  In General Relativity momentum and energy are tied together in the stress-energy tensor, so that if something violates conservation of momentum it is not surprise that it will violate conservation of energy.
I also added that Dr. White's theory assumes a mutable, degradable Quantum Vacuum (in contrast with the prevailing understanding that it is immutable and non-degradable).  I also made a small argument based on the immutability of elementary particles no matter where and when created in the Universe, and also included the objection due to frame-independence.  I was also responsible for the fact that the article never mentions the words "warp-drive", or mentions the word "Alcubierre drive", that nevertheless appeared in other publications.

I hope that others will answer your remaining questions  :)

Take a gander at the section on energy conservation here:

http://emdrive.echothis.com/Generic_EM_Drive_Information
Title: Re: EM Drive Developments - related to space flight applications - Thread 2
Post by: deltaMass on 05/21/2015 08:02 pm
The bottom line is that nobody understands how this thing works.
Title: Re: EM Drive Developments - related to space flight applications - Thread 2
Post by: Reactionless on 05/21/2015 08:04 pm
We can figure out the true thrust now (T=thrust, a=air effect, in gm-wt)
T + a = -0.54
-T + a = +0.54/7
solving by eliminating ‘a’ gives
T = -0.31 gm-wt

so thrusting forward at the small end

Quote
the thrust downwards is around 7 times smaller. difference on the scale is only 0.20 grams

I'm confused.
From Iulian's quote, wouldn't Test 1 = 0.2*7=1.4g and Test 2 = 0.2 . Then the equation would be set up like this?
T=thrust
A=air

T+A=1.4g
T-A=0.2g
T=0.8g
A=0.6g
We are both wrong. He said two contradictory things
"I got 1/7th of the previous thrust, in the opposite direction" and
"I got 2 gm-wt thrust downwards"


Originally he got 0.54 gm-wt upwards, so 1/7th would be 0.08 - yet he says 0.2
I'll go with 0.2. T assumed towards small end.

T + A = 0.54 (up)
-T + A = -0.2 (down)

Solving:
T = 0.37 (thrusts from small end)
A = 0.17 (~50% of the thrust)


Of course, Iulian is welcome to come and correct both of us, but from rewatching his video of test 3, I see that his scale registers around 1.4 when he turns the magnetron on, which is about 7x this new amount. Perhaps he is talking about 0.2 gm registering on the scale itself, not the multiplied amount taking into account the lever multiplier.
Title: Re: EM Drive Developments - related to space flight applications - Thread 2
Post by: Iulian Berca on 05/21/2015 08:05 pm
I just uploaded the video with the upside down test.  in the next days i will start testing the new setup with the adjustable length.

https://www.youtube.com/watch?v=KAMttfMC8PI
Title: Re: EM Drive Developments - related to space flight applications - Thread 2
Post by: TheTraveller on 05/21/2015 08:06 pm
This is also interesting for TM010 mode. Note where the H (magnetic) field is located. At the big end, right where Shawyer feeds in the Rf in the Demonstrator & Flight Thruster EM Drives.

Backs up the Patent mention of TM01 mode.

Fairly clear to me, TM010 is probably Shawyers EM Drive mode.
Title: Re: EM Drive Developments - related to space flight applications - Thread 2
Post by: Rodal on 05/21/2015 08:08 pm
...

The use of HDPE rather than PTFE at MW freqs has been bothering me. Nowhere over 1 GHz have I heard of HDPE, which could indicate high moisture absorption or reflectability. Also, the melting point of HDPE is rather low compared to the 250 deg C of PTFE.

The comparison tables are here: http://www.vanderveerplastics.com/compare-materials.html?sel1=hdpe&sel2=teflon-ptfe-fep

If someone knows why HDPE (typically plastic milk jugs( were initially used, it would be interesting.

"High-density polyethylene (HDPE) or polyethylene high-density (PEHD) is a polyethylene thermoplastic made from petroleum. It is sometimes called "alkathene" or "polythene" when used for pipes.[1] With a high strength-to-density ratio, HDPE is used in the production of plastic bottles, corrosion-resistant piping, geomembranes, and plastic lumber. HDPE is commonly recycled, and has the number "2" as its resin identification code (formerly known as recycling symbol)." per http://en.wikipedia.org/wiki/High-density_polyethylene

HDPE has very interesting properties, it has been used for biomedical applications, for garbage bags and countless commercial applications. It is very tough, it has excellent sliding abrasion resistance and is self lubricating.

As to why it was used as a dielectric for NASA's EM Drive, my recollection is that it was based on prior experience by Paul March, originally working with Prof. Woodward on his Mach Effect theory and experiments, and that it was based, as I noted above, on the imaginary properties acting as absorption coefficients in electromagnetic waves.

By the way, NASA also used PTFE and Nitrile Rubber as dielectrics.  They obtained the highest thrust force with HDPE
Title: Re: EM Drive Developments - related to space flight applications - Thread 2
Post by: TheTraveller on 05/21/2015 08:11 pm
The bottom line is that nobody understands how this thing works.

Once we have solid build dimensions & BOM, working out if it works as per Shawyer & Chinese or not theories should not take long.
Title: Re: EM Drive Developments - related to space flight applications - Thread 2
Post by: deltaMass on 05/21/2015 08:15 pm
I just uploaded the video with the upside down test.  in the next days i will start testing the new setup with the adjustable length.

https://www.youtube.com/watch?v=KAMttfMC8PI
Great! You got a thrust reversal.
T + A = 0.508 gm-wt
-T + A = -0.072 gm-wt
so
T = 0.290 gm-wt (thrust forward from small end)
A = 0.218 gm-wt (air deltaMass comparable in magnitude to thrust, unfortunately)


Interesting that the thrust seems to be "small end forward". Others (Shawyer, NASA, Juan) see opposite thrust, from the large end.
Title: Re: EM Drive Developments - related to space flight applications - Thread 2
Post by: Notsosureofit on 05/21/2015 08:16 pm

Take a gander at the section on energy conservation here:

http://emdrive.echothis.com/Generic_EM_Drive_Information

Just thinking that maybe there should be a third option:  Since a constant acceleration transforms as a 4-space rotational velocity (?), there may be an invariant for the tensor which allows Energy to Momentum conversion. ?  This would be what, a "false force" driven by the dispersion cycle in the cavity ??  I'm visualizing that cycle as distorted compared to symmetrical cavity which would want to make it (the world line) curve in x,t.
Title: Re: EM Drive Developments - related to space flight applications - Thread 2
Post by: Rodal on 05/21/2015 08:27 pm
This is also interesting for TM010 mode. Note where the H (magnetic) field is located. At the big end, right where Shawyer feeds in the Rf in the Demonstrator & Flight Thruster EM Drives.

Backs up the Patent mention of TM01 mode.

Fairly clear to me, TM010 is probably Shawyers EM Drive mode.
What is clear is that TM010 is definitely not the mode shape at the reported frequency and dimensions of Shawyer's Flight Thruster

Even at the lower frequency (almost 1/2 of the Flight Thruster) used by NASA Eagleworks (below 2 GHz with a dielectric) they are into a much higher mode shape: TM212

and look at the natural frequency shown on the image you posted above for NASA Eagleworks: TM010 is below 1 GHz without a dielectric
Title: Re: EM Drive Developments - related to space flight applications - Thread 2
Post by: TheTraveller on 05/21/2015 08:36 pm
This is also interesting for TM010 mode. Note where the H (magnetic) field is located. At the big end, right where Shawyer feeds in the Rf in the Demonstrator & Flight Thruster EM Drives.

Backs up the Patent mention of TM01 mode.

Fairly clear to me, TM010 is probably Shawyers EM Drive mode.
What is clear is that TM010 is definitely not the mode shape at the reported frequency and dimensions of Shawyer's Flight Thruster

Just think about it, even at the lower frequency used by NASA Eagleworks (below 2 GHz with a dielectric) they are into a much higher mode shape: TM212

and look at the natural frequency shown on the image you posted above for NASA Eagleworks: TM010 is below 1 GHz without a dielectric

The EW frustum is not the Flight Thruster. When I wake, will mod the SS with the TM01 cutoff equation & see what I will see.
Title: Re: EM Drive Developments - related to space flight applications - Thread 2
Post by: RareSaturn on 05/21/2015 09:01 pm


Interesting that the thrust seems to be "small end forward". Others (Shawyer, NASA, Juan) see opposite thrust, from the large end.

All tests move towards the small end, including these new tests.  I don't think anyone was shown movement toward the large end... ???
Title: Re: EM Drive Developments - related to space flight applications - Thread 2
Post by: Rodal on 05/21/2015 09:04 pm


Interesting that the thrust seems to be "small end forward". Others (Shawyer, NASA, Juan) see opposite thrust, from the large end.

All tests move towards the small end, including these new tests.  I don't think anyone was shown movement toward the large end... ???
With the possible exception of Prof. Yang in China.

I may not recall this correctly since I have not read those papers in a long time.  Can somebody point out to an explicit reference showing actual measurements (not the computer simulations by Yang) of which way did Yang's device move ?

Notice that in this chart (by Shawyer), Prof. Yang's thruster and Shawyer's Flight Thruster are shown with measurements in the opposite direction:

(http://forum.nasaspaceflight.com/index.php?action=dlattach;topic=36313.0;attach=776944;image)
Title: Re: EM Drive Developments - related to space flight applications - Thread 2
Post by: ThereIWas3 on 05/21/2015 09:12 pm
What I want to know is, can a web site be nominated for the Nobel Prize in physics?
Or does Shawyer et all get all the glory when this pans out?

 :)
Title: Re: EM Drive Developments - related to space flight applications - Thread 2
Post by: Taven on 05/21/2015 09:13 pm
It seems the confusion over thrust direction is stemming from people defining the word thrust to mean "pushing out of the back" or "thrusting in the forward direction".
Title: Re: EM Drive Developments - related to space flight applications - Thread 2
Post by: deltaMass on 05/21/2015 09:16 pm
I think "small end forward" resolves ambiguity
Title: Re: EM Drive Developments - related to space flight applications - Thread 2
Post by: TheTraveller on 05/21/2015 09:20 pm


Interesting that the thrust seems to be "small end forward". Others (Shawyer, NASA, Juan) see opposite thrust, from the large end.

All tests move towards the small end, including these new tests.  I don't think anyone was shown movement toward the large end... ???
With the possible exception of Prof. Yang in China.

I may not recall this correctly since I have not read those papers in a long time.  Can somebody point out to an explicit reference showing actual measurements (not the computer simulations by Yang) of which way did Yang's device move ?

Notice that in this chart (by Shawyer), Prof. Yang's thruster and Shawyer's Flight Thruster are shown with measurements in the opposite direction:

(http://forum.nasaspaceflight.com/index.php?action=dlattach;topic=36313.0;attach=776944;image)

Shawyers Thrust is the direction of the internal force imbalance. As a result of that internal force imbalance, the device moves in the Reaction direction.

This is explained in most of his papers.
Title: Re: EM Drive Developments - related to space flight applications - Thread 2
Post by: RotoSequence on 05/21/2015 09:23 pm
I just uploaded the video with the upside down test.  in the next days i will start testing the new setup with the adjustable length.

https://www.youtube.com/watch?v=KAMttfMC8PI (https://www.youtube.com/watch?v=KAMttfMC8PI)

The measured weight on the scale drifts downward with each successive run of the magnetron, becoming a greater and greater negative value each time the power is cycled. How much does this anomaly impact the measured displacement of the EM drive?
Title: Re: EM Drive Developments - related to space flight applications - Thread 2
Post by: deltaMass on 05/21/2015 09:27 pm
It's not a very stable measurement for sure. But we do know that it's in the opposite direction from before.
Title: Re: EM Drive Developments - related to space flight applications - Thread 2
Post by: zellerium on 05/21/2015 09:31 pm
...
That's fine and depends on the kind of material used as a dielectric. I was considering teflon with mu about 10^-6 but you were using HDPE and things can be quite different...

Some information from NASA on their experiments with the truncated cone with the HDPE insert:


............................................

Forgive me if this has been answered already, but when did they change the dielectric to HDPE?
The Anomalous thrust paper clearly states that a PTFE slug was used in the Cannae drive and there is no mention of HDPE in the paper.

Title: Re: EM Drive Developments - related to space flight applications - Thread 2
Post by: SeeShells on 05/21/2015 09:31 pm

Take a gander at the section on energy conservation here:

http://emdrive.echothis.com/Generic_EM_Drive_Information

Just thinking that maybe there should be a third option:  Since a constant acceleration transforms as a 4-space rotational velocity (?), there may be an invariant for the tensor which allows Energy to Momentum conversion. ?  This would be what, a "false force" driven by the dispersion cycle in the cavity ??  I'm visualizing that cycle as distorted compared to symmetrical cavity which would want to make it (the world line) curve in x,t.

I understand what you're trying to say and I like it on several levels. This is one reason I asked if you had thought of introducing another EM wave into the cavity not in phase, but having the ability to control the phase and frequency. Sorry, it's the old dog with a bone syndrome here.

It somewhat fits this statement.

The general conclusion seems to be that a momentum can be transferred to a body, though the relative magnitudes of the momenta and assumptions appear not to be consistent with one
another.
The main result of this letter is that we demonstrate for
a collection of magnetic dipole scatterers that receive a
classical perturbative correction to its permeability from
an external magnetic field, a non-zero momentum transfer
to the body as a whole. It is a fourth order perturbative
result and requires at least four scattering centres
to be present and held in a rigid configuration. Further,
they should be arranged so that the resulting tetrahedron
(with the four particles placed at the vertices) has
no parity symmetry so that the vacuum photons get to
see a chiral structure. If these conditions are fulfilled
then a non-zero momentum develops which scales as the
fourteenth inverse power of the length scale of the tetrahedron.
Whilst the numerical value of this momentum
for the pure quantum vacuum case is far too small to be
measured experimentally, there are hopes of being able
to measure its classical counterpart. The calculations are
performed using the notation and conventions in [18] and
multiple scattering theory.
http://arxiv.org/pdf/1011.4376.pdf
Title: Re: EM Drive Developments - related to space flight applications - Thread 2
Post by: Rodal on 05/21/2015 10:10 pm
...
That's fine and depends on the kind of material used as a dielectric. I was considering teflon with mu about 10^-6 but you were using HDPE and things can be quite different...

Some information from NASA on their experiments with the truncated cone with the HDPE insert:


............................................

Forgive me if this has been answered already, but when did they change the dielectric to HDPE?
The Anomalous thrust paper clearly states that a PTFE slug was used in the Cannae drive and there is no mention of HDPE in the paper.
That's correct. The Brady et.al. report does not mention the dielectric material used in the truncated cone measurements.

ALL the measurements documented in Brady et.al. with the truncated cone having a dielectric were performed with a HDPE dielectric.  The measurements with the PTFE dielectric in the truncated cone are not documented in the Brady et.al. report.  The information came from Paul March, one of the co-authors of the Brady et.al. report.
Title: Re: EM Drive Developments - related to space flight applications - Thread 2
Post by: Rodal on 05/21/2015 10:17 pm


Interesting that the thrust seems to be "small end forward". Others (Shawyer, NASA, Juan) see opposite thrust, from the large end.

All tests move towards the small end, including these new tests.  I don't think anyone was shown movement toward the large end... ???
With the possible exception of Prof. Yang in China.

I may not recall this correctly since I have not read those papers in a long time.  Can somebody point out to an explicit reference showing actual measurements (not the computer simulations by Yang) of which way did Yang's device move ?

Notice that in this chart (by Shawyer), Prof. Yang's thruster and Shawyer's Flight Thruster are shown with measurements in the opposite direction:

(http://forum.nasaspaceflight.com/index.php?action=dlattach;topic=36313.0;attach=776944;image)

Shawyers Thrust is the direction of the internal force imbalance. As a result of that internal force imbalance, the device moves in the Reaction direction.

This is explained in most of his papers.
The attached table shows a column labeled "force direction" which shows, for Prof. Yang's device and for the Flight thruster to have opposite force direction to all the measurements at NASA Eagleworks.

The table shows:

Thruster             Force Direction

NASA                 Thrust
(China)               Reaction
Flight Thruster    Reaction



The images show:

Thust      --> at the BIG END
Reaction --> at the SMALL END



Therefore, it implies

NASA                 Thrust       BIG END
(China)               Reaction     SMALL END
Flight Thruster    Reaction     SMALL END



The one fact we know is what is the direction of the force and the displacement measured at NASA Eagleworks. That force is in the direction of the small diameter.  (No matter whatever Shawyer has written about it, Shawyer cannot speak with more authority about NASA's experiments than Paul March)

Therefore either:

1) According to Shawyer's chart, Prof. Yang's EM Drive moved towards the Big Diameter

or

2) The chart attached by Shawyer is wrong (with respect to the force direction shown for NASA, which Shawyer shows to be towards the Big Diameter)

 
Title: Re: EM Drive Developments - related to space flight applications - Thread 2
Post by: deltaMass on 05/21/2015 10:23 pm
No. You are muddling everything up.
The diagrams show "thrust" to be BIG END FORWARD.
The diagrams show "reaction" to be SMALL END FORWARD.
*except for the symmetric case where "DIELECTRIC END FORWARD" is the reaction direction.

I recommend you use that nomenclature. It's unambiguous.
Title: Re: EM Drive Developments - related to space flight applications - Thread 2
Post by: kdhilliard on 05/21/2015 10:34 pm
Shawyers Thrust is the direction of the internal force imbalance. As a result of that internal force imbalance, the device moves in the Reaction direction.

This is explained in most of his papers.

So far I've only read Shawyer's Theory Paper V 9.4 (http://www.emdrive.com/theorypaper9-4.pdf), in which he writes:

Quote from: Pg. 2:
Thus the radiation pressure at the larger end plate is higher than that at the smaller end plate.  The resulting force difference (F_g1 - F_g2) is multiplied by the Q of the resonant assembly.
Quote from: Pg. 4:
Now λ_g2 > λ_g1, due to the difference in cross-section, and hence F_g1 > F_g2.  Therefore the resultant thrust T will be T = F_g1 - F_g2 = 2P_0/c (λ_0/λ_g1 - λ_0/λ_g2).

On pg. 5-6 he goes on to discuss thrust reversal at high relativistic speeds of the cavity.  That is not directly relevant to this discussion, but he does argue:

Quote from: Pg. 5:
Thus as the velocity of the waveguide increases in the direction of thrust, the thrust will decrease until a limiting velocity is reached when T = 0.

Reading this paper alone, one would assume that the cavity would accelerate large end first due to the force imbalance on the end plates, and that seems to be what he is suggesting on Pg. 5 with "the velocity of the waveguide increases in the direction of the thrust" (presumably as a result of this thrust), but the 2014 summary slide above suggests that Shawyer somewhere argues that this force imbalance somehow causes a reaction in the opposite direction which accelerates the cavity small end first.

Analogously, I understand that if I were "sitting in a tin can, far above the world" and I pressed against the floor, the only way I could maintain that pressure is by bracing myself against the ceiling, and as a result the forces would balance out and my capsule would not accelerate in any direction.  But if somehow I could magically press on the floor harder than I do on the ceiling, the imbalance in forces would cause my capsule to accelerate floor first.  I understand that without the magic holding me in place, I would accelerate toward the ceiling (and thus loose contact with the floor, countering the thrust imbalance), but what is Shawyer's argument that has, in essence, the capsule accelerate ceiling first?

Traveller, can you recommend another of Shawyer's papers to read where I can find this argument?

~Kirk
Title: Re: EM Drive Developments - related to space flight applications - Thread 2
Post by: Blaine on 05/21/2015 10:37 pm
Take a look at this refreshingly good quote on Iulian's website.  I never thought about this until I read this quote.  Makes sense to me.



Good news, quite intriguing!! :)

When comparing the different thrusts, you will also have to take into account that in test 3.1 the thruster has to fight against the upward force of the spring onto which it is attached. This will lower the observed weight change on the scale, but does not necessarily mean the thrust is lower.
Title: Re: EM Drive Developments - related to space flight applications - Thread 2
Post by: deltaMass on 05/21/2015 10:58 pm
No. You are muddling everything up.
The diagrams show "thrust" to be BIG END FORWARD.
The diagrams show "reaction" to be SMALL END FORWARD.

I recommend you use that nomenclature. It's unambiguous.
I don't know who are you addressing as "muddling everything up", but Shawyer's chart shows for NASA's experiments with the frustum of a cone, with a dielectric:

Force direction: thrust

which according to you,  "thrust" means in the direction of the BIG END FORWARD

which therefore means force direction towards BIG END

and that's wrong.  The force measured at NASA was towards the small end, that's one thing we know for a fact, no matter what Shawyer may write about NASA's experiments.

We have discussed the issue of force measurement and displacement measurement at NASA at length with Paul March in these threads.

Sorry but your nomenclature for me is as ambiguous as all get out
Title: Re: EM Drive Developments - related to space flight applications - Thread 2
Post by: TheTraveller on 05/21/2015 11:25 pm
Traveller, can you recommend another of Shawyer's papers to read where I can find this argument?

~Kirk

Page 4 explains it.
Title: Re: EM Drive Developments - related to space flight applications - Thread 2
Post by: TheTraveller on 05/21/2015 11:26 pm
Sorry but your nomenclature for me is as ambiguous as all get out

Page 4 explains it.
Title: Re: EM Drive Developments - related to space flight applications - Thread 2
Post by: Lobo on 05/21/2015 11:28 pm

Welcome to the thread.   Just a short answer and comment regarding the above points.  It was not Chris's article.  I was one of three authors. 

My Apologies Jose Rodel!  I just now went back and looked at the authors.  My laziness for not having done that first.

Thanks for your response.  I appreciate it.  I don't quite understand all of your references, but that gives me a point to start digging a little further.  :-)

Title: Re: EM Drive Developments - related to space flight applications - Thread 2
Post by: TheTraveller on 05/21/2015 11:28 pm
I don't know who are you addressing as "muddling everything up", but Shawyer's chart shows for NASA's experiments with the frustum of a cone, with a dielectric:

Force direction: thrust

which according to you,  "thrust" means in the direction of the BIG END FORWARD

which therefore means force direction towards BIG END

and that's wrong.  The force measured at NASA was towards the small end, that's one thing we know for a fact, no matter what Shawyer may write about NASA's experiments.

We have discussed the issue of force measurement and displacement measurement at NASA at length with Paul March in these threads.

Page 4 explains it.

Note also Shawyers reference to TM01 cutoff & guide wavelength.
Title: Re: EM Drive Developments - related to space flight applications - Thread 2
Post by: Notsosureofit on 05/21/2015 11:33 pm

Take a gander at the section on energy conservation here:

http://emdrive.echothis.com/Generic_EM_Drive_Information

Just thinking that maybe there should be a third option:  Since a constant acceleration transforms as a 4-space rotational velocity (?), there may be an invariant for the tensor which allows Energy to Momentum conversion. ?  This would be what, a "false force" driven by the dispersion cycle in the cavity ??  I'm visualizing that cycle as distorted compared to symmetrical cavity which would want to make it (the world line) curve in x,t.

I understand what you're trying to say and I like it on several levels. This is one reason I asked if you had thought of introducing another EM wave into the cavity not in phase, but having the ability to control the phase and frequency. Sorry, it's the old dog with a bone syndrome here.



The short answer is yes.  As RODAL mentioned above, putting a pair of frequencies at the half-power points of the resonance.  I did my thesis on cylindrical cavity resonance that way, many long years ago.
Title: Re: EM Drive Developments - related to space flight applications - Thread 2
Post by: Lobo on 05/21/2015 11:40 pm
The bottom line is that nobody understands how this thing works.

Like the Wave Motion Engine that was given to us by Iscandar?

It works, we just don't know how...
Title: Re: EM Drive Developments - related to space flight applications - Thread 2
Post by: phaseshift on 05/22/2015 12:03 am
I've seen reference to the Demonstrator Thrusters DF as .844 in 3 different Shawyer documents now.  Hmmmm something seems amiss
Title: Re: EM Drive Developments - related to space flight applications - Thread 2
Post by: phaseshift on 05/22/2015 12:21 am
I've seen reference to the Demonstrator Thrusters DF as .844 in 3 different Shawyer documents now.  Hmmmm something seems amiss
Can you please provide the links (or attach pdf) to all 3 references havign DF = 0.844 ?
Title: Re: EM Drive Developments - related to space flight applications - Thread 2
Post by: SeeShells on 05/22/2015 12:23 am

Take a gander at the section on energy conservation here:

http://emdrive.echothis.com/Generic_EM_Drive_Information

Just thinking that maybe there should be a third option:  Since a constant acceleration transforms as a 4-space rotational velocity (?), there may be an invariant for the tensor which allows Energy to Momentum conversion. ?  This would be what, a "false force" driven by the dispersion cycle in the cavity ??  I'm visualizing that cycle as distorted compared to symmetrical cavity which would want to make it (the world line) curve in x,t.

I understand what you're trying to say and I like it on several levels. This is one reason I asked if you had thought of introducing another EM wave into the cavity not in phase, but having the ability to control the phase and frequency. Sorry, it's the old dog with a bone syndrome here.



The short answer is yes.  As RODAL mentioned above, putting a pair of frequencies at the half-power points of the resonance.  I did my thesis on cylindrical cavity resonance that way, many long years ago.
Thank you, from one old dog to another. Like I've told others I remember when engineering/science was a rock hitting a rock, then it all turned to dirt. That short answer is long on my understanding.
Title: Re: EM Drive Developments - related to space flight applications - Thread 2
Post by: rfmwguy on 05/22/2015 12:51 am
...

The use of HDPE rather than PTFE at MW freqs has been bothering me. Nowhere over 1 GHz have I heard of HDPE, which could indicate high moisture absorption or reflectability. Also, the melting point of HDPE is rather low compared to the 250 deg C of PTFE.

The comparison tables are here: http://www.vanderveerplastics.com/compare-materials.html?sel1=hdpe&sel2=teflon-ptfe-fep

If someone knows why HDPE (typically plastic milk jugs( were initially used, it would be interesting.

"High-density polyethylene (HDPE) or polyethylene high-density (PEHD) is a polyethylene thermoplastic made from petroleum. It is sometimes called "alkathene" or "polythene" when used for pipes.[1] With a high strength-to-density ratio, HDPE is used in the production of plastic bottles, corrosion-resistant piping, geomembranes, and plastic lumber. HDPE is commonly recycled, and has the number "2" as its resin identification code (formerly known as recycling symbol)." per http://en.wikipedia.org/wiki/High-density_polyethylene

HDPE has very interesting properties, it has been used for biomedical applications, for garbage bags and countless commercial applications. It is very tough, it has excellent sliding abrasion resistance and is self lubricating.

As to why it was used as a dielectric for NASA's EM Drive, my recollection is that it was based on prior experience by Paul March, originally working with Prof. Woodward on his Mach Effect theory and experiments, and that it was based, as I noted above, on the imaginary properties acting as absorption coefficients in electromagnetic waves.

By the way, NASA also used PTFE and Nitrile Rubber as dielectrics.  They obtained the highest thrust force with HDPE

This is significant..the difference in dielectric properties of HDPE and PTFE are curious. While HDPE has a lower temp and freq rating, HDPE has an unusual Dielectric Constant (K or E) variance of 1 to 5 while PTFE has confined K of 2-2.1. This is probably why HDPE is not normally used in MW circuits...unpredictability or randomness, if you will. In essence, a puck/layer of HDPE would present a wide K variance across its surface to EM, unlike PTFE. Few materials have this: http://www.rfcafe.com/references/electrical/dielectric-constants-strengths.htm

Thinking out loud, is this perhaps why more thrust was achieved with HDPE as opposed to PTFE? Permittivity in EM fields comes to mind: http://maxwells-equations.com/materials/permittivity.php

Perhaps some folks can wrap their heads around this K variance versus thrust...I have a brain cramp :o
Title: Re: EM Drive Developments - related to space flight applications - Thread 2
Post by: deltaMass on 05/22/2015 12:55 am
IULIAN:

Hi,
.......snip
Iulian

Another thing to consider; From your video you have the unit on the end of a looped spring hanging from a shelf....  For the Downwards test you are trying to force the unit downwards AGAINST the natural TENSION of the spring...  you need to measure just how much energy it takes to pull the spring down as much as the unit did when you powered it up.!

The original "thrust was with the aid of the spring pulling the unit upwards. 

Placing the complete unit onto a balance board "like a child's see-saw,  American teeter-totter"  with an equal weight on the other end will enable you to perform these types of measurements
That's all true, and hanging one's hat on the value of the thrust and the air mass change is not to be recommended yet. I shouldn't have quoted them to three significant figures because the experimental precision is in no way that good. I also noticed how much the readings were changing during this very latest test.

But we do know that the net force changes direction when the frustum is flipped upside down. That fact alone makes it all worthwhile continuing.
Title: Re: EM Drive Developments - related to space flight applications - Thread 2
Post by: Rodal on 05/22/2015 01:05 am
...HDPE has an unusual Dielectric Constant (K or E) variance of 1 to 5 while PTFE has confined K of 2-2.1. This is probably why HDPE is not normally used in MW circuits...unpredictability or randomness, if you will. In essence, a puck/layer of HDPE would present a wide K variance across its surface to EM, unlike PTFE. Few materials have this: http://www.rfcafe.com/references/electrical/dielectric-constants-strengths.htm
...

HDPE is not random.  It is fairly straightforward to characterize as having well determined properties, based on its molecular weight for example, and method of manufacture.  If the properties of HDPE would be random or difficult to characterize, believe me that HDPE would not be used for biomedical applications, which have higher standards for material properties than many commercial applications do.

What happened here is that you are looking at this row:

High Density Polyethylene (HDPE), Molded   1.0 - 5.0

instead of looking at this one, which is the appropriate row to look at:

Polyethylene LDPE/HDPE     2.26 @ 1 MHz
                                                2.26 @ 3 GHz

Those are the appropriate properties for the NASA Eagleworks dielectric.
Now: that's a narrow range, and it even gives you data at the GHz range for direct application to microwaves.

NASA Eagleworks did not use a molded product (see my previous posts, given the identity of the product used by NASA). 

Having said that, the fact that this website gives properties for HDPE in two completely different rows, with different ranges, and does not explain the differences does not give me a good impression about the quality of the data in this website (which is also the same website I initially found when I was quickly looking for the HDPE properties)

Authoritative handbooks like this one give much more reliable data than these websites: http://bit.ly/1Lr0pSt

Of course, the best thing is to have analytical instruments to properly characterize the material properties of a polymer, which is what we did.



Note that in this same website they also have this funny note, acknowledging that they had the tan delta for PTFE off by a factor of 10 until Craig  ;) found the error !!!!!

Quote
Thanks to Craig B. for correcting the loss tangent for Teflon (0.00028 rather than 0.0028).
Title: Re: EM Drive Developments - related to space flight applications - Thread 2
Post by: WarpTech on 05/22/2015 01:43 am
I've seen reference to the Demonstrator Thrusters DF as .844 in 3 different Shawyer documents now.  Hmmmm something seems amiss
Can you please provide the links (or attach pdf) to all 3 references havign DF = 0.844 ?

These are basically all the "same" paper. Copy and Paste.
Title: Re: EM Drive Developments - related to space flight applications - Thread 2
Post by: LasJayhawk on 05/22/2015 01:46 am

Take a gander at the section on energy conservation here:

http://emdrive.echothis.com/Generic_EM_Drive_Information

Just thinking that maybe there should be a third option:  Since a constant acceleration transforms as a 4-space rotational velocity (?), there may be an invariant for the tensor which allows Energy to Momentum conversion. ?  This would be what, a "false force" driven by the dispersion cycle in the cavity ??  I'm visualizing that cycle as distorted compared to symmetrical cavity which would want to make it (the world line) curve in x,t.

I understand what you're trying to say and I like it on several levels. This is one reason I asked if you had thought of introducing another EM wave into the cavity not in phase, but having the ability to control the phase and frequency. Sorry, it's the old dog with a bone syndrome here.



The short answer is yes.  As RODAL mentioned above, putting a pair of frequencies at the half-power points of the resonance.  I did my thesis on cylindrical cavity resonance that way, many long years ago.
Thank you, from one old dog to another. Like I've told others I remember when engineering/science was a rock hitting a rock, then it all turned to dirt. That short answer is long on my understanding.

Old??? I was looking at ion drives and wondering why we don't use beam focusing plates like a 6L6. :)

But I am wondering something. If this thing works on standing waves, and standing waves require a node on both ends of the cavity, wouldn't this thing have a cylinder of standing waves surrounded by traveling waves?

Title: Re: EM Drive Developments - related to space flight applications - Thread 2
Post by: phaseshift on 05/22/2015 02:16 am
I've seen reference to the Demonstrator Thrusters DF as .844 in 3 different Shawyer documents now.  Hmmmm something seems amiss
Can you please provide the links (or attach pdf) to all 3 references havign DF = 0.844 ?

These are basically all the "same" paper. Copy and Paste.

Maybe this was an intentionally moldy breadcrumb.
Title: Re: EM Drive Developments - related to space flight applications - Thread 2
Post by: WarpTech on 05/22/2015 02:21 am

"Mimics G".. I like that phrase, a lot.

... nice!!.

I would be cautious of closing the door on em-density. Instinct still has me thinking possibly more of an E relationship than B:H, but a rig to test it implies considerable values of E.  Somewhere lurking in here I think polarisation is also a factor.

Thanks! You may be right about the energy density playing a significant roll, but I'm still formulating the equations and exploring various options. With a 3 day weekend, I may have time to write some of this stuff down in MathType.

My understanding of gravity is unique in that I prefer the PV Model of gravity over GR for engineering purposes, and I have a quantum gravity model for engineers that works well enough. Relative to EM waves, it works like a variable refractive index, K. In the case of "real" gravity, it affects all wavelengths equally. In the case of a waveguide, K only affects a narrow bandwidth near the cut-off. It doesn't affect "test particles". However, IMO the cause is the same, attenuation of the wave function due to a change in the refractive index, i.e., a change in the group velocity.

My hypothesis for how the EM Drive works is more like Frame Dragging than a rocket. The photons drag the frustum in the direction they are moving. They do not propel it in the opposite direction like a photon rocket, or how it is being described by Shawyer and others. The "drag" force is asymmetrical because the attenuation cut-offs are asymmetrical.

I think that if there were a charged particle oscillating from end to end inside, it would be more obvious than when it's photons. The charge feels the force of it's oppositely charged reflection in the copper ground plane, and wants to drag it along with it. It would be an interesting problem to calculate the electric field of a point charge, inside a frustum and see what the forces on that test particle would be.  ::)

Todd
Title: Re: EM Drive Developments - related to space flight applications - Thread 2
Post by: SeeShells on 05/22/2015 02:29 am
...HDPE has an unusual Dielectric Constant (K or E) variance of 1 to 5 while PTFE has confined K of 2-2.1. This is probably why HDPE is not normally used in MW circuits...unpredictability or randomness, if you will. In essence, a puck/layer of HDPE would present a wide K variance across its surface to EM, unlike PTFE. Few materials have this: http://www.rfcafe.com/references/electrical/dielectric-constants-strengths.htm
...

HDPE is not random.  It is fairly straightforward to characterize as having well determined properties, based on its molecular weight for example, and method of manufacture.  If the properties of HDPE would be random or difficult to characterize, believe me that HDPE would not be used for biomedical applications, which have higher standards for material properties than many commercial applications do.

What happened here is that you are looking at this row:

High Density Polyethylene (HDPE), Molded   1.0 - 5.0

instead of looking at this one, which is the appropriate row to look at:

Polyethylene LDPE/HDPE     2.26 @ 1 MHz
                                                2.26 @ 3 GHz

Those are the appropriate properties for the NASA Eagleworks dielectric.
Now: that's a narrow range, and it even gives you data at the GHz range for direct application to microwaves.

NASA Eagleworks did not use a molded product (see my previous posts, given the identity of the product used by NASA). 

Having said that, the fact that this website gives properties for HDPE in two completely different rows, with different ranges, and does not explain the differences does not give me a good impression about the quality of the data in this website (which is also the same website I initially found when I was quickly looking for the HDPE properties)

Authoritative handbooks like this one give much more reliable data than these websites: http://bit.ly/1Lr0pSt

Of course, the best thing is to have analytical instruments to properly characterize the material properties of a polymer, which is what we did.



Note that in this same website they also have this funny note, acknowledging that they had the tan delta for PTFE off by a factor of 10 until Craig  ;) found the error !!!!!

Quote
Thanks to Craig B. for correcting the loss tangent for Teflon (0.00028 rather than 0.0028).
Well dang, here I thought we had nailed down the mystery acceleration. Alas, we still have this systemic anomaly ( I love that phrase). All along I thought we could point our fingers at the chirality of twisted polymer crystals.  http://www.esrf.eu/UsersAndScience/Publications/Highlights/2011/scm/scm4

Title: Re: EM Drive Developments - related to space flight applications - Thread 2
Post by: phaseshift on 05/22/2015 02:55 am
Do we have the Flight Thruster Dimensions?

I have found

bD: .265m
height: .164m

freq: 3.85GHz

DF: ?
sD: ?



Title: Re: EM Drive Developments - related to space flight applications - Thread 2
Post by: SeeShells on 05/22/2015 03:00 am

Old??? I was looking at ion drives and wondering why we don't use beam focusing plates like a 6L6. :)

But I am wondering something. If this thing works on standing waves, and standing waves require a node on both ends of the cavity, wouldn't this thing have a cylinder of standing waves surrounded by traveling waves?
I have a couple of 6L6s in the old radio I just rebuilt, because it's just like the one I rebuilt when I was 14. Hats off to you.

I still have an old T-shirt from the 60's that had the year 2000 on the top and a group of flying cars displayed below. We're a few years behind in having flying cars that levitate and spacecraft that zoom around, but I'd sure love to see it happen before...you know. I think we're close, very close to that childhood dream. (sorry if I got off track but we're all dreamers here)

As far as your question of a cavity of standing waves surrounded by standing waves you might have a look at these different modes. 

Title: Re: EM Drive Developments - related to space flight applications - Thread 2
Post by: WarpTech on 05/22/2015 03:12 am
This is also interesting for TM010 mode. Note where the H (magnetic) field is located. At the big end, right where Shawyer feeds in the Rf in the Demonstrator & Flight Thruster EM Drives.

Backs up the Patent mention of TM01 mode.

Fairly clear to me, TM010 is probably Shawyers EM Drive mode.
What is clear is that TM010 is definitely not the mode shape at the reported frequency and dimensions of Shawyer's Flight Thruster

Even at the lower frequency (almost 1/2 of the Flight Thruster) used by NASA Eagleworks (below 2 GHz with a dielectric) they are into a much higher mode shape: TM212

and look at the natural frequency shown on the image you posted above for NASA Eagleworks: TM010 is below 1 GHz without a dielectric

The TM01 mode is the mode shown to have the highest reflection, per Zeng and Fan's paper.

https://www.osapublishing.org/oe/fulltext.cfm?uri=oe-17-1-34&id=175583

It also has fairly high attenuation. To raise Q, Shawyer needed more reflection, so he made the cone angle greater, which may or may not be counter productive at the lower attenuation value depending on how high a Q he can get. You can see why in the attachments.
Title: Re: EM Drive Developments - related to space flight applications - Thread 2
Post by: phaseshift on 05/22/2015 03:27 am
Do we have the Flight Thruster Dimensions?  I have found  bD: .265 m   height: .164m
Based on the photograph, and knowing that bD: .265m; height: .164m; what is your estimate of the small diameter ?

I'll work on it tonight :)
Title: Re: EM Drive Developments - related to space flight applications - Thread 2
Post by: WarpTech on 05/22/2015 03:54 am

"Mimics G".. I like that phrase, a lot.

... nice!!...

...

I think that if there were a charged particle oscillating from end to end inside, it would be more obvious than when it's photons. The charge feels the force of it's oppositely charged reflection in the copper ground plane, and wants to drag it along with it. It would be an interesting problem to calculate the electric field of a point charge, inside a frustum and see what the forces on that test particle would be.  ::)

Todd

To elaborate on this, even without having to solve the electric field problem. In a gravitational field around a planet, where the refractive index is approximated as;

K ~ exp[2*G*M/R*c^2]

Voltage varies as 1/sqrt(K), where R is the distance to the center of the Earth, of mass M for example.

The voltage potential at the radius r from the charge is,

V(r) = Q/4pi*eps0*r

Therefore;

V(r, R) = V(r)*exp[-G*M/R*c^2]

Obviously, a particle closer to the Earth will have a lower voltage potential than one high in orbit. Now, consider a charged particle in a copper cone. It has a voltage potential relative to it's reflection in the copper ground plane. That potential is lowest at the apex of the cone. Once again, a frustum mimic's gravity, very well I might add!

Todd




Title: Re: EM Drive Developments - related to space flight applications - Thread 2
Post by: LasJayhawk on 05/22/2015 04:09 am

Old??? I was looking at ion drives and wondering why we don't use beam focusing plates like a 6L6. :)

But I am wondering something. If this thing works on standing waves, and standing waves require a node on both ends of the cavity, wouldn't this thing have a cylinder of standing waves surrounded by traveling waves?
I have a couple of 6L6s in the old radio I just rebuilt, because it's just like the one I rebuilt when I was 14. Hats off to you.

I still have an old T-shirt from the 60's that had the year 2000 on the top and a group of flying cars displayed below. We're a few years behind in having flying cars that levitate and spacecraft that zoom around, but I'd sure love to see it happen before...you know. I think we're close, very close to that childhood dream. (sorry if I got off track but we're all dreamers here)

As far as your question of a cavity of standing waves surrounded by standing waves you might have a look at these different modes.

I downloaded it when you posted it earlier, but to be honest, when I look at the figures I see a cylinder, not a frustrum. Don't ask for the math, I can't give it to you. :(

But there is something here about power that is worth a mention. I had a pre WWII Stromberg-Carlson with a pair of 6L6's in push pull ( and an tuned labyrinth speaker to boot) and my current baby a 37 Philco 37-116 with a pair of 6B4G's. I doubt either could push much more than 20 watts, but both can move more air than a modern radio rated at 200+ watts, and sound better as well. Note to smart people: AM radio doesn't sound bad, the receivers built in the last 60 years do.

There is something old school about this, and I'm still trying to put my finger on it. But I am leaning toward switching to a bow tie shape, and feeding both ends 90 degrees from what we are doing now ( bounce off the sides, not the ends )
Title: Re: EM Drive Developments - related to space flight applications - Thread 2
Post by: WarpTech on 05/22/2015 05:08 am
I don't think Shawyer's math is correct, but based on some conversations here, I worked out a simple equation to optimize Df to determine the size of the small end;

y = lambda,

y_0/y_g1 = (1 + y_g2*Df/y_0) / (Df + y_g2/y_0)

If Df = 1, then y_0/y_g1 = 1, independent of what y_g2 is.

If Df = 0, then y_0/y_g1 = y_0/y_g2, and we have a cylinder with no taper.


Todd
Title: Re: EM Drive Developments - related to space flight applications - Thread 2
Post by: deltaMass on 05/22/2015 05:52 am
I recall working out a similar result a few pages back
Title: Re: EM Drive Developments - related to space flight applications - Thread 2
Post by: Chrochne on 05/22/2015 07:00 am
Lately. I see a lot of new criticism. I think its fine, but I also think that there were a lot of tests that have proven that there is a thrust.

I am writting this to share this idea. Is it possible to convince some of the main critics of this device to manufacture their own EmDrive and test it? I think that their criticism can not be taken seriously and scientificaly correct, if they do not make their own tests. I say yes to criticism, but I can not take it seriously until they make their own tests.

We also really need FAQ a lot :-P. Too much folks are asking about the questions that were already answered and properly tested before.

I also hope this thread will return to more scientific thread and maybe we can hear from Mr. Paul and White again here in the future.

With kind regards,

Chrochne
Title: Re: EM Drive Developments - related to space flight applications - Thread 2
Post by: zen-in on 05/22/2015 07:12 am
Lately. I see a lot of new criticism. I think its fine, but I also think that there were a lot of tests that have proven that there is a thrust.

I am writting this to share this idea. Is it possible to convince some of the main critics of this device to manufacture their own EmDrive and test it? I think that their criticism can not be taken seriously and scientificaly correct, if they do not make their own tests. I say yes to criticism, but I can not take it seriously until they make their own tests.
...
Chrochne

I am a critic because I have not seen consistent evidence it works.  I also have not seen any experiments done that would rule out alternative explanations.   I am also very handy with metalwork, RF, etc and could build one in a few days.    However I choose not to do this because I have other more interesting experiments I want to do.
Title: Re: EM Drive Developments - related to space flight applications - Thread 2
Post by: TheTraveller on 05/22/2015 08:38 am
Do we have the Flight Thruster Dimensions?  I have found  bD: .265 m   height: .164m
Based on the photograph, and knowing that bD: .265m; height: .164m; what is your estimate of the small diameter ?

Those are OUTSIDE dimensions. We need INSIDE dimensions.
Title: Re: EM Drive Developments - related to space flight applications - Thread 2
Post by: TheTraveller on 05/22/2015 08:46 am
This is also interesting for TM010 mode. Note where the H (magnetic) field is located. At the big end, right where Shawyer feeds in the Rf in the Demonstrator & Flight Thruster EM Drives.

Backs up the Patent mention of TM01 mode.

Fairly clear to me, TM010 is probably Shawyers EM Drive mode.
What is clear is that TM010 is definitely not the mode shape at the reported frequency and dimensions of Shawyer's Flight Thruster

Even at the lower frequency (almost 1/2 of the Flight Thruster) used by NASA Eagleworks (below 2 GHz with a dielectric) they are into a much higher mode shape: TM212

and look at the natural frequency shown on the image you posted above for NASA Eagleworks: TM010 is below 1 GHz without a dielectric

The TM01 mode is the mode shown to have the highest reflection, per Zeng and Fan's paper.

https://www.osapublishing.org/oe/fulltext.cfm?uri=oe-17-1-34&id=175583

It also has fairly high attenuation. To raise Q, Shawyer needed more reflection, so he made the cone angle greater, which may or may not be counter productive at the lower attenuation value depending on how high a Q he can get. You can see why in the attachments.

TM01 also puts the biggest H field at the big end, which is where he couples in his external Rf. I feel the visible fact of where Shawyer feeds in his Rf, in the Demonstrator, Flight Thruster1 and Flight Thruster2 is an important bread crumb.

Engineering is always about compromise.
Title: Re: EM Drive Developments - related to space flight applications - Thread 2
Post by: TheTraveller on 05/22/2015 08:49 am
Do we have the Flight Thruster Dimensions?

I have found

bD: .265m
height: .164m

freq: 3.85GHz

DF: ?
sD: ?

Those are EXTERNAL dimensions. As far as I can find Shawyer has never given the Df for the Flight Thruster. He has stated the Q as 60,000, then later 50,000.
Title: Re: EM Drive Developments - related to space flight applications - Thread 2
Post by: TheTraveller on 05/22/2015 08:54 am
IULIAN:

Hi,
.......snip
Iulian

Another thing to consider; From your video you have the unit on the end of a looped spring hanging from a shelf....  For the Downwards test you are trying to force the unit downwards AGAINST the natural TENSION of the spring...  you need to measure just how much energy it takes to pull the spring down as much as the unit did when you powered it up.!

The original "thrust was with the aid of the spring pulling the unit upwards. 

Placing the complete unit onto a balance board "like a child's see-saw,  American teeter-totter"  with an equal weight on the other end will enable you to perform these types of measurements

Shawyer likes his Teeter Totters.

Paul March said EW is building a version of the Shawyer Teeter Totter balance beam.

May be the way to measure the forces generated.
Title: Re: EM Drive Developments - related to space flight applications - Thread 2
Post by: Rodal on 05/22/2015 11:53 am
This is also interesting for TM010 mode. Note where the H (magnetic) field is located. At the big end, right where Shawyer feeds in the Rf in the Demonstrator & Flight Thruster EM Drives.

Backs up the Patent mention of TM01 mode.

Fairly clear to me, TM010 is probably Shawyers EM Drive mode.
What is clear is that TM010 is definitely not the mode shape at the reported frequency and dimensions of Shawyer's Flight Thruster

Even at the lower frequency (almost 1/2 of the Flight Thruster) used by NASA Eagleworks (below 2 GHz with a dielectric) they are into a much higher mode shape: TM212

and look at the natural frequency shown on the image you posted above for NASA Eagleworks: TM010 is below 1 GHz without a dielectric

The TM01 mode is the mode shown to have the highest reflection, per Zeng and Fan's paper.

https://www.osapublishing.org/oe/fulltext.cfm?uri=oe-17-1-34&id=175583

It also has fairly high attenuation. To raise Q, Shawyer needed more reflection, so he made the cone angle greater, which may or may not be counter productive at the lower attenuation value depending on how high a Q he can get. You can see why in the attachments.

You are correct.  The lowest modes have the highest amplitudes.  This is a basic fact in everything that has to do with resonance, from vibration of structures to electromagnetic resonance.

Here are two more facts:

1) Although the highest thrust is achieved with the highest Design Factor, approaching the value of 1, Shawyer's actual experiments were conducted with geometries having a Design Factor significantly lower than 1.

2) Although the lowest modes are known to have the highest amplitudes and Shawyer has written about mode TM010 the fact is that Shawyer's actual experiments were conducted with geometries and frequencies that excited modes much higher than TM010. 


Fact #1 is not being argued against because everybody can calculate the Design Factor with a simple Excel spreadsheet or a calculator.

Fact #2 is presently argued here by those that presently do not have the capability to calculate the actual mode shapes of a truncated cone.  But this is not a discussion about politics or economics, where people with completely different views can argue indefinitely without ever arriving at a conclusion.  Mother Nature has physical laws that govern the electromagnetic resonance of these cavities based on their geometry. 

There is no TM010 for a truncated cone.  A truncated cone has to have a variation of the electromagnetic field in the longitudinal direction.  What is being labeled as TM010 is a degenerate mode that has a variation in the longitudinal direction and is closest to the cylindrical TM010.

The fact is that to excite cylTM010 (the degenerate mode close to TM010) at 2.45 GHz or higher frequencies will take different geometrical dimensions than Shawyer used in his experiments or that are being discussed in this thread.  This is self-evident to anyone that has the capability to calculate the mode shapes of truncated cones and/or has had experience with analysis of resonance and vibration problems. 

All I have to do is to plug the dimensions of the small and big diameter and the height of the frustum in Mathematica and in a few seconds I get the mode shapes.  Those that do not have the capability to calculate the mode shapes are advised to click at the picture below, identify mode shape cylTM010 (the lowest line at the bottom of the picture) and read the vertical axis to ascertain the frequency associated with this mode shape (here shown for a huge variation in small diameter, going from almost a pointy cone at the left to a perfect cylinder at the right).
Title: Re: EM Drive Developments - related to space flight applications - Thread 2
Post by: Fugudaddy on 05/22/2015 12:29 pm
When i saw the people start thinking i died i was concerned. I wanted to post when i have some results but now i`m forced me to post some unfinished work :)

Something to remember in regards to the latest results Iulian posted; the request from the people here was to first flip the thing over and see if there was still thrust, to counter the effects of buoyancy.

It seemed that his latest tests showed that there was, indeed, some actual thrust effect that can't be attributed to hot air.

That seems like a big enough deal to me, even if, at this time, the actual exact numbers are still being determined. The more results that are out there is the more data that can be analyzed. :)

I don't want to speak for Iulian, but it seems his goal is similar to TheTraveller's; make a practical model and tweak from there.

As an aside- science teachers should show this thread to students. This is science in all it's hands on, get dirty, push ideas back and forth and come up with answers to things that don't seem to have answers glory.
Title: Re: EM Drive Developments - related to space flight applications - Thread 2
Post by: kdhilliard on 05/22/2015 12:38 pm
We also really need FAQ a lot :-P. Too much folks are asking about the questions that were already answered and properly tested before.

Chrochne, a few brave NSF soles are working on that over at http://emdrive.echothis.com/ and they could use some assistance if you can spare some time.

As I'll never have time to read through the previous 200+ pages ...

Lobo, you may be interested in scanning Paul March's posts (March, a.k.a. Star-Drive, is an engineer at Eaglewoks), perhaps starting with his 2015-02-06 report (http://forum.nasaspaceflight.com/index.php?topic=36313.msg1326608#msg1326608) on their recent vacuum results, and the follow-ups discussing the magnitude of the expected thermal effects on their thrust stand in comparison to their thrust measurements.  Unfortunately, if you view them from his profile's posts list (http://forum.nasaspaceflight.com/index.php?action=profile;area=showposts;u=2074), there is no way to tell if there are any attached images short of clicking on the post and viewing it in the appropriate forum, which is important for posts such as with this 2015-02-07 response (http://forum.nasaspaceflight.com/index.php?topic=36313.msg1327012#msg1327012) to Rodal.  You can, of course, view a list of his attachments (http://forum.nasaspaceflight.com/index.php?action=profile;area=showposts;sa=attach;u=2074) separately, but it would be useful if it were possible to pull up a list of one user's posts with attachments displayed, all in one place.

In any case, if you or anyone else wishes to step through this history, please take notes and contribute to the wiki.

~Kirk
Title: Re: EM Drive Developments - related to space flight applications - Thread 2
Post by: Rodal on 05/22/2015 01:46 pm
This is also interesting for TM010 mode. Note where the H (magnetic) field is located. At the big end, right where Shawyer feeds in the Rf in the Demonstrator & Flight Thruster EM Drives.

Backs up the Patent mention of TM01 mode.

Fairly clear to me, TM010 is probably Shawyers EM Drive mode.

(http://forum.nasaspaceflight.com/index.php?action=dlattach;topic=36313.0;attach=831551;image)

There is no TM010 for a truncated cone. TM010 means m=0;n=1;p=0. 

p=0 means constant electromagnetic field in the longitudinal direction. That is a basic fact.

If the mode would be constant in the longitudinal direction, it would make absolutely no difference where to plug the RF connection. 

The picture you posted shows a variation of the electromagnetic field in the longitudinal direction, therefore it is obvious that this cannot be TM010.

Truncated cones do not have the mode shape TM010.

A truncated cone has to have a variation of the electromagnetic field in the longitudinal direction. 

What is being labeled as TM010 is a "degenerate" mode that has a variation in the longitudinal direction and is closest to the cylindrical TM010.

COMSOL's Finite Element Analysis does not give the values of m,n,p, it only gives the natural frequencies and the mode shape images.  NASA's FEA analyst had to come up with a designation for the mode (there is no standard notation for truncated cone mode shapes).  He chose this designation because this degenerate mode had a frequency below that of the other mode that "looks" like TM011.

This degenerate mode has no analogous mode in a cylindrical cavity.

In a truncated cone, TM010 gets cut-off and mode TM011 gets bifurcated into two degenerate modes: one with a lower frequency and another one with a higher frequency.

This mode can only take place in a truncated cone.



Another "bread crumb": concerning "Patent mention of TM01 mode", there are an infinite number of TM modes having m=0, n=1, and p=1,2,3...Infinity.  None of them (for a truncated cone) have p = 0
Title: Re: EM Drive Developments - related to space flight applications - Thread 2
Post by: deltaMass on 05/22/2015 01:57 pm
Do you have a way to estimate feed impedance from feed position, for a given mode?
Or, put another way - given the mode, what is the locus of (50 + j0) ohm feed points?
Title: Re: EM Drive Developments - related to space flight applications - Thread 2
Post by: rfmwguy on 05/22/2015 02:06 pm
...HDPE has an unusual Dielectric Constant (K or E) variance of 1 to 5 while PTFE has confined K of 2-2.1. This is probably why HDPE is not normally used in MW circuits...unpredictability or randomness, if you will. In essence, a puck/layer of HDPE would present a wide K variance across its surface to EM, unlike PTFE. Few materials have this: http://www.rfcafe.com/references/electrical/dielectric-constants-strengths.htm
...

HDPE is not random.  It is fairly straightforward to characterize as having well determined properties, based on its molecular weight for example, and method of manufacture.  If the properties of HDPE would be random or difficult to characterize, believe me that HDPE would not be used for biomedical applications, which have higher standards for material properties than many commercial applications do.

What happened here is that you are looking at this row:

High Density Polyethylene (HDPE), Molded   1.0 - 5.0

instead of looking at this one, which is the appropriate row to look at:

Polyethylene LDPE/HDPE     2.26 @ 1 MHz
                                                2.26 @ 3 GHz

Those are the appropriate properties for the NASA Eagleworks dielectric.
Now: that's a narrow range, and it even gives you data at the GHz range for direct application to microwaves.

NASA Eagleworks did not use a molded product (see my previous posts, given the identity of the product used by NASA). 

Having said that, the fact that this website gives properties for HDPE in two completely different rows, with different ranges, and does not explain the differences does not give me a good impression about the quality of the data in this website (which is also the same website I initially found when I was quickly looking for the HDPE properties)

Authoritative handbooks like this one give much more reliable data than these websites: http://bit.ly/1Lr0pSt

Of course, the best thing is to have analytical instruments to properly characterize the material properties of a polymer, which is what we did.



Note that in this same website they also have this funny note, acknowledging that they had the tan delta for PTFE off by a factor of 10 until Craig  ;) found the error !!!!!

Quote
Thanks to Craig B. for correcting the loss tangent for Teflon (0.00028 rather than 0.0028).
Well dang, here I thought we had nailed down the mystery acceleration. Alas, we still have this systemic anomaly ( I love that phrase). All along I thought we could point our fingers at the chirality of twisted polymer crystals.  http://www.esrf.eu/UsersAndScience/Publications/Highlights/2011/scm/scm4

Agree See, not sure we should give up on the thrust differences between HDPE and PTFE. All else being the same, why the difference? You or Doc might have some insight into the chemistry, but EM properties are what we're interested in.

Base question is: Why does HDPE preform better than PTFE in high EM field experiments? (HDPE does have roughly half the thermal resistance of PTFE).

Here's an old Bulgarian Science Foundation paper where an HDPE was studied using MW (2-10 GHz) : ftp://213.176.96.142/sciencedirect48477b28-4072-20141124031119.pdf

Seems they are quite interested in absorption and reflection performance with the introduction of acetylene carbon black (CB) into HDPE. Doncha love mixing  chemistry and electronics?  :D

"It is obvious that the CB concentration influence on the microwave properties is the strongest in the frequency range 2–3 GHz. At optimum CB concentration (17.5 mass%), the coefficient of attenuation reaches 5 dB/mm,
while the coefficient of reflection is close to the starting HDPE coefficient, i.e., the absorption
activity is more than 10 times greater than the reflection activity."

Title: Re: EM Drive Developments - related to space flight applications - Thread 2
Post by: deltaMass on 05/22/2015 02:17 pm
...Base question is: Why does HDPE preform better than PTFE in high EM field experiments? (HDPE does have roughly half the thermal resistance of PTFE)....
Paul March's answer is that it has to do with the imaginary components of the electric permittivity and magnetic permeability, and the choice was based on his experience working with Prof. Woodward.

I haven't found values for the imaginary components of the electric permittivity and magnetic permeability of HDPE in the literature.

To assess this issue we need actual experimental values of the imaginary components of the electric permittivity and magnetic permeability of HDPE.
Perhaps counterintuitively, these imaginary components are dissipative. Which is A Bad Thing. Probably :)
Title: Re: EM Drive Developments - related to space flight applications - Thread 2
Post by: Rodal on 05/22/2015 02:22 pm
...perhaps counterintuitively, these imaginary components are dissipative. Which is A Bad Thing. Probably :)
Of course they are dissipative, but it is straightforward to show that the time-average value of the Poynting vector, which is zero for a non-dissipative homogeneous cavity becomes non-zero for an asymmetric cavity with a dielectric with dissipative properties (one can show this even just based on the electric Tan Delta, as I did in the thread many pages ago).

Even a cavity just having one end hotter than the other will result in propulsion (due to thermal radiation as in the Pioneer anomaly), albeit very small to justify the researchers claims (solely based on thermal radiation).
Title: Re: EM Drive Developments - related to space flight applications - Thread 2
Post by: phaseshift on 05/22/2015 02:39 pm
I also think the cavity length needs to be adjusted on the Demonstrator Thruster to .187m....

0.187 m is the minimum axial length.  The Demonstrator has a gear-driven mechanism at the small end in order to change the internal length to tune the cavity.

Could you please figure out -from the picture- the maximum length for the Demonstrator so that we can put both these values (minimum length and maximum length) in the wiki (http://emdrive.echothis.com/Experimental_Results) for EM Drive ?

How can the maximum be determined? We have no idea what is inside the cylinder. It could be almost empty except for the plate or chock full of more gears.  .187 is the end of the cone, any additional possible length is up to what's inside.ideas?
Title: EM Drive Developments - related to space flight applications - Thread 2
Post by: Star One on 05/22/2015 03:08 pm
Haven't seen anyone post this to the thread so far.

https://hackaday.io/project/5596-em-drive

Plus interview with the team leader.

http://n-o-d-e.net/post/119343131451/building-a-diy-emdrive
Title: Re: EM Drive Developments - related to space flight applications - Thread 2
Post by: phaseshift on 05/22/2015 03:13 pm

Either use the thickness of the interior plate and gears equal to zero, or use your best guesstimate.


Which is pretty much the length of the cylinder - why don't we put that in the wiki.   It's up to anyone's interpretation of the thickness of the plate and dimensions of internal mechanisms that determine what the maximum is - setting that to zero is the cylinder length. Publishing what we think the cylinder length is allows additional estimates to be worked out by others.

It appears there are two rods entering the cavity, probably for stability.  How they attach to the plate is going to take up a few millimeters and could add a handful of millimeters to the thickness of the plate. 

I'll jump on this in a bit - certainly after, and not before, morning coffee.
Title: Re: EM Drive Developments - related to space flight applications - Thread 2
Post by: phaseshift on 05/22/2015 03:35 pm
Haven't seen anyone post this to the thread so far.

https://hackaday.io/project/5596-em-drive

Plus interview with the team leader.

http://n-o-d-e.net/post/119343131451/building-a-diy-emdrive

I like they went to 25GHz.  :)  Also interesting they are 3D printing plastic parts and then coating with metal. 
Title: Re: EM Drive Developments - related to space flight applications - Thread 2
Post by: deltaMass on 05/22/2015 04:06 pm
Haven't seen anyone post this to the thread so far.

https://hackaday.io/project/5596-em-drive

Plus interview with the team leader.

http://n-o-d-e.net/post/119343131451/building-a-diy-emdrive
Excellent set of strategies these guys are using. In parallel they're doing a Shawyer/Chinese replication attempt @2.4 GHz, and also building a 25 GHz beast with the aim of popping it into a PocketQub and sending it into space (which one of them has already done with a different project).
Title: Re: EM Drive Developments - related to space flight applications - Thread 2
Post by: kdhilliard on 05/22/2015 04:43 pm
Regarding Shawyer's prediction for the direction of thrust vs. acceleration (big-endian vs. little-endian):

Traveller, can you recommend another of Shawyer's papers to read where I can find this argument?

Page 4 [of the Toulouse 2010 paper (http://forum.nasaspaceflight.com/index.php?action=dlattach;topic=36313.0;attach=831580)] explains it.

Thanks Traveller.  I just finished reading it.  The theory portion contains a bit more exposition but gives the same equations as in the V 9.4 Theory Paper (http://www.emdrive.com/theorypaper9-4.pdf), with the exception that V 9.4 also analyzes a cavity with a dielectric-filled end section (so equation (9) from the 2010 paper corresponds with the slightly different equation (11) of V 9.4 since he doesn't introduce the thrust multiplier effect of Q in V 9.4 until the dielectric-filled end section analysis).

As an aside, I found interesting:
Quote from: Pg. 2, while discussing taper geometry
The resulting design must also ensure a low taper slope, to minimise the axial component of side wall forces.
Quote from: Pg. 3
It is clear that if the minimum dimension was the cut off diameter, force F_g2 would be zero. However because there would still be a significant small end plate area, the projected area of the side wall would not equal the area of the large end plate. Thus any attempt to show a resultant zero net force due to equalisation of areas is incorrect.

He seems to be saying that the axial component of the sidewall forces could be an issue, possibly counterbalancing much of the difference of forces between the end plates (all but that proportional to the area of the small end when reduced to the cut off diameter), but I can't find anywhere else in the paper where he accounts for these forces.


Anyhow, back to direction of thrust vs. acceleration:
Quote from: Pg. 4
Note that to maintain the principle of the conservation of momentum, the acceleration of the waveguide due to thrust, is opposite to the actual thrust direction. Thus, in Fig 3, the sign convention for the waveguide velocity axis is:
                      <-----        ----->
Acceleration Vector  Thrust Vector
Quote from: Pg. 4, demonstration his usage of these terms
When the waveguide is accelerated along the acceleration vector, the thrust approaches a maximum of 1. However, as the velocity of the waveguide increases in the direction of thrust, the thrust will decrease to zero.

That's it.

He gives no explanation as to how a device which is expelling no reaction mass is in violation of conservation of momentum when it accelerates in one direction but not when accelerating in the opposite direction.

I'd have understood had he written, "To maintain the principle of conservation of momentum, forces on the sidewalls must counteract the difference of end plate forces, resulting in no net force and no acceleration."  But then there wouldn't be much to discuss.  (I'd also understand him were he to say that his device was constructed from exotic matter, had negative inertial mass, and thus accelerated in the opposite direction of the net force applied to it, but he makes no such claims.)

~Kirk, who is reminded of a "backwards rotating" Crookes radiometer (https://en.wikipedia.org/wiki/Crookes_radiometer).

Edit: Grammar
Title: Re: EM Drive Developments - related to space flight applications - Thread 2
Post by: Rodal on 05/22/2015 05:04 pm
...Base question is: Why does HDPE preform better than PTFE in high EM field experiments? (HDPE does have roughly half the thermal resistance of PTFE)....
Paul March's answer is that it has to do with the imaginary components of the electric permittivity and magnetic permeability, and the choice was based on his experience working with Prof. Woodward.

I haven't found values for the imaginary components of the magnetic permeability of HDPE or PTFE in the literature.

We can ascertain the (non-dimensionalized) imaginary values of the electric permittivity from the electric Tan Delta from here: http://www.rfcafe.com/references/electrical/dielectric-constants-strengths.htm


HDPE             0.00031 @ 3 GHz  (from the later row, not from the molded product row)
PTFE              0.00028 @ 3 GHz
Nitrile Rubber 0.034 @ 3 GHz

It shows HDPE having 11% higher electric Tan Delta than PTFE, which is not much of a difference, but at least in the right direction to produce more thrust.

It shows Nitrile Rubber as having much higher electric Tan Delta that either HDPE or PTFE, which does not agree with Paul March's experiments, as his experiments showed Nitrile Rubber used as a dielectric produced less thrust.  However Nitrile Rubber usually has mineral fillers that strongly affect its properties.  Without knowing exactly the type of Nitrile Rubber compound (including its filler content) used by Paul March it is impossible to extract conclusions.
Also, as dissipation takes place, the temperature of the polymer rises. Materials with higher Tan Delta will get hotter.  As they get hotter, the Tan Delta value changes, (just based on the WLF equation for example). So the effect of Tan Delta is very nonlinear.

Ideally, one would want to have temperature sensors inside the cavity, particularly looking at the dielectric or near it, to understand what is going on.


That reference ( http://www.rfcafe.com/references/electrical/dielectric-constants-strengths.htm ) is not very high quality, as previously discussed, but it is the first one that showed up in a Google search.

Anybody care to look for more references for Tan Delta (electric and magnetic) for HDPE and PTFE and Nitrile Rubber compounds?



Relative electric permittivity

HDPE         2.26 @ 3 GHz
PTFE          2.0 - 2.1
Neoprene   4 @ 3 GHz



Glass Transition Temperature

HDPE    - 110 C  (difficult to determine, dependent on crystallinity)
PTFE     - 115 C  (difficult to determine, dependent on crystallinity)
Neoprene -50 C



Upper Temperature

HDPE        135 C (Melting point)
PTFE         327 C (Melting point)
Neoprene  121 C (Max service temperature)



From the above properties, one concludes:

1) HDPE and PTFE are both semi-crystalline while Neoprene rubber is not.  Perhaps the superior thrust performance of HDPE and PTFE is due to their semi-crystallinity (van Tiggelen effect)

2) HDPE has slightly (~10%) higher real and imaginary components of the electric permittivity than PTFE

3) HDPE and PTFE have similar glass transition temperatures (both difficult to determine due to their crystallinity)

4) PTFE has a much higher melting temperature than HDPE. 

So HDPE may have outperformed due to any of these possibilities:

a) by chance, the extruded HDPE obtained from McMaster Carr by NASA had higher crystallinity than the PTFE (van Tiggelen effect)

b) HDPE gets hot and softens during operation of the EM Drive introducing nonlinearity of its electric permittivity which maybe benefitial to an asymmetric thrust effect. (Neoprene has a huge Tan Delta, therefore it gets much hotter than HDPE, perhaps too hot to operate effectively.)

c) HDPE may have higher imaginary component of its magnetic permeability than PTFE.  (This is completely unknown as to magnitude and relevance).
Title: Re: EM Drive Developments - related to space flight applications - Thread 2
Post by: RonM on 05/22/2015 05:14 pm
Looking at the difference in melting points between HDPE and PTFE, the "improved" performance might be due to out gassing from HDPE as it is over heated.

What is the temperature inside the device during a test run?
Title: Re: EM Drive Developments - related to space flight applications - Thread 2
Post by: TheTraveller on 05/22/2015 05:15 pm
Another "bread crumb": concerning "Patent mention of TM01 mode", there are an infinite number of TM modes having m=0, n=1, and p=1,2,3...Infinity.  None of them (for a truncated cone) have p = 0

After doing some more reading, it seems clear that Shawyer is only interested in resonance between the end plates at the frustum's effective electrical length, which depends on the integral of the constantly changing guide wavelength from one end plate to the other. Operational Rf wavelength is then some multiple of that effective wavelength. For non adjustable Rf generators, having the ability to vary the frustum length will allow operation at resonance. Here we need to fully understand, the physical end plate separation will not have a subharmonic relationship with the Rf generators wavelength.

Shawyers method to calc the frustum's effective electrical length:

Quote
You need to develop a numerical model that calculates the guide wavelength, for the chosen mode, at discrete small increments along the cavity length and then integrate them into an effective wavelength for the whole cavity.

As I want to operate in TM01 mode, as stated in Shawyer's patent and several of his publications, the big and small end cutoff wavelengths and the associated guide wavelengths (as in the quote) are different to TE mode. This  recommended method which only focuses on length resonance, as mentioned in many of the Shawyer publications, seems to be different to what NASA and other have done, are doing.

What this means is Shawyer is treating his frustum as a resonate cylinder that has variable guide wavelengths from one end to the other. His only interest in the radius or diameter is how it effects the guide wavelength and what the integral of all the constantly changing guide wavelength over the length of the cylinder is.
Title: Re: EM Drive Developments - related to space flight applications - Thread 2
Post by: Rodal on 05/22/2015 05:17 pm
Looking at the difference in melting points between HDPE and PTFE, the "improved" performance might be due to out gassing from HDPE as it is over heated.

What is the temperature inside the device during a test run?
No reported internal temperatures for NASA Eagleworks.

Externally, the hottest face was the big diameter covered with fiber-reinforced epoxy facing the outside, hence external outgassing of the epoxy is probably just as or more likely to produce thrust than internal outgassing of HDPE (more free volume in fiber-reinforced epoxy than in HDPE and the epoxy is on the outside)
Title: Re: EM Drive Developments - related to space flight applications - Thread 2
Post by: rfmwguy on 05/22/2015 05:31 pm
Looking at the difference in melting points between HDPE and PTFE, the "improved" performance might be due to out gassing from HDPE as it is over heated.

What is the temperature inside the device during a test run?
No reported internal temperatures for NASA Eagleworks.

Externally, the hottest face was the big diameter covered with fiber-reinforced epoxy facing the outside, hence external outgassing of the epoxy is probably just as or more likely to produce thrust than internal outgassing of HDPE (more free volume in fiber-reinforced epoxy than in HDPE and the epoxy is on the outside)

Thanks Doc, RonM's post was similar to my first series of posts in this thread. Our Romanian friend isn't using a dielectric (to my knowledge) and is getting some validation, so outgassing isn't the explanation for measureable thrust. I was hoping we could pinpoint an EM component of HDPE that would help to explain, but not enough test data on that. Probably NOT a rabbit hole worth digging into right now considering no dielectrics are working...perhaps down the road to amplify the thrust...
Title: Re: EM Drive Developments - related to space flight applications - Thread 2
Post by: SeeShells on 05/22/2015 05:36 pm

Well dang, here I thought we had nailed down the mystery acceleration. Alas, we still have this systemic anomaly ( I love that phrase). All along I thought we could point our fingers at the chirality of twisted polymer crystals.  http://www.esrf.eu/UsersAndScience/Publications/Highlights/2011/scm/scm4
[/quote]

Agree See, not sure we should give up on the thrust differences between HDPE and PTFE. All else being the same, why the difference? You or Doc might have some insight into the chemistry, but EM properties are what we're interested in.

"It is obvious that the CB concentration influence on the microwave properties is the strongest in the frequency range 2–3 GHz. At optimum CB concentration (17.5 mass%), the coefficient of attenuation reaches 5 dB/mm,
while the coefficient of reflection is close to the starting HDPE coefficient, i.e., the absorption
activity is more than 10 times greater than the reflection activity."
[/quote]

It's Michelle, friends call me Shell. Yes, I love mixing and matching different fields. In the real world of building things that need to be used (my company and I built semiconductor equipment) they need to incorporate virtually all the arts and while you might not have a Phd in a subject you better know enough when someone on your design team is full of hockey poo and they are 3 and 2 at the plate (US Baseball term, 3 balls and 2 strikes).
I'm reading a couple papers that are taking me into an idea of why we're seeing something unexpected in these slugs of material inserted in the cavity. Not sure if the two will dovetail into something useful or not or I'm 3 and 2 at the plate.
Thanks for the heavy reading links I'll digest it all and post later.
Thank you for your insight and your help 
Title: Re: EM Drive Developments - related to space flight applications - Thread 2
Post by: rfmwguy on 05/22/2015 06:20 pm
...I'm reading a couple papers that are taking me into an idea of why we're seeing something unexpected in these slugs of material inserted in the cavity. Not sure if the two will dovetail into something useful or not or I'm 3 and 2 at the plate.
Thanks for the heavy reading links I'll digest it all and post later.
Thank you for your insight and your help
[/quote]

Sorry 'bout the heavy reading. That was my brain cramp from yesterday, Shell. HDPE is super cheap and readily available. Its that white cutting board material you find in many stores, like this: http://www.ebay.com/itm/6-x-8-x-1-2-Thick-White-Plastic-HDPE-Cutting-Board-FDA-NSF-USDA-/281539165633 Doc said it was used in medical field as well; easy to sterilize and resistant to abrasion.
Title: Re: EM Drive Developments - related to space flight applications - Thread 2
Post by: Rodal on 05/22/2015 06:33 pm
...
It's Michelle, friends call me Shell. Yes, I love mixing and matching different fields. In the real world of building things that need to be used (my company and I built semiconductor equipment) they need to incorporate virtually all the arts and while you might not have a Phd in a subject you better know enough when someone on your design team is full of hockey poo and they are 3 and 2 at the plate (US Baseball term, 3 balls and 2 strikes).
I'm reading a couple papers that are taking me into an idea of why we're seeing something unexpected in these slugs of material inserted in the cavity. Not sure if the two will dovetail into something useful or not or I'm 3 and 2 at the plate.
Thanks for the heavy reading links I'll digest it all and post later.
Thank you for your insight and your help

...HDPE is super cheap and readily available. Its that white cutting board material you find in many stores, like this: http://www.ebay.com/itm/6-x-8-x-1-2-Thick-White-Plastic-HDPE-Cutting-Board-FDA-NSF-USDA-/281539165633 Doc said it was used in medical field as well; easy to sterilize and resistant to abrasion.

For example

BIOACTIVE CERAMIC-POLYMER COMPOSITES FOR TISSUE REPLACEMENT  Min Wang
http://edge.rit.edu/content/P09021/public/Biocompatible%20Polymers.pdf

Observe emphasis on extruded (as opposed to molded) HDPE to obtain higher crystallinity and better properties (NASA Eagleworks used extruded HDPE):

Quote from: Min Wang
The advantage that HDPE is a linear polymer with very few branches is very important. This is because when advanced polymer processing technology such as hydrostatic extrusion is used, polyethylene chains will be aligned in the extrusion direction and hence high modulus and high strength materials can be produced.
Title: Re: EM Drive Developments - related to space flight applications - Thread 2
Post by: aero on 05/22/2015 06:42 pm
Paul answered a question for me back in mid April.
Quote
The high density polyethylene discs dielectric's relative permittivity is 2.27 at 2.0 GHz with a dissipation factor of ~0.0005.
So that is what Eagleworks used.
Title: Re: EM Drive Developments - related to space flight applications - Thread 2
Post by: Rodal on 05/22/2015 07:21 pm
Another "bread crumb": concerning "Patent mention of TM01 mode", there are an infinite number of TM modes having m=0, n=1, and p=1,2,3...Infinity.  None of them (for a truncated cone) have p = 0

After doing some more reading, it seems clear that Shawyer is only interested in resonance between the end plates at the frustum's effective electrical length, which depends on the integral of the constantly changing guide wavelength from one end plate to the other. Operational Rf wavelength is then some multiple of that effective wavelength. For non adjustable Rf generators, having the ability to vary the frustum length will allow operation at resonance. Here we need to fully understand, the physical end plate separation will not have a subharmonic relationship with the Rf generators wavelength.

Shawyers method to calc the frustum's effective electrical length:

Quote
You need to develop a numerical model that calculates the guide wavelength, for the chosen mode, at discrete small increments along the cavity length and then integrate them into an effective wavelength for the whole cavity.

As I want to operate in TM01 mode, as stated in Shawyer's patent and several of his publications, the big and small end cutoff wavelengths and the associated guide wavelengths (as in the quote) are different to TE mode. This  recommended method which only focuses on length resonance, as mentioned in many of the Shawyer publications, seems to be different to what NASA and other have done, are doing.

What this means is Shawyer is treating his frustum as a resonate cylinder that has variable guide wavelengths from one end to the other. His only interest in the radius or diameter is how it effects the guide wavelength and what the integral of all the constantly changing guide wavelength over the length of the cylinder is.

Mother Nature dictates at what frequencies and mode shapes a truncated cone cavity will resonate at.

The frequencies calculated by NASA using COMSOL FEA have been within a percentage of the actual measured resonant frequencies. 

Concerning mode shapes, the prediction of NASA's COMSOL FEA and my exact solution have both been confirmed by the thermal imaging measurements by Paul March clearly showing the cylTM212 magnetic field distribution that is predicted both by exact analysis and by FEA analysis.

I have not seen any detailed mathematical description of Shawyer's method nor a comparison of his method vs. experimental results so I cannot comment on whether Shawyer's method approaches the confirmed accuracy of COMSOL FEA or the exact solution  :)



NASA COMSOL FEA Thermal Loss calculation (at left) and NASA's thermal imaging of temperature field

(http://forum.nasaspaceflight.com/index.php?action=dlattach;topic=36313.0;attach=635194;image)



NASA's COMSOL FEA calculation of Magnetic Field cylTM212

(http://forum.nasaspaceflight.com/index.php?action=dlattach;topic=36313.0;attach=635195;image)



My exact solution for Magnetic Field cylTM212
 
Title: Re: EM Drive Developments - related to space flight applications - Thread 2
Post by: deltaMass on 05/22/2015 07:39 pm
Impedance?
Title: Re: EM Drive Developments - related to space flight applications - Thread 2
Post by: rfmwguy on 05/22/2015 07:43 pm
Paul answered a question for me back in mid April.
Quote
The high density polyethylene discs dielectric's relative permittivity is 2.27 at 2.0 GHz with a dissipation factor of ~0.0005.
So that is what Eagleworks used.

Thks Aero, this looks like an extruded rod, end sliced HDPE disc: http://www.amazon.com/Density-Polyethylene-Translucent-White-Diameter/dp/B00EVCG9FS

Unfortunately the ASTM rating does not provide the permittivity or dissipation factor.
Title: Re: EM Drive Developments - related to space flight applications - Thread 2
Post by: TheTraveller on 05/22/2015 07:48 pm
I also have a vague memory of someone saying that Sawyer used a T antenna parallel to the major axis of the fulstrum.

That could be another interesting breadcrumb, which may lend support to TM01 mode excitation in the Flight Thruster.

Does anybody have any other info as to Shawyer excites his Flight Thruster?
Title: Re: EM Drive Developments - related to space flight applications - Thread 2
Post by: TheTraveller on 05/22/2015 07:59 pm
My exact solution for Magnetic Field cylTM212

That is all good, except Shawyer seems to excite his Flight Thruster as a TM01 same diameter circular cavity but with variable guide wavelength along the length axis, integrates to find the effective electrical wavelength and then uses an external Rf at a harmonic of the effective electrical cavity wavelength to gain resonance.

Did EW do that to find the wavelength they needed to drive their frustum at effective cavity wavelength?

From my calcs of the Shawyer frustum resonance procedure, the result is not what NASA or you predict, which may suggest using the external wavelength obtained as you indicated may not result in resonance with the effective guide wavelength and little or no thrust.
Title: Re: EM Drive Developments - related to space flight applications - Thread 2
Post by: rfcavity on 05/22/2015 08:12 pm
I also have a vague memory of someone saying that Sawyer used a T antenna parallel to the major axis of the fulstrum.

That could be another interesting breadcrumb, which may lend support to TM01 mode excitation in the Flight Thruster.

Does anybody have any other info as to Shawyer excites his Flight Thruster?

Dude. DUDE. The resonance modes for a spherical tapered cavity are solved, analytically. They are exact solutions. There is no doubt to any of them. Many simple cavity shapes have been solved for decades, and all experimental data backs these solutions very well. This is what Rodal is trying to tell you.

You cannot have a TM01 mode in any cavity. This is a waveguide mode. It is like saying 'I'm going to drive down the road at 100kph in my car, in the garage with the garage door closed'. It makes no sense. You can play semantic games and say it applies to infinitely long cavities, but that's just a waveguide.

The way you ignore salient posts with basic enclosed scientific facts makes you look like a VX Junky, and people won't take you very seriously after a while.
Title: Re: EM Drive Developments - related to space flight applications - Thread 2
Post by: TheTraveller on 05/22/2015 08:15 pm
Not what NASA or I predict ?  ???

what prediction are you referring to ?  ???

Thought I was clear, but seems not.

Calculating the external Rf wavelength that is needed to bring an Em Drive into end plate to end plate resonance.

As EW didn't measure any thrust in their non dielectric frustum, my guess is that they did not drive it at the correct wavelength to gain end plate to end plate resonance.

Shawyer has described how to do this. so why ignore him?

Quote
You need to develop a numerical model that calculates the guide wavelength, for the chosen mode, at discrete small increments along the cavity length and then integrate them into an effective wavelength for the whole cavity.

All SPR cavities were designed using our in-house software which can cover a range of geometries and modes.

BTW in his latest email, the dangers ot microwave radiation and high voltage are again called out:

Quote
Hi Traveller

I have no problems with you disclosing all your work, the more working EmDrives there are in the world the better.

However I would be grateful if you could emphasise the dangers of working with high power, high Q, microwave devices before someone gets seriously injured.

Thanks

Best regards

Roger
Title: Re: EM Drive Developments - related to space flight applications - Thread 2
Post by: TheTraveller on 05/22/2015 08:21 pm
I also have a vague memory of someone saying that Sawyer used a T antenna parallel to the major axis of the fulstrum.

That could be another interesting breadcrumb, which may lend support to TM01 mode excitation in the Flight Thruster.

Does anybody have any other info as to Shawyer excites his Flight Thruster?

Dude. DUDE. The resonance modes for a spherical tapered cavity are solved, analytically. They are exact solutions. There is no doubt to any of them. Many simple cavity shapes have been solved for decades, and all experimental data backs these solutions very well. This is what Rodal is trying to tell you.

You cannot have a TM01 mode in any cavity. This is a waveguide mode. It is like saying 'I'm going to drive down the road at 100kph in my car, in the garage with the garage door closed'. It makes no sense. You can play semantic games and say it applies to infinitely long cavities, but that's just a waveguide.

The way you ignore salient posts with basic enclosed scientific facts makes you look like a VX Junky, and people won't take you very seriously after a while.

I'm just the messenger telling folks here what I have read many times and what Roger Shawyer has shared with me.

Ignore him if you will but his and the Chinese EM Drives are working based on his knowledge.

BTW he treats the Em Drive conic frustum as a infinite series of open circular waveguides, each with a different diameter, that cause the guide wavelength and group velocity to vary as per that diameter. Have you read what he says?
Title: Re: EM Drive Developments - related to space flight applications - Thread 2
Post by: Clifford on 05/22/2015 08:47 pm
Haven't seen anyone post this to the thread so far.

https://hackaday.io/project/5596-em-drive

Plus interview with the team leader.

http://n-o-d-e.net/post/119343131451/building-a-diy-emdrive
Excellent set of strategies these guys are using. In parallel they're doing a Shawyer/Chinese replication attempt @2.4 GHz, and also building a 25 GHz beast with the aim of popping it into a PocketQub and sending it into space (which one of them has already done with a different project).

There might be an even cheaper way of testing an EmDrive in weightlessness than a small space probe: A drop tower like the Fallturm in Bremen, Germany:

http://en.wikipedia.org/wiki/Fallturm_Bremen

At that particular facility you can get up to 9 seconds of weightlessness and the capsule for your experiment can be a lot bigger than a PocketQub for example. It can be up to 0.8m in diameter and up to 2.4m in length. The vacuum inside the structure during the experiment is probably not of a high quality but nothing is stopping you from having an even better vacuum inside your capsule. Using a drop tower would allow for multiple tests with the same hardware with modifocations in between the shots if needed.

I just wanted to put this idea out there.
Title: Re: EM Drive Developments - related to space flight applications - Thread 2
Post by: phaseshift on 05/22/2015 08:59 pm
I also have a vague memory of someone saying that Sawyer used a T antenna parallel to the major axis of the fulstrum.

That could be another interesting breadcrumb, which may lend support to TM01 mode excitation in the Flight Thruster.

Does anybody have any other info as to Shawyer excites his Flight Thruster?

Dude. DUDE. The resonance modes for a spherical tapered cavity are solved, analytically. They are exact solutions. There is no doubt to any of them. Many simple cavity shapes have been solved for decades, and all experimental data backs these solutions very well. This is what Rodal is trying to tell you.

You cannot have a TM01 mode in any cavity. This is a waveguide mode. It is like saying 'I'm going to drive down the road at 100kph in my car, in the garage with the garage door closed'. It makes no sense. You can play semantic games and say it applies to infinitely long cavities, but that's just a waveguide.

The way you ignore salient posts with basic enclosed scientific facts makes you look like a VX Junky, and people won't take you very seriously after a while.

I'm just the messenger telling folks here what I have read many times and what Roger Shawyer has shared with me.

Ignore him if you will but his and the Chinese EM Drives are working based on his knowledge.

BTW he treats the Em Drive conic frustum as a infinite series of open circular waveguides, each with a different diameter, that cause the guide wavelength and group velocity to vary as per that diameter. Have you read what he says?

I really don't want to stick my head in the lion's mouth - but...... In Shawyer's trail of breadcrumbs I think some of them may be moldy and they need to be eaten with Kosher salt.
Title: Re: EM Drive Developments - related to space flight applications - Thread 2
Post by: rfcavity on 05/22/2015 09:03 pm
I also have a vague memory of someone saying that Sawyer used a T antenna parallel to the major axis of the fulstrum.

That could be another interesting breadcrumb, which may lend support to TM01 mode excitation in the Flight Thruster.

Does anybody have any other info as to Shawyer excites his Flight Thruster?

Dude. DUDE. The resonance modes for a spherical tapered cavity are solved, analytically. They are exact solutions. There is no doubt to any of them. Many simple cavity shapes have been solved for decades, and all experimental data backs these solutions very well. This is what Rodal is trying to tell you.

You cannot have a TM01 mode in any cavity. This is a waveguide mode. It is like saying 'I'm going to drive down the road at 100kph in my car, in the garage with the garage door closed'. It makes no sense. You can play semantic games and say it applies to infinitely long cavities, but that's just a waveguide.

The way you ignore salient posts with basic enclosed scientific facts makes you look like a VX Junky, and people won't take you very seriously after a while.

I'm just the messenger telling folks here what I have read many times and what Roger Shawyer has shared with me.

Ignore him if you will but his and the Chinese EM Drives are working based on his knowledge.

BTW he treats the Em Drive conic frustum as a infinite series of open circular waveguides, each with a different diameter, that cause the guide wavelength and group velocity to vary as per that diameter. Have you read what he says?

Those experiments, the modes were not mapped like the NASA one. So I'm not even sure what experimental evidence you're trying to show.

By the way, you don't need treat it as anything: an exact solution exists. Treating it that way doesn't make sense to me anyway, because the number one tool for solving for the modes is constraining the E and B fields to certain values at the boundary.

Please answer me this: Have you read any derivation of the exact solution for ANY cavity? (or waveguide?)
Title: Re: EM Drive Developments - related to space flight applications - Thread 2
Post by: TheTraveller on 05/22/2015 09:45 pm
Quote from: rfcavity
Please answer me this: Have you read any derivation of the exact solution for ANY cavity? (or waveguide?)

I contacted Shawyer & asked him how to calc end plate separation to obtain resonance at the desired external Rf. He provided the information / solution I have shared, which does not match exact solution numbers generated by Dr. Rodal for the same conic frustum dimensions.

Shawyers method does however give Flight Thruster resonance, at 3.85GHz, within 1mm of my calculated, from the photos, length.

BTW I calc the cutoff wavelength of a circular wave guide, at both ends, at different diameters, operating in TM01 mode, then from those values calc the guide wavelengths, and the values for 1,000 points, at each points diameter and numerically integrate them to obtain frustum electrical length.

Title: Re: EM Drive Developments - related to space flight applications - Thread 2
Post by: Rodal on 05/22/2015 09:53 pm
Quote from: rfcavity
Please answer me this: Have you read any derivation of the exact solution for ANY cavity? (or waveguide?)

I contacted Shawyer & asked him how to calc end plate separation to obtain resonance at the desired external Rf. He provided the information / solution I have shared, which does not match exact solution numbers generated by Dr. Rodal for the same conic frustum dimensions...
Precisely what "exact solution numbers generated by" me are not being matched ?

The  TM212 solution that was verified with the thermal imaging ? 

Does Shawyer have a different thermal image of the NASA fustrum ? and the thermal image camera from NASA is defective ?
Title: Re: EM Drive Developments - related to space flight applications - Thread 2
Post by: TheTraveller on 05/22/2015 10:13 pm
Quote from: rfcavity
Please answer me this: Have you read any derivation of the exact solution for ANY cavity? (or waveguide?)

I contacted Shawyer & asked him how to calc end plate separation to obtain resonance at the desired external Rf. He provided the information / solution I have shared, which does not match exact solution numbers generated by Dr. Rodal for the same conic frustum dimensions...
Precisely what "exact solution numbers generated by" me are not being matched ?

The  TM212 solution that was verified with the thermal imaging ? 

Does Shawyer have a method of solution that shows that COMSOL's Finite Element analysis program is wrong ?

Does Shawyer have a different thermal image of the NASA fustrum ? and the thermal image camera from NASA is defective ?

Shawyer & Chinese conic frustums provide thrust that matches their equations predictions.

Can you says the same thing? I thought your / Egan's predictions say no thrust?

But there is thrust. Lots of it. So your / Egan's prediction eqations have an issue.
Title: Re: EM Drive Developments - related to space flight applications - Thread 2
Post by: deltaMass on 05/22/2015 10:18 pm
@TheTraveller - you are nothing if not pragmatic :D

Quote
Shawyer & Chinese conic frustums provide thrust that matches their equations predictions.
Please cite an example set of numbers that instantiates this:
1. frustum big diameter & curvature
2. frustum small diameter & curvature
3. frustum vertical height
4. operating frequency
5. predicted thrust
6. measured thrust

Can you do this?
(no dielectric please, and preferably flat ends)
Title: Re: EM Drive Developments - related to space flight applications - Thread 2
Post by: Flyby on 05/22/2015 10:29 pm
 ???
Starting to wonder if I'm reading an updated version of Hansel and Gretel.... with all those bread crumb references ?

Seriously TheTraveler, I'd rather see you start building your setup instead of getting entangled in endless debates about R.Shawyer's merits.
With your insights and eye for detail, I'm sure your test will be much more informative then the crude (but very interesting/promising) test Iullian made.

So please man, stop digging trenches and go for what you originally planned to do : build a working model...

All this ping-pong stuff about what or what not Shawyer said/did/might have said/ could have meant...sigh... you should not take the criticism on Shawyer's texts as personal "insult"(maybe a big word).... it is all a distraction...let it go...and focus again...

You started so well... :-\
Title: Re: EM Drive Developments - related to space flight applications - Thread 2
Post by: phaseshift on 05/22/2015 10:47 pm
???
Starting to wonder if I'm reading an updated version of Hansel and Gretel.... with all those bread crumb references ?

Seriously TheTraveler, I'd rather see you start building your setup instead of getting entangled in endless debates about R.Shawyer's merits.
With your insights and eye for detail, I'm sure your test will be much more informative then the crude (but very interesting/promising) test Iullian made.

So please man, stop digging trenches and go for what you originally planned to do : build a working model...

All this ping-pong stuff about what or what not Shawyer said/did/might have said/ could have meant...sigh... you should not take the criticism on Shawyer's texts as personal "insult"(maybe a big word).... it is all a distraction...let it go...and focus again...

You started so well... :-\

TheTraveller is doing his due-dilligence to get the dimensions correct - otherwise there will be no thrust (or there will be by just dumb luck). Verification against what has been done before is all part of the process.  It's engineering and art, and I'm very impressed by what he's doing.
Title: Re: EM Drive Developments - related to space flight applications - Thread 2
Post by: Mike-F on 05/22/2015 10:48 pm

OK forgive me if this is total rubbish! But looking at the COMSOL FEA Thermal Loss  diagram and the thermal camera image which verifies the calculated thermal losses. It strikes  me that this shows that internal energy is being converted to heat at the large end but not the side walls or small end. Could it be that in these areas the RF energy is being dissipated in another way i.e. generating thrust? So could lower  heat dissipation in these areas indicate energy transfer to something else unseen?

Hope that makes sense!

Mike.
Title: Re: EM Drive Developments - related to space flight applications - Thread 2
Post by: Lobo on 05/22/2015 11:06 pm
Haven't seen anyone post this to the thread so far.

https://hackaday.io/project/5596-em-drive

Plus interview with the team leader.

http://n-o-d-e.net/post/119343131451/building-a-diy-emdrive
Excellent set of strategies these guys are using. In parallel they're doing a Shawyer/Chinese replication attempt @2.4 GHz, and also building a 25 GHz beast with the aim of popping it into a PocketQub and sending it into space (which one of them has already done with a different project).

There might be an even cheaper way of testing an EmDrive in weightlessness than a small space probe: A drop tower like the Fallturm in Bremen, Germany:

http://en.wikipedia.org/wiki/Fallturm_Bremen

At that particular facility you can get up to 9 seconds of weightlessness and the capsule for your experiment can be a lot bigger than a PocketQub for example. It can be up to 0.8m in diameter and up to 2.4m in length. The vacuum inside the structure during the experiment is probably not of a high quality but nothing is stopping you from having an even better vacuum inside your capsule. Using a drop tower would allow for multiple tests with the same hardware with modifocations in between the shots if needed.

I just wanted to put this idea out there.

Or something like this:

Quote
United States
Peter Diamandis of Zero Gravity Corporation

In late 2004, the Zero Gravity Corporation became the first company in the United States to offer zero-g flights to the general public, using Boeing 727 jets. Each flight consists of around 15 parabolas, including simulations of the gravity levels of the Moon and Mars, as well as complete weightlessness.[19] This profile allows ZERO-G's clients to enjoy weightlessness with minimal motion discomfort.

In 2014, Integrated Spaceflight Services, the Research and Education partner of Swiss Space Systems (S3) in America, began its offering of comprehensive reduced gravity services on S3's Airbus A340 aircraft, as well as FAA certification of science and engineering payloads.

Aurora Aerospace in Oldsmar, Florida offers zero-g flights using a Fuji/Rockwell Commander 700. It is also used to simulate the gravity of the Moon and Mars.[21]
Title: Re: EM Drive Developments - related to space flight applications - Thread 2
Post by: Rodal on 05/22/2015 11:08 pm

OK forgive me if this is total rubbish! But looking at the COMSOL FEA Thermal Loss  diagram and the thermal camera image which verifies the calculated thermal losses. It strikes  me that this shows that internal energy is being converted to heat at the large end but not the side walls or small end. Could it be that in these areas the RF energy is being dissipated in another way i.e. generating thrust? So could lower  heat dissipation in these areas indicate energy transfer to something else unseen?

Hope that makes sense!

Mike.
1) The COMSOL FEA is only solving Maxwell's differential equations to obtain the electromagnetic fields.  The COMSOL FEA thermal loss analysis uses the results of the magnetic field calculations.

2) Heating of the big diameter end is entirely due to induction heating due to the transverse magnetic field producing eddy currents on the inner copper surface of the big end.

3) The small end is internally isolated from the magnetic field by a relatively thick HDPE polymer dielectric and the magnetic field in mode TM212 has higher intensity near the big end. 

4) The lower amount of heating of the walls is due to the direction and magnitude of the magnetic field in mode TM212 (please look at the vector plot at the bottom of my prior post).

5) Hence the heating of the walls and the ends of the truncated cone are entirely explainable by classical physics as embodied in the COMSOL FEA, and nothing exotic needs to be involved to explain the thermal losses.

6) Although you didn't ask this, but apparently others are confused, COMSOL FEA and the exact solution successfully predict the experimentally verified natural frequencies.  The mode shape for mode TM212 is also verified.  If there is any thrust from the EM Drive, such thrust is not affecting the natural frequencies and the mode shapes predicted by classical physics.  If there is thrust such thrust involves a process which is esentially uncoupled from equations governing the natural frequency and mode shapes of the cavity.
Title: Re: EM Drive Developments - related to space flight applications - Thread 2
Post by: PaulF on 05/22/2015 11:16 pm

OK forgive me if this is total rubbish! But looking at the COMSOL FEA Thermal Loss  diagram and the thermal camera image which verifies the calculated thermal losses. It strikes  me that this shows that internal energy is being converted to heat at the large end but not the side walls or small end. Could it be that in these areas the RF energy is being dissipated in another way i.e. generating thrust? So could lower  heat dissipation in these areas indicate energy transfer to something else unseen?

Hope that makes sense!

Mike.
You mean that the IR photons also contribute to the thrust? I believe that was already debunked, but I am just a  layman.
To my knowledge, the total amount of thrust possible from the microwaves + heat dissipation IR photons is on the order of three magnitudes smaller than the measured thrust.
Title: Re: EM Drive Developments - related to space flight applications - Thread 2
Post by: Blaine on 05/22/2015 11:33 pm
I'm beginning to the think the thrust is attributed to a polarizable vacuum as extreme as that may sound.  Whatever is happening is not just because of the microwaves or Maxwell's equations for the magnetic fields.  It is something else.  I think its more like pulling than it is thrust/pushing.
Title: Re: EM Drive Developments - related to space flight applications - Thread 2
Post by: frobnicat on 05/22/2015 11:42 pm
IULIAN:

Hi,
.......snip
Iulian

Another thing to consider; From your video you have the unit on the end of a looped spring hanging from a shelf....  For the Downwards test you are trying to force the unit downwards AGAINST the natural TENSION of the spring...  you need to measure just how much energy it takes to pull the spring down as much as the unit did when you powered it up.!

The original "thrust was with the aid of the spring pulling the unit upwards. 

Placing the complete unit onto a balance board "like a child's see-saw,  American teeter-totter"  with an equal weight on the other end will enable you to perform these types of measurements

Before this get buried...

I agree teeter-totter setup would be better (to get rid of lifting spring, that likely introduces more spurious vibrations), but your comment on going against or with the tension of supporting spring as an explanation for a different apparent force magnitude (upward vs downward) is misleading. It takes no more nor less energy (or force magnitude) to deviate a spring one direction or the other from a rest position, even if there is a constant force bias (in present case, a weight). This is all linear (Hooke's law). Moreover the electronic scale that supports what part of the weight (plus or minus thrust) is left from the up-pulling of the spring, has transducer much stiffer than the spring, meaning that the actual displacement (that makes a force reading possible) is negligible when compared to rest length of the spring : the force exerted by the spring can be considered constant to a good approximation (I would bet well within 1%). Considering the overall setup, delta weight readings on the scale (plus or minus) are just proportional to whatever thrust or force is applied on the lever, with no distinction between upward and downward (same constant of proportionality).
 
That is, provided the upward force is not enough to make the lever leave contact from the electronic scale, which would limit upward readings, not downward, and would hardly go unnoticed. Paul March also mentioned the possibility of an asymmetric force/displacement relation (in their "horizontal" balance) to explain the manifest discrepancy of magnitudes when reversing the frustum 180° : short of support leaving contact (backlash or loose fixation) or dry friction (sliding) I fail to see what could make for such direction dependant non-linearity...

Looks like those frustums don't like being nudged around 180°, and thinking too much about reactionless physics makes people loose basic mechanical common sense. Small displacements => usually linear, otherwise is bad news for a measuring setup (unless carefully designed and understood non-linearity).

Now that I clarified that point on a ground I know, here is a more shaky idea I'd like to see either investigated or debunked by people with more intuition (than me) about aerodynamics : could it be that the convection likely occurring on the outside of the frustum, when small end down (Iulian reversal), would do some kind of Venturi effect that could be "sucking" the conical surface downward ? Also the rising stream of heated air would have a lifting effect from viscosity, maybe of higher magnitude that the hypothetical downward effect I'm contemplating... (this is separate from the previously discussed "hot air balloon" and "warm air jet" effects). Playing with dynamic pressures can be tricky... this would need only fraction of a Pa on the outside of the cone.
Title: Re: EM Drive Developments - related to space flight applications - Thread 2
Post by: aero on 05/22/2015 11:44 pm
Warptech

Quote
PP195:
Quote from: arc on Today at 05:16 AM
        Warptech
        If the thruster had 2 compartments, a cylinder, where resonance was easy to establish at high Q, and a long frustum designed for maximum attenuation connected at one end of the cylinder. Between the two, there is a "shutter" that can rapidly open and close. When closed, the cylinder resonates as a cylinder. When open, energy expands into the frustum chamber where it is attenuated. After the shutter closes again, the energy in frustum attenuates and energy in cylinder recharges.... repeat. I keep looking for ways to decouple the resonant amplifier from the attenuator.

Quote
    Firstly I need to ask what sort of timeframes you are looking at for connecting/ disconnecting cycle, 
micro_sec, milli_sec, seconds?.
    How long do you think the coupling will need to be in place to create resonance inside the thruster cavity...{or are you thinking the resonance is not even needed in that chamber at all, just force fed from the attached cylinder in burst mode}.  I think I see where you are going with this but more info may help clarify the desired method, and help refine a model im working on for mechanical distribution of em waves.

Quote

I don't like posting my equations until I know they're right, but I believe I have shown that the force;

F ~ (alpha) * d(alpha)/dx

Where alpha is the attenuation "variable" of the waveguide. Alpha is larger for a small half-angle taper, so a long tapered pipe like a flagpole should be used for the attenuator. Alpha is variable in a frustum, it is not the same in both directions.

Also, the TC of alpha is Np/m, and it has very little effect over 1/2 a wavelength. Therefore, my thinking is that the resonant amplifier should be just a short cylinder to build up a high Q*P, then release that energy into a very long frustum pipe where all the momentum can be absorbed in the forward direction. Resonance is not needed, we want it to decay quickly, because faster decay is higher dp/dt = Force.

In reply to deltaMass, if you are only considering "reflection" then p = 0. But what happens when a wave is attenuated in a perfectly conducting circular waveguide? That energy is not lost as "heat" because there is no resistance to dissipate it.
Todd

Ok
If Im following your thought train correctly then, from my limited perspective on handling microwaves.

The generation side of the circuit requires cyclic refreshing to achieve resonance. {Unknown time-element at the moment}

Once resonance is achieved, {and not before} the energy is dumped into the "Load".

The operational functionality requires simplicity of operation.

The functionality requires the ability to alter running characteristics in realtime.

The length of the initial resonance chamber {ideally} needs to be automatically configurable {hence shawyers use of piezoelectric actuators inside his system}

The mechanical nature of the beam chopper requires simple operation but also an ability to tune in relation to time taken to achieve resonance compared with port-opening cycles. {potentially a software function to find the maximum thrust by adjusting resonance/port operation timing automatically, Microcontrollers are good for this, I use Arduino}

Thinking about the end product it may be easier engineering wise to have multiple attenuation chambers. This allows simplicity in design and beam chopper operation.

2 attenuators at 180, single beam splitter opening with weight adjusted disk to account for mass removed on one side.{less efficient model}

4 attenuation chambers equally spaced so any two are logically 180 degrees from each other, Both can be fed from the resonant chamber simultaneously via 2 opposed holes in the beam splitter. This also allows for slower rotation rate of the splitter as it has 2 holes not 1.

A variable speed rotary port opening  mechanism. The shape of the beam choppers pass-through-port determines the efficiency of the opening process, square, circle, ellipse, triangular

For shorter port opening times use 2 disks counter rotating with respect to each other. {or just smaller port openings}.

Personally I tend to favour 4 attenuators because 2 will be simultaneously active while the resonator recharges to pump the other 2 attenuators. {this is also because I have no idea how long it will take to attenuate the signals in relation to achieving resonance}

? any use or.. just junk?
Terrible drawing but you get the basic concept...

(http://infinitas.co/r.d.d/project/framework/docs/em-quantum/resonant-attenuator2.png)
Could you perhaps use something like this? (attached)
Although the geometry is hard to see, there exists a 10% of radius gap between the two sections. The resonant cavity shows black because of the high energy level. The cavity resonates as long as the gap is less than about 25% of radius. With the gap closed, call the quality factor 100. It only drops to about 90 before falling out of resonance. While the gap size is increasing and the Q is falling, the resonant frequency is changing, but not in a predictable way. That could be a problem with -- well several things, take your pick.

I added another image without all the RF energy bouncing around. it's easier to see - I hope.
Title: Re: EM Drive Developments - related to space flight applications - Thread 2
Post by: SeeShells on 05/22/2015 11:52 pm

If there is any thrust from the EM Drive, such thrust is not affecting the natural frequencies and the mode shapes predicted by classical physics.  If there is thrust such thrust involves a process which is essentially uncoupled from equations governing the natural frequency and mode shapes of the cavity.
Just got on for a little, have a party to go to.
But first, big kudos to you Dr. Rodal, big kudos! This is the premise I've been fired up about. There is thrust with no adverse change in mode shape,  thermal, dang, there is nothing that shows in the COMSOL or real life tests, but it's there.  Pull the plug (so to speak) out and the thrust stops. Did I get that right, is this what I've been seeing it the data from several weeks ago that you sent me on this site?

Got to go but I'm going with a good feeling.

Shell
Title: Re: EM Drive Developments - related to space flight applications - Thread 2
Post by: Rodal on 05/23/2015 12:21 am
...
Just got on for a little, have a party to go to.
But first, big kudos to you Dr. Rodal, big kudos! This is the premise I've been fired up about. There is thrust with no adverse change in mode shape,  thermal, dang, there is nothing that shows in the COMSOL or real life tests, but it's there.  Pull the plug (so to speak) out and the thrust stops. Did I get that right, is this what I've been seeing it the data from several weeks ago that you sent me on this site?

Got to go but I'm going with a good feeling.

Shell
Shell,

You got it  :)
Yes, if the thrust is real (and the avalanche of replications like Iulian's make it feel more and more real) it looks to be produced by an uncoupled process.

Uncoupled processes are not uncommon, as you know actually more physical problems involve uncoupled physics or negligible amounts of coupling.  Strongly coupled processes are more unusual.  For example, most heat-transfer  effects on structures are essentially uncoupled: thermal expansion, thermal stress, etc.  The coupling in the equations of thermoelasticity is usually negligible.  One can solve Fourier's equations separately, figure out the temperature distribution and from the temperature distribution calculate a thermal stress analysis.  No coupling (with the exception of very thin shells, etc.).

Notsosureofit's formula is an uncoupled formula (notsosureofit please correct me if I'm wrong).  The thrust force is dependent on the mode shapes.  One can first calculate the mode shapes based on standard Maxwell's equations, and from them calculate the thrust force.  I suppose that if the theory matures one can then refine it and explore different types of coupling and nonlinearities like in every theory (publish of perish  :) ) but the main effect, to first order appears uncoupled, based on the experimental frequency and mode shape data.



Now to more salient, on-topic matters.

I hope you had fun in your party.  I have been to a few during the last couple of weeks and whenever I have broached the subject of the EM Drive, while seeping my vodka martini, (even with people I thought would be interested in) I invariably get big stares, faces like "you must be kidding" and the like and I have to embarrassingly switch the topic to something more socially acceptable like other people's private lives, Brangelina, the stock market or the weather :) 
Title: Re: EM Drive Developments - related to space flight applications - Thread 2
Post by: PaulF on 05/23/2015 12:25 am
I'm beginning to the think the thrust is attributed to a polarizable vacuum as extreme as that may sound.  Whatever is happening is not just because of the microwaves or Maxwell's equations for the magnetic fields.  It is something else.  I think its more like pulling than it is thrust/pushing.
I was thinking more in the lines of this: Give the QV a punch (High energy density(flux), which is what sucks up most of the power, being converted into heat. The QV seems not to absorb any energy from this process) and it will react in a way I would like to compare with the rock in a pond example, and that we can hitch a ride on the wave crests. The energy consumption of throwing the rock is also much more than the actual kinetic energy we receive by riding the wave. By tuning the cavity and the frequency we can affect the wave crests and how much energy they carry. Of course the cavity contains standing waves, but the analogy is very good. I would like to think of the QV as an extradimensional or intrainterdimensional singularity (itself being 0-dimensional) from whence all that exists in the universe came from. But like a rocket, it didn't belch out it's entire contents, to prevent problems. Being of 0 dimensions, that also fits into relativity because the universe does not have a middle. QV is nowhere and everywhere underlying the spacetime fabric at any given moment and place, and extrapolating further I'd say that solves the absolute frame reference problem. Plus it does in a logical sense explain why i.e. each and every electron has the exact same rest mass. Because QV is the same QV in every planck volume of spacetime.

The other idea (Warp field) is also very compelling to me, although I am not a scientist and my understanding is limited.
Title: Re: EM Drive Developments - related to space flight applications - Thread 2
Post by: deltaMass on 05/23/2015 12:52 am
I'm more of a fan of The Cosmic Badger theory. If there are systems which have excesses that violate conservation, The Cosmic Badger eats the excesses, which disappear, and The Cosmic Badger has made it all better.
Title: Re: EM Drive Developments - related to space flight applications - Thread 2
Post by: Notsosureofit on 05/23/2015 01:41 am


You got it  :)
Yes, if the thrust is real (and the avalanche of replications like Iulian's make it feel more and more real) it looks to be produced by an uncoupled process.

Uncoupled processes are not uncommon, as you know actually more physical problems involve uncoupled physics or negligible amounts of coupling.  Strongly coupled processes are more unusual.  For example, most heat-transfer  effects on structures are essentially uncoupled: thermal expansion, thermal stress, etc.  The coupling in the equations of thermoelasticity is usually negligible.  One can solve Fourier's equations separately, figure out the temperature distribution and from the temperature distribution calculate a thermal stress analysis.  No coupling (with the exception of very thin shells, etc.).

Notsosureofit's formula is an uncoupled formula (notsosureofit please correct me if I'm wrong).  The thrust force is dependent on the mode shapes.  One can first calculate the mode shapes based on standard Maxwell's equations, and from them calculate the thrust force.  I suppose that if the theory matures one can then refine it and explore different types of coupling and nonlinearities like in every theory (publish of perish  :) ) but the main effect, to first order appears uncoupled, based on the experimental frequency and mode shape data.



For comparison (stronger nonlinearity) here's an acoustic case:

http://www.zainea.com/lowresonances.htm
Title: Re: EM Drive Developments - related to space flight applications - Thread 2
Post by: rfmwguy on 05/23/2015 02:40 am
For comparison (stronger nonlinearity) here's an acoustic case:

http://www.zainea.com/lowresonances.htm
[/quote]

Interesting charts...acoustic waves almost appear to "break" in the frequency domain, leading me to visualize ocean waves: "Breaking Waves -As the wave moves into increasingly shallow water, the bottom of the wave decreases speed. There comes a point where the top of the wave overtakes it and starts to spill forward — the wave starts to break. We're surfing! In general a wave will start to break when it reaches a water depth of 1.3 times the wave height"

http://www.surfing-waves.com/waves/how_waves_break.htm

A breaking ocean wave spills forward along the x axis, transfering amplitude energy (y axis) as the seafloor becomes shallow.

ok, em waves traveling towards the small diameter of the frustum are not ocean waves that "break", sending energy "forward" resulting in a push/thrust. If someone does prove this, however, they can name it after me  ::)
Title: Re: EM Drive Developments - related to space flight applications - Thread 2
Post by: zen-in on 05/23/2015 03:57 am
...
...

If the air stream is going downwards, then everything is vice-versa from what was described above.


(http://cdn4.explainthatstuff.com/laminar-turbulent-flow-wind-tunnel.jpg)

Don't be surprised if tomorrow I read this again and I tell to myself "did I write that" ?

That car could use a spoiler on the back like the one I had on my '86 Mustang GT.   It tracked really well when I got above 120 MPH.
Title: Re: EM Drive Developments - related to space flight applications - Thread 2
Post by: aero on 05/23/2015 03:57 am
Paul answered a question for me back in mid April.
Quote
The high density polyethylene discs dielectric's relative permittivity is 2.27 at 2.0 GHz with a dissipation factor of ~0.0005.
So that is what Eagleworks used.

Thks Aero, this looks like an extruded rod, end sliced HDPE disc: http://www.amazon.com/Density-Polyethylene-Translucent-White-Diameter/dp/B00EVCG9FS

Unfortunately the ASTM rating does not provide the permittivity or dissipation factor.

Well, dissipation factor has a lot of names. I think I recall that I found 3 or 4 different ones that it either equalled or was the inverse of or 1 minus it. Maybe the "loss tangent" is documented. Dissipation factor ~= loss tangent for small factors, and I think 0.0005 is small enough to use that approximation.
Title: Re: EM Drive Developments - related to space flight applications - Thread 2
Post by: WarpTech on 05/23/2015 04:16 am
Impedance?

Impedances of a tapered waveguide are given in Zeng and Fan.
Title: Re: EM Drive Developments - related to space flight applications - Thread 2
Post by: WarpTech on 05/23/2015 04:40 am
I also have a vague memory of someone saying that Sawyer used a T antenna parallel to the major axis of the fulstrum.

That could be another interesting breadcrumb, which may lend support to TM01 mode excitation in the Flight Thruster.

Does anybody have any other info as to Shawyer excites his Flight Thruster?

Dude. DUDE. The resonance modes for a spherical tapered cavity are solved, analytically. They are exact solutions. There is no doubt to any of them. Many simple cavity shapes have been solved for decades, and all experimental data backs these solutions very well. This is what Rodal is trying to tell you.

You cannot have a TM01 mode in any cavity. This is a waveguide mode. It is like saying 'I'm going to drive down the road at 100kph in my car, in the garage with the garage door closed'. It makes no sense. You can play semantic games and say it applies to infinitely long cavities, but that's just a waveguide.

The way you ignore salient posts with basic enclosed scientific facts makes you look like a VX Junky, and people won't take you very seriously after a while.

I'm just the messenger telling folks here what I have read many times and what Roger Shawyer has shared with me.

Ignore him if you will but his and the Chinese EM Drives are working based on his knowledge.

BTW he treats the Em Drive conic frustum as a infinite series of open circular waveguides, each with a different diameter, that cause the guide wavelength and group velocity to vary as per that diameter. Have you read what he says?

Regarding the TM01x modes. Why couldn't you drive them using a simple 1/4-wave stub antenna at the TM011 wavelength, based at the small end or the apex of a cone? The pattern of magnetic field is concentric around the central axis, just like a wire. The resonant modes, p are the same as on a wire. Regardless of which direction current is flowing in the wire, the Lorentz force on the opposing currents flowing in the frustum walls will always have a component in the forward direction, while the magnetic field at the location of the antenna wire will always be zero, according to the mode shapes.

I thought it was an interesting idea...

Todd
Title: Re: EM Drive Developments - related to space flight applications - Thread 2
Post by: deltaMass on 05/23/2015 04:58 am
Impedance?
Impedances of a tapered waveguide are given in Zeng and Fan.
But this is a frustum. I wondered if Roval with his super mode program could trace the locus of 50+j0 ohm feed points?
Title: Re: EM Drive Developments - related to space flight applications - Thread 2
Post by: TheTraveller on 05/23/2015 08:09 am
@TheTraveller - you are nothing if not pragmatic :D

Quote
Shawyer & Chinese conic frustums provide thrust that matches their equations predictions.
Please cite an example set of numbers that instantiates this:
1. frustum big diameter & curvature
2. frustum small diameter & curvature
3. frustum vertical height
4. operating frequency
5. predicted thrust
6. measured thrust

Can you do this?
(no dielectric please, and preferably flat ends)

Before I came on board, no one understood the Df equation, nor guide wavelength nor cutoff wavelength as used by Shawyer. It was all about classic physics and the tools derived from them showing NO THRUST will be generated. The pathway seemed to be:

If there is thrust then as our tools say no thrust, there must be new physics involved, despite Shawyer and the Chinese saying there is thrust and it can be predicted by a non classical use of the existing physics.

Instead of trying to understand what Shawyer and the Chinese are saying about how they successfully use existing physics to predict the thrust, the forum goes off to left field and dreams up ways to not need to try to understand what Shawyer and the Chinese are saying.

Which leave me, who wishes to build a Flight Thruster and have an excel spreadsheet that predicts thrust as the dimensions and frequency are varied in right field, far away from most on this thread who have chosen to play in left field and think what I'm doing is, well a sign of madness, as I will not Hi Five the desire to play in left field and ignore what Shawyer and the Chinese have shared.

Bottom line is both Shawyer and the Chinese have measured thrust on dozens of devices and that thrust is predictable from their equations. Yet those equations are totally ignored. So who is mad? Those who ignore what Shawyer and the Chinese have shared or those who ignore their results and the predictive equation they share?

For me, the pathway is to continue to talk with Shawyer, as he gives me a pat on the head and offers another bread crumb and to continue to put together a spreadsheet that allows me to calc Df and frustum length as per Shawyers shared info and equations so as to get optimal thrust at my desired operational wavelength.

With the greatest respect to others on the forum, why should I engage with equations that predict no thrust as they give me NO feedback to produce an excel spreadsheet to model what I'm attempting to do.

It is ALL about the THRUST (non dielectric) and how to model it so as I change big end and small end diameters, the Df calcs properly and from that, the length calcs properly as per the internal effective wavelength.
Title: Re: EM Drive Developments - related to space flight applications - Thread 2
Post by: TheTraveller on 05/23/2015 08:38 am
???
Starting to wonder if I'm reading an updated version of Hansel and Gretel.... with all those bread crumb references ?

Seriously TheTraveler, I'd rather see you start building your setup instead of getting entangled in endless debates about R.Shawyer's merits.
With your insights and eye for detail, I'm sure your test will be much more informative then the crude (but very interesting/promising) test Iullian made.

So please man, stop digging trenches and go for what you originally planned to do : build a working model...

All this ping-pong stuff about what or what not Shawyer said/did/might have said/ could have meant...sigh... you should not take the criticism on Shawyer's texts as personal "insult"(maybe a big word).... it is all a distraction...let it go...and focus again...

You started so well... :-\

TheTraveller is doing his due-dilligence to get the dimensions correct - otherwise there will be no thrust (or there will be by just dumb luck). Verification against what has been done before is all part of the process.  It's engineering and art, and I'm very impressed by what he's doing.

Comments appreciated.

Another bread crumb is Shawyer's mention in this EM Drive patent and in several of his papers that he excited the frustum at TM01. Originally not knowing what that meant, I googled it and found:

1) TM = the frustum is excited in a way to obtain, inside the frustum, a transverse magnetic field and a electric field that propagates from end plat to end plate. If you excite at the axial centre of the frustum, the e field is centered in the middle of the frustum.

2) 01 = there is a 1/2 wavelength that resonates between the 2 end plates.

I really don't think that going deeper is needed.
Title: Re: EM Drive Developments - related to space flight applications - Thread 2
Post by: Left Field on 05/23/2015 09:48 am
@TheTraveller - you are nothing if not pragmatic :D

Quote
Shawyer & Chinese conic frustums provide thrust that matches their equations predictions.
Please cite an example set of numbers that instantiates this:
1. frustum big diameter & curvature
2. frustum small diameter & curvature
3. frustum vertical height
4. operating frequency
5. predicted thrust
6. measured thrust

Can you do this?
(no dielectric please, and preferably flat ends)

Before I came on board, no one understood the Df equation, nor guide wavelength nor cutoff wavelength as used by Shawyer. It was all about classic physics and the tools derived from them showing NO THRUST will be generated. The pathway seemed to be:

If there is thrust then as our tools say no thrust, there must be new physics involved, despite Shawyer and the Chinese saying there is thrust and it can be predicted by a non classical use of the existing physics.

Instead of trying to understand what Shawyer and the Chinese are saying about how they successfully use existing physics to predict the thrust, the forum goes off to left field and dreams up ways to not need to try to understand what Shawyer and the Chinese are saying.

Which leave me, who wishes to build a Flight Thruster and have an excel spreadsheet that predicts thrust as the dimensions and frequency are varied in right field, far away from most on this thread who have chosen to play in left field and think what I'm doing is, well a sign of madness, as I will not Hi Five the desire to play in left field and ignore what Shawyer and the Chinese have shared.

Bottom line is both Shawyer and the Chinese have measured thrust on dozens of devices and that thrust is predictable from their equations. Yet those equations are totally ignored. So who is mad? Those who ignore what Shawyer and the Chinese have shared or those who ignore their results and the predictive equation they share?

For me, the pathway is to continue to talk with Shawyer, as he gives me a pat on the head and offers another bread crumb and to continue to put together a spreadsheet that allows me to calc Df and frustum length as per Shawyers shared info and equations so as to get optimal thrust at my desired operational wavelength.

With the greatest respect to others on the forum, why should I engage with equations that predict no thrust as they give me NO feedback to produce an excel spreadsheet to model what I'm attempting to do.

It is ALL about the THRUST (non dielectric) and how to model it so as I change big end and small end diameters, the Df calcs properly and from that, the length calcs properly as per the internal effective wavelength.
You need to reverse the polarity captain! Unconventional thinking is definitely left field - hence my monicker.
Title: Re: EM Drive Developments - related to space flight applications - Thread 2
Post by: Chris Bergin on 05/23/2015 10:57 am
Right, that's over a million views for this thread. Let's consider a new, fresh, centralized thread, lead by a post to the relevant links here and with some ground rules.

That'll provide focus (long threads tend to lose that), updates (if and when they arrive) and discussion (which is the meat on the bones of any thread) - moderated to keep any silly posts out of the debate (this site's forum will always put signal before noise). It'll also avoid the potential of lots of threads on this, which is where the conversation becomes diluted and threatens to take over a section that - which advanced - wasn't really set up to cover this subject of EM Drive (ironic, given it became a subject resulting in 1.5 million views on its related threads and a 600,000 read article).

Also, I've had two e-mails worried about folk getting overexcited about setting up their own experiments (which can be hazardous). This subject is being debated on several sites, but we can at least show some responsibility with the guidance to people visiting this site.

So, Dr. Rodal - I'll PM you later about setting this new thread up, but this post is to give a heads up to all that we're going to be efforting this over the coming days.
Title: Re: EM Drive Developments - related to space flight applications - Thread 2
Post by: SeeShells on 05/23/2015 11:30 am


You got it  :)
Yes, if the thrust is real (and the avalanche of replications like Iulian's make it feel more and more real) it looks to be produced by an uncoupled process.

Uncoupled processes are not uncommon, as you know actually more physical problems involve uncoupled physics or negligible amounts of coupling.  Strongly coupled processes are more unusual.  For example, most heat-transfer  effects on structures are essentially uncoupled: thermal expansion, thermal stress, etc.  The coupling in the equations of thermoelasticity is usually negligible.  One can solve Fourier's equations separately, figure out the temperature distribution and from the temperature distribution calculate a thermal stress analysis.  No coupling (with the exception of very thin shells, etc.).

Notsosureofit's formula is an uncoupled formula (notsosureofit please correct me if I'm wrong).  The thrust force is dependent on the mode shapes.  One can first calculate the mode shapes based on standard Maxwell's equations, and from them calculate the thrust force.  I suppose that if the theory matures one can then refine it and explore different types of coupling and nonlinearities like in every theory (publish of perish  :) ) but the main effect, to first order appears uncoupled, based on the experimental frequency and mode shape data.



For comparison (stronger nonlinearity) here's an acoustic case:

http://www.zainea.com/lowresonances.htm
So many things are connected. I just finished scanning this interesting paper, not a new unknown effect (pretty pictures  8) ) and some correlations to what we're doing in harmonics and the effects they can have in a localized environment.
http://xlab.me.berkeley.edu/pdf/245.pdf
I have some reading to do and some coffee to wash it all down with.
Title: Re: EM Drive Developments - related to space flight applications - Thread 2
Post by: Notsosureofit on 05/23/2015 12:27 pm

So many things are connected. I just finished scanning this interesting paper, not a new unknown effect (pretty pictures  8) ) and some correlations to what we're doing in harmonics and the effects they can have in a localized environment.
http://xlab.me.berkeley.edu/pdf/245.pdf
I have some reading to do and some coffee to wash it all down with.

Great find !  Yes, that's an acoustic analog.

"As pointed out in previous studies, the self-bending beam represents an
envelope of rays and there exists a singular gradient mapping between the ray
surface (wavefront) and the self-bending beams (caustics)23. As shown in Fig. 6, the
geometric representation of such a condition is that all the rays forming a caustic
are tangents of the caustic trajectory. As the geometric wavefront is always
perpendicular to the rays, one can construct a unique wavefront family from the
tangential rays of the caustic trajectory. Once the wavefront is determined, the
phase map on a planar source plane can be generated by tracing each point on the
wavefront to the source plane along the ray and calculating the phase retardation28.
To formulate our recipe, let us consider a phased array located in x–z plane
emitting acoustic rays according to a certain phase profile j(x), as illustrated in
Fig. 6. During the propagation along the positive z axis, the rays construct a desired
self-bending wave packet at x ¼ f(z), where the single-valued function f depicts an
arbitrary convex curve. Next we shall use Legendre transformations to construct
the geometric wavefront from a preset beam trajectory."

  In the EM tapered cavity case the phase map (3D this time) is the dispersion profile of the cavity interior.  To get the expected curvature, we must imagine the cavity structure to be massless.  The force arises as the reaction to the (ordinary) inertial mass of the real cavity structure.  That's it in a nutshell.

  To further the analogy, each point in the cavity volume can be considered as a "loudspeaker" w/ a determined amplitude and phase.  (hope that helps to visualize)

As an aside, they only need use the first of the three acoustic wave equations here as the viscous and thermal transport coefficients are small in this case.
Title: Re: EM Drive Developments - related to space flight applications - Thread 2
Post by: StrongGR on 05/23/2015 02:11 pm
...

By contrast, in a closed cavity, there is a reflection at the walls, there is nothing coming out (unless we consider quantum tunneling, or heat dissipation like radiative heat transfer in a vacuum or convective heat transfer in air)

Or gravity maybe?
Title: Re: EM Drive Developments - related to space flight applications - Thread 2
Post by: Notsosureofit on 05/23/2015 02:21 pm
  They are making an acoustic hologram.

Great references.
Title: Re: EM Drive Developments - related to space flight applications - Thread 2
Post by: Notsosureofit on 05/23/2015 02:40 pm
  They are making an acoustic hologram.
Yes, but the topology  seems to be different: the correct hologram should be of a completely enclosed cavity, while the hologram in the paper has two open holes: an entry hole and an exit hole that are not present in the EM Drive (unless one invokes radiation heat transfer, convective heat transfer, perhaps general relativity, etc.)

The topology is different.  The wave packet in the EM case has the shape and phase distribution set by the cavity.  If the cavity walls disappeared the trajectory of the wave packet would curve in 4-space (accelerate).  It can't do that because the cavity is still there and has much more mass-equivalent than the wavepacket, so all you see is the reaction force.
Title: Re: EM Drive Developments - related to space flight applications - Thread 2
Post by: Rodal on 05/23/2015 02:43 pm
  They are making an acoustic hologram.
Yes, but the topology  seems to be different: the correct hologram should be of a completely enclosed cavity, while the hologram in the paper has two open holes: an entry hole and an exit hole that are not present in the EM Drive (unless one invokes radiation heat transfer, convective heat transfer, perhaps general relativity, etc.)

The topology is different.  The wave packet in the EM case has the shape and phase distribution set by the cavity.  If the cavity walls disappeared the trajectory of the wave packet would curve.  It can't do that because the cavity is still there and has much more mass-equivalant than the wavepacket, so all you see is the reaction force.
OK, I would have to work out the math to convince myself that the "walls dissapeared".  If they dissapeared we are in agreement. But to get there I need a proof, as you said  :)
Title: Re: EM Drive Developments - related to space flight applications - Thread 2
Post by: Notsosureofit on 05/23/2015 02:50 pm
  They are making an acoustic hologram.
Yes, but the topology  seems to be different: the correct hologram should be of a completely enclosed cavity, while the hologram in the paper has two open holes: an entry hole and an exit hole that are not present in the EM Drive (unless one invokes radiation heat transfer, convective heat transfer, perhaps general relativity, etc.)

The topology is different.  The wave packet in the EM case has the shape and phase distribution set by the cavity.  If the cavity walls disappeared the trajectory of the wave packet would curve.  It can't do that because the cavity is still there and has much more mass-equivalant than the wavepacket, so all you see is the reaction force.
OK, I would have to work out the math to convince myself that the "walls dissapeared".  If they dissapeared we are in agreement. But to get there I need a proof, as you said  :)

You are still too quick for me !
4-D "curve" is acceleration.  The "holographic" representation is 3-D in the EM cavity.  The fixed plane is time.

It should be reducible to a x,y version w/ z,t in the propagation direction (?) but again the walls must disappear for it to propagate.  ??  does the Poynting vector satisfy that condition if the walls are removed ?  Probably not when I try to visualize it. ? although the standing waves are then propagating waves.  Still sounds like you need to integrate all point spherical waves over the cavity volume using their instantaneous amplitude and phase when the walls disappear.

(Sorry about thinking out loud)
Title: Re: EM Drive Developments - related to space flight applications - Thread 2
Post by: Rodal on 05/23/2015 03:12 pm
...
You are still too quick for me !
4-D "curve" is acceleration.  The "holographic" representation is 3-D in the EM cavity.  The fixed plane is time.

It should be reducible to a x,y version w/ z,t in the propagation direction (?) but again the walls must disappear for it to propagate.  ??  does the Poynting vector satisfy that condition if the walls are removed ?  Probably not when I try to visualize it. ?
The only way I can see having a non-zero period-time-averaged Poynting vector in a cavity is either through a nonlinearity (example: Marco Frasca's second order nonlinearity due to GR, or van Tiggelen's 4th order nonlinearity due to magneto-chiral effect), or through an energy gradient (radiative heat transfer, etc.).

The example you gave with the "backbone curve" (as it is known in the literature, where one has a nonlinear spring) is a nonlinearity.
Title: Re: EM Drive Developments - related to space flight applications - Thread 2
Post by: TheTraveller on 05/23/2015 03:13 pm
@TheTraveller - you are nothing if not pragmatic :D

Quote
Shawyer & Chinese conic frustums provide thrust that matches their equations predictions.
Please cite an example set of numbers that instantiates this:
1. frustum big diameter & curvature
2. frustum small diameter & curvature
3. frustum vertical height
4. operating frequency
5. predicted thrust
6. measured thrust

Can you do this?
(no dielectric please, and preferably flat ends)

Before I came on board, no one understood the Df equation, nor guide wavelength nor cutoff wavelength as used by Shawyer. It was all about classic physics and the tools derived from them showing NO THRUST will be generated. The pathway seemed to be:

If there is thrust then as our tools say no thrust, there must be new physics involved, despite Shawyer and the Chinese saying there is thrust and it can be predicted by a non classical use of the existing physics.

Instead of trying to understand what Shawyer and the Chinese are saying about how they successfully use existing physics to predict the thrust, the forum goes off to left field and dreams up ways to not need to try to understand what Shawyer and the Chinese are saying.

Which leave me, who wishes to build a Flight Thruster and have an excel spreadsheet that predicts thrust as the dimensions and frequency are varied in right field, far away from most on this thread who have chosen to play in left field and think what I'm doing is, well a sign of madness, as I will not Hi Five the desire to play in left field and ignore what Shawyer and the Chinese have shared.

Bottom line is both Shawyer and the Chinese have measured thrust on dozens of devices and that thrust is predictable from their equations. Yet those equations are totally ignored. So who is mad? Those who ignore what Shawyer and the Chinese have shared or those who ignore their results and the predictive equation they share?

For me, the pathway is to continue to talk with Shawyer, as he gives me a pat on the head and offers another bread crumb and to continue to put together a spreadsheet that allows me to calc Df and frustum length as per Shawyers shared info and equations so as to get optimal thrust at my desired operational wavelength.

With the greatest respect to others on the forum, why should I engage with equations that predict no thrust as they give me NO feedback to produce an excel spreadsheet to model what I'm attempting to do.

It is ALL about the THRUST (non dielectric) and how to model it so as I change big end and small end diameters, the Df calcs properly and from that, the length calcs properly as per the internal effective wavelength.
You need to reverse the polarity captain! Unconventional thinking is definitely left field - hence my monicker.

I follow Shawyer.

No need to reverse anything. His equations and explanations cover how this works. He uses conventional physics, just applied a bit different to normal.
Title: Re: EM Drive Developments - related to space flight applications - Thread 2
Post by: Notsosureofit on 05/23/2015 03:19 pm
...
You are still too quick for me !
4-D "curve" is acceleration.  The "holographic" representation is 3-D in the EM cavity.  The fixed plane is time.

It should be reducible to a x,y version w/ z,t in the propagation direction (?) but again the walls must disappear for it to propagate.  ??  does the Poynting vector satisfy that condition if the walls are removed ?  Probably not when I try to visualize it. ?
The only way I can see having a non-zero Poynting vector in a cavity is either through a nonlinearity (example: Marco Frasca's second order nonlinearity due to GR, or van Tiggelen's 4th order nonlinearity due to magneto-chiral effect), or through an energy gradient (radiative heat transfer, etc.)

Of course, if the Poynting vector stays zero then momentum is conserved.  Is that the case in a self-accelerating wavefunction ?  I havn't seen it explicitly mentioned but they do claim CoM.
Title: Re: EM Drive Developments - related to space flight applications - Thread 2
Post by: Rodal on 05/23/2015 03:25 pm
...
You are still too quick for me !
4-D "curve" is acceleration.  The "holographic" representation is 3-D in the EM cavity.  The fixed plane is time.

It should be reducible to a x,y version w/ z,t in the propagation direction (?) but again the walls must disappear for it to propagate.  ??  does the Poynting vector satisfy that condition if the walls are removed ?  Probably not when I try to visualize it. ?
The only way I can see having a non-zero Poynting vector in a cavity is either through a nonlinearity (example: Marco Frasca's second order nonlinearity due to GR, or van Tiggelen's 4th order nonlinearity due to magneto-chiral effect), or through an energy gradient (radiative heat transfer, etc.)

Of course, if the Poynting vector stays zero then momentum is conserved.  Is that the case in a self-accelerating wavefunction ?  I havn't seen it explicitly mentioned but they do claim CoM.
To be specific, let's point out that we are talking about the time-average (over an integer number of periods) of the Poynting vector being zero, as the Poynting vector itself is a non-zero harmonic function of time even as a solution of Maxwell's equations (the Poynting vector in that case having twice the frequency of the electromagnetic field frequency).
Title: Re: EM Drive Developments - related to space flight applications - Thread 2
Post by: Notsosureofit on 05/23/2015 03:31 pm
Yes, only the "disappearance" of the wall for mathematical reasons would be instantaneous.
Title: Re: EM Drive Developments - related to space flight applications - Thread 2
Post by: SeeShells on 05/23/2015 03:35 pm
  They are making an acoustic hologram.
Yes, but the topology  seems to be different: the correct hologram should be of a completely enclosed cavity, while the hologram in the paper has two open holes: an entry hole and an exit hole that are not present in the EM Drive (unless one invokes radiation heat transfer, convective heat transfer, perhaps general relativity, etc.)

The topology is different.  The wave packet in the EM case has the shape and phase distribution set by the cavity.  If the cavity walls disappeared the trajectory of the wave packet would curve.  It can't do that because the cavity is still there and has much more mass-equivalant than the wavepacket, so all you see is the reaction force.
OK, I would have to work out the math to convince myself that the "walls dissapeared".  If they dissapeared we are in agreement. But to get there I need a proof, as you said  :)

You are still too quick for me !
4-D "curve" is acceleration.  The "holographic" representation is 3-D in the EM cavity.

Consider this, let's turn on the acoustic wave guide. Then close off the exit, closing off the Acoustic chamber.  The waves entering into the closed area in the chamber would be reflected back creating another set of set of interference harmonics and possibly destructive to the pattern of the first mode. But let's say I dampened the acoustic waves on the exit door, turning them into a pattern of heat? I see heat in the patterns of the thermal images even though they are RF dissipation in the copper.
So tell me, what is the difference in a system thinking this way? We still maintain the desired mode, whether acoustic or em wave.
Notsosureofit I think we are seeing the same thing but you have a much more eloquent way of stating it, I'm more nuts... well and bolts kinda gal.
Title: Re: EM Drive Developments - related to space flight applications - Thread 2
Post by: Notsosureofit on 05/23/2015 03:42 pm

Consider this, let's turn on the acoustic wave guide. Then close off the exit, closing off the Acoustic chamber.  The waves entering into the closed area in the chamber would be reflected back creating another set of set of interference harmonics and possibly destructive to the pattern of the first mode. But let's say I dampened the acoustic waves on the exit door, turning them into a pattern of heat? I see heat in the patterns of the thermal images even though they are RF dissipation in the copper.
So tell me, what is the difference in a system thinking this way? We still maintain the desired mode, whether acoustic or em wave.
Notsosureofit I think we are seeing the same thing but you have a much more eloquent way of stating it, I'm more nuts... well and bolts kinda gal.

In this case we are ignoring the dissipation by inputting constant power to make up for it.  The EM and (pressure) acoustic systems are similar in many ways.

When I try to do a thermodynamic calculation all bets are off.
Title: Re: EM Drive Developments - related to space flight applications - Thread 2
Post by: TheTraveller on 05/23/2015 03:48 pm
By contrast, in a closed cavity, there is a reflection at the walls, there is nothing coming out (unless we consider quantum tunneling, or heat dissipation like radiative heat transfer in a vacuum or convective heat transfer in air)

Yes nothing is expelled from the frustum.

However in both the Shawyer and Chinese test data thrust is generated, which should say to you that what Shawyer is saying about his theory and what the Chinese are saying about their theory is correct and your and other conventional application of theory is not correct.

I refer to Feynman:
https://www.youtube.com/watch?v=kctmPaCkV0g

"If it [theory] disagrees with experiment it is wrong."

Yet the trust of this forum seems to be to reject Shawyer and Chinese theory, which matches their experimental results, and seek some new theory outside physics when all that is needed is to listen to Shawyer and the Chinese and apply existing theory in a non conventional way.

For an engineer, seeking to model in excel what is happening as dimensions and frequency are varied, the blue sky theories on this forum provide no help, as none can take frustum dimensions and frequency and predict thrust per power applied. In fact most say there is NO THRUST.

However Shawyer's theory and equations does appear to model the thrust but those equations and explanations seem to be almost universally rejected.

Which going back to Feynman, with respect, is about as unscientific as it can get. Others claiming sloppy measurement protocols are just trying to find excuses to deal with reality and avoiding needing to accept the Shawyer and Chinese applications of current theory and thrust measurements are correct.

For me I'll follow what Boyd Bushman & Paul March advised:
http://forum.nasaspaceflight.com/index.php?topic=36313.msg1356062#msg1356062

Quote
Follow the data, theory be dammed
Title: Re: EM Drive Developments - related to space flight applications - Thread 2
Post by: skrock on 05/23/2015 03:51 pm
Hi, I hope this is ok. Just found this interview with Shawyer on youtube https://www.youtube.com/watch?v=4hTdSg47h3k (https://www.youtube.com/watch?v=4hTdSg47h3k)

This thread is very interesting, still deciding what to study later on and this is inspirational stuff.
Greetings from Sweden.

/Marcus
Title: Re: EM Drive Developments - related to space flight applications - Thread 2
Post by: Notsosureofit on 05/23/2015 03:57 pm

Consider this, let's turn on the acoustic wave guide. Then close off the exit, closing off the Acoustic chamber.  The waves entering into the closed area in the chamber would be reflected back creating another set of set of interference harmonics and possibly destructive to the pattern of the first mode. But let's say I dampened the acoustic waves on the exit door, turning them into a pattern of heat? I see heat in the patterns of the thermal images even though they are RF dissipation in the copper.
So tell me, what is the difference in a system thinking this way? We still maintain the desired mode, whether acoustic or em wave.
Notsosureofit I think we are seeing the same thing but you have a much more eloquent way of stating it, I'm more nuts... well and bolts kinda gal.

In this case we are ignoring the dissipation by inputting constant power to make up for it.  The EM and (pressure) acoustic systems are similar in many ways.

When I try to do a thermodynamic calculation all bets are off.


FYI

Here we go:

http://physics.technion.ac.il/~msegev/publications/Maxwell_accelerating_beams.pdf

"For both TE and TM polarizations, the beams exhibit shape-preserving bending which can
have subwavelength features, and the Poynting vector of the main lobe displays a turn of more than 90"
Title: Re: EM Drive Developments - related to space flight applications - Thread 2
Post by: SeeShells on 05/23/2015 04:01 pm

Consider this, let's turn on the acoustic wave guide. Then close off the exit, closing off the Acoustic chamber.  The waves entering into the closed area in the chamber would be reflected back creating another set of set of interference harmonics and possibly destructive to the pattern of the first mode. But let's say I dampened the acoustic waves on the exit door, turning them into a pattern of heat? I see heat in the patterns of the thermal images even though they are RF dissipation in the copper.
So tell me, what is the difference in a system thinking this way? We still maintain the desired mode, whether acoustic or em wave.
Notsosureofit I think we are seeing the same thing but you have a much more eloquent way of stating it, I'm more nuts... well and bolts kinda gal.

In this case we are ignoring the dissipation by inputting constant power to make up for it.  The EM and (pressure) acoustic systems are similar in many ways.

When I try to do a thermodynamic calculation all bets are off.
I know the differences in acoustic pressure waves and EM waves gladly they both have similar phenomena of diffraction, reflection and interference and create standing waves in cavities.
I will be jazzed to see a thermodynamic calculation and no betting. ;)
Title: Re: EM Drive Developments - related to space flight applications - Thread 2
Post by: Rodal on 05/23/2015 04:05 pm
...

I refer to Feynman:
https://www.youtube.com/watch?v=kctmPaCkV0g

"If it [theory] disagrees with experiment it is wrong."

Yet the trust of this forum seems to be to reject Shawyer and Chinese theory, which matches their experimental results, and seek some new theory outside physics when all that is needed is to listen to Shawyer and the Chinese and apply existing theory in a non conventional way.

For an engineer, seeking to model in excel what is happening as dimensions and frequency are varied, the blue sky theories on this forum provide no help, as none can take frustum dimensions and frequency and predict thrust per power applied. In fact most say there is NO THRUST.

However Shawyer's theory and equations does appear to model the thrust but those equations and explanations seem to be almost universally rejected.

Which going back to Feynman, with respect, is about as unscientific as it can get. Others claiming sloppy measurement protocols are just trying to find excuses to deal with reality and avoiding needing to accept the Shawyer and Chinese applications of current theory and thrust measurements are correct.

...
Concerning Feynman, I was educated in the same institution where he studied, under the same scientific principles and approach.  I performed experiments since I was a freshman (I was lucky that they had started the Undergrad Research Opportunity Program and immediately engaged in hybrid chemical rocket propulsion experiments) at that institution until I got my Ph.D.  Nobody at that institution performs experiments following a single researcher's publications as if they were a holy book.

Feynman's famous Lectures, and his professional life, teaches an approach to physical problems that is the diametrical opposite of following a single researcher as a Guru or a Prophet, whose publications have to be revered, obeyed and followed as a religious book. 

Concerning this thread its focus is on an objective, skeptical attitude trying to ascertain whether the experimental reports are an artifact or a real propulsion effect and if so whether they can be used for space applications, and also discussing those possible space propulsion applications.
Title: Re: EM Drive Developments - related to space flight applications - Thread 2
Post by: WarpTech on 05/23/2015 04:06 pm

...
However Shawyer's theory and equations does appear to model the thrust but those equations and explanations seem to be almost universally rejected.
...


I reject them simply because there are numerous, glaring errors in both math and logic, in all of Shawyer's papers. It shows he has a vague engineering understanding of waveguide physics, but has very little understanding of the physical mechanisms that governs "why" those principle can be applied. As I've said, in the end, his results that F1 - F2 > 0, is correct, he's not that far off and IMO he provides a useful approximation. It is the derivation of the Thrust and his concept of how it is achieved that is incorrect. I'm sorry it's taking me so long to write up this data, but life is not easy and neither is physics!

Todd
Title: Re: EM Drive Developments - related to space flight applications - Thread 2
Post by: Notsosureofit on 05/23/2015 04:11 pm


FYI

Here we go:

http://physics.technion.ac.il/~msegev/publications/Maxwell_accelerating_beams.pdf

"For both TE and TM polarizations, the beams exhibit shape-preserving bending which can
have subwavelength features, and the Poynting vector of the main lobe displays a turn of more than 90"

"of the main lobe"

In our case the cavity keeps the shape from changing, so we see the force necessary to maintain the Poynting vector.

Title: Re: EM Drive Developments - related to space flight applications - Thread 2
Post by: deltaMass on 05/23/2015 04:20 pm
Quote from: TheTraveller
However in both the Shawyer and Chinese test data thrust is generated, which should say to you that what Shawyer is saying about his theory and what the Chinese are saying about their theory is correct and your and other conventional application of theory is not correct.
Sure thing. What prevents you, therefore, from writing down those six numbers? Don't we want to look at how the predictions of Shawyer match the engineering truth?

Quote
Shawyer & Chinese conic frustums provide thrust that matches their equations predictions.
1. frustum big diameter & curvature
2. frustum small diameter & curvature
3. frustum vertical height
4. operating frequency
5. predicted thrust
6. measured thrust

(no dielectric please, and preferably flat ends)
Title: Re: EM Drive Developments - related to space flight applications - Thread 2
Post by: SeeShells on 05/23/2015 04:25 pm

Yet the trust of this forum seems to be to reject Shawyer and Chinese theory, which matches their experimental results, and seek some new theory outside physics when all that is needed is to listen to Shawyer and the Chinese and apply existing theory in a non conventional way.

For an engineer, seeking to model in excel what is happening as dimensions and frequency are varied, the blue sky theories on this forum provide no help, as none can take frustum dimensions and frequency and predict thrust per power applied. In fact most say there is NO THRUST.

Quote
Follow the data, theory be dammed
As an engineer I question everything, I look at everything and I mean everything. I look at my coffee cup and I not only see a cup but see how it was made, I look at a radio and see the inside workings of waveforms getting amplified, rectified and how the speaker.  It is the way I think. Yes Shawyer and the Chinese have ideas on how this works and so I question it, it is what I do and so do others, It's how we are built. It's not to be meant to degrade the honest work they have done or discredit them or you in any way. I respect you and them and anyone here who has the dream.

I remember in 1959 on a cold October night watching a twinkling light in the sky and knew even then the world was going to change forever, it was Sputnik. That era was started by people who followed the data, those who dreamed, those who built, those who cheered, but we all in one way or another saw the twinkling light.

Shell
Title: Re: EM Drive Developments - related to space flight applications - Thread 2
Post by: Notsosureofit on 05/23/2015 04:32 pm


FYI

Here we go:

http://physics.technion.ac.il/~msegev/publications/Maxwell_accelerating_beams.pdf

"For both TE and TM polarizations, the beams exhibit shape-preserving bending which can
have subwavelength features, and the Poynting vector of the main lobe displays a turn of more than 90"

"of the main lobe"

In our case the cavity keeps the shape from changing, so we see the force necessary to maintain the Poynting vector.

Added:  in the conclusions...

". To complete
the picture, future work should study the possibility of 3D
accelerating beams, including those with trajectories that
do not lie in a single plane. In practical terms, this work
brings accelerating beam optics into the subwavelength
regime, through the less-than-wavelength features of our
solutions, facilitating higher resolution for particle
manipulation."
Title: Re: EM Drive Developments - related to space flight applications - Thread 2
Post by: phaseshift on 05/23/2015 04:36 pm

Yet the trust of this forum seems to be to reject Shawyer and Chinese theory, which matches their experimental results, and seek some new theory outside physics when all that is needed is to listen to Shawyer and the Chinese and apply existing theory in a non conventional way.

For an engineer, seeking to model in excel what is happening as dimensions and frequency are varied, the blue sky theories on this forum provide no help, as none can take frustum dimensions and frequency and predict thrust per power applied. In fact most say there is NO THRUST.


@TheTraveller,

There are different disciplines at work here.  They do what they know and understand and you do what you know and understand.  I myself am a software engineer (35 years) and likewise I have a different approach to get my head around everything I'm reading and learning. We are all heading in the same general direction just along different paths - this is a good thing.

Accept the differences of the different disciplines, they are necessary. :)
Title: Re: EM Drive Developments - related to space flight applications - Thread 2
Post by: rfmwguy on 05/23/2015 04:54 pm

...
However Shawyer's theory and equations does appear to model the thrust but those equations and explanations seem to be almost universally rejected.
...


I reject them simply because there are numerous, glaring errors in both math and logic, in all of Shawyer's papers. It shows he has a vague engineering understanding of waveguide physics, but has very little understanding of the physical mechanisms that governs "why" those principle can be applied. As I've said, in the end, his results that F1 - F2 > 0, is correct, he's not that far off and IMO he provides a useful approximation. It is the derivation of the Thrust and his concept of how it is achieved that is incorrect. I'm sorry it's taking me so long to write up this data, but life is not easy and neither is physics!

Todd

Well said Todd, for I have read and watched Shawyers papers/presentations and needed more of an explanation. Guess if it were cut & dry, this thread would have stopped long ago.
Title: Re: EM Drive Developments - related to space flight applications - Thread 2
Post by: Mulletron on 05/23/2015 04:59 pm
Brand new. Interview with the inventor of EmDrive. Good info in there.

https://www.youtube.com/watch?v=4hTdSg47h3k
Title: Re: EM Drive Developments - related to space flight applications - Thread 2
Post by: Chris Bergin on 05/23/2015 05:35 pm
This is probably a good place to end this thread, as noted was the plan today :)

Thread 3!
http://forum.nasaspaceflight.com/index.php?topic=37642.0

Farewell Thread 2, and we thank you.